Sei sulla pagina 1di 2812

The

Ultimate
Podiatry Interview Review Manual

© 2010 OZ
Ingredients:

AJM Prism Manual – 100 pages, the best one out there

Crozer-Keystone 2nd edition – Another really good ones

Surgery Review book (OCPM/SCPM) – Simple and easy to read

Classification manual (CCPM) – Quick overview of classifications to know cold

Radiology Review notes (SCPM) – Some topics are over-detailed for the interviews

Mangold manual – Abx, Labs, Pain management, Surgery procedures, EKG (SCPM)

Sean Pimp List – Excellent list of Q and A

OZ manual

Hershey – 1 of 2 classic 1000+ notes

Presby – 1 of 2 classic 1000+ notes

Residency Review manual – Good for some classic articles, lots of pages but a quick read

Trauma manual – Didn’t read this one before interviews

ABPS questions – Good practice run Qs

NBPME 2 questions – Some what easy Qs

Scholl Misc Manual – Sample cases to work up

Classic Articles

<Additional resource you should have is the “Podiatric Medicine and Surgery Part II: Pearls of Wisdom
: that one you will have to buy ~ 50$ @ Amazon/Walmart>
The AJM

PRISM And Externship Guide.


A 100-page Podiatric Residency Interview Study Manual.

This 2009 First Edition was edited by RC.

David Hockney’s Mount Fuji and Flowers


Introduction:
Preparing for externships and the residency interview is one of the most challenging tasks facing the podiatric
medical student. It can be a completely overwhelming exercise unless you are efficient about the way you approach the
process. If you are reading this introduction, then you are already aware that in order to be fully prepared for
residency interviews and externships, you need to study a lot more than what you got from classes and clinic in school.

You should have a strategy going into the residency interview, just as your interviewers should have a strategy about
how to evaluate you. One of the most important ways to prepare is to think about the interview process from the other
side of the table. The interviewers only have a given amount of time to spend with you; no more than 30 minutes in most
cases. This is not a lot of time. What do they really want to know about you? What information can they get from you in 30
minutes that tells them about what kind of doctor you are going to be?
Remember that the attendings of a program are essentially hiring you to help handle their patients, and at the
interview they want to know if they can trust you with this responsibility. Asking inane, esoteric questions during the
academic interview doesn’t really give them this information. On the other hand, asking basic “work-up” based questions
does. These questions allow the interviewer to see how you will be approaching their patients in the future. It gives
them information about how your mind works when dealing with patients on an everyday level. If you were an attending on
the other side of the table, would you rather know if the student can take you through the clotting cascade, or how they are
going to handle your patient in the ED with a suspected post-operative infection?

Another thing to think about is that the interviewers need to compare your answers to the other people you are going
against for the program, and they don’t have a lot of time to do this. They should have some standardized way of
quantitatively grading your performance against the performance of others. I like to think of this as “check marks”. Think
of the interviewers asking the same exact questions to each student and then having a form or a list in front of them. There
are certain “buzz words” that they want you to say and certain questions that they want you to ask during the work-up. The
more things you get correct, the more “check marks” you get on their form. And at the end of the day, they add up all the
“check marks” and see who got the most. Your goal during a 30-minute interview should be to get as many “check marks”
as possible.

This manual was put together based on the way that I studied for interviews. There is certainly no shortage of
material to study, and this manual in no way can replace or even rival some of the other study guides that are out there. The
goal of the PRISM is simply to help you be as efficient as possible with the process and to think about the interview
from the other side of the table. My goal with coming up with the following sheets was to take a given topic and fit
everything that could be asked about that topic during an interview onto a single sheet of paper. Realize that it is not all the
information on a given topic, but all the information that is likely to be asked during an interview. There’s a big
difference there.
Your goal heading into the interview process should be to have a standardized way of handling every question or
situation that you are presented with, and to get as many “check marks” as possible. Think about it. The easiest way for the
interviewers to answer the questions they have about you is to present you with a clinical scenario, and see how you work-up
that situation. Therefore, the most efficient way to study for interviews is to take a given topic, and then “work-up” a patient
in that situation. I made all of the AJM Sheets with this thought in mind (see “Gout” example on next page).

Also included in this manual are AJM Lists. Studying is by nature a passive exercise, but the interview process
involves actively answering questions and talking out loud. The Lists allow you to actively think about a topic like you will
be expected to do during the interview. It takes a broad, clinical situation/subject and asks you to come up with as many
answers as possible. I hope that they help you realize that there is not always one answer to a question, but possibly many
different answers that can all be considered correct. The more answers that you can come up with for a given List, the
more “check marks” you get during the interview.

Again, this manual is far from complete and absolutely does not contain all of the information you will be asked
during an interview. It simply hopes to change the way that you think about the interview process and highlight some
of the information that you are most likely to be asked. I limited it to 100 pages of the most commonly asked information
and the kind of stuff that I’m going to ask if I’m a residency director someday. It is not in any way meant to be
overwhelming.

I also want this to be a “living” document. It is not intended to be commercial and should never be sold. I’m going
to take it with me when I graduate from residency, leave it with the Inova program, and distribute it electronically to anyone
who wants it. Feel free to change/update it in any way that you think would be helpful, but please keep it to exactly 100
pages. In other words, if you think something is important and should be included, you also have to decide what isn’t as
important and should be taken out. It will be interesting to see how it evolves over the years!

Good luck and please do not hesitate to contact me if there is any way that I can be of service to you.

AJMeyr@gmail.com
2
AJM Sheet Example: Gout

-Subjective
CC: Pt classically complains of a “red, hot, swollen joint”. Typical patient is a male in the 6th decade (as much as a 20:1 M:F ratio).
HPI: -Nature: Intense pain out of proportion with swelling/pressure.
-Location: Single or multiple joints. Unilateral or bilateral. Most common is 1st MPJ, but can occur in any joint.
-Acute, abrupt onset; more commonly at night.
-Aggravating Factors: Pressure, WB, diet (red wine, organ meat, lard, seafood).
-History: Recurrent gouty attacks are very common
PMH: -Genetic enzyme defects, obesity, lead poisoning, tumor, psoriasis, hemolytic anemia may all be underlying causes.
-Renal disease (renal disease is 2nd most common complication of gout).
-Kidney stones
Meds: -Diuretics, low dose ASA, TB meds, warfarin may exacerbate.
SH: -Diets high in red wine, organ meat, lard and seafood may exacerbate.
All/FH: -Usually non-contributory
ROS: -May be associated with fever.

-Objective
Physical Exam
-Derm: -Erythema, Calor, Edema present at affected joint
-May see tophi sticking out of skin
-Vasc: -Non-pitting edema at affected site
-Neuro: -Intense pain out of proportion
-Ortho: -Decreased PROM/AROM at affected joint with guarding.

Imaging
-Plain Film Radiograph: -Increased soft tissue density with joint effusion. Tophi may be visible in soft tissue.
-Fine striated pattern of periosteal reaction along the cortex adjacent to tophi
-Lace pattern of osseous erosion
-Round osseous erosion with a sclerotic margin (“rat bite erosion” or “punched-out lesion”).
-Martel’s sign: Expansile lesion with an overhanging osseous margin.

Laboratory
-Joint aspirate is mandatory for diagnosis of gout:
-Needle-shaped monosodium urate crystals
-Negatively birefringent (bright yellow) when viewed under polarizing light microscope parallel to axis of lens.
-Blue when perpendicular to axis of lens.
-Serum uric acid levels > 7.5mg/dl (non-diagnostic)
-Elevated ESR
-Synovial fluid analysis: Elevated leukocytes with a predomination of neutrophils
-Generalized increased white cell count

-General Information
-Definition: Metabolic disorder secondary to the build-up of monosodium urate crystals and supersaturated hyperuricemic extracellular fluids in and around
joints and tendons causing the clinical manifestations of a red, hot, swollen joint.
-It is the most common cause of inflammatory arthritis in men over the age of 30.

-Classification
-Primary: Elevated serum urate levels or urate deposition secondary to inherent disorders of uric acid metabolism.
-Uric Acid Overproduction (Metabolic Gout): 10% of patients
-Excessive amounts of uric acid excreted into the urine
-Occurs secondary to an enzyme defect, tumor, psoriasis, hemolytic anemia, etc.
-Dx: Uric Acid Level >600mg in a 24-hour urine collection
-Uric Acid Undersecretion (Renal Gout): 90% of patients
-Relative deficit in the renal excretion of uric acid.
-Secondary: A minor clinical feature secondary to some genetic or acquired process

-Treatment
-Symptomatic Pharmacology (relieves symptoms, but doesn’t attack underlying pathophysiology)
-Indomethacin: 50mg PO q8
-Colchicine: 0.5-1.0mg PO initially, then 0.5mg PO q1 hour until symptoms (GI) or pain relief
Then around 0.5mg PO qday as prophylaxis
-Active/Physiologic Pharmocology (attacks underlying pathophysiology and prevents recurrence)
-Allopurinol: 100-600mg PO qday as single or divided doses.
-Blocks uric acid production by inhibition of the enzyme xanthine oxidase.
-Probenecid: 250mg PO bid for one week; then 500mg PO bid
-Increases uric acid removal from urine (decreases reabsorption)
-Surgical Intervention (if you get rid of the joint, then you get rid of a potential site for gout to attack!)
-I&D/Washout
-Arthroplasty
-Arthrodesis

-Further Reading
-Roper RB. The perioperative management of the gouty patient. J Amer Podiatry Assoc. 1984 Apr;74(4):168-72.
-Schlesinger N. Management of acute and chronic gouty arthritis: present state-of-the-art. Drugs. 2004;64(21):2399-2416.
-Keith MP. Updates in the management of gout. Am J Med. 2007 Mar;120(3):221-4.

3
Table of Contents:

AJM Lists [Pages 5-29] -45: Common Situational Bugs


-46: Empiric Antibiotic Choices
-5: Introduction and Proposed Schedule -47: IDSA Empiric Recommendations
-48-49: Bugs with Drug of Choice
-Surgery Lists…………………....……….6-10
-50: Antibiotic Dosing Guide
-6: HAV Procedures with Indications
-7: Risks and Complications of Surgery -Trauma…………………………………51-68
-8: Measurement of Radiographic Angles -51: Introduction and Contents
-9: Radiographic Review -52: The Trauma Work-Up
-10: Surgical Layers of Dissection -53-54: General Trauma Topics
-55: Digital Fractures
-Medicine Lists…………………….……11-15
-56: Sesamoid Trauma
-11: Post-Op Fever Etiology -57: Metatarsal Fractures
-12: Lab Infection Diagnosis -58: 5th Metatarsal Fractures
-13: Imaging Infection Diagnosis -59: Metatarsal Stress Fractures
-14: Labs and Why they are important -60: LisFranc Trauma
-15: Vascular and Neurologic Assessment -61: Navicular Trauma
-62: Talar Fractures
-Trauma Lists…………………………...16-20 -63: Calcaneal Fractures
-16: Ankle Fx DDx -64-65: Ankle Fractures
-17: Synthes Chart with Screw Anatomy -66: General Tendon Trauma
-18: Methods of Fixation -67: Achilles Tendon Work-up
-19: Hardware Insertion -68: Achilles Tendon Treatment
-20: Classifications
-Peri-Operative Medicine and Surgery….69-99
-Anatomy Lists……………………….…21-25 -69: Introduction and Contents
-Peri-Operative Medicine
-21: Ossification of Lower Extremity Bones
-70: Admission Orders
-22: 5th Metatarsal Anatomy
-71: Electrolyte Basics
-23: Dorsal Arterial Anastomosis Variations
-72: Glucose Control
-24: Lower Extremity Peripheral Nerve Blockade
-73: Fluids
-25: Dermatomes with Spinal Levels
-74: Post-Op Fever
-Social Interview Lists……………....….26-29 -75: DVT
-76: Pain Management
-General Surgery Topics
-77: AO
-78: Plates and Screws
AJM Sheets [Pages 30-100]: -79: Suture Sheet
-80: Surgical Instruments
-Diabetic Foot Infections……………….30-50 -81: Power Instrumentation
-30: Introduction and Contents -82: Biomaterials
-31: Diabetic Foot Infection History -83: External Fixation
-32: Diabetic Foot Infection Physical Exam -84: Bone/Wound Healing
-33: Wound Classification Systems -Specific Surgery Topics
-34-35: Diabetic Foot Infection Laboratory Results -85: How to “Work-Up” a Surgical Patient
-36: Common Infective Agents -86-87: Digital Deformities
-37: Diabetic Foot Infection Imaging Studies -88: Lesser Metatarsals
-38: Diabetic Foot Infection Pathogenesis -89: 5th Ray
-39: Functional Diabetic Foot Infection Anatomy -90-91: HAV
-40: Osteomyelitis -92: HAV Complications
-41: Osteomyelitis Classifications -93-94: HL/HR
-42: Charcot Neuroarthropathy -95-96: Pes Plano Valgus
-43: Charcot Classifications -97-98: Cavus
-44: Differentiating Charcot vs. Osteomyelitis -99: Equinus

-Page 100: “Can you give me some good articles to read?”

4
Lists Schedule:
AJM Lists were originally created to be done during an externship. Students often have a lot of
down time during the day while the residents are doing work that doesn’t need assistance. The lists give
the students something to do during this time and make it look like they’re busy instead of just standing
around doing nothing (in front of the attendings and residents). It also encourages students to collaborate,
and shows the residents/attendings that they can work well with each other and in groups.
When I was a resident, I would give the students one list and a related article each day, and then
we would try and get together once a week to go over them. It usually generated a great deal of good
discussion. If you are using these lists to study on your own, get together with a group of friends to go over
them and talk about your answers out loud. The way you know if you really understand a topic is if you
can intelligently discuss it and explain it to your peers.
Again, studying is by nature a passive exercise, but at the interview you will be expected to
actively answer questions out loud. Only about half of what the interviewers appreciate from your
answer is the actual content, the other half is how you say it. Remember that the interviewers are
probably asking the same question to every student that walks through the door, so they’ve probably heard
the same answer several times before you even sit down. What they haven’t heard is how you’ve said it!
In other words, you should also be studying “how to say it”.
See the following suggested schedule:

Mondays: Surgery
-HAV Procedures with Indications (page 6)
-Risks and Complications of Surgery (page 7)
-Measurement of Radiographic Angles (page 8)
-Radiographic Review (page 9)
-Surgical Layers of Dissection (page 10)

Tuesdays: Medicine
-Post-Op Fever Etiology (page 11)
-Lab Infection Diagnosis (page 12)
-Imaging Infection Diagnosis (page 13)
-Labs and Why they are Important (page 14)
-Vascular and Neurologic Assessment (page 15)

Wednesdays: Trauma
-Ankle Fx DDx (page 16)
-Synthes Fill-in Chart with Screw Anatomy (page 17)
-Methods of Fixation (page 18)
-Hardware Insertion (page 19)
-Classifications (page 20)

Thursdays: Anatomy
-Ossification of Lower Extremity Bones (page 21)
-5th Metatarsal Anatomy (page 22)
-Dorsal Arterial Anastomosis Variations (page 23)
-Lower Extremity Peripheral Nerve Blockade (page 24)
-Dermatomes with Spinal Levels (page 25)

Fridays: Social Questions


-Social Question Sheets: Part I (pages 26-27)
Part II (pages 28-29)
Part III (page 30)

5
AJM List: HAV Procedures and Indications

Clinical Scenario: You are a first year resident scheduled to be in a “bunion procedure” at a surgery center
tomorrow. You are working with the attending for the first time and want to appear as prepared as
possible. Name as many “bunion procedures” as you can.
Student Goal: Name 20 HAV procedures from distal to proximal. What are the specific clinical and
radiographic indications for each?

6
AJM List: Surgical Complications

Clinical Scenario: You are a first year resident at a surgery center. The attending isn’t there yet, and you
aren’t exactly sure of the specifics of the case, but you want to have all the paperwork filled out for when
the attending gets there (including the consent).
Student Goal: Name as many risks and complications of a generic foot and ankle surgery as possible.
What are some specific complications associated with some specific surgeries? How would you handle
these complications in the post-op period?

7
AJM List: Measurement of Radiographic Angles

Student Goal: Name as many foot and ankle radiographic measurements as possible. What are the normal
values? What clinical information is this giving you?

8
AJM List: Radiology Review

Clinical Scenario: There is a big difference between describing and diagnosing a radiograph. During an
interview, you need to be able to describe the findings you are seeing before you diagnose the pathology.
You also need to be able to do this out loud during the interview process.
Student Goal: Out loud, using as many terms as possible, and in a systematic manner, intelligently describe
the following radiographs before making a diagnosis.

Note: RC and I found it very helpful to practice for interviews by picking up random podiatry textbooks,
and alternating through the pictures describing the radiographs out loud.

9
AJM List: Surgical Layers of Dissection

Student Goal: Identify the 5 surgical planes and 3 surgical intervals of dissection. What structures will you
see in each plane/interval for a standard HAV procedure? For a standard digital procedure?

10
AJM List: Post-operative Fever

Clinical Scenario: On call, you are paged at 3am by a nurse to report a fever in a patient POD#2 of
101.6°F.
Student Goal: Name as many potential causes of fever as possible. How would you diagnose and work-up
each one?

11
AJM List: Laboratory Infection Diagnosis

Clinical Scenario: A patient enters the Emergency Department with a suspected lower extremity infection.
Student Goal: Name as many laboratory tests that you can order to help diagnose an infection. What
clinical information is each test really telling you about the situation?

12
AJM List: Imaging Infection Diagnosis

Clinical Scenario: A patient enters the Emergency Department with a suspected lower extremity infection.
Student Goal: Name as many imaging tests that you can order to help diagnose an infection. Exactly what
are you looking for with each test?

13
AJM List: Routine Lab List

Student Goal: Identify routine labs, their normal values, and what information they are giving you about
the patient. Which labs should be drawn when an infection is suspected and why? Which should be done
in the pre-operative work-up of a patient?

14
AJM List: Vascular and Neurologic Assessment

Student Goal: Name as many subjective and objective vascular and neurologic assessments as possible
while performing a lower extremity examination.

15
AJM List: Ankle Fracture

Clinical Scenario: A patient arrives in the ED exclaiming that they “broke their ankle”. Obviously you
will take an x-ray, but what exactly are you looking for on the radiograph?
Student Goal: Name as many possible fractures that can occur following an ankle sprain.

-Bonus: What do the “Ottawa Ankle Rules” say about getting a radiograph following an ankle sprain?

16
AJM List: Screw Games

Thread Spheric Screwdriver: Core Drill bit Drill bit Tap


Diameter Head Hex or Diameter Thread Gliding Diameter
Diameter Cruciate? Hole Hole
Mini Frag Set
1.5mm

2.0mm

2.7mm

Small Frag Set


3.5mm

4.0mm PT

4.0mm FT

Large Frag Set


4.5mm

4.5mm
Mall Screw
6.5mm PT

6.5mm FT
Hint: pg 76 McGlam’s

-Pitch on a cortical screw?:

-Pitch on a cancellous screw?:

-Draw a screw labeled with as many anatomic landmarks identified as you can (eg. Head, major
diameter, pitch, etc):

17
AJM List: Methods of Fixation

Student Goal: Name as many methods as possible to fixate an osteotomy.

18
AJM List: Hardware Insertion Technique

Student Goal: Describe standard AO lag screw insertion technique. What is the purpose of each step?
Why are they done in that particular order? What is compromised technique? Splintage?

-Bonus: What is the quantitative measurement of “two-finger tightness?”:

-What are some strategies for hardware extraction?:

19
AJM List: Classifications

Student Goal: Name as many different trauma classifications as you can that cover the foot and ankle from
distal to proximal.

20
AJM List: Ossification Dates

Student Goal: Name every bone in the lower extremity in order of ossification date.

21
AJM List: 5th Metatarsal Anatomy

Student Goal: Name as many structures as you can that attach to the 5th metatarsal.

22
AJM List: Dorsal Arterial Anastamosis Variations

Student Goal: Draw out as many different variations as possible for the arterial supply to the dorsum of the
foot.

How does this apply to the angiosome principles? (hint: read Dr. Attinger’s work)

23
AJM List: Local Anesthesia and Peripheral Nerve Blockade

Student Goal: Identify as many named foot and ankle peripheral nerve blocks as possible. Which specific
nerves are being anesthetized with each block?

-Toxic Dose of Lidocaine?:

-Toxic Dose of Marcaine?:

-How and why does epinephrine influence the toxic dose of a local anesthetic?

-What are the reversing agents for local anesthetic toxicity?

24
AJM List: Dermatomes

Student Goal: Draw a lower extremity with all of the dermatomes illustrated with associated spinal levels
and landmarks.

-Bonus: How are dermatomes different than sclerotomes?

25
AJM List: Social Questions
Part I: General Questions

Personal:
Strengths: Be prepared to give at least 3 personal strengths and why they will make you a good resident.
-Strength #1:
-Why it will make you a good resident:
-Strength #2:
-Strength #3:

Weaknesses: Be prepared to give at least 3 weaknesses, and more importantly, how you are remedying them.
-Weakness #1:
-Remedy:
-Weakness #2
-Weakness #3:

Goals: Be prepared to give professional and personal goals, and how you will go about accomplishing them. Another
form this question could take is where you see yourself in a given number of years.
-Professional Goals:
-Goal #1:
-Goal #2:
-Goal #3:

-Personal Goals: -Where do you see yourself in:


-Goal #1: -5 years?:
-Goal #2: -10 years?:
-Goal #3: -25 years?:

Program Specific: For each program that you apply to, you should have a list of strengths and weaknesses for that
program. Obviously be careful with weaknesses, and always have a way that you can improve the situation. You
should be able to answer why you personally are a good fit for that program. I found it helpful to have a list of
priorities that I was looking for in the different programs, and then described how that particular program fit into my
priorities.

-Program #1:
-Strength #1: -What you are able to bring to the program:
-Strength #2: -Favorite attending and why:
-Strength #3: -Least favorite attending and why:
-Weakness #1: -Favorite resident and why:
-Weakness #2: -Least favorite resident and why:
-Best case you saw at the program:
Here’s my list of priorities that I used based on what was important to me. Everyone’s list can (and really should) be
different, this is just to provide an example:
1. Surgery/Academics
-How is this program going to make you a better doctor? You’re going to learn surgery and do
some academic events at any program in the country, is there anything special about this particular
program that sets it apart? Do they really care about academics, or are they just done to meet a
requirement? (Hint: a good way to tell this is to see how often attendings are excited to show up to
and be involved in meetings). Although the quantity of surgery is important (you have be able to
get your “numbers”), also consider the quality and variety of the surgeries at a program.
2. Outside Rotations
-All programs have the same set of “core” rotations that everyone has to do. Is there anything
unique about this particular program that shows that they really care about your complete education
and want you to have outstanding quality and variety to your residency experience?
3. Location:
-This one kind of speaks for itself, but you should consider if you are going to be completely at one
hospital versus traveling to different hospitals and different surgery centers (there are pros and cons
to each). Also consider what the presence is of the program within the hospital. Is the podiatric
surgery department intricately involved in the hospital, or is it more of an afterthought?
4. Independence
-I’m an independent guy who likes to come up with and pursue my own projects and ideas. Other
people really like structure and would prefer to get an exact schedule for the next three years on day
1 of their residency. So this was something that I was really looking for, but someone else may
want exactly the opposite.
5. The Future
-How is this program going to help you accomplish your professional goals now and after you
graduate?

26
AJM List: Social Questions
Part II: Personal Questions

Personal Questions: These questions are hard to answer and often irrelevant, but you should have answers
ready to go (even if they are made up). Answer as specifically as possible to give the interviewer
something tangible to grab onto about yourself. Always answer “Why?” before they have the chance to ask
you. “Why?” may be the most important question you get during the interview process.

-What do you like to do with your free time?


-Answer #1:
-Answer #2:
-Answer #3:
-What professional accomplishments are you most proud of?:
-What personal accomplishments are you most proud of?:
-What was your hardest/most trying experience? What did you learn from this?:
-What is your most embarrassing moment?:
-Name three things that you would bring with you to a deserted island:
-#1:
-#2:
-#3:
-Tell me a joke:
-#1:
-#2:
-#3:
-Favorite Movie:
-Favorite Book:
-Last movie you saw:
-Something you liked about it:
-Something you didn’t like about it:
-Last book you read:
-Favorite Band/Kind of Music:
-Last concert you went to:
-Tell me about the craziest patient you have ever had to deal with:
-What is your funniest medical story?:
-What animal would you be and why?:
-What tree would you be and why?:
-Favorite TV show:
-Favorite actor:
-Favorite actress:
-Favorite surgical instrument:
-Do you have any pets?:
-Favorite pet:
-If you could take a vacation anywhere in the world, where would it be?:
-Tell me something about yourself that few people know:
-Who’s the most important person in your life (real and/or fictional) that you have never met?:

-And of course, “Tell me a little about yourself”:


-Important Note: You should be able to spout off both a 30-second and a 5-minute answer to this
question at the drop of a hat.

-Note: These questions are very easy to answer if you think about them, but you don’t have time to think
during the interview. You don’t want to show any hesitation during this process, especially questions about
yourself. The worst answer you can give to a specific personal question is “I don’t know.” What is that
saying about you?

***Practice answering all of these questions out loud to yourself in the time leading up to interviews! You
may feel silly talking to yourself, but it is undoubtedly the best way to prepare for this line of
questioning.***

27
AJM List: Social Questions
Part III: Academic and Ethical Social Questions

Academic Social Questions: These are hidden academic questions, but ones you can’t study for in any
book. Please plan these questions out because it is very easy to get trapped in your answer if you lie.

-What journals do you read? Which is your favorite?:

-What was the last good journal article you read? (be able to cite it!):

-What was the last thing you built with your hands?:

-Favorite class in school?:

-Least favorite class in school?:

-Favorite teacher in school?:

-What types of things does the field of podiatric surgery need to do to improve in the future?:

-What is something you learned about the field of podiatric surgery since you started school/externships?:

-Have you participated in any research projects? Why or why not? What was your role in this project?:

-What would you do with your life if you couldn’t be a surgeon/physician?:

Ethical Questions: The key to answering an ethical question is to take a step back from the situation.
Pretend that someone else is in the situation and you are going to give that person advice. Don’t pretend
that you are in the situation; it actually makes it more difficult to think through the process. Remember the
concept of chain-of-command and also remember that there is a real patient involved.
Something else that really helped me out was having a clear order of my priorities. Everyone’s
can be different, but mine are:
1. Responsibility to the patient as a physician
2. Responsibility as an employee of a hospital
3. Responsibility as a resident of the residency program
4. Responsibility for my own education
5. Responsibility for the education of junior residents/students

So whatever ethical situation I was put into, I would make decisions based upon this order of
priorities. Remember that usually there is no right or wrong answer when it comes to ethical situations.
Like George Costanza said about beating a lie detector test: “It’s not a lie (wrong), if you believe it.”

The Semistructured Conversation: Many residency programs (and especially general medicine or general
surgery residency programs) have reevaluated the way that they have traditionally selected residents, and
have moved away from a structured academic interview. They have instead moved onto what’s called a
“semistructured conversation” that tries to evaluate if the applicants have the “knowledge, skills and
attitudes deemed necessary for the practice of medicine”. The questions are a kind of mix of academic and
ethical questions that can develop into more of a conversation. So while these are not purely academic
questions, they can lead into a conversation about specific academic topics. There’s actually a couple
articles about it (Neitzschman HR, Neitzchman LH, Dowling A. Key Component of Resident Selection: The
Semistructured Conversation. Acad Radio. 9: 1423-29; 2002.), and I’ve put together a long list of these
type questions on the next List.

28
AJM List: Social Questions
Part IV: The Semistructured Conversation Interview Questions

-Tell me about a patient care situation in which podiatric surgery altered the management of the patient.
-Describe a critical clinical situation and how you communicated with the family.
-Tell me about something you learned during one of your externships.
-How would you respond to a patient who asks, “Am I going to die?”
-Can you recall any time when you disagreed with a patient’s diagnosis or treatment?
-Tell us about the biggest argument/controversy you were involved with in podiatry school.
-Give us an example of a situation when you were pivotal in the resolution of a conflict between two other people.
-Suppose you’re in charge of the call schedule. You need to fill a slot with one of two people, one of whom has told you he
has to be out of town as best man in a wedding, and the other has to present a paper at a meeting. How would you resolve the
conflict?
-Describe a time when you were in a position to give someone a bad evaluation. How did you handle it?
-What features would you add to a medical school curriculum that you think might better prepare you for a podiatric career?
-Can you recall a time when you received an evaluation with which you disagreed?
-If a referring physician insists that you perform a study on a patient and you believe that study could be harmful to the
patient, how would you handle the situation?
-Describe a patient for whom you felt very little empathy but you knew you should.
-What would you do if you saw a senior resident make a mistake that might harm a patient if not corrected promptly?
-A patient acquires your pager number and home phone number and calls several times per day. How do you handle this
patient’s needs?
-A consulting physician asks you a question, and you are not sure of the answer. How do you handle it?
-Tell me about a negative interaction you had during medical school with anyone from a transporter to an attending and how
the two of you dealt with it at the moment and afterwards.
-What do you see as the most challenging aspects of a podiatric residency?
-How would you handle a situation when you know one of your fellow residents has a problem with drugs or alcohol?
-For what reasons do you want to come this particular geographic area?
-You are on call and someone else asks you a question on a subject you know nothing about. How do you gather information
about the topic expeditiously?
-What resources did you use for researching residency programs?
-Outside of the structured lecture, what other formats did you find most helpful as learning tools?
-Describe for me how you deal with a colleague who is exhibiting evidence of substance abuse.
-Describe your response to an episode of someone cheating on the Gross Anatomy final examination.
-How would you decide (and what factors would you consider), as an HMO executive, whether to immunize 2,500 children
at $100 each or provide one liver transplant at $250,000 each?
-In what ways do you maximize your own health and well-being?
-How would you deal with a fellow resident who is not “pulling their own weight” in their work?
-What personal questions most helped you during medical school?
-Tell me about a patient from whom you learned something.
-How would you like to see podiatric surgery develop over the next 5 years?
-How do you see yourself changing between now and the end of residency?
-Tell me about your experience in using online resources, library resources, and internet resources.
-Did you ever feel as a medical student that you were not part of the clinical team? How did you address the situation in
order to optimize your learning experience?
-Can you recall an experience that made you decide to choose podiatry as a profession?
-Describe one of your most challenging cases during your externships.
-Tell me about an experience in medical school where you felt particularly competent.
-What particular skill do you feel you have that makes podiatric surgery the best specialty for you.
-As you examine different programs, what characteristics are you looking for that we might offer you?
-What diseases or topics have fascinated you in medical school and why?
-What topics interest you that you haven’t had time to explore yet?
-Tell me about an experience when you took a risk that ended up being successful.
-Can you tell me about a patient who had an impact on you?
-Can you recall a time in medical school when you had some doubt about the professional path you have chosen? What did
you do?
-A number of residents (15-20%) leave general surgery residency. What do you think influences their choice?
-How would you rate yourself in terms of your ability to establish rapport and maintain healthy relationships with other
health professionals?

29
AJM Sheets: Diabetic Foot Infection Work-Up
The Diabetic Foot Infection work-up is arguably the most important concept that you can study
during the interview process because it is the one topic that you are almost guaranteed of being asked at
some point. My thought process during interviews was that if I’m certain that I will be asked about it, I’m
going to spend extra time and energy knowing everything possible on the subject. Every student at
interviews is going to get something along these lines; therefore it’s important to be the most prepared and
best able to “wow” the attendings when asked. So I put together a collection of AJM Sheets (totaling about
20 pages) that goes through an in-depth work-up of a diabetic foot infection.

This topic is also a classic example of hitting as many “check marks” as possible during the
interview by having a standardized way of going through a work-up. The way this situation is often
presented at interviews is for them to simply ask you:
“There is a diabetic patient in the ED with a suspected foot infection. What do you want to know
about the patient, and what do you want to do?”
By having a standardized way of going through this work-up (or any work-up), you will seem
more prepared during the interviews, hit more check marks, and won’t stumble about thinking what to ask
next. The basics of this work-up can be applied to any clinical situation.

This work-up also highlights taking an active approach and going on the offensive during the
interview process. Take control of the interview from the interviewers. Do not simply ask if the patient
has diabetes; ask specific questions about the patient’s knowledge, management and known complications
of diabetes. This will show that you really understand the concepts and pathogenesis of the disease
process.

This section has a lot of the same information presented in a number of different ways, giving you
a couple ways to study. While there is certainly no shortage of material to study this information from, my
favorite article on the topic is a must-read: Lipsky BA, et al. Diagnosis and Treatment of Diabetic Foot
Infections. IDSA Guidelines. CID 2004; 39: 885-910. You also certainly should read: Frykberg RG, et
al. Diabetic foot disorders. A clinical practice guideline (2006 revision). J Foot Ankle Surg. 2006 Sep-Oct;
45(5 Suppl): S1-66. And finally, the June 2006 Supplement of Plastic and Reconstructive Surgery is a
fantastic resource covering a wide variety of diabetic foot issues, mostly from the Georgetown perspective.

Contents:
-Diabetic Foot Infection History (page 31)
-Diabetic Foot Infection Physical Exam (page 32)
-Wound Classification Systems (page 33)
-Diabetic Foot Infection Laboratory Results (pages 34-35)
-Common Infective Agents with Gram Stain Characteristics (page 36)
-Diabetic Foot Infection Imaging Studies (page 37)
-Diabetic Foot Infection Pathogenesis (page 38)
-Functional Diabetic Foot Infection Anatomy (page 39)
-Osteomyelitis (page 40)
-Osteomyelitis Classifications (page 41)
-Charcot Neuroarthropathy (page 42)
-Charcot Classifications (page 43)
-Differentiating Charcot vs. Osteomyelitis (page 44)
-Common Situational Bugs (page 45)
-Empiric Antibiotic Choices (page 46)
-IDSA Empiric Recommendations (page 47)
-Bugs with Drug of Choice (pages 48-49)
-Antibiotic Dosing Guide (page 50)

30
AJM Sheet: Diabetic Foot Infection Subjective History

Subjective
CC: Pt’s can present with a wide variety of complaints ranging from the systemic signs of infection to increased ulcer
drainage to a change in mental status. Infection should always be in your differential diagnosis dealing with any situation.

HPI: Ask the patient at least the following questions:


-NLDOCAT of chief complaint
-Systemic signs of infection: Nausea, vomiting (quantity and quality), fever, chills, night sweats, ague, loss of
appetite, change in mental status, diarrhea (quantity and quality), constipation, change in sleep patterns, headache, shortness
of breath, chest pain, etc.
-Local signs of infection: Patient reported increases in pain, erythema, swelling, temperature, drainage (quantity
and quality), odor, etc.
-Ulcer specific questions if applicable: Duration of ulcer, changes in size/depth/color, dressing changes, dressing
change schedule, wound care products, last formal evaluation, primary wound care specialist, previous treatments, any
history of hospitalization for infection, etc.
-Remember: The patient probably knows their ulcer better than you!
-It is extremely important to get an antibiotic history from the patient. Are they taking any antibiotic therapy
currently (including dosage and last dose)? When was the last time they were prescribed an antibiotic?, etc. This
information can provide useful information with respect to the development of resistant organisms. Specific risk factors
include antibiotic use in the last 6 months, any fluoroquinolone use, and hospitalization in the last 6 months.
-Richard et al. Risk factors and healing impact of multidrug-resistant bacteria in diabetic foot ulcers. Diabet Metab. 2008 Sep.
-Hartemann-Heurtier et al. Diabetic foot ulcer and multidrug-resistance organisms: risk factors and impact. Diabet Med. 2004 Jul.
-Kandemir et al. Risk factors for infection of the diabetic foot with multi-antibiotic resistant microorganisms. J Infect. 2007 May.
-Tetanus Status
-NPO Status

PMH: -DM: Complete DM history including length of disease, previous complications, glucose monitoring schedule,
normal glucose readings, HbA1c values, medications, last podiatric evaluation, last internal medicine evaluation,
implemented preventative measures, evaluation of patients level of understanding of pathogenesis of disease, evaluation of
patients role in self-treatment, etc.
-Any known complications of diabetes with interventions/treatment: cardiac disease, peripheral vascular disease,
hypertension, retinopathy.
-Specifically ask about renal disease and liver disease (antibiotic implications).
-Any other immuno-compromising conditions.
-Any other PMH issues.

PSH: -Specifically any previous amputations, foot/ankle surgeries and diabetes-related surgeries.
Meds: -Detailed list of drugs, dosages, and patient compliance to schedule.
All: -True allergies and reactions to drugs, food, products, etc.

SH: -Very important and not to be overlooked.


-Work: line of work, quantity of WB and ambulation, hours, ability of the patient to take time off or take it
easy, worker’s compensation issues, etc.
-Diet and exercise.
-Home support network. Includes assessment of patient compliance and family
understanding/education/compliance.
-Smoking, alcohol, drug use.
-House structure: stairs, bathrooms, pets.
-Other wound contamination risk factors.

FH: -Anything applicable.


ROS: -Anything applicable.

***Diabetic foot infections are one of the most challenging aspects of podiatric surgery that will take up a lot of your time,
energy, and stress if you dedicate yourself to the side of limb salvage. Taking a complete history will give you an idea of
how compliant you can expect the patient to be and how actively involved you can expect the patient to be in their care.

31
AJM Sheet: Diabetic Infection Objective Physical Exam
Objective
-Vital Signs: -Temperature: Hyperthermia is a non-descript sign of infection. It is important to monitor temperature on a regular
basis, and follow both current and maximum temperatures. Keep in mind however that Armstrong has documented that 82% of patients
admitted for osteomyelitis were afebrile on admission (JFAS.1996 Jul-Aug; 35(4): 280-3). It has been suggested that diabetic patients,
particularly those with ESRD, are not able to mount an effective immunologic response to the invading pathogen.
-Blood Pressure: Hypotension is a sign of sepsis and non-descript measure of infection.
-Heart Rate: Tachycardia is a sign of sepsis and non-descript measure of infection.
-Respiratory Rate: Increased respiratory rate is a sign of sepsis and non-descript measure of infection.
-Pain Level: Important to document and follow. Has been deemed the “5th vital sign” by JCAHO.
-Glucose Levels: AJM considers blood glucose level the “6th vital sign” and can be one of the most important
quantitative measurements of infection and response to therapy. Research indicates that the immune system is significantly impaired and
essentially not working at levels as high as 150-175 ml/dL. (The Portland Diabetic Project is a good place to start reading about this. Also
see Inzucchi SE. Management of Hyperglycemia in the Hospital Setting. NEJM. Nov 2006. 355;18: 1903-11). Also see the Sheet on
“Glycemic Control” on page 72.
-Ins and Outs: Important in patients with renal compromise.

-Physical Exam
-Derm:
-Wound Characteristics: There are several classification systems you need to know for describing wounds including:
-Wagner Classification
-University of Texas Health System Classification
-PEDIS Classification used by the Infectious Disease Society of America
-Liverpool Classification used by the Musculoskeletal Infectious Disease Society
-Acronym 3D MOBB (depth, diameter, drainage, measure, odor, base, border)
-Regardless of classification, you absolutely must document certain wound characteristics:
-Base: -Exact length, width and depth; consistency (ranging from red/granular to yellow/fibrotic to
black/necrotic. Estimate percentages for mixed bases).
-Depth—Probe to bone? [Grayson JAMA 1995. 89% positive predictive value for OM]. Also…
-Lavery LA. Probe-to-Bone Test for Diagnosing Diabetic Foot Osteomyelitis. Reliable or relic? Diabetes Care. Feb 2007; 30(2): 270-274.
-Wound Edges: -Consider hyperkeratotic, macerated, necrotic, clean, bleeding, epithelial, etc.
-Undermining? Tunneling?
-Drainage: -Consider serous, sanguinous, purulent (describe color), combination, etc.
-Mild, moderate, severe/heavy
-Describe any odor (This is probably Dr. Attinger’s most important variable in infection assessment!)
-Periwound skin: -Consider normal, erythematic (document/draw extent), streaking, stasis changes, trophic changes.
-Vascular:
-Describe extent (anatomic level) and nature (pitting vs. non-pitting) of any edema.
-Pulses (DP, PP, PT, Pop, and Fem every time)
-Always doppler if nonpalpable at each level
-CFT, Pedal Hair
-ABI: -Values >0.9 associated with good healing potential
-Values 0.5-0.9 associated with PVD and delayed healing
-Values <0.5 associated with ischemia and problematic healing
-Be wary of elevated values secondary to vessel calcification
-TcPO2: -Values >30mmHg associated with good healing potential [Mars M. Transcutaneous oxygen tension as a predictor of success
after an amputation. JBJS-Am. 1988; 70(9): 1429-30.]
-Values <20mmHg associated with microcirculatory problems and delayed/problematic healing.
-Absolute Pressures: -Should have 40mmHg at ankle and 20mmHg at the digits for healing potential.
-Absolute/Relative skin temperature: compare B/L (normal around 94° F).
-Any other relevant vascular testing.
-Neurologic:
-Include testing for sensory, motor and autonomic neuropathy
-Sensory testing:
-Posterior column: Vibratory, Proprioception
-Anterior column: Light touch (5.07 SWMF)
-Lateral column: Pain and temperature
-Motor testing:
-Expect intrinsic weakness with advanced neuropathy
-Manual Muscle Testing
-Spinal Reflexes (Achilles, Patellar, Babinski)
-Autonomic:
-Increase in skin temperature
-Lack of sweating leading to xerosis
-Any other relevant neurologic tests (you should have an awareness of Dellon’s work and the PSSD).
-Orthopedic:
-Document any/all foot deformities, especially osseous prominences.
-Expect intrinsic muscle weakness leading to digital deformities.
-Overall foot type
-Equinus
32
AJM Sheet: Specific Wound Classification Systems

-Wagner Classification: [Wagner FW: The dysvascular foot: a system of diagnosis and treatment. Foot Ankle 2: 64–122, 1981]
0: Pre-ulcerative area without open lesion
1: Superficial ulcer (partial/full thickness)
2: Ulcer deep to tendon, capsule, bone
3: Stage 2 with abscess, osteomyelitis or joint sepsis
4: Localized gangrene
5: Global foot gangrene

Modified with the following risk factors:


A: Neuropathic
B: Ischemic
C: Neuroischemic

-So an infected ulcer with localized gangrene and bone exposure on a fully sensate, ischemic foot is: Wagner 4B.

-University of Texas: [Lavery LA, Armstrong DG, Harkless LB: Classification of diabetic foot wounds. J Foot Ankle Surg 35:528–531, 1996]

0: 1: 2: 3:

A: No open lesion Superficial Wound Tendon/Capsule Bone/Joint

B: With infection With infection With infection With infection

C: Ischemic Ischemic Ischemic Ischemic

D: Infection/Ischemia Infection/Ischemia Infection/Ischemia Infection/Ischemia

-So an infected ulcer with localized gangrene and bone exposure on a fully sensate, ischemic foot is: UT-3D.

-Liverpool Classification System:

-Primary: -Neuropathic
-Ischemic
-Neuroischemic

-Modified with: -Uncomplicated


-Complicated (cellulitis, abscess, OM, etc.)

-PEDIS System: [Lipsky BA, et al. Diagnosis and Treatment of Diabetic Foot Infections. IDSA Guidelines. CID 2004; 39: 885-910].
-Recommended by the Infectious Disease Society of America.
-PEDIS is an acronym standing for perfusion (measure of vascular supply), extent/size, depth/tissue loss, infection,
and sensation.
-Each of the 5 categories is graded from 0 (minimal) to 2 (severe).
-Based on10-point scale with 10 being most serious ulcer with greatest difficulty in treatment.

33
AJM Sheet: Diabetic Infection Objective Laboratory Results: Basic

-Complete Blood Count (CBC) with Differential:


-Total Leukocyte Count (~4-10 x 10^3 leukocytes/ul)
-Leukocyte is a generalized term for any WBC including neutrophils/granulocytes, monocytes,
lymphocytes, eosinophils and basophils. So an increased leukocyte count can indicate a rise in any or all of these. This is the
reason why a differential is so important.
-Neutrophils/Granulocytes (Usually ~54%; increased >85%)
-Part of the humoral system.
-Phagocytic cells in the inflammatory process.
-Normally take 8-14 days to mature. Functionally last 1-2 days. Half-life 6 hours.
-Would be increased in an inflammatory state.
-PMNs: Mature neutrophils that you would expect to see in an infection.
-Band cells: Immature neutrophils. Presence indicates active, ongoing infection.
-A left shift is an increased neutrophil percentage in the presence of band cells.
-Monocytes (Usually ~6%)
-Phagocytic, bacteriocidal macrophages in the humoral system.
-Accumulate after neutrophils in acute infection.
-Presence indicates post-inflammatory state or chronic infection.
-Lymphocytes (Usually ~37%)
-Part of the cellular system.
-Produce immunoglobulins and express cellular immunity (T and B cells).
-Not normally increased in bone/soft tissue infections.
-Possibly increased in a foreign body reaction.
-Eosinophils (Usually ~2%)
-Part of the cellular system.
-Generally involved in allergic and immune responses.
-Develop in the same line as lymphocytes.
-Increased with acronym NAACP
-(Neoplasm, Allergy, Addison’s, Collagen vascular disorder, Parasites)
-Basophils (Usually ~0.5%)
-Part of cellular immunity.
-Involved with acute allergic responses and histamine release.

-Leukocytosis is an increased WBC. The absolute count tells you very little, but trending can be very
important. An increased leukocyte count indicates an increased level of inflammation, not necessarily
infection. Keep in mind that there are many other causes of leukocytosis besides infection.
-Drugs: Lithium, Corticosteroids
-Leukopenia is a decreased WBC. This could lead to a normal WBC in the presence of infection.
-Drugs: Methotrexate, Phenybutase, Dilantin, Salicylates
-[Armstrong DG. Leukocytosis is a poor indicator of acute osteomyelitis of the foot in DM. JFAS 1996 Jul-Aug; 35(4): 280-3.]

-Chem-7/Metabolic Panel
-Little information about specific infection, but insight into general health of patient.
-Generally:
-General increased concentrations: Dehydrated state
-Acidosis: Non-descript finding in infection
-Increased BUN: Dehydrated state
-BUN/Cr: Renal function which has antibiotic consequences
-Minerals (Ca, Mg, P)
-Abnormal in renal dysfunction with long term vascular consequences.
-Glucose, HbA1c
-Long-term effects of hyperglycemia discussed in pathogenesis section.
-HbA1C: Measure of glycosylated hemoglobin and long-term glucose control:
-1% equals approximately 20 glucose points (7% equals ~140ug/ul)
-Note that the stress of infection will probably cause a hyperglycemic state.
-H&H, Coags
-Essential to know if you are planning surgery.

34
AJM Sheet: Diabetic Infection Objective Laboratory Results: Advanced

-Erythrocyte Sedimentation Rate (ESR)


-Normal: <20mm/hr Moderate elevation: 20-60mm/hr Severe elevation: >60mm/hr
-Analyzed using the Westergren method, which measures the distance erythrocytes fall in one hour in a vertical
column of anti-coagulated blood under the influence of gravity.
-Sensitive, but not specific for infection as it is increased in any inflammatory state with increased fibrinogen.
-Also elevated in: Pregnancy, DM, ESRD, CAD, CVD, Malignancy, Age, etc.
-[Karr JC. The diagnosis of osteomyelitis in diabetes using ESR. JAPMA 2002 May; 95(5): 314.]
-[Lipsky BA. Bone of contention: diagnosing diabetic foot osteomyelitis. Clin Infect Dis. 2008 Aug; 47(4): 528-30.]

-C-Reactive Protein (CRP)


-Normal: 0-0.6mg/dl
-Measures a liver protein only present in acute inflammation (not normally found at all).
-Sensitive, but not specific for infection.
-Also elevated in: RA, Malignancy, MI, SLE, Pregnancy, etc.
-More expensive and technically difficult to perform.
-[Jeandrot A. Serum procalcitonin and CRP concentrations to distinguish mildly infected from non-infected diabetic foot ulcers: a pilot study.
Diabetologia. 2008 Feb; 51(2): 347-52.]

-Nutrition Analysis
-Albumin:
-Normal Value: 3.6-5g/dl
-Value decreased with inflammation and malnutrition.
-Transport protein in liver with important functions in catabolism.
-Pre-albumin
-Normal Value: 19-36 mg/dL
-Not covered well in PRISM, but check out [Arnold M. Nutrition and Wound Healing. Plast Reconstr Surg. 2006 Jun; 117(7 Suppl):
42S-58S.]

-Wound Culture and Sensitivity


-Wound cultures are still a hotly debated topic because of controversies regarding contamination, colonization and
defensive medicine. Suffice to say that swab cultures are easily contaminated by normal cutaneous flora, so should be taken
as deeply as possible without surface contamination. The ideal situation is a deep wound specimen (not just a swab)
following incision and drainage with pulse lavage before beginning antibiotic therapy.
-Gram Stain (results usually within 24 hours)
-PMNs if present: Do not overlook! Presence indicates inflammation.
-Presence of any organism: essentially irrelevant.
-Preliminary (results usually within 48 hours)
-Gram stain nature (positive/negative) and shape (cocci/bacillus) of any organism
-See chart of common organisms on next page
-Clues you into organism
-Continue Abx unless you are really off-base
-Final (results usually within 72 hours)
-Should always get sensitivities.
-Allows for conclusive Abx planning.

-Blood Cultures
-Should be drawn from 2 sites; 20 minutes apart.
-Indicates bacteremia/septicemia

-Bone Biopsy
-Gold standard for diagnosis of osteomyelitis (discussed further later)

-Consider EKG and CXR if patient is a surgical candidate.

35
AJM Sheet: Gram Stain Results with Common Infective Agents:

-Aerobic Gram Positive Cocci


Staph aureus Strept pyogenes (Group A)
MRSA Strept agalactiae (Group B)
Staph epi Strept bovis (Group D)
MRSE Strept Viridans
Enterococcus
VRE

-Anaerobic Gram Positive Cocci


Peptostreptococcus

-Aerobic Gram Positive Rods


Bacillus anthracis
Corynebacterium diphtheriae
Listeria Monocytogenes

-Anaerobic Gram Positive Rods


Clostridium perfringens
Clostridium difficile
Clostridium tetani

-Aerobic Gram Negative Rods


Pseudomonas Shigella
E. coli Salmonella
Enterobacter Klebsiella
Proteus Serratia
Vibrio E. Corrodens
Y. pestis P. multicide

-Anaerobic Gram Negative Rods


Bacteroides fragilis

-Aerobic Gram Negative Cocci


Neisseria

-Spirochetes
Treponium pallidum
Borrelia burgdorferi

36
AJM Sheet: Imaging in Diabetic Foot Infections

-Plain Film Radiographs


-Soft Tissue: Infection characterized by radiolucent area. One should be able to see a soft tissue deficit if an ulcer is present. It is very important
to rule out emphysema (gas in the tissues) with a plain film. Can also appreciate soft tissue edema.
-Osseous Tissue:
-Early Osteomyelitis signs: No reliable ones. Possible rarefaction and periostitis.
-Subacute OM signs: Brodie’s abscess (lytic lesion surrounded by sclerotic rim).
-Chronic OM: Lysis, Malformation, Involucrum, Cloaca, Sequestra.
-Plain film radiographs are 67% specific, 60% sensitive for OM (Termaat, JBJS 2005)

-MRI
-Cellulitis: T1: Diffuse and infiltrative decreased signal intensity as inflammation replaces fat.
T2 and STIR: Increased signal intensity.

-Abscess: T1/T2/STIR: Homogeneous increased signal intensity.


-Note that pus/necrotic tissue has a decreased intensity compared to inflammatory fluid.

-OM: T1: Decreased signal intensity, cortical lysis and intramedullary changes.
-Increased signal intensity in known OM indicates healing as fat infiltrates.
T2: Increased signal intensity, cortical lysis, and intramedullary changes.
-Rim sign: thin layer of active infection surrounding normal bone.
-60% Specificity, 85% sensitivity per Termaat.

-Bone Scans
-A radio-isotope is injected into the patient and imaged at specific intervals.
-Phases: -Immediate Angiogram (1-3sec): Essentially an arteriogram.
-Blood Pool (3-5min): Demonstrates blood pooling in capillaries and veins.
-Delayed (2-4 hours): Increasingly specific to activity patterns and pathology.
-4th Phase: Increasingly specific to activity patterns and pathology.

-Technetium-99 Bone Scan


-Binds to calcium hydroxyapatite and measures osteoblast/osteoclast activity.
-Half-life: 6 hours
-Excreted through kidneys which will show homogenous control signal.
-Mucomyst 600mg PO q12 day before and of surgery as renal ppx.
-Cellulitis: Focal uptake in blood pool; Negative in delayed phase.
-OM: Diffuse uptake in blood pool; Hot increased uptake in delayed phases.
-45% Specificity; 86% sensitivity per Termaat.

-WBC Scans
-Same principles and phases as bone scan, but WBCs are tagged and followed instead.

-Gallium-67 Citrate
-Uptake by siderophore complex (direct bacteria) and lactoferrin (protein-released by bacteria)
-Scan taken 48-72 hours after injection or done in triphasic manner.
-Has longer half-life
-42% Specificity; 80% Sensitivity per Termaat.

-Indium-111 Oxime
-WBCs isolated from blood sample, labeled and re-injected.
-Scan at 24 hours. Half life: 67 hours.
-Predominantly uptaken by neutrophils, so it demonstrates acute infections better than chronic infections.

-Technetium-99m HMPAO
-WBCs isolated from blood samples, tagged and re-injected. Scan at 3 hours.
-Tagged molecule is HMPAO (hexamethylpropyleneamine oxime)
-Technically easier with less radiation than indium.

-Technetium-99m Sulfur Colloid Marrow Scan


-Specific for bone marrow and neutrophil production
-Has shown promise in differentiating OM from Charcot

-Combination Sequential Technetium-Gallium Scans


-Scan at 4 hours, then at 48-72 hours. Based on half-lives.
-Increased specificity for infection if gallium has higher uptake then technetium.
-Can use any other combination.

-Computed Tomography (CT scans)


-Radiograph altered by computer to highlight specific “windows”. You can isolate soft tissue or different aspects of bone, for example.
-Soft tissue infection: Exact locations and anatomy of abnormal soft tissue density.
-OM: Increased density in the marrow.
-CT scans can be combined with contrast.

-Positron Emission Tomography (PET scans)


-A tracer is injected/inhaled into the patient which releases radioactive positrons. The positrons collide with electrons and produce gamma rays.
->90% Sensitivity and Specificity per Termaat (best in study).

37
AJM Sheet: Diabetic Foot Ulcer Pathogenesis

-The pathogenesis of the diabetic foot ulcer can be described via three mechanisms: neuropathy, trauma and impaired
healing.

-Neuropathy
-30-50% of diabetics have some form of sensory, motor and/or autonomic neuropathy.
-Sorbitol accumulation in Schwann cells leads to hyperosmolarity of the nerve cells leading to swelling and
cellular lysis. This leads to decreased nerve signal conduction. Microvascular damage to the nerve (described later) also
impairs healing of the damaged nerve.
-Sensory Neuropathy
-Loss of light touch/protective sensation (anterior spinothalamic tract)
-Loss of vibratory/proprioception mechanisms (posterior tract)
-Loss of pain/temperature sensation (lateral tracts)
-The patient has no warning of current, developing or impending trauma.
-Motor Neuropathy
-“Intrinsic Minus” foot-type with wasting of the intrinsic muscles and extensor substitution.
-Undetected excess plantar pressures develop.
-Autonomic Neuropathy
-Damage occurs in the sympathetic ganglion
-AV shunting occurs with global LE edema not relieved by diuretics or elevation.
-Increased skin temperature predisposes to ulceration (Armstrong)
-Decreased sweating leads to xerosis and fissuring (portal for infection)

-Trauma leading to Ulceration


-Abnormal anatomy: extrinsic and intrinsic abnormalities secondary to motor neuropathy and glycosylation.
-Decreased joint mobility: secondary to non-enzymatic glycosylation and excess collagen cross-linking of tendons,
ligaments, joint capsules (especially at the STJ and the MTPJ).
-Equinus: Increased cross-linking of collagen in the Achilles tendon (leads to increased forefoot pressures).
-Skin stiffness: secondary to glycosylation of keratin.
-Intrinsic skin weakness: trophic changes associated with PVD.
-All lead to increased plantar pressures, which is the driving force behind ulceration.

-Impaired Wound Healing


-Can be thought of as increased inflammation, decreased vasculature and decreased catabolism.
-Increased Inflammation
-The inflammatory phase of the healing process actually lasts longer than necessary.
-Inflammation initially not as effective due to decreased leukocyte adhesion and morphologic changes to
the macrophages.
-Prolongation occurs due to decreased chemotaxis of growth factors and cytokines.
-MMPs increase their activity and continue to produce an “inflammatory soup.”
-All contribute to a wound “stuck” in the inflammatory phase.
-Decreased Vasculature
-Macroangiopathy: Atherosclerotic obstructive disease of large vessels due to LDL oxidation.
-Microangiopathy: Thickened basement membrane decreases diffusion at capillary level.
-Mechanism behind neuropathy, nephropathy and retinopathy
-Think of it in terms of a decreased TcPO2
-Endothelial dysfunction:
-Decreased NO and prostaglandin to promote vasodilation
-Decreased smooth muscle cell relaxation to promote vasodilation
-AV shunting secondary to autonomic neuropathy
-Decreased vasodilation and membrane permeability in response to trauma/damage/inflammation:
-Usually regulated by substance P and vasomodulators from damaged cells and nociceptors.
-Overall leads to a “sluggish” vasculature with decreased inflow, diffusion, outflow and angiogenesis.
-Decreased Catabolism
-Decreased collagen synthesis, both in peptide production and post-translational modification
-Morphologic changes to keratinocytes
-Decreased angiogenesis
Further Reading:
-Shaw JE, Boulton AJ. The pathogenesis of diabetic foot problems: an overview. Diabetes. 1997 Sep; 46 Sep; Suppl 2:S58-61.
-Rathur HM, Boulton AJ. The diabetic foot. Clin Dermatol. 2007 Jan-Feb; 25(1): 109-20.
-Boulton AJ, et al. The global burden of diabetic foot disease. Lancet. 2005 Nov 12; 366(9498): 1719-24.

38
AJM Sheet: Diabetic Foot Infection Functional Anatomy
From: Essential Questions for Surgical Intervention of Diabetic Foot Infections (http://www.podiatrytoday.com/article/8134)

Dedicating yourself to the side of limb salvage in the fight against diabetic foot disease is a demanding and
personally challenging enterprise. In the face of infection, it often seems as though all variables are against the surgeon and
the patient as together, you struggle against proximal amputation and limb loss. In fact, it often appears as though the only
constant is the unpredictability of the disease progression. But one constant always on the side of the surgeon is anatomic
knowledge. The infection can only work with the anatomy that it is given, and this is certainly something that can be used to
your advantage. Your expert knowledge in lower extremity anatomy is one of the most valuable tools that you have in your
fight. It is a constant, and it is predictable.
There are different anatomic paradigms that must be considered in terms of the evaluation of the infection source.
Certainly depth is one of these paradigms. Absolute depth measurements offer very little clinical information when
compared to a functional view of depth from the surgical layers of dissection. An infection should be evaluated in terms of
whether it extends through the dermis, superficial fascia, deep fascia, musculotendinous structures or to the level of bone
(Table 1). From this general information, specific anatomic structures can then be identified as being within the path of the
infection.
Infections tend to develop and travel along the path of least resistance. This implies that an infection will stay
within the potential space of a given surgical layer or plantar compartment before extravasation into another layer or
compartment. Often, this involves proximal extension along the relatively avascular tendon sheaths or fascial planes between
muscular layers. The studies that have been used to define the number and boundaries of plantar foot compartments have
also given information about relatively consistent fascial clefts where communication between different layers and
compartments is likely. These have involved pressurized injection imaging studies where a known compartment is infiltrated
with a contrast medium and the extravasation into other compartments can be mapped. The findings of these studies are
summarized in Table 2. These communications are obviously numerous and complex. The important concept to realize is
that an infection is likely to initially develop within the potential space of a single layer or compartment. There is a tendency
for the infection to move proximally before communicating with another layer or compartment. Note however, that patterns
of communication are present along known anatomic structures such as tendons and neurovascular structures to each of the
other compartments, as well as the dorsum of the foot and plantar superficial fascia. Intra-operative investigation of an
infection should focus on these structures to trace the extent of plantar involvement. Also note the majority of these
communications are found in the forefoot around MPJ level, so distal infections have an increased likelihood of multi-
compartment involvement.

Table 1: Surgical Layers of Dissection Used for Diabetic Ulcer Depth Measurement

Skin
Superficial Fascia
-First Dissection Interval containing superficial neurovascular structures
Deep Fascia
-Second Dissection Interval containing muscular and deep neurovascular structures
Periosteum
-Third Dissection Interval
Bone

Table 2: Intercompartmental Communications

Medial Compartment Central Compartment


To Central Compartment via: To Medial Compartment via:
-Adductor Hallucis tendon -Adductor Hallucis tendon
-Flexor Hallucis Longus tendon -FHL Tendon
-Peroneus Longus tendon -PL Tendon
-Neurovascular structures penetrating the medial IM septum -NV structures penetrating medial IM septum
To Distal Deep Leg via: To Lateral Compartment via:
-Flexor Hallucis Longus tendon -Long flexor tendon to 5th digit
-Short flexor tendon to 5th digit
Lateral Compartment: -Lumbrical muscle/tendon to 5th digit
To Central Compartment via: -Plantar interosseous muscle to 5th digit
-Long flexor tendon to 5th digit -PL tendon
-Short flexor tendon to 5th digit -NV structures penetrating lateral IM septum
-Lumbrical muscle to 5th digit To Distal Deep Leg via:
-Plantar interosseous muscle to 5th digit -FHL tendon
-Peroneus Longus tendon -FDL tendon
-Neurovascular structures penetrating lateral IM septum To Dorsum of Foot via:
To Dorsal Structures -Interosseous muscles
To Plantar Superficial Fascia -MPJ communications
To Plantar Superficial Fascia

39
AJM Sheet: Osteomyelitis General

-Osteomyelitis is a complicated issue dealing with diabetic foot infections both in diagnosis and treatment. However, there
are several definitions, classification systems, diagnostic modalities and treatment tenets that you should be aware of.

-Definitions per Resnick:


-Periostitis: Inflammation of the periosteum
-Osteitis: Inflammation of the cortex
-Osteomyelitis: Inflammation of the medullary canal
-Sequestrum: piece of dead bone floating in pus/inflammation
-Involucrum: sheath of bone surrounding pus/inflammation
-Cloaca: tract through an involucrum
-Brodie’s Abscess (1832): Chronic abscess in bone surrounded by sclerosis
-Sclerosing OM of Garre: low grade inflammatory condition

-Waldvogel and Lew [Osteomyelitis. NEJM. 1997; 336(14): 999-1007.]


-The classic Waldvogel and Lew article is NOT a classification, rather a serious of definitions. However, it can be
turned into a stratified classification based on their definitions.
-Acute Osteomyelitis: Systemic clinical signs of infection
-Chronic Osteomyelitis: Subacute clinical signs of infection
-Contiguous/Direct Extension: spread of infection to bone from exogenous source or adjacent tissue.
This can be described as an “outside-in” spread invading the cortex and proceeding to the medullary canal.
-Hematogenous Spread: Infective agent reaches medullary canal of bone from the vascular supply. This
can be described as an “inside-out” infection invading the medullary canal first and spreading to the cortex.
-Vascular Impairment: Decreases the effectiveness of the inflammatory response and Abx delivery.

-Others have done a little better job of differentiating acute vs. chronic OM:
-Weiland: Describes chronic OM as lasting > 6months.
-Schauwecker: Describes chronic OM as lasting > 6 weeks and one failed episode of tx.

-AJM has turned these definitions into a classification system that made more sense to him:
-Acute Osteomyelitis
-Contiguous/Direct Extension
-No PVD
-PVD
-Hematogenous Spread
-No PVD
-PVD
-Chronic Osteomyelitis
-Contiguous/Direct Extension
-No PVD
-PVD
-Hematogenous Spread
-No PVD
-PVD

-Cierny-Mader-Penninck Classification [A clinical staging system for adult osteomyelitis. CORR. 2003; (414): 7-24.]
-This is described as a classification, but doesn’t make much sense to AJM.

-Anatomic Stage
1: Medullary: infection of only the medullary canal (Stage 1)
2: Superficial: infection of only the superficial cortex (Stage 2)
3: Localized: infection of only the cortex (Stage 3)
4: Diffuse: infection of both the cortex and medullary canal (Stage 4)
-Physiologic Stage
A: Normal Host
Bs: Compromised Host with systemic risk factors (eg DM)
Bl: Compromised Host with local risk factors (eg smoking)
C: Treatment worse than the disease

-So a smoking DM patient with infection of only the superficial portion of the cortex is: 2Bs
-Obviously much more work needs to be done on the topic of osteomyelitis classification.

40
AJM Sheet: Osteomyelitis Diagnosis and Treatment

-Subjective Findings (See Diabetic Infection Work-up)


-Objective Findings (Diabetic Infection Work-up)
-Probing to bone 89% positive predictive value (Grayson JAMA 1995).
-Gold Standard: Bone biopsy. This is ideally performed when the patient has been free of antibiotics for 2 weeks.
-Imaging Studies:
-Review plain films, MRI, Bone Scans, WBC scans, CT, PET in Diabetic Infection Imaging Sheet.
-These all add evidence, but are rarely exclusively diagnostic.
-Blood Cultures: Hematogenous OM diagnosed with positive BCx and positive bone scan.

Treatment
-There is much controversy regarding long-term Abx (PO vs. IV vs. PMMA) vs. Surgical Debridement.
-The Cierny-Mader Classification makes some general recommendations:
-CM Stage 1: 2 weeks IV + 2-4 weeks PO Abx
-CM Stage 2: Surgical Debridement + 2 weeks IV
-CM Stage 3&4: Surgical Debridement + 4-6 weeks IV Abx

-Antibiotic administration options


-Long-term Abx (4-8 weeks) is a conservative option because many people believe you can never cure OM and that
it can reactivate at any time for years to come.
-PO
-Doxycycline and Ciprofloxacin are reputed to have the best bone penetration.
-Most ID docs would never substitute coverage for bone penetration.
-Your Abx choices should be culture driven.
-IV
-Culture driven
-Access options: IV, PICC, Infusion pump, etc.
-PMMA beads
-PMMA: polymethylmethacrylate
-PMMA is a combination of monomer (liquid) and polymer (powder).
-Comes in 20, 40 and 60g packets.
-7% elusion in the first 24 hours with activity noted for 14 days.
-Demonstrates exponential release.
-Cierny proposes a 1:5 ratio of Abx:PMMA. Another common standard is 4-8g:40-60g.
-Increased Abx means increased elution, but decreased bead hardening.
-Smaller beads means increased surface area and increased elution.
-The Abx must be heat-labile
-Gentamycin, Tobramycin, Vancomycin, Ticarcillin, Cefazolin, Moxalactam, Cefotaxime

41
AJM Sheet: Charcot Neuroarthropathy

-Definition: Neuropathic osteoarthropathy first described by Musgrave in 1703 and named for JM Charcot in 1868.
-Pathogenesis:
-Neurovascular/French Theory/Theory of Charcot
-Trophic centers in the anterior horn of the spinal cord maintain nutrition to joints.
-Trauma to these trophic centers leads to increased blood flow and osteoclastic activity.
-Evidence for the Neurovascular Theory:
-Autonomic neuropathy in DM leads to increased AV shunting, edema and skin temperature.
-Boulton: Increased PO2 in venous system of Charcot pts (63mmHg vs. 46mmHg)
-Shows increased perfusion in neuropathic diabetics
-Edmonds: Increased blood velocity in neuropathic diabetics
-Young: Decreased bone density in patients with decreased nerve conduction velocities
-Cundy: Decreased bone density in Charcot patients
-Gough: Increased osteoclastic activity in Charcot patients
-Neurotraumatic/German Theory/Theory of Virchow and Volkmann
-Repeated trauma from biomechanical stresses during ambulation on an insensate foot.
-Evidence for the Neurotraumatic Theory:
-Eloesser and Johnson: Trauma is the necessary predisposing factor, and not underlying bone weakness, to
create Charcot changes in a neuropathic limb.
-Common sense
-Two opposing, fighting theories? Probably a little bit of both.

-Etiology: Anything that causes neuropathy!


-First described: Tabes Dorsalis (Charcot 1868)
-Most common: DM
-3 most common: DM, Syringomyelia (longitudinal cavities lined by dense tissue), and Tabes Dorsalis
-C: Myelomeningocele, Spina Bifida, CMT, MS, CP, Syringomyelia, Congenital insensitivity
M: DM, Alcoholic neuropathy, Uremia, Pernicous Anemia
I: Tabes Dorsalis, Polio, Leprosy, TB
N: Tumors in brain, spinal cord, peripheral nerve
T: Trauma to brain, spinal cord, peripheral nerve
D: Indomethacin, Intra-articular corticosteroids, phenylbutazone

-DDx: OM, AVN, inflammatory arthritis, PVS, septic arthritis, CPPD, neoplasm, etc.

-Clinical Findings: -Presents similar to infection


-Red, hot, swollen, deformed foot +/- pain
-Neuropathic
-Readily available pulses (often described as bounding)

-Radiographic Findings:
-Atrophic: -With osteopenia, pencil&cup deformities, resorption of bone ends
-Without osteophytes, sclerosis, fragmentation, soft tissue debris
-Hypertrophic: -With joint space narrowing, fractures, fragmentation, ST debris, periosteal rxn, subluxation
-Without osteoporosis
-Be aware of both types.

-Classification Systems (described in detail on next page)


-Eichenholtz Classification (1966)
-Brodsky Classification
-Schon Classification

-Treatment
-Acute: -Strict and immediate NWB and immobilization for 12-16 weeks.
-Edema control (Jones cast, ACE inhibitors, Diuretics, Posterior splint, Elevation, Ex Fix, etc.)
-Education and family support
-FXR every 4-6 weeks with relatively few cast changes
-Transition: -Transition to WB (CAM walker, CROW, Bracing, MAFO, Shoes, etc.)
-Permanent: -Surgical correction of underlying deformity
-Consider TAL, Arthrodesis, Wedging osteotomies, Amputation
-Adjunctive: -Bone stimulators
-Bisphosphonates: -Pamidronate (Aredia): 60-90mg over 24h. 3 doses in 2 weeks.
-Alendronate (Fosamax): 5mg PO q24h.

42
AJM Sheet: Charcot Classifications

-Eichenholtz Classification (1966)


-Based on plain film radiographic findings
-Originally described Stages 13, but Stage 0 added later (Yu given credit, but really Schon).
-[Yu GV, Hudson JR. Evaluation and treatment of stage 0 Charcot’s neuroarthropathy of the foot and ankle. JAPMA. 2002; 92(4): 210-20.]
-Stage 0: High risk pre-Charcot
-Radiograph: Unremarkable. Maybe increased ST density, bone flecks or change in foot architecture.
-Clinical: Sudden onset of non-pitting edema, erythema, calor, +/- pain, bounding pulses, intrinsic atrophy.
-Normal skin temp: 94°F; can increase by 12°
-Uptake in all three phases of Tc-99 bone scan
-Stage 1: Acute/Developmental
-Radiograph: Capsular distention, fragmentation, debris, subluxation
-Clinical: Red, hot, swollen foot with joint laxity
-Stage 2: Coalescence
-Radiograph: Sclerosis, resorption of debris, fusion
-Clinical: Subjectively decreased red, hot, swollen
-Stage 3: Reconstruction
-Radiograph: Decreased sclerosis (with increased vascularity) and remodeling
-Clinical: Decreased joint mobility with increased stabilization

-Brodsky Classification (1992)


-Describes location of deformity

-Type 1: Lisfranc joint (27-60% incidence)


-Type 2: Chopart’s joint and STJ (30-35% incidence)
-Type 3A: Ankle joint (9% incidence)
-Type 3B: Posterior calcaneus
-Type 4: Multiple combinations of above
-Type 5: The forefoot

-Schon Classification [Charcot neuroarthropathy of the foot and ankle. CORR. 1998; 349: 116-131.]
-Describes location and severity of condition
I: Lisfranc Pattern
-AC with increasing deformity to medial rockerbottom and ulceration.
II: Naviculocunieform Pattern
-AC with increasing deformity to lateral rockerbottom and ulceration.
III: Perinavicular Pattern
-AC with lateral rockerbottom, Talar AVN and ulceration.
IV: Transverse Tarsal Pattern
-AC with increasing deformity to central rockerbottom and ulceration.

43
AJM Sheet: Differentiating Osteomyelitis from Charcot

-Please keep in mind that these are not mutually exclusive and both can be present!
-These are just general guidelines and many people will vehemently argue them.

-The gold standard is a bone biopsy which would show infection in OM and not in Charcot.

Subjective
-OM: Constitutional signs and symptoms of infection, infectious risk factors, history of infection.
-Charcot: Uncontrolled DM, history of Charcot, history of recent trauma.

Objective
-OM: Necrosis, pustular drainage, elevated white count, cultures, positive bone biopsy.
-Charcot: Increased joint laxity, non-pitting edema, bounding pulses, rockerbottom deformity, negative bone biopsy.

Imaging
-Not enough evidence yet, but some believe that OM is positive on bone scans and WBC scans for greater than 24 hours
whereas Charcot neuroarthropathy is only positive during the first 24 hours.
-The Tc99 Sulfur Colloid scan would also theoretically be positive for infection, but not for Charcot.

-Not too much here, but check out some further reading:
-Soysal N, et al. Differential diagnosis of Charcot arthropathy and osteomyelitis. Neuro Endocrinol Lett. 2007 Oct; 28(5): 556-559.
-Shank CF, Feibel JB. Osteomyelitis in the diabetic foot: diagnosis and management. Foot Ankle Clin. 2006 Dec; 11(4): 775-89.
-Ledermann HP, Morrison WB. Differential diagnosis of pedal osteomyelitis and diabetic neuroarthropathy: MR Imaging. Semin Musculoskelet Radiol.
2005 Sep; 9(3): 272-83.
-Berendt AR, Lipsky B. Is this bone infected or not? Differentiating neuron-osteoarthropathy from osteomyelitis in the diabetic foot. Curr Diab Rep. 2004
Dec; 4(6): 424-9.
-Yu GV, Hudson JR. Evaluation and treatment of stage 0 Charcot’s neuroarthropathy of the foot and ankle. J Am Podiatr Med Assoc. 2002 Apr; 92(4): 210-
20.
-Schon LC, et al. Charcot neuroarthropathy of the foot and ankle. Clin Orthop Relat Res. 1998 Apr;(349): 116-31.
-Berendt AT, Peters EJ, et al. Diabetic foot osteomyelitis: a progress report on diagnosis and a systemic review of treatment. Diabetes Metab Res Rev. 2008;
24(S1): S145-S161.

44
AJM Sheet: Common Situational Bugs

-Cellulitis with an open wound: -SA (if no streaking present)


-Strept (with streaking and palpable border)
-Usually monomicrobial

-Infected ulcer in Abx naïve pt: -SA


-Strept
-Usually polymicrobial

-Chronically infected ulcer in Abx naïve pt: -SA


-Strept
-Enterobacter
-Usually polymicrobial

-Macerated infected ulcer: -Pseudomonas


-Usually polymicrobial

-Chronic, non-healing ulcer with prolonged Abx therapy: -SA -MRSA


-Staph epi
-Enterococci -VRE
-Diptheroids (Corynebacterium)
-Enterobacter
-Pseudomonas
-Extended GNR
-Usually polymicrobial

-Fetid Foot with necrosis and gangrene: -Resistant Gram positive cocci
-Mixed GNR
-Anaerobes
-Polymicrobial

-Osteomyelitis with hemodialysis: -SA


-Enterobacter
-Pseudomonas

-Osteomyelitis with IVDA: -SA


-Enterobacter
-Pseudomonas

-Osteomyelitis with Decubitus Ulcer: -Gram Negatives

-Osteomyelitis with hemoglobulinopathy: -Salmonella

-Human mouth pathogens (HACEK): -Haemophilus, Actinobacillus, Cardiobacterium hominis, Eikenella corrodens, Kingella
kingae

-Water exposure: -Vibrio


-Aeromonas hydrophila
-Mycobacterium

-Puncture through a shoe: -Pseudomonas

-Any dirt/soil: -Clostridium

-Cat bite: -Pasteurella multocida -Dog bite: -Strept viridans, Capnocytophaga canimorsus

-Immunocompromised pt: -Gram negatives

-Septic bursitis: -SA -Gas gangrene: -Clostridium

-Post-op infection following implant: -Staph epi

-Fruity odor/green hue: -Pseudomonas -Foul smelling discharge: -Anaerobes

-Creamy yellow discharge: -SA -White discharge: -Staph epi

45
AJM Sheet: Empiric Antibiotic Choices

Generalized Gram Positive Coverage:

-2nd Generation PCN -2nd Generation Quinolones -Synercid


-4th Generation PCN -Macrolides -Rifampin
-1st Generation Cephs -Bactrim
-2nd Generation Cephs -Vancomycin
-Carbapenems -Clindamycin
-Tetracyclines -Zyvox

Generalized MRSA Coverage:

-Vancomycin -Synercid
-Clindamycin -Bactrim/Rifampin
-Zyvox -Cubicin

Generalized Gram Negative Coverage:

-3rd Generation PCN -2nd Generation Quinolones


-4th Generation PCN -3rd Generation Quinolones
-Carbapenems -4th Generation Quinolones
-Tetracycline -Bactrim
-Aztreonam

Generalized Pseudomonas Coverage:

-Cephalosporins x 3 (Fortaz, Cefobid, Maxipime)


-PCN x 2 (Zosyn, Timentin)
-Aminoglycosides
-Primaxin
-Quinolones
-Aztreonam

Generalized Anaerobes:

-4th Generation PCN -Aminoglycosides


-1st Generation Cephs -Carbapenems
-2nd Generation Cephs -4th Generation Quinolones
-3rd Generation Cephs -Clindamycin
-4th Generation Cephs -Flagyl

46
AJM Sheet: IDSA Empiric Recommendations

-from Lipsky BA, et al. Diagnosis and Treatment of Diabetic Foot Infections. IDSA Guidelines. CID 2004; 39: 885-910.

-Uninfected Wound
-Definition: No purulence, inflammatory manifestations, or systemic manifestations
-Empiric Therapy: None

-Mildly Infected Wound


-Definition: -2+ Manifestations of Infection (purulence, induration or erythema/pain/warmth)
-<2cm of erythema
-Limited to skin and subcutaneous tissue
-No systemic complaints

-Empiric Therapy Recommendations:


-2-PCN -Bactrim
-Clinda -Augmentin
-Keflex -Levo

-Moderately Infected Wound


-Definition: -As above, in a systemically/metabolically stable patient PLUS
->2cm cellulitis OR streaking OR involvement of deep tissue

-Empiric Therapy Recommendations:


-Bactrim -Invanz
-Augmentin -Ceftin + Flagyl
-Levo -Timentin
-2-Ceph -Zosyn
-3-Ceph -Levo + Clinda
-Daptomycin + Aztreonam -Cipro + Clinda
-Zyvox + Aztreonam

-Severely Infected Wound


-Definition: -Infection as above in a patient with systemic toxicity and metabolic instability

-Empiric Therapy Recommendations:


-Primaxin -Vanco + Fortaz
-Zosyn -Vanco + Fortaz + Flagyl
-Cipro + Clinda
-Levo + Clinda

-If MRSA is likely:


-Zyvox -Vanco + Fortaz
-Zyvox + Aztreonam -Vanco + Fortaz + Flagyl
-Daptomycin
-Daptomycin + Aztreonam

-To cover all bases:


-Vanco + Aztreonam + Flagyl

47
AJM Sheet: Common Infective Agents with DOC:

DOC Alternatives
-Aerobic Gram Positive Cocci
Staph aureus 1-Ceph Vanco, Clinda, Azithromycin
MRSA Vanco Bactrim, Cubicin, Zyvox, Clinda
Staph epi 2-PCN 4-PCN, 1,2-Ceph, Vanco
MRSE Vanco Zyvox, Cubicin, Synercid
Enterococcus 3-PCN Vanco, Tetracyclines, Quinolones
VRE Linezolid Macrobid, Cubicin, Chloramphenicol
Strept pyogenes (Group A) 3-PCN 4-PCN, 1,2-Ceph, Vanco, Clinda
Strept agalactiae (Group B) 3-PCN 4-PCN, 1,2-Ceph, Vanco, Clinda
Strept bovis (Group D) 3-PCN 4-PCN, 1,2-Ceph, Vanco, Clinda
Strept Viridans 3-PCN 4-PCN, 1,2-Ceph, Vanco, Clinda

-Anaerobic Gram Positive Cocci


Peptostreptococcus Clinda 3-PCN, 4-PCN, Carbapenems

-Aerobic Gram Positive Rods


Bacillus anthracis Cipro 3-PCN, Vanco, Clinda
Corynebacterium diphtheriae Macrolide Clinda,
Listeria Monocytogenes 3-PCN Vanco, Bactrim, Carbapenems

-Anaerobic Gram Positive Rods


Clostridium perfringens Ertapenam Vanco, Clinda, 4-PCN, Tetracyclines
Clostridium difficile Flagyl Vanco
Clostridium tetani Clinda Flagyl

-Aerobic Gram Negative Rods


Pseudomonas Zosyn 1,2-Quin, Aztreonam, Primaxin
E. coli 3-Ceph 4-PCN, Bactrim, Quinolones
Enterobacter Bactrim Quinolone, Aztreonam, Carbapenems
Proteus 3-PCN 3-Ceph, 4-PCN, Bactrim, Quinolones
Vibrio Tetracyclines Bactrim, Cipro
Y. pestis Aminoglycosides Bactrim, Cipro
Shigella Cipro Bactrim, Amp, 4-PCN
Salmonella Cipro 3-PCN, 4-PCN, Bactrim
Klebsiella 3-Ceph 4-PCN, Bactrim, 2-Quin, Aminoglycosides
Serratia 3-Ceph Zosyn, Bactrim, Aztreonam, Quin
E. Corrodens Augmentin Tetracyclines
P. multocida Doxycycline Bactrim, 3-PCN

-Anaerobic Gram Negative Rods


Bacteroides fragilis Ertapenam Clinda, Flagyl

-Aerobic Gram Negative Cocci


Neisseria Rocephin 3-PCN, Quinolones

-Spirochetes
Treponium pallidum 1-PCN Tetracyclines, Macrolides
Borrelia burgdorferi 1-PCN Amox, Macrolides

48
AJM Sheet: Antibiotics/Drugs of Choice

1. Staph Aureus
-PO: Keflex -500mg PO tid or 750mg PO bid
Clindamycin -300mg PO qid
Zithromycin -500mg PO day 1, 250mg PO days 2-5
-IV: Ancef -1g IV q8
Vancomycin -1g IV q12
Clindamycin -600mg IV q8

2. Streptococcus
-PO: Keflex -500mg PO tid or 750mg PO bid
Clindamycin -300mg PO qid
-IV: Ancef -1g IV q8
Vancomycin -1g IV q12
Clindamycin -600mg IV q8

3. MRSA
-IV: Vancomycin -1g IV q12
-PO: Bactrim -1 tablet PO bid
Rifampin 300mg + Minocycline 100mg PO bid

4. Enterococcus
-PO: Amoxicillin -250-500mg tid
Augmentin -875mg bid or 500mg tid (or bid)
Zyvox -600mg PO bid
-IV: Vancomycin -1g IV q12
Zyvox -600mg IVq12

5. VRA/VRE
-PO: Zyvox -600mg PO bid
-IV: Zyvox -600mg IV q12
Synercid -7.5mg/kg/hr over 1 hour q12

6. Pseudomonas
-PO: Ciprofloxacin -250-750mg PO bid
-IV: Ciprofloxacin -400mg IV q12
Fortaz -2g IV q12
Aztreonam -1g IV q8

7. E.coli, Proteus
-PO: Keflex -500mg PO tid or 750mg PO bid
Cipro -250-750mg PO bid
Levaquin -500mg PO qday
Tequin -400mg PO qday
-IV: Ancef -1g IV q8
Cipro -400mg IV q12
Levaquin -500mg IV qday
Tequin -400mg IV qday

49
AJM Sheet: Antibiotic Dosing Guide

Penicillins
1st Generation: -Pen V: 500mg q6 PO
-Pen G: 250,000 units/kg/day IV
2nd Generation: -Dicloxacillin: 250mg q6 PO
-Oxacillin: 1-2g q4 IV
-Nafcillin: 1-2g q4 IV
3rd Generation: -Amoxicillin: 500mg q8 PO
-Ampicillin: 1g q4-6 IV
4th Generation: -Augmentin: 875mg q12 PO
-Unasyn: 3g q6 IV
-Zosyn: 4.5g q6 IV
-Timentin: 3.1g q6 IV

Cephalosporins
1st Generation: -Keflex: 500mg q8 PO or 750mg PO bid
-Duricef : 2g q24 PO
-Ancef: 1g q8 IV
2nd Generation: -Ceftin: 500mg q12 PO
-Zinacef: 1.5g q8 IV
-Mefoxin: 1g q6 IV
3rd Generation: -Omnicef: 300mg q12 PO
-Vantin: 400mg q12 PO
-Rocephin: 1g q24 IV
-Fortaz: 1g q8 IV
-Cefobid: 2g q12 IV
4th Generation: -Maxipime: 2g q12 IV

Quinolones
2nd Generation: -Ciprofloxacin: 750mg q12 PO/400mg q12 IV
3rd Generation: -Levofloxacin: 500mg q24 PO/IV
4th Generation: -Tequin: 400 q12 PO/IV
-Avelox: 400 q24 PO

Macrolides -Biaxin: 500mg q12 PO


-Ketek: 800mg q24 PO
-Zithromax: 500 q12 IV/ 500mg PO Day 1; 250 mg PO Day2-5
-Erythromycin 500mg q6 PO

Carbapenems -Invanz: 1g q24 IV


-Primaxin: 500mg q8 IV
-Merrem 1g q8 IV

Aminoglycosides -Amikacin: 1500mg/day


-Tobramycin: 3-5mg/kg/day
-Gentamycin: 3-5mg/kg/day

Tetracyclines -Minocycline: 100mg q12 PO/IV


-Doxycycline: 100mg q24 PO
-Tetracycline: 500mg q6 PO

Misc -Bactrim DS: 160/800mg q24 PO


-Aztreonam: 1g q8 IV
-Vancomycin: 1g q12 IV
-Clindamycin: 600mg q8 IV; 300mg q6 PO
-Zyvox: 600mg q12 PO/IV
-Cubicin: 4mg/kg q12 IV
-Synercid: 7.5mg/kg q8 IV
-Flagyl: 500mg q8 PO
-Rifampin: 300mg q12 PO/IV
-Tygacil: 100mg loading dose; then 50mg q12 IV

50
AJM Sheets: Trauma
Trauma is another area that is often highlighted during the interview process. This section first details a
trauma-specific work-up, and then goes through some specific traumatic conditions.

In terms of the interview, you generally will be expected to work-up, diagnose and classify based on
radiographs, CTs and MRIs. While you should certainly have an understanding of treatment interventions and
protocols, this will probably be less emphasized than diagnosis and classification.
A lot of these classifications are very visual (and I don’t have room for that in 100 pages), so I’ve tried to
include a lot of specific references with pictures of the classifications (mostly to McGlamry’s and Gumann’s texts).
I’ve also tried to include a lot of references to “classic” articles and review articles. Textbooks with good
trauma information for additional reading include specific ones (Gumann’s, Scurran’s, Rang’s, etc), but also general
ones (McGlamry’s, Myerson’s, Hansen’s, etc).

I said that while I was studying for the Diabetic Foot Infection work-up, I tried to learn as much as possible
on the topic and really tried to “wow” the attendings at the interview. However, my strategy was different when
dealing with trauma and the specific surgical work-ups. Here I tried to demonstrate “competence” as opposed to
“mastery” of the material. With specific surgeries, you’re really not supposed to have strong, pre-formed opinions
as a student or as an intern. That’s what your residency is for; developing surgical opinions. If you already know
what to do in every surgical situation, then what’s the point of doing a residency? So while on externships and at the
interview, you should really try to walk a fine line between:
1. Displaying competence in knowledge of the baseline material
2. Displaying that you still have a lot to learn, and that you are eager to learn it

Contents:
-The Trauma Work-Up (page 52)
-General Trauma Topics (pages 53-54)
-Digital Fractures (page 55)
-Sesamoid Trauma (page 56)
-Metatarsal Fractures (page 57)
-5th Metatarsal Fractures (page 58)
-Metatarsal Stress Fractures (page 59)
-LisFranc Trauma (page 60)
-Navicular Trauma (page 61)
-Talar Fractures (page 62)
-Calcaneal Fractures (page 63)
-Ankle Fractures (pages 64-65)
-General Tendon Trauma (page 66)
-Achilles Tendon Work-up (page 67)
-Achilles Tendon Treatment (page 68)

51
AJM Sheet: Trauma Work-up

-The Trauma Work-up is very similar to the regular patient work-up, but with a few things added. You still need to go
through the HPI, PMH, PSH, Meds, Allergies, SH, FH, ROS and complete physical exam in that order. In addition, there
are three other topics that you need to address on every trauma patient for every work-up:

1. ABCDE’s of the Primary Survey


-Airway: Three common forms of airway obstruction are cervical spine injury, swollen tongue and facial fracture.
-Breathing: Note how this is different than an established airway. Someone can have an airway, but still not be
breathing.
-Circulation: Assess vascular status in all four extremities. Two large-bore (18-gauge) IV’s should be started
immediately if fluid replacement is considered necessary.
-Deficits (Neurological): There are two ways to assess this.
-AVPU
-Alert, responds to Verbal stimuli, responds to Painful stimuli, or Unresponsive
-Glasgow Coma Scale
-Based upon three criteria: eye opening, verbal response, motor response.
-Based on scale of 0-15 with a higher score indicating a better prognosis.
-13+ associated with a good prognosis; 7- associated with a poor prognosis.
-Exposure: Complete exposure of the patient to evaluate further, unknown damage.

-Secondary Survey: This is when you go through a normal history including HPI, PMH, etc. and a
comprehensive physical exam.

2. Tetanus Status
-Clostridium tetani is a racquet-shaped gram-positive bacillus. It releases an exotoxin causing a pre-sympathetic
blockade.
-Triad of tetanus symptoms: Trismus, Risus Sardonicus, and Aphagia.
-Characteristics of a tetanus-prone wound: greater than 6 hours old, clinical signs of infection, deep, devitalized
tissue, contamination, traumatic mechanism of injury, etc.

-Basic Tetanus Algorithm:


-Unknown tetanus status: -Clean wound: Give the toxoid; Hold the TIG
-Tetanus-prone wound: Give the toxoid; Give the TIG

-Incomplete tetanus status: -Clean wound: Give the toxoid; Hold the TIG
(No booster within 5 years) -Tetanus-prone wound: Give the toxoid; Give the TIG

-Complete tetanus status: -Clean wound: Hold the toxoid; Hold the TIG
(Booster within 5 years) -Tetanus-prone wound: Hold the toxoid; Hold the TIG

-Dosages: -Toxoid: 0.5ml


-TIG (tetanus immunoglobulin): 250-300 units

3. NPO status
-All trauma patients are potential surgical candidates, so get this information for the weenie anesthesiologists.
-Traditional guidelines recommend:
-Nothing by mouth after midnight the night before elective surgery
-Nothing by mouth within 6-8 hours of any type of surgery

-These strict guidelines are in the process of changing however, particularly with regard to allowing the ingestion
of small amounts of clear liquids up to the time of surgery. If interested, please read:
-[Brady M, Kinn S, Stuart P. Preoperative fasting for adults to prevent perioperative complications. Cochrane
Database Syst Rev. 2003; (4): CD004423.]
-[Murphy GS, et al. The effect of a new NPO policy on operating room utilization. J Clin Anesth. 2000 Feb; 12(1):
48-51.]

52
AJM Sheet: General Trauma Topics

-In addition to having a good trauma work-up, there are a few other things that are helpful to know regarding foot and ankle
trauma.

1. Podiatric Surgical Emergencies


-Infection with emphysema (gas gangrene)
-Open fracture/dislocation
-Compartment syndrome
-Necrotizing Fasciitis
-General Neurovascular compromises

2. Mangled Extremity Severity Score (MESS)


-[Helfet DL, et al. Limb salvage versus amputation. Preliminary results of the Mangled Extremity Severity Score. CORR 1990; 256: 80-6.]
-[Bosse MJ, et al. A prospective evaluation of the clinical utility of the lower-extremity injury-severity scores. JBJS-Am 2001; 83(1): 3-14.]
-Based on 4 criteria: Skeletal/Soft Tissue Injury, Limb Ischemia, Age, and Shock
-Based on a scale from 1-11 with a higher score leading to an increased incidence of amputation.
-A score of 7+ has an increased likelihood of amputation.

3. Open Fractures
-Note that 30% of lower extremity open fractures are associated with polytrauma.
-Mainstays of treatment: Aggressive incision and drainage with copious lavage.
-It is generally recommended to never primarily close an open fracture until devitalized soft tissue has demarcated,
but this certainly isn’t always the case in practice. In fact, the Ortho Trauma service at INOVA routinely primarily closes
open fractures following I&D with ORIF.

-Gustilo-Anderson Classification of Open Fractures [Gustilo RB, Anderson JT. Prevention of infection in the treatment of one
thousand and twenty-five open fractures of long bones: retrospective and prospective analyses. JBJS-Am. 1976; 58(4): 453-8.]
I. Clean Wound <1cm in diameter
-Abx choice: 1st generation cephalosporin (Ancef)
II. Wound 1.0-5.0cm in diameter with minimal soft tissue damage
-Abx choice: Ancef, Clindamycin
III. Wound >5cm in diameter with extensive soft tissue damage
-Abx choice: Ancef (or high dose PCN), Clindamycin and Aminoglycoside
-IIIA: Adequate soft tissue coverage
-IIIB: Extensive soft tissue damage with periosteal stripping and massive contamination
-IIIC: Arterial damage requiring primary repair

4. Fracture Blisters
-Location: Subepidermal
-Note that the fluid is sterile. Fracture blisters are histologically similar to 2nd degree burns.
-Most common LE etiology? Secondary to high-energy trauma such as ankle fx, calcaneus fx or Lisfranc injury.
-2 Common Types of Fracture Blisters
-Clear fluid: Most common (75%). Very tense in appearance.
-Hemorrhagic: Most severe. Roof is flaccid. Takes longer to re-epithelialize.
-Treatment is controversial, but the conservative approach is to never incise through a fracture blister and to delay
surgery until re-epithelialization.
-[Strauss EJ, et al. Blisters associated with lower-extremity fracture: results of a prospective treatment protocol. J Orthop Trauma. 2006 Oct;
20(9): 618-22.]

5. Shock
-Signs/Symptoms of Shock: Tachycardia, Tachypnea, delayed capillary refill, decreased pulse pressure, change in
mental status, decreased systolic pressure, decreased urinary output and decreased H&H.
-Types of Shock:
-Hypovolemic: most common; defined as the acute loss of circulating blood. Treatment is aggressive fluid
replacement.
-Cardiogenic: induced by myocardial dysfunction.
-Neurogenic: secondary to decreased sympathetic tone from head and spinal cord injuries.
-Septic: shock secondary to infection.
-Goal of Treatment: restore organ perfusion.

53
AJM Sheet: General Trauma Topics
-Foreign Bodies/Puncture Wounds
-When should a foreign body be removed?
-Clinical signs of infection, known contaminated object, pain, object close to NV elements, intra-articular
-Recommended imaging studies for a foreign body?
-Plain film radiography (no oblique views!), fluoroscopy, CT, MRI, US
-How will wooden objects appear on US?
-hyperechoic with a hypoechoic dark shadow
-How large must a glass foreign body be to be visible on plain film radiography? Does leaden matter?
-A piece of glass, regardless of whether it is leaden, must be >5mm to be visible.
-Classification for foreign bodies?
-Resnick Classification [Resnick CD. Puncture wounds: therapeutic considerations and a new classification. J Foot Surg. 1990
Mar-Apr; 29(2): 147-53.]
-I. Superficial/cutaneous: usually visible without signs of infection.
-II. Subcutaneous or articular without signs of infection.
-IIIA. Subcutaneous or articular with signs of infection.
-IIIB. Bone penetration without signs of infection.
-IV. Bone penetration with known osteomyelitis.
-Patzakis Classification [Patzakis MJ. Wound site as a predictor of complications following deep nail punctures of the foot. West
J Med. 1989 May; 150(5): 545-7.]
-Zone 1: Toe to met head (50% incidence of osteomyelitis in this limited study.)
-Zone 2: Midfoot (17% incidence of osteomyelitis in this limited study.)
-Zone 3: Calcaneus (33% incidence of osteomyelitis in this limited study.)
-Puncture wound common bugs
-Most common? Staph Aureus
-2nd most common? Beta-hemolytic strept
-Puncture through shoe gear? Pseudomonas
-Puncture involving soil or a farm? Clostridia
-Human bites? Eikenella corrodens
-Cat bites? Pasteurella multocida
-Dog bites? Enterobacter, Pseudomonas, Staph, Bacillus
-Mainstays of foreign body/puncture wound treatment?
-Tetanus status, antibiotics, aggressive I&D with copious lavage

-Gun Shot Wounds


-High velocity GSWs are characterized by speeds >2500 ft/s. This is significant because high velocity GSWs have a tendency
to yaw and tumble leading to increased cavitation.
-Cavitation: Large wound is created under a situation of negative pressure. This negative pressure “sucks” outside
contaminants into the wound.
-[Holmes GB. Gunshot wounds of the foot. CORR. 2003 Mar; (408): 86-91.]

-Compartment Syndrome
-First described by Volkmann. Myerson has good articles/chapters on this topic.
-[Perry MD, Manoli A. Foot compartment syndrome. Orthop Clin North Am. 2001 Jan; 32(1): 103-11.]
-[Myerson M, Manoli A. Compartment syndromes of the foot after calcaneal fractures. Clin Orthop Relat Res. 1993 May: 142-50.]
-Results when interstitial pressure exceeds capillary hydrostatic pressure, so the microcirculation shuts down.
-The foot has anywhere from 3-11 compartments depending on who you read:
-Intermetatarsal Compartments X 4: contains the interossei muscles
-Medial Compartment: Abductor Hallucis
-Lateral Compartment: Abductor digiti minimi
-Superficial Central Compartment: FDB
-Deep Central Compartment: Adductor Hallucis
-Calcaneal Compartment: Quadratus Plantae and lateral plantar artery
-Dorsal Compartment: EHB and EDB
-P’s of Compartment Syndrome (These are very generalized.)
-Pain out of proportion and not controlled by analgesics -Paralysis
-Pain with passive dorsiflexion of the toes -Pulselessness
-Paresthesia -Pressure
-Pallor
-Diagnosis
-Normal compartment pressure? 0-5mm Hg
-When do you start getting worried? 20-30mm Hg
-When do you consider surgical intervention? >30-40mm Hg
-How is diagnosis made? Wick or slit catheter to measure compartment pressures
-Treatment
-Decompression via fasciotomy, debridement of necrotic tissue, copious lavage and delayed closure
-Incision approaches: Consider dorsal vs. medial approaches
-Complications: permanent loss of function with structural deformity (Volkmann contractures), myoneural necrosis, sensory
loss, chronic pain

54
AJM Sheet: Digital Fractures

-Even suspected digital fractures should be worked up according to a standard, full trauma work-up during the interview if
the case is presented as a trauma. The following describes unique subjective findings, objective findings, diagnostic
classifications and treatment.

Subjective
-History of trauma. “Bedpost” fracture describes stubbing your toe while walking at night. Also common are injuries from
dropping objects on the foot.

Objective
-Edema, erythema, ecchymosis, open lesions, subungual hematoma, and onycholysis should all be expected.
-Any rotational/angulation deformities should be identified on plain film radiograph.

Diagnostic Classifications
-Rosenthal Classification [Rosenthal EA. Treatment of fingertip and nail bed injuries. Orthop Clin North Am. 1983; 14: 675-697.]
-Zone I: Injury occurs with damaged tissue completely distal to the distal aspect of the phalanx.
-Zone II: Injury occurs with damaged tissue completely distal to the lunula.
-Zone III: Injury occurs with damaged tissue completely distal to the most distal joint (IPJ in hallux; DIPJ in
lesser).

Treatment
-Zone I Injuries
-If injury involves no exposed bone and a total tissue loss less than 1cm squared, then:
-Allow to heal in by secondary intention.
-If injury involves a total tissue loss greater than 1cm squared, then:
-A STSG or FTSG should be used depending on weight-bearing position.
-Zone II Injuries
-Flaps and Skin Grafts generally employed:
-Atasoy flap: plantar V Y advancement
-[Atasoy E. Reconstruction of the amputated fingertip with a triangular volar flap. JBJS-Am 1970; 52: 921-926.]
-Kutler flap: biaxial V  Y advancement
-[Kutler W. A new method for fingertip amputation. JAMA 1947; 133: 29-30.]
-Zone III Injuries
-Usually requires distal amputation (Distal Symes amputation)

Miscellaneous Notes
-Hallux fracture is regarded as the most common forefoot fracture.
-Digital fractures without nail involvement and displacement/angulation/rotation can be treated conservatively with
immobilization.

-If a subungual hematoma is present, then there is a 25% incidence of underlying phalanx fracture.
-If a subungual hematoma covers >25% of the nail, then the nail should be removed.
-Only 1mm squared of free space from onycholysis is necessary for hematoma development.

-For proper nail function and adherence, there should be no onycholysis within 5mm of the lunula.
-A Beau’s line is a transverse groove often associated with nail trauma.

55
AJM Sheet: Sesamoid Trauma

-The following describes unique subjective findings, objective findings, diagnostic classifications and treatments.

Subjective
-History of trauma is very important in this case. You want to differentiate between acute and chronic conditions involving
the sesamoids. Be careful to elicit any neurologic complaints that could be present.

Objective
-Expect edema, erythema, ecchymosis and open lesions. Take the time for proper palpation.
-Joplin’s neuroma is irritation of the medial plantar proper digital nerve.
-Associated with rigidly plantarflexed first metatarsals, anterior cavus, etc.
-One of the most difficult things to differentiate is an acute sesamoid fracture from a bipartite sesamoid. There are
several generic plain film radiographic characteristics found in acute fractures:
-Jagged, irregular and uneven spacing
-Large space between fragments
-Abnormal anatomy
-Bone callus formation
-Comparison to a contra-lateral view

-Also useful are:


-HISTORY of acute incident
-Bone scan (would show increased osteoblastic/osteoclastic activity with acute fracture).

Diagnostic Classifications
-Jahss Classification [Jahss MH. Traumatic dislocations of the first metatarsophalangeal joint. Foot Ankle. 1980 Jul; 1(1): 15-21.]
-Type I
-Mechanism: Dorsal dislocation of the hallux
-Intersesamoid ligament: Intact
-Fracture?: No sesamoid fracture
-Treatment: Requires open reduction
-Type IIA
-Mechanism: Dorsal dislocation of the hallux
-Intersesamoid ligament: Ruptured
-Fracture?: No sesamoid fracture
-Treatment: Closed reduction/Conservative Care
-Type IIB
-Mechanism: Dorsal dislocation of the hallux
-Intersesamoid ligament: Ruptured
-Fracture?: Fracture of at least one sesamoid
-Treatment: Closed reduction/Conservative Care
-Type II Variant
-Mechanism: Dorsal dislocation of the hallux
-Intersesamoid ligament: Ruptured
-Fracture?: Separation of a bipartite sesamoid
-Treatment: Closed reduction/Conservative Care

Treatments
-Conservative
-Immobilization (NWB SLC, PWB SLC, Surgical Shoe, CAM Walker, etc.)
-Dancer’s Pad
-Surgical
-Excision of the fractured fragment or entire sesamoid

Miscellaneous Notes
-Ilfeld’s Disease: Agenesis of the fibular sesamoid
-[Ilfeld FW, Rosen V. Osteochondritis of the first metatarsal sesamoid. CORR 1972; 85: 38-41.]
-Incidence of Bipartite Sesamoid in Population:
-As much as Kewenter: 35.5%
-As few as Inge: 10.7% with 75% of cases being unilateral

56
AJM Sheet: Metatarsal Fractures
-The following describes unique subjective findings, objective findings, diagnostic classifications and treatments.

-Subjective and Objective


-All will point to some form of traumatic injury. Common injuries leading to metatarsal fracture include direct trauma, blunt trauma, shearing, ankle sprains,
etc.
-Most important in your work-up will be how you read the plain film radiographs. Remember that at least two views are necessary to accurately describe
displacement/angular/rotational abnormalities.

-Metatarsal Head/Impaction Fractures


-MOI: Direct or indirect trauma
-Radiographic findings: -Examine for evidence of displacement/angulation/rotation
-Expect a shortening mechanism
-Examine for intra-articular nature of fracture
-Treatment: -Conservative
-Closed reduction generally unsuccessful
-Surgical
-ORIF with fixation of K-wire, screws or absorbable pins
- immobilization for 4-6 weeks and NWB
-Follow-Up
-Early PROM suggested
-Subsequent arthrosis is a common complication

-Metatarsal Neck Fractures


-MOI: Shearing forces or direct trauma
-Radiographic findings: -Expect elements of shortening, plantarflexion and lateral displacement of the distal segment.
-Treatment: -Conservative
-Closed reduction generally unsuccessful
-Surgical
-ORIF effective in restoring and maintaining alignment with K-wires, IM pinning and plates.
-Follow-up
-NWB in SLC for 4-6 weeks
-General Information:
-Metatarsal neck fractures often involve multiple metatarsals due to the mechanism of injury. Multiple fractures are very unstable due to loss of
function of the deep transverse metatarsal ligament, which usually prevents displacement.
-Vassal Principle: Adjacent fractures generally improve alignment after reduction of the initial fracture because soft tissue structures are
returned to their normal position through traction.

-Midshaft Metatarsal Fractures


-MOI: Result of direct, blunt or torsional injuries
-Radiographic findings: -Expect oblique fracture line, but transverse, spiral and comminuted are all possible.
-Expect elements of shortening, plantarflexion and lateral displacement of the distal segment.
-Treatment: -Based on displacement and fracture type:
-Non-displaced fractures: NWB SLC 4-6 weeks
-Fractures with >2-3mm of displacement and >10 degrees of angulation: ORIF

-Transverse displaced fractures


-Consider buttress plate, compression plate, IM percutaneous pinning, crossed K-wires
-Long oblique or spiral fractures
-Consider screws, plates, IM pinning, cerclage wiring
-Comminution
-Consider screws, plates, cerclage wiring, K-wires and external fixation

-Metatarsal Base Fractures


-MOI: Direct trauma (MVA, fall from height, etc.) Usually associated with Lisfranc’s trauma.
-Radiographic findings: -Generally remain in good alignment/angulation because of surrounding stable structures.
-Treatment: -Conservative
-NWB SLC 4-6 weeks with good alignment
-Surgical
-ORIF with displacement/alignment/angulation

-First Metatarsal Fractures


-MOI: Direct trauma (MVA, fall from height, crush, etc.) and indirect trauma (torsional, twisting, avulsions, etc.)
-Radiographic findings: -Variable
-Examine for distal intra-articular fractures
-Examine for avulsion-type fractures
-Treatment: -Conservative
-SLC 4-6 weeks with non-displaced fractures
-Be wary of closed reduction because extrinsic muscles may displace after apposition.
-Surgical
-Various ORIF techniques detailed above
-Percutaneous pinning and cannulated screws are option in first metatarsal
-ORIF should be utilized if intra-articular fracture involves >20% of articular surface

57
AJM Sheet: 5th Metatarsal Base Fractures

-The following describes unique subjective findings, objective findings, diagnostic classifications and treatments.

Subjective and Objective


-All will point to some form of traumatic injury. Common injuries leading to metatarsal fracture include direct trauma, blunt
trauma, shearing, ankle sprains, etc.
-Most important in your work-up will be how you read the plain film radiographs. Remember that at least two views are
necessary to accurately describe displacement/angular/rotational abnormalities.

Diagnostic Classifications
Stewart Classification
-[Stewart IM. Jones fracture: Fracture of the base of the fifth metatarsal bone. Clin Orthop. 1960; 16: 190-8.]
-Type I: Extra-articular fx at metaphyseal-diaphyseal junction (True Jones Fracture)
-MOI: Internal rotation of the forefoot while the base of 5th met remains fixed
-Radiographic findings: -Usually oblique or transverse fx at metaphyseal-diaphyseal junction
-Treatment: -NWB SLC 4-6 weeks for non-displaced fractures
-ORIF with displacement >5mm
-Misc: -Fracture first described by Sir Robert Jones in 1902 from injuring himself while ballroom
dancing. [Jones R. Fracture of the base of the fifth metatarsal bone by indirect violence. Ann Surg. 1902; 35(6): 776-82.]
-Very unstable fracture with high incidence of non-union/delayed union secondary to variable
blood supply. Remember that the diaphysis and metaphysis are generally supplied by two different arterial sources.
-[Smith JW. The intraosseous blood supply of the fifth metatarsal: implications for proximal fracture healing. Foot Ankle.
1992 Mar-Apr; 13(3): 143-52.]

-Type II: Intra-articular avulsion fracture


-MOI: Shearing force caused by internal twisting with contracture of peroneus brevis tendon
-Radiographic findings: -1 or 2 fracture lines
-Intra-articular in nature
-Treatment: -NWB SLC 4-6 weeks for non-displaced fractures
-ORIF with displacement >5mm
-Type III: Extra-articular avulsion fracture
-MOI: Reflex contracture of peroneus brevis with ankle in plantarflexed position
-Radiographic findings: -Extra-articular; Involvement of styloid process
-Treatment: -NWB SLC 4-6 weeks for non-displaced fractures
-ORIF (pins, screws, tension-band wiring) for displacement >5mm
-Consider excision of fragment and reattachment of peroneus brevis tendon
-Type IV: Intra-articular, Comminuted fracture
-MOI: Crush injuries with base of 5th met stuck between cuboid and the external agent
-Radiographic findings: -Multiple fragments; joint involvement
-Treatment: -NWB SLC 4-6 weeks for non-displaced fractures
-ORIF with displacement
-Consider bone grafting and fragment excision with severe comminution
-Misc: -High rate of non-union/delayed union
-Type V: Extra-articular avulsion fractures of the epiphysis
-MOI and treatment similar to Type II and III fractures
-Note that this can only occur in children (similar to a Salter-Harris Type I fracture)

Torg Classification
[Torg JS, et al. Fractures of the base of the fifth metatarsal distal to the tuberosity. JBJS-Am. 1984; 66(2): 209-14.]
-Radiographic classification of Jones fractures describing potential for non-union development.
-Type I: Acute injuries
-Radiographic findings: Narrow fracture line without intra-medullary sclerosis
-Type II: Delayed Union
-Radiographic findings: Widened fracture intersurface with evidence of IM sclerosis
-Type III: Non-Union
-Radiographic findings: Complete sclerotic obliteration of the IM canal

58
AJM Sheet: Stress Fracture Work-up
Also called: March fx, Hairline fx, Fatigue fx, Insufficiency fx, Deutschlander’s dz, Bone exhaustion, etc.

-Subjective
-CC: Patient presents complaining of a diffuse foot and ankle pain. Classic patient is a military recruit or athlete.
-HPI: -Nature: Pain described as “sharp with WB” or “sore/aching.” May have element of “shooting” pain.
-Location: Described as diffuse, but can be localized with palpation. Common areas include dorsal metatarsal or
distal tib/fib.
-Course: Subacute onset. Usually related to an increase in patient’s physical activity.
-Aggravating factors: Activity
-Alleviating factors: PRICE
-PMH: -Look for things that would weaken bone (eg. Osteoporosis)
-SH: -Look for recent increases in physical activity or a generally active patient
-PSH/Meds/All/FH/ROS: Usually non-contributory

-Objective

Physical Exam
-Derm: -Generalized or localized edema
-Ecchymosis is rare
-Vasc/Neuro: Usually non-contributory
-Ortho: -Painful on localized palpation (positive pinpoint tenderness)
-Possible pain with tuning fork

-Imaging
-Plain Film Radiograph: -Localized loss of bone density and bone callus formation are hallmark signs
-Note that there must be a 30-50% loss of bone mineralization before radiographic presentation
of decreased bone density. This generally takes 10-21 days in a stress fracture.
-Bone Scan: -Increased uptake in all phases regardless of time of presentation

-General Stress Fracture Information


-Somewhere between 80-95% of all stress fractures occur in the LE with the most common sites being the metatarsals (20%
with 2nd metatarsal most commonly involved [11%]) and the distal tibia/fibula.
-Stress fractures can occur via two mechanisms:
-Chronic strain upon a normal bone
-A chronic, normally benign strain upon a weakened bone

-Treatment
-Conservative treatment is mainstay:
-Immobilization and NWB for 4-6 weeks (SLC, Unna boot, surgical shoe, etc.)
-Be certain of anatomic position with no angulation/rotation/displacement (very uncommon)

59
AJM Sheet: Lisfranc Trauma

-History
-Dr. Jacques Lisfranc was a French gynecologist who was called into the service of Napoleon’s army where he served as a trauma surgeon in the 1820’s and
30’s. He also served under Dr. Dupuytren during this time.
-Del Sel first described Lisfranc dislocations following equine injuries (JBJS 1955).

-Anatomy
-Tarsometatarsal joint: 9 bones, ~13 joints, 7 weak dorsal ligaments, 7 strong plantar ligaments, the Lisfranc ligament
-Myerson described three functional columns of the Lisfranc joint. Ouzounian and Shereff described the sagittal plane motion of each of these columns.
-Medial Column: 1st met and medial cuneiform: 4mm of motion in the sagittal plane.
-Central Column: 2nd/3rd mets and central/lateral cuneiforms. 1mm of motion in sagittal plane.
-Lateral Column: 4th/5th mets and cuboid. 10mm of motion in the sagittal plane.

-Mechanism of Injury
-Accounts for 0.2% of all traumatic injuries. Most common in MVA and sports injuries.
-Occurs either by direct crushing (i.e. dropping something on the foot) or indirectly (usually a plantarflexed and abducted foot).

-Diagnosis
-Clinical
-Midfoot pain and tenderness. Possibly exacerbated with pronation, abduction or plantarflexion.
-Plantar ecchymosis
-Be wary of compartment syndrome! Always check neurovascular status.
-Imaging
-Plain Film Radiography
-Pathognomonic “fleck sign” representing an avulsion fx in the 1st IM space.
-Look for deviations from normal in the AP, MO and Lat views. Normal is:
-AP: Medial border of the 2nd met continuous with the medial border of the central cuneiform. Lateral border of the
medial cuneiform continuous with the medial border of the central cuneiform.
-MO: Medial border of the 4th met continuous with the medial border of the cuboid. Lateral border of the 3rd met
continuous with the lateral border of the lateral cuneiform.
-Lat: No sagittal displacement. Look for lateral column shortening with a “nutcracker fracture” of the cuboid.
-“Lisfranc variant” is fracture damage extending proximally into the cuboid-navicular region.
-Consider stress radiographs with the foot in plantarflexion or abduction.
-CT, Bone Scan, MRI could all be utilized.

-Classifications
-Classification originally described by Quenu and Kuss, then modified by Hardcastle, then modified by Myerson. The Myerson Classification is
listed with the Quenu and Kuss equivalent in parentheses.
-Type A: Total incongruity in any plane (QK: Homolateral)
-Type B: Partial incongruity (QK: Isolateral)
-B1: 1st met goes medial
-B2: Lesser mets go lateral
-Type C: Divergent (QK: Divergent)
-C1: Partial (only 1st and 2nd mets involved)
-C2: Total (all mets involved)

-Treatment
-Literature strongly favors ORIF with any displacement (>2mm between the 1st and 2nd mets). Exact anatomic reduction is the key to prognosis.
-Non-operative
-If plain film and stress radiographs show no displacement, then NWB SLC for 6 weeks with films q2 weeks looking specifically for
displacement.
-Operative
-Goal: Reduction and stabilization of the medial and central columns. You must reduce the lateral column, but it is usually left unfixed because
of the pronating mobile adapter mechanism. The medial and central columns do not have as much sagittal plane motion, but you still don’t want excess
compression with associated chondrolysis to develop.
-Fixation:
-1st met to medial cuneiform, 2nd met to central cuneiform, and 3rd met to lateral cuneiform with crossed 0.062” K-wires (removed at 8
weeks), cannulated cancellous screws (removed at ~12 weeks) or 3.5mm corticals. Consider putting a notch 1.5cm distal to the joint for screw to prevent
stress risers. Drill the hole for the screw in the superior aspect of the notch and not the base to prevent splitting the base.

-Consider 4th met to cuboid and 5th met to cuboid with a single 0.062” K-wire
-Lisfranc Screw: Medial cuneiform to 2nd met base, screw in a lag fashion
-Length of the lateral column must be restored following a “nutcracker fracture.” Consider using an H-plate or external fixation.
-Incisions:
-Longitudinal over the 1st IM space (provides access to 1,2,3) and longitudinal over the 4th IM space (provides access to 3,4,5).
-Longitudinal in the 1st IM space, 3rd IM space and lateral to the 5th met.

-Post-Operative
-NWB SLC for 8 weeks transitioned to PWB SLC for 4 weeks transitioned to rehab. High impact activity can usually be resumed at 6 months.

-Complications
-ARTHRITIS! Essentially everyone develops post-traumatic arthritis to some extent.

Additional Reading:
-[Myerson M. The diagnosis and treatment of injuries to the Lisfranc joint complex. Orthop Clin North Am. 1989; 20(4): 655-64.]
-[Hardcastle PH, et al. Injuries to the tarsometatarsal joint. Incidence, classification, and treatment. JBJS-Br. 1982; 64(3): 349-56.]
-[Desmond EA, Chou LB. Current concepts review: Lisfranc injuries. Foot Ankle Int. 2006; 27(8): 653-60.]

60
AJM Sheet: Navicular Trauma

-Suspected navicular trauma should be worked up with a primary and secondary survey. The following describes unique
subjective findings, objective findings, diagnostic classifications and treatments.

Subjective
-History of trauma ranges from contusions to ankle sprains to forced abduction/plantarflexion of the forefoot.

Objective
-MMT of the posterior tibialis tendon is important in these cases.
-Multiple view plain film radiographs are extremely important because of the possible obliquity of some fractures. CT scans
and MR images may also be necessary for complete visualization and analysis of stress fractures.

Relevant Anatomy
-The navicular is surrounded by a number of joints of varying stability. The TNJ proximally is very mobile, while the distal
NCJ and lateral NCJ are very stable. The navicular is also very stable medially because of the insertion of the PT tendon.
-Vascular anatomy to the navicular can be extremely important as described by Sarrafian:
-It has been demonstrated that the central 1/3 of the navicular is relatively avascular.
-The dorsalis pedis artery adequately supplies the dorsal and medial aspects.
-The medial plantar artery adequately supplies the plantar and lateral aspects.
-The central 1/3 has variable, radially-projecting branches from anastomosis of these arteries.

Diagnostic Classification
-Watson-Jones Classification
-Type I: Navicular Tuberosity Fractures
-Occur secondary to eversion and posterior tibialis contracture
-Watch for associated “nutcracker fracture” of cuboid and anterior calcaneal process fractures
-Must be differentiated from accessory navicular
-Treatment: -Displacement <5mm:
-Conservative immobilization
-Displacement >5mm consider:
-Excision of fragment with reattachment of tendon
-ORIF with a cancellous screw
-Type II: Dorsal Lip Avulsion Fractures
-Occur secondary to plantarflexion/frontal plane mechanisms.
-Must differentiate from os supranaviculare and os supratalare accessory ossicles.
-Generally intra-articular
-Generally treated with conservative immobilization
-Type III: Navicular Body Fractures. Described by Sangeorzan.
-[Sangeorzan BJ, et al. Displaced intra-articular fractures of the tarsal navicular. JBJS-Am. 1989; 71(10): 1504-10.]
-IIIA: Coronal Plane Fracture with no angulation
-~100% successful reduction usually achieved
-IIIB: Dorso-lateral to Plantar-medial fracture with adduction of the forefoot
-67% successful reduction usually achieved
-IIIC: Comminution with abduction of the forefoot
-50% successful reduction usually achieved

-Principles of ORIF for Type III fractures:


-Must achieve 60% reapproximation of proximal joint space
-Incision placed dorsal-medial, between the TA and TP
-Complications involve post-traumatic arthritis and/or AVN
-Fixation Options using 3.5mm Cortical Screws:
-Two screws directed lateral to medial
-Two crossed screws directed proximal to distal
-One screw directed proximal-medial to distal-lateral into the middle cuneiform
-Consider FDL transfer in the presence of a weakened PT tendon
-Type IV: Stress Fracture of the Navicular
-Generally occurs secondary to running
-Torg describes typical stress fracture occurring in central 1/3 of body in the sagittal plane.
-DDx: Tibialis anterior tendonitis
-Usually plain films, CT and bone scans are necessary to diagnose

61
AJM Sheet: Talar Fractures

-Talar fractures are generally associated with high energy trauma, and a standard evaluation with primary and secondary surveys
should precede any specific talar evaluation. The following describes unique subjective findings, objective findings, diagnostic
classifications and treatment considerations.

Subjective
-History of trauma with a high incidence of MVC. The classic description of a talar neck fracture comes from a forced dorsiflexion
of the foot on the ankle (“aviator’s astragulus”). Talar fractures account for approximately 1% of all foot and ankle fractures.

Objective
-Important to verify neurovascular status, and rule out dislocations and compartment syndromes.
-Imaging: -Canale View: Plain film radiograph taken with the foot in a plantarflexed position. The foot is also pronated
15 degrees with the tube head orientated 75 degrees cephalad. This view allows for evaluation of angular
deformities of the talar neck.
-CT scan is essential for complete evaluation and surgical planning.

Relevant Anatomy
-An intimate knowledge of the vascular supply to the talus is essential with regard to avascular necrosis (AVN):
-Dorsalis Pedis: Supply the superior aspect of the head and neck (artery of the superior neck)
-Anastomoses with the peroneal and perforating peroneal arteries
-Artery to the sinus tarsi: supplies the lateral aspect of the talar body
-Forms an anastomotic sling with the artery of the tarsal canal
-Posterior Tibial Artery:
-Deltoid branch: medial aspect of the talar body
-Artery of the canalis tarsi: majority of the talar body
-Forms an anastomotic sling with the artery of the tarsal sinus
-Also sends branches to the posterior process
-Peroneal/Perforating Peroneal Artery: supplies posterior and lateral aspects of the talar body
-Anastomoses with the dorsalis pedis artery branches

Classifications/Named Fractures:

Hawkins Classification: Talar neck fractures Berndt and Harty: Talar dome fractures
Type I: Non-displaced (~13% incidence of AVN) Type 1: Chondral Depression
Type II: Displaced fracture with STJ dislocation (~50% incidence) Type 2: Partial chondral fracture, seen on MRI
Type III: Displaced fracture with STJ and ankle dislocation (~95%) Type 3: Nondisplaced complete osteochondral fracture
Type IV: Displaced fracture with STJ/ankle/TN dislocation (>95%) Type 4: Displaced complete osteochondral fracture
-(Type IV added by Canale and Kelly)

Sneppen: Talar body fractures Modified Hawkins: Lateral process fractures


Type 1: Osteochondral fracture Type I: Simple bipartite fracture
Type 2: Sagittal, Coronal, Transverse body fracture Type II: Comminuted fracture
Type 3: Lateral process fracture Type III: Chip fracture of anteroinferior lateral process
Type 4: Posterior tubercle fracture
Type 5: Crush fracture

Others: -Shepherd’s fracture: Acute fracture of posterolateral talar process


-Cedell’s fracture: Acute fracture of the posteromedial talar process
-Snowboarder’s fracture: lateral process fractures

Treatment
-NWB in SLC 6-8 weeks versus ORIF depending on nature of fracture and degree of displacement.
-Titanium hardware may be used so that MRI evaluation may be used in post-operative period to evaluate for AVN!
-Hawkins sign: radiolucency of the talar body noted at 6-8 weeks after fracture. This sign is indicative of intact vascularity.
However, the absence of this sign does not indicate that osteonecrosis and talar collapse are eminent.

Additional Readings:
-Talar fractures are relatively uncommon in the medical literature. Most studies are case reports or small retrospective
reviews leading only to Level IV or V evidence.
-[Ahmad J, Raikin SM. Current concepts review: talar fractures. Foot Ankle Int. 2006 Jun; 27(6): 475-82.]
-[Golano P, et al. The anatomy of the navicular and periarticular structures. Foot Ankle Clin. 2004 Mar; 9(1): 1-23.]
-[Berndt A, Harty M. Transchondral fractures of the talus. JBJS-Am. 1959; 41: 988-1020.]
-[Canale ST, Kelly FB. Fractures of the neck of the talus. Long-term evaluation of seventy-one cases. JBJS-Am. 1978 Mar; 60(2): 143-56.]

62
AJM Sheet: Calcaneal Fractures
-The standard trauma work-up again applies with primary and secondary surveys. The following describes unique subjective
findings, objective findings, diagnostic classifications and treatment considerations.

Subjective
-Demographics: Men>Women; Age range generally 30-60; account for ~2% of all fractures; 2-10% are bilateral; 10% associated
with vertebral fracture (most commonly L1); 1% associated with pelvic fracture and urethral trauma.
-Common mechanisms of injury: Direct axial load, vertical shear force, MVC, gastroc contraction, stress fracture, ballistics,
iatrogenic surgical fracture

Objective
-Physical Exam: -Pain with palpation to heel -Mondor’s Sign: characteristic ecchymosis extending into plantar medial foot
-Short, wide heel -Hoffa’s sign: less taut Achilles tendon on involved side
-Inability to bear weight -Must rule out compartment syndrome
-Imaging: -Plain film Imaging: -Bohler’s Angle: normally 25-40 degrees (decreased with fracture)
-Critical Angle of Gissane: normally 125-140 degrees (increased with fracture)
-Both demonstrate loss of calcaneal height
-Broden’s View: 2 oblique views to view the middle and posterior facets
-Isherwood Views: 3 oblique views to highlight all facets
-Calcaneal Axial View: demonstrates lateral widening and varus orientation
-CT Scan: -Gold standard for evaluation and surgical planning
-The coronal view forms the basis of the Sanders Classification
Classifications
Sanders Classification: Uses widest view of posterior facet on semicoronal CT cut
-Type I: Non-displaced articular fx -A, B and C further describe the fx (lateralmedial)
-Type II: Two-part posterior facet fx -A/B: divide posterior facet into equal 1/3’s
-Type III: Three-part posterior facet fx -C: divides posterior facet from sus tali
-Type IV: Four-part/comminuted fx

-See page 1845 of McGlam’s, or 224 of Gumann’s for actual pictures.


-[Koval KJ, Sanders R. The radiographic evaluation of calcaneal fractures. CORR. 1993 May; 290: 41-6.]
-[Sanders R. Displaced intra-articular fractures of the calcaneus. JBJS-Am. 2000 Feb; 82(2): 225-50.]
Rowe Classification:
-Type I: -Type IA: Plantar tuberosity fractures (medial more common than lateral)
-Type IB: Sus tali fracture (remember FHL: pt will have pain with hallux PROM)
-Type IC: Anterior process fractures (remember your anatomy: bifurcate ligament)
-further divided into three parts by Degan Classification
-Type II: -Type IIA: Extra-articular “beak” fracture of posterior-superior calcaneal body
-Type IIB: Intra-articular “tongue-type” Achilles avulsion fracture
-Type III: Extra-articular calcaneal body fracture
-Type IV: Intra-articular joint depression fracture
-Type V: Intra-articular comminuted fracture

-See page 1830 of McGlam’s or page 223 of Gumann’s for pictures.


-[Rowe CR, et al. Fractures of os calcis: a long term follow-up study of one hundred forty-six patients. JAMA 1963; 184: 920-3.]
-[O’Connell F, Mital MA, Rowe CR. Evaluation of modern management of fractures of the os calcis. CORR 1972; 83: 214-23.]
Essex-Lopresti [Essex-Lopresti P. The mechanism, reduction technique, and results in fractures of the os calcis. Br J Surg 1952; 39: 395-419.]
-Differentiated between extra-articular (~25%) and intra-articular (~75%) fractures and further sub-divided intra-articular fractures
into tongue-type and joint depression fractures (both with the same primary force, but different secondary exit points).
Zwipp [Rammelt S, Zwipp H. Calcaneus fractures: facts, controversies and recent developments. Injury 2004; 35(5): 443-61.]
-Assigns 2-12 points based on: -Number of fragments -Number of involved joints
-Open fracture or high soft tissue injury
-Highly comminuted nature, or associated talar, cuboid, navicular fractures

Treatment
-Appreciate the debate in the literature between cast immobilization vs. percutaneous reduction vs. ORIF vs. primary arthrodesis.
[Barei DP, et al. Fractures of the calcaneus. Orthop Clin North Am. 2002 Jan; 33(1): 263-85.]
-Goals of therapy are to restore calcaneal height, decrease calcaneal body widening (reduce lateral wall blow-out), take it out of
varus, and articular reduction.
-Review the lateral extensile surgical approach [Benirschke SK, Sangeorzan BJ. Extensive intraarticular fractures of the foot. Surgical management
of calcaneal fractures. CORR. 1993 Jul; 292: 128-134.]
-Complications: Wound healing, arthritis, lateral ankle impingement, malunion, nonunion, etc.
-[Benirschke SK, Kramer PA. Wound healing complication in closed and open calc fractures. J Orthop Trauma. 2004; 18(1): 1-6.]
-[Cavadas PC, Landin L. Management of soft-tissue complications of the lateral approach for calcaneal fractures. Plast Reconstr Surg. 2007;
120(2): 459-466.]

63
AJM Sheet: Ankle Fracture Evaluation

-The standard trauma work-up again applies with primary and secondary surveys. The following describes unique subjective
findings, objective findings, diagnostic classifications and treatment considerations.
-Residents and attendings love to ask questions about ankle fractures for whatever reason, so this is certainly a subject where
you should know the classification systems cold, and do a lot of the additional readings. We’ll keep it brief here.

-Relevant Anatomy to Review (not just for this topic; think lateral ankle instability, peroneal tendonopathy, sprains, etc.):
-Ankle Ligaments: -Lateral: ATFL, CFL, PTFL
-Medial: Superficial Deltoid: superficial talotibial, naviculotibial, tibiocalcaneal
ligaments
Deep Deltoid: anterior talotibial and deep posterior ligaments
-Syndesmotic Ligaments: -AITFL, PITFL (and inferior transverse tibiofibular ligament), Interosseous ligament

-Classifications:
-Ottawa Ankle Rules
[Stiell IG, et al. A study to develop clinical decision rules for the use of radiology in acute ankle injuries. Ann Emerg Med. 1992; 21(4): 384-90.]
-Developed by ED docs to minimize unnecessary radiographs following ankle sprains. X-ray only required if:
-Bone tenderness along distal 6cm of posterior edge of fibula or tibia
-Bone tenderness at tip of fibula or tibia
-Bone tenderness at the base of the 5th met
-Bone tenderness on the navicular
-Inability to bear weight/walk 4 steps in the ED

-Lauge-Hansen Classification
-First submitted as a doctoral thesis [Lauge-Hansen N, Anklebrud I. 1942]. Co-authored with a guy named “Ankle”-brud!
-[Lauge-Hansen N. Fractures of the ankle: analytic, historic survey as the basis of new experimental roentgenologic and clinical investigations.
Arch Surg 1948; 56: 259.]
Stage I Stage II Stage III Stage IV
Supination Lateral collateral Near vertical medial NA NA
ligament tear/ avulsion malleolar fx (Muller D)
Adduction fibular fx (Weber A)
Pronation Transverse avulsion fx AITFL syndesmotic Short, oblique lateral NA
medial rupture or avulsion of malleolus fracture
Abduction malleolus/deltoid its insertion (Weber B)
rupture (Muller B) Transverse on lateral
radiograph
Supination AITFL syndesmotic Spiral lateral malleolus PITFL syndesmotic Transverse avulsion fx
rupture or avulsion of fracture (Weber B) rupture or avulsion of medial
External its insertion Long, posterior spike its insertion malleolus/deltoid
Rotation on lateral radiograph rupture (Muller B)

Pronation Transverse avulsion fx AITFL syndesmotic Oblique or spiral fibular PITFL syndesmotic
medial rupture or avulsion of fracture rupture or avulsion of
External malleolus/deltoid its insertion suprasyndesmotic its insertion
Rotation rupture (Muller B) (Weber C)

Pronation
Dorsiflexion
-The problem with the Lauge-Hansen classification: This was an experimental/laboratory study looking at the
result of forced talar movement on a fixed tibia-fibula. But most ankle fractures in real-life occur when a moving
tibia-fibular acts on a fixed foot.

-Danis-Weber/AO Classification for lateral malleolar fractures (From AO Group)


AO Type A: Type B: Type C:
Infrasyndesmotic Transyndesmotic Suprasyndesmotic
1 Isolated Isolated Simple diaphyseal fibular fx

2 With medial malleolar fx With medial malleolar fx or Complex diaphyseal fibular fx


deltoid rupture
3 With posterior-medial fx With medial lesion and posterior- Proximal fibular fx
lateral tibial fx

-Mueller Classification for medial malleolar fractures (From AO group)


A B C D
Avulsion Transverse at level Oblique Near vertical
of mortise

64
AJM Sheet: Ankle Fracture Treatment

-Additional named fractures associated with the ankle:


-Tillaux-Chaput fx: AITFL avulsion from the anterolateral tibia
-Wagstaff fx: AITFL avulsion from the anteromedial fibula
-Volkmann fx: PITFL avulsion from the posterior-lateral tibia
-Bosworth fx: PITFL avulsion from the posterior-medial fibula
-Maisonneuve fx: Weber C-type proximal fibular fracture that occurs within 10cm of the fibular neck
-Pott’s fx: Generic term for a bimalleolar ankle fracture
-Destot fx: Generic term of a trimalleolar ankle fracture
-Dupuytren fx: At least a bimalleolar fracture when the talus gets lodged up between the tibia and fibula
-Posterior Malleolar Fractures: Different than an avulsion fracture of the PITFL; this is a true fx involving a portion of the
tibial plafond cartilage. CT is usually done to estimate a percentage of the involved joint space. The rule of thumb (although
certainly not proven) is that fractures involving >25-30% of the joint space require ORIF.

-Principles of Fixation:
-This is one area where there is a lot of controversy in the medical literature. There are certainly some things you want to
accomplish besides the generic concept of “anatomic reduction”. I can’t get too much into it in this limited space, but I will try
and give you a couple sides of the argument and some reading to do. The question you are really trying to answer is: “How
reduced is reduced enough?” Then we’ll briefly cover some specific aspects of the surgeries themselves. One thing to
appreciate is that most of these arguments are made about SER fractures (because they are the most common):
-Restore fibular length
-Most people agree that the fibular fracture is the dominant fracture. In other words, if you adequately reduce
the fibula, then the other fractures and dislocations more or less fall into line because of the soft tissues (poor
man’s definition of the Vassal Principle). It doesn’t mean that the other fractures don’t require fixation, but
it means there’s no real sense in fixating the other fractures unless you have the dominate fracture fixated (or
at least reduced).
-The other concept is that a fixed fibula is essentially acting as a buttress, keeping the talus within the ankle
mortise.
-The fibula is generally shortened in ankle fractures, so you want to get the full length back with your
reduction (generally visibly seen by reduction of the posterior spike on a lateral view).
-[Yablon IG, et al. The key role of the lateral malleolus in displaced fractures of the ankle. JBJS-Am. 1977; 59(2): 169-173.]
-Restore the ankle mortise (medial clear space and the syndesmotic gap)
-This goes back to the fibula keeping the talus in the ankle mortise. The classic article you need to know is
Ramsey and Hamilton that showed a 42% decrease in the tibiotalar contact area when the talus was displaced
1mm laterally. From this, people inferred that if the talus isn’t perfectly reduced back into the mortise, then
gross instability occurs.
-This is assessed by:
-Medial clear space (from the talar shoulder): Should be ~4mm or less after reduction
-Tib-Fib Overlap: Approximately >10mm on AP view at 1cm superior to the joint line
-Talar Tilt: <10 degrees absolute, or <5 degrees compared to other side
-[Ramsey PL, Hamilton W. Changes in tibiotalar area of contact caused by lateral talar shift. JBJS-Am. 1976; 58(3): 356-7.]
-[Park SS, et al. Stress radiographs after ankle fracture: the effect of ankle position and deltoid status on medial
clear space measurements. J Orthop Trauma. 2006; 20(1): 11-18.]
-Fix the syndesmosis?
-Another area of controversy where there is no clear answer is when and how to fixate the syndesmosis with
internal fixation. One point is clear: the purpose of placing internal fixation across the syndesmosis is to
stabilize the fibula against the tibia to prevent lateral migration of the talus and instability. If the fibula is
stable against the tibia with all of your other fixation, then you don’t really need any additional fixation.
How can you tell? Radiographic findings and the Cotton hook test for instability intra-operatively.
-Other questions where people have opinions, but no clear answers are: What type of screws? How many
screws? How many cortices? How far above the ankle? Temporary vs. permanent fixation? Weight-
bearing? etc.

-Lateral Malleolus:
-Fracture is primarily reduced and fixated with a single 2.7mm cortical screw with interfrag compression.
-Then a generic 1/3 tubular plate or a specialized contoured plate is used for buttress stabilization.
-Attempt for 6 cortices proximal to fracture with 3.5 bicortical screws
-Get as many distal screws as you can. 3.5 bicortical if above the ankle joint. 4.0 unicortical if not.
-Proximal fibular fractures still amendable to 1/3 tubular plating, but may need to double-stack the plates.
-Should appreciate the concept of lateral vs. posterior anti-glide plating.
-Medial Malleolus:
-Several options including 4.0 cancellous, K-wires, plating, cerclage, etc.
-Additional Reading:
-[Mandi DM, et al. Ankle fractures. Clin Podiatr Med Surg. 2006 Apr; 23(2): 375-422.]
-[Mandracchia DM, et al. Malleolar fractures of the ankle. A comprehensive review. Clin Podiatr Med Surg. 1999 Oct; 16(4): 679-723.]
-[Kay RM, Matthys GA. Pediatric ankle fractures: evaluation and treatment. J Am Acad Orthop Surg. 2001; 9(4): 269-78.]
-[Jones KB, et al. Ankle fractures in patients with diabetes mellitus. JBJS-Br. 2005; 87(4): 489-95.]
-[Espinosa N, et al. Acute and chronic syndesmosis injuries: pathomechanics, diagnosis and management. Foot Ankle Clin. 2006 Sep; 11(3): 639-57.]
65
AJM Sheet: General Tendon Trauma

-Mechanism of Injury
-Tendon is actually the strongest part of the muscle-tendon-insertion system. It is much more likely for the complex
to fail at the myotendinous junction or at the tendinous insertion, but acute tendon injuries do occur. They are usually the
result of direct trauma, or overload on an intrinsically weakened tendon.
-Tension overload on a passive muscle
-Eccentric overload on an actively contracting muscle
-Laceration
-Blunt Trauma

-Factors which can intrinsically weaken tendons


-Increased age: -increased cross-linking of collagen fibrils decreases tendon elasticity
-decreased reaction time and muscular contraction speed
-decreased vascularity
-Sex: -M>F
-Systemic inflammatory process: -RA, SLE, Gout, etc.
-Underlying endocrine dysfunction: -Xanthoma (hyperbetalipoproteinemia), DM, Hyperparathyroidism
secondary to renal failure, hyperthyroidism, infection, intratendinous calcifications, etc.
-Medications: -Fluoroquinolones, Corticosteroids

-Tendon Healing
-As with most tissue, there is a generalized inflammation, reparative and remodeling phase.
-Week 1: Severed ends fill in with granulation tissue
-Weeks 2-3: Increased paratenon vascularity; collagen fibril alignment
-Week 4: Return to full activity without immobilization

-Imaging in Diagnosis of Acute Tendon Injury


-Plain Film Radiograph: -May see avulsions, soft tissue swelling, accessory bones/calcifications

-Tenograph: -Radiopaque dye injected into tendon sheath and viewed on plain film radiograph
-Technically difficult with many false positives and negatives

-Ultrasound: -Tendon normally appears hyperechoic to muscle on US.


-Look for discontinuity of fibers, possible alternating hyperechoic/hypoechoic bands, and an area
of intensely hyperechoic hematoma.
-It is very important that the US head is held perpendicular to the long axis of the tendon.

-CT: -Tendon normally appears as a homogenous, well-circumscribed oval surrounded by fat on CT. It
normally has a higher attenuation than muscle.
-Will be able to appreciate discontinuity on CT with injury.

-MRI: -T1: Tendons normally have a uniform low-intensity (very black). Will be uniform with variable
high-intensity signal with injury.
-T2: Tendons are normally relatively low-intensity. Will light up with high-intensity signal with
injury.
-Remember the magic angle phenomenon. Any MRI signal shot at 55 degrees to the course of
the tendon will show a false-positive damage signal. Very common in the peroneals.
-[Mengiardi B, et al. Magic angle effect in MR imaging of ankle tendons: influence of foot positioning on prevalence and
site in asymptomatic patients and cadaver tendons. Eur Radiol. 2006 Oct; 16(10): 2197-2206.]

-Principles of Repair
-It is possible, but rare to get acute tendon injury to any of the long tendons of the leg. An Achilles tendon work-up
will be featured in another AJM sheet, but realize there are some basic principles that apply to any tendon.
-One is generally able to primarily repair the tendon. Non-absorbable suture is preferred.
-Special attention should be paid to vascular supply. Remember that the majority of a tendon’s vascularity comes
from the mesotenon, and therefore should be preserved as much as possible.
-If primary repair is not possible, consider using lengthening tendon slides, tendon grafts, tendon transfers and
biomaterials such as Graft-Jacket (allograft dermal tissue matrix) or Pegasus (equine pericardium) to restore the
integrity of the tendon.
-The goal of treatment should be to allow early PROM without gapping of the tendon.

66
AJM Sheet: Achilles Tendon Rupture Work-Up

Subjective:
CC: Typical complaint is pain, weakness and swelling in the back of the leg following an acute injury. The typical patient is
the “weekend warrior” type. This is a 30-50 y/o male participating in a strenuous athletic activity after a generally inactive
lifestyle.
HPI: Nature: Pain, weakness and swelling. Pain is surprisingly non-intense allowing the patient to ambulate. The patient
may relate an audible “pop” or “snap”. They may also relate feeling like they were “kicked or shot” in the back of the leg.
Location: Distal posterior leg. The left leg is more affected. Some people theorize that this has to do with the
majority of people having right-handedness and a greater strength and proprioception of the RLE.
Duration, Onset, Course: Acute onset with gradually progressive increase in swelling and edema.
Mechanism of Action: -Three classic MOA are described:
-Unexpected dorsiflexion with triceps contraction
-Pushing off during WB with the leg extended (tennis lunge)
-Violent dorsiflexion on a plantarflexed ankle
-Also consider lacerations and blunt trauma
Previous History: obviously more likely to re-rupture
PMH: -Inflammatory conditions: RA, SLE, Gout
-Endocrine dysfunction: DM, Renal failure with hyperparathyroidism, hyperthyroidism, Xanthoma
(hyperbetalipoproteinemia)
-Infection: Syphilis
Meds: -Corticosteroid injection
-Fluoroquinolone use
SH: -Smoking
-Sedentary lifestyle with weekend activity

Objective:
Derm: -Posterior, Medial and Lateral Ecchymosis
-Open lesion associated with laceration
Vasc: -Posterior, Medial and Lateral edema
Neuro: -Sural Neuritis
Ortho: -Palpable gap (“hatchet strike defect”)
-Positive Thompson test
-Negative Jack’s test
-Pain in the area
-Increased PROM ankle dorsiflexion
-Decreased AROM ankle plantarflexion
-Retraction of proximal gastroc belly
-Apropulsive gait
Other specific tests: -Mattles test: Foot should be in plantarflexed position with patient prone and knee at 90°
-Simmonds’ test: Foot should be in plantarflexed position with patient prone
-Various needle tests (O’Brian, Cetti)
-Toygar’s skin angle: Normally 110-125 degrees. Increases to 130-150 degrees with rupture.

Imaging:
-Plain film: -r/o Rowe Type IIB avulsion fracture
-Radiodense gap
-Obliteration of Kager’s triangle
-Soft tissue edema
-US: -Alternating hyperechoic and hypoechoic bands
-Hyperechoic hematoma
-MRI: -TI: -Ill-defined low-intensity with mixed high-intensity signal
-T2: -High-intensity signal from hematoma

67
AJM Sheet: Achilles Tendon Rupture Treatment

-Anatomy Review
-Muscles of the Triceps Surae (origins, insertions, NV supply, action)
-Plantaris (origins, insertions, NV supply, action)
-Segmental Blood Supply of Tendon
-“Twisting” of tendon

-Specific Information regarding the Watershed Area


-Lagergren and Lindholm
-Used microangiographic technique on human cadavers
-Found decreased vascularity 2-6cm proximal to insertion
-Theorized this was secondary to atrophy from inactivity
-Conflicting information from laser Doppler flowmetry studies
-Found uniform vascularity throughout tendon
-Found decreased vascularity with age and in men
-Found decreased vascularity with physical loading/stress of tendon, specifically at insertion
-Leadbetter
-Found increased stress/strain at the watershed area regardless of vascularity

-Kuwada Classification of Achilles Tendon Ruptures


[Kuwada GT. Classification of tendo Achilles repair with consideration of surgical repair techniques. J Foot Surg. 1990; 29(4): 361-5.]
-Type I: Partial tear involving <50% of tendon. Note that in a partial Achilles tear, the posterior fibers are torn first. So the direction of the
tear/rupture is from posterior to anterior.
-Type II: Complete tear with <3cm deficit
-Type III: Complete tear with a 3-6cm deficit
-Type IV: Complete tear with a >6cm deficit

-Puddu Classification of Chronic Achilles Pathology


-Peritendonitis: Inflammation of the surrounding tissues, not the tendon itself. This pain will remain stationary as the tendon is taken through a
range of motion.
-Tendonosis: Intra-tendinous degeneration. This pain will move proximally and distally as the tendon is taken through a range of motion.
-Peritendonitis with tendonosis: combination of the two pathologies.

-The podiatric surgeon is faced with three options: do nothing, cast immobilization and surgical repair. There’s a lot of information out about this in the
medical literature now, particularly with open repair vs. immobilization and when to start weight-bearing/PT.

-Do nothing
-Gap will eventually fill in with fibrotic scar tissue
-Usually requires later surgical intervention

-Cast Immobilization
-AK cast versus SLC
-Schuberth is proponent of AK casting
-Knee should be in a 20 degree flexed position
-General recommendations:
-Gravity equinus cast x 4 weeks
-Reduction of 5 degrees every 2 weeks to a neutral ankle position (~4-6 weeks)
-Heel lift and PT until normal ankle PROM
-Return to full activity at approximately 6 months

-Surgical Repair
-Surgical approach
-Midline to medial incision to avoid superficial neurovascular structures
-Pt in a prone or supine frog-legged position
-Use full-thickness flaps with emphasis on atraumatic technique
-Primary Open Repair
-Keith needles with non-absorbable suture (or fiberwire) with absorbable sutures to reinforce
-There are three common stitches used:
-Bunnell: Figure of 8 or weave stitch
-Krakow: Interlocking stitch
-Kessler: Box stitch
-Augmented Open Primary Repair
-Lynn: Plantaris is fanned out to reinforce
-Silverskoild: 1 strip of gastroc aponeurosis brought down and twisted 180 degrees
-Lindholm: Utilizes multiple strips of gastroc aponeurosis
-Bug and Boyd: Strips of fascia lata are used to reinforce
-V Y lengthening of the proximal segment with primary repair
-Reinforcement with FHL
-Graft Jacket, Pegasus, etc.
-Percutaneous Primary Repair
-Ma and Griffith described a percutaneous Bunnell-type approach
-May be associated with high re-rupture rates

-Post-Op Treatment
-SLC in gravity equinus with gradual reduction over 6-10 weeks
68
AJM Sheets: Peri-Operative Medicine and Surgery
Contents:

Peri-Operative Medicine
-Admission Orders (page 70)
-Electrolyte Basics (page 71)
-Glucose Control (page 72)
-Fluids (page 73)
-Post-Op Fever (page 74)
-DVT (page 75)
-Pain Management (page 76)

General Surgery Topics


-AO (page 77)
-Plates and Screws (page 78)
-Suture Sheet (page 79)
-Surgical Instruments (page 80)
-Power Instrumentation (page 81)
-Biomaterials (page 82)
-External Fixation (page 83)
-Bone, Bone Healing and Wound Healing (page 84)

Specific Surgery Topics


-How to “Work-Up” a Surgical Patient (page 85)
-Digital Deformities (pages 86-87)
-Lesser Metatarsals (page 88)
-5th Ray (page 89)
-HAV (page 90-91)
-HAV Complications (page 92)
-HL/HR (pages 93-94)
-Pes Plano Valgus (pages 95-96)
-Cavus (pages 97-98)
-Equinus (page 99)

This particular section is intended to be more general, as opposed to a specific surgical study
guide. It is ridiculous to think that you could learn foot and ankle surgery in 100 pages, especially with
only 15 pages dedicated to specific deformities. In other words, you should absolutely not be doing all of
your specific surgical preparation for externships and interviews from the PRISM. Many of the Sheets
from the Specific Surgery Topics section are simply summarizations of the 3rd edition of McGlamry’s
chapters for example. This may be an area where you feel the PRISM could be updated in the future.

Again, I said that while I was studying for the Diabetic Foot Infection work-up, I tried to learn as
much as possible on the topic and really tried to “wow” the attendings at the interview. However, my
strategy was different when dealing with trauma and the specific surgical work-ups. Here I tried to
demonstrate “competence,” as opposed to “mastery” of the material. With specific surgeries, you’re really
not supposed to have strong, pre-formed opinions as a student or as an intern. That’s what your residency
is for, developing surgical opinions. If you already know what to do in every surgical situation, then what’s
the point of doing a residency? So while on externships and at the interview, you should really try to walk
a fine line between:
1. Displaying competence in knowledge of the baseline material
2. Displaying that you still have a lot to learn, and that you are eager to learn it.

Page 85’s “How to Work-Up a Surgical Patient” gets into this concept a little deeper.

69
AJM Sheet: Admission Orders/ADC VANDILMAX
(Note: If I wanted to be mean during an interview, I would have you write out a set of admission or post-op orders as I was asking you other questions.)

Admission: Pt is admitted to the general medical floor on the Podiatric Surgery Service under Dr. Attending.
Most patients on the podiatric surgery service are admitted to the general medical floor or a surgical floor. Any pts
admitted to a critical care unit or telemetry unit will probably be on a medicine service with a podiatric surgical consult.

Diagnosis: Infection of bone of right 2nd toe


Always use terminology that everyone in the hospital can understand, but also be as specific as possible.

Condition: Consider: -Stable -Fair -Guarded -Critical


Podiatric surgery pts will generally always be in stable or fair condition.

Vitals: Vitals recorded q8 hours per nursing.


Always designate how often you want them recorded. Also common is “q-shift.”
Consider neurovascular checks to the affected limb if indicated.

Ambulatory Status: Consider: -CBR (Complete bed rest) -As tolerated


-NWB -OOB to chair
-PWB
Always designate which leg the order is for. Be specific with PWB status (“toe-touch” or “heel-touch”). If order is for
CBR, consider DVT ppx and a bed pan order. If the order is OOB, specify # of times and length per day.
Also consider Physical Therapy and/or Occupational Therapy orders here.

Nursing Instructions: Consider: -Accuchecks (how often and when?) -Ice and elevation
-Bedside Commode -Dispense Post-op shoe/Crutches
-Wound Care -Dressing Instructions
-Drain management -Off-loading instructions
-Spirometry

Diet: Consider: -Regular diet -ADA 1800-2200 calorie -Mechanically soft


-Renal diet -Cardiac diet -NPO
-Decreased Na -Decreased K+

Ins/Outs/IVs: Consider: -Measurement and recording of Ins and Outs (especially dialysis pts)
-Foley
-IV Fluids

Labs: Consider: -CBC with diff -Chem-7/Met Panel -Coags


-Type and Screen -Wound cultures -Blood cultures
-D-Dimer -HbA1c -CRP
-ESR
Always detail when the labs should be done. For example, initial CBC and Chem-7 should be taken “upon arrival to the
floor.” Additionally, 2 sets of blood cultures should be taken from 2 different sites.

Medications: Consider: -Write out all at-home medications in full -Pain medication
-Antibiotics -Insomnia
-Anti-emetics -DVT ppx
-Constipation -Diarrhea
-SSI -Fever
-Throat lozenges -Anti-pruritic
Be as specific as possible. SSI needs to be written out in full. Many medications require hold parameters. For example,
fever medications should not be given unless the temperature reaches 101.5° F. Anti-HTN agents should be held if the blood
pressure or heart rate drops too low.

Ancillary Consults: Consider: -General Medicine -Infectious Disease


-Vascular Surgery -Cardiology
-PT/OT -PM&R
-Pulmonary -GI
-Renal -Social Work
-Home Care -Case Manager

X-rays/Imaging: Consider: -Plain film radiographs -CT scans -US Doppler


-MRI -CXR -Bone Scans
-Vascular Studies -EKG
70
AJM Sheet: Electrolyte Basics

-Sodium:
-Hyponatremia
-Manifestations: Primarily neurologic, lethargy, headache, confusion, obtundation
-Treatment: -Restrict water intake and promote water loss -Correct underlying disorder
-Replace Na+ deficits
-Hypernatremia
-Manifestations: Change in mental status, weakness, neuromuscular irritability, focal neurologic deficits, coma,
seizures
-Treatment: -Replace water loss and promote sodium excretion
-Water deficit = ([Na+]-140)/140 x Total Body Water in liters
-Rapid correction of either of these disorders is dangerous due to rapid shifts of water in and out of brain cells. It should
therefore be corrected slowly over 48-72 hours. Aim correction at 0.5 mEq/L/hr with no more than a 12 mEq/L correction over the first 24
hours.

-Potassium:
-An abnormal potassium level is a major reason a surgery will be cancelled and/or delayed. You should have a specific
understanding how to raise and lower potassium levels in the peri-operative setting.
-Hypokalemia
-Manifestations: Fatigue, myalgia, muscular weakness, cramps, arrhythmia’s, hypoventilation, paralysis, tetany
-Treatment: -Minimize outgoing losses -Treat underlying cause
-Correct K+ deficit via oral or IV means (K+ riders added to fluid, oral KCL, etc.)
-Hyperkalemia
-Manifestations: Cardiac toxicity (peaked T waves, prolonged PR, torsades de pointes), muscle weakness, paralysis,
hypoventilation
-Treatment: -Increase cellular uptake of K+
-Insulin (10-20 units) with 50 g IV glucose
-IV NaHCO3 (3 ampules in 1L of 5% dextrose)
-Albuterol (5-10mg nebulized over 30-60 minutes)
-Increase K+ excretion
-Loop diuretic, Thiazide diuretic
-Kayexalate (cation exchange resin) (25-50mg mixed with 100ml 20% sorbitol to prevent constipation)
-Dialysis
-Calcium Gluconate (10ml of 10% solution over 2-3 minutes emergently to reduce membrane excitability)

-Chloride and Carbon Dioxide:


-Not going to talk much about this, but you should have a basic understanding of Acid-Base Regulation.
-Equation for determining Anion Gap:
Anion gap (all units mmol/L) = (Na + K) - (Cl + [HCO3-])
-Normal gap (~8-20mmol/L)
-Negative/lowered gap (<8mmol/L): Alkalotic state
-Positive/elevated gap (>20mmol/L): Acidotic state
-MUDPILES algorithm: methanol/metformin, uremia, diabetic ketoacidosis, propylene glycol,
infection, lactate, ethanol, salicylate/starvation
-BUN and Creatinine:
-Measures of kidney function and hydration status
-BUN (Blood Urea Nitrogen): Protein metabolism waste product eliminated by the kidneys. This can be increased if your
kidneys aren’t eliminating it properly, or in a dehydrated state where it’s a relatively high concentration.
-Creatinine: A more direct measure of kidney function from elimination of this skeletal muscle waste product.
-Creatinine clearance and estimated glomerular filtration rate (GFR) with the Cockcroft-Gault Equation:
[(140-Age in years) x Weight in kg] / [72 x Serum creatinine] x 0.85 if female
-GFR < 60 ml/min indicates chronic kidney disease; < 15 indicates kidney failure
-Antibiotics and other drugs should be dosed appropriately in these situations
-Renal protective agents are utilized prior to procedures that are known to affect the kidneys in patients with kidney
disease. A common example of this is an angiogram with dye to evaluate the vascular status of a patient with diabetic foot disease.
-Pre-procedural hydration -Mucomyst (N-acetylcysteine) (also used for acetaminophen OD)
-Sodium Bicarb Protocols
-[Lawlor DK. Prevention of contrast-induced nephropathy in vascular pts. Ann Vasc Surg. 2007 Sep: 593-7.]

-Glucose covered in another AJM sheet


71
AJM Sheet: Blood Glucose and Glycemic Control

-The importance of in-patient management of blood glucose cannot be overstated. This is an area however
where medicine tends to be very passive with regard to intervention. Rigid control of blood glucose in the in-patient
setting has been definitively shown to:
-Reduce mortality -Reduce in-patient complications
-Reduce infection rates -Decrease length of stay
-Reduce hospital costs
-Specifically with regards to diabetic foot disease, a single blood glucose level higher than 150-175mg/dl
significantly limits the function of the immune system for a period of days, particularly cytokine activation and
recruitment.
-My favorite article of the 2006-7 academic year was Inzucchi SE. Management of Hyperglycemia in the Hospital Setting. NEJM.
Sep 2006; 355: 1903. It is a must-read on this topic. I also strongly recommend obtaining a FREE copy of the Yale Diabetes
Center Diabetes Facts and Guidelines 2006. They will mail it to you (FOR FREE!) by calling 203 737-1932 or emailing
silvio.inzucchi@yale.edu. An online version is also available at http://info.med.yale.edu/intmed/endocrin/yale_diab_ctr.html.
You are a complete sucker if you don’t take advantage of this resource. And if you are really interested in this topic, research
the work of the Portland Diabetic Project.

Oral Agents
-Sulfonylureas: Bind to β-cell receptors stimulating insulin release
-Glyburide (Micronase) -Glipizide (Glucotrol) -Glimepiride (Amaryl)
-Biguanides: Decrease production of glucose in the liver
-Metformin (Glucophage)
-Thiazolidinediones: Increase peripheral cellular response to insulin
-Rosiglitazone (Avandia) -Pioglitazone (Actos)
-α-glucosidase inhibitors: Reduce intestinal carbohydrate absorption
-Acarbose (Precose) -Miglitol (Glyset)

Insulins
Type Onset Peak Duration
Rapid Acting
Lispro (Humalog) 10-15 minutes 1-2 hours 3-5 hours
Aspart (Novolog) 10-15 minutes 1-2 hours 3-5 hours
Short Acting
Regular 0.5-1hr 2-4 hours 4-8 hours
Intermediate Acting
NPH 1-3 hours 4-10 hours 10-18 hours
Lente 2-4 hours 4-12 hours 12-20 hours
Long Acting
Glargine (Lantus) 2-3 hours None 24+ hours
Detemir (Levemir) 1 hour None 24 hours
Combinations
70/30 0.5-1 hour 2-10 hours 10-18 hours
(70% NPH/30% Regular)

In-patient Recommendations
-There is increasing data that sliding scales are completely inefficient at in-patient glucose management.
Sliding scales are passive, reactionary scales that compensate after a hyperglycemic incident occurs. Inzucchi
recommends the following, instead of a sliding scale:
-Basal Rate: Lantus or other long acting
-Start 0.2-0.3 Units/kg/day; then increase 10-20% q1-2 days prn
-Prandial Coverage: Novolog or other rapid acting
-Start 0.05-0.1 Units/kg/day; then adjust 1-2 Units/dose q1-2 days prn

Diabetic NPO Recommendations


-Type 2 DM: -1/2 the normal dose of long acting if they get any
-BG checks q6 hours with short acting agent available for coverage
-D5W or D5-1/2NS at 50-75cc/hr while NPO
-Type 1 DM: - Strongly consider an insulin drip
-1/2 – 2/3 normal dose of long acting agent
- BG checks q6 hours with short acting agent available for coverage
- D5W or D5-1/2NS at 75-100cc/hr while NPO

72
AJM Sheet: Fluids

-Fluid management is a difficult topic to cover because it can be used for a variety of different problems/purposes. It can be
used to maintain fluid balance in a patient who is NPO, correct electrolyte disturbances, and/or provide glucose to name just a
few examples. This sheet will cover the basics of short-term maintenance therapy and show differences in electrolyte
concentrations between the most common fluids.

-Maintenance therapy for the NPO patient


-An NPO patient is still losing water that needs to be replaced to ensure homeostasis. Sources of water loss include:
-Urine output: At least 500ml/day
-Insensible water losses (Skin and Respiration): At least 500ml/day
-This can increase by 150ml/day for each degree of body temperature of 37°C.
-Gastrointestinal losses: Extremely variable
-Direct blood volume loss from the surgery itself
-Electrolytes are also lost to varying degrees. In the short term, it is usually only necessary to replace Na+, K+ and
glucose. The other electrolytes usually do not need replacement until around 1 week of parenteral therapy.

-Pediatric Considerations
-Pediatric patients should be aggressively rehydrated after a surgical procedure for two reasons:
-They will lose a higher percentage of their total fluid volume during a procedure.
-They have a tendency to “third space” and shift fluid balances in the perioperative period.
-To determine the total intravascular volume of a pediatric patient:
-The first 10kg of body weight account for about 80ml/kg.
-So a 7kg kid would be (7x80) = 560ml
-The next kg’s account for about 70ml/kg
-So a 25kg kid would be (10x80 + 15x70) = 1850ml

-General Recommendations:
-At the very least you should replace fluid to account for water loss. This is at least 1L/day, but you can
certainly increase this and lose the excess through the urine.
-It is also recommended to provide some electrolyte supplementation:
-Na+: 50-150 mEq/day
-K+: 20-60 mEq/day
-Glucose: 100-150g/day to minimize protein catabolism and ketoacidosis

-Common parenteral solutions:

IV Solution Osmolality (mOsm/kg) Glucose (g/L) Na+ (mEq/L) Cl- (mEq/L)

D5W 278 50 0 0
D10W 556 100 0 0
D50W 2778 500 0 0
0.45% NaCl 154 (5% available) 77 77
0.9% NaCl 308 (5% available) 154 154
3% NaCl 1026 0 513 513
Lactated Ringer’s 274 (5% available) 130 109
-LR also contains 4 mEq/L K+, 1.5 mEq/L Ca2+, and 28 mEq/L lactate

-Common administrations:
-Normal adult: NS or 1/2 NS or LR at 75-120ml/hr +/- 20mEq KCl
-Diabetic patients: D5-1/2NS at 50-100ml/hr +/- 20mEq KCl while NPO
There usually isn’t a need to deliver extra glucose (D5) to diabetic patients while they are PO.

-The key to fluid management is an understanding and knowledge of exactly why you are giving fluids in the first place, what
you hope to accomplish, what substances you are giving in the fluid and how much you are giving.

-Obvious care needs to be taken with diabetic patients, those with renal pathology, and those with CHF.

Additional Reading:
-[Grocott MP, et al. Perioperative Fluid Management and Clinical Outcomes in Adults. Anesth Anal. 2005 Apr; 100(4):1093-106.]
-[Paut O. Recent developments in the perioperative fluid management for the paediatric patient. Curr Opin Anaesthesiol. 2006 Jun;19(3):268-77.]

73
AJM Sheet: Post-Op Fever

-General Information
-When dealing with a fever work-up, always note what the baseline temperature of the patient is and the method of
measurement.
-Fever in most institutions is defined as greater than 101.5° F.
-Temperatures between 98.6-101.5° are low-grade fevers.

-Intra-operative causes of fever


-Inflammatory process of the surgical procedure itself
-Pain
-Transfusion Reaction
-Malignant Hyperthermia
-Pre-existing Sepsis

-The 5 “W’s” of Post-Operative Fever


-Wind: Atelectasis, aspiration pneumonia, PE
-Wound: Surgical site infection, thrombophlebitis (IV site), pain
-Water: UTI, dehydration, constipation
-Walk: DVT
-Wonder Drugs: Virtually any drug can cause fever, but the most common are antimicrobials and heparin.

-Timeline General Guide


-0-6 hours post-op: Pain, anesthesia rxn, rebound from cold OR, endocrine causes (thyroid crisis, adrenal
insufficiency)
-24-48 hours post-op: atelectasis, aspiration pneumonia, dehydration, constipation
-72+ hours: infection (3-7 days), DVT, UTI, drug allergy, thrombophlebitis

-Temperature General Guide (in degrees F)


-107: Anesthetic Hyperthermia
-106: -
-105: Blood transfusion reaction
-104: Closed abscess
-103: Atelectasis; pneumonia; drug reaction; liver disease
-102: Wound infection
-101: Draining abscess
-100: Benign post-op fever; post-anesthesia overshoot

-General Knowledge
-Usually only two infectious agents can cause a fever within a few hours of surgery:
-Group A Strep (GAS)
-Clostridium perfringens
-Dialysis patients typically run approximately 1 degree F cooler than the normal population, so a fever for HD
patients wound be defined as 100.5° F. This is hypothesized to be due to a resetting of the hypothalamic set point.
-The majority of causes of fever are non-infectious. AJM always carries with him a copy of the DDX of fever
copied from Harrison’s text (it’s 2 pages long!). Common non-infectious causes of post-op fever include:
-Surgical site inflammation
-Seroma
-Hematoma
-Pain
-The purpose of any fever work-up is to find the source!
-If you are thinking infection, then infection from where: Surgical site? Pulmonary? Urine? Blood? Does
the patient have any peripheral vascular access lines?

74
AJM Sheet: Deep Vein Thrombosis (DVT)

-Signs and Symptoms


-Pain
-Homan’s Sign: Pain in calf with dorsiflexion of the ankle
-Pratt’s Sign: Pain with compression of the calf
-Edema -Fever -SOB
-Calor -Palpation of clot

-Risk Factors
-Virchow’s Triad: -History of DVT -Collagen Vascular Dz
-Hypercoagulable state -Family History of DVT -Trauma
-Immobilization -Pregnancy -Infection
-Vessel Wall Injury -Oral Contraceptives -Post-partum
-Age > 75 -Hormone Replacement Therapy
-Malignancy -Obesity -HIV/AIDS
-There is also the acronym I AM CLOTTED: Immobilization, Afib/CHF, Malignancy/MI, Coagulopathy, Longevity (age),
Obesity, Trauma, Tobacco, Estrogen/BCP/HRT, DVT/PE history.
-Diagnosis
-Compression Ultrasound: can actually visualize the clot
-D-Dimer > 500ng/ml: Not sufficient as a stand alone test
-Consider full coagulation work-up for hypercoagulable states
-Contrast venography
-Impedance plethysmography

-Treatment
-Goals of Treatment: 1. Prevent pulmonary embolism
2. Prevent clot extension
3. Prevent recurrence
-Immediate Anti-Coagulation
-IV Unfractionated Heparin
-Law of 8018
-Initial Dose 80mg/kg IV bolus and then 18mg/kg/hour
-PTT should be checked q6 until it stabilizes at 1.5-2.5X normal (46-70s)
-Goal is to get PTT in this range
-LMWH may also be used
-Enoxaparin (Lovenox): 1mg/kg subcutaneous q12

-Heparin Dosing Guide


-Initial Dose: Law of 8018 with PTT checks q6
-If PTT <35s: 80 units/kg IV bolus, then increase infusion rate by 4 units/kg/hr
-If PTT 35-45s: 40 units/kg IV bolus, then increase infusion rate by 2 units/kg/hr
-If PTT 46-70s: No change to dosing. Continue with 18mg/kg/hour infusion rate
-If PTT 71-90s: Decrease infusion rate by 2 units/kg/hr
-If PTT >90s: Hold infusion for 1 hour, then decrease infusion rate by 3 units/kg/hr
-Continued Anti-Coagulation
-Warfarin (Coumadin)
-Load at 10mg or 7.5mg PO qdaily for 2 days
-Decrease/adjust dose to a target INR=2.5
-DO NOT stop heparin infusion until INR reaches 2.5
-INR should be maintained at 2.5 for 3-12 months
-Consider placement of IVC filter (inferior vena cava)

-Pulmonary Embolism (PE)


-PE occurs when a clot from a peripheral location embolizes to the pulmonary vasculature
- <25% of deep vein thromboses distal to the iliac veins go on to develop PE.
-The more proximal the clot, the more likely it is to develop into a PE.
-“Classic Triad” of signs and symptoms of a PE: Dyspnea/SOB, Chest Pain, Hemoptysis
-Please note that less than 14% of patients experience the classic triad
-Diagnosis of a PE
-Gold standard: Pulmonary Angiography, spiral CT
-V/Q study
-CXR
-Treatment of PE
-Thrombolytic Therapy: -Urokinase: 4400units/kg IV over 10 min, then 4400units/kg/hr for 12 hours
-Streptokinase: 1.5 million units IV over 60 minutes
-Pulmonary Embolectomy
-Various filters

75
AJM Sheet: Pain Management
-Pain Management is a subject that you will be dealing with a lot during residency, but something that you
won’t receive much formal education on. Honestly, you probably won’t get many interview questions about it
either, but it’s something that I think is important. This is a shameful plug, but Clinics in Podiatric Med and Surg
had a whole edition to the subject (July 2008) that is worth reading. Specifically for the residency interview, read
Articles 1, 5 and 8. I also wrote the “Perioperative Pain Management” chapter in the 4th edition of McGlam’s.
-The “attack points” are a concept that AJM made up to promote an active approach to multimodal pain
management.

-Acute Operative Pain Physiology “Attack Points” (In Clinics: The Physiology of the Acute Pain Pathway)
Attack Point Physiology Intervention
Stimulus Stimulus: Noxious stimuli resulting in Resolution/Limitation of Stimulus
tissue damage to superficial and deep -Prevention of secondary aggravation
somatic structures.
Anti-Inflammatory Pharmacologics
Transduction: Nociceptor activation by -NSAIDs
chemical, mechanical or thermal means. Carboxylic Acid Derivatives
Proprionic Acid Derivatives
Post-Injury Inflammatory Response: Acetic Acid Derivatives
Normal response to cellular damage with Fenamates
the chance to develop into the Enolic Acid Derivatives
pathophysiologic mechanism of peripheral Naphthylkanones
sensitization. COXII Selectives
-Non-NSAIDs
-Anti-histamines
-Topical Agents
-Local Anesthetics

Anti-Inflammatory physiotherapeutics

Transmission Peripheral sensory afferents carrying the Local Anesthetics


action potential of the noxious stimulus
from the periphery to the CNS. Sodium Channel Blockers
-Normally controlled by myelinated A-
delta and unmyelinated C fibers.
-Allodynia is a pathophysiologic state of
sensitization when A-beta fibers transmit
light touch as painful.
Modulation Spinal Cord Dorsal Horn Opioids
-Peripheral Excitatory Signals
-Peripheral Inhibitory Signals Calcium Channel Blockers
-Central Excitatory Signals
-Central Inhibitory Signals NMDA Receptor Antagonists

Beta-adrenergics

Anti-Inflammatories

Perception Ascending Central Processes Anxiolytics


Descending Central Processes
Patient Emotional Response Anti-Depressants

Patient Education

-Multimodal Approach to Active Pain Management


A passive unimodal therapy like Percocet has very little total effect on the physiology of pain. Opioids and
acetaminophen influence small portions of the modulation attack point, but essentially do not influence any of the other attack
points. A multimodal approach actively intervenes at several attack points with several therapies to interrupt the known
physiologic and pathophysiologic mechanisms.
-Pre-emptive Analgesia (In Clinics: Perioperative Pain Management)
-The concept of pre-dosing pain medications before surgery to interrupt pain pathways before they start
76
AJM Sheet: AO

-AO: Arbeitsgemeinschaft fur Osteosynthese fragen


-History
-Plates and screws for fx fixation first described by Alain Lambotte in 1907.
-Robert Danis (Belgium surgeon) published “The Theory and Practice of Osteosynthesis” in 1949.
-Described use of compression plate called a coapteur.
-Maurice Mueller, a pupil of Danis, founded AO with other Swiss surgeons in 1959.

-Principles of AO
1. Accurate and precise anatomic reduction of fracture fragments (especially in joints).
2. Atraumatic surgical technique with emphasis on preservation of blood supply.
3. Rigid/Stable fixation
4. Early mobilization

-The “Guide to Internal Fixation” by the AO group is a great book that reads fairly quickly. You should also read the text
“Internal Fixation of Small Fractures” and “AO Principles of Fracture Management” from the AO group. General notes
from these books are included throughout the following sheets dealing with specific traumatic fractures.

-General Principles of the Lag Technique


-Why? Generates compression. So? Compression leads to lack of motion and therefore primary bone healing.
Motion disrupts angiogenesis, decreases oxygen tension levels and inhibits osteogenesis. So, it is the lack of motion and
NOT the compression that is osteogenic.

-Orientation of the screw 90° to the fracture line obtains optimal compression.
->20° displacement from perpendicular is significant
-Weakest in translation from axial loading
-Orientation of the screw 90° to the long axis optimally prevents displacement with axial loading.
-Weak in compression
-Ideal screw placement for a long, oblique fracture:
-One central anchor screw 90° to the long axis
-One proximal and one distal compression screw 90° to the fracture line

-Principles of Insertion
-AO Recommendations: Overdrill, Underdrill, Countersink, Measure, Tap, Screw
-Some underdrill before overdrill
-Some don’t overdrill until after tapping
-Two finger tightness = 440-770lbs.
-To prevent thermal necrosis: -Sharp tip
-Fast advancement (2-3mm/sec)
-Slow drill speed (300-400rpm)
-Firm force (20-25lbs)
-Screw Pull-out
-Directly proportional to screw diameter, screw length and bone strength (cortical nature).
-Indirectly related to pilot hole diameter.
-To increase screw pull-out, maximize bone-screw contact.

-Fairly Irrelevant Definitions


-Stress: pressure on a material
-Strain: measurable deformation following a given stress
-Stress-Strain Curve/Load Deformation Curve
-Elastic Range: -Non-permanent strain/deformation with a given stress
-Proportional stress and strain (Hook’s Law)
-Slope of the line is the stiffness (Young’s Modulus of Elasticity)
-Yield Point: -Past the yield point, a given stress causes a non-proportional increase in strain.
-Plastic Range: -Permanent deformation past the yield point
-Ultimate Failure Point
-Fatigue Failure: failure from repetitive cyclic loading
-Creep: temperature dependant permanent deformation of a metal
-Stress Shielding: Internal fixation absorbs physiologic stress from bone and results in bone resorption per Wolff’s
Law.

77
AJM Sheet: Screws and Plates
Screws
-Screw Anatomy/Definitions
-Head: more efficient hexagonal vs. cruciate
-Land: underside of the head which contacts the near cortex. Want as much land-bone contact as possible to reduce
stress at any one location. This is the same principle as washers and countersinking.
-Shank: unthreaded portion of the screw
-Run-out: junction between the shank and the threads. Represents the weakest portion of the screw.
-Thread diameter: diameter of threads + core (major diameter)
-Core diameter: diameter without the threads (minor diameter)
-Pitch: distance between threads
-Tip: can be round, trocar or fluted
-Axis: central line of the screw
-Rake Angle: thread to axis angle
-Thread Angle: angle between the threads

-Cortical and Cancellous Screws


-Please memorize Table 1, page 76 in McGlamry (Also AJM List: page 17)
-Cortical: tighter pitch designed for hard cortical bone
-Cancellous: higher pitch designed for metaphyseal and epiphyseal bone

-Self-Tapping Screws
-Fluted tip that clears debris as it is advanced
-Require larger pilot holes, have decreased thread-bone contact and have the ability to cut its own path different
from the underdrill

-Cannulated Screws
-Classically 3.0, 4.0, and 7.3mm, but really have just about any size available now
-Advantages: self-drilling, self-tapping, good for hard to visualize fractures, avoids skiving of cortical bone on
insertion and has definite co-axial nature with K-wire.
-Disadvantages: hollow core, decreased thread-core ratio, decreased pull-out strength

-Herbert Screws
-Proximal and distal threads separated by a smooth shaft. Headless.
-Leading threads have increased pitch, so it draws the trailing threads.
-Does generate interfragmental compression, but not a lot.

-Interference Screws
-FT, headless screw
-prevents axial displacement. Does not generate compression.

-Malleolar Screws
-Essentially a self-cutting, PT cortical screw.

Plates
-General:
-Quarter Tubular Plate: For use with screws from the mini fragment set
-One-Third Tubular Plate: For use with screws from the small fragment set
-Many other shapes and sizes of plates are available that specifically fit just about any bone/situation.

-General Plate Characteristics:


-Dynamic Compression (DCP): Wider/Deeper holes that allow for eccentric drilling and axial compression
-Limited Contact (LC): Essentially grooves on the underside of the plate that limit periosteal contact
-Locking: see -Miranda MA. Locking plate technology and its role in osteoporotic fractures. Injury. 2007 Sep; 38 Suppl 3:S35-9.
-Egol KA, et al. Biomechanics of locked plates and screws. J Orthop Trauma. 2004 Sep; 18(8): 488-93.

-General Plate Functions:


-Neutralization -Interfragmentary Compression
-Buttressing -Tension Band

-AO Basic Stabilization Rule: Ideally you want 3 or 4 cortical threads in each main fragment distally, and 5 or 6 proximally.

78
AJM Sheet: Suture Sheet

-Suture materials are best classified as to whether they are absorbable vs. non-absorbable, synthetic vs. natural, and
monofilament vs. multifilament.
1. Absorbable (usually used for deep closure)
A. Natural
-Pig collagen, sheep intestine, cow intestine or cat gut
-May be chromic (treated with chromic salts to increase strength and decrease hydrolysis)
-Digested by lysosomal enzymes in 20 days
B. Synthetic
1. Vicryl (Polyglactin 910)
-Braided. May be coated (polyglactin 370 or calcium stearate)
-65% tensile strength at 14 days
-Hydrolyzed (to CO2 and H20) in 80-120 days
-Vicryl Rapid: Hydrolyzed in 42 days; loses strength in 7-10 days
-Vicryl Plus: Coated with broad spectrum antibiotic Triclosan (also found in toothpaste)
2. Dexon (Polyglycolic acid)
-Braided. May be coated (polycaprolate 188)
-Hydrolyzed in 100-200 days
3. PDS (Polydiaxonone)
-Monofilament
-70% tensile strength at 14 days
-Hydrolyzed in 90 days
4. Maxon (Polyglyconate)
-Monofilament
-Hydrolyzed in 180 days; Longest lasting absorbable (“Max”-imum)
5. Monocril (Poliglecaprone)
-Monofilament
-20-30% tensile strength at 14 days
-Hydrolyzed in 90-120 days

2. Non-absorbable (usually used for superficial closure/skin sutures)


-It can be argued that all sutures are eventually absorbable!
A. Natural
1. Silk
-Made from silk worm
-Actually very slowly absorbed (hydrolyzed in 1 year)
-Very low tensile strength
2. Cotton/Linen
-Weakest suture
B. Synthetic
1. Nylon (Ethilon, Surgilon)
-Both monofilament and braided available
-Highest “knot slippage” rate: monofilaments are at a higher risk of knot slippage
2. Polypropylene (Prolene, Surgilene)
-Monofilament
-Can be used in contaminated/infected wounds (Nonabsorbable, synthetic, monofilaments best
in this situation). This is the least reactive suture.
3. Polyester (Ethibond, Dacron)
-Strong suture. May be used for tendon repair.
-Braided. May be coated with silicone.
4. Fiberwire (polyethylene multifilament core with a braided polyester jacket)
5. Stainless Steel
-Monofilament or braided (braided is called Flexon)
-Strongest suture with longest absorption rate
-Used for bone fixation and tendon repair, but may corrode bone at stress points

Other Notes:
-Sutures are also classified according to size. They can range from 0-0 (very thick) to 9-0 (extremely thin).
-Surgeon’s choice is extremely variable and you usually just work with what you are used to, but here are some safe bets:
-Capsule closure: 2-0 or 3-0 Vicryl
-Subcutaneous tissue closure: 3-0 or 4-0 Vicryl
-Skin: 4-0 Nylon or Prolene
-Skin sutures are removed at 10-14 days because at this point the tensile strength of the wound equals the tensile strength of the
suture.
-This was the very first AJM sheet!

79
AJM Sheet: Podiatric Surgery Instrumentation
This sheet is simply a summary of the first chapter of McGlamry’s text by Dr. Malay.

-General Information
-Surgical Instruments are composed of stainless steel which itself is composed of several different metals:
-Carbon: gives instrument “hardness”
-Chromium: chromium oxide layer prevents corrosion of instrument
-Tungsten Carbide: extreme “hardness” for grasping surfaces (teeth of needle drivers)
-Nickel
-Molybdenum
-There are two different series of stainless steel depending on how it is manufactured:
-300 series Austenitic: Implants and internal fixation. Resists corrosion with resilience.
-400 series Martenitic: Cutting instruments. Hardness maintains sharp edges and jaw alignments.

-Categories of Instruments
***Know how to appropriately handle each instrument if handed it during an interview.
***Be able to identify each instrument if handed it or shown a picture during an interview.
1. Surgical Blades
-Most common: 10, 15, 11, 62 on a minihandle
-Purpose: Sharp (blade) and blunt (handle) dissection
-Cutting edge width: 0.015”
2. Scissors
-Most common: Tissue: Metzenbaum, Mayo, Iris, Crown&Collar (Sistrunk)
Non Tissue: Suture, Utility, Bandage
-Purpose: Dissection
3. Hemostats
-Most common: Mosquito (Halsted), Kelly, Crile
-Purpose: Grasping and holding
4. Pick-ups
-Most common: 1-2 (Rat tooth), Adson-Brown, Atraumatic (Potts-Smith)
-Purpose: Grasping and Holding
5. Retractors
-Most Common: Hand Held: Skin Hooks, Senn, Ragnell, Malleable, Army-Navy, Volkmann Rake,
Meyerding
Self-retaining: Weitlaner, Holzheimer, Heiss
-Purpose: Retraction and exposure
6. Elevators
-Most Common: Freer, Sayre, Key, Crego, McGlamry, Langenbeck
-Purpose: Dissection
7. Rasps
-Most Common: Joseph, Maltz, Bell, Parkes
-Purpose: Cutting
8. Miscellaneous
-Osteotomes
-Chisels
-Gauges
-Mallets
-Bone-Cutting Forceps
-Rongeurs
-Trephine
-Curettes
-Bone Handling Clamps
-Reduction Forceps (Lewin, Lane, Lowman, Verbrugge)
-Needle Holders (Mayo-Heger, Sarot, Ryder, Halsey, Webster)
-Suction-Tip (Frazier)

80
AJM Sheet: Power Instrumentation General Information
This sheet is simply a review of McGlamry’s Chapter 2 by Dr. Alfred Phillips.
-General Information
-Hardest material in the human body? Teeth Enamel
-Power instrumentation developed by which medical field? Dentistry

-Power Sources (3)


1. Pneumatic
-Advantages: Delivers high power and torque, does not overheat, cheap
-Disadvantages: Does not operate at slow speeds, bulky, burdensome, cords prone to contamination
-General: -Most commonly driven by compressed nitrogen
-Tank pressure > 500 psi
-Dynamic instrument pressure: 90-110 psi
2. Electric
-Advantages: Light, quiet, small, good for office use
-Disadvantages: Prone to overheating, expensive
-General: -Utilizes an alternating current drive
3. Battery
-Advantages: No cords
-Disadvantages: Loses power quickly, bulky handling
-General: -Utilizes direct current

-Brands: Stryker, Hall-Zimmer, Microaire

-Definitions
-Torque: Measurement of power and force. Units: Newtons/cm^2
-Cortical bone requires more torque to cut through than cancellous bone.
-Speed: Distance per time
-Pod procedures usually require 20,000 rpm.
-Decrease risk of thermal necrosis by decreasing torque and increasing speed.
-Collet: Distal end of a saw where the saw blade attaches
-Stroke: One arc of excursion for a saw blade
-Oscillation: One back and forth motion of a saw blade. (Two strokes equal one oscillation).

-Power Saws
-Types
-Sagittal Saw: Cuts in the same plane as the instrument
-Better for longer and deeper cuts
-4° arc of excursion
-Blade may be positioned anywhere within a 160-180° arc.
-Oscillating Saw: Cuts in plane perpendicular to instrument
-7° arc of excursion
-Blade may be positioned anywhere within a 360° circle.
-Blades
-Vary by cutting depth, width, thickness, shape and number of teeth
-Shapes: straight (most commonly used), inward flair, outward flair
-The angulation of the teeth and NOT the thickness of the blade determine the thickness of a cut.
-Blades may contain holes which collect debris, thereby decreasing heat and friction.

-Wire Drivers
-K-Wires (Kirshner wire)
-Sizes: 0.028”, 0.035”, 0.045”, 0.062”
-Threaded vs. Non-threaded. Note that the direction of the driver only matters with threaded wires.
-K-Wires provide splintage (stability, but no compression)
-Steinman Pins
-Sizes: 5/64”-3/16”
-Rotary Cutting
-Power Drill Bit Sizes: 1.1, 1.5, 2.0mm
-Burrs
-Shapes: Round, Barrel, Straight, Straight-tapered
-Definitions: -shank vs. shaft vs. head
-flute vs. blade
-edge angle vs. clearance angle vs. rake angle
-Surgical Skills Section
-Surgical skills are something best learned by practice, practice, practice. A few hints are listed below:
-The surgeon’s hands provide 3 functions when operating power instrumentation:
-Control of power of the instrument
-Control of direction of the instrument
-Stability between the instrument and the surgical site
-Review concepts of: -Axis guide
-Reciprocal planing
-With a saw or K-wire, always divot perpendicular to the cortex, and then redirect.
-The spin of a burr should be parallel to the grain of the cortex or parallel the ridge of bone to be removed.
-Poor man’s ways to practice handling and control of surgical instruments:
-K-wire through a Nerf ball
-Sagittal saw through a wine cork or wood blocks

81
AJM Sheet: Biomaterials
Summary of McGlamry’s Chapter 3 by Dr. Cicchinelli.

-Properties of the ideal implant material: clinically inert, no inflammatory or foreign body response, noncarcinogenic,
nonallergenic, structurally stable, capable of sterilization, capable of fabrication in desired forms, serve as a scaffold for new
bone growth and gradual biodegradation.

-Host Response to Implant


-Cellular Response (Acute)
-Immediately after implantation, implants are covered with a coat of proteins that denature and elicit an
inflammatory response. Denatured fibrinogen accumulates neutrophils and macrophages.
-Detritic Synovitis: Foreign body reaction to shards of silicone materials in the lymphatic system.
-Environmental Stress Cracking: Surface defects on polyetherurethane implants secondary to chronic
inflammation. Chronic inflammation results from fragmentation and leads to intracortical lysis and cyst formation.
-Tissue Remodeling Response: Normal for implants to have fibrous capsule formation.
-Infection Potential
-Susceptible to S. Aureus and S. Epidermidis infections
-Malignancy and Type III hypersensitivity reactions extremely rare

-Biomaterials
-PLLA (Polylactic-L-Acid: L is enantiomer)
-Degrades to lactic acid via hydrolysis
-Retains strength 36 weeks and degrades in 2-3 years
-Available in FT 2.0, 2.7, 3.5 and 4.5mm screws
-PGA (Polyglycolic Acid)
-Degrades to glycolic acid and glycine
-Elliptical. Provides compression secondary to shape.
-Brittle and rigid
-Highest likelihood of FB rxn or complication (<4%)
-PDS (Poly-para-dioxanone)
-Tapered form swaged on metallic wire. Provides compression secondary to shape.
-Flexible and malleable

-Increased degradation times are good because it decreases the load the body has to clear.
-These screws don’t “bite” like metal screws, but swell 2-4% in the first 48 hours.
-Advantages: decreased stress shielding, no second operation for removal.
-Disadvantages: more expensive than metallic screws, but are cheaper in the long run if you remove >31% of
metallic screws in your practice.

-Metallic Implants
-Surgical Stainless Steel
-316LVM (low carbon vacuum remelting)
-Iron, 17-25% chrome, 10-14% nickel, 2-4% molybdenium, 1% carbon
-Nickel most commonly causes reaction: allergic eczematous dermatitis.
-Titanium
-Very inert, integrates into surrounding bone, resists corrosion, decreased capsule formation
-Addition of 6% aluminum and 4% vanadium increases the strength similar to steel
-Nitrogen implantation forms a stable oxide layer
-Black metallic wear debris is often seen. No toxicity or malignancy associated with this.
-Cobalt Chrome and Alloys
-30% cobalt, 7% chromium, <0.034% moly/carbon
-Used in joint replacement prostheses

-Corrosion: breakdown of metallic alloys because of electrochemical interactions within the environment

82
AJM Sheet: General External Fixation

-Selected History
-377BC: Hippocrates with wood from a cornel tree
-1904: Codvilla (Italy) used unilateral fixator for limb lengthening
-1951-1991: Ilizarov (Siberia, Russia). Father of modern ex-fix and developer of external ring fixator for WWII vets from old bus parts.

-General Principles
-Tension-Stress Effect (Ilizarov)
-Distraction performed at proper rate and in the proper area leads to tissue growth similar to hormone-mediated growth at adolescent
growth plates.
-Too fast: Stretching and traction injuries
-Too slow: Bone callus consolidation preventing future distraction
-An important principle is that all tissues (bone, skin, muscle, NV structures, etc.) become mitogenically active and grow. They proliferate as
opposed to “stretching”. Much of this has to do with the distraction serving as a mechanical stimulus for growth factor release (such as osteoblastic growth
factor) and dramatic increases in vascularity.
-Tension-Stress Effect Influences:
-Stability: increased stability leads to increased osteoblastic activity
-Rate: Ideal is 1mm/day in 4 increments
-Bone Cut: Best to keep medullary canal and as much periosteum intact as possible. Best technique is a percutaneous subperiosteal
corticotomy with a Gigli saw or osteotome/mallet.
-Location of Bone Cut: Metaphysis found to be superior to other areas

-Behrens Principles of External Fixation


-Avoid and respect neurovascular structures
-Allow access to injured area for future fixation
-Meet mechanical demands of the patient and the injury
-Tajana’s Stages of Callus Development
-Colloidal (0-2 weeks): formation of microreticular network
-Fibrillar (2 weeks-1 month): collagen organization
-Lamellar (1 month-years): formation of compact lamellar tissue and calcification

-Advantages of Ex-Fix -Complications of Ex-Fix


-Decreased soft tissue dissection -Pin tract infection vs. irritation
-Immobilization of multiple regions -Pain
-Allows for post-operative adjustment -Cage rage
-Skin grafting and wound debridement available -Non-unions
-Early ROM and WB -Fracture
-NV injury
-Anatomy
-Knowledge of cross-sectional anatomy is essential for the application of external fixation. There are numerous manuals and tests available
demonstrating proper pin and wire placement in a given location.
-The key is to have solid bone with avoidance of neurovascular structures.
-As a general rule, the medial and anterior aspects of the tibia are safe locations.

-Types of External Fixators


-Unilateral Fixators
-EBI Dynafix and Orthofix
-Can be straight (uniplanar) or articulated (multiplanar)
-Allow for compression/distraction in a single plane only
-Attached to bone via half-pins
-Rigidity and stiffness determined by half-pins/bone interface. Want pins spread over a large area.
-Weak in the sagittal plane
-Circular Fixators (Multi-lateral)
-Smith&Nephew
-Generate compression/distraction in multiple planes
-Tensioned wires generate stability; half-pins generate rigidity.
-Best if these are located 90° to each other for optimal stability
-Can be formulated to allow for immediate WB
-Hybrid Fixators
-Orthofix, Dynafix, Smith&Nephew, Rancho
-Combination of unilateral and circular fixators
-Taylor Spatial Frames
-Smith&Nephew
-Allows for reduction of triplanar complex deformities

-Brief Indications
-Limb Lengthening/Distraction
-Percutaneous metaphyseal subperiosteal corticotomy with Gigli saw or osteotome/mallet
-Apply fixation before corticotomy
-Distraction begins 7-14 days after corticotomy at 1mm/day
-Angular Deformities
-CORA principle (center of rotational angulation)
-Double Taylor spatial frame
-Dynamization: release of tension from wires and loosening of half-pins to allow bone a period of introductory WB
-Fracture
-Ligamentotaxis: pulling of fracture fragments into alignment with distraction
-Arthrodesis
83
AJM Sheet: Bone, Bone Healing and Wound Healing
-Bone Properties/Variables
-Bone is a two component system consisting of minerals (increases the yield and ultimate strength of bone) and collagen (mostly
Type II).
-Variables:
-Porosity. Increased porosity leads to increased compressive strength of bone. Cortical bone has <15% porosity and
cancellous bone has ~70% porosity.
-Strength. Strength is defined as the amount of force a material can handle before failure. Bone can handle a 2%
increase in length before failure. Bone is has the greatest strength in compression, followed by tension and is weakest in shear. Strength is
affected by collagen fiber orientation, trabecular orientation, age, presence of defects and osteoporosis.
-Stiffness. Cortical bone has 5-10 times the stiffness of cancellous bone.

-Vascular Supply to Bone


-Blood supply to bone comes from two sources. A nutrient artery feeds the endosteal and medullary vessels and supplies the
inner 2/3-3/4 of bone. The periosteal vessels supply the outer 1/3 of bone from muscle and tendon attachments.
-The amount of vascular disruption following a fracture depends on the force/displacement of the fracture and which vascular
systems are disrupted.

-Phases of Bone Healing


-Inflammation (10%)
-Hematoma fills the area with fibrin, RBCs, neutrophils, platelets, macrophages, fibroblasts (from PMNs).
-Mesenchymal cells from the cambium layer differentiate into osteoblasts and chondrocytes.
-Chemotaxis by growth factors (transforming growth factor beta, platelet derived, and macrophage derived)
-Reparative/Regenerative (40%)
-Soft callus forms and is replaced by bone.
-Cartilage, fibrocartilage, collagen and hydroxyapatite deposition
-Cartilage replaced by bone like endochondral ossification
-Remodeling (70%)
-Callus completely replaced by bone
-Vascular network is normalized
-Remodels according to Wolff’s Law
-Piezoelectric Effect: appearance of electrical potentials within bone in response to the application of an external force
-Compression side: electronegative leading to bone production
-Tension side: electropositive leading to bone resorption
-Types of Bone Healing
-Direct Osseous Repair (Primary Intention, Direct Healing)
-No callus formation; no motion
-Cutting cone: Osteoclasts in the front, osteoblasts in the back. Travels across the fx line (Schenk and Willinegger).
-Gap Healing: Bone deposition at 90° to the orientation of bone fragments
-Indirect Osseous Repair
-Callus formation

-The literature has demonstrated that cyclic loading and dynamization have resulted in decreased healing times, decreased
stiffness, increased torque and increased energy absorption in rabbit and dog bones. A practical means to accomplish this in human
subjects hasn’t been perfected yet.

Wound Healing
Additional Readings:
-[Broughton G, Janis JE, Attinger CE. Wound healing: an overview. Plast Reconstr Surg. 2006 Jun; 117(7 Suppl): 1S-32S.]
-[Broughton G, Janis JE, Attinger CE. The basic science of wound healing. Plast Reconstr Surg. 2006 Jun; 117(7 Suppl): 12S-34S.]
-[Hunt TK, Hopf H, Hussain Z. Physiology of wound healing. Adv Skin Wound Care. 2000 May-Jun; 13(2 Suppl): 6-11.]
-[Lawrence WT. Physiology of the acute wound. Clin Plast Surg. 1998 Jul: 25(3): 321-40.]
-[Falanga V. Wound healing and its impairment in the diabetic foot. Lancet 2005; 366: 1736-43.]

Phases of Wound Healing:


1. Substrate/Lag/Inflammatory Stage (Days 1-4)
-Inflammation characterized by edema/erythema/calor/dolor
-PMNs start out dominating, but are eventually taken over by macrophages
2. Proliferative/Repair Phase (Days 3-21)
-Collagen proliferation and macrophages
-Myofibroblasts also begin working
3. Remodeling/Maturation Phase (Day 21+)

84
AJM Sheet: How to “Work-Up” a Surgical Patient

With regard to specific surgery and the interviews, it’s always important to “know your program”. In other words, programs tend
to have favorite procedures that they routinely do. For a given bunion deformity, one program may primarily do Austin-Akins, whereas
other programs may never do an Akin, and still others may always do a Lapidus in the exact same situation. Some people may feel very
strongly in favor of the lateral release, while others may never do it for any situation. This could even happen between two attendings at
the same program in the same room during your interview! If you give a hard, definitive answer for a procedure choice, one attending may
completely agree with you while another may think it’s completely the wrong choice. So if you are asked what type of procedure you
would do for a given situation, be as general as possible, but always give the reason/specific indications why you are choosing that
procedure or group of procedures. Name a couple different similar procedures instead of sticking by your guns with one procedure.
Additionally, your interviewers may not expect you to know for sure what procedure to choose, but they will definitely expect you to be
able to completely work-up the patient and know which procedures are acceptable for which indications.

The two work-ups that you should have down cold are the HAV and flatfoot work-ups. Practice, practice, practice working
through these situations out loud, and practice, practice, practice going through the radiographic analyses of these deformities out loud.
Again, RC and I found it helpful while studying for interviews to pick up random podiatry textbooks and just flip through the pages,
alternating our description of the radiographs out loud.

There are of course many, many radiographic angles that you can use to describe during either of these work-ups, so focus the
majority of your energy on those that will have the most impact on your treatment choice. Here’s the way that I think about these
deformities. This certainly isn’t the “right” way; it’s just the way that helped me as I first started doing this out loud:

HAV:
I simply use the radiographic angles to define two aspects of the deformity:
-Where is the deformity?
-In which bone or bones, and/or which joint or joints is there deformity?
-Is the deformity mild, moderate or severe?

Once you have successfully answered these questions in your mind, then the remainder of the radiographic work-up
falls into place. For example, if you identify a deformity at the first metatarsal-phalangeal joint, then you can use your
radiographic angles to define it:
“In the area of the patient’s presenting complaint I see a (mild, moderate, or severe) hallux abductovalgus deformity at
the level of the metatarsal-phalangeal joint as defined by a (mildly, moderately, or severely) increased intermetatarsal angle,
(mildly, moderately, or severely) increased hallux abductus angle, and approximate tibial sesamoid position of (1-7). The PASA
and DASA of this joint appear (within normal limits or deviated). There (does or does not) appear to be a hallux interphalangeus
deformity as defined by the (increased or normal) hallux interphalangeus angle. The overall length of the first metatarsal appears
(normal, shortened, or long) compared to the remainder of the lesser metatarsal parabola on the AP view. On the lateral view the
first metatarsal appears (dorsiflexed, plantarflexed, or normal) compared to the second metatarsal using Seiberg’s index. There
(is or is not) an underlying metatarsus adductus as defined by the metatarsus adductus and Engle’s angles. Generally, the
rearfoot appears (rectus, pronated, or supinated) as defined by…”
Now that you have defined the location and severity of the deformity with your angles, suggest procedures based on
these specific abnormal findings. For every abnormality that you described, suggest a procedure (or group of procedures) to
correct it. “I would consider doing a distal metatarsal osteotomy in this case to laterally translate and plantarflex the capital
fragment of the first metatarsal to decrease the intermetatarsal and hallux abductus angles in addition to reducing the sesamoids.”
If you described the DASA and interphalangeus angles as normal, then don’t suggest an Akin procedure! If you described a mild
deformity, then don’t suggest procedures that are indicated for moderate to severe deformities!
I also use the above questions to classify each and every surgical procedure. For each surgical procedure I think: This
procedure will correct for a (mild, moderate, or severe) deformity of this bone or at that joint.

Flatfoot:
Here I use a similar approach, but think of it in terms of planal dominance:
-In which plane does the deformity present?

“Consistent with the patient’s presenting complaint we see a (mild, moderate, or severe) pes planovalgus deformity. In
the sagittal plane I see a (decreased or increased) calcaneal inclination angle, talar declination angle, talar-calcaneal angle, first
metatarsal inclination angle, Meary’s angle, and medial column fault on the lateral view. I would also evaluate the patient for
equinus using the Silfverskiold test to determine a sagittal plane deformity. In the transverse plane I see a (decreased or
increased) talar-calcaneal angle, cuboid abduction angle, talar head coverage, talar-first metatarsal angle, metatarsus adductus
angle on the AP view. In the frontal plane we can see the Cyma Line is (anteriorly displaced, posteriorly displaced or normal) on
the lateral view, and that the subtalar joint alignment, ankle joint alignment and calcaneal position are (normal or abnormal) on
the long leg calcaneal axial views.”
Now that you have defined the deformity on your own terms, you can now suggest how to fix it using the same tools.
“I would consider performing a (Gastroc recession, TAL, Cotton osteotomy, medial column arthrodesis, etc.) to correct for the
sagittal plane deformity, a (Evans osteotomy, CC joint distraction arthrodesis, etc.) to correct for the transverse plane deformity,
and a (medial calcaneal slide, STJ implant, etc.) to correct for the frontal plane deformity.

This is a little philosophic, but radiographic angles aren’t real. They only come into reality if you use them, so only
use them as tools to your advantage. You can use them to first define the deformity on your own terms, and then to show
that your intervention was successful.
85
AJM Sheet: Digital Deformity Work-Up

Subjective
-CC: Pt can complain of generalized “corns, calluses and hammertoes.”
-HPI: -Nature: “Sharp, aching and/or sore” type pain. May have a “tired feeling” in the feet.
-Location: Usually dorsal PIPJ/DIPJ of the toes or submetatarsal
-Course: Progressive onset and course.
-Aggravating factors: WB, shoe gear (especially tight shoes)
-Alleviating factors: NWB, wide shoebox, sandals
-PMH/PSH/Meds/Allergies/SH/FH/ROS: Usually non-contributory

Objective
Physical Exam
-Derm: -Hyperkeratotic lesions can be seen submetatarsal, dorsal PIPJ or DIPJ of the lesser digits, distal tuft of the lesser digits, or
interdigitally. All can have erythema, calor and associated bursitis.
-5th digit is usually dorsolateral at the PIPJ, DIPJ or lateral nail fold (Lister’s corn). Hyperkeratotic lesion of the adjuvant 4th
interspace may also be present (heloma molle).
-Vasc/Neuro: Usually non-contributory
-Ortho: -See discussion on pathomechanics
-Positive Coughlin test: Vertical shift of >50% of the proximal phalanx base on the met head. Also called the “draw sign” or
Lachman’s test.
-Kelikian push-up test: Differentiate between a soft-tissue and osseous deformity
-Specific to the 5th digit:
-Toe usually has a unique triplanar deformity (dorsiflexion, adduction and varus).
-Bunionette, splay foot and equinus may be present
-The 5th digit is in the most susceptible position in terms of a muscular imbalance deformity because the FDL has such
an oblique pull on the 5th digit as opposed to the relatively axial pull of the other digits.

Imaging
-Plain film radiograph: “Gun barrel” sign

Specific Deformities
-Hammertoe: Extension at MPJ level; flexion at PIPJ level, neutral/extended DIPJ
-Mallet toe: Neutral at MPJ and PIPJ level; flexion at DIPJ level
-Claw toe: Extension at MPJ level; flexion at PIPJ and DIPJ level
-Curly toe: Claw/hammertoe deformity with an additional frontal plane component
-Digitus Adductus: Digital deformity with adduction in the transverse plane
-Digitus Abductus: Digital deformity with abduction in the transverse plane
-Heloma Molle: Generally occurs in the 4th interspace with a curly toe deformity of the 5th digit. Using this example, the head of the
proximal phalanx of the 5th digit abuts the base of the proximal phalanx of the 4th digit causing a hyperkeratotic lesion in the proximal 4th
interspace.

Pathomechanics
-Digital deformities are thought to occur via one of three potential mechanisms. Each involves a muscular imbalance at the digital level.
-The way AJM thinks of digits is from distal to proximal. During weight-bearing, the toes cannot function in propulsive gait to aid in load
transfer if the most distal segment is not stabilized. The distal phalanx is stabilized by the long flexor tendons holding it solidly against the
weight-bearing surface. With the distal phalanx stabilized, the short flexor tendon can hold the middle phalanx against the weight-bearing
surface. With the middle phalanx stabilized, the lumbrical muscles hold the proximal phalanx against the ground. The lumbrical muscles
must work against the extensor tendon complex, but this complex is usually not actively firing to extend the MPJ during propulsion. The
interosseous muscles also stabilize the proximal phalanx in the transverse plane. When the proximal phalanx has been effectively
stabilized against the weight-bearing surface, the head of the metatarsal can effectively move through its range of motion and transfer load
across the metatarsal parabola. Any disruption in the stabilization process will lead to abnormal biomechanics and deformity.
-Flexor Stabilization: -Most common origin of hammertoe deformity
-Occurs when the PT muscle is unable to effectively resupinate the midtarsal and subtalar joints at
the beginning of propulsion. To compensate, the FHL and FDL fire earlier, longer and with
greater force to resupinate the foot. This puts too much force on the distal and middle phalanges
causing the toe to “buckle” in a dorsiflexed position at the MPJ. This retrograde buckling puts the
PIPJ in a vulnerable dorsal position and also pushes the metatarsal head plantarly.
-Flexor Substitution: -Occurs when the triceps surae muscle group is unable to effectively plantarflex the foot during
propulsion for whatever reason. To compensate, the muscles of the deep posterior compartment
(PT, FHL, and FDL) again fire earlier, longer and with greater force leading to the same type of
deformity.
-Extensor Substitution: -Can occur in two ways
-One way is when the TA is unable to dorsiflex the foot through the swing phase. In this case the
EDL and EHL fire earlier, longer and with greater force than normal and are actually actively
extending the MPJ. This easily overpowers the lumbricals and leads to retrograde buckling.
-The other way is in a situation with anterior cavus where the EDL is actually at a mechanical
advantage over the lumbricals. Passive stretch of the EDL, rather than active contraction,
overpowers the lumbricals and leads to deformity.

86
AJM Sheet: Digital Deformity Treatment

Conservative
-Do nothing: Digital deformities are not a life-threatening condition and can be ignored if the patient is willing to put up
with it.
-Palliative care: Periodic sharp debridement of hyperkeratotic lesions
-Splints/Supports: -Metatarsal sling pads
-Silicone devices
-Toe crests
-Orthotics: -Cut-outs of high pressure areas
-Metatarsal pads to elevate the metatarsal heads
-Correction of the underlying deformity

Surgical Options
-Two approaches to remembering digital surgical options are the acronym HEECAT, and an anatomic approach thinking of
procedures moving from superficial to deep.
-HEECAT
-Head arthroplasty: Post procedure (1882)
-Extensor hood and PIPJ capsule release
-Extensor tendon lengthening
-Capsulotomy (MPJ)
-Arthrodesis (PIPJ)
-Tendon transfer (flexor longus tendon transfer to function in MPJ plantarflexion)

-Anatomic Approach
-Percutaneous tenotomy
-Both the extensor and flexor tendons can be transected through a percutaneous approach
-Extensor Tendon lengthening
-Done proximal to MPJ level with a Z-lengthening
-Capsulotomy
-Of the PIPJ and MPJ
-Remember the “J” maneuver for release of the collateral ligaments
-Extensor hood release is also usually performed
-Some use the McGlamry elevator in this step to free plantar attachments
-PIPJ Arthroplasty
-Post procedure 1882
-Resection of the head of the proximal phalanx at the surgical neck
-PIPJ Arthrodesis
-Fusion of the PIPJ using a variety of techniques: table-top, V, peg-in-hole, etc.
-Fusion maintained with K-wire crossing the MPJ extending into the distal 1/3 of the metatarsal
-Flexor Tendon Transfer
-Transfer of the FHL tendon dorsally to act as a more effective plantarflexor of the proximal
phalanx
-Girdlestone-Taylor technique: Tendon is bisected, crossed and sutured on the dorsal aspect.
-Kuwada/Dockery technique: Tendon is re-routed through a distal drill hole
-Schuberth technique: Tendon is transferred through a proximal drill hole
-Syndactyly
-Soft tissue fusion of one digit to a normal adjacent digit to help “bring it down”
-Interposing skin is removed and the digits are sutured together

-You should be able to go through the steps of a Post procedure for an interview.
-Please also review the neurovascular elements for each digit and be able to recite which cutaneous nerves supply which
corner of each digit.

5th digit skin incisions


-It is possible to alter your skin incision to incorporate a derotational element to your skin closure. While the
osseous work can be accomplished using a longitudinal or lazy “s” incision (proximal medial to distal lateral), those are
really best for uniplanar deformities. 5th digit HT is usually a triplanar deformity.
-Two semi-elliptical incisions directed proximal lateral to distal medial.
-The more oblique the incision is, the greater transverse plane correction.
-The more longitudinal the incision is, the greater the frontal plane correction.

87
AJM Sheet: Lesser Metatarsal Deformity Work-up

Subjective
-CC: Pt presents complaining of “pain in the ball of my foot.”
-HPI: -Nature: Generalized pain (aching, sharp, sore, etc.)
-Location: Submetatarsal. Can usually be localized to an exact metatarsal.
-Course: Gradual and progressive onset. “Has bothered me for years.”
-Aggravating factors: WB for long periods, shoe gear, etc.
-PMH/PSH/Meds/All/SH/FH/ROS: Usually non-contributory

Objective: Physical Exam


Derm: -Diffuse or focal hyperkeratotic lesions submetatarsal
Vasc/Neuro: Usually non-contributory
Ortho: -Many of the same signs/symptoms as HT digital deformity. HT often present.
-Anterior displacement of the fat pad -Hypermobility of the first ray
-Anterior Cavus foot type -Hypermobility of the fifth ray
-Equinus

Objective: Imaging
-Plain film radiograph: -Look for irregularities of the metatarsal parabola
-Look for excessively plantarflexed or dorsiflexed position on lateral/sesamoid axial views

General Information
-Lesser metatarsalgia has several possible etiologies:
-Retrograde force from hammertoes. Please see AJM Sheet: Digital Deformities.
-An excessively long and/or plantarflexed metatarsal leads to increased load bearing under that particular metatarsal.
-An excessively short and/or dorsiflexed metatarsal can lead to increased load bearing on the adjacent metatarsals.
-Hypermobility of the first ray leads to increased load bearing under at least the second metatarsal.
-Hypermobility of the fifth ray leads to increased load bearing under at least the fourth metatarsal.
-Anterior cavus and equinus deformities lead to increased pressures across the forefoot.
-Before a surgical option is considered, it is extremely important to understand where the increased load is coming from. The goal of
treatment should be to restore a normal parabola and weight-bearing function to the foot. Failure to correct the underlying deformity will
dramatically increase the rate of recurrence and transfer lesions.

Treatment: Conservative
-Do nothing: Lesser metatarsal deformities are not a life-threatening condition.
-Palliative care: Periodic sharp debridement of hyperkeratotic lesions
-Splints/Supports: -Metatarsal sling pads -Toe crests
-Silicone devices
-Orthotics: -Cut-outs of high pressure areas
-Metatarsal pads to elevate the metatarsal heads
-Correction of the underlying deformity

Treatment: Surgical
-Structural correction of lesser metatarsals
-Distal metatarsal procedures
-Duvries: plantar condylectomy on both sides of the MPJ
-Jacoby: “V” shaped cut in the metatarsal neck to allow for dorsiflexion of the head
-Chevron: “V” shaped cut similar to a Jacoby, but with removal of a wedge of bone to obtain metatarsal shortening as
well.
-Dorsiflexory wedge osteotomy: similar to a Watermann of the first metatarsal
-Weil: Distal dorsal to proximal plantar oblique cut to allow for distal metatarsal dorsiflexion and shortening. Can be
made in several planes to obtain desired dorsiflexory/shortening effects.
-Osteoclasis: Through and through cut through the metatarsal neck allowing the distal head to find its own plane.

-Metatarsal shaft procedures


-Cylindrical shortening
-Giannestras step-down procedure: Z-shaped cut which can allow for shortening and distal dorsiflexion.

-Metatarsal base procedures


-Dorsiflexory wedge: (1mm of proximal dorsal shortening equivalent to ~10 degrees of dorsiflexion)
-Buckholtz: Oblique dorsiflexory wedge which allows for insertion of a 2.7mm cortical screw
Complications
-By far, the most common complications are floating toe, recurrence and transfer lesions caused by undercorrection and overcorrection.
While you can evaluate the parabola and transverse plane in the OR with a C-arm, you really can’t appreciate the sagittal plane.
-Studies have demonstrated that osteoclastic procedures allowing the distal segment to find their own plane without internal fixation have
the least occurrence of recurrence and transfer lesions, but they also have a higher rate of malunion, delayed union and non-union.
-[Derner and Meyr. Complications and Salvage of Elective Central Metatarsal Osteotomies. Clinics Pod Med Surg. Jan 2009.]
88
AJM Sheet: 5th Metatarsal Deformity Work-up
Also called: Tailor’s Bunion or Bunionette Deformity

Subjective/Objective
-Very similar to work-ups for lesser metatarsal deformities and digital deformities. Pts may complain of pain related to the
lateral column in general, 5th digit, plantar 5th met head, lateral 5th met head or 4th interspace heloma molle.

Imaging
-Plain Film Radiograph: -4-5 Intermetatarsal Angle > 9 degrees (Normal is 6.47 degrees per Fallat and Buckholtz)
-Lateral Deviation Angle > 8 degrees (Normal is 2.64 degrees per Fallat and Buckholtz)
-[Fallat LM, Buckholtz J. J Am Podiatry Assoc. 1980 Dec; 70(12): 597-603.]
-Splay Foot Deformity
-Plantarflexed 5th metatarsal position
-Structural changes to 5th metatarsal head

General Information
-Etiology
-Numerous authors have chimed in on the etiology of the 5th Metatarsal Deformity:
-Davies: incomplete development of deep transverse metatarsal ligament
-Gray: malinsertion of adductor hallucis muscle
-Lelievre: forefoot splay
-Yancey: congenital bowing of metatarsal shaft
-Root: abnormal STJ pronation
-CMINT, etc, etc, etc.

Treatment
Conservative
-Do nothing: 5th metatarsal deformities are not a life-threatening condition.
-Palliative care: Periodic sharp debridement of hyperkeratotic lesions
-Splints/Supports: -Shoe gear modification with large toe box
-Derotational tapings
-Orthotics: -Cut-outs of high pressure areas
-Metatarsal pads to elevate the metatarsal heads
-Correction of the underlying deformity

Surgical
-Exostectomy: Removal of prominent lateral eminence from 5th met head
-Arthroplasty: Removal of part/whole of 5th met head
-Distal Metatarsal Osteotomies:
-Reverse Hohmann
-Reverse Wilson
-Reverse Austin
-Crawford: “L” shaped osteotomy allows for insertion of cortical screws
-LODO (Long Oblique Distal Osteotomy): similar to Crawford but simply oblique
-Read [London BP, Stern SF, et al. Long oblique distal osteotomy of the fifth metatarsal for correction of tailor's bunion: a
retrospective review. J Foot Ankle Surg. 2003 Jan-Feb;42(1):36-42.] Especially if externing at Inova!
-Medially-based wedge
-Proximal Osteotomies:
-Transverse cuts
-Oblique cuts
-Medially based wedges

89
AJM Sheet: HAV Work-up

Subjective
CC: “Bump pain,” “Big toe is moving over,” Typical patient is female although it is unclear whether there is a higher
incidence among females, or if there is a higher complaint incidence among females.
HPI: -Nature: Throbbing, aching-type pain
-Location: Dorsomedial 1st MPJ is most typical presentation. Pain could also be more medial (suggesting
underlying transverse plane deformity such as met adductus) or dorsal (suggesting OA of 1st MPJ).
-Course: Gradual and progressive
-Aggravating Factors: Shoe wear, WB
PMH: -Inflammatory conditions (SLE, RA, Gout, etc.)
-Ligamentous Laxity (Ehlers-Danlos, Marfan’s, Downs syndrome)
-Spastic conditions (40% incidence of HAV among those with CP)
PSH: -Previous F&A surgery
FH: -Hereditary component (63-68% family incidence among general population, 94% with juvenile HAV)
-Johnston reports an autosomal dominant component with incomplete penetrance
Meds/All: Usually non-contributory
ROS: Usually non-contributory

Objective: Physical Exam


Derm: -Dorsomedial erythema +/- bursa Ortho: -Dorsomedial eminence -Pes plano valgus
-Submet 2 lesion -Varus compensation -Equinus
-Nail bed rotational changes -Underlying met adductus -Hypermobile 1st ray
-Pinch callus -PROM 1st MPJ -LLD
Vasc/Neuro: Usually non-contributory -Tracking vs. Track-bound 1st MPJ

Objective: Radiographic Evaluation


Plain Film Radiographs: -Increased soft tissue density
-In first met head: subchondral bone cysts, osteophytes, hypertrophy of medial eminence
-Overall metatarsal parabola
-1st MPJ joint space: ~2mm of clear space; Congruent vs. Deviated vs. Subluxed
Angular deformities:
-Met Adductus (<15 degrees) -Meary’s Angle
-Engle’s Angle (<24 degrees) -Seiberg’s Angle
-IMA (<8 degrees) -TDA
-HAA (<15 degrees) -CIA
-HIA (<10 degrees) -Cyma Line
-Tibial sesamoid position (1-7) -Calcaneal-Cuboid Angle
-PASA (<8 degrees) -Talar Head Uncovering
-DASA (<8 degrees) -Talar Axis
-Met protrusion distance (<2mm) -Kite’s Angle

HAV Dissection and Capsule Procedures


-Anatomic Dissection
-1st incision is through epidermis and dermis
-Incision is planned along the dorsomedial aspect of the 1st MPJ, just medial to EHL and lateral to the medial dorsal
cutaneous nerve.
-From midshaft of 1st metatarsal to just proximal to the hallux IPJ
-Subcutaneous tissue is dissected to deep fascia/capsular layer
-NV structures: Superficial venous network, medial dorsal cutaneous nerve
-Be wary of the anterior resident’s nerve (Extensor capsularis)!
-Lateral Release
-Sequence of events:
-Release of adductor hallucis tendon from base of proximal phalanx and fibular sesamoid
-Release of fibular metatarsal-fibular sesamoid ligament and lateral capsule
-Tenotomy of the lateral head of the FHB between the fibular sesamoid and the proximal phalanx
-Optional excision of the fibular sesamoid
-Medial Capsulotomies
-Linear
-Washington Monument: Strongest medial capsulotomy allowing for both transverse and frontal plane correction
-Lenticular (Elliptical): Allows for transverse and frontal plane correction with removal of redundant capsule
-Inverted L: Transverse plane correction with removal of redundant capsule
-Medial T: Transverse plane correction with removal of redundant capsule
-Medial H: Transverse plane correction with removal of redundant capsule

90
AJM List: HAV Procedures and Indications
Distal Phalanx
1. Medial Nail Bed Rotation: Corrects soft tissue mal-alignment

Hallux IPJ
2. Amputation of the distal phalanx: Permanent correction of abnormal Hallux Interphalangeus Angle (HIA)
3. IPJ Fusion: Corrects abnormal HAI

Proximal Phalanx
4. Distal Akin: Corrects abnormal HAI with a medially-based wedge osteotomy at distal proximal phalanx
5. Central Akin: Corrects for long proximal phalanx seen with concurrent HL/HR
6. Oblique Akin: Corrects for distal articular set angle (DASA) midshaft proximal phalanx
7. Proximal Akin: Corrects for DASA of the proximal phalanx
8. Keller Arthroplasty: Corrects for abnormal Hallux Abductus Angle (HAA) and with concurrent HL/HR
9. Keller-Brandis Arthroplasty: Same as the Keller, but with removal of 2/3 of the proximal phalanx
10. Bonney-Kessel: Dorsiflexory osteotomy with concurrent HL/HR with modified forms correcting for abnormal DASA
11. Distal Hemi-Implant: Corrects for abnormal HAA or DASA with concurrent HL/HR
12. Regnauld: Allows for correction of DASA and abnormal proximal phalanx length in presence of HL/HR
13. Sagittal Z: Corrects for DASA and abnormal proximal phalanx length in presence of HL/HR

MPJ
14. Total Implant: Correction of HAA in presence of HL/HR
15. McKeever arthrodesis: Allows for permanent correction of DASA, PASA and HAA
16. McBride: Soft tissue reconstruction for correction of HAA
17. Modified McBride: Bone and soft tissue reconstruction for correction of HAA and medial eminence
18. Silver: Correction of medial eminence
19. Hiss: Modified McBride with Abductor hallucis advancement
20. External Fixation: Double Taylor frame for gradual soft tissue realignment
21. Hallux Amputation: Permanent correction of abnormal HAA

Distal 1st Met


22. Proximal Hemi-Implant: Correction of PASA and HAA with concurrent HL/HR
23. Mayo: First met head resection for correction of HAA with HL/HR
24. Stone: Mayo with sesamoid articulation left intact
25. Reverdin: Correction of PASA. Incomplete osteotomy.
26. Reverdin-Green: Correction of PASA with incomplete osteotomy and plantar shelf
27. Reverdin-Laird: Correction of PASA and IMA with complete osteotomy and plantar shelf
28. Reverdin-Todd: Correction of PASA, IMA and sagittal plane deformity (elevatus)
29. Youngswick: Correction of IMA and elevatus
30. Austin: Correction of IMA
31. Bicorrectional Austin: Correction of IMA and PASA
32. Tricorrectional Austin: Correction of IMA, PASA and elevatus
33. Mitchell: Rectangular osteotomy with lateral spicule to correct for IMA, elevatus and metatarsal length. Perpendicular to first met axis.
34. Roux: Wedged Mitchell to also correct for PASA
35. Miller: Mitchell with osteotomy oblique to first met axis for further correction of IM and length
36. Hohmann: Transverse through and through cut to correct for IMA and sagittal plane
37. Wilson: Oblique through and through osteotomy to correct for IMA and metatarsal length
38. Distal L: Similar to a Reverdin-Green without correction of PASA
39. Kalish: Austin with a long dorsal arm to allow for screw internal fixation
40. Mygind: Mexican hat procedure of distal first metatarsal for correction of IM and length
41. Off-set V/Vogler: Proximal Kalish
42. Peabody: Proximal Reverdin
43. Short-arm Scarf: Correction of IMA
44. Percutaneous DMO: Percutaneous Hohmann
45. DRATO (Derotational Abductory Transpositional Osteotomy): Can be used to correct frontal plane, IMA, sagittal plane and wedged for PASA
46. Distal Crescentic: Correction of IMA
47. Distal Crescentic with a shelf: Correction of IMA with greater stability

Central 1st Met


48. Scarf: Correction of IMA
49. Ludloff: Correction of IMA. Dorsal-proximal to distal-plantar cut.
50. Mau: Correction of IMA. Distal-dorsal to proximal-plantar cut.

Proximal 1st Met


51. Cresentic: Correction of IMA
52. Cresentic Shelf: Correction of IMA with greater stability
53. OBWO: Correction of IMA
54. Trethowan: OBWO using medial eminence for graft
55. CBWO (Loison-Balacescu): Closing base wedge proximal osteotomy. Corrects IMA.
56. Logroscino: CBWO with Reverdin. Corrects IMA and PASA.
57. Juvara: Oblique CBWO
58. Proximal Austin: Correction of IMA
59. Lambrinudi: Plantar CBWO to correct for sagittal plane

1st Met-Cunieform
60. Lapidus with internal fixation
61. Lapidus with external fixation
62. Cotton: OBWO of the cuneiform to correct for transverse plane
63. Westman: OBWO of the cuneiform to correct for sagittal plane
64. Cotton-Westman: OBWO of the cuneiform to correct for transverse and frontal plane

Misc.
65. 2nd digit amputation
66. EHL lengthening

91
AJM Sheet: HAV Complications

-Recurrence
-Early (<1 year)
-Usually due to wrong procedure choice, surgical error, or a post-operative complication.
-As little as 1% and as much as 14% rate reported (Kitaoka on 49 feet).
-Late (>1 year)
-Usually due to an unrecognized underlying deformity (such as met adductus, Ehlers-Danlos, equinus, 1st
met hypermobility, etc.)
-Symptoms usually worse than initial presentation
-Treatment: Distal soft tissue procedures or a proximal osteotomy usually indicated

-Hallux Varus
-Defined as a purely transverse plane adduction
-Hallux Malleus: extension at MPJ with flexion at IPJ

-Etiology
-Underlying causes: -Long 1st metatarsal
-Round 1st metatarsal head
-1st MPJ hypermobility

-Iatrogenic causes: -Staking of the 1st metatarsal head


-Overcorrection of the IM angle
-Overzealous medial capsulorraphy
-Fibular sesamoidectomy
-Over extensive lateral release
-Overcorrection of the PASA
-Overzealous bandaging

-Treatment: -Soft tissue rebalancing (medial releases and lateral tightenings)


-EHB tendon transfer
-Reverse distal osteotomies
-Ludloff/Mau
-Resection arthroplasty, implant, arthrodesis

-MalUnion/Delayed Union/Non-Union
-Malunion
-Consolidated osteotomy with an angular or rotational deformity
-Most common is sagittal plane abnormality (“dorsal tilting”)
-Must be corrected with an osteotomy

-AVN

92
AJM Sheet: HL/HR Work-up

Subjective
CC: Pt will generally complain of a “painful big toe.”
HPI: -Nature: Aching, Dull, Throbbing
-Location: Dorsal 1st MTPJ and within the joint
-Course: Usually gradual and progressive. May follow an acute traumatic event.
-Aggravating Factors: Shoe gear, WB
-Alleviating Factors: Ice, NSAIDs, Rest
PMH: -Inflammatory Condition: RA, SLE, Gout
PSH: -Past 1st MTPJ surgery
Meds/Allergies/SH/FH: Non-contributory
ROS: Non-contributory

Objective: Physical Exam


Derm: -Hyperkeratotic lesions: Plantar hallux IPJ, Medial pinch callus hallux IPJ, Submet 2
-Erythema, Calor, Dorsal 1st MTPJ bursa
Vasc/Neuro: Non-contributory
Ortho: -Decreased PROM 1st MTPJ -Varus Deformity Gait: -Early Heel-off
-Dorsal eminence 1st MTPJ -Plantar Contracture -Apropulsive Gait
-Dorsal eminence 1st Met-Cun -Equinus -Abductory Twist
-Hypermobile 1st ray

Objective: Imaging
Plain Film Radiographs: -Osteophytes at 1st MTPJ -Long 1st met
-Irregular Joint Space Narrowing -Long hallux proximal phalanx
-Lateral view: dorsal flag sign, dorsal lipping -Elevated 1st met
-Loose bodies (joint mice) -Osteophytes at hallux IPJ, 1st met-cun
-Square-shaped 1st met head

General HL/HR Information


-Definitions
-Hallux Limitus vs. Hallux Rigidus
-This is a progressive deformity, so what defines rigidus from limitus? Bony ankylosis and sesamoid immobilization.
-Functional HL is defined as a decreased PROM with the foot loading and in a neutral position, and normal PROM when the foot is unloaded.
Dannanberg first defined functional HL.
-Flexor Stabilization of the hallux: Essentially a hammertoe of the hallux with extension at the MTPJ and plantarflexion at the IPJ.
-Axis of rotation of the 1st MTPJ: Normally found in the center of the metatarsal head allowing for a gliding motion of the hallux up and over
the first metatarsal head. In a HL/HR deformity the axis of rotation moves distally and plantarly leading to dorsal jamming of the joint.
-Met Primus Elevatus: Dorsiflexed position of the 1st metatarsal.
-Primary: Structural. Distal segment is dorsiflexed compared to proximal segment.
-Secondary: Global. Due to some extrinsic variable. This can be measured by Meary’s Angle on a lateral plain film radiograph or
using the Seiberg technique comparing the 1st and 2nd metatarsal positions.
-What stimulates osteophyte production in and around the joint?: Loss of functional cartilage.

-1st MTPJ ROM


-Normal PROM of the 1st MTPJ is classically described as 65-75 degrees of dorsiflexion of the hallux referenced to the weight-bearing surface
(same as 85-95 degrees of dorsiflexion referenced to the 1st met shaft). Plantarflexion is 30 degrees to the weight-bearing surface.
-Hetherington contradicts this somewhat by finding an average ROM of 31degrees of dorsiflexion during pain-free gait in asymptomatic patients.

-Compensation Patterns for Lack of Motion


-Distal: Hallux IPJ leading to OA and plantar hyperkeratotic lesions
-Lateral: Lesser metatarsalgia
-Proximal: 1st met-medial cuneiform joint increased motion and OA
-Gait patterns: Abductory twist with roll-off; early heel-off; apropulsive gait

-HL/HR Etiology
-Many people have reported potential causes of HL/HR including Root, Lapidus and Nilsonne:
-Acute Trauma -Chronic degenerative trauma
-Pes planus with 1st met hypermobility -Long first metatarsal
-Short first metatarsal with hallux gripping -Long hallux proximal phalanx
-Iatrogenic -Compensated varus deformity
-Neuromuscular imbalance -Plantar contracture
-Spastic conditions -Square first metatarsal head shape
-Met primus elevatus
-No single characteristic has been shown to reliably lead to HL/HR except acute trauma
-Coughlin (FAI 2003) performed a retrospective analysis and seemed to demonstrate that there are no reliable underlying indicators for
development of HL/HR.

-Classification Systems
-Numerous exist; usually in the mild, moderate, severe format:
-Mild: Mild pain; Normal PROM; Radiographic evidence of osteophytes
-Moderate: Increasing pain; Decreasing PROM; Osteophytes and irregular joint space narrowing on radiograph
-Severe: Increasing pain; Decreasing PROM; Osteophytes, irregular joint space narrowing, subchondral sclerosis on radiograph.
-Rigidus: Increasing pain; Absent PROM; Sesamoid immobility
-Examples of classifications include the Regnauld, Hanft and KLL.

93
AJM Sheet: HL/HR Treatment

-Conservative
-Do nothing
-Activity modification
-Orthotics: First ray cut-out, Morton’s extension, rocker-bottom sole
-Meds: PO NSAIDs, Intra-articular corticosteroid injections

-Surgical
-Surgical options are always divided into joint-sparing and joint-destructive procedures, and further divided into
whether the correction occurs at the proximal phalanx, at the MTPJ, or at the first metatarsal.

-Joint Sparing
-Proximal Phalanx
-Bonney-Kessell
-Regnauld
-Vanore
-Sagittal “Z”
-Central Akin
-1st MTPJ
-Cheilectomy
-1st Metatarsal
-Youngswick
-Watermann
-Watermann-Green
-Jacoby
-Hohmann
-Derner
-Dorsal OBWO
-Lambrinudi
-Westman

-Joint Destructive
-Proximal Phalanx
-Keller
-Keller-Brandis
-Distal Hemi-Implant
-1st MTPJ
-Total Implant
-McKeever arthrodesis
-Valenti
-1st Metatarsal
-Mayo
-Stone
-Lapidus

94
AJM Sheet: Flatfoot Work-up
-This is a lot of information to cover in 2 pages, so these sheets will focus on clinical and radiographic signs, as well as
indications for specific surgeries. Also, will try and include a good amount of additional readings.

Subjective
-Wide range of presenting ages and complaints.
-Always think about posterior tibialis tendon dysfunction when someone complains of “medial ankle pain.”

Objective
-Underlying Orthopedic Etiologies: -Compensated forefoot varus -Forefoot valgus
-Rearfoot valgus -Equinus
-Compensated and uncompensated ab/adduction deformities
-Muscle imbalances (PTTD) -Ligamentous laxity
-Tarsal coalitions
-Planal dominance
-Normal STJ axis: 42° from transverse/16° from sagittal
-Normal MTJ-O: 52° from transverse/57°from sagittal
-Normal MTJ-L: 15° from transverse/9° from sagittal

-Clinical findings: -“Too many toes” sign (forefoot abduction) -Hubscher maneuver
-Evaluation for flexible versus rigid deformity -RCSP
-Single and double heel raise -Subjective gait analysis

-Radiographic evaluation:
-Lateral: -Decreased calcaneal inclination angle -Anterior break in Cyma line
-Increased talar declination angle -Meary’s Angle
-Increased first metatarsal declination angle -Midfoot “breaks” or “incongruity”
-Calcaneal-cuboid “break”

-AP: -Increased talo-calcaneal angle -Talar-first metatarsal axis


-Cuboid-abduction angle -Intermetatarsal angle
-Talar head coverage -Forefoot adduction angle or Engle’s Angle
-Look for “skew foot” deformity

-Harris-Beath: Evaluation of tarsal coalitions


-Long-Leg Calcaneal Axial Views: Evaluation of structural rearfoot deformities

Classifications
-Johnson and Strom [Johnson KA, Strom DE. Tibialis posterior tendon dysfunction. CORR. 1989; 239: 196-206.]
-Later modified by Myerson who added Stage IV (he does that a lot):
-[Myerson MS. Adult acquired flatfoot deformity: treatment of dysfunction of the posterior tibial tendon. JBJS-Am. 1996;
78: 780-92.]
-[Bluman EM, et al. Posterior tibial tendon rupture: a refined classification system. Foot Ankle Clin. 2007 Jun; 12(2):
233-49.]
-Stage I: Tenosynovitis with mild tendon degeneration; flexible rearfoot; Mild weakness of single heel raise and
negative “too many toes” sign
-Stage II: Elongated tendon with tendon degeneration; flexible rearfoot; Marked weakness of single heel raise and
positive “too many toes” sign
-Stage III: Elongated and ruptured tendon; Rigid valgus rearfoot; Marked weakness of single heel raise and positive
“too many toes” sign
-Stage IV: Same as Stage III with a rigid ankle valgus
-Funk: Classification based on gross intra-operative appearance
-[Funk DA, et al. Acquired adult flatfoot secondary to posterior tibial tendon pathology. JBJS-Am. 1986; 68: 95-102.]
-Type I: Tendon Avulsions -Type III: In-continuity tears
-Type II: Complete midsubstance rupture -Type IV: Tenosynovitis
-Jahss or Janis Classifications: There are several MRI classifications generally along the lines of:
-[Conti S, Michelson J, Jahss M. Clinical significance of MRI in preoperative planning for reconstruction of posterior tibial tendon ruptures.
Foot Ankle. 1192; 13(4): 208-214.]
-[Janis LR, et al. Posterior tibial tendon rupture: classification, modified surgical repair, and retrospective study. JFAS. 1993; 31(1): 2-13.]

-Type I: Tenosynovitis, increased tendon width, mild longitudinal splits


-Type II: Long longitudinal splits with attenuated tendon
-Type III: Complete rupture

Additional Reading:
-[Mendicino RW, et al. A systemic approach to evaluation of the rearfoot, ankle and leg in reconstructive surgery. JAPMA. 2005; 95: 2-12.]
-[Lamm BM, Paley D. Deformity correction planning for hindfoot, ankle and lower limb. Clin Podiatr Med Surg. 2004 Jul; 21(3): 305-26.]
-[Greisberg J, Hansen, Sangeorzan. Deformity and degeneration in the hindfoot and midfoot joints of the adult acquired flatfoot. Foot Ankle Int.
2003 Jul; 24(7): 530-4.]
-[Weinraub GM, Saraiya MJ. Adult flatfoot/posterior tibial tendon dysfunction: classification and treatment. Clin Podiatr Med Surg. 2002 Jul;
19(3): 345-70.]

95
AJM Sheet: Flatfoot Treatment
-Again, this is a lot of information to cover, so we’ll just focus on organizing general procedures and indications, but supplement
it with some additional reading.

-Conservative Treatments
-Not going to be discussed here, but try reading:
-[Elftman NW. Nonsurgical treatment of adult acquired flatfoot deformity. Foot Ankle Clin. 2002 Mar; 7(1): 95-106.]
-[Marzano R. Functional bracing of the adult acquired flatfoot. Clin Podiatr Med Surg. 2007 Oct; 24(4): 645-56.]

-Johnson and Strom/Myerson Classification:


-In addition to describing the deformity, this classification system (discussed on a previous sheet) also makes general
treatment recommendations:
-Stage I: Conservative treatment; Tenosynovectomy; Tendon Debridement
-Stage II: Tendon transfer; Rearfoot arthrodesis
-Stage III: Isolated rearfoot arthrodesis; Triple arthrodesis
-Stage IV: TTC arthrodesis; Pantalar arthrodesis

-General Surgical Procedures/Indications:


-Keep in mind that it is very common to do combinations of these procedures.

Soft Tissue Procedures:


-FDL Tendon Transfer: FDL is sectioned as distal as possible (consider anastomosis of stump to FHL) and either
attach proximal FDL to the PT, within the PT sheath or into the navicular under tension.
-Cobb: Split TA tendon, transfer to the PT or into the navicular
-Young’s Tenosuspension: TA rerouted through navicular
-Anastomosis of PB and PL: Removes PB as deforming force
-STJ implant (arthroeresis)
-TAL -Gastroc recession

Rearfoot Osseous Procedures:


-Evans Osteotomy (1975): opening wedge calcaneal osteotomy
-Silver (1967) is a more proximal (and less common) Evans-type opening wedge
-[Sangeorzan BJ, et al. Effect of calcaneal lengthening on relationships among the hindfoot, midfoot and forefoot. Foot Ankle. 1993;
14(3): 136-41.]
-[Raines RA, et al. Evans osteotomy in the adult foot: an anatomic study of structures at risk. Foot Ankle Int. 1998 Nov; 19(11): 743-7.]
-[Weinraub GM. The Evans osteotomy: technique and fixation with cortical bone pin. JFAS. 2001; 40(1): 54-7.]
-[DeYoe BE, Wood J. The Evans calcaneal osteotomy. Clin Podiatr Med Surg. 2005 Apr; 22(2): 265-76.]
-Medial Calcaneal Slide (Koutsogiannis - 1971): medial translation of posterior calcaneus
-[Weinfeld SB. Medial slide calcaneal osteotomy. Technique, patient selection and results. Foot Ankle Clin. 2001 Mar; 6(1): 89-94.]
-[Catanzariti AR, et al. Posterior calcaneal displacement osteotomy for adult acquired flatfoot. JFAS. 2000; 39(1): 2-14.]
-Double Calcaneal Osteotomy: Evans with a medial calcaneal slide
-[Catanzariti AR, et al. Double calcaneal osteotomy: realignment considerations in eight patients. JAPMA. 2005; 95(1): 53-9.]
-[Nyska M, et al. The contribution of the medial calcaneal osteotomy to the correction of flatfoot deformities. Foot Ankle Int. 2001
Apr; 22(4): 278-82.]
-STJ arthrodesis -Triple arthrodesis
-Tibiotalocalcaneal arthrodesis -Pantalar arthrodesis
-Closing wedge of the medial calcaneus: Essentially the opposite of an Evans but with more NV structures
-Dwyer (1960)
-Slakovich: opening wedge behind the sus tali
-Baker-Hill: opening wedge osteotomy with graft horizontally under the posterior calcaneal facet

Midfoot/Forefoot Osseous Procedures:


-Cotton: opening wedge osteotomy with graft in medial cuneiform
-Kidner: advancement and reattachment of PT tendon (+/- resection of portion of navicular)
-TN arthrodesis: called a Lowman when wedged and combined with TAL
-Medial column arthrodeses
-Miller: NC and 1st met-cun arthrodesis
-Lapidus (1931/1960): 1st met-cun arthrodesis
-Hoke: arthrodesis of navicular with 1st/2nd cuneiforms
-Any other combinations
-[Greisberg J, et al. Isolated medial column stabilization improves alignment in adult-acquired flatfoot. CORR. 2005 Jun;
435: 197-202.]

-Additional Reading:
-[Hix J, et al. Calcaneal osteotomies for the treatment of adult-acquired flatfoot. Clin Podiatr Med Surg. 2007 Oct; 24(4): 699-719.]
-[Mosier-LaClair S, et al. Operative treatment of the difficult stage 2 adult acquired flatfoot deformity. Foot Ankle Clin. 2001 Mar; 6(1): 95-
119.]
-[Roye DP, Raimondo RA. Surgical treatment of the child’s and adolescent’s flexible flatfoot. Clin Podiatr Med Surg. 2000 Jul; 17(3): 515-30.]
-[Toolan BC, Sangeorzan, Hansen. Complex reconstruction for the treatment of dorsolateral peritalar subluxation of the foot. JBJS-Am. 1999
Nov; 81(11): 1545-60.]
-[Weinraub GM, Heilala MA. Adult flatfoot/posterior tibial tendon dysfunction: outcomes analysis of surgical treatment utilizing an algorithmic
approach. J Foot Ankle Surg. 2001 Jan-Feb; 40(1): 54-7.]
96
AJM Sheet: Cavus Foot Work-up

-The cavus foot work-up is one of the most feared in the residency interview process because of its complex nature. The most important
technique during this work-up is to use a standardized system to identify several specific variables which will let you best identify the
deformity and decide on a treatment course:
-Underlying Etiology of the Deformity (Spastic vs. Progressive vs. Stable)
-Forefoot vs. Rearfoot driven deformity (Anterior Cavus vs. Posterior Cavus)
-Plane of the Deformity (Sagittal vs. Frontal vs. Transverse vs. Combination)
-Rigid vs. Flexible

-Underlying Etiology of the Deformity (Spastic vs. Progressive vs. Stable)


-Obtained through a good PMH and physical exam
-Brewerton of the Royal Hospital in London identified 75% of 77 patients seen at his pes cavus clinic to have an underlying
neuromuscular disorder.
-Common Congenital Conditions leading to neuromuscular dysfunction:
-Spina bifida -Charcot-Marie-Tooth
-Myelodysplasia -Friedreich’s Ataxia
-Cerebral Palsy -Roussy-Levy syndrome
-Muscular Dystrophy -Dejerine-Sottas
-Poliomyelitis -Etc, etc, etc.
-Also consider MMT, clonus, deep tendon reflexes, EMG studies and nerve conduction studies during your physical exam.

-Forefoot vs. Rearfoot Driven Deformity


-Anterior Cavus: plantar declination of the forefoot in relation to the rearfoot. Subdivided based on apex of deformity:
-Metatarsus Cavus: Apex at Lisfranc’s joint. Generally more rigid.
-Lesser Tarsus Cavus: Apex in the lesser tarsus area
-Forefoot Cavus: Apex at Chopart’s joint
-Combined Cavus: Combination of any of the above

-The apex of the deformity can be found several different ways:


-Intersection of Meary’s lines (longitudinal axes of talus and first met)
-Intersection of Hibb’s Angle (longitudinal axes of calcaneus and first met)
-Dorsal boney prominences
-Joint space gapping
-Posterior Cavus: Dorsiflexion of the rearfoot in relation to the forefoot
-Generally defined as an increased calcaneal inclination angle (>30 degrees) and a varus positioning.
-Usually the result of an anterior cavus; rarely presents as separate entity.

-An anterior cavus and a posterior cavus can be defined based on radiographic evidence and a physical exam measure called the
Coleman Block Test. In this test the forefoot, or the medial and lateral portions of the forefoot, are suspended off of a block. If the
calcaneus returns from a varus to a normal position, the deformity is forefoot driven. A deformity is rearfoot driven only if the varus
positioning of the calcaneus remains after all forefoot elements are removed.

-Biomechanical compensation for a sagittal plane cavus deformity:


-Digital retraction: HT deformity where EDL gains mechanical advantage and uses a passive pull.
-MPJ Retrograde buckling: As per above
-Lesser Tarsal Sagittal Plane Flexibility: The lesser tarsus “absorbs” some of the dorsiflexion. They can be clearly
seen when comparing NWB and WB lateral views of an anterior cavus foot.
-Pseudoequinus: Occurs when the ankle joint must dorsiflex because the lesser tarsus cannot “absorb” all of the
dorsiflexion. Limits the amount of “free” dorsiflexion available during gait.

-Plane of the Deformity:


-Sagittal Plane: -Anterior Cavus (Global, Medial Column, or Lateral Column)
-Posterior Cavus
-Muscular Cavus (Gastroc Equinus, Gastroc-Soleus Equinus)
-Osseous Equinus (Tibiotalar exostosis)
-Transverse Plane: -Met adductus (measured via met adductus angle or Engle’s angle)
-Met abductus (measured via met adductus angle)
-Frontal Plane: -Forefoot Varus
-Forefoot Valgus
-Rearfoot Varus
-Rearfoot Valgus

-Rigid vs. Flexible Deformity


-Flexible deformities can be manipulated out during the physical exam and are obvious comparing NWB and WB lateral
radiographs.
-Rigid deformities show no compensation with manipulation of weight-bearing.
-Defining each of these variables during your work-up will give you a clear enough understanding of the deformity to recommend a
treatment option.

97
AJM Sheet: Cavus Foot Treatment

-Basic principles of treatment based on definition of deformity:


-Underlying Etiology:
-Progressive/Spastic conditions are generally treated with osteotomies and arthrodeses.
-Stable conditions are generally treated with soft tissue procedures and osteotomies.
-Forefoot vs. Rearfoot Driven:
-Forefoot driven conditions are treated with manipulation of the bones and soft tissue of the forefoot.
-Rearfoot driven conditions require rearfoot osteotomies and arthrodeses.
-Plane of the Deformity:
-Procedures are chosen by which plane you want correction in.
-Rigid vs. Flexible:
-Rigid deformities are generally treated with osteotomies and arthrodeses.
-Flexible deformities can usually be managed with soft tissue procedures and tendon transfers.

-Soft Tissue Releases: Reduces contracture of the plantar fascia seen with long standing disease.
-Subcutaneous Fasciotomy: Cuts the plantar fascia at its insertion.
-Steindler Stripping: Removes all soft tissue from the plantar surface of the calcaneus.
-Plantar Medial Release: Releases plantar musculature and ligaments from the plantar-medial foot.

-Tendon Transfers: Used to treat flexible conditions based upon plane of the deformity.
-Jones Suspension: Transfer of EHL through the first metatarsal head.
-Heyman Procedure: Transfer of EHL and EDL tendons through each of the respective metatarsal heads.
-Hibbs Procedure: Transfer of EDL into lateral cuneiform; EHL into first metatarsal; EDB into sectioned tendons.
-STATT: Tibialis anterior is split and sutured into peroneus tertius.
-Peroneus Longus Transfer: Peroneus longus is split and anastomosed to the TA and peroneus tertius tendons.
-Peroneal Anastomosis: Increases the eversion power of the foot.
-PL/PT transfer to calcaneus: Tendons are attached into the calcaneus via bone anchors to aid weak Achilles
tendon.

-Osseous Procedures: Reduction of rigid deformities. Can be used to correct multi-planar deformities.
-Cole Procedure: Dorsiflexory wedge is removed from Chopart’s joint.
-Japas Procedure: “V” shaped osteotomy through the midfoot (apex proximal) to dorsiflex forefoot.
-Jahss Procedure: Essentially a Cole procedure performed at Lisfranc’s joint.
-Dorsiflexory Metatarsal Osteotomies
-Dwyer Osteotomy: Closing wedge osteotomy out of lateral calcaneus to reduce rigid rearfoot varus.
-Dorsiflexory Calcaneal Osteotomy: Must be used with caution

-Arthrodesis Procedures: Used as last resort to correct rigid deformities in the face of progressive disease.
-Hoke: STJ and TNJ arthrodesis
-Ryerson (1923): Triple arthrodesis

-Additional Reading:
-[Younger AS, Hansen. Adult cavovarus foot. J Am Acad Orthop Surg. 2005 Sep; 13(5): 302-25.]
-[Statler TK, Tullis BL. Pes Cavus. JAPMA. 2005; 95: 34-41.]

98
AJM Sheet: Ankle Equinus

-This sheet is not a work-up because equinus rarely presents as a chief concern, but rather as a concomitant and underlying deformity. It
may be seen and deemed correctable in the following deformities:
-Charcot arthropathy -Digital deformities
-Pes plano valgus -Met primus elevatus
-HAV -Plantar fasciitis
-Medial column hypermobility -Diabetic foot ulcerations -Etc.

-History
-First TAL: Paris on Achilles in the “Iliad”
-First medically documented procedure: Stromeyer on Dr. Charles Little. Dr. Little was a prominent physician suffering from
cerebral palsy (CP) who then became an advocate for surgical correction of equinus.

-Anatomy
-Review the origins/insertions/course/action/NV supply of the gastroc and soleus.
-Review the concept of the “twisting” fibers within the Achilles tendon.
-[White JW. Torsion of the Achilles tendon: its surgical significance. Arch Surg 1943; 46: 784-7.]

-Definitions
-Muscular Equinus
-Spastic vs. Non-Spastic
-Gastroc Equinus
-Gastroc-Soleal Equinus
-Osseous Equinus
-Tibio-talar exostosis
-Pseudoequinus
-Combination equinus

-Biomechanic Compensation for Equinus (proximal to distal)


-Lumbar lordosis -STJ pronation
-Hip flexion -MTJ pronation
-Genu recurvatum -Forefoot abduction
-No compensation (toe walking) -Medial column hypermobility

-Testing for Equinus


-Silfverskiold test
-Stress dorsiflexion plain film radiographs
-WB wall test

Treatment
-Conservative Treatment
-Stretching -Heel Lifts (?) -Casting
-Physical Therapy -Neuromuscular blockage injections (Botox)
-Surgical Correction
-Gastroc Equinus
-Neurectomy of motor branches of tibial nerve
-Proximal recession (Silfverskiold procedure)
-Release of muscular heads of gastroc +/- reattachment to tibia +/- neurectomy
-Distal aponeurotic recession
-Vulpius&Stoffel (1913): Inverted “V” shaped incision without suture reapproximation
-Strayer (1950): Transverse incision with proximal dissection and suturing (absorbable)
-Baker (1956): Tongue and groove with suturing (two incisions distal)
-Fulp&McGlamry: Inverted tongue and groove with suturing (two incisions proximal)
-Endoscopic recession
-Gastroc-Soleal Equinus
-Sagittal plane “Z” lengthening: equal medial and lateral portions
-Frontal plane “Z” lengthening: equal anterior and posterior portions
-Hoke Triple Hemisection (1931): 2 medial cuts/1 lateral cut
-White slide technique
-Percutaneous
-Similar to the Hoke procedure
-1cm  3cm  3 cm

Additional Reading:
-[Pinney SJ, Hansen, Sangeorzan. The effect on ankle dorsiflexion of gastrocnemius recession. Foot Ankle Int. 2002 Jan; 23(1): 26-9.]
-[Pinney SJ, Hansen, Sangeorzan. Surgical anatomy of the gastrocnemius recession. Foot Ankle Int. 2004 Apr; 25(4): 247-50.]
-[Lamm BM, Paley, Herzenberg. Gastrocnemius soleus recession: a simpler, more limited approach. JAPMA. 2005; 95: 18-25.]
-[Graham HK, Fixsen JA. Lengthening of the calcaneal tendon in spastic hemiplegia by the White slide technique. A long term review. JBJB-Br. 1988 May;
70(3): 472-75.]
99
AJM Sheet: Page 100

One of the most frequent questions asked by students and externs is “Can you give me some good articles to read?”

-I tried to do this by incorporating current and historical articles throughout this edition. All referenced articles (as well as
other suggested readings) can be found on the Inova Pubmed page:

-www.pubmed.com
-My NCBI link on the left

-User Name: INOVA


-Password: resident
-Although this was specifically designed for the use of externs and residents at the Inova program, the collection of
articles on this page can be used by anyone. Additionally, anyone is welcome and encouraged to update these collections
with articles they feel are valuable.

----------------------------------------------------------------------------------------------------------------------------------

In conclusion, this PRISM was not designed to help you pass the boards or even to directly
make you a better physician; it simply hopes to make you better prepared and more efficient as
you approach externships and the residency interview. Use, change, and pass this guide along as
you see fit, keeping in mind the general goal of selfless education of the next generation. Good
luck, and please don’t hesitate to contact me if there is any way that I can be of service to you.
AJMeyr@gmail.com

100
CROZER-KEYSTONE
RESIDENCY MANUAL
SECOND EDITION
Original Author
Brett Chicko, DPM

Second Edition
Sandi Pollard, DPM
Hubert Lee, DPM
Table of Contents
Introduction William Urbas, DPM ................................................................................. i
Brett Chicko, DPM ................................................................................... ii

Chapter 1 Anatomy ......................................................................................................1

Chapter 2 Antibiotics ...................................................................................................7

Chapter 3 Bugs and Drugs ........................................................................................19

Chapter 4 Labs ...........................................................................................................25

Chapter 5 Medicine ....................................................................................................29

Chapter 6 Clinical Podiatry ......................................................................................51

Chapter 7 Biomechanics ............................................................................................59

Chapter 8 Surgery ......................................................................................................67

Chapter 9 Trauma......................................................................................................85

Chapter 10 Bone Tumors ............................................................................................91

Chapter 11 Wound Care .............................................................................................97

Chapter 12 Classifications
Forefoot ..........................................................................................99
Rearfoot........................................................................................106
Other Classifications ....................................................................121

Chapter 13 Special Studies ........................................................................................127

Chapter 14 Name That Surgery! ..............................................................................131

Chapter 15 Special Surgical Section


Introduction ..................................................................................149
Achilles Tendon
Achilles Tendon Repair ...................................................150
Delayed Repair of the Achilles Tendon ...........................152
Excision of Calcification of Achilles Tendon ..................154
Murphy Procedure ...........................................................155
Arthrodesis
Ankle Arthrodesis ............................................................156
Tibio-Calcaneal Arthrodesis ............................................158
Subtalar Arthrodesis.........................................................160
Talo-Navicular Arthrodesis .............................................161
Triple Arthrodesis ............................................................163
Calcaneal Osteotomy
Calcaneal Slide Osteotomy ..............................................164
Evans ................................................................................165
Tendons
Peroneal Brevis Tendon Repair and Reconstruction .......166
Posterior Tibial Tendon Repair-Substitution ...................167
Posterior Tibial Tendon Transfer .....................................169
Split TA Tendon Transfer (STATT) and
Tibialis Anterior Tendon Transfer (TATT) .....................170
Trauma
Ankle Fracture Weber A ..................................................171
Ankle Fracture Weber B ..................................................173
Medial Malleolar Fractures ..............................................177
Posterior Malleolar Fracture ............................................180
Syndesmotic Repair .........................................................181
ORIF Calcaneus ...............................................................183
ORIF Talus Neck .............................................................185
Tibial Periarticular Fracture Reduction & Fixation .........188
Other Surgeries
Ankle Arthroscopy ...........................................................190
Arthroereisis.....................................................................192
Arthrosurface 1st Metatarsal Implant ...............................193
Brostrom-Gould ...............................................................195
Fibular Derotational and Lengthening Osteotomy ..........196
Ilizarov Method................................................................197
Osteochondral Lesions of the Talus .................................198
Tarsal Tunnel Release ......................................................199

Chapter 16 Case Studies


Case 1 ..........................................................................................201
Case 2 ..........................................................................................203

Chapter 17 The Interviews ........................................................................................205


Introduction

Dear Student:

In the pages that follow is useful information that will help make your externships, interviews
and transition from student to resident a little bit easier. This information contained within is not
the end all on the subject, but the tidbits that need to be on your “Mind’s Fingertips”.

This booklet is a useful guide on the need-to-know, need-to-keep information. Please use it as it
was intended--a guide on the ever-changing world of medical information. My thanks go out to
the Podiatric Surgical Residents at Crozer-Keystone Health System for the formation of this
manual.

Sincerely,

William M. Urbas, DPM


Crozer-Keystone Residency Director

i
Author’s Introduction
This manual is NOT meant to replace “McGlamry’s”, the “Presbyterian Manual”, the “Podiatry
Institute Manual” or any other reference source. Those manuals are excellent resources and
should be used to continue to learn the information. To this day, I still use those texts for
information and reference.

This manual is based on questions I came across as an extern or a resident, either from my own
questions or questions from a superior. I would write these questions down and after I looked up
the answers, I would keep the questions with their answers in a log.

Later, as a resident, I was quizzing a student in order to get her ready for her interviews. The
student asked me, “Why can’t there be a book of these questions?” After that I started to put
together the manual. I also added some additional items to complete the manual.

In no way, shape or form do I claim that the answers written here are the only answers possible,
nor do I even claim that they are all 100% correct. These answers are the ones that I came up
with when I researched the questions. It is up to you to go to the true references--not only to
make sure that the answers are correct, but also to make sure that you understand why.

Therefore, the purpose of this manual is so that the reader can have some questions and answers
so that he or she can go to the sources and really learn podiatry.

I am not able to provide all of my sources because when I started writing down the answers, I
had no idea of turning it into a manual. However, my major sources are, “The Comprehensive
Textbook of Foot Surgery”, “The Presbyterian Manual” and “The Podiatry Institute Manual”. A
special thank you to my attendings and co-residents at the Crozer-Keystone Health Systems in
Springfield, PA, especially Dr. Urbas our residency director, mentor and friend.

Good Luck and Happy Studying,

Brett Chicko, DPM

ii
Anatomy
How many bones are in the foot?
26 (not including sesamoids)

How many joints are in the foot?


35

Name the accessory ossicles


Os Intermetatarsium Between 1st cuneiform and 1st and 2nd metatarsal bases
Os Vesalianum Proximal 5th metatarsal base
Os Tibiale Externum Accessory navicular
Os Supranaviculare Dorsal aspect of navicular
Os Peroneum Sesamoid bone in PB tendon
Os Calcaneus Secondarius Dorsal, anterior process of calc
Os Sustentaculi Posterior aspect of sustentaculum tali
Os Trigonum Posterior aspect of talus (Steida process)
Os Subtibiale Distal to medial malleolus
Os Subfibulare Distal to lateral malleolus

Name the avascular necroses


Renandier Tibial sesamoid
Trevor Fibular sesamoid
Theiman Phalanges
Freiberg Metatarsal heads
Iselen 5th metatarsal base
Buschke Cuneiforms
Kohler Navicular
Lance Cuboid
Diaz Talus
Severe Calcaneus
Blount Proximal, medial tibial epiphysis
Osgood-Schlatter Tibial tuberosity
Legg-Calve-Perthes Femoral epiphysis

What attaches periosteum to bone?


Sharpey fibers

What are the different types of coalitions?


Syndesmosis – fibrous
Synchondrosis – cartilaginous
Synostosis – osseous

CROZER-KEYSTONE RESIDENCY MANUAL – SECOND EDITION 1


What is the difference between a coalition and a bar?
Coalition – intra-articular fusion of two bones
Bar – extra-articular fusion

What is the most common coalition in the foot?


Distal and middle phalanx of 5th digit

What is the most common coalition in the rearfoot?


Talocalcaneal

What is a Steida process?


Enlarged Os Trigonum

What is the only bone in the foot without any muscle origin or tendon insertion?
Talus

What are the plantar muscle layers of the foot from superficial to deep?
1. Abductor hallucis, flexor digitorum brevis, abductor digiti minimi
2. Quadratus plantae, 4 lumbricals
3. Flexor hallucis brevis, adductor hallucis, flexor digiti minimi
4. 3 plantar interossei, 4 dorsal interossei

What layer of the foot does FDL run?


2nd layer – it is the origin of the lumbricals and the insertion of QP

What deformity will result from cutting QP?


Digits 4 and 5 will become adductovarus

How is EDL attached to the proximal phalanxes?


Sling wraps around capsule which attaches to plantar plate, DTML, and flexor tendon sheath
thus attaching to plantar proximal phalanx. No direct insertion to proximal phalanx.

What is the origin and insertion of the capsularis tendon?


Origin – extensor hallucis longus muscle or tendon
Insertion – first metatarsophalangeal joint capsule

What is the Master Knot of Henry?


Fibrous connection between FHL and FDL tendons

What structures attach to the fibular sesamoid?


Plantar metatarsal-phalangeal ligament
Lateral metatarsal-sesamoidal ligament
Intersesamoidal ligament
Phalangeal-sesamoidal ligament
FHB tendon
ADH tendon

2 CROZER-KEYSTONE RESIDENCY MANUAL – SECOND EDITION


Are the sesamoids capsular or extra-capsular?
Capsular

What is the Lisfranc ligament?


Attaches lateral aspect of medial cuneiform to medial base of 2nd metatarsal

What structures in the Lisfranc joint are not connected by ligaments?


1st and 2nd metatarsals

What is the spring ligament?


Plantar calcaneonavicular ligament

What ligaments compose the bifurcate ligament?


Dorsal calcaneonavicular and calcaneocuboid ligaments

Which is stronger – the lateral ankle ligaments or the deltoid ligament?


Deltoid ligament

What are the components of the deltoid ligament?


Superficial – tibionavicular, tibiocalcaneal, posterior tibiotalar
Deep – anterior tibiotalar

What tendons pass over the deltoid ligament?


Tibialis posterior and FDL

What are the lateral ankle ligaments?


Anterior talofibular, calcaneofibular, posterior talofibular

What angle do the ATFL and CFL create?


105°

What is the strongest lateral ankle ligament?


Posterior talofibular

Which ankle ligaments are extra-capsular? Which are capsular?


Calcaneofibular ligament is extra-capsular, all others are capsular

What tendons pass over the lateral ankle ligaments?


Peroneus brevis and longus

What ligaments support the ankle syndesmosis?


Anterior-inferior tibiofibular ligament
Posterior-inferior tibiofibular ligament
Interosseous tibiofibular ligament

CROZER-KEYSTONE RESIDENCY MANUAL – SECOND EDITION 3


What is the Bassett ligament?
Anterior-inferior tibiofibular ligament

What is another name for the flexor retinaculum?


Laciniate ligament

What is another name for the superior extensor retinaculum?


Transverse crural ligament

What is another name for the inferior extensor retinaculum?


Cruciate crural ligament

Where does plantaris insert?


Medial aspect of tendo-Achilles into the calcaneus

What is the incidence of peroneus quartus?


7%

What is the Hoke tonsil?


Fibrous, fatty plug within the sinus tarsi

What is pes anserinus?


Insertion of sartorius, gracilis, and semitendinosus (anteromedial aspect of proximal tibia) where
bursa may cause knee pain (pes anserinus bursitis)

What is a Bakers cyst?


Swelling of the bursa between the tendons of the medial head of the gastrocnemius and the
semimembranosus muscles

What is a fabella?
Sesamoid bone occasionally found in tendon of lateral head of gastrocnemius

What nerves form the sural nerve?


Medial sural cutaneous nerve – branch of the tibial nerve
Sural communicating branch – branch of the lateral sural cutaneous nerve, which originates from
the common peroneal nerve

Does a neuroma lie dorsal or plantar to the deep transverse intermetatarsal ligament?
Plantar

Where do these muscles run in relation to the deep transverse intermetatarsal ligament?
Interossei – dorsal
Lumbricals – plantar

4 CROZER-KEYSTONE RESIDENCY MANUAL – SECOND EDITION


What layers of the foot do the plantar nerves run?
Medial plantar nerve – in the 1st layer (between FDB and abductor hallucis)
Lateral plantar nerve – between the 1st and 2nd

What is the innervation to the plantar muscles of the foot? Blood supply?
(Never LAFF at A FAD)
N – medial plantar Nerve
L – 1st Lumbrical
A – ABH
F – FHB
F – FDB (innervated by both medial and lateral plantar nerves)
A – medial plantar Artery
F – FDB
A – ABH
D – 1st Dorsal interossei

What are the branches of the femoral nerve?


Nerve to femoral artery
Small muscular branch to pectineus
Anterior division (cutaneous)
 Anterior femoral cutaneous
 Nerve to sartorious
 Intermediate femoral cutaneous nerve
 Medial femoral cutaneous nerve
Posterior division (muscular)
 Saphenous nerve
 Infrapatellar branch
 Medial crural cutaneous nerve
 Nerve to rectus femorus
 Nerve to vastus medialus
 Nerve to vastus intermedialus
 Nerve to vastus lateralus

What are the branches of the femoral artery?


Superficial epigastric artery
Superficial circumflex iliac artery
Superficial external pudendal artery
Deep femoral (profunda femoris) artery
Medial femoral circumflex artery
Lateral femoral circumflex artery
Descending genicular artery
Femoral artery continues as the popliteal artery

CROZER-KEYSTONE RESIDENCY MANUAL – SECOND EDITION 5


Trace the path of a drop of blood from left ventricle to the hallux
Ascending aorta → aortic arch → descending aorta → thoracic aorta → abdominal aorta →
common iliac artery → external iliac artery → femoral artery → deep femoral artery →
popliteal artery → anterior tibial artery → dorsalis pedis → 1st dorsal metatarsal artery →
1st dorsal common digital artery → 1st dorsal proper digital artery

What are the sources of blood supply to the talus?


Essentially the 3 major blood supplies to the foot
 Superior surface of head and neck – artery of sinus tarsi and branch from anterior tibial
artery or dorsalis pedis
 Medial side of body – artery of tarsal canal and posterior tibial artery
 Lateral turbercle – anastamosis of branch of peroneal artery with medial calcaneal branch

What are the sources of blood supply to tendons?


Myotendinous junction, paratenon, and at the insertion to bone

6 CROZER-KEYSTONE RESIDENCY MANUAL – SECOND EDITION


Antibiotics
What should you always consider before starting an antibiotic?
 What is the most likely infecting organism?
 Have a gram stain and C&S been done? What are the results?
 Allergies?
 Kidney function (check BUN and Cr)? Many antibiotics are renally metabolized so it is
imperative to make sure the kidneys are functioning properly.
 What medications is the patient currently taking? Be concerned of possible drug
interactions.
 Any other reason you may or may not want to give the antibiotic?

Name That Drug


Augmentin amoxacillin/clavulonic acid
Zosyn piperacillin/tazobactam
Unasyn ampicillin/sulbactam
Timentin ticarcillin/clavulonic acid
Zyvox linezolid
Invanz ertapenem
Cubicin daptomycin
Tygacil tigecycline
Bactrim trimethoprim/sulfamethoxazole (TMP/SMX)
Rocephin ceftriaxone
Avelox moxifloxacin
Zithromax azithromycin
Primaxim imipenem/cilastatin
Synercid dalfopristin-quinupristin
Cleocin clindamycin
Flagyl metronidazole

Augmentin
What is the dose?
500 or 875 mg PO BID

How much clavulonic acid is in Augmentin 500 mg? Augmentin 875 mg?
Both have 125 mg

What is the indication?


PO antibiotic for outpatient therapy of polymicrobial infections

What is the spectrum of activity?


Staph (not MRSA), Strep, Enterococci, Gram negatives, anaerobes

CROZER-KEYSTONE RESIDENCY MANUAL – SECOND EDITION 7


Does it cover Pseudomonas?
No

Zosyn
What is the dose?
3.375 g IV q6h
Renal dose – 2.25 g IV q6h
Alternate dose – 4.5 g IV q6h

What is the indication?


Approved for use in adults for the treatment of moderate to severe diabetic foot infections

What is the spectrum of coverage?


Staph (not MRSA), Strep, Enterococci, Gram negatives, anaerobes

Does it cover Pseudomonas?


Yes

Unasyn
What is the dose?
3.0 IV q6h
Renal dose – 1.5 g IV q6h

What is the indication?


Empiric therapy for polymicrobial diabetic foot infections

What is the spectrum of activity?


Staph (not MRSA), Strep, Enterococci, Gram negatives, anaerobes

Does it cover Pseudomonas?


No

What is an alternative for a patient with a PCN allergy?


Clinda/Cipro
Levaquin
(there are others)

Timentin
What is the dose?
3.1 g IV q4-6h

What is the indication?


Broad spectrum antibiotic for polymicrobial infections

8 CROZER-KEYSTONE RESIDENCY MANUAL – SECOND EDITION


What is the spectrum of activity?
Staph (not MRSA), Strep, Gram negatives, anaerobes

Does it cover Pseudomonas?


Yes

What should you watch for?


Increased Na+ load (5.2 meq/gram)

Penicillins
Which cover Pseudomonas?
(4th and 5th generations)
piperacillin, Zosyn
ticarcillin, Timentin
carbenicillin, mezlocillin, azlocillin

What are IV alternatives for PCN allergic patients?


clindamycin, vancomycin, Levaquin, Bactrim

How are PNC’s excreted?


All are renally excreted except mezlocillin, azlocillin, piperacillin (the ureidopenicillins are 20-
30% renal)

What concern is there of a patient on both PCN and probenecid?


Probenecid will increase duration of serum levels of PCN and most cephalosporins

Cephalosporins
What is the cross-reactivity of cephalosporins and PCN?
1-10% (depending on whom you talk to)

Are cephalosporins contraindicated for a patient with a PCN allergy?


Many people will say yes, and according to Dr. Warren Joseph, ―Cephalosporins should be
avoided entirely in patients with a history of anaphylaxis to penicillin.‖ However, he states that
if there is a questionable allergy history (rash or upset stomach), ―Cephalosporins can be used
with little worry.‖ Personally, I will give a cephalosporin to a patient with a PCN allergy if all
he or she had was an upset stomach and I document this.

How to treat serious hospital acquired Gram negative infections?


3rd generation cephalosporins, aminoglycoside (i.e. Rocephin, gentamycin)

What is the coverage of cephalosporins for each class?


1st Generation
Gram positive – Staph (not MRSA) and Strep
Gram negative – Proteus, E. coli, Klebsiella, Salmonella, Shigella (PECKSS)
Anaerobes – not Bacteroides

CROZER-KEYSTONE RESIDENCY MANUAL – SECOND EDITION 9


2nd Generation
Gram positive – similar to 1st gen
Gram negative – more coverage, H. influenza, Neisseria, Proteus, E. coli, Klebsiella,
Salmonella, Shigella (HEN PECKSS)
3rd Generation
Gram positive – less than 1st and 2nd gen
Gram negative – expanded coverage, ceftazadime covers Pseudomonas
4th Generation
Gram positive – similar to 1st gen
Gram negative – similar to 3rd gen, including Pseudomonas
No anaerobic coverage

Name a couple cephalosporins for each generation


1st Generation – cefazolin (Ancef), cephalexin (Keflex)
2nd Generation – cefaclor (Ceclor), cefuroxime (Ceftin)
3rd Generation – ceftriaxone (Rocephin), ceftazidime (Fortaz), cefdinir (Omnicef)
4th Generation – cefepime (Maxipime)

How are they excreted?


Renally except for ceftriaxone (renal/hepatic) and cefoperazone (hepatic)

Vancomycin
What is the main indication?
MRSA

What is its spectrum of activity?


All Gram positives, including MRSA and MRSE

What is the dose?


1 g IV q12h with slow infusion

When are levels drawn?


Peak taken 30 min after the 3rd dose
Trough taken 30 min before the 4th dose

What should the peaks and troughs be?


Peak 15-30 mg/mL
Trough <10 mg/mL

How do you adjust the dose?


If the peak is too high, decrease the dose
If the peak is too low, increase the dose
If the trough is too high, increase the interval between doses
If the trough is too low, decrease the interval between doses

10 CROZER-KEYSTONE RESIDENCY MANUAL – SECOND EDITION


What happens when you infuse too quickly?
Red Man syndrome – erythema and pruritis to the head, neck, and upper torso. It is caused by an
anaphylactoid reaction where histamine is released by mast cells. (A different Red Man
syndrome is associated with excessive Rifampin that causes a bright reddish-orange
pigmentation of the skin.)

How can you decrease the risks of Red Man syndrome?


Slow infusion over one hour

How do you treat Red Man syndrome?


Antihistamines (Benadryl 25-50 mg IV q2-4h) until symptoms resolve
Symptoms are self-limiting

What are other side effects?


Ototoxicity and nephrotoxicity

Does the duration a patient has been on vancomycin increase the risks of side effects?
Yes. Vancomycin has a reservoir effect: the more often a patient receives vancomycin, the
higher the chance of getting either ototoxicity or nephrotoxicity. Therefore, use vancomycin
carefully; it is a powerful drug with severe side effects.

When should PO vancomycin be used?


Treatment of Pseudomembranous colitis (125 mg PO q6h)

Bactrim
What is the dose?
One tab PO BID

How much is in the single strength tablet? Double strength?


Single strength – TMP 80 mg / SMX 400 mg
Double strength (DS) – TMP 160 mg / SMX 800 mg

How does it work?


Trimethoprim and sulfamethoxazole inhibit folate synthesis in bacteria which prevents DNA
replication

What is the spectrum of activity?


Broad spectrum covering Gram positives (MRSA) and Gram negatives

Does it cover Pseudomonas?


No

What allergy should be avoided?


Sulfa

CROZER-KEYSTONE RESIDENCY MANUAL – SECOND EDITION 11


What are the side effects?
Hemolytic anemia, hypersensitivity

What are the contraindications?


Patient on oral hypoglycemic or with G6PD deficiencies

Zithromax
What is the dose?
250 mg PO, two tabs on the first day then one tab for the next four days

What is the spectrum of activity?


Staph, Strep, and some anaerobes (but not bacteroides)

Can you give it to a patient with a PCN allergy?


Yes

What is the half-life?


68 hours

Primaxin
What is the dose?
500 mg IV q6-8h (most common) or 1 gm IV q6-8h

What is the spectrum of activity?


Very broad spectrum including most Gram positive, Gram negative, and most anaerobes

Does it cover MRSA? Pseudomonas?


No and no

What is a side effect?


Seizure in patients with history of seizures
1% risk with 500 mg dose, 10% risk with 1 g dose

How does it work?


imipenem – antibiotic
cilastatin – renal dehydropeptidase inhibitor, which prevents imipenem from being metabolized
by the kidneys

Which antibiotic is nicknamed ―Gorillamycin‖?


imipenem (because of its very broad of spectrum activity)

Invanz
What is the dose?
1 g IV q24h

12 CROZER-KEYSTONE RESIDENCY MANUAL – SECOND EDITION


What is the indication?
Approved for use in adults for the treatment of moderate to severe diabetic foot infections

What is the spectrum of activity?


Gram positive, Gram negative, and anaerobes

Does it cover Pseudomonas?


No

What class is Invanz?


It is a structurally unique 1-β-methyl-carbapenem related to β-lactams

Zyvox
What is the dose?
400-600 mg PO/IV q12h

What is an indication?
Oral Zyvox may be used for outpatient treatment of MRSA infections

What is the spectrum of activity?


All Gram positives, including MRSA and VRE

What is a major side effect?


Thrombocytopenia (check CBC)

Why isn’t it used more often?


It is expensive

Quinolones
What are some common quinolones?
ciprofloxacin (Cipro), levofloxacin (Levaquin), moxifloxacin (Avelox)

What is the dose of Cipro?


250-750 mg PO q12h
200-400 mg IV q12h

What is the dose of Levaquin?


250-500 mg PO/IV q24h

What is the dose of Avelox?


400 mg PO/IV q24h

What is the spectrum of activity?


Gram negative, including Pseudomonas
Cipro – limited Gram positive
Levaquin and Avelox – better Gram positive

CROZER-KEYSTONE RESIDENCY MANUAL – SECOND EDITION 13


What are side effects?
Tendonitis and tendon ruptures

Who should not be given quinolones?


It is contraindicated in children with open growth plates. Risk of cartilage degeneration.

Aztreonam
What is the dose?
1-2 g IV q8h

What is the spectrum of activity?


Gram negative aerobes and pseudomonas (its main indication)

What are the major side effects?


None

Why isn’t it used more often?


It is expensive

Aminoglycosides
What are some major aminoglycosides?
Gentamycin, Tobramycin, Amikacin

What is the spectrum of activity?


Gram negative aerobes

What are the side effects?


Ototoxicity – irreversible
Nephrotoxicity – reversible
Neuromuscular blockade – prevented by slow infusion

What are the doses, peaks, and troughs?


Dose Peak (µg/mL) Trough (µg/mL)
Gent and Tobramycin 3-5 mg/kg q8h 6-10 2
Amikacin 15 mg/kg q8h 20-30 <10

How to dose gentamycin?


1. Loading dose is 2 mg/kg for Gent and Tobra (7.5 mg/kg for Amikacin)
2. Determine creatinine clearance (CC)
CC = (140 - Age) x Weight (in kg)
72 x Serum Creatinine
For females, multiply the CC by 0.85
3. Maintenance dose is adjusted for CC (e.g. If the CC is 0.75, then the patient has 75%
kidney function. Give 75% of a normal dose.)

14 CROZER-KEYSTONE RESIDENCY MANUAL – SECOND EDITION


Clindamycin
What is the dose?
600-900 mg IV q8h or 150-300 mg PO BID

What is the spectrum of activity?


Most Gram positive and most anaerobes

What is a side effect?


Pseudomembranous colitis

How is clindamycin metabolized?


Liver

Flagyl
What is the dose?
500 mg PO TID

What is the spectrum of activity?


Some Gram positive anaerobes and most Gram negative anaerobes

MRSA
What antibiotics cover MRSA?
PO – linezolid, Minocycline, Cipro/rifampin, Bactrim/rifampin
IV – vancomycin, linezolid, minocycline, Cipro/rifampin, Bactrim/rifampin, Synercid,
tigecyclin, telavancin
Topical – Bactroban

What are the only FDA-approved drugs for treating MRSA?


vancomycin
linezolid
daptomycin
tigecyclin
telavancin (Vibativ)

VRE
How do you treat VRE?
linezolid or dalfopristin-quinupristin

What is the only PO therapy for VRE?


linezolid

CROZER-KEYSTONE RESIDENCY MANUAL – SECOND EDITION 15


Pseudomonas
What drugs cover Pseudomonas?
Aztreonam
Aminoglycosides – gentamycin, tobramycin, amikacin
Cipro
Ceftazidime, cefepime
Timentin
Zosyn

Polymicrobial Infections
What are some empiric therapies for polymicrobial foot infections?
Vanco/Zosyn, Clinda/Cipro, Vanco/Invanz

What are the only FDA-approved drugs for treating diabetic foot infections?
(The 3 Z’s)
Zosyn
Zyvox
Invanz

Antibiotic-Associated Diarrhea
What are two main causes of antibiotic-associated diarrhea?
Pseudomembranous colitis – Clostridium difficile
Non-specific colitis – Staph aureus

How to you test for Clostridium difficile?


Order ―check stool for C diff‖

What is the most common cause of Clostridium difficile colitis?


clindamycin (though any antibiotic can cause it)

How do you treat Clostridium difficile colitis?


Vanco 125 mg PO q6h
Flagyl 500 mg PO TID

Miscellaneous
What antibiotics are metabolized by the liver?
(3 C’s and 1 E)
Clindamycin
Cefoperazone
Chloramphenicol
Erythromycin

Can antibiotics affect PT/INR?


Yes. Antibiotics can affect normal flora, which alters Vitamin K. Therefore, the PT/INR can
increase.

16 CROZER-KEYSTONE RESIDENCY MANUAL – SECOND EDITION


What can β-lactams cause?
Leukopenia

What is the MOA of aminoglycosides? Macrolides?


Aminoglycosides bind to bacterial 30s ribosomes inhibiting protein synthesis
Macrolides bind to bacterial 50s ribosomes inhibiting protein synthesis
(A boy at 30 does not become a Man until 50)

What antibiotics can be safely used with PMMA beads?


Vancomycin, gentamycin, tobramycin, cefazolin
The curing of PMMA is exothermic, therefore the antibiotic must be not be heat-labile

What open fractures should be treated with antibiotics?


Grades 2 and 3

CROZER-KEYSTONE RESIDENCY MANUAL – SECOND EDITION 17


18 CROZER-KEYSTONE RESIDENCY MANUAL – SECOND EDITION
Bugs and Drugs
Gram Positives
What are Gram positive, catalase positive cocci in clusters?
Staphylococcus aureus

DOC for Staph?


Keflex or Ancef

Alternative for Staph?


clindamycin, Levaquin, Vancomycin, Azithromycin, dicloxacillin, nafcillin

Alternative for Staph if PCN allergy?


clindamycin, Levaquin, Vancomycin, Azithromycin

What if the organism is resistant to methicillin?


MRSA (methacillin-resistant Staph aureus)

DOC for MRSA?


Vanco IV, Bactrim PO (if sensitive)

Alternative for MRSA?


Synercid or linezolid

Topical DOC for MRSA?


Bactroban

DOC for Strep?


Keflex or Ancef

What are Gram positive, catalase negative cocci that are in pairs or chains?
Streptococcus

DOC for Strep?


Keflex or Ancef

Alternative for Strep?


clindamycin, Levaquin, vancomycin

Alternative for Strep in PCN allergy?


clindamycin, Levaquin, vancomycin

DOC for Enterococcus?


amoxicillin or vancomycin

CROZER-KEYSTONE RESIDENCY MANUAL – SECOND EDITION 19


Alternative for Enterococcus?
Augmentin, linezolid

What if the organism is resistant to vancomycin?


VRE (vancomycin-resistant Enterococcus)

DOC for VRE?


linezolid or Synercid

DOC for Diptheroids?


vancomycin

Gram Negatives
What is a short, Gram negative rod?
Escherichia coli

DOC for E. coli?


Keflex or Ancef

Alternative for E. coli if PCN allergy?


Cipro or Levaquin

DOC for Proteus?


Keflex or ampicillin

Alternatives for Proteus if PCN allergy?


Cipro or Levaquin

DOC for E/C/S/M group?


Quinolone (Cipro or Levaquin)

Alternatives for E/C/S/M group?


3rd generation cephalosporin, Aztreonam, Bactrim

What is a small Gram negative rod with pili and polar flagella?
Pseudomonas aeruginosa

DOC for Pseudomonas?


Cipro

Alternative for Pseudomonas?


3rd gen cephalosporins, Aztreonam, Zosyn, Timentin

How does Pseudomonas typically present?


blue-green purulence with grape-like odor

20 CROZER-KEYSTONE RESIDENCY MANUAL – SECOND EDITION


What Gram negative spirochete causes Lyme disease?
Borrelia burgdorferi

DOC for Lyme disease?


doxycyline or Rocephin

Alternative for Lyme disease?


amoxicillin

Anaerobes
DOC for Bacteroides?
Augmentin, Zosyn, Unasyn, Timentin

Alternatives for Bacteroides if PCN allergy?


clindamycin/Cipro, Primaxin, Flagyl

What is a large, Gram positive, anaerobic, ―racquet-shaped‖ rod that forms spores?
Clostridium perfringens

DOC for Clostridium?


Penicillin, imipenem, clindamycin, tetracycline

What are two soft tissue clinical manifestations caused by Clostridium?


Anaerobic cellulitis and gas gangrene

Why is gas gangrene a surgical emergency?


It rapidly progresses to shock and renal failure and is fatal in 30% of cases

Less Common Organisms


DOC for Aeromonas?
Cipro PO/IV

Alternative for Aeromonas?


Bactrim

DOC for Pseudomonas cepacia?


Bactrim

Alternative for Pseudomonas cepacia?


Ceftazidime

DOC for Necrotizing Fasciitis?


Primaxin

DOC for superficial thrombophlebitis?


Timentin

CROZER-KEYSTONE RESIDENCY MANUAL – SECOND EDITION 21


DOC for Gonorrhea?
Ceftriaxone or PCN if sensitive

DOC of Cutaneous Larva Migrans?


Promethia under occlusion

Miscellaneous
What organisms may form gas in soft tissue?
Gram positive – Clostridium perfringens, Staphylococcus, Streptococcus, Peptostreptococcus
Gram negative – Bacteroides, E. coli, Klebsiella, Serratia

What are some anaerobes?


Gram positive – Actinomyces, Clostridium, Peptostreptococcus
Gram negative – Bacteroides, Fusobacterium

What is the drug of choice (DOC) for a patient with diabetes and a PCN allergy?
clindamycin

DOC for severe limb-threatening infection?


Primaxin

What are most common organisms of bite wounds?


Human – Eikenella corrodens
Cat and dog – Pasteurella multocida

What is Gram negative rod is associated with dog bites?


DF-2

DOC for cat and dog bites?


Augmentin

What are the most common organisms causing cellulitis?


Staph and Strep

Which type of Strep can cause impetigo, cellulitis, and erysipelas?


Group A Strep

What is the difference between cellulitis and erysipelas?


Cellulitis – confined superficial infection
Erysipelas – superficial infection that extends into the lymphatics

What is the most common organism that causes acute hematogenous osteomyelitis?
Staphylococcus aureus (adults), Gram negative rods (elderly)

What is the most common organism that causes osteomyelitis following a puncture wound?
Pseudomonas aeruginosa

22 CROZER-KEYSTONE RESIDENCY MANUAL – SECOND EDITION


What is an anaerobic Gram positive filamentous bacteria?
Actinomyces

What organism may be found following a puncture wound in the ocean?


Vibrio vulnificus

What type of bacteria is gonorrhea?


Gram negative diplococci

What is gonorrhea cultured on?


Chocolate agar

What is the treatment for gonorrhea?


Ceftriaxone

If a patient is currently on an antibiotic, how long should it be stopped before taking a


wound culture?
At least 48 hours (if possible)

CROZER-KEYSTONE RESIDENCY MANUAL – SECOND EDITION 23


24 CROZER-KEYSTONE RESIDENCY MANUAL – SECOND EDITION
Labs
CBC
What is in a CBC?
WBC, hemoglobin, hematocrit, platelets

What are normal lab values for CBC?


Note: normal values vary between labs
WBC 4.8-10.8 k/µL
Hemoglobin ♂ 14.0-18.0 g/dL, ♀ 12-16 g/dL
Hematocrit ♂ 42-52%, ♀ 37-47%
Platelets 145-400 k/µL

With an infection, what is expected to happen to the WBC count after surgery?
Eventually it should go down, but in post-op days 1-2, the WBC may actually increase a bit.
This is may occur because surgery activates the body's reaction to the infection.

What should be done if the patient’s WBC is over 10?


First, decide if the patient has an infection
 If there is an infection, then antibiotics and possible incision and drainage (I&D) should
decrease the WBC count
 If there is not an infection, then the cause must be determined. Is the increase acute or
chronic? Is there another source of infection (other than the foot)? Is the patient on
corticosteroids? Is there a combination of medical conditions causing this?

What to do if platelets are low (under 150-350 k/mL)?


Can transfuse platelets, but this is not commonly done

What are the minimum levels for hemoglobin and hematocrit for elective surgery?
Hemoglobin 10 gm/dL and Hct 30%

What should be done if the Hemoglobin/Hematocrit (H/H) is below 10/30?


If necessary, transfuse 1-2 units of packed red blood cells (PRBC)

What is the condition called?


Anemia

What are causes of microcytic, hypochromic anemia?


Iron deficiency, thalassemias, lead poisoning

What are causes of macrocytic, megaloblastic anemia?


Vitamin B12/folate deficiency

CROZER-KEYSTONE RESIDENCY MANUAL – SECOND EDITION 25


Following a transfusion of PRBC, when will changes in the H/H be seen?
Approximately 3 hours. Therefore, order new labs to be drawn 4 hours after last unit given.

BMP
What is in a BMP?
Sodium, potassium, chloride, carbon dioxide, BUN, creatinine, glucose

What is in a CMP?
BMP with ALP (alkaline phosphatase), ALT (alanine amino transferase, also called SGPT),
AST (aspartate amino transferase, also called SGOT), bilirubin, albumin, total protein, calcium

What are normal values for BMP?


Note: normal values vary between labs
Sodium 135-146 mmol/L
Potassium 3.5-5.1 mmol/L
Chloride 96-106 mmol/L
CO2 24-32 mmol/L
BUN 10-20 mg/dL
Creatinine 0.7-1.3 mg/dL
Glucose 70-110 mg/dL

What do Na+, K+, Cl and CO2 tell you?


These electrolytes indicate nutritional status

What should be done if Na+ is low?


Give NSS or regular salt

What should be done if K+ is too low?


Hypokalemia may cause cardiac arrhythmias, muscle weakness, paresthesias, cramps
Manage hypokalemia
 Give K-Dur (potassium chloride supplement)
 Give potassium-rich foods (i.e. banana)

What should be done if the K+ is too high?


Hyperkalemia may cause cardiac arrhythmias, lethargy, respiratory depression, coma
Order EKG
Manage hyperkalemia
 Calcium gluconate
 Sodium bicarbonate
 Dextrose with insulin
 Kayexalate

What do BUN and creatinine indicate?


Renal function

26 CROZER-KEYSTONE RESIDENCY MANUAL – SECOND EDITION


What should be done if the creatinine is too high?
Consult renal if creat is over 1.5 for a couple of results
Note: creat may be increased after muscle breakdown or loss

Which is a more important indicator – BUN or creatinine?


Creat is more important, because BUN is influenced by hydration state. If the BUN is high but
creat is normal, then the patient is most likely dehydrated and rehydration should correct the
BUN. However, if both BUN and creat are high, then the patient most likely has renal damage.

PT/PTT/INR
What do PT/PTT/INR tell you?
The coagulable state of the patient. If the levels are high, it will take longer for the patient to
develop a clot and stop bleeding. It requires blockage of only one pathway to anticoagulate the
patient.

What are normal values for PT/PTT/INR?


Note: normal values vary between labs
PT 11.7-14.5 sec
INR 0.9-1.1
PTT 23-36 sec

What can cause an elevated PT/INR?


Coumadin
Malnutrition
Alcoholism
Antibiotics
Vitamin K disorders

What does INR stand for? Why was it developed?


International Normalized Ratio
There are different methods to determine PT, and thus each lab has a different normal value for
PT. INR was devised to standardize all the results.

If the patient is on Coumadin for anticoagulation, what should the INR be?
Intense anticoagulation 2-3

What causes the PTT to be high?


Heparin

Which pathway does PTT check?


Intrinsic (―PITT‖)

Which pathway does PT check?


Extrinsic (―PET‖)

CROZER-KEYSTONE RESIDENCY MANUAL – SECOND EDITION 27


28 CROZER-KEYSTONE RESIDENCY MANUAL – SECOND EDITION
Medicine
What are reasons for post-op fever?
Wind (12-24 h)
 Atelectasis (from muscle relaxers)
 Post-op hyperthermia
Water (~24 h)
 UTI
Walk (~48 h)
 DVT
 PE
Wound (~72 h)
 Post-op infection
Wonder drug (anytime)
 Drug fever

What are treatments of post-op fever?


Wind
 Encourage incentive spirometer
 Chest x-ray
Water
 Straight catheter
 Urine analysis (UA) with Gram stain, culture and sensitivity
 Treat with antibiotics if necessary
Walk
 Heparin or Lovenox protocol
 Use SCDs, TEDs, or get patient out of bed
Wound
 X-ray, Gram stain, culture and sensitivity, blood cultures
 Begin antibiotic
Wonder drug
 D/C drug
 Give reversal drug if necessary

When do fever peaks occur?


Between 4-8 pm

What part of the brain regulates the body's temperature?


Hypothalamus

What is malignant hyperthermia?


A side effect of general anesthesia – tachycardia, hypertension, acid-base and electrolyte
abnormalities, muscle rigidity, hyperthermia

CROZER-KEYSTONE RESIDENCY MANUAL – SECOND EDITION 29


What is the treatment for malignant hyperthermia?
Dantrolene (for muscle relaxation) 2.5mg/kg IV x l, then 1 mg/kg IV rapid push q6h until
symptoms subside or until max dose of l0 mg/kg

If a risk of malignant hyperthermia is suspected, what pre-operative test may be


performed?
CPK – elevated in 79% of patients with malignant hyperthermia

Anesthetics
What is the mechanism of action for local anesthetics?
Block Na+ channels and conduction of action potentials along sensory nerves

What is the toxic dose of lidocaine (Xylocaine)?


300 mg plain (4.5 mg/kg)
500 mg with epi (7.0 mg/kg)

What is the toxic dose of bupivacaine (Marcaine)?


175 mg plain (2.5 mg/kg)
225 mg with epi (3.2 mg/kg)

How to convert percentage of solution to mg/mL?


Move decimal point of percentage one place to right
(e.g. 1% solution has 10 mg/mL)

What are the side effects of lidocaine and bupivacaine associated with systemic exposure?
CNS effects – initial excitation (dizziness, blurred vision, tremor, seizures) followed by
depression (respiratory depression, loss of consciousness)
Cardiovascular effects – hypotension, bradycardia, arrhythmias, cardiac arrest

What can be given to help reverse local anesthetic-induced cardiovascular collapse?


Intravenous fat emulsion (Intralipid)

Is there a risk with intra-articular injections of bupivacaine?


Studies have shown chondrocyte death following prolonged exposure to bupivacaine

In what age group should bupivicaine be avoided?


Children <12 year of age

How are amides (lidocaine and bupivacaine) metabolized?


Liver

How are esters (Novocain) metabolized?


Plasma pseudocholinesterase

What is the only local anesthetic with vasoconstriction?


Cocaine

30 CROZER-KEYSTONE RESIDENCY MANUAL – SECOND EDITION


How is cocaine metabolized?
Plasma pseudocholinesterase (just like other esters)

Can local anesthetics cross the placental barrier?


Yes

What does MAC (as in MAC with local) stand for?


Monitored anesthesia care

For anesthesia, what cannot be given to a patient with an egg shell injury?
propofol (Diprivan)

Pain Medications
Pain management with a codeine allergy?
(STUD or STTUUDDD-N)
S – Stadol
T – Toradol
T – Talwin
U – Ultram
D – Darvon
D – Darvocet
D – Demerol
N – Nubain

First choice for oral?


Darvocet N-100 one tab PO q4-6h prn pain

First choice for non-narcotic oral?


tramadol (Ultram) 50 mg one to two tabs PO q4-6h prn pain, max daily dose of 400 mg per day

First choice for non-narcotic IV?


Toradol 30-60 mg IV

Choice narcotic IV pain med?


Demerol
Note: many hospitals, including our own, do not use Demerol due to its side effects

Name two non-narcotic analgesics


ketoralac (Toradol), tramadol (Ultram)

CROZER-KEYSTONE RESIDENCY MANUAL – SECOND EDITION 31


Drugs and Usual Doses
What schedule are these drugs?
Percocet II high potential for abuse – requires narcotic script
Vicodin III moderate potential for abuse
Tylenol #3 III moderate potential for abuse
Darvocet IV low potential for abuse

Percocet 5/325?
oxycodone/acetaminophen (5 mg/325 mg)
1-2 tabs PO q4-6h prn pain

Roxicet?
oxycodone/acetaminophen (5 mg/325 mg/5 mL)
Essentially a liquid form of Percocet that is good for pediatric patients

What is the difference between Percocet and Percodan?


Percocet has 325 mg of acetaminophen and Percodan has 325 mg of ASA

Vicodin 5/500?
hydrocodone/acetaminophen (5 mg/500 mg)
1-2 tabs PO q4-6h prn pain

Tylenol #3?
codeine/acetaminophen (30 mg/300 mg)
1-2 tabs PO q4-6h

Darvocet-N 100?
propoxyphene/acetaminophen (100 mg/650 mg)
1 tab PO q4h prn pain

Ultram?
tramadol 50 mg
1-2 tabs PO q4-6° prn pain

Toradol?
ketorolac 10 mg
30 mg IV q6h
1 tab PO q4-6h prn pain
An NSAID not to be used more than 5 days due to possible significant side effects

Darvon?
propoxyphene
1 tab PO q4h prn pain

OxyContin?
oxycodone extended release

32 CROZER-KEYSTONE RESIDENCY MANUAL – SECOND EDITION


Morphine sulphate?
2-4 mg IV q2-6h prn mod-severe pain
For very painful dressing change or bedside debridement – 2 mg IV x one dose

MS Contin?
morphine sulfate extended release (15-30 mg)
1 tab PO q8-12h prn pain

Dilaudid?
hydromorphone
2-8 mg PO q3-4h prn severe pain
1-4 mg IV q4-6h prn severe pain
This drug is very strong

Demerol?
meperidine
Our hospitals do not use this due to its side effects

Acetaminophen
What therapeutic effects are seen with acetaminophen?
Analgesic and anti-pyretic

What is the maximum daily dose?


4g

NSAIDs
What therapeutic effects are seen with most NSAIDs?
Analgesic, anti-pyretic, and anti-inflammatory

What pathway do NSAIDs work on?


Cyclooxygenase (COX)
NSAIDs nonselectively inhibit both COX-1 and COX-2 pathways

What is the most common side effect of NSAIDs?


GI disturbance (except with COX-2 inhibitors, because COX-1 protects the stomach lining)

What is the only FDA-approved COX-2 inhibitor?


celecoxib (Celebrex)
Others were withdrawn due to increased risk of heart attack and stroke

Which NSAIDs only have anti-inflammatory effects?


indomethacin, tolmetin

Do NSAIDs decrease joint destruction?


No, they only decrease inflammation

CROZER-KEYSTONE RESIDENCY MANUAL – SECOND EDITION 33


Do NSAIDs affect bone healing?
NSAIDs and COX-2 inhibitors may inhibit bone healing via their anti-inflammatory effects

What NSAID causes irreversible inhibition of platelet aggregation?


aspirin

What NSAID does not inhibit platelet aggregation?


The COX-2 inhibitor, Celebrex

What is the only IV NSAID?


ketorolac (Toradol)

Which NSAID is often given during surgery or immediately post-op to decrease pain and
inflammation?
Toradol 30 mg IV

What are the NSAIDs with the least nephrotoxicity?


Celebrex, Relafen, Lodine

What is the effect of NSAIDs on asthma?


Can increase symptoms of asthma

What are the safest NSAIDs for a patient with asthma?


Diclofenac, ketoprofen

Which NSAIDs treat collagen vascular disease?


Ibuprofen, sulindac, tolmetin

Which NSAIDs are not renally cleared?


Indomethacin, sulindac

What are the cardiovascular effects of NSAIDs?


Can cause vasoconstriction and increase blood pressure

Which NSAIDs have the least cardiovascular effects?


Diclofenac, ketoprofen

Which NSAIDs are the most hepatotoxic?


Ibuprofen, naproxen, diclofenac

What should be given for an indomethacin overdose?


Benadryl – decreases serotonin and histamine release

What is Arthrotec?
diclofenac/misoprostol – an NSAID with protection for the stomach

34 CROZER-KEYSTONE RESIDENCY MANUAL – SECOND EDITION


What is the anti-inflammatory dose of ibuprofen?
1200-3200 mg/day in divided doses

What NSAIDS work on both the lipooxygenase and cyclooxygenase pathways?


Ketoprofen and diclofenac

What is the difference between Cataflam and Voltaren?


Cataflam is diclofenac potassium and has an immediate release
Voltaren is diclofenac sodium and has a delayed release

What are the only pro-drugs for NSAIDs?


nabumetone and sulindac

What is the only nonacidic NSAID?


nabumetone

Which NSAIDs have fewer pulmonary problems?


ketoprofen and diclofenac

What are some once a day NSAIDs?


celecoxib (Celebrex), piroxicam (Feldene), oxaprozin (Daypro), nabumetone (Relafen), others

What drugs do NSAIDs interact with and what are the effects?
Coumadin – increases action of Coumadin
Sulfonylureas – increases action of sulfonylureas
Corticosteroids – increases GI risk
Anti-epileptics – increases anti-epileptic toxicity
Antihypertensives – antagonizes antihypertensive meds
Digoxin – increases digoxin’s effect
Methotrexate – decreases methotrexate’s clearance
Lithium – decreases lithium’s clearance
Probenecid – increases concentration of NSADs

Anticoagulation
What are causes of acute arterial occlusion?
Embolism – detached thrombus, air, fat, or tumor
Thrombus – occlusion of vessel by plaque or thickened wall
Extrinsic occlusion – traumatic, blunt, penetrating

What is the triad of pulmonary embolism?


Dyspnea
Chest pain
Hemoptysis (although tachycardia is more common)

CROZER-KEYSTONE RESIDENCY MANUAL – SECOND EDITION 35


What tests can be ordered to diagnose a PE?
Chest X-ray
Ventilation perfusion scan
Pulmonary angiography

What is Virchows triad?


Venous stasis – tourniquet, immobilization
Endothelial wall damage/abnormality – surgical manipulation, trauma, smoking
Hypercoagulability – birth control, coagulopathy, history of DVT

What does the Virchow triad predict?


Risk of DVT
Previous DVT is #1 risk factor for having another DVT

What are risks factors for DVT?


(I AM CLOTTED)
I – immobilization
A – arrhythmia
M – MI (past history)
C – coagulable states
L – longevity (old age)
O – obesity
T – tumor
T – trauma
T – tobacco
E – estrogen
D – DVT (past history)

How is a DVT diagnosed clinically?


Pain, heat, swelling, erythema of unilateral limb
Positive Pratt sign – squeezing of posterior calf causes pain
Positive Homan sign – abrupt dorsiflexion of foot causes calf pain
Pulmonary embolism

What tests can be ordered to diagnose a DVT?


Doppler ultrasound
Venogram
D-Dimer

For long term DVT prophylaxis, what drugs can be ordered? Why?
Heparin – works right away
Coumadin – takes 3-5 days and causes an initial transient hypercoagulable state

What are treatments for a DVT?


Thrombolytic agents
Heparin 5000 Units IV bolus, then 1000 Units IV q1h and monitor PTT

36 CROZER-KEYSTONE RESIDENCY MANUAL – SECOND EDITION


How to dose Heparin for perioperative DVT prophylaxis?
5000 units SC 2h prior to surgery
5000 units SC q12h until patient ambulates

What is the half-life of heparin?


1.5 hour

How does heparin work?


Intrinsic pathway
Potentiates antithrombin III 100-fold, which inhibits the serine protease in the clotting cascade

How is heparin reversed?


Protamine sulfate 1 mg per 100 units of heparin

What is enoxaparin (Lovenox)?


Low molecular weight heparin

How to dose Lovenox for perioperative DVT prophylaxis?


30 mg SC q12h for 7-10 days (adjust dose to q24h for renal patients)

What is the half-life of Lovenox?


4.5 hours

What are the advantages of using Lovenox vs. regular Heparin? Disadvantages?
Advantages – Lovenox has longer plasma half-life with significant anticoagulation in trough
Disadvantages – increased post-op complications when used with spinal/epidural anesthesia

How do you check Lovenox?


There is no test for the effects of Lovenox

How is Lovenox reversed?


Recombinant Factor VII

How to dose Coumadin?


5-10 mg PO daily for 3-4 days then adjust for INR

What is the half-life of Coumadin?


20-60 hours

How long before Coumadin is therapeutic?


3-5 days

How does Coumadin work?


Extrinsic pathway
Interferes with clotting factors II, VII, IX, X

CROZER-KEYSTONE RESIDENCY MANUAL – SECOND EDITION 37


How is Coumadin reversed?
Vitamin K
Fresh frozen plasma

What are the INR values?


Normal l
Intense anticoagulation 2-3

What are levels of heparin and Coumadin for DVT/anticoagulation prophylaxis?


Heparin – maintain 2-3 times normal PTT
Coumadin – maintain 2 times normal INR

What nonpharmacologic measures are used for perioperative DVT prophylaxis?


Early ambulation – most important
TEDs – thromboembolic deterrent stockings
SCDs – sequential compression devices

What is a surgical treatment for a patient with prior DVTs or recurrent PEs?
Greenfield filter

What level of the body is a Greenfield filter inserted?


Inferior vena cava below the renal veins

What is Pletal?
cilostazol

What is Trental?
pentoxifylline

What is an indication for Pletal or Trental?


Intermittent claudication

CRPS
What is CRPS?
Complex regional pain syndrome (previously known as RSD – reflex sympathetic dystrophy) is
a progressive disease of the autonomic nervous system causing constant, extreme pain that is out
of proportion to the original injury

What are the different types and causes of CRPS?


CRPS Type I (reflex sympathetic dystrophy)
 Nerve injury cannot be immediately identified
 Spontaneous pain not limited to single nerve distribution
 Abnormal response in sympathetic nervous system
 Abnormal reflex leading to vasomotor instability and pain

38 CROZER-KEYSTONE RESIDENCY MANUAL – SECOND EDITION


CRPS Type II (causalgia)
 Distinct, "major" nerve injury has occurred
o Trauma
o Peripheral nerve injury
o Drugs – anti-TB, barbiturates, cyclosporine
 Continued pain not necessarily limited to injured nerve distribution

What are the stages of CRPS?


1. Acute – early (0 to 8-20 weeks)
 Constant pain out of proportion (intense burning)
 Possible edema, muscle wasting
 Hyperhidrosis
 Pain increased by light touch, movement, emotion
2. Dystrophic – mid (2-6 months, possibly up to 1 year)
 Increased edema that is indurated (brawny edema)
 Constant pain by any stimulus
 Skin is cool pale and discolored
 X-ray shows diffuse osteoporosis
3. Atrophic – late (over 6-12 months)
 Intractable pain spreads proximally to involve entire limb
 Decreased dermal blood flow causing cool, thin shiny skin
 Fat pat atrophy
 Joint stiffen, may proceed to ankylosis

What are radiographic findings of CRPS?


Periarticular, mottled, irregular bony demineralization (30-60% of cases) and cortical thinning

What are bone scan findings of CRPS?


The 3-phase bone scan has sensitivity of 96% and specificity of 98%. A normal scan does not
exclude the diagnosis. The findings of the bone scan are based on the phase.
1. Acute
 Increased flow and blood pool activity in the affected extremity
 Increased activity particularly in a periarticular distribution on delayed images
2. Dystrophic
 Flow and blood pool abnormalities begin to normalize
 Increased activity on delayed images persists
3. Atrophic
 Flow and blood pool activity can be normal or decreased (in about 1/3 of patients)
 Normal or decreased activity is commonly seen on delayed images, however,
persistent increased delayed activity has been reported (up to 40%)
 Decreased flow in advanced stages may be related to disuse, which is a common
feature of post-hemiplegic CRPS

CROZER-KEYSTONE RESIDENCY MANUAL – SECOND EDITION 39


What are treatments of CRPS?
Anti-inflammatory drugs
Antidepressant drugs
Local peripheral nerve blocks
Paravertebral sympathetic ganglion blocks
Physical therapy

Diabetes
For diabetic patients, who gets diabetic ketoacidosis and who gets diabetic coma?
Type I (IDDM) – DKA
Type II (NIDDM) – coma

What are signs of hypoglycemia?


Nervousness, tachycardia, diaphoresis, nausea, headache, confusion, tremor, seizures, coma

What are signs of hyperglycemia?


Polyuria, polydipsia, weight loss

What is the function of a biguanide?


Antihyperglycemic (not hypoglycemic)

What is a typical supplemental insulin scale?


BG (mg/dL) Low Medium
<120 0 0
121-150 1 2
151-180 2 4
181-210 3 6
211-240 4 8
241-270 6 10
271-300 8 12
301-350 10 14
351-400 12 16
>400 call physician

What are the only FDA-approved drugs for treating diabetic neuropathy?
duloxetine (Cymbalta)
pregabalin (Lyrica)

Osteoarthritis
What are clinical findings of OA?
 Pain relieved with rest
 Stiffness aggravated with activity
 Crepitus with motion
 Asymmetric joint swelling

40 CROZER-KEYSTONE RESIDENCY MANUAL – SECOND EDITION


What are radiographic findings of OA?
 Asymmetric joint space narrowing
 Broadening and flattening of articular surfaces
 Osteophytes at joint margins
 Subchondral sclerosis

Gout
What is the most common inflammatory arthritis in men over 30?
Gout

What are the stages of Gout?


1. Asymptomatic hyperuricemia
2. Acute gouty arthritis
3. Intercritical gout
4. Chronic tophaceous gout

What are clinical findings of gout?


 Asymmetrical, monoarticular arthritis
 Sudden onset of red, hot, and swollen joint
 Excruciating pain with acute attack
 Tophaceous deposits
 Most commonly affects 1st MPJ

What are radiographic findings of gout?


 Radiographic findings appear late in the disease after multiple attacks
 Bone lysis in acute stages
 Periarticular swelling with preserved joint space
 Tophi at joint margins
 Rat bite – punched-out, periarticular erosions
 Cloud sign – tophaceous material
 Martel sign – periarticular overhanging shelves of bone

What are laboratory tests for gout?


 Uric acid – males >7 mg/dL, females >6 mg/dL, though may be normal during attack
 Synovial fluid analysis provides a more accurate diagnosis

What would a joint aspirate of gout show?


Needle-shaped monosodium urate crystals that are negatively birefringent under polarized light
(CPPD are rhomboid-shaped and positively birefringent)

What is a martini sign?


Histology showing a PMNC engulfing a crystal

If gout is suspected, what should a specimen be sent in?


One in formaldehyde (dissolves gouty tophi) and one in alcohol (does not dissolve gouty tophi)

CROZER-KEYSTONE RESIDENCY MANUAL – SECOND EDITION 41


How to treat acute and chronic gout?
Acute
 Colchicine
 NSAIDS – indomethacin
 Corticosteroids
 ACTH
Chronic
 Colchicine (prophylactically)
 Allopurinol
 Uricosurics – probenecid, sulfinpyrazone

What is the dose colchicine?


0.6 mg PO q1h until symptoms resolve, GI side effects occur, or max dose of 6 mg reached

What is the max daily dose of colchicine?


6 mg

Can allopurinol, probenecid or sulfinpyrazone be used for acute gout?


No, because they may cause an initial hyperuremia

How to determine if patient is an overproducer or underexcretor?


Take a 24 hour uninalysis

Which is more common – to be an overproducer or an underexcretor?


Underexcretors make up approx 90%

What medication should be given if the patient is an overproducer? Underexcretor?


(Over-Achieving, Under-Paid)
Overproducer → Allopurinol
Underexcretor → Probenecid

Rheumatoid Arthritis
What are clinical findings of RA?
 Symmetric, progressive, polyarticular, and degenerative inflammatory arthritis
 Age of onset between 3-4th decades
 Females > males
 Pain first thing in morning
 Stiffness after rest and reduced with activity
 Rheumatoid nodules (25%)
 Nail fold infarcts, splinter hemorrhages
 Swan neck deformities – flexed DIPJ and extended PIPJ
 Boutonniere deformities – extended DIPJ and flexed PIPJ
 Other – bullous dermatosis, Raynaud phenomenon, vasculitis

42 CROZER-KEYSTONE RESIDENCY MANUAL – SECOND EDITION


What are laboratory findings of RA?
 Rheumatoid factor – positive
 RBC – slight to moderate anemia
 WBC – elevated in acute cases and normal to decreased in chronic
 ESR & CRP – moderate to marked elevation
 Synovial fluid analysis – elevated WBCs with cloudy fluid

What are radiographic findings of RA?


 Clinical symptoms may present several years prior to radiographic findings
 Peri-articular edema
 Periosteal elevation and ossification
 Marginal erosions
 Subluxation and contractures (Swan neck deformities)
 Fibular deviation of digits
 Osteoporosis
 Symmetric joint space narrowing and destruction (late stage finding)

What causes the fibular deviation of digits?


Erosive changes of medial plantar metatarsal heads compromises the integrity of medial
collateral ligaments leading to lateral deviation of digits

What is pannus?
Granulation tissue that secretes chondrolytic enzymes which break down articular cartilage

Psoriatic Arthritis
What are clinical findings of PA?
 Polyarthritis including DIPJ involvement
 Sausage digits
 Psoriatic skin changes
 Nail lesions

What are laboratory findings for PA?


 HLA-27 – positive
 Rheumatoid factor – negative

What are radiographic findings of PA?


 Erosions with bony proliferation
 Symmetric narrowing of joint space
 Increased periosteal activity
 Pencil-in-cup appearance
 Osteopenic changes

CROZER-KEYSTONE RESIDENCY MANUAL – SECOND EDITION 43


Reiter Syndrome
What are clinical findings of RS?
 Polyarticular, asymmetric arthritis of lower extremity (mostly affects small bones of feet,
ankle, knee, SI joint)
 Most affects males
 Capsulitis with digital edema
 Bony erosions
 Reiter Syndrome Triad (can't see, can't pee, can't climb a tree)
o Conjunctivitis
o Urethritis
o Arthritis
o Also keratoderma blenorrhagicum

What are laboratory findings for RS?


 HLA-27 – positive
 Rheumatoid factor – negative
 ESR – elevated
 Synovial fluid analysis – Pekin cells

What are radiographic findings of RS?


 Fluffy periosteal reactions
 Large, bilateral heel spur formation
 Inflammation and widening of Achilles tendon insertion
 Deossifications

Ankylosing Spondylitis
What are clinical findings of AS?
 Mostly males affected
 Bilateral sacroiliitis – low back pain and stiffness
 Heel pain
 Peripheral joint pain

What are laboratory findings for AS?


 HLA-27 – positive
 Rheumatoid factor – negative

What are radiographic findings of AS?


 Irregular joint widening with erosions
 Reactive sclerosis
 Bony ankylosis
 Sacroiliac joint fusion
 Bamboo spine

44 CROZER-KEYSTONE RESIDENCY MANUAL – SECOND EDITION


Septic Arthritis
What are clinical findings of SA?
 Painful, hot, swollen joint
 Systemic signs of fever, N/V, tachycardia, confusion

What are laboratory findings for SA?


 WBC – elevated with left shift
 ESR – elevated
 CRP – elevated
 Blood cultures – positive
 Synovial fluid analysis – elevated WBC with cloudy white or gray color

What are radiographic findings of SA?


 Normal in early stages
 Joint effusion
 Juxta-articular osteopenia

What are etiologies of SA?


Contiguous, hematogenous, direct implantation, surgical contamination

What is the most common offending organism of SA?


All ages – Staphylococcus aureus
Neonates – Streptococcus and Gram negatives
Children – H. influenza
Teenagers – Neisseria gonorrhea
Puncture wounds – Pseudomonas aeruginosa
Adults with sickle-cell – Salmonella

What is the treatment for SA?


Needle drainage of joint
Open arthrotomy if osteomyelitis, joint implant, or chronic infection
Initial joint immobilization followed by passive ROM
Appropriate IV antibiotics for 2 weeks followed by 2-4 weeks of oral antibiotics

Other Diseases
What is brachymetatarsia?
Premature closure of epiphyseal plate of metatarsal resulting in a short metatarsal
Usually the 4th metatarsal is affected

What are some conditions associated with brachymetatarsia?


Downs syndrome
Turners syndrome
Cri du chat
Pseudo- or pseudopseudohypoparathyroidism
May be idiopathic

CROZER-KEYSTONE RESIDENCY MANUAL – SECOND EDITION 45


What is the maximum length that a metatarsal may be acutely lengthened for correction of
brachymetatarsia?
1 cm graft allows acceptable stretching of neurovascular structures

If more than 1 cm of lengthening is required, what procedure may be performed?


Callus distraction with Mini-Rail fixation

How much lengthening is typically achieved with callus distraction?


1 mm per day (0.25 mm q6h)

What is achondroplasia?
Dwarfism – all bones short with tibia undergrowth and fibular overgrowth causing genu varum

What is fibular hemimelia?


Aplasia or hypoplasia of the fibula

What is DISH?
Diffuse Ideopathic Skeletal Hyperostosis – characterized by multiple ossifications at tendinous
or ligamentous insertions

What is Apert syndrome?


Multiple bony coalitions

What is Paget disease?


Osteitis deformans – abnormal bony architecture caused by increased osteoblastic and
osteoclastic activity. More common in elderly.

What malignant bone degeneration may be seen with Paget disease?


Osteosarcoma

What are the stages of Paget?


1. Destructive – osteolytic
2. Mixed – osteolytic and osteoblastic
3. Sclerotic – osteoblastic

What are the stages of Charcot?


1. Acute or destructive
2. Coalescence
3. Remodeling

What conditions are associated with positive HLA-B27?


Ankylosing spondylitis, Reiter disease, psoriatic arthritis, reactive arthritis, enteropathic
arthropathies

46 CROZER-KEYSTONE RESIDENCY MANUAL – SECOND EDITION


What are components of CREST syndrome?
Calcinosis
Raynauds phenomenon
Esophageal dysmotility
Sclerodactyly
Telangiectasias

What is the treatment of cutaneous larva migrans?


Promethia under occlusion

What is the treatment for Lyme disease?


Doxycycline 100 mg PO daily or Rocephin l g IV daily

DOC for necrotizing fasciitis?


Primaxin 250-1000 IV q6-8h (most commonly 500 mg IV q8h)

What is Felty syndrome?


Rheumatoid arthritis, splenomegaly, leukopenia

What is mycosis fungoides?


Cutaneous T-cell lymphoma that can resemble eczematoid or psoriasis

What is erythrasma?
Chronic, superficial infection of intertriginous skin caused by Corynebacterium minutissimum.
Interdigital lesions appear as maceration.

What is ecthyma?
Ulcerative pyoderma of the skin often caused by Streptococci. Infection extends into dermis and
is characterized by ulcers with overlying crusts.

What is cellulitis?
Acute spreading infection of dermal and subcutaneous tissue commonly caused by group A Strep
or Staph aureus. Affected area is erythematous, warm, edematous, and tender.

What is erysipelas?
Superficial infection that extends into the lymphatics. Lesions are erythematous, indurated with
sharply-demarcated margins, and have erythematous, ascending streaks.

What is lymphangitis?
Inflammation of the lymphatics as a result of a distal infection

What is psoriasis?
Hereditary disorder with chronic scaling papules and plaques in areas of body related to repeated
minor trauma. Positive Koebner phenomenon and Auspitz sign. Also present are joint pain and
nail changes including pitting, beau lines, oil spot, subungual hyperkeratosis, and discoloration.

CROZER-KEYSTONE RESIDENCY MANUAL – SECOND EDITION 47


What is lichen planus?
Inflammatory dermatosis involving skin or mucous membranes with pruritic, violaceous papules
clustered into large, flat-topped lesions with distinct borders. Lesions possibly covered with
Wickham striae (white streaks). Ridges, onycholysis, subungual hyperkeratosis, and
discoloration.

What is another name for menopausal lipoma?


Juxtamalleolar lipoma

What is the main screening test if AIDS is suspected?


ELISA (Enzyme Linked Immunosorbent Assay)

What test should be performed to confirm the diagnosis of AIDS?


Western blot

Miscellaneous Drugs
What are some effects of steroids?
Anti-inflammatory
 Decreases production of prostaglandins, cytokines, and interleukins
 Decreases proliferation and migration of lymphocytes and macrophages
Metabolic
 Decreases osteoblast activity

What are differences between phosphate and acetate-based steroids?


Phosphate-based – soluble with shorter half-life
 Minimize inflammatory reaction and edema
Acetate-based – insoluble with longer half-life
 May delay inflammatory process or healing and can mask infection

What is a common complication following steroid injection?


Steroid flare – hypersensitivity reaction. Apply ice.

How are glucocorticoids metabolized?


Metabolized in the liver and secreted in urine

What is diazepam?
Valium, a benzodiazepine, is an anxiolytic/anticonvulsant/muscle relaxant

How to reverse diazepam?


Flumazenil (Romazicon) for benzodiazepine reversal
0.2 mg IV over 15 seconds, then 0.2 mg IV prn over 1 minute up to 1 gram total

48 CROZER-KEYSTONE RESIDENCY MANUAL – SECOND EDITION


What are drugs for insomnia?
(BE HARD)
B – Benadryl
E – estazolam
H – Halcion
A – Ambien
R – Restoril
D – Dalmane
Most commonly used are Benadryl 25 mg PO qhs or Ambien 5 mg PO qhs

What drugs leave a metallic taste in the mouth?


Flagyl, Lamisil

What is given for a Tylenol overdose?


acetylcysteine (Mucomyst)

What can cause Gray Baby Syndrome?


Chloramphenicol

What is chloramphenicol?
An antimicrobial

CROZER-KEYSTONE RESIDENCY MANUAL – SECOND EDITION 49


50 CROZER-KEYSTONE RESIDENCY MANUAL – SECOND EDITION
Clinical Podiatry
What are the clinical patterns of tinea pedis? What are common infecting organisms?
Chronic (moccasin or papulosquamous)
 Trichophyton rubrum
Acute (interdigital or vesicular)
 Trichophyton mentagrophytes
Ulcerative
 Trichophyton mentagrophytes with Pseudomonas or Proteus

What are the clinical patterns of onychomycosis? What are common infecting organisms?
Distal subungual onychomycosis (DSO) ~ 90%
 Most common
 Trichophyton rubrum
Proximal subungual onychomycosis (PSO) ~ 1%
 Seen in immunocompromised patients
 Trichophyton rubrum
Superficial white onychomycosis (SWO) ~ 10%
 Trichophyton mentagrophytes
Candidal onychomycosis
 Candida albicans

What test confirms tinea pedis or onychomycosis?


Potassium hydroxide (KOH) preparation of skin or nail specimen
Septate hyphae confirms diagnosis

Who does Lamisil work?


Inhibits ergosterol synthesis

What is phenol?
Carbolic acid

During a P&A procedure, why is alcohol used after phenol?


Phenol is soluble in alcohol, and the alcohol will irrigate excess phenol from the nail groove

For a nail avulsion, what can be done for anesthesia if the patient is allergic to all local
anesthetics?
Saline block (pressure induced block)
Pressure cuff
Benadryl block (blocks histamine release)

CROZER-KEYSTONE RESIDENCY MANUAL – SECOND EDITION 51


In evaluating a bunion, what does the position of the tibial sesamoid indicate? Why isn’t
the fibular sesamoid evaluated?
The tibial sesamoid indicates the abnormal affects of the adductor and flexor brevis tendons.
Once the fibular sesamoid reaches the intermetatarsal space, it travels in the frontal plane (as
opposed to transverse), therefore the tibial sesamoid is a more reliable indicator of deformity.

What are some causes of hallux varus?


Congenital
 Clubfoot
 Metatarsus adductus
Traumatic
 MPJ dislocation
 Fracture
Iatrogenic
 Overcorrection of intermetatarsal angle
 Excessive resection of medial eminence or staking the head
 Fibular sesamoidectomy
 Overaggressive capsulorrhaphy
 Bandaging too far into varus

What is staking the head?


Excessive resection of the 1st metatarsal head with cutting into the sagittal groove may lead to
hallux varus

Describe the types of hammertoes


Flexor stabilization
 Most common
 Stance phase
 Flexors overpower interossei
 Pronated foot
Extensor substitution
 Swing phase
 Extensors overpower lumbricals
 Anterior cavus, ankle equinus, anterior compartment muscle weakness
Flexor substitution
 Least common
 Stance phase
 Deep compartment muscles overpower interossei
 Supinated, high arch foot or weakened Achilles

What is the result of accidentally severing the quadratus plantae?


Adductovarus deformity of digits 4 and 5 as the pull of FDL is unopposed

52 CROZER-KEYSTONE RESIDENCY MANUAL – SECOND EDITION


What are differences between flexible, semi-rigid, and rigid deformities?
Flexible – reducible when NWB and WB
Semi-rigid – reducible when NWB only
Rigid – non-reducible

What is a Haglund deformity?


Pump bump

What x-ray measurements evaluate a Haglund deformity?


Parallel pitch lines
Fowler & Philip
Total angle

What is the Silfverskiöld test?


Determines gastroc vs. gastroc-soleus
Positive test
 Dorsiflexion of the foot to neutral or beyond with the knee in flexion
 Gastroc equinus
Negative test
 Lack of dorsiflexion of the foot to neutral with knee in flexion and in extension
 Gastroc-soleus equinus

What is the Lachman test?


Determines if there is a plantar plate tear or rupture. While stabilizing the metatarsal, a dorsal
translocation of the proximal phalanx greater than 2 mm is suggestive of rupture.

What is the Mulder sign?


Identifies a Morton neuroma by a palpable click when compressing metatarsal heads and
palpating the interspace

What is the Sullivan sign?


Separation of digits caused by a mass within the interspace

What is Q angle?
Angle between the axis of the femur and the line between the patella and tibial tuberosity

What to do if patient has edema with a cast?


If edema goes down in AM → gravity edema → normal
If edema does not go down in AM → abnormal

What is Raynaud phenomenon?


Recurrent vasospasm of digits usually in response to stress or cold

CROZER-KEYSTONE RESIDENCY MANUAL – SECOND EDITION 53


What are the stages of Raynauds Phenomenon?
White → blue → red
 Pallor – spasm of digital arteries
 Cyanosis – deoxygenation of blood pools
 Rubor – hyperemia

What is an ABI?
Ankle Brachial Index – compares ankle to arm pressures
Normal 1
Intermittent claudication 0.6-0.8
Rest pain 0.4-0.6
Ischemic ulcerations <0.4

What may falsely elevate the ABI?


Vessel calcifications/non-compressible vessels

What other tests are typically performed with an ABI?


Segment pressures
 Measured at high thigh, above the knee, below the knee, ankle, midfoot, and toe
 Normal 70-120 mm Hg
 Drop between segments >30 mm Hg indicate disease in vessel above
Pulse volume recordings (PVRs)
 Normal waveforms are triphasic
 Waveforms are widened and blunted with severe disease

What is the most common type of skin cancer?


Basal cell carcinoma – found on sun-exposed parts of the body

What skin cancer may appear cauliflower-like?


Squamous cell carcinoma – found on sun-exposed parts of the body

What is the most common type of melanoma?


Superficial spreading melanoma – found on any part of the body

Most malignant?
Nodular melanoma – may be misdiagnosed as pyogenic granuloma

Most benign?
Lentigo melanoma – typically found on back, arms, neck, and scalp

Typically found on the palms, soles, and nail beds?


Acral lentiginous melanoma

What is a Hutchinson sign?


Pigment changes in the eponychium seen with subungual melanoma

54 CROZER-KEYSTONE RESIDENCY MANUAL – SECOND EDITION


What is the most common vascular proliferation?
Hemangioma

What vascular malignancy appears as red-blue plaques or nodules and has a high
incidence in AIDS?
Kaposi sarcoma

What conditions may be associated with plantar fibromatosis?


Ledderhose disease
Dupuytren contracture
Peyronie disease

What is another name for congenital convex pes valgus?


Vertical talus

What are radiographic findings of CCPV?


Calcaneus in equinus, plantarflexed talus, dorsally dislocated navicular, increased talo-calc angle

What additional radiographic study should be obtained for neonates with CCPV?
Lumbosacral films

Coalitions
What are three coalitions of the rearfoot?
Talocalcaneal, calcaneonavicular, and talonavicular

What percentage of tarsal coalitions are bilateral?


50%

Which is most symptomatic?


C-N

Asymptomatic?
T-N

Which is most common?


T-C > C-N > T-N

Which T-C facet is most commonly fused?


Medial > anterior > posterior

What are the ages of fusion?


T-N (3-5 years)
C-N (8-12 years)
T-C (12-16 years)

CROZER-KEYSTONE RESIDENCY MANUAL – SECOND EDITION 55


What are clinical symptoms of tarsal coalitions?
Pain
Limited ROM of STJ and possibly MTJ
Peroneal spastic flatfoot

What are radiographic findings of tarsal coalitions?


 Rounding of lateral talar process
 Talar beaking due to increased stress on talonavicular ligament
 Asymmetry of anterior subtalar facet
 Narrowing or absence of middle and posterior subtalar facets
 Halo sign – circular ring of increased trabecular pattern due to altered compressive forces
 Anteater sign – C-N coalition in which calcaneus has elongated process on lateral view
 Putter sign – T-N coalition in which neck of talus unites with broad expansion of
navicular

The anterior facet is best seen by which radiographic views?


Medial oblique, Ischerwood

The middle and posterior facets are best seen by which radiographic view?
Harris Beath

What are treatments for symptomatic tarsal coalitions?


 Orthotics or supportive therapy
 Immobilization
 NSAIDs
 Badgley – surgical resection of coalition or bar with interposition of muscle belly
 Isolated fusion or triple arthrodesis

Clubfoot
What are the 3 components of clubfoot?
FF adductus, RF varus, ankle equinus

What ligaments/capsules are contracted?


Posterior
 Posterior tib-fib
 Posterior talo-fib
 Lateral calcaneofibular
 Syndesmosis
Medial
 Superficial deltoid
 Tibionavicular
 Calcaneonavicular
 Talo-Navic, Navic-Cunei, and Cunei-1st MT joints
 Spring ligament

56 CROZER-KEYSTONE RESIDENCY MANUAL – SECOND EDITION


What muscles/tendons are contracted?
Posterior
 Achilles tendon
 Plantaris tendon
Medially
 PT, FDL, and FHL
 Abductor hallucis
Anteriorly
 Tibialis anterior

What is the technique for correction of clubfoot called?


Ponseti technique
 Serial casting
 First correct the FF and RF deformities, and then correct ankle equinus
 During manipulation, pressure is applied to the head of the talus (not the calcaneus)
 4-8 casts, percutaneous Achilles tenotomy (last cast for 3 weeks), occasional TA transfer,
and D-B bar brace until age 3 y/o to prevent relapse

What is the most accepted theory about clubfoot?


Germ plasma defect-malposition of head and neck of talus

What is the Simon rule of 15?


For clubfoot, children <3 years → talo-navicular subluxation
T-C angle is <15° and talo-1st metatarsal angle is >15°

CROZER-KEYSTONE RESIDENCY MANUAL – SECOND EDITION 57


58 CROZER-KEYSTONE RESIDENCY MANUAL – SECOND EDITION
Biomechanics
1st Ray/Bunion Evaluation
Hallux interphalangeal angle
Normal 0-10°

DASA (distal articular set angle)


Normal 7.5°

PASA (proximal articular set angle)


Normal 7.5°

Types of joint deformities


Congruent – joint lines are parallel
Deviated – joint lines intersect outside joint
Subluxed – joint lines intersect inside joint

Types of bunion deformities


Structural
 Bony deformity
 Abnormal PASA and DASA
 PASA + DASA = HA
Positional
 Soft tissue deformity with subluxed or deviated joint
 Normal PASA and DASA
 PASA + DASA < HA
Combined
 Elements of both structural and positional with subluxed or deviated joint
 Abnormal PASA and DASA
 PASA + DASA < HA

Hallux abductus angle


Normal 10-15°

IM angle (intermetatarsal angle)


Normal 8-12°
Head procedure if mild 10-13°
Shaft procedure if moderate 14-17°
Base procedure if severe 18-21°
Lapidus procedure if hypermobile 1st ray

Metatarsus adductus angle


Normal <20°

CROZER-KEYSTONE RESIDENCY MANUAL – SECOND EDITION 59


True IM angle
True IM angle = (metatarsus adductus angle - 15) + IM angle

1st Metatarsal protrusion distance


Normal +/- 2 mm compared to the 2nd metatarsal

Tibial sesamoid position


Normal 1-3

ROM 1st MPJ


Normal 65-75° dorsiflexion and 40° plantarflexion

1st Metatarsal-medial cuneiform angle


Normal 22°

1st ray ROM


Normal 5 mm dorsiflexion + 5 mm plantarflexion = 1 cm total ROM

5th Ray/Tailor Bunion Evaluation


Fallat & Buckholz 4th IM angle
Angle between bisection of 4th metatarsal and proximal-medial cortical border of 5th metatarsal
Normal 6°
Pathologic 8.7°

Fallat & Buckholz Lateral Deviation angle (lateral bowing)


Angle of line bisecting head and neck of 5th met and line adjacent to proximal-medial cortex
Normal 2.64°
Pathologic >8°

Metatarsals/MPJ
Metatarsal length
Longest 2 > 3 > 5 > 4 > 1 shortest

Metatarsal protrusion
Longest 2 > 3 > 1 > 4 > 5 shortest

Lesser MPJ dorsiflexion/plantarflexion


30-40° Dorsiflexion and 50-60° plantarflexion

Metatarsal declination angle


Normal 21°

Metatarsal abductus angle


Normal 0-15°

60 CROZER-KEYSTONE RESIDENCY MANUAL – SECOND EDITION


IM angle of 2nd and 5th metatarsals
Normal 14-18°

Splayfoot
IM angle of 1st and 2nd metatarsals
Pathologic >12°

IM angle 4th and 5th metatarsals


Normal 4-5°
Pathologic >9°
(Schoenhause says normal 4th IMA is 8°)

Splayfoot
1st IM angle >12° and 4th IM angle >8°
With metatarsus primus adductus, there is a high predilection of splayfoot

Talus
Talar neck angle
Long axis of head and neck with long axis of the body
Birth 130-140°
Adult 150-165°

Talar head and neck


Plantarflexed 25-30°
Medially aligned 15° to body

Talar torsion angle


Head is laterally rotated on the body
Fetus 18-20°
Childhood 30°
Adult 40°
Note: this motion brings the supinated foot in utero to a more pronated adult position

Talar declination angle


Normal 21°
Pronation – increases
Supination – decreases

Meary angle – Lateral view


Intersection of longitudinal axis of talus and 1st metatarsal
Normal 0°
Increases with either pronation or supination
Pronation – moves axis of the talus plantar to 1st metatarsal
Supination – moves axis of the talus dorsal to 1st metatarsal

CROZER-KEYSTONE RESIDENCY MANUAL – SECOND EDITION 61


Cyma line
S-shaped line formed by the articulation of T-N and C-C joints
Pronation – moves line anteriorly
Supination – moves line posteriorly

Talo-navicular joint
Normal 75° coverage
Pronation – decreases coverage
Supination – increases coverage

Forefoot abductus
Normal 8° (0-15°)

Calcaneus
Calcaneal inclination angle
Normal 21°
Pronation – decreases
Supination – increases

Hibb angle – Lateral view


Intersection of longitudinal axis of calcaneus and 1st metatarsal

Fowler & Philip angle


Angle formed from the intersection of a line along the anterior tubercle and the plantar tuberosity
and another line along the posterosuperior prominence at the Achilles tendon insertion
Normal <70°
Haglunds deformity >75°

Total angle of Ruch


Fowler & Philip angle + calcaneal inclination angle
Normal 90°
Haglunds deformity >90°

Calcaneal-cuboid abduction
Normal 0-5°
Increases with pronation

Kite angle (Talocalcaneal) – AP view


Infant 30-50°
Adult 20-40°
Pronation – increases
Supination – decreases

Talocalcaneal angle – Lateral view


Normal 25-50° (does not change with age)

62 CROZER-KEYSTONE RESIDENCY MANUAL – SECOND EDITION


Rearfoot Angles
Subtalar joint axis direction
STJ goes through 1st ray in neutral, 2nd ray in supination, and is medial to 1st ray in pronation

STJ axis of motion


Lateral, posterior, plantar → medial, anterior, dorsal
48° from frontal plane
42° from transverse plane
16° from sagittal plane

STJ ROM
From neutral, 2/3 motion in inversion (20°) and 1/3 in eversion (10°)

Longitudinal midtarsal joint


75° from frontal plane
15° from transverse plane
9° from sagittal plane

Oblique midtarsal joint


38° from frontal plane
52° from transverse plane
57° from sagittal plane

Bohler angle
Angle formed by the intersection of a line from the superior aspect of the anterior process to
superior aspect of the posterior facet and another line from the superior aspect of the posterior
facet to superior point of the calcaneal process
Normal 25-40°
Decreases with intra-articular calcaneal fracture

Gissane angle
Angle formed by the intersection of a line along the posterior facet and another line along the
middle and anterior facets
Normal is 125-140°
Increases with intra-articular calcaneal fracture

Toyger angle
Line drawn down posterior aspect
Normal should be a straight line (180°)
Decreases with Achilles rupture

CROZER-KEYSTONE RESIDENCY MANUAL – SECOND EDITION 63


Ankle
Dorsiflexion/plantarflexion
Normal 10-20° dorsiflexion and 20-40° plantarflexion

Axis
Lateral, posterior, plantar → medial, anterior, dorsal

Tibia
Tibial torsion
Birth 0°
6 years 13-18°
Adult 18-23°

Tibial varum/valgum
Compare distal 1/3 of tibia to ground
Birth 5-10° varum
>2 years 2-3° valgum

Femur
Angle of inclination
1 year 146°
4 years 137°
Adult 120-136° (avg 127°)

Angle of declination (antetorsion angle)


1 year 39°
10 years 24°
Adult 6°

Angle of anteversion
Birth 60°
Adult 10-12°

Lower Extremity Joint ROM


Hip flexion/extension with knee extended
Normal flexion 90-100°
Normal extension 10-20°

Hip flexion with knee flexed


Normal flexion 120-130°

Rotation of hip
Adults Children
Internal rotation 35-40° 20-25°
External rotation 35-40° 45-50°

64 CROZER-KEYSTONE RESIDENCY MANUAL – SECOND EDITION


Hip abduction/adduction
Abduction 24-60° (avg 36°)
Adduction <30°

Knee flexion/extension
Flexion 130-150°
Extension 5-10°

Knee rotation with knee flexed


Medial rotation 40°
Lateral rotation 40°

Knee valgum/varus (bow leg, knock knee)


Birth 15-20° (genu varum)
2-4 yrs 0° (straight)
4-6 years 5-15° (genu valgum)
6-12 years 0° (straight)
12-14 years 5-10° (genu valgum)
> 14 years 0° (straight)

Clinical Tests
What is the Ortolani test?
Test for congenital hip dislocation in newborns. With newborn supine and hip and knees flexed,
the hips are adducted while pressing downward and abducted while lifting upward. An unstable
hip will dislocate when adducted and reduce when abducted.

Barlow test?
Test for a hip that is dislocatable but not dislocated in infants. With infant supine and hip and
knees flexed, push posteriorly in line with the shaft of femur. An unstable femoral head will
dislocate posteriorly from acetabulum.

Galeazzi (or Allis) sign?


Sign of unilateral congenital hip dislocation in infants. With infant supine and hip and knees
flexed, the knees should be level. If a knee is lower, that hip is dislocated.

Trendelenberg test?
Test for weak hip abductors. As patient stands on affected limb, pelvis drops to opposite side.

CROZER-KEYSTONE RESIDENCY MANUAL – SECOND EDITION 65


66 CROZER-KEYSTONE RESIDENCY MANUAL – SECOND EDITION
Surgery
Surgical Prophylaxis
What are indications for the use of antibiotics?
 Implants (joint or internal fixation)
 Prolonged surgery (>2 h)
 Trauma surgery
 Revisional surgery
 Immunocompromised patient
 Extensive dissection required
 Intra-operative contamination
 Endocarditis (SBE)

What antibiotics are most commonly used?


Ancef
Clindamycin if PCN allergy
Vancomycin if concerned about MRSA

Peri-operative Management
What pre-op orders are needed for an in-house patient?
 NPO after midnight, except AM meds with sips of water
 Hold all AM hypoglycemics and cover with SSI (if patient with DM)
 Accu-Check on call to OR (if patient with DM)
 Begin ½NSS @ 60 mL/h at 0600 (D5W½NSS if patient with DM)
 Labs – CBC with diff, PT/PTT/INR, BMP
 Chest X-ray, EKG (if necessary)
 Consult medicine for medical clearance (if not already done)
 Anesthesia to see patient (if necessary)

What are indications for ordering a chest X-ray?


>40 years of age, smoker, any history of cardiac or pulmonary disease

What are indications for ordering an EKG?


>40 years of age, any history of cardiac disease

What is the most common time for post-operative myocardial infarction?


Day 3

How long should elective surgery be delayed following an MI or CABG?


6 months

CROZER-KEYSTONE RESIDENCY MANUAL – SECOND EDITION 67


How to calculate daily fluid input requirements?
First 10 kg x 100 = 1000 mL/day
Second 10 kg x 50 = 500 mL/day
Remaining kg x 20 = ___ mL/day
(e.g. 70 kg patient requires 1000 + 500 + 1000 = 2500 mL/day)

How to calculate IV fluid input rate?


―421 Rule‖ calculates IV mL/h
First 10 kg x 4 = 40 mL/h
Second10 kg x 20 = 20 mL/h
Remaining kg x 1 = ___ mL/h
(e.g. 70 kg patient requires 40 + 20 + 50 = 110 mL/h)

What other factors should be considered prior to surgery?


Is the patient on any insulin, anticoagulants, steroids, or anything else that might put them at risk
Note: any non-routine orders should be cleared with patient's primary service

What is the perioperative management for patients with diabetes?


 NPO after midnight
 Start D5W½NSS in AM
 Accu-Check
 If insulin-controlled, hold regular insulin, give ½ NPH dose, and cover with SSI
 If oral-controlled, hold oral meds and cover with SSI
 If diet-controlled, cover with SSI

What should be obtained prior to surgery on a patient with rheumatoid arthritis?


Cervical spine x-ray

What are effects of a long-term, high-dose course of steroids?


Long-term therapy suppresses adrenal function
 Risk of poor or delayed wound healing. Decreased inflammatory process.
 Risk of infection. Low WBC may mask infection.

What is the perioperative management for patients on long-term, high-dose steroids?


Peri-op IV steroid supplementation
Hydrocortisone 100 mg IV given the night before surgery, immediately prior to surgery, and then
q8h until postoperative stress relieved

What is the perioperative management for patients at risk for gout?


Begin colchicine 0.6 mg PO daily 3-5 days pre-op and continue 1 week post-op

What is the perioperative management for patients with hypertension?


If the patient has been on long-term diuretics (e.g. HCTZ, Lasix), check for hypokalemia
Avoid fluids high in sodium; may use ½NSS at low rate

68 CROZER-KEYSTONE RESIDENCY MANUAL – SECOND EDITION


When should aspirin be discontinued prior to surgery?
7 days due to irreversible binding to platelets

When should NSAIDs be discontinued prior to surgery?


3 days due to reversible binding to platelets

When should heparin be discontinued prior to surgery?


8 hours (monitor PTT)

When to Coumadin be discontinued prior to surgery?


3-4 days (monitor PT/INR)

What should the INR be for elective surgeries?


<1.4

What should be done if the INR is >1.4?


If necessary, transfuse Fresh Frozen Plasma (FFP)
One unit of FFP will decrease INR by approximately 0.2
Vitamin K can be given but is slow-acting

When should a patient with an INR >1.4 be allowed to proceed to surgery?


 If the risk of not doing surgery outweighs the risk of excessive bleeding (i.e. if it is an
emergency surgery and you have anesthesia's approval)
 If the patient has PVD and the surgery is a simple debridement or amputation. Note: if
the patient has PVD, make sure you have Vascular Surgery's approval for surgery. In this
case, it is acceptable for the patient to bleed a little extra.

If a patient with a high INR undergoes surgery, what labs should be carefully monitored?
Hgb and Hct

When should a RBC transfusion be given?


If Hgb <8 or Hct<24, consider transfusing 1-2 units PRBC
One unit of PRBC will increase Hct by approximately 3 percentage points

What should be done if the patient is thrombocytopenic?


Order a six pack of platelets, which is a concentration of six pooled platelet units, and consult
hematology

Plastic Surgery
How are relaxed skin tension lines (RSTL) oriented?
Perpendicular to the long axis of the leg and foot

Should a skin incision typically be made parallel or perpendicular to the RSTL?


Parallel incisions will remain approximated and heal better while perpendicular incisions may
gap apart due to increased transverse forces

CROZER-KEYSTONE RESIDENCY MANUAL – SECOND EDITION 69


What is an anti-tension line?
S-shaped or zig-zagged incision when exposure needed is not parallel to RSTL

To close a lesion with minimal tension, what should the ratio of length to width be?
3:1 length:width

How much lengthening can be achieved with a 60° Z-plasty?


75%

To correct a skin contracture, how should the Z-plasty incisions be oriented?


The central arm of the ―Z‖ should be parallel to the contracture

To correct a 5th digit adductovarus rotation, how should the skin incision be oriented?
Distal medial to proximal lateral

What is the order of wound graft closure?


1. Direct closure
2. Graft
3. Local flap
4. Distant flap

What are the stages of skin graft healing?


1. Plasmatic
2. Inosculation of blood vessels
3. Re-organization
4. Re-innervation

What are Blair and Humby knives?


Knives for harvesting skin grafts

What device is more commonly used to harvest skin grafts?


Dermatome

What is the most common complication of skin grafts?


Seroma/hematoma

How do you prevent it?


Mesh or pie crust graft and apply compressive dressing

What are advantages of using a split-thickness skin graft?


Donor site heals spontaneously
May cover large wounds

70 CROZER-KEYSTONE RESIDENCY MANUAL – SECOND EDITION


What are disadvantages?
Grafts are fragile
Contraction of graft during healing
May be abnormally pigmented

What are advantages of using a full-thickness skin graft?


Minimal contraction of graft
Better appearance

What are disadvantages?


More difficult to take
Must close donor site

What is an advantage of using a muscle flap?


It brings immediate increased blood supply to donor site

Fixation Devices
AO principles of internal fixation (2002)
 Anatomic articular reduction, adequate shaft reduction
 Stable/biologic fixation
 Preservation of blood supply
 Early ROM

AO principles (1958)
 Anatomic reduction
 Rigid internal fixation
 Preservation of blood supply
 Early ROM

What are the steps to inserting a fully threaded screw?


1. Overdrill near cortex
2. Underdrill through far cortex
3. Countersink
4. Measure
5. Tap
6. Screw

How much of a screw should pass the far cortex?


1 ½ threads

What is the purpose of tapping?


Creates a path for the screw threads

Why do you countersink a screw?


Prevents stress risers and soft tissue irritation
Provides even compression from screw head (land)

CROZER-KEYSTONE RESIDENCY MANUAL – SECOND EDITION 71


Describe mini fragment screws
Screw sizes of 1.5, 2.0, 2.7 – all fully threaded, cortical screws

What is the screwdriver handle made out of?


Pressed linen

What are the differences between cortical and cancellous screws?


Cortical has smaller pitch
Cortical has smaller rake angle
Cortical has smaller difference between thread diameter and core diameter

Describe a malleolar screw


For fixation of medial malleolus, partially threaded, same thread profile and pitch as cortical
screw, trephedine self-cutting tip

What screw has a fluted tip?


Self-tapping

What are the screw sizes? What are their underdrill sizes? Overdrill? Countersink?

Mini Fragment

Sizes 1.5 2.0 2.7 All are fully threaded


Overdrill 1.5 2.0 2.7
Underdrill 1.1 1.5 2.0
Countersink 1.5 2.0 2.7

Small Fragment

Sizes 3.5 4.0 fully threaded 4.0 partially threaded


Overdrill 3.5 4.0 4.0
Underdrill 2.5 2.5 2.5
Countersink 3.5 4.0 4.0

Large Fragment

Sizes 4.5 4.5 malleolar 6.5 partially threaded 6.5 fully threaded
Overdrill 4.5 4.5 6.5 6.5
Underdrill 3.2 3.2 3.2 3.2
Countersink 4.5 4.5 6.5 6.5

What sizes are in the Synthes modular hand screw system?


1.0, 1.3, 1.5, 2.0, 2.4, 2.7

72 CROZER-KEYSTONE RESIDENCY MANUAL – SECOND EDITION


What are the cannulated screw sizes?
For Synthes 3.0, 4.0
For Smith & Nephew 4.0, 6.5, 5.5 and 7.0

What are the steps for inserting a 4.0 cannulated screw?


1. Insert 1.3 mm guide pin to far cortex
2. Measure
3. Drill near cortex with 4.0 cannulated bit (optional)
4. Drill far cortex with 2.7 cannulated bit (unnecessary for soft bone)
5. Tap (unnecessary with self tapping screws)
6. Countersink
7. Screw

What is a Herbert screw?


Headless screw – can be inserted through articular cartilage. Threaded portion proximally and
distally and smooth in between. Proximal portion has tighter pitch for compression.

What is a Reese screw?


Headless – create compression through arthrodesis. Proximal threads run clockwise, and distal
threads run counterclockwise. Smooth in between.

What are the K-wire sizes and widths in millimeters?


Size 0.028 0.035 0.045 0.062
Width (mm) 0.6 0.9 1.2 1.6

Why is there a question about K-wires in a screw set section?


K-wires can be used for the underdrill if the situation arises (e.g. underdrill bit is missing or it
fell on the floor)
The 0.062 can be used for the 1.5 underdrill (for the 2.0 screw)
The 0.045 can be used for the 1.1 underdrill (for the 1.5 screw)

What are the K wire sizes and their appropriate caps?


0.028 0.035 0.045 0.062
Yellow Blue White Green
(young boys wear green)

What are the sizes of Steinman pins?


Every one from 5/64 to 12/64 except for 11/64

What are the different types of plate fixation?


Compression
 Provides axial compression of fracture
 Pre-bend plate
 Eccentric drilling of hole adjacent to fracture; remaining holes drilled centrally
 Place plate on tension side of bone

CROZER-KEYSTONE RESIDENCY MANUAL – SECOND EDITION 73


Neutralization
 Protects against shear, bending, and torsional forces at the fracture site
 Interfragmental compression obtained by lag screws
 All holes drilled centrally
Anti-glide
 Neutralization plate placed on the posterior aspect of the fibula
Buttress
 Maintains alignment of unstable fracture fragments
 No interfragmental compression

Should a plate be placed on the tension or compression side of a fracture?


Tension

Is the tension side of a metatarsal on the dorsal or plantar aspect?


Plantar

What is a locking plate?


Plate in which threaded screws are secured in to threaded plate holes
Does not rely on the bone for stability but rather forms a fixed-angle construct
Good for osteoporotic, comminuted fractures, or revision surgeries

What is the Hooke law?


For a material under load, strain is proportional to stress

What is the Young modulus?


After a load is removed, the material will spring back to its original shape, the resulting slope
represents the stiffness of a material or the Young modulous

Suture and Absorbable Fixation Devices


What is a Keith needle?
Straight needle

What are some common needle point configurations?


Taper point – for soft, easily penetrated tissue (subcutaneous tissue, fascia)
Cutting – cutting edge on inner curve (skin)
Reverse cutting – cutting edge on outer curve for tough, difficult to penetrate tissue

What is orthofix?
Polyglycolic acid (same as dexon)

How long for orthofix to lose strength/absorb?


Loses strength in 6-12 weeks
Absorbed in 1-3 years

What is orthosorb?
PDS (PDS=orthosorb)

74 CROZER-KEYSTONE RESIDENCY MANUAL – SECOND EDITION


How long before PDS loses its strength? When is it absorbed?
Loses strength in 4-6 weeks
Absorbed in 3-6 months

What are the two sutures that are the least reactive to tissue?
Stainless steel (least reactive), Prolene

What is Vicryl?
Polyglactin 910 (a copolymer of 90% glycolide and 10% lactide)

How is Vicryl broken down?


Hydrolysis

How long does it take to absorb Vicryl?


Tensile strength
75% @ 2 weeks
50% @ 3 weeks
25% @ 4 weeks
Absorbed completely in 10 weeks

Should you use Vicryl with an infection?


Avoid it if possible, since Vicryl is too reactive

Arthroscopy
Who first describe arthroscopy?
Takagi

Who were the first podiatrists to describe a podiatric use for arthroscopy?
Heller & Vogel (1982)

What are the different scope techniques?


Scanning – side to side, up and down
Pistoning – in and out
Rotation – 360°

What are a few indications for an ankle scope?


Synovitis
Osteochondral lesion/fracture
Soft tissue impingement
Osteophytes
Loose bodies

What are other uses for arthroscopy?


Endoscopic plantar fasciotomy (EPF) or endoscopic gastroc recession
Ankle fusion
Arthroscopy of STJ or 1st MPJ

CROZER-KEYSTONE RESIDENCY MANUAL – SECOND EDITION 75


What is the most common complication following an EPF?
Lateral column instability → calcaneal-cuboid joint pain

1st Ray Surgery


What is the most common indication for a Lapidus?
Hypermobile 1st ray

What is the order of the lateral release for a McBride?


1. Extensor hood
2. ADH tendon release
3. Fibular sesamoid ligament
4. Lateral collateral ligament
5. FHB
6. Fibular sesamoid excision (if performing)

What is the difference between a Vogler, Kalish, and Youngswick?


Vogler – offset V (apex at metaphyseal-diaphyseal joint)
Kalish – long-arm Austin with angles of approximately 55° for screw fixation
Youngswick – Austin with a slice taken dorsally to allow decompression and plantar flexion

What procedures correct PASA?


Reverdin
Peabody
Biangular Austin
DRATO
Offset V with rotation

What procedure corrects DASA?


Proximal Aikin

What procedure corrects hallux interphalangeous?


Distal Aikin

What are complications associated with a Keller?


Diminished propulsion of digit, loss of hallux purchase, stress fracture of 2nd metatarsal

What should be done if the capital fragment falls on the floor?


1. Rinse with saline
2. Bacitracin soak for 15 minutes
3. Rinse with saline
4. Bacitracin soak for 15 minutes
5. Rinse with saline
6. Document and inform patient

76 CROZER-KEYSTONE RESIDENCY MANUAL – SECOND EDITION


Post-Op White and Blue Toes
What are causes of a white toe post-operatively?
Arterial in nature, usually acute
Signs – pain, pale, parasthesia, pulselessness

What are treatments for a white toe?


 D/C ice and elevation
 Loosen bandages
 Place foot in dependent position
 Rotate K-wire
 Apply warm compresses proximally
 Apply nitroglycerine paste proximally
 Local nerve block proximally
 Avoid nicotine
 Consult vascular surgery

What are causes of a blue toe?


Poor arterial inflow – toe is cold and doesn't blanch with pressure
Poor venous outflow – toe is warm and will blanch with pressure

What are treatments for blue toe due to arterial insufficiency?


 (Treat like white toe)
 D/C ice and elevation
 Loosen bandages
 Place foot in dependent position
 Rotate K-wire
 Apply warm compresses proximally
 Apply nitroglycerine paste proximally
 Local nerve block proximally
 Avoid nicotine and caffeine
 Thermostat controlled heat lamp, not to exceed 90°
 Vasodilators
 Consult vascular surgery

What are treatments for blue toe due to sluggish venous outflow?
 D/C ice (but not elevation)
 Loosen bandages
 Avoid dependency
 Don’t attempt to increase vascular perfusion
 Consult vascular surgery

CROZER-KEYSTONE RESIDENCY MANUAL – SECOND EDITION 77


Rearfoot Surgery
Describe a Keck & Kelly procedure?
For Haglund deformity with cavus foot and high calcaneal inclination angle. Remove wedge
from posterior-superior aspect of calcaneus. The posterior superior prominence is moved
anteriorly.

What are treatments for equinus?


Stretching/exercises
Night splints
Gastroc recession
 Strayer
 Vulpius
 Baker
 McGlamary & Fulp
Tendoachilles lengthening
 Open/closed Z
 Hauser
 White
 Hoke
 Sgarlato
 Stewart

What is the Murphy procedure?


Achilles advancement for spastic equinus

Name surgical procedures for pes planus


Transverse
 Evans
 Kidner
 C-C distraction arthrodesis
Sagittal
 Cotton
 Young
 Lowman
 Hoke
 Miller
 Cobb
Frontal
 Koutsogiannis
 Dwyer
 Chambers
 Gleich
 Baker-Hill
 Lord

78 CROZER-KEYSTONE RESIDENCY MANUAL – SECOND EDITION


Name surgical procedures for pes cavus
Tendon
 Jones
 Hibbs
 STATT
 PT
Bone
 Dorsiflexory osteotomy of 1st metatarsal
 Cole
 Japas

What is an arthroereisis?
A surgical procedure to limit joint mobility (i.e. MBA implant in sinus tarsi)
Typically want 2-4° of STJ eversion with implant

What is the Valente procedure?


STJ block using a polyethylene plug with screw threads. Allows 4-5° of STJ pronation.

Who first described the triple arthrodesis?


Ryerson

What order do you resect and what order do you fixate the joints in a triple arthrodesis?
Resection
1. Midtarsal joints (T-N, CCJ)
2. Subtalar joint (T-C)
Fixation (opposite order)
1. Subtalar joint
2. Midtarsal joints

What are some types of fixation for a triple arthrodesis?


6.5-7.0 mm interfragmental compression screws, staples, plates

What are FDA-approved total ankle implants?


Two-component devices
 Agility
 Eclipse
 INBONE
 Salto Talaris
Three-component devices
 STAR
 (Not FDA-approved – Buechal-Pappas, TNK, HINTEGRA)

CROZER-KEYSTONE RESIDENCY MANUAL – SECOND EDITION 79


Bone Healing
What are the stages of bone healing?
Bone heals either primarily or secondarily
Primary healing – no motion and no callus formation
1. Inflammation
2. Induction
3. Remodeling
Secondary healing – micro-motion with callus formation
1. Inflammation
2. Induction
3. Soft callus
4. Hard callus
5. Remodeling

What are some factors that negatively affect bone healing?


Smoking, antimetabolite or steroid therapy, anemia, osteoporosis

Name the types of non-unions


Hypertrophic
 Elephant foot
 Horse hoof
 Oligotrophic
Atrophic
 Torsion wedge
 Comminuted
 Defect
 Atrophic

What study can distinguish between a hypertrophic and an atrophic non-union?


Bone scan – positive for a hypertrophic and negative for an atrophic (avascular) non-union

What is a pseudoarthrosis?
Type of non-union in which fibrocartilaginous tissue forms between fracture fragments

What are indications for bone stimulators?


Non-union, failed fusion

What are contraindications for bone stimulators?


Pseudoarthrosis, gap greater than ½ bone diameter

What are the stages of avascular necrosis?


1. Avascular – loss of blood supply, epiphyseal growth ceases
2. Revascularization – infiltration of new blood vessels, new bone deposited on dead bone,
flattening or fragmentation of articular surface
3. Repair and remodeling – bone deposition replaces bone resorption
4. Residual deformity – restoration of epiphysis, sclerosis, deformed articular surface

80 CROZER-KEYSTONE RESIDENCY MANUAL – SECOND EDITION


What is the best study for evaluating avascular necrosis?
MRI – decreased signal intensity within medullary bone in both T1 and T2 images

Bone Grafts
What are the different types of bone grafts?
Osteogenic – able to synthesize new bone
 Mesenchymal stem cells from autologous bone or bone marrow aspirate
Osteoinductive – contains factors that induce host tissue to form new bone
 Demineralized bone matrix
 Bone morphogenic protein
 Platelet-derived growth factors
Osteoconductive – provides scaffold for host tissue to propagate new bone
 Allografts
 Hydroxyapatite
 Calcium phosphate
 Calcium sulfate

What type of bone graft is osteogenic, osteoinductive, and osteoconductive?


Autograft

What are the stages of bone graft healing?


1. Vascular ingrowth
2. Osteoblastic proliferation
3. Osteoinduction
4. Osteoconduction
5. Graft remodeling

What is an early radiographic finding of bone graft healing?


Initial radiolucency of the graft due to increased osteoclastic activity which is followed by
osteoblasts laying down new bone

What is creeping substitution?


Process in which the host’s cutting cone (osteoclasts followed by osteoblasts) invade bone graft

Miscellaneous
What is the ASA classification for general anesthesia?
Class 1 – healthy
Class 2 – mild systemic disease
Class 3 – severe systemic disease
Class 4 – incapacitating systemic disease that is a threat to life
Class 5 – moribund patient who is not expected to live without surgery
Emergency

CROZER-KEYSTONE RESIDENCY MANUAL – SECOND EDITION 81


What is the maximum tourniquet time?
90-120 minutes
After that, allow 5 minutes of perfusion for every half hour over

What are contraindications to using a tourniquet?


Infection
Open fracture
Sickle cell disease
Peripheral vascular disease
Recent arterial graft
Previous DVT
Hypercoagulability
Skin grafts application where bleeding must be distinguished

According to Seddon, what are the different types of nerve damage?


Neuropraxia – nerve contusion resulting in conduction block that recovers promptly
Axonotmesis – interruption of axons with distal Wallerian degeneration. Supporting connective
tissue sheaths remain intact allowing regeneration.
Neurotmesis – complete severance of the nerve that is irreversible

What is the difference between an incisional and excisional biopsy?


Incisional – only a portion of the lesion is removed
Excisional – the entire lesion is removed

What are different biopsy techniques?


Punch, shave, curettage, surgical excision

How does a bone stimulator work?


Piezoelectric principle – side under compression makes a negative charge that leads to bone
growth. Therefore, placing a cathode in a non-union site will stimulate growth.

What is the direction of the cut for reverse Wilson of the 5th metatarsal?
Distal lateral to proximal medial

Who was the first to describe an arthrodesis?


Soule

What is the order for hammertoe surgery?


Note: Perform a Kelikian push-up test to determine if the next step is required
1. PIPJ
 Tendon
 Dorsal capsule
 Collaterals
 Plantar capsule
 Arthroplasty

82 CROZER-KEYSTONE RESIDENCY MANUAL – SECOND EDITION


2. MPJ
 Hood
 Tendon
 Capsule
 Plantar plate
3. PIPJ
 Arthrodesis

Why are joint implants used?


Maintain space between bony surfaces
Relieve pain

What is the lag time for presentation of osteomyelitis on an X-ray?


10-14 days

How to culture osteomyelitis?


Take one culture from the infected bone, and take a second culture proximal to the clearance
margin to ensure remaining bone is not infected

What is a Brodie abscess? What is the treatment?


Subacute osteomyelitic lesion usually found in children. It is a well-circumscribed, lytic lesion
with sclerotic borders found in the metaphysis, epiphysis, and rarely diaphysis. Painful with
periods of exacerbation and remission.
Tx: curettage and packing with autologous bone

What is in antibiotic beads?


PMMA or Poly(methyl methacrylate). Gentamycin or tobramycin are often used since they are
heat stable with good diffusion coefficiencies. Vancomycin and cefazolin may also be used.

CROZER-KEYSTONE RESIDENCY MANUAL – SECOND EDITION 83


84 CROZER-KEYSTONE RESIDENCY MANUAL – SECOND EDITION
Trauma
What should be done when assessing a patient with trauma?
Primary survey (ABCDE)
 Airway
 Breathing
 Circulation with hemorrhage control
 Disability – assess neurologic status
 Exposure of patient and environmental control
Secondary survey
 Full history – medical and drug
 Thorough examination
o Evaluate tenderness and stability as well as neurovascular status of each limb
o Is there injury to joint above or below?
 X-rays and/or CT of all suspected fractures

What should always be asked with a break in the skin?


Tetanus status

Name the appropriate classification


Distal phalangeal/nail Rosenthal
1st Metatarsophalangeal Jahss
5th Metatarsal base Stewart
Lisfranc joint Quenu & Kuss, Hardcastle
Navicular Watson Jones
Posterior tibial tendon Conti (based on MRI findings)
Talar neck Hawkins
Talar body Sneppen
Talar dome Berndt-Hardy, Fallot & Wy
Calcaneus Rowe, Essex-Lopresti, Sanders
Anterior process calcaneal fracture Degan
Ankle sprains O'Donoghue, Leach, Rasmussen
Physeal ankle fracture Dias & Tachdjian
Epiphyseal fracture Salter-Harris
Ankle fracture Lauge-Hansen
Pilon fracture (distal tibia metaphysis) Ruedi & Allgower, Dias & Tachdjian
Achilles rupture Kuwada
Open fracture Gustillo
Non-unions Weber & Cech
Frostbite Orr & Fainer, Washburn

What is a clinical test for a fracture?


Point tenderness over fracture site

CROZER-KEYSTONE RESIDENCY MANUAL – SECOND EDITION 85


What are common fracture patterns?
Transverse, greenstick, torus, oblique (spiral), comminuted

Which is the most stable pattern?


Transverse is most stable

What is the weakest region of the physis?


Zone of cartilage maturation

What is the Vassal principle?


Initial fixation of the primary fracture will assist stabilization of the secondary fractures

What are possible complications of fractures?


Delayed union
Non-union
Pseudoarthrodesis
OA
AVN

What is the most common cause of non-healing for a bone fracture?


Improper immobilization

Who was Lisfranc?


He was a field surgeon in Napoleon's army

Are dorsal or plantar Lisfranc dislocations more common?


Dorsal – the plantar ligaments are stronger than dorsal

What are the Ottowa Ankle Rules?


A series of ankle X-ray films is required only if there is any pain in the malleolar zone and any of
the following findings:
 Bone tenderness at posterior edge or distal 6 cm of lateral malleolus
 Bone tenderness at posterior edge or distal 6 cm of medial malleolus
 Inability to bear weight both immediately and in ED
A series of foot X-ray films is required only if there is any pain in midfoot zone and any of the
following findings:
 Bone tenderness at base of 5th metatarsal
 Bone tenderness at navicular
 Inability to bear weight both immediately and in ED

Talar Fractures
What is the classification for talar dome lesions?
Berndt & Hardy

What stages of Berndt & Hardy are often associated with lateral ankle ligament ruptures?
II, III, IV

86 CROZER-KEYSTONE RESIDENCY MANUAL – SECOND EDITION


What are the common locations of talar dome lesions and their mechanisms of injury?
(DIAL a PIMP)
Dorsiflexion Inversion – Anterior Lateral (unstable, shallow, wafer-shaped lesion)
Plantarflexion Inversion – Medial Posterior (deep, cup-shaped lesion)

What is Hawkins sign?


Presence of subchondral talar dome osteopenia seen 6-8 weeks after talar fracture signifying
intact vascularity. Absence of the sign implies AVN.

What is the Sneppen classification?


Talar body fractures

What percentage of fractures of the talus involve the calcaneus?


60%

Of these fractures, how many involve the joint?


75%

Calcaneal Fractures
What is a Mondor sign?
Plantar, rearfoot ecchymosis that is pathognomonic for calcaneal fractures

How is the Bohler angle affected by a calcaneal fracture?


Decreases with intra-articular calcaneal fracture

How is the Gissane angle affected by a calcaneal fracture?


Increases with intra-articular calcaneal fracture

What fractures are commonly associated with calcaneal fractures?


Vertebral fractures, especially L1
Femoral neck
Tibial plateau

What is the mechanism of injury for an anterior process fracture?


Inversion with plantarflexion

Ligamentous Ruptures
What are tests for ankle ligament pathology?
Anterior drawer test
Calcaneofibular-stress inversion
Abduction stress
Ankle arthrogram
Peroneal tenography

CROZER-KEYSTONE RESIDENCY MANUAL – SECOND EDITION 87


Describe the anterior drawer test
5-8 mm drawer → rupture of ATF
10-15 mm drawer → rupture of ATF + CF
>15 mm drawer → rupture of ATF + CF + PTF

Describe the talar tilt test


>10° → rupture of CFL

Describe the stress inversion test


5° inversion → rupture of ATF
10-30° inversion → rupture of ATF + CF

Achilles Tendon Ruptures


What are clinical symptoms of an Achilles tendon rupture?
Pain with history of ―pop‖
Weakness or loss of function
Palpable dell in area of ruptured tendon
Inability to perform single leg rise
Increased ankle dorsiflexion

What is the Thompson test?


A positive test results when squeezing of the calf muscle does not plantarflex the foot

What is the Hoffa sign?


Increased dorsiflexion compared to the contralateral side along with the inability to perform a
single leg rise test

What is a radiographic finding of an Achilles tendon rupture?


Disruption of Kagers triangle

Where is the most common location for the Achilles tendon to rupture?
1.5-4 cm proximal to the calcaneal insertion

Ankle Fractures
Name the fractures
Pott Bimalleolar fracture
Cotton Trimalleolar fracture
Tillaux-Chaput Avulsion fracture of anterior, lateral tibia from AITFL
Wagstaff Avulsion fracture of anterior, medial fibula from AITFL
Volkman Posterior tibial malleolar fracture from PITFL
Cedell Fracture of posterior medial process
Shepard Fracture of posterior lateral process
Foster Entire posterior process
Bosworth Lateral malleolar fracture with ankle displacement
Maisonneuve Proximal fibular fracture

88 CROZER-KEYSTONE RESIDENCY MANUAL – SECOND EDITION


What is the most common mechanism of injury (MOI) causing an ankle fracture?
SER

What is the MOI causing a transverse lateral malleolar fracture?


SAD I

What is the MOI causing a short, oblique medial malleolar fracture?


SAD II

What is the MOI causing a short, oblique lateral malleolar fracture (AP view)?
PAB III

What is the MOI causing a spiral, lateral malleolar fracture with a posterior spike (AP and
Lateral views)?
SER II

What is the MOI to the ankle with a high fibular fracture? What is this fracture called?
PER III
Maisonneuve fracture

What is a Lauge-Hansen Type V?


Pronation dorsiflexion
1. Vertical tibial malleolar tip fracture
2. Anterior tibial lip fracture
3. Supramalleolar fibular fracture
4. Transverse posterior tibia fracture level with proximal aspect of anterior tibial fracture

When should a posterior malleolar fracture be fixated?


ORIF when fragment is greater than 25% of the posterior malleolus

What direction should transsyndesmotic screws be inserted?


Approximately 30° from the sagittal plane from posterior-lateral to anterior-medial

Should transsyndesmotic screws be inserted using a lag technique?


No. Fully-threaded cortical screws are placed across both cortices of the fibula and the lateral
cortex of the tibia. The goal is stabilization rather than compression.

What do you test clinically test via Jack Toe Test?


Foster fracture – a fracture of the entire posterior process

What is the Thurston-Holland sign?


Epiphysis is separated from the physis with the fracture extending into the metaphysis resulting
in a triangular fracture fragment (AKA Flag sign)

CROZER-KEYSTONE RESIDENCY MANUAL – SECOND EDITION 89


90 CROZER-KEYSTONE RESIDENCY MANUAL – SECOND EDITION
Bone Tumors
What are the different patterns of bone destruction?
Geographic – well-defined, short zone of transition → benign or low-grade malignancy
Moth-eaten – more aggressive, intermediate zone of transition → benign or malignant
Permeative – poorly-defined, wide zone of transition → malignant

What are the different patterns of periosteal reactions?


Single layer – benign but sometimes malignant
Onion skin – malignant, multiple layers of periosteum
Sunburst – spiculated rays
Hair on end – parallel rays
Codman triangle – triangular elevation of periosteum

Name benign bone tumors of the foot


(FOG MACHINES)
F – fibrous dysplasia
O – osteochondroma
G – giant cell tumor
M – myeloma
A – aneurysmal bone cyst
C – chondroblastoma, chondromyxoid fibroma, clear cell
H – hemangioma
I – infection
N – non-ossifying fibroma
E – eosinophillic granuloma, enchondroma, epidermal inclusion cyst
S – solitary bone cyst

Name malignant bone tumors of the foot


Chondrosarcoma
Osteosarcoma
Periosteal sarcoma
Ewings sarcoma
Fibrosarcoma
Multiple myeloma

What is the most common, benign, primary bone tumor?


Osteochondroma

What is the most common, malignant, primary bone tumor?


Multiple myeloma

CROZER-KEYSTONE RESIDENCY MANUAL – SECOND EDITION 91


What primary bone tumors are more frequent in females?
Giant cell tumor
ABC
Parosteal osteosarcoma

What are the most common cancers that metastasize to foot?


Breast, prostate, lung, kidney

What bone tumors do not form matrix?


Bone cysts
Ewings sarcoma
Giant cell tumor

What are bone tumors typically located?


Epiphysis
 Chondroblastoma
 Giant cell tumor (forms in metaphysis)
Metaphysis
 Enchondroma (also diaphyseal)
 Osteochondroma
 Nonossifying fibroma
 Unicameral bone cyst
 Aneurysmal bone cyst
 Giant cell tumor (extends into epiphysis)
 Medullary osteosarcoma
 Parosteal osteosarcoma
 Chondrosarcoma
Diaphysis
 Osteoid osteoma
 Osteoblastoma
 Enchondroma (also metaphyseal)
 Ewings sarcoma (also meta-diaphysis)
 Periosteal osteosarcoma

Centrally located
 Enchondroma
 Unicameral bone cyst
Eccentrically located within medullary canal
 Giant cell tumor
 Chondrosarcoma
 Osteosarcoma
Cortical
 Osteoid osteoma
 Nonossifying fibroma

92 CROZER-KEYSTONE RESIDENCY MANUAL – SECOND EDITION


Periosteal
 Osteochondroma
 Periosteal osteosarcoma

What are characteristics of an osteoid osteoma?


 Benign, osteolytic lesion with central nidus (<1 cm) that may have calcifications
 1st to 2nd decades of life
 Dull pain, worse at night, relieved with ASA

What are characteristics of an osteoblastoma?


 ―Giant osteoid osteoma‖
 Benign tumor that may become malignant
 Osteolytic lesion with well-circumscribed nidus (>1.5 cm) that may have multiple
calcifications
 2nd to 3rd decades of life
 Less symptomatic than osteoid osteoma, pain not relieved by ASA

What are characteristics of an enchondroma?


 Benign, well-defined, intramedullary, cartilaginous lesion
 Geographic lesions with punctuate calcified matrix
 3rd to 4th decades of life
 Painless swelling unless pathologic fracture

What is Ollier disease?


 Multiple enchondromatosis
 May become malignant
 1st decade of life

What is Maffuccis disease?


 Multiple enchondromas with soft tissue hemangiomas
 Most become malignant
 1st decade of life

What are characteristics of a chondroblastoma?


 Benign, geographic, osteolytic, lesion with sclerotic margins
 2nd to 3rd decade of life
 Pain and joint effusion

What are characteristics of an osteochondroma?


 Most common benign primary bone tumor
 Cartilage-capped, hyperplastic bone pointing away from the joint
 2nd to 4th decades of life
 Suspect malignant transformation with growth after skeletal maturity, pain, or cap >2 cm

CROZER-KEYSTONE RESIDENCY MANUAL – SECOND EDITION 93


What are characteristics of nonossifying fibromas?
 Benign connective tissue lesion with fibrous replacement of bone
 Expansive, radiolucent, medullary lesions
 1st to 2nd decades of life
 Lesions typically resolve with age
 Do not biopsy

What are characteristics of a fibrous dysplasia?


 Benign, geographic, fibro-osseous lesion with ground glass matrix
 Presents with deformity
 Sometimes painful 2° to fracture

What are characteristics of a unicameral bone cyst?


 Benign, geographic, medullary lesion that is fluid-filled
 Commonly found in calcaneus
 Fallen fragment sign – pathologic fracture in which cortex lies within lesion
 1st to 2nd decades of life
 Asymptomatic until fracture

What are characteristics of an aneurysmal bone cyst?


 Benin, expansile, lytic lesion with blood-filled cavities
 May extend into soft tissue
 Fluid-fluid levels seen on MRI
 1st to 3rd decades of life
 More common in females
 Painful, especially with pathological fractures

What are characteristics of a giant cell tumor?


 Benign but locally aggressive, lytic lesion with ground glass, ―soap bubble‖ appearance
 May destroy cortex and have soft tissue mass
 More common in females
 3rd to 4th decades of life
 Painful

What are characteristics of a multiple myeloma?


 Most common primary malignant bone tumor
 Punched out lesions or diffusely osteopenic with hair-on-end radiating spicules
 Affect 45-80 y/o
 Painful with weakness or neurologic symptoms
 Bence-Jones protein found within urine

94 CROZER-KEYSTONE RESIDENCY MANUAL – SECOND EDITION


What are characteristics of an osteosarcoma?
 Most common primary malignant bone tumor
 Sunburst periosteal reaction with Codman triangle and cloud-like, dense bone formation
 2nd to 3rd decades of life
 Dull aching pain
 Medullary
o Poor prognosis
 Parosteal
o More common in females
o Better prognosis than medullary
 Periosteal
o Slightly better prognosis than medullary

What is the most common bone tumor associated with Paget disease?
Osteosarcoma

What are characteristics of a Ewings sarcoma?


 Common, malignant, primary bone tumor
 Aggressive, permeative, lytic lesion with hair-on-end, Codman triangle, and onion skin
(wings and onion rings)
 May have large soft tissue mass
 Usually under 20 y/o
 Painful with fever, weight loss, and elevated ESR
 Poor prognosis

What are characteristics of a chondrosarcoma?


 Common, malignant, moth-eaten, lesion with medullary and soft tissue calcifications
 May arise from malignant transformation of enchondromas or osteochondromas
 5th to 6th decades of life
 Painful

What study is most useful in searching for metastatic bone disease?


Total skeletal bone scan
Malignant lesions will show increased uptake

CROZER-KEYSTONE RESIDENCY MANUAL – SECOND EDITION 95


96 CROZER-KEYSTONE RESIDENCY MANUAL – SECOND EDITION
Wound Care
What are the stages of wound healing?
1. Inflammatory (lag) phase
 Days 1-4
 Initial vasoconstriction (minutes) followed by vasodilation (days)
 Neutrophils and macrophages are recruited
2. Proliferative (repair) phase
 Days 3-21
 Collagen synthesis provides tensile strength of wound
 At 14 days, tensile strength of would equals that of suture
3. Remodeling (maturation) phase
 Days 21 up to one year

In what stage of healing do chronic wounds stop progressing?


Proliferative

What is Santyl?
collagenase – an enzymatic debrider that digests collagen in necrotic tissue

What is Regranex?
PDGF-1 (platelet derived growth factor)

Where is Regranex made?


Puerto Rico (I was really asked this once)

What is a normal value for serum albumin?


3.4-5.0 g/dL

What is a low serum albumin level (<3.5 g/dL) associated with?


Decreased wound healing
Edema
Impaired cellular immunity
Decreased collagen synthesis
Decreased fibroblast proliferation

What minimum ABI is necessary for wound healing?


Non-diabetic patient – 0.35
Diabetic patient – 0.45

Using transcutaneous oximetry, what minimum pressure is necessary for wound healing?
Non-diabetic patient – 30 mm Hg
Diabetic patient – 40 mm Hg

CROZER-KEYSTONE RESIDENCY MANUAL – SECOND EDITION 97


How does negative pressure wound therapy (e.g. Wound VAC) assist wound closure?
NPWT applies mechanical shear stress to the wound site. This is believed to promote
granulation by decreasing bacterial bioburden, reducing edema, and inducing capillary budding.

How does hyperbaric oxygen therapy assist wound closure?


HBOT increases the partial pressure of O2 in arterial circulation, which increases diffusion of O2
at the wound site. This is believed to increase growth factors promoting angiogenesis and
collagen synthesis.

Bioengineered Tissue
What is Integra?
Bilayer graft composed of bovine tendon collagen with chondroitin-6-sulfate and a silicone layer
to control moisture loss

Oasis?
Extracellular graft matrix derived from porcine, small intestine submucosa

Apligraf?
Bilayer graft derived from neonatal foreskin with dermal and epidermal layers

GraftJacket?
Extracellular graft matrix derived from human tissue with intact vascular channels

TissueMend?
Acellular collagen matrix derived from fetal bovine dermis

98 CROZER-KEYSTONE RESIDENCY MANUAL – SECOND EDITION


Classifications
Note: Although all of the classifications are important, the ones in bold and capitalized (i.e.
LAUGE-HANSEN) are the most commonly used.

Forefoot
Hallux Valgus
Stage 1
 Excess pronation causes hypermobility of 1st ray. Tibial sesamoid ligament gets stretched
& fibular sesamoid ligament contracts.
 Lateral subluxation of proximal phalanx occurs
Stage 2
 Hallux abductus progresses, touches against 2nd digit
 FHL & FHB gain lateral mechanical advantage
 Crista starts to erode
Stage 3
 Further subluxation at 1st MPJ, formation of IMA
 IMA increases secondary to retrograde forces from abductor hallucis
Stage 4
 Hallux subluxes & dislocates on 1st metatarsal
 Increased crista erosion

Hallux Limitus
REGNAULD (Foot, 1986)
Grade 1 – functional hallux limitus with dorsal spurring
 Intact sesamoids with no associated disease
 Joint enlargement but joint space narrowing and arthrosis
 <40° dorsiflexion and <20° plantarflexion
Grade 2 – broad flat metatarsal head with structural elevatus and significant spurring
 Pain at rest
 Osteochondral defects in metatarsal head and sesamoidal hypertrophy
 Joint space hypertrophy and narrowing
 75% decrease in total ROM
Grade 3 – ankylosis and articular hypertrophy with extensive peri-articular osteophytes
 Osteochondral defect with joint mice and extensive 1st metatarsal-sesamoid disease
 Severe loss of joint space or collapse of joint → bone on bone
 FDL contracture

CROZER-KEYSTONE RESIDENCY MANUAL – SECOND EDITION 99


MODIFIED REGNAULD/OLOFF (ACFAS, 1994)
Stage 1 – functional hallux limitus
 Limited dorsiflexion with weightbearing but normal ROM with non-weightbearing
 No DJD changes on x-ray
 No pain on end ROM
Stage 2 – joint adaptation
 Flattening of metatarsal head with small dorsal exostosis
 Pain on end ROM
Stage 3 – joint deterioration
 Severe flattening of metatarsal head with non-uniform joint space narrowing,
osteophytes, and subchondral sclerosis/cysts
 Crepitus on ROM
Stage 4 – ankylosis
 Obliteration of joint space with osteophyte fragmentation
 Minimal to no ROM

Drago, Oloff, and Jacobs (J Foot Ankle Surg, 1984)


Grade 1 – pre-hallux limitus
 Pain on end ROM
 X-Rays: plantar subluxation of proximal phalanx, met primus elevatus, minimal DJD
 Joint preservation/reconstruction surgery
Grade 2 – flattening of metatarsal head
 Pain on end ROM, limited ROM
 X-Rays: small dorsal exostosis, osteochondral lesion, flattened met head
 Joint preservation/reconstruction surgery
Grade 3 – severe flattening of the metatarsal head
 Pain on full ROM, crepitus
 X-Rays: large dorsal exostosis, marked flattened met head, osteophytic production, non-
uniform joint space narrowing
 Joint preservation/reconstruction surgery
Grade 4 – obliteration on joint space with joint mice
 <10° ROM
 May be asymptomatic if ankylosed
 X-Rays: loss of joint space, loose bodies
 Joint destructive surgery

Hanft (J Foot Ankle Surg, 1993)


Grade 1 – metatarsus primus elevatus, mild dorsal exostosis, and sclerosis around MPJ
Grade 2 – Grade 1 with flattening of metatarsal head, joint space narrowing, and dorsal/lateral
osteophytes
Grade 3 – Grade 2 with DJD findings (osteophytes, subchondral sclerosis, and cysts)
Grade 4 – Grade 2 with severe flattening and sesamoid hypertrophy
Grade 5 – Grade 3 with DJD findings

100 CROZER-KEYSTONE RESIDENCY MANUAL – SECOND EDITION


Ktavitz, Laporta, Lauton (1994)
Stage 1 – normal to mild flattening of the head
Stage 2 – minimal narrowing
Stage 3A – irregular joint space narrowing with dorsal spurring and cysts
Stage 3B – minimal joint space with loose bodies and large dorsal flag
Stage 4 – no joint space with sesamoid fusion and large exostosis formation

1st Metatarsal Dislocations


JAHSS (condensed)
Type l – dorsal dislocation of proximal phalanx and sesamoids with intact intersesamoid
ligament
Type 2 – dorsal dislocation of proximal phalanx and sesamoids
2A – intact sesamoids with ruptured of intersesamoid ligament
2B – transverse fracture of sesamoid with intact intersesamoid ligament

JAHSS (Foot Ankle, 1:15, 1980)


* Secondary to extreme dorsiflexion
Type 1 – dorsal dislocation of proximal phalanx and sesamoids with intact intersesamoid
ligament
 Tx: requires ORIF
Type 2A – dorsal dislocation of proximal phalanx and sesamoids with ruptured intersesamoid
ligament
 Tx: closed reduction and surgical shoe or BK walking cast
Type 2B – dorsal dislocation of proximal phalanx and transverse fracture of sesamoid with intact
intersesamoid ligament
 Tx: closed reduction and surgical shoe or BK NWB cast or excision of the fractured
sesamoid

5th Metatarsal Fractures


Stewart (condensed)
Type 1 – extra-articular fracture at metaphyseal-diaphyseal junction (true Jones fracture)
Type 2 – intra-articular avulsion fracture of 5th metatarsal base
Type 3 – extra-articular avulsion fracture of styloid process of 5th metatarsal base
Type 4 – intra-articular comminuted fracture of 5th metatarsal base
Type 5 – extra-articular avulsion of epiphysis in children

Stewart (Clin Ortho, 1960) – Stewart described only the first 4 types
Type 1 – transverse fracture at the metaphyseal-diaphyseal junction of 5th metatarsal base
approximately 1 cm from the articular cartilage. This is due to rotation of the forefoot
with the base of the 5th metatarsal remaining fixed.
 True Jones fracture (Sir Robert Jones 1902-4 fractures his own)
 This type of injury has a high propensity for non-union
 MOI: internal rotation, PF ankle, and adduction of forefoot
 Tx if non-displaced: BK NWB cast for 4-6 weeks
 Tx if displaced: ORIF

CROZER-KEYSTONE RESIDENCY MANUAL – SECOND EDITION 101


Type 2 – intra-articular avulsion fracture of the 5th metatarsal base (styloid process)
 MOI: shear force. Resulting from contraction of the peroneus brevis.
 Tx if reducible: BK NWB cast for 4-6 weeks
 Tx if non-reducible: ORIF
Type 3 – extra-articular avulsion of the 5th metatarsal base
 ―Tennis fracture‖
 Most common is 5th metatarsal fracture
 MOI: contraction of PB with DF of ankle
 Tx if reducible: BK NWB cast for 4-6 weeks
 Tx if non-reducible: ORIF (possibly tension band wiring)
Type 4 – intra-articular, comminuted fracture of the 5th metatarsal base
 MOI: crush
 Tx: BK NWB cast for 4-6 weeks
 Tx if severely displaced: bone graft and ORIF
Type 5 – extra-articular, avulsion fracture of the epiphysis (in a longitudinal direction)
 Seen in children with open growth plates
 Risk of Iselin AVN
 AKA Salter-Harris type 1
 Tx: BK NWB cast 4-6 weeks

Lawrence – Review Article (Foot Ankle, 1993)


Confusion of 3 fracture
 Jones fracture
 Diaphyseal stress fracture
 Tuberosity avulsion fracture

Shereff (Foot Ankle, 1991)


Spalteholtz tech. of 5th blood supply/nutrient artery proximal and medial 1/3 shaft
On X-ray, fracture heals medially to laterally
Fixation: tension band wire, low profile plate, screws, cross K-wires, 4.5 malleolar screw
Complications: sural nerve entrap
Apophysis fuses at 9-12 years of age

Torg (JBJS, 1984)


Type 1 – acute Jones fracture
Type 2 – delayed union of a Jones fracture or diaphyseal stress fracture
Type 3 – non-union of a Jones fracture or a diaphyseal stress fracture

Champman
Type 1A – Jones fracture
Type 1B – displaced Jones fracture with possible comminution
Type 2 – delayed or non-union of a Jones fracture
Type 3A – avulsion fracture of the styloid
Type 3B – intra-articular fracture of the styloid

102 CROZER-KEYSTONE RESIDENCY MANUAL – SECOND EDITION


Nail Injuries
ROSENTHAL (Ortho Clinics NA, 14(4):695, 1983)
Zone 1 – distal to bony phalanx
Zone 2 – distal to lunula
 Tx: V-Y advancement
Zone 3 – proximal to distal end of lunula
 If nail bed is lacerated, it is considered an open fracture
 Tx: amputation

Metatarsal Head
Freiberg Infarction
Type 1 – metatarsal head dies but heals by replacement. Articular surface preserved.
Type 2 – head collapses but articular surface remains. Peripheral osteophytes (dorsal).
Type 3 – head collapses with articular cartilage loosening. Joint is destroyed.
Type 4 – multiple heads involved

Lisfranc Dislocations
QUENU & KUSS (Rev Chir, 39:281-336,720-91,1093-134, 1909)
Convergent homolateral
 All metatarsals subluxed laterally
 All 5 metatarsals displaced laterally in the transverse plane
Isolateral
 1st metatarsal subluxed medially or metatarsals 2-5 subluxed laterally
 1 or 2 metatarsals displaced laterally in the transverse plane
Divergent
 1st metatarsal subluxed medially and metatarsals 2-5 subluxed laterally
 Displacement in both sagittal and transverse planes

HARDCASTLE (JBJS, 64B:349, 1982)


Type A – total incongruity
A1 – homolateral
A2 – homomedial
Type B – partial incongruity
Bl – partial medial displacement
 1st metatarsal displaced medially and/or in combination with metatarsals 2-4
B2 – partial lateral displacement
 Lateral displacement of one or more lesser metatarsals
Type C – divergent
Cl – partial displacement
 1st metatarsal displaced medially with any combination of metatarsals 2-4
displaced laterally
C2 – total displacement
 1st metatarsal displaced medially with metatarsals 2-5 displaced laterally

CROZER-KEYSTONE RESIDENCY MANUAL – SECOND EDITION 103


Treatment Options
 Cast immobilization (sprains 3-5 weeks)
 Closed reduction and percutaneous pinning
 ORIF

Reduction Sequence
1. First realign 2nd metatarsal on middle cuneiform. Once stabilized, lesser metatarsals will
follow.
2. Next stabilize 1st metatarsal and then lateral metatarsals

Post-op Care
 BK casting for 6 to 12 weeks
 Initial NWB for 6-8 weeks
 Partial WB approximately 6 weeks
 Begin ambulation in stiff-soled shoe
 PT ASAP
 Accommodative orthotics

Complications
 Majority – post-op DJD
 Serious – circulatory compromise

Myerson (Foot Ankle, 6(5):225, 1986)


Type A – total displacement in any plane or direction
Type Bl – medial displacement of 1st metatarsal
Type B2 – lateral displacement affecting 1 or more lesser metatarsals
Type C1 – partial displacement with medial 1st metatarsal and lateral lesser metatarsals
Type C2 – total displacement with a divergent pattern and total incongruity

Navicular Fractures
WATSON-JONES (condensed)
Type 1 – navicular tuberosity fracture
Type 2 – dorsal lip fracture
Type 3 – transverse body fracture
3A – fracture of body without displacement
3B – fracture of body with displacement
Type 4 – stress fracture

WATSON-JONES (Fracture and Joint Injuries, Watson & Jones, 5th ed, p 1200)
Type 1 – navicular tuberosity fracture
 Usually an avulsion fracture by tibialis posterior tendon
 24% of navicular fractures
 Nutcracker fracture – displaced fracture with compression fracture of the cuboid between
4th and 5th metatarsal bases and calcaneus
 MOI: forceful eversion w/medial avulsion of the PT off the tuberosity or

104 CROZER-KEYSTONE RESIDENCY MANUAL – SECOND EDITION


 MOI: direct blow to the tuberosity
 Need to D/Dx Os Tibiale Externum vs. true fracture,
 Best viewed on AP and lateral oblique films
 TX: BK cast with partial WB for 4 weeks
Type 2 – dorsal lip fracture
 Most common
 Tx: BK cast with partial WB for 4-6 weeks
Type 3 – transverse navicular body fracture
3A: without displacement
o Tx: BK walking cast for 6-8 weeks
3B: with displacement
o Tx: ORIF and BK NWB cast for 6-8 weeks
Type IV – stress fracture of the navicular
 Tx if non-displaced: BK NWB cast for 4-6 weeks
 Tx if displaced: ORIF followed by BK NWB x 6-8 weeks

Accessory Navicular – Os Tibiale Externum


Geist (1914), first described by Bahin (1605)
Type l – sesamoid in tendon
Type 2 – articulating os center (Sella Clin Ortho, 1986, Foot Ankle, 1987)
2A – synchondrosis acute angle
2B – synchondrosis obtuse angle
Type 3 – fused accessory os center

Navicular Classifications
Wilson
Chip, comminuted, and crush

Watson Jones
Tuberosity, dorsal lip, and transverse

DePalma
Dorsal lip, avulsion, tuberosity, and fracture dislocation

Rockwood & Green


Body fracture with/without dislocation, chip, and tuberosity

Goldman
Chip, tuberosity, body, displaced, and osteochondral fracture

CROZER-KEYSTONE RESIDENCY MANUAL – SECOND EDITION 105


Rearfoot
Calcaneal Fractures
ROWE (condensed)
Type 1
1A – fracture of plantar tuberosity
1B – fracture of sustentaculum tali
1C – fracture of anterior process
Type 2
2A – ―beak fracture‖
2B – avulsion fracture of Achilles insertion
Type 3
3A – simple oblique fracture of body not involving STJ
3B – comminuted oblique fracture of body not involving STJ
Type 4 – intra-articular fracture involving STJ
Type 5 – intra-articular, comminuted, depression fracture with STJ involvement
Note: Rowe is primarily used for extra-articular fractures. Intra-articular fractures (Rowe 4 & 5)
are usually replaced by Essex & Lopresti.

ROWE (JAMA, 184:98-101, 1963)


Type 1
1A – fracture of the plantar tuberosity due to inverted or everted foot
 Tx of non-displaced: CR and BK WB cast for 6 weeks
 Tx of displaced: ORIF
1B – fracture of the sustentaculum tali due to twist on a supinated foot
 Tx of non-displaced: CR and BK cast for 6 weeks
 Tx of displaced: ORIF
1C – fracture of the anterior tubercle due to plantarflexion on a supinated foot
 Most common type 1 fracture
 Most common in females
 Tx: CR and BK WB cast for 6 weeks. If symptoms persist, excise the fragment
Type 2
2A – ―beak fracture‖ without Achilles insertion involvement
 Tx: NWB BK cast for 6 weeks in plantarflexion
2B – avulsion fracture of the Achilles tendon
 Tx: ORIF or attempt percutaneous pinning
Type 3
3A – simple oblique fracture of body not involving STJ
3B – comminuted oblique fracture of body not involving STJ
 Most common extra-articular
 Tx of non-displaced: NWB AK cast with knee flexed
 Tx of displaced: ORIF
Type 4 – intra-articular fracture involving STJ
Type 5 – intra-articular, comminuted, depression fracture with STJ involvement

106 CROZER-KEYSTONE RESIDENCY MANUAL – SECOND EDITION


ESSEX-LOPRESTI (condensed)
Type 1 – tongue fracture (vertical fracture line) without STJ involvement
Type 2 – joint depression fracture (horizontal fracture line) with STJ involvement

ESSEX-LOPRESTI (Br J Surg, 39:395-419, 1952)


Type 1 – tongue type fracture with a primary fracture line running superior to inferior with a
secondary fracture line exiting the posterior aspect of the calcaneus without STJ
involvement
1A – tuberosity fracture
1B – calcaneal-cuboid joint involvement
Type 2 – joint depression fracture with a primary fracture line running superior to inferior with a
secondary fracture line involving STJ
2A – non-displaced secondary fracture line exits posteriorly
2B – displaced secondary fracture line exits dorsally
2C – gross comminution
** 75% of all calcaneal fractures are intra-articular

Treatment of Intra-articular Fractures


Essex-Lopresti Technique
Percutaneous pinning technique placing a Steinmann pin into the tuberosity. The tongue
fragment is reduced, and a pin is placed into the anterior calcaneus or cuboid. No cast
required, and motion is performed immediately. The pin is removed in 8-10 weeks and
WB is begun. Indicated for Sanders 2C (87% success rate).

Closed reduction
Used if <2 mm displacement

ORIF
Incisional Approaches
 Medial Approach: Burdeaux
 Combined Approach: Stephenson
 Extended Lateral Approach: Benirschke
Procedure
 Goal is to restore the STJ and C-C articulation
 Perform surgery within 6-8 hours of the injury or wait until the swelling is reduced
 Reduction is performed by placing a Steinmann pin through the tuberosity fragment to
restore the STJ posterior facet. Once aligned, the tuberosity fragment is fixated to the
constant fragment (sustentaculum fragment). Various plates can be used as a buttress.
 Before arthrodesis is performed, CR or ORIF should be attempted

DEGAN (J Bone Joint Surg, 64:519, 1982)


Type 1 – non-displaced fracture of the anterior process
Type 2 – extra-articular, displaced fracture of the anterior process
Type 3 – intra-articular, displaced fracture of the anterior process involving C-C joint

CROZER-KEYSTONE RESIDENCY MANUAL – SECOND EDITION 107


SANDERS (Clinics Ortho, 290:87-95, 1993)
 Used for CT evaluation from coronal and axial views
 Classified by number of pieces
 Lines A and B divide the inferior portion of the posterior talar facet into 3 equal portions.
Line C separates the medial and posterior facets.
A – lateral
B – midline
C – medial (at sustentaculum tali)
1 – any number of fracture lines
 All non-displaced, extra-articular fractures
2 – one fracture line
 Two-part fracture of posterior facet
 Use one letter (2A, 2B, or 2C)
3 – two fracture lines
 Three-part, intra-articular fracture of posterior facet with depressed central fragment
 Use two letters (3AB, 3AC, or 3BC)
4 – three fracture lines
 Four-part, intra-articular fracture of posterior facet and sustentaculum fragment with high
degree of comminution

Anterior Calcaneal Process Fractures


Hannover (Clinics Ortho, 290:76-86, 1993)
CT scan evaluation based on fragments involved and number of joint fractures
1 – sustentaculum
2 – tuberosity
3 – STJ
4 – anterior process
5 – anterior STJ
* Most common is the 5 fragment/2 joint fracture

Talar Neck Fractures


HAWKINS (condensed)
Type 1 – vertical fracture of talar neck that is nondisplaced
Type 2 – vertical fracture of talar neck with STJ dislocation/subluxation
Type 3 – vertical fracture of talar neck with STJ and ankle dislocation/subluxation
Type 4 – vertical fracture of talar neck with STJ, ankle, and T-N dislocation/subluxation

HAWKINS (J Bone Joint Surg, 60A:143-156, 1978)


MOI: hyperdorsiflexion of the foot on leg
Type 1 – vertical fracture of talar neck without displacement
 Disruption of 1 blood vessel with 12% risk of AVN
 Tx: BK cast immobilization for 8-12 weeks, NWB for 6-8 wks
 Must have trabeculation across fracture site prior to weightbearing
Type 2 – vertical fracture of talar neck with STJ displacement (ankle joint remains aligned)
 Disruption of 2-3 blood vessels with 42% risk of AVN

108 CROZER-KEYSTONE RESIDENCY MANUAL – SECOND EDITION


 Tx: attempt closed reduction by pushing backward on plantarflexed foot while pulling
forward on the distal tibia. If successful, percutaneous pinning is performed. Cast in
equinus for 4 wks with subsequent casts bringing the foot out of equinus. Requires 3
months of NWB casting.
 Tx: after one unsuccessful attempt at closed reduction, ORIF is indicated. Avoid
multiple attempts at closed reduction. Longitudinal anteromedial incision along the neck
of the talus, just medial to the TA. 6.5 mm cannulated cancellous screws. Use titanium
screws to facilitate the later use of MRI to monitor the progress of osteonecrosis.
Type 3 – vertical fracture of talar neck with STJ and ankle displacement
 Disruption of 3 blood vessels with 91% risk of AVN
 25% are open fractures
 Tx: during ORIF, it is important not to dissect off deep fibers of deltoid ligament which
may remain attached to the talar body (osteotomize the medial malleolus rather than
reflect the deltoid)
Type 4 – vertical fracture talar neck with STJ, ankle joint and talonavicular joint displacement
 Disruption of 3 blood vessels with 91% risk of AVN
 Tx: ORIF

10% incidence of calcaneal fractures associated with talar neck fractures


19-28% incidence of medial malleolar fractures associated with talar neck fractures

X-rays to provide best view of talar neck


 Ankle in maximum equinus
 Foot on cassette pronated 15°
 X-ray tube directed 75° from horizontal

Sclerotic (apparent increase in density) appearance due to surrounding bones becoming


osteoporotic due from disuse and acute hyperemia

Osteonecrosis is the most common complication associated with this injury

Rates of osteonecrosis
 Type 1 (0-13%)
 Type 2 (20-50%)
 Types 3, 4 (83-100%)

MRI can define the presence and extent of osteonecrosis in the talar body as early as 3 weeks

Hawkin sign – presence of subchondral talar dome osteopenia seen 6-8 weeks after talar fracture
signifying intact vascularity. Absence of the sign implies AVN.

Up to 36 months are required for complete creeping substitution of the body after union has
occurred. Protect the patient from WB until complete revascularization occurs. A patellar
tendon brace may partially relieve the load on the talar dome once WB is initiated.

CROZER-KEYSTONE RESIDENCY MANUAL – SECOND EDITION 109


Blair fusion – if the talar dome collapses, excise the avascular talar body and place a sliding
corticocancellous graft from the anterior distal tibia into the residual, viable talar head and neck

Subchondral fenestration to increase vascularity and fibrocartilage production

Talar Body Fractures


SNEPPEN (condensed)
Type 1 – compressive fracture of the talar dome usually involving medial or lateral aspect
Type 2 – shearing fracture of the talar body
2A – coronal shearing force
2B – sagittal shearing force
2C – horizontal shearing force
Type 3 – fracture of the posterior tubercle
Type 4 – fracture of the lateral process
Type 5 – crush fracture

SNEPPEN (Acta Ortho Scand, 45:307, 1974)


Type 1 – transchondral or compression fracture of the talar dome (including osteochondritis of
the talus)
Type 2 – coronal, sagittal, or horizontal shearing fracture involving the entire body
2A – coronal shearing force
2B – sagittal shearing force
2C – horizontal shearing force
 MOA: unknown but thought to be forced dorsiflexion with the foot locked, combined
with axial compression
 Fractures displaced >2-3 mm at trochlear surface require ORIF
 75% incidence of OA of STJ
Type 3 – fracture of the posterior tubercle of the talus
 Shepherd fracture – posterior lateral tubercle fracture
 Sometimes confused with os trigonum. Bone scan can differentiate.
 MOA: hyperplantarflexion or avulsion of posterior talofibular ligament
 Tx: short leg NWB cast with foot in mild equinus. If pain persists, excise fragment.
Type 4 – fracture of the lateral process of the talus
 "Snowborder's ankle"
 MOA: dorsiflexion with inversion
 Tx: 6 weeks of NWB cast immobilization in slight equinus. Large fragments can be
internally fixated.
Type 5 – crush fracture of the talar body
 Poor prognosis
 Primary arthrodesis after 2-3 weeks due to risk of soft tissue envelope if performed
immediately

23% of open talar fractures go on to osteomyelitis and may result in future talectomy

110 CROZER-KEYSTONE RESIDENCY MANUAL – SECOND EDITION


Boyd & Knight (South Med J, 35:160, 1942)
Type 1 – coronal or sagittal shear fracture
1A – non-displaced
1B – fracture with displacement of talo-crural joint
1C – Type 1B with displacement of the STJ
1D – fracture with total displacement of the talar body
Type 2 – horizontal shear fracture
2A – non-displaced
2B – displaced

Talar Dome Fractures


BERNDT & HARDY (condensed)
Stage 1 – nondisplaced compression of talar dome
Stage 2 – partially detached osteochondral lesion
Stage 3 – completely detached, nondisplaced osteochondral lesion
Stage 4 – completely detached, displaced osteochondral lesion

BERNT & HARDY (J Bone Joint Surg, 41A:988-1020, 1959)


Mechanism: – DIAL A PIMP
Stage 1 – nondisplaced, subchondral compression of the talar dome
 Tx: conservative, off-loading patellar tendon brace
Stage 2 – partially detached, nondisplaced osteochondral fracture
 Tx: conservative, off-loading patellar tendon brace
Stage 3 – completely detached, nondisplaced osteochondral fracture
 Tx medial lesion: conservative, off-loading patellar tendon brace
 Tx lateral lesion: surgical excision of the fragment, saucerization of crater, and
fenestration to increase vascularity and fibrocartilage production
Stage 4 – completely detached, displaced osteochondral fracture
 Tx: surgical excision of the fragment, saucerization of crater, and fenestration to increase
vascularity and fibrocartilage production

Mechanism of Injury: DIAL a PIMP


DIAL – Dorsiflexion and Inversion → Anterior-Lateral lesion
 Wafer-shaped lesion, associated with trauma
PIMP – Plantarflexion and Inversion → Posterior-Medial lesion
 Small, deep, round cup-shaped fragment, 80% not associated with trauma

Stages 2-4 require lateral ankle ligament disruption to occur

Treatments
 Tx of Stages 1, 2, and medial 3: NWB Short leg cast for 6-12 wks
 Tx of lateral stage 3 and 4: surgical excision of the fragment, saucerize the crater, and

CROZER-KEYSTONE RESIDENCY MANUAL – SECOND EDITION 111


Lateral Talar Process Fractures
Hawkins (J Bone Joint Surg, 47A:1170, 1965)
Type 1 – simple fracture of lateral process that extends from talofibular articular surface down to
posterior talocalcaneal articular surface of the STJ
Type 2 – comminuted fracture of lateral process that involves both fibular and posterior
calcaneal articular surfaces of the talus and the entire lateral process
Type 3 – chip fracture of anterior and inferior portions of posterior talar articular process

Posterior Lateral Talar Process Fractures


Dobas & Watson (Arch Pod Med Foot Surg, 3:17, 1976)
Stage 1 – normal posterior lateral process; no clinical significance
Stage 2 – enlarged posterior lateral process
Stage 3 – non-fused os trigonum
Stage 4 – synchondrosis of the os trigonum to the talus

McGougall (J Bone Joint Surg, 37B:257-265, 1955)


Stage 1 – line of cleavage occurs at impingement point
Stage 2 – posterior lateral process begins to separate from the main body of the talus
Stage 3 – complete separation of the posterior lateral process from the talar body

STJ Dislocations
Buckingham
Type 1 – medial STJ dislocation (FF moves medially and talar head moves laterally)
Type 2 – lateral STJ dislocation
Type 3 – anterior and posterior STJ dislocation

Tarsal Coalitions
DOWNEY (JAPMA, 81:187-197, 1991)
Juvenile (Osseous Immature)
Type 1 – extra-articular coalition
A – no secondary arthritis
 Tx: Badgley procedure
B – secondary arthritis
 Tx: resection or triple
Type 2 – intra-articular
A – no secondary arthritis
 Tx: resection, isolated arthrodesis, or triple
B – secondary arthritis
 Tx: triple

Adult (Osseous Mature)


Type 1 – extra-articular
A – no secondary arthritis
 Tx: resection or triple

112 CROZER-KEYSTONE RESIDENCY MANUAL – SECOND EDITION


B – secondary arthritis
 Tx: triple
Type 2 – intra-articular
A – no secondary arthritis
 Tx: isolated or triple
B – secondary arthritis
 Tx: triple

Chopart Fractures
Main & Jowett (JBJS, 57B:89, 1975)
Classification based on direction of deforming force and resulting displacement
Medial force
Type A – flake fracture of dorsal talus or navicular and of the lateral calcaneus or cuboid
Type B – medial displacement of forefoot with medial disassociation of T-N and C-C
joints
Type C – forefoot rotates medially around interosseous talocalcaneal ligament, with T-N
disassociation and intact C-C joint
Longitudinal force
Type A – maximally plantarflexed ankle giving a characteristic pattern of through and
through navicular compression fracture
Al – force through the 1st ray crushes medial 3rd with tuberosity displaced
medially
A2 – force through the 2nd ray crushes middle 3rd with middle 3rd and tuberosity
displaced medially
A3 – force through the 3rd ray crushes lateral 3rd with medial 2/3rd and tuberosity
displaced medially
Type B – submaximally plantarflexed ankle resulting in dorsal displacement of the
superior navicular and crush of the inferior portion
Lateral forces
Type A – forefoot forced into valgus with resulting fracture of the navicular tuberosity or
dorsal talus and a compression fracture of the C-C joint (Nutcracker Fracture)
Type B – T-N joint displaces laterally with comminution of the C-C joint
Plantar forces
Type A – avulsion fracture of the dorsal navicular to talus and the anterior process
Type B – impaction fracture of the inferior C-C joint

Ankle Fractures
LAUGE-HANSEN
1st word – position of the foot with respect to the leg
2nd word – motion that causes fracture pattern (how talus moves with respect to tibia/fibula)
* indicates hallmark sign

Supination-Adduction (SAD) – No tib-fib diastasis


Stage l – rupture of lateral collaterals or * transverse fracture of lateral malleous below level of
ankle joint
Stage 2 – * vertical fracture of medial malleolus

CROZER-KEYSTONE RESIDENCY MANUAL – SECOND EDITION 113


Pronation-Abduction (PAB)
Stage 1 – rupture or deltoid ligament or transverse avulsion fracture of medial malleolus
Stage 2 – rupture of anterior and/or posterior distal tib-fib ligaments
Stage 3 – * short fibular fracture (oblique on AP, trans, on lateral) at level of ankle joint

Supination-Eversion (SER) – most common


Stage 1 – disruption of anterior tib-fib ligament with either a tibial avulsion (Tillaux-Chaput) or a
fibular avulsion (Wagstaffe)
Stage 2 – * spiral oblique fracture of the fibula at level of ankle joint
Stage 3 – rupture of posterior tib-fib ligament or tibial avulsion (Volkmann)
Stage 4 – ruptured of deltoid or transverse fracture of medial malleolus

Pronation-External Rotation (PER)


Stage 1 – rupture of deltoid ligament or transverse avulsion of med malleolus
Stage 2 – disruption of anterior tib-fib ligament with rupture of interosseous ligament and
Tillaux-Chaput or Wagstaffe avulsions
Stage 3 – * high fibular fracture above level of ankle joint (Maisonneuve fracture)
Stage 4 – Posterior tib-fib ligament or Volkmann fracture

Pronation-Dorsiflexion (Arch Surg, 67:813-820, 1953)


Describes pilon fracture
Stage 1 – rupture of deltoid ligament or fracture of medial malleolus (oblique or transverse)
Stage 2 – fracture of anterior lip of tibial plafond
Stage 3 – fracture of fibula above the level of the syndesmosis
Stage 4 – transverse fracture of the distal tibia at the same level as the proximal margin of the
large tibial fracture

DANIS WEBER (condensed)


Describes location of fibular fracture
Type A – transverse avulsion fracture below the level of the ankle joint
(corresponds with Lauge-Hansen SAD)
Type B – spiral or oblique fracture at the level of the ankle joint
(corresponds with Lauge-Hansen SER and PAB)
Type C – fracture above the level of the ankle joint (Maisonneuve fracture)
(corresponds with Lauge-Hansen PER)

DANIS-WEBER (Ortho Clinics of NA, 661, 1980)


Type A – transverse avulsion fracture of fibula below the level of ankle mortise
 MOI: SAD
 Tx: K-wire w/ tension band for fibular fracture and 2 interfrag screws for the med
malleolus
Type B – fracture at the level of ankle mortise
 MOI: PAB or SER
 Tx: interfrag screws and/or plate, repair ATFL

114 CROZER-KEYSTONE RESIDENCY MANUAL – SECOND EDITION


Type C – fracture above the level of ankle mortise
 MOI: PER
 Tx: interfrag screws and plate, repair ATFL and interosseous membrane

Pilon Fractures
Lauge-Hansen
Pronation-Dorsiflexion (Arch Surg 67:813-820, 1953)
Stage 1 – deltoid ligament rupture or medial malleolar fracture (oblique or transverse)
Stage 2 – fracture of the anterior lip of the tibial plafond
Stage 3 – fibular fracture above the level of the syndesmosis
Stage 4 – transverse fracture of distal tibia at level of proximal margin of the large tibial fracture

RUEDI & ALLGOWER condensed


Pilon fractures – distal tibial metaphyseal fracture
Type 1 – non-displaced tibial fragments
Type 2 – intra-articular tibial fracture without comminution
Type 3 – comminution and disruption of tibial articular surface

Ruedi & Allgower (Clin Ortho, 138:105-110, 1979)


Type 1 – mild displacement, no comminution, without major disruption of ankle joint
Type 2 – moderate displacement, no comminution, with significant dislocation of ankle joint
Type 3 – "explosion fracture", severe comminution, with displacement of distal tibial metaphysis

Femoral distractor – brings tibia out to length before fixation


If Type 3 fracture, fix tibia first
40-80% failure of ankle fusion

Maale and Seligson (Orthopedics, 3:517-521, 1980)


Modification of Ruedi & Allgower
Type 1 – distal tibial compression fracture
Type 2 – external rotatory fracture with a large posterior fragment
Type 3 – spiral fracture extending from the articular surface into the metaphysis

Ovadia and Beals (J Bone Joint Surg, 68A:543-551, 1986)


Modification of Ruedi & Allgower Type II
Type 1 – non-displaced articular fracture resulting from rotational forces
Type 2 – minimally displaced fracture resulting from articular forces
Type 3 – displaced articular fracture with several large fragments due to compressive forces
Type 4 – displaced articular fracture with multiple fragments including a large metaphyseal
fragment resulting from compressive forces
Type 5 – severe comminution due to compressive forces

CROZER-KEYSTONE RESIDENCY MANUAL – SECOND EDITION 115


AO System (Ankle Fractures)
Type A – extra-articular
Type B – partially articular
Type C – completely articular
All 3 can involve:
 No comminution or impaction in the articular or metaphyseal surface
 Impaction involving the supra-articular metaphysis
 Comminution and impaction involving the articular surface with metaphyseal impaction

Destot System
Subgroup 1 – posterior marginal tibial fracture
Subgroup 2 – anterior marginal tibial fracture
Subgroup 3 – explosive tibial fracture
Subgroup 4 – supra-articular tibial fracture with extension into the ankle joint

Kellam and Waddell (J Trauma, 19:593-601, 1979)


Type A – rotational pattern consisting of two or more large tibial articular fragments, minimal or
no anterior cortical comminution, and a transverse or short oblique fibular fracture at the
level of the tibial plafond
Type B – compressive fracture pattern with multiple tibial fragments and marked anterior tibial
cortical comminution

Mast (Clinics of Ortho, 230:68-82, 1988)


Type 1 – malleolar fracture with significant axial load at the time of the injury producing a large
posterior fragment
Type 2 – spiral extension fracture
Type 3 – centrally compressive injury divided into A, B and C

Medial Malleolar Fractures


Muller
Type A – avulsion of the tip of the medial malleolus, horizontal orientation
Type B – avulsion fracture at the level of the ankle joint, horizontal orientation
Type C – oblique fracture
Type D – vertical fracture

Fibular Avulsion Fractures


Pankovich: Wagstaffe-LeFort Fracture (Clinics Ortho Rel Res, 143:138, 1979)
Type 1 – avulsion fracture maintaining attachment to both the anterior talofibular and anterior-
inferior tib-fib ligaments
Type 2 – avulsion fracture associated with an oblique fracture of the fibula originating distal to
the anterior-inferior tib-fib ligament. Spiral fracture of the fibula with a proximal fibular
spike and a transverse fracture associated with the avulsion fragment.
Type 3 – avulsion fracture of the anterior tibial tubercle followed by a Type 2

116 CROZER-KEYSTONE RESIDENCY MANUAL – SECOND EDITION


Lateral Ankle Trauma
Leach
1st Degree – rupture of the ATF
2nd Degree – rupture of the ATF and CF
3rd Degree – rupture of the ATF, CF, and PTF

O'Donoghue (condensed)
Grade 1 – partial ATF tear
Grade 2 – complete ATF tear
Grade 3 – complete ATF and CFL tear

O'Donoghue (Northwest Med, 1277, 1958)


Grade 1 – partial ATF tear with mild tenderness and swelling
 No loss of function or instability. Pt can walk, play.
Grade 2 – complete ATF tear with moderate pain, swelling, ecchymosis
 Some loss of function and moderate instability. Pt limps after injury.
Grade 3 – complete ATF and CFL tear of with severe pain, swelling, and ecchymosis
 Unable to bear weight and severe instability. Pt cannot walk after injury.

Dias (J Trauma, 19:266-269, 1979)


Grade 1 – partial rupture of the CFL
Grade 2 – rupture of the ATF
Grade 3 – complete rupture of the ATF, CF, and/or PTF
Grade 4 – rupture of all lateral collateral ligaments and partial failure of the deltoid ligament

Posterior Tibial Malleolar Fracture


Volkmans
Type A – large intra-articular fracture (>25% of surface area) with displacement
Type B – small intra-articular fracture (<25% of surface area) with impaction
Type C – small fracture with minimal impaction and articular damage
Type D – avulsion of posterior-inferior tib-fib ligament without articular involvement

Physeal Injuries
Salter-Harris Classification of Fractures
Site – epiphysis, metaphysis, diaphysis
Extent – complete vs. incomplete
Configuration – transverse, oblique, spiral, comminuted
Position – rotated, angulated, distracted, impacted, overriding, lateral shift
Environment – open, closed

CROZER-KEYSTONE RESIDENCY MANUAL – SECOND EDITION 117


SALTER-HARRIS
(SMACK – Same, Metaphysis, Articulation, Continuous, Krush)
(SALTR – Same, Above, Lower, Through, Really bad)
1 – fracture through physis
2 – fracture through physis into metaphysis
3 – intra-articular fracture through physis into epiphysis
4 – intra-articular fracture through epiphysis, physis, and metaphysis
5 – crush injury

SALTER-HARRIS (Skeletal Rad, 6:237-253, 1981)


Type 1 – complete transverse separation of the epiphysis from the metaphysis through the physis
 Epiphysis separates from the metaphysis without any bone fragments
 Germ cells remain with epiphysis
 Common in infants
 Shearing force seen in pathologic fractures
 Growth is not disturbed unless there is associated avascular necrosis or premature closure
of the physis
 Tx: closed reduction if within 7 days of injury, followed by 3-4 weeks of casting
Type 2 – fracture through the physis extending into the metaphysis
 Thurston-Holland sign
 Tx: closed reduction if within 7 days of injury, followed by 3-4 weeks of casting
Type 3 – intra-articular fracture extending from the physis through the epiphysis
 Tillaux fragment
Type 4 – continuous, intra-articular fracture extending from the epiphysis into the physis and
metaphysis
Type 5 – comminuted fracture from impaction of the epiphysis into the physis and metaphysis
 Tx of Types 3-5: attempt closed reduction, but usually requires anatomic reduction of the
physis. Fixation should be kept within metaphysis.

Rang
Type 6 – perichondral injury produced by shearing force resulting in a cup-shaped fragment of
epiphyseal, physeal, and metaphyseal bone with possible degloving
 Tear of the ―ring of Lacroix‖

Ogden
Type 7 – intra-epiphyseal fracture not involving the physis
Type 8 – transverse fracture of the metaphysis only
Type 9 – diaphyseal growth injury resulting in periosteal elevation and possible degloving of the
periosteum

Poland
Type 1 – separation of the epiphysis from the metaphysis
Type 2 – partial separation of the epiphysis from the metaphysis with fracture of the diaphysis
 Thurston-Holland sign
Type 3 – partial separation of epiphysis from the metaphysis with fracture of the epiphysis
Type 4 – complete separation of the epiphysis from metaphysis with fracture of the epiphysis

118 CROZER-KEYSTONE RESIDENCY MANUAL – SECOND EDITION


Peterson (J Ped Ortho, 14:439, 1994)
Type 1 – transverse fracture of the metaphysis with extension to the physis by longitudinal
compression (15.5%)
Type 2 – separation of part of the physis with a part of the metaphysis attached (53.6%)
 Thurston-Holland sign
 Salter-Harris Type 2
Type 3 – separation of the epiphysis from the diaphysis through the physis (13.2%)
 Salter-Harris Type 1
Type 4 – separation of a portion of the physis with extension of a fracture into the joint (10.9%)
 Salter-Harris Type 3

Type 5 – fracture involving the metaphysis, physis, and epiphysis (6.5%)


 Salter-Harris Type 4
Type 6 – fracture involving a missing portion of the physis.
 Often caused by open fractures, lawn mowers, farm machinery, or other power
equipment

Weber
Type A – extra-articular
Al – separation of the epiphysis and metaphysis
A2 – fragments in the epiphysis or metaphysis
Type B – intra-articular
Bl – fracture within the physis extending into the epiphysis
B2 – fracture through the epiphysis, physis, and metaphysis

MRI Rupture
CONTI
Stage 1 – 1-2 fine, longitudinal tears
Stage 2 – intramural degeneration, variable diameter
Stage 3 – diffuse swelling

Achilles Tendon Ruptures


KUWADA
Type 1 – partial tear <50%
 Tx: cast with foot plantarflexed
Type 2 – complete tear with <3 cm defect after debridement
 Tx: end-to-end attachment
Type 3 – complete tear with 3-6 cm defect after debridement
 Tx: end-to-end attachment and tendon flap
Type 4 – complete tear with >6 cm defect after debridement
 Tx: end-to-end attachment, recession, or graft

CROZER-KEYSTONE RESIDENCY MANUAL – SECOND EDITION 119


Radio-opaque Lesions of Achilles Tendon
Morris & Giacopelli (J Foot Surg, 1990)
Type 1 – opacities at the Achilles insertion with calcifications within tendon and partially
attached to the calcaneus
Type 2 – opacities l-3 cm proximal to insertion with lesions separate from calcaneus
Type 3A – opacities > 3 cm proximal to insertion with partial tendon calcification
Type 3B – opacities > 3 cm proximal to insertion with total tendon involvement

Peroneal Tendon Subluxations


Eckert & Davis (J Bone Joint Surg, 58A:670, 1976)
Grade 1 – retinaculum and periosteum ruptures from the cartilaginous lip and lateral malleolus
Grade 2 – distal edge of fibrous lip elevated with retinaculum
Grade 3 – thin fragment of bone with fibrous lip avulsed from deep surface of peroneal
retinaculum and deep fascia
Posterior Tendon Ruptures
MUELLER
1 – direct injury
2 – pathologic rupture (RA)
3 – idiopathic
4 – functional abnormality

Posterior Tibial Tendon Dysfunction


Johnson & Strom
Stage 1 – normal tendon length with mild degeneration
 Medial foot and ankle pain
Stage 2 – supple flatfoot with attenuation or PT rupture
 ―Too many toes‖ sign
 Abducted forefoot, increased talar-1st metatarsal angle, and uncovering of talar head
Stage 3 – rigid flatfoot with complete PT rupture
 Fixed calcaneal valgus with decreased STJ ROM
Stage 4 – rigid flatfoot
 Valgus tilt of talus/ankle mortise leading to lateral tibial/talar degeneration

Wilson (J Bone Joint Surg, 54B:677, 1972)


Inversion Injury
Stage 1 – lateral dislocation of 4 lesser metatarsals with divergent diastasis
Stage 2 – Stage 1 with dorsolateral dislocation of 1st metatarsal and other metatarsals

Eversion Injury
Stage 1 – medial dislocation of 1st metatarsal
Stage 2 – dorsolateral dislocation of lesser 4 metatarsals with divergent diastasis

120 CROZER-KEYSTONE RESIDENCY MANUAL – SECOND EDITION


Other Classifications
Open Fractures
GUSTILLO & ANDERSON (condensed)
Type 1 – wound <l cm without extensive soft tissue damage
Type 2 – wound >l cm without extensive soft tissue damage
Type 3 – extensive skin, soft tissue, muscle, and neurovascular damage
3A – adequate tissue coverage, high energy trauma
3B – periosteal stripping, massive comminution
3C – arterial injury

GUSTILLO & ANDERSON (J Bone Joint Surg, 58A:453, 1976)


Type 1 – open fracture with wound <1 cm without extensive soft tissue damage
 Simple, transverse or short oblique fracture with little comminution
 No crush involved
Type 2 – open fracture with a laceration >1 cm without extensive soft tissue damage
 Slight or moderate crushing injury with moderate comminution
 Moderate contamination
 Minimal foreign material
Type 3 – open fracture with extensive soft tissue damage >5 cm
 Severe comminution associated with high velocity injury
 High degree of contamination
 Significant foreign material
 Gunshot wounds, farm injuries, arterial injuries, motor vehicle accidents
3A – adequate soft tissue coverage
3B – extensive soft tissue loss/damage with periosteal stripping and bone exposure requiring
local or free flap
3C – any open fracture associated with arterial injury requiring repair. Amputation rate of
25-90%.

Non-Union of Fractures
WEBER & CECH
Hypertrophic – hypervascular (90%)
 Elephant foot
 Horse hoof
 Oligotrophic
Atrophic – avascular (10%)
 Torsion wedge
 Comminuted
 Defect
 Atrophic

CROZER-KEYSTONE RESIDENCY MANUAL – SECOND EDITION 121


Ulcerations
WAGNER (Foot Ankle, 2:64-122, 1981)
Grade 0 – no open lesions but bony prominence and/or structural deformity present
Grade 1 – superficial ulcer without penetration to the deep layers
Grade 2 – deep ulcer penetrating to tendon, joint capsule, or bone
Grade 3 – Grade 2 depth with the presence of infection
Grade 4 – gangrene of the forefoot
Grade 5 – gangrene of the entire foot

Wound, Ostomy and Continence Nurses Society (formerly I.A.E.T., Standards of Care, 1987)
Stage 1 – nonblanchable erythema of intact skin
Stage 2 – partial thickness loss of skin involving epidermis, dermis, or both.
 Ulcer is superficial and presents clinically as a blister or shallow crater with erythema and
induration
Stage 3 – full-thickness tissue loss involving damage to or necrosis of subcutaneous tissue that
many extend down to, but not through underlying fascia
 Ulcer presents clinically as deep crater often with undermining, erythema, and drainage
Stage 4 – full-thickness tissue loss with extensive destruction, tissue necrosis, or damage to
muscle, bone or supporting structures (e.g. tendon, joint capsule)
 Undermining and sinus tracts often associated

University of Texas at San Antonio – UTSA (J Foot Ankle Surg, 35:528-531, 1996)
Grade 0 – completely epithelialized pre- or post-ulcerative lesion
Grade 1 – superficial wound not involving tendon, capsule, or bone
Grade 2 – wound penetrating to tendon or capsule
Grade 3 – wound penetrating to bone or joint
Within each grade, there are 4 subtypes:
A – non-ischemic, clean wound
B – infected wound
C – ischemic wound
D – infected and ischemic wound

Diabetic Foot Ulcers


Meade & Mueller (Med Times, 96:154-169, 1968)
Type 1 – diffuse, inflammatory infection of soft-tissue
Type 2 – deep plantar space infection
Type 3 – mal perforans neuropathic foot ulcers (subclassed by Wagner and USATHC)

Charcot
Sanders & Freykberg
1 – IPJ, phalanx, MPJ, and metatarsals
2 – Lisfranc
3 – C-N, T-N, and C-C
4 – ankle
5 – calcaneus

122 CROZER-KEYSTONE RESIDENCY MANUAL – SECOND EDITION


Osteomyelitis
Waldvogel
Hematogenous – spread via blood starting inside the bone and working out towards the cortex
 Seen most commonly in metaphyseal region of children with open growth plates
Direct extension – secondary to trauma or surgery first affecting periosteum, then cortex, and
then marrow
 Proteolytic enzymes destroy Sharpey fibers
Contiguous – spread of infected soft tissue to underlying bone
Vascular insufficiency – PVD

Cierny & Mader


Anatomic Type
Medullary
Superficial
Localized
Diffuse
Physiologic Type
A-Host – good immune system and delivery
 Normal immune response
 Normal metabolic reserve
 Good vascular supply
B-Host – compromised locally or systemically
 Metabolic compromise
 Nutritional compromise
 Immunologic compromise
 Impaired vascularity
 Systemic illness
C-Host – no treatment because treatment is worse than disease
 Minimal disability
 High morbidity
 Poor prognosis for cure

Buckholz (J Foot Surg, 26(1):17, 1987)


Type 1 – wound induced osteomyelitis
1A – open fracture with complete incontinuity
1B – penetrating wound of injury
1C – post-op infections
Type 2 – mechanogenic osteomyelitis
2A – implants and internal fixation
2B – contact instability as bone-to-bone appositional movement
Type 3 – physeal osteomyelitis
Type 4 – ischemic limb disease
Type 5 – combination osteomyelitis, Types 1-4 as acute bone infections
Type 6 – osteomyelitis with septic arthritis
Type 7 – chronic osteomyelitis

CROZER-KEYSTONE RESIDENCY MANUAL – SECOND EDITION 123


Medicare PVD Classification
Class A
A1 – nontraumatic amputation of foot or integral portion thereof
Class B
B1 – absent posterior tibial pulse
B2 – advanced trophic changes such as (3 required)
2a – hair growth diminished or absent
2b – nail changes (thickened)
2c – pigmentary changes
2d – skin texture (thin, shiny)
2e – skin color (rubor, redness)
B3 – absent dorsalis pedis pulse
Class C
C1 – claudication
C2 – temperature changes (cold feet)
C3 – edema
C4 – parathesias
C5 – burning

Soft Tissue Injuries


Tscheme & Gotzen
Grade 0 – little or no soft tissue injury
Grade 1 – significant abrasion or contusion
Grade 2 – deep, contaminated abrasion with local contusion to skin or muscle
Grade 3 – extensive contusion or crushing of skin or destruction of muscle

Malignant Melanoma
Clark (Cancer Res, 29:705-727, 1969)
Based on histological level of invasion
Level 1 – located within epidermis or epidermal-dermal junction
Level 2 – located within papillary dermis
Level 3 – located within papillary-reticular junction
Level 4 – located down into reticular dermis
Level 5 – located within subcutaneous tissue

Breslow (Ann Surg, 172:902-908, 1970)


Based on thickness
Level 1 – <0.75 mm (99% cure)
Level 2 – 0.76-1.5 mm
Level 3 – 1.51-4.0 mm
Level 4 – >4.0 mm

124 CROZER-KEYSTONE RESIDENCY MANUAL – SECOND EDITION


Polydactyly
Blauth & Olason
Radiographic and morphological presentation of the deformity
Describes position of duplication in both the longitudinal and transverse planes
Longitudinal Type – describes degree of duplication of the ray from distal to proximal with a
division into 5 types:
 Distal phalanx
 Middle phalanx
 Proximal phalanx
 Metatarsal
 Tarsal
Transverse Type – indicates which rays are involved in the duplication
 Classification in Roman numerals starting with the 1st ray and ending with the 5th ray

Temtamy & McKusick


Preaxial – located on the medial side of a line that bisects the 2nd digit (15%)
Postaxial – located on the lateral side (80%)
Type A – fully developed
Type B – vestigial duplication involving soft tissue

Venn & Watson


Based on degree of differentiation
 Wide metatarsal head
 T-metatarsal
 Y-metatarsal
 Complete duplication

Syndactyly
Davis & German
Type 1 – Incomplete webbing between two digits
Type 2 – Complete webbing to ends of digits
Type 3 – Simple with no phalangeal involvement
Type 4 – Complex with abnormal phalangeal bones

CROZER-KEYSTONE RESIDENCY MANUAL – SECOND EDITION 125


126 CROZER-KEYSTONE RESIDENCY MANUAL – SECOND EDITION
Special Studies
Bone Scan
What are the phases of bone scan? When is each phase done?
Phase 1 – Immediate, early, blood flow, or angiogram (it goes by all these names)
 2-3 seconds
Phase 2 – Blood pool
 2-3 minutes
Phase 3 – Delayed
 2-3 hours
Phase 4 – Fourth phase
 24 hours

What do each of the phases of the bone scan test for?


Immediate – Blood flow
Pool – Soft tissue
Delayed – Bone activity
Fourth phase – Bone uptake for patient with PVD

What normally lights up on a bone scan?


Epiphysis of a growing child
Fracture
Tips of scapula
Bladder
Sternum
Intercostals (ribs)

What is the half-life of Technetium-99?


6 hours

What does it mean if the bone scan lights up in Phases 1-2 but not in 3?
Cellulitis most likely

Name a way to test between Charcot disease and osteomyelitis


Ceretec scan or Indium-111

What does Indium-111 tag?


WBC’s (as does the Ceretec scan)

What does Indium-111 test for?


Highly sensitive and specific for acute soft tissue and osseous infections

What does Gallium-67 test for?


Acute inflammation and infection

CROZER-KEYSTONE RESIDENCY MANUAL – SECOND EDITION 127


How long does it take for a Gallium-67 test to work?
2-3 days
Note: it’s expensive

Why would you use a Technetium-99 scan with a Gallium-67 scan?


Acute Osteo Chronic Osteo Septic Arthritis
Tech-99
Phase 1 + +/- +++
Phase 2 ++ + +++
Phase 3 +++ +++ +/-
Gallium-67 Positive focal uptake Negative Positive focal uptake
Indium-111 Positive Negative Positive

Fracture Acute Cellulitis Charcot


Tech-99
Phase 1 +++ +++ +/-
Phase 2 +++ ++ +/-
Phase 3 +++ + +++
Phase 4 ++/+++
Gallium-67 Negative Positive diffuse uptake Negative
Indium-111 Negative Positive Negative

MRI
What causes increased signal intensity on a T1 image?
Fat

What causes increased signal intensity on a T2 image?


Fluid, Infection, Inflammation, Tumor (F.I.I.T.)

For MRI, what are the main indications for STIR imaging?
It is useful for evaluation of edema in high lipid regions, such as bone marrow.
It is also useful for evaluating cartilage.

What is fat saturation used for?


Evaluation of fat…c’mon, that’s obvious

What is Gradient Echo also known as?


Steady State Magnetization

What is Gradient Echo used for?


Joint imaging

128 CROZER-KEYSTONE RESIDENCY MANUAL – SECOND EDITION


What are 2 uses for Gadolinium?
Intravenously
 It will be distributed to places with increased vascularity, such as neoplasms and
inflammation
 Cellulitis and walls of abscesses will enhance, but the pus will not.
Intra-articular
 Tests cartilage integrity

What will a stress fracture show up as on MRI?


T1
 Linear zone of decreased signal intensity surrounded by a less defined area of signal
intensity
T2
 Linear zone of decreased signal intensity surrounded by an increased signal intensity due
to edema
STIR
 Increased signal intensity because fatty bone marrow is suppressed

How will osteomyelitis show up on MRI?


T1
 Break in cortex, decreased signal in the bone marrow
T2
 Break in cortex, increased signal in the bone marrow

How will AVN show up on MRI?


T1 and T2
 Decreased signal intensities
STIR and Long T2
 Double rim sign: Inner margin will show an increased signal intensity (this represents
granulation tissue). Outer margin will show decreased signal intensity (this shows
mineralization).

What does MRA stand for?


Magnetic Resonance Angiography

What is MRA used for in the LE?


PVD, DVT, neoplasm and anatomic studies
Most commonly ordered by a vascular surgeon for further description of occlusions/stenosis

CROZER-KEYSTONE RESIDENCY MANUAL – SECOND EDITION 129


CT
What are the 3 planes of a CT Scan?
Coronal
Axial
Sagittal

Which of these planes is computer reconstructed?


Sagittal

What does the coronal plane of a CT scan represent?


Frontal plane
(Memorization tip! 1st vowel in coronal and frontal is ―O‖)

What does the axial plane of a CT scan represent?


Transverse plane
(Memorization tip! 1st vowel in transverse and axial is ―A‖)

Miscellaneous
What must you D/C before an A-gram?
Glucophage because patient may develop metabolic acidosis

What are some tests for sickle cell anemia?


Microscope and observe
Hemoglobin electrophoresis

How many phases in a Ceretec scan?


One!

What does HMPAO stand for?


Hexylmethypropyleneamineoxime (a.k.a. Ceretec scan)

What does MDP stand for?


Methyldiphosphate

130 CROZER-KEYSTONE RESIDENCY MANUAL – SECOND EDITION


Name That Surgery!
Note: This section has a variety (but incomplete list) of surgeries with some indications and brief
descriptions to help you sift through the vast amounts of procedures that are named after their
creators. If you’re going to interview or rotate with a program that has an attending in which a
procedure is named after, please know more than what is listed here! Asterisks are next to the
most common procedures you should know.

1st Ray & Tailor Bunion Procedures


* Akin
Indications: Large DASA → Proximal Akin
Long proximal phalanx → Central Akin
High hallux abductus angle >15° → Distal Akin
Procedure: Medially based wedge osteotomy of the proximal phalanx

* Austin (a.k.a. Distal Chevron Osteotomy)


Note: If you don’t know this one, stop reading this book and find a paper bag to cover your head
in shame.
Indications: HAV (IMA 12°-14°)
Procedure: V-shaped osteotomy with the apex in the center of the metatarsal head and the
arms forming a 60° angle

* Cheilectomy
Indications: Hallux limitus
Procedure: Removal of the dorsal bone spur and dorsal ⅓ of the 1st metatarsal head
OPTIONAL – Removal of bony prominences from proximal phalanx base

* Closing Base Wedge Osteotomy (CBWO, a.k.a. Louisan-Balaceau)


Indication: HAV (high IMA)
Procedure: Closing wedge osteotomy straight across the base of 1st metatarsal (difficult to
fixate)

* Crescentic (a.k.a. Weinstock or Arcuate)


Indication: HAV (IMA >13°)
Procedure: Crescentic osteotomy, (with crescentic blade) concavity directed proximally

DRATO
Indications: Large 1st IMA + Abnormal PASA + Valgus rotation of 1st metatarsal
Procedure: Derotational osteotomy of the 1st metatarsal head (vertical cut through metatarsal
head, cartilage is rotated for realignment, very unstable)

CROZER-KEYSTONE RESIDENCY MANUAL – SECOND EDITION 131


Hohmann
Note: Reverse Hohmann used for Tailor’s bunion
Indications: HAV
Procedure: Through and through transverse osteotomy at the metatarsal neck (unstable
osteotomy)

* Juvara
Indications: HAV (IMA >15°)
Procedure: Oblique CBWO (apex prox-med, wedge laterally with the base ending in mid ⅓
of the metatarsal, direction allows for better fixation)

* Kalish
Indications: HAV (IMA ≤15°)
Procedure: Similar to Austin but with a long dorsal arm for screw fixation (angle reduced to
55° between arms)

* Keller
Note: Used in patients >50-55 years old
Indications: HAV (IMA 16° or less) + Hallux limitus/rigidus
Procedure: Resection of the proximal ¼ to ⅓ base proximal phalanx (⅓ more commonly, cut
perpendicular to long axis of bone), and cheilectomy with capsular tissue sewn
into 1st MPJ space

Kessel-Bonney
Indication: Hallux limitus
Procedure: Removal of a pie-shaped dorsiflexory wedge of bone from proximal phalanx

Lambrinudi
Indication: Hallux limitus
Procedure: Plantarflexory wedge osteotomy of 1st metatarsal base

* Lapidus
Indication: HAV + 1st ray hypermobility
Procedure: Fusion of 1st metatarsal base to medial cuneiform (with the resections of bone
angled to correct the deformity)

Logroscino
Indications: HAV (IMA ≥15° in rectus foot, 13° with adductus) + Abnormal PASA
Procedure:
1. CBWO (or Crescentic) → to correct HAV
2. Reverdin (or Peabody) → to correct cartilage orientation

Loison
Indications: HAV
Procedure: Transverse CBWO

132 CROZER-KEYSTONE RESIDENCY MANUAL – SECOND EDITION


Ludloff
Note: Opposite orientation to Mau
Indication: HAV
Procedure: Oblique bone cut diagonally (dorsal-prox to plantar-dist) through the 1st
metatarsal. Transpositional osteotomy.

Mau
Note: Opposite orientation to Ludloff
Indication: HAV
Procedure: Oblique bone cut diagonally (dorsal-dist to plantar-prox) through the 1st
metatarsal. Rotational osteotomy.

* McBride
Indication: Mild HAV (does not truly correct the HAV deformity)
Procedure: Silver plus soft tissue, capsular releases/tightening

* McKeever (a.k.a. 1st MPJ Arthrodesis/Fusion)


Indications: HAV with dislocation
Hallux limitus/rigidus
Polio, CP, previous joint surgery
Procedure:
1. Removal of cartilage on 1st metatarsal head and base of proximal phalanx
2. Remodel the opposing sides to be a matching cone-in-cup shape
Hallux Position: Abducted 5°-10° (or parallel to lesser digits)
DF 5°-10° off WB surface

Mitchell
Indication: HAV
Procedure: Distal metaphyseal osteotomy with rectangular block of bone removed and
preservation of lateral cortical ―spur‖ (width of spur varied depending on amount
of correction needed) that hangs over shaft when transposed.

* Opening Base Wedge Osteotomy (OBWO, a.k.a. Trethowan)


Indication: HAV
Procedure: Opening base wedge osteotomy (osteotomy across base of 1st metatarsal, then
insert a pie-shaped piece of bone graft into the side of the 1st metatarsal cut)

Peabody
Indication: Abnormal PASA
Procedure: Reverdin done in the 1st metatarsal neck

* Reverdin
Indications: Mild HAV + Abnormal PASA
Procedure: Medially based wedge (proximal cut perpendicular to long axis 1st metatarsal and
distal cut parallel to articular cartilage surface) resection in 1st metatarsal head

CROZER-KEYSTONE RESIDENCY MANUAL – SECOND EDITION 133


* Reverdin-Green
Indications: Mild HAV + Abnormal PASA
Procedure: Reverdin osteotomy but in an L-shape (or trapezoidal) to preserve sesamoid
articulation

* Reverdin-Laird
Indications: Moderate HAV + Abnormal PASA
Procedure: Reverdin-Green with lateral shift of capital fragment to correct IMA

Reverdin-Green-Laird-Todd
Indication: Hallux limitus + HAV
Allows for correction in 3 planes
Procedure: Triangle-shaped wedge removed from both the top and side of the distal 1st
metatarsal

* Scarf
Indication: HAV (IMA 12°-18°)
Procedure: Z-type osteotomy through the shaft of the 1st metatarsal

Silver
Indications: Medial 1st MPJ pain
Mild HAV (but does not correct the true HAV deformity)
Procedure: Isolated resection of medial eminence of 1st metatarsal head

Stamm
Indication: HAV
Procedure: OBWO in medial cuneiform (wedge of graft inserted into medial cuneiform)

Valenti
Indication: Hallux limitus
Procedure: Removal of angled (usually 45°) dorsal wedges from the 1st metatarsal and
proximal phalanx to increase ROM

* Vogler (a.k.a. Offset-V)


Indication: HAV
Procedure: V-osteotomy made in the neck of the 1st metatarsal (similar to Kalish but more
proximal)

Watermann
Indication: Hallux limitus
Procedure: Removal of closing wedge of bone from 1st metatarsal head to DF capital
fragment

Watermann-Green
Indications: Hallux limitus
Procedure: Watermann osteotomy but with a plantar shelf to preserve sesamoid articulation

134 CROZER-KEYSTONE RESIDENCY MANUAL – SECOND EDITION


Wilson
Note: Reverse Wilson can be done for Tailor’s bunion
Indication: HAV (IMA 12°-14°)
HAV + Long 1st metatarsal
Procedure: Oblique (dist-med to prox-lat) through and through osteotomy at the 1st metatarsal
neck, capital fragment slides laterally on shaft (unstable and slow healing)

* Youngswick
Indications: HAV + DF 1st metatarsal
HAV + Hallux limitus
Procedure: Austin but with an extra slice taken out on the dorsal arm to allow the head to
drop plantarly and decompress the 1st MPJ

Gerbert
Indication: Tailor’s bunion
Procedure: Wedge osteotomy (transverse or oblique) at 5th metatarsal base. K-wire or screw
fixation.

Mercado
Indication: Tailor’s bunion
Procedure: Oblique wedge osteotomy at 5th metatarsal neck

Reverse Austin (or Reverse Chevron)


Indication: Tailor’s bunion
Procedure: Transverse plane V-osteotomy in distal 5th metatarsal with medial transposition
and impacted on shaft for fixation (or pin fixation)

Reverse Hohmann
Indication: Tailor’s bunion
Procedure: Transverse osteotomy in distal metaphysis of 5th metatarsal with medial
transposition of capital fragment. Fixation not usually used.

Reverse Wilson
Indication: Tailor’s bunion
Procedure: Osteotomy from dist-lat to prox-med to shorten the 5th metatarsal and medial
transposition of metatarsal head. Fixation not usually used.

Yancey
Indication: Tailor’s bunion
Procedure: Oblique or transverse (most stable) wedge osteotomy at 5th metatarsal prox mid-
diaphyseal area. Fixation used.

CROZER-KEYSTONE RESIDENCY MANUAL – SECOND EDITION 135


Metatarsus Adductus Procedures
Bankart
Indications: Met adductus + Age 8 years or older
Congenital absence of medial cuneiform
Procedure: Excise cuboid (to balance out lack of medial cuneiform)

Brown
Indications: Met adductus + Ages 2-6 years old
Procedure:
1. Transfer TP into navicular
2. Medial capsulotomy of nav-cun joint

Chondrotomy by Johnson
Indications: Met adductus + Ages 6-8 years old
Procedure:
1. Resect 2.5 mm lateral based wedges (apex medial) of cartilage in metatarsals 2-5, enlarge
bases medially
2. Lateral base wedge osteotomy distal to epiphysis of 1st metatarsal
3. Lengthen ABductor hallucis

Fowler
See Clubfoot Procedure section

* Berman & Gartland


Note: Most popular osseous procedure for met adductus
Indications: Met adductus + Age 6 years or older
Procedure:
1. Panmetatarsal base wedges dome-shaped or crescentic osteotomies
a. Optional – rearfoot procedures to correct combined deformities
2. Manipulate foot into corrected position, use pin fixation in all metatarsals and cast for 6
weeks

Ghali
Indications: Met adductus + Ages 2-6 years old
Procedure:
1. Heyman, Herndon & Strong procedure
2. PLUS ant-medial release of naviculocuneiform joint

* Heyman, Herndon & Strong (a.k.a. Tarsometatarsal soft tissue release)


Indications: Met adductus + Ages 2-6 years old
Procedure:
1. 3 dorsal incisions (originally one dorsal transverse incision)
2. Capsulotomies and ligament releases of all tarsometatarsal joints (metatarsals 1-5)
a. Keep plantar lateral ligaments and joint capsules intact (modification from
original to prevent dorsal subluxation)

136 CROZER-KEYSTONE RESIDENCY MANUAL – SECOND EDITION


b. Optional – syndesmotomy of naviculocuneiform joint and release of TA tendon,
also could use K-wires to maintain corrected positions
3. Manipulate metatarsals and foot into rectus position and cast for 3 months

Lange
Indications: Met adductus + Ages 2-6 years old
Procedure: Capsulotomy of 1st metatarsal-cuneiform, followed by serial casting

Lepird
Indications: Met adductus + Age 8 years or older
Procedure:
1. 3 dorsal incisions
2. Transverse plane osteotomies in bases of metatarsals 2-4 from dorsal-dist to plantar-prox,
parallel to WB surface of foot, fixated with compression screws
3. Oblique base wedge osteotomies of 1st and 5th metatarsals, fixed with compression screws

Lichtblau
Note: Same name as procedure for clubfoot
Indications: Met adductus + Ages 2-6 years old
To release abductor hallucis
Procedure: Sectioning of abductor hallucis through a small medial incision

McCormick & Blount


Indications: Met adductus + Age 8 years or older
Procedure:
1. Arthrodesis of 1st metatarsal-cuneiform joint
2. Osteotomy of bases metatarsals 2-4

Peabody & Muro


Indications: Met adductus + Age 8 years old or older
Procedure:
1. Excise bases of metatarsals 2-4
2. Osteotomy of 5th metatarsal
3. Mobilize and reduce subluxation of 1st metatarsal -cuneiform joint
4. Correction of any abnormal insertion of TA tendon
5. Optional – Hoke triple arthrodesis to realign rearfoot when necessary

Steytler & Van der Walt


Indications: Met adductus + Age 8 or older
Procedure: Oblique V-osteotomy (apex of ―V‖ toward rearfoot) of all metatarsal bases
Modified from original to include fixation

CROZER-KEYSTONE RESIDENCY MANUAL – SECOND EDITION 137


Thompson
Indications: Met adductus + Ages 2-6 years old + hallux varus (a.k.a. hallux abductus, severe
contraction of abd hallucis)
To release abductor hallucis
Procedure: Resect abductor hallucis

Achilles Tendon Procedures


* Baker
Indication: Achilles tendon lengthening
Procedure: Tongue-in-groove cut in aponeurosis with the tongue distal, facing upward
Suture aponeurosis bands to one another in retracted position

Bauman
Indication: Gastroc equinus
Procedure: Isolated gastroc recession in the deep interval between soleus and gastroc muscles

Hoke
Indication: Achilles tendon lengthening
Procedure:
1. Incision 5 cm in length over medial aspect of tendon
2. Triple hemisection of Achilles tendon
a. Cut Achilles in half in 3 sections: posteriorly in proximal and distal aspects of
incision and anteriorly in central portion of incision
b. Modification – cuts med/lat instead of ant/post, can be percutaneous
3. Forcibly DF the foot to allow for sliding into lengthened position

Lindholm
Indication: Achilles tendon ruptures
Procedure: Two flaps taken proximally from Achilles and reflected distally to fill defect

Lynn
Indication: Achilles tendon ruptures
Procedure: End-to-end reapproximation of ruptured Achilles (may reinforce with plantaris)

Fulp & McGlamry Modification (of Baker’s technique)


Indication: Achilles tendon lengthening
Correction of non-spastic gastroc equinus
Procedure: Tongue-in-groove cuts in aponeurosis with the tongue distal, facing downward
(inverted version of Baker’s technique)

Silfverskiöld
Note: Makes a 3 joint muscle into a 2 joint muscle
Indication: Achilles tendon lengthening
Procedure:
1. Release the gastroc heads at their attachments to the femoral condyles (above knee joint)
2. Reinsertion into the posterior proximal tibia area (below knee joint)

138 CROZER-KEYSTONE RESIDENCY MANUAL – SECOND EDITION


* Sliding ―Z‖ Lengthening
Indication: Achilles tendon lengthening
Procedure: Cuts most commonly done in frontal plane but can be in sagittal plane
Usually percutaneous, recommended open in McGlamry
DF the foot after cutting to separate and lengthen ends of the tendon
If open procedure, suture ends of ―Z‖ together in lengthened position

Strayer
Indication: Gastroc equinus
Procedure:
1. Distal recession with the complete transverse cutting of gastroc aponeurosis
2. Proximal retracted portion of gastroc is sutured into the deeper soleus

* Vulpius & Stoffel


Note: Originally a transverse cut in aponeurosis
Indications: Gastroc equinus
Procedure: Distal resection of gastroc aponeurosis using an inverted ―V‖
But DON’T suture to soleus

White
Indication: Achilles tendon lengthening
Procedure:
1. Section anterior ⅔ of distal Achilles and medial ⅔ of Achilles (5-7.5 cm proximal to this
point)
2. This lengthens the gastroc in relation to its twisting before its insertion

Pes Planus and PTTD Procedures


Baker & Hill
Indications: Pes planus (to restore alignment of STJ and reduce heel valgus and excess
pronation)
Cerebral palsy
Procedure: Horizontal osteotomy inferior to posterior facet of STJ (in calcaneus, medial
cortex intact as hinge) and a wedge-shaped graft inserted

Chambers
Indication: Flexible pes planus (more often in children, <8 years old)
Rarely performed anymore
Procedure:
1. TAL
2. Bone graft under sinus tarsi (similar to location of arthroereisis to block translocation of
talus on the calcaneus)

CROZER-KEYSTONE RESIDENCY MANUAL – SECOND EDITION 139


Chiappara
Indication: Pes planus
Procedure:
1. Silver (opening wedge calc osteotomy from lateral side) with TP advancement
2. TA tenodesis to TP

Cobb
Note: Good procedure because FDL preserved
Indications: PTTD
Pes valgus
Procedure:
1. Hemi-section of TA (more medial portion released, other half left intact at insertion near
ankle level)
2. Lay released portion of TA along TP tendon and suture together

* Cotton
Indications: Pes planus
PTTD
Medial column repair (to get structural PF of medial column)
Procedure:
1. Medial (and sometimes intermediate cuneiforms) osteotomy dorsal to plantar
(maintaining plantar cortex intact)
2. Triangular shaped bone graft (base measuring 4-7mm) inserted in osteotomy. No
fixation necessary.

* Dwyer
Indication: Pes planus
To produce calcaneal varus
Procedure: Closing wedge osteotomy from medial side (difficult due to possible nerve
entrapment)

* Evans Calcaneal Osteotomy


Note: Same name as procedures indicated for clubfoot and lateral ankle instability
Indications: Pes valgus foot deformity
To lengthen calcaneus
Procedure:
1. Incision over C-C joint, reflect EDB
2. Osteotomy of calcaneus parallel and 1-1.5 cm (dist ⅓) prox to C-C joint
3. Wedge of graft inserted into osteotomy (lateral side of graft up to 1 cm in kids and max 7
mm in adults)

Gleich
Indication: Pes valgus foot deformity (especially frontal plane dominant)
Procedure: Oblique calc osteotomy (posterior calc osteotomy) displaced anteriorly (to
―restore the normal angle of the long axis of the calc to the floor‖)

140 CROZER-KEYSTONE RESIDENCY MANUAL – SECOND EDITION


* Hoke Arthrodesis
Note: Not to be confused with the Hoke Achilles procedure
Indication: Pes planus
Medial column repair
Usually done in conjunction with ankle equinus correction and calcaneal
osteotomies or arthroeresis
Procedure:
1. TAL
2. Fusion of navicular to medial and intermediate cuneiforms

* Kidner
Indications: Pes planus
Kidner foot type (accessory navicular and/or enlarged navicular)
Medial column repair
Procedure:
1. Detach TP from navicular medially
2. Resect accessory navicular and/or bump from navicular
3. Reattach TP to navicular more plantarly (tendon bone anchors commonly used)

* Koutsogiannis (nickname ―Kouts‖)


Note: Sometimes combined with Evans osteotomy for PTTD
Indication: Pes valgus foot deformity
Restores heel valgus, less so in restoring medial longitudinal arch. Also shifts the
functional insertion of the Achilles medially.
Procedure:
1. Medial displacement of an oblique osteotomy of calcaneus from lateral incision
a. Posterior portion of calcaneus ―slides‖ medially ⅓ to ½ the width of calcaneus
until it sits just below sustentaculum tali
2. K-wire, Steinmann pins, or lag screw fixation

Lord
Indication: Pes planus
Procedure: A Gleich (oblique calcaneal osteotomy) displaced anteriorly, medially, and
inferiorly

Lowman
Indications: Pes planus
Medial column repair
Procedure:
1. TAL
2. Talo-navicular wedge arthrodesis
3. Reroute TA under navicular and suture into spring ligament
4. Tenodesis of medial arch by taking slip of TA and reflect downward (leave its insertion
to the calcaneus intact) along medial arch

CROZER-KEYSTONE RESIDENCY MANUAL – SECOND EDITION 141


Miller
Indications: Pes planus (more often appropriate in adults than children)
Medial column repair
Procedure:
1. TAL
2. Medial column fusion (navicular to medial cuneiform to 1st metatarsal)
3. Resect hypertrophy of navicular (use as bone graft for fusion sites)
4. Advance medial soft tissues

Peroneus Brevis Tendon Transfer (PBTT)


Indication: Type 1 vertical talus
Severe pes planovalgus
Procedure: Detach PB and reroute dorsally to talar neck
ALT – Detach PB and transfer to lateral cuneiform or 3rd metatarsal

Selakovich
Indication: Flexible pes planus + Ages 5-9 years old
Congenital vertical talus (flexible/supple deformity) + Ages 5-9 years old
Procedure:
1. Osteotomy and grafting of sustentaculum tali
a. Osteotomy performed midway between interosseous talocalcaneal ligament and
post margin of sustentaculum tali
b. Wedge bone graft inserted to redirect the middle and anterior facets
2. Tightening of medial structures (tightening redundant spring ligament and repositioning
of the TP)
3. Reroute half or all of TA into navicular

Silver or Opening Wedge Dwyer


Indication: Pes planus
Procedures:
1. Opening wedge calcaneal osteotomy from lateral incision
a. Oblique osteotomy from just post to post facet inferiorly to just prox to C-C joint
b. The more proximal and anterior the osteotomy, the greater correction
2. Graft insertion into osteotomy
a. Average wedge size ¼ inch, no fixation needed

* STJ Arthroereisis
Indication: Flexible pes valgus + patient not yet at skeletal maturity (or if arthrodesis not
appropriate in older patient)
Procedure:
1. Incision 2-4 cm long parallel to relaxed skin tension lines over sinus tarsi. Incise deep
fascia to expose lateral talar process, post facet and sinus tarsi floor.
2. Further steps of dissection depend on the specific device you are using
a. MBA (Maxwell-Brancheau Arthroereisis) implant, STA-peg device

142 CROZER-KEYSTONE RESIDENCY MANUAL – SECOND EDITION


Young Tenosuspension
Note: Often done in conjunction with other procedures
Indications: Pes planus + Age 10 years or older
Patients with navicular-cuneiform fault but no DJD yet
Helps to PF 1st ray (takes away TA antagonist action against PL)
Procedures:
1. TAL
2. Reroute TA through keyhole in navicular (do not detach TA from insertion)
a. Alternate – detach TA from insertion and reattach after passing through a trephine
hole in navicular
3. TP reattachment beneath navicular (creates a powerful plantar navicular-cuneiform
ligament)

Lateral Ankle Instability Procedures


* Brostrom-Gould (or just Brostrom)
Note: Common procedure used, see Special Surgery Section for details
Indication: Lateral ankle instability
For primary repair
Procedure:
1. Incise lateral ankle capsule 2-3 cm distal to lateral malleolus
2. Evert foot and tighten capsule including ATFL and CFL in pants over vest fashion with
non-absorbable suture
3. Mobilize extensor retinaculum, pull it over capsule and suture down

* Christman & Snook


Note: Could use PL instead of PB for this procedure
Indication: Lateral ankle instability
To reinforce ATFL and CFL
Procedure:
1. Detach half of PB from its insertion
2. Reroute it through a drill hole in the talar neck and distal lateral malleolus (through
widest part, anterior to posterior). Suture graft tendon to periosteal flap at level of CFL.
3. Distal half of PB then sutured to proximal half

Elmslie
Indication: Lateral ankle instability
To reinforce ATFL and CFL
Procedure: Tensor fascia lata routed through calcaneus, then lateral malleolus, then talus,
back through lateral malleolus and back through calcaneus

Evans
Note: Same name as osteotomies indicated for pes planus and clubfoot. Similar to Nilsonne but
with an osseous tunnel instead of subperiosteal tunnel.
Indication: Lateral ankle instability
Reinforces ATFL only (this does not reconstruct ATFL or CFL anatomically)

CROZER-KEYSTONE RESIDENCY MANUAL – SECOND EDITION 143


Procedure:
1. PB is detached proximally
2. Reroute it through fibular drill hole (anterior-most and distal-most → post-prox location).
PB secured posteriorly at prox aspect of superior peroneal retinaculum.
3. Prox PB is attached to PL

Hambly
Indication: Lateral ankle instability
Reinforces ATFL and CFL
Procedure:
1. Split PL
2. Reroute into talus (or attached through it), through a lateral malleolus drill hole (anterior
to posterior), through calcaneus and attached to the other half of the PL

Kelikian
Indication: Lateral ankle instability
Reinforces ATFL and CFL
Procedure:
1. Isolate the plantaris tendon
2. Reroute it from the calcaneus into lateral malleolus through a drill hole (posterior to
anterior), back through the calcaneus then sutured on itself

Lee
Indication: Lateral ankle instability
Reinforces ATFL only
Procedure:
1. PB detached proximally
2. Reroute it through lateral malleolus drill hole (post → ant) and sutured upon itself
(peroneal anastomosis)
3. Periosteal flap from dist fibula reinforces new ligament
4. Prox PB attached to PL

Nilsonne
Indication: Lateral ankle instability
Reinforces ATFL only
Procedure:
1. PB detached proximally at musculotendon junction
2. Reroute it through subperiosteal groove through fibula (post-superior → ant-inferior),
CFL primarily repaired if necessary
3. PB secured in subperiosteal tunnel (this approximates ATFL course)
4. Prox PB attached to PL

Seeburger
Indication: Lateral ankle instability
Reinforces ATFL and CFL

144 CROZER-KEYSTONE RESIDENCY MANUAL – SECOND EDITION


Procedure:
1. Use a hemi-section of PL
2. Reroute it from talus into lateral malleolus and into calcaneus

Watson-Jones
Indication: Lateral ankle instability
Reinforces ATFL only
Procedure:
1. PB detached proximally
2. Reroute it through lateral malleolus (drill hole posterior to anterior ~2 cm from distal tip
of fibula) into talar neck (vertical drill hole dorsal to plantar), then back through lateral
malleolus (along ATFL course) and sutured on itself posterior to fibula
3. Proximal PB attached to PL

Whinfield
Indication: Lateral ankle instability
Reinforces ATFL and CFL
Procedure:
1. PB detached proximally while maintaining distal attachment
2. The detached portion is rerouted through a lateral malleolus drill hole (anterior to
posterior) and inserted into calcaneus

Clubfoot Procedures
Baja Project
Indication: Clubfoot
Procedure:
1. Cuboid decancellation procedure
2. Laterally based triangular wedge of bone removed from cuboid and lateral cuneiform

* Dwyer
Note: Indicated also for pes planus but wedge done laterally instead of medially (calcaneus goes
into varus)
Indication: Clubfoot
Cavus foot deformity
Procedure: Opening wedge medial calcaneal osteotomy
Calcaneus goes into a more valgus position

* Evans
Note: Same name as procedures indicated for pes planus and lateral ankle instability
Indication: Clubfoot
Procedure: Shorten lateral column by calcaneal-cuboid fusion

CROZER-KEYSTONE RESIDENCY MANUAL – SECOND EDITION 145


Fowler
Note: Often done in conjunction with other procedures, especially if more rigid deformity
Indications: Residual clubfoot deformity
Cavovarus deformity
Met adductus + Age 8 years or older
Procedure:
Bone graft inserted into medial cuneiform with opening wedge osteotomy to lengthen medial
column
Modification – closing wedge osteotomy of cuboid and lateral cuneiform, then use this bone
as the graft for the opening medial cuneiform osteotomy. Good with ages 3-10 years old,
residual adduction, or varus deformity in forefoot/midfoot.

Lichtblau
Note: Same name as a procedure for met adductus
Indication: Clubfoot
Procedure: Closing base wedge osteotomy of anterior calcaneus (base of wedge lateral,
shortens lateral column)

Lund
Indication: Clubfoot (neglected or arthrogryphotic neuromuscular type) + Ages 2-5 years old
(ideally, occasionally in adults)
Procedure:
Talectomy (foot posteriorly displaced allowing for correction in sagittal and frontal planes)
Optional – portions of navicular and fibula may need to be resected. Also may use
midfoot wedges adjunctively. Often multistaged.
Fixation with Steinmann pin from calc to tibia for pseudojoint space. Long leg casting for 1
month to BK cast for 4 months.

Miscellaneous Procedures
Hibbs
Indication: To decrease MPJ buckling and increase DF
Procedure:
1. EDL is detached from insertion and reattached to lateral cuneiform or 3rd metatarsal
2. Distal stubs of EDL are attached to EDB at metatarsal head area

* Hoffman
Note: Often done with Keller arthroplasty
Indications: MPJ subluxation secondary to rheumatoid arthritis and fat pad atrophy
Procedure: Resection of metatarsal heads 2-5

Hoffman-Clayton
Indications: MPJ subluxation secondary to rheumatoid arthritis and fat pad atrophy
Procedure: Resection of metatarsal heads 2-5 and bases of proximal phalanxes

146 CROZER-KEYSTONE RESIDENCY MANUAL – SECOND EDITION


* Jones
Indications: Cock-up hallux
Weak TA (procedure enhances DF)
Procedure:
1. EHL is detached and inserted into 1st metatarsal head via a med → lat drill hole
2. IPJ fusion
3. Stump of EHL is attached to EHB

Murphy
Indication: Spastic equinus
Procedure: Anterior transfer of TA into calcaneus
Modification – route under FHL

OATS (Osteoarticular Transfer System)


Indication: Posterior medial talar dome osteochondral lesion
Procedure:
1. Take a plug of bone with articular cartilage from the knee
2. Through a trans-tibial approach, insert it into the talus (matching the contours of cartilage
on graft to dome of talus)

Peroneus Longus Tendon Transfer (PLTT)


Indication: Drop foot
Anterior muscle weakness
Flexible cavus deformity
Procedure:
1. Cut PL near PB insertion site, suture distal PL to PB
2. Reroute it dorsally to 3rd cuneiform

Silver & Simon


Indication: Spastic equinus
Procedure:
1. Proximal release of gastroc without reinsertion of heads
2. Neurectomy of tibial branches to medial head of gastroc

* Split Tibialis Anterior Tendon Transfer (STATT)


Note: Same as TATT but only half the tendon is used. See Special Surgery Section for details.
Indications: To increase true ankle DF and decrease long extensor swing phase
To decrease adductovarus forefoot
Procedure:
1. Detach half of TA from its insertion
2. Reroute and insert it into peroneus tertius (or cuboid, if peroneus tertius isn’t present)

Stoffel
Indication: Correction of spastic muscular forms of ankle equinus
Procedure: Selective denervation of tibial nerve

CROZER-KEYSTONE RESIDENCY MANUAL – SECOND EDITION 147


Tachdjian-Grice
Note: Grice procedure = STJ arthrodesis
Indications: Congenital convex pes planovalgus (vertical talus!) + Ages 4-6 years old
Procedure:
1. First stage: TAL with posterior ankle and STJ capsular release
2. Second stage: (3 weeks later) STJ extraarticular arthrodesis

Tibialis Anterior Tendon Transfer (TATT)


Note: The STATT is slightly preferred due to fewer complications. See Special Surgery Section
for details.
Indications: To decrease forefoot supinatory twist
To increase true ankle DF
Procedure:
1. TA is detached from its insertion
2. Reroute and insert it into lateral cuneiform or 3rd metatarsal (or inserted into peroneus
tertius if present)

Tibialis Posterior Tendon Transfer (TPTT)


Note: See Special Surgery Section for details.
Indications: Drop foot
To eliminate flexor substitution
Procedure:
1. TP is detached at its insertion site on the navicular
2. It is then rerouted through the interosseous membrane of the tibia and fibula, brought
anteriorly and then inserted into the lateral cuneiform

148 CROZER-KEYSTONE RESIDENCY MANUAL – SECOND EDITION


Special Surgical Section
This section is based on my notes on how to do a select few rearfoot and ankle cases. Basically,
the outlines are a combination of McGlamry’s, Coughlin’s, and Kitoaka that I used in my third
year to prepare for the cases. Obviously, my notes are only one of many ways to do the cases.

I have included my notes here as a quick outline reference. As always, it is up to the reader to go
to the original sources to learn the material.

--Brett

We also used McGlamry and Coughlin & Mann to prepare for cases. We have included a few
additional notes that we found helpful.

--Hubert & Sandi

CROZER-KEYSTONE RESIDENCY MANUAL – SECOND EDITION 149


Achilles Tendon Rupture
 Frequently in ―Weekend Warriors‖ men 30-50 yrs old
 Location of tear usually occurs in the ―watershed area‖ (2-6 cm proximal to the Achilles
insertion) though avulsions and myotendinous junction ruptures are also possible
 Clinical Exam
o Patient can PF because of posterior tibialis but can’t do a one-legged heel raise
o Positive Thompson test (squeezing the calf reveals an absence of ankle plantarflexion)
o Palpable gap, although this may be less obvious with edema after 24 hours
 Surgery vs Conservative Tx
o Younger people tend to do better and recover faster with surgery over conservative
methods
o Older, non-active people tend to do better with conservative methods
 Conservative Treatment
o PF cast for 2 weeks NWB
o Increase DF (but still PF) for 2 more weeks. Continue NWB.
o At 4 weeks, if can cast in neutral, pt can WB
o At 8 weeks, D/C cast and use CAM boot for 4 more weeks. A 2-2.5 cm heel lift can be
put in shoes. Begin passive ROM.
o At 12 weeks, start active exercises in regular shoe (with ¼‖ to ½‖ heel lift)
 Pre-op Tx
o Jones compression dressing with mild PF of foot

 Procedure
o Patient set-up
 Prone
 General or spinal
 Thigh tourniquet
1. Incision: medial aspect of the Achilles tendon from just above the myotendinous
junction to insertion on calcaneus. Incision is down to paratenon.
a. Use very careful dissection as this area is very avascular
b. Watch out for:
i. Plantaris medially
ii. Sural nerve perforates the Achilles tendon centrally at the myotendinous
junction and then courses laterally
iii. Lesser saphenous vein may also run with the sural nerve
2. Incise the paratenon. The paratenon should be opened as part of the full thickness
flap. Get good exposure to the tendon.
 For End-to-end or mid-tendon ruptures
3. Irrigate, debride the mop-top ends of tendon
4. Reapproximate using Bunnel, Kessler or Krackow type of suture using 3-0 or 2-0
Ethibond or other non-absorbable polyester suture
5. Reinforce site with 1-0 or 2-0 Vicryl in a circumferential stitch. Irrigate again.
6. Close paratenon with 3-0 Vicryl, subcutaneous, then skin

150 CROZER-KEYSTONE RESIDENCY MANUAL – SECOND EDITION


 For Myotendinous junction ruptures (Reverse Lindholm’s technique)
3. Rather than inverted strips of tendon being raised from prox → dist (as in Lindholm’s
technique), go dist → prox
4. Weave inverted tendon into place
5. Suture with Bunnel, Kessler or Krackow technique, similar to end-to-end repair
6. Reinforce with Vicryl circumferential suture, close subcutaneous and skin
 For Avulsion ruptures
3. Extend incision distally past insertion of Achilles onto calcaneus
4. Debride calcaneus of fibrous tissue, debride down to cancellous bone
5. Make 2 drill holes longitudinally to each other and put non-absorbable suture in
tendon (i.e. half a Kessler) bring suture through drill holes and tie over itself or (more
commonly now) use suture bone anchors
6. Close

 Post-op
o Jones compression dressing for 7-10 days
o NWB BK cast for 2-3 weeks
 Cast in 20° PF for avulsion and mid-tendon ruptures
 Cast in neutral for myotendinous ruptures
o After the PF cast, cast in neutral position (have the patient rest foot on a footrest for 15-
20 min to gently allow the foot to go to neutral or close). WB BK cast for 2-4 weeks or
CAM boot (removable walking cast) here instead. Pt may also start passive ROM.
o After casts, return to regular shoes with a 1‖ heel lift
o Aggressive walking may begin around 10 weeks post-op
o Return to sports at 14-16 weeks post-op
o Note: Pt may not be able to get full DF of foot for 3-6 months

CROZER-KEYSTONE RESIDENCY MANUAL – SECOND EDITION 151


Delayed Repair of Achilles Tendon
 Clinical Presentation
o At time of rupture, pt feels as if struck in the back of the calf
o Pt complains of weakness in PF
o If pt has progressive degenerative changes to Achilles tendon, insidious onset
o Often rupture is not palpable
o Tendon tends to be thicker as it progressively gets longer
 Areas of Ruptures
o At tendon 2-6 cm from insertion
o At myotendinous junction
o Calcaneal avulsion
 Conservative Treatment
o Heel lifts, lace up shoes, MAFO, braces
o BUT…these won’t restore normal push-off
 Indications for Surgery
o Restore normal push-off power
o Take X-rays and MRI
 If >3 cm defect and >3 months → end-to-end suture (as in Acute Achilles repair)

 Procedure
o Patient set-up
 Prone
 General or spinal
 Thigh tourniquet
o For mid-tendon tear
1. Posterior medial incision over Achilles from just above myotendinous junction to
past calcaneal insertion. Make full thickness incision to paratenon.
2. Incise paratenon, reflect with full thickness flap
3. Irrigate, clean up mop-handle like edges
o If >3 cm tear
4. Kessel, Bunnel or Krackow type suture with 3-0 Ethibond
5. Reinforce ends circumferentially with 2-0 or 3-0 Vicryl
o If gap is approx 3 cm → inverted V-Y advancement is done
4. Inverted V-Y – leaving underlying muscle attached to the paratenon
5. Advance the distal flap distally
6. Close the defect via Kessel, Bunnel, or Krackow with 3-0 Ethibond
7. Reinforce ends circumferentially with 2-0 or 3-0 Vicryl
o If gap is much greater than 3 cm → V-Y advancement, close end-to-end, reinforce
with FHL
 Inverted V-Y will have to be done at an acute angle and will make the tendon
very thin…which is why you have to reinforce with FHL
 FHL Transfer
1. Incision is on medial border of midfoot, from the navicular to head of 1st
metatarsal, just above the level of the abductor muscle (approximately
where plantar skin meets regular skin)

152 CROZER-KEYSTONE RESIDENCY MANUAL – SECOND EDITION


2. Dissect down to the layer of the abductor hallucis fascia. Reflect the muscle
downward. Retract with Weitlaner.
3. FHB is reflected plantarly
4. Identify the FHL (medial) and FDL
a. These are usually covered by a fatty layer
b. Flex the IPJ of the hallux, and the FHL can be identified
5. Section the FHL as distally as possible, generally at midshaft of 1st
metatarsal
6. Tag prox part of FHL. Suture distal FHL to FDL with the toes in neutral
position.
o Posterior Medial Incision
1. Posterior medial incision over Achilles from just above myotendinous junction to
past calcaneal insertion. Make full thickness incision to paratenon.
2. Incise paratenon, reflect with full thickness flap
3. Irrigate, clean up mop-handle like edges
4. Incise the fascia overlying the FHL. By pulling on the suture from the FHL, you
can identify the muscle.
5. Retract the tendon through post-medial incision
6. Make transverse drill hole into posterior calcaneus just distal to insertion of
calcaneus halfway from medial → lateral
7. 2nd drill hole goes from prox → dist to intercept the holes in calcaneus. A large
towel clip is used to connect the 2 holes.
8. A suture passer is inserted in the dist-medial hole upward. The FHL is attached
to it and the FHL tendon is passed from prox → dist-medial.
9. The FHL is then woven into the Achilles tendon from dist → prox and repeated
to use the full length of FHL tendon.
10. The tendon is secured with Ethibond
11. Repair paratenon, then close in layers

 Post-op
o Jones compression dressing and plaster splints with foot in 15° PF until first post-op
visit (7-10 days)
o BK cast with foot in 15° PF for 4 weeks
o Cast foot in neutral with BK walking cast or removable cast boot for 4 weeks
 Put foot on footrest with hip flexed. Allow foot to passively go to neutral.
o At 8 weeks post-op, begin strength training and ROM exercises
o Patient remains in removable cast boot until the 10° of DF and 4/5 PF strength is
achieved
o Half-inch heel lift is added to their shoe. Home exercises are performed at this period.
o Athletic activity restricted for 6 months

CROZER-KEYSTONE RESIDENCY MANUAL – SECOND EDITION 153


Excision of Calcification of Achilles Tendon
 Procedure
o General or spinal
o Prone is ideal
o Thigh tourniquet
 Have Mitek anchor or other kind of anchor in room
1. Incision: from the superior-medial of the Achilles tendon (can go 1 cm medial to
tendon and 3-4 cm proximal to spur or tuberosity)lateral (2-3 cm distal to spur or
tuberosity), with the horizontal part over the spur
a. Note: can go the other direction which may keep away from sural nerve
2. Dissect in layers, tag paratenon
3. Incise Achilles tendon longitudinally (Lateral ½ = Medial ½). Keep distal attachments
of Achilles, if possible.
a. If total resection of Achilles tendon must be performed, remove all bony
prominences and treat it like a ruptured Achilles tendon
i. Can drill holes and use non-absorbable suture (ie fiberwire) or Mitek
bone-tendon anchor
4. Reflect Achilles tendon side to side, remove any intra-tendon calcification
5. Deepen incision—remove retrocalcaneal bursa
6. Release any paratenons fibrosing or scaring
a. Expose any posterior calcaneal exostosis
b. Can resect calcaneal exostosis with an osteotome
7. Repair Achilles tendon with 2-0 Vicryl in running suture
8. Close

 Post-op
o 3-6 weeks NWB in cast (if necessary)
o 3-6 weeks in a WB boot

154 CROZER-KEYSTONE RESIDENCY MANUAL – SECOND EDITION


Murphy Procedure – Achilles Tendon Advancement
 Indication
o Spastic equinus
o Plantarflexory force of gastroc-soleus complex at ankle joint is weakened with minimal
decrease of toe-off force

 Procedure
1. Incision 5 cm slightly medial to midline of Achilles tendon
2. Dissect down to deep fascia and paratenon. Incise and tag the paratenon. Do not
dissect in layers because this will lead to soft tissue necrosis.
3. Detach TAL from its insertion to the calcaneus
a. If child, careful not to disturb the calcaneal apophysitis
4. Reroute the TAL under the FHL (if desired)
5. Divide the fat over the calcaneus, then resect a 0.5 cm wedge of bone from calcaneus
just posterior to posterior facet
6. From that wedge, make 2 drill holes, one exiting medially, one exiting laterally
7. Use a Bunnell technique to the distal end of the TAL with either an absorbable (1-0
Vicryl) or non-absorbable (1-0 Ethibond) suture
8. Bring one of the loose end strands through the medial drill hole, and bring the other
strand laterally
9. With the foot in neutral, guide the tendon into the wedge and tie the sutures over the
dorsal surface of the calcaneus, anterior to Achilles tendon
10. Close, cast in AK cast with knee slightly flexed, foot in neutral

CROZER-KEYSTONE RESIDENCY MANUAL – SECOND EDITION 155


Ankle Arthrodesis
 Ideal position
o Ankle neutral (no DF or PF)
o 5° valgus
o External rotation equal to opposite limb

 Procedure
o Patient set-up
 Prone with sandbags
 Thigh tourniquet
 General or spinal

 Lateral approach
1. Incision is curved starting approx 10 cm proximal to fibula to base of 4th metatarsal
a. Avoid sural and intermediate dorsal cutaneous nerves
2. Create skin flaps
3. Strip periosteum from fibula (reflect anteriorly and posteriorly)
4. Incision is carried down to expose posterior facet of STJ and sinus tarsi
5. Use a periosteal elevator to strip the tibia, ankle joint and prox talar neck (med →
lat)
a. Do NOT dissect talar neck except for prox portion. You don’t want to strip off
the blood supply to the talus!
6. Osteotomize the fibula approx 2 cm prox from ankle joint
a. Bevel the cut proximal-lateral to distal-medial so you don’t leave a sharp edge
b. Remove distal portion of the fibula
c. Reflect peroneal tendons posteriorly
7. Make incision through deep fascia at post tibia. With a periosteal elevator, strip the soft
tissues off the tibia.
a. This is visualized after fibula is removed
8. The initial cut in the tibia is with the short wide blade, then complete with the long
wide blade
a. Cut is perpendicular to long axis of bone
b. Remove as little bone as possible
c. Stop cut where the tibia curves for the medial malleolus

 Medial approach
1. Incision is 4 cm over the anteromedial aspect of the medial malleolus and directed
slightly inferior so that the medial tip of the medial malleolus can be exposed
2. Strip soft tissue anteriorly
a. Do as little damage to deltoid ligament as possible
3. With a size 10 osteotome, cut along the medial malleolus to finish the initial tibial cut
while freeing up the initial portion

156 CROZER-KEYSTONE RESIDENCY MANUAL – SECOND EDITION


 Lateral incision
1. With a broad osteotome, wedge out the tibial cut using gentle levering
2. In the talus, cut 3-4 mm from the superior surface
3. Check alignment and remove more bone if needed

 Fixation
1. Temporary fix with 0.062 K-wire
a. Check position relative to patella
b. Place two 3.2 drill bits, one in sinus tarsi and one just above lateral process
c. Check position
2. Insert two 6.5 mm screws from lat-distal → med-prox
a. Note: There are multiple specialty plates available specifically for this procedure
that may change the way you do your fixation
3. Be sure to engage medial cortex of tibia
4. If soft bone, use a washer
a. Check rigidity of arthrodesis site
b. Optional – Can put 3rd screw through medial incision
c. Optional – Fixation of fibula with 4.0/4.5 mm cancellous screw
5. Closure
a. Use a drain
b. Deep closure, etc.
c. Administer marcaine block
d. Compression dressing and splint

 Post-op
1. Leave post-op dressing in place for 10-12 days, change and remove stitches
2. Put patient in BK cast, NWB
3. Do not use removable cast because they don’t provide enough support
4. At 6 weeks, X-ray. If healing begins to appear, use BK WB cast.
5. At 12 weeks, if satisfactory healing, can WB
6. Avg fusion time: 14 weeks
7. Avg shortening: 9 mm

CROZER-KEYSTONE RESIDENCY MANUAL – SECOND EDITION 157


Tibial-Calcaneal Arthrodesis
 Procedure
o Patient set-up
 Supine with bump under ipsilateral hip
 Thigh tourniquet
 General or spinal

 Lateral Approach (similar to lateral approach in Tibiotalar arthrodesis)


1. Incision 10 cm proximal to tip of lateral malleous across the tip of the lateral malleolus
and toward the 4th metatarsal base
a. Watch out for sural nerve and superficial peroneal nerve
2. Strip the periosteum from the anteroposterior aspect of the fibula, the lateral aspect of
the talus, and the calcaneus
3. The distal portion of the fibula is removed approx 1.5 cm above the level of the distal
tibia
4. Dissect over the anterior portion of the tibia to the medial malleolus
5. An incision is made over the posterior aspect of the tibia, and the periosteal elevator is
passed along the back of the tibia to the level of the calcaneus
a. The entire lateral aspect of the ankle joint and talus is now exposed
6. Using a saw, cut the talar neck, from lat → med just distal to the dorsal articular
cartilage of the talus
7. Remove the talus body
a. The calcaneal articular surface can now be visualized
8. Remove the articular cartilage of the tibia perpendicular to the long axis of the tibia
a. Starting approximately 2 mm above the cartilage, remove as little bone as
possible
9. With the foot in plantargrade position, remove the dorsal aspect of the calcaneus.
This creates a flat surface for the arthrodesis.
a. This includes the posterior and middle facets but leaves the sinus tarsi intact
b. Do not violate the C-C joint or the anterior process of the calcaneus

 Medial approach
1. Incision over the anteromedial aspect of the joint and carry it out distally past the tip of
the medial malleolus for about 2cm to the TN joint
2. Strip the periosteum from the medial malleolus (the portion uncut from the
osteotomy)
3. Remove this portion of the medial malleolus, usually by osteotome
a. Be careful of the neurovascular bundle at the posterior medial portion of tibia

 Remember your alignment (5° of dorsiflexion and 5° of valgus)


1. If necessary, remove bone from tibia or calcaneus to achieve it
2. The fusion site should be posterior enough for normal posterior curvature of the heel
3. Make cut in anterior aspect of the tibia parallel to the cut made in the talar neck
4. Drill surfaces of arthrodesis sites

158 CROZER-KEYSTONE RESIDENCY MANUAL – SECOND EDITION


 Internal Fixation
1. Use 0.062 K-wires for temporary fixation
a. Check alignment
2. Insert 7.0 or 7.3 cannulated screws from posterior calcaneus to anterior portion of tibia
a. If possible, insert two screws
b. The screws are more plantar than in a subtalar arthrodesis, even if this means you
are on a WB surface
c. Can throw a third screw from tibia to calcaneus post → ant or apply a blade plate
3. Fixate the talus to tibia with two 4.0 screws
4. Note: There are multiple specialty plates available specifically for this procedure that
may change the way you do your fixation
5. Check alignment with C-arm
6. Close

CROZER-KEYSTONE RESIDENCY MANUAL – SECOND EDITION 159


Subtalar Arthrodesis
 Procedure
o Patient set-up
 Prone
 Thigh tourniquet
1. Lateral incision over sinus tarsi from tip of lateral malleolus to 4th metatarsal base
2. Free the EDB from its attachment to calcaneus
3. Incise the fatty plug longitudinally
4. Retract the peroneals plantarly
5. With lamina spreader, spread the sinus tarsi.
6. Remove articular cartilage with rasp, curette, osteotome or rongeur
a. Preserve the shape and contour of the bones
b. Be careful not to violate the tibio-talar joint
7. Make a stab incision. Put guide wire in calcaneus.
8. Put heel in 5˚-10˚ of valgus
a. After the heel is in good position, advance wire into talus
b. X-ray and advance 7.0 mm cancellous screw with 16 mm thread length
9. Test strength of fusion
10. Optional – 2nd screw through same incision.
11. If not stable, remove hardware and insert a screw through talar neck into calcaneus.
12. If bone graft is needed, you can take part of the anterior process of calcaneus. Or use
Grafton.

 Post-op
o First 48˚ Jones compression dressing with splint
o Then BK cast NWB for 6 weeks (remove stitches at 3 weeks)
o BK WB cast for 4-5 weeks until radiographic evidence of healing
o Eventually rehab with or without PT

160 CROZER-KEYSTONE RESIDENCY MANUAL – SECOND EDITION


Talo-Navicular Fusion
 Good results in low activity patients
 High demand patients should probably add a C-C fusion (double arthrodesis)
 Indications
o Primary arthrosis secondary to trauma or rheumatoid arthritis (main indication)
o If instability secondary to PT dysfunction or collapse of TNJ from rupture of spring
ligament, isolated T-N fusion is indicated (but Coughlin usually does triple)

 Procedure
o Ideal position of foot
 5° valgus
 TNJ in neutral
 Forefoot 0-5° varus
o Patient set-up
 General or spinal
 Thigh tourniquet
 Patient supine
1. Incision just distal to medial malleolus to 1 cm beyond the navicular-cuneiform joint,
curved slightly dorsal (especially if large dorsal osteophyte is present)
2. Strip joint capsule with periosteal elevator or sharp osteotome
3. Remove osteophytes with rongeur or osteotome
4. Identify articular surfaces, remove with curette or osteotome
a. Can use towel clip in medial navicular for exposure
b. Visualization can be improved with lamina spreader if bone is hard
c. Difficult to see laterally, but this must be exposed and debrided
5. Joint surfaces are heavily feathered and foot is manipulated into anatomic alignment
6. Stabilize calcaneus and place STJ in 5° valgus
a. Manipulate midtarsal joint into a few degrees abduction
b. Forefoot into a plantigrade position that is perpendicular to long axis of tibia
c. Forefoot should not have a residual of more than 7-10° varus or valgus
7. Internal fixation—can use two 4.0 or 4.5 mm canulated cancellous screws. For large
person, can use 7.0 mm screw. If bone is soft, can use multiple staples.
8. Hold foot in corrected alignment, drive guide pin into navicular starting at navicular-
cuneiform joint and drill obliquely across navicular into head and neck of talus
9. Check alignment of foot, C-arm
10. Add second guide wire, C-arm
11. Overdrill navicular, insert 40 to 50 mm long threaded cancellous screw. Screw threads
must pass the intended fusion site. If soft bone, use washer. C-arm
12. Check stability of foot
13. If bone is soft or the fusion is not stable, use staples. This is also useful if there is a
fracture of the navicular.
14. Close
15. Marcaine at the end

CROZER-KEYSTONE RESIDENCY MANUAL – SECOND EDITION 161


 Post-op
o Compressive dressing with two splints
o NWB for at least 6 weeks
o Add cast on 1st visit
o After sutures are removed, place patient in short leg removable cast
o After 6 weeks, and x-rays look good, patient can ambulate with short leg cast
o 3 months after surgery, if x-rays ok, patient can d/c the short leg cast

 Complications
o Non-union—Rate is higher than in CCJ or STJ probably because of inadequate exposure
to joint. Also because navicular is avascular.
o Flatfoot—Results from placing the STJ in too much valgus and forefoot in too much
abduction
 Correct with a triple arthrodesis

 Items needed for surgery


o Periosteal elevator, osteotome, curettes
o Possible saggital saw
o Wire driver
o Towel clips
o Lamina spreader
o 4.0, 4.5 or 7.0 canulated cancellous screw (probably 40-50 mm)
o Possible washers
o Possible staples
o 3-0, 4-0 Vicryl, 5-0 Monocryl or 4-0 prolene
o Splints for stirrup/posterior split

162 CROZER-KEYSTONE RESIDENCY MANUAL – SECOND EDITION


Triple Arthrodesis
 Procedure
o Patient set-up
 Prone
 Thigh tourniquet
 General or spinal
1. See Subtalar Arthrodesis for subtalar steps
2. Lateral incision
a. Incise over sinus tarsi from lateral malleolus to base of 4th metatarsal
i. Avoid superficial peroneal nerve and peroneal tendons
b. Reflect EDB off calcaneus
c. Longitudinally incise sinus tarsi fat plug
d. Find bifurcate ligament → this will show you the entry point to debride the
talonavicular (TNJ)
e. Spread with lamina spreader
f. Mobilizing the soft tissues is necessary for reducing the pes planovalgus
articulation
3. Calcaneal-Cuboid joint (C-C joint)
a. From the above incision, reflect down to the C-C joint
b. Distract with a Hohmann spreader
c. Reflect the soft tissues
4. Talonavicular joint
a. Find the bifurcate ligament and resect it
b. Resect the soft tissues of the TN joint
5. After all the soft tissues are released, the deformity can be corrected → minimally
resect the articular cartilage of the joints
a. Resect TN joint, then C-C joint, then the talocalcaneal joint
6. Dorsomedial incision
a. Incision is centered over TNJ and extends proximally up talar neck
b. Resect the soft tissues and then the cartilage
7. Fixation: Order TC → TN → CC
a. From talar neck aimed post-lat, a 6.5 mm partially threaded cancellous screw
i. Avoid placing the screw too far posteriorly into the talar neck because it will
cause ankle impingement
ii. Could also go from calcaneus into talus.
b. A 4.5 mm cortical screw is placed from the navicular to talus
i. Screw should measure less than 40 mm
c. C-C joint is fixated with a 4.5 mm cortical screw placed from calcaneus to cuboid
i. Screw should measure less than 40 mm

CROZER-KEYSTONE RESIDENCY MANUAL – SECOND EDITION 163


Calcaneal Slide Osteotomy
 Can slide medially for flatfoot, or laterally for ankle instability

 Procedure
o Patient set-up
 General or spinal
 Supine
 Thigh tourniquet
 Bump hip
1. Incision is1 cm posterior to fibula and 2 cm proximal to superior aspect of calcaneous
(behind peroneal tendons and anterior to Achilles tendon). Stay posterior to peroneal
tendons. Ends at junction of plantar and lateral skin at level of peroneal sulcus.
2. Sharp dissection down to bone, careful of peroneal tendons and sural nerve.
3. Bone cut: perpendicular to calcaneous. From the midpoint of tuberosity to 1 cm past
the plantar weight bearing portion of the calcaneous. ***Careful about cutting too far
medial because your nerves and arteries are over there.
a. Score your cut first
b. Get a wider, longer blade than usual.
c. May have to finish with osteotome.
4. Displace tuberosity:
a. If laterally so that midaxis of tibia is slightly medial to the midpoint of the
calcaneus. The lateral wall of calc should be just lateral to the lateral malleolus.
b. If medially, displace approx 1 cm.
c. Can displace with an osteotome or lamina spreader without teeth.
5. If needed, can use a Dwyer wedge for added valgus. A Dwyer wedge is generally 1 cm
laterally.
6. Fixation: angled plate, one (or two) 6.0 or 7.0 partially threaded cannulated screws, or
two 4.0 partially threaded screws.
a. For the screws: insert just off the heel pad posteriorly (about 1.5 cm above
plantar surface). If sliding medially, insert screw just laterally. If sliding laterally,
insert just medially.
b. C-Arm for position.
7. Marcaine post-op block
8. Close.

 Post-op
o Posterior splint in OR, compression dressing
o Cast application after 1st visit for 5 weeks.
o Removable cast: ROM exercises until osteotomy site is healed.

164 CROZER-KEYSTONE RESIDENCY MANUAL – SECOND EDITION


Evans Calcaneal Osteotomy
 Mark out tendons and CC joint before surgery

 Procedure
o Patient set-up
 General or spinal
 Supine
 Thigh tourniquet
1. Get bone graft in saline and starting to reconstitute.
2. Incision is oblique over distal half of calcaneous (cut will be 1-1.5 cm from CCJ)
3. Dissect to bone. Careful of Intermediate Dorsal Cutaneous Nerve (Dorsal) Sural N
and Peroneal Tendons (Plantar)
4. Expose to osteotomy site 1-1.5 cm from CCJ
5. Free up EDB
6. Pass a probe thru the anterior and middle facets.
7. Make cut with sagital saw parallel to CCJ, aimed slightly distal to emerge in between
the anterior and middle facets. Don’t go too far medially or you may damage vital
soft tissues medially. Can use osteotome to finish the medial cut
8. Lamina spreader is put in place of the osteotomy site.
9. The osteotomy site is opened by loading the fifth met and putting foot in adduction.
Load until the hind foot valgus is corrected as well as the forefoot varus.
10. Insert bone graft. Bone graft is probably going to be twice as wide on the outside as
the medial side. Most likely the graft will be about 1cm in width at the widest side
(maximum). Don’t forget to keep the cortical sides with the other cortical sides.
11. Can fixate with staple, or screw. Screw is placed distal dorsal to proximal plantar.
a. Very often no fixation is used.

 Post-op
o NWB BK cast. For adults 6-8 weeks, for adolescents 5-6 weeks.

CROZER-KEYSTONE RESIDENCY MANUAL – SECOND EDITION 165


Peroneal Brevis Tendon Repair and Reconstruction
 Non-operative management—BK cast in neutral to slight inversion for 6 weeks.
o Associated with 30-40% rate of redislocation.

 Procedure
o Pt supine
o Sandbag under hip or lateral decubitous.
o General or spinal
o Thigh tourniquet
1. Incision-curvilinear, approx 5-7 cm behind the fibula, inline with the peroneal tendons.
Half of incision above malleolus, half below.
2. Full thickness flaps, identify superior peroneal retinaculum
3. Inspect peroneal tendons for subluxations, partial or complete tears and tenosynovitis
4. Retract PL anteriorly to visualize PB and often reveals a central split and subluxation
over posterior ridge of fibula
a. If PB tear is found and degenerative tissue is <50% of tendondebride
degenerative tissue. Then tubularize the remaining tissue using a running,
absorbable suture.
b. If peroneus tertius or low-lying muscle belly is presentexcise it.
c. If lateral ligament instabilityuse Brostrom or Chrisman-Snook
d. If PB tear is >50% then resect the whole tendon (not sure about this personally)
and attach to PL.
5. Inspect floor of peroneal groove. If too shallow make larger groove (pg 303)
6. Use rongeur to prepare fresh-bleeding fibular bed, then reattach the superior peroneal
retinaculum through drill holes in lateral ridge. Go from deep thru holes to dorsal.
Suture the rest of superior peroneal retinaculum with pants over vest style.

 Post-op
o NWB splint for 1 week
o BK walking cast for 4-6 weeks

166 CROZER-KEYSTONE RESIDENCY MANUAL – SECOND EDITION


Posterior Tibial Tendon Repair-Substitution
 Stages
o I: Normal length with tendonitis or peritendonitis
 Surgical options: Debridement or repair
o II: PT is elongated, but RF is still flexible
 Surgical options: FDL transfer
o III: PT is elongated, RF rigid
 Surgical options: Triple arthrodesis
 Tendon is usually worse than you thought
 Always try conservative first-rest, arch supports, PT, oral anti-inflamatories, immobilization
with cast or brace
 Steroid injections are contraindicated.

 Procedure
o Almost always done with an osseous procedure
1. Incision is 10 cm proximal to tip of tibial malleolus and 1 cm posterior down behind the
medial malleolus to the navicular tuberosity’s plantar portion. (Follows the TP tendon)
2. At the upper end of the incision, the deep fascia is incised and the TP is exposed.
a. The TP lies very close to the posterior margin. Trace the tendon distally to its
insertion while leaving a 2 cm pulley just posterior to medial malleolus at the
level of the tibial plafond.
3. Determine the length of the TP tendon.
a. If TP is normal length, it’s stage Ithen do tendon debridement,
tenosynovectomy and sheath resections are done and close wound
b. If tendon is elongated, it’s stage II and FDL transfer is needed.
4. For debridement of tendon:
a. If fraying—smooth edges leaving major portion of tendon intact
b. If bulbous enlargement just tip of medial malleolus-an ellipse is removed from
bulb and tendon is sutured burying the knot
c. If longitudinal split exists-clear inner side of tendon of scars and approximate
scars
2. Tenosynovectomy—the outer portion of the tendon sheath distal to the pulley is
removed to prevent a possible reformation of stenotic tendon sheath
3. Inspect tendon for area of tear. Proximal to the region involved, the tendon will be
dull and white if the tear is old. Sometimes there is a transverse tear.
4. Transfer of the FDL: Detach FDL distal to the crossover area of FDL and FHL. Cut
the FDL under direct vision.
a. Optional: Suturing of distal end of FDL to FHL
5. Tag FDL with strong, non-absorbable suture in zigzag suture
6. Identify tuberosity of navicular and expose the inferior and superior surfaces of the
tuberosity
7. With 0.25 or 0.375 inch drill bit, a drill hole is done from superior to inferior. The drill
hole should come out inferior to the main surface of the PT
8. Leave FDL in its own sheath. Bring FDL into navicular drill hole from inferior to
superior. Pull through as tightly as possible with foot PF and supinated.

CROZER-KEYSTONE RESIDENCY MANUAL – SECOND EDITION 167


9. Suture the FDL with its non-absorbable suture into the capsule dorsally. Reinforce the
tendon by suturing the inferior portion of FDL (under navicular) into the TP tendon.
10. Assess the proximal TP muscle. Often the muscle will become fibrotic and stiff after a
TP dysfunction. Test the muscle by pulling on the proximal portion of the tendon.
a. If there is some elasticity (the muscle still has some function)—then do a side-to-
side suturing of FDL to TP with non-absorbable sutures, buried knot.
b. If the muscle is stiff—don’t suture the two muscles together.
11. Optional: Advancement of the spring ligament and the TN capsule
12. Alternative method: TP tendon repair with side-to-side suturing.
a. Limitations-cannot restore significant flatfoot deformity to normal alignment, but
it should relieve pain and improve function
b. Identify and resect the diseased section of the TP tendon. Suture tendon ends
together with non-absorbable suture
c. Then do a side-to-side repair with the FDL with non-absorbable suture.

 Post-op
o Without FDL transfer
 Jones compression cast with foot in PF and inversion for 1-2 days
 BK WB cast for 3 weeks
 Post op shoe and gradually move into shoe. May take months
o With FDL transfer
 Jones compression cast with foot in PF and inversion for 1-2 days
 NWB BK with foot in adduction and inversion for 3 wks.
 Remove sutures, NWB BK with foot in neutral for 3 more weeks
 Progress to WB as tolerated, PT

168 CROZER-KEYSTONE RESIDENCY MANUAL – SECOND EDITION


Tibialis Posterior Tendon Transfer (TPTT)
 Procedure
o Patient set-up
 Prone
 Thigh tourniquet
 General or spinal
o Four incisions
 One at TP insertion site on navicular
 One at middle distal 1/3 of leg, medial to tibial crest
 One at middle distal 1/3 of leg, 1 cm lateral to tibial crest
 One at lateral cuneiform-3rd metatarsal insertion site
1. First incision – at TP insertion on navicular
a. Release the TP from its insertion on navicular
2. Second incision – at middle distal 1/3 of leg medial to tibial crest
a. Pull TP up through this incision
3. Third incision – at middle distal 1/3 of leg 1 cm lateral to tibial crest
a. Separate TA from tibia
b. Expose interosseous membrane and make a window in it
c. Compress the posterior muscle mass. This will expose TP
4. Gently pull TP from medial incision through window in interosseous membrane with
blunt curved Kelly forceps and moist sponges
a. Be careful of NV bundle which lies under TP
b. Often there are muscle fibers attached distally from the window → gently pull
them free
5. Fourth incision – over lateral cuneiform or 3rd metatarsal
a. Insert tendon passer, Bozeman forceps or uterine packing forceps into insertion
site incision and retrograde up extensor sheath. Grab TP and retrograde.
b. Fixate TP to lateral cuneiform with foot in neutral position

CROZER-KEYSTONE RESIDENCY MANUAL – SECOND EDITION 169


Split Tibialis Anterior Tendon Transfer (STATT)/
Tibialis Anterior Tendon Transfer (TATT)
STATT
 Procedure
o Patient set-up
 Prone
 Thigh tourniquet
 General or spinal
1. Three incisions
a. One over TA insertion of medial cuneiform-1st metatarsal
b. One over anterior surface of leg just proximal to transverse cruciate ligament
c. One over peroneus tertius insertion
i. If the peroneus tertius is not present, the tendon can be inserted to cuboid or
sutured to the PB
2. Split the tendon with umbilical tape with a tendon passer in leg incision through to TA
insertion
3. Cut the lateral ½ of the TA tendon and retrograde that through to the proximal
window
4. Insert to peroneus tertius tendon

TATT
 Procedure
o Patient set-up same
o Note: Usually with TATT, the 3rd incision is over the lateral cuneif-3rd metatarsal and
the TA is transferred to this bone. However, it is possible to transfer the TA all the way
to the peroneus tertius sheath.
1. Three incisions
a. One over TA insertion of medial cuneiform-1st metatarsal
b. One over anterior surface of leg just proximal to transverse cruciate ligament.
c. One over lateral cuneiform
2. Tendon is separated from its insertion
3. Tendon is drawn up through insertion onto leg incision
4. With a tendon passer, bring tendon up through peroneus tertius sheath (same as the
EDL tendon sheath)
a. Be sure to be under extensor retinaculum
5. Insert TA into lateral cuneiform via hole and button

170 CROZER-KEYSTONE RESIDENCY MANUAL – SECOND EDITION


Note: These sections on Ankle Fractures are based on my notes from my personal experiences,
the texts I have previously referenced and from my lectures from the AO course in Davos,
Switzerland. The first thing they stated at the AO Course in Fracture Management was that they
used the Weber Classification and not the Lauge-Hansen system because the Lauge-Hansen was
too complicated. Therefore, the surgical choices are broken down by the Weber Classification.
However, you still want to learn the Lauge-Hansen system because that is what you will most
likely be quizzed/tested.

-Brett

Weber A Surgical Procedures


 Screw Placement
o Patient set-up
 Supine with bump under hip
 Thigh tourniquet
 General or spinal
1. Small incision at tip of lateral malleolus
a. Expose tip of malleolus by splitting calcaneofibular ligament longitudinally
b. Avoid tilting lateral malleolus toward the talus
2. Insertion point for medullary fixation is at lateral surface of malleolar tip
a. 4.0 cancellous screw (or malleolar screw) is inserted across fracture into the
proximal medial cortex of fibula above the fracture site
b. Insertion of a long screw (4.0 mm) across the fracture line into the medullary
canal of the proximal fragment
3. Avoid rotation or displacement of distal fragment as screw is inserted
a. K-wire can be added as temporary fixation
b. Note: Since the medullary device (screw) is straight, the lateral malleolus may be
inadvertently tilted toward the talus – this will result in narrowing of ankle
mortise and reduced motion
4. Close

 Tension Band Wiring


o Patient set-up
 Supine with bump under hip
 Thigh tourniquet
 General or spinal
1. Skin incision vertical and parallel to long axis of tibia directly over lateral malleolus
a. Note: A straight incision is often used because it can be extended. Try to avoid J-
shaped incision because they cannot be extended.
2. Dissect sharply down to bone
a. If undermining is necessary, do it just over periosteum

CROZER-KEYSTONE RESIDENCY MANUAL – SECOND EDITION 171


3. With a periosteal elevator or #15 blade, elevate the periosteum 2-3 mm proximal and
distal to the fracture line
a. Remember, be good to the soft tissues
4. Curette and irrigate fracture fragments to remove all hematoma
5. Inspect joint
6. Reduce fracture with towel clip on fracture fragment and guide it with periosteal
elevator
7. Insert two 0.045 or 0.062 K-wires. Insert from dist → prox from tip of lateral
malleolus across the fracture line, up the diaphysis of the fibula.
a. Insert at right angles to the fracture (this is pretty vertical)
b. Be careful not to violate the joint
8. 20-gauge wire is then passed through a transverse drill hole above fracture site and
placed in a figure-8 fashion around bent tips of protruding K-wires. The fragment
should be well-aligned and held securely in place with the wires.
a. Twist the 20-gauge wire and trim the excess wire off and make sure the twisted
tips lie flat against the bone
b. Note: Instead of a transverse drill hole above the fracture site, a screw may be
placed at the same level above the fracture line with a washer and tightened down
after the figure-8 of wire is wrapped around it.
9. Close

172 CROZER-KEYSTONE RESIDENCY MANUAL – SECOND EDITION


Weber B Surgical Choices
 Fixation Options
o 2 lag screws
o Posterior anti-glide plate
o Lateral plate with lag screw
 Weber B Ankle Fracture Characteristics
o Fracture at the joint line
o Corresponds with PAB or SER
 Lateral Malleolus Anatomy
o PB and PL posterior to fibula
o Superficial Peroneal Nerve anterior/medial to fibula
o Sural Nerve and Short Saphenous Vein is posterior and plantar to fibula
o FHL is posterior and mainly muscular at this level
o Proximally incision is between peroneus tertius (anteriorly) and PL and PB (posteriorly)
o Generally safe with an incision centered over fibular fracture site

 Fixation With 2 Lag Screws


o Indications to use 2 Lag Screws
 If fibular fracture is spiral (2x diameter of bone) + not comminuted + not
osteoporotic → then sufficient fixation can be achieved with only 2 lag screws
o Advantages
 Allows for a smaller incision
 Hardware is not prominent and usually does not have to be removed
 Will not interfere with syndesmotic screws (if they are needed)

o Procedure
1. Incision is made slightly anterior to midline of fibula
a. Incision is down to bone without too much layered dissection
2. Irrigate and clean up wound edges
3. Restore fibular length
a. Hold with reduction clamps
b. Check with C-arm
4. Insert two 3.5 screws (or 2.7 if small pt) from ant → post using AO lag
techniques
a. C-arm to confirm position
5. Close

 Lateral Plate and Anterior-to-Post Lag Screw


o Most common surgical approach for Weber B fibular fracture
o Ideally, you want to cross 5 cortices of fixation proximally and distally. If using a
buttress plate, only unicortical screws are used and most likely only 3 cortices will be
fixated distally.

CROZER-KEYSTONE RESIDENCY MANUAL – SECOND EDITION 173


o Procedure
 Patient set-up
 Supine with lateral bean bag/bump
 Thigh tourniquet
 General or spinal
1. Incision directly over fibular fracture site. May make incision slightly anterior to
midline. (If posterior malleolus fracture is present, then make it posterior.)
a. Do not do too much dissection. Make incision down to periosteum
preserving soft tissues. Any undermining should be done at the periosteum
level.
2. Elevate periosteum 2-3 mm from the fracture line. The full anterior and
posterior portions of the fracture line must be exposed.
3. Curette and irrigate fracture fragments and hematoma
4. Explore talus for any osteochondral defects
5. Reposition fracture (increase deformity, distract, reposition) using towel clip and
periosteal elevator to assist
6. Hold fracture in anatomical position with bone clamps
7. Reassess anatomical position
a. Can use bone clamps, lobster claw, or K-wire across fracture to stabilize it
b. Note: Keep in mind the next step with the lag screw, your clamp will most
likely be in the way, so position it thoughtfully!
c. Can use posterior spike of fracture as guide for adequate reduction
d. C-arm for verification of position
8. Insert lag screw (3.5 mm cortical) ant → post, as perpendicular as possible
across the main fracture line
a. Screws must engage posterior cortex, but should not extend so far as to
disrupt the peroneal tendon sheath
9. Apply ⅓ semi-tubular plate laterally
a. Bend plate to appropriately contour of fibula
b. Plate size is based on the number of screws around fracture site and
position on fibula
i. 3 proximally (3.5mm cortical screws)
ii. 2-3 distally 4.0 cancellous screws
iii. Run distal screws short so to avoid violating fibular-talar joint
10. C-arm to check position
11. Close

 Posterior Antiglide Plate


o Advantages
 Achieves strong fixation even in osteoporotic bone
 Hardware generally does not cause symptoms or wound necrosis (or at least less
than a straight lateral plate would)
 Does not interfere with syndesmotic screw insertion
 Distal screws obtain better purchase, because they engage a thicker part of distal
fibula

174 CROZER-KEYSTONE RESIDENCY MANUAL – SECOND EDITION


 Engage 2 cortices without risk of joint penetration (essentially no risk of intra-
articular screw insertion because of plate placement)
 Biomechanically, this construct is stronger than lateral plate fixation, especially in
osteoporotic bone (i.e. good in elderly!)
 Loss of fixation is rare due to the stability of the construct
 Posterior plate provides better fixation with posterior comminution
 Posterior incision allows access to the posterior malleolus, when direct fixation is
required
o Disadvantages
 Technically more challenging
 May irritate peroneal tendons in minority of patients, but this often spontaneously
resolves in 4-8 weeks
 Peroneal tendon subluxation
 Should not be a problem is the tendon sheath is left intact

o Procedure
 Patient set-up
 Supine with lateral bean bag/bump
 Thigh tourniquet
 General or spinal
1. Incision is made along the most posterior border of the fibula at fracture level
a. The plate often lies slightly posterolaterally (rather than directly post), thus
the incision will be away from the plate
b. Incision is carried down to peroneals, but does not violate the peroneal
sheath
c. Incision proceeds over the lateral edge of peroneals which are usually
retracted posteriorly
2. Some of proximal retinaculum may need to be released to expose distal fibula
a. Clear periosteum off fibula
3. Fracture reduction – Ideally, anatomic reduction should be achieved prior to
plate application
a. Reduction can be held with a single K-wire or lag screw
b. Apply lag screw from ant → post so not interfere with plate application
4. Plate application – ⅓ tubular plate is applied to posterior surface of the fibula
a. 4-hole plate – Classically
b. 6-hole plate – More recently
c. Because posterior surface of fibula is straight, contouring of the plate is
usually not necessary
d. Due to the lateral bow of fibula, the plate sits best posterolaterally
5. Proximal screw insertion
a. 1st 3.5 mm cortical screw is placed proximally, through the plate, 2 mm
above posterior fracture line
b. Plate helps prevent proximal gliding of the distal fragment
c. If anatomic reduction is achieved, the proximal screw can be tightened
down

CROZER-KEYSTONE RESIDENCY MANUAL – SECOND EDITION 175


d. If anatomic reduction has not been achieved, then do not fully tighten the
screw yet
i. Apply bone clamps to both fracture fragments and distract out to
length
ii. Apply slight internal rotation to distal fragment
iii. Proximal screw is now tightened, and fragment should be properly
aligned
6. Remaining proximal 3.5 mm cortical screws are inserted
7. Lag screw insertion
a. Lag screw is then inserted post → ant through the first plate hole which is
distal to the posterior fracture line
i. Because the posterior cortex is thin, lag screw must be inserted
through plate (which serves as solid posterior cortex)
b. The screw must be angled slightly proximally in order to be perpendicular to
the fracture site
i. Remember to remove initial lag screw (inserted ant → post) if used
c. Note: Lag screws improve fracture reduction but do not significantly improve
anti-glide strength
d. Okay to leave screw slightly long because screw is directed away from the
peroneal tendons
e. If possible, a 2nd lag screw can be inserted using the same technique
8. Optional – Insert distal screws: Technically the more distal screws are not
necessary, but 4.0 mm cancellous screws may be inserted. Because there is no
risk of joint penetration, longer screws can be used to get a better hold on bone.
9. Close

176 CROZER-KEYSTONE RESIDENCY MANUAL – SECOND EDITION


Medial Malleolar Fractures
 Fixation Options
o 2 Lag Screws
o Medial Plate
o Tension Band Wiring
 Patient set-up for all types
o Supine
o General or spinal
o Thigh tourniquet

 Two Lag Screw Procedures


o Two methods
 Two Screw Fixation either open or percutaneous (a.k.a. two stab incision)
 Lag Screws: True lag screws are used to counteract and neutralize a tension
failure on the medial side
 Brett Chicko Note: I prefer to do an open procedure over a stab incision because
with a fracture of the medial malleolus, there usually is some soft tissue in-
between the fracture fragments

 Two Stab Incision Procedure (Percutaneous)


o Most common procedure for medial malleolus fracture
1. Two stab incisions at tip of medial malleolus
2. Blunt dissect (with hemostat)
3. K-wire from distal tip across fracture line at right angles into proximal section
a. Aim somewhat vertically to avoid the ankle joint
b. C-Arm to check position
4. Insert two 4.0 mm cancellous partially-threaded cannulated screws
a. C-Arm to check position
5. Close

 Open Procedures (Screw/Plate or Tension Band)


o Indications
 For plate insertion
 For two screw insertion with better visualization
 Comminuted fractures
 Difficulty in reduction with percutaneous method

o For Screw Fixation or Plate Insertion


1. Incision – vertical and parallel to with long axis of tibia directly over medial
malleolus
a. A straight incision is often used because it can be extended. Try to avoid J-
shaped incision because they cannot be extended.
2. Dissection sharply down to bone. If undermining is necessary, do it just over
the periosteum.

CROZER-KEYSTONE RESIDENCY MANUAL – SECOND EDITION 177


3. With a periosteal elevator or #15 blade, elevate the periosteum 2-3 mm from the
fracture lines
a. Remember to be good to the soft tissues!
4. Curette and irrigate fracture fragments to remove all hematoma
5. Inspect joint
6. Reduce fracture with towel clip and guide it with periosteal elevator
7. Stabilize with 2 K-wires
a. Insert from distal → proximal from tip of malleolus across the fracture
b. Insert at right angles to the fracture (this is pretty vertical) with care not to
violate the joint
8. Insert two 4.0 mm cancellous bone screws
a. Cannulated screws may be used
b. If not cannulated, remove K-wire and insert screw in K-wire hole
9. If using a plate, insert with 3.5 mm cancellous screws. Run the proximal screws
short so they don’t violate the ankle joint.
a. Other options: ⅓ semi-tubular, DCP, T, Clover Leaf
b. Note: For a "push off" (shear) fracture, the purpose of the plate is to provide
an anti-glide or a buttressing effect
10. Close

o Tension Band Wiring


 This can be used for the very small medial malleolar fragments or comminuted
 Studies differ onto effectiveness of tension band
 Tension bands are rarely used now for medial malleolus
 If you always place two screws, you are going to comminute a few medial
malleoli and you’ll to be very unhappy. Then you will need this tension band
technique for salvage.
 Also, pts hate these wires. Anytime you do a tension band wire and leave a wire
long (in an area where there is movement) patients will hate you a lot.
1. Incision – vertical and parallel to with long axis of tibia directly over medial
malleolus
a. A straight incision is often used because it can be extended. Try to avoid J-
shaped incision because they cannot be extended.
2. Dissect sharply down to bone. If undermining is necessary, do it just over the
periosteum.
3. With a periosteal elevator or 15 blade, elevate the periosteum 2-3 mm from the
fracture lines
a. Remember to be good to the soft tissues!
4. Curette and irrigate fracture fragments to remove all hematoma
5. Inspect joint
6. Reduce fracture with towel clip on fracture fragment and guide it in with
periosteal elevator
7. Two parallel K-wires (0.045 inch or 0.062) are inserted at distal end of fibula
and engage the proximal medial cortex above fracture site

178 CROZER-KEYSTONE RESIDENCY MANUAL – SECOND EDITION


8. 20-gauge wire is then passed through transverse drill hole (or a cortical screw
may be placed instead at this level, which you would wrap the wire around and
tighten the screw down after the wrapping) above fracture site and placed in a
figure-8 fashion around bent tips of protruding K-wires
a. Twist end of wire and trim. Make sure to bend it so it sits close to bone.
9. Let the first year close

CROZER-KEYSTONE RESIDENCY MANUAL – SECOND EDITION 179


Posterior Malleolar Fracture
 When to do an ORIF?
o When > 25% of posterior articular surface is involved as seen on lateral view
o Fracture is displaced >2 mm
o There is posterior subluxation of talus
o If fracture prevents reduction of tibia

 Procedure
o Patient set-up
 Use a posterior lateral approach (similar to the one used for a fibular antiglide
plate)
 Lateral or prone
 General or spinal
 Thigh tourniquet
1. Incision is at posterior border of fibula, if fibula needs to be reduced, do that first
a. Hold with temporary fixation
b. Brett Chicko Note: According to Coughlin – Do the definitive fixation of fibula
after the post malleolus because of lack of exposure after fibula is fixated.
However, I have seen the fibula fixated first and then the post malleolus was
addressed.
2. Bluntly dissect between the PB/PL and the FHL (muscular at this level) to the
posterior surface of the tibia. Must get exposure of entire fracture fragment.
3. Reduce fracture
a. The fracture reduction is determined by palpation and visualization of extra-
articular fracture line and C-arm. Cannot directly visualize the intra-articular joint
because talus is in the way.
b. Note: Reduction of fibular fracture most likely will reduce the posterior malleolus
because of firm attachment of post tibiofibular ligament. Ligamentotaxis!
c. If difficulty reducing fracture, DF foot may give slack to ligaments and posterior
capsule
4. Hold reduction with large reduction clamp
5. Insert 2 K-wires to the fragment in place (for 4.0 mm partially threaded cancellous lag
cannulated screws)
6. Insert screws post → ant
a. Insert at right angles to the fracture
7. Alternate fixation: Stab incision anteriorly, insert 4.0mm cortical screw ant → post
a. Check C-arm for position
8. Let the 1st year close

 Post-op
o NWB until union is solid. This may take up until 3-4 months.

180 CROZER-KEYSTONE RESIDENCY MANUAL – SECOND EDITION


Syndesmotic Repair
 After every ankle fracture repair, evaluate the syndesmosis with intra-op stress exam
 Syndesmotic separations are unstable and should be stabilized
 Remember the AO principle of stable fixation if non-articular
 Indications for Syndesmotic Fixation
o Irreparable medial joint injury w/ disruption of syndesmosis
o High fibular (Weber C) fracture >15 cm above the joint line
o Medial ligament injury, syndesmotic disruption, talar shift w/o fracture of fibula
o Widening of the tibiofibular "clear space" as a result of disruption of the syndesmosis.
The clear space is normally < 5 mm wide.
 Injury Patterns
o Isolated
o Syndesmotic injury with fibular fracture
o Syndesmotic injury with medial injury
 Choices of Syndesmotic Screws
o 4.5 mm screw (most common) x2 or 3.5 mm screws in smaller patients
o Bioabsorbable fixation (polylevolactic acid)
o Fiberwire (Tightrope or others like this…technique as per manufacture’s guidelines)
 Proper level for a Syndesmotic Screw
o Screws should be parallel to joint line
o 1 cm prox to syndesmosis or 4 cm prox to ankle joint
o If too low, can pass through distal tib-fib articulation causing pain
o If too high, may cause tip of fibula to go outward

 Procedure
o Patient set-up
 Prone with hip bump
 General or spinal
 Thigh tourniquet
1. Stabilize the fibular fracture before the syndesmosis
a. Use a plate on the fibula
b. Fibula should be reduced posteriorly into the tibial sulcus
c. The syndesmosis should be reduced before the screw(s) are inserted
2. Dorsiflex foot 5°
3. Stab incision on fibula
a. Use C-arm to find correct level: 1cm proximal to syndesmosis and/or 4 cm prox
to ankle joint
4. Insertion of screw (4.5 mm cortical fully-threaded)
a. Because fibula is posterior to tibia, aim screw through posterior-lateral fibula
to anterior-medial tibia at an angle of 25-30° anteriorly
b. Perpendicular to long axis of bones, parallel to the ankle joint
c. Engage 3 cortices or 4 (depending on doctor preference)
d. Do not lag! Do not over-tighten!

CROZER-KEYSTONE RESIDENCY MANUAL – SECOND EDITION 181


 Post-op
o Screws generally will not loosen or break if the ankle does not dorsiflex past neutral
o Patients are allowed to WB after 6 weeks in a short leg cast or walking boot
o Routine removal of screw 8-12 weeks after surgery

182 CROZER-KEYSTONE RESIDENCY MANUAL – SECOND EDITION


ORIF Calcaneus
 Indications
o Injuries that would do poorly without surgery, such as severely displaced intraarticular,
widening of heel, horizontally oriented talus, severe soft tissue injury, and high-energy
mechanism
 Contraindications
o Severely comminuted
o Impaired vascularity
o Infection
o Severe neuropathy
 Essex-Lopresti Classification
o Primary fracture line runs from ant-lat → post-med through the STJ (mostly through
the posterior facet). As the primary fracture line progresses, this will lead to lateral
wall blowout and decrease in calcaneal height.
o Both Essex-Lopresti types start with the primary fracture line
o Type 1 – vertical force will lead to tongue type
o Type 2 – more horizontal force will lead to joint depression

 Procedure
o Patient set-up
 Lateral decubitus position
 General or spinal
 Thigh tourniquet
o General order of reduction
 Anterior process
 Medial wall
 Posterior facet
 Lateral wall
1. Incision is curved behind the lateral malleolus. The proximal portion is halfway
between the anterior portion of the Achilles and the peroneal tendons. The line
progresses distally, around the lateral malleolus, and then runs parallel to the bottom of
the foot, ending up roughly at the C-C joint.
a. Watch out for sural nerve and peroneal tendons
2. Sharply dissect down to bone, create a flap with the CFL and the peroneals and flap that
anterior-superiorly
3. Insert two 0.062 K-wires into the talus and bend K-wires upward to hold this flap up
4. Expose STJ, remove hematoma and small fracture fragments via irrigation and rongeur
5. Identify the fracture lines in the anterior calcaneus that extend medially
a. Determine if the fracture line progresses to the C-C joint
6. The anterior process is typically elevated. Therefore, the anterior process needs to be
retracted plantarly. Use a retractor or lamina spreader between the talus and the
anterior process. Fix with K-wire.
7. Identify the fracture line progressing from ant-lat → post-med (the primary fracture
lines), separating the posterior facets from the anterior and middle facets

CROZER-KEYSTONE RESIDENCY MANUAL – SECOND EDITION 183


8. The posterior facet is usually PF. With the use of a periosteal elevator, lift up the
posterior facet. Hold with K-wire directed from the anterior process laterally to the
posterior facet medially.
a. The lateral part of the posterior facet is retracted laterally or removed. This allows
visualization of the medial posterior facet
9. A 4.0 Shantz pin is inserted into the tuberosity fragment from post → ant (from the
back of the heel into the posterior tuberosity). This is used as a lever to reduce the
fragment plantarly, medially, and into slight valgus.
10. When the medial wall of the tuberosity lines up with the medial wall of the facet
fragment, it is held with two 0.062 K-wires. The K-wires are inserted from the
posterior aspect of the tuberosity and directed to the sustentaculum tali (be careful not
to damage articular cartilage). Use C-arm for this.
11. After the anterior process and the medial wall are reduced, the posterior facet is
reduced. Match the lateral fragment to the medial fragment. Insert 0.062 K-wires
into the anterior and posterior margins.
12. Get intra-op X-rays. If alignment is good, insert a 2.7 mm cortical lag screw below the
subchondral surface.
a. Reconstruct the lateral wall if necessary. Bend and insert plate.
b. Best bone for a plate is subchondral bone deep to the C-C joint, the subchondral
bone near Achilles tendon insertion, and the dense bone of the sustentaculum tali
c. May need to fill in deficit with bone chips, Grafton or other bone substitute
13. Insert drain and close

 Post-op
o Course depends on the amount of damage
o If minimal displacement, 6-8 weeks ROM exercises and NWB
o If severe displacement, >12 weeks ROM exercises and NWB

 Complications
o Relatively common
o Infection
o Delayed wound healing
o Sural nerve
o Tibial nerve problems (more likely from injury rather than surgery)

184 CROZER-KEYSTONE RESIDENCY MANUAL – SECOND EDITION


ORIF Talar Neck Fractures
 ORIF should be done even if Hawkins type II was close reduced because this type of fracture
will inevitably develop an equinus contracture that happens with prolonged casting in PF

 Talar neck fracture WITHOUT dislocation


o Procedure
 Patient set-up
 Supine with bump under ipsilateral hip
 Have C-arm ready
1. Anterior-medial incision – made from anterior aspect of the medial malleolus to
the dorsal aspect of the navicular tuberosity
a. Dissect carefully down, go dorsal to TP tendon. Don’t disrupt deltoid
ligament because this might disrupt some of the vasculature to the talus.
2. Remove hematoma. Don’t dissect the soft tissues off the talus dorsally and
plantarly because this might disrupt the blood supply to the talar neck.
3. Anterior-lateral incision – starting from anterior margin of lateral malleolus to the
base of the 3rd or 4th metatarsals
a. This allows confirmation of the reduction of the talar neck
b. It also permits removal of foreign bodies
4. Incise inferior retinaculum
5. Retract EDL and peroneus tertius. Retract EDB dorsally.
6. Remove all fragments. Probe STJ blindly for fragments.
7. Reduce the fracture
a. Careful not to have comminution or reduce into varus position.
8. Insert two 2.5mm titanium screws (can be used with MRI). Do not lag! A lag
screw might send talus into varus.

 Talar neck fracture is displaced (as in Hawkin’s Type III – STJ and Ankle joint)
o Procedure
1. Anterior-medial incision – extend the incision over the medial malleolus and the
distal aspect of the tibia
2. Go into the space between tibia and Achilles. The body of the talus will be
visualized.
3. A femoral distractor may be needed. Put pins in the tibia and calcaneus.
4. Manually place the body of the talus back into mortise
a. If the talus will not return to the mortise, a medial malleolar osteotomy
will have to be done
i. For medial malleolar osteotomy
1. Identify the ankle joint for tibia
2. Make 2 retrograde 2.5 mm drill holes in the medial malleolus
across the osteotomy site
3. Release the anterior portion of the capsule off the deltoid
ligament as well as a portion of the TP sheath. Protect the TP
tendon.
4. Incise the periosteum about 5-10 mm superior to the ankle joint

CROZER-KEYSTONE RESIDENCY MANUAL – SECOND EDITION 185


5. With oscillating saw, cut the transverse portion, then the 2
vertical portions
6. Reflect the medial malleolus distally. Don’t damage the deltoid
ligament.
7. Manually place the talus back into its place
5. Anterior-lateral incision – as above
a. Fix medial malleolus with two 4.0 mm cancellous titanium screws

 Post-op
o Post-splint or boot
o Do not do ROM exercises until wound healing is done
o NWB for 8-12 weeks until trabeculae cross the fracture

 Complications
o AVN
o Arthrofibrosis
o Malunion, nonunion
o Skin necrosis

186 CROZER-KEYSTONE RESIDENCY MANUAL – SECOND EDITION


Tibial Periarticular Fx Reduction & Fixation
 Indications
o Pilon or Tibial Plafond fractures
o Articular displacement of >2mm or unacceptable axial alignment
o Open fractures
o Neurovascular injury due to fracture
 Pre-op planning
o Check NV, compartment syndrome, soft tissue injury
o Radiographs: AP, Lat, MO
o Whole tibial shaft
o Foot radiographs
o CT can be useful

 Procedure
o Staged surgery—First part immediate once the patient has stabilized (usually 12-18
hours) with ORIF of fibula and external fixator for tibia. Kitaoka recommends EBI or
Orthofix
o 1st Stage
1. Fibular incision-slightly posterior-laterally to increase the width of the skin bridge
with the later anterior incisions.
2. Do not do calcaneal skeletal traction- (even with a Bohler-Braun frame) this
pulls patient out of bed and displaces foot posteriorly
3. Fix Fibula fracture
4. Apply Ex-Fix onto tibia
5. When the soft tissue edema has subsided (usually 10-21 days), ORIF can be
performed
o 2nd Stage
 Patient set-up
 Pt supine
 General or spinal
 Thigh tourniquet
 Take frame off tibia but don’t remove the pins. Have circulator sterilize the frame.
This may be used later in the case for distraction.
 Anterior-medial incision
1. Locate the fracture fragment. If the fragment is anterior-medial: incision
begins just lateral to the medial crest of tibial shaft. Extend the incision
distally across the ankle joint, staying just medial to tibialis anterior
2. Identify the anterior tibia tendon sheath. Once identified, create a full
thickness flap by incision the tendon sheath and the extensor retinaculum.
Bring this incision down to bone/joint down to the periosteum. Do not strip
the periosteum or remove any fat unnecessarily
3. Identify fracture ends, debride and irrigate. Remove all clots
4. Reduce the fracture. This incision works well with a medial pilon plate

CROZER-KEYSTONE RESIDENCY MANUAL – SECOND EDITION 187


 Anterior Midline Incision
 Use this incision when the fracture is a pure anterior crush injury. This
injury gives good exposure and will allow for easy placement of low profile
anterior tibial plate
1. Incision is made between the TA and the EDL
a. Identify the superficial peroneal nerve, the artery and the deep
peroneal nerve. Retract all of this laterally
 Anterior-Lateral Incision
 Use this with large lateral fragments, such as the Tillaux-Chaput avulsions.
Careful with this incision because it may jeopardize the skin from the fibular
incision.
 This is why you usually use a posterior lateral incision for the fibula.
1. Incision starts proximal to the ankle joint and slightly medial to Chaput’s
tubercle and extends distally in a strait line toward the base of the 3rd and 4th
mets.
a. Superficial peroneal nerve is protected
2. Incise through the superior and inferior extensor retinaculum
3. Mobilize the peroneus tertius and EDL, the deep peroneal nerve and the
anterior tibial/DP artery
4. Distally the EDB is seen and can be retracted laterally or detached
5. Protect the lateral branch of deep peroneal nerve and the lateral tarsal artery
 Fixation
o Fix the jigsaw puzzle, then use reduction clamps followed by 1.6 mm k-wire
o Canulated screws are rarely used
o If a piece is small, a bioabsorbable pin can be used
o Once the screws are in place, a cancellous graft can be used
o Apply plate. Options are:
 Medial pilon plates for rotational and varus valgus injuries
 Anterior pilon plate for anterior crush plates. Note Dr Lutz states to always use an
anterior plate.
 Supplemental washer plates may be needed. Ex: spider washer plate
 Closure
o Small drain, close etc.

 Post-op
o Jones compression dressing and splint
o At 1 week change to compression stocking and removable boot and start ROM.
o Pt is to wear the boot at all times, even sleeping or the patient may develop equinus
deformity
o Start formal PT about 4-6 weeks, only after the wounds have healed
o WB at 3 months if radiographic evidence of healing
o Outcome is based on the fracture at presentation

188 CROZER-KEYSTONE RESIDENCY MANUAL – SECOND EDITION


 Technical Pearls
o Joint surface should be reconstituted first because anatomical malalignment is not
acceptable. The ankle will accept some axial malalignment, but not articular.
o Shaft reconstruction is performed second
o Reconstruct the joint with isolated lag screws followed by neutralization plate of the
metaphyseal-diaphyseal component
o Joint surface needs to be anatomic reconstructed. If there are centrally depressed pieces,
the perimeter fragments need to be retracted and the central ones need to be elevated
 Technical Pitfalls
o Doing definitive surgery too soon because fracture patterns are not clear with all of the
soft tissue swelling
o Unstable EX-FIX
o Failure to stabilize the fibula
 Complications
o Infection
o Wound complications
o Malunions
o Post-traumatic complications at 1-2 years

CROZER-KEYSTONE RESIDENCY MANUAL – SECOND EDITION 189


Ankle Arthroscopy
 Portals
o Most commonly used – Ant-med, ant-lat, and post-lat
o Anterior-medial port
 Medial to TA
 Lateral to saphenous vein and nerve
o Anterior-lateral port
 Lateral to Peroneus Tertius
 Medial to intermediate dorsal cutaneous nerve
o Posterior-lateral port
 Lateral to Achilles tendon, 1-2 cm distal to anterior ports
o Anterior-central port
 Just lateral to FHL
 Medial to DP and deep peroneal nerve, medial dorsal cutaneous nerve crosses
over FDL at this level and may be lateral
 Due to all of the potential complications, this port is usually contraindicated
o Posterior-medial port
 Medial to Achilles tendon
 Also in this area: FHL, FDL, posterior tibial nerve and artery, calcaneal artery
 Due to all of the potential complications, this port is usually contraindicated
o Posterior-central port
 A.k.a. ―trans-Achilles‖ because it is through the Achilles
 This is usually contraindicated…

 Procedure
 Insertion of scope
1. Mark anatomic landmarks (medial and lateral malleolus, superficial peroneal nerve, TA
and peroneus tertius, if this is not present use EDL)
2. Use 2.7 or 4.5 scope
 For Anterior-medial and Anterior-lateral ports
3. Find the ankle joint, insert 18-gauge needle into joint. Fill the joint with 20 cc of NSS
or Lactated Ringers
4. Incise the skin only, use blunt dissection down to the capsule
5. Insert cannula and blunt obturator. Insert scope.
6. With direct vision of scope, insert 18-gauge needle into lateral port, find the needle
with the scope (―Triangulate‖ your position!). Be careful of the superficial peroneal
nerve.
7. Incise the skin over the 2nd port hole, blunt dissect and use obturator to complete the
port
 For Posterior-lateral port
8. Go lateral to Achilles tendon approx 1-2 cm distal to anterior port levels (this will be
just distal to posterior syndesmotic ligament)
9. Cannula is used for dedicated inflow

190 CROZER-KEYSTONE RESIDENCY MANUAL – SECOND EDITION


 Joint examination
10. Look for anterior joint synovitis and shave with 2.9 or 3.5 shaver
11. Always remember to do a good irrigation, often times this may be all that is needed
 Anterior ankle exostosis approach
12. Reflect the capsule by putting the shaver against the osteophyte and lifting the capsule
off of it
13. Use a 4 mm burr from the ant-lat port while viewing from ant-med port. You may
also use a rongeur or osteotome.
14. Switch portals and do lateral portion
15. C-arm or intra-op radiographs may also be used
 Anterior medial exostosis approach at tip of medial malleolus
16. Make secondary port approx 1-2 cm medially and slightly distal to ant-medial port
17. Be sure to suture close all ports used

CROZER-KEYSTONE RESIDENCY MANUAL – SECOND EDITION 191


Arthroereisis
 Procedure
o Patient set-up
 General or spinal
 Thigh tourniquet
 Pt heel is resting over edge of bed
1. Incision over sinus tarsi
2. Blunt dissect to sinus tarsi
3. Insert guide pin so it abuts the anterior aspect of the body of talus. Advance pin until it
tents the skin medially. Can C arm here, or at any step.
a. Axis: distal lateralprox medial. Proper insertion of probe should cause the distal
aspect of the probe to exit just superior to the tibialis post tendon and anterior and
slightly inferior to medial malleolus—this is the direction you’re aiming
1. The torpedo shaped probe is inserted until it tents the skin. Make incision.
4. Rotate probe clockwise and counterclockwise to dilate the tarsal canal.
5. The guide pin is then placed within sinus tarsi.
6. Most often the 8 and 10 mm implants will be used. Use the sizers, the correct size
should allow 2-4° of subtalar eversion.
7. Next, use the trial implant of the above size. Check ROM, and clinical correction are
assessed. Use C-arm at this point.
8. Now use actual implant on screw driver with nose cone. Apply over guide pin. Screw
in clockwise. Insert no more than 1 cm medial to calcaneal wall and no more than ½
way across talus
9. Once inserted, the implant should be resting on floor of sinus tarsi. Take x-ray
10. When satisfied, remove guide pin and inserter. Irrigate with NSS. Re-valuate motion,
close in layers.

 Post-op
o WB in cast for 2 weeks, gradual return to shoe gear.

192 CROZER-KEYSTONE RESIDENCY MANUAL – SECOND EDITION


Arthrosurface 1st Metatarsal Head Implant
 Indications
o Hallux limitus/rigidus + Good bone stock!
o Also a good procedure if arthrodesis is not an option
 Contraindications
o Significant bone demineralization or inadequate bone stock
o Inadequate skin, musculotendinus or NV system status
o Inflammatory, rheumatoid arthritis, sepsis, infection and osteomyelitis
o Pts known to have sensitivity to metal alloys typically used in prosthetic devices

 Simple Technique Guide Steps


o Step 1 – Drill Guides, match with Articular Component
o Step 2 – Cannulated Pin, Drills
o Step 3 – Tap
o Step 4 – Driver
o Step 5 – Tap Cleaner
o Step 6 – Trial Cap
o Step 7 – Centering Shaft
o Step 8 – Contact Probes
o Step 9 – Circle Cutter and Surface Reamer
o Step 10 – Sizing Trial
o Step 11 – Implant Holder, insert Articular Component and Impactor
o Size is usually 15

 Procedure
1. Dissect down and expose 1st metatarsal head (similar to an exposure for a bunion)
2. Use Drill Guide to locate the axis normal to the articular surface and central to the
defect
a. Be sure to choose a Drill Guide where the diameter circumscribes the defect
3. Confirm Articular Component diameter by matching it to the Drill Guide
4. Place Guide Pin through the Drill Guide into bone
a. Make sure its central to defect
b. It is very important to verify that Drill Guide is seated on the curved articular
surface such that four points of contact are established. A normal axis and
correct Articular Component diameter are necessary for proper implant fit.
5. Place Cannulated Drill over Guide Pin and Drill until the proximal shoulder of drill is
flush to the articular surface
6. Tap hole to etched depth mark on tap
a. Optional – insert bone cement into pilot hole
7. Place Driver onto the Taper Post over the guide pin and advance until the line on the
Driver is flush with the height of the original articular cartilage level
a. Optional – advance the Driver further to decompress the joint
8. Remove guide pin
9. Clean taper in Taper Post with Taper Cleaner

CROZER-KEYSTONE RESIDENCY MANUAL – SECOND EDITION 193


10. Place Trial Cap into taper post to confirm correct depth of Taper Post
a. The peak height of the trial cap must be flush or slightly below the existing
articular cartilage surface to avoid the articular component from being place
above the surface of the defect
i. Adjust depth using the driver to rotate the taper post (clockwise to
advance)
11. Place Centering Shaft into Taper Post
12. Place Contact Probe over Centering Shaft and rotate around Centering Shaft. Read
contact probe to obtain offsets at four indexing points.
a. Superior/inferior and medial/lateral
b. Select appropriate Articular Component using Sizing Card
13. Remove Centering Shaft and replace with Guide Pin. Advance Circle Cutter back
and forth.
a. Don’t bend guide pin
b. Score articular cartilage down to subchondral bone
14. Choose appropriate Surface Reamer based on the offsets. Drill Surface Reamer over
Guide Pin until it contacts the top surface of Taper Post.
a. Begin rotation of Surface Reamer prior to contacting bone to avoid chipping
articular rim
15. Remove Guide Pin and clean Taper Post to remove any debris from implant bed
16. Place the Sizing Trial into the defect that matches the offset profile selected
a. Confirm that the Sizing Trial is equal or slightly recessed to edge of articular
cartilage
b. If the Sizing Trial is proud at the edge of articular cartilage, ream with the next
appropriate sized reamer and matching Sizing Trial
17. Use the Implant Holder (attached to suction wall tubing) and align the Articular
Component on the holder with appropriate offsets
18. Insert implant into Taper Post
19. Tap the Impactor gently against the implant to seat it against the bone.
20. Check ROM of 1st MPJ and close in appropriate layers

194 CROZER-KEYSTONE RESIDENCY MANUAL – SECOND EDITION


Brostrom-Gould Repair
 Indications
o Chronic ankle instability unresponsive to conservative treatment
o Athlete with ankle instability
 Contraindications
o Fixed varus heel type (need to correct with Dwyer)
o People over 200-225 lbs (use Evans with PB repair)
o Peroneal weakness (i.e. CMT)

 Procedure
o Patient set-up
 Supine with bump (to internally rotate leg)
 Thigh tourniquet
 General or spinal
 Optional – Bump under foot
2. Incision curvilinear over anterior border of fibula, stop at peroneal tendons
a. Be careful of sural nerve, intermediate dorsal cutaneous nerve, peroneal tendons
b. You will may have to ligate lesser saphenous branch of nerve
3. Dissect down to capsule, incise it from 2-3 mm from border of fibula
a. Leave a cuff for later attachment
4. Find CFL by reflecting peroneals and incise it
5. Put foot in neutral DF and slight eversion
6. Resect necessary capsule, reapproximate using 0 or 2-0 absorbable (or non-absorbable)
suture starting with the CFL then ATFL (extend incision if necessary)
7. Test for full range of DF and PF (gently)
8. Identify extensor retinaculum (should be distal), its fibers run perpendicular to ATFL and
CFL (extend incision if necessary)
9. Mobilize extensor retinaculum. Pull it over the repaired capsule and attach to the tip of
the fibula using 2-0 absorbable suture.
10. Check again for ROM and stability
11. Close

 Post-op
o Posterior splint 3-5 days
o BK walking cast for 3-4 weeks
o Air-type stirrup for an additional month with ROM exercises

CROZER-KEYSTONE RESIDENCY MANUAL – SECOND EDITION 195


Fibular Derotational and Lengthening Osteotomy
 General
o Tramatic displacement of talus is associated with displacement of lat malleolus
o Malalignment is characterized by distal fibular shortening, lateral shift or malrotation
o Increases pressure in mid-lateral and posterolateral quadrants of the talar dome
o Goal of Fibular Derotational and Lengthening Osteotomy is to restore the sensitive WB
area to normal anatomic relationship
 Radiographs (Can also use CT and MRI)
o Abnormalities of talar position is seen on x-rays. Check for (compared to other side)
 Widening of medial joint space
 Talar tilt
 Fibular shortening
o Mortise View—Check for:
 Equidistant and parallel joint space with no medial widening
 Shenton’s line of the ankle
 A dense subchondral supporting bone creates a radiographic line that can be
followed over the syndesmotic space from tibia to fibula (Kitoaka pg. 501)
 This should be even and continuous between the two bones
 Unbroken curve between the lateral part of the articular surface of the talus and
the distal fibular recess
 Talar tilt.
 This should be parallel or within 3° of parallel
 Abnormal seating of fibula in the incisura fibularis of the tibia. (Pg. 501)
 Normal is less than 6mm as measure 1 cm above tibial plafond
 If internal fibular rotation-increase in this measurement
 If external rotation of fibula-measurement is normal or decreased. This is
more common
 Check for DJD
o If no DJD—Fibular Derotational and Lengthening Osteotomy
o If severe DJD-ankle arthrodesis
 Contraindications
o Infection
o Neuropathy
 Surgical choices
o Oblique osteotomy—can only gain 3-5 mm in length
o Transverse osteotomy of fibula, uses iliac bone graft, with plate and syndesmotic
screws through plate

196 CROZER-KEYSTONE RESIDENCY MANUAL – SECOND EDITION


Ilizarov Method
 Insertion of wires
1. Place wire with the frame already built
2. Make stab incision
3. Blunt dissection (with a hemostat)
4. Wire is inserted via safe tract via clamp
5. Remember to pulse the drill to avoid overheating
6. Use a wet Ray-Tec sponge to keep wire cool and stabilized
 Positioning the frame
o Proximal: 2 fingers between the frame and tibia
o Distally: 3 fingers (to allow for swelling)
o Frame should be 2-3 cm from the surface of the ground to allow for WB
 Tibial wire insertions
1. Break the tibia into 6 segments (1st being most proximal and probably out of our scope
of practice)
2. At the 2nd segment
a. Put half pin perpendicular to subcutaneous surface of tibia (pretty much the
general rule for insertion of half pin at the tibia)
b. For the wire, try to engage the widest portion of the tibia. This means inserting
the wire slightly oblique to the transverse plane of the tibia, thus exiting a little
more ant-medial when compared the plane of tibia.
3. At the 3rd and 4th segment
a. Similar to 2nd segment
4. At the 5th segment
a. Wire is inserted almost perpendicular to frontal plane of tibia.
i. Note: Often the tibial wires will be parallel so that the frame can be slid both
medial and lateral
th
5. At the 6 segment
a. Wire options
i. Directly med → lat
ii. More ant-lat → post-med
iii. Through fibula and into tibia

CROZER-KEYSTONE RESIDENCY MANUAL – SECOND EDITION 197


Osteochondral Lesions of the Talus
 Conservative treatment indicated for Stage 1-2 and Stage 3 medial lesions
 Surgery indicated for Stages 3 lateral and Stage 4 lesions

 Procedure
o Patient set-up
 Supine
 Thigh tourniquet
 General or spinal
1. Get scope portals
2. Use 2.7 mm scope, both 30˚ and 70˚. Examine the joint.
3. With a probe, evaluate the articular cartilage. Look for any loose bone fragments
beneath articular surface and the extent of the lesion over the talar dome.
4. Type of surgery is dependent on type of osteochondral defect
a. Acute fracture is usually ant-lat, more substantial bone base and better for
internal fixation
b. Chronic post-med lesion is more likely to have fragmented necrotic bone and
poor articular cartilage. These lesions must be removed.
c. In young patients without skeletal maturity, simple drilling may be enough if
articular cartilage remains intact
5. For drilling, use 0.062 K-wire to depth of 1-1.5 cm
 For post-med lesion
o Old method was to use trans-tibial approach.
o New method is to use a guide and go through sinus tarsi into post-med portion
 For articular cartilage that is fragmented, loose and necrotic
1. For post-med lesions, use post-lat port for 70˚ scope
2. Use probe to lift cartilage
3. Remove cartilage with forceps
4. Use angled cervical curette to debride lesion to healthy, bleeding bone
a. Stopping the inflow will demonstrate bleeding
5. If good bleeding, just remove all remaining bone fragments
6. If not good bleeding, drill as previously described
 For bone grafting
o If cartilage is good and intact, but either there is a viable bone fragment or there is
only edema of the underlying cancellous bone
o Insert in the trans-talar approach (like drilling through subtalar)
o OATS Procedure
 For post-med lesion
 Take a plug of bone with articular cartilage from knee and through a
trans-tibial approach, insert into the talus

198 CROZER-KEYSTONE RESIDENCY MANUAL – SECOND EDITION


Tarsal Tunnel Release
 Most often occurs in the fibro-osseous tunnel (bound by lacinate ligament), most often at the
distal edge of the ligament.
 Division of nerve of medial and plantar nerve occur deep to lacinate ligament in 93% of
people, and proximal in the other 7%
 The nerve is in the third channel (Tom, Dick and A Very Nervous Harry.)
 Need ―peanuts,‖ posterior splint, and Penrose drain.

 Procedure
o Patient set-up
 General or spinal usually
 Thigh tourniquet, deflate before closure.
 Don’t esmarch the foot, just elevate it.
1. Incision 10 cm proximal to the tip of the medial malleolus and 2 cm posterior to the
tibia. After the medial malleolus, gently curve plantar to the level of the talonavicular
joint (plantar to TNJ). This should be about the midpoint of the abductor hallucis
a. McGlamry—2cm proximal to superior edge of lacinate ligament and gently
curving to the proximal margin of the abductor hallucis.
2. With hemostats, blunt dissect the SubQ. A moistened 4x4 can be used. When in the
SubQ, be careful of the medial branch of the nerve as it punctures the lacinate ligament.
3. When at the lacinate ligament, feel for the pulse of the PT artery. Also palpate for the
tendons of the PT and FDL. The FHL (4th compartment) can be palpated by moving the
big toe.
4. From proximal to distal, make incision of the roof of the third canal. May want to use
hemostats and split them, or may use groove channeler/director.
5. Isolate the Tibial Nerve and all 3 branches (medial and lateral plantar nerve, and the
calcaneal branch) of its branches from all tissues.
6. Remove any neoplasm. Be careful!
7. Move varicose veins. Ligate veins if necessary, but make sure that you aren’t ligating
an artery.
8. Follow nerve distally through the abductor canal. Section the abductor canal stricture.
9. Go proximal and follow nerve upwards.
10. Deflate tourniquet.
11. Closure—but don’t reapproximate the lacinate ligament (McGlamry says only partially
reapproximated) and the subQ and skin are closed. If a lot of ooze, then use a drain.
12. Marcaine at the end

 Post-op
o Below the knee compression dressing is applied.
o NWB or partial WB for two weeks.
o Begin DF and PF of ankle after 2 weeks.

CROZER-KEYSTONE RESIDENCY MANUAL – SECOND EDITION 199


200 CROZER-KEYSTONE RESIDENCY MANUAL – SECOND EDITION
Case Study 1
A 23 y/o male presents to the ED with foot trauma.

What should you do first?


Obtain a quick history and check neurovascular status
HPI – patient had foot run over at work. He complains of 10/10 pain and that his toes feel cold
and numb.
PE – pulses are present. Toes feel cold. Patient cannot feel you touching his toes. Toes are
changing color (purple or white). Patient cannot move his toes.

What is your diagnosis?


Compartment syndrome

What is compartment syndrome?


Condition with increased tissue pressure within a limited space which compromises the
circulation and function of the tissues. It can lead to ischemia of the tissues.

Is this a surgical emergency?


Yes. Compartment syndrome is a clinical diagnosis, according to the AO Principles course. It is
better to get the patient up to surgery than to find an instrument to measure the pressure.

What are some causes of compartment syndrome?


Fractures, crush injuries, prolonged limb compression, post-ischemic swelling. Practically any
injury can result in compartment syndrome.

What are the signs of compartment syndrome?


 Pain out of proportion (most important)
 Paresthesia
 Pallor
 Pulses present
 Poikilothermia
 Paralysis

What are some techniques for measuring pressure?


Wick catheter, slit catheter, Synthes catheter, needle technique, continuous infusion technique.
But as stated before, this should be a clinical diagnosis.

What is the treatment?


Fasciotomy of the compartment. In the leg, surgical access should be made to all four
compartments. To do a leg fasciotomy, make one incision medial to the tibia and one lateral.
From the medial incision, open the superficial and deep posterior compartments. From the
lateral incision, open the anterior and lateral compartments.

CROZER-KEYSTONE RESIDENCY MANUAL – SECOND EDITION 201


What are some absolute indications for a fasciotomy?
 Tissue pressure above 30 mm Hg (normal 4 ± 4 mm Hg)
 Sensory and motor loss
 Pain out of proportion

202 CROZER-KEYSTONE RESIDENCY MANUAL – SECOND EDITION


Case Study 2
A patient who you prescribed pain medications has wheels, hives, itching, and trouble
breathing after taking the medication.

What is most likely going on?


Anaphylaxis

What is anaphylaxis?
Rapid, generalized immunologically-mediated event that occurs after exposure to foreign antigen
substances in previously sensitized persons. This syndrome can affect any organ in the body, but
it most commonly affects the pulmonary, circulatory, cutaneous, neurologic, and GI systems.

What are the clinical symptoms of anaphylaxis?


Mild (common)
 Urticaria, weakness, dizziness, flushing, angioedema, congestion, sneezing
Severe
 Upper respiratory tract obstruction, hypotension, vascular collapse, GI distress,
cardiovascular arrhythmias, cardiac arrest

What is the difference between anaphylaxis and anaphylactoid reaction?


Clinically, they present the same, but anaphylactoid reaction is not mediated by the IgE antibody
and does not necessarily require previous exposure to the inciting substance

What is the treatment for anaphylaxis?


Stop the offending agent, and if necessary, D/C all meds
If the patient is having life-threatening problems, get them to the ED
Treat the symptoms
 Airway – bronchospasm
o O2 40-100%
o Epinephrine 0.3-0.5 mL 1:1000 soln SC or IM q15min
o Albuterol 0.5 mL 0.5% in 2.5 mL NS nebulized q15min
o Benadryl 50 mg PO q4-6h
o Methylprednisolone 2-60 mg PO daily
 Cardiovascular – hypotension
o IV fluids 1 L q20-30min prn
o Maintain systolic pressure >80-100 mm Hg
o Epinephrine 1 mg 1:1000 in 500 mL D5W IV at rate of 0.25-2.5 mL/min
o Norepinephrine 4mg in 1 L D5W IV at 0.5-3 mL/min
o Benadryl 50 mg PO q4-6h
 Cutaneous reactions
o Epinephrine 1:1000 0.3-0.5 mL SC or IM q15min
o Benadryl 50 mg PO q4-6h
Document offending agent and educate patient on future avoidance

CROZER-KEYSTONE RESIDENCY MANUAL – SECOND EDITION 203


What is the best way to prevent anaphylaxis?
Thorough history and elimination/avoidance of the offending substance

204 CROZER-KEYSTONE RESIDENCY MANUAL – SECOND EDITION


The Interviews
For some people, the social interview is harder than the academic interview. In my opinion, the
reason that it is harder is because people don’t prepare for the social interview. After sitting
through 2 years of interviews, I’m amazed at how some people don’t seem ready for the social
questions. Many of these ideas are from my past as a salesperson on a job interview. Essentially
that is what you are…a person selling themselves for a job.

-Brett Chicko

The amount of Social vs. Academic Interviewing will vary from program to program. Some will
only ask social questions (but pimp you when you rotate with them) and some may ask only one
social question and a battery of academic questions. Review this section so you have some
baseline answers for common social questions and review the rest of the book for common
academic questions. Overall: be yourself and relax!

-Hubert & Sandi

Hint #1 – Look and dress appropriate!


 Men
o Wear a pressed shirt, tie, and suit. (Suggest a dark suit with a blue or white shirt.)
o Hair should be neat and combed and facial hair clean cut.
 Women
o Wear a conservative suit. Keep the skirt length and shirt neckline appropriate.
o Hair should be neat and combed.
 Sitting in the interview chair
o When you sit in the chair, sit back with good posture. NO SLOUCHING!
o Your hands should be in your lap (when you’re not talking to them).
o Keep your feet flat on the floor for men or legs crossed for women.
o Do not lean your arms on the table in front of you…you’re not at your desk at
home!

Hint #2 – Proper entrance/exit from interviews


 Do not be late for your interview!
 When you enter the room, sit in the chair and give a proper greeting.
o Say hello at least.
o Say ―nice to see everyone again‖ or something similar if you’ve rotated with the
program.
 Make sure to make eye contact with everyone in the room and SMILE! (But don’t be too
cheesy about it.)

CROZER-KEYSTONE RESIDENCY MANUAL – SECOND EDITION 205


 When you leave the room, it is appropriate to only shake hands with the director unless
the other members of the room offer to shake their hands.
 Thank the interviewers for their time.

Hint #3 – Do your homework


 At the end of many interviews, the host will ask the interviewee if he/she has any
questions.
o Brett says: You should not. ―You should have done your homework by this point.
Why would you spend your money and time to interview at a place you don’t
have any information about (other than what is in the CASPR book)? To ask
questions in the interview that should’ve been asked on a visit makes you look
unprepared and uninterested.
o Hubert & Sandi say: It is okay to ask any burning questions you may have about
the program (rumors about taking 4 residents instead of 3, or hearing a program
may be closing next year). Avoid simple questions that may be answered by
reading a program’s handbook or website.
 By the time of your interview, you should have visited the program (and in theory
asked all your questions at that time)
o If you live and attend school near a program you’re interviewing with by golly
you BETTER HAVE VISITED there by the time of the interview!
o If you live too far away (airplane ride distance) then a phone call to one of the
residents will work (track down a resident’s pager number or contact info through
the number given in the handbook for each program)
 During this phone call, get your questions answered.
 As always, be prepared and have a good list of questions for the resident.
 If any questions do arise after the visit/phone call then it is perfectly
acceptable to call or re-page or email the resident.
 A good response to this question: ―Do you have any questions about our program?‖
o ―No Sir/Ma’am, all of my questions were thoroughly answered by Dr. So-and-So
on my visit (or phone call). I feel as if I have a good understanding of the
opportunities of your program.‖

Hint #4 – Ask for the program


 Brett says:
o I know some people feel that by showing up for the program their presence alone
means they want the program. I still feel like one should go the extra yard and
ask for the program. I do not mean suck up to them, but state to the interviewers
that you want the program in a mature and professional manner.
o By asking for the program or stating that you want the program it shows the
program that you want to be there and that this is not a practice interview
o I know personally that if there are 2 women I want to date, the one who expresses
an interest me versus one that is indifferent, I will always choose the one who
expresses an interest
o Proof? All the interviews I’ve been a part of, only 4 people asked for the program
(over a 3 year span) and all 4 have gotten the program.

206 CROZER-KEYSTONE RESIDENCY MANUAL – SECOND EDITION


 Hubert & Sandi say:
o Getting a good program’s spot (like Crozer’s) is highly competitive, and many
students have learned to sneak in ―I love your program‖ or ―I definitely want to
come here‖ as many opportunities as they can.
o Proof that this doesn’t always work? Since this book was written, the occasional
student who has said they wanted the program has turned in to more like 60-75%
of the students coming through with visits or externships that ―ask for the
program.‖ Trust us, your actions speak way louder than your words…
o But if it makes you feel better to say this as a parting goodbye, you wouldn’t be
the first or the last!

Hint #5 – Be enthusiastic
 Don’t be one of those people that walk around looking like they have a hangover or just
took a sleeping pill.
 Get a good night’s rest so your tiredness from studying/drinking doesn’t overcome your
alertness and enthusiasm.
 But don’t be a cheeseball or act unnaturally.

Hint #6 – Act like someone you would want to work with


 If you’ve rotated with the program for a whole month, no matter how you act in the
interview, they already know your personality. But acting like a big goof can hurt your
chances.
 Continuing on Hint #5 and think about #7, be like a mature, responsible adult, or at least
act like one.
 If you’re picked, you’d be the one representing the program so you need to show you’re a
good person to work with.

Hint #7 – Remember you’re in a formal interview


 Even if you joked around a lot on a visit or on a rotation, especially when answering
academic questions, keep the ridiculously silly answers to yourself.
 Although you may have spent a month at a program, not everyone at the interview may
have met or remember you. Make sure your first impression with them a positive one.
 You want to do well because sometimes a good or bad interview makes or breaks your
chances with a program.

Hint #8 – Prepare for the social interview


 Go over some of the sample social questions and formulate some answers for a couple of
them. Go over these answers with a classmate, a resident you’re working with or are
friends with, your mom, anyone really.
 Practice, practice, practice…
 Be creative, but please read your answers out loud to another human being so your
creativity is not interpreted wrong or sound ridiculous.
 Don’t ignore this part because you have a thousand other things to study for the academic
portion!

CROZER-KEYSTONE RESIDENCY MANUAL – SECOND EDITION 207


Hint #9 – Have fun
 Seriously, this is the only way to get through these types of questions.
 This will show the program you are applying to a part of your personality and how you’re
handling the stress of interviewing.

Hint #10 – Don’t over think things


 With nerves and anywhere from 2 to 15 people interviewing and watching you, even the
most confident person can show some nervousness.
 With a few exceptions of certain known hard-core academic programs, most programs
just want to see your scope of basic knowledge and reasoning when working up a case
study or working through the questions.
 If it looks like a dog, smells like a dog...it’s probably a dog. Meaning, if you think you
know the answer, you probably do!
 And if you hear hoofbeats, think horses – not zebras…

Hubert & Sandi’s Bonus Tips


 The Match for students with programs is designed to work in favor of STUDENTS NOT
PROGRAMS
 Rank programs the way YOU want them. Don’t let rumors influence how you rank
your programs. (Ex: If you rank a program you really want and they don’t end up
picking you, it just bumps you down to your #2 pick, but at least this way, if you’ve put
them #1 you’ve given yourself the strongest chance of matching with your first pick!)
After all, no one knows how you ranked programs except YOU (or the people you
tell…).
 When evaluating a residency program, in addition to making sure you’d get the cases
and experience you’re looking for, evaluate the senior resident class and how they’re
performing. Think about it...if they’re good, that’s how you would be if you graduated
that program! (Ex: Are they excellent surgeons who are confident doing a variety of
cases? Are they getting skin-to-skin cases? Are they having trouble getting their
numbers? Are they knowledgeable? Are they happy they went there??)

Sample Social Questions


Tell us about yourself
 Keep this one short. Three sentences is a good length (i.e. who you are, what you are and
what you like to do).
 Keep this to the point and under 15 seconds.

Who is your hero?


 Obviously, besides ―Jazzy‖ Jeff Lehrman, DPM (Crozer attending extraordinaire and
graduate of the Crozer program).
 It’s probably not wise to say someone in your class or a cartoon superhero…but anything
else is probably appropriate.

Why did you pick podiatry?


Foot fetish is NOT okay.

208 CROZER-KEYSTONE RESIDENCY MANUAL – SECOND EDITION


Why should we take you over the other applicants?
Just don’t personally call out any of the other applicants when answering this.

What do you know about our program?

What did you learn on your externships?

What is your favorite externship and why?


Yup, what you’re thinking is correct…it is not wise to name a different program than the
program that asks this silly question.

Did you visit our program?


 If yes, what did you learn (or like) about our program?
 If no, why didn’t you visit our program?
o If you live close to the program (i.e. Temple students and our program), there
really aren’t many good excuses that sound acceptable. The ―I didn’t have time‖
or ―My schedule didn’t allow it‖ just don’t cut it…

Tell us about some of the current events


Just a question to make sure you don’t live in a bubble.

What was the last book you read?


But please have read the book, cause if it’s some unknown title, someone may ask you what it’s
about and it’s kinda strange to say ―I’ve just started it so I don’t know yet.‖

Tell us a joke
 Keep it clean!
 Brett’s joke:
o Sherlock Holmes & Doc Watson were camping. They went to sleep and were
sleeping soundly until Sherlock woke up Watson. Sherlock said ―Watson, look
up. What do you see?‖

Watson looked up at the sky and saw millions and millions of stars and the moon.
Realizing that The Sherlock Holmes asked him the question, Watson wanted to
give an impressive answer.

Watson thought about it for a second and answered:

―From an astronomical point of view, I see the Milky Way and many different
constellations to the North.
―From a holographic point of view, I see the clouds are coming from the East and
we may have a chance of rain tomorrow.
―From a theological point of view, it shows how vast and powerful God is and
how insignificant we are when compared to God.
―From an astrological point of view, I see that the moon is in Jupiter and if you
are a Scorpio, you are going to have a good day tomorrow.‖

CROZER-KEYSTONE RESIDENCY MANUAL – SECOND EDITION 209


Watson then asked, ―Why, what do you see Sherlock?‖

Sherlock took a drag on his pipe and replied, ―Someone stole our tent.‖
 Hubert & Sandi say: Good luck remembering a long joke like that, but only tell a joke if
you can tell the whole thing and remember the punch line. Please!

What do you do in your free time?


 Usually the program isn’t looking for a geeky answer like ―I read podiatry articles.‖
 Mention a hobby or activity you like to do on the weekend (like back in undergrad, when
you had a life…and no, going to frat parties is not an acceptable answer).

Be prepared to answer a question about one of your lower grades (if you have any)
Especially if you have to re-test or re-take a class to pass.

Who was your favorite resident? Who is your least favorite resident?
Proceed to think of an answer with mucho cuidado (much caution)!

What is your favorite color?


Wouldn’t advise you to say black or white, cause technically those aren’t colors and it’s totally
lame if you do say them.

If you can be any animal, what would it be and why?

Who is your favorite clinician in school and why?

Who is your mentor?


Please don’t say someone in your class!

Who is our director?


Please don’t ever mess this one up!

210 CROZER-KEYSTONE RESIDENCY MANUAL – SECOND EDITION


A Surgery
Review
Booklet
(& a few other important
items for externships)

OCPM’s ACFAS
Officers of 2005

Re-typed, Edited, Updated by the JPMSA in 2009

1
All pictures came from class notes / handouts.
Most of the pictures were referenced from Dr. Hetherington’s book.

2
Table of Contents:

Topic Page #
Charting
Surgical Consult 6
Pre-Op Note 6
Post-Op Order 6
Admission Order & Note 6
Post-Op Note 7
Post-Op Visit 7
Layers of the Foot 8
Key Lab Values 8
Hospitalization Indications 9
Post-Op Fever Etiologies 9
Sutures
Types 10
Selection 11
Technique 11
Classification Systems
Stewart, Salter-Harris 12
Gustillo-Anderson, WatsonJones, Freiberg 13
Berndt-Hardy, Hawkin 14
Sneppen, Watson & Dobas, Kuwada 15
Rowe, Sander’s 16
Hardcastle, Dias, Danis-Weber 17
Lauge-Hansen 18
MRI 19
Anesthesia
Anesthetics 20
Dosing 20
Onset/Duration 20
Increasing Comfort 21
Ankle Block 21
Hemostasis = Tourniquet Pressures 21

3
Table of Contents (Continued):
Topic Page #
Corticosteroid Injections
Types 22
Side Effects 22
Cocktails 22
Radiographic Data 23
Joint Deformities 24
Osteotomies
Proximal 25-27
Shaft 29-30
Distal 31-33
Internal Fixation
Principles 34
Rule of 2’s 34
K-Wires 34
Steinmann Pins 34
Monofilament Wire 35
Tension Band Wiring 36
Staples 37
Screws
Anatomy 38-39
Types 40-41
Fixation Technique 42-43
Selection 44
Soft Tissue Anchors 45
Plates 45-46

4
Table of Contents (Continued):

Topic Page #
External Fixation
Principles 47
Complications 48
Types 48-49
Dynamics 50
Care & Management 50
Forefoot Pathologies / Surgical Procedures
Hallux Limitus/Rigidus 51-52
Hammertoes 53-56
Etiologies for Contracted Digits
th
5 Digit Arthroplasty 57
Rearfoot Surgery
Plantar Fasciotomy 58
Haglund’s Deformity 59
Keck & Kelly Osteotomies 59
Tendon Transfers & Indications
Adductor Hallucis 60
Abductor Hallucis 60
Extensor Hallucis Longus 60
Jones Suspension 60
Hibbs 60
Tibialis Anterior 61
STATT 61
Cobb 61
Tibialis Posterior 62
Peroneus Longus 62
Bunions based on Angles 63
Other things to know… 67

5
CHARTING
SURGICAL CONSULT
1. Chief Complaint 8. Primary Care Dr
2. HPI (NLDOCAT) 9. Hospitalizations
3. Allergies 10. RoS
4. Medications  Vitals / Vascular / Neuro / Derm /
5. Social History Musculoskeletal
6. Medications 11. Ancillary (x-rays, labs, ect…)
7. Family History

PRE-OP NOTE
Surgeon Medications
Pre-Op Dx Allergies
Planned Procedure Diagnostic Data – Labs, x-rays, EKG, ect…
Consent Form: Describe Procedure & Care / Complications /
Alleviations / Expected Outcomes / Arrange Pre-Op Testing
“Consent form was reviewed with patient, signed and placed in chart.” “All
risks, possible complication and alternative treatments have been
discussed with the patient in detail. All patients’ questions have been
answered to satisfaction. No guarantees to the outcome have been made.”

POST-OPERATIVE ADMISSION ORDERS &


ORDERS: NOTE:
VANDIMAX ADC – VAANDILMAX Vitals
Date/Time/Signature Date/Time/Signature Activities
Vitals Admit to Allergies
Activities Dx Nursing
Nursing Condition Diet
Diet - Ins/Outs
Ins/Outs Labs
Meds Meds
Ancillary Ancillary
X-ray X-ray

6
POST-OP NOTE: SAPPA HEMI FC2P2
Surgeon Hemostasis – type Findings
Assistants Estimated Blood Loss Pathology
Pre-Op Diagnosis Materials – sutures/hardware Prophylaxis
Post-Op Diagnosis Injectables – any post-incision Complications
Procedure Condition
Anesthesia – type /
how much
“Patient tolerated procedure and anesthesia well. Patient transported to
recovery by anesthesia with vitals stable and vascular status intact.”
Also may include.. Pathology – bone, ST; Condition – stable, guarded,
fair, poor; Prophylaxis

POST-OP VISIT: SOAP


Subjective
1. POV # ______, PVD #_______
2. Procedure
3. N,V,C,F,SOB
4. Activity status
5. Pain / How controlled
6. Other Complaints
Objective
1. How patient presents – walking, wheelchair
2. Vascular, Neuro, Derm, Musculoskeletal
Assessment
1. Status Post-Op
2. Compliance
Plan
1. Treatment
2. Dr & Residents

7
LAYER OF THE FOOT

1st Layer
1. Abductor Hallucis M.
2. Abductor Digiti Minimi M.
3. Flexor Digitorum Brevis M.
2nd Layer
1. Quadratus Plantae M.
2. Lumbricales M.
rd
3 Layer
1. Flexor Hallucis Brevis M.
2. Flexor Digiti Minimi M.
3. Adductor Hallucis M.
th
4 Layer
1. Dorsal Interossei M. (4)
2. Plantar Interossei M. (3)

KEY LAB VALUES


Chem 7
(136-145mEq/L) (97-107mEq/L) (5-20mg/dL)
Na Cl BUN Glucose
(<110mh/dL)
K CO Cr
(3.5-5mEq/L) (23-29mmol/L) (M: <1.2 W: <1.1mg/dL)

CBC (M: 14.4-16.6g/dL)


(W: 12.2-14.7g/dL)

WBC HgB
(4500 - Platelets
11,000/L) HCT (150,000-450,000μL)
(M: 43-49%)
(W: 37-43%)

8
INDICATIONS FOR HOSPITALIZATION POST-OP

1. Fever >101.6o
2. Ascending Cellulitis / Suspect Osteomyelitis
3. Lymphangitis / Lymphademopathy
4. Immunosuppressed
5. Virulent / Resistant Organisms
6. Need for I&D Procedure
7. Need for IV Antibiotics
8. Failed response to outpatient therapy
9. Need a consult

POST-OP FEVER ETIOLOGY

1. Wind – Pulmonary
a. Aspiration / Pneumonia
b. Occurs 24-48h
c. Get chest x-ray

2. Water – UTI
a. Occurs in 2-6d

3. Wound
a. Occurs in 3-5d

4. Walk – DVT / Pulmonary Embolism


a. Within 1st week
b. Virchow’s Triad
i. Hypercoagulation
ii. Venous Stasis
iii. Endothelial Damage

5. Wonder – drugs / fever / benign / medicines

9
SUTURES

Absorbable Sutures Filament Type Total Absorption


Chromic Gut 70d
Monocryl Monofilament 90d
Maxon Monofilament 90-120d
Vicryl Monofilament 56-70d
or Braided
Dexon Mono- or 90-120d
Multifilament
Dexon Plus Mono- or 90-120d
Multifilament
Dexon S Multifilament 90-120d
PDS Monofilament 180d
Non – Absorbable Sutures Filament Type Advantages
Stainless Steel Mono- or High strength, low
Multifilament tissue Rxn
Ethilon Nylon Monofilament Elasticity/Memory
Prolene Monofilament Minimal Tissue Rxn
Novafil Monofilament Elasticity/Tensile
strength
Silk Multifilament Good Handling
Nurolon Nylon Multifilament
Mersilene Multifilament Consistent Tension
Ticron Braided Minimal Tissue Rxn
Ethibond Multifilament Good Handling

10
SUTURE SELECTION
1. Bone 5. Subcutaneous Fat
a. Stainless Steel a. Vicryl
2. Tendon 6. Subcuticular
a. Prolene a. Monocryl
b. Ethibond b. Vicryl
c. Nylon 7. Capsule
d. Polyesters a. Vicryl
3. Muscle 8. Skin
a. PDS a. Nylon
b. Vicryl b. Silk
c. Prolene
4. Fascia Deep Tissue  taper needle; 3-0 suture
a. Prolene Subcutaneous Tissue  taper needle; 4-0 suture
b. PDS Dermal Layer  precision needle; 5-0 suture
Capsule  2-0 or 30 suture
Subcutaneous  4-0 suture
Subcuticular  5-0 clear suture
Skin  4-0 clear suture

SUTURE TECHNIQUES
1. Simple Interrupted
Good for infected wounds
Individual know for each throw
2. Horizontal Mattress
Everts skin edges well
3. Vertical Mattress
Everts tissue edges well
4. Continuous Running
Good to save time
Good for large wound areas
5. Subcuticular (Running Intradermal)
Leaves the best scar
11
STEWART CLASSIFICATION OF 5TH MET FRACTURES
Type I Supra-articular @ metaphyseal-diaphyseal junction
True Jones!
Type II Intra-articular avulsion, 1 or 2 fracture lines
Type III Extra-articular avulsion, PB tears small fragment from the
styloid process
Type IV Intra-articular, comminuted fracture, assoc. with crush injury
Type V Extra-articular avulsion @ of physis in children (SH Type I)

SALTER-HARRIS CLASSIFICATION OF EPIPHYSEAL INJURIES


Type I
Epiphysis is completely separated from metaphysic
Type II
Epiphysis, and the growth plate, is partially separated from the
metaphysis, which is cracked
Type III
Fracture runs through the epiphysis, across the growth plate from
the metaphysic
Type IV
Fracture runs through the epiphysis, across the growth plate, and
into the metaphysic
Type V
The end of the bone is crushed and the growth plate is compressed
Type VI
(Rang’s Addition) Avulsion of peri-chondral ring
Type VII
(Ogden’s Addition) Avulsion fracture of the epiphysis without
involvement of the physis

12
GUSTILLO & ANDERSON OPEN FRACTURE CLASSIFICATION SYSTEM
Type I
Fracture with open wound <1cm in length
Clean, minimal soft tissue necrosis
Usually traverse or short oblong
Type II
Fracture with open wound >1cm in length
Clean, minimal soft tissue necrosis
Usually traverse or short oblon
Type III
Fracture with open wound >5cm in length
Contamination and/or necrosis of skin, muscle, NV, & ST
Comminuted
 Type IIIa
 Adequate bone coverage
 Type IIIb
 Extensive soft tissue loss with periosteal stripping and bone
exposure
 Type IIIc
 Arterial injury needing repair
NAVICULAR FRACTURE – WATSON JONES
Type I Navicular tuberosity fracture
Type II Avulsion fracture of dorsal lip
Type III A: Transverse body fracture – Nondisplaced
B: Transverse body fracture – Displaced
Type IV Stress fracture
FREIBERG CLASSIFICATION – AVN OF 2ND MET
Type I No DJD Articular cartilage intact
Type II Peri-articular spurs Articular cartilage intact
Type III Severe DJD Loss of Articular Cartilage
Type IV Epiphyseal dysplasia; multiple head involvement

13
BERNDT-HARDY CLASSIFICATION OF TALAR DOME LESIONS
Stage I
Compression lesion or non-visible lesion
Stage II
Fragment attached
Stage III
Non-displaced fragment without attachment
Stage IV
Displaced fragment

TALAR NECK FRACTURE CLASSIFICATION – HAWKIN’S


Type I
Non-displaced talar neck
Disrupts blood vessels entering dorsal talar neck and intra-osseous
vessels
20% chance AVN
Type II
Displaced talar neck fracture with subluxed or dislocated STJ
Disrupts dorsal neck arterial branches and branches entering from
inferiorly from sinus tarsi & tarsal canal
40% chance AVN
Type III
Displaced talar neck fracture with dislocated STJ & ankle joint
Disrupts all 3 major blood supplies
100% chance AVN
Type IV
Displaced talar neck fracture with complete dislocation of STJ
Ankle joint + subluxation or dislocation of the talonavicular joint
Disrupts all 3 major blood supplies
100% chance AVN

14
SNEPPEN CLASSIFICATION OF TALAR BODY LESIONS

Group I Transchondral / Compression fracture of the talar dome


Group II Coronal/Sagital/Horizontal shearing fracture of the entire body
Type I Coronal or Sagital
A: Non-displaced
B: Displacement of trochlear articular surface
C: Displacement of trochlear articular surface with
associated STJ dislocation
D: Total dislocation of the talar body
Type II Horizontal
A: Non-displaced
B: Displacement
Group III Fracture of posterior tubercle of talus
Group IV Fracture of lateral process of talus
Group V Crush fracture of the talar body
WATSON & DOBAS CLASSIFICATION – POSTERIOR LATERAL
TUBERCLE OF TALUS (SHEPARD’S FRACTURE)
Stage I Normal Lateral talar process with no clinical significance
Stage II Enlarged posterior lateral tubercle of the talus (Steida’s Process)
Stage III Accessory bone / Os Trigonum that may be irritated by trauma
Stage IV Os Trigonum + cartilaginous/synchrondrotic union with talus
KUWADA CLASSIFICATION OF ACHILLES RUPTURE
Type I Partial rupture
Type II Complete rupture <3cm gap
Type III Complete rupture 3-6cm gap
Type IV Complete rupture >6cm gap

15
ROWE CLASSIFICATION OF CALCANEAL FRACTURES
Type I A Medial Tuberosity fracture
B Sustentaculum Tali fracture
C Anterior Process fracture

Type II A Posterior break fracture without Achilles involved


B Posterior break fracture with Achilles involvement

Type III Extra-articular body fracture

Type IV Intra-articular body fracture without depression

Type V A Comminuted, Intra-articular fracture with depression


B Comminuted fracture with severe joint depression

SANDER’S CT CLASSIFICATION OF CALCANEAL FRACTURES


* Fractures are classified according to the number of intra-articular
fragments and location of fracture lines
# of Fractures
Type I Any non-displaced intra-articular fracture
Type II 1 fracture through posterior facet creating 2 fragments
Type III 2 fractures through the posterior facet creating 3 fragments
Type IV 3+ intra-articular fracture lines

Location of Fracture Lines:

16
LISFRANC’S FRACTURE CLASSIFICATION – HARDCASTLE
Type A: Total or Homolateral
– Disruption of the entire Lisfranc joint
– Transverse or Sagital plane
– Most common type
Type B: Partial
B1 – Medial incongruity with the first met forced medially
– Involves 1st met OR mets 2,3,4 but NOT 5
B2 – Lateral incongruity with lesser mets forced laterally
Type C: Divergent
C1 – Partial divergence with the 1st met medial and 2nd met laterally
displaced
C2 – Total divergence with the 1st met displaced medially and lesser
mets displaced laterally
DIAS CLASSIFICATION OF LATERAL ANKLE LIGAMENT INJURY
Grade I  Partial rupture of CFL
Grade II  Complete rupture of ATFL
Grade III  Complete rupture of ATFL, CFL, &/or PTFL
Grade IV  Complete rupture of all 3: ATFL, CFL, & PTFL
+ Partial rupture of the Deltoid Lig
DANIS-WEBER CLASSIFICATION OF FIBULAR FRACTURES
INVOLVED IN ANKLE FRACTURES

Type A Transverse avulsion fibular fracture BELOW…


(SAD)
Type B Spiral fracture AT…
(SER or PAB)
… the level of
Type C Fibular Fracture ABOVE…
the syndesmosis
(PER)

17
LAUGE-HANSEN CLASSIFICATION OF ANKLE FRACTURES

SUPINATION ADDUCTION (SAD)


¤ Stage I Transverse avulsion of fibula @/below AJ level
Rupture of the Lateral Collateral Ligament
¤ Stage II Oblique to Vertical fracture of the Medial Malleolus

PRONATION ABDUCTION (PAB)


¤ Stage I Transverse avulsion fracture of Medial Malleolus
– or – Rupture of Deltoid Lig
¤ Stage II Rupture of AITFL & PITFL
– or – Tillaux-Chaput / Wagstaffe fracture
¤ Stage III Short oblique fracture of the fibula @ lvl of syndesmosis

SUPINATION EXTERNAL ROTATION (SER) *** Most Common!


¤ Stage I Rupture of AITFL
– or – Tillaux-Chaput / Wagstaffe fracture
¤ Stage II Spiral/Oblique fracture of fibula @ lvl of syndesmosis
¤ Stage III Rupture of PITFL
– or – Avulsion fracture of Posterior Malleolus
(Volkmann’s Fracture)
¤ Stage IV Transverse fracture of Medial Malleolus
– or – Rupture of Deltoid Lig

PRONATION EXTERNAL ROTATION (PER) *** Longest healing time!


¤ Stage I Transverse fracture of Medial Malleolus
– or – Rupture of Deltoid Lig
¤ Stage II Rupture of AITFL & Interosseous membrane
– or – Tillaux-Chaput / Wagstaffe fracture
¤ Stage III High Spiral Oblique fracture (Maisonneuve Fracture)
¤ Stage IV Rupture of PITFL
– or – Avulsion fracture of Posterior Malleolus

18
MRI

T1-Weighted  good for showing anatomical detail


+ Short TE + TR
+ Tissue with short T1 are brighter
+ Fat

T2-Weighted  good for highlighting areas of pathology


+ Long TE + TR
+ Tissue with long T2 are brighter
+ Water, Edema

STIR  Short Tau Inversion Recovery


+ Fat suppression
+ Heavily water–weighted image
+ Very Sensitive for Bone Marrow abnormalities

Gadolinium (best for infection)


+ Contrast–enhanced chemical agent
+ Shortens T1 relaxation times  Increases signal intensity on T1
weighted images
+ Usually used in conjunction with fat suppression
+ Good for identifying ST masses, inflammation processes, & for
staging bone and ST infection

TE = Time to Echo dec TE + dec TR = T1-Weighted


TR = Time of Repetition inc TE + inc TR = T2-Weighted

19
ANESTHETICS

Esters
 Higher incidence of allergies
 Metabolized in Blood (Cholinesterase in plasma)
 Types
~ Cocaine
~ Procaine
~ Cholorprocaine
~ Tetracaine

Amides
 Metabolized by CYP450 system in Liver
 Types
~ Lidociane / Xylocaine (0.5, 1, 1.5, or 2% solutions)
~ Bupivicaine / Marcaine (0.25, 0.5, or 0.75% solutions)  C/I <12y/o
~ Mepivicaine / Carbocaine (1, 1.5, 2, or 3% solutions)

Dosing:
0.25% solution = 2.5 mg/cc drug 1cc = 1mL
0.5% solution = 5 mg/cc drug
1% solution = 10 mg/cc drug

Ex: 5cc of 1% Xylocaine (lidocaine) = 50mg of Xylocaine given


Ex: 3cc of 0.5% Marcaine (bupivicaine) = 15mg of Marcaine given

Toxic Doses: Onset & Duration:


Lidocaine Plain = 300mg Onset: 5min
w/ Epi = 500mg Duration: 1-2h

Marcaine Plain = 175mg Onset: 10-15min


w/ Epi = 225mg Duration: 6-8h

20
6 Ways to Increase Comfort of the Injection: Draw up with 18G
1. Quick Stick Inject with 25 or 27G
2. Slow Injection
3. Small Gauge Needle (large # = small gauge)
4. Small Syringe (less pressure)
5. Cold Spray
6. Warm the Solution (to body temp)

Ankle Ring Block:


Superficial = Saphenous N., Sural N., Superficial Peroneal N. (IDCN, MDCN)
Deep = Posterior Tibial N., Deer Peroneal N.
Deep
** Fact: If you mix Fibular N.
Lidocaine and Marcaine, you
will only have partial Saphenous Superficial
anesthesia deep into surgery. N. Fibular N.
Talar
Only mix to avoid toxic M Trochlea L
doses. M M

** Fact: If you need to inject Post. Tibial Sural N.


N.
more volume, use a small
percent of drug solution.
Achilles Tendon

Ex: 30cc of 1% gives more anesthesia than 15cc of 2%

Saphenous N .5-1cc
Posterior Tibial N 1-3cc give the most here since this N is the largest
Sural N .5-1cc
Deep Peroneal N .5-1cc between 2 Long Extensor Tendons
Superficial N .5-1cc plantarflex & invert

Hemostasis = Tourniquet Pressures:


Ankle: +100 over systolic ~250mmHg
Thigh: +200 over systolic ~ 350mmHg
21
CORTICOSTEROID INJECTIONS
Corticosteroid injections are used to control local inflammatory reactions

Phosphates: short-acting (clear)


Acetates: long-acting (cloudy)

All corticosteroids are collagenilytic and therefore should not be injected


into the same area of soft tissue more than 3-4 times per year.

Side Effects:
~ Soft tissue atrophy
~ Tendon rupture
~ Skin discoloration (lightening)

Cocktails Commonly used in Podiatry: Always draw up the Lido/Marc 1st


1. Plantar Fasciitis followed by Dex or Kenalog!
a. 1cc Kenalog-10 (10mg/mL)
b. 0.75cc 1% Lidocaine
c. 0.75cc 0.5% Marcaine

2. Joint Injections
a. 0.2cc Dexamethasone Phosphate
b. 0.5cc 1% Lidocaine

3. Intermetatarsal Neuromas
a. 0.3cc Dexamethasone Phosphate
b. 0.5cc 1% Lidocaine

22
RADIOGRAPHIC DATA

Angle: Normal:
Hallux Abductus Angle 0-15o
Proximal Articular Set Angle (PASA) 0-8o
Distal Articular Set Angle (DASA) 0-7o
Intermetatarsal Angle
1-2 0-8o
2-5 16o + 4o
4-5 8o + 2o
Hallux Interphalangeal Angle (HIA) 0-10o + 2o
Metatarsal Length + 2mm
Metatarsus Adductus 0-8o
Tibial Sesamoid Position Positions 1-3
1 = Medial to midline of hallux
2 = Touching midline medially
3 = 2/3 medial + 1/3 lateral to midline
4 = 1/2 medial + 1/2 lateral to midline
5 = 1/3 medial + 2/3 lateral to midline
6 = touching midline laterally
7 = lateral to midline of hallux
Calcaneal Inclination Angle 18-22o
Talar Declination Angle 21o
TaloCalcaneal Angle (Kite) 17-21o
Böhler’s Angle 25-40o
Angle of Gissane 125-140o
Calcaneal Fracture resulting in
Joint Depression  Böhler’s Angle ↓
Angle of Gissane

23
CLASSIFICATION OF JOINT DEFORMITIES

Positional Deformities:
PASA + DASA < HAA
PASA and DASA within normal range (0-8o)
Joint is Subluxed

Structural Deformities:
PASA + DASA = HAA
PASA and DASA abnormal
Joint is Congruous

Combined Deformities:
PASA + DASA < HAA
PASA and DASA abnormal
Joint is Dislocated

24
PROXIMAL OSTEOTOMIES
o
IM between 15-22 , normal PASA
NOT for a short metatarsal

I. Closing Base Wedge


1-1.5cm from met-cuneiform joint
4-6weeks NWB

Indications:
 Structural Lg IMA
 Splayfoot
 Juvenile/Recurrent HAV
 Met Primus Elevatus
 HAV + MetAdductus
 C/I in Elderly

II. Juvara – Types A,B,C


A: Oblique, distal lateral to proximal
medial with an intact medial cortical hinge
B: same as A but the medial hinge is
sectioned after wedge resection
C: Oblique, without wedge resection

25
PROXIMAL OSTEOTOMIES CONTINUED…

III. Opening Base Wedge


Good for a short metatarsal
Use medial eminence for the graft

IV. Crescentic
1.5cm from met-cuneiform joint
Easy traverse plane correction
Good for short metatarsal

26
PROXIMAL OSTEOTOMIES CONTINUED…

V. Double Osteotomy
IM and PASA correction

VI. Proximal V
Good screw fixation
Unlikely to get elevates

VII. Lapidus
IM > 18o
Fusion of the base of 1st met to the medial
cuneiform
Indications:
 Pain with motion at met-cuneiform joint
 Hypermobility of 1st met-cuneiform joint

27
28
MIDSHAFT OSTEOTOMIES
** Troughing is unique to midshaft osteotomies

I. Ludloff Osteotomies
IM 1-2 angle: 13-20o
Abnormal HAA
Normal to short 1st metatarsal
Elevatus is a risk

II. Mau
IM 1-2 angle: 13-20o
Abnormal HAA
Normal to short 1st metatarsal
Due to cut, decreases elevates
potential

29
MIDSHAFT OSTEOTOMIES CONTINUED…

III. Scarf – “Z”


Dorsal to Plantar – 50:50 or 66:33 cut
Very Stable, technically difficult
2 screw fixation

IV. Off-Set “V”


Modification of the Austin
Cut angled <55o

30
DISTAL OSTEOTOMIES
HAV angle – 35 o

IM angle -- 16 o
Some PASA correction

I. Reverdin
Some PF possible

II. Hohmann
Very unstable; Rigid Fixation necessary
Shortening occurs with fragment removal

31
DISTAL OSTEOTOMIES CONTINUED…

III. Mitchell – “Step-down Osteotomy”


Used for long 1st metatarsal
Good visualization of possible change

IV. Wilson
Dramatic shortening possible

32
DISTAL OSTEOTOMIES CONTINUED…

V. Austin / Chevron / Distal “V”


Transpositional – PASA, IM, DF/PF possible
Joint preserving
Possible of Juvenile HAV
Displace capital fragment ¼ to ½ bone width

VI. Reverdin Green / Distal “L”


Cut 2/3 way through bone, then plantar cut

33
INTERNAL FIXATION

4 Main Principles of Internal Fixation:


1. Anatomical Reduction
2. Rigid Internal Fixation
3. Atraumatic Technique
4. Early active RoM

Rules of 2’s:
Fracture / Osteotomy site should be 2x’s the diameter of the bone
2 points of fixation is better than 1
2 threads should purchase the distal cortex
2 finger tightness

Kirschner Wires (K-wires):


Steel wires used as permanent or temporary fixation
Dependent on diameter
Available in both smooth and threaded
Threaded wires provide more stable purchase
BUT are weaker & harder to remove
Both are measured by outer diameter
ONLY maintain compression
Sizes: 0.028, 0.035, 0.045, 0.062 inches

Steinmann Pins:
Very similar to K-wires
Larger diameter than K-wires
Provide Inc Stability
Measured in 64ths
Sizes: 8/64 (1/8), 7/64, 6/64…

34
INTERNAL FIXATION CONTINUED…

Monofilament Wire:
Malleable Steel
Provide interfragmentary compression
Measured in Gauges (small gauge = large diameter)
Techniques:
 Cerclage fashion
 circling around a bone
 Interfrag fashion
 placed in between 2 fragments
 always pull on the proximal fragment
 most stable

35
INTERNAL FIXATION CONTINUED…

Monofilament Wire Continued…


 Box Wire fashion
 2 boxes at 90o to each other
 One wire is placed medial to lateral
 Other is placed dorsal to plantar

Tension Band Wiring:


Combines K-wire with MF wire
Requires that there is a soft tissue structural component
Two K-wire’s placed the parallel fashion across fracture site with the
MF wire in a figure 8 pattern around the K-wire on the site opposite to
the tendon’s anatomical pull.
Size of wire measured in gauges -- Lower gauge; thicker wire
26 & 28 are common in Podiatry

36
INTERNAL FIXATION CONTINUED…

Staples:
Primarily used for fractures, osteotomies & fusions
Inserted manually or with pneumatic gun
Provide compression, distraction or maintain compression
Be careful about thickness of bone – DO NOT use if cortical bone is
greater than 2-3mm, may cause cortical fractures or not seat in bone
properly

Pre-Drills:

Neutralization

Compression  = Divergent Lines

Distraction  = Convergent Lines

Richards Staple: GOLD STANDARD for major fusion


Os Staple: Heat activated
Uniclip: Has an aperture
Requires a tool to compress the legs after insertion

37
INTERNAL FIXATION CONTINUED…

Screws:
 These features can differ depending on the function of the screw.

Head Has various configurations; hexagonal, cruciate, slotted…

Land The curve-contoured underside of the screw head


Increases the surface contact between the screw and the bone
Reduces the chance from stress-risers

Shaft/Shank Area of the screw that is void of the thread pattern

Thread The means by which the screw purchases the bone

Thread Diameter The diameter across the thread width


Measurement is the value used to describe the screw size

Core Diameter Diameter between the thread patterns

Pitch Distance between the adjacent threads

Run-Out Junction where the shaft meets the thread


Weakest point on the screw
Avoid placing the run-out near the fracture / osteotomy site

Lead Distance that the screw advances with each turn (360o)

Rake Angle Thread to axis angle

Tip Angle Tip to axis angle

Tip Either rounded (needs pre-tapping) or fluted (self-tapping)

38
INTERNAL FIXATION CONTINUED…

Screw Diagram:

39
INTERNAL FIXATION CONTINUED…

Types of Screws:
1. Cortical Screws
Threaded the entire length of the screw
Have smaller pitch for greater number of contacts between the
screw and the dense cortical bone
2. Cancellous Screws
Partially threaded
Larger pitch to provide greater distance of contact between the
screw and the less dense, porous cancellous bone
3. Cannulated Screws
Hollow center down the length of the screw to be used over a guide
wire
Offers easier placement and less complications
May have decreased pullout strength
4. Herbert Screws
Ho head and two set of threads proximally not distally
Proximal threads have greater pitch than the distal threads
Indicated for intra-articular fractures
Compressive strength of Herbert screw are less than conventional
screws
5. Interference Screws
Fully threaded, headless screw
Does not provide interfragmentary compression but resists axial
displacement of one fragment on another
Indicated for stabilization of tendon grafts to bone and tendon
reattachment

40
INTERNAL FIXATION CONTINUED…

Types of Screws Continued:

6. Absorbable Screws
Available in natural / synthetic polymers
Most common absorbable polymers used are based on alpha-
hydroxy acids such as L-lactic acid, glycolic acid, & para-dioxanone
Need to be able to last 6-8 weeks

Basic Properties for the Ideal Absorbable Implant:


~ Posses and initial strength to meet biomechanical demands
~ Degrades in a predictable manner over time
~ Undergoes complete absorption without harm to surrounding tissues

41
INTERNAL FIXATION CONTINUED…

General Screw Fixation Technique:


~ Place one screw perpendicular to the fracture / osteotomy line for
maximal compression. Place the 2nd screw perpendicular to the
longitudinal axis of the bone this provides greatest resistance to the axial
loading forces on the bone.

~ If only a single screw placement is allowed – place the screw in an


angle that is halfway between the angle that is perpendicular to the
fracture line and perpendicular to the long axis of the bone

Diagram:
A. 2 Screw Technique
B. 1 Screw Technique

42
INTERNAL FIXATION CONTINUED…

General Screw Fixation Technique Continued:


Load Screw Technique
This technique is commonly used in plate fixation.
Involves placement of 2 screws in the plate that is closest to the
fracture line to be drilled offset away from the fracture line.
As the screws are advanced the bone segments between the two screws
are further compressed.

Lag Screw Technique


Placement of the screw so that ONLY the thread engages the distal
cortex of the bone.
Thus further advancement of the screw results in approximation and
subsequent interfragmentary compression.
Most effective in fracture / osteotomy that is 2x’s the width of the
bone or has a fracture angle that is less than 40o.

Partially Threaded Screw Insertion Technique


1. Thread / Pilot Hole
2. Countersink (increases surface contact between screw head and the bone)
3. Depth gauge (measures distance between the proximal and distal cortex)
4. Tap
5. Insert Screw

Fully Threaded Screw Insertion Technique


1. Thread / Pilot Hole
2. Countersink
3. Glide Hole
4. Depth gauge
5. Tap
6. Insert Screw

43
INTERNAL FIXATION CONTINUED…

Screw Selection Chart:

Thread Diameter Thread Hole Gliding Hole Tap Diameter


Mini-Fragment
1.5 1.1 1.5 1.5
2.0 1.5 2.0 2.0
2.7 2.0 2.7 2.7
Small-Fragment
3.5 2.5 3.5 3.5
4.0 (partial/cancel) 2.5 NA 3.5
4.0 (full/cancel) 2.5 NA 3.5
Large-Fragment
4.5 3.2 4.5 4.5
4.5 (mall) 3.2 NA 4.5
6.5 (partial/cancel) 3.2 NA 6.5
6.5 (full/cancel) 3.2 NA 6.5

44
INTERNAL FIXATION CONTINUED…

Soft Tissue Anchors:


 Used for reattachment of tendons or ligaments
 2 basic types: Expandable / Screw type
 Complications: Improper Placement / Failure of Suture / Pullout

Plates:
 Various size and shape – allow alignment of the bones and stability
across the fracture / osteotomy site
 Stability allows for early passive RoM
 Adequate screw fixation is important for the plate to function properly
 Plate designs include semitubular, 1/3 tubular, ¼ tubular, T – plate, L
– plate, calcaneal plate…

Types of Plates:
1. Neutralization Plate
a. Prevents torsional / bending forces from acting on the lag
screws
b. The ridge extension of the plate on the bone proximal & distal
to the fracture / osteotomy site helps neutralize any extra forces
along the bone segment

2. Compression Plate
a. Generate compressive forces along the fracture / osteotomy site
by either placing the plate on the tension side of the bone, off-
set drilling (AKA load screw technique) or pre-bending the plate.

45
INTERNAL FIXATION CONTINUED…

Plates Continued…

3. Dynamic Compression Plate (DCP)


a. Employs the concepts of offset drilling with unique plate
designs to optimize the compressive forces of the plate
b. Disadvantage is it increases periosteal damage and decrease
intramedullary blood supply to the area, decreasing the overall
strength of the bone segment

4. Limited Contact Dynamic Compression Plate


a. Has a series of recessed undercuts on the undersurface of the
plate which allows limited contact between the bone and the
plate
b. Generates less disruption to the vascular supply

5. Buttress Plate
a. Anchored to the main stable fragment
b. Supports the load-bearing bone
c. Indicated in impacted fracture that results in comminution (e.g.
tibial plateau and the tibial pilon fractures)

6. Bridge Plate
a. Useful in unstable comminuted fractures by spanning the
length of the comminution
b. Frequently used with bone grafts to fill the voids in the bone

46
EXTERNAL FIXATION
External fixation implements the use of wires, pins, and rods to keep
bone segments in alignment or compression. Furthermore they allow
distraction of bone segments by the principle of tension-stress effect.

Advantages:
 Use in open fractures, acute, fractures, infected fractures and non-
unions
 Requires minimal tissue dissection
 Allows compression, neutralization, or fixed distraction of bone
segments
 Length can be maintained in a comminuted fracture
 Allows access to the wound site for care, monitoring and dressing
changes
 Full weight bearing is allowed immediately post-operatively

Disadvantages:
 Requires skin and pin tract care
 Difficult frame construction
 Bulky frame
 Fracture through the bone is possible
 Refracture possible after frame removal
 Expensive

Basic Principles of External Fixation:


1. Frame should avoid and respect all vital structures in the area
2. Allow access to the wound site
3. Frame must meet the mechanical demand of the patient and injury

47
EXTERNAL FIXATION CONTINUED…

Complications:
 Pin irritation – avoid pin placement in muscle
 Pin tract infection – most common complication (30%)
 Neurovasculature Impalement – Anterior Tibial A. & Deep Fibular N.
and they are most commonly involved
 Delayed Union / Non-Union – due to faulty frame construction
 Compartment Syndrome – due to increase in the intracompartmental
pressures (mmHg)
 Refracture – once the frame has been removed due to tension
shielding, a rare complication

Types of External Fixators:

1. Unilateral Fixators
 Produces compressive or distraction forces
 Used to fixate fractures, fuse joints, and lengthen
 Available in small or large, it is attached to the bone by multiple
half-pins screwed into the bone and attached to the fixator with the
clamp
 Main disadvantage – not create any sagital plane stability &
therefore should not weight bear immediately post-op

48
EXTERNAL FIXATION CONTINUED…

Types of External Fixators Continued…

2. Circulator Fixators
 Produces compressive and distraction forces
 Used to fixate fractures, treat non-unions, limb-lengthening , soft
tissue lengthening, and correction of congenital deformities.
 Utilizes trans-osseous wires with half-pins to position the wires in
different plane stability
 Limited by the circular frame’s ability to fit the extremity and
patient’s comfort of wearing the apparatus

3. Hybrid Fixators
 Combination of unilateral and the circular fixator
 Used to treat tibial plafond fractures and pilon fractures
 Utilizes trans-osseous wires and half-pins and footplate to allow
early weight bearing

4. Taylor Spatial Frame Fixators


 Newest external fixation device
 Allows for reduction and stabilization of fracture
 Its unique feature allows for reduction of complex triplane
deformities

49
EXTERNAL FIXATION CONTINUED…

Dynamization:
After removal of the plate, the bone may be prone to re-fracture during
weight-bearing because of weakening of the bone from disuse
osteopenia. To prevent this complication it is important to gradually
release tension in the trans-osseous wires and loosen the pins to allow the
bone to gradually strengthen as it bears weight.

Fixator Care & Management:


Pin sites need to be kept clean with sterile solution and applied antibiotic
cream in order to prevent infection and seal the opening around the pins.
Avoid applying Betadine around the pins in order to avoid corrosion.

50
COMMON FOREFOOT PATHOLOGIES AND SURGERIES

Hallux Limitus / Rigidus


Decreased or absent RoM at the 1st MPJ
Normal RoM = 90o (20-25o PF + 60-65o DF)

Radiographic Appearance
AP Lateral
 Focal joint space narrowing  Dorsal Flag Sign
 Joint mice  Spurring
 Spurring  Sclerosis
 Asymmetry  Metatarsus Primus Elevatus
 Squaring of metatarsal head

Etiologies = TIN-MAC
Trauma
Infection
Neoplasm of bone or soft tissue
-
Metabolic
Anatomic
Structural = short/long 1st ray, Met Primus Elevatus
1. Meary’s Angle deviation (b/s talus should b/s 1st met)
2. Parallelism between 1st & 2nd metatarsals
3. Metatarsal parabola / protrusion deviation
Biomechanical = pronation, hypermobile 1st ray
Congenital

51
COMMON FOREFOOT PATHOLOGIES AND SURGERIES CONTINUED…

Hallux Limitus / Rigidus Continued…


Joint Procedures:
Joint Preserving
1. Cheilectomy = Valenti (V-cheliectomy)
2. Osteotomies
 Proximal Phalanx = Bonny-Kessel (proximal DFWO)
 1st Metatarsal
Waterman = Distal DFWO
Mitchell = step-down shortening procedure
Youngswick = chevron double dorsal cut elevates
Sagital Z = corrects for elevates
Lambernudi = diaphyseal PFWO, for elevatus
Joint Destructive
1. Keller = Proximal Phalanx arthroplasty / for elderly / less
functional
 Complications – transfer metatarsalgia, stress fracture of 2nd,
proximal migration of sesamoids
2. Implant = Hemi or Total – must cover cortical surfaces
3. McKeever = 1st MPJ arthrodesis – positioned dorsiflexed and
abducted with no rotation
 DF = 10-15o off weight bearing – one finger under toe
 5-10o of abduction
 Toe will no longer bend so patient cannot squat down

Joint Distraction with External Fixator:


1. Cheilectomy, mini rail
2. 7mm distraction intra-operatively, 2 weeks rest, then 1mm
distraction qd for 7d = Total 14mm Distraction

52
COMMON FOREFOOT PATHOLOGIES AND SURGERIES CONTINUED…

Hammertoes
Function of Lesser Digits:
 Decelerate the foot
 Stabilize the forefoot
 Aid in propulsion
 Provide kinesthetic sensation

Function of Musculature:
 EDL / EDB = dorsiflex MPJ – passive flexion at PIPJ / DIPJ
 FDL / FDB = actively plantarflex MPJ, PIPJ, DIPJ
 Interossei = prevent buckling
 Lumbricales = hold digits rectus (plantarflex MPJ, dorsiflex PIPJ /
DIPJ)

Types of Deformities:
MPJ PIPJ DIPJ
Hammertoe Extension Flexion Extension
Claw Toe Extension Flexion Flexion
Mallet Toe Rectus Rectus Flexion

Etiologies for Contracted Digits:


1. Flexor Stabilization (Most Common)
 Weakness of intrinsic Interossei Ms
 Adv. of Quadratus Plantae
 Pronated foot type – flexors fire longer and harder
 Causes AdductoVarus deformity on 4th and 5th
 Late stance phase biomechanical abnormality
 Tx = Derotational Arthroplasty

53
COMMON FOREFOOT PATHOLOGIES AND SURGERIES CONTINUED…

Hammertoes
Etiologies Continued:
2. Flexor Substitution (Least Common)
 Weakness of Triceps Surae – Flexors gain mechanical advantage
over extensors
 Supinated foot type – late stance phase abnormality
 Tx = suture FDL to Achilles tendon to strengthen muscles
 Must perform Arthrodesis

3. Extensor Substitution
 Weak Tibialis Anterior – extensor gains mechanical advantage
over Lumbricales
 Begins flexible and becomes rigid  reduce early w/ weight
bearing
 Pes Cavus / Ankle Equinus / TA weakness / EDL spasticity and
pain are frequent symptoms
 Swing phase biomechanical abnormality
 Tx = Arthrodesis if Rigid
Hibb’s Tenosuspension if Flexible

54
COMMON FOREFOOT PATHOLOGIES AND SURGERIES CONTINUED…

Hammertoes Surgical Procedures:


SOFT TISSUE
1. Tenotomy = stab incision medial or lateral to tendon deformities only
 PF digit with blade in place – flexible deformities only
 Older population only – lose strength & stability
2. Capsulotomy
3. Tendon Transfer
4. Girdlestone
 Transfer FDL & FDB to dorsal head of proximal phalanx to restore
intrinsic function
5. Hibbs
 Transfer EDL to base of proximal phalanx or met head
6. Kuwada & Dockery
 Modification of Girdlestone – drill hole in base of proximal phalanx
and bring tendons up through it
7. Lengthening
8. Z-Plasty at level of MPJ
9. Percutaneous stab incision and splint
**Complications: Muscle spasm caused by overcorrection, tenosynovitis,
scarring, adhesion, weakness, bowstringing, and nerve entrapment

OSSEOUS
1. Arthroplasty
 Post – resection of base of proximal phalanx
 Gotch & Kreuz – resect base of proximal phalanx and syndactylize
digits
2. Arthrodesis
 Lambrinudi – fusion of PIPJs and DIPJs
 Young-Thompson – Peg-in-Hole Fusion (Peg from Prox.Phalanx)
 High amount of shortening
3. Taylor – PIPJ fusion using K-Wire

55
COMMON FOREFOOT PATHOLOGIES AND SURGERIES CONTINUED…

Hammertoes
Sequential Reduction:
1. Z-Plasty
2. Arthroplasty
3. Extensor Hood Release
4. MPJ Capsulotomy
5. Volar Plate Release
6. Tendon Transfer (Girdlestone, Kuwanda & Dockery, Hibbs)

** Kelikian Push-Up Test: Performed between each step to determine if


sufficient correction has been established. If you get dorsiflexion when
placing GFR on the met head then do the next step.

Hallux Hammertoe:
Etiology:
 Muscle imbalance
 Iatrogenic after sesamoid removal or detachment of FHB
 IPJ sesamoid binding FHL tendon

Treatment:
Flexible – IPJ fusion with EHL lengthening
Rigid – IPJ fusion with Jones Tendon Transfer
~ Cut EHL distally from insertion
~ Drill hole transversely through 1st med head
~ Insert tendon through drill hole and suture back on itself

56
5TH DIGIT ARTHROPLASTY
1. Lazy “S” Incision
 Lateral condylectomy of distal and middle phalanges with resection
of head of proximal phalanx

2. Derotational Arthroplasty
 Distal Medial  Proximal Lateral Incision

3. Complications
 Floppy Digit
 Edema (sausage digit)
 Floating Toe with Metatarsalgia
 Regeneration of Proximal Phalanx
 Infection
 Decreased sensation
 Blue toe

57
REARFOOT SURGERY
Spurs are incidental findings only and are rarely the cause of pain.
1. May be painful if directed plantarly
2. Must be present to be approved for orthotics

Conservative therapies should be used for the first 3-9 months

Plantar Fasciotomy:
1. Plantar L shaped incision at the medial midfoot
2. Release of the medial band of the plantar fascia
3. NWB for 3 weeks
4. Sutures out after 3 weeks

Endoscopic Plantar Fasciotomy:


1. Small incision in the medial rearfoot 3 fingers from the posterior
heel and 2 fingers up from the plantar foot
2. Blunt dissection to the fascia
3. Insert spatula across plantar aspect of foot, dissecting fascia from
plantar fat pad – remove spatula
4. Insert trochar into slotted tube and insert through dissected incision
– remove trochar
5. Insert scope into tube laterally and blunt probe medially – separate
medial and central bands of plantar fascia
6. Insert cutting tool into medial tube and cut medial band of plantar
fascia while pulling instrument out of the tube
7. Visually observe abductor hallucis muscle belly before removing
tube and irrigating incision site

58
REARFOOT SURGERY CONTINUED…

Haglund’s Deformity:
Angles:
Philip-Fowler Angle = normal 44-69o, >75o pathological
Total angle of Ruck = Philip-Fowler + Calcaneal Inclination Angle –
Normal up to 90o, pathological if > 110o
Parallel Pitch Lines – most objective method of determining a
Haglund’s deformity

Procedures:
Longitudinal incision lateral to TA
Dissection down to posterosuperior Calcaneus
Aggressive removal of pathologic bone, but don’t chase the bump
If you need to reflect the TA, reattach with a soft tissue anchor and
remain NWB for 3 weeks

Keck & Kelly Osteotomy:


Indicated for increased CIA angle with no Haglund’s deformity
Dorsal wedge osteotomy of the posterior Calcaneus
Rotate posterior aspect of Calcaneus dorsally after wedge removal –
MAINTAIN PLANTAR HINGE
Secure with cancellous screws
NWB for 6 weeks

59
TENDON TRANSFERS

Tendon Transfer – detachment of the tendon from insertion then relocate


to new position

Tendon Transplantation / Translocation – rerouting the tendon without


detachment from its insertion

Types:
1. Adductor Hallucis
 Resect at insertion, pass under the joint capsule and reattach at
medial aspect of the capsule
 Indicated in HAV to realign the sesamoid apparatus
2. Abductor Hallucis
 Transected at insertion, rerouted inner 1st met head and fixated at
lateral base of proximal hallux
 Indicated in Hallux Varus with an osteotomy
3. Extensor Hallux Longus
 Transected at origin, rerouted under DTIL, fixated to lateral base
of proximal hallux
 IPJ needs fused
 Indicated when have sagital component with Hallux Varus
4. Jones Suspension
 EHL excised from insertion, drill a hole transversely through 1st
met head, rerouted through hole and sutured on itself
 Indicated with cock-up deformity, flexible cavus, lesser
metatarsalgia, chronic ulcers, weak TA, flexible plantarflexion of
1st met
5. Hibb's Tenosuspension
 EDL detached from insertion, bundled together and placed
through midfoot at the base of the 3rd met or lateral cuneiform
 Indicated to release retrograde buckling at MPJs, met equines,
flexible cavus, claw toes

60
TENDON TRANSFERS CONTINUED…

Types Continued...
6. Tibialis Anterior Transfer
 3 incisions at (1) proximal dorsal leg, (2) TA insertion at medial
plantar cuneiform / tubercle 1st met, and (3) the new area of
insertion in the midfoot
 Release from insertion, reroute out the proximal incision, with
tendon, with tendon passer brought to new insertion (usually 3rd
cuneiform)
 Indicated for recurrent clubfoot, flexible forefoot equines,
dropfoot, tarsometatarsal amputation, Charcot Marie Tooth
deformity

7. Split Tibialis Anterior Tendon Transfer (STATT)


 3 incisions at (1) base of 1st met, (2) anterior leg over TA just
lateral to medial malleolus and (3) over peroneus tertius at base of
5th met
 Split tendon through proximal insertion, lateral slip passed
through peroneus tertius sheath and sutured to tendon fixated to
cuboid
 Indicated for spastic RF equines, spastic equinovarus, fixed
equinovarus, FF equines, flexible cavovarus deformity, DF
weakness, excessive supination in gait

8. Cobb Procedure
 STATT but reroute to TA to PA tendon
 Indicated for PT dysfunction

61
TENDON TRANSFERS CONTINUED…

Types Continued...
9. Tibialis Posterior Tendon Transfer
 3 incision (1) insertion of the PT at navicular tuberosity, (2)
anterior leg, middle 1/3 just lateral to tibial crest and (3) one at
new insertion at dorsal midfoot
 Tendon released from navicular Tuberosity, dissected free at the
medial leg insertion to expose the IM and the PT pulled through
this opening then brought to new insertion level (usually 3rd
cuneiform)
 Indicated for weak anterior muscles, equinovarus, spastic
equinovarus, recurrent clubfoot, dropfoot, complications from
Charcot Marie Tooth, peroneal nerve plaste, leprosy, Duchenne’s
MS
 Muscle goes from a stance to a swing muscle during gait

10. Peroneus Longus Tendon Transfer


 3 incisions (1) lateral, lower leg, (2) lateral cuboid and (3) base of
3rd met/lateral cuneiform
 Suture the Peroneus Longus to the Brevis , cut the longus at the
level of the cuboid and the tendon is brought through the
proximal incision and back through the medical incision to the 3rd
cuneiform
 Indicated for anterior muscle weakness, dropfoot

62
TShwer
BUNION PROCEDURES TO KNOW BASED ON ANGLES

IM Angle 12-16o Distal Austin


Osteotomy Hohman (Neck) -- Trapaziodal
o
Normal: 0-8 Mitchell (Neck)
Wilson (Neck)
Reverdin Laird (Distal L)
Short Z
Waterman
Youngswick
o
>16 Proximal Base Wedge
Osteotomy Lapidus (Met-Cuneiform Fusion)Hypermobile
Cresentic
Juavara
Proximal V of Kotzengerb
Comments: with a thin Met shaft  may need to use a proximal procedure
Mitchell – shortens the length of met shaft  used in Long Met Length
( >2mm longer than 2nd met)
Taylor’s Bunion = Symptomatic when IM4-5 >9o
Splayfoot = IM1-2 + IM4-5 >20o

DASA >8o Proximal Proximal Akin


Osteotomy -- cylindrical akin w/ long prox phalanx
Normal: 0-8o -- oblique
-- transverse
-- Bonnel-Kessel  DF wedge
Comments:

PASA >8o Distal Reverdin


Osteotomy Reverdin Green PASA
Normal: 0-8o

Comments:

63
Abnormal IM:12-16o Distal Biplane Austin
PASA + Abn P Osteotomy Reverdin Laird (Distal L)  PASA + IM
+ Reverdin Green
IM Angle Biplane Mitchell  Roux
Hohmann
Shaft Mau
Osteotomy Ludloff
Scarf / “Z”
Klotzenberg
Juvara
IM: >16o Proximal Lapidus w/ Reverdin
+ Abn P Osteotomy V Osteotomy
Logroscino (Base Wedge Reverdin)
Cresentic
Juavara
Proximal V of Kotzengerb
Comments:

HAA > 16o Silver


McBride
Normal: 0-16o Adductor Hallucis Tenotomy
Lateral Capulotomy
Comments: ST or Osseous Abnormality
↑ HAA + IM1-2 13-20o = Lodloff + Mau
(+ ↑ PASA) = Scarf Z

HIA > 10o Distal Akin

Normal: 0-10o
Comments:

64
Tibial Sesamoid Position 4-7 Fibular Sesamoidectomy
Fibular Sesamoid Release
Normal: 0-3
Comments:

Lateral IM: 8-12o Distal Osteotomy Reverse Austin


Deviation Normal 2  exostectomy Reverse Mitchell
Angle Slight  dist. Reverse Hohmann
Normal: 2.5o Increase metaphyseal Reverse Wilson
+ osteotomy Reverse Mercado
IM4-5 LDA: Inc
Normal: 0-8o IM: > 15/16o Proximal Base Wedge
Marked Inc Osteotomy

LDA: ↑
Severe Lat
Bowing
Comments:

65
66
Other Important Things
to Know for 3rd Year
Rotations & Externships
Dr. Bodman’s Drugs p. 68
Dr. Caldwell’s Drugs p. 74
Dr. Caldwell’s Wound Care p. 84
Ankle Scopes p. 90

67
68
69
70
71
72
73
74
TShwer
β – LACTAMASE’S
SENSITIVE PENICILLIN’S
Penicillin V (PO)
G+: Strep
G–: Eikenella corrodensHuman Bites
Penicillin G, Aqueous (IV)
** Tx: Clostridium Tetani if Nisseria gonorrheaSTD Septic Joint
allergic to Tetanus Toxoid Anaerobes: Clostridium perfringens
Penicillin G, Procaine (IM)
** Tx: Treponema Palidum
(Syphilis) HELPS2:
Tx: Strep Throat & Otitis Media Haemophilus
Ampicillin
G+: Strep E. coli
G–: Eikenella corrodens Listeria
Proteus mirabellus
Amoxicillin Nisseria gonorrhea Shigella
** Used in kids instead
of Augmentin!
HELPS2 Salmonella
Anaerobes:
RESISTANT PENICILLIN’S (3RD GENERATIONS) Resist MONDay’s!
M Methacillin (IV)
** Toxic, not used!

O Oxacillin (Bactocil)(PO)
β–Lactamase Resistant Staph Aureus
N Nafcillin (Unipen)(PO)

D Dicloxacillin (PO)
If resistant to this = MRSA!

EXTENDED SPECTRUM PENICILLIN’S (4TH GENERATIONS)


Carboxypenicillins Carbenicillin
High Na+ loads 
avoid pts w/ HTN
Broad SpectrumDo NOT cover β–Lactamase
Beware of Hypokalemia! Ticaricillin
G+: Good Coverage
G–: ↑ Coverage
Ureidopenicillins Mezlocillin Anaerobes: ↑ Coverage

** Tx: Pseudomonas aeruginosa


Piperazine Penicillin Piperacillin
DRUG INTERACTIONS: Warfarin, Oral Contraceptives, Probenecid, Aminoglycosides
C/I with pts on Methotrexate Rheumatiologist
75 Docs tell pts they are C/I!
Pt must take all 10d of meds or else get post-strep glomerulonephritis
β – LACTAMASE’S
COMBINATION DRUGS
Piperacillin / Tazobactam (Zosyn)(IV)
Tx: Pseudomonas aeruginosa & Proteus mirabilis
 Needs 4.5g q6h for pseudomonas
Amoxicillin / Clavulanic Acid (Augmentin)(PO) Broad Spectrum
**Staph aureus is susceptible to G+: Strep, Staph aureus
Augmentin ≤ 35% due to MRSA
G–: Neisseria gonorrhea…
Ticarcillin / Clavulanic Acid (Timentin)(IV) Anaerobes: 

Ampicillin / Sulbactam (Unasyn)(IV)


**Unreliable against G– infections!

DOSAGE BOX:
Zosyn = 3.375g q6h
Augmentin = 875mg q12h
Timentin = 3.1g q6h
Unasyn = 3.0g q6h

76
CEPHALOSPORIN’S
1ST GENERATION
Cephalexin (Keflex)(PO) G+: Good Staph aureus
Staph epidermidis
Cephadroxil (Duricef)(PO) Strep HEN – PEcKS:
Haemophilus influenza
G : Some PEcK

Enterobacter aerogens
Cefazolin (Ancef)(Parenteral) An: Some not B.fragilis Neisseria species
< 80% susceptibility to Ancef Proteus mirabilis
E. coli
2ND GENERATION Klebsiella pneumonia
Serratia
Cefuroxime (Ceftin)(PO) G+: Almost as good as 1st G
(Zinacef)(Parenteral) G–: Extended HEN – PEcKS
Cefoxitin (Mefoxin)(Parenteral) An: 

3RD GENERATION
Cefixime (Suprax)(PO)
G+: Significantly ↓↓↓
Cefpodime (Vantix) (PO)
G–: Superior Coverage
Ceftriaxone (Rocephin)(Parenteral) (Fortaz  Pseudomonas)
*Not good for Staph (Rocephin  Neisseria)
Ceftazidime (Fortaz)(Parenteral)
An: 
Cefdinir (Omnicef)
*Covers Staph & Strep better than 1st G.  MIC levels are superior to Cephalexin
4x’s better for Staph // 7x’s better for Strep
DOSAGE BOX:
4 GENERATION
TH
Omnicef = 300mg q12h
Cefipime (Maxipime)
G+: More active against Staph aureus than 3rd G
G–: Good Coverage
(+ Pseudomonas)
An: 
EXTENDED GENERATION
Ceftobiprole G+:  – Active against MRSA
G–: 
An: 

DRUG INTERACTIONS: Avoid Cephalosporins if pt allergic to Penicillin!


Cefdinir(2ndG) & Cefuroxime(3rdG) are allowed for Penicillin allergy!
 Due to different structure. 77

CARBAPENAM’S

Imipenam–Cilastin (Primaxin)
*ID specialists ONLY! Broad Spectrum
G+:  – Most
G–:  – Most (pseudomonas – resistant)
(mycoplasma – resistant)
An:  – Excellent

Meropenam (Merrem)
*Mostly ID specialists G+: Staph & Strep only (Inferior to Imipenam)
G–:  (Superior to Imipenam)
↓ Seizure Risk! An:  

Ertapenam (Invanz)(IV/IM) DOSAGE BOX:


Invanz = 1g qd
G+:  CrCl < 30ml/min = 500mg 6h pre-dialysis
G–: Limited
An:  

Doripenam
Broad Spectrum
G+: 
G–:  (pseudomonas)
An: 

DRUG INTERACTIONS:
C/I in pts with Penicillin allergy
C/I in pts with Seizure History
Ertapenam interacts with Probenicid.

78
MONOBACTAM

Aztreonam (Azactam)(IV/IM) G+: 


*OK for Penicillin Allergy G–:  (?pseudomonas)
No major renal toxicities; only
dose adjust for renal insufficiency
An: 
or dialysis

AMINOGLYCOSIDES

Gentamycin ADVERSE DRUG REACTIONS:


G+:  MRSA
Tobramycin Ototoxicity (Irreversible)
G–:  Proteus mirabilis
Amikacin Nephrotoxicity (Reversible)
Pseudomonas
* NOT use on Diabetics or MyGravis Klebsiella HypoK+ ––– Gentamycin
E. coli HypoMg+ –– Amikacin
Peak (30min post dose) & Trough
(30min before next dose) levels are Salmonella
recommended Shigella
An: 

GLYCOPEPTIDES
1ST GENERATION
Vancomycin (PO/IV) G+:  IVMRSA

Tx: Endocarditis Prophylaxis G–:  POClostridium difficile


for pts allergic to β-lactams An: 
- Keep for reserve cases! DOSAGE ADJUSTMENT BOX:
50kg: 750mg
DOSAGE BOX: Normal Trough = 5-10mg/dL
50-74kg: 1000mg If trough range >15mg/dL 
Vancomycin = 1g slow push IV 75-90kg: 1250mg double the dose time interval
(over 60min) >90kg: 1500mg

2ND GENERATION G+:  -- MRSA, VRSA, Strep, C.difficile


Dalbavancin(PO/IV) G–: ?
An: ? ADVERSE DRUG REACTIONS:
Ototoxicity (Reversible)
3RD GENERATION Nephrotoxicity (Reversible)
Telavancin G+:  -- MRSA, VRSA, Strep Red Man Syndrome
G–: ? Vestibular Imbalance
An: ? Thrombophlebitis
79
Absorption is limited by:
TETRACYCLINE’S ~ Food ~ Milk
~ Antacids ~ Iron

Doxycycline G+:  Staph aureus


* Some Anti-Inflammatory
properties seen on OA MRSA
Minocyline G–:  E. coli
Klebsiella
Enterobacter
Methacycline Vibrio vulnificansSalt Water
Rickettsia
Tetracycline HCL Chlamydia
An:  
ADVERSE DRUG REACTIONS:
NonSpecific GI Issues ~ Don’t give Doxycylcine before bed  erosive esophagitis
Photosensitivity
Photo-Onycholysis (Doxycycline)
?Acute Pancreatitis
CONTAINDICATIONS:
No Pregnant / Kids ~ tooth discoloration in kids under 8y
C/I for pts on Digoxin -- ↑ Toxicity
C/I for pts on AccutaneAcne -- ↑ ICP, Pseudomotor Cerebri Risk

MACROLIDE’S
* Erythrasma  Coral Red Woods Lamp

Erythromycin G+:  Staph / Strep / Corynebacterium minitussimum*


Rarely used -- poor
ST penetration
G–: 
ADVERSE DRUG REACTIONS:
An:   GI Upset
Oral Dose = 2x Risk of Sudden Death Prolonged Heart Depolarization ~ Torsades de Pointes
Combined with Ca2+ Chanel Blockers DRUG INTERACTIONS: Potent Inhibitors of CYP 3A4
= 5x Risk of Sudden Death Cyclosprine / Sirolimus / Tacromilus
(Verpamil, Diltiazam) C/I for pts on Carbazepine & Theophyline
Azithromycin
(Zithromax) G :  Staph / Strep
+
ADVERSE DRUG REACTIONS:
Detox in Liver G–:  GI Upset
Excreted in Bile An:   Prolonged Heart
DRUG INTERACTIONS:Potent Inhibitors of CYP 3A4
DOSAGE BOX: ↑Digoxin // ↑Coumadin
Zithromax = 500mg 1st Day
(z-pack) 250mg qd next 4 days HMGcoA Reductase ↑
80
LINCOSAMIDES

Clindamycin (Cleocin) G+:  Fulminate Group A StrepNecrotizing Fasciitis


* Good Bone Penetration
* Poor CNS Penetration Group B Strep ~ may show resistance
MRSA ~ may show resistance
DOSAGE BOX:
Cleocin = 600mg 1hr pre-op Staph Aureus** *Staph Aureus resistant to
Given as prophylaxis for G–:  erythromycin on C&S can
bacterial endocarditis An:  B. fragilis develop inducible resistance to
Clindamycin
*C&S of organism is sensitive
to Clindamycin but resistant to
ADVERSE DRUG REACTIONS: erythromycin  do NOT give
Diarrhea Clindamycin because it will
Pseudomembranous Colitis develop resistance
DRUG-DRUG INTERACTION:
↑Respiratory Paralysis with m. relaxants (Baclofen / Diazepam)

CHLORAMPHENICOL

G+:  ADVERSE DRUG REACTIONS:


G–:  Gray Baby Syndrome
An:  Serious Infections Severe Bone Marrow Toxicity
Aplastic Anemia
. Last resort for VRE

SULFONAMIDES

Trimethoprim–Sulfamethoxazole (Bactrim / Septra)* Beware in pts over 50 year old

G+:  Staph & Strep ADVERSE DRUG REACTIONS:


MRSA Acute pancreatitis
G–:  DRUG-DRUG INTERACTION:
An:   T–S + Methotrexate = ↑ Bone Marrow Suppression
T–S + Coumadin/Digoxin = ↑ Toxicity of C/D
T–S + Oral Sulfonylureas = Hypoglycemia

81
5-NITROIMIDAZOLE

Metronidazole (Flagyl) G+:  ADVERSE DRUG REACTIONS:


* Tx Pseudomembranous Colitis Peripheral Neuropathies
G–: 
N/V with Alcohol Consumption
An:  B. fragilis Dark Brown Urine
DRUG-DRUG INTERACTION:
DOSAGE BOX:
Flagyl = 15mg/kg loading dose ↑ Anti-Coagulation effects of Warfarin
7.5mg/kg q6h IV -or- 500mg tid

FLUORINATED 4-QUINOLONES

Ciprofloxacin (Cipro) G+:  CONTRAINDICATIONS:


DOSAGE BOX: G–:  most active Under age 18
Cipro = 750mg bid Pregnant / Nursing
against P. aeruginosa * Attacks joints
infection of bones & joints *Can cause Tendonitis / Rupture
An:  
Levofloxacin (Levaquin)
G+:  Strep ADVERSE DRUG REACTIONS:
DOSAGE BOX: G–:  N. gonorrhea GI / Headache / Phlebitis all are rare!
Levo = 500mg qd (po/IV)
An:  
* Post antibiotic effects (G+)

Moxifloxacin (Avelox)
Broad Spectrum
* Good in ST
* Good for diabetic foot G+:  Staphsome resistance
infections with inoperable Strepenhanced
atherosclerosis G–: 
* May work against TB
An:  B. fragilis

DRUG-DRUG INTERACTION:
Not give within 2hr of: Multivitamins, Antacids, Sulcralfate
MANY interactions! – Theophyline, Caffeine, Warfarin, NSAIDs, ddI (HIV)
May see… Torsades de Pointes & Ventricular Fibrillation

**May produce a false (+) on viral assay for opiates

82
RIFAMYCIN
Rifampin
Broad Spectrum DOSAGE BOX:
* Turns fluids Orange
Not given alone  Give with Cipro / Bactrim
* CYP 450 system G+:  Staph aureus
* Tx Leprosy Strep epidermidis
* Tx Vanco Resistant MRSA
MRSA
G :  N. gonorrhea

Mycobacterium
An:  

STREPTOGRAMINS
Dalfopristin/Quinopristin (Cipro) ADVERSE DRUG REACTIONS:
* Reserve this drug!!! G+:  VRE Arthralgia / Myalgia
MRSA Nausea
MRSE Thrombophlebitis
G–:  ↑ LFT’s
An:   Injection Site Reaction

OXAZOLIDINONES
Linezolid (Zyvox)
* Good bone penetration G+:  VRE ADVERSE DRUG REACTIONS:
* Check weekly CBC’s MRSA Mylosuppresion
VRSA N/V lactic acidosis
G–:  Optic Neuropathy Tx >1mo
An:  

CYCLIC – LIPOPEPTIDE
Daptomycin (Cubicin)
* Check weekly CPK’s G+:  MRSA DRUG INTERACTIONS:
VRSA Tobramycin
DOSAGE BOX:
Statins  Myopathy
4mg/kg qd G–: 
An:  

GLYCYLCYCLINES
Tigecycline (Tygacil)
* Check weekly CPK’s Broad Spectrum ADVERSE DRUG REACTIONS:
G+:  MRSA N/V
DOSAGE BOX:
Tooth Discoloration
IV: 100mg Loading Dose VRSA
50mg bid G :  An: 83 

ORAL ANTIBIOTICS
Itraconazole (Sporanox)
Dermatophytes ADVERSE DRUG REACTIONS:
Onychomycosis
Candida GI upset / Rash / Headache
Tinea pedis(off label) 
Molds  Hepatotoxicity  LFT’s ALT & AST
DRUG INTERACTIONS:
DOSAGE BOX: Statins Ca2+ Channel Blockers
Pulse Dosing = 2x 100mg tabs in AM & PM with food Tikosyn Erythromycin
Take for 1 week of the mo. for ___ months
CONTRAINDICATIONS:
Patient with CHF!
Terbinafine (Lamisil) Dermatophytes ADVERSE DRUG REACTIONS: Rare
Onychomycosis  Headache / Abnormal Taste
Tinea pedis(off label)  Green Vision
DOSAGE BOX: DRUG INTERACTIONS: CYP450 2D6
Pulse Dosing = 250mg qd 1 week/mo over 2mo Cimetidine Cyclosporine
Normal = 250mg qd 3mo Rifampin Nortriptyline
Caffeine

Fluconazole (Diflucan) Dermatophytes ADVERSE DRUG REACTIONS:


DOSAGE BOX: Candida Severe Skin Rash
Pulse = 300mg/week Molds  Alopecia
Drug Interactions: CYP450 3A4

Griseofulvin (Gris-PEG)
Dermatophytes ADVERSE DRUG REACTIONS:
Chronic Tinea pedis  Paresthesia / Rash / Headache
DOSAGE BOX: DRUG INTERACTIONS:
250mg tid (x4-8 weeks) Oral Contraceptives
Warfarin
Barbituates

ORAL ANTIPARASITIC
Thiabendazole (Mintezol) DOSAGE BOX:
* Cutaneous Larva Migrans Mintezol = 10% aqueous solution qid
Ivermectin Ivermectin = 200μg/kg po x1dose for 1-2days

84
TShwer
WOUND CARE & DRESSINGS

ACTICOAT = Nanocrystalline Silver (antimicrobial effect up to 7days)


 Reduces Exudates while maintaining a moist wound environment
 Moisten with sterile water (NOT SALINE!!! Silver reacts with Saline)
 Effective against VRE & MRSA

IODOSORB GEL / IODOFLEX DRESSING = Absorbent Iodine Cadexomer


 Slowly releases small amounts of a 0.9% elemental iodine

ALLEVYN FOAM
 Moderate to High Exudate
 Never use the adhesive type!

HYPAFIX
 Adhesive, non-woven fabric
 Hold post-op dressings / catheters / drainage tubes in place

HYDROGEL SHEETS = ElastoGel, Nu-Gel, Vigilon, Amerigel


 Low Exudate
 Re-Epithelializing wounds
 NonAdhesive
 Gas permeable
 (+) Provides Moisture
 qd change for infected wounds
 (+) No trauma upon removal
 (–) Potential to macerate surrounding skin

HYDRO-GEL = Duoderm Gel, Nu-Gel, Restore, Hypergel


 Low Exudate
 Partial  Full thickness wounds
 Use once granulation tissue is present
 (+) No trauma upon removal
 (+) Provides Moisture
 (–) Potential to macerate surrounding skin

85
TRANSPARENT FILMS = Opsite, Tegederm, Bioclusive, Epivew
 Adhesive, Polyurethane film
 Low Exudate
 May be used over absorptive wound filter or hydrogels
 NOT for INFECTION!
 Superficial Wounds (Blisters)
 (+) Up to 7d wear time (semi-permeable)
 (+) Allows visual assessment
 (+) Provides Moisture
 (–) Potential to macerate surrounding skin with excessive drainage
 (–) NOT absorptive
 (–) Adhesive may tear healthy skin

HYDROCOLLOIDS = Duoderm, Duoderm CGFControl Gel Formula, Tegasorb, Restore


 Adhesive, Occlusive
 Low Exudate
 Granulating & Epithelializing Partial Thickness Wounds
 May be used over absorptive wound filter or hydrogels
 NOT for INFECTION!
 Cover @ least 1inch of surrounding skin
 (+) Up to 3d wear time
 (–) May tear healthy skin
 (–) Potential to macerate surrounding skin with excessive drainage
 Change dressing before it leaks
 Odor/Drainage are Normal

FOAM = Acticoat Moisture Control, Allevyn, Polymem, TeilleAdhesive Border


 Polyurethane
 Adhesive or NonAdhesive
 Moderate to High Exudates
 Varying Thickness
 Infected wounds if changed daily
 Venous Leg Ulcers
 (+) Up to 7d wear time
 (–) May tear healthy skin

86
ALGINATE = Sorbsan, Dermacea Alginate, Kaltostat, Curasorb
 Seaweed Polymer
 Gel formed when fibers interact with wound fluid
 Pad or Rope Form
 Partial/Full Thickness Granulating Wounds
 Moderate to High Exudates
 (+) Haemostatic effect
 (+) Up to 7d wear time
 (–) Requires 2o dressing
 Infected wounds if changed daily
 Tan mucoid appearance upon removal
 ALGINATE WITH COLLAGEN = Fibracol
 90% Collagen, 10% Alginate
 ALGINATE IMPREGNATED WITH SILVER
ABSORPTIVE WOUND FILTERS
 Sheets, Rope, Paste, Granules, Powder made of Starch Polymers
 Deep Wounds
 Heavy Exudate
COLLAGEN BASED PRODUCTS: Medifil Particles/Pads/Gels, SkinTempNylon Mesh over
 Use Collagen Gels for Dry Wounds collagen membrane

 Use Sheets for Low  Moderate Exudative Wounds


 Use Powders, Particles, Pads for Moderate  Heavy Exudative Wounds
 Actions: Absorbent, Hemostasis, Chemotaxis, Provisional Matrix in wounds
for Granulation tissue formation
 PRISMA  colonized or contaminated wounds
 55% Collagen // 44% ORC Oxidized Regenerated Cellulose
 1% Silver
 PROMOGRAN
 55% Collagen // 44% ORC
 Only matrix proven to bind & reduce MMPs Matrix MetaloProtinase
 ORC/Collagen combo binds more MMPs in the dressing the ORC or
Collagen alone
 PEGASUS = Unite Biomatrix
 Enzyme resistant collagen scaffold -- Fenestrated

87
RECOMBINANT DNA TECHNOLOGY
 REGRANEX = Becaplermin
 Recombinant PDGF platelet-derived growth factor
 Attracts monocytes & fibroblasts -- inflammatory phase
 Stimulates granulation tissue
 Refrigerate
 Regranex Gel 0.01%
 15g tube, apply qd, spread evenly and thin (1/16th inch)
 Cover in moist saline gauze dressing
 PROCURAN = Thrombin–Induced Platelet Releasate
 GF from patients own blood
 50-200cc of blood drawn from patient
 Spin down, separate, activate the thrombin
 1 blood draw = 3mo of daily application

GRAFTS
 APLIGRAFT = Bilayered Skin Equivalent
 Epidermis & Dermis Dermis side down
 Newborn foreskin
 FedEx in 24hr in petri dish – use immediately
 Place a compressive wrap over it
 DERMAGRAFT = Human Dermal Replacement
 Newborn foreskin
 Cover with Allevyn & Hypofix tape
 DO NOT use with any other topical agent
 OASIS
 Small Interstine Submucosa Pig/Porcine
 SIS scaffold attracts patients cells
 Store @ room temp up to 18mo
 INTERGRA
 Collagen–GlycosAminoGlycan Biodegradable Matrix Cow/Bovine
 Porous Matrix of cross-linked bovine tendon collagen/GAGs
 Semi-Permeable Polysilxane (Silicone) layer
 Sterile Preperation

88
 GRAFT JACKET
 Processed Human Dermal Membrane
 3-D Bioactive Frame – supports granulation tissue
 Deep Wounds
 GAMMAGRAFT
 Irradiated human skin allograft
 Epidermis & Dermis
 Store @ room temp
 After 24hr in place -- remove secondary covering and allow area to air-
dry for 2-3hr  once dried in place there is no need to recover it
 (+) Patients can do this at home
TOPICAL ENZYMES:
 SANTYL = Collagenase
 Digests collagen in necrotic tissue
 Collagen in healthy tissue or in newly formed granulation tissue is not
attacked
 May be used as an Antibiotic Powder
 Stop use when granulation tissue is well established
 Accuzyme, Gladase
 Papain Proteolytic enzyme from papaya
 Urea  Protein denaturing agent
 May have a burning sensation in patients RARE
 Cleanse with normal saline, NOT water
 Panafil
 Papain
 Urea
 Chlorophyllin Copper Complex Sodium  Inhibits hemagglutinating &
inflammatory properties of protein degradation products in the wound
 Elase = FibrinolysinDesoxyribonuclease RARE 2 Find
TOPICAL AGENTS FOR LOCAL BLOOD FLOW
 XENADERM OINTMENT
 Balsum of Peru  Increased blood flow to wound site
 Castor Oil  Creates a moist environment
 Trypsin  Maintains moist wound bed
 Aluminum Magnesium Hydroxide Stearate  Fluid Repellent

89
ANKLE SCOPES
PORTALS:
 Anterior
o AnteroMedial
 Medial to Tibialis Anterior
 Visualize: medial gutter & medial transchondral margins
 Caution: TA, Saphenous V & N
o Accessory AnteroMedial
o AnteroLateral
 Lateral to EDL or Peronial Tertius
 Visualize: lateral gutter
 Caution: EDL, Peronial Tertius, Superficial Peroneal N
o Accessory AnteroLateral
o AnteroCentral
 Lateral to EHL
 Caution: AntTibial A, Deep Peroneal N, EHL & EDL tendons
o Medial Midline Portal
 Posterior
o PosteroMedial
 Medial to the Achilles Tendon
 Caution: Sural N, Lesser Saphenous V
o Accessory PosteroMedial
o Modified PosteroMedial
o PosteroLateral
 Lateral to the Achilles Tendon
 Visualize: the posterior process of the talus & posterior media talar dome
 Caution: T-D-A-N-H
o Accessory PosteroLateral
o TransAchilles  6 Central Points
o Coaxial Portals o Med / Central / Lat – TibioTalar Artic
o Posterior Inferior TibioFibular Lig
21 POINT EXAM: o Transverse TibioFibular Lig
 8 Anterior Points o Capsular Reflection of the FHL tend
o Deltoid Lig  7 Posterior Points
o AntMed Gutter o PostMed Gutter
o Med / Central / Lat – Talar Dome o Med / Central / Lat – Talar Dome
o Ant TibioFibular Articulation o Post TibioFibular Artic
o AntLat Gutter o PostLat Gutter
o Anterior Gutter 90 o Posterior Gutter
91
92
93
94
95
96
97
98
99
100
PODIATRIC RADIOLOGY also leave epiphyseal scars which may look like stress fxs.
These occur in predictable anatomical areas. Epiphyseal
INTERPRETATION OF PEDAL RADIOGRAPHS plates tend to close from medial to lateral in teenagers.
X-ray interpretation is a written notation of pathological Talar beaking is a finding on lateral of the distal neck of the
findings made in objective terminology. Correlation of talus that occurs secondary to capsular pressure.
radiographic findings along with clinical and lab findings Trabeculae in some circumstances may be accentuated, and
allows a physician to arrive at differential diagnoses. X-ray look possibly like a fx.
interpretation follows the 5-step approach: The calcaneal apophysis is located at the posterior aspect of
1. Quality of Film the calcaneus, and has irregular margins. It is often mistaken
for a fracture. It also may be normally bifid.
• Too light or too dark (background/film density)
Compact bony islands are normal and benign variants.
• Proper projection
A simulated fracture may occur in any bone w/ more than
• Proper positioning one ossification center.
• Adequate contrast The silver dollar navicular is a normal (small) variant of
2. Soft Tissue Assessment navicular development – be sure to r/o Kohler’s dz.
• Increased ST density (edema, obesity) A pronated foot may cause the spaces between bones in the
• ST calcification (vascular, traumatic, chronic) midfoot to be accentuated, causing a simulated lisfranc’s fx.
• ST contour – thicker medially and proximally A pseudo-epiphysis is a secondary ossification center.
• Foreign bodies (r/o artifact w/ multiple views) The gun-barrel effect is seen when a bone (usually phalanx)
3. Bone Assessment is perpendicular to the film plane. It may look like a cyst.
• Number of osseous structures – start distally and Distal phalangeal condyles are normal variants.
work proximally Cone-shaped epiphyses may be normal but are frequently
• Integrity and morphology of osseous structures – are syndromic as well.
cortices intact? Are there structural changes?
• Density of osseous structures – evaluate 2nd MT ACCESSORY OSSICLES AND SESAMOIDS
o Rule of 1/3s – in individuals <40 y/o, middle 1/3 Accessory ossicles and sesamoids are normal variants in
should be medullary, outer 2/3 should be cortical anatomy that occur in predictable locations. Accessory
o Rule of 1/2s – in individuals >40 y/o, half bone ossicles usually arise earlier, are larger, and irregular
should be cortical, half medullary compared to sesamoids (which are usually present in tendons.)
o 1/3 of bone is lost before apparent on X-ray Os Trigonum (2-8%) – occurs posterior to the lateral tubercle
o Cortical bone lost at a rate of 2%/year (>35 y/o) of the talus. When fused (~18 y/o) is called steida’s process.
o Medullary bone lost at a rate or 1% per year Os Tibiale Externum (3-12%) – secondary ossification center
4. Joints and Joint Spaces of the navicular tuberosity. The pre-hallux type is pyramidal
• Is there narrowing? Is it symmetrical/asymmetrical? in shape, while the accessory navicular is in the PT tendon.
• Absence of joint = coalition/joint fusion Os Intermetatarsium (1-10%) – occurs btw the 1st cuneiform
• Increased space is due to joint effusion and 1st and 2nd MT bases. May fuse.
5. Biomechanical Assessment Os Supranaviculare (1%) – located on the dorsal aspect of
• Measure of angular relationships for treatment goals the TN joint (may look like fx/osteophyte).
• Determines surgical procedure and post-op correction Os Calcaneus Secundarius (1%) – located at the anterior
process of the calcaneus – may be confused w/ rowe fx.
• Performed in angle and base of gait
Os Sustentaculi (<1%) – is located posterior to the
• Serial radiographs to check progression of healing
sustentaculum tali, often fused. Is assoc w/ STJ coalition.
Conclusion – use systematic approach to avoid missing
Os Supratalare is located on the dorsum of the talar head.
details. Review the entire film. Do not let your eyes be drawn
Os Vesalinum (<1%) – occurs at the proximal 5th MT base.
to the obvious. Cite only differentials which make sense and
Usually confused w/ apophysitis, or a stewart/jones fracture.
are defensible to you.
Accessory ossicle of the Hallux occurs usually at the base of
the distal phalanx at the medial or lateral aspect.
RADIOGRAPHIC VARIANTS
Os Subtibiale/Subfibulare (4%) - occur distal to the medial
Identification of variants requires knowledge of normal
and lateral malleoli. R/o trauma
anatomy. Often normal variants are mistaken for pathology.
Hallux IPJ Sesamoids (50%) – located at the IPJ of the
The simplest error in this regard is structural overlap due to
hallux, may remain cartilaginous. Assoc w/ plantar HPKs.
wrong projections ordered.
Lesser IPJ Sesamoids occur in the joint capsule and short
Benign fibrocortical defect – small area of discontinuity
flexor tendons of the lesser digits. Does not occur in the 3rd.
usually in long bones. Eventually fades away over time.
Os Peroneum (20%) - a sesamoid in the PL tendon near the
Nutrient foramen – may look like a fx, but discontinuity
cuboid. Often is not ossified.
“stops” halfway through the cortex.
Constant Sesamoids occur in the MPJ of the hallux and
Ward’s triangle – area of low trabecular density in the
ossify around 10-12 y/o. The tibial sesamoid is larger and is
anterior inferior portion of the calcaneus.
commonly bipartite. Partitions occur in cardinal body planes,
Epiphyseal plates can look similar to fractures, if the X-ray
and their combined size is greater than a single sesamoid.
passes through the plane of the plate. Closed epiphyseal plates
You must r/o fx when dxing a bipartite sesamoid.
lymphatic channels), inflammation, surgery, burns, infection,
or helminthic infestation. Inflammatory edema is associated
with systemic disorders, particularly arthridites and metabolic
disorders. D/Dx include A vitaminosis, RCPS, melorheotosis,
collagen vascular diseases, dilantin use, reynaud’s disease, and
thyroid dysfunction (thyroid acropachy – nodular myxedema.)
THE 5 STEP APPROACH TO PLAIN FILM ANALYSIS Traumatic edema will be localized to the site of trauma and
STEP 2: SOFT TISSUE ANALYSIS should be consistent with the history. Besides direct trauma,
The soft tissue analysis of plain film incorporates two main this type of edema is seen in stress fractures, infection, and in
features: surgical sites post-op. Of particular clinical significance is
1. Tissue Density edema in the posterior triangle (bordered by the posterior
2. Tissue Contour flexor hellucis longus, the anterior tendo Achilles, and the
First you want to determine what kind of view you posterior-superior calcaneus.) Edema or hematoma from
are looking at. The most common is of course the DP view of injury to the tendo Achilles will cause this triangle to
the foot, which may be taken WB or NWB. To tell the disappear, which is a pathologic finding. Another pathologic
difference, look at the joint spaces and see if they are clearly finding is the teardrop sign of the ankle, which is visible as
visible. If the MPJs are clearly visible the view is probably increased tissue density surrounding the ankle joint. This is
WB, which is taken at 15 degrees as opposed to NWB, which caused by extravasation of fluid to the ankle joint subsequent
is shot at 0. You may also look to see the ankle position (90 in to trauma. Ligamentous sprain is visible as diffuse edema
WB.) The view is important because WB views will show the surrounding the area of the affected ligament.
soft tissue somewhat distorted due to pressure. On the DP this Burns and frostbite cause areas of devitalized tissue
presents as extra thickness of the S.T. on the lateral column, to show up on radiographs as areas of increased density.
and on the lateral it presents as flatness on the plantar surface. Small localized areas of increased density are visible in the
The first point, density evaluation, requires case of focal gigantism associated with acromegaly. Bony
knowledge of different tissue densities to be of any use. The growth is also affected in this disorder and diagnosis should be
least dense material is of course gas or air, which is easy to make if it has not been made already.
pathologic. The next higher density is fat, typically seen in Heel pad thickening is a finding associated with a
kager’s/bohler’s/toygar’s triangle, as well as on the plantar number of pathologies, and as such measuring the heel pad is
surface. Next dense from fat is water, which is the primary recommended in routine analysis of NWB views. Possible
composition of the soft tissues including muscle and tendon. D/Dx include acromegaly, dilantin use, plantar edema, plantar
Muscular compartments of the foot as well as the tendo infection, obesity, trauma, myxedema, and thyroid acropachy.
Achilles will be of this density. Next dense is atrophied or The accepted limit for normal heel pad thickness is 25mm.
devitalized tissue, which is somewhat dehydrated. Like gas, Soft tissue emphysema is a decrease in radiodensity
tissues of this density are pathologic and should be explained due to the presence of air or gas in the tissues. Always
fully. Next dense is calcific density, which is the density of pathological and often the sign of an emergent situation, S.T.
calcium deposits in the tissues. This is most often pathologic. emphysema is a worthy finding. Gas is almost always
Next dense is ossific density, which is the density of bone. produced by bacteria, especially clostridium perfringens,
Bones are only slightly denser than calcified structures – they which may produce superficial emphysema (radiographic
are normally differentiated by their internal architecture. emphysema) or deep emphysema (deep intramuscular
Finally, the densest structures radiologically are metallic or emphysema). The only non-emergent emphysema finding is
glass structures, which are almost completely radioopaque. of a gas abscess which is a localized ulceration containing gas
Objects of this density are almost always foreign bodies, which does not spread to other areas. Gas abscesses are
although in many cases they simply represent fixation or typically found only in the distal part of the digits. It is
implants from prior surgery. important to distinguish emphysema from the decreased
Pathologic changes in soft tissue density may be radiographic density caused by a sinus tract leading to an
either an increase or decrease in character. Edema causes ulcer. A sinus tract will always track back to the surface of
generally an increased radiodensity which matches that of the skin, whereas emphysema is contained within the soft
muscle or tendon, usually in areas where fat density is normal. tissues.
This is important in the posterior triangle as well as on the Some disorders which affect only soft tissue contour
plantar surface of the foot as extravasation of blood or fluid are worthy of note as well. Ainhum’s disease is a thickening
will show up as an obliteration of normal fat anatomy on film. of fascial bands about the base of the proximal phalanges of
Edema also causes an increase in soft tissue contour, which is the digits, causing a constriction of the underlying tissues that
the second point to evaluate in soft tissue analysis. Edema is is visible on x-ray. This constriction eventually leads to
normally classified by its cause. Obstructive edema is autoamputation, and although it is easy to diagnose from X-
caused by obstruction of the lymphatic channels draining the ray, there is no known treatment for this disorder. The
affected area. This may be normal for the individual with sullivan’s sign sometimes seen on radiograph is a splaying of
congenital lymph blockage, as in lymphedema praecox, but digits which are abducted away from each other at the level of
with insidious onset of obstructive edema a previously the MPJ. This is caused usually by a space-occupying lesion
undiagnosed pathology may be at work. D/Dx include tumor, like a neuroma or interstitial fibrosis following surgery.
trauma, radiation (repeated exposure to radiation damages Sometimes, however, the etiology is biomechanical, due to the
unequal pull of flexor tendons that can result from DM motor particularly CREST syndrome. Calcinosis circumscripta
neuropathy, or simply from closely positioned metatarsal associated with frank scleroderma is termed thiberge-
heads. weissenback syndrome. These distal calcifications
Vascular calcification is seen in varicose veins, predispose the digit to ulceration. Tumoral calcinosis is not
monckeberg’s sclerosis, ASO, thrombophlebitits, and DVT. associated with cancer as the name implies, but rather is a
They appear on X-ray as thin parallel or curvilinear lines. descriptive term referring to the presentation of the
Phleboliths are small round or oval calcifications that appear calcification. Seen in renally compromised patients (usually
within superficial veins. DM type II), these calcifications begin near joints as small
Dracunculosis is a rare finding that is seen in nodules that eventually grow in size to possibly impinge
patients who have had a history of filiaria medinasis infection. neurovascular structures, limit ROM, and cause ulceration.
These helminthes, which burrow into the dermis of the foot, The typical presentation is near the 5th MPJ. This type of
remain after death as bits of foreign tissue that sometimes growth is benign.
calcify. The dracunculosis is the radiographic finding of these The last form of calcinosis is calcified bursa
calcified helminthes, which present as curvilinear lines of secondary to irritation (usually posterior heel, medial 1st MPJ,
radiodensity that do not follow any vascular anatomy. lateral 5th MPJ.)
Hemosiderin deposits may calcify in tissues Soft tissue ossification is distinguished from soft
following the trauma or disease state that caused them. This tissue calcification by the degree of organization within the
type of increased radiodensity is typically seen in periarticular ossified segment, typically with trabecular patterns.
tissue, arthritic joints, or the subcutaneous area of the ankle. Additionally, ossifications that occur near bone
General calcification of the soft tissues can be characteristically do not cause periosteal reaction.
distinguished from ossification by the degree of organization Progressive myositis ossificans (aka fibrodysplasia ossificans
seen in the radiodense area. Calcification is non-homogenous progressive) is a rare childhood disorder in which there is
and disorganized, giving a “salt and pepper” appearance. extensive and unexplained muscular ossification which causes
Ossification, in contrast, is highly organized, often exhibiting extraarticular ankylosis and disuse osteoporosis.
a trabecular pattern, and may have a cortex. In general, soft Neurological myositis ossificans has a similar presentation
tissue calcification is much more common that ossification. but occurs in conjunction with a neurogenic lesion (stroke,
Calcification etiologies are organized into three broad spina bifida, encephalitis, syphilis, brain trauma,
categories: metastatic, dystrophic, and calcinosis. myelomeningocoele, spinal cord hemisection, tuberculosis,
Metastatic calcification is systemic calcification of a and poliomyelitis). The calcifications occur at levels below
metabolic derivative, such as hyperparathyroidism, the neurogenic lesion. Finally, myositis post-traumatica
hypervitaminosis D, renal osteodystrophy, sarcoidosis, or (circumscripta) is an area of localized ossification secondary
cancer. The calcifications are caused by an imbalance in to acute trauma. Ossification begins soon after the traumatic
calcium or phosphorus metabolism. The calcifications may incident, but cortical formation is only seen 6-8 weeks after
appear vascular, but are typically more numerous and less the incident. Ectopic bone within the muscle may also be seen
organized than vascular calcifications. Advanced parallel to the muscle fibers. On X-ray the mass appears
hyperparathyroidism causes chunky calcification, in which similar to an egg, with well-defined borders. Histologically it
separate calcifications conglomerate into calcified patches that contains the same sections as an epiphyseal plate.
are prone to ulceration. Foreign body analysis in soft tissue is vital to
Dystrophic calcification is calcification of localizing the foreign body for surgical removal. Analysis of
devitalized tissues without metabolic imbalance. the film typically calls for the ”tunnel technique” in which a
Theoretically calcium precipitates in devitalized tissue, giving piece of paper is rolled up into a tube, which is used to analyze
rise to this type of presentation. Examples of possible small segments of the film at a time. Often needles are placed
etiologies include ehlers-danlos syndrome, pseudoxanthoma in three planes near the affected area so that when films are
elasticum, fibromatosis, tumor, cyst, hematoma, focal taken the exact location of the object can be triangulated. For
necrosis, trauma of any kind, and vascular disease. One some foreign bodies, particularly wood, advanced imaging
peculiar finding is that of hemarthrosis, which is calcification modalities (ultrasound, CT, MRI, xerogram) should be
of blood exudate in a joint space. These calcifications may employed.
aggregate as “pebbles” within the capsule, visible on x-ray,
that can limit ROM of the affected joint (common in the
ankle.)
Calcinosis is the term applied to califications that are
idiopathic in nature. There are four syndromes associated
mainly with calcinosis. Calcinosis interstitialis universalis
presents as a calcification of subcutaneous structures,
progressing deeper with time. It may affect any soft tissue,
but calcifications are generally laid down in linear
aggregations parallel to the long axis of the limb. This
condition is very rare and is diagnosed in youth. Calcinosis
circumscripta is a distal subcutaneous calcification that is
normally associated with collagen-vascular diseases,
THE 5 STEP APPROACH TO PLAIN FILM ANALYSIS progressive onset of pain, stiffness, swelling, and atrophy
STEP 3: BONE EVALUATION distal to the site of injury over a 3-6 month period. The cause
is hyperactivity of the sympathetic nervous system. In type I
OSTEOLYTIC DISORDERS RCPS no specific nerve damage can be identified. In type II
Osteoporosis is an absolute reduction in bone mass RCPS direct nerve injury has occurred. In 70% of pts, the
and density. Osteopenia is a generalized term to denote a disease can spread up the extremity, sometimes even to the
relative loss in bone density regardless of any etiology. opposite limb. RCPS can also be staged.
Osteoporosis is the most common skeletal pathology, common Stage I describes localized edema, muscle pain, joint stiffness,
in smaller older individuals who are less active. Females are and vasospasm with duration of about 3mos. Additionally,
affected more commonly then males overall. (Age-related, or trigger points form where touch causes pain out of character to
“senile” osteoporosis affects men and women equally.) the stimulus (causalgia.) Osteopenia is noted on x-ray distal to
Environmental factors that contribute to osteoporosis include the lesion.
smoking, alcohol intake, estrogen deficiency, and the use of Stage 2 includes increased pain and area of involvement,
certain medications. The most common presentation of brawny edema, hair loss, brittle nails, and spotty osteopenia.
osteoporosis is vertebral fracture, followed by hip fracture. (Osteopenia w/ areas of normal bone.)
Bone loss in osteoporosis is mostly cortical (80%), Stage 3 trophic damage becomes irreversible at this stage, w/
but 33-50% of bone must be lost before the disease is evident intractable pain, muscle atrophy, joint weakness, flexor tendon
on x-ray. Peak bone mass is realized at age 35, with contracture (producing deformity) and deossification.
involutional bone loss due to age beginning at age 35-40. Tx for RCPS includes pain tx (NSAIDs, opiates), nerve
Cortical bone is lost at a rate of about 1% per year after this blocks, physical therapy, and sx sympathectomy.
point. Bone has a gray appearance on x-ray w/ the primary Osteomalacia is a lack of calcium in osteoid matrix
trabeculae accentuated and the secondary trabeculae reduced. laid down by osteoblasts, resulting in improper bone
Primary osteoporosis is that which has no underlying mineralization. Symptoms include muscle weakness, bone
disorder. Secondary osteoporosis is caused secondary to a pain on palpation, and bone deformity. X-ray shows osteoid
pathology like steroid tx, myeloma, metastasis, gastric sx, seams, lines of unmineralized bone that appear as pseudofxs
anticonvulsants, hyperparathyroidism, heparin, DM, (aka increment fx, looser line, umbua zonen, milkman
alcoholism, disuse, hypogonadism, amenorrhea, and anorexia. syndrome). The cortex is thin and blurs into the medullary
Generalized osteoporosis affects all bones, particularly of the bone (endosteal blurring.)
axial skeleton, and may be post-menopausal or senile. Post- Rickets is osteomalacia seen in children, usually due
menopausal osteoporosis affects females aged 51-75, and is to vitamin D deficiency (but also due to renal dz.) It typically
caused by the increased osteoclastic action present in states of affects children 6-12 months old, and will be accompanied by
estrogen deficiency. Mainly only medullary bone is lost, with muscle tetany, irritability, and weakness. Physical
coarsened trabeculae. It is particularly associated with development is impaired, bone growth is impaired and
vertebral, hip, and colle’s fx (wrist.) Lab values for this type deforms. Cartilage in the epiphyseal plates hypertrophies and
of osteoporosis are normal except for a decrease in PTH. is surrounded by edema. CXR shows a rachitic rosary about
Senile osteoporosis has about an equal distribution btw males the costo-chondral junctions. Proximal calcification of the
and females over 70 y/o. There is a proportionate loss of metaphyses is absent, and margins are frayed and cupped.
cortical and trabecular bone due to generalized loss of Pseudofractures are also seen in rickets as well as
osteoblast activity. Lab values show increased PTH. The tibial/femoral bowing.
most common presenting symptom is extra-capsular hip fx. Scurvy is a disease caused by long-term deficiency
Regional osteoporosis occurs only in a particular segment, of vit. C. It takes 4 months to be apparent on X-ray, and
usually due to disuse atrophy. Localized osteoporosis affects typically affects infants aged 4-8months. Scurvy is
one part of a bone, and is due to tumor, arthritis, or infection. characterized by improper collagen formation, leading to
Diagnosis of osteoporosis is preferred with a device spontaneous capillary hemorrhage, osteopenia, and poor
called a DEXA scanner, or dual-energy x-ray absorbiometry. cartilage. Radiographically, there is a dense zone of
This measures the absolute density of bone via penetrance of provisional calcification (line of frankel), ring epiphyses
x-rays. CT can also be used to diagnose this disease. (wimberger’s sign), irregular metaphyseal margins (corner
Osteoporosis is graded using the calcaneal index: sign), metaphyseal protuberance (pelken’s spurs), scorbutic
Grade V (normal) – all trabeculae are uniformly present zones (trummerfield zones), and sub-periosteal hemorrhage.
Grade IV (normal) – posterior compressive trabeculae are Hyperparathyroidism is ostepenia due to
divided into two separate columns overactivity of the parathyroid gland. Primary
Grade III (borderline) – posterior tensile trabeculae are lost hyperparathyroidism is caused by parathyroid
Grade II (osteoporotic) – anterior tensile trabeculae lost adenoma/carcinoma/hyperplasia, or an ectopic PTH-secreting
Grade I (severe) – all tensile, most compression trabeculae tumor. Secondary hyperparathyroidism is caused by renal
are lost disease. Typical presentation is female, 30-50 y/o, weakness,
Regional Complex Pain Syndrome (RCPS) also lethargy, polydipsia/uria, hypercalcemia and kidney stones.
known as the “great mimicker,” manifests on x-ray as an acute Serum calcium and phosphate are elevated. The radiographic
painful regional osteopenia following minor trauma. Males hallmark is sub-periosteal bone resorbtion (1), followed by
and females are equally affected, and patients are typically accentuated trabeculae and reduced medullary bone (2), and
over age 50 and of type A personality. Presenting s/sx are formation of brown tumors. (3) Bone resorbtion typically
occurs first in the phalanges, and soft tissue ossifications may Hypervitaminosis D presents with nausea, anorexia, polyuria
also be seen. Brown tumors are “holes” in the bone that are and polydipsia. X-rays show extensive calcification of blood
filled with osteoclast-laden fibrous tissue, seen in both primary vessels, soft-tissue, kidney, and peri-articular spaces.
and secondary forms.
OSTEOBLASTIC DISORDERS
Melorheostosis of Leri – “flowing hyperostosis” that looks
like candlewax dripping down the bone. Etiology unknown. BONE AND FRACTURE HEALING
No gender predilection. Joint effusion w/ decreased ROM and Bone Characteristics: Bones support our frame as well as
contracture, one limb (monomelic) affected, muscle wasting, protect the vital organs, provide a source of calcium, a site of
lymphedema all are symptoms. X-ray shows wavy cortical insertion for tendons and ligaments, and provide the rigidity
thickening, narrowing of endosteal space. needed for movement against gravity. Bones comprise 1/10th
Osteopathia Striata – “Voorhoeve’s Disease” most of our body weight. They resist axial stresses very well but
commonly seen in the hip metaphysic. Seen as a linear band are weak against rotational forces. Tubular bones are built for
of bone density of unknown etiology, typically coexisting with axial support and locomotion. Flat bones are built for
sclerosing dysplasias. protection of vital organs.
Osteopetrosis – “Albers-Schoenberg Disease” also known as Bone Healing: Consists mainly of spontaneous regeneration.
“brittle bone disease.” Medulla is completely destroyed – all Tubular bone forms by endochondral ossification, flat bones
bone is cortical. Existing bone thus has no shock absorbing by intramembranous ossification. The periosteum surrounds
ability and is very brittle. It is a hereditary disorder in which and protects the bone, and is made up of an outer fibrous layer
the osteochondroid tissue of developing bone fails to mature and an inner “cambium” layer which contains osteocytes. It is
into medullary and lamellar bone. There are 2 types: this inner layer of the periosteum as well as the endosteum that
osteopetrosis tarda (benign, a dominant trait) and osteopetrosis are most closely associated with bone healing.
congenital (malignant, recessive trait w/ neck LAD and HSM.) FRACTURE TERMINOLOGY
Lack of bone remodeling results in long bones having an Avulsion Fx – pulloff fx consisting of a fragment of bone
Erlenmeyer flask deformity (flared metaphysis.) Variably, pulled off by a muscle, tendon, or ligament.
some persons present with a spotty osteopetrosis which Butterfly Fx – triangular shaped cortical fragment that is part
appears like a “bone within a bone” or as striations of normal of a communuted fracture.
bone with osteopetrotic bone. Chip/Corner Fx – small fragment of bone from a joint
Osteopoikilosis – “Osteopathia condensans disseminata” or margin. Distinguish from a joint mouse, which is a piece of
“spotty bone disease” is a hereditary disorder that presents as osteophytic bone that breaks off into a joint space.
multiple compact bone islands existing within normal bone. Closed Fx – does not penetrate the skin.
The disease is asymptomatic and is usually an incidental Comminuted Fx – consists of more than 2 pieces.
finding. Turnover is also very high, such that two radiographs Fleck Sign – avulsion fx by the lisfranc’s ligament of the base
taken in the same year may show completely different of the 2nd metatarsal.
presentations. 25% of patients have a coexisting Impaction Fx – telescoping of bone segments (also called
dermatological problem (dermatofibrosis, scleroderma, “bayoneting”) that results in shortening of a segment.
syndactyly), 20% have arthralgia. X-ray shows multiple Greenstick Fx – incomplete fx in which one cortex is affected
compact bony islands in long bones with uniform density. and the fracture line follows down the axial line of the bone.
Rarely, bone islands may have a radiolucent center. Torus Fx – trabeculae are buckled inwards with a cortical
Compact Bone Islands are normal variants seen in regular bulge around the fracture site
radiographs. They appear as small areas of radiodensity, Insufficiency Fx – aka pathological fx, caused by normal
usually in flat or tubular bones. What may appear normally as physiological stresses through diseased bone.
a compact bone island can rarely be a osteoid osteoma. Oblique Fx – fx is 45 degrees from the long axis of the bone
Osteoid Osteoma is a painful benign tumor that is typically Occult Fx – see also stress fx; subtle and difficult to identify
<1cm, appearing as a compact bone island with a radiolucent Open/Compound Fx – skin is penetrated
center. They are usually located near the cortex. Symptoms Simple Fx – two bone fragments are involved
include pain at night, relieved by ASA. Dx w/ bone scan. Spiral Fx – fx is circumferential and longitudinal in bone
Heavy Metal Poisoning can be by lead (plumbism), Stress Fx – due to repetitive stresses, no fx line is apparent but
phosphorus, or bismuth. Lead lines at the metaphyses are bony callus forms 10-14d on X-ray. Must rely on clinical Sx.
visible on radiographs one month after exposure. Lead Transverse Fx – fx at 90 degrees to the long axis of bone.
deposition may also be seen at the distal ends of the digits. FRACTURE REPAIR
Lead line density is determined by the amount of lead Displacement/Distraction – requires reduction
ingested, the width is the single-time duration of exposure, and Closed Reduction – realign fragments through manipulation
the number equals the number of exposures. What may of ST, stabilize w/ cast
appear as lead lines in a 4 y/o or younger is actually a normal Open Reduction – realign fragments surgically, using ORIF
finding. Lead foreign bodies do not cause plumbism (unless Inflammatory Phase – Periosteum, haversian system
located within a joint space.) damaged, bleeds to form hematoma. Osteocytes die from
Hypervitaminiosis A presents with dermatitis, pruritis, ischemia, creating necrotic tissue that induces the
alopecia and skin yellowing. X-rays show solid periosteal inflammatory response of vasodilation, edema, and
bone at the ulna and metacarpals. macrophage attraction.
Reparative Phase – governed by vascularity, the reparative This is a variant of non-union characterized by scar tissue and
phase involves the formation of a callus which is mainly fibrin false joint formation. Congenital pseudoarthrosis is present
from inflammation invaded w/ pluripotent cells for healing. A at birth and is assoc w/ neurofibromatosis. It typically occurs
collar callus forms around the bone to stabilize, dictated by in the distal tibia/fibula. Acquired pseudoarthrosis causes
the periosteum, while the internal callus is dictated by the pain, instability, and bowing, with sclerotic bone ends and
endosteum and dictates bridging and union between the two lucency between fragments. It is most common in the distal
segments. Compression encourages bone rather than fibrous tibia.
tissue formation. Necrotic tissue is simultaneously removed. Malunion
Clinical Union – the goal of reduction, clinical union is the Bone healing that occurs in non-reduced fx, in which bones
resolution of symptoms w/ palpable rigidity to the affected are in improper position. It can spontaneously correct in peds.
bone. Clinical union precedes radiographic union. THE 5 STEP APPROACH TO PLAIN FILM ANALYSIS
Remodeling Phase – begins while the reparative phase is STEP 4: EVALUATION OF JOINT SPACE
winding down, in the remodeling phase the callus is resorbed
and the trabecular bone remodeled along lines of stress, PEDAL COALITIONS
resulting in a bone that is stronger than when it broke. A coalition is a union of separate things into a single body or
Cancellous bone heals faster than cortical bone, and has a group; true coalitions are intra-articular fusions of bones,
smaller callus. Cortical healing is limited by vascularity and and bar/bridge coalitions are extra-articular fusions of bone.
the amount of movement permitted during the healing phase. A complete coalition is osseous and limits all motion. An
COMPLICATIONS OF FRACTURE REPAIR incomplete coalition is a union by fibrous or cartilaginous
Delayed Union – union is not achieved w/in the expected time tissue. A rudimentary coalition is an osseous projection
(6 wks.) This is due to inadequate reduction, soft-tissue which limits motion but does not produce a union of parts.
imposition in the fx, excess ST destruction, vascular A synchondrosis is a cartilaginous union, a synostosis is a
disruption, inadequate immobilization, fragment distraction, or bony union, and a syndesmosis is a fibous union of two parts.
inadequate fixation. AO principles dictate that bone healing Coalitions are caused by accessory ossicles (os trigonum, os
fails in tension and heals in compression. sustentaculi proprium), failure of mesenchyme to segment in
Non-Union – complete arrest of bony repair, replaced instead development, trauma, DJD (causing a fibrocartilaginous
by fibrous, cartilaginous, or synovial compensation for the fx fusion), inflammatory rheumatoid conditions, and fxs.
site. It is variably classified as being diagnosable 6-8 months Tarsal coalitions occur in 1-2% of the population and consist
post-fx. Others say non-union is dx if no radiographic healing of the talo-calcaneal coalition (most common), calcaneo-
changes are seen for 3 months. Classification of non-union is navicular, and talonavicular (least common.)
by Weber and Cech, and are divided into two major Ossification of coalitions in the tarsus occur at different times.
categories, hypertrophic and avascular (or hypotrophic.) The age of ossificiation determines when symptoms occur.
Hypertrophic Non-Union may be of the elephant’s foot type, Talonavicular coalitions 3-5 y/o
the horse’s hoof type, or the oligotrophic type. Elephant’s Calcaneonavicular coalitions 8-12 y/o
foot type is usually due to early WB and inadequate Talocalcaneal coalitions 12-16 y/o
stabilization. Horse’s hoof is caused by inadequate internal S/Sx of coalitions include stiffness, decreased ROM, pain,
fixation that has allowed a small amount of movement. peroneal spasm, anterior/posterior tibial spasm, local POP,
Oliogotrophic non-union has poor callus, and is usually due to pronation, cavus deformity, and fixed RF valgus.
inadequate alignment of fx ends. Peroneal spastic flatfoot is characterized by restricted STJ
Avascular Non-Union – consists of four types; torsion motion, RF valgus (in OKC), FF abduction, bowstrung
wedge, comminuted, defect, and atrophic. Torsion wedge is peroneal tendon, and extensor digitorum longus spasm.
incomplete healing of a fx in which there are three or more D/Dx of coalitions include RA, JRA, trauma, neoplasm,
pieces, in which some pieces fuse together but others do not acromegaly, CCPV, casting (causing peroneal contracture),
because of inadequate vascularity. It is sometimes seen with extra-articular arthrodesis, overcorrected flatfoot,
plate fixation, usually in the tibia. Comminuted non-union is osteochondral dystrophy, infection, and N/M disease.
the presence of one or more fragments that become necrotic, Plain film analysis of coalitions is cheap and able to identify
the callus fails, and any involved fixation may break from most symptomatic coalitions.
stress. Defect non-union is the loss of a devitalized fragment IPJ coalition can be viewed on AP projection.
of cortical bone that leaves a dead space within the bone Synphalangism is a congenital fusion of the IPJs.
which cannot be bridged. Atrophic non-union is a long-term Intermetatarsal coalition may be viewed on AP and oblique
result of insufficient vascularity that causes fx ends to become views. Diaphyseal fusions are typically the result of trauma.
osteopenic and atrophic. Metatarsal cuneiform coalition may be viewed with AP,
Radiographic Findings of Nonunion med oblique (medial column), or lat oblique (lateral column.)
Sclerosis of the fracture ends, failure of changes of healing, Navicular cuneiform coalition may be viewed with the AP
progressive bowing, increased bone atrophy above and below and medial oblique projections.
fx site, excessive callus formation, and absence of remodeling. Navicular cuboid coalition may be viwed w/ AP or oblique.
Bone scans show up “hot” in non-union; increased uptake at Calcaneocuboid coalition may be viewed on lateral or lateral
the fracture ends may be indicative of pseudoarthrosis oblique views (typically it is osseous and easy to pick up.)
formation. Talonavicular coalition can be viewed on AP and medial
Pseudoarthrosis oblique. AP shows a “putter sign” of the fusion.
Calcaneonavicular coalition can be viewed on lateral and Conclusion – correlate coalitions w/ pts age. Be suspicious of
lateral obliqe, and shows an “anteater sign” on lateral. These changes in plain films. Consider advanced imaging to
fusions are more commonly cartilaginous or fibrous in nature. investigate further. (Special views, CT, MRI.)
STJ coalition is the most common tarsal fusion, and usually is
the middle facet (the posterior facet is least common.) Fusions
of the sustentaculum tali or posterior process are also possible
(though not common.) Anterior facet coalitions can be seen
on lateral oblique isherwood projections but are most
commonly dx though lateral CT. Middle and posterior
facets are viewed on harris and beath axial projections (30, 35,
40, and 45°) or broden views. The posterior and middle facets
should be parallel to one another, and the posterior facet
should be about 2/3 the width of the calcaneus. Posterior
fusion may be secondary to trauma involving the os trigonum.
(Achilles pain, end ROM plantarflexion, deep ankle pain.)
Radiographic Features of STJ coalition (lateral projection) ARTHRIDITIES
Talar beaking Inflammatory arthridities are RA, erosive arthritis, psoriatic
Posteior TC joint space narrowing arthritis, reiter’s disease, ankylosing spondylitis, and
Rounding of lateral process of the talus enteropathic arthritis.
Obscured middle facet Non-Inflammatory arthritis is degenerative joint disease.
“Halo” effect (Wait a minute – doesn’t arthritis mean joint inflammation?)
Do not confuse talar beaking w/ dorsal hyperostosis, which is Metabolic arthritis is gout and CPPD disease.
caused by pathomechanics. Talar beaking occurs at the Neuropathic arthritis is neuropathic joint disease.
distalmost aspect of the talar neck, near the TN joint. It is Miscellaneous arthridities are DISH, PHO, and pigmented
caused by stress on the talonavicular ligament and nodular synovitis.
impingement of the navicular on DF. The halo sign is a subtle
finding, which is a ring of trabeculae that form around the PSORIATIC ARTHRITIS
coalition secondary to altered compressive forces. Seen in 7-15% of pts w/ psoriasis (80% if pt has nail changes.)
Computerized Tomography is the modality of choice for Typically affects 20-50 y/o, M:F equal. HLA-B27 frequently
evaluation of coalitions (b/c of cortical definition.) It allows positive, particularly w/ SI involvement. Distribution is
planar analysis which may be used to determine the extent of asymmetrical and is most common in the DIPJs. Often
coalition and change to surrounding osseous structures. confused w/ RA (for some reason.)
Nuclear Imaging is not very useful for the diagnosis of Classic radiographic features
coalition. Increased uptake is seen adjacent to areas fusing, Joint margin erosions working in to the center (similar to RA)
and decreased uptake in already fused areas. – eventually leads to “pencil in cup” appearance
MRI allows planar and oblique analysis to more fully explore Increased synovial fluid w/ increased intracapsular pressure
the nature of a coalition. T2 weighted images are particularly Apparent wide joint space (due to erosions)
useful to note presence or absence of synovial fluid in an area. Acroosteolysis – resorbtion of the distal phalangeal tuft
Syndromes associated with coalitions may aid in the Sausage digits
diagnosis of a bony union, and vice versa. Common coalitions Abundant periosteal activity (resulting in “whiskering”)
typically are axial, i.e. distal to proximal fusions, whereas Ivory Phalanx – due to abundant endosteal activity
syndromic coalitions tend to be medial to lateral in position. Osteopenia secondary to reduced activity
Apert’s Syndrome results in massive tarsal synostoses, as Main-en-lorngette (telescoping) – long standing disease
well as cranial synostoses and midfacial hypoplasia (“mouse” process resulting in collapse of diaphyses on themselves.
facies). The distal phalanx of the hallux and pollux are broad. Posterior/Inferior calcaneus hyperostosis (not heel spurs.)
Feet tend to have valgus attitude. Footwear is a problem
Nievergelt-Perlman Syndrome typically has synphalangism REITER’S DISEASE
and carpal fusion, moreso in the hands than feet. This Classic clinical triad – conjunctivitis, urethritis, arthritis.
syndrome may also present with an atypical clubfoot and Also 4th – derm lesions (keratoderma blenorrhagicum)
tarsal fusions, which causes a ball-and-socket type of ankle Usually preceded by diarrhea.
joint. Other findings include congenital dislocation of the Typically males, 18-40 y/o. HLA-B27 positive almost always.
radial head and conduction deafness due to ear ossicle fusion. Asymmetric distribution, primarily lower extremity.
Phocomelia is defective development of the arms or legs, Attributed to chlamydial infection (endemic) or dysentery
resulting in “flipper” extremities. Hemimelia is (epidemic) which are the two main types of Reiter’s.
underdevelopment of a specific extremity, usually a below- Epidemic form may affect women and children as well.
knee deformity. These conditions cause unusual fusions. Arthritis long-term symptom; conjunctivitis/urethritis resolves
Tx of coalition includes immobilization and supportive in weeks. Arthritic changes may also become chronic.
therapy, orthotics, NSAIDs for pain, and possible surgical Acute Stage - weight-bearing joints are more commonly
intervention (resection or arthrodesis.) affected. In the acute stage there is peri-articular ST edema,
deossification of opposing articular structures, Achilles
inflammation/widening, and periosteal reaction (metatarsal crystals are weakly positive birefringent and simulate gout
metaphyses, phalangeal diaphyses, inferior surface of the attacks when precipitated in joint spaces. The typical pt is
calcaneus.) Retrocalcaneal bursitis is also common. over 30 y/o (usually ~60) and there is no gender predilection.
Infracalcaneal exostoses (poorly defined or “fluffy”) form The problem is associated with hyperparathyroidism,
bilaterally and cause heel pain. pseudogout, acromegaly, DJD, and DM. The typically
Chronic stage – periarticular deossification, small marginal affected joint is the knee, where the menisci as well as the
erosions, symmetrical loss of joint space, ankylosis. articular cartilage may calcify as a result. It also can be seen
Specific findings – heel spurs in young men, Achilles edema, in the wrist, hand, ankle, symphysis pubis, and elbow.
small joint space periosteal reactions, primarily foot s/sx. True chondrocalcification is sometimes seen after trauma or
with hyperparathyroidism.
ANKYLOSING SPONDYLITIS
Affects pelvic and spinal joints, ligaments, and tendons. DEGENERATIVE JOINT DISEASE
Affects 15-25 y/o, usually males. Almost always HLA-B27 Also called osteoarthritis, DJD is a local, mechanically
positive. Characterized by lower back pain due to arthritis at caused degeneration of joint function that occurs in
the sacroiliac joint, bony proliferation at enthesiopathies, conjunction with age and trauma, resulting in degradation of
spinal osteophyte formation, and lateral syndesmophytes. the articular cartilage. Primary DJD is the most common form
and is due to aging. Secondary DJD is early onset due to
ENTEROPATHIC ARTHROPATHY trauma or injury to the joint. Joint space narrowing is
Joint changes secondary to GI disease. Seen in young adults, asymmetric in DJD, the subchondral bone exhibits
w/ malaise, anorexia, wt. loss and arthritis. Typically eburnation (pathologic sclerosis), the articular surface
sacroiliac, bilateral periostitis. X-ray findings are nonspecific. becomes broad and flat, and osteophytes form at joint
margins, often breaking off to become joint mice within the
GOUTY ARTHRITIS articular space. Osteophytes typically bend towards the joint
Gout is the result of an inborn error of purine metabolism. It space, which causes locking of the joint in areas where there
typically affects men age 40-50 y/o, and is characterized by are osteophytes on either side of the articulation. Pseudocysts
elevated serum uric acid and negatively birefringent crystals in may also form in the subchondral bone in areas where the
the synovial fluid. The most commonly affected joint is the 1st cartilage is completely worn away. It is important to note that
MPJ due to relative avascularity and lower temperature. the disgnosis of hallux rigidus and limitus, which are seen in
Women only get gout after menopause. Characteristic of gout patients with DJD of the 1st MPJ, cannot be diagnosed
is extreme paroxysmal pain in the middle of the night. radiographically and are purely clinical diagnoses.
Primary gout is hyperuricemia due to over-production of uric Additionally, the hands of patients with DJD clinically may
acid or under-excretion of it by the kidneys. Secondary gout present with enlarged IPJs which are called bouchard’s nodes
is due to medications (particularly HCTZ) and (PIPJ) or heberden’s nodes (DIPJ) that are simply the result
hyperparathyroidism. Urate deposits in relatively avascular of extensive osteophytosis in long-standing disease.
tissues like cartilage, synovial tissue, ligaments, bursae, and Erosive osteoarthritis is a variant of DJD which
subQ tissue. Hyperuricemia may never become symptomatic, affects primarily females aged 40-50 y/o. Clinically an
and in fact may be reduced during an acute phase attack. A 24 increased ESR may be seen because there is an inflammatory
hour urine collection is needed to diagnose accurately. component to the erosive component. The presentation is
X-ray findings of gouty arthritis only occur after several symmetrical and bilateral, and most commonly affects the IPJs
attacks. Thus radiographs in an early attack function mainly of the hands. Radiographically erosive osteoarthritis appears
to r/o other pathologies (septic arthritis.) There is peri- like DJD except that the erosions are symmetric and take on
articular swelling of the joint margins, erosions of the joint the characteristic gull wing appearance that is specific to EO.
margins, punched-out lesions resembling RA, and complete
lysis of bone in the acute stages. Large tophi may form at RHEUMATOID ARTHRITIS
joint margins and produce “rat bite” erosions peri-articularly RA is an inflammatory seropositive joint disease affecting
that leave overhanging shelves of bone called martel’s sign. primarily females between the ages of 20-40 y/o, but affect
This is not seen for 5-10 years after the attack and merely men and women equally beyond the age of 40. 70% of pts w/
signifies that a pt has had gout in the past. Tophus formation RA are seropositive for rheumatoid factor. The small joints of
can occur in the absence of an acute gout attack, and the hands and feet, and articulations of the spine are typically
subsequent erosions can cause subQ calcification and affected. Early presentation in the foot can easily be mistaken
pathological fxs. The joint space is preserved in gouty for gout – most commonly small, C-shaped erosions appear at
arthritis. DJD may accompany the process, and the joint may the base of the proximal phalanges. Radiographic changes are
be somewhat narrowed due to subchondral cyst formation, but seen in 65% of pts who have had some disease process for at
is otherwise unaffected. Metastatic tumors of bone are an least 3 months (85% for >6 mos.) However, no clinical s/sx
important d/dx to r/o when considering asymptomatic gout. may appear until the disease has been present for 3 years.
Early radiographic findings in RA include peri-
CHONDROCALCINOSIS articular edema that is fusiform in shape around the joint,
Pseudogout, or calcium pyrophosphate dihydranate disease, is typically in IPJs and MPJs. Inflammation of the synovium
a symptom similar in presentation to gout that causes the creates pannus which results in bone deossification peri-
radiographic finding known as chondrocalcinosis. CPPD articularly, which progressively extends proximally over time.
Secondary trabeculae are resorbed while primary trabeculae DISH is a disorder of ligamentous ossification and
are enhanced. Fibular deviation of the digits occurs calcification that affects the spinal and extra-spinal
secondary to bony deossification that destroys the medial articulations. Typically it affects males over 50 y/o. It is
collateral ligaments of the MPJs (except on the 5th MPJ.) associatied (20%) w/ DM. 40% are HLA-B8 positive. All
Early on the joint space becomes noticeably widened as parts of the spine, particularly the SI joint, are affected.
effusion and pannus formation characterize the disease Radiographically flowing hyperostosis is seen over 4
process, and often this is the only presenting sign in consecutive vertebral bodies, usually in the area of the anterior
undiagnosed RA. Joint space changes are most easily seen in longitudinal ligament. In any case, the disc is preserved,
the ankle joint radiographically. Near the joint periosteum differentiating this process from syndesmoses seen in
may become irritated and periosteal elevation in the ankylosing spondylitis. Extra-spinal presentations may be at
epiphyseal/metaphyseal region is seen. This creates any enthesis, producing whiskering at the bony attachment and
lamellations which enlarge the bone near the articulation. ossification of the tendon or ligament while preserving the
Entheses and bursae near the joint may also become inflamed joint space.
and enlarge. Localized bony erosions occur near the joint due
to herniation of the pannus through capsule, producing an area PULMONARY HYPERTROPHIC OSTEOARTHROPATHY (PHO)
of radiolucency with poorly defined borders. Erosions seen en PHO is a disease caused by bronchogenic carcinoma that is
face may appear completely within the bone, while laterally described by the triad of digital clubbing, symmetric
they may have a dot-dash appearance. Medial-plantar arthritis, and linear periostitis. It typically affects males 40-
erosions of metatarsal heads 1-4 are most common. Later 60 y/o. The arthritis in this process, like many seronegative
symmetrical joint narrowing is seen, particularly in the IP and diseases, is nondescript in its clinical and lab findings. The
TN joints (but never the CC joint.) arthritis is specific to the extremities only. Periostitis in long
RA-induced subluxations of MPJs and IPJs following bones occurs in the metaphseal/diaphyseal region, appearing
ligamentous involvement cause the characteristic swan-neck as a “double strip sign” on bone scans (dual cortices.)
(like mallet-toe) and boutonneire (like HDS) deformities. PIGMENTED VILLONODULAR SYNOVITIS
These deformities are also seen in SLE and post-rheumatic This disorder is a proliferative pathology of joints that does
fever (jacoud’s) arthritis. Other deformities seen in RA not produce arthritis, but rather a mass within a joint or tendon
include main-en lorgnette deformity (“opera-glass hand”) sheath which may affect the joints secondarily. Typically it
that is also seen in psoriatic arthritis. affects young males. Joint involvement is asymmetric.
Late radiographic findings in RA have obviously Radiographically there is a soft-tissue mass evident with
no diagnostic value, but are used to track the progression of localized bony erosions that have well-defined sclerotic
the disease. Characteristic features include subluxation and borders. Calcification is unusual.
dislocation of the joints, ulnar deviation of the digits,
destruction of joint space, and finally bony fusion and SYSTEMIC LUPUS ERYTHEMATOSUS
ankylosis of the joints altogether. Pathological fxs secondary SLE is a connective tissue disorder involving nearly every
to osteopenia may be seen. Overall, the MPJs are most organ system in the body. It is most common in females 20-
severely affected in RA, with the IPJs of the 1st and 5th digits a 40 y/o, and is characterized by elevated ESR and ANA titer
close second in frequency. levels. Radiographic features are most readily assessed in the
hands, and are characterized by reversible subluxation,
NEUROPATHIC JOINT DISEASE dislocation, and varied deformity. Calcification and atrophy
More commonly known as charcot foot, neuropathic joint can occur secondary to deformity. Most radiographic findings
disease is a destructive disease that occurs secondary to loss of are non-specific and there is no articular degeneration.
proprioception, seen in DM (5%) and syphilitics (20%.)
Clinically there is no protective sensation; instability and SCLERODERMA
crepitation are present in the joint, and neurologically the pt Scleroderma is a connective tissue disease that also involves
suffers from ataxia and loss of DTRs. Besides the more many organ systems and affects females predominantly (30-60
common pathology, neuropathic joint disease can occur y/o.) Radiographic changes in scleroderma are most readily
iatrogenically secondary to administration of indocin or appreciated in the CREST syndrome, in which soft tissue
steroids over long periods of time (notable because of the tx calcification and acro-osteolysis may be seen. (r/o psoriasis)
protocols for RA and gout recommend this.) The hypertrophic
form of the disease affects large, WB joints, while the
hypotrophic (atrophic) form affects NWB joints. Clinically
both undergo 3 stages; fragmentation, coalescence, and
reconstruction. Radiographically the hypotrophic form is
characterized by bone resorption, diabetic osteolysis, and a
“licked candy stick” appearance. The hypertrophic form is
characterized by the six D’s: joint distension, density
increase, debris production, dislocation, disorganization, and
destruction.

DIFFUSE IDIOPATHIC SKELETAL HYPEROSTOSIS (DISH)


line drawn from distal inferior aspect of calcaneus to
anteriormost weight-bearing point of calcaneus

AP ANGLES
Lesser Tarsal Angle = GTA + LTA
Metatarsus Angle = LTA + MTA
Used to evaluate for metatarsus adductus
Digitus Angle = DA + MTA of measured digit
Forefoot Angle = GTA + MTA
Used to evaluate metatarsus adductus in peds
Talo-Calcaneal Angle = collum tali + collum calcanei axes
Used to evaluate pronation/supination
Cuneo-Metatarsal Angle = MTA(1-3) + long axis of
proximal cuneiform
Used to evaluate hypermobility, lisfranc’s injury
Calcaneocuboid Angle = Collum calcanei axis + line CD of
LTA
Used to evaluate lateral column integrity
Talometatarsal Angle = column tali axis + 1st MTA
Called Simmons Angle on AP
Used to evaluate pronation/supination
Intermetatarsal (IM) Angle = 1st MTA + 2nd MTA
Used to evaluate bunion deformity
Hallux Abductus (HA) Angle = hallux axis + 1st MTA
Used to evaluate bunion deformity
THE 5 STEP APPROACH TO PLAIN FILM ANALYSIS Hallux Interphalangeus Angle = hallux axis + bisection of
STEP 5: BIOMECHANICAL EVALUATION distal phalanx of the hallux
The biomechanical evaluation of radiographs is used for pre- Metatarsal Break Angle = line between center of 1st and 2nd
op planning and post-op assessment of correction as well as metatarsal heads + line between center of 2nd and 5th
for the evaluation of orthopedic pathology. Biomechanical metatarsal heads.
readings are most sensitive to positioning errors, and so great Tibial Sesamoid Position
care must be taken to ensure proper positioning in patients 1 = Tibial sesamoid medially clear of 1st MTA
needing a biomechanical evaluation. 2 = Tibial sesamoid laterally abuts 1st MTA
AXES 3 = Tibial sesamoid laterally overlaps 1st MTA
Greater Tarsal Axis (GTA) 4 = Tibial sesamoid bisected by 1st MTA
Line drawn parallel to the distolateral border of the calcaneus, 5 = Tibial sesamoid medially overlaps 1st MTA
most easily identifiable on AP radiograph. 6 = Tibial sesamoid medially abuts 1st MTA
Lesser Tarsal Axis (LTA) 7 = Tibial sesamoid medially clear of 1st MTA
Point A is placed on the distomedial aspect of the medial Proximal Articular Set Angle (PASA) = line btw medial and
cuneiform. Point B is placed at the medial proximal aspect of lateral borders of metatarsal head articulation + line
the navicular articular surface. Point C is placed at the perpendicular to MTA.
proximal lateral aspect of the cuboid and point D at the distal Distal Articular Set Angle (DASA) = line btw medial and
lateral aspect of the cuboid. The midline between points A & lateral borders of proximal phalanx MPJ articulation + line
B and C & D are found and a line drawn between them. The perpendicular to digital axis.
line perpendicular to this is the lesser tarsal axis.
Metatarsal Axis (MTA) LATERAL ANGLES
Line drawn through the midpoint of the shaft of the 2nd Talar Declination Angle = collum tali axis + ground
metatarsal. (Or another metatarsal if specified.) Calcaneal Inclination Angle = calcaneal pitch axis + ground
Digital Axis (DA) Talocalcaneal Angle = collum tali + calcaneal pitch axes
Line drawn down the midpoint of the shaft of the digit in Talonavicular Angle = collum tali + bisection of navicular
question. Used to evaluate for metatarsal breaks
Collum Tali Axis 1st Metatarsal Declination Angle = 1st MTA + ground
Line drawn down the bisection of the talar neck. Talometatarsal Angle = column tali axis + 1st MTA
Collum Calcanei Axis Called Meary’s Angle on lateral projection
Line drawn down the bisection of the distal 1/3 of the Hallux Dorsiflexion Angle = 1st MTA + hallux axis
calcaneus. Cyma Line = position of the talonavicular joint with respect
Hallux Axis to the calcaneo-cuboid joint on lateral projection.
Line drawn down bisection of the shaft of the proximal
phalanx of the hallux. Radiographic Evaluation of Supination
Calcaneal Pitch Axis Talar declination angle decreases
Calcaneal inclination angle increases extent of the cloaca and its sinus tract to the outside of the
Talocalcaneal angle decreases body can be appreciated through the use of a sonogram,
Cyma line posteriorly displaced which is simply radiographic contrast media injected into the
Subtalar joint/sinus tarsi accentuated (bullet hole sinus tarsi) sinus tract that can then be visualized radiographically.
Marjolin’s ulcer is a malignant degeneration of soft tissue
Radiographic Evaluation of Pronation within the cloaca to a squamous cell carcinoma. This occurs
Talar declination angle increases secondary to acidic pus and exudate from the cloaca irritating
Calcaneal inclination angle decreases the tissues of the sinus tract. The infection must be very
Talocalcaneal angle increases chronic (up to 20-30 years) for a marjolin’s ulcer to form, and
Cyma line anteriorly deviated it is only seen in long-standing osteomyelitis. Nevertheless, it
Pseudo-sinus tarsi seen (posterior facet of STJ; real sinus should be kept in mind when treating a patient with history of
tarsi is obscured.) bone infection.
Brodie’s abscess is a localized suppurative infection of bone
Metatarsus Primus Elevatus (MPE) which is successfully walled off and is sterile. A brodie’s
Defined radiographically as an elevation of the 1st metatarsal abscess is usually an incidental finding and typically is seen in
with respect to the 2nd metatarsal. Implied in the development children after osteomyelitis has resolved. A similar finding is
of 1st MPJ OA, sub 2nd metatarsalgia. an abscess that still contains bacteria, which can serve as a
reservoir for reinfection in the future (however, brodie’s
Total Adductory Angle (TAA) abscesses are sterile only.)
Sum of the IM angle + the metatarsus adductus angle, used in Routes of infection in osteomyelitis include hematogenous
the evaluation of bunion deformities and in surgical planning. spread (most common in children), direct
A TAA >30° should warrant consideration of a proximal implantation/inoculation, direct extension, and post-op
osteotomy in surgical bunion correction. infection. The most common infecting organism in children is
H. Influenzae, although it is now much less common than it
Pre-Op Biomechanical Assessment once was. In adults staph aureus is by far the most common
IM angle HA TAA infecting organism for all types of osteo (except plantar
PASA DASA Hallux Interphalangeus puncture wounds.)
Metatarsus angle MPE Sesamoid position Hematogenous osteomyelitis, being primarily a disorder of
Joint status Bone density 1st MT protrusion children, is most commonly seen in the metaphysis of long
RADIOGRAPHIC INTERPRETATION OF INFECTION bones. Bone destruction begins internally, then weakens the
OSTEOMYELITIS cortex, causes periosteal reaction, and finally breaks through
Osteomyelitis is the infection of bone, most commonly by to soft tissue. Being confined within bone initially, the route
bacteria but also possibly by fungus, virus, or parasites. The of pus evacuation can be appreciated on X-ray. Initially pus
anatomy of vascular structures is most important when fills the medullary space for some distance before pushing
considering osteo, and the vascular anatomy of bone changes through the periosteum. The codman’s triangle is a
with age. In infants (0-1 y/o), blood vessels pass through the triangular space of pus bounded on one side by infected cortex
epiphyseal plate in tubular bones. Because of this, it is more and on another by elevated periosteum which attempts to lie
common to see infection in the epiphysis because vascular down new bone. Notation of this on X-ray is usually
channels here are more prominent than at a later age. In diagnostic for hematogenous osteomyleitis.
children (1-16 y/o), metaphyseal vessels end at the epiphyseal Direct implantation osteomyelitis typically occurs as a result
plate, creating loops and blind-ended sacs in the metaphyseal of puncture wounds, human or animal bites, or open fracture.
vasculature. The tortuosity of these vessels means that Staph aureus is the most common infecting organism in all
metaphyseal infection is much more common in children. The cases except those of plantar puncture wounds that pass
adult pattern begins around age 16. As the physis closes, the through a shoe, in which case the infecting organism is
metaphyseal vessels can begin to cross back into the epiphysis pseudomonas. E.Coli is sometimes implicated in direct osteo
again and so infection may be located anywhere in the bone. as well. In direct implantation there is no particular order to
Sequestrum is an infected area that is dead due to the infectious process, as the traumatic injury has already
cortical and medullary infarcts. Small sequestra can be disrupted the cortex.
eventually resorbed by osteoclasts, while large sequestra are Direct extension osteomyelitis is spread of infection to bone
walled off by the body and then expelled gradually through a from a nearby soft tissue infection or ulcer. Nail infections
sinus tract. Involucrum is periosteal new bone formation that and infected ulcers are the most common causative factor in
occurs in response to bone infection, and represents the body’s this type. The order of infection tends to follow the opposite
attempt to prevent the spread of bacteria. Both sequestrum order as hematogenous osteomyelitis.
and involucrum are most commonly seen in hematogenous Post-op infections may be due to staph aureus (again most
osteomyelitis. Cloaca is a defect in the involucrum that common) or pseudomonas. This type of osteo is most
allows for drainage products of the infection to be expelled commonly seen in the hip or knee following arthroplasty,
from the infected area. The cloaca commonly receives small especially when hardware is installed. Loosening of fixation
bits of sequestrum that may have been broken down in the with obvious radiolucency about the device indicates infection
disease process. Exudate from the cloaca with involucrum of this type and surgical removal is indicated.
will feel like sand between the fingers when handled. The
X-RAY FEATURES OF OSTEOMYELITIS Although classically infection and inflammation about a bone
Initially, there are no osseous findings present on X-ray – cause a bone scan to read positive in that area, an increase in
plain film findings lag clinical findings 7-10d. In vertebral vasculature to an area may increase uptake of the isotope, as
bones there may be up to a 21d lag. 30% of bone must be lost well as the articular surfaces of an inflamed joint. Thus,
before the changes can be appreciated on plain films. Initially findings which would otherwise be normal clinically may
the only evidence of infection will be soft tissue changes, show up as positive on bone scan and must be separated from
including deep ST swelling in hematogenous osteo adjacent to pathology. Distal bones seem to be more sensitive to Tc-99
the infected area. Fat planes will be elevated, displaced, or uptake, and can more easily show up hot even if only
obliterated altogether. Lines of lucency between muscles will sustaining minor trauma. Overall, Tc-99 exposes the patient
be obliterated. Infection tracks usually distal to proximal and to only a small radiation dose, and is renally excreted. For this
only rarely travels from medial to lateral (and vice versa.) reason, it is generally recommended that the patient be well-
Breakthrough to a joint space will produce swelling of the hydrated (this also aids in contrast), and care must be taken in
capsule, which in the ankle produces the classic teardrop sign individuals with renal disease, especially if they have DM.
which is also seen in hemarthrosis. Periosteal response to The half-life of Tc-99 is 6 hours. Normal areas of increased
infection occurs usually 7-10d after the initial process has uptake are the lacrimal glands, breasts, SI joint, kidneys,
begun. The pattern of reaction is lamellar and progresses to bladder, and the injection site of the isotope.
formation of an involucrum. Involucrum tends to be less Gallium-67 Citrate is another isotope that resembles the
dense than the established cortical bone, even infected. The ferric ion. It binds to iron-related plasma proteins as well as
most distal the infection is in the body, the most subtle the organelles of leukocytes. This makes Ga-67 useful for
periosteal response will be. Resolution of the infection will visualizing the peripheral tissues because it marks the areas
leave the involucrum to be remodeled and resorbed, however where leukocytes have localized. Ga-67 is also excreted
it may sometimes persist after the healing process is finished. renally, and has a long (78 hour) half-life.
Bony destruction appears “moth eaten” as there is a focal loss Podiatric indications for bone scan
of bone density in the medullary bone, with sequestrum seen Osteomyelitis
3-6 wks after the infectious process has begun. Kissing Trauma
lesions are opposed lesions seen across articular surfaces Inflammatory Arthritis
when the joint has been involved. Often these changes are Stress Fracture (if >7 hours)
subtle and may not be noticed unless serial radiographs are Tumors
taken and analyzed. In slow-growing, rarer cases like fungal Non-specific pain (to check for occult causes)
infections, the bone may be able to mount a more effective Combined use of bone scan isotopes can be used to
response against the causative agent, and sclerosis due to differentiate the infectious process. Soft tissue infection can
endosteal proliferation may be seen. This type of infection- be distinguished from osteomyelitis by decreased uptake in
caused sclerosis is called sclerosing osteitis of Garre. the third phase of the Tc-99 scan, and will be strongly positive
in Ga-67, whereas osteomyelitis shows up hot in all three Tc-
Radiographic Signs Overview 99 phases and is cold in Ga-67 scanning. When performed
Early Signs together, the Tc-99 test is done first because of the shorter
Latent Period (10d extremities, 21d spine) half-life. Gallium is typically used to clarify the issue of
Soft Tissue – fat plane elevation/displacement/obliteration, infection site, but is not as effective at determining the course
increased ST density of an infection. The practice of using a gallium scan to
Bone – motheaten cortical/medullary destruction, periosteal determine when the infectious process (and thus the treatment
new bone formation (involucrum, codman’s triangle) regimen) is finished has been thoroughly disproved.
Late Signs Indium-111 isotope is a rarer, more specialized bone scan
Soft Tissue – draining sinus/cloaca, debris, marjolin’s ulcer that tags WBCs in drawn blood with the isotope and re-injects
Bone – cortical destruction, involucrum, sequestrum, sclerosis them into the bloodstream. I-111 is useful in distinguishing
Joint – loss of joint space, ankylosis, kissing lesions. osteomyelitis from neurotrophic joint changes in the diabetic
individual, which is normally a very difficult clinical
Salmonella Osteomyelitis is a peculiar type of osteo that is diagnosis. A negative I-111 scan would tend to r/o osteo and
sometimes seen in patients with sickle cell anemia. If a sickle provide for a positive diagnosis of charcot, while the reverse is
cell crisis occurs in the intestinal vasculature, small infarcts also true.
can allow salmonella bacteria to travel from the gut to the
bloodstream where they can seed in bones across the body. COMPUTERIZED TOMOGRAPHY FOR INFECTION
The osteomyelitis associated with this bacterial seeding is CT allows planar analysis of anatomy to determine the extent
unique in that it predilects the diaphysis of long bones. of infection. Although sensitive for soft tissue infections, CT
tends to overestimate their involvement, and is much more
BONE SCANS FOR INFECTION reliable at determining bony involvement. Very subtle cortical
Technetium-99m Phosphate is the prototypical bone scan, or periosteal changes may be appreciated on a CT scan.
and is the most commonly used in clinical practice. It reacts
with calcium hydroxyapatite to form insoluble salts which MAGNETIC RESONANCE IMAGING FOR INFECTION
precipitate in areas of bony turnover. This makes the Tc-99 In contrast to CT, MRI is much better at evaluating soft tissue
scan ideal for infectious and/or traumatic processes of bone. infection versus bone infection. MRI cannot distinguish
between infection in a joint space and effusion in a joint, appearance that is almost always malignant. Codman’s
however, and this is a major limitation if joint involvement is triangle is similarly a sign of malignancy in tumors.
suspected. The infected area appears dull on a T1WI, and 5) Tumor matrix description can help determine the tissue
more typically has greater intensity on a T2WI. type involved. Osseous lesions appear cloud-like and
amorphous with usually a “fluffy” appearance. Cartilaginous
THOUGHT PROCESS BEHIND THE RADIOLOGY OF INFECTION lesions appear punctuate w/ C-shaped lesions and numerous
From plain films the distinction must be made as to the small calcifications in the center are typical. Fibroosseous
location of an infection. If soft tissue is suspected, MRI is the lesions have a ground-glass character with no discernible
best modality. If bone is suspected, CT is the best modality. If trabeculae. No visible matrix is a pertinent negative and may
the symptomatology is diffuse, the Tc-99 and Ga-67 bone be a giant cell tumor or cyst.
scans should be considered to determine involvement. CT imaging of tumors is used to evaluate tumor matrix more
effectively and determine cortical vs medullary lesions.
BONE TUMORS AND TUMOR-LIKE LESIONS MRI of tumors in used to asses soft tissue and marrow
Evaluation of plain films – plain film analysis offers the most involvement, but does not provide much for osseous tumors.
diagnostic information for the analysis of bone lesions. Osteoid Osteoma (OO) is a fairly common tumor seen in 10-
Biopsies of bone alone do not give the pathologist enough 30 y/o. It is characterized by a visible central nidus of less
information to make a definitive diagnosis; clinical and then 1cm that is seen in the tibia/fibula or spine (causing
radiographic evidence is needed to support what is seen under painful scoliosis.) The nidus may or may not have a central
the microscope. Some lesions also may become activated or area of calcification. Pain is felt at night and is described as a
react against an invasive biopsy technique. dull pain which responds to ASA. It may take 6-24 months to
Basic approach to solitary bone lesions make a Dx. If near a joint the OO can cause joint effusion and
1) History – age, sex, and symptoms. Age of the patient is pain that can look clinically like JRA. The main D/dx is stress
important because many tumors predilect younger or older fx. MRI is frequently misleading in dx. Tx is sx or
patients. Ewing’s sarcoma, chondrosarcoma, and unicameral radiofrequency ablation (90° C.)
bone cysts are almost exclusively tumors of children, while Osteoblastoma (OB) is a very large osteoid osteoma that is
metastatic tumors and multiple myeloma are only seen in older also very rare. It is seen in the same populations and in the
individuals. Osteosarcoma tends to affect younger patients, same locations but with milder symptoms (despite greater
chondrosarcoma the middle aged, and fibrosarcoma the size.) The nidus may be greater than 1.5cm and can have
elderly. The tumors paraosteal osteosarcoma, giant cell tumor, multiple internal calcifications. It causes less reactive
and aneurismal bone cyst are the only tumors seen more sclerosis than OO.
commonly in women than men. Pain is the most common Osteosarcoma (OS) has several variants. Paraosteal and
symptom in sarcomas. The character, onset, frequency, and medullary (low grade) have good prognoses. Pagetoid and
aggravating factors should be assessed. radiation-induced OS has a poor prognosis. Telangectatic OS
2) Location should be described in relation to the position in causes many complications, but often responds to chemotx.
long bones, cortical or medullary, and eccentric or centrally Classically, OS is a medullary tumor seen in the metaphysis of
located. Epiphyseal lesions include chondroblastoma, intra- 10-25 y/o. Most are seen in the femur near the knee.
osseous ganglion, and clear cell chondrosarcoma. Physeal Diagnosis is by pain, X-ray, and elevated levels of alk phos.
lesions are only seen in giant cell tumors. Metaphyseal Radiographically it appears cloud-like with sunburst periosteal
lesions include metastatic tumors, osteomyelitis, reaction and codman’s triangle. Skip lesions or lung
osteo/chondro/fibrosarcoma, malignant fibrous histiocytoma, metastasis indicates a poor prognosis. Paraosteal
non-ossifying fibroma, and unicameral bone cysts. osteosarcoma (PaOS) is seen 5-10 years later then medullary
Diaphyseal lesions include lymphoma, eosinophilic OS and is more common in females. It causes dull aching
granuloma, metastasis, multiple myeloma, and ewing’s pain and carries a better prognosis than OS, unless it extends
sarcoma. Peripheral/cortical lesions include into the medulla. Periosteal osteosarcoma (PeOS) is a rarer
osteochondroma and para or periosteal osteosarcoma. lesion that has similar age, location, and s/sx as medullary, but
3) Rate of growth is characterized by the borders of the is located in the diaphysis and carries a better prognosis.
lesion. Permeative lesions have a wide zone of transition and Paget’s Disease (P) is thought to have a viral cause, and is
are typical of fast-growing lesions, typically malignant tumors seen primarily in English-speaking countries. Affected bones
and osteomyelitis. Moth-eaten lesions have intermediate have a thick cortex and accentuated trabeculae. Paget’s
zones of transition with irregular borders. They may be predisposes an individual to OS, CS, FS, GCT, and MFH.
malignant or benign tumors. Geographic lesions have well- Enchondroma (E) is a benign, intramedullary, cartilaginous
defined sclerotic borders with a short zone of transition. This lesion. It occurs in any age and is characteristically painless.
finding is typical of benign tumors and cysts (although low- It is normally seen in hands and feet and may cause
grade malignancies may also sometimes look like this.) pathological fx. The lesions appear geographic with central
4) Periosteal reaction appearance may give clues to the type calcifications, possibly with endosteal scalloping or
of tumor. Single-layer periosteal reaction is typical of expansion. Ollier’s disease is the condition of multiple
osteomyelitis, benign tumors, and lower-grade malignant enchondromatosis that may predispose to malignancy. It is
tumors. Onion-skinning is typical of repetitive insults to the more common in children and causes growth disturbances.
periosteum, seen in osteosarcoma and ewing’s. Maffucci’s disease is hereditary multiple enchondromatosis
Perpendicular periosteal reaction has a spiulcated, sunburst
with soft tissue telangiectasias as well. Prognosis for Osteofibrous Dysplasia is an intracortical FD of the tibia
Maffucci’s is very poor, with almost 100% malignancy. (usually) seen in childhood. It is very uncommon. It is
Juxtacotical Chondroma (JC) is a solitary cartilaginous usually painless and has the characteristic ground glass
tumor that causes erosion of surrounding bone. The average appearance. It is associated with adamantinomas.
age is 22 y/o and is seen most commonly in long bones of the Fibrosarcoma (FS) occurs in the 3rd to 6th decades of life and
hands and feet. The tumor is symptomatic, causing pain and present as an aggressive lytic lesion with no matrix and no
swelling in the area. In 50% of cases ST calcification may periosteal reaction. Its location is generally metaphyseal. It
also be seen, with erosion and “saucerization” of the bone. may also arise secondary to other lesions as well (including
Chondroblastoma (CB) is a benign cartilaginous tumor of infarcts.)
the epiphysis. It is seen in individuals with open growth plates Giant Cell Tumor (GCT) is a locally aggressive lesion that is
(teenaged) and is geographic in nature. Calcifications are seen usually benign but not always. It presents as a lytic lesion in
in 50% of cases. Pain and joint effusion may cause CB to be the metaphysis extending into the epiphysis (and is the only
mistaken for arthritis, much like OO. one to occur in this area.) There is no matrix associated with
Chondromyxoid Fibroma (CF) is a very rare tumor that is GCT. The lesion may destroy the cortex and invade the ST as
metaphyseal, geographic, and multiloculated. It occurs in all well. It most commonly occurs in the femur, radius, humerus,
ages and is seen in the LE. Pain is “vague.” It may extend all and sacrum. It is slightly more common in females. S/sx
the way to the physis. include pain. The Giant cell reparative granuloma is a
Osteochondroma (OC) is a cartilage-capped exostosis of cousin of GCT presenting with lucent lesions of the hands and
bone. It is common in the general population, and is usually feet seen in children. It typically does not break through the
discovered in adolescence as a painless, hard “lump.” The periosteum. It may mimic and be hard to distinguish from
location is usually metaphyseal, and the exostoses brown’s tumors, even on histology. It is also called a solid
characteristically point away from the joint. Malignant aneurismal bone cyst.
transformation may have occurred if painful, greater then 2cm, Unicameral Bone Cyst (UBC) is a very common benign
or growing after skeletal maturity. Multiple hereditary lesion often seen in the distal calcaneus. It is medullary and
osteochondromatosis (MHO) is a multiple variant that is fluid-filled, and may be an incidental finding (although it can
autosomal dominant. It frequently causes growth disturbances cause pathological fx.) It is geographic, with sclerotic
in children and carries a 20% chance of malignancy. borders. It typically arises at a growth plate, migrating later
Benign Perosteal Osteochondromatous Proliferation into the diaphysis with age. 90% are seen in the humerus and
(BPOP) is a very rare finding seen in adults aged 20-30 y/o. femur. The hallmark of the UBC is the fallen fragment sign
Sometimes also called “nora’s lesion,” BPOP is benign, but (piece of cortex within the center of the lesion.) Tx is
has a very high rate of recurrence if removed. observation, steroid injection, of sx bone graft packing.
Chondrosarcoma (CS) occurs de novo or secondarily from Aneurysmal Bone Cyst (ABC) is a lytic expansile lesion
conversion of a benign lesion. The average age is 40 y/o. noted for the blood-filled cavities it forms. It may arise de
Pain is the primary s/sx. Lesions are typically geographic and novo or post trauma, or may be associated with CB, GCT,
metaphyseal, with lucent centers filled with small NOF, OB, CMF, or FD. It presents with pain in the long
calcifications. Tx is w/ sx. Variants of CS include clear cell bones and axial skeleton. It is usually seen in teenagers. Its
chondrosarcoma (CCC) which is an epiphyseal lesion (looks hallmark are fluid-fluid levels of two layers seen on MRI
like CB but in older patients) and dedifferentiated representing old coagulated blood and new, unsettled blood.
chondrosarcoma in which part of the lesion is low-grade and Fluid-fluid levels are also seen in telangectatic OS, MFH, and
another part is high grade. FD.
Non-Ossifying Fibroma (NOF) and its relative the Benign Interosseous Lipoma (IL) is a benign asymptomatic lesion
Fibrocortical Defect (BFOD) differ only in their location; the that is typically an incidental finding. It is usually seen in the
NOF is medullary and the BFOD is cortical. It is most calcaneus. It can have a central area of calcification, but is
common in children, and is usually asymptomatic unless there noted for its central area of fat on CT and MRI. It is seen in
is a pathological fx. X-ray shows a sclerotic border in the the same locations as UBCs.
growth plate to the diaphysis. DO NOT BIOPSY NOFs. Intraosseous Ganglion (IG) is a benign lesion of the
With age the lesion usually regresses. Jaffe-Campanacii subcortical bone typically seen in the middle-aged. It can
syndrome is multiple NOFs with café au-lait spots. present with pain. It may resemble a subchondral cyst on x-
Fibrous Dysplasia (FD) is a benign fibroosseous lesion of ray (but in the absence of DJD). It is most commonly seen in
bone that is usually unilateral (75%) and has a geographic the medial malleolus. It may also look like GCT but with a
appearance. The hallmark radiographic feature of FD is a lobulated matrix.
ground glass matrix. Dense areas of calcification may also Ewing’s Sarcoma (ES) is a common primary bone tumor
be seen. FD presents typically with deformity and sometimes seen in children, presenting with pain, low-grade fever, wt
pain, secondary to fx. Albright’s syndrome is polyostotic FD loss, and an elevated sed rate. It is rare in AAs. It has the
with café au-lait spots and endocrine abnormalities. As appearance of a small blue-cell tumor which it shares with
opposed to Jaffe-Campanacci syndrome, where the lesions primary lymphoma and multiple myeloma. It is most common
have a smooth border, albright’s syndrome lesions have a in the lower extremities. The lesion is permeative and has no
rough border. Mazabraud’s syndrome is polyostotic FD matrix. It produces an onion-skin type periosteal reaction.
with intramuscular myxomas. The shepherd’s crook The location can be metaphyseal or metadiaphyseal, and
deformity is also a hallmark of this type of lesion. produces a soft tissue mass surrounding the lesion. A variant
of Ewing’s is primitive neuroendothelial tumor (PNET) diaphyseal region. Marked edema of the hands and feet may
which has the same appearance, prognosis, and treatment, but be seen secondary to this infection. Bone density is lost in the
is associated with a chromosome translocation. spine, flat bones, and long bones with a coarse trabecular
Primary lymphoma of Bone appears similar to ES but in a pattern. The hallmark is fish vertebrae that appears as a
much older patient. Pain is usually the presenting symptom, biconcavity of the vertebral bodies on lateral x-ray (with disc
but there are no systemic signs. space preserved.) Periostitis is also seen in the hands and feet.
Multiple Myeloma (MM) is the most common primary Bones of the hands and feet can also be infracted in children
malignancy of bone, seen in older individuals. It presents with with sickle cell – the hand and foot syndrome causes
pain, weakness, or neurological symptoms. It sometimes localized infarction leading to early closure of growth plates in
presents with a solitary bone lesion called a plasmacytoma. the affected bones. This causes periosteal new bone
On x-ray there are cookie-cutter lesions that appear osteopenic formation (appearing 1-2wks post injury) and may lead to
and without sclerosis. Bone scans and MRI are unreliable in AVN. Cortical infarcts are more common, and the femoral
dx. MM commonly has the POEMS syndrome – head is most affected. Infarcts within long bones may present
polyneuropathy, oganomegaly, endocrine abnormalities, M on x-ray as calcified fat necrosis, which may look like a tumor
proteins, and skin lesions. (fibrous dysplasia.) The bone infarcts can also make the bone
Hemangioma is a common benign or congenital lesion most prone to infection and pathological fx. Infarct of the distal
commonly seen in the skull and spine. It appears to have tibia can lead to premature closure of the epiphysis laterally,
vertical striations in the spine and radial spokes in the skull. causing a characteristic tibiotalar slant with a valgus attitude.
CT shows a polka-dot pattern. It is usually asymptomatic. Thalassemias are gene deletions of primary hemoglobin
Hemangioendothelioma is a rare lesion seen in the lower subunits, causing a microcytic hypochromic anemia.
extremities. It presents as a cortical defect similar to Radiographically they present similarly to sickle-cell anemia,
hemangioma. It is typically seen in the middle-aged. with the notable exception that infarcts are generally not seen.
Eosinophilic Granuloma (EG) is a non-infectious granuloma Trabeculae may take on a “honeycomb” pattern and long
with histiocytes and eosinophils proliferating. Most are seen bones typically take on an Erlenmeyer flask morphology.
in the skull and vertebrae, but it may also occur in the lower Secondary centers of ossification may be delayed. Iron
extremity. Langerhans Histiocytosis is unifocal EG in one Deficiency Anemia characteristically causes osteoporosis of
organ system. Hand-Schuller-Christian Disease is the hands (in severe cases) and transverse striping of the
multifocal in one organ system. Letterer-Siwe Disease is metaphyses. Radiographic changes are more readily apparent
multifocal and multisystem. The average age of EG is 15 y/o. in the pediatric population.
There is little periosteal reaction, and there may be a LIPID STORAGE DISEASES
sequestrum (especially in the skull.) The lesions may appear Gaucher’s Disease is a disorder characterized by the
punched-out. The typical presentation is with a spontaneous accumulation of cerebrosides in reticuloendothelial cells. The
compression fx. They are typically diaphyseal. Bone scans lower extremities are more commonly affected. The classic
are unreliable in dx. presentation is ischemic necrosis of the proximal femur with
Adamantinoma is a rare, locally aggressive lesion seen Erlenmeyer flask deformity.
almost exclusively in the tibia. It appears in the 3rd to 5th Niemann-Pick Disease is an inherited abnormality of
decades as an eccentric, lobulated lesion. Periostitis is rare. sphingomyelin metabolism seen most commonly in persons of
Local pain and swelling are apparent on examination. jewish descent. Radiographically a widening of the medullary
Metastatic bone tumors are the most common type of bone cavity, coarse trabecular pattern, thinned cortex, failure of
malignancy, primarily from the breast, prostate, lung, and tabulation, and loss of bone density are seen. The Erlenmeyer
kidney. Periosteal reaction is rare. Location is based on the flask deformity is also seen.
tissue type. Blow-out lesions are renal and thyroid based. Sarcoidosis is a systemic granulomatous disease involving the
Peripheral and cortical lesions are lung metastases. skin, lungs, lymph nodes, and viscera. 10% of patients have
bone involvement. Children are rarely affected.
Radiographically dx is made with CXR; in the extremities the
RADIOLOGY OF SYSTEMIC DISORDERS middle and distal phalanges can show cyst-like punched out
Changes in the trabecular pattern in bone are indicative of lesions in the cortex.
reconstructive mechanisms and may suggest systemic disease. Pancreatitis causes fat necrosis of the bone marrow (which is
Osteoporosis, osteomalacia, and hyperparathyroidism all cause also seen in cancer of the head of the pancreas.)
a generalized loss and coarsening of the trabeculae. In many Radiographically bone infarction follows the fat necrosis
cases, this can also happen with anemias, storage diseases, and stage, with calcification and ossification of the affected
neoplastic disorders as well. Etiologies of abnormal bone marrow. The long bones of the hands and feet are most
texture include generalized overactivity of cancellous bone affected.
resorption, infarction due to disruption of blood supply, and Dysbaric Osteonecrosis (Caisson disease) is a pressure-
growth retardation due to localized hypoxia (in children.) induced osteoarthopathy seen in deep-sea divers. It presents
ANEMIAS as small localized infarctions of bone due to the formation of
Anemias that affect bone density include sickle cell nitrogen bubbles within the body.
anemia, thalassemias, and iron deficiency anemia. Sickle Cell Hemophilia is a disorder of blood coagulation due to plasma
Anemia puts pts at risk for salmonella infections seeded clotting factor deficiency. Classic hemophilia (A) is due to
hematogenously, with osteomyelitis occurring in the factor VIII, Christmas disease (B) is due to factor IX. Both
are X-linked diseases. Radiographically bleeding is apparent Paget’s Disease (Osteitis Deformans) is a disease of unknown
sub-periostally, intraosseously, and intra-articularly. The etiology. It, as well as fibrous dysplasia, are known as the
chronic hemarthrosis due to joint bleeding causes destruction great masqueraders of bone disease. (Outside of bone, gout
of the hyaline cartilage secondary to lysosomal enzyme and syphilis are also known as great masqueraders.) Paget’s is
release in the breakdown of blood products. Hemophilia can normally seen in the axial skeleton, but can affect any bone.
also cause premature closure of the epiphyseal plates, causing The pelvis, femur, skull, and tibia are most common. Its
tibiotalar slant seen also in sickle cell anemia. incidence is 4-6% in individuals over 40 y/o. Deformities
Metaphyseal and Diaphyseal Infarcts are usually due to associated with Paget’s include femoral bowing (shepherd’s
occlusive vascular disease, sickle-cell anemia, caisson disease, crook), tibial bowing (sabre shin), liontiasis (lion facies), and
infiltrative/collagen diseases, infection and/or idiopathic. The pathological fracture (banana fxs.) Pseudofractures, seen
increase in bone density seen in an infarct is due to crushing of also in osteomalacia/rickets, are also known as umbua zonen
the avascular debris, calcification of fat necrosis, hyperemic and looser’s lines – they represent demineralized areas of bone
removal of viable bone surrounding, bony apposition, and that are not actually fxs, although they appear that way on x-
periosteal involucrum formation. ray. The stages of Paget’s disease are
ENDOCRINE DISORDERS 1) Destructive phase – increased osteoclastic activity and
Acromegaly/Gigantism are both disorders characterized by osteoporosis circumscripta, both difficult to dx early.
excess in growth hormone. The result is joint space 2) Combined phase – bone destruction/turnover and fibrous
narrowing, bony sclerosis, cyst formation, and osteophytes bone replacement.
similar to DJD. It usually affects the acral skeleton most. In 3) Sclerotic stage – uniformly thickened trabeculae, ivory
the foot there is seen sausage digits, enlarged sesamoids/MPJs, appearance of bone (see also psoriasis), and bone expansion.
prominent metatarsal heads, prominent tufts and bases of 4) Malignant degeneration – metaplasia to osteosarcoma (rare)
phalanges, enthesis development, and constriction of proximal Lab findings for Paget’s show elevated alk phos (x20) and
phalangeal shafts. Fat pad thickening is a radiographic hypercalcemia. UA may show urinary hydroxyproline. Bone
hallmark of acromegaly. Hypertrophy of the bases and tufts of scan shows multiple hot spots (sometimes solitary). Course is
the distal phalanges causes secondary bony fusion resulting in slow and treatement is with calcitonin and supportive therapy.
pseudoforamina formation unique to this disease. Massive Osteolysis (Disappearing bone disease) is a disorder
Hyperparathyroidism causes generalized bone resorption causing sudden resorption of large areas of bone without
subperiosteally, intracortically, endosteally, and sub- malignancy. It is associated with lymphangioma or
ligamentous. Bone sclerosis and chondrocalcinosis are also hemangioma and clinically presents with pain. X-ray findings
seen. Brown’s tumors are unique to hyperparathyroidism. show apparent disappearance of bone, (tapering off in long
Renal Osteodystrophy is a disease of chronic renal failure bones), with no periosteal reaction noted.
that causes hyperparathyroidism, rickets/osteomalacia, Diabetic Osteolysis is a disorder characterized by marked
osteoporosis, soft tissue calcification, and vascular resorption with neuropathic changes. It is seen in the absence
calcification. Osteomalacia and Rickets primarily are caused of an infectious process, although that is usually a d/dx. It
by Vitamin D disorders, and cause bowing of the tibia and may be the result of a viral infection. Radiographically a
periarticular calcification. decrease in ST density, local osteoporosis, distal resorption,
Hypoparathyroidism may be primary (deficiency or licked candy stick appearance, and apparent bone
destruction of the gland) or secondary (gland unresponsive.) “regeneration” are apparent on x-ray.
It causes spinal calcifications that appear like osteophytes (of Poliomyelitis radiographically presents with severe cavus
DISH). Pseudohypoparathyroidism is the most common deformity, severe osteoporosis, short metatarsals and
secondary disease, causing a short stature, round face, phalanges, and a ground glass appearance of medullary bone.
brachydactyly, exostoses, cone epiphyses, and wide bones. Neurofibromatosis is a hereditary disease of mesodermal
Women are more affected than men. origin causing café-au-lait spots, skin neurofibromas, bone
Hyperthyroidism is characterized by a general catabolic state deformity, and local gigantism. Bone deformities include
of the body, resulting in loss of connective tissue as well as kyphosis, bony erosion, local overgrowth, overtubulation of
increased bone turnover. Osteoporosis is a common initial long bones, pseudoarthroses, and lytic areas of bone.
finding. Other findings include accelerated skeletal
maturation and myopathy. Thyroid acropachy is a AVASCULAR NECROSIS AND
complication of hyperthyroidism that includes progressive OSTEOCHONDRITIS DESSICANS
exopthalmos, hand and foot swelling, digital clubbing, Avascular Necrosis (AVN, epiphyseal ischemic necrosis,
pretibial myxedema, and periosteal new bone formation (mid- osteonecrosis, osteochondrosis, epiphysitis, and aseptic
diaphyseal), particularly of the 1st metatarsal. The edema has necrosis) is a disruption of the vascular supply of bone leading
a nodular appearance. to the death of an osseous structure. Primary causes are
Hypothyroidism (Myxedema) is caused by a deficiency of unknown. Secondary causes can be fracture, sickle cell
TSH, thyroid hormone, or iodine. In infants the disorder is anemia, long-term corticosteroid use, and alcoholism.
called cretinism. In adults the disorder causes edema, dry Clinically, AVN is variable and relatively minor in
skin, coarse hair, lethargy, fatigue, paresthesia, and symptomatology. Pain and disability progresses as it is
bradycardia. Tarsal tunnel syndrome is also a common untreated. AVN normally lasts about 9 mos, but in some cases
finding. (legg-calve-perthes) it can last up to 8 years. Spontaneous
AVN occurs in the distal femur, femoral head, and metatarsal congenitally small navicular – an incidental finding of a very
heads. Trauma-induced AVN occurs in the talus. small navicular on x-ray may not be kohler’s. The appearance
Alcoholism-induced AVN occurs in the femoral head. should be patchy or uniform sclerosis, collapse (silver dollar
Surgery can also be a cause of AVN. sign), and fragmentation of the bone. The joint space is
The Avascular Phase of AVN is the actual loss of blood usually preserved.
supply, leading to bone death, failure of epiphyseal growth, Iselen’s Disease is a non-AVN of the fifth metatarsal base. It
minimal bony deformity, reactive hyperemia, synovial is actually a tractional apophysitis and is self-limiting.
thickening, and edema. On x-ray you see a small epiphysis, Freiberg’s Disease is a true AVN of the metatarsal heads.
normal bone density, increased joint space, capsular swelling, The second metatarsal is most commonly affected, particularly
metaphyseal osteopenia, and a widened growth plate. in pts w/ a long 2nd metatarsal. There is tenderness and pain
The Revascularization Phase sees infiltration of new blood over the affected MPJ, and the pain is exacerbated with
vessels and fragmentation of the dead parts. On x-ray you see activity. Infarction of the epiphysis leaves necrotic bone that
a sclerotic rim, new bone deposited on old bone (snow cap causes degenerative changes and cortical thickening.
sign), fracture inferior to the articular cortex (crescent sign), Adaptive changes occur on the juxta-articular surface
clefts and fragmentation, flattening of articular surfaces, a (flattening and broadening of the joint space.) Theimann’s
widened metaphysis, and “head within a head” appearance. Disease is a non-AVN of the fingers and toes that is extremely
The Repair and Remodeling Phase includes new bone rare. It occurs secondary to trauma and is attributed to
deposition replacing bone resorption. On x-ray you see multiple ossification centers in the phalanx.
reconstitution of the epiphysis and the disappearance of clefts. Sesamoid AVN is called Trevor’s (fibular) or Renandier’s
The Residual Deformity (stage 4) includes restitution of the (tibial) and occur secondary to trauma and fracture.
epiphysis, with the amount of deformity dependent on Osteochondritis Dessicans (OCD) is a disease of unknown
compressive forces on the affected bone. On x-ray you see a etiology affecting children and adolescents, typically male. A
deformed articular surface, usually flattened. small segment of necrotic bone forms subchondrally, which
Legg-Calve-Perthes Disease is a true AVN of the capital may heal spontaneously or become a joint mouse. Joint mice
fragment of the femoral epiphysis in children. Males are more in these cases are typically symptomatic because the major
commonly affected than females. 3-12 y/o is the average age. component is cartilage, which is not visible on x-ray.
Prognosis worsens with later ages. 1 in 10 are bilateral. Symptoms are joint effusion, pain on ROM, crepitation and
Clincally the pt has limp and a vague pain in the groin that locking and localized tenderness (vague to the ankle). The
may extend to the knee. Pain is exacerbated with activity. disease may be hereditary. There are two types of OCD – In-
There is often muscle atrophy in the thigh and a history of situ and displaced according to the fragment.
trauma in the affected individual. The etiology is unknown. Radiographically they look similar to talar dome fractures.
Blount’s Disease is not a true AVN, seen in infants affecting
the medial tibial epiphysis. The failure of the growth plate in ADULT FOOT DISORDERS
this area contributes to the formation of a tibia/genu varum. Heel Pain – Plantar: R/O soft tissue involvement, fissuring,
Osgood-Schlatter Disease is not a true AVN either, affecting tumor, and nerve entrapment. Plantar fasciitis presents
the tibial tuberosity. It is caused by a partial disruption of the radiographically as increased ST density of the plantar fascia,
tibial patellar ligament (w/ or w/o avulsion). Commonly and deossification of the calcaneus with accentuation of the
ossific nodules are seen with this disease, forming a large, tensile trabecular patterns (do not confuse w/ stress fx.) Heel
prominent tibial tuberosity. The course is self-limiting and spurs are sometimes seen in plantar fasciitis but also can be
affects mainly males 11-15 y/o. caused by the pull of the FDB, Abd hallucis, Abd digiti
Sindig-Larsen-Johannson Disease is not an AVN, but is minimi, and the long plantar ligament. Heel spurs account for
looks similar to Osgood-schlatter disease. It is a traction 15% of all adult foot complaints. They are typically directed
induced avulsion fragment of the lower patella. forward from the medial tubercle. They are assoc w/ obesity,
Diaz Disease is a true AVN of the talus. It is usually caused pes cavus & pes planus, and osteophytosis. It is important to
by trauma, graded by the Hawkin’s classification system. A note that the size of the spur has no effect on the symptoms,
positive Hawkin’s sign indicates revascularization. and oftentimes the contralateral (asymptomatic foot) will have
Sever’s Disease is a non-AVN of the calcaneus. It is a spur of equal or greater size. 90% do not require any type of
calcaneal apophysitis caused by traction of the calcaneus. It sx intervention to tx. Fluffy heel spurs are associated with
presents with heel pain and irregular ossification on x-ray. All certain arthropathies, particularly Reiter’s, ankylosing
radiographic findings, however, may also be seen in normal spondylitis, psoriatic arthritis, RA, and Paget’s disease. Other
children. Examples include a sawtooth metaphysis, decreased causes of plantar heel pain include calcified bursae,
metaphyseal density, increased density of the apophysis, and atrophied fat pad (measuring 15-23mm), steroid-induced
fragmentation of the apophysis. Males and females are calcification, and nerve entrapment. Nerve entrapment can
affected equally. The primary D/dx for sever’s disease is be differentiated from plantar fasciitis by its response to
spring heel (hematogenous osteomyelitis of the calcaneus.) In steroids (relief in plantar fasciitis, no change in nerve
spring heel, the plantar tubercles typically show signs of entrapment), and with nuclear scanning (+ in plantar fasciitis).
deossification, not seen in sever’s disease. Heel Pain – Posterior: Tendo-Achilles (partial tear or
Kohler’s Disease is a true AVN of the navicular seen in males complete tear/AODM), retro-achillean bursa (caused by
around 5 y/o. It presents with localized pain, edema, shoe irritation), or retro-calcaneal bursae/Haglund’s
tenderness, and decreased ROM. Some pts have a deformity (most common.)
Heel Pain – Medial: Tarsal Tunnel Syndrome is caused by Hallucial Sesamoids are located in the tendon of FHB and
chronic irritation of the posterior tibial nerve in the flexor ossify between the ages of 8-12 (earlier in females than
retinaculum and inferior to the medial malleolus (seen w/ RA, males.) Up to 1/3 are multipartite, esp the tibial.
AODM, trauma, fx, and dislocation.) Medial calcaneal nerve Osteonecrosis of the tibial is termed renandier’s, while that of
entrapment and posterior tibial tendon dysfunction are the fibular sesamoid is trevor’s. An interphalangeal sesamoid
other causes of medial heel pain. (Radiographically is present in 43% of individuals (esp males), and is usually
represented by increased TC angle, anterior Cyma line, and bilateral. It may contribute to IPJ HPK formation.
sagging of the metatarsocuneiform joint.
Heel Pain – Lateral: Lateral heel pain is the most rare PEDIATRIC RADIOLOGY
presentation. It can be caused by sural nerve entrapment, Ossification of bones in the foot occurs sequentially from birth
calcaneo-fibular irregularities, peroneal tendon problems, to adolescence, and knowledge of this order is necessary for a
tarsal coalition, and ankle/RF varus deformities. physician to determine the correct skeletal age of a patient.
Midfoot to Rearfoot Relationship: Calcaneal pitch is The talus, calcaneus, metatarsals, and sometimes cuboid are
normally between 10-20 degrees – greater than this is pes ossified at birth. Then the order of ossification is medial
cavus and less is pes planus. A supinated foot will have an cuneiform at 6 months, intermediate cuneiform at 1 year,
accentuated or bullet hole sinus tarsi, while the pronated foot medial cuneiform at 1.5 years, and finally the navicular at 2.5-
will have a pseudo sinus tarsi which is in fact the posterior 5 years. The sesamoids ossify around 7-8 years of age.
subtalar joint facet being brought into the plane of the film. Common variations include an independent ossification center
Cavus Foot Structure: Typically caused by neurological or of the medial malleolus (20%) or of the distal phalanx of the
muscular imbalance. Forefoot cavus has an apex at the hallux, irregular epiphyseal plates, absent epiphyseal
lisfranc’s joint. Midfoot cavus has its apex at the chopart’s ossification in the middle phalanges, 5th digit synphalangism,
joint. Rearfoot cavus is calcaneal in nature and has a bilateral/symmetric cone-shaped epiphyses (which fuse earlier
calcaneal inclination angle of >30 degrees (and claw toes.) than normal epiphyses), and pseudoepiphyses, fissues, and
Foot Faults are sections of the foot where adjacent segments exostoses that may appear similar to fracture.
are improperly aligned. Hindfoot/Subtalar/Midtarsal Fault An accessory navicular is seen in 10% of children, posterior
Syndrome is characterized by an anterior Cyma line, pseudo to the navicular tuberosity, which can cause pes planovalgus
sinus tarsi, increased talar declination angle, decreased deformity that requires sx excision. The navicular itself
calcaneal inclination angle, everted cuboid, and less than 85% ossifies by multiple ossification centers which are
coverage of the talar head. The cuneiform-navicular fault asymmetrical and may look like kohler’s disease.
occurs at the CN joint. The medial cuneiform is plantarly Similarly, the calcaneal apophysis has multiple ossification
displaced, and the CC joint has decreased joint space dorsally centers which may look like sever’s disease. The calcaneus
(with increased space plantarly.) The foot appears longer. may also have a pseudocyst called the ward’s or neutral
Pes Valgo-Planus: The acquired form has a calcaneal triangle which appears as a radiolucent circle/triangle in the
inclincation angle >10 degrees (normal) but the inferior anterior calcaneus (10%). The calcaneus often has compact
surface of the calcaneus is concave. The congenital form bone islands as well. The apophysis of the 5th metatarsal base
(CCPV) has a low calcaneal inclincation angle with a convex runs parallel to the shaft, and should be distinguished from a
inferior surface of the calcaneus. fracture.
Diabetic Neuropathy occurs an average of 16 years after the RADIOGRAPHIC EVALUATION OF THE PEDIATRIC PATIENT
diagnosis of Type 2 DM (AODM). 74% present with a foot The standard AP in infants must use a simulated WB position
ulcer, particularly if loss of proprioception is involved. The which is accomplished with the knees flexed and held together
forefoot ulcerates twice as commonly as the rearfoot. Charcot and an additional 15 degree angulation of the tube. Lateral
neuroarthropathy has three stages of destruction, coalescence, views are positioned similarly, with the beam aimed at the
and remodeling. ankle. CT scanning is used in peds to determine tarsal
Stress Fractures (aka march/fatigue fx) occurs most coalitions as well as growth plate abnormalities. MRI can be
commonly in the metatarsals, esp the 2nd (38%), then the used to evaluate unossified structures.
calcaneus (28%), and tibia (24%). Early X-rays appear GROWTH VARIATIONS
normal, however it may appear radiographically over time and Macrodactyly is a localized gigantism of the digits caused by
as symptoms persist. fat infiltration, long-standing hyperemia (leading to
Hallux Abducto-Valgus (HAV): Is evaluated hypertrophy), or neurofibromatosis.
biomechanically by radiograph. Kippel-Trenaunay Syndrome is a localized gigantism of the
Normals: IM 10-20 PASA 0-8 hands or feet, usually in only one extremity. Varicose veins
HA 0-15 DASA 0-5 and port-wine hemangiomas are also present.
MA 12-15 Larsen’s Syndrome is characterized by a double ossification
Other factors evaluated are tibial sesamoid position, total center of the calcaneus with a cleft palate, joint dislocation,
adductory angle, metatarsal break angle, and bone density. equinovarus/valgus foot, broad thumbs, and vertebral
The joint status is described as congruous, deviated, or abnormalities.
subluxed. Positional HAV is one where PASA+DASA<HA. Gorham’s Disease (vanishing bone disease) is massive
Structural HAV is one where PASA+DASA>HA. osteolysis following episodes of trauma that can to opposing
Combined HAV is if the PASA+DASA >>>HA. bones across joint spaces. Radiographically the bones will
appear tapered with an absence of periosteal reaction. The Amniotic band syndrome (streeter’s bands) is the formation
primary d/dx is charcot foot. of constriction rings around the limbs, present at birth, which
Neurofibromatosis is characterized by multiple leads to distal loss of the limb. The cause is unknown.
neurofibromas with café-au lait spots, lipomas,
kyphoscoliosis, enlargement of the foot bones, multiple ST BIOMECHANICAL ABNORMALITIES
densities, congenital pseudoarthroses (mainly in fibula/tibia), Kite’s (TC) angle is normally 25-50° in peds; however, this
marked cortical thickening, and a hypoplastic pelvis. angle requires the use of estimated axes for involved bones.
Dwarfism is a plastic condition of localized bony Another method is to compare the long axis of the talus and 1st
undergrowth. It occurs when tumor, radiation, or primary MT – they should line up.
developmental defects cause the epiphyseal plates to Rearfoot Valgus is characterized by a high Kite’s and talar
prematurely close. The tibia is typically shorter than the declination angles, and is seen in flatfoot deformities,
fibula, the distal femoral metaphysis is hypoplastic, and the congenital vertical talus, and neuromuscular disease.
tibial and fibular metaphyses are cupped. Kyphosis is the Rearfoot Equinus is a calcaneal inclincation angle >90°. It is
most common skeletal abnormality. Equinus is a common seen in congenital equinovarus and congenital vertical talus.
foot deformity. There are 3 types of short-limb dwarfism: Rearfoot Varus is characterized by a low Kite’s angle and
Rhizomelic Dwarfism is a shortening of the proximal nearly parallel talar and calcaneal axes. It is seen in
segment (humerus or femur) equinovarus and some neuromuscular diseases.
Mesomelic Dwarfism is a shortening of the intermediate Pes Planovalgus is the most common cause of RF valgus.
segment (tibia/fibula or radius/ulna) The etiology is uncertain but thought to be related to
Acromelic Dwarfism is a shortening of the distal ligamentous laxity. On X-ray there is abduction of the
segment (hand, foot) forefoot, laterally displaced navicular, PF calcaneus, DF
Hurler’s Syndrome is a syndrome characterized by metatarsals, and PF talus. If seen w/ equinus or a short
dwarfism, mental retardation, kyphosis, hepatosplenomegaly, Achilles, it is termed hypermobile flatfoot. HAV is also a
and coarse facial features. Radiographically you see shortened common finding. Peroneal spasm may be seen if there is
shafts of the long bones with tapered ends, particularly in the tarsal coalition. Most are asymptomatic and require no sx.
upper extremities. There is also a coax valga and genu varum. Pes Cavus is most commonly seen w/ neuromuscular disease,
Aplasia/Hypoplasia (dysmelia) describes any reduction particularly CMT. Typically there is high calcaneal
malformation. The cause is unknown. They are classified by inclination, PF metatarsals, and clawed digits.
extent and orientation. Phocomelia is loss of the intermediate Talipes Equinovarus (Clubfoot) is most common in males,
segment of a limb. Meromelia is partial absence of a limb. Caucasians & Polynesians, and is hereditary. There are 3
Hemimelia is absence of a half (longitudinally) of a limb. etiologies – intrauterine pressure (most flexible), abnormal
They are a part of Fanconi’s anemia and thrombocytopenia- tendon/ligament attachment (difficult to treat), and teratologic
absent radius synd. Proximal femoral focal deficiency (PFFD) (assoc w/ arthrogryphosis and myeolodysplasia. Clincally you
is assoc w/ absence of the ipsilateral fibula as well. see FF adduction, RF varus, equinus, and TN subluxation. On
Polydactyly is the most common excess malformation. It can X-ray there is low Kite’s angle, FF adduction, PF calcaneus,
be isolated or occur w/ syndromes. There is no sex and stair-stepping of the metatarsals. The talonavicular
predilection and it is most common in the AA population. relationship is estimated – Simon’s angle should be btw 0-20°,
50% of cases are bilateral. The tetamy & McKusick TC angle should be >15 and talometatarsal <15° for normal.
classification is preaxial (tibial side of 2nd digit) or postaxial Tx is initially casting, then sx w/ FF correction first (then RF.)
(fibular side of 2nd digit – 80%). The venn-watson Congenital Vertical Talus is assoc w/ arthrogryphosis,
classification is that preaxial polydactyly has a short myelomeningocoele, and chromosomal abnormalities. It is
metatarsal w/ wide metatarsal head. Postaxial has a Y or T more common in males and it 50% bilateral. On X-ray it is
shaped metatarsal, or complete metatarsal duplication. similar to pes planovalgus, but talus is markedly PF, and
Brachydactyly is shortening of the digits due to hypoplasia or calcaneus may be in equinus. With DF metatarsals this makes
syndactyly. It is seen in pseudo(pseudo)hypoparathryroidism, for a rockerbottom foot.
turner’s synd, sickle-cell, Down’s, and fetal dilantin synd.
Fusion deformities are errors of segmentation. Syndactyly BASIC CXR INTERPRETATION
can be simple (only soft tissues fused) or complex (varying CXR positioning: Standard positioning is with the patient
degrees of bony involvement). facing the cassette for PA, or with their back to the cassette for
Apert’s syndrome (discussed earlier) is manifested AP projections. The patient is instructed to inhale fully to
radiographically by multiple bony coalitions all over the body, standardize the view. AP projections make the heart appear
leading to limitation of motion that is progressive. magnified b/c it is farther from the cassette – the PA
Nievergelt-Perlman syndrome is characterized by clubfoot, projection shows clear lung fields and a normal-sized heart.
synphalangism, conduction deafness, ball&socket ankle joint, Lateral films are taken with the pathological side closer to the
flexion deformities, and phocomelia or hemimelia. film. Normally the position of the patient is standing, w/ arms
Carpenter’s syndrome is polysyndactyly w/ premature raised and chin up. However, depending on the condition of
synostosis of the skull. the patient, sitting or decubitus positioning may be necessary.
Poland’s syndrome is syndactyly and digital hypoplasia. The standard position decreases enlargement and distortion of
the heart and vessels, allows the diaphragm to move down,
and shows air-fluid levels better in the chest. When
positioning the film on a viewbox, it should be placed as either side of the medistinum and are readily appreciable on
though the patient is facing you (i.e. heart facing their left.) X-ray when adenopathy is present.
Osseous Structures to be evaluated in the chest include the LUNG PATHOLOGY
sternum, clavicles, scapulae, ribs, and thoracic vertebrae. On Alveolar processes appear as triangular-shaped areas of
lateral view there are two areas of radiolucency called the increased readiodensity, depending on the extent of
retrosternal airspace and the retrocardial airspace – this is involvement. This presentation is seen in pneumonia,
normal. You should be able to visualize 10 ribs in a normal advanced CHF, ARDS, and PE. Occasionally there may be
individual. Intercostal spaces are named by the rib superior. the addition of fluffy margins in the case of longer standing
The Diaphragm is divided into two on the AP view by the pathology, due to extension of fluid through adjacent acini
vertebrae, making a left and right hemidiaphragm. The right through the pore of Kohn. The silhouette sign is the name
side is normally higher due to the liver (and moreso in HSM.) given to alveolar pathology that exists in close proximity to
The Pleura can only be appreciated with pathology (pleurisy.) the heart or great vessels – the water-density of the pathology
The Lungs should be evaluated in more depth than the rest of makes the outline of the heart and vessels (also water dense) to
the chest X-ray because of subtleties that can be easily missed. be indistinct.
It is recommended to first do a “quick glance” of the lungs to CHF and pulmonary edema cause the hilar lymph nodes and
determine if they are the normal shape and density. Then, vessels to become more distinct on X-ray. The fanning out of
divide each lung into thirds in both the transverse and sagittal the lymphatic vessels bilaterally produces a “batwing” or
planes. Then examine each third separately from each other “butterfly” appearance and is described so radiographically.
with respect to the anatomy of each and densities seen therein. Interstitial processes are those that cause an increase in
If pathology is suspected, determine if it is confined within radiodensity of the interstitium, meaning bronchi, septa, and
lobar divisions – almost all lung pathology except tumors are lymphatics. This type of pathologic process is seen in
lobar in nature. The costophrenic angles are also important to interstitial pulmonary edema, fibrosis, TB, viral pneumonia,
note, as blunting of these angles may be a sign of infection or metastatic tumor, connective tissue disease, and interstitial
left ventricular hypertrophy. Bronchi are not normally seen pneumonitis. Miliary TB is seen as multiple tiny focal areas
on X-ray unless they are thickened, which is the case in cystic of increased radiodensity. Asbestosis appears as an interstitial
fibrosis, CHF, or lymphatic obstruction secondary to tumor. process in the basilar lung fields. Emphysema is a loss of
Supporting structures of the lungs are similarly not seen interstitial density due to loss of tissue in the interstitium.
normally on X-ray, but in pathological processes these lung Besides a change in density, the diaphragm is pulled low
septa, as well as lymphatic channels, can be seen and are (making the lung fields appear “tall”), and the heart is deviated
referred to as Kerley lines. Kerley A lines are apically medially. On a lateral view, the sternal angle may be
located, extending out from the hilum or upper margins. increased.
Kerley B lines are basilar in location, found in the lower lung Primary lung lesions may be seen virtually anywhere in the
periphery, about the diaphragm. Kerley C lines appear lung fields. A general rule of thumb when assessing primary
anywhere in the lung, but have a crosshatch appearance. lung tumor is that dense masses tend to be benign, while fluffy
Kerley lines are seen in interstitial edema/fibrosis, sarcoidosis, and ill-defined masses are more likely malignant. Also, if a
pneumonia, lymphatic obstruction, connective tissue disease, lesion is particularly dense (meaning possibly a calcified
RA, and scleroderma. center), it is increasingly likely that the lesion is a granuloma.
The Mediastinum is the last part of the CXR to examine. Another way to differentiate tumor from granuloma is by
Primarily noted in the mediastinum are the heart and great looking at blood flow to the area – tumors tend to be
blood vessels. An area of increased density is seen usually on associated with blood vessels anatomically, whereas
the left side of the trachea and is called the aortic knob – this granulomas are not. Also important to keep in mind is that
is where the aorta turns back on itself. The size of the heart is (for some reason) nipples are visible radiographically, and it is
estimated using the cardiothoracic ratio which is normally important to note where these are initially, so that they are not
2:1 (the heart should be half the width of the lungs in total.) mistaken for a tumor later.
Cardiomegaly, also known as a “boot-shaped heart,” can be If a lesion is metastatic, it may take on a localized metastasis
caused by sports conditioning (normal), CHF, mitral valve pattern (one large tumor surrounded by many small ones), or if
stenosis, chronic HTN, pericardial effusion, and left from another source may be scattered throughout the lung
ventricular hypertrophy. Masses in the medastinum usually fields. Central cavitation in large tumors is indicative of rapid
represent aortic aneurysms. The trachea, located superiorly growth and carries a poor prognosis.
in the mediastinum, is used to determine the midline of the
body in the CXR. When compared with the aortic knob
(which should be slightly to the left) and the right atrium
(which should be slightly to the right), it can be determined if
the patient was standing in the correct position for the AP/PA
view. In peds it is sometimes necessary to visualize the
thymus, however it is normally obscured by other mediastinal
contents. Therefore, it is the only case in which you take
expiratory CXR views, so that the thymus may be allowed to
deviate away from the midline and be visualized before the
lung fields. Lastly, the hilar lymph nodes are located on
and breast. Ga-67 is taken up in tumors and abscesses w/in
2h, but is ideally scanned at 36-72h. It is less dependent on
vascular supply than Tc-99, but is only sensitive to infection.
Post-surgical sites can stay hot to Ga-67 for up to 2wks.
Chronic osteomyelitis is negative in Ga-67, + in acute. Tc-99
remains positive in both.
Indium is an agent that is tagged to leukocytes in the
patient’s blood and reintroduced via I.V. It is more specific
than Ga-67 but more dependent on leukocyte chemotaxis. T½
is 28d. I-111 is used to evaluate osteomyelitis (acute), DM
NUCLEAR IMAGING neuropathy, joint infections, bone marrow, and ST infection.
Nuclear imaging (scintigraphy) is used to analyze a number of It may show false positives in inflammatory arthritis.
organ systems in the body, most commonly bone. This is due COMPUTERIZED TOMOGRAPHY (CT)
to the fact that bone loss must reach 33-50% to be detected CT is an imaging modality that uses highly focused X-rays to
radiographically, whereas with nuclear imaging changes can create detailed images with subtle changes in tissue density
be detected within hours. Nuclear imaging can also pick up and accurate descriptions of lesion location and size.
pathology that is not clinically visible. Most common Computerization also allows 3D reconstruction. The primary
pathologies indicated are bone tumors, osteomyelitis, stress disadvantages of CT are cost and increased radiation exposure.
fracture, non-specific bone pain, and AVN. The primary It is indicated for trauma, coaltions, ST masses, and infection.
disadvantage to nuclear imaging is poor resolution, which The X-ray tube rotates around the patient in a CT scan, while
makes this imaging modality non-specific (despite high opposite the tube are detectors that are used to create an
sensitivity.) image. The image produced is divided into a grid, of which
Uptake of the nuclear tracer in a bone scan occurs in an each 2D segment is a pixel, which when coupled with the 3D
area where the normal process of bony turnover is upset in any depth is called a voxel. The depth of the voxel is determined
way. Areas where turnover is increased (typical in injury) by slice thickness. Each voxel is then assigned a number
show up “hot” or darker on the image, whereas areas where based on the amount of radiation that passed through that
infarction or compromised blood supply have occurred show segment. If the density of tissue is greater than water, the
up “cold” or blank. number is positive, whereas if the tissue is less dense than
There are a variety of imaging agents available for use in water it gets a negative number. (Bone is 1000-3000, air is -
bone scanning, each with advantages and disadvantages. In 1000.) In an image, the lowest density is represented by
addition to availability and half-life, imaging agents also vary black, while the highest is white, and everything inbetween is
according to the carrier molecule that they bind. Technetium a gray scale, as in X-ray. However, in CT particular densities
is the most common, and may be bound to phosphate (Tc-99, can be windowed, that is, anything below the density window
for bone), pernechate (for brain/blood flow, joint imaging), or is black and anything above is white. This means that great
albumin (for lung perfusion/VQ scans). Also available are levels of contrast can be achieved if only the desired
gallium (Ga-67, for inflammation) and indium (I-111, also for pathological tissue is windowed (others are ignored.) Larger
inflammation). A new agent, ceretec, uses Tc-labeled windows, then, have lower contrast values.
leukocytes and has a shorter t½ as well as increased specificity When ordering CT scans, the specific area of interest
over I-111. For most agents, renal dysfunction is a should be stated (possibly w/ bone scan), along w/ plain films.
contraindication for use. Any lesions should have a measurement request, and
The most basic bone scan uses Tc-99, which incorporates reconstructions and magnification are avalible if appropriate.
as phosphate in areas of new bone formation. The t½ is 6h, CT scans include a scout film, which shows the location of
and requires adequate blood supply. Post-injection, uptake specific slices as well as slice thickness. The scout film looks
will be greatest in the bladder, SI joint, ASIS, kidneys, similar to a plain film x-ray. Image reconstruction in CT is
nasopharynx, lacrimal glands, and epiphyseal plates. used to view sagittal plane slices, which are out of phase with
Scanning for Tc-99 occurs in 4 phases. First is the the CT scanner and thus must be calculated by the computer.
dynamic/blood flow phase which is taken immediately after These reconstructed images are also called secondary slices.
injection. Little is seen in this phase normally. The second Finally, there is a specialized CT you can order called a
phase, blood pool, occurs 1-6 minutes post-injection, and scanogram, which is an image of the entire lower extremity
shows vascularity to a particular region. The third phase, from the pelvis, used to measure LLD. A scanogram is
bone imaging, is taken 2-4h post injection and shows bony basically a scout film with measurements to determine
turnover (usually the most important.) The last phase, absolute length of bony segments.
delayed, is taken 24h post injection, and measures clearance Overall, CT terminology and analysis is very similar to
of the Tc-99 agent from the tissue. For osteomyeltits, Tc-99 plain films. It is most effective at analyzing osseous
can pick up changes w/in 24h (compared to 10d for X-ray.) pathology, and is often used in conjunction with bone scans.
Gallium 67-citrate is a different agent which is not It is less expensive than MRI but inferior to MRI for analysis
sensitive to osteoblastic activity, and has a long (72h) t½. It is of soft tissues.
used to image ST inflammation through binding to iron
binding proteins in WBCs. Normal uptake is seen in the liver,
spleen, stomach, lacrimal glands, intestine, salivary glands,
images or low intensity on T2 images. It should be
homogenous in appearance.
Cortical Bone lacks free hydrogen, so it appears to have low
signal intensity on all images.
Tendons and Ligaments have low hydrogen content also,
(though higher than bone) and has low intensity in all images.
Tendons are contrasted best in T1 weighted images, however.
Subcutaneous Fat and all other fatty areas have a high T1
signal intensity and low T2 signal intensity.
Vascular Supply to an area shows up with low intensity in
MRI due to movement, called a flow void. This is why
contrast media are required to visualize blood vessels on MRI.
Muscle is intermediate in hydrogen density, and thus shows
up intermediate intensity in all images.
MAGNETIC RESONANCE IMAGING (MRI) SAFETY CONSIDERATIONS
MRI is an imaging modality that allows for the visualization MRI cannot be used with electrically or magnetically activated
of hydrogen protons in the human body. Hydrogen protons implants like pacemakers, defibrillators, or cochlear implants.
are present in nearly all materials and are electrically charged, Metallic objects located within the body will not cause harm,
within a magnetic field, and spins on its own axis. This makes but cause artifact on the MRI readout which may render it
it ideal for differentiating between different tissue types and useless. If it is suspected that a patient may have a metallic
analysis of hydrogen proton concentration gives clues to the foreign body in the vicinity of the eyes, an orbital X-ray is
chemical makeup of the tissues analyzed. indicated. Lastly, it has not yet been proven that MRI is safe
The most important component of an MRI scanner is the to use on gravid females; for women in their first trimester, it
magnet, which can be permanent (exerting 0.4T) or super- is advised that a risk/benefit analysis be done before making
conducting (1.5-2.5T) and initiates the process by which the the decision to go ahead with MRI.
hydrogen protons may be visualized. In the first step of the PATHOLOGY ON MRI
process, magnetic field, the magnet is activated around the Osseous trauma, including stress fractures, will typically
patients body in the area to be visualized. The second step, show up as a decrease in signal intensity on T1WI in the area
radiofrequency pulse, is a pulsing of electromagnetic energy of trauma, with diffuse increase in intensity on T2WI due to
in accordance with the precession (spin) of the hydrogen localized inflammation to the area. STIR images can be
atoms, which is a frequency called the larmor frequency. utilized to see marrow edema.
This pulsing at the larmor frequency causes all the hydrogen Avascular necrosis will show a well-defined line of
protons to spin in the direction of the magnetic field. Then, in demarcation between viable bone and devascularized bone on
the last step, relaxation, the magnetic field is turned off, and T1WI.
the hydrogen atoms slowly return to their normal spin axes. Osteochondritis dessicans will show up as a decreased area
The time which it takes for this to happen is termed relaxation of signal intensity on a T1WI in the acute stage, but as the
also, and each tissue type has different relaxation rates. condition becomes chronic intensity increases.
Measurement of spin relaxation is the basic principle ST infection is best visualized as increased intensity on T2 or
behind how the MRI produces an image, and the intensity with STIR images due to inflammation in the area. The degree of
which tissues show up on the image relate to time values contrast seen can differentiate abscess from cellulitis.
associated with relaxation. One of these values is T1, which is Osteomyelitis shows high intensity on T2WI for cortical bone
the time it takes for a particular tissue to regain 63% of its as well as low signal intensity on T1WI for medullary bone.
longitudinal spin. Another value, T2, is the time it takes for a Charcot neuroarthropathy shows up as a decrease in signal
proton to lose 63% of its transverse spin. (Note: T2 is always intensity on T1 and T2WI for affected joints.
a longer value than T2.) Other values include TR (time to Bone tumors typically have a low to intermediate signal on
repeat – the duration of one imaging cycle), and TE (time to T1WI and higher intensity on T2WI, although they may be
echo – time in which magnetization is converted to RF signal.) high on T1WI if fat or blood-filled. Calcification within a
Weighted images are those that emphasize a particular time tumor will also show up as lower intensity on all images.
value, to focus on a particular tissue type. A type of image Soft tissue tumors also show up with low T1WI signal
called proton density is a balance of a long TR and short TE intensity and high T2WI signal intensity, however, the
interval, which does not emphasize any particular time value. composition of the tumor will affect this. Benign lesions tend
A STIR image is a special form of inversion recovery in to be well demarcated, homogenous, and have a less
which the T1 time is very short – this is also called “fat pronounced change in signal intensity.
suppression” because fat tends to show up black. In addition Ganglion cysts are easy to pick out on MRI because of their
to weighting images, contrast can be added (gadolinium, very low T1WI intensity and very high T2WI intensity.
gadolinium DTPA) to visualize vascular lesions. Enchondroma shows up as homogenous and loculated with a
NORMAL ANATOMY ON MRI high T2WI and STIR intensity due to cartilage. Calcifications
Bone Marrow is mostly composed of fat, which has a short may show up as a heterogenous intensity.
relaxation time and thus shows up at high intensity for T1
Lipomas tend to be superficial homogenous lesions with the Other uses for arthrography besides ligament injury may be to
same signal intensity as surrounding fat, unless calcifications visualize joint mice, synovial hypertrophy, joint infection
are present. (which shows uptake of contrast to lymphatics), and capsulitis.
Tenography is the use of contrast media injected into a
MISCELLANEOUS IMAGING MODALITIES tendon sheath to visualize the course of the tendon it encloses.
Xeroradiography is a dry process that uses electrostatic It is useful for visualizing post-traumatic tendon injury.
forces to produce an image. Positive xeroradiographs are used Venography is the use of contrast media to visualize the
to visualize bone and non-metallic foreign-bodies. Negative course of a vein. It is the gold-standard for visualizing DVTs
xeroradiographs visualize soft tissue and metallic foreign but cannot be used in pts w/ an iodine allergy. Side effects
bodies. include allergic rxn, induced DVT, and localized skin
Digital Radiography is an X-ray method by which plain films necrosis.
are replaced with phosphor plates that transfer the image to a Arteriography is the use of contrast media to evaluate
digital signal. This process may produce very detailed images vascular supply. Its use is contraindicated in trauma,
and can zoom in or out to the tastes of the interpreter. athrosclerosis, AV malformations, emboli, or tumors.
Liquid Crystal Thermography produces an image dependent Angiography is visualization of the arterial supply using
on skin surface temperature. This is useful in diabetic computerized digial subtraction to remove non-vascular
neuropathy to predict the location of future ulcers, as well as structures from the picture. It is indicated in evaluation of
vascular and dermatologic pathologies. Digital atherosclerosis and AV malformation.
thermography is a similar process that involves recording an
image with a thermal camera.
Ultrasonography produces an image converted from high-
energy sound waves. Contrast is produced by variations in the
way that different tissues reflect sound. It is most useful for
distinguishing solid from fluid-filled masses, and in most
pathologies tissue alteration is significant enough for
ultrasound to detect.
Sinography is an X-ray taken after injecting a sinus with
radioopaque fluid. It is used to determine the size and extent
of a sinus tract and the location of potential abscesses.
Sinography cannot be used if the patient is allergic to iodine.
Local tissue necrosis and infection are other side effects.
Arthrography is radiographic imaging of a joint space using
injected contrast media. A single contrast arthrogram uses a
single contrast agent, and is used to determine the extent of
acute injury or ligamentous damage. Double-contrast
arthrograms utilize both positive and negative (air) contrast
media, and is used for chronic processes (such as OCD.)
Arthrograms may not be taken if the patient has cellulitis in
the area, and joint sepsis is a potential side effect.
A normal arthrogram of the ankle should show smooth
synovial lining with redundant ankle capsule, medial and
lateral recesses, smooth and uniform articular cartilage, and a
syndesmotic recess ~2.0cm to the interosseous membrane. In
15% of pts, there is a communication btw the ankle and
posterior STJ facet. In 15-20% there is a communication btw
the ankle and FHL/FDL sheaths.
Pathology seen in an arthrogram is typically due to lateral
ankle injury, and must be imaged w/in 48h of the incident.
Contrast leakage is usually indicative of ligamentous injury.
Injury of the ATF ligament shows extravasation anterior and
lateral to the fibula. The CF ligament is close to the peroneal
tendons and thus with CFL injury extravasation into these
tendon sheaths is seen. Chronic injury to the CFL will have a
persistent communication between these two structures. The
PTF ligament is rarely injured and is always seen with either
an ATFL or CFL rupture as well. Medial ankle injuries are
extremely uncommon. Syndesmotic tears are associated with
medial ankle ligament injury, and shows extravasation
superior to the syndesmotic recess.
CLINICAL ANTIBIOSIS IV: CEFOXITIN (MEROXIN), CEFUROXIME (ZINACEF), CEFOTETAN
Infection: Pathologic presence of bacteria in a wound or tissue site, (CEFOTAN)
numbering 106. It is clinically signified by inflammation, erythema, PO: CEFACLOR (CECLOR) 250-500mg TID, CEFUROXIME (CEFTIN),
pain, warmth, and loss of function. CEFPROZIL (CEFZIL)
History: Local signs of infection are edema, erythema, pain,
warmth, and loss of function. Systemic signs of infection include Third-Generation Cephalosporins
fever, chills, shaking, nausea, vomiting, loss of appetite, and More gram -, less gram +, fortaz also anti pseudomonal
shortness of breath. Get history of prior tx, PMH, allergies, social IV: CEFTRIAXONE (ROCEPHIN) 1-2g QD (long half-life)
Hx, travel Hx, and any pets the patient may have. Staph Epi is most Rocephin principal antibiotic in Lyme disease treatment
common in IV/catheter or implant infections. Staph aureus is the CEFTAZIDIME (FORTAZ): some antipseudomonal coverage
most common post-op infection and cause of osteomyelitis. PO: CEFDINIR (OMNICEF) 300mg BID, better staph coverage
D/Dx: Gout, DVT (r/o w/ venous Doppler/venogram), chronic CEFPODOXIME (VANTIN), CEFIXIME (SUPRAX)
venous insufficiency (bilateral pitting edema, hemosiderin
deposition), acute charcot, acute trauma, fungal infection, normal Fourth-Generation Cephalosporins
wound healing, post-surgical healing. Good gram + and gram -, antipseudomonal
Labs: WBC>10, left shift, elevated ESR, CRP. CEFEPIME (MAXIPIME) IV, 1-2g q12h
Hospital Admission: Indicated for osteomyelitis, large draining
wound, sustained fever (over 101F/38C), diabetes, CARBAPENEMS
immunocompromised state, gas present in tissues, failure of PO PRIMAXIN (IMIPENEM + CILASTATIN): PO 500mg q6h
antibiosis, lab results suggestive of sepsis. Gram +, gram -, anaerobes – cilastatin added to protect kidneys.
Expensive, save for life-threatening infections
PENICILLINS MEROPENEM (MERREM) some antipseudomonal coverage
Original Penicillins ERTAPENEM (INVANZ): IV, IM, 1g q24h
Not used often – usually for gonococcus, anaerobes Good against enterobacteria, but not pseudomonas
PEN G: IV or IM – 5-6 million U q4h AZTREONAM (AZACTAM): IV, 1-2g q8h
PEN VK: PO – 250-500mg QID, causes hypokalemia Only good against gram – anaerobes; use in combination

Aminopenicillins AMINOGLYCOSIDES
Good broad spectrum, but useless against staph. Staph, strep, gram -, but not anaerobes
AMPICILLIN: IM, IV, PO – 250-500mg QID or 2g q4h ADRs: reversible nephrotoxicity, irreversible ototoxicity,
AMOXICILLIN: PO only – 250-500mg QID Neuromuscular blockade if infused too quickly
GENTAMYCIN, TOBRAMYCIN, AMIKACIN
Semisynthetic Loading doses: Gentamycin/Tobramycin 2mg/kg
PCNnase resistant, good vs. staph, used in specific situations Amikacin 7.5mg/kg
NAFCILLIN: IM or IV – 1-2g q4-6h, metabolized in liver Maintenance: Gent/Tobra 3-5mg/kg/day (1-3 for children)
DICLOXACILLIN: PO – 250-500mg QID Amikacin 15mg/kg/day
Peak/Trough: Gent&Tobra 10 peak, 2 trough µg/ml,
Uriedopenicillins (Expanded Spectrum) Amikacin 20-30 peak, 10 trough µg/ml
Active against pseudomonas, resistance is common Trough level: Gent/Tobra 2µg/ml, Amikacin 10µg/ml
CARBENICILLIN: Limited use in lower extremity (Peak = immediately after dosing, Trough = 20-30 minutes before
TICARCILLIN: IV – 3-4g q4h, high in sodium dosing)
PIPERICILLIN: Limited use in lower extremity
FLOUROQUINOLONES
Beta-Lactamase Inhibitors CIPROFLOXACIN (CIPRO): PO 500/750mg, IV 400mg BID
First choice antibiotics w/ cephalosporins Good gram – coverage, antipseudomonal
Staph, Strep, Anaerobes, Gram - coverage Not for peds, Sx prophylaxis, or weak/tenotomized tendons
TIMENTIN (TICARCILLIN + 100MG CLAVULANATE) LEVOFLOXACIN (LEVAQUIN): PO or IV 500/750mg BID
IV – 3.1g q6-8h Good for staph and strep, not as strong antipseudomonal
Empiric for DM foot infections, bites MOXIFLOXACIN (AVELOX): PO or IV 400mg QD
AUGMENTIN (AMOXICILLIN + 125MG CLAVULANATE) Good staph, strep, and antipseudomonal
PO – 250/500/875 BID GATIFLOXACIN (TEQUIN): Not for foot infections, lengthens QT
Good for outpatient DM, bites TROVAFLOXACIN (TROVAN): Taken off market for hepatic failure
UNASYN (1 PART AMPICILLIN + ½ PART SULBACTAM)
IV – 3g loading dose, 1.5g following doses SULFONAMIDES
Better at gram + but worse for gram – than timentin BACTRIM/SEPTRA (TRIMETHOPRIM + SULFAMETHOXAZOLE): PO only
ZOSYN (PIPERICILLIN + TAZOBACTAM) - QD dosing, double-strength (DS) 160mg TMX, 800mg
IV – 4.5g q8h sulfamethoxazole
Better against enterococci Broadest possible spectrum, not antipseudomonal
Allergies common
CEPHALOSPORINS
First Generation Cephalosporins MACROLIDES
Good for gram +, most common pre-op prophylaxis ERYTHROMYCIN: PO 250-500mg QD, IV 1g QD
CEFAZOLIN (ANCEF): IV or IM, 1g q8h Good against staph, gram + and gram – anaerobes
CEFALEXIN (KEFLEX): PO, 250-500mg BID/QID AZITHROMYCIN (ZITHROMAX): PO 500mg QD day 1, 250mg QD days
CEFADROXIL (DURICEF): PO, 500mg q12h 2-4. Postbiotic effect for 10 days following dosing.
Gram +, some Gram – organisms, usually for pts who are allergic to
Second-Generation Cephalosporins other antibiotics, or peds (Paronychia)
Used mainly for ear infections, pneumonia – not podiatry CLARITHROMYCIN (BIAXIN): Not often used for foot infections
TETRACYCLINES LOWER EXTREMITY MICOBIOLOGY REVIEW
TETRACYCLINE, DOXYCYLINE, MINOCYCLINE GRAM + COCCI
Limited use in podiatry – mainly used for acne, Lyme disease Staph Aureus – Coagulase +
Incidence: Normal flora, common infection, high resistance
CHLORAMPHENICOL ABx: 1st Cephalosporins, PCNase resistant PCN, Cleocin, Bactrim,
Mainly a historical footnote; rarely used anymore Erythromycin, Vanco (resistant), Cipro (resistant)
Staph Epidermidis/Saprophyticus – Coagulase –
ANTI-ANAEROBIC MISCELLANY Incidence: Normal flora – Epi seen in implant sx, sap in UTI
METRONIDAZOLE (FLAGYL): PO or IV, 500mg TID ABx: Same as S. Aureus
Mainly gram – but some gram +, amebiasis, colorectal Sx Strep Pyogenes – Group A, Strep Agalactiae – Group B
Used also to treat pseudomembranous colitis (see below) Incidence: Pyogenes usually superficial, Agalactiae seen in DM
CLINDAMYCIN (CLEOCIN): PO 150-300mg BID, IV or IM 600-900mg ABx: PCNs, 1st gen Cephalosporins, Cleocin, Vanco
q8h. Good bone penetrance, good for anaerobes Strep Faecalis/Faecium – Group D/Enterococci
Can cause pseudomembranous colitis Incidence: GI flora, highly resistant, ST infection component
ABx: Gentamycin + PCN/Ampicillin/Amoxicillin or Vanco
ANTI-GRAM + MISCELLANY Peptostreptococcus/Peptococcus – Anaerobic
VANCOMYCIN (VANCOCIN): PO 125mg QD (only for c. difficile), IV Incidence: DM foot infections
1g q12h – infuse slowly Good for all gram + except VRSA and VRE. ABx: PCN, 1st gen Cephalosporins, Cleocin, Erythromycin
Peak – 20-500µg/ml, Trough - 10µg/ml, like aminoglycosides
(Trough is more important to determine efficacy of treatment) GRAM + BACILLI
ADRs: nephrotoxicity, ototoxicity, red man syndrome (rash) Clostridium Tetani – Anaerobic, Spore-forming
Can be used for prophylaxis if PCN, clindamycin allergic Incidence: Ubiquitous in environment, puncture wounds
SYNERCID (QUINUPRISTIN + DALFOPRISTIN): IV 7.5mg/kg q12h Used ABx: PCNs (mainly useless b/c of neurotoxin production)
for VRE Clostridium Perfringens – Anaerobic
LINEZOLID (ZYVOX): PO, IV 600mg BID Incidence: Fast growing, gas gangrene (necrotizing fasciitis)
Used for MRSA, all gram +. Can cause thrombocytopenia ABx: Sx debridement indicated – PCN, Clinda, Imipenem
RIFAMPIN: PO 300mg BID Good for resistant staph and strep Corynebacterium – diptheroid
Causes rash, orange discoloration of all body fluids. Incidence: Nosocomial, Immunocompromised infections
ABx: Clinda, Erythromycin, Vanco
ADVERSE REACTIONS
-Pseudomembranous Colitis – Clindamycin, cephalosporins, GRAM – COCCI
uriedopenicillins: Tx w/ metronidazole, oral vancomycin Neisseria Gonorrhoeae
-Tendon rupture, cartilage degeneration – Ciprofloxacin Incidence: Major cause of septic arthritis is LE, resistant
-Ototoxicity (irreversible), nephrotoxicity (reversible), neuromuscular ABx: Rocephin, Cipro (resistant)
blockade – Gentamycin, Tobramycin, Amikacin
-Ototoxicity, nephrotoxicity, red man synd. – Vancomycin GRAM – RODS (ENTERIC)
-Hypokalemia – Pen G, Pen VK: Tx w/ K-exelate Bacteroides
-Thrombocytopenia, bone marrow suppression – Linezolid Incidence: Most common in DM infection, resistant
-Rash, orange discoloration of body fluid – Rifampin ABx: Flagyl, primaxin, clinda, 3rd or 4th gen cephalosporins
-QT interval lengthening – Tequin Enterobacter/Citrobacter/Morganella/Serratia
Incidence: Nosocomial infections, elderly
ANTI-PSEUDOMONAL DRUGS ABx: 3rd gen cephalosporins, cipro, bactrim, aminoglycosides
Penicillins: Ticarcillin (weak), Timentin (weak) Escherichia Coli
Cephalosporins: Fortaz (weak), Maxipime (strong) Incidence: Common in LE infections
Carbapenems: Meropenem (weak) ABx: Any cephalosporin, ampicillin, cipro, bactrim
Quinolones: Cipro (strong), Levaquin (weak), Avelox (strong) Proteus/Providencia
Aminoglycosides: Tobramycin (strong) Incidence: Normal flora, common in interdigital infections
ABx: Cephalosporins, Ampicillin, Cipro, Bactrim
EMPIRIC TREATMENT OPTIONS
Cellulitis: Unasyn, Zosyn, Timentin, Maxipime, Invanz, Avelox, OTHER GRAM – RODS
Cipro, Levaquin, Bactrim. Aeromonas Hydrophilia
Open fractures: Gustillo & Anderson Type I or II – Ancef Incidence: Injuries sustained under water (fresh water)
Gustillo & Anderson type III (any) – add Gentamycin ABx: Cipro, Bactrim, Primaxin, Aminoglycosides
If farm injury/wound contaminated with dirt, add Pen G Haemophilus Influenzae
Incidence: Most common in children, nosocomial infections
SURGICAL PROPHYLAXIS ABx: 3rd/4th gen cephalosporins, bactrim, ampicillin
Indications: Prolonged surgery, immunocompromise, trauma, Pseudomonas Aeruginosa
implant surgery. Incidence: Ubiquitous; puncture wounds, underwater injuries most
Most Common: Ancef, Rocephin, Vancomycin, Clindamycin common.
Never Used: Quinolones ABx: See left
Administration: IV, ½h prior to surgery (usually done in OR) Eikenella Corrodens
Incidence: Found in human bite wounds
ANTIBIOTIC TIMELINE ABx: PCN, Ampicillin, Macrolides, Imipenems
Soft tissue infection only: 10 days – 2 weeks Pasturella Multicida
Osteomyelitis: 4-6 weeks Incidence: Cat and other animal bite wounds
ABx: PCN, Ampicillin, β-lactamase inhibitors, bactrim.
LABS GLUCOSE (Glu)
Normal Range = 60-110 mg/dL
CHEMISTRY PANEL/BASIC METABOLIC PROFILE Glucose is closely regulated so as to provide energy to all cells in the body
without allowing any spillover into the urine. The excess of glucose seen in
Na+ Cl- BUN DM is the underlying cause for all complications of that disease.
Increased in: DM, cushing’s syndrome, chronic pancreatitis, corticosteroids,
Glu
phenytoin, estrogens, thiazides.
K +
HCO3 -
Creat Decreased in: Insulinoma, adrenocortical insufficiency, hypopituitarism, liver
disease, malignancy, insulin, ethanol, propanolol, solfonylureas, tolbutamide.

BMP CALCIUM (Ca2+)


SODIUM (Na+) Normal Range = 8.5-10.5 mg/dL
Normal Range = 135-145 mEq/L Calcium is important in bone mineralization as well as a clotting cofactor. It
Sodium is the predominant extracellular cation. Levels of sodium generally is regulated by calcitonin and parathyroid hormone, and binds to albumin in
reflect the hydration state of the patient. serum.
Increased in: Dehydration, polyuria, hyperaldosteronism, steroids, oral Increased in: Hyperparathyroidism, vitamin D excess, multiple myeloma,
contraceptives, inadequate water intake. paget’s disease, sarcoidosis, vitamin A intoxification, addison’s disease,
Decreased in: CHF, vomiting, diarrhea, sweating, nephropathy, adrenal antacids, thiazides, lithium
insufficiency, thiazides, diuretics, ACE inhibitors, antidepressants, Decreased in: Hypoparathyroidism, vitamin D deficiency, renal insufficiency,
antipsychotics. magnesium/phosphorus deficiency, massive transfusion, hypoalbuminemia.

POTASSIUM (K+) ANION GAP


Normal Range = 3.5-5.0 mEq/L Normal Range = (Na+K) - (Cl+HCO3)<16, or Na - (Cl+HCO3)<12
Potassium is the primary intracellular cation and is the electrical balance to Anion gap is an equation used to measure metabolic acidosis. Anions and
sodium. Potassium imbalance causes disruption in the function of neurons cations in the body sum to zero; anion gap is a measure of the sum of all
and muscles, and the symptoms of DKA are a result of the hypokalemia it minor anions in the body. Increased anion gap indicates acidosis, whose
produces. Potassium movement also accompanies glucose, and in the kidneys etiology is characterized by the acronym SLUMPED; Salicylates, Lactic acid,
potassium is excreted at the expense of sodium. Potassium levels in the blood Uremia, Methanol, Paraldehyde, Ethanol, and Diabetic ketoacidosis.
are affected chiefly by pH; acidosis is accompanied by hyperkalemia, and
alkalosis causes hypokalemia. The action of catecholamines on beta receptors PHOSPHORUS (P+)
has a secondary effect of reducing blood potassium. Normal Range = 2.5-4.5 mg/dL
Increased in: Hemolysis, tissue damage, acidosis, renal failure, addison’s Phosphorus is the other mineral component to bone, and its levels in the blood
disease, exercise, potassium-sparing diuretics, NSAIDs, β-blockers, ACE are regulated chiefly by the parathyroid gland and vitamin D.
inhibitors, and TMX. Increased in: Renal failure, massive blood transfusion, hypoparathyroidism,
Decreased in: Prolonged vomiting/diarrhea, hyperaldosteronism, cushing’s sarcoidosis, neoplasms, adrenal insufficiency, acromegaly, hypervitaminosis
syndrome, osmotic diuresis, trauma, subarachnoid hemorrhage, adrenergic D, leukemia, DKA, cirrhosis, respiratory acidosis, anabolic steroids,
drugs, diuretics. furosemide, clonidine, verapamil, HCTZ.
Decreased in: Hyperparathyroidism, hypovitaminosis D, steatorrhea,
CHLORIDE (Cl-) malnutrition, bone marrow transplantation, alcoholism, diarrhea, vomiting,
Normal Range = 98-107 mEq/L gout, osteoblastic disorders, respiratory alkalosis, pregnancy, hypothyroidism,
Chloride is a secondary player in the maintenance of acid/base balance and is dialysis, anticonvulsants, β-agonists, catecholamines, estrogens, insulin, ASA.
a placeholder in the anion gap. Retention causes acidosis, loss alkalosis.
Increased in: Massive diarrhea, nephrotic syndrome, renal failure, diabetes ALBUMIN
insipidus, hyperparathyroidism, acetazolamide, androgens, salicylates. Normal Range = 3.4-4.7 g/dL
Decreased in: Vomiting, diarrhea, GI suction, DKA, respiratory acidosis, Albumin is the principal carrier protein in the blood and is a measure of
excessive sweating, acute intermittent porphyria, adrenal insufficiency, nutrition. Many hormones depend on albumin for transport in the blood.
laxative use, corticosteroids, diuretics. Increased in: Dehydration, shock.
Decreased in: Liver disease, malnutrition, malabsorbtion, malignancy,
BICARBONATE (HCO3-) nephrotic syndrome, hemorrhage, fistulas, estrogens.
Normal Range = 22-28 mEq/L
Bicarbonate is the main component of the body’s buffer system and is the BILIRUBIN (Bili)
central player in regulation of blood pH. Bicarbonate levels are regulated by Normal Range = 0.1-0.4 (direct), 0.2-0.7 (indirect) mg/dL
the kidneys. Bilirubin is the product of heme metabolism, and is conjugated (direct bili) in
Increased in: metabolic alkalosis, compensated respiratory acidosis, volume the liver for biliary excretion. Direct vs. indirect values predict etiology.
contraction, mineralocorticoid excess, diuretics. Increased in: Hemolysis (indirect), liver failure (indirect), hemorrhage
Decreased in: Metabolic acidosis, compensated respiratory alkalosis, (indirect), biliary obstruction (direct), cirrhosis/hepatitis (both), CHF (both)
fanconi’s synd, volume overload, acetazolamide.
ALKALINE PHOSPHATASE (Alk Phos)
BLOOD UREA NITROGEN (BUN) Normal Range = 41-133 IU/L
Normal Range = 8-20 mg/dL Alk phos is produced by osteoblasts and is an indication of skeletal turnover.
BUN is the end product of protein metabolism, and is directly related to the Increased in: Biliary obstruction, paget’s disease, bone tumor,
amount of protein ingested. It is a relative indicator of kidney function. hyperparathyroidism, rickets, pregnancy, GI disease, hepatotoxic drugs.
Increased in: Renal failure, urinary tract obstruction, degydration, shock,
burns, CHF, GI bleed, nephrotoxic drugs. TOTAL PROTEIN
Decreased in: Hepatic failure, nephritic syndrome, cachexia. Normal Range = 6.0-8.0 g/dL
A group that includes albumin, total protein is an indicator of nutrition and is
CREATININE (Creat) the principal determinant in blood oncotic pressure.
Normal Range = 0.6-1.2 mg/dL Increased in: Gammopathies, dehydration, anabolic steroids, corticosteroids,
Creatinine is a normal product of the breakdown of muscle tissue, and is epinephrine, androgens.
produced at a constant rate. As such, it is an absolute indicator of kidney Decreased in: Burns, nephrotic syndrome, malnutrition, chronic liver disease.
function, and the ratio of BUN to Creat in the blood can give clues to the
etiology of a kidney problem. Increase in BUN and Creat is termed azotemia. ALANINE AMINOTRANSFERASE (ALT, SGPT) Normal Range = 0-40 U/L
10:1 ratio is pre-renal azotemia, caused by insufficient renal blood flow. A ASPARTATE AMINOTRANSFERASE (AST, SGOT) Normal Range = 0-35 U/L
less than 10:1 ratio is renal azotemia, which is indicative of renal failure. These liver function enzymes are an indicator of liver damage or pathology.
Increased in: Renal failure, urinary tract obstruction, nephrotoxic drugs, Increased in: Hepatitis, cirrhosis, liver abscess, biliary obstruction, right-sided
hypothyroidism. heart failure, ischemia/hypoxia, abdominal trauma, hepatotoxic drugs.
Decreased in: Reduced muscle mass. Decreased in: Vitamin B6 deficiency
COMPLETE BLOOD COUNT Hb PROTHROMBIN TIME (PT)
Normal Range = 11-15 sec
WHITE BLOOD CELLS (WBC) PT is the monitor for the extrinsic clotting cascade.
Normal Range = 5,000-10,000/µL WBC Plt Increased in: Coumadin, liver dz, vit K deficiency, DIC, massive transfusion.
Increases in the white count are normally attributed
to infection, however many factors may affect it. PARTIAL THROMBOPLASTIN TIME (PTT)
Increased in: Infection, inflammation, leukemia, Hct Normal Range = 25-35 sec
lymphoma, corticosteroids. PTT is the monitor for the intrinsic clotting cascade.
Decreased in: Aplastic anemia, B12/folate deficiency, sepsis, phenothiazines, Increased in: Heparin, coumadin, clotting factor deficiency (except XIII &
chloramphenicol, aminopyrine. VII), von willebrand dz, DIC, hemophilia.
NEUTROPHILS (Segs) Normal Percentage = 40-60%
Increased in: Infection, stress, inflammation, leukemia, DKA. LACTATE DEHYDROGENASE (LDH)/CREATINE PHOSPHOKINASE (CPK)
Decreased in: Same as WBC count. Normal Range = 88-230 U/L Normal Range = 32-267 U/L
MYELOCYTES (Bands) Normal Percentage = 0-5% LDH is an important enzyme used in carbohydrate metabolism. It is used to
Increased in: Acute infection, toxemia, hemorrhage, pernicious anemia. detect MI – it rises 3-6h post MI and remains high for approximately 1 week.
>20% is called “left shift” and usually signals acute infection, even if the CPK is a muscle enzyme and is released in muscle turnover or muscle
WBC count is normal. damage. It is also used as a marker to detect MI – levels rise immediately
LYMPHOCYTES (Lymph) Normal Percentage = 20-40% after MI and remain high for 2-3 days.
Increased in: Viral infection, thyrotoxicosis, adrenal insufficiency, ALL,
CLL, chronic infection, allergic rxn, autoimmune dz, epinephrine. URIC ACID
Decreased in: HIV, SLE. Normal Range = 2.4-7.4 (♂), 1.4-5.8 (♀) mg/dL
MONOCYTES (Mono) Normal Percentage = 4-8% Uric acid is the end product of purine metabolism. Excess results in gout.
Increased in: Chronic bacterial infection, inflammation, malignancy, Increased in: Gout, renal failure, leukemia, multiple myeloma, lesch-nyhan
TB, multiple myeloma. synd, lead poisoning, pregnancy, menopause, diuretics, ethanol, salicylates.
EOSINOPHILS (Eos) Normal Percentage = 1-3% Decreased in: Xanthine oxidase deficiency, fanconi synd, neoplasms, liver
Increased in: Allergic rxn, drug sensitivity, skin disorders, parasite disease, high-dose salicylates, allopurinol.
infection, hodgkin’s dz, pulmonary infiltrative dz, menstruation.
Decreased in: Inflammation, stress, corticosteroids. Urinalysis
BASOPHILS (Baso) Normal Percentage = 0-1% SPECIFIC GRAVITY
Increased in: Allergic rxn, multiple myeloma, colitis, myxedema. Normal Range = 1.001-1.035
Increased in: DM, dehydration, sweating, vomiting, diarrhea
HEMOGLOBIN (Hb) Decreased in: ↑ fluids, ↓ Na diet, nephritis, diabetes insipidus, aldosteronism
Normal Range = 14-18 (♂), 12-16 (♀) g/dl (Hb = 1/3 Hct)
Central component to O2 transport in blood, required for adequate healing. PH
Increased in: Dehydration, polycythemia, exercise conditioning, high altitude. Normal Range = 5-9
Decreased in: Macrocytic anemia, Normocytic anemia, Microcytic anemia. Acidic: Acetones, DM, gout, nephritis, leukemia, scurvy, saccharin, aspirin
Alkaline: Blood transfusion, vomiting
HEMATOCRIT (Hct)
Normal Percentage = 40-50% (♂), 35-45% (♀) (Hct = RBC x MCV) GLUCOSE
Is a measure of the percentage of whole blood made up of erythrocytes. Normal Range = <300 mg/day
Increased in: Dehydration, polycythemia, exercise conditioning, high altitude. Increased in: DM, Aspirin, hyperpituitarism, hyperthyroidism, chronic liver
Decreased in: Macrocytic, normocytic, microcytic anemias. failure, pregnancy

PLATELETS (Plt) KETONES


Normal Range = 150,000-450,000/µL Normal Range = 10-20 (♂), 5-15 (♀) mg/day
Platelets are required for the cellular component of blood clotting. Increased in: DKA, PKA, starvation
Increased in: Myeloproliferative disorders, bleeding, splenectomy, iron
deficiency, malignancy, alkalosis. PROTEIN
Decreased in: Bone marrow suppression, ethanol, splenomegaly, DIC, Normal Range = 10-200 mg/day
purpura, quinidine, cephalosporins, clopidogrel, heparin. Increased in: Multiple myeloma (bence-jones protein), UTI, hemorrhage,
prostatic secretions, DM, lupus, nephrotic syndrome.
RED BLOOD CELLS (RBC)
Normal Range = 4.8-6.0 (♂), 4.1-5.5 (♀) x106/µL -BILIRUBIN
Increased in: Polycythemia. Presence indicates hepatitis (unconjugated) or biliary tract obstruction (conj.)
Decreased in: Anemia. -UROBILINOGEN
Presence indicates intravascular hemolysis
MEAN CORPUSCULAR VOLUME (MCV) -BLOOD (Hematuria), NITRATES, LEUKOCYTE ESTERASE
Normal Range = 80-100 fL Suggestive of UTI
Increased in: Liver dz, alcoholism, hemochromatosis, megaloblastic anemia, -CASTS
reticulocytosis, methotrexate, phenytoin, zidovudine. Hyaline casts may be normal. RBC/WBC casts suggest pyelonephritis.
Decreased in: Iron deficiency, thalassemias, chronic dz anemia.
Other Tests
MEAN CORPUSCULAR HEMOGLOBIN (MCH) HEMOGLOBIN A1C (HbA1c)
Normal Range = 26-34 pg Normal Range = 4-7% of total hemoglobin
Increased in: Macrocytosis, hemochromatosis. Increased in DM. The equation (33.3 x HbA1c – 27) estimates daily FBS
Decreased in: Iron deficiency, thalassemia, lead poisoning, chronic dz anemia over the last month.

MEAN CORPUSCULAR HEMOGLOBIN CONCENTRATION (MCHC) HUMAN CHORIONIC GONADOTROPIN (HCG/β-HCG)


Normal Range = 31-36 g/dL (MCHC = Hb/(MCV x RBC)) Presence indicates pregnancy or trophoblastic tumor. (Female only)
Increased in: Spherocytosis, hemolysis, lipemia, cellular dehydration.
Decreased in: Hypochromic anemias, sideroblastic anemia, chronic dz anemia RAPID PLASMA REAGIN (RPR)
Presence usually indicates syphilis, but sometimes seen in leprosy, malaria, IV
ERYTHROCYTE SEDIMENTATION RATE (ESR) drug use, the elderly, mono, HIV, autoimmune disease, and pregnancy.
Normal Range = <10 (♂), <15 (♀) mm/h
Increased in: Infection (esp. osteomyelitis), inflammatory dz, malignancy, SICKLE DEX
anemia, pregnancy, chronic renal failure, GI dz, endocarditis. Positive result indicates sickle cell anemia or trait; f/u w/ electrophoresis.
Decreased in: Polycythemia, sickle-cell anemia, spherocytosis, CHF,
hypogammaglobulinemia, high-dose corticosteroids.
PAIN MANAGEMENT * - Indicates avalibility of pediatric dosing
1
Pain Classification: Nociceptive – normal pain response seen – Indicates better soft tissue penetration
in post-traumatic or post-surgical patients. NARCOTIC ANALGESICS
Neuropathic – pathologic pain resulting from nerve lesions, MORPHINE2 – 1-3mg IM q10minutes (or w/ PCA)
neuropathy, entrapment, CRPS, or phantom pain. HYDROMORPHONE (Dilaudid) 2 – 0.1-0.5µg IM q10minutes
Acute pain is managed with analgesics and NSAIDs. MEPERIDINE (Demerol) 2 – 10-30mg IM q1-4h
Chronic pain is managed with NSAIDs and steroid injections. PENTAZOCINE (Talwin) 4 – 30mg IV/IM q4h
Nerve Fiber Types: Aδ fibers – slow, unmyelinated fibers, OXYCODONE (Oxycontin) 2 – 5mg PO q6h, 10-40mg PO q12h
respond to mechanical injury, intense burns/freezing. OXYCODONE w/ Acetaminophen (Percoset)2 – 2.5/325,
C fibers – slow, thinly myelinated, respond to chemical, 5/325, 7.5/500, 10/500mg PO q4-6h
thermal, deep mechanical, “itch” type injuries. OXYCODONE w/ Aspirin (Percodan) 2 – 2.5/325, 5/325mg PO
Spinal Tracts: Spinothalamic tracts to substantia gelatinosa CODEINE w/ Acetaminophen (Tylenol #1-5)3 – 7.5/300,
(releasing substance P, enkephalins, etc.), dorsal 15/300, 30/300, 60/300, 120/300mg PO q4-6h
column/medial lemniscus pathway, anterolateral tracts HYDROCODONE w/ Acetaminophen (Vicodin, Lortab,
(spinoreticular, spinotectal.) Zydone) 3 – 5/500, 7.5/750, 10/660mg PO q4-6h
Clinical Pain Types: HYDROCODONE w/ Ibuprofen (Vicoprofen) 3 – 7.5/200mg PO
Throbbing, dull ache = musculoskeletal pain (esp. w/ activity.) q4-6h
Stabbing, soreness = fracture PROPOXYPHENE w/ Acetaminophen (Darvocet)4 – 50/325,
Burning, tingling, numbness (paresthesia) = neuropathic pain 100/500, 100/650mg PO q4-6h
-Tinel’s sign: Pain radiates proximally w/ percussion PENTAZOCINE w/ Acetaminophen (Talacen) 4 – 25/650mg
-Valleix sign: Pain radiates prox & dist w/ percussion PO q4h (mixed opioid agonist/antagonist)
Itching, superficial burning = skin/epidermal pain
Burning, cramping, tearing = ischemic pain/claudication NON-NARCOTIC ANALGESICS
ACETAMINOPHEN (Tylenol) – 325-1000mg PO q4-6h
NSAIDS Analgesic and antipyretic only- watch for hepatotoxicity
Use carefully w/ bleeding disorders, asthma, pregnancy, peds TRAMADOL (Ultram) – 50, 100mg PO q4-6h
Salicylates TRAMADOL w/ Acetaminophen (Ultracet) – 37.5/325mg PO
ASPIRIN (ASA, Bayer, Ecotrin) – 325-1000mg PO q6h q4-6h
*DIFLUNISAL (Dolobid) – 250, 500mg PO q12h LIDOCAINE (Lidoderm) – 5% patch (transdermal), 1-3 q12h

Propionic Acids INJECTABLE CORTICOSTEROIDS


*IBUPROFEN (Motrin, Advil, Nuprin) – 200, 400mg PO bid/tid Contraindications: Infection, joint instability, AVN,
KETOPROFEN (Oruvail, Orudis) – 25, 50, 75mg PO bid/tid osteonecrosis, neurotrophic pathology, fungal infection.
FLURBIPROFEN (Ansaid) – 50, 100mg PO bid/tid Short-Acting (t½ = 8-12h)
*NAPROXEN (Naprosyn, Aleve) 1 – 250, 375, 500mg PO bid HYDROCORTISONE (Solu-Cortef) Potency: 1† Dose: 20mg
Do not use w/ H2 blockers, Propanolol HYDROCORTISONE VALERATE (West Cort) same as above
NAPROXEN SODIUM (Anaprox) – 275, 550mg PO bid CORTISONE (Cortone) Potency: 0.8 Dose: 25mg
Longer t½ than regular naproxen PREDNISONE (Delta-Cortef) Potency: 4 Dose: 5mg

Indoles Intermediate-Acting (t½ = 12-36h)


*INDOMETHACIN (Indocin) – 25, 50mg PO bid/tid TRIAMCINOLONE ACETONIDE (Aristospan, Kenalog)
Often used for management of gout Potency: 4 Dose: 5mg
SULINDAC (Clinoril)1 – 150, 200mg PO bid TRIAMCINOLONE DIACETATE (Aristocort) Same as above
KETOROLAC (Toradol) – 15, 30mg IM/IV, 10mg PO q4-6h PREDNISOLONE ACETATE Potency: 4 Dose: 4mg
For acute pain only; do not use for >5 days, to max of 60mg METHYLPREDNISOLONE (Solu-Medrol) Potency: 5 Dose:
ETODOLAC (Lodine) 1 – 200, 400mg PO bid/tid, 4mg
-ER (extended release) 400-1200mg PO qid
PIROXICAM (Feldene) 1 – 10, 20mg PO bid Long-Acting (t½ = 36-54h)
DEXAMETHASONE (Decadron) Potency: 25 Dose: 0.75mg
Phenylacetic Acid BETAMETHASONE SODIUM PHOSPHATE (Solu-Span)
DICLOFENAC (Voltaren) – 25, 50, 75mg PO bid, Potency: 25 Dose: 0.6mg
-ER 100mg PO qid BETAMETHASONE ACETATE (Celestone) Same as above

COX2 Inhibitors Pedal Injection Doses


CELEBREX (Celecoxib) – 100, 200, 400mg PO bid Location Corticosteroid Anesthetic Total
Cross-reactive w/ sulfonamide hypersensitivity PIPJ/lesser MPJ 0.25-0.5cc 0.25cc 0.5cc
VALDECOXIB (Bextra) – 10, 20mg PO bid 1st MPJ/STJ 0.5cc 0.5cc 1cc
MIROXICAM (Mobic) – 7.5, 15mg PO qid Neuroma/Heel 0.5-1.5cc 0.5-1.5cc 1-3cc
Achilles Bursa 0.5-1cc 0.5-1cc 1-2cc
2
– Indicates schedule II narcotic (high chance of abuse)
3
– Indicates schedule III narcotic (low chance of abuse)
4
– Indicates schedule IV narcotic (minimal chance of abuse)
† - Corticosteroid potency is calculated in comparison to hydrocortisone
Resect condyles from phalangeal
SURGICAL DISSECTION head, cartilage from joint space
Open intramedullary canal distally
1. ARTHROPLASTY- formation of moveable joint (for HDS) and shape peg proximally
Def- resection of p.p. (Keller-removal of base; Mayo- K-wire/polymeric absorbable pins
resection of 1/3 of MT head for implant; Anatomy- NV bundle
Stone is resection of 2/3 of MT head) Post-Op- SxS 6 wks, K-wire out after 3 wks
Indications-HDS where deforming force has been
removed, DJD, loss o joint cartilage 4. HELOMA MOLLE (soft corn) commonly assoc w/ under
Contra- deforming force is still present (arthroplasty lapping fifth toe or w/ ADDuctus deformity of 5th MPJ
will leave a flexible joint that will eventually deform) Def- 4th interspace HPK
Procedure- Incision (elliptical located dorsally Ind- Interspace HPK due to bony prominence
over PIPJ/DIPJ) Contra- dermatologic conditions at incision site
Excise as much corn as possible Procedure- Linear incision dorsolateral to
Free EDL and retract EDL
Incise joint Lateral condylectomy of 4th p.p.
Free collaterals base (hemiphalangectomy)
Head osteotomy at Sx neck Incision lateral elliptical to expose
Place K-wire head of 5th proximal phalanx
Tuck and suture EDL Arthroplasty of 5th p.p.
Anatomy- NV bundle on either side of digit Anatomy- dorsal common digital n. of intermediate
Post-Op- WB SxS 2-3 wks, k-wire 2-3 wks dorsal cutaneous n.
Complications: 1. Sausage toe- from increased Post-Op- WB SxS for 2-3 wks
swelling + fibrosis. (tx w/ acetate Complications- Flail toe 5th, Destabilization of 4th
steroid inj (≤ .1 cc), coban wrap) MPJ, Inadequate bone resection (recurrence)
2. Flail toe- from increased head
resection or base resection. (tx w/ 5. DIGITI QUINTI VARUS
syndactyly, implant, graft) Procedure- Elliptical incision perpendicular to axis of
3. White Toe; check dressing, deformity (derotational skin plasty), arthroplasty of 5th
dangle leg, bend k-wire, reflex Post-Op- WB SxS 2-3 wks
heat, local anesthetic block, nitro Complications- under correction, flail toe, recurrence
patch
4. Recurrence 6. HALLUX IPJ FUSION
5. Metatarsalgia Indications- DJD, Hallux HDS, assoc w/ hallux varus
6. Floating toe repair, jones tenosuspension
7. Hourglass appearance Procedure- Transverse elliptical incision on
8. Brachymetatarsia dorsal IPJ, release capsule and
9. Hallux Malleus supporting lig. Resect articular
surface of hallux IPJ (w/ rongeur)
2. END-TO-END ARTHRODESIS (OR “V”) Use 4.0 partially threaded
Def- fusion of joint (fusion of 1st MPJ is McKeever) cancellous screw
Ind- HDS where intrinsic muscle function has been Post-Op- BK NWB cast w/ toe plate for 6-8 wks,
compromised & digital/MPJ stability is lacking, k-wire 6-8 wks or until X-ray healing
provides stable lever arm on which the long and short Contra- poor bone stock or medical problems
flexors can function to assist MPJ stability Complications- infection, fixation failure, malposition
Contra- rigid 5th toe does not tolerate shoe pressure
Procedure- Long lenticular incision 7. VALENTI ARTHROPLASTY
Free EDL and retract Indications- Hallux limitus stage II/III, 1st MPJ DJD
Incise joint Contra- PF 1st ray
Free collaterals Procedure- Incision dorsomedial 1st MPJ
Resect 1 mm cartilage from Capsulotomy + cheilectomy
proximal and middle phalanx 45º osteotomy of 1st MT head,
K-wire down axis of digit dorsal proximal to plantar distal
Suture EDL 45º osteotomy of p.p, ┴ to 1st
Anatomy- NV bundle Free sesamoids, close
Post –Op- SxS, K-wire 6-8 wks, WB to tolerance Post-Op- SxS 3-6 wks, immediate ROM exercises
Comp- same ones as arthroplasty + “long” toe, Comp- metatarsalgia, sesamoiditis, loss of stability
immobile joint
8. AKIN OSTEOTOMY
3. PEG-IN-HOLE ARTHRODESIS Distal akin- for high HI (normal ≤ 10˚)
Def- snug fit can be used w/o fixation Proximal akin- for high DASA (normal ≤ 8˚)
Ind- HDS long MT for maximum stability, suspected Oblique akin-facilitates screw fixation, will correct
non-union pre-op, Inability to use K-wire some of HI & DASA
Contra- short toe pre-op Cylindrical- for long phalanx
Procedure- Linear incision Contra- poor bone stock, medical conditions
Free EDL and retract Post- Op- WB or NWB SxS 3-6 wks, K-wire 3-6 wks
Incise joint or loose
Free collaterals Comp- hallux ADDuctus, hinge Fx, fixation failure,
FHL/ FHB severed, tendonitis, IPJ/ MPJ stiffness
HALLUX ABDUCTO-VALGUS HEAD PROCEDURES
-progressive, triplanar deformity w/ frontal plane hallux rotation, 12. REVERDIN (1881)
lateral deviation, & dorsal or plantar deviation Def- medial based incomplete wedge resection of MT head
-Goals 1. Pain free ROM of 1st MPJ Ind- abnormal PASA w/ flexible 1st ray
2. Restore normal osseous angles (PASA + IM) Procedure Two convergent cuts in metaphyseal portion
3. 1st MPJ should be congruent of 1st MT head
4. TSP ≤ 3 (4 or greater = crista erosion) Distal cut is orientated parallel to the
5. Cosmesis (bump should be gone) articular cartilage, proximal cut is
6. PF & shorten 1st ray (decompress 1st MPJ to perpendicular to long axis of MT
increase ROM) Hinge is left intact for added stability
-Skin incision Curvilinear following deformity, located Fixation- K-wire, absorbable pins, or screws
dorsomedial between bump and EHL tendon (1 point normally, 2 if hinge is broken)
-Capsular incision Inverted L, lenticular Comp- sesamoiditis, DJD
-Bumpectomy Angled more medially at proximal aspect to Post-Op- WB/NWB SxS 3-6 wks
avoid sagittal sulcus (see below)
-Indications Evaluate total adductory angle; IM + MT< 13. REVERDIN-GREEN
(see metatarsus ADDuctus) Def- Reverdin w/ transverse plantar “shelf” to avoid
-Procedure All procedures include capsule tendon disruption of articular surface of 1st MT and sesamoids
balance procedures (Silver/McBride)
-Post- Op ROM exercises for all procedures 14. REVERDIN-LAIRD
Def- Reverdin-Green cut thru & thru for lateral
CAPSULE TENDON BALANCE PROCEDURES transposition; corrects PASA and IM
9. SILVER BUNIONECTOMY (1923)
Procedure resection of medial eminence, + 15. REVERDIN-LAIRD-TODD
Lateral release Def- Reverdin-Laird, depress capital fragment to PF 1st ray
Latva lateral capsulotomy as well as correct IM and PASA
ADD hallucis release or transfer
Fibular sesamoid release or “ectomy” 16. AUSTIN (CHEVRON)
EHB tenotomy Def- corrects IM <, MT head is shifted laterally
JFK Deep Transverse MT ligament Ind- mild/moderate HAV, minimal tracking, painless ROM
ADD hallucis release Contra- DJD of 1st MPJ
Lateral capsulotomy Procedure V osteotomy w/ apex located at the mid-
Fibular sesamoid point of a vertical line connecting the ends
EHB tenotomy of articular cartilage of the head.
Indications- pain w/ shoes / bump pain, no abnormal V cuts angled at 60° w/ apex distal
changes, no DJD, positional HAV, osseous <’s WNL Fixation- cancellous screw or K-wire through dorsal
Post-Op- WB SxS for approximately 3 wks, ROM proximal shaft into capital fragment, aiming at crista
exercises Post- Op- SxS 4-6 wks WB
Complications- STAKING THE HEAD, hallux varus, stiff Comp- AVN, dislocation, intra-articular Fx, incorrect <,
toe, under correction overcorrection, undercorrection, stress riser

Sagittal Sulcus 17. YOUNGSWICK (MODIFIED AUSTIN)


Def- will PF & shorten as well as correct IM <
Ind- good for hallux limitus
Procedure- Austin, plus;
resection of an additional rectangle of bone
from the dorsal wing cut
Staking the Head Fixation- same as Austin
Proper

10. MCBRIDE (1928)


Procedure- Silver bunionectomy, plus:
1. fibular sesamoidectomy
2. ADD hallucis & lateral head of FHB
transfer to dorsolateral aspect of 1st MT
Modified McBride- Silver w/ fibular tendon release 18. KALISH (MODIFIED AUSTIN)
Hiss procedure- ABD hallucis transfer dorsal and medial on p.p. Def- long dorsal arm Austin (V = 55˚); reduces higher IM <
Fixate- 2x2.7 cortical from dorsal distal to plantar proximal
11. KELLER ARTHROPLASTY (1877)
Def- Resection of base of 1st proximal phalanx at MPJ
Ind- 1st MPJ HAV/DJD/hallux limitus, w/ osteopenia
Contra- lesser metatarsalgia, unstable hallux
Procedure Silver bunionectomy, plus:
Removal of 1/3 base of proximal phalanx
Reattach FHB tendon and capsule to p.p.
Post-Op- SxS 3-6 wks WB
Comp- Floating toe, flail toe, metatarsalgia
19. GERBERT MASSAD (BI-CORRECTIONAL AUSTIN) (SCARF CONTINUED)
Def- corrects PASA & IM (bi-correctional), 2nd cut is Procedure Longitudinal incision 1st
performed w/ a wide base medially and a wedge removed Transverse cut is 2/3 the width of
from D-M aspect of MT(wedge can also be taken from both MT (1 cm both head & base)
wings) Frontal plane cuts at either end;
Use axis guides to PF/DF metatarsal head (see below) dorsal cut is distal and plantar cut
is proximal
Axis is perpendicular no length
Angles- 45, 60, 70°; can use axis
guide for multiplanar corrections
Modifications- make a shorter arm
(angled 70-90°)
Fixation- two 2.7/2.4 osteomed,
two threaded .062 k-wires
Post-Op- WB or NWB 3-6 wks,
ROM exercises
Complications- troughing, stress
riser, dorsal cortical Fx of proximal
apex, overcorrection
Inverted Scarf

The inverted Scarf provides greater


resistance to disruption from
Axis directed proximally  shorten Axis directed distally  lengthen
weight-bearing forces (decreased
occurrence of stress risers)
SHAFT PROCEDURES
20. HOHMANN (1920) 22. VOGLER OFFSET V
Def- trapezoidal shape osteotomy (cut wider medially to Def- MT shaft osteotomy w/ long dorsal wing & short
reduce PASA) performed at anatomical neck plantar wing
Ind- stage 3 bunion, pain free ROM of 1st MPJ, IM < 18 Indications- bunion w/ ABDucted hallux, IM < 14-17,
Negatives 1. shortens too much PASA 8-35
2. sometimes ends too proximal in shaft Procedure- Apex of osteotomy at meta/diaph jxn (can
3. unstable; requires NWB + 2 pts of use axis guide)
fixation Dorsal cut 1st to exit at basal epicondyle
Positives (same as Reverdin-Laird-Todd) (angle is 40˚)
1. decrease IM < Distal fragment is transposed laterally to
2. decrease abnormal PASA reduce IM < & swiveled to reduce PASA
3. PF capital fragment Fixate- two 2.7, 3.5, .062, or 5/64 K-wires
Procedure Distal cut parallel to abnormal PASA Post-Op- slipper cast 4-6 wks, SxS 6-8 wks
Proximal cut perpendicular to long axis of Comp- AVN
MT shaft
Capital fragment transposed laterally and 23. MAU
plantarflexed Def- sliding MT shaft osteotomy from
Two points of fixation necessary plantar proximal to dorsal distal (capital
Post- Op- NWB w/ slipper/BK cast 4-6 wks, ROM frag is more stable)
exercises Post-Op- NWB 3-6 wks
Comp- elevation of capital fragment, dislocation of MT Comp- AVN, troughing, dorsal
head, PASA overcorrection, hallux varus displacement
Others Fixation- 3.5 screw or Steinmann pin
Mitchell- shortens MT
DRATO- (Derotational Abductory Transverse Osteotomy) 24. LUDLOFF
can have rotation of capital fragment in frontal plane Def- sliding MT shaft osteotomy dorsal
proximal to plantar distal
21. MEYER-SCARF “Z” Post-Op- NWB 3-6 wks
Def- Z-shaped sliding shaft osteotomy Comp- AVN, troughing, dorsal
Ind- painful bunion deformity, hallux abuts 2nd toe, displacement
negative MT <, IM < 14-17 Fixation- 3.5 screw or Steinmann pin
Contra- narrow MT<
BASE PROCEDURES
25. LOISON BALACESCU
Def- transverse base wedge of 1st MT base w/ medial hinge
Ind- stage 3 bunion, IM>15 rectus, IM>12 MTA
Contra- open epiphysis
Procedure- Osteotomy 1-1.5 cm distal to 1st MT-
cuneiform articulation & parallel to MT base
Lateral base closing osteotomy Def- 1st MT-medial cuneiform arthrodesis (stabilizes
perpendicular to WB surface to prevent medial column to restore fulcrum for better function of PL)
dorsiflexion, using axis guides Ind- painful ROM at 1st MT-cuneiform articulation,
Fixation- no screws hypermobile 1st ray, IM correction (angulated), FF splaying
1. .062 longitudinally through MT shaft & Procedure Dorsal incision
1st cuneiform ID & retract nerves, arteries, periosteum and
2. 5/64 medial-distal to lateral-proximal tendons
through cut and into 3rd cuneiform Incise joint, distract laterally
Post-Op- NWB B/K cast 6 wks (NO EXCEPTIONS) Release tarsometatarsal ligament
Comp- 1st MT head elevatus (always), shortening, Resect cartilage minimally from both sides
under/correction, hinge failure, nonunion w/ fixation of articulation
failure Bone graft prn to increase fusion rate &
Other: Trethowan - medial base opening w/ Reverdin prevent shortening (DON’T jam 1st MPJ)
Fixation- two 4.0 cannulated PT screw, side
by side or crossed; two 5/64 pins crossed;
1/3 or ¼ tubular plate medially & one 4.0
PT cannulated screw from dorsaproximal to
plantar-distal
Post-Op- NWB, BK cast up to 12 wks or until X-ray
healing
Comp- malposition,
bone graft failure,
open epiphyseal plate,
sesamoiditis due to
excessive
plantarflexion,
shortening of 1st MT,
metatarsalagia,
transfer lesions,
decreased hallux
purchase, non-union,
26. JUVARA under correction
Def- proximal, oblique 1st MT base osteotomy
Ind- stage 3 bunion, IM>15 rectus, IM>12 MTA
Contra- open epiphysis, narrow MT
Procedure Oblique osteotomy 1 cm distal to 1st MT- Other
cuneiform articulation, apex medially CRESCENTIC (Mann) -
Osteotomy perpendicular to the WB surface prevents shortening
to prevent dorsiflexion using axis guides
Osteotomy oriented from proximal/medial to 28. LAMBRINUDI
distal/lateral at an angle of 45˚-65˚ to the Def- plantar based closing wedge osteotomy, to correct MPE
MT Ind- MPE (resulting in painful hallux limitus)
Distance of cut should be 2X the width of the bone Contra- DJD of 1st MPJ, hypermobile 1st ray, PF 1st ray
Fixation- Procedure- Incision dorsomedial to 1st Met-Cun joint
1. Proximal Anchor Screw (2.7)- goes in 1st , medial Osteotomy w/ base plantar, apex ~1cm from
to lateral perpendicular to shaft 1st cuneiform. Cut is 45° from plantar.
2. Distal Compression Screw (2.7)- perpendicular to Fixation: 3.5/4.0 screw
cut or split the perpendicular Post-op- NWB 4-6wks in BK cast
Post-Op- NWB Comp- PF 1st ray, shortened MT, sesamoiditis, non/malunion
BK cast for 6wks
Complications- 29. WATERMANN
elevatus, Def- dorsal based closing wedge osteotomy of 1st MT head
shortening, under Ind- hallux limitus
/ over correction, Contra- MPE, short 1st MT
hinge failure, Procedure- Incision dorsomedial to 1st MPJ
nonunion, Oblique osteotomy dorsal to plantar at
fixation Fx margins of articular cartilage
Vertical osteotomy from dorsal straight
down to sesamoid apparatus
Juvara A- hinge type Fixation: crossed K-wires
Juvara B1-through & through, wedge -correct TP & SP Post-op- SxS 4-6wks, immediate ROM exercises
Juvara B2- through & through, wedge - correct TP & SP, Comp- stiff toe, non/malunion, sesamoiditis
lengthen
Juvara C1- through & through, no wedge - SP Others
Juvara C2- through & through, no wedge - SP & lengthen Modified Watermann- watermann + cheilectomy. Plantar hinge
is left intact (to protect sesamoids.)
Watermann-Green- Austin Youngswick procedure with a V
27. LAPIDUS angle of 120-140°. Corrects HAV and hallux limitus.
LESSER MT (DORSIFLEXORY)
30. JACOBY (DORSIFLEXORY “V”) 4th IM 6.47 8.71
Def- distal V osteotomy of lesser MT made from dorsal to Lateral deviation 2.64 8.05
plantar allows MT to elevate
Ind- propulsive plantar lesions
Contra- structural plantar lesions
Procedure V-shaped central MT head osteotomy w/
dorsal transposition of frag
Bone cut MUST be proximal to plantar
condyles and IPK
Fixation- percutaneous .045 K-wire
Post-Op- WB SxS 4-6 wks
Comp- nonunion, malposition, unresolved callus, MPJ
stiffness, transfer lesion, post-op DJD, floating toe,
excessive bone callus, AVN, MT coalitions

31. TILT-UP
Def- vertical wedge w/ cortical hinge located plantarly
Fixation- 2.0 or 2.7 K-wire dorsal to plantar

32. JIMENEZ
Def- oblique wedge w/ cortical hinge located plantar
proximal
Fixation- 2.0 or 2.7 K-wire

36. TAILOR’S BUNION EXOSTECOMY


Def- exostecomy of tailor’s bunion at 5th MT head
Ind- F&B type I tailor’s bunion, stable 5th ray, little angular
33. STRAIGHT VERTICAL
deviation, isolated hypertrophy of MT head, Pt. who can’t
Def- through & through vertical cut to slide head up
have an osteotomy, used in conjunction w/ MT osteotomy
Fixation- .045 K-wire proximal dorsal to distal plantar
Contra- poor bone stock
Post- Op- WB SxS 3-6 wks
34. GIANNESTRAS Comp- inadequate bone resection, MTJ subluxation,
Def- step down “Z” to shorten unstable 5th toe, hinge fx, non/malunion
Ind- splay foot (?)
37. REVERSE WILSON (HOHMANN) SLIDE
Procedure Oblique cut lateral distal to medial proximal
35. HELAL / WEIL Slide capital frag over
Def- MT shortening procedure Fixation- .062 K-wire from proximal shaft into head
Ind- predislocation syndrome (usually presents with
metatarsalgia of 2nd MT) 38. CHEVRON (REVERSE AUSTIN)
Procedure- long proximal dorsal to distal plantar Fixation- orthosorb pin
osteotomy
Comp- displacement of capital fragment
39. Z OSTEOTOMY (MINI “Z”)
Fixation- 2.0 or 2.7 screw dorsal to plantar
TAILOR’S BUNION
40. WEDGE (REVERSE REVERDIN)
General- Incision lateral to EDL
Fixation- 2.0 or 2.7 screw
Closing wedges at the base will decrease blood supply
Utilize Fallat & Bucholtz to assess angles
41. NEUROMA
Fallat & Bucholtz (F&B) Dx- Muldur’s sign, X-ray, MRI
Fallat and Buckholz (1980) claim to have devised a more D/Dx- Fx, RA, AVN (Freiburgs), neuropathy,
accurate way of measuring the 4/5 intermetatarsal angle which bursitis/capsulitis
has been adopted by several subsequent authors; it involves Incision Dorsal, longitudinal, interMT that runs up
using the medial border of the shaft as opposed to a bisection of onto either toe
the shaft due to significant variations found between the widths Plantar, longitudinal or transverse in the
of different fifth metatarsal shafts: sulcus
Points are marked on the medial surface at the center and Post-Op- WB SxS 2-3 wks (plantar approach is NWB)
at the base of the metatarsal, and a line is drawn through Comp- stump neuroma, white toe, hematoma, HDS
these points, establishing the longitudinal direction of the possibly
proximal bone. The fourth metatarsal was bisected in the
standard manner, since it is subject to less anatomical
variation.

Normal Pathologic
PLANTAR FASCIITIS
42. HEEL PLANTAR FASCIOTOMY
Dx- find exact location of pain using: nerve conduction & 45. KECK-N-KELLY
EMG, bone scan, lab tests Def- dorsiflexory wedge osteotomy w/ a plantar hinge
Ind- heel pain; initially 3 months of conservative care Procedure Incision lateral, inferior, and parallel to
should be attempted (but is not always practical) peroneal tendons
NSAID Heel cup Resect bump
Pre-fabricated orthotic Ice DF Calcaneal wedge osteotomy
Stretching P.O. steroids Fixate- 2 X staples or orthosorb pins
Steroid injection Acupuncture Post-Op- NWB cast 6wks
P.T. (ultrasound) Taping Comp- tendonitis, hinge breaks, posterior bone
Procedure DuVries medial incision (oblique at fragment
posterior calcaneus) 3-5 cm
Release fascia only
Post-Op- WB SxS
Comp- stress Fx, continued heel pain, increased fibrosis at
fascial origin, severe lateral plantar n., sever nerve to
ADDuctor Digiti Minimi, misdiagnosis
DuVries- removal of heel spur along w/ plantar fascia release
Snook & Chrisman- removal of portion of medial Calcaneal
tubercle (Manoli found that Fx occurred)
Gormley & Kuwada- heel spur resection, fascial release, partial
fasciectomy
RETROCALCANEAL SPURRING
43. ENDOSCOPIC PLANTAR FASCIOTOMY (EPF) 46. RETROCALCANEAL EXOSTECTOMY
(+)- less post-op pain, less soft tissue damage Etiology- equinus, variant arthritides, trauma
(-)- more technically demanding Non- Sx Tx- ¼ “heel lift, NSAID, SLC, steroid injection
Ind- at least 6-9 months of conservative tx w/o relief Try conservative treatment for at least 3 months
Contra- pt w/ atypical heel pain, abnormal EMG of foot, Procedure Incision: split TA or Zadek, lateral to
dysvascular foot Achilles
Procedure Arthroscope inserted through lateral portal Remove spur w/ osteotome (ALWAYS
w/ probe through medial portal (to plantar to dorsal) or w/ rongeur
determine length of plantar fascia) Rasp until smooth
Dorsiflex ankle & toes to put tension on Post-Op- repair TA w/ Mitek anchor p.r.n., BK (some AK)
plantar fascia & bulge FDB cast NWB 4-6 wks
Incise lateral border of pre-measured cut, Comp- stress Fx, inadequate resection, tendonitis, tendon
releasing plantar fascia lateral to medial rupture, anchor failure
Release of portion of deep abductor fascia
METATARSUS ADDUCTUS
CALCANEAL HYPEROSTOSES
Def- “pump bump” at posterior/superior/lateral calcaneus
Etiology- pathobiomechanics in rearfoot + constant, chronic
irritation
Dx Philip-Fowler angle- pathologic if > 75˚
Pavlov’s Parallel Pitch Lines- anything above the
superior line is pathological
Total angle= FF angle +Calcaneal inclination angle-
pathologic if > 90˚
Non Sx Tx- ¼ “heel lift, NSAID, SLC, orthoses, steroid
injection

44. HAGLUND’S RESECTION


Def- resection of dorsal hyperostosis
Procedure Zadek incision on side w/ bump
Osteotome plantar to dorsal
Smooth w/ rasp
Post-op- splint or cast 6-8 wks, Mitek anchor p.r.n.
Comp- “chasing the bump”, Fx, tendonitis, rupture
47. HEYMAN, HERNDON &STRONG (HH&S) Post-Op- NWB, BK
Def- mobilization of Lisfranc’s joint by capsular release cast for 6 wks
Indications- pain, appearance (social concerns of child), Comp- under correction,
difficult fitting in shoes, age- 3-7 y/o mal-union, non-union
Procedure Incision: 1. 3 incision approach: medial to
EHL, 2nd interspace, 4th interspace Other
2. 5 incision: difficult to close Peabody & Muro- excision of
Accurately ID tarsoMT joint spaces the bases of three central MT,
Release all soft tissue & ligamentous osteotomy of the 5th,
structures at level of Lisfranc’s joint except mobilization & reduction of
1/3 of the plantar-lateral aspects of the bases luxation of 1st metcuneiform
of MT 1-4 joint & correction of any
5th MT cuboid joint is left intact for stability abnormal insertion of Lepird
Harris modification- 2nd MT osteotomy tibialis anterior tendon
Post-Op- NWB, AK cast for 3 months (amount of time McCormick & Blount- arthrodesis of the 1st metatarsocuneiform
required for remodeling of MT) joint along w/ an osteotomy of bases of central MT bases, in
Note: NOT used much b/c of bad Sx results (painful scars) neglected cases wedge resection of cuboid
Steytler & Van der Walt- oblique V-shaped osteotomies at
bases of each MT w/ apex of V angled towards the RF

FLATFOOT PROCEDURES - MEDIAL COLUMN STABILIZERS


Soft Tissue
50. KIDNER
Def-soft tissue procedure/medial column stabilizer
Ind- adjunct to osseous procedures for pes valgo planus
deformity, os tibiale externum, hypertrophic navicular
Contra- rigid adult pes valgus, PTTD, normal pes planus
Procedure Medial curvilinear incision over navicular
Resection of hypertrophic navicular
Removal of OTE
TP advancement (reattach plantar-distal)
Spring ligament reefing
Post-op- SLC for 4-6 wks, depends on other procedures
48. BERMAN GARTLAND Comp- PTT rupture, scoring of talar head, prominent talar
Age- 7- adult head as a result of excessive navicular resection
Def- Crescentic osteotomies of all 5 MT bases
Procedure Incision: 3 or 5 incision approach
Crescentic osteotomies of all MT bases 1cm
distal to articular cartilage
Fixation- originally only fixated 1st and 5th
Post-Op- NWB, BK cast for 6 wks
Comp- under correction, mal-union, non-union

51. YOUNG
Def- soft tissue procedure/medial column stabilizer
Ind- adjunct, operative Tx of pes valgo planus, for pt
greater than 10 y/o
Contra- normal pes planus
Procedure Incision: from 1st MT-cuneiform to medial
malleolus
Tendo achilles lengthening
Tib anterior rerouted through key hole in
navicular
TP reefing (reattach plantarly)
49. LEPIRD Modification- Split TA & pass only half through keyhole
Age- 7-adult Post-Op- BK cast 6 wks, f/u w/ shoe wedging and PT
Procedure Incision: 3 or 5 dorsal incision approach Comp- navicular Fx, tendon problems, subluxation
1st & 5th MT closing base wedge w/ base
lateral w/ medial hinge left intact Osseous
2nd, 3rd, 4th MT osteotomy from dorsal- General
distal to proximal-lateral keeping cortex Age- late teen – adult
intact, angled 45˚ to the perpendicular of Ind- long standing flatfoot, faults & DJD on X-ray, RF in
shaft (proximal portion of osteotomy no rigid valgus, PTTD symptoms, tendonitis, & sinus tarsitis
closer than1cm from base) Procedure- usually combined w/ TAL & desmoplasty,
2-4 MT one screw is driven into bone graft used prn to prevent shortening
osteotomy w/o tightening; lateral cortex is Fixate- 6.5, 7.0, 7.3 screws or stables
broken & MT are moved lat Screws are Post-Op- NWB, BK cast for up to 12 wks or until bone
tightened healing on X-ray
Fixation- 2.0, 2.4, or 2.7 K-wire Comp- mal-union, bone graft failure
52. LOWMAN - TN fusion, reroute TA under navicular and 56. SILVER
suture to spring ligament. Def-varus producing posterior
calcaneal osteotomy w/ lateral
53. HOKE opening wedge
Def- navicular, 1st, 2nd cuneiform fusion Ind- Sx correction of pes valgo
Indications- adult and adolescent pes valgo planus planus deformity (frontal plane
stabilizes the medial column to lengthen the lever arm of dominant), stable MTJ
the muscles affecting the longitudinal arch (used almost Procedure Lateral approach w/
exclusively as an adjunct procedure in combination w/ L-shaped incision
ankle equinus correction and calcaneal osteotomy) Opening wedge
osteotomy w/ base
lateral
Comp- dehiscence

57. DWYER
Def- varus producing posterior calcaneal medial closing
wedge osteotomy
Ind- Sx correction of pes valgo planus deformity (frontal
54. MILLER - navicular, 1st cuneiform, & 1st MT base fusion plane dominance)
Procedure Medial approach w/ L incision
Other Closing wedge osteotomy w/ base medial
Modified Hoke/Miller- arthrodesis of the NC joint, opening Post-Op- BK NWB cast 6-12 wks
wedge osteotomy of the first cuneiform and distal advancement Comp- difficulty in approach, nerve entrapment
of an osteoperiosteal flap to include the plantar CN ligament
58. KOUTSUGIANNIS
FLATFOOT PROCEDURES - CALCANEAL OSTEOTOMIES Def- arcuate through and
55. EVANS through posterior osteotomy w/
Def- anterior calcaneal osteotomy w/ wedge insertion medial transposition
lengthens lateral column
Ind- pathologic pes valgo planus (transverse plane
dominant)
Contra- flatfoot caused by neurologic disorders, < 6 y/o,
supra-torsional abnormalities
Procedure Determine compensated MTA prior to Sx ATHROEREISIS
Incision: oblique proximal to Def- “joint lifting” to prevent
calcaneocuboid joint (avoiding sural n.) abnormal joint motion (valgus at STJ), used in conjunction w/
Retract peroneal tendons inferiorly either Kidner or Young procedure
Retract EDB superiorly
Osteotomy 1-1.5 cm proximal to calc-cuboid Goal to prevent lateral talar process from dropping into the
joint (through and through) calcaneal sulcus
Insert trapezoidal graft (ideally from iliac stabilize STJ decrease heel valgus stabilize
crest) MTJincrease efficiency of peroneus longus PF
Perform medial arch tendosuspension & 1st ray stabilize medial column + increase arch
TAL as adjuncts height

Ind pain w/ activity, causing pt to refrain from activity


(+) family history orthoses ineffective
FF deformities >50˚ TNJ disarticulation
clumsy night cramps
everted CSP Kite’s angle > 30˚
FF varus > 8-10˚ ↑ talar declination <
↓ calcaneal inclination < Hx of anterior cyma line
Midtarsal break

Contra rigid (A) significant DJD (A)


ankle valgus (A) prior infection (A)
skewfoot (B) equinus (B)
< 3 y/o structural FF varus

Comp infection implant malposition


Post-Op- NWB, BK cast 8-12 wks sinus tarsitis peroneal tendonitis
Comp- under/ over correction, shift of anterior fragment, implant breaks Fx calcaneus
delayed/non-union, cutting into one of facets, bone graft fx lateral talar process talar beaking
failure (10-12 wks), jamming of medial & lateral columns, over/under correction iatrogenic coalition
DJD, peroneal tendonitis, damage sural nerve
Note: ↑
Trephine- tool used to cut out a hole in calcaneus for implant
Note- if pt is young can do a Dwyer and
59. SELF-LOCKING WEDGE (PERMANENT) just cut plantar fascia
Def- to hold talus and calcaneus apart, screws into sinus Post-Op- Primary- 3 wks NWB BK cast w/ FF dorsiflexed
tarsi & elevates STJ axis Adjunctive- procedure dependent
Ind- any age Comp- excessive fibrosis, nerve entrapment, myositis,
Types: MBA (titanium), Valenti (polypropylene), Viladot fasciculitis, neuritis, damage vessels, HDS
umbrella, Custom-carved plug
Procedure Incision- long lateral over opening of sinus 63. DWYER
tarsi (3-4cm Grice) Age- usually 10-12 y/o (beware of apophysitis)
Cut out deep sinus tarsi plug (Hoke tonsil), Ind- Ruch 2
leave attached anteriorly Procedure Incision: parallel & inferior to peroneal
Square off posterior wall & floor of anterior tendons
calc. process Straight or oblique wedge w/ base lateral
Place K-wire through sinus (Osteotomy decreases supinatory force of
MBA: Screw in correctly sized device 1 cm the TA on calcaneus)
from lateral wall of calcaneus Close by dorsiflexing FF
Check for proper placement and motion of Note- -final position: approximately 0.5˚ valgus
calcaneus -closer the osteotomy is to STJ the greater
Post-Op- WB BK cast 2 wks (MBA) the amount of correction of TA on calc
-can be modified for frontal or transverse
60. AXIS-ALTERING (REMOVABLE) plane deformities
Def- holds bones apart and raises STJ axis back to normal Fixation- two staples (most common), Steinman pin, screw
(lateral process glides on top), drilled into calcaneus Post-Op- NWB, BK or AK cast for 6 wks
Types: Smith Sta-peg (straight top), Lundeen (concave top) Comp- non-union, tendonitis, STJ DJD, hinge Fx, wound
Ind- younger pt; 3-13 y/o dehiscence, nerve entrapment, under correction
Procedure Incision: long lateral (medial to lateral) over
opening of sinus tarsi (Grice) 64. JONES TENOSUSPENSION
Cut out deep sinus tarsi plug (Hoke tonsil), Ind- Ruch 1, hallux hammertoe, hallux
leave attached anteriorly varus, double sesamoidectomy
Square off posterior wall & floor of anterior Procedure Cut & tag EHL at insertion
calc. process Drill hole through MT head
Locate distal-lateral aspect of posterior facet & feed EHL through hole
of calcaneus Reattach tendon to itself w/
Use osteotome to create two vertical & one 2.0 or 3.0 non-absorbable
longitudinal base cuts Fuse hallux IPJ
Drill hole in calcaneus w/ trephine Post-op- NWB BK cast for 6 wks
Place device in hole Comp- rigid PFFR, tendon dislocation, not
Check for motion of calcaneus enough tension on EHL, MT head Fx,
Post-Op- early ROM & WB in SxS gradual increase in
ambulation 3-4 wks return to normal 65. PERONEAL STOP
Def- takes out effect of PL on 1st ray, puts that force
61. DIRECT IMPACT (PERMANENT) through the PB ABDuct & evert foot
Types: Sgarlato mushroom Ind- Ruch 1 or 2
Ind- any age Procedure 1. PL is sutured to the PB w/ 3.0 non-
absorbable while foot is ABDucted and
everted, OR
2. PL is cut (stab incision) along the length
2 of PB & PL is weaved through
CAVUS FOOT PROCEDURES Suture stab ends so they won’t tear
Ruch Classification of Cavus Foot Deformity Post-Op- NWB BK cast for 4-6 wks
Ruch 1- Flexible cavus Comp- peroneus brevis splits longitudinally,
Tx- Rigid fusion, MPJ release, Hibbs, Jones, Orthoses tendon problems, rigid PFFR
Ruch 2- Cavus becoming rigid- PFFR and RF varus
Ruch 3- Rigid (Global) cavus
67% due to neuromuscular (CMT-most common)

62. STEINDLER STRIPPING 66. SPLIT TIBIALIS ANTERIOR TENDON TRANSPOSITION-(STATT)


Def- Soft tissue release/plantar fasciotomy Ind- Ruch 2, swing phase supinatus, invertors
Ind- peds pt. w/ significant contracture of both plantar overpowering dorsiflexers, equinovarus deformity
fascia and musculature, flexible cavus deformity, pt w/ Procedure Incision: 1. linear over cuboid, 5th MT base
neurologic disease 2. linear over 1st MT/cuneiform
Procedure Incision- (DuVries medially) longitudinal 3. linear or transverse over dorsum
along medial side of calcaneus of tibia (5-7 cm proximal to AJ)
Release plantar fascia Make stab incision & divide TA into medial
Strip abductor hallucis, FDB, abductor digiti & lateral halves over tibia
minimi from calcaneal periosteum Pass instrument (uterine packing forceps)
Release long plantar ligament (along w/ it from medial incision to tibia (under skin)
goes Quadratus plantae) Place umbilical tape through incision in TA
Use forceps to grab tape, pull out through REARFOOT ARTHRODESIS
medial incision, and split tendon in process General
Detach lateral half of tendon from its Ind- instability, DJD, pain, trauma, failed implant, end
insertion & pass it up & out of tibial incision stage flatfoot or cavus, RA, S/P infection,
Pass instrument through the lateral incision
to the tibia, grabbing medial TA
Pull back through lateral incision
Attach medial TA to cuboid or peroneus
tertius (if present)
Post-op- NWB BK cast for 6wks
Comp- rigid cavus foot, tendon dislocation, tendon splits

Other
Hibbs- EDL is used for tenodesis of midfoot (3rd cuneiform) Post-Op
Combined Hibbs & Jones tendosuspension 1. Drain for 24-48 hrs
2. Posterior splint for up to 2 wks
67. COLE 3. NWB BK cast for 6 wks
Def- dorsiflexory wedge through midfoot 4. WB BK cast for 6 wks
Ind- Ruch 3, severe anterior cavus, severe NM disease Comp- non-union, mal-union, varus, fuse wrong joint (do
Procedure Incision- same incision as HH&S diagnostic injection)
Dorsiflexory wedge of midtarsus w/ through
& through cut extending from cuboid 69. STJ ARTHRODESIS
through cuneiform-navicular joints Procedure Incision- Ollier or lateral linear (reflect EHB
Fixation- staples Resect cartilage from talus & calcaneus
Post-Op- NWB BK cast for at least 6 wks using osteotome, power or curette
Comp- DJD, deformed foot, neuroma, tendon problems, Pack w/ bone chips to increase healing and
non-union, fixation failure to maintain height
Position at 5˚ valgus
Fixation- 6.5, 7.0, or 7.3 from dorsal to plantar thru
posterior facet (can also use staples)

70. AJ ARTHRODESIS
Procedure Incision- Kocher, or from fibula to 4th MT
base
68. JAPAS Remove at least 2 cm of fibula & retain
Ind- Ruch 3, severe NM disease Remove top of talus & tibial plafond
Procedure Incision: 1. 1st cuneiform-navicular Resect ~1cm medial malleolus (prn)
2. 2nd & 3rd cuneiform-navicular Pack all voids w/ bone chips
3. 4th & 5th MT base-cuboid Position TP: 5-10˚ ext, or matched to opposite limb
Thru & thru “V” cut w/ apex in navicular FP: 5-10˚ of valgus
Medial arm through 1st cuneiform SP: 0˚
Lateral arm through cuboid Talus- 10mm posterior to tibia (will ↑ lever
Slide FF inferior and dorsiflex arm & ↓ stress distally)
Fixation- 2 Steinman pins or 7/64 pins Fixation- 6.5, 7.0, or 7.3 cannulated, crossed angled down
Post-Op- NWB BK cast for at least 6 wks or put both in from talus on lateral side & up into tibia
Comp- DJD, deforms foot, neuroma, tendon problems,
non-union, fixation failure
71. TRIPLE ARTHRODESIS 72. LATERAL STABILIZATION
Procedure Incision- Kocher or 2 Incision approach: Single Ligament Replacement- peroneus brevis is used as an
curvilinear for STJ & CCJ; dorsomedial for autogenous graft for recreating the CFL or ATFL
TNJ (order varies)
Fusion order: CCJ>STJ>TNJ 1.WATSON-JONES
Fixation- 6.5, 7.0, or 7.3 (or can use stables) Procedure Graft is inserted through a drill hole in fibula
1. Ryerson (top of page) (postant) approximately 2 cm from distal
2. Hoke- Head & neck of talus are used as a bone graft tip of malleolus
Pass tendon through hole drilled in talar
3. Lambrinudi- Good procedure for drop foot
neck (dorsal plantar)
Return tendon through fibula (ant post)

2. EVANS
Procedure Drill hole through fibula
Tendon fed through hole & secured
4. Brewster countersink- posteriorly at proximal portion of superior
peroneal retinaculum

5. Triple w/ Dunn modifications- 3. LEE


Procedure Entire PB graft is passed through from
posterior to anterior through fibula
Graft is anchored distally by performing a
peroneal anastomosis
A periosteal flap from the distal most
anterior aspect of fibula reinforces new lig

6. Seiffert (beak)-
4. NILSONNE
Procedure Detach PB at level of musculotendinous
junction
Proximally, PB is sutured to PL
While distal tendon is placed in a
subperiosteal groove through fibula
(posterosuperior to anteroinferior)
Double Ligament Replacement - designed to address chronic 7. SPLIT PERONEUS LONGUS
injury of both ATF & CF ligaments Procedure Tendon identified & severed from muscle
belly at proximal aspect free up distally
1. CHRISTMAN & SNOOK Periosteal channel is created in talar neck
Procedure Split PB tendon graft is harvested from End of tendon is passed through
proximal aspect of tendon Then through a trephine hole in fibula
Graft is routed through talar neck & then (antpost)
through distal fibula at it’s widest (anterior Bone plug is replaced in fibula
 posterior) Then passed through trephine hole in
Graft is then sutured to a periosteal flap calcaneus
created at the level of CF lig Plug replaced recreates CF lig
Last suture the distal end of graft onto itself

2. ELMSLIE TENDO ACHILLES LENGTHENING


Procedure 12 cm incision over PB tendon (5cm Ind- gastroc or gastro-soleal equinus
proximal to tip of lateral malleolus ending Post- Op- NWB BK cast for 3-4 wks followed by P.T.
between base of 5th MT & malleolar tip Comp- tendon rupture, sural neuropathy, shoe irritation,
Retract peroneals inferiorly under/over correction
Insert graft of tensor fascia lata in
osseous canal through talar neck Types:
(sup to inf)
Lace through canal in fibula from 1. White- slide lengthening
superoanterior  posteroinferior
Then through a canal in calcaneus
posterosuperioranteroinferior
Ends are sutured on themselves

3. HAMBLY
Procedure Split PL & suture it to lateral talar neck
Then passed tendon from
anteriorposterior through fibula
Graft is then fixated to calcaneofibular 2. Cummins
ligament insertion

4. WINFIELD
Procedure same as Hambly, but use PB instead of PL
3. Hoke- triple
hemisection slide

5. KELIKIAN 4. Conrad & Frost


Procedure Plantaris tendon is used to recreate lateral
ligaments

6. SEEBURGER
Procedure Hemisection of PL as free graft to
reconstruct ATF & CF lig
5. Baker- distal tongue-in-groove

5.Z- frontal (or sagittal) plane slide


THE ELECTROCARDIOGRAM
Leads: Leads record the electrical activity of the heart occurring
between two electrodes. Placement of leads on the body allows view
of the heart in two planes – frontal and transverse. The 12-lead EKG
views the heart from 12 different angles – 6 frontal and 6 transverse.
There are three types of leads – standard limb leads
(bipolar), augmented leads (unipolar), and chest leads (precordial.)
Each lead has a positive electrode which “sees” the magnitude and
direction of electrical forces in a specific part of the heart. If
electrical activity moves towards the positive electrode, the EKG
shows an upward waveform – if the electrical activity is away, it is a
downward waveform. No electrical activity is represented by a
straight line (isoelectric.) No leads view the posterior surface of the
heart.
Frontal Plane: Has 3 standard leads and 3 augmented leads.
Standard leads:
Lead I (Rt. Shoulder) – views lateral lt. ventricle, QRS is +
Lead II (Lt. Shoulder) – views inferior lt. ventricle, QRS is + ATRIAL DYSRHYTHMIAS: Atrial dysrhythmias reflect abnormal
Lead III (Lt. Foot) – views inferior lt. ventricle, QRS is +, P electrical formation and conduction in the atria. The two
]may be +, -, or both mechanisms whereby electrical impulses are altered are changes in
Augmented leads: automaticity and triggered activity. Factors that cause a change in
AVR (Rt. Shoulder) – view of the base of the heart, QRS is – automaticity include ischemia, drug toxicity, hypocalcemia, and
AVL (Lt. Shoulder) – view of lateral lt. ventricle, QRS is 0 electrolyte imbalance. Factors that cause triggered activity are
AVF (Lt. Foot) – view of inferior lt. ventricle, QRS is + hypoxia, catecholamines, hypomagnesia, myocardial
Transverse Plane: Has 6 precordial (chest) leads. ischemia/infarct, and any medication that increases the repolarization
V1 (4th intercostal space rt.) – P +, -, or 0, QRS is – time. (Triggered activity occurs in runs.) Most atrial dysrhythmias
V2 (4th intercostal space lt.) – same as V1 are non-life threatening, unless they affect the ventricular rate.
V3 (midway btw V2 and V4) – Same as V1, V2
V4 (5th intercostal space, lt. midclavicular line) – P +
V5 (5th intercostal space, lt. ant. axillary line) – P +, QRS +
V6 (5th intercostal space, lt. midaxillary line) – P +, QRS +
EKG stip paper: X axis = Time (sec), Y Axis = Voltage (mV).
Speed = 25mm/sec. 1 box = 1mm = 0.04 sec. One large box = 5mm
= 0.2 sec
P Wave: Smooth and round, no more than 2.5mm tall. No more
than 0.11 sec duration. + in I, II, AVF, V2-V6. PRE-ATRIAL CONTRACTIONS (PACS): Appear early, before an
PR Segment: Line btw P wave end and QRS beginning. expected beat. They are identified by their site of origin – either
PR Interval: P wave + PR segment. Represents depolarization of atrial, junctional, or ventricular. They occur when an irritable focus
the atria and propagation through the AV node, bundles of His and discharges out of sync with the SA node pacemakers – thus the
Purkinje fibers. rhythm and rate are unchanged, but there are premature atrial beats
Normal = 0.12-0.2 sec. coexisting in the EKG. These P waves on the EKG may not be
Long PR interval = delayed impulse (heart block), digitalis toxicity followed by QRS complexes. The PR interval may be normal or
Short PR interval = impulses originating from ectopic focus prolonged. PACs are common findings and may not signify
QRS Complex: Q – downward, depolarization of IV septum pathology – emotional stress, fatigue, and CHF are all causes of
R,S – Simultaneous depolarization of ventricles (mainly lt.) PACs. They do not require intervention unless there is risk of
Overall positive, may be missing one component (variable) progression to atrial flutter, in which case you should use beta-
Normal = 0.06 – 0.10 sec
Long QRS complex = ectopic purkinje/myocardial pacemaker
ST Segment: Line btw QRS complex and T wave. Represents early
ventricular repolarization. Displacement up or down from the
isoelectric line is termed ST “elevation” or “depression.”
ST depression (>1mm) = myocardial ischemia, digitalis tox.
ST elevation (>1mm limb, >2mm chest) = myocardial injury
T Wave: Represents ventricular repolarization, should be in the
same direction as the QRS complex. blockers, calcium channel blockers, or anti-anxiety meds to reduce
Inverted T waves = myocardial ischemia the risk.
Tall, pointed T waves = Hyperkalemia SUPRA-VENTRICULAR TACHYCARDIA (SVT): SVT is a
Rate: Pulse rate measured from the EKG, in 1 of 2 ways: tachyarrythmia originating in the heart above the level of the bundles
1) Count # of QRS complexes in 6 sec, multiply by 10 of His. Sudden onset SVT is described as paroxysmal or PSVT.
2) Count # of large boxes btw QRS complexes, divide into 300 Characteristically, SVTs have a rate of 150-250 bpm, with regular
Rhythm: Regular amount of time btw QRS complexes. rhythm and “different” looking P waves. SVTs are treated with
Normal Sinus Rhythm = 60-100 beats/sec, uniform P waves, PRI synchronized cardioversion (fibrillation.)
0.12-0.2 sec, QRS <0.11
Sinus Bradycardia = Regular rhythm, <60 beats/min
Sinus Tachycardia = Regular rhythm, 100-180 beats/min
Arrhythmia = Irregular rhythm (P waves, QRS uniform)
wave.

ATRIAL FLUTTER: is a rapid atrial rate due to an ectopic pacemaker


– usually an irritable focus that is firing regularly. Pulse rate is >300
bpm. It lasts for seconds to hours, brought on by PE, thyroid
toxicosis, chronic ventilatory failure, ethanol abuse, CAD, hypoxia,
and digitalis/quinidine toxicity. It is precipitated by PACs and is also
assoc. w/ mitral/tricuspid valve dz. 5-10% of post-MI pts develop
atrial flutter. It is treated with meds if cardiac function is normal. If VENTRICULAR FIBRILLATION: is distinguished by the lack of
not, digitalis or cardioversion is organized contraction of the ventricles. 80% of pts post-MI develop
indicated. V-Fib if not tx in a hospital. Rate and rhythm are incalculable. Tx
w/ CPR and cardioversion.

ATRIAL FIBRILLATION: can be spontaneous and last an hour to days.


Pulse rate is 400-600 bpm, which results in a decrease in output due 1ST DEGREE AV BLOCK: Distinguished by a PR interval >0.2
to total dysfunction of the atria. The rate is variable, the rhythm is seconds that is the same for every beat. This situation can be normal
irregularly regular, P waves are indistinguishable, and the PR interval if no Hx of heart dz, ischemia, injury, meds,
immeasurable. It is seen in pts. w/ rheumatic heart dz, CAD, PE, etc.
HTN, ethanol abuse, carbon monoxide poisoning, electrolyte
imbalances, hyperthyroidism. Classic presentation is from over-
imbibing of alcohol at holidays, resulting in “holiday heart
syndrome.” Tx is same as for atrial flutter, except pt also needs to be
put on anticoagulants to prevent blood clot formation in the
atria.

2ND DEGREE AV BLOCK (TYPE I): Here the conduction delay occurs
in the AV node. It is characterized by progressively lengthening PR
intervals that result eventually in entirely missed beats. It usually is a
sign of ischemia, typically in the area of distribution of the rt.
coronary artery, or of
PRE-VENTRICULAR CONTRACTIONS (PVCS): form from an irritable medication.
focus in the ventricles. The normal rate of ventricular contraction
originating from the VA node when the SA node is malfunctioning is
only 20-40 bpm. The distinguishing feature on EKG is that PVCs
show an elongated QRS complex (>0.12 sec) with a compensatory
pause. Like PAC, PVC is a single beat, so rate and rhythm are not
affected. It is also a normal finding that can be brought on by
exercise, hypoxia, stress, and excess catecholamines. Tx depends on
the S/Sx; usually no tx is 2ND DEGREE AV BLOCK (TYPE II): Here the conduction delay
required. occurs somewhere below the AV node, usually the bundle branches.
The bundle branches are supplied by the left coronary artery and are
injured in anterior wall infarcts. The EKG shows variably displaced
or absent P waves and QRS complexes. This condition may progress
suddenly to a complete AV block. Tx is by pacemaker; otherwise the
prognosis is very bad.

VENTRICULAR TACHYCARDIA: is distinguished by a rate of 150-250


bpm. P waves are often absent, and the PR interval is elongated
when they are seen. The QRS is >0.12 sec and is difficult to
distinguish from the T
Drugs To Know – Really…
1kg = 2.2lbs
ANTIBIOTICS:
Broad Spectrum:
PCN ---- ZOSYN (Pipercillin / Tazobactam):
• Staph, Strep, Entero, G -, Anaerobes/Aerobes, Pseudo… NO MRSA
• 3.375 g IV q6h
• 4.5 g IV q8h  Suspected Pseudo
o + Gentamycin if Pseudo / G- / Systemic infxn
PCN ---- AUGMENTIN: 500mg po q8h or 875mg po q12h – ST infxn
INVANZ (Ertapenem) Carbapanem
• Kills EVERYTHING except Pseudo
• 1g IV / IM qd
TYGACIL (Tigecycline) glycylcycline, static, 30S ribosome
• G +, G -, Anaerob, MRSA, NO  pseudo, proteus
• Slow infusion (30-60min)
• SE = derivative of tetracycline (bone, teeth, peds)
• 100mg dose, then 50mg q12h after
• Liver pts lower dose, renal pt ok, no peds yet, no po
MRSA:
VANCOCIN (Vancomycin)
• ALL G +, MRSA, NO G –
• Follow Renal Funcion (Peaks/Trouphs)
• Infuse over at least 45-60min to avoid red man
o If get red man, Tx = Benadryl 10-50mg tid until symptoms go away
• 1g IV q12h (max dose = 2g/day)
• 125mg PO qid for C.dif, better than metro, but $$$
• Peak/Trough = 20-50ug/ml & 5 - 10ug/ml
o Draw 1hr before dose(trough), and 1hr after administration (peak)
• SE = oto and nephro, long term tx is worse
• with aminoglycoside can  nephrotoxic
ZYVOX (Linezolid) Oxazolidinone
• ALL G + (including Entero, staph, VRE, MRSA)
• 600 mg IV / PO bid
• $$$ - don’t use if can use Vanc, or Bactrim + Rifampin
• Better Bone Penetration than Vanc
BACTRIM (TMP/SMX) (160mg/800mg) Sulfonamide
• Good Staph, MRSA, some strep, NO Pseudo
• SULFA
• 1 tab PO bid, tid for severe infxn
• + Rifampin beneficial
o Good staph/strep, mycobac, some G-
o Used in combo b/c of quick resistance to it
o 300mg PO bid
o No adjustment for renal pt
o combo with vanc, bacrim, minocycline for MRSA
Anti-PSEUDOMONAL:
PCN ---- ZOSYN (Pipercillin / Tazobactam):
• Staph, Strep, Entero, G -, Anaerobes/Aerobes, Pseudo… NO MRSA
• 3.375 g IV q6h
• 4.5 g IV q8h  Suspected Pseudo
CIPRO (Ciproflxacin)
• ALL G -, Pseudo, not good staph/strep, NO PEDS
o Levaquin better for staph/strep, less pseudo, same G –
o 500mg PO / IV qd (mild-mod)
o 750mg PO / IV qd (complicated skin infxn)
• SE = tendon ruptures, epiphyseal cartilage damage
• 500mg PO bid, 400mg IV bid (mild-mod infxn) only use IV if pt has to be NPO…
• 750mg PO bid (severe infxn, osteo)
• Decrease to once daily for renal pts…
• Proven Good bone penetration

Other ANAEROBES:
CLEOCIN (Clindamycin) Macrolide
• Most G +, Anaerob, NO  MRSA, Clostridia
• 600 – 900mg IV/IM q8h
• 150-300mg PO bid or tid for outpatient
• OK for renal pt, good bone penetration
Metronidazole: 1g bolus, 500mg IV q6h over 1hr
PCN --- UNASYN (Ampicillin/Sulbactam) B-Lactamase combo
• G +, some G -, great Anaerob, Entero, NO MRSA
• Emperic Rx for bite wounds
• 1.5-3g IV / IM q6h
• dec dose if CrCl drops below 30-50ml/min
Gram + general:
• Ancef: 1g IV/IM q8h 1 hour prior to sx
• Keflex: 500mg po qid
• Dicloxicillin: 500mg po qid
• Clindamycin: 600mg IV q8h (PCN all) 300mg po qid
• Vancomycin: 1g IV q12h over 1hr (PCN all)
Gram – general:
• Levaquin: 500mg po/IV qd

Bug - Drug of Choice / Alternative / 3rd Line


MRSA - Linezolid, Vanc / Rifampin / TMP – SMX
S. Aureus - Keflex, Nafcillin / Vanc, Unasyn, Zosyn…
Strep. Pyogenes - Ampicillin, Pen G/V, Amoxicillin / Unasyn, Zosyn, Ceph(1-3), Macrolide (Erythro),
Vanc
P. Aeruginosa - Zosyn, Aminoglycoside, Cipro, Cefepime / Levofloxacin
E. Coli - Keflex, Ancef, Toby, Gent / Unasyn, Zosyn, Augmentin / TMP-SMX
Clostridium Perfringens - Pen G +/- Clinda / Zosyn, Unasyn, Cefriaxone, Linezolid / Erythromycin
Bacteriodes Fragilis – Metro, Unasyn, Zosyn, Primaxin IM(Imipenem + Cilastin) / Augmentin, Timentin,
Clinda
Gas Causing Bacteria: BECKS
Bacteriodes, Enterococcus, Clostridia (4), Kleibsiella, Staph/Strep

Clostridium (Anaerobic)
Perfringes – gas gangrene, liquefaction necrosis, and hemolysis from α-toxins
Difficile – pseudomem colitis – exotoxin kills enterocytes, need 3 neg toxin screens to clear
Botulinum – food poison – flaccid paralysis – heat-labile toxin inhibits Ach release (anti-
SLUDGE)
Tetanus – CNS – exotoxin blocks glycine release from Renshaw cells in spinal cord – titanic
paralysis

BacteriCIDAL ABx: Pen, Ceph, Vanc, Aminoglycosides, Fluoroquinolones, Metronidazole


Cephalosporin kills: 1st gen (Ancef, Keflex, Duricef) – PEcK
2nd gen (cefoxitin, cefaclor, cefuroxime) – HEN PEcKS
Haemophilis influenza
Enterobacter
Neisseria
Proteus
E. coli
Klebsiella pneumonia
Serratia

GRAM Stain steps: (water rinse after each step)


1) Gentian Violet
2) Alcohol
3) Grams Iodine
4) Safranin

(140 – age) x weight(kg)


Creatinine Clearance = ------------------------------------------
Serum Cr x 72 (for women x 0.85)
*Normal renal function = CrCl around 100 ml/min (it can be much higher)
*The “loading dose” of aminoglycoside Tx is the same regardless of pts renal function. The only thing that
changes is the maintenance dose.
*There is a linear relationship between clearance and percentage of renal function.
*if the CrCl is 50 ml/min, 50% renal function is assumed; therefore 50% of maintenance dose is calculated.
If full dose = 5mg/kg/day, then 2.5mg/kg/day bid or tid should be administered.

Bacteria Growth Curve:


Lag – metabolic activity without growth
Log – rapid cell division
Stationary – nutrient depletion slows growth
Death – nutrient depletion and waste buildup  death

Tetanus Prophylaxis: (0.5 cc Toxoid) (250 units TIG)


Clean, Minor Wound Major or Dirty wound
Toxoid Immunogl Toxoid Immunogl
Unknown or YES NO YES YES 250U
<3 doses
3 + doses  NO, unless NO NO, unless NO
>10y since >5y since
last dose last dose
*Cannot put toxoid and TIG in same site. Must use other arm

Aminoglycosides: Gent, Toby, and Amika


Loading dose: Gent/Toby  2 mg/kg Amika  7.5 mg/kg
Usual Maintenance: Gent/Toby  5-7 mg/kg/d q8-12h Amika  15 mg/kg/d usually q12h
*CrCl must be calculated before maintenance dose given
Usage limited to Life/Limb threatening Aerobic G – infxn
Toby is the most effective against P aeruginosa
Gent is best for Serratia
Amika is RESERVED for organisms resistant to gent and toby
They are active against staph/strep, but there are less toxic Rx
NOT active against ANY Anaerobes!
Amino’s require O2 to cross cell membrane
Great for impregnating in beads because they are heat stable, low reactivity, water soluble. (ratio is 5:1)
Toxicities:
Nephro  reversible starts several days after initial tx, see significant increase in serum Cr. + vanc
 worse
Oto  Irreversible, generally not seen for a few days.. Auditory  tinnitus, loss of hi freq, test by
audiometric testing… Vestibular  dizzy, vomit, loss of balance in dark, nystagmus…
Neuromuscular Blockade: drug accumulates at NM junction, inhibiting Ach release from n. The
only way u get this is with Bolus administration  therefore infuse over 30 minutes

Peaks And Troughs: (management and timing)


Peaks are drawn immediately following administration of dose
Troughs drawn 20-30 min before administration of next dose
Levels should be ordered initially after 3rd dose…
If WNL, do not need to order again unless there is a rise in serum Creatinine level. (Cr should be
monitored about 3x/week)
*Gent and Tobra: peak = 6-10, trough = less than 2 ug/ml
*Amikacin: peak = 20-30, trough = less than 10 ug/ml
Techniques for changing doses: {change amt or interval}
Peak is Hi, trough is normal  Dec. amt
Peak is Lo, trough is normal  Inc. amt
Trough is Hi, peak is normal  Inc. interval
Peak and Trough are HI  Dec dose & interval

PAIN MANAGEMENT & CONCIOUS SEDATION ---------------------------------------------------------


*Remember – the duration of onset of reversal agents and analgesics or sedatives may be different… the
reversal agent may wear off before the agent, leaving the agent to overcome the pt…

VALIUM (Diazepam) – Benzodiazepine


• onset = 1-5 minutes, half life = 30 hrs
• sedative dose = 10-12mg
• lasts 2-3 hrs

VERSED (Midazolam) – Short acting Benzodiazepine


• 4 X more potent than Valium
• Amnesia
• Watch for hypotension, mental and motor defects
• onset = 3-5 min, half life = 1.2-12.3 hrs
• sedative dose = 2.5-7.5mg
• lasts 30-40 min

FLUMAZENIL (Romazicon) – BZD Antagonist / competitive at GABA receptor – Versed reverse


• Lasts 30-60 min
• Onset = 1-2 min
• Initial Dose = 0.2mg. May repeat at 1 min intervals to dose of 1mg

MORPHINE SULPHATE – Opioid Narcotic


• Dose = 1-5mg IV over 1 min, titrated every 6-10 min until analgesia is reached. (max =
12mg/hr)
• Lasts approx 4 hrs
• Watch O2 sats… respiratory depression

DEMEROL (Meperidine) – weaker Narcotic


• Onset = 3-5min, half life = 30-45 min
• Avg Dose = 20-50mg
NALOXONE (Narcan) – Narcotic Antagonist – Morphine reverse
• Lasts 20-30 min
• Initial dose = 0.4 mg. May repeat every 2-3 min @ doses of 0.4-2 mg

PERCOCET: (Oxycodone/Tylenol) class 2


• 2.5/325, 5/325, 7.5/500, 10/650 – 1 tab po q4-6h

VICODIN: (Hydrocodone/Tylenol) class 3


• 5/500 1-2 tabs po q4-6h
• ES (7.5/750) 1 tab po q4-6h
• HP (10/660) 1 tab po q4-6h

Thanks Mikey!:
Remember that all narcotic brand names that end with "-cet" have Tylenol in them, while ending with "-
dan" is ASA. Ex - Percocet has Tylenol, Percodan has ASA

NON-Codeine  STUUD
STADOL (butorphanol) (opioid agonist-antagonist)
0.5-2mg IV or 1-4mg IM q3-4h prn pain
TORADOL (ketorolac/NSAID)
*30mg IV bolus or 60mg IM over at least 15sec
15-30 mg IV/IM q6h or 10mg PO q4-6h prn moderately severe acute pain… do not exceed 5 days
 renal toxic
starts in 30min-1h, duration = 4-6h
10mg PO q4-6h prn pain
max daily dose = 120mg
*ULTRAM (tramadol)
50-100mg PO q4-6h prn mod-sev pain. Max = 400 mg/d
ULTRACET (tramadol 37.5mg + tylenol 325mg)
2 tabs PO q4-6h prn acute pain. Max = 8 tabs/day X 5d
*DARVOCET (propoxyphene + tylenol)
50/325 (N50) 1-2 tabs PO q4h prn pain
100/500 (N100) 1 tab PO q4h prn pain
DEMEROL (meperidine)
1-1.8 mg/kg up to 150mg IM/SC/PO or slow IV q3-4h
75mg IV/IM/SC = 300mg PO

Random:
ETOMIDATE – good to knock out pt for 5-10 min… you cant push this… ER doc has to… 10mg is
good… (sedative like Propofol)

NSAIDS:
COX 1 groups:
Salicylic Acid: ASA, diflunisal
Proprionic Acid: ibuprofen, naproxen
Acetic Acids: ketorolac(toradol), indomethacin, sulindac
TORDADOL – Strong, IM / IV / PO 15-30mg q6h
Do not exceed 5 days in a row  Renal
Fenamates: meclofenamate
Oximcam: meloxicam
3 things you will get called for if not in orders:
Sleep: Ambien PO qhs prn sleep
Pain:
* Morphine IV 2-4mg q2-4h prn severe pain
* Percocet PO 5/325mg q4-6h prn moderate pain
N/V: Phenergan IM 12.5mg q4-6h prn N/V

Local Anesthetic Max single Doses (avg 70kg male/60kg fem)


Drug Class Max w/Epi Max NO epi
Procaine ester 1000mg 750mg
Lidocaine amide 500mg 300-350mg
Marcaine amide 225mg 175mg
1% soln contains 10mg/ml
0.5% soln contains 5mg/ml
0.25% soln contains 2.5mg/ml

Adjustments to Local Anesthetic Doses  Peds


Child - Clarks rule: (weight in lbs / 150) x adult dose
Infant - Fried’s rule: (age in months / 15) x adult dose

EMERGENCY MEDICINES ---------------------------------------


ESMOLOL – Anti-HTN / B-blocker
• Initial dose = 0.25-1.0 mg/kg over 30 sec.

EPHEDRINE – Tx hypotension – sympathomimetic


• Initial dose = 5-10 mg

ATROPINE – Tx Bradycardia / Asystole – Anti-Cholinergic


• Initial dose = 0.25 – 1.0 mg
o Repeat every 3-5 min, max = 0.03 mg/kg
EPINEPHRINE – tx anaphylaxis
• 0.3-0.5 ml subQ of 1:1,000 solution, + antihistamine

Alarm Limits
LOW HIGH
Systolic 85 150
Diastolic 50 100
Rate 50 110
spO2 92 100

BLOOD CLOTTING ---------------------------------------------------


DVT: fat forty female smoker

Risk factors: IAMCLOTTTTED


Immobilization / Iatrogenic / Inflammatory bowel disorder / Inherited / Venous Insufficiency, Arrhythmia,
MI / Malignancy / Mother(pregnant), Coagulopathy / CHF, Long term Immobility, Obesity / Old age,
Trauma / surgery, Tourniquet, Tobacco, previous Thromboembolic event, Estrogen use, Diabetes
Virchows Triad: Endothelial Damage, Hypercoaguable state, Hemostasis

Diagnosis: venogram is gold standard, Doppler is non-invasive, Homan’s and Pratt’s sign, apply tourniquet
above suspected thrombosis  pain at level of clot in 30-45 sec.

PE Dx: gold standard = V/Q scan (ventilation / perfusion), spiral CT is very common 1st line, ST segment
depression, dec pO2, pCO2, pH.
Prophylaxis: TED stockings, elevation of legs with knees flexed, early mobilization
Heparin 5000 units SQ bid or beginning 1 hour pre-op, then 5000 units SQ q12h until pt
ambulates
Lovenox 30mg SQ bid

Labs: (deficiencies in certain factors will INC. values)


PTT – factors 1,2,5,8-12 (all but 7) Intrinsic [norm = 30-45 s]
Prolonged in Heparin therapy, Normal in Von Willenbrand dz (7), initiated by factor 12
PT – factors 1,2,5,7,10 (Vit K) Extrinsic [norm = 11-16 sec]
Prolonged in liver dz, vit K disorder, Coumadin
Bleeding time – inc. in Von Willebrand dz [norm = 1-4 min]
Prolonged in all platelet abnorm, norm in coag disorders
D-Dimer – nonspecific marker in blood for clotting good to rule out, not in… 0-300 = norm, over 300 is
positive
* All are produced in Liver except factor 8 (Intrinsic)
Platelet Count – norm = 250,000/uL of blood (50,000 = minor bleeding)

Treatment: Assess PT/PTT to get baseline,


Then IV Heparin 5,000-10,000 units, then 800-1,500 units per hr, maintaining PTT @ 2-2.5 times baseline
daily. pt maintained at bedrest with LE elevated above hrt.
*It takes approx 1 week for thrombi to become firmly adherent to endothelium and diminish risk of PE.
Coumadin is started when long term therapy is planned. Tx 4-6w for calf DVT, and 3-6 months for
proximal DVT.

Coumadin dose: extrinsic – 2,7,9,10 liver = all but 8


takes 16-48h to cause measurable changes in PT, therefore begin tx 2 days before D/Cing heparin. Don’t
start Coumadin until about day 3 of Heparin therapy… D/C heparin when PT is therapeutic.
Keep INR 2 x normal
Loading dose 10mg qd until PT increases, then smaller dose (5-7.5mg) to maintain PT in therapeutic range.
No adjustments for renal failure

Lovenox prophylaxis: 30mg SQ bid


Lovenox DVT Tx: 1 mg/kg bid

Heparin Reversal: Protamine Sulfate 1mg per 100 units of heparin


Coumadin Reversal: FFP + Vit K
FFP has all coagulation factors in it…
**Peri-Op:
Heparin – hold 8 hours prior to Sx (monitor PTT)
DVT prophylax – give 5,000 u SQ 1hr pre-op, then bid until ambulatory
Coumadin – hold 3-4 days prior to Sx (monitor PT/INR)

STUFF------------------------------------------------------------------------------------------------------------------------
ASA:
1. Healthy
2. Mild systemic Dz
3. Severe systemic Dz
4. Severe systemic Dz + constant threat to life
5. Morbid, will not survive w/o Sx
6. Brain Dead (organ donor)

Mallampati:
1. Entire uvula and tonsilar pillars visible
2. Tip of uvula and pillars hidden by tongue
3. Only soft palate visible
4. Only hard palate visible
History reminders:
CC, HPI – onset, duration, type, trauma, radiating, exacerbates, relieves, pain 10/10, previous episodes,
severity, previous tx…
PMH, Meds, Allergies, Hospitalizations, Illness
SurgHx
SocHx – smoke, drink, drugs
FamHx – DM, HTN, cancer…
ROS – gen, skin, nodes, EENT, Breast, resp, cv, gi, gu, heme, endocrine, m/s, psych, neuron

Residency Interview System of Thought: remember, they would rather you ask too many questions in the
history than not ask enough… they will stop you… don’t ask for stuff you don’t need, cuz then your just
guessing… they will let you dig your own hole, but will not lead you astray…
On the phone with ER  ask  stable?, fever?, N/V intact?, get x-rays, CBC w/diff, CMP, blood
cultures if appropriate (f/n/v, taken from 2 diff spots 20 min apart)… go ahead and get the ball rolling
before u get there… 23h admit, IV Abx… start thinking empiric…
1. NLDOCAT – how long ago, where, pain, cause, did you walk on it, NPO status!, tried to treat it
yet, something makes it worse, better? Infxn – had an ulcer before?, past Tx for it? Failed oral
Abx?...
2. PMH, PSH, All, Med, SocHx, NPO if you forgot
3. Hx of MRSA  Vanc and Zosyn… MRE  Linezolid(Invanz) or Zyvox… 1st time infxn 
Unasyn
4. PE: vitals, vasc, neuro, derm, ortho…
a. if no pulse  Doppler, if not  TcPO2
b. open wound?, edema, eccymosis, erythema, digits cyanotic
c. Probe to Bone? (89% +), undermine, fibrotic, granular, proximal streaking
d. odor? (fruity - pseudo, pungeont – gas gangrene),
e. drainage – purulent, serosanguinous, serous, dirty dishwater  clostridium
f. ortho – dislocated, gross deformity, pain out of proportion, ROM, POP
i. piano key test – DF/PF digits  pain = compartment syndrome
g. Imaging – XR (most everything), MRI (infxn), CT (calc fx, pilon)
h. Labs – CBC w diff, CMP
i. Closed Reduction:
i. Charnley’s 4 steps – increase deformity, distract, decrease deformity,
cast/splint
ii. Put on hrt monitor, pulse Ox, O2 (nasal cannula)
iii. What was given to pt in ER??
iv. Push 2 & 2 or 3 & 3 mg Morphine and Versed or have ER doc give them
Etomodate
5. Be prepared to reverse both of these meds  Naloxone (Initial dose = 0.4 mg. May repeat every
2-3 min @ doses of 0.4-2 mg) and Flumazenil (Initial Dose = 0.2mg. May repeat at 1 min
intervals to dose of 1mg)
6. IV Abx, Pain Meds…
7. Schedule for Sx  I & D, Amputation, ORIF…

PIMP LIST --------------------------------------------------------------------------------------------------------------


1. Indirect dx of exercise induced compartment syndrome?
Put sphygmomanometers on each calf and pump up until pt feels pain… affected side will feel
pain at ½ of the pressure of the unaffected side
2. Volkman’s contracture? Compartment syndrome  ischemic necrosis  contracture of muscles  can
cause HDS if QP is killed by compartment syndrome in Calcaneal compartment… can get renal failure due
to myoglobinuria.
3. Arterial supply to talus? In order of significance? PTa, ATa, Perforating Peroneal a.
4. Principles of AO: Anatomic Reduction, Stable int. fixation, Atraumatic Technique, Early and pain free
ROM (prevent cast dz)
5. Galvanic Corrosion? Rxn b/t titanium and stainless steel
6. Albumin – rising = anabolic, dropping = catabolic(bad). Renal failure  dec albumin, inc BUN/Creat,
dec. urine.
7. TcpO2 – 30 + is ok, ABI over 0.45 is ok, good toe pressure = 30mm Hg
8. RIFLE classification of Acute Renal Failure:
Risk – inc in serum creat X 1.5 or dec in GFR by 25% or Urinary Output <0.5mL/kg/h for 6 hours
Injury – 2.0, 50%, 12 hours
Failure – 3.0, 75%, 24 hours or anuria for 12 hours
Loss – persistent ARF – no kidney function > 4wks
ESRD – loss of kidney function > 3 months
9. Eponym for nerve medial to 1st mpj? Joplin’s n
10. Phalen’s sign? Compress n X 30 sec  re-create symptoms
11. How much force is required to break the lateral ankle ligaments? According to Attarian et al, the
maximum load to failure for the CFL was 2 to 3.5 times greater than that for the ATFL (345.7 versus 139
newtons).
12. Angle between ATF and CF? 105 deg.
13. Post-op course for Brostrom? 3-4w NWB, 3-4w CAM walker, PT @ 6w
14. CFL is extracapsular
15. Brostrom Gould… who came up with what? Brostrom is primary repair of lig’s, Gould is imbrication
(overlapping) & suturing of inferior Extensor retinaculum to reinforce…
Vest over Pants stitch
16. Rix modification? Anchovie procedure… Keller with interpositional tendon or capsule pulled down
into jt.
17. Moberg procedure? Closing base wedge with base dorsal of proximal phalanx of hallux.
18. Difference between community and hospital acquired MRSA?
CA - generally susceptible to a wider range of antibiotics (usually susceptible to fluoroquinolones
and TMP/SMX. Unlike HA-MRSA, CA-MRSA often arises in children and adults without any obvious
risk factors.
HA - 52% of S. aureus isolates recovered from patients in ICUs and 42% of S. aureus recovered
from non-ICU patients are MRSA. Most are resistant to macrolides, fluoroquinolones, clindamycin, and
TMP/SMX.
CA-MRSA can be differentiated from HA-MRSA if the person meets all of the following criteria:
diagnosis of MRSA was made in the outpatient setting or by a culture positive for MRSA within 48 hours
after hospital admission; no past medical history of MRSA infection or colonization; no hospitalizations,
admission to a nursing home, skilled nursing facility, or hospice, dialysis, or history of surgery in the past
year; and no permanent indwelling catheters or medical devices that pass through the skin into the body.
19. Causes for Peroneal Subluxation? Shallow groove, muscle belly extending into groove, trauma…
20. Peroneus Longus ruptures more often in what foot type? Cavus
21. Will it damage the mechanics of the foot if you repair PB rupture by tenodesis to PB? No
22. Most common location for PL rupture? @ bend under cuboid
23. H & H cutoff for doing surgery? 30 & 10 for elective cases
24. % Muscle strength lost per grade? 50% lost in each grade
25. Ok to use self tapping screws in osteoporotic bone? NO
26. “worm gearing” in internal fixation? Crossing 2 screws and having them purchase eachother  better
stability in osteoporotic bone. (to imagine it, hold out your index finger and cross them like swords…
that’s how it would look like inside the bone)
27. Talar position in calc fx? Dorsiflexed
28. Deforming force in varus in calc fx caused by? Achilles & possibly sustentaculum tali.
29. 6 P’s of Compartment Syndrome? Pain, Paresthesia (30 min), Paralysis (late), Pressure (over 30mm
Hg), Pink (good CFT unless severe case), Pulses (present unless arterial dz)
30. Precipitating factors to DKA? Infxn, omission of insulin, new onset DM
31. Drugs that may induce diabetes? Thiazides, Diuretics, Steroids, Phenytoin
32. What should blood glucose be for DM pt the am of Sx? 100-200
33. Mechanisms leading to HDS?
Flexor Stabilization – pronated foot  hypermobility of FF  flexors fire early in gait to
stabilize FF  overpower interosseous muscles leading to hammering of digits… Most common etiology
Extensor Substitution - EDL overpowers lumbricles  a strait dorsal contracture of MPJ’s
Flexor Substitution – Supinated foot  weak triceps surae  deep muscles try to substitute (TP,
FHL, FDL) fire earlier and longer  contracture of lesser digits… Least common etiology
34. AO screw size and order?
Over, under, counter, measure, tap, screw…
Mini frag – 1.5/1.1 2.0/1.5 2.7/2.0
Small frag - 3.5/2.5 4.0/2.5
Standard - 4.5/3.2 6.5/3.2
35. Bone Healing Stages
Inflammation (2d-1w) – pain = protective, edema = splint
Soft Callus (days-2m) – fibrous cartilage forms at each end
Hard Callus (3-4m) – begins to ossify if ends are <1cm apart
Remodeling (yrs) – callus resorbed, wolff’s law  bone will grow to meet the load placed on it…
35. 2 types of Bone Healing?
1o membranous – haversion remodeling – simultaneous remodeling and formation of bone, little/no motion
or callus, good apposition  desirable
2o enchondral – cartilage/fibrous intermediates replaced by bone, involves callus/irritation, motion  NOT
desirable
36. Types of Non-Unions? (Weber & Cech)
Hypertrophic/Vascular – elephant foot, horse hoof, oligotrophic
Tx = stable fixation alone
Atrophic/Avascular – torsion wedge, comminuted, defect, atrophic
Tx = decortication, bone graft, bone stimulator
37. Bunion Sx to correct high IM and PASA? Bicorrectional Austin, reverdin laird and todd, hohmann,
logroscino, Peabody
38. What is Locroscino procedure? Reverdin + Loison-Balacescu. Take wedge from the reverdin and
instet into an opening ABductory wedge at the base
39. Significance of Talar neck Fx (Hawkins)?
AVN STJ arthritis
I – nondisplaced 10% rare
II – STJ 30% 60-75%
III – STJ/AJ 90% 100%
IV – STJ/AJ/TNJ 100% 100%
40. Vascularity to Talus: specific?
PTa  a of tarsal canal (most of body)
 posterior tubercle vessels
DP/ATa  superior neck/head vessels
Perf Per a  a of tarsal sinus (plantar neck/head)
41. Fill in the blanks?
Dale Austin, DPM & Edward Leventen, MD published in 1981
42. Other classification system of AJ Fx? Ashurst and Bromer
43. Where are the following sweat glands found?
Eccrine – palms, soles, axillae
Apocrine – axillae, genitals, scent gland, puberty activates
Sebaceous – all except palms, soles
44. Primary blood supply to long bones? Endosteal
45. IM nail for fibula? Ruch rod
46. Test for Morton’s neuroma other than Mulder’s? Gauthier’s test – side to side compression with
DF/PF toes  pain = +
47. Tx for nail/digit trauma?
Subungual hematoma  up to 25% = trephination (drill), over 25% = avulse nail, vicryl close
Buddy tape 4-6w with fracture
47. Most common closed digit fx? 5th – bedpost fx
48. Most common complication with crush injury to digit? N entrapment/ lasting pain
49. Gissane’s Critical & Bohler’s Angles?
* Gissane’s Angle formed by lateral process of talus and lateral
surface of calcaneus. Norm = 120-140. Angle INCREASES in calc
Fx

* Bohler’s Angle formed by tent like angulation which the sup/post


aspect of the calcaneus makes with sup/ant aspect. Norm = 25-40.
Angle DECREASES in calc Fx.

50. At what stage of intrauterine development do the lower limb buds first appear? 5th Week
51. Where is the tubercle of Tillaux-Chaput? Tibial side of the distal tibio-fibular joint.
52. Ligaments making up the Deltoid?
Tibio-Navicular Ant. Talo-Tib
Calcaneo-Tibial Superficial Deep Post. Talo-Tib
Posterior Talo-Tibial
53. Pneumonic for Diagnosis Options in most cases? TICAN
Trauma, Infxn, Congenital, Acquired, Neoplasm
54. Why can’t you give NSAID’s to Asthma pts? COX inhibition converts Arachidonic Acid Pathway to
produce more Leukotrienes, which are BronchoCONSTRICTORS!
55. Pyoderma Gangrenosum? Nodule or pustule that breaks down  raised, inflammatory border,
boggy, necrotic base, pearly, rolls in, punched out, purulent drainage with hemorrhagic exudate, dusky
red/PURPLE border. Associated with inflammatory disorders (Crohns, IBS, UC, RA). Histo – see mostly
neutrophils. Tx – oral steroids, sulfa drugs, cyclosporine. PAPA syndrome –Pyogenic Arthritis,
Pyoderma, cystic Acne
56. Where can these ligaments be found?
Spring – from sust. Tali  navicular tuberosity
Short Plantar – from ant. Tubercle of plantar calc.  proximal aspect of peroneal ridge on cuboid
Long Plantar – from dist. Posterior aspect of medial calcaneal tuberosity  peroneal ridge (cuboid) and
radiating to base of 2-5 MT’s
57. Nutrient a’s to these bones come from what a?
Tibia – Post. Tibial a
Fibula – Peroneal a
Femur – Perforating branch of profunda femoral a.
58. Major pronator of foot? Peroneus Brevis
59. How does insulin work? Stimulates glycolysis & inhibits gluconeogenesis
60. Clinical findings of Osteogenesis Imperfecta? Multiple fx’s and blue sclerae
61. Syphilis caused by? Treponema Pallidum
62. 3o syphilis characterized by? Neuro-syphilis (tabes dorsalis), gummas, argyl-robertson pupil
63. Boney lesions presenting “codman’s triangle”? osteosarcoma and chondrosarcoma
64. Characteristic radiographic findings of:
giant cell tumor  “soappy bubble appearance” with lytic borders
UBC  fallen fragment sign
OsteoChondroma  pointing away from adjacent joint
65. Dosage for:
Amoxicillin – 250-500 mg pot id
Pen G – 12-24 million u / day divided by q4-6h
Timentin – 3.1 gram IV q4-6h
Zosyn – 3.375 gm IV q4-6h or 4.5 gm IV q8h
Unasyn – 1.5-3.0 gm IV q6h
Augmentin – 875 mg po bid
Erythromycin – 250-500 mg po qid
Vanc – 1g IV q12h (or 125 mg po qid for c. diff only)
Clinda – 600 mg IV q8h or 300 mg PO bid-tid
66. Drugs active against Pseudomonas? Fortaz, Aminoglycosides, Ticarcillin(Timentin), Clindamycin,
Imipenem (Primaxin), Aztreonam
67. Peak/Troughs? (ug/ml)
Vanc - 25-40 / 10
Gent - 4-10 / 2
68. Which agents would you use for a pt with lime dz? Tetracycine’s
69. According to Ruedi & Allgower, what 4 principle steps must be followed in ORIF of comminuted Fxs
of distal tibia?
1. Reconstruction of Fibula
2. Reconstruction of Lower articular surface of Tibia
3. Packing in of cancellous bone
4. Stabilization of medial aspect of tibia
70. Tx for Abx induced Diarrhea? D/C Abx, no anti-diarrheal agents, fluid/electrolyte support, Metro 500
mg po q8h or Vanc 125 mg po q6h
71. Local Anesthetics  Type and metabolism?
Ester (met by plasma esterases): Procaine, Cocaine,
Chlorprocaine, Tetracaine
AmIde (met by liver enzymes) {2 I’s}: Lidocaine, Mepivacaine,
Pilocaine, Bupivicaine, Etidocaine, Ropicacaine
72. Maximum Dose of Lidocaine and Marcaine?
Lidocaine  4.5 mg/kg………7.0 w epi
Marcaine  2.5 mg/kg……….3.2 w epi
73. Abx for MRSA? Vanc, Minocycline, Cipro/Rifampin, Bactrim/Rifampin, Linizolid, Cynercid
74. Dosage for:
Ambien – 5-10 mg po qhs prn insomnia (non-BZD hypnotic)
Benadryl – adult = 25-50mg po tid prn pruritis
Peds = 12.5-25mg pot id prn pruritis
Compazine – 50-100mg po q4-6h or 50mg IM q4-6h prn N/V
Percocet – 1-2 po q4-6h prn pain
75. Analgesic for Codeine/Morphine Allergic pt?
Darvon: 65mg po q4h
Darvon-N: 100mg po q4h
76. Thurston-Holland sign? Spike of metaphyseal bone attached to the fractured epiphysis seen in salter
harris 2 fractures.
77. Johnson & Strom Classification System? PTTD
Stage 1: tendon normal, peritendonitis, mild weakness on single heel raise test, normal hindfoot alignment.
Tx = break cycle of inflammation with 3-6 months of rest, NSAID’s, orthoses, synovectomy, tendon
debridement, BK cast 3w.
Stage 2: tendon elongated, mobile valgus RF, more POP to tendon, FF abduction (TMT sign), marked
weakness on single heel raise test, XR signs of pronation, MRI  discontinuity, balling up of tendon. Tx =
FDL transfer to nav, BK cast 6w + PT, Kidner, Young, STJ arthroeresis, Evans, Dwyer, Medial slide.
Stage 3: elongated tendon, stiff valgus RF, more TMT sign. XR  stage 2 + DJD. Tx = STJ arthroeresis
Stage 4: (added by Myerson) AJ involvement  valgus. Tx = pantalar or tibio-talo-calcaneal fusion.
78. What is Basset’s Lesion? Lesion on ant dorsal lateral aspect of articular cartilage of talus caused by
rubbing from hypertrophic anterior inferior tib-fib ligament.
79. Brodies Abscess? Foci of bone destruction caused by OM filled with pus or connective tissue.
80. Charcot’s Triad? Symptoms of Multiple Sclerosis consisting of nystagmus, intention tremor, scanning
speech
81. Coleman Block Test? Determines if associated varus deformity of the RF is flexible or rigid. Place
pts foot on wooden block one inch thick (placing heel and lateral foot on block and medial MT heads off
block) if RF corrects, it’s a FLEXIBLE condition.
82. What is a Crescent Sign? Early sign of AVN which represents a subchondral Fx through insertion of
the individual trabeculae.
83. Crowe’s Sign? Axillary freckling, pathognomonic for Von Recklinghausen’s dz.
84. LE pathology occurring with Ehlers-Danlos Syndrome? Ligamentous laxity resulting in flat feet, genu
valgum, congenital hip dislocation, scoliosis.
85. Possible causes for foot drop? CVA, Trauma, CMT, Polio, Freidrichs Ataxia, Infxn, Guillian-Barre
Syndrome, Dejerine-Sottas Syndrome, Spinal Tumor/lesion.
86. What is the Hubscher Maneuver? DF the hallux in WB, the arch will rise due to windlass mechanism
if no osseous restrictions are present.
87. What is the Kelikian Test? Tests whether or not the MPJ is reducible. Push up on plantar surface of
MT head and see if toe straitens out.
88. Another name for the Intermediate Dorsal Cutaneous N? Lemont’s nerve
89. What is Marjolin’s Ulcer? A SCC that arises in chronic sinus due to OM. Must be long standing tract.
90. What is MosaicPlasty? OATS, Transplantation of cartilage and bone by way of a plug to fill a defect
caused by OCD.
91. Name the material utilized to make Abx impregnated beads? Polymethylmethacrylate (PMMA)
92. Requirements for Abx to be used in Abx impregnated beads? Broad spectrum, low allergenic, heat
stable, ability to leach from cement.
93. Floor and Roof of Porta Pedis? ABductor Hallucis  floor, Quadratus Plantae  roof
94. Somogyi Effect? Rebound phenomenon occurring in diabetics who take too much insulin in the
evening resulting in hyperglycemia in the morning. When blood glucose levels drop too low, the body
sometimes reacts by releasing counterregulatory hormones such as glucagon and epinephrine. These
hormones spur the liver to convert its stores of glycogen into glucose, raising blood glucose levels. This can
cause a period of high blood sugar following an episode of hypoglycemia.
95. Turk’s Test? Test whether n is entrapped due to varicosities beneath retinaculum. + if symptoms
increase once tourniquet is inflated above venous pressure but below arterial pressure proximal to site of
suspected entrapment.
96. Virchow’s Triad? Stasis, blood vessel injury, hypercoag
97. Valleix’s & Tinel’s sign? V – Proximal. T – Distal
98. Normal WBC count? 5,000 – 10,000 cells/ul
99. Normal Hemoglobin & Hematocrit Values?
Male  14-18 g/dl & 40-54%
Female  12-16 g/dl & 37-47%
100. What tests are in a CBC w/diff? WBC, RBC, Hgb, Hct, MCH, MCHC, MCV, Platelets, Segmented
Neutrophils, Eosinophils, Basophils, Lymphocytes, Monocytes
101. Normal PT and PTT values?
PT = 10.1-13.1 sec PTT = 23.5-34.3 sec
102. Tx for Malignant hyperthermia? Stop Anesthetic, Hyperventilate with 100% O2, Cool the pt,
Dantrolene sodium.
103. Dosing regimen for Dantrolene Sodium?
Intra-Op Crisis: 1mg/kg continuous Rapid IV Push until symptoms subside or max dose of 10mg/kg
reached
Post-Op Crisis: 4-8mg/kg/day po in 4 divided doses X 1-3 days
Pre-Op Prophylaxis: 4-8mg/kg/day po in 4 divided doses for 1-2 days prior to Sx with last dose 3-4 hrs
before scheduled Sx. Or 2.5mg/kg IV approx 1hr before anticipated anesthesia.
104. Causes of Post-Op Fever? (5 W’s)
Wind (12-24h) – atelectasis, post-op hyperthermia
Walk (24h) – Thrombophlebitis, PE
Water (2d) – UTI
Wound (3d) – Infxn
Wonder Drug (anytime) – Drug Fever (PCN…)
105. 4 Reasons for severe intractable pain post-op? Sutures too tight, constrictive dressings, hematoma,
ischemia
106. Tx for White Toe after surgery? D/C ice and elevation, place foot in dependent position, loosen
bandage, warm compress proximal to N/V bundle, piston/rotate toe on k-wire, local nerve block proximal
to area, vascular consult.
107. Differentiate between blue toe due to venous vs arterial insufficiency? Venous  warm & may
blanch with pressure. Arterial  cold and does not blanch with pressure.
108. How many grams of carbohydrate are found in one liter of D5W? 50gm
109. Indications for AJ fusion? Arthrosis, severe angular deformity, irreparable acute trauma, bone tumor,
chronic OM, failed AJ prosthesis, Paralytic deformities, salvage of avn of talar body.
110. Other name for clubfoot? TEV
111. Primary defect in clubfoot? Talar head and neck
112. Adduction of talar head/neck in transverse plane on body?
Norm = 15-20 deg. TEV = 80-90 deg.
113. Types of Clubfoot?
Congenital  Intrinsic(rigid) & Extrinsic(supple)
Acquired
114. Classic Components of Clubfoot? FF ADDuction, RF Varus, RF or Ankle Equinus
115. AP TC angle, norm and Clubfoot?
Norm = 20-40 deg. TEV = 0 deg
116. Factors Delaying Surgical Wound Healing?
Local: incisions improperly placed within RSTL’s, Tissue Trauma to Wound lobes, improper irrigation
(FB in wound), inadequate hemostasis, desiccation of tissues during surgery, infxn, prolonged dependency,
local corticosteroids
Systemic: uncontrolled DM, EtOH, malnutrition/malabsorption, steroids, anemia, platelet inhibiting drugs,
obesity, age, hepatic dz
117. Goals of AO? Anatomic Reduction, Atraumatic Technique, Stable internal fixation, early active
mobilization
118. Ideal fixation for physeal fx’s? smooth K-wire
119. Composition of Orthosorb pin? Poly-O-Dioxanone
120. How are bioabsorbable materials discarded by the body? Degraded by hydrolysis – eventually
entering into KREB’s cycle and excreted into urine, feces, or expired CO2.
121. Advantages of biodegradable fixation?
Avoids stress shielding or stress protection osteopenia
Eliminates burden of secondary procedure
Does not lead to image artifacts when using MRI or CT post implantation
May be left in sites of infection as implants are bacteriostatic!
122. How long will suture maintain its strength? ~60d
123. Secondary causes of DM? Pancreatic dz, hormonal (cushings, acromegaly, pheochromocytoma),
drug induced, genetic syndromes.
124. Diagnostic tests for DM? Hemoglobin A1C, oral glucose tolerance test, fasting BS > 140 mg/dl on
more than 1 occasion
125. What is Whipples Triad? Whipple's triad or Whipple's criteria refers to three conditions that are
considered by physicians necessary for proving hypoglycemia as the cause of a person's symptoms.

1. Symptoms known or likely to be caused by hypoglycemia


2. A low glucose (40 or less) at the time of the symptoms
3. Relief of symptoms when the glucose is raised to normal

126. At what point in BS is pt Hypoglycemic? <50


127. Dosing for Metformin? 500mg po bid
128. Dx test for DVT? Homan’s, Pratt’s, Doppler, Venograph
129. Dx test for PE?
Blood Gas (PaO2 < 80% mmHg with PaCO2 < or = norm
EKG  Tachycardia (acutely)
V/Q scan  area of ventilation but no perfusion
130. Reversal Agent for Heparin overdose?
Protamine Sulfate (1mg / 100u heparin)
131. Peak onset of Coumadin? 3-5d (started once heparin is therapeutic)
132. Best Hematologic index of renal function? Creatinine (CrCl)
133. Dx of Septic Arthritis? Gram Stain, + synovial fluid culture, Juxta-Articular osteopenia after onset
(7-10 days)
134. Phases of Technetium-99 Bone Scan?
1. Angiogram (just after injection)
2. Blood Pool (2-5 min)
3. Delayed (2-4 hrs)
4. Fourth (5-24 hrs)
135. Most common organism causing necrotizing fasciitis? Strep Pyogenes
136. Other name for Necrotizing Fasciitis? Hemolytic strep gangrene
137. Nikolsky’s sign? Shearing of normal epidermis in response to lateral pressure in staphylococcal
scalded skin syndrome.
138. Organisms causing T. Pedis? T. mentagrophytes, T. Rubrum, E. Floccosum
139. Koebner’s Phenomenon? Replication of skin in pressure areas with Psoriasis
140. Abx for Sx Prophylaxis in PCN allergic pt?
Cleocin 600mg IV or Vancomycin 1g IV
141. Dosing for Lamisil? 250 mg/d X 12w (90 days)
142. Sequential Release of hammertoe deformity following ineffective release of contracture demonstrated
by Kelikian Test?
1. Arthroplasty at PIPJ
2. Extensor hood resection
3. Extensor tendon lengthening
4. Dorsal Capsulotomy
5. Plantar Capsule release
6. Arthrodesis / flexor transfer to extensor
143. Which cortex is left intact in a peg-in-hole fusion? Dorsal
144. How should administration of colchicine be altered in pt with renal/hepatic dysfunction as well as the
elderly? Reduce by 50%
145. Xanthine Oxidase inhibitor? MOA?
Allopurinol (for overproducers)
Block the final step in urate synthesis

146. Uricosuric? MOA?


Probenicid
Increase renal excretion of uric acid
147. Dosing for Allopurinol? 300 mg/d
148. Muscles in Layers of Plantar Muscles?
1st – ABDuctor H, FDB, ABDuctor DM
2nd – Quadratus Plantae, Lumbricles
All attach in some way to FDL
3rd – FHB, ADDuctor H, FDM
4th – Plantar and Dorsal Interossei
th
149. 4 IM angle numbers? Buckholtz and Fallot
IM: Norm = 6.47 Pathologic = 8.71 (over 9  symptoms)
LatDev: N = 2.64 Path = 8.05
150. Rate of lengthening with Callus Distraction?
0.5 mm/d to 1 mm/d…. 4 x ½ turns per day
151. Normal Angle in Bunion Management?
MAA = 15, HI = 0-10, HAA = 10-15, IM = 8-12,
PASA/DASA = 7.5, IMA with MT adductus = 8-10
152. Procedure to correct a high HI angle  distal akin
153. The 5 main processes in bone graft healing?
1. Vascular ingrowth (1-2w)
2. Osteoblast proliferation (new bone formation)
3. Osteoinduction (BMP=inducer)(nonosseous tissue produce bone)
4. Osteoconduction (creeping substitution) (bone graft = scaffold)
5. Graft Remodeling (reformation of graft via wolff’s law)
154. Fowler and Philip Angle? Norm = 44-69 >75 = haglunds

Ruch said  if the F/P angle(44-69) + Calc Incl angle (20-52)


If the sum is greater than 90  Pathology
Pavlov’s Parallel Pitch Lines: Line 1 is the Calc Inclination angle line. Line 2 is Parallel to Line 1 and is
at the level of the most superior point of the posterior facet of the calcaneus. If any part of the posterior-
superior aspect of the calcaneus is ABOVE Line 2, that indicates pathology.
155. Procedures for Haglunds?
Keck and Kelly – wedge from post/sup calcaneus
Duvries – Transverse bumpectomy via lat. incision
Fowler & Philip – transverse skin incision through posterior heel
Dickenson
156. Foot type in pseudo-equinus? Cavus
157. Hoke TAL? Triple hemi-section with 1st and last section medially, and the central cut laterally
158. Flatfoot procedures to correct planar abnormalities?
Transverse: Evans, CCJ distraction fusion,
Kidner - resect accessory navicular/tuberosity, transposition of TP insertion from more lateral
navicular attachment to a more plantar navicular attachment
Sagittal:
Lowman - TA inserts into spring lig, TaloNav fusion, Tal), Hoke - plantar base wedge/fusion of
Nav-Cun(1&2) + TAL
Miller – Nav-1stCun-1stMT + PT and spring lig advancement using osteoperiosteal flap
Cotton – Opening Dorsal wedge in 1st Cun
Lapidus – 1st MT-Cun fusion
Young – Re-route TA through keyhole in Navicular
TAL/Gastroc
159. Compartments and associated muscles?
Full length of foot:
Medial – ABd H, FHB
Superficial – FDB, FDL tendons, Lumbricles
Lateral – FDM, ABd DM
FF Only:
Adductor – Adductor H.
4 Interosseous – 4 Dorsal and 3 Plantar Interossei
RF Only:
Calcaneal - Quadratus Plantae, PT vessels and n.
Communicates with Deep post. Comp in leg
160. Jones Tenosuspension? EHL transferred to neck of 1st MT + Hallux IP fusion. For cavus foot, PF 1st
Ray
161. Cole procedure? DF wedge osteotomy Nav Cun jts + cuboid
162. DuVries osteotomy? DF fusion through MTJ for cavus foot
163. Japas osteotomy? Displacement “V” osteotomy through the cuboid, nav, and medial cuneiform
164. Most common jt to go into non-union in triple fusion? TNJ
165. HIBBS tenosuspension procedure? EDL transferred into midfoot. Indicated for functional equinus,
prevents hammertoe formation
166. Surgical Procedures replacing ATF in lateral ankle instability?
Utilizing PB – Watson Jones, Mini Watson Jones, McLaughlin, Lee
Pouzet – PL, Weber – plantaris, Sefton – plantaris and long extensors, Haig – reinserts ATF
167. Surgical Procedures replacing ATF & CFL?
Elmslie, Christman-Snook, Winfield, Storen 2, Split PB lateral ankle stabilization
168. Dosing of Ciprofloxacin? What type of drug?
200-400 mg IV q12h
250-750 mg po q12h 2nd gen quinolone
169. A good drug to give a diabetic pt with foot infxn in ER?
Zosyn
170. Classification systems for OM? Cierny-Mader, Waldvogel
171. Tx for Necrotizing Fasciitis?
1.2 million U /day of Pen G
172. 7 Rules for Callus Distraction?
1. Maximum preservation of marrow and periosseous blood supply
2. Stable external skeletal fixation
3. A latency before commencing distraction
4. Distraction rate of 1mm/day
5. Distraction in small, frequent steps
6. A period of neutral fixation after distraction
7. Normal physiology use of the elongated limb
173. How many cortices of screw purchase on each side of the fracture are required when fixating the tibia
with a plate?
6 (osteoporosis requires an inc in #)
174. Clinical signs of ischemic wound according to UT SanAntonio?
Claudication, Rest pain, Absent pulses, Skin atrophy, absence of pedal hair, dependent rubor (or
pallor on elevation), non-invasive study criteria:
Transutaneous Oxygen < 40 mm Hg
ABI < 0.80
Absolute Toe Systolic Pressure < 45 mm Hg
175. Which group of Abx has Achilles tendon rupture as one of its possible side effects? Flouroquinolones
(cipro)
176. Physical signs following Achilles tendon rupture?
Palpable complete defect of substance of Achilles tendon
Enlargement of gap on passive DF
Loss of spontaneous PF position of foot
Loss of PF power
+ Thompson test
177. 4 Most common Anaerobic organisms to infect DM wounds?
1. Clostridium 2. Peptococcus 3. Peptostreptococcus 4. Bacteriodes Fragilis
178. Blounts Dz? Osteochondrosis of medial portion of proximal epiphyseal ossification center in tibia
causing bowing of leg or legs. Limping and lateral bowing of legs…
Infantile (under 6 yo) – caused by early walking and obesity
True AVN
Adolescent (8-15) – caused by trauma or infxn. No AVN
179. What is INVANZ and what is its dosing?
New PCN derivative
1g IV/IM qd
180. Pain meds for pts allergic to narcotics? STUD
Stadol 0.5-2mg IV or 1-4 mg IM q3-4h
Toradol 10mg po q4-6h
Ultram 50-100mg po q4-6h
Darvocet 50/325 2 tabs po q4h or 100/650 1 tab po q4h
181. Vassal’s Principle? Mitigated by surrounding ST. When multiple fxs involved, after reduction of
initial fx, other segments will relocate into a more favorable position. Accordingly, additional fractures
may be fixed with less rigid or invasive methods or they ma not require fixation. (volkmans fx under 30%)
182. Pre-Dislocation Syndrome? Used to describe the acute, subacute, or chronic inflammatory condition
of a lesser MPJ that is believed is a precursor to a lesser MPJ dislocation.
183. Who described pre-dislocation syndrome? Gerard Yu and Judge
184. What is the ideal position to fuse the AJ?
0-5 DF, 5-10 external rotation, 0-5 valgus, posterior displacement of talus under tibia  smoother
stance phase of gait
185. Ilizarov’s law of tension stress? States that living tissue, when subject to slow steady distraction, can
become metabolically activated in both the biosynthetic and proliferative pathways.
186. Latency period recommended for callus distraction?
4-5 days  2 weeks, depending on secondary factors
187. Time required for ex fix to stay in place once length has been achieved via callus distraction?
1:1 ratio (should equal the amt of time it took to achieve length) osseous consolidation should be
confirmed radiographically.
188. T or F: Initiation of distraction can be initiated earlier when osteotomy is performed in metaphyseal
bone vs diaphyseal bone?
T (metaphyseal  7-10 days) (diap  full 2 weeks)
189. T or F: If radiographically visible callus is not noted on radiographic evaluation a few weeks after
distraction has begun, the physician should d/c the distraction?
F Lengthening process should be continued until desired length is obtained. (once desired length
obtained, distraction ceases and immobilization of foot continues until mineralization is complete)
190. Name the Lateral Ankle Ligaments?
ExtraCapsular – CFL
IntraCapsular – ATF, PTF, Lateral TaloCalcaneal
191. Angle between CFL and ATF? 105 degrees
192. Function of CFL and its orientation in relation to Fibula?
Stabilizes talus during ADDuction. Allows for normal supination of STJ. Oriented 20-45 deg
from vertical bisection of fibula
193. Differentiating single, double, and triple lateral ankle ligament “tears” utilizing radiographic ant
drawer test?
Single  5-8mm anterior talar displacement
Double  10-15mm
Triple  >15mm
194. What is the Tension-Stress Effect?
The governing principle that permits the gradual distraction of osseous and ST to achieve
lengthening of the skeletal system
It is directly affected by rate or frequency of distraction, stability of device you use to assist in
distraction, position, and type of osteotomy.
195. Tx and Abx for Rat bites? Clense wounds, Fortaz (Ceftazidine), Oxacillin, Sodium, Flagyl
(metronidazole)
196. Dosing for Fortaz? 1g IM/IV or 2g IV q8-12h
197. Specific sequence of release to be performed in the 1st IM space during a lateral release for HAV?
ADDuctor H. tendon release, fibular ses. Release, tenotomy of lateral head of FHB, excision of
fibular sesamoid
198. Fluid Resuscitation for pts with 3rd degree burn wounds?
Quantity of fluid required is proportional to size (kg) and extent of burn as a measure of total body
surface area (TBSA)
4 ml per 1% TBSA
1st 24hrs  plain lactated ringers with dextrose
199. Tetanus Prophylaxis in GSW?
Previous immunization  0.5ml toxoid
No previous  250 units Ig + 0.5ml toxoid
200. Torg Classification of 5th MT fractures:
Type 1 – acute fx with absence of intramedullary sclerosis
Tx = NWB BK cast
Type 2 – Fx with delayed union & evidence of intramedullary sclerosis and widened fx margins
Tx = immobilization or medullary curettage plus autogenous inlay bone graft for athletes to
expediate healing
Type 3 – Fx with non-union and complete obliteration of medullary canal by sclerotic bone
Tx = bone graft
201. Treatment for Watson-Jones Fractures of Navicular?
I. tuberosity fx  immobilization 4 weeks
II. dorsal lip  immobilization 4w, 6w if CCJ injury or MTJ subluxation
III. body  closed reduction, ORIF
202. How long before Hawkin’s sign presents on XR? 6-8w
203. What is the name of the lateral hockey stick incision?
Palmer
204. Rowe Classification system?
I A – Tuberosity
B – Sustentaculum Tali
C – Anterior superior process fx
II A – Beak fx of posterior superior tuberosity
B – TendoAchilles avulsion fx of post-sup tuberosity
III Extra-Articular calcaneal body fx
IV Fx involving STJ without jt depression
V Comminuted fx with central or severe depression
205. Essex Lopresti Classification?
Type 1  Tongue type
2o fx line extends posteriorly from 1o line to posterior Calcaneal cortex producing tongue
type (Vertical force)
Type 2  Joint Depression
2o fx line extends posteriorly from 1o line and exits on superior aspect of the calcaneus
causing it to impact into body of the calcaneus
206. When are scout foot XR’s required according to Ottowa Ankle rules?
If there is pain in the Midfoot and:
1) instability to bear weight both immediately and in ED (4 steps)
2) Bone tenderness at navicular or base of 5th MT
207. What are 3 mechanisms under Lauge-Hansen that can lead to Wagstaffe or Tillaux-Chaput fx?
SER - I PAB – II PER – II
208. What are the 2 mechanisms under Lauge-Hansen that can lead to Volkmann’s Fx?
SER – III PER – IV
209. Classification system for ankle diastasis without Fx?
Edwards and Delee
210. What is the Pankovich classification system for?
Maisonneuve fx’s
211. Application of syndesmotic screws?
*2 screws to correct diastasis should engage a minimum of 3 cortices in non-lag fashion (3 point
technique)
*1 screw that engages 4 cortices can be used (4 point)
212. What type of Salter-Harris type injury is the most common acute physeal injury? Type 2
213. What is the Conti Classification used for? MRI – PTTD
214. Eckert & Davis Classification System? (peroneal sublux)
Grade I – superior peroneal retinaculum ruptured from fibrocartilaginous ridge. Most Common
Grade II – Fibrocartilaginous ridge goes with the retinaculum
Grade III – Small amt of bone goes with retinaculum
*Skiing is most common method of injury
215. Surgical Procedures for dislocating peroneals?
Duvries – shelf cut out of fibula and slid back like a peg
Kelly – fibula cut saggitally and rotated to deepen groove
Jones – part of Achilles split and attached to fib. thru tunnel
Zoellner & Clancy – deepening of the groove
216. What is the key diagnostic feature to distinguish RA from OA? Subchondral osteopenia noted in
RA
217. Surgical Procedures for RA?
Hoffman – MT heads 2-5 resected
Clayton – MT heads and PP bases 2-5 resected
Keller – PP of hallux base resection
Gocht – PP bases 2-5 resected without MT heads
218. When should NSAID’s be stopped prior to sx? At least 1w
219. Describe one regimen for corticosteroid supplementation for pre-op RA pts? Methylprednisolone
40 mg IM night before Sx
40 mg IV during Sx
40 mg IM night of Sx
20 mg IM am & pm POD #1
20 mg IM pm POD #2
Continue Pre-op routine
220. What is a normal ABI value? 1.0 – 0.9
221. What is an acceptable ABI for a wound to heal post-op?
>0.6
222. What is an adequate toe pressure to allow for healing?
> 30 mmHg
223. What is an adequate toe pressure to allow elective Sx?
Over 40-45 mmHg (norm = 60-80 mmHg)
224. Drug utilized to counteract Versed?
Flumazenil 0.2 mg IV over 15 sec, then 0.2 mg q/min prn up to 1mg total dose
225. Phases of Skin Graft Healing?
1) Plasmatic Stage (24-48h) – fibrin layer formed between graft and host which serves to anchor
and allow diffuson of nutrients to graft
2) Inosculation Stage (begins @ 48h) – revascularization of graft tissue. Graft will have a PINK
HUE
3) Reorganization (continues for months)
4) Reinnervation (can take 1-2 yrs)
* 3 and 4 start at the same time…
226. Diaz Classification?
Lateral Ankle Ligament injury
I – ATF II – CF III – ATF & CF IV – All 3
227. Degan Classification? For anterior process fx of calcaneus
I – non-displaced involving only anterior process
II – displaced extra-articular fx
III – Large displaced intra-articular fx into CCJ
228. 2 tests that should be ordered with calc fx in ER?
Lumbar spine and Pelvic XR
Urinary Analysis (with fx pelvis, may have severed urethra)
Medical emergency
229. Kuwada Classification? Achilles tendon tears on MRI
I – Partially torn tendon (50%)
II – Less than 3 cm tear in tendon  tx by sew end to end
III – 3-6 cm gapping  graft
IV – over 6 cm gapping  graft
230. 3 types of stitch for Achilles tear?
Kessel, Bonnel, Krakow
231. 4 types of Gastroc Recession?
Strayer, Vulpiun, Baker, McGlammry
232. Islan’s Dz? Osteochondritis of 5th MT base
233. Bushke Dz? Osteochondritis of cuneiforms
234. Goal of reduction in talar neck fx? < 2mm of displacement
235. Gold Standard for Tx of Hawkings Fx? ORIF w lag screws
236. 3 Elements influencing spontaneous (indirect) healing of bone?
1) Callus precursor – Granulation tissue forms first around the fragment and later between the
fragments
2) Resorption – Fracture gap widens due to surface resorption of fragment ends
3) Filling of gap with new bone – bone formation progresses through a series of different steps
from granulation tissue to cortical bone (indirect bone formation) Achieved through a process of internal
remodeling of haversian systems.
237. What is the only process resulting in solid union in primary bone healing?
The process of internal remodeling of the haversian system, uniting the fragment ends…
238. Causes of Hypertrophic and Atrophic Non-unions w Tx?
Cause Tx Options
Mechanical condition correct internal
Hypertrophic of strain stability w splint or
ORIF

Atrophic/ Impaired capacity for Apply bone graft


Non-reactive Biological Rxn + mechanical
Stability

239. Phases of Wound Healing?


Inflammatory Phase (1-7d)
Influx of platelets & leukocytes  coagulation
Release of cytokines and mediators
Proliferative Phase (5-20d) {Re-epithelialization}
Collagen fibers are produced  wound strength
Angiogenesis, Fibroplasia, Wound Contraction
Remodeling Phase (3w – 2y)
Deposition of matrix materials, collagen deposition/remodel
As long as scar is erythematous, remodeling is occuring
240. Which screws should be applied 1st in a pre-bent plate?
The 2 inner screws (closest to the fx line)
241. When applying a buttress plate, where do you begin screwing?
From middle outward
242. How many compartments in the foot?
9, according to Manoli and Weber
243. A devastating Degenerative dz of UMN’s and LMN’s?
Amyotrophic Lateral Sclerosis (ALS) (Lou Gehrig’s dz)
244. Ages and Frequency of Tarsal Coalitions?
#1 STJ(middle) 12 - 16 yo
#2 CN bar 8 – 12 yo
#3 TN 3 – 5 yo
245. Rigid Flatfoot Triad?
Peroneal spasm, Stiffness, Pain  all can indicate tarsal coal.
246. Best radiographic view for CN bar? Med Oblq
247. At what age would sx intervention for a persistently painful CN bar be performed? Before age 14
248. Pre-Axial vs Post-Axial Polydactyly and most common?
Pre – duplication of hallux or medial aspect of 2nd
Post – Duplication of lat aspect of 2nd or anything lateral…
Occurs in 80% of cases
* Sx should be postponed until over 1 yo
249. Difference between UMN and LMN lesions?
UMN LMN
Voluntary control lost lost
Tone spastic flaccid
Reflex arc present absent
Pathologic reflex present absent
Atrophy little or none significant
250. What percentage of cavus foot pathology is linked to neurological problems? 75%
251. Neurological dz leading to “onion bulb” formation of the peripheral nerves leading to distal weakness
and sensory loss?
Dejerine – Sottas dz (hypertrophic interstitial polyneuropathy)
Nerve enlargement is frequently palpable
Nerve biopsy reveals concentric proliferation of Schwann cells around area of demyelinization.
252. Effects of oral corticosteroids on WBC count?
Acute/Short term – Increases WBC count  rule out other causes  UTI, Pneumonia, Cdiff…
*Once taper down is complete from dose of steroid, WBC will stay increased, but taper
down as well for another 2 weeks. WHY!?!?!  NOBODY FRIGGIN KNOWS!!!
Chronic use – Decreases WBC count along with immune system…
253. Neurological dz with weakness of LE anterior and lateral compartments, later upper extremity
weakness primarily ulnar nerve distribution, beginning in late childhood/early adolescence and rapidly
disabling? FREIDRICH’S ATAXIA
254. Neurological dz compares to CMT with addition of resting “tremor of the hands” and clinical finding
of areflexia, intrinsic-pedal muscle atrophy, pes cavus, clumbsy gait, and poor equilibrium?
ROUSSY – LEVY SYNDROME
255. Neurological dz resulting in abnormal lipid metabolism wherein phytanic acid accumulates in serum
with associated findings of ichthyosis, night blindness, and preceeding febrile illness with peripheral
muscle paresis, areflexia, dropfoot, pes cavus, and claw toes? REFSUM’s DISEASE
256. The 5 different types of spinal dysraphism (myelodysplasia)?
Spina Bifida
Spina Bifida Occulta
Spina Bifida Cystica
Meningoradiculocele
Meningomyelocele
257. Neurological dz characterized by progressive disorder and cytoplasmic eosinophilic inclusions in
neurons of substantia nigra?
Parkinsons Dz  characterized by rigidity, resting tremor, bradykinesia, loss of postural reflexes,
and festinating gait
258. Dz affecting anterior horn cells (LMN) of spinal cord, resulting in LE flaccid paralysis (areflexia,
hypotonia, and weakness)? POLIOMYELITIS  central nervous system defect is non-progressive,
however leaps to contracture that changes over time…
259. Neurological dz characterized by inflammatory demyelinating plaques in white matter of CNS and
course relapsingi and remitting with ataxia, scanning dysarthria, and intention tremor? MULTIPLE
SCLEROSIS
260. Dz with Ach receptor antibodies destroying the receptor site on postsynaptic membranes with
hallmark of exercise induced fatigue? MYESTENIA GRAVIS
260. Radiographic angles of clubfoot?
Kite’s(TC)  0-15 (norm = 20-40)
Calc Inc A  ~17 (norm = 20-25)
Talar neck relative to body:
Adduction  80-90 (norm = 10-20)
Plantarflexion  45-65 (norm = 25-30)
261. Muscles innervated by medial plantar nerve? Lat. Plant N?
Medial Plantar Nerve Lateral Plantar Nerve
ABductor Hallucis ABd. DQ brevis
FDB Quadratus Plantae
1st Lumbricle 2nd - 4th Lumbricles
FHB ADDuctor Hallucis
Flexor DQ brevis
Plantar interossei
3rd & 4th Dorsal interossei
262. Coxa Valga?
Angle of femoral inclincation > 128 deg
Most commonly secondary to dysplasia of femoral head & usually bilateral
Occurs concomitantly with Genu varum
263. Coxa Vara?
Angle of femoral inclination < 120 deg
Can occur secondary to trauma or abnormal development of femoral head and neck
Common etiology of this condition is “slipped femoral epiphysis”
May occur with Genu Valgum (knock knees)
264. Normal angle of femoral antetorsion (declinatation)?
Birth  30-35
Adult  10-12
265. Femoral retrotorsion (external femoral torsion)?
Femoral declination angle < 10 deg
Knee jt deviated externally
266. Normal malleolar position and tibial torsion?
13-18  malleolar 18-23  tibial torsion
** tibial torsion is 5 degrees more than malleolar position
267. DOC for Clostridium? Pen G
268. DOC for E. Coli? Ampicillin + Gent
269. DOC for Klebsiella? Cefazolin
270. DOC for Salmonella, Shigella? Ampicillin
271. DOC for Staph. Aureus? Nafcillin
272. DOC for Staph. Epidermitis? Vanc
273. DOC for Strep (non-group D) Pen G
274. Carcinoma developing over chronic, non-healing wounds or ulcers? SCC
275. Most common skin cancer, with 4 types including__ and appearing as shiny nodule with surface
telangiectasia?
BCC
Superficial, Pigmented, Nodular, Morpheaform
276. DOC for animal bites? Augmentin
277. What will decrease specific gravity? Diabetes Insipidus
278. _____ Protein is specific for Multiple Myeloma?
Bence-Jones
279. WBC counts and difference between Leukopenia/-cytosis?
LeukoPENIA (<4000) LeukoCYTOSIS (>10,000)
Overwhelming infxn Bacterial infxn
Viral infxn Inflammatory process
Hypersplenism Tissue Necrosis
Bone Marrow Depression Physical Stress
280. WBC INCREASE in Viral Infxns?
MONOcytes & LYMPHOcytes
281. WBC INCREASE in Allergic Rxn?
Eosinophils & Basophils
282. Hct DECREASES in?
(% vol. of RBC’s in sample of anticoagulated blood)
Anemia & Blood Loss
283. PT INCREASES in?
Vit K Deficiency
Biliary Obstruction
Deficiency of Extrinsic or Common pathway factor
Liver Dz
Coumadin tx
284. PTT INCREASES in?
Hemophilia A & B
Von Willebrand’s Dz
DIC
Deficiency of Intrinsic or Common Pathway Factors
Liver Dz
Heparin tx
285. INCREASE in ALK PHOS?
Liver & Bone dz
Healing Fx / Bone growth
HyperPTHism
Obstructive Biliary Dz
286. Serum Creatinine INCREASES in?
Renal Dz & Nephritis
283. Lactate Dehydrogenase is a reliable test for ____ when viewed with other tests?
MI (elevated 12-24 hrs after MI)
Pulmonary Infarction
284. What will Increase specific gravity?
Dehydration, Fever, Vomitting, Diarrhea
 ALL VOL. DEPLETION
285. Lowman? What plane Corrects?
TNJ fusion, TAL, Transfer of TA under Navicular & sutures into Spring Ligament 
SAGITTAL PLANE
286. Cotton? Which Plane Corrects?
Opening dorsal wedge in 1st Cuneiform  Sagittal
287. Miller? What plane?
Nav-1st Cun-1st MT fusion
PT and Spring Lig advancement with osteoperiosteal flap
 Sagittal
288. Which plane does a Lapidus Correct? Sagittal
289. Orientation of 2 semi-elliptical incisions in 5th digit derotation arthroplasty?
Proximal-Lateral to Distal-Medial
290. Which bone scan is best for subacute and Chronic Bone infxn? GALLIUM
291. Why does cigarette smoking affect bone healing?
Nicotine  vasoconstriction  less blood gets there
 also inhibits osteoblasts
CO  binds hemoglobin molecule irreversibly and stronger than O2  less oxygen in blood
292. Why cant aminoglycosides work on anaerobes?
They require oxygen to cross cell memrane
293. Seeing Glass in the foot?
Fragments as small as 0.5 mm seen if not obscured by bone
No fragment larger than 2mm is completely obscured
294. Localization of FB? Place 3-4 different sized needles 90 deg from each other and take intraop XRs.
Hemostasis is key
295. Most frequently encountered FB in foot? Needle
296. Best for seeing wood? CT is 100x more sensitive than XR
297. Plantar Fibromatosis resection? 5mm on all sides at least
298. Normal Compartment pressure in foot? Less than 6 mmHg
299. % of Calc Fx that induce compartment syndrome? Up to 10
300. Pressure Dx for Compartment syndrome?
30 mmHg or 10-30 mmHg BELOW Diastolic BP
*HypOtensive pts can get compartment syndromes @ sub-normal pressures, so always check
diastolic!
301. Intra-Op tx to prevent superficial infxn after trauma?
5% Acetic Acit (vinegar) to kill Pseudomonas
302. Surgical Technique for compartment syndrome? 3 incisions
2 dorsal incisions over 2nd and 4th MT down to bone
Get med, lat, and interosseous compartments
1 medial incision (6cm long) center of incision should be 2cm distal to med mal.
Get med, adductor, superficial, and calcaneal
303. Post op care of Foot Compartment Syndrome?
Leave open 3-7d, delayed primary closure or split thickness skin graft(20% of time)
304. Watershed area of TA? 2-6cm proximal to insertion (72% of ruptures)
305. Dx of TA rupture? Palpable dell, Thompson/Simmons test, obliteration of Kagers triangle,
Toygar’s Angle less than 150o  the angle of the posterior skin crease of the TA).
Arnar’s sign  TA is angled Away from calc in Insertional zone, and is angled Toward the Talus
in the Supracalcaneal zone
306. Re-Rupture rates with and w/o Sx? Whats better?
W/Sx = 5.4% W/O Sx = 12.7% DO SURGERY
307. Post-Op Care? Early Mobilization  decreases adhesions and makes tendons stronger  More
mature collagen, orient collagen fibers parallel to tendons longitudinal axis, which inhibits adhesions
around tendon, increases tendon strength, vascularity, and number of collagen fibers. DO NOT CAST!
308. Crisman-Snook? PB is cut proximally and left attached at insertion. Then is threaded thru fibula
from anterior to posterior, down thru calcaneus from posterior to anterior and then re-attached back to itself
as it courses into the fibula anteriorly. Re-constructs ATFL and CFL.
309. Brachymetatarsia? Most common is 4th, associated with (Pseudo and PseudoPseudo HypoPTH, polio,
downs…), 25x more common in Females.
310. According to Ilizarov’s data, can distraction osteogenesis increase the blood supply to an entire limb?
Yes, by as much as 330%.
311. Why should cortiotomies be performed metaphyseally?
The bone is wide, and mainly cancellous, therefore new bone will have a wider cross-sectional
area. Also, the nutrient artery has already branched and is thus easier to preserve. Also, muscles with
origins and insertions at metaphyseal regions are already adapted for extremity growth under the influence
of epiphyseal growth zones.
312. Do an oblique or transverse cut for corticotomy? Oblique, because the surface area of growth will be
much more, therefore increasing the regenerative process.
313. Talo-Crural Angle? {8-15o}
*Used to determine normal Lateral malleolus alignment and length.
1 line is drawn along tibial plafond, the other is drawn connecting the distal aspects of the malleoli.
Alternate method: line 1 is drawn perpendicular to tibial plafond, and the other is the intramalleolar line.
{83 +/- 4o}
*Angle should be within 2-5 degrees of the opposite side by either
314. Talar Tilt on stress AP? 6-8 deg difference = rupture
315. Evaluate for Medial ankle sprain?
Medial clear space on Mortise should be less than 4mm
316. Test for Diastasis intra-op?
Cotton test, bone hook around fibula and pull  if lateral shift of talus over 3-4mm, then fix
317. Stress XR for chronic lat ankle instability?
5 deg difference in inversion values = pathological
If over 18 deg, then double ligamentous injury
Anterior drawer subluxation if greater than 4mm
318. % of calcaneal fx associated with spinal fractures?
10-20%, most commonly L1
Always get lumbar spine, hip, and knee views as well as UA for possible bladder trauma.
319. Name of the fragment of calc fx attached to TA? Tuber
320. Triple or Wedge procedure for severe Cavus?
If tibioplantar angle is Greater than 120o, then do Triple
Tibioplantar angle is between long axis of tibia and plantar surface of foot.
321. Burn Classification?
1st Degree: sunburn/down to papillary dermis, no bullae
2nd : Deeper dermis, Bullae, painful, red/moist skin
If over 15% of TBSA  admit
3rd: SubQ fat, destroy dermal structures, painless, chared
If over 2-5% of TBSA  admit
4th: Exposed bone/tendon
*any plantar 2nd/3rd deg burn  admit
*any burn on pt under 2 or over 60  admit
*any electrical burn  admit
322. TBSA (Total Body Surface Area) Rule of 9’s
Trunk = 18%, Leg, arm, head = 9% each
323. Tx for Burns:
1. Fluid Replacement: LR’s for 1st 24 hrs, calculate volume with Baxter’s formula:
4 x (%TBSA) x weight(kg) = mL
1st/2nd 4h period  give ¼ of total each
3rd/4th ¼ of total are given at 8 hour intervals after 2nd dose
*after 24h, give albumin + D5W @ ½ Original volume
2. stabilize fx, tetanus toxoid, Abx
3. get labs (electrolytes, Hct, BUN/Cr)
4. EKG if electrical burn
5. Sx – escharotomy, fascial release for Compartment synd
6. CFT checked q1-2h, dressing changes qd/bid, pain meds, elevation/splinting, PT for ROM
324. 3 most common bugs on burns?
Pseudomonas, STREP, Clostridium
325. Normal Angles for Flatfoot Evaluation?
Lateral: Pronation
TaloCalc: 38-42 Inc
Talar Dec: 21 Inc
Calc Incl: 18-21 DEC
Cyma line Ant displaced
DP:
TaloCalc: 17-22 Inc
TaloCub: 17-25 Inc
FF aDD: 12-15 DEC
Less Tarsal: 5 aBD Inc
Talo1st: 5-10 Inc
326. Order of Reduction in the treatment of Cavus?
1. ST release (Steindler strip)
2. RF fusion and osteotomies (dwyer, triple)
3. Midfoot fusions and osteotomies (Cole wedge = dorsal closing wedge, Japas V – cut midfoot in
half with chevron)
4. MT/Digital sx (DFWO, digit fusions)
5. Tendon transfers (jones, hibbs, heyman, STATT, TPTT)
 FF sx is NOT indicated until RF is fixed first!
327. Ruch Classification of Cavus?
I: MT, MPJ’s, Digital cavus
Tx = digit fusions, MPJ release, tenotomies, flexor transfers
II: Rigid PF 1st ray + RF varus
Tx = DFWO, Dwyer (closing pronatory), STATT (half of TA is relocated to lateral foot),
Peroneal Stop (suture PL to PB)
III: Severe Global RF and FF  N/M origins normally
Tx = osteotomies, triple, tendon transfers
328. Tendon Transfers for Cavus?
Jones – EHL sectioned and fused to 1st MTh c HIPJ fusion
Hibbs – section EDL, attach to 3rd Cun + digit fusions
Heyman – EDL relocated to MT bases + digit fusions
STATT – lat ½ of TA relocated to lat side of foot
TPTT – thru interosseous membrane to 3rd Cun (out of phase)
Peroneal Stop – PL sectioned & fused to PB
329. Structures cut in Steindler Stripping? Plantar fascia, 1st layer of plantar muscles, long plantar lig.
330. Types of Plating Technique?
Neutralization – protects an already reduced fx that has an interfrag screw already in place. It
protects the reduced and stabilized fracture from shear, flexure, and torsion. All screw holes are drilled
Concentrically.
Axial Compression Plate – must be “pre-bent” away from the bone cortex. The fx must be
relatively transverse as oblique fractures will shorten and displace. 1/3 tubular plate must be eccentrically
drilled (load screw principle). When the screw head “seats”, it will bring the near cortex closer to the fx
line, thus giving compression.
Dynamic Compression Plate – thick plate. Holes are designed in oval shape that slope to allow
creation of axial compression as the screws seat. The only holes that must be eccentrically drilled are the 2
closest to the fx. Once those are drilled and screws are seated, the rest can be concentrically drilled. Must
be well-molded to bone. 1/3 tubular plates work best.
Buttress Plating – used for unstable fx to maintain alignment of fragments despite lack of stability
within injured bone. Gap healing may occur. Essentially serves as a bridge between larger fragments with
intervening small fragments leaning against the plate. Devitalized bone should be replaced by cancellous
graft under protection of the buttress.
Tension Band Wire – Combines splintage of 2 smooth k-wires with stainless steel wire tension, to
effect dynamic interfragmental compression. Dynamic interfragmentary compression is created with an
eccentrically positioned tension wire used in conjunction with a load beam that converts the tension in the
wire to compression across the fracture, usually requiring joint positioning that effects wire tension. Good
spots for this are base of 5th, malleolar, and patellar fx’s.
Can be done with plate as well if placed on convex side of bending bone. Used in a relatively neutral
mechanical environment. This principle resists forces and uses them to enhance stability to the fixation
site. Necessary external load will keep the fracture site together. Device is applied to tension side of bone
to resist force on compressions side.
331. 1 unit of blood will raise the Hct how much?
3 percentage points
332. How much fluid does the avg full bladder contain?
400mL
333. Minimum water requirements for most pts?
2000 mL/day
334. Calculate Input of fluid per DAY?
1st 10 kg x 100 mL = 1000 mL +
2nd 10 kg x 50 mL = 500 mL +
Remaining kg x 20 mL = ____ = per day
• eg 70 kg man:
o 1000 + 500 + (50 x 20 = 1000) = 2500 mL/d
335. Calculate Input of fluid per HOUR?
4 2 1 Rule:
4cc x 1st 10 kg = 40 +
2cc x 2nd 10 kg = 20 +
1cc x remaining kg = ___ = per hr
eg 70 kg man:
40 + 20 + (1 x 50 = 50) = 110 cc/hr
Typical rate is 100 cc/h  consult cardio/nephro if dialysis or CHF pt
336. Why does Lidocaine work faster but last less time than Marcaine?
Lidocaine is closer to the body’s natural pH (7.4), therefore it acts more quickly, but it is also
metabolized faster for the same reason.
Marcaine is more basic, and thus takes longer to convert to a more natural pH so the body can
absorb it…
337. What Diabetic Drug is contraindicated when administering IV iodinated contrast to a pt?
Metformin (Glucophage) – may cause kidney damage if combined with contrast. Should hold
metformin 48 hours before and after, with after being more important to withhold. The reason so if the pt
goes into renal failure from contrast, the pts kidneys will not be further harmed by more metformin to deal
with. If metformin DOES build up in the kidneys, can cause fatal lactic acidosis.
338. Tests to perform in office if u suspect LisFranc injury?
FF Abduction stress views (under fluoroscopy if u got it) with ankle block. Look for Fleck sign.
339. How does pH affect K+ levels in blood?
Acidosis  K+ shifts out of cells  Inc [K+] & vice versa
Potassium Exolate gets rid of K+
340. Perioperative Diabetic Medication Management?
IDDM: if on Long acting insulin  take ½ dose in am
If no Short acting insulin  hold day of Sx
NIDDM: Well controlled  d/c orals am of Sx
If on Chlorpropamide or Metformin  d/c 24-48h pre
o Risk of lactic acidosis if patient develops renal
problem perioperatively
Not well controlled  insulin + glucose infusion
IV fluids intra-op: D5W/0.45 NS to prevent hypoglycemia
341. DM pt morning of Sx?
Check K+ levels  Insulin dec. serum K+
Diet controlled Diabetic < 200 mg/dl  no meds needed
Diet controlled Diabetic > 200 mg/dl  consider regular insulin
Type 2 Diabetic < 150 mg/dl  stop meds morning of surgery
Type 2 Diabetic 150-250 mg/dl  take AM dose of oral agent
Type 2 Diabetic >250 mg/dl  start regular insulin
342. Post-Op management of DM pt?
Check post-op BS levels
– 150 - 199 mg/dl  give 2 units regular insulin
– 200 – 249 mg/dl  4 units
– 250 – 299 mg/dl  6 units
– 300 – 349 mg/dl  8 units
– 350 – 399 mg/dl  10 units
– > 400 mg/dl  12 units
Regular oral and insulin therapy may be started again when the patient begins eating
343. HypoLipidemic Agent Peri-Op Management? 3 types: HMG-CoA reductase inhibitors (“Statins”),
Bile acid resins, Fibrates
* Discontinue 1 day before surgery and re-start the day after
surgery, UNLESS IT”S A STATIN.
Recent studies have shown that “statins” benefit patients during surgery by preventing vascular events
through other mechanisms (plaque stabilization, decreased inflammation, decreased thrombogenesis)
Continue Statins
344. Hypertensive Medication Peri-Op management?
β-Blockers (Lopressor, Corgard, Tenormin), α-2 agonists, Ca ++ antagonists (Procardia), and ACE
inhibitors (Capoten) can all be given the am of Sx, unless pt is taking ACE and is hypOtensive or taking the
med for heart failure.
*Sustained arterial HTN damages blood vessels in the kidneys, heart, and brain  increased incidence
of renal failure, coronary disease, cardiac failure and stroke.
345. Post-op HTN episodes caused by?
Pain, Reaction to endotrachial tube, Volume overload,
Emergence excitement
– Treated by eliminating cause and if necessary, giving sub-lingual Procardia, IV diuretics,
nitropatch, sublingual nitroglycerine, or nitroprusside.
346. Dosing Regimens of Steroids?
1) Alternate Daily Dosing: Less side effects, though less effective
2) Daily Low Dose: <15 mg prednisone daily
– Used for inflammatory arthritic processes
– Most common dosing schedule
3) Daily High Dose
– >15 mg prednisone daily
– Used for life threatening autoimmune diseases
4) Pulse Methylprednisone: Used as “last resort” therapy
– 1 gram IV over 45 minutes
347. Who Needs Perioperative Steroid Tx and Why?
* Any pt taking at least 5mg/day of prednisone (or equivalent) for at least 5 days within the past 2
weeks or at least 1 month of sustained cortocosteroid therapy within past year considered at risk for acute
post-op adrenal insufficiency
* Without dose, pt may not be able to respond to surgical stress, resulting in consequences such as
hypotension or death
* Plasma ACTH and cortisol, urinary free Cortisol, and urinary metabolites of cortisol
INCREASE in response to surgery, trauma, and other forms of critical illness in the otherwise healthy
pt.
348. Perioperative Steroid Tx Regimen?
NEW RECOMMENDATIONS Minor Sx Moderate Sx Stress Major Sx
Stress Stress
50-75 mg 100-150 mg
Daily Total 25 mg hydrocortisone hydrocortisone hydrocortisone
equivalent/day equivalent/day x 1-2 equivalent/day x 2-3 days
days
Normal PO prednisone
Pre-op dose or 10mg IV/20-25 25 mg IV/50 mg PO 50 mg IV hydrocortisone
mg PO hydrocortisone hydrocortisone
for non-dependent pts
Case-Dependent Case-Dependent
Intra-op None (Usually 25-50 mg ) (Usually 25-50 mg )
*If complicated, 25 mg *POD 1:12.5-25 mg *POD 1: 50 mg IV
Post-op post-op IV /25-50 mg PO hydrocortisone q 6-8 h for
*If uncomplicated, next hydrocortisone q 6-8 h 24-36 hours post-op
day return to normal *Taper to prior *Taper to prior regimen
dose. regimen over 1-2 days over 2-3 days
349. PeriOperative Anti-Rheumatic Drug Therapy?
Most metabolized through kidneys, so check those pre-op. If kidneys ok, no need to change. If
diminished, hold 1-2 weeks pre-op.
350. Peri-Op Warfarin/Coumadin? (UFH = unfractionated hepar.)
1) D/C oral Warfarin 4-6 days prior to surgery and monitor INR
2) If difficulty getting INR < 2.0, reverse oral anticoagulation with Vit.K (24-48 hours pre-op) or Fresh
Frozen Plasma (immediate reversal)
3) Bridge with UFH / LMWH until 6-12 hours pre-operatively
4) Resume UFH / LMWH after 12 hours post-op along with warfarin
5) D/C UFH / LMWH when warfarin reaches therapeutic levels
351. Anti-Platelet Medication Peri-Op management?
*Recommended to D/C aspirin, clipidogrel, or ticlopidine at least 5-7 days pre-operatively.
– Reinstitute them when bleeding risk has diminished (adequate clotting has occurred)
– Irreversibly inhibit platelet aggregation by inhibiting certain components of the clotting
cascade (COX, TXA2, ADP-binding sites)
– These effects last for the lifetime of the platelet (10 days) and takes at least 7 days for
platelet function to return to 70% of normal (3 days for most NSAIDS).
*Short acting NSAIDS (ex. Ibuprofen) should be stopped at least 1 day before surgery, while longer
acting agents (ex. Naproxen) should be D/C at least 2-3 days pre-op.
– Even though platelets lack COX-2, specific COX inhibitors such as Celebrex should still
be stopped 2-3 days pre-op because of its effects on renal function.
352. Nerve Injury Classification?
Neuropraxia: inject into n, recovery = 2-3 months
Axonotmesis: axon cut, wallerian degeneration, rec is 2-3m if it happens
Neurotmesis: Complete severance of n, irreversible
353. AO stands for?
Arbietgemeinshaft fur Osteosynthesisfragen. In 1958, 15 swiss doctors agreed on this.
354. Osseous Stabilization laws?
Wolf’s Law – bone produced under compression and resorbed under tension
Piezoelectric effect – Compression leads to electro – and tension leads to electro + charges. - =
bone formation
Pauwel’s principle – Eccentric loading of bone causes one side under tension and the other under
compression
Vassal’s principle – Reduction of major fracture components leads to reduction of smaller
components by ST.
Hook’s Law – Amt of strain on bone is directly related to stress applied to bone. This is why we
put pt in Sx shoe
355. Fx Stability Classification?
Intrinsic: Charnley’s Classes
I: transverse fx (most inherently stable)
II: LONG oblq, spiral, or comminuted Fx are Unstable
III: Potentially stable fx (SHORT oblq) when orientation is less than 45% from transverse axis of
bone
356. Rule for screw vs bone size?
Never use a screw greater than 40% of the diameter of bone
357. How much skin can you remove and still be able to close?
3:1 RULE  Length:Width
• If less than 3:1, incision will have too much tension and will not close!
358. Probe to bone test %’s
66% sensitive, 85% specific, 89% predictive value
359. ESR for Dx of OM?
ESR of 70 mm/h, 89.5% sensitive, 100% specific & positive predictive value.
360. Drago, Oloff, and Jacobs Classification of HL?
Grade I: Functional, no DJD, passive ROM ok, loaded will be diminished ROM
Grade II: Joint Adaptation, flattening of head, OCD, pain on EROM, passive ROM limited, dorsal
exostosis, eburnation, lipping
Grade III: Jt Deterioration, grade 2 but worse +, non-uniform narrowing of jt, CREPITUS,
subchondral cysts, pain on FROM
Grade IV: Ankylosis, Hallux Rigidus (<10 deg), deformity
361. Etiologies of HL?
Long 1st, MPE, Hypermobile 1st, Gout, short 1st, trauma, septic, charcot, iatrogenic (mis, shortened
MT, keep immobilized too long, avn)
362. 1st MPJ normal ROM and diff b/t func and structural HL?
Unloaded = 65, Loaded = 20
Functional = only loaded is diminished, structural = both
363. Sx for HL?
Cheilectomy
Kessel & Bonney – for long pp or PFed Hallux – closing base wedge with base dorsal from base
of pp of hallux
Waterman – CBWO c base dorsal in 1st MTh
Waterman/Hohmann – cut plantar cortex and realign head
Regnauld(Enclavement) – cut out base of pp, reshape it, shorten pp, and replace base.
Decompresses jt
Youngswick
Lambrinudi – CBWO c base plantar in 1st MTb (tx MPE)
Keller
Velenti – Mayo/Keller
Fusion – 10-15o ABD, 15-25o DF, parallel to 2nd digit
Implants – total, hemi, swanson
364. Stages of HAV?
1) Lat displacement of pp relative 1st MTh, sagittal groove/sulcus forms
2) Hallux ABductus, PASA inc, HDS of 2nd, EHL/FHL are bowing out into interspace, Abductor
H slips plantarly
 tibial sesamoid position #4  no cristae
3) INC IM angle, cuneiform split
4: Subluxed or Dislocated hallux, RA, 2nd toe lost buttressing power.
365. Reverdin?
Green – shelf, corrects PASA
Laird – shelf, IM and PASA  moves head
Todd – shelf, IM, PASA, MPE  PF’s 1st MTh
366. Shaft osteotomies? IMA of 15-17
Scarf, Vogler
Mau – proximal plantar to distal dorsal strait line
Ludloff – opposite of Mau (would make an X thru bone if both were done)
367. Base Osteotomies? IMA of 18+
Loison Balacescu – CBWO c base lateral, intact med cortex
Juvara – CBWO c base lateral, but oblique (A,B1,B2,C1,C2)
368. Bunionectomy Screw?
4.0 cancellous c 5.0mm head, partially threaded and total length, 16mm-52mm
Normal 4.0 cancellous c 6.0mm head, 10mm-100mm
369. TRUE IM angle?
= IM(measured) + (MAA – 15)
370. Synovial Sarcoma?
Most common primary malignant tumor arising in foot
20-40 yo, grows slowly with metastases
Tx = wide resection or amputation
Survival rate = 5 years
371. Types of Autografts?
Split Thickness – epi and part of dermis – most common
Thin: less than 15 thousandths of an inch thick
Revascularize quicker, contracts more
Less elastic, mesh to allow drainage
Thick: 0.016-0.020” thick – hard to vascularize
Full thickness – epi and dermis, but no subQ
Contracts less, harder to revascularize
Better for WB or flexure points
Skin Flap – transplanted neurovascular structures
Composite Graft – skin and other tissue (bone, tendon, cart)
372. Skin Graft Healing Stages?
I: Plasmatic Stage – (24-48h)
Anchors graft with fibrin, granulation then replaces fibr
II: Inosculation/Revascularization – (48-72h)
Vasc proliferation, Anastomoses from bed, not edges
PINKING UP
Lymph drainage by day 4
Circulation by day 7
III: Reorganization/Epidermal Proliferation – (1w-months)
ST reorganizes, regulation of vascular in/outflow
IV: Reinnervation – (2-4w) can be up to 2y +
373. AMPLE history?
Allergies
Medications
Past Illnesses/Injuries
Last Food/Drink  6-8 h window
Events preceding injury  self medication
Effect of Prehospital tx
374. WBC w diff? INC DEC
Neutrophils (PMN) Acute bacterial draining wound
Bands(Myelocytes) Acute infxn
Eosinophils Allergic rxn
Lymphs Viral or syphilis HIV, Lupus, leukemia
Monocytes Chronic bacterial
SHIFT TO THE LEFT:
Bands (myelocytes) OVER 20%
PMN’s (neutrophil) OVER 70%
*indicates infxn, toxemia, hemorrhage
375. Anemias?
Microcytic: DEC MCV & MCH
Fe deficiency or lead intoxication
Macrocytic: (Megaloblastic) INC MCV & MCH
Vit B12 def(pernicious)
Secondary to malfunction of intrinsic factor or
following gastrectomy
Folic acid def (EtOH, malabsorb, pregnancy)
Drugs – sulfa, methotrexate
Normocytic: Acute hemorrhage, hemolytic, chronic dz
376. BMP Profile?
K+: INC  acute renal failure, if over 7  cancel Sx!
DEC  weakness, cramps, cause = diarrhea
Ca++: Factor 4 in coagulation
INC  bone demineralization – multiple myeloma
DEC  low serum protein
Phos: essential for metabolism of carbs and lipids
INC  renal failure, healing fractures, pagets, mm
DEC  hyperPTH, DM
BUN: end product of protein metabolism produced in liver
INC  kidney dz, DM, chronic gout, CHF
DEC  Liver failure
Creatinine: INC  kidney dz
Uric Acid: 10mg/dL
Total Protein (at least 7): INC  dehydration
Albumin (3.6-5.1gm/100mL)
INC  dehydration
DEC  Liver dz (chronic), malnutrition
Total Bilirubin: byproduct of Hb from destroyed RBC’s.
Indirect INC  liver failure, internal hemorrhage
Direct INC  billiard duct obstruction
Total INC  cirrhosis, acute viral hepatitis, CHF
ALK PHOS: produced by osteoblasts
INC  bone ca, child bone, fx, pagets,
Lactate Dehydrogenase: functions in carb metabolism
INC  kidney damage, acute MI (isoenzyme 1 is elevated 10-14 days post-infarction
SGOT: enzyme acts as catalyst in amino acid metabolism through glycolysis
INC  LIVER dz, MI
377. UA?
Acetone: not normally found in urine  ketoacidosis
Blood: kidney injury
RBC = Glomerulonephritis
WBC = Pyelonephritis
Glucose: DM, excess IVF glucose, chronic liver dz
Acidity:
Acidic  acetone (ketoacidosis), DM
Alkaline  blood transfusion, vomiting
Proteins:
Albuminuria  UTI, hemorrhage
Bence Jones  Multiple Myeloma
Specific Gravity:
INC  DM, dehydration, sweating, vomit
DEC  large fluid intake, dec salt intake
378. Pre-Op Orders?
Admitting Labs: CBC, BMP, PT/PTT, FBS, UA, HcG if female, RPR, Sickle dex
Diet: NPO after midnight
MD or cardiology clearance
CXR for all smokers and pt over 40
EKG: all pt c hx of cad and over 40
XR: foot 3 views, MRI, CT, Bone scan…
Sedative: versed
Anesthesia to see pt
Betadine scrub of foot x 10 min
379. Post-Op Orders?
Admit pt to floor
Dx
Conditon: stable, unstable guarded, critical
Vitals: q 15min until stable then q routine
Activity: ambulatory as tolerated, PWB with sx shoe, NWB c crutches, CBR, BRP
Nursing: elevate foot, ice, check NVS, incentive spirometry to pt with instructions (if general
anesth)
Diet: ice chips to liquids to light snacks previous diet as tolerated
Ins/Outs: D/C IVF when po intake tolerated
Labs: CBC, H&H
Meds: analgesics, symptomatic meds, Abx
Ancillary: reece shoe to pt, PT to crutch train
Xrays: post op
380. Elective major procedure DM bunion periop management?
NPO after midnight
Insulin controlled  FBS pre-op, start D5W IVF, hold regular,can give NPH
Oral controlled  Same + hold oral med, if glucose>300  NPH SQ
Diet controlled  Same + IVF dependant on pts needs
Anesthesia will monitor BS if u ask
Post-Op  post-op glucose stat, may give 2nd ½ of NPH depending on glucose level, continue IVF
until pt eats
381. Hx of gout, peri-op prophylaxis?
If uric acid is >7, r/o kidney damage (BUN/Cr)
Colchicine 0.5mg IV ½ hr pre-op
0.6mg po qd x 3d
Dilute in sterile water, NOT D5W  it will precipitate
382. 5 signs of infxn?
Rubor, Tumor, Dolor, Calor, Functio Laesa
Bacteria  capillaries dilate  inc fluid in extravascular space  swelling  compression of
nerves
383. Bone Scans?
Tech 99: Sensitive, not specific, t½=6h, kidney metabol.
Phase 1 – angiogram
Phase 2 – blood pool
Phase 3 – (3-4h) regional bone metabolism
Phase 4 - (24h) Osteo BEST seen in late phase 4
Gallium 67: detects acute inflamm or infxn, t½=78h
Binds leukocytes, bacteria, plasma proteins
+ trauma, inflammation, active infxn
*if gallium shows more than tech  no osteo
Indium 111: take blood from pt and label leukocytes
Scan 19-24h later
Hi sensitive, mod specific for acute ST infxn + OM
- in noninfectious, inactive, chronic OM
+ in active OM
HMPAO: Ceretic labeled leukocytes c tech 99
90% sensitive, 86% specific
Best WBC scan for OM
Acute OM  all + (gallium focal)
Chronic OM  + tech, - indium, - gallium
Acute cellulitis  + tech, + gall (diffuse), + indium
Infected implant  + tech, + gall
Charcot  + tech, - gall, - indium
384. What are the 2 types of etiologies of metatarsalgia?
Structural (static): Long, short MT, PFed MT, Enlarged lat condyle of MTh, Trauma, Iatrogenic,
Accessory sesamoid
Tx = condylectomy, arthroplasty, implant
Functional (dynamic): Pronation/hypermobility, Equinus, LLD, Compensation for Cavus,
Hammertoe, High heeled shoes, Extensor/Flexor Substitution
Tx = digital or MPJ sx, orthoses, heel lift, shoe mods
385. What are some D/Dx for metatarsalgia?
Neuroma, ST mass, Friedberg, Fx (stress), verrucae, Tendonitis, gout, arthridities, plantar plate
rupture/capsulitis, Pre-dislocation syndrome (do modified lochman test – dorsal drawer of mpj)
386. What are the following numbers?
Success rates for distal and proximal osteotomies for MTalgia? 61% & 46% respectively
Rate of transfer calluses? 40%
What is the proper amt of elevation that you should try to achieve with an elevating MT
osteotomy? 3.5mm – 5mm
Less than 3.5  reoccurance, over 4-5  transfer
387. Name and explain some distal procedures for MTalgia?
Condylectomy, arthroplasty, implant
Jacoby V  v made dorsal to plantar with apex distal
Vertical Tilt up  dorsal CBWO c apex prox/plantar
388. Why is it not a good idea to do shaft procedures for MTalgia?
The nutrient artery to the lesser MT’s is approx 3cm proximal to the head… right where you cut!
** one procedure is the Giannestra’s step down Z – sagittal z – can elongate, PF, DF, Shorten…
versatile, but hi AVN rate
389. Proximal procedures for MTalgia?
DFWO vertical or oblique, proximal V
390. Post-Op care for these procedures?
Head and shaft procedures  WB or NWB + Sx shoe x 3-6w (unless grafting)
Base procedures  B/K NWB cast x 4-6w
391. How can you tell if a Tailor’s Bunion was induced by a PF or a DF 5th MT without touching the foot?
Dorsal MTh hypertrophy  DF 5th, c or s path. Pronation
Lateral MTh hypertrophy  PF 5th, subluxes c pronation
392. Explain what the landmarks are for axes of 5th MT IM & LDA measurement, and what are the
norms/pathological according to Fallat and Buckholtz?
IM: 6.47 / 8.71 – angle between bisection of 4th MT shaft and a line drawn along the Proximal ½
of the Medial surface of the 5th MT shaft.
LDA: 2.64 / 8.05 – angle between the second line for the IMa and a line bisecting the head, neck,
and articular surface of the 5th MT.
393. What is a pathological value for plantar declination angle for the 5th MT? over 10 degrees may be
pathological (FAI 1990)
394. What is the 5th MTh Prominence value? What is the avg? Who came up with it?
The distance between two parallel lines drawn along the lateral cortex of the 5th MT and the lateral
most projection of the 5th MTh. The avg value is 4mm. Steinke and Boll
395. What is the classification system for tailors bunions?
Fallat and Buckholtz  4 types
i) enlarged lat. 5th MTh due to enlarged condyle, round head, true hypertrophy, exostosis
tx = lat condylectomy
ii) lateral bowing of 5th MT (IM = norm, no hypertrophy)
tx = distal osteotomy
iii) Increased IMa (LDA = norm)
tx = Z osteotomy
iv) Combo c at least one structural component (ii & iii) required
tx = lat condylectomy & osteotomy for most proximal deformity
396. What is the rule for MT osteotomies as related to width?
Limit transposition to 1/3rd of the MT width @ osteotomy
397. What are some Distal procedures for tailors bunions?
Duvries lat. condylectomy – destabilize jt + recurrent (good for DM ulcers)
Chevron – leave k-wire in for 3w
Hohmann slide – unstable
Wilson slide – oblq osteotomy from dist/lat to prox/med\ Yu CBWO – base medial  good
to dec. LDA
398. What are some Shaft procedures for Tailors bunions?
Yancey CBWO – intact lat hinge – distal, middle, or proximal placement (a,b,and c)
Z osteotomy – transverse – very stable
399. What are some Base procedures for tailors bunion?
Gerbert CBWO – base medially
Estersohn OBWO – base laterally
*reserved for extremely hi IMA
400. What are some complications for each procedure?
Distal: Over medial translation/dislocation, hinge fx, MPJ dislocation, unstable 5th toe
Shaft: Troughing, hinge fx, dislocation
Base: non-union, hinge fx, elevatus c transfer callus
401. What forms a Heloma Molle?
The head of the proximal phalanx of the 5th rubbing with the Lateral condyle of the proximal
phalanx of the 4th toe.
402. What are the zones of toenail injuries? (nail bed injuries with ST loss)
Zone 1 (distal aspect of nail)
Cleanse & debride wound  try to cover, may let granulate, if over 1cm  skin graft
may be used
Zone 2 (middle of nail)
Exposure of distal phalanx, reduce predominant bone, possible V-Y advancement flap or
partial distal symes
Zone 3 (proximal aspect of nail)
Minimal survival of nail bed, debridement in OR, exise nail plate and groove to prevent
nail spicule from regrowing, many go to distal syme’s amp
403. Good fluid management for HTN pt?
Don’t use fluids with NaCl, NS, 0.5NS, LR
 use D5W @ 5cc/hr
404. Tx for hypoglycemic?
50cc D50W + 1mg glucagon (if unconscious)
10-12g carbs (if conscious)
405. What is CRP? (C-Reactive Protein)
A plasma protein, acute phase reactant produced by liver and adipocytes. Good to get a base
reading to track effectiveness of treatment and disease progress. Viral infxn gives a lower value than
bacterial. Low risk <1mg/L, High risk > 3mg/L.
406. What approaches does Myerson propose for fasciotomy of compartment syndrome in the foot?
Dorsal: 2 longitudinal incisions over 2nd and 4th MT’s. The interosseous and central compartments
are accessed through each webs space and the medial/lateral compartments using curved clamps.
Medial: 1 incision between the aBductor Hallucis muscle and the inferior base of the 1st MT. The
central compartment is decompressed with a curved clamp.
407. How common is pain on passive DF of the toes when trying to diagnose compartment syndrome of
the foot?
In Myerson’s article, it was the most reliable finding next to pain out of proportion (86%). Pain
was difficult to distinguish from pain from injury.
Other findings in this study: DP pulse absent in 12/14 pts, but dopplerable in 13/14. Light touch
loss = 54%, Numbness = 46%, 2 point discrimination = 64%, pin prick = 15%.
408. Lets Talk Bunion Biomechanics?
HI angle: (0-10)  Distal Akin
DASA: (0-8)  Proximal Akin
PASA: (0-8)  Reverdin if trackbound(wedge from 1st MTh)
Green – lat cortex intact c shelf (PASA only)
Laird – cut lat cortex and slide c shelf (PASA & IM)
Todd – Laird + PF head as well (PASA, IM, MPE)
HA angle: (10-15)  ST problem if PASA not involved
HA angle alone  McBride or Silver
Both  capsule tendon balance + head procedure)
IM agle: (8-12 in rectus) (8-10 in adducted foot)
13-14  head (Austin, Reverdin Laird/Todd)
15-17  shaft (Scarf)
18+  Base (Lapidus, CBWO, OBWO, Juvara)
*exception  can Kalish up to 17 deg
MT ADDuctus angle: (<15)
True IM angle = IM measured + (MAA – 15){amt of MT adductus over norm) *as MT adductus
angle inc  more proximal
MT protrusion angle: (+/-2mm)  + = longer 1st MT
Congruous  lines Parallel
Deviated  lines intersect Outside jt
Subluxed  lines intersect Inside jt
Sesamoid Position: (1-3)  by 4, cristae is eroded
Types of 1st MPJ Deformities:
Bne PASA,DASA, PASA Cong
ST or both HA angle +DASA or not
Positional ST angles normal NOT norm <HAa NOT
Structural Bne NOT norm Norm =HAa Cong
Combined both NOT norm NOT norm <HAa NOT

409. Most common AVN site in Navicular?


Middle 1/3
410. What NSAID is contraindicated in sulfa allergic pts?
Celebrex
411. The age old question… What are the effects of NSAID’s on Bone healing? Short answer  nobody
knows for sure
Long answer  COX activity is essential for normal prostaglandin production and is believed to
be the rate limiting enzyme in PG synthesis. All inflammatory responses are primarily mediated by PG’s
(especially PGE2). PGE2 has been shown to increase blood flow to bone, callus formation, mineralization,
trabeculae, osteoid formation, and accelerated remodeling. PGE2 and its receptors appear to be the main
contributors to bone metabolism. Bone healing begins with the onset of inflammatory PG formation
mediated by COX-2. PG’s are also synthesized by osteoblasts and are known to stimulate both bone
formation and resorption. COX-2 is important in early or activated stages of healing, but likely exhibits
little effect on the long-term healing process. COX-1 is mainly expressed in normal bone, while COX-2
expression is up-regulated during initial stages of bone repair process or under conditions of inflammation
or neoplastic bone conditions. COX-2 inhibition is thought to affect bone healing through these pathways.
In the animal models, both non and selective NSAID’s have been found to decrease bone healing.
Indomethacin, rofecoxib, and COX-2 deficient were the three main drugs/gene therapies tested while
rabbits and rats were the animals. Some studies blamed it all on COX-2’s, while others related the change
to COX-1. Still others showed NO significant differences in fracture healing between non, selective, and
control groups. On top of that, the COX-2 inhibitory concentrations in these animals are not known. As
far as humans go, more of the same is true. Many studies were done on spinal fusion pts with toradol and
ibuprofen showing both increased rates of non-union (up to 5x) and normal rates in another study. A study
on fx healing in post-menopausal women with piroxicam tx did not decrease the rate of fx healing and
actually reduced the osteopenia caused by ex-fix. Still other studies (2) on AJ and STJ fusion rates showed
an insignificant INCREASE in union rate when treated with COX-2 inhibitors. These studies showed that
their use in the short term does not negatively affect bone healing. Other studies (Brown et al.) showed that
initial delays in bone healing recovered with time and no significant difference existed between control
groups and tx groups at long term follow-up (12w). Other studies have shown the same. One explanation
offered by Hill et al. for the absence of increased nonunion rates with COX-2 inhibition may be that a
fusion model may not depend on inflammatory phase of bone healing or the pathway affected by COX-2
enzyme. Genetically, COX-2 deficient mice develop normal skeletons but have difficulty with fx repair in
secondary bone healing models, suggesting that COX-2 is really only necessary in posttraumatic states.
COX-2 may not be necessary for all types of bone healing. The general consensus of the literature is this:
Both animal and human data suggest that there is no negative effect on bone healing when used in a short
term manner (d/c after 6-12w post op). Therefore, pts can benefit from the decreased risk of narcotic use
and have no increased risk of a nonunion following arthrodesis procedure. As far as fractures go, results
do suggest that non-selective and selective inhibitors may have some effect on bone fx healing that is
apparent at earlier stages of healing process, but it bears no significant impact on over-all long-term healing
of the fx.
Good read’s 
“Review: Effects of cyclooxygenase inhibition on bone, tendon, and ligament healing” Radi et al.
Iflamm. Res. 54 (2005) 358-366
“The role of cyclooxygenase-2 inhibition in foot and ankle arthrodesis” Hill et al. Foot and Ankle
Clin N Am. 10 (2005) 729-242
412. How much Blood could a lap sponge sponge if a lap sponge could sponge Blood? (sorry… im getting
bored)
A standard dry 18-in. x 18-in. laparotomy sponge containing 25 mL, 50 mL, 75 mL, and 100 mL
of blood will appear 50% saturated, 75% saturated, 100% saturated, and 100% saturated with excess blood
dripping, respectively. A dry 12-ply 4-in. x 4-in. surgical sponge containing 5 mL of blood will appear
completely saturated. A dry 3-in. x 11-in. extra-large perineum pad containing 25 mL of blood will appear
50% saturated, and a completely saturated perineum pad will hold approximately 80 mL of blood.
413. Describe the Trauma/Retention stitch.
Far, then across to Near, then back to Near, then across again to Far, then tie over the entire length
like a big simple.
414. What is the Glasgow Coma Scale?
A neurological scale which aims to give a reliable, objective way of recording the conscious state
of a person

6 5 4 3 2 1

Opens eyes in Opens eyes in


Opens eyes Does not
Eyes N/A N/A response to response to painful
spontaneously open eyes
voice stimuli

Oriented, Utters
Confused, Incomprehensible Makes no
Verbal N/A converses inappropriate
disoriented sounds sounds
normally words

Localizes Flexion / Abnormal


Obeys Extension to painful Makes no
Motor painful Withdrawal to flexion to
Commands stimuli movements
stimuli painful stimuli painful stimuli

Highest Score = 15 (fully awake)


Lowest = 3 (deep coma/death)
Generally, Comas are Classified as:
Severe, with GCS ≤ 8
Moderate, with GCS = 9-12
Mild, with GCS ≥ 13
415. At what point do you fixate a posterior malleolar fx?
When it involves 25-30% of the surface. If any less is involved, Vassal’s Principal will take over
when you correct the lateral malleolus.
416. How many times your normal body weight is placed on your Ankle jt when walking?
5x
417. How much does 1 unit of blood raise the H & H?
1&3
418. What is a condition that a pt can get from receiving too much normal saline?
Hyperchloremic Acidosis
419. Lets talk about Disulfiram-like effects?
Disulfiram (Antabuse) is a drug used to support the treatment of chronic alcoholism by producing
an acute sensitivity to alcohol. Under normal metabolism, alcohol is broken down in the liver by the
enzyme alcohol dehydrogenase to acetaldehyde, which is then converted by the enzyme acetaldehyde
dehydrogenase to the harmless acetic acid. Disulfiram blocks this reaction at the intermediate stage by
blocking the enzyme acetaldehyde dehydrogenase. After alcohol intake under the influence of disulfiram,
the concentration of acetaldehyde in the blood may be 5 to 10 times higher than that found during
metabolism of the same amount of alcohol alone. As acetaldehyde is one of the major causes of the
symptoms of a "hangover" this produces immediate and severe negative reaction to alcohol intake. Some 5-
10 minutes after alcohol intake, the patient may experience the effects of a severe hangover for a period of
30 minutes up to several hours.
Symptoms include flushing of the skin, accelerated heart rate, hypotension (unless its Flagyl, then
HTN), SOB/N/V, abdominal spasms, headache, confusion, flushing, decreased O2 sats, sweating.
Drugs Associated with Disulfiram-like RXN to EtOH: Metronidazole (well documented, but
some question it now), Griseofulvin, Ketoconazole, chloramphenicol, glyburide, glipizide, several third-
generation cephalosporins (including cefotetan and cefoperazone), chlorpropamide, Sulfonamides
(Furosemide).
Remember  many OTC syrups contain over 5% EtOH
Treatment: Obvious  stop EtOH & offending agent. Give anti-emesis agents to stop vomiting.
2010
2010 - 2011
2011
EXTERNSHIP MANUAL

PODIATRIC SURGICAL RESIDENCY


PROGRAM
SAINT FRANCIS HOSPITAL & MEDICAL
CENTER

PROGRESS NOTES

You will write progress notes on both clinic and in-house patients on a daily basis. All
notes must be co-signed by a resident. All notes will be in the S.O.A.P. format. This is
also the way in which you will present patients to attendings and residents when and if the
situation arises. While visiting, we expect the student to be preparing to become a
resident which includes working up and presenting cases including differentials, offering
treatment options and requesting ancillary testing when appropriate.

POST-OP NOTES
The format is easily remembered by the mnemonic SAPPPPAHEMIC:
S: Surgeon
A: Assistants
P: Pre-operative diagnosis
P: Post-operative diagnosis
P: Procedure(s)
P: Pathology specimens sent (bone, C&S, aerobic, anaerobic cx)
A: Anesthesia (general, local w/IV sedation)
H: Hemostasis (tourniquet - location and duration)
E: Estimated blood loss
M: Materials (sutures, wires, pins, implants, drains, etc)
1
I: Injectables (local anesthesia, steroids)
C: Complications (none)
Then, briefly describe the patient’s condition post-op: “The pt. tolerated the procedure(s)
and anesthesia well and was transported from the O.R. to the R.R. (recovery room) with
VSS (vital signs stable) and VSI (vasc. status intact) to all digits of the______ foot.”

ADMISSION/POST-OP ORDERS

Admissions orders are done primarily through the CARE computer system and will be
done by the resident.

As an aid in writing orders, remember the mnemonic ADC VAANDILMAX:


A: Admit to the service of Dr. ________/Podiatry
D: Diagnosis
C: Condition
V: Vital signs (per routine, q4h)
A: Allergies
A: Activity level (CBR, commode to bedside, NWB L foot, BRP)
N: Nursing orders (elevate leg on pillows, dsg changes, foot cradle)
D: Diet (ADA, regular, low Na)
I: IV’s/Ins and Outs (Hep lock)
L: Labs (includes CBC, lytes, cultures, U/A, etc)
M: Medications (regular meds, antibiotics, prn meds)
A: Ancillary consults (Medicine, I.D., Vascular, P.T., etc)
X: Radiological studies (Plain X-rays, nuclear studies, CXR, EKG, PVRs)

REMEMBER TO DATE AND TIME ALL ORDERS!

PRE-OP NOTE

This is a checklist to ensure and document all the necessary preparations have been made.
Surgeon:
Pre-op diagnosis: _____________________
Procedure: _______________________
Consent: (give status--signed & on chart or pending)
CXR: (give results)
EKG: (give results)
Labs: (give all values: chem 7, cbc, PT, PTT, INR)
Abx
Pneumatic stockings
Sub Q heparin
Consult reports: (if necessary)
“Pt. clear for O.R. in AM/this afternoon/etc.”
“All risks, benefits, and complications have been explained to the patient, and
no guarantees have been made.”

ADMISSION NOTE

2
The admission note is the most involved of all the charting requirements. It closely
resembles the detailed H & P’s you perform on new clinic patients at school. This note
must be written within 24 hours of admission, as it provides a guide to the treatment plan.
You will help the resident perform H & P’s, and you will have the opportunity to write
admit notes.

The components of an admit note are:


Chief Complaint Family and Social History
History of Present Illness Review of Systems
Past Medical History Physical Exam
Past Surgical History (including detailed lower extremity exam)
Medications with dosages Assessment and Plan
Allergies

OPERATIVE NOTE

Following each procedure, the surgeon must dictate a detailed narrative of the
proceedings of the surgery. The standard format is:
Patient’s name and Medical Record Number
Date of Surgery, Surgeon, Assistant(s)
Pre-op Diagnosis, Post-op Diagnosis, Procedure(s)
Anesthesia, Hemostasis
Detailed description of procedures (includes induction of sedation or general anesthesia,
local anesthetic techniques, preparation and draping, and the procedure)

LABWORK SKELETONS

Na / Cl / BUN / Glucose WBC / Hgb/ Platelets


K / CO2 / Cr / / Hct/

Any other lab results or values are merely listed. Do not write that a lab is “WNL” and
do not leave empty skeletons on a note.

FINAL WORD ON HOSPITAL CHARTING

The last few pages have been more than enough to swallow at one sitting. We do not
expect you to be proficient at charting prior to your arrival - that is one of the things you
are coming here to learn.

POST-OP NOTE

S: Surgeon
A: Assistants
P: Pre-operative diagnosis

3
P: Post-operative diagnosis
P: Procedure(s)
P: Pathology specimens sent
A: Anesthesia
H: Hemostasis
E: Estimated Blood Loss
M: Materials
I: Injectables
C: Complications
The patient tolerated the procedure(s)
and anesthesia well and was transported
from the O.R. to the R.R. with VSS and
VSI to all digits of the ___ foot.

ADMISSION ORDERS

A: Admit to service of Dr. __/Podiatry


D: Diagnosis
C: Condition
V: Vital signs
A: Allergies
A: Activity level
N: Nursing orders
D: Diet
I: IV’s/Ins and Outs
L: Labs
M: Medications
A: Ancillary consults
X: Radiological studies
The following pages contain information that will be helpful in making your rotation, not
only with St. Francis but all your rotations, go more smoothly. It, by no means, is meant
to supplant the information you can receive by reading texts or journal articles. It is
merely meant to provide you with a good background for what you will be dealing with
most frequently while on rotations and working with patients. Having a good grasp of this
information will make your transition from student to resident a much easier one.

Reading an X-ray

Unfortunately, many students find this to be one of the most difficult concepts to grasp
while in school and on rotations. However, like most educational opportunities in life, if
you can learn to organize your thoughts while staring at a radiograph, reading it becomes
much easier. There are 5 basic steps to reading an x-ray and this includes the way in
which it should be presented.

1. Overall quality- type of view (DP, Lat, Oblique), which foot, weight bearing, if
there is adequate penetration, labeled correctly, etc.
2. Soft tissue- increased contour in a certain area, increased density, tissue
emphysema, foreign body, tissue planes, etc.

4
3. Bone- are all cortices intact, periosteal reaction, density, etc.
4. Joint spaces- increase/decrease in joint space, changes in contour, etc.
5. Biomechanical exam- abnormal angles, cavus foot, pes planus, etc.

The important thing is to have a picture in your head of what “normal” is and work
slowly (to begin with) through each of the steps. Remember not to let your eyes be drawn
to the obvious. Keep in mind the clinical exam and have an idea of what to expect to see
prior to looking at the radiograph. Following those steps gives you an organized way to
not only inspect each radiograph but to organize your thoughts when presenting it.

Dermatology

In your objective examination of a patient, it is necessary to be able to put into words


what you see. Remember that some words such as ulcer, onychomycosis, xerosis, etc. are
considered assessments, not objective statements. For example, it is not appropriate for
you to write the patient has an “ulcer” in the objective. It is more appropriate to write the
patient has a “desquamation of the skin” or something of that nature. The following are
some common words/phrases to objectively discuss lesions of the skin.

Macular- circumscribed, flat lesion


Papular/nodular- circumscribed, solid elevation
Vesicular/bullous- fluid-filled, elevated lesion
Cystic- non-infected, deep-set collection surrounded by a wall of tissue
Scales- thin flakes of dead epidermis
Excoriations- scratch marks
Erosions- essentially thick scratches that do not breach the dermis
Ulcer- deep dermal defect that breaches the dermis or deeper
Fissures- linear, deep epidermal cracks in the skin
Pustule- elevated lesion containing pus, arising from infections of papule or bulla
Abscess- deep collection of pus

Ulcers

Arterial Ulcers
These occur distal to impaired arterial supply most commonly on the lateral aspect of the
leg, when presenting on the lower extremity. However, they can occur anywhere that is
susceptible to trauma such as the IPJ’s of the toes. Wound margins are even, sharply
demarcated and punched out. These have minimum exudate and are very painful.

Venous Ulcers
The medial ankle is the most common location. They are mostly superficial. Moderate to
heavy exudate. The ulcers tend to be large with irregular margins. Often surrounded by
and associated with dermatitis and hyperpigmentation.

Wagner Classification of staging diabetic ulcers:


Grade 0: No ulcer (normal skin flora present)
Grade 1: Superficial skin ulcer (predominantly gram + organisms present when infected
and lesser numbers of gram – aerobes and anaerobes.

5
Grade 2: Deep ulcer through dermis possibly exposing tendon, ligament or joint capsule
Grade 3: Deep ulcer with abscess, osteomyelitis or joint sepsis (polymicrobial infections
with increased number of gram – and anaerobic species)
Grade 4: Localized gangrene (forefoot or heel)
Grade 5: Gangrene of the foot

NPUAP pressure ulcer staging system:


Stage 1: Non-blanchable erythema of intact skin. A defined area of persistent redness,
blue or purple hews. Also known as a deep tissue injury.
Stage 2: Partial thickness skin loss involving epidermis, dermis or both. An abrasion,
blister or shallow crater.
Stage 3: Full thickness skin loss involving damage to or necrosis of subcutaneous tissue
that may extend to underlying fascia.
Stage 4: Full thickness skin loss with extensive destruction, tissue necrosis or damage to
muscle, bone or supporting structures.

Vascular

Evaluating a patient’s vascular status is essential to preventing a wide array of


complications and morbidities. This includes both arterial and venous insufficiency to the
foot. As a podiatric surgeon, one should be able to determine whether there is sufficient
blood flow to a foot in order for it to heal. The following is a list of some of the most
common ways in which vascular status is evaluated in a patient.

A thorough and adequate history of the patient’s symptoms should be taken. This will
include symptoms, location, characterization of the symptoms, effect of treatment,
amount of disability, progression of symptoms, associated symptoms and complaints, any
previous studies done.
Ischemic pain associated with a wound is indicative of arterial disease. This type of pain
may include numbness, coldness, burning, and pallor more severe with exercise.
Intermittent claudication is associated with chronic arterial insufficiency. It is experienced
as transient, exercised induced ischemic myalgia. It is most often seen in the calf muscles.
With arterial blockage, this most often one joint or level above the muscle group in which
the symptoms manifest.
Rest pain usually develops as ischemic changes progress. This reflects severe ischemia.
Blood is shunted from the periphery to a more central location when the person is
sleeping. Metabolites build up in the muscle and cause pain. The patient will usually
relate being awakened at night with pain and having to get up or dangle feet.
There are several other symptoms and pieces of information that are related to vascular
disease:

1. Edema
2. History of emboli
3. Color and temperature changes- pallor, cyanosis, petechiae, telangiectasias
4. Trophic changes and hair growth
5. Past medical history- obesity, tobacco use, HTN, DM, age, etc.
6. Ulcerations
7. Gangrene- impairment of circulation

6
8. Palpation of pulses including femoral, popliteal, posterior tibial, dorsalis pedis
(graded according to rhythm, symmetry and amplitude)

Non-invasive Examination of Vascular Status

Ankle/Brachial Index (ABI)


Measurement obtained by dividing the ankle systolic pressure by the arm systolic
pressure. Some clinical findings associated with the ankle/brachial index:

A/B Index Clinical Finding


> .96 Normal
.71 - .95 Mild obstruction (intermittent claudication)
.31 - .70 Moderate obstruction (possible rest pain)
.00 - .30 Severe obstruction (rest pain/impending gangrene)

Monckeberg’s sclerosis can falsely elevate ABI readings.

Segmental Pressures
These are taken at upper thigh, lower thigh, upper calf and ankle. Criteria based on the 1-
10-20-30 Rule, we can predict potential obstructions.
* An ABI of less than 1 may indicate obstruction.
* The upper thigh pressure should be greater than 10mmHg greater than brachial pressure
* Pressure differences of more than 20 mmHg for adjacent cuff sites on the same leg
* Pressure differences of more than 30 mmHg over the entire leg.

Doppler Signal
These reflect flow velocity. The normal arterial pattern is triphasic. A monophasic signal
indicates pathology.

Digital Blood Pressure


Attained by placing a cuff around the digit, staying away from the joint. Normal digital
pressure is between 70-110 mmHg.

Pulse Volume Recording (PVR)


Reflects the volume of blood that pulses under a cuff. This is obtained by placing cuffs
filled to 60 mmHg pressure around parts to be measured. Closely corresponds to direct
intra-arterial recordings at the level being tested. The waveforms look like high teepees
with a dicrotic notch. Changes in waveforms between segments reflect severe stenosis or
occlusions.

Neurology

Presenting Problems: Can include pain, numbness, tingling, weakness, unsteadiness, and
involuntary movements.

Physical examination should begin with a good history of the problem at hand. After a
good history is obtained, much information can be obtained simply by observation of the

7
muscles of the lower extremity. Observation can demonstrate muscle atrophy or even
hypertrophy. The examiner can also visualize spasticity or even fasciculations. Muscle
strength can be measured and graded:
0 = no movement
1 = trace movement
2 = movement with the aid of resistance
3 = movement against gravity
4 = movement against resistance supplied by the examiner
5 = normal strength

Observing the patient’s gait can also give clues as to where the problem may originate.
Also to be examined are a patient’s deep tendon reflexes (DTR). The symptoms often
present in a dermatomal fashion, which can give certain clues about possible lesions that
may occur at levels along the CNS.
Certain methods of examination for nerve deficits can be done using equipment such as a
tuning fork to test vibratory sense or a Semmes-Weinstein monofilament (10 g/ 5.07) to
test light touch.
Hoffman-Tinel's sign: A tingling in region of the distribution of the involved nerve with
light percussion, results in paresthesias distal to the site of percussion. Due to nerve
entrapment.
Valleix Phenomena: A nerve trunk tenderness above and below the point of compression,
with paresthesias proximal and distal to the point of percussion.

Electromyography (EMG):
When weakness is clinically difficult to attribute to nerve, muscle, or neuromuscular
junction, electrical studies can establish topographically which nerves and muscles are
affected. In EMG, the recording of electrical properties of muscle is displayed on an
oscilloscope during needle insertion. Denervated muscle is recognized by fibrillations and
fasciculations on the screen.
Nerve Conduction Velocities (NCV):
In nerve conduction studies the time for an impulse to travel along the nerve is termed
the conduction velocity. If there is an increase in this conduction velocity, there is damage
to the particular nerve involved.

Types of Neuropathy
D - diabetic
A - alcohol
N - nutritional
G - Guillan-Barre

T - toxic
H - hereditary
R - recurrent
A - amyloidosis
P - porphyria
8
I - infectious
S - systemic
T - tumor

Reflex Sympathetic Dystrophy


This syndrome is characterized by disproportionate pain in intensity, duration, and
location, often from minor or unapparent trauma to an extremity. Clinical diagnosis is
difficult due to the vague subjective data and subtle objective signs and symptoms. Early
diagnosis is important because early treatment gives the best results. The sympathetic
nervous system is always involved and is overactive. Symptoms occur distal to the trauma
site.

Manifestations of RSD include pain, first and foremost. The pain is usually excruciating
and can be described as burning, aching or throbbing. The pain can be localized at first
and then begin to spread. It is distal to the site of trauma to begin with but can spread
proximally in later stages.

RSD can also manifest itself with varied vascular related symptoms. Vasodilation can
exhibit warm skin, dry skin, and hypohidrosis. Vasoconstriction can exhibit cyanosis,
cool skin, edema of the part, and hyperhidrosis.

Trophic changes involving the skin, muscles, bone and joints can occur in later stages.
The examiner may see thin, atrophic skin, absent of hair and witness osteoporotic changes
on x-ray.

Treatment, when suspected, should include a pain management/ neurology consult, first
and foremost. Other professionals to get involved with the case could be psychiatry,
anesthesiology and physical therapy. Local anesthetic blocks have been shown to give
some relief and may help in putting a stop to the pain cycle.

Seddon's classification of nerve injuries:


Neuropraxia: (first degree injuries) a conduction disturbance with complete recovery
Axonotmesis: (second and third degree injuries) an incomplete division of supportive
tissues of the nerve
Neurotmesis: (fourth and fifth degree injuries) a complete division of a nerve

Sunderland's classification of nerve injuries:


First degree: only local changes to the myelin
Second degree: injury to the axons that is incomplete
Third degree: leads to more severe axonal injury with fibrosis
Fourth degree: severe neuronal injury with the axons in complete
disarray (no complete neuronal separation)
Fifth degree: complete transection of the nerve (dismal prognosis)

Neuromas (a misnomer) of the forefoot:


Joplin’s - medial to the first metatarsal

9
Hauser’s - the first intermetatarsal space
Heuter’s - the second intermetatarsal space
Morton’s - the third intermetatarsal space
Islen’s - the fourth intermetatarsal space

Anesthesiology

The American Society of Anesthesiologists (ASA) Physical Status Measure


Class 1: Normal and Healthy - no known diseases
Class 2: Mild Systemic Disease i.e. presence of essential hypertension or mild type
II diabetes
Class 3: Severe systemic disease that is not incapacitating i.e. severe diabetes, type I with
vascular complications
Class 4: Incapacitating systemic disease that is a threat to life i.e. advanced cardiac, renal,
pulmonary, hepatic or endocrine insufficiency
Class 5: Moribund patient who is not expected to live with or without surgery
EMERGENCY OPERATION: any patient in one of the above classes who is operated on
as an emergency (Letter E is placed next to the classification)

Stages of Anesthesia
Stage 1: Analgesia (characterized by variable degrees of analgesia and amnesia)
Plane 1- Pre-analgesia (normal memory and sensation)
Plane 2- Partial analgesia and amnesia
Plane 3-Total analgesia and amnesia

Stage 2: Delirium (extends from the loss of consciousness until the beginning of surgical
anesthesia) (excitement and voluntary activity marked)
Unconsciousness, irregular breathing, pupils dilated

Stage 3: Surgical anesthesia (4 planes)


Plane 1 (sleep)-Rhythmical breathing, eyeball centrally fixed, faint lid reflex
Plane 2 (sensory loss)-Pupils slightly dilated, pulse and blood pressure normal
Plane 3 (muscle tone loss)-Intercostal paralysis begins, increased pulse rate &
decreased BP
Plane 4 (intercostal paralysis)-Provides cessation of all respiratory effort and
requires artificial ventilation for life support

Stage 4: Medullary paralysis


Plane 1- reversible respiratory failure
Plane 2- irreversible cardiovascular collapse
Local Anesthetics
Function in such a way as to prevent sodium migration through the nerve membranes,
which, therefore prevents depolarization of the nerve with inhibition of nerve conduction.
These are divided into 2 classes of drugs: Esters and amides.

Esters:
Procaine (Novocaine): most toxic, and is considered the standard in comparing the
potency and toxicity of other local anesthetics used for injections.

10
Esters are hydrolyzed by pseudocholinesterase in the plasma.
Have a high potential for allergenicity associated with PABA.

Amides:
Lidocaine (Xylocaine): shortest duration & fastest action
Bupivicaine (Marcaine): longest duration, least placental transfer, and greatest cardiac
toxicity if given IV
Amides are hydrolyzed in the liver. There is no cross sensitivity between amides and
esters- can be substituted in case of allergy. Should use ½ dose in elderly, debilitated
patients, and patients with hepatic disease. Pain and temperature lost first following nerve
block, with loss of touch & motor function later. Injection into an acidic area (infection)
converts the anesthetic chemically and does not allow for penetration into the cell
membrane, and lessens its effectiveness.

Maximum Allowable Single Dose in Normal Adults:


Novocaine (1-2%): 750mg plain; 1000mg w/epinephrine
Xylocaine (1-2%): 300mg plain; 500mg w/epinephrine
Marcaine (0.25-0.75%): 175mg plain; 225mg w/epinephrine
(.25% Marcaine= 2.5mg/cc, 0.5%= 5mg/ cc 1 % Xylocaine= 10mg/cc, 2%= 20mg/cc)

Malignant hyperthermia
Thought to be due to reduction in the reuptake of Ca by the sarcoplasmic reticulum
necessary for the termination of muscle contraction

Clinical features:
Unexplained tachycardia, hypercarbia or tachypnea, acidosis, muscle rigidity even in the
presence of neuromuscular blockade, hypoxemia, ventricular arrhythmias, hyperkalemia,
fever is a late sign

Treatment:
1. Discontinue all anesthetics
2. Dantrolene 2.5 mg/kg IV initially
3. NaHCO3
4. Hyperkalemia corrected with insulin and glucose (no calcium)
5. Arrhythmias treated with procainamide
6. Hyperthermia treated with refrigerated IV fluids, gastric, rectal and bladder lavage with
cold saline, surface cooling with ice
7. Maintain urine output

Analgesics
This is just a short list of some of the most commonly used pain medications for post-
operative and inpatient pain management. Also included are some NSAID’s commonly
used.

Morphine sulfate: 2.5-10 mg SC/IM/IV q2-6h prn

11
Dilaudid: 1-4 mg SC/IM/IV q4-6h ; 2-8 mg PO q3-4h
Percocet: (5/325, 7.5/325, 7.5/500, 10/325, 10/650): 1 tab PO q4-6h prn
Oxycontin: (10, 15, 20, 30, 40) 1 tab PO q12h
Darvon: 65 mg PO q4h prn
Darvocet 50 (N-100, A-500): 1 tab PO q4h prn
Demerol: 50-150 mg PO/SC/IM/IV q3-4h (decrease dose if given IV and dilute)
Ketorolac (Toradol): 30-60 mg IM/IV x 1 dose only (do not exceed 5 days duration)
Vicodin (500/5), Vicodin ES (750/7.5), Vicodin HP (660/10): 1-2 tab PO q4-6h
Tylenol #3 (300/30): 1-2 tab PO q4-6h
Ultram (tramadol): 50-100 mg PO q4-6h prn
Motrin (ibuprofen): 400-600 mg PO q6h
Naprosyn (naproxen): 250-500 mg PO q12h
Mobic (meloxicam): 7.5-15 mg PO qd
Celebrex: 200 mg PO q12-24h

Trauma

When describing any fracture, there are 6 characteristics that should be used. This is very
useful when asked to describe a fracture to an attending while on the phone, such as in the
ED at 2 a.m. Just like with an x-ray, organizing your thoughts will allow you to discuss
the fracture in a more decisive manner.

1. Open vs. closed


2. Shape
3. Location
4. Intra-articular vs. extra-articular
5. Complete vs. incomplete
6. Displaced vs. non-displaced

Example: A closed, transverse, through and through, extra-articular fracture of the 5th
metatarsal base that is non-displaced.

Example: A closed, spiral oblique, intra-articular, through and through, displaced fracture
of the distal fibula.

Open Fractures

Classification (Gustillo and Anderson):

Type 1:
- Wound less than 1 cm long and clean
- Minor soft tissue damage
- Minimal comminution

Type 2:
- Wound more than 1 cm long without extensive tissue damage, flaps or avulsions
- Slight crush injury with moderate comminution
- Moderate contamination

12
Type 3: Extensive soft tissue damage, including muscles, skin, and neurovascular
structures with a high degree of contamination
Type 3A: Open fractures with adequate soft tissue coverage of bone despite extensive soft
tissue laceration
Type 3B: Open fractures having extensive soft tissue loss with periosteal stripping and
bone exposure. Severe contamination. Severe comminution.
Type 3C: Open fractures associated with arterial damage requiring vascular repair,
regardless of soft tissue coverage

General Principles of Treatment of Open Fractures

1. Tetanus history
2. Thorough H&P with complete neurological, vascular and musculoskeletal exam
3. Complete x-rays
4. Antibiosis admitted in the ED with cultures
5. Immediate debridement and irrigation
6. All foreign bodies should be removed
7. All marginal and macerated tissue should be debrided and re-debrided as
necessary after 48-72 hours.
8. Types 1, 2 and 3A get delayed primary closure within 5-7 days
9. Types 3B and 3C usually require skin flaps due to such a loss of tissue
10. External fixation for all type 3 and unstable type 2 fractures
11. Internal fixation should be used for articular and metaphyseal open fractures,
preferably within 8 hours.

Tetanus Prophylaxis (for adults)

Clean and minor wounds:


Toxoid given only if last dose was given > 10 years ago.
TIG not given

All other wounds:


Toxoid given if last dose given > 5 years ago
TIG not given

TIG is only given in patients (any age) when they have never received a tetanus shot
(usually pediatric patients)

Compartment Syndrome

This diagnosis should always be considered in a patient experiencing pain and swelling
post trauma. This should be considered a clinical diagnosis (it can be measured, however)
and is also considered a surgical emergency.
Defined as increased compartmental pressure resulting in decreased perfusion and
ultimate ischemic changes to the tissues.
At rest, intramuscular pressure is approximately 5 mmHg. During a muscle contraction,
the compartmental pressure can increase to 150 mmHg. At relaxation, the compartment

13
pressure rapidly drops, and within 5-10 minutes after relaxation, has returned to baseline.

Intracompartmental pressures can be measured with a wicks catheter. During a


compartment syndrome, pressures are over 30 mmHg. Any injury that causes an increase
in compartmental pressure over this amount should undergo an immediate fasciotomy.

Clinical findings include: 1. Pain out of proportion to clinical findings 2. Parasthesias 3.


Pulselessness 4. None of the above.

Perioperative Management of Patients

In a healthy patient, there are 3 main areas to concentrate on in order to minimize the
risks associated with any surgery: coagulation disorders, drug history and previous
anesthetic complications. Certain labs and tests should be ordered for all cases. For local
anesthesia: CBC-Diff, Chem-7, PT/PTT/INR, and EKG. For general anesthesia cases:
CBC-Diff, Chem-7, PT/PTT/INR, EKG, Chest X-ray if smoker or older than 40
(according to some sources).

Diabetic Patient
1. Early morning surgical preference
2. If surgery is delayed, start IV with D5W to avoid hypoglycemia (better sweet than
sour) from remaining insulin or hypoglycemics from the day before.
3. Regular insulin should be used during the perioperative period.
4. Check pre-op potassium levels. Insulin reduces these levels.
5. Autonomic neuropathy can cause increased risks of gastroparesis and aspiration,
resting tachycardia and orthostatic hypotension.
6. Pre-op blood sugar levels:
a. Hypoglycemia in a pre-op patient, consider postponing the case
b. Pre-op BS < 200 in diet controlled patient no meds are needed
c. Pre-op BS 200-300 in diet controlled patient consider regular insulin
d. Pre-op BS in patient taking oral hypoglycemics <150, do not take the drug
on the morning of the surgery
e. Pre-op BS in patient taking oral hypoglycemics 150-250, take morning
dose of oral agent.
f. Pre-op BS in patient taking oral hypoglycemics >250, start regular insulin
7. Avoid post-op hyperglycemia
8. For minor procedures in Type 2 DM patients undergoing minor procedures,
discontinue the oral hypoglycemic one day before surgery and resume taking the
medication when regular diet is resumed. Patient should not get any glucose IV.
9. For long, major procedures, it is safer to utilize IV insulin during surgery.
10. Administering insulin to Type 1 DM patients undergoing surgery:
a. ½ the usual morning dose and start D5W IV and cover with insulin during
case OR….
b. Start 500 ml D5W for the first hour, followed by 125 ml/hr of D5W with
1-2 Units of regular insulin
11. Begin a sliding insulin scale post-op

14
Patient on steroid therapy
Steroids are used to treat many conditions including asthma, COPD, rheumatoid arthritis,
and malignancy. They have an effect on three main areas of importance in the
perioperative patient. Suppression of the hypothalamus/pituitary adrenal axis. Poor
wound healing. Predisposition to infection.
If a patient has taken more than 7.5 mg/ day of prednisone, then exogenous steroids must
be supplied during the perioperative period. Otherwise there could be a resulting
hypotension and cardiovascular collapse.
For procedures, the regimen of exogenous steroids is: Hydrocortisone IV 100 mg pre-op
and 100 mg post-op. Tapering of steroids is only necessary if coverage lasts longer than 3
days.

Patients on anticoagulant therapy

Heparin Therapy- inhibits intrinsic clotting pathway


The half-life of heparin is 1.5 hours. When a patient is undergoing heparin therapy, the
treatment regimen should be stopped the day before surgery.
Effects are reversed with protamine sulfate.
Dosage is regulated according to PTT when treating, not when prophylaxing.
To prophylax for DVT, short term, give 5000 Units SC 2 hours prior to surgery and 5000
Units q12h until patient is ambulatory.

Coumadin Therapy- inhibits extrinsic clotting pathway


Half-life is 20-60 hours and is extremely variable according to rate of clotting factor
catabolism. When a patient is on coumadin therapy, treatment should be stopped 5-7 days
prior to surgery. If the surgery is necessary, heparin therapy can begin at that time and last
until 2-4 hours before surgery.
Effects are reversed by Vitamin K (delayed) or Fresh Frozen Plasma (immediate).
Dosage is regulated according to PT/INR.

Patient with clotting abnormalities


Be sure to ask patient about previous bleeding problems, bruising, nose bleeds.
Certain drugs can alter platelet function: aspirin, NSAID’s, steroids, antihistamines, IV
penicillin and drugs listed above.
Test to determine clotting issues: platelet count, PT, PTT, INR

Cardiac patient
Elective surgery should be postponed until 6 months post myocardial infarct and in
patients with uncompensated congenital heart failure.
All risk factors such as unstable angina and cardiac failure should be stabilized
preoperatively.
Pre-op studies should include the studies for the normal patient undergoing general
anesthesia, plus EKG and chest x-ray.
Nitrates and beta-blockers should be continued during the peri-operative period.
EKG should be considered in any patient with a murmur.

Gout patient
Should receive colchicine 0.5 mg 3 times daily for 2-3 days prior to the operation and 4-5

15
days post-operatievly. Low risk patients do not need to be treated.

Postoperative Care and Complications

Fever (most common post-op problem)


Temp > 101.6 PO.
A simple pneumonic for remembering causes of fever in a post-op patient:
Wind (1 day post-op)- atelectasis, aspiration
Water (2 days post-op)- UTI, IV site phlebitis
Wound (3 days post-op)- infection
Walk (4 days post-op)- DVT
Wonder (> 4 days)- drug reaction
Of course, these are not set in stone and any can happen at any time. Clinical correlation
should be made with all fevers.

Chest Pain
Possible causes include: MI, pulmonary embolus, pneumonia, anxiety, atelectasis,
aspiration, CHF
Treatment: EKG, chest x-ray, ABG’s. May also do ventilation perfusion radionucleotide
lung scan, pulmonary angiography (most specific and sensitive for PE).

Post-operative Infection
Usually occurs 3-7 days post-operatively. Can occur sooner (Strep Group A).
Signs/Symptoms: Pain, swelling, drainage, dehiscence, cellulitis, erythema, fever
Treatment: Patient should be admitted to the hospital if they are experiencing systemic
symptoms (nausea, vomiting, chills, sweats) or if they are immunocompromised or need
surgical debridement. Perform local wound care including removing sutures and incision
and drainage. Obtain a culture and sensitivity test. Obtain necessary lab studies. Begin
empiric therapy with antibiotics and adjust per result of the C&S. Continue antibiotics for
10-14 days.

Pain Management
Pain medications were covered previously, but this information is in regards to pain as a
complication to surgery.
Injecting a long acting local anesthetic (bupivicaine) immediately postoperative will delay
pain past the initial phases.
A short acting steroid (dexamethasone) will help diminish postoperative swelling and
inflammation.
NSAID’s can be given to help reduce inflammation.
Opiates
Sedatives can reduce the anxiety associated with surgery, therefore reducing the pain.

Nausea
Antiemetics – Compazine, Phenergan, Reglan, Zofran

Constipation

16
First, if possible, get the patient moving. Walking can sometimes induce bowel
movements in postoperative patients.
Drugs- senna, lactulose, Colace, Dulcolax

DVT
Symptoms: red, hot, swollen calf with fever
Diagnostic: venous duplex, venogram
Treatment: Heparin IV 5,000-10,000 Units followed by a constant infusion of 1,000-
1,500 Units/hr. PTT is to be kept at 2 times the baseline and the PT is kept at 1.5 times
the baseline. Coumadin should be started on admission (it takes 3-5 days to begin
working). The patient should eventually be discharged home on coumadin.

Internal Fixation

Objectives of fixation devices


1. Eliminate motion at the fracture or osteotomy site.
2. Restore the normal anatomical alignment of the fractured site or the desired
position of an osteotomized segment.
3. Assist in the physiological mechanism of bone healing.
4. Permit early mobilization of the area affected by the fracture or osteotomy.

Cortical screws- function as either a positional screw (plate fixation) or a lag screw
(exerts compression). Compression is only achieved when the threads of the screw do not
engage the cortex of the near osteotomy or fracture fragment, accomplished by
overdrilling.

When screws are used alone for fragment fixation, two smaller screws provide increased
resistance to shear and torsional stresses.
When screws are used for interfragmentary compression, they should be inserted so that
their direction bisects the perpendiculars to the fracture line and the long axis of the bone
involved.
Sufficient screw fixation can usually be obtained with oblique and spiral fracture patterns
only when the fracture line is twice as long as the bone’s diameter. Short oblique or
transverse fractures therefore need an interfragmentary lag screw and a neutralization
plate.

Cancellous Screws- fully or partially threaded. The cancellous screw thread height is
greater than that of cortical. This allows for greater purchase in the softer metaphyseal
and epiphyseal bone for which they were designed. If the threads are left crossing the
interface between two fragments, no compression will be achieved as the lag effect will
be negated. It then acts as a cortical screw.

Herbert Screw- absence of a screw head. Characterized by the presence of threads with
different pitches and leads on both its proximal and distal ends. The distal threads feature
a tighter pitch and smaller lead and are separated from the proximal ones by an
intervening smooth shank. This allows for interfragmentary compression.

17
Plate Fixation

Static compression plates: tension is applied to the implant and compression is achieved
at the fracture interface.
Dynamic compression plates: beyond the compression of the fracture achieved through
static compression, the implant is subjected to a physiological load, which generates
additional compression at the fracture plane.
Neutralization plates: initially a shaft fracture is fixated by interfragmental compression
with a lag screw. A plate is then applied to neutralize or absorb any disruptive forces-
torsional, shear or bending to which the bone and osteosynthesis may be subjected.
Anti-glide plates: used as neutralization plates but placed on the posterior aspect of the
fibula.
Buttress plates: generally used to resist the tendency of metaphyseal fracture fragments to
displace when subjected to compressive forces (tibial plafond or calcaneal plates).

Pre-stressing (bending) the plates results in static interfragmentary compression. Contour


the plate so that the center sits away from the bone to which it is applied. The screws
securing the plate ends are inserted and tightened first (pre-stressing the plate in tension)
so that as sequential screws are applied (progressively closer to the center) axial
compression is developed along the underlying bone.

AO Objectives
1. Atraumatic operative technique
2. Accurate anatomical reduction
3. Rigid internal compression
4. Avoidance of soft tissue damage
5. Early range of motion

Techniques of stable fixation


Single lag screw: A cortical screw with a glide hole or a cancellous screw with all the
threads on the distal side of the fracture fragment. The screw should be placed so that the
angle of the screw bisects the perpendicular of the fracture/osteotomy and the
perpendicular of the longitudinal axis of the bone.

Multiple lag screws: This technique is used in a long/oblique or spiral fracture, where the
length of the fracture is at least twice the diameter of the diaphyseal bone involved. The
first screw should be placed perpendicular to both cortices and be centrally placed. The
second and third screws are placed perpendicular to the plane of the fracture. This
prevents shearing.

AO Technique of Screw Placement


1. Underdrill
2. Overdrill
3. Countersink
4. Measure
5. Tap
6. Screw insertion

18
Other Techniques of Fixation
Circlage wire: The weakest form of internal fixation. Provides apposition of the
osteotomy surfaces, but little stability. Most secure is two loops at 90 degree orientation
to each other.
K-wire: Crossed k-wires provide more rigidity than just one. Distraction of the fracture
fragments can occur. This does not provide interfragmental compression. Threaded k-
wires are mechanically unsound and are rarely used.
Tension banding: Monofilament wire threaded in a figure 8 pattern, used in combination
with two k-wires to give interfragmental compression. This is good with Jones fractures
and some ankle fractures. The plane of insertion of the 2 k-wires must be parallel to the
plane of the drill hole for passage of the monofilament wire.

Infection and Antibiotics (Infectious Disease Society of America suggestions)

Mild Infections
2 of the following characteristics: purulence, erythema, warmth, pain, tenderness,
induration

PO antibiotics: Dicloxacillin, Clindamycin, Cephalexin, TMP-SMZ,


Amoxacillin/Clavulanate, Levofloxacin

Moderate Infections
Characteristics: cellulitis > 2 cm from wound, lymphangitic streaking, abscess, gangrene

PO vs. IV antibiotics: TMP-SMZ, Augmentin, Levofloxacin, Cefoxitin, Ceftriaxone,


Timentin, Ciprofloxacin + Clindamycin, Unasyn, Linezolid, Daptomycin, Ertapenem,
Cefuroxime, Zosyn

Severe Infections
Characteristics: fever, chills, confusion, acidosis, hyperglycemia, tachycardia,
hypotension, vomiting, leukocytosis, azotemia

IV antibiotics: Zosyn, Levofloxacin, Ciprofloxacin + Clindamycin, Primaxin,


Vancomycin + Ceftazidime

Life Threatening
Empirically cover S. aureus, Streptococcus, Enterobacteriaceae, Bacteroides fragilis

IV antibiotics: Unasyn, Zosyn, Primaxin, Ertapenem, Fluorquinolone + 3rd generation


cephalosporin, Clindamycin + Ciprofloxacin or Levofloxacin

Ensure empiric Gram (-) coverage for diabetic foot ulcers:


- Beta-lactamase inhibitors
- 2nd and 3rd generation cephalosporins

19
- fluoroquinolones
- clindamycin

Pseudomonas when suspected


- Ciprofloxacin
- Zosyn

Cellulitis without an open wound


- cover for beta-hemolytic Streptococcus and S. aureus

Infected ulcer not previously treated with antibiotics


- beta-hemolytic Streptococcus and S. aureus

Chronic infected ulcer or previously treated with antibiotics


- S. aureus, beta-hemolytic Streptococcus, Enterobacteriaceae

Macerated ulcer secondary to soaking


- Pseudomonas (often with another organism)

Chronic wounds, prolonged treatment, broad antibiotic therapy


- MRSA, VRE, extended spectrum beta-lactamase gram (-) rods
- Also Dipthiroids, Enterobacteriacea, Pseudomonas, gram (-) rods

Necrosis, gangrenous, stinky foot


- Enterococci, Enterobacteriacea, non-fermentive gram (-) rods, obligate anaerobes

Community-acquired MRSA
- Clindamycin, Doxycycline, TMP-SMZ

Patient with PAD


- think anaerobe; Bacteroides; Metronidazole

20
21
Hershey Board Certification Review
Outline Study Guide

by
Jonathan M. Singer, D.P.M.
Diplomate, American Board of Podiatric Surgery
Fellow, American College of Foot and Ankle Surgeons
Diplomate, American Board of Podiatric Orthopedics & Primary Podiatric Medicine
Fellow, American College of Foot & Ankle Orthopedics & Medicine

Edited by
Larry R. Goss, D.P.M.
Diplomate, American Board of Podiatric Surgery
Fellow, American College of Foot and Ankle Surgeons
Fellow, American College of Foot & Ankle Orthopedics & Medicine
Fellow, American Professional Wound Care Association
Residency Director, Parkview/Roxborough/MCP Hospitals
Clinical Instructor, Temple University School of Podiatric Medicine

Jason Miller, D.P.M.


Other Contributors
Christopher E. Attinger, M.D., Steven J. Berlin, D.P.M., Libby Cone, M.D.,
Douglas Glod, D.P.M., Larry R. Goss, D.P.M., Arthur E. Helfand, D.P.M., Edwin J.
Juda, D.P.M., Peter F. Kelly, D.P.M., Steven Kravitz, D.P.M., Guido LaPorta,
D.P.M., M.S., James Lawton, D.P.M., Jason Miller, D.P.M., Gene K. Potter, D.P.M.,
Ph.D., William H. Simon, D.P.M., John Walter, D.P.M., M.S., Fredrick E.
Youngblood, M.D.

© 2003 Pennsylvania Podiatric Medical Association


Acknowledgements

William Goldfarb D.P.M. and Nancy Sullivan, for supporting me and


believing this book could be done

Larry R. Goss, D.P.M. and Jason Miller, D.P.M. for their academic
contributions and updating this new addition

James Lawton D.P.M. and Gene Potter, D.P.M. for their invaluable
contributions in the general editing these review notes in the previous
addition

Jordon Fersel, M.D., for editing the chapters on Anesthesia and


Emergency Medicine

Deborah Wozniak, M.D., for editing the chapters on Perioperative


Management, Fluid Management, and Postoperative Complications

Warren Joseph, D.P.M., and Steven Maffei, D.P.M. for editing the chapter
on Infectious Diseases

Steven Kravitz, D.P.M., for editing the chapter on Biomechanics

George Vito, D.P.M. for editing the chapter on Internal Fixation

Peter F. Kelly, D.P.M. for writing the chapter on Lasers and Laser Physics

Gene K. Potter, D.P.M., Ph.D. and Steven Berlin, D.P.M. for editing the
chapters on Dermatology, Bone Tumors, and Soft Tissue Tumors

Guido LaPorta, D.P.M., M.S. for writing the chapter on Ankle Arthroscopy

Tracey Vlahovic, D.P.M. for editing the chapter on Dermatology


The Hershey Board Certification
Review Outline Study Guide
is prepared for distribution by the
Hershey Surgical Seminars Committee
Pennsylvania Podiatric Medical Association
Guido La Porta, D.P.M., General Chairman

Author: Jonathan M. Singer, D.P.M.


General Editors: Larry R. Goss, D.P.M. and Jason Miller, D.P.M.
Copy/Content Editors: Larry R. Goss, D.P.M. and Jason Miller,
D.P.M.
Cover Design: Larry R. Goss, D.P.M.
Coordinator: Nancy Sullivan
Published by the Pennsylvania Podiatric Medical Association
ISBN # 0-9639030-04

Copyright © 2003
Pennsylvania Podiatric Medical Association 757 Poplar Church Road
Camp Hill, Pennsylvania 17011 (800)-841-3668
All rights reserved

This book is protected by copyright. No part of this book may be reproduced in any
form or by any means, including photocopying, or utilized by any information
storage or retrieval system without written permission from the copyright owner.

Accurate indications, adverse reactions, and dose schedules for drugs are provided in
this book, but It is possible that they may change. The reader is urged to review the
package information data of the manufacturers of the medications mentioned.

The author has made every effort to ensure the accuracy of the information herein.
However, appropriate information sources should be consulted, especially for new or
unfamiliar procedures. It is the responsibility of every practitioner to evaluate the
appropriateness of a particular opinion in the context of actual clinical situations
and with consideration to new developments. The author, editors, and publisher
cannot be held responsible for any typographical or other errors found In this book.

Printed in the United States of America.


Table of Contents
Chapter 1: Emergency Medicine Water and Electrolyte Imbalance: page 65-
Cardiopulmonary Resuscitation: page 2-3 66
Advanced Life Support: page 3-4 Oliguria: page 66
Office Emergencies: page 5-7 Chest Pain: page 67
Medical Emergencies (Cardiac Postoperative Hypertension: page 67
Dysrhythmias): page 7-9 Postoperative Infection: page 67-68
Summary of CPR: page 9 Anxiety and Pain Management: page 68-69
Other Medical Emergencies: page 9-12 Nausea: page 69
Shock: page 12-13 Constipation: page 69
Blood and Blood Products for Emergency Shivering: page 69
Use: page 13-17 DVT: page 69-70
Compartment Syndrome: page 70-73
Chapter 2: Anesthesia Hemorrhage: page 72-73
Anesthesia Classifications: page 20 Septic Syndrome: page 73
General Anesthesia: page 20-23 Thyroid Storm: page 73-74
Intravenous Anesthesia: page 23
Local Anesthesia: page 23-26 Chapter 6: Infectious Diseases
Special Cases for Lowering the Maximum Specific Diseases: page 76-81
Allowable Dose: page 26 Principles of Antiinfective Therapy: page 81-
Pediatric Anesthesia: page 26-27 84
Lumbar Epidural and Caudal Antibiotics: page 84-90
Anesthesia: page 27-28 Drug Fever: page 90-91
Complications of Endotracheal Intubation: Specific Antimicrobial Therapy: page 91-92
page 31 Surgical Considerations: page 92-93
Nerve Injury During Anesthesia: page 31-32 Necrotizing Infections: page 93-94
Other Medical Compliccations From Osteomyelitis: page 94-102
Anesthesia: page 32-34 Septic Arthritis: page 102
Mycology: page 102-103
Chapter 3: Perioperative Management Viral Diseases: page 104
The Healthy Patient: page 36 Rickettsial Diseases: page 104
The Diabetic Patient: page 36-38 Protozoan and Metazoan Infections: page
The Hypertensive Patient: page 38-39 104-105
The Patient on Steroid Therapy: page 39-40
The Asthmatic Patient: page 40-41 Chapter 7: Wound Healing
The Alcoholic Patient: page 41 Repair of Skin: page 108-111
The Patient on Anticoagulant Therapy: page Factors That Interfere With Wound Healing:
41-42 page 111-113
The Patient With Clotting Abnormalities: Growth Factors in Wound Repair: page 113
page 42 Surgical Approaches: page 113-114
The Rheumatoid Patient: page 43 Wound Dressings: page 114
The Sickle Cell Patient: page 43-44 Suture Materials and Needles: page 115-
The Cardiac Patient: page 44-45 116
The Pulmonary Disease Patient: page 45 Tourniquets: page 116-118
Perioperative Management of the Infant and
Child: page 45-47 Chapter 8: Plastic Surgery
Mitral Valve Prolapse patient: page 47-48 Skin Grafting: page 120-122
The Gouty Arthritis Patient: page 48 Local Cutaneous Flaps: page 122-124
Local Muscle and Cutaneous Flaps: Page
Chapter 4: Preoperative Evaluation 124
Screening Procedures: page 50-61 Fasciocutaneous Flaps: page 124-125
Summary of Perioperative Lab testing: page Microsurgery and Free Flaps: page 125-126
61-62 Diagrams of Skin Flaps: page 126-130

Chapter 5: Postoperative Care & Chapter 9: Bone Healing


Complications Primary Bone Healing: page 132
Fever: page 64-65 Complications of Bone Healing: page 133-
Altered Mental States: page 65 135
Treatment of Nonunions: page 135 218
Fusion: page 135 Principles Affecting Internal Fixation: page
Bone Grafting: page 135-137 218
Aseptic Necrosis Following 1st Metatarsal Internal Fixation Devices: page 219-229
Osteotomy: page 138-139 AO Objectives: page 229
Electrical Bone Stimulation: page 40 AO Principles: page 229-230
Cartilage Healing: page 140 AO Technique: page 230-235
Jumping Screws: page 235-236
Chapter 10: Burns and Frostbite Other Techniques: page 236-238
Assessment and Treatment of Burns: page Complications of Fixation Devices: page 238
142-145 External Fixation: page 239
Burn Deformities: page 145-146 Large Bone Fixation: page 239-252
Other Types of Burns: page 146 Small Bone External Fixation: page 253-
Frostbite: page 146-147 254

Chapter 11: Fluid Management and Chapter 17: Nail Disorders & Surgery
Replacement in the Surgical Patient Nail Entities: page 256-260
Fluid Management: page 150-152 Nail Anatomy: page 260
Complications of Total Parenteral Nutrition: Surgical Nail Procedures: page 262-263
page 152
Replacement Therapy: page 152-153 Chapter 18: Neurology
Blood Products and Indications for Use: Neurological Pathways: page 266
page 153-156 Presenting Features: page 266-270
Composition of IV Fluids: page156-157 Assessment of Clinical Problems: page 270-
278
Chapter 12: Bone Tumors Innervation of the Lower Extremity: page
Introduction: page 160 278-281
Staging Techniques: page 160-161 Tarsal Tunnel Syndrome: page 282-284
Surgical Staging System: page 161 Classification of Nerve Injuries: page 284-
Radiology of Bone Tumors: page 161-162 285
Bone Tumors: Page 163-168 Neuromuscular Causes of Cavus Foot: page
Cystic Lesions of Bone: page 168-169 285
Other Tumors and Cancers: page 169-170 Types of Nerve Surgery: page 285-288
Bone Tumors: A Quick Reference Chart:
page 170-171 Chapter 19: Pediatrics
Phases in Cancer Development: page 171 Evaluation of the Pediatric Patient: page
Benign vs. Malignant: page 172 290-293
Pedicatric Biomechanics-Normal Values:
Chapter 13: Dermatology Newborn to Adult: page 293
The Skin: page 174 Congential Deformities: page 293-302
Dermatological Lesions: page 175-182 Pediatric Radiology: page 302-304
Pediatric Gait Patterns: page 304-305
Chapter 14: The Arthropathies Intoe Gait: page 306-307
Causes of Joint Pain: A Summary: page 184 Pediatric Fractures: page 307-318
The Arthropathies: page 184-195 The Limp in Childhood: page 318-319
Lab Testing for the Arthropathies: page 195 Juvenile Hallux Valgus: page 319-320
Articular Disorders Affecting the Heel: page Biomechanical Examination of the Child:
195-196 page 320-323
Synovial Fluid Analysis: page 197 Corrective Casting Techniques in Infants:
Charcot Foot: page 198-199 page 323-324
The Toe-Walking Child: page 324-326
Chapter 15: Peripheral Vascular Disease
Patient Evaluation: page 202-212 Chapter 20: Drugs and Drug Interactions
The Vascular Diseases: page 212-215 Introduction to Drug Interactions: page 328
Antibiotic Drug Interactions: page 328-329
Chapter 16: Internal & External Fixation Anticoagulant Drug Interactions: page 330
Objectives of Fixation Devices: page 218 Antihypertensive Drug Interactions: page
Requirements of Implant Materials: page 330-331
Antipsychotic Drug Interactions: page 331 Physical Examination: page 418-419
Cardiac Drug Interactions: page 331-332 Preoperative Considerations: page
Diuretic Drug Interactions: page 332 419
Non Steroidal Anti-Inflammatory Drug Arthroplasty Procedures: page 419
Interactions: page 332-333 420
Oral Contraceptive Drug Interactions: page Capsule-Tendon Balancing
333 Procedures: page 421-422
Mechanisms of Drug Interactions: page Implant Arthroplasty Procedures:
333-335 page 422-426
Specific Drugs: page 335-344 Arthrodesing Procedures: page 426
429
Chapter 21: Surgery of the Congenital Foot Proximal Phalangeal Osteotomies:
Flatfoot Surgery (flexible): page 346-355 page 429-431
Subtalar Joint Blocking Procedures: Distal Metatarsal Osteotomies: page
(Arhroereises and Arthrodesis): page 355- 432-436
356 Proximal Metatarsal Osteotomies:
Flatfoot Surgery (rigid): Convex Pes Plano page 436-438
Valgus: page 356-358 Shaft Osteotomies: page 438-439
Metatarsus Adductus Surgery: page 358- Combination Procedures: page 439
362 Other Procedures: page 439-440
Cavus Foot Surgery: page 362-367 Hallux Rigidus and Limitus: page
Clubfoot Surgery: page 368-373 440-445
Hallux Varus: page 445-446
Chapter 22: Generalized Disease Conditions Hallux Hammertoe: page 446
of Bone Diff. Diagnosis of Pain In the
Soft Tissue Overgrowth: page 376 Sesamoid Area: page 446
Abnormalities of Alignment: page 376-377 Complications of HAV Surgery: page
General Increased Bone Density: page 377 447
Osteopenia: page 377-378 HAV Procedure Chart and Their
Marrow Abnormalities: page 378- Indications: page 448
379
The Dysplasias: page 379-380 Chapter 25: Trauma
Medical Management and General
Chapter 23: Radiology Assessment of the Trauma Patient:
Standard Radiographic Techniques page 450-451
of the Foot and Ankle: page 382- Assessment of Lower Extremity
383 Injury: page 451
Specific Radiographic Studies: page General Evaluation and Treatment
383-391 of Fractures: page 451-452
Anatomic Angles: page 391-396 Calcaneal Fractures: page 452-460
Common Structural Measurements Talar Fractures: page 460-463
(Diagrams): page 396-402 Osteochondral Fractures: page 463
Pediatric Radiology: page 402-406 465
The Osteochondritities: page 406-407 Navicular Fractures: page 466-470
The Accessory Bones of the Foot: Cuboid Fractures: page 470-472
page 407 Cuneiform Fractures: page 472
Fifth Metatarsal Base Fractures: page
Chapter 24: Hallux Valgus & 472-474
Related Disorders Metatarsal Fractures: page 474-480
Goals of HAV Surgery: page 410 Ankle Fractures: page 480-493
Anatomical Facts of the 1st MPJ: Midtarsal Joint Dislocations: page 494
page 410-411 Tarsometatarsal Joint
Predisposing Factors: page 412 Dislocations/Fractures: page 494-498
Biomechanics: page 412-414 Ankle Inversion Sprain: page 498
Etiology of HAV: page 414-415 505
Deformity Types: page 416-417 Deltoid Ligament Ruptures: page
Radiological Measurements for HAV: 506-507
page 417-418 Compartment Syndrome: page 507
508 571-572
Open Fracture Classification System Tenosynovitis: page 572
and Tx: page 508-510
Soft Tissue Injuries: page 510-511 Chapter 28: Lesser Metatarsal
Crush, Gunshot, and Lawnmower Surgery
Injuries: page 511-512 Anatomy (Metatarsals 2-3-4): page
Puncture Wounds: page 512 574-575
Epiphyseal Plate Injuries: page 512-514 Differential Diagnosis of
Digital Fractures and Dislocations: Metatarsalgia: page 575
page 514-515 Surgical Treatment of the IPK: page
1st MPJ Trauma: page 516-517 575-578
Nail Bed Trauma: page 517-518 Lesser Metatarsal Joint Replacement:
Toe Tip Injuries With Tissue Loss: page 578-579
page 518-519 Panmetatarsal Head Resection: page
Dog and Cat Bites: page 519 579-581
Metatarsus Adductus: page 581-585
Chapter 26: Digital Deformities Freiberg's Disease: page 585-587
and Surgery Tailor's Bunion: page 587-589
Hammertoe Syndrome: page 522- Splayfoot: page 589
528 Brachymetatarsia
Mallet Toe Syndrome: page 528-529 (Brachymetetopody): page 589-591
Claw Toe Deformity: page 529 Skewfoot: page 591-592
Overlapping 5th Toe: page 529-532
Hallux Hammertoe: page 532-533 Chapter 29: Ankle Conditions
Lesser Digital Arthrodesis: page 533 Differential Diagnosis of Chronic
Hallux Interphalangeal Arthrodesis: Ankle Pain: page 594
page 533 Tarsal Tunnel Syndrome: page 594-
Lesser Digital Arthrodesis: page 533- 597
534 Sinus Tarsi Syndrome: page 597-
Overlapping 2nd Toe: page 534 599
Syndactylization: page 534-535 Peroneal Subluxation: page 599-
Digital Implants: page 535-537 600
Floating Toe Syndrome: page 537 Ankle Arthrodesis: page 600-602
Blue Toe Syndrome: page 538-539 Lateral Ankle Instability: page 603
Polydactylism: page 539-541 Chronic Lateral Ankle Instability:
page 604-607
Chapter 27: Muscle and Tendon Chronic Medial Ankle Instability:
Pathology page 607
Muscle Physiology: page 544-548 Ankle Equinus: page 607-611
Principles of Tendon Repair: page Malunion and Nonunion of the
548-552 Malleolei: page 611-614
Tendon Lengthening and
Tenotomy: page 552-554 Chapter 30: Heel Conditions
Tendon Transfers: page 554-556 Anatomy of the Heel: page 616-617
Tendon Grafts: page 556-557 Radiological Evaluation of the Calcaneus:
Posterior Tibial Tendon Rupture: page 617-618
page 557-559 The Heel in Systemic Disease:
Posterior Tibial Tendon Dysfunction page 618-621
(Acquired Adult Flatfoot Syndrome): Seronegative Arthritis and Heel Pain:
page 559-560 page 621-622
Peroneal Tendon Pathology: page Heel Spurs and Heel Spur
560-562 Syndrome: page 622-623
Achilles Tendon Rupture: page 562- Tumors of the Heel: page 623-626
566 Tarsal Coalitions: page 626-629
Lateral Ankle Stabilization Sever's Disease: page 629-630
Procedures: page 566-571 Causes of Heel Pain (a summary):
Postoperative Care and Training page 630-631
Following Tendon Transfer: page Haglund's Deformity: page 631-632
Chapter 37: Laser Applications in
Chapter 31: Amputations Podiatric Surgery
General Surgical Technique: page Laser and Laser Physics:
634 page 741-751
Digital Amputations: page 634-635 Clinical Applications in Podiatric
MPJ Amputations: page 635 Surgery: page 751-775
Transmetatarsal Amputations: page
635-636 Chapter 38: Specific Radiological
Tarsometatarsal Amputations Pathology
(Lisfranc's): page 637 MRI Pathology: page 777-800
Midfoot Amputations (Chopart's):
page 638
Amputation of the Ankle (Syme's,
Boyd, Pirogoff): page 638-639
Below Knee Amputation: page 639-
640

Chapter 32: Biomechanics


Normal Values: page 641
Criteria of Normalcy in the Lower
Extremity: page 642-643
Adult Biomechanical Examination:
page 643-645
Common Structural Variations: page
646-648
Planes of Motion: page 648-649
Axes of Joint Motion: page 649-653
Angular and Axial Deformities in
Children: page 654-656
Anatomy of Gait: Activity of Muscles:
page 656-663
Observation of Gait: page 663-664
Subtalar Joint Measurements: page
664-666
Subtalar Joint Function: page 666-
667
Midtarsal Joint Function: page 667-
668

Chapter 33: Anatomy


Neuroanatomy: page 670-676
Angiology: page 676-680
Myology: page 680-695
Arthrology: page 695-704

Chapter 34: Soft Tissue Tumors


Soft Tissue Tumors: page 706-715

Chapter 35: Physical Medicine


and Rehabilitation
Physical Modalities and Diseases:
page 718-720
Therapeutic Modalities and
Procedures: page 720-730

Chapter 36: Arthroscopy


Ankle Arthroscopy: page 731-740
Chapter 1: Emergency
Medicine
Cardiopulmonary Resuscitation
Advanced Life Support
Office Emergencies
Medical Emergencies (Cardiac Dysrhythmias)
Summary of CPR
Other Medical Emergencies
Shock
Blood and Blood Components for Emergency Use
EMERGENCY MEDICINE
Cardiopulmonary Resuscitation
1. Rescue breathing: Adult victim
a. Check responsiveness: shake or tap gently and ask "ARE YOU OK ?"
b. If unresponsive yell for help
c. Open the airway: head tilt/chin-lift to open airway, and check for
obstruction. Remove if present
d. Check for breathing: if no breathing then give 4 quick breaths (Observe
chest rise)
e. Check for pulse (Carotid) for 5-10 seconds: if .pulse is present but there is
no breathing then
f. Start rescue breathing: inflate @1 breath every 5 seconds. Continue for 1
minute
g. Reassessment: check pulse and breathing, if breathing then stop CPR. If
no breathing but pulse present, then just continue ventilations. If no
breathing and no pulse, then begin CPR
h. Start chest compressions over sternum: using heel of hand with fingertips
off sternum and with elbows straight; compress 181 /2 to 2 inches @ 80 to
100/ minute
i. Provide proper ventilations: give 2 breaths after 15 compressions if working
alone or 1 breath to every 5 compressions when two rescuers are present

Note* Do not perform a precordial thump in an unwitnessed event

2. Rescue Breathing: Infant victim


a. Check responsiveness
b. If unresponsive call for help
c. Open airway: if obstructed then clear
d. Check for breathing: if no breathing then cover victim's mouth and nose
with rescuer's mouth and give 4 puffs of air (Observe chest rise)
e. Check for pulse (Brachial): if pulse present but no breathing
f. Start rescue breathing: inflate @ 1 breath every 3 seconds, continue for 1
minute
g. Reassessment: check pulse and breathing if victim is breathing then stop
CPR- if victim is not breathing and pulse present then continue ventilations-
if victim is not breathing and has no pulse then start chest compressions
i. Infant chest compressions should incorporate 2-3 fingers on the sternum
centered on an imaginary line between the nipples, compressing vertically
1/2 to 1 inch at a rate of 100/minute ventilations occurring after every 5
compressions

Note* Do not perform a precordial thump in an unwitnessed event


3. Rescue breathing: Child victim
a. All of the above holds true except that the rescuer feels for the carotid
pulse compressions occurring one finger width above the substernal notch
using the heel of the hand at a compression rate of 80-100/ minute-
maintain cycles of 5 compressions to every 1 ventilation

NOTE* Do not perform a precordial thump

Advanced Life Support


1. Advanced cardiac life support: includes the use of drugs, defibrillation, intubation
and military antishock trousers (MAST)- the basic protocol is as follows:
a. Begin basic CPR (Airway-Breathing-Circulation)
b. Determine circumstances (Past history-current medications) c. Begin IV and
intubate
d. Determine cardiac rhythms
e. Determine blood gases
f. Begin appropriate drug therapy. These drugs include:

i. Atropine:
 decreases vagal tone to increase heart rate
 used in sinus bradycardia/high degree AV block
 dosage- .5mg IV Q 15 minutes up to 2mg.

ii. Bretylium:
 used in V-fibrillation and V-tach when lidocaine and countershock fails
 dosage v-fib: 5-10gm/kg bolus Q 15 min to max 30mg/kg
 dosage v-tach: 5-10 mg/kg IV over 10 minutes, then 1 to 2 mg/min IV drip

iii. Calcium:
 should only be used to treat acute hyperkalemia, hypocalcemia, and
calcium channel blocker toxicity (there is no data showing its effectiveness
during CPR)
 increases cardiac contractility and excitability
 used in asystole
 dosage: calcium chloride 2-4 mg/kg Q 10 minutes (Approx 500 mg)
 used in pump failure
 dosage: 2.5-1 Omicrograms/kg/min

iv. Dopamine:
 alpha, beta, and delta agonist
 used to support cardiac output, BP and renal perfusion in shock
states
 dosage: start at 2-5 micrograms/kg/min, titrate to effect 20
micrograms/kg/min
 large dose has mostly alpha effect
v. Epinephrine:
 alpha and beta agonist, increases heart rate and contractility
 used in asystole, V-fibrillation, and cardiac arrest
 dosage: .5 to 1.0mg (5-10 ml of 1-10,000) IV Q 5 min

vi. Furosemide (Lasix):


 loop diuretic and vasodilator
 used for pulmonary edema and congestive heart failure
• dosage: starting dose is 0.5, mg/kg IV and total dose not to exceed 2.0 mg/kg IV

vii. Isoproterenol (Isuprel):


 pure beta agonist
 increases heart rate, contractility and consequently cardiac output
 used in asystole, symptomatic heart block and bradycardia
 dosage: 1 mg in 500 cc D5W to give a starting dose of 2 mcg/min, titrate to
effect (to achieve a heartrate of 60 beats/minute)

viii. Lidocaine (Xylocaine):


 decreases automaticity and raises v-fib threshold
 used to suppress PVC's, v-tach and v-fib
 gm/kg 50-100gm bolus, then 2-4mg/min IV drip

ix. Procainamide (Pronestyl):


 decreases cardiac excitability, decreases automaticity of ectopic pacemakers,
and slows conduction
 used to suppress ventricular ectopics when lidocaine fails
 dosage: 100mg IV with a rate of 20mg/min until dysrhythmia disappears or
hypotension ensues, QRS complex is widened by 50%, or total of 1 gm of
drug is injected

x. Propranolol (Inderal):
 beta blocker (to be used with caution in patients with COPD, diabetes and heart
failure
 used to control recurrent ventricular and atrial tachydysrhythmia
 dosage: 1 mg IV Q 5 minutes to 5 mg total (total dose not to exceed 0.1 mg/kg)
 must administer slowly

xi. Sodium Bicarbonate:


 used to counteract metabolic acidosis
 dosage is based on blood pH or empirically, 1 mEq/kg IV (50-100 mEq or 1-2 amps)

g. Cardioversion as necessary (Defibrillation) for ventricular fibrillation Procedure as


follows:
1. Use paste or pads on skin
ii. Charge defibrillator with SYNCHRONIZATION switch off
iii. Place paddles as directed on the handles: one on the right upper
sternum and the other on the left anterior axillary line
iv. Apply paddles with firm pressure (turn off oxygen)
v. Shout," clear", make sure no one is touching the victim vi. Press both paddle
switches simultaneously to fire the unit
vii. Repeat as necessary : Start at 200-300 joules- maximum output
400 joules

Office Emergencies
1. Syncope- Vasovagal reflex (Primary shock; fainting)
a. Defined as transient loss of consciousness due to sudden release of the
arterial vasomotor tone and temporary insufficiency of cerebral circulation
b. Causes are sudden extreme fear or pain or the effect of severe injury
c. Differential diagnosis: epilepsy, hyperventilation, hysteria , carotid sinus
syndrome, cardiac arrhythmia, drugs and orthostatic hypotension d. Signs
and symptoms: pallor, sweating, slow pulse, yawning and marked transient
hypotension
e. Treatment is supportive: recumbent position, take B.P., pulse, spirits of
ammonia, O2 and drugs (Atropine/Ephedrine) only if previous treatment fails

2. Local Anesthetic Toxicity


a. Causes are too much volume or too concentrated solution; extreme rapid
absorption
b. Reactions- rapid and delayed; with cerebral stimulation and/or depression;
respiratory stimulation; cardiac depression; hypotension; shock
c. Signs and Symptoms: apprehension , nausea, BP elevation, convulsions, perioral
tingling, or most seriously, post-ictal depression, respiratory depression,
hypotension respiratory and cardiac arrest
d. Treatment- mental changes/watch patient, respiratory depression/O2,
hypotension/ vasopressors i.e. Ephedrine 20 mg IM convulsions/Valium 5
mg IV, and CPR if necessary

Note* Ephedrine raises BP and causes tachycardia while Vasoxyl raises BP


without tachycardia (due to the alpha effect)

3. Anaphylactic Reactions
a. These are toxic reactions that occur in persons who are allergic by heredity
or who have become sensitized to a given drug or therapeutic agent after
previous administration. Respiratory obstruction is the cause of death
b. Reactions- anaphylactic shock, angioneurotic edema (swelling of soft tissues
of throat), asthma with acute bronchospasm, urticaria and pruritus
c. Signs and symptoms of anaphylactic shock - skin wheals, itching,
angioedema, laryngeal edema, bronchospasm (wheezing) dyspnea, cyanosis,
apnea, vomiting, hypotension, cardiorespiratory collapse and death
d. Treatment of anaphylactic shock (must be immediate)- .5cc epinephrine IV or
IM (children 0.01 mg/kg), tourniquet and .25cc epinephrine at injection site, O2 &
airway, Solu-cortef 200 mg IV and CPR if necessary
4. Allergic Reactions:
a. End organ response of the skin
b. Symptoms are hives, (urticaria), bronchial asthma, and G.I. upset c.
Treatment: 25-50 mg IM Benadryl- if severe then treat as if anaphylactic
reaction- if tongue swelling use epinephrine

5. Acute asthmatic attack:


a. This is an intermittent airway obstruction, which is reversible- can be
acute and severe leading to respiratory failure
b. Causes- allergies, irritants, infections, extreme cold, drugs and emotion
c. Signs and symptoms- recurrent attacks of wheezing dyspnea and cough
d. Treatment- reassurance, rest, O2, drug therapy (epinephrine .5cc 1:1000
subQ every 20 min up to 3 doses - if no relief then aminophylline 5-6
mg/kg over 20 minutes, hydrocortisone 100mg)

6. Seizures (major convulsions):


a. Defined as convulsive disorders characterized by abrupt transient
symptoms of motor sensory, psychic, or autonomic nature, frequently
associated with change in consciousness. Changes thought to be secondary
to sudden transient alterations in brain function associated with excessive
rapid electrical discharge in the gray matter.
b. Causes: idiopathic, pathological states (brain tumor, CVA, head trauma),
local anesthetic toxicity, and intoxications
c. Signs and symptoms in grand mal (major epilepsy)- aura, severe generalized
clonic, convulsive body movements followed by a period of flaccid coma,
then a period of sleep (post-ictal depression)- with status epilepticus:
recurrent and severe seizures with short or no intervals between seizures
d. Treatment- except in status epilepticus no specific treatment is necessary
except to protect patients from hurting themselves- in status epilepticus start
with Valium, then Dilantin, then phenobarbital prn

7. Hypertension:
a. Defined as persistent elevated BP above normal for the patient (borderline
hypertension BP>140/90)
b. Signs and symptoms- headache, convulsions, visual changes, with acute rise
in BP
c. Treatment- start supportive therapy and reduce BP with sublingual
nifedipine 10mg (Procardia) then send for medical evaluation

8. Insulin Shock (hypoglycemia):


a. Defined as low blood sugar that occurs when a diabetic who has taken
insulin, fails to ingest food or engages in too strenuous exercise
b. Treatment- if conscious and able to swallow then give orange juice- if
unconscious then give IV glucose 20 to 50 ml. of 50% solution

NOTE* If the diabetic patient is seen when unconscious, and if the diagnosis of coma
or insulin reaction is in doubt, give 50% glucose IV- this will overcome insulin
reaction but will not generally harm patient in diabetic acidosis

Medical Emergencies (Cardiac Dysrhythmias)


1. Myocardial Infarct (uncomplicated): Characterized on EKG by big Q waves
a. Monitor EKG
b.100% O2 with nasal cannula
c. Start IV lines with D5W
d. Sublingual nitroglycerin can help
e. Relieve pain with morphine sulfate
f. Do blood gases/pH/electrolytes
g. Consider use of prophylactic lidocaine

2. Asystole: Characterized by a flat line on the EKG


a. Use basic CPR, begin IV, intubate
b. Give epinephrine and bicarbonate
c. If ineffective give calcium chloride
d. If ineffective give atropine
e. If ineffective give isoproterenol'
f. If ineffective repeat steps b-e and as a last resort can give epinephrine
intracardially or using a transvenous or external pacemaker

3. Ventricular Fibrillation: The gravest of all arrhythmias characterized by


irregular and uncoordinated movements of the ventricles
a. Use basic CPR, begin IV's, intubate (if unconscious)
b. Use precordial thump only if witnessed arrest then defibrillate with 200-300
joules- repeat prn
c. If unwitnessed event do not use countershock: first start with epinephrine
and bicarbonate
d. If no response give epinephrine and bicarbonate
e. Defibrillate at 400 joules
f. Use lidocaine or procainamide
g. If no response use bretylium
h. After successful conversion use lidocaine drip

4. Ventricular Tachycardia:
a. Begin lidocaine
b. Use CPR if no pulse/O2/ IV's (and unconscious)
c. Use precordial thump if witnessed event
d. Consider cardioversion and procainamide or bretylium if lidocaine
ineffective

5. Third degree AV heart block:


a. Use atropine .5mg IV followed by isoproterenol prn b. Pacemaker
6. Premature ventricular contractions (PVC's): (frequent) May lead to V-ib (if
untreated)
a. Lidocaine 100mg IV bolus followed by lidocaine IV drip
Wolff-Parkinson-White syndrome:
a. Characterized by a short P-R interval and prolonged QRS time. There is a
40% incidence of episodes of paroxysmal tachycardia, atrial fibrillation and
atrial flutter, as well as the possibility of sudden death. Can occur in healthy
individuals
b. Treatment: Digitalis, quinidine, propranolol, artrial pacing

Summary of Cardiopulmonary Resuscitation


1. Establish the diagnosis: Apnea, no pulse, absence of heart sounds,
absence of responsiveness, ashen gray color
2. Summon help: Time is critical; you only have 4-6 minutes to reestablish
ventilation
3. Do not thump the patient's chest (not part of CPR any more)
4. Check for absence of breathing first. Displace the mandible forward, and
clear the airway manually, then give 2 rapid respirations
5. Place support under patient's back and start mouth-to-mouth breathing and
external cardiac compression (5:1 with two rescuer sequence or 15:2 solo).
Depress the sternum 4-5 cm (1.5-2 inches)
6. Insert ET tube to ventilate with 100% oxygen (only by experienced
personnel)
7. Start IV infusion by needle or cutdown. Administer epinephrine and sodium
bicarbonate and repeat bicarbonate injections until arterial blood gases and
pH results are known
8. Monitor EKG
a. If V-fib: Closed chest compression, epinephrine, sodium bicarbonate,
defibrillate (if not effective repeat countershock)
9. Inject epinephrine, and if defibrillation not successful, repeat
10. Following restoration of heart function inject lidocaine for excessive
ventricular irritability
11. If asystole is present, heart function may resume following myocardial
oxygenation by ventilation and external cardiac compression
12. If asystole persists inject epinephrine and sodium bicarbonate., calcium
chloride and isoproterenol prn
13. If electromechanical dissociation is present take therapeutic steps
as with asystole
14. Corticosteroids may be used to decrease cerebral edema
15. Postcardiac arrest therapy includes corticosteroids, diuretics, hypothermia,
and hyperventilation. Monitor arterial blood gases, BP, EKG, CVP (central
venous pressure), urine, electrolytes, and chest x-ray

Other Medical Emergencies


1. Narcotics Overdose:
a. Give Naloxone (0.2-0.4 mg IV or IM/ in children .01 mg/kg: repeat Q 5
min)

2. Acetaminophen Overdose (APAP):


a. The major toxic effect is centrilobular necrosis of the liver with toxicity likely
to occur after a minimum acute ingestion of about 10g (or 30 acetaminophen
tablets)
b. Perform plasma APAP levels- treatment based accordingly
c. Emesis with ipecac or gastric lavage
d. Administer ACETYLCYSTEINE if elevated APAP plasma level

3. Salicylate Overdose: (the most common cause of fatal drug ingestion in the
pediatric age group) (Reyes syndrome)
a. There is a relationship of signs and symptoms to the amount of ASA ingested
i. Mild: >100mg/kg: hyperventilation, lethargy, tinnitus
ii. Moderate: 200-300mg/kg: hyperventilation, vomiting, sweating, vertigo,
hallucinations
iii. Severe: 300-400mg/kg: hyperventilation, seizures, coma, shock
b. Treatment begins with emesis with ipecac then gastric lavage
c. Alkalinize the plasma ( helps promote secretion of salicylates) with push
doses of bicarbonate
d. If shock present start IV with Ringer's lactate e. Start dialysis if renal failure

4. Poisoning:
a. Each type of poisoning is treated differently a. Begin basic CPR if necessary
b. Determine ingested substance and give antidote if available

NOTE* It is better to call hotline first for specific directions in the treatment of
specific ingested agents

c. If unconscious- protect airway with ET tube, lavage with 28 French NG tube,


use 300m1 normal saline for adults (activated charcoal can be added)
d. If conscious- use syrup of ipecac to induce vomiting, ambulate patient, and
give large quantities of water

5. Pulmonary Embolism: Complete or partial blockage of the pulmonary vessels


from an intravascular clot originating elsewhere in the body, usually the deep
veins of the lower extremity
a. Signs and symptoms can include chest pain, dyspnea, cough, sweats,
syncope, respirations >16/min, pulse >100/min, temp>37.8 degrees C,
phlebitis and edema
b. EKG can show ST-T wave changes
c. Presumptive diagnosis made by lung scan- definitive diagnosis made by
pulmonary angiogram
d. Treat with bedrest, then Heparin 5000 units as loading dose followed by
continuous infusion (25,000 units added to 500 ml D5W administered via
IVAC)
e. Prior to Heparinization do PTT- during treatment adjust to keep PTT 2X
normal
f. If thrombophlebitis present treat with elevation and moist heat (may need
antibiotics)

6. Hypertensive Emergencies:
 Hypertensive encephalopathy
 Malignant Hypertension
 Accelerated Hypertension
 Hypertensive Crisis
a. Diagnosis of Hypertensive encephalopathy or accelerated malignant
hypertension is a clinical one and demands immediate aggressive therapy to
lower BP
b. Treatment initially should be Diazoxide (Hyperstat) 300mg by rapid IV
bolus or can give hydralazine (should give Furosemide simultaneously- prevents
fluid retention)

7. Malignant Hyperthermia (also see Chapter Anesthesia, Section: Other Medical


Complications of Anesthesia):

Defined as a catastrophic reaction to general anesthesia An inherited trait


Incidence of 1 in 20,000

a. With exposure to inhaled anesthetic agent the patient exhibits fasciculations


and increased muscle tone, with jaw clenching during the
induction of anesthesia a typical early sign and body muscles becoming
rigid and excessive body heat produced
b. Anesthesia must be discontinued
c. Patient must be cooled
d. INTRAVENOUS DANTROLENE SODIUM HAS A THERAPEUTIC EFFECT
e. If suspicious of malignant hyperthermia pre-op do CPK LEVEL- THIS
LEVEL IS ELEVATED IN 79% OF THE PATIENTS WITH MALIGNANT
HYPERTHERMIA
f. Early signs:
i. Tachycardia
ii. Tachypnea
iii. Unstable BP
iv. Arrhythmias
v. Dark blood in the surgical field
vi. Cyanotic mottling of the skin
vii. Profuse sweating
viii. Fever
ix. Fasciculations
g. Suggested treatment regimen:
i. Stop anesthesia
ii. Hyperventilate with 100% oxygen (8-10 liters/minute)
iii. Start Dantrolene sodium IV as soon as possible (starting dose 1
mg/kg up to a maximum cumulative dose of 10 mg/kg by rapid infusion)
iv. Start Procainamide IV if required for arrhythmias
v. Initiate cooling
-IV iced saline (not Ringer's)
 surface cooling with ice and hypothermia blanket
 Lavage of stomach, bladder and rectum
vi. Correct acidosis and hyperkalemia with sodium bicarbonate
vii. Monitor EKG, temp, urinary output, electrolytes, arterial pressure and blood
gases, pH, and electrolytes
viii. Maintain urine output of at least 2 ml/kg/hr: administer Mannitol and
Furosemide (if necessary)
ix. If necessary administer Insulin to provide energy to the cells and normalize
the pH
x. Administer oral Dantrolene for 1-3 days after the crisis

NOTE* Avoid amide local anesthetics if a patient has a history of malignant


hyperthermia reaction
NOTE* Malignant hyperthermia is most frequently seen when halothane and
succinylcholine are used together
Shock
The mechanism of shock is poorly understood, however, this phenomenon
results in inadequate tissue perfusion with accompanying cellular injury and
metabolic disturbances. Shock cannot be defined but it can be classified by
etiologic means
1. General clinical presentation:
a. Tachycardia
b. Hypotension
c. Low tension pulse (thready pulse)
d. Collapsed superficial peripheral veins
e. Oliguria
f. Hypothermia
g. Metabolic acidosis

2. Etiology:
a. Hypovolemic: Caused by a reduction in circulating blood as a result of
traumatic injury, GI bleed, crush injuries, burns, massive diarrhea, and
peritonitis
b. Septic: Caused by infections that produce an endotoxic or exotoxic
reaction. Most common gram (-)'s are E. coli, Proteus group, Pseudomonas,
Klebsiella and meningococci. Less often involved are gram (+)'s such as
staphylococci, streptococci, and clostridia
c. Neurogenic: Severe injury to the spinal cord or brain can cause a loss in
vasomotor tone resulting in vasodilation and hypotension from the loss of
peripheral vascular resistance. Also psychogenic factors such as the sight of
blood or surgery can produce shock
d. Cardiogenic: Produced by hypotension arising from inadequate cardiac
output as a result of serious arrhythmias, tamponade, Ml, CHF, and pulmonary
embolism
e. Metabolic: Caused by alterations in the fluid electrolyte balance as a result
of systemic diseases such as diabetic acidosis, renal failure, or chronic
respiratory diseases
f. Anaphylactic: Occurs following the injection of heterologous sera, penicillin
and other medications

3. Treatment of shock:
a. Assess the physical status of the patient
b. Lie the patient down and keep him/her warm
c. Maintain airway administering oxygen at 8-10 liters/minute. If patient
unable to breath on their own use Ambu bag (use- CPR if necessary)
d. IV fluid replacement to avoid dehydration. Do not use lactate solutions

NOTE* Expanding the intravascular volume is the primary goal in the initial
treatment of hypovolemic shock

e. Vasopressor drugs can be used providing there is sufficient blood volume to be


effective (the mechanism and etiology of shock dictates the specific drug)
f. Lab studies should be instituted such as pH, pO2, pCO2, serum electrolytes, BUN,
lactic and pyruvic acids, and hematocrit
g. Measure the urine volume (normally it should be above 30 ml/hour
persistent oliguria below 25 ml/hr for more than 2 hours may cause renal cell
necrosis)
g. If infection is suspected cultures should be performed, and appropriate antibiotics
initiated
h. For allergic/anaphylactic reactions treatment as mentioned above should
be instituted
NOTE* Patients with a history or suspicion of penicillin allergy may be tested as
follows: Dilute penicillin G to a concentration of 1,000 units per ml and place
1 drop on a skin scratch on the forearm. If the test is positive a wheal will be
seen within 15-20 minutes. If the test is negative, inject a small amount of
this solution intradermally to double check. This indicates the decreased
probability of anaphylactic response but does not totally rule out an allergic
state. Keep a "shock kit" immediately available because even a test can
initiate anaphylactic shock. Also the risk of testing is that patients may
become iatrogenically sensitized to future doses

Blood and Blood Components for Emergency


Use (also see Ch. 11 Fluid Management)
1. Red Blood Cells
a. Description:
i. Available as "packed RBCs" of 250 cc, split units of 125 cc, or quad packs
for newborns
ii. Anticoagulants are used to prevent clotting and a small residual amount of
plasma is present
iii. Units can be prepared and combined with special filters to prevent febrile
reactions (leukocyte poor)
iv. Unit can be washed to prevent allergic reactions (washed RBC's)
v. One unit can raise the hematocrit by 3% or Hb by 1 gm.
b. Compatibility:
i. The unit must be ABO compatible, but Rh compatibility not required but preferred
ii. Rh positive blood can be given to patients especially those over 50 years of
age, who are expected to use multiple units (10% will develop Rh antibodies 3-
4 months later, and by this time the transfused Rh+ cells have been
cleared)
iii. When Rh- units are in short supply they should be saved for women of
child bearing age
c. Alternatives:
i. Autologous transfusions: patients can donate up to 3 units of blood prior to
surgery and have these units available if subsequent bleeding occurs during the
procedure (the safest)
ii. Directed transfusions: Patients can elect to have friends and relatives donate
blood for upcoming surgery
iii. Perioperative cell salvage: Patients may elect in certain operative procedures to
have blood lost during surgery, recollected, filtered and transfused (sterile
orthopedic procedures and abdominal aortic aneurysms)
c. Indications for RBC's:
i. Hypovolemia due to acute blood loss and associated with one or more of the
following:
 Acute bleeding with an actual or anticipated blood loss of 750 ml or more
 Systolic blood pressure <90 mm
 Tachycardia (pulse > 100)
 Hct < 30% and documentation of a fall of 5% or more within 24 hours or
10% or more within one week
 Central venous pressure < 3 cm/H20
ii. Chronic anemia:
 Uncomplicated: Hct < 24 or Hgb < 8 mg (and not due to acute blood loss)
with anemia syndrome
 Complicated: Hct < 30% or Hgb < 10% with complications affecting
oxygenation (cardiac or respiratory insufficiency)
 Anesthesia pre-op: Hct < 30% or Hgb < 10mg
iii. Hemodialysis

NOTE* RBC's must be used within 4 hours after removal from the refrigerator
and must return within 20 minutes to the Blood Bank if not used. Warming can
result in bacterial proliferation if allowed to warm to room temperature before
returning to refrigeration

d. Adverse reactions:
i. Infectious reactions:
 AIDS: Risk is 1:20,000 to 1:40,000 for each unit transfused
 Hepatitis (B C): less than 1
 MV
ii. Noninfectious reactions
 Febrile: fever reaction most common. This reaction involving circulating
antibodies in the recipient which react to HLA antigens in infused
granulocytes
 Allergic: associated with circulating serum antibodies within the
recipient to infused immunoglobulins within the small amount of residual
plasma of the red cell unit (hives, serum sickness, anaphylaxis)
 Hemolytic: is a result of circulating naturally occurring antibodies in the
recipient to antigens on the RBC's causing cell lysis
 Graft vs. host disease: engraftment and multiplication of donor blood cells
in an immunosuppressed recipient are possible, and here, immunocompetent
lymphocytes become engrafted and cannot be rejected

2. Platelets:
a. Description: Are a concentrate separated from a single donor by
plasmapheresis from whole blood containing 5.5 x 1011 platelets in 200
300 cc of plasma and anticoagulant, and can be expected to raise the adult
platelets count by 60-80,000 unless platelet antibodies are present
b. Compatibility: ABO compatibility is preferred, but in emergencies or short supply
any ABO group can be used (Rh is not a factor).
c. Alternatives: Random donor platelets are obtained from a single unit of
whole blood and contain 1 /10 the number of platelets in 30-50 cc, and 6-10
units are standard suggested therapy
d. Indications:
L Prophylaxis:
 Platelet count < 20,000/mm3 or anticipated drop below 20,000 in the
 next 24 hours
 Platelet count < 80,000 with surgery anticipated or in the acute post-op
period
 A platelet function defect with surgery anticipated or in the acute post-op
period
i. Bleeding:
 latelet count < 20,000/mm3
 latelet function defect (known or suspected)
e. Adverse reactions: Same risks as RBC's

3. Cryoprecipitate:
a. Description: Prepared by thawing fresh frozen plasma at 4°C and recovering
the cold precipitate. Each bag of 'Cryo' contains 90 or more Factor VIII units
and at least 150 mg of fibrinogen in less than 15 ml of plasma
b. Compatibility: ABO compatibility is preferred but not required in emergency
situations
c. Alternatives: Fresh frozen plasma can be used if there are associated
deficiencies of individual coagulation factors, massive blood transfusion, or
when cryoprecipitate is in short supply
d. Indications:
i. Von Willebrand's Disease
ii. Hypofibrinogenemia associated with bleeding or surgery (perioperative)
iii. Dysfibrinogenemia associated with bleeding or surgery
iv. Uremia associated with bleeding
v. Factor XIII deficiency
e. Adverse reactions: Same as with RBC's

4. Fresh Frozen Plasma:


a. Description: Is the anticoagulated clear liquid portion of blood that is
separated and frozen within a few hours of Whole Blood Collection. A unit of
FFP contains about 200 units of Factor VIII as well as other coagulation
factors. Volume is 250 cc.
b. Compatibility: ABO compatibility required
c. Alternatives:
i. Specific coagulation factors (cryoprecipitate for low fibrinogen or von
Willebrand's disease)
ii. Crystalloid or albumin is the preferred product for volume expansion d.
Indications:
1. Replacement of isolated deficiencies (Factor II, V, VII, IX, XI) ii. Reversal of Warfarin
effect
iii. Massive blood transfusion (greater than 1 blood volume within several
hours)
iv. Antithrombin III Deficiency
v. Thrombotic thrombocytopenia purpura
e. Adverse reactions: Same as with RBC's
NOTE* Allergic reaction, dermal and anaphylaxis can be severe. Treatment
includes Benadryl 50 mg 1M STAT repeated Q 10-20 minutes prn. Steroids
and fluids may be necessary in severe reactions

5. Albumin
a. Description: A solution containing the albumin component of human
blood, which can effect immediate and prolonged restoration of circulating
blood volume by causing a shift of fluid from the interstitial spaces into the
circulation and slightly increasing the concentration. of plasma proteins
b. Action: 25% albumin will draw approximately 3.5 times its volume of
additional fluid into the circulation within 15 minutes, and provides a means
of replacing human plasma proteins
c. Indications:
i. Plasma or blood volume deficit secondary to surgery, hemorrhage, burns, or
trauma: to support BP by expanding the plasma volume
ii. Hemolytic disease in the newborn
iii. Hypovolemic shock: to restore blood volume in increase CO
iv. Hemodialysis: for the treatment of shock or hypotension when the patient
is fluid overloaded
v. Acute or chronic liver disease
d. Contraindications: History of hypersensitivity or severe anemia or CHF
e. Precautions: Solutions containing 5% albumin are usually indicated for
hypovolemic patients, 25% solutions should be used when fluid and Na+
intake must be minimized (cerebral edema and pediatric patients)
f. Adverse reactions: Rare
Chapter 2: Anesthesia
Anesthesia Classifications
General Anesthesia
Intravenous Anesthesia
Local Anesthesia
Special Cases for Lowering the Maximum Allowable Dose
Pediatric Anesthesia
Lumbar Epidural and Caudal Anesthesia General
Complications of Anesthesia Complications of Endotrachial
Intubation Nerve Injury During Anesthesia
Other Medical Complications From Anesthesia
ANESTHESIA
Anesthesia Classifications

The American Society of Anesthesiologists Physical Status Measure ( A classification


system for patients undergoing surgery)
Class 1 Normal and Healthy - no known diseases
Class 2 Mild Systemic Disease i.e. presence of essential hypertension or mild type
II diabetes
Class 3 Severe Systemic Disease That Is Not Incapacitating i.e. severe diabetes, type I
with vascular complications
Class 4 Incapacitating Systemic Disease That Is A Threat To Life i.e. advanced
cardiac, renal, pulmonary, hepatic or endocrine insufficiency
Class 5 Moribund Patient Who Is Not Expected To Live With Or Without Surgery
EMERGENCY OPERATION- any patient in one of the above classses who is operated
on as an emergency (Letter E is placed next to the classification)

General Anesthesia
A reversible state of unconsciousness produced by anesthetic agents, with loss of
sensation of pain over the whole body. The order of descending depression of the CNS
during anesthesia is: cortical and psychic centers, basal ganglia and cerebellum,
medullary centers, and spinal cord
1. Inhalation agents:
a. Volatile liquids:
i. Chloroform (no longer in use)
 Advantages: Rapid induction and recovery, nonflammable, good muscle relaxation
 Disadvantages: Myocardial depressant, hepatotoxic
ii. Diethyl ether (no longer in use)
 Advantages: Reliable signs of anesthesia depth, respiration stimulated,
bronchodilator, circulation not depressed, good muscle relaxation, relatively safe
and nontoxic (has lowest death rate following its use)
 Disadvantages: Prolonged induction and recovery, irritating to mucous
membranes of upper airway, dangerous in patients with full stomachs (emetic),
flammable, and explosive
iii. Halothane (Fuothane®)
 Advantages: Rapid smooth induction and recovery, pleasant smell, nonirritating
(no secretions), bronchodilator, nonemetic, nonflammable
 Disadvantages: Myocardial depressant, may trigger malignant hyperthermia
reaction, arrhythmia-producing drug, sensitizes the myocardium to the action of
catecholamines, possibly toxic to the liver, postoperative shivering
iv. Methoxyflurane (Penthrane(r): It is the most potent and least volatile anesthetic
(no longer in use)
 Advantages: Great margin of safety, good muscle relaxant, not sensitive to
catecholamines, nonflammable
 Disadvantages: Prolonged induction of anesthesia and prolonged recovery,
nephrotoxic
v. Enflurane (Ethraner)
 Advantages: Pleasant smell, rapid induction and recovery, nonirritating (no
secretions), bronchodilator, maintains stability of the cardiovascular system,
nonemetic, compatible with epinephrine
 Disadvantages: Myocardial depressant, smooth muscle relaxant, increase
hypertension with increase depth of anesthesia, CNS irritant, possible
hepatotoxicity

vi. Isoflurane (Forane®): Newest inhalation agent


 Advantages: Rapid induction and recovery, nonirritating, bronchodilator, excellent
muscle relaxation, maintains stable cardiac rhythm, compatible with epinephrine,
nonemetic, nonflammable
 Disadvantages: Depresses the cardiovascular system, shivering postoperatively,
possible acute or delayed liver injury (less likely than with Ethrane or Halothane)

b. Gasseous anesthetic agents


i. Nitrous oxide: The least potent of the anesthetic gases, and the most frequently
used inhaled anesthetic. In the absence of hypoxia, there is little effect on the heart
rate, myocardial contractility, respiration, blood pressure, liver, kidney or
metabolism. Oxygen 100% must be given at the termination of the surgery to prevent
diffusion hypoxia
 Advantages: Rapid induction of anesthesia and emergence, does not sensitize the
myocardium to epinephrine, nonirritating, intense analgesia, nonemetic
 Disadvantages: No muscular relaxation, possible bone marrow depression and
fatal agranulocytosis from prolonged administration or exposure, and increased
risk of spontaneous abortion with prolonged use
ii. Cyclopropane (no longer in use)

2. Preanesthetic or suplementary agents


a. Sedatives
i. Pentobarbital (Nembutal®) and secobarbital (Seconal®): Used before surgery to
relieve anxiety and tension (cerebral cortex depression). Have a short hypnotic effect
and pronounced sedative action. Are used as an inducing agent and have no
analgesic component
ii. Phenobarbital
iii. Chloral hydrate: One of the oldest and best hypnotics, and a very good alternative
to barbiturates in children and the elderly (adult dose is .5-1 gm PO). Excreted by the
lung.
iv. Diazepam (Valium®): Produces a satisfactory sedative and amnesic effect. It is
indicated to prevent and treat convulsions
v. Hydroxazine (Vistaril®): Has sedative antihistaminic, antiemetic, and
bronchodilating properties, but used primarily for its sedative properties.. Excellent
premedication in patients with a history of bronchial asthma
vi. Droperidol-fentanyl (Innovar(r): A 50-1 mixture of droperidol and fentanyl.
Produces effects that are a combination of both drugs
vii. Droperidol (Inapsine®): The main effect is tranquility and peripheral vasodilation,
can cause dysphoria, good antiemetic, and when infused IV produces sleepiness and
mental detachment
b. Narcotic analgesics:
1. Fentanyl (Sublimaze®): Produces depression of ventilation which is short in
duration. Reversed by narcotic antagonist (Naloxone®). Can also produce muscle
rigidity in large doses
ii. Morphine: The standard analgesic narcotic drug for relief of severe pain. It
depresses the CNS, reduces GI motility, constricts the bronchi (due to histamine
release), and lowers the metabolic rate. It has strong sedative and analgesic
properties
iii. Meperedine (Demerol®): Has analgesic, sedative, and spasmolytic properties, and
in conjunction with barbiturates induces amnesia. It can cause tachycardia and is
contraindicated in patients with atrial flutter (and anything that causes increased
intracranial pressure just as with any other narcotic)

NOTE* The adverse side effects from the narcotic analgesics include respiratory
depression, emesis, physical dependence. Asthmatics react poorly to morphine
only due to histamine release (smooth muscle constriction)

c. Tranquilizers:
i. Phenothiazines: Are used for a preanesthetic medication because of their sedative,
antiemetic, antihistaminic, and temperature regulating effects. May produce
postoperative hypertension and lethargy. When given with narcotic analgesics,
increases respiratory depression
 horazine® (15-25 mg)
 ompazine® (5-10 mg)
 henergan® (25-50 mg)

d. Belladonna compounds:
i. Atropine: Decreases secretions and is the drug of choice to reduce bronchial and
cardiac effects of parasympathetic origin. It increases the heart rate by blocking the
vagus nerve, and stimulates the cerebral cortex. Atropine is superior to scopolamine
as a vagolytic agent, therefore, can prevent severe bradycardia and asystole in the
presence of vagotonic agents (halothane). Atropine and scopolamine are potent
bronchodilators. Patients allergic to atropine can be given scopolamine + benadryl
ii. Scopolamine: An effective drug for psychic sedation and amnesia. The drying effect
is better than atropine
iii. Glycopyrrolate (Robinol)

NOTE* Diprivan is a new sedative/hypnotic (used with Versed and Fentanyl for
balanced anesthesia)

3. Stages of Anesthesia
a. Stage 1: Analgesia (characterized by variable degrees of analgesia and amnesia)
i. Plane 1- Preanalgesia (normal memory and sensation)
ii. Plane 2- Partial analgesia and amnesia
iii. Plane 3-Total analgesia and amnesia
b. Stage 2: Delirium (extends from the loss of consciousness until the beginning of
surgical anesthesia) (excitement and voluntary activity marked)
i. Unconsciousness, irregular breathing pupils dilated
c. Stage 3: Surgical anesthesia (4 planes)
i. Plane 1 (sleep)-Rhythmical breathing, eyeball centrally fixed, faint lid reflex
ii. Plane 2 (sensory loss)-Pupils slightly dilated, pulse and blood pressure normal
iii. Plane 3 (muscle tone loss)-lntercostal paralysis begins, increased pulse rate &
decreased BP
iv. Plane 4 (intercostal paralysis)-Provides cessation of all respiratory effort and
requires artificial ventilation for life support
d. Stage 4: Medullary paralysis
i. Plane 1: reversible respiratory failure
ii. Plane 2: irreversible cardiovascular collapse

Intravenous Anesthesia
1. Ultrashort-acting barbiturates: In sufficient amounts these can provide all the
anesthetic stages (may produce serious cardiovascular depression) for short minor
procedures that do not require muscle relaxation
b. Neuroleptoanalgesia: A neuroleptic drug (tranquilizer) plus a narcotic analgesic,
when administered together produce the following psychophysiologic state:
somnolence without total unconsciousness, psychological indifference to the
environment, no voluntary movements, analgesia, and satisfactory amnesia
c. Neuroleptoanesthesia: Combination of nitrous oxide, droperidol, fentanyl and
muscle relaxants (a good choice for patients with little cardiac reserve)
d. Dissociative anesthesia (Ketamine®): Produces a state where the patient becomes
mentally dissociated from the environment

Local Anesthetics
Function in such a way as to prevent sodium migration through the nerve membrane
which, therefore prevents depolarization of the nerve with inhibition of nerve
conduction
1. Chemical Classifications
a. Esters of para-aminobenzoic acids
i.. Procaine (Novocaine): most toxic, and is considered the standard in comparing the
potency and toxicity of other local anesthetics used for injections
ii. Chlorprocaine (Nesacaine®): least toxic, and rapid plasma hydrolysis by pseudo-
cholinesterase
b. Esters of Benzoic Acid
i. Hexylcaine (Cyclaine®)
ii. Tetracaine (Pontocaine): longest duration

NOTE* Esters are hydrolized by pseudocholinesterase in the plasma


Have a high potential for allergenicity due to PABA moiety

c. Amides
i. Lidocaine (Xylocaine®): shortest duration & fastest action
ii. Mepivicaine (Carbocaine®): do not use in presence of renal disease
iii. Bupivicaine (Marcaine®): longest duration, least placental transfer, should not be
used in children under the age of 12 years old (package insert), and greatest cardiac
toxicity if given IV
iv. Etidocaine (Duranest®): four time more potent than lidocaine, but only twice as
toxic

NOTE* Amides are hydrolized in the liver. There is no cross sensitivity between
amides and esters- can be substituted in case of allergy. Should use 1 /2 dose in
elderly, debilitated patients, and patients with hepatic disease. Pain 8 temperature
lost first following nerve block, with loss of touch & motor function later. Injection into
an acidic area (infection) converts the anesthetic chemically and does not allow for
penetration into the cell membrane, and lessens its effectiveness

2. Vasoconstrictors (Epinephrine)
a. Advantages
i. Reduces the vascularity locally at the site of the injection (due to
vasoconstriction)
ii. Reduces the absorption rate of the local anesthetic
iii. Permits a higher allowable single dose dose of local anesthetic to be used
iv. Increases duration of action of the block
b. Disadvantages
i. Use cautiously in patients with hyperparathyroidism, arteriosclerotic
cardiovascular disease, hypertension, and peripheral vascular disease
ii. Creates vasospasm in the end arterioles which could lead to tissue necrosis,
so should be diluted in the digits to 1:200,000-1:400,000 or not used
iii. Can create reactive hyperthermic reaction
iv. Should be avoided in patients receiving Halothane (since Halothane
sensitizes the myocardium in the presence of exogenously administered
catecholamines)

3. Hyaluronidase (Wydase)
a. Permits more rapid spread of solutions into the tissues, to facilitate regional block
anesthesia.
b. There is increased incidence of toxic reactions caused by local anesthetic drugs
when hyaluronidase is used
c. Reduces the duration of action when used with local anesthetics for nerve blocks

4. Regional Nerve Blocks


a. Advantages
i. Causes minimal interference with such preexisting diseases as diabetes, renal
failure, or heart conditions
ii. Eliminates the risk of pulmonary aspiration during induction of general anesthesia
iii. Requires little postoperative nursing
b. Ankle Block
i. Saphenous nerve: the only nerve at the ankle that comes from the femoral nerve,
lies medial to the greater saphenous vein at the ankle
ii. Posterior Tibia] nerve: branch of sciatic nerve, lies in 3rd compartment of lacinate
lig.
iii. Sural nerve: made up from branches of the tibial and common peroneal nerves
iv. Superficial Peroneal nerve: becomes superficial 7-8cm above the ant-lat ankle
v. Deep Peroneal nerve: lies between the EHL and the anterior tibial
5. Maximum Allowable Single Dose in Normal Adults:
a. Novocaine® (1-2%): 750mg plain 1000mg w/epinephrine
b. Pontocaine® (0.1-0.25%): 75mg plain 100mg w/epi
c. Xylocaine® (1-2%): 300mg plain 500mg w/epi
d. Carbocaine® (1-2%): 500mg plain
e. Marcaine® (0.25-0.75%): 175mg plain 225mg w/epi

NOTE* There is no advantage in using a higher % solution- there is no stronger or


longer anesthesia, therefore, with a lower % solution you can inject more volume.
One must know how to convert % solutions to mg/cc (.25% Marcaine= 2.5mg/cc,
0.5%= 5mg/ cc 1 % Xylocaine= 10mg/cc, 2%= 20mg/cc)

6. Complications due to local anesthetics: See Chapter, Medical Emergencies


a. Systemic reactions are associated with high blood levels which ordinarily result
from overdoses, rapid systemic absorption, or inadvertent I.V. administration. The
adverse reactions mainly effect the heart, circulation, respiration, and CNS
i. Effects on the heart and vessels: Direct myocardial depressant , hypertension,
bradycardia, thready pulse, pallor, clammy skin, sweating, cardiac arrhythmias
possibly leading to cardiac arrest
ii. Effects on medullary centers: depressed respiration, apnea, vascular collapse
iii. Effects on the CNS: Nausea, emesis, talkativeness, euphoria, perioral tingling,
restlessness, dizziness, anxiety, excitement, and disorientation. This can be followed
by muscle twitching, convulsions, coma, respiratory failure and heart failure
b. Vasovagal Reflex (Syncope)
c. Anaphylactic Reaction
d. Allergic Reaction (mostly due to esters due to PABA moiety)
e. Reactions. due to epinephrine
f. Local reactions : Skin slough, swelling, abcess, ulceration

NOTE* Therapy for these reactions includes:


a. For convulsions: Valium®, ultrashort-acting barbiturates and artificial ventilation
b. For respiratory depression: Oxygen and artificial ventilation and control of airway
c. For cardiovascular collapse: Vasopressors, IV fluids, and CPR
NOTE* When an allergy is suspected (but unknown) then an amide (frequently
lidocaine) should be chosen. If the patient has a history to paraben sensitivity,
preparations without araben should be tested (single dose vials). Preparations
without epinephrine should be used because it may mask a positive skin test

Special Cases For Lowering Maximum allowable Dose


1. Debilitated geriatric patient: 1 /2 or less of the adult dose
2. Pediatric Patient:
i.. Clark's Rule: used for children older than 1 year

Weight of child in pounds X The adult dose= ADJUSTED DOSE 150

ii. Fried's Rule: used for infants

Age in months X The adult dose= ADJUSTED DOSE 15

iii. Cowling's Rule: Age (next birthday) = Percent Adult Dose 24

Pediatric Anesthesia
1. Preoperative medications: Given up to 1 hour prior to surgery, to decrease
anxiety and to calm the child as well as dry secretions and decrease vagal
stimulation.
a. Sedative/hypnotics: Barbiturates, Chloral hydrate
b. Anticholinergic agents: Scopolamine, atropine
c. Narcotics: Meperidine, morphine

2. Anesthesia:
a. Create a warm environment during anesthesia as children have poor
autothermoregulation mechanisms (inability to shiver)

NOTE* The infant and child lose much of their ability to maintain normal body
temperature during and after anesthesia and their temperature fluctuates with that
of the environment. In most cases, unless vigorous attempts are made to conserve
body heat, the child may become cold and even cyanotic, especially after 2 hours of
surgery. Hypothermia leads to depressed respiration and hypoxia follows. This
predisposes the child to arrhythmias and V-fibrillation, and is the most common
cause of cardiac arrest and shock in the very young

b. Inhalation agents: Halothane and nitrous oxide with .neuromuscular blockade


remain the principle agents (Enflurane® and lsoflurane® are now being used
frequently). The margin of safety of volatile agents is very low, and deep levels of
inhalation anesthesia for intubation of the infant can be quite hazardous
c. Fetanyl in conjunction with halothane-nitrous oxide induction is good for short
surgical procedures. This regimen reduces inhalation requirements, intraoperative
movement, coughing, and laryngospasm while' producing postoperative analgesia
and shortened discharge times. The halothane dosage is then gradually reduced as
the narcotic effects are noted

NOTE Fetanyl is given 1 microgram per kg IV 20 minutes prior to the end of the
surgery (must be administered cautiously to premature infants)

d. Succinylcholine: A muscle relaxant used to quickly establish an airway especially


when regurgitation and aspiration pneumonitis is a real risk. Its downside is:
masseter spasm when used in conjunction with halothane (difficult to differentiate
from malignant hyperthermia), rhabdomyolysis, and cardiac dysrhythmias

Lumbar Epidural and Caudal Anesthesia


Lumbar epidural anesthesia is accomplished by injecting the local anesthetic solution
into the epidural space of the lumbar area of the vertebral canal. Entrance to the
epidural space is usually made at or below the level of the second lumbar vertebrae

1. Indications:
a. Lower extremity surgery when a general anesthetic may be risky for the patient
due to a preexisting medical -problem (i.e. asthma, rheumatoid arthritis affecting the
cervical spine, bronchitis, or emphysema, etc.) b. Patients who are not suitable
candidates for muscle relaxants (myasthenia gravis)

2. Contraindications:
a. Severe hemorrhage or shock
b. Local infection at the proposed puncture site
c. Septicemia
d. Preexisting neurologic disease
e. Extremes of age
f. Chronic backache or preoperative headache
g. Hypotension or marked hypertension

3. Advantages of epidural anesthesia over spinal anesthesia:


a. Allows segmental anesthesia
b. Postoperative headache does not occur
c. Hypotension is less likely
d. Can be maintained 1-2 days into the postoperative period as a useful method for
relief of pain
e. Do not have to remain In bed as long as with spinal anesthesia, therefore, can be
used in outpatient surgery

4. Advantages of spinal anesthesia over epidural anesthesia:


a. Less local anesthetic drug is needed than with epidural
b. Less time is needed to achieve an adequate block than with epidural
c. The level of anesthesia is more predictable d. Easier to perform

5. Anatomy of the epidural space:


a. The spinal cord is located within the spinal canal and is enveloped by the
meninges, the dura being the outermost. The spinal cord seldom extends below the
L1 vertebrae but occasionally extends to the upper level of L2
b. The dura is attached to the margins of the foramen magnum; this prevents the
passage of drugs from the peridural space into the cranial cavity. The dura sac ends
at the lower border of S2
c. The epidural space is located between the spinal dura centrally, and the
ligamentum flavum and the periosteal lining the spinal canal peripherally. It extends
from the base of the skull (foramen magnum), where the periosteum of the skull and
the dura fuse, to the coccyx
d. The epidural space contains areolar connective tissue and fat, arterial and venous
networks, Iymphatics, and the spinal nerve roots

Anatomy of the epidural space: The spinal cord ends at L2. The subarachnoid space
ends at S2

General Complications of Anesthesia


1. Awareness under anesthesia: Frequency of occurrence is greatest in obstetric
procedures, and patients having emergency surgery for trauma have a higher
incidence
a. Periods at risk: Induction, intraoperatively (light anesthesia), postop (muscle
relaxants used without sufficient sedation)
b. Monitoring: EEG, pulse volume plethysmography, clinical signs (decreased chest
compliance, bronchospasm, lacrimation, hypertension,
pupil size, eye movement)

2. Hypoxemia:
a. Definition: Deficient oxygenation of the blood (hypoxia: is reduction of oxygen
supply to a tissue below physiologic levels despite adequate perfusions of tissue by
blood)
b. Control of ventilation:
i. PaCO2 is the most important regulator of ventilation
ii. The respiratory response to hypoxemia is located solely in the peripheral
chemoreceptors, most importantly the carotid bodies
c. Causes of hypoxemia:
i. Hypoventilation: Drugs, medullary disease (encephalitis), anterior horn
cell disease (polio), disease of nerves to the respiratory muscles (Guillain
Barre', diptheria), disease of the neuromuscular junction (myasthenia
gravis) respiratory muscle disease (muscular dystrophy), and sleep apnea
ii. Absolute shunt: perfusion without ventilation
iii. Relative shunt: ventilation perfusion inequality
iv. Diffusion block: impaired diffusion of 02 from alveolus into the pulmonary
capillary blood
d. Biochemical changes of hypoxia: The main effect is cessation of oxidative
phosphorylation at the mitochondria) level, causing conversion to anaerobic
metabolism, reduced energy production and increased production of H+ and lactate
i. Cerebral effects (loss of autoregulation, loss of electrical activity)
ii. Cardiovascular effects (increased heart rate)
iii. Pulmonary effects (pulmonary vasoconstriction)
iv. Renal effects (acute renal failure)
v. Hepatic effects (decreased portal circulation)
e. Compensatory mechanisms:
i. Hyperventilation
ii. Pulmonary redistribution
iii. Increased cardiac output
iv. Increased hemoglobin concentration
v. Changes in oxygen hemoglobin dissociation curve

3. Hyponatremia: Serum sodium less than 136 mEq/L


a. Symptoms: The severity of symptoms depends upon the the rate of decrease of
serum sodium as well as the actual decrease in the serum sodium
i. Symptoms occur when serum sodium falls below 120 to 125 mEq/L
ii. Symptoms include confusion, anorexia, lethargy, nausea, vomiting, coma, and
seizures
b. Treatment:
i. Correct underlying disorder (give insulin if due to hyperglycemia)
ii. If hypovolemic-hypotonic: treat with isotonic saline
iii. If hypervolemic-hypotonic: treat with restriction of water and consider diuretics
iv. Symptomatic hyponatremia: treat with hypertonic saline

4. Hypokalemia: Serum potassium less than 3.5 mEq/L


a. Signs and symptoms: Results in disorders of muscle physiology
i. Respiratory arrest may occur with potassium concentration less than 2 mEq/L
ii. Depressed myocardial contractility
iii. Cardiac arrhythmias
iv. Renal effects (decreased glomerular filtration rate, increased ammonium
production)
v. Endocrine effects (decreased aldosterone and insulin release)
b. Causes of potassium loss:
i. GI
ii. Diuretics
iii. Renal tubular acidosis
iv. Cushing syndrome

5. Hyperkalemia:
a. Etiology:
i. Decreased excretion (renal failure, hypoaldosteronism)
ii. Extracellular shift (acidosis, ischemia, rhabdomyolysis, drugs such as
succinylcholine)
iii. Administration of blood, potassium penicillins, salt substitutes iv. Hemolysis
b. Signs and symptoms:
i. Muscle weakness
ii. Paresthesias
iii. Cardiac conduction abnormalities (become dangerous as K+ levels
reach 7 mEq/L)
c. EKG:
i. Peaked T waves
ii. ST segment depression
iii. Prolonged P-R intervals
iv. Loss of P wave
v. QRS widening
vi. Prolonged Q-T interval
d. Treatment:
i. EKG changes treated with CaCl2
ii. NaHCO2
iii. Glucose and insulin
iv. Kayexalate
v. Dialysis

6. Hypothermia:
a. Effects:
i. Decreases 02 consumption and CO2 production by 7-9%/°C in all
tissues
ii. Effects blood gas transport: shifts the oxygen dissociation curve to the left,
hemoglobin's affinity for oxygen increases 6%/°C decrease in temperature (may put
oxygen delivery at risk)
iii. Respiration: hypoxic ventilatory drive may be depressed or absent in presence of
hypothermia
iv. Cardiovascular function in the anesthetized patient:
 Heart rate and cardiac output decrease as temperature falls
 EKG changes (sinus bradycardia, prolonged PR interval, widened QRS complex,
prolonged QT interval, dysrhythmias at 28°C, ventricular fibrillation or asystole
below 28°C)
 Blood viscosity increases 2-3%/°C decrease in temperature v. Renal and Hepatic:
 Kidneys have largest proportionate reduction in blood flow with glomerular
filtration rate decreased by 60%
 Hepatic blood flow is decreased
vi. Central nervous system
 Function is altered (sedation, cold narcosis, progressive slowing of EEG, or EEG
becoming flat)
b. Treatment: Warming of patient

Complications of Endotracheal Intubation


1. Airway reflexes
2. Laceration or bruising
3. Dental trauma
4. Retropharyngeal dissection
5. Aspiration
6. Esophageal intubation
7. Endobronchial intubation
8. Dislocation of the mandible or arytenoid cartilages
9. Increased airway resistance
10. ET tube obstruction
11. ET tube cuff rupture
12. Laryngeal/tracheal/pulmonary Infection
13. Laryneal or vocal cord ulceration
14. Chronic hoarseness/vocal cord paralysis
Nerve Injuries During Anesthesia
1. Factors predisposing to nerve injury:
a. Tourniquets
b. Hypotension
c. Pre-existing ischemic disease (diabetes)
d. Use of muscle relaxants allowing overstretching of limbs
e. Positioning of the patient which results in stretching or prolonged pressure on a
nerve

2. Postoperative upper extremity complications: a. Brachial plexus injuries are


the most common

3. Postoperative lower extremity injuries:


a. Sciatic nerve injury occurs with external rotation of thighs and legs, or if the knees
are extended
b. Femoral nerve due to excessive angulation of the thigh
c. Common peroneal nerve is most frequently damaged (compressed in a brace of
lithotomy equipment)
d. Saphenous nerve is damaged by compression against the medial tibia] condyle
e. Obdurator nerve is compressed by undue flexion of the thigh to the groin

Other Medical Complications from Anesthesia


1. Pulmonary embolism:
a: Etiology and risk factors:
1. 95% of PE arise from deep venous thrombosis in the lower extremities
ii. PE is responsible for 20% of postoperative deaths.
iii. Older patients undergoing more extensive surgery are at high risk
iv. Highest risk patients have a history of thrombophlebitis, hip or pelvic
fractures, and major lower extremity orthopedic procedures
v. Other risk factors include acute M1, prolonged immobilization major
trauma, oral contraceptive use, CHF, pregnancy
b. Diagnosis: Remains problematic
i. Signs and symptoms:
 Dyspnea, pleuritic chest pain. hemoptysis, tachypnea, cough, wheezing and fever
ii. Major emboli may cause syncope and cardiovascular collapse
iii. Lab studies:

2. Postoperative nausea and vomiting: The vomiting center of the brain is located
in the reticular formation of the medulla. Impulses transmitted by fibers of
sympathetic and parasympathetic nervous system initiate the process of vomiting.
Motor impulses that initiate vomiting are carried in Cranial nerves V, VII, X, and XII
to the upper GI tract and through cervical and thoracic nerves to the diaphragm and
abdominal muscles
a. Predisposing factors:
i. Females more prone (probably estrogen related)
ii. Obesity
iii. Certain anesthetics (opioids, nitrous oxide, volatile anesthetics, barbiturates)
iv. Pain, hypotension, or hypoglycemia in postop period
v. Type of surgery (middle ear, ophthalmic, peritoneal irritation, surgery that results
in blood in the stomach)
b. Effects:
i. Autonomic:
 Tachycardia or bradycardia
 Hypotension or hypertension
ii. Disruption of suture lines
iii. Aspiration
iv. Prolonged hospitalization
c. Prevention:
i. Metroclopramide 10-20 mg IV
ii. Droperidol 0.63-1.25 mg IV
iii. Cimetidine 300 mg IV or po
iv. Ranitidine 150 mg po or 50 mg IV
v. Scopolamine 1.5 mg transdermally (patch)
d. Treatment:
i. Keep patient supine
ii. Antiemetics

3. Malignant hyperthermia: Thought to be due to reduction in the


reuptake of Ca by the sarcoplasmic reticulum necessary for the termination
of muscle contraction
a. Clinical features:
i. Unexplained tachycardia
ii. Hypercarbia or tachypnea
iii. Acidosis
iv. Muscle rigidity even in the presence of neuromuscular blockade
v. Hypoxemia
vii. Ventricular arrythmias
viii. Hyperkalemia
ix. Fever is a late sign
b. Treatment:
i. Discontinue all anesthetics
ii. Dantrolene 2.5 mg/kg IV initially
iii. NaHCO3
iv. Hyperkalemia corrected with insulin and glucose (no calcium)
v. Arrhythmias treated with procainamide
vi. Hyperthermia treated with refrigerated IV fluids, gastric, rectal and
bladder lavage with cold saline, surface cooling with ice
vii. Maintain urine output
c. Anesthesia for MI-L suseptible patients
i. Possible pretreatment with Dantrolene
ii. Local or regional anesthesia should be considered or
iii. General anesthesia with non-triggering agents such as:
 Barbiturates
 Propofol
 Benzodiazepines -Narcotics
 Nitrous oxide
d. Associated syndromes: An increased risk of MH reported in association
with a number of disorders, and therefore, these patients should be treated
as suseptible to MH:
• Duchenne muscular dystrophy
• King-Denborough syndrome (dwarfism, mental retardation, and
musculoskeletal abnormalities)
• Central core disease

4. Pulmonary aspiration:
a. Pathophysiology: Due to passive regurgitation and seen more
commonly in unconcious, obese, pregnant, and patients with full
stomachs
b. Types of pulmonary aspirate:
i. Particulate matter
ii. Liquid gastric contents iii. Blood
c. Incidence: About 10-20% perioperatively and intraoperatively (5%
mortality)
d. Diagnosis: Difficult to differentiate from other causes of pulmonary
insufficiency. Signs and symptoms are tachypnea, tachycardia, cyanosis
and respiratory acidosis
e. Treatment:
i. O2
ii. Tracheal intubation
iii. May need intravascular fluid replacement
iv. Antibiotics if bacterial infection develops
v. Bronchoscopy may be necessary to relieve airway obstruction
Chapter 3: Perioperative
Management

The Healthy Patient


The Diabetic Patient
The Hypertensive Patient
The Patient on Steroid Therapy
The Asthmatic Patient
The Alcoholic Patient
The Patient on Anticoagulant Therapy
The Patient With Clotting Abnormalities
The Rheumatoid Patient
The Sickle-Cell Patient
The Cardiac Patient
The Pulmonary Disease Patient
Perioperative Management of the Infant and Child
Mitral Valve Prolapse Patient
The Gouty Arthritis Patient
PERIOPERATIVE MANAGEMENT

Healthy Patient
The operative mortality for a healthy patient (ASA Class1) undergoing
elective surgery is approx. 1 in 10,000. The risk is minimized by paying
attention to 3 areas of the history: COAGULATION DISORDERS, DRUG
HISTORY AND PREVIOUS ANESTHETIC COMPLICATIONS - For local anesthesia
order: CBC/Diff, UA and PT/PTT. For general anesthesia order: CBC/diff, UA,
PT/PTT, SMA 6, pregnancy test, chest x-ray (if over 40 years old or if HX of
smoking)

The Diabetic Patient


1. Given early morning surgical preference.

2. If surgery is delayed start IV with D5W to avoid hypoglycemia from


remaining insulin or oral hypoglycemics from the day before. Check BS q2-3h.

NOTE* Remember the phrase," BETTER SWEET THAN SOUR".

3. Consult family physician for Insulin requirements.

4. Chlorpropamide has a half-life of 60 hours and thus should be stopped more


than 24 hours prior to surgery (best stopped 2-3 days before).

5. Diabetics with a propensity towards ketoacidosis require IV glucose and


insulin.

6. It is recommended that regular insulin be used during the perioperative


period.

7. Check pre-op potassium levels (the administration of Insulin reduces the


serum potassium levels. Therefore potassium replacement is needed when
insulin is to be administered in patients with hypokalemia.

8. Patients with severe diabetic autonomic neuropathy have an increased


incidence of gastroparesis and aspiration (increased chances for sudden death).
Autonomic neuropathy can result in a resting tachycardia and orthostatic
hypotension (BP drops 20mm Hg from a lying to a sitting position)

9. Preop blood sugar levels (mg/dl):


a. If hypoglycemia is present pre-op consider postponing the case
b. If pre-op BS in diet-controlled diabetic is below 200: no meds needed
c. If pre-op BS in diet-controlled diabetic is about 200 (some say 300): consider
regular insulin
d. If BS in Type II diabetic is < 150: stop the drug on the morning of the surgery
e. If BS in Type II diabetic is between 150-250: take AM dose of oral agent
f. If BS in Type II diabetic is > 250: start regular insulin
10. Monitor long term glucose levels by measuring glycosylated hemoglobin (HbA1C):
should be between 3 and 6% (realistically between 7-10).

11. Avoid post-op hyperglycemia: A BS greater than 250 inhibits phagocytosis,


leading to an increase in post-op infections, esp. gram negative.

12. For minor procedures in NIDDM (TYPE II) patients well controlled on oral agents
(other than chlorpropamide) d/c the drug one day before surgery and follow a NO
INSULIN-NO GLUCOSE protocol during surgery and reinstitute the regular oral
therapy when the patient begins eating

13. Well controlled IDDM patients undergoing short, minor procedures may also be
managed on a NO INSULIN-NO GLUCOSE protocol

14. For long, major procedures it is safer to utilize intravenous insulin therapy during
surgery

15. For IDDM (TYPE I) patients undergoing surgery, the regular insulin
administration can be done a couple of ways:
a. Administer 1 /3 to 1 /2 the patients usual AM dose and start D5W IV and cover
with insulin as per fingerstick BS or
b. Start 500ml D5W for the first hour, followed by 125 ml/hr of D5W with 1-2 units
of regular insulin (1 unit is used in patients taking < 20 units pre-op).
c. Give usual dose of insulin plus 50 gm of carbohydrate for each missed meal
d. SubQ. insulin 6-10 units Q6h

16. Post-op BS levels can be managed as follows:


a. 150-199 2 units
b. 200 -249 4 units
c. 250-299 6 units
d. 300-349 8 units
e. 350-399 10 units
f. >400 12 units
The Hypertensive Patient

1. Two types:
a. Essential hypertension (most common)
b. Secondary hypertension: due to renal disease, endocrine disease, oral
contraceptives, pregnancy and coarctation of the aorta.

2. Evaluate by taking BP 3 different times to eliminate other factors such as


stress.

Note * Take the initial reading in each arm. A significant difference


shows certain diseases.

3. Check the heart: there may be left ventricular hypertrophy with systolic aortic
ejection murmur.

NOTE* An S4 gallop is the most common finding in hypertension

4. Check the optic fundi: they may show retinopathy (indicator of the severity of the
disease).

5. Primarily treated by step-care approach:


a. Step 1- Start with a diuretic (or start with a calcium channel blocker)
b. Step 2- Add a beta blocker
c. Step. 3- Add a vasodilator
d. Step 4- Add a combination drug

6. Types:
a. Diuretics: Thiazides: Dyazide, Maxide, Hydrodiuril
b. Beta-Adrenergic Blocker: Tenormin, Lopressor, Corgard, and Inderal
c. Loop Diuretics: Lasix
d. Potassium Retaining: Aldactone, Triamterene, Amiloride
e. Calcium-Entry Blocker: Cardizem, Procardia
f. Combination- Dyazide, Maxide
g. Central Sympatholytics: Clonidine (Catapres)
h. Angiotensin Converting Enzyme Inhibitor: Capoten, Vasotec
i. Arteriolar dilators: Hydralazine, Minoxidil
j. Alpha andrenergic blockers: Minipress, Hytrin

NOTE* Diuretics are first line drugs used to treat hypertension, CHF and
lymphedema. There is concern regarding diuretics because of the
hypercholesterolemic effect of thiazides linked to ASHD 9 ischemic heart disease.
Thiazides can cause hyperglycemia, hyperuricemia and hypokalemia (pre-op
patients taking thiazides should have potassium checked). Therefore these drugs are
going out of favor as a first line drug for hypertension, replaced by calcium channel
blockers and ACE inhibitors. Loop diuretics (Lasix) are used in hypertensive patients
with fluid retention refractory to thiazides and in patients with impaired renal
function.
Beta blockers are effective in treating hypertension because they decrease the
heart rate, cardiac output and decrease renin release. These drugs prevent angina.
Beta blockers can precipitate bronchial asthma, Raynaud's phenomenon and
aggravate existing peripheral vascular disease.
ACE inhibitors are often used as a first line drug in treating hypertension.
They work by inhibiting the renin-angiotensin-aldosterone system. They are effective
vasodilators for the treatment of CHF.
Clonidine is a second line drug in treating hypertension. A severe rebound in
blood pressure may occur if the drug is abruptly discontinued, therefore, should be
continued perioperatively (4-6 hr pre-op)

7. Secondary hypertension is treated with elimination of the etiological factor,


where possible

8. For severely hypertensive patients (BP of 160/90 or higher), elective surgery


should be postponed until adequate control achieved

9. All antihypertensive medications should not be discontinued preoperatively


EXCEPT GUANETHIDINE and should be taken at 6 AM (FOR AM SURGERY) if
medication is normally taken in the morning

10. Acute elevations of BP are usually seen postoperatively which can be due to:
PAIN, REACTION TO ENDOTRACHIAL TUBE, VOLUME OVERLOAD, AND
EMERGENCE EXCITEMENT

11. Post-op hypertensive episodes are treated by eliminating the cause and if
necessary giving the following: sub-lingual Procardia or can give diuretics IV or
nitropatch, sublingual nitroglycerin, or nitroprusside

NOTE* Procardia given sublingually reduces blood pressure quickly, but does not
drop it below normal

12. These patients are very sensitive to vasopressors: DON'T USE


EPINEPHRINE!!!!

13. Potassium levels in all patients taking diuretics must be measured pre-op
and adequately replaced prior to surgery to prevent hypokalemia associated
cardiac arrhythmias

The Patient on Steroid Therapy


1. Steroids are utilized for the following: asthma, COPD (chronic obstructive
pulmonary disease), autoimmune diseases (RA) and malignancy. They have an effect
on three major areas of importance in the perioperative setting:

a. Suppression of the hypothalamus/pituitary adrenal axis (HPA)


b. Poor wound healing
c. Predisposition to infection

NOTE* There will be suppression of the HPA axis if any patient has taken more than
7.5mg/day of prednisone for longer than one week prior to surgery
2. Therefore, if a patient has taken more than 7.5mg/day of prednisone, then
exogenous steroids must be supplied during the perioperative period. Otherwise
there could be a resulting HYPOTENSION AND CARDIOVASCULAR COLLAPSE

3. For minor procedures the regimen of exogenous steroids is: Hydrocortisone IV


or IM @ 100 mg the evening before surgery, 100 mg prior to the start of the
procedure, and 100 mg Q 8h for 24 hours (there have been modifications of this
utilizing 100mg hydrocortisone pre-op and 100mg post-op)

NOTE* For minor procedures done under local sedation the following can be done: 15
mg prednisone PO 6AM before the surgery, 15 mg prednisone PO that same
afternoon, and 15 mg prednisone the next afternoon following the surgery

4. For major procedures the dosing is the same as above except the doses Q8h
should continue until the stress of the postoperative period has passed.

5. Tapering of steroids is only necessary if coverage lasts longer than 3 days

6. Special care must be taken with patients who have Addison's disease.
The Asthmatic Patient
1. Consider local or spinal anesthesia

2. Continue asthma medication up until 1 & 1 /2 hours before surgery (take


with a sip of water)
3. Inhalant medications are used as prescribed by the M.D. and can be used up
to 1 & 1 /2 hours prior to surgery

4. Aminophylline via continuous infusion of 800 mg In 500 cc D5W at 20cc per


hour (5cc per hour for children). You can adjust the rate according to the
theophylline level

The Alcoholic Patient


1. Should have 3-4 days of rehydration (to prevent DT's), vitamins, proper diet, and
no alcohol intake prior to coming to surgery

2. Nutritional status should be evaluated. This is done via measurement of serum


albumin and total lymphocytes
a. Serum albumin (<2.3 gm/dl): A measurement of nutritional status for the previous
week. Not a good indicator of nutritional status for the day of surgery
b. Total lymphocytes (<900): A good indicator of nutritional status for the day of
surgery. This measures the response to stress

Patients on Anticoagulant Therapy


1. Indications for anticoagulants:
a. PERIOPERATIVE PROPHYLAXIS FOR PREVENTION OF DVT's
b. ISCHEMIC HEART DISEASE,
c. ATRIAL FIBRILLATION
d. MITRAL STENOSIS
e. TIA's (transient ischemic attacks)
f. Prosthetic heart valves

2. Heparin type:
a. Inhibits intrinsic clotting pathway
b. Used as short term therapy for prophylaxis against DVT's

5,000u subQ 2h before surgery & 5,000u subcQ Q 12 h until pt ambulatory

c. DOSAGE REGULATED ACCORDING TO PTT (that's when you are treating, and not
prophylaxing)
d. EFFECTS REVERSED WITH PROTAMINE SULFATE

3. Coumadin type:
a. Inhibits extrinsic clotting pathway
b. Used as long term therapy
c. DOSAGE REGULATED ACCORDING TO PT
d. EFFECTS CAN BE REVERSED BY VIT. K (delayed) OR FFP (immediate)
4. For patients undergoing surgery, stop anticoagulants 3-6 days prior to
surgery and reinstate therapy postoperatively 24 hours after the
procedure. If you are worried about your patient not being on anticoagulants,
you can stop coumadin 3 days prior to surgery and start a heparin drip and stop
that 2-4 hours preoperatively

The Patient With Clotting Abnormalities


1. History is essential. Ask about: previous surgical bleeding problems, bruising
easily, frequent nose bleeds

2. Drugs that alter platelet function: ASPIRIN, NONSTEROIDALS, STEROIDS,


ANTIHISTAMINES, HIGH DOSES OF IV PENICILLIN (ESPECIALLY CARBENICILLIN),
and HEPARIN

3. Tests to determine bleeding diathesis: PLATELET COUNT, BLEEDING TIME


(Lee-White), PT, AND PTT

NOTE* Bleeding time is especially useful if the patient is on aspirin, as aspirin stops
platelet aggregation (must stop aspirin 1 week prior to surgery)

4. Bleeding diseases:
a. Von Willebrand's Disease
i. Abnormal factor VIII, PROLONGED PTT & BLEEDING TIME
ii. Treat with fresh frozen plasma
b. Hemophilia
i. Type a: factor VIII deficiency
ii. Type b: factor IX deficiency
iii. Type c: factor XI deficiency
iv. ALL 3 TYPES HAVE NORMAL BLEEDING TIME AND PROLONGED PTT
v. Treat with: Factor replacement, fresh frozen plasma, whole blood,
cryoprecipitate (4-6 units preoperatively and postoperatively), lyophilized
concentrate
c. Vitamin K Deficiency
i. Vit K effects both intrinsic and extrinsic clotting pathway
ii. PTT & PT are prolonged
iii. Treat with 10 mg of vit K subQ: normalizes pt in 8 hrs.

NOTE The hemophiliac patient must achieve a level of 70-100% prior to surgery,
with a minimum of 40% being attained for 10 days

NOTE* Due to the increased incidence of Hepatitis C and HIV with these patients,
use caution
The Rheumatoid Patient
1. Order CERVICAL SPINE X-RAYS PRE-OP (predisposition for atlas/axis
dislocation)

2. If patient is on steroids, you must supplement as per the previous


recommendations (see The Patient on Steroid Therapy)

3. If patient is on nonsteroidals, discontinue perioperatively as it will alter


bleeding times

4. If the patient is on antimalarials (chloroquine) for RA therapy, since a


frequent complication is eye problems the patient should consider a
preoperative consultation with an ophthalmologist.

5. If the patient is receiving gold salts for RA therapy, their potential toxic side
effects should be looked for (urticaria, skin eruptions, mouth ulcers,
eosinophilia, albuminuria, and leukopenia), especially in the urinalysis which
can document substantial protein loss. If the drugs are not withdrawn after the
onset of the side effects, patients can experience life threatening exfoliative
dermatitis, nephritis, and thrombocytopenia

6. If the patient is on penicillamine for RA therapy, one can possibly expect


potential decreased or slowed wound healing as this drug affects collagen
synthesis. Vitamin C supplementation is useful in these patients.

7. If the RA patient is on immunosuppressive drugs (methotrexate, captopurine,


azothioprine, and cyclophosphamide), the patient is more prone to infections
due to the side effects of these drugs (increased bleeding time, bone marrow
suppression leukopenia, thrombocytopenia). You should consider preoperative
antibiotics in these patients.

Note* These patients have been therapeutically immunosuppressed to decrease the


severity of their autoimmnune response, and are therefore, prone to infections

The Sickle Cell Patient


1. LOCAL ANESTHETICS A PRIORITY

2. Avoid respiratory depression with narcotics and sedatives- avoid hypoxia


with general anesthesia

3. Avoid using a tourniquet in patients with sickle-cell disease

4. Avoid hypoxia, dehydration, and acidosis intraoperatively and


postoperatively

5. Complications of surgery can include: delayed healing, increased incidence of


wound dehiscence, leg ulcerations, bone infection (osteomyelitis is usually
caused by Salmonella), and aseptic necrosis.

NOTE* In the preoperative evaluation of the patient suspected of having sickle cell
disease or trait the following tests are indicated:
a. Sickle-cell prep
b. Hemoglobin electrophoresis (for those with uncertain hemoglobinopathies)

NOTE* There is no specific therapy presently available for this disease. Treatment
consists of symptomatic relief along with adequate hydration and analgesics.
Oxygen therapy, alkalizing measures, and vasodilators have been used in attempts
to shorten the crisis

The Cardiac Patient


1. The cardiac patient is at a higher risk during the perioperative period, and
the factors which give the most post-op cardiac complications are:
a. S3 GALLOP WITH JUGULAR VEIN DISTENTION
b. M.I. WITHIN 6 MONTHS
c. RHYTHM OTHER THAN SINUS ON PRE-OP ECG
d. GREATER THAN 5 PVC's PER MINUTE BEFORE SX.
e. AGE GREATER THAN 70 YEARS OLD
f. PREMATURE ATRIAL CONTRACTIONS
g. SIGNIFICANT AORTIC STENOSIS

2. Elective surgery should be postponed until 6 MONTHS postmyocardial infarct


and in patients with uncompensated congenital heart failure

3. All risk factors such as unstable angina and cardiac failure should be
stabilized preoperatively

4. Pre-op studies should include the studies for the normal patient undergoing
general anesthesia, plus EKG, chest x-ray

5. Nitrates and Beta blockers should be continued during the perioperative


period

6. Echocardiography should be considered in any patient with a pathological


heart murmur

7. Endocarditis prophylaxis (indications):


a. Prosthetic heart valve
b. Prior hx of infective endocarditis
c. Cyanotic congenital heart defect
d. Surgically constructed systemic or pulmonary conduit
e. Hypertrophic cardiomyopathy
f. Mitral valve prolapse with regurgitation, valve thickening, or both

Not recommended for the following:


a. Atrial septal defect
b. Surgically repaired ventricular septal defect
c. Patent ductus arteriosus
d. Isolated Mitral valve prolapse
e. Mild tricuspid regurgitation
f. Previous rheumatic fever or Kawasaki disease without valvular dysfunction
g. Cardiac pacemaker
h. Implantable defibrillator

Antibiotic Regimen: Amoxicillin 2gm IV 30 min pre-op followed by 1gm q8h post-op
IF PENICILLIN ALLERGY: Vancomycin 1gm IV or Clindamycin 300mg PO pre-op and
150mg post-op.

NOTE* Probably the best prophylaxis (when indicated) should be IV penicillin or 1st
generation cephalosporin

NOTE* DRUG ADDICTS ARE PRONE TO SUBACUTE BACTERIAL ENDOCARDITIS


WITH TRICUSPID VALVE INFECTION SO ADMINISTER AN ANTISTAPH/STREP
ANTIBIOTIC PRE-OP (DICLOXICILLIN)

The Pulmonary Disease Patient:


1. The major post-op complications are atelectasis and infection

2. Smoking and obesity increase pulmonary risk

3. Use incentive spirometry post-op to prevent problems (start preop)

4. D/C smoking at least 1 week prior to surgery

5. Treat all respiratory infections prior to surgery

6. Administer Heparin 5,000u subQ in selected cases for prophylaxis for venous
thromboembolic disease. These patients are the ones with previous history of
a. IDIOPATHIC THROMBOPHLEBITIS
b. CHF
c. OTHER DISEASES WHERE VENOUS STASIS MAY OCCUR

7. Consider the use of preoperative incentive spirometry (prevents atelectesis


which can lead to pneumonia)

8. Consider arterial blood gas in patient with pulmonary history


a. If pCO2>45 mmHg and P02<55 mmHg then the patient will require
pulmonary function studies and possible use of bronchodilators (theophylline)
9.Consider the use of sequential compression devices in the pulmonary patient
(stockings) preop, postop, and intraoperatively

Perioperative Management of the Infant and Child


1. Preoperative evaluation:
a. Laboratory evaluation:
i. CBC
ii. ESR
iii. PT/PTT
iv. FBS
v. Sickle-cell test (in young blacks)
vi. Urinalysis

2. History and physical:


a. Examination via the pediatrician
b. Vital signs should be recorded. Temperature should be taken rectally in children
under 3. Rectal temps may run 1 ° higher than oral temps. Pulse rates and
respiration rates may be higher in children, and corresponding systolic blood
pressure readings may be much lower than for adults

3. Preoperative medications: Drug administration in the hospital must be


individualized for the child's level of growth and development, the form of the drug,
and the reason for medication
a. Proper dosage can be calculated via Young's Rule, Cowling's Rule, or Clark's Rule
i. Young: Divide the child's age by the age plus 12= The child's dose
ii. Cowling: Divide the age at the next birthday by 24
iii. Clark: Divide the weight (in lbs) by 150 to give the appropriate fraction.of
the adult dose
b. Sedatives: Used as a preanesthetic agent to decrease apprehension and provide
ease of induction
i. Barbiturates: secobarbital, pentobarbital, and amobarbital. The sedative dose of
barbiturates is 2 mg per kg orally TO

NOTE* Barbiturates are not generally recommended in infants as they can cause
paradoxic restlessness and excitement

ii. Chloral hydrate: A nonbarbiturate sedative-hypnotic, safe for infants and


children. Can be administered in juice or soft drinks to decrease the
unpleasant taste and minimize gastric irritation. The dosage ranges from
250 mg to 500 mg orally, up to three times daily. It may be given at
50 mg per kg at bedtime, up to 1 gm per dose

4. Fluids: Intraoperative fluid replacement is especially important in infants and


children because their reserve available to compensate for fluid loss is small (have a
large surface area for their mass). You must assess the fluid loss carefully. The
estimated fluid loss can be satisfactorily replaced with Ringer's lactate

5. NPO: The routine order of NPO should be avoided


a. No solid food or milk should be given for 12 hours prior to surgery, but fluid intake
should have a minimal interruption
b. Infants younger than 6 months are not placed on NPO status until 4 hours prior to
induction and those from 6 months to 3 years are placed on NPO status 6 hours
prior to surgery. For children over 3 years, NPO status should be 8 hours prior to
induction

6. Postoperative pain: Avoid injections if at all possible (fear)


a. Pentobarbital suppository: Dosage is 30 mg for children 1-5 years, 60 mg for those
6-10 years, and 90 mg for those 11-14 years
b. Codeine (3 mg per kg) for mild pain
c. Meperidine (1-1.5 mg per kg) for moderate pain

7. Antinauseants: Should be used sparingly, and doses should be adjusted


a. Promethazine: Dosage 12.5-25 mg rectally or orally
b. Vistaril®: Dosage 50 mg daily in divided doses for those under 6, and 50-100 mg
daily in divided doses for those over 6
c. Emetrol®: OTC antinauseant, free of toxicity and side effects, and works
immediately to control vomiting (1-2 tsp for infants and young children at 15 minute
intervals until vomiting ceases)
Mitral Valve Prolapse Patient
The majority of these patients are asymptomatic, so when is there the
necessity of preoperative antibiotic prophylaxis to prevent bacterial
endocarditis? Those individuals with evidence of mitral valve
regurgitation (seen on echocardiogram) are at risk for life threatening
complications and are given antibiotic prophylaxis. Asymptomatic
patients are not usually given antibiotic prophylaxis unless it is for a high
risk procedure producing a bacteremia (dental procedures, tonsillectomy
and adenoidectomy, bronchoscopies, I & D of infected tissues, and GI
and GU procedures). Podiatric procedures on low risk patients, are not
usually prophylaxed unless there is infected tissue.
1. Clinical Symptoms:
a. Chest pain
b. Palpitations
c. Syncope
d. Dyspnea
e. Arrhythmias (tachycardia)
f. High levels of anxiety
2. Cardiogenic findings:
a. Late or holosystolic murmur with or without a midsystolic click
b. ST wave changes
c. T wave inversions
d. Q-T elongation

3. Antibiotics:
a. Amoxicillin or clindamycin PO
b. First generation cephalosporin (Ancef), Unasyn, or Vancomycin IV

Gouty Arthritis Patient


The trauma of a surgical procedure can precipitate an acute attack of gouty arthritis,
but the perioperative management of patients with gout has not been standardized.
High-risk patients may include those with attacks in the past year, marked relief
after a test course of colchicine, and a documented need for antihyperuricemic
medication. Moderate-risk patients are those with past episodes of acute monarthritis
resembling gout clinically and hyperuricemia during the acute episode (no joint
aspiration for documentation). Low-risk patients are those with asymptomatic
primary hyperuricemia, asymptomatic hyperuricemia associated with drugs, or some
primary medical state (leukemia or other myeloproliferative disorders or solid tumors)
1. Treatment:
a. High-risk patients: Should receive colchicine in a dose of 0.5 mg 3 times daily for
2-3 days prior to the operation and 4-5 days postoperatively (colchicine or
indomethacin should be used when antihyperuricemic therapy is instituted)

NOTE* Allopurinol rather than uricosuric drugs is indicated for patients excreting
more than 600 mg of uric acid in 24 hours

b. Moderate-risk patients: Given colchicine prophylactically as in high risk patients


c. Low-risk patients: Need not be treated

NOTE* Postoperative attacks of gout are managed as in any patient with acute gouty
arthritis
Chapter 4: Preoperative

Evaluation
Screening Procedures
Summary of Perioperative Laboratory Testing
THE PREOPERATIVE EVALUATION
An adequate pre-op preparation of the surgical patient is of the utmost
importance. The quality of pre-anesthetic care largely determines the outcome,
especially in patients with significant medical problems. What will constitute
proper pre-anesthetic screening will vary from patient to patient according to
their physical status and chronological age. The anesthetic technique
applicable to healthy patients for simple procedures will differ from a more
complex technique for major procedures.

Screening Procedures
1. The History: A medical history obtaining relevant information:
a. DOES THE PATIENT HAVE AN M.D. AND IS HE/SHE PRESENTLY
TREATING THIS PATIENT
b. PRESENT or PAST MEDICAL PROBLEMS
c. PREVIOUS HOSPITALIZATIONS/SURGERY & PROBLEMS WITH
ANESTHESIA OR CLOTTING OR SCAR FORMATION
d. TAKING ANY MEDICATIONS FOR ANYTHING (RX OR OTC'S)
e. ANY KNOWN ALLERGIES
f. FAMILY HISTORY ( SICKLE CELL, DIABETES, HYPERTENSION, REACTION
TO ANESTHESIA, ETC.)
g. SOCIAL HISTORY (SMOKING, DRINKING, DRUGS ETC.)
h. REVIEW OF SYSTEMS

2. The Physical Examination:


a. Vital Signs: B.P., TEMP, PULSE, AND RESPIRATIONS
b. Review of systems: HEAD AND NECK, LUNGS AND HEART
(AUSCULTATION), ABDOMINAL, GI, GU, EXTREMITIES, NEUROLOGICAL,
DERMATOLOGICAL, VASCULAR, AND ORTHOPEDIC/BIOMECHANICAL

NOTE* ONCE THIS INFORMATION IS KNOWN THE PATIENT'S PHYSICAL


CONDITION CAN BE CATEGORIZED ACCORDING TO THE CLASSIFICATION OF
PHYSICAL STATUS ADOPTED BY THE AMERICAN SOCIETY OF
ANESTHESIOLOGISTS (ASA PHYSICAL STATUS)- See section Anesthesia

3. Laboratory Testing: BLOOD WORK, URINALYSIS, EKG AND CHEST XRAY


(CXR)
a. Complete Blood Count (Normal adult values):
i. WBC- 4,800-10,800/mm3
NOTE* A WBC GREATER THAN 11, 000= LEUKOCYTOSIS Leukocytosis
due to:
a. AN ACUTE BACTERIAL INFECTION (Viral infection usually has normal
WBC's)
b. INTOXICATIONS (GOUT, HEAVY METAL POISONING, VACCINES)
c. HEMOLYSIS
d. HEMORRHAGE
e. MYELOPROLIFERATIVE DISORDERS
f. NORMAL VARIANT
g. STRESS
h. DEHYDRATION/ HEMOCONCENTRATION

NOTE* A WBC LESS THAN 4, 400= LEUKOPENIA Leukopenia can be due to:
a. SEVERE INFECTION OR SEPTICEMIA
b. HEPATITIS
c. DRUGS (SULFA, ANALGESICS, BUTAZOLIDIN, THORAZINE)
d. TRAUMA AND AIDS
e. MYELOPROLIFERATIVE DISORDERS (MOST COMMON)

ii. WBC differential: allows for identification of the proportions of each type of
WBC for a more specific diagnosis of a disease entity
Divided into 2 groups: Granulocytes (neutrophils, eosinophils, basophils) and
Nongranulocytes (lymphocytes, monocytes)

NON-GRANULOCYTES:
Lymphocytes (20-40%): involved in antibody production, function in cell-
mediated immunity, delayed hypersensitivity, graft rejection, defense against
intracellular organisms such as tubercle bacillus, brucella, and neoplasms. An
Increase can indicate VIRUSES (MOST COMMON), GERMAN MEASLES,
BRUCELLOSIS, CONGENITAL SYPHILIS, THYROTOXICOSIS, PERTUSSIS, AND
MONONUCLEOSIS. A decrease can indicate HODGKIN'S DISEASE, DRUGS.
OR IRRADIATION, AND IMMUNOLOGICAL DEFICIENCY DISORDERS.
Monocytes (4-8%): transform into macrophages which destroy/ingest
bacteria etc. An Increase can indicate RECOVERY FROM ACUTE INFECTIONS,
SUBACUTE BACTERIAL ENDOCARDITIS,
MYCOTIC/RICKETTSIAL/PROTOZOAL & VIRAL INFECTIONS, HEMATOLOGIC
DISEASE, LEUKEMIA, AND HODGKIN'S DISEASE.

GRANULOCYTES:
Neutrophils (45-65%): same as PMN's (either SEGS 40-60% or Bands 05%).
The first specific line of defense for the body, an increase being called a SHIFT
LEFT (of immature neutrophils, called bands) usually indicating an acute
bacterial infection. A decrease in neutrophils, NEUTROPENIA, may indicate an
OVERWHELMING BACTERIAL INFECTION, SEVERE FUNGAL OR VIRAL
INFECTION, BONE MARROW DEPRESSION, AUTOANTIBODIES, BONE
MARROW REPLACEMENT, HYPERSPLENISM, AND MATURATION DEFECTS
(VITAMIN DEFECTS)
An Increase in neutrophils, NEUTROPHILIA, can indicate: INFECTION
Neutropenia may be the earliest clue to marrow failure
(TYPHOID), TOXIC AGENTS, PHYSICAL OR EMOTIONAL STIMULI, TISSUE
NECROSIS, HEMORRHAGE, HEMOLYSIS, AND HEMOLYTIC DISORDERS, OR
RECENT STRAINED DEFECATION (VALSALVA MANEUVER).

NOTE* Usually with a shift left there is a characteristic relative fall in


lymphocytes with neutrophil leukocytosis and increase in
young forms. When the infection subsides and the fever drops, the
total number of leukocytes decreases

Eosinophils (1 -3%): contain histamine and transport alkaline


phosphatase, an increase indicating INTESTINAL PARASITES, ALLERGIC
REACTIONS, URTICARIAL SKIN ERUPTIONS, LEUKEMIA, GI DISTURBANCES
OR SCARLET FEVER. A decrease can indicate SEVERE INFECTIONS,
CUSHING'S SYNDROME, ELECTRIC SHOCK THERAPY, AFTER INJECTION
WITH ACTH OR EPINEPHRINE.
Basophils (0-1 %): contain heparin and transport alk phos and histamine,
an increase is associated with POLYCYTHEMIA,
LEUKEMIA, CHICKEN POX, SMALL POX. A decrease can be due to ACUTE
INFECTIONS AND ACTH OR CORTICOSTEROIDS

iii. PLATELET COUNT (150,000-400,000): Platelet disorders should be


suspected in patients exhibiting PETECHIAE in the skin or mucosa.
THROMBOCYTOPENIA: Decreased platelet count, is seen with
THROMBOCYTOPENIA PURPURA, APLASTIC ANEMIA, OR SEPTICEMIA.

NOTE* The most common cause of thrombocytopenia is


chemotherapy

Decreased platelet count increases the risk for hemorrhage. If the platelet
count is less than 60,000 give platelets (1 unit will raise the count
5,000/cu mm. For any elective surgery the thrombocytopenic patient
should be postponed

• THROMBOCYTOSIS: Increased platelet count seen with splenectomy,


malignancy, and patient is prone to form clots with this condition
iv. RBC's: (Male, 4,700,000-6,100,000mm3) (Female, 4,200,0005,400,000/mm3)
An increase may be due to POLYCYTHEMIA, METASTATIC CARCINOMA
AFFECTING THE MARROW, THALASSEMIA, INCREASED ALTITUDE,
EXERCISE OR EMOTIONAL STRAIN

v. RETICULOCYTE COUNT (.5-1.5% OF THE TOTAL RED COUNT):


• It is the best indicator to assess marrow activity
• Increases found with IRON DEFICIENCY AND MEGALOBLASTIC ANEMIA
(recent bleeding/hemorrhage is the #1 cause)
• Decreases found with MACROCYTIC AND APLASTIC ANEMIAS

vi. HEMATOCRIT (Males 43-53% Gm/dl) (Females 35-47% Gm/dl):

NOTE* THE HCT IS USUALLY 3 TIMES THE Hb

• That portion of the total blood volume occupied by red cells versus plasma
• Increased with POLYCYTHEMIA, DEHYDRATION OR ADDISON'S DISEASE
• Decreased with anemias and hemorrhage

vii. HEMOGLOBIN (Males 12-17) (Females 11-16):


• Gives red cells their oxygen carrying capacity
• Increased with POLYCYTHEMIA, HEMOLYTIC ANEMIA, AND PORPHYRIAS
• Decreased with other anemias, HEMOGLOBINOPATHY and hemorrhage

viii. MCV=HCT/RBC: Reflects the size of the red cell

Decreased MCV Increased MCV


Iron deficiencies Pernicious anemia
Thalessemia (B12 and Folate deficiency)
Lead poisoning Liver disease
Sideroblastic anemia Hypothyroidism
Hypernatremia Leukemia

ix. MCH=Hb/RBC: Is an estimate of Hb in the average cell

x. MCHC=Hb/HCT: An estimate of Hb In an average cell


Anemias: Anemia may result from inadequate production of RBC's due to
deficiency states of bone marrow failure or to excessive loss or destruction
of erythrocytes. Precise diagnosis is needed for treatment with 4 basic
tests used:
a. Hematocrit and hemoglobin
b. Reticulocyte count: low= marrow is the site of pathology
normal or high count= good RBC production but cellular destruction
peripherally
c. Peripheral smear (with bone marrow biopsy) to examine the shape and
color of the RBC.
d. Platelet count
Also can do COOMBS test (DIRECT): do when the HCT is falling, elevated
bilirubin, don't have obvious bleeding, and have hemolysis on the
peripheral smear
Macrocytic: either megaloblastic (B 12 or folic acid deficiency) or
chronic
liver disease (OH'ism), hypothyroidism
Normocytic: due to sudden loss of blood, hemolytic anemias, anemias
caused principally by impaired production, and anemia of chronic disease
(DM or RA)
Hypochromic -microcytic: seen with iron deficiency (most common cause),
thalassemia, and sideroblastosis
Normochromic-microcytic: seen with bone marrow suppression,
hemolysis, chronic infections, and inflammatory diseases

Note* Pernicious anemia, in which B, 2 is not absorbed when gastric mucosa is


unable to produce intrinsic factor, resulting in a liver depletion of B12 and
subsequently a disturbance of DNA. Diagnosis is via SCHILLING TEST. Pernicious
is part of an autoimmune complex

NOTE* Sickle-Cell Anemia is seen in 1:600 American blacks. Variants of sickle-


cell disease are types of hemolytic anemias caused by the substitution of valine
for glutamic acid in the B-globin chains of HbA, the normal adult
hemoglobin.
Three principle variants exist:
a. Homozygous state (SS) or true sickle-cell anemia b. Heterozygous state
(AS) or sickle-cell trait
c. Double heterozygous state (SC) consists of two abnormal hemoglobin
chains Patients with SS or SC hemoglobinopathies are usually aware of their
disorders because of previous crises, while the AS patient is usually
asymptomatic. Three major factors contribute to red blood cell sickling:
a. The Hgb-S concentration
b. The partial pressure of oxygen
c. The pH
b. Tests for Hemostasis:
i.. Platelet count: for quantification purposes
ii. Prothrombin time: measurement of extrinsic blood coagulation pathway
(normal 11-13 seconds)
PT will be long with deficiencies of prothrombin, Factor V, Factor Il, or Factor X
• Affected by coumadin type anticoagulants

iii. Partial thromboplastin time: measurement of intrinsic blood coagulation


pathway (24-26 seconds is normal) and should be within 5 seconds of the
control (control is up to 45 seconds in some labs).
iv. Bleeding time (Lee-White): a standard in vivo assay that measures the
effectiveness of platelet plug formation. This test is performed in patients who
are suspected of having a qualitative platelet disorder, such as patients having
recently taken ASA or who have von Willebrand's disease

NOTE* The mechanism for blood clotting may be divided into 3 stages:
Stage 1: The production of plasma (extrinsic) or tissue (intrinsic) thromboplastin
to form prothrombin activator.
Stage 2: The conversion of fibrinogen to fibrin by the proteolytic action of
thrombin.
Stage 3: The conversion of fibrinogen to fibrin by the proteolytic action of
thrombin. Calcium is required for all stages.

NOTE* Coumadin prevents the conversion of vitamin K to its active form,


thereby impairing the formation of vitamin K-dependent clotting factors. It is
an anticoagulant indicated for the prophylaxis or treatment of venous
thrombosis, pulmonary embolism, atrial fibrillation with embolization, and
as a prophylaxis of embolism after myocardial infarction.
Coumadin therapy should be monitored by monthly international
normalized ratio (INR) testing. The test represents a standardized or
corrected prothrombin time since results for prothrombin time may vary by
institution. The INR should be obtained preoperatively for any patient on
coumadin (Warfarin) and should range from 2 to 3.
The dosing of the coumadin should be adjusted preoperatively so that the INR
is maintained at the low end of of the therapeutic range. Holding the dose of
coumadin for 2-3 days prior to elective surgery will provide protection
against intraoperative bleeding without compromising prophylaxis of the
patient. If necessary, the coumadin effects on the INR may be reversed by the
administration of vitamin K1 or fresh frozen plasma

c. Sequential Multiple Analyzer (SMA)


SMA 12 has the following tests: ALBUMIN, ALKALINE PHOSPHATASE,
BILIRUBIN, BLOOD UREA NITROGEN, CALCIUM, CHOLESTEROL, LACTATE
DEHYDROGENASE, SGOT, GLUCOSE, PHOSPHATE, TOTAL PROTEIN, AND
URIC ACID
SMA 16 adds the following four electrolytes: NA, K, CL, AND C02
1. Albumin (3.6-5.2 Gm/dl): A blood protein from the liver-a good indicator of
hepatic health
 Reduced levels are a problem and can result in EDEMA
 Increased levels can cause dehydration
 Reduction due to: LIVER DISEASE, MALIGNANCY, MALNUTRITION, KIDNEY
PATHOLOGY AND LARGE SKIN WOUNDS WITH SERUM LOSS SUCH AS
BURNS

ii. Alkaline Phosphatase (35-137 U/dl):


 Found in the liver, bone, placenta and lung as tissue-specific forms
Anything that stimulates osteoblastic activity increases the Alk
Phosphatase: METASTATIC BONE CANCER, GROWING CHILD, OSTEOGENIC
SARCOMA, PAGET'S DISEASE OF BONE, MONONUCLEOSIS, PREGNANCY,
GROWTH AFTER FRACTURE

NOTE* The most common cause of elevated liver alkaline phosphatase is


common duct obstruction, and the second most common cause is bed rest in
the hospitalized patient

iii. Bilirubin (less than 1.2 mg/dl):


 It is a pigment of bile from RBC breakdown in liver, spleen and marrow.
 Problems develop from abnormal production and decreased excretion
Total bilirubin divided into direct and indirect fractions (direct increased
due to direct hepatic obstruction- indirect due to liver disease.
Bilirubin increased with: CIRRHOSIS, ACUTE VIRAL HEPATITIS, CHF,
HEMOLYSIS AND SEPTICEMIA

iv. Blood Urea Nitrogen (5-25 mg/dl):


Is an end product of protein metabolism and produced only in the liver and
excreted by the kidney.
 BUN increased with: DIABETES AND PROTEIN BREAKDOWN, KIDNEY
PATHOLOGY, FEVER AND PROTEIN BREAKDOWN, CHF, RBC
BREAKDOWN AND STARVATION (dehydration is #1 cause)
 BUN decreased with: HYDRATION, LIVER PATHOLOGY AND DECREASED
PRODUCTION

v. Calcium (4.5-5.5 mg/dl):


Serum calcium increased with any disease of bony demineralization such
as: MULTIPLE MYELOMA, CANCER OF BONE, EXCESSIVE CALCIUM INTAKE,
HYPERPARATHYROIDISM AND HYPERVITAMINOSIS D.
Serum calcium decreased with: RENAL FAILURE (#1), POOR ABSORPTION,
MALNUTRITION, HYPOPARATHYROIDISM,
PSEUDOHYPOPARATHYROIDISM AND DIARRHEA

vi. Chlorides (98-110 meq/L):


 Usually lost from the urine or GI fluid which carry the chloride ion
 Chlorides decreased with: DEHYDRATION (#1), VOMITING, DIARRHEA,
ULCER AND FLUID LOSS, EDEMA, DIURESIS, GI OBSTRUCTION AND
INFECTION.
 Rare to have elevated chloride, if so, can indicate CYSTIC FIBROSIS

vii. Cholesterol (150-220 mg/dl):


 Produced from food and endogenous production (mostly in the liver)
 Used for membranes, STEROID hormones and bile acids.
 Increases: IDIOPATHIC, HYPOTHYROIDISM, DIABETES MELLITUS,
PANCREATITIS
 Decreases: MALABSORPTION, LIVER DAMAGE AND POOR PRODUCTION

viii. Creatinine (under 1.2 mg%):


 Produced from creatine in muscle tissue and excreted by the kidneys -Not
elevated in the blood until 50% of renal function obliterated
 Any major reduction in creatinine clearance of urine indicates kidney
impairment

ix. Glucose (70-110 mg/dl):


 The liver produces glucose from protein (gluconeogenesis).
 Blood glucose is filtered through the kidney glomeruli and reabsorbed in the
proximal tubule, and if renal threshold is exceeded glucose is spilled into the
urine.
 Elevated with: DIABETES MELLITUS, CUSHING'S SYNDROME,
ACROMEGALY, STRESS, BURNS, SHOCK, ACUTE PANCREATITIS,
OBESITY, ACTH ADMINISTRATION AND AGE
 Decreased with: INSULIN SHOCK, ISLET CELL TUMOR, NUTRITION,
GLUCAGON DEFICIENCY, ADDISON'S DISEASE AND HYPOTHYROIDISM
(Hypoglycemia may be precursor to DM)

x. Lactate Dehydrogenase (208-378U/L):


 This is a glycolytic enzyme that functions in carbohydrate metabolism which
is found in the kidney, liver, heart, RBC's.
 Increased with any tissue damage, therefore it is NOT SPECIFIC

xi. Total Protein (6.4-8.3 g/dI):


 Contains albumin and globulins and rarely is total protein increased since
most disease states lower body reserves
 Increased with: DEHYDRATION caused by vomiting and diarrhea
 Albumin:Globulin Ratio= 1.5:1-2.5:1

xii. Phosphate (8-12 mg/dl):


 it is absorbed in the intestine, stored in bone, and excreted by the kidneys
 Elevated with: RENAL FAILURE, HYPOPARATHYROIDISM, INCREASE VIT D
INTAKE, BONE DISEASE, FRACTURES, PAGET'S DISEASE OF BONE,
MULTIPLE MYELOMA
 Decreased with: HYPOVITAMINOSIS D, DIABETES,
HYPERPARATHYROIDISM (MOST COMMON IS POOR NUTRITION)
xiii. Potassium (3.5-5.1 mEq/l): Very important element in the pre-op
evaluation
 Decreased K produces muscle cramps and arrhythmias while increased K
produces arrhythmias and cardiac pathology
 increased with: USE OF MINERALCORTICOIDS, RENAL FAILURE,
ACIDOSIS, DIABETIC KETOACIDOSIS.
 Decreased with: ALKALOSIS, DIARRHEA, DIURETICS, INSULIN,
MALABSORPTION AND STARVATION (#1 cause is with excess IV's) -
Symptoms don't start until serum levels fall below 2.5 mEq/L -Cardiac
standstill can occur when levels exceed 7.5-8.0 mEq/L

xiv. SGOT (AST 1-45 IU/L):


 The enzyme used in glycolysis and energy production and found mostly in
the liver, heart, and also in the muscle, kidney, and pancreas.
 Sudden increase seen with MI, also with liver and lung pathology

NOTE* Elevated SCOT (AST) seen in pulmonary embolism postoperatively

 Elevated in: CARDIAC DAMAGE, LIVER DAMAGE, SKELETAL PATHOLOGY,


PANCREATIC PATHOLOGY.

The SGOT parallels the SGPT AL , whose normal values = 1-60 IU/L,
except the SGOT shows higher values with MI and the SGPT has higher
values with LIVER PATHOLOGY

xv. Uric Acid (2.8-8.0 mg/dl):


 Manufactured from purine metabolism
 The pH of urine must be close to 7.4, as uric acid is poorly soluble as the pH
decreases with crystals forming depositing in tissues across the cell
membrane. Therefore alkalinization of the urine increases uricosuric
activity.

NOTE* WHEN TREATING GOUT CHECK THE pH OF THE URINE

xvi. Serum Amylase: NOT USUALLY DONE PREOPERATIVELY


 Enzymes produced in the pancreas and used for the digestion of starch
 Elevation due to cellular destruction of amylase in the pancreas or poor
renal excretion, due to: PANCREATITIS, PERFORATED ULCER,
PANCREATIC DUCT OBSTRUCTION, AND RENAL FAILURE.
 Decreased due to CHRONIC PANCREATIC OBSTRUCTION.

xvi. Acetone: Detected by dipstick urine test- should be negative


 ketone body from fat metabolism
 Elevated with: STARVATION, STRESS, DIABETES, LOW CARBOHYDRATE
DIET

NOTE*AS KETONES INCREASE pH DECREASES


xvii. Acid phosphatase: NOT USUALLY DONE PREOPERATIVELY
 Found in the male prostate, the male urine, RBC's and platelets (female).
 High levels consistent with METASTATIC CARCINOMA OF THE PROSTATE
 Can also be elevated with MULTIPLE MYELOMA AND PAGET'S DISEASE OF
BONE

NOTE*: SERUM PROFILE


LIVER- BILIRUBIN, CHOLESTEROL, SGOT, SGPT, ALKALINE PHOSPHATASE,
ALBUMIN
KIDNEY- CHLORIDE, CREATININE, BUN, PHOSPHOROUS, PROTEIN, TOTAL
PROTEIN, LDH, Na, K, C1, Co 2
CARDIAC- LDH, CPK, SGOT, ELECTROLYTES, BUN
ARTHRITIC- LATEX FIX, ESR, VDRL, ANA, ASO TITER, URIC ACID, LE PREP
(RARELY DONE- ANA MORE ACCURATE)

4. URINE ANALYSIS:
a. A multitest dipstick is used on the urine to get rough readings on glucose,
acetone, bile, urobilinogen, protein and blood. The urine is then centrifuged
and the solid matter goes to the bottom and is then examined microscopically
for red/white/epithelial cells, crystals and casts.
b. Also checked is the color, clarity, pH and specific gravity
c. The specific gravity range is 1.020-1.032. (when concentrated in the AM)
A measure of the kidney's ability to concentrate and the SG is elevated by
extra glucose (diabetes) and protein due to increased concentration in the fluid.
d. All glucose in the urine is competely reabsorbed by the proximal tubules,
therefore the appearance of glucose in the urine when the blood glucose levels
is below 180 (250 IN SOME TEXTS) may signify proximal tubule damage.
e. Small amounts of protein are found in the urine (albumin and globulins),
however, large amounts in excess of 4gms daily indicate glomerular disease
(mostly albumin)
f. Ketone bodies accumulate due to altered lipid metabolism, most frequently as
a consequence of diabetes mellitus or low carbohydrate diet
g. OCCULT hematuria occurs from: EXCERCISE (#1), CYSTITIS,
HEMORRHAGE, MENSTRUATION, GLOMERULONEPHRITIS, HYPERTENSION,
POLYCYSTIC DISEASE AND RENAL THROMBOSIS
i. Microscopic hematuria seen with acute infection, sickle cell anemia,
excercise, menses and SBE.
h. GROSS hematuria associated with stones, tumors, TB, and acute
glomerulonephritis.
i. Normal urine is slightly acidic, pH usually below 5.3, is-altered by diet or
medications
i. An alkaline urine in the presence of metabolic acidosis suggests Renal
Tubular Acidosis.

NOTE* An alkaline urine is most commonly seen in a proteus UTI


j. Normal findings in the urinary sediment includes up to : 1 RBC, 5 WBC'S,
AND AN OCCASIONAL CAST.
k. Abnormal findings in the sediment include: RBC'S, WBC CASTS, YEAST,
CRYSTALS OR EPITHELIAL CELLS.
l. Casts are protein conglomerations in the shape of the renal tubule.
Two types of casts:
i. CELLULAR- RBC, WBC, EPITHELIAL CELLS, OR ANY COMBINATION IN A
PROTEIN MATRIX

NOTE* With WBC casts, the cells become emulsified in the protein matrix and may
be found in pyelonephritis, acute glomerulonephritis, or SLE.
The cells in RBC cell casts are emulsified in protein matrix and are indicative of
glomerulitis.
Casts containing fat droplets are called fatty casts and are associated with
nephrotic syndrome

ii. PROTEIN- are composed entirely of protein


m. Crystals in the urine may indicate stones or certain metabolic diseases
n. Bile present indicates hepatic obstruction or constipation
o. Urobilinogen can be present due to HEPATITIS
p. Catecholamines: elevation in the urine is indicative of pheochromocytoma or
extramedullary chromaffin tumors, malignant hypertension, progressive ms.
dystrophy, myasthenia gravis, and drugs. (this is not usually done
preoperatively)

5. Pregnancy testing:
Should be performed on all women within childbearing years

6. Chest x-ray:
This compliments the H & P as a starting point for the diagnosis and evaluation
of suspected pulmonary disorders. Abnormalities seen on the x-ray are: CHF,
pulmonary masses, pleural effusions, pneumonia. Remember that the chest x-
ray alone is not a good indicator for operative risk

7. Electrocardiogram:
Recommended that all patients over the age of 40 have this test done, however,
a poor predictor of ischemic heart disease and perioperative
cardiac morbidity and mortality. Electrocardiography is a graphic
representation of the electrical currents associated with the contraction of the
heart muscle. The basic function of the electrocardiographic monitor is to
amplify the small voltage formed by the depolarization of the heart so that it
can be presented on the screen for visual monitoring or so that a graphic record
can be made.
a. Conduction mechanism of the heart
i. Sinoatrial node (SA): The electrical impulse is formed in the SA node, which is
the physiologic pacemaker of the heart, located at the junction of the right
atrium and superior vena cava
ii. Atrioventricular node (AV): After the SA node the impulse speeds to the AV
node, at the junction of the atria and ventricles

iii. Bundle of His: After the AV node fires, the impulse travels to the bundle of
His, then the right and left bundle branches, Purkinje's fibers, and the
ventricles
b. The normal electrocardiogram:
i. P wave: Indicates the results of the electrical activity during atrial
depolarization that initiates atrial contractions
 P-R interval: Represents the time it takes the impulse to spread from the SA
node to the ventricles (normal time 0.12-0.20 sec)

Normal EKG Tracing

i. QRS complex: Is due to the depolarization of the ventricles, which triggers


their contraction. It indicates the time it takes for ventricular depolarization
(normal time 0.06-0.10 sec)
ii. T wave: Occurs at the end of ventricular systole
b. The recording paper has horizontal and vertical lines:
i. Horizontal lines: Indicate voltage and are 1 mm apart (1 mm represents 1 mV)
ii. Vertical lines: Indicate time and are separated by an interval of 0.04 sec. The
heavy vertical lines represent 0.2 sec intervals, and the five large squares
represent 1 sec
iii. Heart rate per minute: Is determined by counting the number of QRS
complexes that occur in 3 seconds (15 large squares) and multiplying by 20
c. EKG (ECG) leads:
i. Standard lead (I, II, Ill): Are bipolar indirect leads, that record the difference
between two points on the body, in electrical events formed by the cardiac
action. The electrodes are attached to the left arm, right arm, and left leg. The
right leg is used for a ground. The terminals are placed as follows:
• Lead l: right arm, left arm
• Lead II: right arm, left leg
• Lead III: left arm, left leg
ii. Unipolar extrmity leads: Indicate the electrical potentials at one point. The
electrodes of the three limbs are attached together to form one electrode, called
the central terminal lead. The differences of the electrical potentials are
recorded between the central terminal and each of the three extremities
• Lead aVr: is positioned at the right arm
• Lead aVL: is positioned at the left arm
• Lead aVf: is positioned at the left foot
iii. Precordial leads: Are unipolar leads that indicate the variation of electrical
potentials at a given cardiac anatomic site. The central terminal lead is used,
and the exploring electrode is positioned in the chest and in six different
locations, V1 to V6

Summary of Perioperative Laboratory Testing


1. Blood studies:
a. CBC: Hgb, Hct, RBC, WBC, platelets, red cell indices (MCV MCH etc.), WBC
differential (segs, lymphs, bands, etc.)
b. Serum electrolytes: Sodium, potassium, chloride, calcium, magnesium
c. Liver studies:
i. Hepatocellular enzymes: SGOT/ALT
ii. Studies of obstruction: Alkaline phosphatase, glutamyltransferase (GGT),
and 5' Nucleotidase
iii. Evaluation of jaundice: Bilirubin (conjugated or unconjugated)
iv. Other liver studies: Albumin, prothrombin time (PT)
d. Renal studies: Blood Urea Nitrogen, creatinine, serum electrolytes (mediated
by kidney), creatine
e. Cardiac enzymes: Lactic dehydrogenase (LDH), SCOT (AST), creatine
phosphokinase (CPK)
f. Clotting: Partial thromboplastin, prothrombin, bleeding time, INR

2. Urinalysis:
a. Dipstick: Protein, blood, glucose, ketones, pH
b. Microscopic exam: RBC's, WBC's, cellular casts, bacteria, crystals

3. Pulmonary function tests:


a. Static lung volumes: Total lung capacity, functional residual capacity,
residual volume, vital capacity
b. Dynamic volume measurement: Forced vital capacity (FVC), forced vital
capacity in 1 second (FEV1), and FEV1 /FVC
4. Arterial blood gas: Oxygenation, carbon dioxide, 02 saturation, pH
5. Chest x-rays: Cardiac size, calcifications, lung masses, pleural effusions,
infiltrates, interstitial patterns

6. EKG:
a. Rate
b. Rhythm
c. Intracardiac conduction times (PR interval, QRS interval, QT interval)
d. Chamber enlargement (hypertrophy of myocardium)
e. Myocardial infarction patterns (pathologic Q waves)
f. ST segments of ischemia
Chapter 5: Postoperative Care
&
Complications
Fever
Altered Mental States
Water and Electrolyte Imbalance
Oliguria
Chest Pain
Postoperative Hypertension
Postoperative Infection
Anxiety and Pain Management
Nausea
Constipation
Shivering
DVT
Compartment Syndrome
Hemorrhage
Septic Syndrome
Thyroid Storm
POSTOPERATIVE COMPLICATIONS
The number of potential post-operative problems is large, with some occurring
more frequently. Those are FEVER, ALTERED MENTAL STATUS, WATER &
ELECTROLYTE DISTURBANCES, RENAL FAILURE, CHEST PAIN, SHORTNESS
OF BREATH AND HYPERTENSION (A hospitalized patient developing cough,
fever and an abnormal chest x-ray after prolonged general anesthesia and a
patient with the same fin dins following a procedure done under local
anesthesia must be viewed differently)

Fever
Temperature> 101.6 orally: Fever is the most common post-op problem. The
time of occurrence is an important clinical guide in determining the etiology.
Fever can also occur without any associated pathological conditions.

WIND (PULMONARY EMBOLISM, ATELECTASIS AND PNEUMONIA) WATER


(URINARY TRACT INFECTION)
WALKING (THROMBOPHLEBITIS- between 3-10 days postop; positive
Homan's sign)
WONDER DRUGS (DRUG FEVER- the patient is less ill than the fever
would suggest)
WOUND (SEPSIS)
WOW (BREASTS: MASTITIS) -Usually ObGyn
WOMB (ENDOMETRITIS)-Usually ObGyn

1. Fever within 24 hours (THE MOST COMMON) can be due to:


a. ATELECTASIS FROM HYPOVENTILATION
b. ASPIRATION OF ORAL OR GASTRIC CONTENTS
c. DRUG REACTIONS (IF PATIENT RECEIVED BLOOD THEN ADD
TRANSFUSION REACTIONS)

2. Fever after 24 hours following surgery can be due to:


a. ATELECTASIS & ASPIRATION
b. WOUND INFECTIONS
c. IV SITE PHLEBITIS
d. UTI
e. DVT
f. HEPATITIS
NOTE* IF A FEVER DEVELOPS WITHIN THE FIRST 6-8 HOURS FOLLOWING
SURGERY, THE MOST LIKELY CAUSE IS AN ENDOCRINE IMBALANCE
WHICH MAY LEAD TO A TEMPORARY THYROID CRISIS OR
ADRENOCORTICOID
INSUFFICIENCY. SHOULD FEVER DEVELOP 8-10 HOURS AFTER SURGERY,
A PULMONARY MALFUNCTION SHOULD BE ENTERTAINED AS THE LIKELY
CAUSE. URINARY TRACT INFECTIONS ARE THE MOST LIKELY CAUSE OF
FEVERS AT THE SECOND OR THIRD POSTOP DAY (URGENCY,
FREQUENCY, PAIN, AND BACTERIA). BACTERIAL INFECTIONS OCCUR 3-7
DAYS POST-OP: WITH THE EXCEPTION OF GROUP A STREP, WHICH CAN
OCCUR EARLIER. A SPIKING TEMPERATURE IS INDICATIVE OF AN
INFECTION, ALONG WITH TACHYCARDIA, CHILLS, MALAISE,
LETHARGY, AND LOSS OF APPETITE
ALL IV LINES AND HEPARIN LOCKS SHOULD BE CHANGED EVERY 2-3
DAYS TO PREVENT INFECTION. DIABETIC FEMALES AND ELDERLY MALES
ARE AT HIGHEST RISK FOR UTI'S.

3. DO NOT TREAT THE FEVER WITH ANTIPYRETICS UNTIL THE CAUSE IS


DETERMINED (There are exceptions to this rule: patient discomfort,
delirium or convulsions and the precipitation of heart failure

4. Risk factors:
a. Surgery over 2 hours in time
b. Transfusion
c. Pre-existing infection
d. In place prosthesis or shunt

Altered Mental States


1. Can develop In: ELDERLY PATIENTS, METABOLIC DISORDERS,
HYPONATREMIA, HYPOGLYCEMIA, HYPERGLYCEMIA (MOSTLY HYPO),
ACIDOSIS (SEPSIS, SHOCK AND DIABETIC KETOACIDOSIS), Ml,
ARRYTHMIAS, CVA'S & DRUG PROBLEMS (TOXICITY AND DRUG
INTERACTIONS)

Water and Electrolyte Disturbances


1. Can develop as a result of: IMPROPER FLUID REPLACEMENT,
INCREASED BODY LOSS (Vomiting and fever), MEDICATIONS AND STRESS

2. FLUID ADMINISTRATION OF 2000-3000 MIDDAY IS NEEDED TO


MAINTAIN A DESIRED URINE OUTPUT OF 1000-1500 ML/DAY

3. FEVER INCREASES WATER LOSS (FOR EACH DEGREE ABOVE NORMAL


WATER LOSS IS INCREASED BY 100-150 ML/DAY

4. Sodium disorders (Na):


a. HYPONATREMIA (Na<125 meq/L): Can occur post-op. Symptoms are
lethargy, confusion, coma, muscle twitch, nausea, vomiting and cardiac
problems. Treated by restricting fluids to 1000-1500 ml fluid/day. Can occur
from
i. HYPONATREMIA WITH VOLUME EXCESS
ii. HYPONATREMIA WITH VOLUME DEPLETION
iii. HYPONATREMIA WITH NORMOVOLEMIA
b. HYPERNATREMIA (Na>145 meq/L): Symptoms are confusion, stupor, coma,
muscle tremors, seizures, pulmonary and peripheral edema. Treated with D5W
slowly over 24 hours

5. Potassium disorders (HYPO OR HYPERKALEMIA): can occur post-op, and


can be life threatening
a. HYPOKALEMIA (K<2.5) occurs in the following:
i. PATIENTS GETTING DIURETICS
ii. VOMITING
iii. DIARRHEA
iv. BRITTLE DIABETES
v. ANTI-PSEUDOMONAL ANTIBIOTICS
vi. HIGH DOSES OF STEROIDS (See section Pre-op Evaluation- SMA)
b. HYPERKALEMIA (K>4.5) is seen less frequently but is far more lethal than
HYPOKALEMIA and can manifest itself with weakness, hyperactive reflexes,
and cardiac manifestations/standstill
c. Treatment of hyperkalemia must be immediate: Give Calcium Chloride, then
Sodium Bicarb or even D50 with insulin (and KAYEXELATE if necessary)

Oliguria
1. Due to:
a. BLADDER OBSTRUCTION
b. THE COMBINATION OF BEDREST/PROSTATIC
HYPERTROPHY/ANALGESICS (DEMEROL***)
c. ISCHEMIC INSULTS
d. NEPHROTOXIC DRUGS

2. To approach a patient with acute renal failure you must do the


following:
a. Measure urine volumes and medication use
b. Assess hydration and cardiovascular status
c. Bladder catheterization a possibility
d. Fresh urine analysis- with examination of the sediment
e. Labs: serum and urine sodium, creatinine, osmolality and BUN
f. Special tests: RENAL FAILURE INDEX AND FRACTIONAL EXCRETION OF
SODIUM MEASUREMENT

Chest Pain
1. Etiology:
a. MI
b. PULMONARY EMBOLUS
c. PNEUMONIA
d. ANXIETY
e. ATELECTASIS
f. ASPIRATION
g. CHF

2. Treatment:
a. Must perform EKG, chest x-ray, arterial blood gases (establishes probability
of a diagnosis)
b. May do VENTILATION PERFUSION RADIONUCLEOTIDE LUNG SCAN (A
negative study R/O P.E.)
c. If doubt remains, can do: PULMONARY ANGIOGRAPHY (Most specific and
sensitive test for P.E.)
d. A distinction should be made between ASPIRATION PNEUMONIA AND
ASPIRATION PNEUMONITIS.

NOTE* The former is caused from aspiration of sterile/acidic gastric contents and
results in a chemical pneumonitis. The latter occurs when polymicrobial oral
secretions are introduced into the pulmonary system.

Post-op Hypertension
1. Etiology:
a. Noxious stimuli (most common): Pain, excitement (emergence from general
anesthesia), shivering, and mild hypothermia
b. Drug induced: Withdrawal from beta blockers
c. Metabolic abnormalities: Pheochromocytoma, Cushing's syndrome
d. Miscellaneous: Fluid overload, distended bladder, tight cast

2. Treatment: Try to establish the etiology


a. Treat pain and anxiety first, and warming of the patient
b. If the hypertension persists, it can be treated with sublingual Procardia 20
mg, IV Furosemide (if volume overload, distended bladder), or ideally a drug
which is fast acting, titratable, rapidly reversible, efficacious, and with no side
effects that can offset the cause of the hypertension

NOTE* Sudden decreases in blood pressure accompanying treatment of


hypertension can have catastrophic results: seizures, CVA's, acute Ml, renal
failure, and death

Postoperative Infection
1. Can occur 3-7 days postoperatively
2. Group A Strep can occur earlier
3. Symptoms
a. Increased throbbing pain
b. Swelling
4. Signs
a. Wound drainage
b. Wound dehiscence
c. Erythema
d. Fever
5. Treatment: Decide on hospital or outpatient therapy

NOTE* Admit to hospital with systemic manifestations (fever/shakes/chills,


lymphangitis, lymphadenopathy), debilitated host ( Diabetic, PVD, alcoholic,
immunosuppressive therapy, burn patients), need for IV antibiotics, resistant
organism, risk of deep space infection, deep space necrosis or wet gangrene,
suspicion of osteomyelitis, need for extensive surgical debridement, and
failure of outpatient. therapy

a. Consider ID consult/ internal medicine consult/ vascular consult prn


b. Perform local wound care/release and remove sutures/ incision and
drainage as necessary
c. Perform gram stain and aerobic/anaerobic/fungal/acid fast cultures (as
necessary) from wound site and blood cultures if indicated
d. Obtain necessary lab studies (CBC,Hb, urinalysis, chemistries)
e. Begin antibiotics as per gram stain a clinical evaluation and judgement/
reevaluate as per C & S
f. Continue appropriate antibiotic at least 10 days (especially when treating
strep, to prevent post strep glomerulonephritis. May consider IV antibiotics)

Anxiety and Pain Management


1. Pain complaints reflect not only tissue injury, but many psychological
dimensions of suffering as well.
a. A long acting local anesthetic injected at the end of the surgery will delay
postoperative pain past the initial phases.
b. The injection of a short-acting soluble steroid will help diminish painful
swelling and inflammation associated with the lag phase of healing.
c. NSAID's can many times be the sole postoperative medication because
inflammation is the hallmark of the initial and most painful postoperative
phase.
d. Opiates or opiate-like drugs can be used in combination with ASA or
acetaminophen. These drugs should be considered if NSAIDs have failed to
alleviate the pain.
e. Excessive anxiety will greatly reduce the pain threshold, therefore sedatives
can greatly help these patients.
f. Toradol® IM/IV has been found to be very effective immediately
postoperatively in eliminating pain (non-narcotic)

Note* The side effects of narcotics are: hypotension, respiratory depression,


urinary retention, nausea, and reduced peristalsis.

Nausea
1. Prolonged vomiting can lead to dehydration therefore must be
prevented or controlled.
2. Postoperative nausea responds well to:
a. Phenothiazine antiemetics (Compazine, Phenergan, Trilafon).
b. Other treatment can include the use of antihistamines (Benadryl, Atarax) c.
Tigan 100mg IM, can be of benefit
d. In children you can use Emetrol (5-10ml Q 15 minutes)

NOTE* Can prevent postoperative nausea by using a Scopolamine patch behind the
ear. This is applied preoperatively and worn perioperatively and postoperatively.
The only adverse effect is blurred vision postoperatively

Constipation
1. Can be a problem especially with elderly patients
a. Begin treatment 1st with the mildest treatment, a laxative cathartic, milk of
magnesia (watch the electrolytes)
b. If unsuccessful then try Colace 50-250mg
c. If unsuccessful then use a contact cathartic, Dulcolax

Note* All laxatives and cathartics are contraindicated in patients with cramps,
colic, nausea, vomiting and any undiagnosed abdominal pain.

Shivering
1. Can occur postoperatively and is not associated with fever or infection,
but as a reaction to general anesthesia (22-50% of cases). The incidence
seems to be increased in prolonged cases and in those involving large amounts
of blood loss, fluid administration, or both. Shivering may also be seen after
epidural anesthesia. Shivering is associated with several undesirable
physiologic effects. Oxygen consumption, carbon dioxide production, and
metabolic rate may increase as much as 500% above baseline levels
a. Treatment is IM injection of 12.5-25mg of Demerol
b. Bear hugger forced air warmer

DVT
1. Lower extremity surgery plus the use of pneumatic tourniquets for
hemostatic control, immobilization, obesity, bedrest, and oral
contraceptives are all predisposing factors
a. Symptoms: Fever (101 °F) after 24 hours, calf tenderness
b. Signs: Swelling of the calf in the later stages
c. Diagnostic tests: Duplex scan, venogram

NOTE* A painful swollen leg and a positive duplex scan are criteria to initiate
treatment for a DVT. However, a swollen/painful leg plus a questionable scan
requires a venogram for adequate criteria to initiate treatment

NOTE* Fever may be the only initial warning but also occurs with other
postoperative problems, therefore, must work up the patient completely prior to
initiating treatment

d. Treatment: After drawing blood for a coagulation profile, treat with Heparin
IV 5000-10,OOOU followed by a constant infusion of 1000-1500U/HR. The PTT
is kept at 2 times the baseline and the PT is kept at 1 and 1/2 the baseline.
Coumadin is also utilized (to help prevent pulmonary emboli) and should be
started upon admission, to the hospital as it takes 3-5 days to start working.
The patient can eventually be discharged on p.o. Coumadin

Compartment Syndrome

Two criteria must be fulfilled for this diagnosis to be made:


a. a space that is limited by fascia, skin, or bone must be present
b. increased compartment pressure caused by a decrease in compartment
size or an increase in the size of the contents within that compartment must
be present

Usually diagnosed in the arm and leg, also occurs in the foot, and can follow
several types of injuries, most commonly multiple fractures or crushing
injuries. This entity should be considered in the differential diagnosis in
patients presenting with a painful swollen foot post trauma

1. Definition: Increased compartmental pressure resulting in decreased


perfusion and ultimate ischemic changes to the tissues on the compartment.
This can eventually result in contractures and poorly functioning limbs.
a. Physiology: At rest the intramuscular pressure is approximately 5 mm Hg.
During a muscular contracture the pressure can increase up to 150 mm Hg or
more. At relaxation, the compartment pressure rapidly drops, and within 5-10
minutes, has returned to baseline. With a compartment syndrome, there is no
drop of pressure

2. Types of compartment syndrome:


a. Acute: Occurs when the resting pressure in the compartment exceeds the
available perfusion pressure. This is usually the result of trauma with
hemorrhage or gross muscular edema causing the increased compartmental
pressure. If untreated tissue necrosis is inevitable
b. Chronic: Occurs when the resting pressure is higher than the normal resting
pressure but not so high as to cause hyperprofusion. Following excercise, the
time for pressure to return to baseline is protracted. This results in a relative
prolongation of the ischemic time resulting in symptoms during or following
exercise. Actual muscle necrosis is unusual

3. Diagnosis: Measurement of an increased intramuscular pressure in the


compartment via a wicks catheter (usually greater than 30mmHg)

NOTE * Heppenstal demonstrated that the key element of an alteration in


aerobic metabolism was the difference between the Mean Arterial Pressure
(MAP) and the Intramuscular Pressure (P). He determined this difference as "p.
The lowest "p to maintain normal metabolism was 30mm Hg in normal and 40
mm Hg is traumatized muscle. The formula used to determine ^p is as follows:

MAP = Diastolic + 1/3 pulse pressure MAP


P = Compartment pressure -P
^P
Case 1: A 30 year old female suffered a crushing injury to her foot. Her blood
pressure was 140/80. The patient was essentially stable, but had a painful
and swollen foot. X-ray examination was normal.

With these findings a compartment syndrome is suspected. A Wicks catheter


measured pressure in the central compartment at 50 mmHg. Therefore, does a
compartment syndrome exist?

According to the above Heppenstal's formula a compartment syndrome does


not exist in this case. Using the above formula, the ^p in this case= 45. This
value is greater than 40 and therefore the patient does not have a
compartment syndrome.

Therefore, the most accurate way of determining the presence of a


compartment syndrome would be to measure the compartment pressure and
then use Heppenstal's formula.

4. Clinical Findings:
a. Pain out of proportion to the clinical findings
b. Paresthesias
c. Pulselessness
d. Or none of the above

Note* The patient might present with a pulse because the vascular collapse
occurs first at the arteriolar level

5. Associated complications:
a. Comminuted fractures
b. Severe soft tissue injuries
c. Post-ischemia swelling
d. Intramuscular hematomas associated with bleeding diathesis
e. Crush injuries

6. Compartments of the foot: 4


a. Medial compartment: Its borders are the medial and lateral intermuscular
septum, the medial portion of the plantar aponeurosis, the tarsus
(proximally) and shaft of the first metatarsal (distally). It contains the abductor
hallucis flexor hallucis brevis, and the FDL tendon
b. Central compartment: Its borders are the medial and lateral intermuscular
septum, the central portion of the plantar aponeurosis, the tarsus (proximally)
and interosseous fascia (distally). It contains the flexor digitorum brevis, FDL
tendon with lumbricals, quadratus plantae, adductor hallucis, PT and peroneal
tendons
c. Lateral compartment: Its borders are the lateral intermuscular septum,
lateral portion of the plantar aponeurosis, and the associated osseous
components. It contains the abductor digiti minimi, flexor digiti minimi, and
opponens digiti
d. Interosseous compartment: Its borders are the metatarsals and the interossei
fascia. It contains the interossei

7. Treatment:
a. Fasciotomy
1. Double dorsal technique

NOTE* Midfoot and forefoot: 2 dorsal longitudinal incisions, one over the 2nd
metatarsal and the other over the 4th (deepened down to the metatarsal shaft)
where a hemostat is passed into each adjacent interosseous space, where the
interosseous muscle is stripped from its corresponding metatarsal. The wound
is closed secondarily in 5 days. If closure cannot be completed, then skin
grafting is employed

ii. Extensile medial incision


iii. Combined approach

Hemorrhage
The incidence of major hemorrhage is low considering the total number of
surgeries done in the U.S. The use of blood products is an important decision
that any surgeon must make considering the impact of AIDS seen in
transfusion medicine today.
1. Physiology of anemia:
a. Sufficient amounts of Hb and cardiac output must be present to supply
tissue demands
b. A 20-30% volume loss leads to hypotension and shock, the end result is
tissue hypoxia
c. As the hemoglobin decreases the cardiac output increases (cardiac output is
increased as stroke volume increases)
d. The hematocrit level requiring transfusion is still under debate. However, it is
considered advisable to consider transfusion when the Hb is 7 Gm/dl or less.
Some will transfuse if Hb is under 10 Gm/dl, which is an HCT of 30%

2. Types of transfusion (see chapter, Fluid Replacement)


a. Autologous blood:
i. Preoperative donation: must be done 1 week prior to surgery
ii. Euvolemic hemodilution: removal of the RBC mass at the time of surgery and
replacement with either colloid or crystalloid to maintain intravascular volume
iii. Cell salvage: blood is collected by automated cell washing devices ( it is
contraindicated in cases where infection is present and where cancer cells may
be encountered). The automatic cell washers contain clear plasma and
platelets, and there is no clotting function transfused with the product
b. Homologous blood: Blood bank blood

3. Complications:
a. WBC-mediated allergic type reaction, or immune reaction to minor group
factors
b. Human error causing major cell lysis, mistyping, and hemolytic transfusion
reactions
c. Disease transmission
i. Hepatitis B and C are greatest risks
ii. AIDS (1-50,000 transfusions reported)

NOTE* See chapter 11 Fluid Management, section Blood Bank

Septic Syndrome
In patients who have had recent major surgery, prompt and accurate diagnosis
of sepsis often is difficult. Many of the features commonly attributed to sepsis
(fever, leukocytosis) are normal phenomena in the postoperative period 1.
Criteria: The most important criteria is evidence of an infection based upon a
high index of clinical suspicion using the following
a. Fever > 101 °F or hypothermia < 96°F
b. Heart rate > 90 beats per minute
c. Respiratory rate > 20 breaths per minute
d. Evidence of organ dysfunction
e. A PaO2 < 75 mmHg
f. Elevated plasma lactic acid levels
g. Altered mentation

NOTE* Among the most devastating complications seen in bacteremic shock is


the development of adult respiratory distress syndrome (ARDS)

2. Treatment:
a. Empiric broad spectrum antibiotics in maximum doses

Thyroid Storm
This represents the extreme state of decompensated
thyrotoxicosis. Thyrotoxic patients inadequately prepared are at great risk for
developing thyroid storm from the stress of surgery
1. Signs and symptoms:
a. Tachycardia
b. Fever
c. Arrhythmias
d. Agitation, psychosis, coma (CNS always affected)
e. GI symptoms (abdominal pain, jaundice vomiting
f. Dyspnea
2. Treatment:
a. Propranolol 1-2 mg IV with continuous cardiac monitoring. Repeat every 5
minutes until the pulse rate drops to 90-110
b. PTU 300 mg PO q6h (antithyroid drug: propylthiouracil)
c. Saturated solution of potassium iodide 10 drops PO q8h
d. Hydrocortisone 200 mg IV q8h
e. General supportive therapy with acetaminophen, IV's and peripheral cooling

3. Prevention:
a. Good history
b. Appropriate lab tests (T3, T4, TSH, serum cholesterol)
c. Endocrinology consult to determine if thyroid disease is primary or secondary
and to monitor treatment
Chapter 6: Infectious Diseases
Specific Diseases
Principles of Antiinfective Therapy Antibiotics
Drug Fever
Specific Antimicrobial Therapy Surgical Considerations
Necrotizing Infections
Osteomyelitis
Septic Arthritis
Mycology
Viral Diseases
Rickettsial Diseases
Protozoan and Metazoan Infections
INFECTIOUS DISEASE
Specific Diseases
1. VIRAL HEPATITIS: types A, B, non-A, non-B, C, E, and delta
a. Hepatitis B, non-A and non-B cause chronic liver disease that can lead to
cirrhosis

NOTE* ALL HEALTH CARE WORKERS SHOULD BE INOCULATED AGAINST


THE HBV- IT IS NOW AVAILABLE AS A 3 PART INOCULATION, FOLLOWING
A SIMPLE BLOOD TEST CHECKING FOR HEPATITIS B ANTIBODIES

b. Hepatitis A (Infectious hepatitis) is common and occurs by oral fecal


contamination or ingestion of contaminated food
c. Hepatitis A is usually diagnosed by finding IgG anti-HAV within 2-3 weeks of
illness, and the lgG anti-HAV thereafter
d. Hepatitis B may be associated with a chronic carrier state and chronic liver
disease, with IV drug abuse and homosexuality being common risk factors (
10% of these patients are asymptomatic and become carriers, they can transmit
the virus to others who can get liver disease)
e. Hepatitis non-A and non-B account for 90% of transfusion related hepatitis
f. Drug induced hepatitis by the following:
ACETAMINOPHEN, HALOTHANE, ALPHAMETHYLDOPA, ISONIAZID,
DIPHENYLHYDANTOIN AND PHENOTHIAZINE
IT IS TREATED BY THE WITHDRAWAL OF THE DRUG
g. ACETAMINOPHEN is widely used and can cause irreversible liver damage
when more than 10 grams is ingested (can cause more damage in the alcoholic
or malnourished patient)
h. N-ACETYLCYSTEINE given p.o. within 24 hours of ingestion of
ACETAMINOPHEN enhances the liver's ability to clear the metabolites and
prevent toxicity

2. AIDS: HIV
a. HIV is one of a number of RNA retroviruses possessing a unique enzyme,
REVERSE TRANSCRIPTASE, which allows the virus to synthesize DNA from
RNA or synthesize DNA backwards; as RNA viruses they can make DNA that
may interfere or incorporate into the DNA of the host cell
b. TARGET-THE IMMUNE SYSTEM:
i. HIV infection follows viral recognition of certain molecular receptors (CD-4) on
the surface of human cells, both the T4 lymphocyte and on the macrophages
and monocytes (This is the key cell surface glycoprotein which the AIDS virus
recognizes and renders those cells carrying this marker susceptible to infection).
As a consequence the HIV infection can lead to immunodeficiency
ii. With depression of the circulation of the T-helper lymphocyte population,
host response is impaired to a variety of potential pathogens: bacteria,
mycobacteria, viruses, fungi and helminths
iii. It is known that 40-50% of individuals infected with HIV and having less
than 400/mm circulating T-helper cells will develop the manifestations of AIDS
within 2 years, 80-85% will develop manifestations of AIDS if their T-helper
lymphocyte count is less than 200/mm
iv. Also associated with depression of the T-helper lymphocytes, the natural-
killer (NK) lymphocytes' function are also disrupted (these NK cells are involved
in immune surveillance against neoplasms and virusinfected cells)
v. Also associated with depression of the T-helper lymphocytes is depression of
immunoglobulin (antibody) producing B-lymphocyte, which ultimately leads to
the body's decreased responsiveness to vaccines
c. TARGET -THE CNS: HIV also targets the central nervous system leading to a
subacute encephalitis (AIDS encephalitis)
d. CLINICAL MANIFESTATIONS: A classification for the clinical spectrum of HIV
for adults has been proposed by the C.D.C.: thinking of the potential
consequences of the virus infection on a time line
i. GROUP I- Includes people manifesting acute HIV, transient and selflimited,
characterized by fever, rash, malaise, lymphadenopathy. Incubation period 4-6
weeks after exposure (can be longer). The illness is characterized by serologic
conversion to HIV (+ anti-HIV antibody test)
ii. Group II- Asymptomatic infection with a positive serology and virus culture, a
positive serology but no manifestations of CNS disease (called a Latent state)
iii. Group III- Persistent generalized lymphadenopathy, palpably enlarged lymph
nodes (> 1 cm) at 2 or more extra-inguinal sites persisting for more than three
months in the absence of any other cause: May have other manifestations of the
disease
iv. GROUP IV- Other diseases; individuals in this group have one or more
significant immunodeficiency or CNS disease, divided into 5 .subgroups
according to manifestations:
 Subgroup A: Constitutional disease (fever, weight loss)
 Subgroup B: Neurologic disease (progressive dementia, peripheral
neuropathy and myelopathy)
 Subgroup C: Infectious diseases (pneumonia, histoplasmosis, herpes
simplex, cryptococcosis, et. al.)
 Subgroup D: Secondary cancers ( Kaposi's sarcoma, non-Hodgkins
lymphoma)
 Subgroup E: Other diseases (chronic lymphoid interstitial pneumonitis)
e. CLINICAL MANIFESTATIONS BY ORGAN SYSTEMS:
i. General- fatigue, weight loss, fever and sweats
ii. Dermatologic- rashes (particularly fungal), seborrheic dermatitis, psoriasis,
nail changes and pigmented lesions
iii. Pulmonary- unexplained cough and shortness of breath
iv. Head/ENT- persistent sinus congestion, sore throat, discoloration of the oral
mucosa and tongue
v. Gl- diarrhea
vi. Neurological- headaches difficulty comprehending, behavioral changes,
numbness or tingling
f. TESTING: Seroconversion to a positive HIV antibody test may occur from 4-8
weeks following the acute illness (fever, fatigue, diarrhea, weight loss, night
sweats, and generalized lymphadenopathy).
i. The ELISA (enzyme-linked Immunosorbant Assay) is the primary screening
test to detect the antibody to HIV-1
ii. The Western Blot (Immunoblot) test is used to confirm the validity of the
ELISA test.
iii. A new detection test called the Polymerase Chain Reaction (PCR)
demonstrates amplified proviral DNA in lymphocytes, months to years before
antibody can be detected by ELISA or Western Blot technologies

3. SYPHILIS: a contagious systemic disease caused by the spirochete


Treponema pallidum, characterized by sequential stages and by years of
symptomless latency
a. Primary syphilis: the primary lesion or CHANCRE appears within 4 weeks of
infection and can be seen on the penis, anus, rectum, the vulva and cervix in
women, and also on the lips, tongue and buccal mucosa
b. Secondary syphilis: cutaneous rashes ( of the palms and soles-macules,
papules or pustules- ranging in color from pink to pale red) appear within 6-12
weeks after infection and most are florid after 3-4 months. Some patients
exhibit lymph node enlargement. There are constitutional symptoms of malaise,
headache, anorexia, joint pain and nausea. At this stage some develop syphilitic
meningitis
c. Tertiary syphilis: can be divided into benign tertiary syphilis of the skin bone
and viscera, cardiovascular syphilis and neurosyphilis. The typical lesion is a
GUMMA (a chronic granulomatous reaction that leads to necrosis and fibrosis),
appearing most commonly just below the knee, the upper trunk, the face and
the scalp, but can occur anywhere in the body
d. Tests:
i. VDRL: (a flocculation test where a reagin antibody in the patient's serum
reacts visibly with CARDIOLIPIN, the antigen). Reported as REACTIVE, WEAKLY
REACTIVE, BORDERLINE OR NON-REACTIVE
ii. FTA-ABS: fluorescent treponemal antibody absorption, MORE SPECIFIC
THAN THE VDRL
iii. Darkfield Microscopy: demonstration of the spirochete taken from exudates
iv. Other tests: For secondary and tertiary syphilis can include, biopsy of
lesions, echocardiography and radiology, EKG and CSF
e. Treatment: Penicillin (for all stages) with a serum level of .03 IU/mL for 6-8
days for infectious syphilis.

NOTE* Treat as follows: Benzathine penicillin 1.2 million units IM in each


buttock (primary) with two additional injections Q 2 weeks of above
(secondary). Benzathine penicillin 2.4 million units IM Q 1 week X 3 weeks
(benign tertiary)

4. GONORRHEA: Caused by Gram (-) diplococcus, Neisseria gonorrhea. Has


arthritic and dermatological manifestations (hemorrhagic bullae).
a. Diagnosis: You must use a special collection kit for a C & S that generates
carbon dioxide, called a Jembec plate. Otherwise you must take your specimen
and plate it immediately on chocolate agar, since the organism is so fastidious.
Treatment of uncomplicated urethral gonorrhea is via Ceftriaxone, 250 mg IM X
1 plus Vibramycin 100 mg BID X 7 days (there are also other alternatives

5. MENINGITIS: Most bacteria cause an ACUTE SEPTIC MENINGITIS, but TB


and syphilis cause SUBACUTE MENINGITIS. Viral infections cause ACUTE
ASEPTIC MENINGITIS, while fungal infections and malignancies cause
SUBACUTE ASEPTIC MENINGITIS
a. Acute Septic (Bacterial) Meningitis: 3 bacteria account for 80% of all cases,
NEISSERIA MENINGITIDIS, H. INFLUENZA AND STREP PNEUMOCOCCUS (H.
flu occurs in most children > 1 yr and S. pneumo most common cause in
adults)
b. Signs and symptoms: A sore throat can precede fever, headache, stiff neck
and vomiting which characterize ACUTE MENINGITIS. Adults and children
become desperately ill within 24 hours
c. Diagnosis: Since acute bacterial meningitis, especially meningococcal, can be
lethal in hours, accurate diagnosis and treatment are urgent, therefore any
unexplained fever in infants between age 3 months and 2 years of age warrants
close monitoring and, if necessary, lumbar puncture. When bacterial meningitis
is suspected, antibiotics should not await the results of diagnostic tests.

NOTE* A petechial or purpuric rash can occur in generalized septicemia. Check


blood cultures, CT scan, lumbar puncture or any secretion or serologic test
looking for the bacteria or bacterial antigen

d. Treatment: Antibiotics as per culture and sensitivity, to continue 1 week after


fever subsides
e. Differential diagnosis: Subacute bacterial endocarditis (microbial infection of
the heart valves)

6. SUBACUTE BACTERIAL ENDOCARDITIS: A previous episode of infective


endocarditis, a history of rheumatic valvular disease, patients with congenital
heart lesions (Mitral valve prolapse + murmur) and prosthetic valves are high
risks for developing endocarditis infections. Therefore, should these patients
should be prophylaxed with antibiotics for clean podiatric surgery? According to
Warren Joseph, D.P.M., there are NO recommendations by A.H.A. for
prophylaxing in clean orthopedic surgery
a. Causative organism: Strep more common than Staph aureus
b. Signs and symptoms: Sudden. onset of fever, chills, weakness and often
times aching joints. Findings include petechiae of the oral mucosa and skin,
especially the trunk and painful fingertips and erythematous
lesions of the palms and soles (Janeway's lesions). The most common clinical
finding is a heart murmur
c. Diagnosis: Is by blood culture and echocardiogram (to see vegetations on the
valves). Obtain 6 different blood culture specimens at different fever spikes

7. LYME DISEASE: This is a tick (Ixodes dammini) transmitted, spirochetal


(Borrelia burgdorferi) inflammatory disorder best recognized clinically by a skin
lesion, erythema chronicum migrans (Stage 1), red macule or papule that
expands often with a central clearing to as large as 50 cm. Accompanying the
skin lesion, or preceding it are malaise, fever, chills, headache and stiff neck.
Neurologic abnormalities (Stage 2) like Bell's palsy and arthritis (Stage 3),
especially the knees, can develop in patients within weeks to months of the skin
lesion.
a. Differential diagnosis: In children juvenile RA and adults Reiter's syndrome
and atypical RA
b. Treatment: In adults, tetracycline 250mg QID for 10-20 days and for children
and pregnant women, penicillin V 500 mg QID (early stage)

8. TETANUS: Tetanus is caused by an exotoxin (tetanospasmin) elaborated by


Clostridium tetani, a gram positive, anaerobic, bacillus. The toxin enters the
CNS along the peripheral motor nerves or may be blood borne to the nervous
tissue, and binds to the ganglioside membranes of nerve synapses. Once fixed
the toxin cannot be neutralized.
a. Signs and symptoms: Stiffness of the jaw, difficulty in swallowing,
restlessness, headache, fever, sore throat and later difficulty in opening jaws,
spasm of facial muscles with elevated eyebrows
b. Prognosis: Mortality is high in the young and old patient and drug abuser
c. Prophylaxis: At the time of injury, 0.5 mL of toxoid elicits a protective
antibody level in a previously immunized patient; this booster is not necessary if
is it known beyond doubt that the patient has received a booster within the past
5 years. An inadequately immunized patient should be given tetanus immune
globulin (human) 250-500 u. IM depending upon the wound potential. At the
same time, the first of 3 0.5 mL doses of absorbed tetanus toxoid should be
given sub-cutaneously at another site.
The second and third doses are given at monthly intervals.

Tetanus Immunization Guidelines*


1. If the patient never immunized and the patient has a dirty wound:
a. TIG (tetanus immune globulin) 250 U IM and
b. dT (diptheria tetanus toxoid) IM
2. If immunized and the patient presents with a dirty wound
a. Give dT booster if the last booster was greater than 5 years in the past
3. Every adult should have dT boosters every 10 years

*Center for Disease Control recommendations

9. RABIES: Is an acute infectious disease of mammals, especially carnivores,


characterized by CNS irritation followed by paralysis and death. Rabid animals
transmit the infection by biting animals or humans. Rabies can also be
transmitted by exposure of a mucous membrane or fresh skin abrasion to
infected saliva.
a. Diagnosis: Examination of the infected animal's brain for Negri bodies, or if
the animal is healthy and when practical, observation of the animal by a
veterinarian for 10 days.
b. Prophylaxis: see guide to follow

POSTEXPOSURE ANTIRABIES GUIDE


Animal and Its Condition
SPECIES CONDITION AT TIME OF ATTACK TREATMENT OF EXPOSED HUMAN
Wild animal Regard as rabid unless RIG followed by HDCV
(skunk/fox) proven negative
Domestic dog/cat Healthy & available for None
observation
Escaped (Unknown) Consult with public health
officials
Rabid or suspected rabid RIG followed HDCV

NOTE* RIG= Human rabies immune globulin, gives passive immunization


HDCV= Human diploid cell virus, gives active immunization (Series of 5 IM
deltoid area injections @ day 1,3,7,14 and 28)

c. Wound Care: The wound should be cleansed immediately and thoroughly


with a 20% solution of medicinal soft soap or 1 benzalklonium chloride. Deep
puncture wounds should be flushed not scrubbed, using a catheter and soapy
water (25 ga rubber angiocatheter needle on a large syringe). Do not cauterize or
suture the wound. For large defects, pack open. (This treatment holds true for
all bite wounds, except add Augmentin/tetracycline/Cipro to cover
prophylactically against sepsis)

PRINCIPLES OF ANTIINFECTIVE THERAPY


"The decision to institute antibacterial therapy requires a careful assessment of
the probability of infection; the likely cause of infection; the probable
susceptibilities of the infecting organism to the available antibiotic drugs; and
host factors likely to influence the presentation, clinical course, and outcome of
the infection. If infection is not currently present but the clinical circumstances
in the host suggest that the risk of infection is high, PROPHYLACTIC
antimicrobial therapy may be implemented. If infection is suspected but not
proved, EMPIRIC therapy designed to treat the most likely pathogens is
initiated, until an accurate diagnosis can be made. If the site and causative
organism are known then DEFINITIVE therapy can be initiated".*

*Zier BG, Essentials of Internal Medicine in Clinical Podiatry, Philadelphia, Saunders, 1990, pg
327

1. INFECTION: is contamination above 105 bacteria per gram of tissue in


a healthy adult and as little as 102 in the presence of an artificial joint. A
developing coagulum begins to surround bacteria after 3 hours, protecting it
from antibiotic lavage and prophylactic antibiotic doses
NOTE* Infection is a clinical diagnosis. It is not defined by some artificial lab
finding that most labs do not even do. Just because a contamination is above
105 doesn't mean you cannot close an open wound. It is a clinical evaluation,
not a laboratory one. This numerical value (105) is only utilized here as it has
appeared on numerous board examinations

NOTE* There have been studies proving that 70% of all implants have slime
forming bacteria present in large numbers, yet there is no infection. Therefore
the value as mentioned 102 is only mentioned as a reference point for the
board examinations

a. Diagnosis: Is made by a combination of factors to include signs and


symptoms, prior therapy, medical history, allergies, social and travel history,
physical examination (includes vital signs) and laboratory diagnosis
i. Physical Findings: Warmth and erythema along the incision or wound with
possible exudates

NOTE* The 5 cardinal signs of infection are: RUBOR, TUMOR, DOLOR,


CALOR, and FUNCTIO LASEA

ii. Systemic signs consistent with fever > 100 degrees F or 37.7 C, with
lymphadenopathy, lymphangitis, shakes, chills and malaise

iii. Lab Studies- X-rays, Bilaterally (always serve as a base line for future
reference)

Gas in tissues indicates pathology (could also be H202 lavage, if utilized)


Osteomyelitis takes at least 10-14 days to show on x-ray
Bone scans/ Gallium scans/ Indium scans (IF OSTEO IS SUSPECTED)
CBC & Diff/ SMA 18-24/ ESR/ Urinalysis
Gram Stain (Blue/positive, Red/negative)
Aerobic/ anaerobic/ fungal/ acid fast cultures)
Blood cultures (when septicemia is suspected

Note* A left shift is an increase in the number of immature PMN'S or band cells.
When the percentage of bands is greater than 20 or when the percentage of
bands or segs together add up to 80, this is considered a shift to the left.
Infection, toxemia, and hemorrhage will cause a shift left. Liver disease and
megaloblastic and iron deficiency anemias will cause a shift to the right
NOTE* Anaerobic cultures should be obtained in the following circumstances:
a. A putrid odor from the wound
b. A deep abscess, pain without discharge
c. Significant tissue necrosis (gangrened. Gas formation
e. Failure to recover pathogens in the aerobic culture 8 Gram stain
f. The failure of the infection to respond to antibiotics that are not active against
anaerobes
g. Infection secondary to animal or human bite
h. Previous therapy with amino glycosides
i. Black discoloration of blood containing exudates (may fluoresce red under
UV)

NOTE* Anaerobic bacteria are very difficult to culture even in the best of
conditions. The 4 major anaerobes are:
a. Peptostreptococcus (most common)
b. Peptococcus
c. Clostridium
d. Bacteroides

NOTE* A gram stain can be performed within 10 minutes and provides gross
identification of the pathogen to allow for proper selection of antibiotics. A
culture is most accurate and also checks the resistance of pathogens to the
antibiotics. However, a culture does not diagnose an infection, that is a clinical
finding.

NOTE* For osteomyelitis, the most definitive diagnosis is by bone biopsy. With
an ascending cellulitis and no area of drainage, there is a less than 10%
chance of obtaining a pathogen by injecting .5cc sterile saline (without
bacteriostatic agent) along the leading edge of the cellulitis and reaspirating the
solution back into the syringe and sending it for culture. If septicemia is
suspected, blood cultures should be ordered from 2 different sites at the start
of a fever spike.

2. Indications for hospitalization:


a. Systemic manifestations (fever/shakes/chills, lymphangitis,
lymphadenopathy)
b. Debilitated host ( Diabetic, p.v.d., alcoholic, immuno-suppressive therapy,
burn patients)
c. Need for IV antibiotics (Resistant organism, risk of deep space infection, deep
space necrosis or wet gangrene)
d. Osteomyelitis (most times)
e. Need for extensive surgical debridement
f. Failure of outpatient therapy
3. Indications for consultations:
a. For medical management of systemic disease
b. Utilizing a toxic antibiotic
c. Gram negative septicemia
d. Patient on immuno-suppressive therapy
e. Patient with abnormal renal/hepatic function

4. Management of soft tissue post-op Infections:


a. Decide on hospital or outpatient therapy
b. Consider ID consult/ internal medicine consult/ vascular consult prn
c. Perform local wound care/release and remove sutures/ incision and drainage
as necessary
d. Send specimen for gram stain and aerobic/anaerobic/fungal/acid fast
cultures (as necessary)
e. Obtain other necessary lab studies
f. Begin antibiotics as per gram stain & clinical evaluation and judgement/
reevaluate as per C & S
g. Continue appropriate antibiotic at least 10 days (especially when treating
strep, to prevent post strep glomerulonephritis)

5. Preoperative antibiotics are indicated in the following circumstances:


Valvular disease (when indicated), surgery in the presence of an artificial joint
or the implantation of a new one, contamination, prolonged procedure or
extensive dissection, debilitated host and patients with shunt
a. Consider using: Ancef 1 gm IV preoperatively (can also use postoperatively, 1
gram Q8H in 2 additional doses prn)
b. If penicillin allergic- then Vancomycin 500mg tot gm IV, or Clindamycin
300mg po pre-op
c. Prophylactic antibiotics in the diabetic: Postop infections in diabetics are
usually monomicrobial. Most common pathogens are Staph aureus, Staph epi,
Strep pyogenes and Pseudomonas.
Best drugs are 1st gen. cephalosporins, vancomycin or clindamycin in the
penicillin allergic patient
d. Antibiotics should be administered no less than 5 minutes before inflation of
the tourniquet and no more than one hour before surgery.

Note* The current literature states that there is no significant difference in


postoperative infection rates between one dose, two dose, three dose or one
week of prophylaxis

6. Identification of organisms by morphology (Gram stain):


Organism Morphology
 Gram Positive: Staphylococcus Cocci in grape-like clusters
Streptococcus Cocci In chains
Clostridium Bacilli (rods) with a "racquet shape"
Corynebacterium Rods In "Chinese characters"
 Gram Negative: Neisseria Diplococcus (usually Intracellular)
Pseudomonas Slightly curved rod
Haemophilus/Pasturella Coccold rods
E. Coll/Shigella/Serratla/
Enterobacter/Bacteroldes Straight rods

Antibiotics

The best antibiotic is the one that has the narrowest spectrum, the safest, the
cheapest

1. The Penicillins
a. Penicillin G: Parent compound introduced in the 1940's
i. Good gram(+) and weak gram(-) coverage
ii. Fallen out of favor since many resistant strains (Staph 100% Beta lactamase
producing)
iii. 1 mg PenG= 1667 units
iv. Available as Aqueous (10-30 million u/day) and Procaine (600,000 u Q1 2h)
b. Penicillin VK:
i. Used in severe erysipelas and rheumatic fever prophylaxis
c. Methicillin:
i. For PCN-ase resistant organisms
ii. IV form only
iii. Can cause thrombophlebitis
d. Oxacillin/Dicloxicillin/Cloxacillin/Nafcillin: (PRP's)
i. PCN-ase resistant
ii. Good gram(+) coverage
iii. Oral form can cause diarrhea
iv. Requires frequent dosing, Q4-6 hours
e. Ampicillin: Increased gram(-)coverage
i. Not PCN-ase resistant
ii. Used with UTI, typhoid fever and salmonella infections
iii. Used pre-op for endocarditis prophylaxis
iv. Used in combination with aminoglycosides for gram(-) septicemia
f. Carbenicillin: The original anti-pseudomonal penicillin
i. Can be combined with aminoglycoside for pseudomonas infection
ii. Not used much now since has high sodium content, hepatotoxic,
neurotoxic and causes bleeding disorders
iii. Oral form: Geopen (UTI's only)
g. Ticarcillin: A 4th generation penicillin active against pseudomonas
i. 2-4 times more potent than carbenicillin vs pseudomonas
ii. Has increased anaerobic activity
h. Piperacillin: As above, gram(+) 9 (-) activity
i. Azlocillin: As above but superior to ticarcillin/piperacillin vs pseudomonas
aeruginosa
i. Neurotoxic/Hepatotoxic
j. Mezlocillin (Mezlin): A 4th generation penicillin with good gram(-) and
anaerobic activity
i. Can be used for Pseudomonas/B.fragilis

NOTE* The adverse effects of the penicillins can manifest as hypersensitivity,


hematologic, renal, hepatic, G.l., and C.N.S. reactions.
a. Hypersensitivity (approaches 10%):
Type 1: anaphylaxis, laryngeal edema, accelerated urticaria
Type 2: Coombs positive hemolytic anemia
Type 3: serum sickness; urticaria after 72 hours
Type 4: contact dermatitis usually after accidental exposure
b. Hematologic:
Neutropenia: nafcillin/carbenicillin
Platelet dysfunction: carbenicillin
c. Renal:
Interstitial nephritis/cystitis: methicillin
Electrolyte imbalance: ticarcillin/carbenicillin/ penicillin G
d. Hepatic
SGOT elevation: oxacillin
e. G.1.
N.V.D.: amoxicillin/ampicillin
f. C.N.S.
Seizures (rare): penicillin G

2. The Clavulanates
a. Amoxicillin/clavulanate (Augmentin): Adds clavulanic acid to ampicillin
which inactivates the beta-lactamase enzymes:
i. PCN-ase resistant
ii. Spectrum of activity increased vs gram (-) to include E.Coli & Klebsiella, also
good Staph and Bacteroides coverage
iii. The oral drug of choice for cat, dog and human bites
iv. Dosed at 250-500 Q 8h (for other than endocard prophylaxis)
b. Ticarcillin/clavulanate (Timentin): Has greater gram(-) coverage than any 4th
gen. penicillin
i. Has good gram(+) coverage and covers anaerobes well (i.e. B. fragilis)
ii. Good drug for initial therapy for moderate diabetic foot infections
iii. Has high sodium load/use cautiously in hypertensive-renal pt's
iv. Dosed at 3.1 Q 6-8h (3gm ticarcillin + 100mg clavulanate)

3. The Sulbactams
a. Ampicillin/ sulbactam (Unasyn):
i. Similar to Timentin but has much lower sodium load
ii. Adds sulbactam, a beta-lactam inhibitor
iii. 99% coverage against B.fragilis/not good against
pseudomonas and good against enterococcus
iv. Dosed at 1.5-3g Q 4-6h.

NOTE* Unasyn has better gram (+) coverage than Timentin, but weaker gram (-)
coverage

4. The Tazobactams
a. Piperacillin/Tazobactam (Zosyn)
i. Similar to Timentin in coverage and spectrum
ii. Adds Tazobactum a Beta lactamase inhibitor
iii. Has greater activity than pipericillin
iv. Dosed @ 3.375 gm Q 6 hrs

5. The Cephalosporins
Semi-synthetic compounds derived from the mold, cephalosporum acremonium.
There is a cross reactivity with penicillin allergic patients from 5-20% depending
upon the source. As a whole, these antibiotics
are well tolerated, non-toxic and broad spectrum. They are categorized in
generations, which define their spectrum.

As you go from first generation to third generation you INCREASE GRAM


NEGATIVE COVERAGE AND DECREASE GRAM POSITIVE COVERAGE.

a. 1st generation:
i. Keflin (cephalothin), Keflex (cephalexin), Ancef (cefazolin), Cefadyl
(cephapirin), Anspor & Velocef (cephradine), Duricef (cephadroxil)
ii. ACTIVITY vs gram (+) cocci: S. aureus and epidermidis, Strep pyogenes and
pneumonia
iii. Activity vs gram (-): Proteus mirabilis, E. coil, Klebsiella pneumonia (PECK)
b. 2nd generation:
i. Mandol (cefamandole), Mefoxin (cefoxitin), Ceclor (cefaclor), Zinacef
(cefuroxime), Ceftin (cefuroxime axetil), Monocid (cefonicid), Cefotan (cefotetan),
Lorabid (loracarbef), Cefzil (cefprozil)
ii. ACTIVITY vs gram (+): is variable to Staph , still OK to Strep

NOTE* Zinacef® and Mandol® are better than 1st generation for Staph
coverage

iii. Activity vs gram (-): as with 1st generation (PECK) plus H. flu, Enterobacter
& Neisseria (HENPECK)
c. 3rd generation:
i. Claforan (cefotaxime), Cefobid (cefperazone), Cefizox (cefizoxime), Rocephin
(ceftriaxone), Fortaz (ceftazidime), Suprax (cefixime), Vantin (cefprodoxime
proxetil)
ii. ACTIVITY vs gram (+): variable to both Staph and Strep
iii. Activity vs gram (-): as with 2nd generation (HENPECK) plus Serratia,
Morganella, Providencia, Citrobacter and Pseudomonas

NOTE* ROCHEPHIN HAS THE LONGEST HALF LIFE OF ANY 3RD


GENERATION CEPHALOSPORIN, THEREFORE CAN BE DOSED AT ONCE A
DAY.
FORTAZ and CEFOBID HAVE THE BEST ACTIVITY AGAINST PSEUDOMONAS.
METHICILLIN RESISTANT STAPH AS WELL AS GROUP D STREP
(ENTEROCOCCI) ARE RESISTANT TO ALL CEPHALOSPORINS

NOTE* Probenecid 500 mg QID blocks tubular secretion doubling the antibiotic
concentration of penicillin and cephalosporins

NOTE* The top 8 questions and answers about cephalosporins:


1. Are the 1st generation cephalosporins more effective against grampositive
organisms than 2nd or 3rd generation cephalosporins? The most commonly
used 1st generation cephalosporin is cefazolin. This drug is excellent against
group A streptococci, pneumococci but it less effective against these gram
positive organisms than some of the 2nd and 3rd generation cephalosporins.
3rd generation cefotaxime, ceftizoxime, and ceftrriaxone, are more active
against pneumococci and some hemolytic and non hemolytic streptococci than
the first generation cephalosporins. These 3rd generation cephalosporins inhibit
resistant streptococcus pneumoniae as compared to 1st and 2nd generation
cephalosporins.
2. Why do cephalosporins not inhibit enterococci? Enterococci to not bind to
cephalosporins adequately. Enterococci produce a f-lactamase which makes a
cult wall to enter. Enterococcal superinfection is more common after cefriaxone
or 2nd generation cefuroxime.
3. What is the best way to administer Rocephin? A 2g. dose exceeds the
ability of the drug to bind to protein, therefore, more drug reaches the tissues
during a single 2g. dose than 1g. dose given in two doses.
4. Which of the cephalosporins are effective against Pseudomonas
aeruginosa? Ceftazidime has the highest activity of all the cephalosporins. Also
cefoperazone has activity against many P. aeruginosa but must be given on a 6
hour interval.
5. Can cephalosporins be used in patients who are allergic to penicillin?
Cephalosporins may be administered to patients who have hat only cutaneous
reactions to penicillin. Anyone who has hat anaphylaxis to penicillin should be
skin tested with the breakdown products of penicillin prior to any
cephalosporin administration, ant if no wheal or flare it would be safe to
administer. Most reactions occurred during the 1960's when penicillins ant
cephalosporin were packaged in the same plant, which probably caused cross
contamination. It is also possible to be allergic to cephalosporin ant not to
penicillins.
6. Can children with osteomyelitis be treated with oral cephalexin?
Cephalexin has excellent absorption ant may be used to treat Staphylococcus
aureus osteomyelitis in children. Cephalexin produces very high blood levels
ant is well tolerated by children, Studies show that 4-6 weeks of this therapy is
as effective as parenteral therapy.
7. Which cephalosporins inhibit anaerobes? Clinical studies show that
ceftizoxime ant ceftoxitin are both equally effective against anaerobes which
cause human infections. Those cephalosporins totally ineffective against
anaerobes are ceftriaxone ant ceftazidime.
8. What would be the 1st choice for meningitis at present? When dealing with
Neisseria meningitis in the U.S., penicillin is still effective. If the meningitis is
caused by H. influenza or by S. pneumoniae, the choice of therapy would be
cefotaxime or ceftriaxone. In the newborn cefotaxime combined with an
amino glycoside toes not cause the alteration in gut flora that ceftriaxone toes.
For S. marrewscens ant Acinobacter, ceftizoxime is the 3rd generation
cephalosporin of choice. Ceftazidime is useful in treating meningitis caused by
P. aeruginosa.

6. Other Beta-Lactams:
a. Imipenem/Cilastatin (Primaxin): Is an extremely potent antibiotic with the
broadest spectrum of an available beta lactam including anaerobic coverage/
most expensive antibiotic on the market. Cilastatin is added to prevent renal
hydrolysis (destruction of imipenem)
i. May be the drug of choice in severe/limb threatening diabetic infections ( as
initial therapy) other than clinda/genta/ampi
ii. Major therapeutic use for Gram (+) cocci and aerobic gram (-) bacilli
iii. A 3% cross sensitivity with penicillin allergic patients
Dosed at 0.5-1 gram Q 6h IV up to 4gm/day
b. Azreonam (Azactam): Is ONLY effective against gram (-) aerobes, including P.
aeruginosa
i. Can be combined with clindamycin in penicillin allergic patients when
gram(+)and anaerobes are suspected

7. Quinolones:
a. Ciprofloxacin (Cipro):
i. Its main benefit is it's p.o. gram (-) coverage
ii. Can be used for methicillin resistant staph but should be combined with
rifampin (300 mg BID) in the treatment of these infections
iii. Contraindicated in its use with children as it can cause cartilage
degeneration
iv. Can be combined with clindamycin (Cleocin) or metronidazole (Flagyl) in the
treatment of diabetic foot infections
v. Oral therapy for osteomyelitis when caused by Pseudomonas
vi. Rarely a first line antibiotic
b. Ofloxacin (Floxin):
i. As with the above, but with better gram (+) coverage
ii. Dosed for soft-tissue infections at 400 mg Q 12 h

8. Aminoglycosides:
a. Streptomycin: (used in treatment of TB)
b. Kanamycin (used as an irrigant)
c. Gentamycin: used with methylmethacrylacte beads (PMMA) for osteomyelitis
and in triple antibiotic therapy for serious infections
d. Potentially ototoxic in patients with renal problems

NOTE* Gentamycin is a good drug for use with PMMA because of its good
water solubility, heat stability, and broad antibacterial spectrum. Most studies
have fabricated PMMA beads utilizing 1-2 g of antibiotic powder to 40-60 g of
PMMA

d. Tobramycin (less ototoxic than gentamycin)


e. Amikacin (reserved for serious infections against aminoglycoside resistant
organisms)
f. As a group these antibiotics have well documented toxicities
(ototoxicity/hepatotoxicity). g. They are essentially anti-gram negative agents,
but do have gram positive coverage. When using these antibiotics it is beneficial
to, have an ID consult and you should perform peak/trough serum levels as
well as creatinine clearance and BUN tests (if BUN elevated increase time span
between doses or lower the dose)
NOTE* Aminoglycosides (gentamycin/tobramycin) must be monitored to keep
the peak level just below 10 micrograms/ml and the trough level below 2
micrograms/ml and Amikacin peak at 20-30 µ/ml and trough less than 10
µ/ml. The peak should be drawn within 30 minutes after infusion of the drug
and the trough level should be determined by drawing another blood sample
15 minutes prior to the next dose. The calculated dose of aminoglycosides
should be given Q 12 or 24 h (never Q 8)

NOTE* Two techniques can be used to change the dose of amnoglycosides


when inappropriate levels are found:
a. Raise or lower the amount of the dose
b. Increase or decrease the time interval
So…. if the peak is high and trough is normal: decrease the amount
if the peak is low and the trough normal: increase the amount
if the peak is normal and trough high: increase the time interval
if the peak is high and trough high: decrease the dose/increase
the time*
Joseph WS Handbook of Lower Extremity Infections, Churchill Livingstone, N.Y., 1990, pg. 182

9. Other antibiotics:
a. Vancomycin:
i. Indicated in penicillin allergic patients or those patients needing coverage
against gram (+) organisms, including methicillin resistant Staph
ii. It is possibly nephrotoxic and should be monitored carefully
iii. Red neck syndrome occurs if infused too quickly (not an allergy)/severe
hypotension can result
iv. Oral form is for pseudomembranous colitis only

NOTE* Peaks and troughs for vancomycin can be done. The peak is 20-30,
and the trough is <10

b. Clindamycin:
i. It is used extensively for anaerobic infections and in the penicillin allergic
patient for gram (+) coverage
ii. Can cause pseudomembraneous colitis
c. Tetracycline:
i. A broad spectrum antibiotic used for rocky mountain spotted fever,
Lyme disease, and H. pylori infection
ii. To be avoided in children and pregnant/ nursing mothers (brown
teeth)
d: Metronidazole (Flagyl):
i. An amebicidal drug also with excellent anaerobic coverage
ii. Can be combined with Cipro for more complete coverage
e. Erythromycin, clarithromycin (Biaxin), azithromycin (Zithromax)

Drug Fever
Fever is considered to be drug induced if no other infections or noninfectious
cause is present on the basis of clinical or laboratory evidence, if there is no
known underlying disease or condition, if the temperature elevation is
temporarily associated with the administration of a "sensitizing" medication and
if the fever disappears within 72 hours of discontinuation of the medication.

The most reliable diagnostic clue is a relative bradycardia or


PULSE/TEMPERATURE DISSOCIATION in patients with 102 degrees F or
above. This pulse/temperature dissociation is unreliable in neonates, patients
on cardiac medications and patients with systemic diseases
accompanied by bradycardia. The treatment is to discontinue the causative
agent and avoid antipyretics.

Specific Antimicrobial Therapy


1. Gram (+) cocci (Penicillinase resistant): When a gram stain report is
received and initial therapy is to be started prior to receiving a C & S the
following should be considered due to the increasing number of betalactamase
organisms: Dicloxicillin 500 mg qid p.o., Nafcillin 1 gm IV Q6h, Ancef 1 gm IV
Q8h, Duricef 500mg bid, Timentin 3.1 gm Q6h IV and Unasyn 1.5-3.0 gm Q6h
IV

2. Gram (+) cocci + penicillin allergy: Vancomycin 500mg or 1 gm Q6 and Q1


2 H IV or Clindamycin 300 Q 6h po, Erythromycin 500 mg Q6h po

3. Methicillin (nafcillin/oxacillin) resistant gram (+) Staph: Vancomycin


500 mg Q 6h IV or 1 gm Q12h, Cipro 500 Q 12h po + Rifampin 300 mg Q1 2h
po (Rifampin synergizes the anti-gram (+) effect of Cipro when in combination
with it), Minocycline, Trimethoprim/sulfa

4. Gram (-): When a gram stain is received and initial therapy is to be started
prior to a C & S the following should be considered: Cipro 750 mg Q1 2h,
Azactam 1 gm Q8h IV, Gentamicin 3-5 mg/kg _IV following a loading dose,
Timentin and Fortaz 1-2 gms Q8h IV, Zosyn (tazobactam/piperacillin) 3.375 gm
Q6 IV

5. Anaerobic coverage: Flagyl 500 mg Q8h po, Clindamycin 600-800 mg Q8h


IV or 300 mg bid-tid po, Primaxin, Timentin, and Unasyn

6. Antipseudomonal coverage: 4th generation penicillin (in combination with


another antibiotic), Fortaz 1-2 gm Q 8h, Azactam 1 gm Q8h, Gentamycin 3-5
mg/kg IV following a loading dose, Cipro 750 mg Q 12h

7. Antifungal coverage: Diflucan 100 mg od, Amphotericin B (very severe side


effects), Ketoconazole (Nizoral), Griseofulvin, and Sporanox 100mg

8. Antihelminthic Coverage: Thiabendazole (Mintezole) for cutaneous larva


migrans and hook worms, and Gamma Benzene Hexachloride (Kwell) for
parasitic skin infestations caused by scabies.
9. Pregnancy: Antibiotics that are probably safe to use are aztreonam,
cephalosporins, erythromycins (not estolate), and penicillins. However,
conversely, the antibiotics that are definitely not safe are the tetracyclines and
metronidazole. Always check with a patient's physician before giving antibiotic
to a pregnant women

NOTE* When administering antibiotics to females on birth control pills, you


must advise them that the activity of these medications are reduced and that
they must utilize some other form of birth control during the time the antibiotics
are utilized.
P.S. Document that you said this in your chart

Note* MIC is measured in microgram per milliliter, the amount of antibiotic


needed to inhibit a specific organism and each organism will have its own MIC
to a given antibiotic
Note* The dosage for children is calculated based on age and body weight
according to Young's Rule, Clark's Rule and Cowling's Rule (see anesthesia
section)

Note* Adjusting the dosage of antibiotics in patients with compromised renal


function is based on a calculation called the Estimated clearance of Creatinine
(ml/mm) is as follows:
ECC (men)= (140-age) X weight (kg)
Creatinine concentration (mg/dl) X 72
ECC (women)= ECC (men)x 0.9

Surgical Considerations
Probably the single most important factor in the control of a postoperative
infection is local care of the wound. Unless all areas of abscess formation and
pus are drained and devitalized tissue excised the adjunctive therapy of moist
dressings, antibiotics etc., will be of little value. The exception to this is a
rapidly ascending cellulitis without areas of abscess formation in which surgical
debridement would be of no value.
1. Outline of General Surgical Principles: try not to use tourniquets, try using
regional anesthesia and not local anesthesia which can spread the infection:
a. Sterile skin prep of foot
b: If a post-op infection, release all skin sutures
c. Explore the extent and depth of the infection, breaking up any loculations if
present, not extending into uninvolved areas
d. If the infection has gone below the subcutaneous layer explore and remove
any deep sutures or necrotic tissue, opening the wound to the deepest portion
of the infection
e. If the infection penetrated the joint capsule, the capsule must be opened, if
an implant is used it must be removed, the bone and cartilage must be carefully
inspected to determine the involvement
f. If an infection extends proximal and distal to a joint both the proximal
phalanx and metatarsal neck must be explored
g. If osteotomies have been done external fixation devices should be removed
h. Once the extent of the infection is determined all devitalized soft tissue and
bone must be removed
i. Samples of deeper tissue and bone, if excised, should be submitted to
pathology for microscopic .examination as well as to microbiology for culture
and sensitivity tests
j. All implant material when removed should be sent to microbiology for culture
and sensitivity tests in order to isolate organisms which may be firmly attached
to the device and not found within the soft tissues k. The wound should then be
copiously lavaged with large volumes of 1% Betadine solution followed by sterile
saline or Ringer's lactate
l. The wound should be packed open with 1/4 or 1/2 inch iodoform gauze for
hemostasis and drainage and covered with gauze soaked in sterile saline or
Ringers lactate. When this dries it acts as a mechanical debriding agent
m. Immobilize and elevate the extremity
n. Keep extremity warm (can use Microtemp heating unit at 102°F)
o. Post surgically, the packing should be changed on a daily basis, and as the
wound starts to granulate less and less packing is necessary p. When
granulation tissue has completely filled the defect, closure of the wound is
attempted, however, for small to moderate surface defects, delayed primary
closure with skin adhesive strips to gradually bring the skin edges together is
attempted within three days after surgical debridement
q. If repeated cultures are negative and clinical appearance of the wound is
clean and well granulated, then the patient can return to the operating room for
irrigation and resuturing of the wound, which is attempted within 4-5 days.
This type of delayed primary closure should be avoided if the defect is large
and/or if the suture closure places excessive tension on the wound
r. If the defect is large the patient may undergo skin grafting when conditions
are right

Necrotizing Infections
The classification of necrotizing infections is difficult. Prompt and oftentimes
empirical medical and surgical therapy must be initiated to save life and limb.
Even with early intervention the rate of amputation and mortality is high, with
all of these patients having some underlying predisposition: trauma, surgery,
burns, malignancy, diabetes mellitus, immunosuppression, human bite
wounds, injection injuries, puncture wounds and PVD.
1. Necrotizing Fasciitis: Widespread necrosis of subcutaneous tissue
YOU MUST TREAT THIS WITHIN 48 HRS
a. Signs and Symptoms:
i. Fever, tachycardia, anemia, shift left and > bilirubin
ii. Hot/edematous/indurated/erythematous extremity
iii. The skin is shiny and smooth with vescicles & bullae filled with a
reddish-brown fluid
iv. Cutaneous anesthesia
v. Later: Tissue slough with grey necrotic subcutaneous fat and fascia b.
Causative organisms- Strep predominates
c. Diagnosis:
i. Extensive necrosis of the superficial fascia
ii. Toxic reaction/altered mental states iii. Absence of muscle involvement
iv. Absence of clostridium as the primary organism
v. No major vascular occlusion is present
d. Differential diagnosis- Lymphangiitis, Clostridial gas gangrene,
synergistic (the aerobes help the anaerobes proliferate) necrotizing
fasciitis/cellulitis and progressive bacterial synergistic gangrene
e. Treatment:
i. Stabilize the patient medically
ii. Penicillin/Gentamycin/Clindamycin or Primaxin
iii. Aggressive incision and drainage with multiple incisions
iv. Daily irrigation and packing

2. Synergistic necrotizing cellulitis/fasciitis is closely related to necrotizing


fasciitis, with a pathogenesis related to diabetes and obesity, however this
condition involves the entire muscle compartment resulting in muscle necrosis.
There are systemic manifestations with presence of the reddish-brown
"dishwater" pus. The gram stain is mixed with gram(+) & (-) and there is a mixed
aerobic & anaerobic infection (Bacteroides/ Peptococcus). The prognosis is poor
with amputation a probability.

3. Clostridial Myonecrosis is also known as gas gangrene or clostridial


myonecrosis. It can present itself after injury, bums or trauma. There is a tense
swelling of the skin with a reddish-brown drainage containing gas bubbles.
There is the appearance of gas on soft tissue x-ray. The muscles undergo
necrosis and exhibit a color change to a reddish-purple. Clostridium perfringens
is usually the causative organism. The diagnosis can be made by gram stain
and Fluorescent Antibody Test. Prognosis and treatment as above, with massive
I & D and IV antibiotics (Penicillin).

4. Nonclostridial Myonecrosis is similar to the aforementioned but slower to


progress. The causative organisms can be anaerobic Strep (Peptostreptococcus)
or a combination of aerobic and anaerobic organisms (synergistic). Treatment is
similar to that of all necrotizing infections.

Osteomyelitis
1. Definitions:
a. Osteomyelitis- infection of bone (chronic OM & acute OM, the distinction of
which will determine treatment regime)
b. Sequestrum- dead necrotic bone separated from the affected bone
c. Involucrum- a chronic process where new bone is laid down around dead
bone
d. Cloaca- an opening in along the cortex from where the pus drains
e. Rarefaction- localized loss of bone density (earliest radiological finding when
30-50% of the osseous mineralization has been lost)
f. Bone abscess ( Brodie's abscess)- localized focus of infection within bone
usually found in the metaphyseal region of tubular bones, but can be
occasionally found in the diaphysis (Chronic/subacute OM)
f. Chronic Sclerosing Osteitis (Garre's)- a low grade infection causing sclerotic
reaction without destruction or sequestration

2. Classifications:
a. Hematogenous (AHO)- a form of OM caused by spread of bacteria via the
bloodstream, originating within cancellous bone, which will result in
radiographic findings that start inside the bone and eventually work out to the
cortex and periosteum. Seen mostly in the metaphyseal region in children
with open epiphyseal plates, occurring most commonly in the calcaneous and
femur. In adults it is most commonly seen in the metatarsal heads
b. Direct Extention- is secondary to trauma or surgery, will first effect the
periosteum, then the cortex and finally the marrow. Proteolytic enzymes destroy
Sharpey's fibers
c. Contiguous- is the spread of infection from contiguous soft tissue to the
underlying bone, also will first effect the cortex and finally the marrow
d. Vascular insufficiency

3. Pathophysiology (Small Vessel Osseous Anatomy)


I. From the last stages of intrauterine life up to the first six months, growth
cartilage is established but not yet limited by bone on the epiphyseal side
a. Vessels from the metaphysis penetrate the end of the anlage, perforating the
pre-existing plate
b. At their ends, those vessels expand forming large venous lakes resembling
metaphyseal sinusoids, which are situated close to the surface of the epiphysis
c. This explains the frequency of infection of the joint and of the epiphyseal side
of the preliminary growth cartilage in the infant
d. Any severe damage to the cells at the epiphyseal side of the growth plate is
irreversible and consequently, joint damage and arrest or disorganized growth
may result in the infant
e. The isolation of the epiphysis from the metaphysis caused by the epiphyseal
plates provides protection both for the epiphysis and the joint. This explains the
rarity of joint infections and epiphysitis in children
II. Physiological factors
a. Intrinsic host defense: Acute hematogenous osteomyelitis (AHO) localizes in
the metaphyseal region because of the phagocytic function in the area of bone.
The normal individual displays abundant phagocytic activity in the medullary
region of bone. AHO occurs in the medullary region of the diaphysis in sickle
cell disease because of inpaired host defense
b. The cortex of the child's bone is thick in the diaphyseal region but thin and
porotic in the metaphyseal region, therefore, an infection
usually spreads laterally through the metaphysis toward and through the thin
porotic cortex.
c. Pus usually spreads through the Haversian system to the subperiosteal
space, with the periosteum being stripped from the cortex: The blood supply to
the cortex is eliminated either by rupturing as the periosteum is elevated or by
bacterial emboli. The periosteum in the child displays 3 differences from the
adult: it is thick, well, vascularized, and loosely attached to the bone.
d. There are 3 commonly involved metaphyses whose adjacent joints are not
safe from an early infection (child). The following are the joints whose
metaphysis is intra-articular: hip joint, shoulder joint and ankle joint (distal
lateral aspect of the tibial metaphysis is intra-articular).

4. Clinical Presentation and Diagnosis


The clinical presentation is variable and may depend on the age of the patient,
the virulence of the organism, the stage and type of disease process.

NOTE* The tenets of treatment include identification of infecting organism,


selection of appropriate antibiotic and surgical excision of dead bone

a. Symptoms (AHO):
i. Local warmth over the infected area
ii. Pain and tenderness on ambulation and palpation
iii. Pseudoparalysis (often the only complaint in kids) and possible subtle gait
changes
iv. Palpable swelling if the infection has ruptured the cortex into the periosteum
v. Ulceration or soft tissue lesion(s) may produce contiguous OM
vi. Concurrent infection: Measles/chicken pox may give strep AHO.
Middle ear infections may give hemophilus/pneumococcus/staph

AHO
b. History & Physical: In pediatric AHO recent infections may be implicated as
causative pathogens
c. Laboratory Exams:
i. Same basic studies as with management of inpatient infection
ii. Leukocytosis or a shift left is not commonly seen in acute or chronic OM
iii. Sed Rate is usually increased (a nonspecific exam useful to monitor the
response to treatment)
d. Radiographs:
i. Always take bilateral x-rays for comparative purposes
ii. The lytic process in bone is not visable on x-ray until at least 30%50% of the
osseous mineralization of the area has been lost
iii. The first bone changes occur 10-14 days following the onset of symptom
iv. Baseline radiographs must be taken- (look at the soft tissue first, may find
edema in the contiguous soft tissues, with swelling of one tissue layer as
compared to another)
v. Three stages of soft tissue changes have been discribed in children with AHO:
small deep local. soft tissue in region of metaphysis, swelling of muscles and
obliteration of lucent plane between them and superficial subcutaneous soft
tissue edema which occurs 3 days after onset of symptoms (if aspirate bone
during this phase, should be able to extract pus).
vi. Additional radiographic findings are: periosteal elevation, metaphyseal
destruction (children), regional osteoporosis, sequestrum and involucrum.
vii. Radiographic findings in chronic OM are:
thickened/irregular/sclerotic bone, elevated periosteum, chronic draining sinus
and sequestrum
e. Aspiration (bone/joint biopsy): Confirms the diagnosis, identifies the
organism and aids in determining the treatment plan.
f. Blood Cultures:
i. Are positive in 50% of cases of septic arthritis and osteomyelitis.
ii. The joint fluid in septic arthritis is sterile in 30% of the cases- look at the
blood cultures.
g. Wound cultures:
i. Sinus tract cultures frequently yield gram(-) organisms which are not
responsible for the underlying bone infection.
ii. If Staph aureus is cultured out, there is a 50% chance that this organism is
producing the associated osteomyelitis.
h. Bone cultures:
i. Take specimens for gram stain, aerobic, anaerobic, fungal and acid fast
cultures
ii. Take a specimen for histology

i. Bone Scans:
i. In most patients with osteomyelitis, bone scintograms become positive within
48-72 hours after onset of clinical symptoms (precedes the x-ray changes by
10-14 days)
ii. Predominant scintigraphic finding is a "hot spot" (increased tracer
localization)
iii. The exception to this are "cold spots" which is due to NO delivery of the
tracer due to necrosis or a fulminating destructive osteomyelitis not
accompanied -by significant reparative processes
 99m Technetium MDP: is currently the most frequently used radionuclide
It is renally excreted, has a half-life of 6 hours, deposits more in trabecular than
cortical bone and provides more anatomic information with less time, exposure
and expense than Gallium 67 imaging. 99m TC MDP may continue to show
abnormal isotope accumulation after infection subsides as a result of continued
bone repair.
 Three Phase Bone Scan: is used to differentiate OM from cellulitis.
Phase 1- at time of injection shows an immediate radionuclide angiogram or
dynamic blood flow, OM and cellulitis both show increased uptake at this point.
Phase 2: 5-10 minutes after injection looking for focal increases (blood pool
image) cellulitis and OM are still positive at this point.
Phase 3: 4 hours after injection (delayed static scan), cellulitis becomes
quiescent at this point.
j. Gallium 67: Almost all IV injected Gallium binds to transferrin, with 1/3
excreted by the G.I. tract and 1/3 by the kidneys. More tracer is localized in the
metaphyseal area. Imaging is done 48 hours after tracer administration.
Resolution of anatomical detail for bone lesions is considerably less with
Gallium than with 99m TC MDP, with higher radiation exposure. This scan is
preferred to 99m TC MDP in evaluating the response to treatment and is not as
dependant on blood flow. More specific in differentiating. bone tumors from
osteomyelitis and more reliable in assessing subacute and chronic infections.
k. Multiple Tracer Studies: When both types of study are needed the bone scan
scan should precede the Gallium scan by 24 hours. The following lists some
scenarios when multiple studies are used:

(+) 99m Tc MDP & - Ga67= chronic osteomyelitis


diabetic osteoarthropathy
implant loosening/ectopic bone (-) 99m Tc MDP & (+) Ga67= cellulitis

(+) 99m Tc MDP & (+) Ga67= acute osteomyelitis


implant loosening

(+) 99m Tc MDP & (+++) Ga67= infection in the presence of an implant

l. 111 Indium White Cell Scanning: This scan is much more specific for
infection. With this scan, white blood cells are labeled with the tracer (Indium)
and injected intravenously. This technique was developed to detect leukocyte
accumulation at sites of inflammation and abscess formation. Scans are
performed 24 hours after injection. A positive scan is defined as a focal
accumulation of leukocytes that is higher than in the surrounding bone. This
technique is reserved for complicated posttraumatic or post-surgical patients
with equivocal conventional bone scans, in cases where 99m TC MOP scanning
reveals false positive results because of rapid turnover. Therefore, it may be
more accurate in detecting acute infections.

NOTE* Limitations of Scans: Some patients show multiple hot spots at an early
stage of S. aureus septicemia but do not progress to OM. You can have a
negative scan with a confirmed OM due to impaired blood supply (false-
negative). You can have difficulty in differentiating OM from cellulitis. You can
have difficulty in differentiating normal bone repair from bone infection (false-
positive).

m. Computerized Tomography: An additional technique that can be used to


evaluate OM, especially the articular surfaces and periarticular soft tissues.
Also with OM, elevation of attenuated values in the marrow space is an early
sign of an acute bone infection. The non-specific destruction of cortical bone
and new bone formation can be seen in this way and the progression and/or
recession of disease may be monitored.
n. Magnetic Resonance Imaging: MRI distinguishes soft-tissue structures more
readily than any other imaging modality. It can differentiate osteonecrosis and
aseptic necrosis since most artifacts of bone occur in fatty tissue which is easily
imaged by MRI. Can be used on OM by evaluating the marrow. This is because
water has a high proton content and MRI measures water content of various
body tissues (CT measures specific gravity). The more water the higher the
signal intensity. The following tissues are ranked on the relative Spin-Echo
Grayscale from brightest (short T1 and/or long T2) to darkest (long T1 and/or
short T2 and/or low proton density): fat, marrow, cancellous bone, brain, spinal
cord, viscera, muscle, fluid filled cavities, ligaments, tendons, blood vessels,
compact bone and air. (SEE SECTION ON RADIOLOGY)

5. Acute Hematogenous Osteomyelitis


a. Is a result of blood stream dissemination of bacteria emanating from an
identifiable focus of infection or developing during transient bacteremia
unrelated to a discernable infection
b. Blunt trauma to long bones precedes AHO 33% of the time, femur, tibia and
humerus in that order.
c. Infections usually begin in the metaphysis
d. Large cortical involucrum formation is typical of AHO in the child
e. Three common sites of damage based on age of the patient: epiphyseal anlage
and joint in the infant, cortex of shaft in the child and joint and marrow in the
adult
f. AHO (infants):
i. Causes severe and often permanent epiphyseal damage and joint infection
ii. Only transient damage to shaft and metaphysis
iii. Adjacent joint effusion 60-70% of the time
iv. Group B Strep, S. aureus and E.coli are most frequent organisms
v. Calcaneal AHO reported secondary to repeated venipunctures
g. AHO (child):
i. Extensive cortical damage with involucrum formation
ii. Permanent damage to growth cartilage and to joints is rare iii. Chronic OM is
a rare complication
iv. S. aureus and epidermidis 60-90% cases
v. Salmonella seen with sickle cell (in the diaphyseal area)
vi. H. flu implicated in kids under 2 years
h. AHO (adult):
i. Relatively common in IV drug users with #1 pathogen being Pseudomonas
ii. Usually affects axial skeleton and/or irregularly shaped bones (wrist/ankle)
iii. The cortex is absorbed instead of sequestering
iv. The whole bone is invaded and frequently results in chronic infection of the
whole marrow

6. Contiguous Osteomyelitis:
a. Most common form of OM seen in podiatry.
b. Chronic ulcerations may frequently lead to contiguous OM.
c. May develop when an infection arises in an area adjacent to bone in which a
malignancy is present or bone has received radiotherapy.
d. Staph aureus is the most frequent pathogen.

7. Osteomyeltis associated with peripheral vascular disease:


a. Commonly found in patients with diabetes mellitus or with severe p.v.d.
b. Usually a mixed aerobic and anaerobic infection.
8. Chronic Osteomyelitis:
a. Long standing OM is associated with sequestration.
b. Indolence of the infection is sometimes related to the suppressive effects of
antibiotics in low doses.
c. Staph aureus is the most common pathogen.
d. The phalanges and metatarsal heads are the most common sites. e. The sed
rate is elevated in 57% of these patients. f. Increased warmth over the infected
area found in 94% patients.
g. Pathogens residing in the "dead bone" if not removed along with the
sequestra, can cause acute flare ups as late as 50 years after the initial episode.

NOTE* Dead bone (sequestrum) acts as a foreign body


The sequestrum must be surgically removed
15-30% of individuals with acute OM or those with chronic OM
who have undergone surgical debridement, develop active flare ups

h. Squamous cell carcinoma has been reported as a late sequel of chronic OM


(foul smelling discharge, increased pain, increased drainage and enlarged
mass), in local draining skin sinuses.

9. Osteomyelitis secondary to puncture wounds:


a. Usually caused by a nail driven into the foot.
b. Suspicion should arise with patients who present with persistent cellulitis
despite soaks and antibiotics and when an I&D produces a foreign body.
c. Pseudomonas is the most common organism.

10. Brodie's Abscess:


a. A small metaphyseal lesion with well defined margins which is often tibial
and which occurs in young males prior to epiphyseal closure.
b. The complaint is recurrent pain, with relief from aspirin, therefore must r/o
osteoid osteoma.
c. S. aureus is the predominant organism.
d. Treated with debridement and curettage and antibiotics e. Can act as site for
local flareups and systemic infections

11. Treatment: Conceptually, It is easy to formulate and describe principles to


follow in planning therapy for a patient with osteomyelitis.
a. The principles that should be considered in the treatment of OM are:
i. A definitive bacterial diagnosis is required
ii. The choice of antibiotics depends on the susceptibility of the etiologic agent,
and the least toxic, least expensive, bacteriocidal antibiotic should be
administered
iii. Surgical intervention must remove devitalized bone that is inaccessible to
antibiotics and may provide a culture medium for continued bacterial growth.
NOTE* Despite the apparent clarity and simplicity of these statements,
diffilculties can arise in their application. This standard set of principles is
aimed at providing a clinical and bacteriological cure. However, in treating
osteomyelitis, one must also consider the best functional result for the patient.
The patient may decide that intermittent short course antibiotic therapy for
control of chronic osteomyelitis is more acceptible for functional needs than
amputation of the affected limb. It becomes imperative to involve the patient in
these difficult decisions.

b. A debate rages regarding the length of parenteral antibiotic therapy (36


weeks).
c. Surgical debridement generally expedites the resolution of the infection and
can decrease the duration of parenteral antibiotic therapy.
d. The spectrum of pathogens causing OM secondary to a contiguous focus or
PVD is usually broader than found in AHO. The selection of antimicrobials for
contiguous focus OM must be based on C&S from the infected bone.
e. OM secondary to PVD is commonly associated with a mixture of aerobic and
anaerobic organisms. The isolation of anaerobes on C&S, presence of organisms
on a gram's stain that fail to subsequently grow in vitro, and the existence of a
foul odor should guide therapy towards anaerobic bacteria. Gram positive
anaerobes such as Clostridium or peptostreptococci can be adequately treated
with penicillin. However, anaerobic OM usually involves gram negative
organisms such as Bacteroides fragilis which can be treated by clindamycin or
metronidazole. Mixed aerobic/anaerobic infections can be treated with
combined antimicrobial therapy. Third generation cephalosporins (except
Claforan for B. frag) are resistant to 20% of the anaerobes so should not be
used.
f. In a study of oral therapy it was found that S. aureus OM was treated
successfully with 1-2 weeks of IV antibiotics to stem the acute flare up followed
by oral therapy (cloxacillin/dicloxicillin or clindamycin + probenecid) for as long
as 6-12 months (J. Medicine 1976).
g. Treatment for puncture wound OM involves a good history (including a
tetanus history), x-rays, cleanse foot with Betadine, remove any foreign body
making the entrance wound larger if necessary, take deep cultures, flush well
and if gross contamination pack the wound open

NOTE* Oral antibiotic therapy prescribed after the original puncture wound
may contribute to a later OM infection

h. Implantation of gentamycin/ methylmethacrylate beads has been shown to


be helpful as adjunctive therapy for contiguous focus OM.

Septic Arthritis
1. Occurs most commonly in infants and children
2. Adults will generally have an underlying disease
3. Pathogens vary according to the age of the patients with S. aureus
predominating in all age groups
a. H. influenza 2nd most common organism in patients under 2
b. N. gonorrhea 2nd most common organism in adolescents
c. Enterobacter and pseudomonas 2nd most common organisms in patients
older than 50
4. Diagnosis is made via sterile aspiration of pus with a positive gram stain and
C&S.

Radiographic diagnosis reveals loss of subchondral cortical bone with


preservation of underlying supporting trabeculae

5. Treatment:
a. Serial aspirations vs. surgical decompressions
b. Suction irrigation
c. Parenteral antibiotics
d. Do not use intra-articular injections of antibiotics (can cause a superinfection
or a post-infection synovitis)

Mycology

Fungi are heterotrophic, eukaryotic, chlorophyll-free, thallophytic organisms.


They reproduce by spores, which germinate into long filaments called hyphae.
The hyphae grow and branch and develop into a mat of growth called the
mycelium. Spores are produced from the mycelium. Fungi are divided into 4
groups
 Phycomycetes
 Blastomycetes
 Ascomycetes
 Fungi Imperfecti (most pathogenic fungi are in this group)
Those fungi affecting humans can be divided into either superficial
(dermatophytes) or deep fungi
Diagnosis is based upon microscopic examination (KOH prep) and culturing
the organism on Sabouraud's medium. Additionally, a Wood's light can be
used for the diagnosis of some organisms, Calcoflour White stain is used with
fluorescent microscopy, India ink smear for detecting cryptococci in the CSF,
Gomori Methamine Silver and Neutral Counter stain for histopathology slides

1. Superficial Mycoses
a. Tinea Versicolor
b. Candidiasis
c. Onychomycosis
d. Tinea capitis
e. Tinea corporis
f. Tinea pedis
g. Tinea cruris
h. Trichomycosis axillaris
2. Deep Subcutaneous Fungal Infections
a. Sporotrichosis: granulomatous infection of the skin and subcutaneous
tissue, spread via lymph channels, skin lesions present as raised erythematous
plaque-like lesions, a chancre will develop at site of entry and has rare systemic
manifestations (Amphotericin B). Often occurs in those working with roses,
whose thorns inoculate the organisms.

b. Blastomycosis: begins as a papular lesion which ulcerates and spreads with


a pustular border. Treatments include local excision of the early lesion or the
drug 2-Hydroxystilbamidine.

c. Mycetoma: is a chronic granulomatosis infection of the soft tissue and bone.


It requires the triad of an indurated swollen lesion, draining sinus and
unilateral presentation for a clinical diagnosis. It is seen in endemic proportions
in some third world countries. "Sulfur granules' describes material that exudes
from the sinus. It is a form of 'Madura Foot`.

d. Madura foot: may either be fungal (Madurella mycetoma/Pertriellidum


Boydii) or bacterial (Nocardia/Actinomyces/Streptomyces/Achnomadura).
Diagnosis is via gram stain, KOH prep, and testing with Saubouraud's (fungal)
or blood agar (bacterial). Longstanding infections will produce multiple
osteolytic lesions in bone. Treatment involves sulfonamides, local wound care,
and debridement or amputation in severe cases. Infectious disease specialist
should be consulted

NOTE* In addition to the above there are actinomycosis, candidiasis,


chloromycosis, coccidioidomycosis, cryptococcosis, geotrichosis, histoplasmosis,
nocardiosis, aspergillosis, and mucormycosis. These are more common in
immunosuppressed patients, e.g. AIDS, uncontrolled Diabetes mellitus,
malignancies (especially of hematopoietic and lymphoid tissues)

NOTE* Antifungal therapy can consist of the following:


1. Imidazole and Triazoles: inhibit ergosterol synthesis in the fungal cell wall
2. Imidazoles: econazoles, ketoconazoles, sulconazoles, terconazole,
clotrimazole, oxiconazole
3. Triazoles: itraconzole, fluconazole
4. Miscellaneous: allylamine, terbinafine (Lamisil), griseofluvin
Viral Diseases

Viruses are the smallest and most simple biologic units that can manifest the
essential aspects of a living substance. Some important facts about viruses are:
 They are composed of an outer coat of protein and an inner core of nucleic
acid
 The viruses reproduce heritable characteristics in a predictable manner
during multiplication and demonstrate genetic continuity
 A virus infected cell is immune to reinfection by the virus and also
immune to infection from other viruses usually related to the one which
infected it
 The virus infected cell can support the reproduction of the virus which
entered it and may produce new virus particles in a relatively short time
 The most common form of damage in cells supporting viral reproduction is
a cytopathic effect which leads to deterioration and results in death and
disintegration

Viruses are classified according to the target organ and similarity of action.
They are:
1. The pox group (vaccinia and smallpox)
2. Neurotrophic viruses (poliomyelitis, rabies, and arthropod-transmitted
encephalitis including Japanese, St. Louis encephalitis, equine encephalitis and
aseptic meningitis)
3. Viseotrophic viruses (yellow fever, infectious hepatitis and homologous
serum hepatitis)
4. The herpes group
5. Varicella and herpes zoster
6. The myxoviruses (mumps and influenza)
7. The lymphogranuloma psittacosis group
8. Miscellaneous viruses (measles, rubella, Coxsackie viruses, the common
cold, warts, etc.)
9. Bacteriophage

Rickettsial Diseases
Rickettsial diseases are caused by intracellular microorganisms which are
classed between bacteria and viruses, and have characteristics of both. These
pleomorphic, cocco-bacillary organisms can cause acute, febrile, self-limited
symptoms usually accompanied by a skin rash. The group of diseases includes
typhus, Rocky Mountain spotted fever, tsutsugamushi, and Q fever. Diagnosis
is via serologic testing (the Weil-Felix test is useful)

Protozoan and Metazoan Infections


These infections are common in the warmer climate countries. When a parasite
invades a host, there are 4 possibilities:
 The parasite may die at once
 It may survive without causing symptoms
 It may survive and cause disease
 It may kill the host
Protozoa are one-celled organisms and metazoa are invertebrates made up of
many cells (such as worms) replicating directly within a human host

1. Protozoan infections
a. Amoebic dysentery
b. Malaria
c. Trypanosomiasis
d. Leishmaniasis
e. Toxopiasmosis

2. Metazoan infections (parasitic)


a. Nemathelminthes (roundworms)
b. Platyheiminthes (flatworms)
i. Trematoda (flukes)
ii. Cestoda (tapeworms)
c. Hirudinea (leeches)
d. Arthropoda (insects and related species), as in scabies, fleas, tunga
penetrans, maggots (myiasis) etc.

NOTE* Stool examination for the egg larvae and the organisms themselves are
diagnostic for some intestinal protozoa and metazoa. Blood smears using
special staining techniques aid in the diagnosis of some protozoan infections.
Some special stains can detect protozoa in tissue biopsies
Chapter 7: Wound Healing
Repair of Skin
Factors That Interfere With Wound Healing Growth
Factors in Wound Repair Surgical Approaches
Wound Dressings
Suture Materials and Needles Tourniquets
WOUND HEALING

Repair of Skin

There are three basic phases of wound repair: Inflammatory, Fibroblastic and
Maturation

1. Phase 1-Inflammatory Phase Substrate or Lag Phase : This stage lasts 3-4 days
and has 3 parts, vascular, hemostatic and cellular.
a. Hemostasis is obtained via active vasoconstriction of blood vessels damaged
in the wound. Aggregation of platelets also leads to the formation of a
hemostatic plug.
b. Platelet adhesion is in part stimulated by exposure of the platelets to the
proline and hyroxyproline matrix of mature collagen and other connective
tissue components exposed by the injury.
c. Once platelets are exposed to and adhere to the connective tissue matrix,
the platelets are activated. This can only occur in the presence of von
Willebrand components of factor VIII which is released from adjacent
endothelial cells. Activation involves the release of ADP from the platelet. The
ADP stimulates other platelets to stick to one anotherplatelet aggregation.
d. Platelets store calcium and 5-hydroxytryptamine in intracytoplasmic
granules as well as many other growth factors. These are released upon
adhesion and promote further platelet aggregation and vasoconstriction. This
process is termed "degranulation"
e. Platelet stimulation results in activation of phospholipase and hydrolyzed
membrane lipids and the liberation of arachidonic acid. Arachidonic acid is
converted by platelet cyclo-oxygenase into
thromboxane A2. Thromboxane A2 further stimulates platelet aggregation
and is also a potent vasoconstrictor.
f. Contractile protein in the platelet, thrombosthenin, promotes clot retraction.
Clot retraction will not occur unless platelets are present.
g. Coagulation occurs due to the activation of clotting factors.
i. Intrinsic system
ii. Extrinsic system
The end result is the activation of factor X which then converts prothrombin
to thrombin. Thrombin then converts fibrinogen to fibrin monomers, which
polymerize to form a fibrin clot. Fibrin besides promoting hemostasis, provides
a scaffolding for the ingrowth of cells at a later stage.
h. Platelets release a number of other factors at this point which promote
wound healing. These include:
i. Proteolytic enzymes activate the complement system. Also released is 12-HPETE
which in turn stimulates the release of leukotriene B4 an
important chemotaxic agent.
ii. Various platelet derived growth factors which promote various components of
wound healing. (See below)
h. Other substances in the plasma increase vascular permeability. Histamine is
released by mast cells. Histamine increases vascular permeability by causing
contraction of endothelial cells and uncovering
gaps between the cells. Histamine is also a powerful vasodilator. Serotonin
released from the platelet and kinins made from plasma alpha globulins at
the site of injury, also increase vascular permeability.
i. Neutrophils, attracted by the chemotaxic factors arrive in the wound about
6 hours after the injury. They reach their highest numbers at 1-2 days post
injury. If no infection is present, their numbers decline after this.
Neutrophils are responsible for wound debridement through the release of
collagenolytic and fibrinolytic enzymes. Additionally, neutrophils ingest
bacteria.
j. Lymphocytes reach their maximum number in the wound at day 6.
The most important role of the lymphocyte is the synthesis of lymphokines.
Two of the best known lymphokines are the migration inhibition factor
(MIF) and macrophage activation factor (MAF). MIF attacts macrophages
to the wound and MAF activates them.
k. The macrophages attracted to and activated in the wound are actually
derived from monocytes in the blood. They are the most important
inflammatory cells involved in wound healing for the following reasons:
i. They are the only cells able to tolerate the low oxygen tensions at the wound
edge.
ii. They appear in the wound during the first 5 days and have a long life span
(7-10 days).
iii. Wound healing is severely inhibited in the absence of monocytes.
iv. They process and present antigens to the lymphocytes to initiate immune
response.

NOTE* Activated macrophages have a number of roles.


a. Wound debridement- accomplished by the production and secretion of
proteinases, such as collagensase, etc.
b. Ingestion of bacteria and cell debris
c. Capillary formation-neovascularization is a direct effect of macrophage
stimulation through cytokines
d. Antigen processing and presentation

l. Migratory fibroblasts originate from mesenchymal cells near the wound


edge. These cells become bound to the fibrin laid down in the wound and
proliferate. They then produce glycoproteins. Collagen synthesis begins on the
5th day post injury and lasts 2-4 weeks.
m. Eosinophil concentration reaches a peak in the injured area between
days 7-14. They may be associated with collagen remodeling and synthesis
occurring at the same time. There are very few and their role is unclear.
n. Fibronectin is a glycoprotein produced by fibroblasts, endothelial cells and
hepatocytes. Among the functions attributed to fibronectin are:
i. Fibronectin coats macrophages, aiding in opsonization and phagocytosis.
ii. Fibronectin is found on the surface of fibroblasts, where it may aid on the
adhesion of these cells to the extracellular matrix.
iii. Fibronectin cross-links with collagen and glycosaminoglycans. This results
in increased adhesion of epidermal cells and endothelial cells to the dermis.
iv. The matrix formed by fibroblasts and fibronectin creates a framework over
which epidermal cells may migrate.

2. Phase 2- The Proliferative Phase (Migratory/Lag Phase): This stage lasts from 5-
20 days and has three parts: epidermal regeneration, neoangiogenesis, and
collagen synthesis.
a. Epidermal regeneration:
i. The cells at the wound edges flatten out and develop pseudopods (extensions
of their cytoplasm), then migrate across the wound, only migrating over viable
tissue, at a rate directly proportional to the oxygen tension of the tissues
which is highest under hyperbaric conditions. The aforementioned fibrin-
fibronectin network serves as a framework over which this migration occurs.
ii. Intracellular contractile filaments (actin) develop at the periphery of the
migrating cells. These filaments align themselves with the fibrinectin strands
in the extracellular matrix. The interaction of these strands actually
pulls the epithelial cells along.
iii. Other changes occur- the basement membrane under the epidermal cells
changes; the epidermal cells themselves elongate in the direction of the wound
defect; mitotic activity of the epidermal cells dramatically increases, and the
division and movement of epidermal cells may be under the direction of
epidermal growth factor (EGF).
b. Neoangiogenesis: The formation of new capillary buds from blood vessels
near the wound occurs at the same time as the migration of the epidermis. The
development of capillaries towards the center of the wound may be under the
influence of growth factors released by macrophages. As oxygen tension
increases with the opening of new vascular channels, these growth factors are
inhibited and capillary growth slows and then stops.
c. Collagen Synthesis:
i. Within the injured dermis, fibroblasts (surgeon's cell) begin to appear at the end
of the inflammatory process, and adhere to the dermal collagen and fibrin. As
the capillary structure returns to the wound and oxygen tension increases,
fibroblast replication slows. As oxygen tension further increases, fibroblasts
begin synthesizing collagen.
ii. There are at least 5 types of collagen.
iii. Collagen production by the fibroblasts are under control of at least 5
growth factors.
iv. Collagen at this point represents 50% of the scar.
v. The amount of collagen in the healing wound reaches a maximum at two to
three weeks post injury. Remodeling now begins.

Note* The wound reaches 35% of its original strength around the 14th day.
The tensile strength of the wound now equals the strength of the suture and
sutures can be removed at this point.
Tension on the wound stimulates production of collagen which is initially
beneficial.
Therefore excessive tension should be avoided making skin incisions along
the relaxed skin tension lines and using steri-strips when possible.

NOTE* A number of factors influence epithelial repair: It has been shown that
cortisone delays formation of granulation tissue and wound closure; causes
thinning of the dermis and atrophy of collagen fibers; and causes a decrease
in fibroblast and new blood vessel proliferation. This effect can be reduced by
vitamin A. Data is conflicting and there are no dosage amounts or periods of
time described as being a minimum for steroids to have their deleterious
effect on wound healing. Therefore, injections of post-op steroids which is
done routinely in podiatry to reduce post-op pain and edema should be
studied more scientifically.

3. Phase 3-The Remodeling Phase (Maturation)- Can last up to a year At two


weeks post injury, a wound has regained only 35% of its tensile strength. By
one month this has increased to 40-50%. A number of processes occur
during the remodeling phase:
a. The entire remodeling process is really an equilibrium between enzymatic
processes lysing and resorbing old collagen and forming new collagen.
b. Wound contraction is part of this remodeing stage of healing. Contraction
progresses at 0.6 to 0.7 mm/day independent of the wound size, but certain
shapes heal faster. Round wounds do not contract as quickly as rectangular or
stellate wounds. Skin grafting diminishes wound contraction.

Note* Only after 6 months and preferably not before a year do you really
know what a scar will look like, therefore, any attempt at scar revision should
wait at least this long.

Factors That Interfere With Wound Healing


1. Age: Growth rate and multiplication of fibroblasts decrease with age.

2. Inadequate Perfusion: Inadequate perfusion of skin, subcutaneous tissue, or


bone results in a decrease in oxygen delivery to the wound, thereby impairing
healing.

3. Infection: Infection leads to tissue destruction and edema, both of which


interfere with the healing process.

4. Edema: Interferes with tissue perfusion and leads to tissue destruction.

5. Poor Nutrition: Protein depletion results in alterations in collagen


synthesis and cross-linking.

6. Vitamin and Trace Element Deficiencies:


a. Vitamin A deficiency can interfere with wound healing, and may reverse
wound healing problems associated with steroids.
NOTE* Vitamin A taken p.o. to help reverse the effects of steroids can also
reverse the effectiveness of steroids in the condition for which they were
originally intended (kidney transplant; vitamin A can cause rejection).
However, topically vitamin A can be very effective. Have the pharmacist
mix 1000 units of vitamin A per gram of Bacitracin ointment for steroid
patients to apply topically to a wound

b. Vitamin C deficiencies lead to scurvy, a disease associated with the failure


of collagen synthesis.

7. Steroid and Cytotoxic Medications:


a. Steroids slow protein synthesis when given exogenously. Steroids
interfere with capillary budding, slow fibroblast proliferation as well as the
rate of epithelialization.
b. Cytotoxic drugs commonly used in chemotherapy, inhibit cellular
proliferation. In general, wound healing is slowed but not prevented.

8. Radiation: Microvascular changes occurring after tissue is exposed to


radiation at therapeutic doses will lead to perfusion problems if that tissue
is later injured. All cell types involved in healing may be adversely affected
by radiation. Malignant change may also occur.

9. Diseases Which are Associated With or Predispose to Chronic Wounds:


a. Diabetes Mellitus:
i. Deposits in the arteries interfere with tissue perfusion.
ii. Diabetic neuropathy leads to reduced sensation and gait abnormality iii.
Metabolic problems lead to a reduction in nutrients available for
wound healing.
iv. Impaired phagocytosis seen as part of the disease spectrum on diabetics
leads to an increase in bacterial infections and subsequent tissue destruction.
b. Venous Stasis: Poor venous return leads to an increase in tissue pressure.
The increase in tissue pressure results in underperfusion of the skin and
wounds, as well as accumulation of inhibitory metabolites
c. Collagen Vascular Disease: These diseases have an autoimmune basis and
result in capillary damage leading to poor tissue perfusion and hypoxia,
and immune response to other cells or cell constituents

10. Treatment of Non-healing Wounds:


a. Debridement of necrotic tissue
b. Control of infection
c. Control of diabetes mellitus
d. Nutritional support
e. Avoidance of trauma
f. Aggressive intermittent compression and elevation to eliminate limb
edema
g. Tapering of steroids (paradoxically, topical steroids applied to wound in
patients with collagen vascular disease may control the vasculitis and actually
stimulate wound healing)
h. Revascularization of an ischemic wound through angioplasty or
reconstructive vascular surgery
i. Use of hyperbaric oxygen
j. Plastic reconstructive surgery
k. Application of growth factors (still experimental)

Growth Factors in Wound Repair


With the production of a platelet derived wound healing formula (PDWHF)
known as PROCUREN, a mixture of 5 platelet-produced growth factors,
angiogenesis and other aspects of wound healing are stimulated. The 5
growth factors are
1. Platelet derived growth factor (PDGF)
2. Platelet derived angiogenesis factor (PDAF)
3. Platelet derived epidermal growth factor (PDEGF)
4. Transforming growth factor (TGFB)
5. Platelet factor 4 (PF-4)

Surgical Approaches
Skin incisions are planned according to established principles:
a. Anatomical landmarks including vital structures should be identified and
marked with a skin scribe prior to the initial incision (listen with a doppler for
arterial flow)
b. Tension on the healing incision must be avoided
c. Easy access to all structures involved must be possible
d. The incision should be long enough to avoid excess traction on the wound
margins (remember that skin heals from side-to-side and not from end-to-end)
e. Vital nerve and blood vessels should be identified and protected, with skin
incisions made parallel to them
f. Incisions should be parallel to relaxed skin tension lines (RSTL) whenever
possible
g. Excessive manipulation and damage to the deeper tissues must be avoided
by following the lines of cleavage (RSTL) and planes of fascia
h. Scars over bony protuberances or weight bearing surface points should be
avoided
i. Longitudinal incisions or incisions perpendicular to the RSTL should be
located in areas of relatively little skin movement
j. Longitudinal incisions that cross flexion or extension surfaces of joints
should be avoided or at least made in a curvilinear fashion to allow for
flexibility and scar contracture
NOTE* An incision perpendicular to the RSTL will gap widely whereas an
incision made parallel to the RSTL will remain approximated. The RSTL are
created by the directional pull of structures underlying the relaxed skin. The
RSTL are important for the direction they convey. Optimal healing can be
produced by ensuring that the collagen in the scar forms parallel to the
adjacent dermal collagen. Skin expands and contracts along the of direction
of muscle pull, so that if an incision is made parallel to this direction, then the
body responds by depositing increased amounts of collagen to strengthen the
scar.

To find the RSTL one simply relaxes the skin in the area by passive manipulation
or in areas where there is not much motion the RSTL can be found by
pinching the skin over the area. This will form furrows. Pinching in line with the
RSTL will form a regularly shaped furrow and pinching oblique to the RSTL
will give rise to an S -shaped furrow

Wound Dressings
Optimum wound dressings should remove excess exudate and toxic
components, maintain a high humidity at wound/dressing interface,
allow gasseous exchange, provide thermal insulation, afford protection
from secondary infection, be free from particulate or toxic contaminants
and allow removal without trauma at the dressing change.
1. Categories:
a. Alginates:
i. Sorbsan®
ii. Kaltostat®
b. Hydrogels:
i. Carrington®
ii. Core Care®
iii. MPM®
iv. Intrasite®
v. Sososite®
vi. Biolex®
vii. Vigilon®
c. Foams:
i. Flexan®
ii. Hydrosorb®
iii. Mitraflex®
iv. Polymem®
v. Epilock®
d. Hydrocolloids:
i. Ultec®
ii. Triad®
e. Isotonic Saline:
i. Dermagran®
f. Zinc Saline:
i. Dermagran®
g. Transparent dressings:
i. Tegaderm®
ii. Proclude®
h. Hydrophilic dressings:
i. Dermagran®

Suture Materials and Needles


1. Classification of surgical sutures:
a. Absorbable:
i. Natural filament (from animals):
 Plain gut
 Chromic gut
ii. Synthetic multifilament:
 Dexon-S® (polyglycolic acid)
 Vicryl® (polyglactin 910)
iii. Synthetic monofilament:
 PDS® (polydioxanone)
 Maxon® (polyglyconate)

b. Nonabsorbable:
i. Natural filament
 Surgical silk
 Dermal silk (Perma-hand)
 Surgical linen
ii. Metal
iii. Synthetic monofilament:
 Dermalon®, Ethilon® (nylon)
 Surgilene® (polypropylene)
 Novafil® (polybutester)
iv. Synthetic multifilament
 Neurolon®, Surgilon® (nylon)
 Mersilene®, Dacron®, Tevdek®, Ethibond®, Ticron® (polyester)

NOTE* Two principles should be kept in mind when choosing suture and the
caliber of suture. The first one is that the less reactive the suture the less
risk that it may potentiate infection (woven sutures tend to be more
reactive than monofilamentous ones). The second one is that the less the
amount and smaller the caliber of the suture used, the less foreign
material is left inside the wound to potentiate infection. Among the
absorbable stitches the monofilament polyglycolic PDS (looses strength in
60 days and is absorbed in 180 days) is the least reactive. The woven
polyglycolic, Dexon/Vicryl (looses strength in 30 days and is absorbed in 70-
90 days) is minimally reactive. Among the permanent stitches, nylon is
minimally reactive. Nonabsorbable woven silk, woven cotton, or
absorbable plain/chromic gut should not be used (with specific exceptions)
as they cause acute inflammatory reactions.
2. Biologic properties: All suture materials elicit a foreign body reaction within
the tissues, which lasts for 5-7 days and is relatively the same for all suture
materials. It has been reported that monofilament sutures and uncoated
sutures produce far less reaction than multifilament and coated sutures. Also
synthetic sutures cause less tissue reaction than sutures derived from natural
materials. The best suture appears to be synthetic, monofilament and
nonabsorbable
3. Needle characteristics:
a. Design:
I. Eye
 Closed eye
 French eye
 Swaged (eyeless) (most commonly utilized)
II. Body
 Straight (Keith needle)
 Half curved
 Curved (either 1/4, 3/8, 1/2, or 5/8 circle)
III.Point
 Tapered
Cutting
 Reverse cutting
 Paper cut or diamond point
 Precision-point or hand-honed reverse cutting
Tourniquets
1. Indications: To exert enough pressure on the arterial blood flow to a limb
to produce a bloodless field. Single cuff tourniquets are generally used for
operations lasting less than 90 minutes

2. Contraindications: The final decision whenever to use a tourniquet rests


with the attending physician
a. Open fractures of the leg
b. Post-traumatic hand reconstruction
c. Severe crushing injuries
d. Elbow surgery (with excess swelling)
e. Severe hypertension
f. With skin grafts in which all bleeding points must be readily
distinguished
g. Compromised vascular circulation or in the presence of an arterial graft
h. Diabetes mellitus
i. Sickle cell disease (relative contraindication) as severe postoperative pain
could result

NOTE* Test the hemoglobin and type and level before using a tourniquet on patients
with sickle cell disease or trait

3. Precautions:
a. Carefully exsanguinate the extremity (this prolongs the pain free
tourniquet time)
b. In the presence of malignant tumors, painful fractures, or infection
elastic bandage (Esmarch) exsanguination must not be done, only elevating
the limb for 3-5 minutes is acceptable
c. Do not use an elastic bandage for exsanguination in cases where
bacteria, exotoxin, or malignant cells could spread through the general
circulation
d. Inflation must be done rapidly to occlude arteries and veins simultaneously
e. Frequent irrigation is recommended when working under tourniquet
control to prevent tissue drying
f. Prolonged tourniquet time can produce permanent or temporary
tissue/nerve/blood vessel damage. It can also produce changes in the
coagulability of blood with an increase in the clotting time
g. If the blood does not return to the limb within 3-4 minutes after release
of the tourniquet, the limb must be placed in a dependent position

4. Adverse Effects:
a. A dull aching pain
b. Stiffness, weariness, reactive hyperemia, skin discoloration
c. Pathophysiological changes due to pressure, hypoxia, hypercarbia, and
acidosis of the tissue can occur after one and one-half hours of tourniquet use.
Symptoms of tourniquet paralysis are:
i. Motor paralysis and loss of touch sensation
ii. Loss of proprioceptive responses
d. Intraoperative bleeding due to:
i. An underpressurized cuff
ii. Insufficient exsanguination
iii. Insufficient cuff pressure
iv. Blood entering through the nutrient vessels of the long bones v.
Inadequate cuff size

5. Pressure Settings: For each patient, the tourniquet should be set to the
minimum effective pressure, taking into account such factors as obesity
preoperative systolic pressure, maximum anticipated rise in the
patient's blood pressure during the surgery, and the presence of hypertension
a. Published literature suggests an effective tourniquet setting be 75-100 mm
Hg above preoperative systolic pressure above the upper limbs, and two
times the preoperative systolic pressure for the lower limbs when using single
bladder cuffs

6. Inflation Time:
a. Depends upon the patient's anatomy, age, and absence of vascular disease
b. For reasonably healthy adults, there is general agreement that 2 hours
should not be exceeded without releasing the tourniquet for 15-20 minutes
before reinflating, and during this time the limb should be elevated about 60°
with steady pressure to the incision
7. Release of Tourniquets:
a. When the tourniquet is released the wound should be protected from
blood surging back by applying pressure dressings and if necessary elevating
the limb
b. If color is not back back within 3 minutes, lower the extremity
c. It has been reported in the literature then upon the release of bilateral
thigh tourniquets (at the same time) a patient developed asystole.
Therefore, this article suggests that when utilizing bilateral thigh tourniquets,
that there be a 5 minute delay between deflation of the cuffs
Chapter 8: Plastic Surgery
Skin Grafting
Local Cutaneous Flaps
Local Muscle and Myocuteneous Flaps
Fasciocutaneous Flaps Microsurgery and Free Flaps
Diagrams of Skin Flaps
PLASTIC SURGERY

When considering soft tissue reconstruction one always needs to go


through a decision tree in evaluating options from the simplest to most
complex reconstructive technique depending upon the patient's health,
resultant changed biomechanics, and soft tissue defects location:
1. Allow the soft tissue defect to heal by secondary intention
2. Close the wound primarily
3. Apply a split thickness or full thickness skin graft
4. Use a local fascial, fasciocutaneous, muscle or musclocutaneous flap
5. Use a microvascular free flap transfer

Skin Grafting
1. Definitions:
a. Consists of harvesting epidermis with a varying thickness of
accompanying dermis and placing it on a recipient base.,
b. Split thickness: Includes epidermis and a portion of dermis (the more
dermis included, the thicker the graft)
c. Intermediate split thickness: contain more dermis
d. Thick split thickness grafts: contains most dermis
f. Full thickness grafts: contain all the epidermis and dermis, no fascia or
fat but has the sweat and sebaceous glands
g. Grafts can be autografts (same individual), allografts (same species), isografts
(twins), and xenografts (different species)
h. A free skin graft is completely detached from the body during its transfer
from the donor site

2. Anatomy of skin:
a. The epidermis represents 95% of the skin thickness and dermis 5%
b. The dermis contains sebaceous glands, except in the palms and soles
c. The dermis is principally made up of collagen and elastin
d. The subcutaneous tissue contains the sweat glands and hair follicles
(except in the palms and soles that lack hair)
e. The blood supply arises out of a vascular network that lies on top of
fascia and sends vertical branches up through the subcutaneous tissue and
dermis. The vessels arborize along the way and terminate as capillary buds
between the dermal papillae (the thinner the graft, the more vessels are
transected)

3. Preparation of the recipient site:


a. Must have no infection for a successful graft (a bacterial count of less
than 100,000/gram of tissue) To sterilize a wound:
i. Surgical debridement
ii. Topical Sylvadine for 4-5 days plus IV antibiotics: or
iii. Biological dressings (pig skin or amnionic membranes) plus IV antibiotics
b. Must be well vascularized for a successful graft (pH at 7.4, Tcp 02> 40 mm Hg
and epithelialization at the border)
c. If the wound is fresh then you can graft onto dermis, fat, fascia, paratenon,
or periosteum
d. You cannot graft onto cortical bone or tendon
e. Granulation tissue contains bacteria and must be removed at the time of
grafting
f. Hemorrhage must be controlled
g. Thorough irrigation

4. Split thickness skin graft:


a. The thinner the graft the higher the chances of a successful take, due to in
part to the higher number of transected blood vessels through which primary
revascularization can be established
b. The thinner graft will shrink more at it heals (about 50-70% of its size)
because the decreased amount of dermis is less effective in inhibiting secondary
contraction
c. The thinner the graft the greater the chance for hyperpigmentation
d. The thinner the graft, the more susceptible it is to trauma, because of the
absence of rete pegs and the loss of lubricating sebaceous glands
e. A thin skin graft is usually .012-.013 inches thick
f. Donor site bleeding minimized with topical thrombin or dilute concentration
of epinephrine and dressed with Xeroform or Scarlett Red, Opsite or Tegaderm,
and/or Biobrane (semipermeable dressing)
g. Recipient site bleeding must be controlled (topical thrombin or dilute
epinephrine) and if not place the skin graft back onto the donor site and
return to the OR in 24-48 hours
h. Graft cut with a BrownAire or Zimmer air driven dermatome are reliable
(can be cut by a Humby or Goulian knife by hand but is difficult). For power:
i. Set desired width by using a width guard (either 5 cm, 8 cm, 10 cm)
ii. Set thickness by turning knob (usually .012-.013") and introduce a
#15 scapel blade between the cutting blade and base to check thickness

NOTE* A # 15 scapel blade is the proper thickness of the graft, so by placing this
blade into the dermatome, provides a double-check

i. Meshing the donor graft allows for removal of hematomas or seromas and
increases its size (do not mesh at a ratio greater than 1 1 /2 to 1)
j. Inset the graft using a Stent tie-over dressing (or a bolster dressing). The
bolster is built by first placing Xeroform® on the wound then normal saline
soaked cotton in the center. The nylon suture used at the wound's periphery
are then crossed over and tied to each other, forcing out the water in the
cotton. This allows the graft to conform exactly to the recipient site
h. The foot and leg should then be placed in a posterior splint to eliminate
movement/shearing
i. Dressing changed in 5-7 days if meshed, and if unmeshed at 48 hours
to check for fluid accumulation (if accumulation occurs, it should be aspirated
with a needle
j. The graft on the extremity should be kept elevated for 7-10 days, until
venous circulation is fully established

5. Physiologic phases in skin graft take:


a. Plasmatic imbibation phase during first 48 hours (graft is ischemic at this
time)
b. Inosculatory and capillary growth phase phase starts after 48 hours when
capillary budding from the recipient bed makes contact with the graft.
c. Circulation occurs between the 4th and 7th day

6. Full thickness skin graft:


a. Best donor site for full thickness grafts are the flexor surfaces such as the
groin, anticubital fossa, and popliteal fossa, and then closed primarily, leaving a
linear scar.
b. The length to width ratio of the donor graft should be at least 3-1
c. The donor graft is then sewn into the recipient site
d. Perforations should be made into the donor graft to allow for seroma
removal
e. Revascularization is more tenuous with a full thickness graft than with a
split thickness graft
f. Full thickness graft usually takes primarily, there is no contraction of the
wound, lubrication of the skin is normal, and there is no change in skin color
or texture
g. Neurotization occurs in the following order: pain, light touch, then
temperature
h. Sensory recovery starts at 4 weeks and can take up to 1-2 years

7. Reasons for graft failure:


a. Lack of compression of the graft to the recipient site
b. Movement/shearing
c. Infection (second most common cause)
d. Seroma
e. Hematoma (most common cause)

NOTE* It is important that the pressure on the graft does not exceed 30 mm Hg
or else blood flow to the graft will be compromised

Local Cutaneous Flaps


1. Anatomic principles of skin flaps:
a. It is not the length to width ratio but rather the presence of an artery at
the base of a flap that determines its success. Therefore the preoperative use of
a doppler enables one to determine whether the flow to a particular area of the
foot is antegrade or retrograde (the flow can be redirected due to an occlusion
of either the anterior tibial or posterior tibial artery)
b. A defined area of skin receives blood from 3 sources
i. Cutaneous artery: direct cutaneous arteries run in subcutaneous fat parallel
to the skin, and are usually accompanied by two venae commitantes (veins
that drain the area supplied by the cutaneous artery)
ii. Musculocutaneous artery
iii. Fasciocutaneous arteries
c. The flaps that have a direct blood supply (direct cutaneous artery flaps)
have a larger length to width ratio, than traditional random flaps and can be
used as:
i. Axial pattern flaps (traditional name) and can be used as a pedicle flaps (a
flap dissected free at its base, of most of the tissue surrounding the artery and
veins thus giving the flap added mobility)
ii. An island flap: where the vascular pedicle is dissected completely free for a
certain length and the flap is transferred to a local site separate from the
donor site while the pedicle is buried under intervening tissue
iii. A free flap: where the pedicle is totally detached and then hooked up by
microsurgery to recipient vessels anywhere in the body
d. Musculocutaneous flaps consist of muscle, subcutaneous fat and
skin, with the muscle receiving its blood supply according to one of 5
patterns

NOTE* In the foot it is preferable to harvest the muscle without the overlying
skin paddle as skin graft over the muscle, as the blood supply to the overlying
skin has a very narrow range and can cause significant donor defects if harvested
as a unit

e. The fasciocutaneous system is the chief source of blood supply to the


skin. It arises from a major regional artery as perforators that pass along the
fascia between muscle bellies and then fan out at the level of the deep
fascia (an example is along the long axis of the 3 arteries, posterior tibial,
anterior tibial, and peroneal
i. Example of a faciocutaneous flap of the foot is the medial plantar flap

NOTE* A random flap with is obligate 1:1 length to width ratio in the foot is a flap
based on unknown vascular anatomy. Axial pattern flaps have an
identifiable blood flow at their base and have a length to width ratio that
depends upon the angiotome which the artery serves. These flaps must be
preplanned, and can be extended beyond their angigtomes using delay
principles

2. Local flaps:
a. Local flaps are adjacent to the defect and are either rotated on a pivot
point or are advanced forward from their base to cover a defect. They
include a minimum of the epidermis, dermis and subcutaneous tissue. The
donor site is either closed primarily or skin grafted
b. Flaps that rotate about a pivot point
i. Rotation flap: is designed when a pie shaped triangle defect is created to
remove a lesion or preexistent defect. The flap includes skin and
subcutaneous tissue
ii. Transposition flap: are rectangular in shape with rounded edges and can be
rotated 900
iii. Limberg flap: is a type of transposition flap that depends on the looseness of
the adjacent tissue, and is used when the defect has a rhomboid shape (angles
of 60° and 1200)
iv. Z-plasty is a type of rotation flap that is used to lengthen an existing scar
and to reorient them along lines of minimal tension. The Z-plasty consists of 3
limbs of equal length on the shape of a Z, and the angles between the limbs
can vary from 30° to 90 0, and the wider the angle the more the theoretical
gain in length

NOTE* Clinically, 60° has been found to be the most useful and yields a theoretical 75%
gain in length, however, the actual gain in length is anywhere from 28% to 45% less
than calculated. The length of the center of the limb also determines the amount of
length gained, and the longer it is, the larger the gain

v. Interpolation flap: has a soft tissue pedicle with a distal skin island which is
rotated into a defect that is close to but not adjacent to the donor site
vi. Island flap: is a specialized interpolation flap where the only link between
the cutaneous flap and its bed is the neurovascular bundle. This can
be very useful in the foot, as the results are aesthetic, sensate, and
very functional
c. Advancement flaps:
i. Advancement flaps: are moved directly forward to fill a defect without
rotation or lateral movement. A rectangular incision of skin dissected out and
advanced into the defect thereby creating a folding of tissue at both ends of
its base (burrow's triangles), which are removed so that the skin can be
sutured together
ii. V-Y flap: is a V shaped flap whose sides are advanced creating a Y when the
incisions are closed. Can also use a double V to Y flaps when a defect is to
large for one (defects 3-4 cm wide)

Local Muscle and Myocutaneous Flaps


One can transfer simple muscle or muscle with overlying skin to cover a soft
tissue defect. It is critical to know the anatomic blood supply of the muscle
and skin

1. Abductor digiti minimi flap: Is a muscle flap used to fill defects of the lateral ankle
joint or skin

2. Abductor hallucis brevis muscle flap: The medial counterpart of the abductor
digiti minimi. If more bulk is needed it can be harvested with the medial half
of the flexor hallucis brevis muscle

3. Flexor digitorum brevis muscle flap: can only be considered in the well
vascularized non-diabetic foot, as its harvesting may lead to charcot changes
by weaking the midfoot arch
Fasciocutaneous Flaps
These flaps are thin, pliable, and reliable, however, are not as useful as muscle in
treating osteomyelitis because the blood flow per centimeter2 i s
3-5 times less

1. Dorsalis pedis flap (direct cutaneous blood supply): Is a direct skin flap in the foot,
can be used as a free flap, its advantage is that it is thin and can be used as a
sensory flap if the superficial nerve is incorporated
a. The potential flap territory overlies the artery
b. It is mandatory to know when using this flap whether the arterial flow is
antegrade or retrograde, which vascular system supplies the first dorsal
metatarsal artery, and whether the anterior branch of the peroneal artery is
dominant (if the distal portion of the flap is supplied by the vascular blood
supply from the sole of the foot, then a delay of that portion of the flap
should be done to avoid distal flap necrosis)
c. Should be only used in well vascularized patients as a 2nd resort because of
donor site morbidity

2. Filet of toe flap (direct cutaneous blood supply): Similar characteristics as that of
the sole of the foot (it is filling and sensate), a toe has to be sacrificed. It can be
rotated locally, or carried with its neurovascular bundle for more proximal
placement

3. Lateral calcaneal artery fasciocutaneous flap (direct cutaneous blood supply):


Derives its blood supply from the calcaneal branch of the peroneal artery. In
order to dissect this flap, it is critical to doppler out the artery along its full length,
and the artery should lie along the mid axis of the flap which allows for a
8x4cm vertical flap to be harvested. If the viability is questionable, dissection
is stopped and the flap delayed for 5-7 days. It is best to start with a lateral
incision down to the periosteum, and dissect up in a retrograde fashion

4. Plantar flaps: The blood supply to the sole of the foot is supplied by the
medial and lateral branches of the tibialis posterior artery.
a. Lateral plantar artery flap
b. Medial plantar artery flap: better than the lateral artery flap because it is
based on the less important medial plantar artery and is designed over a
nonweight-bearing portion of the sole, and can include the abductor hallucis
ms. to give the flap extra bulk

Microsurgery and Free Flaps


This has revolutionized the ability to cover soft tissue defects. Can include
fasciocutaneous, musculocutaneous, osteocutaneous, and
osteomusculocutaneous flaps

1. Donor site does not include the foot, and donor site morbidity is minimal

2. The free flap has to have adequate inflow through one of the three distal
arteries (preferably the distal posterior tibial or dorsalis pedis). If adequate flow
does not exist, then revascularization via in-situ by-pass graft is mandatory first
3. Free flap anastamosis, whenever possible, should always be done end to
side so that the distal flow is not compromised

4. For the dorsum of the foot a fasciocutaneous free flap from either the
parascapular area, the radial forearm, the lateral arm or the temporalis fascia
with STSG (the advantage are that these flaps are thin, have minimum donor
morbidity, and have reliable vascular pedicles

5. The muscle flap for the sole of the foot comes from either the serratus
anterior or gracilis muscle

6. If metatarsals and skin need to be replaced then a osteocutaneous flap from


the contralateral fibula is used, or parascapular osteocutaneous flap

NOTE* The advantage of using vascularized bone is that the risk of infection is
diminished and the bony union is more rapid

Diagrams of Skin Flaps


Chapter 9: Bone Healing
Primary Bone Healing
Complications of Bone Healing Treatment of Nonunions
Fusion
Bone Grafting
Aseptic Necrosis Following 1st Metatarsal Osteotomy
Electrical Bone Stimulation Cartilage Healing
BONE HEALING
Primary Bone Healing
The two basic requirements for primary bone healing are an intact vascular
supply to the bone and rigid stable fixation of the fracture fragments.
Primary bone healing consists of simultaneous remodeling and formation
of new bone at the fracture site. The intermediary phase of fibrocartilage
formation as seen in callus bone healing is actually bypassed, as new bone is
formed intentionally at the fracture margins. Primary bone healing is the
direct reconstruction of the fragment edges by haversian remodeling. This is
the same homeostatic mechanism that occurs in the living intact osseous
skeleton on a daily basis.

There are six phases of the reparative process:


1. Hematoma: will form between the fracture fragments immediately
after the injury up until 1-3 days
2. Organization of the hematoma: slow shrinkage of the hematoma
resorption of devitalized bone, periosteal and endosteal cells from
osteoblasts at wound edges, from 3-10 days
3. Formation of fibrous/cartilaginous callus: this time of this phase will be
directly proportional to the proper apposition of fracture fragments and
immobilization of the affected body part, from 10-40 days
4. Primary bone callus: the bone callus becomes denser and fracture
clefts fill in with bone matrix to bridge the fracture site, from 40-80
days 5. Absorption of the primary bone callus: transformation of this
material to secondary bone callus, from 80-120 days
6. Remodeling of new bone: responding to the stresses placed upon it,
after 120 days

NOTE* When compression and rigid fixation devices are utilized, bone will
repair across the fracture gap by direct primary bone repair, with bone callus
being minimal or nonexistent.
Healing at points of contact along the fracture surface begins with the advance of
a capillary bud from the haversian canal. This organized structure advances
in a linear direction and crosses the fracture line depositing lamellar new
bone along its path. The tip of the complex is a group of large multinucleated
cells that function as osteoclasts. They cut their way through existing osteoid
and cross the fracture line into the surface of the opposite fracture fragment
(called the cutting cone).
The osteoclasts are followed closely by a capillary loop as the cutting cone creates a
tunnel in the existing bone substrate. The wall of this tunnel is lined by cells with
osteoblastic activity. These cells produce a concentric pattern of new lamellar
bone as the cutting cone passes and this is loosely deposited along the course
of the haversian canal to exist as mature lamellar bone.
NOTE* Underlying diseases such as rickets, osteomalacia, Paget's Disease of
Bone, hyperparathyroidism, osteoporosis and osteitis fibrosa cystica may all
cause a delay in bone healing

Complications of Bone Healing


Without an intact vascular supply to bone or rigid stable fixation problems
occur: irritation callus or secondary bone healing, malunion, delayed
union, non-union or pseudoarthrosis.

1. Secondary bone healing Involves the formation of fibrous tissue (Phase 3),
primary bone healing does not.

2. Malunion is a misalignment of the fracture fragments along any of the three body
planes including axial rotations, caused by inadequate reduction and
inadequate immobilization.

3. Delayed union is the inability of the fracture to heal within an acceptable time
period.
a. Diagnosis is by radiographic evaluation
b. Treatment is by rigid external or internal immobilation
c. A good prognosticator for healing is the formation and evaluation of
the bone callus
d. Treatment involves stimulating bone callus formation and can be
enhanced by bone drilling, onlay grafts, external electric current and
magnetic field induction e. Bone scans are of little value in differentiating
delayed from non- unions

4. Non-union can be an end stage of a delayed union, due to inaccurate


reduction, interrupted or inadequate immobilization, severe local trauma
to soft tissue and blood supply in the fracture site, impairment of bony
circulation following open reduction, infection with secondary
osteomyelitis of bony fragments, loss of bony substance and distraction of
bony fragments, age, compromised host, anemia or anticoagulant therapy.

a. Unless a non-union is aggressively treated, it will usually result in a


pseudoarthrosis; the difference between them is that an atrophic non-
union does not have the fibrocartilaginous surface seen in a
pseudoarthrosis
b. Time frame for a non-union is usually 8 months following the initial
fracture
c. Diagnosis is by radiographic evaluation revealing gapping at the fracture
site with no bone callus; in some cases the fracture ends will begin to
round off (atrophic or non-reactive pseudoarthrosis); or will mushroom
developing a flaring (hypertrophic or elephant foot type pseudoarthrosis)
and form a joint space with synovial fluid filling the gap.
NOTE* Hypertrophic non-unions can be either Elephant type (maximum callus
/hypertrophy-best chance for healing), Horsefoot type (moderate
callus/hypertrophy) and Oligotrophic type (minimal callus/hypertrophy-least
reactive) NOTE" Atrophic non-unions can be either Torsion wedge type
(butterfly fragment with unilateral healing), Comminuted (gap+numerous
pieces), Defect type (no osseous integrity) and Atrophic type (rounded edges)
no osseous integrity.

d. Pseudoarthrosis is classified as non-infected, previously infected or


actively infected; non-infected are further subclassified according to their
location, diaphyseal vs metaphyseal (most diaphyseal pseudoarthroses are
atrophic)
e. Radiological studies:
i. Serial x-rays should be taken spanning time demonstrating a lack of
progress.
ii. Tomograms studies and CT scans can provide information of the bone
callus and the healing process.
iii. Bone scans are used to differentiate hypertrophic vs. atrophic
nonunions/pseudoarthrosis whereby a (+) bone scan would indicate a
hypertrophic/elephant foot type; a (-) bone scan would indicate an
atrophic/nonreactive type since it is relatively avascular.
f. Treatment of hypertrophic pseudoarthrosis: rigid immobilation and
electrical bone stimulation (cathode is inserted in the operative site set at 20
microamps). Bone forms under electronegative conditions.

NOTE* Bone stimulation is either implantable/invasive or non-invasive. Neither


will work for a synovial pseudoarthrosis. Non-invasive devices will not work
where the fracture gap is wider than 1 cm., and do not use during pregnancy or in
the presence of a pacemaker. Invasive stimulators may be used with pacemakers
and when the fracture gap is greater than 1 cm., also do not use during pregnancy.
Types of stimulators: constant direct current (semi-invasive), pulsed direct current
(invasive), inductive coupling electromagnetic stimulation (non-invasive),
capacitive coupling electromagnetic stimulation.

Note* Implantable bone stimulators have no known contraindications or adverse


effects, however, it is recommended that they not be used in the presence of active
OM or pathologic fracture from malignant bone tumors.

g. Treatment of atrophic non-union/ pseudoarthrosis: must resect the area


involved and replace with new bone using rigid internal fixation for approx. 16
weeks minimum
h. Metaphyseal pseudoarthrosis, because of its proximity to the joint space
cannot be easily tension banded or fixated with a plate
5. Previously infected pseudoarthrosis/open infected pseudoarthrosis
a. The infectious process within bone will respond much quicker to
antibiotics only in the presence of a stable fracture fragment (stabilizing the
fracture fragment stabilizes the vascular bed), therefore, it is proper medical
practice to maintain rigid internal fixation in the treatment of
osteomyelitis
b. The goals of treatment are to provide a wide surface contact between the
fracture fragments under high compression, using whatever means is
necessary
c. Dead bone and non-viable soft tissue is removed
d. When bone loss is identified, the defect is filled with decorticated bone
or pure autogenous cancellous bone graft
e. Metal implants and fixation devices should be left undisturbed as long
as they afford rigid immobilization of the fragments. As soon as a
sufficiently strong bony bridge develops both the fixation devices and the
remaining sequestered bone are removed

6. Assessment of fracture healing


a. Standard x-rays
b. Stress fluoroscopy
c. Tomography/CT scans
d. Intraosseous venography: intramedullary injection of Renografin-76,
with sequential x-rays taken at 30 second intervals to access the presence or
absence of medullary flow across the fracture
e. Signs and symptoms: pain, warmth and tenderness

Treatment of Nonunions
The nonunion must be viewed as more than a fracture that has not healed.
There is often edema, pain, joint stiffness and deformity in the bone, resulting
in impaired function. Consequently, the first principle in the repair of
nonunions is the restoration of function. Treatment of nonunions follows 4
basic concepts:
I. Resection of useless tissue is required to allow healing
II. The osteogenic capacity must be augmented by bone grafting
III. The osteogenic capacity must be stimulated by electricity
IV. The bone has the capacity to unite but has had inadequate
immobilization, so the foot now should be adequately immobilized during
healing.
1. Open nonunions vs. closed: When considering the above concepts one must
first consider whether the nonunion is open (with extensive soft tissue
damage) or closed.
a. Open nonunions are less frequent and more difficult to manage
b. Open nonunions frequently require multiple autogenous cancellous grafts
(poor vascularity so cortical is a poor choice).
c. Stabilization of an open nonunion is critical- can use external skeletal
fixators (stability with minimal trauma to surrounding soft tissues)
d. Following the above stages split/full thickness skin grafting can be applied
next over a bed of granulation tissue.
e. The type of treatment of a closed nonunion (most common) depends upon
the type and etiology, which as previously discussed is based on vascularity
f. Proper evaluation of the nonunion is made via technetium scanning
g. Approx. 20% of nonunions require bone grafting
h. Electrical stimulation can be used but requires patient selection (satisfactory
position of bone fragments, no interposed soft tissue, and positive bone scan)

Fusion
The basic concept of fusion is to eliminate motion where motion normally
occurs through primary bone healing. This is accomplished by removing
cartilagenous surfaces, obtaining anatomical apposition and utilizing some
form of rigid fixation to obtain primary bone healing. Primary bone healing
means membranous bone formation not endochondral bone formation,
without evidence of fibrous tissue, cartilage and no evidence of callus
formation. External callus is evidence of motion at the fusion site and
endochondral bone formation.
Fusion occurs quicker in cancellous bone where there are more osteons and.
better blood supply available than in cortical bone.

Bone Grafting
1. There are essentially two types available:
a. Autogenous (isograft): from the same person or twin
b. Allograft: from the same species- two types available fresh and lyophilized
2. Indications for bone grafting are: osteogenesis, Immobilization, and
replacement
a. Treatment of delayed and nonunions, and pseudoarthroses.
b. Augmentation of defects.
c. Facilitation of arthrodesis
d. Bone blocking procedures
e. Reconstructive procedures (for opening wedge and bradymetatarsia
procedures)
f. Autogenous grafting to treat OM

3. Cortical vs. Cancellous


Cortical Cancellous
a. Denser ....................................................................... Less dense
b. Few viable cells .......................................................... Many viable cells
c. Gives stability ............................................................ Fills defects
d. Never revascularizes ................................................... There are vascular
channels that unite at
the graft-host junction immediately
e. Does not facilitate osteogenesis ................................ Facilitates
osteogenesis (by
osteoconduction and osteoinduction
f. Used with fixation devices ........................................ Not used with
fixation devices (soft)
g. Incorporation is slow ................................................. Incorporation is
fast
h. Incompletely repaired/incorporated ......................... Completely
replaced by new bone
i. Allows for creeping substitution ................................ Allows for creeping
substitution
j. Fenestration of graft helpful (drill holes) .... ............. Fenestration not
done
if made parallel to long axis of bone
k. Radiolucency when healing (immediate).... .............. Radiodensity when
healing because it quickly revascularizes
l. Has haversion systems ............................................... No haversion
systems

Note* Corticocancellous grafts (from the iliac crest) gives the best
combination. It looks like the first metatarsal head. Cortical graft looses 80%
of its strength immediately. The weakest point in the cortical graft is at 8
weeks.

4. Autogenous bone
a. is the material for most situations, the advantages are: viable cells and
immunological compatibility
b. the disadvantages are: donor site morbidity, insufficient quantity,
increased OR time, additional risks arising from surgery at the donor site.
c. Soaking in sterile saline prior to use is detrimental; proper short-term
storage should be to place the graft in a closed container covered with a
moistened saline sponge without immersion.
d. Sources are: iliac crest (much bleeding and pain), fibula, and lateral
calcaneus (small amounts).
e. Procedure to remove graft from the calcaneus is as follows:
A lateral incision is made over the calcaneus, posterior to the neutral
triangle, avoiding the sural nerve and deepened by layers to bone. Drill
holes are made outlining a cortical window which is then removed with a
power saw. If cortical bone is not needed it is replaced on the lateral side,
after packing the defect with lyophilized bone and covering the area with
periosteum. Must keep the calcaneus nonweight bearing until evidence of
healing is present.

5. Lyophilized Allograft (bone bank bone)


a. To reduce the antigenicity and increase immunological compatibility the
dead bone is freeze dried; this reduces the moisture content to less than 5%

NOTE* It is important that potential donors do not transmit disease so must be


carefully screened by serological tests, autopsy, and blood and tissue
bacteriological cultures, before and at implantation surgery

b. Lyophilized allografts vs. autogenous isografts

Lyophilized Autogenous
Freeze dried/devoid of water Fresh/water present
Osteogenic precursor present Same
Non-cellular bony matrix Cellular bony matrix
Osteoinductive property is lost Osteoinductive property
present
Requires good recipient bed Not as critical
Allows for healing by creeping Same
substitution
Unlimited amounts Limited amounts
No morbidity of donor site Morbidity of donor site
Less OR time Increased OR time
Try using autogenous first The preferred
material for repair of
nonunions, especially
avascular nonunions

Note* Place lyophilized allografts in Ringer's lactate for 45 minutes prior to use

c. No difference in post-op infection rate of lyophized grafts vs. autogenous


grafts

6. Healing of grafts is by creeping substitution, osteoconduction and


osteoinduction
a. Creeping substitution: the temporal and spatial repair activities whereby
new viable bone replaces necrotic bone.
b. Osteoconduction: the scaffolding effect of the bone graft that acts as a
conduit for migration of viable cells (allows creeping substitution to take
place). c. Osteoindudion : the presence of a bone morpheogenic protein
(inductor substance) that causes nonosseous tissue to become osteogenic.
7. Grafting techniques Include: cortical onlay, peg, inlay, and peg-in-hole
a. Papineau technique used to treat OM and infected nonunions: excision of
necrotic bone, cancellous bone grafting and skin coverage

8. Healing is monitored with x-rays, Tc-99 scans, tomograms and CT. A


gradual blurring of cortical margins occurs, with eventual crossing of the
trabecular pattern across the graft-host junction. Impending disaster is
signaled by sclerosis or dissolution of the graft.

Aseptic Necrosis of the First Metatarsal Head Following Osteotomy


Aseptic bone necrosis following metatarsal surgery can present itself. Stripping
away too much periostium or allowing the bone to "heat up" by the bone
saw are potential sources. A diagnosis can be made by bone scans and later x-
rays, after the onset of pain, swelling, and erythema.
1. Etiology of bone necrosis:
a. Major trauma (osteotomy)
b. Minor trauma (contusion)
c. Steroid therapy
d. Arteritis
e. Occlusive PVD
f. Antecedent phlebitis
g. Post-irradiation
h. S/P renal transplant
i. Hyperuricemia/gout
j. Collagen vascular diseases (RA)
k. Sickle-cell disease
l. Alcoholism/pancreatitis
m. Hyperlipidemia
n. Osteoporosis/osteomalacia

2. Bone Circulatory Networks:


a. Nutrient artery
b. Endosteal artery
c. lntracortical systems
d. Metaphyseal-epiphyseal
e. Diaphyseal
f. Marrow capillaries
g. Periosteal
h. Venous pathw

3. Arterial Circulation: Six groups of vessels


a. Nutrient
b. Periosteal
c. Proximal metaphyseal
d. Proximal epiphyseal
e. Distal metaphyseal
f. Distal epiphyseal
4. Radiological Classification of Bone Necrosis:
a. Stage
1: Pre-radiologic (avascularity/precollapsed)
i. The joint space/epiphyseal contour/trabeculae are WNL
ii. Joint space may be stiff and painful
b. Stage 2: Probable (revascularization phase with bone deposition and
resorption/collapsed)
i. Joint space/epiphyseal contour are WNL
ii. Altered trabecular pattern (osteoporotic/sclerotic/sclero-cystic)
c. Stage 3: Definite (bone healing phase/early and late degenerative arthritic
changes)
i. Joint space is WNL
ii. Epiphyseal contour is disrupted and flattened
iii. Trabecular collapse, arcuate sclerosis, cysts, and sequestrum
d. Stage 4 (phase of deformity)
i. Joint space narrowed (cartilage disruption)
ii. Epiphyseal collapse
iii. Trabecular destruction (sclerosis and cysts)
iv. Epiphyseal collapse

5. Clinical Signs and Symptoms:


a. Stage 1: Usually asymptomatic with minimal pain and/or stiffness
b. Stage 2: Usually significant pain and stiffness, but can be asymptomatic
c. Stage 3: Pain, stiffness, and limitation of motion, but can be asymptomatic

6. Clinical Lab Findings:


a. Elevated ESR (nonspecific finding and not diagnostic)

7. Clinical Radiological Findings:


a. Bone scan: Early (pre-collapse)- localized cold scan
b. Bone Scan: Later- Hot

8. Differential Diagnosis:
a. Arthrosis
b. RSD
c. Infection

9. Treatment of Aseptic Necrosis:


a. Eliminate weight bearing
b. Peripheral vasodilators
c. Indomethacin
d. Avoid steroids
e. Core decompression, autogenous cancellous graft
f. Pedicle muscle graft, decompression, bone graft
g. Chilectomy, decompression or osteotomy, decompression
h. Joint replacement (prosthesis)
i. Arthrodesis
10. Recommendations/Precautions:
a. Preserve capsular and periosteal attachments
b. Accurate hemostasis, avoid tamponade
c. Caution with modified Austin osteotomies (long-arm), Offset V osteotomy,
and long Z osteotomies
d. Caution with distal subchondral osteotomies, especially in the elderly
e. Avoid hemi-implant in combination with distal osteotomy
f. Use stable fixation, protective ambulation
g. Use routine serial radiographs
h. Consider bone scan early

Electrical Bone Stimulation


The use of electrical energy in an effort to effect the healing of bone was
initially described in 1812. A variety of therapeutic devices are currently
available to provide electrical stimulation in the treatment of nonunions of
bone. The discovery of stress related and steady-state electrical potentials,
resulted in the hypothesis that Wolff’s law of bone was in effect mediated by
electrical impulses.
Therefore the application of exogenous electropositive currents to stimulate
osteogenesis is the basis for bone stimulation.
1. A variety of methods have been employed to develop exogenous electrical
potentials:
a. Constant direct current:
i. EBI:
 Implantable
 Contraindicated in osteomyelitis
 Available in either 24 or 36 weeks life span
b. Pulsed direct current:
i. EBI:
 Used for a 6 hour treatment/day
ii. AME:
 3 hour treatment/day
 Has even distribution
c. Capacitively coupled electrical field
i. Bioelectron:
d. Low intensity ultrasound
i. Exogen®

Cartilage Healing
1. Normal Cartilage:
a. Consists of chondrocytes in a glycoprotein hydrated matrix that may
contain collagen and/or elastin
i. Type Il collagen
ii. Ground substance (glycosaminoglycans and water)
b. Nutrition is through synovial fluid (there is no blood supply to the cartilage)

2. Healing phase:
a. Necrosis
b. Inflammation
c. Repair via metaplasia and replication

3. Types of injury:
a. Partial thickness: Usually these injuries do not heal, using shaving and
drilling of articular surface may lead to the development of fibrocartilage
ingrowth
b. Osteochondral heals via:
i. Type II collagen: similar to hyaline cartilage
ii. Type I collagen: fibrocartilage
Chapter 10: Burns and
Frostbite
Assessment and Treatment of Bums
Bum Deformities
Other Types of Burns
Frostbite
BURNS AND FROSTBITE
Assessment and Treatment of Burns

A tissue injury caused by thermal, chemical, or electrical contact


resulting in protein denaturation, wound edema, and loss of
intravascular fluid volume due to increased vascular permeability.
Systemic effects such as hypovolemic shock, infection, or respiratory tract
injury pose a greater threat to life than do local effects.

NOTE* R.J. Hinshaw has proven that the. bum progresses over a 48 hour period
following the initial insult. D.M. Jackson has outlined the histopathology in the
burn wound, showing 3 zones: the zone of hyperemia, zone of stasis, and the zone
of coagulation.

1. The severity of the burn is judged by the quantity of tissue involved,


which is represented by the percentage of the body surface area (% BSA)
burned and by the depth of the burn.
a. Critical burns: Hospitalization required
i. Second degree bums exceeding 30% BSA
ii. Third degree burns exceeding 10% BSA
iii. Burns complicated by respiratory tract injury, major soft tissue injury,
and
fractures
iv. Third degree burns involving critical areas (hands, face, feet)
b. Moderate burns: Hospitalization recommended for observation
i. Superficial second degree of > 15% BSA
ii. Deep second degree burns of 15-30% BSA
iii. Third degree burns of less than 10% BSA excluding the hands, feet, and
face
c. Minor burns: Outpatient management
i. First degree burns
ii. Second degree bums < 15% BSA
iii. Third degree bums < 2% BSA

2. The depth of the burn is described as first, second, third degree, and recently
fourth and fifth degree. Burn depth is difficult to determine in the early
stages of the injury.
a. First degree (epidermis only, does not blister, painful, heals in 5-10 days):
Involves only a redness of the skin, very sensitive to touch, usually moist,
no blisters, and the surface blanches to light pressure. This indicates mild
inflammation. The most common 1st degree burn is sunburn. The rate and
severity at which a person will burn is dependent upon the radiation, the
absorptivity of the skin, and the length of exposure. First degree burns from
fires may result from a much greater amount of radiation received during a
shorter period of time. However, the two do not vary directly. The same
total amount of heat received by radiation may be more damaging if
received during a shorter period of time. Freezing of skin and body tissues
may produce similar effects to those of heat burns
b. Second degree (epidermis and dermis, mostly hyperemic, moist, may
blister, painful, can take a month to heal: More serious than 1st degree
burns with the formation of blisters (usually), and in the more severe cases,
fluid collects under the skin. The skin beneath the blisters is extremely
sensitive, red in color, and exudes considerable amounts of fluid. Second
degree burns are many times more painful than 1st degree burns, since nerve
endings may be exposed (in 3rd degree bums the nerve endings are deadened
by injury and edema). The broken blisters will expose the body to possible
sources of infection
c. Third degree (destruction of entire dermis and all deep epidermal elements,
avascular, pale to black in color, dark red in children, either heals by secondary
intention or needs skin grafting, painless): Generally do not have blisters, the
surface may be white and pliable with pressure, or black and charred and
leathery, the subdermal vessels do not blanch on pressure, and the
area is usually anesthetic/hypoesthetic. The skin, subcutaneous tissue, red
blood cells, capillaries, and sometimes muscle are destroyed. Burns may be
white, light gray, brown or even charred black. Burns caused by direct contact
between the skin and high temperature substances, may be hundreds-of-times
more damaging than heat flow by radiation or convection of air because of
tissue destruction, loss of plasma from the body, and disturbance of the
body's fluid and chemical balance. All of these cause shock. Destruction of
capillaries can cause gangrene. Pain may be present at more shallow wound
edges.
d. Fourth degree: Burning of muscle (electrical burns)
e. Fifth degree: Burning of muscle (electric and radiation burns)

NOTE* Laser Doppler flowmetry may have some advantage in predicting or


delineating areas of burned skin that will heal spontaneously vs. those that
will require skin grafting.

3. 85% of burns are small and may be treated on an outpatient basis, more
extensive burn patients (>1 5% BSA in adults, >10% BSA in children) treated as
inpatients.

4. Objectives of burn wound care:


a. To prevent progression of the injury
b. To prevent infection
c. To provide an environment in which healing will proceed as rapidly as
possible (remove dead tissue and protect viable tissue)
NOTE* According to Jackson's theory, three zones of injury exist:
a. The zone of hyperemia: minimally injured tissue at the periphery of the
bum wound
b. The zone of stasis: less seriously injured tissue which may or may not
necrose
c. The zone of coagulation: the center of the bum consists of dead tissue
which must be debrided

5. Early treatment:
a. The wound should be cooled with cold wet compresses or by immersing it
in ice water, especially if the patient has to wait for debridement. Continue
this treatment as long as the patient does not get frostbite
b. If the wound is not cooled, oxygen free radicals continue to be released by
the drying cells causing arachidonic acid release
6. Treatment of severe burn patients:
a. Inpatient:
i. Stop the burning process: removing all clothing and smoldering material
ii. Establishment of airway: intubate when breathing is rapid and shallow and
patient is in apparent respiratory distress
iii. Replacement of acute plasma loss: especially in 3rd degree burns, Ringer's
lactate first, and plasma later
iv. Manag3m3ociated life-threatening major trauma
v. Diagnosis of metabolic abnormalities especially proteins, fluid, and electrolytes
vi. Protection from bacterial contamination: Pen G 5 million units is given
daily for 3 days as prophylaxis against streptococcal cellulitis. Topical
therapy includes the use of one of the following: silver nitrate solution 0.5%,
mefanide acetate 10%, and silver sulfadiazine 1 %.
vii. Tetanus prophylaxis is mandatory
viii. Control of pain
ix. Operative management: excision of skin (escharotomy) results in burn wound
bed that requires closure with skin graft material Technique of skin grafting is as
follows: the wound is treated with topical antibiotics until the day, before surgery;
saline wet dressings are then applied every 3-4 hours to reduce the debris and
bacterial count of the wound. A wound C&S is taken 48 hours before surgery.
The debridement is carried out surgically with a Goulian or Humby knife to
tangentially excise the eschar until punctate bleeding is encountered. If this
is not available or it is a large area a motor driven dermatome may be used
to debride, the eschar. In small wounds a #20 blade is sufficient. Hemostasis
can be provided with topical thrombin or a lidocaine with epinephrine soaked
gauze. Absolute hemostasis and a bacterial count of less than 105 is essential.
A skin graft is taken with a Pagget or Brown power dermatome, and is meshed
via a meshing machine to permit expansion of the graft. It is rarely necessary to
go beyond an expansion ratio of 1:3 (See section on plastic surgery). A graft
with a larger expansion ratio will give a more disfiguring scar. Staples are used
on the wound edges, and Xeroform® is used to cover the area. A plaster splint is
applied to protect the area from shearing.
b. Outpatient:
i. Remove the burning agent
ii. Wound cleansing
iii. Debridement of the blisters if present and debridement of all debris
iv. Application of Silvadene® cream covered with sterile non-adherent
dressing changed twice a day, thoroughly cleansing the wound with sterile
saline
prior to the next application of antibiotic cream.
v. Elevation of the extremity
vi. Penicillin p.o. or erythromycin or 1st generation cephalosporin
vii. Prophylaxis against tetanus
viii. Analgesics
7. Dressings:
a. Avoid occlusive dressings as they can promote infection
b. For partial thickness, clean superficial burn use a biosynthetic bilaminar
membrane dressing (Biobrane®). Place tightly against the wound and wrap
with compressive gauze (contraindicated in infected wounds)
c. Nonadherent gauze (Xeroform®, Scarlet Red®) if the burn is too old or the
biosynthetic dressing is not available
d. Heterografts (EZ Derm®), a pigskin graft can be used in deep second or dirty third
degree wounds

NOTE* The deeper the bum the more frequently the dressing should be
changed

8. Topical antibacterials: Effective for maintaining low subeschar bacterial


counts
a. Silver sulfadiazine (Silvadene): Poor penetration of eschar, good against
pseudomonas, enterobacter, E. coli, Candida, Staph, avoid in sulpha
anaphylactic allergy
b. Mefenide (Sulfamylon®): Best penetration of eschar, good against
pseudomonas and broad range of bacteria
c. Gentamycin (Garamycin®): Can cause renal toxicity d. Nitrofurazone
(Furacin): Can cause renal toxicity

NOTE* These creams are contraindicated in clean partial thickness wounds


such as donor sites where healing by epithelialization is desired. These
medications may inhibit epithelialization

9. Complications:
a. Adhesions and contractures
b. Late developmental deformities
c. Chronic burn ulceration
d. Squamous cell/basal cell carcinoma, and melanoma
e. Burn scar with nerve entrapment
f. Burn dermatosis, pigmentary changes
g. Polyarthrosis of fingers and toes
h. Traumatic neuralgias and neuropathies

NOTE* Bacteremia is the major cause of death in patients hospitalized with


bums. These infections may result in pulmonary infections, UTI's, and
endocarditis

Burn Deformities
Forces must be exerted to prevent bum scar formation and eventual
contracture. Prevention of deformities begins with the initial care. This
includes consulting a physiatrist to help in the fabrication of splints,
expeditious surgery, aggressive rehabilitation, orthoses, and Jobst® type
pressure stockings.
1. The dorsal foot burn (most common) wound contracture results in
dorsiflexion of the rear foot and hyperextension of the m.p.joints, with the
toes being pulled toward the midaxial line. Plantar deformities consist of
flexion of the foot and toes, and an equinus deformity.
2. Surgery for Late Deformities:
a. This requires precise planning
b. For most burn scar deformities, release of the contracture and split/full
thickness grafting will usually be sufficient. Placing the incision across the area
of maximum tension assures the greatest release.
c. TAL lengthenings can be performed as necessary d. Extensor tenotomies
may or may not be indicated
Other Types of Burns
1. Electrical: Can be either low tension or high tension. The type of current,
amperage, voltage, area of the body through which it passes, duration of the
exposure, and the resistance of the body structures all influence the
magnitude of the injury.
a. Low Tension: When the voltage is less then 1000 V. Mostly seen in the
hand while in the home.
b. High Tension : Usually industrial in nature. The burn is the result when the
electrical energy upon meeting resistance is converted to thermal energy. These
injuries require immediate escharotomy and a decompression fasciotomy to
salvage an extremity.

2. Chemical Burns: Factors which influence the severity of a chemical injury are
the concentration or pH of the solution, the mode of action of the agent, the
vehicle, the volume, and the duration of contact. Emergency treatment of a
chemical injury requires knowing precisely what agent caused the injury and
the immediate application of the proper antidote. In nearly all chemical injuries
the most effective expedient is water (immediate and copious). Water washes
away the chemical, reduces the rate of reaction, mitigates the hygroscopic
action of the chemical, and restores the wound pH to normal.
Frostbite
1. Classifications:
a. By degree:
i. First degree: Freezing without blistering (peeling may be present)
ii. Second degree: Freezing with clear blistering
iii. Third degree: Freezing with skin death (ulcers, blisters, and subcutaneous
involvement)
iv. Fourth degree: Freezing with full hemorrhagic thickness involvement and
ultimate loss or deformity of body part (frostbitten tissue appears white to blue
white, and is firm or hard)
b. Superficial vs deep
i. Superficial frostbite: Chillblains
 Mildest form of cold injury
 Edema of the dermis, vasculitis of the deep and superficial vessels,
blanching followed by white, waxy frozen appearance
 Beneath the superficial skin, tissues are normal
 Bullae may occur after warming
 Rewarm in whirlpool 108°F 20-40 minutes with analgesic support
ii. Deep frostbite:
 Mostly involves the extremities
 Tissues are icy hard without deep tissue resilience (resembles frozen meat) -No
sensation
 Rewarm rapidly in whirlpool as above but with narcotic analgesia
c. Trench foot: Caused by exposure to wet, cold weather, with the temperature
not necessarily below freezing
i. Initially, the foot becomes edematous due to tissue hypoxia and vascular
damage. The tissue becomes cold, white or cyanotic, pulses and sensation are
decreased
ii. The predominating pathology is vasospasm
iii. Warming the extremity causes a cascade of events, which leads to the
second phase, the hyperemic state.
iv. The hyperemic state is one where the extremity is hot, dry, reddened,
marked by bounding pulses and severe pain (this state is often identical to
frostbite)
v. Recovery from trench foot is characterized by decreasing edema and the
return of normal pulses
d. Hypothermia: The basis of hypothermia is dehydration. The difference
between mild and severe hypothermia is the core temperature. Severe
hypothermia (below 90°F) the heart and lungs begin to fail and metabolic
problems appear

2. Rewarming procedure:
a. Before thawing protect the injured part b. Do not rub or massage the area
c. Take core temperature with hypothermia thermometer
d. Rewarm extremities 100-108°F in a whirlpool with thermometer control
e. Leave blisters and blebs intact
f. Administer narcotics prn for pain
g. Do not debride the injury
h. Tetanus prophylaxis
i. Antibiotics?
j. After thawing, do not allow the area to refreeze
k. Protect the area from mechanical damage

3. Complications:
a. Can result in burn-like injuries
b. Sensitivity of the affected part
c. Raynaud's-like phenomenon
d. Muscle atrophy and fat tissue loss
e. Skin texture affected with the epidermis generally dry with fissuring and
depigmentation
f. Punched out lytic lesions in the bone seen on x-ray
g. Paresthesias
h. Hyperhidrosis
i. Pain may be secondary to permanent vasoconstriction, suggesting loss of
autonomic vascular control and resulting in hypoxia to nerve endings and
other local tissues.
Chapter 11: Bone Tumors
Introduction
Staging Techniques
Surgical Staging System
Radiology of Bone Tumors Bone Tumors
Cystic Lesions of Bone
Other Tumors & Cancers
Bone Tumors: A Quick Reference Chart Phases in Cancer
Development Benign vs. Malignant
TUMORS OF BONE, BONE MARROW
AND CARTILAGE
Introduction
Diagnosis of bone tumors is not the singular effort of the clinician, but rather the work of a team
consisting of attending surgeon, radiologist, and pathologist who will evaluate the history,
provide a comprehensive examination, and order appropriate tests. Differential diagnosis on x-
ray is an interesting academic excercise, but actual diagnosis is more important. Radiologic study
has been compared to physical examination of the lesion in its context. Laboratory adjunctive
testing will provide clues to some conditions. MRI and scans will help to delineate tumors.
Radionuclide uptake procedures can help to locate occult metastatic or synchronous lesions.
Occasionally, even angiography can be helpful.
Bone tumors are either primary or metastatic. Primary bone tumors can be either benign or
malignant. After a complete history and physical examination, and lab studies, the
management of bone tumors can be divided into 3 phases: (1) radiographic staging, (2) diagnosis,
and (3) treatment. The radiographic staging is designed to provide information concerning the
diagnosis and the anatomic extent of the lesion. The definitive diagnosis always requires
biopsy. Only after these first 2 phases are complete can a rational treatment plan be established
and carried out.
In formulating a treatment plan, the. histogenetic type of tumor, local extent, and possibility of
distant disease must be considered. Clinical factors such as age, size, occupation, life-style, and
expectations also play a key role in the treatment options available
Staging Techniques
The purpose of radiographic staging of a bone tumor is two-fold. The first is to obtain
information concerning a possible diagnosis, and the second is to define the anatomical extent
of the lesion
1. Staging techniques:
a. Plain radiograph demonstrates the bone involved, the region of the bone, the extent and type
of destruction, and the amount of reactive bone formed. It also gives some information of
benign vs malignant tumors (plain film x-rays are a must in the staging of bone tumors)
b. Radioisotope scans (99m Tc MDP) can estimate the local intramedullary extent of the tumor
and screen for other skeletal areas of involvement. It can give information about the biologic
aggressiveness of the tumor via its uptake. This study is also important in post-treatment followup
c. CT: Greatly aids in the diagnosis and anatomic location. Using contrast with CT aids in the
identification of the major neurovascular structures as well as the enhancement of well-
vascularized lesions. CT is the best study for evaluating cortical penetration and osseous detail, and
valuable in the assessment of matrix calcification or ossification. CT is the best technique for
detecting pulmonary metastasis, replacing whole lung tomography.
d. MRI: The best information is obtained when MRI and CT are interpreted together. MRI
more accurately demonstrates the local intramedullary extent of tumors than with gallium-
67 citrate or any other method. MRI is the best technique for demarcating the soft tissue
component of bone tumor and its relationship to neurovascular structures without utilization
of contrast material
e. Arteriography: Since the advent of CT anti MRI with contrast, this technique is not used
much, except in a difficult anatomic location such as the shoulder girdle or pelvis
f. Biopsy:, The purpose of biopsy is to obtain adequate tissue for accurate diagnosis and
grading while avoiding potential detrimental effects to the patient or compromise of the
definitive surgical procedure
Surgical Staging System
The information generated by the history, physical examination, radiographic studies, and
biopsy has been incorporated into a staging system for musculosketetal tumors. The surgical
staging system reflects the biologic behavior of the lesion and its degree of aggressiveness. It
is based on a combination of the histologic grade (G), anatomic site (T), and the presence of
distant metastasis (M) 1. Staging system:
a. Grade (G): Divided into benign (GO), low grade malignant (G1), and high grade malignant
(G2). Grade is based upon a combination of histologic, radiographic, and cytologic
characteristics and biologic characteristics
b. Anatomic site (T): Is divided into intracapsular (TO),
extracapsular /intracompartmentaI (T1), and extracapsular/ extracompartmentaI
(T2)
c. The stages of malignant lesions are designated by numerals 1, II, and III
Radiology of Bone Tumors
1. Making a diagnosis: When attempting to diagnosis bone tumors it is important to know:
the age and sex of the patient, which bone is involved, is it well demarcated, its size and
shape and location, type of periosteal reaction, any soft tissue involvement, any pain and
what relieves the pain, pattern of bone destruction, appearance of the tumor matrix,
appearance of trabeculation, cortical erosion penetration or expansion, and a good medical
history. a. Pattern of destruction: geographic, motheaten or permeative
i. Geographic- usually seen with benign tumors, least aggressive pattern of bone destruction,
indicative of a slow growing lesion, has a smooth or irregular but well-defined margin which
is easily separated from the surrounding normal bone by a short zone of transition. Some
metastatics can look like this.
ii. Motheaten- may be demonstrated by a malignant bone tumor, OM, or . hematopoietic
tumors, indicative of rapidly growing lesion, less well-defined with a longer zone of
transition from normal to abnormal bone
iii. Permeative- demonstrated by certain malignant bone tumors and rarely OM, most
aggressive pattern of bone destruction, rapid growth potential, poorly demarcated and not
easily separated from the normal surrounding bone b. Size, shape and the margin of the
tumor
1. Size- in general, malignant primary bone tumors are larger than benign ones and when
first discovered primary malignant tumors may be greater than 6 cm in size (in some cases)
ii. Shape- tumor with greatest diameter at least 1.5 times its least diameter may include
Ewing's sarcoma, chondrosarcoma and large cell lymphoma (reticulum cell sarcoma)
iii. Margin- sclerosis of the surrounding normal bone indicates a slow growing lesion,
indicating pressure with periosteal or endosteal reaction c. Tumor matrix: calcification or
ossification
1. Matrix calcification- cartilage tumor, may appear centrally located, ring-like, or fleck-like
radiodensity, seen with chondromas, chondrosarcomas and chondroblastomas
ii. Matrix ossification- bone tumor, seen with osteoid osteoma and osteogenic sarcoma
d. Trabeculation:
i. Giant cell- delicate and thin
ii. Chondromyxoid fibroma- course and thick
iii. Aneurysmal bone cyst- delicate and horizontal iv. Non-ossifying fibroma- loculated
v. Hemangioma- striated and radiating
e. Cortical Erosion: Penetration and Expansion
The bone cortex serves as a barrier to the further lateral growth of "certain tumors such as non-
aggressive medullary lesions which generally expand in the direction of least resistance
within the medullary canal. Other lesions, however, may penetrate the cortex in variable
lengths to those aggressive bone lesions which may penetrate the entire thickness of the
cortex in one or more places
f. Type of periosteal response: Buttressing, Codman's triangle, sunburst, onion skin and hair-on-
end
i. Buttressing (thick periostitis)- indicative of a slow growing tumor whereby an expansile mass
presses against the periosteum, which thickens the cortex with new layers of bone
ii. Codman's triangle- a triangular elevation of the periostium seen in osteogenic sarcoma
and other conditions (see Quick Reference Chart)
iii. Sunburst pattern- delicate rays of periosteal bone formation separated by spaces containing
blood vessels, seen with hemangioma, Ewing's sarcoma and osteogenic sarcoma
iv. Onion skin pattern- production of multiple layers of new periosteal bone,
exemplified by Ewing's sarcoma, eosinophilic granuloma, and lymphoma of bone
v. Hair-On-End pattern- similar to sunburst pattern, indicating the most malignant type of
tumor, where rays of periosteal bone project in a perpendicular direction to the underlying
bone (Ewing's sarcoma)
g. Part of the bone involved: Epiphyses, metaphyses or diaphysis
i. Epiphyses- chondroblastoma, giant cell tumor, subchondral cyst, and hemangioma
ii. Metaphyses- solitary bone cyst, osteogenic sarcoma, chondromyxoid fibroma, non-
ossifying fibroma, and chondrosarcoma
iii. Diaphysis- solitary bone cyst, aneurysmal bone cyst, non-ossifying fibroma, Ewing's
sarcoma, enchondroma, fibrous dysplasia, giant cell tumor, and eosinophilic granuloma
Bone Tumors
1. Osteoma (hyperostosis, periosteal osteoma):
a. Is a benign, reactive, expansile tumor-like lesion that may be pedunculated
b. It usually arises from intramembranous bones and may occur posttraumatically
c. Some may represent osteochondromas whose cartilage cap has ossified d. May occur
subungually
e. These may be:
i. Ivory: compact
ii. Spongy: trabecular
iii. Mixed
f. Multiple osteomas may occur in Gardner's Syndrome g. Excision is curative

2. Enostosis:
a. An uncommon, benign, intramedullary island of ossification
b. It may be reminiscent of osteoblastic metastatic visceral cancers c. These are often
asymptomatic, incidental radiologic findings
d. They may occur with Multiple Enchondromatosis and with keloid formation

3. Osteoid Osteoma:
a. Is a benign, osteoblastic, slow-growing lesion that may occur in cortical or medullary
bone
b. The soft tissue center is its NIDUS
c. Some (but not all) patients experience nocturnal, intermittent pain that becomes a
"boring" type of constant pain
d. It many cases (but not all), pain is relieved by salicylates
e. X-rays often show a sclerotic rim of bone around the radiolucent nidus f. Sometimes
the nidus is sclerotic and not visable unless MRI or scans are done
g. Pain may be referred to adjacent joints
h. Lesions adjacent to joints may erode the cartilage
i. Surgery consists of en-bloc resection to include the nidus, (x-rays should be done of the
specimen removed to make sure the nidus was removed) j. Lesions usually are under 2 cm
in diameter, affecting patients in the 1025 year age group (mostly males)

4. Osteoblastoma (osteogenic fibroma, spindle cell variant of giant cell tumor, giant osteoid
osteoma):
a. Is a benign, fast-growing tumor that usually affects males (ages 10-20) b. Pain is
not relieved by salicylates in most cases c. Lesions are usually metaphyseal or diaphyseal
d. It usually exceeds 2 cm in diameter, and lacks the sclerotic rim around the soft tissue
center
e. Older lesions may show a mottled pattern of thin calcifications
Chapter 12: Bone Tumors
Introduction
Staging Techniques
Surgical Staging System
Radiology of Bone Tumors Bone Tumors
Cystic Lesions of Bone
Other Tumors & Cancers
Bone Tumors: A Quick Reference Chart Phases in
Cancer Development Benign vs. Malignant
TUMORS OF BONE, BONE MARROW
AND CARTILAGE

Introduction
Diagnosis of bone tumors is not the singular effort of the clinician, but rather
the work of a team consisting of attending surgeon, radiologist, and
pathologist who will evaluate the history, provide a comprehensive
examination, and order appropriate tests. Differential diagnosis on x-ray is
an interesting academic excercise, but actual diagnosis is more important.
Radiologic study has been compared to physical examination of the lesion in its
context. Laboratory adjunctive testing will provide clues to some conditions.
MRI and scans will help to delineate tumors. Radionuclide uptake procedures
can help to locate occult metastatic or synchronous lesions. Occasionally,
even angiography can be helpful.
Bone tumors are either primary or metastatic. Primary bone tumors can
be either benign or malignant. After a complete history and physical
examination, and lab studies, the management of bone tumors can be
divided into 3 phases: (1) radiographic staging, (2) diagnosis, and (3) treatment.
The radiographic staging is designed to provide information concerning the
diagnosis and the anatomic extent of the lesion. The definitive diagnosis always
requires biopsy. Only after these first 2 phases are complete can a rational
treatment plan be established and carried out.
In formulating a treatment plan, the. histogenetic type of tumor, local extent, and
possibility of distant disease must be considered. Clinical factors such as age, size,
occupation, life-style, and expectations also play a key role in the treatment
options available
Staging Techniques
The purpose of radiographic staging of a bone tumor is two-fold. The first is
to obtain information concerning a possible diagnosis, and the second is to
define the anatomical extent of the lesion
1. Staging techniques:
a. Plain radiograph demonstrates the bone involved, the region of the bone, the
extent and type of destruction, and the amount of reactive bone formed. It also
gives some information of benign vs malignant tumors (plain film x-rays are a
must in the staging of bone tumors)
b. Radioisotope scans (99m Tc MDP) can estimate the local
intramedullary extent of the tumor and screen for other skeletal areas of
involvement. It can give information about the biologic aggressiveness of the
tumor via its uptake. This study is also important in post-treatment
followup
c. CT: Greatly aids in the diagnosis and anatomic location. Using contrast with
CT aids in the identification of the major neurovascular structures as
well as the enhancement of well-vascularized lesions. CT is the best study for
evaluating cortical penetration and osseous detail, and valuable in the
assessment of matrix calcification or ossification. CT is the best technique for
detecting pulmonary metastasis, replacing whole lung tomography.
d. MRI: The best information is obtained when MRI and CT are Interpreted
together. MRI more accurately demonstrates the local intramedullary extent of
tumors than with gallium-67 citrate or any other method. MRI is the best
technique for demarcating the soft tissue component of bone tumor and its
relationship to neurovascular structures without utilization of contrast material
e. Arteriography: Since the advent of CT anti MRI with contrast, this technique
is not used much, except in a difficult anatomic location such as the shoulder
girdle or pelvis
f. Biopsy:, The purpose of biopsy is to obtain adequate tissue for accurate diagnosis
and grading while avoiding potential detrimental effects to the patient or
compromise of the definitive surgical procedure

Surgical Staging System


The information generated by the history, physical examination, radiographic
studies, and biopsy has been incorporated into a staging system for
musculosketetal tumors. The surgical staging system reflects the biologic
behavior of the lesion and its degree of aggressiveness. It is based on a
combination of the histologic grade (G), anatomic site (T), and the presence of
distant metastasis (M)
1. Staging system:
a. Grade (G): Divided into benign (GO), low grade malignant (G1), and high
grade malignant (G2). Grade is based upon a combination of histologic,
radiographic, and cytologic characteristics and biologic characteristics
b. Anatomic site (T): Is divided into intracapsular (To),
extracapsular/intracompartmental (Ti), and extracapsular/
extracompartmental
(T2)
c. The stages of malignant lesions are designated by numerals I, II, and III

Radiology of Bone Tumors


1. Making a diagnosis: When attempting to diagnosis bone tumors it is
important to know: the age and sex of the patient, which bone is involved, is
it well demarcated, its size and shape and location, type of periosteal reaction,
any soft tissue involvement, any pain and what relieves the pain, pattern of
bone destruction, appearance of the tumor matrix, appearance of
trabeculation, cortical erosion penetration or expansion, and a good medical
history. a. Pattern of destruction: geographic, motheaten or permeative
i. Geographic- usually seen with benign tumors, least aggressive pattern of
bone destruction, indicative of a slow growing lesion, has a smooth or
irregular but well-defined margin which is easily separated from the
surrounding normal bone by a short zone of transition. Some metastatics can
look like this.
ii. Motheaten- may be demonstrated by a malignant bone tumor, OM, or
hematopoietic tumors, indicative of rapidly growing lesion, less well-defined
with a longer zone of transition from normal to abnormal bone
iii. Permeative- demonstrated by certain malignant bone tumors and rarely OM,
most aggressive pattern of bone destruction, rapid growth potential, poorly
demarcated and not easily separated from the normal surrounding bone b.
Size, shape and the margin of the tumor
i. Size- in general, malignant primary bone tumors are larger than benign ones
and when first discovered primary malignant tumors may be greater than 6 cm
in size (in some cases)
ii. Shape- tumor with greatest diameter at least 1.5 times its least diameter may
include Ewing's sarcoma, chondrosarcoma and large cell lymphoma (reticulum
cell sarcoma)
iii. Margin- sclerosis of the surrounding normal bone indicates a slow growing
lesion, indicating pressure with periosteal or endosteal reaction
c. Tumor matrix: calcification or ossification
i. Matrix calcification- cartilage tumor, may appear centrally located, ring-like, or
fleck-like radiodensity, seen with chondromas, chondrosarcomas and
chondroblastomas
ii. Matrix ossification- bone tumor, seen with osteoid osteoma and osteogenic
sarcoma
d. Trabeculation:
i. Giant cell- delicate and thin
ii. Chondromyxoid fibroma- course and thick
iii. Aneurysmal bone cyst- delicate and horizontal
iv. Non-ossifying fibroma- loculated
v. Hemangioma- striated and radiating
e. Cortical Erosion: Penetration and Expansion
The bone cortex serves as a barrier to the further lateral growth of certain tumors
such as non-aggressive medullary lesions which generally expand in the
direction of least resistance within the medullary canal. Other lesions,
however, may penetrate the cortex in variable lengths to those aggressive bone
lesions which may penetrate the entire thickness of the cortex in one or more
places
f. Type of periosteal response: Buttressing, Codman's triangle, sunburst, onion skin
and hair-on-end
i. Buttressing (thick periostitis)- indicative of a slow growing tumor whereby an
expansile mass presses against the periosteum, which thickens the
cortex with new layers of bone
ii. Codman's triangle- a triangular elevation of the periostium seen in
osteogenic sarcoma and other conditions (see Quick Reference Chart)
iii. Sunburst pattern- delicate rays of periosteal bone formation separated by spaces
containing blood vessels, seen with hemangioma, Ewing's sarcoma and
osteogenic sarcoma
iv. Onion skin pattern- production of multiple layers of new periosteal bone,
exemplified by Ewing's sarcoma, eosinophilic granuloma, and lymphoma of
bone
v. Hair-On-End pattern- similar to sunburst pattern, indicating the most malignant
type of tumor, where rays of periosteal bone project in a perpendicular
direction to the underlying bone (Ewing's sarcoma)
g. Part of the bone involved: Epiphyses, metaphyses or diaphysis
i. Epiphyses- chondroblastoma, giant cell tumor, subchondral cyst, and
hemangioma
ii. Metaphyses- solitary bone cyst, osteogenic sarcoma, chondromyxoid
fibroma, non-ossifying fibroma, and chondrosarcoma
iii. Diaphysis- solitary bone cyst, aneurysmal bone cyst, non-ossifying fibroma,
Ewing's sarcoma, enchondroma, fibrous dysplasia, giant cell tumor, and
eosinophilic granuloma

Bone Tumors
1. Osteoma (hyperostosis, periosteal osteoma):
a. Is a benign, reactive, expansile tumor-like lesion that may be
pedunculated
b. It usually arises from intramembranous bones and may occur post-
traumatically
c. Some may represent osteochondromas whose cartilage cap has ossified
d. May occur subungually
e. These may be:
i. Ivory: compact
ii. Spongy: trabecular
iii. Mixed
f. Multiple osteomas may occur in Gardner's Syndrome
g. Excision is curative

2. Enostosis:
a. An uncommon, benign, intramedullary island of ossification
b. It may be reminiscent of osteoblastic metastatic visceral cancers c. These
are often asymptomatic, incidental radiologic findings
d. They may occur with Multiple Enchondromatosis and with keloid
formation

3. Osteold Osteoma:
a. Is a benign, osteoblastic, slow-growing lesion that may occur in cortical
or medullary bone
b. The soft tissue center is its NIDUS
c. Some (but not all) patients experience nocturnal, intermittent pain
that becomes a "boring" type of constant pain
d. It many cases (but not all), pain is relieved by salicylates (asprin)
e. X-rays often show a sclerotic rim of bone around the radiolucent nidus
f. Sometimes the nidus is sclerotic and not visable unless MRI or scans are
done
g. Pain may be referred to adjacent joints
h. Lesions adjacent to joints may erode the cartilage
i. Surgery consists of en-bloc resection to include the nidus, (x-rays should
be done of the specimen removed to make sure the nidus was removed)
j. Lesions usually are under 2 cm in diameter, affecting patients in the 10-
25 year age group (mostly males)
k. Associated pain relieved by apsrin

4. Osteoblastoma (osteogenic fibroma, spindle cell variant of giant cell tumor,


giant osteoid osteoma):
a. Is a benign, fast-growing tumor that usually affects males (ages 10-20)
b. Pain is not relieved by salicylates in most cases
c. Lesions are usually metaphyseal or diaphyseal
d. It usually exceeds 2 cm in diameter, and lacks the sclerotic rim around
the soft tissue center
e. Older lesions may show a mottled pattern of thin calcifications
f. There is a debate as to whether these lesions can undergo malignant
transformation

5. Osteogenic Sarcoma (osteosarcoma):


a. Is the second most common primary malignant bone tumor of the body
b. It is not common in the feet (occurs 2% of the time)
c. The most common site is in the region of the knee
d. Most lesions are solitary, a few are multiple, metachronous or synchronous
e. Most patients are teenage males
f. Cortical lesions seem to have a better prognosis than medullary lesions
g. There are many subtypes and radiographic appearances vary
h. Codman's triangle in x-rays represents a cuff of subperiosteal new bone at
the borders of a lesion that rapidly elevates periosteum, forming a bone
spicule

NOTE* Codman's triangle is seen in osteogenic sarcoma, but is not diagnostic


for it as it is also seen in other bone lesions, including chondrosarcoma,
Ewing's sarcoma, giant cell tumor of bone, simple bone cyst and any
sarcomas arising in the context of Paget's disease of bone

i. The "sunburst" appearance (on x-ray) of some early osteogenic sarcomas also
is not diagnostic, since it can also be seen in Ewing's sarcoma and in
hemangioma of bone
j. The serum alkaline phosphatase is quite elevated in active osteogenic
sarcoma and can be used as an index of recurrence or late metastasis
(i.e. if it rises from normal levels after tumor resection)
k. Secondary osteogenic sarcoma arises after age 40 from pre-existing conditions
such as Paget's disease of bone, fibrous dysplasia of bone, nonossifying
fibroma of bone, bone infarcts, solitary osteochondroma, and multiple
enchondromatosis
l. Radiographic diagnostic aids include angiography, MRI and Gallium scans

NOTE* Extraosseous osteogenic sarcoma is a well recognized entity. The rare


juxtacortical type may show a periosteal "string sign" on x-rays

6. Osteochondroma (osteocartilaginous exostosis):


a. Is the most common benign bone tumor
b. The most frequent form in the foot is the subungual (Dupuytren's) exostosis
c. Multiple forms exist and are known as multiple cartilaginous exostoses,
hereditary multiple exostosis, hereditary deforming dyschondroplasia, and
diaphyseal aclasis
d. While the classic type has a hyaline cartilage cap, the subungual types have
fibrocartilage caps
e. Excision is curative if the overlying soft tissue which acts as perichondrium
(and therefore is a source of recurrence) is removed with the lesion
f. Malignant transformation is rare, but is recorded

7 . Chondromas:
a. Are benign neoplasms
b. Solitary enchondromas appear as intramedullary (central) lucencies
derived from rests of epiphyseal cartilage
c. They may become symptomatic after local trauma, usually in adults
between 30 & 40 years of age
d. Pain without history of local trauma suggests malignant transformation
e. Multiple enchrondatosis (Ollier's disease) is a cartilage dysplasia that can
be quite deforming, and carries a high risk for malignant transformation
f. Multiple enchrondromatosis occur as part of Maffuci's (Kast-Maffuci)
syndrome (angiochondromatosis). Persons with this syndrome may show
areas of vitiligo, as well as pigmented lesions
g. Metachondromatosis is a dominantly inherited mime of multiple
enchondromatosis, but the lesions regress
h. Periosteal (juxtacortical/eccentric) chondroma is a rare lesion and is also
called ecchondroma
i. The lesions are sharply circumscribed on x-ray
j. Size and location determine surgical approach

8. Chondroblastoma (Codman tumor):


a. Is a benign lesion of immature cartilage
b. Most lesions are epiphyseal
c. Most patients are males, ages 10-20 years
d. Secondary changes may resemble aneurysmal bone cyst
e. Lesions are painful and appear cystic on x-ray, with a thin overlying
shell
f. Fine trabeculations ("chickenwire calcifications") may present in older
lesions
g. Curettage followed by packing with bone chips may be curative

NOTE* Some prefer curettage followed by freezing with liquid nitrogen


and packing with bone chips

h. Recurrences are more frequent when there is a concurrent aneurysmal


bone cyst or arteriovenous malformation
i. Radiation therapy may work, but presents a risk of radiation-induced
sarcoma

9. Chondromyxoid Fibroma:
a. In general, is a rare tumor, but does affect the feet
b. Most frequently seen between the ages of 10 & 20 years
c. Foot lesions seem to be more frequent in the 30-40 age group
d. These are painful lesions that are lucent on x-rays, and which may
appear lobular or bubbly
e. Some lesions are quite aggressive
f. En-bloc resection or amputation may be necessary

10. Chondrosarcoma:
a. Is a malignant, potentially metastatic cartilage tumor that may show
myxoid and/or osseous elements
b. It Is the third most common malignant bone tumor
c. Most frequently seen between the ages of 40 & 60
d. Lesions may be primary or secondary, the latter arising in such pre-
existing lesions as Paget's disease of bone, osteochondroma (single or multiple),
Maffuci's (Kast-Maffuci) syndrome, Ollier's multiple enchondromatosis, and bone
cysts
e. The types range from the well differentiated through the clear cell and
mesenchymal (anaplastic) types
f. Patients demonstrate a painful limp and a mass may be palpable
g. On x-ray, the tumors may be lucent or. may demonstrate calcifications
h. Angiograms, scans, and MRI are helpful adjuncts
i. Amputation is the indicated treatment,- the level depending upon the
location and size of the tumor

11. Desmoplastic Fibroma of Bone (endosteal fibroma):


a. Is rare and occurs between the ages of 10 & 20 years of age (in most cases)
b. The borders appear indistinct on x-ray, suggesting malignancy
c. These may be aggressive (although benign) causing pathologic fractures and
pain
d. En-bloc resection with bone grafting seems the best approach to prevent
recurrence
e. Periosteal desmoids are related and are aggressive, and have to be differentiated
from parosteal fasciitis

12. Fibrous Dysplasia of Bone:


a. May be monostotic or polyostotic (the polyostotic form affects the feet the
feet more often than the monostotic form, and may be part of the McCune-
Albright syndrome in some cases)
b. There may be pain and swelling with pathologic fractures
c. The lucent lesion has a sclerotic rim and may expand surrounding bone
d. Polyostotic forms may give rise to osteogenic sarcoma, chondrosarcoma, or
intraosseous fibrosarcoma
e. Monostotic forms may be cured by en-bloc resection and packing with bone
chips, or curettage depending upon size and location of the lesion

13. Nonossifying Fibroma:


a. Is a benign process seen most frequently between the ages of 10 El 20 years
b. Occurs in the metaphyseal areas of bone
c. It is the active and proliferating form of fibrous cortical defect of infants
and young children ("metaphyseal fibrous defect")
d. Symptoms may be absent, or with growth of the lesion, there may be pain,
swelling, and pathologic fracture
e. X-rays show a lucent, eccentric lesion that may be loculated within a
sclerotic rim
f. Treatment consists of curettage and packing with bone chips
g. Fibrous cortical defect may be associated with osteochondritides of the tibia
and femur

14. Paget's Disease of Bone:


a. Is a benign, premalignant condition of bone that may be monostotic or
polyostotic and which is rare in the feet
b. Most cases occur after age 40 years of age
c. There are well defined, sharply bordered areas of bone resorption and bone
deposition, the latter in a haphazard structure ("woven bone")
d. The osteoid and bone deposition is sufficient to thicken the affected bones,
sometimes causing deformities
e. Some suspect a viral infection of osteoclasts as the etiology
f. In the active stages, levels of urinary hydroxyproline are significantly
elevated
g. The highest levels of serum alkaline phosphatase occur in this condition
h. Development of pain may herald malignant transformation
i. Paget's sarcoma refers to any sarcoma that develops in the setting of known
Paget's disease of bone (these include osteogenic sarcoma, chondrosarcoma,
fibrosarcoma, and malignant fibrous histiocytoma)

15. Eosinophilic Granuloma (Langerhans cell granulomatosis, Taratynov's disease):


a. Is an uncommon bone lesion that may occur as solitary or multifocal
lesions
b. Involvement of foot bones is extremely rare
c. Once considered malignant, this condition is now considered benign even
though irradiation and chemotherapy are used in some cases of multifocal
disease
d. Diagnosis is confirmed by open biopsy, during which small defects
may be curetted and packed with bone chips
e. A "tempo phenomenon" is seen, such that osseous lesions come and go
without treatment
f. Large lesions may require en-bloc resection
g. On x-ray, lesions may appear "punched out" or irregular and ragged
h. Lesion edges may appear slanted or bevelled, suggesting depth to the lesion
i. Overlapping lucent lesions suggest a "hole within a hole"

16. Soft Tissue Malignant Tumors:


a. Occur as primary bone lesions and differ from their soft tissue counterparts
only by location
b. Examples include intraosseous fibrosarcoma, malignant fibrous histiocytoma,
leiomyosarcoma, angiosarcoma, Kaposi's sarcoma, adamantinoma, and
angioendothelioma
NOTE* Secondary malignant bone tumors are metastases from visceral
carcinomas or other soft tissue sarcomas. These intraosseous malignancies
cause pain and bone destruction, appearing irregular on x-ray

17. Soft Tissue Benign Tumors:


a. May also arise as primary bone tumors
b. These include hemangiomas (solitary or multiple), lymphangiomas, glomus
tumors, and giant cell tumors of bone
c. Tumor-like lesions affecting bone include penetrating epidermal inclusion
cysts, pseudoepitheliomatous hyperplasia in fistulas leading from bone,
hypertrophic pulmonary osteoarthropathy, and giant cell reparative
granuloma of the digits (considered by some to be a variant of giant cell tumor
of bone)

Cystic Lesions of Bone


These have to be differentiated from bone tumors
1. Unicameral (Simple) Bone Cyst:
a. Generally are asymptomatic lytic lesions, that are thought to be
interosseous synovial cysts as a result of entrapment if synovium during bone
development
b. Classically, these have been treated by curettge and packing. Curettings are
very sparse. Cryosurgery is risky due to possible damage to nearby epiphyses
c. Recently, treatment success has been reported with one or more aspirations
of fluid followed by introduction of a steroid
d. Most cases arise in males over-20 years old
e. Those in the calcaneus arise in Ravelli's triangle
f. Pain may suggest a fracture of the wall, and a "fallen fragment sign"
represents a fracture chip within the cyst

2. Intraosseous Ganglion:
a. May give a similar x-ray appearance as a unicameral cyst
b. May be painful
c. A synovial cell lining is seen in the excised tissue (which is not seen in the
curettings from a simple bone cyst)

3. Aneurysmal Bone Cyst:


a. These are arteriovenous malformations grafted onto some other lesion, such
as a chondroblastoma, chondromyxoid fibroma, giant cell tumor of bone,
osteoblastoma, fibrous dysplasia and hemangioma of bone
b. The cyst is filled with unclotted blood
c. Curettage and packing yields a high recurrence rate
d. Cryosurgery can be more successful if bleeding is stopped first, so that there are no
warm areas where tissue can survive the treatment
e. Irradiation is the last resort
f. For long bones en-bloc resection followed by packing with bone chips may
be warranted
g. Radiographic examination supplimented by MRI can aid in the diagnosis
h. These cysts lack a sclerotic wall, and may demonstrate flecks of bone
formation as well as septae
i. These cysts show irregular borders and may cause bulging of the overlying
cortex
j. Sometimes a Codman's triangle is present
k. It is a very rapidly growing lesion, hence, it shoes a "quick tempo
phenomenon"

Other Tumors and Cancers


1. Ewing's Sarcoma:
a. Is a very malignant, small-cell bone tumor whose histogenesis is unclear
b. It is rare in the black population
c. It is the fourth most common malignant bone tumor, and is seen most
frequently under the age of 20 years
d. Symptoms and signs are nonspecific, and pathologic fractures may occur
e. On x-rays, nonspecific "onionskin" changes and lytic lesions may be
present, and affected areas may be swollen, painful, and tender
f. Primary treatment is amputation since irradiation may result in changes
causing pain and impaired function
g. The foot can (uncommonly) be the primary site, so that consultations are
necessary to determine (and watch for) lesions elsewhere
h. Chemotherapy is also used in treatment

2. Leukemias and Lymphomas:


a. Rarely arise as primary intraosseous foot lesions but may appear there as
secondary lesions
b. Bones may show rarefactions and cortical thinning
c. Affected bones may be painful and tender, and pathologic fractures may
occur
d. The patient may be anemic, debilitated and otherwise quite ill
e. Lymph nodes and spleen may be enlarged
f. The CBC, differential, bone marrow smear, biopsy and lymph node biopsy
may establish the diagnosis

3. Plasma Cell Dyscrasias (multiple myeloma):


a. This can manifest itself in the feet
b. Multiple myeloma is the most common primary malignant bone tumor
c. Most cases occur in males 40-70 years old
d. Bone pain (worse on weightbearing) and tenderness are present
e. Lytic "punched out" bone lesions may be seen in earlier stages, the lytic
lesions being more diffuse later
f. Pathological fractures may occur
g. Serum protein electrophoresis reveals a monoclonal gammopathy
h. Urine shows Bence-Jones protein
i. Amyloid is produced and carpel tunnel syndrome may develop
j. There is a progressive anemia, erythrocyte aggregation (Rouleaux
formation), cryoglobulinemia, hypercalcemia, reversal of the A/G ratio, M
proteinemia (light chains) and an elevated erythrocyte sedimentation rate
k. Skin manifestations may include plane normolipemic xanthoma and
necrobiotic xanthogranuloma, the latter (when it does occur) often
precedes the onset of the disease

4. Metastatic Cancers:
a. Metastatic cancers from visceral organs may occur in the bones of the
feet
b. They may be osteolytic or osteogenic
c. These are rapidly developing lesions that may cause pain and pathologic
fractures
d. There are no specific signs or symptoms, but a history of treatment for a
malignant tumor (even years earlier) may provide a clue

NOTE* Of course, even with such a history, a suspect lesion may not be a
metastasis, it may be an independent and entirely coincidental lesion

Bone Tumors: A Quick Reference Chart


1. Most Common Malignant Primary Bone Tumors:
#1 multiple myeloma
#2 osteogenic sarcoma
#3 chondrosarcoma
#4 Ewing's sarcoma

NOTE* The most common bone tumor is metastatic tumors from other
sites, but this group does not constitute primary malignancy in bone

II. Most Common Benign Primary Bone Tumor:


#1 osteochondroma (bone spur, subungual exostosis, and other variants)

III. "Onionskin" Periosteal Reactivity Can Be Seen In:


#1 osteomyelitis (especially Garre sclerosing type
#2 eosinophilic granuloma
#3 Ewing's sarcoma
#4, lymphoma of bone

IV. Codman's Triangle Is Seen In Some Cases Of:


#1 osteogenic sarcoma
#2 chondrosarcoma
#3 Ewing's sarcoma
#4 "Paget's" sarcoma
#5 unicameral bone cysts
#6 giant cell tumor of bone

V. "Sunburst" Periosteal Reaction Can Be Seen In:


#1 osteomyelitis
#2 Ewing's sarcoma
#3 hemangioma of bone

VI. "Paget's" sarcomas that arise in bone In the setting of Paget's disease of bone:
#1 osteogenic sarcoma
#2 chondrosarcoma
#3 fibrosarcoma
#4 malignant fibrous histiocytoma

NOTE* They may also arise in bone independent of Paget's disease of bone

VII. "Tempo Phenomenon" Is Associated With:


#1 aneurysmal bone cyst: (rapid growth=quick tempo phenomenon)
#2 eosinophilic (Langerhans cell) granuloma: (interosseous lesions may appear
and disappear without treatment=tempo phenomenon)

NOTE* Carcinoma vs. sarcoma


A carcinoma is a malignant tumor demonstrating a sheet-like cellular
arrangement, usually arising from epidermis and from visceral organs. A sarcoma
is a malignant tumor arising from supportive tissues of the body, such as
muscle, bone, cartilage, tendon, synovium, fat, endothelium,
schwann cells, histiocytes, fibroblasts, blood cells, and other immuno cells

Phases in Cancer Development


1. Definitions:
a. Preneoplastic: A long phase with no known identifiable cellular features to
predict malignant transformation
b. Incipient: May be a long phase. A proliferative process is seen that has to be judged
as to whether it is benign or malignant, by histology and biological behavior.
In some cases it is not clear, and the lesion must be watched. It is in this
context that dysplasias are viewed with suspicion and followup (many
carcinomas do not show such a phase)
c. In-situ: A short phase, in which the malignant process has not passed
through the basement zone of its tissue of origin to another tissue (carcinomas
show this phase, sarcomas do no )t
d. Local invasion: A short phase, in which malignant cells have passed to an
adjacent tissue through the basement zone bordering the tissue of origin, while
attached to the tumor source. At this time it is considered locally infiltrative
e. Extention: A short phase during which the malignant tumor has largely
replaced local tissues by infiltrating broadly in all directions, but
remains contiguous with the original tumor mass
f. Metastasis: A short phase, in which sites of malignant tumor (of the original
type) appear in other parts of the body, but they are not contiguous with the
original tumor site. Metastases may take many routes and appear in any one
or more sites

Benign vs. Malignant Tumors


1. General Characteristics of Benign and Malignant Tumors:

Benign Tumor Malignant Tumor


Uniformity of cell nuclei Atypical Cell nuclei (1)
Architectural nuclear and cellular order Architectural disorder (loss
of polarity)
Restraint in growth Growth often rapid (poorly
restrained growth)
Usually not recurrent Frequently recurrent
Does not metastasize Frequently metastasizes (2)
Very difficult to culture cells Relatively easy to culture
cells (3)
Does not develop resistance to Resistance can develop to
antineoplastics antineoplastics
May kill patient if impinging on vital May kill the patient for
structures many reasons (4)

KEY
1. Pleomorphism, hyperchromasia, abnormal mitoses, abnormal number of
mitoses, multinucleation, bizarre forms
2. The single most definitive determinant of malignancy
3. Cells develop 'immortality' in culture, and may show changes in
cytogenic patterns, medication resistance, cell morphology and behavior,
and subclones may emerge
4. Impingement on vital structures, size, metastasis to multiple sites, bleeding,
immunosuppression, unusual secretions that induce or inhibit other
functions of nontumor cells, necrotic changes, other metabolic changes
Chapter 13: Dermatology
The Skin
Dermatological Lesions
DERMATOLOGY
The Skin
The skin is one of the largest organs of the body exhibiting a wide range of
functions which include: mechanical protection, formation of a barrier to water
transfer, immune responses, thermoregulation, perception of the environment,
excretion, limitation of harmful radiation, and nutrition. The skin is composed
of two layers the epidermis and the dermis.
1. Epidermis:
a. This is composed of a stratified squamous epithelium containing cells that
become specialized for the production of keratin.
b. There are five strata in the thickness of the epidermis
i. Stratum basale
ii. Stratum spinosum
iii. Stratum granulosum
iv. Stratum lucidum
v. Stratum corneum
c. Specialized cells of the epidermis and their role:
i. Melanocytes are specialized cells for the production of melanin pigment and
are located in the basal cell layer. Melanin offers protection to UV-B radiation.
There are two forms of melanin in the human skin: Eumelanin (brown or black)
and Pheomelanin (red and yellow).
ii. Langerhans cells residing in the epidermis play an important role in skin
immunity.
iii. Merkel cells demonstrate dense core neurosecretory granules and are
thought to be part of the cutaneous sensory system.

2. The Dermas:
a. The dermis is composed primarily of connective tissue and consists of two
layers: a papillary layer and reticular layer
i. The papillary layer lies between epidermal rete ridges and contains many
nerve endings and capillaries
ii. The reticular dermis lies below the papillary dermis and above the
subcutaneous fat. It contains arterioles, venules, capillaries, larger nerves, and
adnexal structures

3. Skin Adnexal Structures:


a. Pilosebaceous structures: (hair, sebaceous glands, arrector pill muscle) are
present on the dorsal aspect of the foot and toes. Hereditary factors and
various acquired conditions contribute to the amounts
b. Sweat glands: Are present throughout the foot. These are eccrine types
c. Nails: Are present on the dorsal-distal aspects of all toes

4. Special Structures:
a. Vater-Pacini Corpuscles: In the deep dermis are sensors for pressure
b. Glomus bodies: In the toe tips function in blood shunting as an aid to
temperature regulation. The shunt is a narrow branch of the arterioles that
connect directly to a venule, bypassing capillaries. This shunt is the Suquet-
Hoyer canal
Dermatological lesions
1. Primary lesions:
a. Macules: Circumscribed flat lesions measuring up to 1 cm. in diameter
b. Patch: Flat lesions measuring more than 1 cm. in diameter. They may form
as a consequence of coalescence of macules
c. Papules: Circumscribed, solid elevations measuring up to 1 cm. in diameter,
and elevated due to an intradermal infiltrate
d. Plaque: A circumscribed, solid elevation exceeding 1 cm. in diameter, but
usually not over 2 cm. in diameter
e. Nodules: Circumscribed, solid elevations exceeding 2 cm in diameter, but not
usually exceeding 3 cm. in diameter
f. Tumors: Circumscribed, solid elevations of larger size than nodules

NOTE* The definitions may be somewhat artificial because of colloquial usage


such as "tumor nodules", and some informal overlap occurs in the use of the
terms "nodule" and "tumor".

g. Vesicles: Fluid-filled, elevated lesions, under 1 cm. in diameter (small


blisters)
h. Bullae: Fluid-filled, elevated lesions, exceeding 1 cm. in diameter (large
blisters)
i. Cysts: Non-infected, deep-set collection of material surrounded by a
histologically definable wall (sebaceous cyst, mucous cyst, epidermal
inclusion cyst, etc.) -
j. Burrow: An intraepidermal tunnel usually caused by insects (scabies,
tunga penetrans/chigoe)

2. Secondary lesions:
a. Scales: Products of imperfect and frequently rapid epidermal turnover,
occurring in "papulosquamous" diseases, with a great deal of exfoliation.
Examples include psoriasis, lichen planus, dermatophytosis
b. Excoriations: Scratch marks, usually seen where there is pruritus. These
show epidermal discontinuities.
c. Erosions: These are deep excoriations in the epidermis, but the dermis is not
breached. These leave no scars upon healing
d. Ulcers: Deep epidermal defects in which the dermis or deeper tissues are
exposed. These may leave scars when healed
e. Crusts: These are "scabs", i.e. aggregations of dried serum or blood with
other cellular debris
f. Fissures: Linear, deep epidermal cracks in the skin, penetrating to the
dermis and common in areas of dry skin
g. Scar: Also called "cicatrix", and resulting from inflammatory or traumatic
destruction of subepidermal connective tissue. Scarring is a normal reaction,
and that final portion of dermal healing called " fibrosis"
i. Hypertrophic scarring represents an abnormal response that will eventually
reduce itself
ii. Keloids represent abnormal scarring responses that continue beyond the
borders of the inciting injury, and progress to cause contractures
andinterference with function. They also may cause cosmetic problems, and
can be quite deforming. Treatment is a major problem. Before any elective
surgery is undertaken, a careful history and examination should elicit the risk
of keloid formation
h. Pustule: An elevated, circumscribed lesion containing pus, and arising from
infections of papules, vesicles or bullae.
i. Abcess: is a deep circumscribed collection of pus
ii. Furuncle (Boil): is an abcess originating in a hair follicle
iii. Carbuncle: an abcess involving several adjacent hair follicles, with
interconnecting sinuses

NOTE* Sinus tracts connect cavities, abcesses, etc. under the skin; fistulas
connect abcesses to the body surface

3. Vesiculobullous Disorders:
a. Viral: Herpes simplex, herpes zoster (shingles), Kaposi's varicelliform
eruption, varicella (chicken pox), variola (small pox), molluscum contagiosum,
hand-foot mouth disease (coxsackie Al 6 virus)

NOTE* Tzanck smear of fluid from vesicles or bullae for identification of


multinecleated giant cells or other viral cytopathic effects

b. Fungal: T. mentogrophytes, T. rubrum, T versicolor (etiological agent is


Malassezia furfur), other fungi and yeasts
c. Other:
i. Benign familial pemphigus (Hailey-Hailey disease), pemphigus vulgaris and
its variants (autioimmunity to intercellular epidermal glycoproteins), bullous
pemphigoid (eosinophils in bullae), dematitis herpetiformis (neutrophils in
bullae) often present with gluten enteropathy, Darier's keratosis follicularis,
bullous impetigo

NOTE* Nikolski Sign is present in pemphigus and in bullous impetigo. It


consists of pressing an existing bulla vertically with the finger tip and seeing
an adjacent bulla form as the fluid is gently forced peripherally to cleave the
epidermis

ii. Epidermolysis bullosa (all 16 types) represents a defect in skin development


such that trauma results in blisters, some forms being fatal. The feet are
affected by many types, such as the Weber-Cockayne and EB dystrophica
dominant types

4. Dermatitis (Eczema):
a. Contact dermatitis:
i. Primary irritant: acids caustic chemicals, etc.
ii. Allergic: poison ivy, poison oak, shoe ingredients, sock dyes, etc.
b. Atopic dermatitis: Affected individuals show atopy, asthma, hayfever, and
other forms of allergic rhinitis, blood serum with >IgE. Three stages, infantile (2
months-2 yrs), childhood (4-10), and adult (12-25). It is usually symmetrical,
and is diagnosed by the area of involvement, and the familial history of allergy
not the rash itself (biopsy non-specific)
c. Dermatophytid (ID) reaction: Allergic reaction incited by a fungal infection.
There are other ID reactions as well, in other dermatoses
d. Dyshidrosis: Pompholyx of palms and soles (difficult to treat)

5. Papulosquamous diseases: Represent a category of dermatitis that


produces an inflammatory papule and scaling. This goup includes
a. Psoriasis: Chronic disease characterized by epidermal hyperplasia and a
greatly accelerated turnover rate of cells. Discrete papules may coalesce to form
erythematous plaques. The lesions are covered by "micaceous" silver scales.
Seen on the elbows and knees most frequently, lesions tend to favor extensor
surfaces. The Koebner phenomenon is the occurrence of a lesion at an area of
trauma, and also occurs in other skin diseases b. Pustular psoriasis: Sterile
pustules on the soles. Very difficult to treat. Nails may be pitted
c. Lichen Planus: Inflammatory and pruritic disease of the skin and mucous
membranes. The lesions appear violaceous and may show a network of white
lines (Wickham's striae). Oral lesions have white lace appearance. Nail changes
may include pterygium. Lesions tend to favor the flexor surfaces. Bullous
lichen planus on the soles of the feet may become squamous Ca.
d. Pityriasis Rosea: Pruritic macules and papules, oval shaped, lesions appear
with distinctive 'collarette' of scaling on a pink base. Onset of herald patch, a
solitary lesion on the buttock or trunk that precedes the others and that
follows the skin lines. The disease is self limiting
e. Secondary Syphilis: Oval pink macules (non-pruritic) occur on the palms
and soles, associated lymphadenopathy, malaise, sore throat and low grade
fever. (+VDRL). Hyperkeratotic pitted papules may occur on the palms and
soles
f. Pityriasis Rubra Pilaris: Type I shows small follicular papules, distinct
location dorsal portion of digits, 'nutmeg grater' appearance; lesions coalesce to
form plaques. Palmoplantar hyperkeratoses arise. Most patients are clear of
lesions within 3 years. There are 4 other disease subtypes.

6. Painful tumors of the skin (ANGEL): Pain frequently occurs but not in all
cases
a. Angiolipoma: Vascularized tumors of adipose tissue
b. Neurilemoma: A benign tumor of Schwann cells (Schwannoma)
c. Glomus tumor: Arises from glomus body in the nail bed
d. Eccrine spiradenoma: An eccrine sweat duct tumor, paroxysmal spasms of
pain
e. Leiomyoma: A smooth muscle tumor (arrector pili muscle and vascular
smooth muscle pilar leiomyoma and angioleiomyoma, the latter also called
vascular leiomyoma)

7. Non-malignant pigmented lesions:


a. Epidermal melanocytes: Nevus spilus (present at birth), solar lentigo, and
ephilides (freckles)
b. Dermal melanocytic lesions: Blue nevus, junctional nevus (can become
malignant melanoma), compound nevus, halo nevus (Sutton's nevus,
leukoderma acquisitum centrifugum) and Spitz's nevus (benign juvenile
melanoma, spindle and epitheloid cell nevus). About half the cases occur in
adults

8. Pre-malignant lesions:
a. Actinic keratoses: Solar keratoses, pre-malignant, can become squamous
cell Ca, usually scaly and telangiectatic and confined to the epidermis. TX: 5
FU, surgical excision
b. Xeroderma pigmentosum: Genetic disease with sensitivity to sun,
basal/squamous carcinomas can develop, as well as melanomas
c. Bloom's syndrome: Genetic defect involving skin with sun sensitivity, growth
retardation and sometimes immunodeficiencies. A high risk for leukemia and
lymphoma, as well as GI carcinomas
d. Ataxia telangiectasia: Genetic defect involving skin and nervous system.
Numerous ectasias appear, many affecting the lower extremities.
Sinopulmonary infections are common. High risk of leukemia and lymphoma
e. Porokeratosis of Mibelli: Papular lesion with central keratosis that expands
to form a circinate lesion with a furrow containing keratin. Some may precede
development of squamous carcinoma. Some are hereditary.

9. Malignant Lesions:
a. Basal Cell Carcinoma:
i. There are 5 clinical subtypes, the most common of which is the
noduloulcerative or "rodent ulcer" type
ii. These are essentially nonmetastastic (with the exception of a few situations)
iii. The Basal Cell Nevus Syndrome (Nevoid Basal Cell Epithelioma
Syndrome/Gorlin's Syndrome) demonstrates many abnormalities including
"ham-colored pits" on the palms and soles that are basal cell carcinomas. In
this syndrome, basal cell carcinomas are aggressive
iv. Bazex Syndrome demonstrates hyperkeratoses of the palms and soles
preceding and associated with visceral carcinomas (acrokeratosis neoplastica)
and basal cell carcinomas resembling tricoepitheliomas
v. Basal cell carcinomas also occur in the Linear Unilateral Basal Cell Nevus
vi. In general lesions should be excised in toto, in full depth and with a border
about 3 mm of clinically normal skin
vii. Contributors to the development of basal cell carcinoma include old
wounds, burns, ultraviolet irradiation, and x-irradiation
b. Squamous cell carcinoma:
i. The in-situ form is Bowen's disease which has not invaded the dermis, and
which is confined to the epidermis
ii. Marjolin's ulcer is a squamous cell carcinoma arising in a scar or in the
epithelium at the edge of a chronic ulceration
iii. Bowen's disease may appear as a reddish, irregular, sharply bordered lesion
with crusting or scaling. It may be present for many years. Invasive lesions may
be solid, ulcerated, or verrucous (verrucous carcinoma, epithelioma
cuniculatum). Some are more aggressive than others. Metastases may develop.
Lesions should be excised in full depth and with a clinically normal boarder of
3-5 mm. Contributors to the development of squamous cell carcinoma include
old wounds, burns, chronic ulcers or fistulas, x-irradiation, ultraviolet
irradiation, certain arsenical and other organic compounds
iv. Lesions that resemble squamous cell carcinoma:
• Keratoacanthoma: A lesion thought to arise from hair follicles, in some
cases it may occur as single or multiple lesions. Single lesions develop quickly,
grow and ulcerate. They may be very difficult to differentiate clinically and or
histologically from well differentiated squamous cell carcinoma. The central
area usually contains a keratinous plug. Treatment is excision in full
thickness. Use of retinoids is preferred by some for treatment. Some
investigators consider keratoacanthoma to be a very low grade squamous cell
carcinoma that is "self healing" if left alone. Some types heal with scarring if
left alone
• Pseudoepitheliomatous hyperplasia: A thickening of epidermis due to
hyperplasia of keratinocytes. It is clinically and histologically benign, and
occurs in chronic wounds such as the edges of ulcers or fistulas
c. Sweat gland (eccrine) carcinoma: There are many types. These are
uncommon, but can be slow-growing and aggressive. Metastases do occur in
many cases. Many are asymptomatic small papules or nodules that have been
present for many years and suddenly enlarge. Some ulcerate. Suspect lesions
should be excised in full depth and with 3-5 mm borders of normal skin
d. Sebaceous carcinoma: Extremely rare in the feet, these have no distinct
presenting symptoms or signs. Metastasis is a risk. These lesions, if primary in
the feet (not metastatic to the foot) will not be found in the plantar tissues
which lack pilosebaceous structures. These should be excised in full depth
with a 3-5 mm boarder of clinically normal skin
e. Merkel cell carcinoma: Also called trabecular carcinoma, these cancers are
quite uncommon, but have been reported in the foot. They are very aggressive
and metastatic. There is no distinct presentation. These lesions should be
excised in full depth and with a 3-5 mm boarder of clinically normal tissue.
Also called Primary Neuroendocrine Carcinoma of Skin (PNCS)
f. Melanoma: A highly malignant tumor of melanocytes showing strong
association with ultraviolet irradiation of high intensity. Lighter skinned
persons are more apt to develop melanomas in sun-exposed areas. The types
are:
i. Lentigo maligna melanoma: Almost never occurs in the foot and is most
common on the face. They are the least aggressive (in-situ form=lentigo
maligna)
ii. Superficial spreading (pagetoid) melanoma: Most common type in all body
areas. Moderately aggressive. Histologically can resemble Paget's disease of the
breast (hence its name)
iii. Nodular melanoma: Arises anywhere in the body. Most aggressive and
malignant
iv. Acral melanoma: Occurs on the extremities and is the most common type
seen in the feet of black and oriental patients. Aggressive type

NOTE* Some consider the nodular type to represent a late phase of all the
other types, i.e. the "vertical growth phase" as compared to the earlier "radial
growth phase". Also, the true nodular type may represent a very aggressive
form with a very short radial growth phase

Clark's Classification (according to histologic level of invasion)


Level 1: Intraepidermal (including adnexae) to dermoepidermal border Level
2. Intradermal and into the papillary dermis
Level 3: Intraepidermal and down to border of papillary and reticular or
dermal areas
Level 4: Intraepidermal and down into the reticular dermis
Level 5: Intraepidermal, through all dermal layers and into subcu. Tissue

NOTE* Difficulties with Clark's classification as a method of prognosis is that


the skin thickness varies in different areas. Tumors in the papillary dermis
may push reticular dermis down, only giving the appearance of penetration so
that level may be overestimated. Identification of levels, and therefore
interpretation, is subjective. The border between papillary and reticular dermis
is not always clear

Breslow's Classification (according to the thickness of the melanoma)


NOTE* This is determined by direct measurement of numerous tissue
sections to obtain a figure. The criteria for measurement are not the same as
depth of invasion so that Breslow's and Clark's classifications may not
necessarily correspond
Survival is in terms of 5 years disease free
1. Under 0.76 mm: Generally excellent survival (virtually 100%)
2. 0.76-1.5 mm: Moderate survival (possible lymph node invasion)
3. Over 1.5 mm: Poor survival and increasingly poor with greater thickness
(probable lymph node and perhaps visceral invasion)

g. Amelanotic melanoma: Not different from other melanomas except for the
lack of visible pigment. Therefore, it is extremely dangerous because it is often
misdiagnosed and therefore is deeply invasive and thick by the time of
diagnosis, and probably has metastasized to the lymph nodes and viscera
h. Subungual melanoma: This must be confirmed with biopsy. The clues to this
lesion are
i. Pigmented lesion of recent origin with no history of local trauma to cause a
hematoma
ii. Pigmented lesion that does not move distally as the nail grows
iii. Sudden development of melonychia striata (pigmented stripe in the nail)
iv. Chronic non-pigmented lesion that does not respond to treatments (based
on clinical impression) within a reasonable period
v. Dusky, irregular pigmentation in eponychial tissue (Hutchinson's sign)
h. Biopsy:
i. Ideally, should be excisional including full depth to the subcutaneous fat
ii. Incisional biopsy from one or more edges, including clinically normal-
appearing skin, and always in full depth
iii. Incisional biopsy of nodular portion of lesion (if present) as well as edge(s),
in full depth

NOTE* For all malignant tumors:


1. Excise as indicated by the type and extent (surgical consultation)
2. Consult with oncologist for, chemotherapy, irradiation (if indicated)
3. Arrange to detect metastases to other areas/organs (consult with diagnostic
radiologist, pathologist, and others as needed)
4. Follow-up care by those involved as attending team

10. Miscellaneous conditions:


a. Discoid lupus: Exacerbated by sunlight, lesions are
erythematous/scaling/telangiectic
b. Drug induced SLE: SLE can be induced by hydralazine, procainamide,
isoniazid, penicillamine, griseofulvin, phenylbutazone, methyldopa and oral
contraceptives
c. Dermatomyositis: Adult form may be associated with visceral malignancy
d. Scleroderma:
i. CREST syndrome represents one type- C (calcinosis cutis), R (Raynaud's), E
(esophageal involvement), S (sclerodactyly) and T (telangectasias). May be
associated with Sjogren's syndrome
ii. Localized scleroderma= morphea
iii. Generalized type= progressive systemic sclerosis
e. Sarcoid: A chronic granulomatous Inflammatory disease affecting various
organ systems with erythema nodosum of the legs (plaques on the extremities)
Dx: chest x-ray, Kveim test and other immunologic tests
f. Erythema nodosum: Acute inflammatory/immunologic disorder with
panniculitis and painful nodules on the anterior shins. It may be associated
with infections, drugs, sarcoidosis, ulcerative colitis
g. Granuloma annulare: Self-limiting chronic inflammation of dermis with
annular papules. The generalized papular form has been associated with
diabetes mellitus
h. Necrobiosis lipoidica: Very similar histologically to granuloma annulare and
frequently associated with diabetes mellitus
i. Pigmented purpuric diseases: There are many types that may be of the
"palpable purpura" type, or it may be nonpalpable (Shamberg's dermatosis,
Majocchi's purpura, Henoch Schonlein purpura)

NOTE* Purpuric lesions can also be seen with thrombocytopenia,


meningococcemia and gonococcemia
j. Diabetic dermopathy: May include Bullosus Diabeticorum, ulcers,
necrobiosis lipoidica, cutaneous reaction to insulin, trophic changes (preulcer)
related to circulation, etc.
k. Petechial hemorrhages: May be seen in meningococcemia, gonococcemia,
platelet deficiencies, leukemias, scurvy, salicylate poisoning, bacterial
endocarditis (including subungual splinter hemorrhages), anticoagulant
overdosage, other bacterial infections (systemic) etc.
12. Ulcers: The following table lists the more common causes of lower
extremity ulcerations
Chapter 14: The Arthropathies
Causes of Joint Pain
The Arthropathies
Lab Testing for the Arthropathies Articular Disorders
Affecting the Heel Synovial Fluid Analysis
Charcot Foot
THE ARTHROPATHIES
Causes of Joint Pain: A Summary
1. Joint disease: the arthropathies

2. Bone disease: fractures, primary or secondary tumors, osteochondritis,


osteomyelitis, etc.

3. Soft-tissue lesions: sprains and strains, tenosynovitis, overuse syndrome,


direct trauma, bursitis, "soft tissue rheumatism"

4. Arthralgia: defined as joint pain in the absence of objective joint disease,


seen with
a. The Arthropathies- either preceding the development of local signs or in
some conditions in which there may be no local signs. Important examples are
polymyalgia rheumatica and temporal arteritis, SLE, and polyarteritis nodosa.
b. Infections- particularly viral and rickettsial
i. Viral: influenzia (25% cases), glandular fever, psittacosis, yellow fever,
sandfly fever
ii. Rickettsial: all types of typhus
iii. Bacterial: septicemia, subacute bacterial endocarditis, thyphoid salmonella,
gonorrhea
iv. Spirochaetal infections: secondary syphilis, leptospirosis, relapsing fever
v. Protozoan/metazoan: kala-azar and other tropical diseases
c. Drugs- immunizations, serum sickness
d. Protein abnormalities- e.g. mixed IgG IgM cryoglobulinemia

5. Referred pain: particularly common in the knee due to hip and prostate
disease and in the shoulder due to some abdominal conditions, esophageal
conditions, cardiac conditions, neurologic conditions neurological conditions
and pulmonary conditions

6. Psychogenic: Joint pain may be a manifestation of psychological


disturbance and "rheumatism" may become a source of complaint in the
anxious or neurotic patients

The Arthropathies
1. Anklosing Spondylitis (Marie Strumpell Disease): a chronic condition of
the spine (bamboo spine) and sacro-iliac joints in which early inflammatory
changes are followed by progressive restriction of spinal movement, with
radiological calcification of spinal ligaments (seronegative disorder).
a. Signs: chest expansion reduced (< 5cm), limited spinal movements, tender
bony points (heel)
b. Symptoms: gradual onset of low backache and/or pain in buttocks
c. X-ray: shows bamboo spine, and "whiskering" of ischial tuberosities
d. Labs: ESR >, mild anemia, (-) latex fixation, synovial fluid shows
inflammatory/neutrophils
e. Tx: Exercises, NSAIDS
2. Arthrogryphosis Multiplex Congenita: a rare congenital disorder
characterized by stiff, deformed joints and muscle wasting.
a. Clinical features: painless deformities (equinovarus feet, flexion deformities
of knees), stiff joints
b. Tx: correction of deformities, surgical and nonsurgical

3. Avascular necrosis: a group of conditions in which bone infarction is not


associated with sepsis but is usually due to interference with blood supply,
either by abnormalities of the vessel wall such as arteritis, pressure on the
vessels from outside, trauma, thrombosis, or embolism. Clinical disease results
only when the surface of the joint is involved causing arthritis. The condition
may be:
a. Traumatic: following fractures
b. Secondary to existing arthropathies: RA, severe OA, psoriatic arthropathy,
neuropathic joint
c. Secondary to systemic conditions: sickle cell disease (particularly sicklecell
thalassemia and sickle-cell-hemoglobin C disease), high dose steroid therapy,
pregnancy, extensive burns endocarditis, Caisson disease, and scleroderma
d. Idiopathic: avascular necrosis may occur in middle aged men and is the
cause of Perthes' disease in children
e. Types: Legg- Calve'- Perthe's disease (femoral capital epiphyses), Blount's
disease (proximal tibial epiphysis), Osgood Schlatter's disease (tibial
tuberosity), Sever's disease (calcaneal apophysis), Freiberg's disease (2nd
metatarsal head), Diaz/ Mauchet (talus), Islen's disease (base of 5th
metatarsal), Thieman's disease (phalanges), Kohler's disease (navicular and
patella), Treve's disease (medial sesamoid), Buchman's disease (medial
cuneiform)
f. X-rays: small areas of infarction which appear sclerotic or porotic, areas of
joint surface collapse into underlying infarction producing flattening, necrotic
osteochondral fragments may separate completely or partially
g. Tx: immobilation, physical therapy, surgery

4. Drug Induced SLE (the Hydralazine Syndrome): a condition resembling


SLE but induced by procainamide, hydralazine, isoniazid, oral contraceptives,
penicillin, sulfonamides, tetracycline, griseofulvin, phenylbutazone, reserpine
and methyldopa. It is suggested that patients with this condition have a
heredity "lupus diathesis" which is made manifest only when the drug is given.
Arthritis occurs in 50% of the cases and can occur synchronously or up to 4
weeks prior to the skin rash.
a. X-rays: normal
b. Labs: L.E. cells (90%), A.N.A. (100%)
c. Tx: stop the drug, steroids for symptoms prn

5. Erthema Nodosum: an acute self-limiting condition characterized by the


development of crops of tender nodules in the skin of the lower leg and arthritis
occurring in 50% of the cases occurring with the nodules or preceding them by
up to 4 weeks. The nodules are first bright red, later dark red, and then fade
like bruises. This condition occurs in sarcoidosis (common in Great Britain),
various infections (particularly strep, measles, etc.), ulcerative colitis,
malignant diseases, and drug sensitivities.
a. Joints affected: knees and ankles most commonly
b. Symptoms: sudden onset of pain without swelling
c. Signs: variable (none to red swollen joints), fever common
d. X-rays: normal
e. Labs: >ESR, LA Fix occasionally (+), must do complete work up to find cause
f. Tx: rest, MAIDS

6. Gaucher's Disease: a rare condition transmitted as an autosomal recessive


trait characterized by the abnormal accumulation of glucocerebrosides in
reticulo-endothelial cells. The characteristic feature is the Gaucher cell, a lipid
filled macrophage that contributes to the arthritis and eventually avascular
necrosis. Severity is variable and can occur at any age.
a. Joints affected: hips are most common, 70% unilateral
b. Symptoms: pain and stiffness
c. Signs: hip held in flexion and adduction with limb shortening
d. Course: untreated hip involvement will progress to complete destruction of
the joint with secondary OA
e. X-ray: areas of porosis/sclerosis, aseptic necrosis, Ehrlenmeyer flask
appearance of lower end of femur
f. Labs: anemia, sometimes leukopenia and thrombocytopenia, bone biopsy
shows Gaucher cells (lipid-filled "bubbly' macrophages)
g. Tx: rest, immobilization with no weight- bearing in early cases, surgery in
advanced cases

7. Gonococcal arthritis: arthritis due to gonococcal infection of the joints,


beginning 3-17 days after being infected.
a. Joints affected: knee is the most common area, ankles(40%), asymetrical,
polyarticular (75%), "Gonorrheal heel"
b. Symptoms: sudden onset of severe pain and swelling, fever, rigors
c. Signs: Warmth, erythema, tenderness, edema of joint and periarticular soft
tissues
d. Course: effective treatment produces complete cure in 1-4 weeks
e. Associations: skin lesions (50% cases) maculopapular, hemorrhagic
f. X-ray: normal in the acute stage
g. Labs: mild leukocytosis, >ESR, gonococcal antibodies detectable (90%),
organism may be recovered from the blood and genital tract
h. Tx: Penicillin (procaine penicillin 1-2 megaunits + benzylpenicillin 1
megaunit daily for 5 days) or tetracycline 250mg. q.d.s. for 7 days. VDRL
should be done before treatment, since antibiotics may mask early signs of
concurrent Syphilis.

8. Gout: a disorder of purine metabolism, characterized by hyperuricemia and


the deposition of urate crystals in joints, resulting in acute attacks of arthritis.
In the later stage there is deposition of urate in the soft tissues and the kidney,
with a chronic arthritis.
a. Joints affected: 1st m.p.joint (75%), monoarticular (90%)
b. Symptoms: prodromal irritability with sudden onset of pain often 3-6 am,
precipitated by surgery, trauma, starvation, alcoholic excess, and drugs (ASA,
thiazides), usually males>females
c. Signs: red/hot/swollen/tender joint, between attacks the joints are normal
until tophaceous stage, must differentiate from septic arthritis
d. Variants: Secondary gout (5%) cases due to myeloproliferative disorders
(polycythemia rubra/vera) and during the treatment of malignant disease.
Other variants are renal failure, Lesch-Nyhan syndrome and glycogen storage
disease (von Gierke's disease)
e. X-ray: normal in early stages, with the joint space preserved until late in the
disease, periarticular swelling and punched out/radiolucent areas in the
affected joints
f. Labs: synovial fluid and tophaceous deposits contain needle-shaped crystals
which are strongly negatively birefringent, uric acid greater than 6mg./100ml,
>ESR (uric acid serum levels may not be elevated during acute attack)
g. Tx: responds to indocin, phenylbutazone or colchicine (.6 mg./hr until relief
or NVD), if no response then ACTH. Later uricosurics or allopurinol (used in
tophaceous gout)

9. Hemophilia: a group of disorders of blood coagulation mechanisms due to


deficiencies of various factors. The most common form is inherited by males,
the females being unaffected carriers. It is inherited as an X-linked recessive
trait. Attacks of arthritis are due to hemorrhage into joints and recurrent
hemoarthrosis leads to a degenerative arthropathy which is characteristic of
hemophilia.
a. Joints affected: knee (70%), ankle (20%), monoarticular, asymmetrical
b. Symptoms: sudden onset of very severe pain sometime after mild trauma
c. Signs: joint is red, warm, swollen and very tender, after repeated attacks
there is limited ROM and crepitus, eventually muscle wasting
d. Course: complete recovery but later deformities and degenerative changes
e. X-ray: normal with early attacks, later small superficial erosions and cysts
appear with a loss of joint space and flattening of joint surfaces
f. Labs: prolonged PTT time, confirmed by Factor VIII assay, blood stained
synovial fluid
g. Tx: complete immobilization while in the acute stage, give missing factor
(cyroprecipitate or fresh plasma), analgesics, avoid aspiration, gentle
physiotherapy

10. Hypothyroidism: deficiency of thyroid hormone may cause a variety of


rheumatic manifestations: pain and stiffness in proximal muscles,
polyarthritis, monoarthritis associated with osteolytic lesions usually in
children, carpal tunnel syndrome, secondary gout, hypercholecterolemia.
a. Joints affected: knees, wrists, ankles, usually bilateral and symmetrical
b. Labs: confirm by > cholesterol, < thyroxin level, abnormal EKG
c. Tx: thyroxin

11. Neuropathic Joint (Charcot): disorganization of a joint and destruction of


joint surfaces associated with diminished pain sensation which is most
commonly due to: tabes dorsalis, diabetes mellitus, syringomyelia, Charcot-
Marie-Tooth disease, meningomyelocele, hemiplegia, and leprosy.
a. Joints affected: monoarticular, depends on localization of pain loss
b. Symptoms: 50% have acute onset of pain and swelling, pain is absent in the
later stages
c. Signs: acute stage, red/warm/tender/swollen joint; chronic stage, bony
swelling and recurrent effusion/instability/crepitus/grotesque deformities
d. Course: acute inflammatory stage lasts for up to 6 months, with subsequent
slow progression for years until deformities occur
e. X-ray: sclerosis of the bone ends, loss of the joint space, loose bodies,
massive osteophytes, periarticular calcification, fractures
f. Labs: essentially normal chemistries. Do immunologic tests (antibodies to
listed bacterial infections)
g. Tx: rest in the acute stage, MAIDS, stabilization with braces and splints,
arthrodesis for instability

12. Conditions associated with neuropathic joints:


a. Tabes Dorsalis: knee is most common site (70%), with ankle and feet (30%),
Argyll Robinson pupil (80%), absent DTR's
b. Syringomyelia: 50% have cervical spondylosis on x-ray, loss of pain and
temperature in the upper extremities, equinus deformities c. Diabetes Mellitus:
1 % of diabetics and 5% with diabetic neuropathy have a neuropathic joint,
foot is most common site (80%), absent ankle jerk, sensory loss

13. Osteoarthritis: a common degenerative condition of a joint surface


associated with aging. OA may be primary or secondary to: obesity,
unrecognized congenital hip dislocation, any condition causing irregularity of
the joint surface including avascular necrosis, repeated trauma, septic or other
arthritis. There is a wide spectrum of clinical manifestations:

a. Secondary OA Primary OA Primary generalized OA


one joint one to six joints polyarticular
definite cause no definite cause Heberden's nodes
Heberden's nodes usually
(sometimes)
b. Joints involved: D.I.P. joints of the hands, lumbar spine, 1st M.P.J., P.I.P. or
M.C.P. of hands

NOTE* Heberden's nodes occur over the distal ipj's, and Bouchar’ s nodes occur
over the proximal ipj's (Bouchard's nodes are associated with gastric dilatation)

c. Symptoms: pain with or after movement/relieved by rest, worse towards


evening, stiffness of affected joints after immobility, but not generalized
morning stiffness (post static dyskinesia)
d. Signs: bony swelling and tenderness, acute episodes may be accompanied by
warmth/erythema/effusion
e. Course: chronic with exacerbations, sometimes related to trauma
f. X-ray: degenerative changes with non-uniform loss of joint space/loss of
subchondral bone (subchondral sclerosis)/subchondral cysts (do not break
through the cortex)/osteophytes/later, irregularity of the joint surface/joint
mice
g. Labs: ESR normal, synovial fluid: see chart
h.Tx: NSAIDS, physiotherapy, surgery if symptoms severe and persistent

14. Osteochondritis Dissecans: a condition characterized by the separation of


avascular osteochondral fragments from the surface of the joints. If separation
is complete, loose bodies are found within the joint.
a. Joints affected: knee is most common (85%), monoarticular (75%), ankle
frequently
b. Symptoms: pain (mild) worse after exercise
c. Signs: often none, effusion occasionally
d. Course: acute a isode recovers after a few months with OA following after a
few years (50%)
e. X-ray: separation of a fragment or fragments of bone from the joint surface,
sclerosis of the fragment and the crater from where it comes from,
osteoarthritic changes later
f. Labs: uneventful
g. Tx: immobilization in non-weight bearing cast/ spontaneous healing in
younger cases, with surgery for fixation of a fragment or removal of loose
bodies

15. Paget's Disease of Bone (osteitis deformans): a disorder of unknown


etiology affecting one or more bones particularly the skull, femur, and tibia.
The condition is rare before the age of thirty. Approx. 80% of patients with
radiological changes also have symptoms, usually bone pain and
deformities.Areas of increased density and of decreased density.
a. Symptoms: pain indistinguishable from OA, limp
b. Course: chronic and slowly progressive leading to OA
c. X-ray: 1st change is localized porosis (in skull called osteoporosis
circumscripta), bone often expanded and thickened, with fractures of the long
bones. Distinct border between normal and abnormal bone
d. Labs: >Alk Phos, Ca normal (ALK PHOS reaches extremely high serum levels)
e. Tx: analgesics, disodium etidronate (inhibits bone resorption and
mineralization)

16. Palindromic Rheumatism: an uncommon condition, probably a variant of


RA, characterized by recurrent/acute/self-limiting attacks of arthritis.
a. Joints affected: hands, wrists, knees and feet, with each patient having 1-3
prominent sites
b. Symptoms: sudden onset of severe pain and stiffness
c. Signs: swollen/red/tender joint
d. Course: attacks occur at irregular intervals, averaging 20 per year. There
may be years of freedom between attacks or hundreds of attacks in one year.
Attacks last a few hours/days, seldom more than 1 week, with complete
recovery follows the attack
e. X-rays: normal
f. Labs: >ESR, latex fix (+) 50% cases
g. Tx: indomethacin, phenylbutazone or ASA for acute attacks, gold therapy
may produce remission

17. Psoriatic Arthritis: a common skin disease (psoriasis) with associated


arthritis in 10% of cases, with the skin manifestation preceeding the arthritis
by many years. A seronegative disease. Sometimes joint symptoms precede
skin lesions
a. Joints affected: polyarticular, with small joints of the hands predominating
(70%). There are three patterns (distal type- d.i.p.j.'s and p.i.p.j.'s, seronegative
indistinguishable type- joint involvement indistinguishable from RA, deforming
type- causes arthritis mutilans)
b. Symptoms: pain is seldom prominant except in the deforming type
c. Signs: acute stage- red/hot/swollen joint (sausage toe), chronic stageswollen
joints
d. X-rays: erosions within the joint, sclerosis of the joint margins, proliferation
of bone, cysts, ankylosis, destruction of bone ends in the deforming type,
mushrooming of the joint surfaces
e. Labs: ESR normal except in the acute stage, (+) HLA-B27 (in many cases),
latex fix (-)
f. Associations: pitting of the nails (90%), hyperuricemia (30%)
g.Tx: analgesic and NSAIDS, avoid systemic steroids, avoid antimalarials cause
exfoliative dermatitis)

18. Pyrophosphate Arthropathy: a condition caused by the deposition of


crystals of calcium pyrophosphate dihydrate (CPPD) into joints. This condition
does not resemble gout and should not be called pseudogout. The term
chondrocalcinosis articularis should be reserved for the radiological
appearance of cartilage calcification. There are three clinical patterns of the
disease: recurrent acute arthritis, chronic degenerative arthritis (resembles
RA), and chronic arthritis with acute episodes. Typical onset is age 60.
a. Joints affected: knee is most common site, can affect ankles/toes/fingers,
usually monoarticular and symmetrical b. Symptoms: pain and stiffness,
sudden onset
c. Signs: acute red/ hot/ swollen joint
d. Course: attacks last weeks/months
e. Associations: diabetes mellitus present in up to 50% cases, hypercalcemia,
hemochomatosis
f. X-ray: affected joints eventually show calcification of cartilage and
degenerative changes
Labs: synovial fluid changes (see synovial fluid chart), >ESR, leukocytosis,
crystals seen microscopically in fluid and biopsy-crystals polarize
g. Tx: injection of joints with hydrocortisone, indomethacin

19. Reiter's Disease: a triad of arthritis/conjunctivitis/urethritis which


usually follows non-specific (non-gonococcal) urethritis, occurs in males (20-1).
A dermatitis may also be present. A seronegative disease.
a. Joints affected: knee (90%) and ankle (75%) most common, feet (40%),
polyarticular/asymetrical
b. Symptoms: acute onset of joint pain and swelling, dysuria, and penile
discharge
c. Signs: red/hot/swollen joints in the acute stage
d. Course: 1st attack resolves within 6 months, rarely becomes chronic, 50%
relapse, 20% have continued relapsing/chronic arthritis
e. Associations: urethritis, conjunctivitis, circinate balanitis, keratoderma
blenorrhagica, tenosynovitis (usually achilles), plantar fasciits, heel pain, back
pain
f. X-ray: periarticular osteoporosis with erosions, fluffy periosteal new bone
formation (calcaneal spur)
g. Labs: >ESR, (-) latex fixation, (+) HLA-B27 (in many cases), joint fluid is
inflammatory, gonococcus found in GU tract in some cases. Other bacterial
infections may be present. Reiters disease is also seen in AIDS
h. Tx: rest, analgesics, MAIDS, antibiotics (if indicated)

20. Rheumatoid Arthritis: a chronic and widespread chronic deforming


polyarthritis characterized by bilateral/symmetrical joint involvement, erosions
on x-ray, positive latex fixation tests, and pathologically a chronic proliferative
synovitis with villous hypertrophy, infiltration of lymphocytes and plasma cells,
and lymphoid nodules. Occurs in 6% females and 2% males world-wide. An
autoimmune disease a. Diagnostic criteria:
• morning stiffness
• pain on motion or tenderness in 1 joint
• swelling in 1 joint, swelling in another joint
• symmetrical joint swelling
• subcutaneous nodules
• x-ray changes typical of RA
• poor mucin ppt. from synovial fluid
• characteristic histologic changes in the synovial membrane
• positive Rheumatoid factor
• characteristic histologic changes showing palisading granulomas

Definite rheumatoid arthritis= 5 of the above


Probable rheumatoid arthritis= 3 of the above

b. Joints affected: small joints of the hands and feet (75%) especially the
m.p.j.'s, and the p.i.p.j's, usually bilateral and symmetrical. Also calcaneus
affecting the posterior-superior surface near the achilles insertion (forms a
posterior calcaneal recess) with non-well demarcated heel spur. Larger,
proximal joints are frequently involved.
c. Symptoms: insidious onset of joint pains and stiffness, generalized morning
stiffness, accompanied by general ill health, fatigue and weight loss which can
precede joint symptoms by a few months.
d. Signs: swollen/tender joints with limited ROM due to effusion or synovial
thickening, muscle wasting around the affected joints
e. Special features: ulnar deviation hands, boutonniere deformity of p.i.p.j.,
baker's cyst of knees, atlanto-axial subluxation of cervical spine, crico-
arytenoid joint involvement causeshoarseness/dyspnea/dysphasia/bronchitis
f. Course: either episodic or persistent, episodic has average patient having 3
attacks of arthritis once every 2 years each lasting 6 months; persistent has
chronic arthritis with partial remissions and exacerbations, joints tend to
become burnt out
g. Non-articular manifestations:
i. Periarticular soft tissues- nodules (Haygarth's nodes at affected joints),
tenosynovitis, bursitis, synovial cysts, muscle wasting, ligamentous laxity
ii. Skin- tight like scleroderma, leg ulcers
iii. Eyes- Sjogren's syndrome
iv. Heart- pericarditis, granulomatous lesions in myocardium
v. Nerve- carpal tunnel syndrome/tarsal tunnel syndrome
vi. Blood- anemia, Felty's syndrome (splenomegaly/leukopenia)
h. X-ray: early changes include periarticular osteoporosis, joint space widening;
later changes include uniform joint space narrowing thinning of the cortices
synovial erosions, pseudocysts that erode through the cortex (OA cysts have a
sclerotic boarder), subluxations and deviations joint fusions and widespread
osteoporosis
i. Labs: anemia, >ESR, latex fixation (+) 80%, ANA (+), RA (+), synovial fluid
changes (see chart), blood protein (plasma fibrinogen and globin increased;
decreased total protein)
j. Tx: NSAIDS, rest modalities, intraarticular steroid injections, synovectomy,
gold injections (Myocrisin) @ 10, 20, 30, 40 mg I.M. at weekly intervals then 50
mg. weekly up to 1 g., then a maintenance dose of 50 mg. monthly if results
good (must check WBC's)

21. Septic Arthritis: Infection of a joint with pyogenic bacteria. Most common
organism is Staph aureus (50%). Can occur in any age group but more
frequent in kids and geriatrics (should be considered when examining any
monarticular erythematous, hot, swollen joint)

NOTE* The bacteriology in septic arthritis is most frequently as follows:


Neonates: Streptococcus and gram negative organisms
Children (6months to 5 yrs): H. influenza common
Teenagers: Neisseria gonorrhea have higher incidence
Adults: P. aeruginosa common after puncture wound
Adults with sickle-cell: Salmonella common
Compromised patients (burn victims, drug users): Serratia marcescens
pyarthrosis

a. Joints affected: knee most common, 90% monarticular


b. Symptoms: rapid. onset of pain and swelling, can have elevated temperature,
malaise, tachycardia, and confusion
c. Signs: swelling with effusion/tenderness/warmth and painful limitation of
motion, febrile
d. Course: untreated, leads to joint destruction (OM/ankylosis)
e. X-rays: normal initially, osteoporosis after 2 weeks
f. Labs: >WBC's, synovial fluid (see chart), blood culture and synovial fluid
culture (+)

NOTE* The joint aspirate should undergo the following studies:


C&S, gram stain, examination for crystals, WBC count and differential

NOTE* The WBC's in joint aspirate in a septic joint is usually higher then
100,000, however, in gonococcal arthritis the WBC's is usually below 50,000

g. Tx: antibiotics, rest, aspirations (?? damage to the joint), open debridement
(permits lysing of adhesions and debridement of necrotic bone or soft tissue
present, arthrotomy performed in patients with advancing osteomyelitis

22. Sickle-Cell Disease: a condition inherited as an autosomal intermediate,


in which hemoglobin A is replaced by hemoglobin S. Heterozygotes are not
anemic and do not have arthritis. Homozygotes are severely anemic and have
thrombotic crises which commonly cause arthritis as a result of local bone
infarcts. Usually seen in blacks with the onset in the first 10 years of life.
a. Joints affected: polyarticular, hands and feet common sites, often migratory
b. Symptoms: sudden onset of severe joint pains or backache
c. Signs: often none, occasionally affected joints are swollen/red/tender, fever
common in a crises, painful ankle ulcers
d. Course: joint pain subsides spontaneously within days, life span greatly
reduced
e. X-rays: areas of osteoporosis or sclerosis, periosteal proliferation affecting
the shafts of the metatarsals
f. Labs: in vitro sickling tests and hemoglobin electrophoresis confirm the
diagnosis
g. Tx: analgesics, IV bicarbonate may help, urea compounds, pentoxophylline,
avoid hypoxia/hypotension/tourniquets

23. Still's Disease (Juvenile Rheumatoid Arthritis): a chronic polyarthritis


resembling R.A. clinically and histologically, but beginning before the age of 16
(peak ages, 1-3 & 10-15).
a. Joints affected: as with the adult + involvement of the cervical
spine/TMJ's/d.i.p.s. (more)
b. commonly affected, less symmetry than RA
c. Symptoms: pain often mild
d. Signs: joints warm/tender/red/swollen with effusion and synovial
thickening, fever (50%)
e. Course: better prognosis than in adults especially if earlier onset
f. X-ray: erosions less common, periosteal reaction may occur in phalanges and
mets
g. Labs: >ESR, latex fixation (+) in 20% cases
h. Tx: ASA, rest, maintain full ROM, gold or immunosuppressive drugs only in
severe progressive cases, short course A.C.T.H. useful for severe relapse

24. Systemic Lupus Erythematosis (S.L.E.): an autoimmune condition


characterized by the presence of antinuclear factor and other autoantibodies.
The diagnosis is often made on the basis of multi-system involvement.
a. Joint pain occurs in 90% cases, and arthritis usually occurs at onset but
may precede other systemic features by up to 20 years.
b. Joints affected: Commonly seen in the p.i.p.j.'s, also in the wrists, knees,
ankles and elbows, is polyarticular with a distribution resembling RA
c. Symptoms: sudden onset of pain and stiffness which may be precipitated by
exposure to sunlight or stress, morning stiffness (50%)
d. Signs: 50% have none, but the characteristic finding is slight soft tissue
swelling, fever common
e. Course: 70% survive 5 years and 50% 10 years, prognosis is much worse
with renal involvement
f. Associations: rash of face (butterfly), renal disease, pericarditis, Raynaud's
phenomenon, and liver disease
g. X-ray: usually normal
h. Labs: >ESR, ANA (+) 100%, LE cells present (80%), latex fixation (+) 30%
i. Tx: prednisone 30-60 mg/day initially reducing slowly to a maintenance dose
of about 15 mg.day

NOTE* Some medications precipitate SLE (see earlier chapter)

25. Ulcerative Colitis: a condition in which severe ulcerative inflammation of


the colon causes fever, anemia, and the passage of blood/mucus from the
bowel. Arthritis is of 4 types, colic arthritis, ankylosing spondylitis, arthritis
associated with erythema nodosum, and pseudohypertrophic osteo-
arthropathy. In ulcerative colitis 10% develop arthritis, in Crohn's disease 4%
develop arthritis
a. Joints affected: knees and ankles, usually 1-3 joints in each attack, usually
asymetrical
b. Symptoms: sudden onset of pain/swelling, often associated with relapse of
colitis
c. Signs: effusion, painful limitation of motion
d. Associations: skin lesions (erythema nodosum/pyoderma gangrenosum)
e. X-ray: joints normal
f. Labs: >ESR, Synovial fluid shows WBC's up to 40,000
g. Tx: NSAIDS and analgesics, appropriate tx for colitis
h. Younger patients may go on to develop GI adenocarcinomas after 10 or more
years of disease.

Laboratory Testing for the Arthropathies


1. Synovial Fluid analysis: should be approached with caution so as to
prevent hemoarthrosis or septic joint. When the joint is punctured an attempt
should be made to extract all the fluid present. The analysis of the joint fluid is
of some importance in the diagnosis of certain diseases. (See chart)

2. Serologic tests:
a. Erythrocyte Sedimentation Rate: nonspecific, reflects the presence of
tissue injury and inflammation, as a screening test may be overrated
b. C-reactive protein (CRP): nonspecific test for inflammation, a most
sensitive indicator of rheumatic fever
c. Rheumatoid factor: are a heterogenous group of antiglobulin antibodies
primarily found in patients with rheumatoid arthritis. The antibodies react to
specific antigenic determinants on the crystallizable fragment of human IgG.
RF has been found among several classes of immunoglobulins, including IgM,
IgA, IgG, and IgE
d. Latex fixation: are also used to detect RF, with IgG used as the antigen
e. Antinuclear antibodies (ANA): are a heterogenous group of antibodies
detecting many nuclear constituents; including deoxyribonucleic acid (DNA),
deoxynucleoprotein (DNP), histone, and ribonucleicacid (RNA). The ANA
antibodies were 1st noted in the sera of patients with SLE. Only a test reactive
for double- stranded DNA is diagnostic for SLE
f. Histocompatibility antigens (HLA-B27): is associated with the seronegative
diseases
g. VDRL: screening test for syphilis (but not specific for it)
h. ASO titer: done with CRP where Rheumatic fever is suspected
i. LE prep: rarely done today because of the accuracy of the ANA

3. CBC and differential

4. Uric Acid: May be normal during acute gouty attacks 5. Test for Lyme
disease (Lyme titer)

6. Test for AIDS: ELISA (and If reactive then do a Western Blot)

Articular Disorders Affecting the Heel


Systemic inflammatory disorders such as RA, anklosing spondylitis, psoriatic
arthritis, Reiter's syndrome, gout, and Behcet's syndrome have all been
reported to cause heel pain. Signs and symptoms of these disorders include
local soft tissue swelling and occasional erythema associated with diffuse
aching type pain. Various symptom complexes highlight certain arthritides (like
Reiter's). Radiographic erosive and proliferative bone changes in these patients
include posterior and inferior calcaneal spurring, retrocaicaneal swelling,
Achilles tendon thickening, and posterior and inferior calcaneal erosions. These
changes occur in various combinations and each disease entity has its own
usual but not exclusive calcaneal target site.
Calcaneal target sites
A: Rheumatoid Arthritis
B: Ankylosing spondylosis and psoriasis
C: Reiter's syndrome
McGlamry ED: Comprehensive Textbook of Foot Surgery, Williams & Wilkins, Baltimore, 1987, with permission
SYNOVIAL FLUID
After aspiration note volume, clarity, turbidity, blood-staining, and viscosity. Divide as follows:

2 ml. in a tube containing At least 1 ml. in a sterile Remainder: examine a wet


EDTA for cell count and container for microscopy and film for crystals under
differential culture. Ask for urgent Gram polarized light. Note presence
stain or special culture media or absence of clot formation
if indicated after standing

RESULTS: The characteristics of synovial fluid depend upon the presence or absence of
inflammation of synovium. Non-inflammatory fluid is clear, viscous, fails to clot on standing,
and contains less than 1000 cells per c.mm., predominantly mononuclears. Inflammatory fluid
is non-viscous, may clot, and contains an increased number of white blood-cells. These
changes may be slight or gross, depending on the severity of the inflammatory process. Fluid
with a high whitecell count is turbid and this does not necessarily mean that it is septic. The
characteristics of inflammatory and non-inflammatory fluid are summarized in the following
table. Synovial fluid characteristics in individual arthropathies are shown with other laboratory
features of the conditions.

NON-INFLAMMATORY INFLAMMATORY

E.g., Osteo-arthrosis Gout or


Rheumatoid Septic
or Traumatic Pyrophosphate
Arthritis, Arthritis
-Arthritis Arthropathy
Appearance Clear Often turbid Turbid Clear with Bakes
of fibrin

Colour Yellow Yellow/green Brown/green Yellow

Viscosity High Low Low Low

Clots ? No Yes Yes Yes

Approx. W.B.C. 1,000 30,000 100,000 10,000


(per mm.3)

Predominant cell Mononuclears Neutrophils Neutrophils Neutrophils

Crystals No No No Yes

Culture Sterile Sterile Positive Sterile

IDENTIFICATION OF CRYSTALS. Uric acid crystals are needle-shaped and strongly negatively
birefringent (blue across the plane of the first-order red compensator); pyrophosphate crystals
have square ends and are weakly positively birefringent (blue along the plane of the
compensator).

SPECIAL TESTS
1. Complement levels are low in rheumatoid arthritis and high in Reiter's disease.
2. Latex tests parallel serum titres in patients with rheumatoid arthritis; there is a high
incidence of false positives in other conditions and the test has little diagnostic value.
Charcot Foot
1. Disorders producing a Charcot joint: Diabetes mellitus (most common
cause), tabes dorsalis, leprosy, syringomyelia, spina bifida, meningomyelcoele,
congenital insensitivity to pain, chronic alcoholism, spinal cord injury and
compression (all have decrease in pain sensation in the presence of
uninterrupted physical activity in common)

2. Mechanism of destruction: Precipitated by a single injury or by repetitive


moderate stress applied to bones and joints. The results are fractures,
effusions, and ligamentous laxity followed by erosion of articular cartilage,
fragmentation, luxation, distintegration, and finally collapse of the foot. The
consequence of trauma is a hyperemic response, which promotes additional
resorption of bone and increases the susceptibility to further injury and
progressive deterioration

NOTE* Eichenholtz divided the disease process into 3 radiographically distinct


stages:
a. Development: The acute destructive period, which is distinguished by joint
effusions, soft tissue edema, subluxation, formation of bone and cartilage
debris (detritus), intra-articular fractures, and fragmentation of bone
b. Coalescence: Noted by a lessening of edema, absorption of fine debris, and
healing of fractures (reparative phase of healing has begun)
c. Reconstruction: Further repair and remodeling of bone take place in an
attempt to restore stability and homeostasis

Neuropathic osteoarthropathy can be arrested during the Development stage if


diagnosed before the disease has a chance to mature

3. Pathogenesis:
a. Sensory-motor neuropathy: Loss of protective sensation, absent DTR's,
diminished vibratory sense, muscle weakness, ankle equinus b. Autonomic
neuropathy: Sympathetic denervation loss of vasomotor control, increased
peripheral blood flow, increased artiovenous shunting, increased bone blood
flow, hyperemia
c. Minor trauma: Repetitive moderate stress, repetitive impulse loading,
trabecular microfractures, inadequately protected fractures and sprains,
surgery
d. Other factors: Metabolic abnormalities which weaken bone, renal
transplantation, immunosuppressive treatment, steroid-induced osteoporosis,
decreased cartilage growth activity, glycosylation of collagen

4. Diagnosis:
a. Medical history, clinical manifestation, and radiologic findings
b. Technium and Gallium scans, MRI (low signal Intensity in T, and T2-
weighted images within the bone marrow space adjacent to the involved
joint), and bone biopsy and cultures in equivocal cases
5. Treatment: Based on the acuteness of symptoms, the anatomic pattern of
bone and joint destruction, the degree of involvement (deformity, fractures
instability, etc.) and the presence of infection a. Immobilization: At least 3
months of non-weight-bearing cast immobilization prior to resumption of
partial weight-bearing (always check the asymptmatic limb during this time)
b. Orthoses and shoes: A patella tendonbearing brace with molded shoes to
decrease the load on the foot
c. Surgery: Ostectomy, arthrodesis
i. Criteria for surgery: Instability, deformity, recurrent ulceration, refractory to
conservative treatment, must be quiescent, circulation intact, no active
infection, medically stable
Chapter 15: Peripheral Vascular
Disease
Patient Evaluation
The Vascular Diseases
PERIPHERAL VASCULAR DISEASE

The podiatric physician should be able to decide whether the patient is at


risk for developing complications based on arterial and venous insufficiency
to the tissues of the foot. The podiatrist should be able to evaluate whether
there is sufficient blood flow entering the foot and leg to sustain its normal
nutrition, to heal, and to sustain nutrition following surgery.
The lower extremity is predisposed to the development of vascular disease.
Some of the causes are: hypertension, hypercoagulability of blood, the
natural aging of the blood vessel walls, diet, tobacco use, oral contraceptives,
prostaglandins and even gravity. A misdiagnosis of vascular disease may
result in significant morbidity.

Patient Evaluation
1. Vascular History: the chief complaint and history of the present illness often
preclude the immediate diagnosis of vascular disease. Some clues to follow:
a. Attempt to establish time relationships accurately (date of onset, etc.)
b. Attempt to separate symptoms that appear to be superficially similar
c. Try to identify the exact location of the symptom
d. Characterize the symptom as to type and severity
e. Delineate factors that aggravate or alleviate symptoms
f. Determine the effect of self-treatment or treatment by other physicians
g. Determine the progression/rate of symptoms
h. Evaluate the amount of disability imposed by the symptoms
i. Analyze any associated conditions or complaints
j. Document any previous studies and their results

NOTE* A study by Hershey et al (1984) revealed that majority of patients with


PVD undergoing major vascular reconstruction will have concomitant problems:
60% had an abnormal EKG, 20% had a previous Ml, 7% had CHF, 4% had
previous arrhythmia and 7% had a previous CVA. Thus, PVD occurs most
frequently in the older population or as part of conditions that compromise
vascular integrity (such as diabetes mellitus)

2. Pain: Arterial, venous, and even lymphatic diseases present with


manifestations, however, painful extremities are not necessarily a
manifestation of a disturbance in peripheral circulation.
a. History: Mode of onset, characteristics, progression, location, duration,
excerbating and alleviating factors, pain on walking flat surfaces/hills, distance
walked
b. Ischemic Pain: Severe persistent pain associated with ulcerated or gangrenous
ischemic tissue is indicative of a more severe arterial disease. Ischemic
complaints in order of frequency are pain, numbness, coldness,
tenderness, burning, fullness, and pallor, more severe during excercise and
localized in the muscles
c. Intermittent Claudication: The classic pain associated with chronic arterial
insufficiency is that of intermittent claudication. This symptom is defined as a
transient, exercise induced ischemic myalgia, characterized by aching,
cramping, tiredness, or tightness of the affected muscle group. Most
often seen in the calf muscles.
The extent of claudication is determined by measuring the number of blocks
walked, prior to the onset of symptoms (a treadmill can be substituted -set at
120 steps/minute and claudication time measured). Generally with arterial
blockage, the lesion occurs one joint or level above the muscle group in which
the symptoms manifest.
The differential diagnosis of intermittent claudication is- musculoskeletal
disorders, venous claudication, shin splints, tendonitis, Morton's neuralgia,
vitamin B1 deficiency, metabolic disorders (McCardle's disease),
and pseudoclaudication of spinal disease.
Intermittent claudication generally is not an indication for reconstructive
surgery unless the symptoms are debilitating.
d. Rest Pain: As the disease and ischemic changes progress, rest pain develops,
usually insidiously.
 Rest pain reflects severe ischemia.
Rest pain is produced by the body's shunting blood from the periphery to a
more central circulation when the person is sleeping. A buildup of
metabolites occurs in the muscles, resulting in pain.
Usually the patient is awakened every night at the same time, and must get
up or dangle the feet to alleviate the pain, sometimes these patients sleep in a
recliner to avoid this pain;

Note* No podiatric elective surgery should be done in patients with these


symptoms.

e. Ischemic neuropathy: Ischemia due to chronic/acute arterial occlusion will


result in an hypoxic condition of the peripheral nerves.
Pain is sharp, shooting, poorly localized, radiating throughout the entire
extremity following no distinct nerve root.
Pain described as tearing, pulling, or agonizing discomfort.
Areas of occurence of intermittent claudication
Clinics in Podiatric Medicine and Surgery: Peripheral Vascular Disease, January 1992, Volume 9:1, Saunders,
Philadelphia, with permission

3. Edema: Can be a manifestation of many diseases, so must be carefully


evaluated as to whether it is bilateral from systemic disease or unilateral
from a regional condition, is the edema pitting (can be graded) or
nonpitting, and when/if does the edema disappear. The edema associated
with systemic disease is generally pitting and bilateral, involving the entire
lower leg/foot/toes
Common causes of Edema in the lower Extremity
a. Systemic: CHF, nephritis, nephrosis, hypervolemia, hypoproteinemia,
hypothyroid, hyperadrenalism, lymphedema
b. Local: lymphangiectasis, lymphatic obstruction (filaria, tumors), pelvic
or abdominal masses, varicose veins, venous stasis, thrombophlebitis,
popliteal vein obstruction
c. Drug induced: corticosteroids, estrogen, progesterone, androgens,
phenylbutazone, MAO inhibitors, hydralazine, methyldopa
d. Other causes: angioneurotic edema and pretibial myxedema, Milroy's
disease

4. History of Emboli: Arterial emboli should be suspected whenever there is a


sudden and marked decrease in circulation to an extremity, with sudden
cyanosis of the foot or toes;
a. The source of arterial embolus is the heart in over 90% cases.
b. Conditions associated with emboli are:
i. mitral stenosis
ii. acute MI
iii. bacterial endocarditis iv. prosthetic valves v. aneurysms vi. stroke

5. Color and Temperature Changes: The color of the skin is directly proportional
to the degree of perfusion of arterial blood and oxygen saturation of
hemoglobin in the subpapillary venous plexus.
a. Pallor associated with sudden arterial occlusion, results from the absence of
arterial perfusion
b. Cyanosis occurs with decreased arterial perfusion of chronic arterial
insufficiency
c. Red/warm skin of erythromelalgia is the vasodilatory effect of the disease
d. Petechiae or pinpoint areas of cutaneous bleeding indicate capillary
fragility, which may arise in a number of conditions
e. Purpura, bluish/black areas of subcutaneous bleeding is the result of clotting
abnormalities or other diseases (see section Dermatology)
f. Telangectasias are visible dilations of capillaries which can result from
arterial or venous insufficiency
g. A drop in skin temperature of 6-8 degrees may indicate ischemia

6. Trophic Changes and Hair Growth: The nutritional changes of the skin and
appendages is directly affected by cutaneous circulation. Alterations in
arterial- flow can cause thickening, brittleness, longitudinal ridging and
slow growth of the nails. This can be confused with onychomycoses. The
disappearance of hair on the foot can be indicative of reduced arterial flow.

7. Past Medical History: Ten risk factors have been associated with
arteriosclerosis- chronic cigarette smoking, hypertension, diabetes mellitus,
hypercholesterolemia, hyperlipidemia, obesity, stress, lack of exercise, age, and
genetics.
a. Family history: Of any of the above
b. Medications/allergies
c. Medical care
d. Social factors
e. Past medical history: Important to assess the healing ability. Factors that reduce
the healing potential are anemia, alcoholism, uncontrolled diabetes,
neuropathy, rheumatoid arthritis, SLE, scleroderma, severe pulmonary disease,
polycythemia, sickle cell disease, treatment with antineoplastic drugs,
radiation therapy, high doses of prednisone, dialysis, and poor nutrition.

Note* Adequate blood flow does assure adequate healing potential, but
inadequate blood flow will contribute to poor healing

8. Ulcerations: Commonly, ulcerations of the lower extremity may be the


first sign of PVD, however, they may occur secondary to hematologic,
endocrinic, dermatologic or systemic disease. Vascular ulcerations can be
due to ASO, venous stasis, thromboangiitis obliterans, acute arterial
occlusion, Raynauds phenomenon, emboli, and systemic hypertension.
(See Section Dermatology)

9. Gangrene: Due to impairment in circulation, can be either wet or dry.

Remember, pedal gangrene is not a diagnosis, but it is a symptom.

An adequate workup is required to determine the causes


a. Drugs- amphetamines, amobarbital, pentobarbital, meperidine HCL,
hydroxyzine HCL, sodium thiopental, propoxyphene HCL and ergotamine
(oral). When injected IV to the tissues these can produce extensive necrosis.
b. Infectious Processes- meningococcic, staphylococcic or streptococcic
bacteremia (SBE), cholera, typhus fever, typhoid fever, pneumonia, and
trichinosis can produce gangrene of the digits.
c. Spontaneous Acute Occlusive Arterial Disorders- arterial embolism
(arteriosclerosis), spontaneous arterial thrombosis (thromboangiitis obliterans,
CHF, cardiac arrthymias)
d. Venous Thrombosis
e. Trauma to blood vessels
f. Compartment Syndrome
g. Exposure to low temperatures- frostbite
h. Mechanical, neurotrophic and metabolic factors
i. Vasospastic disease
j. Connective tissue diseases

10. Palpation of Peripheral Pulses: Peripheral pulses that are evaluated are
the radial, femoral, popliteal, posterior tibial, anterior tibial (dorsalis pedis),
and peroneal. These pulses should be graded according to rhythm,
symmetry, and amplitude.
a. Factors that reduce pulse amplitude: CHF, rapid atrial fibrillation,
numerous premature contractions, paroxymal tachycardia, aortic stenosis,
shock, and myocarditis.
Also included are: coarctation of the aorta, Leriche's syndrome, dissecting
aneurysm of the aorta, tumors of the abdominal cavity compressing
arteries, and edema overlying the artery.
Also included are: Vasospasm (Raynaud's disease, acrocyanosis, livedo
reticularis, later stages of frostbite, causalgia, post-traumatic vasomotor
disorders, disuse atrophy), chronic occlusive arterial disorders affecting the
large arteries, acute occlusion of large arteries, organic occlusion of small
arteries, congenital/acquired A-V fistula, Monckeberg's sclerosis, and trauma
to large arteries
b. Factors that increase pulse amplitude: Hyperthyroidism, hypertension,
aortic insufficiency, marked anemia, fever, physical exertion, and
erythromelalgia.
Also included are: lumbar sympathectomy, early stage of causalgia, Paget's
disease of Bone.
Also, Quincke's pulse is seen in the toenails in cases of aortic regurgitation,
patent ductus arteriosis, very slow heart rate with increased central pressure,
extreme increase in venous return and extreme dilatation of the small arteries
and arterioles. In cases of coarctation of the aorta, Quincke's capillary
pulsations are seen in the fingernails (dueto the increased pressure above the
coartation) but not in the toenails (due to the decreased pressure below the
coarctation)

Note* Please refer to Chapter 33: Anatomy, for description of the complete
circulation of the foot, however, some important arterial anatomic facts are
presented here:
1. The posterior tibial is the largest of the three pedal arteries and bears major
responsibility for perfusing all the plantar intrinsic muscles.
2. The dorsalis pedis is second in importance and is responsible for perfusing
the dorsum of the foot.
3. Ten percent of the normal population does not have a dorsalis pedis
artery, so the perforating peroneal can take its place.
4. Lack of pedal pulses may suggest anatomic variation rather than PVD, if no
other suggestions of vascular insufficiency are present

Palpation of peripheral pulses and corresponding arteriograms (femoral and


popliteal)
Palpation of peripheral pulses and corresponding arteriograms (anterior and
posterior tibial)

Note* When the PT is absent the DP can function as primary artery of the foot. The
entire foot can even be perfused by one pedal artery since they all interconnect
(an important concept in vascular reconstruction). Each toe has 4 digital
arteries, with the plantar arteries being larger than the dorsal ones.
The lateral plantar digital artery of the hallux is larger than the medial
plantar digital artery of the hallux.
The medial plantar digital artery of the lesser toes is larger than their lateral
plantar diqital artery.
Vascular anatomy to foot showing alternate circulation with absent
DP artery
Clinics in Podiatric Medicine and Surgery: Peripheral Vascular Disease, January 1992, Volume 9:1, Saunders,
Philadelphia, with permission

11. Non-Invasive Measurements of Arterial Flow: The most important


measurements of arterial inflow are easy to perform and consist of the following
a. Ankle/Arm Pressure (Ankle/Brachial Index)- is the measurement obtained by
dividing the ankle systolic pressure by the arm systolic pressure. Use a 8MHz
Doppler

Ankle/Arm Ratio Clinical Finding


A/A ratio= 0.96 or more Normal
A/A ratio= 0.71-0.95 Mild obstruction (intermittent claudication)
A/A ratio= 0.31-0.70 Moderate obstruction (intermittent
claudication/rest pain)
A/A ratio= 0.00-0.30 Severe obstruction (rest pain/impending
gangrene)
NOTE* Falsely elevated ankle pressures are seen with diabetes mellitus. This
is due to either ASO or Monckeberg's medial calcific sclerosis. The ABI should
not be the sole criteria for vascular assessment in these patients.

Note* If there is doubt about whether the ABI is falsely elevated, the modified
Carter exercise test can be used to differentiate. Additionally, listen to the
sounds of the artery, it should be either bi or triphasic. Any abnormal
swishing sound would indicate a proximal obstruction.

b. Segmental Pressures- taken at the upper thigh, lower thigh, upper calf,
and ankle. The ABI is noted.
The criteria for this test are based on the 1-10-20-30 Rule of Thumb.

Note* The ABI index if less than 1 may indicate an obstruction.


The upper thigh pressure should be greater than 10mmHG greater than the
brachial pressure, if less there may be an obstruction. Pressure differences
between adjacent cuff sites on the same leg that exceed 20mmHg may
indicate obstruction (some sources use a 30 mmHg drop between sites).
Pressure differences of 30mmHg over the entire leg may indicate obstruction.

c. Doppler Signal- Doppler recordings reflect flow velocity in a specific vessel,


the frequency of the emitted beam is altered by any object moving faster than 6
cm/sec. If waveforms are being recorded, the faster the blood can flow, the
steeper is the recorded waveform. The qualitative audible Doppler signal,
when made into a tracing, shows that the normal arterial pattern is a triphasic
picture. A monophasic sound indicates arterial pathology.
d. Photoplethysmography- is a modality used to provide an indication of skin
blood flow. The emitted beam of the PPG sensor is reflected by hemoglobin
molecules located in the cutaneous microcirculation. A photoelectric detector
measures this reflected beam and the signal is transformed to be displayed as
a recorded waveform looking more like a narrow teepee. This waveform is
representative of pulsatile flow in the subpapillary plexus of the skin. This test
should be correlated with digital Doppler tests, noting discrepancies in the
waveforms.
e. Digital Blood Pressure- this is obtained by placing an appropriate-sized cuff
around the digit, staying away from the joint. In normal digits without any
pathology the pressure is generally between 70-110 mmHg, and have
waveforms that look more like teepees than igloos.
f. Pulse volume recording (PVR)- pulse volume recording, reflects the volume of
blood that pulses under a sensor cuff, obtained by placing cuffs filled to 60
mmHg pressure around the parts to be measured. The PVR closely corresponds to
direct intra-arterial recordings at the level being tested. The waveforms look
like high teepees with a dicrotic notch. Changes in waveform morphology
between segments reflect severe stenosis or occlusions. The waveforms should
be analyzed for changes existing between segments and for differences
between left and right.
g. Five-Minute Reactive Hyperemia Test- used to aid in the differentiation of
vasospastic foot pathology from organic disease. In vasospastic disease, blood
flow is limited by increased tone of smooth muscles surrounding the blood
vessels; in organic disease by material clogging the lumen of the vessels. This
test can also be used to predict the success of a contemplated sympathectomy.

12. Invasive Measurements of Assessing Arterial flow:


a. Angiography for PVD is generally performed as a preoperative or screening
evaluation of the arterial tree in patients with non-healing ulcers, rest pain,
severe claudication, gangrene, and absence of pulses. Non-invasive
examinations should precede angiography to select the appropriate
candidates since this test is not without its potential
complications. The compounds used are either ionic or nonionic, both with
high iodide contents for its radiodensity. They cannot be done in anyone
who has iodine allergy. History of use of similar compounds (such as an IVP
for renal studies) and of thyroid disorders/medications is necessary, with
appropriate consultations.

Ionic Contrast vs. Nonionic Contrast


More painful on injection Less painful
Use in patients < 60 Patients > 60
Use with patients with previous
contrast reaction, history of asthma,
allergy to shellfish, cardiovascular
disease, renal failure, a diabetes

Both are toxic and can cause renal failure, hypotension, direct
myocardial depression, pulmonary edema, bronchospasm, convulsions
and even stroke

NOTE* Adequate preparation of the patient is essential with hydration to prevent


renal complications. This involves infusing approx. 1000cc sterile fluids within 24
hours of the procedure. Also given is Mannitol (12.5 g) in patients with an
elevated creatinine to effect an osmotic diuresis to minimize nephrotoxicity. Oblique
and lateral views are taken.

b. Digital substraction angiography (DSA) is a method of performing


angiography that uses computerized techniques to enhance contrast
visualization. The image before contrast injection is electronically
subtracted from the contrast images, leaving only opacified vessels on the
final image. A DSA image can only be obtained at one position per
injection of contrast. DSA complements regular angiography.

13. MRI: This study is now being done to assess distal runoff in the foot
when angiography is questionable or to double check angiography when
attempting to save a limb.
The Vascular Diseases
1. Treatment of Chronic Arterial Occlusive Disease: Patients with chronic lower
extremity ischemia should be divided into those with claudication and
those with limb-threatening ischemia.
a. Claudicators: Treatment can consist of conservative management
including exercise, elimination of tobacco/caffeine etc., control of medical
conditions. Pharmacological management includes Trental (pentoxifylline)
which alters blood flow characteristics by "softening" the RBC.
b. Limb Threatening Ischemia: Gangrene, rest pain, or tissue loss is an
indication for revascularization. Options for revascularization include
endarterectomy, balloon angioplasty, atherectomy, and bypass grafting (in-
situ autologous vein or prosthetic material-Gortex).

NOTE* In diabetics the most common site of occlusion is just above the trifurcation in
the leg.
Femoral popliteal and popliteal-peroneal bypasses are most common procedures
in these individuals.

The treatment goal is to bypass all hemodynamically significant disease


down to the level of a continuous outflow artery to the foot.

2.. Venous Disease: The venous system is characterized by low pressure,


distensibility and high capacity. It is driven by the effects of respiration
on intraabdominal pressure, the calf muscle pump, and the venous valves.
Veins are superficial or deep, connected, by perforators. a. Venous
Thrombosis (DVT): Conditions predisposing a patient to DVT include prolonged
immobilization, bedrest with a recent MI, women confined to bed during
pregnancy or post Caesarean section, obesity, an underlying malignancy, use
of a tourniquet, a history of ulcerative colitis and Behcet's syndrome, inherited
hypercoagulative states and oral contraceptives. Patients can present with pain
and swelling of the leg, but additionally DVT's can be silent. A patient with
suspected DVT should undergo venography for a definitive diagnosis.
Treatment is with heparin initially (full anticoagulation via IV administration),
then coumadin to prevent pulmonary emboli, and to inhibit further DVT

NOTE* Treatment with heparin is as follows: after drawing blood for a


coagulation profile, 5000 to 10,000 U of intravenous heparin is given
followed by a constant infusion of 1000 to 1500 U/hour. The activated
thromboplastin time is kept at 2-2.5 times the baseline.
DVT prophylaxis is with subcutaneous heparin Q 8-12 hours.

A long term sequelae of DVT is postphlebitic syndrome characterized by failure of


the calf muscle pump, increased ambulatory venous pressure, pain, edema, skin
and soft tissue changes, and ulceration.
b. Pulmonary Embolism: A potentially lethal complication of DVT. Heparin
coagulation is the treatment of choice. There are vena cava devices to prevent
pulmonary embolism while the patient is being treated for DVT.
c. Superficial Venous Insufficiency: Results in primary varicose veins.
d. Deep Venous Insufficiency: Results in secondary varicose veins are
related to incompetent perforator veins or deep venous system insufficiency.

3. Diabetes and PVD: Diabetes is the seventh leading cause of death in the
United States. Vascular complications of diabetes are accelerated by
hypertension, cigarette smoking, hypercholesteremia, hyperglycemia, the
duration of diabetes, and the degree of blood sugar control.
a. Pathogenesis: The three most important factors that influence the vascular
complications of diabetes are- the duration of diabetes after puberty, elevated
blood sugar levels, and blood pressure. Diabetic microangiopathy is a result of
metabolic abnormalities caused by an absolute or relative insulin deficiency.
The development of neuropathy is a critical factor in the rapid progression
of peripheral arterial disease with the concurrent development of autonomic
denervation.

b. Pathophysiology: Microangiopathy is characterized by the thickening of the


basement membrane and arteriovenous shunting.
Hypercoagulability of blood components is recognized as a significant
factor in diabetic angiopathy. Hypertension can cause a threefold increase
in atherosclerosis in the diabetic.

Note* The use of Beta blockers for the treatment of hypertension is contraindicated in
the diabetic because of the unopposed alpha action. Calcium channel blockers or
ACE inhibitors are the recommended drugs for diabetics.

c. Signs end Symptoms: Pain is the most common symptom. The three
signs are- blenching of the foot upon elevation, delayed venous filling time
after elevation, end rubor on dependency. Other signs end symptoms ere
pulseless feet, subcutaneous fat tissue atrophy, shiny skin, absent hair
growth on the lower extremities, and thickened nails.
d. Treatment: Exercise and diet, cholesterol control, cessation of cigarette
smoking, daily aspirin (?) (suppress prostaglandin production in
platelets/suppresses platelet aggregation), blood sugar control, blood
pressure control, drugs (PV dilators).

Note* Treatment for venous stasis ulceration is almost always conservative, 'with
compression therapy with graduated elastic compression stockings (30-40 mm
Hg), Unna Boot with DuoDERM hydroactive dressings etc., or the "Oregon Protocol".

4. Vasospastic Disorders:
a. Raynaud's Vasospasticity: Divided into Raynauds phenomenon, syndrome
and disease. Phenomenon is the clinical presentation of the discoloration
sequence of pallor, cyanosis, end rubor accompanied by paresthesias.
Syndrome is the bilateral end symmetric recurrence of the phenomenon
that persists for longer periods of time with each incident, and is
associated with an underlying collagen or autoimmune disorder, or with
organic arterial disease. Disease is the state of a consistent recurrence of
discoloration with associated paresthesias upon exposure to cold or
emotional crisis that has not been identified to coexist with any other
disorder 2 to 3 years after the original presentation. All of these forms of
vasospesticity ere precipitated by exposure to cold water, ice, cold ambient
temperature, cold humid air, or a chilly wind.
b. Acrocyanosis: A condition characterized by persistent uniform cyanosis of
the feet and toes end associated plantar hyperhidrosis. Unlike Raynaud's,
the vesospesticity extends proximally beyond the digits.
c. Livedo Reticularis: A condition involving the arterioles of the skin
presenting with livid discoloration in a reticular pattern (mottled or lacelike
red-blue discoloration). Has an annular form, livedo annularis, and a grape-
like form, livedo annularis racemosa
d. Medications: The traditional vasodilators, isoxsuprine, pepavarine, niacin
are used in Raynaud's cases. Nitroglycerine 2% ointment applied in a 0.75
inch strip on the dorsum of the foot Q 5 hours has been beneficial when
oral vasodilators have failed. Calcium channel blockers (nifedipine end
diltiezem) have recently been studied for these disorders.
5. Lymphatic Diseases: Can be either primary or secondary lymphedema a.
Primary lymphedema: The classic indication of primary lymphedema is soft
tissue swelling as a result of lymph accumulation. This swelling arises without
cause and may not present bilaterally. It is associated with aplasia, hypoplasia,
or hyperplasia of the lymphatic channels, trunks, or nodes.
i. Milroy's Disease: Congenital/hereditary form of lymphedema, with the edema
noted at birth to be firm but pitting, chronic, permanent, and confined to the
lower extremity.
ii. Congenital Lymphedema: Congenital although no family history is
determinable, more prevalent than Milroy's disease, edema may be present in
the lower or upper extremities.
iii. Lymphedema Praecox: Hereditary, noncongenital manifesting itself in early
life (ages 9-25), seen predominantly in girls, a rapid onset involving one
foot and ankle, progresses to a nonpitting form. Treatment is with an
intermittent compression pump and compression stockings.
b. Secondary Lymphedema: Results from acute, repetitive or chronic
lymphangitis secondary to a known cause. The inflammation may be due to
direct trauma, foreign bodies, infection, or as a complication of venous
insufficiency with associated venous hypertension. It also may result as an
extention of acute cellulitis or deep thrombophlebitis.
i. Traumatic Lymphedema: From direct impact or lacerating injury to the large
pre- and post nodal collecting lymphatics, nodes, and major trunks.
ii. Infectious Lymphedema: From bacterial or filarial infections
iii. Neoplastic and Foreign Body Lymphedema
iv. Postphlebitic Lymphedema
v. Neuroplegic Lymphedema
c. Direct Lymphangiography: The gold standard of lymphatic system imaging
Chapter 16: Internal &
External,
Fixation
Objectives of Fixation Devices
Requirements of Implant Materials
Principles Affecting Internal Fixation
Internal Fixation Devices
AO Objectives
AO Principles
AO Technique
Jumping Screws
Other Techniques
Complications of Fixation Devices
External Fixation
Large Bone External Fixation (Ilizarov Technique)
Small Bone External Fixation of the Foot
INTERNAL & EXTERNAL FIXATION
Objectives of Fixation Devices
1. Eliminate motion at a fracture or osteotomy site.
2. Restore the normal anatomical alignment of the fractured site or the
desired position of an osteotomized segment.
3. Assist in the physiological mechanism of bone healing.
4. Permit early mobilization of the area affected by the fracture or osteotomy.

Requirements of Implant Materials


1. Materials must be resistant to corrosive environment of the body, yet inert
to any foreign body reaction.
2. Material must have strength and durability to endure the stress loads
placed upon it during implantation, bone healing, and subsequent function
of the involved part.
3. Material must be available in various sizes and shapes and practical,
enabling fabrication into fixation devices suitable for implantation, without
the need for complicated hardware or technique.
4. Metals must be compatible with the surrounding environment, thus
reducing the pitting and crevice corrosion phenomena which would lead to
fatigue fracture of the implant device.
5. Use of similar metals within the fixation device to prevent the anode-
cathode "battery affect", or the production of hydrogen ions from saline fluid
within the body. Acidic environment leads to rapid corrosion and fatigue
fracture of implant devices.
6. Should be relatively inexpensive.

Principles Affecting Internal Fixation


1. Alignment and stability across the fracture site must be developed and
maintained during fracture healing to effect bone healing. 2. Tension Band
Principle: Load-bearing through a bone creates one convex and concave
surface subjected to compressive and tension forces. Accordingly, implant
devices are applied to the convex surface of bone or to the side of tension,
to prevent gapping from tensile forces. The gapping forces are counteracted
with proper positioning and selection of the device. This causes
counterreactive force of compression across the fracture site, enhancing
proper fracture healing.
3. Neutralization Principle: Specific anatomic sites are exposed to multiple
stresses, which include torsional and axial loads. These forces may change
with dynamics of muscle and joint activity. The various load forces are
neutralized at the fracture site with plating in combination with bone screws
to minimize movement, especially with multifragmented fractures.

Internal Fixation Devices


1. Suture Material:
a. Absorbable and non-absorbable sutures are used to re-approximate an
osteotomy site. This is done when the osteotomy can be closed without any
tension. If a non-absorbable material is being used, this fixation device is
then considered a permanent type.
b. The only advantage of this material is that it is very easy to use.
c. The disadvantages are that is provides poor compression and low tensile
strength.

2. Stainless steel wire: 316 LVM surgical steel


a. Monofilament is better than braided to achieve compression as it is
twisted down on itself.
b. Its advantages are:
i. Its simplicity
ii. Adequate compression when used properly
iii. Minimal amount of foreign material left in the bone
iv. Acceptable in various anatomic locations independent of surface
irregularities and bone cuts.
v. Easily retrieved, if necessary, postoperatively, and visable on x-ray
c. Its disadvantages are:
i. Difficulty in achieving equal compression along the fracture/osteotomy
site
ii. Possible trauma to bone as the wire is pulled through
iii. Requires good -bone stock
iv. Becomes a permanent fixation device.
v. Fatigue fracture of wire with motion at the fracture site
d. Size used is generally 28 gauge.
e. Tension band wiring using monofilament and K-wires. This technique
provides greater stability than that provided by either component used
separately. The tension band principle applies to bones that are eccentrically
loaded. The application of a tension band device on the tension side of a
bone allows dynamic compression to be generated on the opposite cortex.
f. MRI may be a problem if wire is present in the foot.

3. Kirschner Wire: 316 LVM surgical steel


a. Designs:
i. Come in different lengths and can also be cut to size
ii. Come as single or double ended
iii. Come as threaded or smooth
iv. Tips are either trochar (slip the least along the cortical bony surface and
have the greatest holding power), diamond, or cut tip (poorest holding
power)
b. Sizes include: .028", .035", .045", .062".

c. The advantages are:


i. Application to many sites requiring minimal dissection for fracture
immobilization
ii. Can be inserted percutaneously without the need for surgical exposure,
specifically for implantation
iii. Can be easily removed following surgery once fracture healing is
accomplished
iv. Ability to fixate multiple small fragments
v. Can prevent motion on all three body planes, including axial rotation by
using multiple pins
vi. Can immobilize joints by passing wire through a joint surface, thus
preventing undesirable motion
vii. Can be incorporated within the cast to protect fixation and maintain
position
viii. Is considered a temporary device
d. Its disadvantages are:
i. Creates a track from the external surface of the wound into the bone
ii. Can be a potential source for introducing bacteria
iii. Requires good patient compliance during the postoperative phase
iv. Threaded wires are difficult to remove
v. Threaded wires across a fracture site will maintain separation of
fracture fragments after expected necrosis occurs at fracture.
vi. Can break with the bone if exposed to excessive pressure
vii. Can migrate or slip out of the bone

4. Stelnmann Pins: 316 LVM surgical steel


a. Very similar to Kirschner wires except for their size.
b. Size ranges from 5/32 to 1 /8 inches in diameter (1.9 mm to 4.7 mm).
c. Their rigidity is proportional to the fourth power of their diameter (as with
K-wires).
d. Advantages are the same as with K-wires.
e. Disadvantages are the same as with K-wires.
f. The primary stabilization of subtalar arthrodesis has frequently been
performed using these pins.
g. Are well suited to providing provisional fixation of subtalar and ankle
arthrodesis as well as calcaneal fractures.

5. Absorbable Pins (Polydioxanone/Polyglycolide): At the present time


there are two types of pins available. They were originally designed for
fixation of osteochondral fragments, which were previously treated via
excision and abrasion or fixation with K-wires, screws, or adhesives, which
would leave extensive osteochondral defects.
a. Orthosorb (polydioxanone) (Johnson & Johnson):
i. This pin is available in only one length (1.3 mm x 40 mm long) and is very
flexible.
ii. A tapered variety allows for better compression of osteotomies as they are
inserted into the pilot hole
iii. Has been used with success in digital fusions because of its flexibility
iv. Can be cut with a bone cutting forceps
v. Lose their strength in 4-8 weeks and are totally absorbed in 9-12 months
b. Biofix (self reinforced polyglycolide)(Acuflex):
i. Various diameters from 1.5 mm to 4.5 mm
ii. Various lengths from 10 mm to 70 mm and is very rigid
iii. Lose their strength in 4-8 weeks and are totally absorbed in 6
months
iv. Must be cut with a bone saw or it will shred

6. Staples: Various 2 prong and 4 prong staples are available and are
supplied with templates to assure proper implantation. Available in surgical
steel and titanium.
a. Their application is limited and are best suited for bones with high
cancellous/cortical ratios.
b. When used primarily in diaphyseal bone there is a propensity for the
cortical bone around the staple legs to become communited as the staple
is inserted, resulting in compromised fixation.
c. Advantages are:
i. Easily removable
ii. Can be a permanent implant
iii. Provides fixation on one plane developing static compression across
the fracture fragment site.
d. Disadvantages are:
i. Should not be used in incomplete fractures independently, unless
secondary devices or complete non-weight bearing are utilized.
ii. Staples can dislodge
iii. Staples can fatigue fracture
e. Have been used for calcaneal osteotomies, triple arthrodesis, tib-fib
syndesmosis diathesis, medial and lateral malleolar fractures (with malleolar
screw), and epiphyseal plate injuries.

7. Osteoclasps: 3/16 LVM stainless steel


a. Available in five sizes, 8 mm, 10 mm, 12 mm, 14 mm, 16 mm.
b. The device requires secondary instrumentation for template positioning of
drill holes, and additionally, a tension stat to implant the osteoclasp under
proper tension.
c. Advantages:
i. Can be used in various anatomic locations without the need for additional
surgical exposure
ii. Completely internal and can be considered a permanent implant
iii. Creates fixation with dynamic compression across the fracture site
d. Disadvantages:
i. Limited to incomplete osteotomy where cortical hinge is intact on the
opposite side of placement of the osteoclasp device
ii. Technical difficulty with implantation
iii. Implant may have to be remodeled which weakens its compression force
and may result in spontaneous loosening
8. Bone Screws: Are used to reappose fracture fragments, their primary
advantage over any other type of fixation device is that they can provide
compression and thus more rigid fixation. Cortical bone screws require
pretapping of drill holes and the thread is finer, whereas cancellous bone
screws can be used for self-tapping locations to create a lag effect across a
fracture site. Cancellous screw threads are much larger and grasp greater
surface area of bone to achieve fixation.
a. Four basic structural dimensions are employed to precisely characterize
screws:
i. Root or Core diameter is the minimal diameter of the screw not including
the threads (Fig. 2).

NOTE* The tensile strength of screws is proportional to the square of the


root/core diameter, and the shear strength of screws is proportional to the
cube of the root/core diameter.

ii. Thread diameter is the maximal diameter including the screw threads (Fig.
1).
iii. Screw pitch is the distance between two successive threads (fig. 3).
iv. The lead is the distance a screw advances when turned one complete
revolution.
b. Other screw parts are:
i. Screw head: either cruciform or hexagon
ii. Screw land: the undersurface of the screw head
iii. Screw tip: either round, pointed, or fluted
Al 4.0 mm Partially threaded cancellous screw 1.75 mm pitch

B2 4.0 mm Fully threaded cancellous screw 1.75 mm pitch (Formally 3.5 mm


cortical screw

C3 3.5 mm Fully threaded cortical screw 1.25 mm pitch

D4 3.5 mm Fully threaded cancellous screw 1.75 mm pitch

Reprinted from Ruch JA, Vito GR Corey SV (eds); Podiatry Institute Internal Fixation Workbook. 8th ed.,
Podiatry Institute Publishing, Tucker, Georgia, 1992, with permission
iv. Screw shank: the distance between the land and the start of the screw
runout (Fig. 4)
v. Screw runnout: the distance from the end of the shank to the first thread (
Fig. 5).
vi. Screw thread: either assymmetric (buttress) or symmetric (Fig. 6)

NOTE* Bone possesses a significantly lower modulus of elasticity than metal


alloys. The buttress AO thread is designed to maximize the volume of bone
between threads and increase the holding potential of the screw in the weaker
bone matrix.

c. Cortical Screws: (see figure 7)


i. Function as either a positional screw (provide plate fixation) or a lag screw
(exerts compression)
ii. Compression is only achieved when the threads of the screw do not engage
the cortex of the near osteotomy or fracture fragment, accomplished by
overdrilling.
iii. Cortical screws measuring 3.5 mm in diameter are used in lag fashion to
provide interfragmentary compression in the distal fibula, rearfoot, and
occasionally the metatarsals.
iv. Screws measuring 2.7 mm, 2.0 mm, and 1.5 mm are also employed to
stabilize metatarsal fractures or osteotomies. Screws measuring 1.5 mm are
used in the proximal phalanx of the hallux for fracture fixation.
v. When screws are used alone for fragment fixation, two smaller screws provide
increased resistance to shear and torsional stresses.
vi. When screws are used for interfragmentary compression, they should be
inserted so that their direction bisects the perpendiculars to the fracture line
and the long axis of the bone involved.

Note* If one screw is used for a base wedge osteotomy with an intact cortical
hinge, the angle of insertion of the screw should bisect the perpendiculars of
the long axis of the osteotomy and the long axis of the bone. If the screw is
placed at an angle greater than this, the cortical hinge will disrupt.

vii. Sufficient screw fixation can usually be obtained with oblique and spiral
fracture patterns only when the fracture line is at least twice as long as the
bone's diameter.
viii. Short oblique or transverse fractures, therefore need an interfragmentary
lag screw and neutralization plate.
d. Cancellous Screws: (see figure 7)
i. Come either fully or partially threaded
ii. Cancellous screw thread height is greater than that of cortical. This allows
for greater purchase in the softer metaphyseal and epiphyseal bone for which
they were designed.
iii. Screw head fixation can be augmented in osteoporotic bone with a washer.
iv. If the threads of a cancellous screw are left in a position crossing the
interface between two fragments, no compression will be achieved, as the lag
affect that is desired from this screw will be negated. It then acts as a cortical
screw.
v. Cancellous screws 6.5 mm in diameter are used in ankle and subtalar
arthrodeses.
vi. Lisfranc's injuries are amenable to 4.0 mm cancellous screws.
vii. Fractures of the talus and calcaneus are frequently stabilized with
cancellous screws (in these locations are generally augmented with washers or
small plates).
e. Washers:
i. Generally used in osteoporotic bone.
ii. Used with cancellous screw for increasing the purchase power on the near
fracture cortex.
iii. Can be used with screws to provide increased surface area as well as barbs
for the reattachment of ligaments or transferred tendon insertions.
f. Malleolar Screws:
i. Are self-tapping and possess a sharp pointed tip that was designed to allow
insertion without predrilling.
ii. Due to their large size, 4.0 cancellous screws have replaced them.
g. Cannulated screws: The complications involving placement of screws in
complicated fractures can be greatly minimized with this type of screw.
i. This type of screw can be inserted over a guidewire through its entire length,
after the guidewire is properly placed in the bone. This minimizes bony trauma.
ii. When a cannulated screw is to be used, the K -wire (guidewire) serves a dual
purpose of maintaining reduction and providing a guide for screw placement.
h. Herbert Screw:
i. Originally designed for osteochondral fractures (also used for scaphoid
fractures of the hand), due to the absence of a screw head.
ii. Characterized by the presence of threads with different pitches and leads on
both its proximal and distal ends. The distal threads feature a tighter pitch and
smaller lead and are separated from the proximal ones by an intervening
smooth shank. This allows for interfragmentary compression.
i. Reese Arthrodesis Screw: Right/lefthanded threaded screws which are used
for digital fusions.

Note* A screw can be used alone for internal fixation whenever the fracture or
osteotomy is at least twice as long as the diameter of the bone at the level of
the fracture or osteotomy.
A screw inserted at right angles to the fracture or osteotomy plane gives the
best interfragmental compression, but provides no stability under axial
loading. A screw inserted at right angles to the long axis of the bone gives the
best resistance to axial loading, but decreases the interfragmental
compression. Based on the previous 3 principles, a cortical lag screw is
inserted so that it bisects the angle formed by the perpendicular to the fracture
plane and the perpendicular to the long axis of the bone
9. Intermedullary Fixation (Nails): These are long pieces of metal of various
available diameters which are placed in the medullary canal of a
fractured/osteotomized long bone to stabilize the site.
a. They are:
i. Rush pins
ii. K-wires
ii. Inyo nails (tapered V-shaped stainless device used for fractures of the distal
fibula)
b. Are wedged into the medullary canal after the canal is reamed to a diameter
slightly smaller than the nail to be used, and then removed after healing is
completed.
c. Of all the internal fixation devices used, this one delays bone healing the
most by damaging the medullary blood vessels when it is inserted.
d. The other major drawback it is limited control of the rotational forces of the
fracture fragment.

10. Plate Fixation: Are temporary fixation devices which serve a particular
function and then are removed. Plates can function in several fashions,
depending upon how they are applied and the resulting bone-plate construct
geometry. These functions include rigid fixation through interfragmentary
compression, buttressing, and neutralization.
Depending upon the mechanical circumstances, a plate may provide more than
one of these functions.
a. The following plates are utilized:
i. Static Compression Plates: Tension is applied to the implant and
compression is achieved at the fracture interface.
ii. Dynamic Compression Plates: Beyond the compression of the fracture
achieved through static compression, the implant is subjected to a
physiologic load which generates additional compression at the fracture
plane
iii. Neutralization Plates: Initially a shaft fracture may be fixated by
interfragmental compression with a lag screw. A plate is then applied to
neutralize or absorb-any disruptive forces; torsional, shear, or bending to
which the bone and osteosynthesis may be subjected
iv. Anti-Glide Plates: Are used as neutralization plates but placed on the
posterior aspect of the fibula.
v. Buttress Plates: Are used to maintain separation of bone during bone
grafting procedures to gain or maintain length. Are generally used to resist
the tendency of metaphyseal fracture fragments to displace when subjected
to compressive forces. Specifically designed plates by the AO group are
spoon and cloverleaf plates for the distal tibia, and the malleable H or
double-H plates for the calcaneus.
b. Pre-stressing the plate results in static interfragmentary compression,
and is performed by contouring the plate so that its center sits away from
the bone to which it is applied. The screws securing the plate ends are
inserted and tightened first (pre-stressing the plate in tension) so that as
sequential screws are applied (progressively closer to the center) axial
compression is developed along the underlying bone. In addition,
eccentrically plated screws may be inserted (as a compression device) for
interfragmentary compression.
c. Plates also function to protect lag screw fixation. Oblique or spiral fracture
of the metatarsals or the distal fibula can be stabilized with
interfragmentary lag screws. The addition of a plate then serves to neutralize
the bending, torsional, and shear forces that would otherwise jeopardize the
fixation obtained by lag screws alone.
d. The AO group has developed 1 /3 and 1 /4 tubular plates which are
easily contoured.
e. The advantages of plates are:
i. Allows for complete reduction of fracture fragments and proper
anatomical alignment.
ii. Can be implanted permanently or removed at a later date.
iii. Creates rigid fixation with stabilization and/or dynamic compression
across the fracture site.
f. The disadvantages are:
i. Significant amount of surgical dissection for implantation of plate and
screws.
ii. High degree of difficulty with irregular or multifragmented bone
fractures.
iii. Technical difficulty for implantation, potential fatigue fracture of bone
plate with motion.
iv. Should be applied to the tension side of the fracture to avoid
breaking the bone plate.
iv. Results in a degree of bone necrosis beneath the plate.

11. External Fixator Devices: These devices are available in a variety of


sizes depending upon the location to be used. Their prime indication is
severe trauma, especially associated with open fractures. Also can be used
in the treatment of infected fractures, non-unions involving the ankle,
arthrodesis of the subtalar joint or ankle joint, acute and chronic OM, and
chronic septic arthritis.

NOTE* Specific guidelines for their use have been outlined by Kenzora and
Edwards and associates in The Foot and Ankle. They recommended the use of
various configurations of Hoffman's external fixators in order to:
a. Stabilize open fracture-dislocations
b. Maintain length where bone is lost or extensively comminuted
c. Prevent soft tissue contractures
d. Control joint position for delayed ankle arthrodesis
e. Provide easy access for bone and soft tissue reconstruction

a. Charnley compression clamp: Has been utilized in combination with


Steinmann pin fixation. It is applied on each side of the extremity and
attached to an exiting pin. Turn-buckle style adjustments are made on each
side of the extremity forcing compression across the fracture site.
b. Hoffman fixator devices: Were designed for the small bones of the hands
and feet, and have greatly enhanced the use of external fixator techniques in
fracture repair and bone grafting techniques.
c. Advantages:
i. Ability to be adjusted during the healing phase
ii. It is only a temporary device
iii. Its ability to provide rigid fixation while allowing ready access to
surrounding soft tissues for debridements and dressing changes as necessary
iv. Neighboring joint motion can be preserved
d. Disadvantages:
i. Difficult to use and requires special instrumentation
ii. Pin-tract loosening and infection
iii. Requires good patient compliance
iv. Creates a bulky external apparatus which will hinder the activity of the
patient

AO Objectives
1. Atraumatic operative technique
2. Accurate anatomical reduction
3. Rigid internal compression fixation 4. Avoidance of soft tissue damage
5. The AO tenet: "Life is movement, movement is life".

AO Principles
1. Intrinsic Factors Affecting Stable Fracture Reduction:
a. Stable fractures:
i. Are transverse fractures
b. Unstable fractures:
i. Long oblique fractures
ii. Comminuted fractures
iii. Spiral fractures
c. Potentially stable fractures:
i. Short oblique

2. Extrinsic Factors Affecting Stable Fracture Reduction:


a. The disruptive mechanical forces are bending, shear, and torsion.

3. Mechanical Basis for Stable Fixation:


a. Types of interfragmental compression
i. Static compression: a constant and uniform force across a
fracture/osteotomy site, accomplished by lag screw technique, a preloaded
plate, or external fixator.
ii. Dynamic compression: is the combination of a statically loaded fixation
device to a functionally loaded fracture configuration (the tension band
concept).
b. Splintage: A technique applied when interfragmental compression is not
possible and is used in combination with interfragmental compression when it
alone is not adaquate to provide stable fixation. c. Combinations: combination
of techniques of interfragmental compression and splintage (i.e. a single lag
screw plus reinforced by a plate).

4. Techniques of Stable Fixation:


a. Single lag screw: A cortical screw with a glide hole or a cancellous screw with
all the threads on the distal side of the fracture fragment. A single lag screw can
provide adequate interfragmentary compression, however, is not able to
withstand shearing and bending loads, so the fracture/osteotomy must be
protected.
i. Angle of screw insertion: Should be placed so that the angle of the screw
bisects the perpendicular of the fracture/osteotomy and the perpendicular of
the longitudinal axis of the bone. If the angle of the screw deviates from the
plane of the fracture, there is a shift of the near fragment in the direction of the
course of the screw as the screw is tightened and compression is created.
b. Multiple lag screws (two or more): Are used in a long/oblique or spiral
fracture, where the length of the fracture is at least twice the diameter of the
diaphyseal bone involved.
i. Angle of screw insertion: when fixating a fracture/osteotomy with several
screws, the first screw should be perpendicular to both cortices and be
centrally placed. The second and third screws (placed on either side of the
first screw) are placed perpendicular to the plane of the fracture. This
prevents a frontal plane shift, called the shear effect, of the longitudinal
relationship of the fracture fragments. These secondary screws can also be
placed so that they bisect the angle between the perpendicular of the
fracture and the perpendicular of the cortical surface. This reinforces
interfragmentary compression.

AO Technique
1. Instrumentation:
i. Thread hole drill bit: (1.1, 1.5, 2.0, 2.5, 3.2 mm)
ii. Glide hole drill bit: (1.5, 2.0, 2.5, 2.7, 3.5, 4.5 mm)
iii. Countersink: (Mini Fragment Set 1.1 and 2.0 mm tip) (Small Fragment
Set 2.0 mm tip) (Large Fragment Set 3.2 and 4.5 mm tip)
iv. Depth gauge: (Mini/Small/Large)
v. Tap: (1.5, 2.0, 2.7, 3.5 mm @ 1.25 pitch) (3.5 mm @ 1.75 pitch) (4.5 and
6.5 mm)
vi. Screw Driver: (Cruciform/Hexagon head)
v. Drill and Tap Sleeve: (protection for the soft tissue/ reduces the need for
excessive retraction when the drill bit Is aimed obliquely at the bone/the
serrated end anchors well to cortical bone and prevents slippage of the drill
bit)
2. Sequence For Screw Insertion:
a. 1.5 mm Cortical Screw
i. Pre-drill (0.035 K-wire= 0.9 mm)
ii. Thread Hole (1.1 mm)
iii. Countersink (Mini)
iv. Overdrill near cortex (1.5 mm)
v. Depth gauge (Mini)
vi. Tap (1.5 mm)
vii. Screw Placement
b. 2.0 Cortical Screw
i. Pre-drill (0.045 K-wire= 1.1 mm)
ii. Thread Hole (1.5 mm)
iii. Countersink (Mini)
iv. Overdrill near cortex (2.0 mm)
v. Depth gauge (Mini)
vi. Tap (2.0 mm)
vii. Screw Placement
c. 2.7 Cortical Screw
i. Pre-drill (0.062 K-wire= 1.6 mm)
ii. Thread Hole (2.0 mm)
iii. Countersink (Small)
iv. Overdrill (2.7 mm)
v. Depth Gauge (Small)
vi. Tap (2.7 mm)
vii. Screw Placement
d. 3.5 Cortical Screw
i. Pre-drill (0.062 K-wire= 1.6 mm)
ii. Thread Hole (2.5 mm)
iii. Countersink (Small)
iv. Overdrill (3.5 mm)
v. Depth Gauge (Small)
vi. Tap (3.5 mm)
vii. Screw Placement
e. 3.5 mm Cancellous Screw
i. As with 3.5 Cortical Screw but eliminate the 2.5 mm Thread Hole
f. 4.0 Cancellous (partially threaded)
i. Pre-drill (0.062 K-wire= 1.6 mm)
ii. Thread Hole (2.0 mm)
iii. Countersink (Small)
iv. Overdrill (3.5 mm)
v. Depth Gauge
vi. Tap (3.5 mm)
vii. Screw Placement
g. 4.0 Fully Threaded Cancellous Screw
i. Pre-drill (0.062 K-wire= 1.6 mm)
ii. Thread Hole (2.0)
iii. Countersink (Small)
iv. Depth Gauge
v. Tap (3.5 mm)
vi. Screw Placement
h. 4.5 Cortical Screw
i. Pre-drill (0.63 K-wire= 1.6 mm)
ii. Thread Hole (3.2 mm)
iii. Countersink (Large)
iv. Overdrill (4.5 mm)
v. Depth Gauge
vi. Tap (4.5 mm)
vii. Screw Placement
i. 6.5 mm Cancellous Screw (partially threaded)
i. Pre-drill (5/64 K-wire)
ii. Thread Hole (3.2 mm)
iii. Countersink (Large)
iv. Depth Gauge
v. Tap (6.5)
vi. Screw Placement
j. 3.5 mm Cortical Screw (using a T-Sleeve)
i. 3.5 mm drill (proximal cortex only)
ii. 3.5 mm x 2.0 mm drill sleeve
iii. 2.0 mm thread hole of the far cortex
iv. Countersink
v. Depth Gauge
vi. Tap (3.5 mm)
vii. Screw Placement
k. Exercise: Modified Austin With 2 x 2.7 mm Cortical Screw Placement
i. Osteotomy performed with lateral shift of the capitol fragment
ii. Temporary fixation (0.045 K-wire)
iii. Temporary fixation: pilot hole for the proximal screw (0.062 K-wire)
iv. Pilot hole for the distal screw (0.062 K-wire)
v. 2.0 mm thread hole (distal screw)
vi. Countersink (distal screw)
vii. 2.7 mm over drill (distal screw)
viii. Depth gauge (distal screw)
ix. 2.7 mm tap (distal screw)
x. Insert 2.7 mm distal screw
xi. Remove proximal temporary fixation
xii. Proximal screw insertion (as just described)
xiii. Remove distal temporary fixation
xiv. Tighten screws

3. Plating Procedures:
a. Prestressed Plate: Because of the linear design of these plates, this
technique is best used in long bone fractures. The axial load created by a
prestressed plate is a form of static compression, and can be accomplished
three ways:
1. Load Screw Technique:
 The most common plate technique in the foot and ankle.
 These plates are thin and are semi 1 /3 and 1 /4 round.
 They are to be used for tension only and do not provide rigidity against
bending (provided by the thicker Dynamic Compression plate).
 Performed by using eccentrically drilled holes, (for the initial two holes)
one just proximal and one distal to the fracture/osteotomy site. As the
screws are tightened down they move the fracture fragments together.
This can be done as a result of the ovoid design of the screw holes in the
plate. These are the load screws. The other screws must be centrally
placed or they will dislodge these load screws.
 To prevent the fracture surface from gapping on the opposite side of the
tension surface, the plate can (and should) be prebent at its center. It
now acts as a leaf spring and resists gapping of the opposite cortex.
 The use of this concept is limited to transverse metatarsal fractures and
arthrodesis of first metatarsal-cuneiform arthrodesis.

ii. Dynamic Compression Plate: This incorporates the load screw technique
with the added effect of geometrically designed slots within a plate.
 These plates are thicker and stiffer than tension plates.
 These geometrically designed slots have two features: they are oblong and
longitudinally placed, and they have two different slopes (the first acute slope is
the compression slope/the second slope is the gliding slope). Both of these
features allow for linear motion.
 All the screws can be used as load screws (because there is space for the
first two screws to glide after the other screws are tightened down).
 There are individual plates corresponding to the 2.7 mm/3.5 mm/4.5 mm
cortical screws.
iii. Tension Device: This can be done only with large bones. It is done by
anchoring a tension device to one of the fragments and to a free end of a plate,
then anchoring the plate to the other side of the fragment and then tightening
the tension device. This causes interfragmental compression.
Jumping Screws
In case of screw failure you must have a backup or alternative. This explains
how to change screws properly
1. To go from a 1.5 to a 2.0 mm screw: Use a 1.5 mm thread hole, followed
by a 2.0 overdrill

2. To go from a 2.0 to a 2.7 mm screw: Use a 2.0 thread hole followed by a


2.7 overdrill (may need to re-countersink)

3. To go from a 2.7 to a 3.5 mm cancellous screw: Use a 3.5 overdrill

4. To go from a 2.7 to a 3.5 mm cortical screw: Use a 2.5 thread hole


followed by a 3.5 overdrill

5. To go from a 2.7 to a 4.0 mm cancellous screw (may be the best choice):


Needs no instrumentation

6. To go from a 3.5 cancellous to a 3.5 mm cortical screw.: Use a 2.5 mm


thread hole

7. To go from a 3.5 cortical to a 3.5 mm cancellous: Needs no


instrumentation

8. To go from a 4.0 cancellous to a 3.5 mm cortical screw: Use a 2.5 thread


hole followed by a 3.5 overdrill

NOTE* Never retap after the first screw fails

Other Techniques
1. Splintage: A technique used to splint or protect a reduced fracture. The
primary uses of splintage are: when interfragmental compression cannot be
used, epiphyseal fractures, and to protect a tenuous interfragmental
compression.

2. Circlage Wiring: The classic application in podiatry is the dorsal loop


technique for an abductory closing wedge osteotomy, even though is has proven
to be the weakest form of internal fixation. It does provide apposition of the
osteotomy surfaces, but provides little stability. The most secure fixation is two
loops in a 90 degree orientation to each other

3. K-wires: A single K-wire rarely provides any rigidity, however, crossed Kwires
are best. This is not without its shortcomings as distraction of fracture
fragments can occur. K-wire fixation alone does not afford interfragmental
compression. K-wires do offer stability when used in combination with
intraosseous loop techniques. Threaded K-wires are rarely used as they are
mechanically unsound.

4. Tension Banding: Monofilament wire threaded in a figure 8 fashion, used in


combination with two K-wires to give interfragmental compression (Figure 11).
Good with Jones fracture, and some ankle fractures. The plane of insertion of
the 2 K-wires must be parallel to the plane of the drill hole for passage of the
monofilament wire
Figure 11: Tension Band Wiring Techniques
Reprinted from Ruch JA, Vito GR, corey SV (eds); Podiatry Institute Internal Fixation Workbook. 8th ed.,
Podiatry Institute Publishing, Tucker, Georgia, 1992, with permission
Note* The tension band principle is applied when an eccentric load is placed on
a bone, and reduction is attempted. The eccentric load creates a concavity on
one side (which is under tension), and a convexity on the other side which is
under compression. The tension band absorbs the tensile force, and the bone
(load beam) absorbs the compressive forces. The 2 areas that are easily
accessible to this principle in podiatry are the 5th metatarsal and the two
malleoli Principles:
a. Neutralize the distracting force and convert to a compressive force
b. Apply the tension band to the tension side only (convex side)
c. K-wires eliminate the rotational instability

Complications of Fixation Devices


1. Infection: Despite long-standing efforts directed at eliminating this
complication, there appears to be an irreducible minimal infection rate destined
to plague both patient and physician. The potential exists for the growth of
resistant bacterial strains or superinfections as a result of increased use of
antibiotics. Studies involving the use of prophylactic antibiotics have shown a
decrease in the incidence of postoperative infections when fixation devices are
used (first-generation cephalosporins given preoperatively provide good
coverage against Staph aureus and many gram(-) rods, and are most widely
used). Implants frequently need to be removed in the presence of a deep
infection, however, should be left in place in the absence of bony union even
with the presence of an infection (infections are difficult to manage without
stability, and so do better with stabilization).

2. Slippage of the Fixation Device: Screws, K-wires, as well as the rest of the
fixators can lose purchase and slip out of place. When this happens, the device
must be removed and replaced with an alternate.

3. Mechanical Failure: Has not been a frequent problem.

4. Inappropriate Use of Fixation Devices: This is a significant problem. Next


to infection this is the most common cause of implant failure.

5. Stripping of a Screw Head/ Breakage of a Screw: When stripping occurs a


vise grip is used. When a cancellous screw is removed after many months it can
break. A cancellous screw is unable to cut bone when it is backed out, and if
excessive torque is applied when bone has grown in around the smooth shank,
the screw can break.

External Fixation
An external fixator can be used in many different ways in the fixation of the
osseous skeleton. However, the use of an external fixator is presently limited in
foot an ankle surgery. With the understanding of the techniques and training
now available, should become a more popular method in the surgeon's
armamentarium. The techniques discussed will be divided into large bone and
small bone fixation

Large Bone Fixation


1. Ilizarov technique: This method of external fixation was developed in
Kurgon, Russia and has been used successfully to treat surgical and
traumatic fractures, osteomyelitis (without sequestrectomy or even
antibiotics), non-unions, osteotomies, fusions, pseudoarthrosis, angular
deformities, limb shortenings, and joint contractures using a surgical
technique that respects osteogenic tissues and their vascular supply while
preserving the weightbearing function of the limb. This is all due to the
massive neovascularization coupled with mechanical control of the limb
permitting not only histogenesis of bone, muscles, nerves, and skin, but also
transformation of pathological states such as osteomyelitis, fibrous
dysplasia and pseudoarthrosis into normal bone. This technique requires
strict adherence to certain surgical, anatomic, and mechanical principles.
Surgically, it is necessary to maintain the periosteum, endosteum, and bone
marrow with its blood supply, via transection of only the bony cortices.
It is important to understand that this technique is very difficult to master,
has a long learning curve, and is technically demanding. Because of the
complexity of the different methods of assembly, no surgical technique
brochure could possibly explain all of the variations of usage.
a. Components: The Ilizarov external fixator is a modular apparatus consisting
of parts that can be assembled in an unlimited number of configurations. With
one or more rings affixed to each bone fragment, the frame can be used to
compress, distract, angulate, or rotate bone segments with respect to each
other. In this manner, deformities can be overcome while at the same time the
limb is made stable enough to permit weightbearing and functional use.
i. Transfixation wires: The mainstay of the Ilizarov system, consisting of 1.5 and
1.8mm the latter utilized for lower extremity adult pathology/deformity.

NOTE* After a transfixation wire is inserted, one end of the wire is secured to
the frame, the other end of the wire is tensioned before final fixation

ii. Olive wires: A Kirschner wire with a small bead on it used to abut
against cortical bone to stabilize or pull bone segments. Can serve several
functions: can act as a stabilizing element, can act as a fulcrum or rotation
point around which a deformity correction occurs, or can act as a traction
element to pull bone in a desired direction
iii. Rings: Can be either half-rings, full rings, 5/8 circle rings and Omega rings
(for the shoulder). The half-rings can be bolted together (sizes from 80-240 mm
in diameter) and then the wires are secured to them. One ring can be bolted to
another ring via threaded rods tightened by a nut on the end.
iv. Arches: Arches are large, heavy, curved plates used most commonly for
fixation of the upper femur, and come is three diameters, 90, 110, and 140mm
v. Nuts and bolts: Among other things, these secure the half-rings together. The
bolts come in 10, 16, and 30mm lengths and the head of the bolt fits a 10mm
metric wrench
vi. Fixation bolts: Are used to secure wires to the rings and are either
cannulated, grooved, or cannulated with a tapped head. A cannulated bolt is
used when a wire passes across the center if a hole at the point of fixation, a
grooved bolt is used whenever a wire is tangential to a fixation hole, and a
cannulated bolt with a tapped head is used when wire fixation is needed in a
crowded situation where a connecting rod, socket, plate, or other hardware
must be attached to the same ring position as a wire.
vii. Washers: There are plain washers, grooved washers, and paired spheric
washers. A grooved washer can serve for wire fixation anywhere, and if a wire is
far off a ring's plane for fixation, enough washers can be stacked on a long bolt
to secure the wire. A pair of grooved washers surrounded by a pair of nuts on a
threaded rod can also secure a wire. A pair of spheric washers are useful in
compensating for angulation between a ring and a threaded rod and allow
about 7.5° of anglation in a hole
viii. Threaded rods: Are the basic connectors between the support rings and
come in lengths from 30 to 400 mm
ix. Telescopic Tubes: Are used to prevent frame deformity when there is a long
distance between the support rings
x. Ratchet telescopic rods: Incorporates a ratchet mechanism to simplify
distraction, which the patient can rotate to extend the rod 0.25 mm, and is
calibrated so that the surgeon can assess the elongation or shortening
xi. Posts: Come with one, two, three, or four holes, can have many functions.
Two being to act as a fixation point for wires off the plane of a ring, to act as a
swivel for ring rotation as well as points for pushing or pulling a ring
xii. Buckles: Were Ilizarov's original fixation device
xiii. Plates: Can function as wire attachment points, as a stable supporting
element in push configurations designed simultaneously to angulate and to
translate ring clusters with respect to each other, or to simply enlarge the
diameter of, a small ring
 Straight plates
 Paddles
 Twisted Plates
xiv. Sockets: They function not only as interconnectors between threaded rods,
but also as spacers to raise a point of attachment off the plane of a ring or plate
xv. Bushings: Due to its configuration, will slide along any rod in a fixation
frame. The free movement of such an assembly is used to build a
mechanism for counterrottation of rings or traction on threaded rods or as a
slide assembly to move componants along threaded rods
xvi. Wire tensioners: Are either spring-loaded or threaded. Are used to, apply
tension to the transfixation wires

b. Ring selection: Allow 2-3 cm of clearance between the inner edges of rings
and the “skin”
c. Wire and pin placement: The pins and wires must be placed in certain
locations at specific anatomical levels. The following diagrams show proper pin
placement at different levels
d. Wire tensioning: To achieve enough stiffness in the wires to maintain
stability and overcome intrinsic tissue resistance, the wires must be stretched
like a tightrope. The multiplanar fixation with tensioned wires provides an
optimal environment for bone formation. The fixation resists bending and
torsion, thus minimizing shear forces at the bone-healing interface. The use of
a Richards dynamometric wire tensioner is preferred to accurately tension each
wire. A calibration scale is noted on this instrument from 50 to 130 kg of force.
A tension of 70-110 kg is utilized with a 1.5 mm diameter wire and 70-130 for a
1.8 mm wire
e. Hinge placement: Complex deformities consist of more than one of the
following deformities: length, rotation, angulation, and translation. The actual
sequence of correction of complex deformities can vary, however, in general,
length must be achieved prior to offset and translation, and rotation should be
accomplished last. Once the plane of deformity and the maximum angulation
and translation have been determined, determination of hinge placement is
necessary. This is worked out by geometry (see Figs 1-5)
f. Ilizarov corticotomy: The method of limb lengthening or bone
lengthening consists of external distraction of a surgically created
osteotomy or corticotomy via a percutaneous, subperiosteal incision,
perserving periosteum and endosteum. After a latency period after a
corticotomy anywhere from 7-14 days, distraction of the bone can begin,
anywhere from .5 to 1.0 mm per day. This is called distraction
osteogenesis
g. Techniques in fracture reduction: Several factors are taken into
consideration when a frame is constructed: size and number of fracture
fragments, plane of the fracture lines, condition of the soft tissues, and
proximity of the fracture fragment of the joint and intra-articular
involvement. As a general rule one should achieve 2 levels of fixation in
each major fracture fragment (2 rings applied to any bone segment). The
diatance from a fracture line to a ring is usually 3-4 cm, giving enough
room for compression or distraction, or angulation and translation. The
angle formed by 2 wires crossing a fracture fragment should approach 900
for maximum stability. Intra-articular fractures should be reduced prior to
diaphyseal fractures
h. Illustration of techniques for the foot and ankle:
Malleolar fractures

Standard assembly for a transsyndesmotic fibular fracture of the posterolateral


rim with medial lesion or in presence of Volkmann fragment:
a) Ilizarov technique (Malzev-Kirienko);
b) Hybrid Advanced technique with internal osteosynthesis (1 = open reduction,
2 = centralization, 3 = fixation) (Catagni).

Catagni, M.A., Malzev, V., Kirienko, A., Advances In Ilizarov Apparatus Assembly, A. Bianchi Maiocchi (ED),
Medicalplastic srl, Milan, 1994
The universal joint allows movement of the ankle during the postoperative
period.

Standard assembly for a malleolar fracture (Malzev-Kirienko).

Lateral view of the assembly for reduction and compression of the medial
malleolus.

Catagni, M.A., Malzev, V., Kirienko, A., Advances In IlIzarov Apparatus Assembly, A. Bianchi MaIocchi (ED),
Medicalplastic srl, Milan, 1994
Distal articular (tibial plafond fractures)

Standard configuration for a typical comminuted fracture of the tibial plafond


(Malzev-Kirienko).

Catagni, M.A., Malzev, V., Kirlenko, A., Advances In IlIzarov Apparatus Assembly, A. Bianchi MaIocchi (ED),
Medicalplastic srl, Milan, 1994
CatagnI, M.A., Malzev, V., Kirlenko, A., Advances in Ilizarov Apparatus Assembly, A. Bianchi Maiocchi (ED),
Medicalplastic srl, Milan, 1994
Equinus foot

Apparatus assembly after correction of the equinus foot;

a) Standard apparatus assembly for correction of an equinus foot deformity.


b) Diagram of wire insertion at trascalcaneal and transmetatarsal sites. A
fourth wire in the tarsus may be placed according to severity of the case
(Malzev-Kirienko).

Catagni, M.A., Malzev, V., KIrienko, A., Advances In IlIzarov Apparatus Assembly, A. Bianchi Maiocchi (ED),
Medicalplastic srl, Milan, 1994
Small Bone External Fixation of the Foot
1. Orthofix modulsystem (Pennig minifixator): Allows for secure fragment
fixation, with minimum of invasive surgery
a. Can be applied under fluorscopy, using minimally invasive threaded rods
b. Allows for 2 pairs of wires to be placed as little as 6 mm apart
c. Fracture reduction is possible on all planes
d. Allows for lengthening, treatment of non-unions, soft tissue correction, and
any technique for compression or distraction

Indications for use:

 Fractures
 Aseptic and infected non-unions
 Corrective osteotomies
 Lengthening
 Replantation

PROXIMAL FRACTURES OF THE FIRST METACARPAL


ASEPTIC AND INFECTED NON-UNIONS
CORRECTIVE OSTEOTOMIES
LENGTHENING
Chapter 17: Nail Disorders &
Surgery
Nail Entities Nail Anatomy
Surgical Nail Procedures
NAIL DISORDERS AND SURGERY
Nails are excellent indicators of systemic disease and may provide invaluable
diagnostic information. The nails are equally sensitive to environmental and
physical stimuli and may provide vital clues that indicate toxic exposure and
traumatic insult.
Owing to the great cosmetic value of the nail, any physical derangement to the
structure can bring the patient to your office. You should check for the following
nail presentations: discoloration, anonychia, brittleness, hypertrophy,
koilonychia, onycholysis, pitting, pterygium, onychomadesis, splitting,
striations, nail thinning, ridging, change in nail consistency, change in nail
configuration, and nail clubbing.

Nail Entities
1. Anonychia: Is the complete absence of one or usually more than one nail.
This condition is a rare congenital anomaly.
i. Caused by ischemia, frostbite, toxic and infectious states, Raynaud's disease,
Darier's disease, lichen planus, subungual neoplasm, fungal infections,
psoriasis, and injuries.

2. Paronychia: Is an infection usually accompanying onychocryptosis.


Staphlococcus is most common organism. Candida is a common pathogen
also. Cultures for bacteria and for fungi are indicated.

3. Beau's Lines: Transverse ridges in the nail plate, 0.1-0.5 mm wide by


0.1 mm deep appear in the lunula and progress forward. Beau's lines are
a sudden arrest of function of the nail matrix. The width of the Beau's lines are
directly related to the duration of the illness.
i. Caused by typhus, diptheria, syphilis, malaria, leprosy, influenza, scarlet fever,
vascular diseases, diabetes mellitus, hyperthyroidism, ACTH therapy, psroriasis,
diffuse alopecia, and exfoliative dermatitis.

4. Clubbed Digits: A characteristic bulbous deformation of the terminal


phalanges topped by marked convexity of nails, with the nail becoming hard,
lustrous and thick. As the disease progresses, advanced clubbing and
angulation of the nail results in Lovibond's angle (> 160 angulation of the
nail).
i. May be subclassified as hippocratic nails, pulmonary hypertrophic
osteoarthropathy, pachydermoperiostosis, or other diseases
ii. Can be caused by: congenital heart defects, cirrhosis of the liver, chronic
diarrhea, SBE, neoplasms of the lung and emphysema. 80% of clubbing is seen
in conjunction with respiratory ailments, but it may be seen in many
nonpulmonary acquired and hereditary conditions.

5. Darier-White Disease: Similar to alopecia areata. This condition will cause


changes such as red and white longitudinal streaks that run the length of the
nail.

6. Eczematous Conditions: Many types of eczematous dermatitis such as atopic


and contact dermatitis frequently affect the nail folds causing damage to the
nail matrix. As a result, many changes 'to the nail plate occur, such as Beau's
lines, onychorrhexis, subungual hyperkeratosis, and onycholysis. Nail color
may change to yellow, green, grey, or black.

7. Onychomycosis: Fungal infections are common, usually caused by


dermatophytes. In the cases of monilia and yeast it is reported that onycholysis
will develop without destruction of the nail plate. Other diseases mimic this
condition, such as psoriasis.
i. Distal and lateral onychomycosis: Most common type, caused by T. rubrum,
T. mentogrophytes, and E. Floccosum.
ii. Proximal subungual onychomycosis: caused by T. rubrum, T. megnini, T.
schoenleinii, and T. tonsurans.
iii. White superficial onychomycosis: usually T. mentagrophytes and mold
fungi
iv. Diagnosis by KOH prep and growth on Sabouraud's agar

8. Glomus Tumor: Neoplasia of the arteriovenous shunts (Suquet-Hoyer canals)


in the nail beds that constitutes the glomus bodies leads to a purplish tumor
that causes extensive pain. The nail bed will appear as a blue-red distortion
and the nail plate can have variable distortions. The lesion may be tiny and
colorless.

9. Green Nails: Usually caused by a local Pseudomonas infection or Candida


albicans. Pseudomonas produces a green pigment called pyocyanin.

10. Keratoacanthoma: This may develop in the nail beds with serious
consequences to the nail and subungual structures. The lesion appears
suddenly, ulcerated, and both clinically and histologically resembles
squamous carcinoma. Underlying bone may be involved.

11. Koilonychia: Means spoon shaped nails and describes a characteristic


deformity in the form of a concave shape.
i. Occurs with hypochromic anemia, Plummer-Vinson syndrome,
thyrotoxicosis, Raynaud's disease, gastrointestinal disorders, nail bed
tumors, and syphilis.

12. Leukonychia: A transverse spot or striation porcelain white in color of the


nail plate. There are 5 types:
i. Leukonychia punctuate (pinhead spot): associated with psoriasis, dyshidrosis,
typhus, scarlet fever, measles, arsenic and lead poisoning and
microtraumas.
ii. Leukonychia striata (transverse): looks like Mee's lines and found with
scleroderma.
iii. Leukonychia totalis (entire nail): Seen in leprosy, hypochromic
anemia, arsenical poisoning, cachexia, and Bart-Pumphrey Syndrome.
iv. Partial Leukonychia (nail is all-white but has a distal pink band) As found in
Hodgkin's disease, chilblains, metastatic Ca, and leprosy.
v. Longitudinal Leukonychia (longitudinal white stripes): Found in Darier's
disease and arsenic poisoning.

13. Lichen Planus: Will result in atrophy of the nail plate and pterygium
formation, which is considered pathognomonic for the disease.

14. Malignant Melanoma: Acral lentiginous melanoma is the most dreadful of


malignancies found under or around the nails. When the melanotic whitlow
is present, there is an elevation of the nail that can be mistaken for glomus
tumor, subungual exostosis, or profuse granulation (see chapter 13, Dermatology:
Malignant tumors)

15. Mee's Lines: Is an eponym for horizontal striations that appear in the nails
as a consequence of arsenic and thallium poisoning.

16. Onychauxis: This is the thickened, elongated, raised irregular nail. The
color can be changed from white to a mixture of green, yellow, brown, or
black, all of which may obliterate the lunula.
i. Can be caused by trauma, fungal infection, nutritional disturbances, circulatory
disorders, acute rheumatic fever, secondary syphilis, TB, psoriasis, ichthyosis,
eczema, hyperuricemia, RA, venous stasis, hyperglycemia, hyperthyroidism,
leprosy, peripheral neuritis, tabes dorsalis, and scleroderma.

17. Onychogryphosis: Is an exaggeration of onychauctic condition.

18. Onychoheterotopia: Means ectopic or abnormal position of the nails.

19. Onychomalacia: Refers to softness of the nails and is synonymous with


hapalonychia.

20. Onycholysis: Detachment of the nail bed from the overlying plate creates a
space between nail plate and nail bed in which keratin forms. This occurs in
numerous conditions:
i. Due to trauma, contact dermatitis due to nail polish, cement and topical drugs,
fungal infection, Pseudomonas infection, psoriasis, hyperthyroidism,
pregnancy, iron deficiency anemia, lichen planus, and many others.

21. Onychomadesis: The shedding of nails from the proximal to the distal free
edge. The pathology in this condition involves lesions to the matrix and the
hyponychium.
i. Due to epilepsy, peripheral neuritis, peripheral thrombosis, embolic
occlusions, diabetes mellitus, syphilis, hemiplegia, syringomyelia, and many
others.

22. Onychorrhexis: Means the breakage of nails, the nail becoming thin
and fragile with exaggerated dermal epidermal subungual sulci.
i. Due to hypochromic anemia, hypocalcemia, lichen planus, RA, radiation,
arsenic and lead poisoning, leprosy, and syphilis.
23. Onychophagia: Means nail biting.

24. Onychoschizia: The nail becomes very fragile, and as a result, distal
splitting of the nail occurs. There are two or more laminations overlying
each other. The nail appears multilayered.
i. Due to acromegaly, chronic eczema, metabolic acidosis, peripheral nerve lesions,
trauma, infectious diseases, hyperthyroidism, and hypochromic anemia.

25. Pterygium Ungium: Overgrowth of the eponychium so that the lunula


and much more of the nail plate is covered by a wing (pterygium) of soft
tissue.
i. Due to scleroderma, Raynaud's disease, vasospastic disorders, leprosy,
dermatomyositis and peripheral neuritis.

26. Squamous Cell Carcinoma: This occurs under nail plates usually as a
result of a progression from squamous cell Ca in situ (Bowen's disease). It
must be differentiated from keratoacanthoma. (see Dermatology section)

27. Splinter Hemorrhages: These are caused by capillary fragility in the


longitudinal vessels of the nail bed.
i. Due to: scurvy, SBE, CHF, vasculitis, tetanus, hemophilia, and
hypoparathyroidism

28. Periungual Fibroma: May be either acquired or congenital.


Congenital fibromas are often associated with tuberous sclerosis, a disorder
characterized by periungual fibromas, mental retardation, seizures, and
adenoma sebaceum. (see Dermatology section)

NOTE* Changes in nail color are markers for disease of blood, endocrine system,
cardiovascular and peripheral vascular systems, and toxicity of drugs,
chemicals or metals:
a. Yellow nails: may suggest Addison's disease or diabetes mellitus b. Blue
nails: may be a sign of cyanosis or heavy metal poisoning c Green nails:
Pseudomonas infection
d. Black-brown discoloration: may be normal pigmentation, Peutz Jaegher disease
(also have brown macules on the palms and soles), Addison's disease,
junctional nevus, or melanoma
e. White nails: may be hereditary, a sign of anemia, fungal infection, BartPumphrey
Syndrome
f. White striae: arsenic poisoning or drug toxicity
g. White spots: injury to the nail plate, psoriasis
h. Alternating white lines with pink nail (Muercke's lines): may suggest
anemia, chronic liver disease, nephrotic syndrome, or Darier- White disease
i. Reddish nails (or combination of red/white/brown): may be signs of localized
neoplastic disease
Nail Anatomy
1. The matrix is a stratified epithelium that produces hard keratin. Proximal
matrix forms the superior nail and the distal matrix forms the lower nail.
2. Hyponychium is an epithelial layer of the nail bed and really does not
produce much nail plate keratin. It does help, however, in subungual debris
production.
3. Predominantly nail develops from the matrix, but the proximal nail fold,
lateral grooves, bed, and hyponychium can all be onychogenic.
4. The nail plate can be separated into 3 zones with predominantly different
beginnings. The uppermost layer is generated by the proximal nail fold, the
plate by the matrix, and the deepest section of the nail plate is contributed
to by the nail folds and bed.
5. The nail matrix is found on the proximal slope of the distal phalanx and
extends medial and lateral to the phalanx. The germinal matrix extends
laterally as far as the width of the nail plate just distal to the lunula and with
the same curvature. Proximally it extends to 1 1 /2 to 2 1 /2 times the length
of the visible lunula.
6. The lunula is a white semi-lunar area corresponding to the anterior matrix.
7. The nail bed consists of the hyponychium and corium over the matrix.

Surgical Nail Procedures


Nail prob lems t h at dicta te sur gica l interve ntion are abscess ,
persiste nt pai n, regrowth or spicule development. Chronic nail infections
may lead to OM due to the close proximity of the nail and the dorsum of the
proximal phalanx.
1. Phenol and Alcohol Chemical Matrixectomy:
i. Should always do preoperative CBC/Diff and FBS.
ii. This procedure has been done in well controlled diabetics, but should be
avoided in patients with PVD.
iii. Phenol and lidocaine are teratogenic and should not be used in the
pregnant patient.

2. Sodium Hydroxide Matrixectomy:


i. Acetic acid used to neutralize
ii. Apply 10% NaOH till capillary coagulation
iii. Same criteria apply as for Phenol-Alcohol procedure

NOTE* The following are cold steel procedures. The indications are: chronically
recurring ingrown toenail, failed Phenol-Alcohol procedure, chronic
hypertrophic ungual labia, subungual exostosis in combination with
dystrophic, hypertrophic or mycotic nail, patient in whom chemical or thermal
bum is contraindicated (diabetes?), excision and biopsy of nail/nail
bed/matrix tissue, in conjunction with bunion procedures, patients who will
not comply with postoperative regimen of Phenol-Alcohol procedures,
cosmetic reconstruction of deformity, and surgeon's preference.
3. Frost Procedure:
i. For ingrown toenail with "proud flesh" and chronically hypertrophic ungual
labia
ii. Excise a piece of matrix through an inverse "L" shaped incision
iii. Nail and matrix along the problem labia is excised with the second incision
iv. Closure is with suture after curettement of the phalanx
v. Due to the tissue necrosis that occurs, this procedure requires both
primary and secondary wound healing, therefore of little advantage over non-
cold steel procedures

Figure 1: Illustration of Frost Procedure


Clinics in Podiatric Medicine and Surgery: Nail Disorders, Saunders, April 1989, Volume 6:2, with permission

4. Zadek Procedure: Based on the premise that excision of the nail bed was
not necessary in preventing regrowth of the nail, therefore Zadik directed his
attention only to the nail matrix.
i. The incisions utilized are perhaps this procedures greatest contribution
ii. Utilized more for total nail excisions in the lesser digits
iii. Not recommended for the difficult onychauxic nail (where nail bed removal
may also he necessary)

Figure 2: Illustration of Zadek procedure


Clinics In Podiatric Medicine and Surgery: Nail Disorders, Saunders, April 1989, Volume 6:2, with permission

5. Kaplan Procedure: The most well documented toenail surgery in the


literature. Unlike the Zadek, the Kaplan stressed the need to remove both the
nail matrix and nail bed.
i. This procedure is indicated for correction of severely onychauxic,
mycotic, or chronically deformed or ingrown toenails. It is also the procedure
of choice when the former are combined with painful or deforming subungual
exostosis or osteochondroma.
ii. The original procedure describes an "H" incision carried out at two
tissue depths. The proximal half of the "H" includes only the nail matrix and
stresses the preservation of periosteal tissue. The distal half of the "H" is carried
directly down to the distal phalanx and involves stripping of the nail bed and
thus exposing the distal phalanx.
iii. A modified Kaplan has been described, whereby, the "H" incision is replaced by the
Zadek-type incision. This allows for maximum exposure of the nail matrix
area.

Figure 3: Kaplan Procedure


Clinics In Podiatric Medicine and Surgery: Nail Disorders, Saunders, April 1989, Volume 6:2, with permission

Figure 4: Wound closure for Winograd Procedure


Clinics In Podlatric Medicine and Surgery: Nail Disorders, Saunders, April 1989, Volume 6:2, with permission

6. Winograd procedure: Dr. Winograd was the first to describe a linear


incision into the posterior nail fold with excision and curettage of the nail
matrix tissue. The incision he discribed is the same used today.
i. Contrary to popular belief, the most important component of wedge
resection is wound closure and not whether the entire matrix is encompassed
within the width of the tissue ellipse.
ii. When a modified Winograd is considered, 3 preoperative components must
be considered:
 must be sure that an adequate amount of soft tissue exists for good wound
closure.
 the length of the ellipse must be at least 3 times the width
 wound closure is by primary intention (preferably)

7. Suppan Procedure:
i. Frees the eponychial fold and removes the nail
ii. Visualizes the nail matrix proximally
iii. Cut the lateral borders and the anterior borders
iv. Hold tag and remove the proximal attachment
v. Curette down to bone into the lateral cul de sac

8. Complications From Nail Surgery:


a. Recurrence
b. Excessive drainage
c. Excessive bleeding
d. Poor technique and excessive tissue destruction e. Infection
f. Exuberant granulation tissue
g. Insufficient amount removed
h. Soft tissue migration upward to the dorsum of the toe
Chapter 18: Neurology
Neurologic Pathways Presenting Features Assessment of
Clinical Problems
Tarsal Tunnel Syndrome
Classification of Nerve Injuries
Neuromuscular Causes of Cavus Foot
Types of Nerve Surgery
NEUROLOGY
It is not uncommon for a neurologic illness to affect the feet initially, thereby
making patients seek podiatric consultations first. This chapter will help you
understand disorders affecting the central and peripheral nervous systems with
emphasis on the lower extremity.
Neurological Pathways
1. Cerebral Cortex: Divided into frontal, parietal, temporal, and occipital
lobes
2. Descending pathways: Divided into corticospinal (pyramidal) and
extrapyramidal
3. Thalamus and hypothalamus
4. Cerebellum
5. Brain Stem
6. Spinal Cord
7. Nerve Roots
8. Peripheral Nerves
9. Neuromuscular Junction and Muscle
Presenting Features
1. Presenting Problems: Can include pain, numbness, tingling, weakness,
unsteadiness, and involuntary movements. Neurologic disorders should always
be suspected in patients with:
a. A foot deformity (i.e. pes cavus)
b. Leg or foot weakness
c. Difficulty in walking
d. Pain or paresthesias in the legs

2. The Physical Examination:


i History: with children get a developmental history which includes questions
such as:
When did the child get head and neck control? (normal at 2 months) When was
the child sitting independently? (normal at 6 months) When did the child start
to walk? (normal by 9-15 months) Low or normal birth weight?
Any problems at birth?
ii. Motor System (Muscle tone/strength): Specific examination of muscles
includes observation, palpation and strength testing. For muscle tone look for
atrophy (loss of muscle bulk), fasciculations (brief, fine irregular twitches), and
myotonia (decreased relaxation of muscle following a sustained contraction).
Sometimes muscle hypertrophy is abnormal To test muscle strength, the
specific muscle must be tested against resistance. It is graded from zero (no
movement), 1 (trace movement), 2 (movement with the aid of resistance), 3
(movement against gravity), 4 (movement against resistance supplied by the
examiner), and 5 (normal strength).
Abnormalities of Movement
Fasciculations- visable twitching movements of muscle bundles.
Tremors- involuntary rhythmic tremulous movements.
Tics- repetitive twitching of muscles often in the face and upper trunk.
Chorea- involuntary movements of the face and extremities that are rapid,
jerky, and unpredictable. They occur at rest or during any purposeful
movement.
Athetosis- involuntary movements of the face, extremities that are slower,
more twisting and writhing than chorea. They occur at rest or during any
purposeful movement.
Myoclonus- involuntary, sudden and rapid unpredictable jerks; faster than
chorea.
Asterixis- involuntary and brief loss of muscle tone in the outstretched
fingers and hands, resulting in nonrhythmic flapping of the fingers or the
entire hand. (chronic liver disease)
Tardive dyskinesia- choreiform movements affecting primarily the tongue,
lips, and face, producing repetitive, involuntary grimacing, protrusion of the
tongue, opening and closing of the mouth, and deviations of the jaw.

iii. Stance and gait: There are specific gait patterns associated with specific
diseases.
Abnormal Gait Patterns
Spastic gait- Manifested by internal rotation and adduction of the entire
limb, with hip/knee/ankle in marked flexion. Seen with cerebral palsy,
familial spastic diplegia, paraplegia, and hemiplegia.
Dyskinetic gait- A constant movement abnormality with a high degree of
variability from patient to patient and gait cycle to gait cycle. It is
characterized by motion involving considerable effort, often with deliberate,
almost concentrated step. Seen with cerebral palsy. Huntington's chorea, and
dystonia musculorum deformans.
Ataxic gait- Characterized by a marked instability during single limb stance
with an alternating wide/narrow base during double support. During swing
phase the limb will swing widely and cross the midline. Seen with multiple
sclerosis, tabes dorsalis, diabetic polyneuropathy, Friedreich's ataxia.
Waddling gait- A labored walking, exhibiting difficulty with balance,
proximal pelvic instability, leading to a lumbar lordosis. May see an
associated equinovarus foot type. Seen with Duchenne muscular dystrophy,
limb-girdle muscular dystrophy, Beckers muscular dystrophy, spinal
muscular atrophy, and congenital dislocated hips.
Steppage gait- Gait exhibits a swing phase drop foot. Seen with Charcot-
Marie-Tooth, polio, Guillain-Barre syndrome, CVA, paralytic drop foot, and
fascioscapulohumeral dystrophy.
Vaulting gait- Gait changes include a high cadence, increased lateral trunk
movement, scissoring and instability from step to step, suggesting a loss of
balance. Seen with myotonic dystrophy.
Equinus gait- Gait exhibits a swing phase ankle plantarflexion with no heel
contact. Seen with cerebral palsy, Charcot-Marie-Tooth, muscular dystrophy,
spinal muscular atrophy, schizophrenia, osseous block of the ankle, and
habitual toe walking.
Festinating gain- Gait changes include loss of reciprocal arm swing,
decreased velocity, shuffling, decreased stride length, and increased cadence.
Seen with Parkinson's disease.
Trendelenberg gait- Stance phase of each step leads to a contralateral tilt of
the pelvis with a deviation of the spine to the affected side. Seen with
dislocated hip or weakness of gluteus medius.

iv. Deep tendon reflexes (DTR's): Assesses the afferent nerve, the synaptic
connections within the spinal cord, the motor nerves, and the descending motor
pathways. It is important to note right and left asymmetries and the degree of
activity, measured from zero to five (zero= no activity, 1= hypotonia, 2= normal,
3= exaggerated response, 4= multiple contractions, 5= sustained contractions).
Hyperreflexia indicates a lesion in the corticospinal thalamic tracts.
Hyporeflexia indicates a lesion in the lower motor neurons or an intrinsic
muscle weakness. The DTR's tested are the:
 Biceps/brachioradialis (C5 and C6)
 Triceps (C7 and C8)
 Patellar (L2, L3, and L4)
 Achilles (S1).
v. The plantar response, or the Babinski reflex is the response to striking the
sole (extention of the great toe), and when present represents an upper motor
neuron lesion of the pyramidal tract.
vi. Rossolimo Sign (reflex): involves flicking the plantar aspects of the toes,
distally. Flexion response occurs in pyramidal tract disease. This same
response may accompany a Babinski reflex.
vi. Rhomberg's sign: assesses balance (cerebellar function) and if abnormal the
patient will not be able to stand with feet together and eyes closed.
Proprioceptive control is lost.
vii. Sensory system testing:
*Pain and temperature (lateral spinothalamic tract) testing is done with a pin
and an ice cube placed on various dermatomes (See Figure 1).
*Vibratory testing (posterior columns) is done with a tuning fork (128
cycles/sec) over a bony prominence or joint.
*Joint position (posterior columns) is performed by moving the first m.p. joint
into either extension or flexion with the patient's eyes closed.
*Light touch (anterior spinothalamic tracts) is performed using a nylon filament
over various dermatomes.

Lower Extremity Dermatomes (Figure 1)


L1 and L2 ennervate the anterior thigh. L4 ennervates the lateral thigh, then
crosses the patella to innervate the medial anterior calf and foot (medial side
of the hallux). L5 ennervates the lateral anterior leg and central aspect of the
foot plantarly and dorsally. S1 ennervates the posterior lateral thigh, leg, and
lateral border of the foot plantarly and dorsally. S2 ennervates the posterior
medial thigh and leg , and the medial-posterior portion of the heel
3. Neurologic Diagnostic Procedures:
a. Lumbar Puncture: Tapping of the lumbar subarachnoid space between L3
and L4 provides important information about intercranial pressure and allows a
diagnostic analysis of the CSF (the CSF count is abnormal if more than 5 cells
are present).
b. CT Scan: Rapid noninvasive imaging for the brain, spinal cord and their bony
enclosures.
c. MRI: Provides extraordinary resolution in imaging of the neural structures
without any known risk to the patient. Helpful in identifying brainstem lesions
and other abnormalities. MRI cannot be used to examine patients with
pacemakers, or patients who are pregnant, who have metal prostheses, and
who are dependent on respirators.
d. Electroencephalography (EEG): Voltage vs. time recordings of electrical
currents in the brain. Good for detecting epilepsy and metabolic and structural
encephalopathies.
e. Electromyography (EMG) and Nerve Conduction Velocities: When weakness is
clinically difficult to attribute to either nerve, muscle, or neuromuscular
junction, electrical studies can establish topographically which nerves and
muscles are affected. In EMG, the recording of electrical properties of muscle is
displayed on an oscilloscope during needle insertion. Denervated muscle is
recognized by fibrillations and fasciculations on the screen.
In nerve conduction studies the time for an impulse to travel along the nerve is
termed the conduction velocity. If there is an increase in this conduction
velocity, there is damage to the particular nerve involved.

Assessment of Clinical Problems


Finding the site of the lesion is extremely useful in diagnosing the cause. Once
a possible location(s) have been identified, one can review a checklist of disease
processes, such as traumatic, vascular, infectious, metabolic, immunologic,
neoplastic, and inherited problems that may be responsible.

1. Seizure Disorders: A seizure is a sudden disturbance of cerebral function


due to a paroxysmal neuronal discharge in the brain.
a. Epilepsy: implies a chronic condition of recurring seizures. Subdivided into
two types
i. Generalized: Grand mal and petite mal
ii. Partial
b. Status epilepticus: recurring seizures, one following another without full
recovery from the preceding seizure.
c. Etiology: may be metabolic (hypoglycemia), hypocalcemia, phenylketonuria,
drug toxicity, drug withdrawal, or a focal abnormality of the brain as with
trauma or stroke.
d. Time sequence:
1. Prodrome: time preceding the seizure that the patient does not feel well.
ii. Aura: A symptom or sign signaling the beginning of the seizure (visual, aural
or other sensory change)
iii. Postictal period: The period after the seizure while the patient returns to
normal.
e. Management: During the seizure administer first aid, preventing the patient
from biting his tongue and keeping the airway open. If the seizure does not stop
within 5 minutes, emergency medical care should be
instituted (IV's w/ D5W/ diazepam/phenytoin PRN). Control should be
achieved with as few anticonvulsants as possible.

2. Cerebrovascular Disorders: Cerebral vascular problems usually appear


dramatically with sudden onset and often result in permanent loss of
neurologic function. There are two basic types
i. Intracranial hemorrhage: (types) hypertensive, ruptured aneurysm,
arteriovenous malformations, traumatic, secondary to brain tumor, and
secondary to hematologic disorders
ii. Ischemic stroke: (types) thrombotic and cerebral embolism. The prognosis
after stroke is usually dependent upon the blood vessel involved and its
perfusion territory.

3. Peripheral Neuropathies: In terms of subjective complaints, peripheral


neuropathies are the most common causes of foot and toe dysesthesias
(burning foot syndrome). In addition patients complain of extremity weakness,
muscle atrophy, or both. In addition to motor function, peripheral nerves are
responsible for sensory and autonomic function.
a. There are three general forms:
i. Segmental demyelination: refers to destruction of the myelin segments with
survival of the myelin segments with survival of axons until late in the course of
the illness.
ii. Axonal degeneration: destruction of the axon.
iii. Wallerian degeneration: occurs when the nerve is injured or severed at a
focal point, and the distal segment breaks down and is reabsorbed. b.
Classification of Neuropathies: Remember the mnemonic "Dang Thrapist"
D-Diabetic
A-Alcoholic
N-Nutritional
G-Guillain-Barre

T-Toxic
H-Hereditary
R-Recurrent
A-Amyloidosis
P-Porphyria
I-Infectious
S-Systemic
T-Tumor

i. Diabetic neuropathy: 2/3 of all diabetics show evidence of peripheral nerve


dysfunction (clinically or subclinically), which progresses in the face of poor
diabetic control. Patients can present with a mononeuropathy, polyneuropathy
and can have sensory impairment, or both. At the earliest stage the patient
experiences pain, usually worse at night.
 Symmetric Distal Polyneuropathy: loss of sensation (predominantly) and
motor weakness in a stocking-glove configuration, that occurs bilaterally.
The distal portion of the longest nerves are affected first and the feet affected
before the hands. Later muscular atrophy can occur with intrinsic muscle
wasting.
 Mononeuropathies: both cranial and spinal mononeuropathies can occur
which includes femoral mononeuropathy (characterized by pain, motor and
sensory loss, and absent knee jerk), peroneal mononeuropathy
(characterized by a sudden foot drop), and tarsal/carpal tunnel syndrome.
 Autonomic Neuropathies: disturbances may be seen in the cardiovascular
system, bowel, bladder, and sexual function. Symptoms include, diarrhea,
incontinence, impotence, decreased sweating, and orthostatic hypotension

NOTE* The diagnosis of orthostatic hypotension is made by demonstrating a


decrease of approximately 25 mm Hg in systolic or 10 mm Hg in diastolic blood
pressure after 2 minutes of upright posture without a compensatory increase in
heart rate (treated with fluorocortisone or ephedrine).

 Neuroarthropathy: foot ulcers occur secondary to neuropathy,


microangiopathy, large vessel atherosclerosis, or combinations of these
factors. The Charcot joint seen in the lesser tarsus or tarso-metataral area
has an unknown etiology. It is unfortunately seen after revascularization
procedures of the extremities. The most common bony deformities are the
medial convexity, plantar deformity, dorsal midfoot deformity, and
plantarflexed metatarsals. A differential diagnosis includes tabes dorsalis,
osteomyelitis, leprous neuropathy, and peripheral nerve injuries. Initial
treatment for active arthropathy includes 1-2 weeks of bed rest, nonsteroidal
anti inflammatory drugs (?), the use of crutches, and a nonweight-bearing
cast or splint until the soft tissue swelling and erythema subsides. Gradual
weight-bearing can be resumed once radiographic evidence of an arrest is
visualized. Patients with fractures of the lower 1 /3 of the tibia are prone to
develop ankle arthropathy.

Note* Treatment of diabetic neuropathy: For burning pain and


dysesthesia (combination of Elavil h.s../ Prolixin t.i.d or Valium t.i.d, Axain
topically), lightning-like pain (Tegretol or Dilantin), carpal/tarsal tunnel
syndrome (splints/orthotics), itching (diphenhydramine), and diarrhea
(codeine).

4. Infectious Diseases:
a. Meningitis: Causes symptoms such as fever, neck stiffness, disorders of
consciousness, and seizures. 80-90% of cases of meningitis are caused by one
of three organisms, H. influenzae, N. meningitis, and D. pneumoniae. Diagnosis
is confirmed by lumbar puncture (CSF is cloudy, pressure
elevated, WBC > 1000/mm3, and positive CBS). Treatment is usually penicillin
G for N. meningitis and D. pneumoniae, and ampicillin for H. influenzae.
Positive Brudzinski and Kernig signs clinically.
b. Tuberculosis: Can result in multiple cranial nerve palsies.
c. Neurosyphilis: Occurs in 25% of patients with syphilis. Diagnosis is by FTA-
ABS test. Penicillin is the treatment of choice.
d. Fungal Infections: Cryptococcus neoformans (most common organism) which
can produce a subacute meningitis.
e. Acute Viral Infections: Poliomyelitis, starts as a flu-like illness, followed by
meningitis, and then flaccid paralysis of the limbs and trunk. Herpes zoster
(shingles) has a dermatomal distribution of vesicles and demonstrates
segmental weakness and pain, sometimes for years.
5. Movement Disorders: Are generally extrapyramidal
a. Parkinson's Disease: Characterized by hypokinesia, tremor, rigidity and
disorders of gait and balance. On examination there is a "pill rolling" tremor of
the hands. The drugs used to treat this disease either act by decreasing
cholinergic activity (trihexyphenidyl) or by increasing dopaminergic activity (L-
dopa usually combined with carbidopa).

NOTE* Neuroleptics such as chlorpromazine and haloperidol may produce a


Parkinson-like syndrome. The syndrome stops with withdrawal of the drug.

b. Chorea: A variety of neurologic diseases are associated with chorea such as,
childhood rheumatic fever (Sydenham's chorea), systemic disorders
hyperthyroidism, hypoparathyroidism, and SLE), drugs (oral contraceptives),
pregnancy, and hereditary disorders (Huntington's chorea).
i. Huntington's chorea: an inherited disease than begins to manifest itself
between the ages of 30-40 with. progression to death within 20 years.
Manifested by the combination of chorea and dementia.
c. Dystonia: Is a movement disorder likely to be seen first by podiatrists,
presenting with a slow sustained contraction of muscle groups, resulting in
abnormal postures of the trunk and extremities, and of more rapid, twisting
movements.
i. Dystonia musculorum deformans: a primary hereditary disorder, presents
usually with an equinovarus foot.
d. Tremor: Essential tremor is a benign disorder that frequently accompanies
other neurologic conditions.

6. Tumors: Symptoms depend upon the location of the lesion.

7. Demyelinating and Degenerative Diseases:


a. Multiple Sclerosis: characterized by a remitting and exacerbating course of
multiple neurologic symptoms, including blindness, diplopia, ataxia,
nystagmus, spastic weakness, dysesthesia, and difficulty with bladder and
bowel function. Electroimmunodiffusion reveals migration of gamma G
immunoglobulin on discrete bands. There is no specific treatment.
Corticosteroids can shorten the span of an acute attack. Speech is often
explosive.

b. Amyotrophic Lateral Sclerosis: a group of diseases characterized by


progressive weakness, atrophy, spasticity, hyperreflexia, and fasciculations
occurring in a widespread distribution. There is a wide variation in
symptoms, with some patients showing more atrophy and some showing more
spasticity. Some patients have only cranial nerve involvement. There is no
specific therapy. There is an inherited pattern.

c. Friedreich's Ataxia: An autosomal-recessive disorder with cerebellar


degeneration. Characteristics include gait changes, decreased position and
vibratory sense in the legs, absent DTR's, nystagmus, kyphoscoliosis, pes
cavus, and hypertrophic cardiomyopathy.

NOTE* Pes cavus is the major foot type associated with neurologic illness.
Patients with pes cavus can be divided into four groups:
1. Patients with hereditofamilial disease: Friedreich's ataxia and Charcot-
MarieTooth disease.
2. Those who have isolated pes cavus but whose family members have one of
the aforementioned hereditofamilial neurologic diseases.
3. Those with isolated, or idiopathic, familial pes cavus with no family history
of hereditofamilial neurologic disease.
4. Those with familial pes cavus and lymphedema (very rare syndrome).

8. Cerebellar Disorders: These patients present with an imbalance and a


difficulty with walking.
a. Friedreich's ataxia
b. Dandy-Walker deformity
c. Arnold-Chiari malformation
d. Infections

9. Disorders of Muscles and Nerves: Myopathies are disorders of nerves


characterized by progressive weakness. Laboratory tests for diagnosis include
measurement of serum muscle enzymes, creatine phosphokinase (CPK),
electromyography, nerve conduction studies, and muscle biopsy. a. Muscular
Dystrophies:
i. Duchenne's muscular dystrophy: An X-linked recessive disorder with a
frequent mutation rate, which appears with a slowly progressive proximal
weakness beginning in the lower extremities and later involving the upper
extremities. It is often associated with early toe walking and causes a waddling
gait. The gait changes include decreasing cadences, increasing anterior pelvic
tilt, increased hip flexion in swing, decreasing ankle dorsiflexion, and increased
shoulder sway. These patients have a progressive muscle atrophy and
weakness (there is often pseudohypertrophy of the calf muscles). The serum
CPK is markedly elevated in these patients early in the disease and a muscle
biopsy establishes a definitive diagnosis. Pathognomonic for this disease is a
positive Gower's sign, where the affected child rises from a sitting position on
the floor by climbing on his own legs.
ii. Becker's muscular dystrophy: Is a more benign form of X-linked muscular
dystrophy clinically similar to Duchenne's MD, and is manifested by
progressive proximal limb weakness.
iii. Facioscapulohumeral dystrophy (Landouzy-Dejerine): A variant of muscular
dystrophy with a swing phase drop foot and compensatory
increase in hip and knee flexion. This disease gives a "Popeye the sailor"
forearm appearance.
iv. Limb-girdle muscular dystrophy: Mostly autosomal recessive disorders
characterized by progressive proximal weakness.
v. Myotonic muscular dystrophy: An autosomal dominant disorder with
symptoms that involve a combination of weakness and myotonia (myotonia is a
delayed relaxation of muscle following contraction and is associated with
abnormal EMG discharges).
vi. Spinal muscular atrophy: An autosomal recessive condition with progressive
degeneration of the anterior horn cells. There are three types-
 Group I: Werdnig-Hoffman disease (most severe, diagnosed in infancy
 Group II: intermediate form
 Group III: Kugelberg-Welander disease (mildest form). Also called Wohlfart-
Kugelberg-Welender disease
b. Acquired Myopathies: The following can cause a slow progressive and
prominent muscle weakness:
i. Endocrine disorders: Hypo/hyperthyroidism
ii. Drugs: Corticosteroids, antibiotics, and alcohol.
iii. Collagen vascular diseases: Polymyositis, dermatomyositis, scleroderma, RA,
and SLE.

10. Perinatal or Gestational CNS Damage Disorders:


a. Cerebral Palsy: A nonprogressive brain lesion usually due to a perinatal
insult (hypoxia) resulting in a pyramidal tract lesion. There are various types
i. Spastic
ii. Athetotic
iii. Ataxic
iv. Rigid
b. Familial Spastic Diplegia: Has a strong family history of lower limb spasticity,
no perinatal insult, and is progressive.

Note* The type of spasticity is based on the anatomical areas involved:


Diplegia- all four extremities, primarily lowers
Paraplegia- lower extremities only
Quadriplegia- all four limbs equally involved
Hemiplegia- one side of the body (upper and lower extremities)

11. Disorders of the Spinal Cord and Nerve Roots:


a. Spinal Cord Disorders: Localizing the level of a focal lesion of the spinal cord
is easier if one remembers the Brown-Sequard syndrome.

NOTE* The Brown Sequard syndrome occurs after hemisection of the spinal
cord, which results in an ipsilateral spastic paralysis and loss of postural
sense, and on the opposite side a loss of pain and temperature sensations

i. Spinal cord dysfunction may be acute or chronic


 Acute due to: trauma, compression, inflammation, infarction, vascular
malformation, and hemorrhage.
 Chronic due to: Syringomyelia
b. Radiculopathies:
i. A fifth lumbar nerve radiculopathy in addition to back pain, often causes
radiating pain down the lateral aspect of the leg with numbness or paresthesias
of the lateral calf and the dorsum of the foot.
ii. A first sacral nerve radiculopathy characteristically causes radiating pain
down the posterior leg with sensory changes on the lateral and plantar aspects
of the foot. (absent achilles reflex).

12. Pain Syndromes:


a. Reflex Sympathetic Dystrophy Syndrome: Originally termed "causalgia".
Other variants include Sudek's atrophy and post-traumatic reflex dystrophy.
This syndrome is characterized by disproportionate pain in intensity, duration,
location, often from minor or unapparent trauma to an extremity. Clinical
diagnosis is difficult due to the vague subjective data and subtle objective signs
and symptoms. Early diagnosis is important because early treatment gives the
best results. The sympathetic nervous system is always involved and is
overactive. Symptoms occur distal to the trauma site.
i. Manifestations: (signs and symptoms)
 Pain- most prominent characteristic feature
Quality of pain: burning, aching or throbbing
Severity of pain: mild to excruciating, usually continuous Paroxysmal
aggravations: emotional stress, movement, and touch Localized first then
spreads
Not limited to a dermatome or peripheral nerve distribution

 Vasomotor disturbance
Vasodilation: warm skin, dry skin, and hypohidrosis
Vasoconstriction: cyanosis, cool skin, edema of the part, and hyperhidrosis

 Delayed return to function


 Trophic changes- are late changes (atrophy, and osteoporosis)
Involves the skin, appendages, muscle, bone and joints (RSDS arthropathy)

NOTE* The striking feature is that while all signs and symptoms are usually
resent, a patient often manifests one out of proportion to all the others.

i. There are three grades based upon mode of onset, Intensity, and
preponderance of symptoms
 Grade 1 (SEVERE): rapid onset, severe burning/knifelike pain, severe
vasomotor disturbance, no mobility, atrophy early.
 Grade 2 (MODERATE): slow onset, dull/throbbing diffuse pain, aggravated
by walking (and relieved with rest and immobilization), edema, atrophy, and
osteoporosis.
 Grade 3 (MILD): most common type, the border zone between normal
response and exaggerated response so is often overlooked, usually seen after
surgical procedures
ii. There are three stages of the disease divided as per the time frame -
 Stage 1 (days to weeks): Characterized by
Pain
Hyperesthesia
Hyperalgesia
Localized edema
Muscle spasm and tenderness
Vasomotor disease
No x-ray changes
Trophic changes of hair, nails and skin begin
In mild cases (GRADE 3) this stage lasts a few weeks and then subsides
spontaneously
In severe cases (GRADE 1) symptoms become progressively worse

 Stage 2 (3-6 months):


Gradual decrease of pain
Spread of edema, soft to brawny
Hair scant, nails brittle/cracked and heavily grooved Muscle wasting
X-rays reveal spotty osteoporosis (early), and diffuse osteoporosis (late)

 Stage 3 (greater than 6 months):


Marked trophic changes which eventually become irreversible
Skin is smooth/glassy/drawn/pale or cyanotic with a loss of subcutaneous fat
Nail changes
Digits are thin and pointed
Muscle atrophy, especially interossei
Limited ROM of joints
Tendon contractions
Subluxations
Bone atrophy is diffuse and marked

NOTE* The goal is to restore functional and anatomical integrity ASAP and
break the sympathetic response.

NOTE* According to Van Wyngarden and Bleyart in the Journal of Foot


Surgery, Volume 31- Number 1, their diagnostic test of choice is a sympathetic
and sensory epidural nerve block. If there is a response, then their treatment of
choice is frequent sympathetic blocks (bupivacaine+methylprednisolone), in
combination with physical therapy, and with oral clonazepam throughout the
treatment period.

iii. Treatment:
 Neurology consult
 Psychiatric consult
 Anesthesiology consult
 Sympathectomy (GRADE 1) -Local blocks (GRADE 2 and 3)
 Physical therapy
 Systemic steroids (?)
 Beta blockers (?)
 TENS (?)
 Axain + topical lidocaine (?)
 Procardia (?)
 Analgesics (?)
 Medication to reduce patients' stress

Innervation of the Lower Extremity


Lower extremity innervation is supplied by branches of the sciatic nerve.
1. The sciatic nerve is the interconnection of spinal nerves from L1, L2, L3, L4,
L5, S1, S2, and S3 (S2, S3, and S4 make up the pudendal nerve which is
localized to the pelvis), passes through the greater sciatic notch, between the
greater trochanter of the femur and ischial tuberosity, and rests on the
posterior surface of the adductor magnus. In the lower third of the thigh, the
sciatic nerve splits into the tibial nerve and common peroneal nerve.
2. The common peroneal splits into the deep peroneal, superficial peroneal
and gives off a branch called the lateral sural cutaneous in the popliteal fossa.
3. The lateral sural cutaneous meets with a branch of the tibial nerve (medial
sural cutaneous) to form the sural nerve.
4. The deep peroneal nerve (anterior tibial) descends with the vessels anterior
to the ankle joint, where it divides into medial and lateral branches.
5. The medial branch of the deep peroneal nerve follows the course of the
dorsalis pedis artery and stays lateral to it. This nerve splits in the first
interspace where it supplies the adjacent sides of toes 1 and 2.
6. The lateral branch of the deep peroneal nerve passes across the lateral
tarsal area where it supplies the extensor digitorum brevis, then splits into
three interosseous branches that supply the 2nd, 3rd, and 4th interosseous
muscle.
7. The superficial peroneal nerve supplies the peroneal muscles, comes down
to the ankle lying between the peroneii and the EDL. It divides into the medial
and intermediate (lateral branch) dorsal cutaneous nerves.
8. The medial dorsal cutaneous nerve divides in front of the ankle into two
dorsal digital nerves, the medial and lateral dorsal digital nerves. The medial
dorsal digital nerve supplies the medial side of the hallux and the lateral
dorsal digital branch supplies the adjacent sides of the 2nd and 3rd toes.
9. The intermediate dorsal cutaneous nerve, the smaller branch of the
superficial nerve, divides into two dorsal digital branches over the dorsolateral
aspect of the foot. The more medial branch supplies the adjacent sides of the
3rd and 4th toes. The lateral branch joins with the terminal branch of the sural
nerve to form the lateral dorsal cutaneous nerve.
10. The tibial nerve descends at the back of the thigh to the popliteal fossa,
where it passes with the popliteal artery beneath the soleus muscle,
and descends to the back of the leg with the posterior tibial vessels. It is located
between the FDL medially and the FHL laterally. It enters the lacinate ligament
(3rd compartment) and divides into medial and lateral plantar nerves to
innervate the sole of the foot.
11. The medial plantar nerve is the larger branch and its branches. It gives off
a cutaneous branch to the medial side of the hallux, adjacent sides of the
hallux and 2nd toe, the adjacent sides of the 2nd and 3rd toes, and the
adjacent sides of the 3rd and 4th toes. The 3rd and 4th common digital nerves
communicate in the third interspace and is the site for Morton's neuromas. The
muscular attachments are as per Fig. 4. 12. The lateral plantar nerve supplies
the medial and lateral side of the 5th toe and the lateral side of the 4th toe. The
muscular attachments are as per Fig. 4.
Tarsal Tunnel Syndrome
Is an entrapment or compression neuropathy of the posterior tibial nerve or one
of its three branches, the medial and lateral plantar nerves and/or medial
calcaneal nerve.
1. Anatomy: Nerve entrapment occurs either in the porta pedis or lacinate
ligament
a. The flexor retinaculum (lacinate ligament) extends from the medial
malleolus to the medial process of the calcaneal tuberosity and the plantar
aponeurosis. The deep fibrous septa form four compartments, and converts
bony grooves into canals from anterior-medial to posterior lateral: #1 contains
tibialis posterior tendon (most superficial), #2 FDL tendon, #3 posterior tibial
nerve artery and vein, and #4 FHL tendon. These compartments are unyielding
spaces.
b. The porta pedis is a canal created by the abductor hallucis muscle belly
through which the medial and lateral plantar nerves pass. c. Division of the
posterior tibial nerve into its 3 terminal branches may occur proximal to the
lacinate ligament, which is most common; within the lacinate ligament, as
described in most texts; or distal to the lacinate ligament, which is rare.
d. The medial calcaneal nerve is entirely sensory, and innervates the medial
and plantar aspect of the heel. It may arise from either the posterior tibial or
lateral plantar nerve.
e. The medial plantar nerve gives sensory innervation to the plantar aspect of
the hallux, second and third toes, medial half of the fourth toe, and the medial
half of the plantar aspect of the foot. It gives motor innervation to the abductor
hallucis, flexor digitorum brevis, flexor hallucis brevis, and the first lumbrical.
f. The lateral plantar nerve gives sensory innervation to the plantar lateral half
of the fourth toe, plantar aspect of the fifth toe, and plantar lateral aspect of the
foot. Initially it sends motor fibers to the quadratus plantae and abductor digiti
quinti before dividing in a superficial and deep branch. Superficial branch
supplies motor innervation to the flexor digiti quinti brevis and the dorsal and
plantar interossei of the fourth intermetatarsal space. The deep branch supplies
the remaining intrinsic muscles of the foot.

2. Pathology: Compression of the nerve initially causes only sensory


involvement with possibly partial involvement of motor fibers. Continuation of
the irritation, ischemia, and compression may lead to secondary hyperactivity
of the autonomic nervous system, manifested by coldness and numbness from
the altered sympathetic activity. Eventual structural changes in the nerve
result in the development of muscle wasting, paresis, and objective sensory
loss.

NOTE* Reflexes are unaffected

3. Etiology: In the majority of cases no etiology can be found at the time of


surgical decompression.
a. Dilated posterior tibial veins: can also cause severe night discomfort.
b. Trauma: Fracture, dislocation, sprain, post-traumatic edema and fibrosis. c.
Systemic disease: Gouty arthritis with urate deposits, rheumatoid arthritis,
diabetes mellitus, and myxedema.
d. Space occupying lesions: Ganglions, neurofibromas, neurilemmomas, and
synovial cysts.
e. Hypertrophy of abductor hallucis muscle belly.
f. Biomechanical: excessive pronation

4. Clinical Symptoms: Symptoms can be either distal to the metatarsal area,


or the medial and lateral heel depending on the branch involved. a. Early:
i. Intermittent burning pain, numbness and paresthesias over the medial side
of the heel, the toes, and the plantar aspect of the foot. b. Late:
i. A paresis that will develop into paralysis of the pedal intrinsic muscles.
ii. Proximal radiations of pain may develop in the posterior calf.
iii. Pain that is proportional to the amount of activity during the day.
iv. May develop some sensory loss

5. Diagnosis: Not always easy, as the signs are not always definitive a. History
of paresthesias
b. History of trauma
c. History of systemic disease
d. Hoffman-Tinel's sign: A tingling in region of the distribution of the involved
nerve with light percussion, results in paresthesias distal to the site of
percussion.
e. Valleix Phenomena: A nerve trunk tenderness above and below the point of
compression, with paresthesias proximal and distal to the point of percussion.
f. Turk's test: Application of a venous tourniquet to the lower extremity will
elicit positive symptoms on the affected side, by producing a venous occlusion.
g. Forced eversion of the foot.
h. Positive radiographic evidence of previous injury
i. Positive lab studies for any specific disease
j. EMG's and nerve conduction studies are only useful for late stage disease.

Note* EMG may show fibrillation potentials which indicate denervation of


muscle. Nerve conduction studies may reveal an increased distal latency.
Placement of nerve conduction study surface electrodes are as follows:
1. Proximal stimulation point: distal aspect of popliteal fossa
2. Distal stimulation point: behind the medial malleolus
3. Recording electrode (for conduction of the medial plantar nerve) through the
abductor hallucis ms. belly.
4. Recording electrode (for the lateral plantar nerve) through the abductor digiti
quinti muscle belly.

6. Treatment: Conservative
a. Local blocks: Posterior tibial nerve blocks with steroids
b. Unna boot: can be combined with nerve blocks
c. Support hose: for varicosities
d. Functional orthoses
7. Treatment: Surgical Decompression (positive EMG's and nerve conduction
studies mandate surgical decompression). Involves the complete exploration of
the tarsal tunnel with release of the flexor retinaculum and its fibrous bands,
and resection and ligation of any dilated veins in the area.
The surgical technique is as follows:
a. Without a tourniquet, a curvilinear incision is made posterior and inferior to
the medial malleolus by 1 cm.
b. The subcutaneous tissue is incised and the superficial vessels are ligated as
necessary.
c. The neurovascular structures superior to the retinaculum are identified,
preserved, and retracted (especially the medial calcaneal branch).
d. The flexor retinaculum is incised and the posterior tibial nerve or its terminal
branches are identified and mobilized.
e. The nerve(s) is retracted with a penrose drain.
f. The nerve(s) is followed proximally, incising the flexor retinaculum as you go.
g. The nerve(s) is followed distally to the point where the medial and lateral
plantar nerves pass through the fibrous canals superior to the abductor
hallucis ms. belly.
h. The abductor hallucis ms. is examined for any abnormality, and any
hypertrophy is excised.
i. If there are any posterior tibial vein varicosities, they should be ligated.
j. The retinaculum is not reapproximated and no deep closure is done.
k. The superficial fascia is reapproximated and the skin reapproximated
l. Sterile compression dressing and a non-weight-bearing BK cast applied for 3
weeks.

8. Complications:
a. Recurrence: due to fibrosis
b. Severing the PT artery : if done then tie off and prepare patient for
microvascular repair later.
c. Severing a nerve
d. Tenosynovitis
e. Hematoma
f. Wound dehiscence

Classification of Nerve Injuries


1. Seddon and Sunderland classified nerve injuries:
a. Seddon's classification
i. Neuropraxia: (first degree injuries) a conduction disturbance with complete
recovery
ii. Axonotmesis: (second and third degree injuries) an incomplete division of
supportive tissues of the nerve
iii. Neurotmesis: (fourth and fifth degree injuries) a complete division of a nerve
b. Sunderland's classification
i. First degree: only local changes to the myelin
ii. Second degree: injury to the axons that is incomplete
iii. Third degree: leads to more severe axonal injury with fibrosis
iv. Fourth degree: severe neuronal injury with the axons in complete
disarray (no complete neuronal separation)
v. Fifth degree: complete transection of the nerve (dismal prognosis)

Neuromuscular Causes of the Cavus Foot


The cavus foot is classified according to the level of the CNS that is affected
1. Cerebral Cortex: Hysteria
2. Pyramidal and Extrapyramidal: C.P., athetosis, dystonia musculorum
deformans
3. Spinocerebellar Tracts: Friedreich's ataxia, Roussy-Levy syndrome
4. Spinal Cord Level: Polio, myelomeningocele, diastematomyelia, cord tumor
5. Peripheral Nerve or Spinal Nerve Root: C.M.T., polyneuritis-Sotas
6. Muscle: Muscular dystrophy

Types of Nerve Surgery


1. Neurolysis: Frees the nerve from adhesions or scar tissue that obstruct the
growth of regenerating axons or block the conduction of nerve impulses.
Immature nerve fibers may suffer temporary conduction block with neurolysis.
If the entrapped nerve and branches are found in dense scar tissue, the nerve
may be rerouted to a more favorable bed which minimizes the risk of
subsequent compression
a. It is indicated in a complicated first degree injury in which scarring or
adhesions have interrupted conduction
b. In a second degree injury, the nerve is intact and normal in appearance, but
the interfunicular tissue is scarred. Internal neurolysis may be necessary to
split the, sheath and release the bundles from the interfunicular scar tissue
(difficult)

2. Neurorrhaphy: Nerve repair is justified when conservative care fails and


nerve function deteriorates. When it is determined that a traumatized
nerve is partially or completely severed neurorrhaphy can be attempted.
The above diagrams A,B, and C show the technique of partial neurorrhaphy.
Diagram D shows the exposed nerve with orientation sutures, the ends
mobilized, bulbous distal segments removed (E) and epineural sutures inserted
circumfrentially (G,H,I) to repair the nerve
Neurons retain for several years the capacity to regenerate a new axon. The
regenerative process of the nerve remains intact, and the new axons enter the
endoneurial tubes in the stump distal to the trauma. The earlier the
reinnervation, the better the prognosis
a. If a small nerve is partially or virtually severed, repair of the severed section
by partial sutures or resect the damaged segment, mobilize the nerve
proximally and distally to gain the added length, and perform an end to end
anastomosis
b. In a large nerve, if one half is disrupted, partial neurorrhaphy is advisable,
and if a neuroma is encountered, resect back to normal tissue and do an end-
to-end anastomosis
c. Whenever a nerve is transected, it retracts approximately 4% of its normal
length between excision points

3. Neuroma (Morton's):
a. Definition: A neuroma represents hyperplasia of Schwann cells, axonal
elements and fibroblasts in an area where proximal elements cannot relocate to
their distal pathways
b. Histopathology: The term neuroma refers only to nodules that are formed by
hyperplasia of axons and Schwann cells. This process is characterized by
endoneural and neural edema (early stages); perineural, epineural, and
endoneural fibrosis (late stages); and eventually demyelination. It is a reactive
lesion, not a tumor. The term 'Morton's Neuroma' refers to a lesion in the third
intermetatarsal space only.

NOTE* Endoneural edema, fibrosis and demyelination are diagnostic for


Morton's neuroma and other types

c. Anatomy: lies in the 3rd intermetatarsal space, plantar to the transverse


intermetatarsal ligament, where the communicating branch of the lateral
plantar nerve joins the communicating branch of the medial plantar nerve
d. Signs and symptoms: Burning, radiating, lacinating pain and paresthesia.
Can cause calf and heel pain. Palpation can produce pain upon squeezing the
intermetatarsal space, and often a "click" is felt upon lateral pressure (Mulder's
sign)
e. Differential Diagnosis: Metatarsal stress fractures, RA, osteochondritis
dissecans (Freiberg's), localized vasculitis, ischemia, tarsal tunnel, nerve root
compression syndromes, peripheral neuropathy (especially diabetic
neuropathy), and intermetatarsal bursitis
f. Etiology: Compression trauma, and stretching of the interdigital nerve, with
micro-tears of axons
g. Treatment:
i. Conservative: Injections, padding, strapping, orthoses, and wider shoes
ii. Surgery:
 Surgical approach is either dorsal longitudinal, web splitting, plantar
longitudinal, and plantar transverse
NOTE* It is important to do the following when doing this procedure
a. Achieve meticulous hemostasis
b. Identify the digital branches before completing the resection
c. Remove the neuroma without damaging the intermetatarsal artery of
lumbrical
d. Cut the nerve proximal enough to avoid stump neuroma and if possible,
bury the cut end in local muscle
e. Close the dead space

 Other surgical option is decompression of intermetataral neuroma via


cutting the transmetatarsal ligament. Can be done open or endoscopically
(E.D.I.N., as described by Steven Barret, D.P.M.)
h. Complications: The most annoying complication is the formation of a stump
or amputation neuroma. The pain perceived is more proximal plantarly on the
foot than with the pre-existing neuroma. There is a positive Tinel's sign. This
complication does not usually respond to conservative care, and must be
surgically resected. The operation should be done under general or spinal
anesthesia, with a longitudinal plantar incision. The nerve should be cut back
proximally, bathed in steroid, and finally buried in a muscle belly with a non-
absorbable suture. Additionally, nerve caps, and vein grafts have been used
successfully.

NOTE* Local inflammatory reactions (intermetatarsally) may produce


symptoms and signs reminscent of neuroma. These may be chronic, leading to
surgical intervention with a clinical impression of neuroma. Histologic studies
may reveal vascular fibrofatty tissue with inflammation, but no neuroma
Chapter 19: Pediatrics
Evaluation of the Pediatric Patient Pediatric
Biomechanics:
Normal Values (Newborn to Adult)
Congenital Deformities
Pediatric Radiology
Pediatric Gait Patterns
Intoe Gait
Pediatric Fractures
The Limp in Childhood
Juvenile Hallux Valgus
Biomechanical Examination of the Child
Corrective Casting Techniques in Infants
The Toe-Walking Child
PEDIATRICS
Evaluation of the Pediatric Patient
1. Examination of the Neonate:
a. Apgar score (Dr. Virginia Apgar: 1953): It is based upon observations made at
1 and 5 minutes after birth of the heart rate, respiration, muscle tone, reflex
irritability, and skin color (scores of 0-1-2 are given for each). The lower the
score the more depressed the infant. Low scores indicate severe acidosis.

b. Proportional measurements
i. Head circumference (averages 13 inches/slightly larger than the chest)
ii. Chest circumference
iii. Sitting height (crown rump height roughly equal to the head circumference)

c. General postural considerations: depends on the in-utero position


i. Cephalic delivery
ii. Brow delivery
iii. Breech delivery

d. Examination of the head and neck


i. Check for equal eye movements
ii. Head should be held in the midline
iii. ROM of the neck tested

e. Examination of the upper extremities


i. Palpation of the humerus: see if the anterior bulge of the humeral head
absent (produces a dislocated shoulder), and check for fracture of humerus.
ii. Cleidocranial dysostosis: clavicles absent (this condition is detected
later in life due to gait changes from femoral neck deformity).
iii. Check ROM shoulders: if limited then possible dislocation
iv. Check ROM elbows: if limited possible arthrogryphosis
v. Check fingers and nails the absence of which are caused by
congenital abnormalities

f. Examination of the spine: The infant is placed on his abdomen, and the hand
is run over the spine and palpation for scoliosis/kyphosis is performed.
i. Congenital scoliosis: is concerned with scoliosis arising in association with
congenital vertebral anomalies. Some children with congenital scoliosis show
curvature at birth, but many do not. The anomalies and variations in
development of the vertebrae may be single or multiple and may be associated
with other anomalies, particularly in the ribs, which are frequently combined
with spina bifida.
ii. Congenital kyphosis: kyphosis at birth is almost always due to congenital
anomaly of the vertebral column.
iii. Klippel-Feil syndrome: absence of one or more cervical vertebrae, or
two or more fused together, with brain stem abnormalities
iv. Vertebral agenesis: absence of only the coccygeal segment to
absence of all the lumbar and sacral vertebrae.
v. Spina bifida: see section Congenital Deformities
g. Examination of the lower extremities
i. Gross abnormalities
ii. Examination of the hip (dislocation/ femoral rotation/neonatal osteomyelitis)
iii. Examination of the knee and leg (dislocation/bowing of tibia/tibial torsion).
Bowing of the tibia in the infant is present in achondroplasia.
iv. Examination of the foot (metatarsus adductus/calcaneovalgus/ convex pes
planus/ talipes equinovarus)

h. Examination of CNS (muscle tone/reflexes): Usually, at birth, the infant is


limp, but after the first or second cry he develops good muscle tone.
i. Shortly after birth, the newborn flexes himself into a position of comfort, the
position he occupied in-utero. Motor activity should be observed for symmetry,
myoclonic jerks, and convulsive movements. Poor muscle tone in an infant after
a few minutes of age is a grave sign. Infants who remain limp longer than a few
minutes should be suspected of having anoxia, necrosis, CNS lesions, vascular.
collapse, hypoglycemia or mongolism.
ii. Some important-reflexes present at birth include:
 Blink reflex: which occurs when a bright light is directed to each eye
 Rooting reflex: elicited by stroking the angles of the lips with the finger
 Normal suck reflex: elicited by placing a sterile nipple in the mouth
 Grasp reflex: elicited in the palms and soles by placing a finger at the bases
of the fingers or toes
 Moro reflex: is very important and is elicited by startling the baby while he
is on his back. Normally the infant reacts by first extending his arms, then
flexing his arms, clenching his hands, and flexing his knees and hips. The
Moro reflex should demonstrate for symmetry and overall vigor.

NOTE* Absence of this response in the newborn indicates severe CNS injury or
deficiency. Absence of the Moro reflex in one arm indicates a fractured
humerus, brachial nerve palsy, or fractured clavicle. Absence of the Moro reflex
of one leg indicates lower spinal injury, myelomeningocele, avulsion of the
cord, or dislocated hip. A hyperactive Moro reflex indicates tetany, tetanus (not
clostridium), or CNS infection. This reflex should disappear by the 5th month.

2. Neurological Examination of the Infant:


a. Plantar response
i. Either normal or
ii. Babinski (should disappear by age 1 year)
b. Palmar grasp
i. Either normal or
ii. Abnormal (should be normal by age 4 months)
c. Plantar grasp
i. Either normal or
ii. Abnormal (should disappear by age 1 year)
d. Deep tendon reflexes (hypo or hyperreflexia/absent/symmetrical)
i. Patellar
ii. Achilles
iii. Biceps/triceps/flexor carpi radialis
e. Ankle clonus (absent or present)
f. Examination of the anterior fontanelle (should close by age 2 years)
i. Bulging indicates excessive cranial pressure
ii. Depression indicates dehydration
g. Head and neck control (infant should have control by age 2 months)
h. Muscle power
i. Motor skills of the lower extremities: developmental average landmarks
i. Crawls: 3-5 months
ii. Creeps: 7-9 months iii. Stands: 9-14 months
iv. Cruises: 9-12 months
v. Walks: 7-18 months (average is 13 months)

2. Orthopedic examination of the Infant:


a. Examination for dislocated hip (discussed under the heading: Congenital
Deformities)
b. Determination of femoral position (normal=2 x external rotation)
c. Frontal plane knee motion
d. Transverse plane knee motion
e. Determination of malleolar position
f. Ankle dorsiflexion/plantarflexion (should be equal)
g. Examination of the digits and nails
h. Check for spina bifida (check the lower back for any abnormalities)

3. History Taking:
a. Pregnancy:
i. Maternal trauma ?
ii. Any medications?
iii. Complications? Hemorrhaging?
iv. Delivery: Full-term, premature (<38 weeks- 1500 grams), natural,
prolonged, C-section, fetal distress?
Pediatric Biomechanics-Normal Values: Newborn to
Adult
1. Chart of normal findings:
Part Birth Position Adult Position
Thigh/Femur/Hip Angle of head and neck of femur=150° At age 6 years=125°
(angle of Inclination)
Externally rotated 60° At age 6 years=0
Femoral torsion (angle declination)=30° At age 6 years=10°
Total range of motion=150° Past puberty=100°
Knee (Birth) (1&1/2-3yr) (3-6yr) (7-puberty) (puberty-18) (over 18) (over 60)
Genu Varum Straight Genu valgum Straight Genu valgum Straight G. valgum
Genu Recurvatum=5-10° At age 6 years =0°
Externally rotated=30° At age 6 years=0°
Frontal plane rotation=5-10° At age 6 years=0-5°
Transverse plane rotation=5-15° At age 6 years=0-5°
Leg/Tibia Varum(approx. 15°) At age 18 years=0-2° varum
Tibial torsion at birth=0° 1 yr=6°, 2-3 yr=10-15°, 5-6 yr=18-23°
Malleolar torsion at birth=0° 5-6 yr=13-18°
Rearfoot At birth= 10° varus approx. At 6 years=2-5° varus
Talocalcaneal angle=30-50° At 6 yrs=25-30°
Calcaneal inclination angle=approx. 14° At 6 yrs=20° approx.
Talar declination angle=approx. 30° At 6 yrs=21° approx.
Calcaneal stance @ 1 yr=5-10° 5 yr=3-8° 8 yr=<2° external rot.
Dorsiflexion=45° approx. After age 18=10V min
Forefoot Varus 10-15° At age 6=0-2° varus
Metatarsus adductus=15-35° Adult=15-22°

Congenital Deformities
1. Dislocated Hip:
At birth there may be no clinical sign of dysplasia of the hip, so periodic
examinations during the first year are therefore necessary. No clinical sign is
diagnostic by itself. This can present with an older child that limps.
a. Girls are more susceptible than boys
b. Unilateral cases are more common than bilateral, with infants born by
breech more at risk.
c. Diagnostic tests:
i. Ortolani's sign: with adduction of the thighs, a click when the femoral head
is either made to enter the acetabulum or emerge from it, or telescoping of the
flexed and adducted thigh on the pelvis.
ii. Barlow's sign: like Ortolani's sign, except using abduction.
iii. Anchor sign: The infant should be examined while on his abdomen and a
difference in gluteal folds of the two thighs is checked. A difference in the
number of folds can indicate a dislocated hip (not accurate).
iv. Galezzi's sign: while the hips are flexed and the knees flexed while the baby
is on his back, a dislocated hip results in a lower knee position on the affected
side.
v. Abduction test: while the infant is on his back, flex the knees and then
bring them down to the table in the flexed position. A dislocated hip will have
limitation of abduction on the affected side.
d. Radiological examination: The value of radiography of the hip of young
babies is limited by 3 important factors: that only AP views of the pelvis
can be taken; that much of the hip joint is cartilaginous; and that excessive
radiation should be avoided.

NOTE* Two AP views should be taken: one with the lower limbs lying together
and a second with the hips medially rotated and abducted by 45 degrees.

i. Acetabular index: the angle formed by a line touching the inferior margin of
the ilium and Hilgenreiner's line. Greater than a 30 degree angle is indicative of
a dislocated hip (only useful in a unilateral dislocations).
ii. Hilgenreiner's horizontal reference line: across the upper edges of the
ischiopubic joint. In the normal child, the metaphysis should lie well below this
line, and the epiphysis should only just reach to this line
iii. Perkin's vertical reference line: a perpendicular line dropped through the
anterior inferior iliac spine at right angles to Hilgenreiner's line. Normally, the
femoral epiphysis and the beak-like medial side of the epiphysis on its inner
side.
iv. Shenton's line: lies along the upper margin of the obdurator foramen and
continues outward and downward along the under surface of the femoral neck
and the medial aspect of the shaft of the femur. This forms an even curve in the
normal baby, but is interrupted in the dislocated hip.
v. Simon's line: from the lateral margin of the ilium to the upper and outer edge
of the acetabulum, and then continues downwards and outwards along the
upper margin of the femoral neck. This forms an even curve in the normal
baby, but is interrupted in the dislocated hip.

NOTE* Shenton's line and Simon's line can be identified even before the
femoral epiphysis is present. When the epiphysis has appeared, then other
information can be obtained by drawing Hilgenreiner's line and Perkin's line.

vi. Von Rosen's Sign: an AP radiograph is taken with the hips extended and the
thighs abducted 45 degrees and medially rotated. A line drawn along the axis of
the shaft of the femur running close to the anterior superior iliac spine and
crossing the center of the spinal column above the lumbosacral junction
constitutes a positive sign for dislocated hip.
e. Trendelenberg Gait: seen with bilateral dislocated hips
f. There are three forms of congenital hip:
i. Acetabulum dysplasia
ii. Neuromuscular problems
iii. Capsular laxity
Measurements from the above diagram for detection of congenital hip:
Keats TE, Lusted LB: Atlas of Roentgenographic Measurements, Yearbook Medical Publishers, Chicago, 1985, with
permission
1= Hilgenreiner's line, Y line, or symphyseal line, drawn horizontally through the cotyloid
notches of the acetabula
2 & 3= Distances from the apex of the femoral head to Hilgenreiner's line (1), normally these
distances are equal
4= Shenton's line. Follows the upper arched contour of the obdurator foramen, thus marking
the lower margin of the pubic bone, and is continued as a regularly curved line Into the lower
boarder of the femoral neck
5= Break in the continuity of Shenton's line, indicating a dislocation or fracture
6= Fusion of the ischiopubic syndrosis, may be delayed with a dislocation
7= The angle of the acetabulum. If this angle is more than 300 in the newborn and 25° in the
one year old it Is said that there is a "steep acetabular roof" and indicative of a dislocation
(should decrease to 20° by age 2)
8= The diaphyseal interval, the distance between the diaphysis of the femur and Hilgenreiner's
line. This distance should be less than 6 cm
9= If in the newborn the distance of the pivotal point (point of intersection of line 8 and
Hilgenreiner's line) from the tip of the acetabular angle Is more than 16 mm, subluxation must
be suspected
10= This is the horizontal distance between the vertical line of Ombredanne (Perkin's line) and
line 8. This distance is normally less than one-half the epiphyseal width (not illustrated)
11= The vertical line of Ombredanne (Perkin's line) which intersects the upper jutting edge of
the acetabular roof and is perpendicular to Hilgenreiner's line. The center of ossification of the
normal femoral head lies below the horizontal line and medial to the vertical line. In cases of
dislocation this center will be above and lateral respectively
12= The parallelogram of Kopitz. In cases of dislocation a rhomboid will be observed, and the
head of the femur will have an eccentric position
13= The guide line of the Y symphysis down from the center of the acetabulum to the center of
the head
14= The axis of the neck of the femur
15= The angle between 13 and 14 is normally 120°-125°
2. Spina Bifida:
a. Spina bifida occurs in about 1 out of every 1000 live births, the common
problem with this deformity is neuropathy (sensory deficit)
b. Minor degrees of spina bifida affecting the 5th lumbar vertebrae are
insignificant and pose no problems.
c. Spina bifida can be grouped into three clinical entities:
i. Simple meningocele: which may be present anywhere in the spine, however,
most common in the lumbosacral/sacral region, and is caused by a failure of
fusion of the vertebral arches with cystic degeneration of the meninges. It
presents itself with a swelling in the back covered by skin or a thin membrane.
It is flaccid and capable of being transilluminated. The swelling consists of
herniation of the dura and arachnoid filled with cerebral spinal fluid. The lower
limbs show no deformity or any abnormal reflexes.
ii. Open myelomeningocele: most commonly seen in the lumbar/lumbosacral
spine. Patients seen at one day old present with an oval area of red glistening
tissue at the center of the lesion constituting the dysplastic portion of the
spinal cord. The skin is often thin or shows pigmentation in the region
adjoining its junction with the membranous area. In 50% of the children born
with this there may be one or more abnormalities depending upon the cord
level: adduction/lateral rotation of the hip, fixed or limited flexion of the
knee or fixed recurvatum, equinus, equinovarus, calcaneus,
calcaneovalgus, equinovalgus, vertical talus, and clawtoes. The deformity
is always bilateral and symmetrical
iii. Closed myelomeningocele and spina bifida occulta: presents with a
lipomatous or cystic swelling, abnormal pigmentation, hair, and formation of a
dermal sinus. The vertebral arches are unfused, but there is no gross distention
of the meninges. This generally occurs at L5 or S1.
3. Congenital Calcaneovalgus:
This is the most frequently encountered congenital podiatric deformity. It is the
prime cause of the flexible flatfoot deformity.
a. It can be unilateral or bilateral, but is usually bilateral, one side being more
severe than the other.
b. Etiology:
i. Abnormal intrauterine position
ii. Excessive internal limb rotation in fetal life
c. Physical findings:
i. The dorsal surface of the foot is in contact with the anterior surface of the
lower leg.
ii. Redundant skin folds in the lateral malleolar area and tight skin at the
anterior ankle.
d. Clinical findings:
i. The foot lies in acute extention and slight valgus.
ii. Forefoot varus
iii. Rearfoot valgus
iv. Tendo achilles is tight
v. Plantarflexion restricted/ dorsiflexion greater than 15 degrees
vi. Prominent talar head (plantarmedial prominence)
e. Pathology:
i. Talocalcaneal ligaments are relaxed or lacking
ii. The navicular is laterally displaced to the talus
iii. The distal aspect of the calcaneus is laterally displaced
iv. External tibial torsion co-exists in majority of cases

NOTE* According to Ganley, in infants, cartilage is stronger than bone and


therefore osseous structures do not cause the deformity but rather adapt to the
deforming forces.

f. Radiological changes:
(Lateral View)
i. Increased talar declination angle
ii. Plantarflexed talus
iii. The talar head overlaps the distal superior calcaneal surface
iv. The bisection of the talus falls below the cuboid ossification center
(D-P View)
i. Grossly excessive-talocalcaneal angle
g. Treatment:
i. Manipulation: the foot is brought down perpendicular to the ankle
with the heel in neutral position (5 times per correction, 3 times per day
for mild cases)
ii. Casting: applying a BK or AK cast, the foot is held in equinus with
plantarflexion of the first metatarsal and adduction of the forefoot to align
the T-N joint. The rearfoot held in neutral position. Casting weekly for 3-6
months. Following casting, Ganley splints used for 6 months @ 12
hours/day.

NOTE* The key to the treatment is gentle, gradual, and persistent correction.

iii. Surgery: reserved when the T-C angle exceeds 30-35 degrees

4. Convex Pes Valgus (Vertical talus/congenital rigid flatfoot):


Is a rare condition which presents clinical findings at birth similar to congenital
calcaneovalgus, but is very different and more severe. Abnormal radiographic
findings consistent with rigid flatfoot will appear with patients at rest and
during weightbearing. The range of motion of the subtalar joint is the key
clinical finding in the evaluation of this condition.
a. Etiology: Talonavicular dislocation where the navicular subluxes over the
talus.
b. Usually found with other congenital deformities and/or endocrine
disturbances.
i. Arthrogryposis multiplex congenita
ii. Myelomeningocele
iii. Cerebral palsy
iv. Coarctation of the aorta and high palate
v. Congenital dislocation of the hip vi. Clubfoot
vii. Nail-patella syndrome
viii. Multiple pterygium syndrome
ix. Marfan's syndrome
x. Down's syndrome (trisomy 21)
xi. Trisomy 13-15, and 18
xii. Neurofibromatosis
xiii. Mental retardation
c. Clinical Findings:
i. A rigid deformity with the heel in neutral/vertical
ii. A depression if found anterior to the lateral malleolus
iii. A convex plantar aspect (rocker bottom foot)
iv. Forefoot is abducted and there is dorsiflexion of the forefoot on the
rearfoot.
v. Tendoachilles is tight and the anterior muscle group is tight
vi. Talar movement is impossible (no supination)
vii. Calcaneal equinus
d. Pathology: Soft tissue and osseous

NOTE* The structures attached to the navicular plantarly will be stretched,


while those structures attaching dorsally will be contracted. The tibionavicular,
talonavicular, and calcaneofibular ligaments hold the deformity so that the
calcaneus cannot be manually inverted.

Soft Tissue Pathology: structures contracted, due to dorsiflexion at the


midtarsal joint
i. Tibionavicular ligament (Deltoid)
ii. Talonavicular ligament
iii. Anterior ms. group
iv. Peroneus brevis/longus
v. Triceps surae
vi. Calcaneofibular ligament
vii. Posterior ankle and subtalar capsule
Soft Tissue Pathology: structures stretched
i. Spring ligament
ii. Tibialis posterior, FDL, and FDB
Osseous Pathology:
i. Subluxation of the navicular on the talar neck
ii. Talus locked in the vertical position
iii. Calcaneus in a 20-25° equinus position
iv. Talar head subluxed below the navicular
v. Talar head has an abnormally flat surface
vi. Talar neck is hypoplastic, develops an abnormal facet
vii. Talus has an hourglass shape
viii. Anterior surface of the calcaneus is deviated laterally
ix. Subtalar facets are abnormal due to the abnormal T-C articulation (the
anterior facet is absent/middle facet is hypoplastic/posterior facet is malformed
x. Calcaneocuboid subluxation (dependent on the type of deformity)
e. Radiographic findings:
I. T-C angle increased (usually >45°)
ii. Talus perpendicular to the tibial longitudinal axis iii. Equinus calcaneal
position
iv. Dorsal dislocation of the navicular on the talus v. Lateral forefoot
subluxation
v. Talocalcaneal overlap
vi. Navicular is dorsiflexed and laterally displaced to the talar head (it is usually
cartilaginous until age 3 years so its position must be evaluated via position of
the talar head and the medial cuneiform f. Treatment: conservative vs. surgical
i. Conservative: Serial long-leg casting up to 6 months of age, with the foot in a
position of ankle plantarflexion, an inverted heel, forefoot in adduction to
reduce the T-N dislocation, and the knee flexed 90°. The key to successful
results is lining up the T-N joint. If a proper result occurs, the patient will have
an equinovarus foot type which would then have to be corrected.
ii. Surgery: Herdon-Hyman and Craig

NOTE* Aggressive surgical approach is indicated when the T-N joint is non-
reducible

5. Metatarsus Adductus (Metatarsus varus):


The latter is the more severe form of this condition. Met. adductus is
where the anterior part of the foot deviates medialward and there is a varus
angulation at the T-M joints. The heel may be neutral or in valgus and if it is in
valgus it is a more severe form., This condition is not usually noticed at birth
but about 8 weeks later. The incidence of metatarsus varus has increased 4x in
the past 25 years.
a. Classification:
i. Flexible
ii. Rigid
iii. Functional dynamic
iv. Skew foot
b. Etiology:
i. Arrested ontogenetic development
ii. Uterine pressure
iii. Abnormal muscle/tendon insertions of the adductor hallucis/FDB
c. Clinical Assessment:
i. Degree of inversion/adduction
ii. Medical concavity/lateral convexity
iii. Associated met primus adductus
iv. No rearfoot involvement except with a skew foot v. Use "V" finger test
d. Radiographic findings: Angle of met adductus on DP view is normally 15-35°
at birth and should decrease to 25°.
NOTE* The metatarsus adductus angle is the angle formed by the intersection
of two lines, the bisection of the second metatarsal and the perpendicular of
the lesser tarsus. The lesser tarsus is is bisected in the infant using the
following
technique:
1. A line is drawn from the distal medial talus to the medial base of the 1st
metatarsal, and its midpoint marked.
2. A line is drawn from the calcaneus to the base of the 4th metatarsal, and its
midpoint marked.
3. A line is drawn connecting the two midpoints from 1 and 2, and this line is
bisected.
4. A perpendicular line is drawn at the midpoint of line 3, which is the final
reference line that is the bisection of the lesser tarsus.

e. Treatment:
i. Manipulation
ii. Shoe therapy
iii. Wheaton Brace (works well in a flexible deformity in child under 1
year old)
iv. Serial cast immobilization (cast with the calcaneus in neutral/with an
abductory forefoot force) Use 2 sets of casts .
v. Orthoses
vi. Surgery: Should postpone surgery until after age 2-3 years (HHS: soft
tissue; Johnson: cartilaginous; Lepird and Berman 8 Gartland: osseous)

6. Talipes Equinovarus:
Clubfoot may be acquired or congenital. At birth, other disturbances such as
hip dysplasia/neuromuscular disorders/poliomyelitis/tibial epiphysis
abnormality must be ruled out in with patients with clubfoot.
a. Incidence:
i. 1:1000 births (according to Kite)
ii. Increases to 1:35 if a sibling has clubfoot
iii. Males to female ratio is 2:1
b. Etiology:
i. Arrest of fetal development (Max Bohm: JBJS, 1929)
ii. Combination of inheritance and environmental factors (Ruth WyneeDavis:
JBJS, 1929)
iii. Primary germplasm defect (George Settle: JBJS, 1963) iv. Genetic factors
(Palmer)
c. Physical findings:
i. Medial border concave/lateral border convex ii. Talar head prominent
laterally with inversion iii. Palpation of a medially displaced navicular
iv. Deep furrowed creases along the medial longitudinal arch area and the
posterior ankle.
d. Clinical findings:
i. Inversion and adduction of the forefoot
ii. Inversion of the rearfoot
iii. Talocalcaneal equinus/tibiotalar equinus
iv. No dorsiflexion
v. Associated with tibial torsion and cavus deformity (with cavus deformity
there is also plantarflexion of the forefoot on the rearfoot).
e. Types:
i. Rigid: usually genetic and very resistant to treatment (seen with small,
inverted, plantarflexed calcaneus, and posterior leg atrophy)
ii. Flexible: usually due to intrauterine position and usually responds to
casting.
f. Pathology:
i. The major defect is in the talus, with the head and neck medially and
plantarly deviated, the superior surface is anterior displaced out of the ankle
mortise, and the sinus tarsi is opened as the anterior facet is tilted medially.
ii. The T-N and C-C joints face medially and plantarly, with the C-C lying below
the T-N joint.
iii. Tibia is usually OK, occasionally, tibial torsion involved iv. Soft tissue
contractures:
 Posterior- ankle capsule, STJ capsule, calcaneofibular ligament,
tendoachilles.
 Medial- spring ligament, deltoid ligament, ms. TP, FDL, FHL.
 Plantar- plantar fascia, abd. hallucis, FHD.
g. Radiographic findings:
i. DP view reveals S angle of Kite (T-C angle): 0-15° with clubfoot
ii. DP view reveals superimposition of talar head and the anterior surface of the
calcaneus.
iii. DP view reveals ossification center of the navicular is medially displaced
iv. Lateral view with dorsiflexion and forced equinus, reveals calcaneal
inclination angle of approx. 17° (normal is 45°).
v. Lateral view reveals a bisection of talus is above the metatarsals h.
Treatment: Conservative vs. surgical
i. Serial casting (Kite: JBJS 1963): must be done early and as follows
• First, correct the forefoot adduction- Position the forefoot in abduction
and push in and up under the calcaneocuboid joint and forcing the forefoot
laterally. Abduction is positive and will cause a flatfoot condition
• Second, correct the inverted heel- Before the equinus can be corrected
these 2 deviations must be treated and corrected first (if not a rockerbottom
foot will develop). The cuboid must be brought up to the calcaneus and the
navicular must be brought up to the talus.

NOTE* Do not attempt to fix the equinus until the foot is mildly flat

Third, correct the equinus- By dorsiflexing at the ankle.

NOTE* Maintain the casts until a correction is achieved and then follow with a
Ganley splint for the same amount of time that casting was utilized
i. Radiographic findings consistent with a failed treatment:
i. Forefoot is dorsiflexed on the rearfoot at the midtarsal joint
ii. Rocker-bottom deformity
iii. Flat-topped talus (can be due to talar head ischemic necrosis from
compression)

7. Congenital Deformities of the Forefoot:


a. Overlapping lesser toes
b. Juvenile hallux valgus:
i. Treat with splinting
ii. Treat with surgery- (soft tissue/chondrotomy)
c. Syndactyly (webbed toes)- usually 2nd and 3rd toe:
i. Due to embryonic developmental arrest
ii. Associated with Apert's syndrome (acrocephalosyndactyly)
iii. Treat with corrective plastic surgery (elective plastic surgery)
d. Polydectyly (supernumerary toes):
i. Associated with Ellis-van Creveld syndrome (chondroectodermal dysplasia)
ii. Treat with surgery: resection of non-functional digit (no muscle/tendon
attachments)

NOTE* Functional digit articulates with metatarsal head properly and has
better blood supply

e. Congenital short metatarsals:


i. Short 4th metatarsal can be associated with pseudohyperparathyroidism or
pseudopseudohyperparathyroidism
ii. Treat with metatarsal lengthening procedure
f. Ectrodactyly (lobster claw foot): absence of 2 or more metatarsals and their
associated phalanges.
g. Long toes: Associated with Marfan's syndrome (also seen are visual problems,
ligamentous laxity, long fingers, and mitral valve prolapse)
h. Edematous feet: Associated with Milroy's disease
i. Enlarged or giant toes (macrodactyly): Associated with von Recklinghausen's
disease (CT scan of CNS to check for neurofibromas)

Pediatric Radiology
1. Roentgenographic Development of the Foot:
a. Important ossification points to remember:
i. 1st bone to ossify before birth: calcaneus
ii. Last bone to ossify before birth: cuboid
iii. 1st bone to ossify after birth: lateral cuneiform
iv. Last tarsal bone to ossify after birth: navicular at 3.5 years
v. Calcaneal apophysis appears at age 7 years
vi. Sesamoids appear at age 12 years
b. Ossification at birth:
i. Talus
ii. Calcaneus
iii. Cuboid (can be absent in the premature baby)
iv. Metatarsals
v. Proximal phalanges
vi. Middle and distal phalanges 2-4
vii. Distal phalanx 1
c. Age 3 months: lateral cuneiform
d. Age 4 months: tibial epiphysis
e. Age 6 months: cuboid and lateral cuneiform articulate
f. Age 7 months: talar neck appears, base of metatarsals widen
g. Age 11 months: fibular epiphysis appears
h. Age 18 months: phalangeal epiphyses appear
i. Age 24 months: medial cuneiform and ossification of epiphysis of metatarsal
1
j. Age 30 months: intermediate cuneiform ossifies
k. Age 36 months: ossification of epiphysis of metatarsals 2, 3, and 4
l. Age 3.7 years.: ossification of navicular
m. Age 4.2 years: ossification of epiphysis metatarsal 5
n. Age 4.9 years: alignment of tarsal and metatarsal bones

NOTE* Boys lag behind girls with regard to skeletal age

o. Age 6.7: ossification of calcaneal epiphysis


p. Age 12 years: sesamoids appear
q. Age 13 years: os trigonum and os vesalianum appear
r. Age 14 years: fusion of epiphyses of distal phalanges of toes 2, 3, and 4
s. Age 15 years: epiphyseal fusion of tibia/fibula, metatarsals 2/3/4, and
phalanges 1, 3, 4, and hallux.
t. Age 17.5 years: epiphyseal fusion complete

2. The Osteochondroses: (also see chapter: The Arthropathies)


These are a group of related disorders which effect the primary or secondary
centers of ossification. Its etiology probably relates to some type of vascular
disturbance to the ossification center, during the time of their greatest
developmental activity.
a. Osgood-Schlatter's disease (osteochondrosis of the tibial tubercle):
i. Clinical appearance: Complaint of pain at the anterior aspect of the knee.
Occurs between the ages of 11-15 most commonly in boys who participate in
sports. On examination there is enlargement of the tibial tubercle, with a
maximum area of tenderness at the insertion of the patellar tendon.
ii. Etiology: Tendonitis of the distal portion of the patellar tendon with
secondary hypertrophic new bone formation
iii. X-ray findings: Soft tissue swelling anterior to the tibial tuberosity (acute
stage), irregularity in the ossification center of the proximal tibial tubercle is a
normal variation, late stages a prominent/irregular tibial tuberosity with or
without a small particle of bone anterior and superior to the tibial tuberosity.
iv. Differential diagnosis: Must R/O osteogenic sarcoma
v. Treatment: Restriction of excessive physical activities, severe cases
immobilization of the knee.
b. Kohler's Disease (osteochondritis of the tarsal navicular):
i. Clinical appearance: Affects boys (80%) 3-6 years old. Can be a
spontaneous onset (pain and swelling over the navicular) or brought on by
injury. A limp (antalgic gait) may be present with the child walking on the
lateral side of the foot.
ii. Etiology: Abnormalities in ossification of the navicular from compression of
the bony nucleus at a critical stage of growth of the navicular.
iii. X-ray findings: A disc-like navicular is visualized, with irregular ossification,
intermingled plaques and hazy fibrocystic degeneration. There is soft tissue
swelling around the bone.
iv. Treatment: In mild cases stop strenuous exercises and use rigid
orthoses. In severe cases a BK cast in supination for 6-8 weeks.
c. Freiberg's Disease (osteochondritis of the second metatarsal):
i. Clinical appearance: Usually seen in young active adolescents (after
age 13). It is more common in females, and is usually unilateral affecting
the 2nd metatarsal head primarily (can affect others). Pain is localized under
the affected metatarsal head, with swelling and limitation of motion at the
m.p.j.
ii. Etiology: Due to a vascular insufficiency from trauma to the epiphyseal area.
iii. X-ray findings: Widening of the joint space with effusion and soft tissue
swelling in the early stages. Later there is a flattening of the head of the affected
metatarsal, a narrowing of the joint space, and subarticular sclerosis. There is
never any ankylosis.
iv. Treatment: Orthoses to remove pressure from the metatarsal head in
mild cases. For severe cases use a BK cast for 4-6 weeks, or if symptoms
persist into adult life a metatarsal head resection or total joint replacement.
d. Sever's Disease (calcaneal apophysitis):
i. Clinical appearance: Seen in children ages 8-14 years. Most common
in boys 10-11 with a cavus foot type. Complaints are pain in the heel especially
after rigorous activity. Pain is exacerbated by squeezing the medial/lateral
epiphyseal margins of the calcaneus.
ii. Etiology: Excessive traction on the calcaneal apophysis
iii. X-ray findings: Multiple centers of ossification, a moth eaten appearance of
bone, and apophyseal sclerosis (all of which also can be seen in a normal
apophysis)
iv. Treatment: Cessation of rigorous physical activity, stretching of posterior
muscles, shock absorbing heel pad, and/or orthoses. If severe then a BK cast.

Pediatric Gait Patterns


1. Parameters of Normal Gait:
a. Early independent ambulator (1 year) characteristics:
i. Wide base of gait
ii. Prolonged hip and knee flexion
iii. Arms abducted with elbows extended to increase stability
iv. High cadence
v. Low velocity
vi. Short step length
b. Toddler (1-2 years) characteristics:
i. Narrowing base of gait
ii. Reciprocal arm swing
iii. Increased step length
iv. Increasing velocity
v. Mild foot drop
c. 2 year old characteristics:
i. Reduced external rotation during arm swing
ii. Decreased pelvic tilt
iii. Knee flexion in early stance
iv. Knee extention in late stance
v. Emerging heel strike
vi. Disappearance of mild foot drop
d. 3 year old (early mature gait) characteristics:
i. High cadence
ii. Low velocity
iii. Increasing stride length with growth of limb
iv. Increasing single support phase
e. 7 year old characteristics:
i. Decreasing cadence
ii. Increasing stride length
f. Full mature gait characteristics:
i. Increased single support phase
ii. Increased velocity
iii. Increased stride length
iv. Increased ratio of pelvic span to ankle spread
v. Decreasing cadence

2. Abnormal Gait Patterns:


a. Spastic gait: Manifested by internal rotation and adduction of the entire
limb, with hip/knee/ankle in marked flexion. Seen with cerebral palsy, familial
spastic diplegia, paraplegia, and hemiplegia.
b. Dyskinetic gait: A constant movement abnormality with a high degree of
variability from patient to patient and gait cycle to gait cycle. It is characterized
by motion involving considerable effort, often with deliberate, almost
concentrated step. Seen with cerebral palsy, Huntingtons chorea, and dystonia
musculorum deformans.
c. Ataxic gait: Characterized by a marked instability during single limb stance
with an alternating wide/narrow base during double support. During swing
phase the limb will swing widely and cross the midline. Seen with multiple
sclerosis, tabes dorsalis, diabetic polyneuropathy, Fredrich's ataxia.
d. Waddling gait: A labored walking, exhibiting difficulty with balance,
proximal pelvic instability, leading to a lumbar lordosis. May see an associated
equinovarus foot type. Seen with Duchenne muscular dystrophy, limb-girdle
muscular dystrophy, Beckers muscular dystrophy, spinal muscular atrophy,
and congenital dislocated hips.
e. Steppage gait: Gait exhibits a swing phase drop foot. Seen with Charcot
Marie-Tooth, polio, Guillain-Barre syndrome, CVA, paralytic drop foot, and
fascioscapulohumeral dystrophy.
f. Vaulting gait: Gait changes include a high cadence, increased lateral
trunk movement, scissoring and instability from step to step, suggesting a
loss of balance. Seen with myotonic dystrophy.
g. Equinus gait: Gait exhibits a swing phase ankle plantarflexion with no
heel contact. Seen with cerebral palsy, Charcot-Marie-Tooth, muscular
dystrophy, spinal muscular atrophy, schizophrenia, osseous block of the
ankle, and habitual toe walking.
h. Festinating gait: Gait changes include loss of reciprocal arm swing,
decreased velocity, shuffling, decreased stride length, and increased
cadence. Seen with Parkinson's disease.
i. Trendelenberg gait: Stance phase of each step leads to a contralateral tilt
of the pelvis with a deviation of the spine to the affected side. Seen with
dislocated hip or weakness of gluteus medius.

Intoe Gait
1. Evaluation:
a. History:
i. Onset of the problem: What age? Sudden onset? How long?
ii. Severity of the problem: Difficulty walking? Falling? Does not want to
play? More pronounced in rigid shoes?
iii. Developmental history: as previously discussed
iv. Hospitalizations/ injuries v. Family history
b. Observation of gait:
i. Angle of gait: in-toe or out-toe or rectus
ii. Position of the knee at midstance: externally/ internally deviated,
straight
iii. Presence of a limp?
iv. Presence of equinus?
v. Early heel lift off?
vi. Bouncing gait?
vii. Rapid out-toe after midstance?
viii. Excessive pronation? No arch present?
c. Examination of hips: (performed in younger children: < 2 years)
i. Should be able to abduct hips at least 70° (flexing hips and knees)
d. Determination of femoral position:
i. Mark the center of both knees and position them straight up
ii. With the child lying down" and the hip and knee extended, rotate the
femur inward (estimate the number of degrees)

NOTE* This is done by rotating the knee and visualizing this by noting the
numbers on a clock, whereby each hour equals 30°. So if the knee moves from
the straight up position to 2 o'clock (external) that equals 60° of external
rotation.

iii. Rotate the femur outward (estimate the number of degrees of motion)
iv. Perform the same examination with the hip flexed an then with the knee
flexed (to eliminate soft tissue elements)

NOTE* If the measurements vary with positional changes then the abnormal
endings are due to soft tissue tightness. If the measurements are the same
then the problem is torsional within the femur.

e. Average rotation of the hip joint:


i. Up to 1 year old: twice external to internal rotation
ii. At 5 years old: internal = external rotation
f. Internal femoral position/can produce an in-toe gait:
i. Anti-torsion and retroversion
External femoral position/can produce an out-toe gait:
ii. Retrotorsion and retroversion g. Examination of the knee joint:
i. Normal frontal plane motion: birth/5-10°,decreases to 0-5° at 6 years
old
ii. Normal transverse plane motion: birth/5-15°, 0-5° at years old
iii. Pseudomalleolar torsion (pseudolock): excessive transverse plane
motion at the knee can result in in-toe gait (usually 15-35°)
h. Examination of malleolar position (tibial torsion: add 5° to malleolar
torsion):
i. measure the malleoli with a goniometer
ii. Values: birth/0-5°, 6 years old/13-18° (less than this = in-toe gait)
i. Other deformities causing in-toe gait:
i. Talar neck adductus
ii. Clubfoot
iii. Talipes varus
iv. Metatarsus adductus
v. Spastic posterior tibial ms.
vi. Genu varum
vii. Tibia varum
j. Treatment:
i. Not all cases warrant treatment
ii. The cases that should be treated are those in which 8° of improvement would
not bring their feet into a straight-forward position,
those children having difficulty walking, or a destructive foot type
iii. Manipulation of the soft tissues by the parent
iv. Functional orthoses to control pronation
v. Gait plates (child must have a propulsive gait)
vi. D-B bar (inhibits abnormal sleeping positions- use 1 " wider than hips)
vii. Ganley splint (like D-B bar but neutralizes pronation at the STJ)
viii. Plaster cast for pseudolock
viii. Surgery
Pediatric Fractures
1. Anatomical Considerations:
a. Epiphysis
i. Pressure epiphysis: At the articular surface, involved with growth
ii. Traction epiphysis: Not at the articular surface and not involved with growth
(femoral trochanter, tibial tuberosity, calcaneus, base 5th metatarsal)
b. Diaphysis
c. Metaphysis
d. Epiphyseal plate: Zone between the epiphysis and the metaphyseal plate
Microscopic appearance (multiple cell types)
i. Resting cells
ii. Proliferating cells
iii. Endochondral cells
e. Points to remember:
i. Matrix makes the plate the strongest at areas close to the epiphysis and
metaphysis.
ii. The zone of provisional calcification, where cells are dividing and calcification
begins, is the weakest part of the region, poor in matrix; epiphyseal injuries
usually occur at this part.
iii. Due to the separation at the middle of the physis, it leaves the proliferating
cell zone uninterrupted; thus there is usually no interference in bone growth,
as long as circulation to the physis remains intact.
iv. Diaphyseal blood supply will nourish the layer of endochondral ossification
without crossing the epiphyseal plate.
v. Epiphyseal blood supply will enter through the joint capsule and through the
perichondrium to the epiphysis zone area of the epiphyseal plate (2 types:
intracapsular & extracapsular).
vi. When making a diagnosis a minimum of 3 views should be taken and
bilateral x-rays should be taken.

Note* The physis contains 3 parts:


a. Zone of Growth: containing the dividing and resting cells
b. Zone of Maturation: for calcification
c. Zone of Transformation: for ossification
The Zone of Ranvier is a groove surrounding the periphery of the physis and is
composed of..
a. Fibroblasts
b. Mesenchymal tissue (undifferentiated cells-multipotential)
c. Osseous ring of Lacroix (an extention of the metaphyseal cortex)
The Zone of Ranvier functions so that longitudinal and latitudinal growth of the
physis proceeds in harmony
Note* Remember that the periosteum in the child is more vascular, thicker,
loosely attached and stronger than in the adult

2. Results of a Swedish study of epiphyseal fractures:


a. The risk of fractures increases at ages 11-12 in females and 13-14 in males
b. Chance of epiphyseal fracture is 42% till age 16 in males and 27% in
females.
c. Fractures at the epiphysis peak at 2 years of age and again between 11-14
years of age.
d. 13.4% of the fractures occur in the foot and ankle e. 21 % of the injuries are
sports related
f. Pathological fractures should be taken into account occurring due to minor
trauma in diseases such as: renal disease, cystic fibrosis, growth hormone
deficiency, diabetes mellitus, Turner's syndrome.

3. Important points to remember:


a. Haversian canal system in children is more extensive as compared to that of
adults, especially in the metaphysis.
b. Children's fractures can be of compression or tension type; in adults
compact bone fails in tension only.
c. Incomplete failure in tension results in a greenstick fracture
d. In. children, the strongest skeletal segment is the periosteum, the weakest
being the growth plate.
e. Due to the strong periosteum one rarely sees open fractures in children
Intermediate strength is the epiphysis which is protected by joint congruency
and cartilage.
f. Capsule and ligaments are a continuation of the periosteum, which mostly
bypasses the epiphysis, distributing forces to the growth plate and metaphysis.

4. Classifications of epiphyseal fractures:


a. Salter-Harris: Is an anatomic classification applicable to most physeal
injuries. There are problems associated with this classification: the size of the
epiphyseal/metaphyseal fragment is not quantified. There are 5 types:

Type 1: complete separation of the epiphysis with no fracture of bone Type 2:


fracture along the epiphysis then through the metaphysis producing a
triangular fragment
Type 3: extends from the joint surface to the epiphyseal plate, along the plate
to the periphery
Type 4: extends from the joint surface to the epiphysis and plate and through
a portion of the metaphysis
Type 5: severe crush injury and compression of the plate

NOTE* Type 6 was added by Rang, a bruising of the peripheral growth plate
secondary to blunt trauma, resulting in a peripheral osseous bridge causing
angular deformity (treat with resection of the osseous bridge with correction of
the angular deformity)
i. S-H Type 1: Fibular fracture
 occurs due to inversion stress
 may appear as an isolated injury or in combination with a medial corner
injury to the ankle (Salter 3 Tibial)
 diagnosis is difficult (there should be at least one cm of lateral soft tissue
swelling around the plate area)
 these fractures may or may not be displaced
 good prognosis
 apply short-leg walking cast for at least 3 weeks
 occurs in ages < 12
ii. S-H Type 1: Tibial fracture
 occurs from shear stress due to external rotation injuries
 can occur with fibula shaft fracture which is usually transverse
 may be displaced or nondisplaced
 good prognosis
 treat with closed reduction (prn) and immobilze in NWB A-K cast for 3 weeks
plus 2 weeks in a B-K cast

iii. S-H Type 2: Fibular fracture


 caused by eversion and external rotation of the foot
 treat with 3-4 weeks in a B-K cast
 occurs in the older age group where the epiphyseal plate is more bonded
(age 13)

iv. S-H Type 2: Tibial fracture


 caused by eversion (small medial fragment), external rotation (large medial
fragment), or plantarflexion (posterior fragment)
 associated with a fibular fracture
 may be displaced or non-displaced
 average age is 13
 triangular fragment called Thurston-Holland sign medially
 opposite disruption called Lamellar sign laterally, is the overhang of the
proximal end of the tibia over its distal end as the fracture shifts laterally
 prognosis is good
 application of B-K cast for 3-4 weeks and mild manipulation by reversing the
mechanism of injury (avoid excessive manipulation because the germinal
cells are already damaged)

iv. S-H Type 3: Tibial fracture


 caused by inversion
 may or may not be displaced (take MO x-ray of the ankle to determine
amount of displacement)
 treat if non-displaced with A-K cast for 3 weeks and a B-K cast for 3 weeks
 treat if displaced (by more than 2 mm) with ORIF (the fixation must not
cross the plate)
 good prognosis if alignment is restored

v. S-H Type 4: Tibial fracture


 caused by a continuing inversion force greater than that in a Salter 3 injury
 treat same as Salter 3 tibial for non-displaced and displaced, except that
additional fixation is needed for the metaphyseal fragment (proper
anatomical reduction/alignment is necessary)
 these are rare
 possible growth disturbances

vi. S-H Type 5: Tibial fractures


 caused by severe axial loading across the plate resulting in multiple fracture
lines
 these fractures may lead to premature fusion across the whole plate causing
a limb length discrepancy
 may lead to fusion on one side or from front to back causing asymetrical
 angulation of the limb
 treat if displaced by ORIF to restore articular congruity
 prognosis is poor

vii. S-H Type 6:


 due to bums or avulsions
 loss of growth plate substance
 epiphyseodesis can result with premature epiphyseal plate closure with
progressive shortening and angular deformity

viii. Summary of S-H fractures:


 the younger the patient the greater the chance of deformity
 type 1 and 2 have a better prognosis than types 3 to 6
 a non-displaced epiphyseal fracture has a better prognosis
 a compression fracture has the poorest prognosis
 blood supply interruption leads to growth arrest
 infection in addition to a fracture can be disastrous
 reduction (open or closed) should be done gently within 10 days after
trauma
 healing of epiphyseal injuries takes 3 weeks as compared to 4-6 weeks for
bone
 the resulting deformity has to be treated surgically
 follow-up for growth retardation should be done for at least 3 years
 radiological studies with markers indicate 5 types of post-traumatic growth
patterns: symmetrical growth, initial/temporary growth stimulation,
initial/temporary growth retardation, initial/ progressive growth retardation,
and initial/permanent growth arrest.
b. Dias and Tachdjian: Added a classification of physeal injuries based in
Lauge-Hansen and Salter-Harris. The first term describes the position of the
foot at the time of impact, and the second term refers to the direction of
the talus taken by the injuring force. It involves 4 types:
i. Supination-plantarflexion
 Result in Salter 2 fracture of the distal tibia with the metaphyseal fragment
posteriorly
ii. Supination-external rotation
 Stage 1:Characterized by Salter 2 of the distal tibia
 Stage 2: Characterized by a short oblique fibula fracture above the distal
fibula physis
iii. Supination-adduction
 Stage 1:Characterized by Salter 1 or 2 of the distal fibula
 Stage 2: Characterized by Salter 3 or 4 of the distal tibia
iv. Pronation-external rotation
 Stage 1: Characterized by a Salter 1 or 2 of the tibia
 Stage 2: Characterized by a fibula fracture above the level of the tibiotalar
joint with the metaphyseal fragment lateral

c. Juvenile Tillaux fracture:


i. An avulsion fracture of the distal lateral tibial tubercle by the anterior
tibiofibular ligament due to an external rotation force
ii. This fracture is seen on an A-P view as a S-H type 3 fracture of the lateral
distal tibial epiphysis, involving from 20-50% of the width of the epiphysis
iii. Usually occurs in ages 12-14, because of closure of the medial half of the
epiphysis and not the lateral half

iv. Treat if non-displaced with a A-K cast for 6-8 weeks with some internal
rotation of the lower end of the cast

NOTE* Treatment is ORIF if displaced with lag screw technique from distal
lateral to proximal medial (can cross the physis since growth is already
completed)

Note* To best view this fracture, take an oblique x-ray in which the fibula is
rotated internally to eliminate the overlap of the tibia to better see the entire
distal tibial epiphysis

vii. Triplane fractures:


 combination of a central vertical Salter 2 and Salter 3 tibial fracture
 can be missed on routine x-rays
 treat as per Tillaux type
The Limp in Childhood
Limp is an exaggeration or deficiency of normal gait which may be painful or
painless. It may be associated with paralysis, spasticity, contractures, loss of
supporting structures, limb length discrepancy, and ataxia. It should be
considered to reflect serious disease until proven otherwise. Except in the most
obvious causes, a complete review of history and general physical
exam is necessary and if indicated appropriate lab studies, x-rays, and
consultations. Looking at limp by age groups is helpful.
1. The Beginning Walker:
a. Congenital hip dislocation
b. Trauma (always consider abuse)
c. Septic joint (diagnosis urgent especially in the hip)
d. Osteomyelitis
e. Synovitis
f. Neuromuscular disorders

2. The Walking Child:


a. Congenital dislocated hip
b. Cerebral palsy (may only manifest itself as intoeing)
c. Discitis (pain on straight-leg raising)
d. "Toxic" synovitis (must r/o septic hip, labs are WNL, a small percentage
develop Legg-Perthes disease)
e. "Growing pains"
f. Legg-Calve-Perthes disease

3. Preteens-Teens:
a. Congenital dislocated hip
b. Slipped proximal femoral capital epiphysis (the patient is usually larger and
male; holds leg in internal rotation; in gait patient lurches over the involved hip

SUMMARY OF TYPES OF LIMP


Age (Yr) Antalgic Paralytic Short Leg Contracture Loss of Support
Birth-4 Trauma, infection Cerebral palsy, Congenital, coxa Spina bifida, cerebral CHD, trauma,
spinal vara, palsy, CHID, Infec- van, spine I:
muscular atrophy, CHD, infection tion
spin bifida, hemi-
5-10 Trauma, infection, atrophy palsy,
Cerebral Spina bifida, trauma, Trauma, CHD, Legg- Trauma, polio, infec-
rheumatoid arthritis, spina CHD, Infection, Calve-Perthes dis- tion, muscular dys-
Legg-Calve-Perthes bifida, muscular dys- rheumatoid arthritis ease, infection, rheu- trophy
disease, hemophilia, trophy, polio matoid arthritis
transient synovitis
11-14 Trauma, synovitis, Muscular dystrophy, infection, trauma, Infection, trauma, Trauma, slipped capi-
slipped capital femo- polio, peripheral slipped capital femo- Legg-Calve-Perthes tal femoral epi-
ral epiphysis, infec- nerve trauma, CNS ral epiphysis, neo- disease, slipped capi- physis, inadequate
tion, Osgood-Schlat- neoplasm, ischemic plasm tal femoral epiphysis treatment, CHD
ter disease contracture
CHD, Congenital heart disease.
Juvenile Hallux Valgus
1. Classification:
a. Congenital:
i. Aplasia: small toes, cleft foot, or amputated toes (a secondary HAV/
interphalangeus, no met primus varus)
ii. Congenital vertical talus: overgrowth of the medial column
iii. Residual met adductus
b. Hyperlaxity (hypermobile pes planus):
i. Physiologic
ii. Pathologic: Down's and Marfan's syndrome
c. Adolescent:
i. Advanced met primus varus secondary to
 trapezoidal medial cuneiform
 oblique 1st met-epiphyseal line
 oblique 1st met-medial cuneiform joint
ii. Associated hypermobile pes planus
d. Neurologic:
i. Equinovalgus: contracted achilles with overpull of the peroneals
ii. Spasticity of the adductor hallucis
e. Superimposed factors:
i. Extrinsic: shoes, ballet
ii. latrogenic

2. Treatment: Treatment depends upon the underlying etiology, age, and


clinical findings.
a. Akin procedure: for hallux interphalangeus
b. Tendon sling with Modified McBride
c. Lapidus procedure
d. Cotton procedure (sagittal plane opening wedge of medial cuneiform for 1st
metatarsal elevatus)
e. Hemiepiphysiodesis at the proximal phalanx and base of the first metatarsal
f. Proximal 1st metatarsal osteotomies (Austin, Mitchell) good up to 140 IMA
g. Distal 1st metatarsal osteotomies

3. Complications:
a. The most common complication seen with correction of juvenile bunion is
shortening of the first metatarsal. This is a result of interruption of growth of
the physis from
i. Base wedge osteotomies
ii. Epiphysiodesis

NOTE* The extent of shortening is dependent on the amount of dissection about


the physis and the age of the child

b. Avascular necrosis from metatarsal head osteotomies


c. Over or undercorrection
Biomechanical Evaluation of the Child
1. The initial visit:
a. Perinatal history:
i. Was the child full term or premature (premature children are at more risk for
neurological problems)
ii. Standard delivery, breech delivery (dislocated hip), or cesarean section
b. Developmental landmarks:
i. When did the child start to ambulate? (the normal range is 9-16 months) If a
child is delayed in this then you must explore mechanical pathology,
musculoskeletal pathology, and neurologic deficit

2. Gait evaluation:
a. Head tilt: Observe the position of the head in gait. An obvious tilt away from
the midline would indicate a possible limb length deficit
b. Shoulder and pelvis level: If the child walks with an obvious shoulder drop,
one should observe the level of the pelvis.

NOTE* Generally, an apparent shoulder drop in a young child may be


consistent with a shortened limb on that side. In a child under the age of 12 or
13, the shoulder drop is to the shortened side as there is generally no scoliosis
present until that age. This is in contrast to the adult where the shoulder drop
is typically on the contralateral side from the shortened limb

c. Patellar position: An important landmark when exaluating the child with


transverse plane deformities of the legs.

NOTE* If a child has an adducted gait and the patellar position is internally
rotated ("squinting patella") then at least a portion of the deformity is at the
femoral level. If the patellar position is normal, this indicates a problem within
the knee, tibia, or the foot

d. Angle of gait: Typically a child's angle of gait should be more in an external


position. If a child's gait is adducted on the transverse plane because of a
torsional abnormality, that angle of gait is generally consistent. However, some
children demonstrate an angle of gait deformity that varies considerably from
step to step. In these cases, special attention should be directed to the amount
and direction of transverse plane knee rotation present on the off-weight-
bearing examination. In other instances, some children will exhibit an
"adducting" (rather than an adducted) gait at heel contact which is due to tight
medial musculature.
e. Calcaneal position: Most children have a pes planus on weight-bearing. A
child's calcaneus is normally everted at the onset of ambulation which can
persist till the age of 6-7 years old.
NOTE* Calcaneal eversion should reduce approximately 1° per year. Therefore
a child with a calcaneal eversion of 6-7° should reduce to perpendicular by the
age of 7 (a rough estimate)

3. Off-weight-bearing examination:
a. Hip range of motion: External vs. internal rotation measured with the child
supine has about a 2:1 ratio for the first few years. The total ROM measures
more than 1000. Both the amount of external rotation and the total ROM
diminishes over the first few years and essentially becomes symmetrical at the
age of 5-6 through adulthood. This is measured with a Martin's goniometer
placed on the epicondyles of the femur. If a marked amount of internal hip
rotation is noted compared with external rotation, the examination should
include evaluation of motion with the hip extended and hip flexed positions
(determines whether structural vs. positional)
b. Knee range of motion: At birth through 4 years there may be a total of 20-30°
of transverse plane rotation available at the knee. This decreases drastically at
age 3-4. In a normal child under the age of 3 there should be a symmetrical
amount of internal and external rotation of the tibia relative to the femoral
segment. However, if there is 300 of external rotation available from a resting
position with minimal internal rotation this indicates that the tibia is being
maintained in an internally rotated position (congenitally short hamstring or
medial head of the gastrocnemius)
c. Tibial torsion: This is assessed by measuring malleolar position, by
determining the amount of anterior rotation of the tibial malleolus relative to
the fibular malleolus (measured by a goniometer placed on the malleolei or
tractograph placed on the plantar aspect of the foot). At birth there is no tibial
torsion present, however, this gradually increases in an external direction
through the first 7-8 years to a normal adult value of 13-18°. Tibial torsion is
generally 5° greater than malleolar position.
d. Ankle joint dorsiflexion: At birth there is unrestricted ankle joint dorsiflexion
(can approach 75°) , reduces to 20-25° by age 3, reduces to 15° by age 10, and
reduces to 10° by age 15. When obtaining this measurement the subtalar joint
is held in neutral position or the midtarsal joint will unlock thereby introducing
additional forefoot dorsiflexion. If dorsiflexion is limited with the knee extended,
retake the measurement with the knee flexed. If there is limited ankle
dorsiflexion with the knee extended but greater than 150 with the knee flexed,
the child has a gastrocnemius equinus. If the limitation is present with the
knee extended and flexed then a gastroc-soleus equinus is most likely
responsible (an osseous equinus is uncommon in this age group)
e. Subtalar joint range of motion: Since a child under the age of 3 has an
apropulsive gait, measurement of the STJ ROM is not essential. Only after the
age of 3 when a heel-toe propulsive gait is initiated, does a functional orthoses
become useful, and therefore so does the STJ measurement. Rangle of motion
is accomplished by bisecting the calcaneus and measuring full eversion and
inversion. This is generally more than the adult values. The foot must be
measured in a slight dorsiflexed position for accuracy.
f. Midtarsal joint range of motion: The STJ is placed in neutral position and the
MTJ is locked (in this fashion the 2 axes of the MTJ cross each other and limit
extraneous midtarsal joint mobility upon examination). A forefoot varus or
valgus deformity upon examination will not be outgrown, and any degree of
forefoot deformity should be supported to prevent abnormal compensation

4. Weight-bearing examination
a. Relaxed and neutral calcaneal stance position: The calcaneal bisector on
weight bearing should be noted. Abnormal calcaneal eversion in the child can
be due to compensated forefoot varus (frontal plane),
compensated gastrocnemius equinus (sagittal plane), and internal
tibial/femoral torsion (transverse plane).
b. Tibia varum: The angle that the distal one-third of the tibia makes relative to
the ground with the STJ in neutral position is tibial varum (generally 0-5° in
the child)
c. Genu varum/genu valgum, genu recurvatum: The child demonstrates
changes in frontal plane position at the genicular region at different ages . If
excessive genu varum is detected and does not reduce with time, the possibility
of Blount's disease or juvenile rickets should be investigated. If genu valgum is
noted and does not decrease with time, then functional orthoses should be
used to reverse the severe pronatory force to the feet. A posterior deflection of
the femur into the tibia may be present measuring 5-10° until the age of 5. If
any amount greater than this is present before the age of 5, or any amount
present at all after 5 should be checked for pathology
d. Limb length inequality: This is done by palpating the anterior superior iliac
spine, and measuring to the tip of the medial malleolus. Repeat measurement
2-3 times consecutively for consistency. If there is asymmetry in their levels,
further investigation is necessary

Corrective Casting in Infants


1. Application of the cast:
a. Only gentle force is exerted
b. Webril® is utilized and should be no more than 2 layers thick with 3-4 layers
at the posterior-of the heel and at the upper limit of the cast. It should be
wrapped under a slight stretch making a snug fit. It should extend one-half
inch beyond the plaster
c. Use extra-fast setting plaster
d. Babies usually require only 1 roll of 3 inch plaster and 2 inch rolls for
newborns

2. Calcaneovalgus deformity:
a. The assistant grasps the thigh and toes and then holds the hip, knee, and
foot in the anteroposterior plane with the foot in equinus
b. With the foot held in equinus, the metatarsal adducted, apply pressure in
three areas:
i. The posterior aspect of the heel
ii. Over the talus medially
iii. Laterally over the 5th metatarsal area
3. Metatarsus adductus:
a. Failure to appreciate the rearfoot malalignment (talocalcaneal breach on AP
x-ray) will result in failure of treatment and produce a severe flatfoot deformity
b. Pressure should be applied in three areas:
i. Pressure laterally over the cuboid area
ii. Counter pressure medially on the talar head (attempts to close down the T-C
angle)
iii. Pressure along the 1st metatarsal medially
iv. If the hallux is in adduction due to contraction of the abductor hallucis, a
strip of plaster is used to hold the hallux in a rectus or slightly abducted
position

4. Talipes equinovarus:
a. Apply 2-3 casts consecutively,, with the foot in "unlocked" equinus position
b. Apply three point pressure:
i. Evert the heel by pressure medially
ii. Lateral counter-pressure over the talar head in an attempt to open up the T-
C angle (to try to move the calcaneus laterally from beneath the talar head)
iii. Pressure along the 1st metatarsal medially

NOTE* Response to this should be a dramatic improvement in the transverse


and frontal plane alignment. Later casts are applied with the forefoot to
rearfoot in a neutral position and the knee flexed to stretch the posterior
structures (equinus is overcome by pulling down on the posterior of the heel
and applying dorsiflexion pressure, making sure some of the pressure is on the
calcaneocuboid area). Being overzealous will result in a rocker-bottom foot with
the calcaneus in an equinus attitude.

5. Tibial torsion:
a. The cast is applied from the toes to midthigh with the foot in mild equinus,
the knee flexed to at least 300
b. The corrective force is exerted by holding the thigh stable and rotating the
foot to the end range of motion (avoid inversion or eversion while doing this)

The Toe-Walking Child


Toe-walking may be a result of idiosyncratic or idiopathic factors with no
pathological background, or can follow serious neuromuscular,
psychological, and skeletal pathologies. Therefore, the etiology of habitual
toe-walking is determined by exclusion:
1. Examination:
a. Medical history: As above
b. Gait Evaluation:
c. Physical examination:
i. Musculoskeletal
ii. Biomechanical
iii. Neurologic
2. Differential diagnosis:
a. Cerebral palsy: A fixed nonprogressive neurologic deficit acquired before,
during or in the months after birth. The spastic form is most likely to produce a
toe-walking gait
b. Pseudoscissor gait: Must be recognized and differentiated from the scissor
gait of CP. This condition occurs when there is a combination of habitual toe-
walking and adducted limb position due to internal tibial
and/or femoral position. The child with a pseudoscissor gait will have more
stability than a child with a scissors gait
c. Mental retardation: There are behavioral clues to a diagnosis such as
dependency on routine, distractibility, fear, lack of spontaneity, poor judgment,
and repetitive physical activities that are disturbing to others (rocking, head
banging, temper tantrums, etc.)
d. Clumsy child syndrome: A behavioral problem characterized by poor control
of motor function, impulsiveness, short attention span, and hyperkinesis
e. Autism: Autistic children are principally disturbed in their lack of emotional
rapport and in their behavioral characteristics. They show seclusiveness,
irritability when seclusiveness is disturbed, day dreaming, bizarre behavior,
decrease of interest, regression of interpersonal interests, and sensitivity to
criticism. There is a gradual withdrawal from affective contact with people
f. Dystonia musculorum deformans: A disease of the basal ganglia
characterized by slow, powerful twisting and writhing movements
g. Delayed maturation of the corticospinal tracts: Results in spastic toewalking
due to the lack of inhibition of the stretch reflexes. This condition disappears by
the age of 6-8
h. Diastematomyelia: Consists of a partial or complete division of the spinal
cord by tissue located in the midline of the spinal canal. There is a mixed upper
and lower motor neuron deficit involving the bladder and bowel function and
progressive disturbances in gait develop by the age of 2-3
i. Muscular dystrophy (Duchenne and limb-girdle: Toe-walking is the result of a
disturbance of antagonistic balances of the variously affected muscle groups.
With limb-girdle MD the first symptoms appear in the 2nd decade. With
Duchenne's MD the child may walk later than expected with frequent falls
when learning to walk
j. Peroneal muscular atrophy: This disorder begins in the feet and legs
producing difficulty in walking, paresthesias of the legs, and muscle cramps.
There is early weakness of the intrinsic muscles of the feet, ankle dorsiflexors,
and peroneals. The patellar DTR is lost, vibratory and position sense are
diminished
k. Gastrocnemius soleus muscle equinus: The most common entity is the
habitual toe-walking child. The Silferskjold test first used to differentiate
spastic gastrocnemius equinus from spastic gastrosoleus equinus can be
usefully applied to evaluate nonspastic short calf muscles. Clinically, a child
with a gastrocnemius soleus muscle equinus will stand with an abducted
stance angle and will often exhibit a genu recurvatum and significant midtarsal
pronation
1. Clubfoot: A deformity in which the leg and foot are said to resemble a club,
and is associated with congenital hip dislocation, arthrogryphosis multiplex
congenita, spinal abnormalities, and neuromuscular disease. The form of
clubfoot that leads to toe-walking is talipes equinovarus.
a. Shoe therapy: Rigid sole high top shoe
b. Orthoses therapy: Heel lifts, gait plates, and knee-ankle-foot orthoses c.
Short leg casting
c. Auditory feedback: A method of cognitive muscle management
d. Surgical intervention: With a significant structural gastrosoleus muscle
equinus
Chapter 20: Drugs & Drug
interactions
Introduction to Drug Interactions
Antibiotic Drug Interactions
Anticoagulant Drug Interactions
Antidiabetic Drug Interactions
Antihypertensive Drug Interactions
Antipsychotic Drug Interactions
Cardiac Drug Interactions
Diuretic Drug Interactions
NonSteroidal anti-inflammatory Drug Interactions
Oral Contraceptive Drug Interactions
Mechanisms of Drug Interactions Drugs
DRUGS & DRUG INTERACTIONS
Introduction to Drug Interactions
1. Types of drug interactions:
a. Addition (1 + 1 = 2): The response elicited by combined drugs is equal to the
combined responses of the individual drugs
b. Synergism (1 + 1 = 3): The response elicited by combined drugs is greater
than the combined responses of the individual drugs:
c. Potentiation (0 + 1 = 2): A drug which has no effect enhances the effect
of a second drug
d. Antagonism (1 + 1 = 0): A drug inhibits the effect of another drug

2. Mechanisms of drug interactions:


a. Altered absorption
b. Altered metabolism
c. Plasma protein displacement
d. Altered excretion

3. Top 10 interaction drug problem list:


a. Antibiotics
b. Anticoagulants
c. Antidiabetic agents
d. Antipsychotics
e. Beta blockers
f. Digoxin
g. NSAIDS
h. Oral contraceptives

Antibiotic Drug interactions


1. Aminoglycosides with:
a. Cephalosporins (parenteral): Effect is increased nephrotoxicity, mechanism
unknown
b. Penicillins (parenteral): Effect is inactivation of certain aminoglycosides,
mechanism is unknown
c. Loop diuretics: Effect is increased 8th cranial nerve damage with possible
irreversible hearing loss, mechanism is synergistic auditory toxicity

2. Imidazole antifungals (ketoconazole and Itraconazole) with:


a. Seldane: Effect is Cardiotoxicity, arrhythmia, death, mechanism is that
imidazoles exert a quinidine-like effect with prolongation of Q wave on the EKG
b. Hismanal : Same as with Seldane
c. H2 antagonists: Effect is inactivation of the imidazoles, mechanism is
alteration of the pH by the H2 antagonists and antacids
d. Phenytoin: Effect is decreased action of imidazoles, mechanism is increased
metabolism by hydantoin induction of hepatic enzymes
3. Macrolide antibiotics (erythromycin, clarithromycin and clindamycin)
with:
a. Seldane (with erythromycin and clarithromycin): Effect is cardiotoxicity,
arrhythmia, death, mechanism is macrolides exert a quinidine-like effect with
prolongation of Q wave on the EKG
b. Hismanal: Same as with Seldane
c. Theophylline (with erythromycin and clarithromycin): Effect is increased
theophylline toxicity, mechanism is decreased theophylline renal clearance
d. Oral anticoagulants (with erythromycin only): Effect is increased
anticoagulant activity with hemorrhage, mechanism is unknown
e. Kaopectate (with clindamycin only): Effect is decreased absorption of
clindamycin, mechanism is binding absorption of the antibiotic

4. Quinolones with:
a. Multivalent cations (antacids): Effect is decreased effects of the quinolones,
mechanism is chelation binding decreases the GI absorption
b. Theophylline: Effect is increased theophylline toxicity, mechanism is
inhibition of hepatic metabolism of theophylline
c. Carafate: Effect is decreased effects of quinolones, mechanism is chelation
binding decreases GI absorption
d. Caffeine: Effect is increased effect of caffeine

5. Miscellaneous antibiotics:
a. Penicillins (ampicillin) with allopurinol: Effect is markedly increased rate of
ampicillin retention and induced skin rash, mechanism is unknown
b. Metronidazole with alcohol: Effect is a disulfiram-like reaction, mechanism is
metronidazole inhibits aldehyde dehydrogenase

NOTE Disulfiram reaction (with OH) is nausea, vomiting, hypotension, and


possible coma later on

c. Tetracyclines with multivalent cations: Effect is decreased effects of


tetracyclines, mechanism is chelation binding decreases GI absorption
d. Ampicillin with oral anticoagulants: Effect is increased anticoagulant activity

Anticoagulant Drug Interactions


1. Anticoagulants (oral) with:
a. Aspirin: Effect is increased bleeding and hypoprothrombinemic effects of
Warfarin, mechanism is protein binding displacement combined with anti
platelet effects
b. Erythromycin: Effect is increased anticoagulant activity with hemorrhage,
mechanism is unknown
c. Cephalosporins: Effect is augmented Warfarin effects, mechanism is
antiplatelet effects of certain parenteral cephalosporins
d. H2 antagonists:: Effect is increased bleeding with possible hemorrhage,
mechanism is inhibition of hepatic metabolism of Warfarin
e. Phenytoin: Effect is increase in both bleeding and phenytoin activity,
mechanism is unknown
f. Sulfonylureas: Effect is hypoglycemia, mechanism is the hepatic metabolism
of the sulfonylureas is inhibited
g. Barbiturates: Effect is decreased effect of Warfarin, mechanism is barbiturate
induced heptic metbolism of Warfarin
h. Griseofulvin: Effect Is decreased effect of Warfarin, mechanism is unknown

Antidiabetic Drug Interactions


1. Insulin with:
a. Alcohol: Effect is hypoglycemia, mechanism is inhibition of gluconeogenesis
b. Beta Blockers: Effect is hypoglycemia with the masking of hypoglycemic
symptoms, mechanism is that beta blockers blunt sympathetic mediated
responses to hypoglycemia
c. MAO inhibitors: Effect is hypoglycemia, mechanism is inhibition of
gluconeogenesis
d. Salicylates: Effect is hypoglycemia, mechanism is increased circulating
concentration of insulin
e. Steroids/Estrogen/Thyroxin: Effect is increased hypoglycemia

2. Sulfonylureas with:
a. Alcohol: Effect is hypoglycemia and disulfiram-like reactions, mechanism is
decrease elimination and altered metabolism involving aldehyde dehydrogenase
b. Anticoagulants: Effect is hypoglycemia, mechanism is inhibition of hepatic
metabolism of sulfonylureas
c. MAO inhibitors: Effect is hypoglycemia, mechanism is unknown
d. NSAIDS: Effect is hypoglycemia, mechanism is protein binding displacement
e. Sulfinpyrazone: Effect is hypoglycemia, mechanism is inhibition of hepatic
metabolysm of sulfonylureas
f. Thiazide diuretics: Effect is hyperglycemia, mechanism is thiazide diuretics
decrease insulin secretion

Antihypertensive Drug Interactions


1. ACE Inhibitors with:
a. Indomethacin: Effect is decreased hypotensive effect of ACE inhibitor,
mechanism is interference with plasma renin activity and vasopressor response
b. Potassium supplements: Effect is hyperkalemia, mechanism is unknown
c. Capsaicin (Zostrix): Effect is exacerbation of coughing associated with ACE
inhibitor, mechanism is unknown

2. Calcium Channel Blockers (Verapamil and Nifedipine) with:


a. Calcium salts (with Verapamil): Clinical effect of Verapamil is reversed,
mechanism is pharmacologic antagonism
b. Quinidine (with Verapamil): Effect is hypotension
c. H2 antagonists (with Nifedipine): Effect is hypotension, mechanism is
inhibition of hepatic metabolism of nifedipine

Antipsychotic Drug Interactions


1. MAO Inhibitors with:
a. Meperidine: Effect is seizures coma apnea, and death, mechanism is
unknown
b. Tricyclic antidepressants: Effect is seizures, hypotension, coma, and death,
mechanism is unknown
c. Amine containing foods: Effect is hypertensive crisis or stroke, mechanism is
impaired metabolism of tyramine by inhibition of monoamine oxidase

2. Lithium with:
a. Iodide salts: Effect is hypothyroidism or goiter, mechanism is pharmacologic
synergism
b. Thiazide diuretics: Effect is increased lithium toxicity, mechanism is
decreased lithium renal clearance
c. NSAIDS: Effect is increased lithium toxicity, mechanism is decreased lithium
renal clearance

3. Phenothiazines with:
a. Meperidine: Effect is excessive hypotension and sedation, mechanism is
pharmacologic synergism
b. Anticholinergics: Effect is decreased effect of phenothiazines, mechanism is
accelerated metabolism of phenothiazines

Cardiac Drug Interactions


1. Beta Blockers with:
a. Epinephrine: Effect is initial hypertensive episode, followed by bradycardia,
mechanism is beta blockade allowing alpha receptor stimulation increasing BP
which stimulates baroreceptors causing
bradycardia
b. Verapamil: Effect is that the effects of both drugs are increased, mechanism
is pharmacologic synergism
c. Insulin: Effect is prolonged hypoglycemia with masking of hypoglycemic
symptoms (tachycardia), mechanism is that beta blockers blunt sympathetic
mediated responses to hypoglycemia
d. NSAIDS: Effect is impaired antihypertensive effect of beta blockers,
mechanism is NSAIDS inhibit renal prostaglandin synthesis allowing
unopposed vasopressor activity
e. Theophylline: Effect is antagonistic activity increasing airway resistance,
mechanism is pharmacologic antagonism
f. H2 Antagonists: Effect is increased effects of beta blockers, mechanism is H2
antagonists can inhibit first pass hepatic metabolism of beta blockers

2. Digoxin with:
a. Diuretics: Effect is electrolyte disturbances predispose to digitalis-induced
arrhythmias, mechanism is increased loss of potassium affects cardiac muscle
action
b. Verapamil: Effect is increased digoxin toxicity, mechanism is additive effects
c. Erythromycin: Effect is increased digoxin toxicity in about 10% of patients,
mechanism is in about 10% of patients, digoxin is metabolized by GI bacteria
d. Tetracycline: Effect is the same as above, mechanism is the same as above
e. Quinidine: Effect is increased digoxin toxicity, mechanism is reduced renal
and biliary clearance of digoxin
f. Quinine: Effect is same as above, mechanism is same as above

Diuretic Drug Interactions


1. Loop diuretics with:
a. Aminoglycosides: Effect is increased 8th cranial nerve damage with possible
irreversible hearing loss, mechanism is synergistic auditory toxicity b.
Phenytoin: Effect is decreased effect of the diuretic, mechanism is decreased
absorption and increased metabolism

2. Thiazide diuretics with:


a. Lithium: Effect is increased lithium toxicity, mechanism is decreased lithium
renal clearance

3. Potassium sparing diuretics with:


a. Potassium supplements: Effect is hyperkalemia. mechanism is decreased
elimination of the potassium ion

4. Spironolactone with:
a. Salicylates: Effect is blockage of spironolactone-induced diuresis, mechanism
is blockage of the renal tubular secretion

Non Steroidal Anti-Inflammatory Drug


Interactions
1. NSAIDS with:
a. Anticoagulants: Effect is increased bleeding and hypothrombinemic effects of
Warfarin, mechanism is protein binding displacement combined with anti-
platelet effects
b. Beta Blockers: Effect is impaired antihypertensive effect of beta blockers,
mechanism is NSAIDS inhibit renal prostaglandin synthesis allowing
unopposed vasopressor activity
c. Lithium: Effect is increased lithium toxicity, mechanism is decreased lithium
renal clearance
d. ACE inhibitors (Indomethacin): Effect is decreased hypotensive effect of ACE
inhibitors, mechanism is interference with plasma renin activity and
vasopressor response
e. Methotrexate: Effect is increased methotrexate toxicity, mechanism is
decreased renal clearance of methotrexate

Oral Contraceptive Drug Interactions


1. Oral contraceptives with:
a. Phenytoin: Unintended pregnancy
b. Barbiturates: Unintended pregnancy
c. Griseofulvin: Unintended pregnancy
d. Corticosteroids: Effect is therapeutic and adverse actions of corticosteroids
are enhanced
Mechanisms of Drug Interactions
1. Alteration of Intestinal Absorption:
a. Antacids diminish absorption of: (Antacids contain Ca++, Mg++, or
Al +++)
i. Phenylbutazone
ii. Sulfonamides
iii. Barbiturates
iv. Salicylates
v. Oral anticoagulants
vi. Indomethacin
vii. Ampicillin/penicillin
viii. Tetracycline
ix. Cipro®

b. Antacids increase the absorption of:


i. Talwin®
ii. Antihistamines
iii. Phenergan®
iv. Darvon®

c. Drugs that alter vitamin K levels:


i. antibiotics (penicillin, ampicillin, erythromycin, and tetracycline) reduce
vitamin K production, therefore may increase anticoagulant activity.

d. Drugs that alter GI motility:


i. Decrease motility: codeine, morphine, and other opiates
ii. Increase motility: milk of magnesia, cathartics

2. Competition for Plasma binding protein:


a. Oxacillin/cloxacillin/dicloxacillin/nafcillin can displace uric acid which
can precipitate a gouty attack.
b. Dicoumarol can sustain tolbutamide activity (causes severe hypoglycemia)

3. Metabolism or Biotransformation Alteration:


a. Inhibition of metabolism:
i. Anticholinesterases inhibit metabolism of ester-type local anesthetics
resulting in a prolonged effect of the drug.
ii. Oral anticoagulants prolong the effects of tolbutamide and increase the
toxicity of diphenhydramine by inhibiting metabolism.

b. Acceleration of metabolism:
i. Phenobarbital accelerates the metabolism of coumadin, anticoagulants,
steroids, and griseofulvin.

Note* Patients on anticoagulants and barbiturates must be carefully monitored


when the barbiturate is discontinued, as prolonged hemorrhage can result.
4. Alteration of Electrolytes:
a. Hypokalemia may be caused by long term usage of digoxin, thiazide, and
furosemide diuretics, and corticosteroids.
b. Hyperkalemia may result from ACE inhibitors

5. Alteration of Renal Excretion:


a. Probenecid inhibits renal secretion of penicillin and other medications (see
#6, this chapter)
b. Salicylates (in small doses) interfere with the uricoscuric action of probenecid

6. Drugs With Opposing Actions:


a. Thiazide diuretics elevate blood glucose levels. This may partially counteract
the hypoglycemic action of an antidiabetic drug, necessitating adjustment of
the dose.
b. Many diuretics produce hyperuricemia, requiring dose adjustments

7. Drug Interactions That Can Be Life Threatening:


a. Warfarin + phenobarbital
b. MAO inhibitor + wine and cheese (tyramine)
c. Neomycin + succinylcholine
d. Tolbutamide + dicumarol
e. Digitalis + thiazide diuretic
f. Seldane or Hismanil + macrolide antibiotics
g. Seldane or Hismanil + Imidazole antifungal agents

8. Potential Drug Interactions During the Perioperative Period:


a. Increase anesthetic requirements (Halothane/enflurane/isoflurane) on the
following:
i. Patients on MAO inhibitors
ii. Alcoholism
iii. Cocaine

b. Decrease anesthetic requirements:


i. Alcoholism
ii. Alpha-methyldopa
iii. Clonidine
iv. Verapamil

c. Potentiate neuromuscular blockers:


i. Magnesium
ii. Aminoglycoside antibiotics
iii. Lithium
iv. Lidocaine
v. Quinidine

9. Other Drug Interactions:


a. Tagamet®: Interacts with benzodiazepines: sedation may be enhanced
b. Morphine and meperidine: Interacts with CNS depressants: enhanced
sedation
c. Valium®: Interacts with alcohol, antihistamines, barbiturates, narcotics:
increased CNS depression

10. Possible adverse reactions to antibiotics in the elderly:


a. Pen G potassium: Hyperkalemia in patients with renal insufficiency
b. Pen G sodium: Sodium retention, volume overload
c. Carbenicillin: Pulmonary edema, hypokalemia associated with serious
arrhythmias, postop hemorrhage
d. Ticarcillin: Pulmonary edema, neurotoxicity
e. Tetracyclines: Exacerbation of pre-existing abnormal renal function
f. Minocycline: Vestibular disorders (nausea, vomiting, dizziness, vertigo)
g. Cephalothin: Nephrotoxicity, acute renal failure
h. Aminoglycosides: Nephrotoxicity, ototoxicity
i. Clindamycin: Pseudomembranous colitis
j. Chloramphenicol: Bone marrow suppression, aplastic anemia
k. Trimethoprin-sulfamethoxazole: Blood dyscrasias in patients with borderline
folate stores

Drugs
1. Topical antibacterials:
a. Muprocyn (Bactroban®): Effective against gram (+) cocci
b. Iodochloroquine (1 % cream/ointment): Effective against gram (+) and (-)
organisms
c. Gentamycin (Garamycin®): An aminoglycoside effective against gram (+) and
(-) organisms
d. Chloramphenicol (Chloromycetin®): Effective against gram (+) and gram (-)
organisms. Blood dyscrasias have been reported

2. Topical antiseptics:
a. Povidone-lodine (Betadine): Antibacterial and antifungal scrub
b. Chlorhexidine (Hibiclens: Antibacterial and antifungal, and nonallergenic
c. Hexachlorophene (Phisohex: Antibacterial and antifungal, a neurotoxin
(especially in infants)
d. Benzalkonium Cl (Zephiran Chloride®): A quaternary ammonium compound.
Antifungal and antibacterial depending on the concentration
e. Dakin's Solution (sodium hypochlorite): Active ingredient of commercial
liquid bleaches

3. Astringents: Have drying effects secondary to evaporation. Astringents


precipitate protein, thereby decreasing oozing
a. Burrow's solution: 5% aluminum acetate
b. Domeboro's powder (Pedi-boro®/Bluboro®): Aluminum sulfate and calcium
acetate
c. Dalibour solution: Copper and zinc sulfates with camphor
d. Silver nitrate (0.1-0.5%): Also has antiseptic effects
e. Acetic acid (1-2%): May decrease pseudomonas colonization
4. Drugs to treat Rheumatoid Arthritis;
a. Hydroxychloroquine (Plaquenil®): Can cause muscle weakness that can affect
the DTR's
b. Penicillamine (Cuprimine): Cytotoxic and immunosuppressant (cytotoxic to
lymphocytes and can cause bone marrow depression)
c. Gold salts: Toxicity may affect the CBC with a lowering of the Hb, WBC, and
platelets. Skin reaction in sun-exposed areas
i. Parenteral: Solganal®, Myochrysine®
ii. Oral: Auranofin®
d. Cytotoxics/lmmunosuppressants: Cytotoxic to lymphocytes and can cause
bone marrow suppression, making patients more susceptible to infections
i. Methotrexate (Rheumatrex)
ii. Azathioprine (Imuran®) e. Glucocorticoids

5. Adrenocorticosterolds:
a. Control mechanism: Corticotropin (ACTH) released from the anterior
pituitary stimulates the adrenal cortex to secrete glucocorticoids (cortisol) and
other corticosteroids. The release of ACTH is affected by corticotropin releasing
factor (CRF) from the hypothalamus. ACTH and CRF are subject to negative
feedback inhibition by cortisol/glucocorticoids. ACTH is at its highest level in
the early morning
b. Actions/Side effects of glucorticoids: Any dose of glucocorticoids may exert a
physiologic (replacement) or pharmacologic (anti-inflammatory,
immunosuppressive) effect. Adverse effects can result from too rapid
withdrawal, manifesting as acute renal insufficiency. Glucocorticoids inhibit the
early and the late phases of the inflammatory response. The physiologic
antagonism of histamine, prostaglandin, and kinin-induced vasodilation
occurs. There is an inhibition of prostaglandin and leukotriene synthesis. Also,
leukocyte adherence, chemotaxis, and bacteriocidal/fungicidal activities are
reduced. An inhibition of interleukin1 and T-cell lymphokine release occurs:
i. Hepatic gluconeogenesis/glycogen deposition
ii. Suppression of the HPA axis (after 5-7 days of high dose therapy)
iii. Decrease resistance to infection (signs of infection may be masked)
iv. Possibility of perforation in patients with peptic ulceration or
inflammatory bowel -disease
v. Myopathy: characterized by weakness of proximal muscles of the arms and
legs (tendon rupture)
vi. Osteoporosis (postmenopausal females are at greatest risk), fractures,
aseptic necrosis of the femoral head
vii. Fluid and electrolyte imbalances (sodium and water retention, hypokalemia)
viii. May increase blood pressure and exacerbate K+ sensitive arrhythmias
ix. Exacerbation of diabetes mellitus
x. Impaired wound healing and skin fragility
xi. Increased frequency of subcapsular cataracts or glaucoma
xii. With high doses, psychologic disturbances (steroid psychosis) may
occur
c. Agents:
i. Short-acting:
 Betamethasone sodium phosphate: Celestone®,
 Dexamethasone sodium phosphate: Decadron®, Hexadrol®
ii. Long-acting:
 Hydrocortisone:
 Dexamethasone acetate: Decadron-LA®
 Triamcinolone acetonide: Kenalog®
 Methylprednisolone acetate: Depo-Medrol®
iii. Mixtures:
 Betamethasone acetate and phosphate: Celestone Soluspan®

6. Drugs to treat gout:


a. Colchicine: This inhibits PMN migration and phagocytosis of urate crystals,
PMN lactic acid production is reduced. Colchicine is used to treat acute gout. It
can be used IV for a more rapid response when there is gastric intolerance
precluding oral therapy (oral dose is .5-1.2 mg followed by .5-1.2 mg every 1-3
hours until pain is relieved or GI distress develops. No more than 8 mg should
be given) (IV dose is 2 mg administered slowly over 2-5 minutes, followed by .5-
1.0 mg every 6 hours. No more than 4 mg should be given in 24 hours). This
medication inhibits spindle
formation during mitosis, therefore decreases DNA activity (DNA activity is a
source of urates)
b. Uricosuric agents:
i. Probenecid (Benemid®): inhibits renal tubular absorption of urate,
approximately doubling the daily excretion of uric acid ( also inhibits the
tubular excretion of most penicillins/cephalosporins, naproxen, indomethacin,
methotrexate, and oral hypoglycemics

NOTE* In diabetics using probenecid, a false positive test may result for
urinary glucose using Clinitest® reagents may be observed

ii. Sulfinpyrazone (Anturane): similar to probenecid in action


c. Xanthine oxidase inhibitors:
i. Allopurinol (Zyloprim): Inhibits xanthine oxidase which converts xanthine into
uric acid. This drug is contraindicated in an acute attack (acts as a competitive
inhibitor of the enzyme)
d. NSAIDs: Used for symptomatic relief of acute gouty arthritis

7. NSAIDs:
a. Actions:
i. Analgesia
ii. Anti-inflammatory
iii. Antipyresis
iv. Gastric symptoms
v. Inhibition of platelet aggregation (reversible)
vi. Exacerbation of symptoms of aspirin-intolerant patients
vii. Other: CNS symptoms, hepatic injury with elevation of serum
transaminase, cutaneous reactions
b. Indications:
i. Arthritis
ii. Tendonitis
iii. Soft-tissue inflammation
iv. Mild to moderate pain

NOTE* The action of NSAIDs are compared to aspirin in reducing symptoms

c. Groups:
i. Salicylates:
 Diflunisal (Dolobid®)
ii. Propionic acids:
 Ibuprofen (Motrin®)
iii. Acetates:
 Naproxen (Naprosyn®/Anaprox®)
 Fenoprofen (Nalfon®)
 Ketoprofen (Orudis®)
 Etodolac (Lodine®)
 Flurbiprofen (Ansaid®)
iv. Pyrole acetic acids:
 Indomethacin Indocin®)
 Sulindac (Clinoril®)
 Tolmetin (Tolectin®)
 Nabumetone (Relafen®)
v. Oxicams:
 Piroxicam (Feldene®)
vi. Phenylacetic acids
 iclofenac (Voltaren®)
vii. Pyrazoles:
 Phenylbutazone (Butazolidin®)
viii. Fenamates:
 Meclofenamic acid (Meclomen®)

d. Drug interactions with NSAIDs:


i. Anticoagulants: May prolong prothrombin time
ii. ACE inhibitors: Reduce the antihypertensive effect
iii. Beta Blockers: Reduce the antihypertensive effect
iv. Lithium: Lithium levels increased
v. Loop diuretics: Effect of diuretics decreased
vi. Methotrexate: Increased risk of methotrexate toxicity
v. Thiazides: Decreased antihypertensive/diuretic actions
vi. Probenecid: Increased concentrations of NSAIDs

8. Laxatives/Antidiarrheals:
a. Laxatives: Are used for the treatment of constipation secondary to the use of
opiate analgesics for postoperative pain relief. Are classified by differences in
mechanism (active or passive), intensity of effect, and time to the onset of action
i. Passive (24-72 hours): Metamucil®, Colace®, Chronulac®
ii. Active (6-8 hours): Ex-lax®, Senokot®, Dulcolax®
iii. Saline cathartics (30 minutes-3 hours): Magnesium citrate, Milk of
Magnesia®)
iv. Suppositories (15 minutes-1 hour): Glycerin, Dulcolax
b. Antidiarrheals should not be used for the treatment of acute diarrhea
associated with broad-spectrum antibiotics because of possible exacerbation of
pseudomembranous colitis
i. Passive (non-opiate): Kaopectate®, Pepto-Bismol®
ii. Opiates and others (decrease GI motility): Lomotil®, Imodium®

9. Muscle relaxants: Decrease skeletal muscle tone for use in acute muscle
spasms. Can be used after muscle or tendon transfer. All agents cause a degree
of sedation or CNS dysfunction. Most agents are combined with acetominophen
or aspirin
a. Centrally acting agents:
i. Metaxalone (Skelaxin®): contraindicated in patients prone to hemolytic
anemia
ii. Carisoprodol (Soma®): Contraindicated in acute intermittent porphyria
iii. Cyclobenzaprine (Flexeril®): Contraindicated in patients who have cardiac
pathologies or those receiving MAO inhibitors
iv. Orphenadrine (Norflex®): Contraindicated in glaucoma or prostatic
hypertrophy
v. Chlorzoxazone (Paraflex®)

10. Antiemetics: May be required postoperatively to control nausea and


vomiting. Drug-induced vomiting is believed to be mediated by dopamine
impulses within the chemoreceptor trigger zone of the CNS, therefore dopamine
receptor antagonists are generally effective
a. Phenothiazines: These drugs exert dopamine-receptor antagonism, plus
provide variable blockade of alpha-adrenergic (hypotension), chlolinergic
(atropine-like), and histamine receptors. Therefore these drugs should be used
with caution in patients with cardiovascular disease
i. Prochlorperazine (Compazine®)
ii. Chlorpromazine (Thorazine®)
iii. Promethazine (Phenergan)
b. Nonphenothiazines:
i. Trimethobenzamide (Tigan®): Contraindicated in patients who have
hypersensitivity to local anesthetics
ii. Benzquinamide (Emete-Con): May increase blood pressure, especially
in patients receiving epinephrine or other pressure agents
iii. Hydroxyzine (Atarax®/Vistaril®):

11. Opioid analgesics and antagonists: Opioid analgesics exert their actions
by combining with one or more subtypes of opioid receptors within the CNS,
peripheral nervous system, or smooth muscles. The receptors designated mu,
kappa, or sigma are the most significant. Opioid analgesics are classified into 3
groups based on the action at these receptors
12. Morphine/Morphine-like agents/Antagonists: Morphine exerts its
actions largely through agonistic behavior at the mu and kappa receptors
a. Actions:
i. Analgesic response characterized by a reduction in the sensory intensity and
the emotional response to pain
ii. Sedation and euphoria (behavioral)
iii. Depression of autonomic reflexes yields respiratory depression,
pupillary constriction, suppression of cough reflex, orthostatic hypotension
iv. Peripheral effects: constipation, increased urethral tone, weak
bronchoconstriction
b. Precautions:
i. Contraindicated in diarrhea characterized by pseudomembrane formation
ii. Use cautiously in patients with bronchial asthma, chronic obstructive
pulmonary disease or cor pulmonary
c. Drug interactions:
i. May potentially affect other CNS depressants
ii. May cause toxic reactions in patients receiving MAO inhibitors
d. Agonists-Antagonists:
i. Agonists: Dependence/abuse are less with these, and withdrawal symptoms
are less than with morphine-like drugs
ii. Antagonists (Naloxone/Narcan®): this antagonizes the mu, kappa, and sigma
receptors, thereby preventing or reversing opiate-induced respiratory
depression, sedation, hypotension and psychomimetic effects of sigma agonists
such as pentazocine. Should be used to achieve complete or partial reversal of
opiate induced CNS depression and can be used to diagnose suspected opioid
overdose

13. Analgesic combinations: The analgesic effect of the opioids can be


enhanced by the concurrent administration of a non-narcotic analgesic. Also
these may contain caffeine, antacids, or barbiturates
a. Codeine combinations: Tylenol® with codeine, empirin with codeine, APC with
codeine, Fiorinal®
b. Oxycodone combinations: Percodan®, Tylox®, Percocet®
c. Hydrocodone combinations: Vicodin®, Lortab®, Dolo-Pap®
d. Pentazocine combinations: Talwin Compound®, Talacen®
e. Propoxyphene combinations: Darvon® with ASA, Darvon-N® with ASA,
Darvocet-N 50/ N-100®, Darvon Compound®

14. Antidepressants for chronic pain syndromes: Any antidepressant may be


used to treat pain but the heterocyclic compounds are more popular. The
dosages are generally 1 /2 of those used for the treatment of depression
a. Amitryptyline (Elavil®): 25-100 mg
b. Doxepin (Apadin): 25-100 mg
c. Imipramine (Tofranil®): 50-100 mg

15. Antihistamines: Nearly all mammalian tissue contain histamine, with the
highest concentration found in the lungs, skin, and stomach. Histamine is also
stored in the mast cells and in circulating basophils where it is released by
antigenic stimuli or indirectly by organic bases. Responses to stimuli are
mediated by 2 distinct populations called H1 and H2. Antihistamines are H1-
receptor antagonists
a. Actions:
i. Reduce prominent vasodilation, vasodilation, increased microvascular
permeability, and bronchoconstriction caused by histamine
ii. Variable anticholinergic, antipruritic, sedative, and antiemetic effects
iii. Since related to local anesthetics, demonstrate local anesthetic
activity

b. Indications:
I. Adjunct in the treatment of pruritus and allergic reactions ii. Nocturnal leg
cramps
iii. As local anesthesia in patients allergic to amides and ester-linked
compounds
iv. Perioperative medication

16. Anxineytics (Sedative-Hypnotics): Benzodiazepines


a. Major effects:
i. Anxiolytic action (that is distinguished from the effects of reduced
conscious excitability)
ii. Sedation/sleep induction
iii. Skeletal muscle relaxation
iv. Anticonvulsant action
v. Antiemetic effect
b. Indications:
i. Perioperatively, for the relief of anxiety
 Valium®- 10 mg PO or IM
 Ativan®-0.05 mg/kg IM up to 4 mg
 Librium®- 50-100 mg PO or IM 1 hour before surgery
ii. Conscious sedation
iii. Sleep induction:
 Dalmane®- 15-30 mg
 Restoril®- 15-30 mg
 Halcion®- .25-.5 mg
 Ativan®- 2-4 mg
iv. Skeletal muscle relaxation:
 Valium®- 5-10 mg 3-4 times daily
v. Termination of acute seizures
 Valium®- 5-10 mg IV, repeated every 10-15 minutes prn, or 2-3 mg IV
repeated every 5 minutes
c. Precautions:
i. Contraindicated in acute narrow angle glaucoma
ii. IV administration may result in apnea or hypotension
Chapter 21: Surgery of the
Congenital Foot
Flatfoot Surgery (flexible)
Subtalar Joint Blocking Procedures:
(Arthroereisis and Arthrodesis)
Flatfoot Surgery (rigid): Convex Pes Piano Valgus
Metatarsus Adductus Surgery
Cavus Foot Surgery
Clubfoot Surgery
SURGERY OF THE CONGENITAL
FOOT
Flatfoot Surgery (Flexible),
1. Etiology of Flexible Flatfoot (pes planovalgus):
a. Rare syndromes: (Down's, Ehler's-Danlos, Marfan's, and Morquio's
syndromes)
b. Calcaneovalgus: (not all cases)
c. Biomechanical:
i. Forefoot varus
ii. Forefoot valgus
iii. Equinus
iv. Torsional abnormalities
v. Muscle imbalance (weakness of the supinators)
vi. Ligamentous laxity
d. Neurotrophic feet (early stages)
e. Enlarged or accessory navicular
f. Limb length inequality

2. Biomechanical alterations:
a. The subtalar joint is pronated
b. The midtarsal joint becomes unstable and unlocked: this is because the STJ
is in a pronated position with the calcaneus everted, the T-N joint and the C-C
joint become divergent from each other, their axes become more parallel. This
allows for independent range of motion of each of these joints and increases the
range of motion of the MTJ itself. The pronated STJ allows for compromised
function of the peroneus longus and tibialis posterior muscles.
This results in loss of osseous stability as the heel comes off the ground. The
reactive force of gravity produces a dorsiflexory force on the forefoot. The
following are the changes to the foot:
i. Arch fatigue
ii. Hypermobile first metatarsal
iii. Subluxation of the first ray
iv. Contraction of the digits
v. A medial distribution of body weight when the calcaneus everts beyond 4-5°
of eversion.
vi. Collapse of the MTJ
vii. The axis of the STJ in the normal patient is approx. 42° from the transverse
plane 16° from the sagittal plane. Any change in this will result in changes to
motion on the various planes. In clinically examining each patient, it may be
necessary to estimate the primary plane of motion of the STJ to predict the
ability (biomechanically or surgically) to control the STJ. In examining the ROM
of the joints of the foot, the predominant axis of motion can be estimated.
NOTE* IF frontal plane motion (inversion/eversion) is predominant the joint
axis will be more horizontal. If transverse plane motion
(abduction/adduction) is more predominant, the joint axis is more vertical. If
sagittal plane motion (dorsiflexion/plantarflexion) is more predominant, the
axis will lie closer to the frontal and horizontal planes.

3. Radiographic Alterations:
a. If transverse plane dominance:
i. Increase in the dorsoplantar T-C angle
ii. Increase in the cuboid abduction angle
iii. Decrease in the percentage of T-N congruency
b. If frontal plane dominance:
i. Widening of the lesser tarsus on dorsoplantar view
ii. Decrease of the first metatarsal declination angle
iii. Decrease in the height of the sustentaculum tall
c. If sagittal plane dominance:
i. Increase in the talar declination angle
ii. Naviculocuneiform breach
iii. Increased T-C angle on the lateral view
iv. Decreased calcaneal inclination angle
d. The stress dorsiflexion lateral view (charger view) is used to determine osseus
blocks of the ankle joint.
e. Harris-Beath views are helpful in determining T-C coalitions (taken from
posterior and superior with the x-ray beam at 35°, 40°, and 45° to the
perpendicular).

4. Evaluation, Criteria. and Goals:


a. First ascertain the available range of motion: then differentiate rigid vs.
flexible flatfoot.

Note* Procedures that are effective for flexible flatfoot are usually ineffective for
rigid flatfoot, and the foot with bony adaptation secondary to forefoot
varus/supinatus also requires a different approach

b. Determine the planar dominance: because the foot that presents a high
degree of transverse plane motion is extremely difficult to control nonsurgically.
c. Surgery must be avoided in the normal low arched foot (pes planus), which
must be distinguished from the collapsing pes planovalgus deformity as we are
describing.
d. Consider the age of the patient and the percentage of bone growth remaining.
e. Consider the presence of other related medical conditions
f. Consider the presence of other superstructural deforming forces (tibial
torsion)
g. Other surgical criteria for flexible flatfoot:
i. Symptoms are resistant to conservative therapy
ii. The unstable foot is not controllable by mechanical devices
iii. Secondary changes are present or can be definitely predicted
h. Goals of surgery:
i. Relief of pain
ii. Biomechanical control of excessive pronation
iii. Prevention of progression of the deformity

5. Soft Tissue Approaches: (Medial Column Procedures)


a. Kidner procedure:
i. Requires the removal of an accessory navicular (changes the leverage of the
tibialis posterior ms.).
ii. Removal of any hypertrophied tuberosity of the navicular
iii. Transposition of the insertion of the tibialis posterior tendon into the
underside of the navicular.
b. Lowman procedure:
i. Achilles tendon lengthening
ii. T-N wedge arthrodesis
iii. Rerouting the tibialis anterior tendon under the navicular and suture to the
spring ligament.
iv. Tenodesis of the medial arch with a slip of the Achilles tendon, which is left
attached to the calcaneus and folded forward along the medial arch as an
accessory ligament (this helps maintain the calcaneus and forefoot in
adduction).
v. Desmoplasty of the T-N ligaments
c. Young procedure:
i. Tendoachilles lengthening
ii. Rerouting of the tibialis anterior tendon through a slot in the navicular
without detaching the tendon from its insertion.
iii. Tibialis posterior reattachment beneath the navicular
6. Osseous Approaches: (medial column procedures)
a. Hoke arthrodesis:
i. Tendo achilles lengthening
ii. navicular to the medial and intermediate cuneiform

Note * The Hoke procedure is now used as an adjuctive procedure in


combination with a TAL, calcaneal osteotomy, or arthroereisis. It is utilized in
the presence of severe N-C sagging, and is reserved for patients whose bone
growth is complete and when secondary changes. have occurred in a joint

b. T-N arthrodesis:
i. Generally used in combination with other procedures (TAL, calcaneal
osteotomies, medial column tendon balancing
ii. Blocks all MTJ motion and almost all STJ motion
iii. When the T-N joint is wedged the procedure can reduce some
forefoot varus deformity, and can be combined with an Evans calcaneal
osteotomy.
iv. Most useful in degenerative joint changes/severe collapse at the T-N joint,
part of a repair of a ruptured tibialis posterior, and paralytic deformity.
c. Miller procedure:
i. Lengthening of the tendo achilles
ii. Raising an osteoperiosteal flap left in place proximally, along the
medial arch (this includes the spring ligament and tibialis posterior tendon
insertion) and reattaches in an advanced position.
iii. Arthrodesis of the navicular-medial cuneiform joint
iv. Arthrodesis of the 1st metatarsal-medial cuneiform joint
d. Osteotomies of the talus:
i. Stokes:
ii. Perthes: closing wedge at the talar neck
e. Osteotomies of the medial cuneiform:
i. Anderson and Fowler: plantar flexory wedge in conjunction with an Evans
calcaneal osteotomy.
ii. Cotton: opening wedge to produce plantarflexion of the medial column

7. Osseous Approaches: Calcaneal osteotomies (3 types): Calcaneal


osteotomies allow some margin for error, because joint motion is still present.
You must be specific in choosing procedures for the appropriate problem. You
must correct the etiology, not the symptoms. During the procedure, dorsiflex
the foot and make sure it is in neutral in all planes before being satisfied on
wedge size and fixation. These procedures are basically designed to replace the
triple arthrodesis and allow maintenance of joint motion after the foot is
corrected.
a. Extra-articular:
i. Chambers:
 A procedure to limit STJ motion by placing a bone graft under the sinus
tarsi (similar to arthroereisis)
ii. Baker-Hill: (to reduce heel valgus and excessive pronation)
 A refined Chambers concept for use in patients with CP
 They used a vertical-lateral approach to perform a horizontal osteotomy
inferior to the posterior facet of the STJ
 A wedge shaped graft is inserted.
iii. Selakovich:
 Through a medial approach performing an osteotomy and grafting of the
sustentaculum tali
 Tightening of the spring ligament
 Repositioning of the tibialis posterior
 Transfer of all/part of the tibialis anterior into the navicular

b. Anterior:
i. Evans (refined by Ganley): good when the forefoot abducts severely when the
STJ is in neutral. This procedure is contraindicated in neurological disorders
that may generate spasticity and varus due to functional overcorrection. The
rationale for this procedure is that as the lateral column is lengthened, the
entire forefoot is forced to pivot around the head of the talus, effectively
adducting the forefoot and tightening the structures of the arch

NOTE* If there is excess lengthening of the lateral column, an equinus may be


produced (talus abducts and dorsiflexes)

 Due to scarring the incision has been changed, to one parallel to the skin
tension lines in an oblique/transverse fashion on the lateral side of the foot
over the C-C joint (avoid the sural nerve inferiorly and the intermediate
dorsal cutaneous nerve superiorly; they are found at the extreme poles of the
incision)
 The peroneals are retracted inferiorly
 Reflection of the EDB ms. dorsally while preserving the dorsal
calcaneocuboid ligament

NOTE* This ligament is critical in limiting the dorsal shift of the anterior beak of
the calcaneus

 Osteotomy of the calcaneus parallel and 1.5 cm proximal to the C-C joint
from lateral to medial
 insertion of tibial bone graft or bone bank iliac crest, which is tapped into
position, and any void filled with bone chips
 Suture the EDB back into place: impossible to achieve good closure of this
muscle layer

Note* This procedure straightens the lateral column with reduction of heel
valgus. In most cases it is still necessary to reduce the amount of forefoot
varus and stabilize the medial column by doing a T-N fusion, wedge osteotomy
of the cuneiform, naviculocuneiform fusion, or medial arch tenosuspension
(predominant)

 Complications include undercorrection/overcorrection, delayed/nonunion,


and if there is a met. adductus present It will be made worse.

Note* One of the most difficult pes planovalgus foot types to treat is one with a
vertical STJ axis. This foot compensates for deforming forces mainly in the
transverse plane. This foot is recalcitrant to mechanical control and medial
column procedures. Lateral column lengthening procedures are indicated
(Evans procedure)

 When the Evans procedure is done in conjunction with a medial arch


tenosuspension the cuboid abduction angle is decreased, the forefoot
abduction angle is decreased by an average of 170 and the calcaneal
inclination angle is increased by 7°.
c. Posterior (varus producing osteotomies): Are designed to place the weight-
bearing surface of the calcaneus in neutral or varus, preserve STJ motion while
changing the ratio of available inversion/eversion
i. Gleich
 Oblique calcaneal osteotomy displaced anteriorly, helps increase the
calcaneal inclination angle
ii. Dwyer
 Most commonly performed as an opening wedge (can be closing wedge also)
from the lateral side
 Slightly overcorrect with the osteotomy
iii. Silver
 Lateral opening wedge osteotomy with the direction from just posterior to the
posterior facet running inferiorly.
iv. Koutsogiannis
 Lateral approach with a transection osteotomy of the calcaneus (oblique) -
Posterior fragment is medially displaced until it lies below the
sustentaculum tali

Note* Posterior calcaneal osteotomies are most useful in the least prevalent
type of pes planovalgus, where there is frontal plane dominance. These
osteotomies are most useful in conjunction with medial column procedures

v. Additional procedures:
 Sullivan (a dorsal anterior sliding osteotomy of the posterior aspect of the
calcaneus to treat equinus)
 Moeller (triplane closing wedge osteotomy of the lateral cortex, for cavus foot)
 Keck and Kelly (dorsal closing wedge osteotomy just anterior to the achilles;
it will decrease the inclination angle and remove pressure from a Haglund's
deformity)
 Reinhart (for pes cavus deformity where a through and through osteotomy is
done)

8. Ancillary Procedures:
a. Tendo Achilles lengthening: is indicated in almost all of the above flatfoot
procedures

9. Rearfoot Arthrodesis: Is usually used with severe DJD, severe triplane


deformity with pain, paralytic deformity, or for long-standing rupture of the
tibialis posterior, with collapse of the foot.
a. Subtalar arthrodesis: Restores the appropriate T-C relationship while
preserving midtarsal motion
i. The procedure involves resection of the sinus tarsi with packing with
autologous bone chips, and screw fixation.
ii. Objections to this procedure include:
 Fusion of one portion of the STJ results in DJD of the other joints
 No correction occurs in the forefoot which is usually in varus
 Potential for fatigue failure of the screw
 Long term loss of the correction

b. Triple arthrodesis: Because this is a long-standing and reliable procedure,


it will be discussed in great detail.
i. Definition: fusion of the T-C, C-C, and T-N joints
ii. History: originally used for paralytic deformities, modified by Ryerson in
1923 with a two incision approach/internal fixation/above knee casting.
iii. Indications: pain, instability, structural deformity (rigidity)
iv. Specific etiological conditions:
 Valgus foot deformity
 Collapsing pes valgus deformity
 Ruptured posterior tibialis tendon
 Tarsal coalitions
 Tarsal arthritis
 Cavus foot
 Talipes equinovarus
vi. Preoperative considerations:
 Procedure must be delayed until the patient reaches skeletal maturity
 Patient must have adequate ankle dorsiflexion available
 Due to the amount of dissection prophylactic antibiotics should be
considered
 Hemostasis necessary during the procedure
v. Surgery:
 The dissection is started laterally passing between the sural and the
intermediate dorsal cutaneous nerves. The contents of the sinus tarsi are
vacated, the extensor digitorum brevis muscle is reflected distally, and the
peroneals are mobilized and protected from the lateral surface of the
calcaneus. This gives exposure to the posterior facet, the C-C joint, and the
lateral aspect of the T-N joint. The medial incision is dissected down to the
dorsomedial aspect of the T-N joint. The periosteum and posterior tibial
tendon are dissected inferiorly off the navicular, and the periosteum is
reflected off the dorsal surface of the navicular and the head and neck of the
talus until it connects with the lateral incision. If a subtalar wedge is to be
taken, the medial incision dissection is carried posteriorly, reflecting the
periosteum and the deltoid ligament of the sustentaculum tali, exposing the
anterior and middle facets.
 A minimal amount of bone is resected off all joint surfaces. The MTJ -is
resected first, which relaxes tissues and makes it easier to manipulate the
foot. Any wedging of the MTJ should be done after the STJ is resected and
temporarily fixed. In any wedging of the STJ, the most bone should come off
the calcaneus.
 The rearfoot and forefoot must be fixed in slight valgus, because a valgus
foot can be accommodated to be comfortable and a varus foot can rarely be
made comfortable. If inadequate dorsiflexion is available at the ankle then
jamming of the anterior ankle will occur causing development of chronic
synovitis and degenerative destruction of the foot. Too much resection
posterior to the STJ will cause dorsiflexion of the ankle joint and jamming.
Anterior displacement of the talus on the calcaneus will plantarflex the
forefoot, while posterior displacement of the talus will dorsiflex the forefoot.
Plantarflexion of the forefoot will cause more ankle joint dorsiflexion and
jamming of the ankle. The forefoot can be rotated on the rearfoot to
accommodate frontal plane problems.
 Temporary fixation is achieved with K-wires or Steinmann pins. The STJ is
fixated first. Rigid compression fixation gives more constant joint fusion. A
6.5mm cancellous or cannulated screw is used to fixate the STJ. Using
screws in the MTJ is difficult due to the angulation, and so staples may be
used for fixation of the T-N and C-C joints. Fluoroscopy or intraoperative x-
rays must be taken to assure proper position.
vi. Postoperative management:
 Jones compression cast for 2-3 days
 Drain pulled at 48-72 hours
 A B-K NWB cast is applied for 8 weeks
 Change cast and suture removal at 2-3 weeks
 Change cast every 2-3 weeks thereafter
 Take serial x-rays to evaluate healing every 4 weeks
 A B-K WB cast is then used for 4 weeks
 Physical therapy continued for 3 months
 Goal is a return to normal function 6 months postoperatively
vii. Complications:
 Fracture
 Wound dehiscence
 Peroneal tendonitis
 Entrapment neuropathy
 Nonunion
viii. Postoperative gait pattern:
 Abducted gait
 Shorter stride
 Difficulty in going down stairs

Subtalar Joint Blocking Procedures (Arthroereisis


and Arthrodesis) for Flatfoot
There are 2 major categories: extra-articular arthrodesis which actually fuses
the joint by means of a bone graft eliminating all motion, and arthroereisis
which limits excessive valgus motion of the STJ and retains the varus range of
motion.

1. Arthrodesis:
a. Grice Green (EASTA: extra-articular subtalar joint arthrodesis): This
procedure allows you to fuse the STJ without disturbing growth.
i. Indications
 Paralytic instability and equinovalgus
 Peroneal spastic flatfoot
 Unresectable tarsal coalitions
 Age bracket 3-14 years old
ii. Contraindications
 Ankle valgus flatfoot
iii. Procedure
 Use corticocancellous graft placed at 900 to the STJ axis

2. Arthroereisis:
a. Indications
i. To see if a patient needs an arthroereisis: have the patient stand on the toes,
the heel must supinate
ii. Age is 4-8 years
iii. Cavovalgus foot
iv. A foot that has been unresponsive to treatment for 2 years
v. Eversion of the heel at least 8°
vi. Predominant frontal plane deformity
vii. Flexible forefoot varus deformity above 10°
b. Contraindications
i. Rigid flatfoot
ii. Significant arthritis/trackbound tarsal joints
iii. Ankle valgus
iv Equinus (must be released)
v. Skewfoot
vi. Torsional problems
vii. Frontal plane knee deformity
c. 3 types
i. Self-locking wedge
 Viladot
 Valenti
 Addante
 Valgus stop
ii. Axis altering (not for adults): elevates a low STJ axis
 STA-peg
iii. Direct impact (can be used for all ages)
 Sgarlato
 Pisani
d. Anatomical placement
i. Sinus tarsi
 Valgus stop
 STA-peg
 Sgarlato
ii. Canalis tarsi
 Valenti
 Viladot
e. Postoperative care
i. Cast for 2 weeks
ii. Orthoses and high top sneakers for 1 year
f. Complications
i. Extrusion of the implant
ii. Fracture of the implant
iii. Fracture of the calcaneus
iv. Improper placement of the implant
v. Over/undercorrection
vi. Infection
vii. Erosion of the bone-implant interface

Flatfoot Surgery (rigid): Convex Pes Planovalgus


1. Etiology of Rigid Flatfoot:
a. Vertical talus
b. Congenital T-N dislocation
c. Arthrogryphosis
d. Tarsal coalition/peroneal spastic flatfoot
e. Tarsal arthrosis caused by trauma
f. Cerebral palsy
g. Spina bifida
h. Improper correction of clubfoot
i. Post-traumatic
j. Neurotrophic (late stages)
2. Clinical Presentation of Convex Pes Planovalgus (vertical Talus): (see
pediatrics)
a. Rocker-bottom deformity with prominent talar head bulge on the medial and
plantar aspect of the foot.
b. The forefoot may actually touch the anterior surface of the tibia
c. Valgus rearfoot that is in equinus

3. Radiographic Presentation of Convex Pes Planovalgus:


a. Definitive diagnosis when it is evident that the tarsal navicular is dorsally
dislocated on the neck of the talus even when the foot is maintained in a stress
plantarflexed attitude (forced plantarflexion and inversion will not reduce the
dislocation of the T-N joint in a true convex pes piano valgus deformity).
b. Talus is vertical, lying parallel to the longitudinal axis of the tibia
c. Calcaneus is in an equinus position
d. T-C angle is abnormally increased on the D-P view

4. Pathology of Convex Pes Planovalgus:


a. Severe dislocation of the T-N joint (navicular is articulating with the dorsal
aspect of the talus
b. Neck of the talus is hypoplastic
c. Talar head is flattened
d. Calcaneus is displaced posteriolaterally
e. Calcaneus is convex on its plantar surface
f. The tibialis posterior and peroneals are located anterior to their normal
position and are contracted
g. The tibialis anterior, EHL, EDL, and the triceps surae are contracted
h. The tibionavicular ligament and dorsal talonavicular ligament are contracted
(the problem in the repair of this foot type)

Surgery of the Convex Pes Planovalgus Foot: Surgery is dependent upon


patient's age, clinical and radiographic features, type and degree of previous
treatment, and experience of the surgeon. The common objective of all
procedures is release of all soft tissue contractures, establishment of a rectus
forefoot to rearfoot relationship, and the production of equal medial and lateral
foot columns.
a. Talar procedures: resect the head and neck of the talus
b. Navicular procedures: naviculectomy
c. Tendon transfers: transfer of the tibialis anterior/posterior and peroneals
d. Open surgical reduction: T-N joint reduction along with a peritalar release,
heel cord lengthening, extensor and peroneal tendon lengthening, STJ and
ankle joint capsulotomy, calcaneocuboid joint reduction, and transfer of the
tibialis anterior into the navicular (K-wires removed at 6 weeks and A-K cast
removed at 3 months)

Metatarsus Adductus
1. Indications
a. Failure to respond to conservative treatment
b. Residual deformity after treatment of talipes equinovarus
c. Newly diagnosed metatarsus adductus deformity

2. Considerations: (see section Pediatrics)


a. Age of patient
b. Osseous development
c. Severity of deformity
d. Presence of concomitant deformities
e. Extent of malfunction and disability

3. Soft Tissue Surgery:


a. Heyman, Herndon, and Strong:
i. Indications:
 For flexible met. adductus which is reducible on manipulation (stress x-ray)
 Usually children less than 5 years old
 Deformity present at Lisfranc's joint, without significant bowing present in
the proximal portion of the metatarsal bones themselves ii. Procedure:
 2 or 3 longitudinal dorsal incisions, or a transverse incision
 Release of the dorsal, interossei, and plantar ligaments of the
tarsometatarsal joints and intermetatarsal joints
 Preserve the plantar-lateral ligaments, especially 5th metatarsocuboid
articulation and the peroneus brevis tendon
 Manipulate the foot into abduction
 K-wire fixation of the first met-cuneiform joint and 5th met-cuboid joint
 Release of the naviculocuneiform and intercuneiform joints is rarely needed
 Consider abductor hallucis release or tenotomy in conjunction with HH&S
iii. Precautions
 Avoid damage to the 1st metatarsal epiphyseal growth plate (do not confuse
this with the met-cuneiform joint)
 Be careful not to introduce latrogenic dorsal dislocations at the met-
cuneiform joints
iv. Postop care
 Cast for 6-12 weeks
 Manipulate the foot and recast every 3-4 weeks depending upon the severity
 Monitor the foot carefully for the development of a flatfoot deformity
v. Complications
 Dorsal dislocation
 Degenerative arthritis
 Damage to the growth plates

b. Thompson procedure (modified):


i. Indications
 Congenital hallux varus primarily
 Flexible met. adductus secondarily
 Hyperactivity of the abductor hallucis ms.
ii. Procedure
 Medial longitudinal 1st m.p.j. skin incision approach
 Dissection to level of deep fascia over the abductor hallucis muscle
 Transection of the abductor hallucis tendon with resection of a segment of
the tendon and portion of the distal muscle
 Consider lesser m.p.j. release medially if lesser digits are also adducted
 Release of the medial head of the flexor hallucis brevis if adduction of the
hallux is still present
iii. Precautions
 Do not reduce varus of the hallux without insuring correction of any
adduction deformity of the first metatarsal
 Place the medial incision over the 1st m.p.j. strategically; if too superior or
inferior, may damage the medial neurovascular bundle
 Avoid the procedure as a primary mode of correction for met. adductus
unless clinical findings and x-rays strongly support hyperactivity of the
abductor hallucis as the primary etiology
iv. Postoperative care
 Weightbearing in a surgical shoe for 3-6 weeks
 Splinting of the hallux and the first ray
v. Complications
 Hallux abductovalgus
 Hallux hammertoe (hallux malleus)

c. Johnson osteochondrotomy: cartilaginous procedure


i. Indications
 Met. adductus deformity in children between the ages of 5-8 years (can be
younger)
ii. Procedure
 3 dorsal longitudinal incisions
 Closing abductory base wedge osteotomy of the 1st metatarsal
 Wedge resection of cartilage and bone from the bases of the lesser
metatarsals, distal to the proximal articular surface (base is lateral with the
apex medial)
 Fixation of the osteotomies with stainless steel wire, k-wires, or staples
iii. Precautions
 Avoid damage to the epiphyseal growth plate of the 1st metatarsal
 Overcorrection/undercorrection of individual ray segments
iv. Postoperative care
 Non-weightbearing with cast immobilization for 6-8 weeks
 Serial x-rays to assess healing

4. Osseous Surgery:
a. Modified Berman-Gartland procedure:
i. Indications
 Met. adductus in the child older than 6-8 years old
 Residual deformity following treatment of talipes equinovarus
ii. Procedure
 3 dorsal longitudinal incisions
 Transverse or oblique-type closing abductory wedge osteotomy of the 1st
metatarsal
 Similar type of osteotomies of the lesser metatarsals with the cortical hinge
medially
 Fixation of osteotomies with SS wire, K-wires, staples, AO screws or
combinations
iii. Precautions
 Avoid damage to growth plate of 1st metatarsal
 Meticulous subperiosteal dissection is critical to avoid heavy callus
formation and undesirable synostosis between adjacent metatarsals
 Preservation of the medial cortical hinge is important to insure stability
 Careful planning to avoid over/undercorrection
iv. Postoperative care
 Non-weightbearing cast immobilization 6-8 weeks
 Convert the cast to posterior splint and start PT
 Orthotics when patient resumes weightbearing
 Serial x-rays to assess bone healing at 6 weeks, 12 weeks, 24 weeks and 1
year
v. Complications
 Over/undercorrection
 Delayed union/nonunion/pseudoarthrosis
 Fracture of cortical hinge
 Damage to growth plate
 Elevatus of metatarsals
 latrogenically induced flatfoot deformity
b. Lepird procedure:
i. Indications
 Met. adductus in the child greater than 6-8 years old
 Residual talipes equinovarus deformity
ii. Procedure
 3 dorsal longitudinal incisions
 Oblique closing-abductory wedge osteotomy (Juvara type) of the 1st
metatarsal with AO/ASIF screw fixation
 Rotational osteotomy of each lesser metatarsal with AO/ASIF screw fixation
(2.7 mm cortical used mostly) perpendicular to the plane of the osteotomy
 An oblique closing wedge osteotomy may be used on the 5th metatarsal in
place of the rotational type (if preferred)
 Rotational osteotomies are performed from dorsal-distal to plantarproximal
with temporary preservation of the cortical hinge (facilitates fixation). The
osteotomy is approximately 45° from the weightbearing surface. The precise
angle will depend on the declination of the metatarsal segment. As the
declination of the metatarsal increases, the osteotomy will be more parallel
to the weightbearing surface of the foot
 Area of the cortical hinge preserved is most commonly proximal/plantar
 The screws are then removed and the osteotomy is completed
 The screws are reinserted, the distal fragments are rotated laterally, and the
screws are tightened
 The alignment of the foot is assessed; if realignment is necessary the
screw(s) can be loosened and the bone adjusted
iii. Postoperative care
 Same as Berman-Gartland
iv. Complications
 Same as Berman-Gartland
 if the osteotomy is performed too vertically the rotation of the osteotomy will
be around the longitudinal axis of the metatarsal bone itself, resulting in
inversion/eversion of the bone itself
v. Advantages
 This procedure is amenable to rigid internal fixation and primary bone
healing
 Over/undercorrection can be corrected during surgery
 Biplanar correction can be achieved
 Eliminates pin tract infections

5. Ancillary Procedures:
a. Equinus Deformity
i. TAL
ii. Gastrocnemius recession
b. Flatfoot Deformity:
i. STJ arthroereisis
ii. Evans calcaneal osteotomy
iii. Modified Young's tenosuspension/ Modified Kidner procedure
iv. T-N joint arthrodesis/ N-C arthrodesis

Cavus Foot Type


1. Description: Pes cavus is primarily a sagittal plane deformity and can best
be described by the area of involvement.
a. Anterior pes cavus: Types
i. Metatarsus cavus (at Lisfranc's joint)
ii. Lesser tarsal cavus (lesser tarsus)
iii. Forefoot cavus (Chopart's joint)
iv. Combined anterior cavus (occurring at 2 or more of the aforementioned
areas)
b. Rearfoot cavus: may not truly be a separate cavus deformity. It may be a
compensation for anterior cavus. It etiology is:
i. Pseudoequinus: reverse buckling of the ankle joint
ii. Muscle weakness/spasticity
iii. Congenital
c. Combined: both rearfoot and forefoot

Note* The apex of the deformity should be located on a lateral x-ray

2. Etiology:
a. Neuromuscular: 66%

Note* Because there is such a high correlation between neuromuscular disease


and pes cavus, and because the cavus foot is often an early manifestation of
such disease, a neurology consult is mandatory prior to any surgical
intervention

i. Muscle lesion: muscular dystrophy


ii. Peripheral nerve lesion: Charcot-Marie-Tooth, polyneuritis, traumatic
lesion
iii. Spinocerebellar tract: Friedreichs ataxia
iv. Anterior horn cell: poliomyelitis, cord tumors
v. Pyramidal/extrapyramidal: cerebral palsy
vi. Cerebral cortex: hysteria
b. Congenital
i. Pes arcuatus (rare)
ii. Spina bifida
iii. Myelomeningocele
iv. Clubfoot
v. Congenital syphilis
c. Idiopathic: 33%
i. Trauma
ii. Infection
iii. Ledderhose's disease
iv. Spinal cord tumors

3. Classification:
a. Flexible deformity (mild): Non-weight bearing, contracted digits, high arch
and varus deformity of the heel may be noticed. With loading, digits appear
normal, arch is flattened and heel may go into valgus. Minimal clinical
symptoms at this point
b. Semi-rigid deformity (moderate): Weight-bearing does not completely reduce
the contracture of the digits, arch appears higher and heel is in more varus
attitude. Soft tissue contractures and bony adaptation begin to take place.
Symptoms are more prominent.
c. Rigid deformity (severe): Joint motion is limited. The foot is similar in
appearance both weight/non-weight bearing. Digits are contracted dorsally and
painful keratomas are present. Difficult to fit shoes.
d. Progression of deformity: Often progresses from flexible to rigid as the patient
gets older.

4. Compensation for Anterior Cavus:


a. Retraction of the toes at the m-p joints (due to extensor substitution)
b. Reverse bucking of the m-p joints
c. Forefoot reduction of the "flexible" anterior cavus
d. Ankle joint dorsiflexion

5. Associated Conditions:
a. Forefoot varus
b. Forefoot valgus
c. Plantarflexed 1st ray
d. Metatarsus adductus
e. Rearfoot pseudoequinus
f. Rearfoot varus

6. Principles of Surgical Judgement:


a. The presence of neurological disease will dictate what types of procedures
you can and cannot do. You must determine whether the disease is progressive
b. For idiopathic pes cavus the following flow chart from McGlamry ED (ed)
Comprehensive Textbook of Foot Surgery , 2 ed, Williams & Wilkins, (with
permission) illustrates the surgical decision making plan best.
c. You must determine whether the deformity is flexible or rigid and follow the
preceding flow chart
d. You must consider the age of the patient (soft tissue procedures are best in
the child)

7. Preoperative Evaluation:
a. Neurology consult
b. Spinal x-rays
c. EMG's and nerve conduction studies
d. X-rays of the foot (lateral)
i. Calcaneal inclination angle > 30°
ii. Talar-1st metatarsal angle > 6° (Meary's angle)
iii. Increased pitch of the 5th metatarsal
iv. Note apex of the deformity
v. Cyma line broken posteriorly (can be normal)
vi. Sinus tarsi is clear and accentuated
vii. Coalition views should be used to assess the subtalar joint
viii. An axial view of the calcaneal angle should be taken to rule out a structural
varus
8. Pathogenesis:
a. With weak tibialis anterior the long extensors substitute for ankle
dorsiflexion, causing hyperextension of the MPJ's and retrograde pressure on
the metatarsal heads (extensor substitution)
b. Weak peroneus brevis and strong posterior tibial ms. will create a varus heel
c. Paralysis of intrinsics causes the development of clawtoes
d. Weak gastrocnemius causes flexor substitution which causes clawtoes.
9. Surgery: Soft tissue
a. Plantar release
i. Subcutaneous fasciotomy
ii. Steindler stripping: through a medial incision the abductor hallucis, -flexor
digitorum brevis, and the abductor digiti quinti are stripped from the
periosteum of the calcaneus. The plantar fascia is released and the long plantar
ligament is released.
iii. Tachdjian: describes a plantar medial approach used with a fixed anterior
cavus. Includes releasing the long/short plantar ligaments, spring ligament
(calcaneonavicular), the calcaneonavicular portion of the bifurcate ligament,
and the plantar fascia. Lengthening of the long flexors and the tibialis posterior
is performed if on intraoperative reduction of the cavus foot bow-stringing of
these tendons is noted
iv. Complications:
 Plantar fasciitis
 Forefoot may become splayed

(Note * These procedures are followed by serial casting

b. Tendon transfers (give best results when the patient is > 10 years old)
i. Jones suspension: transfer of the extensor digitorum longus to the
neck of the 1st metatarsal with i.p.j. fusion
Complications:
 Transfer lesion to the 2nd metatarsal if 1st ray is raised too high
 Hallux limitus
 Tendon may not hold the correction
 Failure of the fusion site
ii. Heyman procedure: transfer of all five extensor tendons to their
respective metatarsal heads.
Complications:
 Clawtoes may result
 Tendonitis of the EDL
 Tendons may not hold the correction
 Same as Jones complications
iii. Hibbs procedure (modified): transfer of the long extensors to the
second and third cuneiform, a Jones transfer of the EHL, anastomosis of
the distal stumps of the long extensors to the EDB tendons. This procedure
helps load the midtarsal joint in dorsiflexion.
Note* Although the Hibbs procedure is a classic procedure described for the
correction of anterior cavus, it should not be performed for pes cavus, because
it fails to provide the dynamic force necessary to elevate the metatarsal heads

iv. Split tibialis anterior tendon transfer (STATT): the lateral half of the
tibialis anterior tendon is sectioned and anastomosed to the peroneus tertius
tendon near its insertion into the base of the 5th metatarsal.
v. Peroneus longus tendon transfer
vi. Tibialis posterior tendon transfer

Note* The above procedures are utilized with flexible deformities

10. Surgery: Osseous

a. Cole procedure: dorsal wedge tarsal osteotomy to reduce a fixed anterior


cavus, which extends from the cuboid laterally through the
naviculocuneiform joints medially. The width of the wedge is determined by the
severity of the deformity.
Note* The disadvantage of the Cole procedure is that it can result in a shorter,
wider, and thicker foot

b. Japas procedure: a midtarsal V-osteotomy (apex of the V is proximal and at


the highest point of the cavus). The lateral limb of the V extends through the
cuboid, and the medial limb of the V extends through the cuneiform

Note* Difficult to control the amount of correction


Note* The primary difference in indications between the Cole and Japas is that
the Cole is for more severe deformities and is only performed in the skeletally
mature foot

c. 1st ray Dorsal Wedge Flexory Osteotomy


d. DWFO all metatarsals
Complications:
i. If 1st metatarsal is done alone, may get transfer lesion to 2nd
ii. May decrease ROM of 1st MPJ
iii. May get a metatarsal on different planes
e. Truncated tarsometatarsal wedge osteotomies: first described by Jahass,
excision of a truncated wedge of bone. across the tarsometatarsal articulations.
This procedure is contraindicated in subtalar joint abnormalities, moderate-
severe rearfoot varus, or muscular imbalance secondary to Charcot-Marie-
Tooth disease
f. McElvenny-Caldwell procedure: elevation of the first metatarsal by fusing
the 1st met-cuneiform joint. If the deformity is too severe then fusion of the N-C
joint is added.
g. Calcaneal osteotomies (Dwyer: opening or closing and biplane):

Note* The biplane osteotomy permits reduction of the calcaneal inclination


angle or sagittal plane deformity, as well as frontal plane varus

h. Triple arthrodesis: previously described under Section: Flatfoot Deformity


i. Hoke procedure: a combination of subtalar arthrodesis with a
resection/reshaping/reimplantation of the head and neck of the talus
j. Dunn procedure: a tarsal arthrodesis that obtained posterior displacement of
the foot by excising the navicular and part of the head and neck of the talus

Note* The Dunn and Hoke procedures are useful when posterior displacement
of the calcaneus beneath the talus is needed

Talipes Equinovarus (clubfoot)


The reason to treat a clubfoot is to obtain a pliable, plantargrade, cosmetically
acceptable foot in a short treatment time with minimal risk. The indications for
operative treatment are incomplete correction of the varus and equinus
components, and you may see the bony pathology progressing if continued
conservative treatment is followed. When surgery is done early there is less
deformity to the talus
1. Types:
a. Rigid
b. Non-rigid
c. Neural component foot with joint problems

Idiopathic: only the foot is deformed, the musculoskeletal system is otherwise


normal. In this foot type you will probably go on to surgical intervention
before a non-idiopathic foot since the disorder should be known and the
progression of the disease should be known. You should wait to treat this
foot till the full deformity has developed as the treatment is more effective

Non-idiopathic: the deformity is a local manifestation of a systemic skeletal


syndrome. The foot deformity and associated skeletal anomalies are due to
the same etiologic factors that caused the failure of normal musculoskeletal
development. Can be caused by:
a. Congenital diseases: congenital constricting band syndrome (Streeter
disease), hereditary onycho-osteodysplasia (nail-patella syndrome),
arthrogryphosis multiplex congenita
b. Neurologic diseases: meningomyelocele, spina bifida, hydrocephalus, CP
c. Myopathy: muscular dystrophy

2. Etiology:
a. Unknown- but many theories (see Chapter: Pediatrics)

3. Anatomical Presentation:
a. The talus is supinated over the top of the distal calcaneus so that the
calcaneus is plantarflexed and inverted
b. Master Knot of Henry is the deforming force on the medial side of the foot. It
is the fibrous junction of the FHL/FDL sheaths to the navicular and fascia of
the FHB. It must be severed to allow the navicular to re-establish itself laterally
with the calcaneus
c. The talus is usually deformed: the head and neck are medially displaced and
downward (this is consistent in rigid clubfoot)
d. The T-N joint is subluxed with the navicular medially
e. Posterior medial structures are tight along with the STJ tissues
f. Cuboid is displaced inwardly along with the navicular at the M-T joint and
the anterior calcaneus following to go down and under the talar head
g. The anterior surface of the calcaneus faces more medially, so the lateral
column must be corrected to give a rectus foot
h. Two accessory joints are regularly found
i. The navicular rests on the anterior portion of the medial malleolus
ii. The posterior-lateral calcaneus rests on the posterior fibular malleolus
i. The tendo achilles is slightly medial on the calcaneus
j. The most consistent bony deformity is at the talar neck. It is short and
medially deviated
k. Arteriography:
i. PT artery is most prominent (so must protect during surgery)
ii. Deep plantar arch is supplied primarily by the PT not the DP (as in the
normal foot)
iii. A majority of the TEV feet develop without the DP artery
l. The calcaneofibular and posterior talofibular ligaments are tight due to the
equinus position
m. The medial submalleolar skin is contracted and heals poorly

4. Pre-operative Evaluation:
a. Angle of Kite (normal= 20-40°) approaches 00
b. A-P view shows talar bisection lateral to 1 st metatarsal
c. If foot has marked forefoot adductus, corrective surgery is indicated for this
d. Flat-top talus may be present due to aggressive conservative treatment,
which may later produce osteochondral fracture and later arthritis
e. Preoperative vascular assessment is important to determine the amount of
correction able to be done: the limiting factor to any correction is the stress
placed on the medial soft tissue and the neurovascular bundle
f. Make sure the preoperative x-ray is taken with the knee/leg/foot vertical and
not abducted
g. A lateral stress dorsiflexion view is the most accurate judge of a clubfoot
correction
h. The long axis of the talus is directed downward toward the 3rd met instead of
medial to the 1st metatarsal as in the normal foot
i. Should consider a plantar release in children older than 6 due to cavus
deformity that progresses from accommodation contractures of the plantar
fascia, abductor hallucis, intrinsic toe flexors, and abductor digiti minimi
j. Talus is too far forward in the mortise, therefore, increased equinus and
decreased dorsiflexion
k. Tibia shows increased lateral torsion
l. Ossification centers usually appear later in the clubfoot
m. Parallel talus and calcaneus
n. No overlap of the anterior ends of the talus and calcaneus
o. In the normal foot the T-C angle increases with dorsiflexion, but with the
clubfoot this angle does not change
p. If surgery is delayed till after the age of 10, many adaptive changes will have
taken place, and a triple arthrodesis may be the procedure of choice

5. General Symptoms:
a. Lateral callosities
b. Tiring easily
c. Thin calf ms. on the affected side
d. Smaller foot
e. Small 1st metatarsal with larger 4th and 5th metatarsals
f. Limb length difference with adaptive scoliosis
g. Hyperextended knees
h. Metatarsus adductus
i. No wrinkles over the achilles insertion (this helps diagnosis idiopathic from
non-idiopathic as so if wrinkles are present the foot did have
plantar/dorsiflexion at one time indicating a non-idiopathic clubfoot)
j. Genu valgum
k. External rotation of the leg

6. General Order of Surgical Corrections:


a. Posterior release
b. Posterior medial release
c. Plantar release
d. Subtalar release
e. Metatarsus adductus procedure
f. Tendon transfers
g. Calcaneal osteotomies
h. Triple arthrodesis with soft tissue release
i. Amputation (if all else fails)
7. Surgical Treatment (soft tissue): Performed after 3 months of failed
conservative care- the next step.
a. Posterior Medial Subtalar Release (TURCO procedure)
i. Skin incision is classically a hockey stick incision from the base of the 1st
metatarsal continuing under the medial malleolus and partially up the medial
aspect of the leg. Due to severe skin necrosis from this, a linear incision is now
used
ii. Isolate the posterior tibial ms., FHL, FDL, neurovascular bundle, and medial
achilles
iii. Loosen the abductor hallucis from the medial calcaneal tuberosity
iv. Release the Master Knot of Henry (this obstructs a good view of the medial
side of the foot)
v. Posterior release is done first to allow good visualization of anatomical
structures

 Inferior/posterior achilles release with sagittal plane Z TAL


 Release FHL sheath and retract with the neurovascular bundle medial
and anterior
 Apply dorsiflexory pressure and transect ankle and STJ capsule
 Resect calcaneofibular and posterior talofibular ligaments
 Resect posterior superficial part of the deltoid ligament (talotibial)
 Lengthen the FDL to prevent clawtoe deformity

vi. Medial release now tries to reposition medially displaced navicular laterally
onto the talar head

 Posterior tibial ms. is sectioned retaining good control of the distal


segment to help isolate the T-N joint and medial structures
 Resect the calcaneofibular ligament (spring)
 Resect the superficial part of the deltoid leaving the deep tibiotalar intact
anteriorly

vii. Plantar release

 incise the plantar fascia


 incise the first layer of intrinsics
 incise the long plantar ligament

viii. Subtalar release allows the anterior calcaneus to move lateral-dorsal so the
talus may reposition to a more appropriate pronated position
 Evert the heel and intersect the interosseous talocalcaneal ligament
 May need to sever the bifurcate ligament to help reposition the talus and
calcaneus
ix. After all soft tissue releases and the foot is corrected, the T-N and T-C joints
should be stabilized with K-wires
x. Resected tendons should now be repaired except the posterior tibial
xi. Skin closure: may need skin graft in the severely deformed foot or overly
corrected foot
xii. If metatarsus adductus present now do HHS procedure
Note* Turco states that best results are obtained when:
a. the child is 1-2 years old
b. Good results decrease with age
c. Good results if the child is walking due to Wolfs Law to help bone remodel
d. Previous surgery is a hindrance

b. Tendon transfers: usually done as an adjunctive procedure, and not primary


ones. Transfers are only useful to help hold the correction of the flexible foot
i. Stewart TAL
ii. STATT
iii. Anterior tibial transfer
iv Posterior tibial transfer

8. Surgical Treatment (osseous): Usually done after the child is 4 or if soft


tissue surgery has failed
a. Evans calcaneal osteotomy (corrects the anterior medial position of the
anterior calcaneal surface-an adaptive change)
b. Dwyer calcaneal osteotomy (tries to establish a perpendicular heel and a
weight bearing center of gravity)
c. Triple arthrodesis is the procedure of choice when all else fails
d. Talectomy is a possibility in the severely deformed foot
e. Amputation can be a viable alternative when most other procedures fail. With
the aid of bracing, many times the extremity is more functional with a good
device as compared to a very deformed and painful foot

9. Postoperative Evaluation:
a. Calcaneus is rotated out of plantarflexion to dorsiflexion
b. Posterior tubercle moves down when the anterior process moves up and
laterally away from under the talus
c. T-C angle now approx. 400 (lateral x-ray) and 250 on A-P view
d. Intraoperative lateral should shoe dorsiflexion of the calcaneus, overlap of
the talus on the anterior calcaneus if correction achieved

Note* Failure of the calcaneus to dorsiflex is evidence of incomplete subtalar


correction regardless of what the A-P x-ray and clinical exam reveal

10. Postoperative Care:


a. Cast in neutral
b. Prophylactic antibiotics given
c. Change cast every month, first cast change at 3 weeks-remove sutures
d. At second cast change, remove K-wires
e. Weight bearing allowed when wires removed
f. Straight last shoes used for 1 year
g. Physical therapy

11. Complications:
a. Rocker bottom foot if conservative treatment of equinus is corrected before
the varus component or if the internal fixation is removed too soon
b. Flap/skin necrosis
c. Inability to close the skin after reduction
d. Damage to growth plates
e. Relapse of the deformity
f. Loss of the longitudinal arch
g. Stiffness
h. Hammertoe deformities
i. Skewfoot can develop secondary to a valgus correction of the forefoot

12. General Facts:


a. Treatment must be related to the type of clubfoot and age
b. The first step in the correction of clubfoot should be the replacement of the
navicular on the talus
c. Soft tissue procedures will fail if secondary bony changes are present
d. In the older child an adaptive contracture can produce a cavus foot. This is
different from the average cavus foot because the calcaneus is plantarflexed in
clubfoot-cavus and usually dorsiflexes in idiopathic or neuromuscular cavus.
This is due to plantar contracture of the aponeurosis, abductor hallucis,
intrinsics, and deep plantar ligaments
Chapter 22: Generalized
Disease
Conditions of Bone
Soft Tissue Overgrowth
Abnormalities of Alignment
General Increased Bone Density
Osteopenia
Marrow Abnormalities
The Dysplasias
GENERALIZED DISEASE
CONDITIONS OF BONE
Soft Tissue Overgrowth
1. Acromegaly: An acquired condition resulting in excess of growth hormone
due to a pituitary eosinophilic adenoma. Radiographically, it is characterized by
overgrowth of bone, cartilage, and soft tissue. With adult onset the hands and
feet demonstrate the most pronounced skeletal changes

2. Pachydermoperiostosis (Idiopathic Osteoarthropathy): A hereditary


condition transmitted as an autosomal dominant gene. It manifests itself in
adolescence as overgrowth of the soft tissues, periostitis, and synovitis.
Clinically, this is characterized by thickening of the skin, clubbing of the
terminal digits, and painful swollen joints. It does resemble Acromegaly except
the terminal tufts tend to be reabsorbed rather than enlarged. The scalp
demonstrates cutis verticus gyrata-longitudinal wrinkles.

3. Neurofibromatosis-1 (von Recklinghausen's Disease): An autosomal


dominant multisystem disease. Neurofibromas of the skin, bones and. soft
tissues result in grotesque deformities. Neurofibromas occur on the peripheral
and cranial nerves and have the potential for sarcomatous degeneration. "Cafe-
au-lait" spots are present on the skin. Half of these patients develop skeletal
abnormalities which include kyphoscoliosis, scalloping of the vertebral bodies,
rib notching, and a characteristic defect of the orbit of the eye. The peripheral
skeleton can develop overgrowth of the soft tissues and bones or
overtubulation, giving the bones a thin bowed appearance.

4. Macrodactyly: A localized enlargement of the soft tissues of a digit, which


can be idiopathic or non-idiopathic.

Abnormalities of Alignment
1. Ehlers-Danlos Syndrome: A group of inherited connective tissue disorders
resulting in hyperelasticity of the skin and fragility of blood vessels.
Ligamentous and capsular laxity as well as muscular weakness lead to
subluxation and dislocation. In the feet pes planus is very common. Atrophic
scarring occurs: There are 9 subtypes. Types I, II, and III are the most common,
and are autosomal dominant types.

2. Marfan's Syndrome: An inherited autosomal dominant connective tissue


disorder resulting in abnormal elastic tissue and collagen production and
excessive laxity. Hyperextensible joints and ligamentous laxity result in hallux
valgus and pes planus, and kyphoscoliosis. The most serious complication of
Marfan's syndrome is a dissecting aneurysm of the aorta.
The radiographic hallmark of Marfan's is exaggerated length of the bones. The
lens of the eye Is also displaced usually upward.

General Increased Bone Density


1. Osteopetrosis (Albers-Schonberg Disease): An inherited bone disease
causing diffusely dense bones. The basic defect is a failure of osteoclasts to
absorb primary spongiosa during enchondral bone formation. This leads to a
"bone-within-a-bone" appearance. The bone is actually very weak.

2. Melorrheostosis: An acquired condition causing cortical thickening. Its


etiology is unknown. The cortical thickening is smooth and thick and involves
the periosteal surface or the endosteal surface, encroaching on the medullary
canal.

3. Osteopoikilosis: A hereditary "spotted" bone disorder transmitted as an


autosomal dominant. There are multiple round or oval bone densities occurring
in the spongiosa of bone. There is a predilection for the tarsus in the foot. There
is no uptake with this via bone scan (differential for blastic metastases).

4. Bone Islands: Are histologically identical to osteopoikilosis but occur as


single lesions.

5. Paget's Disease of Bone: The destruction of bone with subsequent repair


results in thickened, disorganized trabeculae and increased size of bone, giving
the appearance of increased density. Usually the tibia is affected, but can
involve the bones of the feet. The coarse trabeculae often have a different
appearance from blastic metastases and melorrheostosis. Malignant
degeneration is a rare complication (malignant fibrous histiocytoma,
osteosarcoma, fibrosarcome, chondrosarcoma-all called "Paget's sarcoma" if
they occur in a setting of Paget's Disease of Bone).

Osteopenia
1. Rickets and Osteomalacia: Due to deficient mineralization of bone
specifically due to insufficient vitamin D from malabsorption, renal disease,
lack of adequate sunlight, liver disease, and metabolic disorders. In the child
this can cause widening of the growth plate, cupping of the epiphyses, and
bowing of the legs. The pathognomonic sign of osteomalacia is the presence of
Looser's lines or pseudofractures. These lucent lines are perpendicular to the
cortex, are often bilateral and symmetrical.

2. Osteoporosis (decreased bone mass): Loss of trabeculae and thinning of the


cortex
i. Scurvy: Lack of vitamin C results in failure to produce intracellular
substances, therefore, osteoporosis of the adjacent metaphysis, seen as a
transverse radiolucent bands. This is called the scurvy line. Fractures can
occur through this zone with complete separation of the epiphyseal plate.
The vascular endothelium is abnormal, which leads to hemorrhage.
Subperiosteal bleeding causes wide separation of the periosteum, and healing
manifests as a thin shell of peripheral periosteal new bone. It takes a minimum
of 4 months of lack of vitamin C for the disease to become apparent. This is
very rare today in developed countries.
ii. Osteogenesis Imperfecta (Ekman-Lobstein disease): A generalized
connective tissue disorder that is inherited as an autosomal dominant gene. A
deficiency of osteoblasts results in severe generalized osteoporosis, the long
bones are overtrabeculated and bowed, and protrusio acetabuli are frequently
seen. Additionally there is abnormal dentition, blue sclerae and early deafness.

iii. Disuse: Diffuse, severe osteoporosis accompanies disuse or immobilization,


with changes resulting in thinning of the cortex and loss of trabeculae giving
the bone a ground glass appearance.

iv. Reflex Sympathetic Dystrophy Syndrome (Sudek's Atrophy): Generally


follows minor trauma, with severe pain and soft tissue swelling and rapid
demineralization. Rapid loss of mineral results in a patchy"motheaten" pattern
of demineralization. (see Chapter: Neurology).

The basic radiological changes inherent to all the marrow affecting


disorders includes expansion of the marrow cavity, thinning of the
trabeculae, and the appearance of rectangular, osteopenic bone.
Marrow Abnormalities
1. Anemias:
i. Thalassemia major: causes microcytic/hypochromic anemia. Generalized
involvement of all the bones of the feet occurs as mentioned above.
ii. Sickle cell disease: produces changes similar to thalassemia. Those changes
can sometimes be mistaken for osteomyelitis. Can cause infarctions in bone
due to vascular blockage by abnormal RBC's.

2. Storage Diseases:
i. Glycogen storage diseases (Von Gierke's disease)
ii. Cerebrosides (Gaucher's disease and Niemann-Pick disease)
iii. Mucopolysaccharides (Hurler's, Hunter's, Sanfiiippo's. Morquio's,
Scheie's, and Maroteaux-Lamy syndromes)

3. Fibro-Osseous Dysplasia:
i. Fibrous dysplasia: A developmental anomaly that results in fibrous tissue
replacement of the marrow cavity

4. Granulomatous Diseases:
i. Sarcoidosis: a systemic granulomatous disease of unknown etiology that can
affect the bones in a small percentage of cases, and when involved, almost
always is seen in the phalanges causing small punched-out defects. Severe
involvement may result in cortical destruction, fractures, and collapse of bone,
giving an arthritis mutilans scenario.
ii. Tuberculosis: Granulomatous disease due to hematogenous spread of the
TB bacillus. Acid fast stains show TB bacilli, which are not seen in Sarcoid.

5. Infarction: Any type of vascular obstruction may result in marrow


infarction. The radiographic changes are frequently those of bone destruction.
i. Pancreatic disease

The Dysplasias
Dysplasia is a disturbance in the formation and modeling of bone; it is
usually hereditary and most commonly occurs as a result of inborn errors in
metabolism. Foot dysplasias are an expression of a generalized skeletal
anomaly.
1. Spondyloepiphyseal Dysplasia (Morquio-Brailsford disease)
2. Multiple Epiphyseal Displasia (Fairbank's disease)
3. Chondrodysplasia punctata (Conradi's disease)

The Occurence of Cone Shaped Epiphyses


a. Normal variant
b. Chondroectodermal dysplasia (Ellis-van Crevald syndrome)
c. Cleidocranial dysostosis
d. Dyschondrosteosis
e. Hand-foot-uterus syndrome
f. Oto-palato-digital syndrome
g. Osteopetrosis
h. Pycnodysostosis
i. Tricorhinophalangeal syndrome

Occurence of Tarsal Coalition


a. Isolated variant (usually in the hindfoot)
b. Juvenile rheumatoid arthritis
c. Apert's syndrome
d. Arthrogryphosis multiplex congenita
e. Hand-foot-uterus syndrome
f. Oto-palato-digital syndrome (Taybi's syndrome)

Occurence of Polydactyly
a. Arthrogryphosis
b. Basal cell nevus syndrome
c. Cleidocranial dyostosis
d. Ellis-van Crevald syndrome
e. Gorlin-Chaudhry-Moss syndrome
f. Larsen's syndrome
g. Myositis ossificans progressiva
h. Rubenstein-Taybi syndrome
i. Trisomy 13-15 (Perth's syndrome)
Occurence of Syndactyly
a. Normal variant
b. Apert's syndrome
c. Carpenter's syndrome
d. Bloom's syndrome
e. Down's syndrome
f. F syndrome
g. Laurence-Moon-Biedl syndrome
h. Taybi's syndrome
i. Popliteal pterygium syndrome
j. Prader-Willi syndrome
k. Silver's syndrome
l. Trisomy 13

Occurence of Short Hallux


a. Myositis ossificans progressiva
b. Taybi's syndrome
c. Hand-foot-uterus syndrome
d. Apert's syndrome
e. Larsen's syndrome
f. Popliteal pterygium syndrome
g. Franconi's syndrome
h. Holt Oram's syndrome

Occurence of Short Metatarsals


a. Normal variant
b. Enchondromatosis
c. Juvenile RA
d. Osteochondromatosis
e. Pseudohypoparathyroidism
f. Pseudopseudohypoparathyroidism
g. Trauma
h. Turner's syndrome*

*Turner's syndrome is-one of the more common chromosomal syndromes


resulting in a sex chromosome" abnormality that causes a XO pattern.
Important clinical findings are coarctation of the aorta, horseshoe kidneys,
short neck, and hypogonadism. Radiographically, a short 4th metatarsal is
commonly found.
Chapter 23: Radiology
Standard Radiographic Techniques
Specific Radiographic Studies
Anatomic Angles
Common Structural Measurements (Diagrams)
Pediatric Radiology
The Osteochondritities
The Accessory Bones of the Foot
RADIOLOGY
Standard Radiographic Techniques of the Foot and Ankle

The standard of our profession's technique (especially for pre-operative


planning) is a weight-bearing x-ray of each foot individually, in the
angle and base of gait, with proper shielding to the patient. This allows
us to reproduce the same bony architectural relationships consistently.

1. Dorsoplantar Projection: X-ray tube angled 15° from vertical and aimed at
the lateral aspect of the navicular.

2. Weight-bearing Lateral Projection: X-ray tube angled at 900 from vertical

3. Non-weight-bearing Medial Projection: Useful in examining the talus,


calcaneus, and lesser tarsal bones for the effects of trauma.

4. Lateral Oblique Projection: Gives a magnified and slightly distorted


representation of the bones of the foot. Gives the most accurate picture of the
shape of the proximal phalanx of the fifth toe.

5. Medial Oblique Projection: Has limited value in examining the foot.

6. Axial Sesamoidal Projection: Good for examining the plantar aspect of the
sesamoids and their relationship with the 1st metatarsal head.

7. Axial Calcaneal Projecton: Useful when examining the calcaneus for


fractures, abnormalities of shape, or neoplasms

8. Harris and Beath (ski jump/coalition) Projection: Useful in examining the


STJ (T-C coalitions of the posterior or middle facets), Calcaneal fractures, and
sustentaculum tali. 3 exposures taken with the x-ray tube set at 35°, next at
40°, and then at 45°

9. Ankle Mortise Projection: The ankle is internally rotated 15° with the xray
tube at 90° from vertical. Good for evaluating the joint space. The view for
measuring the angles of the ankle (see Anatomic Angles below).

10. Lateral & Oblique Projections of the Ankle: Used in looking for the effects
from trauma.

11. Stress Inversion Projection of the Ankle: Usually taken following


inversion sprains. With the ankle joint anesthetized, bilateral views should be
taken in plantarflexion and at right angles. It can sometimes be difficult to
correlate the number of degrees of talar tilt with the number of ligaments
ruptured.
Note* Some feel that a tilt of 159= rupture of the anterior talo-fib ligament, 15-
309= rupture of the anterior talo-fib ligament and calcaneofib ligaments, and
over 30° of talar tilt= all three ligaments ruptures (this is a guide, not a hard
and fast rule)

12. Anterior Drawer Projection: Is taken following trauma to the ankle joint,
with a distraction force placed on the forefoot. (looking for an anterior excursion
of the talus out of the mortise). If the excursion is greater than 4 mm. as
compared to the opposite side, then disruption of the anterior talofibular
ligament is assumed.

13. Stress Dorsiflexion Projection: Can be useful for the evaluation of talipes
equinovarus (measure the T-C angle).

14. Isherwood Projections: Consist of 3 projections to fully visualize the STJ.


They are: the lateral oblique, medial oblique axial, and lateral oblique axial.
Positioning is very difficult with these positions, so better use tomograms or CT.

15. Broden Projections: For examination of the STJ (Broden #1 examines the
posterior facet & Broden #2 examines the sinus tarsi). Consists of 4 views with
the beam angled at 10°, 20°, 30°, and 40° while centering the x-ray tube on a
point 2 cm anterior and distal to the tip of the lateral malleolus. Has generally
been replaced by tomography and CT.

16. Anthansen Projections: To view the medial and posterior facet of the STJ.

Specific Radiographic Studies


1. Bone Scanning: Bone scintigraphy using technetium-99m MDP is excellent
for screening as it provides extremely good sensitivity but at the expense of
being very nonspecific.
Increased tracer uptake occurs as a result of hyperemia. A positive bone scan
does not necessarily represent an osseous lesion, because juxtacapsular joint
lesions, periosteal inflammation, or inflammation at tendonous insertions can
also produce positive results. The predominant scintigraphic finding is a "hot
spot" (increased tracer localization). The exception to this are "cold spots" which
is due to NO delivery of the tracer as a result of poor circulation, necrosis, or a
fulminant destructive osteomyelitis not accompanied by significant reparative
processes which is also due to poor circulation. To obtain good-quality
scintigraphic results the following guidelines should be followed: Request close
up views of the foot and ankle with multiple angles of view (not whole body
shots), and have the studies supervised by a radiologist who can ensure that
the images are filmed at appropriate densities (this avoids white-outs or black
outs).
a. 99m Technetium-MDP is currently the most frequently used radionuclide.
b. It is renally excreted
c. Has a half-life of 6 hours,
d. MDP chemiadsorbs to bone with the hydroxyapatite crystal (99mTc is just a
label)
e. Provides more anatomic information with less time, exposure and expense
than Gallium 67 imaging.
f. May continue to show abnormal isotope accumulation after infection
subsides as a result of continued bone repair.
g. Uptake of technetium 99m MDP will occur in any focus of increased bone
turnover, whether due to physical stress (osteoarthritis), repair (fracture,
reactive periostitis, rheumatoid arthritis, plantar fasciitis), or tumor (primary or
secondary; multiple myeloma is a relative exception)
h. When to order: Persistent pain with negative x-rays, osteomyelitis from
overlying soft tissue infection (x-ray negative), extent of osteomyelitis, progress
of healing of osteomyelitis and fracture, osteomyelitis ve Charcot, and
unexpected x-ray or MRI abnormality

Three Phase Bone Scan: is used to differentiate OM from cellulitis.


Phase 1: at time of injection shows an immediate radionuclide angiogram or
dynamic blood flow, OM and cellulitis both show increased uptake at this
point.
Phase 2: 10 minutes after injection looking for focal increases (blood pool
image) cellulitis and OM are still positive at this point.
Phase 3: 4 hours after injection (delayed static scan or bone image), cellulitis
becomes quiescent at this point- ONLY CLASSICALLY
Phase 4: 24 hours later used for patients with poor vascular flow and the 3rd
phase looks almost exactly like the 2nd phase (increased sensitivity for
detecting osteomyelitis in the diabetic foot)

NOTE* False negatives have been reported in infants and children

Osteomyelitis
NOTE* Bone has
scans are intense
a good and focal
indicator if the uptake in all 3 phases
Reflex Sympathetic Dystrophy
patient will be responsive to treatment

2. Gallium Scanning: The isotope, Gallium-67 citrate, was originally developed


as a marker for certain tumors, i.e. lymphoma, and is considered an imaging
marker for inflammation. It is less dependent upon blood flow than technetium.
a. There are 3 mechanisms for Gallium-67 localization:
i. Leukocyte localization or incorporation
ii. Direct lactoferrin and transferrin binding at the site of infection (also Gallium
binds to the siderophores of bacteria)
iii. Direct bacterial uptake by phagocytes
b. The usual dosage of is 3-5 mCi, and is usually performed 24 hours after
injection
NOTE* Gallium is more reliable in differentiating benign from malignant
tumors, assessing subacute and chronic infections. The only tumor Gallium is
specifically used for is lymphoma

c. Combination of gallium scanning following technetium scanning by 24 hours


(due to the short half life of Tc-99m)is helpful in distinguishing chronic from
acute OM, and cellutitis from OM.
d. Gallium is valuable in monitoring disease activity and response to treatment
in patients with chronic osteomyelitis (gallium is not as sensitive to bone
remodeling as technetium)
e. Gallium imaging is for chronic infection, predominantly lymphocytes

3. 111 Indium White Cell Scanning (Indium is just the label): This scan is
much more specific for infection (especially acute infections) and involves
predomininantly granulocytes. With this scan, the patient's white blood cells
are labeled with the tracer and injected intravenously. This technique was
developed to detect leukocyte accumulation at sites of inflammation and
abscess formation. Scans are performed 24 hours after injection. A positive
scan is defined as a focal accumulation of leukocytes that is higher than the
surrounding bone activity. This technique is reserved for complicated post-
traumatic or post-surgical patients with equivocal conventional bone scans, in
cases where 99mTc MDP scanning reveals false positive results because of rapid
turnover. Therefore, it may be more accurate in detecting acute infections. VIII.
Limitations of Scans: Some patients show multiple hot spots at an early stage
of S. aureus septicemia but do not progress to OM. You can have a negative
scan with a confirmed OM due to impaired blood supply (false-negative). You
can have difficulty in differentiating OM from cellulitis. You can have difficulty
in differentiating normal bone repair from bone infection (false-positive).

4. Xeroradiography: A process in which an image is produced on a selenium-


coated plate. This process tends to emphasize the characteristics of borders
between tissues, making detailed information more easily seen. It is sensitive
enough to visualize non-metallic foreign bodies. (no longer readily available)

5. Fluoroscopy: An imaging modality in which x-rays are produced


continuously on demand to give a real-time, dynamic image that is displayed on
a television screen. The C-arm fluoroscope is the usual unit used
intraoperatively.

6. MRI: MRI gathers information (imaging the nucleus of the atom) in the
form of low energy radiowaves and transduces this energy into images
with the use of computers. Four components are necessary for the
production of such images:
 Magnetic nuclei (the sample)
 The strong magnetic field
 Coils to transmit and receive radio frequency waves
 Magnetic gradiance (small magnetic fields with known, carefully
controlled spatial variation)

NOTE* Protons spin on their long axis, making a magnetic field. In the human
body, these protons spin randomly. When the body is placed in the MRI
machine, most (or more) of the protons line up parallel to the magnetic field of
the machine. Feeding radiowaves into the body (RF), the protons are excited
and energy is released producing an image.

Most clinical MR imaging is performed with spin-echo pulse sequences.


A spin-echo sequence consists of a 90° pulse followed after a time period by a
180° pulse, with a consequent production of a signal (echo). The echo-time (TE)
is the time elapsed from the beginning of the 90° pulse to the peak of the echo.
The repetition time (TR) is the time elapsed between successive 90° pulses. In
typical MRl foot ankle studies the spin echo sequence is performed twice
(referred to as 2 averages or excitations)

By varying both TR and TE, images that primarily reflect TI-relaxation, T2-
relaxation, or proton density may be obtained.
a. A signal that reflects primarily T1 images is produced by using a spin-echo
sequence of a short TE (20-30 msec) and short TR (300-800 msec)
b. T2 images are produced with a long TE (60-120 msec) and long TR
(15003000 msec)
c. A proton image (or balanced image containing properties of both T1 and T2)
is produced with a short TE and long TR

With all other factors being equal, imaging time is directly proportional to TR,
with T2 images taking the longest time to obtain (prone to degradation due to
motion by the patient)

a. On T1-weighted images, tissue characterized by a short T1 relaxation time


produces a high-intensity signal, whereas, long T1 tissue yields a low-intensity
signal. Conversely, on T2-weighted images, a long T2 relaxation time which
results in high signal intensity, and a short T2 results in low signal intensity.

NOTE* Adequate characterization of musculoskeletal disease usually requires


both T1 & T2-weighted images, because each provides complimentary
information in terms of soft tissue contrast and anatomic detail.

b. T1-weighted images provide the best signal-to-noise ratio, resulting in


superior anatomic definition. A typical T1 weighted image : repetition time of
500 millisec and echo time of 30 millisec (TR500/TE30). These numbers can be
seen to the right of the image on the film, which allows you to determine if the
image is T1 or T2 or or a proton image.
c. T2-weighted images provide the best soft tissue contrast, and are therefore,
best for many pathologic processes, such as neoplasms and inflammation.
d. High density (bright) areas most often reflect a high density of mobile protons
or tissues with a short T1 or a long T2 (bone marrow and subcutaneous tissues
that contain large amounts of fat). Low intensity (dark) areas have fewer mobile
protons (cortical bone and tendons). Most soft tissue tumors (with the exception
to those composed of fat) have an appearance to those of muscle (low to
intermediate signal intensity) on T1 weighted images, but can be differentiated
from muscle on a T2 weighted image.

NOTE* Evaluation of T1 weighted images:


a. Dark (black) areas are: tendons, subchondral cortex of bone, blood vessels
with moving blood, ligaments, muscles, tumors
b. Light (white) areas: fat (whitest), medullary bone, stationary blood
Evaluation of T2 weighted images:
a. Dark areas: ligaments, compact bone
b. Intermediate areas (gray): subcutaneous fat, bone marrow, muscle
c. Light areas: stationary blood, tumor
Evaluation of proton image:
a. Intermediate (gray): fluid , muscles
NOTE* There are other MRI Stues: STIR mage wcs gooor tumors anGradient
echo (also known as magnetic resonance angiography) which is good for
hyaline cartilage

e. Benefits of MRI:
i. MRI is better than CT in evaluating bone tumors of the medullary canal (CT
has higher resolution for tumors of the cortex). It permits better delineation of
the tumor and has superior soft tissue contrast
NOTE* MRI cannot predict malignancy vs. non-malignancy

ii. MRI is excellent in diagnosing trauma with patients that have preexisting
metallic implants.
iii. MRI is the imaging modality of choice for avascular necrosis.
iv. MRI gives direct multiplanar imaging capabilities in any desired plane
(sagittal, axial, and coronal)

NOTE* When ordering an MRI you must specify what you are trying to look for,
and if you want a certain view, you must specify this too

f. Precautions:
i. MRI not to be done in the first trimester of pregnancy (no studies showing
fetal abnormalities with MRI to date)
ii. MRI should be avoided with cerebral aneurysm clips (may become dislodged),
cardiac pacemaker (interfere with function), and implanted metallic objects
near the orbit of the eye)
iii. Patients with claustrophobia may require sedation
7. CT Scanning: Can establish the presence, nature, size, margination, and
exact location of tumors. Muscle and soft tissue involvement can be
determined. If a tumor is located next to blood vessels, a contrast medium is
needed to enhance its identification. Is excellent to evaluate metabolic bones
diseases (osteoporosis, aseptic necrosis, osteomalacia). It is excellent in
evaluating trauma especially the calcaneus and STJ. The CT can dictate
whether open reduction would be beneficial and whether one or a twosided
approach is indicated to effect the reduction. CT is excellent in the diagnosis of
tarsal coalition and degenerative changes of the tarsus or lesser tarsus where
superimposition has always been a problem.

8. Arthrography: Following the injection of contrast medium into a joint, x-rays


are taken. Good for the diagnosis of capsular or ligamentous tears. Has been
replaced by MRI when available. Contrast media can cause anaphylactic
reactions

9. Tomography: This procedure requires a complex reciprocal motion of both


the radiographic tube and cassette around the patient. It requires a relatively
large number of radiation exposures and demands exacting technique, but has
the advantage of providing excellent bony detail in areas of complex osseous
anatomy. The most useful applications of tomography are in the evaluation of
osteochondral fractures in the dome of the talus; arthritic changes or loose
bony fragments of the STJ and the tarsometatarsal joints, stress fractures of
the navicular, union vs. non-union of an arthrodesis site, and the status of
metallic implants (metallic implants affect the quality of CT and MRI).

10. Tenography: Is most often used on the ankle tendons. It can also
document calcaneofibular ligament tears, because this ligament is contiguous
with a part of the peroneal tendon sheath. It has been used to identify
irregularities of the peroneal tendons themselves. Has generally be replaced
with MRI (non-invasive).
a. Tenogram shows narrowing and irregularity of the involved tendon
b. Is only useful in tendons that go around a bone (like the malleoli)
c. Tenography of the posterior tibial tendon reveals 3 types of pathology: central
swelling, thinning of the tendon, and rupture
d. The results of tenography can be:
i. Normal
ii. Mild marginal irregularity
iii. Moderate marginal irregularity
iv. Marked marginal irregularity
v. Occlusion of the tendon sheath
e. Contrast is injected into the proximal portion of the tendon sheath (Conray
43®) mixed 50/50 with Xylocaine®. Upon completion of the
tenogram, a steroid is injected

Anatomic Angles
1. Angles of the Ankle: Are helpful when evaluating ankle trauma. on the D-P
projection.
2. Angular Relationships on the D-P Projection:
a. Talocalcaneal Angle or Angle of Kite (normal for ages 0-5 years= 35-50°
& ages 5-adult= 15-35°): Has long been used as an index of relative foot
pronation and supination. It is a measure of the transverse plane angular
relationship between the longitudinal bisectors of the talus and calcaneus. It
becomes increased with STJ pronation and reduced with supination.

b. Cuboid Abduction Angle (normal= 0-5°): Lines along the lateral surfaces of
both the cuboid and calcaneus. With pronation (increased abduction: Lenoire's
sign) the angle increases. This is important with serial casting.

c. Forefoot Adductus Angle (normal=0-15° In the rectus foot): Is the angle


formed by the longitudinal reference of the rearfoot and the bisection of the
second metatarsal.

d. Metatarsus Adductus Angle (normal= 0-15°): The relative position of the


forefoot to the rearfoot. It is measured on D-P view by comparing the
longitudinal bisection of the second metatarsal, with the bisection of the lesser
tarsus. An increase of the angle results in medial deviations of the first
metatarsal.

e. Metatarsus Primus Adductus Angle or First Intermetatarsal Angle


(normal in rectus foot--8-1 2° and 8-10° In adductus foot): Represents the
medial deviation of the first metatarsal relative to the second. The angle is
measured on D-P view by the intersection of longitudinal bisections of the first
metatarsal and second metatarsal.

NOTE* In general, angular increases up to 15° are correctable by distal


osteotomies, while increases greater than 15° require proximal osteotomies

f. Tibial Sesamoid Position (positions 1-3 can be considered normal): This


position is sometimes used to decide the necessity for fibular sesamoid removal
during HAV surgery. The change in sesamoid position occurs relative to the 1 st
metatarsal head. TSP is measured relative to the 1 st metatarsal bisector. There
are 7 positions.

NOTE* A severely dislocated fibular sesamoid becomes a strong deforming


force maintaining the hallux in the laterally rotated position, and impeding
correction of a high metatarsus primus adductus angle
NOTE* Removal of the fibular sesamoid plus cutting the adductor tendon will
result in a hallux varus. Prior to its removal a plantar-axial view should be
evaluated, and if there are no degenerative changes and lateral subluxation is
not severe then the sesamoids can be relocated.

g. Hallux Abductus Angle (normal= 10-15°): Represents the transverse plane


position of the hallux relative to the long axis of the first metatarsal. This angle
is produced by the intersection of the first metatarsal and first proximal
phalangeal bisectors. This measurement quantifies the lateral deviation of the
hallux in HAV.

h. Hallux Interphalangeal Angle (normal= 0-10°): Represents the lateral


hallux deviation at the level of the IP joint. It is the measurement of the angle
produced by the intersection of the proximal and distal phalangeal bisectors.
Increases in this value produce a lateral curvature of the hallux that become
clinically significant, and becomes an important assessment of HAV when
attempting to determine whether a proximal or distal Akin or Akin arthrodesis
should be performed.

i. Proximal Articular Set Angle (normal= 7.5°): An angle formed by a


perpendicular to a bisection of the 1 st metatarsal and a line representing the
effective articular cartilage. This represents the effective cartilage in relation to
the shaft of the metatarsal. Any increase in the PASA is pathological and may
either add to a structural deformity or combined deformity (lateral deviation of
the cartilage).
j. Distal Articular Set Angle (normal= 7.5°): The angle that measures the
relationship of the effective articulating cartilage of the base of the proximal
phalanx to a mid-line bisection of the proximal phalanx of the hallux. If the
DASA is highly abnormal, this may indicate the need for some type of
osteotomy to reduce the angulation of the proximal phalanx.

k. Metatarsal Protrusion distance (normal = +/- 2 mm.): The measure in the


difference on length between the 1st and 2nd metatarsals. This parameter is of
primary concern when 1 st metatarsal surgery is being considered on a patient
with an abnormally increased negative protrusion distance.

I. Metatarsus Quintus Abductus Angle (normal = 8-10°): The angle created by


the intersection of the 4th and 5th metatarsal bisectors. This angle is increased
with a splayed foot.

NOTE* The intermetatarsal angle between the 4th and 5th metatarsals can
also be drawn using a line parallel to the proximal medial portion of the 5th
metatarsal as the lateral arm and bisected the 4th metatarsal as the medial
arm of the angle

NOTE* When examining for lateral bowing of the 5th metatarsal an angle
calledthe lateral deviation angle of the 5th metatarsal is examined. This
angle isformed by a line bisecting the head and neck of the 5th metatarsal
and the line previous described to simulate the proximal 5th metatarsal
shaft. Normal= 2.640 With pathology of the 5th metatarsal this number
usually = 8°. When this is present the structural deviation should be
considered as a significant contributing factor in the tailor's bunion
deformity, and addressed surgically.

m. Metatarsal Parabola (normal= 142.5°): The angle formed by the


intersection of lines touching the 1 st and 2nd metatarsal heads intersecting
with 2nd-5th metatarsal heads.

3. Angular Relationships on the Lateral Projection:


a. Calcaneal Inclination Angle (normal= 18-21°): Is a measurement of the
sagittal plane position of the calcaneus as seen on the lateral x-ray. This angle
is increased in rearfoot cavus deformities and decreased (or negative) in flatfoot
deformities.

b. Talar Declination Angle (normal=21°): Is the angle formed by the plane of


support and the column tali axis (bisector of the head and neck of the talus).
This axis will be colinear with the 1st metatarsal declination axis. This angle
increases with pronation and decreases with supination.

Note* The Talar Declination Angle and the Calcaneal Inclination Angle are
inversely proportional
c. Cyma Line: A lazy S curve formed by the T-N and C-C joints (Chopart's
joint). Pronation causes the T-N joint to be anteriorly displaced, and supination
causes the T-N joint to be posteriorly displaced.

d. Sinus Tarsi: In the normal foot it is seen on lateral view as an oval area of
decreased bone density, separating the posterior from middle subtalar facets.
When pronation occurs, as the talus rides anteriorly on the calcaneus and
plantarflexes, the sinus tarsi is obliterated.

e. Fowler-Phillip Angle (normal=44-69°): This is used to evaluate the


posterosuperior tuberosity of the calcaneus (Haglund's deformity). Symptoms
are common when this angle exceeds 700

f. Talocalcaneal Angle (normal=15-35°): Compares the long axis of the head


and neck of the talus to the inferior surface of the calcaneus. This angle is
decreased in the supinated foot and increased in the pronated foot. It is useful
in determining the treatment with talipes equinovarus.

g. Bohler's Angle (normal=25-40°): Is used to describe the calcaneal


architecture by defining the contour of the dorsal calcaneal surface. The angle
is decreased in joint compression and beak fractures of the calcaneus as well as
with Haglund's deformity.

h. Talometatarsal Angle or Meary's Angle (normal 0-10°): The angle formed


from the bisection of the talus and the bisection of the first metatarsal. Is used
in evaluating whether a plantarflexed or hypermobile ray is present.

i. Critical Angle of Gissane (normal=120-145°): This angle is created by the


subchondral bone of the posterior calcaneal facet and of the middle and
anterior calcaneal processes, as seen on the lateral view. It is utilized when
evaluating calcaneal injuries.

j. Neutral Triangle: For reference, the sparsely trabeculated neutral triangle


occurs just below, and posterior to the apex of the Critical Angle of Gissane.

4. Arthromorphic Variants: In HAV surgery there are further considerations


that must be given to the structural anatomy of the 1st MPJ and the shape of
the 1 st metatarsal head.
a. Congruous joint: The measurement of the articulating cartilage of the head
of the 1st metatarsal and proximal phalanx is parallel

b. Deviated joint: The lines intersect outside the joint

c. Subluxed joint: The lines intersect within the joint

d. Dislocated joint: see diagram to follow

d. Round head: The weakest variant and deviates easily.


e. Square head: A more stable shape of joint

f. Square head with a central ridge: The most stable variant. It is believed that
the central ridge is an extention of the plantar crista.

5. Hallux Abductus Deformity:


a. Structural Deformity (bony): A deformity in which there is osseous change
in either the PASA the DASA or both. The structural deformity has as its
characteristics the congruous joint. The HA deformity is also equal to the
summation of the PASA and the DASA.
b. Positional Deformity (soft tissue): There is an abnormality in the HA angle.
The PASA and DASA are normal. The joint is either deviated or subluxed. The
summation of the PASA and DASA are less than the HA angle.

c. Combined Deformity: Has elements of both structural and positional


deformities. Either the PASA or the DASA or both are abnormal, and when
added together they do not equal the HA angle. The joint is either deviated or
subluxed.

Common Structural Measurements (diagrams to follow)


Common Structural Measurements (A to W)
A: Metatarsus adductus angle (normal 15°)
B: Metatarsus adductus angle (alternate)
C: Metatarsus primus adductus angle (normal 8-12°)
D: Proximal articular set angle (normal 0-10°)
E: Distal articular set angle (normal 5.2°)
1st MPJ articulation: (F-I)
F: Congruous
G: Deviated
H: Subluxed
I: Dislocated
J: Hallux abductus angle (normal 10-15°)
K: Hallux abductus interphalangeal angle (normal 0-10°)
L: Metatarsal protrusion distance (normal +/- 2mm)
M: Tibial sesamoid position
N: Tangential angle to the second axis +5° to -5°
O: Metatarsus quintus abductus angle (normal 7°)
P: 1st metatarsal declination angle
Q: Calcaneal inclination angle (normal 18-21°)
R: Fowler-Phillip angle (normal 44-69°)
S: Bohler's angle (normal 28-40°)
T: Critical angle of Gissane (normal 120-145°)
U: Dorsoplantar talocalcaneal angle (Kite) (normal 20-40°)
V: Lateral talocalcaneal angle (normal 35-50°)
W: Anterior cyma line
Pediatric Radiology
1. Roentgenographic Development of the Foot:
a. Important ossification points to remember:
i. 1st bone to ossify before birth: calcaneus
ii. Last bone to ossify before birth: cuboid
iii. 1st bone to ossify after birth: lateral cuneiform
iv. Last tarsal bone to ossify after birth: navicular at 3.5 years
v. Calcaneal apophysis appears at age 7 years
vi. Sesamoids appear at age 12 years
b. Ossification at birth:
i. Talus
ii. Calcaneus
iii. Cuboid (can be absent in the premature baby)
iv. Metatarsals
v. Proximal phalanges
vi. Middle and distal phalanges 2-4
vii. Distal phalanx 1
c. Age 3 months: lateral cuneiform
d. Age 4 months: tibial epiphysis
e. Age 6 months: cuboid and lateral cuneiform articulate
f. Age 7 months: talar neck appears, base of metatarsals widen
g. Age 11 months: fibular epiphysis appears
h. Age 18 months: phalangeal epiphyses appear
i. Age 24 months: medial cuneiform and ossification of epiphysis of metatarsal
1
j. Age 30 months: intermediate cuneiform ossifies
k. Age 36 months: ossification of epiphysis of metatarsals 2,3, and 4
l. Age 3.7 years: ossification of navicular
m. Age 4.2 years: ossification of epiphysis metatarsal 5
n. Age 4.9 years: alignment of tarsal and metatarsal bones

NOTE* Boys lag behind girls with regard to skeletal age

o. Age 6.7: ossification of calcaneal epiphysis


p. Age 12 years: sesamoids appear
q. Age 13 years: os trigonum and os vesalianum appear
r. Age 14 years: fusion of epiphyses of distal phalanges of toes 2, 3, and 4
s. Age 15 years: epiphyseal fusion of tibia/fibula, metatarsals 2/3/4, and
phalanges 1, 3, 4, and hallux.
t. Age 17.5 years: epiphyseal fusion complete

2. The Talocalcaneal Angular Relationships in the Diagnosis of Normal and


Pathological Conditions:
3. Secondary Centers of Ossification:
a. Metatarsals 2-5: at the metatarsal heads
b. Metatarsal 1: at the base
c. Phalanges: at the bases
d. Calcaneus: only constant tarsal bone

The Osteochondritities
1. These are a group of related disorders which effect the primary or secondary
centers of ossification. Its etiology probably relates to some type of vascular
disturbance to the ossification center, during the time of their greatest
developmental activity.

2. The radiographic findings of lucency and fragmentation of the articular


surface with collapse of a wedge shaped fragment of epiphysis and subsequent
sclerosis are the manifestations of ischemic bone.

3. Originally classified together as a spontaneous osteonecrosis, but are a


heterogenous group of entities with several different etiologies.

4. The Aseptic Necroses:


a. Freiberg- metatarsal heads
b. Kohler- navicular
c. Bunchke/Buckman/Lewin/Wagner- cuneiforms
d. Theimann- phalanges
e. Sever- calcaneus (apophysis)
f. Diaz/Mouchet- calcaneus
g. Lance- cuboid
h. Iselin- base of 5th metatarsal
i. Relander- tibial sesamoid (1st metatarsal)
j. Lewin/Wagner/Theimann- epiphysis of toe phalanges
k. Legg-Calve'-Perthes- femoral capital epiphyses
1. Mandl/Buchman- greater trochanter of femur
m. Felix/Monde- lesser trochanter femur
n. Kohler/Sindins/Larsen- patella
o. Osgood-Schlatter- tibial tuberosity
p. Blount- proximal tibial epiphysis

The Accessory Bones of the Foot

Illustration of oblique and A-P projection with most common accessory


bones:
1. Os trigonum
2. Os sustentaculi
3. Os tibiale externum
4. Os supranaviculare
5. Pars peronaea metatarsalis primi
6. Os intermetatarseum
7. Calcaneus secundarium
8. Cuboides secundarium
9. Os vesalianum
Chapter 24: Hallux Valgus &
Related Disorders
Goals of HAV Surgery
Anatomical Facts of the 1st MPJ
Predisposing Factors
Biomechanics
Etiology of HAV
Deformity Types
Radiological Measurements for HAV
Physical Examination
Preoperative Considerations
Arthroplasty Procedures
Capsule-Tendon Balancing Procedures
Implant Arthroplasty Procedures
Arthrodesing Procedures
Proximal Phalangeal Osteotomies
Distal Metatarsal Osteotomies
Proximal Metatarsal Osteotomies
Shaft Osteotomies
Combination Procedures
Other Procedures
Hallux Rigidus and Limitus
Hallux Varus
Hallux Hammertoe
Diff. Diagnosis of Pain in the Sesamoid Area
Complications of HAV Surgery
HAV Procedure Chart and Their Indications
HALLUX VALGUS AND RELATED
DISORDERS
Goals of Hallux Valgus Surgery
1. Pain free joint
2. IM angle less than 100
3. Congruent joint
4. Good range of motion
5. Sesamoids in good position (TSP 3 or less) 6. Cosmetic result acceptable

Anatomical Facts of the 1st M.P. J.


1. The articular surface of the base of the proximal phalanx is larger
plantarly than dorsally.

2. The medullary canal of the proximal phalanx Is dorsal to middle of the


proximal articular surface; this is due to the concave nature of the plantar
curvature.

3. The bony prominence on the dorsum of the proximal phalanx is for the
EHB.

4. 1st metatarsal has cristae on the plantars

5. The nutrient foramen on the lateral aspect of the shaft is 2.7cm from
the M.PJ.urface of the head to separate the sesamoids.

6. The epiphysis is on the base, so head osteotomies are permissible on


children; but watch the nutrient artery.

7. The ligaments of the 1 st MPJ are:


a. Medial collateral: intracapsular
b. Lateral collateral :intracapsular
c. Tibial plantar sesamoidal
d. Fibular plantar sesamoidal
e. Tibial sesamoidal: intracapsular
f. Fibular sesamoidal: intracapsular
g. Inter-sesamoidal: intracapsular
h. Deep transverse metatarsal
i. Capsule
8. The capsule itself:
a. The sesamoids are invested in the capsule through which the FHB runs
b. The capsule blends with the periosteum and may be removed easily from the
base with an elevator
c. The capsule is strongly attached to the metatarsal head
d. There are medial and lateral collateral ligaments
9. EHL attaches through the hood ligament and lifts the proximal phalanx
into extention (the EHB is under the hood ligament).
10. The hood ligament attaches the dorsal to plantar structures

11. The capsularis is actually a branch of the tibialis anterior

12. The only structure that never changes is the interosseous ligament

13. The bunion bump Is the abnormal position of the cartilaginous surface

14. Blood Supply to the 1st MTP and 1st metatarsal are via the 1st dorsal
and plantar metatarsal arteries and the superficial branch of the medial
plantar artery

Predisposing Factors
1. Biomechanical
a. Primarily genetic in nature
b. Acquired factors
2. Arthritic
a. RA
b. Psoriatic arthritis

3. Neuromuscular
a. Cerebral palsy

4. Traumatic (primarily hallux limitus and hallux rigidus)

5. Genetic:
a. Down's syndrome
b. Ehler-Danlos syndrome
c. Marfan's syndrome

Biomechanics
1. The first metatarsal dorsiflexes due to hypermobility and when
dorsiflexed it also inverts (the 1st ray axis is medial/proximal/dorsal to
plantar/lateral/distal).

NOTE* There is almost no transverse plane motion due to the


horizontal positioning of the axis

2. A foot with a low axis to the transverse plane gets a small amount of
abduction/adduction of the first ray as does a rectus foot. This type will
develop more of a dorsal bunion. The metatarsus adductus foot with a
higher 1st ray axis gets a more medial bunion.

3. As the 1st metatarsal dorsiflexes and inverts, the hallux which is held
to the ground by muscle power, is everting and dorsiflexing as well as
subluxing laterally due to adduction of the first metatarsal.

4. Due to poor weight bearing of the 1st metatarsal, the second gets
transfer lesions and the 1st develops a dorso-medial metatarsal head
hypertrophy to form a bunion.

5. This is an acquired condition due to abnormal pronation in an adducted


foot type or inflammatory disease of the joint which changes mechanics
and muscle direction around the joint due to edema.

6. There must be a propulsive phase of gait for HAV to develop.

7. A hypermobile 1st ray is the primary cause due to a pronatory force and
weakening of the peroneus longus.

8. Pronatory problems may develop from neuromuscular pathology, but a


neuromuscular problem may not show bunion development even with
pronation due to poor propulsion.
9. The loss of the normal tibial sesamoid position or excision of the tibial
sesamoid will hasten the development of HAV.

10. Factors that affect the rate of progression of HAV are:


a. The amount of pronation during propulsion
b. The amount of adduction of the forefoot
c. The amount of calcaneal eversion (a flatter STJ axis increases eversion and
hastens HAV development)
d. The amount of STJ and MTJ subluxation
e. The presence of inflammation of the 1 st MTPJ
f. The angle and base of gait
g. The stride length (amount of time in propulsion)
h. Obesity
i. Terrain
h. Poor fitting shoe gear

11. If you see HAV prior to age 3-4 suspect an anatomical anomaly or
neuromuscular problem (there is no active propulsion In this age group)

12. The hallux is prevented from following the metatarsal head due to the
need for ground purchase, therefore, at the 1st MTPJ torque is formed to
sublux the joint. A normal 1st MTPJ does not have frontal plane motion,
so as the metatarsal dorsiflexes and inverts, the stable hallux dorsiflexes
and everts, and subluxation begins.

13. The hallux loses stability at weightbearing due to the hypermobile


base, causing the lesser mets to carry more of the load.

14. Lateral subluxation of the hallux occurs due to weakening of the


peroneus longus and dorsiflexion of the metatarsal with the transverse
head of the adductor pulling on the hallux and enhancing its lateral
migration.

15. The sesamoids migrate laterally to change the abductory forces on the
hallux. This causes bony adaptation on the plantar metatarsal head so
that the crista is gradually worn away, to allow further lateral shift of the
sesamoids.

16. Bony adapation shows medial deposition and lateral absorption of the
metatarsal head.

17. The metatarsal head develops a groove (sagittal groove) where the
phalanx now articulates. This allows for normal plantar/dorsiflexion in
the plantar part of the metatarsal head, but on the dorsal part you get
abduction and eversion due to the new bony adaptation.

18. As metatarsus adductus increases, HAV develops. The angulation in


the met-cuneiform joint helps this progression.

19. If the bunion develops lateral pressure on the lesser toes look for:
a. Onychocryptosis of the fibular nail groove of the hallux
b. Interdigital soft corn
c. Hammertoes and underlapping digits.

20. A stable hallux against the second digit causes the retrograde muscle
contraction to cause the 1st ray to adduct, therefore, the IM angle
Increases (this causes the met-cuneiform split).

21. As long as you have a propulsive gait the HAV worsens. It stops when
you have a plodding type of gait.

22. After a long time of dorsiflexion of the 1st ray, there is an adaptation
of the 1st ray into a position of elevatus that is somewhat rigid.

23. The medial branch of the superficial peroneal nerve may develop
neuritis due to trauma.

24. Rheumatoid complications that help the development of HAV are:


a. Intra-articular swelling causes the long tendons to migrate laterally
b. Intrinsic muscle spasm causes the sesamoids to migrate laterally
c. Pain of the lesser metatarsals may cause excess pronation to get the weight
off the painful area

Etiology of Hallux Abducto Valgus


1. Excessive pronation during stance phase results in
a. Hypermobile 1st ray due to loss of proper P. longus mechanics
i. Results in dorsiflexion/varus of the 1st MTPJ and mild adduction of the
metatarsal
ii. A stretching and loosening of the medial sesamoid ligaments and adaptive
tightening of the fibular sesamoid ligament (also some lateral subluxation of the
hallux)(GRADE 1)
iii. Valgus hallux due to continuation of 1st metatarsal medially and abnormal
lateral tracking of the sesamoid
iv. Crista starts to erode and transverse adductor helps pull hallux over along
with increased ground reaction force from hypermobile first ray
v. Abductor hallucis becomes displaced plantarly, as the crista continues to
erode and the sesamoids move laterally to change the mechanical advantage of
the adductor hallucis and flexor hallucis brevis to decrease the stability of the
hallux
vi. The hallux moves laterally and buttresses the second toe (GRADE 2)
vii. The IM angle becomes increased due to the retrograde force from the
abductor hallucis position (GRADE 3)
viii. The PASA increases adaptively, the crista erodes more as the joint pain
increases and the lateral metatarsal head degenerates
ix. Eventually there is complete erosion of the crista and fast exacerbation of
the HAV deformity
x. The hallux becomes subluxed and dislocated on the 1st metatarsal (GRADE
4)
xi. Marked HAV deformity

NOTE* The longer it takes to change from one stage to the next, the less severe
the resultant deformity will be

b. Tibial sesamoid tracking problems


i. Crista erodes

NOTE* When the crista erodes you can transfer the adductor hallucis to the
tibial sesamoidal ligament to hold the sesamoids in the proper position

ii. Chondromalacia
iii. Osteoarthritis
iv. Fibrosis of the 1st MTPJ
v. Marked HAV deformity
1. Structural Deformity (bony): A deformity in which there is osseous change
in either the PASA the DASA or both (so either the PASA or DASA are
abnormal). The structural deformity has as its characteristics the congruous
joint. The HA deformity is also equal to the summation of the PASA and the
DASA.

2. Positional Deformity (soft tissue): There is an abnormality in the HA angle.


The PASA and DASA are normal. The joint is either deviated or subluxed. The
summation of the PASA and DASA are less than the HA angle.
3. Combined Deformity: Has elements of both structural and positional
deformities. Either the PASA or the DASA or both are abnormal, and when
added together they do not equal the HA angle. The joint is either deviated or
subluxed.

NOTE* Structural deformities may exist at levels other than the 1st
metatarsophalangeal joint and contribute to the metatarsophalangeal
deformity

Radiological Measurements For HAV


1. Hallux abductus interphalangeal angle = normal 0-10°
i. When abnormal, a structural abductus deformity of either the head of the
proximal phalanx or the distal phalanx base or both may be present. However,
the abnormality is most often seen at the head of the proximal phalanx, and
regardless of the site is corrected via osteotomies. Valgus rotations falsely affect
the HAI

2. Hallux abductus angle= normal 15°


i. Significance: This measurement is the prime method of quantifying the
abductus in a HAV condition. Can be either structural, positional, or combined

NOTE* The HA angle is a combination of PASA+DASA+JOINT DEVIATION

3. Distal articular set angle= normal 0-10°


i. Significance: DASA is a structural component of HA. When abnormalities are
detected with this measurement, osteotomies of the hallux proximal phalanx
are indicated

4. Proximal articular set angle= normal 7.5°


i. Significance: PASA represents a structural component of hallux abductus,
and is an attempt to quantify the structural adaptation of the 1st MTPJ

NOTE* Situations exist where PASA of 12° is normal.


Functional vs. Dysfunctional PASA:
PASA remains functional so long as PASA is less than the IM angle or
Dysfunctional PASA= PASA - IM
Therefore, if the PASA exceeds the IM angle, the PASA should be corrected by
at least the number of degrees difference between the two angles. In cases
that require correction of the IM angle by base/shaft procedures, the
anticipated postoperative IM angle must be determined.

5. 1st Metatarsal Declination Angle = normal 15-20°


i. Significance: Gives information regarding sagittal plane position

6. Joint position: The 1st MTPJ is either parallel, deviated or subluxed


7. Metatarsus primus adductus angle (intermetatarsal angle)= normal 8-12°
in the rectus foot and 8-10° In the adductus foot
i. Significance: When the angle is abnormally increased the condition may be
termed metatarsus primus adductus. The MA angle may determine the
significance of the IM angle. Adducted foot types require correction of the IM at
a lesser angle, therefore, the adducted foot type will require a base/shaft
procedure at lower degrees than a rectus foot type.

8. Metatarsus adductus angle (MA)= normal under 15°


i. Significance: The MA determines the significance of other measurements (IM,
HA, and PASA). MA is influential in the choice of procedures. MA feet tend to
develop HAV more rapidly than rectus feet

9. Metatarsal protrusion distance (MPD)= normal +2 mm. to -2 mm.


i. Significance: An excessively long metatarsal may be the etiologic factor in
hallux limitus or HAV, and an excessively short metatarsal can cause 2nd
metatarsalgia. Procedures should be selected that avoid shortening the MPD or
if unavoidable combine with a plantarflexing procedure

10. Tibial sesamoid position= normal 1-3


1. Significance: TSP documents the stage of progression of HAV. The plantar-
axial view is best to determine the exact position of the sesamoids with regard
to their position in their respective grooves

11. First metatarsal base gapping= normal Is less than a 2mm gap

Physical Examination
1. The foot of the patient is evaluated on a weightbearing and on a
nonweightbearing attitude:
a. Weightbearing:
i. EHL contracture
ii. Hallux purchase
iii. Hallux position with regard to resting calcaneal stance
position/neutral calcaneal stance position
iv. Radiographic analysis (including mineralization, osteophytes, and
cysts)
b. Nonweightbearing:
i. 1st ray ROM
ii. Manual reduction of IM
iii. 1 st MTPJ ROM, quality of motion, and axis of motion
iv. Location of pain
v. Location of bunion
vi. Hallux position relative to 2nd toe
vii. Calluses
viii. EHL contracture (rare)
ix. Associated deformities (2nd hammertoe, etc.)
x. Trackbound (position the toe in a rectus position and dorsiflex and
plantarflex- there is inability for adequate ROM with resultant pain)
2. A complete vascular, neurologic, dermatological, and biomechanical
examination should additionally be performed.

Preoperative Considerations (planning)


1. Decide upon the right choice of procedure based upon radiological and
clinical assessment
2. Remember that capsulotomies will not correct a structural deformity
3. Use a template in preoperative planning if you are unsure
4. Use fixation method which you are most comfortable with and that will
provide the best stability
5. Try to minimize complications with proper execution of procedure and
normal postoperative recovery period
6. Review the risks and consequences thoroughly with the patient

Arthroplasty Procedures
1. Keller
a. Indications
i. HAV with degenerative joint disease
ii. Second and third degree hallux rigidus

Note* Second degree hallux rigidus


a. Established arthrosis
b. Painful ROM
c. Loss of central articular cartilage
d. Proliferation and enlargement of peripheral osteophytes
e. Ankylosis
Third degree hallux rigidus
a. Complete disappearance of the joint space
b. Ankylosis of the MTPJ
c. Sign (cant hypertrophy of the joint
d. Irregular hypertrophic osteophytes

iii. Geriatric bunion


b. Advantages
i. Relief of pain
ii. Establish/restore 1st MTPJ ROM
iii. IM reduction of 3-5°
iv. Minimal rehabilitation
v. Early return to shoes
c. Disadvantages
i. Short hallux
ii. Loss of purchase
iii. Sesamoid retraction
iv. Altered gait
v. Limited ROM
vi. Limited IM reduction
d. Complications
i. Hallux extensus
ii. Loss of purchase
iii. Retraction of the hallux with loss-of joint space
iv. Metatarsalgia/stress fracture 2nd metatarsal and/or 3rd metatarsal
v. Edema/telescoping of the hallux base
e. Modifications
i. Reattachment of the flexor brevis
ii. EHL lengthening
iii. Purse-stringing the capsule
iv. Capsular flap
v. K-wire splinting
vi. Implant arthroplasty
vii. Anchor the FHL to the sesamoids and the base of the proximal
phalanx

2. Hueter: Complete resection of the 1 st metatarsal head, usually performed


as part of the Hoffman-Clayton

3. Mayo: Resection of one-quarter inch of the 1 st metatarsal head and medial


eminence.

4. Stone: Remodeling of the 1st metatarsal head. An oblique plane resection of


the head, leaving a three-quarter inch space at the 1st MTPJ

5. Regnauld: Resection of a portion of the base of the proximal phalanx with


the fashioning of a "hat-shaped". inverted, or "cork shaped" autogenous graft.
This procedure besides shortening a long proximal phalanx and increasing
ROM in the presence of DJD, will also correct a mildly abnormal DASA, mildly
abnormal HIA by angling the surface of the base of the proximal phalanx.

6. Valenti "V" Resection:


a. Indications
i. Hallux valgus
ii. Hallux limitus in the elderly
iii. Nonactive patient
iv. Hallux rigidus in cases of implant salvage
b. Surgical technique: A "V" shaped osteotomy at the head of the 1st metatarsal
and the base of the proximal phalanx (a 45° angular resection of the metatarsal
and phalanx) on the dorsal and plantar aspect. A dorsal U-shaped capsular flap
perserves intrinsic attachments at the base of the proximal phalanx. EHL
lengthening
c. Advantages:
i. Restoration of ROM
ii. Preservation of hallux purchase
iii. Immediate weightbearing

Capsule-Tendon Balance Procedures


1. McBride: Originally described with an incision lateral to the EHL, excision of
fibular sesamoid, medial eminence removed, adductor tendon transferred to the
lateral side of the metatarsal head, and the medial capsule and adductor
tendon shortened (several modifications to date)
a. Criteria: Structural deformities of the 1st ray should not be present unless
corrected by other procedures
i. Normal HIA
ii. Normal DASA
iii. Normal PASA
iv. Normal IM angle
v. Hypertrophied medial eminence
vi. Deviated to subluxed 1st MTPJ
vii. Deviation of sesamoids >4 (fibular sesamoid may be arthritic)
viii. Increased HA angle
b. Surgical technique:
i. A 6 cm. dorsolinear incision parallel and medial to the
ii. Inverted "L" capsulotomy
iii. Medial eminence removed
drilling with .045 k-wire to produce fibrocartilage
iv. If erosions in the cartilage are found then perform subchondral
v. Removal of the fibular sesamoid only if it is acting as a deforming force and
an adductor transfer is not being done (just freeing the attachments is not
enough)
vi. Release OR transfer of the Adductor tendon
 The tendon is pulled over the 1st metatarsal neck and sutured into the
medial capsule. This repositions the sesamoids
 The tendon is tied into the metatarsal neck and into the medial capsule (this
helps close the IM angle
vii. Cut EHB

NOTE Do not cut the adductor and remove the fibular sesamoid, you will
predispose to a hallux varus

b. Contraindications:
i. Structural deformities
ii. Hallux rigidus
iii. DJD
c. Complications:
i. Stiff joint
ii. Undercorrection of the deformity
iii. Hallux varus

2. Silver:
a. Indications:
i. Hypertrophied medial eminence
ii. Mild HA angle
iii. No other structural abnormalities
b. Surgical technique: Removal of the medial eminence
c. Complications:
i. Stiff joint
ii. Increase on HA angle following surgery due to cutting of medial stabilizing
structures

3. Hiss: Like the Silver, plus the abductor hallucis is repositioned on the medial
aspect of the 1st MTPJ (trying to balance the abductory component)

Implant Arthroplasties

Implants utilized for the 1st MTPJ are static spacers which relieve pain, allow
for limited painless motion, and give some internal stability to the joint.
Ideally, biocompatibility requires the implant to be:
 Chemically inert or free from biodegradation and sterile
 capable of withstanding stresses imposed upon it
 Durable or possess an integrity of structure without modification of its
physical properties because of the biological environment
 Non-irritation, eliciting only a benign local tissue response
General indications for 1 st MTPJ implant arthroplasty:
 Hallux valgus with subluxation and painful limited ROM
 End stage hallux rigidus
 Revisional surgery
 Rheumatoid arthritis
 Painful DJD
 Gouty arthrosis/arthritis
 Osteochondral fractures
 Intra-articular fractures
General contraindications for Implant arthroplasty:
 infection
 Salvagable joint

1. Hemi implant:
a., Criteria:
i. Joint pain
ii. Adequate bone stock of the proximal phalanx
iii. Normal IM angle unless reduced by another procedure
iv. Normal articular cartilage of the head of the 1st metatarsal (no DJD)
v. Adequate capsular tissue to allow for implant coverage
b. Complications:
i. Implant instability: Pistoning can occur from removal of too much bone as
well as axial rotation of the implant
ii. Implant failure: Mechanical stress can produce microfragmentation with
migration of the silicone particles into the lymphatic system. With this there
will be obvious loss of function and possible deformity. Not so with titanium
implants
iii. Foreign body reaction: lymphadenopathy which is reminiscent of metastatic
Ca clinically
iv. Osteochondritis dessicans: From excessive stripping of the periosteum and
resultant avasular necrosis
v. Detritic synovitis reaction: The surgical area will become red and swollen
with a chronic low grade pain. Once infection is ruled out the patient can be
treated with NSAIDS or remove the implant device (less chance of this with
titanium implants)
vi. Infection: Implant must be removed and not replaced for at least 6 months
to 1 year. If gram negative infection was present, implant should not be
replaced for longer period of time if at all
vii. Pistoning of the implant into cancellous bone (if implant chosen is too small)
viii. Chronic edema
c. Contraindications:
1. DJD of the 1st metatarsal head contraindicates a hemi implant
ii. Severe osteoporosis of the involved bones
iii. Inability to correct a high IM angle
iv. History of a prior joint infection within the last 6 months
v. History of allergic reaction to implant material

2. Hemi-angled Implant (Well): for accommodation up to 15° lateral


deviation of the 1st metatarsal articular surface

NOTE* This device eliminates the "lateral gap sign" as seen with the standard
hemi-implant, which is. the nonarticulation between hemi- implant and lateral
metatarsal articular surface

a. Criteria: As above plus


i. Abnormal PASA
ii. Revisional surgery for hallux varus (reverse the implant)
b. Complications: As above

3. Total Swanson Implant: Manufactured by Dow-Corning Wright, comes in


7 anatomical sizes plus the addition of titanium grommets. A central
hinge in a "U" configuration allows for dorsiflexion. An intramedullary
stem on both sides of the implant stabilizes the implant within the
medullary canals. There is no angulation of the stems in the sagittal or
transverse plane. There is a short stem version available in 6 sizes
a. Criteria: As above plus
i. Good bone stock of the 1 st metatarsal head
ii. Degenerative changes of the 1 st metatarsal head iii. Abnormally low metatarsal
declination angle
b. Complications: As above
c. Contraindications: As above

4. Total Lawrence Implant: Manufactured by Sutter Biomedical and is marketed In


4 sizes. The proximal stem is angled 150 dorsally in the sagittal plane to allow for
normal 1 st metatarsal declination, without sacrificing excursion of the implant to
compensate for this. It is designed for a ROM of 85° of dorsiflexion
a. Criteria: As above plus
i. Normal 15° declination angle
b. Complications: As above
c. Contraindications: As above

5. Total LaPorta Implant: Manufactured by Sutter Biomedical, comes in


right, left, and neutral, referring to the deviation of the stems on the
transverse plane. The right and left implant show a 100 angulation
(lateralward) on the transverse plane. It is designed for a ROM of 60°
a. Criteria: As above plus
i. Normal 150 declination angle or use a neutral LaPorta implant for an
abnormally low metatarsal declination angle.
ii. Can be used for a mildly elevated IM angle without performing a procedure to
reduce it, due to the 100 transverse plane abduction built into the proximal
stem
b. Complications: As above
c. Contraindications: As above

6. Total Bioaction Implant: Newer two-piece device made of high density


polyethelene, cobalt steel and titanium.
a. Criteria: As per total Swanson implant
b. Complications: As above
c. Contraindications: As above

7. Kinetic Great Toe Implant: Two piece titanium/cobalt chromium


implant with an anatomic dorsal flange providing an anatomic range of
motion
a. Criteria: as above
b. Complications: as above
c. Contraindications: as above

NOTE* This device also may be used to replace previous implants that have
failed. This system offers implants that are anatomically and biomechanically
accurate, with instrumentation that is precise to aid in its installation.
The system consists of four phalangeal components which articulate with all
six metatarsal (three left/three right) components. The metatarsal component is
made from cobalt chromium (CoCr) which is the same material used for the
femoral component of the total knee inn plant. The phalangeal component is
made form titanium alloy as the backing to a high molecular weight
polyethylene bearing surface

NOTE* It is mandatory that all patients after receiving the implant arthroplasty
be advised that prior to any invasive procedure (medical/ dental) that the
patient be given prophylactic antibiotics, as is the case with all implanted
prosthetic devices

Arthrodesis
1. McKeever Type (fusion of the 1st m.p.j.): Originally described as a peg-in-
hole 1st mpj fusion.
a. Criteria:
i. Flail toe
ii. Failed implant arthroplasty
Iii. Arthritis/gout
iv Severe hallux valgus deformity
v. Loss of extensor and/or flexor function
vi. Intra-articular fractures with post-traumatic arthritis
vii. Previously failed bunion procedures
viii. Failed Keller arthroplasty procedure
ix. Prior infection/septic arthritis
x. Hallux limitus/rigidus
xi. Rheumatoid arthritis
xii. Charcot joint/osteoarthropathy
xiii. latrogenic hallux varus
xiv. Tumor
b. Ideal position of fusion
i. Slight dorsiflexion (5-10° from the ground supporting surface)
ii. Slight abduction (parallel to the 2nd toe) or 10-15° on the transverse plane
iii. No frontal plane valgus or varus rotation

NOTE* Position will vary depending on activity, lifestyle, and shoe gear

c. Surgical technique:
i. Cartilage denuding
ii. Metatarsal head resection ( as part of pan-metatarsal head resection)
iii. With bone grafting (i.e. failure of implant cases)
d. Possible ancillary procedures:
i. Tibial or fibular sesamoidectomy
ii. IPJ arthrodesis (most common)
iii. Relocation and arthrodesis of the lesser toes
iv. Possible metatarsal osteotomies
v. Possible Hoffman-Clayton
vi. EHL lengthening
vii. Excision of ipj sesamoid
viii. CBWO/ Lapidus/ OBWO
e. Fixation types:
i. Insertion of crossed .045 k-wires
ii. Monofilament 28 gauge wire loops
iii. 4.0 cancellous screw
iv. 3.5 Cortical screw in a lag technique
v. 2.7 mm. screw in a lag technique
vi. Herbert screw
f. Advantages:
i. Preserves adductor, short flexor, and EDB ms. function
ii. Improved cosmetic appearance
iii. Improved stability
iv. Improved overall balance and gait
vi. Improved position of lesser toes
vii. Restores and maintains weightbearing function to the 1st ray
viii. May be converted to Keller or implant arthroplasty
ix. Relief of pain
x. Simultaneous reduction of IM angle
xi. Recurrence of the deformity is unlikely
g. Disadvantages:
i. May promote arthritic changes at the ipj
ii. Optimum position may be difficult to achieve as it is technically
difficult to perform
iii. May require autogenous graft
iv. May limit shoe gear
v. May result in gait alterations
v. Difficulty in kneeling
h. Complications:
i. IPJ arthritis of the hallux
ii. Delayed union/non-union/malunion/pseudoarthrosis
iii. Fracture
iv. Onychocryptosis
v. Medioplantar calluses
vi. Hallux flexus
vii. Impaired gait
viii. Balance problems
ix. Possible subluxation of toes 2 and 3 x. Improper positioning
2. Lapidus Type (fusion of the 1st metatarsal-cuneiform joint):
a. Anatomy:
i. The joint is arthrodial with its own synovial membrane
ii. The medial cuneiform distal articular surface is reniform in shape with a
convex medial border and concave lateral border
iii. The surface is elongated in the vertical direction with an inferior and medial
inclination
iv. The proximal articular surface of the 1st metatarsal is reniform with upper
and lower parts
v. There is a variable articulation between the 2nd metatarsal at its
proximal medial aspect and the first metatarsal at its proximal lateral aspect
vi. Dorsal and plantar 1st metatarsal-1st cuneiform ligaments are present
vii. There are no interosseous ligaments between the base of metatarsals
1 and 2
viii. There is a strong interosseous ligament between the 1st and 2nd cuneiform
and between the 1st and 2nd cuneiform and 2nd metatarsal
ix. Tibialis anterior tendon inserts at the proximal junction of the medial and
inferior surfaces of the 1st metatarsal
x. Peroneus longus tendon inserts at the proximal junction of the lateral and
inferior surfaces of the first metatarsal
b. Indications:
i. Extreme hypermobility associated with HAV
ii. Correction of sagittal plane deformity
iii. Correction of severe metatarsus primus adductus associated with a
hypermobile or structurally medially deviated M-T-C joint
iv. Repair of fracture or dislocation
v. Hypermobile flatfoot with medial column sag
c. Surgical technique: Via a dorsal longitudinal incision medial to the EHL
tendon, with capsular incision dorsal linear or transverse. Articular cartilage is
resected from the joint surfaces. The metatarsal is then adducted and slightly
plantarflexed and then fixated appropriately with a compression screw, staple
or k-wires. A BK NWB cast is applied until radiographic signs of fusion and
stability are seen. Originally, Lapidus fused the 1st and 2nd metatarsal bases,
but this is rarely necessary. Modifications of this procedure include
i. Transfixation of the 1st to 2nd metatarsals with a threaded k-wire
ii. Fusion of the 1st to 2nd metatarsal base with bone graft obtained from the
medial exostosis of the metatarsal head

NOTE* There exist 3 surgical alternatives for extreme hypermobility cases:


a. Cartilage abrasion and subchondral perforation, manual reduction,
temporary fixation followed by bone screw fixation
b. Joint wedge resection, minimal bone resection, lateral plantar wedge with
bone-screw fixation and -stress receiving graft
c. Joint resection, minimal bone removal with temporary reduction of the
lengthening by means of allogeneic bone, double screw fixation, and occasional
temporary plates.
With each, there must be at least two points of fixation
d. Complications:
i. Prolonged healing time
ii. Malalignment in the frontal, sagittal or transverse planes
iii. Severence of the vascular structures in the proximal intermetatarsal space
iv. Non-union and pseudoarthrosis

Proximal Phalangeat Osteotomies


The Akin-type procedures are generally combined with other types of bunion
procedures to correct deformities around the 1st MTPJ, however, the cylindrical
Akin is often used independently to shorten a long proximal phalanx
1. Proximal Akin: It is a medial closing wedge osteotomy at the metaphysis of
the proximal phalanx. Is used to correct a high DASA. There should be
adequate bone stock and adequate length of the proximal phalanx. The IM and
HA angles will be unaffected. The lateral hinge is kept intact and the osteotomy
should be fixated.

2. Distal Akin: It is a medial closing wedge osteotomy of the distal part of the
proximal phalanx used to correct a high HI angle. IM angle unaffected.
a. Indications:
i. Pressure of the hallux on the 2nd digit
ii. Epiphysis may be open
iii. Good bone stock
iv. HIA > 100
v. Adequate length of proximal phalanx
vi. DASA is normal
vii. Congruous 1st mpj unless corrected by another procedure
b. Disadvantages:
i. Long healing phase
ii. Fixation needed
iii. Elimination of propulsive phase of gait for 3-6 weeks
c. Complications:
i. Poor correction
ii. Pain postop due to poor fixation
iii. Non-union
iv. Short hallux
v. Hallux elevatus from FHL damage

3. Cylindrical Akin: Removal of cylindrical section to shorten a long proximal


phalanx. The proximal osteotomy is one and one-half cm. from the base of the
proximal phalanx, and the second osteotomy is made distally to the first cut.

4. IPJ fusion: Used to correct a hallux hammertoe or injury to the ipj. Must
have good ROM of the 1st MTPJ.

5. Kessel-Bonney: A dorsal wedge osteotomy of the base of the proximal


phalanx, used for hallux limitus. This extends pre-existing joint motion more
dorsally.
a. Advantages:
i. Allows the hallux to be in a dorsal position at the propulsive phase of gait
ii. Allows for immediate ambulation
b. Disadvantages:
i. Requires an osteotomy with fixation
ii. Does not really increase overall 1 st MTPJ ROM iii. Often creates a lack of toe
purchase
iv. Does not correct the underlying etiology of the deformity
v. Requires elimination of the propulsive phase of gait for 3-6 weeks
Distal Metatarsal Osteotomies

The major criteria for all distal osteotomies of the 1st metatarsal head are
adequate bone density, adequate ROM, and arthritis-free joint. The potential
complication for all distal 1st metatarsal osteotomies is avascular necrosis.

1. Austin:
a. Criteria
i. Normal to mildly abnormal PASA
ii. Increased IM angle (max 14°/depends on other factors)

NOTE* 1 mm in lateral shift of the capital fragment equals a reduction of 1° of


IM angle.

NOTE* One can safely shy the metatarsal head laterally one-third of the width
of the bone. So if the metatarsal head measures 21 mm. across, you can safely
shy the bone 7 mm. and thereby close the IM angle 7° (since the wider the
bone the more lateral shifting you can perform)

iii. Normal metatarsal declination


b. Surgical technique: V osteotomy with 60° angular cuts, originally
described with no fixation, but commonly fixation devices utilized.
c. Advantages
i. Reduces the IM angle
ii. Performed in cancellous bone
iii. Stable in the sagittal plane allowing early ambulation
iv. Avoids the sesamoids
d. Disadvantages
i. Technically challenging (easier with Reece osteotomy guide)
ii. Dislocation potential
e. Complications
i. Dislocation of capital fragment
ii. Intra-articular fracture
iii. Aseptic necrosis
NOTE* It has been reported in the literature that an unusual sequel of this
procedure has been for the capital fragment to pop out of the wound and fall on
the operating room floor. The suggested protocol for this is to:
a. Pick up the fragment with a sterile forceps and place in a basin containing 1
liter of sterile saline + 1 cc. Neosporin G. U. irrigant + 1:100, 000 Bacitracin for
5 minutes.
b. Then transfer to another basin with the same type of solution for another 5
minutes.
c. Then transfer to a third basin containing the same solution and swirl for an
additional 1 minute and replace back into the foot with fixation.
d. Advise the patient of the occurrence.
e. Prophylaxis with 1 gm IV Cefadyl at 8, 16, and 24 hours postoperatively.

2. Austin/Kalish modification: As above but with a smaller angular cut to


allow for longer dorsal wing for placement of 2-2.7mm screws or one 3.5 mm
screw

3. Austin/Youngswick modification:
a. Criteria
i. Normal to slightly abnormal PASA
ii. Increase IM angle (mild)
iii. Metatarsus elevatus
iv. Limited ROM
b. Operative technique: Similar to other Austin procedures except with
removal of rectangular wedge from dorsal arm of the 600 angle of the V.
Additionally the wedge can be placed in the plantar arm of the V cut for
additional plantarflexion of the capital fragment.
c. Advantages
i. Plantarflexes
ii. Shortens- relaxes the tension around the joint, thereby increasing
ROM
iii. Stable if fixated
iv. Done in cancellous bone
d. Disadvantages
i. Potential displacement
ii. Difficult to perform
iii. Needs fixation
iv. Should be non-weight bearing 4-6 weeks
e. Complications
i. Intra-articular fracture
ii. Excessive shortening
iii. Dorsal displacement
iv. Metatarsalgia

4. Austin/Bicorrectional modification:
a. Criteria
i. Abnormal PASA
ii. Increased IM angle
iii. Normal metatarsal declination
b. Operative technique: Same as traditional Austin except another cut is made
which is thicker medially so as to remove a trapezoidal wedge of bone from the
dorso-medial metatarsal head. When! the capital fragment is shifted laterally
and impacted the IM and PASA is subsequently reduced.
c. Advantages:
i. Same as the Austin plus
ii. Reduces the PASA
d. Disadvantages
i. Technically challenging (more so than the traditional Austin)
ii. Dislocation potential
e. Complications
i. Same as the traditional Austin

5. Reverdin:
a. Criteria
i. Increased PASA
ii. Normal IM angle
iii. Normal metatarsal declination
b. Advantages:
i. Reduces the PASA
ii. Performed in cancellous bone
iii. Fixation optional
iv. Good visualization
c. Disadvantages:
i. Potential sesamoid trauma
ii. No IM angle reduction or sagittal plane reduction
d. Complications: DJD

6. Reverdin-Green (distal "L" osteotomy): This is identical to the Reverdin


except with an additional plantar osteotomy cut made parallel to the weight-
bearing surface to protect the sesamoids

7. Reverdin-Laird: Adds transposition of the metatarsal head to the Reverdin-


Green procedure, with reduction of the IM angle

8. Reverdin-Todd: Combined with the Reverdin-Green and Reverdin-Laird, this


modification adds correction of the sagittal plane deformity of the 1st
metatarsal

9. Hohmann: Its modifications are the Mitchell and the DRATO


a. Criteria: Sagittal and transverse plane deformities
i. Increased PASA
ii. Increased IM angle
iii. Elevatus
b. Precautions: Cannot be used for frontal plane deformities, in the presence of
degeneration of the crista or any DJD
c. Advantages:
i. Reduces PASA
ii. Reduces IM angle
iii. Plantarflexes
d. Disadvantages:
i. Highly unstable
ii. Needs 2 point' fixation
iii. Cast required
iv. Extracapsular
v. Performed in cortical bone
vi. Does not remove the medial eminence
e. Complications:
i. Dislocation
ii. Delayed or non-union
iii. Metatarsus elevatus

10. Mitchell: A step down transpositional/angulation osteotomy at the


metatarsal neck
a. Criteria:
i. Normal PASA
ii. Moderate increase in IM angle
iii. Positive metatarsal protrusion
iv. Metatarsal elevatus
b. Advantages:
i. Reduces the IM angle
ii. Removes the medial eminence
iii. Plantarflexes the met head
iv. Avoids the sesamoids
v. More stable than the Hohmann
c. Disadvantages:
i. No PASA correction
ii. Shortens the metatarsal
iii. Performed in cortical bone
iv. Dislocation potential

11. DRATO:
a. Criteria:
i. Abnormal PASA
ii. Mild increase of IM angle
iii. Valgus rotation of the metatarsal head
iv. Sagittal plane deviation of 1st metatarsal head articular cartilage (facing
either plantarward or dorsalward)
b. Advantages: It addresses 4 deformities in one procedure (PASA, sagittal plane
1st met head, valgus rotation, and IM angle)
c. Disadvantages:
i. Very difficult to perform
ii. Done in cortical bone
iii. Cast/ NWB 6-8 weeks
d. Complications: Same as with other procedures of this type

12. Waterman: Distal metaphyseal dorsal wedge osteotomy that raises the
declination of the metatarsal head. Utilized for hallux limitus.
a. Disadvantages:
i. Does not actually increase the 1st MTPJ ROM
ii. Can create a lack of hallux toe purchase
iii. Does not correct the underlying etiology
iv. Requires removal of the propulsive phase of gait for 3-6 weeks postop.

13. Wilson: An oblique displacement osteotomy that shortens the 1st


metatarsal and decreases the IM angle.

14. Peabody: Similar to the Reverdin except performed more proximally.

15. Lambrinudi: A plantarfiexory osteotomy of the 1st metatarsal. Reserved for


the younger active individual without severe joint disease with hallux limitus
complaints.

Proximal Metatarsal Osteotomies:

These procedures are used when the true IM angle should be reduced
(greater than or equal to 140 in the rectus foot) and/or when a sagittal plane
deformity of the first ray needs to be corrected. These procedures are
performed transversely across the metatarsal, perpendicular to the long axis,
and 1-1.5cm distal to the met-cuneiform joint.

1. Trethoan (opening base wedge): Used when the metatarsal is relatively short.
Must utilize a bone graft (best to fixate with a staple to avoid compression of the
bone graft).

2. Louisan-Balacescu (closing base wedge): A transverse closing base wedge


osteotomy. Fixated with a staple. osteoclasp, K-wires, or monofilament wire.
Complications have been elevatus of the metatarsal, non-unions, and delayed
unions. The Juvara modifications allows for AO fixation.

3. Juvara Type A: Oblique base/shaft osteotomy from medial-proximal to


lateral-distal, a wedge of bone removed to close the IM angle, medial hinge is
kept intact, and screw fixation utilized.

NOTE* For screw fixation the osteotomy cut is performed obliquely and must be
at least twice as long as the width of the metatarsal shaft

4. Juvara Type B: As in type A, a wedge is removed for IM angle reduction, also


the hinge is broken after screw insertion for sagittal plane rotation.

5. Juvara Type C: No wedge is removed but the hinge is broken, so that


sagittal plane rotation is possible and axial transposition is possible
(lengthening or shortening).

NOTE* Biomechanics of the Axis:


a. When the axis lies in the plane, no motion occurs in that plane.
b. Motion around an axis occurs in a plane that is perpendicular to the axis.
c. An osteotomy hinge will act as an axis of rotation.
d. A hingeless osteotomy may be rotated in the plane in which the osteotomy
lies (an axis of rotation will exist perpendicular to the plane of the osteotomy)
e. A hinge should be made perpendicular to the weight bearing surface to
prevent subsequent dorsiflexion of the metatarsal.
f. More significant sagittal plane errors will occur when the hinge erroneously
deviates in the frontal plane
g. Virtually any combination of planal deviations may be obtained if the hinge
(or in the case of hingeless osteotomies) the plane of the osteotomy is correctly
manipulated (the problem is achieving the exactly desired deviation on each
plane).

6. Arcuate (Weinstock): A proximal dome shaped osteotomy that reduces a high


IM angle and sagittal plane deformity. No cortical hinge is left intact. Its benefit
is that it avoids shortening the metatarsal. Its drawback is that it is unstable
and occasionally difficult to fixate.

7. Van Ness: Plantar closing base wedge osteotomy for reduction of metatarsal
elevatus. K-wire fixation utilized.

NOTE* Fixation studies of proximal metatarsal osteotomies revealed:


a. Dorsal loop: failed at 1 lb pressure
b. Dorsal loop with .062 k-wire: failed at 3.5 lb pressure
c. Right angle loops: failed at 6 lb pressure
d. Crossed. 062 k- wires: failed at 6 lb pressure
e. 4 mm. cancellous screw: failed at 8 lb pressure
f. 3.5 mm. cortical screw: failed at 9.5 lb pressure
g. Two 2.7 mm. cortical screws: failed at 10 lb pressure (gives better and more
even compression)
h. Right angle loops with .062 k-wires: failed at 10 lbs pressure

Shaft Procedures
1. Offset V (Vogler):
a. Criteria:
i. IM angle of 17-18°
ii. PASA between 8-30°

NOTE* It has been reported that there is an average reduction in IM angle of


9.1° and PASA of 7.99°

iii. Minimal to no joint pain


iv. No excessive DJD
b. Operative technique: A long V osteotomy that extends almost to the met-
cuneiform joint enabling reduction of the IM angle and PASA. The apex of the V
is more proximal than the standard Austin (done at the metaphyseal-
diaphyseal junction) with the angle measured at 35°. The dorsal arm is longer
extending between midshaft and the proximal one-third of the metatarsal.
Fixation is achieved with 1 K-wire or 2.7 mm. screw.
b. Complications: As per most osteotomies of this type

2. Scarf:
a. Criteria:
i. Increased IM angle
ii. Increased PASA
b. Operative technique: A long Z osteotomy that reduces the IM angle and
the PASA. Modifications of this procedure include:
i. A medially based incision
ii. The length of the horizontal osteotomy is in direct proportion to the
width of the IM angle
iii. The direction of the osteotomy has been modified such that approx.
two-thirds of the width of bone is dorsal to the osteotomy, proximally,
and one third is plantar, in order to prevent stress fractures from occurring
c. Advantages:
i. The long plantar fragment possesses stability in two planes
ii. Lends itself to AO fixation (tension band effect)
d. Disadvantages:
i. Difficult to perform
ii. Possible vascular compromise
iii. Performed in diaphyseal bone
e. Complications:
i. Aseptic necrosis

3. Mau (Gudas modification for screw fixation): An oblique shaft osteotomy from
plantar-proximal to dorsal-distal. Indicated for IM angle reduction.

4. Ludloff (Engelman modification for screw fixation): An oblique shaft


osteotomy from dorsal-proximal to plantar-distal

Combination Procedures
1. Stamm (opening base wedge + Keller)
2. Logroscino (closing wedge + Reverdin)

Other Procedures
1. Cotton (1st cuneiform opening wedge osteotomy):
a. Criteria:
i. Metatarsus primus adductus
ii. Pronounced obliquity of the 1st met-cuneiform joint
iii. Whenever a double osteotomy is indicated
b. Contraindications:
i. Excessive bony bridging at the opposing surfaces of the osteotomy
c. Operative technique: A wedge is removed from the 1st cuneiform. The cut is
made parallel to the joint and does not enter the met-cuneiform joint plantarly,
and is placed tibial to the intercuneiform joint at the bases of the 1st and
second metatarsals. The 1st cuneiform osteotomy is then performed on the
distal 1/3 of the bone. The more distal the osteotomy the more effect it has on
the joint direction. The graft comes from the medial eminence and is fashioned
prior to removal of the cuneiform wedge.

NOTE* Dissection of the 1st cuneiform must be done carefully to avoid


avascular necrosis of this bone

d. Complications:
i. Inadequate reduction of the deformity
ii. Malalignment of the osteotomy
iii. latrogenic tenotomy of the FHL
iv. Metatarsus primus elevatus
e. Advantages:
i. This procedure can be added if the distal metaphyseal osteotomy proves to be
inadequate to repair the deformity
ii. Postoperative recovery is far easier than a closing base wedge or Lapidus
iii. No cast is required and partial weight bearing can begin several days
postoperatively
iv. Allows surgery in the presence of a short 1st metatarsal as this procedure
lengthens the 1st ray

2. Cheilectomy: Removal of osteophytes from the 1st MTPJ

Hallux Rigidus and Hallux Limitus


1. Etiology:
a. Dorsiflexed 1st metatarsal secondary to abnormal pronation and
hypermobility of the 1st ray.
b. Dorsiflexed 1st metatarsal secondary to muscle imbalance affecting the 1st
ray.
c. Dorsiflexed 1st metatarsal secondary to sagittal plane structural
malalignment of the 1st metatarsal.
d. Abnormally long 1st metatarsal.
e. Prolonged 1st MTPJ immobilization.
f. Arthritic conditions of the 1st MTPJ either traumatic or metabolic
g. latrogenic secondary to previous foot surgery affecting the 1st ray.
2. Preoperative Symptoms:
a. Spasms or tendonitis of the EHL.
b. Inability to move the hallux normally and or pain on motion.
c. Inability to wear high heeled shoes.
d. Painful hyperkeratotic lesion under the IPJ of the hallux.
e. Painful hallux nail plate.
f. Enlargement over the dorsal aspect of the 1st MTPJ
3. Preoperative signs:
a. Dorsal bunion with or without skin irritation.
b. Limited or absent 1st MTPJ motion
c. Hallux extention distal to the IPJ
d. Deformation of the hallux nail plate
e. Weakness of the peroneus longus and/or hyperactivity of the anterior
tibial ms.
f. Crepitation and/or pain with 1st MTPJ ROM.
4. Preoperative radiographic signs:
a. Dorsal osteophytic proliferation
b. Dorsiflexed 1st metatarsal relative to talar bisection
c. Narrowing of the 1st MPJ joint space
d. Flattening of the 1st metatarsal head
e. Arthritic changes of the 1st MPJ
f. Subchondral sclerosis

5. Biomechanical considerations: One must determine if there is a functional


component that is producing the deformity, as a functional limitus is a common
finding in the patient with significant uncontrolled pronation producing
hypermobility of the 1st ray. To determine this examine the patient in a relaxed
vs. neutral calcaneal stance position. A functional limitus only occurs in the
relaxed position.

6. Conservative treatment:
a. Orthoses
b. Metatarsal bar
c. Rocker-bottom shoes

7. Surgical treatment- Joint preservation techniques:


a. Soft tissue release: Should be followed by immediate ROM excercises
i. Release of fibrosis on the dorsal aspect of the joint
ii. Release of the dorsal capsule
iii. Release of the medial capsule if previously overcorrected
iv. Release of the plantar adhesions between the sesamoid apparatus and the
plantar aspect of the metatarsal head

b. Cheilectomy: The osseous proliferation around the joint is excised


i. Advantages:
 Easily performed
 Reduces the dorsal enlargement
 Allows for increase in ROM in many cases
 Creates minimal postoperative disability
 Allows for immediate postop propulsive phase ambulation
ii. Disadvantages:
 Potential for capsulodesis
 Does not correct the underlying etiology
c. Kessel-Bonney osteotomy:
i. Advantages:
 Allows the hallux to be in a dorsal position at the propulsive phase of gait
 Allows for immediate postoperative ambulation
ii. Disadvantages:
 Requires an osteotomy with no fixation
 Does not actually increase the overall ROM
 Often creates a lack of hallux purchase
 Does not correct the underlying etiology
 Requires the elimination of the propulsive phase of gait for 3-6 weeks postop

d. Mitchell-type osteotomy: This produces shortening and plantarflexion of the


metatarsal head
i. Advantages:
 Allows for correction of an abnormally long metatarsal
 Allows for mild plantarflexion of the 1st metatarsal
 Increases overall 1st MTPJ motion by relaxing tension around the joint
produced by a long metatarsal
 Allows for mild correction of an abnormal IM angle
 Does not interfere with an open epiphysis
ii. Disadvantages:
 Requires an osteotomy with fixation
 Allows only minimal plantarflexion of the head
 Requires non-weightbearing for 4-6 weeks postop

e. Watermann osteotomy:
i. Advantages:
 Allows the hallux to become more dorsal without using any additional joint
motion
ii. Disadvantages:
 Does not correct the underlying etiology of the deformity
 Can create a lack of toe purchase, and requires the elimination of the
propulsive phase of gait for 3-6 weeks postop

f. Austin-Youngswick osteotomy: This procedure produces a shortening and


plantarflexion of the head
i. Advantages:
 Allows for correction of an abnormally long 1st metatarsal
 Allows for mild plantarflexion correction
 Relaxes tension around the joint via the shortening
 Allows for immediate postoperative ambulation
 Can be utilized to correct abnormal IM angle
 Does not interfere with an open epiphysis
ii. Disadvantages:
 Requires an osteotomy
 Allows only minimal plantarflexion
 Requires elimination of the propulsive phase of gait for 3-6 weeks postop

g. Plantarflexory wedge osteotomy (Van Ness): A wedge of bone is removed from


the plantar aspect of the 1st metatarsal base i. Advantages:
 Allows for true correction of a structurally dorsiflexed 1st metatarsal
 Increases overall 1st MPJ ROM
ii. Disadvantages:
 Requires non-weightbearing with immobilization for 6 weeks postop
 Does not allow for easy repositioning of the 1st ray intraoperatively if too
much bone is resected

h. Juvara type C osteotomy:


i. Advantages:
 Allows for true correction of a structurally dorsiflexed 1st metatarsal
 Increases overall 1st MTPJ ROM
 Does not require removal of a bony wedge
 Can be modified for reduction of an high IM angle
 Allows for easy intraoperative repositioning of the 1st metatarsal on the
sagittal plane
ii. Disadvantages:
 Requires non-weightbearing with immobilization for 6 weeks postop
 Requires the use of two cortical screws for fixation
8. Surgical treatment- Joint destructive procedures:
a. Keller arthroplasty
i. Advantages:
 Elimination of joint pain
 Allows for increase ROM
 Easily performed
 Allows for immediate propulsive-type gait
ii. Disadvantages:
 Destroys the joint
 Creates instability of the 1st ray
 Does not correct the underlying etiology
 Creates lesser metatarsalgia

b. Arthroplasty with joint prosthesis:

c. Arthroplasty with joint prosthesis and proximal plantarflexory osteotomy

d. Arthrodesis:
i. Advantages:
 Eliminates joint pain
 Creates significant internal stability of the 1st ray
ii. Disadvantages:
 Eliminates all motion of the 1st MTPJ
 Requires fixation for 6-8 weeks with immobilization
 Restricts the type of shoes which can be worn postoperatively depending
upon hallux position in the sagittal plane

Hallux Varus
1. Etiology:
a. Congenital and often accompanied by other congenital abnormalities
b. Most commonly latrogenic following surgical treatment of HAV
i. Excessive resection of the medial eminence
ii. Excision of the fibular sesamoid and release of the adductor tendon
iii. Overcorrection of the IM angle
iv. Overtightening of the medial capsule
v. Overcorrection of the PASA

2. Preoperative Symptoms of latrogenic Hallux Varus:


a. Inability to wear conventional shoe gear comfortably
b. Pain along the medial aspect of the hallux
c. Pain along the medial side of the arch (ms. contracture)
d. Pain may be present at the MPJ with or without shoes

NOTE* Capsulitis of the 2nd MPJ may be the only initial complaint

3. Preoperative Signs of latrogenic Hallux Varus:


a. Presence of an adducted positioned hallux
b. Contracture of the hallux IPJ (sometimes)
c. Contracture of the EHL (sometimes)
d. Contracture of the abductor hallucis (sometimes)
e. Pain and crepitation may be present at the 1st MTPJ
f. Hallux limitus may be present at the 1st MTPJ

4. Preoperative Radiographic Signs:


a. The hallux will be in an adducted position at the 1st MTPJ
b. The IM angle will usually be reduced
c. The head of the 1st metatarsal may be staked
d. The fibular sesamoid may be absent
e. The evidence of a previous osteotomy on the 1st metatarsal may be seen
f. A negative PASA may be present
g. Arthritic changes may be present at the 1st MTPJ
h. Arthritic changes may be present at the hallux IPJ

5. Operative Considerations:
a. The deformity should be corrected as soon as possible to prevent DJD of the
joint
b. There is no one surgical procedure. The causative factor(s) must be
determined and corrected along with any secondary changes that had
developed, by selecting the proper procedures (i.e. reverse Austin, reverse Akin,
reverse hemi-angulated implant , soft tissue balancing, arthrodesis, and
Keller/total implant arthroplasty)

Hallux Hammertoe (etiology)


1. Cavus foot (extensor substitution)
2. Removal of the sesamoids
3. Detachment of the flexor tendons
4. Overzealous HAV surgery
5. Plantar hallux IPJ sesamoid

Differential Diagnosis of Pain in the Sesamoid


Area
1. Joplin's neuroma
2. Sesamoiditis
3. Osteochondritis of the sesamoids
4. Ruptured bipartite sesamoid
5. DJD with an eroded crista
6. Hypertrophic sesamoid
7. Fractured sesamoid
8. Tumor of the sesamoid (a giant cell tumor has been reported)

NOTE* Most bipartite sesamoids are tibial, 75% are unilateral, and ossification
of the sesamoids occurs at age 8-10 years

Complications of HAV Surgery (General)


1. Staking the metatarsal head and producing hallux varus
2. Hallux hammertoe
3. Longitudinal fracture when removing the exostosis
4. Sesamoiditis from invasion of the met-sesamoid articulation by tumor
5. Fracture of the articular cartilage of the MTPJ
6. Unstable osteotomy
7. Non or delayed union
8. Damage to the neurovascular structures
9. Elevatus and iatrogenic hallux rigidus
10. Over or undercorrection
11. Osteoporosis from disuse
12. Infection
13. Problems with the fixation devices
14. Hallux limitus/rigidus
15. Avascular necrosis
Chart of Procedures and Their Indications
Abbreviations:

HI= Hallux Interphalangeus


IM= Intermetatarsal Angle
DASA= Distal Articular Set Angle
MP= Metatarsal Protrusion
PASA= Proximal Articular Set Angle
SES= Sesamoid Position
HA= Hallux Abductus Angle
D/PF= Motion of the 1st ray (dorsi/plantarflexion)
JNT= Condition of the 1st m.p. joint
TYPE= Type of Procedure
N= Normal I= Increased D= Decrease G= Good P= Positional
S=Structural
A= Arthritic
Chapter 25: Trauma
Medical Management and Assessment of the
Polytrauma Patient
Assessment of Lower Extremity Injury
General Evaluation and Treatment of Fractures
Calcaneal Fractures
Talar Fractures
Osteochondral Fractures
Navicular Fractures
Cuboid Fractures
Cuneiform Fractures
Fifth Metatarsal Base Fractures
Metatarsal Fractures (1st, 2nd, 3rd & 4th)
Tarsometatarsal Joint Dislocations/Fractures
Subtalar Joint Dislocations
Ankle Fractures
Ankle Inversion Sprain
Deltoid Ligament Ruptures
Compartment Syndrome
Open Fracture Classification System and Treatment
Soft Tissue Injuries
Crush, Gunshot, and Lawnmower Injuries
Puncture Wounds
Epiphyseal Plate Injuries (also see chapter 19, Pediatrics)
Digital Fractures and Dislocations
1st MPTJ Trauma
Nail Bed Trauma
Toe Tip Injuries With Tissue Loss
Dog and Cat Bites
TRAUMA
Medical Management and General Assessment of
the Trauma Patient
1. First Priorities:
a. Evaluate and establish an appropriate airway and ventilate with 100%
oxygen (intubate if ventilation is inadequate, but stabilize cervical spine with
Philadelphia collar)
b. Control external hemorrhage
c. Inspect patient for skin color, alertness, chest wall motion, and extremity
motion
d. Auscultate the chest for breath sounds and establish adequate ventilation (if
suspect pneumothorax with respiratory distress, insert chest tube without
waiting for x-ray confirmation)
e. Obtain vital signs

NOTE* Carotid pulse is palpable at systolic BP of 60 mm Hg, femoral pulse


at 70 mm Hg, and radial pulse at 50 mm Hg

f. If pulselessness/hypotensive from blunt trauma to chest or a penetrating


wound of precordium with distended neck veins not relieved by thoracostomy
tube, open chest for effective CPR
g. Establish IV lines and begin infusion (if patient is hypotensive use femoral
lines via cutdown in conjunction with upper extremity infusion). Use
crystalloid, colloid and/or blood as indicated (lactated Ringer's is preferred
because it prevents metabolic acidosis)

NOTE* In general, blood transfusions should be instituted when crystalloid


infusion exceeds 50 ml/kg

NOTE* If systolic BP is less than 100 mm Hg, place IV in both antecubital


spaces, and if inaccessible do greater saphenous cutdown

h. If patient still hypotensive insert arterial line to monitor BP and blood gases
i. Initiate EKG monitoring
j. Assess neurologic status by Glasgow coma scale
i. Check pupillary response, extremity posturing, and response to commands
ii. Evaluate motor function of all extremities and trunk
iii. If sensory and/or motor deficit, establish spinal cord level of functional
changes
k. Obtain admission blood work (CBC and coagulation profile, arterial blood
gases, urinalysis, and venous blood chemistries)

2. Second priorities:
a. Obtain a H&P if possible
b. Secondary examination of the head, chest, abdomen, pelvis, and extremities
with attention paid to life threatening injuries
c. Obtain appropriate x-rays (cervical, chest, etc.)
d. Place a Foley catheter and check for occult blood in the urine
e. Place a NG tube and check for occult blood in the GI tract
f. Splint extremity fractures
g. Complete neurologic examination
h. Tetanus prophylaxis
i. Culture and sensitivity of open wounds
j. No antibiosis unless specific indication is apparent
k. Perform emergency surgery if required

3. Third priorities:
a. Systematic evaluation of the body
b. Specialty radiology (CT, angiograms)
c. Specialty consultations
d. Invasive monitoring (Swan-Ganz)
e. Urgent Surgery

Assessment of Lower Extremity Injury


1. Examination:
a. Rapid neurovascular assessment

NOTE* Lower extremity injuries are of low initial priority unless there is frank
bleeding. With an arterial injury, there is a pulsatile flow or spurt of bright red
blood. if present exert manual pressure initially, surgical repair later

b. Quantity and quality of pulses


c. Observe motor function
d. Inspect for lacerations, swelling, deformities e. Joints palpated

NOTE* Signs of ischemia are pain, pallor, paresthesias, paralysis, pulselessness.


You have 6 hours to reverse before permanent pathological changes occur

2. Traumatic limb or digital amputation salvage:


a. The avulsed part should be placed in sterile saline soaked gauze, sealed in a
plastic bag, and immersed in ice water
b. The avulsed part that has been properly cooled may last up to 24 hours c.
The avulsed part that is not cooled within about the first 8 hours has a poor
chance of being replanted

General Evaluation and Treatment of Fractures


1. Determine type of fracture:
a. Classification of fractures:
i. Transverse
ii. Oblique
iii. Spiral
iv. Comminuted
b. Stable or unstable:
i. How much bone to bone contact is there?
ii. Are the fragments well aligned?
iii. Is the area subject to movement?
iv. How extensive is the soft tissue involvement?
v. How is the blood supply to the fractured segment?
vi. What kind of bone is involved in the fracture?
c. Open or closed
d. Intraarticular or extraarticular
d. Cortical or cancellous bone involvement

NOTE* Characteristics of cortical and cancellous bone


Cortical Cancellous
Osteogenic properties poor good
Fractures surfaces small large
Soft tissue support poor (few) good
Vascularization poor good
Inherent stability poor good

e. Description of a fracture based upon the 4 basic relationships (described by


the mnemonic LARD):
i. Length and location
ii. Angulation
iii. Rotation
iv. Displacement

2. Treatment of fractures: After determining the location and evaluating


the patient's physiological status, a treatment regimen is tailored to the
patient's needs:
a. Rest, Ice, Immobilization, Compression, Elevation
b. Closed reduction (with or without internal fixation). The mechanisms of
closed reduction are:
i. Step 1- Increase the deformity
ii. Step 2- Distract the fragments
iii. Step 3- Reverse the mechanism of injury
c. Open reduction (with or without internal fixation)
d. Excision of fracture fragments
e. Amputation

Calcaneal Fractures
A disabling injury of the foot. There are a number of associated injuries when
dealing with a calcaneal fracture including: compression fracture of the lumbar
vertebrae, laceration of the kidney-renal damage, fractures of the lower
extremity, and compartment syndrome
1. Anatomical considerations:
a. The largest tarsal bone that has a thin cortical shell enclosing cancellous
bone that contains traction trabeculae radiating from the inferior cortex and
pressure lamellae converging to support the posterior and anterior facets.
b. The calcaneus articulates with the talus through 3 facets, the largest being
the posterior
c. The middle and anterior facet have a common joint cavity with the TN joint
and are separated from the larger posterior facet by the sulcus calcaneus.
d. The lateral end of the tarsal canal gives attachment to the bifurcate ligament,
the EDB, and the inferior extensor retinaculum.
e. Boehler's tuber joint angle overlies the posterior articular facet and is a
measurement of the sagittal plane between the anterior and posterior aspect of
the calcaneus (normal 20-400)
f. Gissane's critical angle is the position that abuts with the lateral process of
the talus and which under compression force acts as a wedge creating the
primary fracture line in the calcaneus

2. Radiological Examination
a. Plain film x-rays

NOTE* Evaluating integrity of the bone plus Boehler's angle and the Critical
angle of Gissane is essential in the diagnosis

i. A-P view
ii. Lateral view
iii. Anthensen's view (demonstrates medial and posterior facets of the STJ)
iv. Isherwood views:
 Oblique lateral (anterior process and calcaneocuboid)
 Medial oblique axial (medial and posterior facet)
 Lateral oblique axial (posterior facet) v. Broden's projections
 Broden 1 (shows the posterior facet from anterior)
 Broden 2 (shows the sinus tarsi to posterior)
b. Tomography
c. CT scanning (The Gold Standard)

3. Classification: Due to the fact that two types of fractures exist (intra-
articular and extra-articular) the classification that is best used is a
combination of the Rowe (inclusive for extra-articular) and Essex-Lopresti
(inclusive for intra-articular, replacing Rowe 4 8 5)
a. Rowe:
i. Type 1a: Fracture of the tuberosity
ii. Type 1b: Fracture of the sustentaculum tali
iii. Type 1c: Fracture of the anterior process (most common; female
patients predominate, mostly related to wearing high heeled shoes)
iv. Type 2a: Beak fracture
v. Type 2b: Avulsion fracture involving the tendo Achilles insertion
vi. Type 3: Oblique body fracture not involving the STJ
vii. Type 4: Body fracture involving the STJ
viii. Type 5: Joint depression fracture with comminution

b. Essex-Lopresti:
i. Fractures not involving the STJ: Tuberosity fractures
 Beak type
 Avulsion medial border -Vertical
 Horizontal
ii. Fractures involving the calcaneal-cuboid joint:
 Parrot-nose type
 Various
iii. Fractures involving the STJ:
 Without displacement
 Tongue-type with displacement
 Centro-lateral depression of the joint
 Sustentaculum tali fracture alone
 Comminuted

NOTE* The primary fracture line is vertical from the vertex of the critical angle of
Gissane to the plantar aspect. The secondary fracture line is determined by the
direction of the force

c. Sanders: A new classification that utilizes CT scanning rather than plain


radiographs for its identification. This is the first system to have a prognostic
value
i. Type 1: nondisplaced
ii. Type 2: two part posterior facet fracture
(a) Fracture through the lateral column
(b) Fracture through the central column
(c) Fracture through the medial column
iii. Type 3: three part posterior facet fracture with central depression
(ab) Fracture through lateral and central columns
(the bone between FX a and b is depressed)
(ac) Fracture through lateral and medial columns
(the bone between FX a and c is depressed
(bc) Fracture through the central and medial columns
(the bone between FX b and c is depressed)
iv. Type 4: Four part posterior facet fracture

4. Mechanism of injuries:
a. Torque injuries (extra-articular fractures)
i. Anterior process fx
ii. Avulsion fx
iii. Sustentacular fx

b. Direct impaction (extra-articular fractures)


i. Tuberosity fx
ii. Beak fx
c. Falls from a height (intra-articular fractures)
d. Concussive force from below (intra-articular fractures)

5. Clinical Diagnosis of Calcaneal Fractures:


a. Anterior process fractures:
i. Swelling- well defined 3-4 cm. anterior to the lateral malleolus
ii. Inversion and adduction increase pain

NOTE* Must R/O 5th metatarsal base fractures

b. Beak and avulsion fractures:


i. "Pop" sound heard/felt on the heel with sudden pain
ii. Pes planus antalgic gait
iii. Weakness of plantarflexion
iv. Edema, ecchymosis, bullous lesions (Mondor's sign)
c. Fractures of the medial and lateral process:
i. Heel thickens, edema, and ecchymosis
ii. ROM of ankle, STJ, and MTJ within normal limits
d. Fracture of the body no STJ involvement
i. Inability to bear weight
ii. Edema/ecchymosis
iii. Generalized pain around the heel
iv. Pain with ROM of ankle and STJ
e. Fracture of sustentaculum tali
i. Pain and edema on the medial aspect of the foot 1 inch below the medial
malleolus
ii. Pain inferior to medial malleolus on dorsiflexion (FHL)
iii. STJ ROM decreased and painful
iv. Ankle joint ROM WNL
f. Tongue depression fractures
i. Rapid edema and severe pain and inability to bear weight
ii. Severe bleeding under fascia
iii. Discoloration of the heel extending to the calf
iv. Blister formation
v. Flattened arch
vi. Decreased plantarflexion (Hoffa's sign)
vii. Widened heel
viii. R/O compression fracture of lumbar vertebrae and ankle (10% occurrence)

6. Treatment:
a. Anterior process fractures:
i. Small fragment fractures
 Soft cast NWB for 2-4 weeks (early mobilization important)
ii. Large displacement fragment
 RIF or excision (it is recommended to wait 1 year before excision)

b. Beak or avulsion fractures:


i. Beak fractures
 BK cast 4-6 weeks (weight bearing if fragment not displaced)
 If fragment displaced closed reduction followed by BK NWB cast for 6 weeks
in plantarflexion with gradual weightbearing
 RIF (screw) if closed reduction unsuccessful followed by BK NWB cast
ii. Avulsion fractures
 ORIF followed by screw fixation followed by BK NWB cast in plantarflexion
for 4 weeks followed by a neutral position BK cast for 2 weeks
c. Fractures of the medial and lateral process:
i. For nondisplaced treat with compression dressing/ice/elevation NWB. After
edema subsides, follow with well molded BK cast for 4 weeks
iii. For displaced fragment closed reduction followed by BK NWB cast for 6
weeks

d. Fracture of the body not involving the STJ:


i. For non displaced use compression dressing/ice/elevation with ROM
exercises immediately (NWB 4-6 weeks)
ii. If displaced, closed reduction with BK NWB cast 6-8 weeks (Steinmann pin
can be used with proximal displacement)

e. Fracture of the sustentaculum tali:


i. Compression dressing/ice/elevation
ii. ROM excercises immediately for FHL
iii. BK cast with progressive weightbearing
iv. Firm shoe with orthoses
v. If displaced closed reduction followed by BK cast 4 weeks NWB followed by
weight-bearing 2 weeks

f. Tongue and joint depression fractures:


i. Closed reduction
ii. Open reduction (extensive)

NOTE* Treatment for significant calcaneal fractures has traditionally been


conservative (either closed reduction or posterior percutaneous pin fixation).
However, calcaneal fractures treated by these methods resulted in a marked
disability that gradually resolved to a tolerable level but with many sequelae
(widening of the heel, significant malalignment in varus or valgus, and lateral
impingement syndromes). If the talus was left impacted into the posterior facet
region, anterior ankle arthritis developed. Because of this, new techniques
have been advocated.
Open Reduction Technique:
The lateral approach is used most for the primary incision. The medial
approach is used when needed for more accurate reduction and rigid
stabilization, or when CT scan demonstrates that most of the pathology lies
medial in the fracture. A wide lateral flap is made incorporating the peroneal
tendons and sural nerve, down to the subperiosteal layer.
Timing is important to give a satisfactory result. Some advocate waiting 4-7
days to allow the swelling to resolve, but immediate fixation can also be done
if the fracture is open or is associated with a compartment syndrome Several
technical options are available with regard to hardware: a. Either 3.5 mm.
cortical or 4.0 mm. cancellous screws can be used to fix the reduced posterior
facet to the sustentacular fragment b. Neutralize the entire calcaneus with 3.5
mm. reconstruction plates, flattened 1/3 tubular plates (possible in
combination), or cervical plates (plates are recommended for neutralizing the
interfragmental screw repair and for holding the lateral wall
c. A transverse K-wire is inserted through the calcaneus to attach a Kirschner
traction device for manipulation (traction reduces the medial wall) d. A bone
graft would not be necessary if adequate lateral to medial wall reduction is
performed. However, some feel that a bone graft is necessary under the
posterior facet after it has been elevated from the body of the calcaneus

b. A plate stabilizing the lateral wall (the plate is the key in preventing the
posterior heel from drifting into varus)
c. A transverse K-wire or Schanz screw is inserted into the posterior body and
helps reduce the Medial wall indirectly to pull the heel out of varus, and lock in
the medial cortices before the lateral-to-medial lag screws are inserted

7. Further considerations of treatment of intra-articular fractures:


a. Displaced intra-articular calcaneal fractures require open reduction in order
to restore joint congruency
b. The Essex-Lopresti maneuver can be used in elderly patients with intra-
articular tongue-type fractures who cannot tolerate surgery
NOTE* The Essex-Lopresti maneuver is a technique of reduction. The patient is
taken to the OR and using fluroscopic control, a Steinmann pin is driven into
the posterior tuberosity from the posterior aspect of the heel. The pin is then
used as a lever to elevate the impacted and depressed joint surface and
tongue portion. This is accomplished by pushing the protruding portion of the
pin downward toward the plantar aspect of the heel. If the articular surface
and posterior tuberosity are noted to fall into proper alignment the pin is
advanced into the anterior calcaneus to fixate the fracture. A second pin may
be driven parallel to the first for added fixation. A BK cast is applied

c. Summary of correction: If there is visual congruency of the subtalar joint,


visual alignment of the fracture through Gissane's angle, and radiographic
absence of an intra-articular step off at the posterior facet with reduction of the
medial wall on the axial view, then reduction must be considered anatomic

NOTE* The 4 areas to consider when evaluating the intra-articular fracture of


the calcaneus are posterior facet disruption, medial wall (determines height),
lateral wall blow out (determines width) and calcanel cuboid joint
a. Posterior facet: Indication for ORIF is with more than a 3 mm step off
involving the posterior facet or if there is an angulation of the tuberosity
fragment greater than 10 0
b. Medial wall pathology: 1 cm or more displacement of the medial wall is
indicative of increased shortening and increased width, and should be treated
with ORIF
c. Lateral wall pathology: Lateral wall disruption can cause sural nerve
irritation and peroneal tendon dysfunction, therefore should be reduced
d. Calcaneal cuboid joint: Involvement can be treated with closed reduction

8. Surgical incisions:
a. Medial approach: The main advantage of this approach is in the direct
visualization of the reduction of the posteriolateral fragment and the
superiomedial fragment and sparing the peroneals and sural nerve
i. McRenolds horizontal incision
ii. Stephenson vertical incision
iii. Zwipp medial "L" incision
b. Lateral approach: Gives expansile exposure to the lateral wall of the
calcaneus, the calcaneocuboid joint and most importantly the posterior facet
(site of major pathology)
i. Modified -Kocher incision
ii. Oilier incision
iii. Right angle incision (consists of a vertical and horizontal arm; an excellent
expansile incision which exposes the entire lateral rearfoot complex via a
subperiosteal flap, and can be extended proximally if a concomitant ankle
fracture is present)
NOTE* The most advantageous approach includes the use of a primary lateral
incision with an ancillary medial incision if there is difficulty reducing the
posteriolateral and superiomedial fragments

9. Complications of Calcaneal Fractures:


a. Heel pain
b. Peroneal tendonitis
c. Osteoarthritis of the STJ, MTJ, and ankle joint
d. Heel pad damage
e. Bony prominence
f. Flexor tenosynovitis
g. Sural or posterior nerve entrapment
h. Calcaneus gait (weak plantarflexion)
i. Rigid pes planus
j. Reflex sympathetic dystrophy
k. Infection

NOTE* In cases of severe comminution, the question of primary subtalar fusion


or triple arthrodesis is still debated

Talar Fractures
1. Anatomy:
a. 2nd largest tarsal bone with more than 1 /2 the surface being cartilage b. No
muscular or tendinous attachments
c. The 3 main parts are the head, body and neck. The neck deviates medially
15-200 and is its most vulnerable part
d. The FHL lies within a groove on the posterior talar tubercle held by a
retinacular ligament
e. Inferiorly 3 facets are present; between the posterior and middle is a
transverse groove which (with the calcaneus) forms the tarsal canal that exits
laterally into the sinus tarsi

2. Blood supply:
a. Extraosseous blood supply comes from:
i. Posterior tibial (#1)
ii. Anterior tibial (#2)
iii. Perforating peroneal (#3)
b. The talar neck is supplied by an anastamosis of 2 vessels:
i. Artery of the tarsal sinus
ii. Artery of the tarsal canal

3. Classification:
a. Chips and avulsions
b. Compression fractures
c. Fractures of the body:
i. Non displaced
ii. Displaced
iii. Comminuted
d. Fractures of the neck (Hawkins' classification): All caused by some fall or
accident resulting in a severe dorsiflexory force to the foot
i. Group 1: Vertical fracture of the neck that is undisplaced
 One of the three sources of the blood supply is disrupted (a 13% chance
of avascular necrosis has been reported)
ii. Group 2: Vertical fracture of the neck that is displaced, the STJ is
subluxed or dislocated, and the ankle joint is WNL
 Two main sources of blood supply are interrupted (a 42% chance of
avascular necrosis has been reported)
 Prognosis is related to the development of avascular necrosis
iii. Group 3: A vertical fracture of the neck that must be displaced and the body
of the talus must be dislocated from both the ankle and subtalar joints
 All three sources of blood supply are disrupted (91 % chance of avascular
necrosis)
iv. Group 4: The fracture of the talar neck is associated with dislocation of the
body from the ankle and the subtalar joints with an additional dislocation or
subluxation of the head of the talus from the T-N joint
 Avascular necrosis reported in 100% of cases

NOTE* Early anatomical reduction in displaced fractures yields the most


favorable long term results

NOTE* Hawkins' sign is an area of translucency of subarticular or subchondral


bone seen on x-ray, following injury, which indicates healing is occurring

e. Fractures of the talar dome (Berndt and Harty): see section


Osteochondral fractures
f. Dislocations:
i. Anterior dislocations
ii. Posterior dislocations
iii. Lateral dislocations
iv. Medial dislocations
v. Total dislocation

4. Treatment:
a. Talar neck:
i. Group 1:
 BK/NWB cast for 6-12 weeks, followed by NWB with no cast for an
additional 2-5 months with ROM ankle excercises (prognosis is excellent)
 ii. Group 2:
 Closed reduction with BK/NWB cast until evidence of union
 Early ORIF when and if closed reduction fails or the original reduction is
unstable (prognosis related to the development of avascular necrosis)
iii. Group 3:
 ORIF with accurate anatomical reduction must be achieved followed by
BK/NWB cast for 3-4 months (prognosis is poor)
 iv. Group 4:
 As per Group 3

NOTE* If the talus must be removed, a Blair procedure is recommended

NOTE* Arthrodesis procedures have been stated to give better results as a


secondary procedure than a talectomy alone

NOTE* Hawkins grades 3 and 4 fractures were thought to be unsalvagable but


with modem ORIF techniques there are improved chances of restoring normal
function after injury.

a. The operative incision to the talus must not inflict any additional harm to the
arteries bringing blood to the body and the neck the most critical blood supply
coming from the posterior tibial in the deltoid: ligament attachment. An additional
blood supply courses into the undersurface through the talocalcaneal ligament
b. The talus must be reduced as quickly as possible to protect any remaining
blood supply by untwisting and reducing tension in the deltoid ligament, and to
encourage revascularization
c. An atraumatic operative approach is needed that allows adequate
visualization for anatomic reduction (Oilier lateral incision, transverse Cincinnati
incision or a posterolateral vertical incision work well)
b. Lateral process:
i. Undisplaced:
 BK cast partial weight bearing 4 weeks
i. Displaced:
 Excision of bone fragment if symptomatic

c. Total talar dislocations (out of the ankle mortise and STJ, anterior to the
fibula, head directed medially, talus rotated on the longitudinal axis so its
inferior aspect points posteriorly):
i. Manipulation:
 Usually not successful but should be attempted
ii. Skeletal traction:
 Steinmann pin through calcaneus attached to traction apparatus to achieve
an open space between the tibia and calcaneus. The assistant inverts and
plantarflexes the foot, as the surgeon presses both thumbs on the posterior
aspect of the talus by inward and backward movement to rotate the talus.
Afterward, the pin is removed and the foot is immobilized in an anterior and
posterior splint for 7 days with the knee bent to 300 and ankle at 90°. This
is followed by a BK cast for 6-8 weeks. Avascular necrosis is inevitable.
 If there is an open wound treat appropriately.

Osteochondral Fractures
1. Classification (Berndt and Harty):
a. Stage 1: A small area of compression of subchondral bone
b. Stage 2: A partially detached osteochondral fragment
c. Stage 3: A completely detached osteochondral fragment remaining in the
defect.
d. Stage 4: A displaced osteochondral fragment

NOTE* It has been found that 44% are lateral and anterior, and 56% are
medial and posterior. Lateral lesions are shallow wafer shaped and medial
lesions are deep cup shaped

2. Mechanism of injury:
a. Lateral lesions: Inversion and dorsiflexion
b. Medial lesions: Inversion, plantarflexion and lateral rotation of the tibia on
the talus

3. Diagnosis:
a. Stage 1:
i. Usually no symptoms, and has been diagnosed as an ankle sprain
ii. ROM of the ankle is WNL and painless
b. Stage 2:
i. Painful with associated collateral ligament damage
 Lateral dome lesions have pain over the lateral collateral ligaments
 Medial dome lesions have pain over the deltoid
ii. Ankle ROM may be limited due to traumatic synovitis
c. Stage 3 8 4:
i. Pain is more severe
ii. Decreased ROM of the ankle, joint locking or crepitus, and/or instability of
the collateral ligaments

Note* The diagnosis can be made on x-ray (the A-P view shows the medial talar
dome clearly, the lateral dome is obscured but can be visualized in the medial
oblique), but the use of tomograms or CT are best

4. Treatment:
a. Conservative: For stage 1, 2, 3 medial lesions via NWB BK cast for 6 weeks
followed by a patellar-bearing brace until the fracture heals
b. Surgical: For stage 3 lateral and 4, surgery to remove fragment, or stabilize
fragment using K -wire or Herbert screw

NOTE* Review of the literature reveals that surgically treated patients have
better results in preventing post-traumatic arthritis. However, conservative vs.
surgical treatment depends upon the size/location/stage of the fracture
fragment

Staging System for Osteochondral Lesions


Radiographs T2W-MRI Arthroscopy
Stage 1 Normal Marrow edema Normal or
(diffuse high signal irregularity and
intensity softening of
cartilage
Stage 2 Semicircular Low signal line Articular cartilage
fragments surrounds fragment breached, definable
but nondisplaced
fragment
Stage 2A Subcortical lucency High-signal fluid
within fragment
Stage 3 Semicircular High signal line Displacable
fragment surrounds fragment fragment
Stage 4 Loose body Defect talar dome Loose body
Navicular Fractures

Fractures of the navicular are easily missed,, and are important to diagnosis
quickly as a delay in treatment could lead to traumatic arthrosis of Lisfranc's
joint as well as the T-N joint. Isolated fractures are uncommon, and usually
occur in conjunction with Lisfranc's dislocations and fractures of the
rearfoot. Stress fractures of the navicular have been seen in runners but
more frequently in basketball players, and this problem is often
misdiagnosed as anterior tibial tendonitis

1. Anatomy:
a. Cancellous bone which is convex distally where it articulates with the three
cuneiforms and is concave proximally to accomodate the talar head
b. The dorsal navicular surface is roughened and serves as an attachment for
the dorsal talonavicular ligament, cuneonavicular ligaments and the
cubonavicular ligament
c. The plantar surface is so roughened and is invested by the plantar
calcaneonavicular ligament (spring ligament)
d. The lateral surface serves the attachment for the navicular portion of the
bifurcate ligament
e. The navicular tuberosity provides the major attachment site for the posterior
tibial tendon
f. The blood supply is from the dorsalis pedis and the medial plantar artery
which form an arcade of 6-8 randomly arranged vessels that penetrate the
navicular surface (the central 1 /3 is relatively avascular)

2. Navicular Fracture Classification by Watson-Jones


a. Type I- Fracture of the tuberosity
i. Relatively common as compared to other types
ii. The mechanism of this fracture is acute eversion of the foot causing an
avulsion-type fracture, caused by increased tension placed on the tibialis
posterior tendon
iii. These fractures are generally non-displaced because of the multiple soft
tissue attachments to the tuberosity
iv. Best demonstrated radiographically on the AP and oblique x-ray with the
foot in moderate equinus
v. It is important to differentiate this fracture from a type II accessory navicular
vi. If the type I fracture is severely displaced you should suspect calcaneocuboid
involvement
vii. The combination of a severely displaced fracture and compression fracture
of the cuboid and/or calcaneus is referred to as the NUTCRACKER SYNDROME
vii. Treatment is with an Unna-type boot of BK partial weight-bearing cast x 4
weeks.
viii. If a symptomatic non-union occurs it is recommended that the fragment be
removed and reattachment of the tibialis posterior performed

b. Type II- Fracture of the dorsal lip

i. Most frequent fracture of the navicular and is intraarticular


ii. The mechanism of injury is plantarflexion of the foot followed by
either forced inversion or eversion
iii. Best seen on the lateral x-ray

iv. Can be confused with 2 accessory ossicles in the same area, the os
supratalare and os supranaviculare
v. Treatment is with a BK partially weight-bearing cast for 4-6 weeks
vi. If late problems such as a painful dorsal prominence occurs, excision of the
fragment is recommended

c. Type IIIA - Fracture of the body without displacement


d. Type IIIB - Fracture of the body with displacement

i. A severe injury that causes disruption of the talonavicular and cuneonaviclar


joints
ii. Can be either displaced (type A) or nondisplaced (type B)
iii. The mechanism of injury can result from either direct crush or blow,
or indirect from a fall from a height with the foot in a marked
plantarflexion position at the moment of impact
iv. These fractures are usually intraarticular
v. DP, lateral, and oblique x-rays will demonstrate the fracture
vi. A differential diagnosis for a type Ill navicular fracture include a bipartite
tarsal navicular and lithiasis of the navicular
vii. Treatment of nondisplaced fractures is with a BK walking cast for 6-8 weeks
viii. Treatment of displaced fractures is with ORIF and a BK non-weightbearing
cast for 6-8 weeks

e. Type IV- Stress fracture of the navicular


i. Usually an athletic injury, most commonly track and field
ii. Symptoms are increased pain with activity, and decreasing pain
following the activity
iii. Usually intraarticular
iv. Usually found with either a bone scan, CT scan, or MRI
v. Early diagnosis is important to prevent a complete fracture and an eventual
nonunion
v. Treatment is a BK non-weight-bearing cast for 4-6 weeks if nondisplaced,
and if displaced ORIF with a BK non-weight-bearing cast for 6-8 weeks

NOTE* Watson-Jones navicular fracture classification described 3 types:


Type 1 (tuberosity fx), Type 2 (dorsal lip fx), and Type 3 (transverse body fx)
Cuboid Fractures
The cuboid is a key bone in the rigid lateral column of the foot. Its position is
stabilized by several structures to ensure its structural and functional integrity
1. Anatomy:
a. The cuboid is locked in articulation with 5 bones of the foot (4th and 5th
metatarsals, calcaneus, lateral cuneiform, and a fibrous articulation with the
navicular)
b. Primarily cancellous
c. Dorsally, the bifurcate ligament attaches the calcaneus to the cuboid, the
dorsal cuneocuboid ligaments tether the cuboid to the lesser tarsus
d. Dorsolaterally, the dorsal tarsometatarsal ligaments attach the cuboid to the
metatarsal bases
e. Plantarly, the long and short plantar ligaments attach to and cross the
cuboid while adding to the maintenance of the longitudinal arch
f. The sural nerve and lesser saphenous vein cross over the cuboid area
g. The peroneus longus courses plantarly under the peroneal groove in the
cuboid
h. The arterial supply is made up of an arterial rete system supplied by the
lateral malleolar artery, the lateral tarsal artery and the arcuate artery

2. Classification system:
a. Type 1: Stress fracture
b. Type 2: Avulsion fractures
(a) Bifurcate ligament area
(b) Tarsometatarsal ligament area
c. Type 3: Body fracture, nondisplaced
d. Type 4: Indirect crush fracture or nutcracker fracture
e. Type 5: Plantar dislocation
f. Type 6: Direct crush

3. Avulsion Fractures:
a. Most common on the lateral aspect at the calcaneocuboid joint and the 5th
met-cuboid articulation.
b. Avulsion fracture of the tuberosity due to tension on inferior calcaneocuboid
ligament.
c. Adduction of the cuboid on the calcaneus will result in avulsion due to
tension on lateral calcaneocuboid ligament.

NOTE* Avulsion fractures of the cuboid must be differentiated from os cuboid


secondarium, os peroneum and/or os vesalianum

d. Treatment is closed reduction with casting

4. Fractures of the body of the cuboid:


a. Due to axial rotary forces while the foot contacts the ground in a
plantarflexed position- usually associated with fracture of the base of the 5th
metatarsal and calcaneus
b. Crush fracture as above mechanism but with more force (a nutcracker effect)
c. Treatment is closed reduction with BK cast 6-8 weeks or arthrodesis in case
of crush fracture

5. Stress fractures:
a. Should be suspected if concerned about peroneus longus tendonitis,
calcaneocuboid arthritis, dropped cuboid, and capsulo-ligamentous strain in
the cavus foot type.
b. Treatment: BK cast 6-8 weeks (first 2 weeks NWB)
NOTE* In general, treatment for type 1, 2, and 3 injuries is usually NWB BK
cast for 6-8 weeks. Avulsion fractures are sometimes opened if the dislodged
fragment is felt to be intraarticular or will cause impingement on the peroneal
tendons. Type 5 dislocations must be reduced, with closed reduction under
general anesthesia attempted first with an inversion-adduction force on the
forefoot while pushing the cuboid up from the arch. If this fails, open reduction
is advised. Type 4 fractures usually require autogenous bone grafting for
anatomic alignment of the calcaneocuboid and tarsometatarsal joints

Cuneiform Fractures
1. Avulsion fractures:
a. Usually located on the medial aspect of the internal cuneiform as an avulsion
due to pull of the tibialis anterior

2. Fractures of the body:


a. Mechanism: Either by direct trauma or rotational force

3. Stress fractures:
a. Diagnosed by bone scans, CT, or tomography b. Treated with BK WB cast

4. Treatment: Requires traction to reduce the dislocation and allow anatomical


reduction of the cuneiforms to prevent chronic pain and arthritis

NOTE*
a. Cuneiform fractures are usually associated with Lisfranc dislocations
b. The mechanism of the dislocation and fractures of the cuneiforms involves
the forefoot and rearfoot acting as levers, with the lesser metatarsals displaced
laterally and dorsally
c. Fracture of the 2nd metatarsal base is an important factor in causing
dislocation or fracture of the middle cuneiform
d. Lisfranc's ligament interruption has an effect on a middle cuneiform
fracture/dislocation

Fifth Metatarsal Base Fractures


Fractures of the 5th metatarsal are commonly associated with inversion sprains
of the ankle, therefore, with any ankle inversion injury, the 5th metatarsal base
should always be evaluated. This is the Stewart Classification
1. Type 1: A true Jone's fracture which occurs between the epiphysis and
diaphysis (metaphyseal level).
a. Usually oblique or transverse in nature
b. Situated at the distal end of the articular capsule above the intermetatarsal
ligaments
c. The mechanism of injury is internal rotation of the forefoot while the base of
the 5th metatarsal remains fixed, The capsule is only stretched and the
peroneus brevis takes practically no part on the injury
d. Upon physical examination, extreme mobility of the shaft of the 5th
metatarsal is found
e. This is an unstable fracture with a very poor blood supply and because of
this, this fracture has a very high propensity for non-union
f. Treatment is with a non-weight-bearing cast BK cast for 4-6 weeks, or If
displaced, then ORIF

2. Type 2: Intraarticular fracture of the 5th metatarsal base with one or two
fracture lines
a. A result of shearing force caused by the internal twisting of the forefoot while
the peroneus brevis is contracted
b. Displacement of the fragments depends upon the extent of the damage to the
capsule and ligaments
c. Treatment is with an Unna-type boot or BK non-weightbearing cast for 4-6
weeks, or if nonreducible, then ORIF

3. Type 3: This is an avulsion fracture of the base of the 5th metatarsal


a. This is oftentimes mistaken in the literature for a Jones fracture. It is the
most proximal injury, where a small fragment is torn away, the fracture is
extraarticular, and the fracture line is usually at right ankles to the long axis of
the metatarsal base
a. The mechanism of injury is primarily a sudden sharp contraction of the
peroneus brevis when the ankle is in plantarflexion
b. The treatment is with an Unna-type boot of BK non-weightbearing cast for 4-
6 weeks, or if nonreducible then ORIF with tension bend wiring or screw
fixation. If the fragment is too small for fixation, then excision of the fragment
and reattachment of the peroneus brevis tendon is recommended

4. Type 4: This is a comminuted intraarticular fracture of the 5th metatarsal


base
a. The mechanism of injury is similar to Type 2, but in this case the 5th
metatarsal base gets crushed between the cuboid and the ground causing
fragmentation
b. There is a high rate of non-union
c. Treatment is an Unna-type boot or a BK non-weight-bearing cast for 46
weeks, or if fracture fragments are severely displaced, bone grafting and ORIF
may be required

5. Type 5: A fracture that occurs in children, where there is a partial avulsion


of the epiphysis with or without a fracture line or hairline crack as seen in Type
2. This fracture can also be classified as a Salter-Harris I.
a. The treatment is a BK non-weight-bearing cast for 4-6 weeks

Metatarsal Fractures (1st and 2nd-3rd-4th)


1. Classification:
a. Site:
i. Epiphyseal
ii. Diaphyseal
iii. Metaphyseal
b. Type:
i. Incomplete separation of the bony fragments
ii. Complete separation
iii. Greenstick fracture
iv. Buckle type fracture
c. Configuration:
i. Transverse
ii. Spiral
iii. Oblique
iv. Comminuted
v. Stress fracture

d. Relationship of the fragments:


i. nondisplaced
ii. Displaced:
 Shifted sideways
 Rotated
 Distracted
 Overriding
 Angulated
 Impacted
e. Relationship to outside environment:
i. Simple
ii. Compound

NOTE* If, wound is open treat accordingly :(check blood loss, shock etc)
tetanus prophylaxis, antibiotic therapy, skin coverage as necessary, rigid
reduction of fracture, and fluid replacement as necessary

NOTE* Salter devised a classification system describing fractures of long bones


1. Location of the fracture: diaphysis, metaphysis, physis, epiphysis,
intraarticular
2. Extent of the fracture: complete or incomplete
3. Arrangement of the fracture: transverse, spiral, oblique, compression,
comminuted

2. Treatment:
a. Closed reduction with BK NWB cast 4-6 weeks
b. Open reduction:
i. Monofilament wire
ii. K-wires
iii. AO technique

NOTE* Complications include pseudoarthrosis, avascular necrosis, and


malposition

NOTE* Radiographic diagnosis of this fracture should not be confused with the
normal apophysis present in children (closed b age 15 in boys and 12 in girls).
and Iselin's disease (osteochondrosis) Also differentiation should be made
between an avulsion fracture and an os vesalianum and os perineum
NOTE* Radiographic evidence of chronicity is manifested by a wide radiolucent
fracture line, periosteal reaction, thickening of the lateral margin of the cortex
adjacent to the fracture with or without callus, and intramedullary sclerosis

3. First metatarsal fractures:


a. Anatomy:
i. Articulates laterally with the 2nd metatarsal, proximally with the medial
cuneiform, and distally with the base of the proximal phalanx of the hallux
ii. 2.7 cm proximal to the head of the 1 st metatarsal (on the lateral aspect) is
the foramen for the nutrient artery
iii. Plantar surface is concave, causing this side to be under tension during
weight bearing
iv. Holds the sesamoids, the tibial being larger separated by the crista or central
ridge (if more than 2 sesamoids, bipartite indicating multiple ossification
centers)
v. Muscles around the 1st metatarsal:
 Peroneus longus: involved in 1st metatarsal base avulsion fractures
 Tibialis anterior
 EHL
 EHB
 Adductor hallucis
 Abductor hallucis
 FDB
 Tibialis posterior
vi. Arterial supply is the dorsalis pedis and 1 st plantar metatarsal

b. Classification:
a. Salter classification is based on 6 categories
i. Site: anatomical location
ii. Configuration: transverse, oblique, spiral, etc.
iii. Open or closed
iv. Location
v. Extent: complete or incomplete
vi. Relationship of the fracture fragments to each other: displaced, angulated,
rotated, etc.

c. Treatment:
i. Simple fractures with no displacement are treated NWB BK cast for 6-8 weeks
ii. Displaced fractures should be anatomically reduced (usually open reduction
ORIF)
iii. Open fractures: treated as per open fracture classification
4. External fixator device for metatarsal fractures: A miniature external
fixation device can be utilized in the treatment of metatarsals. Maintains the
normal metatarsal parabola pattern
a. Indications: When a metatarsal fracture is severely comminuted or when a
significant loss of bone stock is present (gunshot)
b. This supplies rigid fixation giving stability to the fracture and can be
combined with other forms of fixation
c. It is capable of both compression and lengthening the metatarsal fragments
and can be combined with bone grafting as needed
d. Should be reserved for patients for whom reduction by any other means
cannot be obtained

5. Internal metatarsal fractures (metatarsals 2-3-4): Treated like the other


metatarsal fractures
A: Is a diaphyseal fracture with straight plate fixation without lag screw
B: is a diaphyseal/metaphyseal neck fracture with application of L plate with
lag screw fixation
C: Is a metaphyseal base fracture with application of T-plate with lag screw
fixation
Ankle Fractures
1. Classifications:
a. Lauge-Hansen: A two-word description indicating the position of the.
foot at the time of injury, and the direction of the talus. Five types of
injuries listed:
i. Supination-Adduction
ii. Pronation-Abduction
iii. Supination-External Rotation
iv. Pronation-External Rotation
v. Pronation-Dorsiflexion
NOTE* The major advantage of the classification is to enable the examiner to
assess the stability of the ankle from the x-rays by predicting ligamentous
injuries
In external rotation injuries, the progression of lesions simply follows the
anatomic sequence around the ankle joint: deltoid-medial malleolus
complex, anterior syndesmosis, fibula, and posterior syndesmosis.

In supination injuries, the sequence starts with the anterior syndesmosis, and
in pronation injuries, with the deltoid-medial malleolus complex.

b. Danis-Weber: Based on the location of the fracture of the fibula and is useful
for determining the appropriate form of treatment for ankle fractures.
i. Type A: Below the joint level
ii. Type B: At the level of the joint
iii. Type C: Above the level of the joint

2. Lauge-Hansen:
a. Supination-Adduction:
NOTE* The hallmark of this injury is an avulsion fibular fracture at the level
of the ankle or below.
i. Stage 1: Transverse fracture of the lateral malleolus usually below or at the
level of the ankle mortise or lateral collateral ligamentous rupture (pulloff)
ii. Stage 11: Stage I plus an oblique fracture of the medial malleolus (pushoff)

NOTE* Variants of Stage If S -A injuries are: rupture of the deltoid ligament


rather than fractures of the lateral malleolus, concomitant damage to the
tibiofibular syndesmosis with fracture of the medial and lateral malleolus,
avulsion fracture of the lateral malleolus proximal to the A.T.F. with damage to
the lateral collateral ligament, S-A fracture of the medial malleolus without
injury to the lateral side,

NOTE* Healing is more favorable with supination injuries (less overall damage)
b. Pronation Abduction:
1. Stage I: Fracture of the medial malleolus or tear of the deltoid ligament
ii. Stage II: Stage I plus rupture of the anterioinferior tibiofibular and
posterioinferior tibiofibular ligaments and transverse tibiofibular ligament,
with fracture of the posterior lip of the tibia
iii. Stage III: Stage II plus an oblique supramalleolar fracture of the fibula (the antero-
posterior tibiofibular ligaments tear but the interosseous ligament does not)

c.

Supination-External Rotation:

NOTE* This is the most common fracture of the ankle, and its hallmark is a spiral
fracture of the fibula
i. Stage I: Rupture of the anteroinferior tibiofibular ligament, sometimes
with avulsion of the bony fragment between the tibia and fibula. (tibia:
Chaput, fibula: Wagstaff)
ii. Stage II: Stage I plus a spiral oblique fracture of the lateral malleolus.
iii. Stage III: Stage 11 plus a fracture of the posterior lip of the tibia
(Volkmann's fracture)
iv. Stage IV: Stage III plus a fracture of the medial malleolus

d. Pronation-External Rotation:

NOTE* 'The hallmark is a high fibular fracture.

i. Stage I: Fracture of the medial malleolus or a tear of the deltoid ligament.


ii. Stage II: Stage I plus a tear of the anteroinferior tibiofibular ligament and
interosseous ligament.
iii. Stage III: Stage II plus an interosseous membrane tear and a spiral fracture
of the fibula 7-8 cm. proximal to the tip of the lateral malleolus
iv. Stage IV: Stage III plus a fracture of the posterior lip of the tibia.
e. Pronation-Dorsiflexion:
i. Stage I: Fracture of the medial malleolus
ii. Stage II: Fracture of the anterior inferior aspect of the tibia
iii. Stage Ill: Supramalleolar fracture of the fibula (transverse)
iv. Stage IV: Fracture of the posterior aspect of the tibia (Pilon fracture) Reudi
and Allgower divided these into:
 Grade I: Cleavage fracture of the distal tibia with no disruption of the
internal surface
 Grade II: Internal surface disruption with no comminution
 Grade III: Impaction and comminution

NOTE* In a fall from a height, where there is pronation-dorsiflexion injury,


axial compression is present, which will result in the following fractures of the
tibial plafond:
a. if the talus is dorsiflexed upon impact, anterior portion of tibial plafond is
fractured
b. if the talus is plantarflexed upon impact, posterior portion of the tibial
plafond fractures
c. if the talus is in neutral position upon impact, central shattering of the
articular surface of the tibial plafond takes place
3. Danis-Weber fractures:
a. Type A (Supination-adduction Lauge-Hansen): The fibular fracture occurs
below the level of the tibial plafond and therefore below the level of the
syndesmotic ligaments. It is associated with a vertical fracture of the medial
malleolus.
b. Type B (Supination-external rotation or Pronation-abduction LaugeHansen):
An avulsion fracture of the medial malleolus and fracture of the fibula that
begins at the level of the tibial plafond. The posterior rim of the tibia might also
be fractured
c. Type C (Pronation-external rotation Lauge-Hansen): Characterized by rupture
of the syndesmosis and a fibular fracture that is located above the tibial
plafond. Associated injuries are an avulsion fracture of the medial malleolus or
deltoid ligament rupture and a large or small posterior malleolar fracture.

4. Other fractures:
a. Tillaux fracture: Fracture of the anterior tubercle of the tibia due to tension
of the IATF ligament. Also a type 3 epiphyseal injury of the anterolateral distal
tibia.
b. Wagstaff-Lefort fracture: Vertical fracture of the anterior margin of the
lateral malleolus due to an avulsion of either the anteroinferior tibiofibular or
anterior talofibular ligaments.
c. Maisonneuve fracture: Fracture of the proximal fibula , associated with
tibiofibular diastasis.
d. Pankovich classification of Wagstaff fractures:
i. Type l: Avulsion fracture and fibular fragments remaining attached to the
anterior talofibular ligament and IATF ligament
ii. Type Il: Oblique fracture with fragment remaining attached to the IATF
ligament
iii. Type III: Oblique fracture of the fibula in addition to a fracture of the
anterior tibial tubercle
e. Bosworth fracture: Fibular oblique fracture caused by external rotation but
the fracture occurs after posterior dislocation of the fibula. This causes closed
reduction to be impossible.
f. Frost. fracture: A triplane fracture which is a combination of Tillaux and
Salter-Harris Type 2 occurring at the distal tibia
g. Pott's Fracture: A fracture of the distal fibula and disruption of the deltoid
ligament (or medial malleolar fracture)
h. Cooperman's fracture: This is a Salter-Harris triplane type 4 epiphyseal
ankle fracture which consists of 2 fragments: the first is composed of the tibial
shaft, medial malleolus, and the anteromedial portion of the epiphysis; the
second consists of the remainder of the metaphysis, epiphysis, and attached
fibula.
i. Chaput's tubercle: The anterolateral tubercle of the distal tibia
j. Shepherd's fracture: Fracture of the posterolateral tubercle of the talus.
k. Volkmann's fracture: A fracture of the posterolateral corner of the distal
tibia (Volkmann's triangle), medial malleolus, fibular shaft, and tibiofibular
diastasis.
l. Ashurst's sign: The overlap of the anterior tibial tubercle and the medial 2/3
of the distal fibula normally is found on the A-P x-ray of the ankle. Ashurst's
sign is present with a lessening of this overlap due to widening of the ankle
mortise due to disruption of the anterior tibiofibular ligament
m. Thurston-Holland sign: The spike of metaphyseal bone attached to the
fractured epiphysis seen with Salter-Harris 2 fractures.

4. Treatment of ankle fractures:


a. General considerations: A decision to perform surgery takes in account all
aspects of the patient's condition. In general, the best long term results in
terms of restoration of function and avoidance of posttraumatic arthritis are
directly related to treatment that restores anatomy and allows for early
range of motion and early weight-bearing. Early ORIF should be done provided
that the initial evaluation of the patient reveals a satisfactory .neurovascular
status and skin condition of the foot. Early ORIF reduces swelling by stabilizing
the fracture and also reduces bleeding. If the fracture is open, the wound
should be cultured and broad spectrum IV antibiotics started, followed by
wound debridement, irrigation, and ORIF as indicated. The wound is left open
and delayed primary closure is performed at least 5 days later. Closed
reduction of displaced ankle fractures rarely accomplishes restoration of normal
anatomy without repeated forced manipulations, and does not allow for early
ambulation and range of motion.

NOTE* ORIF is indicated for all ankle fractures with a greater than 2 mm.
lateral or posterior displacement of the lateral or medial malleolus

b. Absolute criteria:
i. Fractures and dislocations must be reduced immediately
ii. All joint surfaces of the ankle must be anatomically reduced iii. Reduction
must be maintained while the fractures are healing iv. Motion of the joints
should be started as soon as possible

c. Other criteria:
i. ORIF of the fibula should precede fixation of the medial malleolus because it
provides a buttress to the talus, which tends to displace laterally and pull along
the medial malleolus. Shortening of the fibula must be prevented (see chapter
29: Ankle Conditions, Nonunion of Malleoli)
ii. Repair of the deltoid is difficult and rarely necessary, and should be reserved
for more severe injuries in which soft tissues around the ankle are damaged.
iii. Large displaced fragments of the anterior and posterior processes of the
tibia, which are present in some indirect ankle fractures, should be
anatomically reduced (if at least 1 /4th the weight-bearing surface) in order to
restore congruity of the articular surface (reduction of these fragments prevents
subluxation of the talus)
iv. Fracture of Chaput tubercle and Wagstaff fractures should always be
reduced and fixed
v. A displaced yet essentially intact fibula requires syndesmotic screws for
proper reduction

NOTE* Stability of the syndesmosis is tested by pulling the fibula laterally with
a bone hook. When there is more than 2-3 mm of lateral displacement of the
fibula, instability is present and the use of a syndesmotic screw(s) is indicated.
It is desirable to insert this screw 2-3 cm above the tibial plafond. When there
is a plate attached to the fibula, one cortical screw can be removed and
replaced with a syndesmotic screw. When drilling for a syndesmotic screw, the
direction must be anteromedial to avoid inserting the screw posterior to the
tibia
NOTE* Observe for tendon dislocations, i.e. posterior tibial tendon into the
ankle joint

d. Fixation of the fibula: Should be fixed prior to the medial malleolus


i. AO Technique:
Interfragmentary screws are inserted most often from the anterior edge in a
posteroinferior direction
 Overdrilling is unnecessary and may cause comminution of the fragment
 3.5 mm cortical screws are most often used
ii. Cerclage Wiring:
 It is useful in comminuted fractures while a plate is being applied to the
lateral side of the fibula
 Can be used as an adjunctive device for an oblique fracture of the fibula
while an intramedullary nail is being used
iii. Inyo Nails:
 Excellent for transverse fractures of the distal fibula
 Useful in osteoporotic bone but requiring cerclage wiring of an oblique
fracture of the fibula prior to insertion
 When inserting the nail, it is critical to reduce the fracture anatomically in
order to avoid penetration outside the bone

e. Fixation of the medial malleolus: Requires fixation with a device that provides
compression between the fracture fragments I. AO technique:
 Essentially only 4.0 mm cancellous screws should be used for fixation of the
medial malleolus (self-tapping 4.5 mm malleolar screws are not practical
because the head of the screw is too large and prominent after insertion)
 A screw usually 40-45 mm in length is usually used
 In osteoporotic bone a washer would be used to prevent penetration of the
head of the screw head
 A second screw is used when the malleolar fragment is large (a K -wire can
be used first to prevent rotation of the fragment and retained for additional
stability)

f. Closed reduction:
i. The main advantage of closed reduction is lack of postoperative wound
complications
ii. Closed reduction is contraindicated in unstable ankle fractures in which
both malleoli are fractured
iii. Closed reduction is acceptable in fractures when open reduction is
contraindicated (vascular compromise, neglected open fractures, pyoderma,
skin necrosis or contusion)
iv. Gravity is utilized by positioning the leg horizontally and in external rotation
while holding the foot in one hand with the heel resting in the palm. This
effectively produces internal rotation and adduction of the talus and in that way
reduces the fibula and brings in position the medial malleolus
 A short leg cast is first applied while the fracture is being reduced, then is
extended to a long leg cast with the knee in 30° of flexion. A minimum of 6
weeks of immobilization is required
5. Factors that result in irreducible fractures:
a. Interpositon of the deltoid ligament
b. Trapping of the tibialis posterior tendon
c. Trapping of the medial tendon(s)
d. Dislocation and fracture-dislocation of the fibula behind the tibia
6. Soft tissue complications of fractures and dislocations of the ankle:
a. Skin: Blistering, decubitus breakdown, slow wound healing
b. Massive Edema: Treat with compression immediately, cold application,
elevation, rigid internal fixation and early ROM
c. Fracture blisters: Direct result of edema. ORIF must be delayed for 3-7
days
d. DVT's: Due to plaster immobilization, venous insufficiency, sickle-cell.
Treat casted patients with sub-Q heparin 2500-5000 units Q 8-12 hours if
they are at risk
e. Chondrolysis of the ankle (cartilage necrosis): Leads to posttraumatic
arthritis
f. Avascular necrosis of the talus
g. Infection following open fractures (5-30%)
h. Nerve injuries
i. Nerve disruptions (complete and incomplete)
ii. Reflex sympathetic dystrophy syndrome
i. Arterial injuries
j. Tendon injuries
k. Ligament injuries
i. Medial deltoid
ii. Lateral ligament (chronic thickening, local tenderness, inversion
instability, anterior subluxation)

7. Bony complications of fractures and dislocations of the ankle:


a. Fractures of the lateral malleolus:
i. Non-union and malunion (external rotation) with chronic swelling and
widening of the ankle
b. Fractures of the medial malleolus:
i. Non-union: More common than the lateral malleolus due to soft tissue
interposition between the fragments
 Chronic diastasis
 Loose bodies
 Arthritic changes
c. Fractures of the posterior tibial margin: Greater than 25% margin renders
the ankle unstable leading to posterior subluxation
i. Posterior subluxation
 Medial and lateral malleoli and syndesmosis torn
ii. Malunion with posterior subluxation
 Most common complication
d. Fractures involving the distal tibiofibular syndesmosis:
i. 2nd only to plafond injuries, leads to arthrodesis of the ankle
ii. Mortise widening
e. Vertical or Pilon fractures of the distal tibia:
i. Varus or valgus deformity
ii. Traumatic arthritis
iii. Articular incongruity
f. Epiphyseal injuries:
i. Articular incongruity is the main concern
ii. Vascular embarassment
iii. Posttraumatic arthritis
iv. Varus or valgus deformity of the ankle
v. Angular deformity
vi. Leg length discrepancy vii. Bone and joint sepsis

Midtarsal Joint Dislocations


The talonavicular and the calcaneocuboid joints function as a single unit in
movements (functions with STJ in inversion and eversion) so are considered
together as the MTJ. Injuries are rare
1. Classification (Main and Jowett): Midtarsal joint injuries are defined
according to the direction of the force producing the dislocation. a. Medial force
results in three grades of injuries:
i. Fracture sprain of calcaneus, talus, navicular, or cuboid
ii. Fracture/subluxation or dislocation with medial subluxation or dislocation of
forefoot while the talocalcaneal relationship remains normal
iii. Swivel dislocation with only the T-N joint dislocating
b. Longitudinal force injuries:
i. Fracture of the navicular takes place
c. Lateral force injuries:
i. Fracture sprain of the navicular tuberosity, dorsal chip of the talus or
navicular, and lateral fracture of the cuboid
ii. Fracture/subluxation will result in T-N lateral subluxation and nutcracker
fracture of the cuboid
iii. Swivel dislocation with the talus dislocating laterally relative to the navicular
d. Plantar force injuries:
i. Result in dorsally dislocated talus and calcaneus relative to the navicular,
and cuboid chip fractures also present dorsally, as well as anteroinferior
calcaneal fractures
e. Crush injuries: Have variable patterns. Usually associated with open wounds

2. Treatment:
a. Medial force injuries: reduced by traction and reversal of the mechanism of
injury with casting (WB or NWB)
b. Lateral force injuries: closed reduction first. C-C fusion is recommended for
persistent symptoms. Triple arthrodesis has been the traditionally
recommended treatment if conservative care has failed

Tarsometataral Joint Dislocations/Fractures (Lisfranc


dislocation)

This type of injury occurs in conjunction with high energy trauma


(equestrian injuries) as well as minor twisting injuries, which are also
associated with injuries to the cuneiforms, cuboid, and the navicular. The
key to understanding this injury is the structural integrity provided by the
slotting in the base of the second metatarsal, which is surrounded by 5
adjacent bones that create a tight mortise. There is no ligament between the
base of the 2nd and 1st metatarsals, but a ligament extends from the medial
base of the 2nd metatarsal obliquely to insert into the medial cuneiform.
Soft tissue loss and vascular impairment can be a major problem in this
setting
The 2nd metatarsal is the key to stability of Lisfranc's joint.
The dorsal tarsometatarsal ligaments are weaker than the plantar
tarsometatarsal ligaments.

1. Mechanism of injury:
a. Abduction and plantarflexion: 2nd metatarsal is fractured, the remaining
ligaments give way and the forefoot is subluxed laterally
b. With continued abduction, there may be a nutcracker-like fracture of the
cuboid.
2. Classifications:
a. Hardcastle et. al.
i. Type A: Total incongruity (the metatarsals displace in a unit in one plane
ii. Type B; Partial incongruity ( at least one of the tarso-metatarsal joints is not
displaced)
iii. Type C: Divergent (the 1 st metatarsal is displaced medially and the other
metatarsals are displaced laterally)
b. Quenu and Kuss
c. Myerson: Further subdivided Hardcastle's classification
i. Type A: Total incongruity
ii. Type 131: Partial incongruity, medial metatarsals
iii. Type B2: Partial incongruity, lateral metatarsals
iv. Type C: Divergent patterns
v. Type Cl: Partial displacement
vi. Type C2: Total displacement

3. Diagnosis: Via x-ray (A-P & lat)/tomography (A-P)/CT scan


a. Widening between the base of the 1 st and second metatarsals or the middle
and medial cuneiforms is often present. This widening can also be between the
base of the second and third metatarsals or middle and lateral cuneiforms.
b. An avulsion fragment referred to as the "FLECK SIGN" is often present
between the base of the 1st and 2nd metatarsals or middle and medial
cuneiforms.
c. Projected lines from the base of the metatarsals should not intersect the
corresponding cuneiforms or cuboid.
d. Angulation of the metatarsals can occur without apparent fracture at the
base.
e. Widening between the base of the 5th metatarsal and the cuboid can occur.

NOTE* Due to the spontaneous relocation that this fracture dislocation can
produce, x-rays do not usually show the true magnitude of the severity of this
injury

4. Treatment: Includes splinting, casting, closed reduction and casting, closed


reduction and percutaneous pinning, or open reduction and percutaneous
pinning (according to type)
a. If instability is present but alignment is anatomic with release of the force,
immobilizing the extremity in a BK NWB cast is appropriate.
b. If instability is present and alignment is not anatomic with release of force,
then pinning (open or closed) is mandatory followed by casting.
c. If severely unstable, then open anatomic reduction with heavy gauge K -
wires or screw fixation is the procedure of choice.
d. With severe joint comminution, primary arthrodesis may be considered
NOTE* Pin placement according to Myerson classification type: a. Type A: 2 K-
wires (medial and lateral)
b. Type B 1: 2 medial K-wires
c. Type B2: 1 wire laterally
d. Type C: 3 or more wires
The medial pin should go across the 1st metatarsal-cuneiform joint, the 2nd
pin should go across the 3rd and 4th tarsometatarsal joints, and a 3rd pin
across the 2nd ray articulation. The pins should remain in place for a
minimum of 6 weeks

NOTE* Always pin the 2nd metatarsal base


NOTE* Most feel that stability and anatomic reduction depends upon the 2nd
metatarsal, so that if the 2nd metatarsal relocates, all the other metatarsals
will follow if there is no damage to the intermetatarsal ligaments

5. Complications:
a. Amputation
b. Sepsis
c. Thrombophlebitis
d. Compartment syndrome
e. Neuroma formation (either traumatic or postsurgical “amputation” type)
f. Post-traumatic arthritis

Ankle Inversion Sprain


Also see Chapter 29, Ankle Conditions: Chronic Lateral Ankle Instability

Definition: A sprain is a disruption of fibers, a strain is plastic deformation


with elongation

Inversion sprains by definition involve lateral ligament disruption. The anterior


talofibular ligament resists internal rotation, plantarflexion, and anterior
subluxation of the talus. The posterior talofibular ligament functions to
reinforce the ankle joint and the calcaneofibular ligament functions to resist
adduction forces.
1. Anatomy:
a. Lateral ligaments:
i. Anterior talofibular (ATFL) (intracapsular): The primary stabilizing structure
preventing anterior displacement of the talus (fan shaped).
ii. Calcaneofibular (CFL) (extracapsular)
iii. Posterior talofibular (PTFL) (intracapsular/extrasynovial): The thickest and
strongest and the least likely to be injured
iv. Lateral talocalcaneal ligament

NOTE* PTFL is 20-45° posteroinferior to the fibular bisection so allows STJ


range of motion.

NOTE* Inversion of the ankle is resisted primarily by the ATFL when the ankle
is plantarflexed and by the CFL when the ankle is dorsiflexed.
NOTE* Angular relationships between the ATFL and the CFL is 100° in the
frontal plane and 105° in the sagittal plane. This sagittal plane angle
decreases with STJ supination and increases with STJ pronation. The angular
relationship between these two lateral ligaments is very d cult to attain during
reconstructive ankle stabilization repair tending to cause a decrease in
allowable STJ supination at the expense of attaining stability against inversion
stress.

2. Ligament composition:
a. 67% water
b. Remaining 33%: 90% collagen type 1, elastin, and glycosaminoglycans
3. Causative Factors:
a. Tibial varum
b. Calcaneal varum
c. Plantarflexed -1st ray
d. Rigid forefoot valgus
e. STJ varum
f. Uncompensated equinus
g: Muscle imbalance (peroneal insufficiency)
h. Previous sprains (elongated ligaments no longer restrain inversion)
i. Torsional abnormalities
j. Short leg syndrome
k. Ankle varus

4. Mechanism of injury:
a. Internal rotation, plantarflexion, and adduction of the talus beyond normal
physiologic limits

5. Classifications of inversion ligamentous injuries (mechanism of


injuries):
a. Leach (1983)
i. 1 st degree (ATFL rupture)
ii. 2nd degree (ATFL, CFL, and capsule rupture)
iii. 3rd degree (all three ligaments and capsule)
b. Diaz (1st word describes the position of the foot and ankle and the second
word the direction of the force applied: similar to Lauge Hansen classification)
i. Supination-inversion (with a plantarflexed ankle or a neutral ankle)
ii. Supination internal rotation
iii. Supination plantarflexion

NOTE* Additionally, ankle sprains can be classified into:


Type I: stretching of the ligaments or tearing of the ATFL
Type II: a partial rupture or tearing of the ATFL and CFL
Type III: a total rupture of the ATFL, CFL, PTFL, and capsule or tearing of the
ligaments

6. Diagnosis:
a. Scout films
b. Stress views (local anesthesia: peroneal block + local ankle infiltration):
Can use a Telos® apparatus for better quality control
i. Inversion stress: A 5-6° difference between the injured and uninjured ankle
signifies ligamentous rupture

NOTE* Always take bilateral inversion stress films when examining a patient
radiographically with potential grade III ruptures

NOTE* Degree of talar tilt is not a true indication of which ligament is ruptured

ii. Push-pull stress (anterior draw sign): The ability to pull the ankle out of the
mortise more than 4 mm. usually indicates a rupture of the anterior talofibular
ligament

NOTE* There are certain situations where the stress test may be invalid:
genetic ligamentous laxity, history of chronic ankle instability, inability to
achieve adequate anesthesia, or inability to properly maneuver the uninjured
ankle. In these cases, ankle arthrography would be indicated

NOTE* The main indication for ankle arthrography in a soft tissue injury is to
evaluate a possible ankle diastasis and to confirm ligament tears

c. Ankle arthrography: In performing this test you must consider the following
i. The patient must have no allergy to iodine
ii. The injection should be administered at the anterior-medial aspect of
the ankle (to prevent confusion from the actual injury)
iii. The test must be performed within the first 5-7 days following the
injury
iv. Dye that is found within the normal anatomical confines of adjacent tendon
sheaths and not within the surrounding soft tissue should be considered a
normal anatomical variant
d. Peroneal tenography: A diagnostic technique for evaluation of the calcaneo-
fibular ligament. If dye is injected into the peroneal tendon sheath and is found
to enter the ankle joint but no dye is seen in the soft tissue surrounding the
ankle, a negative test.

NOTE* For some patients there is a normal communication between the


peroneal tendon sheath and the ankle joint capsule: gives a false positive

7. Differential diagnosis and associated findings:


a. 5th metatarsal base fractures (avulsion and Jones)
b. Stieda's process fracture (talar posterior process)
c. Calcaneal avulsion of the EDB
d. Calcaneal anterior process fracture
e. Talar dome fractures (medial or lateral)
f. Sinus tarsi syndrome
g. Peroneal stenosing synovitis
h. Peroneal tendon dislocations
i. Peroneal neuropathy

NOTE* A Jones fracture is a transverse fracture secondary to a triplane load


with pull of the peroneus brevis
8. Treatment:
a. Symptomatic therapy: Used for patients with negative stress x-rays, patients
with a significant medical history which would contraindicate more definitive
therapy,. geriatrics with a sedentary lifestyle, and patients who present to
treatment 3-4 weeks following injury.
i. Elastic compression
ii. Ice
iii. Analgesics
iv. Weight-bearing to tolerance
v. Physical therapy (proprioceptive excercises and strengthening)

NOTE* TEMPER is an acronym for ankle sprain rehabilitation:


T: Timely diagnosis/temporary immobilization
E: Edema reduction
M: Muscle strengthening
P: Proprioceptive excercises
E: External stabilizing devices
R: Return to activity

b. Definitive therapy: Either immobilization (preferred) or surgery


i. Immobilization:
 48 hours following the injury a BK weightbearing cast is applied for 3-6
weeks
 This is followed by an Aircast® for an additional 3 weeks (for athletes this is
continued for 6-9 weeks)
 Stress x-rays should be repeated in 6 months to evaluate the treatment

ii. Surgical treatment 2-3 days following injury (must be young and athletic who
need complete stability):

 Single ligament rupture:


Watson-Jones*: This uses the peroneus brevis, which passes through the
fibula from posterior to anterior, through the neck of the talus from dorsal to
plantar, back through the fibula, from anterior to posterior, and sutured
back onto itself.
Lee Procedure (modified Watson-Jones)*: This uses the peroneus brevis
tendon, which is then passed through the fibula, from posterior to anterior,
and then sutured back onto itself.
Evans*: This utilizes the peroneus brevis through an oblique hole through
the fibula sutured back onto the belly of the peroneus brevis.
Storren
Nilsonne
Pouzet
Haig
Castaing and Meunier
Dockery and Suppan
 Double ligament rupture:
Elmslie*: Originally described as using the fascia lata and passed through a
drill hole in the lower aspect of the fibula, through the calcaneus, back
through the same drill hole, and tied onto itself, after passing through the
neck of the talus.
Chrisman and Snook*: This uses the split peroneus brevis, which is passed
through the fibula from anterior to posterior through a flap in the calcaneus,
and is then sutured back to the peroneus brevis tendon. Stroren
Hambly
Winfield
Gschwend-Francillon

 Triple ligament rupture:


Spotoff
Rosendahl and Jansen

8. Inversion injuries can result in the following:


a. Sprains of the STJ ligaments
b. Medial STJ subluxation
c. Dislocation of the talus
d. Osteochondral fractures of the talar dome
e. Shear fractures of the head of the talus medially
f. Shear fractures of the navicular laterally
g. Avulsion fractures of the posterior aspect of the talus
h. Avulsion fracture of the base of the 5th metatarsal
i. Fracture of the cuboid
j. Avulsion fracture of the lateral malleolus

9. Complications:
a. Inappropriate diagnosis and lack of treatment
b. Early complications
i. Painful hemarthrosis
ii. Hematoma
iii. Rarely, gangrene of the skin of the lateral ankle in cases of rupture of the
perforating peroneal artery
c. Neuropraxia in grades II and III with damage to the intermediate dorsal
cutaneous nerve

NOTE* With greater than 20% of stretching, fascicular interruption may occur,
causing permanent neurotmesis of the intermediate dorsal cutaneous nerve
and producing a profound lateral foot and ankle sensory loss

d. The most common surgical complication of primary ankle repair, involves the
intermediate dorsal cutaneous nerve
i. Entrapment neuropathy
ii. Laceration
e. Late complications of surgical repair result from overzealous tightening of the
lateral ankle structure (grade III ankle sprains should be fixed in neutral not in
eversion
f. Painful sinus tarsi can occur later from an everted ankle position
LIGAMENTOUS INJURIES

Lateral Medial

Common Rare
Adduction (inversion) Rarely alone
Type I (Anterior Talofibular Ligament) palpable tenderness Abduction or External Rotation
Talus Stable - Anterior drawer sign Edema
- Inversion stress (occhymosis in deltoid area)
- X-Ray (Bilateral)
Deltoid ligament ruptured to variable degree
Treatment 1 strap assesment made as per
2 physical therapy Close (1956)
1. Mortise view
Type II Anterior Talofibular Ligament Palpable tenderness 2. If lateral displacement of
Calcaneal Fibular Ligament diffuse ankle pain talus
If plantar flexion • possible anterior deltoid ligament 2mm + Type I
rupture 3mm + Type II
Talar stability 4mm + Type III
+ Anterior drawer Treatment 1. Type I strap
- Inversion stress 2. Type II
- X-Ray 3. Type III -cast B, K., W. B.
Treatment 1. strap (occ. B. K.. W.B. cast)
2. physical therapy
3. youth - possible ligament

Type III AnteriorTaloflbular ligament


Calcaneofibular ligament
Post Deltoid if plantar flexion
1. Talar stability
+ Anterior drawer
+ Inversion stress
- X-Ray
Syndesmotic
Treatment (usually occurs in young athlete) Rare
1. ligament repair with cast Rarely alone
2. physical therapy Abduction external rotation
Edema
ecchymosis -in syndesmotic area

Anterior + Post Tibiofibular ligament


interosseous ligament
usually with Maisonneuve fracture
Treatment 1. Dap. on other assoc injury
2. Mild -cast B.K., W.B.
3. Severe, A.O. Fixation
Deltoid Ligament Ruptures
1. Anatomy: The deltoid takes origin off the medial malleolus, which ends
structurally in two colliculi (one anterior and one posterior), divided by an
intercollicular groove. There is a superficial and a deep deltoid: a. Superficial
deltoid
i. Naviculotibial
ii. Calcaneotibial (strongest)
iii. Superficial talotibial
b. Deep deltoid
i. Deep anterior talotibial
ii. Deep posterior talotibial

2. Mechanism of Injury: Solitary injury to the deltoid is rare, it is usually


accompanied by other ligament injuries or fractures.

NOTE* Most common are fractures of the fibula and ruptures of the tibiofibular
ligaments

a. Types of injuries:
i. Supination-external rotation
ii. Pronation-external rotation
iii. Pronation-abduction

3. Signs and symptoms:


a. Pain and swelling on the medial and anterior aspects of the ankle
b. Since there are usually associated injuries, the usual presentation is a
completely edematous and ecchymotic ankle that is being splinted

4. Diagnosis:
a. Scout films (with pronation injuries a high fibular x-ray)
b. Stress x-ray (local anesthesia): Can be done by hand or using a Telos®
apparatus
i. Mortise view of the ankle where the foot is abducted and everted in relation to
the leg
ii. Lateral view where the foot is anteriorly displaced in relation to the leg

NOTE* Stress views are done bilaterally and the clear space is what is
compared

Note* A clear space of 1 cm. or greater is diagnostic of a complete rupture, and


a displacement of 3mm. or more means tearing of part of the deltoid

5. Treatment:
a. Usually closed reduction and with BK NWB cast with the foot in inversion is
sufficient for 3-6 weeks, followed by a BK weight-bearing cast for another 3-6
weeks.
b. Surgical repair is indicated if closed reduction does not replace the talus to
its proper position.

NOTE* This can occur if the deltoid gets rolled up or inverted, or if the posterior
tibial tendon gets trapped.

Compartment Syndrome
Usually diagnosed in the arm and leg, also occurs in the foot, and can follow
several types of injuries, most commonly multiple fractures or crushing
injuries. This entity should be considered in the differential diagnosis in
patients presenting with a painful swollen foot post trauma
1. Definition: Increased compartmental pressure resulting in decreased
perfusion and ultimate ischemic changes to the tissues on the compartment.
This can eventually result in contractures and poorly functioning limbs,
a. Physiology: At rest the intramuscular pressure is approximately 5 mm Hg.
During a muscular contracture the pressure can increase up to 150 mm Hg or
more. At relaxation, the compartment pressure rapidly drops, and within 5-10
minutes, has returned to baseline. With a compartment syndrome, there is no
drop of pressure

2. Types of compartment syndrome:


a. Acute: Occurs when the resting pressure in the compartment exceeds the
available perfusion pressure. This is usually the result of trauma with
hemorrhage or gross muscular edema causing the increased compartmental
pressure. If untreated tissue necrosis is inevitable
b. Chronic: Occurs when the resting pressure is higher than the normal resting
pressure but not so high as to cause hyperprofusion. Following excercise, the
time for pressure to return to baseline is protracted. This results in a relative
prolongation of the ischemic time resulting in symptoms during or following
excercise. Actual muscle necrosis is unusual

3. Diagnosis: Measurement of an increased intramuscular pressure in the


compartment via a wicks catheter (usually greater than 30 mm Hg)

Two criteria must be fulfilled for this diagnosis to be made: a space that is
limited by fascia, skin, or bone must be present; second increased
compartment pressure caused by a decrease in compartment size or an
increase in the size of the contents within that compartment must be present
Any injury with a pressure greater than 30 mm Hg should
undergo an immediate fasciotomy

Note* The patient might still present with a pulse because the vascular collapse
occurs first at the arteriolar level

4. Compartments of the foot:


a. Medial compartment: Its borders are the medial and lateral intermuscular
septum, the medial portion of the plantar aponeurosis, the tarsus (proximally)
and shaft of the first metatarsal (distally). It contains the abductor hallucis
flexor hallucis brevis, and the FDL tendon
b. Central compartment: Its borders are the medial and lateral intermuscular
septum, the central portion of. the plantar aponeurosis, the tarsus (proximally)
and interosseous fascia (distally). It contains the flexor digitorum brevis, FDL
tendon with lumbricals, quadratus plantae, adductor hallucis, PT and peroneal
tendons
c. Lateral compartment: Its borders are the lateral intermuscular septum,
lateral portion of the plantar aponeurosis, and the associated osseous
components. It contains the abductor digiti minimi, flexor digiti minimi, and
opponens digiti
d. Interosseous compartment: Its borders are the metatarsals and the interossei
fascia. It contains the interossei

5. Clinical Findings:
a. Pain out of proportion to the clinical findings b. Paresthesias
c. Pulselessness
d. Or none of the above

6. Treatment:
a. Fasciotomy
i. Double dorsal technique:
 Midfoot and forefoot: 2 dorsal longitudinal incisions, one over the 2nd
metatarsal and the other over the 4th (deepened down to the metatarsal
shaft) where a hemostat is passed into each adjacent interosseous space.
The wound is closed secondarily in 5 days.
ii. Extensile medial incision
iii. Combined approach

7. Associated complications:
a. Comminuted fractures
b. Severe soft tissue injuries
c. Post-ischemia swelling
d. Intramuscular hematomas associated with bleeding diasthesis
e. Crush injuries

Open Fracture Classification System and


Treatment
1. Gustillo and Anderson described an open fracture classification system:
This depends upon the mechanism of injury, degree of soft tissue damage,
the configuration of the fracture, and the level of contamination
a. Type 1:
i. Wound less than 1 cm long and clean
ii. Minor soft tissue damage is present
ii. Fracture is simple, transverse, or short oblique with minimal
comminution
b. Type 2:
i. Wound more than 1 cm. long without extensive tissue damage, flaps
or avulsions
ii. There is a slight crushing injury, moderate comminution of the fracture ill.
Moderate contamination
c. Type 3: Extensive soft tissue damage, including muscles, skin, and
neurovascular structures, with a high degree of contamination (high velocity
injuries, farm injuries)
i. Type 3A: Open fractures with adequate soft tissue coverage of bone despite
extensive soft tissue laceration
ii. Type 3B: Open fractures having extensive soft tissue loss with periosteal
stripping and bone exposure. Severe contamination and severe comminution.
Usually a local or free flap is needed for bony coverage
iii. Type 3C: Open fractures are associated with arterial injury requiring
microvascular repair, regardless of the soft tissue coverage

2. General principles of treatment:


a. Tetanus history and therapy administered
b. Thorough H & P conducted (blood loss measured-CBC,HCT, Hb) with
neurological, musculoskeletal, and vascular assessment of the lower extremities
c. Complete x-rays
d. Appropriate antibiosis should be administered in the E.R. (cultures and gram
stains should be taken)

NOTE* The primary bacteria encountered in open fractures is Staph. aureus.


However, the choice of antibiotic is determined by the extent of the soft tissue
injury. Gustillo and Anderson recommend cefazolin for type 1 and 2 open
fractures (2 gm initially followed by 1 gm Q 6 hr for 3 days). For type 3 injuries
a cephalosporin plus an aminoglycoside (1.5 gm/kg body weight then 3 to 5
gm/kg body weight in divided doses) is used. Penicillin is added for farm
injuries to cover Clostridium sp.

e. Immediate debridement and irrigation, with repeat debridement and


irrigation in 24-48 hours

NOTE* The irrigant can contain either 1 gm cefazolin in 1 liter of sterile saline,
or 50,000 units of bacitracin and 1 million units of polymyxin B in 1 liter of
sterile saline.

f. All foreign debris should be excised


g. All marginal, macerated skin, and soft tissue should be debrided
h. Fluorescein (non-toxic dye) may be used to assess the viability of the soft
tissue structures
i. The wound should be kept moist and re-evaluated in 48-72 hours, with
repeated debridements, especially if soft tissue coverage is necessary
NOTE* 10-15 mg/kg of fluorescein is injected IV and observed under UV light
after 10-20 minutes. Vascularized tissue will fluoresce yellow-green and
nonvascularized tissue will appear dark blue

j. For type 1, 2, and 3A open fractures delayed primary closure, using skin
grafts within 5-7 days
k. For type 3B and 3C open fractures, the soft tissue loss is so great that the
use of skin flaps is necessary and a delay in using them becomes apparent
because of the repeated debridements
l. External fixation should be used for all type 3 and unstable type 2 fractures

The advantages of external fixators include the ease of application without


additional trauma, allows for daily wound inspections and care, allows for
grafting procedures, accomplishes compression/reduction of the angulation/
stabilization of the fracture without much surgical trauma

m. Internal fixation (screws, plates, pins, etc.) should be used for articular and
metaphyseal open fractures. This is done preferably within 8 hours of the injury

NOTE* An open fracture untreated in the initial 7-8 hours (golden period) is
generally considered to convert from a contaminated wound to an infected
wound

3. Absolute indications for open reduction:


a. Irreducible fractures where function and alignment can not be obtained
otherwise
b. Displaced intra-articular fractures where incongruity will lead to
degeneration of the joint
c. Displaced epiphyseal fractures with a large potential for growth disturbance
d. Major avulsion fractures with muscle ligament attachments
e. Nonunions that do not have the capacity to unite (pseudoarthrosis and
avascular nonunions

4. Antibiotic considerations:
a. Limiting the duration of the initial antibiotic therapy is important to minimize
the emergence of resistant nosocomial bacteria
b. Type 1 fractures are treated with Cefazolin 2 gm STAT followed by 2 gm Q 8
h for 48 to 72 hours
c. Type 2 and 3 are treated with combined therapy, using cefazolin as above,
plus an aminoglycoside (Gentamycin or Tobramycin) dosed at 1.51.7 mg per kg
on admission, followed by 3.0 to 5.0 mg per kg per day in divided doses. The
duration of therapy is 3 days unless overt infection develops
d. Administer 10 million units of Pen G if the injury was sustained on a farm, to
cover for Clostridium sp.

Soft Tissue Injuries


1. Classification:
a. Tidy wound: Surgical incision, laceration
b. Untidy wound: Crush, avulsion, abrasion
c. Wound with tissue loss: Excision, burn, ulcer, avulsion
d. Infected wound: Established (cellulitis, lymphangitis, abscess, bum, or
vasculitis) or Incipient (bum, contaminated wound, abrasion)

2. Treatment (general):
a. Tetanus prophylaxis
b. Antibiotic prophylaxis
c. H 8 P, including vascular, neurological, musculoskeletal and integumentary
status
d. Inspection of the wound under local or regional anesthesia
e. Initial gentle cleansing of the wound with a mild soap (no strong antiseptics
that can cause tissue damage)
f. X-rays, CBC, and urinalysis as necessary
g. Primary wound care: remove all foreign and devitalized material
copious flushing, atraumatic tissue handling, avoid tourniquet
h. Skin closure when appropriate

3. Treatment (specific):
a. Tidy wound: Once debrided, can be closed after appropriate skin cleansing
(skin edges may be freshened)
b. Untidy wound: Deep damage must be repaired and skin closure should be
delayed until wound demarcation has progressed to the point where viability is
reasonably assured. Secondary or delayed primary closure may be indicated.
Swelling within closed compartments may indicate the need for the release of
damaged fascia or skin
c. Wound with tissue loss: Must prevent the wound from drying out and must
cover exposed vital structures using biological dressings, porcine xenografts, or
appropriate autograft
f. Infected wound: Prior to closure the wound must be debrided and converted
to a contaminated wound, and then a clean wound (check with C&S and colony
counts: less than 105 bacteria per millimeter means
contamination)

Crush Gunshot and Lawnmower Injuries


1. General protocols:
a. Priority is given to prevention of infection especially Clostridia sp.
b. Therefore tetanus prophylaxis is given (see chapter Infectious Disease)
c. Antibiotic therapy is started after cultures are taken
d. Debridement and copious lavage under local/regional, or general anesthesia
e. Depth and extent of the wounds carefully explored and inspection with
removal of all foreign bodies and all non-viable tissue and packed open
f. The wound is reexamined under regional/general anesthesia in 24-48 hours,
and after further debridement the wound is packed open
g. The wound should not be closed before 5-7 days (check cultures and use
clinical judgement)
NOTE* The most important criterion is the clinical appearance of the wound in
the decision to close a wound. The number 105 bacteria present in the wound
is mentioned as a criteria of active infection, as it has been seen on the board
exams (this is unreliable)

h. Use xenograft as necessary to prevent further contamination


i. Use split thickness skin flap immediately on the dorsum of the foot if the
tendons are exposed without the paratenons (this is the only time immediate
coverage is utilized)
j. Rigid stabilization of fractures

Puncture Wounds

These wounds deserve special attention because they characteristically have


a benign presentation that can rapidly progress to OM if not treated
appropriately. Complications run as high as 10%. Pseudomonas is the most
common pathogen isolated

1. General protocols:
a. Tetanus prophylaxis
b. Remove all foreign material, leave wound open, do C&S
c. Start broad spectrum antibiotics
d. If no improvement in 3 days suspect a gram (-) infection
e. If bone or joint is penetrated or if wound is deep, surgical exploration
and debridement are necessary
f. If pain persists after 4 days of treatment use bone/gallium scans, sed rate,
WBC to follow patient progress

Epiphyseal Plate Injuries: Also see section: Pediatrics


(Pediatric Fractures)
1. Anatomic differences: Since the growth plate is radiolucent, acute injury
can only be inferred from widening of the growth plate or from displacement of
the adjacent bones on plain x-ray. The periosteum is stronger, thicker, and
produces callus more quickly than in adults

2. Biomechanical differences: Pediatric bone is less dense, more porous with


a smaller lamellar content than adult bone. It also will fail not only in tension,
as adult bone, but in compression as well. Hence there are certain pediatric
fracture patterns: buckle fractures, plastic deformation of bone, and greenstick
fractures

3. Physiological differences: Growth provides the basis for a greater degree of


remodeling than is possible with the adult (a bump of a malunion is corrected
by periosteal resorption; a concavity is filled out by periosteal new bone). This is
an example of Wolff's Law. Also, a fracture through the shaft of long bone
stimulates longitudinal growth (increased nutrition of the growth cartilage),
which can result in a longer bone as a result of a fracture. Because of this,
pediatric fractures can be treated more conservatively than with an adult

4. Growth plate injuries: Problems after injury are rare, but when growth is
disturbed, the reason is from avascular necrosis of the plate, crushing or
infection of the plate, formation of a bone callus bridge between the bony
epiphysis and metaphysis, and hyperemia producing local overgrowth. There
are 2 types of growth plates, epiphyseal (those that form under pressure) and
apophyseal (those that form under traction)

5. Anatomy: The growth plate is a cartilagenous disc situated between the


epiphysis and the metaphysis. The germinal cells are attached to the epiphysis
and their blood supply is from the epiphyseal vessels. As the germinal cells
multiply, the cell population of the plate increases. The plane" of separation in
the physis is most frequently at the junction of the calcified and uncalcified
cartilage, known as the zone of transformation. With an epiphyseal separation,
most of the important germinal part of the plate usually remains with the
epiphysis. If much of the germinal layer is disturbed, growth may be affected

6. Classification: Salter-Harris
a. Type 1: A complete separation of the growth plate at the zone of
transformation, no disruption of growth, treated with closed reduction and
immobilization for 3 weeks
b. Type 2: Separation of the growth plate with extention of the fracture line into
the metaphysis. This extension creates the 'Thurston Holland Sign'. There is
usually no growth disturbance and it is treated the same way as type 1
c. Type 3: Separation of the growth plate with extension of the fracture line into
the epiphysis so that it is intraarticular. Potential for growth disturbance as the
fracture line crosses the entire growth plate, and must not be left displaced by
ORIF
d. Type 4: A fracture from the metaphysis through the growth plate and into the
epiphysis, and can result in growth disturbance. This fracture is unstable and
requires ORIF
e. Type 5: A crush type injury usually with subsequent growth disturbance.
Treated with closed reduction (if displaced) and immobilization 3-6 weeks NWB
f. Type 6: A scooping out of a portion of the growth plate, via some type of
projectile causing osseous and soft tissue damage. Any large fragments
of bone are reduced. Bony bridging causing growth disturbances can be a
complication here. Treat the bony bridge with resection and interposition of fat
or silicone rubber
g. Type 7: An intraarticular fracture that does not involve the physis. Very
difficult to diagnosis in the very young. Treat with immobilization if
nondisplaced and ORIF if the fragment is large and displaced (or excision of the
fragment if too small to reduce)

7. Apophyseal Injuries: Either an inflammatory process secondary to traction


vs. a Salter-Harris fracture type 1
a. Calcaneal apophysitis: Involves reduction of stress to the apophysis, (heel
lifts, orthoses, and local anti-inflammatory measures) with severe cases
requiring BK casting. Later calf muscle stretching is helpful
b. Tuberosity of 5th metatarsal: Injury from direct impact or forced inversion of
the foot. If the tuberosity is displaced, closed reduction with immobilization for
3 weeks

8. Treatment: Always advise of the long term sequelae of the fracture

9. Internal fixation devices: Smooth K-wires should be employed and should


be buried to avoid infection of the plate which can cause an autolysis of the
plate. Never use threaded pins or screws across a growth plate

10. Specific ankle fractures:


a. Tillaux fracture: A Salter-Harris type 3 of the tibia involving the lateral aspect
of the tibia. This fracture is unique to the age group of 12-13 year olds.
Treatment is ORIF if displaced or unstable, and closed reduction with
immobilization for 6 weeks if stable and in anatomic alignment
b. Triplane: A Salter-Harris type 4 fracture of the tibia. Diagnosis is made by
visualizing the fracture on at least 2 views. An unstable fracture, requiring
ORIF

Digital Fractures and Dislocations


1. Fractures of the hallux:
a. Communited fracture of the distal phalangeal tuft i. Mechanism: Direct
trauma
ii. Treatment:
 Local anesthesia and prep
 Avulse the nail atraumatically
 If closed fracture reduce any gross prominences, replace nail plate as part of
compression dressing, patient instructed to check vascular status
frequently, Reece© shoe, ice packs
 If open fracture, tetanus prophylaxis, IV antibiotics, debride necrotic tissue
and loose exposed bone leaving no prominences, irrigate copiously, open
drainage, Reece© shoe, and follow appropriately

b. Intra-articular dorsal avulsion fracture of the distal phalangeal base:


i. Mechanism: Forced plantarflexion of the hallux IPJ (stubbing) resulting in
avulsion of the EHL insertion
ii. Treatment of displaced fracture (most common): ORIF followed by
Reece© shoe
iii. Treatment of nondisplaced (uncommon): Closed reduction with slipper cast,
or BK cast (with or without percutaneous pinning)

c. Hallux IPJ intra-articular fractures of the distal or proximal phalanx:


i. Mechanism: Transverse plane torque (stubbing) resulting in a push-off
fracture of the medial or lateral condyle of either the distal phalangeal base or
the head of the proximal phalanx
ii. Treatment of displaced fracture (most common): First attempt closed
reduction, and if successful pad 1st interspace with felt or cotton and tape
to the 2nd toe. If closed reduction fails, ORIF larger fragments and excise
smaller fragments. Reese© shoe for 4-6 weeks

d. Proximal phalanx shaft fracture:


i. Mechanism: Direct or indirect trauma resulting in a transverse oblique
fracture
ii. Treatment of displaced fracture: Closed reduction (with or without
percutaneous pinning), splinting, Reese© shoe
iii. Treatment of nondisplaced fracture: Buddy splinting to 2nd toe and
Reece© shoe

e. First MPJ intra-articular condylar fractures of the proximal phalanx:


i. Mechanism: Transverse plane torque resulting in avulsion of the insertion of
the medial or lateral intrinsics
ii. Treatment of displaced fracture: ORIF followed by Reece© shoe for 6 weeks
(smaller fragments maybe excised and the intrinsics reinserted)
iii. Treatment of nondisplaced: If no change in hallux abductus, then buddy
splint to the 2nd toe, if the hallux abductus changed, consider surgical repair.

2. Fractures of the lesser toes:


a. Fractures of the distal and intermediate phalanges: Rare unless crush type

b. Non-articular proximal phalangeal fractures:


i. Mechanism: Direct or indirect trauma (stubbing most common) resulting in a
transverse, oblique or spiral fracture appearing sub-capitally, mid-shaft, or at
the base or epiphysis
ii. Treatment of displaced fracture (less common): Closed reduction followed by
buddy splinting, or ORIF for gross reduction failures
iii. Treatment of nondisplaced fractures (common): Buddy splinting

c. PIPJ intra-articular proximal phalangeal fractures:


i. Mechanism: Usually a stubbing injury with axial forces resulting in an
oblique push off or comminuted fracture
ii. Treatment of displaced and non-displaced fractures: Closed reduction aimed
at restoring alignment, followed by buddy splinting. If failure then primary
arthroplasty.

d. MPJ intra-articular proximal phalangeal fractures: i. Mechanism: Usually a


stubbing injury
ii. Treatment of displaced fracture: ORIF depending on the size of the fragment
and comminution
iii. Treatment of nondisplaced fracture: Reece© shoe for 6 weeks

1st MPJ Trauma


1. Turf Toe:
a. Mechanism: Hyperdorsiflexion, hyperplantarflexion, hyperadduction, or
hyperabduction stress resulting in a - 1st MTPJ sprain without alignment
changes
b. X-ray evaluation: Rule out dislocation, osteochondral injury, or sesamoid
fracture (take MO, LO, Lateral, AP, plantar axial)
c. Treatment: Ice, rest, Reece© shoe, modify the athletic shoe.

2. First MTPJ dislocation:


a. Mechanism: Hyperdorsiflexion
b. X-ray evaluation: Rule out osteochondral fracture and sesamoid fracture
(take AP, lateral, plantar axial)
c. Classification (Jahss):
i. Type 1:
 Joint capsule torn transversely under the metatarsal neck
 Proximal phalanx, plantar capsule, and sesamoids dislocated dorsally on the
first metatarsal head
 First metatarsal protrudes through the capsule, depressed plantarly by the
retrograde forces of the hallux
 Hallux IPJ is flexed
 Usually not reducible by closed technique
ii. Type 2A:
 Same as type 1 except that rather than the entire plantar capsule and
sesamoid apparatus dislocated dorsally, the intersesamoidal ligament
ruptures and the sesamoids sublux to each side of the metatarsal head
 Easier to reduce than type 1
iii. Type 2B:
 Same as type 2A except sesamoid fractures occur instead of the
intermetatarsal ligament rupturing
 Easier to reduce than type 1

3. Treatment:
a. Type 1: Open reduction

NOTE* Closed reduction can be tried under anesthesia as follows: traction and
increase dorsiflexion, then push the proximal phalanx into contact with the
metatarsal head, then push (don't pull) the proximal phalanx into the reduced
position, maintaining contact with the metatarsal head

b. Type 2A: Closed reduction followed by Reece shoe or BK walking cast


c. Type 2B: Closed reduction followed by Reece shoe or BK NWB cast (sesamoid
may have to be excised at a later date prn symptoms) or open reduction with
excision of the fractured sesamoid

3. Sesamoid fractures:
a. Mechanism: Fall from a height, repetitive direct trauma (dancing), and
repetitive indirect trauma (traction of the intrinsics)
b. Presentation:
i. Sesamoid involved: Tibial more than fibular (tibial is larger), rarely both
injuried, almost never bilateral
ii. Clinical presentation: Pain on direct palpation and pain on hallux
dorsiflexion
iii. Differential diagnosis of pain in the sesamoid area: Joplins neuroma,
sesamoiditis, osteochondritis dissecans, osteochondrosis, ruptured bipartite
sesamoid, turf toe, DJD/eroded crista, hypertrophic sesamoid, and fractured
sesamoid
c. X-ray evaluation: Order bilateral AP, lateral and plantar axial (MO for tibial
and LO for fibular sesamoid) 75% of bipartite sesamoids are unilateral. Bone
scan if in doubt
d. Normal sesamoids:
 Ossification appears at 8-10 years
 Bipartite sesamoids more common in tibial than in fibular
 Sesamoids may be multipartite

Nail Bed Trauma


1. Classification (S. Malay):
a. Primary onycholysis:
i. A separation of the nail plate from the bed
ii. Partial avulsions cause posterior nail fold friction injury, subungual
bleeding, and digital sepsis especially in compromised patients
iii. Removal of the nail plate, antisepsis, and antibiotics (prn)
iv. No adverse sequelae
b. Subungual hematoma:
i. Blood clot under the nail plate
ii. Must check for fractures of the plate from impaction
iii. Treat like open fractures, the nail plate must be removed from the tissue to
decompress the area
iv. X-rays should be taken to r/o fracture
v. The nail plate can be removed if the hematoma comprises more than 25% of
the nail plate
vi. Drill holes can be made if feasible
c. Simple nail. bed laceration:
i. Tetanus coverage
ii. Systemic antibiotics
iii. Surgical cleansing and lavage (no epinephrine utilized in seriously
traumatized digits)
iv. If you are avulsing a salvagable nail plate, remove it in one piece and save
for subsequent splinting
v. Nail bed injuries are usually repaired with a 6-0 absorbable suture on an
atraumatic needle
vi. The root and the bed must be accurately aligned on the toe
vii. Periosteal irregularities must be debrided
viii. Reusing the nail plate involves scraping all soft tissue from the nail plate,
drilling holes through the body, then soaked in Betadine until the bed repair is
accomplished, and then the nail plate is replaced on the nail bed and anchored
with Steri-strips®
ix. Avulsive lacerations of the bed are treated with intermediate thickness skin
graft
d. Complex nail bed laceration: As above plus
i. If a major segment of the proximal nail fold over the matrix is avulsed with a
skin defect, rotational flaps are utilized

e. Nail bed laceration with phalangeal fracture: As above plus


i. Subungual fractures must be accurately reduced
ii. Remove all bone spicules and nail fragments

2. Complications:
a. Split nail
b. Adhesions of the skin fold to the nail root
c. Chronic ingrown nails
d. Widening of the nail
e. Narrowing of the nail
f. Protruding or non-adherent nail
g. Malaligned nail

Toe Tip Injuries With Tissue Loss


These injuries are secondary to crush forces, and should be considered and
treated like open fractures. Tissue loss increases the likelihood of poor
cosmetic result
1. Classification (Rosenthal): According to the level and direction of tissue
loss
a. Level of nail bed tissue loss
i. Zone 1: distal to bony phalanx
ii. Zone 2: distal to the lunula
iii. Zone 3: proximal to the distal end of the lunula
b. Direction of tissue loss
i. Dorsal oblique
ii. Transverse guillotine
iii. Plantar oblique
iv. Tibial or fibular axial
v. Central or gouging

2. Treatment:
a. Zone 1:
i. Flush, debridment and appropriate wound closure (usually secondary
intention)
ii. Occasionally skin graft large defects (split thickness less durable, full
thickness more durable)
b. Zone 2:
i. Reduction of bone with debridement of necrotic tissue
ii. Coverage of nail bed and phalanx tip usually achieved by local
neurovascular advancement flap
c. Zone 3:
i. Not suitable to initial treatment in ER or office
ii. Usually complete nail bed loss
iii. OR debridement of necrotic tissue and matrix
iv. Delayed revision of the digit
v. Attempt to maintain tendon function
v. Terminal Symes may be necessary

Dog and Cat Bites


Both dog and cat bites are susceptible to infection because of direct inoculation
of bacteria from the animals into the bite wound. In addition to tearing of
tissue, dogs can also cause a crushing injury. Most patients with bite wounds
harbor bacteria, so that aggressive therapy should be undertaken initially.
1. Pasturella multocida (gram negative bacillus) is present in 50% of cat bites
and 25% of dog bites Other organisms should also be considered:
Pseudomonas, Staphylococcus, and beta streptococcus

2. Some authors believe that the culture of the bite wound offers little
information because of the multiplicity of organisms found and the absence of
an established infection

3. X-rays of the involved area should be obtained

4. Thorough and aggressive debridement and irrigation (manual lavage using


Ringer's lactate or dilute Betadine®)

5. Elevation and immobilization with the ankle at 900, and after 72 hours
improvement occurs, then initiate ROM and adjunctive PT

6. Leave any potentially contaminated wound open for 4-6 days, and at that
time, if the wound is clean, without redness or swelling, it is reasonably safe to
perform primary closure

7. The use of prophylactic antibiotics is still controversial


a. For cat bites, dicloxicillin, penicillin, or Augmentin® is recommended
(erythromycin in penicillin allergy)
b. For dog bites dicloxicillin, cephalexin, or Augmentin® is adequate (one study
showed a 95% cure of infected dog bites with cephadrine)

8. Rabies is of concern with any animal bite. See Chapter 6, Infectious Disease,
section on Rabies
Chapter 26: Digital Deformities
and Surgery
Hammertoe Syndrome
Mallet Toe Syndrome
Claw Toe
Overlapping 5th Toe
Hallux Hammertoe
Hallux Interphalangeal Arthrodesis
Lesser Digital Arthrodesis
Overlapping 2nd Toe
Syndactlyization
Digital Implants
Floating Toe Syndrome
Blue Toe Syndrome
Polydactylism
DIGITAL DEFORMITIES AND
SURGERY
Hammertoe Syndrome
There is a necessity for surgical procedures not only to relieve symptoms, but
also to preserve function. The toe functions to decelerate the foot, stabilize, help
in propulsion, and for kinesthetic sensation. To prevent a recurrence, it is
necessary to investigate and neutralize the deforming forces for full correction
of the deformity

1. Definitions:
a. Hallux hammertoe: This is a deformity whereby there is a dorsal contracture
of the 1st MTP joint and a plantar contracture of the hallux IPJ. There is a
dorsal contracture of the MTP joint capsule and plantar contracture of the
hallux IPJ capsule
b. Lesser hammertoe: Plantarflexion of the PIP joint with dorsiflexion of the MTP
joint.
c. Clawtoe: Dorsiflexion of the MTP joint and plantarflexion of the DIP and PIP
joints.
d. Mallet toe: Plantarflexion of the DIP joint
e. Clinodactyly: Curly toe (transverse plane deviation)
f. Digiti quinti varus: Overlapping 5th toe

2. Classification: Digital deformities are classed according to their flexibility.


This is determined by dorsiflexing the MTP joint and noting the amount of
reduction of the digit (Kelikian push up test) at the MTPJ
a. Flexible: Reducible on weight bearing and with the push up test
b. Semi-rigid: Slightly reducible by hand
c. Rigid: No change in the deformity when examiner attempts manual correction

3. Anatomy: Electromyographic studies have shown that the long flexor fires
approximately at 15% of stance phase, the short flexor at 60%, the EDB at
40%, the EDL during swing and heel contact, and the intrinsics (interossei) at
50%.
a. Extensors:
i. The EDL goes to each lesser digit by a separate tendon
ii. The EDB goes to the middle three lesser toes
 EDL and EDB form a common tendon that passes over the proximal phalanx
to split into three slips. The middle slip goes to the middle phalanx and the
lateral two rejoin to insert over the distal phalanx
 The sling apparatus is part of the extensor hood. The sling mechanism
wraps around the base of the proximal phalanx and attaches to itself. It can
lift up on the proximal phalanx like a sling. There are no specific
attachments of the extensor apparatus into the proximal phalanx
 The EDL and EDB dorsiflex the MTP joint by the sling and cause weak
extension of the DIP and PIP joints, but in vivo cause passive flexion due to
the passive stretch on the fibers.
b. Flexors:
i. Goes to each digit to insert on the distal phalanx after passing deep and
superficial through the FDB under the proximal phalanx. It has the quadratus
plantae attached to its lateral border to help align its pull and the four
lumbricals attached distally and medially.
ii. The FDB is a 1st layer muscle that goes to each toe and inserts on the
middle phalanx.
 Flexor tendons do not insert on the proximal phalanx

 Action of the FDL and FDB gives active flexion of the IPJ's and secondary
passive extension of the MTP joint from the passive pull on the extensor
complex
 These are stance phase muscles that help in stabilization and propulsion of
the foot and digits
 Predominant action of the brevis vs. longus may determine whether the PIPJ
or DIPJ is contracted
c. Intrinsics:
i. There are 4 dorsal interossei. They abduct the digits around a central ray.
Their insertion is on the medial side of the 2nd and the lateral side of the 2nd,
3rd, and 4th proximal phalanx. There is also a point of attachment of both
plantar and dorsal interossei to the plantar plate under the MTP joint.
ii. Plantar interossei are 3 in number. They adduct the 3rd, 4th and 5th digits
toward the 2nd by attaching to the medial side of the phalanx and plantar
plate.
 The interossei run dorsal to the deep transverse metatarsal ligament, but
inferior to the axis of flexion of the MTP joint, and both function to
plantarflex the MP joint and extend the PIPJ and DIPJ
 When both interossei on each side of the MTP joint fire concurrently,
transverse plane abduction/adduction is stabilized
 These are stance phase muscles that help prevent buckling of the toes due
to the mechanism of the sling and production of passive stretch on the
longer extrinsic muscles
iii. Lumbricales are 4 muscles that originate from the medial side of each of the
4 FDL tendons, run beneath the deep transverse intermetatarsal ligament to
insert into the base of the proximal phalanx and form the distal extent of the
extensor expansion.
 They plantarflex the MTP joint and dorsiflex the PIPJ's and DIPJ's
 Not recorded on EMG
 By limiting dorsiflexion of the MTP joint they help hold the digit in a more
rectus position
iv. Quadratus plantae is attached to the FDL laterally coming off the calcaneus.
Its proximal pull helps align the FDL pull to reduce the adductovarus
component to the lateral digits.

d. Arterial and venous supply:


i. The majority of the arterial supply is via the medial plantar artery
digital branches to each digit.
ii. Dorsally the digits are supplied by the dorsal digital proper branches
iii. The venous supply runs parallel to the arterial supply

e. Neurological supply: Divided into plantar and dorsal


i. Dorsal aspect:
 Saphenous nerve runs on the medial aspect of the foot to the 1st MTP joint
 The deep peroneal nerve supplies the adjacent sides of the lateral and
medial aspects of the 1 st and 2nd toes
 The medial dorsal cutaneous branch of the superficial peroneal nerve
supplies the medial aspect of the hallux and the contiguous sides of the 2nd
and 3rd digits

 The intermediate dorsal cutaneous nerve from the superficial peroneal nerve
supplies the adjacent sides of the 3rd and 4th digits, and the 4th and 5th
digits
 The sural nerve supplies the lateral aspect of the dorsum of the foot and the
lateral aspect of the 5th digit
ii. Plantar aspect:
 The medial plantar nerve of the posterior tibial nerve supplies the hallux,
2nd, 3rd, and medial aspect of the 4th digit
 The lateral plantar nerve supplies the lateral aspect of the 4th and 5th digit

f. Summary:
i. Stance:
 FDL and FDB are deforming digital forces, but the FDB is the primary
deforming force
 Interossei and lumbricales have the potential for stabilizing the MTP joint
and neutralizing the deforming forces
 With intrinsic pathology, hammertoes form
 When the intrinsics function properly, stable digital function ensues
 Interossei are stance phase, lumbricales are not well documented (swing
phase hypothesized)
ii. Swing:
 EDL and EDB are active deforming forces that could create a hammer toe in
swing phase by creating MTP joint dorsiflexion and passive plantarflexion of
the IPJ's
 It is assumed in normal foot function, that lumbricales prevent hammertoes
from occurring

4. Etiology: The etiology of the hammertoe will depend to a degree on the time
in the gait cycle when the toe becomes initially deformed. There are 3 basic
mechanisms.

NOTE* These all have a common mechanism: abnormal extention of the


proximal phalanx and secondary passive stretch of the flexors and flexion of
the PIPJ's and DIPJ's

a. Flexor stabilization: Is an increased pull of the long flexors as they gain


mechanical advantage over the intrinsics due to
i. Early flexor firing to help stabilize the hypermobile and flat foot
ii. Intrinsic pathology
 This is a stance phase condition seen from midstance on
 The foot is flexible and forefoot abducted
 The long flexors are firing early and longer to stabilize a pronated mobile
forefoot
 The intrinsics are not able to counter the deforming forces so hammertoe
deformities develop
 The flattened foot may come from forefoot varus, equinus, calcaneal valgus,
torsional problems, muscle imbalances, ligament laxity, and neuromuscular
problems
 Adducto-varus deformity of the 5th and sometimes the 4th toes

b. Extensor substitution: Is a swing phase condition due to weak anterior


muscles (due to ankle equinus, weak lumbricales, and spastic EDL)
i. Marked dorsiflexion of the MTP joint that may straighten on ground contact.
The extensor tendons and the metatarsal heads are prominent
ii. Progresses to a rigid deformity with time
iii. The extensors gain a mechanical advantage over the intrinsics (lumbricales)
when the anterior muscles are firing to dorsiflex the foot at the ankle in swing
to gain ground clearance and at heel contact to prevent foot slap
iv. Patients may be diagnosed NWB by having them dorsiflex their foot.
Normally the digits will dorsiflex approximately 300 at the MTP joints. With
extensor substitution there will be more dorsiflexion of the proximal phalanx
v. Extensor substitution can occur in an equinus foot. There is an increased
declination of the front part of the foot
vi. Even though the digits are rectus on weight-bearing, during swing they curl
vii. Results in an anterior pes cavus
viii. Whatever will allow the long extensors to fire early or gain a mechanical
advantage over the lumbricales will result in extensor substitution

c. Flexor substitution: Occurs where there is weakness of the triceps surae.


i. The posterior deep muscles and the peronei attempt to produce heeloff in
place of the weakened triceps. This may be due to overlengthening of the
achilles or a congenital problem.
ii. A calcaneal gait is common
iii. Produces a hammertoe deformity without the adductovarus component
iv. A supinated high arch foot type is seen

5. Preoperative considerations:
a. If the Kelikian push-up test allows the digit to straighten, then the EDL and
MTP capsule is not so taut so that only a flexor tenotomy may be done
b. 90% of the hammertoe deformities are a result of FLEXOR STABILIZATION
compensation secondary to hypermobile flat feet.
Therefore following surgery orthoses must be used to neutralize the etiology
c. With EXTENSOR SUBSTITUTION an arthroplasty would only be temporary
due to lack of neutralization of the deforming forces, soft tissue releases would
only add a little time, and orthoses function during stance phase and won't
work here. Therefore an arthrodesis is the procedure of choice. A Hibbs
procedure would only be useful in a flexible deformity
d. With FLEXOR SUBSTITUTION strengthening of a weakened triceps by
tendon transfer and fusion of the digits would be in order
6. Surgical procedures:
a. Post procedure: Arthroplasty with proximal head resection
b. Lambrinudi procedure: Fusion of PIP and DIP joints
c. Young-Thompson procedure: Peg-in-hole fusion
d. Gotch procedure (or Gotch and Kreuz): Resection of the base of the proximal
phalanx and syndactylization of the digits
e. Girdlestone procedure: Transfer of the flexor tendons to the dorsum of the
proximal phalanx
f. Sgarlato procedure: Transfer of the FDL dorsally with capsular resection
through a 3 incision approach
g. Taylor procedure: PIPJ arthrodesis using a K-wire
h. Hibbs procedure: A tenosuspension transferring the EDL to the met heads or
base conjointly
i. Collins procedure: Repositioning of the medial and lateral extensor slips
dorsally on the digits
j. Suppan CAP procedure: Indicated for hammertoe correction in children. Two
transverse semielliptical incisions are made over the head of the proximal
phalanx, skin section and tendon and capsule removed, the collaterals are left
intact, metaphyseal osteotomy performed with cylinder of bone removed. The
capital fragment will fit snugly against the shaft of the proximal phalanx and
held snugly by the skin repair

7. Correction of the non-reducible hammertoe: Know the etiology and


neutralize it and follow a stepwise approach during surgery.
a. Arthroplasty to release PIPJ pressure and reduce the corn. Now do the
Kelikian push up test. (If the toe still does not straighten go to the next
procedure)
b. Extensor recession for release of the hood and sling fibers to slacken the
extensor apparatus to the proximal phalanx: If the toes still does not straighten
go to the next procedure

NOTE* When doing this do not cut the lumbricales to the base of the proximal
phalanx

c. Hood release or EDL lengthening: If no straightening go on to the next


procedure
d. Capsulotomy of the MTP joint: If no straightening go on to the next
procedure
e. Plantar hood release
f. If a problem still exists fuse the PIPJ with a K-wire and extend through the
MTP joint held in a rectus position: Allows the digit to function as a rigid beam
and the deforming flexor to pull the entire toe into plantarflexion
NOTE* Test the proximal phalanx for any dorsal resistance after each step. If
the proximal phalanx springs dorsally after it is placed in a corrected position,
go on to the next step

8. Correction of reducible hammertoe deformity:


a. Flexor tenotomy: When there is a flexion at the PIPJ which can be reduced,
the long flexor tendon may be the only pathological entity which needs
correction. If the skin is contracted, it is done through a plantar incision,
otherwise done through a medial or lateral approach.
b. Extensor tenotomy and capsulotomy: When the extensor tendons are
contracted along with the dorsal capsule of the MTP joint, this may be the only
pathological entity which needs correction. Care is made not to injure the
cartilage of the joint. The toe is splinted for 4-6 weeks.

NOTE* It is important to lengthen both the long and short extensors

c. Repositioning of the extensor slips: In digits in which the PIPJ is buckled but
completely reducible, the medial and lateral extensor slips may be repositioned
dorsally on the digits.

NOTE* By performing this procedure, one avoids resection of bone, shortening of


a digit, and flailness. The toe becomes straight, but very little motion is present at
the IPJ.

9. Complications:
a. Floppy digit with phalangeal base resection
b. Edema and sausage toe
c. Floating toe with metatarsalgia
d. Short toe
e. Regeneration of the phalangeal head
f. Infection
g. Decreased sensation
h. Blue toe secondary to venous congestion
i. White toe secondary to arterial spasm

NOTE* The difference between a floppy (flail) toe and a floating toe is that a
floating toe does not purchase the ground while a floppy toe may purchase the
ground however it is unstable.

Mallet Toe Syndrome


A sagittal plane deformity in which the distal phalanx is flexed on the middle
1. Surgery (adults):
a. Two semi-elliptical incisions encompassing the middle phalangeal head will
allow good access for resection of this deformity
b. Must be careful of the neurovascular structures with this procedure
c. Usually the deforming forces emanate from the contracture of the FDL or

abnormal morphology of the bony middle phalanx


2. Surgery (children):
a. Suppan CAP procedure: Performed similar to the hammertoe procedure,
except the metaphyseal osteotomy is done at the head of the intermediate
phalanx, and no subcutaneous sutures are used

1. Definition: Dorsiflexion of the proximal phalanx with plantarflexion of the


middle an distal phalanx

2. Surgery of severe claw toe with MPJ deformity:


a. Incision from DIPJ to metatarsal neck (curving across the metatarsal neck)
b. Z-plasty EDL and retract
c. Prepare bone for arthrodesis (now do Kelikian push up test)
d. Extensor hood release (Kelikian test)
e. Capsulotomy at MPJ
f. Fuse toe with K-wire across the MPJ
g. Repair EDL
h. If medial or lateral dislocation of the flexor plate present, you will need
capsulorrhaphy on one side of the MPJ

Adductovarus 5th Toe Deformity (overlapping 5th toe


This condition is usually hereditary and present most often bilaterally. Before a
procedure is done it is necessary to determine if any functional adaptation has
taken place in the 5th MTP joint. If this has occurred it is then necessary to
perform an osseus procedure (adults only). If there is any skin contracture, a
plastic release must be additionally performed
1. Etiology:
a. Proximal phalangeal base removed
b. Intrauterine position
c. Result of tailor's bunion procedure (tissue contracture)
d. Short EDL
e. Hammertoe toe repair sequelae

2. Diagnosis:
a. Adduction of the toe
b. Contracture of the MTP capsule
c. Medial contracted EDL
d. Varus rotation
e. Extention of MTP joint
f. Subluxed MTP joint

3. Surgical planning:
a. Skin incisions:
i. Z-plasty or V-Y

NOTE* The central arm of the Z-plasty is in line with the direction you want to
lengthen the skin
ii. Plantar elliptical
iii. Plantar V-Y
iv. Longitudinal incisions with dog ear resection
v. Syndactylization
b. Tendon/soft tissue:
i. Release and lengthening of the EDL
ii. Capsulotomy
iii. Plantar capsule release
iv. Transfer EDL to distal stump of abductor digiti quinti
v. Suspend EDL tendon around the metatarsal neck
v. FDL split and reattached dorsally
c. Bony procedures:
i. Resect 5th metatarsal head
ii. Proximal phalangeal head removal
iii. Removal of the base of the proximal phalanx
iv. Abductory wedge removal of phalanx
v. K -wire to hold position
4. Procedures:
a. Lapidus procedure: Extensor tenotomy with transfer of the distal stump
under the proximal phalanx to attach to abductor digiti quinti
b. Kelikian procedure: Syndactyly of the 4th and 5th after capsule release and
arthroplasty of the 5th toe
c. Ruiz Mora procedure: Resection of the proximal phalanx and semieliptical
plantar crease incisions to hold in corrected position
d. Lanzonis procedure: Extensor tenosuspension of the 5th metatarsal head
and fusion of the PIPJ and MTP capsulotomy
e. Goodman-Swisher procedure: A V-Y plasty, Z-tenotomy and capsulotomy
f. Butler procedure: Two concurrent racket shaped incisions completely
encircling the toe so to derotate and plantarflex
g. McFarland procedure: Proximal phalangeal head removed, Jones suspension
and syndactylization
h. Jahss procedure: Ruiz Mora incision with diaphesectomy of the proximal
phalanx

NOTE* In a child osseous procedures usually do not have to be performed. A Z-


plasty or V-Y plasty may be utilized to release the skin contracture. An
extensor tenotomy at the level of the MTP joint and capsulotomy are performed.
A plantar skin wedge can be removed. Toes are splinted for 4 weeks

NOTE* In adults it is usually necessary to perform an osseous procedure at the


level of the MTP joint. If the base of the proximal phalanx is resected, then
syndactylism of the 4th and 5th digits should be performed. If a partial
metatarsal head resection is performed, then a Z-plasty or V-Y skin plasty is
performed. Some also advocate the removal of a transverse skin ellipse
plantarly to help hold the toe in position

Hallux Hammertoe Deformity


1. Etiology:
a. Muscle imbalance seen with a cavus foot type
b. Following surgical procedures of the 1 st MTP joint
i. Especially with removal of both sesamoids
ii. Detachment of the flexor brevis tendons at their insertion onto the base of
the hallux tendons
iii. Overzealous HAV surgery with medial subluxation of the tibial sesamoid
c. In the presence of IPJ sesamoids which bind down the long flexor tendon in a
shortened position
2. Flexible deformity: An IPJ fusion with EHL lengthening may be done,
approached through 2 semi-eliptical incisions,

NOTE* Fixation for fusion is either with 2 K-wires, AO fixation (4-0 cancellous,
3.5 cortical, 2.7 cortical), or 28 gauge monofilament wire loops

NOTE* AO fixation of the IPJ cannot be used with a total joint replacement
unless 2.7 mm cortical screw modification is utilized, but can be difficult
NOTE* Monofilament wire fixation and crossed K-wires are the best choices
when planning to utilize a total joint replacement

NOTE* When doing an IPJ fusion, the propulsive phase of gait should be
eliminated for 6 weeks

3. Rigid hammertoe deformity: Jones tendon transfer plus IPJ fusion

4. Rigid hammertoe deformity plus rigid plantarflexed 1st ray: IPJ fusion
plus dorsal wedge osteotomy of the 1st metatarsal

4. Postoperative complications:
a. Non-union
b. Hallux limitus or rigidus
c. Hallux extensus
d. Elevatus of the first metatarsal with IPK sub 2nd metatarsal

Hallux Interphalangeal Arthrodesis


1. Fixation techniques:
a. Stainless steel monofilament wire, 28 gauge
b. Two 0.045 Kirschner wires
c. 4.0 cancellous screw
d. 3.5 cortical screw (lag)
e. 2.7 cortical screw (lag)

2. Indications for fusion:


a. Semi or non-reducible IPJ contracture
b. Hyperkeratosis overlying the IPJ
c. Transverse or frontal plane deformity of the hallux
d. Clawtoe deformity
e. Abnormally long or short toe

Lesser Digital Arthrodesis


1. Biomechanics:
a. Flexor substitution
b. Extensor substitution

2. Signs and symptoms:


a. Semi-rigid or rigid deformity
b. Dorsal hyperkeratosis
c. Transverse plane deformity may be present
d. Clawtoe deformity may be present
e. Abnormally long or short toe may be present
f. Painful PIPJ motion may be present
g. Flail toe secondary to previous surgery
3. Fixation:
a. Stainless steel monofilament 28 gauge wire loops
b. 0.045 K-wire
c. Reese arthrodesis screw
d. Orthosorb®

NOTE* Fusion of the 2nd toe will not stop the formation of a hallux abductus
deformity

Overlapping 2nd Toe


1. Etiology:
a. Chronic biomechanical forces
b. Intra-articular steroid injections
c. Inflammation of the joint capsule (seen with RA)

2. Surgery:
a. Resection of phalangeal base
b. Flexor tendon transfer
c. Proximal IPJ arthrodesis
d. Partial met head resection
e. Relocation of the flexor plate
f. Freeing the base of the proximal phalanx from attachments and freeing
the metatarsal head from attachments and fixating with K-wire
g. Repositioning of a 2nd MTP capsular flap
h. Total implant arthroplasty

Syndactylization
1. Classification:
a. Type 1 (zygodactyly): Most common
i. 2nd and 3rd digits most frequently involved (followed by the 3rd and
4th digits)
ii. Asymptomatic and requires repair primarily for cosmetic reasons
b. Type 2 (synpolydactyly):
i. Associated with duplication of a part or entire digit (the duplicated digit
usually intervenes between two essentially normal digits)
ii. 3rd and 4th digits primarily affected, followed by the 4th and 5th
toes
iii. Usually discomfort due to shoe irritation when the 5th toe is involved
c. Type 3: Fingers only
d. Type 4: Fingers only
e. Type 5
i. Syndactyly with concomitant metatarsal (or metacarpal) synostosis

2. Surgery: Plastic flap repairs (see following diagram)


Digital Implants
1. Signs:
a. Deformity involves the 2nd and or 3rd toe at the PIPJ
b. Semi-rigid or rigid hammertoe deformity
c. Painful PIPJ motion may be present
d. Hyperkeratosis may be present
e. Involved toe is of normal or shortened length when placed in its proper
position
f. Absence of significant MTP joint or DIP joint contracture of the involved
toe
g. Absence of significant frontal plane deformity of the involved toe
h. Normal skin condition, vascular status, and neurological status

2. Radiographic findings
a. Adequate bone stock to receive the stems of the implant
b. Adequate width of the proximal and intermediate phalanx to receive the
stems of the implant
c. Adequate length of the intermediate phalanx to receive the stem of the
implant
d. Absence of MTP and/or DIPJ contracture
e. DJD of the PIPJ may be present

3. Implant product selection:


a. Silastic H.P. 100, (Swanson Type) Weil Design, Dow Corning Wright:
A double stemmed flexible implant with cylindrical central body
b. Sutter Lesser Toe Proximal Interphalangeal Joint Prosthesis (Sgarlato
Design), Sutter Biomedical: A double stemmed very flexible implant with a
central hinge and rectangular stems with a polyester mesh internal fabric for
reinforcement
c. Sgarlato Hammertoe Implant Prosthesis, Sgarlato Labs: The newest device,
also a double stemmed with a trapezoidal solid central portion

4. Implant procedure:
a. Two longitudinal semi-elliptical incisions (to prevent fat toe syndrome)
b. The dorsal tendinous structure is dissected free from the base of the distal
phalanx to the middle of the shaft of the proximal phalanx, and is retracted
medially or laterally
c. The PIPJ is entered by severing the capsular ligaments
d. The head of the proximal phalanx is excised at the surgical neck (a little
more bone is removed than with a traditional arthroplasty)
e. The proximal phalangeal stump is reamed first, and the middle phalangeal
stump is reamed
f. The implant is inserted, and there should be a 2-3 mm space between the
implant and each bone (very important)
g. The tendon and skin are then repaired
5. Advantages of digital implants:
a. Relief of pain
b. Maintenance of toe stability
c. Maintenance of toe length
d. Restoration of function
e. Allows for PIPJ motion and plantar gripping power of the toe
f. Minimal postop disability and early toe motion

6. Disadvantages (versus regular arthroplasty):


a. Cannot be performed in an abnormally big toe
b. Difficult to perform in the 4th and 5th toe due to small bone stock

NOTE* The Sgarlato (S.H.I.P.) and Sutter device have been used in the 5th toe
due to their size.. The Sutter which has the advantage of having a small
central portion whose thickness is less than its width, can be placed either
angled or vertical to prevent pressure from the shoes or adjacent tissues. The
S.H.I.P. can be placed normally

c. Need good bone stock and adequate width of bone


d. Need to have normal. sagittal plane position of the MTP joint
e. Needs specialized equipment
f. Cannot be used with frontal plane deformity
g. Need to remove implant if infection occurs
h. The Silastic Swanson design could permit lesion recurrence and digital
swelling due to the large diameter central portion
i. The Sgarlato implant is also available in a longer stemmed version which is
useful in revisional surgery as well as digits with longer phalanges

7. Contraindications:
a. Nonreducible contracture of MTP joint and/or DIPJ of the involved toe
b. Inadequate bone stock
c. Infection
d. Inadequate vascular status
e. Significant frontal plane deformity
f. Presence of an implant at the MTP joint of the involved toe
g. Inadequate skin coverage

Floating Toe Syndrome


1. Etiology:
a. Bradymetatarsia
b. Excessively elevated metatarsal
c. Dislocated flexor plate
d. Procedures which reduce the internal cubic content of the joint
e. Resection of the base of the proximal phalanx

2. Surgery:
a. Correct the underlying etiology
b. PIPJ fusion
c. Total joint replacement

Blue Toe Syndrome


1. Definition: Blue toe syndrome/purple toe syndrome results from
atheromatous embolization, which can eventually lead to gangrene if the cause
is not eliminated

2. Causes:
a. Atherosclerosis (most common)
i. Thrombosis formation,
b. Infection
i. Microthrombi formation
ii. Secondary syphilis
c. Atheroembolism
i. Cholesterol emboli from ulcerated plaques in the more proximal
vessels
ii. Mural wall thrombi
iii. Endocarditis
iv. Myxoma
v. Vascular surgery
vi. Angiography
vii. Meningitis
d. Anticoagulation
i. Coumadin
e. Thrombolytic activity
i. Tissue plasminogen activator
ii. Streptokinase
f. Drugs
i. Dopamine
ii. Beta blockers
iii. Steroids
iv. Epinephrine (in local anesthetics)
g. Hyperviscosity syndromes
i. Cryoglobulinemia
ii. Cold agglutinins
iii. Polycythemia vera
h. Hypercoagulable states
i. Malignancies
ii. Diabetes mellitus
i. Vasculitis
i. Polyarteritis nodosa group
ii. Hypersensitivity group
ii. Wegener's granulomatosis group
iv. Giant cell arteritis
j. Foot surgery

3. Signs and Symptoms:


a. Pain
b. Bluish mottling of the digit
c. Pedal pulses tend to be present
d. Can be either bilateral or unilateral

4. Origin of the Emboli:


a. Bilateral signs and symptoms in the toes suggests ulcerated plaques in the
aorta.
b. Unilateral signs suggests ulcerated plaques in the iliac, femoral, of
popliteal arteries.

5. Treatment:
a. Angiography to determine the location of the plaque
b. Photoplethysmography of the digits
c. Removal of the atheromatous plaque
d. Endarterectomy
d. Risk factor modification
e. Amputation as necessary
f. Medical therapy as needed (i.e. D/C anticoagulant or other suspected
causative agent, use of antibiotics or other drug)

Polydactylism
A hereditary malformation, transmitted as an autosomal dominant trait. It may
occur as a single deformity in the foot (nonsyndromatic) or may be associated
with accessory digits in the hand, and there may be other congenital
malformations as well (syndromatic). The digital deformities may be pre-axial
(hallux) or post-axial (5 toe) or central toes 2,3,4). The duplication of the toe
may be complete or involve the distal phalanx or the distal and middle phalanx.
The metatarsal may be partially or completely duplicated. Duplicated digits may
share a common metatarsal. Shoe fit is the major problem.
1. Classification (Temtamy and McKusick): Adapted from the classification of
the hand (less applicable to the foot)
a. Pre-axial:
i. Type 1 to type 4
b. Post-axial:
i. Type A: A fully developed accessory digit that articulates with either the 5th
metatarsal or with a duplicated 5th metatarsal
ii. Type B: Characterized by an accessory digit devoid of osseous tissue which
represents a vestigal digit

NOTE* Venn-Watson further divided postaxial polydactyly into 5 specific


morphological patterns, based on the degree of metatarsal duplication

1. Surgical tenets:
a. The most rudimentary digit (least important) should be excised when
possible leaving 5 toes
b. Try to achieve a normal functioning foot as well as a cosmetically pleasing
one
c. Avoid scar on the medial or lateral side of the foot where shoe pressure will
irritate them

2. Surgical excision of lateral pre-axial toe:

a. If any other congenital deformities are present, they should be corrected first

b. The above diagram shows an accessory pre-axial great toe that is in varus
secondary to a metatarsus adductus. The metatarsus adductus is treated first
via casting (which will help stretch the medial skin of the hallux prior to
surgery). In the above case it is best to remove the lateral toe so the scar line is
on the lateral aspect. Redundant soft tissue can be excised from the 1st
interspace, and the adductor hallucis stump from the amputated toe is sutured
to the base of the remaining proximal phalanx, and the intermetatarsal
ligaments are repaired. This helps straighten the toe and close the
intermetatarsal angle

3. Surgical excision of a medial postaxial toe:


a. The following diagram illustrates the lesser developed medial 5th toe.
Excision is made via two longitudinal eliptical incisions around the accessory
5th toe which meet at approximately midshaft of the 5th metatarsal
Chapter 27: Muscle and
Tendon Pathology
Muscle Physiology
Principles of Tendon Repair
Tendon Lengthening and Tenotomy
Tendon Transfers
Tendon Grafts
Posterior Tibial Tendon Rupture
Posterior Tibial Tendon Dysfunction (Acquired Adult
Flatfoot Syndrome)
Peroneal Tendon Pathology
Achilles Tendon Rupture
Lateral Ankle Stabilization Procedures
Postoperative Care and Training Following Tendon
Transfer
Tenosynovitis
MUSCLE AND TENDON PATHOLOGY
Tendon repair and tenoplasty are integral parts of many podiatric procedures,
thus it is imperative that the podiatric surgeon be familiar with the principles of
tendon healing and repair. Knowledge about tendon healing will allow the
surgeon to make appropriate decisions concerning the procedure performed,
materials used, postoperative care, and potential complications

Muscle Physiology
1. Anatomy:
a. Connective tissue surrounding the muscle
i. Intact muscle enclosed by the epimysium
ii. Muscle fascicles enclosed by perimysium
iii. Individual muscle fiber enclosed by endomysium
b. Muscle's structural and functional subunits:
i. Fasciculus
ii. Muscle fiber
iii. Myofibril: The myofibrils complex protein structure is the basic contractile
unit:
 Actin: Thin protein filament containing contractile proteins tropomysin and
troponin
 Myosin: Thick protein filament
iv. Sarcomere: the smallest functional unit of the muscle fiber extending from
one "Z" line to the next
 The myofibril is composed of alternating "A" bands corresponding to the
thick myosin filaments and "I" bands corresponding to the thin actin units
 The "A" band encloses the "H" band (where cross bridges are absent) and in
the middle of the "I" band is the "Z" line
 T-tubules are invaginations of the sarcolemma which form an
interconnected network
 The sarcoplasmic reticulum extends from one T-tubule to the next forming
the terminal cisternae
v. Myoneural junction:
 The axon gives rise to several terminal twigs, the end of each is dilated and
unmyelinated
vi. Organelles:
 Mitrochondria are found between the myofibrils and appear in varying
amounts depending upon the type of muscle fiber
 Cytochromes for oxidation, and glycogen appear in varying amounts
c. Tendons:
i. Dense connective tissue between connective tissue in muscle, and insertion
area
ii. Golgi tendon organs transmit information concerning tendon tension

2. Physiology of muscle fiber:


a. Muscles function according to the Sliding Filament Theory of Muscular
contraction: rest, excitation coupling), contraction, recharging, and relaxation
b. Initiation of contraction:
i. The axons released acetylcholine which initiates an action potential
along the sarcolemma
ii. The action potential is propagated into the depths of the myofibril via
the T-tubule system
iii. This in turn mediates a release of calcium from the sarcoplasmic
reticulum (the sarcoplasmic reticulum has an active pump to recover
calcium once it is released)
iv. In the presence of repeated neural stimulation calcium will remain in
the sarcoplasm
v. Following the action potential is an absolute refractory period, during
which no action potential may be initiated
vi. Next follows a relative refractory period during which a greater than
normal neural stimulus may initiate another action potential in the
sarcolemma

c. Myofibril contraction (the ratchet mechanism):


i. In the absence of calcium, tropomycin blocks the myosin-binding sites on the
F-actin
ii. When calcium is released by the sarcoplasmic reticulum into the sarcoplasm,
it bonds to the Tn-C portion of the troponin, which mediates a conformational
change in tropomyosin which uncovers the myosin binding sites
iii. ATP binds to heavy meromycin (form of myosin), which releases it from the
actin. The ATP-ase activity of the heavy meromysin then cleaves the phosphate,
and the myosin can once again bind to actin
iv. Upon binding with actin, the heavy meromysin changes conformation,
thereby pulling the actin along
v. As calcium is reabsorbed by the sarcoplasmic reticulum, the myofibrils again
relax

NOTE* Motor unit response is an all or nothing

d. Fast twitch vs. slow twitch muscle

Feature Fast Twitch Slow Twitch


# of type 1 fibers + +++
# of type 2 fibers +++ +
Speed of response +++ +
Strength + +++
Stamina + +++
Major energy source anaerobic aerobic

e. Type 1 vs. Type 2 muscle

Feature Type 1 Type 2


Color RED WHITE
Myoglobin content +++ +
Mitochondrial content +++ +
Glycolytic enzymes + +++
Glycogen content + +++
Complexity of T-tubules + +++
Speed to respond to stimulus + +++

f. Energy system function:


i. Phosphagen system (anaerobic)
 ATP and phosphocreatine
 Availability of energy is rapid
 Small amounts of energy available for a few seconds (30 seconds)
 Utilized in sprinting, jumping, swimming etc.
 Primary source of energy muscle storage
ii. Glycolytic system (anaerobic)
 For activities 30 seconds to 1.5 minutes
 No oxygen required
 Formation of lactate takes place
 Lactate accumulation results in oxygen debt and muscle fatigue
iii. Aerobic phosphorylation
 For activities greater than 1.5 minutes
 Carbohydrates, proteins and fats utilized through the Krebs cycle and
electron transport system (oxidative phosphorylation)
 Site of energy formation is the mitochondria -No lactate formation

NOTE* Sport activities require all 3 sources of energy but in different


proportions depending on the sport characteristics

NOTE* Rigor mortis occurs due to exhaustion of A TP in the presence of calcium


(in the absence of ATP, myosin becomes tightly bound to actin)

3. Training programs:
a. Most training programs are an adaptation of sprinting and endurance
training
i. Endurance training results in hypertrophy of type 1 fibers
ii. Sprint training results in hypertrophy of type 2 fibers

NOTE* Fiber type can change in response to training programs, but there is no
proof that one fiber can transform to another fiber type

iii. Training results in cardiac hypertrophy, increase in cardiac stroke volume,


and decrease in heart rate

4. Types of training:
a. Isometric excercises:
i. Contraction in which a muscle maintains a constant length
ii. Contraction against stationary objects
iii. Maximal isotonic contraction increases strength to a greater extent than
submaximal contractions
iv. Isotonic training does not require a long time of excercising
v. Maximal strength gained is very specific for the joint angle at which the
training is performed
vi. Motor performance is not increased by isotonic exercise
vii. Isometrics are static excercise
viii. Strength gained by isometrics decrease the maximal speed of a limb

b. Isotonic excercises (concentric): Implies constant tension


i. During isotonic excercise the amount of force exerted on a weight being lifted
depends on the acceleration of that weight (Newton's 2nd law: F+W+MA)
ii. In case of isotonic training, voluntary maximal contraction (VMC) takes place
somewhere during training
iii. The maximal lift is really the weakest point in the ROM of the joint. If the
weight could be lifted quickly on each repetition, the VMC would be possible to
perform through the entire ROM
iv. Motor performance is increased
v. There is an increase in lean body mass and a decrease in body fats
c. Eccentric excercises: Negative weight training
i. A contraction in which the muscle lengthens upon contraction
ii. Strength gained is not different than in concentric or isotonic contraction
iii. Less effort is required to gain the same strength as in concentric training
iv. Tension developed in the muscle during eccentric contractions (as compared
to concentric contraction) utilizing equal weights indicates that more tension is
developed during concentric contraction
v. To cause equal tension as that during concentric training, more weight has to
be used during eccentric training
vi. Eccentric training has the following disadvantages:
 Muscular soreness
 Trainer is necessary to use heavier weights
 Weight handled can be hazardous
 Training time is longer than with other training methods

d. Isokinetic excercises: Constant velocity excercise


i. The machine is set at a constant velocity but the resistance is not set;
whatever force the trainee applies to the machine set at a certain velocity, is
offered as a resistance to the trainee by the machine
ii. The above allows the VMC throughout the entire ROM
iii. To be strong at fast speed of movement, the athlete should be training at
fast speed of movement
iv. Isokinetics increases motor performance
v. Motor performance is increased to a greater extent at fast speed than at slow
speed
vi. Muscular soreness is minimal
vii. No weight is lifted
vii. Length of the workout is decreased

e. Variable resistance exercises:


i. Percentage of increase in resistance that an individual can tolerate and
complete a particular movement depends upon the following variables:
 Limb length
 Muscle length
 Point of attachment of the tendons on the bone
 Bony position
ii. Motor performance increases with variable resistance machinery (high cost)

f. Comparisons
i. Isotonic vs. isokinetics:
 Strength increase is greater in isokinetic as compared to isotonics
 Isokinetic contractions are preferred over isotonics
 Isokinetic training increases isokinetic and isotonic strength more than
isotonics
 Isokinetic training (at fast speeds) increases motor performance more than
isotonics
 Isokinetics is as effective as isotonics in decreasing fat and increasing lean
body mass
 Less muscle soreness with isokinetics over isotonics
ii. Isokinetics vs. isometrics:
 Isokinetics causes a greater increase in isometric and isokinetic strength
than does isometrics
iii. Isotonics vs. Isometrics:
 Motor performance is Improved to a greater extent with isotonics than
isometrics
iv. Variable resistance vs. isometrics:
 Probably better motor performance with variable resistance

NOTE* The strength training choice will depend upon the cost of the equipment,
amount of strength gains, and motor performance increase

Principles of Tendon Repair


1. Histology:
a. Tropocollagen: The most basic molecular unit of tendon

NOTE* Successive molecules of tropocollagen are assembled and eventually


form collagen fibers. These longitudinally anastomosed fibers constitute a
tendon

b. Tendons are composed of 3 anatomical coverings:


i. Endotenon: Surrounds groups of collagen fibers and forms units called
fascicles
ii. Epitenon: This covers groups of fascicles. It is also the visceral layer and is
responsible for the intrinsic repair response
iii. Paratenon: A loose filmy structure that covers the entire tendon and has a
rich vascular supply that communicates with the, tendon itself. The paratenon
allows the tendon to glide
NOTE* Without the paratenon, the tendon would stick to the surrounding
tissue, so care must be made during surgery not to damage this structure

2. Tendon healing:
a. 4 stages each taking approx. one week
i. Stage 1: The severed ends being joined by a fibroblastic splint. At the end of
this stage the repair site is in its weakest state consisting of serous material
and granulation tissue (termed zone of degeneration)
ii. Stage 2: Shows an increase in paratenon vascularity and collagen
proliferation. Immobilization is still necessary.
iii. Stage 3: Collagen fibers begin to form longitudinally and give the tendon a
moderate degree of strength. At this time controlled passive motion is beneficial
to decrease the formation of fibrous adhesions (CPM)
iv. Stage 4: Exhibits fiber alignment which imparts increased strength to the
tendon. At this point active mobilization can be initiated

b. Tendon lengthenings will often result is a loss of muscle strength roughly


equal to one grade of manual examination once healed

3. Suture selection: Plays a vital role in uneventful tendon surgery


a. Surgilon®: This non-absorbable non inflammatory suture allows for increased
strength during the end of stage 1 when the tendon is the weakest
b. Stainless steel: Excellent to anchor tendon to bone and then removed when
healing occurs. It is the strongest and least reactive, and best in contaminated
wounds. Its drawbacks are it can "kink" up and "saw" through a tendon.
c. Silk: Was used for years but has been replaced with less reactive
nonabsorbable and absorbable sutures.
d. Tevdek®/Ticron®: Nonabsorbable braided polyester that retains greater
ability to resist gap-producing forces at 3 weeks than either nylon or
polypropylene
e. Vicryl®/Dexon®: Absorbable polygalactic acid and polyglycolic acid usually
provide strength long enough for the repair

4. Methods of tendon repair: tendon to tendon suture techniques (see


following diagrams)
a. Bunnell end-to-end: Excellent technique but can cause tissue restriction
b. Double right angle: Good for quick repair of small tendons
c. Lateral trap: Firmly grips the outside of the tendon without constricting the
microcirculation in the center. The central mattress suture acts as a temporary
anchor.
d. Chicago: A simple x-stitch described by Mason and Allen
e. Robertson: An excellent method of anastamosing tendons of unequal
diameter
f. Interlace: Another method for attaching smaller to larger tendons as in
tendon grafting
g. Herringbone stitch and insertion: A method of grafting one tendon into the
center of another
h. Bunnell pull-out suture: A pull-out stitch is a non-absorbable suture that
anchors a deep stitch to the outside of the skin so it can be removed once
healing it complete. Anchored to the outside with a button.

NOTE* Side-to-Side anastamosis of a transferred tendon provides the most


physiological pull, the greatest danger is that of slippage, so the adjoining
surfaces should be roughened and the epitenon scraped free (encourages
fibrous union) and then sutured
NOTE* When suturing a tendon it is important to preserve the microcirculation
and therefore not encircle or strangle large amounts of tendon tissue. Close
apposition is important but the tendon ends must not bunch or overlap
excessively. Tendon-to-tendon approximation must be equal otherwise fibrous
tissue extrusions will bind to the surrounding tissues. Necrotic ends must be
debrided

5. Securing tendon to bone: Most secure form of fixation


a. Trephine plug: Using a Michele vertebral trephine a hole is drilled into the
bone with the tendon pressed inside and the resultant plug is later tapped into
place securing the tendon
b. Three hole suture (see diagram prior page): Anchoring the transposed tendon
with a double armed suture and placing it in a drill hole. The sutures (a
nonabsorbable polyester suture is recommended) are then tied into 2 adjacent
small drill holes.
c. Buttress and button anchor: For tenodesis using a nonabsorbable suture
(stainless steel) that is removed once the healing is complete
d. Tunnel with sling: Can only be used with a tendon with sufficient length.
Made via a tunnel in a bone with the tendon passed through and sutured on
itself. Used with a Jones suspension of the EHL
e. Screw and washer (cleated polyacetyl): Useful where there is little soft tissue
for the transferred tendon can be sutured.
f. STATAC Device (Zimmer® Inc.): Titanium implant that is drilled into the bone
with non-absorbable sutures attached that can be threaded to Keith needles
and sewn through a tendon.

6. Objectives of tendon transfer:


a. To improve motor function where weakness and imbalance exist and
thereby prevent contractures and further deformity
b. To eliminate deforming forces
c. To provide active motor power
d. To provide better stability
e. To eliminate the need for bracing
f. To improve cosmesis

7. Principles of tendon transfers:


a. Select suitable cases, do not create a new imbalance
b. Understand the anatomy and physiology
c. Correct the fixed or structural deformities first
d. Select the proper timing (age of patient)
e. Select a suitable tendon for the transfer (adequate length and strength)
f. Provide a direct or mechanically efficient line of pull
g. Perform stabilizing procedures first (if needed)
h. Preserve the gliding mechanism
i. Use atraumatic technique
j. Preserve blood supply and innervation
k. Provide adequate muscle-tendon tension on fixation
l. Use secure fixation techniques
m. Provide detailed postoperative management

8. Grading system of manual muscle testing:


5 Normal Full resistance at end range of motion
4 Good Some resistance at end ROM
4+ Moderate resistance at end ROM
4- Mild resistance at end ROM
3 Fair Able to move against gravity alone
2 Poor Able to move with gravity eliminated
1 Trace Can palpate or visualize muscle contraction
0 Zero No evidence of muscle contraction

Tendon Lengthening and Tenotomy


Tendon lengthening and tenotomy have limited indications when abnormal
contracture of a musculotendinous unit compromises normal function.
Absolute tenotomy has few applications in reconstructive foot surgery (severing
the adductor tendon in HAV surgery, the FDL for mallet toe, and tenotomy for
lengthening of the Achilles tendon)
1. Common procedures:
a. Strayer technique (distal recession): Modification of the Volpius-Stoffel
procedure. Lengthening the gastrocnemius ms., requiring the complete
severence of the aponeurosis, suturing the retracted proximal aponeurosis to
the underlying soleus, and casting the foot in neutral to allow for healing at the
new length
b. Silverskiold procedure (proximal recession): Release of the muscular heads
of the spastic gastrocnemius from the femoral condyles and reinsertion to the
proximal tibial area (a 3 joint muscle is converted to a 2 joint muscle)
c. Fulp and McGlamry tongue-in-groove procedure (distal recession):
Modified Baker procedure. The tongue-in-groove cuts are inverted in the
gastrocnemius
NOTE* These procedures are utilized for the correction of non-spastic ankle
equinus secondary to gastroc. shortening. If a spastic gastroc. equinus is
present then you must also perform a concomitant excision of the central soleus
aponeurosis
d. White tenotomy: Tenotomy of the anterior 2/3 of the distal end of the
Achilles tendon and the medial 2/3 of the tendon, performed 5-7.5 cm proximal
to the insertion (presumption of torque)
e. Hoke's tenotomy: A triple tenotomy of the Achilles tendon starting 2.5 cm
from the insertion and the others at 2.5 cm intervals extending proximally
f. Hibbs procedure: Tendo Achilles lengthening via a lateral skin incision, with
the medial 2/3 of the tendon divided proximally and then split longitudinally in
the distal direction at the lateral end of the incision. The lateral 2/3 of the
tendon is then divided near the point insertion and it is split longitudinally in
the proximal direction at the medial end of the incision
g. Sliding Z-plasty: Can be used for the Achilles (Hauser or White procedures)
or extensor tendons
h. Abductor hallucis tenotomy: Tenotomy of the abductor hallucis in the
treatment of congenital hallux varus and metatarsus adductus.

NOTE* Tendon lengthenings (once healed) will often result in a loss of muscle
strength roughly equal to one grade of manual examination

Tendon Transfers
1. Common procedures:
a. Murphy modification (for advancement of the tendo Achilles): Is utilized
in young patients with CP where the spasticity of the triceps is causing ankle
equinus. This procedure is performed by transecting and rerouting the achilles
tendon into the calcaneus distally just proximal to the subtalar joint
b. Peroneus brevis tendon transfer: This muscle is transferred to aid in
dorsiflexion via rerouting the tendon medially into the 3rd cunieform.
c. Peroneus longus tendon transfer: This muscle is transferred when
additional dorsiflexory power is needed via rerouting the tendon medially into
the 3rd cuneiform. It can also be rerouted into the posterior calcaneus when
paralytic calcaneal deformities are present

NOTE* The peroneus longus tendon transfer to the cuneiform is utilized with a
drop foot deformity and weakness or paralysis of the anterior muscle group

d. Tibialis posterior tendon transfer: Has the potential to be a good


dorsiflexor when replacement is needed via rerouting the tendon laterally, and
inserting it into the 3rd cuneiform

NOTE* The tibialis posterior tendon transfer is indicated when a weak or


paralyzed anterior muscle group is present, equinovarus deformity, drop foot,
Charcot-Marie-Tooth deformity, and permanent peroneal nerve palsy

e. Tibialis anterior tendon transfer: To reduce the supinatory forces in the


foot via detaching the tibialis anterior over the navicular and rerouting it
laterally into the 3rd cuneiform.

NOTE* Tibialis anterior tendon transfer can be used for recurrent clubfoot,
flexible forefoot equinus, drop foot, and Charcot-Marie-Tooth deformity

f. Split tibialis anterior tendon transfer (STATT): Its goal is to increase true
dorsiflexion of the foot by balancing its power laterally via splitting the tibialis
anterior and suturing the lateral portion to the peroneus tertius (see chapter
21, Surgery of the Congenital Foot)

NOTE* The STATT is recommended for spastic rearfoot varus, fixed equinovarus,
excessive invertor power, forefoot equinus with swing phase extensor
substitution and claw toes, flexible cavovarus deformity, and dorsiflexory,
weakness

h. Hibbs tenosuspension: Is performed to release the retrograde bucking at the


MPJ's causing the flexible forefoot equinus, is done via detaching all 4 tendons
of the EDL distally enough and fused at the base of the 3rd metatarsal to the.
corresponding EDB

NOTE* EDL slips 4 and 5 must be attached to EDB tendon 4

i. Jones suspension: Used for treatment of a cocked-up hallux by transecting


the EHL at the IPJ of the hallux and rerouting it through a medial to lateral
drill hole in the head of the 1st metatarsal
NOTE* The Jones suspension has been utilized for flexible cavus foot flexible
plantarflexed 1st ray (with or without hammered hallux), and prophylaxis
when both hallucal sesamoids are removed

j. Young procedure: A tendon transposition (rerouted through a "keyhole" in


the navicular) for flatfoot (see chapter 21, Surgery of the Congenital Foot)
k. Kidner procedure: Advancement of the tibialis posterior either inferior to the
navicular bone or modified by attachment to the medial cuneiform to increase
its adductory influence on the forefoot (see chapter 21, Surgery of the
Congenital Foot)
l. Lowman procedure: For flatfoot, a rerouting of the a medial band of the
tibialis anterior tendon under. the navicular and sutured to the spring ligament
and transfer of a section of tendo Achilles (see chapter 21)
m. Heyman procedure: A panmetatarsal suspension for equinus foot via
suturing the EDL to their respective metatarsal heads (see chapter 21, Surgery
of the Congenital Foot)
n. Flexor digitorum longus transfer: Transferring the FDL to the proximal
phalanx of the involved digit will convert it into a strong plantarflexor of the
MPJ
o. Peroneal anastomosis: Involves securing the peroneus longus to the
peroneus brevis at the level of the midcalf (for pes cavus deformity to decrease
the plantarflexory force on the 1st ray and to increase the eversion force to the
foot)
p. Joplin sling procedure: To narrow the forefoot (used with children where
you do not want to do an osseous procedure). It is done via cutting the EDL
tendon 5 and passing it through and underneath the 5th MPJ joint capsule to
the abductor hallucis and back around and over the EHL suturing it to the 1st
MPJ joint capsule. The EDL tendon 4 is sutured to stump of the EDL 5. The
adductor is transected.
2. Healing for tendon transfers:
a. Casting for 4 weeks (foot in neutral position and at right angles to the leg)
b. Early passive ROM at about 3 weeks by bivalving the cast
c. Later, progressive weight-bearing excercise (isometrics)

Tendon Grafts
1. Donor tendons: Are usually from the plantaris, peroneus tertius, strips of
the Achilles, and slips of the EDL or EDB

2. Carbon Implants: The carbon acts as a scaffold on which new tendon can
develop, which makes it appropriate for filling large gaps as can be present in
the Achilles tendon (experimental as of now)

3. Silastic sheets: Used to protect a tendon anastomosis in one study

4. Silicone rod Implant: Used in staged gliding tendon transplant in patients


where the gliding bed has been damaged. This creates a pseudosheath for
delayed tendon grafting
5. Tendon xenografts: Bovine grafts are experimental at this point

6. Dacron mesh (Dacron Cooley graft): The dacron vascular graft is split to
provide a band of material of the desired length that can be woven through or
around a ruptured Achilles tendon as a lattice for further healing, In the same
manner as the plantaris tendon is used

Posterior Tibial Tendon Rupture


1. Anatomic considerations:
a. Deep posterior compartment muscle
b. Originates from the tibia, fibula and interosseous membrane
c. Extrinsic insertions into all bones of the midfoot except the talus, 1st and 5th
metatarsal
d. Passes retromalleolarly with the flexor retinaculum and functions as a two
pulley system (medial malleolus and navicular) providing a mechanical
advantage to the tendon

2. Functional considerations:
a. Open kinetic chain:
i. Supination (plantarflexion-,adduction-inversion)
b. Closed kinetic chain:
i. Deceleration of STJ pronation
ii. Acceleration of STJ and oblique MTJ supination in midstance phase of gait
iii. Rigid lever for gastro-soleus function

3. Etiology:
a. Traumatic forces and injuries
b. Progressive degeneration due to excessive demand (severe forefoot varus,
equinus, obesity)
c. Severe degeneration secondary to systemic disease (RA, mixed connective
disease, DM, etc.)
d. Neoplasms

4. Subjective findings:
a. Medial arch and/or ankle pain
b. Diffuse swelling and tenderness along the course of the TP tendon
c. Symptoms aggravated by proloned weightbearing and ambulation
d. May be more painful on initial arising in the AM (post-static dyskinesia)
e. Progressive flatfoot deformity
f. Sedentary/decreased activity

5. Clinical findings:
a. Edema and increased warmth of the medial aspect of the foot and ankle
b. Palpable tenderness along the course of the tibialis posterior tendon
c. Tenosynovitis may be present
d. Collapse of the medial arch
e. Palpable defect with complete ruptures
f. Increased heel valgus and midfoot abduction
g. Decreased muscle strength with guarding
h. Positive single heel rise test
i. Apropulsive/antalgic gait without resupination
j. Flexible to rigid depending upon the duration

6. Radiographic findings:
a. DP view:
i. Increased T-C angle (angle of Kite)
ii. Increased calcaneocuboid angle (cuboid abduction angle)
iii. Degenerative arthritic changes
b. Lateral view:
i. Decreased calcaneal inclination angle (can be normal)
ii. Increased T-C angle
iii. Increased talar declination angle
iv. Significant medial column faulting
v. Forefoot supinatus
vi. Degenerative arthritic changes
c. Special studies:
i. MRI: T1-weighted images provide images about the tendon itself, T2
weighted images are useful to highlight fluid within the tendon sheath or
adjacent edema

NOTE* MRI is most revealing. Three patterns of rupture have been reported:
a. Type 1: Intrasubstance tears noted on MRI with longitudinal splits and
hypertrophy of the tendon (increased signal on T1)
b. Type 2: Progression of intrasubstance tears noted on MRI as decreased girth
and attenuation of the tendon
c. Type 3: Complete rupture (noted as discontinuity of the tendon on MRI)

ii. CT
iii. Tenogram

7. Conservative treatment: Not usually helpful


a. NWB cast 4-6 weeks
b. Shoe modifications/orthotic devices
c. NSAIDS

8. Surgical treatment: Depends upon the time since the rupture, degenerative
changes taken place, rigidity of the deformity, and expected functional demands
a. Soft tissue procedures:
i. Early tendon repair:
 Excision of all scar tissue
 Excision of inflamed synovium
 Z-plasty shortening repair technique or transfer of tibialis anterior
 Primary reattachment to the navicular tuberosity
ii. Delayed primary repair with tendon free graft and desmoplasty
iii. Delayed primary repair with tendon transfer and desmoplasty
iv. Evans type procedure

NOTE* The indications for primary soft tissue repair alone are limited

b. Osseous procedures:
i. Isolated STJ arthrodesis
ii. Evans calcaneal osteotomy
iii. Talonavicular arthrodesis
iv. Combinations of above
v. Triple arthrodesis
vi. Ankle arthrodesis
vii. Pantalar arthrodesis
viii. Talonavicular arthrodesis with lateral column lengthening
c. Ancillary procedures:
i. TAL
ii. Gastrocnemius recession
iii. Medial column suspension procedures iv. Bone grafting
v. Subtalar joint arthroereisis

Posterior Tibial Tendon Dysfunction (acquired adult flatfoot


syndrome)
1. Etiology:
a. Tenosynovitis of tendon
b. Shallow or absence of malleolar groove
c. Attenuation of tendon
d. Rupture

2. Differential diagnosis:
a. Residual calcaneal valgus
b. Torsional abnormalities
c. Limb length discrepancy
d. Post-traumatic arthritis
e. Charcot arthropathy
f. Lisfranc dislocation

3. Signs and symptoms:


a. Pain
b. Edema
c. Abducted forefoot
d. Apropulsive gait
e. Loss of inversion power
f. Progressive flatfoot
g. Antalgic gait
h. Difficulty on toe raising
i. Heel not inverting with standing on toes

4. Diagnostic studies:
a. Tenogram
b. CT
c. MRI

5. Treatment (conservative)
a. BK cast immobilization in equinovarus x 4 weeks
b. Orthoses
c. NSAIDS

6. Treatment (Surgery):
a. Tendon repair
b. FHL tendon transposition
c. Secondary stabilization:
i. Medial column fusion
ii. Modified Young procedure
iii. STJ arthroereisis
iv. Evans calcaneal osteotomy
v. Triple arthrodesis

NOTE* Posterior tibial inflammation can be divided into peritendonitis, chronic


tenosynovitis, and stenosing tenosynovitis.
a. Peritendonitis: elicits pain at the musculotendinous junction, and is
consistant with an overuse syndrome. Treatment is physical therapy and
orthoses
b. Chronic tenosynovitis: elicits pain around the tendon between the tip of the
malleoli and the navicular (seen with rheumatic disease patients), requiring
orthoses and steroid injections
c. Stenosing tenosynovitis: elicits pain around the malleoli, and requires
surgical intervention

Peroneal Tendon Pathology


1. Types of pathology:
a. Dislocation:
i. Etiology:
 Eversion/dorsiflexion trauma
 Congenital absence of groove in the lateral malleolus
 Direct blow to the lateral ankle with the ankle inverted
ii. Signs and symptoms:
 Ankle edema
 Tendonitis
 Pain
 “Clicking” sound
 Avulsion flake from the fibula noted on x-ray
i. Treatment:
 Strapping (acute and chronic cases)
 Cast immobilization 3--6 weeks (acute)
 Surgical repair (acute and chronic) followed by BK cast x 4 weeks and
physical therapy

b. Stenosing Tenosynovitis:
i. Etiology:
 Direct trauma
 Lowgrade/chronic trauma
 Enlarged peroneal tubercle
 Calcaneal fracture
 Arthritis
ii. Signs and symptoms:
 Pain
 Trigger point pain
 Thickened tendon sheath
 Pain with ankle inversion
 Chronic edema
iii.Diagnostic studies:
 X-ray (calcaneal axial view)
 CT
 MRI
 Peroneal tenogram
iv. Treatment:
 Surgical repair of osseous pathology
 Surgical repair of the tendon sheath
 Iontophoresis
 Physical therapy

c. Tendon rupture:
i. Etiology:
 Laceration
Chronic degeneration
ii. Signs and symptoms
 Pain
 Edema
 Loss of eversion strength
 Inability to plantarflex the 1st ray
 Increased soft tissue mass
iii. Diagnostic studies:
 Peroneal tenogram
 MRI
 CT
iv. Treatment:
 Cast x 6 weeks
 Surgical repair (either primary repair or secondary with a graft)

Achilles Tendon Rupture


In most cases rupture results in longitudinal tearing of the tendon tissue into
irregular strips either at the musculotendonous junction (younger patient) or at
the point of insertion into the calcaneus (middle-aged people), the 2 most
common sites of rupture. The areas most susceptible to rupture are areas of
decreased circulation, myotendonous junctions, and the area 4-6 cm proximal
to the tendo Achilles insertion

1. Etiology:
a. Direct blow
b. Laceration
c. Abnormal muscle pull

2. Clinical diagnosis:
a. Pain at the site
b. Palpable tendon gap
c. Increased soft tissue mass
d. Loss of plantarflexory strength
e. Inability to walk on toes
f. Doherty-Thompson Test (+) (or just Thompson Test): The patient attempts to
plantarflex the foot while the calf is being squeezed. The inability to perform
this plantarflexion is a strong indication of Achilles tendon rupture
NOTE* Plantaris rupture often mimics tendo Achilles rupture but with this the
Thompson test is normal, and the pain is usually located along the course of
the ruptured plantaris tendon

3. Radiographic findings:
a. Obliteration of Kager's triangle
b. Increased soft tissue density
c. Toyger's angle (130-150°)
d. CT scan
e. MRI

4. Treatment: Partial rupture


a. BK cast x 3-4 weeks in plantarflexion
b. Followed by another BK cast with less plantarflexion x 4 weeks

5. Treatment : Complete rupture (24 hours-5 days)


a. Full equinus BK cast x 3 weeks, followed by
b. Gravity equinus BK cast x 3 weeks, followed by
c. Heel lifts

6. Treatment: Complete rupture (5 days or longer)


a. Surgical repair
b. BK NWB cast x 3 weeks, followed by
c. BK weight-bearing cast x 3 weeks, followed by
d. Heel lifts (19 mm to 13 mm to 6 mm)

NOTE* If a diagnosis of a distal rupture is made within 10 days, the Lynn


procedure is ideal.
Lynn procedure: A 7 inch medial/longitudinal incision parallel to the medial
border of the TA. The paratenon is opened in the midline and with the foot held
in 20° plantarflexion, and without excising the irregular ends, the TA is sutured
using an absorbable suture. If the plantaris is intact, its insertion at the
calcaneus is divided and the tendon is fanned out to form a membrane, which
is then placed over the TA repair and sutured into place (covering the TA 1 inch
above and below the repair). The TA paratenon and skin are closed. Cast
applied

NOTE* If a diagnosis of a proximal rupture is made within 10 days, a


McLaughlin procedure is preferred.
McLaughlin's procedure: A midline incision curving laterally is made. The
frayed tendon edges are trimmed back to healthy tissue. A drill hole is made
medially to laterally through the calcaneus, and a stab wound is made at its
point of emergence. A long screw is passed through the drill hole in the
calcaneus. A wire suture is inserted into the proximal tendon fragment, which
is then pulled into position by the 2 ends of the wire suture which are fastened
to the projecting ends of the screw. With retraction thus counteracted, the
trimmed tendon ends are sutured together. The superfluous portion of the
screw is cut free and removed. A twisted wire with a split lead shot (for
palpable localization of the mattress suture) is attached to the proximal portion
of the wire suture. Cast applied.

NOTE* The Bosworth procedure or the Lindholm procedure is used for late
repair of a rupture.
Bosworth procedure: Ruptured tendon exposed through a posterior
longitudinal midline incision from the calcaneus to the proximal 1/3 of the calf.
Excision of scar tissue at the ruptured ends. Free up from the medial raphe of
the gastrocnemius a strip of tendon 1/2 inch wide and 7 inches long, leaving
this strip attached just proximal to the rupture site. The strip is turned down
and passed transversely through the proximal tendon and then passed
transversely through the distal tendon, and then passing the tendon through
the distal end from anterior to posterior, while holding the knee at 90° and the
ankle in plantarflexion. Once again the strip is brought proximally and passed
through transversely and sutured onto itself. Cast applied
Lindholm procedure: A posterior curvilinear incision is made from the midcalf
to the calcaneus. The rupture is exposed, the ragged ends are debrided and
apposed with a box-type mattress of heavy silk or other non-absorbable suture.
From the proximal tendon and gastrocnemius aponeurosis, 2 flaps are
fashioned, each approx. 1 cm wide and 7-8 cm long. These flaps are left
attached at a point 3 cm proximal to the site of rupture,- and each flap is
twisted 1800 on itself so that its smooth external surface lies next to the
subcutaneous tissues as it is turned distally over the rupture. Each flap is
sutured to the distal stump of the tendon and to the other flap, completely
covering the site of the rupture. Wound closed. Cast applied.
5. General surgical principles:
a. Functional length restoration
b. Approximation of clean ends
c. Avoid the sural nerve
d. Preserve the tendon sheath
e. Evacuate the hematoma
f. Use proper anchoring sutures for the tendon
g. Tendon graft as necessary

6. Complications:
a. Nonoperative
i. Occurs from long term cast immobilization in equinus (needs aggressive
isokinetic rehabilitation). At the 10-15 week mark atrophy of the triceps occurs
b. Operative:
i. Intratendinous hemorrhage and irreparable damage to the paratenon
ii. Every attempt must be made to cover the newly repaired tendon with the
paratenon complex, because if not, this will become immobile and nongliding
iii. Rerupture
iv. Infection, wound dehiscence, sinus tarsitis, and STJ damage

Lateral Ankle Stabilization Procedures (tendon


transfers)

Single ligament rupture:


Watson-Jones*: This uses the peroneus brevis, which passes through the
fibula from posterior to anterior, through the neck of the talus from plantar
to dorsal, back through the fibula, from anterior to posterior, and sutured
back onto itself.
Lee Procedure (modified Watson-Jones)*: This uses the peroneus brevis
tendon, which is then passed through the fibula, from posterior to anterior,
and then sutured back onto itself.
Evans*: This utilizes the peroneus brevis through an oblique hole through
the fibula sutured back onto the belly of the peroneus brevis. Storren
Nilsonne
Pouzet
Haig
Castaing and Meunier
Dockery and Suppan

Double ligament rupture:


Elmslie*: Originally described as using the fascia lata and passed through a
drill hole in the lower aspect of the fibula, through the calcaneus, back
through the same drill hole, and tied onto itself, after passing through the
neck of the talus.
Chrisman and Snook*: This uses the split peroneus brevis, which is passed
through the fibula from anterior to posterior through a flap in the calcaneus,
and is then sutured back to the peroneus brevis tendon.
Stroren Hambly
Winfield
Gschwend-Francillon

Triple ligament rupture:


Spotoff
Rosendahl and Jansen
NOTE* In the case of lateral instability, both the Watson-Jones and the
Evans procedures are utilized.
a. The Watson-Jones restores the function of the calcaneofibular and
talofibular ligaments by rerouting the peroneus brevis tendon. The
chief drawback of this procedure is that it involves drilling a hole
through the neck of the talus (difficult to accurately accomplish). A
second problem can arise with this technique if the peroneus brevis is
too short to be threaded through the tunnels fashioned to receive it.
b. The Evans technique, which was designed to obviate the potential
difficulties of the Watson-Jones, has the disadvantage of involving
reconstruction of only the calcaneofibular ligament.
c. The Chrisman and Snook procedure was designed to repair the
anterior talofibular and inferior calcaneofibular ligaments, with
preservation of the peroneus brevis tendon

Postoperative Care and Training Following


Tendon Transfer
1. Age of the patient at the time of the transfer:
a. Should be old enough to cooperate in training (>4 years old)
b. Earlier transfer is indicated when delay would result in structural deformity

2. Support In overcorrected position:


a. Until full function is restored and no tendency for reccurrence
b. Bivalved cast will help hold tendon in relaxed position during this period

3. Preoperative training to localize contracture In muscle to be transferred

4. Instruct patient to contract transferred muscle:


a. Voluntary contracture through the original arc while guiding the part in the
direction provided by the transfer
b. Initially palpate the belly of the ms. and tendon to ensure proper contraction
c. Initially excercises are performed in the bivalved cast
d. Mild gentle tension on the transferred tendon may be used to assist patient
in "finding" the transfer
e. Electrical stimulation may also be used to assist patient in "finding" transfer

5. Establish motion in the new function provided

6. Development of motor strength:


a. Once motor strength becomes fair, the bivalve cast is gradually discontinued
during the day
b. Controlled activities are permitted to develop function
c. Resistance excercises are begun to develop strength when a normal ROM and
fair strength are established
d. Important to excercise antagonistic muscles also

7. Incorporation of the transfer into the new functional pattern:


a. Action of the transfer may be good through full range and moderate
resistance and yet during walking, voluntary control is lost
b. Use of crutches during this period is helpful
c. Careful supervision is required
d. Walking periods are gradually increased until gait pattern becomes a
conditioned reflex

8. Use of bracing:
a. Should be judicious and for specific reasons
b. Standing and walking excercises must also be performed without a brace to
stimulate function in the transfer

9. Bivalved casts:
a. Prolonged use is very important
b. Continue until the muscle has developed full strength and balanced function
with no tendency for reccurrence of the original deformity

10. Triple arthrodesis:


a. If dynamic balance can be established prior to the development of structural
deformity, arthrodesis can be avoided
b. Perform the osseous correction/stabilization and then perform tendon
transfer and muscle reeducation after bone union has taken place

Tenosynovitis
An inflammation of the synovial lining of the tendon sheath
1. Etiology:
a. Acute infectious tenosynovitis: Caused by a pyogenic organism. The bacterial
invasion and the resultant purulent exudate can involve the entire length of the
tendon sheath. Treatment with antibiotics must be prompt, I and D may be
necessary when the purulent material organizes
b. Chronic infectious tenosynovitis: Caused by diseases such as syphilis and
TB. The synovial wall becomes thickened and there is a fibrinous exudate
which affects the peroneal and extensor tendons most frequently.
c. Acute simple synovitis: Results from overuse most commonly affecting the
EHL, TA, and tendo Achilles
d. Chronic simple tenosynovitis: Caused by continuous shoe friction on the
extensors or Achilles tendon
e. Stenosing tenosynovitis: Usually affects the anterior and posterior tibial,
EDL, and the peroneals below the lateral malleolus and in the inferior
retinaculum. Caused by friction with-in the "pulley system" of the ankle within
the fibrous sheath. In digits, "trigger toe" occurs.
f. Hemorrhagic tenosynovitis: Caused by trauma in which the epithelial lining of
the sheath is ruptured followed by hemorrhage and clot formation (excision of
the hematoma is recommended)
g. Paratendonitis: Results from excessive friction between the tendon and the
paratenon caused by overuse, crepitation can occur
h. Acute tenosynovitis caused by rheumatoid arthritis: Nodular masses can
form within the tendon sheath, which may be rheumatoid nodules.
Chapter 28: Lesser Metatarsal
Surgery
Anatomy (Metatarsals 2-3-4)
Differential Diagnosis of Metatarsalgia
Surgical Treatment of the IPK
Lesser Metatarsal Joint Replacement
Panmetatarsal Head Resection
Metatarsus Adductus
Freiberg's Disease
Tailor's Bunion
Splayfoot
Brachymetatarsia (Brachymetopody)
Skewfoot
LESSER METATARSAL SURGERY
The central 3 metatarsals are usually grouped together because they do not
have individual axes of motion.

Anatomy (Metatarsals 2-3-4)


1. The deep transverse metatarsal ligament attaches to the plantar pad and
head of the central metatarsals on both sides. This affords greater stability to
the metatarsals 2-4 rather than to metatarsals 1 & 5

2. The plantar plate attaches to the metatarsal heads and the extensor hood
runs from dorsal to plantar to join at the inferior junction of the hood, capsule,
and deep transmetatarsal ligament

3. Blood supply to a long bone is via 3 sources:


a. Nutrient
b. Metaphyseal: In the metaphyseal region, there is an additional advantage of
having metaphyseal vessels adding a vascular system to bone
c. Periosteal: overlap entirely with the nutrient artery and so in most places
there is at least two supplies
4. Surgical neck is distal to the anatomical neck, and the condyles are directly
plantar to the flare between the two

5. There is a normal declination of the metatarsal of approximately 15°

Differential Diagnosis of Metatasalgia,


1. Local factors:
a. Stress fractures
b. Neuroma or neuritis
c. Intermetatarsal bursitis
d. Freiberg's infraction
e. Biomechanical factors:
i. Abnormality of metatarsal parabola resulting in plantarflexion,
shortening or elevation of a metatarsal
ii. Pes planus or pes cavus
g. Tumors
h. Arthritis (local as well as systemic)
i. Sesamoiditis
j. Tendonitis

2. Referred or systemic etiology:


a. Compression neuropathy of spinal cord L5-S3
b. Compression neuropathy of tarsal tunnel
c. Peripheral neuropathy
d. Ischemia

Surgical Treatment of the IPK


One should try to biomechanically evaluate why a lesion is present so that the
chances for return are reduced after correction of both the actual lesion and the
underlying cause.
1. Etiology of plantar lesions (metatarsals 2-4):
a. Biomechanical forces: Equinus, rearfoot varus, FF varus/valgus, adductory
twist
b. Hammertoe syndrome: Causes a retrograde plantarflexory force on the
metatarsal head. As the MPJ dorsiflexes due to muscular imbalances around
that articulation, the dorsal sling mechanism causes the proximal phalanx to
dorsiflex and apply a downward vector to the metatarsal. As time progresses,
there is a soft tissue contracture of the area and the metatarsal is exposed to
abnormal stresses
c. Atrophy or displacement of the plantar fat pad
d. Long or short metatarsal (even at the same declination angle)
e. Sagittal misalignment (abnormal declination angle): Abnormal plantarflexed
position, abnormal adjacent metatarsal, hypermobility
f. Abnormal bone shape (prominent plantar condyle)

2. Etiology of plantar lesions (metatarsal 5):


a. Biomechanical: Rearfoot varus, rigid forefoot valgus, forefoot varus
b. Sagittal malalignment: Abnormal plantarflexed 5th metatarsal, plantarflexed
cuboid, dorsiflexed 4th metatarsal
c. Congenitally long 5th metatarsal or short 4th metatarsal
d. Abnormal bone shape or size (prominent plantar lateral condyle)
e. Fat pad atrophy

3. Differential diagnosis:
a. Verruca plantaris: pinpoint bleeding, usually not directly on weightbearing
area, fast development, skin lines surround the lesion
b. Inclusion cyst: history of trauma (foreign body, puncture)
c. Scar tissue (history of trauma)
d. Foreign body

4. Preoperative considerations:
a. Mark the lesion with a x-ray opaque marker
b. Take x-ray in the angle and base of gait in full weight-bearing
c. Evaluate the metatarsal parabola (141.5°)
d. Check the axial view to evaluate the condyles
e. Look at the morphology of the metatarsal head and the relative position of
the fat pad

5. Summary of procedures: Can be done at the head, shaft, or the. base of


the metatarsal.
It is a good idea to enucleate the lesion prior to, or at the time of surgery to
hasten recovery of the plantar skin.
a. Procedures at the neck or head:
i. Percutaneous metaphyseal osteotomy (PMO):
 Osteotomy at the metaphyseal region where the capital fragment is
dorsiflexed
 Done so that the capital fragment is forced to the appropriate level when
walking
 The deep transverse ligament helps hold the the head in the correct position,
not allowing it to dislocate dorsally
 The cut is dorsal-distal to plantar-proximal to avoid the condyles and lift the
entire distal segment (fixation at the proper level)
ii. Transverse osteotomy:
 Similar to the PMO but done visually
 May be modified to shorten a metatarsal or fixated to control position
iii. "V" osteotomy:
 Done at the anatomical neck
 Gives good transverse and frontal plane stability due to the "V" cut
 Cut must include the condyles
 Apex is distal so the head and the phalanx act as one unit, so the head is
not free to dislocate
 Ambulation is allowed to force the head into the appropriate position
 Must debride or excochleate the lesion preop to avoid forcing the head too
high
 May impact on the shaft to control position
iv. Dorsiflexory wedge osteotomy (DFWO): A tilt up osteotomy
•Done at the anatomical neck or at the base (1 cm from the metatarsal-
cuneiform joint)
 Apex is plantar and the base-is dorsal
 Shortens the metatarsal
 Must be proximal to the condyles at the neck
 Must fixate
 Ambulation in a Reese® shoe
v. Arcuate osteotomy:
 Bone at the neck or base
 Special blade is needed (1800 arc)
 Allows for transverse and sagittal motion
 Must be fixated
vi. McKeever peg-in-hole:
 Shortens the metatarsal significantly
 Technically difficult
vii. Cylindrical stepdown osteotomy:
 For long metatarsal
 A cylindrical segment of bone is removed to cause shortening of the
metatarsal that needs fixation and NWB
viii. Chevron:
 A double "V" osteotomy with section removed used to shorten the metatarsal
 Fixation must be used
 Cannot take too much bone due to the soft tissue attempting to maintain
length
 Cuts must be parallel and congruent
ix. Osteoclasis:
 Surgical fracture at the anatomical neck by forceps
 Semi-free floating head
 No heat from the power equipment, therefore, little bone necrosis
x. Metatarsal head resection and condylectomy:
 Helpful for subluxed and/or deformed joint
 Joint is basically removed
xi. Metatarsal head resection:
 For deformed or destroyed MPJ
 Shortens the ray
 Allows contracture of the toe but the pain from the lesion disappears
 Best done in the elderly
 Close and purse string the capsule
xii. Plantar condylectomy:
 Open the MPJ and elevate the metatarsal head to allow access to the plantar
condyles
 Condyles are resected and area rasped smooth
 Osteoarthritis can develop and joint limitus may develop as disruption of the
integrity of the MPJ is necessary
 No bone healing needed, therefore, early ambulation

b. Procedures at the shaft:


i. Giannestras step down osteotomy:
 For a long metatarsal
 Z" shortening of the metatarsal that needs fixation

c. Procedures at the base:


i. Cresentic: As described above
ii. DFWO: As described above

6. Complications of metatarsal osteotomies:


a. Transfer lesions develop (so try not to overcorrect by indiscriminate
elevation)
b. Dorsal bump develops from too much elevation without having
remodeled the head
c. Floating toe develops from destroying the internal cubic content of the
joint
d. Non-union occurs if ischemia is produced or no fixation or stabilization
of the osteotomy site exists
e. Flail toe from transection of the musculotendonous tissues surrounding
structures
f. Dislocation of the metatarsal head and deformed position of the head
or toe
g. Edema
h. Return of the original deformity due to not enough elevation of the
metatarsal segment

Lesser Metatarsal Joint Replacement


1. Indications:
a. Inflammatory arthritides: RA
b. Degenerative arthrosis secondary to:
i. Osteochondral fractures
ii. Osteochondritis dissecans
iii. Orthopedic deformity
iv. Joint subluxation
v. Malaligned fractures of the foot
vi. Trauma
vii. Previous surgery
viii. Congenital deformity
c. Congenital deformity- Brachymetatarsia
d. Flail toes
e. Floating toes
f. Revisional surgery

2. Types:
a. Swanson flexible hinge toe implant
b. Sgarlato double-stem cup implant (hinge avoided)
c. Swanson condylar implant

3. Surgical technique:
a. Lazy "S" Incision over the MPJ (less skin contracture)
b. Linear or "U" shaped capsulotomy
c. Preoperative soft tissue contractures eliminated via extensor/flexor
tenotomies and/or plantar plate/hood release
d. Bony resection (mostly metatarsal head)
e. Reaming the medullary canals (caution in the proximal phalanx)
f. Sgarlato recommends centralizing the flexor tendons via a drill hole in the
plantar portion of the phalangeal base and attaching the tendon by suture
g. Check fit with a sizer
h. Flush copiously
i. Wound closed in layers

4. Complications:
a. Implant instability: Pistoning can occur from removal of too much bone as
well as axial rotation of the implant
b. Implant failure: Mechanical stress can produce microfragmentation with
migration of the silicone particles into the lymphatic system. With this there
will be obvious loss of function and possible deformity
c. Foreign body reaction
d. Osteochondritis dissecans: From excessive stripping of the periosteum and
resultant avasular necrosis
e. Detritic synovitis reaction: The surgical area will become red and swollen
with a chronic low grade pain. Once infection is ruled out the patient can be
treated with NSAIDS or remove the implant device
f. Infection: Implant must be removed and not replaced for at least 6 months to
one year. If gram negative infection was present, implant should not be
replaced for longer period of time if at all
g. Pistoning of the implant into cancellous bone (if implant chosen is too small)
h. Chronic edema
i. Fracture of the base of the proximal phalanx

5. Contraindications:
a. Severe osteoporosis of the involved bones (seen with RA)
b. History of a prior joint infection within the last 6 months
c. History of allergic reaction to implant material
d. Medically compromised patient (diabetic neuropathy, Charcot joint)

Panmetatarsal Head Resection


This procedure can be gratifying but must be performed only when the proper
criteria are met.
1. Historical:
a. Hoffman (1911): Transverse plantar approach
b. McKeever (1952): Dorsal longitudinal approach
c. Clayton (1963): Transverse dorsal approach for metatarsal head and
phalangeal base resections

2. Preoperative signs:
a. IPK's under most metatarsal heads
b. Atrophy of plantar fat pad
c. Ability to palpate prominent metatarsal heads
d. Dorsally contracted toes at the MPJ's
e. Possible contracted toes and proximal interphalangeal joint or distal
interphalangeal joint with associated lesions
f. Range of motion at the MPJ may be limited or painful or may be absent
g. Range of motion of the MPJ may elicit crepitus
h. Signs of degenerative disease and deformity
i. Patient ambulates with an apropulsive type gait
j. Ulceration of sub-metatarsal head area

3. Preoperative symptoms:
a. Moderate to severe pain on the plantar aspect of the forefoot when the
patient ambulates with or without shoes
b. Painful multiple hyperkeratotic lesions
c. Painful plantar ulcerations under the metatarsal heads area
d. Patient may complain of painful dorsally contracted toes when wearing shoes
e. Patient complains of pain when most of the MPJ's are moved
f. Patient may complain that the toes cannot be straightened
g. History of metabolic disease (RA, psoriatic arthritis, etc.)

4. Preoperative x-ray evaluation:


a. Evidence of DJD
b. Dorsally contracted MPJ
c. Most of the MPJ's show evidence of DJD
d. Bone loss evident secondary to severe DJD
e. Proliferation of bone at the MPJ's
f. Loss of normal joint space
g. Cystic and erosive changes in the metatarsal heads
h. Generalized osteoporosis
i. Moderate to severe angulation deformity of the toes and metatarsals may be
present

5. Surgical procedure of choice:


a. 5 dorsal linear incisions: 3 dorsal linear incisions; or transverse incision
b. Maintain normal metatarsal parabola: The second is the longest, followed by
the 1st and third, followed by the 4th, and finally the 5th
c. The 1st metatarsal head is resected more medially than laterally
d. The lesser extensor tendons are usually tenotomized
e. Angulate the dorso-plantar cuts on all the metatarsal heads in order to
remove more bone plantarly than dorsally
g. Release the tourniquet prior to closing to prevent hematoma formation
h. K-wires can be used (helps eliminate the need for syndactylism)
i. Betadine soaked gauze can prevent postoperative edema and infection and
helps keep the toes in an aligned position

6. Advantages:
a. Eliminates painful MPJ's
b. Ability to ambulate without pain
c. Allows patient to wear regular shoes
d. Allows reduction of dorsally contracted toes in most cases
e. Elimination of plantar pressure points

7. Disadvantages:
a. Loss of propulsive gait
b. Flail toes postoperatively
c. Incidence of hematoma formation with resulting fibrosis
d. Destroys the function of the MTPJ's
e. Loss of digital stability

NOTE* if one excises a large amount of the metatarsal and one is already
dealing with short toes (especially the 5th), then syndactylism will aid in
achieving some stability of the area distally in the forefoot. This procedure can
be an adjunct to panmetatarsal head resection

Metatarsus Adductus
1. Clinical evaluation:
a. Adducted forefoot in the transverse plane with the apex of the deformity at
LisFranc's joint
b. Medial border concave with a deep vertical skin crease
c. Hallux widely separated from the 2nd toe d. The lesser digits will be adducted
at their bases
e. Occasionally the abductor hallucis may be palpably taut

2. Radiographic evaluation:
a. Increase in metatarsus adductus angle (greater than 200)
NOTE* Not always accurate as the lesser tarsal bones in the neonate are not
measurable as they are radiographically "silent", and in many cases the T-C
relationship is abnormal. Therefore It is best to use the calcanealsecond
metatarsal angle (normal parameters pending)

3. Indications for surgery


a. Failure to respond to conservative treatment
b. Residual deformity after treatment of talipes equinovarus
c. Newly diagnosed metatarsus adductus deformity

4. Considerations: (see section Pediatrics)


a. Age of patient
b. Osseous development
c. Severity of deformity
d. Presence of concomitant deformities
e. Extent of malfunction and disability

5. Soft Tissue Surgery: (current procedures will be discussed in detail)


a. Heyman, Herndon, and Strong:
i. Indications:
 for flexible met. adductus which is reducible on manipulation (stress x-ray)
 usually children less than 5 years old
 deformity present at Lisfranc's joint, without significant bowing present in
the proximal portion of the metatarsal bones themselves
ii. Procedure:
 2 or 3 longitudinal dorsal incisions or transverse incision
 release of the dorsal, interossei, and plantar ligaments of the
tarsometatarsal joints and intermetatarsal joints
 preserve the plantar-lateral ligaments, especially 5th metatarsocuboid
articulation and the peroneus brevis tendon
 manipulate the foot into abduction
 K-wire fixation of the first met-cuneiform joint and 5th met-cuboid joint
 release of the naviculocuneiform and intercuneiform joints is rarely needed
 consider abductor hallucis release or tenotomy in conjunction with HH&S
iii. Precautions:
 avoid damage to the 1st metatarsal epiphyseal growth plate (do not confuse
this with the met-cuneiform joint)
 be careful not to introduce iatrogenic dorsal dislocations at the met-
cuneiform joints
iv. Postop care:
 cast for 6-12 weeks
 manipulate the foot and recast every 3-4 weeks depending upon the severity
 monitor the foot carefully for the development of a flatfoot deformity
v. Complications:
 dorsal dislocation
 degenerative arthritis
 damage to the growth plates

b. Thompson procedure (modified):


i. Indications:
 congenital hallux varus primarily
 flexible met. adductus secondarily
 hyperactivity of the abductor hallucis ms.
ii. Procedure:
 medial longitudinal 1st MTPJ skin incision approach
 dissection to level of deep fascia over the abductor hallucis muscle
 transection of the abductor hallucis tendon with resection of a segment of
the tendon and portion of the distal muscle
 consider lesser MTPJ release medially if lesser digits are also adducted
 release of the medial head of the flexor hallucis brevis if adduction of the
hallux is still present
iii. Precautions:
 do not reduce varus of the hallux without ensuring correction of any
adduction deformity of the first metatarsal
 place the medial incision over" the 1st MTPJ strategically; if too superior or
inferior, may damage the medial neurovascular bundle
 avoid the procedure as a primary mode of correction for met. adductus
unless clinical findings and x-rays strongly support hyperactivity of the
abductor hallucis as the primary etiology
iv. Postoperative care:
 weightbearing in a surgical shoe for 3-6 weeks
 splinting of the hallux and the first ray
v. Complications:
 hallux abductovalgus
 hallux hammertoe (hallux malleus)

c. Johnson osteochondrotomy: cartilaginous procedure


i. Indications:
 met. adductus deformity in children between the ages of 5-8 years (can be
younger)
ii. Procedure:
 3 dorsolongitudinal incisions
 closing abductory base wedge osteotomy of the 1st metatarsal
 wedge resection of cartilage and bone from the bases of the lesser
metatarsals, distal to the proximal articular surface (base is lateral with the
apex medial)
 fixation of the osteotomies with stainless steel wire, K-wires, or staples
iii. Precautions:
 avoid damage to the epiphyseal growth plate of the 1st metatarsal
 overcorrection/undercorrection of individual ray segments
iv. Postoperative care:
 non-weightbearing with cast immobilization for 6-8 weeks
 serial x-rays to assess healing
6. Osseous Surgery:
a. Modified Berman-Gartland procedure:
i. Indications:
 met. adductus in the child older than 6-8 years
 residual deformity following treatment of talipes equinovarus
ii. Procedure:
 3 dorsolongitudinal incisions
 transverse or oblique-type closing abductory wedge osteotomy of the 1st
metatarsal
 similar type of osteotomies of the lesser metatarsals with the cortical hinge
medially
 fixation of osteotomies with SS wire, K -wires, staples, AO screws or
combinations
iii. Precautions:
 avoid damage to growth plates of 1st metatarsal
 meticulous subperiosteal dissection is critical to avoid heavy callus
formation and undesirable synostosis between adjacent metatarsals
 preservation of the medial cortical hinge is important to insure stability
 careful planning to avoid over/undercorrection
iv. Postoperative care:
 non-weightbearing cast immobilization 6-8 weeks
 convert the cast to posterior splint and start PT
 orthotics when patient resumes weightbearing
 serial x-rays to assess bone position and healing at 3 weeks , 6 weeks, 12
weeks, 24 weeks and 1 year
v. Complications:
 over/undercorrection
 delayed union/nonunion/pseudoarthrosis
 fracture of f cortical hinge
 damage to growth plate
 elevatus of metatarsals
 iatrogenically induced flatfoot deformity

b. Lepird procedure:
i. Indications:
 met. adductus in the child older than 6-8 years
 residual talipes equinovarus deformity
ii. Procedure:
 3 dorsolongitudinal incisions
 oblique closing-abductory wedge osteotomy (Juvara type) of the 1st
metatarsal with AO/ASIF screw fixation
 rotational osteotomy of each lesser metatarsal with AO/ASIF screw fixation
(2.7 mm cortical used mostly) perpendicular to the plane of the osteotomy
 an oblique closing wedge osteotomy may be used on the 5th metatarsal in
place of the rotational type (if preferred)
 rotational osteotomies are performed from dorsal-distal to plantar proximal
with temporary preservation of the cortical hinge (facilitates fixation). The
osteotomy is approximately 45° from the weightbearing surface. The precise
angle will depend on the declination of the metatarsal segment. As the
declination of the metatarsal increases, the osteotomy will be more parallel
to the weightbearing surface of the foot
 area of the cortical hinge preserved is most commonly proximal/plantar
 the screws are then removed and the osteotomy is completed
 the screws are reinserted, the distal fragments are rotated laterally, and the
screws are tightened
 the alignment of the foot is assessed; if realignment is necessary the
 screw(s) can be loosened and the bone adjusted
iii. Postoperative care:
 same as Berman-Gartland
iv. Complications:
 same as Berman-Gartland
 if the osteotomy is performed too vertically the rotation of the
 osteotomy will be around the longitudinal axis of the metatarsal bone itself,
resulting in inversion/eversion of the bone itself v. Advantages: this
procedure is amenable to rigid internal fixation and primary bone healing
 over/undercorrection can be corrected during surgery
 biplanar correction can be achieved
 eliminates pin tract infections

7. Ancillary Procedures:
a. Equinus Deformity:
i. TAL
ii. Gastrocnemius recession
b. Flatfoot Deformity:
i. STJ arthroereisis
ii. Evans calcaneal osteotomy
iii. Modified Young's tenosuspension/ Modified Kidner procedure
arthrodesis/ N-C arthrodesis

Freiberg's Disease
Also known as osteochondrosis of the metatarsal head or avascular (aseptic)
necrosis of the bone, most commonly affects the 2nd metatarsal
1. Etiology:
a. Trauma (or trauma followed by fracture)
b. Ischemia
c. Prominent plantar metatarsal head with excessive loading with a compromise
to the circulation to the subchondral bone
d. Often appears after age 13, affecting women 3 times more frequently than
men

2. Signs and symptoms:


a. Pain in the MPJ (usually dorsally), either sharp, dull, or aching In character
b. Edema with increased activity
c. Limitation of motion of the involved digit and MPJ
d. Palpable irregularities may be present dorsally
e. Distal distraction of the toe will cause pain
f. Adjacent MPJ hyperkeratoses may be present as the Involved metatarsal
bears less weight

3. X-ray evaluation:
a. The initial findings include a joint space widening 3-6 weeks after the onset
of symptoms
b. This is followed by increased density of subchondral bone
c. As the disease progresses, a zone of rarefaction develops surrounded by a
sclerotic rim
d. With time, the epiphyseal bone weakens and collapses with the formation of
spicules and loose bodies
e. Flattening of metatarsal head with osteophytic lipping
f. Joint narrowing
g. Peripheral soft tissue swelling
h. Bone margins are sclerotic

4. Treatment:
a. Directed toward preventing further damage and displacement of the MPJ
(casting and cortisone shots followed by orthoses))
b. Later stages:
i. Implant arthroplasty: If symptoms are due to joint arthritis
ii. Metatarsal head remodeling (must preserve the alignment of the toeuse splint
3 months postoperatively)
iii. Bone grafts (Smillie): To restore the contour of the metatarsal head by
inserting a cancellous graft (good for stage 1-3)
iv. Rotational osteotomies (Gauthier and Elbaz): Rotates the lower aspect of the
metatarsal head dorsally after a section of damaged cartilage has been excised.
This allows the plantar cartilage to articulate with the proximal phalanx

NOTE* Dr. Freiberg's only surgical treatment involved removing the loose
bodies
5. Classification (by Smillie into 5 stages):

a. Stage 1: Fissure fracture


b. Stage 2: Absorption of bone. Central aspect of bone is sinking into the
metatarsal head
c. Stage 3: Further. progression with projections remaining on either side of the
metatarsal head. The plantar articular cartilage remains intact
d. Stage 4: Fractures and loose bodies may occur. Plantar cartilage no longer
intact
e. Stage 5: Flattening of the metatarsal head.

Tailor's Bunion
1. Etiology:
a. Any uncompensated varus position of the forefoot or rearfoot in a fully
pronated foot
b. A congenital plantarflexed 5th ray deformity
c. A congenital dorsiflexed 5th ray deformity
d. Idiopathic
e. Lateral deviation or wide 5th metatatarsal head
f. Combined influences

2. Clinical findings:
a. Prominence over the 5th metatarsal head with pain
b. Hyperkeratosis and erythema over the 5th metatarsal head area
c. 5th toe assumes a varus or adducto varus attitude

NOTE* Must determine if a splayfoot deformity is present. Evaluate on


weight-bearing and x-ray. Radiographically, splayfoot deformity is
characterized by an IM angle between the 1st and 2nd metatarsal of greater
than 12° and between the 4th and 5th metatarsals of greater than 8°. In
association with varus of the 1st metatarsal, the slant of the distal articular
surface of the medial cuneiform is more than 105°

3. Radiological findings:-6 types according to the findings


a. Rotation of the lateral plantar tubercle into a lateral position
b. Increased IM angle (normal 6.47°): People with tailor's bunion have an IM of
8.71 or greater (Fallat and Buckholz)
c. Increased lateral deviation angle (normal 2.64°) People with tailor's bunion
have a lateral deviation angle of 8.05° (Fallat and Buckholz)
d. A large "dumbell-shaped" 5th metatarsal head
e. Arthritic changes resulting in exostosis formation at the 5th MPJ
f. Any combination of the above conditions, the 1st three being most common

4. Surgical management:
1. Hohmann osteotomy: Single transverse osteotomy at the level of the
metatarsal neck with medial displacement of the capitol fragment
b. Oblique osteotomy from distal lateral to proximal medial with displacement
of the capital fragment proximally and medially (reverse Wilson procedure)
c. Modified Mitchell: Step down osteotomy
d. Austin type osteotomy: 2 mm of medial transposition
e. Mercado osteotomy: Medially based closing wedge osteotomy at the
metatarsal neck
f. Yancy osteotomy: Midshaft medially based closing wedge osteotomy
g. Gerbert et al osteotomy: Proximal diaphyseal closing wedge osteotomy
h. Buchbinder osteotomy: DRATO
i. McKeever: Partial metatarsal head resection
j. Kelikian: Partial metatarsal head resection with syndactylization of the 4th
and 5th toes
k. Distal oblique osteotomy with intramedullary K-wire fixation
NOTE* Excessive 5th metatarsal head resection results in laxity of the internal
cubic content of the joint leading to further varus or adducto varus
malalignment of the 5th toe, and more retrograde pressure on the 5th
metatarsal head

Splayfoot
As this deformity consists of high IM angles for the 1 st and 2nd , and 4th and
5th, surgical repair is focused on reducing the IM angles. This is accomplished
via a closing base wedge osteotomy of the 1 st metatarsal with AO fixation, and
distal oblique osteotomy of the 5th metatarsal with K-wire fixation.

Brachymetatarsia (Brachymetapody)
1. Etiology:
a. Congenital: Premature idiopathic closure of the distal epiphyseal growth
plate
NOTE* The congenital pattern has also been associated with neonatal
hyperthyroidism, pseudohypoparathyroidism, pseudo-
pseudohypoparathyroidism, malignancy, Down's syndrome, Albright's
syndrome, myositis ossificans, Turner's syndrome, sickle-cell anemia, Still's
disease, and enchondromatosis

b. Traumatic
c. Infectious

2. Clinical presentation:
a. Symptoms usually appear in adolescence when full growth discrepancy is
most apparent
b. In the younger patient the only complaint will be the appearance of a
shortened or "floating" toe
c. The adjacent toes underlap the involved toe
d. Calluses under the adjacent metatarsal heads with metatarsalgia
e. The amount of associated disability typically depends upon the amount of
weight that Is transferred to the adjacent metatarsal heads
f. A deep sulcus is present underneath the short metatarsal

NOTE* The iatrogenic and traumatically induced types of brachymetatarsia


are usually more acute and severe in their presentation

3. Radiological findings:
a. Short, underdeveloped metatarsal with deficient bone content
b. Osteoporosis of the metatarsal head

4. Operative planning:
a. Consider the amount of length needed to restore the normal metatarsal
parabola
b. Must consider whether to lengthen and plantarflex the involved metatarsal or
shorten and dorsiflex the adjacent metatarsals
c. Soft tissue mobility and neurovascular status of the involved ray
d. Use of a bone graft either autogenous or allogeneic

5. Procedure:
a. Bone lengthening procedure (frontal plane "Z" osteotomy)
b. Insertion of corticocancellous bone graft
c. Extensor tenotomy
d. " V" to "Y" skin plasty
e. BK NWB cast until osseous healing

6. Complications:
a. Risk of neurovascular compromise
b. Non-union
c. Absorption or collapse of the graft
d. Painful pseudoarthrosis
e. Painful limitation of motion at the joint

Skewfoot
1. Description: A metatarsus adductus forefoot-type with a pathological
rearfoot valgus component

2. Etiology:
a. After serial casting for metatarsus adductus in which the rearfoot was in a
pronated position
b. Untreated metatarsus adductus which has compensated by excessive
subtalar joint pronation
c. Congenital metatarsus adductus with associated calcaneovalgus

3. Clinical evaluation:
a. The metatarsals are angulated medially
b. The base of the 5th metatarsal is prominent
c. A large space is noted between the hallux and 2nd toe
d. A metatarsus varus may be present
e. The digits are abducted in stance
f. Talar bulging (ptosis) on weight-bearing with low medial arch
g. Abducted midfoot position with internal rotation of the malleoli
h. Rearfoot equinus may be present

4. Types:
a. Simple skewfoot: An adducted forefoot with an abnormally pronated rearfoot
b. Complex skewfoot: An adducted forefoot, abducted midfoot, and abnormally
pronated rearfoot

5. Radiological evaluation:
a. Increased metatarsus adductus angle (MA angle greater than 21 °)
b. Increased cuboid abduction angle (greater than 5°)
6. Indications for surgery:
a. Too old for correction by conservative means
b. Deformity is increasing despite conservative treatment
c. Deformity is obviously not manageable by conservative means
d. Deformity is beginning to cause secondary deformities
e. Patient is experiencing painful compensatory symptoms
f. Patient is accommodating to life style because of related symptoms
g. Increased difficulty with standard shoegear

7. Surgical repair: As this Is a complex deformity, multiple procedures


must be employed as necessary
a. Equinus correction: Gastrocnemius recession or TAL as indicated
b. Pes valgoplanus correction: Evans opening calcaneal osteotomy and medial
arch tenosuspension. The Evans osteotomy lengthens the lateral column and
therefore, realigns the midtarsal joint

NOTE* This procedure can unmask a previously unappreciated metatarsus


adductus

c. Metatarsus adductus correction: Modified Berman-Gartland or Lepird


d. Subtalar joint instability and bony adaptation: STJ arthrodesis
Chapter 29: Ankle Conditions
Differential Diagnosis of Chronic Ankle Pain Tarsal
Tunnel Syndrome Sinus Tarsi Syndrome
Peroneal Subluxation
Ankle Arthrodesis
Lateral Ankle Instability
Chronic Lateral Ankle Instability Chronic Medial Ankle
Instability Ankle Equinus
Malunion and Nonunion of the Malleoli
ANKLE CONDITIONS AND
TREATMENT
Differential Diagnosis of Chronic Ankle Pain
1. Lateral:
a. Peroneal tendon and nerve:
i. Peroneal tendon stenosing tenosynovitis
ii. Peroneal dislocation
iii. Peroneal neuropathy b. Sural nerve injury

2. Dorsolateral
a. Sinus tarsi syndrome
b. Extensor digitorum brevis (myositis, avulsion)
c. Calcaneus anterior process fracture

3. Intracapsular:
a. Talar posterior (lateral) process fracture
b. Talar dome (osteochondral) defects
c. Intra-articular fibrous bands, hypertrophic synovitis, meniscoid bodies

4. Medial:
a. Tarsal tunnel syndrome
b. Posterior tibial tendon tendonitis, rupture
c. Flexor hallucis longus tendonitis
d. Os trigonum fracture

5. Other:
a. Ligamentous injury (instability) b. Traumatic DJD of STJ or ankle

Tarsal Tunnel Syndrome


This is an entrapment or compression neuropathy of the posterior tibial nerve
or one of its three branches, the medial and lateral plantar nerves and/or
medial calcaneal nerve.
1. Anatomy: Nerve entrapment occurs either In the porta pedis or lacinate
ligament
a. The flexor retinaculum (lacinate ligament) extends from the medial
malleolus to the medial process of the calcaneal tuberosity and the plantar
aponeurosis. The deep fibrous septa form four compartments, and converts
bony grooves into canals from anterior-medial to posterior lateral: #1 contains
tibialis posterior tendon (most superficial), #2 FDL tendon, #3 posterior tibial
nerve artery and vein, and #4 FHL tendon. These compartments are unyielding
spaces.
b. The porta pedis is a canal created by the abductor hallucis muscle belly
through which the medial and lateral plantar nerves pass.
c. Division of the posterior tibial nerve into its 3 terminal branches may occur
proximal to the lacinate ligament, which is most common; within the lacinate
ligament, as described in most texts; or distal to the lacinate ligament, which is
rare.
d. The medial calcaneal nerve is entirely sensory, and innervates the medial
and plantar aspect of the heel. It may arise from either the posterior tibial or
lateral plantar nerve.
e. The medial plantar nerve gives sensory innervation to the plantar aspect of
the hallux, second and third toes, medial half of the fourth toe, and the medial
half of the plantar aspect of the foot. It gives motor innervation to the abductor
hallucis, flexor digitorum brevis, flexor hallucis brevis, and the first lumbricalis.
f. The lateral plantar nerve gives sensory innervation to the plantar lateral half
of the fourth toe, plantar aspect of the fifth toe, and plantar lateral aspect of the
foot. Initially it sends motor fibers to the quadratus plantae and abductor digiti
quinti before dividing in a superficial and deep branch. Superficial branch
supplies motor innervation to the flexor digiti quinti brevis and the dorsal and
plantar interossei of the fourth intermetatarsal space. The deep branch supplies
the remaining intrinsic muscles of the foot.

2. Pathology: Compression of the nerve initially causes only sensory


involvement with possibly partial involvement of motor fibers. Continuation of
the irritation, ischemia, and compression may lead to secondary hyperactivity
of the autonomic nervous system, (manifested by coldness and numbness) from
the altered sympathetic activity. Eventual structural changes in the nerve
result in the development of muscle wasting, paresis, and objective sensory
loss.

NOTE* Reflexes are unaffected

3. Etiology: In the many cases no etiology can be found at the time of surgical
decompression.
a. Dilated posterior tibial veins: can also cause severe night discomfort.
b. Trauma: Fracture, dislocation, sprain, post-traumatic edema and fibrosis.
c. Systemic disease: Gouty arthritis with urate deposits, rheumatoid arthritis,
diabetes mellitus, and myxedema.
d. Space occupying lesions: Ganglions, neurofibromas, schwannomas, synovial
cysts, etc.
e. Hypertrophy of abductor hallucis muscle belly.
f. Biomechanical: excessive pronation

4. Clinical Symptoms: Symptoms can be either distal to the metatarsal area,


or the medial and lateral heel depending on the branch involved.
a. Early:
i. Intermittent burning pain, numbness and paresthesias over the medial side
of the heel, the toes, and the plantar aspect of the foot.
b. Late:
i. A paresis that will develop into paralysis of the pedal intrinsic muscles.
ii. Proximal radiations of pain may develop in the posterior calf.
iii. Pain that is proportional to the amount of activity during the day.
iv. May develop some sensory loss

5. Diagnosis: Not always easy, as the signs are not always definitive
a. History of paresthesias
b. History of trauma
c. History of systemic disease
d. Hoffman-Tinel's sign: A tingling in region of the distribution of the involved
nerve with light percussion, results in paresthesias distal to the site of
percussion.
e. Valleix Phenomena: A nerve trunk tenderness above and below the point of
compression, with paresthesias proximal and distal to the point of percussion.
f. Turk's test: Application of a venous tourniquet to the lower extremity will
elicit positive symptoms on the affected side, by producing a venous occlusion.
g. Forced eversion of the foot.
h. Positive radiographic evidence of previous injury i. Positive lab studies for
any specific disease
j. EMG's and nerve conduction studies are only useful for late stage disease.

Note* EMG may show fibrillation potentials which indicate denervation of


muscle. Nerve conduction studies may reveal an increased distal latency.

Note* Placement of nerve conduction study surface electrodes are as follows:


1. Proximal stimulation point: distal aspect of popliteal fossa
2. Distal stimulation point: behind the medial malleolus
3. Recording electrode (for conduction of the medial plantar nerve) through the
abductor hallucis ms. belly.
4. Recording electrode (for the lateral plantar nerve) through the abductor
digiti quinti muscle belly.
6. Treatment: Conservative
a. Local blocks: Posterior tibial nerve blocks with steroids
b. Unna boot: can be combined with nerve blocks
c. Support hose: for varicosities
d. Functional orthoses

7. Treatment: Surgical Decompression (positive EMG's and nerve conduction


studies mandate surgical decompression). Involves the complete exploration of
the tarsal tunnel with release of the flexor retinaculum and its fibrous bands,
and resection and ligation of any dilated veins in the area.
The surgical technique is as follows:
a. A curvilinear incision is made posterior and inferior to the medial malleolus
by 1 cm.
b. The subcutaneous tissue Is Incised and the superficial- vessels are ligated as
necessary.
c. The neurovascular structures superior to the retinaculum are identified,
preserved, and retracted (especially the medial calcaneal branch).
d. The flexor retinaculum is incised and the posterior tibial nerve or its terminal
branches are identified and mobilized.
e. The nerve(s) is retracted with a penrose drain.
f. The nerve(s) is followed proximally, incising the flexor retinaculum as you go.
g. The nerve(s) is followed distally to the point where the medial and lateral
plantar nerves pass through the fibrous canals superior to the abductor
hallucis ms. belly.
h. The abductor hallucis ms. is examined for any abnormality, and any
hypertrophy is excised.
i. If there are any posterior tibial vein varicosities, they should be ligated.
j. The retinaculum is not reapproximated and no deep closure is done.
k. The superficial fascia is reapproximated and the skin reapproximated
i. Sterile compression dressing and a non-weight-bearing BK cast applied for 3
weeks.

8. Complications:
a. Recurrence: due to fibrosis
b. Severing the PT artery : if done then tie off and prepare patient for
microvascular repair later.
c. Severing a nerve
d. Tenosynovitis
e. Hematoma
f. Wound dehiscence

Sinus Tarsi Syndrome


1. Symptoms: A diffuse deep aching pain on the dorso-lateral aspect of the foot
over the sinus tarsi. Relief of the discomfort after an injection of anesthesia
deep into the sinus tarsi while maintaining superficial sensation is a diagnostic
feature.

2. Etiology:
a. Post traumatic:
i. Inversion ankle sprain
ii. Fibular fracture
iii. Calcaneal fracture
iv. Talar neck fracture

NOTE* Arthroscopic studies of the posterior facet showed an absence of the


normal synovial recesses in front of the talocalcaneal interosseous ligament.
This may indicate synovial hyperplasia, scarring, and/or synovitis

b. Biomechanical fault:
i. Pes piano valgus (stretching of the cervical ligament)
ii. Pes cavus
c. Systemic arthritic/metabolic:
i. RA
ii. Gout
iii. Seronegative arthropathies

3. Findings:
a. Subjective:
i. Diffuse pain on the lateral side of the foot
ii. Feeling of rearfoot instability especially on uneven terrain
iii. Dramatic relief of symptoms with an anesthetic block b. Objective:
i. Pain reproduced with direct pressure over the sinus tarsi
ii. Discomfort with forced inversion and plantarflexion
iii. X-rays normal
iv. Ankle joint is WNL
v. STJ motion painful but not limited
vi. MTJ motion mostly pain free
vii. Palpation of the intermediate dorsal cutaneous nerve and sural nerve
uneventful
viii. Palpation of the anterior talofibular and calcaneofibular ligaments is
uneventful
ix. Elicit pain by digital pressure on both sides of the sinus tarsi simultaneously
c. Diagnostic:
i. X-rays
ii. Diagnostic anesthetic block
iii. Arthrography of the posterior facet of the STJ
iv. Ankle stress films (to RIO ankle instability)
v. 3 phase bone scan to R/O fracture
vi. MRI to evaluate the soft tissues, fracture, and infectious disease or arthritic
process
vii. CT scan to R/O coalitions

4. Differential diagnosis:
a. Entrapment neuropathy of the intermediate dorsal cutaneous nerve
b. Entrapment neuropathy of the sural nerve
c. Damage to the anterior talofibular ligament d. Damage to the calcaneofibular
ligament
e. Peroneal tenosynovitis
f. DJD of the STJ (posterior or middle facet)
g. Coalition
h. Talar neck fracture
i. RA, gout, seronegative arthritides
j. Talar dome fractures
k. Space-occupying lesion

5. Treatment:
a. Conservative:
i. Injection of series of local anesthetic/steroid, once every 2 weeks x 3 times
ii. NSAID's
iii. Tape immobilization
iv. BK cast
v. Foot orthoses (if biomechanical)
vi. Physical therapy
b. Surgery: Sinus Tarsi evacuation as follows:
1. Transverse incision over the sinus tarsi 4 cm.
ii. Resection of portions of the extensor retinaculum, a fatty plug, the
cervical ligament, and a synovectomy of the adjacent middle and
posterior articulations of the STJ for a total distance of 2 cm.
iii. Evaluation of the STJ articulations (ATFL, CFL, and peroneals)
iv. Postop: Orthopedic shoe for 2 weeks, followed by physical therapy

NOTE* if the pain persists despite appropriate care then a STJ fusion or triple
arthrodesis is indicated

6. Complications:
a. Entrapment neuropathies
b. Avascular necrosis
c. Subtalar instability

Peroneal Subluxation
1. Clinical presentation:
a. Post-traumatic state: Evident after acute injury or later with a history of
recent antecedent trauma. The post-traumatic state represents:
i. Either a subperiosteal dissection of the superior peroneal retinaculum or
ii. An avulsion fracture of the fibula with the adherent superior peroneal
retinaculum and deep fascia
b. Chronic subluxing state with an insidious progression of pain about the
peroneals (not necessarily associated with trauma). This state represents:
i. A stretching, redundancy, or pouching of the superior peroneal retinaculum
and deep fascia permitting displacement of the peroneal tendons within an
intact compartment

NOTE* The key difference between the two forms of tendon dislocation is
based upon the relationship of the deep fascia and superior peroneal
retinaculum to the peroneal tendons and the distal fibula

2. Classification of peroneal injuries (Eckert and Davis):


3. Surgical technique (one type): A reduction of the deep fascial
redundancy with insertion Into the fibula
a. 8 cm incision placed posterior to the palpable posterior division of the
anterior border of the fibula overlying the peroneal compartment
b. Incision into the superficial and deep fascia
c. Periosteal incision just superior to the posterior division of the anterior
border of the fibula
d. 5 drill holes into the fibula from the lateral surface to the posterior surface,
with the most distal drill hole just overlying the peroneal groove area
e. Redundant deep fascia containing the peroneals is plicated
f. Sutures inserted (twice around) with the knots below the deep fascia
g. BK weight-bearing cast applied for 6 weeks

NOTE* A Jones procedure for peroneal stabilization involves detaching a


small strip of Achilles tendon proximally, and rerouting it from posterior to
anterior through a drill hole in the fibula, then sutured onto itself
Arthrodesis of the Ankle
1. Indications: This procedure is indicated primarily in patients with severe
pain and deformity, Including: DJD, RA, talar collapse, failed ankle joint
prostheses, infection of the ankle joint, drop foot, invasive tumors, and
congenital deformities

2. Preoperative evaluation:
a. Patients selected for ankle fusion should be those for whom it will be
reasonable to expect a significant reduction in pain and deformity, and an
increase in activity
b. Evaluate integrity of adjacent joints (STJ and forefoot): To R/O the necessity
for a secondary STJ fusion or triple arthrodesis
c. Ankle joint is fused at right angle to the leg, and compensation for heel
height must then come from plantarflexion at the midtarsal and
tarsometatarsal joints
d. Stability at the knee joint is important in those patients undergoing a
pantalar fusion
e. Good bone stock a necessity

3. Surgical approaches: Dictated by the exposure necessary to perform the


desired technique
a. Transverse anterior approach (Charnley): Severs the extensor tendons and
anterior neurovascular bundle
b. Midline longitudinal anterior approach: Inadequate visualization of the
posterior ankle joint
c. Lateral approach via hockey-stick incision: When combined with a fibular
osteotomy this approach gives good exposure of the posterior, lateral, and
anterior aspects of the ankle
d. Medial malleolar approach: When combined with medial malleolar osteotomy
gives good exposure of the anteromedial, medial, and posteromedial aspects of
the ankle joint

NOTE* Procedure of choice for good visualization is the lateral hockeystick


incision plus medial incision

4. Surgical technique categories:


a. Articular wedging with or without grafting
b. Anterior arthrodesis with inlay grafting
c. Articular wedging combined with malleolar osteotomy
d. Dowel or other subtotal fusions
e. Compression arthrodesis

5. The requirements for a successful fusion:


a. Complete removal of all the cartilage, fibrous tissue, and any other material
that may prevent contact of raw bone to raw bone
b. Accurate and close fitting of the fusion surfaces
c. Optimal position of the ankle joint
d. Maintenance of the bone apposition in an undisturbed fashion until fusion is
completed
Chrenshaw AH (ed): Campbell's Operative Orthopedics, Volume 2, CV. Mosby, St.Louis, 1971, p. 1126, with permission

Lateral Ankle Instability


1. Plantar flexion inversion injury classification (Leach):
a. 1st degree: ATFL
b. 2nd degree: ATFL, CFL, and capsule
c. 3rd degree: ATFL, CFL, PTFL, and capsule

2. Factors which predispose to recurrent ankle sprains:


a. Tibial varum
b. Ankle varum
c. Calcaneal varum
d. STJ varus
e. Plantarflexed 1st ray
f. Rigid forefoot valgus
g. Uncompensated equinus
h. Muscle imbalance
i. Weak peroneals
ii. Overactive tibialis anterior and tibialis posterior
i. Previous ankle sprains
j. Torsional abnormality
k. Short leg syndrome
Chronic Lateral Ankle Instability
Many surgical procedures have been devised to reconstruct the lateral
ligaments, but because of their unique configuration, accurate anatomic
reconstruction is nearly impossible.

The above diagram shows the relationship of the anterior talo-fibular ligament to the calcaneo-
flbular ligament. This relationship is hard to recreate with any surgical procedure. This is why
there is limitation of STJ range of motion following stabilization surgery

1. Etiology:
a. Post-traumatic ligamentous disruption
b. Osteochondral dome fractures
c. Degenerative joint disease
d. Peroneal subluxation
e. Muscular weakness or paralysis
f. Talofibular meniscoid
g. Generalized ligamentous laxity
h. Tibio-fibular diastasis
i. Non-union of previous fracture
j. Poorly reduced/healed fracture
k. Fixed calcaneal varus
l. Tibial varum
m. Rigid plantarflexed 1st ray
2. Anatomy of the lateral ankle:
a. The peroneal muscles terminate in tendons proximal to the ankle joint and a
common synovial sheath surrounds the peroneals at this point above the ankle
and contains them in a fibro-osseous tunnel
b. Posterolaterally, they are contained by the superior peroneal retinaculum
and medially by the lateral ankle joint ligaments
c. At the level of the ankle joint the peroneus brevis lies anterior and deep
against the fibular groove, with the peroneus longus posterior
d. As the peroneus brevis tendon passes distally, below the tip of the fibula, it
turns anterior, plantar, and lateral, crossing the CFL superficially
e. As the peroneus longus passes distally it runs plantar and anteriorly, then
passes in an inferior tunnel formed by the inferior peroneal retinaculum at the
level of the peroneal tubercle of the calcaneus. The peroneus brevis tendon lies
in a similar tunnel superior to the tubercle on the calcaneus
f. The ATFL is intracapsular
g. The CFL, PTFL, and fibulotalocalcaneal (not always present) are
extracapsular

NOTE* Due to the angular relationships of the ATFL and the ankle, inversion
of the ankle is primarily resisted by the ATFL when the ankle is
plantarflexed, and by the CFL when the ankle is dorsiflexed

3. Clinical and radiographic evaluation:


a. Common complaints: persistent instability, pain, edema, weakness, and
associated insecurity on uneven surfaces
b. Radiology: Talar tilt and anterior draw test (need comparison views of the
contralateral ankle), arthrography, tenography, and MRI

NOTE* Controversy exists over the interpretation of inversion stress views.


The anterior draw test is most useful in assessing the ATFL integrity (0-5 mm
of anterior displacement is normal, 8-10 mm consistent with a single
ligament rupture, 10-1.5 mm consistent with a double ligament rupture, and
greater than 15 mm anterior displacement is consistent with a triple
ligamentous rupture).
These two tests are complementary.
They should be done under common peroneal and sural nerve block. A
Telos® stress device is used for more of a constant/gradually applied
force.

4. Surgical Procedures: Stabilization procedures depend upon the patient's


needs
a. Delayed primary repair: Suturing of the ATFL/CFL with a non-absorbable
suture (0 or 2-0), with NWB BK casting for 6 weeks b. Delayed secondary
repairs (utilizing fascial grafts): Are categorized according to the number of
ligaments ruptured
i. Single ligament rupture:
• Watson-Jones*: This uses the peroneus brevis, which passes through
the fibula from posterior to anterior, through the neck of the talus from plantar
to dorsal, back through the fibula, from anterior to posterior, and sutured back
onto itself.
• Lee Procedure (modified Watson-Jones)*: This uses the peroneus
brevis tendon, which is then passed through the fibula, from posterior to
anterior, and then sutured back onto itself.
• Evans*: This utilizes the peroneus brevis through an oblique hole
through the fibula sutured back onto. the belly of the peroneus brevis.
• Storren
• Nilsonne
• Pouzet
• Haig
• Castaing and Meunier
• Dockery and Suppan

ii. Double ligament rupture:


• Elmslie*: Originally described as using the fascia lata and passed
through a drill hole in the lower aspect of the fibula, through the calcaneus,
back through the same drill hole, and tied onto itself, after passing through the
neck of the talus.
• Chrisman and Snook*: This uses the split peroneus brevis, which is
passed through the fibula from anterior to posterior through a flap in the
calcaneus, and is then sutured back to the peroneus brevis tendon.
• Stroren
• Hambly
• Winfield
• Gschwend-Francillon

iii. Triple ligament rupture:


• Spotoff
• Rosendahl and Jansen

NOTE* A Split Peroneus Brevis Lateral Ankle Stabilization Procedure was


developed at Doctor's Hospital, which has proven to have minimal morbidity
and to be dependable, especially for the athletic individual The surgical
procedure is as follows:
a. Patient lying in the lateral position with thigh tourniquet
b. Single incision beginning approx. 10-12 cm proximal to the lateral
malleolus, extending distally just posterior to the fibular malleolus
c. A subperiosteal channel is created from the neck of the talus to the base of
the 5th metatarsal, and a wire loop is placed within the channel to facilitate
passage of the tendon later
d. The peroneal retinaculum is incised just posterior to the fibula, peroneals
retracted posteriorly, CFL exposed, and a 4 mm hole is made from the
anterior edge of the fibula, angled slightly inferiorly (bone plug is saved)
e. A 6 mm hole is made in the body of the calcaneus adjacent to the insertion
of the CFL
f. A final subperiosteal channel is made from the posterior fibula to the hole
created in the calcaneus, and a wire loop is inserted for later passage of the
tendon
g. The peroneus brevis tendon is then split (at the start of the muscle belly),
tagged with a suture and pulled distally within its tendon sheath to the base
of the 5th metatarsal
h. The tendon is brought through the 1st subperiosteal channel to the neck
of the fibula and passed through from anterior to posterior, sutured to the
anterior fibula
i. The tendon is brought through its 2nd subperiosteal channel and buried
into the hole created in the calcaneus

NOTE* Either dacron mesh (Dacron Cooley graft) or Marlex surgical mesh
(porous film) can be used as an adjunct in the repair of the lateral ligaments

5. Complications after reconstruction:


a. Questions that should be asked to determine problems:
i. Is there still a sense of ankle instability and repeated inversion injuries?
ii. Is the primary problem chronic pain? If the problem is the former, then
reconstruction is the problem.. If the problem is the latter then there might be a
secondary lesion
b. A varus rearfoot can contribute to reinjuring the ligament and causing a
failed reconstruction over time. Also subtalar instability can cause failure (if
this is the reason, the patient might. require additional surgery involving the
calcaneofibular ligament)
c. If pain is the patient's main complaint you must rule out a coexistent lesion
such as an osteochondral fracture, tarsal coalition, ankle arthrosis, chronic
tear in the peroneal tendon, or postoperative neuroma of the sural nerve

Chronic Medial Ankle Instability


Deltoid injuries are rare and occur when the foot is forcibly everted on the leg
(see chapter 25: Deltoid Injuries). If an external rotary component is also
present, a concomitant fibular fracture with disruption of the T-F syndesmosis
will occur. Therefore, when confronted with an unstable medial ankle check for
a displaced fibular fracture or T-F diastasis
1. Anatomy:
a. Superficial deltoid: Tibiocalcaneal
b. Deep deltoid: Anterior tibiotalar, tibionavicular, and posterior tibiotalar

2. Surgical repair:
a. Delayed primary repair

Ankle Equinus
Primary neuromuscular spasticity of the posterior muscle group needs to be
approached differently than a secondary acquired contracture of the
gastrocnemius. The goal of the surgery will also depend on the etiology.
The resulting gains in ankle joint dorsiflexion must be weighed against the loss
of a grade of muscle strength
1. Patient complaints:
a. Low back pain
b. Calcaneal apophysitis in children
c. Heel spur syndrome in adults
d. Arch pain
e. Inability to stand for long periods without pain/fatigue
f. Juvenile HAV
g. Digital contractures

2. Anatomy:
a. The gastrocnemius is a muscle spanning two joints, and forming the belly of
the calf attached by two heads to the femoral condyles, the medial head being
larger
b. The gastrocnemius and soleus are innervated by the tibial nerve from S1 and
S2
c. The soleus is a broad flat muscle deep to the gastrocnemius, arising from the
back of the head and the upper fourth of the posterior surface of the fibula.
d. The soleus joins with the gastrocnemius to form the tendo Achilles. At the
insertion, the gastrocnemius component usually comprises the lateral side of
the superficial surface and a small portion of the lateral aspect of the deep or
anterior surface of the tendon
e. Superficial to soleus are gastrocnemius and plantaris; deep are flexor
digitorum longus, flexor hallucis longus, tibialis posterior, and the posterior
tibial vessels and tibial nerve, all separated from soleus by the deep transverse
fascia
f. The plantaris arises in close association with the lateral head of the
gastrocnemius, has a small fusiform belly, that ends in a long slender tendon,
which crosses obliquely between gastrocnemius and soleus and runs along the
medial border of the tendo Achilles,-to be inserted with it. The plantaris limits
dorsiflexion of the ankle
g. The tendo Achilles is the thickest and strongest human tendon, surrounded
by a paratenon (highly vascular areolar tissue which bathes the tendon in
synovial fluid). The tendo Achilles attaches to the posteriorsuperior surface of
the calcaneus
h. The anatomy of the ankle joint is a modified ginglymus joint because in full
plantarflexion, the narrow posterior aspect of the trochlear surface of the talus
allows the frontal plane motion of inversion and eversion to occur in the mortise
(normal dorsiflexion should be 10-200, normal plantarflexion 30-500)

3. Muscular activity:
a. The muscles of the calf are the major plantarflexors, the gastrocnemius
also extends the knee, and the soleus steadies the leg on the foot in standing
b. Phasic activity of the triceps is from shortly after heel contact until just
before toe-off, all functioning to extend the knee during a normal gait
cycle (it does this through the soleus, by slowing the forward progression of the
tibia, thus allowing the femur to rotate over it, extending the knee joint)
c. The soleus fires at about 15%-20% of the stance phase of gait, slightly ahead
of the gastrocnemius

4. Pathomechanics:
a. In normal function, with the knee fully extended and the STJ in neutral
position, at least 100 of dorsiflexion of the foot to the leg are needed
b. When there is pronation past perpendicular, this shortens the origin to
insertion distance of the gastrocnemius (the distal aspect of the calcaneus
lowers, the midtarsal joint lowers to the ground, the calcaneal inclination angle
decreases) and over a period of time there is secondary adaptation, and the
amount of dorsiflexion is limited
c. With a primary equinus deformity at 50%-60% of midstance, with the hip
and knee extended, you cannot get 100 of dorsiflexion and, therefore, the
subtalar joint and midtarsal joints pronate to give more dorsiflexion (the oblique
axis of the MTJ gives more dorsiflexion than any other joint in the foot as it
compensates for the equinus
d. This midtarsal joint pronation will also induce the forefoot into a supinatus
deformity

NOTE* Tendon lengthening should not be done unless the etiology of the
equinus is absolutely certain because there will be an automatic loss of 10%
of its strength

5. Classification:
a. Uncompensated: The STJ remains supinated, therefore, the lack of
dorsiflexion at the ankle joint cannot be compensated by abnormal STJ and
MTJ pronation (the CP patient)
b. Fully compensated: Presents with STJ and MTJ pronation, seen as the
hypermobile flatfoot, with the rearfoot maximally everted to the floor and the
forefoot everted on the rearfoot (the most severe symptom complex). Creates a
spinal lordosis, excessive transverse plane motion about the knee, low back
pain, chondromalacia, exaggerated genu valgum, juvenile bunions, and digital
contractures
c. Partially compensated: The patient has sufficient combination of ankle joint
dorsiflexion and dorsiflexion about the oblique axis of the MTJ to permit heel
contact during the early portion of the stance phase of gait. However, there is
insufficient dorsiflexory motion to permit the leg to angulate 100 to the floor
later in stance. Early heel-off is seen during gait. Generally, with a partially
compensated equinus, the calcaneus will strike the floor but will evert only
minimally, and then will rapidly achieve an early heel-off

6. Etiologies:
a. Muscular (gastrocnemius equinus, gastrosoleus equinus)
ii. Congenital shortness: Toe walker for 1 st 6 months
iii. Acquired shortness: From casts, high-heeled shoes, excessive pronation
b. Osseous
i. Osseous equinus: Dorsal exostosis of the talar neck from a flat top talus (can
be from treatment of a clubfoot)
ii. Pseudoequinus (cavus foot type)

7. Clinical findings:
a. With the patient supine and the knee extended and the STJ in neutral
position, the foot is dorsiflexed. If dorsiflexion is less than 100 an ankle equinus
to exists
b. Silverskiold test is then used to differentiate gastrocnemius equinus from the
remaining types of posterior equinus: by flexing the patient's knee to a right
angle and again dorsiflexing the patient's foot. If more than 100 of dorsiflexion
is found a gastrocnemius equinus still exists. Thus after performing the
Silverskiold test there is still limitation of ankle joint dorsiflexion, other sources
of ankle joint dorsiflexion remain to be evaluated (either gastrosoleal equinus or
osseous equinus).

NOTE* To help evaluate osseous equinus a stress dorsiflexion lateral x-ray is


used. Comparison is made with the regular lateral view and checked for 2
items:
a. The excursion of the tibia over the talus
b. How much motion of the tibia and talus is occurring as a unit

8. Surgical procedures: See Chapter 27, Tendon pathology: Tendon


Lengthening and Tenotomy
a. Distal recession (Volpius and Stoffel)
b. Slide lengthening (White)
c. Distal recession (Strayer)
d. Lengthening the apneurotic tendon of. the gastrocnemius (Baker)
e. "Z" plasty (Sgarlato)
f. Frontal plane "Z" plasty (McGlamry)
g. Modified Baker tongue and groove (Fulp and McGlamry): For nonspastic
gastrocnemius equines

Malunion and Nonunion of the Malleoli


According to Ramsey and Hamilton, 1 mm of ankle asymmetry causes a 42%
decrease in contact area between the talus and the tibia. Therefore, because
stress per unit area increases as the total contact area decreases, even mild
malunion of the fibula and the ensuing talar instability can result in significant
disability
1. Malunion of the fibula:
a. Might not become symptomatic for several years, with the patient
complaining of a dull ache over the anterior aspect of the ankle which worsens
with use, swelling, onset of arthritis, and a limp
b. Radiographic criteria used to diagnose a malunion of the lateral malleolus
are similar to those used in evaluating acute ankle fracture
i. Evaluation of the tibiofibular articulation
ii. Fibular length and rotation
iii. Ankle mortise symmetry
iv. Talar tilt
v. Bimalleolar angle: measures the exact amount of fibular shortening
(comparison of both ankles is necessary, and a 10 change in the angle
corresponds to a 1 mm change in fibula length).
NOTE* Three views should be used (AP), (LAT), and 15° internally rotated
mortise view
c. Conservative treatment:
i. AFO
ii. NSAIDs
iii. Injections
d. Surgical treatment: When conservative measures fail, reconstruction can be
attempted. If that fails. then a fusion can always be done.
i. The concurrent presence of a medial malleolar malunion makes
reconstruction more difficult. A malleolar malunion has no substantial effect on
stability of the ankle and only needs to be corrected if symptomatic or- with a
step-off in the weight-bearing joint surface of more than 2 mm.
ii. A concomitant malunion of a large posterior malleolar fragment (more than
30% of the joint) usually precludes reconstruction (posterior displacement of
the talus, soft tissue scarring, and osteoporosis of the displaced piece make
reduction difficult)
iii. The fundamentals are that there is a shortened and externally rotated lateral
malleolus, with lateral subluxation. The procedure to accomplish correction is
as follows:
• The fibula must be freed leaving only the collateral ligaments intact
• An oblique or transverse osteotomy is made at the level of the malunion
and an AO compression device set in the distraction mode (to increase fibular
length)
• A plate is fixed to the fibula
• The syndesmosis is held in reduction with a 4.5 mm cortical screw

2. Nonunion of the medial malleoli:


a. Occurs more frequently in fractures that are treated with closed reduction
b. Surgery for a nonunion should only be considered if the clinical examination,
including pain, tenderness at the fracture site, correlates with the radiographic
findings
c. Nonunion of the medial malleolus can occur anywhere but proximal lesions
nearer the weight-bearing aspect of the tibial plafond are more likely to be
symptomatic
d. If a malunion is present it should be taken down and anatomically reduced
with compression screws and a local cancellous bone graft from the metaphysis
of the distal tibia
e. If the nonunion is in anatomic alignment, it can be left in place with a trough
trough created across the nonunion site and packed by a cancellous bone

3. Nonunion of the lateral malleoli:


a. Similar approach should be taken as with medial malleolar nonunions,
although these are more prone to produce a malunion and instability
b. A small, distal, non-united fragment (smaller than 1 cm) should be excised
rather than trying to obtain osseous union. The collateral ligaments should be
carefully reattached
Chapter 30: Heel Conditions
Anatomy of the Heel
Radiological Evaluation of the Calcaneus
The Heel in Systemic Disease
Seronegative Arthritis and Heel Pain
Heel Spurs and Heel Spur Syndrome
Tumors of the Heel
Tarsal Coalitions
Sever's Disease
Haglund's Deformity
Causes of Heel Pain (a summary)
HEEL CONDITIONS

Anatomy of the Heel


1. Blood supply:

2. Attachments:
a. The plantar calcaneal tuberosity consists of 2 smaller tubercles, the larger,
the medial process, and the smaller the lateral process, separated by a sulcus.
b. The plantar aponeurosis is made up of a medial, central, and lateral band
(the central band being divided into a superficial and a deep plantar fascia).
c. The central band is attached to the medial process of the plantar tuberosity,
posterior and plantar to the origin of the flexor digitorum brevis.
d. The band divides into 5 slips as it approaches the digits (the superficial
stratum inserts into the skin of the transverse sulcus separating the digits from
the sole, and the deeper stratum divides into two slips which embrace the sides
if the flexor tendons and blend with the sheaths of these tendons).
e. The abductor hallucis originates from the medal tubercle of the calcaneal
tuberosity and plantar aponeurosis and extends along the medial side of the
foot until it inserts into the medial side of the proximal phalanx of the hallux.
f. The FDB attaches at the medial calcaneal tubercle just superior to the
plantar aponeurosis, and extends along the aponeurosis where it sends tendon
slips to the 4 lateral digits, with each tendon splitting into 2,
allowing the FDL to pass between them on its way to the distal phalanx (these
medial and lateral slips insert into the base of the middle phalanx)
g. The abductor digiti minimi originates at the medial and lateral tubercles of
the calcaneal tuberosity and plantar aponeurosis and extends along the lateral
aspect of the foot to insert into the lateral side of the base of the proximal
phalanx of the 5th toe
h. The long plantar ligament attaches at the medial and lateral tubercles of the
calcaneal tuberosity and extends across the plantar aspect of the calcaneus
where it branches into 4 ligamentous slips to the bases of the 4 lesser
metatarsals
i. Dorsally interconnecting the talus and calcaneus and in reinforcing the
functional subtalar joint are the interosseous talocalcaneal ligament (located in
the sinus tarsi) and the cervical ligament (located lateral to the sinus tarsi
j. Other ligaments include: the lateral, medial, and posterior talocalcaneal
ligaments, and the calcaneotibial and calcaneofibular portions of the collateral
ligaments of the ankle joint
k. The calcaneal dorsal surface has 3 facets, anterior, middle and posterior. The
posterior is the largest, separated from the middle by the sulcus calcanel (or
sinus tarsi when including the sulcus tali of the talus)

3. Innervation:
a. The tibial nerve gives off the medial calcaneal branches that innervates the
heel while continuing through the tarsal tunnel, and as it exits the tunnel it
divides into the medial and lateral plantar nerves
b. The lateral plantar nerve runs along the medial side of the lateral plantar
artery where it innervates the quadratus plantae and abductor digiti minimi
ms., and then divides into superficial and deep branches. The lateral plantar
nerve travels deep to the plantar aponeurosis as it leaves the tibial nerve from
the tarsal tunnel to travel distally and laterally and crosses the aponeurosis
where it inserts into the calcaneal tuberosity
c. The inferior calcaneal nerve branches from the lateral plantar nerve just
distal to the bifurcation of the tibial nerve into medial and lateral plantar nerve,
and courses between the abductor hallucis ms. and medial head of the
quadratus plantae ms., and continues laterally remaining 5.5 cm anterior to
the calcaneal tuberosity coursing between the FDB and long plantar ligament.
It finally crosses over the lateral head of the quadratus plantae and terminates
in the abductor digiti minimi ms.

Radiological Evaluation of the Calcaneus


The Heel in Systemic Disease
Many diseases manifest themselves in the heel. These include the following:
1. Rheumatoid arthritis:
a. Sources of heel pain in RA have been attributed to plantar and posterior
spurring, calcaneal erosions, valgus deformity of the STJ and heel, associated
sites of soft tissue inflammation (Achilles tendonitis, plantar
myofasciitis, inferior and posterior calcaneal bursitis), or the presence of
rheumatoid nodules
b. Most commonly affected sites in the heel are the posterior surface near the
insertion of the Achilles tendon and the inferior surface near the origin of the
plantar fascia
c. Bony changes involve bony proliferation or spurring, erosive changes,
sclerosis, and osteoporotic changes

NOTE* Rheumatoid arthritis according to a study by Resnick et al, produces


erosive bony changes at the posterior/superior surface and the posterior
surface of the calcaneus immediately above the site of attachment of the
Achilles tendon. It also produces well-developed posterior and plantar spurs
at the posterior surface at the site of insertion of the Achilles, and the plantar
surface anterior to the site of attachment of the plantar aponeurosis

2. Gout:
a. The calcaneus may be affected in gout by pressure erosion from adjacent
tophaceous deposits penetrating bone, classically producing wellmarginated
bony erosions with sclerotic margins and over-hanging edges
b. Mineralization of the calcaneus in gout is unaffected

3. Calcium pyrophosphate dihydrate deposition (CPPD):


a. This disorder mimics acute gouty attacks, and rarely affects the calcaneus,
however, can affect the talocalcaneonavicular region producing an
osteoarthritis profile demonstrating joint space narrowing, and subchondral
cyst formation. This is sometimes called chondrocalcinosis.

4. Osteoporosis:
a. It is estimated that 30% of bone must be lost before osteoporosis can be
identified radiographically
b. It has been suggested by numerous authors that the calcaneus be used in
the evaluation of osteoporosis by grading changes (as bone mass diminished,
there was a reproducible change in the trabecular appearance which was
progressive as bone became more porotic)

5. Diffuse idiopathic skeletal hyperostosis (DISH):


a. This is recognized as an ossifying diathesis, most commonly encountered in
middle-aged males, characterized by areas of hyperostosis at points of
attachment of tendon, ligament or fascia to bone (axial spinal symptoms
predominate).
b. Pedal conditions involve the talus (beaking), heel pain, and plantar and
retrocalcaneal spurs. The spurs are large, irregularly shaped, with well defined
margins, without reactive sclerosis, periosteal reaction, or erosions producing a
noninflammatory appearance (calcification of the plantar fascia may be seen)

6. Diabetes mellitus:
a. Periarticular calcifications of the calcaneus
b. Calcifications of the long plantar ligament
c. Osteophytosis of the calcaneus
d. Diabetic osteoarthropathy
e. Pathologic fractures
7. Hypertrophic osteoarthropathy:
a. Involves the clinical triad of clubbing of the nails, periostitis with new bone
formation, and arthritis. Often seen secondary to pulmonary neoplasms or
pulmonary suppurative conditions, bowel disorders, heart disorders, thyroid
disorders, and other conditions (some genetic)
b. It has been reported that the calcaneus has been involved with this disease,
manifesting itself with a band of increased density elevated from the lateral
aspect of the tuberosity, in the region of the calcaneo-cuboid joint (acute
periostitis with subperiosteal new bone formation)

8. Paget's disease of bone (osteitis deformans):


a. This condition involves accelerated bone resorption and destruction followed
by disorganized repair, leaving an irregular mosaic pattern of well defined
mature and immature bone
b. The calcaneus has been seen to be involved, with the chief symptom of pain
in the heel
c. Radiographic demonstration of areas of decreased density with widening,
destruction, and disorganization of the calcaneal trabecular pattern amid
irregular, patchy, sclerotic areas creating the typical irregular mosaic type
pattern

9. Sarcoidosis:
a. Is a multisystem, multiorgan disorder of a autoimmune etiology and with
associated immunologic abnormalities, typified by the development of
noncaseating granulomas in various organs
b. Increased observance of HLA B8 antigen
c. Sarcoid arthropathy occurs 3-15%, and presents as an acute polyarthritis
d. This disease can affect the calcaneus with the symptom of heel pain.
Radiographically there can be cortical defects or cyst formation

10. Sickle cell anemia:


a. An autosomal dominant disorder characterized by an abnormality in
hemoglobin, producing hemolytic crises and a variety of clinical complications
related to vascular occlusive phenomena. b. Calcaneal involvement has been
reported manifested by aseptic necrosis documented by Tc-99m bone scans
(decreased uptake), and an erosive process on the superior surface of the
calcaneus (may be pathognomonic for sickle-cell)

11. Acromegaly:
a. Occurs secondary to an excessive amount of growth hormone present after
epiphyseal closure which results in excessive growth of various body parts,
(hands, feet, jaw, internal organs, etc.)
b. Clinical there is thickening of the skin (increased heel pad thickness)

Seronegative Arthritis and Heel Pain


Patients who do not respond to conservative treatment or present with an
atypical picture might cause the clinician to consider some of the less common
causes of heel pain as these.
1. General features of the seronegatives:
a. Unlike RA, these disorders have a greater affinity for the spinal and sacroiliac
areas, and involve the insertions of tendons and ligaments on bone which
produce painful enthesopathies (these are thought to produce the heel pain
associated with the seronegatives)
b. Increased incidence of HLA B27

NOTE* HLA B27 is thought to be linked to genes that regulate new bone
formation

2. Ankylosing spondylitis (Marie Strumpell disease): Peripheral arthritis may


be the initial presentation and the heel may be involved.

3. Reiter's Syndrome: Involvement of the enthesis of the calcaneus, presenting


with mild, moderate swelling without redness.

4. Psoriatic arthritis

5. Treatment:
a. NSAIDs usually work well
b. Methotrexate may be used in severe cases of Reiter's syndrome and psoriatic
arthritis
c. Physical therapy
d. Orthoses
Heel Spurs (Heel Spur Syndrome)
1. Etiology:
a. Disease processes
b. Biomechanical abnormalities (physiologic reaction to constant stress forming
new connective tissue which eventually converts to bone) includes all types of
feet with an abnormal pronation component and even supinated foot types
NOTE* A heel spur (exostosis) need not be painful; it is only significant that
the patient has pain at the anatomic site and that we determine the pain is
caused by a mechanical abnormality and not to any other sources of heel
pain (heel spur syndrome). Abnormal pull of the plantar calcaneal
periosteum at the tuberosity causes separation of periosteum from bone and
an inflammatory reaction (hence pain). The pulled periosteum fills in forming
the spur. Hence, it is an adaptive response. No pull=no separation=no
inflammation=no pain. Once a spur forms and there is no new pull, etc.,
there is no pain despite the presence of a spur (barring fractures, etc.).
Pronation and supination place a twisting pull on the calcaneal periosteum
at the tubercles, hence spurs form.

2. Diagnosis:
a. Radiographically
b. Palpation with execution of the Hubscher maneuver over the medial band of
the plantar fascia

3. Treatment: Heel spurs should only be treated if symptomatic


a. Orthoses
b. Oral anti-inflammatory medications and steroid injections
c. Shoe accomodations
d. Strappings
e. Physical therapy
f. Surgery:
i. Surgical approaches:
• Medial horizontal incision (DuVries)
• Posterior horizontal "U" incision (Griffith)
• Transverse plantar incision (Michetti)
• Longitudinal plantar incision
• Lateral horizontal incision
• Minimal incision approach (Mercado)
• Endoscopic Plantar Fasciotomy (Barrett and Day)
ii. Whatever the approach, careful dissection is mandatory to avoid
transection of the lateral plantar nerve
iii. Other complications
• Wound dehiscence
• Hematoma
• Phlebitis
• Infection
• Fracture
NOTE* Endoscopic Plantar Fasciotomy by Barrett and Day has been reported
to give excellent results via the transection of a portion of the plantar fascia.
The bony exostosis is left intact. This procedure utilizes two small incisions
with a slotted canula passed just inferiorly to the fascia. Through one end of
this canula a camera (scope) is passed and through the other end a small knife
is passed.

Tumors of the Heel


As the largest bone in the foot, the calcaneus has many anatomic features
which make it unique and potentially more prone to develop tumors and tumor-
like conditions. Internally, it has an abundant vascular supply, which may
explain the increased incidence of metastatic malignant lesions. Also
the large cancellous component may sequester these tumors for long periods of
time before they become symptomatic. Externally, the extensive surface area of
the calcaneus and the numerous points of attachment of intrinsic and extrinsic
muscles, tendons, and ligaments lend themselves to a vast array of tumors.
Pain has been shown to be the primary complaint of patients presenting with
tumors.
1. Pseudotumors:
a. Inclusion cysts: in the soft tissues
b. Traumatic neuromas: in the soft tissues
c. Ganglionic cysts: in the soft tissue or bone
d. Keloids: in soft tissues
e. Foreign body granulomas: in soft tissues
f. Piezogenic papules (protrusions of adipose tissue surrounding the heel)
g. Rheumatoid nodules: in soft tissues

2. Skin and soft tissue tumors:


a. Verrucae
b. Molluscum contagiosum
c. Keratoacanthoma
d. Squamous cell carcinoma
e. Plantar fibromatosis (Ledderhose's or Dupuytren's disease)
f. Lipomas
g. Eccrine poroma
h. Leiomyoma
i. Glomus tumor
j. Malignant melanoma
k. Kaposi's sarcoma
l. Many others

3. Bone tumors:
a. Solitary bone cyst: Most frequently seen bone lesion other than the heel
spur, found in the anteroinferior and lateral regions of the calcaneus. Can be
found as a result of pathologic fracture of the thin wall (“Fallen Fragment” sign
on x-ray)
b. Ewing's sarcoma and other sarcomas
c. Osteoid osteoma
d. Intraosseous lipoma
e. Giant cell tumor of bone

f. Multiple myeloma: the most common primary bone malignancy


g. Aneurysmal bone cyst
h. Chondroblastoma
i. Osteoblastoma
j. Chondromyxoid fibroma
k. Hemangioma
l. Osteogenic sarcoma

Tarsal Coalitions
It is usually a congenital anomaly that represents a failure of differentiation and
segmentation of primitive mesenchyme, which results in failure of joint
formation. These coalitions cause a limitation or absence of motion of the
involved joint and can affect the entire foot in gait
1. Classifications:
a. Intra-articular vs. extra-articular (usually accessory bone fusion)
b. Fibrous, cartilaginous, or osseous
c. Developmental pattern
d. Congenital or acquired (acquired cases include trauma, previous surgery,
infection, RA, and OA).

2. Types:
a. Talocalcaneal (fuses between the ages of 12-16): Talar beaking; broadening
lateral talar process; middle subtalar joint not visualized; asymmetric anterior
subtalar joint; "ball and socket" ankle joint. Usually the middle facet.
b. Calcaneonavicular (fuses between the ages of 8-12): Close approximation of
the calcaneus and navicular; irregularity and indistinctness of cortical surfaces;
hypoplastic head of the talus c. Talonavicular
d. Calcaneocuboid (rare)

3. Pathognomonic Signs and Symptoms:


a. Peroneal Spasticity (peroneal spastic flatfoot) b. Talonavicular beaking
c. Halo sign (in cases of T-C coalition, occasionally this sign can be observed on
the lateral projection, seen around the sinus tarsi) d. Broadening and flattening
of the lateral process of the talus e. Decrease in ROM of the subtalar joint f.
Sudden onset of pain after excessive activity
g. Children begin to complain between the ages of 12-15 (with T-C bars) h.
Sinus tarsi syndrome
i. If an osseous bar is present then there will be no motion at the STJ and no
pain over the bar but pain can be present distally j. Adults may be
asymptomatic but show degenerative changes

4. Radiological Diagnosis:
a. Normal Harris Beath projections: the posterior and middle facets should be
present and parallel to each other. With a subtalar coalition the facets are no
longer parallel.
b. CT scanning provides the best diagnostic tool (it is the gold standard)
c. Calcaneonavicular coalitions can best be seen on the 45° medial oblique x-
ray

d. Talonavicular and calcaneocuboid coalitions can be seen on the lateral view

5. Treatment: Surgery
a. Calcaneonavicular coalition:
i. A modified Oilier approach
ii. Origin of the extensor digitorum brevis muscle belly is detached
proximally and reflected distally (bar now exposed)
iii. An osteotome or saw is used to resect the osseous segment (at least 1
cm of bone is removed)
iv. The EDB belly is now placed Into the defect created and then sutured to the
plantar medial aspect of the foot using Keith needles, nonabsorbable suture,
and button fixation
v. The wound is closed in layers
vi. A BK cast is applied for 4 weeks

NOTE* The calcaneonavicular bar resection is contraindicated in the


presence of degenerative changes In the talonavicular joint with
accompanying talar beaking, with complete ossification of the bar, and when
there is a second coalition between the talus and calcaneus. If this procedure
fails, a triple arthrodesis may be indicated to relieve the patient's symptoms
b.

Talocalcaneal coalition:
i. Conservative therapy 1 st since most of these are asymptomatic and when
symptomatic respond to conservative care.
• BK walking casts for 3-6 weeks
• Injection into the sinus tarsi with steroids
• Orthoses
ii. Surgery (resection of the bar vs. triple arthrodesis)
•Medial Incision for resection of middle facet coalition, with the incision starting
just behind the medial malleolus and following the top of the calcaneocuboid
joint
• The posterior tibial and FDL are retracted superiorly; the FHL and
neurovascular bundle are retracted inferiorly
• K-wires are used to locate the joint
• A rotary drill with a burr is used to resect the coalition
• If successful there will be an immediate increase in motion

c. Associated Conditions: (Tarsal coalitions have been reported to be


associated with the following)
i. Phocomelia and hemimelia
ii. Nievergelt-Pearlman syndrome
iii. Arthrogryposis multiplex congenita

Sever's Disease
Sever's disease was first described by Haglund who noted irregularities of the
calcaneal apophysitis, similar to those observed in osteochondrosis of the tibial
tuberosity. It is the only bone in the body whose epiphysis assumes the entire
weight before it is ossified
1. Secondary ossification:
a. First appears in females (ages 4-6) and later in males (ages 7-8)
b. It can be divided into a bipartite or tripartite apophysis
c. Fusion takes place as early a 12 years old in females and 15 years old in
males

2. Etiology:
a. Tension from the Achilles tendon and plantar fascia (equinus)
b. Acute and chronic trauma
c. Infection
d. Embolism
e. Stress fracture
f. Obesity
g. Congenital and hereditary factors
h. Endocrine disturbances
i. Diet disturbances

3. Symptoms:
a. Increased pain with activity especially sports
b. Demonstrable pain when the posterior aspect of the heel is squeezed from
side to side when direct pressure is exerted on the lower one-third of the
posterior calcaneus

4. Treatment: Always conservative


a. Rest and cessation of sports
b. Heel lifts
c. BK cast in resistant cases
d. Follow-up with an orthoses
e. Stretching excercises

(NOTE* The symptoms will resolve when the apophysis fuses


Haglund's Deformity
Haglund's syndrome, a common cause of pain in the posterior heel, consists of
a painful swelling of the local soft tissues with or without the prominence of the
calcaneal. bursal projection.
1. Clinical presentation: It is a chronic, sometimes painful condition,
characterized clinically by a tender swelling in the region of the Achilles tendon
near its insertion and radiographically by an osseous proturberance at the
superio-posterior aspect of the calcaneus

2. Radiographic analysis:
a. Fowler-Phillip angle: 44°-69° is normal (>75° is pathologic)

NOTE* Since the calcaneal inclination angle can influence the usefulness of
the Fowler-Phillip angle, the C-I angle should, therefore, be taken into
account (see the following diagram)

b. Using the following diagram is more accurate to represent this disorder

3. Treatment:
a. Conservative: Removing pressure either with a heel lift or orthoses

b. Surgical:
i. Resection of the posteriosuperior border of the calcaneus and bursa
ii. Wedge (osteotomy with the base dorsally) is removed from the
proximal half of the calcaneus posteriorly to the posterior articular facet (the
vertical cuts to be two-thirds the height of the calcaneus, but through and
through from medial to lateral)

NOTE* The incision most commonly employed for both procedures is a lateral
para-Achilles tendon approach with the incision being linear, lazy "L", or
reversed "J" shaped
Causes of Heel Pain (a summary)
1. Inflammatory:
a. Juvenile RA
b. Rheumatoid arthritis
c. Ankylosing spondylitis
d. Reiter's syndrome
e. Gout

2. Metabolic:
a. Migratory osteoporosis
b. Osteomalacia

3. Degenerative:
a. Osteoarthritis
b. Atrophy of fat pad

4. Nerve entrapment:
a. Tarsal tunnel syndrome
b. Entrapment of the medial calcaneal branch of the PT nerve
c. Entrapment of the nerve to the abductor digiti quinti

5. Traumatic:
a. Calcaneal fractures
b. Calcaneal malunions
c. Traumatic arthritis
d. Rupture of the fibrous septae of the fat pad
e. Puncture of the fat pad

6. Overuse syndromes:
a. Plantar fasciitis
b. Stenosing tenosynovitis of the FDL and FHL
c. Calcaneal apophysitis
d. Subcalcaneal bursitis
e. Periostitis
f. Calcaneal stress fractures
g. Achilles tendonitis
h. Haglund's deformity

7. Infectious:
a. Osteomyelitis

8. Other:
a. Tumors
Chapter 31: Amputations
General Surgical Technique
Digital Amputations
MPJ Amputations
Transmetatarsal Amputations
Tarsometatarsal Amputations (Lisfranc's)
Midfoot Amputations (Chopart's)
Amputation of the Ankle (Syme's, Boyd, Pirogoff)
Below Knee Amputation
AMPUTATIONS
Amputation that includes disarticulation is an ancient surgical procedure, with
techniques that have evolved over the centuries. In the last 20 years research
and advances in surgical technique have led to a more scientific approach, so
that the indications are more exact, and the level of amputation is more distal.
The combined approach of vascular surgeon, podiatrist, and plastic surgeon
have helped prevent major amputations, or allowed amputations to be
performed more distally on the extremity, resulting in less morbidity and lower
mortality.
General Surgical Technique
1. Before surgery the general condition of the patient must be stabilized (control
of infection, blood sugar, hydration, etc.)

2. Surgical handling of skin must be meticulous

3. A tourniquet is used except in the presence of vascular disease, and the limb
is exanguinated if no infection or tumor is present

4. Flaps should be planned to provide sensate skin over the stump 5. The scar
must not adhere to the underlying bone

6. Blood vessels should be carefully ligated and nerves divided as proximally as


possible under gentle traction to allow them to retract (these divided nerves
should not lie close to the bone)

7. The bone end should be rounded and beveled appropriately

8. Drainage of the stump is mandatory

9. If infection is present, and the viability of the tissues is in question, the


wound should be left open

Digital Amputations
1. Indications:
a. In gangrene of the toes (need adequate proximal arterial flow)
b. Malignant or large, deforming, benign tumors of the digits

2. Effects of digital amputation:


a. Amputation of all the toes allows the patient to have a normal gait while
walking slowly
b. Amputation of the 2nd toe alone results in the hallux going into a valgus
position
c. Amputation of the hallux only does not alter gait when the patient is walking
slowly but causes a limp with a quick walk because of loss of push-off
3. Procedure:
a. Short dorsal flap at the level of the bone resection and a long plantar flap can
then be reflected over the stump
b. The tendons and nerves should be sectioned after gentle traction to allow
them to retract
c. Round the bone end to prevent a distal keratoma

MPJ Amputations
1. Indications:
a. Severe trauma
b. Gangrene

2. Procedure:
a. A long posterior flap should be used and the dorsal incision made
approximately 1 cm distal to the MPJ
b. Specifically in the 1st ray, the flexor and extensor tendons and intrinsic
muscles should be sutured over the metatarsal head to prevent retraction of the
sesamoids to stabilize the 1 st ray (groove the met head to hold the tendons)
c. If the sesamoids are not present the prominence on the underside of the 1st
metatarsal head should be removed
d. After the articular cartilage of the 1st metatarsal is removed, the metatarsal
is beveled medially to provide better shape to the final amputation stump
e. Also the lateral third of the 5th metatarsal head should be removed with this
disarticulation
f. If there is a sub-metatarsal callus or ulceration present, a plantar
condylectomy should be performed

Transmetatarsal Amputations
1. Indications:
a. Ischemia
b. Gangrene
c. Osteomyelitis
d. Frostbite
e. Trauma

2. Procedure:
a. Can be single ray resection for the treatment of infection, and is most
successful in the 1 st and 5th ray
b. A racket incision with a dorsal limb
c. The metatarsals should be sectioned at the level of the dorsal skin incision,
in a line curving proximally from the medial side
d. Careful attention should be given to the digital nerves, and must be seen to
retract or a painful "stump" neuroma can develop
e. The plantar surface of the metatarsals should be beveled to allow better
weight distribution
1. Complications: This procedure has fallen into disfavor because muscle
imbalance resulted in severe equinus deformity. A tendo Achilles lengthening
and reattachment of the extensor tendons is recommended to prevent this

2. Procedure: Reattachment of the extensors should be considered. The base of


the second metatarsal should be left in place in line with the medial and lateral
cuboid joints
1. Complications: This amputation has a greater tendency to result in an
equinus deformity because of loss of the insertion of the tibialis anterior tendon
and toe dorsiflexors

2. Procedure: The tibialis anterior tendon must be dissected free as distally as


possible and anchored through or around the neck of the talus. A tendo
Achilles lengthening is a useful adjunct. The intrinsic muscle bellies can be
used to cover the distal stump

Amputation at the Ankle (Syme's, Boyd, Pirogoff)


1. Syme's amputation:
a. Indications: A Syme's through-ankle amputation is indicated in some
congenital deformities, tumors, and management of the diabetic foot. Protective
sensation to the heel pad must be present because this is the end stump. The
heel pad must be stable and adhere to the the end of the tibia to prevent
migration
b. Procedure: The skin is 1st marked 1 cm below the lateral malleolus and 2 cm
below the medial malleolus. These marks are joined by a horizontal skin
incision. A vertical skin incision is then made to joint these points. The
incisions are deepened cutting structures to expose the calcaneus and the neck
of the talus. After the anterior capsule of the ankle joint is sectioned, the foot is
forcibly plantarflexed and the talus pulled forward and downward. The
collaterals are sectioned. The Achilles once exposed is divided near its insertion
near the calcaneus. The calcaneus is dissected free from the soft tissues. A
transverse saw cut is made through the lower end of the tibia and lateral
malleolus. The anterior tibial, posterior tibial and greater saphenous are
ligated. The calcaneal nerve branch of the posterior tibial nerve must be
preserved

2. Boyd amputation: Part of the calcaneus is retained and fused to the lower
end of the tibia

3. Pirogoff amputation: A technique based on rotation of the calcaneus 90°


and a fusion of the calcaneus to the lower end of the tibia

Below Knee Amputation


1. Indications:
a. Vascular disease
b. Trauma
c. Tumor
d. Diabetes complications
e. Infection
2. Procedure: Suitable flaps should be planned. The stump should measure
between 12.5 cm-17.5 cm depending upon body height. The anterior surface of
the tibia should be beveled, and the fibula should be cut 3 cm proximal to this
Chapter 32: Biomechanics
Normal Values
Criteria of Normalcy in the Lower Extremity
Adult Biomechanical Examination
Common Structural Variations
Planes of Motion
Axes of Joint Motion
Angular and Axial Deformities
Anatomy of Gait: Activity of Muscles
Observation of Gait
Subtalar Joint Measurements
Subtalar Joint Function
Midtarsal Joint Function
BIOMECHANICS
Normal Values
1. Quick reference chart of normal biomechanical findings:
Part Birth Position Adult Position
Thigh/Femur/Hip Angle of head and neck of femur=150° At age 6 years=125°
(frontal plane) (angle of Inclination)
At age 6 years=0
(transverse plane) Externally rotated 60° At age 6 years=10°
Femoral torsion (angle declination)=30° Past puberty=100°
Total range of motion=150°
Knee (Birth) (1&1/2-3yr) (3-6yr) (7-puberty) (puberty-18) (over 18) (over 60)
Genu Varum Straight Genu valgum Straight Genu valgum Straight G. valgum
Genu Recurvatum=5-10° At age 6 years =0°
Externally rotated=30° At age 6 years=0°
Frontal plane rotation=5-10° At age 6 years=0-5°
Transverse plane rotation=5-15° At age 6 years=0-5°
Leg/Tibia Varum(approx. 15°) At age 18 years=0-2° varum
Tibial torsion at birth=0° 1 yr=6°, 2-3 yr=10-15°, 5-6 yr=18-23°
Malleolar torsion at birth=0° 5-6 yr=13-18°
Rearfoot At birth= 10° varus approx. At 6 years=2-5° varus
Talocalcaneal angle=30-50° At 6 yrs=25-30°
Calcaneal inclination angle=approx. 14° At 6 yrs=20° approx.
Talar declination angle=approx. 30° At 6 yrs=21° approx.
Calcaneal stance @ 1 yr=5-10° 5 yr=3-8° 8 yr=<2° external rot.
Dorsiflexion=45° approx. After age 18=10V min
Forefoot Varus 10-15° (birth) At age 6=0-2° varus
Metatarsus adductus=15-35° (birth) Adult=15-22°

Criteria of Normalcy in the Lower Extremity


1. Nonwelghtbearing:
a. Malleolar torsion should be 13°-18° externally rotated
b. Ankle joint dorsiflexion should be at least 100 with the knee extended
c. Ankle joint plantarflexion should be at least 20°
d. Total STJ ROM should be 30° (with 20° of inversion and 10° of eversion)
e. When the STJ is in neutral position, the calcaneal bisection should be
parallel to the bisection of the lower 1/3 of the leg and perpendicular to the
supporting surface
f. When the MTJ is maximally pronated and locked the forefoot should be
perpendicular to the calcaneal bisection and parallel to the supporting surface
and the knee on the frontal plane with the hip neutral at 0°
g. There should be equal excursion dorsally and plantarly (5mm) of the 1st ray
from a level equal with the 2nd metatarsal when the STJ is in its neutral
position and MTJ maximally pronated
h. The 5th ray should have equal excursion dorsally and plantarly from a level
equal with the central 3 metatarsals when the STJ is in its neutral position and
MTJ maximally pronated

2. Weightbearing:
a. The distal 1 /3 of the leg should be vertical
b. The knee, ankle, and STJ should lie in transverse planes parallel to the
supporting surfaces
c. The STJ should rest in its neutral position
d. A bisection of the posterior surface of the calcaneus should be vertical
e. The MTJ should be locked in a maximally pronated position about both its
axes
f. The plantar plane of the forefoot and rearfoot should be parallel to each other
and to the supporting surface
g. The central 3 metatarsals should be completely dorsiflexed and describe a
plane parallel to the supporting surface
h. The 1st and 5th metatarsals should describe a common transverse plane
with the central 3 metatarsals

Adult Biomechanical Examination


1. Non-weightbearing assessment:
a. Ankle dorsiflexion:
i. Taken with the patient supine
ii. The minimum ankle joint dorsiflexion that is necessary for normal
ambulation is 10° with the knee extended.
iii. Dorsiflexion is also measured with the knee flexed

NOTE* If the amount of dorsiflexion is less than or equal to 100 whether the
knee is extended or flexed, then there is an osseus or soleus equinus; if the
amount of dorsiflexion is decreased only with extention, then there is a
gastrocnemius equinus

b. Subtalar joint ROM: to calculate neutral position


i. The posterior calcaneus is bisected as is the posterior /distal 1 /3 of the leg
ii. The foot is dorsiflexed to resistance, the MTJ is pronated and locked against
the rearfoot, and the rearfoot is supinated maximally and pronated maximally.
The total ROM is measured with a goniometer (or tractograph) placed parallel to
the bisection of the lower 1/3 of the leg
iii. The STJ neutral position is defined as that point that divides the medial 2/3
of motion from the lateral 1/3 of motion

NOTE* Calculated neutral position= eversion - total ROM-3


For example:
From an examination there is revealed to be: 20° inversion + 10° eversion = 30°
total ROM. So following the formula from above:
Neutral position= eversion (100)- TROM-3 (100)= 0 °
A positive number means a valgus or neutral position, while a negative number
means a varus or neutral position

iv. The minimal STJ ROM is 8-12° for normal ambulation


c. Midtarsal (forefoot):
i. The foot is held by the 5th metatarsal and dorsiflexed to resistance and then
slowly everted until the STJ reaches neutral position
ii. The plantar plane of the rearfoot should be perpendicular to the calcaneal
bisector
d. First ray:
i. Dorsiflex the foot to. resistance and bring the STJ into its neutral position,
stabilize the 2nd-5th metatarsal heads and grasp the 1st metatarsal head and
move it in a dorsal to plantar direction to resistance (this distance should be
about 5 mm or 100 to either side of its resting position and its resting position
should lie on the same transverse plane as the lesser metatarsals)
e. Malleolar torsion:
i. Reference marks are placed on the malleoli, the knee placed on the frontal
plane, and a measurement made with a gravity goniometer
ii. Normal malleolar torsion is 13-18° externally rotated
f. Hip motion (transverse plane): internal and external rotation of the femur
i. The hip joint functions around its neutral position, in a transverse plane,
with the femur rotating the same number of degrees from a neutral position
(same with the hip flexed and extended). Total range of internal and external
rotation must be equal to be considered normal
ii. Mark the patella or the femoral condyles and measure internal and external
rotation with a goniometer
iii. A normal hip rotates 45° internally and 45° externally from a zero degree
starting point. This results in a total ROM of 90° and a neutral position of zero
(from the neutral position, the hip rotates 45° in each direction)

NOTE* Other examples of hip ROM:


a. Internal rotation = 55° external rotation= 35°
Total ROM= 90° 90-2= 45°, 55°(int)-45= 10° therefore, neutral position= 100
internal
(from neutral position, the hip rotates 45° in each direction)
b. Internal rotation=10° external rotation= 40°
Total ROM= 50°, 50-2=25° 10°(int)- 25°=-15° therefore, neutral position= 15°
external
(from neutral position, the hip rotates 25° in each direction)

NOTE* When there is a variance in the degree of neutral position between the
flexed and extended hip position, this indicates soft tissue abnormality limiting
hip rotation. This abnormality is called internal or External Femoral Rotation
Example of a 15° internal femoral position: Internal rotation with hip extended=
45° external rotation with the hip extended= 15°, total ROM=60° therefore,
neutral position with the hip extended=15° internal
Internal rotation with the hip flexed=45° external rotation with the hip
flexed=45°, total ROM=90° therefore neutral position with the hip flexed=0
NOTE* Lack of symmetry in total ROM between the flexed and extended
positions results in asymmetry of the neutral position measurement. This
indicates soft tissue abnormality at the hip.

2. Weight-bearing assessment:
a. Angle and base of gait:
i. The angle and base of gait are necessary to measure NCSP, RCSP and tibial
varum. This allows for standardization and reproducibility of values
ii. The angle of gait is the number of degrees that the foot is deviated from the
line of progression of gait (mid-sagittal plane of the body). Normally the foot is
between 7°-10° abducted from the line of progression
iii. The base of gait is defined as the space between the malleoli during
midstance (normally 1 & 1 /2 inches)

b. Neutral calcaneal stance position (NCSP):


i. Defined as the angular relationship between the calcaneus and the ground
with the STJ in its neutral position and the patient standing in the angle and
base of gait
ii. The calcaneus is bisected, the foot is placed in STJ neutral position, and the
angular relationship between the calcaneus and (perpendicular to) the ground
is assessed

c. Resting or relaxed calcaneal stance position (RCSP): This is measured with


the patient standing in the angle and base of gait with the STJ in a relaxed
position. A measurement is taken of the number of degrees the calcaneal
bisector deviates from perpendicular with the ground

d. Tibial varum: Measured by placing the goniometer on the bisection of the


lower 1 /3 of the leg, with the feet in angle and base of gait, and the foot placed
in the NCSP

e. Gait evaluation: Things to look for


i. What is the position of the calcaneus at heel strike?
ii. Does the foot pronate excessively?
iii. What is the position of the calcaneus and foot at midstance?
iv. Is the heel lifting at the proper time?
v. In propulsion:
• Is it vigorous and active?
• Does the hallux participate?
• Does the foot roll medially?
• Is there a plantarflexion of the ankle?
vi. Is the knee joint on the frontal plane at heel strike, internally deviated, or
externally deviated?
vii. Does the knee joint flex and extend normally or not?
viii. Does the pelvis function around the transverse plane or tilt excessively to
one side?
ix. Does the patient lead with one side excessively?
x. Do the arms swing symmetrically?
xi. Are the trunk and head in the sagittal plane?

Common Structural Variations: Signs and Symptoms


1. Rearfoot varus: A condition in which the calcaneus is inverted relative to
the ground with the STJ in its neutral position. Symptoms:
a. Callus, plantar 4th and 5th metatarsal heads
b. Tailors bunion
c. Haglund's deformity
d. Inversion ankle sprains
e. Adductovarus 4th and 5th hammertoes
f. Mild HAV deformity

NOTE* With rearfoot varus the changes during gait are as follows: The knee
would function fully extended both at heel contact and during midstance. The
leg may be somewhat internally rotated at heel contact, and some internal
rotation would occur if STJ motion were available from this point. During the
latter half of stance phase normal external rotation of the leg occurs. Position of
the heel at contact would be inverted. If the rearfoot varus is not fully
compensated, the heel remains in an inverted position. The STJ is normally
supinated at heel contact

NOTE* Rearfoot varus is not a major pronator. It only allows pronation until the
heel is vertical

2. Rearfoot valgus: A condition in which the calcaneus is everted relative to


the ground with the STJ in its neutral position. Symptoms:
a. Callus, plantar 2nd metatarsal head (occasionally)
b. Fatigue muscle of foot and leg
c. Arch pain
d. HAV deformity

3. Forefoot varus: A structural abnormality in which the plantar plane of the


forefoot is inverted relative to the supporting surface and a vertical bisection of
the posterior surface of the calcaneus when the STJ is in its neutral position
and the MTJ is maximally pronated and locked (rearfoot is normal).
Symptoms:
a. Callus, plantar 2nd, 4th, and/or 5th metatarsal heads
b. Muscle fatigue in foot and leg
c. Tailor's bunion
d. AdductovaruS 4th and 5th hammertoes
e. HAV deformity
f. Plantar fascitis/heel spur syndrome
NOTE* With forefoot varus the changes to the gait cycle are as follows: During
gait the individual with compensated forefoot varus functions with the knee
internally rotating during forefoot loading (contact). The external rotation of the
knee that occurs later in stance phase is decreased. There may be some
reduction in the amount of flexion of the knee during stance phase because of
the pronated position of the foot. The knee would be fully extended at heel
contact and midstance. Following heel contact, normal ankle joint
plantarflexion occurs with smooth contact of the forefoot. At heel contact the
calcaneus is inverted. The heel everts during the stance phase of gait as the
forefoot loads. The STJ will be in a pronated position through the rest of the
stance phase of gait

4. Forefoot valgus (plantarflexed 1st ray with compensation by MTJ


longitudinal axis supination): A structural abnormality in which the plantar
plane. of the forefoot is everted relative to the supporting surface and the
posterior bisection of the rearfoot when the STJ is in its neutral position and
the forefoot is maximally pronated and locked about both MTJ axes (the
rearfoot is normal). Symptoms:
a. Callus, plantar to 1st and 5th metatarsal heads
b. Tibial sesamoiditis
c. Muscle fatigue in foot and leg
d. Flexion contractures of the lesser digits
e. Lateral knee strain

5. Forefoot valgus (plantarflexed 1st ray with compensation by supination


of the STJ and MTJ longitudinal and oblique axes): Symptoms
a. Callus, plantar to the 1st and 5th metatarsal heads
b. Tibial sesamoiditis
c. Flexion contraction of the lesser digits
d. Lateral knee strain
e. Inversion ankle sprains
f. Haglund's deformity
g. Intoe gait seen in children

6. Metatarsus primus elevatus: A structural abnormality in which the 1st ray


has a resting position above the plane of the lesser metatarsals. Symptoms:
a. Callus, plantar 2nd metatarsal and hallux IP joint
b. Fatigue of muscles of the foot and leg
c. Dorsal bunion
d. Hallux limitus/rigidus

7. Equinus deformity (adults): Symptoms:


a. Corn, 5th toe
b. Adductovarus deformities of the 4th and 5th toy
c. Fatigue of the muscles of the foot and leg
d. HAV deformity
e. Plantar fasciitis/heel spur syndrome
f. Neuroma symptoms
g. Contracture of all the digits (extensor substitution)

NOTE* The gait variations seen are as follows: The compensatory changes for
equinus are an early heel-off with the knee slightly flexed throughout the
stance phase of gait. The knee will be somewhat flexed at heel contact, the
flexion might increase during midstance, but it never fully extends at heel lift.
As the leg swings forward, it actually becomes hyperextended relative to the
femur and then begins to flex, and is flexed by the time the heel contacts the
ground. Rather than being an actual compensatory mechanism for equinus,
this seems to protect the knee from abnormal stress in a fully extended
position. In severe equinus the knee may go into hyperextention during the
stance phase of gait (back-knee function). The ankle will be at 90° at heel
contact unless the equinus is severe. Following foot flat, the ankle will then
dons flex to the limit of its ROM, at which time heel-off will occur (the earlier the
heel lift, the sooner the load to the forefoot, the more stress induced
symptomatology there is. The STJ will typically be neutral or slightly inverted
at heel contact.

8. Forefoot supinatus: A relatively fixed supinated position of the forefoot


relative to the rearfoot with the STJ in its neutral position and the forefoot
maximally pronated and locked about both MTJ axes, caused by soft tissue
adaptation. The MTJ ROM is typically decreased secondary to soft tissue
contracture

Planes of Motion
There are three planes of motion in the body, one perpendicular to the other
two, corresponding to the three dimensions in space. The position is one with
the body erect, elbows extended, palms facing forward, _and feet slightly
separated and parallel
1. Sagittal plane: A vertical plane passing through the body from front to back,
dividing it into right and left half. The cardinal plane divides the body into equal
symmetrical halves

2. Frontal plane: A vertical plane passing through the body from side to side,
dividing it into a front and back half. The cardinal frontal plane passes through
the center of gravity dividing the body into equal but asymmetrical halves

3. Transverse plane: This is a horizontal plane, which passes through the body
from side to side and from front to back, dividing it into an upper and lower
half. The cardinal transverse plane passes through the center of gravity and
divides the body into equal but asymmetrical halves

NOTE* It is these planes of motion that are used as coordinates to describe


where the axis of motion lies and what motion will occur around the axis
Axes of Joint Motion
1. Either single or triplane joint motion:
a. An axis can be defined as an invisible line around which all motion takes
place
b. The axis of motion is always perpendicular to the plane in which the motion
takes place (the motion takes place in one plane and the axis lies in the other 2
planes)
c. The frontal-transverse axis is horizontal (its motion is sagittal plane), the
sagittal-transverse axis is horizontal (the motion is frontal plane), and the
frontal-sagittal axis is vertical (the motion is transverse plane)
d. The majority of joints in the lower extremity are hinge-like, therefore when
motion occurs around a joint it occurs by rotation about an axis

2. Position of the joint axes:


a. Single joint motion: The plane of motion is perpendicular to the axis of
motion
b. Triplane joint motion: The amount of motion that can occur in that plane will
depend upon the degree of angularity the joint axis makes to each individual
body plane
NOTE* The greater the amount of degrees (up to 90°) between an axis of motion
and a cardinal body plane, the greater the amount of motion that will occur in
that body plane

3. Motion of specific joints:


a. Hip:
i. Transverse plane axis (controls sagittal plane motion): Controls the
movements of flexion and extension. The range of hip flexion depends upon the
position of the knee:
• When the knee is extended the hip can flex 90° (active flexion) and 140°
(passive flexion)
• With the knee flexed the hip can be flexed 120° (active flexion) and 120°
(passive flexion)
ii. Anteroposterior axis: lies in the sagittal plane and controls the movements of
abduction and adduction
iii. Vertical axis (controls transverse plane motion): controls internal and
external rotation
b. Knee:
i. Frontal plane axis (controls sagittal plane motion): Controls the motion of
flexion and extension
• Active flexion of the knee with the hip joint extended can result in 120° of
flexion, and 140° with the hip joint flexed
• Passive flexion of the knee may attain a range of 160°
ii. Longitudinal plane axis (controls transverse plane motion): Motion of internal
and external rotation
• Active internal rotation has a range of 30°, while active extended rotation
has a range of 40°
c. Ankle: The primary motions at the' ankle joint are flexion and extension in
the sagittal plane
i. The axis of the ankle joint passes lateral, plantar, and posterior to medial,
dorsal, and anterior passing through the tips of the malleoli. It is deviated from
the frontal plane due to malleolar torsion
• The range of dorsiflexion available at the ankle joint is 20-30°, while the
range of plantarflexion is 30-50°

NOTE* Due to slight deviations in the transverse frontal axis of the ankle joint
we will see transverse plane motion of abduction and adduction

d. Subtalar: The axis is oblique to all 3 body planes which allows for triplanar
motion (pronation and supination). Pronation allows the motion of abduction,
eversion, and dorsiflexion. Supination allows for adduction, inversion, and
plantarflexion
i. Subtalar joint axis passes from a plantar, posterior, lateral direction to a
dorsal, anterior, medial direction. It enters the heel and exits through the
dorsomedial surface of the talar neck
ii. The axis is directed 42° from the transverse plane and 16° from the sagittal
plane
iii. As the axis is deviated 42° from the transverse plane positioning it
approximately equidistant from being completely vertical or completely
horizontal, and equal amount of transverse and frontal plane motion will occur
(i.e. Equidistant from both the frontal and transverse planes)
iv. Additionally, as the axis is deviated 16° from the sagittal plane, this allows a
minimal amount of dorsiflexion and plantarflexion
v. The normal values for passive ROM from neutral is 20-35° of inversion to 10-
15° of eversion for a total ROM of 30-35°

e. Midtarsal: Has 2 axes of motion which allow for triplanar motion


i. The longitudinal axis passes 150 from the transverse plane and 9° from the
sagittal plane. It passes dorsal, anterior, medial to plantar, posterior and
lateral. It enters the calcaneus and exits medially through the 1st metatarsal-
cuneiform joint. Since the longitudinal axis is more longitudinal (close to the
transverse-sagittal axis) its permits frontal plane motion of inversion and
eversion. The normal ROM is 4-6°
ii. The oblique axis passes 52° from the transverse plane and 57° from the
sagittal plane. It enters the lateral aspect of the calcaneus (plantarly) and exits
the talonavicular joint (dorsally). The motion that occurs is adduction and
abduction, and dorsiflexion and plantarflexion. Inversion and eversion does
occur, but is minimal

f. First ray: Consists of the 1st metatarsal and medial cuneiform


i. The axis of motion passes dorsal, medial, posterior (enters the medial aspect
of the talonavicular joint) to plantar, lateral, anterior (exits third metatarsal-
lateral cuneiform area
ii. It angles approximately 45° to both the frontal and sagittal planes and
slightly from the transverse plane. It is a triplanar axis, with most of the motion
in the sagittal and frontal planes in a 1:1 ratio
iii. When the first ray dorsiflexes, it inverts and when it plantarflexes it everts.
For every degree the first ray dorsiflexes it also inverts 1°. The first ray axis is
now a pronatory axis. The ankle, STJ, MTJ (longitudinal and oblique) and fifth
ray all possess pronatory-supinatory axes. The 2nd, 3rd, and 4th rays as well
as the digital IPJ's are all uniplanar and produce sagittal plane motion only

NOTE* The digital MPJ's contain 2 axes, a transverse and a vertical. While
the vertical axis allows for transverse plane motion, it is the transverse axis
which allows for dorsiflexion/plantarflexion to occur. The transverse axis
moves in a dorsal-proximal direction with MPJ dorsiflexion. The first MPJ
must allow for 65% of joint dorsiflexion. The joint acts as a ginglymus joint
for the initial 25° of dorsiflexory motion, and acts as arthrodial joint with the
first metatarsal plantarflexing for and MPJ dorsiflexion after the Initial 25°

iv. The average ROM is 5 mm for dorsiflexion and 5mm for plantarflexion

NOTE* The term "degrees of freedom" refers to the number of axes a joint
has. While the ankle and most joints of the foot have one degree of freedom,
the MPJ's and the MTJ each have 2 axes and thus 2 degrees of freedom

g. Fifth ray: Consists of the 5th metatarsal


i. It has an axis of motion from proximal inferior, lateral to distal, superior
medial and angles to all body planes
ii. It angles 20° from the transverse plane and 35° from the sagittal plane,
therefore exhibiting triplane motion. The degree of sagittal and frontal plane
motion is large, and transverse plane motion is minimal
iii. When the 5th ray dorsiflexes it also everts and abducts (pronatory axis)
h. Lesser rays: The central 3 rays along with the IPJ's, have axes that are
parallel with the frontal and transverse planes, therefore will have motion only
in the sagittal plane
Angular and Axial Deformities of the Lower
Extremity in Children
1. Definitions:
a. Torsion: is the twisting of a long bone on its longitudinal axis
b. Rotation is an axial change in the limb due to changes at the joint
c. Anteversion is an anterior axial change in the femur with relation to the head
and neck to the distal condyles (refers to rotational and positional changes of
the limb at the hip)
d. Antetorsion (femoral): Refers to twists in the axis of bone. It is a twisting of
the head and neck of the femur on its own body axis while the portion of the
femur on its own body axis entering Into the knee joint area remains (more or
less) in a fixed position, lining up with the lower leg

NOTE* Normally, antetorsion at birth is approximately 390, and eventually


untwists to 12° by adulthood. If antetorsion is greater than it should be (femur
does not untwist enough), then we have one of the factors influenceing a
pigeontoe gait

NOTE* While the bony femur is untwisting after birth, there is a simultaneous
inward rotation of the thigh (anteversion) in the region of the hip. The bony
external twisting (torsion) is being neutralized by the inward rotation of the
thigh to align the extremity for forward progression

e. Retroversion: Is a lack of normal torsion in the femur, less than normal by


10-12°

2. Axial Deformities:
a. Tibial torsion: Normally 18-23° (external) in the adult. Measured by malleolar
torsion which is 5° less
i. Etiology: Soft tissue origin? osseous origin ?
ii. Treatment: The decision whether to treat or not is determined by the degree
of the deformity (if any), and the age of the child. In the infant with rigid tibial
torsion, serial-above knee casting is effective, followed by maintenance with a
Ganley splint or D-B bar

NOTE* The actual amount of tibial torsion cannot be measured, so therefore we


measure malleolar torsion. From birth to 1 year it measures 0° to 10° (external),
1 year to 5 years it measures 8°to 13 ° (external), and 6 years to adult it
measures 13-180 external torsion

3. Femoral torsion:
a. Anteversion (medial femoral torsion): Is an axial deformity within the neck
and shaft of the femur, resulting in a medial functioning knee
i. Normal femoral alignment at birth is approximately 35-40° of anteversion,
which gradually derotates to the adult alignment of 12-15°
ii. Any delay in this progression should be considered abnormal and treated
owing to positional imbalance caused by soft tissue constraints and the
adaptability of the epiphysis to external forces. It is usually seen is the older
female child and manifested by the "reverse tailor's" sitting position
iii. With this, there is a persistent medial effect of the femoral growth plate
iv. Examination reveals excessive medial angle of the femur associated with a
limited lateral range of motion suggestive of femoral anteversion
v. Treatment:
• In the older child who presents with a knock knee conformation and who,
on examination, shows excessive medial femoral ROM, treatment initially
consists of change in sitting habits
• Later (if necessary) a Ganley femoral derotation splint (this splint is of
benefit in the 4-8 year child with excessive medial ROM)
• In true persistent osseous anteversion in the older child a femoral
derotation osteotomy or epiphyseal stapling may be necessary

NOTE* The treatment approach to femoral problems has been difficult since
any splinting device attached to the feet or legs is nullified once the knee flexes

b. Retroversion (lateral femoral torsion): Felt to be a continuation of lateral


rotated position of the femur in infancy. Lateral ROM is excessive. Often found
in females, usually obese with heavy thighs. The gait effect is knock-knee

4. Angular deformities:
a. Knock knee (genu valgum): An angular problem found more often in the
obese female child, it also may be associated with coxa vara, and is often the
visual result of the proximal axial problems of medial and lateral femoral
torsion. These changes are often associated with torsional changes of the lower
leg and valgus foot deformity. Depression of the lateral tibial condyle in
persistent knock-knee in the older child may also lead to soft tissue strain often
reflected as quadriceps and calf pain. This can lead to DJD in the adult

i. Etiology:
• Physiologic
• Medial femoral torsion
• Lateral femoral torsion
• Anteversion syndrome (Kleiger)
• Trauma: microtrauma (Hueter-Volkman law of epiphyseal response to
pressure states that increased pressure across the epiphyseal plate will
decrease the rate of growth), direct trauma, infection
b. Bowleg (genu varum): May, be associated with coxa valga
i. Types:
• Physiologic: noted at birth
• Rickets: result of vitamin D deficiency causing a disturbance in the
metabolism of phosphorous and calcium
• Osteochondritis deformans tibiae (Blount's disease): an angulation is
noted only at the proximal tibia causing a characteristic "beaking" of the medial
tibial plateau. This disease shows fragmentation of the proximal tibial
metaphysis and may not become evident before 24-30 months of age.
Additionally, infants with a metaphyseal-diaphyseal angle of 110 or greater
(created from the intersection of a bisection of tibial and a bisection of the
femur) will eventually develop Blount's disease

Anatomy of Gait: Phasic Activity of Muscles


The human body has evolved bipedalism as the most efficient means of
locomotion. Bipedal gait is a repetitive sequence of alternating movements of
the lower limbs; one complete sequence represents a gait cycle
1. Subdivisions of the gait cycle:
a. Stance phase (62% of gait): The period of ground contact and weight support
of the foot
i. Contact period (27% of stance and 17% of the entire gait cycle): initiated by
heel strike, the fully dorsiflexed foot Is lowered to the ground as the body moves
from a posterior position to one more directly over the foot. The key locomotor
events of contact are:
• Subtalar joint pronation (for shock absorption) which is normal pronation
• Subtalar supination (closed chain, begins at the end of contact)
• Internal rotation of the leg and femur (concurrent with STJ pronation)
• Full loading of the metatarsus is completed by the end of contact
• Peak vertical ground reactive forces occur for the first time at the end of
contact (the first of the two periods where the ground reactive force rises above
body weight during late contact phase
• The foot functions as a mobile adaptor in contact
• Internal leg rotation initiates STJ pronation
• At the end of contact the STJ begins supinating and it is initiated
primarily by the posterior tibial and is aided somewhat by the other calf
supinators and leg external rotators
NOTE* At the beginning of weightbearing, during the entire contact period, the
STJ pronates in order to make the foot more flexible and, as such, a better
mobile adaptor to variances in terrain.

ii. Midstance period (40% of stance): Foot flat begins midstance, when it singly
bears the body weight (single support), and the alternate foot is in the swing
phase. The key locomotor events of midstance are:
• Conversion of the foot from a mobile adaptor to a rigid lever for
propulsion. The primary condition for a rigid lever to occur is STJ supination
(when this does not occur you have all types of problems (i.e. flatfoot). This
leads to a poor propulsive unit
• A decrease in vertical ground reaction force to about 75% of body weight,
but begins to increase again prior to heel lift
• Continued external leg rotation
• The contralateral limb is in swing phase
• STJ supination as a result of external leg rotation and the supinating calf
muscles (especially the posterior tibial and swinging limb). The foot passes
through STJ neutral shortly before heel lift. From this point on, the STJ is
supinated

NOTE* During midstance the STJ is still pronated but starts supinating to
convert the foot to a rigid lever

iii. Propulsive period (33% of stance): Continuation of the forward shifting body
results in heel lift and the initiation of the propulsive period, whereby the
weight is shifted to the forefoot and at the end, opposite foot regains contact
with the ground by heel strike. The key locomotor events of propulsion are:
• Continued STJ supination which increases skeletal rigidity and creates a
rigid lever
• Continued external leg rotation
• Second peak vertical ground reaction force (about 125% of body weight)
• Shift of forefoot weightbearing from lateral to medial
• The opposite foot begins to bear weight just after lateral to medial shift
and by toe-off the opposite foot is in full contact phase

b. Swing phase (38% of gait): That portion of the gait cycle when the foot is off
the ground. During swing the foot pronates first and then supinates. The key
events of swing phase are:
• During swing, the foot is transported from one step to the next
• The leg continues to externally rotate momentarily after toe-off. Then it
begins an internal rotation during the swing
• Pronation of the foot to aid ground clearance and then resupination to
prepare for contact
• Ankle joint dorsiflexion, and hip/knee flexion to shorten the length of the
leg (there would be a tremendous amount of pelvic motion during gait if there
were no mechanisms to flex and shorten the leg length)

c. Double support: Both feet are in ground contact at the beginning and end of
each stance phase in the walking gait. Both feet are on the ground 25% of the
gait cycle (0-12% and then again from 50-62% of the entire gait cycle)

NOTE* In running gaits, the swing phase proportionally increases in duration,


an instead of stance phase overlap, the limbs overlap in off-ground motion in a
period of float; i.e. there is no double support phase during running, also there
is an airborne phase with no ground contact and there is never more than one
foot in ground contact at one time

2. Hip and knee motion:


a. Motions in gait: At toe off, both the hips extended and knee is flexed while
the limb Is posterior to the body. At heel strike the hip is flexed, the knee
extended and the limb forms a lever for the movement of the body onto the heel.
During stance the limb extends at the hip as the body moves over the
stationary foot. Internal rotation of the limb continues from swing phase into
the contact period of stance. In midstance, the limb begins external rotation as
the opposite side of the pelvis and hip shift forward and medially rotate during
swing phase of the opposite limb b. Swing phase muscle action:
i. Hip flexors:
• The primary flexor of the hip is the iliopsoas which is activated shortly
after toe off: it is comprised of the iliacus and the psoas major to form a
common muscle belly
ii. Hip stabilizers and rotators: In swing phase, the hip on- the swing side has
lost support from the ground; it needs to shift body weight medially to the
stance limb and to resist downward tilt. The unsupported hip, therefore, needs
greater muscular stabilization in swing phase than it does when supported in
stance
• Transverse stabilization of the unsupported hip is produced by the
erector spinae on the swing side
• The hip abductors (gluteus medius and minimus) pull down the pelvis on
the supported side, and lift and level the pelvis op the swing side. Their primary
function on the swing side is to abduct the-thigh
• The medial hip adductors (adductor longus, adductor brevis, adductor
magnus, and pectineus), stabilize the hip and thigh (with the abductors) in both
stance and swing, and they contribute to flexion, extension, and internal and
external rotation of the hip
iii. Knee extensors:
• The quadriceps femoris (rectus femoris, vastus medialis, vastus lateralis,
vastus intermedius) is a biarthrodial muscle acting on hip flexion and knee
extension at the same time. The quadriceps is the only extensor of the leg, and
in the walking gait, extends the leg near the end of swing (this extention is
decelerated by the hamstrings (semimembranosis, semitendinosis, biceps
femoris)
iv. Knee flexors and hip extensors:
• The hamstrings, prior to heel strike, exert a flexor force on the leg,
decelerating ongoing extension, and extend the hip in early stance. The
semimembranosus and semitendinosus medially and biceps femoris laterally,
are synergists of each other in stabilizing rotation at the knee at heel contact.
The hamstrings also stabilize both the hip and knee joints at heel strike

NOTE* The gluteus maximus (the largest and most powerful of lower extremity
muscles) contributes to hip extensor stabilization

c. Stance phase muscle action: In the contact period, those muscle groups that
decelerated and stabilized these joints at heel strike continue to perform these
functions until the foot achieves full foot support. At midstance most of the
weight-support functions are managed by bone and ligament, requiring little
primary muscular action. As. the body moves over the standing foot, the limb
externally rotates in all segments down to the ankle. This rotation is a reaction
both to subtalar supination and to medial rotation of the opposite limb swing
phase. After the stance limb passes beneath the hip joint, the hamstrings are
again activated to help extend it

3. Ankle motion:
a. Motions in gait: Functions on a pronatory/supinatory joint axis with the
majority of motion consisting of sagittal plane dorsiflexion and plantarflexion.
Any rotation of the leg carries the talus with it when the foot is off the ground,
and any subtalar action involving the talus affects leg rotation when the foot is
weight bearing
b. Dorsiflexors: Dorsiflexion occurs from late contact to the propulsive period,
whose action is the result of kinetic forces of the body moving over the limb.
The 4 anterior crural tendons pass anterior to the transverse axis of the ankle
joint, and therefore dorsiflex the ankle. Distally, the tendons pass to either side
of the subtalar and midtarsal joint axes where the tendon passing medially
(tibialis anterior) exerts an inverting force, supinating the STJ and MRJ axes.
The tendons passing laterally (EDL, peroneus tertius) exert an everting force
(these apply a pronatory force to the STJ and MTJ longitudinal axes). The
anterior crurals function mainly during swing to dorsiflex the foot to clear the
ground. The toe extensors begin to act at the end of the propulsive. period,
when they help stabilize the toes. They are joined by the tibialis anterior and
peroneus tertius, and together lift. the foot at toe off. After heel strike, they
decelerate the foot while it is lowered to the ground.

NOTE* The tibialis anterior is the main dorsiflexor because of its insertion at
the base of the first ray, and acts together with the EHL in elevating the first
ray and hallux above the groung during swing (its insertion is also medial to
the STJ and MTJ axes making it an effective supinator, and invertor during
dorsiflexion)
NOTE* Summary of anterior aural function: They become active at toe-off to
dorsifex the foot. They remain active throughout swing, and show a peak
activity at heel strike as they decelerate the forefoot as it strikes the ground.
They are active for the first 10% of stance

c. Plantarflexors: Consist of 6 muscles divided into superficial and deep groups


(superficial: gastrocnemius, soleus, plantaris) (deep: tibialis posterior, FDL,
FHL). The tendo Achilles passes 2 cm posterior to the STJ axis giving it
more leverage to act on the ankle than any other muscle (supinates the ankle).
The combined action of the triceps surae (2 heads of the gastroc and the
soleus.) together with the deep plantarflexors, produce 4 times as much power
as the combined dorsiflexors, and mainly act on the fully loaded limb in single
support. The tibialis posterior is the strongest invertor and adductor of the foot.
The plantarflexors begin to function in the contact period and continue through
midstance and into propulsion. They act to decelerate the momentum of the
body as it moves across the fixed foot dorsiflexing the joint. The gastrocnemius
also resists hyperextension of the knee prior to heel lift

NOTE* Summary of posterior aural function: The, triceps surae are active
during the middle of stance phase. They begin to fire during contact and
terminate during propulsion in order to achieve heel-off

4. Tarsal joint motion:


a. Motions in gait: The definitive feature of the contact period is progressive
loading of the foot from its initial contact at the heel, along the lateral border, to
full foot support. At this stage, the foot is undergoing pronation, and the leg
continues to rotate internally. These joints are characterized by their flexibility
(if the MTJ is unlocked, the 1 st ray is not stabilized). By midstance, the weight
is progressively more medially distributed as the body center of gravity
continues to move over it. From the lateral border of the foot, weight is shifted
medially along the metatarsal heads. The stresses on the foot now require
rigidity in tarsal structure to transfer weight to the forefoot. As the STJ begins
to supinate, the leg, with the hip and the thigh, externally rotate. This
supinatory process involves the windless effect as the foot proceeds through
propulsion.
b. Tarsal stabilizers: In midstance, the soleus, gastrocnemius, and tibialis
posterior are the prime movers to supinate the foot with concurrent external
rotation of the leg. The peroneus brevis and longus (lateral crural ms.) are
inactive in the contact period and only begin to act well into midstance and
early propulsion. The peroneus brevis is the prime evertor (pronator) of the foot.
The peroneus longus exerts a force on the first ray, pressing the first metatarsal
to the ground as well as on the intervening tarsals. The combined result is a
compact tarsus. The continuing secondary action of the soleus and supinating
tendons from the posterior compartment, maintain pressure on the lateral
border of the foot. This maintains a fixed position of the cuboid, which acts as a
pulley for the peroneus longus tendon. At the end of midstance, the heel is
lifted, and the cuboid is released, permitting the peroneus longus to act more
directly on the 1st ray as body weight is lifted from the lateral metatarsal heads

5. Forefoot motion:
a. Motions in gait: Stability in the forefoot begins to develop in the latter part of
midstance when the foot is supinating, the MTJ is locked, and the tarsal and
metatarsal bones are in a close packed position. At heel lift, the foot forms a
lever from the plantarflexing ankle joint to the met heads that form its fulcrum.
The MPJ's are transversely stabilized while the toes are firmly braced against
the ground forming a stable platform for the fulcrum. The loaded extension at
these joints activates the windless effect of the plantar fascia, which is
stretched by its attachment to the MPJ and hallux

b. Extrinsic muscle action. The FHL continues to act almost until toe off as the
hallux is the last part of the foot to leave the ground and requires a longer
period of stabilization than do the lesser toes. The tibialis posterior and the
peroneals compress the metatarsal bases as well as the tarsal bones. The
peroneus longus stabilizes the 1 st ray through the propulsive period

c. Intrinsic muscle action: The main function of the intrinsic muscles of the foot
is to transversely and axially stabilize the digits against the metatarsal heads
and against ground reaction forces. All intrinsic muscles begin to contract in
midstance and most continue throughout the propulsive period. Transverse
stabilization of the toes is accomplished by the plantar and dorsal interossei.
The dorsal interossei are bipennate and 4 in number, originating from the
corresponding adjacent sides of their respective intermetatarsal spaces. The
first dorsal interosseous attaches medially into the base of the proximal
phalanx of the second toe while the 2nd, 3rd, and 4th attach laterally into digits
2, 3, and 4. The plantar interossei are unipennate. These three muscles attach
medially into the 3rd, 4th, and 5th digits and originate from the medial aspect
of their respective metatarsals. Their combined action resists displacement of
the toes to either side. Transverse stability of the hallux is provided by the
abductor hallucis on one side and adductor hallucis on the other side. The
abductor digiti minimi mimics the function and attachment of the dorsal
interosseus on the lateral side of the 5th toe. The FHB, FDB and flexor digiti
minimi brevis act synchronously with the long flexors, stabilizing the toes
against the ground. The flexor digiti minimi brevis attaches laterally into the
5th toe, as a unipennate muscle and functions with the interossei to provide
transverse plane digital stability. The interossei are stance phase muscles and
function to plantarflex the MPJ's against the retrograde dorsiflexing buckling
force that accompanies the FDL and FDB contraction. The axial tension of the
FDL is aided by the quadratus plantae. The lumbricals are 4 muscles
originating from the medial aspect of the corresponding FDL slip, and attaching
medially into the base of the extensor hood of the lesser toes, as they pass
plantar to the deep transverse metatarsal ligament. These have been described
as swing phase muscles, stabilizing the MPJ's plantarly while assisting in
extending the PIPJ's and DIPJ's, limiting excessive swing phase contraction.
This provides a stable insertion to allow the EDL to be an important dorsiflexor
of the ankle during swing phase of gait
NOTE* The greatest combined effect of all these muscles is achieved in
conjunction with the extensor expansion mechanism, which links the IPJ's &
the MPJ's in each toe so that tension on the long extensor extends a row of
digital joints at a time

NOTE* Summary: The abductor digiti minimi and EDB become active at about
30% of stance, the FHB, abductor hallucis, and FDB become active at about
40%, 50% and 60% of stance respectively, and the interossei become active at
about 25% of stance. Activity of all these muscles ceases near toe-off

d. Muscles acting on the 1st ray and hallux: During supination in the
midstance period, the peroneus longus is uniquely significant in plantarflexing
the 1st ray. The FHL assists in supination during the earlier part of midstance,
plantar stabilization of the, metatarsal head together with the action of the
peroneus longus, and stabilization of the hallux in propulsion

6. Summary of joint motion during gait:


a. Hip joint motion-sagittal plane:
i. Contact: hip extends from flexed position
ii. Midstance: hip continues to extend
iii. Propulsion: hip flexion

b. Hip joint motion-transverse plane:


i. Contact: internal rotation of the thigh in the pelvis, corresponding to internal
leg rotation with STJ pronation
ii. Midstance and propulsion: external rotation of the thigh on the pelvis

c. Knee joint motion sagittal plane:


i. Contact: flexion (the major shock absorbing mechanism of the body)
ii. Midstance: body weight passes over the knee, so there is extension
iii. Propulsion: the knee flexes again for pushoff

d. Knee joint motion-transverse plane: Corresponds to supination and


pronation in the STJ
i. Contact: with STJ pronation comes internal rotation of the tibia
ii. Stance: external rotation of the tibia on the femur

e. Ankle joint motion-sagittal plane:


i. Contact: plantarflexion until midcontact
ii. Late contact to midstance: dorsiflexion
iii. Propulsion: plantarflexion

f. Subtalar joint motion:


i. Contact: pronation
ii. Midstance and propulsion: supination

g. Midtarsal joint motion: There are 2 separate axes


Longitudinal axis Oblique axis
i. Contact: Supinated Pronated
ii. Midstance: Fully pronated by heel lift* Pronated
Ill. Propulsion: Remains pronated and locked Supinates

NOTE* Longitudinal axis pronation during midstance* is very important for


normal propulsion. Negative plaster casts are taken of the feet with the
longitudinal axis of the MTJ in pronation

Observation of Gait
1. Stance phase:
a. Posterior view:
i. Contact:
• Heel contact is inverted but rapidly everts
• Contact is slightly lateral to the midline of the heel
i. Midstance:
Posterior bisection of the posterior of the heel goes from everted to vertical
i. Propulsion:
• Posterior bisection of the heel inverts as heel lifts

b. Lateral view:
i. Contact:
• Posterior portion of the heel strikes the ground with the foot dorsiflexed
on the ankle
• Plantarflexion of the ankle begins slightly after contact
• At contact the knee is extended and flexes rapidly for shock absorption
ii. Midstance:
• Ankle dorsiflexion of 5°-10° as the body weight passes over the planted
foot
• Knee returns to full extension
• Late midstance heel lifts as the trunk passes over the planted foot,
literally peeling the heel up from the floor
iii. Propulsion:
• Ankle plantarflexes to facilitate toe-off
• Knee flexion as the trunk advances further

c. Anterior view:
i. Contact:
• Forefoot is markedly inverted as the heel contacts the ground
• Leg is slightly inverted
ii. Midstance:
• Forefoot has everted bringing the metatarsals to the ground. The forefoot
is loaded 5-4-3-1-2 or 5-4-3-2-1- depending upon the metatarsal length and
muscle functioning
• Leg is slightly internally rotated
iii. Propulsion:
• Marked dorsiflexion of the MPJ's
• Lateral digits lift-off first
• Body weight passes through the center of the hallux
• Leg externally rotated

2. Swing phase:
a. Lateral view:
i. Trunk muscles advance the leg forward
ii. Ankle dorsiflexion to decrease leg length
iii. Hip and knee flexion to assist in shortening

Subtalar Joint Measurements


1. Open kinetic chain measurement and neutral position calculation: In
the non-weightbearing patient, the bisection of the posterior distal 1 /3 of the
leg is the point from which calcaneal inversion and eversion are measured.
a. Example 1: The calcaneus can evert 80 from the leg bisection and invert 22°
from the lebisction. The total STJ ROM is 30°

To find the neutral position of this STJ we need to find the point from which
there is twice as much supination as there is pronation. In this case the
neutral position is 2° inverted, as from this point there is 100 of pronation and
20° of supination available

Total STJ ROM X 2= Inversion from neutral position


3
Inversion from the leg -MINUS- Inversion from neutral= Neutral position
30°(total ROM) X 2= 20° 22°(inversion from leg)- 20°= 20 varus
3
b. Example 2: Calcaneal inversion= 32°, calcaneal eversion= 7°, total ROM= 39°
39°+ 3 x 2= 26° inversion from neutral, 32°-26°=6° varus

NOTE* Another method for calculation is: Total ROM- Eversion = Neutral
3
If the resulting number is (+), then there is a varus or neutral position
If the resulting number is (-), then the resulting number is valgus or neutral

2. Closed kinetic chain measurement and neutral position calculation:


a. Example 1:
Maximum calcaneal inversion 12° (right), 150 (left), maximum calcaneal
eversion 6° (right), 3° (left), tibial varum 1 ° bilaterally
i. To calculate neutral position: Total ROM 18° bilaterally
18+ 3 x 2= 12° 12°- 12°= 00 (right), 15°- 12°= 30 varus (left)

ii. To calculate NCSP: Add the tibial varum component to the neutral position
measurement
Tibial varum is 1 ° (left) + 30 varus (left)= 40 rearfoot varus (left)
Tibial varum is 1 ° (right) + 0° (right)= 1° rearfoot varus (right)

iii. To calculate the RCSP: Since these values indicate a rearfoot varus
component, this individual will compensate at the STJ to bring the calcaneus
perpendicular to the ground, by using all STJ pronatory ROM that it needs (the
pronatory ROM of the STJ is 1/3 the total ROM or 6°).
On the right the NCSP= 1° varus so the RCSP= 0°
On the left the NCSP= 40 varus, so the RCSP= 0° (using 2/3 available pronatory
ROM)

b. Example 2:
Maximum calcaneal inversion 16° (right), 15° (left), maximum calcaneal
eversion 2° (right), 3° (left), tibial varum 30 (right), 2° (left)
i. Calculate the neutral STJ position:
Total ROM (right)= 18° Total ROM (left)= 18°
18-3 x 2= 12° (bilateral) inversion from neutral
16°- 12°= 40 varus (right), 15°- 12°= 30 varus (left) neutral position

ii. To calculate the NCSP:


Tibial varum 3° (right) + 4° varus neutral (right)= 7° rearfoot varus NCSP (right)
Tibial varum 2° (left) + 3° varus neutral (left)= 50 rearfoot varus NCSP (left)

iii. To calculate the RCSP: Know that 1/3 the total STJ ROM is 6°. 7° rearfoot
varus (right)- 6° (1/3 available pronation)= 1° RCSP (right)
5° rearfoot varus (left)- 6° (1/3 available pronation)= 0° RCSP (the STJ still has
1° more of available compensatory motion left)
NOTE* Forefoot varus is compensated (mostly) by STJ pronation, and
minimally by some MTJ pronation. If the amount of forefoot varus is 3° or less,
the STJ will only compensate that sped lc number of degrees. lf, however, the
forefoot varus is greater than 3° the STJ will (usually) maximally pronate to the
end of its ROM. Therefore it will pronate more than the number of degrees
required to bring the forefoot's medial surface into contact with the ground. The
reason the STJ maximally pronates with a forefoot varus deformity greater
than 3° is that once the calcaneus is everted more than 3° the force of the
body's weight pushes it to the end of the STJ's pronatory ROM. If, however, the
STJ cannot completely compensate the forefoot varus deformity, then and only
then will the MTJ pronate to help with the compensation (on the longitudinal
axis), leading to first ray dorsiflexion and inversion

NOTE* In a rearfoot valgus greater than 2° the body weight on the everted
calcaneus will cause the STJ to pronate to the end of its ROM. A rearfoot
valgus of less than 2° does not change the STJ position from the NCSP. If a
greater than 10° rearfoot valgus exists, the head of the talus will usually
plantarflex toward the ground before the STJ completely pronates. While this
produces a severe flatfoot, the STJ may not be pronated to the end of its ROM

Subtalar Joint Function


During the gait cycle, the STJ functions during the weightbearing (closed
kinetic chain) and nonweightbearing portions (open kinetic chain).
1. Open kinetic chain (OKC):
a. During the 1st half of the swing phase, the STJ pronates, and during the last
half of swing the STJ supinates
b. In OKC function, the STJ pronatory and supinatory components are
exhibited exclusively by the calcaneus (with OKC pronation, the calcaneus
abducts, everts, and dorsiflexes)
c. In OKC motion, the calcaneus moves around the talus, which functions as
an immobile extension of the leg

2. Closed kinetic chain (CKC):


a. In CKC the STJ motion, the calcaneus and talus both move, the calcaneus
moves only in the frontal plane (inversion and eversion), and the talus moves in
the transverse and sagittal planes

NOTE* The bones of the STJ move around the STJ's axis of motion, and if any
motion takes place in a bone which is proximal to that axis, the motion will be
in the opposite direction of the named major motion.

b. In CKC STJ pronation, the calcaneus will still evert, but the talus will
plantarflex and adduct

c. In CKC STJ supination, the calcaneus will invert and the talus dorsiflex and
abduct (the talus abducts and dorsiflexes because it is proximal to the STJ joint
axis) (transverse plane talar excursion reflects the transverse plane movement
of the leg)

NOTE* During the contact period (STJ pronation) the calcaneus is everting,
while the talus is plantarflexing and adducting. During midstance and
propulsion the calcaneus is inverting while the talus abducts and dorsiflexes
(STJ supination)

d. Internal rotation of the tibia is associated with CKC STJ pronation, and the
converse is true with CKC STJ supination

e. The 2 major functions of CKC pronation are shock absorption and adapting
to uneven terrain

3. Measurement of STJ motion:


a. There is really no good way to measure STJ motion in all 3 planes, therefore,
the frontal plane motion of the calcaneus is used as an index of STJ motion (a
bone distal to the STJ axis)
b. From the STJ neutral position, the normal foot can supinate twice as
much as it can pronate. The average total ROM for the STJ is about 30° of
calcaneal frontal plane motion (minimum normal STJ ROM is 8-12° for normal
ambulation)

Midtarsal Joint Function


Although triplane motion occurs about both the MTJ axes, some planes of
motion are so small as to be clinically insignificant
1. MTJ function:
a. Motion about the longitudinal axis will occur primarily in the frontal plane
(inversion and eversion)
b. The oblique axis allows primarily for sagittal and transverse plane motion

NOTE* Since 2 planes of motion occur about the MTJ oblique axis, it is
necessary to know which motions are coupled. As the axis is a pronatory/
supinatory axis, the following occurs by necessity:
a. With plantarflexion: adduction also occurs
b. With dorsiflexion: abduction also occurs

NOTE* The MTJ's total ROM is dependent upon the STJ's position. The axes of
the articular facets are just about parallel when the STJ is maximally
pronated. This allows for a certain congruity to the 2 joints (T-N and C C joints).
As the STJ goes from a maximally pronated position toward a more supinated
position, the axis of the 2 joints progressively diverge from one another,
congruity is lost, and with it ROM decreases
c. The MTJ longitudinal axis has an average ROM of 4°-6° (ROM of the oblique
axis is unknown)

d. When the STJ is maximally pronated, the MTJ's ROM is increased and the
forefoot becomes mobile. When the STJ is maximally supinated the MTJ's ROM
is decreased and the forefoot inverts with the rearfoot
Chapter 33: Anatomy
Neuroanatomy
Angiology
Myology
Arthrology
ANATOMY
Neuroanatomy
1. Femoral nerve:
a. Formed from the ventral rami of lumbar nerve roots (L2, L3, and L4)
b. Divides into anterior and posterior division (the posterior division gives rise
to the saphenous nerve which supplies cutaneous innervation to the medial leg
and foot)

2. Sciatic nerve:
a. Formed from the ventral rami of lumbar and sacral nerve roots (L4, L5, S1,
S2, and S3)
b. The largest nerve of the body
c. Divides into its terminal branches, the tibial and common peroneal nerve
d. Supplies all musculature of the leg and foot (except 'for that supplied by the
saphenous nerve)

3. Common peroneal nerve (L4, L5, S1, S2):


a. One of 2 terminal branches of the sciatic nerve, and winds superficially
around the neck of the fibula
b. Gives off the sural communicating near the head of the fibula, which joins
with a communicating branch of the tibial nerve to form the sural nerve
c. Terminal Branches:
i. Deep peroneal nerve: This nerve runs with the anterior tibial artery (laterally)
in the lower leg to give muscular innervation to the EDL, EHL, TA, peroneus
tertius, EDB, and ends in the first interspace
ii. Superficial peroneal nerve: In the distal 1 /3 of the leg, the nerve pierces the
anterolateral intermuscular septum and deep fascia to become cutaneous, and
divides into 2 branches, the medial and intermediate dorsal cutaneous nerves
before reaching the ankle. This gives innervation to the peroneus longus and
brevis

4. Tibial nerve (L4, L5, S1, S2, S3):


a. The larger of the 2 branches of the sciatic nerve
b. Travels between the two heads of the gastrocnemius to leave the popliteal
fossa, descends along the back of the leg with the posterior tibial artery and
veins
c. The nerve lies medial to the blood vessels in the upper 1 /3 of the leg and
lateral to them in the lower 1 /3 of the leg d. The nerve supplies the
gastrocnemius, popliteus, tibialis posterior, FHL, and FDL
e. Terminates into medial and lateral plantar nerves at the level of the lacinate
ligament (prior to this, gives off medial calcaneal branch)

5. Sural nerve (L5, S1,S2):


a. Formed by union of medial sural cutaneous (tibial) and sural communicating
branch of the lateral sural cutaneous of the common peroneal
b. Pierces the deep fascia to become cutaneous at the upper mid-calf region,
travels distally near the lateral side of the achilles with the small saphenous
vein, and passes forward below the lateral malleolus
e. Becomes the lateral dorsal cutaneous nerve on the lateral aspect of the foot

6. Saphenous nerve (L3,L4):


a. This is the only nerve in the foot that originates from the lumbar plexus
(comes off the posterior division of the femoral nerve and is its longest branch)
b. Travels with the greater saphenous vein along the medial side of the leg
c. It divides into 2 branches in the lower 1 /3 of the leg giving sensory
innervation to the skin of the medial ankle and medial side of the foot (it may
reach distally as far as the metatarsophalangeal joint)

7. Deep peroneal nerve (L4, L5, S1):


a. Travels with the anterior tibial artery and lies lateral to it in the foot
b. Divides into its terminal branches and passes into the dorsum of the foot in
the middle compartment
c. Its terminal branches are the lateral terminal nerve (passes across the tarsal
area to innervate the EHB) and the medial terminal nerve (to the 1st interspace)
where this divides into 2 dorsal digital nerves (2nd dorsal digital proper nerve &
3rd dorsal digital proper nerve)

8. Superficial peroneal nerve (L5, S1, S2):


a. Primarily a cutaneous nerve with 2 terminal branches, the medial and
intermediate dorsal cutaneous nerves
b. Medial dorsal cutaneous nerve divides into a medial and lateral branch.
These supply the medial side of the hallux, lateral side of the 2nd toe, and the
medial side of the 3rd toe.
c. Intermediate dorsal cutaneous nerve divides into medial and lateral
branches. These supply the lateral side of the 3rd toe, medial side of the 4th
toe, lateral aspect of the 4th toe, and the medial aspect of the 5th toe

9. Lateral dorsal cutaneous nerve (S1, S2):


a. A continuation of the sural nerve below the ankle, and communicates with
the intermediate dorsal cutaneous nerve, and divides into 2 branches (one to
the lateral side of the 5th toe, and the other anastomoses with branch of the
superficial peroneal nerve)
10. Medial calcaneal nerve (S1, S2):
a. Last branch given off by the tibial nerve before dividing into medial and
lateral plantar nerves
b. It perforates the lacinate ligament and divides into anterior and posterior
branches (posterior branch supplies the heel)
c. The anterior branch continues distally into the plantar aspect of the foot,
where it anastomoses with the lateral calcaneal branch of the sural nerve
laterally, anastomoses with the saphenous nerve anteromedially and
anastomoses with the lateral and medial plantar nerves distally

11. Lateral Calcaneal Nerve (S1, S2):


a. Given off before the sural nerve goes around the lateral malleolus
b. Supplies the cutaneous innervation to the lateral aspect of the heel and
sensory innervation to the lateral proximal aspect of the plantar surface of
the foot
12. Medial plantar nerve (S3):
a. Larger of the 2 terminal branches of the tibial nerve running in the 3rd
compartment of the lacinate ligament
b. Enters the foot at the porta pedis and emerges from between the abductor
hallucis and FDL
c. Supplies the cutaneous innervation to the medial plantar aspect of the
midfoot, abductor hallucis, FDB
d. Its terminal branches are the 1st plantar digital nerve and 1st, 2nd and 3rd
common plantar digital nerves
]

13. Lateral plantar nerve (S2, S3):


a. Smaller of the 2 terminal branches of the tibial nerve, lying posterior to the
posterior tibial artery, eventually lying medial to the lateral plantar artery with
which it travels into the plantar aspect of the foot b. The nerve passes through
the porta pedis lying between the FDB and quadratus plantae. Between the
FDB and abductor digiti minimi the nerve divides into its 2 terminal branches,
superficial and deep c. Superficial branch comes off the main branch at the
level of the 5th metatarsal and supplies the flexor digiti minimi brevis,
opponens digiti
minimi (when present), 3rd plantar interosseous, and 4th dorsal interosseous
d. Deep branch follows along with the lateral plantar artery between the 3rd
and 4th muscle layers (deep to the adductor hallucis and superficial to the
plantar interossei) and supplies the transverse/oblique heads of the adductor
hallucis, lateral 2 (or 3) lumbricales, and interosseous muscles of the 2nd and
3rd intermetatarsal space (2nd and 3rd dorsal, 1st and 2nd plantar), and 1st
dorsal interosseous

NOTE* The 1st and 2nd dorsal interossei receive additional innervation from
the deep peroneal nerve

Angiology
1. The arteries: Blood flows from the left ventricle of the heart, through the
ascending aorta, aortic arch, descending aorta, thoracic aorta, and abdominal
aorta. In the lower abdomen, the aorta divides into paired common iliac
arteries. Each common iliac artery divides distally into internal and external
iliac arteries. The external iliac artery becomes the femoral artery as it passes
under the inguinal ligament,
a. Femoral artery: Courses through the medial thigh through the femoral
triangle into the adductor canal
b. Deep femoral (profunda femoris): The last branch of the femoral artery before
leaving the femoral triangle
c. Popliteal: When the deep femoral crosses over the medial femur it then
becomes the popliteal and continues to the lower border of the popliteus where
it divides into anterior and posterior tibial (before its bifurcation it gives off a
sural branch which is the only source of blood to the gastrocnemius)
d. Anterior tibial: Branches off the popliteal artery at the level of the soleal line,
travels into the anterior compartment of the leg, lies between the tibialis
anterior and EDL muscle bellies in the proximal 1/3 of the leg. Immediately
above the the ankle joint the EHL tendon crosses over the anterior tibial artery
and at this level the artery is found between the EHL and EDL. The anterior
tibial artery can terminate as the dorsalis pedis or becomes insignificant before
reaching the ankle joint, and if this happens, the dorsalis pedis will be absent
or arise as a branch of the perforating peroneal artery
i. Branches of anterior tibial artery
• Posterior recurrent tibial
• Anterior recurrent tibial
• Muscular branches (to the tibialis anterior, EDL, EHL, peroneus tertius,
and muscles of the deep posterior compartment of the leg)
• Anterior medial malleolar (superficial and deep branches)
• Anterior lateral malleolar (transverse portion meets with branches from
perforating peroneal; descending portion anastomoses with the descending
retromalleolar branch of the peroneal artery)
e. Dorsalis pedis: Begins as it crosses the ankle joint continuous with the
anterior tibia] artery, and gives off branches
i. Branches of the dorsalis pedis
• Medial branches (medial tarsal arteries)
• Lateral branches (artery to the sinus tarsi, lateral tarsal, arcuate, anterior
perforating, 4th dorsal metatarsal, and 1 st dorsal metatarsal artery)
f. Posterior tibial: Branches off the popliteal artery at the level of the soleal line,
remains within the posterior compartment of the leg, runs superficial to the
tibialis posterior proximally and over the FDL distally as it descends into the
medial ankle into the 3rd compartment of the flexor retinaculum, and
bifurcates into medial and lateral plantar arteries while in the 3rd compartment
deep to the muscle belly of the abductor hallucis
i. Branches of the posterior tibial artery
• Circumflex fibular
• Peroneal (gives off muscular branches, perforating branch,
communicating branch, posterior peroneal, anterolateral transverse branch,
collateral branch, and recurrent calcaneal branches)
• Nutrient artery (supplies the tibia)
NOTE* This is the largest nutrient artery in the body

• Muscular branches (to the soleus, FDL, FHL, and tibialis posterior)
• Communicating artery
• Posterior Medial malleolar
• Medial calcaneal
• Artery of the tarsal canal
g. Medial plantar: Smaller of the 2 terminal branches of the posterior tibial and
gives rise to 2 branches (supplies the abductor hallucis, FDB, 1st dorsal
interosseous)
i. Branches of the medial plantar artery:
• Superficial branch (gives rise to medial marginal plantar artery of the
hallux and the common plantar digital artery)
• Deep branch (divides into a medial and lateral branch)

h. Lateral plantar: Travels into the intermediate compartment laterally and


distally between the quadratus plantae and FDB muscle bellies, and terminates
by connecting the perforating branch of the dorsalis pedis

NOTE' This artery serves as a landmark, separating the 3rd and 4th muscle
layers of the plantar aspect of the foot

i. Branches of the lateral plantar artery:


• 3 posterior/perforating arteries
• 4 plantar metatarsal arteries
• Proper digital arteries

2. The veins: Two sets of veins are found in the lower extremity, superficial
and deep. Most arteries are associated with a pair of deep veins (venae
commitantes). Valves are present in the veins, but are more numerous in the
deep veins. The valves are located at the termination of the great and small
saphenous veins, above and below the knee and ankle joints, and in the leg.
Perforating veins connect superficial veins to deep veins
a. Superficial veins:
i. Dorsal digital veins
ii. Dorsal venous arch
iii. Superficial plantar veins
iv. Greater saphenous vein
v. Lesser saphenous vein
b. Deep veins:
i. Deep dorsal venous network
ii. Deep venous plantar network
iii. Medial and lateral plantar vein
iv. Femoral vein

3. The lymphatics:
a. Lymph nodes: The largest group of lymph nodes are found in the inguinal
region, and are divided into superficial and deep
i. Superficial nodes: There are 2 groups of superficial nodes totalling 1525 in
number
ii. Deep nodes: 1 to 3 in number, found in the femoral canal and occasionally
medial to the femoral vein, where the great saphenous vein drains into it

Myology
1. Osteofascial compartments of the leg
a. Anterior
i. Boundaries: Anterior and laterally by the crural fascia, medially by the tibia,
posteriorly by the fibula. and interosseous membrane
ii. Contents: tibialis anterior, EHL, EDL, peroneus tertius, anterior tibial artery
and veins, deep peroneal nerve
b. Lateral
i. Boundary: Laterally is the crural fascia, posteriorly by the peroneal
septum, medially by the fibula, and anteriorly by the anterior peroneal septum
ii. Contents: peroneus longus and brevis
c. Posterior (divided into superficial and deep) i. Superficial
• Boundaries: Medially/laterally/posteriorly by the crural fascia, and
anteriorly by the deep transverse intermuscular septum
• Contents: Gastrocnemius, soleus, and plantaris
ii. Deep
• Boundaries: Posteriorly by the deep transverse intermuscular septum,
laterally by the fibula, and medially by the tibia
• Contents: FDL, FDB, tibialis posterior, posterior tibial artery and vein,
and tibial nerve

2. Retinacula of the ankle: Thickened portions of crural fascia which bind


tendons of extrinsic muscles of the foot closely to bone. They are considered to
be superficial ligaments of the ankle
a. Extensor Retinaculum: Is divided into two distinct portions, superior portion
(transverse crural ligament) and inferior portion (cruciate crural ligament)
i. Transverse crural:
• Located entirely on the leg (attached to the tibia and fibula above the
malleoli)
• Separates the tendons of the EDL, the peroneus tertius, and EHL from
each other
ii. Cruciate crural:
• Forms a transverse "Y", with a lateral stem and 2 medial arms
• Lateral stem attaches to the lateral surface of the calcaneus, deep fascia,
and talus
• The superior arm is separated into superficial and deep layers by the EHL
which runs through it
• The inferior arm passes around the medial side of the foot to blend with
the deep fascia of the sole, and attaches to the medial aspect of the plantar
fascia near the tuberosity of the navicular. Medially, the fibers bifurcate to
envelope the muscle belly of the abductor hallucis, forming a tunnel
b. Flexor Retinaculum (lacinate ligament):
i. Forms (in part) the tarsal tunnel
ii. The apex is attached superiorly to the crural fascia and the anteromedial
portion of the medial malleolus. The base it attached inferiorly to the medial
surface of the calcaneus, the medial process of the calcaneal tuberosity, and
tendo Achilles
iii. Contents of the tarsal tunnel:
• 1st (medial compartment): tibialis anterior crosses the medial surface of
the talus and deltoid ligaments
• 2nd compartment: flexor digitorum longus crosses the posteromedial
tubercle of the talus and passes over the medial surface of the sustentaculum
tali
• 3rd compartment: tibial vessels (the nerve runs deep to the artery and
vein)
• 4th (lateral) compartment: flexor hallucis longus grooves the inferior
surface of the sustentaculum tali
c. Peroneal Retinaculum: Divides into superior and inferior surfaces
i. Superior peroneal retinaculum:
• Is quadrilateral in shape
• Splits to enclose the tendons of the peroneus longus and brevis ii. Inferior
peroneal retinaculum;
• Continuous with the main stem of the inferior extensor retinaculum
where it attaches to the lateral surface of the calcaneus
• Some fibers bind to the peroneal tubercle, forming a septum that
separates the 2 peroneal tendons

3. Extrinsic muscles of the foot:


a. Muscles of the anterior compartment:
i. Tibialis anterior:
• Origin: Lateral condyle of the tibia, the upper 2/3 of the lateral surface of
the shaft of the tibia, the interosseous membrane, the deep surface of the
crural fascia, and the intermuscular septum
• Insertion: Into the medial and plantar aspects of the medial cuneiform
(90%) and the medial and plantar aspects of the base of the 1st metatarsal
(10%)
• Vascular supply: Anterior tibial artery
• Innervation: Deep peroneal nerve
• Functions: Statically in dorsiflexion of the foot on the leg, and
dynamically to accelerate and decelerate the foot during gait, assist in
dorsiflexion at toe off, dorsiflex the 1 st ray during swing, resists plantarflexion
of the foot at heel strike, prevents excessive pronation during swing, and
supinates the midtarsal joint around its longitudinal axis prior to heel strike
ii. Extensor hallucis longus:
• Origin: Anterior surface of the fibula at its middle half (medial to the
origin of the EDL) and the interosseous membrane
• Insertion: Inserts at the base of the distal phalanx of the hallux
• Vascular supply: Anterior tibial
• Innervation: Deep peroneal nerve
• Functions: Statically it dorsiflexes the foot on the leg, extends the hallux,
extends the distal phalanx on the proximal phalanx, and assists in inverting
the foot. Dynamically, during stance it creates a "rigid beam effect" for the
hallux by stabilizing the 1st MTPJ and the IPJ, and during swing it acts as the
strongest dorsiflexor of the foot and accelerates the foot immediately after toe
off
iii. Extensor digitorum longus:
• Origin: Lateral condyle of the tibia, upper 3/4 of the lateral condyle of the
tibia, upper 3/4 of the anterior surface of the tibia, upper lateral part of the
interosseous membrane, deep surface of the crural fascia, and the anterior
peroneal septum
• Insertion: It divides into 4 tendons at various distances proximal to the
ankle joint, and sends a tendon to each lesser toe. Near the metatarsal heads,
each tendon forms a membranous expansion (extensor hood) which covers the
MTPJ dorsally and laterally and receives tendons of insertion from the
lumbricales and interossei muscles to the lesser toes. Tendons to the 2nd, 3rd,
and 3rd toes receive tendons from the EDB on their lateral sides. The EDL
tendons to each toe then divide into 3 parts distal to the extensor hood
apparatus at the head of the proximal phalanx
• Vascular supply: Anterior tibial artery
• Innervation: Deep peroneal nerve
• Functions: Statically the EDL dorsiflexes the foot on the leg, extends the
lateral 4 toes, and extends the phalanges of the toes upon each other.
Dynamically, it stabilizes and accelerates the foot during gait, and during the
propulsive period of gait it helps create a "rigid beam effect" in each lesser toe
by assisting the lumbricales in rigidly extending the IPJs during propulsion.
During swing phase it assists in dorsiflexion
iv. Peroneus tertius:
• Origin: From the lower border of the anterior surface of the fibula, and
adjoining portion of the interosseous membrane
• Insertion: On the dorsum of the base of the 5th metatarsal
• Vascular supply: Anterior tibial
• Innervation: Deep peroneal nerve
• Functions: Statically in flexion of the foot on the leg and eversion of the
foot. Dynamically it assists in dorsiflexion of the foot for clearance of the toes
during swing, and helps prevent excessive supination of the foot during swing
b. Muscles of the lateral compartment:
i. Peroneus longus:
• Origin: From the head and upper 1/2 of the lateral surface of the fibula,
the crural fascia, and the anterior/posterior septa
• Insertion: Posterolateral inferior angle of the base of the 1st metatarsal
(90%) and the adjacent portion of the medial cuneiform (does give a slip that
gives origin to the 1st dorsal interosseous muscle)
• Vascular supply: Anterior tibial and peroneal arteries -Innervation:
Superficial peroneal nerve
• Functions: Statically in plantarflexion of the foot on the ankle, eversion of
the foot. Dynamically it is a stance-phase muscle by stabilizing the base of the
1st ray against the tarsal bones, stabilizes the 1st metatarsal head plantarly
against the ground, resists the adductory force exerted on the first ray by the
tibialis posterior, and assists (with the peroneus brevis) in transferring body
weight from lateral to medial side of the forefoot during propulsion
ii. Peroneus brevis:
• Origin: from the lower 2/3 of the lateral surface of the fibula and the
anterior/posterior personeal septa
• Insertion. Lateral aspect of the tuberosity of the 5th metatarsal
• Vascular supply: Peroneal artery
• Innervation: Superficial personeal nerve
• Functions: Statically it plantarflexes the foot on the leg and everts the
foot. Dynamically it is a stance phase muscle by assisting the peroneus
longus in producing an abductory stabilizing force on the foot during
midstance and propulsion, stabilizes the lateral column during midstance
and propulsion, resists supination of the foot and external rotation of the
leg by the calf muscles, provides a pronatory force at the subtalar and
midtarsal joints and assist the peroneus longus in transferring body weight
from the lateral to the medial forefoot
c. Muscles of the posterior compartment (divided into superficial and deep
compartment)
i. Gastrocnemius (superficial):
• Origin: The medial head (larger) of the gastrocnemius from the posterior
portion of the medial condyle of the femur, the lower part of the supracondylar
line, and the posterior aspect of the knee joint capsule. The lateral head from
the posterior portion of the lateral condyle of the femur and the lower part of
the lateral supracondylar line
NOTE* The tendon of the lateral head of the gastrocnemius may contain a
sesamoid bone (fabella)
The soleus attaches to the medial 2/3 of the deep surface of the tendo- Achilles
The plantaris is absent 7% of the time
The tendo Achilles is 15 cm in length

The gastrocnemius passes across 3 joints (knee, ankle, STJ) The soleus passes
across 2 joints (Knee and ankle)

• Insertion: Fuse with the soleus to form the Achilles tendon


• Vascular supply: Sural branch of the popliteal artery to each head
entering the origin of the muscle
• Innervation: Tibial nerve
• Functions (swing phase): Prevents hyperextension of the knee, assists in
deceleration if the internal rotation of the leg toward the end of contact, to
prevent torque forces from developing in the knee, assists in developing
supination of the STJ during midstance and early propulsion, and flexes the
knee and lifts the heel to initiate propulsion
ii. Soleus (superficial):
• Origin: From the upper 1 /3 of the posterior portion of the fibula,
posterior surface of the fibula, posterior portion of the head of the fibula, soleal
line on the posterior surface of the tibia, and the middle 1 /3 of the medial
border of the tibia
• Vascular supply: Posterior tibial artery, peroneal and sural arteries -
Innervation: Tibial nerve
• Functions (stance phase): Stabilizes the lateral forefoot against the
ground during late contact and midstance, assists in decelerating knee flexion,
assists in decelerating STJ pronation and internal leg rotation at the end of
contact, assists in extending the knee during midstance, assists in heel lift
during propulsion by stopping ankle joint dorsiflexion

iii. Achilles tendon:


• The thickest and strongest tendon in the body in the lower 1 /4 of the
leg. The fibers of the soleus are anterior to those of the gastrocnemius
proximally, and as the tendon passes distally, the gastrocnemius fibers take a
lateral turn to become lateral to the soleus fibers
iv. Plantaris (superficial):
• Origin: From the lateral supracondylar line of the femur, slightly medial
to the lateral head of the gastrocnemius
• Insertion: Into the medial edge of the tendo Achilles -Vascular supply:
Sural arteries
• Innervation: Tibial nerve
• Functions: Statically it flexes the leg on the thigh and plantarflexes the
foot on the leg. Dynamically, it assists the gastrocnemius in its function during
locomotion
v. Flexor digitorum longus (deep):
• Origin: From the posterior surface of the tibia below the popliteal line and
medial to the vertical line and from the fascia of the tibialis posterior muscle
belly
• Insertion: Into the middle of the bases of the distal phalanges of the
lateral 4 toes plantarly
• Vascular Supply: Posterior tibial artery
• Innervation: Tibial nerve
• Functions: Statically, plantarflexing the foot on the leg, flexes the lateral
4 toes upon their metatarsals, and flexes the phalanges of each toe upon
each other. Dynamically (stance phase) assists with deceleration of STJ
pronation and internal leg rotation during contact, assists with deceleration of
the forward motion of the tibia during midstance, assists with STJ supination
and external leg rotation during midstance, assists with deceleration of ankle
joint dorsiflexion (contributes to heel lift), maintains stability of the lesser digits
against the ground during propulsion, assists in plantarflexion of the foot upon
the leg during propulsion, and assists the abductor hallucis and FHL in
supination of the MTJ about its oblique axis during early propulsion
vi. Flexor hallucis longus (deep):
• Origin: From the lower 2/3 of the posterolateral portion of the posterior
surface of the fibula, posterior peroneal septum, deep transverse intermuscular
septum
• Insertion: Into the base of the distal phalanx of the hallux, plantarly
• Vascular supply: Peroneal and posterior tibial
• Innervation: Tibial nerve
• Functions: Flexes the hallux at the MTPJ and IPJ, and plantarflexes the
foot on the leg. Dynamically it assists with deceleration of the forward
momentum of the tibia during midstance, assists with STJ supination and
external leg rotation during midstance, assists in deceleration of forward
momentum of the tibia, maintains stability of the hallux against the ground
during propulsion, assists in posterior stabilization of the phalanges on each
other against the 1st metatarsal head, and assists the abductor hallucis and
FDL with supination of the MTJ about its oblique axis
vii. Tibialis posterior (deep):
• Origin: From the posteriomedial surface of the fibula, posterior surface of
the interosseous membrane between the tibia and fibula, posterior surface of
the tibia
• Insertion: Primarily onto the tuberosity of the navicular, however,
insertions reach all the tarsal bones except the talus
• Vascular supply: Sural, peroneal, and posterior tibial arteries -
innervation: Tibial nerve
• Functions: Plantarflexes the foot on the leg and inverts the foot.
Dynamically it is a stance phase muscle that decelerates STJ pronation and
internal rotation of the leg during the contact period, assists with deceleration
of forward momentum of the leg during the contact and midstance periods,
accelerates STJ supination and external rotation of the leg during midstance,
assists in heel lift by decelerating forward motion of the tibia in ankle joint
dorsiflexion
4. Plantar aponeurosis and plantar compartments of the foot:
a. Plantar aponeurosis/Plantar fascia: Is divided in 3 portions (central, lateral,
and medial)
i. Central portion: Is the thickest and strongest, triangular in shape, with its
apex attached to the posteromedial portion of the tuberosity of the calcaneus. It
extends distally and separates into 5 bands. At the level of the metatarsal
heads, each band divides into a deep and superficial process.

NOTE* There are several aponeurotic structures in the distal forefoot


associated with the superficial processes:
a. Natatory ligament: Made up of 6-8 bands of transversely oriented
aponeurotic tissue deep to the superficial portion of the plantar aponeurosis. It
divides the area between the web spaces and dermis in the plantar aspect of
the metatarsal head region
b. Fasciculus aponeurotica transversum
c. Sagittal septa
d. Vertical fibers
e. Mooring ligaments
f. Superficial transverse metatarsal ligament

b. Plantar compartments of the foot: Are divided into medial, central and lateral
i. Central compartment: Is the largest and contains 4 fascial spaces
• Contains tendons of the FDL, tibialis posterior, peroneus longus, and
FDB, flexor accessorius, 4 lumbricales, and adductor hallucis
ii. Medial compartment:
• Contains the tendons of insertion of the tibialis posterior and FHL and
muscle bellies of the adductor hallucis and FDB
iii. Lateral compartment:
• Contains the abductor digiti minimi, flexor digiti minimi brevis, and
opponens

5. Intrinsic muscles of the dorsum of the foot:


a. Extensor digitorum brevis:
i. Origin: From the tubercle at the lateral end of the calcaneal sulcus and
cervical ligament
ii. Insertion: To the lateral side of the EDL to toes 2, 3, and 4. It runs superficial
to the dorsalis pedis artery
iii. Vascular supply: Dorsalis pedis (lateral tarsal branch/perforating peroneal)
iv. Innervation: Lateral terminal branch of the deep peroneal and occasionally
the superficial peroneal
v. Functions: Extends toes 2, 3, and 4 and extends the phalanges on each
other. Dynamically it is a stance phase muscle that stabilizes the MTJs oblique
axis in a pronatory direction during propulsion, and stabilizes the 2nd, 3rd,
and 4th metatarsals against the lesser tarsus in a posterior direction, assists
the lumbricales and EDL in extending the 2nd, 3rd, and 4th toes during
propulsion
b. Extensor hallucis brevis: This is the medial portion of the EDB muscle belly
i. Origin: From the tubercle at the lateral end of the calcaneal sulcus and
cervical ligament
ii. Insertion: Onto the dorsal aspect of the proximal phalanx of the hallux or the
lateral aspect of the EHL tendon
iii. Vascular supply: Dorsalis pedis (lateral tarsal branch perforating peroneal)
iv. innervation: Lateral terminal branch of the deep peroneal, and occasionally
the superficial peroneal
v. Functions. Extends the hallux at the MTPJ and extends the distal
phalanx on the proximal phalanx of the hallux. Dynamically the EHB is a
stance phase muscle that stabilizes the 1st metatatarsal against the lesser
tarsus during midstance and propulsion, and assists in stabilizing the hallux
against the 1st metatarsal in a posterior and abductory direction
6. Intrinsic muscles of the plantar aspect of the foot:
a. First layer (superficial):
i. Abductor hallucis:
• Origin: Medial process of the calcaneal tuberosity, lacinate ligament,
plantar aponeurosis
• Insertion: Mainly to the plantar part of the MTPJ joint capsule and the
medial sesamoid
• Vascular supply: Medial plantar artery
• Innervation: Tibial nerve
• Functions: Statically it flexes and abducts the hallux. Dynamically
(stance phase) it stabilizes the hallux in an abductory direction, assists in
stabilizing the proximal phalanx against the 1st metatarsal head, plantarflexes
the 1st ray, and stabilizes the oblique axis of the MTJ in a supinatory direction
during propulsion against the pronatory force of the ground reaction
ii. Flexor digitorum brevis:
• Origin: Medial process of the calcaneal tuberosity, and the deep surface
of the plantar aponeurosis
• Insertion: Into the medial and lateral portions of the plantar aspect of the
middle phalanx of each lesser toe

NOTE* Just proximal to the MTPJ the tendon to each toe divides longitudinally
to allow passage of the FDL. It reunites to form a grooved channel for the FDL
under the proximal phalanx, and then divides again at the head of the
proximal phalanx into 2 tendons

• Vascular supply: Medial plantar artery


• Innervation: Medial plantar nerve
• Functions: Statically it flexes the lesser toes and flexes the middle
phalanx upon the proximal phalanx. Dynamically (stance-phase) it stabilizes
the oblique axis of the MTJ in a supinatory direction, stabilizes the 2nd, 3rd,
and 4th rays posteriorly and plantarly, plantarflexes the lesser rays during
propulsion, assists the FDL in maintaining the stability of the lesser digits
iii. Abductor digiti quinti (minimi):
• Origin: Lateral process of the calcaneal tuberosity, plantar surface of the
calcaneus, plantar aponeurosis
• Insertion: The lateral side of the base of the proximal phalanx of the 5th
toe plantarly with the tendon of the flexor digiti quinti brevis
• Vascular supply: Lateral plantar artery
• Innervation: Lateral plantar nerve
• Functions: Statically it flexes and abducts the 5th toe. Dynamically
(stance phase) stabilizes the 5th toe posteriorly, plantarly, and in an abductory
direction
b. Second layer:
i. Quadratus plantae: Has 2 heads with the medial head being larger. The 2
heads come together to form a sheet-like 4 sided muscle
• Origin: The medial surface of the calcaneus (lateral head originates
anterior to the lateral process of the calcaneus on its plantar surface from the
long plantar ligament)
• Insertion: Attaches to the lateral aspect of the FDL tendon, before it
divides
• Vascular supply: Lateral plantar artery
• Innervation: Lateral plantar nerve
• Functions: Statically it aids in flexing the 4 lateral toes by assisting the
FDL. Dynamically it stabilizes the oblique axis of the MTJ in a supinatory
direction during midstance and propulsion, stabilizes the tendons of the FDL,
and stabilizes the lesser metatarsals

ii. Lumbricals: Are numbered 1-4 from medial to lateral


• Origin: 1 st from the medial margin of the FDL to the 2nd toe. 2nd from
adjacent sides of of the 1st and 2nd FDL tendons. 3rd from adjacent sides of
2nd and 3rd FDL tendons, 4th from adjacent sides of 3rd and 4th FDL tendons
Insertion: Into the medial aspect of middle of proximal phalanges
• Vascular supply: Plantar metatarsal artery
• Innervation: 1st lumbrical L5 and S1 (medial plantar nerve), the rest via
roots S1 and S2
• Functions: Statically they function to flex the 4 lesser toes, adduct the
lesser toes toward the hallux and extend the phalanges of each toe.
Dynamically they function to extend the IPJs of each lesser toe, assist in
stabilizing the proximal phalanx of each lesser toe against the ground

NOTE* All intrinsic muscles of the 2nd layer are related to tendons of the FDL
which pass through it

c. Third layer:
i. Flexor hallucis brevis: Has a "Y" shaped configuration
• Origin: Originates from the medial aspect of the plantar surface of the
cuboid (stem), the lateral and plantar aspects of the 3rd cuneiform (lateral arm),
peroneus longus sheath, (lateral arm), tibialis posterior to the 3rd cuneiform
and cuboid (lateral arm), and tendinous slips of the tibialis posterior to the
metatarsal bases (medial arm)
• Insertion: 2 heads of insertion into the plantar plate of the 1st MTPJ and
sesamoid area, then passes forward to insert with the tendon of the abductor
hallucis into the base of the proximal phalanx
• Vascular supply: 1st plantar metatarsal artery
• Innervation: Medial plantar nerve
• Functions: Statically it aids the hallux in flexion. Dynamically (stance
phase) it stabilizes the base of the proximal phalanx of the hallux against the
ground during propulsion, assists in stabilizing the entire hallux against the
ground, and assists in stabilizing the 1st, 2nd, and 3rd metatarsals at the
metatarsocuneiform joints during propulsion
ii. Adductor hallucis: This consists of 2 different muscle bellies of origin
(oblique and transverse heads)
• Origin: Oblique head from the medial sides of the shafts and bases of
metatarsals 2-4, adjacent portions of associated cuneiforms, and peroneal
sheath. The transverse head originates from the deep transverse metatarsal
ligaments, plantar plates, joint capsules, and plantar metatarsal ligaments 3-5
MTPJs
• Insertion: Both heads insert by a common tendon which inserts into a
bony prominence on the inferolateral aspect of the base of the proximal phalanx
of the hallux with the tendon of the lateral head of the FHB
• Vascular supply: 1 st plantar metatarsal artery
• Innervation: Deep branch of the lateral plantar nerve
• Functions: Statically flexes hallux. Dynamically the oblique head is a
stance-phase muscle that stabilizes the hallux in an adductory direction as well
as a posterior direction against the metatarsal head and assists in stabilizing
the proximal phalanx against the ground during propulsion. The transverse
head is a stance-phase muscle that prevents elongation of the deep transverse
metatarsal ligament, and transverse stability to the forefoot at the MTPJs
during propulsion
iii. Flexor digiti quinti (minimi) brevis:
• Origin: From the base of the shaft of the 5th metatarsal, crest of the
cuboid, and peroneal sheath
• Insertion: To the base of the proximal phalanx of the 5th toe laterally and
plantar (along with the tendon of the abductor digiti quinti)
• Vascular supply: Lateral plantar artery
• Innervation: Superficial branch of the lateral plantar nerve
• Functions: Statically it abducts and flexes the 5th toe. Dynamically it
functions as an interosseous muscle during gait

d. Fourth layer:
i. Plantar interossei: Three in number
• Origin: 1 st from the base and medial surface of the shaft of the 3rd
metatarsal. 2nd medial surface of the shaft and plantar surface of the base of
the 4th metatarsal. 3rd from the plantar aspect of the base and medial surface
of the shaft of the 5th metatarsal
• Insertion: 1st inserts into the 3rd toe medially near the base of the
proximal phalanx, and the medial side of the 3rd MTPJ capsule. 2nd is into the
4th toe medially near the base of the proximal phalanx, and the medial side of
the 4th MTPJ capsule. 3rd inserts medially into the 5th toe near the base of the
proximal phalanx, and the medial side of the 5th MTPJ capsule
• Vascular supply: 2nd, 3rd, and 4th plantar metatarsal arteries
• Innervation: Lateral plantar nerve
• Functions: Statically the plantar interossei aid in flexing the toes into
which they are inserted and aid in extension of the IPJs. Dynamically they draw
the toes medially toward the 2nd digit

ii. Dorsal interossei: Four in number


• Origin: 1st from the adjacent surfaces of the first and second metatarsals.
2nd from the adjacent surfaces of the second and third metatarsals. 3rd from
the adjacent surfaces of the third and fourth metatarsals. 4th from the adjacent
surfaces of the fourth and fifth metatarsals
• Insertion: 1st inserts into the membranous expansion of the EDL to the
2nd toe and the medial side of the base of the proximal phalanx of the 2nd toe.
3rd into the membranous expansion of the EDL to the 3rd toe and the lateral
side of the base of the proximal phalanx to the 3rd toe. 4th into the the
membranous expansion of the EDL to the 4th toe, and the lateral side of the
base of the proximal phalanx of the 4th toe
• Vascular supply: 2nd, 3rd, and 4th plantar metatarsal arteries
• Innervation: Lateral plantar nerves (superficial and deep)
• Functions: Statically they aid in flexing the toe to which they insert and
in extending the IPJs. Dynamically these are stance-phase muscles that
stabilize the bases of the proximal phalanges posteriorly and transversely
against their metatarsal heads

7. Extensor hood expansion:


a. It is a membranous sheet covering each lesser toe from the MTPJ to (and
including) the distal phalangeal joint
b. It receives contributions from the:
i. EDL tendon
ii. Plantar and dorsal interossei tendons
iii. Tendons of the lumbricales
iv. EDB tendon (toes 2-4)
v. Deep process of the plantar aponeurosis
c. The hood apparatus provides a means for the EDL to extend the proximal
phalanx without having a direct connection to it by controlling the actions of
the interossei and lumbricales muscles
d. The hood apparatus is made up of 2 parts that are continuous with each
other, the sling and wing
e. The sling is more proximal, and the wing is more distal
f. The action of the lumbricales and interossei of the phalanges is intergrated
with that of the EDL by the extensor apparatus attachments

Arthrology
1. Tibiofibular joints: The tibia and fibula are joined in 3 areas proximal,
distal, and crural interosseous

a. Proximal tibiofibular joint: A plane synovial joint between the facet on the
medial aspect of the head of the fibula and the fibula facet on the lateral
condyle of the tibia
i. Anterior superior tibiofibular ligament
ii. Posterior superior tibiofibular ligament
b. Distal tibiofibular joint: A syndesmosis between the distal ends of the tibia
and fibula
i. Interosseous tibiofibular ligament
ii. Anterior inferior tibiofibular ligament
iii. Posterior inferior tibiofibular ligament

c. Crural interosseous membrane (middle tibiofibular joint): A sheet of fibrous


connective tissue between the interosseous borders of the tibia and fibula
d. Functional anatomy of the tibiofibular joint:
i. Motion is limited to slight rotation and gliding
ii. In ankle dorsiflexion, the talus is forced into the ankle mortise, separating
the 2 bones (adding stability). Distal separation is limited by the fibrous
syndesmosis of the distal joint and the crural interosseous membrane
2. Ankle joint (talocrural): A modified ginglymus or hinged synovial joint. In
addition to dorsiflexion/plantarflexion, a small amount of abduction/adduction
as well as eversion/inversion
a. Articular surfaces (superior concavity): The ankle mortise
i. Distal end of the tibia (tibial plafond): wider anteriorly (produces stability in
stance)
ii. Comma shaped facet on the lateral surface of the medial malleolus
iii. Triangular shaped facet on the medial surface of the lateral malleolus iv
Anterior tibiofibular ligament
b. Medial collateral ligaments (deltoid): Fans out into one deep and 3 superficial
portions, with attachments to the talus, calcaneus, and navicular
i. Anterior tibiofibular ligament: Deepest
ii. Posterior tibiofibular ligament
iii. Tibiocalcaneal ligament: Blends with the spring ligament
iv. Tibionavicular ligament
c. Lateral collateral ligaments: 3 subdivisions
i. Anterior talofibular ligament: Intracapsular, blends with the ankle joint
anteriorly, shortest lateral ligament, deep
ii. Posterior talofibular ligament: Strongest, intracapsular but extrasynovial
iii. Calcaneofibular ligament: Extracapsular, found deep to the tendons of the
peroneals

NOTE* This ligament crosses the ankle and STJ as does the tibiocalcaneal
ligament
d. Fibrous capsule: Surrounds the ankle joint and is attached to the margins of
the articular surfaces
e. Synovial capsular membrane:
i.Closely attached to the fibrous capsular membrane medially and laterally
ii. Bulges are seen at the anterior tibiofibular ligament and parts of the lateral
ligament
iii. Synovial membrane is exposed near the medial malleolus
iv. Loose fold of synovial membrane is present in the ankle mortise between the
articulation of the lateral malleolus and inferior surface of the tibia (allows for
accommodation of the talar dome) f. Functional anatomy:
i. Oblique axis of motion running from posteroinferolaterally to
anterosuperomedially (pronatory/supinatory)
ii. The axis changes as the foot moves with dorsiflexion and plantarflexion

3. Subtalar joint (anatomic): A modified ginglymus joint

NOTE* The "anatomic" STJ is defined as the synovial articulation between the
posterior talar facet on the superior surface of the calcaneus and the posterior
calcaneal facet on the inferior surface on the body of the talus. The 'functional"
STJ includes the middle and anterior facets
The anterior and middle facets of the calcaneus are part of the
talocalcaneonavicular, a separate Synovial joint.
a. Articular areas: The posterior facets are roughly triangular in shape. The
articular surface of the the talus is concave, and the calcaneal surface is convex
alond its longitudinal axis
b. Sinus tarsi: Formed by the articulation of the sulcus calcanei between the
posterior and and anterior-middle facets of the calcaneus and the sulcus tali on
the inferior surface of the talar neck. It is wider laterally
c. Fibrous capsule: Completely surrounds the joint and reinforced by capsular
ligaments
d. Capsular ligaments:
i. Posterior calcaneal ligament: "Y" shaped. Medial band forms a roof over the
groove for the FHL tendon
ii. Lateral talocalcaneal ligament
iii. Medial talocalcaneal ligament
iv. Anterior talocalcaneal ligament
v. Interosseous talocalcaneal ligament (ligament of the tarsal canal): Located
within the tarsal canal and strengthens the STJ posteriorly
vi. Cervical ligament: Found laterally in the sinus tarsi, and resists supination
of the STJ
e. Functional anatomy:
• Most important STJ supinators are the tibialis anterior and tibialis
posterior
• Most important pronators are the peroneus longus and peroneus brevis
• The long axis of the posterior talar facet of the calcaneus is at an angle of
about 40° to the long axis to the foot

4. Talocalcaneonavicular joint: The head of the talus fits into a socket formed
by the posterior articular surface of the navicular and the anterior and middle
talar articular facets of the calcaneus, as well as the plantar calcaneonavicular
ligament
a. Classification: A modified condylar joint (capable of gliding and rotatory
motion)
b. Articular surfaces:
i. Head of talus:
• Convex in all directions
ii. Acetabulum pedis (joint socket):
• Anteroinferior portion: Concave in all directions, and formed by the
posterior surface of the navicular
• Posterior portion: Formed by the anterior and middle articular facets of
the calcaneus
• Inferomedial portion: Formed by the plantar calcaneonavicular ligament

NOTE* The articular area of the talar head is greater than the socket, and as a
result, in normal anatomic position, the head protrudes slightly dorsally at the
joint

c. Capsular ligaments:
i. Dorsal talonavicular ligament
ii. Plantar calcaneonavicular ligament (spring): Important in supporting the
talar head
iii. Calcaneonavicular portion of the bifurcate ligament: "Y" shaped with the
stem attached to the calcaneus and one arm attached to the cuboid and the
other arm to the navicular
d. Synovial membrane: Lines the fibrous capsule
e. Functional anatomy:
i. Some gliding and rotational movements are possible, but the TCN joint
cannot act independently
ii. Any motion at the STJ causes motion in the TCN joint. Additionally, motion
between the talus and navicular also involves the C-C joint
5. Calcaneocuboid joint: Formed by the articulation of the posterior surface of
the cuboid and the anterior surface of the calcaneus
a. Classification: A saddle-shaped synovial joint
b. Fibrous capsule: Surrounds the entire joint and has 3 capsular ligaments
i. Dorsal calcaneonavicular ligament
ii. Lateral calcaneocuboid ligament
iii. Calcaneocuboid portion of the bifurcate ligament
c. Extracapsular ligaments:
i. Long plantar ligament (long calcaneocuboid):
• Superficial and deep fibers
• Longest ligament in the foot
• Posterior fibers attached to plantar surface of the calcaneus and runs
distally to bases of metatarsals 2-5
d. Synovial capsule: Lines the entire joint
e. Functional anatomy: Has two axes of rotation
i. One axis through the calcaneal process
ii. One axis through the head of the talus
• Pronation results in the plantar, dorsal, and lateral ligaments being taut
• Supination results in the opposite
6. Great tarsal joint: Made up of the following joints
a. Cuboideonavicular joint:
i. Classification: Syndesmosis or planar synovial
ii. Fibrous capsule: Present only when a synovial joint is present. Has 3
capsular ligaments
• Dorsal cuboideonavicular
• Interosseous cuboideonavicular
• Plantar cuboideonavicular
i. Function: Very limited motion, but the motion that does occur is with
pronation/supination of the STJ
b. Cuneonavicular joint:
i. Classification: Planar in configuration, with a slight convexity in the
navicular
ii. Fibrous capsule: Continuous with all 3 navicular facets, but absent
laterally. Has 3 main capsular ligaments
• Dorsal cuneonavicular
• Medial cuneonavicular
• Plantar cuneonavicular (divided into 3 ligaments)
c. Intercuneiform joints: Forms part of the transverse pedal arch, with the
intermediate cuneiform being the highest point of the arch
i. Classification: Planar synovial
ii. Articular areas:
• Medial intercuneiform joint
• Lateral intercuneiform joint
• Cuneocuboid joint
iii. Fibrous capsule: Blends with the cuneonavicular capsule
d. Cuneocuboid joint
e. Tarsometatarsal joints (2nd and 3rd)
f. Intermetatarsal joints (bases of 2nd and 3rd)

7. Tarsometatarsal joints (LisFranc's joint): Formed by the articulations


between the anterior surfaces of the cuneiforms and cuboid proximally and
posterior surfaces of the metatarsals distally. The 2nd metatarsal base is
recessed proximally for added stability and restricted motion.
a. Classification: Has 3 synovial joints
i. Medial tarsometatarsal joint: Between the base of the 1st metatarsal and
medial cuneiform
ii. Intermediate tarsometatarsal joint: Between the base of the 2nd and 3rd
metatarsals and intermediate and lateral cuneiforms
iii. Lateral tarsometatarsal joint: Between the bases of the 4th and 5th
metatarsals and cuboid
b. Fibrous capsule: Each T-M synovial joint has its own capsule
c. Capsular ligaments:
i. Dorsal tarsometatarsal ligaments
ii. Plantar tarsometatarsal ligaments
d. Inferior surface of the medial cuneiform to the base of the 1st metatarsal
i. Interosseous tarsometatarsal ligaments (attach the cuneiforms or cuboid to
the metatarsal bases)
• Medial interosseous ligament (LisFranc's ligament): Strongest, runs from
the lateral surface of the medial cuneiform to medial surface of the 2nd
metatarsal base, and is important for stability of the T-M joint
• Intermediate interosseous ligament (2nd)
• Lateral interosseous ligament (3rd)
e. Function: Slight gliding motion of the lesser metatarsal on their adjacent
tarsal bones
Chapter 34: SOFT TISSUE
TUMORS
Soft Tissue Tumors
SOFT TISSUE TUMORS
Soft tissue tumors are those originating in the supportive and connective
tissues of the body, other than bone or cartilage. Benign and malignant forms
occur. In addition, there are tumor-like lesions (“pseudotumors”) that may
clinically suggest a neoplasm. The difference is based upon a good history,
examination and biopsy
Soft Tissue Tumors
1. Scar (cicatrix), Hypertrophic scar, Keloid:
a. Reactive connective tissue lesions, secondary to penetrating trauma, and
therefore secondary lesions (see Chapter 13, Dermatology)

2. Fibroma:
a. A benign, self-limiting proliferation of fibroblasts, producing collagen, most
being subcutaneous single lesions
b. They occur at any age
c. Koenen's Periungual Fibromas are seen on the hands and feet of about one-
third of patients with tuberous sclerosis

NOTE* Patients with tuberous sclerosis show an acneform facial rash whose
lesions are called adenoma sebaceum, but which are angiofibromas. Some
patients also are epileptic due to intracranial calcifications. This form of
tuberous sclerosis is called Epiloia. Fibromas may develop independent of
other diseases and are firm, discrete lesions. Sometimes other
elements are involved, e.g. fibrolipoma, fibrokeratoma, angiofibroma,
fibromyxoma, xanthofibroma, neurofibroma, etc. Lesions may recur after
excision

3. Fibromatosis: This is a group of diseases demonstrating infiltrative growth


that makes them less discreet than fibromas
a. Lesions are frequently multiple, and patients may have more than one type
b. Plantar fibromatosis usually affects the medial fascial slip, but any area may
be affected. This is correctly called Ledderhose's or Dupuytren's disease
c. A cure may be impossible because the lesion is infiltrative so that residual
unseen lesion is always left behind after excision
d. Variants affecting the feet may include juvenile hyalin fibromatosis, extra-
abdominal desmoid tumors, infantile digital fibromatosis, and juvenile
aponeurotic fibromatosis of Keasbey (which also occurs in adults)
e. Desmoids also occur in Gardner's syndrome
f. Fibromas may histologically resemble well diferentiated fibrosarcoma
g. Older Keasbey lesions may become chondrified or ossified. Fibroosseous
pseudotumor of the digits affects the fingers and toes, showing soft tissue
ossifications. It is not the same as a Keasbey lesion
NOTE* A differential diagnosis of fibrosarcoma vs. plantar fibromatosis
should be kept in mind on initial examination of the foot

4. Myxomas:
a. Are soft tumors of fibrous origin that show a loose fibrous stroma and much
mucin and mucopolysaccharide within
b. This group includes digital mucus cysts and the synovial cysts such as
ganglion (only when the tendon sheath is present, hence, never distal to the
tarsus), and bursa
c. Synovial cysts can also be intraosseous lesions
d. This group are pseudotumors

5. Fibrosarcoma:
a. A malignant, collagenous, metastatic tumor that can occur at any age and
which is uncommon in the foot
b. They may be deep or intradermal
c. They also may be well differentiated or poorly differentiated
d. They may show a histologic "herringbone" pattern of cellular arrangement

6. Histiocytomas:
a. Are similar to fibromas, but show less collagen
b. The histiocyte is considered a facultative fibroblast, a cell state (rather
than a true cell type)
c. There are 3 major groups:
i. Fibroblastic fibrous: includes the following types
• Dermatofibroma
• Xanthomas (some)
• Myxomas
• Dermatofibrosarcoma protuberans (malignant): show a storiform pattern
histologically
ii. Histiocytic fibrous:
• Giant cell tumor of soft tissue (and bone)
• Xanthogranuloma
• Pigmented villonodular synovitis
• There are also malignant forms recognized
iii. Pleomorphhic fibrous: A group of unusual xanthomas,
xanthogranulomas, and atypical fibroxanthomas

NOTE* Tendon xanthomas may suggest lipoproteinemia. Necrobiotic


xanthogranulomas may be associated with multiple myeloma. Multicentric
reticulohistiocytosis may be a cause of polyarthritis

7. Synovial Chondromatosis (loose bodies, joint mice, joint rice,


osteochondritis dissecans):
a. A rare condition that follows trauma to synovium of large joints such as the
ankle
b. Some of the bodies may ossify
c. X-rays and scans are helpful in the diagnosis

8. Synovial sarcoma (tendosynovial sarcoma):


a. A highly malignant tumor of synovial tissues, often not arising from a joint
b. Young people are affected
c. They may be slow or fast in growing and are very metastatic, especially if 5
cm or larger in size
d. Radiographic techniques and biopsy aid in the diagnosis
e. Treat aggressively (chemotherapy, surgery, irradiation)
f. This is one of the few tumors for which there is no benign equivalent (i.e.
there is no synovioma)

9. Subcutaneous Nodular Fat Necrosis (pancreatic panniculitis):


a. Consists of reddish, tender nodules on the feet and ankles often
accompanied by ankle arthralgias
b. This accompanies pancreatitis and pancreatic carcinoma
c. Elevated blood levels of trypsin increase vascular permeability so that
elevated serum lipase escapes and damages the fat, causing inflammatory
tumor-like nodules
d. If biopsy proven, these lesions suggest need for immediate consultation

10. Lipogranulatomatosis (Farber's disease):


a. Very rare, recessively inherited sphingolipidosis, showing periarticular and
tendon masses, especially at the wrists and ankles
b. Other organs are affected and patients rarely live beyond the first year
c. The masses may, therefore, be diagnostic, requiring immediate consultation

11. Piezogenic Papules:


a. Represent cystic herniations of subcutaneous heel fat into the dermis
b. They are seen on weightbearing in obese patients
c. They may be painful and can undergo necrosis
d. Control with heel cupping devices and diet

12. Lipoma:
a. A benign tumor of adipose tissue and multiple lesions may occur
b. These may be present independently or as part of Gardner's syndrome and
neurofibromatosis-1
c. Lipomas do not reduce in size as fat is lost due to diet or illness
d. Variants include fibrolipoma, angiolipoma, and myelolipoma (producing
myeloid hematopoietic cells)
e. They can be excised locally without recurrence
f. Tendon sheath lipomas (endovaginal lipomas) may occur in the feet and/or
hands
g. Lipoblastoma may affect the feet of children

13. Liposarcoma:
a. Is a malignant, metastatic tumor of fatty tissue
b. It is probable the most common lower extremity sarcoma in adults, but it is
uncommon in the feet
c. There are gradations from well to poorly differentiated
d. They may be slow growing and asymptomatic until a nerve or other tissue is
sufficiently affected to result In pain or altered function e. These are deep
seated tumors that should be excised followed by chemotherapy and irradiation

14. Myositis Ossificans (Munchmeyer's disease):


a. A benign reactive lesion deep in striated muscle resulting from trauma
b. Early stages may be hard to tell on x-ray and histologically from the
parosteal or extraosseous forms of osteogenic sarcoma
c. Older lesions appear fully ossified and may be excised if they impede function
or cause pain
d. Myositis ossificans progressiva is a rare hereditary disease of children and
young adults in which ossified lesions affect many tissues and are not related
to trauma (patients may show shortened digits, absent thumbs and great toes,
or hallux valgus bilaterally)

15. Rhabdomyoma:
a. Is a benign tumor of striated muscle and has not been reported in the foot
b. Patients showing signs and symptoms of tuberous sclerosis (see Fibroma
section this chapter) may have cardiac rhabdomyoma and should, be sent for
examination

16. Supernumerary Muscles:


a. Have been reported in the ankle region and may simulate tumors

17. Rhabdomyosarcoma:
a. Very malignant metastatic tumor of striated muscle
b. Rare in the foot, there have been reported cases
c. There are 2 main types:
i. Embryonal: seen in mostly younger persons
ii. Pleomorphic: seen in older persons
d. These tumors are fast-growing and they erode bone
e. Amputation, chemotherapy, and irradiation are all treatment methods

NOTE* Probably the most common lower extremity malignant tumor of


children

18. Leiomyoma:
a. Is a benign tumor arising from smooth muscle

NOTE* The sources of smooth muscle in the feet are the arrector pill muscles
associated with pilosebaceous units in the dorsal skin of the feet and toes,
and blood vessels (frequently veins) anywhere in the foot.

b. The tumors are pilar leiomyomas and angioleiomyomas


c. These lesions are often painful
19. Leiomyosarcoma:
a. A malignant smooth muscle tumor rare (but does occur) in the ankle or foot
area
b. Lesions may be superficial (dermal in origin) or deep (subcutaneous)
c. Metastases from other sites may occur in the foot

20. Neuromas:
a. Are not tumors
b. They are reactive lesions arising as a result of trauma
c. Rudimentary (supernumerary) digits may contain neuromatous tissues

21. Schwannoma:
a. Is the benign peripheral nerve sheath tumor seen in the lower extremities
b. It is considered a more accurate term than the older "neurilemoma" or
"neurinoma"
c. Slow growing, sometimes painful and mainly occurring on the flexor surfaces
d. They may fluctuate due to cystic changes
e. These fall into the peripheral nerve sheath tumor (PNST) group
f. Histologically one sees Verocay bodies that may be well developed (Antoni
type A) or that may show myxoid degeneration (Antoni type B)
g. The "ancient schwannoma" is one that shows cystic degeneration and/or
calcification
h. These lesions can be excised and are not recurrent
i. In association with neurofibromatosis-1 there may be multiple schwannomas
j. The neurothekeoma is a benign peripheral nerve sheath tumor

22. Neurofibroma:
a. A benign, slow-growing, solitary, superficial, usually asymptomatic nodule
b. Excision is curative
c. Most cases occur in women
d. Neurofibromatosis-1 (von Recklinghausen's disease, multiple
neurofibromatosis) is a hereditary malformation involving neural, cutaneous,
and fibrous elements
i. Many organ systems may be involved
ii. There are strict criteria for diagnosis of neurofibromatosis-1 that involve cafe-
au-lait spots
iii. Patients show a high rate of development of unrelated malignant tumors
iv. There is a high risk of transformation of neurofibromas to
neurofibrosarcomas in neurofibromatosis-1
v. Diseases associated with neurofibromatosis-1 include:
• Scoliosis
• Erosions of bone (due to neurofibromas)
• Frohlich's syndrome
• Addison's disease
• Endocrine tumors
• Increased incidence of malignant melanoma
• Connective tissue tumors
• Multiple lipomas
• Meningocele and syringomyelia

23. Malignant Peripheral Nerve Sheath Tumors (MPNST):


a. These include malignant schwannoma and malignant neurofibrosarcoma
b. These are extremely rare in the feet but are a risk in the patient with
neurofibromatosis-1 (when present there is a high mortality rate
c. These tumors may also arise as a result of overexposure to irradiation
(therapeutic or occupational)
d. These tumors are metastatic and may be painful
e. Treatment consists of amputation (improvements in chemotherapy and
irradiation are still being assessed)
f. Malignant triton tumor is a malignant schwannoma with evidence of
concurrent rhabdomyosarcoma

24. Malignant Neuroepithelioma (adult peripheral neuroblastoma,


primative neuroectodermal tumor):
a. Rare in the feet but has been reported
b. Neuroectodermal tumor of infancy (pigmented neuroectodermal tumor of
infancy, melanotic progonoma) is also extremely rare in the foot but has been
reported

25. Granular Cell Tumor (Abrikossoff tumor):


a. Occurs in benign and malignant forms and is uncommon
b. It was once thought to be a tumor of muscular origin, but has been shown to
be of peripheral nerve origin
c. It does occur in the foot

26. Angiomas:.
a. May arise from the endothelium of blood vessels and/or lymphatic vessels
b. Predominant picture based on the presence of blood cells determines
whether the lesion is a hemangioma or a lymphangioma

27. Congenital Hemangiomas:


a. May be localized or may involve large areas of limb and or other organs
b. Most are capillary hemangiomas and after what appears to be a spurt of
rapid growth (in the first 6 months of life) most involute within a year or two
c. Patients should be checked to determine whether a larger (venous)
hemangioma underlies the surface lesion
d. There are rare cases in which capillary hemangiomas in infants are
accompanied by thrombocytopenia (Kasabach-Merritt syndrome)

28. Acquired Tufted Hemangioma:


a. Is the adult form of congenital capillary hemangioma
b. Verrucous hemangioma (angiokeratoma circumscriptum) underlies
hyperkeratoses and may clinically resemble warts or angiokeratoma
c. Cherry angiomas (De Morgan spots, senile angiomas) are tiny papular
angiomas that arise (mostly on the trunk) after adolescence
d. Nevus flammeus (nevus telengiectaticus, “port wine stain”) is a dermal lesion
with many dilated capillaries that usually suggests an underlying vascular
anomaly such as arteriovenous malformation and/or varicosities in the Parkes-
Weber syndrome (osteohypertrophic hemangiectasia, Klippel-Trenauney
syndrome). The A-V malformations usually produce bruits audible with a
stethoscope, local temperature changes, limb hypertrophy, and a risk of “high
output” heart failure

29. Cavernous Hemangiomas:


a. Composed of arterial and venous channels and are deep, large, diffuse
lesions that may clinically feel like a “bag of worms”
b. X-rays may show intravascular calcifications called phleboliths (associated
with thrombi)
c. Venous hemangiomas are clinically similar but do not show the arterial
elements
i. These types of hemangiomas are seen in the blue rubber bleb nevus, Maffuci
(Kast-Maffuci) syndrome, ataxia telangiectasia, von Hippel-Lindau disease, and
Sturge-Weber syndrome

30. Angiokeratoma:
a. The name given to a number of syndromes that affect the feet (and
other areas) showing dilated capillaries (ectasias) with secondary epidermal
hyperplastic changes
b. There are various types, all (except the Fordyce type) affecting the feet
i. Mibelli type
ii. Papular type
iii. Fabry type

NOTE* Angiokeratoma of Fabry is a storage disease whereas the others are


not

c. Cobb's syndrome involves a cutaneous hemangioma in a dermatomal


distribution resembling angiokeratoma and associated with a spinal cord
angioma

31. Angioma Serpiginosum:


a. Consists of red, flat, grouped, asymptomatic, dilated capillaries, forming net-
like or macular patterns
b. Most occur in postadolescent women

32. Pyogenic Granulomas:


a. Eruptive hemangiomas that are friable, easily traumatized, and that may
become secondarily infected
b. Most are on a stalk

33. Glomus Tumors (glomangiomas, angiomyoneuromas):


a. Arise in the digits as a tiny, painful and tender, frequently subungual tumors
b. They arise from the Suquet-Hoyer canal, the shunts arising from arterioles
that bypass the capillary network to the venules
c. They may be red to purple and may be accompanied by thrombocytopenia
(hence a variant of the Kasabach-Merritt syndrome)
d. Most occur as single lesions
e. Glomangiosarcoma is extremely rare, but has been reported

34. Synovial Hemangiomas:


a. Arise in joints and tendon sheaths
b. Rare and difficult to excise
c. There is pain, swelling, joint effusion and hemarthrosis

35. Hemangiopericytoma:
a. Rarely occurs in the feet
b. There are benign and malignant forms, and histologic characteristics do not
necessarily correlate with the benign or malignant nature of the lesion
c. The patients must be watched for recurrence and/or metastases

36. Lymphangiomas:
a. As a group are rare
b. They may arise in the context of chronic and extensive lymphedema, e.g.
Milroy's disease, elephantiasis due to Wucheria infestation, etc.
c. Cystic hygroma represents a cavernous lymphangioma

37. Papillary Endothelial Hyperplasia (Masson's pseudoangiosarcoma):


a. An intravascular proliferation of endothelial cells in association with a
thrombus, and usually follows trauma
b. It is not common
c. Most cases show single lesions and these may be difficult to differentiate
histologically from angiosarcoma

38. Ectasias (vascular dilatations):


a. May occur in the feet and be mistaken for angiomas
b. The most common is the “spider nevus” (spider angioma, nevus araneus)

39. Angioendothelioma:
a. The name given to group of lesions that are considered by some to be of
intermediate malignancy
b. Types:
i. Spindle cell type:
• Is most apt to occur on the lower extremities
• Uncommon, seen frequently in young males
ii. Reactive proliferating angioendotheliomatosis:
• A vascular, benign lesion in response to infection, (especially subacute
bacterial endocarditis) that truly involves endothelial proliferation
• Usually clears n 6-24 months
• AIDS patients may develop this lesions n response to cat scratch fever
organisms (Rochalimaea henselae)
iii. Malignant angioendothelioma:
• A malignant intravascular lymphoma and not an angioma or
angiosarcoma

40. Angiosarcomas:
a. Very rare in the lower extremities
b. Very malignant endothelial tumors, usually occurng in the elderly
c. They may be associated with chronic edema
d. They may grow rapidly and may metastasize
e. Most cases are fatal
f. Amputation a good distance proximal to the tumor is the surgical approach
g. Irradiation has not been very effective
h. Some superficial forms (not the deep forms) have been treated successfully
with carbon dioxide laser
i. Patients must be checked for recurrences and metastases

41. Kaposi's Sarcoma (multiple idiopathic hemorrhagic sarcoma):


a. Is related to immunodeficiency, occurs n older patients, AIDS patients, and
patients on chemotherapy or post-transplant immunosuppression therapy
b. May occur n the presence of leukemias and lymphomas
c. Lesions may appear as purpuric macules, papules, patches, nodules,
plaques, and tumors
d. Silent visceral lesions may be present
e. It is presently divided into the following types:
i. Classic (sporadic) epidemic (non AIDS)
• Attributed to the elderly eastern and southern European men
• Skin associated lesions frequently begin on the feet
ii. African- affects adults and children and not restricted to the feet
iii. Allograft-associated (immunosuppression)
iv. Epidemic (AIDS-associated)

NOTE* On occasion, malignant lower extremity tumors may develop that


cannot be clearly identified as to tissue of origin. These include the alveolar
soft-part sarcoma, the mesenchymoma, and any undifferentiated malignant
soft tissue tumor
42. Nodules of Metastatic Lymphoma (non-Hodgkins) or Myeloid Leukemia
(granulocytic sarcoma, chloroma):
a. May occur n the dermis as metastatic site to feet
b. Biopsy is diagnostic
c. Mycosis fungoides is a T-cell lymphoma originating n the skin
i. Woringer-Kolopp disease: Is the localized epidermotropic form formerly called
Pagetoid reticulosis
ii. Sezary syndrome: is the systemic form
• Circulating abnormal lymphoid cells
• Peripheral lymphadenopathy
• Erythroderma
• Intense pruritus
43: Mastocytomas:
a. Are tumors formed by mast cells
b. Some localize to the dermis others are systemic
c. Dermal lesions urticate (Darier's sign)
d. Pruritis and flushing may occur due to local vasodilatory effects of mastcell
derived histamine shock
e. A single local lesion is known as urticaria pigmentosa

44. Metastatic Cancers:


a. Metastatic cancers from visceral organs may occur in the dermis
b. They are usually rapidly growing, multiple (various sites) and otherwise
nonspecific clinically
c. A history if malignant tumor (even years ago) is a clue, but not always
present
d. Even with such a history, a new lesion may be an independent entity
Chapter 35: Physical Medicine
and Rehabilitation for the
Management of Foot
Conditions
Physical Modalities and Diseases
PHYSICAL MEDICINE AND
REHABILITATION FOR THE
MANAGEMENT OF FOOT
CONDITIONS
Physical medicine can broadly be defined as that element of health care that
utilizes physical agents such as light, heat, cold, water, electricity, mechanical
agents such as excercises, aids to ambulation, gait training, tests and
measurements in the diagnosis and treatment of disease.
Physical Modalities and Diseases
1. Physical modalities (used for foot conditions)
a. Ultrasound
b. Short wave diathermy
c. Infrared lamps
d. Radiant light bakers
e. Whirlpool
f. Contrast baths
g. Paraffin baths
h. Massage
i. Ultraviolet radiation
j. Electrical stimulation
k. Hi-lo galvanic current
l. Faradic current
m. Sinusoidal current
n. Transcutaneous electrical nerve stimulation (TENS)
o. Cold packs
p. Lasers
q. Traction
r. Intermittent compression
s. Exercise
i. Passive
ii. Active assistive
iii. Voluntary active
iv. Active resistive
v. Coordinative
t. Gait training
u. Crutches/walkers/canes/braces/parallel bars
v. Prosthetics/orthotics
w. Activities of daily living
x. Diagnostic procedures and tests

2. Chronically disabling conditions which usually present with pedal


manifestations:
a. Arthritis
b. Burns
c. Cerebral palsy
d. Cerebral vascular accidents
e. Congenital deformities
f. Dermatomyositis
g. Degenerative spinal cord disease
h. Joint deformities
i. Muscular contractures
j. Muscular atrophy
k. Paraplegia
l. Quadraplegia
m. Peripheral nerve injuries
n. Peripheral vascular disease
o. Anterior poliomyositis
p. Polymyositis
q. Polyneuritis
r. Residuals of severe trauma
s. Scleroderma
t. Spina bifida
u. Spinal cord injuries
v. Stasis dermatitis
w. Ulcerations
x. Chronic renal failure
y. Alcohol/drug abuse
z. Chemical dependency

3. Treatment considerations: Examples of conditions which involve the


foot that benefit from the use of physical medicine and rehabilitation
a. Adhesive capsulitis
b. Antalgic gait
c. Arthritis (acute or chronic)
d. Arthritis with nerve root pressure
e. Ataxic gait
f. Atrophy
g. Burns
h. Bursitis
i. Causalgia
j. Capsulitis
k. CVA's
l. Compression syndromes (entrapments)
m. Contractures
n. Contusions
o. Degenerative joint disease
p. Dislocations
q. Fasciitis
r. Fibrosis
s. Foot drop (peroneal palsy)
t. Fractures
u. Hemiplegia
v. Inflammation (acute or subacute)
w. Multiple sclerosis
x. Muscle spasm (acute. or severe)
y. Muscular dysfunction
z. Myositis
A. Nerve palsy
B. Neuralgia
C. Neuritis
D. Osteoarthritis (acute or severe)
E. Osteomyelitis
F. Osteoporosis
G. Pain
H. Paralysis from nerve disease or injury
I. Paraplegia
J. Parkinson's disease
K. Periostitis
L. Peripheral neuritis
M. Peripheral neuropathy
N. Post joint surgery
O. Quadriplegia
P. Radiculitis
Q. Scleroderma
R. Sesamoiditis
S. Severe or generalized weakness
T. Severe osteochondritis dissecans
U. Sprains/strains
V. Sudeck's atrophy
W. Synovitis
X. Tendonitis/tenosynovitis
Y. Trauma/post-trauma
Z. Ulcerations

Therapeutic Modalities and Procedures


1. Therapeutic cold:
a. Indications:
i. To reduce pain and edema following trauma
ii. To reduce pain
iii. To create a reactive hyperemia
iv. To help reduce inflammatory reactions
v. To provide for local tissue destruction
vi. To decrease local tissue metabolism
vii. To help facilitate muscle contraction in some forms of neurogenic weakness
b. Basic contraindications and precautions:
i. Hypertension
ii. Raynaud's disease or phenomenon
iii. Vascular disease
iv. History of frostbite or pernio
v. Rheumatoid arthritis
vi. Cold hypersensitivity
vii. Any condition that produces a cold pressor response
c. Methods of application:
i. Immersion
ii. Cold packs
iii. Ice Massage
iv. Cryokinetics
v. Contrast applications with heat

2. Therapeutic Heat:
a. Physical agents (superficial heat): These modalities cause a rise in tissue
temperature generally limited to the skin and subcutaneous tissue. The effect is
increased circulation, with histamine release and enhanced phagocytosis and
lymph flow. Also there is a sedative type effect and increased connective tissue
extensibility
i. Hot water
ii. Hot air
iii. Infrared
iv. Radiant light
v. Whirlpool
vi. Paraffin
vii. Hot packs
b. Physical agents (deep heat): These modalities cause the same physiological
effects as with superficial heat agents, but they also have the ability to heep
deep structures such as muscle and joint capsule. These modalities are
extremely useful for helping regain lost motion in contracted joints or shortened
muscles
i. Short wave diathermy:

NOTE* High frequency electrical energy extends from electrode to electrode


through human tissue causing a heating effect. The diathermy may be either
short wave (heating within a magnetic field) or microwave energy beamed from
an "antenna" to, an area of the body

ii. Microwave diathermy: not to be used if a patient has a pacemaker


iii. Ultrasound:

NOTE* High frequency sound produced from the vibration of a piezoelectric


crystal. Sound energy is passed directly through the skin and causes heating
at interfaces such as junctions between different tissues. An example of this is
the junction between bone and muscle.

c. Primary physiologic effects:


i. Hyperemia
ii. Sedation
iii. Analgesia
iv. Increased regional vascularity
v. Increased tissue temperature
vi. Increased metabolic rate
vii. Arterial dilatation
viii. Increased capillary flow and hydrostatic pressure
ix. Increased collagen extensibility
x. Increased tissue inflammatory response
xi. Increased edema
d. Indications:
i. Analgesia
ii. Increased local vascular cutaneous flow
iii. Sedation
iv. Hyperemia
v. Accelerate the suppurative process
vi. Muscle spasticity
vii. Joint stiffness
vii. Contractures
e. Contraindications and precautions:
i. Tendency to hemorrhage
ii. Active hemorrhage
iii. Impaired sensation to pain and temperature
iv. Ischemia and vascular impairment
v. Non-inflammatory edema
vi. During the immediate (24-48 hour) post trauma period
vii. Known malignancies
viii. Patients with coma or paralysis
ix. Use with extreme caution if there is a threat of hemorrhage, in the
very old or the very young, and in those patients who have debilitated or
insensitive conditions
f. Modalities for superficial therapeutic heat:
i. Baths
ii. Electric heating pads
iii. Hot packs
iv. Hot water bottles
v. Compresses
vi. Paraffin
vii. Radiant heat
viii. Lamps
ix. Bakers
g. Modalities for superficial heat by convection:
i. Hot air baths
ii. Moist air baths
iii. Agitated water baths or whirlpool
h. Modalities for deep heat:
i. Short wave diathermy
ii. Microwave diathermy (caution in patients with pacemakers)
iii. Ultrasound
i. Infrared radiation:
i. Superficial heat source
ii. Bulbs
iii. Bakers
iv. Heat lamps
v. Heated carborundum rods
vi. Glowing wire coils with proper reflectors
j. Infrared radiation indications:
i. Subacute and chronic inflammation
ii. Contusions (if no threat of hemorrhage)
iii. RA
iv. Early joint stiffness
v. Degenerative or osteoarthritis
vi. Sprains/strains
vii. Neuritis
viii. Myositis
k. Infrared contraindications and precautions:
i. Acute inflammation
ii. Malignancy
iii. Ischemia
iv. Impaired sensation (pain and temperature)
v. The very young/elderly who are unable to respond to the effects of heat
l. Deep heat (short-wave diathermy) contraindications and precautions:
i. Hemorrhage or suggestion of hemorrhage
ii. Sensory loss
iii. Application over moist dressings
iv. Malignancy
v. TB
vi. Areas of ischemia
vii. Arteriosclerosis
viii. Thromboangiitis obliterans
ix. Metal implants
x. Foreign bodies
xi. Pregnancy/menstruation
xii. Suppuration
xiii. Special care in patient with pacemakers
xiv. Special care in the geriatric and pediatric patient
xv. Metal contact with skin or other tissues
xvi. Epiphyseal and developing bone
m. Deep heat indications:
i. Post trauma (after the threat of hemorrhage)
ii. Analgesia
iii. Pain and spasm
iv. Inflammation (subacute or chronic) associated with: bursitis, periostitis,
arthritis, neuritis, neuralgia, myositis, capsulitis, postfracture care, fibrositis,
tenosyovitis, tendonitis, sprains, strains
v. Rheumatoid arthritis
vi. Postdislocation
vii. Reflex vasodilation
n. Microwave diathermy contraindications and precautions:
i. Ischemia
ii. Hemorrhagic areas
iii. Tumors
iv. Impaired sensation
v. Debilitation
vi. Edema
vii. Use over wet dressings
viii. Metallic implants
ix. Pregnancy
x. Use over adhesive dressings
xi. Special care over bony prominences
xii. P.V.D.
xiii. Synovitis with excessive areas of synovial fluid
xiv. Systemic and local infections
xv. Patients with pacemakers
xvi. Care in the pediatric/geriatric patient
o. Microwave diathermy indications:
i. Deep heating of subcutaneous tissue and superficial musculature
ii. Musculoskeletal system for joint diseases (DJD, RA, bursitis and
tendonitis)
iii. Post trauma if no threat of hemorrhage
iv. Strains/sprains
v. Neuritis
p. Ultrasound indications:
i. Joint contractures associated with immobilization, trauma, scarring, RA
and DJD
ii. Pain
iii. Muscle spasm
iv. Tendonitis
v. Acute inflammation due to trauma
vi. Adjunct to exercise therapy
vii. Phonophoresis with indicated drugs
viii. Subacute sprains/strains ix. Calcific bursitis
x. Resolution of hematomas xi. Neuroma/neuritis
q. Ultrasound contraindications and precautions:
i. Basic contraindications for all forms of deep heat
ii. Avoid large fluid areas
iii. Hemorrhagic diseases
iv. Decreased sensation and insensitivity
v. Vascular insufficiency
vi. Tumors
v. Malignancies
vi. Over epiphyses
vii. Pacemakers
viii. With caution over implants both from a thermal and mechanical
concern
r. Hydrotherapy indications:
i. Muscle weakness
ii. Following amputation
iii. Following joint injury
iv. Paraplegia
v. Burns
vi. Selected post-surgical debridement
vii. Selected spastic palsy
viii. Selected flaccid paralysis
s. Whirlpool bath indications:
i. Chronic post-traumatic conditions
ii. Early joint stiffness
iii. Pain
iv. Painful scars
v. Adhesions
vi. Neuritis
vii. Arthritis
viii. Tenosynovitis
ix. Strains/sprains
x. Painful stumps
xi. Preliminary to massage, exercise and electrical stimulation
x. Burns
xi. Post CVA
xii. After fracture care
xiii. Vascular insufficiency
xiv. Decubitus ulcers
xv. Industrial medicine
xvi. After peripheral nerve injuries
xvii. Dermatologic debridement
xviii. Sports and dance medicine
xix. Post surgical rehabilitation
t. Whirlpool bath contraindications/precautions:
i. Acute phlebitis
ii. Osteoporosis (caution)
iii. Cardiac (caution)
iv. Immediate trauma
v. Excessive edema
vi. Vascular or neurologic impairment
vii. Non-inflammatory edema
viii. Caution in the geriatric/pediatric patient
u. UV radiation precautions:
i. Photo-ophthalmia of patient and operator
ii. Hypersensitive areas (skin folds)
iii. Unusual skin sensitivities
iv. Photosensitive drugs (sulfonamides, tetracyclines, green soap, dyes,
coal tar preparations, oral hypoglycemics, chlorthiazide diuretics,
phenothiazines)
v. UV radiation indications:
i. Local UV radiation for its anti bacterial /fungicidaI effects for superficial
ulcers, infections, dermatophytosis, and sterilization of fresh wounds
ii. Decubitus ulcers
iii. Boils/carbuncles/furunculosis
iv. Burns
v. Eczema
vi. Contact dermatitis
v. Erysipelas
vi. Felon
vii. Granulating tissue
viii. Infectious eczemoid dermatitis
ix Intertrigo
x. Psoriasis
xi. Leukoderma and vitiligo
xii. Urticaria
ix. Diabetic ulcerations (caution)
w. UV radiations contraindications/precautions:
i. Excessive exposure
ii. Photosensitivity
iii. Encapsulated pus without drainage
iv. Acute generalized dermatitis
v. Drug photosensitivity
vi. Malignancies
vii. Sarcoidosis
viii. Lupus erythematosus
ix. Herpes simplex
x. Caution with the cardiac, renal or with hepatic insufficiency

3. Manual modalities:
a. Massage indications: Classical massage, transverse friction massage
i. Relief of pain
ii. Arthritis
iii. Periarthritis
iv. Bursitis
v. Neuritis
vi. Fibrositis
vii. Mobilization of contracted tissue
viii. Reduction of swelling and induration
b. Massage contraindications:
i. Infection
ii. Cutaneous inflammation
iii. Tumors
iv. Burns
v. Skin diseases in relation to contamination and irritation
vi. Clotting disorders
vii. Fractures
viii. Thrombophlebitis
ix. Aneurysms
x. Thrombosis
xi. Fever
xii. Acute systemic disease
c. Manipulation indications:
i. Loss of range of motion
ii. The residuals of long periods of immobilization
iii. The residuals of trauma and/or capsular inflammation
d. Manipulation contraindications:
i. Immediate post trauma
ii. Joint effusion
iii. Acute inflammation
iv. Ligamentous rupture
v. Dislocation (unless used for repositioning)
vi. Fracture (unless used for repositioning)

4. Kinetic Procedures
a. Therapeutic excercise: Prescribed to increase and maintain joint and soft
tissue mobility, prevent adhesions and fibrosis, and to aid in soft tissue healing
i. Techniques:
• Resistive: Isotonic, isometric, and isokinetic (Kin-Com, Cybex, and
Biodex)
• P.N.F.: Proprioceptive neuromuscular facilitation (exercise which employs
combined neuromuscular training techniques
• Neuromuscular re-education (secondary to nerve injury)
ii. Indications:
• Preventing joint contractures
• Preventing joint adhesions
• Increasing joint range of motion due to trauma, muscle spasm, disease,
or states that limit function and mobility and prevent activity
• Arthritis and related conditions
• Post-amputation
• Neuromuscular and collagen diseases that produce contractures and
limited ranges of motion
• Paresis due to multiple conditions, such as CVA,
• Weakness and atrophy due to disuse and/or after immobilization
• Muscular weakness
• To strengthen muscles
iii. Contraindications/precautions/limitations:
• Acute active inflammation
• Active post-trauma
• Severe cardiovascular disease
• Severe pulmonary disease
• Metastatic malignancies
• Nonunion fracture sites (caution)
• Pain in the arthritides (caution)
• Severe joint and/or muscle pain (caution)
• Infection
b. Range of motion exercise:
i. Active: patient exercises on their own
ii. Passive: Joint mobilization or manipulation
iii. Active assist
5. Electrical modalities: This group of modalities delivers an electrical current
to the patient with various physiological effects. Among these effects are pain
relief, reduction of swelling, and muscle stimulation for enhancement of
strengthening or retardation of atrophy.
a. TENS: Used for reduction of pain. The Gate Control theory of pain states that
stimulation of large diameter nerve fibers may effect transmission of a noxious
stimuli before it reaches the cortex. This theory was used to explain TENS for
many years. More recently, other theories of central modulation have been used
to explain why TENS causes a reduction of pain
i. Indications:
• Chronic pain
• Acute pain associated with postoperative control
• Arthritis
• Bursitis
• Sprains/strains
• Tenosynovitis
• Metatarsalgia and related foot pain
• Painful motor and sensory nerve lesions
• Post amputation pain including phantom pain
• Peripheral neuritis
ii. Contraindications:
• Cardiac pacemakers
• Carotid sinus
• Insulin pump
• In the presence of electronic life support systems
• Over denuded skin
b. Iontophoresis: An electrical generator with DC current used for ion transfer
of medication into areas beneath the skin. This is often used for the purpose of
infliltrating a tendon, ligament or joint capsule with an anti-inflammatory or
anesthetic
c. Electrical stimulation: Is a high frequency current used primarily for
enhancement of shortening exercised
i. Indications:
• Injury to the muscles/tendons/joints
• Paralysis
• Sprains/strains
• Postfracture and after dislocation to help retard atrophy and restore
muscle function
• To help restore muscle tone and strength
• To help provide tissue contraction to milk tissues of excess fluids
• To relieve muscle spasm from trauma and associated muscle conditions
• Muscle pain and spasm
• Myositis
• Metatarsalgia
• Post polio
• Muscle re-education
• Following surgery to help prevent clots and embolism
• With ultrasound for trigger points and pain
• Denervated muscles
• To help retard muscle atrophy
• Synovitis
• Disuse
• Fibrositis
• Contractures
• Pain with ankylosis
• Stress and anxiety
ii. Contraindications/precautions:
• Individual patient reactions
• Allergy
• Placement of electrodes over malignancies
• Severe inflammation
• The head area
• Cardiac area
• Pacemakers
• Pregnancy
• Special concerns in the presence of vascular and/or neurologic
deficiencies
d. Galvanic currents:
i. Indications:
• Reduce pain
• Reduce swelling
• Increase local circulation
• Aid in nerve regeneration
• Soften scar tissue
e. Hi-volt: This high voltage, low amperage modality is used for pain reduction,
swelling reduction and muscle stimulation. Tissue impedance is overcome more
easily due to the high voltage, allowing deeper penetration of current. Some
theories state that inflammation may also be reduced by using this modality
f. Interferential: This modality relies on an extremely high frequency (2500-
4000Hz) to allow deeper penetration for the purpose of stimulation of nerves for
pain relief.

MODALITY INDICATIONS PRECAUTIONS


Electrical Stimulation Pain reduction Pacemakers
Monophasic D/C Muscle reeducation and Thrombophlebitis
High volt or interferential strengthening Malignancy
Muscle pump for edema Skin sensitivity
reduction Pregnancy
Increase ROM
Acute inflammation
Fracture healing
Electrical Stimulation Wound healing As above
Low voltage Fracture healing
Monophasic lontophoresis
Electrical Stimulation Pain reduction Pacemakers
Pulsed Muscle spasm Skin sensitivity
Biphasic A/C Increase ROM
TENS
TEAM
Diathermy Improve circulation Metal implants
Short wave Increase local metabolic Pacemakers
Micro Wave activity Malignancy
Increase tissue temperature Wet dressings
Reduce muscle guarding or Decreased sensation
spasm Acute inflammation
Reduce chronic Poor circulation
inflammation Pregnancy
Facilitate wound healing Over the eyes
Analgesia/sedation
Increase connective tissue
stentibility
Cryotherapy Acute inflammation Cold hypersensitivity
Ice Chronic inflammation Cold urticaria
CO2 Vasoconstriction Poor circulation
Cold Compression Control edema Healing wounds
Analgesia Hypertension
Reduce muscle guarding or
spasm
Improve ROM
Reflex vasodilation
Decrease local metabolic
activity
Superficial Heat Vasodilation Acute inflammation
Whirlpool Analgesia/Sedation Poor circulation
Paraffin Reduce muscle guarding or Malignancy
Hot Packs spasm
Infrared Increase metabolic activity
Facilitate tissue healing
Chronic inflammation
Ultrasound Increase connective tissue Infection
distensibility Malignancy
Deep tissue involvement Acute inflammation
Increase circulation Epiphyseal areas
Increase metabolic activity Early fractures
Reduce chronic Thrombophlebitis
inflammation Poor circulation
Reduce muscle guarding or Poor sensation
spasm Over the eyes
Improve healing Over the pregnant uterus
Phonophoresis Medication allergies
Intermittent Decrease edema Pain
Compression Reduce acute bleeding Circulatory impairment
Ultra Violet Acne Poor circulation
Wounds (septic or aseptic) Generalized dermatitis
Folliculitis Renal or hepatic
Tinea Pedis insufficiency
Pityriasis rosea Hyperthyroidism
Psoriasis Lupus
Pellagra
Herpes
Eczema
Diabetes
Psoriasis
Sensitizing medications
Chapter 36: Arthroscopy
Ankle Arthroscopy
ARTHROSCOPY
The most important indication for an arthroscopic procedure is less disability
than an open joint procedure. The advantages of arthroscopy are its use as a
diagnostic tool to see the pathology present when there is pain or disability in
the absence of a positive on MRI or CT, and ability to simultaneously correct
the condition. Hence it is a diagnostic and therapeutic procedure
Ankle Joint Arthroscopy
1. History:
a. Takagi (1918): First use in cadaveric knee- Tokyo, Japan
b. Bircher (1921): First arthroscopic knee examination (meniscus)
c. Burman (1931): First use in the USA (a cadaveric knee)
d. Takagi (1939): First successful ankle arthroscopy
e. Watanabe and Takagi: First operative procedure
f. Watanabe (1945): Developed smaller scopes to be used in joints
g. Heller and Vogler (1982): Podiatric applications of ankle arthroscopy

2. Patient evaluation and selection: Arthroscopy of the foot and ankle should
only be done after all conservative measures have been exhausted
a. Diagnostic indications:
i. Unexplained pain
ii. Swelling
iii. Stiffness
iv. Instability
v. Hemarthrosis
vi. "Popping"
b. Therapeutic indications:
i. Debridement of osseous bodies
ii. Repair of the ATFL iii. Irrigation
iv. Septic joint (flush out bacteria)
v. Osteochondral defects
vi. Soft tissue impingement
vii. Synovitis
viii. Arthrofibrosis
ix. Arthrodesis
c. Contraindications:
i. Cellulitis and local infections
ii. Fused joint
iii. Moderate DJD with restricted range of motion
iv. Severe edema
v. High risk medical patients

3. Instrumentation:
a. Arthroscopes: 2.7 mm, 4.0 mm with 30° viewing
b. Trochars, canulas and obturators
c. Accessory instruments (biopsy forceps, scissors, grasping forceps, suction
punch, meniscal knives, cutter, shavers, probes, suture delivery systems)
d. Irrigation system (normal saline, Ringer's lactate or acetate)
e. Other: Electrosurgical units, lasers, distraction devices (noninvasive or
invasive)

4. Ankle portals:
a. Anteromedial:
i. Medial to the anterior tibial tendon, saphenous nerve and vein
ii. Visualization of the medial gutter, medial transchondral bone margins
b. Anterolateral:
i. Lateral to the peroneus tertius and EDL
ii. Care must be taken to preserve the superficial peroneal nerve
iii. Visualization of the lateral gutter
c. Anterocentral:
i. Lateral to the EHL
ii. Care must be taken to preserve the anterior tibial artery and deep
peroneal nerve
d. Posterolateral:
i. Patient is usually prone
ii. Incision is lateral to the Achilles tendon
iii. Care must be taken to preserve the sural nerve and lesser saphenous
vein
e. Posteromedial:
i. Medial to the Achilles tendon
ii. Care must be taken to preserve the posterior tibial artery and nerve
iii. Visulalization to the posterior process of the talus and transchondral
lesions to the back of the talus
f. Accessory portals

5. Anatomy:
a. Anterior joint pouch:
i. Medial gutter:
• Medial malleolus
• Adjacent talar medial articular surface
• Anterior tibiotalar ligament (floor of the gutter)
• Posterior tibiotalar ligament (with valgus stress)
a. Anterior ankle (continued):
i. Medial bend
• Anterior tibial lip
• Medial talar shoulder
• Tibial plafond
a. Anterior ankle (continued):
i. Anterior joint line
• Sagittal groove of the talus
• Synovial recess (tibia)
• Capsular reflection (tibia)
a. Anterior ankle (continued):
i. Lateral talar shoulder
• Tbiofibular synovial recess
• Tibiofibular synovial fringe
• Anterior inferior tibiofibular ligament

b. Anterior ankle (continued):


i. Lateral gutter
• Mdial fibular articular surface
• Anterior talofibular ligament
• Posterior talofibular ligament (with varus stress deep in the lateral gutter)
b. Posterior ankle:
i. Posterior joint pouch:
• Posterior tibial lip
• Posterior talar dome
• Sagittal groove of the talus
• Psterior tibiofibular ligament
• Labrum of the posterior tibiofibular ligament
• Medial bend
• Medial malleolus
• Posterior tibiotalar ligament
b. Posterior ankle (continued):
i. Posterior ankle via an anterior view
• Posterior tibiofibular ligament
• Posterior capsule wall
• Medial bend
• Medial gutter
• Posterior dome of the talus

6. Ankle Pathology:
a. Soft tissue:
i. Synovitis
ii. Fibrous bands
iii. Meniscoid bodies
• Wollins lesion
iv. Adhesive capsulitis
b. Cartilage:
i. Subchondral erosions
ii. Chondromalacia:
• Collins classification:
Grade 1 : Fraying
Grade 2: Fibrillation, fissuring
Grade 3: Extensive fissuring
Grade 4: Cartilage loss
• Goodfellows classification (superficial degeneration):
Type 1 : Superficial erosion
Type 2: Loss of superficial layer
Type 3: Exposed subchondral bone
Type 4: Deep matrix exposed
• Goodfellows classification (basal degeneration):
Stage 1 : Cartilage softening
Stage 2: Blister formation
Stage 3: Exposed matrix
Stage 4: Bone exposed
iii. Chondral lesions:
• Bauer and Jackson classification:
Type 1: Linear crack
Type 2: Stellate lesion
Type 3: Cartilage flap
Type 4: Avulsed with exposed bone
Type 5: Fibrillation
Type 6: Fibrillation plus subchondral erosion
c. Osseous pathology:
i. Osteochondral bodies
ii. Subchondral bone cysts

7. Specific ankle joint pathology:


a. Osteochondral lesions
b. Avulsion fractures of the ankle:
i. Anterior/posterior tibial lips
ii. Ligamentous insertions

c. Medial impingement lesions


d. Transchondral ankle fractures:
i. Berndt and Harty classification:
Stage 1: Compression
Stage 2: Partially detached
Stage 3: Totally detached
Stage 4: Avulsed
e. Tibial lip fractures
f. Impingement exostosis

8. Specific procedures done arthroscopically:


a. Ankle arthrodesis
b. Lateral ankle stabilization

9. Post-operative care and rehabilitation:


a. With exploration and general debridement: compressive dressing and
immediate weightbearing
b. With ligamentous repair: range of motion within 24 hours, patient in an
aircast 6 weeks post-weightbearing
c. With debridement for chondral defect: non-weightbearing 8 weeks with active
range of motion
d. With arthroscopic arthrodesis: 8 weeks in hard non-weightbearing cast
followed by 2 weeks in weightbearing splint

10. Complications of ankle arthroscopy:


a. Scope breakage
b. Infection
c. Cartilage injury or creation of multiple small bodies
d. Hemarthrosis due to trochar damage
e. Compartment syndrome
f. Polyneuritis of the ankle
g. Venous laceration
h. Painful scar
i. Nerve laceration
j. Recurrence of symptoms
Chapter 37: Laser Applications in
Podiatric Surgery
Lasers and Laser Physics
Fundamentals
Tissue Interraction
Laser Safety

Clinical Applications in Podiatric Surgery


The CO2 Laser
The Nd: YAG Laser
The ARGON Laser
The KTP Laser
Other Surgical Lasers
LASER APPLICATIONS IN PODIATRIC SURGERY
Applications of lasers to medicine and surgery have increased exponentially over
the past decade. This technology has become established in the medical community
and has become the standard of care for many procedures. Lasers have justified their
utilization by the improved clinical outcome in the delivery of comparably more traumatic
and invasive procedures. Some procedures are not possible without the precision or
uniqueness of this modality.
There are a great variety of laser types and delivery systems, each having
indications unique to the desired tissue response. Fundamental to the surgeon in
selecting the wavelength, power and control to produce the intended effect, with safe
handling of the instrument, is a knowledge of laser physics for this tissue interaction.

LASERS AND LASER PHYSICS

HISTORY
1. The Quantum Theory:
Max Planck
1910
Light is quantified in Photon units
the basic unit of light (6.625 x 10-27 erg sec (cm2/sec))
2. Stimulated Emission Theory:
Albert Einstein
1917
Basis of laser light
3. First laser developed, demonstrated and patented
Theodore Maiman
Ruby Laser
1960

UNITS OF MEASUREMENT
1. Frequency Expressed in Cycles per Second (CPS)
Hertz (Hz)
2. Wavelength The measurement of one crest to another of a particular frequency

3. Length Meter = the basic measurement unit


Prefix
centi (cm) = 1 x 10-2 meters = .01 meters
-3
mili (mm) = 1 x 10 meters = .001 meters
micro (um) "micron" = 1 x 10-6 meters = .000001 meters
nano (nm) = 1 x 10-9 meters = .000000001 meters
4. Time Second = the basic measurement unit
Prefix
mili (ms) = 1 x 10-3 seconds = .001 seconds
micro (us) = 1 x 10-6 seconds = .000001 seconds
-9
nano (ns) = 1 x 10 seconds = .000000001 sec
pico (ps) = 1 x 10-12 seconds = .000000000001 sec
5. Power Watts (W) = The basic measurement unit
Power density = Watts per centimeter squared (W/cm2)
Joules (J) = Watts x Seconds of power on tissue
FUNDAMENTALS
1. The wavelength is the key to tissue absorption, laser delivery systems and to laser
safety.
2. Comparison with other modalities:
Scalpel --> Mechanical pressure. Local effect. Controlled crushing.
Electrocautery --> Electrons. Conduction through isotherms. Global effect.
Radiosurgery --> Radio Frequency transmission. Local effect.
Laser --> Photon absorption. Specific to tissue content. Thermal precision.
3. The mnemonic "LASER":
L ight
A mplification by
S timulated
E mission of
R adiation
4. Laser frequencies most commonly used are in the infrared and visible spectra.
5. These are non-ionizing photonic radiation.
6. No lead shielding is required.
7. Exception: Excimer (UV) lasers are ionizing.
8. Laser light is NOT a natural phenomena.

UNIQUE CHARACTERISTICS OF LASER LIGHT


Coherent
Monochromatic
Collimated

Coherent = All crests


of wavelengths line up.
Crests and troughs are
equidistant in
time and space. This eliminates wavelengths canceling each other out and producing
interference patterns which would decrease its intensity. This enables very efficient
power production. Coherent light, (compared with incoherent, conventional light) can be
focused to an exact single point. ie: 200 W of incoherent conventional light will illuminate
a room. 200 W of coherent laser light will rapidly carve through the cement wall of the
room.

Monochromatic = Pure, single color. Responsible for the interaction of tissue


chromophores producing a specific effect. ie: CO2 laser to incise and ablate amelanotic
tissue, Nd:YAG for deep tissue penetration, Argon penetrates epidermis.

Collimated = Emitted stream of photons is linear, and does not diverge. This also
eliminates wavelengths producing interference patterns reducing power.

COMPONENTS OF A LASER
1. Partially reflecting mirror 97%
2. Fully reflecting mirror 100%
3. Lasing media
4. Xenon flash lamp
5. high frequency Switching system
6. High voltage power supply
7. Delivery system - Articulating Arm, Fiberoptic, waveguide 8. Lense
9. Aiming Laser (HeNe), if required, depends on laser type

Conventional light radiation Laser light radiation


+ Multiwavelength - polychromatic + Pure - monochromatic
+ Divergent + Collimated
+ Coherent + In phase
+ Spontaneous Emission + Stimulated Emission

THEORY of LASER OPERATION


1. Spontaneous Emission (conventional)
2. Stimulated Emission (laser)
CREATION OF LASER LIGHT V. CONVENTIONAL LIGHT
1. Lasers are classified by the type of active media used in the laser tube.
ie: CO2 laser tube filled with CO2 (excitable media), N2, and He gasses.
Nd:YAG is a Yttrium, Aluminum and Garnet crystal
doped with Neodymium as the excitable media.
2. Atoms are stimulated to rise from a lower energy shell to a higher shell,
3. Then fall back to emit a specific monochromatic wavelength of light.
4. These waves reflect in the laser media randomly at first, then become coherent
together by being amplified by reflecting between the mirrors.
5. Once their energy exceeds the threshold of transmission through the partially
reflecting mirror, laser radiation is emitted in a linear, collimated, array.
6. Frequency doubling media is also used to change laser wavelength.
ie: Tunable dye or KTP (Potassium, Titanium, Phosphate) laser.
The KTP crystal pumps a KTP crystal. Efficiency drops to about 30% of input.

Nd:YAG Laser -----------> KTP Crystal------------> output


1060 nm 532 nm

DELIVERY MECHANISMS
1. Low frequencies = longer wavelengths = far- and mid-infrared.
Articulating arms, or internally reflecting waveguides are used.
2. At near-infrared, 2100 nm and above (Ho:YAG laser) fiberoptics contain these
frequencies having a higher index of refraction.
3. Fiberoptics are constructed of quartz (Aluminum dioxide), silicon dioxide or silver
halides, coated with a plastic sheath.
4. Lenses, or contact light scalpels of selective focal lengths, can be integrated into the
terminal end of the fiberoptic system.
5. The bare fiber is also used for free beam ablation work.
TRANSMISSION MODES
1. Desirable laser energy distribution energy follows a Gaussian curve.
2. Energy decay falls exponentially on either side of the curve.

TEM00 has a narrow spot size TEM01 small spot 0.3 mm at best
true Gaussian curve. called "near Gaussian"
0.2 mm diameter spots not desirable
appropriate for cutting can be used for ablation.

TISSUE INTERACTION
1. This is THE most important aspect of lasers in medical science.
2. Tissue interaction with the specific laser wavelength is the KEY to laser selection.

TRANSMISSION CHARACTERISTICS THROUGH TISSUE


1. reflection
2. transmission
3. scattering
4. adsorption

** Absorption of specific wave-


length by specific chromophores
is key.
CLINICAL TISSUE INTERACTION PHENOMENA
The effect on tissue by thermal lasers commonly used in Podiatry is both:
1. power and
2. time dependent:

POWER DENSITY
1. Is the standard of expression in documenting laser power to tissue.
2. Expressed in W/cm2.
3. P.D. maybe constant while tissue spot size and power varies.

This allows physicians to communicate standard terminology, allows for preference. It is


the STANDARD OF CARE: in operative reports describing laser use
It is necessary for communicating standard measurement in the scientific community.

A typical example using 14 Watts with a 0.2 mm diameter contact tip or spot size (which
is 0.1 mm radius)
Traditional Algebraic:
WATTS 14 4.46 446
----------- = --------------------- = ----------------- = --------------------- = 44,600
pi x r2 0.12 0.01 0.01
3.14 x ----- -----
102 100

where: 0.1 is the radius


10 is the conversion factor of 10 mm/cm

a shortcut algebraic:
WATTS 14
127 x ----------- = 127 x --------- = 44,450
d2 0.22
WATTS PER CM2 Chart

WATTS TIP DIAMETER (mm) or CO2 Spot Size (mm)


0.1 0.2 0.4 0.6 0.8 1.0 2.0 3.0
-----------------------------------------------------------------------------------------------------------------------
4 50,955 12,739 3,185 1,415 796 510 127 57
5 63,694 15,924 3,981 1,769 995 637 159 71
6 76,433 19,108 4,777 2,123 1,194 764 191 85
7 89,172 22,293 5,573 2,477 1,393 892 223 99
8 101,911 25,478 6,369 2,831 1,592 1,019 255 113
9 114,650 28,662 7,166 3,185 1,791 1,146 287 127
10 127,389 31,847 7,962 3,539 1,990 1,274 318 142
11 140,127 35,032 8,758 3,892 2,189 1,401 350 156
12 152,866 38,217 9,554 4,246 2,389 1,529 382 170
13 165,605 41,401 10,350 4,600 2,588 1,656 414 184
14 178,344 44,586 11,146 4,954 2,787 1,783 446 198
15 191,083 47,771 11,943 5,308 2,986 1,911 478 212
16 203,822 50,955 12,739 5,662 3,185 2,038 510 226
17 216,561 54,140 13,535 6,016 3,384 2,166 541 241
18 229,299 57,325 14,331 6,369 3,583 2,293 573 255
19 242,038 60,510 15,127 6,723 3,782 2,420 605 269
20 254,777 63,694 15,924 7,077 3,981 2,548 637 283
21 267,516 66,879 16,720 7,431 4,180 2,675 669 297
22 280,255 70,064 17,516 7,785 4,379 2,803 701 311
23 292,994 73,248 18,312 8,139 4,578 2,930 732 326

TIME
The gating of the flash lamp may be:
1. C.W. Continuous Wave - Continuously on
2. Single Pulsed - Continuous on for a preset period
3. Superpulsed - Rapid pulsing at peak power at 250 - 1000 Hz.
Average power is determined by
1. pulse width and
2. repetition rate
This allows tissue to undergo "thermal relaxation"
4. Ultrapulsed - Much higher RF (Radio Frequency) switching
nanosecond pulse width.
More thermal precision.
5. Q-switched - Very high peak power with picosecond pulse width

THERMAL RELAXATION = Interval between pulses to allow dissipation of energy.


Minimum interval is 1:10 ratio on-off.
LASERS APPLICABLE TO PODIATRIC SURGERY

WAVELENGTH USE IN PODIATRY DEPTH OF PENETRATION FUNCTION


10,600 um CO2 Noncontact: 0.1 mm Cutting
Far IR Dissection Ablation
Derm.Pathologies Coagulation
Nail Pathologies
1,060 um Nd:YAG Bare Fiber 6-8 mm Ablation
Near IR Deep tumor destr. Coagulation

Nd:YAG Contact-tip: 50-200 u Cutting


Dissection

2,100 um Ho:YAG Near-contact: 0.4-0.6 mm Ablation


Mid IR Cartilage and bone

4881514 um Argon Noncontact Dermal vessels Photoablation


Verruca

532 um KTP Noncontact: Dermal vessels Photoablation


Cutaneous vascular Cutting
Verruca
Contact: 1-2 mm
Dissection

478 urn Copper- Noncontact: Dermal vessels Photoablation


Vapor Cutaneous vasc.
lesions

ie: CO2 is strongly


absorbed by water, therefore
superficial penetration.
Holmium is absorbed by
water but not as much as
C02, so deeper tissue penetration. Argon and KTP are absorbed by Hb and
chromophores.
Nd:YAG (bare fiber) is not absorbed by anything, so it penetrates.

LASER SAFETY
1. Reference: "ANSI 136.3 Publication" on laser safety
2. Four Classes of Lasers:
Class I - No ocular damage with direct viewing
Class II - Ocular damage with prolonged exposure
Class III - Ocular damage to the eye before the eye can blink
Class IV - Medical lasers. Great potential and hazard to the eye
and skin. Ignites combustible materials. Beam = fire hazard.
3. Dedicated laser nurse in O.R. controlling use and monitoring laser safety
4. Key operated, controlled access.
5. Room shields to outside personnel.
6. Eye protection for patient and all OR personnel.
7. Wavelength specific eye protection hung outside the door
so that people can enter the room safely.
8. Adequate smoke evacuation appropriate to tissue atomization.
9. Dual stage filtration, carbon and 0.2 um filter.
10 Coaxial visible aiming beam for use with invisible light lasers.
11 Proper filtering mask.

EYE PROTECTION
1. Impact is direct or reflective.
2. Minimal hazard zone is determined to be 6 feet away from the reflected zone.
3. O. D. = Optical Density, expressed as an exponent of power of 10.
ie: O. D. of 5 is 100 x as absorbent as O. D. of 3
4. Always expressed as an O.D. at a specific wavelength
5. Recommended to surgeon (direct viewing field) - minimum O.D. of 5 @ wavelength.
6. Recommended to OR personnel (indirect) - minimum O.D. of 3 @ wavelength.
7. Conventional glass or plastic
glasses will stop CO2
8. Recommend UV coating to
stop the ultraviolet fluorescence
off carbonization.
9. Eye protection still necessary
for endoscopic procedures, fibers
could break.
10. High density filters on
endoscopes, arthroscopes,
waveguides.
11. The reflected light transmits
directly back to the surgeon.

CO2 - corneal burn.


Surgeon's cornea replaceable.
Argon, KTP - retinal damage,
Irreversible.

Nd:YAG absorbed in the vitreous humor causing posterior cataract formation.


Reversible? Doubtful.

DRAPING FOR LASER SURGERY


1. Surgical site draping for CO2 laser use should be wet towels.
2. Drapes should be dry for the Nd:YAG procedures.
(Water is a transmitter at the YAG frequency and absorbed at CO2 frequency.)
3. Laser nurse - laser on standby when not immediately using the instrument avoids
accidental discharge.
4. Multiple foot switches - Bovey, power saws, table, can be confused with the laser.
5. A defocused beam has more of a tendency to start fires.
6. A focused beam will have a tendency to drill.
7. A prefocused beam will have a tendency to accelerate the hole it is drilling.
The power density increases approaching the focal point.
8. Always have water on hand, to extinguish a potential flame.
9. Anodized instruments (blackened or roughened) are helpful to diffuse the beam,
minimizes reflections but does not eliminate.
10. Endotracheal tubes should be coated with Mirasil (noncombustible material).

HAZARDS OF THE LASER SMOKE PLUME


1. Epidermis with the CO2 laser creates a great deal of smoke.
2. The shock waves backscatter verrucoid particles which can be inhaled.
3. Vaporized tissue and debris is liberated by tissue atomization
4. Studies show live intact DNA recovered from the laser plume.
5. Hazard in AIDS and hepatitis patients
Hazard in patients with infectious lesions, i.e. warts.
6. Lesions have been reported by Dermatologists, Podiatrists, and Gynecologists.
7. Formaldehyde also produced, large number of other carcinogens.

High power plume evacuators are required with dual stage filters.
1. Charcoal filter for carcinogens, smell.
2. Filtering down to 0.2 microns to filter out virus
a. The key is good technique in smoke evacuation.
b. Keep the smoke evacuator close to the area of surgery.
c. A laser mask will filter down to 0.3 microns.
d. These measures reduce nearly all of the hazards of the viral particles

II - CLINICAL LASER APPLICATIONS IN PODIATRIC SURGERY


STANDARD OF CARE
1. OPERATIVE REPORT - Include laser type power density calculation.
ie: "Procedure: Austin Bunionectomy, left foot (Soft tissue with CO2 laser): With the CO2
laser set at 33,000 W/cm2 power density, a linear incision was ..."

2. CONSENT FORM - Include the laser type or wavelength used and the intended
application of the laser if there is conventional instrumentation used. ie: "(Usual
description of surgery), soft tissue with CO2 laser"

3. ETHICS IN ADVERTISING - Differentiate the application of the laser ie: "Laser


assisted" bunionectomy, or "Laser for soft tissue"

Advertise straightforward what laser procedures (warts, nails) are done if also
advertising conventional procedures (bunionectomy) that are not performed with laser
assistance.

Public misconceptions: No incision, laser cuts bone.


You will never lose a patient because of an honest disclosure of a procedure.

THE CO2 LASER


PROPERTIES OF THE CO2 LASER
1. Active media is C02, helium, nitrogen
Carbon dioxide is the excited media
Helium and neon are catalysts
2. High absorption in water, Tissue mostly water
therefore superficial absorption
"What-you-see-is-what-you-get"
Low scattering in tissues
3. Invisible beam at 10,600 nm far-infrared, helium-neon aiming beam necessary

ADVANTAGES OF USING THE CO2 LASER


1. Thermal precision
Maximum impact on target and minimum damage to adjacent tissue
2. Absolute hemostasis minimizing postoperative edema. Coagulates small blood
vessels, lymphatics (<0.5 mm diameter) Minimizes pathways spreading malignant cells
3. Accelerated healing of internal tissue
because of lack of mechanical trauma.
Fibroblasts are less stimulated.
therefore skin sutures need to be left in tissue a few days longer, however internal
scarring is less
tissue remodeling is minimized due to little scar formation. earlier joint range of motion.
4. Minimal postoperative pain
sealing axonal tubules in small cutaneous nerves
5. Pain may increase several days postoperatively if patient weightbearing sealed
exoplasm from nerve endings is under increased hydrostatic
pressure
electrolytes stimulate neural discharge
6. Sterilization of the target sight
Inactivate any bacteria, fungi, or virus
7. No foreign body reaction
8. Versatile - Operates in CW or pulse mode to vaporize or incise tissue
9. Portable and inexpensive relative to other lasers
10. Minimal amount and cost of disposable laser items per case.
11. Handpieces easily sterilizable
12. Least expensive laser
SELECTION OF LASER PARAMETERS
1. Appropriate power, spot size, power duration, and angle to tissue
2. Ablational work: spot size less than 2-3 mm in diameter
3. Incisional work: spot size less than 0.3 mm in diameter

DISADVANTAGES
1. Cost, power, alignment, control, additional informed consent
2. Smoke evacuation system
3. Combustible materials risk, extra drapes, higher protection
4. Special training for physician/staff
5. Learning curve
6. Credentialling process/extension of privileges if hospital use

PROCEDURES PERFORMED USING THE CO2 LASER ASSIST


1. Plantar Verruca Ablation
2. Porokeratoma Ablation
3. Nail Matrixectomy Ablation
4. Fungal Nail Treatment - Drilling through nail plate
5. Heel Fissure Debridement
6. Ulcer Debridement/Sterilization
7. Incisional Procedures for soft tissue component (of neuroma, bunion, etc.)

THEORY OF CO2 LASER TISSUE INTERACTION


1. Controlled, highly localized vaporization.
2. Energy is absorbed by water.
3. High conductivity minimal to adjacent tissue damage.
4. Avoid tissue carbonization - increases and conducts thermal effects Immediately
seen. Worse problem at low power densities. Global tissue temperature and thermal
conductivity. Wipe this off with a damp gauze.

CO2 LASER PROCEDURES


TECHNIQUE OF CO2 LASER ABLATION
1. Power Density over 1000 W/cm2
2. Larger spot size- 2-3 mm

The following diagrams, illustrate two methods: linear and circular overlap.
The goal is an evenly ablated surface.
1. Circumscribe lesion by 2 mm peripherally
2. Curette representative area and send biopsy for pathology.
3. Deep channels should be avoided.
4. Do not penetrate dermis in verrucoid lesions.
5. If you have a 0.2 mm spot size at focal point, defocus to 1.0 mm.
For example, 20 watts with a 1 mm spot size equals 2540 watts/cm2 power density.
Scarring results from dermal penetration
IPK's and porokeratosis are focally penetrated to the dermis.
1. Need to lase to subdermal fat.
2. 75% cure rate, somewhat higher than conventional applications.
3. Little scarring.
4. More focal treatment is required at higher power levels.
5. Remove char by lavage or sponge

TECHNIQUE OF CO2 LASER FOR INCISION/EXCISION


1. Power density greater than 6,000 watts/cm2 preferred
2. Small spot size, maximum 0.3 mm diameter,
3. TEM01 lasers are not able to produce less than 0.3 mm spot at focal point.
Thus they are not appropriate for making incisions.
4. TEM00 lasers are available to deliver 0.1 mm, but commonly 0.2 mm.
Example:
a. 20 watts with 0.2 mm spot size equals 63,500 watts/cm2 power density.
b. Technique: smooth rapid continuous motion
c. In focus
d. Traction and countertraction perpendicular to incision.
5. Traction/countertraction of the incised area will enable smooth tissue plane
delineation.
6. Retrace path to achieve desired depth.
7. Important: Characteristics of individual lasers vary greatly.
8. Test on a tongue blade first.
Depth should be a little over halfway through with minimal charring.

NOTES:
1. A TEM00 laser produces a very different effect compared to a TEM01 machine
2. A superpulsed laser has a variety of pulse settings to achieve the same P. D.
3. The ultrapulsed lasers cut faster at lower power settings.
4. These are characterized by very short duration RF pulsed power supplies

Power densities are a general rule of thumb and should be adjusted to


1. each wavelength,
2. the particular instrument and
3. the type of tissue undergoing surgery.

HEMOSTASIS
1. By Coagulation:
Defocus to a spot size greater than twice the vessel diameter
Use a Power density less than 1500 watts/cm2
Technique: defocus beam to increase spot size and direct beam at site
2. By Dessication (thermal contraction):
Spot size 1 mm
Power density as with coagulation
Technique: direct beam to tissue immediately adjacent

FOCUSED, FREE BEAM LASER APPLICATIONS


1. In Focus: Incision
2. Defocus: Debulking
3. Greatly defocused: Coagulating
4. Prefocused: Avoid altogether

OVERLASING
Significant problem to inexperienced user is "Overlasing"
Definition: delivery of an inappropriate amount of laser energy to target tissue
or to the surrounding tissues
producing unintended tissue destruction.
(Immediately visualized with CO2 lasers.)

CAVERNOUS HEMANGIOMA
1. Considered ablative surgery requiring high power densities.
2. This is a highly vascular tumor.
3. Nd:YAG (bare fiber) is appropriate for deep penetration
4. Causes deep thermal vascular stenosis.
5. CO2 is not good for coagulation for these tumors, but it can be used.
6. KTP and Argon are more appropriate for superficial vascular lesions.

KELOID AND HYPERTROPHIC SCAR


1. Excellent indication for CO2 laser excision because of lack of fibroblast stimulation.
2. Superficial epidermal incision with the CO2 laser, NOT with the steel scalpel.
3. Avoid charring, will delay healing.
4. Refer to technique for incision/excision

LASER ASSISTED OSSEOUS PROCEDURES


1. Advertise as laser assisted bunionectomy.
2. Lasers used for soft tissue dissection only.
3. Not FDA approved for osseous work.
4. Used for incision, soft tissue dissection and capsular work.
5. Result is less postoperative pain, edema, and earlier range of motion.
6. Fascial layers - very little water content
therefore is more transmissible at this wavelength
7. Excellent for capsular incision. Earlier ROM.
contraindicated for periosteal dissection hemostasis of ALL vessels.
This seals the metaphyseal arteries and slows periosteal healing
8. Remember the delayed skin healing effect
9. Leave the sutures in a few days longer
10. Fibroblast stimulation is minimal thus scar formation is minimal
11. Better cosmetic result.

BONE AND CARTILAGE


1. Accidentally hitting the bone cortex will take 16-20 weeks to heal. Solution: Debride
damaged cortex immediately. Damage is usually superficial.
2. Carbonization in a joint will set up severe chronic inflammation. Solution: Lighten up
on capsular dissection in this area Irrigate thoroughly postoperatively, as always
3. Excellent application for subchondrodesis procedures
Instead of using K-wire to drill use CO2 at high P.D. for 0.5 seconds. Space closer
together with less mechanical disturbance to cartilage

LASER TREATMENT OF VERRUCA


1. CO2, Nd:YAG, Argon, KTP 532 can be used.
2. Selection or combination treatments depend upon clinical presentation.

The technique is to ablate in a layering method


1. Anesthesia, avoid epinephrine.
Avoid directly sublesional.
2. Drape area using moist towels or laser safe drapes.
3. Power density CO2 laser: 6,000 to 21,000 watts/cm2.
Decrease for light skinned and thin skinned individuals
Also reduce power density for thin areas on dorsal areas of the foot
4. Circumscribe lesion taking 2 mm min border of normal appearing tissue
at the periphery. Viable verruca in this tissue.
5. Do this in focus.
6. Submit representative biopsies.
7. Plow multiple interspersing furrows and crosshatch these to an even base.
8. Next wipe area with a sterile, moist gauze to remove char. Avoid relasing char.
9. Repeat lasing and wiping until dermal/epidermal separation occurs. Epidermis will
appear to peel away from the dermis.
10. Several passes are required on the plantar surface of the foot. Desired depth is
papillary dermis.
11. Healing will occur from basal cells in the dermal papillae.
12. Relase superficial areas until an homogeneous depth is encountered
to rete ridges.
13. Photocoagulate in a defocused mode.
Coagulate the surface to a very light haze.
This also sterilizes the surgical bed of viral particles.
14. Work is complete. Do not revaporize. Inspect with magnification.
15. Silvadene cream and sterile dressing for 24 hours.
Avoid occlusive dressings.
Extra strength Tylenol for small lesions
Hydrocodone 2.5 mg i-ii Q 4 h prn for large masses
16. Expect moderate drainage for 3 days to 1 week.
Wound closes completely in 1 month entirely healed.
In 2 months no signs of treatment are usually visible.

* Treat lesions less than 1 cm from each other as one lesion


* Do not leave a bridge of healthy skin between.

Handling large lesions:


i.e. large, mosaic verruca.
1. Keep depth of penetration even.
2. Circumscribe and divide the lesion into quadrants.
3. Lase each quadrant individually.
4. If the patient is supine, work from posterior to the anterior
If bleeding is encountered be sure it does not drain over the surgical site.
5. Be prepared with extra smoke evacuation filters.

To accomplish hemostasis, if needed::


1. reduce the power density and "brush" hemorrhagic area.
2. Power density can be reduced by backing off to a defocused mode.
3. Suction blood away first - laser does not coagulate free blood

Postoperative Care:
1. Patient seen 3 days to 1 week
2. Patient allowed to clean twice daily with H202 and bandaid
exception: large lesions require redressing until drainage decreases.
3. Normal bathing after first redressing.
Accommodative pad if needed.
4. Stop dressing when drainage ceases, no dressing at night.
5. Monitor patient for at least 6 months due to the nature of HPV.
6. Success rate easily 90% after learning curve reached.

Complications:
1. Infection--rare, laser sterilizes the bed.
2. Overlasing

NOTES:
3. Increased pain--result of overlasing.
4. Increased bleeding--result of overlasing
5. Increased scarring--result of overlasing
6. Scarring--Penetration of dermis

LASER NAIL MATRIXECTOMY


1. No epinephrine
2. No tourniquet - will have good hemostasis
3. Avulse the nail, do not ablate with laser
4. Power settings: 0.2 mm spot size, 125 mm focal length lense, 10 watts CW
5. Aim at 45 degrees, under proximal nail fold for acisional technique
6. Outline matrix and circumscribe
to periphery of distal phalanx
condyle avoid lasing bone.
7. Lase the matrix in layers
achieving a uniform layer of
desiccated tissue. Debride with a
dermal curette to the next layer of
matrix. Stop when coming close
to bone. Several passes are
necessary
8. Keep site very dry and free
from blood.
9. Dilute phenol may be used as
an adjunct, but the laser replaces
the blade.

1. Techniques for missing part of the matrix are just a probable with laser or blade
2. Characterized as a blind procedure.
3. Burning bone may result in periostitis, very rare.
4. Recurrence after learning curve partial permanent procedure, hallux, 0.5%.
5. These rate of results after learning curve reached.
6. Usually recurrence is keloid, hypertrophic scar formers, and psoriatic patients
7. Patients with high epidermal growth turnover
8. Total permanent if recurrent in these patients

POSTOPERATIVE CARE
1. Leave sterile dressing on 24 h
Patient to change at home
Patient to clean twice daily with H202. No soaks.
2. Some tissue necrosis 1 week
3. Patient to keep dry for 3 days
4. Patients seen 24 hours - 3
days postop
5. Bandaid dressing
6. Normal healing
7. Discontinue dressing and soaks
when drainage ceases, generally 2 weeks
8. Allow it to drain 1-3 weeks until it stops draining spontaneously.
9. Total permanent drain more on the 3 week margin, lesser digit partials for a week or
so.
COMPLICATIONS
1. Increased pain
2. Increased drainage
3. Delayed healing
4. Soft tissue infection
5. Thermal osteitis
6. Osteomyelitis
7. Overlasing is generally the culprit of all those complications.

PREVENTION OF COMPLICATIONS
1. Use appropriate power density
2. Keep the hand piece moving or apply power with periodicity
3. Keep exposure time on a given spot to a minimum
4. Don't relase over char
5. Always know where the beam is going, especially these blind procedures

FROST AND WINOGRAD TECHNIQUE


1. Do not use lasers to cut the nail- excessive heat.
Use incisional power densities as described for incisional procedures
2. Laser is 90 degrees to the skin, P.D.= 40,000 W/cm2
Then decrease when performing matrixectomy
3. Incision would be the same otherwise as the Winograd
please refer to that section within this review book
4. Laser incision is made straight back past the eponychium
Second curvlinear incision around soft tissue pathology
Remove hypertrophied nail lip and granuloma tissue.
5. Closure with 4-0 Nylon suture.
6. Tourniquet is not necessary.

LASER TREATMENT OF ONYCHOMYCOSIS


1. No anesthesia required
2. Laser "mottling" techniques
3. Object is to punching holes in the top nail plate
4. This allows topical medications to penetrate
a. Laser settings to just barely fire through a tongue depressor.
b. These settings should be just subthreshold for patient feeling any heat
c. Laser must be in a pulsed mode
d. holes drilled 4-5 mm apart
e. Three separate treatments 6 weeks apart.
f. Topical antifungal applied BID

SUBTOTAL MATRIXECTOMY
1. Anesthesia as before
2. The plate is always removed conventionally
3. Lasing is performed on the total matrix
4. however only scanned to 50% of the depth
5. The idea is to remove only part of the nail matrix to result in a thinner nail
SUBUNGUAL HEMATOMA
1. No anesthesia
2. Same procedure as mottling technique
3. Slightly higher power Density may be used
4. Lase a couple of holes until the nail plate is penetrated.
5. Hematoma will isolate thermal effects.

LASER TREATMENT OF GRANULOMAS


These respond very well to laser treatment
1. Ablate the granuloma in a crisscross pattern, the same as verruca
2. Alternate with a moist gauze until normal tissue is encountered
3. Good hemostasis should be encountered throughout the procedure
4. No chemocautery, bovey, or hemostatic solutions are necessary
5. Once the granuloma is gone the minimal bleeding encountered stops
6. Defocus, relase, apply sterile dressing.
7. Home treatment and followup as with verruca.

CAUTION IN REVISIONAL PROCEDURES


Scar tissue, if encountered, has less water content.
Therefore reduce power density when you relase this type of tissues. Otherwise excess
vaporization penetrating tissue planes may occur.

THE Nd:YAG LASER


GENERAL DESCRIPTION
1. 1060 nm, near-infrared, separate HeNe aiming beam
2. Most frequently used laser besides the CO2 laser
3. This is a general surgical instrument used most of the time by general, thoracic,
plastic, and urologic surgeons
4. See absorption chart - Nd:YAG is centered between other common medical lasers
5. Unique characteristic - the "window" of low absorption
and high transmissibility
YAG is poorly absorbed by hemoglobin, chromophores, protein, or water.
6. 99% of Podiatric use is with contact laser scalpels
7. Able to coagulate vessels < 0.2 mm diameter
8. User friendly, but tip selection, type and size must be understood
9. Power settings are very important
10. Endoscopically/arthroscopically compatible

MODES OF OPERATION
1. Non-contact mode - used for debulking and treating deep tumors.
higher power levels required ie: 40 W
2. Contact-tip mode - highly localized scalpel form similar to CO2 laser
lower power ie: 12-16 W (frosted tip)
4-6 W (nonfrosted)
3. Contact-tip is very superficial absorption, cutting only at the tip.
"What-you-see-is-what-you-get"
Noncontact is indicated for deep
Contact is used for incision and tumors.
excisional work.

THE INSTRUMENT
1. Instrument is portable, conventional nondedicated power OK
2. C.W. mode only
3. Flashlamp excites Neodymium-doped crystal of Yttrium, Aluminum, Garnet
4. Fiberoptic delivery system, air or water cooled within the sheath
5. Large variety of handpieces, general surgical one is used
6. Large variety of contact tips, fiber and handpiece combinations

THE CONTACT TIP


1. Developed for 3-dimensional feedback, feels similar to a conventional scalpel Better
control of dissection
2. Converts light energy from a laser into heat energy.
3. Very precise hot knives, tissue effect 50-200 microns
4. Types of conical tips: Frosted, clear, ceramic, titanium coated

There are many combinations of tips:


1. Sapphire scalpels interchangeable - screw onto handpiece
2. Integrated quartz tips with fiber and handpiece also used
3. Scalpel must be in contact with tissue when power on
or flare out of expensive tip will occur
4. Flare-out threshold temperature:
Sapphire scalpel - 2000 degrees F
Quartz scalpel - 1000 degrees F
5. Tip shapes - Chisel, flat, round, cylindrical, hook
a. Long or short conical used in Podiatry
b. Tapered conical tip concentrates energy
c. Polished lense at distal end
d. Available radii are 0.2-1.2 mm diameter
i. when calculating P.D. don't forget diameter -> radius
ii. and mm to cm conversions
6. Frosted and nonfrosted available
a. Frosted - distal end roughened to allow lateral radiation except at lense and allows
coagulation during dissection
b. Nonfrosted - tip is clear
i. radiation only at distal lense
ii. appropriate for very fine dissection at low power levels
c. Procedures are scalpel specific

SURGICAL APPLICATIONS

LASER SCALPEL STEEL SCALPEL


Rapid dessication Controlled crushing
Seals small nerves Smears small nerve endings
Seals small vessels No microcoagulation
Cell necrosis is small Cell necrosis is moderate
Cuts with Light Energy Cuts with physical pressure
High precision Normal tactile feedback

ADVANTAGES OF Nd:YAG OVER SCALPEL


1. Less postoperative pain
2. Less bleeding of smaller vessels/lymphatics - less swelling
3. Sterilizes surgical site reducing chance of infection
4. Less cell necroses
5. Less fibroblastic stimulation - faster tissue remodeling

Nd:YAG MEDICAL INDICATIONS


1. Situations contraindicating tourniquet
particularly where a dry field is essential
2. Dissection of delicate tissue planes in all axis requiring contact
3. Patients where surgical trauma may stimulate adverse reactions
Collagen-vascular disease (ie: SLE), gout, R.A., etc.
3. Patients with platelet, hematogenous and vessel disease,
sickle cell, phlebitis
4. Elderly patient exhibiting capillary fragility

PODIATRIC MEDICAL INDICATIONS FOR Nd:YAG SCALPEL


1. The Nd:YAG laser scalpel decreases the surgical pathophysiology of
a. edema in a dependent extremity
b. leakage of intravascular fluid in the surgical site
of the foot from hydrostatic pressure
c. nerve microtrauma and axonal depolarization
d. local surgical cell necroses (v blade)
e. scar formation
f. nosocomial infection
2. Extremity surgery
a. Hydrostatic pressure, terminal perfusion. weightbearing
b. structure undergoing reconstructive surgery are all
c. factors complicating foot and ankle surgery
3. Hypertensive patient with peripheral edema
4. Plastic reconstruction
5. Wet cases
6. Cases where visualization must be optimized
ie: Nerve decompression within ganglion complex
7. Any situation where cell necrosis must be minimized

CONTRAINDICATIONS
Defer these cases until the learning curve plateau is reached
1. Digital surgery - cannot justify utilization
Instrument overkill for procedure
2. Revisional surgery - actually indicated
but these surgeries carry a higher risk by default
If successful laser gets the credit
If not successful surgeon gets blammed
Public expectations of lasers are high
3. Any bone work - not FDA approved
This is not a wavelength for this
4. Medical-Legal cases - Same idea as revisional Surgery
5. Amputation - Same idea as revisional surgery
6. PVC cases - Same idea
7. Acute Trauma cases - no time to call in laser team

INDICATIONS FOR FROSTED AND NONFROSTED CONTACT TIPS

PROCEDURE CONTACT-TIP COMMENT


Nerve releases and Neuroma Nonfrosted Thermal Radiation
Neurectomy Nonfrosted Lateral Radiation
Bunionectomy (soft tissue) Frosted Limit capsular dissection
Ganglions Nonfrosted Particularly those
adjacent to muscle
Tendon transplants/lengthening Nonfrosted Long remodeling time
Heel spur (soft tissue) Frosted Excellent visualization
Plantar Fasciotomy Nonfrosted Around calcaneus only
General Podiatric Surgery Frosted Coagulation during
dissection

INAPPROPRIATE Nd:YAG PROCEDURES


1. Wrong tip = wrong procedure
ie Neuroma sx with frosted tip
Induces thermal periostitis in adjacent metatarsals
This can be done with frosted at short power applications
2. Nail matrixectomy = burns periosteum
use CO2
3. Warts = can use, but it's more easily treated with CO2
4. See contraindications
GENERAL CONSIDERATIONS IN APPLICATION OF THE Nd:YAG LASER
1. Drapes are dry. Water transmits this wavelength.
2. Separate mayo stand for fiberoptics with expensive tips and power equip
3. Notch filter glasses are the best eyewear protection, best visibility
4. Select general surgery handpiece
5. Select tip size and frosting based on presurgical plan
6. Laser nurse will connect fiberoptics to launch pad on laser
7. Calibrate instrument and hand off calibration cone, now contaminated
Select power level, C.W. mode
8. Incision is made with a steel blade only to the dermis
When you see the whiteness of the dermis - stop
Contact tip is held 45 degrees to tissue
New frosted tips need 2 seconds at full power to "age"
9. Traction - countertraction throughout procedure
10. Room suction be used for the minimal smoke plume
much less than CO2 laser
11. Deeper dissection now before using forceps Use traction - countertraction
12. Dissect in linear strokes. Avoid burying the tip.
This laser needs less pressure than a steel blade Tactile feel is like a hot knife through
butter So let the laser do the work
13. Repeat dissection strokes until each plane of tissue complete
14. Dissection may be adjacent to vessels, stay 3-4 mm from nerves at high power
levels 14-16 W
15. Hemostasis of larger vessels can be with the laser
alternate on either side, observe coagulation
16. Capsular dissection may be made right over cartilage
with no damage to cartilage
17. Dissect only the periosteum/capsule you intend to discard
Good hemostasis, but seals off periosteal vessels
18. Remainder of capsule/periosteum done with blade
19. Bone work with conventional power instrumentation
20. Watch on-time during intermetatarsal neuroma surgery
Avoid thermal periostitis in adjacent metatarsals
21. Seal nerve endings in neuroma sx. with the contact tip
Prevents stump neuroma formation
22. Minimal char formation seen
Very dry and atraumatic surgical site seen
23. Closure is conventional, dressings conventional
24. Sutures remain in a few days longer
Expect macroscopic bleeding due to tourniquet reflux hyperemia

REALISTIC EXPECTATIONS
1. Learning curve is steeper than CO2 Laser
2. Postoperative bruising still seen
3. Swelling, and pain still seen - although diminished
4. Macroscopic bleeding present but diminished
5. Be ready for the unexpected -
New technology presents new situations
6. Do simple cases first
THE ARGON LASER
GENERAL DESCRIPTION
1. Dual wavelength output:
Blue 488 nm
Green 514 nm very close to KTP 532 nm (pure green)
2. 1 to 2 mm depth of penetration.
3. Operates as a coagulation device, not used for cutting
4. Argon and KTP pass epidermis to absorb in the dermal hemoglobin selectively
Nd:YAG and CO2 do not absorb in the region of the Hb curve
5. Fiberoptic delivery system
6. collimated handpiece, freebeam fiber, contact
7. Aiming beam is a low power argon beam, hard to see through OD 3 or 5 glasses
8. 30 degree divergence on the KTP fiber, 2 degree divergence on the Argon fiber.
9. 488 nm filter is used to filter out green component
10. Hemoglobin Absorption is a bimodal curve
11. Ideally the wavelength should fall on the peak absorption of this curve and be
maximally transmissible through other tissues

MECHANISM OF ACTION
1. Chromophores on the bottom of the foot are minimal
2. They pose little problem because the epidermis, dermis
basal layer is transparent to this wavelength
3. Absorption at this wavelength is low
first absorbed in the hemoglobin within the vessels of the reticular dermis
4. vessels are stenosed via selective photoablation.
a. Able to coagulate vessels less than 1 mm in diameter.
b. Indicated for tissue coagulation and necrosis procedures (acisional)
c. KTP laser, 532 nm can be used also for vascular stenosis.
d. Deeper dermal structures, such as capillary hemangioma, other lasers are indicated
for this such as the free beam Nd:YAG.

EYE PROTECTION
1. Optical Density (O.D.) minimum of 5 at 488 nm.
2. Unfortunately, these glasses block out the aiming beam
The aiming beam is a low level intensity treatment beam.
3. Visible light eye protection radically alters the colors of the surgical field

SURGICAL APPLICATIONS
INDICATIONS FOR THE ARGON LASER
1. This treatment is very useful for incisionless surgery
It is highly favored by the patient,
particularly in the large verrucae on the plantar aspect of the foot and the posterior
aspect of the heel normally a CO2 laser would leave an ulcerative defect Immediate
shoe gear
2. Multiple disseminating lesions or mosaic warts on the plantar foot
3. Vascular lesions of a superficial nature

4. Patients having communicable diseases when a bloodless field is desired


5. It is not indicated for highly fibrotic and scarred verrucoid lesions. Scar tissue
transmits this frequency giving a painful result

ADVANTAGES
1. Minimal exposure to blood--this is an incisionless procedure.
2. Decreased laser plume about 5% of that with CO2 laser A smoke evacuator is still
required
3. Good treatment for immunocompromised patients
4. Faster than CO2 laser, i.e. a 45 minute procedure for the CO2 laser for
verruca plantaris would take 5 minutes with the Argon laser
5. It is repeatable
6. Sterile preparation unnecessary.
Surgeon still should be gloved for isolation from lesion contaminants.

ARGON LASER DESTRUCTION OF VERRUCA


1. Object = delivery of energy to the superficial dermis - papillary plexus These are the
vessels feeding the wart.
The wart is an epidermal structure, not a dermal structure. It is however fed by vessels
from the dermis.
2. Anesthesia peripheral to lesions and without epinephrine
3. Thick sections of epidermis should be debrided previous to treatment This minimizes
epidermal carbonization.
4. Inject peripherally - do not blanch skin from the injection pressure
5. Collimated handpiece is used with 600 nm to 1 mm diameter fiber
6. 5 degree to 30 degree divergence. Focusing handpieces are available.
7. Bare fiber is held 1-2 cm from tissue
8.2-4 mm spot, 5.5 watts, 0.5 seconds for
the plantar foot.
May be used continuous mode and
brushed when a good technique is
adapted.
9. Selection of appropriate power density
is very important.
3 watts for thin skin, 6.5 watts for thick skin
10. Include 2 - 3 mm border peripheral to
the wart,
11. Carefully check this tissue for a
"blanching effect".
12. Allow for a 3 to 5 second delay in this
blanching
13. This is a result of the coagulation
of the superficial dermal vessels. No vaporization occurs

NOTES:
1. Some carbonization is normal in thick epidermis
Avoid charring this by continuous circular motions.
2. When blanching occurs, this is the proper setting.
Also the proper rate of hand piece movement.
3. This is a time dependent phenomena.
4. After the vasculature is coagulated the chromophores have absorbed the wavelength.
5. If blanching is not encountered, do not increase power,
do not slow down handpiece movement.
6. Repeat the same movement of the handpiece over the area.
7. When proper parameters are determined, continue treatment beyond the test area.
8. The result is not only power and spot size, giving P.D., but time dependent.

POSTOPERATIVE CARE
1. Accommodative pad prn
No dressing necessary.
Patient can put his shoe and sock on and walk out of the operating room.
2. Hydrocodone 2.5 mg i-ii Q 4-6 h prn pain
3. Blistering likely to occur in 3 to 5 days.
Patient may incise and drain this at home.
After I&D, patient is to leave the skin on, for a protective barrier.
4. At one week a black necrotic skin component will form
This lasts 3 weeks and spontaneously sheds.
5. Check patient in 3-4. Recheck in 10 weeks.
6. Should fully heal within 4 to 5 weeks.
No scarring should be seen.
A slight hypopigmentation may be observed.
7. Ulceration is not possible with this laser as the chromophores, hemoglobin
and oxyhemoglobin stops the absorption in the superficial papillary plexus.

THE KTP LASER


GENERAL CHARACTERISTICS
1. Very similar to Argon laser. Single wavelength 532 nm (v. 514 nm Argon).
2. Difference: KTP can cut tissue, contact mode
3. Very useful in selected tissue (below)
4. A frequency doubled Nd:YAG laser
5. Blanching of the skin also seen similar to the Argon laser
6. Note indications for vascular tumors:
Argon superficial
KTP moderate
Nd:YAG (bare fiber) deep
7. Absorption coefficient is slightly increased over the Argon
A.C. - The distance it takes for the radiation to diminish 90% in tissue.
8. Contraindicated in surgery in close proximity (0.5 cm) to thin cortical bone
Vessels of the bone will absorb this wavelength, necrose vascular supply
9. Contact mode and free beam mode

FIBER PREPARATION
Fiber preparation is done previous to each case
1. Fiber must be cleaved:
a. Optical fibers have a crystalline nature
b. A cleaver is used to penetrate the sheath and score the fiber cortex this sets up a
stress riser so the fiber can be snapped
c. the end of the fiber is inspected for a flat, even surface
emitted laser light should be circular and symmetrical
d. The sheath must be stripped.
e. 4 mm recommended by manufacturer, but emits too much lateral light
f. so strip 2-3 mm instead, but enough so the sheath doesn't melt
g. So leave it to Cleaver to strip off the sheath for fiber preparation

SURGICAL APPLICATIONS
KTP TREATMENT OF VERRUCA
1. Same as for Argon
2. Treatment is slightly deeper
3. Blanching is similar but has more significance of penetration depth

KTP APPLICATIONS TO PLANTAR FASCIOTOMY


1. MECHANISM OF ACTION
a. KTP is a nonthermal laser
b. Selective wavelength absorption
c. Operates in contact mode by specific photoablation of protein and hemoglobin.
d. Pericalcaneal tissue = adipose, muscle, and plantar fascia
Advantage - transmission through clear adipose.
Advantage - plantar muscle contact - very little bleeding
e. Muscle is highly vascular and plantar fascia is separated off
Usually muscular bleeding is considerable
Absorption in muscle is superficial - stopped by Hb
f. Lower power levels required

2.. THERMAL LASER PROBLEMS INDICATING KTP LASER


a. Thermal lasers, C02, Nd:YAG --> adipose photohydrolysis and liquefaction
b. This water and fat liquefaction interferes with laser cutting
c. Disadvantage CO2 laser - strongly absorbed by water and no cutting occurs
d. Disadvantage Nd:YAG laser - water transmits and energy is disseminated.
e. Surgical site in heel spur work is deep and visibility must be optimized
Particularly with endoscopic size incisions

3. DISADVANTAGES OF KTP LASER


a. Retinal hazard
b. Fiber preparation before every case
c. Cannot dissect in close proximity to bone
d. Inefficient laser - requires dedicated 220 V 50 A line
e. Larger zone of necrotic damage and zone of coagulation than the CO2
f. Takes a long time to dissect through vertical septa and plantar fascia
g. Must take care not to deliver much energy into muscle (well absorbed)
h. Learning curve
OTHER SURGICAL LASERS
Other medical lasers available with properties, delivery systems unique to each Podiatry
is already using these, but be familiar with basic lasers first

Ho:YAG LASER
1. a near-contact laser being used for resecting calcaneal spurs
2. good for endoscopic and arthroscopic work
3. transmits through water
4. also indicated for cartilage ablative procedures for joint restoration

COPPER VAPOR LASER


1. Known largely for treating port wine stain
and congenital cutaneous vascular pathologies
2. Two wavelengths - yellow and green
3. yellow for superficial vascular
4. green for deeper vascular
5. dermatological surgery
6. very inefficient, large laser, long warmup period

Q-SWITCHED LASERS
1. used to be used in ophthalmic procedures
2. Q-switched Ruby and Q-switched Nd:YAG for tatoo removal
3. no anesthesia necessary
4. multiple treatments necessary
5. little scarring results, mild hypopigmentation

EXCIMER LASER
1. UV laser, 0.2 mm absorption
2. Ionizing radiation
3. Cardiac catheterization, osteotomy, corneal sculpting
4. High frequency, short wavelength means high precision

Er:YAG LASER
1. Mid infrared, 0.1 mm absorption
2. Bone surgery
3. Difficulty in fiberoptic delivery - fragile and toxic
BIBLIOGRAPHY
1. Arndt, Kenneth A., "Argon Laser Therapy of Small Cutaneous Vascular Lesions",
Journal of the American Academy of Dermatology, vol. 118, April 1982, pp. 220-224.

2. Borovoy, Mathew; Fuller, Terry A.; Elson, Lawrence M.; Laser Safety in Podiatry", The
Journal of Foot Surgery, 1985, vol. 24, no. 2, pp. 136-138.

3. Borovoy, Mathew; Klein, Jeffery T.; Fuller, Terry A.; "Carbon Dioxide Laser
Methodology for Ablation of Plantar Verrucae", vol. 24, no. 6, 1985, pp. 431-437.

4. Cacciaglia, G.B.: Reigelhaupt, R.W.; "Effectiveness of Lasers on Plantar Papillomas:


A Preliminary Study", Journal of Foot Surgery, vol. 24, no. 1, 1985, pp. 477-481.

5. Carlson, Bruce A., and Pyrcz, Robert A., "Lasers in Podiatry and Orthopaedics",
Nursing Clinics of North America, v. 25, No. 3, September 1990, Pg. 719-723.

6. Carlson, Bruce A.; Pyrcz, Robert; "Human Papilloma Virus-Induced Lesions: Their
Treatment and the Evolution of an Alternative Laser Application", Current Podiatric
Medicine, November 1989, pp.9-12.

7. Carlson, Bruce A., "Complications Associated with Laser Surgery", Clinics in Podiatric
Medicine and Surgery, vol. 4, no. 4, October 1987, pp. 823-828.

8. Chromey, Paul A., "The Significance of Power Density in Applying the CO2 Laser",
Current Podiatric Medicine, September 1986, pp. 20-22.

9. Chromey, Paul A., "The Application of CO2 Laser to Soft Tissue Tumors", Current
Podiatric Medicine, May 1986, pp.24-27.

10. Collis, Sheldon; Rowland, Roberta N.; "Lasers For Podiatry Principles and
Language", Current Podiatry, April 1984, pp. 33-34.

11. Kaplan, Isaac, "The CO2 Laser In Clinical Surgery: Past, Present, and Future",
Journal of Clinical Laser Medicine and Surgery, pp. 341-343, vol. 9, no. 5, 1991.

12. Kelly, Peter F.; "Nd:YAG Contact-Tip Laser Reduces Pain from Foot Surgery",
Clinical Laser Monthly, Volume 10, No. 1, January, 1992.

13. Kelly, Peter F., "Nd:YAG Contact Tip V. Cold Steel Applications in Podiatric Foot and
Ankle Surgery", American Society for Laser Medicine and Surgery, Supplement 4, 1992.

14. Kelly, Peter F.; "The Light Scalpel - Nd:YAG Laser Contact-Tip", Issue 24, 1992, The
Laser Letter, International Society of Podiatric Laser Surgery, Doylestown, PA.

15. Kelly, Peter F.; "The Nd:YAG Laser for the Podiatric Surgeon", The Laser Letter,
Issue 25, 1992, International Society of Podiatric Laser Surgery, Doylestown, PA.
16. Kelly, Peter F.; "KTP Laser Application to Calcaneal Spur Surgery", Clinical Laser
Monthly; Volume 11, No. 3, April, 1993.

17. Kelly, Peter F., "KTP Laser Application to Calcaneal Spur Resection and Plantar
Fasciectomy", American Society for Laser Medicine and Surgery, Supplement 5, 1993

18. Landsman, Mark J.; Mancuso, John E.; Abramow, Steven P.; "Laser's Use in Bone
and Joint Surgery, Clinics in Podiatric Medicine and Surgery, vol. 9, no. 3, July 1992, pp.
721-737

19. McDowell, Brian A., "Carbon Dioxide Laser Excision of Benign Pedal Lesions,
Clinics in Podiatric Medicine and Surgery, vol. 9, no. 3, July 1992, pp. 617-632.

20. Mueller, Terrance J.; Carlson, Bruce A.; Lindy, Marc P.; "The Use of the Carbon
Dioxide Surgical Laser for the Treatment of Verrucae", Journal of the American Podiatry
Association, vol. 70, no. 3, March 1980, pp. 136-141.

21. Nicholson, Ronald A., "Two Techniques Described using C02 Laser for
Matrixectomy", Laser Practice Report, vol. 7, no. 7, pp. 1 S-2S.

22. Pyrcz, Robert A.; Carlson, Bruce A.; "Lasers in Podiatry and Orthopedics", Nursing
Clinics of North America, vol. 25, no. 3, September 1990, pp. 719-723.

23. Wasserman, Gerald, "Treatment of Morton's Neuroma with the Carbon Dioxide
Laser", Clinics in Podiatric Medicine and Surgery, vol. 9, no. 3, July 1992.

SPEED-READING BIBLIOGRAPHY
1. "Continued efforts to enhance the utilization of laser technology and the ability of laser
education at teaching hospitals are key for the future." Lanzafame, Raymond, J.;
Hinshaw, Raymond, J.; "Laser Education, Laser Usage, and Surgical Attitudes: A
Challenge for the Future", Pg. 279-81, Journal of Clinical Laser Medicine and Surgery,
Volume 10, No. 4, 1992.

2. "It (the CO2 laser) routinely provides a bloodless surgical field as well as unusual
surgical precision." Fairhurst, Mark V.; Roenick, Randall K.; Brodland, David G.;
Subspecialty Clinics: Dermatology, "Carbon Dioxide Laser Surgery for Skin Disease",
Mayo Clinical Proceedings, Vol. 67, Pg. 49-58, 1992.

3. "Thermally induced tissue destruction is accurate with little damage to surrounding


normal tissue because the coherent, collimated, monochromatic beam of light can be
focused to a very tiny point using an optical lens system. Histologically, the area of
tissue necrosis adjacent to the laser incision is less than 0.1 mm, usually 50-70 microns.
This facilitates healing with reduced scarring. The zone of cellular damage varies from
0.3 to 0.5 mm." Chromey, Paul A., Current Podiatric Medicine, September 1986, Pg. 20-
22.
4. "Carbon dioxide laser offers many advantages demonstrated in this study, including
minimal bleeding, improved healing, reduced edema, improved postoperative
discomfort, minimal scarring and minimal infection." Cacciaglia, G.B., Reigelhaupt, R.W.,
"Effectiveness of Lasers on Plantar Papillomas: A Preliminary Study", Journal of Foot
Surgery, Vol. 24, No. 1, 1985, Pg. 477-481.

5. "Since the beam affects well defined areas of the skin, there is minimal necrosis of
adjacent tissue; consequently, postoperative pain, edema, and scarring are minimized."
Mueller, Terrance J.; Carlson, Bruce A.; Lindy, Mark P.; "The Use of the Carbon Dioxide
Surgical Laser for the Treatment of Verrucae", Journal of the American Podiatry
Association, Vol. 70, No. 3, March 1980, Pg. 136-141.

6. "Podiatry started using the laser for the excision of Morton's neuroma and for
incisional approaches to bunionectomy and other podiatric procedures." "The same
advantages were found to be present for incisional procedures: less bleeding, pain, and
postoperative edema." Peyrcz, Robert A., Carlson, Bruce A., "Lasers In Podiatry and
Orthopedics", Nursing Clinics of North America, Vol. 25, No. 3, September 1990, Pg.
719-723.

7. "Utilizing the CO2 laser in neuroma surgery for making incisions and sealing the nerve
stump decreases postoperative pain and healing, allowing patients to resume normal
ambulation faster than with conventional scalpel surgery." Wasserman, Gerald, Clinics
in Podiatric Medicine and Surgery, "Treatment of Morton's Neuroma With the Carbon
Dioxide Laser", Vol. 9, No. 3, July 1992, Pg. 671-686.

8. "The contact method of performing endoscopic and open surgery with the Nd:YAG
laser opens a new era in laser surgery ... with cutting capabilities previously only seen
with the CO2 laser." "The CO2 laser operating at a wavelength of 10,600 nm with energy
outputs of 100 watts is effective at cutting and coagulation of SUPERFICIAL blood
vessels." Joffe, Stephen N.; Schroder, Tom; Lasers in General Surgery, Year Book
Medical Publishers, Inc., Laser Center of America, Cincinnati, OH, Pg. 125-130, 1987.

9. "We concluded that CO2 laser surgery for hemophiliacs has a confirmed place for
modern laser technology." Santo-Dias, A.; "CO2 Laser Surgery in Hemophilia
Treatment", Journal of Clinical Laser Medicine and Surgery, Pg. 297-301, Volume 10,
No. 4, 1992.

10. "Because of the large amount of water in body tissue, this laser (CO2) will cause a
vaporization of the tissue at the focal point and seal the small blood vessels and
lymphatics. The laser creates an incision that leaves residual tissue undamaged."
Kaplan, Isaac; "Twenty Years of CO2 Laser Surgery: A Review and Update", Journal of
Clinical Laser Medicine and Surgery, Pg. 57-60, Volume 11, No. 2, 1993.

11. "Certain advantages of the use of the CO2 laser stand out: absence of hemorrhage
and cellular vaporization which permit, due to a perfect visibility, appreciation at every
moment of the quantity of tissue that needs to be removed." Dourov, Nicolas; Nammour,
Samir; "Removal of Benign Tumors Using the CO2 Laser", Journal of Clinical Laser
Medicine and Surgery, Pg. 109-113, Volume 10, No. 2, 1992.
12. "This action results in a fine hemostatic incision leaving the residual tissue relatively
undamaged." "The treated areas heal rapidly because the skin appendages escape
permanent damage." Kaplan, Isaac; "The CO2 Laser In Clinical Surgery: Past, Present,
and Future", Journal of Clinical Laser Medicine and Surgery, Pg. 341-343, Volume 9,
No. 5, 1991.

13. "Surgical laser technology has been available for nearly 30 years and is being used
increasingly in many surgical disciplines including orthopedic surgery." Cahill, Sandy;
Kopta, Joseph A.; Kosanke, Stanley D.; Rayan, Ghazi M.; Stanfield, Denver T.; "Effects
of Rapid Pulsed CO2 Laser Beam on Cortical Bone In Vivo", Lasers in Surgery and
Medicine, Pg. 615-620, Volume 12, No. 6, 1992.

14. "The lack of wound contraction, scarring, and good reepithelialization combined with
precise tissue destruction makes CO2 laser surgery ideal for this procedure when
compared with conventional techniques." Keng, S. B.; Loh, H. S.; "The Treatment of
Epulis Fissuratum of the Oral Cavity by CO2 Laser Surgery", Journal of Clinical Laser
Medicine and Surgery, Pg. 303-306, Volume 10, No. 4, 1992.

15. "The advantage of the CO2 laser technique was that it produced minimal thermal
damage to the surrounding tissues." Fallouh, Hayel; Sultan, Raymond A.; "Combined
CO2-Nd:YAG Radiation in Liver and Anorectal Diseases", Journal of Clinical Laser
Medicine and Surgery, Pg. 255-263, Volume 10, No. 4, 1992.

FURTHER READING
1. Sherk, Henry H., Editor, Lasers in Orthopaedics, J.B. Lippincott Company,
Philadelphia, PA, 1990.

2. Ballow, Edward B., D.P.M., Editor, Laser Surgery of the Foot, First Edition,
International Society of Podiatric Laser Surgery, Doylestown, PA, 1988.

3. Joffe, Stephen N.; Schroder, Tom; Lasers in General Surgery, Year Book Medical
Publishers, Inc., Laser Center of America, Cincinnati, OH, 1987.

4. "Lasers In Podiatry and Orthopedics", Nursing Clinics of North America, Vol. 25, No.
3, September 1990.
PHOTODYNAMIC THERAPY "PDT"

MECHANISM OF OPERATION
1. Requires an injection, dissemination and systemic absorption of protoporphyrins
2. Malignant cells take up the protoporphyrin
These can be visualized under UV light
Patient must remain away from all light during this treatment session and have a
photosensitivity up to six weeks post treatment
3. Laser radiation is applied at a specified frequency, usually red light area. and for a
specified time - result is Joules to tissue
4. Wavelength depends on protoporphyrin
5. Results is a single oxygen produced which destroys malignant tissue selectively

BIOSTIMULATION "BIOSTIM"
1. A few milliwatts hitting mitochondria stimulating thermal mechanism.
2. No heat is produced.
3. Mitochondrial chromophores.
4. This is FDA investigational.
5. Applied to wound healing, to nerve regeneration, and chronic pain--analgesic.
6. Elicits a systemic effects
7. 0 milliwatts, tunable dye, helium-neon, 12 joules per week for 30 weeks
8. Applied to rheumatoid arthritis, trigeminal neuralgia, osteoarthritis, sciatica, diabetic
neuropathy.
9. Has 60 to 85% of pain relief
10. 5-hydroxyindolacetic acid, 5-HIAA produced
Urinary HIAA output increase correlated with pain relief
11. Systemic effects:
stimulation of human lymphocytes
Decreased pain and inflammation of distant ion irradiated sites Increased urinary 5-
HIAA, product of serotonin metabolism
12. Dose:
50 milliwatts, tuneable dye or helium-neon laser, 12 joules per week for 30 weeks.
13. Proposed mechanism:
absorption of light by photoreceptor or chromophore in the mitochondria activates the
respiratory chain, resulting in a cellular response.
14. Primary process:
a. Electron train excitation
b. acceleration of electron transfer in redox pairs (activates and stimulates).
c. Transfer of excitation energy from oxygen
to single oxygen (oxidative effect inhibiting healing).
d. Respiratory chain components are probably the primary photoacceptors.
e. Flavins, cytochromes, cytochrome oxidase.
f. Respiratory chain in a unitary dynamic system can be acted upon
at various points causing change in the whole state of response.
Summary:
1. Effects are dose dependent. Higher energies seem to be damaging.
2. Coherent light is preferred versus non-coherent.
3. Narrowed band monochromatic light preferred, i.e. helium-neon coherent and
monochromatic light
4. Competing wavelengths cancel the effects, therefore coherent light is preferred.
5. Transcutaneous irradiation penetrates deeply enough to
produce generalized effect in many cases.
6. 4 joules/cm2 penetrates approximately 1 cm.
7. Local radiation with systemic effects.
i.e., 5-HIAA production from serotonin metabolism.
8. Depends upon the physiological status of cell before radiation.
9. The biostimulation effect is not always possible.
Chapter 38: Specific
Radiological Pathology
MRI Pathology
SPECIFIC RADIOLOGICAL STUDIES
MRI Pathology
1. Posterior tibial tendon tear (grade 2): Photographs 1-4
Discussion: Photograph #1 is a T1 weighted image with a low signal around the
posterior tibial tendon. The dark center is tendon, (clear arrow) which is not
completely dark but has some areas of grey, which indicated an intrasubstance
tear. The low signal around the tendon (dark arrow) indicates that there is fluid
around the tendon. Photograph #2 is a T2 weighted image, which shows
significant fluid around the posterior tibial tendon, indicating a Grade 2 tear.

Photograph # 1
Photograph #2
Photograph #3 is a T1 weighted image (sagittal view) showing irregularity with
in the tendon (lobulated dark area) with the white area below indicating fluid
accumulation. Photograph #4 is a T2 weighted image (sagittal view) showing a
white area of increased signal intensity behind the posterior tibial tendon
consistent with a grade 2 tear

Photograph #3
Photograph #4
2. Achilles tendon tear: Photograph 5
Discussion: A T1 weighted image (sagittal view) revealing a linear signal within
the Achilles tendon of higher signal intensity, which is consistant with an
intrasubstance tear and overall thickening about 4-6 cm above the insertion of
the tendon

Photograph #5
3. Achilles tendon rupture: Photograph 6 & 7
Discussion: Photograph #6 is a T1 weighted image (sagittal view) revealing a
large signal of low intensity surrounding the Achilles tendon in Kager's triangle.
Photograph #7 is a T2 weighted image (sagittal view) showing a signal of bright
intensity (acute hemorrhage) with complete loss of continuity of the Achilles
tendon

Photograph #6
Photograph #7
4. Aneurysmal bone cyst: Photograph 8 & 9
Discussion: Photograph #8 is a T2 weighted image (axial view) through
the calcaneus and tarsus. In the calcaneus there is a higher signal above and a
lower signal below in each of the pockets. This is consistant with an
aneurysmal bone cyst, which is multiloculated. The reason for the difference in
signal intensity within the pockets, is the blood constituents settling to the
bottom and serosanguinous materials settling to the top. Photograph #9 is a T1
weighted image (sagittal view) with a low homogenous signal from the
calcaneus. Diagnosis can be made from the T2 image

Photograph #8
Photograph #9
5. Tophaceous gout: Photograph 10
Discussion: Photograph # 10 is a T1 weighted image (sagittal view) with a low
intensity signal visable over the dorsal aspect of the foot invading into the lesser
tarsus. The T2 signal (not shown) has a brighter (higher) signal but not what
fluid filled lesion would look like. This is consistent with tophaceous gout.

Photograph #10
6. Giant cell tumor: Photograph 11
Discussion: Photograph #11 reveals a T1 weighted image (axial view) showing a
low instensity signal mass inferior to the sesmoid and flexor apparatus. This is
a giant cell tumor over the flexor hallucis longus

Photograph #11
7. Avascular necrosis of tarsal navicular: Photograph 12 Discussion:
Photograph #12 is a T1 weighted image (sagittal view) revealing a signal of low
intensity throughout the navicular. This is consistent with a avascular necrosis

Photograph #12
8. Osteochondritis dissecans: Photograph 13-15
Discussion: Photograph #13 is a T1 weighted (sagittal view) demonstrating a
low signal lesion on the lateral talar dome. Photograph #14 is a T2 weighted
image (sagittal view) demonstrating a high signal from the lateral talar dome.
Photograph #15 is a T1 (coronal view) demonstrating a low signal intensity
lesion of the anterior talar dome. This is consistent with osteochondritis
dissecans

Photograph #13
Photograph #14
Photograph #15
9. Osteogenic sarcoma: Photograph 16 & 17
Discussion: Photograph #16 is a proton density image (combination of a T1 and
T2) demonstrating increased signal intensity with loss of normal fatty marrow
signal. Also there is an increase in dark signal intensity of the posterior aspect
of the tibia. Photograph #17 is a T1 weighted image through the tibia (sagittal
view) showing increased signal intensity centrally. This lesion was proven by
biopsy to be an osteogenic sarcoma
Photograph #17
10. Brodie's abcess/yeast infection: Photograph 18 & 19
Discussion: Photograph #18 is a T1 weighted image (sagittal view)
demonstrating loss of fatty marrow signal in the tibia with a loculated-type
lesion. Photograph #19 is a STIR image (a fat supression technique)
demonstrating increased signal within the lobulated lesion, and increased
signal intensity within the marrow of the distal tibia. This was consistent of a
Brodie's abcess due to a yeast infection in a previously undiagnosed HIVpositive
patient

Photograph #18
Photograph #19
11. Diabetic osteolysis: Photograph 20 & 21
Discussion: Photograph #20 is a T1 weighted image (sagittal view)
demonstrating loss of cortical margins of the first metatarsal head, with loss of
signal intensity of the fatty marrow and destruction of the 1st metatarsal head.
There is a plantar ulceration underneath the first metatarsal head. Photograph
#21 is a STIR image revealing no increased signal within the meatarsal shaft
which helps eliminate the diagnosis of osteomyelitis. This is consistent with
diabetic osteolysis.

Photograph #20
Photograph #21
References
Abramson DI, Circulatory Problems in Podiatry, Karger, New York, 1985 ACFS,
Complications in Foot Surgery, Williams & Wilkins, Baltimore, 1976 Advances
in Ilizarov Apparatus Assembly, A. Bianchi Maiocchi (ED), Medicalplastic srl,
Milan, Italy, 1994
Arrancia LJ, Rinaldi FT, Atlas of Rearfoot Surgery, Futura, New York, 1978
Balderston RA, Operative Techniques in Orthopedics: Foot and Ankle
Procedures, Vol 2: Number 3, Saunders, Philadelphia, 1992
Bartlett JG, Pocketbook of Infectious Disease Therapy '91-'92, Williams &
Wilkins, Baltimore, 1991
Berg D, The Resident's Quick Reference to Internal Medicine, Lippincot,
Philadelphia, 1990
Clinics in Podiatry, Podopediatrics, Saunders, Philadelphia, December 1984
Clinics in Podiatric Medicine and Surgery, Plastic Surgery, Saunders,
Philadelphia, April 1986
Clinics in Podiatric Medicine and Surgery, Pediatric Surgery of the Foot and
Ankle, Saunders, Philadelphia, January 1987 Clinics in Podiatric Medicine and
Surgery, Rheumatology, Saunders, Philadelphia, January 1988
Clinics in Podiatric Medicine and Surgery, Applied Biomechanics, Saunders,
Philadelphia, July 1988
Clinics in Podiatric Medicine and Surgery, Nail Disorders, Saunders,
Philadelphia, April 1989
Clinics in Podiatric Medicine and Surgery, Neurologic Disorders of the Lower
Extremity I, Saunders, Philadelphia, October 1989
Clinics in Podiatric Medicine and Surgery, Neurologic Disorders Affecting the
Lower Extremity II, Saunders, Philadelphia, January 1990 Clinics in Podiatric
Medicine and Surgery, Heel Pain, Saunders, Philadelphia, April 1990
Clinics in Podiatric Medicine and Surgery, Infections in the Lower Extremity,
Saunders, Philadelphia, July 1990
Clinics in Podiatric Medicine and Surgery, Wound Healing, Saunders,
Philadelphia, October 1991
Clinics in Podiatric Medicine and Surgery, Peripheral Vascular Disease in the
Lower Extremity, Saunders, Philadelphia, January 1992 Clinics in Podiatric
Medicine and Surgery, Pharmacology, Saunders, Philadelphia, April 1992
Cole DR, DeLauro TM, Neoplasms of the Foot and Leg, Williams and Wilkins,
Baltimore, 1990
DiNapoli DR, Reconstructive Surgery of the Foot and Leg, Update '90, Podiatric
Institute, Tucker GA
Draves DJ, Anatomy of the Lower Extremity, Williams & WIlkins, Baltimore,
1986
Fenstermacher K, Dysrhythmia Recognition and Management, Saunders,
Philadelphia, 1989
Forrester DM, et. al., Imaging of the Foot and Ankle, Aspen, 1988
Jahass MH, Disorders of the Foot and Ankle, Saunders, Philadelphia, 1991
Jay RM, Current Therapy in Podiatric Surgery, BC Decker, Toronto, 1989
Joseph WS, Handbook of Lower Extremity Infections, Churchill Livingstone,
N.Y., 1990
Keats TE, Lusted LB, Atlas of Roentgenographic Measurement, Year Book
Medical Publishers, Chicago, 1985
Kominsky SJ, Yearbook of Podiatric Medicine and Surgery 1992, Mosby, St.
Louis, 1992
Marchino DE, Advanced Protocol for the Diagnosis and Treatment of Pedal
Infections (Revised Edition), PICA
Marcinko DE, Comprehensive Textbook of Hallux Abducto Valgus
Reconstruction, Mosby, St. Louis, 1992
Marcinko DE, Medical and Surgical Therapeutics of the Foot and Ankle, Williams
and Wilkins, Baltimore, 1992
McGlamry ED, Comprehensive Textbook of Foot Surgery (1st and 2nd edition),
Williams ff Wilkins, Baltimore, 1987
McGlamry ED, Reconstructive Surgery of the Foot and Leg, Update '89, Podiatric
Institute, Tucker GA
Muller ME, Allgower M, Schneider R, Willenegger: Manual of Internal Fixation,
3rd ed, Springer Verlag, 1991, N.Y.
Orthopedics Clinics, Bone Tumors: Evaluation and Treatment, Saunders,
Philadelphia, 1989
Politz MJ, Clinical Laboratory Diagnosis, PMS vol 9, Futura, N.Y. 1977
Smith JW, Aston SJ, Grabb and Smith's Plastic Surgery, Little Brown, Boston,
1991
Root ML et. al., Biomechanical Examination of the Foot Volume 1, Clinical
Biomechanics Corporation, Los Angeles, 1971 Samitz MH, Dana AS, Cutaneous
Lesions of the Lower Extremities, Lippincott, Philadelphia, 1971
Sammarco, GJ, Foot and Ankle Manual, Lea & Febiger, Philadelphia, 1991
Sarrafian SH, Anatomy of the Foot and Ankle, Lippincott, Philadelphia, 1983
Seibel MO, Foot Function: A Programmed Text, Williams & Wilkins, Baltimore,
1988
Snow JC, Manual of Anesthesia, Little Brown, Boston, 1988
Vanore J, Osteosynthesis Techniques in the Foot and Ankle, P.P.M.A., 1989
Vender JS, Post Anesthesia Care, Saunders, Philadelphia, 1992
Weissman SD, Radiology of the Foot, Williams & Wilkins, Baltimore, 1989
Zang K: Traumatic Ankle Conditions, Futura, Mount Kisco, N.Y., 1976
Zier BG, Essentials of Internal Medicine in Clinical Podiatry, Saunders,
Philadelphia, 1990
Hershey Board Certification Review
Outline Study Guide

by
Jonathan M. Singer, D.P.M.
Diplomate American Board of Podiatric Surgery
Fellow American College of Foot and Ankle Surgeons
Diplomate American Board of Podiatric Orthopedics & Primary Podiatric
Medicine
Fellow American College of Foot & Ankle Orthopedics & Medicine

Edited by
James Lawton, D.P.M., Gene K. Potter, D.P.M., Ph.D.,
and Samuel L. Deitch, D.P.M.

Contributors
Peter F. Kelly, D.P.M., Gene K. Potter, D.P.M., Ph.D.,
Steven J. Berlin, D.P.M., Steven Kravitz, D.P.M., Guido
LaPorta, D.P.M., M.S., James Lawton, D.P.M., Arthur E.
Helfand, D.P.M., Fredrick E. Youngblood, M.D., Douglas
Glod, D.P.M., Christopher E. Attinger, M.D., Libby Cone,
M.D., Edwin J. Juda, D.P.M., William H. Simon, D.P.M.,
John Walter, D.P.M.
© 1998 Pennsylvania Podiatric Medical Association
Acknowledgements
William Goldfarb D.P.M. and Nancy Sullivan, for supporting
me and believing this book could be done

James Lawton D.P.M., and Gene Potter, D.P.M. for their


invaluable contributions in the general editing these
review notes

Jordon Fersel, M.D., for editing the chapters on


Anesthesia, and Emergency Medicine,

Deborah Wozniak, M.D., for editing the chapters on


Perioperative Management, Fluid Management, and
Postoperative Complications

Warren Joseph, D.P.M., and Steven Maffei, D.P.M. for


editing the chapter on Infectious Diseases

Steven Kravitz, D.P.M., for editing the chapter on


Biomechanics

George Vito, D.P.M. for editing the chapter on Internal


Fixation

Peter F. Kelly, D.P.M. for writing the chapter on Lasers


and Laser Physics

Gene K. Potter, D.P.M., Ph.D. and Steven Berlin, D.P.M.


for editing the chapters on Dermatology, Bone Tumors,
and Soft Tissue Tumors

Gene K. Potter, D.P.M., Ph.D., for proof reading, editing,


and adding his phenomenal talent to this book

Guido LaPorta, D.P.M., M.S. for writing the chapter on


Ankle Arthroscopy
The Hershey Board Certification
Review Outline Study Guide
is prepared for distribution by the
Hershey Surgical Seminars Committee
Pennsylvania Podiatric Medical Association
Guido La Porta, D.P.M., General Chairman

Author: Jonathan M. Singer, D.P.M.


General Editor: James Lawton, D.P.M.
Copy/Content Editor: Gene K. Potter, D.P.M., Ph.D.
Cover Design: Marissa Singer
Coordinator: Nancy Sullivan
Published by the Pennsylvania Podiatric Medical Association
ISBN # 0-9639030-04

Copyright © 1998
Pennsylvania Podiatric Medical Association 757 Poplar Church Road
Camp Hill, Pennsylvania 17011 (800)-841-3668
All rights reserved

This book is protected by copyright. No part of this book may be reproduced in any
form or by any means, including photocopying, or utilized by any information
storage or retrieval system without written permission from the copyright owner.

Accurate indications, adverse reactions, and dose schedules for drugs are provided in
this book, but It is possible that they may change. The reader is urged to review the
package information data of the manufacturers of the medications mentioned.

The author has made every effort to ensure the accuracy of the information herein.
However, appropriate information sources should be consulted, especially for new or
unfamiliar procedures. It is the responsibility of every practitioner to evaluate the
appropriateness of a particular opinion in the context of actual clinical situations
and with consideration to new developments. The author, editors, and publisher
cannot be held responsible for any typographical or other errors found In this book.

Printed in the United States of America.


Table of Contents
Chapter 1: Emergency Medicine Water and Electrolyte Imbalance: page 65-
Cardiopulmonary Resuscitation: page 2-3 66
Advanced Life Support: page 3-4 Oliguria: page 66
Office Emergencies: page 5-7 Chest Pain: page 67
Medical Emergencies (Cardiac Postoperative Hypertension: page 67
Dysrhythmias): page 7-9 Postoperative Infection: page 67-68
Summary of CPR: page 9 Anxiety and Pain Management: page 68-69
Other Medical Emergencies: page 9-12 Nausea: page 69
Shock: page 12-13 Constipation: page 69
Blood and Blood Products for Emergency Shivering: page 69
Use: page 13-17 DVT: page 69-70
Compartment Syndrome: page 70-73
Chapter 2: Anesthesia Hemorrhage: page 72-73
Anesthesia Classifications: page 20 Septic Syndrome: page 73
General Anesthesia: page 20-23 Thyroid Storm: page 73-74
Intravenous Anesthesia: page 23
Local Anesthesia: page 23-26 Chapter 6: infectious Diseases
Special Cases for Lowering the Maximum Specific Diseases: page 76-81
Allowable Dose: page 26 Principles of Antiinfective Therapy: page 81-
Pediatric Anesthesia: page 26-27 84
Lumbar Epidural and Caudal Antibiotics: page 84-90
Anesthesia: page 27-28 Drug Fever: page 90-91
Complications of Endotracheal Intubation: Specific Antimicrobial Therapy: page 91-92
page 31 Surgical Considerations: page 92-93
Nerve Injury During Anesthesia: page 31-32 Necrotizing Infections: page 93-94
Other Medical Compliccations From Osteomyelitis: page 94-102
Anesthesia: page 32-34 Septic Arthritis: page 102
Mycology: page 102-103
Chapter 3: Perioperative Management Viral Diseases: page 104
The Healthy Patient: page 36 Rickettsial Diseases: page 104
The Diabetic Patient: page 36-38 Protozoan and Metazoan Infections: page
The Hypertensive Patient: page 38-39 104-105
The Patient on Steroid Therapy: page 39-40
The Asthmatic Patient: page 40-41 Chapter 7: Wound Healing
The Alcoholic Patient: page 41 Repair of Skin: page 108-111
The Patient on Anticoagulant Therapy: page Factors That Interfere With Wound Healing:
41-42 page 111-113
The Patient With Clotting Abnormalities: Growth Factors in Wound Repair: page 113
page 42 Surgical Approaches: page 113-114
The Rheumatoid Patient: page 43 Wound Dressings: page 114
The Sickle Cell Patient: page 43-44 Suture Materials and Needles: page 115-
The Cardiac Patient: page 44-45 116
The Pulmonary Disease Patient: page 45 Tourniquets: page 116-118
Perioperative Management of the Infant and
Child: page 45-47 Chapter 8: Plastic Surgery
Mitral Valve Prolapse patient: page 47-48 Skin Grafting: page 120-122
The Gouty Arthritis Patient: page 48 Local Cutaneous Flaps: page 122-124
Local Muscle and Cutaneous Flaps: Page
Chapter 4: Preoperative Evaluation 124
Screening Procedures: page 50-61 Fasciocutaneous Flaps: page 124-125
Summary of Perioperative Lab testing: page Microsurgery and Free Flaps: page 125-126
61-62 Diagrams of Skin Flaps: page 126-130

Chapter 5: Postoperative Care & Chapter 9: Bone Healing


Complications Primary Bone Healing: page 132
Fever: page 64-65 Complications of Bone Healing: page 133-
Altered Mental States: page 65 135
Treatment of Nonunions: page 135 218
Fusion: page 135 Principles Affecting Internal Fixation: page
Bone Grafting: page 135-137 218
Aseptic Necrosis Following 1st Metatarsal Internal Fixation Devices: page 219-229
Osteotomy: page 138-139 AO Objectives: page 229
Electrical Bone Stimulation: page 40 AO Principles: page 229-230
Cartilage Healing: page 140 AO Technique: page 230-235
Jumping Screws: page 235-236
Chapter 10: Burns and Frostbite Other Techniques: page 236-238
Assessment and Treatment of Burns: page Complications of Fixation Devices: page 238
142-145 External Fixation: page 239
Burn Deformities: page 145-146 Large Bone Fixation: page 239-252
Other Types of Burns: page 146 Small Bone External Fixation: page 253-
Frostbite: page 146-147 254

Chapter 11: Fluid Management and Chapter 17: Nail Disorders & Surgery
Replacement in the Surgical Patient Nail Entities: page 256-260
Fluid Management: page 150-152 Nail Anatomy: page 260
Complications of Total Parenteral Nutrition: Surgical Nail Procedures: page 262-263
page 152
Replacement Therapy: page 152-153 Chapter 18: Neurology
Blood Products and Indications for Use: Neurological Pathways: page 266
page 153-156 Presenting Features: page 266-270
Composition of IV Fluids: page156-157 Assessment of Clinical Problems: page 270-
278
Chapter 12: Bone Tumors Innervation of the Lower Extremity: page
Introduction: page 160 278-281
Staging Techniques: page 160-161 Tarsal Tunnel Syndrome: page 282-284
Surgical Staging System: page 161 Classification of Nerve Injuries: page 284-
Radiology of Bone Tumors: page 161-162 285
Bone Tumors: Page 163-168 Neuromuscular Causes of Cavus Foot: page
Cystic Lesions of Bone: page 168-169 285
Other Tumors and Cancers: page 169-170 Types of Nerve Surgery: page 285-288
Bone Tumors: A Quick Reference Chart:
page 170-171 Chapter 19: Pediatrics
Phases in Cancer Development: page 171 Evaluation of the Pediatric Patient: page
Benign vs. Malignant: page 172 290-293
Pedicatric Biomechanics-Normal Values:
Chapter 13: Dermatology Newborn to Adult: page 293
The Skin: page 174 Congential Deformities: page 293-302
Dermatological Lesions: page 175-182 Pediatric Radiology: page 302-304
Pediatric Gait Patterns: page 304-305
Chapter 14: The Arthropathies Intoe Gait: page 306-307
Causes of Joint Pain: A Summary: page 184 Pediatric Fractures: page 307-318
The Arthropathies: page 184-195 The Limp in Childhood: page 318-319
Lab Testing for the Arthropathies: page 195 Juvenile Hallux Valgus: page 319-320
Articular Disorders Affecting the Heel: page Biomechanical Examination of the Child:
195-196 page 320-323
Synovial Fluid Analysis: page 197 Corrective Casting Techniques in Infants:
Charcot Foot: page 198-199 page 323-324
The Toe-Walking Child: page 324-326
Chapter 15: Peripheral Vascular Disease
Patient Evaluation: page 202-212 Chapter 20: Drugs and Drug Interactions
The Vascular Diseases: page 212-215 Introduction to Drug Interactions: page 328
Antibiotic Drug Interactions: page 328-329
Chapter 16: Internal & External Fixation Anticoagulant Drug Interactions: page 330
Objectives of Fixation Devices: page 218 Antihypertensive Drug Interactions: page
Requirements of Implant Materials: page 330-331
Antipsychotic Drug Interactions: page 331 Physical Examination: page 418-419
Cardiac Drug Interactions: page 331-332 Preoperative Considerations: page
Diuretic Drug Interactions: page 332 419
Non Steroidal Anti-Inflammatory Drug Arthroplasty Procedures: page 419
Interactions: page 332-333 420
Oral Contraceptive Drug Interactions: page Capsule-Tendon Balancing
333 Procedures: page 421-422
Mechanisms of Drug Interactions: page Implant Arthroplasty Procedures:
333-335 page 422-426
Specific Drugs: page 335-344 Arthrodesing Procedures: page 426
429
Chapter 21: Surgery of the Congenital Foot Proximal Phalangeal Osteotomies:
Flatfoot Surgery (flexible): page 346-355 page 429-431
Subtalar Joint Blocking Procedures: Distal Metatarsal Osteotomies: page
(Arhroereises and Arthrodesis): page 355- 432-436
356 Proximal Metatarsal Osteotomies:
Flatfoot Surgery (rigid): Convex Pes Plano page 436-438
Valgus: page 356-358 Shaft Osteotomies: page 438-439
Metatarsus Adductus Surgery: page 358- Combination Procedures: page 439
362 Other Procedures: page 439-440
Cavus Foot Surgery: page 362-367 Hallux Rigidus and Limitus: page
Clubfoot Surgery: page 368-373 440-445
Hallux Varus: page 445-446
Chapter 22: Generalized Disease Conditions Hallux Hammertoe: page 446
of Bone Diff. Diagnosis of Pain In the
Soft Tissue Overgrowth: page 376 Sesamoid Area: page 446
Abnormalities of Alignment: page 376-377 Complications of HAV Surgery: page
General Increased Bone Density: page 377 447
Osteopenia: page 377-378 HAV Procedure Chart and Their
Marrow Abnormalities: page 378- Indications: page 448
379
The Dysplasias: page 379-380 Chapter 25: Trauma
Medical Management and General
Chapter 23: Radiology Assessment of the Trauma Patient:
Standard Radiographic Techniques page 450-451
of the Foot and Ankle: page 382- Assessment of Lower Extremity
383 Injury: page 451
Specific Radiographic Studies: page General Evaluation and Treatment
383-391 of Fractures: page 451-452
Anatomic Angles: page 391-396 Calcaneal Fractures: page 452-460
Common Structural Measurements Talar Fractures: page 460-463
(Diagrams): page 396-402 Osteochondral Fractures: page 463
Pediatric Radiology: page 402-406 465
The Osteochondritities: page 406-407 Navicular Fractures: page 466-470
The Accessory Bones of the Foot: Cuboid Fractures: page 470-472
page 407 Cuneiform Fractures: page 472
Fifth Metatarsal Base Fractures: page
Chapter 24: Hallux Valgus & 472-474
Related Disorders Metatarsal Fractures: page 474-480
Goals of HAV Surgery: page 410 Ankle Fractures: page 480-493
Anatomical Facts of the 1st MPJ: Midtarsal Joint Dislocations: page 494
page 410-411 Tarsometatarsal Joint
Predisposing Factors: page 412 Dislocations/Fractures: page 494-498
Biomechanics: page 412-414 Ankle Inversion Sprain: page 498
Etiology of HAV: page 414-415 505
Deformity Types: page 416-417 Deltoid Ligament Ruptures: page
Radiological Measurements for HAV: 506-507
page 417-418 Compartment Syndrome: page 507
508 571-572
Open Fracture Classification System Tenosynovitis: page 572
and Tx: page 508-510
Soft Tissue Injuries: page 510-511 Chapter 28: Lesser Metatarsal
Crush, Gunshot, and Lawnmower Surgery
Injuries: page 511-512 Anatomy (Metatarsals 2-3-4): page
Puncture Wounds: page 512 574-575
Epiphyseal Plate Injuries: page 512-514 Differential Diagnosis of
Digital Fractures and Dislocations: Metatarsalgia: page 575
page 514-515 Surgical Treatment of the IPK: page
1st MPJ Trauma: page 516-517 575-578
Nail Bed Trauma: page 517-518 Lesser Metatarsal Joint Replacement:
Toe Tip Injuries With Tissue Loss: page 578-579
page 518-519 Panmetatarsal Head Resection: page
Dog and Cat Bites: page 519 579-581
Metatarsus Adductus: page 581-585
Chapter 26: Digital Deformities Freiberg's Disease: page 585-587
and Surgery Tailor's Bunion: page 587-589
Hammertoe Syndrome: page 522- Splayfoot: page 589
528 Brachymetatarsia
Mallet Toe Syndrome: page 528-529 (Brachymetetopody): page 589-591
Claw Toe Deformity: page 529 Skewfoot: page 591-592
Overlapping 5th Toe: page 529-532
Hallux Hammertoe: page 532-533 Chapter 29: Ankle Conditions
Lesser Digital Arthrodesis: page 533 Differential Diagnosis of Chronic
Hallux Interphalangeal Arthrodesis: Ankle Pain: page 594
page 533 Tarsal Tunnel Syndrome: page 594-
Lesser Digital Arthrodesis: page 533- 597
534 Sinus Tarsi Syndrome: page 597-
Overlapping 2nd Toe: page 534 599
Syndactylization: page 534-535 Peroneal Subluxation: page 599-
Digital Implants: page 535-537 600
Floating Toe Syndrome: page 537 Ankle Arthrodesis: page 600-602
Blue Toe Syndrome: page 538-539 Lateral Ankle Instability: page 603
Polydactylism: page 539-541 Chronic Lateral Ankle Instability:
page 604-607
Chapter 27: Muscle and Tendon Chronic Medial Ankle Instability:
Pathology page 607
Muscle Physiology: page 544-548 Ankle Equinus: page 607-611
Principles of Tendon Repair: page Malunion and Nonunion of the
548-552 Malleolei: page 611-614
Tendon Lengthening and
Tenotomy: page 552-554 Chapter 30: Heel Conditions
Tendon Transfers: page 554-556 Anatomy of the Heel: page 616-617
Tendon Grafts: page 556-557 Radiological Evaluation of the Calcaneus:
Posterior Tibial Tendon Rupture: page 617-618
page 557-559 The Heel in Systemic Disease:
Posterior Tibial Tendon Dysfunction page 618-621
(Acquired Adult Flatfoot Syndrome): Seronegative Arthritis and Heel Pain:
page 559-560 page 621-622
Peroneal Tendon Pathology: page Heel Spurs and Heel Spur
560-562 Syndrome: page 622-623
Achilles Tendon Rupture: page 562- Tumors of the Heel: page 623-626
566 Tarsal Coalitions: page 626-629
Lateral Ankle Stabilization Sever's Disease: page 629-630
Procedures: page 566-571 Causes of Heel Pain (a summary):
Postoperative Care and Training page 630-631
Following Tendon Transfer: page Haglund's Deformity: page 631-632
Chapter 37: Laser Applications in
Chapter 31: Amputations Podiatric Surgery
General Surgical Technique: page Laser and Laser Physics:
634 page 741-751
Digital Amputations: page 634-635 Clinical Applications in Podiatric
MPJ Amputations: page 635 Surgery: page 751-775
Transmetatarsal Amputations: page
635-636 Chapter 38: Specific Radiological
Tarsometatarsal Amputations Pathology
(Lisfranc's): page 637 MRI Pathology: page 777-800
Midfoot Amputations (Chopart's):
page 638
Amputation of the Ankle (Syme's,
Boyd, Pirogoff): page 638-639
Below Knee Amputation: page 639-
640

Chapter 32: Biomechanics


Normal Values: page 641
Criteria of Normalcy in the Lower
Extremity: page 642-643
Adult Biomechanical Examination:
page 643-645
Common Structural Variations: page
646-648
Planes of Motion: page 648-649
Axes of Joint Motion: page 649-653
Angular and Axial Deformities in
Children: page 654-656
Anatomy of Gait: Activity of Muscles:
page 656-663
Observation of Gait: page 663-664
Subtalar Joint Measurements: page
664-666
Subtalar Joint Function: page 666-
667
Midtarsal Joint Function: page 667-
668

Chapter 33: Anatomy


Neuroanatomy: page 670-676
Angiology: page 676-680
Myology: page 680-695
Arthrology: page 695-704

Chapter 34: Soft Tissue Tumors


Soft Tissue Tumors: page 706-715

Chapter 35: Physical Medicine


and Rehabilitation
Physical Modalities and Diseases:
page 718-720
Therapeutic Modalities and
Procedures: page 720-730

Chapter 36: Arthroscopy


Ankle Arthroscopy: page 731-740
Case Study #1
CC: 65 yr old female presents with an open sore on the bottom of her right foot.

HPI:
N: no pain
L: corresponding to the area under the 1st metatarsal head
D: x 3 weeks but probably longer
O: gradual onset; started out as a blister
A: none, except that it is starting to smell. (This is why her husband made her come in)
T: clean it with hydrogen peroxide and apply Vasaline to it
S: fever and nausea x 2 days.
Pt arrived with a bandaid covering the area. Yellow drainage observed.

PMH:
1. DM2 x 18 years. Last seen by MD 2 months ago. FBS at that time was 138 mg/dl.
She was advised to check her blood sugar each morning using an accucheck
machine, but does not do this regularly
2. HTN x 12 years. MD tells her that it is under control
3. Hyperlipidemia x 10 years

PSH:
1. Fx ulna as a child
2. Hysterectomy, 20 years ago

Allergies:
1. PCN
She was told this as a child. She doesn’t know what happens if she takes this.

Medications:
1. 40 units of NPH insulin q AM
2. HCTZ
3. Captopril
4. K-lor
5. Lipitor

SH:
1. + Tobacco use – 1 ppd x 30 years
2. Denies EtOH and illicit drug use
3. Occupation: housewife

FH:
2 children who are alive and well
Father deceased from diabetic complications
Mother alive and well with HTN
ROS:
General: increased fatigue over the past 3 – 4 days
Eyes: wears glasses
Lungs: SOB upon exertion
PV: leg cramps after walking 2 blocks
Neuro: tingling and burning of the feet

Physical Exam

T = 101.5
P = 100
RR = 20
BP = 100/65
Accucheck = 280
Ht: 61 inches
Wt: 185 lbs (84.1 kg)

Derm/Integ:
The right foot is substantially warmer than the left. The entire right foot is erythematous
to the level of the ankle joint with increased intensity around the first and second rays.
Deep ulceration 1.5 cm in diameter located under the 1st metatarsal head, right foot. The
base is a mixture of granular and fibrotic tissue. Some dry yellow pus is apparent upon
inspection of the wound. When the area proximal to the wound is squeezed,
approximately 3cc of creamy yellow pus is expressed through the ulcer. With the use of a
cotton tip applicator, the wound probes to the 1st MT head as well as laterally into the 1st
interspace. Malodor is present but no specific smell can be distinguished. The borders are
mildly hyperkeratotic.

Vasc:
DP and PT pulses are non palpable bilateral. Popliteal pulses are palpable bilateral. The
entire right foot is edematous corresponding to the erythema.

Neuro:
Diminished protective sensation to the level of the ankle joint using SWMF. Achilles and
patellar DTRs are 2/4 and symmetric. Muscle mass is WNL and muscle strength is 5/5.

M/S:
Plantarflexed 1st ray right greater than left. Dorsally contracted digits 2 – 5 bilateral.
Slightly decreased medial arch bilateral.
Relevant Questions – Case Study #1
1. What are some tests that you may want to order & some consults to consider?
- Order the standard 3 x-rays (AP, lateral and lateral oblique). You may also want to
consider an axial sesamoidal view do to the location and way that this ulcer probes. There
may be an infection in the sesamoids and this is a great way to see them unobsecured.
This view may not be necessary, however.

- Order labs: CBC with a differential, ESR, CRP, HbA1C, Chem 7, Albumin.
CBC with diff: looking for a left shift. This means that there are more immature
neutrophils (Bands) hanging out.
ESR: This is a non-specific marker of inflammation. An ESR >70 is highly correlated
with the presence of osteomyelitis. This is a measurement that can be followed to
determine the effectiveness of treatment but BEWARE! ESR can lag behind clinical
presentation.
CRP: Used mainly as a marker of inflammation. Measuring and charting C-reactive
protein values can prove useful in determining disease progress or the effectiveness of
treatments.
HbA1C: This is a marker of the long term glucose control in diabetic patients.
Chem 7: There are several values that can be of interest in this panel. Potassium is
important in heart function. Make sure that it isn’t going crazy. This is important if you
are going to consider surgical intervention. BUN and creatinine are markers of kidney
function. The nephrons of the kidney can be affected by the disease course of diabetes.
These values can also be used to calculate any renal dosing of antibiotics. Anion gap can
be calculated from several values in this panel
Albumin: This is a marker of nutrition. Some clinicians use this as a measurement of
whether or not a patient has the ability (base on a nutritional standpoint) to heal.

- Gram stain, culture & sensitivity: If you are unsure of the infecting organism, this can
be helpful. In this particular case, make sure that a deep culture is taken. It may be
advantageous to consider a quantative culture as well.

- Non-invasive Vascular studies: Doppler, ABI, TcPO2.


Doppler: Audible exam of lower extremity arteries
ABI: Ankle Brachial index.
TcPO2: Transcutaneous oxygen pressure.
All three of these tests, especially when their results are taken together, can provide
insight into the vascular status of the patient and help determine at what level the patient
can heal.

- Pre-Op Testing
As with all things this can be hospital or surgery center dependent. Some general tests
may include the following:
CXR: This patient is a smoker. Chest x-rays are always good in smokers.
EKG: Takes a peak at the electrical activity of the heart and can help determine if any
previous infarcts or other pathology exist. Acute MI are large killers of DM pts
UA: Does she have a UTI?
Labs – Case Study #1
X-rays:
No sign of osteomyelitis
No soft tissue emphysema
Edematous soft tissues around the entire right foot

WBC 15.38 (Lymph = 7.5%) (TLC = 1154)


HGB 11.13 HGB (11 – 16)
HCT 39.0 WBC ( 5 – 10) >------------------<PLT (150 – 400)
PLT 220 HCT (40 – 50)

Na 135
K 3.2 Na+ (135 – 145)| Cl – (98 – 109) | BUN (10 – 20)
Cl 101 --------------------|--------------------|------------------< Glucose (70 – 120)
CO2 26 K+ (3.5 – 5.5) | CO2 (20 – 30) | Cr (0.5 – 1.5)
BUN 22
CR 2.9
Glucose 291

Albumin 2.6

HbA1C 11.5

ESR: 92

UA: + protein

Microbiology
Gram stain: Many gram + cocci
Few gram – rods
Cultures: S. aureus
E. coli
B. fragilis

Vascular Studies:
Handheld Doppler reveals monophasic waveforms

ABI: L = 0.47
R = 0.31
(Should be greater than 0.45)

TcPO2 L = 36.5
R = 20.7
(Should be greater than 30)
Relevant Questions – Case Study #1
1. If you were to write admit orders for this patient, what headings would you cover?
Admit to service
Diagnosis
Condition
Vitals Acronym: ADC VAN DILMAX
Activity
Nursing
Diet
Ins & Outs
Labs
Medications
Ancillary
X-rays

2. What are your radiographic findings for osteomyelitis?


- soft tissue changes (swelling that obliterates the fat planes, possible soft tissue
emphysema)
- periosteal reaction
- cortical destruction
- sequestrum
- involucrum

3. What is the lag time associated with x-rays findings of osteomyelitis?


10 – 14 days

4. What is a left shift?


Neutrophils + bands > 80
Increase in the number of immature neutrophils present

5. What color denotes gram + organisms? Gram – organisms?


(+) are pink
(-) are purple

6. Name some gram + cocci


Staphylococcus Enterococcus
Streptococcus Micrococcus

7. Name some gram - rods S2P3HEK (S/P HEK)


Escherichia Pseudomonas
Hemophilus Klebsiella
Serratia Salmonella
Pasteurella Proteus
8. Name 5 anaerobes.
Clostridium Bacteroides
Peptostrepococcus Peptococcus
Fusobacterium

9. Name 7 gas forming organisms BECKS+


Bacteroides
Escherichia
Clostridium
Klebsiella
Staph/Strep
Serratia
Fusobacterium
Peptostrepococcus
Peptococcus

10. Name some anti-pseudomonal drugs. FATZ – CIA


Fortaz
Aztreonam
Ticracillin (Timetin)
Pipercillin (Zosyn)
Ciprofloxacin
Imipenem
Aminoglycosides

11. How do you calculate creatinine clearance?


(140 – age) x weight (kg) (x 0.85 in women)
72 x serum Cr

12. What are some commonly used antibiotics that do not require renal dosing?
Clindamycin, Erythromycin, Chloramphenicol, Cefoperazone

13. Name 4 common PCN combination drugs used in diabetic foot infections and their
dosages
Unacyn (Ampicillin/Sulbactam) 1.5, 3.0 g IV q6h
Zosyn (Piperacillin/Tazobactam) 3.375 g q 4 – 6h; 4.5 g q4-6h for pseudomonas
Timentin (Ticracillin/Clavualanate) 3.1 g q4 – 6h
Augmentin (Amoxicillin/Clavualanate)500 – 879 PO BID or 250 – 500 PO TID

14. Estimate this patient’s blood glucose over the past 120 days.

15. At what HbA1C is a patient diagnosed with diabetes?


8.5%

16. What are some parameters used to evaluate a patient’s probability for healing?
vascular (ABI > 0.45); nutrition (albumin > 3.0); immune status (TLC >1500)
17. What are the minimum levels of albumin and TLC required for healing?
Albumin of 3.0 g/dl
TLC of 1500
From: Dickhaut et al, JBJS 1984

18. What antibiotics are safe to use during pregnancy?


Penicillins
Cephalosporins
Erythromycin

19. What antibiotics are not safe to use during pregnancy?


Ticracillin
Metronidazole
Tetracycline

20. Which antibiotics are associated with ototoxicity?


Aminoglycosides – irreversible
Erythromycin – reversible with d/c of the drug
Vancomycin – may or may not cause ototoxicity

21. Which class of antibiotics can cause a neuromuscular blockade and what do you give
to reverse this?
Aminoglycosides
Give Calcium to reverse the blockade

22. Which antibiotics are most commonly associated with seizures?


Penicillin
Ampicillin
Imipenem

23. Which antibiotics are associated with nephrotoxicity?


Aminoglycosides
Vancomycin
Penicillins – interstitial nephritis
Cephalosporins – intersititial nephritis

24. Which antibiotics are associated with hepatotoxicity?


Tetracycline (may cause death in pregnant women)
Erythromycin – reversible

25. What are possible antibiotics to treat C. diff associated diarrhea?


Metronidazole: 500 mg PO q8h x 10 days
Vancomycin: 125 mg PO q6h x 10 days

26. Which anaphylactoid response is unique to Vancomycin?


Red man syndrome
27. Who studied the relationship between ABI and healing and what did he/them find?
Wagner
ABI > 0.45

28. What is the minimum TcPO2 needed for healing? Who said this?
30 mmHg
Wyss, Harrington & Burgess, JBJS, 1988

29. What physical exam finding may lead to a decreased TcPO2?


Edema due to cellulitis

30. Name several etiologies of peripheral neuropathies


DANG THERAPIST
Diabetics/drugs
Alcoholic
Nutritional
Guillain-Barre Syndrome
Toxic
Hereditary
Recurrent
Amyloidosis
Porphyria
Infectious (syphilis, hansen’s dz)
Systemic
Tumor
Meds

31. Three Diabetic Foot Classifications.


Wagner (Foot & Ankle, 1981)
0: potential, preulcerations, no open lesions
1: superficial ulceration
2: deep ulceration penetrating to tendon, joint capsule of ligament
3: penetration to bone with presence of infection
4: gangrene of forefoot
5: gangrene of rearfoot

University of Texas, JFAS 1996


This classification is based on wound depth and the presence or absence of ischemia
and/or infection. The benefit of this classification system is that it has predictive value
and can indicate treatment protocol.

0: pre- or post-ulcerative lesion that is completely epithelialized


1: superficial wound not involving tendon, capsule or bone
2: wound penetrating to tendon or capsule
3: wound penetrating to bone or joint
A: non-ischemic or clean wound
B: infected wound
C: ischemic wound
D: infected and ischemic wound

Meade & Mueller (Med Times, 1968)


1: dorsal foot phlegmon (non localizing, cellulitic, infectious process)
2: deep plantar space infection
3: mal perforans neuropathic foot ulcers (subclassed above)

32. Name 4 Different Insulin Preparations


Ultra fast/ultrashort acting:
-Lispro insulin
-Aspart
Short Acting
-Regular insulin (crystalline zinc) SQ/IV
-Semilente SQ
Intermediate Acting
-NPH (neutral protamine hagedorn) SQ
-Lente
Long Acting
-Ultralente SQ
-Protamine zinc
Ultra Long Acting
-Glargine

Onset Peak Duration


Rapid 10 – 30 minutes 1 – 2 hours 2 – 4 hours
Intermediate 1 – 4 hours 6 – 12 hours 18 – 24 hours
Long 4 – 8 hours 18 – 24 hours 24 – 36 hours
Ultra long acting 4 hours None
33. Name some hypoglycemic drugs.
Suflonureas
-MOA: increased insulin secretion from the beat cells of the pancreas
-1st generation: chlorpropamide
-2nd generation: glipizide (glucotrol); glyburide (micronase)
Biguanides
-inhibits glyconeogenesis in the liver; increases peripheral insulin action
-Metformin(glucophage)
Alpha-Glucosidse Inhibitors
-inhibit glucose uptake in the gut blunting the post prandial hyperglycemia
-Acarbose (precose)
Thiazolidinediones (TZDs)
-antihyperglycemics that increase insulin sensitivity
-Rosiglitazone
-Troglitazone
Other
-Repaglinide: insulin secretagogue whose binding site differs from that of the
suflonureas. Insulin release is rapid and brief. Very useful for post prandial
hyperglycemia

34. What type of anti-hypertensive may cause hyperglycemia?


Thiazide diuretics (HCTZ)
Elective Surgery and Pre-Operative Labs – The Chem 7
Sodium (Na): Normal 125 – 145 mEq/L
Hypernaturemia: dehydration, sodium overload, diabetes insipidus or volume
depletion (diarrhea, osmotic diuresis).
Zebra: Cushing’s syndrome
Hyopnaturemia: volume excess/edema, fluid retention (CHF), IV fluids,
vomiting/diarrhea, renal failure (nephritic syndrome), heart failure, cirrhosis.
Zebra: Addison’s disease
Potassium (K): Normal 3.5 – 5.5 mEq/L
Increases: dehydration, trauma (hemolysis), acidosis, renal disease, use of K
sparing diuretics. Membrane potential will become closer to threshold  can lead to
fibrillation and cardiac stand still
Decreases: Vomitting/diarrhea, diuretics, excessive sweating, cellular shifts.
Zebra: Cushing’s disease. Less excitable membranes can lead to bradycardia.
Chloride (Cl-): Normal 89 – 109 mEq/L
Strongly mirrors sodium. See above.
Bicarbonate: Normal 23 – 32 mEq/L
Why this is valuable: information as to overall perfusion and saturation of RBC.
Significant alteration in either positive or negative directions is not desirable and may
indicate the presence of an anemia or significant blood loss recently.
BUN (Blood Urea Nitrogen): Normal 10 – 20%
BUN is a produced by protein breakdown in liver and excretion by the kidney.
Elevated BUN (azotemia) indicates possible crush injury, infection, excessive
protein intake or an inability of kidneys to excrete BUN (renal dz, hypotension).
Decreases: liver is unable to breakdown protein or malnutrition.
Not as reliable as creatinine because BUN is dependent on age, gender, lean body mass
and etc.
Creatinine (Cr): Normal 0.7 – 1.4
Byproduct of creatine produced by the liver and kidneys and is used as an energy
storehouse in skeletal muscle. Creatinine is completely filtered by the kidneys and is not
reabsorbed or secreted, thus its an excellent measure of GFR (glomerular filtration rate).
Elevated serum creatinine is most specific for renal dz, creatinine clearance,
usually greater than 80 ml/min is the best way to assess renal function. It requires a 24 hr
urine specimen, but it can be estimated using the equation:
[(140 – age) x wt (kg) (0.85 for women)] / 72 x creatinine clearance
Glucose: Normal 80 – 120 mg/dl
Most offen used in diagnosing and managing diabetes. Diabetes can be diagnosed
as a fasting blood sugar >126 mg/dl and 2 hour post prandial blood sugar over 200 mg/dl.
Glucose can also be elevated by hyperthyroidism or from thiazide diuretics
Glucose can be decreased in pancreatic neoplasms and Addison’s disease
Hemoglobin A1C normal: 3 – 6%
Can be 3 – 6% in non diabetics. In uncontrolled diabetics it can be as high as 10%
(or higher).
Good estimate for judging a patient’s long term control
Elective Surgery & Pre-Operative Labs – The CBC
Can be done with or without a differential

Agranulocytes: Lymphocytes, Monocytes


Granulocytes: Neutrophils (PMN), Eosinophils, Baseophils

WBC Normal 5000 – 10000 / mm


Leukopenia (<4000/mm): drugs (most commonly); acute infection, vit B12
deficiency, SLE, RA.
Leukocytosis (>12000/mm): infection, pregnancy, splenectomy, drugs
(corticosteroids, lithium, epi, abx, sulfonamides)

Neutrophils: Normal 40 – 75%


Increases: bacterial infections
Bands: immature neutrophils
Segs: mature neutrophils
Bands + Segs > 80  left shift has occurred

Lymphocytes: Normal 15 – 45%


Diverse group of cells
Includes T cells and B cells
Increase: usually a viral infection

Eosinophils: Normal 1 – 6%
Increase: allergic reactions, parasitic diseases

Basophils: Normal 1 – 2%
Source of histamine; contain heparin
Increase in basophils is usually associated with an increase in Mast cells
(basophils in tissue)
Increase: myxedema, blood vessel injury

Monocytes: Normal 1 – 10%


Increase: protozoan infections, TB
Elective and Pre-Operative Labs – The H & H
Most valuable in cases of malnutrition or suspected anemia.

Hemoglobin (Hgb): 13 – 17 males


11 – 15 females
Hemocrit (Hct): 39 – 51
3 x Hgb
Mean corpuscular volume Hct/RBC
82 – 92 mm3

Microcytic MCV<80 Normocytic MCV 80 – 100 Macrocytic MCV > 100


Iron deficiency Chronic dz Folate def/Vit B12 def
Blood loss Bone marrow Dz Liver dz
Thalassemia Hemolysis Anticonvulsants

Microcytic anemias (Fe deficiency anemias) D/Dx pneumonic = TICS

T – Thalassemias
I – Iron Deficiency
C – Chronic Inflammation (anemia)
S – Sideroblastic anemia (check pt for Pb toxicity)

Normocytic anemias D/Dx pneumonic = NORMAL S(I)ZE

N - Normal pregnancy (volume expands up to 30% plasma vol)


O - Over hydration
R - Renal (chronic renal disease => lower EPO production)
M – Myelophistic: replacement of bone marrow w/ tumor
A – Acute blood loss
L – Liver disease

S – Systemic Inflammation
Z – Zero production
E – Endocrine disorders e.g. hypothyroid disease
Macrocytic anemias D/Dx pneumonic = BIG FAT RED CELLS

B – B12 deficiency
I – Inherited problem
G – G/I

F – Folic Acid deficiency


A – Alcoholism
T – Thymine responsive

R – Reticulocytes (larger than mature RBC), therefore inflate MCV


E – Endocrine
D – Dietary problems (lack of dietary folate, B12 etc.)

C – Chemotherapeutic Rx
E – Erythroleukemia
L – Liver Disease
L – Lesch- Nyan Syndrome (“zebra”)
S – Splenectomy

Hemolytic Anemias (Increased rate in destruction of RBC) D/Dx pneumonic =


HEMATOLOG(I)ST

H – Hemoglobinopathy (most common is sickle cell)


E – Enzyme deficiency
M – Medication (drug induced)
A – Ab
T – Trauma to RBC
O – Ovalocytosis (Autosomal Dom disorder in pop w/ south east asia)
L – Liver Disease
O – Osmotic Fragility
G – Glucose 6-P DH Deficiency
S – Splenic destruction
T – Transfusion related
Elective and Pre-Operative Labs – Coagulation Panel
Bleeding Time: Normal 2 – 9 minutes
Normal time is via a controlled forearm scratch.
Increase: ASA and NSAIDs

Platelet Count: Normal 150,000 – 450,000


Indication of cessation of bleeding following an operation.
Below 25,000 may indicate spontaneous bleeding

PT: Normal 11 – 16 sec


Measures the extrinsic coagulation pathway
Normal in Hemophilia A & B
Increase in Vitamin K, impaired fat absorption, liver dz, warfarin (coumadin)

PTT: Normal 25 – 35 sec


Measures the intrinsic coagulation pathway
Normal in thrombocytopenia, platelet dysfunction, von Willebrand’s dz
Increased: Hemophilia A & B, prothrombin complex disorders, heparin

Radiographic Analysis
- always obtained for any patient undergoing osseous resection, penetrating wounds,
ulcerations
- several standard views (usually 3)
- good quality
-available at time of surgery

Cultures
- any open wound, especially in diabetic or immunocompromised patients
- will help with individual tailoring of abx treatment

Ancillary Tests
- Women under the age of 50 – pregnancy or HCG test should be performed
- Patients over the age of 40 – EKG to rule out arrhythmias
- Over 50 with history of smoking – Chest X-ray
Case Study #2
HPI: 22 yr old female patient comes to the office seeking treatment for a three day history
of left foot and ankle swelling, redness and warmth. Pt does not complain of pain and
denies trauma or recent illness.

PMH: Type I diabetes x 12 years with associated neuropathy


Anorexia nervosa
Chronic diarrhea
Osteoporosis due to anorexia, chronic diarrhea and prolonged amenorrhea
Hospitalization: several times for tx of DKA and exacerbation of her psychiatric
problems

PE: Vitals: Ht 160 cm; Wt 52 kg; VSS, afebrile


Integ: right foot and ankle is erythematous, warm, swollen; no open lesions
Vasc: palpable pedal pulses
Neuro: diminished perception of light touch in a stocking distribution
M/S: normal muscle strength; no pain on palpation

Labs: ESR 130mm/hr


WBC 8.7 with normal differential
Hgb: 8.6
Hct: 29%
Platelet count: 700,000/mm3

X-ray:

Diagnosis: Acute charcot arthropathy

Treatment: Posterior splint with compressive dressing x 3 days


TCC applied once swelling subsided x 6 weeks
Increased WB in short leg cast x 4 – 6 months
Eventually went on to full WB in an AFO
Related Questions – Case Study #2
1. What are some differentials for this case study?
Acute charcot Acute septic arthritis
Osteomyelitis Avascular necrosis
Cellulitis Pigmented villonodular synovitis
DJD Psoriatic arthritis
Gout Rheumatoid arthritis

2. What are the four factors necessary for charcot to develop?


Peripheral neuropathy Hypervascularization
Repetitive microtrauma Unrecognized injury

3. Eichenholtz’s three stages of Charcot are….


I – Acute, Developmental, Destructive, Dissolution – foot is inflamed, edematous and
erythematous; radiographs demonstrate bone fragmentation (subchondral bone and
articular cartilage) and debris formation; joint dislocation/subluxation may be present;
typically lasts 6 – 8 weeks
II – Coalescence – healing begins and inflammation subsides; radiographs show
coalescence and resorption of bony fragments; larger fragments begin to fuse
III – Remodeling, Resolution, Reconstructive, Quiescence – inflammation is resolved;
return to normal radiographic density; reformation of the joint architecture.

4. What are the treatments associated with each stage?


Stage I – resolution of edema with compression, elevation and rest; total immobilization
of the limb by total contact casting, regular casting or pre-fabricated cast boot
Stage II – progressive weight bearing in regular or total contact cast; as the disease
progresses move from a cast to bracing such as a Charcot restraint orthotic walker
(CROW), double upright metal AFO or other AFO with progressive weight bearing
Stage III – long term AFO bracing; extra depth shoes or other types of accommodative
foot wear
***Remember that you must use clinical judgment to decide when to progress to the next
stage of treatment***

5. What are the 6Ds of Charcot?


Distension of joints Density increases
Dislocation Debris production
Disorganization Destruction

6. What disorder has the highest percentage of Charcot joints?


Syringomyelia; up to 25% will have charcot joints

7. What percentage of diabetic patients will develop charcot joints?


1 in 680
From .13% to 25%
This number depends on who is reporting it.
8. What are some disorders associated with Charcot?
- Diabetes - Spina bifida
- Tabes dorsalis (syphilis) - Meningomyelocele
- Syringomyelia - Congenital insensitivity to pain
- Chronic alcoholism - CMT
- Hansen’s disease (leprosy) - Multiple sclerosis

9. Describe the two theories associated with the development of Charcot joints.
Neurovascular (French) Theory  damage to the trophic nervous centers with an
alteration in the sympathetic control of blood flow to bones and joints leads to persistent
hyperemia and active bone resorption
Neurotraumatic (German) Theory  An extreme progression of degenerative joint
disease following loss of proprioception and protective pain sensation

10. How do you tell the difference between Charcot and osteomyelitis?
- use of the MRI is becoming popular
- definitive way: bone biopsy with microbiological diagnosis
- use of the WBC labeled bone scan – osteomyelitis will be hot, charcot will not be hot
- Plain radiographs:
Charcot Osteomyelitis
Bone destruction and fragmentation Bone destruction present
Increased soft tissue density Increased soft tissue density
Sclerosis may be present; osteoporosis may Early sclerosis with osteoporosis in later
be present as diabetic osteolysis progression
Progressive resorption is usually not Progressive resorption is present
present
Sequestrum formation may be mimicked Sequestrum formation occurs
by fragmentation
Subperiosteal new bone formation Subperiosteal new bone formation

11. What are the patterns of bone and joint involvement?

Sanders & Frykberg. The Charcot foot, 1991.


I – Forefoot Often atrophic and destructive findings Reported to occur
mimicking osteomyelitis 26 – 27%

II – Tarsometatarsal Often with collapse of the midfoot, plantar 15 – 43%


ulceration often occurs at the apex of the
deformity
III – Talonavicular, Characterized by dislocation of the 32%
CC, navicular; can be solo or with other
Navicularcuneiform patterns
IV – Ankle joint Almost always results in severe deformity 3 – 10%
and instability. STJ can also be involved
V – Calcaneus Insufficiency avulsion fx, least commonly
reported
Brodsky J et al. Patterns of breakdown in the Charcot tarsus of diabetes and relation to
treatment. Foot Ankle 5(12):353, 1986.

Type 1 (Midfoot) Metatarsocuneiform Most common ~ 60%


&
naviculocuneiform
Type 2 (Hindfoot) STJ, CC and TN jt Less common 30 – 35%
Type 3A (Ankle) Ankle joint Serious long lasting Associated with
changes discrete trauma
Type 3B Tubercle of the Least common Associated with
(Calcaneus) calcaneus Secondary discrete trauma
deformity and
collapse of the distal
portion of the foot

Schon LC et al. Charcot Neuroarthropathy of the foot and ankle. Clinical orthopedics and
related research. 349, 1998.

Type I: Lis franc pattern


Type II: Naviculocuneiform pattern
Type III: Perinavicular pattern
Type IV: Transverse tarsal pattern
Subtypes A, B and C with increasing severity from A to C

12. During what stage(s) should surgical intervention be attempted?


Well, like everything else in medicine recently, it depends on who you read.

Eichenholtz: early stage I or late stage III


Charcot joints pp 7 – 8. Springfield, IL 1996

Mann: early stage I (before development of osteopenia from local hyperemia) or late
stage II when new bone formation is occurring
Foot and Ankle Surgery, 1999.

Johnson: operative treatment is usually carried out in Stage III after conservative care
has been exhausted. An acute charcot fracture can be treated with ORIF in stage I if
performed early before bone stock become insufficient for fracture.
Johnson JE. Operative treatment of neuropathic arthropathy of the foot and ankle. JBJS
80A Nov 1998.

Myerson: surgery is necessary in the acute phase of skin necrosis resulting from the
pressure of a dislocated bone is imminent or other skin problems are likely to occur as a
result of casting. Surgery is also indicated at this time for marked dislocation of the foot
that is likely to cause problems with shoewear at a later date. Surgery can only be
performed if bone stock is adequate, therefore, he rarely operates if more than 6 weeks
after the beginning of the acute phase. In the setting of acute neuroarthropathy associated
with infection and surgery is indicated, Myserson uses an external fixator to stabilize the
foot. Surgery should probably be reserved until Stage II or III.
Foot and Ankle Disorders, 1999.

Simon: First paper to demonstrate successful results in operative treatment in stage I


Charcot. Authors operated on 14 patients in the acute phase of the disease with good
anatomical reduction and good results. All patients had anatomic reduction, clinical union
and stability without an increased risk of adverse outcome. Given this outcome along
with decreased cost when compared to ulcer care, authors believe that early arthrodesis
should be considered in patients with early stage I charcot.
Simon SR et al. Arthrodesis as an early alternative to non-operative management of
charcot arthropathy of the diabetic foot. JBJS 82A July 2000.

13. What is the natural history of Charcot arthropathy?


- the foot goes numb
- the bones become soft
- the patient walks on the numb foot with soft bones
- the soft bones fracture
- the fractures heal with malposition of the fragments and deposition of fracture
fragments in the adjacent joint capsule
- the process repeats
- the foot becomes shorter and wider and develops angular bony prominences
- the foot becomes ulcerated and infected as the patient walks on the angular prominences
- all hell breaks loose

14. What is the primary deforming force in many cases of Charcot?


Ankle joint equinus

15. What is the history of Charcot in the literature?


Musgrave 1703 – first to report neuropathic osteoarthropathy as an arthralgia secondary
to venereal disease

Jean Marie Charcot 1868 – French neurologist. Concisely describe the neuropathic
component of the disease. Linked the condition to syphilis which was very common at
the time.

Jordan 1936 – Linked this condition to diabetes mellitus.

Etiology of the Condition:


Charcot 1868 – Deficient trophic centers in the spine

Volkman, Virchow and the “German School” – neurotraumatic in nature. The


neuropathic foot would fracture with exuberant bone formation.
Eloesser (1917) – sectioned posterior nerve roots in cats and noted neurotrophic changes
in 71% of the animals
Finsterbush & Friedman (1975) – repeated Eloesser’s experiment in rabbits but casted the
rabbits hind legs. Different response noted in denervated and sensate rabbits. Concluded
that trauma important but not primary factor.

Edmonds et al (1985) – Increased blood flow was partially responsible for the osteopenia
and ‘washing out of bone’ seen in Charcot. Neurovascular theory.

16. What are the relevant lab tests that can help lead to a diagnosis of charcot?
WBC will be elevated with a left shift on the differential  acute osteomyelitis
ESR is non specific  may not be helpful
Tech 99 bone scans  may or may not be helpful
Indium 111 bone scans  much more specific.
If +  do bone biopsy to confirm osteomyelitis and r/o Charcot
If -  dx is charcot until proven otherwise
Certec scan  labeled white blood cells. May also be helpful

Probe to bone
If +  procede to bone biopsy
If -  continue with other tests

17. What is the ‘gold standard’ for diagnosis?


Bone biopsy
Will histologically differentiate between osteo and charcot.

18 . What is a plan for the management of Acute Charcot?


- Immobilization
- Reduction of stress
- Armstrong advocates the TCC (surprise, surprise) The reduced stride length and
walking cadence will decrease the possibility of causing charcot in the other limb

19. What are the radiographic signs of Stage III Charcot ?


trabecular bridging on serial radiographs
Hammer Digit Syndrome
Three main categories of pathomechanical causes of hammer digits:
- flexor stabilization
- extensor substitution
- flexor substitution

What must happen in order for the rigid beam effect of the digits to occur?
The muscles involved with gait must fire in the correct order. Breaks in the sequence lead
to inability of the toes to purchase the ground.

Flexible deformities  may be able to be managed with biomechanics; may not need
surgery

Rigid deformities  surgical correction probably necessary

Flexor stabilization:
- occurs in stance phase of gait
- FDL gains mechanical advantage of the interossei
- Can occur in flexible flatfoot, neuromuscular dz that leads to weakness of the interossei,
spasticity of the FDL
- Flexible flatfoot: quadratus plantae also looses mechanical advantage; FDL fires longer
and sooner to help stabilize the floppy and abducted forefoot; oblique pull of FDL leads
to adductovarus deformity of the 5th, 4th and possibly 3rd. SELDOM the 2nd. Contraction
of these digits will be seen in mid to late stance phase of gait.

Extensor substitution:
- excessive dorsiflexion of digits during swing phase of gait and at heel strike
- EDL has gained mechanical advantage over the lumbricales
- causes: weak lumbricles, spastic EDL, pes cavus (especially flexible pes cavus), weak
anterior group, contracture of the Achilles tendon, pain in the anterior part of the foot

Surgical Correction:
- Hibbs tenosuspension
- detach the EDL at the MPJ
- EDL attached to 3rd cuneiform
- EDB transected and sewn to the EHL
- may work to control HDS caused by extensor substitution
- arthrodesis of the proximal interphalangeal joint
- prox phalanx head resection with soft tissue release will lead to rapid recurrence
- AFO or dropfoot leg brace

Flexor substitution:
- occurs when there is weakness of the triceps surae
-FDL gains mechanical advantage as in flexor stabilization but b/c there is weakness in
the Achilles tendon there is late stance phase supintaion rather than pronation
Flexor substitution:
- arthrodesis of the PIPJ is often required
- transfer the PT tendon and the PL tendon to the calcaneus
- combination of the above

Surgical Incisions for HDS surgery:


- longitudinal
- longitudinal oval
- transverse
- transverse oval
- curvilinear
- derotational ellipse for 5th digit

Intraoperative Stepwise Approach


- IP joint tenotomy/capsulotomy
- resect the head of the proximal phalanx (post procedure)  reduces bony prominence
and reduces the painful dorsal corns
- release of the extensor sling and hood in longitudinal fashion parallel to the EDL and
EDB tendons
- Z plasty of the extensor tendons
- capsulotomy of the MPJ joint
- plantar release
- flexor tendon transfer
- proximal phalanx arthrodesis of the PIPJ
The Wondrous World of Hallux Abducto Valgus
Goals of HAV Surgery
1. The establishment of a congruous 1st MPJ
2. The reduction of the abnormal osseous angles to normal parameters
3. Realignment of the sesamoid bones beneath the 1st metatarsal head – TSP <3
4. Restoration of weight bearing function of the 1st ray
5. Maintenance of the 1st MPJ pain free ROM
6. Repositioning of the hallux in rectus alignment
7. Decreasing ‘the bump’
8. Control or correction of the factors which lead to the deformity

Etiology of HAV
1. Primary  Pathobiomechanics
a. Pronation is the root of all evil. We should know this by now.
b. Hypermobile 1st ray
c. Long 1st ray
d. LLD
e. Metatarsus primus elevatus (this is more a hallux limitus issue, however)
2. Secondary
a. Trauma (like gun shots)
b. Rheumatic: look at the RA x-rays, this is pretty obvious
c. Neuromuscular: tendon/muscle imbalance
d. Genetic or collagen disorders: Marfan’s or Down Syndrome
e. Flat foot
f. Gout
g. Iatrogenic (i.e. tibial sesamoidectomy)

Pathomechanics of HAV
This is a progressive disorder characterized by 4 stages. The longer that it takes to
progress from one stage to the next, the less severe the resulting deformity will be. The
factors which affect the HAV deformity are:
1. walking surface
2. shoe gear
3. length of patient’s stride (increasing the propulsive phase)
4. STJ subluxation
5. MTJ subluxation
6. obesity
7. degree of calcaneal eversion
8. forefoot adduction
9. inflammation of the MPJ
Patho-Anatomy of HAV
There are 4 articular facets within the synovial capsule:
1. metatarsal head
2. base of the proximal phalanx
3. tibial sesamoid
4. fibular sesamoid
The 1st MPJ has 9 ligaments
1. 2 collateral ligaments
2. 2 metatarso-sesamoidal ligaments
3. 2 phalangeal-sesamoidal ligaments
4. Intersesamoidal ligament
5. Deep transverse metatarsal ligament
6. Capsule

The 1st MPJ capsule incorporates 5 tendons:


1. FHB
2. Abductor hallucis
3. Adductor hallucis conjoined tendon
4. EHL
5. EHB
 The FHL is the only tendon that does not have an attachment to the MPJ capsule
 The weakest part of the MPJ capsule is dorsomedial.

Stages of HAV
Stage I Lateral displacement of the proximal phalanx relative to the first
metatarsal head
Sagittal groove/sulcus where the corner of the proximal phalanx rubs into
the metatarsal head
Stage II Development of hallux abducto valgus deformity; the hallux abuts the 2nd
toe
1st MPJ is fairly congruent
Hallux abductus = abnormal
IM = normal
PASA = increased  more lateral deviation of the cartilage
Stage III Increase in 1st IM angle
Stage IV Subluxed or dislocated hallux on the 1st metatarsal head
Axis Guides
Guides (usually .062 K-wire) designed to change the direction of the osteotomy.

So why do I care?
- this allows correction in the 2 planes
- these can change the direction of the osteotomy cut

What would you like to do? Direct the Axis Guide like this…
Shorten the met Distal medial to proximal lateral
Lengthen the met Proximal medial to distal lateral
Plantarflex the met No angulation from anterior to posterior
Dorsal medial to plantar lateral
Dorsiflex the met No angulation from anterior to posterior
Plantar medial to dorsal and lateral
Angle Formed By: Importance/Location of Normal Value Procedure
Deformity
Hallux - bisection of distal phalanx IPJ of hallux 0 – 10 Osseous
Interphalangeus - bisection of proximal phalanx Osseous deformity
Ex. Distal Akin
DASA - perpendicular to bisection of Fixed deformity 7.5 Osseous
prox phalanx Prox phalanx  hallux abduction (0 – 8) in some Phalanx
- cartilage prox phalanx base literature
Ex. Proximal Akin
PASA - perpendicular to 1st met Fixed deformity 7.5 Osseous at 1st met head
bisection Trackbound joint (0 – 8) in some
- line connecting effective Lateral deviation/ abduction of prox literature Ex. Head wedgie procedure
cartilage of 1st met head phalanx on 1st met head
Hallux Abductus - prox phalanx shaft MPJ soft tissue deformity 10 – 15 Capsule tendon balance – or –
- 1st met head Osseous
Depending on the other angles
involved
IM Angle - bisection 1st met 1st ray fixed deformity 8 – 12 in rectus <14 – head
- bisection 2nd met foot 15 – 17 shaft
8 – 10 in >17 base
adducted foot
Metatarsus - lesser tarsal axis Increase in MA = increase in <15 Look at the IMt
Adductus - bisection 2nd met significance of other angles
Large MA  HAV develops faster CBWO
Metatarsal - arc of 1st +  longer 1st met +/- 2 mm Chose carefully
Protrusion - arc of 2nd -  shorter 1st met
-
difference in mm
Tibial Seasmoid - position of tibial seasmoid > 4 may indicate fib seasmoid 1-3
- bisection of 1st met removal or degenerating cristae
Metatarsal Head Shapes
Round – usually most prone to developing HAV,
most reducible yet most the most unstable
Square – more stable
Square with a ridge – oblique or central ridge,
helps prevent lateral deviation of hallux;
compensatory hallux interphalangeus may be
seen with this; more commonly associated with
hallux rigidus or limitus; most stable

Metatarsal Phalangeal Joint Positions


Congruous – two articular surfaces are wholey
articular and the joint space is equal
Deviated – the lines representing the articular
surfaces intersect outside the joint
Subluxed – the lines representing the articular
surfaces intersect inside the joint

Reading a Radiograph to Evaluate HAV


1. Remove the hypertrophied eminence
2. Bring 1st metatarsal head closer to the second metatarsal head – IM angle
3. Obtain or maintain a congruous joint – PASA/DASA
4. Straighten the toe – HA/HAI

Capsulotomies
A – Vertical
B – U shaped
C – H shaped
D – T shaped
E – L shaped

A – Linear
B – T shaped
C – Inverted L

E – Lenticular from dorsal view


F – Oblique suture closure  corrects a frontal plane deformity
G – Horizontal suture closure  corrects a transverse plane deformity

C – Washington Monument type capsulotomy


D – Suture to close the capsule
E – Replace the capsular flap; aids in correction
Capsule Tendon Balancing Procedures
Schede’s Exostectomy (1870s) Lothrop’s Lateral Release (1873)
- remove the bump - lateral release

Silver (Silver, David: The operative treatment of hallux valgus. JBJS 5:225, 1923)
1. curved medial incision
2. removal of bursal sac
3. resection of dorsomedial eminence
4. lateral capsulotomy with medial capsulorraphy designed to reinforce the medial
collateral ligament and transpose dorsally the abductor hallucis tendon
5. lateral release

McBride (McBride, Earl: A conservative operations for bunions. JBJS 10:735, 1928)
1. 5 cm incision from the toe web along the lateral border of EHL
2. resection of the dorsomedial eminence
3. lateral release consisting of lateral capsulotomy with fibular sesamoid excision
4. transfer of the adductor hallucis and lateral head of FHB to the dorsolateral aspect
of the 1st met head
 Adductor tendon transfer
1. dorsally through the capsule to reposition the sesamoids
2. into the 1st metatarsal neck to reduce the IM angle
 transfer into the neck can be either dorsal to plantar or lateral to medial

Hiss (Hiss, J: Hallux valgus, its cause and simplifies treatment, Am J Surg 11:51, 1931)
1. Resection of the dorsomedial eminence
2. lateral capsulotomy
3. transfer of abductor hallucis from plantar to medial to the 1st MPJ (abductor
sutured into hole in the base of the proximal phalanx; acts as a ligament to
strengthen the capsule medially)

Stein (1938)
1. lateral release
2. resection of the medial eminence
3. Medial capsuloplasty to reposition to abductor hallucis more medially

Joplin’s Sling (1950)


1. Modified McBride procedure by an adductor hallucis transfer through the
metatarsal head to the medial side, instead of laterally on the head.
2. Very similar to the mini-tightropes being now used for juvenile HAV

Component Procedures
1. adductor transfer
2. capsulorraphy
3. EHL tendon lengthening
4. EHB tenotomy
Phalangeal Osteotomies
Proximal Akin (Akin, OF: The treatment of hallux valgus: a new
operative approach and its results. Med Sentinel 33:678, 1925.)

1. Corrects for DASA


2. Osteotomy performed 5 – 10 mm from the MPJ
3. Proximal cut is parallel to the base of the
proximal phalanx
4. Distal cut is perpendicular to the bisection of the
proximal phalanx

Distal Akin
1. Corrects for a large hallux interphalangeus angle
2. Performed 5 – 10 mm from the IPJ of the hallux
3. Distal cut is parallel to the surface of the head of the proximal phalanx
4. Proximal cut is perpendicular to the long axis of the shaft
5. Base medial; apex lateral
6. Fixation with 28-guage wire, single 0.045 K wire or 0.062 K wire with hinge
intact, if broken, 2 crossed K wires

Cylindrical Akin
Type A – Corrects for long proximal phalanx
Type B – Corrects for a long proximal phalanx and a
high DASA and/or HI angle
1. Proximal cut 1 – 1.5 cm distal to the
phalangeal base
2. Distal cut parallel to first cut or angulated to
reduce abductus
3. Fixation with crossed K-wires (0.045 or 0.062)

Oblique Akin
1. Indicated for and used to correct an increased hallux interphalangeus angle and an
increased DASA
2. Leave the hinge intact
 The following two procedures are named for where the apex is (proximal vs
distal). The apex of the cut is always lateral.
o Oblique Proximal
 Base: medial and distal
 Apex: lateral and proximal
o Oblique Distal
 Base: proximal and medial
 Apex: distal and lateral
Shortening Sagittal Z
1. Good for a long proximal phalanx
2. Remove the darkened areas of the picture
3. Shortens the long hallux and reduces forces at the 1st met
head  enhanced motion

Distal First Metatarsal Head Osteotomies


Traditional Austin (Austin DW, Leventen EO: A New Osteotomy for hallux valgus: a
horizontally directed “V” displacement osteotomy of the metatarsal head for hallux
valgus and primus varus. Clinical Orthop. 157:105, 1981)

1. Resection of medial eminence


2. Osteotomy performed in the center of the metatarsal head with 60
degree angle between arms (plantar cut is made first)
 Unicorrectional: normalizes IM angle
 Bi correctional: normalized IM angle and PASA
4.Axis guides – utilize to lengthen, shorten, plantarflex or dorsiflex

Youngswick – Austin Modification (Youngswick FD: Modifications of the Austin


bunionectomy for treatment of metatarsus primus elevatus associated with hallux limitus.
J Foot Surg 21:114, 1982)

1. Indicated for both hallux valgus and hallux limitus


(early stages)
2. Resection of the medial eminence
3. Angle of the cut is 60 degrees
4. Plantar cut is the same as the Austin procedure.
5. Rectangular piece of bone is excised dorsally (Shortening the
metatarsal head and decompressing the MPJ)

Gerber-Massad (Bicorrectional) Modification to the Austin


1. Indicated for increased IM angle (mild) and increased PASA (mild)
2. Resection of the medial eminence
3. Plantar wing cut is the same and previously mentioned procedures
4. Dorsal wing is a medially based wedge of bone
5. Lateral movement decreases the IM angle
6. Closing the wedge rotates the head medially and decreases PASA
Kalish (Long Dorsal Arm) Modification to the Austin
1. Indicated for a larger IM angle
2. The cut is made at a 55 degree angle
3. Same plantar cut as previously mentioned procedures
4. Long dorsal arm that increases stability & allows for screw fixation
5. Reduce the larger IM angle (you are moving more bone)
6. Unable to swivel capital fragment to correct PASA

Reverdin (Reverdin J: De la deviation en dehors du gros orteil et de son traitment


chirugical. Tr Internat Med Congr 2:408, 1881)

1. Primarily used in conjuction with other procedures.


2. Indicated for an increased PASA and used by itself will only correct
PASA.
 Performed in metaphyseal bone
 Vertical wedge cut in the distal portion of the metatarsal
head
 Distal cut (1st cut) is parallel to abnormal PASA
 Proximal cut (2nd cut) is perpendicular to long axis of the
1st met
 Lateral hinge should be preserved to increase stability

Reverdin-Green (1977)
1. Indicated for an increased PASA
2. First cut is made parallel to the weight-bearing surface and
exits the bone just proximal to the 1st MTH avoiding the
articular surface
3. This preserves the articulation between the metatarsal heads
and the sesamoids

Reverdin-Green-Laird (1977)
1. Indicated for high PASA and high IM angle
2. Lateral cortex is cut, allowing lateral transposition of the capital fragment

Reverdin-Todd (1978)
1. Corrects PASA & IM angle
2. Can be used to PF the metatarsal (for met primus elevatus)
3. Proximal ostetomy is angled either dorsal-distal to plantar-proximal
(plantarflexing the capital fragment) or dorsal proximal to plantar distal
(dorsiflexing the capital fragment)

Peabody
1.Same as the Reverdin expect
osteotomy is performed at the anatomical neck.
Hohmann
1. Indicated for high PASA, IM angle and sagittal plane displacement
2. Trapezoidal osteotomy (base orientated medially) at the level of the metatarsl neck.
Through and through cut
3. Avoids sesamoidal irritation because this is a neck procedure.

Mitchell
1. Similar to the Hohmann.
2. Medial eminence resection
3. Lateral cortical strut left intact
4. No lateral release performed with this procedure.
5. Indications: Long first metatarsal (This procedure shortens the
1st met the most of any procedure)

Wilson
1. Oblique osteotomy oriented distal medial to proximal lateral
(through and through)
2. Capital fragment is shifted proximally and laterally along the
osteotomy
3. Capital fragment can also be translated dorsally or plantarly

DRATO (Smith, 1971)


1. Derotational Angulated Tranpositional Osteotomy
2. Distal osteotomy – perpendicular to the long axis to the 1st
metatarsal; through and through cut
3. Proximal osteotomy – angled proximal medial to distal lateral
(PASA correction)
4. Dorsiflexes capital fragment to place the articular cartilage in a
more functional position attempting to increase dorsiflexion at the 1st
MPJ
Shaft Procedures for HAV
Vogler Offset V
1. Apex of osteotomy at the metaphyseal-diaphyseal junction
2. Angle is 40 degrees
3. Longer dorsal arm allows for greater bone to bone contact
4. Length of dorsal wing determines how much swivel is
possible
5. This is NOT the same procedure as a long arm Austin.

Scarf or Z Osteotomy
1. Distal cut dorsal; proximal cut
plantar  avoids the possibility of
scoring the sesamoid apparatus;
easier to fixate
2. Troughing effect: dorsal segment
of the Z (cortical bone) wants to sink
into the plantar segment when screws are tightened down.

Mau (1926)
1. The cut is distal dorsal to proximal plantar
(‘the airplane is taking off to Maui’)
2. You can use the angle of the cut to swivel the bone and reduce the
PASA

Ludloff (1918)
1. Cut medial to lateral through the bone from dorsal and proximal to
plantar and distal
2. You can use the angle of the cut to swivel the bone and reduce the
PASA
Proximal Osteotomies for HAV
Crescentic (Weinstock)
1. Indicated for high IM angle, short metatarsal
2. ½ moon shaped osteotomy created approximately 1.5 cm
distal to 1st met-cuneiform joint
3. When looking at the metatarsal from dorsal to plantar,
the concave portion of the osteotomy faces distally
4. The distal segment can be dorsiflexed or plantarflexed
5. The distal segment can be rotated until desired
correction is obtained

Juvara (Oblique closing base wedge, 1919)


1. Long oblique osteotomy with a medial hinge
2. Originally described as a ‘double oblique section of the metatarsal’ performed at the
junction of the middle and proximal thirds of the metatarsal shaft
3. Modifications
 Oblique osteotomy from distal lateral to proximal medial
 Proximal cortical apex is maintained
 Fixation with one or two screws
4. Type A – Corrects the transverse plane
Type B1 – Corrects the transverse and sagittal
planes
Type B2 – Corrects the transverse, sagittal
planes & long or short met
Type C1 – Sagittal correction only
Type C2 – Sagittal and long/short met
correction

Loison – Balacescu
(Loison M: Note sur le traitment chirurgical du hallux valgus, d’apres
l’etude radiographique de la deformation, Bull Soc Chir 27:528, 1901.)
(Balacescu J: Un caz de hallux valgus simetric, Rev Chir Orthop 7:128,
1903)
1. Originally described as being at a point just distal to the insertion of the
peroneus longus tendon
2. Closing base wedge 1 cm from the met-cuneiform junction with medial
cortex left intact

Logroscino Double Osteotomy (Logroscino, D: IL trattemento


chirugico dell’aluce valgo. Chir Organi 32:81, 1948)
1. Double osteotomy of the first metatarsal
2. Indicated for increased PASA (distal wedge)
3. Indicated for increased IM angle (proximal closing wedge)
4. For short first met: Reverdin and Trethowan
5. For long first met: Reverdin and Loison
Trethowan (Trewthowan J: Hallux valgus. In Choyce CC, ed: A System of Surgery,
New York 1923, PB Hoeber)
1. Resection of the medial eminence
2. Use this resected bone for an opening base wedge (base of
which is medial) of the 1st met base

Opening Base Wedge

3:1
1. Capsule tendon balance procedure
2. Closing abductory base wedge
3. Proximal akin osteotomy

Opening wedge of the medial cuneiform

Lapidus
(Lapidus PW: Operative correction of metatarsus varus
primus in hallux valgus, Surg Gynecol Obstet 58: 183, 1934)
1. Denuded cartilage of 1st met-cuneiform joint
2. Decorticated lateral aspect of the base of the 1st met and the
adjacent surface of the 2nd metatarsal base
3. Packed bone chips obtained from the resected medial
eminence
4. Indicated for very high IM angles, a medially orientated
proximal edge of the medial cuneiform and a hypermobile
first ray
Complications
1. Hallux varus
A. Etiology
1. Staking the head
2. Negative IM angle
3. Aggressively biasing the capsule
4. Negative PASA
5. Fibular sesamoidectomy
6. Aggressive post-op bandaging
B. Sequence of surgical correction
1. Scar revision
2. Dissection in planes
3. Capsuloplasty, V-Y or Z plasty or advancement (not simple linear
capsulotomy)
4. Degloving the articulation
5. Adductor tendon reefing
6. Abductor tendon transfer to lateral side
7. EHL lengthening
8. If dislocated, tibial sesamoidectomy (if, at this point, both sesamoids
have been removed do an IPJ arthrodesis)
9. Reverse Akin, Austin, Hohmann
10. Arthrodesis
2. Tibial sesamoid dislocation
A. Etiology
1. Staking the head and removing the sagittal groove
2. Overtightening the medial capsule
3. A negative IM angle
3. Capsular defect
4. Staking the head – excessive bone removal on the medial 1st MT head, losing the
plantar tibial sesamoid groove
5. Capital fragment on the floor (Christenson C: Intra operative contamination of the
bone and cartilage during an Austin bunionectomy 31:285, 1992)
A. Mix the following: 1 L normal saline, 1 ml Neosporin GU irrigant and
1:100,000 bacitracin
B. Place fragment in this solution x 5 minutes
C. Transfer to second basin with same solution x 5 minutes
D. Transfer to third basin with same solution x 1 minute
E. 1 gm Cefadyl IV @ 8, 16 and 24 hrs post op (1st generation cephalosporin)
6. Severing the EHL or the FHL 14. AVN
7. Infection 15. Nerve entrapment
8. Breaking the hinge 16. Sesamoiditis
9. Intra-articular osteotomy 17. Hallux limitus
10. Recurrence 18. Fixation failure
11. Delayed or non-union of the osteotomy site
12. 1st met shortening
13. 1st met elevatus
HAV Questions
What are the goals of HAV surgery?
1. The establishment of a congruous 1st MPJ
2. The reduction of the abnormal osseous angles to normal parameters
3. Realignment of the sesamoid bones beneath the 1st metatarsal head – TSP <3
4. Restoration of weight bearing function of the 1st ray
5. Maintenance of the 1st MPJ pain free ROM
6. Repositioning of the hallux in rectus alignment
7. Decreasing ‘the bump’
8. Control or correction of the factors which lead to the deformity

Which of the above factors do the patients care most about?


1. Decreasing ‘the bump’
2. Hallux in rectus alignment

What are the etiologies of HAV?


hypermobile 1st ray
genetic disorders
neuromuscular disorders
rheumatic disorders
tibial sesamoidectomy

Which bones in the 1st met capsule have articular facets?


Metatarsal head
Proximal phalangeal base
Tibial sesamoid
Fibular sesamoid

Name the ligaments in the 1st MPJ apparatus.


2 collateral ligaments
2 metatarsal-sesamoidal ligaments
2 phalangeal-sesamoidal ligament
intersesamoidal ligament
deep transverse metatarsal ligament
capsule

Name all of the tendons incorporated into the 1st MPJ.


FDB
Adductor hallucis
Abductor hallucis
EHL
EHB

Which hallucial tendon has no attachement to the 1st MPJ capsule?


FHL
What is the weakest part of the 1st MPJ capsule?
The dorsomedial aspect of the capsule

What are these fibers called?


Sharpey’s fibers

Name the radiographic parameters that are used when evaluating a radiograph for HAV
surgery and give the normal values.
Met adducuts angle normal = 15
1st IM angle normal = 8 – 12
Hallux abductus normal < 15
PASA normal < 7.5
DASA normal < 7.5
HAI normal < 10
Met protrusion distance < 2 mm
Tibial sesamoid position normal 1 – 3

What are the different shapes of the 1st metatarsal head and why are they important?
Round – usually most prone to developing HAV, most reducible yet most the most
unstable
Square – more stable
Square with a ridge – oblique or central ridge, helps prevent lateral deviation of hallux;
compensatory hallux interphalangeus may be seen with this; more commonly associated
with hallux rigidus or limitus; most stable

What are the different metatarsal phalangeal joint positions?


Congruous – two articular surfaces are wholey articular and the joint space is equal
Deviated – the lines representing the articular surfaces intersect outside the joint
Subluxed – the lines representing the articular surfaces intersect inside the joint

What are axis guides and how do you orient them to get the desired effect?
K-wire inserted primarily to define the apex and orientation of the osteotomy; most
importantly serves to determine the direction of displacement for the capital fragment in
all three body planes
Distal Osteotomies
Frontal plane: describes the lateral movement of the capital fragment in respect to the
weight-bearing surface
- from medial to lateral the K wire is directed dorsally, the capital fragment will shift
dorsally
- from medial to lateral the K wire is directed plantarly, the capital fragment will shift
plantarly
Transverse osteotomies: dictates lengthening or shortening of the metatarsal with lateral
shifting
- K-wire orientated parallel to the shaft of the 2nd metatarsal, with lateral displacement, the
met will be shortened approximately the width of the saw blade
- K-wire orientated distal medial to proximal lateral, shortens the met
- K-wire orientated proximal medal to distal lateral, lengthens the met
Name the Medial capsulotomies. (VW HULT)
Vertical
H shaped
U shaped
L shaped
T shaped
Lenticular
Washinton Monument

Name the dorsal capsulotomies. (LiLT)


Linear
Inverted L shaped
T shaped

Describe the Washington Monument. (Not the one in Washington, the capsulotomy)
Incision into the capsule medially
Reflect the flap proximally
Close the capsule
Replace the capsule flap
This aids in the correction of hallux valgus

Name all of the Akin procedures & describe them to the best of your ability.
Proximal Akin (Akin, OF: The treatment of hallux valgus: a new operative approach and
its results. Med Sentinel 33:678, 1925.)
Corrects for DASA
Osteotomy performed 5 – 10 mm from the MPJ
Proximal cut is parallel to the base of the proximal phalanx
Distal cut is perpendicular to the bisection of the proximal phalanx
Distal Akin
Corrects for a large hallux interphalangeus angle
Performed 5 – 10 mm from the IPJ of the hallux
Distal cut is parallel to the surface of the head of the proximal phalanx
Proximal cut is perpendicular to the long axis of the shaft
Base medial; apex lateral
Fixation with 28-guage wire, single 0.045 K wire or 0.062 K wire with hinge intact, if
broken, 2 crossed K wires
Cylindrical Akin
Type A – Corrects for long proximal phalanx
Type B – Corrects for a long proximal phalanx and a high DASA and/or HI angle
Proximal cut 1 – 1.5 cm distal to the phalangeal base
Distal cut parallel to first cut or angulated to reduce abductus
Fixation with crossed K-wires (0.045 or 0.062)
Oblique Akin
Indicated for and used to correct an increased hallux interphalangeus angle and an
increased DASA
Leave the hinge intact
 The following two procedures are named for where the apex is (proximal vs
distal). The apex of the cut is always lateral.
o Oblique Proximal
 Base: medial and distal
 Apex: lateral and proximal
o Oblique Distal
 Base: proximal and medial
 Apex: distal and lateral

Name all of the Reverdin modifications and describe the modification.


Reverdin (Reverdin J: De la deviation en dehors du gros orteil et de son traitment
chirugical. Tr Internat Med Congr 2:408, 1881)
1. Primarily used in conjunction with other procedures.
2. Indicated for an increased PASA and used by itself will only correct PASA.
 Performed in metaphyseal bone
 Vertical wedge cut in the distal portion of the metatarsal head
 Distal cut (1st cut) is parallel to abnormal PASA
 Proximal cut (2nd cut) is perpendicular to long axis of the 1st met
 Lateral hinge should be preserved to increase stability
Reverdin-Green (1977) – added plantar shelf
1. Indicated for an increased PASA
Reverdin-Green-Laird (1977) – lateral cortex is cut, allowing for lateral transposition of
the capital fragment
Reverdin-Todd (1978) – Proximal osteotomy is angled either dorsal-distal to plantar-
proximal (plantarflexing the capital fragment) or dorsal proximal to plantar distal
(dorsiflexing the capital fragment)
1. Corrects PASA & IM angle
2. Can be used to PF the metatarsal (for met primus elevatus)

What does DRATO stand for?


De-rotational angulated transpositional osteotomy

Describe the orientation of the osteotomy for a Scarf Z and a reverse (inverted) Z
osteotomy.
For both of these osteotomies, the osteotomy is horizontally directed displacement Z cut
that is performed at the diaphyseal junction level in an upper 2/3rd , lower 1/3rd manner.
The distal apex is placed 1 cm proximal to the articular surface. The horizontal bone cut
is approximately 2.5 – 3.0 cm in length and is angled 5 – 10 degrees in relation to the
transverse plane in a plantar lateral direction.
Scarf Z – Distal cut is dorsal, plantar cut is proximal. Both are angulated 70 – 80 degrees
to the horizontal cut
Reverse Z - Distal cut is plantar, proximal cut is dorsal. Both are angulated 70 – 80
degrees to the horizontal cut

Differentiate between a Mau and a Ludloff.


Mau – osteotomy oriented from plantar proximal to dorsal distal (Looking at this cut
from the medial aspect of the foot, it looks like a plane is taking off to MAUI and flies
over the toes before hitting the ocean)
Ludloff – osteotomy oriented from dorsal proximal to plantar distal

What are the Juvara subtypes and what do they correct?


Type A – Corrects the transverse plane
Type B1 – Corrects the transverse and sagittal planes
Type B2 – Corrects the transverse, sagittal planes & long or short met
Type C1 – Sagittal correction only
Type C2 – Sagittal and long/short met correction

Name as many bunion procedures as you can and categorize them according to
osteotomy placement (i.e. head vs shaft vs base, etc)
1. Phalangeal
Proximal Akin
Distal Akin
Transverse Akin
Proximal Oblique Akin
Distal Oblique Akin
Cylindrical Akin
Sagittal Shortening Z
Keller
2. Capsule tendon balance
Silver
McBride
Adductor tendon transfer
3. Head
Schede
Austin
Traditional
Youngswick
Gerber – Massad (bicorrectional)
Kalish (long dorsal arm)
Logriscino (distal part)
Reverdin
Green
Laird
Todd
4. Neck
- Mitchell
- Hohman
- Peabody
- DRATO
5. Shaft
Scarf
Off-set V
Mau
Ludlow
6. Base
- Loison – Balacescu
- Juvara
- Cresecentic
- Trethowan
- Logriscino (proximal part)
7. 1 met/1 cuneiform
- Lapidus

Why might you want to dorsiflex the hallux when making the initial incisions of an
Austin?
Protects the articular cartilage of the 1st metatarsal head

Describe the hinge axis principle.


(Smith TF: The hinge concept in base wedge osteotomies. In Schlefman B (ed): Doctors
Hospital Podiatry Institute Seminar Manual. Tucker, GA. 1983, 66 – 68)
The function of an intact cortical hinge in a closing base wedge osteotomy acts as an axis
of rotation.
Traditional axis of the base wedge osteotomy was perpendicular to the metatarsal,
subsequent lateral displacement of the metatarsal will result in sagittal plane dorsiflexion.
If the osteotomy is made perpendicular to the weight bearing surface, then lateral
displacement of the distal metatarsal will not dorsiflex or plantarflex the metatarsal head
and should remain parallel to the weight bearing surface.

What are the differences between a Silver and a McBride procedure?


Silver, D: The operative treatment of hallux valgus. JBJS 5: 225, 1923
McBride E: A conservative operation for bunions. JBJS 10:735, 1928
McBride Silver
Incision Lateral border of EHL Medial curved incision
Capsule Dorsal to planter incision Y shaped incision
Sesamoids Fibular sesamoidectomy No sesamoidectomy
Tendon transfer Adductor tendon to lateral Adductor release
st
aspect of 1 met shaft

What is the McBride test and what do the results indicate?


From: McBride E. JBJS 1967.
Patient is seated and toes are relaxed. Move the hallux towards the midline of the body.
Can’t be brought into alignment with a little bit of overcorrection? There is capsular
contraction. Surgically: remove fibular sesamoid, tenotomize the lateral capsule and
transfer the conjoined tendon.
Can be brought into alignment with a bit of overcorrection? Repeat the test standing.
Can’t be brought into alignment? Capsule is not the deforming force, the adductor is.
Surgically may be possible to just tenotomize the adductor tendon.
What are the mini frag, small frag and large frag set and what size screws do they
contain?
Mini frag set: 1.5, 2.0, 2.7
Small frag set: 3.5, 4.0 (partially threaded), 4.0 (fully threaded)
Large frag set: 4.5, 4.5 (malleolar screw), 6.5 (partially threaded), 6.5 (fully threaded)

What is a bunionectomy screw? A Herbert screw? A Reese screw?


Bunionectomy screw: 4.0mm partially threaded cancellous thread diameter with a 2.7
screw head
Herbert screw: a unique screw that has two threaded portions that are separated by a
smooth portion centrally. The pitch difference in the threaded portions is the design
mechanism that draws together two bone fragments and creates compression across an
osteotomy or fracture.
Ford T: The Herbert bone screw and its application in foot surgery. JFAS 33:346, 1994
Herbert TJ et al: Management of the fractured scaphoid using a new screw. JBJS 66B: 55
– 58, 1984)
Reese screw: headless screw with clock-wise threads proximally and counter clock wise
threads distally. Used for 1st IPJ fusions

What material is the surgical screw driver handle made from?


Pressed linen

Name the modifications of the Austin bunionectomy.


Youngswick – Austin
Bicorrectional Austin
Kalish
Vogler Offset V

What is meant by staking the head in a bunion surgery?


Excessive bone removal on the medial 1st MT head which causes the loss of the tibial
sesamoid groove

Which structures attach to the fibular sesamoid?


Lateral head of the FHB
Lateral metatarso-sesamoidal ligament
Lateral phalangeo-sesamoidal ligament
Deep transverse metatarsal ligament
Lateral conjoined tendon of the adductor hallucis
Intersesamoidal ligament

If you are planning to do a bunion procedure on a person with an already short 1st
metatarsal, name as many procedures as you can that would take this into account and
not shorten the metatarsal more.
Bi-correctional Austin Logriscino
Scarf Z Trethowan
Juvara B2 and C2 Opening wedge of the met or the cuneiform
Loison-Balacescu
Describe the anatomy of the MPJ complex

What are some common capsule tendon balance procedures?


Silver (Silver, David: The operative treatment of hallux valgus. JBJS 5:225, 1923)
1. curved medial incision
2. removal of bursal sac
3. resection of dorsomedial eminence
4. lateral capsulotomy with medial capsulorraphy designed to reinforce the medial
collateral ligament and transpose dorsally the abductor hallucis tendon
5. lateral release
McBride (McBride, Earl: A conservative operations for bunions. JBJS 10:735, 1928)
1. 5 cm incision from the toe web along the lateral border of EHL
2. resection of the dorsomedial eminence
3. lateral release consisting of lateral capsulotomy with fibular sesamoid excision
4. transfer of the adductor hallucis and lateral head of FHB to the dorsolateral aspect
of the 1st met head
 Adductor tendon transfer
1. dorsally through the capsule to reposition the sesamoids
2. into the 1st metatarsal neck to reduce the IM angle
 transfer into the neck can be either dorsal to plantar or lateral to medial
Hiss (Hiss, J: Hallux valgus, its cause and simplifies treatment, Am J Surg 11:51, 1931)
1. Resection of the dorsomedial eminence
2. lateral capsulotomy
3. transfer of abductor hallucis from plantar to medial to the 1st MPJ (abductor
sutured into hole in the base of the proximal phalanx; acts as a ligament to
strengthen the capsule medially)

Name three neck procedures.


Peabody, Hohmann, Mitchell

Name five base procedures.


Loison – Balasecu
Juvara
Crescentic
Trethowan
Logriscino

Starting at the rearfoot, explain in as much detail as possible Root’s theory on the
formation of a bunion. Include all of the stages.
(Root et al. Forefoot deformity caused by abnormal subtalar joint pronation. In Normal
and Abnormal function of the foot. Clinical Biomechanics. Volume 2. Los Angeles. 1977
pg 349 – 462)

List four broad categories of problems that can lead to HAV.


NM conditions, arthritidies, trauma, biomechanical problems
What are the biomechanical relationships that can lead to hallux valgus?
Compensated forefoot and rearfoot varus
Pes valgus deformity
Hypermobile 1st ray
Excessive pronation resulting from ankle joint equinus
Torsional deformity or rotational abnormalities of the LE
Any condition that leads to abnormal subtalar joint pronation

According to proper AO principles, explain the two screw placement for a CBWO of the
1st metatarsal.
Proximal screw – anchor screw that is placed perpendicular to the metatarsal
Distal screw – compression screw, placed perpendicular to the shaft of the metatarsal

Why was the Herbert screw originally developed?


Scaphoid fractures of the wrist

How would you fixate an Austin bunionetomy?


Original procedure fixation was not used and metatarsal head was allowed to ‘seek its
own level’
Can use 1 screw, 2 screws, 2 K-wires, 1 screw and 1 K-wire, etc.

Name complications of HAV surgery.


1. Hallux varus
2. Tibial sesamoid dislocation
3. Capsular defect
4. Staking the head – excessive bone removal on the medial 1st MT head, losing the
plantar tibial sesamoid groove
5. Capital fragment on the floor (Christenson C: Intra operative contamination of the
bone and cartilage during an Austin bunionectomy 31:285, 1992)
6. Severing the EHL or the FHL
7. Infection
8. Breaking the hinge
9. Intra-articular osteotomy
10. Recurrence
11. Delayed or non-union of the osteotomy site
12. 1st met shortening
13. 1st met elevatus
14. AVN
15. Nerve entrapment
16. Sesamoiditis
17. Hallux limitus
18. Fixation failure
Hallux Limitus and Rigidus
Definition of Hallux Limitus:
The proximal phalangeal base is plantarly subluxed upon the first metatarsal head. The
hallux has limited dorsiflexion motion at the first MPJ during propulsion.
There is a decrease in 1st MPJ ROM to < 20 degrees.

Definition of Hallux Rigidus:


Ankylosis of the first MPJ secondary to repetitive trauma during dorsiflexion.
< 10 degrees of dorsiflexion

Definition of Functional Hallux Limitus:


Functional hallux limitus exists when the first MPJ DF ROM is reduced when the
forefoot loads. Stage 1.

Definition of Structural Hallux Limitus:


Structural hallux limitus exists when first MPJ ROM is reduced with the forefoot loaded
and unloaded.

Etiology:
Long first met or proximal phalanx Neuromuscular disease
Short first met Gout
MPE Osteomyelitis/septic arthritis
Trauma with DJD Charcot neuroarthropathy
Hypermobile 1st ray Neoplasms
Immobile 1st ray Septic arthritis
Neurogenic inflammation Foreign bodies (implants)

Iatrogenic (i.e. Minimal invasive surgery, AVN, Joint immobilization, Poor


dissection technique, Inadvertent metatarsal lengthening, failure to correct DF 1st
ray previously, over aggressive capsulorraphy, osteotomy displacement, etc)
Systemic arthridities (RA, PA, Gouty arthritis)

Classification of Hallux limitus/rigidus


Primary v Secondary
1. Primary
a. Occurs in adolescent patients and is associated with a long first metatarsal
b. Congenitally long first metatarsal
2. Secondary
a. Occurs in older patients
b. Usually caused by DJD, trauma, septic arthritis and systemic arthridities
Structural v Functional
1. Structural
a. DF of the hallux is decreased when the forefoot is loaded and unloaded
2. Functional
a. DF of the hallux is decreased when the forefoot is loaded only
Pathobiomechanics
- restricted first ray plantarflexion
- first MPJ sagittal plane subluxation
- bony adaptation
- first MPJ DJD

Clinical Findings – Symptoms


1. gradual onset of pain and limitation of 1st MPJ ROM
2. Pain localized to the dorsal aspect of the 1st MPJ
3. Pain with prolonged weightbearing or ambulation
4. Pain with shoe gear
5. Painful plantar IPJ callous
6. Lateral foot pain
7. Subungual hematoma

Clinical Findings – Physical Exam


1. Dorsal or dorsomedial bump
2. Hallux is rectus
3. Tenderness with passive DF of hallux
4. Pain and crepitus first MPJ
5. Decreased painful ROM
6. Excessive IPJ dorsiflexion
a. Associated with EHL spasm
b. Ganglion cyst or adventious bursa associated with EHL tendon secondary
to irritation from shoe gear
c. Plantaremedial IPJ hyperkeratotic lesion
7. Check sesamoid mobility
a. Load the 1st ray and palpate the sesamoids while mobilizing the first MPJ
b. Arthrosed or fibrosed sesamoids prevent normal mobility and MPJ
dorsiflexion
8. Antalgic, apropulsive gait with an early heel off

Radiographic findings
1. Non uniform joint space narrowing
1. Flattening of the first metatarsal head
2. Osteophytes on the met head and the proximal phalanx (i.e. dorsal flag or sailboat
sign)
3. Subchondral sclerosis (eburnation)
4. Loose bodies within the joint space (joint mice)
5. MPE on the lateral view
a. Proximal phalanx articulates with the plantar aspect of the 1st metatarsal
head
b. Bisection of the first met is superior to the talar bisection (Meary’s angle)
6. Positive 1st met protrusion distance
Stages and Classifications
1. Drago, Oloff and Jacobs 2. Regnauld
a. Functional limitus a. Development
b. Joint adaptation b. Established arthrosis
c. Joint deteriortation c. Ankylosis
d. Ankylosis

Non-Surgical Management
 determine whether the deformity is functional or structural hallux limitus.
 Functional responds better to orthotic control.
 Shoe modifications
o Rocker bottom
o Metatarsal bar
 NSAIDs
 Orthoses
o First ray cutout
o Heel life
o Kinetic wedge device
 Periarticular v intra-articular steroid
 ROM exercises
 Manipulation and stretching
 Physical Therapy

Goals of Surgical Treatment


1. restore pain free ROM
2. re-establish joint space
3. remove osteophytic spurring and loose bodies
4. reduce osseous deformities like MPE or a long first metatarsal
5. attempt joint preservation
o Must have 50% of the cartilage to do a joint preservation procedure
6. creation of slack in the flexor apparatus

Indications for joint preservation procedures


1. Need to reduce osseous deformities
2. Need to remove loose bodies
3. Being able to have enough viable cartilage to salvage the articulation (at least
50%)
Joint Preservation Procedures
- Curettage and Drilling
o .028 or .035 k-wires
o Multiple drill holes to get to subchondral bone
o Adjunctive procedure; not curative
o Goal: promote the growth of fibrocartilage
o Keep NWB for but do ROM
- Cheilectomy
o Remove dorsal bone spur on met head or
phalangeal base
o Adjunctive procedure not curative
o Doesn’t give you more joint space or fix the
osseous deformities
- Bonney & Kessel
o Use for a long proximal phalanx
o Indicate for a plantar flexed hallux
o Take a dorsal wedge of bone at proximal phalanx
 close it
o Does not address the MPE or redirect the
cartilage on the 1st met head
- Waterman
o Metatarsal procedure
o Take a dorsal wedge from the head of the met
leaving the plantar hinge intact
o Rotate the met head up  redirecting the
cartilage
o Does not address the MPE
- Waterman – Hohmann
o Do the Watermann procedure but break the plantar
hinge
o Plantar flex the head of the metatarsal and fixate
o Shortens and decompresses the joint
o Re-direct the cartilage
o Addresses mild to moderate elevatus
- Watermann – Green
o Similar to the bicorrectional Austin
o Dorsal cut: trapezoidal wedge that is wider medially
o Plantar cut: oblique, transverse osteotomy
o Angle = 45 degrees
o Shortens and plantarflexes the 1st met head
- Regnauld
o Enclavement procedure
o Base of the proximal phalanx is fashioned into a peg and
then reinserted into the proximal phalanx (looks like a
Mexican hat)
o Decompress the joint
o Good for long prox phalanx or short 1st met
o Shortens the proximal phalanx
- Youngwick modification of the Austin bunionectomy
o Use an Austin bunionectomy
o Use parallel dorsal cuts & remove rectangle
- Sokoloff dorsal V
o Create a V shaped osteotomy driving the drill
from dorsal to plantar
o Performed at the metatarsal neck
o Good for MPE
o Plantarflexes the first met head
o Doesn’t decompress the joint or shorten the
metatarsal
- Sagittal Logriscino
o Waterman
o Lamburnudi
- Lambrinudi
o Proximal procedure
o Take a wedge (either transverse or oblique
cut) from the base of the 1st met
o Gets rid of HUGE elevatus
o Leave the dorsal cortex intact
o Doesn’t redirect the cartilage

Joint Destructive Procedures


- Keller
o Resect 1/3rd of the base of the proximal phalanx
o Problem: toe purchase
o Good for old people or DM with ulceration under the base of the proximal
phalanx; those who aren’t very active
- Valenti
o Take a V out
o One cut in the metatarsal head that is directed proximal dorsal to distal
plantar. This leaves the plantar one third of the cartilage intact.
o One in the proximal phalanx base that is directed proximal plantar to
dorsal distal
o Immediate post-op complications: sesamoiditis
o Doesn’t correct elevatus or reposition cartilage
- Stone
o Dorsal to plantar wedge resection of 1/4th of the metatarsal head
- Mayo
o Excision of 5 mm of the 1st metatarsal head including the articular surface
- McKeever (MPJ funsion)
o Remove remaining cartilage and fixate
o Stress moves to the IPJ
o Optimal positioning:
 15-25 degrees DF; 10-15 degrees Abducted; no frontal plane
deviation
- Lapidus
o See bunion procedures
- Arthrosurface
o Similar to a one sided joint implant
o Looks like a giagantic thumb tak has been inserted into the first met head
- Implant
o Interpositional
 Joint spacers
 Good to use with keller procedures
• Swanson flexible hinge
• Sutter-Lawrence design
• Sutter-LaPorte design
• Hemi design
o Total joint replacement
 Implant properties:
• Inert
• Non-biodegradable
• Durable
• Non-irritating
 Fabricated from:
• Stainless steel
• Silicone rubber
• Cobalt chromium alloy
• UHMW polyethelene
 Semiconstrained
• Only sagittal plane motion
 Nonconstrained
• Motion in more than one plane
• Fabricated from:
o Cobalt chromium
o Titanium alloy
o UHMW polyethylene
Implant complications:
- biomechanical failure
- implant failure
- soft tissue
- bone failure
- alignment failure

ROM exercises are of the utmost importance and are the most important part of the
post op course for HL/HR.
Ankle Fractures
Oblique fracture: produced with combined loading of compression and torque. Usually a
45 degree fracture orientation. This is considered to be unstable and is usually fixed with
ORIF

Spiral fracture: this is always a type of torque fracture. It usually takes a 360 degree
course with a vertical connecting segment between the two opposing sharp spikes of the
fracture. This is also usually orientated at 45 degrees

Comminuted fracture: this suggests the presence of two or more fracture planes and has
at least three fragments. This is usually due to very high velocity loading, it is a high
energy failure with load concentration over a small area of the bone.

Eponyms for Fractures


Percival Pott (1768): Described a fibular fracture 2 – 3 inches proximal to distal aspect
fibula with a deltoid ligament fracture and an intact syndesmosis.
Now: bimalleolar fracture

Dupuytren (1819): Described a medial deltoid ligament tear or a medial malleolar


fracture with a fibular fracture 2.5 inches proximal to most distal aspect of the fibula with
diastasis always being present.

Maisonneuve (1840): The New House fracture. Fracture of the proximal 1/3rd of the
fibula.

LeFort (1886): Described a vertical fracture of the fibula due to SAD type forces.

Chaput – Tillaux (1872): In 1872 Tillaux described a medial deltoid injury or medial
malleolar fracture, a fibular fracture 2 inches proximal to the ankle joint, AND an
anterior lateral tibial tubercle fracture. In 1907, Chaput conducted laboratory studies of
this fracture which is why the fracture of the anterior lateral tibial tubercle contains both
names.

Wagstaffe (1873): Distal fibular fracture due to avulsion of the anterior inferior tibial-
fibular ligament. An SER type injury.

VonVolkman (1875): Originally described as an anterior lateral lip of the fibula fracture
due to avulsion of the anterior tibial-fibular ligament and interosseous membrane due to
PAB type forces. Now: posterior lateral tibial fracture

Cotton (1915): Described a posterior malleolar fracture with a medial malleolar fracture
and a fibular fracture just proximal to the joint level. Trimalleolar fracture.
Correlating Danis-Weber and Lauge-Hansen

Danis—Weber Lauge—Hansen Where to start the Lauge—Hansen


Category LH Clock Type
SAD Start lateral 1 – lateral ankle
ligament injury – or
– transverse fibular
A – fracture of fracture distal to the
fibula distal to the level of the ankle
level of the ankle joint
joint 2 – near vertical
fracture of the
medial malleolus
SER Start anterior and 1 – rupture of the
central anterior inferior
tibial-fibular
ligament
B – fracture of the 2 – fibular fracture
fibula originating at with posterior
the level of the spike
ankle joint 3 – rupture of the
posterior tibial –
fibular ligament
ALWAYS ask for a 4 – fracture of the
lateral film!!!!! medial malleolus
PAB Start medial 1 – deltoid ligament
rupture or medial
malleolar fracture
2 – anterior tib-fib
ligament rupture
with posterior
tibial fracture
3 – Danis Weber B
fracture
PER Start medial 1 – medial malleolar
fracture or a deltoid
ligament rupture
2 – anterior inferior
C – Fibular fracture tib-fib ligament
originating proximal rupture
to the ankle joint 3 – high fibular
mortise fracture
4 – posterior tib-fib
ligament rupture or
posterior malleolar
fracture
Ankle Fracture Questions
1. What are the radiographic hallmark of each Lauge Hansen Injury?
PER – high fibular fracture
SER – spiral oblique fracture of the distal fibula
PAB – short oblique fracture of the distal fibula
SAD – short oblique fracture (near vertical) of the medial malleolus

2. What is the big difference between a SER type fracture and a PAB type fracture?
SER has a posterior fibular spike while a PAB has a posterior tibial fracture

3. What fragment do you reduce first during ORIF of an ankle fracture?


The fibular fracture

4. What do the two words of the Lague-Hansen classification system signify?


The first word is the position of the foot at the time of injury.
The second word is the direction of the deforming force or the direction the talus moves
in the mortise.

5.What is the Danis-Weber classification system based on?


The anatomic position of the fibular fracture in relationship to the tib-fib syndesmosis

6. You are called to see a patient in the ER. The ER nurse tells you in a stressed out near
hysterical voice that this patient has a transverse fracture of the fibula and a near
vertical fracture of the medial malleolus. What type of fracture is she describing?
SER 2 or Danis Weber A

7. At what stage of the SER injury, would you see a Wagstaffe fracture?
SER 1

8.When does a posterior malleolar fracture need to be fixated?


If the fracture involves 25 – 30% of the articular surface

9. Apply Vassal’s principle as it applies to the ankle joint.


When you reduce and fixate the fibular fracture to the appropriate length, the talus should
fall back into the mortise. This doesn’t always work perfectly for the medial malleolus.

10. Who wrote the original article relating the displacement of the talus in the mortise
and the affect of this displacement on the ankle joint congruity? What did this article
conclude?
Ramsey and Hamilton. JBJS. 1976.
This article concluded that 1 mm of lateral displacement of the talus leads to a 42% reduction in
contact area of the ankle joint. Meaning: reduce your freaking fibular fracture.However, when
doing their cadaveric experiments, they removed all soft tissue from around the ankle joint and
did not allow the talus to compensate for being displaced. More recent studies have indicated that
the talus tends to move back into the mortise when compressed if allowed to do so.
11. What are the radiographic criteria for adequate reduction of displaced ankle fractures?
- no widening of the medial clear space (<4mm) and symmetric joint space on mortise view
- no displacement of malleoli on AP views
- less than 2 mm of posterior displacement of lateral malleolus on lateral films
- no angulations
- fracture of less than 25 – 30% of the posterior malleolus

12. The key points for fixation are…


- fibular fracture is the most important
- restoration of fibular length takes precedent over repair of the inferior tibial-fibular syndesmosis
- realign the ankle mortise
- evaluation the talar done and tibial plafond
- reapproximation of soft tissue supporting structures

13. Put the following in order according to priority


- blood flow
- reduction of marked dislocation or deformity
- care of open wounds or other soft tissue injuries
- precise anatomic reduction of bony structures
- repair of damaged tendons and nerves
- rehabilitation
- prompt identification and treatment of any complications that may develop

14. What is the Cotton test and what does it tell you?
- a way to evaluate the syndesmosis for rupture or injury
- after fibular fixation, use a large bone hook to try to laterally distract the fibula from the tibia
while observing the relationship of the two bones
- if > 3 – 4 mm of lateral sift of the talus occurs, significant instability is present and a
syndesmotic screw is recommended

15. When should you use a syndesmotic screw?


- when there is a high fibular fracture or a positive Cotton test
- more recent studies, however, suggest that if you have a fibular fracture 5.0 cm from the ankle
joint and distal do not need to be fixated with a syndesmotic screw
(Kennedy et al. J Orthop Trauma 14(5), 2000)

16. How do you insert a syndesmotic screw?


- from the lateral fibula angulate the screw about 25% anteriorly and penetrate three cortices with
the foot at 90 degrees to the leg.
- Olerud (Arch Ortho Trauma Surg 104:299, 1985) demonstrated a 0.1 degree loss of dorsiflexion
for every degree of plantarflexion that the ankle was in at the time of fixation of the syndesmosis

17. When is a syndesmotic screw removed?


- No one agrees on this
- Usually at 6 – 8 weeks post op or immediately before weight bearing
- It can be left in for longer (12 – 14 weeks) for increased stability in syndesmotic ruptures
- Or you can use a tightrope for syndesmosis fixation and you wouldn’t have to remove that at all

18. Which two classes of Lauge Hansen fractures will most commonly produce posterior
malleolar fractures?
SER and PER
19. What is the best way to assess for a posterior malleolar fracture?
CT or a lateral film (x-ray)

20. When should a posterior malleolar fracture be fixated?


> 25 – 30% of the joint surface is involved when ORIF is indicated

21. What are the names of two approaches to fixate a posterior malleolar fracture?
- Anterior approach through the same incision used to fix the medial mall fractures
- Posterolateral approach of Henry (1945). Incision between the peroneal and Achilles tendons;
avoids damage to the sural nerve and avoids the NV bundle medially. The FHL muscle is used as
a guide to the fragment

22. What is the posterior malleolus?


A tubercle at the posterior border of the fibular notch on the lateral surface of the distal tibia

23. What is a triplane fracture and why does it occur?


- Pediatric fracture that appears as a SH2 on the lateral (with the Thurston Holland sign) and a
SH3 on the AP
- Occurs in kids around 14 yrs of age where the medial aspect of the growth plate is closed and
the lateral side remains open
- Vertical fx of the epiphysis from the joint space to the physis that is orientated in the sagittal
plane
- This fracture changes directions when it reaches the physis and orientates itself in the transverse
plane
- It then changes direction again and continues into the metaphysic in the coronal plane and exits
the bone posteriorly.

24. What should you do with a pediatric fracture?


Order a CT to r/o a triplane fracture and to determine growth plate involvement

25. With which Saltar-Harris fracture pattern in a Juvenile Tillaux fracture associated?
SH 3

26. How does a Danis-Weber B fracture displace?


Posterior and superior

27. What is an antiglide plate?


- This plate was developed because of the difficulty encountered with accurately reducing and
securing the Danis-Weber B fibular fractures by traditional methods
- This plate is put on the posterior side of the fibula
- A 5 hole 1/3rd tubular plate is used with three holes above and two holes below the fracture
- An interfragmentary screw can be used through one of the holes

28. Name some disadvantages of using a lateral fibular plate.


- Frequent plate and screw irritation due to the superficial nature of this bone
- Closure problems? Also due to superifical nature of the bone
- Hard to fit the shape of the fibula due to the torque required
- Possibility for penetration of the distal screws into the talofibular and tibiofibular articulations
29. What are the four principles of pilon fracture reduction?
- reconstruction of the fibular fracture
- reconstruction of the tibial articular surface
- cancellous graft to fill the distal tibial metaphyseal defect
- buttress plate application to the medial or anterior aspect of the tibia

30. Classifications for Pilon fractures

a. Ruedi and Allgower


Type I: Mild displacement and no comminution without major disruption of the ankle
joint
Type II: Moderate displacement and no comminution with significant dislocation of the
ankle joint
Type III: “Explosion fracture”; severe comminution and displacement of the distal tibial
metaphysis; significant displacement and loss of cancellous bone

b. Lauge Hansen, PDF


Stage I – medial malleolar fracture
Stage II – fracture of the anterior lip of the tibial plafond
Stage III – fibular fx above the level of the syndesmosis
Stage IV – transverse fracture of the distal part of the tibia at the same level as the
proximal margin of the large tibial fracture

c. Mueller (AO system)


Type A – Extra articular
Type B – Partially articular
Type C – Completely articular
All of the above can include:
A: no comminution or impaction in the articular or metaphyseal surface
B: impaction involving the supra-articular metaphysis
C: comminution and impaction involving the articular surface with metaphyseal
impaction
31. A pt presents to the ER and you are the resident on call. X-rays show a short oblique fracture
of the fibula at the level of the syndesmosis. What else do you want to know about the x-rays and
what do you tell your attending on the phone?
X-rays:
Fibular displacement
Relationship of the talus to the tibia
Medial clear space
Posterior malleolus and syndesmosis involvement
Tell your attending that you have bi mallelous fracture that needs ORIF

32. Radiographic evaluation of ankle fractures should include:


- medial clear space
< 4 mm on the mortise views with relatively symmetric joint space
- talocrural angle
83 + 4 degrees - or – within 2 degrees of the contralateral side
- talar tilt angle
- Shenton’s line
The continuous curve between the lateral talus and the recessed tip of the distal fibula
- Syndesmotic width
Less than 5 mm on the AP view
33. You have a 70 y.o. active female patient who sustains a Danis-Weber B type fracture that
needs ORIF. Pt has a smoking history. What are you worried about?
Quality of her bone – will it hold fixation?
Healing potential

34. During her surgery, you attempt to apply a lateral plate and screws to fixate the fracture.
After several attempts, it is deemed that the fixation will not help in her soft bone. What do you
do?
Insert an IM rod (Rush rod) from anterolateral at the distal end of the fibula and proceed
proximally
IM fixation is great in elderly patients; they can weight bear soon with this type of fixation.
Pritchett (Ortho Review June 1993) showed 88% of pts treated with Rush rods had a good or fair
functional result compared with 76% treated with the AO Method. FWB was possible 6 weeks
earlier with Rush rods than with plates and screws. Study was done on pts over 65 y/o.

35. Muller classification of medial malleolar ankle fractures.


Type A: avulsion of the tip
Type B: avulsion at the level of the ankle joint
Type C: oblique fracture
Type D: vertical fracture

36. What are three ways to fixate the medial malleolus?


a. single or double screw fixation
b. K-wires
c. Tension band wiring technique
Johnson and Fallat JFAS, 1997, showed that cancellous screws (two) exhibited only
47.16% of the strength of tension band wiring at clinical failure.

37. What are some complications of ankle fractures?


Delayed or non union NV injury
Post traumatic arthritis Infection (particularly if open fracture)
RSD

38. Using Lauge Hansen what is the most common ankle injury?
SAD

39. Using Lauge Hansen, what is the most common ankle fracture?
SER
Pilon Fractures
(As presented by Roy Sanders, MD, 9/21/2000 at Loyola University)
Treatment:
1. Plate the fibula
2. External fixation; wait 10 – 21 days (edema resolution)
3. Fixate the tibia
a. Metaphyseal plate (M-plate; Sanders & Bone)
b. Spider plate
c. Spring plate; spoon plate (6.5 cancellous screws)
d. 1/3 tubular plate hammered flat
Anatomic reduction: a congruous joint is the goal; the only way to achieve a congruous joint is to
ORIF the injury
Recommended text: Planning and Reduction Techniques in Traumatic Fractures
Calcaneal Fractures

How often do intra-articular fractures occur?


Approximately 75% are intra-articular

What three factors determine the pattern of communition and location of the fracture lines?
1. position of the foot at impact
2. force at impact
3. bone quality

Describe the Rowe classification.


Ia: plantar tuberosity fracture
Ib: sustentaculum tali fracture
Ic: anterior process fracture

IIa: fracture of the posterior aspect of the calcaneus not involving the Achilles tendon; ‘beak
fracture’
IIb: avulsion fracture of the posterior aspect of the calcaneus

III: fracture of the body without STJ involvement

IV: fracture of the body with STJ involvement

V: comminution of the body of the calcaneus

Describe the Essex-Lopresti classification?


- intra-articular fracture classification only
- Tongue type fracture: primary fracture line runs superior to inferior and secondary
fracture line exits from the posterior aspect of the calcaneus
- Joint depression: primary fracture line runs superior to inferior with a second fracture
line surrounding the STJ (posterior facet)

On which imaging modality is the Sanders classification based and what slice is used?
- CT imaging
- Based on the widest section of the sustentaculum tali in 3 mm coronal slices

Describe Sanders classification.


- I: all non displaced articular fractures irrespective of the fracture lines
- II: two part fracture of the posterior facet
- III: three part fracture of the posterior facet with central joint depression
- IV: four part articular fracture; often more pieces and highly comminuted

How many stages are in the Sander’s classification (including subtypes)?


Eight (I, IIA, IIB, IIC, IIIAB, IIIAC, IIIAB, IIIAC, IV)

In the Sander’s line classification, is line A medial or lateral?


Lateral

The ecchymosis seen in calcaneal fractures is known as what?


Mondour’s sign; this usually occurs plantarly but can also occur distal to both malleoli
Where are fracture blisters most commonly located in calcaneal fractures?
The medial side because during the fracture there is predominantly shearing and stretching of the
soft tissues on the medial side of the foot.

What two important angles are associated with calcaneal fractures?


1. Bohler’s angle: normally 20 – 40 degrees; decreases with depression of the posterior STJ
2. Gissane’s angle: normally 120 – 140 degrees; will increase with the depression of the
joint

What plain film views would you order and what would you see on each?
1. Lateral foot: see joint depression; evaluate the two angles in the question above; check
for loss of height of the posterior STJ
2. AP foot: to evaluate all other foot bones for additional fractures/pathology
3. Harris-Beath and/or Broden view: to evaluate the posterior facet of the STJ
4. Lateral oblique: anterior process of the calcaneus to check for CC joint involvement

What is a Broden’s view and how is it taken? (Broden projection I)


- a way to evaluate the posterior STJ on plain films
- pt is supine with cassette under the foot; leg is internally rotated 30 – 40 degrees
- x-ray beam is centered over the malleoli and four consecutive projections are made with
the tube angled at 40, 30, 20 and 10 degrees toward the head of the patient

What are the fragments usually seen in calcaneal fractures?


1. * Superomedial fragment (Constant or sustentacular fragment)
2. * Posterior facet fragment (superolateral, semilunar or comet fragment)
3. * Tuberosity fragment (main fragment)
4. Anterior process fragment
5. Anterior STJ fragment
* The three important fragments that must be reduced

What are the goals of ORIF with calcaneal fractures?


1. Restoration of length, width and height of the calcaneus
2. Anatomic reduction of all involved joint surfaces
3. Restitution of function by stable osteosynthesis without joint transfixition
In July 2000 in the Journal of Orthopedic Trauma there was a report of using injectable bone
cement for augmentation of ORIF of calcaneal fractures. The authors report using an
injectable cement in the area of the neutral triangle under the posterior facet. This will allow
for good resistance from compression that ORIF alone can’t give. At the end of their study,
they were having patients fully weight bear at 3 weeks post op. This is about 10 weeks earlier
than some authors report. This could potentially be a great tool for augmentation or internal
fixation in these fractures.

What are the four ways to treat calcaneal fractures?


1. non-operative
2. ORIF
3. Ex-fix
4. Primary STJ arthrodesis

What else should be evaluated when dealing with calcaneal fractures?


1. proximal injuries (lower back, spine, neck and head)
2. bladder rupture
Which vertebrae are most commonly injured?
1. L1, L2
2. Wong (1966) found that 11% of males with calcaneal fractures also had vertebral
compression fractures

What is the ‘wrinkle test’?


- A way to evaluate if the soft tissue swelling has reduced enough for surgical intervention
- Dorsiflex and evert the foot and the skin on the lateral side of the foot will wrinkle

What are the locations for the incisions of ORIF and the advantages of each?
1. Lateral extensile, Modified Ollier
a. Easy visualization of posterior facet and calcaneocuboid joint
b. Avoids neurovascular bundle
2. Medial
a. Initially popularized by McReynolds
b. Easy reduction of the sustentacular fragment
3. Seligson’s lateral extensile
a. Described by Giouild (F&A, 1984)
Some authors use both approaches so each fragment can be adequately visualized
Many authors also use the lateral approach for which there are many variations.

When should primary arthrodesis be done?


In a highly comminuted intra articular fracture (i.e. Sander’s type IV)

Name ten complications of treatment of calcaneal fractures.


1. Nerve damage
2. Post traumatic arthritis
3. RSD
4. Compartment syndrome
5. Nerve entrapment
6. Wound dehissance (with or without calcaneal osteomyelitis)
7. Malposition after fixation
8. Calcaneal malunion
a. Classified by Stephens and Sanders
b. Type I: large lateral exostosis with or without extremely lateral arthrosis of the
STJ
c. Type II: a lateral exostosis combined with major arthrosis across the width of the
STJ
d. Type III: a lateral exostosis, severe arthrosis of the STJ and malunion of the
calcaneal body with the hindfoot in varus or valgus angulation
9. Peroneal tendonitis/subluxation
10. Heel pad pain
a. Damage to the fatty plantar heel pad

What is the most frequent post-op complication with ORIF of calcaneal fractures?
Wound dehissance (cited numerous places in the literature)

Note: There seems to be a large discrepancy in outcomes following treatment of intra-articular calcaneal fractures. This
is seen between those injuries suffered while at work and those that are not. Since this injury frequently occurs in the
working population, it is difficult not to include these subjects in studies. Recently, there have been reports alluding to
this idea so hopefully in the future we will see studies on injuries that are not sustained at work.
Flexible Flatfoot Procedures
Transverse plane dominant deformities:
- Evans (bone insertion, lateral calcaneus)
- CCJ distraction arthrodesis (similar to Evans but you fuse the CC joint)
- Kidner (resection of the hypertrophic navicular and accessory bones and reattachment of
the posterior tibial tendon to the navicular)

Sagittal plane dominant deformities


- Lowman (TNJ)
- Hoke (NCJ)
- Miller (NCJ, 1st MCJ)
- Cotton (open PF wedge of the 1st cuneiform)
- Lapidus (1st MCJ)
- Young (Tib ant in keyhole of the navicular)
- Cobb (hemisection of the tib ant to the PTT)
- TAL

Frontal plane deformities


- Orthotics
- Chambers (opening wedge inferior to posterior facet)
- Baker (osteotomy inferior to STJ with graft)
- Selakovich (opening wedge osteotomy of STJ)
- Arthroeresis (MBA, Sta-peg, CSI)
- Coleich (posterior oblique through and through osteotomy)
- Lord (displaces the calcaneus anterior, inferior and medial)
- Silver (lateral opening wedge)
- Dwyer (medial closing wedge)
- Koutsogiannis (posterior transpositional)

Rigid Flatfoot
3. Tarsal coalition (see next page)
4. Congential Vertical Talus (treat with serial casting)
Pedal Coalitions
Terminology
Coalition – the union of things separate into single body or group
True coalition – intra-articular fusion of two bones
Bar or Bridge – extra articular fusion of two bones

Etiology
1) Accessory Ossicles
- Os Sustantaculi Proprium
o Can be incorporated into the middle facet
- Os Trigonum
o Can be incorporated into the posterior facet
2) Failure Of Segmentation Of Primitive Mesenchyme
- Autosomal Trait
- Variable Penetrance

Congenital vs. Acquired


CONGENITAL - Mesenchymal Differentiation.

ACQUIRED -
- Trauma
- DJD can cause erosion of articular surface
- Inflammatory rheumatoid conditions
- Fractures
- Fusion of accessory bones

Classification by Development
Complete Coalition - two bones joined by osseous union that limits all motion.
Incomplete Coalition - two osseous projections united by intervening fibrous or cartilaginous
tissue.
Rudimentary Coalition - osseous projections from one or more involved bones which limits
motion, without any tissue union between those segments.

Histological Classification
- SYNCHONDROSIS - Cartilaginous
- SYNOSTOSIS - Osseous
- SYNDESMOSIS – Fibrous

Anatomical - Defined by osseous segments involved; best way to assess coalitions.

Forefoot Coalitions Subtalar Coalitions


• Interphalangeal - Anterior facet
• Intermetatarsal - Middle facet
• Metatarsal – cuneiform - Posterior facet
• Navicular – cuneiform - Posterior process
- Sustentaculum tali
Midtarsal / Tarsal Coalitions
• Calcaneo-navicular – common Navicular – cuboid - underreported
• Talonavicular – more common Cuneiform – cuboid – fairly rare
• Calcaneo-cuboid – relatively common Talo – cuboid – very rare
Incidence
• Males > Females
• May be b/c males have traditionally been
more active than women
• Race: Equal
• Bilateral in 50% of patients
• Overall < 1% of total population
*Jahss and Tachdjian
• Fusion of middle/distal phalanx 5th digit most common pedal coalition
(Approximately 50% of general population)
• Tarsal coalition -
- talo-calcaneal
- calcaneal navicular
- talonavicular
• 1-2% of the general population

Ossification of Coalition
• Talonavicular 3-5 years old
• Calcaneal navicular 8- 12 years old
• Talocalcaneal 12-16 years old
- Age of ossification of coalition corresponds to the onset of symptoms

Clinical Exam
•Peroneal spasm – most common sign of •Pain
involvement •Local tenderness to palpation
•Anterior/posterior tibial spasm •Stiffness/decreased ROM
•Pronation
•Fixed rearfoot valgus
•Very vague complaints of pain
•Cavus deformity
•Either cavus or planus feet
Peroneal “Spastic Flatfoot”
• Restricted Subtalar Motion • Forefoot Abduction
• Rearfoot Valgus – occurs b/c they • “Taught Peroneal Tendons”
are trying to get the FF to the • Ext. Digitorum Brevis Spasm
ground (severe cases

Differential Diagnosis
•Rheumatoid arthritis •CCPV
•Trauma – needs to be R/O especially in •Peroneal muscle contracture
kids • 2 TO Localized ISCHEMIA which
•Acromegaly can be caused by decreased padding
•Extra articular arthrodesis in BK case
•Overcorrected clubfoot •Neuromuscular disease
•Osteochondral defect
•Infection
•Neoplasm
•Juvenile RA – very rare
Coalition Incidence View Associated Signs Trivia/Other
Interphalangeal Common AP May be
synphalangism

Intermetatarsal Not very AP Diaphyseal fusions may


common Oblique be result of trauma

Metatarsal - cuneiform AP 3rd metatarsal cuneiform


Medial oblique (for coalition
medial column)
Lateral oblique (for
lateral column)

Navicular – cuneiform AP Coalition may


Oblique continue through the
intercuneiform joints
as well

Navicular – cuboid Under AP


reported Lateral oblique
Calcaneocuboid Relatively Lateral Can be
common Lateral oblique bilateral in
nature but
can have
unilateral
symptoms
Talo-navicular More AP Putter Sign On lateral, be sure to count your
common Medial oblique medial column bones

Calcaneonavicular Common Lateral oblique Ant-eater Sign Long snouted anterior process of
Lateral calcaneus
Ratty radiolucency and irregular
sclerosis  fibrous union

S – shaped appearance of the


calcaneus an navicular on lateral
oblique

Anterior Facet Lateral oblique 1. talar beaking


Isherwood lateral 2. narrowing of the posterior TC joint
oblique dorso-plantar space
3. rounding of the lateral talar process
4. middle STJ may not be visable
5. Halo effect
Middle Facet Most Harris and Beath 1. talar beaking
common of 2. narrowing of the posterior TC joint
the talo- space
calcaneals 3. rounding of the lateral talar process
4. middle STJ may not be visable
5. Halo effect
6. Posterior and middle facets should
be parallel
7. Middle facet is angulated in an
inferior medial direction in a coalition
Posterior Facet Harris and beath 1. talar beaking
Broden medial oblique 2. narrowing of the posterior TC joint
space
3. rounding of the lateral talar process
4. middle STJ may not be visable
5. Halo effect

Posterior Process Fusion Lateral

Flare
Lack of differentiation of lateral
process
Os trigonum? (has a role)
Specialized Plain Film Studies
• Harris & Beath:
• identifies posterior/middle facets of STJ
• Isherwood:
• oblique lateral-anterior facet STJ
• Broden:
• medial oblique posterior STJ
• Lateral oblique:
• anterior facet
• X-ray positioning: Podiatry Today, Sept. 1989 (Baron & Strugielski)

Computed Tomography
• Modality of choice for the assessment of
coalitions
• Planar analysis
• Extent of coalition
• Assess subtle cortical changes in
surrounding osseous structures
• Picture: bilateral coalitions
• One side fibrous
• One side bony

Nuclear Imaging
Technetium bones scans will assist in identification of “maturing coalitions”
- see uptake on adjacent aspects of the involved bones
- lack of uptake in region of “mature” coalition
MR Imaging
• Planar analysis
• Identification of tissue matrix (may be better than a CT)
• Loss of fluid signal intensity in area of coalition on T-2 weighted images

Associated Syndromes
- Isolated fusions usually occur between bones of the same row (distal to proximal),
those associated with other abnormalities occur across rows (medial to lateral).
• Distal to proximal: isolated fusions
• Medial to lateral: associated with other abnormalities
- Cuneiform involvement points to possible associated syndrome.
• Apert’s syndrome
• Nievergelt-Pearlman syndrome
Apert’s Syndrome
• Massive tarsal synostosis –
• one big block of bones
• Craniosynostosis –
• born with normal mentation,
• premature closure of the cranial bones  mental
deficiency.
• Midfacial hypoplasia –
• mouse facies,
• think mickey mouse  bones close early 
decrease in size
• Broad distal phalanx of thumb and hallux
• Adaptive changes seen at the ankle joint
• Sagittal plane disorganization
• Same pt in the picture as in the radiographs
• Use palliative care
• Surgical intervention: less surgery = better

Nievergelt – Pearlman Syndrome


• Synphalangism
• Carpal fusion
• Atypical clubfoot with massive tarsal fusions
• Congenital dislocation of radial head
• Conduction deafness
• Three inner ear bones are fused  true conduction
deafness
• Ball and socket ankle joints
• Gives the frontal plane motion at the ankle joint
that the pt would have previously had at the STJ
• Simple inversion may lead to dislocation
• Lack normal flexor creases

Treatment:
- Immobilization
- Supportive therapy
- Orthotic therapy (pronated devices?)
- NSAIDs
- Surgical Intervention (resection v fusion)
Cavus Deformity

Three guidelines should be followed:


1. All fixed deformities must be reduced
2. Proper muscle and tendon balance should be restored
3. Limit the likelihood of the recurrence of the deformity

Bilateral Cavocarus Deformity:


1. Spinal cord tumor
2. CMT
3. Spina bifida
4. Diastematomyelia
5. CNS trauma
6. Poliomyelitis (uncommon)
7. Friedreich’s ataxia (uncommon)

Unilateral Cavovarus Deformity:


1. spinal cord injury
2. poliomyelitis
3. deep posterior compartment syndrome
4. crush syndrome of the foot

Classification Systems
The classification systems for pes cavus are many, and often times confusing.

 Japas classified the deformity as:


 Anterior (local v. global),
 Anterior pes cavus is an increased plantarflexed attitude of the forefoot and is divided
into four types, based upon the apex of the deformity.
* Metatarsus cavus: at LisFranc’s
LisFrancs palpable prominence
* Lesser tarsus cavus: the entire lesser tarsal region
Prominence in the lesser tarsal area
* Forefoot cavus: at Chopart’s, with a distant prominence at the dorsal/lateral
talar head
* Combined anterior cavus: at 2 or more levels
 Posterior
 Combined

Ruch Classification
Stage I: Deformity primarily restricted to the metatarsals, MPJ’s, or digits. Surgical
management includes digital fusions, MPJ release, extensor tendon tenotomies, or flexor
transfers.

Stage II: Deformity primarily consists of a rigid PF first ray and RF varus. Surgical
management included a DFWO, Dwyer, STATT, or Peroneal Stop.

Stage III: A severe global RF and FF deformity of probable N/M etiology. Surgical
management includes midtarsal osteotomies, triple arthrodesis, and tendon transfers.
 Other systems take into account combined sagittal / transverse plane or sagittal / frontal
plane deformities, making the cavus foot even more perplexing.
 Biggest question: IS IT NEUROMUSCULAR DZ?????

What are the goals of tendon transfers in patients with drop foot?
1. Pt should be able to clear the foot during gait
2. Muscle transferred should be in phase with paralytic muscle
3. Transferred tendon should be in line with the muscle and should not undergo angulation

Bridle Procedure: done for anterolateral paralysis resulting in foot drop. The PL tendon and the
TA tendon are anchored to the PTT which is transferred to the anterior side of the ankle.

After injury to the common peroneal nerve, equinovarus develops because of the unopposed pull
to the PT tendon:
- Transfer of the PT tendon will restore DF to the foot and remove some of the deforming
force of the tendon
- Transfer is indicated to correct a foot drop with equinovarus deformity (transfer of the PT
tendon through the interosseous membrane and insert it into the 3rd cuneiform)

In paralysis of all lower leg muscles patients usually require some type of brace
- It is possible to use the flaccid tendons to create a tenodesis for the foot and to minimize
brace use (used for patients who want to be brace free)
- Tenodesis of the extensor tendons and anchor them to the tibia; this will keep them in 10
degrees of dorsiflexion

Paralysis of the tibial nerve usually results in a calcaneus deformity when the anterior leg muscles
are preserved.
- Over time with paralysis of the Achilles tendon and functioning dorsiflexors, the
calcaneus will assume a vertical position as the intrinsic muscles of the foot continue to
contract leading to a cavus deformity
- There will be a marked increase in the pitch of the calcaneus, thickening of the plantar
heel pad, dorsiflexion of the midfoot and slight flexion of the forefoot as a result of
contraction of the plantar fascia.
- Tendon transfer is difficult because no tendon has the strength to oppose the calf
musculature
- Arthrodesis (triple, tibiotalocalcaneal or pantalar) is usually performed
Surgical Options – Soft Tissue Procedures

1. Plantar fascia release


a. Usually done with other bony procedures
b. Steindler striping
c. May need to release the long plantar ligament
2. Jones Tenosuspension
a. Used for cock-up deformity of the hallux
b. Usually seen in TA weakness

c. EHL tendon is rerouted through a drill hole


in the head of the 1st met
d. IPJ fusion of the hallux is also done
e. This procedure will DF the 1st met
3. Extensor shift procedure
a. This is a modification of the Heyman
procedure
b. Done with Jones procedure
c. EDL tendons are transected distally and
tendons 1 & 2 are re-routed through a
transverse drill hole in the shaft of the 3rd
met; tendons 3 & 4 are re-routed through a
transverse drill hole in the shaft of the 5th met
4. Hibbs tenosuspension
a. Transfer of the EDL through a drill hole in the 3rd cuneiform

5. Girdlestone-Taylor transfer
a. Transfer of the FDL tendons to the dorsal aspect of the digits
b. Done for treatment of claw toes
c. Usually done in conjunction with an arthroplasty
6. Heyman Procedure
a. Transfer of all five long extensors to the respective metatarsal heads
7. Split Tibialis Anterior Tendon Transfer (STATT)
a. Indicated for flexible cavus deformity
b. The lateral half of the TA tendon is sectioned and
anastamosed to the peroneus tertius near its
insertion
8. Peroneal Anastamosis
a. Transfer of PL to PB
b. May be done in CMT when the PB function is diminished but the PL function is
normal
9. Peroneus longus tendon transfer
a. PL tendon is sectioned near the cuboid and the tendon is then re-routed down the
EDL tendon sheath and inserted into the lesser tarsal area
b. If split, the medial half is anastamosed to the TA tendon and lateral half is
anastamosed to the peroneus tertius tendon near their insertions

10. Tibialis posterior tendon transfer


a. TP tendon is sectioned at its insertion
into the navicular
b. The tendon is threaded through the
interosseous membrane and down
the EDL tendon sheath
c. The tendon is then inserted into the
lesser tarsal area

11. PL and TP tendon transfer to the calcaneus


a. Both tendons are resected (PL near the cuboid and TP near insertion into
navicular) and rerouted to reinforce the Achilled tendon or be inserted into a hole
in the calcaneus
b. Indicated for a weak Achilles tendon
Osseous Procedures
1. Metatarsal osteotomies
a. Dorsally based wedge in the proximal metatarsal
b. Usually just done to the 1st met
c. Proximal cut is perpendicular to the long axis of the foot
d. Distal cut is perpendicular to the long axis of the first met
e. Proximal cut should be 1 cm distal to the metatarsocuneiform joint
f. Usually done in conjuction with a plantar fascia release

2. Cole osteotomy
a. Dorsally based closing wedge
b. Proximal cut is made through the navicular and the
cuboid
c. Distal cut is made through the cuneiforms and the
cuboid

3. Japas osteotomy
a. V shaped osteotomy with the apex at the middle cuneiform and
the medial border just proximal to the 1st MC joint and lateral
border proximal to the cuboid 5th metatarsal joint
b. The distal section is raised and the proximal portion is lowered

4. Jahss midfoot osteotomy


a. Truncated tarsometatarsal wedge arthrodesis
b. Dorsally based wedge osteotomy at the tarsometatarsal joints with arthrodesis

5. McElvenny-Caldwell Procedure
a. Arthrodesis of the 1st MT – CN joint
b. Procedure done after a serious of plaster casting with the heel in valgus and
forefoot in supination

6. Dwyer calcaneal osteotomy


a. Lateral based closing wedge
b. Osteotomy is made 1 cm proximal to the posterior facet
c. Fixation: canulated screws, staples

7. Dorsiflexory calcaneal osteotomy (Samilson)


a. A cresentic, biplanar osteotomy of the posterior aspect of the calcaneus
b. Section is rotated dorsally and out of varus
c. May cause anterior impingement of the talus on the tibia and exacerbate anterior
cavus
8. Triple arthrodesis
a. Done for rigid deformities
b. Siffert triple works well
i. Moritising the navicular into the head of the
talus
ii. Depressing the navicular, cuboid and
cuneiform while allowing the elevation of the
metatarsal
iii. Laterally closing wedge at the subtalar joint
Neurological Disorders/Pes Cavus
Hereditary Peripheral Neuropathies

Charcot Marie Tooth Disease


- Inherited familial neuromuscular disease that affects peripheral nerves, causing slowing
of sensory and motor nerve conduction
- Progressive in nature
- HMSN I: Classic CMT; usually beings in the 2nd decade of life
- HMSN II: Clinical picture is similar although less pronounced. Manifests later in life.
Usually causes bilateral symmetric neurogenic weakness and atrophy of the distal
musculature of the lower extremity
- Genetics:
• Autosomal dominant (most common)
• X-linked recessive
• Autosomal recessive (least common)
- Diagnosis:
- Clinical
• Claw toes:
• Weak intrinsics and to a lesser degree EDL with normal long flexors lead to
MPJ extension and IPJ flexion
• EDL and EHL are recruited to DF the AJ which pull against the weaker
intrinsics
• Cavus:
• Strong PL overpowers the weak TA causing plantarflexion of the first ray and a
FF cavus
• Strong TP overpowers the weak PB causing an adducted forefoot
• There is then shortening and fibrosis of the short plantar muscles and
contraction of the plantar fascia
• RF varus:
• Overpowering of the TP and the long flexors
• May be a secondary deformity
• May become rigid over time
• Ankle instability:
• Weak PB contributes
• Varus heel puts ankle at risk
• Foot drop:
• Thin lower legs (chicken legs)
• Stork leg deformity, inverted Champaign bottle appearance
• Sensation loss may occur: vibratory and proprioception diminished
• Pattern of muscular weakness:
• Anterior group; peroneus brevis
• Posterior muscles, PL retain normal strength
• Marionette gait
• Multiple lateral ankle sprains
• Pes planovalgus is possible in a flexible deformity
- Family history (genetic testing is now available)
- Slow NCV
Treatment:
- Non operative
• Night splints
• AFO
• Orthotics
• Extra depth shoes
- Surgical
• Multiple options depending on the deformity
• Soft Tissue procedures:
• Plantar fascia release
• Peroneus longus to brevis transfer
• TP transfer through the interosseous membrane
• TAL
• Jones procedure
• Long extensory transfers (to cuboid, base of the 4th met)
• Osseous Procedures
• Metatarsal dorsal closing wedge osteotomies
• Cole midfoot osteotomy
• Triple arthrodesis: results seem to deteriorate with time
• Dwyer calcaneal osteotomy

Note: If an arthrodesis is performed, a tendon transfer to balance out the deformity should also be
done. This is a progressive deformity and the fusion will deteriorate faster if a tendon transfer
isn’t performed.

Dejerine-Sottas Disease (Type III)


- hypertrophic interstitial polyneuropathy
- autosomal recessive
- distal muscle weakness in the lower extremities
- pes cavus and foot drop
- progressive
- similar to CMT
- peripheral nerve enlargement and frequently palpable due to onion bulb formation

Rouse-Levy Syndrome (CMT types V or VI)


- similar to CMT
- Clumsy gait, abnormal equilibrium, absent DTR, acquired cavus
- Essential tremor in the hands that occurs during movement and is absent during rest

Refsum’s Disease
- progressive paresis of the distal aspects of the extremities
- drop foot, pes cavus, decreased DTR, ataxia, night blindness
- first symptom: night blindness
- autosomal recessive
- metabolic neuropathy caused by a disturbance of lipid metabolism which leads to an
accumulation of phytanic acid
- waxes and wanes
Friedrich’s Ataxia
- pes cavus and drop foot deformity
- more severe than CMT
- by age 30 pts are incapacitated
- degeneration of the post and caudal regions of spinal cord of unknown etiology
- instability and ataxia due to cerebellar involvement
- scoliosis and kyphoscoliosis are common
- peroneal nerve weakness will occur followed by ulnar nerve weakness
- glove and stocking distribution of decreased sensation
- Trendelenburg gait or lurching due to weakness of gluteus medius

Cerebrovascular Accidents
- Condition is static and non-progressive
- Surgical repair is very helpful
- An UMN disease
- Contractures of unopposed muscle groups is common
- Foot deformities such as equinovarus, claw-toes, spastic equinus and drop foot
- Bracing is part of treatment for joint imbalances
- Pt are usually on anti-coagulant therapy

Muscular Dystrophy
- Group of disorders were muscle tissue dies or becomes dystrophic
- Ankle equinus and equinovarus are common
- Duchenne’s
• The most severe
• Sex linked disease that tends to affect males
• Protein dystrophin is not produced
• Pseudohypertrophy is seen as fatty deposits in the muscle; often seen in the calf
• Goal of treatment: maintain ambulatory status of patient for as long as possible
• Gower’s sign is seen
- Becker’s
• More benign
• Abnormal dystrophin

Myelodysplasia
Spina Bifida:
- group of disorders that have a congenital manifestation of the spine
- incomplete closure of the midline of the osseous, mesenchymal and ueral tissues
- most often in the lumbar or sacral area
- results in dynamic imbalance in the muscles of the lower extremity
- progressive disorder
- commonly a pes cavus develops but equinus, equinovarus and pes valgus are possible
- many times these patients have a paralyzed bladder so catheterization is required during
surgery
Spina Bifida Occulta:
- isolated defect of the fusion of the posterior vertebral arch without disruption of the
underlying neural tissue
- many time asymptomatic
- most common in posterior arches of L5 and S1 leading to a cavus foot
- derm markers:
- hairy tuft, dimple or teratoma (lipoma) of skin of lower back. These lesions may grow
and compress cord
- Etiology: genetic, environment, unknown
- Hydrocephalus is common; shunts are used for treatment
- Adhesions may cause symptoms as child grows:
• The attachment of the spinal cord to the meningocele sac and the skin prevents
normal cephalad migreation of the spinal cord with growth. This produces the so
called tethered cord. Even after surgical release from the spinal cord, reattachement
of the cord is likely during the healing process. As the child grows, the spinal cord is
stretched, causing increasing neurological damage.
- Sensory and proprioception abnormalities may be worse than motor abnormalities
• The posterior aspect of the cord is exposed
• Posterior section of the cord is responsible for sensory and proprioceptive function
whereas the anterior part of the cord is involved in motor function

Poliomyelitis
- not very common today
- viral infection of the anterior horn cells
- paralysis of the muscles are involved; may be partial depending on the many cells or
sequential portion of the cord is damaged
- Tibialis anterior which is innervated by a short column of anterior horn cells is more
vulnverable to complete paralysis
- Paralysis is asymmetrical
- Most common cause of posterior cavus (due to loss of Achilles tendon function)
- Acute stage:
• Fever, myositis, paralysis
• Muscles are tender
• Pt will contract and shorten the muscle to relieve pain and if held in that position then
permanent contracture may remain
- Subacute or convalescent stage:
• Improvement of symptoms
• Aggressive physical therapy should be initiated
- Chronic Stage
• Period of stability
• Surgical intervention can be attempted
• Non-progressive once in this stage
Cerebral Palsy
- congenital neuromuscular disease caused by brain lesion
- lesions may be formed before, during or after birth
- common causes:
• maldevelopment of the brain
• cerebral anoxia during the prenatal period
• trauma during birth
• erythroblastosis secondary to Rh incompatability
• neonatal encephalitis
• prematurity
- non progressive syndrome but muscle imbalances may progress over time
- types
• spastic (65%)
• responds well to surgical repair of the deformities
• muscles that cross more than one joint are often infected
• the muscles of flexion, adduction and internal rotation tend to overpower those
muscles of extension, abduction and external rotation
• scissor gait is seen
• TEV commonly develops
• Other foot deformities such as talipes calcaneus, pes cavus, hallux valgus and
hammer toe deformities
• Athetoid (20%)
• Other (15%)
Questions about Lis Franc Injuries
How do you assess Lis Franc’s joint radiographically?
AP: Medial border of the 2nd metatarsal base should be aligned with the medial border
of the middle cuneiform
30 degree lateral oblique: Medial border of the 4th metatarsal base should be aligned
with the medial border of the cuboid
Lateral: Dorsal border of the 2nd metatarsal base should be aligned with the middle
cuneiform base
Desmond, FAI, 2006.
Myerson, Foot & Ankle, 1986.

What is the most common finding in a lis franc’s injury?


Diastasis of the bases of the 1st and 2nd metatarsal
What is the pathoneumonic finding for a Lis Franc’s injury?
Fleck sign (90%)
Desmond, FAI, 2006.
Myerson, Foot & Ankle, 1986.

How do you assess for calcaneal cuboid joint congruity?


Lateral oblique: No overlap with continuous trabecular patterns
What classification systems exist for Lis Franc injuries?
Quenu and Kuss (1909)
►Type A: Homolateral or total incongruity
►Type B: Isolateral or partial incongruity
►Type C: Divergent dislocation

Hardcastle Classification (1982)


►Type A – Total incongruity in one plane (may be
sagittal, coronal or combined)
►Type B – Partial incongruity: Displaced segment is in
one plane which may be sagittal, coronal or combined.
Medial displacement affects the first met either in isolation
or combined with displacement of one or more of the
second, third or fourth met. Lateral displacement affects one
or more of the lateral four mets but the first met is not
affected
►Type C – Divergent: partial or total incongruity. AP
radiograph shows the first met displaced medially while any
combination of the lateral metatarsals are displaced
laterally. Sagittal and coronal plane displacement.
Hardcastle PH, Reschauer R, Kutscha-Lissberg E, Schoffmann W. Injuries to the
Tarsometatarsal Joint. JBJS. 1982, Vol 64-B, Jun(3):349-56
Myerson’s Modification (1986)
►Type A – Total incongruity
st
►Type B1 – Partial incongruity 1 metatarsal
►Type B2 – Partial incongruity lesser metatarsals
►Type C1 – Divergent pattern, partial incongruity
►Type C2 – Divergent pattern, total incongruity
►B1, C1 and C2 also involve tarsal bones and the
cuneiform joints
Myerson MS, Fisher RT, Burgess AR, Kenzora JE.
Fracture Dislocations of the Tarsometatarsal Joints:
End Results Correlated with Pathology and
Treatment. Foot Ankle. 1986, Vol 6; Apr(5):225-42

What other injuries are accociated with Lis Franc Fx/Dislocation?


►Nutcracker type fracture of the cuboid
►Chopart subluxation/dislocation
►Calcaneal fractures
►Proximal injuries
►Compartment syndrome
►Disrupted neurovascular supply
►Soft tissue envelope compromise

What is the goal of reduction in displaced lis franc injuries?


Anatomic alignment of the bases of the metatarsals with their respective tarsal bones.

What are Charnley’s four steps to closed reduction?


1. Re-create/exaggerate the deformity
2. Distract the deformity
3. Reduce the deformity
4. Cast the body part

What are principles to reduction that are specific to lis franc’s complex?
►Key to reduction: 2nd metatarsal
►Step 1: Medial column
►Step 2: Lis Franc ligament complex
►Step 3: Lateral rays
From: Kelkikian. Operative Treatment of the Foot and Ankle. 1998.
If you take a lis franc injury to the OR, what are your fixation options?
►Closed reduction, cast immobilization
►PerQ fixation with K-wires only
►PerQ fixation with K-wires 4 – 5; ORIF of 1-3 PRN
►ORIF (no K-wires) with screws
►ORIF with plates
►External fixation
►Partial arthrodesis
►Full arthrodesis

Outcome for this injury depends on…


►Injury
►Anatomic reduction
►Patient education
►Surgeon’s experience

What are some of the major complications associated with this injury?
DJD
CRPS
Compartment syndrome
Cuboid syndrome
Additional surgery/fusion
Some good references for Lis Franc fracture/dislocations….
► Aitken AP, Poulson D. Dislocations of the Tarsometatarsal Joint. JBJS. 1963, Vol 45-A;
Mar(2):246-383
► Chandran P, Puttaswamaiah R, Dhillon MS, Gill SS. Management of Complex Open Fracture Injuries
of the Midfoot with External Fixation. JFAS. 2006, Vol 45; Sep/Oct (5):308-15
► Chang T. Master Techniques in Podiatric Surgery: The Foot and Ankle. Lippincott, Williams and
Wilkins, Philadelphia, PA. 2005:189-210
► Desmond EA, Chou, LB. Current Concepts Review: Lisfranc Injuries. Foot and Ankle. 2006,Vol
26;Aug(8):653-60
► Ebraheim NA, Haman SP, Lu J, Padanilam TG. Radiographic Evaluation of the Calcaneocuboid
Joint: A Cadaveric study. FAI. 1999, Vol 20; Mar(3):178-81
► English TA. Dislocations of the Metatarsal Bone and Adjacent Toe. JBJS. 1964, Vol 46-B;
Nov(4):700-4
► Gissane W. A Dangerous Type of Fracture of the Foot. JBJS. 1951, Vol 33-B; Nov(4): 535-8
► Hardcastle PH, Reschauer R, Kutscha-Lissberg E, Schoffmann W. Injuries to the Tarsometatarsal
Joint. JBJS. 1982, Vol 64-B, Jun(3):349-56
► Kelkikian AS. Operative Treatment of the Foot and Ankle. The McGraw-Hill Companies. 1998:455-
79.
► Kuo RS, Tejwani NC, DiGiovanni CW, Holt KS, Benirschke SK, Hansen ST, Sangeorzan BJ.
Outcome After Open Reduction and Internal Fixation of Lisfranc Joint Injuries. JBJS. 2000, Vol82-
A; Nov(11):1609-18
► Lin SS, Bono CM, Treuting R, Shereff MJ. Limited Intertarsal Arthrodesis Using Bone Grafting and
Pin Fixation. Foot Ankle. 2000, Vol 21; Sep(9):742-8
► Ly TV, Coetzee JC. Treatment of Primarily Ligamentous Lisfranc Joint Injuries: Primary Arthrodesis
Compared with Open Reduction and Internal Fixation, a Prospective Randomized Study. JBJS.
2006, Vol 88-A; March(3):514-20
► Mulier T, Reynders P, Dereymaeker G, Broos P. Severe Lisfranc Injuries: Primary Arthrodesis or
ORIF? Foot and Ankle. 2002,Vol 23; Oct(10):902-5
► Myerson MS, Fisher RT, Burgess AR, Kenzora JE. Fracture Dislocations of the Tarsometatarsal
Joints: End Results Correlated with Pathology and Treatment. Foot Ankle. 1986, Vol 6; Apr(5):225-
42
► Peicha G, Labovitz J, Seibert FJ. The Anatomy of the Joint as a Risk Factor for Lisfranc Dislocation
and Fracture-Dislocation. An Anatomical and Radiological Case Control Study. JBJS. 2002, Vol 84-
B; Sep(7):981-5
► Richter M, Wippermann B, Krettek C, Schratt HE, Hufner T, Thermann H. Fractures and Fracture
Dislocations of the Midfoot: Occurance, Causes and Long-term Results. Foot Ankle. 2001, Vol 22;
May(5):392-8
► Saxena A. Bioabsorbable Screws for Reduction of Lisfranc’s Diastasis in Athletes. JFAS. 2005, Vol
44; Nov/Dec(6):445-9
► Thompson MC, Mormino MA. Injury to the Tarsometatarsal Joint Complex. J Am Acad of Orthop
Surg. 2003, Vol 11; Jul/Aug(4):260-7
► Zgonis T, Roukis TS, Polyzois VD. Lisfranc Fracture-Dislocation: Current Treatment and New
Surgical Approaches. Clin Podiatr Med Surg. 2006, Vol 23; Apr(2):303-22
Case Study
CC: 22 y/o woman presents to our office c/o left ankle pain

HPI: Hurt it at track practice 2 days ago; trainer iced the ankle and applied ACE wrap and
told me to use crutches; was pushing off for a spring and it felt like someone kicked me
in the back of the leg; stabbing pain; didn’t do my usual stretching and was late for
practice.

PMH: Severe pyelonepthritis diagnosed 1 month ago

Meds: Ciprofloxacin 250 mg PO BID

Allergies: none

Family Hx: unremarkable

Social Hx: student at the local university; scholarship track team – sprinter; denies EtOH,
tobacco and drug use

ROS: unremarkable

Physical Exam:
Integ: diffuse ecchymosis posterior calcaneus, left
Vasc: palpable pedal pulses bilaterally; diffuse edema surrounding left ankle
Neuro: intact bilaterally
M/S: unable to do single leg toe raise on the left size. + Thompson test, left. + Knee
flexion test

Thompson-Doughtery Test
Positive – patient prone, no plantarflexion while the calf is being squeezed

Knee Flexion Test


Patient asked to actively flex the knee to 90 degrees while lying prone and the foot on the
affected side fails to neutral or dorsiflexion

Differential Diagnosis
Rupture of the Achilles tendon
Partial rupture of gastrocnemius muscle
Avulsion fracture of calcaneus (Rowe IIB)

Radiographs:
Increased soft tissue density and volume obliterating Kaeger’s triangle
No fractures, dislocations or foreign bodies noted.
MR:
T1: Disruption of signal within the tendon
T2: Generalized increase in signal intensity with edema and hemorrhage at the site of the
rupture with an area of high signal intensity

Treatment for Partial Rupture


1. Jones compression dressing for 24 – 72 hours
2. BK cast in plantarflexion for 3 – 4 weeks
3. Another BK cast x 4 weeks with less plantarflexion

Complete Rupture Operative Repair


1. Primary repair with suture/implant
a. Ma and Griffith – percutaneous suture
with non absorbabale suture on a straight
needle

b. Single suture Krakow

c. Double suture Krakow

d. Modified Bunnell

e. Double suture Bunnell

f. Double suture Kessler

g. V-Y Graftoplasty

h. Turn down fascial graft

i. Bosworth

Achilles Blood Supply


1.Muscle belly
2.Paratenon
3.Calcaneus

Normal Achilles tendons do not rupture – 98% of the time a degenerative process
preludes a rupture (i.e. fibrosis, peritendonitis or chronic tendonitis)
Anatomically – 3 distinct areas of concern:
- myotendinous origin: 4 – 14% of ruptures
- tendon: 72 – 73% of ruptures
- osseous insertion – 14 – 24% of ruptures

Watershed area – 2 – 6 cm proximal to the Achilles insertion; degenerates most rapidly


here due to a physiological twist in the tendon, not blood supply
Although…there may be some relation to blood supply because this area of the tendon is
the least vascular

MRI – Increases in intratendinous signal intensity should be regarded as abnormal


Evaluate tendon
- Signal response
• Increase intratendonous signal on T1 (fibrosis) – not seen on T2
- Morphology
• Increase thickness – fusiform/bulbous
• Loss of concavity of the anterior portion of the tendon

Pre-disposing factors to rupture


- Collagen vascular diseases
- Long term systemic steroid therapy
- Local steroid injection
- Flouroquinolones
Fifth Metatarsal Base Fracture Questions
1. What is the eponym associated with 5th MTB fractures?
- Dancer’s fracture
- Jones’ fracture

2. What are the two general types of the 5th MTB fractures?
- avulsion fracture of variable sizes
- Fracture of the metaphyseal – diaphyseal junction
There is also the diaphyseal stress fracture that is sometimes included in this list, though
it doesn’t occur in the base region.

3. What was Sir Robert Jones doing when he injured himself and subsequently described
the fracture associated with his name?
- 1902. Dancing around a tent pole at the military garden party in 1896.
- He described a transverse fracture of the 5th metatarsal that did not heal quickly.

4. What is the suggested treatment for non-displaced avulsion fractures of the 5th MTB?
- Heal sufficiently when immobilized in a cast or supported with elastic wrap. 3 – 4
weeks
- Delayed and non-union are rare complications and in the event that satisfactory bony
healing does not occur, most patients will remain asymptomatic because of fibrous tissue
bridging across the defect.

5. Name three structures that insert into the 5th MTB from proximal to distal.
- lateral cord of the plantar aponeurosis
- peroneus brevis
- peroneus tertius

6. Describe the Stewart Classification for 5th MTB fractures.


Type I – Proximal metaphyseal – diaphyseal fracture (true Jones)
Type II – Intra-articular avulsion fracture
Type III – Extra-articular avulsion fracture
Type IV – Intra-articular comminuted fracture
Type V – Partial avulsion fracture of the epiphysis (located in a longitudinal direction);
risks of Iselin’s AVN

Where is a Jones fracture located?


- at the metaphyseal-diaphyseal junction of the 5th MTB approximately 1.5 cm
distal to the tuberosity
- distal to the intermetatarsal ligament at the bases of the 4th and 5th metatarsals

What are three ways that you can fixate a displaced avulsion fracture?
- K-wires
- Screws
- Tension band wire
Case Study
HPI: 45 y/o WF is seen at bedside at AM rounds. Pt complains that she feels lousy and
had a difficult time sleeping last night. Pt is 2 days s/p triple arthrodesis of the right foot
for severe acquired flatfoot. Pt denies F/V/CP/SOB and states that she feels cold and
lousy.

Allergies: PCN; codeine


Meds: Clinda 600 mg q6h
Atenolol 50 mg qd
Glucotrol XL 5 mg qd
ASA 325 mg every other day
Premarin .625 mg qd
Depomedrol injection every three months
PMH: HTN, DM2 x 10 years
SHX: Smokes 2ppd x 25 yrs. EtOH occasionally
FHX: Mother deceased at 82 gunshot to the head; Father deceased at 74 from massive MI
ROS: HEENT: occasional headaches; CV: dx with a heart murmur in 1998

PE:
Vitals: Ht 62 inches; Wt 168 lbs; T 101.1; P: 92; R 24; BP 185/90
Integ: upon removal of the cast, medial and lateral incisions appear intact with some
tension noted; suture intact; some periwound erythema and edema; right calf appears
swollen compared to the left.
Vasc: Non-palpable, right due to edema. Palpable pedal pulses, left.
Neuro: decreased to SWM b/l
M/S: pain to palpation of the right posterior calf

What do you have to rule out at this point? And how do you rule it out?
1. Atelectasis – chest x-ray
2. UTI – UA with cultures
3. DVT – venous duplex doppler
4. Wound infection – clinical correlation
5. Drug fever

What are the five causes of post – op fever and what is the time period in which they
occur?
Wind: 12 hours
Water: 24 hours
Walking: 12 – 48 hours
Wound: 2 – 4 days
Wonder Drug: anytime
Results:
1. Chest x-ray is negative; pt has been using the incentive spirometer once every
hour for five minutes; pt does not complain of shortness of breath or chest pain
2. UA results are the following
pH: 6.5 color: straw
specific gravity: wnl WBC 2/field
bacteria: 0 nitrities: negative
Leukocyte esterase: negative
3. Duplex Doppler was positive for actute DVT in the posterior tibial vein of the
right leg
4. No work up needed because no wound infection suspected. Normal post op
changes to the peri-wound areas.

Treatment:
- Pt started on 15,000 u bolus of heparin IV
- Pt given 1400 u heparin IV/hr
- PTT was checked four hours after initiation of therapy and found to be 46 sec
(normal = 25 – 35s)
- Pt was given a bolus of 3000i IVP and 1452u/hr
- PTT was checked again four hours after adjustment of therapy: 75 sec (you want
the PTT to be 70 – 95here)
- Pt was started on 10 mg of coumadin 24 h after initial diagnosis of DVT; pt was
give 10 mg of coumadin the next day
- PT was taken and INR was found to be 2.5 (you want it between 2 and 3)
- Heparin was continued for five days after coumadin was started
- Coumadin was continued for 3 months
Case Study
HPI: 24 yr old WM presents to the ER c/o a thigh cast on his right leg. Pt had a haglund’s
deformity removed 3 days prior. Pt states that the pain woke him up last night and has
been present every since. Pt took two Tylenol #3 which didn’t help.

PMH: unremarkable
Medications: none
Allergies: none
SHx: denies tobacco, drugs; social EtOH
ROS: unremarkable

PE:
Vitals: Ht 68 inches; Wt 215 lbs; VSS; afebrile
Integ: R leg warm, slight erythema, diameter in center of calf is 3 cm > L calf; incision
intact, no soi
NV: intact
MS: unremarkable

A/P:
R/O DVT
Duplex Doppler was positive for DVT in the right calf
Pt admitted to hospital and began anti-coagulant therapy
DVT Related Questions
1. What is virchow’s triad?
- states of hypercoaguability
- venous stasis
- damage to the endothelial lining in the blood vessel

2. Pneumotic for risk factors of developing DVT.


I AM CLOTTED
Inactivity
A Fib/Age
MI
Coaguable state
Longevity of surgery
Obesity
Tobacco/tourniquet
Tumor/trauma
Estrogen (oral contraceptives)
DVT hx

3. What are the common locations for DVT?


- 20 % of calf emboli will become thigh emboli
- 1/5th of PE come from the calf

4. How do you diagnose a DVT?


Clinically: red, hot, swollen, painful calf
- edema is the most reliable sign of DVT (compare suspected calf to the contralateral
side)
Homan’s test: DF foot elicits pain in the calf
Pratt’s sign: compression of calf elicits pain

Diagnostic Tests:
1. Non-invasive
a. Duplex Doppler: lack of venous compression indicates DVT
i. Can have color flow imaging to enhance sensitivity
ii. Allows to determine direction of blood flow and the amount of reduction in
lumen diameter
iii. Good for symptomatic DVT
iv. Grady-Bensmetal JBJS, 1994: duplex ultrasound has the positive predictive
value of 7/9
v. Ciccone et al JBJS 1998: duplex ultrasound with color flow imaging is
unreliable in detecting asymptomatic DVT post THR/TKR
b. Impedence plethysmography
2. Invasive
a. Contrast venography
i. Gold standard for detecting DVT
ii. Not as reliable in detecting recurrent DVT
iii. Has been known to cause PE during testing
iv. New contrast media has decreased this risk
5. How do you diagnose a PE?
Clinically: sudden onset of chest pain, dyspnea, hemoptysis, tachycardia
Pt may be febrile, hypotensive and cyantic

Diagnosis:
1. Blood gasses: PaO2 < 80 mmHg
2. Chest x-ray: 50% are normal; a normal or near normal chest x-ray in a dyspneic
patient suggests PTE. Well – established abnormalities include focal oligemia
(Westermark’s sign), a peripheral wedged shaped density above the diaphragm
(Hampton’s hump) or an enlarged right descending pulmonary artery.
3. Ventilation – Perfusion Scan (V/Q Scan)
a. A mismatch demonstrating an area of ventilation but no perfusion suggests
PE
b. Ventilation: inhalation of xenon 133
c. Perfusion: T99 labeled albumin
d. V/Q mismatch: acute PE, previous PE, centrally located cancer, radiation
therapy
4. Pulmonary angiography
a. Definitive test
b. Indicated if V/Q scan is inconclusive
c. Diagnostic signs: intraluminal filling defect, abrupt vessel cutoff, loss of
side branches

6. Prophylaxis
a. Non-pharmacologic
- compression stockings
- intermittent compression pumps
1. increases levels of prostacyclin and fibrinolytic byproducts
2. prevents stasis due to increased venous return
b. Heparin
- 5000u SQ q2h pre-op
- 5000u SQ q8-12h

7. Treatment
a. Heparin IV
- loading dose: 10,000 – 15,000u or 80u/kg
- maintenance dose: start with 1,000 u/hr (18u/kg/hr)
- adjust according to PTT (goal 57 – 90 seconds)
- protamine sulfate reversed heparin
- 1 mg protamine pre 100 u heparin
- use with care in pts on NPH insulin
- complications: hemorrhage, thrombocytopenia, ostoporosis with long term use
b. Coumadin
- start after heparin is therapeutic
- commonly 2.5 mg qd
- adjust according to PT (1 – 1.15 x normal/INR 2 – 3)
PT normal = 11 – 13 INR normal 0.8 – 1.2
- vitamin K reversal
c. LMWH
- Safer than regular heparin
- no need to monitor PTT, easier dosing
i. Lovenox
Theraputic: 30 mg SQ BID
Prophylatic: 1 mg/kg
ii. Fragmin
2500 u SQ qd
iii. Normiflo
5000 u – 10000 u SQ BID
Compartment syndrome has been implicated with their use (McLaughlin et al
JBJS 1998)
d. Thrombolytic therapy
- urokinase, streptokinase to dissolve clot
- must be initiated within 24 – 48 hours
e. Surgical therapy
1. venous interruption operation
- Greenfield filter: placed in IVC below renal veins
2. Pulmonary embolectomy

8. Post-phlebetic syndrome (post thrombotic syndrome)


- result of venous HTN due to recanalization of major thrombi which lead to patent but
scarred/incompetent valves
- occurs in 50 – 60% of patients with proximal DVT; 30% of patients with symptomatic
calf DVT
- blood can now flow from the deep to superficial veins which leads to persistent LE
edema, stasis dermatitis may occur and breakdown of skin and ulceration develops

9. Pt has a DVT in his lower extremity. Clots are thrown and pt is found dead at home.
Upon autopsy pt is found not to have died from a pulmonary embolism but from a
massive CVA. How is this possible?
Pt had an undiagnosed patent foramen ovale. This is called a paradoxical embolism.

10. What is usually the immediate cause of death from a PE?


Right sided heart failure.
Right ventricular dysfunction. Progressive right heart failure is the usual immediate cause
of death from PTE. As pulmonary vascular resistance increases, right ventricular wall
tension rises and perpetuates further right ventricular dilation and dysfunction.
Consequently, the inter-ventricular septum bulges into and compresses an intrinsically
normal left ventricle. Increased right ventricular wall tension also compresses the right
coronary artery and may precipitate myocardial ischemia and right ventricular infarction.
Underfilling of the left ventricle may lead to a fall in left ventricular output and systemic
arterial pressure, thereby provoking myocardial ischemia due to compromised coronary
artery perfusion. Eventually, circulatory collapse and death may ensue.
Case Study
CC: Pain at the 1st MPJ

HPI: 34 y/o healthy female s/p bunionectomy x 18 months presents to your office c/p
pain, stiffness and swelling at the base of the left big toe joint. The symptoms began
gradually approximately 6 months ago. Her condition has gradually worsened and now
her big toe joint is almost as painful as it was pre-op. Symptoms are triggered
predominately by weight-bearing. She has been treated with anti-inflammatory
medication and rest by the physical that performed the surgery. This provides temporary
relief of her symptoms.

PMH: Seasonal allergies

PSxH: Bunionectomy, left foot – 18 months ago

Allergies: NKDA

Meds: Claritin

SH: Denies tobacco; EtOH and ilicits

FH: All alive and well

ROS: Non-contributory

Physical Exam
INT: warmth and erythema noted at the 1st MPJ

VASC: palpable pedal pulses, edematous 1st MPJ

NEURO: intact

M/S: pain on palpation of the 1st MPJ, pain on passive and active ROM, decreased 1st
MPJ ROM (approximately 15 degrees)

List your differential diagnosis.


You can list a million things but you MUST include: AVN and infection

What would you like to order? X-rays

Describe your radiographic findings.

What is your diagnosis? AVN of the first metatarsal (Assman’s disease)

Plan: What is your treatment plan?


Metatarsal head AVN Related Questions
1. List and describe the radiographic stages of AVN.
Stage 1: Avascular Stage (several weeks to 3 months)
- Relatively few clinical and radiographic changes
• Soft tissue inflammation (increase in soft tissue density)
• Thickened synovium (appears as capsulitis/synovitis)
- The ossific nucleus of the epiphysis ceases to grown due to the interrupted blood
supply
• Smaller epiphysis on affected side
• There is a relative increase in the density of the epiphysis which is illusional due
to the adjacent osteopenia seen in the metaphysis
• Joint space appears increased secondary to decreased size of the epiphysis
• Articular cartilage is unaffected since it is nourished by synovial fluid
- Minimal deformity
- Mild arthralgia with associated edema, muscle atrophy and antalgic gait
Stage 2: Revasculization Stage (1 – 4 years)
- revascularization of the dead epiphysis, which further undermines its integrity due
to significant vascular inbudding
- there is a true increase in density of the epiphysis due to condensation and
impaction, as the epiphysis collapses on itself
- “head within head” appearance – the epiphysis appears very irregular due to
deposition of new bone around existing dead bone
- Subchondral fractures may occur during this extremely vulnerable stage, in which
osteogenesis produces primary woven bone (soft bone)
Stage 3: Remodelling Stage/Bony Healing
- bony deposition predominates over bony resorption, replacing old necrotic bone
- osseous structures still suseceptible to injury and deformity, but to a lesser degree
than stage 2
- radiographic finding: return of the contour and outline of the epiphyseal center,
which can only adequately be assessed when re-ossification is complete
Stage 4: Residual Deformity
- complete bony healing has occurred
- epiphysis may be normal if sufficient treatment was rendered
- resultant end stage deformity is assessed at this point

How do you diagnose AVN?


Sequential radiographs and clinical correlation

If not evident on plain films, what other imaging modality can be used to detect AVN and
how does it appear?
MR imaging. Since most bone infarct occur in fatty tissue, MR imaging is very useful.
Decreased signal intensity within medullary bone would be noticed in T1 and T2 weighted
images. More sensitive and has better resolution than a bone scan.
How do you treat AVN?
This is a self limiting process. We are trying to prevent residual deformity.
Conservative Treatment:
- NSAIDs
- Orthotics
- Analgesics
- Protected/off weightbearing (casting, bracing, padding)
Surgical Treatment:
- arthrodesis

According to the literature, what is the incidence of post-operative AVN following a


distal 1st metatarsal osteotomy?
- Lateral release of the fibular sesamoid, conjoined adductor tendon, deep
transverse intermetatarsal ligament and joint capsule and excessive dissection,
especially the periosteal tissues can compromise the blood supply to the first
metatarsal head.
- Wilkinson et al using MR imaging found a 50% incidence of AVN following
Austin bunionectomy, with 10% of plain film in this study showing signs of
AVN.
- Wallace et al using a mailed survey to 45 podiatry surgeons, reported an incidence
of only 0.11% in 13,000 head osteotomy bunionectomies.
- Although AVN is a recognized complication of first metatarsal head osteotomy
and may occur at a higher incidence than first recognized, clinical signs and
symptoms are relatively rare.

How do you reduce the incidence of AVN when performing a first metatarsal osteotomy?
- rigid fixation
- minimal dissection
- minimal lateral release

Name the true AVNs.


- Renandier’s disease – tibial sesamoid
- Trevor’s disease – fibular sesamoid
- Freiberg’s disease – second metatarsal head
- Kohler’s disease – navicular
- Diaz/Mouchet disease – talus
- Blount’s disease in adolescents – medial proximal tibial epiphysis
- Legg Calve Perthes Disease – femoral head

Name the AVNs that are NOT true.


- Sever’s disease – calcaneus
- Theimann’s disease – phalanges (hand and foot)
- Islen’s disease – 5th MT base
- Haglund’s disease – accessory navicular
- Osgood Schlatter’s disease – tibial tuberosity
- Blount’s disease in children – proximal medial tibial epiphysis
- Scheurman’s disease – ring epiphysis of the vertebral body
- Calve’s disease – solitary vertebral body
- Buchman’s disease – iliac crest
- Van Neck’s disease – ischiopubic synchondrosis
- Mandle’s disease – greater trochanter of the femur
- Singling-Larson-Johannson disease – lower patella

Name the AVNs that may or may not be true AVNs.


- Buschke’s disease – cuneiform(s)
- Lewis/Liffert/Arkin – distal tibial epiphysis
- Ritter’s disease – proximal fibula

The AVNs by Bones in the foot.


- Calcaneus – Sever’s disease
- Talus – Diaz disease
- Navicular – Kohler’s disease
- Cuboid – Lance’s disease
- Cuneiforms – Buschke’s disease
- 1st MT Head – Assman’s disease
- Tibial sesamoid disease – Renandier’s disease
- Fibular sesamoid disease – Trevor’s disease
- Lesser met heads – Frieberg’s disease
- Phlanages – Theimann’s disease
Puncture Wounds
Haverstock & Grossman. Puncture wounds of the foot. Clin Pod Med: Oct 1999

2 most important complications: Soft tissue infection and


Osteomyelitis (calcaneus is most common)

Most common puncture objects: Nails


Glass

Clinical Presentation
- Patients usually present immediately due to pain and the inability to bear weight.
- The puncture wound penetration site usually exhibits signs of acute injury: localized erythema and
edema
- If a patient delays in seeking medical care, infection may already be established in the wound.
- When a foreign body is retained in the puncture wound, infection can manifest in 24 hours but
may have a delayed presentation of 3 to 4 days

Clinical Evaluation
- Thorough history (time of injury, type of shoe, environment and a good description of the
penetrating object. It is also important to find out if the patient has attempted to remove the
object.)
- Patient’s tetanus status must be ascertained and appropriately addressed (see tetanus prophylaxis)
- Determining the location and depth of the wound is important
- It must be determined if the wound was close to a joint, tendon, or any other NV structure
(Patzakis, et al – Classification system)
- Determining the depth of the puncture is essential because extension of the puncture wound to
deeper structures may cause penetration of structures such as the intrinsic and extrinsic tendons,
intermetatarsal bursae or joint capsules. These structures are less vascular and have a lower
metabolic rate, allowing infectious organisms the opportunity to replicate and the infection to be
established. Nerve damage should be evaluated by testing distal sensation. Tendons should be
evaluated as well.
- Inspect the borders of the wound. Jagged, irregular borders have an increased potential for
infection due to decreased vascular perfusion of the apices.

Radiographic Evaluation
- Plain films should always be obtained to determine if any object has been retained or to see if any
osseous structures have been involved.
- A retained foreign body can be a nidus for infection
- Can glass be seen on plain films? The size of the piece is the limiting factor
- Plain film may be inadequate to show small pieces of glass, wood, plastic or material from socks
and shoes
- Xeroradiography, ultrasound, CT or MR may also be used.
- CT is good to visualize wood in deep tissues
- Ultrasound is good to detect non-radio-opaque bodies. May be useful when the puncture wound is
near a vessel
- Plain film, CT, MR, and radionuclide imaging also can be used to detect osteomyelitis

Wound Microbiology
- Most common gram positives responsible for soft tissue infections:
• S aureus
• S epidermidis
• Streptococcus
- Most common gram negatives responsible for soft tissue infections:
• E coli
• Proteus
• Klebsiella
- Pasteurella multocida in dog/cat bites
- Pseudomonas aeruginosa is the most common organism isolated in osteomyelitis following
puncture wounds (especially in gym shoes). This is well documented.
- Patients without systemic illnesses usually have one pathologic organism
- Patients with systemic illnesses usually have more than one pathologic organism

Treatment
- The goal of treating puncture wounds is to convert a contaminated or dirty wound into a clean
wound.
- Begin by cleansing the wound with sterile saline. If local anesthetic is needed, perform the block
proximal to the wound. You do not want to spread the soft tissues around the wound.
- The wound should be explored with a blunt sterile probe. Wounds extending beyond the deep
fascia must be treated aggressively to present deep abscess formation and possible osteomyelitis.
- Incision and drainage are required in all deep, contaminated or infected wounds. Once aggressive
debridement is completed, the wound should then be irrigated, preferably with pressure irrigation.
Deep wound cultures then are taken by removing deep tissue samples and sending them for culture
and sensitivity. The wound should be packed open or closed over a drain to prevent hematoma or
establishment of an anaerobic infection. Intraoperative radiographic techniques may be utilized,
such as triangulation or a metal grid system.

Antibiotics
- Controversial issue
- If the wound has not penetrated the plantar fascia and is clear of any foreign matter or necrotic
tissues, then observation without antibiotics is sufficient
- If the wound has penetrated deep through the plantar fascia, the likelihood of contamination may
necessitate the use of prophylactic antibiotics
- When puncture wound occurs in a shoe, the antibiotic must cover pseudomonas aeruginosa.
• Recommend:
• oral fluoroquinolones, such as cipro
- Late puncture wounds with abscess formation or ascending cellulitis must be treated with IV
antibiotics.
• Recommend:
• anti-pseudomonal penicillin class, such as ticracillin and pipercillin;
• third generation cephalosporins, such as cefoperazone and ceftazidine;
• aminoglycosides with clindamycin
- Wound causing osteomyelitis must receive IV antibiotics x 6 weeks

Wound Contamination
Clean – No frank contamination. No break in aseptic technique. Most surgical wounds

Clean – Contaminated – Minimal wound contamination owing to a minor break in technique

Contaminated – Significant bacterial contamination. Most traumatic wounds.

Dirty – Grossly contaminated with signs of frank infection.


Tetanus Prophylaxis
Taken from Joseph, Warren S. Handbook of Lower Extremity Infections. 2nd edition. Chruchill Livington; St Louis,
2003. pg 87.
Clean, Minor Wound Other Wounds
Hx of Tetanus Toxoid Immunoglobulin Toxoid Immunoglobulin
Toxoid Admin
Unknown or less Yes, and proceed No Yes, and Yes (250 U of
than 3 with basic proceed with human tetanus
immunization basic immunoglobulin)
immunization
More than 3 No, unless >10 No No, unless >5 No
yrs since last years since last
dose dose

When determining what to give in regards to tetanus, there are three things to look at:
1. Immunization status
2. Timing of the patient’s last booster
3. Wound and wound treatment
4.
Taken from: Brooks JW. Management of Pedal Puncture Wounds. JFAS 1994. 33(5):463 – 466.
Immunization Booster Status Wound Tetanus
Status
Completed Within the last year None
Completed in the None .5 ml toxoid (Td)
previous 10 years
Completed more Within the previous .5 ml Td
than 10 years 10 years
previously
Completed more None within the Minor wound .5 ml Td
than 10 years previous 10 years Relatively clean
previously Treated promptly
and adequately
Completed more No booster in the Something other .5 ml Td
than 10 years previous 5 years than minor wound 250 units tetanus
previously Not clean immune globulin
Not treated 500 units TIG if
promptly wound is clostridia-
prone
No record or hx of No record of booster Minor wound .5 ml Td
immunization Clean wound 2nd injection 4 – 6
Treated promptly weeks later
and adequately 3rd injection 6 – 12
months after second
No record or hx of No record of booster Wound not clean, 250 units of TID
immunization not minor and not (500 units TID if
treated promptly or claustridium prone)
adequately w/above Td sequence
Hallux Varus
Classification:
1. Congenital hallux varus (Tachdjian, 1982/1985)
a. Primary: isolated deformity where a taught medial band of tissue extends
from the medial side of the great toe to the base of the first metatarsal
b. Secondary: associated with other congenital deformities: metatarsus varus
and broadening and shortening of the first metatarsal.
c. Tertiary: associated with developmental afflictions of the skeleton, i.e.
diastrophic dwarfism
2. Acquired hallux varus
a. Static: muscle balance undisturbed (i.e. met osteotomy)
b. Dynamic: muscle imbalance (i.e. adductor transfer of the McBride type)
Etiology
1. Excessive resection of medial eminence with disruption of the sagittal groove
2. Plantar lateral release
3. Excessive tightening of the medial capsular structures
4. Osseous over correction
5. Aggressive post operative bandaging or splinting
6. Excision of the fibular sesamoid
7. Muscle imbalance caused by previous HAV correction
Relevant History:
1. HAV surgery

Symptoms
1. Unable to wear conventional shoe gear
2. Pain present at medial aspect of hallux secondary to shoe pressure
3. Pain along the medial aspect of the arch secondary to muscle contracture
4. Pain at the first MPJ with or without shoe gear.

Clinical Signs of Hallux Varus


1. Adducted position or varus rotation of the hallux
2. Contracture at the IPJ or involving the EHL or abductor hallucis tendon (cocked
up hallux)
3. Pain on palpation of abductor hallucis
4. Pain and crepitus at the 1st ray
5. Hallux limitus or rigidus

Radiographic Signs of Hallux Varus


1. Hallux abducted at 1st MPJ
2. Staked head of the 1st metatarsal with a peeking tibial sesamoid
3. Absent fibular sesamoid
4. Previous osteotomy of the 1st metatarsal
5. Reduced or negative IM angle
6. Negative PASA
7. Arthritic changes at the 1st MPJ and/or IPJ of hallux
Correction of Hallux Varus
Conservative
a. Adductovalgus strapping and splinting
Surgical
b. Soft tissue release
i. Medial capsulotomy
ii. Lateral capsulotomy
c. Dynamic soft tissue procedure
i. Abductor hallucis tendon transfer
ii. Retransfer adductor hallucis tendon laterally
d. Osseous procedures
i. Correct abnormal IM angle or PASA (reverse Austin, etc.)
e. Joint destructive procedures
i. Arthroplasty (Keller)
ii. Arthrodesis (McKeever)

Stepwise Approach (Presby)


Total soft tissue release at the 1st MPJ
Medial capsulotomy
Tibial sesamoidectomy (if 30 – 50% of sesamoid is peeking)
Address abductor hallucis as a deforming force – transfer abductor hallucis to the
plantar – lateral flexor apparatus and lateral base of the proximal phalanx
Osseous correction
a. Determine level of osseous deformity
b. Revise the previous metatarsal osteotomy
c. If the IM angle is negative, reverse the overcorrection with the appropriate
osteotomy
(Reverse Austin, reverse off-set V, reverse CBWO)
Arthroplasty of the 1st MPJ for severe deformity and for non functional joint surfaces
d. Keller
e. Implant
f. Keller with fusion of IPJ
Arthrodesis of 1st MPJ
g. McKeever
h. Lapidus
i. If both sesamoids are removed, fuse the IPJ of the hallux

Systemic Repair of Hallux Varus (McGlamry)


Complete soft tissue release
Correction of structural deformity (IM angle)
Tendon transfers
Tibial sesamoidectomy
Joint arthroplasty
Step-wise correction of Hallux Varus.

1. Skin incision – If the skin incision is contributing to the deformity, scar revision (e.g.
Z-plasty) instead of reincision should be performed.
2. Soft tissue dissection will be more difficult and requires precise establishment of
tissue planes.
3. Release all soft tissue from the joint – will help to determine of the deformity is
positional, structural or both.
4. Release the medial capsule. V shaped capsulotomy is ideal for this.
5. Tenotomize the abductor hallucis tendon if it is contributing to the deformity.
6. Evaluate the IM angle and the PASA at this point.
7. Choose osseous procedure based on which procedure was performed before, length of
the 1st metatarsal and the amount of correction needed.
8. If the tibial sesamoid is dislocated, relocation should be attempted. If this fails, remove
the sesamoid and fuse the HIPJ.
Taken from: Zahari & Girolamo. Hallux varus: a step-wise approach for correction. JFAS, 1991. 30(3):264
– 266.
Questions about Tarsal Tunnel Syndrome
1. What are the contents of the tarsal tunnel?
- Tibialis posterior tendon
- Posterior tibial artery
- Posterior tibial nerve
- Posterior tibial companion veins
- Flexor digitorum longus
- Flexor hallucis longus

2. Name some of the specific nerves that can be affected by tarsal tunnel syndrome.
- Medial plantar nerve
- Lateral plantar nerve
- Nerve to the abductor digiti minimi (Baxter’s nerve)
- 1st branch of the lateral plantar nerve

3. What are the attachments of the flexor retinaculum?


- medial malleolus & medial surface of the calcaneus blending with plantar aponeurosis

4. Where does the bifurcation of the tibial nerve usually occur?


- deep to the flexor retinaculum
- a variety of branching patterns have been described; rarely does it branch proximal to
the retinaculum

5. What are the borders in all directions as the tibial nerve courses under the flexor
retinaculum?
- superficially: the flexor retinaculum
- deep: the medial side of the talus, sustentaculum tali, medial side of the calcaneus
- superiorly: the deep aponeurotic fascia of the leg
- inferiorly: the abductor hallucis muscle

6. Name ten etiologies of tarsal tunnel syndrome.


- trauma (fracture, ruptured tendon)
- varicosities (engorgement of the veins)
- hypermobile flat foot
- idiopathic tight tarsal canal
- obesity
- space occupying lesion
- hypertrophic abductor hallucis muscles
- RA
- Diabetes
- Post surgical fibrosis
- Accessory FDL muscle
7. What two athletes have been cited as having a high incidence for tarsal tunnel
syndrome?
- long distance runners
- jockeys: forced eversion against a flat foot while in stirrups

8. What are some conservative options for treatment?


- Orthoses
- NSAIDs
- Injection with local anesthetic or corticosteroids
- Ice
- Physical therapy
- Stretching exercises

9.What is anterior tarsal tunnel syndrome?


- compression of the deep peroneal nerve within the fibro-osseous tunnel created by the
inferior extensor retinaculum

10. What are the contents of the anterior tarsal tunnel?


- deep peroneal nerve
- dorsalis pedis artery and veins

11. Name some causes of anterior tarsal tunnel syndrome excluding those mentioned
before.
- tight laced shoe gear
- osteophytes at the ankle or talonavicular joint
- pt with cavus feet

12. Why is that the symptoms in a patient with anterior tarsal tunnel syndrome are
accentuated during sleep?
- foot drops into plantarflexion during sleep

13. Name the three types of nerve injury according to Seddon.


a. Neuropraxia: physiological disruption rather than anatomic disruption. Often caused
by compression and distortion of the myelin sheath and a transient loss of salutatory
conduction. Some common causes are contusions, hematoma, unreduced fracture or
dislocation pressure, iatrogenic. There is usually full recovery of nerve function.
b. Axonotmesis: Anatomic disruption of the axon without disruption of the endoneurial
tube. Axon undergoes Wallerian degeneration where the axonal material distal to the site
of injury degenerated. Since the endoneurial tube is intact regeneration is possible.
c. Neurotmesis: Anatomic disruption of the axon and the endoneurial tube and is usually
irreversible

14. Name two iatrogenic causes of neuropraxia.


- tourniquet
- cast
15. Name the stages of Sunderland’s classification of nerve injury.
- 1st degree: neuropraxia
- 2nd degree: axonotmesis
- 3rd degree: loss of axon and endoneurium with intact perineurium
- 4th degree: 3rd degree plus loss of perineurium with intact epineurium
- 5th degree: complete loss of entire nerve trunk

16. Name the two major types of nerve repair (neurorrhaphy)


a. epineurial repair: suture the epineurium together
- try to orientate the fascicles as closely as possible
- advantages include technical ease, short OR time, minimal Rahimi
(magnification) requirements
b. fasicular (perineurium) repair: suture the perineurium together
- this offers the potential for a better alignment of fascicles
- disadvantages include the need for greater Rahimi requirements, longer
surgical time, an increased potential for intraneural fibrosis
Chronic Lateral Ankle Instability
Etiology
Inversion injuries of ankle
a. Most common soft tissue injury
b. Athletes
i. 16 – 21% of all athletic injuries
ii. Basketball – 43% of injuries
iii. Soccer – 31% of injuries
iv. Running and jumping sports – 25%
Proper diagnosis in the acute injury is critical
c. Inadequate treatment = poor healing and chronic instability

Anatomy
Anterior talofibular ligament
a. Intracapsular
b. Origin: distal anterior fibula
c. Insertion: body of the talus – anterior to articular facet
Calcaneofibular ligament
d. Extracapsular
i. Confluent with peroneal tendon sheath
e. Origin: anterior border of distal malleolus below the origin of the ATFL
f. Insertion: lateral body of the calcaneus superior to peroneal tubercle
Posterior talofibular ligament
g. Intracapsular
h. Origin: medial surface of lateral mallelous
i. Insertion: broad, majority of posterior lip of talus

Pathology of the Chronic Lateral Ankle Instability


- Undiagnosed or untreated acute ankle sprains
- Chronic ankle sprains
- Leads to scarred or elongated ligamentous structures

Diagnosis – Clinical Evaluation


History
a. Ankle ‘giving way’
b. Recurrent acute ankle sprains
c. Restricted physical activity as a result of pain
Physical Exam
a. Positive anterior drawer test – ATFL
b. Positive inversion stress test
i. Sulcus between talus and lateral malleolus (sucking sign)
c. Anterior and lateral aspect of the lateral malleolus
ii. Tenderness
iii. Chronic edema
Diagnosis – Radiographic Evaluation
Routine radiographs
a. AP, lateral and mortise
Stress radiographs – Bilateral
b. Talar tilt (inversion stress)
i. Tests CFL
ii. Normal up to 4 – 7 degrees
iii. 5 degrees greater than contralateral side is pathological
c. Anterior drawer
i. Tests ATFL
ii. Greater than 4 mm displacement is positive

Classification of Lateral Ankle Sprains


Anatomic System
a. Grade I: ATF sprain
b. Grade II: ATF and CF sprain
c. Grade III: ATF, CF and PTF sprain
AMA standard nomenclature system
d. Grade 1: Ligament stretched
e. Grade 2: Ligament partially torn
f. Grade 3: Ligament completely torn

Treatment Options
Conservative
a. Activity and shoe modification
i. Lateral heel wedge, lowering heels, stiff soles
b. Orthotics with lateral heel wedge
c. Aircast, ankle corset, strapping
d. Peroneal strengthening exercises
e. Jones compression dressing
f. RICE
g. NSAIDs
Surgical
a. Success of conservative therapy
b. Degree of ankle instability
c. Activity level
d. Compliance
e. Surgical risk factors
f. Primary repair or not primary repair
i. Degree of ligament scarring and elongation
ii. How old is the instability?
g. Drill hole difficulty
h. Tendon harvest length
i. Single ligament repair vs double ligament repair
j. Regional anatomy considerations
iii. Intermediate dorsal cutaneous nerve
iv. Sural nerve
v. Small saphenous vein
vi. Peroneal tendons
Surgical Goals:
Restore proper geometric alignment of the ligaments
Restore mechanical and functional stability to the lateral side
Full ROM

Surgical Management – Primary Repair of Ligaments


- Brostrom – 1966
- Modified Brostrom – 1980
- Arthroscopic repair – 1985 (Lundeen)

Surgical Management – Replacement of Ligaments


- Nilsonne – 1932
- Elmslie – 1934
- Hambly – 1945
- Evans – 1953
- Winfield – 1953
- Watson-Jones – 1955
- Lee – 1957
- Christman and Snook – 1969
- Kelikian and Kelikian – 1985
- Split peroneus brevis lateral ankle stabilization

Brostrom – 1966
- Imbrication of ATFL and CFL
- Gold standard for procedure comparison
- Reported satisfactory results in 85% of patients
- Disadvantage: difficult to expose and suture torn ends due to scarring and elongation of
the ligaments
- Summary: anatomic reapproximation of the ATF and CF with possible ATF
augmentation utilizing the lateral talocalcaneal ligament. ATF is not incised during the
capsular incision.
- Brostrom’s Logic for anatomic reconstruction: “previously used tenodesis operations
are technically fairly complicated, that the healing time is long, that the normal anatomy
is not fully restored, that increased stability is obtained at the price of diminished
supination mobility , that they are unsuitable in children because of the risk of epiphyseal
damage, and that in most cases they entail weakening of the peroneus muscles.”
- Original article citation: Brostrom, Lennart. Sprained ankles VI. Surgical treatment of
“chronic” ligament ruptures. Acta Chir Scand 132: 551-565, 1966.
-
Modified Brostrom (Gould & Seligson) – 1980
- Imbrication of the inferior extensor retinaculum to reinforce repair
- Suture whatever one can find
- Reinforcing the ATF ligament repair with overlap of lateral
talocalcaneal ligament plus inferior extensor retinaculum
- 30 minute procedure
- Small incision
- Summary: Anatomic reconstruction/reapproximation of the lateral
ankle ligaments with the use of the inferior extensor retinaculum for
augmentation.
Modified Brostrom Procedure – 1994
o 1994, JFAS, Paden, Stone & McGarry from PSL Denver CO
o Indications: Chronic lateral ankle instability
o Summary: anatomic reconstruction of the ATF and CF (possibly) that differs from the
original Brostrom in that 1) the ATF is incised during the capsular incision; 2) Mitek
anchors are used and 3) lateral talocalcaneal ligament is not used for augmentation of the
ATF.

Arthroscopic Primary Repair


- Intracapsular staple fixation to lateral aspect of talar dome
- Standard lateral and medial portal with a third portal directly over AFTL for staple
insertion
- Subchondral bone denudation must be obtained for staple to hold

Nilsonne – 1932
- Repair of ATF
- Detach PB at musculotendinous junction
- Suture in subperiosteal groove in lateral aspect of lateral malleolus
- Does not recreate normal anatomical orientation of ATF

Elmslie – 1934
- Repair of ATF and CF
- Tensor fascia lata graft
- Osseous canal in talar neck, lateral malleolus and lateral body of the
calcaneus

Hambly – 1945
- Repair of ATF and CF ligaments
- Split PL
- Suture to the lateral talar neck, pass through fibula anterior to posterior
and fixated to CF ligament insertion

Haig – 1950
- Summary: Used the bony attachment of the anterior inferior tibiofibular ligament to re-
inforce the ATF.

Evans – 1953
- Summary: Peroneus brevis cut proximally and musculotendinous
junction and passed through fibular drill hole from anterior-inferior
to posterior-superior. Tendon sutured to the posterior fibular
periosteum.

Winfield – 1953
- Summary: Peroneus brevis tendon through osseous fibular tunnel, passed through
calcaneal osseous tunnels and sutured to itself.

Pouzet – 1954
- Summary: Peroneus longus tendon cut proximally, routed through the fibula from
anterior to posterior and sutured back onto itself.
Watson – Jones – 1955
- Repairs ATF ligament
- Detach PB at musculotedinous junction
- Two holes in fibula in posterior superior direction
- Trephine hole in the lateral talar neck
- Route PB through superior fibular hole, talar trephine, inferior
fibular hole
- Suture tendon to itself

Lee – 1957
- ATF ligament repair
- Modification of Watson – Jones because of technical difficulty
and need for excessive tendon length
- One hole in fibula
- Tendon is entered posteriorly to anteriorly and sutured to PB and
PL tendons distally

Storen – 1959
- Summary: Two stages. First stage utilized the medial 1/3rd of the tendo Achilles to pass
posterior to anterior through the fibula and the talar neck from superior to inferior and
sutured to the bifurcate ligament. Second stage cut the medial 1/3rd of the tendo Achilles
at its attachment to the calcaneus and rerouted this proximally superficial to the peroneal
tendons and into the calcaneal attachment of the CF ligament.

Casting & Meunier – 1961


- Summary: Modification of the Lee. Peroneus brevis is rerouted in a more oblique fashion
through the fibula (posterior inferior to anterior superior).
Christman and Snook – 1969
- ATF and CF repair
- Modification of the Elmslie
- Split PB
- Trephine hole in lateral talar neck, hole in fibula, trephine in
lateral calcaneus
- PB through talar neck, through fibula anterior to posterior through
calcaneal trephine posterior to anterior
- Remaining tendon directed anteriorly and sutured to PB

Kelikian & Kelikian – 1985


- ATF and CF ligament
- Plantaris tendon
- Insertion intact
- Drill hole from CF ligament towards the plantaris, fibula from
anterior to posterior, trephine hole in lateral talar neck
- Plantaris passed form posterior to anterior lateral calcaneus,
through fibula posterior to anterior, through talar neck, through
fibula anterior to posterior suture on itself.
- 7 – 18% of population has absent plantaris
Split Peroneus Brevis Lateral Ankle Stabilization
- ATF and CF ligament
- Split PB at musculotedinous junction
- Subperiosteal channel created in talar neck
- Trephine hole anterior to posterior fibula and lateral calcaneal
wall
- Route tendon subperiosteally through talar neck, through
fibular trephine anterior to posterior, through calcaneal
trephine
- Suture tendon to itself

Common Procedures
- Watson-Jones, Evans, Christman & Snook are the most widely used procedures and have
a high rate of success
- Disadvantages:
• Do not place graft in anatomically correct position
• Loss of eversion & plantarflexion due to use of peroneus brevis
• Extensive surgical exposure

Post – Op Care
- 10 – 12 weeks of protection
- NWB BK cast 4 – 6 weeks
- WB BK cast 3 weeks
- Physical therapy modalities and strengthening following cast removal

Summary
- Repair of ATF ligament only
• Modified brostrom
• Nilsonne
• Evans
• Watson Jones
• Lee
- Debate of only one vs two ligaments repair
• ATF is the primary lateral stabilizer

- ATF and CF Repair


• Brostrom
• Elmslie
• Hambly
• Winfield
• Christman and Snook
• Kelikian and Kelikian
• SPBLAS
- Full Tendon Graft
• Nilsonne
• Evans
• Winfield
• Watson-Jones
• Lee
• Kelikian & Kelikian
Ankle Fractures
Oblique fracture: produced with combined loading of compression and torque. Usually a
45 degree fracture orientation. This is considered to be unstable and is usually fixed with
ORIF

Spiral fracture: this is always a type of torque fracture. It usually takes a 360 degree
course with a vertical connecting segment between the two opposing sharp spikes of the
fracture. This is also usually orientated at 45 degrees

Comminuted fracture: this suggests the presence of two or more fracture planes and has
at least three fragments. This is usually due to very high velocity loading, it is a high
energy failure with load concentration over a small area of the bone.

Eponyms for Fractures


Percival Pott (1768): Described a fibular fracture 2 – 3 inches proximal to distal aspect
fibula with a deltoid ligament fracture and an intact syndesmosis.
Now: bimalleolar fracture

Dupuytren (1819): Described a medial deltoid ligament tear or a medial malleolar


fracture with a fibular fracture 2.5 inches proximal to most distal aspect of the fibula with
diastasis always being present.

Maisonneuve (1840): The New House fracture. Fracture of the proximal 1/3rd of the
fibula.

LeFort (1886): Described a vertical fracture of the fibula due to SAD type forces.

Chaput – Tillaux (1872): In 1872 Tillaux described a medial deltoid injury or medial
malleolar fracture, a fibular fracture 2 inches proximal to the ankle joint, AND an
anterior lateral tibial tubercle fracture. In 1907, Chaput conducted laboratory studies of
this fracture which is why the fracture of the anterior lateral tibial tubercle contains both
names.

Wagstaffe (1873): Distal fibular fracture due to avulsion of the anterior inferior tibial-
fibular ligament. An SER type injury.

VonVolkman (1875): Originally described as an anterior lateral lip of the fibula fracture
due to avulsion of the anterior tibial-fibular ligament and interosseous membrane due to
PAB type forces. Now: posterior lateral tibial fracture

Cotton (1915): Described a posterior malleolar fracture with a medial malleolar fracture
and a fibular fracture just proximal to the joint level. Trimalleolar fracture.
Correlating Danis-Weber and Lauge-Hansen

Danis—Weber Lauge—Hansen Where to start the Lauge—Hansen


Category LH Clock Type
SAD Start lateral 1 – lateral ankle
ligament injury – or
– transverse fibular
A – fracture of fracture distal to the
fibula distal to the level of the ankle
level of the ankle joint
joint 2 – near vertical
fracture of the
medial malleolus
SER Start anterior and 1 – rupture of the
central anterior inferior
tibial-fibular
ligament
B – fracture of the 2 – fibular fracture
fibula originating at with posterior
the level of the spike
ankle joint 3 – rupture of the
posterior tibial –
fibular ligament
ALWAYS ask for a 4 – fracture of the
lateral film!!!!! medial malleolus
PAB Start medial 1 – deltoid ligament
rupture or medial
malleolar fracture
2 – anterior tib-fib
ligament rupture
with posterior
tibial fracture
3 – Danis Weber B
fracture
PER Start medial 1 – medial malleolar
fracture or a deltoid
ligament rupture
2 – anterior inferior
C – Fibular fracture tib-fib ligament
originating proximal rupture
to the ankle joint 3 – high fibular
mortise fracture
4 – posterior tib-fib
ligament rupture or
posterior malleolar
fracture
Questions Related to Ankle Fractures

1. What is the big difference between a SER type fracture and a PAB type fracture?
SER has a posterior fibular spike while a PAB has a posterior tibial fracture

2. What fragment do you reduce first during ORIF of an ankle fracture?


The fibular fracture

3. How do you perform Cotton’s test and what does it signify?


After fixating a fibular fracture, use a large bone hook to pull the fibula laterally. If
everything moves, the test is negative. If only the fibula moves, the test is positive and
indicates that the syndesmosis is torn or not intact. If this is the case, you would use a
syndesmotic screw or a mini tightrope to fix this problem.

4. What do the two words of the Lague-Hansen classification system signify?


The first word is the position of the foot at the time of injury.
The second word is the direction of the deforming force or the direction the talus moves
in the mortise.

5.What is the Danis-Weber classification system based on?


The anatomic position of the fibular fracture in relationship to the tib-fib syndesmosis

6. You are called to see a patient in the ER. The ER nurse tells you in a stressed out near
hysterical voice that this patient has a transverse fracture of the fibula and a near vertical
fracture of the medial malleolus. What type of fracture is she describing?
SER 2 or Danis Weber A

7. At what stage of the SER injury, would you see a Wagstaffe fracture?
SER 1

8.When does a posterior malleolar fracture need to be fixated?


If the fracture involves 25 – 30% of the articular surface

9. Apply Vassal’s principle as it applies to the ankle joint.


When you reduce and fixate the fibular fracture to the appropriate length, the talus should
fall back into the mortise. This doesn’t always work perfectly for the medial malleolus.

10. Who wrote the original article relating the displacement of the talus in the mortise and
the affect of this displacement on the ankle joint congruity? What did this article
conclude?
Ramsey and Hamilton. JBJS. 1976.
This article concluded that 1 mm of lateral displacement of the talus leads to a 42% reduction in
contact area of the ankle joint. Meaning: reduce your freaking fibular fracture.However, when
doing their cadaveric experiments, they removed all soft tissue from around the ankle joint and
did not allow the talus to compensate for being displaced. More recent studies have indicated that
the talus tends to move back into the mortise when compressed if allowed to do so.
AO Fixation
Stress risers: areas of concentrated stress which can predispose the segment to fracture. This can
be caused by screw holes, bone graft excision, entrance points of blood vessels, tumors or
iatrogenic damage to the bone.

AO Principles of Internal Fixation


atraumatic operative technique
accurate anatomic reduction
rigid internal compression fixation
early and pain free ROM

Dynamic v Static Interfragmentary Compression


Dynamic: achieved with the use of a tension band wire or tension band plate

Static: generated by ‘pre load’ of lag screws or tubular plates that prevent separation of two
surfaces as long as the functional load produced by limb performance is less than the preload
generated by the screws or plates. The compression is constant in nature. This can also be
accomplished by rigid external fixation.

Interfragmentary compression usually requires


multiple screws. Proximal anchor screw (perpendicular
to the cortex of the bone) and the distal compression
screw (perpendicular to the osteotomy site).

Pauwels principle (tension band principle): an imposed


load applied eccentrically is altered by application of a
‘band’ on the opposing side, canceling out bending and
altering it into pure compression. This principle is
dynamically created by functional limb loading. (ex
putting a plate on the tension side of a fracture)

Load screw technique: This technique is performed by using an offset (eccentric) drilled hole for
the initial screws used to fixation the plate to the fracture fragments. The plate is aligned on the
reduced fracture site, the thread hole is drilled at the far side of the plate hole or at the edge of the
hole farthest from the fracture site. As the screw is inserted, the head of the screw will seat itself
into the center of the hole in the plate by shifting the bone fragment in the direction of the
fracture. This motion is countered by a similar shift from the opposite fragment, and as the two
fragments are pushed toward each other, interfragmental compression is created. Only the initial
two screws adjacent to the fracture line are used as load screws. This is the most common
application of plate fixation used in the foot and ankle region. ½, 1/3, ¼, tubular plates have great
tensile strength but do not resist against bending. Application of a pate plus interfragmentary
compression with screw is a great way to created rigidity
within the fracture area.

Prebending the plate: This is used so the cortex opposite the


plate does not gap. The offset holes are drilled before the
plate is bent. The plate is bent slightly at its center. As the
load screws are inserted, the bone fragments are drawn up
against the undersurface of the plate, and the opposite
cortices come into contact before the surface of the fracture
that is adjacent to the plate. As the load screws are tightened,
the fragments are drawn closer together and the gap at the
adjacent cortex decreases. As the gap reduces, the plate is
bent back into a straightened position with even contact and
pressure existing along the entire fracture surface.

The load screw concept and the technique of prebending are


fundamental principles from the creation of axial
compression with plate fixation.

Types of Plates
Neutralization: Allowing the pathologic forces to be
‘transmitted’ through the implant (plate) rather than through
the fracture interface, thus the plate takes the strain rather
than the fracture (as in a neutralization plate for a fibular
fracture.) This is a protection plate.
The difference between a neutralization plate and a buttress
plate is the way in which the plate is used.

Buttressing: Used to ‘hold together’ a comminuted fracture. May times this is used in the
metaphyseal area of long bones since they fail in a more random fashion than cortical bone.

Dynamic compression plate: DCP incorporates the load screw technique with the added effect of
geometrically designed slots within the body of the fixation plate. This plate is significantly
thicker and stiffer than tension plates. DCP is suitable for a solitary fixation device for a
transverse fracture. There are individual DCP for use with 4.5 mm, 3.5 mm and 2.7 mm cortical
screws. The major problem with the DCP is an alteration in periosteal blood supply. This is due
to contact of the undersurface of the plate for extended periods of time. Also, these plates are
thick and may cause hardware irritation problems.

Limited Contact Dynamic Compression Plates (LC-DCP): These plates have grooves on the
underside of the plates so that the blood supply to the bone is not impaired.

Lag screws: When the screw is placed across the failure plane so that its threads do not contact
the near cortex side of the bone. Thus the threads of the screw will only purchase the far or distal
cortex and pull the two surfaces together when the undersurface of the screw head contacts the
proximal bone cortex. This is a principle and results from the use of the screw, not the design of
the screw.

Types of Internal Fixation


K wires
Monofilament wires
Steinmann pins
Intramedullary nails (rods)
Screws
Staples
Plates
Herbert Bone Screw
- headless screw that is threaded at both ends
- proximal threads have a smaller pitch than the distal threads
- this pitch difference provides compression between adjacent bony fragments
- screw diameter is 4 mm
- just need to tap the distal threads (proximal threads are self tapping)

Reese Arthrodesis Screw:


- design to provide greater compression across fusion site
- clockwise (right handed) screw threads proximally and counterclockwise (left handed)
threads distally

Plate Sizes:
semi tubular plate: fits 4.5 mm screws
one third tubular plate: fits 3.5 mm & 4.0 mm
one quarter tubular plate: fits 2.7 mm screw
posterior anti-glide plate
a. placed posteriorly on the fibula
b. indicated for Danis-Weber type B fracture
c. why? These fractures tend to displace posteriorly and superiorly
d. the plate will resist the displacement and provide some compression

External Fixation

7 Principles of Callus Distraction (some of these principles are relative to the particular
anatomical correction)
1. Preservation of blood supply
- atraumatic corticotomy
2. Rigid external fixation
3. Latency period before distraction
- usually 7 – 14 days; allows 1st stage of bone healing to begin
4. Steady rate of distraction
- about 1 mm / day
5. Ideal frequency of distraction
- about .25 mm per turn (4 turns per day)
6. WB on limb during distraction
7. Post distraction healing time
- for every 1 mm of lengthening you must rest for 1 week
General Advantages
• applicable to closed, open or limited open surgical techniques
• device can be conformed to the shape of the leg and foot which will allow
multifunctional correction
• allows acute or gradual lengthening
• can be adjusted post operatively
• can be stable enough to permit early weight bearing
General Disadvantages
• Disliked by patients
• Disliked by surgeons
• Lengthy treatment time
• Increase risk of infection compared to internal fixation (ex: pin tract infections)
Largest role in foot & ankle surgery is with pilon fractures
Types of External Fixators
Circular
a. Ilizarov frame
b. Taylor spatial frame
i. Six color coordinated knobs
ii. Radiographic parameters put into a computer
iii. Knobs are turned to a certain number each day according to the computer
calculations
iv. New device
Monolateral
a. Orthofix
b. Hidleburg
c. Hoffman
Hybrid
a. Combination of the ilizarov and monolateral frames

Functions of External Fixators


1. compression
2. distraction
a. new bone that is formed is called the regenerate
3. angulation
4. translation
5. rotation

Dynamimzation: process by where stress is gradually applied to the new bone that has formed so
that the trabecular bone will remodel according to Wolf’s law making the regenerate strong and
suitable for weight bearing without the device. This is initially done by loosening up the device
gradually. If the is not done the regenerate will fracture very easily.

Corticotomy (osteotomy) can be done differently


• Osteotome (classic iliazarov method)
• Pre-drilling and osteotome
• Gigli saw method

The Ilizarov method consists of passing numerous perpendicular wires through the bone at
regular intervals and attaching them to the external rings and bars to stabilize the bone segment.
This configuration makes it possible to move bone at a specific rate and frequency, while
simultaneously providing rigid fixation of the adjacent bone for ambulation.

The Ilizarov technique is called the callous distraction method (distraction histiogenesis). While
rigid stabilization of two ends of a long bone is provided, the bone itself may be lengthened to
correct a short extremity. Rapid healing takes place in the site from which the bone was stretched.
This technique requires only a few small holes in the skin large enough for the passage of a small
osteotome. This narrow, chisel-like tool is used with a mallet to cut circumferentially the cortex
of the bone. Such a corticotomy leaves the marrow space intact. The low tensile strength of this
portion of the bone allows it to be easily stretched. The blood supply to the segment is
‘transported’ 1 mm/day as the patient turns the screw on the device. As the cortex moves apart,
rapid cortical healing takes place. The results in new bone formation at the donor site and in a
longer extremity.
The basis for 1 mm/day is so the neurovascular bundles aren’t acutely stretched and so the
cortical bone can heal as distraction occurs.
Specific uses of external fixation in the lower extremity
• non unions
• trauma (pilon fractures, etc)
• osteomyelitis (useful when large amounts of bone is resected)
• lengthening (tibial, metatarsal)
• arthrodesis (ankle, subtalar)
- advantages: 1) longer lever arm than internal fixation; 2)post-op adjustability
• correct malunions
• severe deformities (clubfoot, etc)
• joint preservation
o re alignment of arthritic joints
• supramalleolar osteotomy to realign the ankle joint
o the anterolateral aspect of the ankle joint is the most loaded part and is usually
where arthritis will begin
o distraction of arthritis joints can decrease pain, though no studies have show that
articular cartilage will regrow
o CORA: center of rotation of angulation
 Osteotomy at this point will lead to a full realignment of the foot & ankle
Bone Grafting in the Foot and Ankle
Two basic types of bone grafts
Cancellous
• Used to fill smaller defects
• Cancellous bone typically has superior osteogenic potential
• Osteoinductive in nature; to some extent osteoconduction
• Large amount of viable cells and proteins that increase the bone induction potential
 Bone morphogenic protein (BMP) is present in bone matrix that acts as
an inductor substance
• Initially describe by Marshall Urist in the early 70s
• Now is being produced through recombinant gene technology
• They incorporate quicker than cortical graft
• They are radiodense as they heal
Cortical (structural)
• generally used to fill large defects or where structural segments of the WB axis of the RF
are missing (talus, calcaneus, tibia)
• marked higher structural strength
• osteoconductive in nature
• can be obtained from the iliac crest, proximal tibia, distal tibia and fibula
• after grafting, there is resportion of the graft during revascularization but some structural
integrity usually remains during the process of bone graft incorporation to allow the graft
to withstand the loads of weightbearing
• since some osteoclastic resorption must precede osteogenesis, cortical graft will appear
radiolucent
• slower to incorporate compared to cancellous graft

Osteoconductive: Osteo" means "bone". Osteoconduction refers to the ability of some materials
to serve as a scaffold on which bone cells can attach, migrate (meaning move or "crawl"), and
grow and divide. In this way, the bone healing response is "conducted" through the graft site, just
as we say that electricity is conducted through a wire. Osteogenic cells generally work much
better when they have a matrix or scaffold to attach to. Associated with the term creeping
substitution.

Osteoinductive: Induction of bone formation refers to the capacity of many normal chemicals in
the body to stimulate primitive "stem cells" or immature bone cells to grow and mature, forming
healthy bone tissue. Most, but not all of these signals are protein molecules called, as a group,
"peptide growth factors" or "cytokines". Many of these growth factors are present in normal
human bone. For this reason, methods have been developed to process human bone and prepare
bone matrix which retains the normal growth factors, but limit, if not eliminate, the risk of
transmitting diseases or viruses. "Allograft Bone" processed in this way is currently the only
approved way in which surgeons can use an osteoinductive stimulus.

Corticocancellous bone graft: combination of cortical and cancellous graft. This is the optimal
graft to use in the foot and ankle. Tricortical graft taken from the iliac crest is an example of this.

The choice of which bone graft to use depends on the biologic and mechanical properties of the
graft required.

Autograft: viable bone tissue taken from one area of the host and implanted into another
Allograft: non living bone graft from someone other than the host.
- only functions as a substrate for osteoconduction
- several types
• frozen allograft
• freeze dried allograft
• demineralized or decalcified allograft
• artificial allograft

Techniques of Bone Grafting


1. onlay bone graft technique
- uses large amount of autogenous cortical bone graft to bridge a non-union
- graft is fixated with screws or wires
- Phemister described an onlay graft which was not fixated
- this technique is inferior to other because it doesn’t provide rigidity in the area
2. inlay bone graft technique
- formation of a slot or window in which the bone graft is placed
3. sliding bone graft technique
- create a graft from the shaft of a long bone
- slide the graft across the non-union or arthrodesis site
4. papineau bone graft technique
- originally described for treatment of chronic osteomyelitis and infected non-unions
- involves three steps:
1. excision of necrotic bone
2. cancellous bone graft
3. skin coverage
- this technique was designed for rapid revascularization

Locations for autogenous bone graft


Iliac crest: corticocancellous and cancellous bone can be harvested from this area
Proximal tibia: excellent source for both cortical and cancellous bone; two locations are used:
a) over the tibial tuberosity and b)over the anterior medial tibia
Distal tibia: source of single cortical struts and cancellous bone; the amount of bone obtained
from the area is limited
Fibula: strictly used as a cortical strut; can be harvested as a vascular bone with the nutrient
artery and vein intact
Calcaneus: both cortical and cancellous bone can be obtained
Other sites: navicular tuberosity, medial eminence of the 1st MT, phalangeal bone from
arthroplastic procedures
Random Questions
1. Name as many classifications as you can for osteomyelitis as you can and explain them
Waldvogel
- hematogenous
- contiguous focus
- osteo secondary to PVD
- chronic

Cierny – Mader
Anatomic Stage Physiologic Stage
I: medullary A: good systemic defense & local vascularity
II: superifical B: systemic & local compromise
III: localized C: non candidate for surgery with tx increasing
IV: diffuse morbidity and mortality than non treatment

Buckholtz
7 surgical classifications for osteo
I: wound induced
II: mechanicogenic infection (implants)
III: physeal osteo
IV: ischemic limb disease
V: combination (I – IV as acute osteo)
VI: osteitis with septic arthritis
VII: chronic osteo

2. What are the 5 D’s for the treatment of infection?


Debridement
Decompressions
Drainage
Dressings
Drugs

3.What is the D-dimer assay used for?


This is a new way to detect PE and more recently DVT. D-dimer is a fragment that is
specific for the degradation of fibrin. (Barbe et al Am J Clin Path 1998)

4. Name 5 causes for bilateral edema.


CHF
Protein deficiency
Prolonged standing
Chronic venous valve insufficiency
Myedema
Drugs (corticosteroids, NSAIDs, BCP, Abx with increased sodium)
Pregnancy
5. Name 5 causes for unilateral edema.
Venous obstruction (DVT, post phlebotic syndrome)
Lymphatic obstruction
Infection
Trauma
Tumor
Previous surgery
Popliteal aneruysm
Charcot
AV fistula
Milroy’s disease (swelling secondary to lymphatic obstruction)
Filariasis
Baker’s cyst
Retroperitoneal fibrosis

6. Name the stages of wound healing.


Inflammatory: PMN predominate
Features – hemostasis, debridement, decontamination of the wound
Day 3 – 4
Fibroblastic or Proliferative: macrophages predominate
Features – collagen synthesis, neovascular formation, repithelialization
Day 3 – 4 to day 14 – 21
Maturation or Remodelling:
Features – remodeling and reorganization of collagen bundles; wound
contraction, increase tensile strength
Week 3 post injury to 1 yr post injury

7. What is Charnley’s description of closed reduction?


Recreate the deformity
Distract
Reduce the deformity and realign
Maintain correction with immobilization

8. What disease is the CREST syndrome associated with and what does it stand for?
Associated with Scleroderma (systemic sclerosis); this syndrome occurs in the
limited form of the disease
C: calcinosis
R: raynaud’s phemenon
E: esophygeal dysmotility
S: sclerodactyly
T: telangectasia

9. What are the phases of bony healing?


Inflammatory
Reparative
Remodelling
Secondary bone healing has been said to heal in six phases:
Hematoma formation
Hematoma organization
Fibrocartilagenous callus
Primary bone callus
Primary bone callus resorption
Remodelling (maturation)

10. Name the layers of skin.


Stratum basale (germinativum)
Stratum spinosum
Stratus granulosum
Stratus lucidum
Stratus corneum

11. What clinical conditions occur in each stage of syphilis?


Primary: chancre
Secondary: skin lesions that resemble many other diseases (the Great
Masquerader)
Tertiary: tabes dorsalis, CNS effects, neuropathy

12. What is the Jarisch-Herxheimer reaction?


This occurs when patients are being treated with antibiotics (Lyme disease). During the
initial 24 hours of therapy, the patient becomes sicker with worsening of the symptoms.
This is due to the massive destruction of the bacteria and the release of toxins into the
blood as the bacteria dies.
This can also occur in any treponemal infection (Syphillis)

13. Name 5 anaerobes.


Clostridium Bacteroides
Peptostrepococcus Peptococcus
Fusobacterium

14. If a pt is taking Rifampin and notice a red color in their urine, should they be
worried?
No, this is a normal side effect. Rifampin may turn body fluids red

15. Name 5 gas forming organisms.


Bacteroides Escherichia
Clostridium Klebsiella
Staph/Strep Serratia
Fusobacterium Peptostrepococcus
Peptococcus

16. Which side effect seen in aminoglycoside therapy is reversible?


Kidney function
17. What are the peaks and troughs for gentamycin and vanco and what do they tell you?
Gentamycin Trough – 2 Peak – 10
Vancomycin Trough – 10 Peak – 20

The peak gives information regarding the amount of the dosage


The trough gives information regarding the timing of the dose (frequency)

18. Name all of the beta-lactamase penicillins and their dosages


Augmentin (amoxicillin/clav) 250, 500, 875 mg PO (500 TID; 875 BID)
75 mg amoxicillin
125 mg clavulanic acid
Timentin (ticracillin/clav) 3.1 g IV q6-8h
3 g ticracillin
100 mg clavulanic acid
Unasyn (ampicillin/sulbactam) 1.5/3.0 g IV q6-8h
2:1 ratio
Zosyn (pipercillin/tazobactam) 3.375 IV q6h –or—4.5 g IV q8h
Pipercillin 3g
Tazobactam 0.375g

19. Name some treatments for MRSA and VRE.


MRSA: Vancomycin, Zyvox, Bactrim/Rifampin, Cipro/Rifampin, Minocycline
VRE: Zyvox, Syndercid, Amp/Aminoglycoside

20. What is the characteristic color pattern in Raynaud’s phenomenon?


White-blue-red

21. Explain the glucose tolerance test.


Pt ingests 75 g of oral glucose. Blood sugar is tested at 0, .5, 1, 1.5 and 2 hours
after ingesting. If the reading at 2 hrs and 1 other measurement is >200mg/dl then
diabetes can be diagnosed.

22. Name 5 Disease modifying anti-Rheumatic drugs (DMARDS)


Methotrexate
Gold salts
Penicilliamine
Axathioprine
Sulfasalazine
Leflunonide

23. What is the hallmark of ankylosing spondylitis?


Bilateral sacro-illitis (bamboo spine)

24. What is the half life of Tc-99, Gd and Indium?


Tc 99 – 6 hours Gd – 79 hours
Indium – 67 hours
25. What is the major distinction between Roussy-Levey Syndrome and CMT?
Essential tremor expressed primarily in the hands (RLS)

26. What are the three stages of RSD?


1. Acute: hyperemic; lasts 1 – 3 months
2. Dystrophic: ischemic; lasts 3 – 6 months
3. Atrophic: 6 – 9 months

27. What are the three types of biopsy?


- Shave: not commonly done
- Incisional: usually a punch biopsy; done to remove a section of the lesion; pick
the most aggressive appearing part of the lesion
- Excisional: removal of the entire lesion

28. What are the four main types of malignant melanoma?


- superficial spreading: the most common type
- lentigo: most commonly seen in older men
- acral lentiginous: see Hutchinson’s sign (color changes in the eponychium of
subungual melanomas): most common in non-caucasians
- nodular: the most malignant type

29. What are the three radiographic patterns of bone destruction?


Geographic
Moth eaten
Permeative

30. Name types of periosteal reaction.


Single layer
Onion skin
Codman’s triangle
Velvet
Sunburst
Perpendicular (hair on end)

31. 5 D/Dx for chest pain.


MI Aortic dissection
PE Aortic aneurysm
Angina Pericarditis
Heartburn Herpes Zoster
Pneumothorax Musculoskeletal pain
Cardiac tamponade

32. What is Felty’s syndrome?


RA, splenomegaly, neutropenia
33. Which specific ascending pathway is responsible for vibratory, pressure, light touch
and conscious proprioception?
Dorsal column medial lemniscus

34. What is responsible for pain, temp, light touch and itch?
Anterolateral spinothalamic tract
Social questions

1. What 5 adjectives would you use to describe yourself?


2. Tell me about yourself.
3. Why didn’t you choose a hard science major? Why <insert your major here>?
4. Why do you want to work here?
5. Why should we pick you over another applicant?
6. Are you married/single?
7. Do you have any questions?
8. Who is forcing you to become a podiatrist?
9. Why should we not take you for our program?
10. If we accepted you, how would you contribute?
11. Who do you respect most in podiatric medicine?
12. What do you know about me and my career?
13. What non-medical books have you read recently? Why those? What did you learn
from it?
14. Would you sleep with anyone to get this residency?
15. There’s room for only 3 residents in this program…why should we pick you?
16. What are you worst attributes?
17. How do you feel about us reviewing your qualifications tonight over a drink?
18. Have you visited our program/facility before? Why or why not?
19. What do friends say behind your back?
20. How would your best friend describe you?
21. Who did you vote for President? Do you think that was a good decision?
22. What is your strongest quality?
23. What current movies describe your approach to life?
24. When you’re not studying, how do you relax?
25. What’s your GPA? Are you satisfied with that?
26. What do you like best about podiatric medicine?
27. What do you like least about podiatric medicine?
28. What is your favorite podiatric subject? Explain.
29. With which recreational drugs have you ever experimented?
30. What do women do better than men?
31. What do men do better than women?
32. What makes our program special compared to others?
33. How much time do you spend with your family?
34. If you weren’t a podiatrist, what other career path would you have pursued?
35. Do you prefer working alone or as part of a team?
36. What was your last complete medical check-up and what did they find?
37. What new skills have you developed over the past year?
38. What epitaph would you want inscribed on your tombstone?
39. What is your favorite TV program and why?
40. What mistakes have you made in your life that you really regret?
41. What major trends do you see in our specialty over the next 2-4 years?
42. What’s your idea of success?
43. What trade or professional journals do you subscribe to?
44. If you won the lotto, how would you spend it?
45. Everybody has a skeleton in their closet. What’s yours?
46. What’s the nicest thing you’ve ever done in your life? What’s the meanest?
47. What’s the most memorable moment in your life?
48. What’s one accomplishment are you especially proud of?
49. Are you smart? Would your fellow students agree?
50. What is the most important thing your parents/family taught you?
51. If you could change one thing about yourself, what would it be?
52. What makes you special?
53. Rate yourself on a scale of 1 – 10.
54. How would you describe your character?
55. Why should I consider you a strong candidate for this residency program?
56. What can you do for us that someone else can’t?
57. What will your strengths be as a resident?
58. What characteristics do you possess that will allow you to fit in at this program?
59. What characteristics do you think residents need to have to be successful at this
program?
60. What do you want to be doing 5 years from now?
61. What are your most important long term objectives?
62. If you could change one thing about your personality what would it be?
63. Tell me about the one thing in your life of which you are proudest.
64. Tell me about the worst decision that you ever made.
65. Describe your work philosophy.
66. What does ‘success’ mean to you?
67. What does ‘failure’ mean to you?
68. With what extra-curricular activities were you involved? What made you choose
these activities?
69. Which ones did you enjoy the most? Why?
70. Which ones did you enjoy the least? Why?
71. Which ones do you regret not choosing? Why?
72. Why did you choose podiatry?
73. Which courses did you like the most?
74. Which courses did you like the least?
75. What other professions did you consider? And why did you choose one and reject the
others?
76. Where did you extern?
77. Tell us what you know about our hospital.
78. What did you learn from your externship here?
79. What have you learned at other externships?
80. Why did you pick those externships?
81. Why didn’t you visit this program?
82. Why are you applying to this program?
83. Are grades a good measure of ability?
84. What is your GPA?
85. What is your class rank?
86. Why didn’t you get better grades?
87. What is the most memorable accomplishment that you have had in your life?
88. What is the biggest failure that you have had in your life? What steps have you taken
to make sure that this haven’t happen again?
89. What is your greatest weakness?
90. What would you say is the biggest problem that you have failed to overcome?
91. Tell me about the type of people that you have trouble getting along with.
92. What types of people have trouble getting along with you?
93. What other two programs do you think are equal to ours?
94. Are you an organized person?
95. Paint me a picture of your current desk.
96. Do you manage your time well?
97. How do you handle a challenge?
98. How do you go about making important decisions?
99. Do you work well under pressure?
100. Tell me about the last time pressure led you to indecision, a poor decision or a
mistake. What would you have done differently? Have you found yourself in a similar
situation since? What did you do?
101. Do you anticipate problems or just react well to them?
102. Are you a risk taker or do you prefer to play it safe?
103. Tell me about the last time you took a risk. Was it the right decision? What would
you have done differently?
104. Do you plan on practicing with yourself or with others?
105. How you do get along with your superiors?
106. How do you generally handle conflict?
107. How do you behave when you have a conflict with a co-worker?
108. Tell me about the last time you lost your temper.
109. When was the last time you were upset? Why were you so upset?
110. How do you motivate people?
111. Tell me about the last time that you made a good decision.
112. Tell me about the last time that you failed to complete a project.
113. Tell me about the last time that you found a unique decision to a problem.
114. Tell me about the last time that you aimed too high.
115. Tell me about the last time that you aimed too low.
116. What do you do when you have trouble solving a problem?
117. What do you do when things are slow?
118. What do you do when things are hectic?
119. What do you do when you are burned out?
120. What do you do when you have multiple priorities?
121. What are the skills that you most need to acquire/develop?
122. What have your superiors criticized most about you?
123. How do you ‘stay in the loop’?
124. Have you received offers from other programs? If no, why not? If yes, are you
willing to decline those offers to accept an offer from our program?
125. How do your classmates describe you?
126. The successful candidate for this residency program will be working with people
who have been with this program for a long time, how will you mesh with them?
127. If you are unfairly criticized by an attending, what would you do?
128. Would you like to be a residency director? Why or why not?
129. What do you know about our residency program?
130. What was the last paper that was published out of this program?
131. What have you heard about this residency?
132. What aspect of this residency excites you the least?
133. What is the most important problem in podiatry today as you see it?
134. Are you in good health? What do you do to stay in shape?
135. Do you have any physical problems that will affect your ability to perform?
136. How do you manage to balance your career and family?
137. What turns you on in your off hours?
138. What do you like to do when you are not at work?
139. What is the last movie that you saw?
140. What is the last CD that you bought?
141. Aren’t you a little old to be a resident?
142. Why did it take you so long to get interested in medicine?
THINGS I SHOULD KNOW OZ

Gram + Ancef (Staph/Strep/Bacilli) Vanco


Gram – Gentamycin (Ecoli) Supramax
Anaerobe Clindamycin Flagyl

Two Abx to give for OM: Clinda and Cipro – two of the best bone penetrators + coverage for all bugs;
then consult Infectious D.

Augmentin (Amoxcillin + Clavulonic acid)


Timentin (Ticaricillin + Clavulonic acid)
Unasyn (Ampicillin + sublactam)
Zosyn (Pipercillin + Tazobactam)
Bactrim (Sulfamethoxazole + trimethoprim) in a 5:1 ratio

Ancef 1G (Prophylaxis), Keflex 1G (Nail), Cefepime 4G (Serious/Pseudomonas – fruity smell)

Clindamycin – Pseudomembranous colitis – Vanco PO or Flagyl PO

MRSA Killers: Vanco/Bactrim/Linezolid/Clinda (if comm. Acquired)/Primaxin/Daptomycin/Tygacil

Principles of AO fixation: Stable internal fixation, Anatomic reduction, early return to ADL, atraumatic
technique >>> WHY? To prevent cast disease/disuse syndrome – arthritis, osteopenia

Axis of evil forces at fixation site: bending, torsion, shear


Primary bone healing is what we want, secondary = callus formation

Mini frag set (1.5, 2.0, 2.7) Small (3.5, 4.0) Large (4.5,6.5)
Bunion screw 4.0: No OD, 2.5UD, 2.7Countersink, 4.0Tap
1.5 screw UD1.1, 2.0 screw UD1.5, 2.7screw UD2.0

Capsacin – blocks substance P – for neuropathic patient who is having a lot of pain

Toradol (IM/IV NSAID), Tramadol (abuser opioid like). Strength: Hydro then Oxy
So: Morphine – Hydromorphone (dilaudid) – oxymorphone, then Hydrocodone (vicodin)- Oxycodone,
then Darvocet (Propoxphene + acetaminophen)
Vicodin (hydrocodone + acetominphen)

Ester (PABA – allergies), Amide (Liver breakdown)

Transfuse patient if <7/21or 6/18


FFP for DIC/Liver issues/Warfarin aka Coumadin <NOT Heparin>

INR: Surgery pt (1.0), Normal pt (0.8-1.2), DVT pt (2.0-3.0) – PT/Vit K/ F2,7,9,10


Intrinsic pathway – injury contact platelet and starts this pathway – PTT heparin (↑Antithrombin III)
Extrinsic pathway – (more important) fibrin produced – PT – Coumadin aka warfarin
Higher the INR = bleeder, Lower = clotter
↑INR – Coumadin, ASA, Abx (NSAID has no affect)
↓ INR – vitamin K, birth control, barbtituates (NSAID has no affect)
D-dimer test: measures fibrin – to DX DVT
THINGS I SHOULD KNOW OZ

Lovenox, Fragmin – low molecular weight Heparin – prophylaxis for DVT

Heparin for PTT – 5000U load, 1000U maintain – Protamine Sulfate to reverse

Ibuprofen (proprionic a), ASA (salicylates), Indomethacin (indole), Celebrex (COX2inhib)

Lidocaine 4mg/kg – 15cc (2%) or 30cc (1%) depending on %.


Marcaine 2.5mg/kg – 35cc (0.5%) 70cc (0.25%) depending on %

Lidocaine w/epi – 25cc or 50cc depending on %


Marcaine w/epi – 45cc or 90cc depending on %

1cc has 10mg of Lidocaine/Marcaine

Nausea/Vomit: Zofran (ondasetron), Dexamethasone, promethazine (antihistamine)


Diarrhea: Codeine
Constipation: MOM, Metamucil, Doculax, Docusate, colace, WALK PATIENT WALK!

Vancomycin is renal toxic! Hence peaks/trough: draw 30 minutes before or after the 3rd admin dose
Peak – Shouldn’t be 30+, Trough – Shouldn’t be more than 10+ <more important number> + keep em
hydrated with IV fluids

GAS produces: BECKS Bacillus, Enterococc/Ecoli, Clostridium, Klebsiella, Staph/Strep

Albumin should be 3.0 or your flap/sx won’t heal. Lab value lags 7 days – give ensure/protein powder

Anion gap = (Na+K) – (Cl + bicarbonate) – if more than 15 = DKA

High power (Dexamethasone, Betamethasone) Medium (Medrol dose pack) Low (Hydrocortisone)
Decadron = dexamethasone

DKA (Diabetic ketoacidosis) – extreme high glucose – no more insulin available: body switches to
burning fat = ketone bodies. Symptoms: Kussmaul breathing, vomit, nausea, confusion, dehydration
Diagnose via BMP (acetone in blood), ABG (= acidosis Allen test), UA (ketone bodies, glucose spill too)
Treat via: IV FLUIDS + INSULIN + K + BICARBONATE
THINGS I SHOULD KNOW OZ

Not shown are A and P perforating arteries! Rmr the the posterior ones are out of sync since the first
one is actually the deep plantar!
THINGS I SHOULD KNOW OZ
THINGS I SHOULD KNOW OZ

Tibial nerve: LPN and MPN, and M calcaneal nerve


Sural nerve: Lat. Dorsal cutaneous and L calcaneal nerve

Note: malleolar fossa on fibula: Sup: inferior transverse ligament, Inf: PTFL
Deltoid ligament:
Superficial – tib-na, tib-calc, SUPERFICIAL POSTERIOR tib-talar
Deep – DEEP POSTERIOR tib-talar, ANTERIOR tib-talar

Spinal stenosis = pseduo claudication


Monckenberg = calcifications w/o claudication = not occlusive!

Fibromuscular dysplasia = SM in BV screwed = kidney HTN~!

Trental - increase RBC flexibility + antiplatelet agg


Pletal - antiplatelet agg

TAO - thromboangiitis obliterans = inflammation of A and V! = smoker (Burgeurs disease) = 40 yo MALE


Corkscrew arteries (collaterals) seen in beurger disease (TAO)

Raynauds phenomenon - pain due to cold/stress pallor>cyanosis>rubor

DIC disseminated intravasc coagulopathy: widespread activation of coag system = emboli + bleed
treat w/ two things: infuse plates/plasma + IV heparin!

TCPO2 >50mmHg = good


THINGS I SHOULD KNOW OZ

Huge ulcers on the medial aspect of the leg - secondary to venous insuff > edema > skin looses nutrients
= ulcer!

KENDALL PUMP - pumps up - for venous insuff edema and elephantitis

Homan's sign - DF foot = pain cuz of DVT

IV HEPARIN - PTT - 5000units to load, 1000 per hour thereafter 4-6days REVERSE VIA protamine
sulphate: half life is 1.5 hours
Coumadin - VIT K factors: 7,9,10 - 10mg qd 6months REVERSE VIA vitK or FFP(faster reversal): half life is
20-60 hours (change dose as per INR)
Streptokinase or Urokinase

embolus: air/fat/shit = fracture


thrombus: plaque

Ischemia6Ps: polar, pulseless, pallor,paresthesia,poikliothermia

Venous stasis ulcers: fibrin cuff - barrier to O2 diffusion

GLYCOSYLATION of skin = loose flexibility

Normal albumin = 3.5 / total protein = 7

Symes amputation = amputation at malleoli!

AV fistula = thrill/bruit at auscultation

Visatril - antiemetic + opioid booster

Bone healing FIVE stages:


(1 and 2) initial inflammation/hematoma = decreased o2 + acidity = promotes osteoprogenitors to
differentiate
(3) neovascularization
(4) Clasts come in and remodel
(5) remodel

All American Procedure (Flatfoot/Severe PTTD): TAL, Medial displacement calc osteotomy, CCjoint
lenghthening(Evans), FHL or FDL transfer to the PT
PTTD = RF Valgus = TendoAchilles pulls more valgus onto it!

Syndesmotic lig complex: ATFL, PTFL, Interosseous, inf transv tibfib = syndesmotic injury!
Syndesmotic lig injury repair: debride the whole area, mitek to recreate ATFL, Syndesmotic screw
thrown across tib and fib

Spring ligament = calcaneo-navicular ligament

Intramedulary rod/nailing - big ass rod in medulla <infection rate is high>


THINGS I SHOULD KNOW OZ

Stevens–Johnson syndrome - autoimmune/hypersensitivity epidermis seperates from the dermis


by Bactrim (Sulfamethoxazole and trimethoprim)

Floroquin and/or steroid = collagen necrosis followed by repair = tendon rupture

3-wea (wetting, emulsifying, antiseptic)

Keratoderma - horn like skin

Grafts:
Real: Autologous (self), Allogenic (same species - apli and derma), xenogenic (porcine)
DERMAGRAFT: Dermis + fibroblasts <foreskin>
APLIGRAF: Living skin/epidermis
Synthetic: Collagen, hylauronic acid

Allopurinol: Xanthine oxidase inhibitor (uric acid producer)


Colchicine: raises pH, anti-inflamm properties, inhibit MT polymerization

FHB: origon: M (from TP), L(3rd Cunei and Cuboid)

Extensor Hallucis Capsularis/Peronues Digiti Minimi/ Woods muscle <From abdDM - inserts into 5th
base>

Third plantar common digital branch <MPN> is where you get Mortons neuromas (often comm with
Plantar common DB off LPN)

CFL only ankle extracapsular ligament <attaches to peroneal spine on calc>

I - Randomized dbl-blind controlled, systematic/meta analysis


II - Cohort
III - Case
IV - animal research, expert opinion

Macular- circumscribed, flat lesion


Papular/nodular- circumscribed, solid elevation
Vesicular/bullous- fluid-filled, elevated lesion
Cystic- non-infected, deep-set collection surrounded by a wall of tissue
Scales- thin flakes of dead epidermis
Excoriations- scratch marks
Erosions- essentially thick scratches that do not breach the dermis
Ulcer- deep dermal defect that breaches the dermis or deeper
Fissures- linear, deep epidermal cracks in the skin
Pustule- elevated lesion containing pus, arising from infections of papule or bulla
Abscess- deep collection of pus

Arterial ulcers: lateral leg (or at trauma site – IPJ) – Lots of pain, no exhudate
THINGS I SHOULD KNOW OZ

Venous ulcers: medial – little pain, lots of exhudate

1) ABI: 0.9-0.7 IC, 0.7-0.3 Rest pain, 0.3-0 Gangrene


2) Segmental pressures: no drop more than 20mmHg, plus all waveforms triphasic - These are taken at
upper thigh, lower thigh, upper calf and ankle. 10-20-30 rule
* The upper thigh pressure should be greater than 10mmHg greater than brachial pressure
* Pressure differences of more than 20 mmHg for adjacent cuff sites on the same leg
* Pressure differences of more than 30 mmHg over the entire leg.
3) Doppler – should be triphasic
4) Digital blood pressure – should be 70-110mmHg
5) PPG - Photoelectric plethysmography- light intensity of blood = norm is 50mmHg
6) TCPO2 –

0 = no movement
1 = trace movement
2 = movement with the aid of resistance
3 = movement against gravity
4 = movement against resistance supplied by the examiner
5 = normal strength

Types of Neuropathy
D - diabetic
A - alcohol
N - nutritional
G - Guillan-Barre

T - toxic
H - hereditary
R - recurrent
A - amyloidosis
P - porphyria
I - infectious
S - systemic
T - tumor

CRPS: pain is distal to the site of trauma to begin with but can spread proximally in later stages.
Vasodilation can exhibit warm skin, dry skin, and hypohidrosis. Vasoconstriction can exhibit cyanosis,
cool skin, edema of the part, and hyperhidrosis. Trophic changes involving the skin, muscles, bone and
joints can occur in later stages. The examiner may see thin, atrophic skin, absent of hair and witness
osteoporotic changes on x-ray.

Neuromas (a misnomer) of the forefoot:


Joplin’s - medial to the first metatarsal
Hauser’s - the first intermetatarsal space
Heuter’s - the second intermetatarsal space
Morton’s - the third intermetatarsal space
Islen’s - the fourth intermetatarsal space
THINGS I SHOULD KNOW OZ

ASA1 – normal patient


ASA2 – Systemic disease w/o manifestation (DM)
ASA3 – Systemic disases w/ manifestation (DM w/ neuropathy)
ASA4 –Systemic disease that is a threat to life (Renal failure)
ASA5 – SuperObese Cannot survive w/o operation, emergency
ASA6 – Brain dead, lets harvest organs

Stages of anesthesia:
I – Analgesia/Amnesia (pre, partial, total)
II – Delirium (excitement etc)
III – Surgical Anesthesia
IV – Medullary paralysis

Ester (pseudocholinesterase – Procaine), Amides (Lidocaine, Marcaine)

Malignant hyperthermia
Thought to be due to reduction in the reuptake of Ca by the sarcoplasmic reticulum necessary for the
termination of muscle contraction
Dantrolene 2.5 mg/kg IV initially – 175mg for 70kgmale
Also do ice bath, bicarbonate, insulin

Fracture:
1. Open vs. closed
2. Shape
3. Location
4. Intra-articular vs. extra-articular
5. Complete vs. incomplete
6. Displaced vs. non-displaced

Gustillo and Anderson:


I <1cm, clean, minor damage
II >1cm, moderate damage
III extensive damage and dirty (comminutated, crushed, skin missing, avulsion)
a enough ST coverage to fracture site
b not enough ST coverage
c blood supply screwed!

General Principles of Treatment of Open Fractures

1. Tetanus history
2. Thorough H&P with complete neurological, vascular and musculoskeletal exam
3. Complete x-rays
4. Antibiosis admitted in the ED with cultures
5. Immediate debridement and irrigation
6. All foreign bodies should be removed
7. All marginal and macerated tissue should be debrided and re-debrided as necessary after 48-72
hours.
8. Types 1, 2 and 3A get delayed primary closure within 5-7 days
THINGS I SHOULD KNOW OZ

9. Types 3B and 3C usually require skin flaps due to such a loss of tissue
10. External fixation for all type 3 and unstable type 2 fractures
11. Internal fixation should be used for articular and metaphyseal open fractures, preferably
within 8 hours.

Tetanus:

Clean and minor wounds:


Toxoid given only if last dose was given > 10 years ago.
TIG not given

All other wounds:


Toxoid given if last dose given > 5 years ago
TIG not given

TIG is only given in patients (any age) when they have never received a tetanus shot (usually pediatric
patients)

Compartment syndrome: normal 5mmHg, exercise 150mmHg (returns to normal in 5 min), 30+ = with
wick catheder

Pre-op patients: CBC (for H/H) + EKG + CXR + HCG or UCG

Insulin REDUCES potassium levels, Steroids RAISE Glucose levels.

PRE-OP: DM PATIENT
Early morning Sx is best. If not start D5W.
Want glucose 150-200. Don’t take meds if <200. If 200+ take insulin.
Minor procedure: can d/c oral meds for the day of surgery.
For major procedures: ½ insulin dose on the morning of. Start D5W during case. And then give 1-2 units
in surgery.

If patient is taking more than 7.5mg of steroid per day – cannot d/c steroid! Give 100mg IV
hydrocortisone pre op and then 100mg post op.
Steroid: poor wound healing, infection.

Heparin therapy: d/c day before sx


Coumadin therapy: d/c 5-7 days before sx (20-60 hour half life!!)

Heart attack? No surgery for another 6 months

Gout patient for sx: give Colchicine 0.5mg TID for 5 days before, 5 days after Sx

5W’s:
Wind (1 day post-op)- aspiration
Water (2 days post-op)- UTI
Wound (3 days post-op)- infection
Walk (4 days post-op)- DVT
THINGS I SHOULD KNOW OZ

Wonder (> 4 days)- drug reaction

Screw is either: positional (ie holding the plate) OR is a lag screw (compressing – only if proximal threads
don’t purchase – ie overdrill)

Plate types:
Static compression: normal compression
Dynamic compression: static + physiological stress = more compression <has the sliding screw or
ECCENTRIC DRILLING>
Neutralization plate: fracture first fixated via a lag-screw, then this plate is put on on top – to neutralize
3 evil forces: bending, torsion, shear
Buttress plate: maintains length in case of big defect

Note if the fracture length is TWICE that of the bone: use multiple lag screws!

Tagaderm: to cover ulcer in OR

PCA example: 4mg initial dose of Morphine. 1mg per 20 minute, with 10 minute lockout etc.
D/C <8 breaths per min or systolic is <90mmHg

Etomidate – Short term IV anesthetic for bedside ORIF

Lidocaine is acidic (lower pKa) = burns, Marcaine is buffered

MRI w/ contrast: used at: tendonopathy or fluid lesion

Pre-Op: NPO + cardiac clearance + defib company rep in OR!

Talus supplied by: PT (A of tarsal canal), DP (A of sinus tarsi), Perf peroneal: neck branches

Lateral ankle ligaments: CFL, ATFL, PTFL, and Lateral Talo-Calc ligament

Master knot of henry connects FHL and FDB to the navicular

Sural nerve = medial sural cutaneous nerve + comm. Branch from lateral sural cutaneous nerve
THINGS I SHOULD KNOW OZ

Nail and tip of toes are mostly by the plantar proper digital arteries

Interossei: 4XDAB, 3XPAD

Stopped at #10 on the first page


THINGS I SHOULD KNOW OZ

DMC lecture
Chronic wound definition: reduction in 50% area in 4 weeks (has not happened)
>65N/m2 = ulcer since O2 tension is reduced
TCPO2 30mmHg = good O2 – do it to determine amputation level
MMPs – not good since they deactivate GF in wound
Calciphylaxis is a syndrome of vascular calcification, thrombosis and skin necrosis (@ ESRD patient)
Pyoderma gangrenosum – huge deep ulcers that are due to auto-immune diseases
Dakin solution – bleach + water + baking soda
Santyl – collegnase/self debrider
Regranex – GF paste
Apligraf – living epidermis (ALL GF) – after application don’t mess with it for one week
Dermagraft: dermis + fibroblasts (MISSING SOME GF)
Aquacell: absorbs exhudate + has silver in it

Uncontrolled diabetic is >12Hb1Ac


Residency Q: if start with a Wagner 3, and treat and it gets smaller: what is it now? Still a Wagner 3
Diabetic:
Non-fasting > 200mg/dl
Fasting > 126
Hb1Ac >6.5% (pre-diabetic 5.7 – 6.4)

Glitazones – cause EDEMA

Diabetic Drugs:
Metformin – STANDARD DRUG – ↓ carb resportion + ↑ insulin sensitivity (WRONG)
Glitazone - ↑ insulin sensiƟvity
Acarbose - ↓ carb resporƟon
Meglitinide - ↑ Bcells to secrete insuline
Sulfonyureas -↑ Bcells to secrete insuline
GLP analog – Glucagon like peptide – give sensation of being full
Dpp4i inhibitor – DPP4 break down GLP

First drug will lower Hb1ac by 1.5, second 0.7, third 0.5

Types of insulin: (short term) R, NPH, L, UL (long term) – Old method


NEW: RAA or LAA rapid or Long acting analop

LISPRO (Humalog) RAA – take with meals 5 units


GLARGINE (Lantus) LAA – take at night time 20 units

Sliding scale – controversial – basically see what patients glucose level is BEFORE meal and then give
them units of insulin. However post-food glucose may be huge! Therefore trial and error method – give
5 units Lispro, see post-food glucose! If reached 900, try 10 units etc etc

All surgery patients need to maintain 140-180 in the ICU.


Note COPD patients take prednisone! = raise glucose and decrease insulin sensi! Therefore give em
more insulin!
THINGS I SHOULD KNOW OZ

D5W = 5% glucose = 50mg over 8 hours.

BUN and Creatinine Story


BUN Blood urea nitrogen – filtered and REABSORBED
Creatinine – only filtered
Normal ratio 10-20:1 BUN:Creatinine
↑>20:1 Acute renal fail, GI bleed, old person (low muscle mass) – PRE-Renal problem (BUN resorb high)
↓<10:1 Liver disease – Renal damage (No resorb of BUN)

Before Abx: kidney function, allergies, C+S, other abx

Primaxin = imipenem + cilastatin (protect kidney via inhibiting enz that break down imipenem)
= Gorillamycin since broadest spectrum
SE: Seizures!

PCN allergy: use CLINDA or CIPRO (cross sensitivity of PCN and CEF is 1-5%)

Augmentin: 500 or 875mg, both have 125mg of Clauvulonic acid

Timentin: raises Na levels!!!

All Abx renal excreted: minus: Pipercillin aka ZOSYN


Liver excreted: Clinda and Erythromycin

1st Gen Ancef(IV), Keflex(PO) -G+ and G- (Peckss)


2nd Gen Ceclor -G+ and G- (Hen Peckss)
3rd Gen Rocephin -G+ and G- and Pseudomonas
4th Gen Cefepime (aka Maxipime) -G+ and G- and Pseudomonas

Hinflu, Neisseria, Proteus, Ecoli, Klebsiella, Shigella,Salmonella

MRSE: Meth Resis Stap Epidermidis

Vanco IV 1g q12 slow infuse(over 1 hour) or get red-neck syndrome (Red man syndrome = Rifampin)
Tx Redman/neck with Benadryl
Peak is high(lower dose), low(higher dose). Trough is high(increase interval), low(lower interval)
Peak30/Trough10

Vanco SE: Ototox, Nephrotox, Redneck

Aminoglycosides: Tobra/Gentamycin (G- Lover) 2mg/kg load, 4mg/kg maintain


SE: IRREVERSIBLE OTOTOXCITIY
Peak10/Trough2

Kidneyfuction: CC Creatinine clearance = (140)(weight in KG)/(72)(Creatinine)


*0.85 if female
Bactrim (Sulfamethaxole + Trimethoprim) – one tab PO BID – both kill folic acid production
THINGS I SHOULD KNOW OZ

Bone penetration: Clinda + Flouroquinolones

Bactrim contraindicated in DM patients with PO hypoglycemic

Z pack aka Azithromycin – 250mg tab, 2 for first day, 1 each for 4 days

Aztreonams: for pseudomonas – but super expensive

Quinolones: Cipro/Levaquin – NOT FOR KIDS, Kills Cartilage!

Diarrhea – Pseudo colitis (by Clostridium difficile) OR regular colitis (by S Aureus)
TX with Vanco PO 125mg or Flagyl PO 500mg

VRE – TX with Linezolid

Pseudomonas KILLERS: Cefepime, Zosyn, Timentin, Cipro, Gentamycin

MRSA KILLERS:
IV – Vanco, Clinda, Linezolid, Primaxin, Bactrim, Rifampin
PO – Clinda, Linezolid, Bactrim, Rifampin
Topical - Bactroban

Bactroban: isolated from Pseudomonas!!!

Surgical prophylaxis: 2+hour surgery, implant, AIDS pt, Emergency/trauma, endocarditis

ABx – change flora of GI  reduces Vit K absorption  increases INR

Fever: mostly between 4pm and 8pm

Augmentin for BITES


Enterococcus: Augmentin!

G+ and G- 1st choice: Keflex or Ancef, 2nd choice: Clinda or Cipro


Anaerobe – 1st choice: Augmentin, 2nd choice: Clinda or Genta

C. Perfingen (GAS): clinda or augmentin

Xanthoma – Bactrim
Lyme disease – Doxcyyline
Necrotizing fasciitis – Primaxin

Gas gangrene = deadly since: kidney fail + shock

28 bones in the foot, 35 joints, 107 ligaments, 19 muscles


THINGS I SHOULD KNOW OZ

Remember the difference between Os Subtibiales and Os TibExternum

Sharpey fibers: attach periosteum to bone

Strongest lateral ankle ligament: Posterior Talo-fib

Deepest medial ankle ligament: posterior-Tibio-talar

Flexor retinaculum aka Laciniate ligament


SERetinaculum aka Transverse Crural Ligament
IERetinaculum aka Crural Cruciate ligament

MPN – courses in 1st layer muscles: between ABD H and FDB


LPN – courses between 1st and 2nd layer: between FDB and QP

MEDIAL of foot – EHL – Deep peroneal nerve – Dorsalis Pedis artery – LATERAL of foot

Pes Anseurinus – Semitendinous, Gracilis, Sartorius (longest muscle in body)

DTML is deeper or dorsal to neuroma

Circulation coming down from the heart: AA to common iliac to external iliac to femoral to popliteal - …

NEVER LAFF at A FAD

MPN: 1st lumbricle, ABD H, FDB, FHB


MPA: FDB, ABD H, 1st dorsal interossei

Remember the triangle: Talus


Calcaneus Navicular
Most symptomatic is CN Coalition

3 causes for brachymetatarsia: Down syndrome, Polio, HEP B, Turners

Hallux varus is post-fibular sesamoid removal

Saggital groove is an articulation for the tibial sesamoid

Skin graft healing stages: plasmatic (nutrient thru plasma), neovascular, reorganize, reinnevate
Wound healing stages: inflamm, proliferative, remodel
Bone graft stages: vascularization, blastic, osteoinduc, osteoconduc, remodel

Complications of grafting: #1: seroma, #2: hematoma, #3: autoimm

Primary bone heal: inflamm, osteoinduc, remodel


Secondary bone heal: inflamm, osteoinduc, soft callus, hard callus, remodel
Tailor’s bunion: IM 6 (9+ not good), Lateral deviation 8+ not good
Haglunds: fowler phillip, PPP, total angle of Vega
THINGS I SHOULD KNOW OZ

Von Recklinghausen aka Neurofibromatosis


Kayexalate for hyperkalemia

Hollow Mill: if you strip the head of a screw, use this – its drills out both screw and surrounding bone

Charcot vs. OM: how to diagnose?


Easy option: BIOPSY IT!!
Difficult option: 3 phase bone scan with Indium 111 (more specific for OM) however very long half life!

When to use Gadilinium for MRI?


Neuroma (will be darkened)
Fibroma (will light up – especially important pre-surg since want to know extent)

Hemophiliac arthritis: Similar to PVNS – hemosiderin deposits in joints.

GIANT CELL TUMOR: MOST LIKELY tumor or tendon sheath

MRI: PT tendon is about 1.5-2X larger than the FDL tendon


MRI: Peroneus Quartus = mass laterally! Inserts on Calc tubercule
MRI: Accessory Soleus = mass posteriorly!
MRI: no tendon sheaths for anterior tendons, TA, distal PT
MRI: Fatty infiltrates in ABD H = Tarsal tunnel syndrome
MRI: Fatty infiltrates in ABD digi quinti = Plantar fasciitis <> if not = baxter neuritis
MRI: FHL tendon sheath communicates with Ankle joint

DVT: compressible vs. non-compressible vein


Compressible: No DVT
Non-compressible: your probe is ON the DVT and the vein wont collapse!
DVT: Augmentation: compress the foot and blood goes gushing by! Therefore no DVT!

Iontophoresis: Therapeutic ultra sound w/steroid cream (supposed to make the steroid absorb deeper)

Discoid meniscus: the lateral meniscus of the knee isn’t C shaped, and does taper, instead its thick (rare
anamoly)

ADA diet: low carb, AHA diet: low Na

Vibrio Vulfincus: fresh water bacteria

4 grams is max does of Tylenol – therefore Vicodin(500/5): can only take 8 in a day!

Integra: flowable wound matrix


Promogran: collagen dressing (wet it first)

Valium for muscle spasm – IE: TA lengthening!


THINGS I SHOULD KNOW OZ

DVT: Least you can do is ASA 81mg


Heparin SE: (within 5 days) HIT heparin induced thrombocytopenia! – basically heparin+platelet is
thought of as a foreign body and is attacked by immune system = more and more thrombi! = MI or CVA
Tx: Coumadin/discontinue heparin?

ARGATROBAN: treatment of HIT


Don’t forget: streptokinase: dissolves clot (nothing else dissolves clots!) Also TPA (tissue plasminogen
activators – TP breaks down clots naturally)

Coumadin/Warfarin SE: Warfarin necrosis – skin necrosis! BLACK. Associated with high loading dose,
This is because Coumadin initially increases coagulation (paradoxical) – this is because Coumadin also
kills protein C!!!!!

Coumadin kills Vit k associated factors AND Protein C. (Rmr Protein C is also an anti-coagulator!)

Pyoderma Grangrenosum: Auto-immune disease = deep ulcers w/ papules

Necrotizing Fasciitis: C Perf, S Aureus, Vibrio Vulf

LMA – Laryngeal mask airway – like General, except no intubation, more like a triangular mask over
larynx

Opiate strengths: Morphine then HYDRO then OXY


Morphine > Hydromorphone (dilaudid) > Oxymorphone > Hydrocodone (vicodin)- Oxycodone> Darvocet
(Propoxphene + acetaminophen)
Vicodin (hydrocodone + acetominphen)
Darvocet (Propoxphene + acetaminophen)

Malalignment syndrome – keep it in mind, pelvis. Always look 2 joints above!

Caffeine increases uptake of codeine

Osteoporosis: Bisphospanates (Fosamax for example) Stops Clasts


1200 units of Ca, 2000 units of D
Or can do Zoledronic acid IV injections = ONCE A YEAR!

Qhs = Take nightly!

ORIL – open reduce insert lock (IM aka INTRAMEDULARRY ROD)

Voltaren – NSAID cream (PO is higher then normal)


Sulindac 200mg PO BID – easy on the kidneys

OTO-sil: silicone impression system for corns within interspaces.

Verrucae plantaris: wart on plantar aspect


Verrucae vulgaris: wart on dorsal aspect
THINGS I SHOULD KNOW OZ

Onychomycosis: DDx may include: Lichen planus

Monckenberg or plaques: BIG BV killed by DM


Microangiopathy: Small BV killed by DM <ie eye, kidney>

Plantar fasciosis: BAD BAD Plantar fasciitis


Ultrasound thickness of P Fasciia: normal <3mm, >6mm is 

Prolotherapy: inject with sugar water or PRP to instigate inflammation

Liver enzymes:
ALT – high also due to musc break down
Creatinine Kinase – high due to MI (CK-MB more specific)
Alk phosp – high due to biliary duct blockage

Sporanox aka itraconazole – PO antifungal take 1 day 7 days daily, then 3 weeks off, then 7 days, then 3
weeks off, then 7 days

Monsel’s solution – use for hemostasis in P+A

Corti-sporin: ophthalmic solution used for P+A, I+D

Reverse flow gradient: @ Monckenberg sclerosis Pressures: 100, 212, 104 = back flow!

CHEST PAIN – MONA


Morphine (reduce pain, reduce breaths per min)
Oxygen (2L nasal canula)
Nitroglycerin Slingual
Aspirin

Also get CXR(so its not a lung thing) and EKG and ABG (pulmonary gases, Allen test)
Draw a MI Screen (troponin, CK-MB)

Poly-ol pathway and neuropathy:


Hyperglycemia = ↑polyol pathway  ↓NADPH  ↓NO  vasocon  ischemia
Hyperglycemia = ↑polyol pathway  ↓NADPH = sorbitol accumulaƟon
Hyperglycemia = ↑polyol pathway  ↓NADPH oxidative stress (as NADPH is needed in
Gluthatione reductase)

NOTE: NADPH used up in converting Glucose to Sorbitol


THINGS I SHOULD KNOW OZ

↑ levels of homocysteine = KIDNEY, EYES, NEUROPATHY (retinopathy, glomerulopathy)


B Vitamins lowers levels of homocysteine
Cymbalta – neuropathy (ANTI-DEPRESSANT)
Lyrica – neuropathy (pain kill)
Metanx – Vitamin B6 B12 and folate

Transfusion reaction: IGE mediated (Benadryl), Fever (Tylenol) – Stop if bronchospams


Store blood = Citrate toxicity

Give one unit over FOUR hours (this is for our patients, if emergency bleeding = give bolus/over one
hour etc)
99% patients: 2 units over 8 hours – Do loop diuretic in between (so then can pee ins and outs)
LASIX = FUROSEMIDE

packedRBC raises hemoglobin by one 1%, hematocrit by 3%

Types of transfusions:
Whole blood – never given, give em what they need
packedRBC – most common
Albumin – increases volume only (wont help with healing)
Platelet – to increase PT/PTT/INR for bleeder surgery
FFP – for Coumadin
Cryoprecipitate – (=fibrinogen) for VonWilder disease

10/30 is norm, transfure when <7/21

150-500K is normal platelet. 100K is minimum for Sx. <50 is = spontaneous bleed

STJ: 42T 48F 16S

Bohlers angle: top of anterior process to top of calc post facet to top of calc tuberosity
Norm: 25-40
Gissane angle: (strut) ant and middle facet compared
Norm: 120 – changes with calcaneal fractures
Talo-calcaneal angle (on DP): 21
Talar declination angle: 21
Calc inclincation angle: 21
Note talar head is 15 degrees medial, and 25 degrees plantarflexed

Cyma line: S shape line formed by CC and TN joints (supination moves it post, pron moves it anteriorly)

Splay foot: increased IM angles for all (12+ for 1st and 2nd, 8+ between 4th and 5th)
THINGS I SHOULD KNOW OZ

Toygar angle: usually achilles is 180 (its straight), when ruptured, it would be less

Surgical Dissection: <further names in the Crozer-Keystone manual>

Keller – base removed


Mayo – head removed @ surgical neck - capsule <which is distal to anatomical neck-collats>

Distal Akin – hallux interphalangeus


Proximal Akin - DASA
Oblique Akin – long PP

Silver – bumpectomy + lateral release (Add H, FDB, DTML cut, lat capsulotomy, freeing fib sesamoid)
Mcbride – Silver + fibular sesmoidectomy

Head
Note True IM = Measured IM + (TAA -15)
IM 12-14 head, 15-17 shaft, 18+ base
THINGS I SHOULD KNOW OZ

Austin – <
Youngswick – < with extra dorsal cut
Kalish – long dorsal arm
Reverdin – thru n thru wedge
Reverdin Green – w/shelf
Reverdin Laird – thru n thru
Reverdin Laird Todd – thru n thru and PF fragment

Shaft
Hohman – trapezoid
Vogler – long dorsal arm
Mau/Ludloff

Base
Juvara – thru n thru
CWBO/OWBO

Watermann – wedge taken out at head (base dorsal)


Jacobi V – V looking DP
Weil – straight cut into MT, slide head back

Keck n Kelly – Wedge removed, base dorsal in calc

MT adductus procedures:
Heyman Herdon Strong – cut off all tarsal-MT ligaments (except 5th-Cuboid)
Bertman Gartland – cresenteric osteotomies of all MT

Flatfoot procedures:
All American – Evans + TAL + Calc slide + FHL to TP
ST procedures <KIDS>
Young – TAL + TA re-rout thru nav
Hoke – Nac-cunei fusion
Arthroeresis <KIDS> “lift joint”
MBA – SELF LOCKING – keeps the lateral process of talus off the sinus tarsi
STA-PEG - AXIS ALTERING – eleveates the floor of the sinus tarsi
Mushroom – IMPACT REDUCER – reduces impact
Osteotomies
Evans – Insert bone in CC joint
Dwyer – remove wedge, base is medial
Silver – insert wedge, into lateral
Koutsugiannis – cresenteric

Cavus procedures:

ST procedures
THINGS I SHOULD KNOW OZ

Steindler stripping
Jones tenosuspension
STATT
Peroneal stop – PL taken out via, suturing to PB, therefore more eversion

Osteotomies
Triple/STJ/AJ fusion
Dwyer – opposite dwyer, wedge taken out, base medial
Cole – wedge taken out at cuneis and cuboid, base is dorsal
Japas – A V (looking DP) cut made and whole FF PFed

Hallux limitus:
Jones Tenosuspension
STATT

Lateral ankle stabilization:


Brostrom: debride + tighten ligaments + Use IER to reinforce

Watson-Jones – fibula and neck


Evans - fibula
Lee – Split PB thru fibula

<Dbl ligament> Chrisman Snook split PB thru fib and calc

TAL
Hoke – 3 small incisions 2 medial 1 lateral
Baker tongue in groove – tongue is on the distal
Frontal plane Z #1

Silverskoid Test:
Limited DF @ both - GASTRO-SOLEAL tight: do Frontal plane Z
Limited DF @ knee extended - GASTROC tight: Baker tongue in groove

OATS procedure: for talar OCD – remove bone w/ cart from knee
THINGS I SHOULD KNOW OZ

NON ABSORBABLE
Ethilon - poly proplene
Prolene - nylon

ABSORBABLE
Vicryl - Lasts 4 weeks
Monocryl - Lasts 2 weeks

RARE/Less commonly used


PDS – absorbable – Lasts 6 months
Chromic gut – absorbable – Lasts 3 months
Dexon - absorbable
Silk - nonabsorbable
THINGS I SHOULD KNOW OZ
THINGS I SHOULD KNOW OZ

Classification systems:

Danis Weber
A = SAD
B = SER, PAB
C = PER, PDF

Note: PDF = describes a Pilon Fx

Wagstaffe: Fibula avulsed ant


Tillaux-Chaput: tib ant
Volksman: tib post

Reudi + Allgower: Pilon fx – 1(no displacement) 2(intra-artic) 3(comminuted + intra-artic)


AO for Pilon: extra/partial/complete intra-articular
A no comm/no impact, B some, c both yes

Sneppen: Talar body – 1(OCD) 2(body) 3(post tubercule) 4(lateral process) 5(crush)

Essex-Loperesti: 1(tongue fx) 2(joint depression)


Tongue fx: 2 fx lines, one goes D to P, other exits posteriorly
Joint depression: same as above: plus line around STJ

Johnson + Strom:
I mild degen
II attenuation
III rupture
IV rigid valgus

Conti (MRI)
I longitudinal tears
II variable degen
III diffuse swelling

Note: Stewart: Mostly Jones (so type 1), Other most common is either 2 or 3 (cuz PB avulsion – 2Intra,
3Extra)

JHASS = dorsal dislocation of PP and sesmoids (in all categories)


1 intersesm lig intact
2 not intact A(no fx), B(sesm fx)

Rosenthal I(distal to PP), II(distal to lunula), III(PP)

Drago, Olaff, Jacob


I pain end ROM
II adapt - flatten
III deteriorate – osteophytes/sclerosis
IV ankylosis
THINGS I SHOULD KNOW OZ

Etiologies of Tailors Bunion:


Incomplete development of DTML, malinsertion of add hallucis, FF splay, Congenital bowing of MT,
abnormal STJ supination

Wound Care Products: <some products skipped from G document>


All Silver products need to be removed before MRI
Can change almost everything every 3 days
MMPs – inactivated cuz sucked up into ie promogran

Necrotic Wound Debrider:


Accuzyme (papaine-urea)
Panafil (Papaine-urea-chlorophill) Chloro for smell/swelling
Santyl - debrider

Antimicrobial dressing:
Acticoat – Silver 100ppm for 3 days
Actisorb – Charcoal and silver
Aquacell – Aquacell Ag contains Silver
Hydrofera Blue – Gentian Violet/Methylene Blue

Compression dressings:
Unna boot
Pro-Fore (5 layers: contact layer, webril, bandage, elastic compression, Coban)
Soft cast: Webril/Unna/Coban – Passive compression only (only when they walk)

Non-Antimicrobial dressings:
Iodosorb – beads with Iodine – bacteria and debris become trapped in it also – therefore wash
wound then changing dressing with iodosorb application last time. 3x a week. Contra:
allergy/hashimoto/goiter
Prisma/Promogran – matrix/collagen
Oasis – ECM
Regranex – Platelet derived GF
Xeroform – gauze + Vaseline + abx salt

Grafts:
Alloderm - collagen
Dermagraft – dermis/fibroblast
Apligraft – living epidermis
Graft Jacket – human dermis

High K – CBIGKFAL – Calcium/Bicarb/Insuline/Glucose/K-axelate/Furosemide/Albuterol


THINGS I SHOULD KNOW OZ

Normal Lab Ranges:

Admit after Sx:


ADC VAANDIMAX
Admit to s/p
Diagnoses
Condition
Vitals
Activity
Allergies
Nursing order
Diet
IV
Meds
Ancillary – PT/Vascular
X-ray
Same Day Sx:
Admit to PACU
s/p L 5th MT ORIF
Please dispense Sx Shoe
Non WB w/ crutches
Elevate and Ice L LE
D/C per anesthesia
F/U with Dr. X on 2/3/21
Rx Percocet 5/325
THINGS I SHOULD KNOW OZ

TBI = toe brachial index – more accurate than ABI for foot and calcified arteries. Normal value 0.7

Prevalone boot – decubitus heel ulcers


Soft cast – webril, unna, coban

Amitryptilyline – TCA for neuropathy

Prominent calcaneal exostosis on the lateral aspect: peroneal tubercule

Hba1c: 120mg/dl = 6, add 30 for each above

Nickel allergy = allergy to stainless steel and/or titanium

Titanium screw is weaker than stainless, however it is more elastic, and less scatter on MRI
Titanium plates = best, since want some flexibility or it will break

Lesser MT:
head procedure = may cut nutrient a.
shaft procedure = no blood supply
base procedure = too much correction

Bunion re-surgery rate = 5% for Austin


Bunion infection rate <1%

Tight rope for bunion – biggest problem: 2nd MT stress Fx (since you have a hole in the 2nd head), will
loosen up, will rupture = fiberwire

We are born adductovarus  become adductovalgus

MT adductus is NON-CONTRIBUTORY to HAV!

Shaffer plate = arch support

Ped devices: C = equal flanges = UCBL = Anti Splay


A = M flange = Anti-Pron
Whitman = cause pain at navic tuberosity at pronation

Hollow Mill = Trephine

Neural tube = cover both ends of the cut nerve to connect a cut nerve

Bipolar cautery – no need to ground patient, spark only to the vessel


Monopolar cautery – need to ground patient, whole local area sparked
THINGS I SHOULD KNOW OZ

Double crush syndrome – same nerve damaged twice at two different spots – rmr neuropathic nerves
are swelled up, hence tarsal tunnel + back injury + neuroma etc

TRANSMALLEOLAR SCREWS: need to be removed in 12WEEKS (since motion between tibia and fibule)
otherwise will break
Neuropathy: (1) Demylinate via sorbitol (2) myelin production is down since need microvessels (3)
Osmotic effect (hyperglycemia cuz nerve loves sugar – water comes/follows in) (4) oxidative stress

Tyloma = HPK0

Thompson test – squeeze the calf for TA rupture


Pratt test – squeeze the calf to test for DVT
Hohman test – DF foot for DVT

For ankle Fx: fix diastasis first and then the LM or MM fx

ESR: 100+ Acute OM, <100 Chronic OM


>60 Acute OM, 30-60 Chronic, 0-15 Normal

Sepsis if 2+ of the following are raised: Temp, PR, RR, WBC

Vassal rule: Fixate the most unstable fx first (the others will take care of themselves)

Vertical stress test: test for subluxable MTPJ – hold MT and PP and try to push up the proximal phalanx

Plantar fasciitis is always on LEFT – since right arm and left leg are dominant and longer

Torsion: head n neck vs fem chondyles


Version: head n neck vs frontal plane

Antetorsion - int
Retrotorsion -ext

Anteversion -ext
Retroversion –int

Note that physis in kids that are compressed, grow faster!

In-toeing caused by (1) MT adductus (2) Femoral retroversion (3) Antetorsion (4) Int tib torsion (5) Int
malleolar position
THINGS I SHOULD KNOW OZ

Hip externally rotate 2.5degrees every 10 years.

Constipation can cause low grade fever!

Young – FHL to Navicular

Fault at Nav-Cunei? Fuse it!


Orthoses is to change GRF, not to correct deformity

UCBL <6yo, Whitman 6-12yo

SPRING LIGAMENT ALWAYS RIPS IN FLATFOOT

TAL = reduces strength by 60%

Bony union 6-8w, bone heals in 18months

Wolf law: bone under stress will adapt


Davis law: ST under stress will adapt

Metaphyseal fx – ie 5th base – need WB to heal! Cast for 6 weeks (normal cast time 8w)

Fx heal: adult – 8w, 12yo = adult, 6yo 4w, 2yo 2week!

Physeal fx are at calcification zone

OCD that is painful, and casting wont help: retrograde drilling

Fragmin – low molecular weight heparin

Abx not used on wounds, cuz caustic


Abx beads = 90 days leach
Abx beads taken out in 2 weeks (unless absorbable)
Bone is NEGATIVELY charged – S.Aures sticks to it!!!!

Ring sign: on X-ray for chronic OM


Heat labile Abx = Clinda

Charcot OM Arthritis
T1 Normal Decreased Normal
T2Increased Increased Increased

PMNA Poly-methyl-acrylate-beads

Lateral process of the talus fracture = shepherd fx


Medial process of the talus fracture = Cedell fx
THINGS I SHOULD KNOW OZ

30 to 70 degree angle for arthroscopy (between camera and cutting tool)

Remember that Sural nerve is formed by: M sural cutaneous (+ comm. Branch off L sural cutaneous)

Distal syndesmotic tib-fib ligament: FOUR


AITF, PITF, Interosseous, Transverse Tib-fib ligament

Medial ankle ligament: FIVE


Tib-Calc, Tib-Nav, Tib-Talus, Also: deep deltoid (deep tibio-calc) is deeper to tibio-calc (Sustentaculum
Tali)

AVN – Possible treatment options: Core decompression, Vascularized bone graft, non-vascularized bone
graft, fusion (talo-calc, talo-tib, blair), talectomy. ALSO: percutaneous drilling w/Steinman pin

Krischner wire diameter(1, 1.5, 2, 2.5, 3mm)


STEINMAN PIN diameter(3, 3.5, 4, 4.5mm)

Piezoelectric phenomenon: Negative Charge @ Compression = bone deposition (Thus Bone stim – apply
cathode at where you want bone to be deposited!)

Vanco and all other Abx cause Neutropenia: Daptomycin is the least causing of em all – good for OM
Tx for Neutropenia is G-CSF

True MTa = (Measured MT – 15) + IM

Dermatome:
Sciatic nerve: Ventral Rami of L4, L5, S1, S2, S3
Tibial: Ventral division of L4, L5, S1, S2, S3
Common Peroneal: Dorsal division of L4, L5, S1, S2
THINGS I SHOULD KNOW OZ

Charcot vs. OM: Certec labeled Tech99


Lachman test = vertical push up test = for predislocation syndrome
Reudi and Allgower (Pilon)
I (no comm/no displacement) II (no comm/some disp) III(comminuted of joint and displacement)

Neuropathy drugs:
Metanx: Vit B12 supplements, Nuerontin: Gabapentin, Lyrica: Pre-Gaba, Cymbalta: depression

3:1 skin rule for incisions


Creatinine increase: Kidney failed, BUN decreased: Liver failed

Eccentric and Concentric drilling.


DCP: only 2 holes nearest to fx are eccentric

Hooks law – amount of strain = stress on bone


Pauwels law – eccentric load = compression and tension on oppo sides.

Charnley fx stability classification: (I: transverse most stable, II oblique most unstable, III short oblique
<45 degrees medium stability)

Coumadin dosages 2.5, 5, 7.5, 10


Close reduce: 3mg of Dilaudid, 3mg of Versed, 10mg of Zofran (Etomidate by ER doc)

Bone healing stages:


Primary infl, repair, remodel
Secondary hematoma(1 week), soft callus(3m), hard callus(6m), remodel
Skin graft healing stages: plasmatic (nutrient thru plasma), neovascular, reorganize, reinnevate
Wound healing stages: inflamm, proliferative, remodel
Bone graft stages: vascularization, blastic, osteoinduc, osteoconduc, remodel

Codeine allergy: STUD: Toradol 10mg IV/IM, Ultram 100mg, Darvocet N-50/100

HDS proper technique: plasty PIPJ, hood release, tendon lengthen, capsulotomy, pp release,
girdlestone/taylor and desis

FF procedures:
Young – TA rerouted thru Navicular
Kidner – Rid of accessory navicular and reattach

Porta Pedis: Roof QP, Floor ABD H

STRESS-Sheilding: decresed stresses cause osteopenia since screw is bearing weight

Lemont’s nerve: interm DC


Baxter’s nerve: Nerve to the abdDM
THINGS I SHOULD KNOW OZ

Thompson test – squeeze the calf for TA


Pratt test – squeeze the calf to test for DVT
Hohman test – DF foot for DVT

A- Asymmetry, B-border, C-color, D-diameter, E-elevation


Types: Superficial spreading (most common), acral lentigous (hutchingson sign), nodular

True AVN’s:
Koehlers, Friedburg, Assman, Diaz, Leg-Calve-Perthes

AVN stages: Avasc, Revasc, bony healing, residual deformity


X-ray finding on each: none, head within head and increase density, more bone deposited, residual

CRPS stages:
(1) Hyperemic – Pain+edema+hyperhydrosis
(2) Dystrophy – more pain+brawny edema+spotty penia
(3) Atrophic - less pain+diffuse penia+flexure contractures (volksman)

Pagets: Osteolytic>combined>osteoblastic>Malignant degeneration(6%)

Gout stages:
Asymptomatic: elevating levels of uric a
Acute: WOAH! Deposit = pain
Intercritical: inbetween acute attachs
Chronic tophaceous: 10 years+

Treat Charcot @ early stage 1 (right after dislocations/fractures) – cuz right after = osteopenia
Or Stage 3: new bone laid down and no more inflammation

Etiologies of HAV:
Biomechanical (equinus, pronation, hypermobility, FF varus, DFFR, Hallux limitus)
More Biomechanical (long 1st MT, MPE)
Arthritic (psoriatic, gouty, rheumatoid)
Others (Ehler-Danos, CMT, MS, CP)

Shaft osteotomies: Mau/Ludloff, Lambrinudi(wedge taken out with apex prox-dorsal and base disal-
plantar –PF the head!), Kalish(shaft vogler)

3 fragments of Calcaneus: constant(S. Tali), Comet(post facet), main fragment(tuberosity)

Reudi and Allgower – I (mild disp and comm) II (mod disp) III (severe disp and comm, AJ impacted)
AO – I extra-artic, II partial, III complete intra-art

1/3 tubular plate for fibula, 3 screws each side of fx


T-plate for tibia

Hawkins % 12/42/91/91
THINGS I SHOULD KNOW OZ

Posterior tubercule fx of talus – Watson


Normal, enlarged, os trigonum, syn-chondrosis

Lateral – Hawkin
Simple, comm, chip

Sanders CT: Coronal


Charnley = fracture stability

Tillaux fragment @ Salter-Harris 3

Sedden: nerve neuropraxia, axontmesis, neurotmesis

Eckert and Davis: Peroneal subluxation – retinaculum/fibrous tip/bone

AVN stages: avasc, revasc, bony healing, residual

French theory: neurovascular


German: neurotrauma

Avascular (0-3months) – ST swelling only/smaller epiphysis/illusional inc in density


Revascular (1-3years) – very fragile!/increase in density as epiphysis collapses on itself head within head
Remodel – bony deposition
Residual –

Raynauds phenomenon = Pallor, Cyanosis, Rubor

Stratum basale, spinosum, granulosum, lucidum, corneum

Shyphillis: primary: chancre, secondary: great masquredar/mimicks others, tertiary: tabes dorsalis/CNS

Peak and trough is mg/L

Malignant Melanoma – Superficial spreading (common), Acral lentigous, Lentigo, nodular(deadly)

Bone destruction patterns: geographic, moth eaten, permeative


OZ Interview Scenario List
Patient walks into ER with a DFU w/Puss, what do you do?

1. History and Physical


• History
o NLDOCAT
o Ask about Systemic SOI – N/C/V/F
o Local ulcer site SOI – Pus/Odour/swelling/erythema/pain
o Ulcer history
o ABx history
o Tetanus status
o NPO status
o Meds and Allergies
o PMH – Get DM history in detail
o Others PSH, FH, SH, ROS
• Physical
o Vitals
o Know how to work up non-palpable pulses
2. Start empiric ABx
• Local SOI (augmentin), Cellulitis (Zosyn), Bone (Clinda/Cipro), Systemic
(Vanco/Fortaz/Flagyl or Ancef/Gent/Clinda), Life threatening (Primaxin)
3. Order Labs
• CBCwdiff/CMP/ESR/CRP/Albumin/a1C – know why for each
4. Order 3 views X-rays, possibly MRI if severe (need for Sx planning)
5. Depending on NPO status schedule OR time for I+D
• Don’t forget things like consent/Fluid management/What drugs to DC/EKG/UCG/CXR
• Cultures
o Wound/ C+S
o Bone
• Ulcer = defect, hence cannot be closed, purpose is to clean it out. Then schedule follow
up visits in office/wound clinic for wound healing.
Patient with Trauma presents to the ER, what do you do?

Primary
• ABCD
• Tetanus
• NPO
• Is it polytrauma (Calc fx – get spinal and pelvic films)
• NVS of your LE (for compartment syndrome etc)

Secondary
• History and Physical
• X-rays/CT/MRI/Bone scans
• Labs

Surgery as appropriate
o Open Fracture
o Compartment syndrome
o AJ
o Pilon
o Calc
o LisFranc
o Talus
o MT fracture
o Tendon
o LawnMower
o Sesamoid
o GSW
o Nec Fasciitis
o FB

Open Fx:
Primary and Secondary survey
C+S
Empiric Abx
Debride in OR, Stabilize the fracture (temporary)
Repeat debridement in 48-72hrs for G+A 3
Stabilize ORI (permanent)
Abx start per C+S
Delayed primary closure 5-7 days @ Type 1 and 2
Closed fracture
 Check NV status
 Perform local block
 Close reduce (increase deform6ity, distract, decrease deformity)

Compartment Syndrome
Remove all dressings/casts
Dependence, hydrate, Nasal O2
Fasciotimize
DPC
(most important: pain, second most: Wick)

GSW
Velocity more important than mass
High velocity (military >2000ft/s) – Radical debridement due to cavitation
Low velocity (<2000 ft/s)
Do primary and secondary survey (earlier in document)
Fracture: consider it open, treat per G+A
Otherwise debride, remove bullet if superficial or palpable
Small wound closed, others left open with C+S and prophylactic Abx

ST trauma
Do primary and secondary survey
I+D, Remove FB (if applicable)
Control bleeding with Surgicel or Thrombin pad.
Ensure H+H fro EBL
Deep cultures always
DPC (delayed primary closure)

Lawn mower/nail/Rosenthal
Do primary and secondary survey
Treat per Rosenthal
Rosenthal I (let it granulate or primary closure if <6hrs) II(V-Y or Z skin plasty) III (distal symes amp)
If <25% subungual hematoma (trephinate it), if >25% nail avulsion + suture nail bed if its torn w/ abs
suture
Don’t forget to take xrays with any subungual hematoma. I know you know this though.

Puncture wounds/FB
Do primary and secondary survey
May triangulate FB position with THREE needles (one in each plane)
Xeroradiograph is excellent for FB (wood) – now out of date
I+D and removal
Three things to ensure did not happen: OM, Cellulitis, Abscess
DPC w/ packing + deep cultures + Empiric Abx

Human/dog bite (Cats don’t bite, they are gentle creatures (unless pissed off))
Bugs looking for: E. corrodens/P. Multicoda/Rabies/Tetanus
Augmentin 875 q12 (not the 500mg)
Follow puncture guidelines: I+D, pack, DPC and Deep cultures
*human bite also includes punching someone in the face, and their teeth opening skin on your fist

Burns and Frost bites


Remove agent
Cool w/ NS, Rewarm with 104 degree Saline
Do primary and secondary survey
Debridements everyday (depending on area involved)
Escharectomy in burn patient (once eschar tissue is widespread)
Blisters: do not pop, instead apply silvadene
If amputation is required: allow skin to demarcate
*Could not find article for prophylaxis (Presby says: NO! don’t want yeast/pseudomonas superinfection)

Calcaneus fracture
Do primary and secondary survey
Surgerize intra-articular fx only (75% of all) – can do traction initially
Remember Essex-Loperesti technique for tongue type
Schanz pin into tuberosity. Pull proximal for length, plantarward to DF the P. Facet, evert to take out of
valgus
Otherwise: always pin or screw comet fragment to constant fragment

Pilon fracture
Remember the four steps: restore fib length, reconstruct distal tibial articulation, graft tibia, restore
medial aspect of tib

Ankle fracture
LM: place that inter-frag screw first (2.7mm) AP (pointing plantarly). Then throw a 1/3 neutralizing
plate on the lateral side (6 hole, 3.5mm), ensure one hole is not screwed, do cottons test and screw in
4.5malleolar screw if need be
MM: 2 crossed k-wires or 2 4.0 screws
PM: usually not ORIF’d (unless greater than 25% articular involvement)

LisFranc fracture
Remember to re-create the lisfranc ligament

JHASS dislocation
Don’t forget that type one needs to be OPEN reduced!

Patient walks into your office with:


HAV
HDS
HL/HR
Cavus
Flat

ALWAYS TRY YOUR CONSERVATIVE CARE FIRST


ABPS 420

Examination Study Guide

American Board of Podiatric Surgery


445 Fillmore Street
San Francisco, CA 94117-3404
(415) 553-7800

April 2009
ABPS 420 (April 2009) Page 1

Table of Contents
Page

Introduction .................................................................................................................................. 2
How to Study for ABPS Examinations ........................................................................................ 3
Written Examinations .................................................................................................................. 4
General Information .................................................................................................................. 4
Format .......................................................................................................................................... 4
Subject Areas ............................................................................................................................ 4
Board Qualified and Recertification in Foot Surgery ............................................................... 5
Board Qualified and Recertification in Reconstructive Rearfoot/Ankle Surgery ..................... 5
Self-Assessment in Foot and Ankle Surgery ............................................................................ 6
Oral Examinations........................................................................................................................ 6
General Information .................................................................................................................. 6
Certification in Foot Surgery .................................................................................................... 8
Certification in Reconstructive Rearfoot/Ankle Surgery .......................................................... 8
Sample Written Questions ......................................................................................................... 10
Foot Surgery............................................................................................................................ 10
Reconstructive Rearfoot/Ankle Surgery ................................................................................. 25
Answer Key ............................................................................................................................ 30
Suggested Reading List .............................................................................................................. 31

Copyright 2009 ABPS Any unauthorized use, modification, reproduction, display, publication, performance, or
distribution of this work without the prior written consent of the ABPS is prohibited and may violate the Copyright
Act, 17 U.S.C. § 101 et seq. Such prohibited actions may result in legal action or the loss of privileges, benefits,
qualifications, or certifications granted by the ABPS.

U:\DOCS\ABPS Documents History\420\2009-ABPS420_09_v1.doc 4/29/09

Nondiscrimination policy. In accordance with applicable federal laws, the American Board of Podiatric Surgery does not discriminate in
any of its policies, procedures, or practices on the basis of race, color, national origin, sex, sexual orientation, age, or disability.

Americans with Disabilities Act. In compliance with the Americans With Disabilities Act, the American Board of Podiatric Surgery will
make special provisions for individuals with disabilities if written request is made no later than thirty (30) days prior to the date(s) of the
examination for which appropriate application has been made, fees paid, and credentialing completed.
ABPS 420 (April 2009) Page 2

Introduction

While obtaining board certification is an important step in the career of every podiatric surgeon, board
status is also important to hospitals and the general public. The profession of podiatric medicine has
chosen to use the certification process and its examinations component to indicate that a board-certified
podiatrist has demonstrated a level of knowledge appropriate to her/his specialty area.

This Study Guide is provided to assist candidates in their preparation for the examinations of the
American Board of Podiatric Surgery (ABPS). ABPS uses both written, oral, and Computer Based
Patient Simulation (CBPS) examinations in its assessment of candidates' knowledge of podiatric surgery.
While this guide is designed to represent the types of questions used in certification examinations, the
examination content is subject to changes that reflect current technology and medical practice.

The Examinations Committee, in consultation with the ABPS’ independent psychometric consultant,
prepares the examinations. Scoring, interpretation of scores, and determination of passing scores are the
responsibility of the Board of Directors.

Candidates are tested in specific subject areas applicable to the practice of podiatric medicine and
surgery and the perioperative care of the podiatric surgical patient. The written parts of the
examinations use objectively scored (i.e., multiple-choice, true-false, etc.) questions to test declarative
knowledge. The written examination may be adaptive or linear and will be administered by computer at
testing centers throughout the United States. The oral examinations employ face-to-face interaction
between the examiner and the candidate to test the candidate's ability to obtain and interpret clinical
information, to reason logically, and to arrive at a diagnosis and treatment plan for each patient
presentation. The CBPS examinations are computer-driven using an interactive screen to gather patient
information, order tests and procedures, make a diagnoses and administer treatments.

Before written, oral, and CBPS questions are incorporated into an examination, they are studied and
field tested for efficacy. Each question is viewed statistically immediately after the examination. Some
questions may be eliminated, or the number of acceptable responses broadened, if evidence of significant
ambiguity is discovered.
ABPS 420 (April 2009) Page 3

How to Study for ABPS Examinations

1. Read this Study Guide to learn as much as you can about the examination
process.

2. Get maximum benefit from the sample written questions by answering them
without looking at the answer key. Then score yourself and note any pattern
showing a weakness in any particular subject area.

3. It is imperative that candidates sitting for the CBPS examinations access the
practice questions sometime during the weeks prior to the actual examination.
Studies have shown that candidates who do not practice the CBPS examination
often do not have time to complete the actual CBPS examination. You must
become familiar with the computer screens and how they function if you expect
to efficiently move through the actual examinations.

Candidates for the CBPS examination access the practice examination through
their personal page on the ABPS web site using their user ID and passwords.
Instructions for using the practice examinations are also found on the personal
page.

4. Use references listed on the Suggested Reading List (beginning on page 31) to
review the subject areas tested in ABPS examinations.

5. While ABPS does not endorse board certification review courses, candidates have
reported that the review of subject areas and mock orals included in some review
courses have been helpful in preparing for the examinations. Board review
courses are advertised in national podiatric publications and are often associated
with the colleges of podiatric medicine and state podiatric medical associations.

6. Make time to prepare for the examinations. Cramming and intensive study may
not be helpful. Candidates who prepare by carefully reviewing are sometimes
less anxious at examination time and may perform better.
ABPS 420 (April 2009) Page 4

Written Examinations:

Board Qualified Foot Surgery


Board Qualified Reconstructive Rearfoot/Ankle Surgery
Recertification Foot Surgery
Recertification Reconstructive Rearfoot/Ankle Surgery

General Information

a. Format. The written examinations consist of objectively scored (i.e., multiple-choice, true-false,
etc.) questions administered by computer. Adaptive testing uses a large pool of questions, the
difficulty level of which was determined by the questions’ statistical performance in ABPS tests. In
an adaptive test, the computer dynamically selects questions as the candidate takes the test and
adjusts question selection based on the candidate’s demonstrated ability level. As each question is
answered, the computer assesses the response and selects the next question based on whether the
previous answer was right or wrong. Questions become progressively more or less difficult as the
program assesses and establishes the test taker’s ability level. Each question has only one best
answer. In this computer adaptive format, returning to previous questions for review and/or
changing answers is not possible. It should be noted that in an adaptive test, each question is
weighted according to its difficulty level and other statistical properties, and not by its position in the
test.

b. Subject Areas. The written examinations are based on the following subject areas applicable to the
practice of podiatric medicine and surgery:

Basic Science.
Pathoanatomy.
Pathomechanics.
Pathophysiology.
Diagnostic Evaluation/Medical Imaging.
Surgical Principles.
Surgical Procedures and Techniques.
Procedural Perioperative Management.
Surgically Relevant Medical Management.
General medical.
Anesthesia.
Emergency Medicine.
Lower extremity conservative.
Complications.
ABPS 420 (April 2009) Page 5

NOTE: The following specific conditions may be tested under each response subject area:

Deformities.
Infections.
Neoplasm/tumors/masses.
Other conditions.
Acute trauma.

c. Time considerations. Candidates with average reading skills should have ample time to complete
each written examination session. Extra time has been factored in to allow most candidates to work
steadily through the questions without running out of time.

d. Scoring. Examination results are provided as a scale score. The score is determined by the difficulty
level of the questions you are able to answer correctly. The examinations are not graded on a curve.
All examinations will be administered at computer sites throughout the United States.. Except for
the Reconstructive Rearfoot/Ankle Surgery Recertification Examination, all written examinations are
adaptive. The Reconstructive Rearfoot/Ankle Surgery Recertification Examination is a linear
examination. The Board of Directors sets passing scores.

e. Review of Scores. Each examination is scored by computer, and failed examinations are
automatically rechecked and re-scored. Therefore, once examination results are mailed to
candidates, there is no further review of scores.

f. Inclusion of Non-scored Questions. Some questions may be included in the ABPS examinations to
improve the examination system and to provide data for investigations into the examination process.
Such questions are not counted in determining your scores.

Board Qualified and Recertification in Foot Surgery

a. Content. The Foot Surgery examinations will test the diagnostic evaluation of conditions
pertaining to the foot and ankle and intraoperative/perioperative management of both forefoot
and non-reconstructive rearfoot procedures.

b. Schedule. Both the Board Qualified and the Foot Surgery Recertification examinations will
consist of 125 questions, given in one session of no more than three hours in length.

Board Qualified and Recertification in Reconstructive Rearfoot/Ankle Surgery

a. Content. The Reconstructive Rearfoot/Ankle examinations will test diagnostic, intraoperative,


and perioperative management pertaining to reconstructive rearfoot and ankle procedures.

b. Schedule. The Board Qualified examination will consist of 125 questions given in one session of
no more than three hours. The Recertification examination in Reconstructive Rearfoot/Ankle
Surgery will contain 100 questions, given in one session of no more than three hours.
ABPS 420 (April 2009) Page 6

Self-Assessment in Foot and Ankle Surgery

a. Content. The Foot and Ankle Surgery Self-Assessment examination will test diagnostic,
intraoperative, and perioperative management pertaining to foot and ankle procedures.

b. Schedule. The self-assessment examination in Foot and Ankle Surgery will consist of 100
questions. The examination is administered as a secured examination at compute test sites.

Oral and CBPS Examinations:

Certification Foot Surgery


Certification Reconstructive Rearfoot/Ankle Surgery

The oral and CBPS examinations serve to evaluate the candidate's knowledge and skill in obtaining and
interpreting clinical information as well as their ability to reason logically and to arrive at a diagnosis
and a treatment plan for a specific patient presentation.

General Information

a. Overall Process. Each candidate is assigned to a group with a unique schedule for the oral and
CBPS examination sessions. Groups gather in a central meeting area until their schedule
begins at which time they are escorted to the examination hallways in the hotel. Each oral
examination session is held in a private room where the candidate and the oral examiner
(selected by the Board of Directors) sit across from each other at a table. Candidates move
between rooms until the group schedule is complete, and then return to the central meeting
area. Every attempt is made to equalize waiting times for all groups. The CBPS questions are
administered in larger groups with each candidate having access to a laptop computer.

b. Oral Question Format. At the beginning of each question, the candidate receives a Candidate
Information Sheet describing a clinical situation. Based on the described situation, the
candidate must request additional information (such as patient history, physical examination,
radiographs, laboratory reports, etc.) in a manner simulating a real patient encounter. The
examiner will respond as a patient would and will only report, not interpret, any requested data.
The examiner will not volunteer information, so the candidate must be thorough and specific
when asking for all information needed to reach a diagnosis and treatment plan. The examiner
scores each question by noting which predetermined essential areas are satisfactorily covered
by the candidate.
ABPS 420 (April 2009) Page 7

Example of an examiner/candidate interchange:

Candidate: Is there a history of injury?


Examiner: Specifically?
Candidate: Is there history of injury to the foot or ankle?
Examiner: The patient says that he hurt his foot when he was very small.

c. Scoring. Examiners fill out scorecards for each candidate. Each oral question has predetermined
essential areas that must be adequately covered to pass the question. A passing score is set by
the Board of Directors based on psychometric evaluation of the examination.

d. Review of Scores. The scorecards of failing candidates are automatically reviewed and
rechecked for accuracy. Therefore, once examination results are mailed to candidates, there is no
further review of scores.

e. Challenge Examination. If a candidate feels that an oral examiner has been unfair, or that a
personality conflict has interfered with the examination, the candidate may request a challenge
examination by another examiner. The candidate must request the challenge immediately upon
leaving the examiner's room. The Chairman of the Examinations Committee will determine
whether a challenge examination is warranted. Specific instructions about this process will be
given to candidates during the candidate orientation session prior to the examination.

f. CBPS Examination. A CBPS examination is “Candidate-Process Driven.” You will drive the
examination by performing actions to collect and analyze information. You will select the
necessary information for problem solving. For example, if you wanted to palpate a foot mass,
you would select “palpate mass” in the “Physical Exam” section. If you wanted to aspirate the
mass, you might select “aspirate needle” in the “Diagnostic Procedures” section. If you wanted
to order a magnetic resonance image (MRI) on the foot, you would select the appropriate MRI in
the “Imaging” section. Performing a surgical technique (procedure) on the mass is done in the
“Treatment” section. Follow-up care is also selected, when appropriate, in the “Treatment”
section. By practicing with the software and the simulated case examination, you will be become
familiar with how to navigate through the simulation.

You are to complete the CBPS as best you can by taking into account the relevant aspects of
patient management, (case history, physical examination, imaging, labs, diagnostic procedures,
diagnosis, treatment; and in some cases follow-up diagnoses and treatments). While collecting
patient information, you must balance thoroughness with efficiency, as well as balancing quality
versus quantity. You will need to pace yourself and be careful to not take too much time on any
one point or decision as the CBPS is a timed examination. Field testing has demonstrated that
users who have practiced the CBPS will have ample time to complete each case. As you collect
information regarding your simulated case, keep in mind that relevancy holds the key to
successful resolution of a clinical problem. For example if you are hesitant about whether a
procedure is warranted, make the decision based on clinical indications. CBPS scoring is based
on the relevancy of the processes or actions performed.
ABPS 420 (April 2009) Page 8

Certification in Foot Surgery

a. Schedule. There are six traditional (examiner/candidate) oral questions and six Computer Based
Patient Simulation (CBPS) given in three sessions, over one and one-half days.

b. Content. The questions will test the diagnostic evaluation of conditions pertaining to the foot
and ankle, and intraoperative/perioperative management of both forefoot and non-
reconstructive rearfoot procedures.

c. Subject Areas. The Foot Surgery Certification Oral Examination (oral and CBPS) focuses on
those deformities and conditions involving the forefoot and non-reconstructive rearfoot
procedures. Questions may emphasize the following areas:
Biomechanical/Acquired deformities.
Congenital and/or pediatric deformities.
Infections.
Metabolic conditions and/or emergency medical management.
Neoplastic (primary or metastatic) conditions.
Traumatic conditions.
Surgical or traumatic complications.

d. Timing. Candidates receive six tradition oral questions divided between three sessions. Each
question lasts 14 minutes. Candidates receive six CBPS questions divided between three
sessions. Each question lasts 25 minutes.

e. Passing Score. Each year a passing score is set by the Board of Directors based on psychometric
evaluation of the examination.

Certification in Reconstructive Rearfoot/Ankle Surgery

a. Schedule. There are four traditional (examiner/candidate) oral questions and four Computer
Based Patient Simulation (CBPS) questions given over one day.

b. Content. The examination will involve diagnostic, intraoperative, and perioperative


management pertaining to reconstructive rearfoot and ankle procedures.
ABPS 420 (April 2009) Page 9

c. Subject Areas. The Reconstructive Rearfoot/Ankle Surgery Certification Examination (oral and
CBPS) focuses on those deformities and conditions involving the rearfoot and ankle. Questions
may emphasize the following subject areas:
Common rearfoot/ankle trauma.
Adult rearfoot/ankle deformities.
Pediatric rearfoot/ankle trauma or deformities.
Surgical or traumatic complications.
Neoplasms or infections.

d. Timing. Each oral question lasts for 20 minutes, while each CBPS lasts 25 minutes.

e. Passing Score. Each year a passing score is set by the Board of Directors based on psychometric
evaluation of the examination.

NOTE: Both the qualification examination and the certification examination in Reconstructive
Rearfoot/Ankle Surgery must be passed for Certification in Reconstructive Rearfoot/Ankle
Surgery. If a candidate sits for both the qualification and the certification examination and
passes the qualification only, he/she is Board Qualified in Reconstructive Rearfoot/Ankle
Surgery, provided the candidate has completed a PSR-24 or PSR-24+ or PM&S-36
residency (Board Qualified or Certification in Foot Surgery is a prerequisite for attaining
Board Qualified status in Reconstructive Rearfoot/Ankle).
ABPS 420 (April 2009) Page 10

SAMPLE FOOT SURGERY WRITTEN QUESTIONS

Note: These questions are representative of the various formats used by the ABPS in
its examinations. These questions are not meant to be representative of the scope or
level of difficulty of any specific examination.
Answers found on page30.

1. A patient presents with painful hallux abductovalgus deformity. There is pain-free range of motion of the
first metatarsophalangeal joint (MPJ). Radiographs reveal:

hallux abductus angle: 35 degrees


intermetatarsal angle: 15 degrees
proximal articular set angle: 22 degrees
distal articular set angle: 4 degrees
metatarsus adductus angle: 8 degrees

What is the most appropriate procedure?

A. closing wedge with McBride.


B. proximal Akin with McBride.
C. closing base wedge with Reverdin-Green.
D. Austin with proximal Akin.

2. Following removal of the proximal phalanx of the fifth digit, what is an appropriate surgical procedure for
prevention of fifth digit flailing?

A. syndactylism of the fourth digit to the fifth digit.


B. arthrodesis of the distal interphalangeal joint.
C. shortening of the flexor tendons.
D. flexor tendon transfer.

3. A patient presents with an acute dorsal dislocation of the third metatarsophalangeal articulation.
Radiographs show no fractures and closed reduction is performed under local anesthetic. What is the proper
sequence of maneuvers to reduce the dislocated digit?

A. longitudinal traction and plantarflexion of the digit.


B. plantarflexion and then longitudinal traction of the digit.
C. dorsiflexion, longitudinal traction, and plantarflexion of the digit.
D. longitudinal traction, dorsiflexion, and plantarflexion of the digit.

4. A patient with a history of true penicillin allergy is scheduled for a total joint implant. Antibiotic prophylaxis
would best be served with which of the following medications?

A. piperacillin (Pipracil).
B. cephalothin (Keflin).
C. ciprofloxacin (Cipro).
ABPS 420 (April 2009) Page 11

D. vancomycin (Vancocin).

5. A 60-year-old male suddenly begins complaining of a "crushing" sensation in his chest. The episode lasts
very briefly then subsides. It begins again and is more intense. What should you administer to this patient?

A. nitroglycerin (Nitrogard) 0.5 mg.


B. diazepam (Valium) 5 mg.
C. ephedrine (Racephedrine) 25 mg.
D. morphine sulfate 15 mg.

6. Radiographs taken of a 25-year-old male with a painful bunion indicate the following:

interphalangeal abductus angle: 18 degrees


hallux abductus angle: 30 degrees
distal articular set angle: 6 degrees
proximal articular set angle: 7 degrees
metatarsus primus adductus angle: 14 degrees
deviated first metatarsophalangeal joint

Based upon this information, what is the procedure of choice?

A. proximal Akin with Austin.


B. proximal Akin.
C. distal Akin with Austin.
D. distal Akin with Reverdin.

7. What procedure should be performed on a patient with a 25 degree metatarsus primus adductus angle and a
severe first ray hypermobility?

A. Lapidus.
B. lateral closing wedge osteotomy.
C. crescentic osteotomy.
D. Logroscino.

8. A 32-year-old woman has a history of a slowly enlarging, subcutaneous, multinodular, painless mass beneath
the flexor surface of the great toe. There is radiographic evidence of marginal bone erosion. What is the
most likely diagnosis?

A. glomus tumor.
B. enchondroma.
C. giant cell tumor of the tendon sheath.
D. ganglion.

9. What would a dorsoplantar radiograph of the foot with a plantarflexed first ray taken in the angle and base of
gait show the position of the hallucal sesamoids to be?

A. medial to the metatarsal head.


B. proximal to the metatarsal head.
C. distal to the metatarsal head.
D. lateral to the metatarsal head.
ABPS 420 (April 2009) Page 12

10. A patient presents with an inversion ankle sprain. Stress inversion radiographs reveal a 15 degree difference
between the symptomatic and asymptomatic ankle. A peroneal tenogram reveals contrast media within the
ankle joint and extravasation anterior, lateral, and distal to the lateral malleolus. These findings are
consistent with rupture of the:

A. anterior talofibular ligament.


B. anterior talofibular and calcaneofibular ligaments.
C. anterior talofibular ligament and peroneal tendons.
D. anterior talofibular, calcaneofibular, and posterior talofibular ligaments.

11. A 65-year-old female diabetic on insulin therapy arrives at the surgeon’s office. While waiting she begins
to feel faint and loses consciousness. What would be the treatment of choice?

A. administer phenytoin (Dilantin) 200 mg. intramuscularly (I.M.).


B. administer epinephrine 0.5 cc. intramuscularly (I.M.).
C. protect patient from injury and observe.
D. administer 25 cc. of 50 percent dextrose in water intravenously (I.V.)

12. A 25-year-old female has unremitting numbness in the lesser digits. The pain becomes more severe while
walking and is difficult to relieve. Over the past several months, the pain has been sharp with radiating
sensation in the arch. What is the most likely diagnosis?

A. neuroma of the third intermetatarsal nerve.


B. sciatica.
C. diabetes mellitus.
D. compression of the posterior tibial nerve.

13. What is an alternate approach to hallux limitus that does not require a joint destructive procedure? (note
negative format)

A. dorsiflexory osteotomy of the proximal phalanx.


B. Jones tenosuspension.
C. lengthening of extensor hallucis longus.
D. dorsiflexory osteotomy of first metatarsal base.

14. A plantar fibroma is removed. Two months later the wound has healed, but the three medial digits begin to
hyperextend at the metatarsophalangeal joints (MPJ). A possible cause would be the severance of which of
the following nerves or muscles?

A. medial plantar nerve.


B. quadratus plantae.
C. saphenous nerve.
D. flexor digiti minimi.
ABPS 420 (April 2009) Page 13

15. A 30-year-old male complains of a painful right ankle after sustaining a forced plantarflexion injury.
Plantarflexion of the foot and dorsiflexion of the hallux greatly exacerbate the symptoms. What is the most
probable diagnosis?

A. flexor digitorum longus tendinitis.


B. fracture of the sustentaculum tali.
C. fracture of the posterior lip of the tibia.
D. fracture of the posterior tubercle of the talus.

16. Three weeks following excision of a neuroma in the third intermetatarsal space, the third digit drifts into an
adducted position. The most probable cause would be severance of which of the following anatomical
features?

A. second lumbrical muscle.


B. third dorsal-interossei tendon.
C. second plantar interossei muscle.
D. lateral capsule of the third metatarsophalangeal joint.

17. In using a Z-plasty to correct a skin contracture, what should the surgeon do?

A. keep the Z as small as possible.


B. make the central incision of the Z parallel to the contracted skin.
C. place the Z perpendicular to Langer's lines of tension.
D. make the wings of the Z angled 25 degrees from the central incision.

18. A 36-year-old female has a history of recurrent masses in the medial longitudinal arch of the right foot.
What is the most likely primary lesion of the foot causing metastasis to the lung?

A. melanoma.
B. rhabdomyosarcoma.
C. fibrosarcoma.
D. neurofibromatosis.

19. A 15-year-old female sustained an injury resulting in a Salter-Harris type V epiphyseal fracture of the first
metatarsal base. What should initial treatment include?

A. cast immobilization.
B. open reduction and ASIF fixation.
C. soft cast and wooden shoe.
D. no treatment is necessary.

20. Which of the following procedures is not an appropriate treatment of a dog bite on the foot which occurred
eight hours ago? (note negative format)

A. debridement of wound and primary closure.


B. pulsed lavage irrigation of wound.
C. saline web-to-dry daily dressing changes.
D. OR debridement.
ABPS 420 (April 2009) Page 14

21. Keratoses beneath the second metatarsal head are frequently observed in association with hallux valgus
deformities. What is the most common cause of these keratoses?

A. hypermobility of the first ray.


B. short second metatarsal bone.
C. long first metatarsal bone.
D. rigid plantarflexed first ray.

22. A patient has a history of hypertension which is controlled by furosemide (Lasix) 40 mg. (q.d.). What is the
most frequent electrolyte disturbance observed in this type of patient?

A. hypokalemia.
B. hyponatremia.
C. hyperkalemia.
D. hypernatremia.

23. Which benign soft-tissue lesion may be invasive into bone?

A. synovioma.
B. histiocytoma.
C. villonodular synovitis.
D. neurilemmoma.

24. What is the condition that must be present before attempting an extensor suspension (Jones) procedure of
the first metatarsal?

A. contracted hallux.
B. flexible plantarflexed first metatarsal.
C. forefoot supinatus that is reducible by manual pressure.
D. adequate interphalangeal joint motion.

25. A 25-year-old female sustained an inversion ankle sprain 24 hours ago. The area is severely edematous and
ecchymotic. Diagnostic tests reveal rupture of the lateral collateral ligaments. What should the treatment
consist of at this time?

A. compression dressing for 24-to-48 hours.


B. open ligament repair.
C. posterior splint and warm compresses for 24-to-48 hours.
D. tape strapping with high top athletic shoe for 24-to-48 hours.
E. short-leg walking cast.

26. To derotate an adductovarus fifth digit, how is the lenticular incision made?

A. distal medial to proximal lateral.


B. distal lateral to proximal medial.
C. lateral to medial.
D. medial to lateral.
ABPS 420 (April 2009) Page 15

27. A 40-year-old female presents with a ganglion at the base of the first metatarsal medially. After surgical
excision of the ganglion, the patient complains of anesthesia along the medial aspect of the foot, up to the
first metatarsal head. Which peripheral nerve was involved?

A. medial plantar.
B. lateral plantar.
C. saphenous.
D. common peroneal.

28. What coalition does the Harris-Beath view (axial view of the calcaneus) best demonstrate?

A. talonavicular coalition.
B. calcaneonavicular coalition.
C. talocalcaneal coalition.
D. calcaneocuboid coalition.

29. When performing a Lapidus procedure for hallux abductovalgus, what structure should be avoided when
exposing the first metatarsocuneiform articulation?

A. deep perforating artery.


B. flexor hallucis longus tendon.
C. deep peroneal nerve.
D. anterior tibial tendon.

30. A 41-year-old female has a unilateral flatfoot of six-months duration. The patient relates a history of
multiple cortisone injections by another doctor for "os tibial externum." What is the most likely cause of
this patient's condition?

A. dorsal tear of the plantar fascia.


B. subtalar degenerative arthritis.
C. rupture of the peroneus brevis tendon.
D. rupture of the posterior tibial tendon.

31. A 60-year-old male develops an anaphylactic reaction secondary to a drug injection. Epinephrine
(Adrenalin) 0.5 cc. is injected intramuscularly (I.M.) immediately. What is the purpose of administering
this drug in this situation?

A. slow the rate of absorption of the offending drug.


B. strengthen myocardial contractions.
C. combat cardiac arrhythmias.
D. dilate the bronchioles.

32. What is most often the cause of anterior process fractures of the calcaneus?

A. inversion with the foot plantarflexed.


B. eversion with the foot plantarflexed.
C. inversion with the foot dorsiflexed.
D. eversion with the foot dorsiflexed.
ABPS 420 (April 2009) Page 16

33. When is electrical stimulation of the bone best indicated?

A. synovial pseudoarthrosis.
B. hypertrophic nonunion.
C. fibrous nonunion.
D. fibrous malunion.

34. Three months after resecting a ganglion from the dorsum of the midfoot, a patient presents complaining of
persistent numbness along the adjacent sides of the second and third digits. The most likely diagnosis is
iatrogenic severance of which nerve?

A. saphenous.
B. deep peroneal.
C. lateral dorsal cutaneous.
D. medial dorsal cutaneous.

35. What procedure most effectively transfers the action of the flexor digitorum longus to the
metatarsophalangeal joint (MPJ) of the lesser digit?

A. arthroplasty of proximal interphalangeal joint.


B. arthrodesis of proximal and distal interphalangeal joints.
C. flexor lengthening.
D. extensor tenotomy.

36. What is the major reason for countersinking?

A. reduce screw prominence.


B. disperse compression force of screw head evenly.
C. allow for shorter screw utilization.
D. provide proper angulation of screw.

37. In the early stages of superficial spreading malignant melanoma, which of the following conditions is not
present? (note negative format)

A. change in size.
B. pain and bleeding.
C. variation in color.
D. irregular borders.

38. On a peripheral blood smear stained with hematoxylin and eosin, the presence of "rosettes" (clusters of
polynuclear leukocytes surrounding an extracellular hematoxylin body) most likely indicates which of the
following systemic disorders?

A. rheumatoid arthritis.
B. lupus erythematosus.
C. hematogenous infection.
D. thrombocytopenic purpura.
ABPS 420 (April 2009) Page 17

39. Shortly after general anesthesia, a patient is nauseous and treatment is required. Which of the following
would be the most appropriate treatment?

A. diazepam (Valium) 10 mg. intramuscularly (I.M.).


B. prochlorperazine (Compazine) 10 mg. intramuscularly (I.M.).
C. fentanyl (Sublimaze) 0.05 mg. intramuscularly (I.M.).
D. meprobamate (Meprospan) 100 mg. intramuscularly (I.M.).

40. A patient presents with a brachymetatarsia on the fourth metatarsal right foot. What is the limiting factor
when considering the maximum length that may be added to the metatarsal?

A. tendon length.
B. nerve length.
C. adjacent metatarsal length.
D. vascular length.

41. The dynamic compression plate is most suitable as a solitary fixation device for which of the following
fractures?

A. oblique.
B. spiral.
C. transverse.
D. compression.

42. Why should burning of bone with a power saw be avoided?

A. it decreases phagocytosis at operative site.


B. it causes an increased zone of resorption at the osteotomy site.
C. it results in necrotic bone in which white blood cells fail to migrate.
D. additional inflammation causes a decrease in phosphate and displacement of calcium in bone
matrix.

43. A 30-year-old female complains of a painful, six-month-old lesion on the bottom of her foot. Examination
shows a hyperkeratotic lesion beneath the third metatarsal head without a central nucleated core. The third
digit is contracted at the proximal interphalangeal joint and is nonreducible. The metatarsal parabola is
normal. What is the procedure of choice?

A. flexor tendon transfer.


B. flexor tenotomy of the third digit.
C. arthrodesis fusion of the third digit.
D. resection of the third metatarsal head.

44. A patient presents eight weeks after sustaining a fracture through the neck of the talus. What finding is a
prognostic indicator that the vascular supply is intact?

A. resorption of subchondral bone of talar dome.


B. increased density of the talar body.
C. increased trabecular pattern across fracture.
D. absence of degenerative arthritis.
ABPS 420 (April 2009) Page 18

45. Which procedure for hallux abductovalgus has the least effect on growth centers in a young child? (note
negative format)

A. Austin.
B. opening abductory wedge.
C. Lapidus.
D. closing abductory wedge.

46. Which complication is not associated with the Keller bunionectomy? (note negative format)

A. loss of hallux purchase.


B. diminished propulsion of digit.
C. stress fracture of second metatarsal.
D. tibial sesamoiditis.

47. In a Steindler stripping, which group of muscles is sectioned?

A. flexor digitorum brevis, abductor digiti quinti, abductor hallucis.


B. flexor digitorum brevis, adductor hallucis, abductor hallucis.
C. abductor hallucis, adductor hallucis, quadratus plantae.
D. quadratus plantae, abductor digiti quinti, flexor digitorum brevis.

48. Twenty-one hours after an Austin bunionectomy under general anesthesia, the patient develops a
temperature of 102 degrees Fahrenheit. What is the most likely cause?

A. pulmonary atelectasis.
B. postoperative wound infection.
C. superficial thrombophlebitis.
D. constipation.

49. The preoperative complete blood count (CBC) on a 45-year-old male reveals the following:

platelets: 1.0 million


leukocytes: 10,000
hemoglobin: low

What is the most likely cause of these abnormal values?

A. myelofibrosis.
B. polycythemia vera.
C. thrombocythemia.
D. leukemia.

50. Bence-Jones proteinuria is most commonly associated with which of the following diseases?

A. lymphoma.
B. systemic lupus erythematosus.
C. scleroderma.
D. multiple myeloma.
ABPS 420 (April 2009) Page 19

51. What is the local anesthetic that should be used on a patient with cirrhosis of the liver?

A. lidocaine (Xylocaine).
B. mepivacaine (Carbocaine).
C. procaine (Novocain).
D. bupivacaine (Marcaine).

52. The most severe nerve injury is complete disruption of the axon, Schwann cell, and endoneurial tubes with
varying disruption of the perineurium and epineurium. What is the term used to designate this condition?

A. neuropraxia.
B. axonotmesis.
C. neurotmesis.
D. neuritis.

53. The most probable etiology of heloma molle of the fourth interdigital space is pressure between which of
the following structures?

A. head of the fifth proximal phalanx and the base of the fourth proximal phalanx.
B. head of the fourth proximal phalanx and the base of the fifth proximal phalanx.
C. head of the fourth metatarsal and the base of the fifth proximal phalanx.
D. head of the fifth metatarsal and the base of the fourth proximal phalanx.

54. For the traditional Austin procedure to be modified to produce both shortening and plantarflexion of the
capital fragment, how must the osteotomy be angulated?

A. proximal-medial dorsal and distal-lateral plantar.


B. distal-medial dorsal and proximal-lateral plantar.
C. dorsal proximal-lateral and plantar-medial distal.
D. medial plantar and dorsal lateral.

55. Which of the following results is seen in simple chronic anemia?

A. hemoglobin and hematocrit increase.


B. hemoglobin and hematocrit decrease.
C. hemoglobin decreases and hematocrit increases.
D. hemoglobin increases and hematocrit decreases.

56. A 50-year-old nondiabetic male cigarette smoker presents with one block right calf claudication. Bilateral
femoral pulses are palpable with no bruits. The left pedal pulse is present, but the right popliteal and pedal
pulses are absent. What is the most likely level of arterial occlusion?

A. abdominal aorta.
B. right common iliac artery.
C. right popliteal artery.
D. right superficial femoral artery.
ABPS 420 (April 2009) Page 20

57. In performing a McBride bunionectomy under local anesthesia, which of the following nerves should not be
anesthetized? (note negative format)

A. saphenous.
B. deep peroneal.
C. medial dorsal cutaneous.
D. intermediate dorsal cutaneous.

58. Charcot joints are not commonly associated with which disease? (note negative format)

A. syringomyelia.
B. gout.
C. diabetes mellitus.
D. alcoholism.

59. A 43-year-old male with Addison's disease of 20 years duration is on a maintenance dose of 30 mg.
hydrocortisone per day. Prior to surgery, how should this patient be treated?

A. be supplemented with additional hydrocortisone.


B. discontinue hydrocortisone.
C. be supplemented with thyroxin.
D. maintain his daily hydrocortisone dosage.

60. A two-and-one-half-year-old has been under treatment for the past year for a unilateral metatarsus adductus.
The most recent treatment included a series of eight weekly casts with manipulation. Radiographs now
show a metatarsus adductus angle of 25 degrees. What is the indicated treatment?

A. additional casting and manipulation.


B. tarsometatarsal and intermetatarsal base releases and casting.
C. Denis-Browne night splints.
D. metatarsal base osteotomies with casting for six-to-eight weeks.

61. A 45-year-old male sustains a crushing-type injury to the left foot. Radiographs are negative for fracture
and dislocation. Which of the following tests is the most helpful in determining skin flap viability?

A. fluorescein dye study.


B. disital plethysmography.
C. segmental pressure gradients.
D. indium scan.

62. Which radiographic finding is most important in evaluating placement of an osteotomy for correction of
hallux valgus?

A. tibial sesamoid position.


B. hallux abductus angle.
C. first intermetatarsal angle.
D. first metatarsal protrusion distance.
ABPS 420 (April 2009) Page 21

63. Surgical excision of the hallucal sesamoids would most directly result in what condition?

A. hallux limitus.
B. lack of propulsive stability of the hallux.
C. inability of the first ray to dorsiflex.
D. inability of the forefoot to evert.

64. A man is coughing and wheezing from a foreign body lodged in his airway. What should be done?

A. give him four back blows.


B. give him four back blows and four manual thrusts.
C. give him water.
D. attempt to dislodge the foreign body by sweeping your fingers through his mouth.
E. encourage him to continue coughing.

65. Which malignant melanoma is most commonly misdiagnosed as a pyogenic granuloma?

A. superficial spreading.
B. nodular.
C. lentigo malignant.
D. acral lentiginous.

66. A 35-year-old male presents with a hallux abductovalgus deformity of the left foot. Radiographs reveal the
following:

intermetatarsal angle: 13 degrees


proximal articular set angle: 12 degrees
distal articular set angle: 8 degrees
hallux abductus angle: 20 degrees
metatarsal adductus angle: 27 degrees
metatarsal protrusion: +1 mm.
medial exostosis is present

Which statement is true regarding findings of the intermetatarsal angle?

A. less important because of the hallux abductus angle.


B. less important because of the metatarsus adductus angle.
C. more important because of the hallux abductus angle.
D. more important because of the metatarsus adductus angle.

67. A 31-year-old female presents with a complaint of pain of the dorsal distal aspect of the hallux.
Radiographs reveal a radiolucent area at the distal phalanx with surrounding sclerosis. There is no sign of
infection, no sinus tract, and no edema. What is the clinical diagnosis?

A. subungual exostosis.
B. enchondroma.
C. osteoid osteoma.
D. osteochondroma.
E. aneurysmal bone cyst.
ABPS 420 (April 2009) Page 22

68. Excision of a second intermetatarsal neuroma results in an abducted third digit. Damage to which intrinsic
muscles would produce this deformity?

A. first plantar interossei.


B. third dorsal interossei.
C. second lumbrical.
D. flexor digitorum brevis.
ABPS 420 (April 2009) Page 23

The following are examples of VISUAL questions.


69. The radiograph shown below is consistent with which type of Charcot diabetic osteoarthropathy?

A. developmental atrophic.
B. developmental hypertrophic.
C. coalescence atrophic.
D. coalescence hypertrophic.

70. Shown below is a radiograph of a 14-year-old male who fell six feet and presented to the emergency
department two hours after the injury. Which of the following diagnostic tests is the most important to
order?

A. axial radiograph of the foot.


B. computed tomography (CT) scan of the foot.
C. magnetic resonance imaging (MRI) of the foot.
D. spine radiograph.
ABPS 420 (April 2009) Page 24

71. The radiograph shown below was taken 29 days postoperatively. Which treatment is appropriate?

A. a minimum of 13 additional days of fixation is required.


B. the pin can be removed as clinical healing is noted.
C. the pin must remain in place since a nonunion is developing.
D. re-evaluate the patient’s condition in one week.
ABPS 420 (April 2009) Page 25

SAMPLE RECONSTRUCTIVE REARFOOT/ANKLE WRITTEN QUESTIONS

Answers found on page 30.

72. A 20-year-old male sustains his first ankle injury which is diagnosed as a rupture of the lateral collateral
ligaments of the ankle. What is the most appropriate treatment plan?

A. splinting and functional rehabilitation


B. elastic-type ankle support for six weeks.
C. open repair of lateral ligaments.
D. short-leg nonweightbearing cast for eight-to-ten weeks.

73. A 27-year-old man develops a deep wound infection three weeks following open reduction and internal
fixation of a pilon ankle fracture with interfragmental screws and a medial buttress plate. The wound is
debrided and the patient is placed on intravenous (I.V.) antibiotics. What should be done with the
hardware?

A. removed regardless of the degree of fracture healing.


B. removed and an external fixator applied.
C. left in place until fracture union is obtained, unless loosening occurs.
D. removed only if exposed.

74. Why is the trephine technique to fuse the posterior facet during a triple arthrodesis contraindicated in a
cavus foot? (note negative format)

A. varus deformity in rearfoot cannot be corrected.


B. cavus deformity creates too much stress through posterior facet.
C. calcaneal and talar posterior facets are oblique to each other.
D. it is too difficult technically to perform a trephine technique in a cavus foot type.

75. What is a contraindication to performing a Young tenosuspension flatfoot procedure? (note negative
format)

A. talonavicular fault.
B. patient age 12.
C. rigid deformity.
D. dorsiflexed first metatarsal.

76. How is chronic peroneus longus rupture resulting in a dorsiflexed and inverted foot best treated?

A. lateral transfer of anterior tibial tendon to cuneiforms.


B. transfer of anterior tibial tendon to plantar aspect of foot.
C. Young tenosuspension.
D. Kidner procedure.
ABPS 420 (April 2009) Page 26

77. The advancement of the Achilles tendon more anteriorly toward the posterior aspect of the subtalar joint on
the dorsum of the calcaneus is useful in treating which of the following conditions?

A. spastic equinus.
B. spastic heel varus.
C. talipes calcaneus deformity.
D. peroneal spastic flatfoot.

78. Axial load forces creating instability in diaphyseal fibular and tibial fracture surfaces may be dissipated by
the use of which of the following mechanisms?

A. buttress plate.
B. neutralization plate.
C. tension band cerclage wiring.
D. double compression plate.

79. In a patient with Charcot-Marie-Tooth disease (peroneal muscular atrophy) and weak anterior group
muscles, which of the following tendons should be transferred to improve foot function?

A. posterior tibial.
B. flexor digitorum longus.
C. anterior tibial.
D. flexor hallucis longus.

80. What is the most appropriate fixation technique for a small, displaced avulsion fracture of the lateral
malleolus?

A. tension band.
B. single lag screw.
C. single lag screw and a one-third tubular plate.
D. dynamic compression plate.

81. What is the muscle that is most commonly transferred through the interosseous membrane to function as a
dorsiflexor of the foot?

A. posterior tibial.
B. anterior tibial.
C. peroneus longus.
D. flexor digitorum longus.

82. A 21- year- old female presents with pain and swelling to the right ankle. The patient states that last week
she jumped from a high fence on to uneven ground, twisted her foot and fell forward. Assuming that the
foot was inverted and then forced into dorsiflexion, this type of trauma would suggest what type of
frequently missed fracture?

A. lateral process of the talus.


B. avulsion of the medial malleolus.
C. oblique proximal fibula.
D. sustentaculum tali of the calcaneus.
ABPS 420 (April 2009) Page 27

83. In performing a pantalar arthrodesis, which materials would be least effective in compression fixation of the
ankle and subtalar joint? (note negative format)

A. external fixator.
B. intramedullary nail.
C. large cancellous screws crossing both joints.
D. crossed Steinman pins.

84. A 30-year-old male, who had a resection of a calcaneonavicular bar at age 11, exhibits fixed valgus
deformity of his foot. Radiographs demonstrate the recurrence of a calcaneonavicular coalition with
subtalar joint degeneration and talar beaking. What is the appropriate surgical treatment?

A. resection of coalition with interposition of extensor digitorum brevis muscle.


B. subtalar joint arthrodesis.
C. resection of coalition with subtalar arthroereisis.
D. triple arthrodesis.

85. For a severe rigid and painful talipes equinovarus in an adult, what primary surgical approach is
contraindicated? (note negative format)

A. soft-tissue release and tendon transfer.


B. talectomy and foot-to-leg arthrodesis.
C. tarsal osteotomies.
D. triple arthrodesis.

86. In performing a posteromedial release for a congenital clubfoot deformity, what is the most important joint
to release to prevent recurrence of the deformity?

A. ankle.
B. subtalar.
C. talonavicular.
D. calcaneocuboid.

87. In a six-month-old child, aggressive casting for the equinus component of a clubfoot deformity may result in
which of the following conditions?

A. dislocation of the ankle.


B. subluxation of the midtarsal joint.
C. compression of the subtalar joint.
D. rupture of posterior talotibial ligament.

88. What condition is a contraindication in performing a lateral subtalar arthroereisis? (note negative format)

A. 15 degrees of calcaneal valgus.


B. plantarflexed talus.
C. significant internal femoral torsion.
D. weakness of the posterior tibial muscle.
ABPS 420 (April 2009) Page 28

89. What is the ankle fusion technique that is indicated in a child to preserve the potential for growth of the
distal tibial and fibular physis?

A. Blair fusion.
B. transfibular fusion.
C. distraction-compression fusion.
D. Charnley compression fusion.

90. What is the first step in the open reduction and internal fixation of an ankle pilon fracture?

A. bone grafting of the metaphyseal defect.


B. fixation of the fibular fracture.
C. fixation of the medial malleolar fracture.
D. restoration of the articular surface of the distal tibia.

91. Hypertrophy of synovial villi with impingement between capsule and joint surfaces and resultant thickening
of the tissue may lead to which of the following pathological intra-articular structures encountered in ankle
arthroscopy?

A. chondral lesions grade I-IV.


B. osteochondral lesions.
C. meniscoid bodies.
D. impingement exostosis.

92. Which phrase best describes the physeal fracture of Tillaux?

A. occurs more frequently in adolescents than in children.


B. represents a supination-external rotation mechanism.
C. represents a Salter-Harris type III injury.
D. all of the above.
E. A and B only.

93. For a one-year-old patient with calcaneal valgus, what is an acceptable treatment plan?

A. manipulation only.
B. manipulation with serial casting.
C. soft-tissue release.
D. arthroereisis.
E. triple arthrodesis at skeletal maturity.

94. In the treatment of the equinovarus foot as described by Ponsetti, what is the last stage to be corrected by
casting?

A. varus component.
B. forefoot adductus component.
C. supination at the subtalar joint component.
D. equinus deformity.
ABPS 420 (April 2009) Page 29

95. What is the primary indication for the Evans calcaneal osteotomy with insertion of a bone graft?

A. congenital vertical talus.


B. residual talipes equinovarus.
C. transverse plane flatfoot deformity.
D. tarsal coalition.

96. A 16-year-old male with peroneal spastic flatfoot has 8 degrees tibia varum. What is the proper position of
the calcaneus to the tibia for triple arthrodesis?

A. 0 degrees.
B. 2 degrees inverted.
C. 6 degrees everted.
D. 10 degrees everted.

97. A 52-year-old female presents with a unilateral dropfoot condition. Following electromyelography, nerve
conduction studies and full neurological work-up the etiology remains obscure. Physical examination notes
loss of the anterior tibial and long extensors, and weakness of the posterior tibial, peroneal, and triceps. A
flaccid pes valgus dropfoot gait is noted. What is the most appropriate method of treatment?

A. subtalar fusion.
B. triple arthrodesis.
C. ankle fusion.
D. pantalar fusion.

98. Which of the following techniques is not appropriate fixation for a Salter-Harris type III fracture of the
distal tibial physis? (note negative format)

A. two smooth Kirschner wires through the fragment, across the physis, and into the metaphysis.
B. one cancellous screw through the fragment and into the epiphysis.
C. tension band wiring using two Kirschner wires through the fragment, across the physis, and into
the metaphysis.
D. one cancellous screw and one Kirschner wire through the fragment and into the epiphysis.

99. Which radiographic sign is not consistent with congenital convex pes planovalgus? (note negative format)

A. talus parallel with the longitudinal axis of the tibia.


B. dorsiflexion of the calcaneus.
C. navicular dorsal to the talus.
D. talocalcaneal angle greater than 40 degrees on the anteroposterior view.

100.What is an indication for primary fusion of the subtalar joint following a calcaneal fracture?

A. concomitant peroneal injury.


B. widening of the calcaneal body.
C. large osseous defect.
D. severe comminution.
ABPS 420 (April 2009) Page 30

Answers for Sample Written Questions

1. C 27. C 53. A 79. A


2. A 28. C 54. B 80. A
3. C 29. A 55. B 81. A
4. D 30. D 56. D 82. A
5. A 31. D 57. D 83. D
6. C 32. A 58. B 84. D
7. A 33. B 59. A 85. A
8. C 34. D 60. A 86. C
9. C 35. B 61. A 87. B
10. B 36. B 62. C 88. C
11. D 37. B 63. B 89. C
12. D 38. B 64. E 90. B
13. A 39. B 65. B 91. C
14. A 40. D 66. D 92. D
15. D 41. C 67. B 93. B
16. B 42. B 68. A 94. D
17. B 43. C 69. D 95. C
18. C 44. A 70. D 96. D
19. A 45. A 71. B 97. D
20. A 46. D 72. A 98. C
21. A 47. A 73. C 99. B
22. A 48. A 74. A 100. D
23. C 49. C 75. C
24. B 50. D 76. A
25. A 51. C 77. A
26. A 52. C 78. B
ABPS 420 (April 2009) Page 31

SUGGESTED READING

The following references are a sampling of books from which readings may be helpful in preparation for ABPS written and
oral examinations. The list is suggested only, and may not be exhaustive for any particular examination.

Abramson, C. (ed.): Infectious Diseases of the Lower Extremities. Philadelphia, Williams & Wilkins, 1991. *

Adelaar, R.S.: Disorders of the Great Toe. Rosemont, IL, AAOS, 1997.

Adelaar, R.S.: Complex Foot and Ankle Trauma. Philadelphia, Lippincott, 1999.

Alexander, I.: The Foot: Examination, Diagnosis and Conservative Care, 2nd ed. New York, Churchill Livingstone, 1996.
Andreoli, T.E. (ed.): Cecil Essentials of Medicine, 5th ed. Philadelphia, Saunders, 2000.

Baxter, D.E.: Foot and Ankle in Sport. St. Louis, Mosby, 1995.

Berquist, T.H. (ed.): Radiology of the Foot and Ankle, 2nd ed. New York, Raven Press, 2000.

Birrer, R.B.: Common Foot Problems in Primary Care, 2nd ed.. St. Louis, Mosby, 1998.

Bouysset, M. (ed.): Bone and Joint Disorders of the Foot and Ankle: a Rheumatological Approach. New York, Springer-
Verlag, 1998.

Butterworth, R.: Color Atlas and Text of Forefoot Surgery. St. Louis, Mosby, 1992.

Cailliet, R.: Foot and Ankle Pain, 3rd ed. Philadelphia, Davis, 1997.

Canale, S.T.: Campbell’s Operative Orthopaedics, 9th ed. St. Louis, Mosby, 1997.

Carrel, J.M. (ed.): Complications in Foot and Ankle Surgery, 3rd ed. Baltimore, Williams & Wilkins, 1992. *

Cole, D.R. and DeLauro, T.M. (ed.): Neoplasms of the Foot and Leg. Baltimore, Williams & Wilkins, 1990. *

Condon, R. (ed.): Manual of Surgical Therapeutics, 9th ed. Boston, Little-Brown Co., 1996.

Coughlin, M.J. (ed.): Surgery of the Foot and Ankle, 7th ed. St. Louis, Mosby, 1999.

Coussons, T.R. (ed.): Manual of Medical Care of the Surgical Patient, 4th ed. Boston, Little-Brown Co., 1990. *

Crim, J.: Imaging of the Foot and Ankle. Philadelphia, Lippincott, 1996.

Deutsch, A.L. (ed.): MRI of the Foot and Ankle. New York, Raven Press, 1992.

DeValentine, S.J. (ed.): Foot and Ankle Disorders in Children. New York, Churchill Livingstone, 1991. *

Dockery, G.L.: Color Atlas of Foot and Ankle Dermatology. Philadelphia, Lippincott, 1999.

Dockery, G.L.: Cutaneous Disorders of the Lower Extremity. Philadelphia, Saunders, 1997.

* Out of print
Compiled: September 2000
ABPS 420 (April 2009) Page 32

Downey, M.S. and Malay, D.S.: Manual of Digital Surgery of the Foot. New York, Churchill Livingstone, 1991. *

Draves, D.J.: Anatomy of the Lower Extremity. Baltimore, Williams & Wilkins, 1986. *

Drennan, J.C. (ed.): The Child's Foot and Ankle. New York, Raven Press, 1992.

Falanga, V.: Leg & Foot Ulcers: a Clinician’s Guide. St. Louis, Mosby, 1995. *

Ferkel, R.D.: Arthroscopic Surgery: the Foot and Ankle. Philadelphia, Lippincott, 1996.

Forrester, D.M., et al.: Imaging of the Foot and Ankle. Rockville, Aspen Publ., 1988. *

Frykberg, R.G. (ed.): The High Risk Foot in Diabetes Mellitus. New York, Churchill Livingstone, 1991. *

Gerbert, J. (ed.): Textbook of Bunion Surgery, 2nd ed. Mt. Kisco, NY, Futura Publ. Co., 1991. *

Gilman, A.G.: Goodman and Gilman's Pharmacological Basis of Therapeutics, 9th ed. New York, Macmillan and Co.,
1996.

Goldman, D. (ed.): Perioperative Medicine: Medical Care of the Surgical Patient, 2nd ed. New York, McGraw-Hill, 1994.

Gould, J.S. (ed.): Operative Foot Surgery. Philadelphia, Saunders, 1994.

Greenfield, G.: Radiology of Bone Diseases, 5th ed. Philadelphia, Lippincott, 1990.

Guhl, J.F. (ed.): Foot and Ankle Arthroscopy, 2nd ed. . Thorofare, NJ, Slack Inc, 1993. *

Hansen, S.: Functional Reconstruction of the Foot and Ankle. Philadelphia, Lippincott, 2000.

Harkless, L.B.: Foot and Ankle Secrets. Philadelphia, Hanley & Belfus, 1997.

Harkless, L.B. and Krych, S.M. (eds.): Handbook of Common Foot Problems. New York, Churchill Livingstone, 1990. *

Heim, U.: Internal Fixation of Small Fractures. New York, Springer-Verlag, 1988.

Helal, B. (ed.): Surgery of Disorders of the Foot and Ankle. Philadelphia, Lippincott, 1996.

Hetherington, V.J. (ed.): Hallux Valgus and Forefoot Surgery. New York, Churchill Livingstone, 1994. *

Holmes, G.B. (ed.): Surgical Approaches to the Foot and Ankle. New York, McGraw-Hill, 1994.

Jahss, M.H.: Disorders of the Foot and Ankle, 3 Vols., 2nd ed. Philadelphia, Saunders, 1991.

Jay, R.M. (ed.): Current Therapy in Podiatric Surgery. Philadelphia, B.C. Decker, Inc., 1989. *

Jay, R.M. (ed.): Pediatric Foot and Ankle Surgery. Philadelphia, Saunders, 1999.

Johnson, K.A. (ed.): Foot and Ankle. New York, Raven Press, 1994.

Johnson, K.A.: Surgery of the Foot and Ankle. New York, Raven Press, 1989.

Joseph, W. S.: Handbook of Lower Extremity Infections. New York, Churchill Livingstone, 1990.

Kang, H.S. and Resnick, D.: MRI of the Extremities: An Anatomic Atlas. Philadelphia, Saunders, 1990.

* Out of print
Compiled: September 2000
ABPS 420 (April 2009) Page 33

Katzung, B.G.: Basic and Clinical Pharmacology, 7th ed. Stamford, CT, Appleton & Lange, 1998.

Kelikian, H.: Disorders of the Ankle. Philadelphia, Saunders, 1985. *

Kelikian, A.S. (ed.): Operative Treatment of the Foot and Ankle. Stamford, CT.: Appleton & Lange, 1999.

Klenerman, L.: Foot and Its Disorders, 3rd ed. Oxford, Blackwell, 1991.

Kominsky, S.J. (ed.): Advances in Podiatric Medicine & Surgery. St. Louis, Mosby, 1995-96. 3 Volumes *

Kominsky, S.J. (ed.): Medical and Surgical Management of the Diabetic Foot. St. Louis, Mosby, 1994.

Kozak, G.P. (ed.): Management of Diabetic Foot Problems, 2nd ed. Philadelphia, Saunders, 1995. *

Lankhorst, G.J. (ed.): Management of Ankle Injuries. Kirtland, WA, Hogrefe & Huber Publ., 1991. *

Levy, L.A. and Hetherington, V.J. (eds.): Principles and Practice of Podiatric Medicine. New York, Churchill Livingstone,
1989. *

Longnecker, D.E., et al. (eds.): Dripps’ Introduction to Anesthesia, 9th ed. Philadelphia, Saunders, 1997.

Luces, J.R. (ed.): Color Atlas of Foot Disorders. Mt. Kisco, NY, Futura Publ. Co., 1990. *

Lundeen, R.O.: Manual of Ankle and Foot Arthroscopy. New York, Churchill Livingstone, 1992. *

Lutter, L.D.: Atlas of Adult Foot and Ankle Surgery. St. Louis, Mosby, 1997.

Mandel, S. (ed.): Handbook of Lower Extremity Neurology. New York, Churchill Livingstone, 2000.

Marcinko, D.E. (ed.): Comprehensive Textbook of Hallux Abducto Valgus Reconstruction. St. Louis, Mosby, 1992. *

Marcinko, D.E.: Infection of the Foot: Diagnosis, Management and Prevention. St. Louis, Mosby, 1998.

Marcinko, D.E. (ed.): Medical and Surgical Therapeutics of the Foot and Ankle. Baltimore, Williams & Wilkins, 1992. *

Masquelet, A.C. (ed.): Atlas of Surgical Exposures of the Lower Extremity. Philadelphia, Lippincott, 1993.

Mayer, D.D., et al.: Foot and Ankle: a Sectional Imaging Atlas. Philadelphia, Saunders, 1993.

McDermott, J.E.: The Diabetic Foot. Rosemont, IL, AAOS, 1995.

McGlamry, E.D. (ed.): Comprehensive Textbook of Foot Surgery. 2 Vols., 2nd ed. Baltimore, Williams & Wilkins, 1992.

McMinn, R.M.H.: Color Atlas of Foot and Ankle Anatomy, 2nd ed. St. Louis, Mosby, 1996.

Merriman, L. (ed.): Assessment of the Lower Limb. New York, Churchill Livingstone, 1995.

Mizel, M.S.: Orthopedic Knowledge Update: Foot and Ankle, 2nd ed. Rosemont, IL, AAOS, 1998.

Morrissy, R.T. (ed.): Lovell and Winter's Pediatric Orthopedics, 2 vols., 4th ed. Philadelphia, Lippincott, 1996.

Muller, M.E. (ed.): Manual of Internal Fixation, Techniques Recommended by the AO-ASIF Group, 3rd ed. New York,
Springer-Verlag, 1991. *

* Out of print
Compiled: September 2000
ABPS 420 (April 2009) Page 34

Myerson, M. (ed.): Current Therapy in Foot and Ankle Surgery. St. Louis, Mosby, 1993. *

Myerson, M.S. (ed.): Foot and Ankle Disorders. Philadelphia, Saunders. 2000

Negus, D.: Leg Ulcers: A Practical Approach to Management, 2nd ed. Newton, MA., Butterworth, 1995.

Oestreich, A.E.: How to Measure Angles from Foot Radiographs. New York, Springer-Verlag, 1990. *

Oloff, L.M. (ed.): Musculoskeletal Disorders of the Lower Extremity. Philadelphia, Saunders, 1994.

Omer, G.E. (ed.): Management of Peripheral Nerve Problems, 2nd ed. Philadelphia, Saunders, 1994.

Ouriel, K.: Lower Extremity Vascular Disease. Philadelphia, Saunders, 1995.

Peacock, E.E., Jr.: Wound Repair, 3rd ed. Philadelphia, Saunders, 1984. *

Pfeffer, G.B. and Frey, C.C. (eds.): Current Practice in Foot and Ankle Surgery. New York, McGraw-Hill.
Volume 1: 1993 *, Volume 2: 1994.

Ranawat, C.S. (ed.): Disorders of the Heel, Rearfoot, and Ankle. New York, Churchill Livingstone, 1999.

Resnick, D.: Bone and Joint Imaging, 2nd ed. Philadelphia, Saunders, 1996.

Robbins, J.M.: Primary Podiatric Medicine. Philadelphia, Saunders, 1994.

Root, N.L.: Normal and Abnormal Function of the Foot. Los Angeles, Clinical Biomechanics Corp., 1977.

Salter, R.: Textbook of Disorders and Injuries of the Musculoskeletal System, 3rd ed. Baltimore, Williams & Wilkins, 1998.

Sammarco, G.J. (ed.): Foot and Ankle Manual, 2nd ed. Philadelphia, Lea & Febiger, 1998.

Sammarco, G.J. (ed.): Rehabilitation of the Foot and Ankle. St. Louis, Mosby, 1995.

Sarrafian, S.K.: Anatomy of the Foot and Ankle, 2nd ed. Philadelphia, Lippincott, 1993. *

Schwartz, S. (ed.): Principles of Surgery, 7th ed. New York, McGraw-Hill, 1999.

Scurran, B.L. (ed.): Foot and Ankle Trauma, 2nd ed. New York, Churchill Livingstone, 1995.

Shereff, M.J.: Atlas of Foot and Ankle Surgery. Philadelphia, Saunders, 1993.

Sherk, H.H. (ed.): Lasers in Orthopaedics. Philadelphia, Lippincott, 1990. *

Simons, G.W. (ed.): Clubfoot: the Present and a View of the Future. New York, Springer-Verlag, 1994. *

Stiehl, J.B. (ed.): Inman's Joints of the Ankle. Baltimore, Williams & Wilkins, 1991. *

Tachdjian, M.O.: Atlas of Pediatric Orthopedic Surgery. Philadelphia, Saunders, 1994.

Tachdjian, M.O.: Clinical Pediatric Orthopedics. Stamford, CT, Appleton & Lange, 1997.

Tierney, L.M. (ed.): Current Medical Diagnosis and Treatment, 40th ed. Norwalk, Appleton & Lange, 2001.

* Out of print
Compiled: September 2000
ABPS 420 (April 2009) Page 35

Tollafield, D.R. (ed.): Clinical Skills in Treating the Foot. New York, Churchill Livingstone, 1997.

Valmassy, R. L. (ed.): Clinical Biomechanics of the Lower Extremities. St. Louis, Mosby, 1995.

Witkowski, J.: Color Atlas of Cutaneous Disorders of the Lower Extremities. New York, Igaku-Shoin, 1993. *

Weinstein, S.L. (ed.): Turek's Orthopedics, 2 vols., 5th ed. Philadelphia, Lippincott, 1994.

Williams, P.L. (ed.): Gray's Anatomy, 38th ed.. New York, Churchill Livingstone, 1995.

Wulker, N. An Atlas of Foot and Ankle Surgery. St. Louis, Mosby, 1998.

Yao, J.S.T.: Ischemic Extremity: Advances in Treatment. Norwalk, Appleton-Lange, 1995.

Young, J.R. (ed.): Peripheral Vascular Diseases, 2nd ed. St. Louis, Mosby, 1996.

Zier, B.G. (ed.): Essentials of Internal Medicine in Clinical Podiatry. Philadelphia, Saunders, 1990. *

* Out of print
Compiled: September 2000
NATIONAL BOARD
OF
PODIATRIC MEDICAL EXAMINERS

PART II
Clinical Science Examination

PRACTICE TEST 1

The Part II Practice Tests are representative of the content covered in the Part II Examination. They
include question formats found in the actual examination. They also include questions of varying
difficulty. A candidate’s performance on a Practice Test does not guarantee similar performance on
the actual examination.

Copyright © 2008 by the National Board of Podiatric Medical Examiners.


All rights reserved.
3 Practice Test 1

CLINICAL SCIENCE EXAMINATION


PRACTICE TEST 1

60 questions

Directions for questions 1-56: These questions are followed by four suggested answers. Select the one answer that
is best in each case.

NOTE: Throughout this test, the term “medial oblique foot” refers to a nonweightbearing medial oblique position
in which the film is flat on the orthoposer, the medial side of the foot is closest to the film and the sole forms a 45°
angulation with the film, and the central beam is 90° to the film (the tubehead is angulated 0°). The converse is true
for the term “lateral oblique.”

1. A 6-month-old child exhibits a limitation of hip 4. Which of the following is the most appropriate
abduction and an increase in thigh folds on the antibiotic therapy for pseudomembranous colitis?
left side. The most likely diagnosis is
(A) Vancomycin
(A) cerebral palsy (B) Gentamicin
(B) transient synovitis of the left hip (C) Ciprofloxacin
(C) a congenitally dislocated left hip (D) Ticarcillin
(D) a fracture of the left femur
5. Vertical talus presents with which of the
2. In a podiatric physician’s office, the best method following x-ray changes?
to prevent HIV infection and/or hepatitis B
(A) Dorsiflexed talus
infection is to use
(B) Dislocated navicular
(A) an eyewash station (C) Increased first metatarsal declination angle
(B) thorough handwashing (D) High calcaneal inclination angle
(C) appropriate handling and disposal of sharps
(D) appropriate handling and disposal of waste 6. Which of the following is a true statement about a
fracture of the styloid process of the fifth
3. Following fifth-toe surgery, a patient reports that metatarsal?
the toe is unstable and feels “floppy.” X-rays
(A) It is also known as a Jones fracture.
reveal loss of bone mass including the distal
(B) The mechanism of injury usually involves a
two-thirds of the proximal phalanx and most of
strong force applied by the fibularis
the middle phalanx. A flail toe condition is
(peroneus) brevis.
diagnosed. The best procedure for this patient
(C) It needs to be treated surgically due to an
would be
increased likelihood of nonunion.
(A) a distal Syme's amputation (D) Treatment may include immobilization, with
(B) amputation of the metatarsophalangeal joint the foot in an inverted position.
(C) removal of the base of the proximal phalanx
(D) surgical syndactylization of the fourth and 7. The most important factor for the treatment of a
fifth toes navicular stress fracture involves early
(A) short leg casting
(B) internal fixation
(C) nonweightbearing
(D) use of electrical stimulation
Practice Test 1 4

8. Which of the following is a radiographic 14. Which of the following impressions from an MRI
hallmark of dominant sagittal plane pronatory would be most consistent with a diagnosis of
compensation? osteomyelitis of the first metatarsal?
(A) Navicular bulge (A) A decreased signal intensity in a
(B) Increased cuboid abduction angle T1-weighted image
(C) Decreased calcaneal inclination angle (B) An increased signal intensity in a
(D) Decreased talar declination angle T2-weighted image
(C) A decreased signal intensity in a
9. A 4-year-old girl who has asthma presents with a T2-weighted image
persistent, pruritic, vesiculating, erythematous (D) A signal void in a T1-weighted image
eruption of both feet. Which of the following is
the most likely diagnosis? 15. The most common mechanism of ankle fracture
is
(A) Psoriasis
(B) Herpes simplex virus infection (A) supination adduction
(C) Atopic dermatitis (B) supination external rotation
(D) Lichen planus (C) pronation external rotation
(D) pronation abduction
10. All of the following conditions contribute to
hallux varus deformity EXCEPT 16. A 62-year-old obese patient has been hospitalized
for the treatment of an infected foot ulcer. The
(A) a negative intermetatarsal angle
patient is placed on subcutaneous heparin for
(B) excision of the fibular sesamoid
prophylaxis against deep vein thrombosis. Which
(C) aggressive medial capsulorrhaphy
of the following laboratory findings would
(D) lengthening of the extensor tendon
indicate effective heparin management?
11. The scope of practice of podiatric medicine is (A) Prothrombin time (PT) = 14.0 sec
primarily determined by (B) Partial thromboplastin time
(PTT) = 60.0 sec
(A) act of Congress
(C) Bleeding time = 5 min
(B) state law
(D) Platelet count = 150,000/cu mm
(C) the American Podiatric Medical Association
(D) the U.S. Department of Health and Human
17. A patient complains of pain in the posterior
Services
aspect of the ankle with palpation. Pain is
particularly aggravated by dorsiflexion and
12. Which of the following is the most sensitive
plantarflexion of the hallux. Which of the
blood test for neuromuscular disorders?
following would be the best radiographic view to
(A) Creatine phosphokinase confirm the diagnosis?
(B) Lactate dehydrogenase
(A) An anteroposterior view of the affected ankle
(C) Alkaline phosphatase
(B) A medial oblique view of the foot
(D) Acid phosphatase
(C) A lateral view of the foot and ankle
(D) A Harris-Beath (calcaneal axial) view
13. Proprioception and vibratory sensations are
mediated by which of the following pathways?
(A) Posterior column
(B) Corticospinal tract
(C) Extrapyramidal tract
(D) Lateral spinothalamic tract
5 Practice Test 1

18. In the application of a short leg cast, the main 21. An individual with bronchial asthma triggered by
advantage of plaster of Paris over synthetic an allergic reaction is likely to have elevated
materials is that plaster of Paris is more
(A) monocytes in the WBC differential
(A) durable (B) lymphocytes in the WBC differential
(B) moldable (C) eosinophils in the WBC differential
(C) porous (D) sodium in the serum chemistry
(D) radiolucent
22. Which of the following is a true statement about
19. Which of the following is an appropriate rigid forefoot valgus?
cleansing agent for a surgeon who is allergic to
(A) It is the cause of medial ankle sprains.
iodine?
(B) It is compensated for by subtalar joint
(A) Acetic acid supination.
(B) Hexachlorophene (C) It is confirmed by a positive anterior drawer
(C) Dakin’s solution sign.
(D) Povidone-iodine (D) It can be identified on a weightbearing x-ray
if the talocalcaneal angle is increased.
20. A doctor may NOT refuse to treat a prospective
patient because the patient
(A) has AIDS
(B) is unable to pay the doctor
(C) appears to be under the influence of drugs
(D) has been a plaintiff in previous malpractice
suits
Practice Test 1 6

23.

Initial treatment of the condition shown in this radiograph would most appropriately include

(A) resection of the spur and plantar fasciotomy


(B) transcutaneous nerve stimulation
(C) nonweightbearing casting for 2 weeks
(D) mechanical support of the plantar fascia

____________________________________________________________________________________________

24. Blood pressure may drop when spinal anesthesia 26. In a medical malpractice action, the plaintiff must
is used because prove that the podiatric physician did not meet
the standard of care, which is defined as
(A) the patient is recumbent
(B) sympathectomy may cause vasodilatation (A) that care necessary to correct the patient’s
(C) motor blockade affects the vasoconstrictor problem so the patient is cured
muscles (B) the care a reasonable podiatric physician
(D) sensory blockade obviates the need for would provide under the same or similar
higher pressure circumstances
(C) the care a compassionate podiatric physician
25. Which of the following casting techniques allows would provide under the same or similar
the best visualization of the neutral subtalar joint circumstances
position and the forefoot-to-rearfoot relationship? (D) the best care that could be provided by any
podiatric physician under the same or
(A) Prone suspension
similar circumstances
(B) Supine modified Langer
(C) Biofoam partial weightbearing
(D) BioVac inshoe vacuum
7 Practice Test 1

27. Arrhythmia may occur from the toxic effects of 31. A 12-year-old patient who is being evaluated for
which of the following medications? hip pain of several weeks’ duration has other
complaints that are more acute. The patient
(A) Hydrochlorothiazide
complains of headache and fever, some
(B) Nifedipine
photophobia, and “just not feeling well.” These
(C) Digitalis
symptoms began yesterday and appear to be
(D) Quinine
getting worse. The podiatric physician notices
that the patient moves slowly and carefully and
28. Balancing the heel of a positive cast used to
that the neck is painful and somewhat stiff.
fabricate a functional orthotic in an inverted
Temperature is 103°F. There is no sign of head
position effectively decreases
trauma, and the pupils are equal and reactive.
(A) supination around the subtalar joint axis The lungs and heart are normal. There is no skin
(B) supination around the midtarsal joint axis rash. The left hip is slightly tender over the
(C) pronation around the subtalar joint axis trochanter but has full range of motion. The
(D) dorsiflexion around the first patient appears neurologically intact. Which of
metatarsophalangeal joint axis the following is the most likely diagnosis for
these acute symptoms?
29. The laying down of new bone surrounding an
(A) Osteomyelitis
area of infection is referred to as
(B) Lyme disease
(A) a cloaca (C) Meningitis
(B) an involucrum (D) Viral upper respiratory infection
(C) a sequestrum
(D) emphysema 32. A lateral weightbearing roentgenogram of a
flexible flatfoot will demonstrate
30. Wound dehiscence may result from all of the
(A) a well-visualized sinus tarsi
following EXCEPT
(B) a plantarflexed attitude of the talus
(A) infection (C) an increased angle of declination of the
(B) hematoma metatarsals
(C) sutures that are too tight (D) a decrease in the talocalcaneal angle
(D) everted wound margins
Practice Test 1 8

33. Which of the following intravenous medications 38. A town has a population of 500,000 in a
is indicated for nausea? particular year. During that year there are 500
live births and 500 deaths, 100 of which are of
(A) Midazolam
children under 1 year of age. What is the infant
(B) Meperidine
mortality rate?
(C) Phenobarbital
(D) Promethazine (A) 1/1,000
(B) 5/1,000
34. Ultrasound therapy is an appropriate treatment (C) 200/1,000
for which of the following? (D) 500/1,000
(A) Arterial insufficiency
39. Which of the following bone tumors is
(B) Enthesopathy
malignant?
(C) Thrombophlebitis
(D) A stress fracture (A) Chondroma
(B) Giant cell tumor
35. Which of the following would be an appropriate (C) Ewing’s sarcoma
cast for a patient who has undergone repair for an (D) Osteoid osteoma
Achilles tendon rupture?
40. A 65-year-old woman has undergone an
(A) A long leg cast with the ankle plantarflexed
emergency incision and drainage of a plantar
20º
ulcer associated with ascending cellulitis. She
(B) A long leg cast with the foot 90º to the leg
has a history of insulin-dependent diabetes,
(C) A short leg cast with the ankle plantarflexed
hypertension, and chronic venous insufficiency.
20º
On the third postoperative day, the patient
(D) A short leg cast with the foot 90º to the leg
complains of severe substernal chest pain
radiating to the back. Which of the following is
36. The primary function of neutrophils in the body’s
the LEAST likely cause of the pain?
defense is to
(A) Costal chondritis
(A) assist lymphocytes in response to
(B) Pulmonary embolism
anaphylaxis
(C) Myocardial infarction
(B) phagocytize and destroy microorganisms
(D) Dissecting aortic aneurysm
(C) initiate the IgE immunologic response
(D) release histamine
41. After administration of a general anesthesia, a
podiatric physician injects 8 cubic centimeters of
37. Which of the following phases of wound healing
2% lidocaine in Mayo fashion into a 42-year-old
is characterized by the release of vasoactive
patient in order to perform a modified McBride
amines and initial vasoconstriction?
procedure. During the prep and drape, the patient
(A) Proliferative experiences increased temperature, arrhythmias,
(B) Inflammatory and muscle rigidity. The patient is most likely
(C) Maturation experiencing
(D) Fibroblastic
(A) anaphylaxis
(B) malignant hyperthermia
(C) a hypertensive crisis
(D) a myocardial infarction
9 Practice Test 1

42. Performing an inversion stress test with the ankle 46. Fever that occurs during the first postoperative
in a neutral position is useful in the detection of day after a general anesthetic has been
injury to which of the following ligaments? administered is most likely the result of which of
the following?
(A) Calcaneofibular
(B) Posterior talofibular (A) Atelectasis
(C) Anterior talofibular (B) Infection
(D) Anterior inferior tibiofibular (C) Malignant hyperthermia
(D) Urinary tract infection
43. Which of the following nail changes is
characteristic of lichen planus? 47. Tibial varum should be measured with the
rearfoot
(A) Pitting
(B) Pterygium (A) parallel with the leg
(C) Thickening (B) perpendicular to the ground
(D) Oil drop staining (C) in the neutral calcaneal stance position
(D) in the relaxed calcaneal stance position
44. If all other factors remain unchanged, a blacker
radiograph can be produced by 48. Which of the following tendons is exposed during
neuroma surgery performed on the third
(A) increasing the source-to-image distance
interspace through a dorsal approach?
(B) increasing the mA
(C) decreasing the kVp (A) Plantar interosseous to the third toe
(D) decreasing the exposure time (B) Flexor digitorum brevis to the third toe
(C) Lumbrical to the fourth toe
45. An increase in the thickness of the calcaneal fat (D) Dorsal interosseous to the fourth toe
pad is a cardinal sign of which of the following
conditions?
(A) Hyperparathyroidism
(B) Cushing’s syndrome
(C) Acromegaly
(D) Hypervitaminosis A
Practice Test 1 10

49.

Which type of image is depicted in this slide?


(A) MRI T1
(B) MRI T2
(C) CT scan
(D) Bone scan
____________________________________________________________________________________________

50. A 65-year-old patient presents for a 52. After surgery for a dislocated hammer toe, the
bunionectomy. The patient has been healthy digit is still slightly dorsiflexed at the
and takes no medications. A reasonable metatarsophalangeal joint, despite the fact that
intervention to prevent pulmonary emboli a complete sequential reduction was performed.
postoperatively would be At the time of surgery, the podiatric physician
should have done which of the following?
(A) elastic stockings
(B) warfarin (A) Performed a flexor tendon lengthening
(C) subcutaneous heparin procedure.
(D) an inferior vena cava filter (B) Performed an extensor tendon lengthening
procedure.
51. All of the following are appropriate drugs for the (C) Stabilized the digit to the base of the
treatment of rheumatoid arthritis EXCEPT proximal phalanx with a Kirschner wire.
(D) Stabilized the digit to the base of the
(A) penicillamine
proximal phalanx and across the
(B) prednisone
metatarsophalangeal joint with a Kirschner
(C) pentoxifylline
wire.
(D) piroxicam
11 Practice Test 1

53. Lidocaine can be used to 56. Peroneal muscular atrophy is best known as
(A) increase cardiac contractility (A) Charcot-Marie-Tooth disease
(B) control premature ventricular contractions (B) Refsum’s disease
(C) control atrial flutter (C) Dejerine-Sottas disease
(D) control atrial fibrillation (D) Guillain-Barré syndrome

Questions 54-55 refer to the following case. 57. Which of the following are mandates of state
departments of health? Select the two that apply.
A 67-year-old woman presents with complaints of a
(A) Manage educational services
tender, progressively flattening arch on the left foot.
(B) Manage environmental services
She has no pain in the right foot, and modifying her
(C) Set public health policies and standards
shoe gear to softer, more “comfortable” styles has not
(D) Collect, analyze, and disseminate health
helped alleviate her symptoms. Her symptoms began
information
approximately 6 weeks ago when she heard a
(E) Set drug dispensing standards
popping sound in her foot.
58. Which of the following accessory bones are
54. Which of the following is the most likely
sesamoid bones? Select the two that apply.
diagnosis?
(A) Os peroneum
(A) A Lisfranc’s fracture dislocation
(B) Os trigonum
(B) A form of arthritis
(C) Os supra naviculare
(C) A stress fracture of the fifth metatarsal
(D) Accessory navicular type I
(D) A ruptured tibialis posterior tendon
(E) Accessory navicular type II
55. Which of the following diagnostic modalities
59. Which of the following are potential signs of
would be best for confirming the diagnosis?
early sepsis in the elderly? Select the three that
(A) An MRI scan of both feet apply.
(B) A CT scan of both feet
(A) Agitation
(C) Multiple x-ray views
(B) Respiratory acidosis
(D) An arthritis panel to include SMAC-12,
(C) Respiratory rate of 24
ANA, ESR, C-reactive protein, RA, and
(D) Polyuria
LE prep
(E) Nocturnal disorientation (“sundowning”)

____________________________________________________________________________________________

60. List the following steps in a McBride bunionectomy in the order in which they are performed (from first to
last).
(A) Section of the deep transverse intermetatarsal ligament 1. _____
(B) Release of the conjoined adductor tendon 2. _____
(C) Medial capsulotomy 3. _____
(D) Removal of the fibular sesamoid 4. _____
(E) Exostectomy 5. _____

END OF PRACTICE TEST 1


NATIONAL BOARD
OF
PODIATRIC MEDICAL EXAMINERS

PART II
Clinical Science Examination

PRACTICE TEST 2

The Part II Practice Tests are representative of the content covered in the Part II Examination. They
include question formats found in the actual examination. They also include questions of varying
difficulty. A candidate’s performance on a Practice Test does not guarantee similar performance on
the actual examination.

Copyright © 2008 by the National Board of Podiatric Medical Examiners.


All rights reserved
15 Practice Test 2

CLINICAL SCIENCE EXAMINATION


PRACTICE TEST 2

60 questions

Directions for questions 1-56: These questions are followed by four suggested answers. Select the one answer that
is best in each case.

NOTE: Throughout this test, the term “medial oblique foot” refers to a nonweightbearing medial oblique position
in which the film is flat on the orthoposer, the medial side of the foot is closest to the film and the sole forms a 45°
angulation with the film, and the central beam is 90° to the film (the tubehead is angulated 0°). The converse is true
for the term “lateral oblique.”

1. Generalized osteopenia is a characteristic feature 5. All of the following conditions may cause pes
of planus EXCEPT
(A) osteomyelitis (A) vertical talus
(B) osteosarcoma (B) gastrocnemius equinus
(C) osteoporosis (C) posterior tibial tendon rupture
(D) osteoarthritis (D) Charcot-Marie-Tooth disease

2. Which of the following anesthetics is normally 6. The film/cassette is placed vertically in the
administered intravenously? orthoposer for which of the following views?
(A) Thiopental (A) Weightbearing lateral
(B) Enflurane (B) Weightbearing dorsoplantar
(C) Halothane (C) Weightbearing calcaneal axial
(D) Nitrous oxide (D) Nonweightbearing medial oblique

3. A 19-year-old male patient presents with an itchy 7. Which of the following tests is used in the
and burning rash on the bottom of his foot. management of patients on warfarin?
Physical examination reveals patchy scaling on
(A) Erythrocyte sedimentation rate (ESR)
an erythematous base. The lesions are dry and
(B) Partial thromboplastin time (PTT)
extend to the sides of the foot in a moccasin
(C) Prothrombin time (PT)
distribution. Which of the following would be an
(D) Rumpel-Leede test
appropriate presumptive diagnosis?
(A) Psoriasis 8. When implant surgery is performed, the most
(B) Tinea pedis appropriate antibiotic prophylaxis is provided by
(C) Lichen planus which of the following?
(D) Pityriasis rubra pilaris
(A) First-generation cephalosporins
(B) Third-generation cephalosporins
4. Which of the following is the single most
(C) Aminoglycosides
important preventable risk factor for peripheral
(D) Quinoline
vascular disease?
(A) Stress
(B) High fat diet
(C) Cigarette smoking
(D) Alcohol abuse
Practice Test 2 16

9. Which of the following would be most useful in 13. Ankle stress radiographs of an isolated rupture of
the treatment of a patient with ankle fusion the calcaneofibular ligament will reveal a
secondary to trauma?
(A) positive anterior drawer sign and negative
(A) A stiff sole shoe talar tilt
(B) A rocker-bottom shoe (B) positive anterior drawer sign and positive
(C) A shoe with a Thomas heel talar tilt
(D) A lateral heel flare (C) negative anterior drawer sign and negative
talar tilt
10. A 28-year-old female patient presents with pain (D) negative anterior drawer sign and positive
in the left forefoot that began after a long hike on talar tilt
a mountainous trail. There is no history of
trauma. Physical examination reveals nontender 14. A 38-year-old patient with diabetes mellitus is in
range of motion of the metatarsophalangeal the preoperative holding area and surgery has
joints. Deep palpation of the intermetatarsal been delayed for 2 hours. One-half of the
spaces does not reproduce the patient’s patient’s prescribed AM NPH insulin dose was
symptoms; however, palpation along the second administered at 7:00 AM. It is now 9:30 AM and
metatarsal is painful. Which of the following is the patient suddenly becomes disoriented and
the most likely diagnosis? begins to perspire profusely. Which of the
following actions would be most appropriate?
(A) Intermetatarsal space neuroma in continuity
(B) Capsulitis of the second metatarsophalangeal (A) Obtain an electrocardiogram (EKG).
joint (B) Perform a finger-stick blood glucose
(C) Stress fracture of the second metatarsal evaluation.
(D) Degenerative joint disease of the second (C) Administer the remaining half of the
metatarsophalangeal joint patient’s prescribed AM NPH insulin.
(D) Administer midazolam 2 mg to the patient.
11. The radiographic weightbearing lateral view of
the foot requires 15. All of the following are radiographic features
of closed-kinetic-chain subtalar joint pronation
(A) placing the film flat on the orthoposer
EXCEPT an
(B) placing the lateral side of the foot against the
film (A) increased metatarsus adductus angle
(C) directing the central beam at the ankle (B) increased talar declination angle
(D) angling the tubehead 90° from vertical (C) increased talocalcaneal angle
(D) anterior break in the cyma line
12. Which of the following findings is most
commonly observed in the first phase of 16. A podiatric physician who submits a report of
neuropathic arthropathy (Charcot’s joint)? child abuse will be
(A) Ankylosis (A) liable if no abuse in fact occurred
(B) Cyanosis (B) liable if the report was true but made with
(C) Pain malice
(D) Erythema (C) immune from liability if there was
reasonable cause to suspect abuse
(D) immune from liability only if the podiatric
physician personally examined the child
17 Practice Test 2

17. Which of the following antibiotic combinations, 21. Which of the following statements is true
given orally, would offer the best overall concerning therapeutic ultrasound?
coverage for gram-positive, gram-negative, and
(A) It is best used without a coupling agent.
anaerobic infections?
(B) It will dissolve intratendinous calcifications.
(A) Penicillin and ofloxacin (C) It can be used to diagnose acute and chronic
(B) Erythromycin and tendon ruptures.
trimethoprim/sulfamethoxazole (D) Sonic energy increases the temperature in
(C) Clindamycin and ciprofloxacin deep tissues.
(D) Metronidazole and oxacillin
22. Decreased metatarsal bone girth is seen in which
Questions 18-19 refer to the following case. of the following?
(A) Osteogenesis imperfecta
A 12-year-old patient presents with pain on the
(B) Thyroid acropachy
lateral side of the right ankle when standing. The
(C) Acromegaly
subtalar joint is restricted in motion, and pain is
(D) Paget’s disease
elicited on palpation of the fibular (peroneal)
tendons.
23. The most common cell types seen in cases of
immediate hypersensitivity are
18. A lateral x-ray would most probably demonstrate
(A) lymphocytes and monocytes
(A) a large os trigonum
(B) neutrophils and eosinophils
(B) a posteriorly broken cyma line
(C) mast cells and basophils
(C) an increased calcaneal inclination
(D) band cells
(D) dorsal talar head and neck beaking
24. Which of the following local anesthetics has the
19. If the medial oblique view of the foot is negative,
shortest duration of action?
which of the following views will probably be
positive? (A) Tetracaine
(B) Lidocaine
(A) Harris-Beath (calcaneal axial)
(C) Bupivacaine
(B) Anteroposterior
(D) Etidocaine
(C) Sesamoidal axial
(D) Mortise ankle
___________________________________________

20. Dual, curvilinear soft tissue calcifications that are


parallel to one another and somewhat serpiginous
in appearance are radiographic features of
(A) phleboliths
(B) atherosclerosis
(C) Mönckeberg’s arteriosclerosis
(D) tumoral calcinosis
Practice Test 2 18

25.

The radiograph shown is most characteristic of

(A) osteoarthritis
(B) gouty arthritis
(C) psoriatic arthritis
(D) rheumatoid arthritis
19 Practice Test 2

26. Which of the following is the first step in basic 31. Stabilization of toes at the metatarsophalangeal
life support (BLS)? joint during midstance is aided by which of the
following muscles?
(A) Ventilation
(B) Airway restoration (A) Gastrocnemius
(C) Restoration of circulation (B) Quadratus plantae
(D) Assessment for unresponsiveness (C) Tibialis posterior
(D) Extensor digitorum longus
27. A 42-year-old patient underwent second
interspace neuroma surgery 2 years ago. The 32. Running in a shoe with a sole that is too rigid
patient now complains that the second toe is across the metatarsophalangeal joints increases a
deviated medially. Damage to which of the person’s risk of developing
following muscles during surgery would result in
(A) piriformis syndrome
this finding?
(B) posterior tibial tendinitis
(A) First plantar interossei (C) fibularis (peroneus) longus tendinitis
(B) Second dorsal interossei (D) anterior tibial shin splints
(C) Third dorsal interossei
(D) Lumbricales 33. The hinge axis concept is useful when planning
which of the following procedures?
28. Which of the following is a
(A) A hallux interphalangeal joint arthrodesis
CONTRAINDICATION to the resection
(B) A subtalar joint arthroereisis
of a calcaneonavicular coalition?
(C) A metatarsal basilar osteotomy
(A) Peroneal spasm (D) A Kidner procedure
(B) Talar beaking
(C) A completely ossified bar 34. Intravenous barbiturates can produce all of the
(D) Degenerative disease of the subtalar joint following EXCEPT
(A) hypnosis
29. An apparently nervous 25-year-old female patient
(B) unconsciousness with increased doses
demonstrates lid lag during examination, along
(C) myocardial excitation with increased doses
with a fine tremor of her outstretched hands.
(D) interference with the sensitivity of the
Her skin is warm, moist, and smooth. Her
medullary respiratory center to carbon
nervousness is most likely the result of which of
dioxide
the following?
(A) Addison’s disease 35. Capitation is generally part of which of the
(B) Cushing’s syndrome following health care delivery systems?
(C) Pheochromocytoma
(A) PPO
(D) Hyperthyroidism
(B) IPA
(C) HMO
30. The proliferative phase of wound healing is
(D) Fee-for-service
characterized by
(A) neutrophilic infiltrates 36. From a plantar approach, the common digital
(B) remodeling and scar tissue formation nerve divides into two proper digital nerves
(C) epithelialization and connective tissue repair which, in relation to the deep transverse
(D) inflammation and decreased tensile strength metatarsal ligament, are
(A) superficial
(B) superior
(C) proximal
(D) deep
Practice Test 2 20

37. A 64-year-old male patient complains of a cold 41. Which type of exercise should be recommended
left foot and cramping pain in his left calf and to minimize muscle atrophy in a patient with a
buttock when walking. He also states that he is short leg cast?
impotent. Examination reveals a weak femoral
(A) Concentric
pulse on the left side and nonpalpable pulses at
(B) Isotonic
the popliteal and pedal arteries. It is most likely
(C) Isokinetic
that an occlusion has occurred in which of the
(D) Isometric
following arteries?
(A) Deep femoral 42. When a dressing is applied after a flexor to
(B) Superficial femoral extensor tendon transfer for stabilization, it is
(C) Common femoral most important to
(D) Common iliac
(A) allow the toe to seek its own level
(B) apply povidone-iodine to prevent infection
38. A 21-year-old basketball player sustained a
(C) plantarflex the proximal phalanx at the
complete rupture of the Achilles tendon. It is
metatarsophalangeal joint
now 5 days after the injury and the patient has
(D) dorsiflex the proximal phalanx at the
been immobilized in a posterior splint. The most
metatarsophalangeal joint
appropriate treatment at this time is
(A) a primary anastomosis with an augmentation 43. Which of the following therapies is
graft if necessary recommended for initial treatment of an acute
(B) resection of the fibrous plug and an asthmatic attack?
augmentation graft of the defect
(A) Intravenous theophylline
(C) a long leg cast for 4 weeks followed by a
(B) Inhaled glucocorticoids
short leg cast for 4 weeks
(C) Inhaled beta-adrenergic agonists
(D) a short leg cast for 6 weeks followed by
(D) Intramuscular antihistamines
aggressive physical therapy
44. A podiatric physician suspects that a patient
39. Radiographs of a college basketball player who
may be abusing the hydrocodone prescribed for
presents with a foot injury reveal a minimally
the patient’s foot surgery. All of the following
displaced fracture of the proximal shaft of the
signs and symptoms would indicate hydrocodone
fifth metatarsal. Treatment should consist of
abuse EXCEPT
which of the following?
(A) miosis
(A) Tape and immobilization
(B) diarrhea
(B) A nonweightbearing cast
(C) drowsiness
(C) An Ace bandage and ambulation as tolerated
(D) pruritus of the nose
(D) An Unna’s boot and partial weightbearing
with crutches
45. Which of the following is true about the
talocalcaneal angle?
40. All of the following are clinical signs of anabolic
steroid use in young athletes EXCEPT (A) It is 8°-10° in the normal foot.
(B) It is increased in a flatfoot deformity.
(A) alopecia
(C) It is increased in a supinated foot deformity.
(B) hypertension
(D) It is decreased in a pronated foot deformity.
(C) osteoporosis
(D) temporary sterility
21 Practice Test 2

46. In civil court, the performance of a procedure on 51. Which of the following is the best treatment for a
a patient without the patient’s informed consent dermatofibroma on the anterior ankle?
could be considered
(A) Fulguration or laser ablation
(A) assault (B) Salicylic acid treatments with debridement
(B) malpractice (C) Surgical excision and pathological
(C) abuse examination
(D) battery (D) Wide excision, pathological examination,
and probable chemotherapy
47. Which of the following gait patterns is most
commonly seen in cases of lower motor neuron 52. The doctor-patient privilege may be waived
pathology?
(A) by the patient only
(A) Steppage (B) by the doctor only
(B) Circumduction (C) only if both the patient and the doctor agree
(C) Festinating to waive it
(D) Ataxic (D) only with the consent of a judge

48. A 30-year-old male patient with a history of 53. How often should a patient’s vital signs be
seizures since childhood has a seizure during a monitored following general inhalation
routine office visit for a wound check. The anesthesia?
podiatric physician should immediately
(A) Every 15 minutes until stable
(A) insert a bite block (B) Every hour
(B) turn the patient on his side (C) Twice a day
(C) administer phenytoin, 500 mg, IV (D) Every nursing shift
(D) administer diazepam, 10 mg, IV
54. The risk of which of the following is increased
49. Which of the following is a typical radiographic most for patients who undergo hip and knee-joint
finding in cases of osteochondroma? replacement procedures?
(A) Epiphyseal location (A) Pneumonia
(B) Moth-eaten appearance (B) Atelectasis
(C) Osseous protuberance that grows toward the (C) Pulmonary embolism
joint (D) Myocardial infarction
(D) Osseous protuberance with the cortex
continuous with the parent bone 55. A patient’s blood laboratory results reveal
elevated levels of creatine phosphokinase,
50. In podiatric radiology, the standard anode-film aldolase, and alanine transaminase. Which of the
distance usually falls within a range of following is the most likely diagnosis?
(A) 12-17 inches (A) Muscular dystrophy
(B) 18-23 inches (B) Rheumatoid arthritis
(C) 24-30 inches (C) Aneurysmal bone cyst
(D) 31-36 inches (D) Cerebral palsy
Practice Test 2 22

58. Which of the following radiographic angles in a


56. If a person falls from a height, causing the lateral dorsoplantar view are most indicative of talipes
process of the talus to drive into the calcaneus equinovarus? Select the three that apply.
over the neutral triangle, the most likely calcaneal
(A) Talo-first metatarsal angle of 10°
fracture would be
(B) Talo-first metatarsal angle of 5°
(A) a beak fracture (C) Kite’s angle (talocalcaneal angle) of 5°
(B) a sustentaculum fracture (D) Kite’s angle (talocalcaneal angle) of 25°
(C) a joint depression fracture (E) Bohler’s angle of 30°
(D) an avulsion fracture
59. Typical presentations of multiple myeloma
57. In a T1-weighted MRI of the foot, which of the include which of the following? Choose the three
following lesions will have the lowest signal that apply.
intensity? Select the three that apply.
(A) Bone pain
(A) Ganglion cyst (B) Renal failure
(B) Unicameral bone cyst (C) Hypocalcemia
(C) Adamantinoma (D) Normocytic anemia
(D) Interosseous lipoma (E) Multiple areas of uptake on a bone scan
(E) Hemangioma

___________________________________________________________________________________________

60. List the order in which the components of the talipes equinovarus deformity should be corrected during casting
treatment (from first to last).
(A) Adduction component 1. _____
(B) Inversion component 2. _____
(C) Equinus component 3. _____
(D) Internal torsional component at the tibia 4. _____

END OF PRACTICE TEST 2


NATIONAL BOARD
OF
PODIATRIC MEDICAL EXAMINERS

PART II
Clinical Science Examination

PRACTICE TEST 3

The Part II Practice Tests are representative of the content covered in the Part II Examination.
They include question formats found in the actual examination. They also include questions of
varying difficulty. A candidate’s performance on a Practice Test does not guarantee similar
performance on the actual examination.

Copyright © 2008 by the National Board of Podiatric Medical Examiners.


All rights reserved.
25 Practice Test 3

CLINICAL SCIENCE EXAMINATION


PRACTICE TEST 3

60 questions

Directions for questions 1-55: These questions are followed by four suggested answers. Select the one answer that
is best in each case.

NOTE: Throughout this test, the term “medial oblique foot” refers to a nonweightbearing medial oblique position
in which the film is flat on the orthoposer, the medial side of the foot is closest to the film and the sole forms a 45°
angulation with the film, and the central beam is 90° to the film (the tubehead is angulated 0°). The converse is true
for the term “lateral oblique.”

1. Diazepam is used in the anesthetic setting to 5. When should a podiatric physician prescribe
reduce ambulation and range-of-motion exercises for a
postoperative patient with a history of deep vein
(A) cardiac depression
thrombosis?
(B) respiratory tract secretions
(C) postoperative pain (A) Before symptoms develop, for prevention
(D) anxiety (B) After symptoms develop but before a
definitive diagnosis is made
2. Which of the following classes of drugs is used to (C) After a definitive diagnosis is made
treat osteoporosis? (D) Three days after a venous thrombus has been
identified
(A) Calcium channel blockers
(B) Bisphosphonates
6. Which of the following conditions has a strong
(C) Antiestrogens
(short) T1 signal on MRI?
(D) Steroids
(A) Lipoma
3. Radiographically, the inversion ankle stress view (B) Osteomyelitis
is most useful in evaluating which of the (C) Hemangioma
following conditions? (D) Ganglionic cyst
(A) Achilles tendon rupture
7. In football players, the most common mechanism
(B) Lateral ligament rupture
of metatarsophalangeal dislocation is
(C) Osteoarthritis
(D) Osteochondral defect (A) hyperdorsiflexion
(B) hyperplantarflexion
4. Which of the following is the phase of gait during (C) hyperadduction
which the body’s center of mass is directly over (D) hyperabduction
the metatarsophalangeal joints and the sesamoids
are most compressed? 8. The main concern during surgery following an
inadvertent scrape or needle stick is
(A) Just after heel contact
(B) Midstance (A) hepatitis A infection
(C) Just before heel lift (B) hepatitis B infection
(D) Swing (C) a positive TB test
(D) a gram-positive infection
Practice Test 3 26

9. A 60-year-old male patient is scheduled for a 13. In closed-chain kinetics, internal rotation of the
neurectomy in a podiatric physician’s office. On leg results in
preoperative examination his blood pressure is
(A) locking of the midtarsal joint
160/115 mm Hg on three consecutive readings.
(B) effective weightbearing of the first ray
In this situation, the podiatric physician should do
(C) pronation of the subtalar joint
which of the following?
(D) abduction of the talus
(A) Cancel the surgery.
(B) Administer calcium channel blockers and 14. Twelve hours postoperatively, a patient with a
proceed with the surgery 15 minutes later. cast complains of severe pain, bluish
(C) Change the procedure to an in-hospital discoloration of the digits, numbness, and
setting for the next day. throbbing. Which of the following would be the
(D) Proceed with the surgery as planned. most appropriate treatment?
(A) Apply heat.
10. A patient presents with interdigital lesions that
(B) Apply ice and elevate.
fluoresce coral pink under a Wood’s light. The
(C) Apply a bivalve cast.
drug of choice for treatment is
(D) Administer intramuscular meperidine.
(A) tetracycline
(B) erythromycin 15. The establishment of the doctor-patient
(C) ciprofloxacin relationship requires a
(D) terbinafine
(A) written contract between doctor and patient
that must be witnessed or notarized
11. A 26-year-old patient presents to the emergency
(B) written contract that must be signed by both
department with a puncture wound of the right
doctor and patient
foot, sustained while working in a construction
(C) written or oral contract that must be
zone. Emergency department records confirm a
acknowledged by both doctor and patient
similar injury 1 year ago, at which time the
(D) written, oral, or implied contract between
patient was given a tetanus immunization. Which
doctor and patient
of the following is a true statement about tetanus
prophylaxis at this time?
16. Which of the following conditions would benefit
(A) No additional tetanus immunization is most from immobilization alone?
necessary.
(A) Atrophic nonunion
(B) The patient will require half of the normal
(B) Hypertrophic nonunion
dose of tetanus toxoid.
(C) Infected nonunion
(C) The patient will require tetanus
(D) Pseudoarthrosis
immunoglobulin only.
(D) The patient will require tetanus
17. Patients with rheumatoid arthritis frequently have
immunoglobulin and tetanus toxoid.
a subluxation of
12. All of the following are risk factors (A) the subtalar joint
for the development of deep vein (B) the lumbar spine
thrombophlebitis EXCEPT (C) C1 and C2
(D) T2 and T3
(A) obesity
(B) varicose veins
(C) use of local anesthetics
(D) smoking
27 Practice Test 3

18. Aggressive dissection of the first interspace at the 23. In order to classify a neuromuscular disorder as
time of a fibular sesamoid release is most likely cerebral palsy, there must be
to damage the intrinsic muscle belly of which of
(A) cranial trauma
the following muscles?
(B) a brain tumor
(A) Quadratus plantae (C) a nonprogressive lesion
(B) Abductor hallucis (D) paralysis of the lower extremity
(C) First dorsal interosseous
(D) First plantar interosseous 24. On lateral weightbearing radiographs, markedly
increased density is noted in the area of the
19. In the oral treatment of onychomycosis, which of subtalar joint and sustentaculum tali.
the following drugs is fungicidal at low Osteophytosis is noted at the level of the talar
concentrations? neck. The differential diagnosis should include
all of the following EXCEPT
(A) Griseofulvin
(B) Terbinafine (A) osteoarthritis
(C) Fluconazole (B) tarsal coalition
(D) Itraconazole (C) compression fracture of the subtalar joint
(D) osteomyelitis
20. In a Lisfranc’s fracture of the midfoot, the most
common displacement seen is 25. The inflammatory phase of wound healing
normally lasts approximately
(A) dorsal and lateral
(B) dorsal and medial (A) 24 hours
(C) plantar and lateral (B) 3-5 days
(D) plantar and medial (C) 14 days
(D) 6 weeks
21. After the primary repair of a ruptured anterior
tibial tendon, the most appropriate casting 26. In the normal development of the foot, the center
technique would be a of ossification appears last in the
(A) short leg cast with the ankle neutral (A) talus
(B) short leg cast with the ankle plantarflexed (B) calcaneus
(C) long leg cast with the ankle plantarflexed (C) lateral cuneiform
(D) long leg cast with the ankle neutral (D) navicular

22. OSHA regulations require that an employer do 27. When surgical scars are planned, it is important
which of the following? to remember that skin tension lines appear in
which direction relative to the direction of muscle
(A) Provide hepatitis B vaccinations for all
movement?
employees.
(B) Post OSHA regulations in a highly visible (A) Oblique
area for all employees. (B) Parallel
(C) Require that all employees receive CPR (C) Tangential
certification. (D) Perpendicular
(D) Establish a contractual agreement with an
OSHA-approved management program.
Practice Test 3 28

28. The initial treatment of choice for reflex 31. A 58-year-old female patient has a septic ankle
sympathetic dystrophy in the lower extremity is joint that requires irrigation. She has a negative
cardiac history and takes no medications. The
(A) surgical sympathectomy
patient has had fevers to 103.2°F and blood
(B) the administration of vasoconstrictors
cultures are positive for Staphylococcus aureus.
(C) immobilization of the affected extremity
Which of the following anesthesia options would
(D) physical therapy of the affected extremity
be CONTRAINDICATED in this patient?
29. Which of the following instruments has strong, (A) Spinal anesthesia
heavily constructed opposing jaws, each of which (B) Sciatic block anesthesia
is scooped out like the tip of a curette? (C) General endotracheal anesthesia
(D) Laryngeal mask anesthesia
(A) A gouge
(B) A malleable retractor
32. A patient presents with a second toe that is
(C) A rongeur forceps
dislocated in the sagittal plane. One year ago,
(D) A double-action bone cutting forceps
the proximal phalangeal base of the toe was
removed. The most likely cause of the
30. Transcutaneous electrical nerve stimulation
dislocation is loss of attachment of the
(TENS) is used for
(A) first lumbrical and the first and second dorsal
(A) neuromuscular reeducation
interossei
(B) edema reduction
(B) first lumbrical and the first and second
(C) pain relief
plantar interossei
(D) biofeedback to overcome postinjury muscle
(C) first and second lumbricals and the first
inhibition
dorsal interosseous
(D) first and second lumbricals and the first
plantar interosseous
29 Practice Test 3

33.

Which of the following organisms would most likely produce the finding displayed in the radiograph?
(A) Staphylococcus aureus
(B) Pseudomonas aeruginosa
(C) Streptococcus pyogenes
(D) Clostridium perfringens
Practice Test 3 30

34. Below the knee, osteoid osteomas are most Questions 39-40 refer to the following case.
commonly found in the
A 47-year-old patient has pain in the right ankle of
(A) calcaneus
several weeks’ duration. The patient remembers no
(B) tibia
inciting event but says the pain worsens with
(C) metatarsal
increased activity. The right foot appears more
(D) navicular
pronated than the left foot on weightbearing; on
attempts to rise to the toes, the patient has
35. Which of the following findings is most
considerable pain and the rearfoot does not invert.
suggestive of child abuse?
Pain is found on palpation just proximal to the
(A) Limping gait navicular tuberosity.
(B) Multiple leg bruises
(C) A spiral fracture of the humerus 39. Which of the following is the most likely
(D) Fractures in different stages of healing diagnosis?
(A) Popliteal rupture
36. A 30-year-old patient presents with bilateral
(B) Talonavicular bar
ankle pain. A systems review is positive for
(C) Calcaneal stress fracture
shortness of breath and a dry cough. Physical
(D) Tibialis posterior tendinitis
examination reveals multiple tender erythematous
subcutaneous nodules on both legs. The probable
40. Which of the following examination techniques
etiology of the ankle pain is
would be most appropriate?
(A) sarcoidosis
(A) Side-to-side compression of the calcaneus
(B) psoriatic arthritis
(B) Supination of the subtalar joint against
(C) enteropathic arthritis
resistance
(D) rheumatoid arthritis
(C) A calf squeeze
(D) A Coleman block test
37. All of the following are considered seronegative
spondyloarthropathies EXCEPT
(A) psoriatic arthritis
(B) ankylosing spondylitis
(C) erythromelalgia
(D) Reiter’s syndrome

38. An intra-articular epiphyseal fracture extending


through the physis and epiphysis would be
classified as which type of Salter-Harris fracture?
(A) Type I
(B) Type II
(C) Type III
(D) Type IV
31 Practice Test 3

41. Rheumatic heart disease is characterized by 46. Which of the following immune reactions best
describes allergic contact dermatitis?
(A) mitral valve damage
(B) tricuspid valve damage (A) Immune-complex formation
(C) holosystolic murmur (B) Hapten-mediated cytotoxicity
(D) a history of Staphylococcus aureus infection (C) IgE-mediated allergic reaction
(D) Delayed cutaneous hypersensitivity
42. Which of the following sets of parameters will
result in the lowest radiation dose to the patient? 47. A 20-year-old male patient with hemophilia
presents for podiatric treatment of a paronychia.
(A) 15 mA, 10/60 sec, 69 kVp
He is currently under treatment by his physician
(B) 15 mA, 10/60 sec, 60 kVp
and requires routine transfusion and Factor VIII
(C) 15 mA, 5/60 sec, 69 kVp
administration. Which of the following would
(D) 30 mA, 5/60 sec, 60 kVp
provide the most pertinent information?
43. Absorbable pins are an appropriate method of (A) Platelet count
fixation for which of the following procedures? (B) Reticulocyte count
(C) Prothrombin time (PT)
(A) Proximal metatarsal osteotomy
(D) Partial thromboplastin time (PTT)
(B) Austin bunionectomy
(C) Base wedge osteotomy
48. If a patient presents with the classic symptoms of
(D) Metatarsophalangeal joint fusion
a stress fracture, but the radiographs are initially
normal, the next special imaging study to order
44. A 53-year-old female patient presents with a
would be
1-year history of an unsightly painless “bump”
on the dorsum of the right foot. She has worked (A) ultrasound
outdoors in sandals all of her adult life. The (B) xeroradiography
lesion is 1 centimeter in diameter. It has an (C) bone scintigraphy
elevated, “rolled” translucent border and a central (D) angiography
umbilication with a central crust that bleeds on
debridement. Which of the following would be 49. Which of the following is the most common bone
an appropriate presumptive diagnosis? lesion?
(A) Red ant bite (A) An aneurysmal bone cyst
(B) Verruca vulgaris (B) An enchondroma
(C) Basal cell carcinoma (C) An osteochondroma
(D) Molluscum contagiosum (D) An osteoblastoma

45. A nerve block procedure may NOT be successful 50. A competitive collegiate basketball player
in the presence of infection for which of the presents with a 1-week history of lateral midfoot
following reasons? pain and swelling. Radiographs reveal a
nondisplaced transverse fracture at the proximal
(A) Local anesthetics are bases and may be
metaphyseal-diaphyseal junction of the fifth
neutralized by the acidic environment of an
metatarsal. No intramedullary sclerosis is present
infection.
at the fracture site. The most appropriate
(B) Local anesthetics may be degraded by the
treatment would be
increased amounts of pseudocholinesterase.
(C) Local anesthetics may be degraded by the (A) a posterior splint
bacteria. (B) a bone stimulator with partial weightbearing
(D) Edema and inflammation may prevent the (C) fracture shoe immobilization for 6-8 weeks
anesthetic from crossing the nerve’s lipid (D) open reduction and internal fixation with an
barrier. intramedullary screw
Practice Test 3 32

51. Which of the following is considered a violation 56. A T1 image on an MRI scan is described by
of sterile technique? which of the following parameters? Select the
two that apply.
(A) Double-gloving
(B) Hands above the head (A) TE of 25 msec
(C) Hands below the waist (B) TE of 125 msec
(D) Switching positions back-to-back (C) TE of 1000 msec
(D) TR of 25 msec
52. A 45-year-old construction worker sustains a (E) TR of 500 msec
severe crush injury to the right foot, and on
admission to the emergency department is noted 57. Which of the following complications are
to be hypotensive. The paramedics report that the commonly caused by sickle cell disease? Select
patient lost approximately 1,000 cubic the three that apply.
centimeters of blood during transit to the hospital.
(A) Osteoporosis
Packed red cell transfusions are instituted.
(B) Early puberty
Approximately 4 hours later, the patient is in
(C) Chronic hematuria
shock with severe back pain, flushing, and fever.
(D) Aseptic necrosis of the hips
Which of the following is a likely diagnosis?
(E) Chronic osteomyelitis caused by Salmonella
(A) Sepsis
(B) Pulmonary edema 58. A patient with a unilateral flatfoot deformity
(C) Myocardial infarction presents with pain centered over the medial
(D) Major hemolytic reaction navicular. Radiographic studies of the involved
foot demonstrate the presence of a small rounded
53. Following repair of an Achilles tendon rupture, ossicle-like structure adjacent to the medial
which type of cast should be applied? navicular. Which of the following are likely
conclusions? Select the three that apply.
(A) A short leg cast with the foot in a neutral
position (A) This is an os tibiale externum type I.
(B) A short leg cast with the foot in an equinus (B) This is an os tibiale externum type II.
position (C) This is an avulsed fracture fragment.
(C) A long leg cast with the foot in a neutral (D) This has nothing to do with the
position symptomatology.
(D) A long leg cast with the foot in an equinus (E) Contralateral studies may make MR
position evaluation unnecessary.

54. The number of electromagnetic waves that pass a


given point per unit of time is referred to as the
(A) wavelength
(B) wave frequency
(C) photon length
(D) speed of propagation

55. In the United States, the largest percentage of the


health dollar is utilized in which of the following
age categories?
(A) 0-1 year
(B) 2-8 years
(C) 19-64 years
(D) 65 years and older
33 Practice Test 3

59. Which of the following statements regarding 60. Informed consent may be legally obtained from
normal first ray function and anatomy are true? which of the following? Select the three that
Select the three that apply. apply.
(A) The first ray axis has a supinatory-pronatory (A) A geriatric patient
nature. (B) A legal guardian
(B) The first ray consists of the first metatarsal (C) A 19-year-old patient
and proximal phalanx. (D) An intoxicated patient
(C) Inversion and adduction motions are coupled (E) A minor patient
with dorsiflexion of the first ray.
(D) The first ray axis orientation runs from
posteromedial and dorsal to anterolateral
and plantar.
(E) The fibularis (peroneus) longus stabilizes the
first ray against the lesser tarsus and ground
in midstance.

END OF PRACTICE TEST 3


35 Practice Test 1

PRACTICE TEST 1
ANSWER KEYS AND RATIONALES

Sequence: Key Sequence: Key


1 C 7 C
In congenital hip dislocation, the femoral head is Uncomplicated navicular stress fractures will usually
usually posterior and superior to the acetabulum, heal with strict nonweightbearing for 6 to 8 weeks;
resulting in shortening of the limb with an increase in surgical treatment is usually not recommended unless
thigh folds and limited hip abduction on the affected the navicular is unstable.
side.
Sequence: Key
Sequence: Key 8 C
2 C The greater the angle formed between the axis and
Even though actual transmission is rare, because the plane of motion, the more motion is available in
sharps injuries are so commonly associated with that plane. As the calcaneal inclination angle
occupational transmission of HIV and HBV, sharps decreases, the sagittal plane compensation becomes
management is the best method to prevent infection. more significant.

Sequence: Key Sequence: Key


3 D 9 C
Surgical syndactylization of the fourth and fifth toes The presence of asthma along with these symptoms
provides stability and a predictable position for a flail makes atopic dermatitis the most likely etiology.
fifth digit.
Sequence: Key
Sequence: Key 10 D
4 A Lengthening of the extensor tendon will affect
Vancomycin is most appropriate in this case because sagittal plane positioning but will not affect
it produces a rapid response to treat the disorder. transverse plane (hallux varus) deformities.

Sequence: Key Sequence: Key


5 B 11 B
Congenital vertical talus presents as the talus fixed in The extent to which a podiatric physician can
a vertical position with hypoplasia of the talar neck practice is determined by the state legislature in a set
and head. The navicular is dislocated and articulates of laws called the State Practice Act.
with the dorsal aspect of the talar neck. The
tibionavicular and dorsal talonavicular ligaments are Sequence: Key
contracted preventing reduction of the navicular. 12 A
Creatine phosphokinase is an enzyme found in heart,
Sequence: Key brain, and skeletal muscle tissue. It is used to detect
6 B muscle disorders in serum blood testing.
The fibularis (peroneus) brevis tendon inserts on the
styloid process of the fifth metatarsal and can cause Sequence: Key
fracture of the process with excessive traction applied 13 A
to the bone by the tendon. The posterior columns of the spinal cord carry the
sensory nerve fibers for position, vibration, and
proprioception.
Practice Test 1 36

Sequence: Key Sequence: Key


14 B 21 C
MRI T2-weighted images of osteomyelitis have an Allergic or atopic diseases are some of the most
increased signal intensity. common causes of eosinophilia.

Sequence: Key Sequence: Key


15 B 22 B
The most common mechanism of a fractured ankle is Rigid forefoot valgus that cannot be compensated for
supination external rotation. At the time of injury the by forefoot inversion and first ray dorsiflexion will
foot is in a supinated position and an external rotatory require supination of the subtalar joint.
force is applied.
Sequence: Key
Sequence: Key 23 D
16 B A heel spur or plantar fasciitis is shown on the
The typical normal PTT range is 28-30 sec. Effective radiograph. Plantar fasciitis is a biomechanical
prophylaxis against deep vein thrombosis is typically condition caused by overuse and is best treated
considered to require a 1.5-fold to 2.5-fold elevation mechanically. Mechanical support initially includes
in the PTT. shoe modification, stretching, and taping and later
includes orthotics to support the plantar fascia.
Sequence: Key
17 C Sequence: Key
A lateral radiograph is the most appropriate view to 24 B
evaluate the posterior aspect of the ankle. The Hypotension during spinal anesthesia is the result of
posterior process of the talus is clearly visualized on arterial and venous dilation.
a lateral view of the foot and ankle.
Sequence: Key
Sequence: Key 25 A
18 B With the prone suspension technique, the forefoot-to-
The primary advantage of plaster of Paris compared rearfoot relationship can be visualized from behind,
to synthetic materials is that it is more moldable. allowing the subtalar joint to be manipulated through
This is particularly important for pediatric orthopedic its range of motion to help determine the joint’s
conditions which often require close contouring and neutral position.
molding of the cast to the extremity in order to
maintain proper alignment and accurate positioning. Sequence: Key
26 B
Sequence: Key The definition of standard of care is the level of
19 B practice of the average, prudent provider in any given
Hexachlorophene does not contain iodine and is community.
sufficient for surgical preparation.
Sequence: Key
Sequence: Key 27 C
20 A Digitalis poisoning can cause a variety of
Patients with AIDS have been found to be protected arrhythmias.
by the Americans with Disabilities Act. It is illegal
to discriminate against these patients by refusing to
treat them or referring them to others based on their
HIV status.
37 Practice Test 1

Sequence: Key Sequence: Key


28 C 35 A
Since calcaneal inversion (frontal plane motion) is a A long leg cast with the ankle plantarflexed 20º will
component of subtalar joint supination, placing the immobilize the medial and lateral heads of the
heel in an inverted position also puts the subtalar gastrocnemius muscle across the knee and also
joint in a supinated position, thus limiting subtalar reduce the tension of the Achilles tendon distally.
joint pronation.
Sequence: Key
Sequence: Key 36 B
29 B Neutrophils evolve from stem cells with the
An involucrum is new bone formed beneath an specialized ability to move toward and completely
elevated periosteum surrounding necrotic bone in engulf bacteria and fungi. They are capable of
osteomyelitis. producing a number of different granules containing
potent chemicals, which results in the destruction of
Sequence: Key the engulfed organisms.
30 D
Generalized causes of wound dehiscence include Sequence: Key
infection, hematoma, injury to the wound, and 37 B
incorrect suturing technique. During the inflammatory phase of wound healing
collagen exposed during wound formation activates
Sequence: Key the clotting cascade. After injury to tissue occurs the
31 C cell membranes, damaged from wound formation,
The symptoms of acute meningitis include fever, release thromboxane A2 and prostaglandin 2-alpha,
headache, and a stiff neck. potent vasoconstrictors. This helps to limit
hemorrhage. After a short period, capillary
Sequence: Key vasodilatation occurs secondary to local histamine
32 B release.
A lateral roentgenogram of a flatfoot pronated in
stance will reveal talar plantarflexion. Sequence: Key
38 C
Sequence: Key Infant mortality rate in a population is defined as the
33 D number of deaths of children under the age of 1
Promethazine is a drug used prophylactically against during a year divided by the number of live births
postoperative nausea and vomiting. that year. The rate is expressed per 1,000 live births.

Sequence: Key Sequence: Key


34 B 39 C
Ultrasound therapy is used to treat enthesopathy, By definition, Ewing’s sarcoma is a malignant bone
which is inflammation at the site of attachment of tumor.
muscle tendons and ligaments to bones or joint
capsules. Ultrasound therapy reduces tightness and Sequence: Key
spasms, decreases inflammation, and assists in 40 A
healing. Costal chondritis will not usually be localized to the
substernal area. It will normally present as moderate,
generalized chest pain.
Practice Test 1 38

Sequence: Key Sequence: Key


41 B 48 C
Malignant hyperthermia is a potentially Of the options listed, the only tendon encountered in
life-threatening though rare disorder characterized the third interspace is the lumbrical to the fourth toe.
by extremely high fever, muscle rigidity, cardiac
arrhythmias, and acidosis. It may be precipitated by Sequence: Key
inhaled anesthetics, both depolarizing and 49 C
nondepolarizing, neuromuscular blocking agents and CT scans use special x-ray equipment to produce
by other means including stress. multiple images of the inside of the body. A
computer is used to join the images together to
Sequence: Key produce cross-sectional views of the area of interest.
42 A The image in this question was taken at the level of
The calcaneofibular ligament is specifically stressed the foot.
when the ankle is in a neutral position and inversion
force is applied against the foot relative to the tibia. Sequence: Key
50 A
Sequence: Key In the absence of any hypercoagulable state or
43 B significant risk factors, the mechanical compression
Pterygium is frequently seen when nails are affected provided by elastic stockings is usually sufficient to
by lichen planus due to involvement of the nail prevent blood clot formation postoperatively.
matrix.
Sequence: Key
Sequence: Key 51 C
44 B Pentoxifylline is used for the treatment of arterial
When a radiograph is taken, current is measured in insufficiency.
milliamperes (mA). Increasing the current increases
the number of electrons emitted, which in turn Sequence: Key
increases the intensity of the rays produced, thus 52 D
resulting in a blacker radiograph. A complete sequential reduction will reduce the
deformity, but the position must be maintained with
Sequence: Key temporary fixation to prevent redislocation.
45 C
Enlargement of the hands and feet is an early feature Sequence: Key
of acromegaly. 53 B
Lidocaine helps suppress ectopic ventricular rhythms
Sequence: Key by depressing automaticity which decreases the slope
46 A of phase IV depolarization.
Of the options presented, the most likely cause of a
fever that occurs during the first postoperative day is Sequence: Key
atelectasis. 54 D
A rupture of the posterior tibial tendon is followed by
Sequence: Key symptoms which often include pain, swelling, a
47 C flattening of the arch, and an inward rolling of the
To properly assess tibial varum, which is the inward ankle. Generally these symptoms do not improve
angulation of the distal third of the shaft toward the with conservative treatment such as shoe
midline in the frontal plane, the rearfoot should be in modification.
the neutral calcaneal stance position.
39 Practice Test 1

Sequence: Key Sequence: Key


55 A 58 AD
An MRI scan is the imaging study of choice to Accessory bones are bones that are not regularly
confirm the diagnosis. An MRI scan can reveal a present. A sesamoid is a type of accessory bone that
complete or incomplete rupture of the tibialis is embedded within a tendon or joint capsule. On
posterior tendon. x-ray, this type of variant anatomy may be mistaken
for pathology.
Sequence: Key
56 A Sequence: Key
Charcot-Marie-Tooth disease is also known as 59 ACE
peroneal muscle atrophy. As the disease progresses, Urinary incontinence and dehydration are often early
symmetrical muscular atrophy and weakness are signs of sepsis in elderly patients. Polyuria is rare.
apparent in peroneal muscles and toe extensors. Because of the increased respiration rate, respiratory
alkalosis is a risk.
Sequence: Key
57 CD Sequence: Key
Health departments pursue a legislative policy 60 The order is: C E A B D
agenda promoting health. They also design and The McBride procedure is an important historical
implement a health care delivery system. procedure for hallux valgus correction in which the
sequence of steps is medial capsulotomy,
exostectomy, section of the deep transverse
intermetatarsal ligament, release of the conjoined
adductor tendon, and removal of the fibular
sesamoid.
Practice Test 2 40

PRACTICE TEST 2
ANSWER KEYS AND RATIONALES

Sequence: Key Sequence: Key


1 C 8 A
Generalized osteopenia on a radiograph is a First-generation cephalosporins are most effective
characteristic feature of osteoporosis; however, it against Staphylococcus aureus, the most common
cannot be used as the only diagnostic criterion for the cause of postoperative infection.
condition.
Sequence: Key
Sequence: Key 9 B
2 A A rocker-bottom shoe acts as a dynamic lever to
Thiopental is an intravenous sedative-hypnotic that is decrease the need for required ankle motion in the
used as an induction agent. sagittal plane during the gait cycle, allowing the
patient to ambulate with a more normal gait.
Sequence: Key
3 B Sequence: Key
Tinea pedis most often appears as dryness and 10 C
scaling in a moccasin distribution. Stress fracture of the metatarsal can occur after
excessive cyclic loading of the bone, such as after
Sequence: Key prolonged, strenuous weightbearing activity.
4 C
The use of tobacco is the most prevalent and Sequence: Key
preventable risk factor involved in lower extremity 11 D
ischemia and disease. When a lateral radiograph is taken, the patient stands
with the medial side of the foot placed against the
Sequence: Key film and the tubehead angled 90 degrees from
5 D vertical.
Charcot-Marie-Tooth disease specifically increases
the spasticity of the fibularis (peroneus) longus Sequence: Key
muscle, resulting in the supination of the entire foot 12 D
around the subtalar joint, producing a pes cavus Swelling and erythema are seen in the initial phase of
appearance of the foot. Charcot arthropathy.

Sequence: Key Sequence: Key


6 A 13 D
In a weightbearing lateral view, the film is placed An isolated rupture of the calcaneofibular ligament
vertically in the orthoposer with the medial aspect of will allow the talus to invert relative to the tibia due
the foot and ankle against the plate. to lateral instability but will not allow anterior
displacement of the talus to the tibia if the anterior
Sequence: Key talofibular ligament is intact.
7 C
PT measures blood clotting ability and is increased
with the use of warfarin.
41 Practice Test 2

Sequence: Key Sequence: Key


14 B 20 C
Blood glucose levels should be measured Mönckeberg’s arteriosclerosis occurs in peripheral
preoperatively and postoperatively. The need for arteries of the lower limbs as calcification of the
additional measurements is determined by the tunica media. The radiographic appearance has been
duration and magnitude of surgery and the stability of called “rail tracking” and described as a pipe stem
the diabetes. Signs of hypoglycemia include pattern.
tachycardia, hypertension, and diaphoresis.
Sequence: Key
Sequence: Key 21 D
15 A In therapeutic ultrasound, high frequency sound
An increased metatarsus adductus angle is associated waves are absorbed primarily by connective tissue,
with the podopediatric condition of metatarsus such as tendons and ligaments, heating the tissue,
adductus. It involves medial displacement of the increasing blood flow, and reducing chronic
metatarsals and is not a feature of subtalar joint inflammation.
pronation.
Sequence: Key
Sequence: Key 22 A
16 C Osteogenesis imperfecta is a genetic disease that
Child abuse legislation in most states provides for results in weakness of the bones secondary to a
immunity from prosecution arising from reporting malfunction of the body’s production of collagen.
suspected child abuse in good faith. Pedal manifestations include decreased metatarsal
girth.
Sequence: Key
17 C Sequence: Key
Clindamycin is used for anaerobic infections. 23 C
Ciprofloxacin is primarily used to treat Immediate hypersensitivity is the result of antigen
gram-negative infections but also provides binding to IgE, which is attached to circulating tissue
gram-positive coverage. mast cells and basophils.

Sequence: Key Sequence: Key


18 D 24 B
X-ray findings in tarsal coalition include dorsal talar Lidocaine is commonly used for peripheral nerve
head and neck beaking. Pain is a common symptom. blocks and has a usual duration of 1-3 hours.
Limitations of subtalar motion and valgus deformity
vary in severity. Sequence: Key
25 A
Sequence: Key Common radiographic findings of osteoarthritis
19 A include subchondral sclerosis, asymmetric joint space
The Harris-Beath (calcaneal axial) view can allow narrowing, and marginal osteophyte formation.
visualization of the middle facet of the talocalcaneal
joint. The joint space is obliterated in the case of Sequence: Key
osseous coalition. If there is fibrocartilaginous 26 D
coalition, the joint space appears narrow and The American Heart Association Basic Life Support
subchondral sclerosis may be present. Guidelines establish the assessment for
unresponsiveness as the initial action to be
performed.
Practice Test 2 42

Sequence: Key Sequence: Key


27 B 34 C
It is critical to understand the surgical anatomy of the The cardiovascular effects of barbiturates include a
second interspace in order to avoid potential decrease in blood pressure and a compensatory
complications. Severing of the second dorsal increase in heart rate.
interossei will cause the second toe to deviate
medially. Sequence: Key
35 C
Sequence: Key Capitation is a fixed periodic HMO payment
28 D calculated to cover the expected cost of providing
Degenerative disease of the subtalar joint with a services to patients over a period of time.
calcaneonavicular coalition requires arthrodesis, not
resection of the coalition. Sequence: Key
36 A
Sequence: Key When an excision is performed through a plantar
29 D approach, the two proper digital nerves that are
Clinical features of hyperthyroidism include warm divisions of the common digital nerve are found
moist skin, lid lag, and fine tremors. superficial to the deep transverse metatarsal ligament.

Sequence: Key Sequence: Key


30 C 37 D
The second phase of wound healing is the Symptoms of iliac artery occlusion include weak
proliferative phase, which is characterized by femoral pulses, sexual dysfunction, and buttock
epithelialization, angiogenesis, granulation tissue claudication.
formation, and collagen deposition.
Sequence: Key
Sequence: Key 38 A
31 B Young healthy athletes have a high rate of success
Stabilization of the toes in midstance is assisted by with surgical repair of an Achilles tendon rupture.
the plantar intrinsic musculature, which includes the
quadratus plantae. Sequence: Key
39 B
Sequence: Key The patient is at risk for delayed union or
32 D nonunion of the fracture due to instability and
A shoe that does not flex at the metatarsophalangeal limited blood supply in the region of the fracture.
joints has the effect of increasing the lever arm of Nonweightbearing is a necessary part of the
ground reactive force at the ankle. The longer the conservative post-injury treatment to allow for
lever arm of ground reactive force, the greater the complete healing in a timely manner.
tension on the Achilles tendon in plantarflexing the
ankle. This results in overuse of the tibialis anterior Sequence: Key
muscle as it attempts to counteract the resistance of 40 C
the Achilles tendon, causing inflammation where the Anabolic steroids are FDA-approved for use to
muscle of the tibialis anterior attaches to the anterior relieve the bone pain associated with osteoporosis.
aspect of the tibia. They do not cause osteoporosis in young athletes.

Sequence: Key
33 C
Maintaining the hinge axis for a metatarsal basilar
osteotomy is critical to provide another point of
stabilization.
43 Practice Test 2

Sequence: Key Sequence: Key


41 D 49 D
Muscle atrophy due to disuse is a major concern A key radiographic feature that defines an
when a patient is in a cast. Isometric exercises, a osteochondroma is the continuity of the cortex of the
form of resistance training where the muscle lesion with the cortex of the involved bone.
contracts but does not change length while exerting a
force, allows muscles to be exercised while the limb Sequence: Key
is immobilized. 50 C
The standard distance between the film and anode
Sequence: Key when a podiatric foot x-ray is performed is 24-30
42 C inches.
A flexor to extensor tendon transfer for stabilization
requires healing with the proximal phalanx Sequence: Key
plantarflexed at the metatarsophalangeal joint to ease 51 C
tension on the tenodesis. Dermatofibromas are common benign tumors but
some pigmented basal cell carcinomas and even some
Sequence: Key melanomas can present with a similar appearance, so
43 C excision with pathological examination is often
Inhaled beta-adrenergic agonists provide immediate indicated.
relief of asthma symptoms by causing direct dilation
of the constricted air passages. Sequence: Key
52 A
Sequence: Key In many states, the doctor can invoke a legal
44 B privilege on the patient’s behalf when asked to
Opioid agonists cause constipation. disclose or divulge information about the patient.
This privilege belongs to the patient, not the doctor,
Sequence: Key so only the patient may waive it, usually by written
45 B consent.
The talocalcaneal angle is important in the evaluation
of a flatfoot deformity. When the talocalcaneal angle Sequence: Key
is markedly increased, heel valgus is said to be 53 A
present, which indicates a more significant deformity. After a general anesthetic is administered, a minimal
level of monitoring must be provided in the post-
Sequence: Key anesthesia care unit. Vital signs including blood
46 D pressure, heart rate, breathing rate, airway patency,
Under civil law, the performance of a procedure and level of consciousness should be monitored every
without consent is an offense referred to as battery. 15 minutes.

Sequence: Key Sequence: Key


47 A 54 C
Lower motor neuron lesions cause weakening of The risk of deep vein thrombosis associated with
muscles and paralysis which leads to a steppage gait. total hip arthroplasty ranges from 20% to 80%; for
total knee replacement the risk is approximately 50%.
Sequence: Key Patients who undergo these procedures are at
48 B increased risk for the development of pulmonary
The greatest immediate risk to this patient is embolism.
aspiration of vomit, which is minimized when the
patient is turned on his side. Vomiting is not
uncommon in patients experiencing seizures.
Practice Test 2 44

Sequence: Key Sequence: Key


55 A 59 ABD
All three tests when elevated suggest myopathy or a Lytic lesions are frequently seen on bone scans as
neuromuscular disease such as muscular dystrophy. “cold” areas. Hypercalcemia is frequently seen due
to increased bone destruction and release of calcium
Sequence: Key into the circulation.
56 C
The mechanism of injury produces an intra-articular Sequence: Key
fracture of the calcaneus, creating a posterior facet 60 The order is: A B D C
fragment that is impacted or “depressed” into the To reduce a talipes equinovarus deformity accurately,
calcaneal body. the casting treatment should correct the components
in the following order: adduction, inversion, internal
Sequence: Key torsion at the tibia, and equinus.
57 ABC
Tissues consisting primarily of water have the lowest
signals on T1-weighted images.

Sequence: Key
58 ABC
In talipes equinovarus, the increase in the talo-first
metatarsal angle from the normal range of 0° to -20°
is due to the adductus of the forefoot, while the
decrease in the talocalcaneal angle from the normal
range of 20° to 50° reflects the inversion (varus) of
the heel.
45 Practice Test 3

PRACTICE TEST 3
ANSWER KEYS AND RATIONALES

Sequence: Key Sequence: Key


1 D 7 A
In the anesthetic setting, diazepam is used as a Extreme dorsiflexion of the metatarsophalangeal joint
preoperative medication and adjuvant drug partly causes the metatarsal head to rupture the plantar joint
because of its anxiolytic properties. capsule with dorsal and proximal displacement of the
digit. The injury is common among football players
Sequence: Key due to the combination of artificial playing surfaces
2 B which require the use of more flexible shoegear and
Bisphosphonates are a class of compounds that slow the frequency with which the player is positioned
bone resorption and are used to treat osteoporosis. with the toes fixed on the playing surface and the
heel raised.
Sequence: Key
3 B Sequence: Key
The inversion ankle stress view or “talar tilt” test is 8 B
most useful for evaluating injury to the lateral ankle The hepatitis B virus (HBV) is a blood borne
ligaments. pathogen that causes serious viral disease and targets
the liver. It can cause chronic infection, cirrhosis,
Sequence: Key and death. HBV is the longest known occupational
4 C pathogen, and infection is largely preventable
The metatarsophalangeal joints and sesamoids are through vaccination.
regions that receive maximal ground reactive force at
the very end of the midstance phase of gait, just Sequence: Key
before heel lift. 9 A
Patients with poorly controlled hypertension may
Sequence: Key be at an increased risk for intraoperative or
5 A postoperative myocardial infarct or stroke. A
Deep vein thrombosis is a serious medical traditional recommendation has been to delay
complication that needs to be addressed preventively, surgery if the diastolic blood pressure is greater than
prior to formation. 110 mm Hg.

Sequence: Key Sequence: Key


6 A 10 B
A fat-containing lesion like a lipoma has a short Erythromycin is the drug of choice for treatment of
T1 signal and a long T2 signal. erythrasma which fluoresces coral pink under a
Wood’s light.

Sequence: Key
11 A
Tetanus toxoid immunization booster administration
is recommended every 10 years by the CDC.
Practice Test 3 46

Sequence: Key Sequence: Key


12 C 20 A
The use of local anesthetics is not a risk factor for Displacement after a Lisfranc’s fracture most
deep vein thrombophlebitis. commonly occurs in a dorsal and lateral direction.
The dorsal ligaments are intrinsically weak and more
Sequence: Key likely to rupture than the plantar ligaments. Lateral
13 C deviation occurs due to rupture of Lisfranc’s
In closed-chain subtalar joint pronation, the talus ligament.
adducts and plantarflexes, and the leg internally
rotates. Sequence: Key
21 A
Sequence: Key A short leg cast with the ankle neutral will provide
14 C zero tension and ideal positioning for healing of a
The application of a bivalve cast will provide repaired anterior tibial tendon.
immediate relief of the patient’s symptoms and
restore blood flow. Sequence: Key
22 B
Sequence: Key OSHA regulations state that it is “mandatory to post”
15 D regulations in a location all employees routinely visit.
The establishment of a doctor-patient relationship
requires an individual to voluntarily seek a doctor’s Sequence: Key
care and expect that the communication be held in 23 C
confidence. The relationship can be defined in Cerebral palsy is defined as a nonprogressive lesion
writing but can also be oral or implied. in the brain that alters motor control, leading to
disorders of movement and posture.
Sequence: Key
16 B Sequence: Key
Hypertrophic nonunion results from excessive 24 D
motion, so eliminating movement will allow Markedly increased density is not a radiographic
complete bone healing. feature of osteomyelitis; therefore, it can be excluded
from the differential diagnosis.
Sequence: Key
17 C Sequence: Key
Clinical features of rheumatoid arthritis include 25 B
tendon and ligament erosion which in the cervical The inflammatory phase of wound healing lasts
spine leads to instability between the C1 and C2 approximately 3-5 days. Blood fills the wound,
vertebrae. epithelial cells mobilize, and venules are more
permeable.
Sequence: Key
18 C Sequence: Key
The only muscle belly present in the first interspace 26 D
is the first dorsal interosseous. A long leg cast immobilizes both knee and ankle
joints, reducing strain on the gastrocnemius-soleus
Sequence: Key complex and Achilles tendon. Casting the foot in an
19 B equinus position reduces the risk that the tendon will
Terbinafine is fungicidal rather than fungistatic at the heal in an elongated, weakened position.
minimum therapeutic dose.
47 Practice Test 3

Sequence: Key Sequence: Key


27 D 35 D
Flexion-extension creases in the skin form tension Whenever a child presents with multiple fractures at
lines perpendicular to the direction of muscle pull various stages of healing, child abuse should be at the
over joints. top of the differential diagnosis.

Sequence: Key Sequence: Key


28 D 36 A
Physical therapy is the best initial treatment for reflex Erythema nodosum and subcutaneous nodule
sympathetic dystrophy because inactivity can formation are common in the latter stages of
exacerbate the disease and perpetuate the pain cycle. sarcoidosis. The presence of shortness of breath and
nonproductive cough are also manifestations of
Sequence: Key sarcoidosis.
29 C
A rongeur forceps has strong, heavily constructed Sequence: Key
opposing jaws, each of which is scooped out like the 37 C
tip of a curette. Erythromelalgia is a neuropathic pain syndrome, not
a seronegative spondyloarthropathy.
Sequence: Key
30 C Sequence: Key
TENS is a relatively safe, noninvasive method of 38 C
pain management. A Salter-Harris type III injury occurs through the
physis and the epiphysis. The fracture passes through
Sequence: Key the hypertrophic layer of the physis and extends to
31 A split the epiphysis, inevitably damaging the
A spinal anesthetic is not recommended for a patient reproductive layer of the physis.
with sepsis due to the increased risk for meningitis.
Sequence: Key
Sequence: Key 39 D
32 A The tibialis posterior muscle acts as a supinator of the
The first lumbrical and the first and second dorsal subtalar joint, counterbalancing the effects of the
interossei all insert in the base of the proximal fibularis (peroneus) brevis muscle, which acts as a
phalanx of the second digit. pronator. Posterior tibial tendinitis will result in
weakness of the muscle, causing a more pronated
Sequence: Key appearing foot, as well as discomfort at the tendon’s
33 D insertion on the navicular tuberosity.
Soft tissue “emphysema” or gas on a radiograph is
most commonly associated with Clostridium Sequence: Key
perfringens. 40 B
Since the tibialis posterior muscle is a supinator of
Sequence: Key the subtalar joint, having the muscle contract against
34 B resistance should demonstrate both pain and
Half of all osteoid osteomas are seen in the tibia and weakness associated with tendon dysfunction.
femur, making the tibia the most likely location
below the knee. Sequence: Key
41 A
A common complication of rheumatic fever is
thickening and stenosis of the mitral valve.
Practice Test 3 48

Sequence: Key Sequence: Key


42 C 49 C
The least time of radiation exposure while Osteochondroma is an overgrowth of cartilage and
minimizing the current (mA) takes precedence over bone. It is the most common benign bone tumor
the kVp when trying to minimize radiation dose to accounting for 20-50 percent of benign bone tumors
the patient. and 10-15 percent of all bone tumors.

Sequence: Key Sequence: Key


43 B 50 D
Absorbable pins provide stability for inherently In a competitive athlete, this type of fracture should
stable osteotomies, such as the commonly performed be treated aggressively due to the increased incidence
Austin bunionectomy. of delayed union or nonunion associated with this
injury.
Sequence: Key
44 C Sequence: Key
The presence of asymptomatic central ulceration, 51 C
which may be dry and crusted, is highly suggestive of Hands below the waist is considered a violation of
basal cell carcinoma. This is frequently associated sterile technique.
with sun exposure.
Sequence: Key
Sequence: Key 52 D
45 A These symptoms occurring during this time frame
Acidosis from local infection retards the diffusion of after transfusion are most likely the result of acute
local anesthetics because of increased ionization. hemolysis resulting from a reaction to the
transfusion.
Sequence: Key
46 D Sequence: Key
Contact dermatitis is a T cell mediated phenomenon 53 D
wherein the T cells are sensitized with exposure to The ossification center of the navicular bone appears
a specific antigen, then activated by subsequent at approximately 3 years of age and is the last center
re-exposure to the antigen. of ossification to appear in the foot.

Sequence: Key Sequence: Key


47 D 54 B
PTT is elevated when an inadequate level of Factor Wave frequency is simply defined as the number of
VIII is present. electromagnetic waves that pass a given point per
unit of time.
Sequence: Key
48 C Sequence: Key
Scintigraphy records on film the distribution of 55 D
radioactivity in tissue following the use of radioactive Health costs for those age 65 and older are more than
tracer substances. When a stress fracture is strongly three times higher than for younger age groups.
suspected and the initial radiographs are normal, the
next study to order would be a triple phase nuclear
medicine bone scan.
49 Practice Test 3

Sequence: Key Sequence: Key


56 AE 59 CDE
In clinical practice the TE is always shorter than the The first ray axis runs through all three planes of
TR and usually it is lower than 30 msec. The TR is motion from a posteromedial and dorsal direction to
usually lower than 500 msec. TE and TR scan an anterolateral and plantar direction. As a result,
parameters can help determine if the image is when the first ray dorsiflexes, it also inverts and
T1-weighted or T2-weighted. adducts. The fibularis (peroneus) longus stabilizes
the first ray in midstance.
Sequence: Key
57 CDE Sequence: Key
Sickle cell disease complications are due to vascular 60 ABC
effects caused by the altered red blood cell shape and Consent is regulated by laws that vary from state to
function. Osteoporosis and delayed puberty are state but generally define competent adults eligible to
primarily endocrine effects and are not commonly make health care decisions.
seen in sickle cell disease.

Sequence: Key
58 ADE
In this case, the os tibiale externum is a normal
finding. A smooth rounded appearance categorizes
this ossicle as type I and disassociates it from the
flatfoot deformity. When there is doubt about normal
anatomical x-ray findings, contralateral studies can
eliminate the need for MRI.
NATIONAL BOARD
OF
PODIATRIC MEDICAL EXAMINERS

PART II
Clinical Science Examination

PRACTICE TEST 1

The Part II Practice Tests are representative of the content covered in the Part II Examination. They
include question formats found in the actual examination. They also include questions of varying
difficulty. A candidate’s performance on a Practice Test does not guarantee similar performance on
the actual examination.

Copyright © 2005 by the National Board of Podiatric Medical Examiners.


All rights reserved.
-3- Practice Test 1

CLINICAL SCIENCE EXAMINATION


PRACTICE TEST 1

60 questions

Directions: Each of the questions or incomplete statements below is followed by four suggested answers or
completions. Select the one that is best in each case.

NOTE: Throughout this test, the term “medial oblique foot” refers to a non-weight-bearing medial oblique position
in which the film is flat on the orthoposer, the medial side of the foot is closest to the film and the sole forms a 45°
angulation with the film, and the central beam is 90° to the film (the tubehead is angulated 0°). The converse is true
for the term “lateral oblique.”

1. Venous stasis ulcers of the lower extremities 5. Which of the following imaging modalities results
usually respond best to which of the following in no ionizing radiation exposure to the patient?
types of treatment?
(A) CT scan
(A) Oral antibiotics (B) MRI
(B) Topical antibiotics (C) Xeroradiography
(C) Arterial revascularization (D) Technetium bone scan
(D) Compressive contact dressings
6. One week after the application of a short leg
2. Hemoglobin A1c is most useful in the evaluation fiberglass cast, a patient complains of pain and
of which of the following conditions? burning in her right foot. Homans’ sign is
(A) Diabetes mellitus positive. Her past medical history is
(B) Diabetes insipidus unremarkable with the exception of a 15-year
(C) Adrenal cortical hyperplasia history of cigarette smoking (1 pack per day) and
(D) Hyperparathyroidism oral contraceptive use. The cast is removed and
the leg is noted to be hot, red, and indurated with
3. The most common location for an Achilles tendon superficial venous engorgement. Which of the
rupture is following is the most appropriate preliminary
diagnosis?
(A) the myotendinous juncture
(B) the posterior aspect of the calcaneus (A) Phlebothrombosis
(C) 2 to 6 cm proximal to its insertion (B) Deep vein thrombosis
(D) 8 to 10 cm proximal to its insertion (C) Superficial thrombophlebitis
(D) Thromboangiitis obliterans
4. A stratum granulosum is found in which of the
following anatomical locations? 7. A hemi-implant is used with the Keller procedure
primarily to
(A) Plantar skin
(B) Hair (A) establish intrinsic muscular stability
(C) Nail bed (B) reestablish the function of the flexor hallucis
(D) Oral mucosa brevis
(C) reestablish normal range of motion
(D) act as a spacer following bone removal

GO ON TO THE NEXT PAGE.


Practice Test 1 -4-

8. Twenty minutes into a procedure, a patient 12. A 10-year-old girl is brought to a hospital
develops laryngoedema, loses consciousness, and emergency department. She is seen by a podiatric
becomes cyanotic. Which of the following is the physician for treatment of a large abscess on the
most likely diagnosis? dorsum of the foot, where she was bitten by an
(A) Vasovagal attack insect 3 days ago. Physical examination reveals a
(B) Anaphylaxis temperature of 103°F with a grade IV/VI
(C) Insulin-induced hypoglycemia holosystolic murmur radiating all over the chest
(D) Myocardial infarction wall. In this situation, it would be most
appropriate for the podiatric physician to do which
9. The most common adverse reaction to intravenous of the following?
sedation with morphine is (A) Request a cardiac consultation and take no
(A) hypertension further action until the consultation
(B) anaphylaxis (B) Order an echocardiogram and take no further
(C) respiratory depression action until the results are reviewed
(D) renal failure (C) Take prophylactic measures for endocarditis
and proceed with surgery
10. The running gait cycle differs from the walking (D) Admit the patient to the hospital, begin
gait cycle in that the runner intravenous antibiotics, and wait for a stat.
cardiac consultation before taking further
(A) supinates rather than pronates at heel contact action
(B) pronates during the propulsive phase of gait
(C) does not have a double support phase of gait 13. Podiatric services that are eligible for payment
(D) does not have a float phase of gait under Medicare are determined by which of the
following?
11. If a skin incision is made directly over a tendon,
there is an increased risk of (A) The state insurance commission
(B) The state podiatric medical association
(A) adhesion (C) The Centers for Medicare & Medicaid
(B) infection Services
(C) synovitis (D) The American Podiatric Medical Association
(D) skin slough
14. Which of the following is true with respect to
magnetic resonance imaging of the Achilles
tendon?
(A) The normal diameter generally does not
exceed 5 mm.
(B) Close to the calcaneal insertion, the anterior
aspect is convex in the axial view.
(C) The signal intensity is normally isointense
with muscle on a T2-weighted image.
(D) A high intratendinous signal intensity on a
heavily water-weighted image is suggestive
of an acute rupture or injury.

GO ON TO THE NEXT PAGE.


-5- Practice Test 1

15. Secondary radiographic findings of a subtalar joint 20. A 12-year-old patient presents with arch pain.
coalition include all of the following EXCEPT Gait evaluation reveals hyperpronation throughout
(A) talar beaking the stance phase of gait. Resting calcaneal stance
(B) loss of sinus tarsi position is 2 degrees valgus. Radiographic
(C) increased calcaneal inclination angle evaluation reveals an anterior break in the cyma
(D) blunting of the lateral process of the talus line and an increased Kite's angle. This patient
most likely has
16. In a 75-year-old patient with bowel incontinence (A) a flexible flatfoot
and an infected heel decubitus ulcer, which of the (B) a clubfoot deformity
following is the most likely pathogen? (C) a cavovarus foot
(A) Escherichia coli (D) an equinovarus foot
(B) Bacteroides fragilis
(C) Neisseria gonorrhoeae 21. The primary goal of implant surgery of the first
(D) Pseudomonas aeruginosa metatarsophalangeal joint is to
(A) eliminate joint pain
17. The four elements of the tort of medical (B) improve joint motion
malpractice are (C) increase the stability of the first ray
(A) duty, breach, causation, abandonment (D) preserve the plantar intrinsic musculature
(B) duty, breach, causation, damages
(C) duty, consent, causation, damages 22. A 2-year-old child presents with symptoms of
(D) duty, standard, relationship, damages jaundice, chronic anemia, and episodes of
abdominal, back, and limb pain. The child's
18. A wet-to-dry dressing for an infected wound is symptoms seem to be precipitated by dehydration.
used primarily to Physical examination reveals swelling of the
hands and feet. Treatment consists of fluid
(A) cover the wound in order to prevent further maintenance and the administration of analgesics.
contamination This case description is characteristic of
(B) apply antibacterial solutions to the wound
(C) mechanically debride the wound (A) juvenile diabetes
(D) keep the wound moist (B) juvenile arthritis
(C) sickle cell anemia
19. In an anteroposterior radiographic view, a (D) collagen disease
talonavicular articulation of less than 70 percent is
a sign of 23. Which of the following provides the best
compression and alignment for an Austin
(A) pronation bunionectomy?
(B) supination
(C) metatarsus adductus (A) Cerclage wire
(D) skew foot (B) Threaded Kirschner wire
(C) Cancellous screws
(D) Orthosorb pins

24. A complication of halothane anesthesia is


(A) bronchospasm
(B) hepatic necrosis
(C) peripheral vasoconstriction
(D) hypertension

GO ON TO THE NEXT PAGE.


Practice Test 1 -6-

25. The female athlete triad is composed of 29. The inflammatory arthritis associated with
hemophilia most commonly affects which of the
(A) eating disorders, stress fractures, and
following joints?
infections
(B) eating disorders, amenorrhea, and (A) Shoulder
osteoporosis (B) Hip
(C) amenorrhea, stress fractures, and infections (C) Knee
(D) osteoporosis, stress fractures, and increased (D) Metatarsophalangeal
incidence of endometrial cancer
30. The Thompson test is useful in the detection of
26. A 5-year-old boy presents with a slight equinus
(A) an Achilles tendon rupture
deformity. His parents state that he has had a
(B) a calcaneofibular ligament sprain
tight gastrocnemius-soleus complex ever since he
(C) an anterior talofibular ligament sprain
was an infant. He appears to be a healthy,
(D) a bucket handle tear of the medial meniscus
well-nourished, and well-developed child.
Duchenne type muscular dystrophy is suspected.
31. A 52-year-old man experienced pain in his
All of the following tests would provide useful
left arm, chest, and jaw for 5 minutes. He
information relative to this diagnosis EXCEPT
took nitroglycerin sublingually with no relief.
(A) a muscle biopsy He is taken to the emergency department where
(B) a muscle strength evaluation it is determined that he has had an acute
(C) a genetic screening for a sex-linked recessive myocardial infarction. Which of the following is
disorder a life-threatening dysrhythmia he will most likely
(D) an acetylcholine receptor (AChR) antibody experience during the first hour following the
test myocardial infarction?
(A) Atrial flutter
27. A runner presents with a history of posteromedial
(B) Atrial fibrillation
shin splints due to overuse of the deep posterior
(C) Ventricular flutter
muscles of the leg as a compensation for a
(D) Ventricular fibrillation
structurally long limb on the involved side.
Evaluation of the patient's shoe wear pattern on
32. The nutrient artery of the first metatarsal enters
the involved side would reveal
the midshaft
(A) excessive lateral forefoot outsole wear
(A) medially
(B) heel counter tipping medially in a valgus
(B) laterally
position
(C) dorsally
(C) compression of the lateral rearfoot midsole
(D) plantarly
(D) abnormal wear of the uppers in the region of
the fifth digit due to lateral crowding
33. An asymptomatic, fragmented, irregular, sclerotic,
calcaneal apophysis in an 8 year old is most
28. In a nursing home population of 200, 12 people
indicative of
are diagnosed with onychomycosis in one year. In
the first 4 months of the next year, 5 more people (A) Köhler's disease
are diagnosed with onychomycosis. What is the (B) calcaneal apophysitis
incidence rate for the second year? (C) a nonunion fracture
(D) a normal anatomic variant
(A) 5/17
(B) 12/17
(C) 5/200
(D) 17/200

GO ON TO THE NEXT PAGE.


-7- Practice Test 1

34. Somatization disorder is a neurotic illness 38. A 56-year-old woman is seen for pain involving
characterized by all of the following EXCEPT her right ankle. The patient reports no specific
traumatic incident. Recently she has noticed mild-
(A) weakness and paralysis of muscle groups
to-moderate pain across her midfoot and her ankle
(B) dramatic, emotional presentation of
with increased activity. Physical examination
complaints
reveals pain and swelling along the medial ankle.
(C) multiple, vague complaints referable to any
On the single heel rise test, she is unable to invert
part of the body
the right hindfoot. On weight bearing, the right
(D) dissatisfaction with care from one physician
forefoot is abducted on the rearfoot and the arch
to the next
appears to flatten significantly more on the right
foot than on the left. Which of the following
35. A patient with a long-standing varicose ulcer over
therapies would NOT be appropriate?
the medial malleolus is at higher risk for the
development of which of the following within the (A) Steroid injection
ulcer? (B) Walking brace/boot
(C) Prescription foot orthosis
(A) Melanoma
(D) Nonsteroidal anti-inflammatory drugs
(B) Hemangioma
(NSAIDs)
(C) Basal cell carcinoma
(D) Squamous cell carcinoma
39. Neuromuscular causes of cavus foot include all of
the following EXCEPT
36. Which of the following cells normally develops
into a macrophage when it infiltrates tissue? (A) cerebral palsy
(B) myelomeningocele
(A) Monocyte
(C) muscular dystrophy
(B) Mast cell
(D) Charcot-Marie-Tooth disease
(C) Lymphocyte
(D) Basophil
40. The presence of Bence Jones proteins in the urine
is diagnostic of
37. Yellow discoloration of radiographic film is most
likely due to inadequate (A) amyloidosis
(B) multiple myeloma
(A) development
(C) monoclonal gammopathy
(B) fixation
(D) Waldenström's macroglobulinemia
(C) mAs
(D) kVp
41. Which of the following is a periosteal elevator?
(A) Sayre
(B) Hohmann
(C) Weitlaner
(D) Seeburger

GO ON TO THE NEXT PAGE.


Practice Test 1 -8-

42. A 23-year-old man is seen in the emergency 46. Which of the following fractures has the best
department for a diagnosed right ankle prognosis?
fracture/dislocation. Vascular evaluation reveals
(A) Rowe type Ia
an absence of all pulses below the knee. Closed
(B) Rowe type IV
reduction is not successful. The patient has just
(C) Rowe joint depression
eaten a large meal. Assuming a thigh tourniquet
(D) Essex-Lopresti joint depression
will be used, what is the most appropriate type of
anesthesia for this patient? 47. A 6 year old presents with a painful ankle injury
(A) Spinal of 3 days’ duration. There is swelling and pain on
(B) Local infiltration direct palpation of the lateral malleolus. X-rays
(C) General (inhalation anesthesia) are unremarkable. Which of the following would
(D) Intravenous sedation be the treatment of choice?
(A) A short leg cast
43. The minimal acceptable treatment for a diastasis (B) An Ace bandage
of the ankle joint would include which of the (C) Ligamentous repair
following? (D) Activity, as tolerated
(A) Low Dye strapping with a crossover J
(B) A reinforced Unna's boot and a surgical shoe 48. Which of the following statements about Lyme
(C) Internal fixation disease is true?
(D) Immobilization with a short leg cast (A) The disease is transmitted via feline to human
contact.
44. In radiology, the Harris-Beath (calcaneal axial) (B) The diagnostic test of choice is the
projections are indicated to determine the presence erythrocyte sedimentation rate (ESR).
of a talocalcaneal coalition that involves which of (C) The drug of choice is metronidazole.
the following anatomic locations? (D) The most commonly associated dermatologic
(A) Anterior facet only lesion is erythema chronicum migrans.
(B) Posterior facet only
(C) Anterior and middle facets 49. A runner who twisted her ankle 6 months ago still
(D) Middle and posterior facets has ankle pain and an occasional sensation of the
ankle giving way and locking. She has no
45. Which of the following is normally associated apparent swelling or pain on range of motion. The
with Paget's disease? most likely diagnosis is
(A) Increased osteoblast activity only (A) sinus tarsi syndrome
(B) Increased osteoblast and osteoclast activity (B) osteochondral talar dome fracture
(C) Decreased serum alkaline phosphatase levels (C) synovitis of the fibular (peroneal) tendons
(D) A male-to-female ratio of occurrence of 4:1 (D) fracture of the os trigonum

GO ON TO THE NEXT PAGE.


-9- Practice Test 1

50. Pyoderma gangrenosum is most often found in 55. Which of the following tumors is associated with
individuals with which of the following diseases? night pain that is relieved by salicylates?
(A) Ulcerative colitis (A) Osteoblastoma
(B) Sarcoidosis (B) Chondroblastoma
(C) Lupus erythematosus (C) Osteoid osteoma
(D) Psoriasis (D) Chondromyxoid fibroma

51. Which of the following ligaments is most 56. A podiatric physician can best minimize drug
commonly damaged in an inversion sprain of the interactions by
ankle?
(A) limiting the number of prescription refills
(A) Anterior talofibular (B) performing an adequate physical examination
(B) Posterior talofibular (C) taking a thorough medical history
(C) Calcaneofibular (D) ordering appropriate laboratory tests
(D) Deltoid
57. Which of the following is an inhibitor of
52. Radiographic findings indicative of Freiberg's prostaglandins?
infraction are initial joint-space
(A) Aspirin
(A) widening followed by a long-term increase in (B) Warfarin
subchondral bone density (C) Insulin
(B) widening with no long-term effect on (D) Heparin
subchondral bone density
(C) narrowing followed by a long-term increase 58. Which of the following is the most common
in subchondral bone density postoperative infective organism in podiatric
(D) narrowing with no long-term effect on surgery?
subchondral bone density
(A) Streptococcus
(B) Pseudomonas
53. Multiple sclerosis is characterized by all of the
(C) Staphylococcus
following signs and symptoms EXCEPT
(D) Clostridium
(A) ataxia
(B) impaired vision
(C) bladder dysfunction
(D) fibular (peroneal) muscular atrophy

54. A custom molded shoe is the primary choice for


treatment of which of the following?
(A) Multiple sclerosis
(B) Metatarsal stress fracture
(C) Advanced Charcot foot deformity
(D) Severe osteoarthritis of the tarsus and lesser
tarsus

GO ON TO THE NEXT PAGE.


Practice Test 1 -10-

The next 2 questions refer to the following case.

A male patient presents with an induration, pain, and


bluish discoloration over the second interspace area of
the left foot. The pain and size of the lesion have been
increasing over the last month. X-rays show a
calcification of portions of the lesion. There is no
history of trauma and the skin texture is normal.

59. Which of the following is the most likely


diagnosis?
(A) Neuroma
(B) Fibroma
(C) Glomus tumor
(D) Cavernous hemangioma

60. The correct treatment would be


(A) surgical excision of the lesion
(B) steroid injection of the lesion
(C) padding of the lesion
(D) ultrasound and whirlpool

END OF PRACTICE TEST 1


NATIONAL BOARD
OF
PODIATRIC MEDICAL EXAMINERS

PART II
Clinical Science Examination

PRACTICE TEST 2

The Part II Practice Tests are representative of the content covered in the Part II Examination. They
include question formats found in the actual examination. They also include questions of varying
difficulty. A candidate’s performance on a Practice Test does not guarantee similar performance on
the actual examination.

Copyright © 2005 by the National Board of Podiatric Medical Examiners.


All rights reserved
Practice Test 2 -12-

CLINICAL SCIENCE EXAMINATION


PRACTICE TEST 2

60 questions

Directions: Each of the questions or incomplete statements below is followed by four suggested answers or
completions. Select the one that is best in each case.

NOTE: Throughout this test, the term “medial oblique foot” refers to a non-weight-bearing medial oblique position
in which the film is flat on the orthoposer, the medial side of the foot is closest to the film and the sole forms a 45°
angulation with the film, and the central beam is 90° to the film (the tubehead is angulated 0°). The converse is true
for the term “lateral oblique.”

1. Which of the following diagnostic tests is 5. Which of the following agents is associated with
appropriate for a patient with suspected herpes drug-induced systemic lupus erythematosus?
simplex on the foot?
(A) Procainamide
(A) Gram’s stain (B) Lidocaine
(B) Tzanck smear (C) Bupivacaine
(C) Ova and parasite smear (D) Etidocaine
(D) Dermatophyte test medium (DTM)
6. In a histologic section of normal skin of the
dorsal foot, melanocytes are located
2. Which of the following is associated with an
increased incidence of osteosarcoma? (A) in the cornified layer
(B) in the subcutaneous fat
(A) Paget's disease
(C) deep in the reticular dermis
(B) Ankylosing spondylitis
(D) in the basal cell layer of the epidermis
(C) Rheumatoid arthritis
(D) Osteomalacia
7. A 23-year-old runner presents with anterior leg
pain that has been present for 3 weeks and is
3. The best modality to confirm an osseous subtalar
greatest when running on hills. Pain is elicited on
joint coalition is
palpation of the anterior aspect of the lower leg,
(A) a bone scan lateral to the anterior crest of the tibia. The most
(B) a CT scan likely diagnosis is
(C) an MRI
(A) peroneal tendinitis
(D) tomography
(B) anterior shin splints
(C) iliotibial band syndrome
4. A 68-year-old woman undergoes a complex
(D) anterior tibial compartment syndrome
reconstructive rearfoot surgical procedure
with general anesthesia without complication.
Within 48 hours the patient develops dyspnea,
tachypnea, and a temperature of 101.5°F. Which
of the following is the most likely diagnosis?
(A) Septicemia
(B) Pulmonary emboli
(C) Deep vein thrombosis
(D) Acute appendicitis

GO ON TO THE NEXT PAGE.


-13- Practice Test 2

8. Which of the following is a chronic inflammatory 13. Differential diagnosis of Kaposi's sarcoma
disease characterized by proliferative synovitis includes
with subsequent bony erosion and destruction of
(A) stasis dermatitis
the articular cartilage?
(B) hypertrophic lichen planus
(A) Gouty arthritis (C) malignant melanoma
(B) Osteoarthritis (D) dyshidrotic eczema
(C) Rheumatoid arthritis
(D) Psoriatic arthritis 14. Which of the following bone lesions is
characterized by dense bone surrounding a lytic
9. A 45-year-old overweight patient with a pes central nidus?
planus deformity reports “aching” pain at the
(A) Giant cell tumor
tuberosity of the navicular that has progressed
(B) Aneurysmal bone cyst
over the past 3 months. Radiographs show an
(C) Osteoid osteoma
enlarged navicular medially without fracture.
(D) Osteosarcoma
The patient wears poorly supported shoes to work
at a department store. The podiatric physician’s
15. In a patient with bacterial endocarditis who has
next step should be
no known allergies, the most appropriate
(A) foot and ankle strapping, a nonsteroidal anti- prophylactic drug is
inflammatory drug (NSAID), and a
(A) tetracycline
discussion about shoes
(B) gentamicin
(B) a short leg non-weight-bearing cast for
(C) erythromycin
6 weeks
(D) amoxicillin
(C) an Unna's boot for 4 weeks with ice and
elevation
16. Semi-weight-bearing impression foam foot
(D) a technetium-99m 3-phase bone scan
castings are most appropriate for which of the
following?
10. A patient manifests a serpiginous eruption on the
dorsal aspect of the foot that is successfully (A) Rigid foot with fixed deformity
treated with topical thiabendazole. Which of the (B) Flexible flatfoot deformity
following is the most likely diagnosis? (C) Flexible cavus foot deformity
(D) Hypermobile first ray deformity
(A) Tuberculosis verrucosa cutis
(B) Erythema nodosum leprosum
17. When a soft-tissue lesion on the foot is excised,
(C) Cutaneous larva migrans
the standard length-to-width ratio for the
(D) Erosio interdigitalis blastomycetica
incision is
11. Which of the following is true of Sever’s disease? (A) 2:1
(B) 3:1
(A) It rarely affects females.
(C) 3:2
(B) It is a self-limiting disease.
(D) 5:2
(C) Ecchymosis and edema are pathognomonic.
(D) X-rays reveal a crescent sign in the Achilles
18. Which of the following is a muscle that directly
tendon.
prevents the longitudinal arch from collapsing?
12. The Bunnell technique is used for (A) Abductor hallucis
(B) Plantar aponeurosis
(A) tenorrhaphy
(C) Fibularis (peroneus) longus
(B) arthrodesis
(D) Posterior tibial
(C) capsulodesis
(D) capsulorrhaphy

GO ON TO THE NEXT PAGE.


Practice Test 2 -14-

19. Following lesser digital arthroplasty, the digit should be splinted in which of the following positions?
Metatarsophalangeal Proximal Interphalangeal Distal Interphalangeal
Joint Joint Joint
(A) Dorsiflexed Straight Straight
(B) Dorsiflexed Plantarflexed Straight
(C) Slightly Plantarflexed Straight Straight
(D) Slightly Plantarflexed Plantarflexed Plantarflexed

20. Which of the following is (are) most sensitive to 25. In addition to gloves, OSHA-designated personal
radiation? protective equipment includes
(A) Spermatogonia (A) gowns and eye protection
(B) Neutrophils (B) steam sterilizers and masks
(C) Neurons (C) self-sheathing needles and gowns
(D) Skin (D) sharps containers and eye protection

21. On a lateral radiograph, a decreased calcaneal 26. A patient with a painful limb, who will be using a
inclination angle with an increased talar single cane, should be instructed to use the cane
declination angle would indicate
(A) with the elbow fully extended
(A) supination of the foot (B) in a manner to shift weight away from the
(B) pronation of the foot cane
(C) talipes equinovarus (C) on the side opposite the painful/injured side
(D) metatarsus adductus (D) angled away from the body about 40 degrees

22. Radiographic subject contrast is best controlled 27. Serial casting is likely to be successful in the
by the manipulation of treatment of all of the following EXCEPT
(A) kVp (A) metatarsus adductus
(B) mAs (B) calcaneovalgus
(C) source-to-image distance (C) windswept deformity
(D) developer temperature (D) congenital vertical talus

23. Which of the following Salter-Harris fractures 28. Which of the following tendons is found plantar
has the best prognosis? to the deep transverse metatarsal ligament?
(A) I (A) Lumbrical tendon
(B) II (B) Abductor tendon
(C) III (C) Plantar interosseous tendon
(D) IV (D) Dorsal interosseous tendon

24. The fifth toe can clearly be seen in all of the


following standard radiographic views EXCEPT
the
(A) lateral
(B) lateral oblique
(C) medial oblique
(D) dorsoplantar

GO ON TO THE NEXT PAGE.


-15- Practice Test 2

29. A podiatric physician is examining a 57-year-old 34. The formation of Heberden’s nodes at distal
patient who has a 21-year history of poorly interphalangeal joints is associated with
controlled diabetes mellitus. A neurosensory (A) osteoarthritis
examination reveals a loss of proprioception. (B) rheumatoid arthritis
Further examination of the patient would most (C) systemic lupus erythematosus
likely reveal a (D) ankylosing spondylitis
(A) symmetric absence of deep tendon reflexes
(B) loss of protective sensation with 10-gram 35. Which of the following local anesthetics is NOT
monofilament recommended for use in children under 12 years
(C) loss of sharp-dull discrimination of age?
(D) plantar reflex with a bilateral flexion (A) Lidocaine
response (B) Mepivacaine
(C) Bupivacaine
30. An individual can functionally increase limb (D) Procaine
length by
(A) dorsiflexing the ankle joint 36. A patient presents with a single, nonpruritic
(B) supinating the subtalar joint vesicle on the dorsum of the third toe. There is
(C) supinating the longitudinal midtarsal joint no history of trauma and the patient has not made
(D) pronating the oblique midtarsal joint any change in shoe gear. Inspection reveals an
umbilicated vesicle. The most likely diagnosis is
31. All of the following radiographic signs are (A) verruca plana
associated with excessive pronation EXCEPT (B) acute contact dermatitis
(A) obliteration of the sinus tarsi (C) molluscum contagiosum
(B) anterior displacement of the talus (D) bullous diabeticorum
(C) a talocalcaneal angle of 10°
(D) a metatarsus primus adductus angle of 20° 37. Which of the following is most characteristic of
venous stasis ulcerations?
32. Which of the following general anesthetics is (A) A punched-out border
most likely to sensitize the myocardium to (B) A gray mucoid base
catecholamines? (C) A red, granulating base
(A) Enflurane (D) Pain
(B) Isoflurane
(C) Halothane 38. The most prominent physical finding of
(D) Nitrous oxide right-sided congestive heart failure is
(A) S3 gallop
33. A Young procedure performed on the tibialis (B) rapid breathing
anterior is best described as (C) moist rales in the lungs
(A) tendon transfer (D) lower extremity dependent edema
(B) tendon translocation
(C) tendon lengthening 39. In a flexor tendon transfer for hammer toe
(D) tenolysis correction, the flexor tendon is inserted into the
(A) plantar aspect of the distal phalanx
(B) plantar aspect of the middle phalanx
(C) dorsal aspect of the proximal phalanx
(D) dorsal aspect of the middle phalanx

GO ON TO THE NEXT PAGE.


Practice Test 2 -16-

40. Which of the following statements applies to a 44. In a lateral osteochondral talar dome fracture, the
podiatric physician who suspects that an elderly most common mechanism of injury is
patient has been abused? (A) inversion, plantarflexion
(A) The podiatric physician should report the (B) inversion, dorsiflexion
suspicion only if he or she is convinced (C) eversion, plantarflexion
beyond a reasonable doubt that abuse has (D) eversion, dorsiflexion
occurred.
(B) The podiatric physician should report the 45. Which of the following is associated with
suspicion as a matter of moral but not legal normochromic, normocytic anemia?
obligation. (A) Iron deficiency
(C) The podiatric physician may be criminally (B) Hemolysis
liable for failure to report the suspicion. (C) Folate deficiency
(D) The podiatric physician may not report the (D) Lead poisoning
suspicion because of the doctor-patient
privilege. 46. A patient with 3º forefoot valgus was casted for
functional orthoses, but the negative cast
41. An orthopedic shoe with a steel shank from heel reflected 10º of forefoot valgus. This most likely
to toe and a rocker bar would be appropriate for occurred because the clinician
treating
(A) supinated the long axis of the midtarsal joint
(A) drop foot (B) dorsiflexed the fourth and fifth
(B) spastic flatfoot metatarsophalangeal joints
(C) flexible hammer toes (C) pronated the subtalar joint when locking the
(D) an arthritic ankle and subtalar joint midtarsal joint
(D) did not check to make sure the patient was
42. Which of the following is a true statement about not contracting the anterior tibial muscle
healing by first intention?
(A) It implies suppuration or necrosis. 47. A 35-year-old patient fell from the roof of a
(B) It involves excessive formation of house and sustained a fracture of the talus.
granulation tissue. X-rays reveal fracture of the talar neck and
(C) It profoundly diminishes the strength of the dislocation of the talar body from the ankle
surgical wound. mortise and subtalar joint. Which of the
(D) It is demonstrated in the closure of an following describes this type of fracture and the
uncomplicated skin incision. most likely complication associated with it?
(A) Berndt-Harty stage II fracture with a severe
43. A patient is experiencing anterior leg pain. On risk of osteochondritis
examination of muscle strength, the anterior (B) Stewart type III fracture with a slight risk of
compartment muscles are graded 3/5. Which of avascular necrosis
the following nerves is most likely responsible (C) Hawkins type III fracture with a high risk of
for this finding? avascular necrosis
(A) Tibial (D) Sanders fracture (Tongue type) with a risk of
(B) Saphenous degenerative arthritis
(C) Deep fibular (peroneal)
(D) Superficial fibular (peroneal)

GO ON TO THE NEXT PAGE.


-17- Practice Test 2

48. Which of the following conditions may cause 51. Thrombophlebitis of the small veins frequently
delayed relaxation of Achilles tendon reflexes? occurs with intravenous administration of
(A) Diabetes (A) diazepam
(B) Porphyria (B) propofol
(C) Alcoholism (C) droperidol
(D) Hypothyroidism (D) fentanyl

49. A 33-year-old woman presents with heat 52. Which of the following is the correct ventilation-
intolerance headaches and excessive sweating of to-chest-compression ratio in one-rescuer CPR?
2 months’ duration. She has a visible hand (A) 2 ventilations : 3 chest compressions
tremor and her handwriting is uneven. She jogs (B) 2 ventilations : 15 chest compressions
5 miles a day, 5 times a week. Over the past (C) 4 ventilations : 8 chest compressions
2 months her tolerance for exercise and her (D) 8 ventilations : 15 chest compressions
running times have decreased. Physical
examination reveals a thin, well-developed and 53. Generalized subperiosteal bone resorption is
anxious woman. The patient's blood pressure is characteristic of
130/50 mm Hg and her pulse is 120. Ocular
inspection reveals lid retraction and stare. She (A) osteomalacia
also presents with proximal muscle weakness. (B) hyperparathyroidism
Examination of the neck reveals diffuse (C) osteoporosis
enlargement of the thyroid and audible bruit. (D) osteosarcoma
There is no history of drug intake. Which of the
following is the most likely diagnosis? 54. Unusual infections such as Mycobacterium
avium-intracellulare are commonly seen in
(A) Subacute thyroiditis association with
(B) Exogenous thyroid hormone
(C) Graves’ disease (A) HIV
(D) Multinodular goiter (B) pelvic inflammatory disease
(C) sarcoidosis
50. Emergency treatment of an acute myocardial (D) cystic fibrosis
infarction includes all of the following EXCEPT
55. In a prospective epidemiologic study of a disease,
(A) thrombolysis the cohort originally selected consists of persons
(B) heparin
(C) warfarin (A) with the disease
(D) angioplasty (B) without the disease
(C) with a family history of the disease
(D) without a family history of the disease

GO ON TO THE NEXT PAGE.


Practice Test 2 -18-

56. Which of the following is NOT a characteristic of The next 2 questions refer to the following
plantar fasciitis? information.
(A) Post-static dyskinesia
(B) A tight gastrocnemius-soleus complex A 40-year-old African-American man presents with
(C) Pain at the medial plantar aspect of the an injury to his right foot sustained 5 days ago when
calcaneus a heavy sheet of glass fell onto the top of his foot.
(D) Occurrence in boys between the ages of An examination reveals a laceration over the first
4 and 7 years metatarsophalangeal joint with erythema, edema, and
drainage. A loss of active dorsiflexion of the first
57. The patellar reflex is largely mediated by which metatarsophalangeal joint is noted, indicating that
of the following nerve roots? some tendon damage may have occurred. The patient
has not had past tetanus immunization.
(A) L1-L2
(B) L3-L4
(C) L5-S1 59. Conservative treatment fails and surgery is
(D) S2-S3 performed. Three days postoperatively, the
patient presents with pain. Physical examination
58. Which of the following medications is most reveals severe localized edema, erythema,
likely to contribute to the formation of a deep serosanguineous drainage, induration, and pain
vein thrombosis? on palpation. The most likely diagnosis is
(A) Multivitamins with zinc (A) hematoma
(B) Oral contraceptives (B) causalgia
(C) Enteric-coated aspirin (C) Freiberg's infraction
(D) Dipyridamole (D) normal postoperative course

60. Preoperative laboratory tests should include


which of the following?
(A) Uric acid
(B) Sickle cell trait
(C) Hemoglobin A and hemoglobin C
(D) Erythrocyte sedimentation rate (ESR)

END OF PRACTICE TEST 2


NATIONAL BOARD
OF
PODIATRIC MEDICAL EXAMINERS

PART II
Clinical Science Examination

PRACTICE TEST 3

The Part II Practice Tests are representative of the content covered in the Part II Examination.
They include question formats found in the actual examination. They also include questions of
varying difficulty. A candidate’s performance on a Practice Test does not guarantee similar
performance on the actual examination.

Copyright © 2005 by the National Board of Podiatric Medical Examiners.


All rights reserved.
Practice Test 3 -20-

CLINICAL SCIENCE EXAMINATION


PRACTICE TEST 3

60 questions

Directions: Each of the questions or incomplete statements below is followed by four suggested answers or completions.
Select the one that is best in each case.

NOTE: Throughout this test, the term “medial oblique foot” refers to a non-weight-bearing medial oblique position in
which the film is flat on the orthoposer, the medial side of the foot is closest to the film and the sole forms a 45°
angulation with the film, and the central beam is 90° to the film (the tubehead is angulated 0°). The converse is true for
the term “lateral oblique.”

1. Deep vein thrombosis is diagnosed in a 5. The Hardcastle classification system was


hospitalized, 3-day postoperative patient. developed for classifying
The initial treatment of choice is
(A) subtalar joint dislocations
(A) oral warfarin (B) Lisfranc's joint dislocations
(B) intravenous heparin (C) metatarsophalangeal joint dislocations
(C) compression wraps (D) extra-articular fractures of the os calcis
(D) local heat and elevation
6. When a short leg cast is applied, which
2. Metatarsus adductus is evaluated radiographically superficial nerve needs to be protected in the area
by utilizing which two axes? of the fibular neck?
(A) Navicular and first metatarsal (A) Sural
(B) Calcaneal and second metatarsal (B) Saphenous
(C) Second metatarsal and long axis of the talus (C) Femoral
(D) Second metatarsal and lesser tarsal (D) Common fibular (peroneal)

3. When griseofulvin is prescribed, it is important to 7. In the normal development of the foot, the center
perform which of the following tests in addition of ossification appears last in the
to a complete blood cell count (CBC)?
(A) talus
(A) Culture and sensitivity (B) calcaneus
(B) KOH or fungal culture (C) lateral cuneiform
(C) Wood's light examination (D) navicular
(D) Gram's stain
8. Which of the following types of padding would
4. Which of the following is a postoperative be best for an arthritic hammer toe?
nonadherent dressing utilized in immediate
(A) A metatarsal pad
wound care?
(B) A crest pad
(A) Adaptic dressing (C) A Morton's pad
(B) Kerlix fluff dressing (D) A heel-lift pad
(C) Kling roller bandage dressing
(D) Sterile 4" x 4" gauze dressing

GO ON TO THE NEXT PAGE.


-21- Practice Test 3

9. True acquired clubbing of the fingernails is 13. An avulsion fracture of the anterior process of the
associated with which of the following diseases? calcaneus is associated with which of the
following ligaments?
(A) Lung cancer
(B) Osteoarthritis (A) Spring
(C) Rheumatoid arthritis (B) Bifurcate
(D) Ulcerative colitis (C) Long plantar
(D) Interosseous talocalcaneal
10. Chemical exposure in the podiatric laboratory is
controlled by the 14. Which of the following radiographic findings is
most consistent with chronic tophaceous gout?
(A) Public Health Service
(B) National Institutes of Health (A) Osteopenia
(C) Drug Enforcement Agency (B) Osteophytosis
(D) Occupational Safety and Health (C) Early joint destruction
Administration (D) Large bony erosions

11. Adverse reactions associated with nonsteroidal 15. Which of the following types of Salter-Harris
anti-inflammatory drugs (NSAIDs) include all of epiphyseal injuries do NOT involve a fracture
the following EXCEPT through the metaphysis?
(A) nausea (A) I and II
(B) interstitial nephritis (B) I and III
(C) respiratory depression (C) II and III
(D) peptic ulcer disease (D) III and IV

12. A 45-year-old man presents with a history of pain 16. In the 18- to 30-year-old age group, the most
in the medial aspect of the right ankle and foot common infective organism in septic arthritis is
for the past 6 months. The pain has become (A) Staphylococcus epidermidis
progressively worse and causes the patient to (B) Staphylococcus aureus
limp at the end of the workday. The pain is sharp (C) Haemophilus influenzae
at times; otherwise it is a throbbing, aching pain. (D) Neisseria gonorrhea
Examination findings are pain on palpation
posterior and proximal to the medial malleolus 17. Which of the following blood tests measures the
and also at the tuberosity of the navicular and coagulation time via the intrinsic pathway?
stance heel valgus with a collapse of the
longitudinal arch. The remaining examination is (A) Bleeding time
unremarkable. To form a treatment plan, it (B) Sedimentation rate
would be most helpful for the podiatric physician (C) Prothrombin time (PT)/INR
to review the results of (D) Partial thromboplastin time (PTT)
(A) an MRI 18. A Charcot joint is caused by
(B) a CT scan
(C) a soft tissue ultrasound (A) motor neuropathy
(D) an EMG/NCV study (B) sensory neuropathy
(C) vascular insufficiency
(D) dystrophic ulceration

GO ON TO THE NEXT PAGE.


Practice Test 3 -22-

19. The Hoffman-Clayton procedure is most 23. Pulmonary embolism is most definitively
commonly indicated for which of the following diagnosed by
conditions? (A) pulmonary angiography
(A) Pes cavus (B) ventilation perfusion scans
(B) Hallux valgus (C) chest x-ray studies
(C) Hammer toe (D) arterial blood gas studies
(D) Rheumatoid arthritis
24. A 68-year-old man reports coldness and
20. Which of the following is the usual mechanism of numbness of both feet for the past 6 months and
injury in football turf-toe injuries to the first leg pain with increased activity that is relieved
metatarsophalangeal joint complex? with rest. Physical examination shows absent DP
and PT pulses. Subpapillary venous filling time
(A) Hyperextension with compression
is more than 8 seconds in all digits. The skin of
(B) Hyperflexion with tension
both feet is cool to the touch, shiny, and pale, and
(C) Hyperflexion with shear
there is a lack of digital hair growth. Doppler and
(D) Axial load on the sesamoid complex
plethysmography show minimal blood flow to
both feet and lower legs. The likely diagnosis is
21. A 35-year-old patient with rheumatoid arthritis
has a severe hallux valgus deformity with a (A) arterial embolism
dislocated first metatarsophalangeal joint. She (B) arteriosclerosis obliterans
takes ibuprofen, prednisone, and methotrexate for (C) dissecting aortic aneurysm
the arthritis and has a severe penicillin allergy. (D) Raynaud's phenomenon
Her bunion-last, extra-depth shoes no longer
relieve her symptoms and she requests surgical 25. Which of the following findings should be
intervention. The podiatric physician decides to expected in a 14-year-old patient with congenital
perform a first metatarsophalangeal joint fusion soleus equinus?
under general anesthesia using internal fixation. (A) Limited dorsiflexion of the ankle with the
In addition to a complete blood cell count (CBC) knee flexed and extended
and a urinalysis, preoperative studies should (B) Limited dorsiflexion of the ankle with the
include knee extended but not flexed
(A) a thyroid profile (C) Normal dorsiflexion of the ankle with the
(B) cervical radiographs knee flexed but not extended
(C) patch testing for other antibiotic allergies (D) Normal dorsiflexion of the ankle with the
(D) muscle biopsy to evaluate her potential for knee extended but not flexed
malignant hyperthermia with general
anesthesia 26. A 50-year-old patient presents with a tender mass
on the lateral forefoot. A radiograph reveals an
22. Which of the following is a true statement about expansile “soap bubble” lesion in the fifth
gout? metatarsal. The most likely diagnosis is
(A) Acute septic joint disease mimics gout. (A) giant cell tumor
(B) Blood uric acid levels above 7 mg/dL are (B) osteochondroma
pathognomonic of gout. (C) Ewing’s sarcoma
(C) Early acute gout is best treated by uricosuric (D) unicameral bone cyst
agents.
(D) Salicylates are administered to potentiate the
effects of uricosuric agents.

GO ON TO THE NEXT PAGE.


-23- Practice Test 3

27. When a dorsal wedge osteotomy is performed at 32. Postanesthesia headache is most likely to occur
the base of the third metatarsal, which after which of the following types of anesthesia?
interosseous muscle belly is first seen between
(A) Spinal
the second and third metatarsals?
(B) Local
(A) Second dorsal (C) Epidural
(B) Third dorsal (D) Subarachnoid
(C) Second plantar
(D) Third plantar 33. A 30-year-old patient is scheduled to have
major rearfoot reconstruction. The patient has
28. An abnormal “Q” angle is most commonly a history of asthma and is currently on an
associated with albuterol nebulizer and has taken prednisone,
7.5 milligrams, daily for the past 2 years. Prior
(A) anterior cruciate tears
to surgery, the podiatric physician should do
(B) posterior cruciate tears
which of the following?
(C) bucket handle meniscus tears
(D) patellar malalignment (A) Begin aspirin therapy 24 hours prior to
surgery.
29. A 40-year-old woman complains of a painful (B) Begin antibiotic prophylaxis to prevent
hammered hallux of the right foot. Examination endocarditis.
shows a rigid hammering of the interphalangeal (C) Increase the prednisone dose to at least
joint of the right hallux with an associated 20 milligrams per day.
flexible plantarflexed first ray. Which of the (D) Stop the prednisone 2 days prior to surgery
following is the most appropriate operation for to aid in wound healing.
this patient?
34. Which of the following steps should be followed
(A) Jones tenosuspension with interphalangeal
when an individual is using an automated
fusion
external defibrillator?
(B) Arthroplasty of the interphalangeal joint
(C) Dorsiflexory wedge osteotomy of the first (A) Wipe the patient's chest dry with alcohol.
metatarsal with interphalangeal fusion (B) Remove any medication patch prior to use.
(D) First metatarsophalangeal joint fusion (C) Place the pads over implantable devices.
(D) Delay use until after hypothermia, if present,
30. Penicillin dose adjustment must be considered is treated.
when
35. Chest compressions in infant cardiopulmonary
(A) renal excretion is impaired
resuscitation should be
(B) hepatic excretion is impaired
(C) hepatic metabolism is impaired (A) 0.5-1.0 inch
(D) oral absorption is increased (B) 1.0-1.5 inches
(C) 1.5-2.0 inches
31. An increased risk of deep vein thrombosis is (D) 2.0-2.5 inches
associated with
36. Which of the following skin lesions is derived
(A) diabetes mellitus
from a distinctive cell line of melanized cells?
(B) iron deficiency anemia
(C) polycythemia vera (A) Dermal nevus
(D) rheumatoid arthritis (B) Junctional nevus
(C) Compound nevus
(D) Blue nevus

GO ON TO THE NEXT PAGE.


Practice Test 3 -24-

37. Which of the following is NOT a side effect of 42. A 42-year-old patient is seen by a podiatric
glucocorticoids? physician 48 hours after an ankle sprain.
Examination reveals moderate to severe edema
(A) Psychosis
and moderate ecchymosis. Which of the
(B) Edema
following would be the best course of treatment?
(C) Hypokalemia
(D) Chronic active hepatitis (A) Primary repair
(B) Lateral ankle stabilization
38. A 60-year-old resident of a skilled nursing (C) A short leg weight-bearing cast
facility suffers from osteomyelitis of her (D) A Jones compression dressing, non-weight-
calcaneus. Her past medical history includes bearing
stroke syndrome, inability to swallow, and
chronic nasogastric tube feeding. She undergoes 43. In order to attain better x-ray penetration of the
a 2-hour procedure. Two days later the patient is tarsal bones and avoid an increase in the overall
noted to have fever with shortness of breath and film density the
production of fetid sputum. The most likely (A) kilovoltage and the mAs both must be
diagnosis is increased
(A) acute bronchitis (B) kilovoltage must be increased and the mAs
(B) atelectasis must be decreased
(C) aspiration pneumonia (C) kilovoltage must be decreased and the mAs
(D) pulmonary embolism must be increased
(D) kilovoltage and the mAs both must be
39. Hand-foot-and-mouth disease is a mild infectious decreased
disease of childhood caused by a
44. Two measurements used for the evaluation of
(A) staphylococcus bacteria
Haglund’s deformity are the
(B) streptococcus bacteria
(C) herpesvirus (A) parallel pitch lines and Böhler’s angle
(D) coxsackievirus (B) parallel pitch lines and the Fowler-Philip
angle
40. In normal radiographic anatomy, the lateral (C) Fowler-Philip angle and Böhler’s angle
(third) cuneiform bone is LEAST visible with (D) angle of Gissane and Böhler’s angle
which of the following standard pedal studies?
45. Which of the following materials would be
(A) Anteroposterior foot
LEAST appropriate for extrinsic posting of
(B) Lateral foot
functional orthotics?
(C) Medial oblique foot
(D) Lateral oblique foot (A) Acrylic
(B) Crepe
41. Bone healing with rigid internal fixation is (C) Cork
known as (D) Leather
(A) secondary bone healing
46. The ability of a test to give a negative finding
(B) periosteal bone healing
when the person screened is truly free of the
(C) primary bone healing
condition is called the test’s
(D) cartilage phase healing
(A) validity
(B) sensitivity
(C) specificity
(D) variation

GO ON TO THE NEXT PAGE.


-25- Practice Test 3

47. Fog on a developed radiographic film most likely 51. A patient presents with a chief complaint of
indicates interdigital itching and maceration. Physical
examination reveals fissuring and maceration of
(A) a light leak
the third and fourth web spaces bilaterally. A
(B) excessive fixing
KOH preparation is performed and reveals
(C) low developer temperature
multiple budding pseudohyphae. The most likely
(D) weak developing solution
diagnosis is infection caused by
48. Statutes of limitations are determined by (A) Candida albicans
(B) Trichophyton rubrum
(A) state laws
(C) Trichophyton mentagrophytes
(B) state podiatry associations
(D) Corynebacterium minutissimum
(C) the Department of Justice
(D) the American Podiatric Medical Association
52. Which of the following dysplasias is
characterized by multiple bone islands in
49. Which of the following is an appropriate
periarticular areas?
treatment for a Salter-Harris type V fracture?
(A) Osteopetrosis
(A) An Ace bandage
(B) Osteogenesis imperfecta
(B) A functional orthosis
(C) Osteopoikilosis
(C) Casting immobilization
(D) Melorheostosis
(D) Open reduction internal fixation
53. Common precipitating triggers of asthma include
50. A 25-year-old runner presents with achy soreness
all of the following EXCEPT
poorly localized to the lateral border of the foot
and complains of something “going out of” and (A) smoke
“popping into” place. This condition has been (B) exercise
present for several months with periods of (C) dust mites
exacerbation and remission. The area is not (D) topical cortisone
warm or swollen and the patient has never
noticed edema or redness. Palpation localizes 54. In long bones, primary ossification occurs at
tenderness to the calcaneocuboid joint. The most which of the following sites?
likely diagnosis is (A) Metaphysis
(A) plantar fasciitis (B) Diaphysis
(B) a midtarsal stress fracture (C) Epiphysis
(C) cuboid subluxation syndrome (D) Metaphyseal-diaphyseal junction
(D) peroneal tendinitis

GO ON TO THE NEXT PAGE.


Practice Test 3 -26-

55. A patient who presents with an adducted, The next 2 questions refer to the following case.
toe-walking, scissor-type gait most likely has
(A) cerebral palsy A patient with a documented penicillin allergy
(B) muscular dystrophy inadvertently receives an intravenous infusion of
(C) multiple sclerosis cefazolin. The patient complains of shortness of
(D) amyotrophic lateral sclerosis breath and develops a rash and pruritus.

56. A patient presents with a hemoglobin A1c of 8.0


and a fasting blood glucose level of 210 mg/dL. 59. Which of the following medications is indicated
Clinical examination strongly indicates that as the first line of therapy?
sensory polyneuropathy is present. A symptom (A) Epinephrine
of this would be (B) Sodium bicarbonate
(A) Babinski’s reflex (C) Verapamil
(B) a positive Thompson test (D) Nifedipine
(C) diminished deep tendon reflexes
(D) an inability to feel a 10-gram Semmes- 60. As part of the workup a complete blood cell
Weinstein monofilament count (CBC) is obtained. Which of the following
cell types are most likely to be elevated?
57. Standards for quality of care by health systems (A) Monocytes
are developed by the (B) Lymphocytes
(A) National Institutes of Health (C) Eosinophils
(B) Centers for Medicare & Medicaid Services (D) Basophils
(C) National Committee for Quality Assurance
(D) Joint Commission on Accreditation of
Healthcare Organizations

58. The most common side effect of fentanyl,


sufentanil, and thiopental is
(A) tachycardia
(B) hypokalemia
(C) respiratory depression
(D) hyperglycemia

END OF PRACTICE TEST 3


-27- Practice Test 1

PRACTICE TEST 1
ANSWER KEYS AND RATIONALES

Sequence: Key Sequence: Key


1 D 7 D
Stasis ulcers are caused by blood pooling in the legs Removal of the proximal phalangeal base of the
secondary to venous distention and a failure of the hallux may result in shortening and postoperative
venous valves to maintain an upward flow of venous contracture of the hallux. Maintaining this space
blood against gravity. Compression dressings are an with a hemi-implant may reduce this complication.
effective way to compensate for this lack of proper
venous function. Sequence: Key
8 B
Sequence: Key Stridor, laryngospasm, wheezing, angioedema, hives,
2 A possible hypotension, and circulatory collapse are
Hemoglobin A1c is an end product produced when symptoms of anaphylaxis. A lack of oxygen due to
hemoglobin glycosylates due to hyperglycemia. bronchospasm is the primary cause of death.
Assessment of hemoglobin A1c provides useful
information related to a patient’s regulation of blood Sequence: Key
glucose during the previous 3 months. 9 C
Morphine causes depression of ventilation.
Sequence: Key
3 C Sequence: Key
Approximately 2 to 6 cm proximal to its insertion, 10 C
the Achilles tendon has an area of decreased blood Stance phase is shortened as running speed increases,
flow, making this region more vulnerable to strain which decreases the time in stance and results in the
and rupture. elimination of the brief time when both feet are in
stance at the same time.
Sequence: Key
4 A Sequence: Key
The stratum lucidum is one of the five layers of the 11 A
normal plantar skin epidermis. The stratum Fibrous scar tissue often causes adhesions between a
granulosum lies just above the mucosal layer of the tendon and its sheath and the overlying skin.
epidermis.
Sequence: Key
Sequence: Key 12 D
5 B Although the intensity of the murmur does not
MRI uses nonionizing radio frequency pulses that are indicate the severity of a valve defect, a patient
absorbed and then emitted by tissue lying inside a with cardiac murmur should be medically cleared by
magnetic field. an M.D. or D.O. prior to undergoing any invasive
treatment such as surgical intervention. In this case,
Sequence: Key there is the potential for incision and drainage.
6 B
The patient has several risk factors for deep vein
thrombosis (DVT), i.e., smoking, immobilization,
and oral contraceptive use. The description of the leg
as “hot, red, and indurated” is most consistent with
DVT. Homan’s sign is a diagnostic sign, although
unspecific for DVT.
Practice Test 1 -28-

Sequence: Key Sequence: Key


13 C 20 A
The Centers for Medicare & Medicaid Services The clinical and radiographic features described in
(CMS) is the government agency responsible for this question are associated with an excessively
determining the fees and services that are eligible for pronated foot, such as flexible flatfoot.
payment for Medicare and Medicaid recipients.
Federal law to provide that direction defines this Sequence: Key
agency. 21 A
The implant functions as a spacer between the
Sequence: Key proximal phalanx and the first metatarsal after
14 D the painful joint has been resected.
On MRI, a rupture or an injury appears as an area of
increased signal in the tendon. Sequence: Key
22 C
Sequence: Key Sickle cell anemia manifests itself in infancy.
15 C Clinical manifestations include all the symptoms
Subtalar joint coalition is most often associated with described in the question. Management is largely
pes planus and peroneal spasm, with available motion symptomatic and supportive.
seen usually in the pronatory direction. All of these
findings are associated with a decreased calcaneal Sequence: Key
inclination angle. 23 C
A partially threaded screw is used to create
Sequence: Key interfragmental compression by the lag principle.
16 A This technique is commonly used to provide the best
Escherichia coli is transmitted from fecal material compression and alignment for an Austin
and isolated from stool cultures. In a patient with bunionectomy.
bowel incontinence, the ulcer is most likely infected
from the patient’s fecal material. Sequence: Key
24 B
Sequence: Key In genetically susceptible patients, an oxidative
17 B trifluoroacetyl metabolite of halothane may evoke the
A tort is an act of wrongdoing, which is civil in production of neoantigens directed against
nature rather than criminal. It is a wrong independent hepatocytes.
of contract or criminality. Health care providers have
a duty to do what is right and reasonable. The Sequence: Key
elements of the tort of malpractice address situations 25 B
in which there is a breach of duty that causes or The female athlete triad, which affects women and
results in harm. girls in many sports, is defined by the following
symptoms: disordered eating (bulimia or anorexia),
Sequence: Key amenorrhea, and osteoporosis.
18 C
When a wet-to-dry dressing is removed, superficial Sequence: Key
necrotic tissue is also removed from the wound. 26 D
Acetylcholine is found in the autonomic nervous
Sequence: Key system, which is not involved in the development of
19 A muscular dystrophy.
The talus adducts and plantarflexes in closed chain
pronation, and actually decreases the amount of
articulation of the talus and navicular.
-29- Practice Test 1

Sequence: Key Sequence: Key


27 B 34 A
Compensation for a structurally long limb produces Weakness and paralysis of muscle groups are not
excessive pronation of the foot on the involved side. characteristic of somatization disorder.
The wear pattern of the shoe on the involved foot
would, therefore, be greatest on the medial side. Sequence: Key
35 D
Sequence: Key Many types of chronic skin irritations and long-
28 C standing ulcers, such as venous stasis ulcers, can
Incidence is the number of new cases of a specific undergo malignant transformations, the vast majority
disease occurring during a given period in a specified of which result in well-differentiated squamous cell
population. In this question there are 200 nursing carcinoma.
homes residents. In one year, there were 12 patients
who presented with onychomycosis. During the next Sequence: Key
year, there were 5 new cases diagnosed. The 36 A
incidence rate for the second year is 5/200, A monocyte is a large phagocytic cell that transforms
representing only new cases. into a macrophage once it enters tissues.

Sequence: Key Sequence: Key


29 C 37 B
Hemarthrosis from hemophilia occurs in up to two- Flaws in the fixation process are the most common
thirds of patients and is heralded by pain, warmth, cause of yellow discoloration of radiographic film.
and stiffness. The knee, elbow, and ankle are the
joints most frequently affected. Sequence: Key
38 A
Sequence: Key The systematic evaluation and examination findings
30 A are indicative of posterior tibial tendinitis. The
The Thompson test is performed by squeezing the tendon is intact due to the inversion noted with the
calf muscle of a patient lying in the prone position, heel rise test. Excessive calcaneal eversion with
and observing for plantarflexion of the foot. Absence progressively increasing forefoot abduction is a
of foot plantarflexion is indicative of disruption of hallmark of posterior tibial tendinitis. Injection of a
the Achilles tendon. steroid near or within a tendon increases the risk of
iatrogenic rupture. Injected corticosteroids (and even
Sequence: Key oral steroids) have been well documented to
31 D predispose a tendon to rupture because they weaken
Ventricular fibrillation is a life-threatening collagen cross-linking.
dysrhythmia that occurs in the early phases of a
myocardial infarction. Sequence: Key
39 C
Sequence: Key In muscular dystrophy, contractures of the
32 B gastrocnemius muscles appear early and result in
The nutrient artery of the first metatarsal enters the tightening of the heel cords, which causes the patient
midshaft on the lateral aspect. to have an equinus deformity and walk on the toes.

Sequence: Key Sequence: Key


33 D 40 B
The apophysis of the calcaneus can develop normally Bence Jones proteins in the urine are diagnostic of
from multiple ossification centers with the adjoining multiple myeloma (a malignancy of plasma cells).
metaphysis appearing sclerotic, irregular, or even These proteins are produced by plasma cells and are
serrated. considered the first tumor marker.
Practice Test 1 -30-

Sequence: Key Sequence: Key


41 A 48 D
Of the choices listed, only the Sayre is a periosteal The dermatologic lesion most commonly associated
elevator. with Lyme disease is erythema chronicum migrans.

Sequence: Key Sequence: Key


42 A 49 B
Open reduction and internal fixation are now The anatomic location of the patient’s symptoms
indicated for the patient. Spinal anesthesia would narrows the diagnosis to an intra-articular injury of
provide anesthesia at the appropriate level for lower the ankle joint.
extremity surgery with a thigh tourniquet. Also, with
spinal anesthesia there is no risk that the food Sequence: Key
recently eaten by the patient will be aspirated. 50 A
Pyoderma gangrenosum is thought to be an
Sequence: Key autoimmune disease and can be caused by many
43 D systemic diseases, although the exact etiology is
Latent ankle diastasis requires no reduction and is unknown. It is associated with inflammatory bowel
treated by cast immobilization. Distal syndesmotic diseases such as ulcerative colitis and Crohn’s
ligamentous injuries without fractures or gross disease.
widening of the ankle mortise are treated
nonoperatively with a short leg cast, followed by Sequence: Key
physical therapy. 51 A
Most inversion ankle sprain injuries occur when the
Sequence: Key foot is plantarflexed and inverted at the ankle joint,
44 D which results in instability. The anterior talofibular
The orientation of this view is 45 degrees from the ligament is the first ligament of the lateral ankle area
long axis of the foot. This orientation most closely to be maximally stretched and torn with this type of
parallels the plane of the posterior and middle injury.
calcaneal facets, thus visualizing the joint space.
Sequence: Key
Sequence: Key 52 A
45 B Freiberg’s infraction is a true osteonecrosis of the
Osteoblast and osteoclast activity both increase in second metatarsal head, with the inflammation
Paget’s disease. causing the initial joint space widening. The
resultant osteoarthritis causes the subchondral
Sequence: Key sclerosis commonly seen.
46 A
Rowe classifications I through III describe extra- Sequence: Key
articular calcaneal fractures, which comprise about 53 D
25 percent of calcaneal fractures and have the best Patients with multiple sclerosis do not usually have
prognosis. signs and symptoms affecting the lower motor
neurons. Lower extremity reflexes are generally
Sequence: Key hyperactive.
47 A
Since there is no visible fracture on x-ray, an
epiphyseal fracture of the lateral malleolus is likely.
A short leg cast for 3 weeks would be the treatment
of choice.
-31- Practice Test 1

Sequence: Key Sequence: Key


54 C 57 A
Charcot neuroarthropathy is a progressive Aspirin inhibits prostaglandins.
deterioration of a joint characterized by peripheral
neuropathy. The goal of treatment is to maintain a Sequence: Key
stable foot in order to decrease the tendency of the 58 C
foot to subluxate or dislocate without support. A Coagulase-positive Staphylococcus aureus has
custom molded shoe will help maintain the foot in a consistently been the most common infecting
stable position and help protect the extremity from organism in postoperative infections. This also
repetitive microtrauma and skin breakdown. applies to foot and ankle surgery. Closely following
this is the less but increasingly virulent organism
Sequence: Key coagulase-negative Staphylococcus epidermidis.
55 C
Osteoid osteoma is classically described as associated Sequence: Key
with pain at night, relieved by the use of salicylates. 59 D
Pain and the rapid increase in size without trauma
Sequence: Key make cavernous hemangioma the most likely
56 C diagnosis.
A thorough medical history is the only method listed
that covers the identification of all of the prescription Sequence: Key
and over-the-counter medications a patient has taken 60 A
or is taking. An aggressive lesion is best treated with surgical
curettage and packing.
Practice Test 2 -32-

PRACTICE TEST 2
ANSWER KEYS AND RATIONALES

Sequence: Key Sequence: Key


1 B 7 B
Herpes simplex is diagnosed with a Tzanck smear in The anatomic location of the patient’s pain narrows
which fluid from an intact vesicle is smeared on a the diagnosis to the anterior muscle compartment.
glass slide, dried, and stained with Giemsa’s stain. Shin splints are a common injury in runners and are
most symptomatic when a person is running on hills.
Sequence: Key Overpronation is often the etiology.
2 A
The most serious complication of Paget’s disease is Sequence: Key
sarcomatous degeneration. Osteosarcomas are most 8 C
common but fibrosarcomas and chondrosarcomas can Rheumatoid arthritis is characterized by synovial
also be seen. inflammation. Joint destruction in rheumatoid
arthritis targets articular cartilage, ligaments, tendons,
Sequence: Key and bone.
3 B
A CT scan is definitive in the diagnosis of osseous Sequence: Key
subtalar joint coalition, particularly in terms of 9 A
preoperative planning. Conservative therapy that consists of rest, anti-
inflammatory medication, and shoe modification
Sequence: Key is the first step in the treatment of a painful pes
4 B planus deformity.
Pulmonary emboli are usually released from deep
venous structures of the lower extremity. They lodge Sequence: Key
in pulmonary vasculature, which causes a decrease in 10 C
oxygen levels and an increase in pulmonary blood Cutaneous larva migrans is a disorder caused by
pressure. The result is air hunger, tachypnea, and nematode larvae, is serpiginous in appearance, and
dyspnea. responds to treatment with thiabendazole.

Sequence: Key Sequence: Key


5 A 11 B
Patients who take procainamide for prolonged Sever’s disease is limited to the posterior aspect of
periods often develop anticardiolipin antibodies as the heel and resolves by the time the apophysis fuses
well as anti-DNA and anti-histone antibodies. Drug- with the body of the calcaneus.
induced systemic lupus erythematosus is rare and
usually subsides once the medication is discontinued. Sequence: Key
12 A
Sequence: Key The Bunnell suturing technique is an effective and
6 D commonly used end-to-end technique for the repair
Melanocytes are contained in the stratum of tendon ruptures.
germinativum, also known as the basal layer of the
epidermis. Sequence: Key
13 C
Malignant melanoma would produce the type of
pigmented skin changes that could mimic Kaposi’s
sarcoma.
-33- Practice Test 2

Sequence: Key Sequence: Key


14 C 21 B
Osteoid osteoma is a small benign osteoblastic tumor. In pronation, the talus is positioned more distally and
It initially appears as a sclerotic bone island with a plantarly, which increases the talar declination angle.
central lucent defect. The calcaneus is lowered and everted, which
decreases the calcaneal inclination angle.
Sequence: Key
15 D Sequence: Key
Prophylaxis against Staphylococcus aureus and 22 A
Staphylococcus epidermidis consists of amoxicillin Manipulation of the kVp will affect radiographic
3 gm orally 1 hour before the procedure and 1.5 gm subject contrast. A lower kVp will increase the
orally 6 hours later. contrast; a higher kVp will decrease the contrast.

Sequence: Key Sequence: Key


16 A 23 A
The goal of the semi-weight-bearing technique is to A Salter-Harris type I fracture is simply a fracture of
capture an accurate impression of the size and shape the growth plate without joint involvement.
of the foot. The cast produced via this technique is
most appropriate for an accommodative orthosis. Sequence: Key
24 A
Sequence: Key In the lateral view, the fifth toe is juxtaposed against
17 B the other four toes and the metatarsal heads;
Generally, excisional biopsies for skin lesions are therefore, it is not clearly visible.
performed in the shape of an ellipse with pointed
ends. The long axis of the ellipse should be roughly Sequence: Key
three times its central and greatest width to prevent 25 A
buckling of the skin. In addition to gloves, OSHA-designated personal
protection includes gowns and eye protection, which
Sequence: Key provide protection especially from blood splatter.
18 D
The posterior tibial muscle, an inverter of the foot, is Sequence: Key
the only muscle listed that is powerful enough to 26 C
support the arch and counteract the effect of fibularis The cane is held on the stronger side of the body and
(peroneus) brevis, an everter of the foot. One of the the weight is shifted away from the weaker/painful
main characteristics of posterior tibial dysfunction is limb.
loss of the arch.
Sequence: Key
Sequence: Key 27 D
19 C Congenital vertical talus is a complex deformity
The positions listed in the answer slightly overcorrect involving bone, tendon, capsule, and soft tissue
for the preoperative hammer toe deformity and are abnormality. It has a similar appearance at birth to
the positions most likely to prevent recurrence. talipes calcaneovalgus, but there is lack of motion at
the subtalar joint and ankle joint within 6 months
Sequence: Key after birth, a negative calcaneal inclination angle, and
20 A complete dislocation of the talonavicular joint. It
This sensitivity refers to the significant genetic effect cannot be reduced with serial casting.
radiation has on the chromosomes in germ cells.
Sequence: Key
28 A
The lumbrical tendon is located plantar to the deep
transverse metatarsal ligament.
Practice Test 2 -34-

Sequence: Key
Sequence: Key 36 C
29 B Molluscum contagiosum is a viral infection of the
A patient with a loss of proprioception will also have skin characterized by the appearance of a few to
a loss of protective sensation, which is best detected numerous small, pearly, umbilicated papular
with a 10-gram monofilament. epithelial lesions that contain numerous inclusion
bodies known as molluscum bodies.
Sequence: Key
30 B Sequence: Key
In closed kinetic chain supination of the subtalar 37 C
joint, the talus and tibia externally rotate, the Practically all venous stasis ulcers exhibit a red,
calcaneus inverts, and the longitudinal arch of the granulating base.
foot is elevated, all of which result in a functionally
longer limb. Sequence: Key
38 D
Sequence: Key Lower extremity dependent edema is a sign of right-
31 C sided congestive heart failure.
The normal value for the talocalcaneal angle is
20 to 40 degrees. The talocalcaneal angle becomes Sequence: Key
smaller with supination, so a value of 10 degrees 39 C
would be indicative of supination, not excessive Surgically changing the insertion of the flexor
pronation. digitorum longus tendon from the base of the distal
phalanx to the dorsal aspect of the proximal phalanx
Sequence: Key will correct the proximal interphalangeal joint
32 C contracture of the hammer toe by pulling the
With the exception of halothane, all volatile proximal phalanx into a more congruous position
anesthetics contain an ether linkage. Volatile with the middle phalanx.
anesthetics with an ether linkage are less likely than
halothane to produce cardiac dysrhythmias in the Sequence: Key
presence of exogenous epinephrine injection. 40 C
The failure to report a suspicion of patient abuse may
Sequence: Key potentially result in harm or death. In such cases, the
33 B practitioner may be criminally liable.
The tibialis anterior tendon remains attached at its
insertion and is simply routed through a groove Sequence: Key
created in the navicular. Since its insertion has not 41 D
changed, it is a translocation. Rocker sole modification is used for any type of
pathologic or pathomechanical condition that limits
Sequence: Key normal movement of the ankle, metatarsal, or
34 A metatarsophalangeal joint. A rocker sole is a
People with osteoarthritis frequently have bony dynamic lever that assists with the normal movement
enlargement of the distal interphalangeal joints, of the lower extremity during gait and prevents
referred to as Heberden’s nodes. flexing of the sole of the shoe during all phases of
gait.
Sequence: Key
35 C Sequence: Key
Bupivacaine is not recommended for use in children 42 D
under 12 years of age. It is long acting, and long- Routine primary suturing is categorized as healing by
acting local anesthetics carry an increased risk of first intention.
cardiac and CNS toxicity.
-35- Practice Test 2

Sequence: Key Sequence: Key


43 C 50 C
Motor innervation to the anterior compartment Warfarin is an oral anticoagulant and therefore has no
muscles of the lower leg is supplied by the deep effect on established thrombi, nor does it reverse
fibular (peroneal) nerve. ischemic tissue damage.

Sequence: Key Sequence: Key


44 B 51 A
During ankle joint dorsiflexion, the anterior and Diazepam is insoluble in water and contains
widest portion of the talar dome is wedged in the propylene glycol, a tissue irritant that causes pain
ankle mortise. With inversion, the lateral border of on injection and venous irritation.
the talar dome is compressed against the lateral
malleolus. Sequence: Key
52 B
Sequence: Key The correct ventilation-to-chest-compression ratio in
45 B one-rescuer CPR is 2 ventilations for every 15 chest
Hemolytic anemia is normochromic and normocytic. compressions.

Sequence: Key Sequence: Key


46 C 53 B
A pronated subtalar joint would result in pronation of Hyperparathyroidism, which can affect bone in many
the midtarsal joint, and forefoot valgus would ways, appears on radiographs as osteopenia. Excess
increase. parathyroid hormone results in an increase in bone
metabolism, which may appear as microfractures,
Sequence: Key bone cysts, brown tumors, and pathologic fractures.
47 C
The Hawkins classification of talar neck fractures is Sequence: Key
universally accepted as a way of defining the 54 A
different fracture patterns of this injury. The severity Mycobacterium avium-intracellulare is one of the
of the injury and the associated risk of avascular most common bacterial infections in people with
necrosis of the talus increases in a stepwise fashion HIV. One study demonstrated the presence of these
from type I to type IV. The severity of the injury is bacteria in 43 percent of people within 2 years of an
directly correlated with anatomic disruption of blood HIV diagnosis. This is a disseminated infection that
supply to the talus. thrives in the presence of a weakened immune system
and low CD4+ counts.
Sequence: Key
48 D Sequence: Key
In hypothyroidism, the relaxation of Achilles tendon 55 B
reflexes is delayed. This is sometimes referred to as A cohort represents a designated group of individuals
hypothyroid neuropathy. who are followed or traced over a period of time.
Epidemiology is the study of how a disease presents
Sequence: Key itself or occurs within a population. Prospective
49 C studies examine patients rather than a review of the
The patient’s presentation is fairly classic for Graves’ records. To study a disease and how it presents from
disease. The audible bruit described is generally one point in time forward, one looks at those who are
found only in Graves’ disease. The classic triad in free of the disease.
this common disorder is hyperthyroidism with goiter,
ophthalmopathy, and dermopathy.
Practice Test 2 -36-

Sequence: Key Sequence: Key


56 D 59 A
Heel pain in boys 4 to 7 years old is more likely to be Severe localized edema, erythema, serosanguineous
caused by irritation of the growth plate of the drainage, induration, and pain on palpation 3 days
calcaneus. postoperatively all indicate that a vessel was
damaged, causing a hematoma.
Sequence: Key
57 B Sequence: Key
The patellar reflex tests the femoral nerve, which 60 B
innervates the quadriceps tendon. The nerve is made Sickle cell crisis is a medical emergency in African-
up of spinal roots L2, L3, and L4. American patients, and the risks associated with this
condition can be minimized by using a sickle cell
Sequence: Key trait screening test.
58 B
The use of oral contraceptives puts a woman at risk
to develop deep vein thrombosis.
-37- Practice Test 3

PRACTICE TEST 3
ANSWER KEYS AND RATIONALES

Sequence: Key Sequence: Key


1 B 8 B
The initial treatment of choice for deep vein The crest pad is used when one or several digits of a
thrombosis is intravenous heparin. patient’s foot are in moderate contracture and are
becoming irritated at the distal end of the digit. Its
Sequence: Key purpose is to mechanically dorsiflex and extend the
2 D affected digits. It creates an upward thrust and places
Measurement of the angle formed by the bisection of weight bearing on the padded plantar aspect of the
the second metatarsal and the line perpendicular to toes, alleviating painful lesions.
the lesser tarsus bisection gives the metatarsus
adductus angle. Sequence: Key
9 A
Sequence: Key The clubbing seen in lung cancer and certain other
3 B pulmonary diseases is generally considered to be a
A KOH preparation and/or fungal culture should be secondary effect of hypoxemia-induced local
done to ensure that a fungal infection exists before vasodilatation and is thus an acquired deformity.
griseofulvin is prescribed due to the risk of side
effects. Sequence: Key
10 D
Sequence: Key The Occupational Safety and Health Administration
4 A is the government agency responsible for chemical
Adaptic dressing is the only nonadherent dressing exposures.
listed.
Sequence: Key
Sequence: Key 11 C
5 B Nonsteroidal anti-inflammatory drugs do not cause
The Hardcastle classification is universally respiratory depression.
recognized as a diagnostic and therapeutic system
specific only to Lisfranc’s joint dislocation injuries. Sequence: Key
12 A
Sequence: Key MRI will assess the ligaments and tendons as well as
6 D the integrity of the associated osseous structures of
The common fibular (peroneal) nerve winds around the foot.
the neck of the fibula as it travels distally, lying
superficial to the bone and deep to the skin, distal to Sequence: Key
the knee. If the proximal edge of a short leg cast is 13 B
not properly padded, it can put pressure on the nerve The bifurcate ligament, which originates on the
and damage it. anterior process of the calcaneus and attaches to the
navicular and cuboid, is associated with this type of
Sequence: Key fracture.
7 D
The ossification center of the navicular bone appears
at approximately 3 years of age and is the last center
of ossification to appear in the foot.
Practice Test 3 -38-

Sequence: Key Sequence: Key


14 D 20 A
Radiographic evaluation of chronic tophaceous gout A plantar capsular-ligamentous sprain of the first
usually reveals large bony erosions, which are caused metatarsophalangeal joint occurs when the hallux
by pressure necrosis of bone adjacent to a is hyperextended and compressed at the
periarticular accumulation of monosodium urate metatarsophalangeal joint. This can be caused by the
crystals. friction of artificial surfaces resulting in fixation of
the hallux, which causes hyperextension and
Sequence: Key jamming of the joint on the already dorsiflexed
15 B hallux.
The Salter-Harris classification of epiphyseal injury
defines fracture pattern on the basis of which zone or Sequence: Key
zones of the epiphysis in a long bone are disrupted. 21 B
By definition, the fracture pattern of type I and Atlantoaxial subluxation is present in 40% of patients
type III injuries does not extend into or through the with rheumatoid arthritis, and marked flexion of the
metaphysis of a long bone. neck during anesthesia may cause severe
neurological interruption. The anesthesiologist
Sequence: Key should be consulted preoperatively for evaluation,
16 D and flexion-extension lateral cervical neck
Neisseria gonorrhea is the infective organism that radiographs should be obtained for patients at risk.
most commonly causes septic arthritis in young
adults. Commonly polyarticular in onset, it is often Sequence: Key
associated with skin lesions. 22 A
Acute septic joint disease produces a red, hot,
Sequence: Key swollen, and painful joint, which mimics the
17 D symptoms of gout.
Partial thromboplastin time (PTT) evaluates the
intrinsic coagulation system. It is commonly used to Sequence: Key
monitor heparin therapy. 23 A
The gold standard for the diagnosis of pulmonary
Sequence: Key embolism is the pulmonary angiogram.
18 B
Charcot joint is caused by impairment of deep pain Sequence: Key
sensation or proprioception, which affects the joint’s 24 B
normal protective reflexes. It commonly occurs in Arteriosclerosis obliterans refers to the occluded
patients with diabetes in joints formed by the tarsal peripheral arterial system hallmarked by the
bones. examination findings listed in this question.

Sequence: Key Sequence: Key


19 D 25 A
The severe debilitating effects of rheumatoid arthritis The soleus muscle does not cross the knee joint.
commonly require complete excision of the lesser Limited ankle joint dorsiflexion with the knee flexed
metatarsophalangeal joints, a technique known as the is due to soleus equinus, since the influence of the
Hoffman-Clayton procedure. gastrocnemius and plantaris muscles has been
eliminated.
-39- Practice Test 3

Sequence: Key Sequence: Key


26 A 32 A
Giant cell tumors are benign lesions, usually solitary The headache associated with a spinal anesthetic is
and locally aggressive. They classically appear with believed to result from the loss of cerebrospinal
a radiolucent center and denser periphery. They are fluid through the meningeal needle hole, which
eccentric and expansile, with a border that is well results in decreased buoyant support for the brain.
defined but not sclerotic, and have a “soap bubble” The headache is characteristically mild or absent
appearance. when the patient is supine, but head elevation results
in fronto-occipital headache. In the upright position,
Sequence: Key the brain sags in the cranial vault, putting traction on
27 A the pain-sensitive structures.
When a dorsal incision for a dorsal wedge osteotomy
is performed at the base of the third metatarsal, the Sequence: Key
second dorsal interosseous muscle belly is seen first 33 C
between the second and third metatarsals. Long-term steroid usage suppresses the
hypothalamic-pituitary axis (HPA) and increases
Sequence: Key the risk for hypotensive shock. Preoperative
28 D supplementation with high-dose steroids will help
The Q angle is formed in the frontal plane by two line avert this complication.
segments: one segment from the tibial tubercle to the
middle of the patella, and one segment from the Sequence: Key
middle of the patella to the anterior superior iliac 34 B
spine (ASIS). Patellofemoral tracking problems and If an automated external defibrillator is to be used on
anterior knee pain are associated with an increased a patient who has a medication patch, the patch
Q angle. should be removed prior to use of the defibrillator.
Small burns to the skin may result if a defibrillator
Sequence: Key pad is placed on a medication patch.
29 A
Extensor hallucis longus transfer to the first Sequence: Key
metatarsal head will elevate the plantarflexed first 35 A
ray, and fusion of the hallux interphalangeal joint will In infant cardiopulmonary resuscitation, the chest
prevent a hallux hammer toe. compressions should be 0.5-1.0 inch.

Sequence: Key Sequence: Key


30 A 36 D
Penicillin excretion occurs mainly by glomerular Blue nevi arise from dendritic melanocytes. This
filtration. If renal excretion is impaired, the dose benign nevus is believed to form from nerves and not
must be adjusted to accommodate the decreased renal from the basal layer of the epidermis.
function.
Sequence: Key
Sequence: Key 37 D
31 C Glucocorticoids (steroids) do not cause hepatitis.
The combination of intravascular hyperviscosity due
to a high red cell mass and a high platelet count with Sequence: Key
functionally abnormal platelets puts patients with 38 C
polycythemia vera at high risk for stroke, myocardial Swallowing difficulty, prior stroke, chronic
infarction, and venous thromboembolism. nasogastric tube feeding, and fetid sputum 2 days
postoperatively are all clues of an infection in the
lung.
Practice Test 3 -40-

Sequence: Key Sequence: Key


39 D 46 C
Hand-foot-and-mouth disease has a viral etiology and The specificity of a test is the degree to which it
is caused by a coxsackievirus. accurately provides a negative result for persons who
are free of the disease.
Sequence: Key
40 B Sequence: Key
In the standard lateral view, the third metatarsal- 47 A
cuneiform joint is usually not visible. Light leakage is the most common cause of fogging
on developed radiographic film.
Sequence: Key
41 C Sequence: Key
The two basic requirements for primary bone healing 48 A
are an intact vascular supply to the bone and stable Statues of limitations are a segment of an individual
rigid fixation of the fracture fragments. state’s rights and are, therefore, determined by state
law.
Sequence: Key
42 D Sequence: Key
Treatment of acute ankle injury is generally 49 C
conservative and consists of rest, ice, compression, Salter and Harris described the type V injury as an
and elevation. A modified Jones compression uncommon injury secondary to a crushing force to
dressing to control edema and immobilize the the physis through the epiphysis. There is no visible
extremity for 24 to 73 hours is recommended. fracture and no displacement. It is recommended that
this injury be treated by placing the patient in a non-
Sequence: Key weight-bearing cast for 3 to 4 weeks, followed by
43 B range-of-motion exercises to prevent premature
Increasing the kilovoltage increases the penetrating fusion secondary to immobilization.
power of the x-rays emitted from the x-ray tube, with
more photons reaching the x-ray film through the Sequence: Key
target tissue. This increases the exposure of the film 50 C
and thus increases radiographic density. To keep the The calcaneocuboid joint is prone to abduction in
radiographic density constant, it is, therefore, runners with pronation. This can then progress to
necessary to reduce the mAs, which controls the total subluxation, unless it is controlled with an orthotic
number of x-rays produced at the x-ray tube. device.

Sequence: Key Sequence: Key


44 B 51 A
In Haglund’s deformity, an increase in the Fowler- Candida albicans is a yeast that will reveal
Philip angle of more than 75 degrees can be seen on a pseudohyphae on a KOH preparation.
lateral radiograph. Positive parallel pitch lines or
excessive bone on the calcaneus are also present. Sequence: Key
These measurements are used to evaluate the extent 52 C
of the deformity and to plan surgical correction. Osteopoikilosis is a sclerosing bone disorder
characterized by numerous, small, symmetrical
Sequence: Key bone islands prominent in metaphyseal areas.
45 D
Leather is not a good material for extrinsic posting of Sequence: Key
a functional orthotic due to its tendency to wear and 53 D
compress with weight bearing. Topical cortisone is an anti-inflammatory agent
applied to the skin to treat various dermatologic
conditions and is not known to induce asthma.
-41- Practice Test 3

Sequence: Key Sequence: Key


54 B 58 C
Each long bone usually has separate centers of Fentanyl, sufentanil, and thiopental cause depression
ossification: the shaft (diaphysis) and each end of ventilation. They also cause cardiovascular
(epiphysis). The first (primary) center to ossify is depression and bradycardia, slowed gastric emptying,
always the diaphysis. and antidiuretic hormone release.

Sequence: Key Sequence: Key


55 A 59 A
An adducted, toe-walking, scissor-type gait is most Epinephrine is the drug of choice for shock,
often associated with the spastic disorder of cerebral angioedema, airway obstruction, bronchospasm,
palsy. and urticaria in severe anaphylactic reactions. Its
action is via adrenergic effect.
Sequence: Key
56 D Sequence: Key
The 5.07 (10-gram) monofilament is used to screen 60 C
for protective sensation loss in patients with diabetes. Mild to moderate eosinophilia is associated with
drug allergy. When drug-induced eosinophilia is
Sequence: Key suspected, termination of the offending agent
57 C generally terminates the problem.
The National Committee for Quality Assurance sets
the standards for patient care.
Admission orders (ADC VANDILMAX)
● Admit
● Diagnosis
● Condition
● Vitals
● Ambulatory
● Nursing orders
● Diet
● Ins/outs
● Labs
● Medications
● Ancillary
● Xray
Post op Notes (SAPPPAHEMICCC) (SAPPPAHEMIPC)
● Surgeon Surgeon
● Assistants Assistants
● Pre-op dx Pre-op diagnosis
● Post-op dx Post-op diagnosis
● Procedure Procedure
● Anesthesia Anesthesia
● Hemostasis Hemostasis
● EBL (estimated blood loss) EBL
● Materials Materials
● Injectables Injectables
● Culture Pathology
● Condition Complications
● Complications

Diuretics [“Aces For Bilal’s Ethnic Spirit And Try Hard Insight”]
● Proximal Tubule:
○ Acetazolamide
● Loop
○ Furosemide
○ Bumetamide
○ Ethacrynic Acid
● K+ sparing (Collecting Duct)
○ Spironolactone
○ Amiloride
○ Triamterene
● Thiazides (Distal Convoluted Tubule)
○ Hydrochlorothiazide
○ Indapamide

Triple Arthrodesis (order of fusion according to Coughlin and Mann)


1. STJ
2. TNJ
3. CCJ

Pes Cavus
- Anterior Cavus
 Metatarsal (Lis francs)
 Lesser Tarsal (middle of lesser tarsal bones)
 Forefoot (Choparts)
 Combined
- Rearfoot Cavus
- Pseudoequinus
 In the rigid form of anterior pes cavus, compensatory dorsiflexion cannot be absorbed by the
forefoot. Therefore, the entire foot must dorsiflex at the ankle to allow the heel to contact the
ground
- Radiographic Angles
st
 Meary’s – 1 MT and talar neck
st
 Hibb’s – 1 MT and calcaneus
 Calcaneal inclination angle
 Talar declination angle
 TC angle (Lateral and AP-Kite’s)
 CYMA line
- Coleman Block Test
 To determine if the deformity is flexible or rigid.
 Lateral foot except heel and medial forefoot in on 2inch wooden block.
st
 If heel moves from varus to neutral  deformity is due to PF 1 ray
 If heel stays in varus  deformity is rigid
- If flexible, do ST correction
 Steindler stripping (release PF, abd hallucis, FDB, abd digiti minimi, long plantar ligament)
 STATT
st st
 Jones tenosuspension (EHL through 1 MT head: removes windlass mech.. no PF of 1 MT)
- If rigid, do osseous correction
 Cole
 Japas
 Dwyer (CBWO base lateral)
 Reverse Koutsogiannis (lateral calcaneal slide)

Hallux Limitus
- Drago, Oloff, and Jacobs
o I – functional HL
o II – Joint adaptation
o III – Joint deterioration
o IV – Hallux Rigidus (ankylosis)
- Etiologies
o Trauma**
st
o Short 1 MT
st
o Long 1 MT
o MPE
o Arthritic (DJD)
st
o Hypermobile 1 ray
st
o Round or square 1 MT head
- Seiberg index (evaluation of MPE on lateral xray)
nd st
o The perpendicular distance from the dorsum of the 2 MT to the dorsum of the 1 MT shaft is
st st
measured at the 1 MT neck and 1.5 cm from the 1 MT base.
o The proximal measurement is subtracted from the distal measurement to give the Seiberg index.
o A positive number indicates an elevated first metatarsal.
o On average, the distal measurement is about 8mm in elevatas
- Treatment
o Joint preserving
 Cheilectomy
 Watermann-Green – PF and shortens… note: can also translate capitol frag laterally for IMA
1. Cheilectomy (dorsal and medial prominences)
rd
2. Plantar arm (plantar 2/3 of MT)
3. Distal dorsal arm parallel to articular surface (PASA)
st
4. Proximal dorsal arm (perpendicular to shaft of 1 MT)
 Watermann
nd
 Youngswick (Austin with 2 dorsal arm) – PF and shortens
 Lambrinudi
 Valenti
 Hohmann
o Joint destructive
 Keller
 Mayo
 Stone
 Implants (total and hemi)
 Mc Keever
 Lapidus

DDX List:
Posterior heel pain
1. Achilles tendon rupture
2. Achilles tendonitis or –osis (insertional or non-insertional)
3. Fracture
a. Avulsion fracture of superior posterior calcaneus (Rowe 2)
b. Shepard’s fracture
c. Cedell’s fracture
d. Calcaneal fracture
4. Stress fracture of calcaneus (pathologic or insufficiency fx)
5. Bursitis (retrocalcaneal or subcutaneous calcaneal bursa)
6. Haglund’s deformity (pump bump)
7. Calcaneal apophysitis
8. Osteomyelitis
9. Bone tumor
a. ABC
b. UBC
c. Intraosseous lipoma
d. Enostosis
Dorsal foot pain (proximal)
1. Anterior process of calcaneus fracture (Rowe 1c)
2. Sinus tarsi syndrome (compensation from pes planus?)
3. Stress fracture (any proximal dorsal bones)
4. Fracture
5. Tendonitis or tendon rupture
6. Arthritis
7. Gout
8. Charcot
9. Osteomyelitis
10. AVN
11. Bone tumor
12. CRPS
13. Neuropraxia
14. Ganglion cyst
Dorsal foot pain (distal)
1. Predislocation syndrome
2. Stress fracture (any distal dorsal bones)
3. Fracture
4. AVN
5. Tendonitis or tendon rupture
6. Arthritis
7. Gout
8. Charcot
9. Neuroma
10. Neuropraxia
11. Ganglion cyst
12. Mucoid cyst (if in toes)
13. Osteomyelitis
14. Bone tumor
a. Enchondroma
b. Osteochondroma or chondrosarcoma
Plantar foot pain
1. Tarsal tunnel syndrome
2. Plantar fasciitis
3. Baxter’s neuritis
4. Stress fracture
5. Fracture
a. Calcaneal tuberosity fracture (Rowe 1a)
b. Sesamoid fracture
6. Tendonitis or tendon rupture
7. Neuropraxia
8. Plantar’s wart
9. Plantar fibroma
10. Neuroma
11. Bone tumor
a. UBC
b. ABC
c. Intraosseous lipoma
d. Enostosis
Medial foot pain
1. PTTD
2. Tendon rupture or tendonitis
a. TA
b. TP
3. Medial ligamentous sprain (deltoid)
4. Sustentaculum tali fx (Rowe 1b)
5. Avulsion of navicular tuberosity
6. Stress fracture
7. Tarsal tunnel syndrome
8. Neuropraxia
9. Bone tumor
Lateral foot pain
1. Tendon rupture or tendonitis
a. PB
b. PL
2. Lateral ligamentous sprain (CFL, ATFL, PTFL)
3. Neuropraxia (sural nerve lateral dorsal cutaneous nerve)
4. Stress fracture
5. Fracture
a. Nutcracker fracture
6. AVN
th
7. Avulsion fracture of tuberosity of 5 MT (Stewart)
8. Avulsion fracture of lateral malleolus
9. Peroneal tendon dislocation (Ekert and Davis)
a. SER rupture
th
10. Iselin’s disease (apophysitis of 5 MT base)
11. Tarsal Coalition
12. Bone tumor

Aside:
● 3 main principles of ORIF for ankle fractures
○ Restore length of fibula
○ Fix ankle mortise
○ Fix syndesmosis
● 4 main principles of ORIF for pilon fractures
○ Restore fibular length
○ Fix ankle mortise
○ Fill in cancellous defects
○ Support tibia with plate
● 4 main principles of ORIF for calcaneal fractures
○ Fix height of calcaneus
○ Fix width of calcaneus
○ Take heel out of varus
○ Articular reduction

Journal Articles
● Subtalar Fusion After Displaced Intra-Articular Calcaneal Fractures: Does Initial Operative Treatment
Matter?
○ JBJS 2009: Craig S. Radnay, Michael P. Clare and Roy W. Sanders
○ 2 patient groups: conservative non-ORIF vs. non-conservative ORIF after intraarticular calcaneal fracture.
○ Both groups got post traumatic arthritis and underwent STJ fusion
○ Better functional outcomes and fewer wound complications in patients who originally underwent ORIF
 This is due to the fact that ORIF restored calcaneal shape, height, and alignment which facilitated
fusion.
● Screw Fixation Compared with Suture-Button Fixation of Isolated Lisfranc Ligament Injury
○ JBJS 2009. Panchbhavi
○ Suture-button fixation can provide stability similar to that provided by screw fixation in cadaver specimens
after isolated transection of the Lisfranc ligament.
○ Fixation with a suture button may be an acceptable alternative to screw fixation in the treatment of
isolated Lisfranc ligament injuries, avoiding subsequent surgery to remove the hardware prior to weight-
bearing.
● Cast-Saw Burns: Evaluation of Skin, Cast, and Blade Temperatures Generated During Cast Removal
○ JBJS 2008. Shuler
○ The highest skin temperatures were recorded for fiberglass casts with two layers of padding. The lowest
skin temperatures were recorded for plaster casts with four layers of padding. Four layers of cast padding
compared with two layers significantly reduced skin temperatures for both plaster and fiberglass casts.
● Fractures of the Lower end of the Tibia into the Ankle Joint
○ Ruedi and Allgower 1969
○ 4 main principles of ORIF in Pilon Fractures:
 Restore length of fibula
● Incision: longitudinal parallel to posterior edge of fibula
 Reconstruct articular surface of tibial plafond
● Incision: anterior tibia curving to medial malleolus
 Fill in cancellous defects using bone graft
 Support medial tibia with plate to prevent varus
● T- plate allows 2 cancellous screws to be inserted into lower articular fragments
○ Classification of Pilon Fractures (Ruedi and Allgower)
 Type 1 – minimal displacement
 Type 2 – significant displacement. Not comminuted
 Type 3 – comminuted, displaced, and impacted
● Ankle Fractures
○ Clinics in Podiatric Medicine and Surgery 2006. Mandi et al
○ Always check NV status!! If not intact… maybe think compartment syndrome
○ If vascular compromise exists, it most commonly is caused by severe malalignment or dislocation of ankle
 Emergent reduction should be performed.
○ 3 views always: AP, mortise (15deg internally rotated), lateral
○ On mortise view, assess:
 If medial gutter space is >4mm think lateral displacement of the talus
● Accd to Ramsay and Hamilton… 1mm displacement = 42% loss of tib-talar contact area
 The tibia-fibula overlap should be at least 10 mm.
● <10mm overlap would lead the observer to expect syndesmotic rupture.
○ If >25-30% of articular surface is involved in posterior malleolus fracture fixate
○ Protocol
 Check NV status
 Perform local block
 Close reduce (increase deform6ity, distract, decrease deformity)
 If need to go to OR:
● Debride devitalized tissue
● Aggressive irrigation
● Empiric antibiotics … cephs and aminoglycosides
○ If too swollen…
 Jones compression with posterior splint
 Elevation and ice
 2-3 days is usually sufficient to decrease edema. Look for skin wrinkles to return!!!
○ Fibula usually shortens in ankle fractures causing lateral displacement of the talus 1mm lateral
displacement = 42% loss of tibiotalar contact ankle joint instability!!
○ Fibula Fixation
 Bone reduction forceps to hold alignment
 Interfrag screw (4.0 partially threaded)
rd
 1/3 tubular plate bent to contour the fibula
● SER: Posterolateral surface of fibula to prevent proximal gliding (anti-glide)
● PAB: dynamic compression plate used as buttress
 3.5 screws (6-8 cortices purchased proximal to the fracture line and 4-6 distal) into plate
● 3-4 screws proximal
● 2-3 screw distal (note this cannot always be done due to location of fracture so in this
case you insert what you can)
○ Syndesmosis Fixation
 Cotton test (aka Hook test) can be used to evaluate syndesmosis after fibular fixation
 Repair is recommended if talus shifts laterally 3-4mm
 Approach laterally
 1-2 screws 1.5 – 3.0cm proximal to tibial plafond
● AO recommends use of a fully threaded screw (3-4 cortices)
○ In this paper: 3.5 corticol screws with 4 cortices purchased
○ Some use 4.0 cancellous screws with 3 cortices purchased
 Allows earlier weight bearing
● Tightrope, partially threaded screws, or bioabsorbable screws can also be used
○ Tightrope allows the earliest return to weight bearing and no need for removal
 Angle screws posterolateral to anteromedial (15 degrees) to account for the fact fibula is a little
posterior to tibia
 Make sure foot is in maximal dorsiflexion during repair!
● Fibula moves ~2mm laterally with ankle dorsiflexion
● You lose 0.1 degrees of DF for every degree of PF of the ankle at time of syndesmosis
repair
 Remove syndesmotic screw after signs of radiographic healing
● Reports from 2-4 months before removal
● Beware of full weight bearing… may cause screw breakage
○ Medial malleolar fixation
 Muller classification
● Type A – Avulsion fx distal to ankle mortise
● Type B – Avulsion fx at the level of the ankle mortise
● Type C – Oblique fx of the medial malleolus
● Type D – Vertical fx of medial malleolus (ie. SAD)
 1 or 2 screws (lag fashion) for compression
○ Posterior malleolar fixation
 Most often associated with SER and PER fractures
 If >25-35% of joint surface is involved, posterior frag should be reduced and fixated
 Usually anterior incision (through same incision used for medial malleolar reduction)
● Posteromedial and posterolateral approaches have also been used.
 1 or 2 cannulated screws aided by fluoroscopy
○ Post-op
 SLC with strict NWB 4-6 weeks
 ROM exercises begin at 2 weeks (sagittal plane)
○ Complications
 Post-traumatic osteoarthritis
 Delayed union, malunion, non-union (rare)
 Hardware irritation
● Management of the Diabetic Charcot Foot
○ Diabetes/Metabolism Research and Reviews, 2000. Robert Frykberg
○ Sanders and Frykberg anatomic classification of Charcot neuroarthropathy
■Pattern I - IPJs, phalanges, MTPJs, metatarsals
■Pattern II - Tarsometatarsal joints
■Pattern III - NC, TN and CC joints
■Pattern IV - Ankle joint
■Pattern V - Calcaneus
○ Effective conservative management of Charcot neuroarthropathy is dependent on its presentation pattern.
Once the patient exits the acute phase, management is directed at prolonged or permanent bracing for
ambulation.
■Pattern I-III - Custom full-length inserts + extra depth shoes; severe midfoot deformities: fabrication of
custom shoes to accommodate the misshapen foot
■Pattern IV – Mild: high-top custom-molded shoe with a full-length orthotic device; moderate: solid
AFO + therapeutic shoe; severe: patellar tendon bearing brace in custom shoe
■Pattern V – Immobilization via casting for minimally displaced fractures (NWB until radiographic
evidence of union, 8 weeks)
○ The goal of treatment: to achieve stable plantigrade foot with intact skin
JBJA Journal of Bone and Joint Surgery - these joints on the excursion of the posterior tibial
American 1996 - 1998 tendon in order to determine when it is
February 1997, Volume 79-A, Number 2 appropriate to reconstruct the tendon
241 Motion of the Hindfoot after Simulated concomitantly with these arthrodeses.
Arthrodesis*
Article Motion of the so-called triple joint complex (the
subtalar, talonavicular, and calcaneocuboid joints) is
AUTHOR(S): ASTION, DONNA J., M.D.†; necessary for the foot to accommodate variations in
DELAND, JONATHAN T., M.D.‡; ground surface and rotation from the lower extremity.
OTIS, JAMES C., PH.D.‡; Although gross differences in the amount of motion
KENNEALLY, SHARON, B.S.‡, NEW YORK, N.Y. of the triple joint complex remaining after different
arthrodeses has been recognized in vivo and in
Investigation performed at The Hospital for Special vitro (2-4,9), the amount of motion remaining has not
Surgery, New York City been measured accurately, to our knowledge.
While it is difficult to measure this motion
J Bone Joint Surg [Am] 1997; 79-A; 241-6 precisely in the clinical setting, Mann and
Baumgarten estimated clinically that about 50 per
ABSTRACT: Arthrodesis of the subtalar joint, cent of the motion of the transverse tarsal joint
triple arthrodesis (involving the subtalar, remained after arthrodesis of the subtalar joint. They
talonavicular, and calcaneocuboid joints), double therefore advocated the use of isolated arthrodesis of
arthrodesis (involving the talonavicular and the subtalar joint rather than triple arthrodesis, when
calcaneocuboid joints), arthrodesis of the possible, to leave the foot as flexible as possible.
talonavicular joint, and arthrodesis of the Fogel et al. found that isolated arthrodesis of the
calcaneocuboid joint were simulated in a cadaver talonavicular joint provided satisfactory results but
model, and the range of motion of each joint not clinically limited subtalar motion compared with that
involved in the simulated arthrodesis was of the contralateral foot. With use of a standard
measured with a three-dimensional magnetic space manual goniometer and reference pins in the tibia,

Redistribution of this article permitted only in accordance with the publisher’s copyright provisions.
tracking system. The excursion of the posterior calcaneus, and second metatarsal, Gellman et al.
tibial tendon was also measured under all of these measured the motion of the foot and the ankle in vitro
conditions. We found that any combination of after various combinations of simulated arthrodeses
simulated arthrodeses that included the of the ankle and tarsal joints. Those authors did not
talonavicular joint severely limited the motion of include arthrodesis of the talonavicular or
the remaining joints to about 2 degrees and limited calcaneocuboid joints, or both, or measure the motion
the excursion of the posterior tibial tendon to 25 of individual joints.
per cent of the preoperative value. Simulated Unlike motion of the major joints of the lower
arthrodesis of the calcaneocuboid joint had little extremity, motion of the joints of the foot cannot be
effect on the range of motion of the subtalar joint, measured accurately with an external goniometer. It
and it reduced the range of motion of the is possible, however, to measure the change in
talonavicular joint to a mean of 67 per cent of the position of individual bones of the foot in three-
preoperative value; a mean of 73 per cent of the dimensional space with a three-dimensional magnetic
excursion of the posterior tibial tendon was space tracking system; the range of motion of the
retained. After simulated arthrodesis of the joints can then be calculated. This system has been
subtalar joint, a mean of 26 per cent of the motion used in the past to study the mechanics of the wrist(7,
8)
of the talonavicular joint, 56 per cent of the motion .
of the calcaneocuboid joint, and 46 per cent of the Most pronation and supination of the foot occurs in
excursion of the posterior tibial tendon was the triple joint complex. The posterior tibial muscle
retained. and the peroneal muscles are the principal muscles
The talonavicular joint is the key joint of the acting on these joints. Arthrodesis of any one of the
triple joint complex. The talonavicular joint had joints of the triple joint complex affects the excursion
the greatest range of motion, and simulated of the posterior tibial tendon.
arthrodesis of this joint essentially eliminated The purpose of the present study was to quantify,
motion of the other joints of the complex. in vitro, the effect of simulated arthrodesis of selected
CLINICAL RELEVANCE: Arthrodesis of any of the joints of the triple joint complex on both the range of
joints of the so-called triple joint complex (the motion of the remaining, unfused joint or joints and
subtalar, talonavicular, and calcaneocuboid joints) the excursion of the posterior tibial tendon. We
limits the motion of the remaining, unfused joints. believe that these in vitro data will help a practitioner
However, clinical estimates of these limitations are to determine when to perform a transfer of the flexor
imprecise and motion of the individual joints digitorum longus tendon to replace the function of a
cannot be specifically measured clinically. It is ruptured or otherwise non-functional posterior tibial
important to be able to measure the limitation of tendon concomitantly with arthrodesis.
motion of each joint after arthrodesis of the other
joints in order to understand the clinical Materials and Methods
implication of the arthrodesis. Also, it is necessary Ten fresh-frozen cadaveric foot specimens,
to ascertain the effect of limiting the motion of amputated at the middle of the tibia, were used in this

Downloaded from www.ejbjs.org on October 23, 2006


study. All of the specimens had a grossly normal muscles acting on the triple joint complex. In
posterior tibial tendon and had no radiographic preliminary testing, a pull on the peroneus longus
evidence of osteoarthrosis. The exact ages of the tendon did not cause greater motion in the triple joint
donors were not known, but all feet were from adults complex than a pull on the peroneus brevis tendon
and all growth plates were closed. did.
The soft tissue about the ankle and the malleoli Stainless-steel wires were sutured to the posterior
was left intact, and the more proximal soft tissue was tibial and peroneus brevis tendons proximal to the
cleared from the tibia and the fibula. The ankle was malleoli. The sutures were attached to gauges that
fixed in a neutral position with three large screws. measured linear displacement through wire cables
Elimination of motion of the ankle was necessary to threaded through pulleys. Use of the pulleys allowed
isolate motion of the triple joint complex and to allow a direct line of pull on each tendon and allowed the
reliable and reproducible measurement of the tendon excursion of these tendons to be measured as the foot
excursion. was moved (Fig. 1).
The three-dimensional locations of the talus, As neither the peroneus brevis tendon nor the
calcaneus, navicular, and cuboid were monitored with posterior tibial tendon inserts into the calcaneus,
a magnetic space tracking system (Fastrak; Polhemus testing of the initial specimens was performed with a
Navigational Sciences Division, Colchester, load placed on the calcaneus to stabilize it against the
Vermont). This system consists of a source that talus and to prevent it from acting as a free body.
generates a magnetic field and four sensors, the Subsequent testing showed no difference in joint
positions of which are detected within the magnetic motion or tendon excursion with or without a load on
field. The accuracy of the system is dependent both the calcaneus. However, for consistency, all of the
on the amount of metal in the field and on the specimens were tested with the calcaneus loaded.
distance between the sensor and the source(10). Metal For the purpose of this study, the motion produced
objects of a certain composition, size, and shape can in the triple joint complex by placement of a load on
be used in the space tracking system without causing the peroneus brevis tendon is defined as pronation
interference. The metal components (screws, wires, and the motion produced by placement of a load on
and threaded pins) used during this experiment were the posterior tibial tendon is defined as supination.

Redistribution of this article permitted only in accordance with the publisher’s copyright provisions.
placed in various locations about the source to verify To achieve the pronated position of the foot, a thirty-
that they did not create any detectable distortion. The six-newton load was placed on the peroneus brevis
manufacturer determined the accuracy of the tracking tendon and an eleven-newton load was placed on the
system (0.8 millimeter along each of the three posterior tibial tendon (Fig. 1). The loads were
translational axes and 0.15 degree about each of the exchanged to achieve the supinated position of the
three rotational axes) with the sensor thirty inches foot. These loads were chosen because preliminary
(76.2 centimeters) from the source. The present study testing had demonstrated that greater loads did not
was conducted with the sensor no more than twelve produce noticeable additional motion of the joints
inches (30.5 centimeters) from the source. The under investigation.
accuracy with the shorter distance was determined, After each simulated arthrodesis, the linear
before the study was conducted, to be 0.1 millimeter displacement reading for the excursion of the
along each translational axis and 0.1 degree about posterior tibial tendon and the three-dimensional
each rotational axis. position of the bones were recorded for each of three
The magnetic source of the space tracking system supination trials and three pronation trials. The
was fixed to the test frame, and the sensors were simulated conditions included no arthrodesis,
attached to each of the four bones (Fig. 1). The arthrodesis of the subtalar joint, triple arthrodesis
position of the sensor in each bone was selected so (involving the subtalar, talonavicular, and
that the sensors did not come into contact with each calcaneocuboid joints), double arthrodesis (involving
other throughout the range of motion of the foot. To the talonavicular and calcaneocuboid joints),
attach the sensors rigidly to each bone, each sensor arthrodesis of the talonavicular joint, and arthrodesis
was mounted on a small Plexiglas plate that was of the calcaneocuboid joint. The order of the
connected to two 3.0-millimeter carbon vinyl ester simulated arthrodeses was maintained to minimize
pins. The pins were press-fit into two undersized the number of times that hardware was placed into the
holes in the bone six millimeters apart. Two pins bones to be fixed together, thereby allowing more
were used with each sensor to prevent rotation at the rigid stabilization of the bones. During the
pin-bone interface. experiment, the foot was held in a neutral plantigrade
The specimens were placed plantar side up in the position while each joint was fixed. The subtalar joint
testing apparatus, which stabilized the tibia and was fixed with the heel in the neutral position, as
allowed unrestricted motion of the foot (Fig. 1). fixing of this joint in varus results in more restricted
Placement of the foot plantar side up also simplified motion of the talonavicular and calcaneocuboid
the use of weights and wires for the application of joints(11). To simulate an arthrodesis, a combination
loads. of at least two screws or threaded pins, with a firm
Tendon pulls were used to move the foot through a hold in bone, were placed across the joint to be fixed.
reproducible range of motion without imposing an No cartilage was removed from the joints.
artificial axis of motion. The tendons of the posterior From the data collected, the range of motion of the
tibial and peroneus brevis muscles were selected subtalar, talonavicular, and calcaneocuboid joints and
because they are antagonists and the principal the excursion of the posterior tibial tendon were

Downloaded from www.ejbjs.org on October 23, 2006


calculated for each condition from full supination to (arthrodesis of the talonavicular joint, double
full pronation of the foot. arthrodesis, and triple arthrodesis) (p < 0.0001 for
all). Simulated arthrodesis of the subtalar joint and
Statistical Analysis simulated arthrodesis of the talonavicular joint
Statistical analysis was performed with each foot decreased the tendon excursion, in the nine feet for
acting as its own control. Analysis of variance with which it was recorded, to a mean of 46 and 25 per
pair-wise comparisons with Bonferroni corrections cent, respectively, of the excursion before the
were applied to account for multiple measurements. simulated arthrodeses. After simulated double
A level of significance of p ≤ 0.05 was used. If 3 arthrodesis (in the nine feet for which it was
degrees of motion or more occurred at the site of a recorded) and after simulated triple arthrodesis (in the
simulated arthrodesis, data from that arthrodesis as eight feet for which it was recorded), the remaining
well as data from subsequent conditions that included excursion was a mean of 25 per cent of that before the
the inadequately fixed site, were excluded. For simulated arthrodeses (Fig. 5).
example, if excessive motion was present after the
simulated subtalar arthrodesis, this condition as well Discussion
as the simulated triple arthrodesis were excluded for There have been several clinical studies involving
that foot. However, if excessive motion occurred after patients who had various arthrodeses of the triple
simulated arthrodesis of the calcaneocuboid joint (the joint complex. Lapidus reported on a patient who had
last condition simulated), the measurements of the arthrodesis of the subtalar joint in one foot and
previous simulations were not excluded. arthrodesis of the talonavicular joint in the other. He
reported that the foot that had had arthrodesis of the
Results subtalar joint had better compensatory motion
Before the simulated arthrodeses, the talonavicular compared with the foot that had had arthrodesis of the
joint had the greatest range of motion (36.7 ± 13 talonavicular joint; however, the amount of motion
degrees [mean and standard deviation]), followed by remaining was not quantified in either foot. Mann and
the subtalar joint (20.4 ± 8 degrees) and the Baumgarten retrospectively evaluated eleven feet that
calcaneocuboid joint (14.4 ± 6 degrees), in all ten had had isolated arthrodesis of the subtalar joint.

Redistribution of this article permitted only in accordance with the publisher’s copyright provisions.
feet. With use of the contralateral side for comparison,
Simulated arthrodesis of the talonavicular joint and they estimated that about 50 per cent of the transverse
simulated double arthrodesis significantly limited the tarsal (talonavicular and calcaneocuboid) motion had
range of motion of the subtalar joint, in the nine feet been maintained. Hall and Pennal clinically estimated
for which it was recorded, to a mean of 8 and 9 per that 25 to 50 per cent of transverse tarsal motion
cent, respectively, of the subtalar motion before the remained after arthrodesis of the subtalar joint. In the
simulated arthrodeses (p < 0.0001 for both) (Fig. 2). present study, a mean of 26 per cent of the motion of
Simulated arthrodesis of the subtalar joint resulted in the talonavicular joint and 56 per cent of the motion
a significant decrease in the range of motion of the of the calcaneocuboid joint remained after simulated
talonavicular joint (p < 0.0001) and the arthrodesis of the subtalar joint. Fogel et al., in their
calcaneocuboid joint (p = 0.005), in the nine feet for follow-up study of patients who had had arthrodesis
which it was recorded, to a mean of 26 per cent of the talonavicular joint, noted that subtalar as well
(Fig. 3) and 56 per cent (Fig. 4), respectively, of the as transverse tarsal motion was severely limited. This
range of motion before the simulated arthrodeses. clinical result is in accordance with our in vitro
Simulated arthrodesis of the calcaneocuboid joint had finding that the subtalar and calcaneocuboid joints
little effect on the range of motion of either the retained 3 degrees of motion or less after simulated
talonavicular joint, which retained a mean of 67 per arthrodesis of the talonavicular joint. Our in vitro
cent of the motion that it had had before the measurements are consistent with the in vivo
simulated arthrodeses (Fig. 3), or the subtalar joint, estimates of motion retained by the transverse tarsal
which retained a mean of 92 per cent of its motion joint after arthrodesis of the subtalar joint and by the
(Fig. 2), in the five feet for which these data were subtalar joint after arthrodesis of the talonavicular
recorded. joint.
The mean excursion of the posterior tibial tendon The mean range of motion of the subtalar joint
in the ten feet was 17 ± 5 millimeters (range, ten to (20.4 degrees) before the simulated arthrodeses in the
twenty-seven millimeters) before the simulated present study was within the 18-to-24-degree range
arthrodeses. In the five feet for which it was recorded, determined by Inman in two cadaveric studies(11). The
a mean of 73 per cent of the excursion was results of the present study, however, cannot be
maintained after simulated arthrodesis of the compared with those reported by Gellman et al.
calcaneocuboid joint (Fig. 5). With the number of because those authors determined the motion
specimens in this study, we could not show a remaining in the foot as a whole after various
significant difference between the tendon excursion simulated arthrodeses. Also, Gellman et al. moved the
before the simulated arthrodesis of the foot manually in a predetermined direction, whereas
calcaneocuboid joint and that after it. However, the we used tendon pulls with constant loads to move the
excursion of the posterior tibial tendon was feet.
significantly decreased, compared with before the With our experimental model, we were able to
simulated arthrodeses, after all of the simulated quantify several important clinical observations. The
arthrodeses that involved the talonavicular joint degree to which simulated arthrodesis of a joint of the

Downloaded from www.ejbjs.org on October 23, 2006


triple joint complex affects motion of the remaining joint (a mean of 25 per cent remained after
joints is associated with the amount of motion that arthrodesis of the talonavicular joint, after double
the joint had before the arthrodesis. The arthrodesis, and after triple arthrodesis). These data
calcaneocuboid joint had the least motion before the suggest that excursion of the posterior tibial tendon is
simulated arthrodeses, and the simulated arthrodesis most related to mobility of the talonavicular joint and
of this joint had the least effect on the motion of the that any arthrodesis that preserves mobility of the
other joints. Conversely, the talonavicular joint had talonavicular joint, such as arthrodesis of the
the most motion before the simulated arthrodeses and calcaneocuboid or subtalar joint, also preserves
the simulated arthrodesis of this joint restricted excursion of the posterior tibial tendon. It follows
motion more than the simulated arthrodesis of any of that transfer of the flexor digitorum longus tendon to
the remaining joints did. Simulated arthrodeses that the navicular tuberosity, to restore the function of a
included fixation of the talonavicular joint ruptured or otherwise non-functional posterior tibial
(arthrodesis of the talonavicular joint or double tendon, will also maintain its excursion if the
arthrodesis) eliminated nearly all motion of the talonavicular joint remains mobile. Therefore, this
remaining joint or joints. After either simulated tendon transfer can be performed to restore the
arthrodesis of the talonavicular joint or simulated function of the posterior tibial tendon in conjunction
double arthrodesis, the motion of the subtalar joint with arthrodesis of the calcaneocuboid joint or,
was limited to about 2 degrees (about 9 per cent of possibly, arthrodesis of the subtalar joint, both of
the motion of the subtalar joint before the simulated which preserve motion of the talonavicular joint.
arthrodeses), which was the same as the motion of the In summary, this cadaveric study demonstrated that
subtalar joint after simulated arthrodesis of the motion of the talonavicular joint is the key to motion
subtalar joint. After simulated arthrodesis of the of the triple joint complex. Any simulated arthrodesis
subtalar joint, 26 per cent of the motion of the that includes fixation of the talonavicular joint
talonavicular joint (about 9 degrees) remained and 56 essentially eliminates the motion of the other joints in
per cent of the motion of the calcaneocuboid joint the complex and severely limits excursion of the
(about 7 degrees) remained. Simulated arthrodesis of posterior tibial tendon. After simulated arthrodesis of
the calcaneocuboid joint affected the motion of the the subtalar joint, some motion of the talonavicular

Redistribution of this article permitted only in accordance with the publisher’s copyright provisions.
remaining joints the least. We previously and calcaneocuboid joints is retained, as is a mean of
demonstrated that most of the motion of the subtalar 46 per cent of the excursion of the posterior tibial
and talonavicular joints was retained after simulated tendon. Simulated arthrodesis of the calcaneocuboid
arthrodesis of the calcaneocuboid joint with a bone joint preserves a good deal of the motion of the
block (lengthening of the lateral column)(1). talonavicular joint, most of the motion of the subtalar
The mean excursion of the posterior tibial tendon joint, and most of the excursion of the posterior tibial
before the simulated arthrodeses in the present study tendon. These results are consistent with clinical
is consistent with that determined by Hintermann et observations that the amount of motion remaining in
al. In their study, the excursion of all of the extrinsic these joints influences the patient's postoperative
muscles of the plantigrade foot were determined in function.
various degrees of plantar flexion and dorsiflexion
with varying degrees of inversion and eversion. References: (1-11)
Those authors demonstrated that the excursion of the
posterior tibial tendon was less affected by flexion of
the ankle from -20 to 30 degrees (mean, 4.4
millimeters of excursion; range, 1.8 to 9.3 millimeters 1. Deland, J. T.; Otis, J. C.; Lee, K.-T.; and
of excursion) than by inversion and eversion of the Kenneally, S. M.: Lateral column lengthening
foot. With 0 degrees of plantar flexion, they found the with calcaneocuboid fusion: range of motion
mean excursion of the posterior tibial tendon to be in the triple joint complex. Foot and Ankle
16.3 millimeters (range, 6.8 to 22.3 millimeters) from 1995; 16:729-733.
inversion to eversion. In the present study, the mean 2. Fogel, G. R.; Katoh, Y.; Rand, J. A.; and
excursion of the posterior tibial tendon before the Chao, E. Y. S.: Talonavicular arthrodesis for
arthrodeses was 17 ± 5 millimeters (range, ten to isolated arthrosis. 9.5-year results and gait
twenty-seven millimeters) with the ankle fixed in analysis. Foot and Ankle 1982; 3:105-113.
neutral. 3. Gellman, H.; Lenihan, M.; Halikis, N.; Botte,
As far as we know, the effect of various arthrodeses M. J.; Giordani, M.; and Perry, J.: Selective
on excursion of the posterior tibial tendon has not tarsal arthrodesis: an in vitro analysis of the
been studied previously. In the present study, the effect on foot motion. Foot and Ankle 1987;
amount of excursion remaining after a particular 8:127-133.
simulated arthrodesis was associated with the amount 4. Hall, M. C.; and Pennal, G. F.: Primary
of motion remaining in the triple joint complex. A subtalar arthrodesis in the treatment of severe
mean of 73 per cent of the excursion of the posterior fractures of the calcaneum. J. Bone and Joint
tibial tendon was retained after simulated arthrodesis Surg. 1960; 42-B(2):336-343.
of the calcaneocuboid joint, whereas a mean of 46 per 5. Hintermann, B.; Nigg, B. M.; and Sommer, C.:
cent was retained after simulated arthrodesis of the Foot movement and tendon excursion: an in
subtalar joint. Excursion was most restricted by vitro study. Foot and Ankle 1994; 15:386-
simulated arthrodeses that included the talonavicular 395.

Downloaded from www.ejbjs.org on October 23, 2006


6. Lapidus, P. W.: Subtalar joint, its anatomy
and mechanics. Bull. Hosp. Joint Dis. 1955;
16:179-195.
7. Logan, S. E.; Groszewski, P.; Krieg, J. C.; and
Vannier, M.: Upper extremity kinematics
assessment using four coupled six degree of
freedom sensors. Biomed. Sci. Instrum. 1988;
24:75-81.
8. Logan, S. E.; Vannier, M. W.; Bresina, S. J.;
and Weeks, P. M.: Wrist kinematic analysis
using a 6 degree of freedom digitizer. In
Proceedings of the Seventh Annual
Conference of the IEEE Engineering in
Medicine and Biology Society, pp. 13-16.
Piscataway, New Jersey, IEEE Service Center,
1985.
9. Mann, R. A.; and Baumgarten, M.: Subtalar
fusion for isolated subtalar disorders.
Preliminary report. Clin. Orthop. 1988;
226:260-265.
10. Polhemus Navigational Sciences Division:
Personal communication, 1992.
11. Sangeorzan, B. J.: Subtalar joint: morphology
and functional anatomy. In Inman's Joints of
the Ankle, edited by J. B. Stiehl. Ed. 2, p. 31.
Baltimore, Williams and Wilkins, 1991.

Redistribution of this article permitted only in accordance with the publisher’s copyright provisions.

Downloaded from www.ejbjs.org on October 23, 2006


Fig. 1 Illustration of the foot in the apparatus used to
test the range of motion of the subtalar, talonavicular,
and calcaneocuboid joints as well as excursion of the
posterior tibial tendon.

Redistribution of this article permitted only in accordance with the publisher’s copyright provisions.
Fig. 2 Graph of the mean range of motion (ROM)
(and standard deviation) of the subtalar joint before
the simulated arthrodeses (black bars) and after
simulated arthrodeses (gray bars) of the talonavicular
(TN) joint, the calcaneocuboid (CC) joint, and both
joints (double arthrodesis). The numbers below the
bars are the number of feet included for each
simulated arthrodesis.

Fig. 3 Graph of the mean range of motion (ROM)


(and standard deviation) of the talonavicular joint
before the simulated arthrodeses (black bars) and
after simulated arthrodeses (gray bars) of the subtalar
(ST) joint and the calcaneocuboid (CC) joint. The
numbers below the bars are the number of feet
included for each simulated arthrodesis.

Downloaded from www.ejbjs.org on October 23, 2006


Fig. 4 Graph of the mean range of motion (ROM)
(and standard deviation) of the calcaneocuboid joint
before the simulated arthrodeses (black bars) and
after simulated arthrodeses (gray bars) of the subtalar
(ST) joint and the talonavicular (TN) joint. The
numbers below the bars are the number of feet
included for each simulated arthrodesis.

Redistribution of this article permitted only in accordance with the publisher’s copyright provisions.
Fig. 5 Graph of the mean excursion (and standard
deviation) of the posterior tibial tendon before the
simulated arthrodeses (black bars) and after simulated
arthrodesis (gray bars) of the subtalar (ST) joint,
triple arthrodesis, double arthrodesis, arthrodesis of
the talonavicular (TN) joint, and arthrodesis of the
calcaneocuboid (CC) joint. The numbers below the
bars are the number of feet included for each
simulated arthrodesis.

Downloaded from www.ejbjs.org on October 23, 2006


CLINCIAL PRACTICE GUIDELINE

Diagnosis and Treatment of First


Metatarsophalangeal Joint Disorders.
Section 1: Hallux Valgus
Clinical Practice Guideline First Metatarsophalangeal Joint Disorders Panel:
John V. Vanore, DPM,1 Jeffrey C. Christensen, DPM,2 Steven R. Kravitz, DPM,3
John M. Schuberth, DPM,4 James L. Thomas, DPM,5 Lowell Scott Weil, DPM,6
Howard J. Zlotoff, DPM,7 and Susan D. Couture8

T his clinical practice guideline (CPG) is based upon Diagnosis and Treatment of First MTP Joint
Disorders (Pathway 1)
consensus of current clinical practice and review of the
clinical literature. The guideline was developed by the Clin-
ical Practice Guideline First Metatarsophalangeal (MTP) Pathology of the first MTP joint encompasses a variety
Joint Disorders Panel of the American College of Foot and of disorders from acquired orthopedic deformities and
Ankle Surgeons. The guideline and references annotate traumatic injuries to overuse problems and systemic dis-
each node of the corresponding pathways.
orders. These clinical problems typically involve the first
MTP joint and are encountered daily by the foot and
1
Chair, Gadsden, AL; 2 Everett, WA; 3 Richboro, PA; 4 San Francisco, ankle surgeon. This document will review current prac-
CA; 5 Board Liaison, Birmingham, AL; 6 Des Plaines, IL; 7 Camp Hill, tice guidelines for the diagnosis and treatment of: hallux
PA; and 8 Park Ridge, IL. Address correspondence to: John V. Vanore,
DPM, Gadsden Foot Clinic, 306 South 4th St, Gadsden, AL 35901; e-mail: valgus (Pathway 2), hallux rigidus (Pathway 3), hallux
jvanore@prodigy.net varus (Pathway 4), sesamoid disorders (Pathway 5),
Copyright © 2003 by the American College of Foot and Ankle Surgeons
1067-2516/03/4203-0002$30.00/0
trauma (Pathway 6), and other disorders of the first MTP
doi:10.1053/jfas.2003.50036 joint (Pathway 7).

PATHWAY 1

112 THE JOURNAL OF FOOT & ANKLE SURGERY


PATHWAY 2
Hallux Valgus (Pathway 2) may also occur with a lesser degree of transverse plane
deformity with limitation of joint movement and degenera-
Hallux valgus deformity is one of the most common foot tive changes. This is referred to as hallux valgus rigidus.
problems seen today by foot and ankle specialists. Hallux
valgus is a deformity at the first MTP joint with abduction
and valgus rotation of the great toe combined with a medi- Associated Findings (Node 3)
ally prominent first metatarsal head. A complete spectrum
of pathology and symptoms is seen with this condition. A Hallux valgus can present with numerous associated find-
vast array of surgical procedures has been described for the ings (8) that are part of a syndrome of forefoot derangement
symptomatic patient. The treatment may be more challeng- and may include:
ing than one may initially expect, because the deformity
● Hammertoe deformity of the second toe—Lateral devia-
may range from benign and asymptomatic to severe and
tion of the great toe may cause instability of the second
disabling.
toe and the second MTP joint. The second toe may
overlap (dislocation syndrome) or under lap the hallux, or
the toes may abut each other. Interdigital lesions are not
Significant History (Node 1) uncommon.
● Plantar callus—Hyperkeratotic lesions may vary from a
Patients presenting with this deformity often have a diffuse callus subsecond, -third, and -fourth metatarsal
significant family history of bunion deformity (1,2). The heads to a localized or nucleated lesion usually under the
deformity may be noted in adolescence, although it is second metatarsal head. Formation of these lesions is
more prevalent in women older than 30 years (2– 6). believed to be caused by abnormal metatarsal pressures
Symptoms may occur early or they may not manifest during gait (20,21).
until later in life. Pain is generally associated with irri- ● Central metatarsalgia—Generally, bunion formation is
tation at the medial subcutaneous bunion, although even associated with first ray insufficiency, leading to lateral
significant deformities may be asymptomatic. The con- weight transfer and symptoms that may include metatar-
dition may be aggravated by short or tight footwear, salgia or a localized capsulitis of the second MTP joint
particularly with regard to women’s shoe gear. Hallux (22–25).
valgus is considered to be a progressive condition and ● Pronated foot type—Pronation may be responsible for the
patients present with varied degrees of deformity from weak foot characteristics, leading to forefoot deformities
mild enlargement of the metatarsal head to severe dislo- such as bunions and hammertoes (10,11).
cation of the first MTP joint. ● Ankle equinus—Ankle or gastrosoleus equinus is de-
fined clinically as a limitation of ankle dorsiflexion.
Significant Findings (Node 2) Ankle equinus is a detrimental force leading to prona-
tion of the foot and development of foot deformities
Hallux valgus is readily apparent with clinical inspection (26).
of the patient. Significant findings may include a subcuta- ● Ingrown toenail—Ingrown toenails may occur on either
neous bony prominence or medial bump. The great toe is nail border and are caused by the valgus and abducted
abducted or deviated laterally, often with a degree of axial position of the great toe or from abutment with the second
or valgus rotation (7,8). Shoe irritation may result in a toe (11).
painful medial bursitis with inflammation surrounding the
first MTP joint or neuritis of the adjacent medial dorsal Radiographic Findings (Node 4)
cutaneous nerve (6). Patients note widening of the forefoot;
this contributes to difficulty wearing shoes comfortably. Radiographic evaluation should include assessment of
First MTP joint range of motion should occur completely general radiographic parameters and angular relationships
within the sagittal plane, but with hallux valgus, the motion of the osseous segments involved in this deformity (2,5,13,
may be in an oblique manner with abduction and eversion 27–29).
during dorsiflexion (9 –11). Adaptation occurs at the meta- These parameters allow assessment of the severity of
tarsal articular surface with lateral deviation, and joint mo- deformity and provide a basis for surgical procedural selec-
tion may become track-bound laterally (12–14). An assess- tion (Node 9). Radiographs should be weightbearing views
ment of the patient standing and limited gait analysis is of the feet and taken with the patient standing in the angle
undertaken. First ray hypermobility secondary to rear-foot and base of gait (30).
pronation is generally considered responsible for elevation
of the intermetatarsal (IM) angle (15–19) Hallux valgus ● Medial prominence of the first metatarsal head—The

114 THE JOURNAL OF FOOT & ANKLE SURGERY


with increases in the IM angle. The relationship of the
tibial sesamoid to the first metatarsal bisection is deter-
mined (7,35).
● Axial rotation of the hallux—Valgus rotation of the great
toe occurs with advanced hallux valgus deformity. This is
a qualitative determination and is assessed by the sym-
metry of the proximal phalanx on the dorsoplantar radio-
graph (13).
● Hallux abductus interphalangeus angle—This angle is
defined by the longitudinal bisections of the hallucal
proximal and distal phalanges. Normal values are consid-
ered less than 10° (28).
● Degenerative joint disease—With long-standing or severe
hallux valgus, degenerative joint disease may develop.

Initial Treatment Options (Node 5)

When symptoms begin to interfere with a patient’s life-


FIGURE 1 The HA angle is formed by the angle between the style, initial treatment (eg, wider, lower-heeled shoes; bun-
longitudinal axis of the hallux and first metatarsal; it is a reliable
ion pads; ice; and over-the-counter analgesics) is often
indicator of the severity of a bunion or a hallux valgus deformity.
self-directed. Patients who are unresponsive to the initial
treatment or unable to fulfill the self-directed regimen
should be directed to a podiatric foot and ankle surgeon for
medial prominence may be caused by the increased IM
evaluation.
angle and/or hypertrophy of the medial epicondyle. The
Nonsurgical care involves patient education, including a
metatarsal head may square off or present with erosions
discussion of the natural history of the disorder, evaluation
(2,7,31).
of footwear, and prior treatment. Prescription anti-inflam-
● Joint space—The joint space may be normal or show
matory nonsteroidal drugs may be indicated for symptom-
narrowing with or without degenerative changes. End-
atic arthralgias or bursitis. Nonsurgical alternatives include
stage deformities may show dislocation of the joint (13).
shoe modifications, with pocketing of the medial shoe con-
● Hallux abductus (HA) angle—This angle is defined by
tour or wider causal shoes. Although there is no scientific
the longitudinal bisections of the hallucal proximal pha-
lanx and the first metatarsal, and is measured on the
dorsoplantar radiograph (Fig. 1). Values greater than 20°
are often pathologic and correlate with severity of defor-
mity (32).
● Proximal articular set angle—Lateral adaptation of the
distal first metatarsal articular surface occurs with long-
standing HA, and this can be assessed radiographically.
This angular value is described by a perpendicular line
drawn to the longitudinal bisection of the first metatarsal
at a point intersecting a line drawn to the functional
articular surface of the metatarsal head. This has also
been described as the distal metatarsal articular angle.
Normal values are 0° to 8°, but interobserver variability
exists and the value may not correlate with intraoperative
observations (29,33).
● IM angle—The IM angle, formed by the longitudinal
bisections of the first and second metatarsals, increases
with severity of deformity (Fig. 2). Abnormal values are
greater than 10°. Metatarsus adductus may alter the sig-
nificance of this measurement (5,34). FIGURE 2 The IM angle is formed by the longitudinal axis of the
● Lateral displacement of the sesamoids—Displacement of first and second metatarsals, respectively; this angle is also gener-
the sesamoids occurs with hallux valgus and correlates ally elevated in more severe deformities.

VOLUME 42, NUMBER 3, MAY/JUNE 2003 115


TABLE 1 Relative corrective ability of hallux valgus procedures

Surgical Procedure IM Type of Correction Able to Be Achieved


Sagittal Linear PASA Frontal Postoperative
Plane Metatarsal Plane Care
Deformity Deformity Deformity
Capsule tendon balance ⫹ E E E ⫹a WB
Green-Reverdin ⫹ E E ⫹⫹⫹⫹ E WB
Hohmann ⫹⫹ ⫹ ⫹⫹ ⫹⫹⫹⫹ ⫹⫹ PWB
Chevron ⫹⫹ ⫹ ⫹⫹ ⫹⫹⫹⫹ E WB
Long arm V ⫹⫹⫹ ⫹ ⫹⫹ ⫹⫹ E PWB/WB
SCARF ⫹⫹⫹ ⫹⫹ ⫹⫹⫹ ⫹⫹ E WB
CAWO (intact hinge) ⫹⫹⫹⫹ ⫹ ⫹ E E NWB
Crescentic ⫹⫹⫹⫹ ⫹ ⫹ E E PWB/NWB
Proximal chevron ⫹⫹⫹ ⫹ ⫹ E E PWB/NWB
Lapidus ⫹⫹⫹⫹ ⫹⫹⫹⫹ ⫹⫹⫹⫹ E ⫹ NWB
Keller ⫹⫹ E E E ⫹⫹a WB
MTP arthrodesis ⫹⫹⫹ E ⫹⫹⫹ E ⫹⫹⫹a PWB

NOTE. E, no corrective value/no effect; ⫹ to ⫹⫹⫹⫹, relative value scale indicative of potential to correct type of deformity.
Abbreviations: CAWO, closing abductory wedge osteotomy; NWB, nonweightbearing; PASA, proximal articular set angle; PWB, partial
weightbearing; WB, weightbearing.
a
Frontal plane correction at first MTP joint.

evidence to support the efficacy of orthotic devices in the Surgical Treatment: Stage 1 (Node 10)
treatment of hallux valgus, symptomatic relief may be re-
alized by some patients (36,37). Stage 1 hallux valgus deformity is defined as an IM angle
Surgical recommendations might be considered on the ⬍12° and an HA angle ⬍25°. Although the appearance of
initial evaluation of hallux valgus deformity. Because hal- the deformity may not be significant, there is often deviation
lux valgus is a progressive disorder and is often evaluated in of the joint and medial enlargement of the first metatarsal
the second or third stage, surgical consideration can be head. Typically, soft tissue tendon balance (38 – 45) and
undertaken early in the course of treatment. exostectomy with/without a distal osteotomy are performed
to correct the deformity (46 –77). If hallux abductus inter-
phalangeus is present, a phalangeal osteotomy may be in-
Clinical Response (Nodes 6, 7, and 8) dicated (28,78 –91) (Table 1; Figs. 3 and 4).

When nonsurgical care is rendered, the clinical response


is assessed (Node 6). If the patient is doing well, initial Surgical Treatment: Stage 2 (Node 11)
treatment may be continued (Node 7). If there has been little
or no improvement or if initial improvement deteriorates, Stage 2 deformities are more significant and have an IM
surgical treatment is appropriate. If a primary care physician angle ⱕ16° with an HA angle of ⱖ25°. The joint congru-
performed the initial evaluation and treatment, referral to a ency must be evaluated. Capsule-tendon balancing is per-
podiatric foot and ankle surgeon is indicated (Node 8). formed with or without osteotomy of the first metatarsal
and/or proximal phalanx (92–99). When hypermobility of
the first ray is encountered or is in the presence of severe
Assessment of Deformity and Arthrosis (Node 9) deformity, a metatarsal cuneiform arthrodesis may be con-
sidered (17,25,34,100 –106). If hallux abductus interphalan-
Hallux valgus deformity may be classified into stages 1, geus is present, a phalangeal osteotomy may be indicated
2, and 3 (Nodes 10, 11, and 12). These stages are based on (Table 1; Figs. 5 and 6).
the progression and degree of deformity of HA and the IM
angle.
In each stage, surgical intervention includes a capsule- Surgical Treatment: Stage 3 (Node 12)
tendon balancing procedure (which may include medial
exostectomy), a lateral release, and a medial capsulorrha- Stage 3 deformities are considered severe and generally
phy. more disabling. These deformities have an IM angle that is

116 THE JOURNAL OF FOOT & ANKLE SURGERY


FIGURE 3 Distal metatarsal
osteotomy is a common pro-
cedural alternative and may
be performed with a variety of
techniques including the Aus-
tin or chevron osteotomy. (A)
Preoperative anteroposterior
(AP) radiograph, (B) postoper-
ative radiograph.

FIGURE 4 Phalangeal os-


teotomy has been performed
as variations of the Akin os-
teotomy. (A) Preoperative and
(B) postoperative oblique Akin
with Kirschner wire fixation.

VOLUME 42, NUMBER 3, MAY/JUNE 2003 117


FIGURE 5 Proximal first
metatarsal osteotomy is se-
lected for cases of significant
elevation of the first inter-
metarsal angle. (A) Preopera-
tive AP radiograph and (B)
postoperative AP radiograph.

FIGURE 6 The Lapidus or


first metatarsal cuneiform joint
fusion is useful in cases of hal-
lux valgus with large IM angles
or first ray hypermobility. (A)
Preoperative AP radiograph
and (B) postoperative AP ra-
diograph.

118 THE JOURNAL OF FOOT & ANKLE SURGERY


FIGURE 7 The Keller bun-
ionectomy with resection of
the proximal phalangeal base
still is a useful procedure for
hallux valgus deformities. (A)
Preoperative and (B) postop-
erative AP views.

FIGURE 8 First MTP joint fu-


sion is not only a useful sal-
vage technique for recurrent
(A) preoperative, (B) postoper-
ative, or arthritic deformities
but also useful as a primary
procedure for the correction of
hallux valgus.

VOLUME 42, NUMBER 3, MAY/JUNE 2003 119


FIGURE 9 Reconstruction of the rheumatoid forefoot may be described as a forefoot arthroplasty combining (A) resection arthroplasty
(Keller), (B) first MTP joint implant, or (C) first MTP joint fusion with lesser metatarsal procedures.

usually ⬎16° and an HA angle ⱖ35°. The MTP joint may References
be deviated or subluxed. Severe deformities often present
with associated findings (Node 3) in addition to hallux 1. Coughlin MJ, Roger A. Mann Award. Juvenile hallux valgus: etiol-
valgus. ogy and treatment. Foot Ankle Int 16:682– 697, 1995.
2. Hardy R, Clapham JCR. Observations on hallux valgus. J Bone Joint
Deformities in this stage may be corrected through cap-
Surg 33B:376 –391, 1951.
sule-tendon balancing with an osteotomy of the first meta- 3. Gould N, Schneider W, Ashikaga T. Epidemiological survey of foot
tarsal and/or proximal phalanx. Double osteotomy of the problems in the continental United States: 1978-1979. Foot Ankle
first metatarsal provides an additional option (56,107–113). 1:8 –10, 1980.
Determination of location of the osteotomy is influenced by 4. Kilmartin TE, Barrington RL, Wallace WA. Metatarsus primus varus.
A statistical study. J Bone Joint Surg 73B:937–940, 1991.
the degree of deformity and/or the presence of associated 5. Piggott H. The natural history of hallux valgus in adolescence and
degenerative arthritis. Metatarsal cuneiform arthrodesis early adult life. J Bone Joint Surg 42B:749 –760, 1960.
may also be considered. 6. Rosen JS, Grady JF. Neuritic bunion syndrome. J Am Podiatr Med
If hallux abductus interphalangeus is present, a phalan- Assoc 76:641– 644, 1986.
geal osteotomy may be indicated. In certain situations, first 7. Haines RW, McDougall A. The anatomy of hallux valgus. J Bone
Joint Surg 36B:272–293, 1954.
MTP joint resection arthroplasty (114 –120), with or with- 8. Kelikian H. Hallux Valgus, Allied Deformities of the Forefoot and
out a joint implant (121–132), or arthrodesis (133–139) may Metatarsalgia. WB Saunders, Philadelphia, 1965.
be performed, as in the case of patients with rheumatoid 9. Eustace S, Byrne JO, Beausang O, Codd M, Stack J, Stephens MM.
arthritis or degenerative joint disease or in patients requiring Hallux valgus, first metatarsal pronation and collapse of the medial
revision surgery (123–125,140 –157) (Table 1; Figs. 7–9). longitudinal arch—a radiological correlation. Skeletal Radiol 23:
191–194, 1994.
In summary, hallux valgus deformity is an inherited, 10. Shaw AH. The biomechanics of hallux valgus in pronated feet. J Am
progressive deformity often associated with certain foot Podiatry Assoc 64:193–201, 1974.
types, with symptoms aggravated by shoe wear. Although 11. Root ML, Orien WP, Weed H. Forefoot deformity caused by abnor-
conservative measures may be used initially to reduce the mal subtalar joint pronation. In Normal and Abnormal Function of
symptomatology associated with this deformity, surgical the Foot, pp 349 – 462, Clinical Biomechanics, Los Angeles, 1977.
12. Coughlin MJ. Hallux valgus in men: effect of the distal metatarsal
repair is often necessary to correct the hallux valgus and its articular angle on hallux valgus correction. Foot Ankle Int 18:463–
associated deformities. 470, 1997.

120 THE JOURNAL OF FOOT & ANKLE SURGERY


13. Laporta G, Melillo T, Olinsky D. X-ray evaluation of hallux abducto 38. McBride ED. The conservative operation for “bunions.” End results
valgus deformity. J Am Podiatry Assoc 64:544 –566, 1974. and refinements of technique. J Am Med Assoc 105:1164 –1168.,
14. Evans RD, Lile LN. Proximal articular set angle. Radiographic versus 1935.
intraoperative measurement. J Am Podiatr Med Assoc 90:199 –202, 39. McBride ED. The McBride bunion hallux valgus operation. J Bone
2000. Joint Surg 49A:1675–1683, 1967.
15. Prieskorn DW, Mann RA, Fritz G. Radiographic assessment of the 40. McGlamry ED, Feldman MH. A treatise on the McBride procedure.
second metatarsal: measure of first ray hypermobility. Foot Ankle Int A review of the McBride publications on hallus valgus correction
17:331–333, 1996. with observations on rationale of the original procedure and the
16. Rush SM, Christensen JC, Johnson CH. Biomechanics of the first ray. current modifications. J Am Podiatry Assoc 61:161–173, 1971.
Part II: metatarsus primus varus as a cause of hypermobility. A 41. Pfeffinger LL. The modified McBride procedure. Orthopedics 13:
three-dimensional kinematic analysis in a cadaver model. J Foot 979 –984, 1990.
Ankle Surg 39:68 –77, 2000. 42. Lawton J, Evans R. Modified McBride bunionectomy. J Am Podiatry
17. Faber FW, Kleinrensink GJ, Verhoog MW, Vijn AH, Snijders CJ, Assoc 65:670 – 688, 1975.
Mulder PG, Verhaar JA. Mobility of the first tarsometatarsal joint in 43. Kempe SA, Singer RH. The modified McBride bunionectomy utiliz-
relation to hallux valgus deformity: anatomical and biomechanical ing the adductor tendon transfer. J Foot Surg 24:24 –29, 1985.
aspects. Foot Ankle Int 20:651– 656, 1999. 44. Schwitalle M, Karbowski A, Eckardt A. Hallux valgus in young
18. Carl A, Ross S, Evanski P, Waugh T. Hypermobility in hallux valgus. patients: comparison of soft-tissue realignment and metatarsal osteot-
Foot Ankle 8:264 –270, 1988. omy. Eur J Pediatr Surg 8:42– 46, 1998.
19. Inman VT. Hallux valgus: a review of etiologic factors. Orthop Clin 45. Mann RA, Pfeffinger L. Hallux valgus repair. DuVries modified
North Am 5:59 – 66, 1974. McBride procedure. Clin Orthop 272:213–218, 1991
20. Gibbs RC, Boxer MC. Abnormal biomechanics of feet and their 46. Shapiro F, Heller L. The Mitchell distal metatarsal osteotomy in the
cause of hyperkeratoses. J Am Acad Dermatol 6:1061–1069, 1982. treatment of hallux valgus. Clin Orthop 107:225–231, 1975.
21. Eulry F. Static metatarsalgia. Rev Prat 47:37– 42, 1997. 47. Dovey H. The treatment of hallux valgus by distal osteotomy of the
22. Viladot A. Metatarsalgia due to biomechanical alterations of the first metatarsal. Acta Orthop Scand 40:402– 408, 1969.
forefoot. Orthop Clin North Am 4:165–178, 1973. 48. Funk FJ, Wells RE. Bunionectomy—with distal osteotomy. Clin
23. Roy KJ. Force, pressure, and motion measurements in the foot: Orthop 85:71–74, 1972.
current concepts. Clin Podiatr Med Surg 5:491–508, 1988. 49. Grill F, Hetherington V, Steinbock G, Altenhuber J. Experiences with
24. Raymakers R, Waugh W. The treatment of metatarsalgia with hallux the chevron (V-)osteotomy on adolescent hallux valgus. Arch Orthop
valgus. J Bone Joint Surg 53B:684 – 687, 1971. Trauma Surg 106:47–51, 1986.
25. Hansen ST. Hallux valgus surgery. Morton and Lapidus were right! 50. Hyytinen T, Lantto E, Kallio P, Salo S, Kaukonen JP. Modified distal
Clin Podiatr Med Surg 13:347–354, 1996. metatarsal osteotomy for hallux valgus. Ann Chir Gynaecol 84:81–
26. Yu GV, Johng B, Freireich R. Surgical management of metatarsus 84, 1995.
adductus deformity. Clin Podiatr Med Surg 4:207–232, 1987. 51. Johnson JB, Smith SD. Preliminary report on derotational, angula-
27. Palladino SJ. Preoperative evaluation of the bunion patient: etiology, tional, transpositional osteotomy: a new approach to hallux abducto
biomechanics, clinical and radiographic assessment. In Textbook of valgus surgery. J Am Podiatry Assoc 64:667– 675, 1974.
Bunion Surgery, pp 1– 87, edited by J Gerbert, Futura Publishing, 52. Auerbach AM. Review of distal metatarsal osteotomies for hallux
Mount Kisco, NY, 1991. valgus in the young. Clin Orthop 70:148 –151, 1970.
28. Sorto LA, Balding MG, Weil LS, Smith SD. Hallux abductus inter- 53. Austin DW, Leventen EO. A new osteotomy for hallux valgus: a
phalangeus: etiology, x-ray evaluation and treatment. J Am Podiatry horizontally directed “V” displacement osteotomy of the metatarsal
Assoc 66:384 –396, 1976. head for hallux valgus and primus varus. Clin Orthop 157:25–30,
29. Whitney AK. Radiographic Charting Tecnhique, PCPM Press, Phil- 1981.
adelphia, 1978. 54. Chou LB, Mann RA, Casillas MM. Biplanar chevron osteotomy. Foot
30. Spinner SM, Lipsman S, Spector F. Radiographic criteria in the Ankle Int 19:579 –584, 1998.
assessment of hallux abductus deformities. J Foot Surg 23:25–30, 55. Christensen PH, Hansen TB. Hallux valgus correction using a mod-
1984. ified Hohmann technique. Foot Ankle Int 16:177–180, 1995.
31. Mann RA, Coughlin MJ. Hallux valgus— etiology, anatomy, treat- 56. Coughlin MJ, Carlson RE. Treatment of hallux valgus with an
ment and surgical considerations. Clin Orthop 157:31– 41, 1981. increased distal metatarsal articular angle: evaluation of double and
32. Root ML, Orien WP, Weed JH. Motion of the joints of the foot. In triple first ray osteotomies. Foot Ankle Int 20:762–770, 1999.
Normal and Abnormal Function of the Foot, pp 1– 63, edited by 57. Mitchell CL, Fleming JL, Allen R, Glenney C, Sanford, GA. Osteot-
Orien WP, Root ML, Weed JH, Clinical Biomechanics, Los Angeles, omy-bunionectomy for hallux valgus. J Bone Joint Surg 40A:41– 60,
1977. 1958.
33. Amarnek DL, Mollica A, Jacobs AM, Oloff LM. A statistical analysis 58. Peabody CW. Surgical cure of hallux valgus. J Bone Joint Surg
on the reliability of the proximal articular set angle. J Foot Surg 13:273–282, 1931.
25:39 – 43, 1986. 59. Warrick JP, Edelman R. The Hohmann bunionectomy utilizing A-O
34. Lapidus PW. Operative correction of the metatarsus varus primus in screw fixation: a preliminary report. J Foot Surg 23:268 –274, 1984.
hallux valgus. Surg Gynecol Obstet 58:183–191, 1934. 60. Wilson JN. Oblique displacement osteotomy for hallux valgus.
35. Judge MS, LaPointe S, Yu GV, Shook JE, Taylor RP. The effect of J Bone Joint Surg 45B:552–555, 1963.
hallux abducto valgus surgery on the sesamoid apparatus position. 61. Laird PO, Silvers SH, Somdahl J. Two Reverdin-Laird osteotomy
J Am Podiatr Med Assoc 89:551–559, 1999. modifications for correction of hallux abducto valgus. J Am Podiatr
36. Kilmartin TE, Barrington RL, Wallace WA. A controlled prospective Med Assoc 78:403– 405, 1988.
trial of a foot orthosis for juvenile hallux valgus. J Bone Joint Surg 62. Beck EL. Modified Reverdin technique for hallux abducto valgus
76B:210 –214, 1994. (with increased proximal articular set angle of the first metatarsopha-
37. Moraros J, Hodge W. Orthotic survey. Preliminary results. J Am langeal joint). J Am Podiatry Assoc 64:657– 666, 1974.
Podiatr Med Assoc 83:139 –148, 1993. 63. Kalish SR, Spector JE. The Kalish osteotomy. A review and retro-

VOLUME 42, NUMBER 3, MAY/JUNE 2003 121


spective analysis of 265 cases. J Am Podiatr Med Assoc 84:237–242, 91. Meyer HR, Muller G. Regnauld procedure for hallux valgus. Foot
1994. Ankle 10:299 –302, 1990.
64. Barker AE. An operation for hallux valgus. Lancet 1:655, 1884. 92. Lynch FR. Applications of the opening wedge cuneiform osteotomy
65. Hohmann G. Der hallux valgus und die uebrigen Zehenverkruem- in the surgical repair of juvenile hallux abducto valgus. J Foot Ankle
mungen. Ergeb Chir Orthop 18:308 –348, 1925. Surg 34:103–123, 1995.
66. Hohmann G. Fuss und Bein, pp 145–192, JF Bergmann, Munich, 93. Ludloff K. Die besetigung des hallux valgus durch die schraege
1951. planto-dorsale osteotomie des metatarus 1. Arch Klin Chir 110:364 –
67. Pollack RA, Bellacosa RA, Higgins KR, Sharp BE, McCloskey TF. 387, 1918.
Critical evaluation of the short “Z” bunionectomy. J Foot Surg 94. Weil LS. Scarf osteotomy for correction of hallux valgus. Historical
28:158 –161, 1989. perspective, surgical technique, and results. Foot Ankle Clin 5:559 –
68. Quinn MR, DiStazio JJ, Kruljac SJ. Herbert bone screw fixation of 580, 2000.
the Austin bunionectomy. J Foot Surg 26:516 –519, 1987. 95. Gudas CJ. Compression screw fixation in proximal first metatarsal
69. Reverdin J. Anatomie et opération de l’hallux valgus. Internat Med osteotomies for metatarsus primus varus: initial observations. J Foot
Congr 2:408 – 412, 1881. Surg 18:10 –15, 1979.
70. Reverdin J. De la déviation en dehors du gros orteil (hallux valgus, 96. Schuberth JM, Reilly CH, Gudas CJ. The closing wedge osteotomy:
vulg. “oignon,” “bunions,” “Ballen”) et de son traitement chirurgical. a critical analysis of first metatarsal elevation. J Am Podiatry Assoc
Tr Internat Med Congr 2:408 – 412, 1881. 74:13–24, 1984.
71. Bass SJ, Gastwirth CM, Green R, Weinstock RE. A modification of 97. Mau C, Lauber HT. Die operative Behalung des hallux valgus (Na-
the Austin bunionectomy. (The tongue and groove osteotomy bun- chuntersuchungen). Dtsch Ztschr Chir 197:361–377, 1926.
ionectomy). J Am Podiatry Assoc 69:608 – 615, 1979. 98. Amarnek DL, Juda EJ, Oloff LM, Jacobs AM. Opening base wedge
72. Boc SF, D’Angelantonio A, Grant S. The triplane Austin bunionec- osteotomy of the first metatarsal utilizing rigid external fixation. J
tomy: a review and retrospective analysis. J Foot Surg 30:375–382, Foot Surg 25:321–326, 1986.
1991. 99. Zettl R, Trnka HJ, Easley M, Salzer M, Ritschl, P. Moderate to severe
73. Wu KK. Modified Mitchell’s bunionectomy (Wu’s bunionectomy). hallux valgus deformity: correction with proximal crescentic osteot-
Orthopedics 20:253–257, 1997. omy and distal soft-tissue release. Arch Orthop Trauma Surg 120:
74. Keogh P, Jaishanker JS, O’Connell RJ, White M. The modified 397– 402, 2000.
Wilson osteotomy for hallux valgus. Clin Orthop 255:263–267, 1990.
100. Lapidus PW. Dorsal bunion: its mechanics and operative correction.
75. Winston L, Wilson RC. A modification of the Hohmann procedure
J Bone Joint Surg 22:627– 637, 1940.
for surgical correction of hallux abducto valgus. J Am Podiatry Assoc
101. Lapidus PW. The author’s bunion operation from 1931 to 1959. Clin
72:11–14, 1982.
Orthop 16:119 –135, 1960.
76. Vega MR, Jackson-Smith JL. A variable first metatarsal distal “L”
102. Rutherford RL. The Lapidus procedure for primus metatarsus adduc-
osteotomy with adductor tendon transfer. J Foot Ankle Surg 34:384 –
tus. J Am Podiatry Assoc 64:581–584, 1974.
388, 1995.
103. Catanzariti AR, Mendicino RW, Lee MS, Gallina MR. The modified
77. Miller S, Croce WA. The Austin procedure for surgical correction of
Lapidus arthrodesis: a retrospective analysis. J Foot Ankle Surg
hallux abducto valgus deformity. J Am Podiatry Assoc 69:110 –118,
38:322–332, 1999.
1979.
104. Frankel JP. The misuse of the Lapidus procedure. J Foot Ankle Surg
78. Barouk LS. Osteotomies of the great toe. J Foot Surg 31:388 –399,
36:71–72, 1997.
1992.
105. Grace D, Delmonte R, Catanzariti AR, Hofbauer M. Modified Lapi-
79. Boberg JS, Menn JJ, Brown WL. The distal akin osteotomy: a new
approach. J Foot Surg 30:431436, 1991. dus arthrodesis for adolescent hallux abducto valgus. J Foot Ankle
80. Bordelon RL. Technique and use of Akin osteotomy. Contemp Or- Surg 38:8 –13, 1999.
thop 23:38 – 41, 1991. 106. Sangeorzan BJ, Hansen ST. Modified Lapidus procedure for hallux
81. Frey C, Jahss M, Kummer FJ. The Akin procedure: an analysis of valgus. Foot Ankle 9:262–266, 1989.
results. Foot Ankle 12:1– 6, 1991. 107. Lidge RT. Hallux valgus—surgical correction by three-in-one tech-
82. Donahue WE. The proximal phalangeal osteotomy. A technically nique. In Foot Science, pp 188 –210, edited by JE Bateman, WB
advanced approach. Clin Podiatry 2:449 – 455, 1985. Saunders, Philadephia, 1976.
83. Gerbert J, Melillo T. A modified Akin procedure for the correction of 108. Evins RJ. Alternate method of reducing metatarsus primus adductus
hallux valgus. J Am Podiatry Assoc 61:132–136, 1971. angle. J Foot Surg 16:9 –11, 1977.
84. Gohil P, Cavolo DJ. A simplified preoperative evaluation for Akin 109. Mitchell LA, Baxter DE. A Chevron-Akin double osteotomy for
osteotomy. J Am Podiatry Assoc 72:44 – 45, 1982. correction of hallux valgus. Foot Ankle 12:7–14, 1991.
85. Green AH, Bosta SD. Akin osteotomy of the hallux proximal phalanx 110. Osterwalder A, Maestretti G. Metatarsus primus double osteotomy, a
utilizing Richards mini staple fixation. J Foot Surg 25:386 –389, logical and non-disabling surgical technique for treatment of hallux
1986. valgus. Helv Chir Acta 60:245–248, 1993.
86. Kinnard P, Cantin S. The Akin procedure in hallux valgus. Can 111. Peterson HA, Newman SR. Adolescent bunion deformity treated with
J Surg 34:491– 493, 1991. double osteotomy and longitudinal pin fixation of the first ray. J Pe-
87. Levitsky DR. Percutaneous osteoclasp fixation of Akin osteotomy: an diatr Orthop 13:80 – 84, 1993.
alternative fixation technique. J Foot Surg 20:163–166, 1981. 112. Tollison ME, Baxter DE. Combination chevron plus Akin osteotomy
88. Levitsky DR, DiGilio J, Kander R, Rubin B. Rigid compression for hallux valgus: should age be a limiting factor? Foot Ankle Int
screw fixation of first proximal phalanx osteotomy for hallux abducto 18:477– 481, 1997.
valgus. J Foot Surg 21:65– 69, 1982. 113. Wattling WO, Cox KL. Double osteotomies for structural correction
89. Schwartz NH, Iannuzzi PJ, Thurber NB. Derotational akin osteot- of hallux abducto valgus. J Foot Surg 15:61– 64, 1976.
omy. J Foot Surg 25:479 – 483, 1986. 114. Ganley JV, Lynch FR, Darrigan RD. Keller bunionectomy with
90. Akin OF. The treatment of hallux valgus—an operative procedure fascia and tendon graft. J Am Podiatr Med Assoc 76:602– 610, 1986.
and its results. Med Sentinel 33:678 – 679, 1925. 115. McGlamry ED, Kitting RW, Butlin WE. Keller bunionectomy and

122 THE JOURNAL OF FOOT & ANKLE SURGERY


hallux valgus correction. An appraisal and current modifications skeletal Disorders of the Lower Extremities, pp 545–558, edited by
sixty-six years later. J Am Podiatry Assoc 60:161–167, 1970. LM Oloff, WB Saunders, Philadelphia, 1994.
116. Keller WL. Further observations of the surgical treatment of hallux 138. Kampner SL. Total joint replacement in bunion surgery. Orthopedics
valgus and bunions. N Y Med J 95:696 – 698, 1912. 1:275–284, 1978.
117. Keller WL. The surgical treatment of bunions and hallux valgus. N Y 139. Kampner SL. Total joint prosthetic arthroplasty of the great toe: a
Med J 80:741–742, 1904. 12-year experience. Foot Ankle 4:249 –261, 1984.
118. Rix RR. Modified Mayo operation for hallux valgus and bunion—a 140. Andre S, Champetier de Ribes B, Sauzieres P, Tomeno B. Swanson
comparison with the Keller procedure. J Bone Joint Surg 50A:1368 – silastic implant arthroplasty in pathology of the metatarsophalangeal
1378, 1968. joint of the great toe. Rev Chir Orthop Reparatrice Appar Mot
119. Vitek M, Steinbock G. Value of cerclage fibreux for the Keller- 71:95–100, 1985.
Brandes procedure. Arch Orthop Trauma Surg 108:104 –106, 1989. 141. Baskwill D, Kanat IO. Surgical considerations in hallux abducto
120. Cosentino GL. The Cosentino modification for tendon interpositional valgus with rheumatoid arthritis. J Foot Surg 26:429 – 433, 1987.
arthroplasty. J Foot Ankle Surg 34:501–508, 1995. 142. Beauchamp CG, Kirby T, Rudge SR, Worthington BS, Nelson, J.
121. LaPorta GA, Pilla P, Richter KP. Keller implant procedure: a report Fusion of the first metatarsophalangeal joint in forefoot arthroplasty.
of 536 procedures using a Silastic intramedullary stemmed implant. Clin Orthop 190:249 –253, 1984.
J Am Podiatry Assoc 66:126 –147, 1976. 143. Benson GM, Johnson EW. Management of the foot in rheumatoid
122. Kalish SR, McGlamry ED. The modified Keller hallux valgus repair arthritis. Orthop Clin North Am 2:733–744, 1971.
utilizing Silastic implants with some comments on implants in gen- 144. Brattstrom H, Brattstrom M. Resection of the metatarsophalangeal
eral. J Am Podiatry Assoc 64:761–773, 1974. joints in rheumatoid arthritis. Acta Orthop Scand 41:213–224, 1970.
123. Cracchiolo A, Weltmer JB, Lian G, Dalseth T, Dorey F. Arthroplasty 145. Budiman-Mak E, Roach KE, Stuck R, Spencer F, Polizos T, Conrad
of the first metatarsophalangeal joint with a double-stem silicone KJ. Radiographic measurement of hallux valgus in the rheumatoid
implant. Results in patients who have degenerative joint disease arthritic foot. J Rheumatol 21:623– 626, 1994.
failure of previous operations, or rheumatoid arthritis. J Bone Joint 146. Clayton ML. Surgery in the forefoot in rheumatoid arthritis. Clin
Surg 74A:552–563, 1992. Orthop 16:136 –140, 1960.
124. Vanore JV. Implants. In McGlamry’s Comprehensive Textbook of 147. Cracchiolo A, Swanson A, Swanson GD. The arthritic great toe
Foot and Ankle Surgery, pp 373– 417, edited by SJ Miller, Lippincott, metatarsophalangeal joint: a review of flexible silicone implant ar-
Williams & Wilkins, Philadelphia, 2001. throplasty from two medical centers. Clin Orthop 157:64 – 69, 1981.
125. Vanore J, O’Keefe RG, Pikscher I. Current status of first metatarso- 148. Groulier P, Curvale G, Piclet-Legre B, Kelberine F. Arthrodesis of
phalangeal joint implants. Foot Ankle Q 8:121–134, 1995. the first metatarsophalangeal joint. Rev Chir Orthop Reparatrice
126. Swanson AB. Implant arthroplasty for the great toe. Clin Orthop Appar Mot 80:436 – 444, 1994.
85:75– 81, 1972. 149. Laird L. Silastic joint arthroplasty of the great toe. A review of 228
127. Swanson AB. Implant arthroplasty in disabilities of the great toe. In implants using the double-stemmed implant. Clin Orthop 255:268 –
The American Academy of Orthopaedic Surgeons Instructional 272, 1990.
Course Lectures, CV Mosby, St Louis, 1972. 150. Mann RA, Thompson FM. Arthrodesis of the first metatarsophalan-
128. Albin RL, Weil LS. Flexible implant arthroplasty of the great toe: an geal joint for hallux valgus in rheumatoid arthritis. J Bone Joint Surg
evaluation. J Am Podiatry Assoc 64:967–975, 1974. 66:687– 692, 1984.
129. Arenson DJ. The angled great toe implant (Swanson design/Weil 151. Mann RA, Thompson FM. Arthrodesis of the first metatarsophalan-
modification) in the surgical reconstruction of the first metatarsopha- geal joint for hallux valgus in rheumatoid arthritis. 1984. Foot Ankle
langeal joint. Clin Podiatry 1:89 –102, 1984. Int 18:65– 67, 1997.
130. Weil LS, Pollak RA, Goller WL. Total first joint replacement in 152. Moeckel BH, Sculco TP, Alexiades MM, Dossick PH, Inglis AE,
hallux valgus and hallux rigidus. Long-term results in 484 cases. Clin Ranawat CS. The double-stemmed silicone-rubber implant for rheu-
Podiatry 1:103–129, 1984. matoid arthritis of the first metatarsophalangeal joint. J Bone Joint
131. Vanore JV, Lawrence BR, Weil LS, Zang K. First MPJ implant Surg 74A:564 –570, 1992.
symposium. Foot Ankle Q 8:1995, 1995. 153. Pfeiffer WH, Cracchiolo A, Grace DL, Dorey FJ, Van Dyke E.
132. Zlotoff H. Swanson flexible hinge toe implant. A preliminary report. Double-stem silicone implant arthroplasty of all metatarsophalangeal
J Am Podiatry Assoc 73:347–355, 1983. joints in patients with rheumatoid arthritis. Semin Arthroplasty 3:16 –
133. McKeever DC. Arthrodesis of the first metatarsophalangeal joint for 24, 1992.
hallux valgus, hallux rigidus and metatarsus primus varus. J Bone 154. Swanson AB, Lumsden RM, Swanson GD. Silicone implant arthro-
Joint Surg 34A:129 –134, 1952. plasty of the great toe. A review of single stem and flexible hinge
134. Harrison MHM, Harvey FJ. Arthrodesis of the first metatarsophalan- implants. Clin Orthop 142:30 – 43, 1979.
geal joint for hallux valgus and rigidus. J Bone Joint Surg 45A: 155. Trnka HJ. Arthrodesis procedures for salvage of the hallux metatar-
471480, 1963. sophalangeal joint. Foot Ankle Clin 5:673– 686, 2000.
135. Bourdillon JF. Metatarsophalangeal fusion for hallux valgus. Can 156. Vanore JV, Pikscher I. Forefoot arthroplasty. In Musculoskeletal
Med Assoc J 121:1351, 1979. Disorders of the Lower Extremities, pp 496 –515, edited by LM
136. von Salis-Soglio G, Thomas W. Arthrodesis of the metatarsophalan- Oloff, WB Saunders, Philadelphia, 1994.
geal joint of the great toe. Arch Orthop Trauma Surg 95:7–12, 1979. 157. Merkle PF, Sculco TP. Prosthetic replacement of the first metatarso-
137. Vanore JV. First metatarsophalangeal joint arthrodesis. In Musculo- phalangeal joint. Foot Ankle 9:267–271, 1989.

VOLUME 42, NUMBER 3, MAY/JUNE 2003 123


This is an enhanced PDF from The Journal of Bone and Joint Surgery

The PDF of the article you requested follows this cover page.

Hallux Rigidus. Grading and Long-Term Results of Operative


Treatment
Michael J. Coughlin and Paul S. Shurnas
J Bone Joint Surg Am. 2003;85:2072-2088.

This information is current as of September 5, 2008

Letters to The Editor are available at


http://www.ejbjs.org/cgi/content/full/85/11/2072#responses
Reprints and Permissions Click here to order reprints or request permission to use material from this
article, or locate the article citation on jbjs.org and click on the [Reprints and
Permissions] link.
Publisher Information The Journal of Bone and Joint Surgery
20 Pickering Street, Needham, MA 02492-3157
www.jbjs.org

COPYRIGHT © 2003 BY THE JOURNAL OF BONE AND JOINT SURGERY, INCORPORATED

Hallux Rigidus
GRADING AND LONG-TERM RESULTS OF OPERATIVE TREATMENT
BY MICHAEL J. COUGHLIN, MD, AND PAUL S. SHURNAS, MD
Investigation performed at St. Alphonsus Regional Medical Center, Boise, Idaho

Background: There have been few long-term studies documenting the outcome of surgical treatment of hallux rigi-
dus. The purposes of this report were to evaluate the long-term results of the operative treatment of hallux rigidus
over a nineteen-year period in one surgeon’s practice and to assess a clinical grading system for use in the treatment
of hallux rigidus.
Methods: All patients in whom degenerative hallux rigidus had been treated with cheilectomy or metatarsophalan-
geal joint arthrodesis between 1981 and 1999 and who were alive at the time of this review were identified and
invited to return for a follow-up evaluation. At this follow-up evaluation, the hallux rigidus was graded with a new five-
grade clinical and radiographic system. Outcomes were assessed by comparison of preoperative and postoperative
pain and AOFAS (American Orthopaedic Foot and Ankle Society) scores and ranges of motion. These outcomes were
then correlated with the preoperative grade and the radiographic appearance at the time of follow-up.
Results: One hundred and ten of 114 patients with a diagnosis of hallux rigidus returned for the final evaluation.
Eighty patients (ninety-three feet) had undergone a cheilectomy, and thirty patients (thirty-four feet) had had an arthro-
desis. The mean duration of follow-up was 9.6 years after the cheilectomies and 6.7 years after the arthrodeses.
There was significant improvement in dorsiflexion and total motion following the cheilectomies (p = 0.0001) and sig-
nificant improvement in postoperative pain and AOFAS scores in both treatment groups (p = 0.0001).
A good or excellent outcome based on patient self-assessment, the pain score, and the AOFAS score did not corre-
late with the radiographic appearance of the joint at the time of final follow-up. Dorsiflexion stress radiographs dem-
onstrated correction of the elevation of the first ray to nearly zero. There was no association between hallux rigidus
and hypermobility of the first ray, functional hallux limitus, or metatarsus primus elevatus.
Conclusions: Ninety-seven percent (107) of the 110 patients had a good or excellent subjective result, and 92%
(eighty-six) of the ninety-three cheilectomy procedures were successful in terms of pain relief and function. Cheilec-
tomy was used with predictable success to treat Grade-1 and 2 and selected Grade-3 cases. Patients with Grade-4
hallux rigidus or Grade-3 hallux rigidus with <50% of the metatarsal head cartilage remaining at the time of surgery
should be treated with arthrodesis.
Level of Evidence: Therapeutic study, Level IV (case series [no, or historical, control group]). See Instructions to Au-
thors for a complete description of levels of evidence.

H
allux rigidus is a term used to describe symptoms position arthroplasty23. Some authors reported a technique
commonly associated with degenerative arthritis of consisting solely of the removal of the osteophytes in line
the first metatarsophalangeal joint. Surgical treatment with the dorsal metatarsal cortex2,6,8-10,12,16,21,23,27,29,48, and several of
of protracted symptomatic hallux rigidus includes cheilec- them recommended cheilectomy as a treatment for early dis-
tomy1-13, excisional arthroplasty14-20, interposition arthroplasty13,21-25, ease only2,6,8,12. Although Nilsonne21 and Bonney and Macnab70
phalangeal osteotomy26-30, first metatarsal osteotomy16,31-40, im- found cheilectomy to be unsuccessful, others have reported
plant arthroplasty16,41-47, and arthrodesis14,48-68. successful results with cheilectomy for all levels of disease3,7,11
Cheilectomy has been recommended in a number of and have recommended a more aggressive resection of the
reports1-4,6-8,11-13,69; however, in many of the studies, the duration metatarsal head. Radiographic signs of deterioration of the
of follow-up was less than one year2,3,12,69 or some or all of the metatarsophalangeal joint over time have been reported after
patients were not examined at the time of final follow-up12,69. cheilectomy1,4,7,8, but continued good clinical function despite
In some studies, a substantial number of patients were lost to the worsening radiographic appearance has been noted in
final follow-up11,13; in others, cheilectomy was combined with these studies.
phalangeal osteotomy13,27,29, metatarsal osteotomy34 , or inter- Arthrodesis of the first metatarsophalangeal joint has

THE JOUR NAL OF BONE & JOINT SURGER Y · JBJS.ORG HAL LUX RIG IDUS
VO L U M E 85-A · N U M B E R 11 · N O VE M B E R 2003

TABLE I Clinical-Radiographic System for Grading Hallux Rigidus

Grade Dorsiflexion Radiographic Findings* Clinical Findings


0 40° to 60° and/or 10% to Normal No pain; only stiffness and loss of motion
20% loss compared with on examination
normal side

1 30° to 40° and/or 20% to Dorsal osteophyte is main finding, Mild or occasional pain and stiffness,
50% loss compared with minimal joint-space narrowing, minimal pain at extremes of dorsiflexion and/or
normal side periarticular sclerosis, minimal flattening plantar flexion on examination
of metatarsal head

2 10° to 30° and/or 50% to Dorsal, lateral, and possibly medial Moderate-to-severe pain and stiffness
75% loss compared with osteophytes giving flattened appearance that may be constant; pain occurs just
normal side to metatarsal head, no more than 1/4 of before maximum dorsiflexion and maxi-
dorsal joint space involved on lateral mum plantar flexion on examination
radiograph, mild-to-moderate joint-space
narrowing and sclerosis, sesamoids not
usually involved

3 ≤10° and/or 75% to 100% Same as in Grade 2 but with substantial Nearly constant pain and substantial
loss compared with normal narrowing, possibly periarticular cystic stiffness at extremes of range of motion
side. There is notable loss of changes, more than 1/4 of dorsal joint but not at mid-range
metatarsophalangeal plantar space involved on lateral radiograph,
flexion as well (often ≤10° sesamoids enlarged and/or cystic
of plantar flexion) and/or irregular

4 Same as in Grade 3 Same as in Grade 3 Same criteria as Grade 3 BUT there


is definite pain at mid-range of passive
motion

*Weight-bearing and anteroposterior and lateral radiographs are used.

been evaluated as a treatment for hallux rigidus by several Materials and Methods
authors48-68. However, in many series, the duration of follow- ne hundred and forty consecutive patients were treated,
up was less than one year50,51,55-57, multiple surgeons were in-
volved52,56,61, or more than one technique was performed53,56.
O by the senior author (M.J.C.), with either a cheilectomy
or an arthrodesis of the first metatarsophalangeal joint as sur-
Some studies included patients with other diagnoses such as gical treatment for a symptomatic hallux rigidus deformity
rheumatoid arthritis or hallux valgus49,52,55,57,58,67, and others between November 1981 and November 1999. Twenty-one
provided inadequate information on the patients or the patients were excluded from the present study because of a
method of evaluation48,51,53,56,58,59,65-67 or patients were not eval- diagnosis of gout, rheumatoid arthritis, systemic lupus erythe-
uated at the time of final follow-up66. We found only two matosis, poliomyelitis, or previous pyarthrosis, and five other
reports50,68 that dealt exclusively with the treatment of hallux patients died during the study period. Of the 114 remaining
rigidus, and both included patients with less than one year of patients with a diagnosis of hallux rigidus, four were unavail-
follow-up. able or had moved away; 110 (96%) returned for a final follow-
On the basis of their individual beliefs about the etiol- up evaluation.
ogy of hallux rigidus, authors have proposed several grading Of these 110 patients (127 feet), eighty (ninety-three
systems with either three1,2,12,22,71-73 or four stages35,41,50,71,74 deter- feet) were treated with cheilectomy and thirty (thirty-four feet)
mined according to radiographic criteria only12,71 or a combi- were treated with arthrodesis. Thirteen patients had bilateral
nation of clinical and radiographic criteria1,2,17,22,41,50,72,73. Several cheilectomy, and four patients had bilateral arthrodesis. No pa-
of these grading systems add modifications to an existing tient underwent a cheilectomy on one side and an arthrodesis
scheme2,4,6,8,11,69, while in others the grades or treatment rec- on the contralateral side. Sixty-nine (63%) of the 110 patients
ommendations are based on poorly studied concepts such as were female. The average age at the onset of symptoms was
metatarsus primus elevatus41,50,71 or functional hallux limi- forty-three years (range, thirteen to seventy years), and the
tus17,41,50. All of these variations make comparisons between average age at surgery was fifty years (range, sixteen to seventy-
studies difficult. six years).
The purpose of the present study was to evaluate the
predictability of a clinical-radiographic grading scale for choos- Grading
ing a surgical procedure, and to examine the long-term results At the time of final follow-up, the patients were evaluated with
of cheilectomies and arthrodeses performed by one surgeon. a five-level clinical-radiographic grading system (Table I and

THE JOUR NAL OF BONE & JOINT SURGER Y · JBJS.ORG HAL LUX RIG IDUS
VO L U M E 85-A · N U M B E R 11 · N O VE M B E R 2003

Fig. 1-B

Figs. 1-A and 1-B Radiographs demonstrating Grade-1 hallux rigidus. Fig. 1-A The
anteroposterior radiograph demonstrates a small lateral marginal osteophyte
(arrow) but a well-preserved joint space. Fig. 1-B The lateral radiograph demon-
strates a small dorsal spur (arrow).
Fig. 1-A

Fig. 2-B

Figs. 2-A and 2-B Radiographs demonstrating Grade-2 hallux rigidus. Fig. 2-A
The anteroposterior radiograph demonstrates a substantial lateral marginal
osteophyte, mild flattening of the metatarsal head, mild joint-space narrowing,
and sclerosis. Fig. 2-B The lateral radiograph demonstrates narrowing of less
than one-fourth of the joint space (primarily dorsally) and a more prominent
dorsal spur.
Fig. 2-A

THE JOUR NAL OF BONE & JOINT SURGER Y · JBJS.ORG HAL LUX RIG IDUS
VO L U M E 85-A · N U M B E R 11 · N O VE M B E R 2003

Figs. 1-A through 4-B) that incorporates many features of all served and routinely recorded at the time of surgery. A cheil-
of the classification systems used by previous authors1,2,12,22,72,74 ectomy was always recommended for Grade-1 and Grade-2
but is not based on hypothetical concepts or notions17,41,50,71. All hallux rigidus, whereas a cheilectomy or an arthrodesis was
range-of-motion measurements used in the system refer to recommended for Grade-3 hallux rigidus. An arthrodesis was
passive motion. Pain at mid-range of passive motion refers to recommended when radiographs demonstrated end-stage ar-
pain that is elicited not only at the extremes of passive dorsi- throsis and the clinical examination demonstrated a painful
flexion and plantar flexion of the metatarsophalangeal joint range of motion. When an arthrodesis was performed, such
but also in between. Loose bodies or osteochondral defects patients were typically found to have <50% of the cartilage
can occur with any grade. Their presence does not affect the surface remaining on inspection at surgery. Determining the
assigned grade. There is intentional overlap of the ranges of best type of treatment was the most difficult for patients with
motion of the different grades as range of motion is only one similar radiographic findings who had minimal pain during
of the three general factors determining the eventually as- the range of motion except at maximum plantar flexion and
signed grade of hallux rigidus. Range-of-motion, clinical, and dorsiflexion. If it was acceptable to them, such patients gave
radiographic examinations are utilized to determine the indi- consent for both arthrodesis and cheilectomy, and the surgeon
vidual grade. chose the procedure at the time of the operation on the basis
of the amount of cartilage surface that remained. Although an
Preoperative Findings arthrodesis was always recommended to patients with end-
The indication for surgery was intractable pain isolated to the stage arthritis, which we later characterized as Grade-4 hallux
first metatarsophalangeal joint that was refractory to shoe rigidus, eight of these patients refused the arthrodesis and un-
modifications, use of rigid shoe inserts, nonsteroidal anti- derwent cheilectomy instead.
inflammatory medications, and modification of activities. Early in the study, the magnitude of preoperative pain
Symptoms referable to the great toe and foot were compiled was quantified as none, mild, moderate, severe, or quite se-
from a chart review. The operative reports were also evalu- vere; later, a visual analog scale was used by the patients to
ated to record the estimated percentage of the cartilage surface quantify the level of pain numerically. In order to compare the
of the metatarsal head that remained, which had been ob- preoperative and postoperative pain scores in the treatment

Fig. 3-B

Figs. 3-A and 3-B Radiographs demonstrating Grade-3 hallux rigidus. Fig. 3-A The
anteroposterior radiograph demonstrates substantial joint-space narrowing and
sesamoid irregularity. Fig. 3-B The lateral radiograph demonstrates narrowing of
more than one-fourth of the joint space and a prominent dorsal spur.
Fig. 3-A

THE JOUR NAL OF BONE & JOINT SURGER Y · JBJS.ORG HAL LUX RIG IDUS
VO L U M E 85-A · N U M B E R 11 · N O VE M B E R 2003

Fig. 4-B

Figs. 4-A and 4-B Radiographs demonstrating Grade-4 hallux rigidus. Both the an-
teroposterior (Fig. 4-A) and the lateral (Fig. 4-B) radiographs demonstrate more
advanced degenerative changes than the radiographs showing the Grade-3 dis-
ease. (Grades 3 and 4 may look identical radiographically and must be distin-
guished clinically.)
Fig. 4-A

groups (cheilectomy and arthrodesis) on the basis of the 10- the onset of the symptoms, and duration of pain or other
point visual analog score that was eventually used, a number symptoms.
was assigned retrospectively to the preoperative pain de- Patients were asked to rate their satisfaction with the re-
scribed by the patients seen early in the study. None was quan- sult of the surgery according to a previously published scale76.
tified as 0; mild, as 3; moderate, as 6; severe, as 9; and quite In that scale, a result is considered excellent if the patient has
severe, as 10. no problems related to the foot, is very satisfied, has mild or
Preoperatively, twenty-one patients had moderate pain no pain, walks with mild or no difficulty, and would have the
and the remainder had severe or quite severe pain. Although surgery again under similar circumstances. A result is consid-
the AOFAS (American Orthopaedic Foot and Ankle Society) ered good if the patient has a few problems, is satisfied, has
hallux metatarsophalangeal scale75 was not available until mild pain, walks with no or mild difficulty, and would have
1994, the chart information and radiographs allowed us to the surgery again under similar circumstances. A fair result
also calculate a preoperative AOFAS score retrospectively for means that the patient has moderate pain, some difficulty
each patient. with walking, and reservations about the success of the sur-
gery. A poor result indicates that the patient has continued
Follow-up Evaluation pain, has little or no improvement in walking, and regrets hav-
The mean duration of follow-up was 9.6 years (range, 2.3 to ing had the surgery.
20.3 years) after the cheilectomies and 6.7 years (range, 2.1 More specific questions regarding shoe wear were asked
to 12.2 years) after the arthrodeses. At the time of follow-up, to determine whether the patient could wear fashionable shoes
patients were assessed with a standardized questionnaire and (a 2-in [5-cm] or higher heel) postoperatively, felt pressure
examination. from constricting shoes, or had any other difficulty with shoe
Patients were asked to characterize their postoperative wear. Patients were asked to characterize their shoe-wear re-
pain as none, mild, moderate, severe, or quite severe and also strictions (preoperatively and postoperatively) as none (could
to rate it on a 10-point visual analog scale in which 0 indicated wear fashionable shoes), could wear comfortable shoes only,
no pain and 10 indicated the most severe pain. Patients were or required modified custom shoes or orthotic devices.
also asked to localize the pain, if they had any, and to specify Patients were asked if they could walk on tiptoe and the
their main symptom (pain, stiffness, cosmetic appearance, lock- time to maximal comfort and recovery from the surgery. They
ing, or a gait problem). In addition, patients were questioned were asked to characterize any activity restrictions (preop-
about their clinical history, including use of orthotics, age at eratively and postoperatively) as none (no restriction of sports

THE JOUR NAL OF BONE & JOINT SURGER Y · JBJS.ORG HAL LUX RIG IDUS
VO L U M E 85-A · N U M B E R 11 · N O VE M B E R 2003

or recreational activity), mild (not enough to interfere with termined for all patients at the time of final follow-up. This is
everyday activity but some limitation of sports or recreational a 100-point scale composed of separate sections for pain (40
activity), moderate (limitation of daily and recreational or points), function (45 points), and alignment (15 points). In
sports activity), or severe (major limitation of any activity). the scale for pain, 40 points is given for no pain; 30 points, for
The physical examination included inspection and pal- mild, occasional pain; 20 points, for moderate, daily pain; and
pation of the foot with attention to posture, sensation, mo- 0 points, for severe, constant pain. Function is graded on the
tion, motor function, strength, and appearance. Both feet of basis of activity (10 points for no limitation, 7 points for limi-
all patients were examined. Stance and gait were assessed with tation of recreational activity but not daily or job-related
attention to the position of the foot when the patient walked. activities, 4 points for limitation of daily and recreational ac-
Passive motion of the metatarsophalangeal and interpha- tivities, and 0 points for severe limitation of daily and rec-
langeal joints of the great toe was measured with a goniometer, reational activities), shoe wear (10 points for the ability to
with the plantar aspect of the foot considered to be neutral. wear fashionable and regular shoes with no insert, 5 points
These joints were then assessed for stability in the sagittal for the ability to wear only comfortable shoes or the need for
plane. Pain that occurred in the mid-range, between maxi- an insert, and 0 points for the need to wear modified shoes or
mum plantar and dorsiflexion, of a passive range of motion of a brace), motion (dorsiflexion plus plantar flexion) of the
the hallux metatarsophalangeal joint was noted. The plantar metatarsophalangeal joint (10 points for ≥75°, 5 points for
aspect of the foot was assessed for callosities or areas of ten- 30° to 74°, and 0 points for <30°), motion (plantar flexion) of
derness. Motor strength of the hallux was quantitated on a the interphalangeal joint (5 points for no restriction and 0
5-point scale in which 1 indicated no strength; 2, active move- points for <10°), stability (in all directions) of the metatarso-
ment with gravity eliminated; 3, active movement against grav- phalangeal joint (5 points for stable and 0 points for un-
ity only, without resistance; 4, active movement against gravity stable), and callus related to the metatarsophalangeal and
with some resistance; and 5, active movement against gravity interphalangeal joints (5 points for no or an asymptomatic
with full resistance. callus and 0 points for a symptomatic callus). In the align-
An AOFAS hallux metatarsophalangeal score75 was de- ment section of the scale, 15 points is given for a well-aligned

Fig. 5-B
Radiographs demonstrating the technique for measuring the joint width. On the
preoperative and postoperative radiographs, three points were placed along the
corresponding articular surfaces of the base of the proximal phalanx and the
distal part of the metatarsal. On the anteroposterior radiograph (Fig. 5-A), these
points were placed at the medial, central, and lateral aspects of the joint sur-
Fig. 5-A
faces. On the lateral radiograph (Fig. 5-B), these points were placed at the dor-
sal, middle, and plantar aspects of the joint surface. On a line connecting each
pair of corresponding points, the joint width was measured in millimeters. The
six scores were added and then divided by six, to provide an average width for
each joint.

THE JOUR NAL OF BONE & JOINT SURGER Y · JBJS.ORG HAL LUX RIG IDUS
VO L U M E 85-A · N U M B E R 11 · N O VE M B E R 2003

lateral radiograph was made for all feet3. This is a weight-


bearing lateral radiograph made with manual dorsiflexion
stress applied by grasping the proximal phalanx and passively
extending the metatarsophalangeal joint (Figs. 7-A and 7-B).
The dorsiflexion measured on the weight-bearing stress ra-
diograph was compared with the dorsiflexion measured with a
goniometer without weight-bearing, and any discrepancy was
recorded in order to measure the magnitude of functional
Fig. 6 hallux limitus81-83. In addition, metatarsus primus elevatus
Diagram demonstrating measurement of metatarsus primus elevatus was measured on the stress radiograph and compared with
and the first metatarsal declination angle on weight-bearing lateral ra- the metatarsus primus elevatus on the non-stress standard
diographs. To measure metatarsus primus elevatus, a line is drawn weight-bearing lateral radiograph to evaluate the extent
along the distal dorsal metaphyseal cortex of the first and second to which the metatarsus primus elevatus was a secondary
metatarsals. A perpendicular line is drawn between the two cortical change.
lines, and the distance between the two lines is measured in milli-
meters. To measure the first metatarsal declination angle, the lateral
longitudinal axis of the first metatarsal is drawn with use of mid-
diaphyseal reference points. A second line estimating the plantar sur-
face of the foot is drawn, intersecting reference points on the plantar
aspect of the calcaneus and the medial sesamoid. The intersection of
these two lines forms the first metatarsal declination angle (angle A).

hallux; 8 points, for fair alignment with some deformity but


no symptoms; and 0 points, for symptomatic malalignment.
With use of this scale, 90 points is the highest score attainable
after an arthrodesis.

Radiographic Evaluation
Standardized preoperative weight-bearing radiographs77 were
reviewed and compared with standardized postoperative weight-
bearing radiographs made at the time of the final follow-up.
The width of the metatarsophalangeal joint was determined Fig. 7-A
by a summation method based on six separate measurements Figs. 7-A and 7-B Metatarsus primus elevatus and dorsiflexion stress
(Figs. 5-A and 5-B). Recurrence of the dorsal osteophyte was test. Fig. 7-A A postoperative standing lateral radiograph demonstrat-
assessed on the lateral radiograph by drawing a line along the ing 9 mm of metatarsus primus elevatus.
dorsal cortex of the metatarsal and measuring any osteophyte
above that line in millimeters.
Interphalangeal joint arthritis was graded on the antero-
posterior radiograph with use of a previously described meth-
od78. With this method, Grade 1 indicates no degenerative
changes; Grade 2, mild changes with <1 mm of chondrolysis;
Grade 3, moderate changes with 1 to 2 mm of chondrolysis;
and Grade 4, severe changes with malalignment, cysts, and/or
joint destruction. Periarticular sclerosis at the metatarsopha-
langeal joint was recorded as 0 (no sclerosis), +1 (mild), +2
(moderate), or +3 (severe), as seen on the preoperative and
postoperative anteroposterior radiographs of the patients
treated with cheilectomy.
Elevation of the first ray (metatarsus primus elevatus)
was measured on the weight-bearing lateral radiograph79. The
difference between the dorsal cortices of the first and second
metatarsals measured at the head-neck junction was recorded Fig. 7-B
in millimeters. A normal value is ≤8 mm79. The first metatarsal A standing lateral radiograph of the same foot with application of
declination angle was measured as well79. Normal values are dorsiflexion stress, demonstrating an absence of metatarsus primus
reported to range from 19° to 25°80 (Fig. 6). A dorsiflexion stress elevatus.

THE JOUR NAL OF BONE & JOINT SURGER Y · JBJS.ORG HAL LUX RIG IDUS
VO L U M E 85-A · N U M B E R 11 · N O VE M B E R 2003

Surgical Techniques mark bandage as a tourniquet1,52,76,84. A dorsal longitudinal


A cheilectomy is performed with use of regional anesthesia incision, 4 to 5 cm long, is centered over the metatarsopha-
and with an Esmark bandage (Medline Industries, Mundelein, langeal joint and is deepened through the capsule on the me-
Illinois) employed as a tourniquet1,3,7. A 3-cm dorsal longitudi- dial aspect of the extensor hallucis longus tendon. The capsule
nal incision is centered over the metatarsophalangeal joint and is preserved for later repair. The joint is débrided, and osteo-
is deepened through the capsule on the medial aspect of the phytes and loose bodies are removed. The proximal phalanx is
extensor hallucis longus tendon. The capsule is preserved for plantar flexed, a 0.62-mm Kirschner wire is proximally cen-
later repair. Hypertrophic synovial tissue and loose bodies tered on the first metatarsal articular surface and driven in,
are fully débrided from the joint, and the percentage of viable and an appropriately sized cannulated cylindrical reamer is
cartilage remaining on the metatarsal head is estimated. The used to create a cylinder-shaped metatarsal. A concave can-
proximal phalanx is plantar flexed, exposing the metatarsal nulated metatarsal reamer (small joint reamer; Howmedica,
head. An osteotome is used to remove the dorsal, medial, and Rutherford, New Jersey) is then utilized to create a convex
lateral osteophytes along with 25% to 33% of the metatar- cancellous metatarsal surface. The Kirschner wire is removed,
sal head dorsally. Usually, all or almost all of the cartilage- and attention is then directed to the proximal phalanx. A
deficient surface of the metatarsal head is resected. At least 70° Kirschner wire is centered on the articular surface of the base
of dorsiflexion should be achieved intraoperatively. The osteo- of the proximal phalanx and is driven distally to prepare for
phytes are then removed from the dorsal aspect of the base of cannulated reaming. Convex phalangeal reamers are used to
the proximal phalanx, and the joint is lavaged. Bone wax is ap- create a concentric matching surface on the phalangeal base to
plied to the dorsal region of the metatarsal head. The capsule match the metatarsal head. The hallux is placed in neutral ro-
is repaired beneath the extensor hallucis longus tendon with tation, 15° of valgus, and 20° of dorsiflexion in reference to the
interrupted absorbable sutures, and the skin is approximated axis of the first metatarsal. It is stabilized with a six-hole Vital-
in a routine fashion. lium mini-compression plate and a lag screw. A gentle dorsal
A gauze-and-tape compression dressing is applied at the bend in the plate allows better conformity to the dorsal os-
conclusion of the surgery and is changed every ten days. Pas- seous surfaces to achieve approximately 20° of dorsiflexion.
sive range-of-motion exercises are begun within ten days after Closure is identical to that used for a cheilectomy.
surgery, and aggressive stretching is allowed as pain and swell- A gauze-and-tape compression dressing is applied at the
ing subside. Walking is permitted following surgery with the conclusion of the surgery and is changed every ten days for
patient wearing a stiff-soled postoperative shoe and bearing eight to twelve weeks, until there is radiographic evidence of a
weight as tolerated. successful fusion. The foot is placed in a stiff-soled postopera-
Arthrodesis of the first metatarsophalangeal joint is per- tive shoe after surgery, and weight-bearing on the heel and the
formed with use of regional anesthesia and with use of an Es- lateral aspect of the involved foot is permitted. The first ray is

TABLE II Preoperative and Postoperative Symptoms and Functional Factors in Patients Treated with Cheilectomy or Arthrodesis*

Postop.
Symptom or Preop. Cheilectomy Arthrodesis Total
Functional Factor (N = 110) (N = 80) (N = 30) (N = 110)
Stiffness 109 10 (13%) 14 (47%) 24

Pain 110 46 (58%) 9 (30%) 55

Locking 20 0 0 0
Dissatisfaction with cosmetic appearance 60 0 0 0

Eversion gait 108 10 (13%) 2 (7%) 12

Able to rise up on toes 11 66 (83%) 15 (50%) 81


Pain at metatarsophalangeal joint (main symptom) 55 46 (58%) 9 (30%) 55

Painful dorsal bump (main symptom) 55 0 0 0

Insert or orthotic 20 9 9 18
Able to wear fashionable shoes 50 56 21 77

Able to wear comfortable shoes 110 80 30 110

Discomfort from shoe pressure 110 10 3 13

*The values are given as the number of patients.



THE JOUR NAL OF BONE & JOINT SURGER Y · JBJS.ORG HAL LUX RIG IDUS
VO L U M E 85-A · N U M B E R 11 · N O VE M B E R 2003

TABLE III Preoperative and Postoperative Clinical Scores and Findings on Examination for Patients Treated with Cheilectomy
or Arthrodesis

Cheilectomy Arthrodesis
Score or Finding Total Series
on Examination Preop. Postop. Preop. Postop. (Postop.)
Pain score on visual analog 8 (6-10) 1.5 (0-8) 8.7 (6-10) 0.4 (0-5)
scale* (points)
AOFAS score* (points) 45 (24-70) 90 (67-100) 38 (24-60) 89 (72-90)
Subjective patient self- All fair or poor 58 excellent, All fair or poor 26 excellent,
assessment score (no.) 19 good, 3 fair, 4 good, 0 fair
0 poor or poor
MTP dorsiflexion*† (deg) 14.5 (0-45) 39 (10-65) 7 (−15-10) 0 Dorsiflexion
improved 24.5°
MTP total motion*† (deg) 39.2 (5-80) 63.7 (15-110) 22.1 (5-30) 0 Total motion
improved 24.5°
Callus† (no. of patients/ 5/2nd MTP, 5/IP joint, plantar 4/2nd MTP, 4/IP joint, plantar 9/IP joint, plantar
location) plantar plantar and medial

*The values are given as the mean with the range in parentheses. †MTP = metatarsophalangeal joint and IP = interphalangeal joint.

unweighted in this fashion until there is radiographic evidence operatively. However, there was a significant reduction (p =
of a fusion. 0.0001) in the postoperative pain score indicated on the visual
analog scale. Postoperatively, patients no longer complained
Statistical Analysis of a dorsal bump or pressure from the shoe, but they con-
Descriptive and comparative statistical analysis was performed tinued to have metatarsophalangeal joint pain. The time to
with use of InStat software (GraphPad Software, San Diego, maximum postoperative improvement averaged 2.4 months
California). Standard chi-square analysis was performed on (range, one to 4.5 months) after the cheilectomies and 2.8
continuous variables. Pearson and binary correlation coef- months (range, 1.5 to four months) after the arthrodeses.
ficients were used to evaluate the noncontinuous data, and
positive coefficients (r values) closer to one indicate strong Clinical Scores (Table III)
correlation while values closer to zero indicate weak or no cor- One hundred and seven (97%) of the 110 patients had an ex-
relation. Differences were considered to be significant when cellent or good self-assessment (subjective) score at the time
the p value was ≤0.05. of follow-up, whereas all patients had a fair or poor score pre-
operatively (as estimated retrospectively). A good or excellent
Results outcome based on the patient self-assessment score, visual an-
ighty patients (ninety-three feet) treated with a cheilec- alog score for pain, and AOFAS score did not correlate with
E tomy and thirty patients (thirty-four feet) treated with an
arthrodesis were evaluated at the final examination. At the
the radiographic appearance of the joint (loss of joint space)
at the time of final follow-up (r = 0.08, p = 0.34).
time of follow-up, at a mean of 9.6 years postoperatively, seven At the time of final follow-up, the mean AOFAS score
(8%) of the ninety-three cheilectomies had failed. At the time was significantly improved in both the group treated with
of follow-up, at a mean of 6.7 years postoperatively, thirty-two cheilectomy (p = 0.0001, difference in the means = 45.7, 95%
(94%) of the thirty-four arthrodeses had successfully fused. confidence interval = 43.3 to 48.1) and the group treated with
There were sixteen associated surgical procedures, in- arthrodesis (p = 0.045, difference in the means = 50.4, 95%
cluding four repairs to treat a hammertoe deformity of the confidence interval = 46.5 and 54.4). The cheilectomy group
second toe, nine repairs to treat capsular instability of the sec- had a significantly higher preoperative mean AOFAS score
ond metatarsophalangeal joint, and three Akin phalangeal os- than did the arthrodesis group (45 compared with 38 points,
teotomies performed for severe hallux valgus interphalangeus p = 0.025, difference in the means = 6.6, 95% confidence in-
at the time of metatarsophalangeal joint arthrodesis. terval = 1.8 and 9.1), but there was no difference in the mean
postoperative AOFAS scores (89 compared with 90 points, p =
Historical Data (Table II) 0.3). However, the maximum possible AOFAS score in the ar-
The most common primary preoperative symptoms were throdesis group was 90 points, as 10 points are unavailable be-
metatarsophalangeal joint pain (fifty-five patients) and a pain- cause of loss of motion.
ful dorsal bump (fifty-five patients). These symptoms decreased When the patients treated with cheilectomy were grouped
postoperatively in both treatment groups. Interestingly, the according to the grade of the hallux rigidus, a significant dif-
same number of patients listed their primary symptom as ference in both preoperative and postoperative AOFAS score
metatarsophalangeal joint pain both preoperatively and post- was found among the subgroups (r = 0.3, p = 0.02). Also, a

THE JOUR NAL OF BONE & JOINT SURGER Y · JBJS.ORG HAL LUX RIG IDUS
VO L U M E 85-A · N U M B E R 11 · N O VE M B E R 2003

TABLE IV Mean Range of Motion and Follow-up Scores for Patients Treated with Cheilectomy

Postop.
Dorsiflexion (deg) Total Motion (deg) Postop. Pain Score on
AOFAS score Visual Analog
Grade Preop. Postop. Change Preop. Postop. Change (points) Scale (points)
1 33.3 56.7 23.4 53.3 76.7 23.4 95.7 1.1

2 21.1 46.4 25.3 38.9 62.1 23.2 92.9 1.5

3 9.5 34.5 25 21.5 44.7 23.2 89.8 1.7


4* 5.8 −5.8 12.5 −12.5 88.9 0.4

*Five of nine Grade-4 feet underwent arthrodesis and had a mean loss of motion.

TABLE V Mean Radiographic Values for Patients Treated with Cheilectomy or Arthrodesis

Cheilectomy* Arthrodesis*
Radiographic Parameter Preop. Postop. Preop. Postop.
Metatarsus primus elevatus (mm) 5.3 (0-10) 6.1 (0-15) 5.6 (0-12) 1.8 (0-10)
First metatarsal declination angle (deg) 20.4 (15-27) 21.1 (12-30) 23.4 (19-30) 20 (15-26)

Hallux valgus angle (deg) 12.2 (0-20) 12.6 (0-24) 12.2 (0-20) 11.7 (5-19)

1st-2nd intermetatarsal angle (deg) 7.3 (2-24) 7.7 (2-15) 8.1 (4-14) 8.1 (4-13)
Joint width (mm) 1.6 (0.3-2.7) 1.2 (0-3) 0.9 (0.5-1.6) 0.01 (0-0.5)

Periarticular sclerosis 1.8 (0-3) 2.2 (0-3) 2.6 (2-3) Fused

Interphalangeal joint width (mm) 1.1 (1-2) 1.1 (1-2) 1.1 (1-2) 1.2 (1-2)

*The values are given as the mean with the range in parentheses for the feet.

correlation was found between increasing grade and lower diographs demonstrated no progression of arthritis in either
preoperative AOFAS scores (r = 0.4, p = 0.01). treatment group.
There was significant progression of periarticular scle-
Results of Clinical Examination (Table IV) rosis of the metatarsophalangeal joint (p = 0.0001, difference
The total range of motion of the metatarsophalangeal joint at in the means = 0.5, r = 0.84, 95% confidence interval = 0.3 to
the time of final follow-up after the cheilectomies averaged 0.6) and of loss of the metatarsophalangeal joint width (p =
64°: the average dorsiflexion of the metatarsophalangeal joint 0.0001, difference in the means = 0.4 mm, r = 0.8, 95% con-
improved from 14.5° preoperatively to 38.4° postoperatively fidence interval = 0.25 and 0.46) in the cheilectomy group.
(p = 0.0001, difference in the means = 23.8°, 95% confidence Twenty-one metatarsophalangeal joints were noted to have a
interval = 20.9° to 26.6°). Postoperatively, no significant dif- loose body on preoperative radiographs and confirmatory
ference was noted between passive dorsiflexion and dorsi- findings at the time of surgery, but no loose bodies were noted
flexion with stress at either the metatarsophalangeal or the on final follow-up radiographs. The average size of the recur-
interphalangeal joint of the hallux (p = 0.4, difference in the rent dorsal osteophytes was 0.72 mm (range, 0 to 3 mm) after
means = 1.2°). Dorsiflexion of the interphalangeal joint aver- the cheilectomies and 0 mm after the arthrodeses.
aged 8° in both treatment groups at the time of final follow-
up. Motor strength was graded as 5/5 (normal) in all subjects. Grading of the Hallux Rigidus (Table VI)
All first metatarsophalangeal joints were stable on manual ex- In the cheilectomy group, the mean clinical-radiographic
amination following cheilectomy. Three patients had a Tinel grade of the hallux rigidus was 2.50 (range, 1 to 4) preopera-
sign over the dorsal-medial aspect of the metatarsophalangeal tively compared with 2.59 (range, 1 to 4) postoperatively
joint preoperatively, but no patient had a sensory disturbance (p = 0.1). Nine patients had an increase of one grade at the
postoperatively. time of final follow-up. In the cheilectomy group, patients
with a lower grade preoperatively had a better AOFAS score
Radiographic Data (Table V) both preoperatively and postoperatively. No patient had an
Comparison of the extent of arthritic changes in the interpha- increase in the grade after an arthrodesis, and no patient in
langeal joint between the preoperative and postoperative ra- the series had an increase of more than one grade. Following

THE JOUR NAL OF BONE & JOINT SURGER Y · JBJS.ORG HAL LUX RIG IDUS
VO L U M E 85-A · N U M B E R 11 · N O VE M B E R 2003

TABLE VI Preoperative and Postoperative Clinical-Radiographic Grades for Patients Treated with Cheilectomy or Arthrodesis*

Grade 1 Grade 2 Grade 3 Grade 4


Before cheilectomy 6 (8%) 32 (40%) 34 (43%) 8 (10%)

After cheilectomy 4 (5%) 31 (39%) 33 (41%) 12 (15%)

Before arthrodesis 10 (33%) 20 (67%)

*The values are given as the number of patients with the percentage in parentheses.

the cheilectomies, the patients with Grade-1 hallux rigidus ment groups (r = 0.6, p = 0.01). Preoperatively, 120 (94%) of
had a mean AOFAS score of 95.7 points and a mean pain the 127 feet had <8 mm of elevation, which was within the
score on the visual analog scale of 1.1 points, and all four feet range of normal.
were subjectively rated as excellent. Patients with Grade 2 had The mean preoperative and postoperative measurements
a mean AOFAS score of 92.9 points and a mean pain score of elevatus were 5.3 mm and 6.1 mm in the cheilectomy group.
on the visual analog scale of 1.5 points; of the thirty-eight There was a correlation between the postoperative grade and
feet, thirty-five were rated as excellent and three were rated the amount of elevatus (r = 0.44, p = 0.02). Moreover, the
as good. Patients with a Grade-3 rating had a mean postop- elevatus reduced to a mean of 1.2 mm on dorsiflexion stress
erative AOFAS score of 89.8 points and a mean pain score on examination in the cheilectomy group at the time of final
the visual analog scale of 1.7 points; of the thirty-four feet, follow-up. This value was significantly different from the mea-
twenty-nine were rated as excellent and five were rated as surements of elevatus on weight-bearing lateral radiographs
good. both preoperatively (p = 0.001, difference in the means = 3.9
mm, 95% confidence interval = 1.6 to 5.2 mm) and postop-
Integrity of the Articular eratively (p = 0.001, difference in the means = 4.5 mm, 95%
Surface at the Time of Surgery confidence interval = 1.3 to 5.1 mm).
As estimated by inspection of the metatarsophalangeal joint at Both before and following cheilectomy, an increased
the time of surgery, an average of 17% (range, 0% to 40%) of value for elevatus was associated with a higher grade of hallux
the articular surface of the metatarsal head remained in the ar- rigidus (p = 0.04, r = 0.44). In addition, elevatus decreased
throdesis group and an average of 60% (range, 55% to 90%) postoperatively in patients with Grade-1 or 2 hallux rigidus
remained in the cheilectomy group. There was a correlation but it increased in those with Grade-3 or 4. The mean pre-
between an estimation of <50% of the metatarsal head carti- operative elevatus in the arthrodesis group was 5.6 mm, and
lage remaining and failure of cheilectomy (p = 0.002, r = 0.4). this was significantly reduced postoperatively to 1.7 mm (p =
Also, an estimation of >50% of the metatarsal head cartilage 0.009, difference in the means = 3.9 mm, 95% confidence in-
remaining in a patient undergoing a cheilectomy correlated terval = 2.7 to 4.9 mm).
with a long-term AOFAS score of >80 points and a good or ex-
cellent subjective score (p = 0.01, r = 0.4) at the time of final Complications
follow-up. Five of the eighty patients in the cheilectomy group and two
of the thirty in the arthrodesis group required oral antibiotics
Metatarsus Primus Elevatus (Table VII) for the treatment of mild postoperative cellulitis. There were
There was good correlation between the first metatarsal de- no deep wound infections. No patient in either group had
clination angle and metatarsus primus elevatus in both treat- tenodesis or scarring of the extensor hallucis longus, concerns

TABLE VII Metatarsus Primus Elevatus, According to Grade of Hallux Rigidus, in the Patients Treated with Cheilectomy or
Arthrodesis

Metatarsus Primus Elevatus (mm)


Grade 1 Grade 2 Grade 3 Grade 4
Before cheilectomy* 2.6 (0-8) 5.2 (0-8) 5.2 (0-10) 6 (5-10)

After cheilectomy* 2.5 (0-5) 4.7 (0-8) 8 (0-15) 8.4 (5-10)

P value 0.086 0.0001 0.0001 0.0001


Before arthrodesis* 5.3 (0-10) 6.2 (0-12)

After arthrodesis* 1.1 (0-5) 1.5 (0-6)

*The values are given as the mean with the range in parentheses.

THE JOUR NAL OF BONE & JOINT SURGER Y · JBJS.ORG HAL LUX RIG IDUS
VO L U M E 85-A · N U M B E R 11 · N O VE M B E R 2003

about the cosmetic appearance of the foot, neuritis, or a hy- metatarsophalangeal joint pain (mean pain score, 4.4 points).
pertrophic dorsal scar. However, the cheilectomy unexpectedly failed in two of thirty-
Seven failed cheilectomies in six patients resulted in four patients with Grade-3 hallux rigidus. Both had radio-
seven additional surgical procedures. There were two outcomes graphic evidence of chondrolysis within one year after the
that were unexpected on the basis of the grades assigned with surgery, with progressive pain, and subsequently underwent
our rating system during the study period. These two outcomes arthrodesis of the metatarsophalangeal joint at seven and
consisted of rapid chondrolysis (within one year after the sur- eight years following the cheilectomy.
gery), and both patients had a metatarsophalangeal joint ar- To improve the accuracy and predictability for Grade-3
throdesis (at seven and eight years following the cheilectomy). hallux rigidus, the percentage of metatarsal head cartilage re-
The other four patients (five feet) in which the cheilectomy maining, as estimated with direct surgical inspection, should
failed had originally been advised to have an arthrodesis for be considered, as <50% of the cartilage remaining correlated
the treatment of Grade-4 disease; a metatarsophalangeal joint with failure.
arthrodesis was eventually performed in these four patients. The indications for cheilectomy described in the litera-
The other four patients with Grade-4 changes (who did not ture have varied greatly. While some authors have recom-
have additional surgery after the initial cheilectomy) had a mended cheilectomy for early disease only2,12, others have used
mean long-term AOFAS score of 74 points (range, 67 to 80 the procedure to treat both early and moderate disease4,5; still
points); three rated the result as fair and one rated it as good others have used it for all levels of disease3,7,11,69. The results of
after a mean duration of follow-up of 7.4 years (range, 2.5 to this study support the use of cheilectomy for all levels of dis-
8.4 years). Thus, of the nine feet with Grade-4 changes for ease except Grade 4.
which arthrodesis was recommended but cheilectomy was per- As a result of all of the different grading systems (or a
formed at the patient’s request, five had failure of the proce- lack of grading) and differences in the technique of cheilec-
dure and later underwent arthrodesis (at a mean 6.9 years tomy, it is difficult to compare the results of different studies.
after the cheilectomy) as initially recommended. The other However, rates of satisfaction after cheilectomy have been fa-
four had inferior subjective results (three fair and one good) at vorable, ranging from 72% to 90%2-4,11-13,69. We favor a uniform
the time of follow-up but had radiographic signs of deteriora- grading system to allow comparisons between studies and to
tion of the joint space and a mean pain score of 4.7 points on distinguish between a Grade-3 and a Grade-4 metatarsopha-
the visual analog scale. langeal joint.
Two plates were removed because of pain following a Of interest was a significant progression in objective evi-
successful fusion. Two of the thirty-four feet that had under- dence of periarticular sclerosis as well as loss of joint space
gone arthrodesis had a painless fibrous union, and the AO- width in our patients who had undergone cheilectomy; how-
FAS score was 90 points for both of these feet at the time of ever, there was no correlation between loss of joint width and
follow-up. the AOFAS score, pain score, or patient self-assessment.

Discussion Recurrence and Chondrolysis


Grading System Easley et al.1 reported that dorsal osteophytes recurred in
he grading system used in this study had been modified twenty-one of sixty-eight feet following cheilectomy, al-
T on the basis of the findings of Easley et al.1 in order to
add Grade 4 for advanced hallux rigidus. A grade-0 stage was
though the authors did not specify how the recurrence was
quantified. Several authors have noted that the metatarso-
also added to include asymptomatic patients with early loss phalangeal joint deteriorates radiographically following chei-
of metatarsophalangeal joint motion. The initial grading sys- lectomy1,4,7,11. Others have reported that cheilectomy hastens
tem was a clinical-radiographic scheme described by one of deterioration of the joint2, and still others have reported that
us (M.J.C.)73. deterioration is uncommon3. Smith et al.85 reported on the
The classification system shown in Table I and Figures natural history of hallux rigidus treated nonoperatively and
1-A through 4-B incorporates many of the best elements of observed that the metatarsophalangeal joint deteriorated ra-
prior grading systems1,12,22,72 and requires both subjective and diographically and clinically with time (in sixteen of twenty-
objective examination and radiographic data to determine the four feet). The results of our study support the finding of
grade. When applied retrospectively, the system appeared to those authors that a metatarsophalangeal joint with hallux
be reliable as it correctly predicted a successful outcome in 108 rigidus naturally deteriorates if followed for a long enough pe-
of 110 patients. Moreover, it accurately predicted a fair or riod of time. Cheilectomy does not appear to alter the natural
poor outcome in patients with Grade-4 hallux rigidus treated progression of the disease process, but it enables a patient to
with cheilectomy. Its key utility was in the distinction between be more comfortable during the course of degeneration.
Grade-3 and Grade-4 hallux rigidus. Cheilectomy uniformly
failed in patients with Grade-4 hallux rigidus, as predicted; Metatarsophalangeal Joint Motion
five of the nine feet in which Grade-4 hallux rigidus was Improved metatarsophalangeal joint motion following cheil-
treated with cheilectomy subsequently had an arthrodesis, and ectomy has been emphasized in several studies1,2,6,11. Dorsiflex-
two of the four remaining patients had moderate to severe ion has reportedly been improved by 28° to 50°, depending on

THE JOUR NAL OF BONE & JOINT SURGER Y · JBJS.ORG HAL LUX RIG IDUS
VO L U M E 85-A · N U M B E R 11 · N O VE M B E R 2003

the individual study and the specific surgical technique used higher grades of disease2,12,13. A cheilectomy alters the joint in
for the cheilectomy1,3,7,13. Authors using the technique de- some fashion, providing pain relief even in the presence of
scribed by Mann et al.3 have reported 20° to 30° improvements more advanced disease. However, there is a point (Grade-4
in dorsiflexion and total range of motion1,3,4,7,11,13, whereas those disease) at which reliable results are more difficult to obtain.
using a minimal resection technique have reported about 50% We support the notion of Easley et al.1 that a clinical finding
less improvement in dorsiflexion2,6,12. Several authors have re- of pain at the mid-range of motion (Grade 4) is a harbinger of
ported diminishing improvements in motion with increases in a poor result following cheilectomy and that it is critical to
the grade of the disease2,6,12, whereas others1, including us, have recognize this finding especially in the presence of advanced
found fairly uniform improvement in motion even in patients radiographic changes. However, as noted by others7,69, radio-
with more advanced disease. We reported an increase in mean graphic findings alone do not correlate with the final clinical
dorsiflexion from 14.5° preoperatively to 38.4° postoperatively result, and we believe that purely radiographic grading sys-
and an increase in mean total motion from 39.2° to 64°. This tems should be discarded.
improvement occurred consistently across all grades, and it
corresponds very closely with the findings reported by several Arthrodesis
authors1,3,7,11,13, confirming the reliability of our cheilectomy There is little information on the performance of arthrodesis
technique. solely for the treatment of hallux rigidus52,53,57,58. Fitzgerald53 re-
Thus, it appears that minimal resection techniques ported that arthritis developed in the interphalangeal joint
are not as versatile, especially for more advanced grades of when the metatarsophalangeal joint had been fused in <15°
disease2,6,8,12. With our more aggressive resection technique, the of valgus. One of us (M.J.C.)78 previously reported that arthri-
indications can be extended to include more advanced disease tis was more common in interphalangeal joints when the
without compromising the result or creating an unstable metatarsophalangeal joint had been fused in <22° of dorsiflex-
joint3,7,13,69. ion. Other studies have shown progression of interphalangeal
joint arthritis after metatarsophalangeal joint arthrodesis78,86,87.
Pain Relief However, these studies included a large number of patients
Pain relief has been reported consistently following cheilec- with rheumatoid arthritis. In the current study, which dealt
tomy1,3,7,69, but some authors have noted less pain relief with only with hallux rigidus, we noted no progression of degener-

Fig. 8-A
Figs. 8-A and 8-B Radiographs of a foot with a long-term good result
of cheilectomy. Fig. 8-A Preoperative anteroposterior radiograph
showing a foot with Grade-3 hallux rigidus. Fig. 8-B Anteroposterior
radiograph made eighteen years following cheilectomy; the foot still Fig. 8-B

has Grade-3 hallux rigidus.



THE JOUR NAL OF BONE & JOINT SURGER Y · JBJS.ORG HAL LUX RIG IDUS
VO L U M E 85-A · N U M B E R 11 · N O VE M B E R 2003

Fig. 9-A Fig. 9-B


Figs. 9-A and 9-B Failure of cheilectomy. Fig. 9-A The pre-
operative anteroposterior radiograph shows a foot with
symptomatic hallux rigidus. Fig. 9-B Two years following
cheilectomy, painful arthrosis developed.

ative changes in the interphalangeal joint following cheilec-


tomy or arthrodesis.

Complications
Arthrodesis resulted in a 94% fusion rate, and neither of the
fibrous unions was painful. Two patients underwent hard-
ware removal because of pain. There were no failures of hard-
ware or other complications.
Although Easley et al.1 suggested that a medial approach
is preferable for cheilectomy because of the risk of tenodesis or
scarring of the extensor hallucis longus tendon, because of
cosmetic concerns, and to avoid creating a hypertrophic dor-
sal scar, we found no evidence of these complications in any
patient.

Metatarsus Primus Elevatus


The concept of metatarsus primus elevatus as a cause of hallux
rigidus has been endorsed in several reports2,8,23,28,30,34,36,37,41,50,70,81,82,88,89,
whereas radiographic evidence to the contrary79,80,90 has also
been reported. Functional hallux limitus (reduction of dorsi-
flexion of the first metatarsophalangeal joint with loading of
the foot compared with passive dorsiflexion with non-weight- Fig. 9-C
bearing) has also been proposed as a cause of hallux rigidus81-83. Radiograph made following arthrodesis of the metatarsophalangeal
Attempts to quantitate the magnitude of elevatus have been joint.

THE JOUR NAL OF BONE & JOINT SURGER Y · JBJS.ORG HAL LUX RIG IDUS
VO L U M E 85-A · N U M B E R 11 · N O VE M B E R 2003

made with use of two different measurements, the first meta- an average of 9.6 years after cheilectomy and 100% of those
tarsal declination angle80 and the difference in elevation between followed for an average 6.7 years after arthrodesis had a good
the dorsal cortices of the first and second metatarsals as seen on or excellent subjective result. Pain scores in both groups were
a weight-bearing lateral radiograph79,90. significantly reduced compared with the preoperative scores,
Normal elevatus is considered to be ≤8 mm, and the and AOFAS scores were significantly improved.
normal first metatarsal declination angle has been reported to The clinical-radiographic grading system used in this
be between 19° and 25°79,90. The mean preoperative elevatus study appears to be reliable. We believe that cheilectomy can
was 5.5 mm in our patients, well within the limits of normal. be used with success for Grades 1, 2, and 3 hallux rigidus
Moreover, the mean first metatarsal declination angle was (Figs. 8-A and 8-B), but patients with Grade 4 or with <50%
within normal limits both preoperatively and postoperatively of the cartilage surface of the metatarsal head remaining at
in our patients. the time of surgery should be treated with arthrodesis (Figs.
Metatarsus primus elevatus is rarely a structural prob- 9-A, 9-B, and 9-C). For patients who desire preservation of
lem that needs to be corrected by an osteotomy, as has been motion and are willing to accept less than total pain relief,
recommended by others32,34,36-39,74. In our series, it consistently cheilectomy provides a high proportion of good and excel-
decreased after cheilectomy in patients with lower grades of lent long-term results. However, arthrodesis should be con-
disease. We noted that these patients had the least amount of sidered for patients who clearly have pain in the mid-range
elevatus. Elevatus corrected to nearly zero with the dorsiflex- of motion on examination. A high proportion of these pa-
ion stress test (with any grade of hallux rigidus). While the ele- tients can be expected to have a good or excellent long-term
vatus decreased with passive dorsiflexion stress, we found no result after arthrodesis performed with the technique de-
difference between first metatarsophalangeal joint motion scribed in this study. 
with the dorsiflexion stress test and passive metatarsopha- NOTE: The authors thank Mary Sampson, MS, for assistance with the statistical analysis.
langeal joint motion. These findings call into question the
concept of functional hallux limitus that has been reported in
the literature81-83. However, we hypothesized that functional
hallux limitus may represent the residual elevatus that we oc- Michael J. Coughlin, MD
casionally noted on the dorsiflexion stress radiographs of pa- 901 North Curtis Road, Suite 503, Boise, ID 83706. E-mail address:
footmd@aol.com
tients with more advanced disease.
This study demonstrates that a simple procedure such as Paul S. Shurnas, MD
cheilectomy tends to reduce elevation of the first ray as the Regional Orthopaedic Health Care, No. 3 Medical Plaza, Mountain
joint functions more normally. We believe that when the joint Home, AR 72653
has deteriorated clinically to the point where elevation of the
first ray is pronounced, it is an indication for metatarsopha- The authors did not receive grants or outside funding in support of their
langeal arthrodesis because first-ray elevation significantly di- research or preparation of this manuscript. They did not receive pay-
ments or other benefits or a commitment or agreement to provide such
minished after arthrodesis in our series. benefits from a commercial entity. No commercial entity paid or
Our study had the longest mean duration of follow-up directed, or agreed to pay or direct, any benefits to any research fund,
after both cheilectomy and arthrodesis for the treatment of foundation, educational institution, or other charitable or nonprofit
hallux rigidus. Ninety-six percent of the patients followed for organization with which the authors are affiliated or associated.

References
1. Easley ME, Davis WH, Anderson RB. Intermediate to long-term follow-up of 9. Feldman RS, Hutter J, Lapow L, Pour B. Cheilectomy and hallux rigidus.
medial-approach dorsal cheilectomy for hallux rigidus. Foot Ankle Int. 1999; J Foot Surg. 1983;22:170-4.
20:147-52.
10. Gould N. Hallux rigidus: cheilotomy or implant? Foot Ankle. 1981;1:
2. Geldwert JJ, Rock GD, McGrath MP, Mancuso JE. Cheilectomy: still a useful 315-20.
technique for grade I and grade II hallux limitus/rigidus. J Foot Surg. 1992;
31:154-9. 11. Feltham GT, Hanks SE, Marcus RE. Age-based outcomes of cheilectomy for
the treatment of hallux rigidus. Foot Ankle Int. 2001;22:192-7.
3. Mann RA, Coughlin MJ, DuVries HL. Hallux rigidus: a review of the literature
and a method of treatment. Clin Orthop. 1979;142:57-63. 12. Hattrup SJ, Johnson KA. Subjective results of hallux rigidus following treat-
ment with cheilectomy. Clin Orthop. 1988;226:182-91.
4. Mulier T, Steenwerckx A, Thienpont E, Sioen W, Hoore KD, Peeraer L,
Dereymaeker G. Results after cheilectomy in athletes with hallux rigidus. 13. Lau JT, Daniels TR. Outcomes following cheilectomy and interpositional ar-
Foot Ankle Int. 1999;20:232-7. throplasty in hallux rigidus. Foot Ankle Int. 2001;22:462-70.
5. Shereff MJ, Baumhauer JF. Hallux rigidus and osteoarthritis of the first meta- 14. Giannestras NJ. Hallux rigidus. In: Foot disorders: medical and surgical man-
tarsophalangeal joint. J Bone Joint Surg Am. 1998;80:898-908. agement. 2nd ed. Philadelphia: Lea and Febiger; 1973. p 400-2.
6. Mackay DC, Blyth M, Rymaszewski LA. The role of cheilectomy in the treat- 15. Kurtz DH, Harrill JC, Kaczander BI, Solomon MG. The Valenti procedure for
ment of hallux rigidus. J Foot Ankle Surg. 1997;36:337-40. hallux limitus: a long-term follow-up and analysis. J Foot Ankle Surg. 1999;
38:123-30.
7. Mann RA, Clanton TO. Hallux rigidus: treatment by cheilectomy. J Bone Joint
Surg Am. 1988;70:400-6. 16. Chang TJ. Stepwise approach to hallux limitus. A surgical perspective. Clin
Podiatr Med Surg. 1996;13:449-59.
8. Pontell D, Gudas CJ. Retrospective analysis of surgical treatment of hallux
rigidus/limitus: clinical and radiographic follow-up of hinged, silastic implant 17. Saxena A. The Valenti procedure for hallux limitus/rigidus. J Foot Ankle Surg.
arthroplasty and cheilectomy. J Foot Surg. 1988;27:503-10. 1995;34:485-8.

THE JOUR NAL OF BONE & JOINT SURGER Y · JBJS.ORG HAL LUX RIG IDUS
VO L U M E 85-A · N U M B E R 11 · N O VE M B E R 2003

18. Cleveland M, Winant EM. An end-result study of the Keller operation. J Bone DH, Cestari VA. The use of a grommet bone liner for flexible hinge implant ar-
Joint Surg Am. 1950;32:163-75. throplasty of the great toe. Foot Ankle. 1991;12:149-55.
19. Grady JF, Axe TM. The modified Valenti procedure for the treatment of hallux 47. Townley C, Taranow W. A metallic hemiarthroplasty resurfacing prosthesis
limitus. J Foot Ankle Surg. 1994;33:365-7. for the hallux metatarsophalangeal joint. Foot Ankle Int. 1994;15:
20. Jordan HH, Brodsky AE. Keller operation for hallux valgus and hallux rigidus. 575-80.
An end result study. Arch Surg. 1951;62:586-96. 48. Bingold AC, Collins DH. Hallux rigidus. J Bone Joint Surg Br. 1950;32:214-22.
21. Nilsonne H. Hallux rigidus and its treatment. Acta Orthop Scand. 1930;1: 49. Smith NR. Hallux valgus and rigidus treated by arthrodesis of the metatarso-
295-302. phalangeal joint. Br Med J. 1952;2:1385-7.
22. Barca F. Tendon arthroplasty of the first metatarsophalangeal joint and hallux 50. Lombardi CM, Silhanek AD, Connolly FG, Dennis LN, Keslonsky AJ. First
rigidus: preliminary communication. Foot Ankle Int. 1997;18:222-8. metatarsophalangeal arthrodesis for treatment of hallux rigidus: a retrospec-
23. Cosentino GL. The Cosentino modification for tendon interpositional arthro- tive study. J Foot Ankle Surg. 2001;40:137-43.
plasty. J Foot Ankle Surg. 1995;34:501-8. 51. Chana GS, Andrew TA, Cotterill CP. A simple method of arthrodesis of the
24. Hamilton WG, O’Malley MJ, Thompson FM, Kovatis PE. Capsular inter- first metatarsophalangeal joint. J Bone Joint Surg Br. 1984;66:703-5.
position arthroplasty for severe hallux rigidus. Foot Ankle Int. 1997;18: 52. Coughlin MJ, Abdo RV. Arthrodesis of the first metatarsophalangeal joint with
68-70. Vitallium plate fixation. Foot Ankle Int. 1994;15:18-28.
25. Ganley JV, Lynch FR, Darrigan RD. Keller bunionectomy with fascia and ten- 53. Fitzgerald JAW. A review of long-term results of arthrodesis of the first meta-
don graft. J Am Podiatr Med Assoc. 1986;76:602-10. tarsophalangeal joint. J Bone Joint Surg Br. 1969;51:488-93.
26. Moberg E. A simple operation for hallux rigidus. Clin Orthop. 1979;142: 54. Fitzgerald JA, Wilkinson JM. Arthrodesis of the metatarsophalangeal joint of
55-6. the great toe. Clin Orthop. 1981;157:70-7.
27. Thomas PJ, Smith RW. Proximal phalanx osteotomy for the surgical treatment 55. Gimple K, Amspacher JC, Kopta JA. Metatarsophalangeal joint fusion of the
of hallux rigidus. Foot Ankle Int. 1999;20:3-12. great toe. Orthopedics. 1978;1:462-7.
28. Kessell L, Bonney G. Hallux rigidus in the adolescent. J Bone Joint Surg Br. 56. Harrison MH, Harvey FJ. Arthrodesis of the first metatarsophalangeal joint for
1958;40:668-73. hallux valgus and rigidus. J Bone Joint Surg Am. 1963;45:471-80.
29. Blyth MJ, Mackay DC, Kinninmonth AW. Dorsal wedge osteotomy in the 57. Johansson JE, Barrington TW. Cone arthrodesis of the first metatarsopha-
treatment of hallux rigidus. J Foot Ankle Surg. 1998;37:8-10. langeal joint. Foot Ankle. 1984;4:244-8.
30. Citron N, Neil M. Dorsal wedge osteotomy of the proximal phalanx for hallux 58. Marin GA. Arthrodesis of the metatarsophalangeal joint of the big toe for
rigidus. Long-term results. J Bone Joint Surg Br. 1987;69:835-7. hallux valgus and hallux rigidus. A new method. Int Surg. 1968;50:175-80.
31. Davies-Colley M. Contraction of the metatarso-phalangeal joint of the great 59. McKeever DC. Arthrodesis of the first metatarsophalangeal joint for hallux
toe. Br Med J. 1887;1:728. valgus, hallux rigidus, and metatarsus primus varus. J Bone Joint Surg Am.
1952;34:129-34.
32. Youngswick FD. Modifications of the Austin bunionectomy for treatment of
metatarsus primus elevatus associated with hallux limitus. J Foot Surg. 60. Moynihan FJ. Arthrodesis of the metatarso-phalangeal joint of the great toe.
1982;21:114-6. J Bone Joint Surg Br. 1967;49:544-51.
33. Southgate JJ, Urry SR. Hallux rigidus: the long-term results of dorsal 61. Riggs SA Jr, Johnson EW Jr. McKeever arthrodesis for the painful hallux. Foot
wedge osteotomy and arthrodesis in adults. J Foot Ankle Surg. 1997;36: Ankle. 1983;3:248-53.
136-40.
62. von Salis-Soglio G, Thomas W. Arthrodesis of the metatarsophalangeal joint
34. Cavolo DJ, Cavallaro DC, Arrington LE. The Watermann osteotomy for hallux of the great toe. Arch Orthop Trauma Surg. 1979;95:7-12.
limitus. J Am Podiatry Assoc. 1979;69:52-7.
63. Thompson FR, McElvenny RT. Arthrodesis of the first metatarsophalangeal
35. Viegas GV. Reconstruction of hallux limitus deformity using a first metatarsal joint. J Bone Joint Surg Am. 1940;22:555-8.
sagittal-Z osteotomy. J Foot Ankle Surg. 1998;37:204-11.
64. Tupman S. Arthrodesis of the first metatarsophalangeal joint. J Bone Joint
36. Lundeen RO, Rose JM. Sliding oblique osteotomy for the treatment of hallux Surg Br. 1958;40:826.
abducto valgus associated with functional hallux limitus. J Foot Ankle Surg.
65. Turan I, Lindgren U. Compression-screw arthrodesis of the first metatar-
2000;39:161-7.
sophalangeal joint of the foot. Clin Orthop. 1987;221:292-5.
37. Ronconi P, Monachino P, Baleanu PM, Favilli G. Distal oblique osteotomy of
66. Wilkinson J. Cone arthrodesis of the first metatarsophalangeal joint. Acta
the first metatarsal for the correction of hallux limitus and rigidus deformity.
Orthop Scand. 1978;49:627-30.
J Foot Ankle Surg. 2000;39:154-60.
67. Wilson JN. Cone arthrodesis of the first metatarso-phalangeal joint. J Bone
38. Davies GF. Plantarflexory base wedge osteotomy in the treatment of func-
Joint Surg Br. 1967;49:98-101.
tional and structural metatarsus primus elevatus. Clin Podiatr Med Surg.
1989;6:93-102. 68. Wilson CL. A method of fusion of the metatarsophalangeal joint of the great
toe. J Bone Joint Surg Am. 1958;40:384-5.
39. Feldman KA. The Green-Watermann procedure: geometric analysis and
preoperative radiographic template technique. J Foot Surg. 1992;31: 69. Keogh P, Nagaria J, Stephens M. Cheilectomy for hallux rigidus. Ir J Med Sci.
182-5. 1992;161:681-3.
40. Kissel CG, Mistretta RP, Unroe BJ. Cheilectomy, chondroplasty, and sagittal 70. Bonney G, Macnab I. Hallux valgus and hallux rigidus. A critical survey of op-
“Z” osteotomy: a preliminary report on the alternative joint preservation ap- erative results. J Bone Joint Surg Br. 1952;34:366-85.
proach to hallux limitus. J Foot Ankle Surg. 1995;34:312-8.
71. Hanft JR, Mason ET, Landsman AS, Kashuk KB. A new radiographic classifi-
41. Drago JJ, Oloff L, Jacobs AM. A comprehensive review of hallux limitus. J Foot cation for hallux limitus. J Foot Ankle Surg. 1993;32:397-404.
Surg. 1984;23:213-20.
72. Regnauld B. Disorders of the great toe. In: Elson R, editor. The foot: pathol-
42. Albin RK, Weil LS. Flexible implant arthroplasty of the great toe: an evalua- ogy, aetiology, semiology, clinical investigation and therapy. New York:
tion. J Am Podiatry Assoc. 1974;64:967-75. Springer; 1986. p 269-81, 344-9.
43. Blair MP, Brown LA. Hallux limitus/rigidus deformity: a new great toe im- 73. Coughlin M. Arthritides. In: Coughlin MJ, Mann RA, editors. Surgery of the
plant. J Foot Ankle Surg. 1993;32:257-62. foot and ankle. 7th ed, vol 1. St. Louis: Mosby; 1999. p 605-50.
44. Hanft JR, Merrill T, Marcinko DE, Mendicino R, Gerbert J, Vanore JV. Grand 74. Selner AJ, Bogdan R, Selner MD, Brunch EK, Mathews RL, Riley J. Tricorrec-
rounds: first metatarsophalangeal joint replacement. J Foot Ankle Surg. 1996; tional osteotomy for the correction of late-stage hallux limitus/rigidus. J Am
35:78-85. Podiatr Med Assoc. 1997;87:414-24.
45. LaPorta GA, Pilla P Jr, Richter KP. Keller implant procedure: a report of 536 75. Kitaoka HB, Alexander IJ, Adelaar RS, Nunley JA, Myerson MS, Sanders M.
procedures using a Silastic intramedullary stemmed implant. J Am Podiatry Clinical rating systems for the ankle-hindfoot, midfoot, hallux, and lesser
Assoc. 1976;66:126-47. toes. Foot Ankle Int. 1994;15:349-53.
46. Swanson AB, de Groot Swanson G, Maupin BK, Shi SM, Peters JG, Alander 76. Coughlin MJ. Arthrodesis of the first metatarsophalangeal joint with mini-

THE JOUR NAL OF BONE & JOINT SURGER Y · JBJS.ORG HAL LUX RIG IDUS
VO L U M E 85-A · N U M B E R 11 · N O VE M B E R 2003

fragment plate fixation. Orthopedics. 1990;13:1037-44. limitus concept. In: Reconstructive surgery of the foot and leg update ’93.
Tucker, GA: The Podiatry Institute; 1993. p 27-32.
77. Smith RW, Reynolds JC, Stewart MJ. Hallux valgus assessment: report of re-
search committee of American Orthopaedic Foot and Ankle Society. Foot 84. Coughlin MJ. Arthrodesis of the first metatarsophalangeal joint. Orthop Rev.
Ankle. 1984;5:92-103. 1990;19:177-86.
78. Coughlin MJ. Rheumatoid forefoot reconstruction. A long-term follow-up 85. Smith RW, Katchis SD, Ayson LC. Outcomes in hallux rigidus patients
study. J Bone Joint Surg Am. 2000;82:322-41. treated nonoperatively: a long-term follow-up study. Foot Ankle Int. 2000;
21:906-13.
79. Horton GA, Park YW, Myerson MS. Role of metatarsus primus elevatus in
the pathogenesis of hallux rigidus. Foot Ankle Int. 1999;20:777-80. 86. Mann RA, Thompson FM. Arthrodesis of the first metatarsophalangeal joint for
hallux valgus in rheumatoid arthritis. J Bone Joint Surg Am. 1984;66:687-92.
80. Bryant A, Tinley P, Singer K. A comparison of radiographic measurements in nor-
mal, hallux valgus, and hallux limitus feet. J Foot Ankle Surg. 2000;39:39-43. 87. Mann RA, Schakel ME 2nd. Surgical correction of rheumatoid forefoot defor-
mities. Foot Ankle Int. 1995;16:1-6.
81. Dananberg HJ. Gait style as an etiology to chronic postural pain. Part I. Func-
tional hallux limitus. J Am Podiatr Med Assoc. 1993;83:433-41. 88. Jack EA. The aetiology of hallux rigidus. Br J Surg. 1940;27:492-7.
82. Dananberg HJ. Gait style as an etiology to chronic postural pain. Part II. Pos- 89. Lambrinudi C. Metatarsus primus elevatus. Proc R Soc Med. 1938;31:1273.
tural compensatory process. J Am Podiatr Med Assoc. 1993;83:615-24.
90. Meyer JO, Nishon LR, Weiss L, Docks G. Metatarsus primus elevatus and the
83. DiNapoli D. Gait analysis based on first MTPJ function: the functional hallux etiology of hallux rigidus. J Foot Surg. 1987;26:237-41.
OSTEOTOMY OF THE CALCANEUM FOR PES CAVUS

F. C. DWYER, LIVERPOOL, ENGLAND

Since the publication of my earlier paper (Dwyer 1955) describing the treatment of pes
cavus in children by osteotomy of the calcaneum, many more patients have been operated
upon and the procedure has been extended to embrace the more severe deformities encountered
in older patients. Over five years have now elapsed since the first operation was performed, and
the late results indicate that the procedure is sound.
In children pes cavus develops at any time after the age ofthree, the age ofonset depending
on whether the condition is idiopathic or neurological in origin. In the earlier stages what is
little more than a high arch is well controlled by a metatarsal bar, but in many cases the
deformity progresses to embrace the following characteristics : 1) dropping of the forefoot;
2) contracture of the plantar fascia ; 3) clawing of the toes and callosity formation ; 4) the
gradual development of a varus deformity of the heel and forefoot.
It should be appreciated that, contrary to some text-book descriptions and as pointed
out by Todd (1935), contracture of the calcaneal tendon is not a usual accompaniment of
uncomplicated pes cavus; equinus is confined to the forefoot, and the heel is usually well
developed.
Operations such as those described by Steindler (1921), Lambrinudi (1932), Todd (1935)
and others for the relief of pes cavus have, on the whole, proved disappointing, especially
for well established deformity in adolescents (Fig. 1).

Pes cavus in a patient aged twelve, two years after a Steindler operation.

Whatever the underlying etiology, it is certain that, with the onset of varus, the deformity
rapidly becomes worse. With adduction of the heel three deforming influences appear. First
the calcaneal tendon becomes an active invertor and, since its pull is concentrated largely
on the inner border of the heel, much of its force is transmitted to the thick medial portion
of the plantar fascia (Fig. 2). In consequence the forefoot is pulled towards the heel.
Secondly, the child walks on the outer border of the foot, thus increasing the varus deformity.

80 THE JOURNAL OF BONE AND JOINT SURGERY


OSTEOTOMY OF THE CALCANEUM FOR PES CAVUS 81

Finally, with the heel in varus, weight


bearing fails to stretch the plantar fascia
and contracture results. The operation
that I have described aims at overcoming
these deforming influences by shifting the
heel into valgus and allowing weight
bearing to produce correction.

THE OPERATION
First the plantar fascia is divided
subcutaneously to reduce the obvious
dropping of the forefoot. The outer aspect
of the calcaneum is then exposed through
a curved incision and the whole flap
turned forwards until the tendon of the
peroneus longus muscle is exposed. The
upper and lower surfaces of the bone are
cleared. A wedge, with its base eight to
twelve millimetres in width, is removed
from the calcaneum just below the peroneus
longus, which is used as a guide (Fig. 3).
FIG. 2
This wedge tapers down to the medial With inversion of the heel, weight bearing and the pull
cortex, which must be broken to ensure of the calcaneal tendon become deforming forces, allowing
the plantar fascia to contract with resultant approximation
complete closure of the resulting gap. of heel and forefoot.
Pressing the forefoot up into dorsiflexion
against the pull of the calcaneal tendon brings the bony surfaces together nicely. Failure to
obtain closure is invariably due to leaving a small piece of the apex behind. (It is important

FIG. 3
Technique of operation. A wedge is removed from the lateral aspect of the calcaneum.
The osteotomy is made just below the peroneus longus tendon and the posterior tab-
calcaneal joint. The medial cortex must be cracked to allow the distal fragment to close
easily.

VOL. 41 B, NO. 1, FEBRUARY 1959


F
82 F. C. DWYER

FIG. 4
Case l-Pes cavus with early varus deformity complicating poliomyeitis in a boy of five.

FIG. 5

Case 1-Three years after operation. Foot virtually normal and toes straight.

THE JOURNAL OF BONE AND JOINT SURGERY


OSTEOTOMY OF THE CALCANEUM FOR PES CAVUS 83

FIG. 7
Case 2-Deformity of foot in a boy of six, three years after accidental division of both peroneal tendons and
a year after transplantation of tibialis anterior to base of fourth metatarsal.

FIG. 8
Case 2-The same foot a year after removal of a lateral wedge from the calcaneum.

at this stage to be sure that the varus deformity has been properly corrected and that the
heel is in a neutral or even slightly valgus position.)

DISCUSSION
An early effect of the operation is that walking and shoe wear are remarkably improved.
With the varus deformity of the heel corrected, the forefoot gradually follows and the whole
foot becomes plantigrade. With certain exceptions that will be mentioned later, relapse is
rare and steady improvement takes place provided that the heel has been fully corrected
(Figs. 4 to 12).
The principle underlying this type of correction was well illustrated in a boy of three,
first seen eight months after irreparable division of both peroneal tendons. Transplantation
of the tibialis anterior to the base of the fourth metatarsal two years later, when varus was
already well established, failed to halt the progress of the deformity, which was severe at the
age of six (Fig. 7). Osteotomy of the calcaneum was performed, and, with the heel squarely
on the ground, progressive improvement took place with weight bearing (Fig. 8).

VOL. 41 B, NO. 1, FEBRUARY 1959


84 F. C. DWYER

FIG. 9
Case 3-Recurrent cavo-varus deformity in a boy aged twelve, two years after the performance of a Steindler
operation. (Same patient as in Figure 1.)

FIG. 10
Case 3-The same patient four years after wedge osteotomy of the calcaneum. The toes were not operated
upon, and correction of the clawing has been brought about simply by weight bearing after correction of the heel.

Failure to correct fully the varus deformity of the heel will inevitably lead to recurrence
or progress ofthe initial deformity (Fig. 9), and this explains the high incidence of recurrence
after the older types of operation.
Since it was first performed in May 1952 this operation has now been undertaken sixty-
three times in forty-one children aged from three to sixteen years. In all the patients the
improvement, not only in gait and shoe wear, but also in the dropping of the forefoot and
clawing of the toes, has been striking. The early promise has been so well fulfilled by the
passage of time that it seems unnecessary to perform any other operation for established pes
cavus. In patients presenting early with a high arch only and no varus of the heel, a plantar
fasciotomy alone is indicated.
in a few cases it has been necessary to repeat the operation either because complete
correction was not obtained at the first operation or because of real recurrence. The reason
for recurrence may in some instances be the primary tendency, but muscular imbalance seems
to be an important cause because it is particularly in cases of poliomyelitis and spina bifida
that one encounters such recurrence. This was well illustrated in a patient of seven, with

THE JOURNAL OF BONE AND JOINT SURGERY


OSTEOTOMY OF THE CALCANEUM FOR PES CAVUS 85

FIG. II
Case 4-Severe deformity of feet in a girl of sixteen.

II. 12
Case 4-Two years after correction of the varus of the heel together with multiple filleting and arthrodesis of
the toes.

marked varus of the heel after poliomyelitis. Good correction was obtained but, two years
later, a second osteotomy was necessary because varus had recurred and, even after that,
there was further recurrence, which was dealt with by transplanting the tibialis anterior to
overcome the relative weakness of the evertors.
Recurrence occurred also in one patient with spina bifida who suffered infection of the
calcaneum after the operation. A persistent sinus cleared up after removal of a sequestrum.
There was then still some varus of the heel and it was rather small and did not come properly
to the ground. Three years after the original operation the varus deformity was corrected by
inserting a bone wedge into the inner side of the calcaneum, which had the effect of increasing
the height of the heel and at the same time improving the deformity.
In six heels the operation was repeated at an interval varying from six to eighteen months
after the original operation because there was some varus deformity, probably because the
initial wedge had been too narrow.
Even in older children, with well established bony deformity of the forefoot, quite
appreciable flattening of the longitudinal arch can be demonstrated some time after correction

VOL. 41 B, NO. 1, FEBRUARY 1959


86 F. C. DWYER

of the heel (Figs. 1 1 and 12). The degree of correction seen in Figure 12 is better than average,
and the extent offalling ofthe arch which takes place depends on the age at the time of operation
and on the strength of the bones. In boys, with rather stronger bones than in girls, one does
not expect to see so much change in the cavus element. Nevertheless correction of the heel
overcomes the important problem of excessive wear of the outer sides of the shoes, and also
allows the toes to straighten out if they are malleable. The subjective improvement is much
greater than the objective because the feet are properly balanced.
In patients with pes cavus one seldom encounters real contracture of the calcaneal tendon.
lfsuch contracture is present it must be dealt with first by a separate operation, before correction
of the cavus deformity.
In adults with a high fixed arch, gross varus of the heel and forefoot and clawing of the
toes, good correction can be obtained by the same means. A large lateral wedge is removed
from the heel to correct inversion, and the plantar fascia is divided. The dropped and distorted
forefoot can then be brought up to full correction by taking an appropriate wedge from the
tarso-metatarsal region. It must be appreciated, however, that to achieve sound fusion at
this site takes a good deal longer than is the case with a mid-tarsal wedge. This is a small
price to pay for preservation of mid-tarsal-subtalar movement.

SUMMARY
1 . A new surgical approach to the treatment of pes cavus is suggested. The operation consists
in a subcutaneous division of the contracted plantar fascia and correction of the varus
deformity of the heel by removing a wedge from its lateral aspect. It is submitted that, by
approaching the deformity from behind and overcoming the varus of the heel, the foot is
rendered plantigrade and that thereafter weight bearing exerts a corrective influence which
results in progressive improvement of the deformity. The operation is essentially a prophylactic
one and, for the best results, it should be performed before there is gross structural deformity
and while active growth is still taking place.
2. Even in patients over the age of fourteen, improvement is obtained by doing nothing more
than this simple operation. In the presence of fixed deformity of the forefoot, as encountered
in older patients, inversion is corrected by removing a lateral wedge from the calcaneum and
the cavus by taking a dorsal wedge from the tarso-metatarsal region. This has the double
advantage of producing good correction of deformity, while at the same time preserving
movement at the mid-tarsal-subtalar joint. Fixed clawing of the toes will require appropriate
corrective treatment, but if the toes are malleable the simple effect of weight bearing on the
plantigrade foot produces gradual correction.

REFERENCES

DwYER, F. C. (1955): A New Approach to the Treatment of Pes Cavus. Sixi#{232}me Congr#{232}s de Chirurgie
orthopCdique, Berne, 30 ao#{252}t-3septembre 1954. Soci#{233}t#{233}
Internationale de Chirurgie Orthop#{233}dique et de
Traumatologie, p. 551. Bruxelles: Imprimerie Lielens.
HALLGRLMSSON, 5. (1939): Pes Cavus, seine Behandlung und einige Bemerkungen #{252}ber
seine Atiobogie. Acta
Orthopaedica Scandinavica, 10, 73.
LAMBRINUDI, C. (1932): Use and Abuse of Toes. Post-Graduate Medical Journal, 8, 459.
STEINDLER, A. (1921): The Treatment of Pes Cavus (Hollow Claw Foot). Archives of Surgery, 2, 325.
TODD, A. H. (1935): The Treatment of Pes Cavus. Proceedings of the Royal Society of Medicine (Section of
Orthopaedics), 28, 117.

THE JOURNAL OF BONE AND JOINT SURGERY


CALCANEO-VALGUS DEFORMITY

DILLWYN EVANS, CARDIFF, WALES

A discussion of the essential deformity in calcaneo-valgus feet develops a theme originally put forward
in 1961 on the relapsed club foot (Evans 1961). Whereas in the normal foot the medial and lateral columns
are about equal in length, in talipes equino-varus the lateral column is longer and in calcaneo-valgus shorter
than the medial column. The suggestion is that in the treatment of both deformities the length of the columns
be made equal. A method is described of treating calcaneo-valgus deformity by inserting cortical bone grafts
taken from the tibia to elongate the anterior end of the calcaneus.

The material in this article arises out of a mistake removal of too much bone produced a short rigid valgus
made in the treatment of club foot and develops a theme foot with a convex medial border. The radiological
put forward in a previous article in this Journal (Evans features of such a case are shown in Figures 1 to 3. Logic
1961). In that article I described an operation which could suggested that if this shape had been produced by exces-
correct club foot in the older child. It was based on the sive shortening of the lateral column, it should be possible
concept that one element in the deformity of club foot is to improve the shape by lengthening the lateral column
relative overgrowth of the lateral column of the foot, by the insertion of a bone graft. The calcaneo-cuboid
and I suggested that in the older child it might be arthrodesis was therefore undone, the calcaneus and
necessary deliberately to equalise the columns by excising cuboid bones were prised apart and the gap was plugged
bone from the lateral column at the level of the calcaneo- with cortical bone taken from the tibia.
cuboid joint. It was important to excise the correct The result was gratifying and the experience was
amount of bone because removal of too little bone illuminating because it seemed to justify two theoretical
resulted in under-correction of the deformity, whereas assumptions : first, that varus and valgus are opposite

Figure 1-A radiograph of the original relapsed club foot showing the medial displacement of the navicular bone on the head of the talus.
Figure 2-After wedge resection of the calcaneo-cuboid joint. Too much bone has been removed and the talo-navicular dislocation is
over-corrected. The clinical effect is a rigid valgus deformity of the foot. Figure 3-After lengthening the calcaneus. The normal
talo-navicular relationship has been restored and the clinical shape of the foot is satisfactory.

270 THE JOURNAL OF BONE AND JOINT SURGERY


CALCANEO-VALGUS DEFORMITY 271

Fio. 8 Fio. 9
The operation. Figure 4-The incision. Figure 5-The exposure. Figure 6-The calcaneus has been divided and the “spreader” is in
position. This instrument, which has proved invaluable, is described in the text. Figure 7-Insertion of the first graft between the blades
of the instrument. Figure 8-Three grafts in position. Figure 9-The wound is usually closed easily.

VOL. 57-B, No. 3, AUGUST 1975


272 D. EVANS

deformities ; and, second, that the difference between the point at which the lateral column could be effectively
two in terms of tarsal structure lay in the relative lengths shortened in club foot was at the calcaneo-cuboid joint,
of the two columns of the foot. A long lateral column because of the need to pull the navicular bone laterally
was associated with varus deformity of the tarsus, includ- in relation to the talus. It was obviously desirable,
ing a varus heel and possibly also equinus, whereas a however, to preserve the calcaneo-cuboid joint, and it
short lateral column was associated with valgus deformity seemed reasonable to think that if the calcaneus itself
of the tarsus, including a valgus heel and possibly also could be lengthened near its anterior end this might have
calcaneus deformity. the effect of pushing the navicular bone medially and so
If these assumptions are sound it should be possible straightening the foot. It was reasonable, therefore, to
to improve other calcaneo-valgus deformities by lengthen- do an osteotomy of the anterior end of the calcaneus
ing the lateral border ofthe foot; but at what level should about 1 .5 centimetres behind the calcaneo-cuboid joint
it be lengthened ? Experience had shown that the only and in a plane parallel with that joint. The two parts of

.a 12
A boy, born in November 1949, developed anterior poliomyelitis at the age of I 5 months which caused a calcaneo-valgus deformity of
the left foot. This deformity, which was passively correctable, was treated by talo-navicular arthrodesis in the hope that this would hold
the foot in the corrected position. It failed to do so. In July 1959, when he was 10, the left calcaneus was elongated. It was found that this
corrected the deformity but only after the talo-navicular arthrodesis had been undone to free the midtarsal joint. Figures 10 and I 1 show
the clinical appearance before and after the operation and Figures 12 and 13 show the radiographs before and after the lengthening
of the calcaneus.

THE JOURNAL OF BONE AND JOINT SURGERY


CALCANEO-VALGUS DEFORMITY 273

the calcaneus could then be forced apart to lengthen the foot, but before discussing indications and contra-
lateral column, and the gap could be plugged with bone. indications, I shall describe the operation.
The first case chosen for this operation was that of
a calcaneo-valgus deformity resulting from poliomyelitis
THE OPERATION
-in a foot that had been selected for triple arthrodesis-
and it was found that what had been anticipated in theory The operation is constant in principle but the practical
came about in practice. As the anterior part of the details vary with the aetiology of the valgus deformity.
calcaneus was pushed forward, the valgus deviation of The constant factors are shown in Figures 4 to 9.
the forefoot disappeared, the heel took up a more varus An incision is made over thelateralsurface ofthe calcaneus
position and passive extension at the ankle became more parallel with, andjust above, the peroneal tendons, avoid-
restricted. it was apparent as this was happening that ing the sural nerve lest it become involved in the scar.
if the calcaneus were lengthened enough the equinovarus The anterior half of the bone is exposed and the calcaneo-
deformity of club foot would be produced. cuboid joint is identified. The anterior end of the cal-
The clinical result was encouraging and it seemed caneus is then divided through its narrow part in front
justifiable to apply the operation to other types of valgus of the peroneal tubercle by an osteotome, the line of

A boy sustained a cut over the inner side of the left foot at the age of 6 which divided the tendon of tibialis posterior. He developed a
secondary valgus and planus deformity ofthe foot. The calcaneus was elongated at the age of 15 years. Figures 14 and 15 show the clinical
appearance before and after operation, and Figures 16 and 17 the corresponding radiographs.

VOL 57-B, No. 3, AUGUST 1975


274 D. EVANS

division being parallel with and about 1 .5 centimetres CLINICAL MATERIAL


behind the calcaneo-cuboid joint. The cut surfaces of
the calcaneus are then prised apart by means of a spreader The operation was first done in 1959 and it has been
and a graft of cortical bone taken from the tibia is found to be ofvalue as an alternative to triple arthrodesis
inserted between the blades of the spreader to maintain in valgus deformity from four causes-over-corrected
separation of the two pieces of the calcaneus. The talipes equino-varus, calcaneo-valgus following poliomy-
spreader* that I use (Fig. 6) was designed for this purpose elitis, rigid flat foot, and gross idiopathic calcaneo-valgus.
by Mr Q. S. Otto, now of Johannesburg; its blades are These deformities all show a radiological feature which
so arranged that they not only enable the cut surfaces of indicates a need for the operation ; an antero-posterior
the calcaneus to be prised apart but they also allow the radiograph of the foot in the standing position shows
first, or holding, graft to be inserted before the instrument that the talus points in a medial direction and that the

A girl born in 1949 developed anterior poliomyelitis at the age of 7


in 1956. She was first seen in 1961 and found to have 25 centimetres
of shortening in the left leg, weakness throughout the limb, valgus
and pronation of the left foot and much weakness of tibialis anterior
and posterior muscles. The extensors of the toes were strong and
there was “dropping” of the forepart of the foot. In May 1961 the
calcaneus was elongated. This produced an equinus deformity
(despite the fact that extension was possible to 10 degrees above
the right angle before the operation) and the calcaneal tendon was
therefore elongated. The long extensor tendon of the great toe was
transferred into the neck of the first metatarsal bone to improve the
“dropping” of the forefoot. Figure 1 8 shows the appearance before
operation, and Figure 19 afterwards.

is withdrawn. Inspection of the foot at this stage will navicular bone is displaced laterally in relation to the
reveal that the forepart of the foot has become adducted, head of the talus-that is, the reverse of the deformity
that the heel has moved into varus and that extension of of club foot.
the ankle:has become less free. The spreader is removed The operation has been done on fifty-six feet. Four
and further grafts are inserted above and below the first operations were for over-corrected talipes equino-varus,
graft to ensure that the two cut surfaces of the calcaneus twenty-five for deformities resulting from poliomyelitis
remain apart. All grafts are obtained from the tibia of (Figs. 10 to 13 and 18 and 19), two for deformity following
the same side. The wound is then closed and the foot traumatic division of the tendon of tibialis posterior in
immobilised comfortably in plaster in a position of slight infancy (Figs. 14 to 17), nine for rigid flat foot, and
equino-varus. The plaster is retained for about four eighteen for idiopathic valgus (Figs. 20 to 29) including
months to allow consolidation of the new calcaneus, but one case of Marfan’s syndrome (Figs. 30 to 33). It has
weight-bearing is allowed at four weeks. No after-care been found (Figs. 18 to 25) that the operation restricts
is needed when the plaster is removed. extension of the ankle and that it reduces the range of

* Obtainable from Messrs Downs Surgical Ltd.

THE JOURNAL OF BONE AND JOINT SURGERY


CALCANEO-VALGUS DEFORMITY 275

A boy presented at the age of 15 because of pain in the calves of both legs after activity. His feet were found to be of the plano-valgus-
abductus type and the calcaneal lengthening was done on both feet. Figures 20 and 21 show the clinical appearance before and after
operation and the range of movement before and after this is shown in Figures 22 and 23, from which it is seen that there has been restriction
of dorsifiexion. Figures 24 and 25 are the radiographs before and after operation.

VOL. 57-B, No. 3, AUGUST 1975


276 D. EVANS

side-to-side movements in the foot by eliminating exces- be possible to obtain full correction at a second operation
sive eversion. An occasional, and unnecessary, error has done two or three years later.
been damage to the sural nerve, which produces a painful Experience has shown that it is not possible to over-
scar and sensory impairment along the lateral border of correct valgus deformity ofthis aetiology. On the contrary,
the foot. adequate correction may be difficult because sufficient
Calcaneo-valgus from poliomyelitis (Figs. 10 to 13 and separation of the divided parts of the calcaneus may not
18 and 19)-The ideal age for correction is between eight be possible without dividing all the soft tissues on the
and twelve years, but the operation can be done earlier lateral side of the foot, including the peroneal tendons.
if the severity of the deformity makes this necessary. If When this has been done, difficulty in skin closure becomes
done early in life, or if the deformity is very severe, the limiting factor.
the operation may have to be repeated between the ages Rigid flat foot-These cases tend to present in early
of eight and twelve. Full correction may not be possible adolescence ; the foot is rigid and, as in paralytic cases,
with severe deformity at the first attempt, but it should it tends to resist correction ; the soft tissues have to be

I._. __ 1-._. 29
A child of 12 had idiopathic calcaneo-valgus feet, with no symptoms but the muscles were weak and the movements of inversion and eversion
were restricted. The parents were concerned about the shape of the feet. Figures 26 and 27 show the clinical appearance before and after
calcaneal lengthening and the radiograph before operation is shown in Figure 28 and another, taken ten years afterwards, in Figure 29.

THE JOURNAL OF BONE AND JOINT SURGERY


CALCANEO-VALGUS DEFORMITY 277

divided and over-correction is not possible. The opera- in radiographs taken standing. Such cases do not usually
tion has succeeded in feet in which a calcaneo-navicular present until about the age of eight, and it is important
bar has been present. The shape of the foot is slightly to know that over-correction is possible and that it is all
improved but the most gratifying features are relief of too easy to produce an equinovarus deformity. In this
pain and a subjective feeling of freedom within the foot. group the calcaneus should be lengthened only as far as is
Severe idiopathic valgus (Figs. 20 to 33)-Here it is necessary to produce a normal shape ; the soft tissues must
necessary to distinguish between simple mild valgus which not bedivided and the peroneal tendons must not be injured.
is a variant of normal, and severe valgus which is clearly Conditions in which the operation is confra-indicated-
abnormal. Correction is necessary only when deformity The operation is inappropriate for neurological disorders
is severe and the foot is obviously abnormal, with marked including spasticity in children and spina bifida. Over-
valgus of the heel and of the forefoot and with a convex correction is too prevalent in spastic disorders, and in
bulging medial border; lateral displacement of the navi- spina bifida the calcaneus is too soft to allow correction
cular in relation to the head of the talus will be seen and the grafts tend to sink into the bone.

A boy, born in February 1957, presented in June 1961. He had Marfan’s syndrome, with long feet, plano-valgus in shape, and hypermobile.
There was also valgus deviation and pronation at the midtarsal joints with a valgus deformity of the heels on weight-bearing. The head of
the talus was prominent on the medial side ofeach foot. The left calcaneus was elongated in June 1961 and the right calcaneus in June 1962.
The wound on the right foot failed to heal by first intention and a skin-graft was necessary. Figures 30 and 31 show the clinical appearance,
and Figures 32 and 33 the radiographs, also before and after calcaneal lengthening.

VOL. 57-B, No. 3, AUGUST 1975


278 D. EVANS

DISCUSSION calcaneus suggests that equinus may be a more complex


The operation has proved to be of practical value as a subject than it has appeared to be.
means of averting triple arthrodesis but it also has Thirdly, the lateral column is the foundation of the
theoretical implications which are of some interest, be- skeletal structure of the foot. It is the base on which
cause it throws some light on the nature and structure the foot stands. It does not vary much in shape but it
of some deformities of the tarsus. Three conclusions are varies in length, and the length of this column relative to
drawn from this study. the length of the medial column has an enormous
Firstly, the deformities of equino-varus and calcaneo- influence on the shape of the foot, even if it is not the
valgus are opposites. Some believe that the opposite only factor. It plays no part in some deformities, such
of club foot is congenital vertical talus, but I have found as the cavo-varus foot so well described and so effectively
little to support this view, and most of the evidence, treated by Dwyer (1959).
theoretical and experimental, points to calcaneo-valgus It is also doubtful if the lateral column of the foot
as being the opposite of equino-varus. is in fact the primary factor in producing a deformity;
Secondly, in regard to an equinus deformity, it has more likely is it a secondary, or adaptive, consequence
been assumed that it is a deformity at the ankle produced of deformity initiated by other factors such as congenital
by a short calcaneal tendon and because of this belief, abnormality or the forces of muscle imbalance acting on
it is accepted by many that this is produced in a club a plastic growing skeleton. But whatever its origin, once
foot by contracture of the calf muscles. This, however, this factor of inequality has developed-and it is betrayed
is not necessarily so ; the experiences recorded in this radiologically by relationship of the talus and navicular
article suggest that rearrangement of tarsal relationships bones--the foot cannot be restored to a good shape until
may produce immediate equinus on the operating table the inequality has been eliminated by equalising the
without anything being done to the ankle or to the calf columns. It is possible, sometimes, to twist a weakened,
structures (Figs. 18 to 25). It is therefore possible that paralytic foot into a good shape and to feel that arthro-
there may be two kinds of equinus : one produced desis of the talo-navicular joint should hold it there, only
primarily by contracture of the calf structures, and to find in practice that it does not do so (Figs. 10 to 13);
another produced primarily by deformity of the tarsus, the foot falls back into valgus when it has to take the
such as in club foot. weight of the body and this is because the foundation of
Against this it may be said by some that it is its structure-the lateral column-is unsound. It appears
unacceptable because experience has shown that tran- from this study that the lateral column of the foot is the
section of the calcaneal tendon in a baby’s foot will key to structural equino-varus and calcaneo-valgus.
reduce equinus. This, of course, is true but it is true only The operation has been successful in over-corrected
under certain conditions ; these are, first, that the tarsal talipes equino-varus (four cases), calcaneo-valgus caused
deformity is corrected by other means such as by manipu- by poliomyelitis (twenty-five cases), by old injury to the
lation or by division of other tight structures, and second, tendon of tibialis posterior (two cases), in painful rigid
that the operation is done at an early age when the tarsal flat foot in young people (nine cases), and in severe
bones are still cartilaginous and plastic. It is demonstrably disabling idiopathic calcaneo-valgus (eighteen cases). It
not true in the older child when the bones have ossified is of no value in spastic disorders (in which there is a
and lost much of their plasticity. This, combined with the tendency to over-correct and so produce equino-varus) or
fact that it is possible to produce a club foot (including in cases of spina bifida in which the bones of the foot are
the equinus deformity) simply by over-lengthening the soft and too yielding.

I wish to thank Miss B. Wales and Mr C. M. Walker of the Departments of Radiology and Clinical Photography at the Prince of Wales
Orthopaedic Hospital for their contribution to this article and Miss L. M. Thomas, Miss M. A. Angove and Miss H. R. Taylor for their
secretarial help.

REFERENCES
Dwyer, F. C. (1959) Osteotomy of the calcaneum for pes cavus. Journal ofBone and Joint Surgery, 41-B, 80-86.
Evans, D. (1961) Relapsed club foot. Journal ofBone andJoint Surgery, 43-B, 722-733.

THE JOURNAL OF BONE AND JOINT SURGERY


INJURIES TO THE TARSOMETATARSAL JOINT
INCIDENCE, CLASSIFICATION AND TREATMENT

P. H. HARDCASTLE, R. RESCHAUER, E. KUTSCHA-LISSBERG, W. SCHOFFMANN

From the Department ofAccident Surgery, Landeskrankenhaus, Graz, Austria

Injuries to the tarsometatarsal (Lisfranc) joint are not common, and the results of treatment are often
unsatisfactory. Since no individual is likely to see many such injuries, we decided to make a retrospective
study of patients from five different centres. In this way 119 patients with injuries of the Lisfranc joint have
been collected. This paper classifies these injuries and describes their incidence, mechanism of production,
methods of treatment, results and complications. Sixty-nine of the patients attended for review: 35 of these
had been treated by closed methods, 27 had had an open reduction and seven patients had had no treatment.
On the basis ofour study we suggest that these injuries should be classified according to the type of injury
rather than the nature of the deforming force and that their treatment be based upon this classification. It
seems that, whatever the severity of the initial injury, prognosis depends on accurate reduction and its
maintenance.

The tarsometatarsal joint, at which the bases of the


five metatarsals articulate with the three cuneiforms and
the cuboid, was named after Lisfranc a French surgeon TYPE A QO’3
. . . . TOTAL INCONGRUITY C
serving in the Napoleonic Army, who described ampu-
tation through that joint (Cassebaum 1963).
Dislocation and fracture-dislocation are both rare tAT MED tAT. MED

and are said to occur at the rate of one person per 55 000 #{231}\
per year (Aitken and Poulson 1963 ; English 1964). They TYPE B

result from direct or indirect forces (Rainaut, Cedard PARTIAL INCONGRUITY Li


and d’Hour 1966) acting on or through the Lisfrancjoint. Medal dislocation

Although the anatomy and the various mechanisms of


the injury have been well described (Gissane 1951;
Aitken and Poulson 1963; Jeffreys 1963; Wiley 1971;
Bonnel and Barth#{233}l#{233}my 1976), there is considerable J
difference of opinion regarding classification and treat- Lateral dislocation

ment (BOhler 1958; Granberry and Lipscomb 1962;


Wilppula 1973 ; Engelhardt and Ganz 1 975). The purpose
of this present study was to try to clarify both.

CLASSIFICATION
TYPE C %“IIil “UM’
Previous classifications have been based on the mecha- DIVERGENT

nisms ofinjury (Francesconi 1925; Jeffreys 1963; Wilson


1972; Bonnel and Barth#{233}l#{233}my 1976). These mechanisms
are complex and varied. Direct forces may crush the Total
displacement
Partial
displacement
metatarsals displacing them plantarwards, with second- Fig I
ary medial or lateral displacement, depending on the Classification of Lisfranc injuries (modified from Qu#{233}nu and KOss
nature of the applied force. With indirect rotational 1909).

P. H. Hardcastle, MD, FRCS Ed, FRACS, Senior Orthopaedic Registrar


Hollywood Orthopaedic Service, Sir Charles Gairdner Hospital, Nedlands, Perth, Western Australia.
E. Kutscha-Lissberg, MD, Dozent
Unfailabteilung A.O. Krankenhaus, A-2620, Neunkirchen, Austria.
R. Reschauer, MD, Dozent
W. Schoffmann, MD
Landeskrankenhaus, Auenbruggerplatz, Graz 8036, Austria.
© 1982 British Editorial Society ofBone and Joint Surgery 030l-620X/82/3065-0349 $2.00

VOL. 64-B. No. 3, 1982 349


350 P. H. HARDCASTLE, R. RESCHAUER, E. KUTSCHA-LISSBERG, W. SCHOFFMANN

forces the foot must be in plantarfiexion for dislocation affects one or more of the lateral four metatarsals but the
to occur (Wiley 197 1 The dorsal aspect of the Lisfranc
). first metatarsal is not affected.
joint is unable to resist tensile forces of any magnitude, Type C. Divergent. There may be partial or total
in contrast to the much stronger soft-tissue support on incongruity. On an anteroposterior radiograph the first
the plantar aspect. Forces applied to the plantarfiexed metatarsal is seen to be displaced medially, while any
foot may be angular or rotational; since they are applied combination of the lateral four metatarsals may be
at a point distant from the joint they are collectively displaced laterally. Sagittal displacement occurs in
called indirect forces. conjunction with the coronal displacement.

Number of patients
35 42

40

I
20

10

2 I I 2 44 2 II 3 I 2 42

Ml-4 M2 M2-5 M3-5 M4-5 MS (1-2

No fracture of the toot M : metatarsal

Fracture of one or more metatarsals C: cuneiform

Fracture of cuneitorms. cuboid. navicular


talus and or caicaneus

Fig. 2
Concomitant injuries involving the foot in 1 19 cases of Lisfranc injury.

MATERIAL AND METHODS


Such classifications give information regarding the For this retrospective study the notes and radiographs of patients with
nature of the deforming force but their value is limited injuries of the tarsometatarsal joint treated between 1965 and 1978 in
since they do not provide information which would the Accident Departments at Graz, Kiagenfurt, Vienna. Linz and
influence treatment. Our study shows that treatment and Salzburg, were reviewed. There were 1 19 patients with adequate
documentation and radiographs; of these, 69 attended for review,
prognosis depend not so much on the direction of the
between 18 months and I 2 years after the injury. The incidence of these
causal force as upon whether there is incongruity (partial injuries was approximately one person per 60 000 of the population
or total) of the tarsometatarsal joint. each year.
The classification described by Qu#{233}nuand K#{252}ss There were 86 men and 33 women with a mean age of 39 years.
(1909) has the virtue of being simple to apply. It divides Forty-five injuries were due to industrial accidents, 48 to road traffic
accidents, 10 occurred at sport and 16 involved a fall from a height, a
the injuries into three groups (homolateral, isolated and
twisting injury or a household accident. In addition to the tarsometa-
divergent) but does not include every variety of displace- tarsal injury, there were often fractures in other parts of the same foot.
ment. Their classification forms the basis for the one we Figure 2 shows both the incidence of the different segments that were
now present (Fig. 1), upon which treatment can be based. dislocated and that of concomitant fractures of other bones.
Type A . Total. There is incongruity of the entire Only in those patients who were operated upon was it possible to
assess plantar or dorsal displacement accurately. in many of the
tarsometatarsaljoint. Displacement is in one plane which
patients who were not operated upon, oblique radiographs, which could
may be sagittal, coronal or combined. have shown displacement (B#{246}hler 1958; Bonnel and Barth#{233}l#{233}my
1976)
Type B : Partia!. There is incongruity of part of the joint. were not available.
Again the displaced segment is in one plane which may Treatment. The initial treatment at all five centres was to attempt
be sagittal, coronal or combined. Partial injuries are of closed reduction either by manipulation under general anaesthesia, or
by using skeletal traction (B#{246}hler 1958). Percutaneous Kirschner wire
two kinds whose treatment and prognosis differ : medial
fixation was used if the reduction was clinically unstable. Open
displacement affects the first metatarsal either in isolation reduction was performed when congruent closed reduction was not
or combined with displacement of one or more of the achieved. In 14 cases where there was an associated open wound its
second, third or fourth metatarsals ; lateral displacement care was considered the primary object of treatment.

THE JOURNAL OF BONE AND JOINT SURGERY


INJURIES TO THE TARSOMETATARSAL JOINT 351

Table I. Treatment and results weeks after injury. An arch support or surgical shoe was then fitted and
worn for three to six months. Most patients liked surgical shoes, but

Results few wore the supports.


Assessment. We graded results as good, fair or poor on the basis of
Type of injury Treatment Good Fair Poor residual pain, function, gait, deformity and radiological features.
‘Good” implied that symptoms were trivial or absent, the patient
A. Total Closed 9 I -

Operative reduction 6 3 3 could stand on tip-toe, had a normal gait, no deformity, and a
radiograph which showed at most minimal degeneration. “Fair”
B. Partial implied moderate pain on activity, difficulty standing on tip-toe, a
Medial Closed 14 4 -
limp, a foot of reasonably good shape, and slight to moderate
Operative reduction 2 3 -
radiographic evidence of degeneration. “Poor” implied marked pain
Lateral Closed 10 4 2 which affected the ability to walk, inability to stand on tip-toe, limp,
Operative reduction 4 - - deformity, and radiographic evidence of moderate to severe
degeneration.
C. Divergent Closed - - I
Operative reduction I I -
Obviously not all patients were in the same group for every one of
these features, so an overall assessment was used. Table I summarises

The patient with above-knee amputation has not been included in the relationship between our classification, the treatment used and the
these results. end result.

RESULTS
Of the I 19 patients, primary closed reduction was successful in 69;
Type A: Total (22 patients). There were 15 good, four fair
43 required open reduction (which, in 16 cases was performed 2 to 12
weeks after injury); and seven patients were left untreated. and three poor results in this group. In one case where
The subsequent management was in most instances the same, reduction was held in a plastercast without any Kirschner
whether reduction was by closed or open methods. A plaster cast, open wires, redisplacement occurred (Figs 3 to 10). In eight
anteriorly, was applied and the leg elevated. Two weeks later a below-
patients closed reduction was held with percutaneous
knee walking plaster was applied. Where internal fixation had been
used, progressive weight-bearing was allowed unless prevented by other Kirschner wires, and all obtained good results. Open
injuries. The cast and any Kirschner wires were removed six to eight reduction was performed in 12 patients ; three had fair

Fig. 7 Fig. 8
Figures 3 and 4 -Anteroposterior and lateral views of Type A with
lateral displacement found at operation. With only these views it is
difficult to define whether the displacement is plantar or dorsal. 5 and 6-Redisplacement
Figures is evident on the films in plaster. Figures
7 and 8- Anteroposterior and lateral radiographs demonstrating the ideal positioning of the two Kirschner wires with Type A injuries.
Figures 9 and 10-Three years after injury milddegenerative changes are seen in the entire Lisfranc joint.

VOL. 64-B. No. 3. 1982


352 P. H. HARDCASTLE, R. RESCHAUER, E. KUTSCHA-LISSBERG, W. SCHOFFMANN

and three had poor results. One patient with a fair result redisplacement after closed reduction without fixation
had redisplaced after closed reduction and plaster; open are the main causes of unsatisfactory results.
reduction was not performed until three months after Type B: Partial (43 patients). There were 30 good, 1 1 fair
injury. A second patient with a fair result had a total and two poor results. There was little difference between
medial and dorsal displacement with open fractures of those treated by closed and those treated by open
the cuboid bone; his postoperative course was compli- methods ; nor did the direction of displacement (plantar
cated by sepsis. The third patient with a fair result had or dorsal) affect the results. When the first metatarsal was
dorsolateral displacement with fractures of the second, openly reduced two patients had a good result and three
third and fourth metatarsals which were reduced, but fair. Two patients with fair results were men aged 48 in
only a single Kirschner wire was used for stabilisation; whom delayed open reduction and Kirschner wire
at review he had a cavus deformity with associated pain fixation had been performed two and eight weeks
in the Lisfranc joint and a limp. respectively after the injury (Figs 1 1 to 16 ; 17 to 22) ; both
Two of the poor results in this group were in patients soon redisplaced with consequent pain, mild deformity
who required early amputation of the forefoot for and severe osteoarthritis. The other patient had open
ischaemia. The third patient with a poor result had a reduction and stabilisation with a single Kirschner wire
total dislocation medially, with associated fractures of as a primary procedure ; however, at follow-up redisplace-
the navicular and medial cuneiform. Redislocation ment was evident.
occurred in plaster following the initial closed reduction Two patients with fair results had had a closed
and a second reduction was performed two weeks later; reduction held with a single percutaneous Kirschner
although this was anatomically accurate and was held wire; at follow-up there was clinical evidence of deform-
with percutaneous Kirschner wires, he developed severe ity, though when redislocation occurred is unknown.
degenerative changes. The two poor results, when only a single metatarsal
Comment. In this type of injury, forefoot ischaemia and was displaced (Type B lateral injuries), were treated by

Figures I 1 and I 2-Type B. There is medial displacement involving


the first three metatarsals. The diastasis between the first two
cuneiforms was due to interposition of the tibialis anterior tendon.
Note the large articular fragment from the first metatarsal (L).
Figures 13 and 14-Open reduction was necessary and the
anteroposterior and lateral radiographs show satisfactory alignment
held with Kirschner wires. Figures 15 and 16-At five years the
result is fair, with mild degenerative changes.

THE JOURNAL OF BONE AND JOINT SURGERY


INJURIES TO THE TARSOMETATARSAL JOINT 353

closed reduction and plaster. At review dorsal angulation ment, with or without fracture of the first metatarsal, is
was apparent at the fracture site and was associated with an unstable injury and redisplacement occurred unless
metatarsalgia. more than one stabilising wire was used (Figs 23 to 27).
Comment. Redisplacement in the lateral group resulted Type C: Divergent (three patients). Only three patients
from failure to use Kirschner wire stabilisers. Displace- with this rare injury were reviewed. One had a good

Fig. 19 Fig. 20

Figures 17 and 18 Type B. This patient had medial and plantar


displacement of the first metatarsal. Note the concomitant fractures
of the metatarsal shafts. Figures 19 and 20- Open reduction was
necessary because of inadequate closed reduction, and was stabilised
with Kirschner wires. Figures 21 and 22-Anteroposterior and
lateral radiographs four years later show mild degenerative changes
localised to the metatarsocuneiform joint, with malunion of the
fractures of the metatarsal shafts.

I,

Fig. 23 Fig. 24 Fig. 25 Fig. 26

I Fig. 27

Figures 23 and 24-Type B showing dislocation of the lateral three


metatarsals. The second metatarsal has not been displaced although this
was an indirect rotational injury. Figures 25 and 26-Position of the
percutaneous Kirschner wires following adequate reduction by closed
methods. Figure 27-Lateral photograph of the same foot showing the
deformity which may result ifdorsal displacement recurs.

VOL. 64-B. No. 3. 1982


354 P. H. HARDCASTLE, R. RESCHAUER, E. KUTSCHA-LISSBERG, W. SCHOFFMANN

Figures 28 and 29-Type C. This injury was due to an indirect force.


There is medial displacement of the first metatarsal and lateral
displacement of the second and third. Figures 30 and 31 Open
reduction was necessary. Good alignment was obtained and held by
internal fixation. Figures 32 and 33-Four years later the alignment of
the foot is good and degenerative changes only mild.

Fig. 34 Fig. 35 Fig. 36 Fig. 37


Figure 34-Type C. Direct injury (open). The first metatarsal was carried in by the patient. Concomitant injuries to the metatarsophalangeal
joints are also seen. Figure 35-After open reduction with osteosynthesis; a delayed split-skin graft was performed. Figures 36 and 37- Eighteen
months later a “fair” result has been achieved with revascularisation of the first metatarsal. The deformity of the four lateral metatarsals resulted
from inadequate reduction and stabilisation at the time of operation.

THE JOURNAL OF BONE AND JOINT SURGERY


INJURIES TO THE TARSOMETATARSAL JOINT 355

result the initial


; closed reduction was inadequate and redisplacement was diagnosed nine days after the initial
open reduction was performed two weeks after injury, closed reduction, was successfully treated by a further
when the swelling had subsided (Figs 28 to 33). The closed reduction and percutaneous wires. In these five
patient with a fair result (Figs 34 to 37) had a severe open cases, redisplacement had occurred before the patients
crushing injury with complete devascularisation of the had been allowed to take weight. A further five patients
first metatarsal, which the patient brought into hospital; with clinical and radiological evidence of redisplacement
open reduction, reimplantation and osteosynthesis with were seen, but it was not possible to ascertain when this
a small AO plate was performed. His postoperative had occurred. When Kirschner wires had been used to
course was uncomplicated but he developed a painful stabilise the reduction, redisplacement occurred only in
valgus deformity of the hallux. His final outcome is Type B medial injuries where the first metatarsal was
unknown as he was seen only a few weeks after an displaced (at least three cases) and in Type C injuries
operation to correct the hallux valgus. (two cases); in all these, only a single wire had been used
The third patient had a poor result. Closed reduction to stabilise the medial segment.
failed, but, at operation, there was marked comminution Kirschner wires are undoubtedly valuable as stabi-
involving the Lisfranc joint and an adequate reduction lisers. In their absence redisplacement may follow when,
was not obtained, so this injury was treated without as the swelling diminishes, the plaster becomes loose.
fixation. The strong plantar muscles and tendons bowstring across
DISCUSSION the tarsometatarsaljoint and, on contraction, shorten the
Since the time of Lisfranc, the causes of injury to the plantar aspect of the foot, leading to displacement if the
tarsometatarsal joint have changed ; road accidents and dorsal ligaments have been torn. This bowstring effect is
industrial injuries are now the commonest. The most less marked in the more lateral injuries where the
common segmental displacement in our series was one curvature of the Lisfranc joint is less. In those injuries
involving only the first metatarsal and resulting from a with plantar displacement the peroneus longus tendon
direct crushing force or an indirect pronation force acting acts as a deforming force, depressing the medial side of
on a fixed forefoot in equinus. This contrasts with the tarsometatarsal joint and elevating the lateral side.
previous reports where either dorsolateral displacement Osteoarthritis is an almost inevitable sequel of
of all five metatarsals (Aitken and Poulson 1963; tarsometatarsal displacements because of damage to the
Cassebaum 1963 ; Wilson 1972), or of the four lateral articular surface at the time of injury (Jeffreys 1963). It is
metatarsals (Wilppula 1973) were the commonest usually confined to the displaced segment and may be
injuries. associated with the formation ofan exostosis, particularly
Radiographs in three planes (anteroposterior, lateral on the dorsum of the foot. Moderate to severe degenera-
and 30-degree oblique) are essential in order to diagnose tive changes in the Lisfranc joint occurred in approxi-
the initial displacement and also to assess whether mately 30 per cent of our patients.
reduction is sufficiently accurate. Unhappily, in many of
our cases oblique views had not been taken. It is also Treatment
important to include the entire foot and ankle, otherwise With regard to treatment, we feel that, whenever there is
concomitant injuries may be missed. displacement, closed reduction should be attempted. It is
Our study defined three patterns of displacement best achieved by longitudinal traction in the line of the
and our classification is based on that ofQu#{233}nuand K#{252}ss foot and must be checked radiographically. Providing
(1909). This has, however, been expanded to include reduction is adequate it should be stabilised with
plantar and medial displacement injuries, and disloca- percutaneous wires. With Type A injuries (total incon-
tions where four metatarsals are involved. This modified gruity) one Kirschner wire is placed from the first
classification is simple to apply and gives information metatarsal to the medial cuneiform bone, and a second
regarding prognosis and treatment. placed laterally from the fifth metatarsal to the cuboid.
Of the Type B injuries (partial displacements) those of
Complications the lateral segment need only a single Kirschner wire.
The complications which accounted for most of the But if the first metatarsal is displaced, the injury is
unsatisfactory results were vascular impairment, redis- inherently unstable and two wires are needed. With Type
placement, skin complications (necrosis or cicatrisation C injuries(divergent displacements)one or two Kirschner
if transverse or plantar incisions were used), and wires are used to stabilise the medial fragment, with
osteoarthritis. another single wire for the lateral displacement.
At follow-up we were often unable to diagnose early It is clear from the results in our series that when an
redisplacement (within six weeks) because radiographs adequate reduction can be achieved by closed methods,
were not available. In four of the 27 patients reviewed open reduction, as advocated by some authors (Gissane
who required open reduction the operation became 1951 ; Jeffreys 1963; Engelhardt and Ganz 1975), is not
necessary because of redisplacement after the initial necessary. Early closed reduction is unlikely to succeed
closed reduction and plaster. A further case, in which where there is soft-tissue interposition, marked commi-

VOL. 64-B, No. 3, 1982


356 P. H. HARDCASTLE, R. RESCHAUER, E. KUTSCHA-LISSBERG, W. SCHOFFMANN

nution, or a large articular fragment. Diastasis between In several cases good results followed open reduction
the medial and intermediate cuneiform bones suggests up to six weeks after injury. However, the two patients
interposition of the tibialis anterior tendon. In such cases treated by open reduction and fixation after this period
and, indeed, whenever closed reduction is inadequate, had only fair results. It is probably better not to attempt
open reduction should be performed as soon as possible reduction after six weeks.
and certainly within six weeks of the injury. Primary arthrodesis is advocated by Granberry and
The absolute indication for open reduction is Lipscomb (1962) and by Bonnel and Barth#{233}l#{233}my (1976).
vascular insufficiency that does not improve after closed We have had no experience with this procedure but it is
reduction. We suggest the technique advocated by possible that it may have a place in those injuries where
Gissane 1 95 1 ), where
( both dorsalis pedis and posterior there is considerable comminution and where mainte-
tibial arteries are explored. In the two cases of forefoot nance of reduction by internal fixation is difficult.
ischaemia in our series, open reduction was performed Niederecker (1956) recommended arthrodesis of the
but the posterior tibial artery was not explored and metatarsocuneiform joint as a standard procedure in all
gangrene developed. dislocations involving the first metatarsal. Certainly in
For open reduction in the absence of ischaemia, the our series redisplacement sometimes occurred when only
longitudinal incisions advocated by Aitken and Poulson a single Kirschner wire was used, and we therefore
(1963) give adequate exposure. The first incision is recommend using two wires.
between the first and second metatarsals, and the second With regard to after-care, previous reports recom-
(if a second is needed) is made more laterally over the mend non-weight-bearing for a period of at least six
tarsometatarsal joint. With these incisions skin necrosis weeks after injury. We found, however, that taking
and contracture, which may follow transverse or plantar weight as soon as the swelling subsided (approximately
incisions, do not occur. A single midline incision also two weeks) did not seem to affect the final outcome,
may lead to necrosis because excessive skin retraction is provided the foot was protected in a plaster cast and
necessary to obtain access. reduction stabilised by internal fixation.

The authors would like to thank Professor Sydney Nade, Professor ofOrthopaedic Surgery, Queen Elizabeth Medical Centre. Nedlands, Western
Australia, for his assistance in preparing this paper.

REFERENCES

Aitken AP, Poulson D. Dislocation of the tarsometatarsaljoint. J Bone Joint Surg [Am] 1963 45-A : 246-60.
B#{246}hlerL. Dislocations in Lisfranc’s joint. In : Wallner A and Russe 0, translators. The treatment o/fracture.s, 5th English edition. New York:
Grune and Stratton, l958;3:213&-7.
Bonnel F, Barth#{234}l#{234}my
M. Traumatismes de l’articulation de Lisfranc : entroses graves, luxations, fractures : #{233}tudede 39 observations personnelle
et classification biomecanique. J Chir (Paris) 1976; 1 1 1 : 573-92.
Cassebaum WH. Lisfranc fracture-dislocations. C/in Orthop l963;30: 1 16-28.
Engelhardt P, Ganz R. Die Luxationsfrakturen des Lisfranc Gelenkes. Orthopadische praxis der haden badt’nr.s rei/ze 1975 9 : 896--8.
English TA. Dislocations of the metatarsal bone and adjacent toe. J Bone Joint Surg [Br] 1964:46-B :700-4.
Francesconi F. Sopra un caso di lussazione di Lisfranc. Chir Organi Moe 1925;9:589-604.
Gissane W. A dangerous type of fracture of the foot. J Bone Joint Surg [Br] 1951 :33-B: 535-8.
Granberry WM, Lipscomb PR. Dislocation of the tarsometatarsaljoints. Surg Gvneco/ Obstet 1962 ; 114: 467-9.
Jeifreys TE. Lisfranc’s fracture-dislocation : a clinical and experimental study of tarso-metatarsal dislocations and fracture-dislocations. J Bone
Joint Surg [Br] I963 :45-B: 546-51.
Niederecker K. Operative Behandlung der MittlefussknochenbrOche. Verh Dtsch Ges Orthop 1956:44: 358-63.
Qu#{234}nuE, KBss G. #{201}tudesur les luxations du metatarse (luxations m#{233}tatarsotarsiennes) du diastasis entre le I et le 2e metatarsien. Ret Chir I 909:
39: 281-336, 720-91, 1093-134.
Rainaut JJ, Cedard D, d’Hour JP. Les luxations tarsometatarsiennes. Rev Chir Orthop 1966:52:449-62.
Wiley ii. The mechanism of tarso-metatarsaljoint injuries. J Bone Joint Surg [Br] 1971 :53-B :474-82.
Wilppula E. Tarsometatarsal fracture-dislocation. Acta Orthop Scand 1973:44:335-45.
Wilson DW. Injuries of the tarso-metatarsal joints. J Bone Joint Surg [Br] 1972 54-B : 677-86.

THE JOURNAL OF BONE AND JOINT SURGERY


ORIGINAL RESEARCH

Effects of Isolated Weber B Fibular


Fractures on the Tibiotalar Contact Area
John Harris, DPM,1 and Lawrence Fallat, DPM, FACFAS2
Fractures of the lateral malleolus can occur without rupture of the deltoid ligament or fracture of the
medial malleolus. Controversy exists regarding the necessity of surgery on supination-external rotation
stage II ankle fractures. Theoretically, as long as the medial structures are intact, the talus cannot displace
enough to cause degenerative arthritis of the ankle joint. The purpose of this study was to measure
changes in contact area between the tibial plafond and the talar dome with serial displacement of the
distal fibula in both a lateral and a superolateral direction. Twelve cadaver lower extremities were used.
Distal fibular fractures were replicated by creating an osteotomy. Displacement was accomplished with
a customized apparatus that displaced and held the distal fibula in a malaligned position. Tibiotalar
contact area was measured with pressure sensitive film at the following intervals of fibular displacement:
0 mm, laterally 2 mm and 4 mm, and then posteriorly and superiorly 2 mm and 4 mm. A servohydraulic
testing apparatus was used to apply the same physiologic load to all limbs while measuring contact area.
Key independent variables included the direction and amount of displacement of the distal fibula. Mean
tibiotalar contact area decreased from baseline (no displacement) 361.1 mm2 (SD ⫾ 49.0) to 162.2 mm2
(SD ⫾ 81.3) and 82.6 mm2 (SD ⫾ 30.6) for 2 mm and 4 mm lateral displacement of the distal fibula
respectively. With posterior/superior displacement of 2 mm and 4 mm mean tibiotalar contact decreased
to 219.3 mm2 (SD ⫾ 56.7) and 109.2 mm2 (SD ⫾ 39.0), respectively. Statistical significance was found
( P ⬍ .001) when comparing normal ankle alignment with displaced fractures at all levels of displacement.
( The Journal of Foot & Ankle Surgery 43(1):3–9, 2004)

Key words: SER II fractures, tibiotalar contact area, Weber B fractures, ankle fracture

S upination-external rotation (SER) injuries of the ankle distal fibula. Ramsey and Hamilton (8) concluded that lat-
eral displacement of the talus can be significant enough to
comprise 40% to 55% of malleolar fractures (1). Approxi- alter tibiotalar joint dynamics in isolated fibular fractures
mately 85% of fractures of the lateral malleolus occur and an intact deltoid ligament, thereby supporting the need
without substantial injury to the medial side of the ankle for surgical repair. Curtis et al (17) also found a decrease in
joint (2, 3). There has been an extensive debate over
tibiotalar contact area with shortening and external rotation
whether or not surgical correction is needed for Lauge-
of the fibula with an intact deltoid ligament.
Hansen SER stage II ankle fractures (1, 4 –16). Many of
There are several reports that have shown that the tibio-
these studies focus on the need for anatomic reduction of the
talar contact area decreases only when the deltoid ligament
is severed (1, 7, 18). Michelson et al (18) reported that
From the Department of Podiatric Surgery, Oakwood Healthcare Sys- tibiotalar contact area does not significantly change with
tem, Dearborn, MI. lateral displacement of the fibula without sectioning the
1
Submitted During Second-Year Surgical Residency, Oakwood Health-
care System. deltoid ligament. Moody et al (1), while investigating SER
2
Diplomate, American Board of Podiatric Surgery; Director, Podiatric IV ankle fractures without medial malleolar fractures, found
Surgery Residency, Oakwood Healthcare System.
Address correspondence to: Lawrence Fallat, DPM, FACFAS, Director,
that tibiotalar contact area decreases with lateral shift of the
Podiatric Surgical Residency, Oakwood Healthcare System, 20555 Ecorse talus. Thordarson et al (7) found increased tibiotalar contact
Rd, Taylor, MI 48180. E-mail: lfallatdpm@aol.com pressures with shortening, lateral displacement, and external
Copyright © 2004 by the American College of Foot and Ankle Surgeons
1067-2516/04/4301-0002$30.00/0 rotation of the distal fibula after sectioning the deltoid
doi:10.1053/j.jfas.2003.11.008 ligament.

VOLUME 43, NUMBER 1, JANUARY/FEBRUARY 2004 3


On the other hand, there have been challenges to the
concept that ankle joint dynamics are altered after a distal
fibular fracture, with or without lateral displacement of the
talus (3, 10, 18 –29). These and other authors credit the
deltoid ligament for maintaining the position of the talus
when the fibula is fractured (18, 25–27, 29, 30). Some of
these studies show no significant change in contact pres-
sures with lateral displacement of the fibula (10, 20, 21, 23,
24).
The fundamental basis for examining tibiotalar contact
area is that, as the total contact area decreases, stress per
area increases (31). This contact area helps determine ef-
fects on ankle joint dynamics. Additional researchers have
investigated the contact area and pressures of the normal
ankle joint, which can serve as a baseline for comparison to
the pathologic or fractured ankle (32, 33). Given the dis-
crepancies regarding tibiotalar contact characteristics after FIGURE 1 Apparatus in place on cadaveric limb.
fracture of the fibula (8, 10, 17, 18, 20, 21, 24), this study
examines tibiotalar contact area with lateral and posterosu-
perior displacement of the distal fibula while the deltoid A saggital saw and a 2108-110 blade (Stryker, Kalamazoo,
ligament remains intact. MI) were used to create the simulated fracture. Osteotomies
were directed from anterior-distal, beginning at the level of
the ankle joint, to posterior-proximal. An apparatus was
Materials and Methods designed and built to precisely displace and to hold the
distal fibula in a malaligned position. The distal tibia was
Six paired fresh-frozen, above-knee cadaveric lower ex- firmly fixated to the device with 4 brass 5/16⬙ screws (Fig.
tremities were obtained for the study. The 12 specimens 1). A 5/64⬙ Steinmann pin was inserted through the distal
were thawed by placing them in a refrigerator at 4°C for 2 fibula from anterior to posterior, while either end of the pin
days. After the specimens were thawed, they were kept in was tightly fastened to the apparatus (Fig. 1). Two ma-
ice water for the duration of the study. Average age of the chined tracks in the apparatus ensured displacement in the
specimens was 72 years (range, 48 to 91 years; 4 women proper plane. Millimeter markings on the tracks confirmed
and 2 men). Exclusion criteria included radiographic and the amount of displacement along that track.
gross evidence of arthritis and/or previous trauma. Controlled variables were the direction and the amount of
All skin and soft tissue were removed from the anterior displacement of the distal fibula. Tibiotalar contact area was
and lateral ankle of each limb with the exception of tendons recorded sequentially at the 0-mm, 2-mm, and 4-mm inter-
and ligaments. The anterior joint capsule was dissected from vals of lateral fibular displacement. Contact area was also
its attachments on the anterior surface of the tibia, the determined at 2 mm and 4 mm of posterior/superior fibular
superior surface of the talus, and the medial and lateral displacement. The right specimen was tested first followed
malleoli. The deltoid ligament and the anterior talofibular by the left for each trial.
ligament were kept intact. A small window of skin and Tibiotalar contact area was measured with ultra–super-
subcutaneous tissue was also removed from the lateral mal- low Fuji (Fuji Photo Film Co, Tokyo, Japan) (0.5 MPa to
leolus, leaving a 9 ⫻ 5 cm rectangular void that was 6.0 MPa) pressure-sensitive film. Because this study exam-
centered over the lateral malleolus. The superior extensor ined contact area only, there was no need to use film that
retinaculum was separated from its attachment to the ante- measured higher pressures. The film was shaped in a man-
rior aspect of the lateral malleolus and the anterior tibiofib- ner to maximize contact between tibia and talus (1, 7, 10,
ular ligament was severed sharply as in a supination exter- 19, 32, 33). The film was then covered with 2 layers of thin,
nal rotation injury (4). The peroneal tendons, superior clear packing tape to keep cadaveric fluid from corrupting
peroneal retinaculum, calcaneofibular ligament, and poste- the film. The thickness of the film and tape together, mea-
rior talofibular ligament were all kept intact. sured with digital calipers, was approximately 0.27 mm.
An osteotomy was performed in the proximal tibia per- The film was inserted into the ankle anteriorly between
pendicular to the long axis of the tibia and 1.5 to 2 cm distal extensor hallucis longus and extensor digitorum longus.
to the articular surface. Through the window made over the While the limb was being loaded, the film was held in place
lateral malleolus, an osteotomy was created in the distal posteriorly by the posterior ankle joint capsule, medially
fibula to replicate and oblique SER stage II fibular fractures. and laterally by folding into the medial and lateral gutters of

4 THE JOURNAL OF FOOT & ANKLE SURGERY


TABLE 1 Mean tibiotalar contact area (mm2) by degree and
direction of displacement

Positions of Fibular Displacement Mean SD


Baseline (0 mm) 361.1 49.0
2 mm posterior/superior 219.3 56.7a
4 mm posterior/superior 109.2 38.9a
2 mm lateral 162.2 81.3a
4 mm lateral 82.6 30.6a
a
Statistically significant change from baseline.

tibia was held in the crosshead with three 1 ⫻ 1 ⫻ 5 inch


aluminum blocks cemented to the crosshead plate (Fig. 3).
The crosshead on the tensiometer was lowered into po-
sition at a manually controlled rate, not exceeding 5 mm/
min, until a static force of 686 N was observed. The load
was manually decreased back to 0 at the same rate. The limb
was removed and the film was retrieved and immediately
photographed and stored for future measurement. The pho-
tographs were taken with the film placed in a grid, with the
camera centered directly over the grid. Each pressure film
was photographed on a separate piece of numbered paper. A
separate log was kept of each limb, along with amount and
direction of displacement correlating to the numbered pres-
sure film. This insured no bias when measuring surface area
on each film. This exact technique was used for each limb
with each fibular displacement. All testing was performed
by the same individual (J.H.).
FIGURE 2 Saw bone model showing film insertion. Data were analyzed by using 2 repeated-measures anal-
ysis of variance (posterosuperior and lateral displacement)
by using SPSS (SPSS, Chicago, IL). All post-hoc compar-
isons used a Bonferroni correction for inflated error rate

Results

Descriptive statistics for contact area by type and degree


of displacement are presented in Table 1 and Fig 4. The
overall change in mean contact area for posterosuperior
displacement was significant (P ⬍ .001). Post hoc compar-
ison of mean contact area change from baseline to 2-mm
posterosuperior displacement (361 ⫾ 49 vs. 219.3 ⫾ 56.7)
was highly significant (P ⬍ .001) as was 4-mm displace-
ment (P ⬍ .001) (361 ⫾ 49 vs. 109.2 ⫾ 38.9). The change
in contact area from 2-mm to 4-mm posterosuperior dis-
placement was also significant (P ⬍ .001).
A similar pattern was found for lateral displacement by
FIGURE 3 Apparatus on limb, with limb in the Instron tensiometer.
using the same data analytical strategy. Lateral displace-
ment of the distal fibula on tibiotalar contact area showed an
the ankle, and anteriorly by the hemostat used to place the overall decrease (P ⬍ .001). Post hoc comparisons of con-
film into the joint (Fig. 2). tact area change from baseline for lateral displacement at 2
The limb was then loaded with an Instron model 5866 mm (P ⬍ .001) (361 ⫾ 49 vs. 162.2 ⫾ 81.3) and 4 mm (P ⬍
servohydraulic tensiometer (Instron, Canton, MA) with the .001) (361 ⫾ 49 vs. 82.6 ⫾ 30.6), respectively. The change
ankle in neutral position and the limb perpendicular to the from 2-mm to 4-mm lateral displacement was also signifi-
supporting surface to minimize slippage. Proximally, the cant (P ⬍ .001).

VOLUME 43, NUMBER 1, JANUARY/FEBRUARY 2004 5


FIGURE 4 Mean surface area (mm2).

Discussion loading of the ankle joint allows for decreased tibiotalar


contact area when the lateral bony support is absent.
Baseline contact area between all limbs was similar and This decrease in contact area gives an increase in force
was comparable to other studies of tibiotalar contact area per unit area (31). Generation of high-stress concentrations
without pathology (1, 32). Lateral displacement of the fibula can damage not only articular cartilage but also osseous
resulted in a slightly greater decrease in contact area than components of the distal tibia, diminish function through
did posterosuperior displacement. Our data indicate that loss of congruence, and may be considered a decisive factor
lateral displacement of only 2 mm results in a 55% decrease in the pathogenesis of osteoarthritis (34, 35). Changes in
in contact area and lateral displacement to 4 mm results in joint alignment may lead to degenerative changes and ankle
a 77% decrease (Fig. 5). Two millimeters of posterosuperior pain (36, 37). Indeed, several clinical investigators have
displacement yields a 39% decrease in contact area, and, at shown radiographic signs of ankle osteoarthritis in ankle
4 mm, shows a 70% decrease. A nearly linear relationship malunions with greater than 1 or 2 mm of lateral talar shift
exists between increasing fibular displacement and decreas- (12, 15, 38 – 40). Others have stated that the major indica-
ing amount of tibiotalar contact area (Fig. 4). These findings tion for open reduction with internal fixation of fibula frac-
support prior studies that show a decrease in tibiotalar tures is lateral shift of the talus of 2 mm or greater (12–16).
contact area with SER II ankle fractures (8, 17). Our results support the notion that open reduction with
This study clearly shows that tibiotalar contact area de- internal fixation be preformed on SER II ankle fractures
creases with simulated SER II ankle fractures despite intact with 2 mm or more of lateral and/or posterior and superior
superficial and deep fibers of the deltoid ligament and all displacement of the fibula. This investigation differs from
other medial structures. A recent study reported that axial previous studies in several key areas. In our study, tissue
loading stabilizes the ankle joint and prevents lateral dis- dissection was minimal and only involved the lateral and
placement of the talus, thereby maintaining tibiotalar con- anterior ankle. All ligaments and tendons that would be
tact area (23). In contrast, this study shows physiologic axial intact after a SER II ankle fracture were kept intact to

6 THE JOURNAL OF FOOT & ANKLE SURGERY


FIGURE 5 Percent of tibiotalar contact area lost.

maintain the structural integrity of the limb. Other studies into contact with the cadaveric specimen only on the tibia
reviewed removed all soft tissue with the exception of and the fibula. This apparatus held the distal fibula in a fixed
ligaments from their cadaveric specimens (7, 10, 17–24, 31, space, in relation to the rest of the limb, during axial loading
33, 41). in an attempt to replicate actual conditions as close as
Effects on tibiotalar contact area of simultaneous dis- possible. This most likely resulted in a more realistic stres-
placement posteriorly and superiorly of the fibula have not sor.
been studied. In contrast, other studies (7, 10, 17, 24) There are several limitations in this study. First, the
examined just shortening of the fibula, without posterior sample size of 12 specimens was relatively small. Never-
displacement as this study did. The displacement of the theless, our findings were highly statistically significant.
fibula in 2 planes, as in this study, verses 1 affords a There is an inherent degree of imprecision when dealing
possible difference in results between this study and others with cadavers and fitting of pressure film in the joint;
(7, 10, 24). Lateral displacement alone had an increased loss therefore, some variability is unavoidable. Regardless, we
of tibiotalar contact area compared with the posterior and feel this study was accurate enough to indicate that displace-
superior displacement without lateral displacement. Perhaps ment of the lateral malleolus both laterally and posterosu-
with lateral displacement of the fibula, the eventual restrain- periorly results in significant loss of ankle joint contact area
ing mechanism of displacement of the talus becomes the with the deltoid ligament intact.
deltoid ligament. There are no osseous structures that phys- Second, true spiral oblique fractures are unable to be
ically block lateral displacement of the talus. With pure replicated with a saw. As such, the difference between the
superior displacement only, there may still be some residual experimentally induced and the clinical fracture configura-
impairment of lateral talar shift. tions cannot be determined.
Also, the apparatus that was designed and built for this Last, we tried to minimize slippage by ensuring that the
investigation held the distal fibula in a malaligned position, limb was perpendicular to the supporting surface. However,
allowing for axial loading of the limb without artificially limb migration in increments not visible to the naked eye
inducing the talus to displace. Artificially inducing the talus could have occurred and modified the results.
to move laterally is quite often considered a shortcoming of Future areas of research may involve measuring the ac-
other studies (1, 8). The apparatus used in this study came tual stresses such as shear stresses within the pathologic

VOLUME 43, NUMBER 1, JANUARY/FEBRUARY 2004 7


ankle joint. Instability may result in shear stresses that may tion of adult ankle fractures: rational, technique, and early results.
be more damaging to cartilage than elevated contact stresses Orthop Clin North Am 11:661– 679, 1980.
15. Pettrone FA, Gail M, Pee D, Fitzpatrick T, Van Herpe LB. Quantita-
(19). There are also discrepancies in the literature as to tive criteria for prediction of the results after displaced fracture of the
which side of the joint should be examined (42). ankle. J Bone Joint Surg 65A:667– 677, 1983.
16. Phillips WA, Schwartz HS, Keller CS, Woodward HR, Rudd WS,
Spiegel PG, Laros GS. A prospective, randomized study of the
Summary management of severe ankle fractures. J Bone Joint Surg 67A:67–
78, 1985.
Findings of this study show that lateral and posterosupe- 17. Curtis MJ, Michelson JD, Urquhart MW, Byank RP, Jinnah RH.
rior displacement of the lateral malleolus significantly affect Tibiotalar contact and fibular malunion in ankle fractures. Acta Orthop
Scand 63:326 –329, 1992.
ankle joint dynamics and allow for decrease in tibiotalar 18. Michelson JD, Clarke HJ, Jinnah RH. The effect of loading on tibio-
contact area. This may have in important role in the man- talar alignment in cadaver ankles. Foot Ankle Int 10:280 –284, 1990.
agement of SER II ankle fractures. 19. Vrahas M, Fu F, Veenis B. Intraarticular contact stresses with simu-
lated ankle malunions. J Orthop Trauma 8:159 –166, 1994.
20. Clarke HJ, Michelson JD, Cox QG, Jinnah RH. Tibiotalar stability in
Acknowledgment bimalleolar ankle fractures: a dynamic in vitro contact area study. Foot
Ankle Int 11:222–227, 1991.
21. Kimizuka M, Kurosawa H, Fukubayashi T. Load-bearing pattern of
The authors thank Steven Schwartz, PhD, for help with
the ankle joint. Contact area and pressure distribution. Arch Orthop
statistical analysis and Kurt Miller and Larry Maciag from Trauma Surg 96:45– 49, 1980.
Axel Products (Ann Arbor, MI) for help with design and 22. Yablon IG, Heller FG, Shouse L. The key role of the lateral malleolus
testing of the apparatus. in displaced fractures of the ankle. J Bone Joint Surg 59A:169 –173,
1977.
23. Sasse M, Nigg BM, Stefanyshyn DJ. Tibiotalar motion— effect of
References fibular displacement and deltoid ligament transaction: in vitro study.
Foot Ankle Int 20:733–737, 1999.
24. Zindrick MR, Knight GW, Gogan WJ, Patwardhan AG, Vanderby R,
1. Moody ML, Koeneman J, Hettinger E, Karpman RR. The effects of
fibular and talar displacement on joint contact areas about the ankle. Hoyt JS. The effect of fibular shortening and rotation on the biome-
Orthop Rev 21:741–744, 1992. chanics of the talocrural joint during various stages of stance phase.
2. Michelson JD, Helgemo SL. Kinematics of the axially loaded ankle. Orthop Trans 8:259 –260, 1984.
Foot Ankle Int 16:577–782, 1995. 25. Pankovich AM. Fractures of the fibula proximal to the distal tibiofib-
3. Harper MC. The short oblique fracture of the distal fibula without ular syndesmosis. J Bone Joint Surg 60A:221–229, 1978.
medial injury: an assessment of displacement. Foot Ankle Int 16:181– 26. Pankovich AM, Shivaram MS. Anatomical basis of variability in
186, 1995. injuries of the medial malleolus and the deltoid ligament. Acta Ortho
4. Lauge-Hanson N. Fractures of the ankle. II. Combined experimental- Scand 50:225–236, 1979.
surgical and expalimental-roentgenologic investigation. Arch Surg 60: 27. McCullough CJ, Burge PD. Rotatory stability of the load-bearing
957–985, 1950. ankle. An experimental study. J Bone Joint Surg 62B:460 – 464, 1980.
5. Kleiger B. The treatment of oblique fractures of the fibula. J Bone Joint 28. Yde J, Kristensen KD. Ankle fractures: supination-eversion fractures
Surg 43A:969 –979, 1961. stage II primary and late results of operative and non-operative treat-
6. Magnusson R, On the late results in non-operative cases of malleolar ment. Acta Orthop Scand 51:695–701, 1980.
fractures. I. Fractures by external rotation. Acta Chir Scand 90(Suppl 29. Burns WC, Prakash K, Adelaar R, Beaudion A, Krause W. Tibiotalar
84):11–116, 1944. joint dynamics: indications for the syndesmotic screw, a cadaver study.
7. Thordarson DB, Motamed S, Hedman T, Ebramzadeh E, Bakshian S. Foot Ankle Int 14:153–158, 1993.
The effect of fibular malreductionon contact pressures in an ankle 30. Michelson JD. Current concepts review fractures about the ankle.
fracture malunion model. J Bone Joint Surg 79A:1809 –1815, 1997. J Bone Joint Surg 77A:142–152, 1995.
8. Ramsey P, Hamilton W. Changes in tibiotalar area of contact caused 31. Gere JM, Timoshenko SP Axially loaded members, chapter 2. In
by lateral talar shift. J Bone Joint Surg 58A:356 –357, 1976. Mechanics of Materials, 3rd ed, PWS Publishing, Boston, 1990.
9. Maynou C, Lesage P, Mestdagh H, Butruille Y. Is surgical treatment 32. Driscoll HL, Christensen JC, Tencer AF. Contact characteristics of the
of deltoid ligament necessary in ankle fractures? Rev Chir Orthop ankle joint. Part 1: the normal joint. J Am Podiatr Med Assoc 84:491–
Reparatrice Appar Mot 83:652– 657, 1997. 498, 1994.
10. Yoshimine F. Effects of fibular malunion on contact area and stress 33. Hartford JM, Gorezyea JT, McNamara JL, Mayor MB. Tibiotalar
distribution at the ankle with six simulated loading conditions. Nippon contact area contribution of posterior malleolus and deltoid ligament.
Seikeigeka Gakkai Zasshi 69:460 – 469, 1995. Clin Ortho Rel Res 320:182–187, 1995.
11. DeSouza LJ, Gustilo RB, Meyer TJ. Results of operative treatment of 34. Lieberman DE, Devlin MJ, Pearson OM. Articular area responses to
displaced external rotation-abduction fractures of the ankle. J Bone mechanical loading: effects of exercise, age, and skeletal location.
Joint Surg 67A:1066 –1073, 1985. Am J Phys Anthropol 116:266 –277, 2001.
12. Joy G, Patzakis MJ, Harvey JP Jr. Precise evaluation of the reduction 35. Glaser C, Putz R. Functional anatomy of articular cartilage under
of severe ankle fractures. Technique and correlation with end results. compressive loading: quantitative aspects of global, local and zonal
J Bone Joint Surg 56A:979 –993, 1974. reactions of the collagenous network with respect to the surface
13. Cedell CA. Is closed treatment of ankle fractures advisable? Acta integrity. Osteoarthritis Cartilage 10:83–99, 2002.
Orthop Scand 56:101–102, 1985. 36. Trias A. Effect of persistent pressure on the articular cartilage: an
14. Mast JW, Teipner WA. A reproducible approach to the internal fixa- experimental study. J Bone Joint Surg 43B:376 –386, 1961.

8 THE JOURNAL OF FOOT & ANKLE SURGERY


37. Wagner KS, Tarr RR, Resnick C, Sarmiento A. The effect of simulated 40. Brunnel HN, Charnley AD. The treatment of displaced fractures at the
tibial deformities on the ankle joint during gait cycle. Foot Ankle ankle by rigid internal fixation and early joint movement. J Bone Joint
5:131–141, 1984. Surg 47B:634 – 660, 1965.
38. Hughes JL, Weber H, Willenegger H, Kaner EH. Evaluation of ankle 41. Kura H, Kitaoka HB, Luo ZP, An KN. Measurement of surface contact
fractures: non-operative and operative treatment. Clin Orthop 138: area of the ankle joint. Clin Biomech 13:365–370, 1998.
111–119, 1979. 42. Michelson JD, Checcone M, Kuhn T, Varner K. Intra-articular load
39. Wilson FC, Skilbren LA. Long-term results of displaced bimalleolar distribution in the human ankle joint during motion. Foot Ankle Int
fractures. J Bone Joint Surg 48A:1065–1078, 1966. 22:226 –233, 2001.

VOLUME 43, NUMBER 1, JANUARY/FEBRUARY 2004 9


Tibialis Posterior Tendon Dysfunction
KENNETHA. JOHNSON,M.D., AND DAVIDE. STROM,M.D.

Dysfunction of the tibialis posterior tendon evolves casions, have only an aching along the medial
through a series of stages. The pain symptoms, aspect of the ankle that is exacerbated by
clinical signs, and roentgenographic changes for physical activity and has probably modified
each of these stages are characteristic. This stag-
ing system permits clarificationand individualiza- his or her activities to be less strenuous. Al-
tion of dysfunction, expected pathologic changes, though it may be difficult for patients to local-
and surgical treatment. The importance of the tibi- ize the discomfort, with specific questioning
alis posterior tendon in normal hindfoot function they will point along the course of the TPT
and its treatment when injured are now being prop- from a few centimeters proximal to the tip of
erly appreciated.
the medial malleolus to its major attachment
Afflictions of the tibialis posterior tendon at the undersurface of the navicular. The on-
(TPT) are now being recognized and treated set of pain will have been gradual, and only
with increased frequency and ',I2 infrequently can an inciting episode be re-
It is important to be aware of this problem called. Occasionally, a young athlete will re-
and its stages of presentation in order to make member a twisting episode with subsequent
a proper diagnosis. This article reviews the persistent pain.
evolving concepts of presentation and formu- On examination, the points of maximal
lates a plan of rational care. tenderness will be detected along the tendon
from just before it passes around the medial
DIAGNOSTIC AND RADIOGRAPHIC malleolus to its navicular insertion. When lo-
CONSIDERATIONS calized, the site of tenderness corresponds
The stages of TPT dysfunction are quite well to areas of TPT pathologic changes.
distinct (Table 1). Each stage will be discussed Swelling is best appreciated by viewing the
with regard to pain symptoms, physical find- standing patient from a posterior vantage.
ings, and roentgenographic changes. Fullness of the region just inferior to the me-
dial malleolus is evident when compared
STAGE1-TENDON LENGTHNORMAL with the unaffected foot. The alignment of
Many TPT problems remain unrecognized the hindfoot-forefoot will still be normal at
because pain symptoms are only mild to this stage.
moderate. The patient may, on multiple oc- In the authors' experience, manual testing
for weakness has been essentially useless. It
has been suggested2that testing the TPT with
Dr. Johnson is Consultant in Orthopedics, Mayo the hindfoot in eversion and the forefoot in
Clinic Scottsdale, Scottsdale, Arizona, and Professor of abduction with toe flexors relaxed will elimi-
Orthopedic Surgery, Mayo Medical School, Rochester,
Minnesota. nate the synergistic action of the tibialis ante-
Dr. Strom is a Fellow in Foot Surgery, Mayo Clinic rior and flexor digitorum longus, thus allow-
Scottsdale, Scottsdale, Arizona. ing determination of the strength of the TPT.
Reprint requests to Kenneth A. Johnson, M.D., Mayo
Clinic Scottsdale, 13400 East Shea Blvd., Scottsdale, AZ However, because the TPT power is so
85259. strong, and probably because of substitution

196
Number 239
February, 1989 Tibialis Posterior Tendon Dysfunction 197

TABLE 1. Changes Associated With Various Stages of TPT Dysfunction


Stage 1 Stage 2 Stage 3

TPT condition Peritendinitis andfor Elongation Elongation


tendon degeneration
Hind foot Mobile, normal Mobile, valgus position Fixed, valgus position
alignment
Pain Medial: focal, mild to Medial: along TPT, Medial: possibly
moderate moderate lateral, moderate
Single-heel-risetest Mild weakness Marked weakness Marked weakness
“Too-many-toes’’ sign Normal Positive Positive
with forefoot
abduction
Pathology Synovial proliferation, Marked degeneration Marked degeneration
degeneration
Treatment Conservative, 3 months; Transfer FDL* for Subtalar arthrodesis
surgical, 3 months TPT
with synovectomy,
tendon debridement,
rest

* Flexor digitorurn longus.

by the other extrinsic muscle-tendon units in much in the way of secondary deformity, and
spite of efforts to negate them, this method of overall forefoot-hindfoot alignment will be
testing has not been accurate when correlated unaffected. On routine standing roentgeno-
with surgical findings. grams the changes will be minimal. If the di-
The single-heel-rise test is very good for agnosis is in question, magnetic resonance
testing TPT strength. The patient is asked to imaging (MRI) may demonstrate if tendon
rise up on the ball of the foot while the other degeneration is present.
foot is held off the ground (the patient may
use a door or wall for balance). The normal STAGE 2-TENDON ELONGATED,
sequence for a single-heel rise is as follows. HINDFOOTMOBILE
First, the TPT is activated, which inverts and
locks the hindfoot, thus providing a rigid From Stage 1 to Stage 2 the change in
structure. Next, the gastrosoleus muscle symptoms evolves over several months to
group pulls up the calcaneus and the heel rise years. During Stage 1, when pain is mild to
is completed. With elongation of the TPT, moderate, patients have often seen a physi-
however, the initial heel inversion is weak cian but were not given a specific diagnosis.
and the patient either rises up incompletely Often these patients have been told to live
without locking the heel or does not get up on with the discomfort. In Stage 2, the pain in-
the ball of the foot at all (Figs. 1A and 1 B). In creases in seventy and distribution, is present
the Stage 1 condition, the patient usually will even after cessation of weight bearing, and is
be able to get up on the ball of the affected significantly troublesome. The patient has al-
foot and will be able to tell that it is more ready applied for a handicapped parking
painful and somewhat weaker than the other sticker and actively wants relief. Pain is lo-
side. At this early stage, there will not be cated along the TPT for a greater length. The
ClinicalOrthopaedics
198 Johnson and Strom and Related Research

FIGS.1A AND 1B. The single-heel-rise test. (A) Normal. The TPT inverts the hindfoot and the patient
rises on the forefoot. (B) Abnormal. Ability to rise on the forefoot is decreased or absent. Instead, the
patient just rolls to the outside ofthe foot. (By permission of Mayo Foundation.)

tendon has been disrupted, and secondary a direct posterior midline vantage, the exam-
changes are developing. Swelling with tender- iner counts the number of toes on each foot
ness is still present inferior to the medial mal- that are visible laterally. As the heel goes into
leolus when viewed posteriorly. increased resting eversion and the forefoot
The single-heel-rise test becomes even goes into abduction, too many toes are seen
more abnormal because the tendon is weak- on the affected side. The number of extra toes
ened. Another helpful diagnostic sign is that seen is a recordable measurement of the de-
of “too many toes,” i.e., seeing more toes gree of deformity and is surprisingly repro-
than normal from this view (Fig. 2). In this ducible.
test, the patient is asked to assume a comfort- Changes on routine standing roentgeno-
able knee-leg alignment toward a wall. From grams now become evident. To understand
these changes, it is useful to think of the foot
as consisting of only two pieces (Figs. 3A-
3C). One piece, the talus, is fixed in the ankle
mortise. It can move only in flexion-exten-
sion, not in a varus-valgus plane. The second
piece of the foot is everything else, i.e., the
calcaneus, cuboid, navicular, and bones dis-
talward move as a unit with motion being in
a varus-valgus plane through the subtalar
joint. With elongation ofthe TPT, the second
piece rotates from beneath the talus laterally
to produce a hindfoot in valgus and a forefoot
in abduction. When the head of the talus is
left unsupported, the talus flexes.
The changes on routine roentgenograms
FIG.2. “Too many toes.” This patient had right are then predictable. On the anteroposterior
TPT dysfunction. Four toes show on the right with
forefoot abduction, but only two toes show on the (AP) view, the forefoot will be abducted in re-
normal left foot. (From Johnson.* By permission lation to the hindfoot. Also, the navicular will
of J. B. Lippincott.) have subluxed off the head of the talus and
Number 239
February, 1989 Tibialis Posterior Tendon Dysfunction 199

A
Frontal View
B I Superior Vlew I
I FIGS. 3A-3C. Two-piece
concept of foot. (A) Talus is

II i
!
held in the ankle mortise while
the calcaneus rotates laterally
because of loss of TPT func-
tion. (B) The second piece (cal-
caneus, cuboid, navicular, and
bones distalward) moves to
produce forefoot abduction.
(C) As the sustentaculum tali of
the calcaneus moves from be-
neath the talus, the talus rotates
A
into a plantar-flexed position.
B

Lateral Vlew
C

the angle between the long axes of the talus helpf~l.~.In the authors' experience, however,
and the calcaneus will be increased. In a lat- this is not the case. Either the diagnosis is so
eral view, there will be sagging at the talus- evident that it is not necessary or in cases of
navicular joint and divergence of the long difficult diagnosis the tenogram is equivocal.
axis of the talus from the long axis of the cal-
caneus, STAGE 3-TENDON ELONGATED,
If the diagnosis is in question, MRI' can HINDFOOT AND STIFF
DEFORMED
show a tendon discontinuity as well as a prox-
imal balling-up ofthe tendon (Fig. 4). At first, The pain may transfer to the lateral aspect
it might be thought that a tenogram would be of the hindfoot and be located over the sinus
Clinical Orthopaedics
200 Johnson and Strorn and Related Research

margin of the posterior facet will impinge on


the superior aspect of the calcaneus in the si-
nus tarsi. Pressing on the sinus tarsi will re-
produce the patient’s pain symptoms. In
Stage 3, the pain also may be more suggestive
of degenerative arthritis, exhibiting an activ-
ity-related sharp pain that is present long after
the activity ceases.
Deformity is the most prevalent change.
When viewed posteriorly, the fullness be-
neath the medial malleolus may not be so evi-
dent, but the hindfoot eversion and forefoot
abduction are significant. The single-heel-rise
test will demonstrate the absence of locking
hindfoot inversion along with a diminished
ability to rise on the ball of the foot. Also, too
FIG.4. MRI shows the balled-up tendon (arrow) many toes will be seen. At this stage, the foot
just behind the medial malleolus. (From Alexan-
der ef al.’ By permission of the American Ortho-
appears severely flat (Fig. 5).
paedic Foot and Ankle Society.) The hindfoot valgus and forefoot abduc-
tion will be seen on standing AP and lateral
tarsi because the tendon is now completely roentgenograms, much the same as in Stage
disrupted and intrinsically less painful. This 2 but more marked. Secondary degenerative
has allowed the hindfoot to go into eversion. changes with joint narrowing and osteophyte
As this occurs, the bone projection from the formation may also appear in the subtalar, ta-
inferior surface of the talus at the anterior lonavicular, and calcaneocuboid joints.
If that anterior margin of the posterior
facet projecting inferiorly off the talus is im-
pinging on the superior aspect of the calca-
neus, sclerosis will appear in the midsuperior
calcaneus at the site of impingement. This
bony condensation is called the “sinus tarsi
impingement sign.”
If a technetium scan is done, the delayed
views will show uptake at the site of sinus tarsi
impingement as well as at sites of secondary
degenerative arthritis.
PATHOLOGIC AND TREATMENT
CONSIDERATIONS
With an understanding of the diagnostic
signs and symptoms along with roentgeno-
graphic abnormalities as presented above, the
pathologic changes can be anticipated and a
treatment program offered.

STAGE1-TENDON LENGTHNORMAL
FIG.5 . Acquired pes planus as a late result of
TPT rupture. (From Johnson.*By permission of At this stage,
- the length of the tendon ap-
J. B. Lippincott.) pears normal. This early presentation, how-
Number 239
February, 1989 Tibialis Posterior Tendon Dysfunction 201

ever, is the most varied and hardest to de- months, trials of antiinflammatory agents,
scribe and explain. In some instances, there is rest, arch supports, and perhaps medial shoe
almost a pure peritendinitis. That is, when wedges or orthotics may be appropriate. Be-
the TPT retinaculum is opened, much clear cause steroid injection has been implicated in
amber synovial fluid will escape.I3 The ten- , ~use is not suggested. If
tendon ~ e a k e n i n gits
don itself will look and feel essentially nor- improvement does not occur, then the surgi-
mal, but a luxuriant synovial proliferation cal treatment just outlined should be used to
will be present within the tendon sheath. In break the inflammation cycle. This applies
other cases, there will be minimal synovial also to patients with rheumatoid arthritis in
fluid and synovial thickening; however, in whom loss of TPT function and subsequent
this case the tendon, in a location just distal deformity can be as disabling as some large
to the medial malleolus, will be firm and have joint involvement.
an off-white color (Fig. 6A). There may be
some longitudinal split tears within the ten-
don substance, and it may be enlarged in a STAGE 2-TENDON ELONGATED,
bulbous configuration to almost twice the HINDFOOTMOBILE
normal tendon width.” This would be a pure
tendon degeneration. Combinations of peri- In this situation, the tendon may show
tendinitis and degeneration may also be seen. marked degeneration over several centime-
Still, the tendon length appears normal. It is ters. The tendon is enlarged and has multiple
unclear whether the different presentations longitudinal tears with secondary adhesions
represent different causes, e.g., peritendinitis to the tendon sheath. It will be a varied yel-
from systemic inflammatory disease or de- lowish white-brown color and firm in consis-
generation from acute traumatic interstitial tency. Proximal to the directly involved re-
tears. gion, the tendon will have a peculiar white,
The current treatment in this situation fish-flesh appearance if the tear is old and ten-
(Fig. 6B) is to open the sheath from the TPT sion has not been transmitted through the
musculotendinous junction all the way to its tendon for some time. In other situations,
insertion, leaving only a 1-cm pulley just pos- there will be a single complete transverse tear
terior to the medial malleolus.18 Synovec- of the tendon with rounding off of the tendon
tomy is performed and the tendon is de- ends. The tendon that remains will then have
brided. If the tendon is enlarged to more than the off-white, fish-flesh appearance.
1.5 times its normal size, a wedge is removed When the TPT has been elongated, the
from the substance to debulk the tendon and flexor digitorum longus (FDL) is transferred
the gap is sutured closed.” Small flap tears to substitute for the TPT.4,6,8*’2 This transfer
are debrided and larger tears are sutured. As entails detaching the FDL distally and rein-
the wound is closed, some betamethasone is serting it into the undersurface of the navicu-
left around the tendon. A short-leg walking lar through a drill hole (Figs. 7A-8G).
cast is then applied and worn for three weeks. The distal portion of the FDL does not
This treatment method seems to be suc- need to be tenodesed to the adjacent flexor
cessful in stopping the inflammation and ten- hallucis longus tendon. The intrinsic toe
don degeneration that would otherwise pro- flexors are So good in the foot that leaving the
ceed to Stage 2 changes. In a few cases, excis- distal stump alone will cause no functional
ing or reattaching the tendon was not loss in lesser toe function later. Avoiding this
~atisfactory.~ tenodesis also allows a greater length of FDL
It is important that the patient with Stage to be used for transfer.
1 physical findings does not progress to Stage Should the FDL be left in its own sheath
2 or 3 with conservative care. For three to six or rerouted through the flexor hallucis longus
Clincal Onhopaedlcs
202 Johnson and Strom and Related Research

FIGS.6A AND 6B. Medial view of right foot. (A) TPT is exposed, leaving a pulley just posterior to the
medial malleolus. The off-white discoloration of the tendon just distal to the medial malleolus is seen.
(B) At completion of the surgical procedure, the tendon sheath has been released and synovectomy is
completed.
Number 239
February, 1989 Tibialis Posterior Tendon Dysfunction 203

sheath? It does not make sense to reroute a


healthy tendon through a diseased tendon
sheath. The FDL tendon is probably strong - -TlBlALlS P0STERK)R
enough for substitution. The anatomic cross- - - FLEXOR DlGlTORUM LONGUS
sectional area of the FDL is only about one-
- - FLEXOR HALLUCIS LONGUS
third that of the TPT,I5 but it seems to be
holding up well clinically. Perhaps it hyper-
trophies.
The FDL should be pulled as tight as rea-
sonably possible. With the ankle in equinus
and the forefoot in varus, the tendon is pulled
quite taut and sutured distally. (There is no
anatomic rationale for this but it seems to
work.)
A proximal attachment of the TPT muscle
should not necessarily be made to the FDL.
If, when the proximal stump of the TPT is -1IBIALIS POSTERIOR
pulled, the muscle seems to be nonyielding - FLEXOR DlGlTORUM LONGUS
and fibrotic, the proximal attachment is not
done because the muscle is nonfunctional.
A soft-tissue static transfer is also usually
unnecessary. Although some authors6 have
advocated reefing the talonavicular capsule
and the calcaneonavicular (spring) ligament,
this has not been done as a part of the repair
procedure. Suturing of the FDL to the under-
surface of the talonavicular region may ac-
complish this to some degree. Static transfers B
alone, historically, have not been successful FIGS.7A AND 7B. Diagrams showing (A) the
in maintaining the arch of the foot. discontinuity in the TPT and the position of adja-
A negative exploration for a suspected TPT cent tendons and (B) the transfer of the FDL to
substitute for the TPT. (By permission of Mayo
dysfunction has become more and more in- Foundation.)
frequent. Usually what is found at operation
is more extensive than suspected clinically. It
is necessary to carry the surgical dissection all opened several centimeters proximal to the
the way to the TPT insertion where the tear- medial malleolus, the tendon appears white
ing and elongation may be located. compared to the adjacent FDL, the tendon
The results of transfer of the FDL for the will be completely disrupted distally. The sec-
elongated TPT have been quite Ini- ond is the resident sign and is only possible
tially, resuturing or reattachment of the elon- when practicing in a training program with
gated tendon was recommended but the re- residents. If, upon entering the operating
sults were not satisfactory. Because the FDL room one hears the resident muttering
seems to be so expendable, liberal use of this “Tom, Dick, and Harry,” followed by the
transfer seems reasonable. statement that the patient does not have a
There are two signs that will indicate a TPT, then the tendon is torn. The reason is
complete disruption of the TPT before the that the tendon has changed color and is
whole tendon is visualized. The first is the scarred closely behind the tibia and has not
white sign. If, when the sheath of the TPT is yet been correctly identified.
ClinlcalOrthopaedics
204 Johnson and Strom and Related Research

FIGS.8A-8G. Transfer of FDL to substitute


for TPT. (A) Medial view of left foot, showing
incision and TPT apparent just beneath the
flexor retinaculum proximally. (B) Fish-flesh
discoloration of TPT in contrast to normal
FDL just beneath it. (C) FDL tendon is uncov-
ered and retracted prior to transection distally.
(D) Zigzag suture is placed in the end of the
FDL tendon. (E) Drill hole is made from supe-
rior to inferior through navicular tuberosity.
(F) Tendon of the FDL is drawn through the
drill hole and sutured to the surrounding soft
tissues. (G) Transfer of the FDL is completed.
Distally, the transferred tendon runs beneath
the TPT and is sutured to it. Proximally, tenodesis of the TPT to the FDL was not done because of the
fibrotic changes within the tibialis posterior musculotendinous structure.
Number 239
February. 1989 Tibialis Posterior Tendon Dysfunction 205

STAGE 3-TENDON ELONGATED, version of the hindfoot. The tendon excur-


HINDFOOTDEFORMED
AND STIFF sion is short and the muscle is powerful.
Thus, elongation of the tendon will decrease
The tendon changes are much the same as
its function significantly.
in Stage 2. The static supports of the foot also
The recent concepts of the TPT difficulties
have been damaged, and fixed flatfoot has de-
have evolved through the efforts of multiple
veloped. A dynamic tendon transfer against a
authors. Two particularly important contri-
fixed deformity is not reasonable. Instead, a
butions were the paper by Kettelkamp and
realignment followed by an arthrodesis is
Alexander’ clearly describing the problem,
done. Controversy rages as to whether sub-
and that by Goldner et aL6 suggesting tendon
talar arthrodesis,16 talonavicular arthrode-
transfers. Since then, the experience and vari-
sis,I6 talonavicular with calcaneocuboid ar-
ations of others have been added. It is inter-
throdesis,” or triple arthrodesis should be
esting to speculate that perhaps the most
done. In fact, it probably does not make
efficacious pain relief aspect of the tendon
much difference. Arthrodesing any one of the
transfer procedures is really the release of the
hindfoot joints will effectively block hindfoot
TPT sheath. Earlierlo.’491* it was reported that
motion. It is important that the method ofar-
release of the sheath of the TPT alone was sat-
throdesis not malalign the adjacent joints.
isfactory. Still, it is reasonable to add the
The authors’ personal preference is for an
strength of the FDL to substitute for the TPT
isolated subtalar arthrodesis in most cases.
and to avoid progression of the flatfoot defor-
Based on the two-piece concept of the foot
mity.
(Fig. 3), the talus and everything else distally,
When this information is presented at a
with the subtalar arthrodesis being the every-
meeting someone will inevitably say that he
thing else, is repositioned beneath the talus.
or she has been in practice for 20 years and
The subtalar arthrodesis should not disrupt
has never seen a case of TPT dysfunction.
t t e relative positions at the talonavicular and
Then, a few weeks later, this same doctor sur-
calcaneocuboidjoints.
prisingly finds his or her first case. These pa-
For the subtalar arthrodesis, a morcellated
tients are out there and can be helped.
bone graft from the anterior iliac crest is in-
There is probably also a Stage 4 TPT dys-
serted into the subtalar joint with temporary
function. This is when the hindfoot has be-
fixation by a Steinmann pin across the neck
come fixed in eversion, over a number of
of the talus into the calcaneus.
years, to produce a valgus tilt of the talus
An arthrodesis with ten weeks of cast im-
within the ankle mortise and lateral tibiotalar
mobilization and then gradual recuperation
degeneration. This pattern may be seen with-
may seem a bit drastic. But the FDL tendon
out prior history of trauma. For these pa-
transfer requires six weeks of casting followed
tients, an arthrodesis from the tibia to the cal-
by an even longer period of recuperation. If
caneus can be done. With awareness of the
there is any question as to which of the two
TPT problem, such an extensive reconstruc-
should be done, based on the degree and ri-
tive procedure could have been avoided.
gidity of hindfoot change, the authors select
the arthrodesis. REFERENCES
1. Alexander, 1. J., Johnson, K. A., and Berquist, T. H.:
DISCUSSION Magnetic resonance imaging in the diagnosis of dis-
ruption of the posterior tibia1 tendon. Foot Ankle 8:
The tibialis posterior muscle-tendon unit 144,1987.
is a prime stabilizer of the hindfoot. By virtue 2. Citron, N.: Injury of the tibialis posterior tendon: A
of its position posterior to the axis of the an- cause of acquired valgus foot in childhood. Injury
16:610, 1985.
kle joint and medial to the subtalar axis, it 3 . Ford, L. T., and DeBender, J.: Tendon rupture after
provides plantar flexion at the ankle and in- local steroid injection. South. Med. J . 72:827, 1979.
Clinical Orlhopaedics
206 Johnson and Strorn and Related Research
-

4. Funk, D. A., Cass, J. R., and Johnson, K. A.: Ac- nor tibial muscle. J. Bone Joint Surg. 35A:1006.
quired adult flat foot secondary to posterior tibial- 1953.
tendon pathology. J. Bone Joint Surg. 68A:95, 1986. II. Leach, R. E., DeIorio, E., and Harney, R. A,: Patho-
5. Gilula, L. A., Oloff, L., Caputi, R., Destouet, J. M., logic hindfoot conditions in the athlete. Clin. Or-
Jacobs, A,, and Solomon, M. A.: Ankle tenography: thop. 177:I 16, 1983.
A key to unexplained symptomatology. Radiology 12. Mann, R. A., and Thompson, F. M.: Rupture of the
151:581, 1984. posterior tibial tendon causing flat foot. J. Bone
6. Goldner, J. L., Keats, P. K., Bassett, F. H. 111, and Joint Surg. 67A:556, 1985.
Clippinger, F. W.: Progressive talipes equinovalgus 13. Mueller, T. J.: Ruptures and lacerations of the tibi-
due to trauma or degeneration of the posterior tibial alis posterior tendon. J. Am. Podiatr. Med. Assoc.
tendon and medial plantar ligaments. Orthop. Clin. 74: 109, 1984.
North Am. 939, 1974. 14. Nonis, S. H., and Mankin, H. J.: Chronic tenosyno-
7. Jahss, M. H.: Spontaneous rupture ofthe tibialis pos- vitis of the posterior tibial tendon with new bone for-
mation. J. Bone Joint Surg. 60B:523, 1978.
terior tendon: Clinical findings, tenographic studies,
15. Sutherland, D. H.: An electromyographic study of
and a new technique of repair. Foot Ankle 3:158,
the plantar flexors of the ankle in normal walking on
1982.
the level. J. Bone Joint Surg. 48A:66, 1966.
8. Johnson, K. A.: Tibialis posterior tendon rupture. 16. Thomas, W. H.: Personal communication, 1986.
Clin. Orthop. 177:140, 1983. 17. Trevino, S., Gould, N., and Korson, R.: Surgical
9. Kettelkamp, D. B., and Alexander, H. H.: Spontane- treatment of stenosing tenosynovitis at the ankle.
ous rupture of the posterior tibial tendon. J. Bone Foot Ankle2:37, 1981.
Joint Surg. 5 lA:759, 1969. 18. Williams, R.: Chronic non-specific tendovaginitis of
10. Key, J. A,: Partial rupture ofthe tendon of the poste- tibialis posterior. J. Bone Joint Surg. 45B542, 1963.
This is an enhanced PDF from The Journal of Bone and Joint Surgery

The PDF of the article you requested follows this cover page.

Treatment of Acute Achilles Tendon Ruptures. A Meta-Analysis of


Randomized, Controlled Trials
Riaz J.K. Khan, Dan Fick, Angus Keogh, John Crawford, Tim Brammar and Martyn Parker
J Bone Joint Surg Am. 2005;87:2202-2210. doi:10.2106/JBJS.D.03049

This information is current as of June 10, 2007

Supplementary material Commentary and Perspective, data tables, additional images, video clips and/or
translated abstracts are available for this article. This information can be accessed
at http://www.ejbjs.org/cgi/content/full/87/10/2202/DC1
Letters to The Editor are available at
http://www.ejbjs.org/cgi/content/full/87/10/2202#responses
Subject Collections Articles on similar topics can be found in the following collections

Adult Trauma (574 articles)


Foot/Ankle (161 articles)
Ankle (82 articles)
Soft Tissue Trauma (213 articles)
Reprints and Permissions Click here to order reprints or request permission to use material from this
article, or locate the article citation on jbjs.org and click on the [Reprints and
Permissions] link.
Publisher Information The Journal of Bone and Joint Surgery
20 Pickering Street, Needham, MA 02492-3157
www.jbjs.org
2202
COPYRIGHT © 2005 BY THE JOURNAL OF BONE AND JOINT SURGERY, INCORPORATED

Treatment of Acute
Achilles Tendon Ruptures
A META-ANALYSIS OF RANDOMIZED, CONTROLLED TRIALS
BY RIAZ J.K. KHAN, FRCS(TR&ORTH), DAN FICK, MBBS, ANGUS KEOGH, MBBS,
JOHN CRAWFORD, FRCS(TR&ORTH), TIM BRAMMAR, FRCS(TR&ORTH), AND MARTYN PARKER, MD
Investigation performed at Perth Orthopaedic Institute, Department of Surgery and Pathology, University of Western Australia, Perth, Australia

Background: There is a lack of consensus regarding the best option for the treatment of acute Achilles tendon rup-
ture. Treatment can be broadly classified as operative (open or percutaneous) or nonoperative (casting or functional
bracing). Postoperative splinting can be performed with a rigid cast (proximal or distal to the knee) or a more mobile
functional brace. The aim of this meta-analysis was to identify and summarize the evidence from randomized, con-
trolled trials on the effectiveness of different interventions for the treatment of acute Achilles tendon ruptures.
Methods: We searched multiple databases (including EMBASE, CINAHL, and MEDLINE) as well as reference lists of
articles and contacted authors. Keywords included Achilles tendon, rupture, and tendon injuries. Three reviewers ex-
tracted data and independently assessed trial quality with use of a ten-item scale.
Results: Twelve trials involving 800 patients were included. There was a variable level of methodological rigor and re-
porting of outcomes. Open operative treatment was associated with a lower risk of rerupture compared with nonoper-
ative treatment (relative risk, 0.27; 95% confidence interval, 0.11 to 0.64). However, it was associated with a higher
risk of other complications, including infection, adhesions, and disturbed skin sensibility (relative risk, 10.60; 95%
confidence interval, 4.82 to 23.28). Percutaneous repair was associated with a lower complication rate compared
with open operative repair (relative risk, 2.84; 95% confidence interval, 1.06 to 7.62). Patients who had been man-
aged with a functional brace postoperatively (allowing for early mobilization) had a lower complication rate compared
with those who had been managed with a cast (relative risk, 1.88; 95% confidence interval, 1.27 to 2.76). Because
of the small number of patients involved, no definitive conclusions could be made regarding different nonoperative
treatment regimens.
Conclusions: Open operative treatment of acute Achilles tendon ruptures significantly reduces the risk of rerupture
compared with nonoperative treatment, but operative treatment is associated with a significantly higher risk of other
complications. Operative risks may be reduced by performing surgery percutaneously. Postoperative splinting with
use of a functional brace reduces the overall complication rate.
Level of Evidence: Therapeutic Level I. See Instructions to Authors for a complete description of levels of evidence.

T
he Achilles tendon, which is formed by the merging of topical corticosteroids5,6, fluoroquinolone antibiotics (e.g., cipro-
the tendons of the gastrocnemius and soleus, is the floxacin)7, exercise-induced hyperthermia8, and mechanical
thickest and strongest tendon in the human body. abnormalities of the foot9.
Acute ruptures occur most commonly in men in the third and Treatment of acute Achilles tendon ruptures can be
fourth decades of life who participate in sports intermittently, broadly classified as operative (open or percutaneous) or non-
and the left side is ruptured more commonly than is the right operative (cast immobilization or functional bracing). Gener-
side1. The mechanisms of injury include sudden forced plantar ally, open operative treatment has been used for athletes and
flexion of the foot, unexpected dorsiflexion of the foot, and vi- young, fit patients; percutaneous operative treatment has been
olent dorsiflexion of a plantar flexed foot2. The prevalence is used for those who do not wish to have an open repair (e.g.,
approximately 18 per 100,000 per year (in Finland) and is for cosmetic reasons); and nonoperative treatment has been
thought to be rising3. used for the elderly10-13.
It is generally accepted that ruptures occur in previously Previous reviews have examined the relative advantages
abnormal tendons2,4. A number of etiological theories have of operative and nonoperative treatment14-17. However, to our
been proposed, including the adverse influence of oral and knowledge, there has not been a systematic review of different
2203
THE JOUR NAL OF BONE & JOINT SURGER Y · JBJS.ORG TRE A T M E N T O F A C U T E
VO L U M E 87-A · N U M B E R 10 · O C T O B E R 2005 A C H I L L E S TE N D O N R U P T U RE S

methods of nonoperative treatment, operative treatment, and and manual search results. In MEDLINE (OVID-WEB), the fol-
postoperative splinting. The aim of the present meta-analysis lowing subject-specific search was combined with all three lev-
was to identify and summarize the evidence from random- els of the optimal trial search strategy18: Achilles Tendon,
ized, controlled trials on the effectiveness of all treatment in- (achill#s or tendoachill#s).tw., or/1-2, Rupture/, rupture$.tw.,
terventions for acute Achilles tendon ruptures. or/4-5, and/3,6, Tendon Injuries/, and/3,8, or/7,9. Articles in all
languages were considered for inclusion and were translated
Materials and Methods when necessary. We excluded retrospective studies, studies with
ll randomized, controlled trials comparing operative and insufficient reporting of primary outcomes, studies with inade-
A nonoperative methods for the treatment of acute Achilles
tendon ruptures were considered for inclusion. Quasi-random-
quate methods of randomization, and unique randomized,
controlled trials (where pooling of data was not possible, mak-
ized trials (for example, those involving allocation by alterna- ing them unsuitable for meta-analysis).
tion or the date of birth) and trials in which the treatment Participants included adults with acute ruptures of the
allocation was inadequately concealed were also considered. We Achilles tendon. Patients with delayed presentation (more than
searched the Cochrane Musculoskeletal Injuries Group special- three weeks after the injury) and rerupture were excluded. The
ized register, reference lists of articles, and proceedings of rele- types of interventions included operative repair (open and per-
vant conferences, and we contacted authors when additional cutaneous) and nonoperative treatment (cast immobilization
clarification was required. This register is compiled from multi- and functional bracing). The primary outcomes were complica-
ple databases and includes the results of regular searches of the tions of treatment and rerupture. Other outcomes, such as the
Cochrane Central Register of Controlled Trials in the Cochrane level of sporting activity, patient satisfaction, and the length of
Library, MEDLINE (which combines subject-specific terms hospital stay, were omitted because they lack quantity and uni-
with the optimal trial search strategy18), EMBASE, CINAHL, formity to support rigorous meta-analysis.

Fig. 1
System used for the scoring of methodology.
2204
THE JOUR NAL OF BONE & JOINT SURGER Y · JBJS.ORG TRE A T M E N T O F A C U T E
VO L U M E 87-A · N U M B E R 10 · O C T O B E R 2005 A C H I L L E S TE N D O N R U P T U RE S

Fig. 2
Flowchart depicting the method by which the twelve randomized, controlled studies were chosen.

Trials were independently assessed for inclusion by four semi-mobile)25-29. Two studies, involving ninety-four patients,
reviewers (R.J.K.K., D.F., A.K., and T.B.). Data on the outcomes compared percutaneous repair with open operative repair24,30.
listed above were extracted by three reviewers (R.J.K.K., D.F., Two studies, involving ninety patients, compared different
and T.B.). Differences were resolved by discussion. Ten aspects nonoperative treatment regimens31,32. The group of thirteen
of methodology were used to assign a maximum score of 12 to patients managed with open operative treatment as described
each study, similar to the scale described by Detsky et al.19 (Fig. by Schroeder et al.24 were analyzed in two sections of the
1). In addition, the risk of pre-allocation disclosure of assign- present review, which accounts for the discrepancy in the
ment was rated as A, B, or C according to the Cochrane Review- overall number of patients and studies involved.
ers’ Handbook20. The methodological quality scores of the included
For each study, relative risks and 95% confidence inter- studies are detailed in Table I. Low scores indicate poor
vals were calculated for dichotomous outcomes, and weighted methodology.
mean differences and 95% confidence intervals were calculated
for continuous outcomes. The results of individually random-
TABLE I Methodological Quality Scores
ized trials were pooled whenever possible with use of the fixed-
effects model of Mantel-Haenszel. Heterogeneity between Scores for Total Score
comparable studies was tested with the use of a standard chi- Study Ten Items (Maximum, 12)
square test. The random-effects model of DerSimonian and Cetti et al.21 1110110010 6
Laird was used when there was statistical or graphical evi- Cetti et al.25 1110010011 6
dence of heterogeneity. 29
Kangas et al. 3110100111 9

Results Kerkhoffs et al.26 0110110011 6


hirty-six articles were identified with use of our search
T
30
Lim et al. 0100010101 4
strategy; of these, twenty-four were excluded from the Maffulli et al.28 0110111010 6
meta-analysis (Fig. 2). Moller et al. 22
3110110111 10
Details on the twelve randomized, controlled trials that
Mortensen et al.27 2110110010 7
were included in the review are documented in the Appendix.
23
A total of 800 patients were involved. Four studies, involving Nistor 0010010011 4
356 patients, compared open operative treatment with non- Petersen et al.32 3110010010 7
operative treatment21-24. Five studies, involving 273 patients, Saleh et al. 31
1100110011 6
compared postoperative splinting in a cast alone (i.e., rigid)
Schroeder et al.24 1100100000 3
with splinting in a cast followed by a functional brace (i.e.,
2205
THE JOUR NAL OF BONE & JOINT SURGER Y · JBJS.ORG TRE A T M E N T O F A C U T E
VO L U M E 87-A · N U M B E R 10 · O C T O B E R 2005 A C H I L L E S TE N D O N R U P T U RE S

Fig. 3
Illustration indicating the prevalence of rerupture associated with open operative and nonoperative treatment. The values are given as the num-
ber of patients with a rerupture (n)/number of patients in the group (N), with a summation of the totals and the relative risk (RR) and 95% confi-
dence intervals (95% CI).

Fig. 4
Illustration indicating the prevalence of complications other than rerupture associated with open operative and nonoperative treatment. The val-
ues are given as the number of patients with a complication (n)/number of patients in the group (N), with a summation of the totals and the rel-
ative risk (RR) and 95% confidence intervals (95% CI).

Open Operative Treatment Compared in the nonoperatively treated group (relative risk, 0.27; 95%
with Nonoperative Treatment21-24 confidence interval, 0.11 to 0.64). The mean duration of follow-
The rate of rerupture is summarized in Figure 3. One study22 up used for the calculation of rerupture rates ranged from eight
revealed a significant difference between the operative group to thirty months.
and the nonoperative group with regard to the rate of rerup- Nistor23 and Moller et al.22 reported more adhesions, dis-
ture (one of fifty-nine compared with eleven of fifty-three; p = turbed sensibility, and deep or superficial wound infection in
0.0013). Of the three remaining studies, one showed no dif- the operative group. No statistical analysis was performed. Cetti
ference between operative and nonoperative treatment21, one et al.21 reported a significant increase in the rates of delayed
included no reruptures24, and one did not involve statistical wound-healing, adhesions, and disturbed sensibility in the op-
analysis23. Results gave a pooled rate of 3.5% (six of 173) in the erative group (p = 0.004). The pooled rate of reported compli-
operatively treated group and of 12.6% (twenty-three of 183) cations (other than rerupture) was 34.1% (fifty-nine of 173) in

Fig. 5
Illustration indicating the prevalence of wound infection associated with open operative and nonoperative treatment. The values are given as the
number of patients with a wound infection (n)/number of patients in the group (N), with a summation of the totals and the relative risk (RR) and
95% confidence intervals (95% CI).
2206
THE JOUR NAL OF BONE & JOINT SURGER Y · JBJS.ORG TRE A T M E N T O F A C U T E
VO L U M E 87-A · N U M B E R 10 · O C T O B E R 2005 A C H I L L E S TE N D O N R U P T U RE S

Fig. 6
Illustration indicating the prevalence of rerupture associated with open surgery and percutaneous surgery. The values are given as the number of
patients with a rerupture (n)/number of patients in the group (N), with a summation of the totals and the relative risk (RR) and 95% confidence
intervals (95% CI).

the operative group and 2.7% (five of 183) in the nonoperative 95% confidence interval, 0.19 to 21.00). The mean duration of
group (relative risk, 10.60; 95% confidence interval, 4.82 to follow-up used for the calculation of the rerupture rates
23.28). Similar analysis demonstrated an overall rate of wound ranged from six to eight months. These results are summa-
infection of 4.0% (seven of 173) in the operative group; there rized in Figure 6.
were no infections in the nonoperative group (relative risk, The pooled rate of reported complications (excluding
4.89; 95% confidence interval, 1.09 to 21.91). These results are rerupture) was 26.1% (twelve of forty-six) in the open group
summarized in Figures 4 and 5. and 8.3% (four of forty-eight) in the percutaneous group (rel-
ative risk, 2.84; 95% confidence interval, 1.06 to 7.62). Lim et
Open Compared with al. reported a significantly higher rate of wound infection in
Percutaneous Operative Repair24,30 the open group as compared with the percutaneous group (p =
Schroeder et al.24 reported no reruptures, whereas Lim et al.30 0.01)30. The pooled rate of infection was 19.6% (nine of forty-
found no significant difference between the groups with re- six) in the open group and 0% (zero of forty-eight) in the per-
gard to the rate of rerupture. The pooled rate of rerupture was cutaneous group (relative risk, 10.52; 95% confidence interval,
4.3% (two of forty-six) in the open group and 2.1% (one of 1.37 to 80.52). The results of these analyses are summarized in
forty-eight) in the percutaneous group (relative risk, 2.00; Figures 7 and 8.

Fig. 7
Illustration indicating the prevalence of complications other than rerupture associated with open surgery and percutaneous surgery. The values
are given as the number of patients with a complication (n)/number of patients in the group (N), with a summation of the totals and the relative
risk (RR) and 95% confidence intervals (95% CI).

Fig. 8
Illustration indicating the prevalence of wound infection associated with open surgery and percutaneous surgery. The values are given as the
number of patients with an infection (n)/number of patients in the group (N), with a summation of the totals and the relative risk (RR) and 95%
confidence intervals (95% CI).
2207
THE JOUR NAL OF BONE & JOINT SURGER Y · JBJS.ORG TRE A T M E N T O F A C U T E
VO L U M E 87-A · N U M B E R 10 · O C T O B E R 2005 A C H I L L E S TE N D O N R U P T U RE S

Fig. 9
Illustration indicating the prevalence of rerupture associated with postoperative splinting with casting alone and casting followed by functional
bracing. The values are given as the number of patients with a rerupture (n)/number of patients in the group (N), with a summation of the totals
and the relative risk (RR) and 95% confidence intervals (95% CI).

Postoperative Splinting: Cast Immobilization val, 0.59 to 21.76). The mean duration of follow-up used for the
Alone Compared with Cast Immobilization calculation of the rerupture rates was twelve months for both
Followed by Functional Bracing25-29 studies. The results are summarized in Figure 11.
Rerupture rates are summarized in Figure 9. No individual
study demonstrated a significant difference between the groups. Discussion
The pooled rate of rerupture was 5.0% (seven of 140) in the cast welve prospective randomized studies involving the treat-
immobilization group and 2.3% (three of 133) in the functional
bracing group (relative risk, 2.04; 95% confidence interval, 0.59
T ment of acute Achilles tendon rupture fulfilled the inclu-
sion criteria for this meta-analysis. Quality assessment scores
to 7.06). The mean duration of follow-up used for the calcula- were calculated for each study in order to assess the level of
tion of the rerupture rates ranged from five months to 6.7 years. methodological rigor. They were not used as a criterion for ex-
Complications were more common in the cast-only clusion or to weight the pooled data. However, the findings of
group; specifically, the cast-only group had higher rates of ad- studies with higher methodological quality should naturally
hesions (18.6% compared with 9.7%), disturbed sensibility be considered to be of greater importance. The scores indicate
(8.6% compared with 3.8%), keloid or hypertrophic scarring a variable level of methodological rigor, particularly with re-
(5% compared with 3%), and infection (3.5% compared with gard to the method of randomization and concealment of al-
3%). The pooled rate of reported complications (other than location. The quality of the primary study used for pooled
rerupture) was 35.7% (fifty of 140) in the cast immobilization analysis influences the results of a meta-analysis33. The inclu-
group and 19.5% (twenty-six of 133) in the functional bracing sion of poorly randomized trials can lead to over-reporting of
group (relative risk, 1.88; 95% confidence interval, 1.27 to treatment effect, and thus all recommendations should be
2.76). The results are summarized in Figure 10. critically appraised. Recommendations have been made on the
basis of analysis of pooled data extracted from what were be-
Nonoperative Treatment: Casting Immobilization lieved to be the most rigorously conducted studies.
Compared with Functional Bracing31,32 The rate of rerupture was consistently higher among
Pooled data revealed a rerupture rate of 2.4% (one of forty-one) nonoperatively treated patients as compared with operatively
in the functional bracing group and of 12.2% (six of forty-nine) treated patients. The most methodologically sound study22 pro-
in the casting group (relative risk, 3.59; 95% confidence inter- vided the most favorable rerupture rate with operative interven-

Fig. 10
Illustration indicating the prevalence of complications other than rerupture associated with postoperative splinting with casting alone and casting
followed by functional bracing. The values are given as the number of patients with a complication (n)/number of patients in the group (N), with a
summation of the totals and the relative risk (RR) and 95% confidence intervals (95% CI).
2208
THE JOUR NAL OF BONE & JOINT SURGER Y · JBJS.ORG TRE A T M E N T O F A C U T E
VO L U M E 87-A · N U M B E R 10 · O C T O B E R 2005 A C H I L L E S TE N D O N R U P T U RE S

Fig. 11
Illustration indicating the prevalence of rerupture associated with nonoperative treatment (casting alone and casting followed by functional brac-
ing). The values are given as the number of patients with a rerupture (n)/number of patients in the group (N), with a summation of the totals and
the relative risk (RR) and 95% confidence intervals (95% CI).

tion, suggesting that there is a legitimate advantage associated Using a technique of analysis not previously applied to rup-
with surgery. However, there was a consistent finding of in- ture of the Achilles tendon, Kocher et al.37 performed an ex-
creased rates of complications (other than rerupture) in the op- pected-value decision analysis of operative and nonoperative
eratively treated group, with all studies demonstrating similar management. Expected-value decision analysis involves allo-
rates. In summary, nonoperatively treated patients have a more cating utility scores to outcomes and allows quantitative anal-
than three times higher risk of rerupture but have a minimal ysis of decision-making. A decision tree was constructed, and
risk of other complications resulting from treatment. One-third prospective patients progressed through the various alter-
of operatively treated patients have a complication. natives. Article selection for the generation of outcome proba-
A meta-analysis by Bhandari et al.15 comparing open op- bilities followed the criteria of Lo et al.14. With use of this
erative treatment with nonoperative treatment of acute Achil- technique, operative treatment was found to be the optimal
les tendon ruptures did not provide a strong recommendation strategy. However, the authors stressed that the decision-
for surgery. The authors suggested that patients who are reluc- making process should be shared between doctor and patient.
tant to undergo an operation may choose nonoperative treat- In our analysis of studies comparing open and percuta-
ment. Their meta-analysis included six studies, three of which neous repair, we noted a tendency for a lower overall rate of
were omitted from the present meta-analysis because of inad- complications (particularly infection) in the percutaneously
equate reporting of results34, discontinuation of treatment in treated group. However, this finding is based on pooled data
the control group because of a high recurrence rate and no al- from a small number of patients, and there is some discrep-
location concealment (personal communication with the au- ancy between studies with regard to the rate of infection in the
thor)35, and inadequate randomization36. We included an extra open treatment group. In a previous review of prospective and
study24 that did fulfill our strict inclusion criteria. Neverthe- retrospective studies regarding operative and nonoperative
less, the results reported by Bhandari et al.15 are similar to treatment, Wong et al.17 reported a lower rate of wound com-
ours; specifically, the rerupture rate was 3.1% for operatively plications in patients undergoing percutaneous repair. How-
treated patients and 13% for nonoperatively treated patients ever, they also noted that patients in the percutaneous group
(p = 0.005) and the infection rate was higher among opera- had relatively high rates of complications (notably sural nerve
tively treated patients (4.7% compared with 0%; relative risk, injury), particularly when the procedure was combined with
4.6; p = 0.03). early active mobilization.
Lo et al.14 compiled two prospective trials and seventeen One of the most important aspects of the present review
case series for their comparison of operative and nonoperative is that pertaining to postoperative splintage. This subject has
treatment. The studies by Moller et al.22 and Schroeder et al.24 not been previously evaluated with use of meta-analysis. The
were not included in that analysis. Lo et al. found it difficult to functional bracing group had a significantly lower rate of
recommend one treatment over the other on the basis of the complications (p = 0.001), particularly with regard to adhe-
combined complication rates14. They suggested that patients sion formation. The early mobilization group also tended to
with poor healing potential should be managed nonopera- have a lower rerupture rate. Conclusions made on the basis of
tively and that active patients should be offered both operative the pooled data must be interpreted with caution because of
and nonoperative treatment. In other reviews, Lynch16 and the variety of regimens used.
Wong et al.17 included sixteen studies and 125 studies, respec- Limited conclusions can be drawn from the two studies
tively. Both groups of investigators included nonrandomized comparing nonoperative treatment in a cast and functional
trials in their analysis. The studies concurred in their recom- bracing because of the small numbers involved (ninety patients),
mendation for surgical intervention combined with early differences in regimens, and minimal reporting of outcomes31,32.
functional mobilization for the treatment of acute Achilles It is interesting to note, however, that the pooled rate of rerup-
tendon ruptures; nonoperative management with early func- ture in the functional bracing group (2.4%) was lower than that
tional mobilization may be an acceptable alternative for pa- for patients managed with operative treatment (3.5%). Indeed, it
tients who are reluctant to undergo a surgical procedure. was almost equivalent to that seen for patients managed with
2209
THE JOUR NAL OF BONE & JOINT SURGER Y · JBJS.ORG TRE A T M E N T O F A C U T E
VO L U M E 87-A · N U M B E R 10 · O C T O B E R 2005 A C H I L L E S TE N D O N R U P T U RE S

immobilization in a functional brace after open repair (2.3%). Riaz J.K. Khan, FRCS(Tr&Orth)
This apparent discrepancy may be explained by the small num- Department of Orthopaedics, Norfolk and Norwich University Hospital,
bers of patients managed nonoperatively. Colney Lane, Norwich, Norfolk, NR4 7UY, United Kingdom. E-mail ad-
dress: riazkhan@aol.com
In conclusion, open operative treatment of acute Achil-
les tendon ruptures significantly reduces the risk of rerupture Dan Fick, MBBS
compared with nonoperative treatment but has the drawback Angus Keogh, MBBS
of a significantly higher risk of other complications, including Department of Surgery and Pathology, University of Western Australia,
wound infection. Complications may be reduced by perform- Perth, WA 6009, Australia
ing surgery percutaneously. Postoperative splinting in a cast
followed by a functional brace rather than a cast alone reduces John Crawford, FRCS(Tr&Orth)
the overall complication rate. Tim Brammar, FRCS(Tr&Orth)
Addenbrooke’s Hospital, Cambridge, CB2 2QQ, United Kingdom
Additional rigorously conducted prospective random-
ized trials with larger sample sizes, full reporting of outcomes,
Martyn Parker, MD
and blinding of assessors are required. Increased transparency Peterborough District Hospital, Peterborough, PE3 6DA, United Kingdom
is needed if the same cohort of patients is reported on in
different studies, and avoidance of multiple publications is The authors did not receive grants or outside funding in support
strongly recommended. of their research or preparation of this manuscript. They did not
receive payments or other benefits or a commitment or agreement
Appendix to provide such benefits from a commercial entity. No commercial
A table presenting the characteristics of the included entity paid or directed, or agreed to pay or direct, any benefits to any
studies is available with the electronic versions of this ar- research fund, foundation, educational institution, or other charita-
ble or nonprofit organization with which the authors are affiliated
ticle, on our web site at jbjs.org (go to the article citation and or associated.
click on “Supplementary Material”) and on our quarterly CD-
ROM (call our subscription department, at 781-449-9780, to
order the CD-ROM).  doi:10.2106/JBJS.D.03049

References
1. Hattrup SJ, Johnson KA. A review of ruptures of the Achilles tendon. Foot An- 16. Lynch RM. Achilles tendon rupture: surgical versus non-surgical treatment.
kle. 1985;6:34-8. Accid Emerg Nurs. 2004;12:149-58.
2. Arner O, Lindholm A. Subcutaneous rupture of the Achilles tendon: a study of 17. Wong J, Barrass V, Maffulli N. Quantitative review of operative and nonopera-
92 cases. Acta Chir Scand. 1959;116:1-51. tive management of achilles tendon ruptures. Am J Sports Med. 2002;30:565-75.
3. Leppilahti J, Puranen J, Orava S. Incidence of Achilles tendon rupture. Acta Or- 18. Alderson P, Green S, Higgins JPT, editors. MEDLINE highly sensitive search
thop Scand. 1996;67:277-9. strategy for SilverPlatter-MEDLINE, OVID-MEDLINE, and PubMed. Cochrane re-
viewers’ handbook 4.2.1; Appendix 5B. In: The Cochrane library, Issue 1. Chich-
4. Tallon C, Maffulli N, Ewen SW. Ruptured Achilles tendons are significantly
ester, UK: John Wiley and Sons; 2004.
more degenerated than tendinopathic tendons. Med Sci Sports Exerc. 2001;
33:1983-90. 19. Detsky AS, Naylor CD, O’Rourke K, McGeer AJ, L’Abbe KA. Incorporating vari-
ations in the quality of individual randomized trials into meta-analysis. J Clin Epi-
5. Mahler F, Fritschy D. Partial and complete ruptures of the Achilles tendon and
demiol. 1992;45:255-65.
local corticosteroid injections. Br J Sports Med. 1992;26:7-14.
20. Alderson P, Green S, Higgins JPT, editors. Assessment of study quality. Co-
6. Newnham DM, Douglas JG, Legge JS, Friend JA. Achilles tendon rupture: an
chrane reviewers’ handbook 4.2.1; Section 6. In: The Cochrane library, Issue 1.
underrated complication of corticosteroid treatment. Thorax. 1991;46:853-4.
Chichester, UK: John Wiley and Sons; 2004.
7. Royer RJ, Pierfitte C, Netter P. Features of tendon disorders with fluoroquinolo-
21. Cetti R, Christensen SE, Ejsted R, Jensen NM, Jorgensen U. Operative ver-
nes. Therapie. 1994;49:75-6.
sus nonoperative treatment of Achilles tendon rupture. A prospective randomized
8. Wilson AM, Goodship AE. Exercise-induced hyperthermia as a possible mech- study and review of the literature. Am J Sports Med. 1993;21:791-9.
anism for tendon degeneration. J Biomech. 1994;27:899-905.
22. Moller M, Movin T, Granhed H, Lind K, Faxen E, Karlsson J. Acute rupture of
9. Clement DB, Taunton JE, Smart GW. Achilles tendinitis and peritendinitis: eti- tendon Achillis. A prospective randomised study of comparison between surgical
ology and treatment. Am J Sports Med. 1984;12:179-84. and non-surgical treatment. J Bone Joint Surg Br. 2001;83:843-8.
10. Bossley CJ. Rupture of the Achilles tendon. J Bone Joint Surg Am. 2000; 23. Nistor L. Surgical and non-surgical treatment of Achilles tendon rupture. A
82:1804. prospective randomized study. J Bone Joint Surg Am. 1981;63:394-9.
11. Maffulli N. Rupture of the Achilles tendon. J Bone Joint Surg Am. 1999; 24. Schroeder D, Lehmann M, Steinbrueck K. Treatment of acute achilles tendon
81:1019-36. ruptures: open vs. percutaneous repair vs. conservative treatment. A prospective
randomized study. Orthop Trans. 1997;21:1228.
12. Martinelli B. Rupture of the Achilles tendon. J Bone Joint Surg Am. 2000;
82:1804. 25. Cetti R, Henriksen LO, Jacobsen KS. A new treatment of ruptured Achilles ten-
dons. A prospective randomized study. Clin Orthop Relat Res. 1994;308:155-65.
13. Raisbeck CC. Rupture of the Achilles tendon. J Bone Joint Surg Am. 2000;
82:1804-5. 26. Kerkhoffs GM, Struijs PA, Raaymakers EL, Marti RK. Functional treatment af-
ter surgical repair of acute Achilles tendon rupture: wrap vs walking cast. Arch Or-
14. Lo IK, Kirkley A, Nonweiler B, Kumbhare DA. Operative versus nonoperative
thop Trauma Surg. 2002;122:102-5.
treatment of acute Achilles tendon ruptures: a quantitative review. Clin J Sport
Med. 1997;7:207-11. 27. Mortensen NH, Skov O, Jensen PE. Early motion of the ankle after operative
treatment of a rupture of the Achilles tendon. A prospective, randomized clinical
15. Bhandari M, Guyatt GH, Siddiqui F, Morrow F, Busse J, Leighton RK, Sprague
and radiographic study. J Bone Joint Surg Am. 1999;81:983-90.
S, Schemitsch EH. Treatment of acute Achilles tendon ruptures: a systematic
overview and metaanalysis. Clin Orthop Relat Res. 2002;400:190-200. 28. Maffulli N, Tallon C, Wong J, Lim KP, Bleakney R. Early weightbearing and an-
2210
THE JOUR NAL OF BONE & JOINT SURGER Y · JBJS.ORG TRE A T M E N T O F A C U T E
VO L U M E 87-A · N U M B E R 10 · O C T O B E R 2005 A C H I L L E S TE N D O N R U P T U RE S

kle mobilization after open repair of acute midsubstance tears of the achilles ten- operative treatment. Am J Sports Med. 2003;31:685-91.
don. Am J Sports Med. 2003;31:692-700.
43. Weber O, Schmidgen A. Semifunctional postoperative treatment of Achilles
29. Kangas J, Pajala A, Siira P, Hamalainen M, Leppilahti J. Early functional treat- tendon rupture. Trauma und Berufskrankheit. 1999;1:258-63.
ment versus early immobilization in tension of the musculotendinous unit after
44. Wellner K. [Modified Kirchmayr suture in comparison with the Silfverskjold
Achilles rupture repair: a prospective, randomized, clinical study. J Trauma.
folding-plasty in primary management of covered Achilles tendon rupture]. Beitr
2003;54:1171-81.
Orthop Traumatol. 1990;37:107-12. German.
30. Lim J, Dalal R, Waseem M. Percutaneous vs. open repair of the ruptured
45. Kakiuchi M. A combined open and percutaneous technique for repair
Achilles tendon—a prospective randomized controlled study. Foot Ankle Int.
of tendo Achillis. Comparison with open repair. J Bone Joint Surg Br. 1995;
2001;22:559-68.
77:60-3.
31. Saleh M, Marshall PD, Senior R, MacFarlane A. The Sheffield splint for con-
46. Kern O, Bouillon B, Lobach R. On the therapy of fresh rupture of the achilles
trolled early mobilisation after rupture of the calcaneal tendon. A prospective,
tendon operative-functional vs. conservative functional therapy. Hefte zur der Un-
randomised comparison with plaster treatment. J Bone Joint Surg Br. 1992;
fallchirurg. 1996;262:199.
74:206-9.
47. Thermann H, Hufner T, Tscherne H. [Achilles tendon rupture]. Orthopade.
32. Petersen OF, Nielsen MB, Jensen KH, Solgaard S. [Randomized comparison
2000;29:235-50. German.
of CAM walker and light-weight plaster cast in the treatment of first-time Achilles
tendon rupture]. Ugeskr Laeger. 2002;164:3852-5. Danish. 48. Thermann H, Zwipp H, Milbradt H, Reimer P. [Ultrasound sonography in the
diagnosis and follow-up of Achilles tendon rupture]. Unfallchirurg. 1989;92:266-
33. Moher D, Pham B, Jones A, Cook DJ, Jadad AR, Moher M, Tugwell P, Klassen
73. German.
TP. Does quality of reports of randomised trials affect estimates of intervention ef-
ficacy reported in meta-analyses? Lancet. 1998;352:609-13. 49. Thermann H, Zwipp H, Sudkamp N. Operative versus conservative func-
tional treatment of rupture of the achilles tendon. Hefte zur Unfallheilkunde.
34. Coombs RRH. Prospective trial of conservative and surgical treatment of
1989;207:398.
Achilles tendon rupture. J Bone Joint Surg Br. 1981;63:288.
50. Thermann H, Zwipp H. [Achilles tendon rupture]. Orthopade. 1989;18:321-
35. Majewski M, Rickert M, Steinbruck K. [Achilles tendon rupture. A prospective
33. German.
study assessing various treatment possibilities]. Orthopade. 2000;29:670-6.
German. 51. Thermann H. Conservative functional treatment. Chirurgische Praxis. 2001;
58:83-95.
36. Thermann H, Zwipp H, Tscherne H. [Functional treatment concept of acute
rupture of the Achilles tendon. 2 years results of a prospective randomized 52. Thermann H. Functional treatment concept for fresh rupture of the achilles
study]. Unfallchirurg. 1995;98:21-32. German. tendon. Hefte zur der Unfallchirurg. 1996;248:4873.
37. Kocher MS, Bishop J, Marshall R, Briggs KK, Hawkins RJ. Operative versus 53. Thermann H. Rupture of the Achilles tendon. Conservative vs operative.
nonoperative management of acute Achilles tendon rupture: expected-value deci- Trauma und Berufskrankheit. 2000;2:160-6.
sion analysis. Am J Sports Med. 2002;30:783-90.
54. Thermann H, Zwipp H. Conservative treatment of fresh rupture of the achil-
38. Haggmark T, Liedberg H, Eriksson E, Wredmark T. Calf muscle atrophy and les tendon. Hefte zur Unfallheilkunde. 1992;222:83-94.
muscle function after non-operative vs operative treatment of achilles tendon rup-
55. Zwipp H, Thermann H, Sudkamp N, Tscherne H, Milbradt H, Reimer P, Heintz
tures. Orthopedics. 1986;9:160-4.
P. [An innovative concept for primary functional treatment of Achilles tendon rup-
39. Helgeland J, Odland P, Hove LM. [Achilles tendon rupture. Surgical or non- ture]. Sportverletz Sportschaden. 1990;4:29-35. German.
surgical treatment]. Tidsskr Nor Laegeforen. 1997;117:1763-6. Norwegian.
56. Mortensen NH, Saether J, Steinke MS, Staehr H, Mikkelsen SS. Separation
40. Paes E, Weyand F, Tuncay N. [Management of Achilles tendon rupture. A com- of tendon ends after Achilles tendon repair: a prospective, randomized, multi-
parative study of the adaptation suture and the plantaris tendon interweaving center study. Orthopedics. 1992;15:899-903.
technic]. Unfallchirurg. 1985;88:303-7. German.
57. Mortensen NH, Sorensen L, Pless S. Below-knee versus above-knee cast
41. Steele GJ, Harter RA, Ting AJ. Comparison of functional ability following per- after Achilles tendon repair—a prospective controlled trial. Acta Orthop Scand.
cutaneous and open surgical repairs of acutely ruptured Achilles tendons. J Sport 1996;67:38.
Rehab. 1993;2:115-27.
58. Maffulli N, Tallon C, Wong J, Peng Lim K, Bleakney R. No adverse effect of
42. Weber M, Niemann M, Lanz R, Muller T. Nonoperative treatment of acute early weight bearing following open repair of acute tears of the Achilles tendon.
rupture of the achilles tendon: results of a new protocol and comparison with J Sports Med Phys Fitness. 2003;43:367-79.
TREATMENT OF MOBILE FLAT FOOT BY DISPLACEMENT OSTEOTOMY
OF THE CALCANEUS

E. KOUTSOGIANNIS, LARISSA, GREECE

From the Southmead Hospital, Bristol, Englaizd

The purpose of this paper is to advocate simple calcaneal osteotomy as an effective


treatment to relieve fatigue, improve shape and prevent abnormal shoe wear in patients with
mobile flat foot. It is reserved for those cases in which conservative measures have failed
and the symptoms warrant surgical treatment.
The concept of calcaneal osteotomy is an old one. The first description was given by
Gleich iii 1893 (quoted by Silver, Simon, Spindell, Litchman and Scala 1967). Gleich’s
technique involved displacement of the posterior fragment forwards, medially and downwards
in an attempt to restore the normal angle between the long axis of the calcaneus and the floor.
This was achieved by excision of a medial wedge. In 1923 Lord reported thirteen cases of
calcaneal osteotomy for severe flat foot. Only one patient, however, had calcaneal osteotomy
alone; the others all required supplementary procedures.
Dwyer (1960) advocated calcaneal osteotomy with insertion of a tibial graft in the outer
side. He believed that the basic principles were the same whether the patient suffered from

FIG. 1
Radiograph of a foot in the standing position shows that the inferior
surface of the calcaneus is almost parallel with the ground, the talus is
less horizontal than normal and there is a subluxation at the tab-
navicular joint.

mobile flat foot or from flat foot in cerebral palsy. In many patients with the latter condition
he elongated the tendo calcaneus as a supplementary procedure at the same time.
Displacement osteotomy of the calcaneus has been practised in the Bristol clinical area
for some years, the operation having been first performed here by the late Mr Pridie.

ANATOMY OF THE FLAT FOOT


In mobile flat foot the calcaneus lies in valgus; its superior articular surface is tilted
medially, lying on a lower level than normal. The talus is rotated medially and downwards,
its head producing a prominence medially just behind the navicular bone. Hence the alignment
of the bones in the medial pillar of the foot is lost, and subluxation is produced between talus
and navicular bone, to which is sometimes added a subluxation between the navicular bone
and the cuneiform (Fig. 1).

96 THE JOURNAL OF BONE AND JOINT SURGERY


TREATMENT OF MOBILE FLAT FOOT BY DISPLACEMENT OSTEOTOMY OF THE CALCANEUS 97

The forefoot is pronated and slightly abducted at the mid-tarsal joints (Fig. 2).
The distorted anatomy produces the well known clinical deformity comprising: 1)
flattening at the longitudinal arch (Fig. 3); and 2) valgus deformity of the heel, in consequence
of which the tendo calcaneus acquires an evertor action,
which may tend to aggravate the condition (Fig. 7).
Either the planus or the valgus may predominate or
they may be equally represented.
Consequent upon the valgus deformity, the line of
weight-bearing transmitted through the talus passes
medial to the calcaneus (Figs. 4 and 5).

THE OPERATION
The aim of the operation is to displace the posterior
part of the calcaneus medially and thus to restore normal
weight-bearing (Fig. 6).
Technique-A tourniquet is used. The patient lies prone
with a sandbag under the lower shin to allow free
movement of the ankle. A lateral incision is made,
parallel to and a little behind and inferior to the peroneal
tendons, care being taken not to damage the sural nerve.
The upper end of the incision overlies the lateral margin
of the calcaneal tendon and its lower end reaches the
inferior margin of the calcaneus. The skin edges are
held apart by a self-retaining retractor and two small
bone spikes are inserted above
and below the calcaneus in
order to expose its superior, lateral and inferior surfaces.
The periosteum is incised and elevated in the line of
the skin incision to allow subperiosteal osteotomy with
a broad osteotome. The osteotomy is prised open and FIG. 2
the periosteum is divided along the medial aspect to Radiograph of a weight-bearing foot
showing that the mid-talar line passes
allow the displacement envisaged. It may be necessary medial to the forefoot and not through
to divide the long plantar ligament to obtain sufficient the shaft of the first metatarsal as it
should in the normal.
displacement.
When the osteotomy has been completed the posterior fragment of the calcaneus is
displaced medially until its medial border lies in a line with the sustentaculum tali. This usually

FIG. 3
Severe flattening of the longitudinal arch in a mobile flat foot.

entails a displacement between a third to a half of the width of the calcaneus. The displacement
is held by one or two Kirschner wires, inserted obliquely from the postero-inferior surface of

VOL. 53 B, NO. 1, FEBRUARY 1971


G
98 F. KOUTSOGIANNIS

FIG. 4 FIG. S FIG. 6


The weight-bearing line and relation of the talus to the calcaneus seen from a postero-superior
viewpoint in a normal foot (Fig. 4) and a flat foot (Fig. 5). The displacement at operation is shown
in Figure 6.

Before operation there is valgus of the heels (Fig. 7) but after operation on both heels the position
is neutral (Fig. 8).

V
-- FIG. 9 FIG. 10
The arch before operation (Fig. 9) and afterwards (Fig. 10).

THE JOURNAL OF BONE AND JOINT SURGERY


TREATMENT OF MOBILE FLAT FOOT BY DISPLACEMENT OSTEOTOMY OF THE CALCANEUS 99

FIG. II FIG. 12
A lateral radiograph before operation to show the talo-navicular subluxation and the flat arch (Fig. II). After
operation (Fig. 12) the talo-navicular subluxation is decreased and there is improvement of the longitudinal
arch: the mid-talar line approximates to the normal.

the bone, care being taken to ensure that the ends of the wires are buried beneath the skin.
Routine closure of the skin only is undertaken and a padded below-knee plaster is applied with
the ankle held in neutral position.
Post-operative care-Three weeks after operation the sutures and the wires are removed and
a new below-knee plaster is applied. The patient is allowed to bear weight in this for three
weeks. Radiographs are then taken, out of plaster, and if union has occurred the limb is left
free to mobilise. Occasionally physiotherapy is required.

FIG. 13 FIG. 14
Marked abduction of the forefoot before operation
(Fig. 13). After operation (Fig. 14) there is a
decrease of the abduction of the forefoot and the
mid-talar line approximates to the normal.

CLINICAL MATERIAL
In the Bristol clinical area there was a total of thirty-four feet operated on in nineteen
patients. All the patients were traced and examined. The follow-up time ranged from six
years to a few months. The age range was nine to fifty-five years but only two patients were
over seventeen at the time of operation; one was thirty-nine years and the other was fifty-five
years. The mean age was twelve years when these two are excluded. There were eleven females
and eight males. All had simple mobile flat foot with a marked valgus element and without
neurological abnormalities.

VOL. 53 B, NO. 1, FEBRUARY 1971


100 E. KOUTSOGIANNIS

RESULTS
The results were judged on three criteria: the shape of the foot, fatigue and shoe wear.
Shape of the foot. Hee/-Seventeen patients (thirty feet) lost the valgus deformity of the heel
and with this the eversion element in the pull of the calcaneal tendon (Figs. 7 and 8). Arch-
The longitudinal arch showed some improvement in twenty-five feet (thirteen patients), most
of whom had some arch before operation (Figs. 9 and 10). Nine severely deformed flat feet
(six patients) failed to show any improvement. Forefoot abduction-This was found to run
parallel with the improvement of the longitudinal arch. Although the improvement was
difficult to demonstrate in the photographs, it was easily apparent on the radiographs.
Fatigue-This was recorded as definitely improved in seventeen patients. They were very
pleased to be able to indulge in normal activities such as walking, running, football and
dancing. In addition the spring in the gait was considerably improved.
Shoe wear-In seventeen patients there was no problem with footwear after the operation.
The parents of the young children were particularly appreciative.
Failures-None of these features improved in the two patients (four feet) with persisting valgus
deformity of the heel. These failures were probably due to a tight calcaneal tendon or to
inadequate displacement atthe osteotomy. In both cases there was inability to dorsiflex to
the right angle with the heel in neutral position and the knee straight.
Radiographic appearance-Standing radiographs before and after operation are essential for
a proper assessment of the effect of operation.
Improvement in the appearance of the foot was always associated with decrease in the
tabo-navicular subluxation. This improvement can be divided into components : the longitudinal
arch and the forefoot. Figures 1 1 to 14 illustrate the typical radiographic appearance before
and after operation. Only very slight improvement in the elevation of the anterior end of
the calcaneus was seen in the lateral radiographs.

CONCLUSION
1 . Thirty-four cases of calcaneal osteotomy for mobile flat foot in nineteen patients are
reviewed.
2. The function was markedly improved in seventeen of the nineteen patients.
3. The operation was successful in correcting the valgus deformity of the heel in thirty of
the thirty-four feet. it was less successful in improving the longitudinal arch, especially when
the flat foot deformity was severe.

My thanks are due to Mr A. L. Eyre-Brook and Mr D. M. Jones for their help and criticism in the preparation
of this paper. They and Mr H. K. Lucas, Mr M. P. McCormack and Mr H. E. D. Griffiths kindly gave me
access to their patients and records.

REFERENCES
DWYER, F. C. (1960): Osteotomy of the Calcaneum in the Treatment of Grossly Everted Feet with Special
Reference to Cerebral Palsy. In Huiti#{232}me Congr#{233}s Internationale de Chirurgie Orthop#{233}dique, New York,
Septembre
4-9 1960. Soci#{233}t#{233}
Internationale de Chirurgie Orthop#{233}dique et de Traumatologie, p. 892. Bruxelles:
Imprimerie des Sciences.
Low, J. P. (1923): Correction of Extreme Flatfoot. Value of Osteotomy
Displacement of Os Calcis and Inward
of Posterior Fragment (Gleich Operation). Journal of the 81, 1502.
American Medical Association,
SILVER, C. M., SIMON, S. D., SPINDELL, E., LITCHMAN, H. M., and SCALA, M. (1967): Calcaneal Osteotomy for
Valgus and Varus Deformities of the Foot in Cerebral Palsy. Journal of Bone and Joint Surgery, 49-A, 232.

THE JOURNAL OF BONE AND JOINT SURGERY


Pathophysiology/Complications
O R I G I N A L A R T I C L E

Probe-to-Bone Test for Diagnosing


Diabetic Foot Osteomyelitis
Reliable or relic?
LAWRENCE A. LAVERY1 EDGAR J.G. PETERS3 expensive and not universally available
DAVID G. ARMSTRONG2 BENJAMIN A. LIPSKY4 (9 –21). In 1995, Grayson et al. (22) de-
scribed a clinical technique they used in
diabetic patients with a foot infection con-
OBJECTIVE — We sought to assess the accuracy of the probe-to-bone (PTB) test in diagnos- sisting of exploring the wound for palpa-
ing foot osteomyelitis in a cohort of diabetic patients with bone culture proven disease. ble bone with a sterile blunt metal probe.
Their most important finding was that the
RESEARCH DESIGN AND METHODS — In this 2-year longitudinal cohort study, we probe-to-bone (PTB) test had a positive
enrolled 1,666 consecutive diabetic individuals who underwent an initial standardized detailed predictive value of 89%, leading them to
foot assessment, followed by examinations at regular intervals. Patients were instructed to im- conclude that a positive test usually made
mediately come to the foot clinic if they developed a lower-extremity complication. For all
imaging studies for diagnosing osteomy-
patients with a lower-extremity wound, we compared the results of the PTB test with those of a
culture of the affected bone. We called PTB positive if the bone or joint was palpable and defined elitis unnecessary (22). Since then, many
osteomyelitis as a positive bone culture. have considered a positive PTB sufficient
evidence for osteomyelitis. In the study by
RESULTS — Over a mean of 27.2 months of follow-up, 247 patients developed a foot wound Grayson et al., however, the prevalence of
and 151 developed 199 foot infections. Osteomyelitis was found in 30 patients: 12% of those osteomyelitis in their population with “se-
with a foot wound and 20% in those with a foot infection. When all wounds were considered, the vere limb-threatening infections” was
PTB test was highly sensitive (0.87) and specific (0.91); the positive predictive value was only 66%. Furthermore, the investigators did
0.57, but the negative predictive value was 0.98. not obtain a bone specimen for analysis,
the criterion standard for the diagnosis,
CONCLUSIONS — The PTB test, when used in a population of diabetic patients with a foot
from all patients and used histopatholog-
wound among whom the prevalence of osteomyelitis was 12%, had a relatively low positive
predictive value, but a negative test may exclude the diagnosis. ical rather than microbiological confirma-
tion to diagnose osteomyelitis. To assess
Diabetes Care 30:270 –274, 2007 the value of the PTB test in an unselected
population of individuals with diabetes,
we conducted the test as part of a prospec-

I
ndividuals with diabetes have an tion and prolonged antibiotic therapy
tive cohort study of foot complications in
⬃25% lifetime risk of developing a foot (6,7).
diabetic patients and confirmed the pres-
complication (1), the most common of While diagnosing osteomyelitis is im- ence of osteomyelitis by bone culture.
which is skin ulceration. Over half of portant, it is unfortunately also difficult.
these foot wounds may eventually be- Clinical and laboratory signs and symp-
come infected, which greatly increases the toms are generally unhelpful (6,7). Bone RESEARCH DESIGN AND
risk of lower-extremity amputation (2–5). infection may not show up on plain radio- METHODS — As part of a diabetes
While most diabetic foot infections in- graphs in the first 2 weeks, and any X-ray disease management program to study
volve only the soft tissue, bone involve- abnormalities detected may be caused by and prevent lower-extremity complica-
ment occurs in 20 – 66% of cases (6 – 8). the neuropathic bone disorders that fre- tions and in cooperation with two large
Furthermore, foot infections complicated quently occur in diabetes. More accurate primary care physician groups in south
by osteomyelitis generally have a worse imaging studies, such as radionuclide Texas, we prospectively enrolled 1,666
outcome and often require surgical resec- scans or magnetic resonance imaging, are patients in an observational trial over an
8-month period. As part of a systematic
● ● ● ● ● ● ● ● ● ● ● ● ● ● ● ● ● ● ● ● ● ● ● ● ● ● ● ● ● ● ● ● ● ● ● ● ● ● ● ● ● ● ● ● ● ● ● ● ●
screening program, we documented each
patient’s medical history for all potential
From the 1Department of Surgery, Scott and White Hospital, Texas A&M University Health Science Center
College of Medicine, Temple, Texas; 2Scholl’s Center for Lower Extremity Ambulatory Research (CLEAR) at
foot complications and screened them for
Rosalind Franklin University of Medicine and Science, North Chicago, Illinois; the 3Department of Infectious established risk factors (23). Patients were
Diseases, Leiden University Medical Center, Leiden, the Netherlands; and the 4VA Puget Sound Heath Care then seen at regular intervals (i.e., every
System and the Department of Medicine, University of Washington, Seattle, Washington. 2–12 months, depending on their foot
Address correspondence and reprint requests to Lawrence A. Lavery, Department of Surgery, Scott and risk classification) for routine foot care
White Hospital, 703 Highland Spring Lane, Georgetown, TX 78628. E-mail: llavery@swmail.sw.org.
Received for publication 26 July 2006 and accepted in revised form 19 October 2006. and repeat evaluations (24). In addition,
Abbreviations: PTB, probe to bone. all patients were instructed to immedi-
A table elsewhere in this issue shows conventional and Système International (SI) units and conversion ately return to the foot clinic if they devel-
factors for many substances. oped any foot complication. We followed
DOI: 10.2337/dc06-1572
© 2007 by the American Diabetes Association.
the patients for an average of 27.2 months
The costs of publication of this article were defrayed in part by the payment of page charges. This article must therefore be hereby (range 4 –32) and tracked all pertinent
marked “advertisement” in accordance with 18 U.S.C. Section 1734 solely to indicate this fact. clinical outcomes, verifying all hospital

270 DIABETES CARE, VOLUME 30, NUMBER 2, FEBRUARY 2007


Lavery and Associates

Table 1—Characteristics of patients with diabetic foot wound by osteomyelitis status

Osteomyelitis
Yes (n ⫽ 30) No (n ⫽ 217) P value Relative risk 95% CI
Age ⬎70 years 51.3 52.6 0.85 0.99 0.97–1.01
Male 58.6 52.1 0.51 1.26 0.63–2.53
Diabetes duration (years) 17.0 ⫾ 10.1 13.0 ⫾ 9.6 0.03 1.03 1.00–1.06
BMI (kg/m2) 28.7 ⫾ 6.2 29.6 ⫾ 7.3 0.53 0.98 0.93–1.04
Peripheral neuropathy 83.3 72.4 0.2 1.79 0.71–4.47
Peripheral vascular disease 43.3 37.8 0.6 1.22 0.62–2.40
Wound depth
Full thickness skin 10.0 73.7 1.00
To fascia or tendon 3.3 17.1 0.75 1.43 0.15–13.37
To bone/joint 86.7 9.2 ⬍0.001 30.71 9.73–96.93
Ulcer duration (days) 267 ⫾ 284 169 ⫾ 268 0.06 1.001 1.000–1.001
Data are percent or means ⫾ SD.

admissions and lower-extremity amputa- tage, or rongeur at the time of debride- mented in 30 patients, representing 20%
tions with claims data. The disease man- ment or through sites that were of the 150 infected patients and 12% of all
agement program’s foot clinic was the noncontiguous with the wound. Speci- 247 with a foot wound.
primary source for foot care, as well as for mens were transferred to a sterile con- The PTB test was performed in all of
referral and consultation for diabetes- tainer or transport tube with culture the 247 patients with a wound; it was pos-
related lower-extremity complications. media and quickly transported to the clin- itive in 46 (18.6%), 26 (56.5%) of whom
This project was approved by our institu- ical microbiology laboratory. We used the had osteomyelitis. The test was positive in
tional review board. results of bone culture to determine the 26 (86.7%) of the 30 with culture-proven
We defined a foot wound as a full presence or absence of bone infection. A bone infection, as well as in 20 (9.2%) of
thickness lesion involving any portion of positive culture was defined as growth of the 217 without osteomyelitis. Among the
the foot or ankle (25–27). We excluded any organism from the bone specimen. 150 patients with a clinically infected
wounds characterized as blisters, minor Although our data forms did not specifi- wound, the test was positive in 46
lacerations, or abrasions (n ⫽ 16). We de- cally record information on antimicrobial (30.7%). There were no complications at-
fined a wound infection clinically, by cri- treatment in all cases, most patients pre- tributable to the PTB test.
teria consistent with the International sented with an acute wound and were not The values for sensitivity, specificity,
Working Group guidelines (28), i.e., the receiving any antibiotic therapy. We fol- and positive and negative predictive val-
presence of wound purulence or at least lowed all patients with a foot wound until ues of the PTB test for all patients with a
two signs or symptoms of local inflamma- it either healed or required surgical foot wound and for the patients with a
tion or systemic symptoms of infection intervention. clinically infected foot wound are shown
with no other apparent cause. We evalu- To assess the value of PTB in diagnos- in Table 2. The sensitivity was 87% for
ated all wounds to determine the extent of ing osteomyelitis, we calculated the sen- both groups (i.e., all wounds and infected
soft tissue involved and for any evidence sitivity, specificity, and positive and wounds), while the specificity was 91%
of bone infection (osteomyelitis) negative predictive values of the test using for all wounds and 87% for infected
(6,29,30). As part of this evaluation, each the results of the bone culture as the cri- wounds. The negative predictive value
patient underwent the PTB test, con- terion standard. We calculated statistical was extremely high (96 –98%), but the
ducted by one of two experienced podia- values using SPSS version 11.0 for Mac- positive predictive value was only 57–
trists using a sterile probe to gently intosh (SPSS, Chicago, IL) and Diagnostic 62%. The positive likelihood ratio was 9.4
explore the wound. We defined a positive and Agreement Statistics DAG Software for all wounds and 6.5 for infected
test as palpating a hard or gritty substance (Mental Health Research Institute, wounds, similar to the values for the neg-
that was presumed to be bone or joint Parkville, Victoria, Australia). ative likelihood ratios for both popula-
space. Each patient with a clinically in- tions.
fected wound also underwent a series of RESULTS — The demographic and
plain radiographs and had additional im- clinical characteristics of the patients we CONCLUSIONS — Osteomyelitis of
aging studies as indicated. If, based on the enrolled are shown in Table 1. Over a the foot in individuals with diabetes is of-
clinical examination (other than the PTB mean of 27.2 months of follow-up, 247 ten difficult to diagnose. Bone biopsy is
test) and imaging studies, we thought (14.8%) of the 1,666 enrolled patients de- considered the criterion standard for the
bone infection was possible, the patient veloped a foot wound and 151 (9.1%) de- diagnosis. While histopathological defini-
underwent bone biopsy. Using aseptic veloped 199 foot infections. One patient tions may be useful for diagnosing osteo-
techniques, we obtained specimens for with cellulitis did not have a wound, pre- myelitis, most prefer microbiological
culture, either in the clinic or operating cluding conducting the PTB test. All of the methods (6,31). Many clinicians (and pa-
room, following standard surgical skin patients with osteomyelitis presented tients) are hesitant to undertake this inva-
preparation with betadiene. We obtained with signs and symptoms of a soft tissue sive and rather expensive procedure.
bone specimens by needle aspirate, curet- foot infection. Bone infection was docu- Thus, clinicians have sought clinical evi-

DIABETES CARE, VOLUME 30, NUMBER 2, FEBRUARY 2007 271


PTB test for diabetic foot osteomyelitis

Table 2—Statistical analysis of the PTB test for diagnosing osteomyelitis in all foot wounds biopsy was performed by Grayson et al.,
and in clinically infected wounds they histologically defined osteomyelitis
(in 46 of 50 cases by the presence of in-
All wounds value Infected wounds value flammatory cells, fibrosis, necrosis, and
Statistic (n ⫽ 247) (n ⫽ 150) reactive bone), while we defined it micro-
biologically (by a positive culture of a
Sensitivity 0.87 (0.71–0.96) 0.87 (0.69–0.96) bone specimen). Because most of our pa-
Specificity 0.91 (0.89–0.92) 0.87 (0.79–0.92) tients presented with an acute foot
Positive predictive value 0.57 (0.46–0.62) 0.62 (0.46–0.76) wound, we believe that few were receiv-
Negative predictive value 0.98 (0.96–0.99) 0.92 (0.91–0.99) ing antibiotic therapy, enhancing the
Positive likelihood ratio 9.40 (6.05–14.61) 6.50 (4.03–10.48) value of a microbiologically based diagno-
Negative likelihood ratio 6.81 (2.73–16.97) 6.50 (2.60–16.23) sis. Thus, it is possible that they missed
Data in parentheses are 95% CI. cases of osteomyelitis that did not have
histological changes (false negatives) or
that we included cases that represented
dence to help them determine what pa- diological imaging (in the rest). They microbial contamination (false positives)
tients were likely to have diabetic foot calculated that the PTB test had a sensitiv- of the bone specimen. Our patient popu-
osteomyelitis. Unfortunately, local in- ity of 66%, specificity of 85%, positive lation is probably more representative of
flammatory signs and symptoms may be predicative value of 89%, and a negative those in a typical clinical practice where
blunted because of diabetes-related vas- predictive value of 56% (22). Our study the PTB would be most commonly used.
cular insufficiency, peripheral neuropa- evaluated more than three times as many Our study had several potential limita-
thy (32), and leukocyte dysfunction (33). patients with a foot wound and more than tions. First, we did not perform histological
While clinical findings (34) or elevations twice as many with a foot infection. Un- examination of the bone specimens to
in hematological inflammatory markers like in the study by Grayson et al., our compare against the culture results.
(e.g., white blood cell count, erythrocyte patients were identified (and largely Rather, we elected to use a positive bone
sedimentation rate [35], or C-reactive treated) in an outpatient setting. Further- culture as our criterion standard. We did
protein [36]) may be helpful (37,38), more, in all of our patients, osteomyelitis so because it is often difficult to obtain an
these are not sufficiently accurate for di- was defined exclusively by a positive bone
adequate core of bone from the small
agnosis (3,4,35,39 – 45). Furthermore, culture. We found very little difference in
bones of the feet (especially toes) to allow
evaluating published reports of the sensi- positive and negative predictive values
histopathological analysis and because
tivity, specificity, and predictive value of when we compared PTB results in all pa-
the criteria for histologically diagnosing
various diagnostic methods is compli- tients who had a wound with the subset
osteomyelitis are not well-defined. Fur-
cated by inconsistent operational defini- who had clinical signs of infection. In our
thermore, because we believe that most of
tions and outcome measures, as well as patient population, the PTB had high sen-
the variability in the prevalence of osteo- sitivity and specificity, but because of the our patients were not receiving antibiotic
myelitis in the populations studied (46). lower prevalence of osteomyelitis, our therapy at the time the bone biopsy was
It is not surprising, therefore, that the positive predictive value was only 57– taken and they underwent careful wound
clinical assessment for diagnosing osteo- 62%. Thus, a positive PTB only slightly cleansing and debridement before the
myelitis has a reported sensitivity ranging increased the probability of osteomyelitis procedure, we thought that the risk of
from 0 to 54% (9,20,47,48). Various im- over tossing a coin. The negative predic- false negative or positive results was low.
aging studies, especially magnetic reso- tive value, however, was considerably Additionally, for samples collected in this
nance, certainly enhance the accuracy of higher, at 96 –98%. A negative test, there- study and in our greater clinical experi-
diagnosing osteomyelitis, but these are fore, argues strongly against the diagnosis ence, readings of histological specimens
expensive, time-consuming, and not uni- of osteomyelitis. These results confirm often refer to signs of inflammation or in-
versally available (49,50). Thus, clini- the importance of disease prevalence in flammatory cells but do not specifically
cians have sought a simple inexpensive assessing any test for making the diagno- describe osteomyelitis. Second, we did
bedside test to help determine which pa- sis of diabetic foot osteomyelitis (46). not conduct a bone biopsy on patients
tients should undergo more extensive At least three factors may have con- with a foot wound in whom there was no
evaluations. tributed to the apparent disparity in out- suspicion of bone involvement. While
Since its introduction, the PTB tech- comes between our study and that of work in this area suggests that bone bi-
nique has been widely used for evaluating Grayson et al. (51). First, the lower posi- opsy is both safe (52,53) and helpful (49),
diabetic patients with a foot wound. Pal- tive predictive value in our population we believed it would be unethical to do
pation of bone with a metal probe is a may be attributable to their lower preva- this procedure on patients with no suspi-
simple bedside procedure predicated on lence of osteomyelitis (20 vs. 66%) (46). cion of osteomyelitis. As previously
the concept that if the probe can reach the Second, all of the patients in the Grayson stated, none of the patients who did not
bone, so can infectious bacteria. In the et al. study required hospitalization for undergo a biopsy were later found to have
report by Grayson et al. (51) on 76 hos- severe foot infections, which required pa- developed osteomyelitis. Because the av-
pitalized patients enrolled in a diabetic rental antibiotics. Our study population erage follow-up for patients in this popu-
foot infection antibiotic trial, 66% were was derived from patients who mostly lation was 27 months and our group was
found to have osteomyelitis, defined by presented in a clinic setting, and only the sole source of diabetic foot referral, it
histology on bone biopsy (in most sub- 61% of patients with a foot wound had was unlikely that we missed any cases of
jects) and by surgical exploration or ra- evidence of infection. Third, when bone bone infection. Third, the PTB was con-

272 DIABETES CARE, VOLUME 30, NUMBER 2, FEBRUARY 2007


Lavery and Associates

ducted by one of two podiatrists, but we 14. Unal SN, Birinci H, Baktiroglu S, Cantez
References S: Comparison of Tc-99m methylene
did not test the interrater reliability. 1. Singh N, Armstrong DG, Lipsky BA: Pre-
We were only able to find two other diphosphonate, Tc-99m human immune
venting foot ulcers in patients with diabe- globulin, and Tc-99m-labeled white
studies in the literature of the PTB test in tes. JAMA 293:217–228, 2005 blood cell scintigraphy in the diabetic
patients with a diabetic foot wound. In a 2. Lavery LA, Armstrong DG, Wunderlich foot. Clin Nucl Med 26:1016 –1021, 2001
recent brief report, Shone et al. (54) de- RP, Boulton AJM, Tredwell JL: Diabetic 15. Poirier JY, Garin E, Derrien C, Devillers A,
scribed 81 patients with 104 foot wounds foot syndrome: evaluating the prevalence Moisan A, Bourguet P, Maugendre D: Di-
on whom they did the PTB test. They did and incidence of foot pathology in Mexi- agnosis of osteomyelitis in the diabetic
can Americans and non-Hispanic whites foot with a 99mTc-HMPAO leucocyte
not diagnose osteomyelitis by bone bi- from a diabetes disease management co-
opsy but rather clinically (mostly by phys- scintigraphy combined with a 99mTc-
hort. Diabetes Care 26:1435–1438, 2003 MDP bone scintigraphy. Diabetes Metab
ical examination and plain X-rays, with 3. Armstrong DG, Lipsky BA: Advances in 28:485– 490, 2002
bone histology in a minority). Their pa- the treatment of diabetic foot infections. 16. Sarikaya A, Aygit AC, Pekindil G: Utility
tients included both those in whom the Diabetes Technol Ther 6:167–177, 2004 of 99mTc dextran scintigraphy in diabetic
diagnosis had already been made and 4. Armstrong DG, Lipsky BA: Diabetic foot patients with suspected osteomyelitis of
those in whom it was made later. Interest- infections: stepwise medical and surgical the foot. Ann Nucl Med 17:669 – 676,
ingly, their results were similar to ours, management. Int Wound J 1:123–132, 2003
2004 17. Yuh WT, Corson JD, Baraniewski HM,
i.e., PTB had a positive predictive value of 5. Lavery LA, Armstrong DG, Wunderlich
53% and a negative predictive value of Rezai K, Shamma AR, Kathol MH, Sato Y,
RP, Mohler MJ, Wendel CS, Lipsky BA: el-Khoury GY, Hawes DR, Platz CE, et al.:
85%. They diagnosed osteomyelitis in 19 Risk factors for foot infections in persons Osteomyelitis of the foot in diabetic pa-
(24%) of their patients, a prevalence sim- with diabetes mellitus. Diabetes Care 29: tients: evaluation with plain film, 99mTc-
ilar to that in our study (20%). Balsells et 1288 –1293, 2006 MDP bone scintigraphy, and MR imaging.
al. (55) performed a PTB test in a series of 6. Lipsky BA: Osteomyelitis of the foot in AJR Am J Roentgenol 152:795– 800, 1989
33 episodes of foot ulceration (on 28 dia- diabetic patients. Clin Infect Dis 25:1318 – 18. Enderle MD, Coerper S, Schweizer HP,
betic patients) that required the patient to 1326, 1997 Kopp AF, Thelen MH, Meisner C, Pressler
7. Lipsky BA, Berendt AR, Deery HG, Embil H, Becker HD, Claussen C, Häring HU,
be hospitalized. Among the 21 who had JM, Joseph WS, Karchmer AW, LeFrock
osteomyelitis (defined by either positive Luft D: Correlation of imaging techniques
JL, Lew DP, Mader JT, Norden C, Tan JS: to histopathology in patients with dia-
nuclear medicine scans or characteristic Diagnosis and treatment of diabetic foot betic foot syndrome and clinical suspi-
X-ray changes associated with a foot ul- infections. Clin Infect Dis 39:885–910, cion of chronic osteomyelitis: the role of
cer), only 7 (33%) had a positive PTB. 2004 high-resolution ultrasound. Diabetes Care
Unfortunately, they reported no data on 8. Norden C: Bone and joint infection. Curr 22:294 –299, 1999
the results of PTB in the 12 patients who Opin Infect Dis 9:109 –114, 1996 19. Williamson MR, Quenzer RW, Rosenberg
did not have bone infection, limiting the 9. Vesco L, Boulahdour H, Hamissa S, Kretz RD, Meholic AJ, Eisenberg B, Espinosa
S, Montazel JL, Perlemuter L, Meignan M, MC, Hartshorne MF: Osteomyelitis: sen-
ability to evaluate the test’s accuracy in Rahmouni A: The value of combined ra- sitivity of 0.064 T MRI, three-phase bone
this study. dionuclide and magnetic resonance imag- scanning and indium scanning with bi-
If we are to use the PTB test in clinical ing in the diagnosis and conservative opsy proof. Magn Reson Imaging 9:945–
practice, we must understand both its management of minimal or localized os- 948, 1991
value and limitations. Unfortunately, teomyelitis of the foot in diabetic patients. 20. Newman LG, Waller J, Palestro CJ, Her-
some have inappropriately generalized Metabolism 48:922–927, 1999 mann G, Klein MJ, Schwartz M, Har-
the results from the study by Grayson et 10. Rubello D, Casara D, Maran A, Avogaro A, rington E, Harrington M, Roman SH,
Tiengo A, Muzzio PC: Role of anti-granu- Stagnaro-Green A: Leukocyte scanning
al. to all foot (and even other) wounds in
locyte Fab’ fragment antibody scintigra- with 111In is superior to magnetic reso-
various clinical settings (56,57). We have phy (LeukoScan) in evaluating bone nance imaging in diagnosis of clinically
also observed clinicians using devices and infection: acquisition protocol, interpre- unsuspected osteomyelitis in diabetic foot
methods for the test that are quite differ- tation criteria and clinical results. Nucl ulcers. Diabetes Care 15:1527–1530,
ent from those described in the original Med Commun 25:39 – 47, 2004 1992
study. Furthermore, there are no data on 11. Harvey J, Cohen MM: Technetium-99- 21. Lipsky BA, Baker PD, Landon GC, Fernau
the interrater or intrarater reliability of the labeled leukocytes in diagnosing diabetic R: Antibiotic therapy for diabetic foot in-
test. Perhaps most importantly, clinicians osteomyelitis in the foot. J Foot Ankle Surg fections: comparison of two parenteral-
36:209 –214, 1997 to-oral regimens. Clin Infect Dis 24:643–
must realize that the prior prevalence of
12. Devillers A, Garin E, Polard JL, Poirier JY, 648, 1997
osteomyelitis greatly affects the useful- Arvieux C, Girault S, Moisan A, Bourguet 22. Grayson ML, Gibbons GW, Balogh K,
ness of the PTB test. In a population with P: Comparison of Tc-99m-labelled anti- Levin E, Karchmer AW: Probing to bone
“limb-threatening” infections and a high leukocyte fragment Fab’ and Tc-99m- in infected pedal ulcers: a clinical sign of
prevalence of osteomyelitis, a positive HMPAO leukocyte scintigraphy in the underlying osteomyelitis in diabetic pa-
PTB is probably quite helpful in diagnos- diagnosis of bone and joint infections: a tients. JAMA 273:721–723, 1995
ing bone infection. In more typical clini- prospective study. Nucl Med Commun 21: 23. Armstrong DG, Lavery LA, Vela SA,
cal settings, however, this is less likely to 747–753, 2000 Quebedeaux TL, Fleischli JG: Choosing a
13. Devillers A, Moisan A, Hennion F, Garin practical screening instrument to identify
be true, and the PTB test is a better tool to
E, Poirier JY, Bourguet P: Contribution of patients at risk for diabetic foot ulcer-
exclude osteomyelitis. We need further technetium-99m hexamethylpropylene ation. Arch Intern Med 158:289 –292,
studies on this test to answer the remain- amine oxime labelled leucocyte scintigra- 1998
ing questions and to help understand its phy to the diagnosis of diabetic foot infec- 24. Peters EJG, Lavery LA, Armstrong DG: In-
value in different settings. tion. Eur J Nucl Med 25:132–138, 1998 ternational Working Group’s diabetic foot

DIABETES CARE, VOLUME 30, NUMBER 2, FEBRUARY 2007 273


PTB test for diabetic foot osteomyelitis

risk classification: validation in a large An acute phase reaction in diabetic pa- Stagnaro-Green A: Unsuspected osteomy-
population based cohort. Diabetologia 48 tients with foot ulcers. Cardiovasc Surg elitis in diabetic foot ulcers: diagnosis and
(Suppl. 1):A97, 2005 5:32–36, 1997 monitoring by leukocyte scanning with
25. Lavery LA, Armstrong DG, Harkless LB: 37. Pakarinen TK, Laine HJ, Honkonen SE, indium in 111 oxyquinoline. JAMA 266:
Classification of diabetic foot wounds. J Peltonen J, Oksala H, Lahtela J: Charcot 1246 –1251, 1991
Foot Ankle Surg 35:528 –531, 1996 arthropathy of the diabetic foot: current 49. Ertugrul MB, Baktiroglu S, Salman S, Unal
26. Armstrong DG, Lavery LA, Harkless LB: concepts and review of 36 cases. Scand S, Aksoy M, Berberoglu K, Calangu S: The
Validation of a diabetic wound classifi- J Surg 91:195–201, 2002 diagnosis of osteomyelitis of the foot in
cation system: the contribution of depth, 38. Lee SS, Chen CY, Chan YS, Yen CY, Chao diabetes: microbiological examination vs.
infection, and ischemia to risk of amputa- EK, Ueng SW: Hyperbaric oxygen in the magnetic resonance imaging and labelled
tion. Diabetes Care 21:855– 859, 1998 treatment of diabetic foot infection. leucocyte scanning. Diabet Med 23:649 –
27. Oyibo SO, Jude EB, Tarawneh I, Nguyen Changgeng Yi Xue Za Zhi 20:17–22, 1997 653, 2006
HC, Harkless LB, Boulton AJ: A compari- 39. Leichter SB, Allweiss P, Harley J, Clay J,
50. Tan PL, Teh J: MRI of the diabetic foot:
son of two diabetic foot ulcer classifica- Kuperstein-Chase J, Sweeney GJ, Kolkin
differentiation of infection from neuro-
tion systems: the Wagner and the J: Clinical characteristics of diabetic pa-
pathic change. Br J Radiol. In press
University of Texas wound classification tients with serious pedal infections. Me-
systems. Diabetes Care 24:84 – 88, 2001 tabolism 37 (Suppl. 1):22–24, 1988 51. Grayson ML, Gibbons GW, Habershaw
28. International Working Group on the Di- 40. Armstrong DG, Lavery LA, Sariaya M, GM, Freeman DV, Pomposelli FB, Rosen-
abetic Foot: International Consensus on the Ashry H: Leukocytosis is a poor indicator blum BI, Levin E, Karchmer AW: Use of
Diabetic Foot. Apelqvist J, Bakker K, Van of acute osteomyelitis of the foot in diabe- ampicillin/sulbactam versus imipenem/
Houtum WH, Nabuurs-Franssen MH, tes mellitus. J Foot Ankle Surg 35:280 – cilastatin in the treatment of limb-threat-
Schaper NC, Eds. Maastricht, the Nether- 283, 1996 ening foot infections in diabetic patients.
lands, International Working Group on 41. Lavery LA, Armstrong DG, Quebedeaux Clin Infect Dis 18:683– 693, 1994
the Diabetic Foot, 1999 TL, Walker SC: Puncture wounds: the fre- 52. Senneville E, Melliez H, Beltrand E, Le-
29. Lipsky BA: Diabetic foot infections: quency of normal laboratory values in the gout L, Valette M, Cazaubiel M, Cordon-
pathophysiology, diagnosis, and treat- face of severe foot infections of the foot in nier M, Caillaux M, Yazdanpanah Y,
ment. Int J Dermatol 30:560 –562, 1991 diabetic and non-diabetic adults. Am J Mouton Y: Culture of percutaneous bone
30. Lipsky BA: Evidence-based antibiotic Med 101:521–525, 1996 biopsy specimens for diagnosis of diabetic
therapy of diabetic foot infections. FEMS 42. Armstrong DG, Perales TA, Murff RT, foot osteomyelitis: concordance with ul-
Immunol Med Microbiol 26:267–276, Edelson GW, Welchon JG: Value of white cer swab cultures. Clin Infect Dis 42:57–
1999 blood cell count with differential in the 62, 2006
31. Zuluaga AF, Galvis W, Saldarriaga JG, acute diabetic foot infection. J Am Podiatr 53. Kessler L, Piemont Y, Ortega F, Lesens O,
Agudelo M, Salazar BE, Vesga O: Etiologic Med Assoc 86:224 –227, 1996 Boeri C, Averous C, Meyer R, Hansmann
diagnosis of chronic osteomyelitis: a pro- 43. Bagdade JD, Root RK, Bulger RJ: Impaired Y, Christmann D, Gaudias J, Pinget M:
spective study. Arch Intern Med 166:95– leukocyte function in patients with poorly Comparison of microbiological results of
100, 2006 controlled diabetes. Diabetes 23:9 –17, needle puncture vs. superficial swab in
32. Edmonds M, Foster A: The use of antibi- 1974 infected diabetic foot ulcer with osteomy-
otics in the diabetic foot. Am J Surg 187: 44. Molinar DM, Palumbo PH, Wilson WR, elitis. Diabet Med 23:99 –102, 2006
25S–28S, 2004 Ritts RE: Leukocyte chemotaxis in dia- 54. Shone A, Burnside J, Chipchase S, Game
33. Oncul O, Yildiz S, Gurer US, Yeniiz E, betic patients and their first degree rela- F, Jeffcoate W: Probing the validity of the
Qyrdedi T, Top C, Gocer P, Akarsu B, tives. Diabetes 25:880 – 889, 1976
probe-to-bone test in the diagnosis of os-
Cevikbas A, Cavuslu S: Effect of the func- 45. Lipsky BA, Berendt AR, Embil J, De Lalla
teomyelitis of the foot in diabetes (Letter).
tion of polymorphonuclear leukocytes F: Diagnosing and treating diabetic foot
Diabetes Care 29:945, 2006
and interleukin-1 beta on wound healing infections. Diabetes Metab Res Rev 20
in patients with diabetic foot infections. (Suppl. 1):S56 –S64, 2004 55. Balsells M, Viade J, Millan M, Garcia JR,
J Infect. In press 46. Wrobel JS, Connolly JE: Making the diag- Garcia-Pascual L, del Pozo C, Anglada J:
34. Schinabeck MK, Johnson JL: Osteomyeli- nosis of osteomyelitis: the role of preva- Prevalence of osteomyelitis in non-heal-
tis in diabetic foot ulcers: prompt diagno- lence. J Am Podiatr Med Assoc 88:337– ing diabetic foot ulcers: usefulness of ra-
sis can avert amputation. Postgrad Med 343, 1998 diologic and scintigraphic findings.
118:11–15, 2005 47. Darouiche RO, Landon GC, Klima M, Diabetes Res Clin Pract 38:123–127, 1997
35. Kaleta JL, Fleischli JW, Reilly CH: The di- Musher DM, Markowski J: Osteomyelitis 56. Caputo GM, Joshi N, Weitekamp MR:
agnosis of osteomyelitis in diabetes using associated with pressure sores. Arch Intern Foot infections in patients with diabetes.
erythrocyte sedimentation rate: a pilot Med 154:753–758, 1994 Am Fam Physician 56:195–202, 1997
study. J Am Podiatr Med Assoc 91:445– 48. Newman LG, Waller J, Palestro CJ, 57. Embil JM, Trepman E: Microbiological
450, 2001 Schwartz M, Klein MJ, Hermann G, Har- evaluation of diabetic foot osteomyelitis.
36. Upchurch GR Jr, Keagy BA, Johnson G Jr: rington E, Harrington M, Roman SH, Clin Infect Dis 42:63– 65, 2006

274 DIABETES CARE, VOLUME 30, NUMBER 2, FEBRUARY 2007


This is an enhanced PDF from The Journal of Bone and Joint Surgery

The PDF of the article you requested follows this cover page.

Changes in tibiotalar area of contact caused by lateral talar shift


PL Ramsey and W Hamilton
J Bone Joint Surg Am. 1976;58:356-357.

This information is current as of December 24, 2007

Reprints and Permissions Click here to order reprints or request permission to use material from this
article, or locate the article citation on jbjs.org and click on the [Reprints and
Permissions] link.
Publisher Information The Journal of Bone and Joint Surgery
20 Pickering Street, Needham, MA 02492-3157
www.jbjs.org
Changes in Tibiotalar Area of Contact
Caused by Lateral Talar Shift
BY PAUL L. RAMSEY, M.D.*, AND WILLIAM HAMILTON, M.D.t, WILMINGTON, DELAWARE

Iro,,i t/U Alfred I. duPont Institute, Wilmington

ABSTRACT: A carbon black transference technique ferred to graph paper to measure the area of contact by
was used to determine the contact area in twenty-three counting the enclosed squares.
dissected tibiotalar articulations, with the talus in Carbon black does not leave a permanent stain on the
neutral position and displaced one, two, four, and six cartilage and therefore permits repeated examinations of
millimeters laterally. The greatest reduction in contact the same specimen. The contact area between the talus and
area occurred during the initiah one millimeter of hat- the tibia was determined with the talus first in normal rela-
eral displacement, the average reduction being 42 per tionship to the tibia and then with it laterally displaced
cent. With further lateral displacement of the talus the one, two, four, and six millimeters (Fig. 1).
contact area was progressivehy reduced but the rate of To maintain the exact displacement during testing,
change for each increment of shift was less marked. metallic spacers milled to the appropriate width were
placed between the talus and medial malleolus, with the
Fractures or ligament injuries about the ankle may re- leading edge of the spacer flush with the anterior border of
sult in widening of the medial part of the mortise as seen the articular cartilage. Each spacer was fixed in place by
on roentgenograms 2 Long-term follow-up studies after means of a threaded central extension that was passed
ankle injuries have shown that significant residual talar through a drill hole in the medial malleolus and held se-
displacement predisposes to an unsatisfactory result 1,3.6 curely with a nut tightened against the medial aspect of the
In an effort to explain such unsatisfactory results, we pos- malleolus. In this way, the position of the spacer and its
tulated that the area of contact between the articular sur- apposition to the articular surface of the medial malleolus
faces of the tibiotalar joint is altered and may contribute to was maintained during each test.
the poor result. We therefore attempted to measure the With respect to position, the talus was adjusted so
changes occurring in this contact area as the talus is dis- that it was as near neutral as could be determined by in-
placed laterally in the ankle mortise. spection based on the tibiotalar relationship while the foot
was in neutral position prior to removal of the soft tissues.
Materials and Methods
The design of the compression device ensured that when
Twenty-three lower extremities amputated for periph- each specimen was retested it was in the same degree of
eral vascular disease were studied. Any specimen with in- plantar flexion-dorsiflexion and of medial-lateral tilt for
fection or gangrene involving the articuhar cartilage of the each increment of lateral displacement.
ankle joint was not used.
Results
The fibuha was removed and all soft tissues about the
ankle were excised, including the ligaments and capsule. The contact areas (range, mean, and standard devia-
The tibia was transected five centimeters above the ankle tion) are listed in Table I. The considerable variation in the
joint. The anatomical unit to be tested thus included the areas of contact can be explained by the normal biological
distal end ofthe tibia and the talus as separate components, variations in the different specimens. This variation ren-
with both completely free of all soft-tissue attachments. ders the measured contact areas of little value when com-
To determine the area
of contact, the distal tibial ar- paring the changes in area in different specimens. There-
ticular surface was coated with powdered carbon black and fore, the measured contact areas with each increment of
rearticulated with the talus, which was placed in neutral lateral displacement were expressed as percentages of the
position with respect to plantar flexion, dorsiflexion, and area obtained with no displacement.
medial and lateral tilt. A compression clamp mounted on a The mean decrease in contact area was 42 per cent
bathroom scale was used to deliver an axially-directed load with one millimeter of lateral talar shift, 14 per cent with
of seventy kilograms on the joint for thirty seconds. When between one and two millimeters, 9 per cent with between
the load was released, a deposit of carbon black was pres- two and four millimeters, and only 3 per cent with between
ent on the
talus, marking the area of contact with the distal four and six millimeters.
end of the tibia. The outline of the deposit was traced on With no talar displacement, the area of tibiotalar con-
wax wrapping paper placed on the talus, and then trans- tact extended across the breadth of the talus and was wide
S Box 269, Wilmington. Delaware 19899.
on the lateral side and narrow on the medial side. Once the
± Thomas Jefferson Hospital. Philadelphia. Pennsylvania 19104. talus was displaced laterally, contact was only apparent on

356 THE JOURNAL OF BONE AND JOINT SURGERY


CHANGES IN TIBIOTALAR AREA OF CONTACT CAUSED BY LATERAL TALAR SHIFT 357

FIG. I
A left
talus viewed from above. showing the contact areas with zero. one, two, four, and six millimeters of talar displaccnient (from left to right).
Note the rapid decrease in contact area and the change in its pattern. With no displacement the major area of contact is on the lateral side but with
lateral displacement the major contact is on the medial side.

the medial and lateral prominences of the talus, there being TABLE I
TIBI0TAI.AR CONTACT ARIA
no demonstrable contact in the mid-portion. Also, the pat-
tern of contact was reversed so that the medial contact area Lateral Talar
was broad and the lateral contact area was narrow (Fig. 1). Displacement Range Mean Standard Deviation
(,z,ii) ((.,p2) (ei2) (,p2)

Discussion 0 2.3-6.7 4.40 1.21

The findings in this study expand those in a previous I 1.0-5.0 2.50 0.95

investigation , in which the talus was tilted laterally 2 and 2 1.1-3.3 I .89 0.50

4 0.9-3.0 1.53 0.59


4 degrees and displaced laterally two millimeters. A
6 0.7-2.5 1.37 0.45
marked decrease of the tibiotahar contact area was noted
but no measurements were recorded.
The unsatisfactory clinical results sometimes as-
sociated with slight widening of the ankhe mortise and mahleolus after ankle injuries. Since the stress per unit area
the 42 per cent reduction in the area of contact between the increases as the total contact area decreases, a decrease in
tibia and talus with one millimeter of lateral displacement contact area may be a factor contributing to a poor result
emphasize the importance of restoring the normal after ankle fracture or dislocation when talar displacement
roentgenographic relationship of the talus to the medial is one millimeter or more.

References
1. BR0DIF. I. A. 0. D.. and DENIJASI, R. A.: The Treatment of Unstable Ankle Fractures. J. Bone and Joint Surg.. 56-B: 256-262. May 1974.
2. DENHAM, R. A.: Internal Fixation for Unstable Ankle Fractures. J. Bone and Joint Surg., 46-B: 206-211. May 1964.
3. Jo, GREGORY: PATZAKIS. M. J.: and HARVEY, J. P., JR.: Precise Evaluation ofthe Reduction ofSevere Ankle Fractures. Technique and Correla-
tion with End Results. J. Bone and Joint Surg. . 56-A: 979-993. July 1974.
4. MullER. M. E.: ALLGOWER, M.; and WII.I.ENEGGER, H.: Technique of Internal Fixation of Fractures, p. 1 14. New York, Springer. 1965.
5. WII.l.ENE;ER, H.: Die Behandlung der Luxationsfrakturen des obenen Sprunggelenks nach biomechanischen Gesichtspunkten. Helvetica chir.
acta, 28: 225-239, 1961.
6. WilSoN, F. C.. and SKII.BREw, L. A.: Long-Term Results in the Treatment of Displaced Bimalleolar Fractures. J. Bone and Joint Surg.. 48-A:
1065-1078, Sept. 1966.

VOL. 58-A, NO. 3, APRIL 976


FOOT& ANKLEINTERNATIONAL
Copyright O 2002 by the American Orthopaedic Foot &Ankle Society, Inc.

Incidence of DVT Following Surgery of the Foot and Ankle

Gregory Solis, M.D.; Terence Saxby, FRACS (Orth)


Jacksonviffe, FL and Brisbane, Austrafia

ABSTRACT return for the first postoperative follow-up, had a history


of DVT or pulmonary embolus, or were currently taking
A prospective study was undertaken to establish the inci- a blood thinner (warfarin or heparin).
dence of deep vein thrombosis (DVT) in patients who had
undergone surgery of the foot and ankle. All consecutive MATERIALS AND METHODS
patients who underwent foot and ankle surgery in the
senior author's practice had duplex ultrasound per- The study group consisted of all patients undergoing
formed of the bilateral calves at the first postoperative foot and ankle surgery by the senior author (T.S.) com-
visit. Of 201 patients, deep calf clots were found in seven mencing in May 1999 in a practice that is exclusive to
patients (3.5%), but none of these showed progression on disorders of the foot and ankle. During this period, 209
follow-up ultrasound or extension proximal to the calf. By patients were treated but seven patients did not have
the authors' criteria, none of the studied patients required the first follow-up in the author's rooms, as they lived in
treatment. The authors feel that the rate and progression country towns some distance away, and one patient had
of DVT after foot and ankle surgery is low and does not surgery on the tibia, leaving 201 patients in the study
require routine prophylaxis. Factors associated with risk group. All of the study patients answered a survey of
of DVT formation were postoperative immobilization, potential risk factors for DVT. All patients undergoing
hindfoot surgery, tourniquet time and advancing age. surgery had a duplex ultrasound of the bilateral lower
extremities from the popliteal vein distally upon the first
INTRODUCTION postoperative visit (average 10.55 days, range six to 18
days after surgery). The senior author's standard rou-
Deep vein thrombosis (DVT) following orthopaedic tine at the time was to screen all postoperative patients
surgery can be a significant cause of morbidity and mor- with ultrasound. All ultrasounds were performed in the
tality. The risks have been studied extensively in the lit- same institution using Advanced Technology
erature, particularly with respect to total joint arthroplas- Laboratories Ultrasound System with a Linear Array 7.4
ty of the lower extremities. This problem has not MHz Broad-band Transducer. Patients were only
received nearly as much attention for surgery of the foot excluded if they did not return for their postoperative
and ankle and is generally not regarded as a significant visit, if they had a history of prior DVT or pulmonary
problem in this field of surgery. The actual incidence of embolism, or patients on anticoagulants (heparin or
DVT following foot and ankle surgery is largely warfarin) for other reasons.
unknown.'O By prospectively studying a series of 201 The senior author's standard practice is to give pro-
eligible, consecutive patients undergoing surgery of the phylactic anticoagulants to patients with prior DVT or
foot and ankle with postoperative ultrasound of the legs, PE, which is the reason these patients were excluded.
the authors hoped to determine the incidence of, and No patients in the study were treated with perioperative
risk factors for, DVT formation in this population. DVT prophylaxis, nor were any patients found to be on
Patients were ineligible for this study if they failed to blood thinners (heparin/coumarin) prior to the surgical
procedure. Therefore, no patients were excluded based
on these criteria.
Corresponding Author:
The procedures were classified with regard to loca-
Terency S. Saxby, FRACS tion using the same criteria as Mizel et aI.l4 Forefoot
Holy Spirit Hospital was defined as extending from the proximal metatarsal
259 Wickham Terrace
Brisbane, Qld. 4000 to distal phalanx, midfoot from proximal metatarsal to
Australia Chopart's joint, hindfoot as talus and caicaneus and
Phone: 0738346640
Fax: 07 3834 6641
ankle as tibiotalar joint and distal leg. Any procedure on
E-mail: forefootQbigpond.com more than one area was graded as being the most prox-
41 1
4 12 SOLS AND SAXBY

imal location that was involved. Procedures were also each affected patient, with a range from one to th{ee. One
scored as being bony or soft-tissue only, with combined of the seven had thrombus in the contralateral limb but not
procedures being placed in the bony category. the operated side. None of the seven patients had any
If a DVT was detected and restricted to the caif, a clinical symptoms associated with the DVT (pain, palpable
repeat ultrasound was performed in one week. Where cords, swelling or Homans' sign).
no progression was found, no treatment was institut- Wald Chi-square testing was used to analyze the vari-
ed."7Progressionof the clot, or any clots found proximaf ables for association with DVT formation, and also with
to the calf, would be treated with heparinization followed occlusive DVT (ODVT) formation (Table 2). Factors found
by ~ournadin.~' No patients in this study had progres- to be associated with DVT formation were: hindfoot sur-
sion of thrombus or DVT proximal to the calf, and there- gery fp=0.02), hindfoot surgery combined with immobi-
fore none required treatment. lization (p=0.02), and increased tourniquet time fp=0.03).
All results were analyzed with statistical software Age was also associated, both as a continuous variable
@AS {2QOl] SASe Proprietary Software Release 8.2 (p=0,051f and a variable categorized by decade @=0.04).
[TS2MO] SAS fnstitute Inc. Cary, NC, USA). Wald Chi- Testing was also done to analyze the incidence of
square testing was used to analyze the association of a occlusive DVT as a separate entity, scoring nonocclu-
number of variables (Table I ) with the formation of DVT, sive DVT as normal. Hindfoot surgery was again asso-
as well as occfusive DVT as a subset of DVT. Variables ciated (p=0.04) as was combined hindfoot surgery with
were tested as Binomial Factors, Continuous Variables immobilization (p=0.03). Unlike the risks for all DVT, nei-
and Categorized Variables. ther age nor tourniquet time were associated with the
Characteristics of risk factors analyzed are outlined in subgroup of occiusive DVT formation.
Table 1. Body Mass lndex was calculated as Body Mass lndex was not associated with overall
BMI=Weight(Kg)/Height2(meters)and stratified into the DVT formation, but was associated with occlusive DVT
following categories: 48.5, 18.5 to 25, 25 to 30, 30 to (ODVT). BMI was associated strongly with ODVT in all
40 and 40+, according to National Institute of Health three modes tested: BMb30 (p=0.04), as a continuous
obesity guidelines10 Age was categorized by decade variable (p=O.Ol) and when grouped in the N!H cate-
(10-19, 20-29 etc.), The variables of hindfoot proce- gories of obesity (p=0.02).
dures and immobilization appeared linked and were
tested as four groups:
1. Nonhindfoot surgery without immobilization,
2. Hindfoot surgery without immobilization, Deep vein thrombosis can be a significant cause of
3. Nonhindfoot surgery with immobilization, and morbidity and mortality following orthopaedic surgery.
4. Hindfoot surgery with immobilization. Studies of total hip and knee replacements have found
high incidences of DVT in untreated populations, but the
RESULTS incidence foilowing foot and ankle surgery is mostly
unknown'4and is largely regarded as a low The
The study population consisted of 201 patients, 83 practical questions most surgeons would wish to know
male and 118 female, with an average age of 45.69 are what is the risk of thromboembolism, and is routine
years (range, 13 to 83). Forefoot surgery accounted for prophylaxis indicated.
46.6% of the procedures, midfoot 8.9%, hindfoot 18.32% Mizel et al.'"ave undertaken the most extensive
and ankle 26.18%. Bony procedures accounted for 550A study to date regarding DVT following foot and ankle
of the total procedures, 80% done under general anes- surgery, evaluating over 2,700 patients. In this study
thetic, and 33.5% had postoperative immobilization. they found an incidence of DVT of 0.22% and an inci-
In the present study population, the rate of DVT was dence of nonfatal pulmonary emboli of 0.15O%. While
found to be low. Of 201 patients, 53 (6.5%) had positive this multicenter study was very significant in terms of
ultrasound. In six f3.074) of these patients the findings the large number of patients, it did suffer two flaws.
were only in the superficial veins and muscular plexus- First, radiological studies to detect DVT were used only
es. These were not considered clinically significant and on patients who had clinical symptoms of thrombi or
were not included in the final analysis. embolism. Because clinical detection alone is inaccu-
Seven of 201 (3.5% total population) had true DVT (per- rate and many thrombi may be a s y m p t o m a t i ~ , ' ~ " " ~ ~ ' ~ ~ ~ ~
oneal, anterior tibial and posterior tibial venous thrombi). the true incidence of thrombi is likely to have been
Five of these seven (2.5% of total population) patients had under-reported. The incidence of clinically significant
thrombi that were occlusive, eliminating ail blood flow in events, however, would be expected to be accurate.
the vein. DVTs involved an average of 1.57 of the six veins Second, the use of thromboembolism prophylaxis was
(two each peroneal, anterior tibia1 and posterior tibialf in "determined by the treating surgeon." This means that
Foot & Ankle InternationalNol. 23, No. 5/May 2002 DEEP VEIN THROMBOSIS AFTER SURGERY 413

Table 1: Variables analyzed for association with the occurrence of Deep Vein Thrombosis (DVT) and Occlusive
Deep Vein Thrombosis (ODVT)
Binomial Factors Continuous Categorized
Risk Factor (yeslno) Variables Variables
Gender (Male) X
Smoker X
Oral contraceptive pill X
Hormone replacement therapy X
History of cancer X
Family history of DVT X
Blood disorderlclotting problems X
Procedure on Bone (v. soft tissue alone) X
General anesthetic X
Postoperative immobilization X
Location-forefoot X
Location-midfoot X
Location-hindfoot X
Location-ankle X
Body Mass Index (BMI) x (>=30, obese) x (BMI units) x (NIH categoriesa)
Age x (in years) x (by decadeb)
Tourniquet time (min) X

"ational Institute of Health BMI categories: 4 8 . 5 , 18.5 to <25, 2 5 to <30, 3 0 t o <40 a n d 40+
"0-19 years, 20-29 years, 30-39 years etc.

the incidence of DVT in untreated patients undergoing ciated with occlusive DVT following foot and ankle sur-
foot and ankle surgery is still unknown, as the two gery were hindfoot surgery with or without immobiliza-
groups were not separated. tion and increasing BMI. Age was not found to be asso-
All patients in this study underwent postoperative ciated with occlusive DVT, and BMI was not found to be
ultrasound to detect thrombosis, and no patient associated with overall DVT formation, only occlusive
received any perioperative DVT prophylaxis. The DVT. A similar study with larger numbers of patients in
patients only had calf ultrasound performed. While this the various groups would give more power to the study.
may have missed proximal clots, the authors feel that This study shows that:
the majority of proximal clots propagate from the 1. DVT following foot and ankle surgery is rare, but
The authors believe that checking the calf only would more prevalent than previously suggested,
give a higher incidence of thrombosis than in studies 2. They are mostly asymptomatic, and
that check only the thigh. Studies have shown that ultra- 3. They do not progress proximally.
sound may be less sensitive (48 to 57%) in the calf Based on these findings and review of the literature,
compared with the thigh (62%) as well as in asympto- the authors agree with Mizel et aI.l4 that routine DVT
matic patient^'^.'^ of which both conditions apply to this prophylaxis is not indicated in patients undergoing foot
study. Thigh veins were not tested as part of the senior and ankle surgery. Even in higher risk patients (hindfoot
author's protocol, partly as a cost-saving measure. surgery, obese, elderly or increased tourniquet time) the
This study was a prospective evaluation of all patients lack of clot progression or symptoms would lead us to
undergoing surgery of the foot and ankle without any believe that prophylaxis is not indicated routinely in
DVT prophylaxis. A low incidence (3.5%) was found in these patients.
this study of thrombus formation in the deep venous Based on the results of this study, the senior author
system of the calf, none of which showed any progres- no longer performs routine ultrasound screening of
sion on follow-up ultrasound. No patients were found to postoperative patients.
have clinical symptoms consistent with thrombosis or
pulmonary embolism at the postoperative visit. REFERENCES
In this study, the risk factors found to be associated
1. Agnelli, G; et a!: Clinical Outcome of Orthopaedic Patients with Negative Lower
with DVT formation following foot and ankle surgery Limb Venography at Discharge. Thrombosis and Haemostasis, 74(4):1042-4, 1995.
were: hindfoot surgery with or without immobilization, 2. Agnelli, G: PostdischargeProphylaxis for Venous Thromboernbolism Among High-
increasing age and tourniquet time. Factors found asso- risk Surgery Patients. Vascular Medicine 351-56, 1998.
4 14 SOLS AND SAXBY foot & Ankle Infernztional/Voi.23, No. 5/May 2002
--- - - - -
Table 2: Results of logistic regression of incidence of Deep Vein Thrombosis (DVT) or DVT (ODVT) on
risk factors
DVT O DVT
Wald Chi-square Wald Chi-square
X
i'" )'(: '"
Risk Factor -- (Probability)
-- --. rubabi
it .I ityf
Gender 0.47 fn.s.1 0 (ns.)
Smoker 0 (n.s.) 0 fns.)
Oral contraceptive pill 0 (n.s.) 0 (n.s.)
Hormone replacement therapy 1. l 0 (n.s.) 0.13 (n.s.)
History of cancer 0.31 (ns.) 0.81 (ns.)
Family history of DVT 0 fn.s.) 0 (8s.)
Blood disorder/ clotting problems 0 Ins.) 0 fns.)
Procedure on Bone fv. so3 tissue alone) 2.06 fn.s.) 1.02 (ns.)
General anesthetic 0.12 (n.s.j 0 (ns.)
Postoperative immobilization 3.76 (P=0.053) 3.50 (P=0.06)
Location- forefoot 2.52 (n.s.) 0 (ns.)
Location- midfoot 0.25 (n.s.1 0.71 (n.s.)
Location- hindfoot 5.45* (P=0.02) 4.29* (P=0.04)
Location- ankle 0.42 (n.s.1 0.06 fn.s.1
Body Mass Index fBMl)>30 2.23 ( n s f 4.07* (P=0.04)
Body Mass Index (continuous] 3.37 fP=0.07) 6.43* (P=O.Ol)
Body Mass Indexb(categorized by National Health Instjfute standards) 2.53 (n.s.) 5.68* (P=0.02)
Age in years (continuous) 3.80* (P=0.051) 0.99 (ns.)
Age (categorized by decade)" 4.39* (P=0,04) 1.51 (ns.)
HindfooVlmmobilizationd(categorize4 5.64* (P=0.02) 4.88' (P=0.03)
Tourniquet time in minutes (cantinrraus) 4.80* (P=0.03) 3.23
"All Chi-squared values h a v e o n e degree of freedom
bl=underweight, ...6=profoundly o b e s e
...
"Decade 0=1-9. 1 = 10-19.. 8=80-89.
i=
a~indfootllmmo'bilization ~ e i t h e rHindfoot nor Immobilization 1= Either Hindfoot o r Immobilization 2 = Both Hindfoot a n d Immobilization
"Signif~cantat 5% level
ns,: Non-significant
- - ..

3. Ascani, 8; e i at: Distiibution and Occiusiveness of Thrombi in Patients with Ubmsound a Sensitive Screening Methd in Diagnosing Deep Vein Thrombosis
Sunteillance Detected Deep Vein Thrombosis after Hip Surgery. Thrombosis and after Hlp Surgery? Thrombosis and Haemostasis: 75(2):242-5. 1396.
Haemostasis 75(2):239-41, 1996. 13. Mitcheti, LJ: Thron>boembolicComplications of Cast Immobtlrzation lor injuries of
4. Bates, SM; Hirsh, J: Treatment of Venous Throboernboiism. Thrombosis and the lower extremities. Clin. Grthop., 108:191-5,1975.
Haemostasis 82(2):870-7, 1999. 14. Mizel, MS; Temple, HT; Michelson, JD; Alvarez, RG; Clanton, TO; Frey, CC;
5. Davidson, HC; Mauu, D; Gage, BF; Jeffrey, RB: Screening for Deep Venous Gegenheimer, AP; Hurwitz, SR; Lutter, LD; Mankey, MG; Mann, RA; Miller, RA;
Thrombosis in Asymptomatic Postoperative Orthopedic Patients Using Color Richardson, EG; Schon, LC; Thompson, FM; Yodlowski, ML:
Doppjer Sonography- anaiysis of prevalence and risk factors. Am. J. Roentgenol, ThromboemboiismAfter Foot and Ankle Surgery- a muliicenier study. Cim. Orihop.,
166:659-62, 1996. 348:160-85. 1998.
6. Hatch, DJ; Magnusson, PG; DiGiovanni, JE: Mini-dose heparin prophylaxis for 15. Paiement, GO; Mendelsohn, C: The Risk of Venous Thromboemboism in the
high-risk patients in pediatric surgeiy J. Am. Podiatry Assoc., 70(2):7343, 1980. Grihopedic Patient: Epidemiologicaland Physioiogical Data. Oithopedics. Feb, 20
7. Hyers TM; Agnelii G; Hull RD; Weg JG; Morris T A Samama M; Taspon V: Suppl:7-9, 1997.
AntithromboticTherapyfor Venous Thromboembolic Disease. Chest, 114(5):561S- 16. Servatjoo, P: Deep venous thrombosis. The dilemma of diagnosis. J Am. Podiatr.
78S, 1998 Supplement Med. Assoc., 87(5):224-32 1997.
8. Kakkar, VV; Howe CT; Flanc C; Clarke MB: Natural History of Postoperative 17. Simon, MA; et al: The Effect of Thigh Tourniquet on the Incidence of Deep Venous
Deep-Vein Thrombosis. Lancet ,Aug 2 ik230-32, 1962. Thrombosis after Operations on the Fore Part of the Foot J. Bone Joint Surg., 64-
9. Kroll, HR; Odderson, IR; Alten, FH: Deep vein thrombi associatedwith the use of Af2]:188-91. 1982.
plastic ankleloot orthoses. Arch. Phys. Med. Rehabi1.,79:576-578, 1938. 16. Thomas, AIIL; O'Dwyer, JA: A Phlebographic Study of the lnadence and
10, National Institute of Health: The Practical Guide: Identification. Evaluation and Significance of Venous Thrombosis in the Foot. Am. J. Roentgenoi. 135 751-754,
Treatment of Oveweight and Obesity in Adults, NIH Publication Number 00-4084, 1978
October 2000. 19. Wells, PS; et al: Accuracy of Ultrasound for the Diagnosis of Deep Venous
11. Paiement, GD; Mendelsahn, C: The risk of venous thrornboembolismin the ortho- Thrombosis in Asymptomatic Patients after Orthopaedic Surgery- a rneta-analysis.
pedic patient: Epidemiologicaland physiological data. Orthopedics, Feb., 20 Suppl: Ann Intern Med. 122:47-53, 1995.
7-9, 1997. 20, Wolf, DW: Case records of the California Podiatry Hospital clinicopatholog~cexer-
12,Magnusson, M; Eriksson, 81; Kaiebo, P; Sivertsson, R: Is Colour Doppier cise- postoperative thrombophlebitis. J. Am. Podiatry Assn , 69(3):207.:0. 1979.
ARTICLE IN PRESS

Journal of Diabetes and Its Complications xx (2008) xxx – xxx


WWW.JDCJOURNAL.COM

Charcot arthropathy of the foot and ankle: modern concepts and


management review
Dane K. Wukich a,⁎, Wenjay Sung b
a
UPMC Comprehensive Foot and Ankle Center, University of Pittsburgh School of Medicine, Pittsburgh, PA 15203, USA
b
UPMC South Side Podiatric Residency Program, Pittsburgh, PA 15203, USA
Received 4 April 2008; received in revised form 14 August 2008; accepted 13 September 2008

Abstract

Charcot arthropathy (Charcot neuroarthropathy, diabetic neuropathic osteoarthropathy, or neuropathic arthropathy) remains a poorly
understood disease, although recent research has improved our level of knowledge regarding its etiology and treatment. The effects of
Charcot arthropathy are almost exclusively seen in the foot and ankle, and the diagnosis is commonly missed upon initial presentation. It has
been well established that this complication of diabetes mellitus severely reduces the overall quality of life and dramatically increases the
morbidity and mortality of patients. However, there are few high-level evidence studies to support management and treatment options at this
point in time. The goal of this study is to evaluate the modern concepts of Charcot arthropathy through a review of the available literature and
to integrate a perspective of management from the authors' extensive experience.
© 2008 Elsevier Inc. All rights reserved.
Key words: Charcot arthropathy; Neuroarthropathy; Neuropathic osteoarthropathy; Diabetes mellitus; Current treatment concepts

1. Introduction clinicians (Rajbhandari, Jenkins, Davies, & Tesfaye, 2002).


The effects of Charcot arthropathy are almost exclusively
Charcot arthropathy of the foot and ankle is a deforming seen in the foot and ankle (Frykberg & Belczyk, 2008);
and destructive process that can lead to increased patient however, the supply of scientific data and evidence-based
morbidity due to gross instability, recurrent ulcerations, treatments is largely based on retrospective studies. Known
and/or amputation (Armstrong & Peters, 2002; Frykberg et as Charcot neuroarthropathy and diabetic neuropathic
al., 2006; Saltzman, Hagy, Zimmerman, Estin, & Cooper, osteoarthropathy (Sanders & Frykberg, 2007), this compli-
2005). Charcot arthropathy has been associated with leprosy, cation of diabetes mellitus severely reduces the overall
toxic exposure, syringomyelia, poliomyelitis, rheumatoid quality of life and dramatically increases the morbidity and
arthritis, multiple sclerosis, congenital neuropathy, and mortality of patients (Gazis et al., 2004; Lee, Blume, &
traumatic injury (Gupta, 1993; Johnson, 1967; Sanders & Sumpio, 2003).
Frykberg, 2007). However, diabetes mellitus has become the
most common etiology in the modern era (Miller &
Lichtman, 1955). The diagnosed cases of Charcot arthro- 2. Historical review and epidemiology
pathy associated with diabetic patients range from 0.08% to
7.5% (Sanders & Frykberg, 2007); however, true prevalence Sir William Musgrave is believed to have first recorded
is likely unknown due to cases undiagnosed by untrained descriptions of “neuropathic arthritis” as a complication of
venereal disease in 1703 (Armstrong & Peters, 2002; Gupta,
1993; Kelly, 1963), but most of the world's medical
⁎ Corresponding author. Tel.: +1 412 586 1546; fax: +1 412 586 1544. communities remained unaware of this disease. In 1831, an
E-mail address: wukichdk@upmc.edu (D.K. Wukich). American physician, John Kearsley Mitchell, reported a case

1056-8727/08/$ – see front matter © 2008 Elsevier Inc. All rights reserved.
doi:10.1016/j.jdiacomp.2008.09.004
ARTICLE IN PRESS
2 D.K. Wukich, W. Sung / Journal of Diabetes and Its Complications xx (2008) xxx–xxx

of “caries [tuberculosis] of the spine” that correlated spinal though diabetes has emerged as the primary cause (Fabrin
disease with hot, swollen, and asymmetrical joints (Mitchell, et al., 2000; Lavery et al., 2003; Miller & Lichtman, 1955;
1831). Silas Weir Mitchell supported J.K. Mitchell's findings Sinha et al., 1972).
by observing similar clinical features from wounded soldiers The prevalence of Charcot arthropathy, defined as the
with spinal injuries during the American Civil War (Mitchell, number of patients currently with the disease divided by the
Morehouse, & Keen, 1864). In 1868, Jean-Martin Charcot study population (Le & Boen, 1995), is unknown, and this
recognized the importance of their findings and described a may be due to the lack of standardized clinical or
hypertrophic process of destructive arthritis. Fifteen years radiographic diagnostic criteria (Rajbhandari et al., 2002).
later, Charcot and Féré (1883) published the first observa- In a series of 68 patients treated for midfoot Charcot
tions of this process in the “short bones and small joints of arthropathy (Myerson, Henderson, Saxby, & Short, 1994),
the foot [la pied tabetique]” (Sanders, 2004). Charcot the clinicians noted that 25% of the patients referred to their
received universal acclaim at the Seventh International facility had not been diagnosed correctly with active Charcot
Medical Congress in London, and the prominent Sir James arthropathy by the referring institution. These patients were
Paget, recognizing the discovery of a distinct pathologic initially diagnosed with infection, gout, arthritis, fracture,
entity, declared that the disease be known as “Charcot's venous insufficiency, or tumors. There have been estimates
disease” (MacCormac & Klockmann, 1881; Sanders & of prevalence by various authors ranging from 0.08% of the
Frykberg, 2007). However, it was Jordan (1936) who general diabetes patient population to 13% of patients
established the link between Charcot arthropathy and presenting at a high-risk diabetic foot clinic (Frykberg &
diabetes mellitus. Belczyk, 2008). Although there has been much published
The incidence of Charcot arthropathy, defined as the work on Charcot arthropathy in the literature, there remains
number of new cases over a set period of time divided by little work on actual evidence for drawing conclusions about
the study population initially without the disease (Le & its prevalence.
Boen, 1995), was first reported by Miller and Lichtman Charcot arthropathy has also been reported in diabetic
(1955) to occur more often in diabetic neuropathy patients patients who undergo transplant surgery. Matricali, Bam-
than in those with other neurologic disorders. The authors mens, Kuypers, Flour, and Mathieu (2007) retrospectively
accounted that out of 33 patients presenting as new cases of reviewed the medical records of 66 patients with diabetes
Charcot arthropathy, 17 had diabetes while only 4 had who underwent simultaneous pancreas–kidney (SPK) trans-
syphilis. Sinha, Munichoodappa, and Kozak (1972) plants. They found that 8 of 66 patients (12%) developed a
reported an incidence of 1/680 in their series of 68,000 Charcot foot, and they attributed this to long-standing
consecutive admissions of diabetic patients over a 21-year diabetes, high glycosylated hemoglobin (HbA1c) values,
period. The authors suggested that the true incidence was and nephropathy. Pretransplant HbA1c values were sig-
much higher due to several patients in the series who were nificantly higher for the SPK group that developed Charcot
discovered after radiographs had been taken due to arthropathy; however, the study did not mention trauma
unrelated reasons; however, the authors did not specify history, neuropathy, or the role of immunosuppressive
what those unrelated reasons were. Fabrin, Larsen, and therapies in these patients.
Holstein (2000) reported epidemiological data of sponta- Despite the fact that it has been over 300 years since the
neous-onset Charcot arthropathy in a consecutive series of first reported case, Charcot arthropathy of the foot and ankle
diabetic patients. In this long-term observational study from remains a difficult and complex disease to identify, manage,
Denmark, the authors followed an average of 4000 patients and treat. With the increasing prevalence of diabetes
over a 10-year period and found the incidence of Charcot worldwide (Wild, Roglic, Green, Sicree, & King, 2004),
deformity to be 0.3% annually. In a smaller cohort study, health-care providers need to be aware of Charcot arthro-
Lavery, Armstrong, Wunderlich, Tredwell, and Boulton pathy and its potentially limb-threatening complications in
(2003) prospectively reported the incidence of diabetic foot the foot and ankle. The goal of this current concepts review is
complications in a consecutive series of Americans to examine the available literature and to provide the reader
diagnosed with diabetes while enrolled in a diabetes guidance in managing and treating this potentially devastat-
management program. They found a total incidence of ing complication of diabetic neuropathy.
Charcot arthropathy of 8.5/1000 patients per year. However,
they suggested that further studies were needed to explain
the significant differences between non-Hispanic White 3. Etiology and pathogenesis
Americans (11.7/1000 patients per year) and Mexican
Americans (6.4/1000 patients per year). The lower Diabetic Charcot arthropathy typically presents as a
incidence reported in Mexican Americans may be due to warm, swollen, and erythematous foot and ankle. The
failure to recognize the diagnosis, especially in the early appearance of the extremity may be indistinguishable from
stages, thereby underestimating the true rate of incidence. infection, and almost all afflicted patients have severe
The reported incidence of new Charcot arthropathy cases peripheral neuropathy. It is this lack of protective sensation
has remained relatively low among diabetic patients even that delays identification of bony stress injuries that may
ARTICLE IN PRESS
D.K. Wukich, W. Sung / Journal of Diabetes and Its Complications xx (2008) xxx–xxx 3

overload the insensate limb, leading to an active Charcot their somatosensory neuropathy was proportional to that of
process (Chantelau, 2005; Schon & Marks, 1995; Sella & autonomic neuropathy. However, they did not elaborate as to
Barrette, 1999). how the autonomic neuropathy led to increased osteopenia.
Charcot (1868) and Charcot and Féré (1883) believed that Nevertheless, Young et al. concluded that their findings
the disease occurred due to degeneration in the central supported the assertions of Edmonds et al. (1985). The
nervous system, resulting in a neurogenic deficit in bone reduced BMD in Charcot arthropathy patients results in
nutrition. This “neurotrophic theory” stimulated research and reduced bone strength, predisposing patients to the devel-
interest even though they could not provide any evidence, opment of fractures.
outside of observational studies, to support this idea Herbst, Jones, and Saltzman (2004) separated the pattern
(Chantelau & Onvlee, 2006). After many years and several of Charcot arthropathy in the foot and ankle into three
transformations of Charcot's theory, the amount of evidence- groups: fracture, dislocation, and combined fracture–dis-
based studies supporting the “neurotrophic theory” has been location. Although not all Charcot arthropathy patients had
debatable (Banks & McGlamry, 1989). However, there is no reduced BMD in their lower extremity, those within
debate that his work brought the attention of the medical the fracture-pattern group had significantly lower BMD
world to this disease process. compared to the dislocation group and the combination
Although there have been others who suggested that group. They concluded that reduced BMD in the lower
autonomic denervation would lead to a hyperemic state extremity was a specific risk factor for patients developing
(Leriche & Fontaine, 1927), Johnson (1966) is credited for the fracture pattern, especially in the ankle, but those with
describing a variation of Charcot's theory known as the normal BMD were more likely to develop a dislocation or
“neurovascular” theory. From his observations of bed- combination pattern. Their findings also acknowledged why
ridden patients with spinal cord injury, he found radio- successful results vary with early surgical treatment between
graphic evidence of osteolysis. Johnson contended that a patients within the different groups (Bibbo, Lin, Beam, &
central sympathetic failure initiated a hypervascular reflex Behrens, 2001).
that led to a state of overactive bone resorption and Research into the relationship between reduced BMD and
mechanical weakening, thus instigating Charcot arthro- the pathogenesis of Charcot arthropathy has led to the
pathy (Chantelau & Onvlee, 2006; Sanders, 2004). In investigation of bone resorption mediators. Increases in
support of this theory, Edmonds, Clarke, Newton, Barret, certain proinflammatory cytokines, which are known
and Walkins (1985) found a significant increase in the mediators of bone resorption, are a contributing factor to
pattern of isotope uptake on bone scans of patients increased osteoclastic activity in ACA patients (Baumhauer,
diagnosed with Charcot arthropathy of the foot compared O'Keefe, Schon, & Pinzur, 2006). In a study of Charcot-
to those of diabetic neuropathy patients without the arthropathy-reactive bone, osteoclasts were disproportio-
disease. The authors attributed the increase in uptake to nately increased compared to osteoblasts, and the osteoclasts
sympathetic denervation. Others have found the same demonstrated immunoreactivity for interleukin (IL) 1, IL-6,
pattern of isotope uptake in patients experiencing acute and tumor necrosis factor (TNF) α (Baumhauer et al., 2006;
Charcot arthropathy (ACA), but attributed the findings to Jeffcoate, Game, & Cavanagh, 2005). Reacting from an
bony destruction from an acute inflammatory process initial insult, bone resorption due to a disproportionate
(McGill et al., 2000). increase in cytokines is believed to factor into the
Using duel-energy X-ray absorptiometry, Young, Mar- pathogenesis of Charcot arthropathy (Baumhauer et al.,
shall, Adams, Selby, and Boulton (1995) demonstrated that 2006). By measuring serologic markers, Petrova et al. (2007)
the bone mineral densities (BMD) of Charcot-arthropathy- demonstrated that the acute inflammatory Charcot foot is a
afflicted lower extremities were significantly less than those peripheral event and is dissociated from any systemic
of non-Charcot-arthropathy-afflicted diabetic patients. inflammatory response. They concluded that the local
Although there have been other BMD studies that did not inflammatory response demonstrated by the increase in
show differences between diabetic patients with Charcot skin temperature must be related to an increased expression
arthropathy and diabetic patients without Charcot arthro- of proinflammatory cytokines that did not consequently lead
pathy (Clasen, 2000), Young et al.'s findings were in line to a classical systemic acute-phase response.
with those of others who observed bone resorption in up to The role of proinflammatory cytokines and the “receptor
81% of Charcot arthropathy patients (Brower & Allman, activator of nuclear transcription factor κB ligand
1981; Cundy, Edmonds, & Watkins, 1985). All of the (RANK-L)/osteoprotegerin (OPG)” signaling pathway in
patients with Charcot arthropathy were qualitatively diag- Charcot arthropathy pathogenesis provided a modern
nosed with cardiovascular autonomic neuropathy and direction for research and theory (Jeffcoate, 2004; Jeffcoate,
demonstrated significantly reduced BMD in the afflicted 2005a; Jeffcoate et al., 2005). Jeffcoate (2004) observed that
feet. However, there was no difference in lumbar spine BMD in a wide variety of other medical conditions such as
values between patients with Charcot arthropathy and osteoporosis and cardiovascular disease, the RANK-L/OPG
patients without Charcot arthropathy. In agreement with signaling pathway regulates both bone turnover and
previous studies, Young et al. believed that the severity of vascular calcification. He hypothesized that this pathway
ARTICLE IN PRESS
4 D.K. Wukich, W. Sung / Journal of Diabetes and Its Complications xx (2008) xxx–xxx

may play the decisive role in Charcot arthropathy. Increased there are studies linking sympathetic denervation—induced
osteoclastogenesis and subsequent osteolysis result in and noninduced—to this calcification (Edmonds, Morrison,
response to RANK-L up-regulation of nuclear transcription Laws, & Watkins, 1982; Goebel & Fuessl, 1983). There is
factor κB (NF-κB). The glycoprotein OPG, a member of the also evidence that the bones of patients afflicted with syphilis
TNF receptor superfamily, inactivates RANK-L by serving who undergo ACA changes have widened haversian canals
as a decoy receptor, therefore moderating RANK-L activity as a result of the sustained increase in blood flow (Jeffcoate,
and NF-κB expression (Jeffcoate, 2004). RANK-L and OPG 2005a). However, the obvious sympathetic neuropathic
syntheses are essential features of the regulation of the fine consequence leading to bilateral changes in ACA patients
balance of bone homeostasis. A disturbance in the balance should be consistently expected (Jeffcoate, Lima, &
between RANK-L and OPG, which might include trauma, Nobrega, 2000; Young et al., 1995), but unilateral distribu-
surgery, foot ulcers, or iatrogenic factors, is believed to tion is the normal finding (Jeffcoate, 2005b). Bilateral
trigger excessive osteolysis (Frykberg et al., 2006; Jeffcoate, involvement has ranged from 5.9% to 39.3% in hetero-
2004). This cycle is fully realized in the insensate limb of a geneous population studies (Sanders & Frykberg, 2007), but
patient who continues to bear weight on the affected foot, simultaneous bilateral involvement has been reported to be
thereby continually reactivating the inflammatory process. only 0.7% (Fabrin et al., 2000). Another facet of Charcot
Mabilleau, Petrova, Edmonds, and Sabokbar (2008) arthropathy that is not well understood is whether the process
compared osteoclastic activity in active Charcot arthropathy may recur in the same or uninvolved limb (Eichenholtz,
patients to osteoclastic activity in general diabetes patients 1966). Fabrin et al. reported on 20 cases of Charcot
and healthy control patients using in vitro samples of arthropathy recurring in the ipsilateral foot and suggested
peripheral blood monocytes (PBM). From these samples of that recurrent stress injuries to the neuropathic foot,
PBM, the investigators used a previously described techni- regardless of location, may initiate breakdown. This self-
que to generate functional osteoclasts in the presence of limiting process of Charcot arthropathy should not be
macrophage colony-stimulating factors and soluble RANK- expected in someone with irreversible autonomic sympa-
L (Fujikawa, Quinn, Sabokbar, McGee, & Athanasou, 1996; thetic neuropathy (Jeffcoate, 2005b).
Sabokbar & Athanasou, 2003). They then compared the The belief that sympathetic neuropathy acts as a
sample of nine consecutive patients with recent-onset precursor for diabetic bones and joints undergoing a
Charcot arthropathy to the samples of eight diabetic patients “neurotrophic,” vulnerable, and hyperemic state remains
with no history of Charcot arthropathy and eight healthy popular among investigators. There is evidence to suggest
control subjects. After excess concentrations of OPG had that patients with type 1 diabetes mellitus and peripheral
been added to all the samples, the results indicated that the neuropathy have a greater tendency for osteopenia
RANK-L-dependent pathway played a significant role in (Petrova, Foster, & Edmonds, 2005; Rix, Andreassen, &
osteoclastic activity in active Charcot arthropathy, but it also Eskildsen, 1999). Some authors also suggest that auto-
suggested that an alternative non-RANK-L dependent path- nomic sympathetic neuropathy results in increased periph-
way may be involved in combination. eral blood flow (Edmonds et al., 1985; Watkins &
Evidence of unregulated activation of RANK-L derived Edmonds, 1983; Young et al., 1995) and that increased
from sympathetic denervation has only been circumstantial blood flow from arteriovenous shunting occurs in all
(Jeffcoate, 2004). Although some authors have implicated diabetic neuropathy patients with or without Charcot
relationships among peripheral somatosensory neuropathy, arthropathy (Boulton, Scarpello, & Ward, 1982). However,
autonomic sympathetic neuropathy, and osteoclastic activity, investigators have not been able to demonstrate the link
there have been no conclusive studies linking them between autonomic sympathetic neuropathy and increased
(American Diabetes Association, 2008; Young et al., osteoclastic activity in ACA through bone circulatory
1986). Investigators have discovered evidence linking hyperperfusion (Chantelau & Onvlee, 2006). The perspec-
sympathetic autonomic activity to increased osteoclastic tive of Bloomgarden (2005) on clinical diabetic neuropathy
activity (Togari, 2002). The findings indicate that decreased was that the effects of autonomic neuropathy on the foot
sympathetic activity leads to decreased osteoclastic activity, would lead to a state of inefficient circulation and abnormal
or, in other words, sympathetic failure leads to an sweat gland and oil gland functions. Although ulceration
unregulated increase in osteoblastic activity. In light of would likely occur due to this dysfunctional state,
these findings, Chantelau and Onvlee (2006) postulated that Bloomgarden did not attribute bone destruction in Charcot
peripheral osteopenia of the foot and ankle is not the result of arthropathy to autonomic neuropathy. In a case series of 47
decreased autonomic sympathetic activity. Charcot arthropathy patients by Pinzur, Sage, Stuck,
Historically, many have believed that the loss of Kaminsky, and Zmuda (1993), it was noted only half of
sympathetic innervation was associated with increased all patients were also diagnosed with autonomic neuro-
peripheral blood flow, leading to an ACA process (Sanders pathy; however, all 47 patients were diagnosed with
& Frykberg, 2007). Vascular calcification of smooth muscle peripheral sensory neuropathy. In fact, only peripheral
cells (Monckeberg's arteriosclerosis) is commonly found in somatosensory neuropathy has been consistently seen as a
diabetic Charcot arthropathy patients (Jeffcoate, 2005b), and predisposing factor for developing Charcot arthropathy
ARTICLE IN PRESS
D.K. Wukich, W. Sung / Journal of Diabetes and Its Complications xx (2008) xxx–xxx 5

(Armstrong, Todd, Lavery, Harkless, & Bushman, 1997; (Armstrong et al., 1997; Eloesser, 1917; Horwitz, 1948;
Sinha et al., 1972). Kimmerle & Chantelau, 2007; O'Connor, Palmoski, &
Shapiro et al. (1998) examined the patterns of peripheral Brandt, 1985; Sinha et al., 1972).
blood flow and the rhythmic contractions of small arteries The current majority of clinicians believe that a
and arterioles in three groups of patients: those with diabetic combination of these theories may contribute to the
neuropathy and ACA, those with diabetic neuropathy pathogenesis of this disease. The debate between “neuro-
without Charcot arthropathy, and healthy control subjects. vascular" and “neurotraumatic" exclusivity has become an
Despite severe neuropathic findings in Charcot arthropathy antiquated sidebar to the mystery of Charcot arthropathy.
patients, they found that the ACA group did not differ in the Whether these theories equally contribute to—or whether
rate of blood flow when compared to a control group of active Charcot arthropathy is—the sum of two or more
healthy subjects. Also, there was no significant between- unequal parts continues to be explored.
groups difference in their respective vasomotion, defined as
spontaneous rhythmic oscillations of venules and arterioles
that theoretically serve to direct the distribution of blood 4. The course of Charcot arthropathy: modified
flow (Shapiro et al., 1998). This suggested that these patients Eichenholtz stages
would have similar, if not exaggerated, responses to
repetitive trauma. They further suggested that the lack of Early literature contributions concerning Charcot arthro-
inflammatory response seen with diabetic neuropathy pathy lacked clinical findings that correlated with radio-
patients without Charcot arthropathy, when challenged graphic descriptions of the disease. Eichenholtz (1966)
with increased local temperature, may actually act as a published a landmark article on Charcot arthropathy based
preventive factor against the development of the disease. on radiographic appearance and its physiologic course.
Inflammation in response to trauma may explain the Dividing the condition throughout its process, he described
central role for the exaggerated hyperemia (Jeffcoate, three separate but linear stages: developmental, coalescent,
2005b). In addition, the reported low incidence among and reconstructive stages. Shibata, Tada, and Hashizume
diabetic patients may be explained by the high prevalence of (1990) modified the Eichenholtz system to include an
macrovascular disease, which limits the capacity of an earlier stage prior to “development.” Several authors have
inflammatory response. Although there is evidence to proposed that this early inflammatory phase following
suggest that peripheral somatosensory neuropathy and injury be called “Charcot in situ,” “pre-Stage 1,” or “Stage 0
vascular changes are involved in Charcot arthropathy Charcot” (Schon & Marks, 1995; Sella & Barrette, 1999;
(Armstrong et al., 1997), increased blood flow may be Shibata et al., 1990; Yu & Hudson, 2002). This classifica-
more likely the consequence—rather than the cause—of tion is currently being applied by the majority of foot-and-
bony destruction (Chantelau & Onvlee, 2006). ankle physicians to Charcot arthropathy patients in the
In 1886, three notable German physicians, Virchow, staging of the disease. Table 1 shows a summary of this
Rotter, and Volkmann, disputed Charcot's theory of Charcot arthropathy classification.
neurotrophic bone (Anonymous, 1886). They believed that Stage 0 Charcot (inflammatory) arthropathy begins with
a repetitive trauma sustained by a joint that was unable to an initial perceived or nonperceived insult to the neuropathic
sense pain would lead to this disease—the “neurotraumatic” foot-and-ankle complex, producing localized swelling, red-
theory. Gupta (1993) noted that the common thread of ness, and warmth (Chantelau, 2005; Shibata et al., 1990).
pathogenesis included continual injuries from minor trauma Minimal, if any, radiographic abnormalities may be
or isolated major trauma to neuropathic joints. There are detected, and Classen, Rolley, Carneiro, and Martire
many authors who believed that Charcot arthropathy may be (1976) noted that clinical symptoms may precede radio-
triggered in diabetic patients by some type of joint trauma, graphic changes by up to 1 year. Diagnosis of this stage with
and they are supported by the identification of the lack of magnetic resonance imaging (MRI) or technetium bone
protective sensation as a predisposing factor to the disease scans may prove to be critical in preventing the disease from

Table 1
Modified Eichenholtz stages a
Stage Phase Description
0 Inflammatory Localized warmth, swelling, and redness; minimal to no radiographic abnormalities; MRI may show nondisplaced pathologic fracture(s)
and increased marrow edema to the foot and/or ankle
1 Development Localized warmth, marked swelling, and redness; radiographic presence of bony debris, fragmentation of subchondral bone, periarticular
fracture, subluxation, and/or dislocation
2 Coalescence Continued but decreased warmth, swelling, and redness; radiographic presence of absorption of fine debris, new bone formation,
coalescence of fragments, fusion of joints (ankylosis), and/or sclerosis of bone ends
3 Remodeling Marked decrease or absence of warmth, swelling, and redness; physically enlarged fixed (“healing”) deformity; radiographic appearance
of remodeled and new bone formation, decreased sclerosis, and/or possible gross residual deformity
a
Modified from Eichenholtz (1966).
ARTICLE IN PRESS
6 D.K. Wukich, W. Sung / Journal of Diabetes and Its Complications xx (2008) xxx–xxx

progressing into the development phase (Chantelau & Poll, case reports of isolated involvement within the body that do
2006). Schon and Marks (1995) proposed that neuropathic not include the foot or the ankle (Bayne & Lu, 1998;
diabetic patients be identified as being at risk for Charcot Edison & Finger, 2005; Lambert & Close, 2002); however,
arthropathy even after only minor injury to limbs with loss less attention is paid to these anatomical sites due to lack of
of protective sensation. Repetitive cumulative injuries to readily available data. When multiple joints are involved in
insensate patients may progress into the destructive stage of Charcot arthropathy, these may be seen at different
Charcot arthropathy and may lead to gross foot deformity, Eichenholtz stages, and these multiple joint attacks may
ulceration, infections, and/or amputation (Armstrong et al., even course at different progressive rates of destruction
1997; Chantelau, Richter, Ghassem-Zadeh, & Poll, 2007). (Eichenholtz, 1966). However, a study by Fabrin et al.
In contrast, patients with intact protective sensation limit the (2000) showed that recurrent and/or multiple joint attacks
progressive course of repetitive injury by offloading the were not influenced by any particular location. The lack of
injured limb. Identification of this prodromal stage may data and standardization of assessment for ACA may fuel
deter further progression into the latter stages of Charcot debate as to whether these cases of “recurrence” may
arthropathy, thus preventing further deformity and compli- actually be, in fact, true recurrence.
cations (Chantelau, 2005; Frykberg et al., 2006). Frequently,
Stage 0 patients are misdiagnosed as cellulitis, gout, or deep
vein thrombosis due to its clinical appearance and normal 5. Anatomical classifications
radiographic findings, but increased clinical awareness and
education of the health-care provider can potentially limit Several authors have anatomically classified the char-
the progression of the disease. acteristics of Charcot arthropathy by observing the patterns
Stage 1 Charcot arthropathy (development) is distin- of destruction to the foot and ankle (Brodsky & Rouse, 1993;
guished by marked warmth, swelling, and redness, and Sanders & Frykberg, 2007; Sanders & Mrdjenovich, 1991;
radiographic changes are evident in the bone-and-joint Schon, Weinfeld, Horton, & Resch, 1998; Sella & Barrette,
architecture (Eichenholtz, 1966). The appearance of bony 1999). Although this disease has been associated with other
debris, fragmentation of subchondral bone, subluxation, bodily sites in the diabetic patient (Bayne & Lu, 1998;
dislocation, and periarticular fractures are common findings Lambert & Close, 2002), it almost exclusively affects the
during Stage 1. This stage may also have abnormal foot and the ankle.
laboratory values that indicate overactive osteoclastic and Sanders and Frykberg (1991) divided the foot and the
osteoblastic activities (Chantelau & Onvlee, 2006). ankle into five patterns of destruction for diabetic Charcot
Stage 2 Charcot arthropathy (coalescence) follows, with arthropathy and correlated different anatomical patterns with
decreasing warmth, swelling, and redness, and radio- the frequency of complications: Pattern I (forefoot)=15%;
graphic signs of healing fracture fragments (Eichenholtz, Pattern II (tarsometatarsal joints)=40%; Pattern III (navicu-
1966). Although sometimes difficult to distinguish, Stage 2 locuneiform, talonavicular, and calcaneocuboid joints)=30%;
Charcot arthropathy is considered separate from Stage 1 Pattern IV (ankle and/or subtalar joint)=10%; Pattern V
Charcot arthropathy, and the clearest detection of Stage 2 (calcaneus)=5% (Sanders & Frykberg, 1991). The most
is radiographic appearance of osseous debris absorption, severe structural deformity and functional instability are
coalescence of bone fragments, and/or new periosteal found at the Lisfranc joint and at the ankle/subtalar joints.
bone formation. Destruction at the calcaneus is not commonly seen with
Stage 3 Charcot arthropathy (remodeling) results as the Charcot arthropathy (Sanders & Frykberg, 2007), but it may
final consolidation of fractured and deformed bone without be associated with an isolated pathologic fracture or avulsion
the warmth, swelling, and redness seen previously (Eichen- injury of the posterior tuberosity (Brodsky & Rouse, 1993). A
holtz, 1966). This stage is characterized by osseous summary of this anatomical classification is described
deformity that can be either stable or unstable. Considered in Table 2.
the chronic stage of Charcot arthropathy, osteoclastic and Brodsky (2007) described an anatomical classification
osteoblastic activities are no longer increased, as confirmed based on the four areas most commonly affected by Charcot
by laboratory test (Chantelau & Onvlee, 2006). Radio- arthropathy. Representing about 60% of the anatomical sites
graphic appearance of a mature fracture callus, bony that develop Charcot arthropathy, Type I (“midfoot”) is
remodeling of major fragments, and decreased sclerosis considered the most common site and includes the
signify the finality of the permanent deformity. metatarsocuneiform and naviculocuneiform joints. These
Although Eichenholtz described the Charcot course in are also noted to be associated with symptomatic bony
linear stages, the time of progression through each stage prominences. The second most common site, Type 2
can vary from weeks to months to years. However, (“hindfoot”), includes the subtalar, talonavicular, or calca-
neuropathy remains unchanged throughout the Charcot neocuboid joints, where bony prominences are less common.
process, whether active or quiescent (Armstrong et al., It accounts for 30–35% of anatomical incidence. Type 3 is
1997; Sinha et al., 1972). ACA initially begins in one joint divided into “A” and “B,” or the ankle and the posterior
and may progress to other proximal joints. There have been calcaneus, respectively. Comprising 9% of Charcot
ARTICLE IN PRESS
D.K. Wukich, W. Sung / Journal of Diabetes and Its Complications xx (2008) xxx–xxx 7

Table 2
Charcot arthropathy anatomical classification a
Pattern Location Description
I Forefoot Involving the interphalangeal joints, phalanges, metatarsophalangeal joints, and/or distal metatarsal bones;
commonly occurring pattern, also seen with plantar ulceration; seen as osteopenia, osteolysis, juxtaarticular cortical
bone defects, subluxation, and destruction on radiographs
II Tarsometatarsal joints Involving the tarsometatarsal joints and metatarsal bases, cuneiforms, and cuboid; commonly occurring pattern,
with greater frequency in diabetic patients than in patients with leprosy; may be associated with plantar ulceration at
the apex of deformity; seen as subluxation or fracture–dislocation, collapse of midfoot, and resultant rocker-bottom
foot deformity (consistent with initial features of osteoarthritis) on radiographs; may have dorsal prominence at
metatarsal bases; late changes include fragmentation
III Naviculocuneiform, talonavicular, Involving usually the naviculocuneiform joint and navicular bone but also the other midtarsal joints and bones;
and calcaneocuboid joints ulceration may occur at the apex of deformity and may be in combination with Pattern II; on radiographs, seen as
osteolysis of naviculocuneiform joints with fragmentation; with osseous debris both dorsally and plantarly
IV Ankle and subtalar joints Involving the ankle joint with or without the subtalar joint and medial or lateral malleolar fracture; considered a
severe structural deformity with instability—may even be associated with minor ankle sprain; on radiographs, seen
as malleolar fractures, erosion of bone and cartilage with collapse of joint, free bodies in ankle, extensive
destruction, and lateral dislocation of ankle
V Calcaneus Rarely involving only the calcaneus bone and usually involving an avulsion fracture of the posterior tubercle;
although no joint is involved, the pattern develops in patients with Charcot arthropathy; on radiographs, seen as
osteolytic changes in the posterior calcaneus attachment of the Achilles tendon; avulsion fracture of the posterior
tubercle may ensue; osteolytic changes may also occur at the naviculocuneiform joint due to additional stress
during liftoff in the gait cycle (this may be due to lack of an Achilles tendon attachment to the calcaneus)
a
Adapted from Sanders and Frykberg (2007).

arthropathy presentations, Type 3A involves the tibiotalar laterally at the fourth and fifth metatarsocuneiform joints.
joint and associated bones, and Type 3B, representing less The “perinavicular pattern” includes the navicular and its
than 2% of presentations (Sanders & Frykberg, 2007), surrounding bones, and patients with this pattern may have a
involves a pathologic fracture of the tuberosity of the tendency toward an adducted forefoot. The “transverse tarsal
calcaneus. Brodsky and Rouse (1993) also noted that the pattern” involves significant deformity in the talonavicular
breakdown of soft tissues from bony prominences occurred joint, including the medial and central columns. Unlike the
most commonly in Type I patterns, and these ulcers were other patterns, the “transverse tarsal pattern” has much larger
located on the plantar surface of the foot. Trepman, Nihal, talocalcaneal and talonavicular coverage angles. By examin-
and Pinzur (2005) have noted the obvious lack of inclusion ing the severity of the mechanical malalignment, the health-
for the forefoot and multiple regions of involvement, and care provider could assess the treatment appropriate for the
have modified Brodsky's anatomical classification to include midfoot using radiographic and clinical parameters. It was
these sites. also noted that the most severe “rocker-bottom” phase in any
Because of their increased proportion of involvement in of the four patterns of Charcot midfoot could lead to
Charcot arthropathy, the midfoot and medial column of the ulceration and/or infection at the midfoot (Schon, Easley, &
foot have been further classified into separate schemes. Weinfeld, 1998).
Schon et al. (1998) evaluated 131 feet from 109 patients to The medial column of the Charcot foot was also further
describe the patterns of acquired midfoot deformities, and divided into five clinical stages using clinical findings,
although many etiologies of midfoot deformities were radiographs, and bone scans (Sella & Barrette, 1999).
included, 89 of the 109 patients were diagnosed with Similar to the modified Eichenholtz classification, the first
Charcot arthropathy. Assessed from the three standard of the five stages is called Stage 0, and the medial column is
radiographic views, the anatomical location with the most swollen, warm, and often painful; however, radiographs are
clinically significant involvement would determine the negative for any abnormal findings. Clinical symptoms
pattern type. Each of the four patterns in this classification persist into Stage 1 with radiographic findings of periarti-
was divided into three separate clinical stages ranging in the cular cysts, erosions, localized osteopenia, and sometimes
degree of midfoot collapse, regardless of etiology. diastases. Once subluxation is seen, Stage 2 is reached, with
The “Lisfranc pattern” has radiographic breakdown the second metatarsocuneiform joint usually involved. Stage
involving the first three metatarsocuneiform joints, and as 3 is identified with joint dislocation and arch collapse, while
the stages progresses in severity from A to C, the fourth and Stage 4 represents the healed stage wherein the end result is a
fifth metatarsocuboid joints become involved. The “navicu- sclerotic fusion of involved bones and joints.
locuneiform pattern” is seen when the majority of radio- Although several authors have provided classification
graphic deformities occur more proximally and medially at schemes with some clinical importance, none has been able
the naviculocuneiform joint. Less significant anatomical to predict outcome (Rajbhandari et al., 2002). These different
deformities can also be seen in the central column and classifications do add insight into the understanding of
ARTICLE IN PRESS
8 D.K. Wukich, W. Sung / Journal of Diabetes and Its Complications xx (2008) xxx–xxx

diabetic Charcot arthropathy and do not enable clinicians to Lipsky, 2005). It may be easy to clinically diagnose an
accurately describe the stage and location of the disease. infection with an obvious open wound, but this does not
exclude a concomitant Charcot process. Some have observed
cases of Charcot arthropathy that may have been triggered by
6. Clinical presentation and standard diagnostics cellulitis, osteomyelitis (OM), and even synovitis (Arm-
strong et al., 1997; Jeffcoate, 2005b). Positive systemic signs
The American Diabetes Association (2008) recently of infection include leukocytosis, elevated C-reactive protein
released its position statement for standards of medical and erythrocyte sedimentation rate (ESR) levels, and recent
care and foot care recommendations for patients. The unexplained hyperglycemia. However, unremarkable clin-
American Diabetes Association (2004) has long recognized ical tests may not necessarily exclude infection, making
the need for the screening and evaluation of the foot and distinction between the two difficult. The treatment regimen
ankle of patients with long-standing diabetes, especially is therefore dependent on the accurate diagnosis of infection.
those who have developed neuropathy. These recommenda- The Charcot arthropathy patient may present for treatment
tions rely on a keen understanding of the diabetic foot and at any stage of the disease, but the common reasons for
ankle, with all their possible complications. If the treating seeking medical consultation may be mild and moderate
physician can recognize Charcot arthropathy without delay, pain, severe edema of the foot, and inability to put their shoe
the chances of devastating morbidity and mortality decrease on due to swelling (Wilson, 1991). Chantelau (2005)
substantially (Chantelau, 2005). reported that 19 of 24 patients (80%) were misdiagnosed
The diagnosis of Charcot arthropathy is primarily reliant as sprain, OM, Sudeck's atrophy, deep vein thrombosis,
on clinical presentation, but a physician's high index of cellulitus, or rheumatoid arthritis instead of Charcot arthro-
suspicion should not be underappreciated (Armstrong & pathy, and all continued to bear weight on the involved limb
Peters, 2002; Jeffcoate, 2005b; Sanders & Frykberg, 2007). prior to referral to a specialty clinic. Although radiographs
A thorough patient history is essential to any assessment; may not reveal any bone or joint abnormalities during the
however, a neuropathic patient's history can be unintention- prodromal inflammatory stage of the Charcot process,
ally misleading. It is therefore up to the clinician to know radiographic studies, in correlation with clinical exam, are
what questions to ask and what information is important one of the tools widely used to differentiate the disease
when making an assessment. Special attention must be given (Sanders & Frykberg, 2007). When this initial stage is
to the patient especially if history of any trauma, history of suspected but not proven, the patient should be prevented
neuropathy, recent swelling, redness in the limb, or a from incurring any additional injury to the suspected limb.
combination of the like is uncovered. With immobilization and offloading, the patient is protected
Counterintuitively, the history may include pain sensa- while awaiting the results of further investigation.
tions in an insensate limb but no recollection of any sustained Gouty arthritis and rheumatoid arthritis can be separated
trauma. In a study of 55 patients with Charcot arthropathy, from Charcot arthropathy by their radiographic and clinical
more than 75% complained of pain in the foot upon features (Trepman et al., 2005). Radiographs of the foot and
presentation even though all subjects had a clinical loss of ankle that are taken in the non-weight-bearing position can
protective sensation to the 10-g Semmes–Weinstein mono- have obvious variability in image quality and may not show
filament wire (Armstrong et al., 1997). Repetitive trauma to subtle instability compared to radiographs in the weight-
the foot and ankle may be entirely absent from verbal history bearing position. It is recommended that all foot-and-ankle
even though clinical symptoms prove otherwise. The study radiographic examinations be obtained in a weight-bearing
found that only 22% of patients were able to recall some position if possible.
specific traumatic event prior to the onset of Charcot MRI exams are increasingly being used and recom-
arthropathy. The loss of protective sensation may leave the mended for diagnosing Charcot arthropathy, especially at the
patient unaware of any particular event or reoccurring minor earliest stage (Chantelau & Poll, 2006; Sanders & Frykberg,
events (Chantelau, Richter, Schmidt-Grigoriadis, & Scher- 2007). Chantelau et al. (2007) evaluated 12 consecutive
baum, 2006). diabetic patients with polyneuropathy who presented with
It is important to investigate any previous history of only mild pain and a swollen foot. All of these patients had
infections or ulcers to rule out a recurring acute or chronic plain radiographs that did not demonstrate any osseous
infection. The usual presentation of Charcot arthropathy abnormality. MRI studies in all these patients revealed bone
involves a warm, swollen, erythematous foot or ankle in an stress injury with a median involvement of 2.5 bones per
insensate patient and, because of its similarity to an acute foot, which were previously undetected using plain radio-
soft-tissue infection, heightened awareness is needed when graphs. Although bone scintigraphy and white blood cell
dealing with the diabetic neuropathy patient population scans have been traditionally advocated for the differentia-
(Armstrong et al., 1997; Pinzur et al., 1993). Most infections tion and diagnosis of Charcot arthropathy, there is clear
in the diabetic foot involve a direct source of inoculation evidence that MRI offers the highest diagnostic accuracy
through an opening in the skin, usually caused by (Tan & Teh, 2007). Differentiating between OM and ACA
neuropathic ulcers (Sanders, 1994; Singh, Armstrong, & has been noted to be difficult due to similar signal intensity
ARTICLE IN PRESS
D.K. Wukich, W. Sung / Journal of Diabetes and Its Complications xx (2008) xxx–xxx 9

changes (Marcus et al., 1996). However, there are several nonhealing wound or suspected bone infection (Sanders &
MRI features that—with clinical correlation, anatomical Frykberg, 2007).
distribution, and abnormal appearance—help distinguish
these two diagnoses (Tan & Teh, 2007). In OM, the pattern
of bone marrow edema tends to involve a single bone with 7. Nonoperative therapies and medical management
diffuse marrow involvement, but in ACA, the pattern tends
to be periarticular and subchondral (Tan & Teh, 2007). The treatment of Charcot arthropathy depends on many
Distribution of OM has a focal involvement, usually the factors, including the course or stage of Charcot arthropathy
weight-bearing surfaces of the toes, metatarsal heads, or (Eichenholtz, 1966), location(s) of involvement (Sanders &
calcaneus (Ledermann, Morrison, & Schweitzer, 2002), Frykberg, 2007; Sinacore, 1998), presence of ulcers (Saltz-
while ACA usually has several joints/bones involved. man et al., 2005), and ability to achieve a stable and
Differentiating based on site involvement is helpful; plantigrade foot (Harrelson, 1993). Other factors that could
however, it cannot be relied on solely. Deformity is usually affect treatment options are comorbidities (American
present with ACA along with bone debris, but OM does not Diabetes Association, 2008) such as cardiovascular disease,
typically involve deformity unless there is an underlining morbid obesity, nephropathy, or infected ulcer (Armstrong
Charcot process. The clinical presence of soft-tissue disease, et al., 1997; Saltzman et al., 2005). The goals for every
ulcerations, soft-tissue abscess, or sinus tract in the foot can patient undergoing treatment for an acute or quiescent
improve the overall diagnostic accuracy for OM (Morrison, Charcot process should be to maintain or achieve structural
Schweitzer, Batte, Radack, & Russel, 1998); however, MRI stability of the foot and ankle, to prevent ulceration, and to
changes without these clinical findings may sway the preserve a plantigrade foot.
diagnosis away from OM. The limitations to an MRI exam The initial treatment for Stage 0 or Stage 1 Charcot
include the presence of internal fixation devices, associated arthropathy is typically offloading in a total contact cast
cost, and lack of available MRI equipment. (TCC) (Sanders & Frykberg, 2007). Persistent warmth and
Bone scintigraphy is highly sensitive, but lacks specificity increased temperature at the site involved indicate that the
in the diagnosis of Charcot arthropathy (Schauwecker, Park, Charcot process has yet to progress to Stage 2. One specialty
Burt, Mock, & Wellman, 1988). It is mainly used to rule out center's experience found that 60% of patients with midfoot
OM in diabetic patients with open wounds that may or may Charcot arthropathy had minimal deformity and were treated
not have bone destruction on radiographs. The determination successfully without surgery (Pinzur, 2004). This finding
to use bone scans should be based on “clinical suspicion.” emphasized that if ACA is treated judiciously, achievement
However, when bone destruction is evident on radiographs of a stable midfoot without incurring surgery or skin
without an open wound, then there is less need to undergo a breakdown is possible. It may also be prudent to monitor
three-phase 99technetium scan. However, when MRI is not temperature changes at the site of involvement to detect any
available for neuropathic diabetic patients with open wounds areas that may be experiencing repetitive stress (Lavery et
and if an MRI is not available, the use of a bone scan to rule al., 2004) perpetuating the Charcot process.
out OM may be warranted. Some authors recommend using Although alternatives devices for immobilization and
leukocyte-labeled bone scans (111indium or 99mtechnetium offloading have been studied, the TCC is considered by
hexamethylpropyleneamine oxime) to help exclude OM many clinicians as the treatment of choice (Armstrong et al.,
(Rogers & Bevilacqua, 2008a). 1997; Frykberg et al., 2006; Myerson, Papa, Eaton, &
Histological specimens can be obtained through wound or Wilson, 1992; Pinzur, 2007; Pinzur et al., 1993). Some
surgical intervention, but this is primarily used for research clinicians have recommended an 8- to 12-week non-weight-
methods only. In diagnosing Charcot arthropathy, Horwitz bearing immobilization in a TCC, while others have allowed
(1948) found multiple shards of bone and soft tissue weight bearing as tolerated from the start of treatment
embedded in the deep layers of the synovium. One review (Sanders & Frykberg, 2007). Although ambulation after an
described findings of abnormal values of bone-specific initial period of non-weight-bearing is often recommended,
alkaline phosphatase, type 1 collagen carboxyterminal weight bearing during the acute stage may not adversely
telopeptide, and urinary desoxypiridinoline cross-links, affect their Charcot limb. A preliminary study of 10 patients
indicating increased osteoclastic/osteoblastic activities examined whether allowing patients with ACA localized to
throughout Eichenholtz Stages 1 and 2 (Chantelau & Onvlee, the midfoot to bear weight was detrimental to their outcome
2006). Baumhauer et al. (2006) found that Charcot- (Pinzur, Lio, & Posner, 2006). The patient population was
arthropathy-reactive bone had osteoclasts disproportionately considered “overweight,” and their large stature did not
increased compared to osteoblasts, and the osteoclasts permit these patients from maintaining a non-weight-bearing
demonstrated immunoreactivity for IL-1, IL-6, and TNFα. status. All 10 patients were permitted to bear weight as
Although these laboratory findings may assist a clinician's tolerated in a TCC, utilizing assistive devices as needed. The
assessment, it is impractical to obtain histological samples authors concluded that the use of “conscientious total contact
from patients presenting with the initial symptoms. It is therapy” and biweekly cast changes allowed all patients to
recommended that biopsies be collected only when there is a successfully progress into custom shoes. Whether or not
ARTICLE IN PRESS
10 D.K. Wukich, W. Sung / Journal of Diabetes and Its Complications xx (2008) xxx–xxx

bearing weight, cast changes every week to every 2 weeks is possible complications and patient compliance issues before
needed to accommodate for decreased edema. proceeding with Charcot arthropathy management.
Factoring in the increased load stress on the contralateral Saltzman et al. (2005) found that “intensive, nonopera-
limb, Clohisy and Thompson (1988) suggested that the non- tive” treatment for Charcot arthropathy patients varied based
weight-bearing TCC may actually have unfavorable con- on whether the patient presented initially with or without
sequences on the weight-bearing non-Charcot limb. Others ulceration. The standardized regimen for treatment included
have also noted that the increased pressure attributed to a immobilization of the foot and ankle and administration of
three-point gait may induce unnatural stress patterns, antibiotics, if there was a presenting infected ulcer. After
resulting in ulcerations (Lesko & Maurer, 1989). Using a resolution of the infection, patients were placed in a non-
patellar tendon-bearing cast or orthosis can help negate the weight-bearing TCC. After erythema and warmth had
increased load on the contralateral limb (Saltzman, Johnson, resolved, the patients were braced in a weight-bearing foot-
Goldstein, & Donnelly, 1992). and-ankle orthosis. Their treatment regimen identified a 3%
Sinacore (1998) found that the different anatomical annual amputation risk due to infection and a 49% annual
locations affected by Charcot arthropathy would also affect amputation risk for recurrent ulceration. This retrospective
healing times in TCC. Although patient compliance and study suggested that not all patients at Stage 0 or Stage 1
delay from diagnosis to treatment were essential factors Charcot arthropathy would benefit from initial TCC
leading to shorter healing times, Sinacore found that Charcot treatment, especially those with a history of prior ulcerations.
arthropathy of the hindfoot (mean, 97±16 days), midfoot Bisphosphonates are popular as antiresorptive drugs
(mean, 96±11 days), and ankle (mean 83±22 days) took against osteoporosis, Paget's disease, and other diseases
significantly longer to heal in TCC than Charcot arthropathy with increased bone turnover (Rogers, 2003), and there have
of the forefoot (mean, 55±17 days). The total time of non- been reports on the possibility of pharmacologic therapy for
weight-bearing TCC and the immobilization period in a ACA (Anderson, Woelffer, Holtzman, & Jacobs, 2004;
weight-bearing TCC or Charcot restraint orthotic walker Selby, Young, & Boulton, 1994). One study infused
(CROW) device may last up to 4–6 months (Frykberg et al., pamidronate intravenously in ACA patients while comparing
2006). Once there is bony consolidation, custom inserts or its effects to saline (Jude et al., 2001). The authors performed
extra-deep shoes can be worn, followed by proper physician a randomized double-blinded placebo-controlled study in 39
visits to ensure no uneven distribution of plantar pressures. patients and found significant reductions in bone turnover
Although the TCC is an effective treatment for the markers, temperature, and pain symptoms. However,
ACA process, there have been complications associated significant findings in time to ambulation and time to
with this treatment, usually related to weight-bearing radiographic consolidation were not reported. Pitocco et al.
allowance (Guyton, 2005; Wukich & Motko, 2004). In a (2005) evaluated the oral efficacy of bisphosphonate
study of 70 neuropathic patients with 389 TCC changes, compounds for the treatment of ACA patients during a
Guyton found a complication rate of 6% per cast, and the 6-month randomized controlled trial. Their results showed a
greatest contributing factor for complications was the significant reduction in serum collagen COOH-terminal
diagnosis of Charcot arthropathy. The study concluded telopeptide of type 1 collagen and hydroxyprolin (known
that TCC was a safe and reliable technique for offloading bone resorption markers) and noted clinical improvements in
and immobilizing the neuropathic foot because of the the Charcot foot at 6 months. Although some consider these
predictable low rates of minor and reversible complica- studies as strong evidence supporting the use of bispho-
tions. Wukich and Motko concluded that minor complica- sphonates in the treatment of early-stage Charcot arthropathy
tions such as irritation and cast tightness should be (Pinzur, 2007; Tan, Greenstein, Jarrett, & McGonagle,
anticipated, but the avoidance of major complications 2005), these drugs have not been approved by the US
relied mostly on well-trained casting technique, appro- Food and Drug Administration for use in Charcot arthro-
priate follow-up, and proper patient education. pathy patients. The US Food and Drug Administration
After TCC, some patients may benefit from CROW (2008) has issued an alert regarding bisphosphonate use and
device (Frykberg et al., 2006). The CROW device offers the the “possibility of severe and sometimes incapacitating bone,
capacity to offload the forefoot and may replace the initial joint, and/or muscle (musculoskeletal) pain in patients.”
TCC because of its similar offloading capability (Hartsell, Black et al. (2007) studied the use of bisphosphonates for
Fellner, & Saltzman, 2001). However, the design of the postmenopausal osteoporosis and reported that “serious
removable walker device also relies heavily on patient atrial fibrillation occurred more frequently in the zoledronic
compliance. Armstrong, Short, Nixon, and Boulton (2002) acid [bisphosphonate] group” than in the placebo-controlled
described the “instant TCC” technique that transformed a group. To be clear, atrial fibrillation has not been reported
removable cast walker to one that is less easily removed. with the use of pamidronate or other bisphosphonates
Although it addresses the disadvantages of removable approved by the US Food and Drug Administration, but
walkers in Charcot arthropathy management, the additional only with zoledronic acid. However, with these data, the use
plaster around the walker creates a heavier, bulkier, and less of bisphosphonates in ACA patients must be carefully
practical device. Patients and physicians must be aware of all evaluated and all consequences must be examined fully.
ARTICLE IN PRESS
D.K. Wukich, W. Sung / Journal of Diabetes and Its Complications xx (2008) xxx–xxx 11

A randomized controlled study compared bone turnover individual clinical expertise with the best available external
and temperature between a study group that received salmon evidence (Sackett, Rosenberg, Gray, Haynes, & Richardson,
calcitonin nasal spray daily with calcium supplementation 1996). Individual clinical expertise relies on the proficiency
and a control group that received only calcium supplementa- and judgment acquired through clinical experience and
tion (Bem, Jirkovska, Fejfarova, Skibova, & Jude, 2006). practice with Charcot arthropathy patients. The best
Although both groups received offloading treatments by a available external evidence relies on relevant basic science
removable contact cast or a cast walker, the study group research and patient-centered clinical research into the
showed significant reduction in bone turnover compared accuracy, precision, and safety of therapeutic treatments.
with the control group during the 3-month follow-up. The However, there is a lack of high-level evidence supporting
advantage of calcitonin may be its direct impact on the the basis of surgical therapies in the treatment of Charcot
RANK-L/OPG system, with fewer complications compared arthropathy patients. Levels of evidence for medical
to bisphosphonate use (Bem et al., 2006). therapeutic studies—from Level 5 (expert opinion) to
Other adjunct therapies have also been offered to help Level 1A (systematic review of homogenous randomized
manage ACA. Electric bone growth (EBG) stimulators controlled trials)—were established to better interpret the
(Grady et al., 2000; Petrisor & Lau, 2005) have been quality of studies, with grades of recommendations (from A
experimentally applied and clinically tested to promote to D) based on the consistency and quality of these studies
healing of fractures in the acute phase. In theory, the EBG (Phillips et al., 2001; Wright, Einhorn, & Heckman, 2005).
stimulator generates an increased number of hydroxyl ions, The difficulty of structuring a blinded prospective rando-
increasing the pH and thus decreasing tissue oxygen mized study in a surgical specialty limits the availability of
pressure (Lavine & Grodzinsky, 1987). This scenario “high-level” evidence, and the relatively low prevalence of
correlates with what has been found in animal studies of Charcot arthropathy restricts the number of patients per
bone repair (Brighton & Heppenstall, 1971). In a small case study. Currently, surgeons are carefully constructing an
series of 10 patients, Grady et al. found that when their EBG integrated strategy for operative treatment in Charcot
stimulator was used during the initial period of immobiliza- arthropathy based on the best available evidence and
tion, there was a clinical decrease in ACA symptoms. The individual clinical experience.
authors also reported radiographic consolidation in all From a literature search on MEDLINE (using the key
patients in an average of 3.5 months. However, this limited terms Charcot, arthropathy, neuroarthropathy, osteoar-
case series did not examine the efficacy of a EBG stimulator thropathy, surgery, and diabetes) from 1960 to July 2008,
compared to immobilization with a TCC during ACA approximately 430 articles were cited. Of these articles, 80
(Pinzur, 2007). Although little evidence has been provided reported on operative treatments and/or discussed operative
to support the use of EBG stimulators in ACA treatment, its management. Over half of the articles (43 of 80) were
use has been supported as an adjunct therapy during the noncontrolled retrospective case series (Level 4), one article
postsurgical reconstruction period in Charcot arthropathy was a non-controlled prospective study (Level 4), and the
patients (Hockenbury, Gruttadauria, & McKinney, 2007; rest were expert opinions or case reports (Level 5). There
Petrisor & Lau, 2005; Saxena, DiDomenico, Widtfeldt, were no controlled or randomized prospective (Levels 1 and
Adams, & Kim, 2005). 2) or controlled retrospective (Level 3) studies reported on
A series of case reports has described the use of adjunct this topic. The current knowledge base for operative
low-intensity ultrasound for Charcot arthropathy treatment management of Charcot arthropathy is based on approxi-
(Strauss & Gonya, 1998). The authors reported a patient who mately 1000 patients who have been reported in the
underwent five surgical procedures at the tibiotalar joint and literature between 1960 and July 2008. However, this
calcaneus without success of healing. Provisional surgery number may be less on closer inspection because some
using a retrograde nail across the subtalar and tibiotalar joints authors may have reported data on the same patients in
was elected, and low-intensity ultrasound was begun post- separate articles. The surgical procedures that were men-
operatively after initial soft-tissue healing. The outcome tioned in conjunction with Charcot arthropathy operative
revealed complete arthrodesis 5 1/2 months after the management include amputations; arthrodesis; debridement
application of the ultrasound device. Although these findings of ulcers, drainage, or infections; and exostectomies.
were promising, there have been no subsequent studies to The optimal surgical management has not reached a
validate this method. consensus due to the lack of robust data supporting any
particular approach. However, this has not deterred surgeons
from addressing concerns of deformity, ulceration, and joint
8. Integrated strategy for operative treatment stability due to the effects of Charcot arthropathy on the foot
and management and ankle. Most clinicians agree that the indications for
surgical intervention in Charcot arthropathy include, but are
As the discipline of evidence-based medicine continues not limited to, recurring ulcer(s), joint instability, pain
to improve patient care and clinical practice, operative associated with malalignment, offending exotosis, and
treatment for Charcot arthropathy must also integrate potential skin complications from inability to brace or from
ARTICLE IN PRESS
12 D.K. Wukich, W. Sung / Journal of Diabetes and Its Complications xx (2008) xxx–xxx

a nonplantigrade foot (Burns & Wukich, 2008). At specialty Marcus, & Heiple, 1995). Successful surgical intervention
referral centers, Charcot arthropathy patients who undergo for ACA has not been established in the hindfoot or ankle,
minor and/or major surgical procedures have ranged from and this may be due to the peripheral BMD deficiency
14% (Fabrin et al., 2000) to 51% (Pinzur, 1999). Amputation associated with certain Charcot arthropathy patients (Herbst
rates of the lower extremity have ranged from 3% to 9%, et al., 2004). Overall, the timing of surgical intervention
depending primarily on avoidance of ulceration (Saltzman remains controversial due to the lack of high-level evidence-
et al., 2005). One center hypothesized that amputation risk based literature on reconstructive surgery in Charcot
increased with a more proximal location of deformity and arthropathy (Burns & Wukich, 2008).
complexity of infection (Rogers & Bevilacqua, 2008b). The Treatment decisions and expected outcomes are usually
authors believed that the more proximal is the Charcot based on the anatomical pattern of Charcot arthropathy, stage
deformity and with increasing degree of infection, the more of the disease, and associated comorbidities. After a mutual
likely is the chance of amputation. However, they did not decision to proceed with surgical treatment has been made,
correlate this hypothesis with patient data. Saltzman et al. preoperative assessment of adequate vascularity, nutritional
found that the majority of amputations were performed in status, and BMD is prudent to achieve the optimal
patient(s) who had developed Charcot arthropathy distal to environment for wound healing and successful surgery
the midfoot (nine cases), compared to more proximal (Burns & Wukich, 2008).
amputations (six cases). This cohort study of 127 limbs The type of surgical approach depends on many factors,
reported that the location of Charcot arthropathy did not play including the amount and location of deformity, ulceration
a significant role in whether ulceration would occur or with and without infection, stability, and skill of the surgical
whether the likelihood of amputation was necessary. Instead, team. Although the senior author has vast surgical and
they found that those with chronic recurrent ulcerations were nonsurgical experience with Charcot arthropathy patients,
significantly more likely to require amputations (36%) than the complexity and diverse presentations of the foot and
those without chronic recurrent ulcerations (6%). ankle that this center encounters may not mirror those of
Specialty referral centers have the most clinical and other specialty referral centers. Fig. 1 summarizes an
surgical experience with Charcot arthropathy patients. These integrated treatment strategy based on the available litera-
centers may be more “aggressive” in their surgical treatment ture, coupled with our experience at the comprehensive foot-
of Charcot arthropathy. Using benchmark analysis to and-ankle center.
measure morbidity and resource consumption, Pinzur Certain Charcot arthropathy patients requiring surgery
(1999) justified the use of aggressive therapies to reduce may have bony prominences without other major deformities
the costs and complications associated with this high-risk that require only simple exostectomy (Brodsky, 2007). The
patient population. This may include a more intensive exostectomy technique has been used successfully in many
offloading regimen or even “early” surgical intervention in studies of surgical treatment in Charcot midfoot deformity
order to reduce the morbidity associated with this disease and ulceration (Brodsky & Rouse, 1993; Catanzariti,
(Saltzman et al., 2005; Simon, Tejwani, Wilson, Santner, & Mendicino, & Haverstock, 2000; Laurinaviciene, Kirke-
Denniston, 2000). terp-Moeller, & Holstein, 2008; Myerson et al., 1994; Pinzur
The decision to proceed with surgery should never be et al., 1993; Rosenblum, Giurini, Miller, Chrzan, &
taken lightly even if the patient has one or more indications Habershaw, 1997). One recent case series of 20 patients
for surgical intervention. Surgical reconstruction of the reported that exostectomy procedures were effective and safe
deformed foot and ankle has historically been recommended for treating Charcot midfoot deformities and ulcerations
after all nonoperative measures to prevent further breakdown (Laurinaviciene et al., 2008). Their results were comparable
had been exhausted (Pinzur et al., 1993). This is due to the to those of previous studies in terms of exostectomy success.
perceived greater risk of potential malunion and complica- Although there are no controlled studies utilizing this
tions of surgery in patients with Charcot arthropathy. During technique, there is a fair amount of evidence to support the
the acute inflammation and destruction stage, optimal use of exostectomy for Charcot midfoot deformity and/or
fixation can be difficult to achieve, thus discouraging ulceration treatment.
surgeons from operating during Eichenholtz Stage 1. If exostectomies fail to prevent recurring ulcerations and
Simon et al. (2000) reported their results of surgical if the deformity is unstable, realignment arthrodesis may be
management of acute midfoot Charcot arthropathy (Eichen- used to stabilize the foot and ankle (Zgonis, Roukis, &
holtz Stage 1 or early Stage 2). All 14 patients were treated Lamm, 2007). The forefoot and midfoot prominences may
with adequate anatomical reduction and primary arthrodesis, develop from hindfoot influences such as subtalar or ankle
and all achieved a successful clinical outcome. There are joint valgus instability. The literature regarding arthrodesis in
other case series involving arthrodesis for limb salvage that Charcot arthropathy provides fair evidence to support its use.
have resulted in relatively successful results; however, these As the potential for major complications is higher for
studies involved relatively few patients and arthrodesis of Charcot deformity in the hindfoot and ankle because of
different anatomical sites of the foot and ankle (Bono, Roger, greater instability (Brink, Eickmeier, Levitsky, & Solomon,
& Jacobs, 1993; Papa, Myerson, & Girard, 1993; Tisdel, 1994; Papa et al., 1993), there is a trend toward the use of
ARTICLE IN PRESS
D.K. Wukich, W. Sung / Journal of Diabetes and Its Complications xx (2008) xxx–xxx 13

Fig. 1. Charcot arthropathy treatment algorithm.

supplemental fixation to increase stability at the surgical site. months after the procedure (Mueller, Sinacore, Hastings,
However, there is insufficient medical evidence to support Strube, & Johnson, 2003). This is supported by studies on
one form of fixation (i.e., internal vs. external) over another. the beneficial biomechanical effect of Achilles tendon
Because the role of ankle equinus may determine the lengthening that have been published in various medical
severity of Charcot deformity, most surgical reconstructions journals; however, it appears that the same authors may be
are performed with concomitant lengthening of the Achilles reporting the same results and the same patient data in these
tendon or gastrocnemius muscle (Burns & Wukich, 2008). studies (Hastings et al., 2000; Maluf et al., 2004; Mueller
Achilles tendon contracture has been examined as a factor et al., 2004; Salsich et al., 2005).
leading to midfoot collapse in Charcot arthropathy due to Catanzariti et al. (2000) used adjunctive Achilles tendon
lack of adequate dorsiflexion of the foot (Armstrong & lengthening with ostectomies in 9 of 10 Charcot arthropathy
Lavery, 1998; Mueller et al., 2004). In order to decrease patients with lateral midfoot ulcers, but only 38% healed
pressure on the midfoot and forefoot, lengthening procedures primarily. They also found a 92% primary healing rate with
of the Achilles tendon have been performed to decrease its medial midfoot ulcers. Mueller et al. (2003) randomized and
power and to provide more available dorsiflexion (Maluf, controlled an outcome comparison study of diabetic patients
Mueller, Strube, Engsberg, & Johnson, 2004). However, this with neuropathic ulcerations using TCC with and without
decrease in power and forefoot peak pressure has not been Achilles tendon lengthening. Their study, which also
shown to be permanent and may return to baseline several included patients with Charcot arthropathy midfoot
ARTICLE IN PRESS
14 D.K. Wukich, W. Sung / Journal of Diabetes and Its Complications xx (2008) xxx–xxx

deformity, found that those with Achilles tendon lengthening 5. Reducing the incidence of amputations and mortality
and TCC were 75% less likely to have ulcer recurrence at associated with this disease process.
7 months and 52% less likely to have ulcer recurrence at 6. Obtaining scientific evidence to identify those patients
2 years compared to the TCC-only group. Their recommen- who are most at risk, to facilitate prompt and early
dations were to consider using Achilles tendon lengthening diagnosis, and to initiate routine care.
as an adjunctive procedure for limited dorsiflexion of less
than 5° and plantar ulcerations at the forefoot of diabetic The current evidence for surgical intervention for Charcot
neuropathy patients. This adjunctive procedure, however, arthropathy includes the following:
must be done with caution and not indiscriminately, as
Achilles tendon lengthening has been shown to increase the • There is fair evidence for surgical excision of bones
risk for heel ulcerations (Holstein, Lohmann, Bitsch, & (exostectomy) to reduce pressure from bony promi-
Jorgensen, 2004). Mueller et al. also found in their study that nences. There are Level 4 studies with a consistency of
peak pressure on the heel increased by 34% and skin successful results (Grade C recommendation).
breakdown was noted in 4 of 31 patients in the Achilles • There is fair evidence for the arthrodesis of unstable
tendon lengthening group. Even in light of possible Charcot deformity of the foot and/or ankle as indicated
complications, there appears to be good evidence to support in patients who fail nonoperative treatment. There are
the use of adjunctive Achilles tendon lengthening for certain Level 4 studies with a consistency of successful results
Charcot arthropathy patients with neuropathic ulcers. (Grade C recommendation).
Postoperatively, patients should be immobilized until • There is not enough evidence available to determine
edema and erythema have resolved. Immobilization may be any advantage of internal fixation over external
necessary for 5–6 months, at which time patients can be fixation, and vice versa, in the surgical reconstruction
placed into braces or therapeutic shoes with custom inserts of Charcot deformities (insufficient data available to
(Trepman et al., 2005). Extended duration of bracing may be grade recommendation).
necessary especially in hindfoot and ankle reconstruction. It • There is good evidence to support the use of tendo-
is evident from long-term follow-up of reconstructive Achilles or gastrocnemius muscle lengthening to
Charcot surgical techniques that complications such as decrease forefoot pressure and to improve ankle
nonunions, malunions, or amputations can occur despite equinus. There is one Level I study and several Level
excellent surgical technique (Fabrin et al., 2000; Pinzur, III and IV studies with a consistency of successful
1999; Saltzman et al., 2005). Other postoperative complica- results (Grade B recommendation).
tions may include development of Charcot arthropathy in
adjacent joints, pseudoarthrosis, postoperative infection,
recurrence of ACA, and transfer lesions. References

American Diabetes Association. (2004). Preventive foot care in diabetes.


9. Conclusion Diabetes Care, 27(Suppl 1), S63−S64.
American Diabetes Association. (2008). Standards of medical care in
In the era of evidence-based medicine, Charcot arthro- diabetes—2008. Diabetes Care, 31(Suppl 1), S12−S54.
pathy of the foot and ankle remains a poorly understood Anderson, J. J., Woelffer, K. E., Holtzman, J. J., & Jacobs, A. M. (2004).
disease, although recent clinical and basic science research Bisphosphonates for the treatment of Charcot neuroarthropathy. Journal
of Foot and Ankle Surgery, 43, 285−289.
has improved our level of knowledge regarding its etiology Anonymous (1886). Sitzung vom 17. November 1886. (in German). Berli-
and treatment. However, there are few high-level evidence ner Klinische Wochenschrift, 23, 5.
studies to support management and treatment options at this Armstrong, D. G., & Lavery, L. A. (1998). Elevated peak plantar pressures
point in time. in patients who have Charcot arthropathy. Journal of Bone and Joint
The current goals for clinicians treating Charcot arthro- Surgery, American Volume, 80, 365−369.
Armstrong, D. G., & Peters, E. J. (2002). Charcot's arthropathy of the foot.
pathy include the following: Journal of the American Podiatric Medical Association, 92, 390−394.
Armstrong, D. G., Short, B., Nixon, B. P., & Boulton, A. J. M. (2002).
1. Maintaining a high index of suspicion in diabetic Technique for fabrication of an “instant” total contact cast for treatment
patients with peripheral neuropathy who experience of neuropathic diabetic foot ulcers. Journal of the American Podiatric
pain or edema with or without a history of trauma. Medical Association, 92, 405−408.
Armstrong, D. G., Todd, W. F., Lavery, L. A., Harkless, L. B., & Bushman, T. R.
2. Recognizing ACA and promptly offloading the (1997). The natural history of acute Charcot's arthropathy in a diabetic foot
affected limb in patients in whom the diagnosis is specialty clinic. Journal of the American Podiatric Medical Association,
suspected but not proven. 87, 272−278.
3. Maintaining a stable plantigrade foot that remains free Banks, A. S., & McGlamry, E. D. (1989). Charcot foot. Journal of the
of ulcerations and infection. American Podiatric Medical Association, 79, 213−235.
Baumhauer, J. F., O'Keefe, R. J., Schon, L. C., & Pinzur, M. S. (2006).
4. Educating patients, primary care physicians, and Cytokine-induced osteoclastic bone resorption in Charcot arthropathy:
emergency medical doctors about this disease process An immunohistochemical study. Foot & Ankle International, 27,
and the urgency of early diagnosis. 797−800.
ARTICLE IN PRESS
D.K. Wukich, W. Sung / Journal of Diabetes and Its Complications xx (2008) xxx–xxx 15

Bayne, O., & Lu, E. J. (1998). Diabetic Charcot's arthropathy of the wrist. Charcot, J. M., & Féré, C. (1883). Affections osseuses et articulaires du pied
Case report and literature review. Clinical Orthopaedics and Related chez les tabétiques (Pied tabétique). Archives de Neurologie, 6,
Research, 122−126. 305−319.
Bem, R., Jirkovska, A., Fejfarova, V., Skibova, J., & Jude, E. B. (2006). Clasen, S. (2000). Is diabetic Charcot foot related to lower limb osteopaenia?
Intranasal calcitonin in the treatment of acute Charcot neuroosteoar- Foot and Ankle Surgery, 6, 255−259.
thropathy: A randomized controlled trial. Diabetes Care, 29, Classen, J. N., Rolley, R. T., Carneiro, R., & Martire, J. R. (1976).
1392−1394. Management of foot conditions of the diabetic patient. American
Bibbo, C., Lin, S. S., Beam, H. A., & Behrens, F. F. (2001). Complications Surgeon, 42, 81−88.
of ankle fractures in diabetic patients. Orthopedic Clinics of North Clohisy, D. R., & Thompson, R. C., Jr. (1988). Fractures associated with
America, 32, 113−133. neuropathic arthropathy in adults who have juvenile-onset diabetes.
Black, D. M., Delmas, P. D., Eastell, R., Reid, I. R., Boonen, S., Cauley, Journal of Bone and Joint Surgery, American Volume, 70, 1192−1200.
J. A., Cosman, F., Lakatos, P., Leung, P. C., Man, Z., Mautalen, C., Cundy, T. F., Edmonds, M. E., & Watkins, P. J. (1985). Osteopenia and
Mesenbrink, P., Hu, H., Caminis, J., Tong, K., Rosario-Jansen, T., metatarsal fractures in diabetic neuropathy. Diabetic Medicine, 2,
Krasnow, J., Hue, T. F., Sellmeyer, D., Eriksen, E. F., & Cummings, 461−464.
S. R. (2007). Once-yearly zoledronic acid for treatment of Edison, J., & Finger, D. R. (2005). Neuropathic osteoarthropathy of the
postmenopausal osteoporosis. New England Journal of Medicine, shoulder. Journal of Clinical Rheumatology, 11, 333−334.
356, 1809−1822. Edmonds, M. E., Clarke, M. B., Newton, S., Barrett, J., & Watkins, P. J.
Bloomgarden, Z. T. (2005). Clinical diabetic neuropathy. Diabetes Care, 28, (1985). Increased uptake of bone radiopharmaceutical in diabetic
2968−2974. neuropathy. Quarterly Journal of Medicine, 57, 843−855.
Bono, J. V., Roger, D. J., & Jacobs, R. L. (1993). Surgical arthrodesis of the Edmonds, M. E., Morrison, N., Laws, J. W., & Watkins, P. J. (1982). Medial
neuropathic foot. A salvage procedure. Clinical Orthopaedics and arterial calcification and diabetic neuropathy. British Medical Journal
Related Research, 14−20. (Clinical Research Edition), 284, 928−930.
Boulton, A. J., Scarpello, J. H., & Ward, J. D. (1982). Venous oxygenation in Eichenholtz, S. N. (Ed.). (1966). Charcot joints (pp. 3−8). Springfield, IL:
the diabetic neuropathic foot: Evidence of arteriovenous shunting? Charles C. Thomas.
Diabetologia, 22, 6−8. Eloesser, L. (1917). On the nature of neuropathic affections of the joints.
Brighton, C. T., & Heppenstall, R. B. (1971). Oxygen tension in zones of the Annals of Surgery, 66, 201−207.
epiphyseal plate, the metaphysis and diaphysis. An in vitro and in vivo Fabrin, J., Larsen, K., & Holstein, P. E. (2000). Long-term follow-up in
study in rats and rabbits. Journal of Bone and Joint Surgery, American diabetic Charcot feet with spontaneous onset. Diabetes Care, 23,
Volume, 53, 719−728. 796−800.
Brink, D. S., Eickmeier, K. M., Levitsky, D. R., & Solomon, M. G. (1994). Frykberg, R. G., & Belczyk, R. (2008). Epidemiology of the Charcot foot.
Subtalar and talonavicular joint dislocation as a presentation of diabetic Clinics in Podiatric Medicine and Surgery, 25, 17−28.
neuropathic arthropathy with salvage by triple arthrodesis. Journal of Frykberg, R. G., Zgonis, T., Armstrong, D. G., Driver, V. R., Giurini, J. M.,
Foot and Ankle Surgery, 33, 583−589. Kravitz, S. R., Landsman, A. S., Lavery, L. A., Moore, J. C., Schuberth,
Brodsky, J. W. (2007). The diabetic foot. In M. J. Coughlin, R. A. Mann, & J. M., Wukich, D. K., Andersen, C., & Vanore, J. (2006). Diabetic foot
C. L. Saltzman (Eds.), Surgery of the foot and ankle (pp. 1281−1368). disorders: A clinical practice guideline (2006 revision). Journal of Foot
Philadelphia: Mosby Elsevier. and Ankle Surgery, 45, S1−S66.
Brodsky, J. W., & Rouse, A. M. (1993). Exostectomy for symptomatic bony Fujikawa, Y., Quinn, J. M., Sabokbar, A., McGee, J. O., & Athanasou, N. A.
prominences in diabetic Charcot feet. Clinical Orthopaedics and (1996). The human osteoclast precursor circulates in the monocyte
Related Research, 21−26. fraction. Endocrinology, 137, 4058−4060.
Brower, A. C., & Allman, R. M. (1981). Pathogenesis of the neurotrophic Gazis, A., Pound, N., Macfarlane, R., Treece, K., Game, F., & Jeffcoate, W.
joint: Neurotraumatic vs. neurovascular. Radiology, 139, 349−354. (2004). Mortality in patients with diabetic neuropathic osteoarthropathy
Burns, P. R., & Wukich, D. K. (2008). Surgical reconstruction of the Charcot (Charcot foot). Diabetic Medicine, 21, 1243−1246.
rearfoot and ankle. Clinics in Podiatric Medicine and Surgery, 25, Goebel, F. D., & Fuessl, H. S. (1983). Monckeberg's sclerosis after
95−120. sympathetic denervation in diabetic and non-diabetic subjects. Diabe-
Catanzariti, A. R., Mendicino, R., & Haverstock, B. (2000). Ostectomy for tologia, 24, 347−350.
diabetic neuroarthropathy involving the midfoot. Journal of Foot and Grady, J. F., O'Connor, K. J., Axe, T. M., Zager, E. J., Dennis, L. M., &
Ankle Surgery, 39, 291−300. Brenner, L. A. (2000). Use of electrostimulation in the treatment of
Chantelau, E. (2005). The perils of procrastination: Effects of early vs. diabetic neuroarthropathy. Journal of the American Podiatric Medical
delayed detection and treatment of incipient Charcot fracture. Diabetic Association, 90, 287−294.
Medicine, 22, 1707−1712. Gupta, R. (1993). A short history of neuropathic arthropathy. Clinical
Chantelau, E., & Onvlee, G. J. (2006). Charcot foot in diabetes: Farewell to Orthopaedics and Related Research, 43−49.
the neurotrophic theory. Hormone and Metabolic Research, 38, Guyton, G. P. (2005). An analysis of iatrogenic complications from the total
361−367. contact cast. Foot & Ankle International, 26, 903−907.
Chantelau, E., & Poll, L. W. (2006). Evaluation of the diabetic Charcot foot Harrelson, J. M. (1993). The diabetic foot: Charcot arthropathy. Instruc-
by MR imaging or plain radiography—An observational study. Ex- tional Course Lectures, 42, 141−146.
perimental and Clinical Endocrinology & Diabetes, 114, 428−431. Hartsell, H. D., Fellner, C., & Saltzman, C. L. (2001). Pneumatic bracing and
Chantelau, E., Richter, A., Ghassem-Zadeh, N., & Poll, L. W. (2007). total contact casting have equivocal effects on plantar pressure relief.
“Silent” bone stress injuries in the feet of diabetic patients with Foot & Ankle International, 22, 502−506.
polyneuropathy: A report on 12 cases. Archives of Orthopaedic and Hastings, M. K., Mueller, M. J., Sinacore, D. R., Salsich, G. B., Engsberg, J.
Trauma Surgery, 127, 171−177. R., & Johnson, J. E. (2000). Effects of a tendo-Achilles lengthening
Chantelau, E., Richter, A., Schmidt-Grigoriadis, P., & Scherbaum, W. A. procedure on muscle function and gait characteristics in a patient with
(2006). The diabetic Charcot foot: MRI discloses bone stress injury as diabetes mellitus. Journal of Orthopaedic and Sports Physical Therapy,
trigger mechanism of neuroarthropathy. Experimental and Clinical 30, 85−90.
Endocrinol & Diabetes, 114, 118−123. Herbst, S. A., Jones, K. B., & Saltzman, C. L. (2004). Pattern of diabetic
Charcot, J. M. (1868). Sur quelques arthropathies qui paraissent dépendre neuropathic arthropathy associated with the peripheral bone mineral
d'une lésion du cerveau ou de la moelle épinière. Archive de Physiologie density. Journal of Bone and Joint Surgery, British Volume, 86,
Normale Pathologique, 1, 161−178. 378−383.
ARTICLE IN PRESS
16 D.K. Wukich, W. Sung / Journal of Diabetes and Its Complications xx (2008) xxx–xxx

Hockenbury, R. T., Gruttadauria, M., & McKinney, I. (2007). Use of principles of treatment. Clinical Orthopaedics and Related Research,
implantable bone growth stimulation in Charcot ankle arthrodesis. Foot 226−231.
& Ankle International, 28, 971−976. Mabilleau, G., Petrova, N. L., Edmonds, M. E., & Sabokbar, A. (2008).
Holstein, P., Lohmann, M., Bitsch, M., & Jorgensen, B. (2004). Achilles Increased osteoclastic activity in acute Charcot's osteoarthropathy: The
tendon lengthening, the panacea for plantar forefoot ulceration? Dia- role of receptor activator of nuclear factor-kappaB ligand. Diabetologia,
betes Metabolism Research and Reviews, 20(Suppl 1), S37−S40. 51, 1035−1040.
Horwitz, T. (1948). Bone and cartilage debris in the synovial membrane; its MacCormac, W., & Klockmann, J. W. (Eds.). (1881). Transactions of the
significance in the early diagnosis of neuro-arthropathy. Journal of Bone international medical congress: Seventh session held in London, August
and Joint Surgery, American Volume, 30A, 579−588. 2–9, 1881 (p. 26). London: Balantyne, Hanson, & Co.
Jeffcoate, W. (2004). Vascular calcification and osteolysis in diabetic Maluf, K. S., Mueller, M. J., Strube, M. J., Engsberg, J. R., & Johnson, J. E.
neuropathy—Is RANK-L the missing link? Diabetologia, 47, 1488−1492. (2004). Tendon Achilles lengthening for the treatment of neuropathic
Jeffcoate, W., Lima, J., & Nobrega, L. (2000). The Charcot foot. Diabetic ulcers causes a temporary reduction in forefoot pressure associated with
Medicine, 17, 253−258. changes in plantar flexor power rather than ankle motion during gait.
Jeffcoate, W. J. (2005a). Abnormalities of vasomotor regulation in the Journal of Biomechanics, 37, 897−906.
pathogenesis of the acute Charcot foot of diabetes mellitus. Interna- Marcus, C. D., Ladam-Marcus, V. J., Leone, J., Malgrange, D., Bonnet-
tional Journal of Lower Extremity Wounds, 4, 133−137. Gausserand, F. M., & Menanteau, B. P. (1996). MR imaging of
Jeffcoate, W. J. (2005b). Theories concerning the pathogenesis of the acute osteomyelitis and neuropathic osteoarthropathy in the feet of diabetics.
Charcot foot suggest future therapy. Current Diabetes Reports, 5, 430−435. Radiographics, 16, 1337−1348.
Jeffcoate, W. J., Game, F., & Cavanagh, P. R. (2005). The role of Matricali, G. A., Bammens, B., Kuypers, D., Flour, M., & Mathieu, C.
proinflammatory cytokines in the cause of neuropathic osteoarthropathy (2007). High rate of Charcot foot attacks early after simultaneous
(acute Charcot foot) in diabetes. Lancet, 366, 2058−2061. pancreas–kidney transplantation. Transplantation, 83, 245−246.
Johnson, J. T. (1967). Neuropathic fractures and joint injuries. Pathogenesis McGill, M., Molyneaux, L., Bolton, T., Ioannou, K., Uren, R., & Yue, D. K.
and rationale of prevention and treatment. Journal of Bone and Joint (2000). Response of Charcot's arthropathy to contact casting: Assess-
Surgery, American Volume, 49, 1−30. ment by quantitative techniques. Diabetologia, 43, 481−484.
Johnson, L. (1966). Comment to Williams et al. Arthritis and Rheum, 9, Miller, D. S., & Lichtman, W. F. (1955). Diabetic neuropathic arthropathy of
358−359. feet; summary report of seventeen cases. AMA Archives of Surgery, 70,
Jordan, W. R. (1936). Neuritic manifestations in diabetes mellitus. Archives 513−518.
of Internal Medicine, 57, 307−366. Mitchell, J. K. (1831). On a new practice in acute and chronic rheumatism.
Jude, E. B., Selby, P. L., Burgess, J., Lilleystone, P., Mawer, E. B., Page, S. R., American Journal of the Medical Sciences, 8, 55−64.
Donohoe, M., Foster, A. V., Edmonds, M. E., & Boulton, A. J. (2001). Mitchell, S. W., Morehouse, G. R., & Keen, W. W. (Eds.). (1864). Gunshot
Bisphosphonates in the treatment of Charcot neuroarthropathy: A wounds and other injuries of nerves (pp. 119−133). Philadelphia: J. B.
double-blind randomised controlled trial. Diabetologia, 44, 2032−2037. Lippincott & Co..
Kelly, M. (1963). De arthritide symptomatica of William Musgrave (1657– Morrison, W. B., Schweitzer, M. E., Batte, W. G., Radack, D. P., & Russel,
1721): His description of neuropathic arthritis. Bulletin of the History of K. M. (1998). Osteomyelitis of the foot: Relative importance of primary
Medicine, 37, 372−377. and secondary MR imaging signs. Radiology, 207, 625−632.
Kimmerle, R., & Chantelau, E. (2007). Weight-bearing intensity produces Mueller, M. J., Sinacore, D. R., Hastings, M. K., Lott, D. J., Strube, M. J., &
Charcot deformity in injured neuropathic feet in diabetes. Experimental Johnson, J. E. (2004). Impact of Achilles tendon lengthening on
and Clinical Endocrinol & Diabetes, 115, 360−364. functional limitations and perceived disability in people with a
Lambert, A. P., & Close, C. F. (2002). Charcot neuroarthropathy of the knee neuropathic plantar ulcer. Diabetes Care, 27, 1559−1564.
in Type 1 diabetes: Treatment with total knee arthroplasty. Diabetic Mueller, M. J., Sinacore, D. R., Hastings, M. K., Strube, M. J., & Johnson, J.
Medicine, 19, 338−341. E. (2003). Effect of Achilles tendon lengthening on neuropathic plantar
Laurinaviciene R., Kirketerp-Moeller K., Holstein P.E. (2008). Exostectomy ulcers. A randomized clinical trial. Journal of Bone and Joint Surgery,
for chronic midfoot plantar ulcer in Charcot deformity. Journal of American Volume, 85-A, 1436−1445.
Wound Care, 17, 53–55, 57–58. Myerson, M., Papa, J., Eaton, K., & Wilson, K. (1992). The total contact cast
Lavery, L. A., Armstrong, D. G., Wunderlich, R. P., Tredwell, J., & for management of neuropathic plantar ulceration of the foot. Journal of
Boulton, A. J. (2003). Diabetic foot syndrome: Evaluating the Bone and Joint Surgery, 74A, 261−269.
prevalence and incidence of foot pathology in Mexican Americans Myerson, M. S., Henderson, M. R., Saxby, T., & Short, K. W. (1994).
and non-Hispanic whites from a diabetes disease management cohort. Management of midfoot diabetic neuroarthropathy. Foot & Ankle
Diabetes Care, 26, 1435−1438. International, 15, 233−241.
Lavery, L. A., Higgins, K. R., Lanctot, D. R., Constantinides, G. P., O'Connor, B. L., Palmoski, M. J., & Brandt, K. D. (1985). Neurogenic
Zamorano, R. G., Armstrong, D. G., Athanasiou, K. A., & Agrawal, C. acceleration of degenerative joint lesions. Journal of Bone and Joint
M. (2004). Home monitoring of foot skin temperatures to prevent Surgery, American Volume, 67, 562−572.
ulceration. Diabetes Care, 27, 2642−2647. Papa, J., Myerson, M., & Girard, P. (1993). Salvage, with arthrodesis, in
Lavine, L. S., & Grodzinsky, A. J. (1987). Electrical stimulation of repair of intractable diabetic neuropathic arthropathy of the foot and ankle.
bone. Journal of Bone and Joint Surgery, American Volume, 69, 626−630. Journal of Bone and Joint Surgery, American Volume, 75, 1056−1066.
Le, C. T., & Boen, J. R. (Eds.). (1995). Health and numbers: Basic Petrisor B., & Lau J. T. (2005). Electrical bone stimulation: An overview and
biostatistics methods (pp. 5−14). New York: Wiley-Liss. its use in high risk and Charcot foot and ankle reconstructions. Foot and
Ledermann, H. P., Morrison, W. B., & Schweitzer, M. E. (2002). MR image Ankle Clinics, 10, 609–620, vii–viii.
analysis of pedal osteomyelitis: Distribution, patterns of spread, and Petrova, N. L., Foster, A. V., & Edmonds, M. E. (2005). Calcaneal bone
frequency of associated ulceration and septic arthritis. Radiology, 223, mineral density in patients with Charcot neuropathic osteoarthropathy:
747−755. Differences between Type 1 and Type 2 diabetes. Diabetic Medicine, 22,
Lee, L., Blume, P. A., & Sumpio, B. (2003). Charcot joint disease in diabetes 756−761.
mellitus. Annals of Vascular Surgery, 17, 571−580. Petrova, N. L., Moniz, C., Elias, D. A., Buxton-Thomas, M., Bates, M., &
Leriche, R., & Fontaine, R. (1927). Experimental researches upon Edmonds, M. E. (2007). Is there a systemic inflammatory response in the
vasomotricity. Annals of Surgery, 85, 641−646. acute Charcot foot? Diabetes Care, 30, 997−998.
Lesko, P., & Maurer, R. C. (1989). Talonavicular dislocations and Phillips, B., Ball, C., Sackett, D., Badenoch, D., Straus, S., Haynes, B.,
midfoot arthropathy in neuropathic diabetic feet. Natural course and & Dawes, M. (2001). Levels of evidence. Oxford Centre for
ARTICLE IN PRESS
D.K. Wukich, W. Sung / Journal of Diabetes and Its Complications xx (2008) xxx–xxx 17

Evidence-Based Medicine. Oxford: www.cebm.net/levels_of_evidence. Schauwecker, D. S., Park, H. M., Burt, R. W., Mock, B. H., & Wellman, H.
asp (Accessed August 6, 2008). N. (1988). Combined bone scintigraphy and indium-111 leukocyte scans
Pinzur, M. S. (1999). Benchmark analysis of diabetic patients with neuropathic in neuropathic foot disease. Journal of Nuclear Medicine, 29,
(Charcot) foot deformity. Foot & Ankle International, 20, 564−567. 1651−1655.
Pinzur, M. (2004). Surgical versus accommodative treatment for Charcot Schon, L. C., Easley, M. E., & Weinfeld, S. B. (1998). Charcot
arthropathy of the midfoot. Foot & Ankle International, 25, 545−549. neuroarthropathy of the foot and ankle. Clinical Orthopaedics and
Pinzur, M. S. (2007). Current concepts review: Charcot arthropathy of the Related Research, 349, 116−131.
foot and ankle. Foot & Ankle International, 28, 952−959. Schon, L. C., & Marks, R. M. (1995). The management of neuroarthropathic
Pinzur, M. S., Lio, T., & Posner, M. (2006). Treatment of Eichenholtz Stage I fracture—Dislocations in the diabetic patient. Orthopedic Clinics of
Charcot foot arthropathy with a weight-bearing total contact cast. Foot North America, 26, 375−392.
& Ankle International, 27, 324−329. Schon, L. C., Weinfeld, S. B., Horton, G. A., & Resch, S. (1998).
Pinzur, M. S., Sage, R., Stuck, R., Kaminsky, S., & Zmuda, A. (1993). A Radiographic and clinical classification of acquired midtarsus defor-
treatment algorithm for neuropathic (Charcot) midfoot deformity. Foot mities. Foot & Ankle International, 19, 394−404.
& Ankle, 14, 189−197. Selby, P. L., Young, M. J., & Boulton, A. J. (1994). Bisphosphonates: A new
Pitocco, D., Ruotolo, V., Caputo, S., Mancini, L., Collina, C. M., Manto, A., treatment for diabetic Charcot neuroarthropathy? Diabetic Medicine, 11,
Caradonna, P., & Ghirlanda, G. (2005). Six-month treatment with 28−31.
alendronate in acute Charcot neuroarthropathy: A randomized controlled Sella, E. J., & Barrette, C. (1999). Staging of Charcot neuroarthropathy
trial. Diabetes Care, 28, 1214−1215. along the medial column of the foot in the diabetic patient. Journal of
Rajbhandari, S. M., Jenkins, R. C., Davies, C., & Tesfaye, S. (2002). Charcot Foot and Ankle Surgery, 38, 34−40.
neuroarthropathy in diabetes mellitus. Diabetologia, 45, 1085−1096. Shapiro, S. A., Stansberry, K. B., Hill, M. A., Meyer, M. D., McNitt, P. M.,
Rix, M., Andreassen, H., & Eskildsen, P. (1999). Impact of peripheral Bhatt, B. A., & Vinik, A. I. (1998). Normal blood flow response and
neuropathy on bone density in patients with type 1 diabetes. Diabetes vasomotion in the diabetic Charcot foot. Journal of Diabetes and Its
Care, 22, 827−831. Complications, 12, 147−153.
Rogers L.C., Bevilacqua N.J. (2008a). Imaging of the Charcot foot. Clinics Shibata, T., Tada, K., & Hashizume, C. (1990). The results of arthrodesis of
in Podiatric Medicine and Surgery, 25, 263–274, vii. the ankle for leprotic neuroarthropathy. Journal of Bone and Joint
Rogers, L. C., & Bevilacqua, N. J. (2008b). The diagnosis of Charcot foot. Surgery, American Volume, 72, 749−756.
Clinics in Podiatric Medicine and Surgery, 25, 43−51. Simon, S. R., Tejwani, S. G., Wilson, D. L., Santner, T. J., & Denniston,
Rogers, M. J. (2003). New insights into the molecular mechanisms of action N. L. (2000). Arthrodesis as an early alternative to nonoperative
of bisphosphonates. Current Pharmaceutical Design, 9, 2643−2658. management of Charcot arthropathy of the diabetic foot. Journal of
Rosenblum, B. I., Giurini, J. M., Miller, L. B., Chrzan, J. S., & Bone and Joint Surgery, American Volume, 82-A, 939−950.
Habershaw, G. M. (1997). Neuropathic ulcerations plantar to the Sinacore, D. R. (1998). Acute Charcot arthropathy in patients with diabetes
lateral column in patients with Charcot foot deformity: A flexible mellitus: Healing times by foot location. Journal of Diabetes and its
approach to limb salvage. Journal of Foot and Ankle Surgery, 36, Complications, 12, 287−293.
360−363. Singh, N., Armstrong, D. G., & Lipsky, B. A. (2005). Preventing foot ulcers
Sabokbar, A., & Athanasou, N. S. (2003). Generating human osteoclasts in patients with diabetes. Journal of the American Medical Association,
from peripheral blood. Methods in Molecular Medicine, 80, 101−111. 293, 217−228.
Sackett, D. L., Rosenberg, W. M., Gray, J. A., Haynes, R. B., & Richardson, Sinha, S., Munichoodappa, C. S., & Kozak, G. P. (1972). Neuro-arthropathy
W. S. (1996). Evidence based medicine: What it is and what it isn't. (Charcot joints) in diabetes mellitus (clinical study of 101 cases). Me-
British Medical Journal, 312, 71−72. dicine (Baltimore), 51, 191−210.
Salsich, G. B., Mueller, M. J., Hastings, M. K., Sinacore, D. R., Strube, M. Strauss, E., & Gonya, G. (1998). Adjunct low intensity ultrasound in Charcot
J., & Johnson, J. E. (2005). Effect of Achilles tendon lengthening on neuroarthropathy. Clinical Orthopaedics and Related Research, 132−138.
ankle muscle performance in people with diabetes mellitus and a Tan, A. L., Greenstein, A., Jarrett, S. J., & McGonagle, D. (2005). Acute
neuropathic plantar ulcer. Physical Therapy, 85, 34−43. neuropathic joint disease: A medical emergency? Diabetes Care, 28,
Saltzman, C. L., Hagy, M. L., Zimmerman, B., Estin, M., & Cooper, R. 2962−2964.
(2005). How effective is intensive nonoperative initial treatment of Tan, P. L., & Teh, J. (2007). MRI of the diabetic foot: Differentiation of infection
patients with diabetes and Charcot arthropathy of the feet? Clinical from neuropathic change. British Journal of Radiology, 80, 939−948.
Orthopaedics and Related Research, 0, 185−190. Tisdel, C. L., Marcus, R. E., & Heiple, K. G. (1995). Triple arthrodesis for
Saltzman, C. L., Johnson, K. A., Goldstein, R. H., & Donnelly, R. E. (1992). diabetic peritalar neuroarthropathy. Foot & Ankle International, 16,
The patellar tendon-bearing brace as treatment for neurotrophic arthro- 332−338.
pathy: A dynamic force monitoring study. Foot & Ankle, 13, 14−21. Togari, A. (2002). Adrenergic regulation of bone metabolism: Possible
Sanders, L. J. (1994). Diabetes mellitus. Prevention of amputation. Journal involvement of sympathetic innervation of osteoblastic and osteoclastic
of the American Podiatric Medical Association, 84, 322−328. cells. Microscopy Research and Technique, 58, 77−84.
Sanders, L. J. (2004). The Charcot foot: Historical perspective 1827–2003. Trepman, E., Nihal, A., & Pinzur, M. S. (2005). Current topics review:
Diabetes Metabolism Research and Reviews, 20(Suppl 1), S4−S8. Charcot neuroarthropathy of the foot and ankle. Foot & Ankle
Sanders, L., & Frykberg, R. (2007). The Charcot foot (Pied de Charcot). In J. International, 26, 46−63.
H. Bowker, & M. A. Pfeifer (Eds.), Levin and O'Neal's the diabetic foot US Food and Drug Administration. (2008). Bisphosphonates (marketed as
(7th ed., pp. 257−283). Philadelphia: Mosby Elsevier. Actonel, Actonel+Ca, Aredia, Boniva, Didronel, Fosamax, Fosamax+D,
Sanders, L. J., & Frykberg, R. G. (1991). Diabetic neuropathic Reclast, Skelid, and Zometa) 2008 safety alerts for drugs, biologics,
osteoarthropathy: The Charcot foot. In R. G. Frykberg (Ed.), The medical devices, and dietary supplements. Available at: www.fda.gov/
high risk foot in diabetes mellitus (pp. 297−338). New York: cder/drug/infopage/bisphosphonates/default.htm, January.
Churchill Livingstone. Watkins, P. J., & Edmonds, M. E. (1983). Sympathetic nerve failure in
Sanders, L. J., & Mrdjenovich, D. (1991). Anatomical patterns of bone and diabetes. Diabetologia, 25, 73−77.
joint destruction in neuropathic diabetics. Diabetes, 40(Suppl 1). Wild, S., Roglic, G., Green, A., Sicree, R., & King, H. (2004). Global
Saxena, A., DiDomenico, L. A., Widtfeldt, A., Adams, T., & Kim, W. prevalence of diabetes: Estimates for the year 2000 and projections for
(2005). Implantable electrical bone stimulation for arthrodeses of the 2030. Diabetes Care, 27, 1047−1053.
foot and ankle in high-risk patients: A multicenter study. Journal of Foot Wilson, M. (1991). Charcot foot osteoarthropathy in diabetes mellitus. Mi-
and Ankle Surgery, 44, 450−454. litary Medicine, 156, 563−569.
ARTICLE IN PRESS
18 D.K. Wukich, W. Sung / Journal of Diabetes and Its Complications xx (2008) xxx–xxx

Wright, J. G., Einhorn, T. A., & Heckman, J. D. (2005). Grades of Young, R. J., Zhou, Y. Q., Rodriguez, E., Prescott, R. J., Ewing, D. J., &
recommendation. Journal of Bone and Joint Surgery, American Volume, Clarke, B. F. (1986). Variable relationship between peripheral somatic
87, 1909−1910. and autonomic neuropathy in patients with different syndromes of
Wukich, D. K., & Motko, J. (2004). Safety of total contact casting in high- diabetic polyneuropathy. Diabetes, 35, 192−197.
risk patients with neuropathic foot ulcers. Foot & Ankle International, Yu, G. V., & Hudson, J. R. (2002). Evaluation and treatment of Stage 0
25, 556−560. Charcot's neuroarthropathy of the foot and ankle. Journal of the
Young, M. J., Marshall, A., Adams, J. E., Selby, P. L., & Boulton, American Podiatric Medical Association, 92, 210−220.
A. J. (1995). Osteopenia, neurological dysfunction, and the Zgonis, T., Roukis, T. S., & Lamm, B. M. (2007). Charcot foot and ankle
development of Charcot neuroarthropathy. Diabetes Care, 18, reconstruction: Current thinking and surgical approaches. Clinics in
34−38. Podiatric Medicine and Surgery, 24, 505−517.
1570
C OPYRIGHT Ó 2008 BY T HE J OURNAL OF B ONE AND J OINT S URGERY, I NCORPORATED

Current Concepts Review


The Management of Ankle Fractures in
Patients with Diabetes
By Dane K. Wukich, MD, and Alex J. Kline, MD

ä Patients with diabetes mellitus have higher complication rates following both open and closed management of
ankle fractures.

ä Diabetic patients with neuropathy or vasculopathy have higher complication rates than both diabetic patients
without these comorbidities and nondiabetic patients.

ä Unstable ankle fractures in diabetic patients without neuropathy or vasculopathy are best treated with open
reduction and internal fixation with use of standard techniques.

ä Patients with neuropathy or vasculopathy are at increased risk for both soft-tissue and osseous complications,
including delayed union and nonunion. Careful soft-tissue management as well as stable, rigid internal fixation are
crucial to obtaining a good outcome.

ä Prolonged non-weight-bearing and subsequently protected weight-bearing are recommended following both op-
erative and nonoperative management of ankle fractures in patients with diabetes.

Ankle fractures are among the most common injuries treated by diabetes that pose a problem in the management of ankle frac-
orthopaedic surgeons, with an estimated 260,000 occurring per tures (impaired wound-healing, delayed fracture-healing, and
year in the United States1,2. With an ever-aging population, the neuropathy), the information in the literature as it relates to
number of ankle fractures continues to rise3. While protocols outcomes and complications following the management of these
for management of ankle fractures are generally well established, fractures in diabetics, and current evidence regarding the op-
treatment of these injuries in patients with diabetes mellitus timal management of ankle fractures in patients with diabetes.
poses a challenge for orthopaedists. These patients have unique A critical analysis of the existing evidence regarding the impact
characteristics, including delayed fracture-healing, impaired of diabetes on ankle fractures will lead to a better understanding
wound-healing, vasculopathy, and neuropathy, that all must be of this problem and the ability to make better decisions for
taken into account when formulating a treatment plan. patient management.
As the prevalence of diabetes mellitus has continued to
increase, so too has the number of ankle fractures seen in this Epidemiology of Diabetes
patient population. While much has been published on the im- According to statistics from the Centers for Disease Control and
pact of diabetes on the treatment of ankle fractures, the majority Prevention in 2005, 20.8 million people (7% of the population) in
of studies have included a small number of patients and have been the United States had diabetes mellitus4. In the population over
either retrospective reviews or case-control studies. In this review, the age of sixty years, 10.3 million (20.2% of the population)
we will report the impact of diabetes on the management of ankle were affected. Additionally, 1.5 million new cases of diabetes
fractures. Specifically, we will examine the epidemiology of dia- were diagnosed in 2005 alone4. The prevalence of diabetes in-
betes as it relates to ankle fractures, the specific characteristics of creased by 61% between 1990 and 2001, and it is estimated that

Disclosure: The authors did not receive any outside funding or grants in support of their research for or preparation of this work. Neither they nor a
member of their immediate families received payments or other benefits or a commitment or agreement to provide such benefits from a commercial
entity. No commercial entity paid or directed, or agreed to pay or direct, any benefits to any research fund, foundation, division, center, clinical practice,
or other charitable or nonprofit organization with which the authors, or a member of their immediate families, are affiliated or associated.

J Bone Joint Surg Am. 2008;90:1570-8 d doi:10.2106/JBJS.G.01673


1571
THE JOURNAL O F B O N E & J O I N T S U R G E RY J B J S . O R G
d
THE MANAGEMEN T OF AN K L E FR AC T U R E S IN P AT I E N T S
V O L U M E 9 0-A N U M B E R 7 J U LY 2 008
d d
WITH DIABETES

this number will further increase by 165% between 2000 and all diabetic patients over the age of sixty have some degree of
2050, with the fastest rates of increase occurring in older and peripheral neuropathy 13. The subgroup of patients with a loss of
minority subpopulations5. Overall, the estimated cost incurred protective sensation in the foot and ankle is at particular risk for
by the United States economy as a result of diabetes and its complications following an ankle fracture. In general, peripheral
complications is more than 100 billion dollars annually 6. Similar neuropathy must be profound prior to the loss of protective
increases in the prevalence of diabetes and its associated costs sensation. Once protective sensation is lost, the risk of foot
have been reported throughout the world7,8. ulcerations increases sevenfold because of the increased vul-
The authors of a recent study estimated the overall nerability to unrecognized trauma14,15. While the so-called gold
lifetime risk of diabetes developing in an individual born in the standard for diagnosing peripheral neuropathy remains nerve
United States in the year 2000 to be 32.5% for males and 38.5% conduction studies, the most commonly used instrument clin-
for females9. The highest rates were predicted to occur in the ically is the 10-g (5.07) nylon Semmes-Weinstein monofilament
Hispanic population (a 45.4% risk for males and a 52.5% risk test16-19. This simple test can identify persons at an increased risk
for females). The diagnosis of type-2 diabetes at the age of forty for foot ulceration with a sensitivity of up to 91% and a speci-
was predicted to be associated with a decreased life expectancy ficity of up to 86%20-22. Vibration testing with a 128-Hz tuning
of 11.6 life-years for males and 14.3 life-years for females. Ad- fork can also be used and may be an even more sensitive pre-
ditionally, the number of quality-adjusted life-years was pre- dictor of early neuropathy 16. It is important to identify neu-
dicted to be decreased by 18.6 for males and 22.0 for females. ropathy in all diabetic patients and particularly in those who
have sustained an ankle fracture.
Pathophysiology of Diabetes The severity of diabetic neuropathic complications is di-
Diabetes mellitus constitutes a heterogeneous group of metabolic rectly related to both the lack of control and the chronicity of
disorders that share the common manifestation of hyperglyce- abnormal glucose metabolism23. The importance of control is
mia. The diagnosis of diabetes is contingent on a fasting blood highlighted by the fact that a 1% reduction in the hemoglobin
glucose level of ‡126 mg/dL (‡7 mmol/L) measured on two A1C level results in approximately a 25% to 30% reduction in
separate occasions, random glucose levels of ‡200 mg/dL (‡11 the rate of complications10. Under normal glucose homeostasis,
mmol/L) with symptoms (polyuria, polydipsia, or unexplained the entry of glucose into the cell is tightly regulated by insulin.
weight loss), or a positive glucose challenge of ‡200 mg/dL10. However, in diabetic patients, insulin impairment leads to in-
Broadly, diabetes can be considered to present in two forms creased glucose levels in the bloodstream and subsequently to
(type 1 and type 2). Type-1 diabetes is caused by the autoim- increased diffusion of glucose into the cells. The end product of
mune destruction of insulin-producing b-cells in the islets of this is the formation of advanced glycosylation end products,
Langerhans in the pancreas. This leads to an absolute decrease in which are stable and irreversible. In red blood cells, these can be
the amount of circulating insulin. Circulating insulin is virtually measured as the hemoglobin A1C level, a marker of long-term
absent, and pancreatic b-cells fail to respond to normal insu- glucose control. Ultimately, the effects of hyperglycemia on
linogenic stimuli. Therefore, exogenous insulin must be ad- peripheral nerves are twofold, direct neuronal injury and mi-
ministered in order to maintain adequate glycemic control. crovascular damage.
Type-2 diabetes (formerly known as adult-onset diabetes) On a molecular level, cellular homeostasis relies on the
is characterized by increased peripheral insulin resistance com- tightly regulated use and production of certain reactive oxygen
bined with a secretory defect in insulin by the pancreatic b-cells. species (nitric oxide, hydrogen peroxide, and superoxide) that
Both components must be present for the clinical manifestation play crucial roles in the normal functioning of the cell. These
of diabetes. Overall, 90% of the cases of diabetes in the United processes are very sensitive to glycemic control. In the presence of
States are type 2, and 90% of these patients are clinically obese. excess glucose, the tight regulation of reactive oxygen species is
Regardless of the underlying cause of diabetes, the com- disrupted and excess reactive oxygen species are produced23,24.
mon manifestation is systemic hyperglycemia. In turn, this Within the nerve cells, the excess reactive oxygen species cause
leads to the glycosylation of proteins and the increased forma- direct injury to both cellular proteins and membrane lipids.
tion of intracellular sorbitol and other polyols. The end product Additionally, toxic peroxidation products accumulate and bind
is tissue damage in a number of end organs. Patients all show to normal cellular nuclear material, leading to increased apo-
some degree of immune dysfunction, peripheral neuropathy, ptosis, DNA damage, and decreased axonal transport. Hyper-
nephropathy, retinopathy, and arthropathy. glycemia also ultimately results in a decreased production of
neurotrophic factors that are responsible for the health and
Diabetic Neuropathy maintenance of normal nerve function25,26. On the vascular level,
Diabetic neuropathy profoundly impacts the management of chronic excess production of superoxide species leads to a loss of
ankle fractures. In the United States, 10% of diabetic patients normal nitric oxide function, resulting in vasoconstriction and
have some degree of neuropathy at the time of the initial diag- nerve ischemia.
nosis of the diabetes, and up to 40% will be diagnosed with
peripheral neuropathy within the first decade following that di- Charcot Arthropathy
agnosis1,11,12. In older patients, in whom ankle fractures are more Charcot arthropathy is defined as a noninfectious, destructive
common, neuropathy is even more prevalent. More than 50% of process culminating in eventual dislocation and periarticular
fracture in patients with peripheral neuropathy and the loss of dismissed as a sprain, contusion, cellulitis, gout, or deep vein
protective sensation27-29. While diabetes is the most common thrombosis.
cause of Charcot arthropathy today, it can potentially result The initial classification system described by Eichenholtz
from any clinical entity that renders a patient insensate to was based on clinical and radiographic data of patients with
protective sensation. Although Charcot arthropathy is relatively Charcot arthropathy 36. Stage I (development-fragmentation)
uncommon, the potentially serious complications following a refers to the initial, acute phase in which the cardinal signs of
delayed diagnosis of this condition make early detection ex- hyperemia, warmth, and swelling are present. Radiographically,
tremely important. As recent evidence has highlighted, a high a continuum of joint subluxation and dislocation, articular
index of suspicion and early intervention can potentially avoid erosions, and osseous destruction is observed. Stage II (coales-
some of the historically poor outcomes30. cence) is heralded by the resolution of the cardinal clinical signs
Charcot arthropathy was originally described in the 1860s and the transition from destruction to the reparative process.
by the neurologist Jean-Martin Charcot 28. The hypertrophic, Radiographically, sclerosis is present and, as the bones begin to
destructive neuroarthropathy that he initially described has coalesce, their inherent stability increases and the deformity
since been shown to occur in diabetic patients with peripheral ceases to rapidly progress. Stage III (reconstruction-consolidation)
neuropathy. The prevalence of Charcot arthropathy is esti- refers to the hypertrophic, remodeling phase in which tra-
mated to be approximately 0.3% among all diabetic patients, becular remodeling and the maturation of the osseous archi-
although it is thought that this may be an underestimate of its tecture into a stable, almost ankylosed state occur.
true burden31. The only symptom of diabetes that has been The Eichenholtz classification ignores the initial phase of
shown to be predictive of the potential for the development of inflammatory swelling in which the radiographic appearance
Charcot arthropathy is the loss of protective sensation32,33. remains normal or nearly normal. It is patients with this stage of
Two distinct theories have been postulated as potential the disease who are often initially dismissed as having a bruise or
causes of Charcot arthropathy: the neurotraumatic theory and strain, and unfortunately this is also the group that can most
the neurovascular theory 27. In reality, the cause is likely mul- likely be successfully treated if diagnosed early. Shibata et al. were,
tifactorial, and overlap between the theories is certainly feasi- to our knowledge, the first to describe Stage-0 neuroarthropathy
ble. The neurotraumatic theory postulates that an initial injury in their series of patients with leprosy 37. In 1995, Schon and
(either a single macrotraumatic event or repetitive micro- Marks referred to Stage 0 as occurring in patients with neu-
trauma) activates the process. The loss of protective sensation ropathy who sustain an acute fracture38. Chantelau et al. sub-
leads to continued weight-bearing by the patient and the lack sequently defined Stage-0 Charcot arthropathy as occurring in
of protective unloading. Eventually, there is a loss of structural the six to twelve-month period in which the clinical signs of
integrity, and finally a reparative process begins. The neuro- a Charcot foot develop without any abnormality seen on plain
vascular theory predicates a state of hyperemia resulting from radiographs39,40. Magnetic resonance imaging at this stage will
abnormal vasomotor control secondary to the underlying neu- show a pattern of bone-marrow edema consistent with osseous
ropathy. This leads to an increase in local blood flow and, stress injury. When diagnosed at this early stage, the Charcot
secondarily, to an increased level of osteoclast stimulation. foot can be managed with immobilization and non-weight-
Eventually, this results in increased bone turnover and a relative bearing and this can often successfully prevent the development
local osteopenia, which leaves the osseous architecture suscepti- of osseous destruction and deformity 39.
ble to minor trauma. This theory is supported by the fact that
local tissue samples from Charcot joints show an increased
osteoclast-to-osteoblast ratio mediated by a local cytokine path-
way 34. Recently, overexpression of RANKL (receptor activator of
nuclear factor kappa-B ligand) has been implicated as a mech-
anism for the development of Charcot arthropathy35.
The classic clinical description of an early Charcot foot
or ankle is one with painless erythema, warmth, and swelling
of the involved joints. Many patients still have some degree of
pain on initial presentation. Often, a patient with an acute
Charcot foot is initially misdiagnosed with an infection sec-
ondary to the erythematous, inflamed appearance of the foot.
However, these patients lack signs of systemic illness, such as
fever or elevated white blood-cell counts. Additionally, the
erythema will resolve rapidly with elevation of a Charcot foot
but not with elevation of an infected foot. In the absence of an
open, draining ulceration as a conduit for deep infection, in-
fection is unlikely to be the cause of the acute inflammation.
When observed prior to the development of osseous changes
on radiographs, these clinical symptoms are often initially
ERROR: undefined
OFFENDING COMMAND: get

STACK:
/3
-dictionary-
1
This is an enhanced PDF from The Journal of Bone and Joint Surgery

The PDF of the article you requested follows this cover page.

The key role of the lateral malleolus in displaced fractures of the ankle
IG Yablon, FG Heller and L Shouse
J Bone Joint Surg Am. 1977;59:169-173.

This information is current as of November 7, 2007

Reprints and Permissions Click here to order reprints or request permission to use material from this
article, or locate the article citation on jbjs.org and click on the [Reprints and
Permissions] link.
Publisher Information The Journal of Bone and Joint Surgery
20 Pickering Street, Needham, MA 02492-3157
www.jbjs.org
The Key Role of the Lateral Malleolus in
Displaced Fractures of the Ankle*
BY ISADORE G. YABLON, M.D.t, FREDERICK G. HELLER, M.D.t, AND
LEROY SHOUSE, M.D.t, BOSTON, MASSACHUSETTS

From the Boston University Medical Center, Universiti Hospital, Boston

ABSTRACT: The reason why late degenerative ar- Materials and Methods
thritis developed in some patients who had sustained Laboratory Studies
displaced bimalleolar fractures of the ankle was inves-
Ankles from fresh cadavera were stripped of skin,
tigated. The roentgenograms indicated that incomplete
muscles, and tendons, and two heavy Kirschner wires
reduction of the lateral malleolus and a residual talar
were inserted transversely through the talus parallel to the
tilt were present. When bimalleolar fractures were
anklejoint. These wires were used to stabilize the foot and
created in cadavera the talus could be anatomically re-
to measure rotatory and sagittal displacements of the talus.
positioned only when the lateral malleolus was accu-
Four individual specimens were used for each set of exper-
rately reduced. Fifty-three patients with bimalleolar
iments and each maneuver was repeated five times. Four
fractures were treated by anatomically fixing the lat-
separate studies were done, to assess the effects on ankle
eral malleolus with a four-hole plate. There was an
stability after: (1 ) isolated division of the deltoid ligament,
anatomical reduction of the talus and medial malleolus
(2) isolated division of the fibular collateral ligaments, (3)
in each instance and there were no late cases of de-
transverse osteotomy of the medial malleolus at the level
generative arthritis when these patients were followed
of the joint with all ligaments intact, and (4) short oblique
for from six months to nine years. We concluded that
osteotomy of the lateral malleolus with all ligaments in-
the lateral malleolus is the key to the anatomical reduc-
tact. The amount of rotatory displacement was measured
tion of bimalleolar fractures, because the displacement
with a protractor and determined by the amount of rotation
of the talus faithfully followed that of the lateral mal-
of the Kirschner wires from the longitudinal axis of the
leolus.
tibia and fibula. Varus deformity was measured by noting
the angle between the tibial plafond and the wires. The
It has generally been accepted that reduction of the
final result was expressed as the mean of the values ob-
medial malleolus is of paramount importance in treating
tamed from each of the individual observations.
unstable bimalleolar fractures of the ankle 1,2 It is further
In a fifth series of experiments, after the pins had
usually believed that even if the lateral malleolus is intact
been inserted the medial malleolus was osteotomized at
stability of the ankle is lost when the medial malleolus is
the level of the joint line and an oblique osteotomy was
fractured, especially if the fracture is transverse and at the
performed on the lateral malleolus to simulate an external
level of the joint Most relevant. publications emphasized
rotation-abduction fracture. The ankle was placed in val-
that reduction of the medial malleolus should be anatomi-
gus and 30 degrees of external rotation to create the de-
cal and that once it is achieved there is complete reduc-
formities seen on the initial roentgenograms of such in-
tion of the displaced talus 4.5.8.10.12,16.19.22.25.26#{149} It was
juries. Manual reduction was then attempted by applying a
also stated that any residual talar displacement can
varus stress and internally rotating the ankle, as would be
be corrected by manually reducing the lateral malleolus
done clinically.
after the medial malleolus has been securely fixed in
place ‘ 1 In following patients who had sustained displaced Clinical Material
abduction-external rotation bimalleolar fractures of the
Fifty-three patients who had sustained abduction-
ankle, we became aware that a number of the patients had
external rotation injuries of the ankle were studied. There
late degenerative changes in the ankle. The arthritis had its
were thirty-one women and twenty-two men. The age
onset one to eight years after the injury. The present study
range was from nineteen to eighty-six years, with an aver-
was undertaken in an attempt to determine why this com-
age of thirty-four years. Open fractures were not included
plication occurred.
in this series. Forty-two patients had a bimalleolar fracture
* Read in part at the Annual Meeting ofThe American Academy of and eleven had a fracture of the lateral malleolus and a tear
Orthopaedic Surgeons, New Orleans, Louisiana, February 2, 1976.
t University Hospital, 75 East Newton Street, Boston, Ma.ssachu-
of the deltoid ligament, as evidenced by lateral talar shift.
setts 02118. This latter group was treated by open reduction and inter-

VOL. 59-A, NO. 2. MARCH 1977 I 69


170 I. G. YABLON, F. G. HELLER, AND LEROY SHOUSE

nal fixation of the lateral malleolus using a four-hole plate.


The deltoid ligament was not repaired. The stability and
quality of reduction of the fracture were assessed
roentgenographically.
The first seventeen patients in this series with bimal-
leolar fractures had the medial malleolus reduced and fixed INVERSION
with a screw in the conventional manner, in order to de-
termine whether primary reduction of the medial malleolus
would reposition the talus. Intraoperative roentgenograms
were then obtained. In fourteen patients there was an in-
complete reduction of the talus and lateral malleolus. In
these patients the screw in the medial malleolus was re-
FIG. 1-A
moved, following which the lateral malleolus was exposed
Anterior view of displacement, with the fibula intact but the foot
through a separate incision and accurately reduced with a obliquely positioned. The vertical line represents the axis of the fibula.
four-hole plate without replacing the screw in the medial
malleolus. In the three patients in whom intraoperative
roentgenograms revealed that the talus was anatomically
reduced, the screw in the medial malleolus was left in
place but a four-hole plate was also applied to the lateral
malleolus . Postreduction roentgenograms were obtained
in all cases.
In order to determine what effect primary anatomical
INVERSION
reduction of the lateral malleolus would have on these dis-
placed fractures, in ten patients in this series the lateral
malleolus was exposed and reduced under direct vision.
The reduction was held in place with a blunt periosteal
elevator and intraoperative roentgenograms were made to
observe the position of the talus and medial malleolus. A
four-hole plate was then applied to the lateral malleolus
FIG. 1-B
and intraoperative stress roentgenograms were made to
The distal end of the fibula has been resected.
test the stability of the ankle.
Postoperatively, in all cases the extremity was ele-
vated in a canvas sling suspended from a Balkan frame for began to move into position but its progress was arrested
five days. A below-the-knee cast was then applied and the because the lateral malleolus began to impinge on the
patient was allowed to walk, non-weight-bearing, on proximal fibular fragment, preventing further internal rota-
crutches. A below-the-knee walking cast was applied after tion. Securely fixing the medial malleolus with a screw did
four weeks. not improve the position of the talus. However, when the
The patients were followed for six months to nine ankle was forcefully internally rotated after the medial
years (from 1966 to 1975), with an average of 5.5 years. malleolus had been fixed, the talus could be anatomically
Twenty-nine patients were followed for more than six repositioned; but this occurred by stretching the lateral
years. malleolar ligament complex. When the internal rotatory
force was discontinued the talus resumed its displaced p0-
Results
sition.
Experimental Findings
When the deltoid ligament alone was divided, no in- Clinical Findings

stability of the ankle resulted. Resection of the medial In the fifty-three patients in whom the lateral mal-
malleolus at the level of the joint line allowed about 10 de- leolus was accurately reduced and fixed with a four-hole
grees of rotatory displacement of the ankle but very little plate, postreduction roentgenograms revealed an anatomi-
valgus instability. When only the fibular collateral liga- cal reduction of the talus. In the seventeen patients who
ments were divided there was approximately 30 degrees of had primary fixation of the medial malleolus, fourteen (as
external rotatory instability and marked talar instability already mentioned) showed evidence of a residual talar
(Figs. 1-A and 1-B). Resecting the lateral malleolus pro- tilt. When the screw was removed and the lateral mal-
duced marked rotatory and valgus instability, and this in- leolus was anatomically reduced, the talus and medial mal-
creased as progressively greater force was applied. leolus were seen to be accurately repositioned (Figs. 2-A
When abduction-external rotation fractures were through 2-D). No instability was demonstrated on the
created and manual reduction was attempted by applying stress roentgenograms when the lateral malleolus was re-
varus stress and internally rotating the ankle, the talus duced and fixed with a four-hole plate (Figs. 3-C and 4-B).

THE JOURNAL OF BONE AND JOINT SURGERY


ROLE OF LATERAL MALLEOLUS IN DISPLACED FRACTURES OF THE ANKLE 171

FIG. 2-A FIG. 2-B FIG. 2-C FIG. 2-D


FIg. 2-A: Anteroposterior roentgenogram showing a displaced bimalleolar fracture.
Fig. 2-B: lntraoperative roentgenogramof the same ankle. An attemptwas made to reduce the talus manually and the medial malleolus is fixed with
a screw . Although the repositioning of the medial malleolus is accurate, there is residual talar displacement and incomplete reduction of the lateral
malleolus.
Fig. 2-C: lntraoperative photograph of the same ankle, demonstrating incomplete reduction of the lateral malleolus.
Fig. 2-D: Postreduction roentgenogram. The screw was removed from the medial malleolus and a four-hole plate was applied over the distal part of
the fibula after the lateral malleolus was anatomically reduced. The talus is in its normal position and the medial malleolus has been reduced. The
radiolucent area between the medial malleolus and the tibia is due to traumatic loss of bone.

There was no instance of periosteal or posterior tibial- One patient had a superficial skin slough over the plate,
tendon interposition preventing reduction of the medial but the wound did not become infected and closed by sec-
malleolus. There was no evidence of late degenerative ar- ondary intention. The total immobilization time averaged
thritis, the talus remained in its anatomical position, and ten weeks.
there were no cases of non-union. There were two infec- All patients experienced swelling of the ankle after
tions which were treated by closed suction-irrigation, and the cast was removed, but this resolved in an average of
the fractures eventually united after four and five months. three weeks. Seven patients still had mild asymptomatic

FIG. 3-A FIG. 3-B FIG. 3-C


Fig. 3-A: Oblique roentgenogram of a right ankle showing a displaced bimalleolar fracture.
Fig. 3-B: lntraoperative roentgenogram showing repositioning of the talus and medial malleolus by accurately reducing the lateral malleolus and
holding it in position with a blunt periosteal elevator.
Fig. 3-C: Postoperative roentgenogram. An anatomical reduction was achieved by the application of a four-hole plate.

VOL. 59-A, NO. 2, MARCH 1977


172 I. G. YABLON, F. G. HELLER, AND LEROY SHOUSE

FIG. 4-A FIG. 4-B FIG. 5


F ig . A : Anteroposterior
4 - roentgenogram of a d i splaced bimalleolar fracture.
Fig. 4-B: lntraopcrative roentgenograrn of the same fracture. An anatomical reduction was achieved by reducing the lateral malleolus and applying
a tour-hole plate. A valgus and external-rotation stress was applied. The ankle retains its stability despite the fact that the medial malleolus was not
fixed.
Fig. 5: Anteroposterior rocntgenogram of the ankle of a patient who had sustained a fracture of the lateral malleolus and a tear of the deltoid
ligament. with lateral and rotatory talar displacement. The talus follows the dIsplacement of the lateral malleolus.

swelling. Forty-nine patients became ambulatory without ruptured or lax, and the paucity of communications ap-
support. Three patients had walked with a cane prior to the pearing in the literature describing late reconstruction of
accident and continued to do so after their fractures had the deltoid ligament may also correlate with these observa-
healed. Thirty-eight patients regained a full range of pain- tions. On the other hand, lateral ligament laxity is a well
less motion within six weeks after cast removal. Nine of established clinical entity and a number of effective surgi-
the remaining fifteen patients lacked 10 degrees of dor- cal techniques to restore ankle stability have been de-
siflexion one year postoperatively. Six patients lacked an scribed 7.13

average of 10 degrees of full plantar flexion: four were It was of interest to observe experimentally why
asymptomatic, but two continued to complain of pain on anatomical reduction of a displaced bimalleolar fracture
walking. Roentgenograms revealed demineralization of could not always be achieved manually. Impingement of
the tarsal bones suggestive of Sudeck’s atrophy. With the the lateral malleolus on the proximal fibular fragment pre-
exception of these two instances, all patients were able to vented the talus from resuming its anatomical position.
resume their normal preinjury activities.

Discussion

Unsatisfactory long-term results of treatment of dis-


placed bimalleolar ankle fractures were reported by
others. Cedell reviewed the results of treatment of over
400 abduction-external rotation fractures. He postulated
that late pain, swelling, and degenerative changes were
due to persistent laxity of the anterior tibiofibular liga-
ment. Phillips and associates reported that 44 per cent of
thirty-six bimalleolar fractures treated conservatively con-
tinued to cause pain. Several authors 14.15.17.21.24 demon-
strated that incomplete reduction of the lateral malleolus
was the most common fault in the treatment of these frac-
tures. In all of the symptomatic patients reported in the lit-
erature, it was found that there was incomplete reduction
of the lateral malleolus and residual talar tilt.
The cadaver studies showed, surprisingly, that divi-
sion of the deltoid ligament or medial malleolus did not
create an important degree of instability of the ankle. This Anteroposterior and oblique roentgenograms of the ankle of a patient
who sustained a tear of the deltoid ligament and a diastasis of the inferior
observation fits in well with the clinical findings of a low
tibiofibular joint. The talus follows the displacement of the lateral mal-
frequency of ankle instability when the deltoid ligament is leolus.

THE JOURNAL OF BONE AND JOINT SURGERY


ROLE OF LATERAL MALLEOLUS IN DISPLACED FRACTURES OF THE ANKLE 173

Secure fixation of the medial malleolus with a compres- ankle in such cases, but this stretches the fibular collateral
sion screw did not improve the displaced position of the ligament. When external immobilization is discontinued,
talus, and the talus could be anatomically reduced only by the lateral ligaments remain in a stretched position and
forced abduction and internal rotation of the ankle; but this slight to moderate talar instability, which predisposes to
occurred by stretching the lateral collateral ligament com- the development of late degenerative arthritis, may be the
plex. The lateral malleolus, however, still maintained its result.
displaced position. On discontinuation of the manual On the basis of these studies it would appear that the
force, the talus resumed its displaced position. lateral malleolus is the key to the anatomical reduction of
In reviewing our clinical material of other injuries to displaced bimalleolar fractures, and that restoring the in-
the ankle which involved talar displacement with fracture tegrity of the lateral malleolus establishes stability of the
only of the lateral malleolus or talofibular diastasis, not in- ankle 18.23 This does not necessarily lessen the importance
eluded in the series described, we found that in many in- of the medial malleolus in contributing congruity to the
stances the talus followed the lateral malleolus (Figs. 5 medial aspect of the ankle, but it does serve to emphasize
and 6). These observations strongly suggest that in un- that the lateral malleolus should no longer be ignored or
stable bimalleolar fractures, or those fractures in which the disregarded in the treatment of these injuries.
lateral malleolus is fractured and the deltoid ligament is There was no instance in our fifty-three cases in which
torn, the talus remains attached to the lateral malleolus. soft-tissue interposition prevented replacement of the me-
Reducing the medial malleolus alone may prevent anatom- dial malleolus when the lateral malleolus was reduced9.
ical repositioning of the talus, because in some cases the One can only assume that this complication is not as corn-
lateral malleolus cannot be accurately reduced when it im- mon as is generally believed. Perhaps the interposing tis-
pinges on the proximal fibular fragment. Repositioning of sues are replaced in their original sites by the talus as it
the talus can be achieved by forcibly internally rotating the resumes its anatomical position.

References
1. ASHHURST, A. P. C., and BROMER, R. S.: Classification and Mechanism of Fractures of Leg Bones Involving the Ankle. Arch. Surg.. 4: 51-
129. 1922.
2. BONNIN, J. G.: Injuries to the Ankle. London, Heineman, 1950.
3. BRAUNSTEIN. P. W., and WADE. P. A.: Treatment of Unstable Fractures of the Ankle. Ann. Surg. , 149: 217-226, 1959.
4. BURGESS, ERNEST: Fractures of the Ankle. J. Bone and Joint Surg., 26: 721-732, Oct. 1944.
5. BURWELL, H. N., and CHARNLEY, A. D.: The Treatment of Displaced Fractures at the Ankle by Rigid Internal Fixation and Early Joint Move-
ment. J. Bone and Joint Surg., 47-B: 634-660, Nov. 1965.
6. CEDELL, C-A.: Supination-Outward Rotation Injuries of the Ankle. A Clinical and Roentgenological Study with Special Reference to the Op-
erative Treatment. Acta Orthop. Scandinavica, Supplementum I 10. 1967.
7. CHRISMAN. 0. D. , and SNOOK. G. A.: Reconstruction of Lateral Ligament Tears of the Ankle. An Experimental Study and Clinical Evaluation
of Seven Patients Treated by a New Modification of the Elmslie Procedure. J. Bone and Joint Surg. , 51-A: 904-912, July 1969.
8. CONWELL, H. E., and REYNOLDS, F. C.: Key and Conwell’s Management of Fractures, Dislocations, and Sprains. Ed. 7. St. Louis, C. V.
Mosby. 1961.
9. COONRAD, R. W.. and BUGG. E. I., JR.: Trapping ofthe PosteriorTibialTendon and Interposition ofSoftTissue in Severe Fractures about the
Ankle Joint. J. Bone and Joint Surg. . 36-A: 744-750. July 1954.
10. Cox, F. J., and LAXON, W. W.: Fractures About the Ankle Joint. Am. J. Surg. , 83: 674-679, 1952.
11. CRENSHAW. W. H. [editor]: Campbell’s Operative Orthopedics. Ed. 5. St. Louis, C. V. Mosby, 1971.
12. DENHAM, R. A.: Internal Fixation for Unstable Ankle Fractures. J. Bone and Joint Surg., 46-B: 206-211, May 1964.
13. FREEMAN. M. A. R.: Treatment of Ruptures of the Lateral Ligament of the Ankle. J. Bone and Joint Surg., 47-B: 661-668, Nov. 1965.
14. lSEi.IN, MARC, and DE VELLIS. H.: La primaut#{233}du p#{233}ron#{233} dans les fractures du cou-de-pied. M#{233}m.Acad. chir.. 87: 399-408, 1961.
15. KLEIGER, BARNARD: The Treatment of Oblique Fractures of the Fibula. J. Bone and Joint Surg., 43-A: 969-979, Oct. 1961.
16. LEE, H. G.. and HORAN. T. B.: Internal Fixation in Injuries ofthe Ankle. Surg., Gynec. and Obstet.. 76: 593-599. 1943.
17. MAGNUSSON. RAGNAR: On the Late Results in Non-operated Cases of Malleolar Fractures. A Clinical-Roentgenological-Statistical Study. 1.
Fractures by External Rotation. Acta Chir. Scandinavica, Supplementum 84, 1944.
18. MULLER, M. E.; ALLGdWER, M.; and WILLENEGGER, H.: Manual ofinternal Fixation. Berlin, Springer, 1970.
19. PATRICK. JAMES: A Direct Approach to Trimalleolar Fractures. J. Bone and Joint Surg. , 47-B: 26-29, May 1965.
20. PHILLIps, R. S.; BALMER,G. A.; and MONK, C. J. E.: The ExternalRotation Fracture oftheFibular Malleolus. British J. Surg., 56: 801-806,
1969.
21. PICAUD, A.-J.: R#{233}fiexions a propos dun traitement chirurgical simple des fractures r#{233}centesde Ia cheville. Rev. chir. orthop.. 39: 570-580,
1953.
22. QUIGLEY, T. B.: Analysis and Treatment of Ankle Injuries Produced by Rotatory, Abduction, and Adduction Forces. In Instructional Course
Lectures. The American Academy ofOrthopaedic Surgeons. Vol. 19, pp. 172-182. St. Louis. C. V. Mosby. 1970.
23. SNEPPEN, 0.: Pseudoarthrosis ofthe Lateral Malleolus. Acta Orthop. Scandinavica, 42: 187-200, 1971.
24. SOLONEN. K. A.. and LAUTTAMUS. LEO: Operative Treatment of Ankle Fractures. Acta Orthop. Scandinavica, 39: 223-237, 1968.
25. STAPLES. 0. S.: Injuries to the Medial Ligaments of the Ankle. Result Study. J. Bone and Joint Surg.. 42-A: 1287-1307, Dec. 1960.
26. WIlSoN. F. C.. JR.. and SKILBRED, L. A.: Long-Term Results in the Treatment ofDisplaced Bimalleolar Fractures. J. Bone and Joint Surg.,
48-A: 1065-1078, Sept. 1966.

VOL. 59-A, NO. 2, MARCH 1977


Predislocation Syndrome
Progressive Subluxation/Dislocation of the
Lesser Metatarsophalangeal Joint
Gerard V. Yu, DPM*
Molly S. Judge, DPM†
Justin R. Hudson, DPM‡
Frank E. Seidelmann, DO‡

Progressive subluxation/dislocation of the lesser toes resulting from idio-


pathic inflammation about one or more of the lesser metatarsophalangeal
joints is a common cause of metatarsalgia that is frequently unrecognized
or misdiagnosed. The disorder results from a failure of the plantar plate
and collateral ligaments that stabilize the metatarsophalangeal joints
and is typically associated with abnormal forefoot loading patterns. The
authors refer to this condition as predislocation syndrome and have de-
vised a clinical staging system that is based on the clinical signs and
symptoms present during examination. A thorough review of predisloca-
tion syndrome and an overview of the conservative and surgical treat-
ment options available for this disorder are presented. (J Am Podiatr
Med Assoc 92(4): 182-199, 2002)

Painful instability of the lesser metatarsophalangeal migration of the proximal phalanx. A hammer toe de-
joint has been recognized in both the orthopedic and formity may be found in association with the disor-
the podiatric literature as a common cause of meta- der, but is usually not an early or a significant com-
tarsalgia.1-6 The condition is typically described as a ponent. Unlike in the classic hammer toe deformity,
progressive, painful subluxation or dislocation of the a dorsal corn overlying the proximal interphalangeal
proximal phalangeal base on the metatarsal head joint is typically not observed (Fig. 1). Although any
that occurs as a result of weakening of the periarticu- lesser metatarsophalangeal joint can be affected, the
lar structures that stabilize the metatarsophalangeal second metatarsophalangeal joint is affected most
joint—in particular, the plantar plate. The subluxa- often.7, 8 Hallux valgus and first-ray insufficiency may
tion/dislocation process can be slow or rapidly pro- contribute to this problem but are not necessarily
gressive. Medial or, less commonly, lateral deviation precursors or prerequisites.1-6 A long second metatar-
of the digit may also occur and accompanies dorsal sal has also been implicated in the development of
this entity.1
*Diplomate, American Board of Podiatric Surgery; Fel-
low, American College of Foot and Ankle Surgeons; Direc-
While pain about any lesser metatarsophalangeal
tor of Podiatric Medical Education and Residency Training, joint may be associated with several systemic and
St Vincent Charity Hospital, Cleveland, OH; Faculty Mem- local processes, the idea of idiopathic pain and insta-
ber, The Podiatry Institute, Tucker, GA. Mailing address: bility of the metatarsophalangeal joint is relatively
23823 Lorain Rd, Ste 280, North Olmstead, OH 44070. new.6, 9 The literature is replete with numerous terms
†Director of Externship Program, St Vincent Charity Hos-
pital, Cleveland, OH; private practice, Toledo, OH.
describing the components of lesser toe derangement.
‡Submitted during third-year podiatric surgical residency, The following terms, for example, are slightly modi-
St Vincent Charity Hospital, Cleveland, OH. fied from a list appearing in Miller10:
‡Adjunct Professor of Radiology, Ohio College of Podi-
atric Medicine, Cleveland; Clinical Professor of Radiology, ● Sub–metatarsal 2/neuroma syndrome
Ohio University, Athens; private practice, Cleveland, OH. ● Chronic lesser metatarsophalangeal dislocation

182 April 2002 • Vol 92 • No 4 • Journal of the American Podiatric Medical Association
plain that they feel as if they are walking on a “stone
bruise” or “lump” on the bottom of the foot in spite
of the absence of any obvious plantar lesion. Patients
may also complain of swelling about the base of the
toe, more impressive plantarly than dorsally. Some
patients will note that the toe “feels as though it is
trying to go out of position,” and others may have
even noted a distinct change in position over the
course of several weeks. Rarely is a lesion present
over the dorsal aspect of the proximal interphalange-
al joint, although mild erythema may be present as a
result of shoe irritation of the knuckle pad. Tyloma
formation plantarly is rarely present.
Some patients mention a recent increase or change
in activity level prior to the development of symp-
toms.1 The current authors have seen this problem
develop following participation in such activities as
jogging, tennis, and basketball. In other cases, pa-
tients may recall a minor traumatic event, such as a
Figure 1. Classic hammer toe deformity of the second
misstep on a stair, hole, or protruding object such as
toe with chronic metatarsophalangeal joint subluxa- a garden hose or an electrical cord prior to the devel-
tion. Note the presence of a classic heloma durum opment of symptoms. In a number of cases, the se-
overlying the proximal interphalangeal joint, which is nior author (G.V.Y.) has encountered a history of a
not typically seen in patients with predislocation syn- misstep climbing up or down stairs or a ladder as an
drome.
inciting factor.
In later stages of predislocation syndrome, severe
pain may lead to an antalgic gait, with the patient
compensating by walking on the lateral aspect of the
● Floating toe syndrome
foot. Barefoot weightbearing often becomes intolera-
● Lesser metatarsophalangeal joint instability syn-
ble, especially on hard surfaces. The authors have
drome
found that secondary manifestations, such as lateral
● Second metatarsophalangeal joint dislocation/
column overload and calcaneocuboid joint syndrome,
subluxation
may occur, sometimes becoming as significant as the
● Monoarticular nontraumatic synovitis of the sec-
original chief presenting complaint. A constant lin-
ond metatarsophalangeal joint
gering or long-lasting throbbing sensation may also
● Crossover second-toe deformity
develop, regardless of weightbearing status.
● Second metatarsophalangeal joint instability
Frequently, the patient had sought treatment for
The authors, however, consider many of these to the problem in the past, often after being treated by a
be part of a distinct clinical entity, which they refer to variety of medical and surgical specialists to no avail.
as predislocation syndrome. Once a frank dislocation Treatments may have been either conservative or
occurs, more appropriate descriptive terminology surgical. It is not uncommon for the patient to have
should be employed. This article presents the natural received a diagnosis of second interspace or adjacent
history of predislocation syndrome and reviews the interspace neuromas in spite of the obvious absence
conservative and surgical treatment options available of the subjective complaints classically associated
for this disorder. with neuromas. Many patients will have undergone
neurectomy, some with revisional procedures under-
Clinical Presentation taken for the presumed recurrence of neuroma or
stump neuroma (Fig. 2). The failure of these various
Patients suffering from predislocation syndrome typ- treatments and continuing disability are very frustrat-
ically present with acute, subacute, or, less common- ing for patients and may give rise to anxiety about
ly, chronic focal pain at the plantar aspect of the in- their prognosis.6 Some patients may have been ac-
volved lesser metatarsophalangeal joint or joints. In cused of malingering or suspected of having a psy-
general, pain is most noticeable during ambulation chosomatic illness. A few patients or their spouses
and subsides during rest.11 Frequently, patients com- may even question whether they are imagining the

Journal of the American Podiatric Medical Association • Vol 92 • No 4 • April 2002 183
examination, the focal point of tenderness is clearly
and distinctly the metatarsophalangeal joint area, not
the adjacent interspace. An intermetatarsal injection
of local anesthetic may be helpful when symptoms
are generalized or when the patient has difficulty pin-
pointing the target area of pain.
Active flexor and extensor tendon functions of the
affected toe are present with minimal or no compro-
mise. Palpation and range of motion of the metatar-
sophalangeal joint may elicit capsular crepitus or
mobility of a well-circumscribed, bursa-like projec-
tion. A painful decrease in range of motion of the
joint, particularly in plantarflexion, is often noted.13
Significant malposition is absent in the early stages;
however, the astute clinician readily notes subtle
changes in position over time. Not uncommonly, the
toe manifests a clear tendency toward dorsal migra-
tion (not necessarily dorsiflexion), with or without
transverse plane deviation toward or away from the
hallux (Fig. 3).
Figure 2. Foot of a 45-year-old patient who previously
underwent two neuroma resections from the second
In most cases, evaluation of weightbearing stance
interspace within a 3-month period. This patient sub- demonstrates subtle but obvious malalignment of the
sequently developed complete dislocation of the sec- involved digit. Loss of toe purchase, with or without
ond metatarsophalangeal joint. Note the persistent medial or lateral deviation, is typically present. If this
swelling of the second metatarsophalangeal joint and condition is left unchecked, the toe will continue to
toe. The patient denied having had the symptoms at-
tributable to a typical neuroma.
progressively subluxate dorsally, and when this oc-
curs, increased pain plantarly is predictable. Rarely,
a callus plantar to the metatarsal head or a corn over-
lying the proximal interphalangeal joint of the affect-
problem and feel as if they are acting like hypochon- ed toe (as is typically seen with the classic hammer
driacs. Some patients, out of desperation, will plead, toe deformity that has undergone chronic metatarso-
“Please tell me I’m not crazy!” Not uncommonly, this phalangeal joint subluxation/dislocation) is present
unfortunate chain of events results in significant de- on examination. These keratoses are due to friction
pression and anxiety, further complicating the clini- and shear of associated osseous structures and to
cal course of the complaint. loss of intrinsic muscle balance; they develop only
after long-standing dislocation of the toe.
Physical Examination Change in local topography is common and often
impressive (Fig. 4). Local edema and warmth result-
Specific and accurate pain localization during a phys- ing from inflammation about the metatarsophalangeal
ical examination is paramount for determining the joint are typically present; the increased temperature
structures affected. On physical examination of the may be measured with skin-surface thermometers
patient with predislocation syndrome, one finds ex- and recorded. Subtle edema may be indicated by the
cruciating pain on palpation just distal and plantar to obliteration of normal extensor tendon contours. In
the metatarsal head that is disproportionate to any most cases, the plantar edema is obvious, even pro-
other objective clinical findings. Focal pain in this found, and encompasses the entire metatarsophalan-
area is thought to result from bursitis or inflamma- geal joint area, frequently extending distally into the
tion of the plantar plate.12 Occasionally, inflammation sulcus and base of the toe or toes.
of an intermetatarsal bursa may cause local nerve ir- In the initial stages of predislocation syndrome,
ritation, resulting in neuritic or neuroma-type symp- the lesser metatarsophalangeal joint may be in good
toms. However, patients presenting with pain caused anatomical position without malalignment or con-
by inflammation of the plantar plate generally do not tracture. However, as the condition progresses and
have subjective complaints of numbness or shooting joint instability ensues, dorsal or transverse plane de-
pain or objective sensory deficits in the affected dig- viation develops, with subsequent loss of toe pur-
its on clinical examination.11 In addition, on physical chase. The stages of predislocation syndrome have

184 April 2002 • Vol 92 • No 4 • Journal of the American Podiatric Medical Association
A B

Figure 3. Nonweightbearing (A) and weightbearing (B) views of a patient with predislocation syndrome of the right
second toe. Note the edema surrounding the metatarsophalangeal joint, obliteration of the extensor tendons to the
second digit, and the absence of a significant lesion over the proximal interphalangeal joint. Mild irritation is pres-
ent. Absence of toe purchase and medial deviation are seen on weightbearing.

been described by Yu and Judge6 and are based on tends into the toe. More pronounced deviation and
the clinical findings present at the time of examina- possible subluxation/dislocation of the toe are pres-
tion. In stage 1, there is mild edema plantar, and often ent. While the edema and inflammation eventually
dorsal, to the metatarsophalangeal joint. Extreme subside altogether, the deformity remains unchanged
tenderness is present when the joint is manipulated. or continues to progress. The end result can be a
No anatomical malalignment is noted clinically. In crossover second-toe deformity, with or without a
stage 2, there is moderate edema with noticeable de- concomitant hallux abducto valgus deformity.
viation of the affected digit both clinically and radio- Thompson and Hamilton5 have suggested the use
graphically. Often the patient does not realize that of a vertical stress test, akin to the anterior drawer
the affected toe does not purchase the ground. This sign in the unstable ankle joint, to identify and stage
becomes quite evident in stance evaluation. In stage sagittal plane metatarsophalangeal joint instability.
3, moderate edema is present about the entire cir- In this maneuver, the foot is placed in neutral posi-
cumference of the metatarsophalangeal joint and ex- tion, and the metatarsal head is stabilized between
the examiner’s thumb and index finger. The con-
tralateral hand is used to grip the dorsal and plantar
aspect of the corresponding proximal phalanx base.
The digit to be tested is manipulated upward with
vertical pressure applied by the thumb plantarly. A
purely vertical force is applied to the base of the
proximal phalanx in the dorsal direction. This is dif-
ferent from testing for dorsiflexion (Fig. 5). A posi-
tive vertical stress test result is present when the
proximal phalanx can be translocated 2 mm dorsally
above the metatarsal head. According to Thompson
and Hamilton’s classification scheme, in stage 0 there
is no dorsal translocation of the proximal phalanx. In
stage 1, the phalangeal base can undergo subluxation
dorsally but not dislocation. In stage 2, the pha-
langeal base can be dislocated but may be manually
reduced. In stage 3, the phalangeal base is in a fixed
Figure 4. Edema in a patient with predislocation syn- dislocated position. Accurate assessment of the
drome of the left third metatarsophalangeal joint. The
patient has a congenital syndactyly of the second and amount of dislocation of the base of the proximal
third digits, confusing the clinical presentation. Note phalanx on the metatarsal head may be quantified on
the absence of similar findings in the contralateral foot. a lateral x-ray.

Journal of the American Podiatric Medical Association • Vol 92 • No 4 • April 2002 185
ond metatarsal (or a short first metatarsal) may alter
normal forefoot loading patterns by transferring load
from the first to the second metatarsal head (Fig. 6).
Hallux abducto valgus may also cause abnormal
forefoot loading patterns.2 Lateral deviation of the
hallux into the second digit may cause multiplanar
deviation of the second toe that results in retrograde
buckling at the second metatarsophalangeal joint
level on weightbearing. The use of high-heeled shoes
or heel lifts, which increase forefoot loading by main-
taining the metatarsophalangeal joints in a hyperex-
tended position, have also been implicated in the de-
velopment of predislocation syndrome in one or
more of the lesser digits.1
Figure 5. Positive vertical stress test result with dor- Biomechanical hypermobility may also predispose
sal translocation of the proximal phalanx. Note main-
tenance of the toe parallel to the plantar aspect of the a patient to altered forefoot loading patterns. Exces-
foot while this maneuver is performed. The examiner sive pronation will alter the axis of insertion of the
must be careful not to confuse dorsal translocation peroneus longus tendon in such a way that it loses its
with dorsiflexion. ability to stabilize the first ray during the forefoot-
loading phase of gait. 13 Accordingly, metatarsus
primus elevatus resulting from either biomechanical
dysfunction or structural deformity may cause exces-
A thorough history and physical examination are sive load transfer to the adjacent lesser metatarso-
required to rule out all systemic and local problems phalangeal joint or joints. Similarly, first-ray insuffi-
that may contribute to pain about the metatarsopha- ciency caused by generalized joint hypermobility
langeal joint. Degenerative joint disease, avascular syndromes, such as Ehlers-Danlos syndrome or Mar-
necrosis, rheumatoid arthritis, fat pad atrophy, stress fan syndrome, may result in overload of one or more
fracture, neuromuscular dysfunction, and neuromas of the lesser metatarsophalangeal joints.14
are all common causes of lesser metatarsalgia.2, 12 The senior author has also witnessed numerous
Only after a detailed history and physical examina- cases of predislocation syndrome in patients who
tion and a radiographic evaluation can a primary di- had undergone surgical correction of hallux abducto
agnosis of predislocation syndrome be made (Table valgus deformity in which subtle, but discernible, ele-
1). Selective laboratory testing may be needed for vation of the first metatarsal head occurred (Fig. 7).
completeness. It is important to listen carefully to the In other cases, the initial presenting complaint was
patient’s subjective complaints, as these will often sub–metatarsal 2 discomfort attributed to an asymp-
provide sufficient information for the clinician to sus- tomatic hallux abducto valgus deformity. While
pect this entity as the cause of the patient’s pain, surgery for the hallux abducto valgus was unevent-
even prior to a physical examination of the area. ful, progressive dislocation of the second toe ensued.
Patient frustration in such cases is very high.
Pathomechanics Coughlin1 has identified two distinct populations
at risk for developing instability of the second meta-
The key developmental factor for idiopathic subluxa- tarsophalangeal joint: The first group consisted main-
tion/dislocation of the lesser toe is progressive in- ly of sedentary women aged 50 to 70 years, with an
flammation about the metatarsophalangeal joint with average age of 60 years. The high proportion of elder-
subsequent attenuation and rupture of the plantar ly women in this group was attributed to the preva-
plate and collateral ligaments.2, 5 Thus any structural lent use of high-heeled shoes in this population. The
or biomechanical deformity that increases loading second group consisted predominantly of athletic
forces within the forefoot and results in inflamma- men aged 25 to 64 years, with an average age of 50
tion of the plantar plate can predispose an individual years. A majority of this group were found to have a
to progressive weakening of the periarticular struc- long second metatarsal. Coughlin hypothesized that
tures and resultant joint instability. in the second group repetitive physical activity re-
A long second metatarsal is often cited as a com- sulted in inflammation about the second metatarso-
mon finding in individuals with instability of the sec- phalangeal joint that eventually resulted in a weaken-
ond metatarsophalangeal joint.8, 11 An elongated sec- ing of the periarticular structures that stabilize the

186 April 2002 • Vol 92 • No 4 • Journal of the American Podiatric Medical Association
Table 1. Differentiating Predislocation Syndrome from Common Lesser Metatarsophalangeal Joint Problems
Predislocation Syndrome Other Common Problems
Stress Fracture
Pain localized to plantar metatarsophalangeal joint Pain at metatarsal shaft dorsally
Negative tuning fork test Tuning fork test elicits pain
X-rays nondiagnostic in early stages Conventional x-rays eventually diagnostic
Bone scan uptake at metatarsophalangeal joint Bone scan uptake in metatarsal shaft

Hammer Toe
Minor trauma or inciting event may be recalled No inciting event or trauma
Onset distinctly acute, subacute, or chronic Onset insidious over years
No digital contracture until late stage Contracture at proximal or distal interphalangeal joint
preceded pain
Positive vertical drawer test Negative vertical drawer test
Usually no lesions; occasional irritation of the dorsal Lesions frequently present: heloma durum at proximal
proximal interphalangeal joint interphalangeal joint and/or sub–metatarsal head tyloma

Neuroma
Plantar to metatarsophalangeal joint, often the second Classic third interspace
interspace
Neuritic symptoms absent Neuritic symptoms: tingling, cramping, burning, shooting pain
Moulder sign absent Positive Moulder sign: palpable click of soft-tissue mass
Subtle or obvious malalignment or malposition of the Toes usually have normal alignment and position
involved toes
May demonstrate significant inflammation of No edema present
metatarsophalangeal joint and toe, especially plantarly

A B

Figure 6. Anteroposterior (A) and lateral (B) radiographs of a patient who developed sudden dislocation of the
second metatarsophalangeal joint following a period of severe pain and inflammation of the plantar aspect of the
joint. Note the slightly elongated second metatarsal segment as a possible contributing factor and the presence of
a hallux abducto valgus deformity.

Journal of the American Podiatric Medical Association • Vol 92 • No 4 • April 2002 187
A B

Figure 7. Anteroposterior (A) and lateral (B) radiographs demonstrating dorsal dislocation of the second toe following
a distal metaphyseal osteotomy for correction of a hallux abducto valgus deformity. The patient’s initial complaint was
pain beneath the second metatarsal segment. An arthroplasty procedure was performed. Owing to persistent symp-
toms, the patient received multiple injections of steroid and local anesthetic, which resulted in acceleration of the de-
formity and a steroid-induced avascular necrosis of the third proximal phalangeal base. Note the development of a
stress fracture of the second metatarsal and the shortening and elevation of the first metatarsal segment.

toe. The authors believe that both structural alter- also serves as an insertion for both the interosseous
ation and biomechanical dysfunction of the first ray and the lumbrical tendons. Inferiorly, the plantar
can contribute to the development of this entity at plate has a smooth grooved surface for the passage
the second metatarsophalangeal joint. The condition of the flexor tendons.
can, however, occur in the absence of either of these An anatomical dissection by Johnston et al 15
predisposing factors. demonstrated that the plantar plate of the foot is sim-
ilar in both form and function to the volar plate of
Role of the Plantar Plate in the hand. However, those authors noted that the
Predislocation Syndrome plantar plate is larger and thicker than the volar plate
and has a weaker proximal attachment. Deland et al16
Stability of the lesser metatarsophalangeal joint is also performed an in-depth anatomical dissection of
derived from the plantar plate, the collateral liga- the plantar plate and outlined several possible func-
ments, and the intrinsic and extrinsic foot muscula- tions of the structure. First, the plantar plate merges
ture.15-17 Static stabilization of the metatarsophalan- with the plantar fascia distally and is an integral part
geal joint is derived primarily from the plantar plate of the windlass mechanism. Second, its fibrocarti-
and collateral ligaments. The plantar plate is de- laginous structure helps to absorb compressive loads
scribed as a fibrocartilaginous thickening of the acting on the metatarsal head. Finally, its central posi-
metatarsophalangeal joint capsule plantarly that is tion and multiple attachments act to stabilize the less-
firmly attached to the base of the proximal phalanx er metatarsophalangeal joint in the plantar direction.
but only loosely attached to the metatarsal head.15 It Through mechanical testing, Bhatia et al17 found
is composed of type I collagen that is histologically the plantar plate to be the main stabilizing force of
identical to the collagen present in the meniscus of the metatarsophalangeal joint. The study concluded
the knee.16 The plantar plate acts as the major distal that the metatarsophalangeal joint collateral liga-
attachment of the plantar fascia and has attachments ments, which have an insertion into the plantar plate,
to the deep transverse metatarsal ligament and are the second most powerful structures that stabi-
metatarsophalangeal joint collateral ligaments. It lize the metatarsophalangeal joint. Deland and Sung18

188 April 2002 • Vol 92 • No 4 • Journal of the American Podiatric Medical Association
performed an anatomical dissection of a medial cross- metatarsophalangeal joint in the plantarflexed posi-
over second toe in a cadaveric specimen in an attempt tion during weightbearing. However, like the interos-
to fully describe the underlying pathologic anatomy seous muscles, they too lose their ability to provide
found in the condition. The specimen exhibited rup- flexion of the metatarsophalangeal joint when the
ture of the lateral collateral ligaments, shortening of proximal phalanx is in a subluxated (dorsal) posi-
the medial collateral ligaments, attenuation of the tion.3 Thus, in chronic subluxation of the metatarso-
plantar plate, and dorsal medial displacement of both phalangeal joint, the long and short flexor, interosse-
the flexor tendons and the plantar plate. ous, and lumbrical muscles provide no dynamic
While static stabilization of the metatarsophalan- restraint to dorsal dislocation of the toe.
geal joint is primarily the function of the plantar The second toe is anatomically unique, having in-
plate, dynamic stabilization is provided by the extrin- sertions of two dorsal interosseous tendons but no
sic and intrinsic musculature of the foot. It should be plantar interosseous tendon. The lumbrical tendon
noted, however, that the ability of the intrinsic and has a medial insertion and provides a slight medial
extrinsic foot musculature to stabilize this joint is de- pull in the second toe. With chronic inflammation
pendent largely on the integrity of the plantar plate. about the metatarsophalangeal joint, progressive
When the metatarsophalangeal joint is in a flexed or dorsal subluxation of the second toe ensues as the
neutral position, the extensor digitorum longus and plantar plate and collateral ligaments fail in static
extensor digitorum brevis tendons act to dorsiflex stabilization of the joint. As a result of the dorsal po-
the metatarsophalangeal joint through their insertion sition of the toe, the normal axes of the intrinsic mus-
into the extensor hood apparatus.19 However, with cles are lost, rendering them incapable of dynamical-
rupture of the plantar plate, the proximal phalanx as- ly stabilizing the joint. The extensor digitorum longus
sumes a subluxated (dorsal) position, and the exten- and lumbrical muscles subsequently become a de-
sor tendons are mechanically unable to extend the forming force, contributing to dorsal medial disloca-
proximal and distal interphalangeal joints. When the tion of the toe.
metatarsophalangeal joint is maintained in a sublux- Further explanation for a dorsal medial disloca-
ated (dorsal) position for a prolonged period, the ex- tion following failure of the plantar plate has been
tensor digitorum longus becomes a deforming force provided by Hatch and Burns,23 who performed an
at the metatarsophalangeal joint and results in pro- anatomical dissection of seven cadaveric specimens
gressive dorsal subluxation and frank dislocation of exhibiting a crossover second-toe deformity. The au-
the toe.2 thors found that four of the seven specimens demon-
The flexor digitorum longus and brevis tendons strated an accessory medial slip of the extensor digi-
provide flexion at the proximal and distal interpha- torum brevis tendon to the second toe and believed
langeal joints, but are relatively inefficient flexors of that this accessory tendon was a contributing factor
the metatarsophalangeal joint.3 In a weightbearing sit- in the etiology of the crossover second-toe deformi-
uation, these muscles act to stabilize the toes against ty. The current authors have not found this anatomi-
the ground and, in doing so, may cause slight dorsi- cal variant in several cases of predislocation syn-
flexion of the metatarsophalangeal joint.19 The flexor drome that have undergone surgical correction.
tendons are therefore unable to aid in stabilization of Anatomically, it is easy to understand why dorsal
the dorsally subluxated toe. medial dislocations are most common at the second
Under normal circumstances, the interosseous metatarsophalangeal joint. However, dorsal medial
muscles provide most of the flexion that occurs at dislocations are also more common than dorsal later-
the lesser metatarsophalangeal joints.20 However, al dislocations at the other lesser metatarsophalange-
with dorsal subluxation of the metatarsophalangeal al joints. The rationale behind this observation is not
joint, the interosseous tendons migrate dorsal to the easily understood, as the other lesser metatarsophalan-
axis of the metatarsophalangeal joint and subse- geal joints have both dorsal and plantar interosseous
quently lose their mechanical ability to provide flex- tendon insertions that directly oppose one another in
ion of the joint.21 The lumbrical muscles also play an the transverse plane. Subluxation and dislocation of
important role in the dynamic stability of the lesser the lesser toe at the level of the lesser metatarsopha-
metatarsophalangeal joint. These muscles arise from langeal joint, not attributable to the normal progres-
the long flexor tendons and insert into the medial as- sion of the classic hammer toe deformity, are still
pect of the extensor hood. According to an anatomi- somewhat of an enigma and continue to challenge
cal and functional study performed by Jarrett et al,22 physicians who treat foot and ankle problems. The
the interosseous muscles stabilize the interphalange- senior author has diagnosed and treated predisloca-
al joints in the extended position and maintain the tion syndrome involving one or more of the following

Journal of the American Podiatric Medical Association • Vol 92 • No 4 • April 2002 189
digits: second, third, and fourth (Fig. 8). While dorsal dysfunction. Weightbearing lateral or oblique radio-
subluxation is seen in all of them, the medial or later- graphs clearly indicate dorsal subluxation of the
al deviation is a peculiar finding not well understood. proximal phalanx upon the metatarsal head as the
The authors suggest that an underlying structural condition progresses. Weightbearing anteroposterior
metatarsus adductus deformity may influence the radiographs will demonstrate dorsal as well as trans-
type of transverse plane deformity that develops. verse deviation of the affected toe. The normal clear
space of the lesser metatarsophalangeal joint is 2 to 3
Diagnostic Modalities mm, and the joint surfaces should be congruent.2
When dorsal subluxation or dislocation is present,
Several imaging modalities are available to assess in- the clear space is obliterated as the concave base of
stability of the lesser metatarsophalangeal joint, al- the proximal phalanx migrates dorsally over the con-
though rarely is extensive or expensive imaging re- vex metatarsal head. Incongruency of the joint space
quired. When there is clinical evidence of subluxation is noticed as the digit deviates in the sagittal and
or dislocation, standard radiography is the most transverse planes. The anteroposterior radiograph
practical diagnostic modality and is essential in rul- will also demonstrate any abnormal metatarsal
ing out local or systemic pathologies that contribute length patterns that might contribute to the develop-
to lesser metatarsophalangeal joint instability and ment and progression of this deformity. Cortical hy-

A B

Figure 8. Predislocation syndrome of the third metatar-


C sophalangeal joint. A, Initial radiographic findings were
unremarkable; B, a subsequent radiograph demon-
strated a complete dislocation of the third metatarso-
phalangeal joint (arrow); C, note the swelling of the
digit and corresponding metatarsophalangeal joint.

190 April 2002 • Vol 92 • No 4 • Journal of the American Podiatric Medical Association
pertrophy of the affected metatarsal shaft may be sophalangeal joints obtained parallel to the second
present and is believed to be indicative of abnormal through fifth metatarsal heads. The oblique axial se-
forefoot loading.1 However, the authors have not quence is aligned to the dorsal surface of the foot
found this to be a consistent finding. Finally, a radio- with a lateral down-sloping orientation. To obtain the
graphic vertical stress test may also be used to sup- scan, merely instruct the technologists to perform a
port the clinical findings of sagittal plane instability. STIR sequence in the plane defined by laying their
More advanced imaging modalities may be war- hand on the dorsum of the forefoot region. This com-
ranted when there is no clear clinical or radiographic bination of pulse sequences provides a simple yet de-
evidence of instability. An arthrogram with 0.5 to 1 tailed evaluation of the metatarsophalangeal joints
mL of iodinated contrast material injected dorsally and their associated capsuloligamentous complexes,
into the metatarsophalangeal joint under fluoroscopy including the plantar plate.
may be used to evaluate the integrity of the metatar- The plantar plate and its associated structures are
sophalangeal joint capsule.24 Rupture of the metatar- best visualized on T1 and gradient echo image se-
sophalangeal joint capsule is demonstrated by opaci- quences.25 In T1-weighted images, the plantar plate is
fication of the flexor tendon sheath while leakage of seen as a smooth, low-signal structure running be-
the contrast material outside the joint capsule into neath the metatarsal head and proximal phalangeal
the intermetatarsal bursa is indicative of collateral base. However, the plantar plate may be difficult to
ligament rupture.25 distinguish from the thicker, underlying flexor ten-
Although not typically used, a nuclear medicine dons on T1-weighted images. For this reason, it is
three-phase bone scan will show increased tracer up- helpful to obtain gradient echo images in which the
take at the metatarsophalangeal joint during all three plantar plate is slightly hyperintense relative to the
phases.6, 11 The first two phases are decidedly more flexor tendons.
intense than the delayed phase. This pattern strongly The MRI findings of plantar plate rupture have
suggests a local inflammatory process, rather than an been well documented by Yao et al.25, 26 A rupture of
osseous problem. Additionally, a conventional bone the plantar plate will demonstrate increased signal
scan may be helpful in identifying a stress fracture intensity within the plate with a loss of continuity.26
not identified on conventional radiographs. The pat- The area of rupture will be isointense with synovium
tern of uptake is different from that seen in patients and joint fluid and is typically located adjacent to the
with predislocation syndrome. A stress fracture metatarsal head (Fig. 9). Synovitis of the flexor ten-
would show increased uptake in all phases of the don sheath and distention of the metatarsophalange-
bone scan with the greatest uptake in the delayed al joint capsule are common associated findings in
phase (3 to 6 hours), which demonstrates a linear plantar plate rupture and, if present, are readily iden-
pattern within the shaft of the affected metatarsal. In tified on MRI.
contrast, the pattern of uptake in patients with pre-
dislocation syndrome is localized to the metatarso-
phalangeal joint area.
Magnetic resonance imaging (MRI) of the mani-
festations of predislocation syndrome can be demon-
strated on high-field, as well as low-field, magnets.
This includes imaging on high-field superconducting
extremity images. However, adequate MRI visualiza-
tion demands a tailored approach. The necessary
pulse sequences include sagittal longitudinal relax-
ation time (sagittal T1), gradient echo transverse re-
laxation time (gradient echo T2), and short tau inver-
sion recovery (STIR) images and coronal T1 and
STIR images, as well as an oblique axial STIR se-
quence for imaging of the metatarsals and intrinsic
muscles of the forefoot.
The selection of pulse sequences is very impor-
tant. Sagittal images are obtained perpendicular to
the long axis of the metatarsals. Coronal images, Figure 9. Typical MRI findings of a patient with pre-
however, require alignment of the first and second dislocation syndrome involving the second and third
metatarsal heads, with imaging of the lesser metatar- metatarsophalangeal joints.

Journal of the American Podiatric Medical Association • Vol 92 • No 4 • April 2002 191
Conservative Treatment pain-free ambulation are achieved; 2) a prolonged
course of taping may result in edema and possible ul-
The goal of conservative treatment for predislocation ceration of the digit.11 In short, it is simply not practi-
syndrome is to prevent progression of the deformity cal. Taping is, however, useful in helping to establish
and alleviate symptoms. Successful treatment is most the diagnosis of predislocation syndrome at the ini-
often accomplished with a combination of several tial office visit, provided that the metatarsophalange-
treatment modalities. Nonsteroidal anti-inflammatory al joint has undergone only deviation or subluxation,
drugs and corticosteroids have been a mainstay for not dislocation. Taping the toe in the correct position
the management of the pain and inflammation associ- often provides immediate relief of the patient’s symp-
ated with this disorder. While a tapered course of toms on weightbearing, confirming the diagnosis of
oral corticosteroids has been found to be safe and ef- predislocation syndrome.
fective, the same cannot be said about repeated Splinting may also be used to maintain proper ana-
intra-articular steroid injections. Dislocation of the tomical alignment of the digit. A metatarsal sling pad
toe following repeated intra-articular administration stabilizes the proximal phalanx in a plantar direction
of corticosteroids has been documented in the litera- at the metatarsophalangeal joint level by augmenting
ture.27 The senior author has witnessed this in sever- the function of the plantar plate and preventing hy-
al cases where dislocation of the metatarsophalange- perextension or dorsal migration of the proximal
al joint was seemingly accentuated by the repeated phalanx. Accommodative padding may also be used
use of intra-articular steroid injection. to alleviate symptoms at the affected metatarsopha-
The adverse effects of corticosteroid injection di- langeal joint. A longitudinal metatarsal pad with a
rectly into periarticular structures have been record- lesser metatarsophalangeal joint cutout can be used
ed in the literature. In one study, a single injection of to effectively decrease the weightbearing stress on
methylprednisone (4 mg) deposited directly into liga- the metatarsal head. A metatarsal pad applied to the
mentous tissues destroyed fibrocytes and decreased foot or used in the shoe is also helpful, but far less ef-
the tensile strength of the tissues for as long as 1 fective than the metatarsal sling pad. It will offload
year.28 The physician must realize that such an injec- the metatarsal but is insufficient in providing direct
tion may weaken the already compromised plantar splinting of the toe in a plantar direction and fails to
plate. However, if intra-articular steroids are deemed prevent further dorsal migration and dislocation.
necessary, the smallest effective dose of a phosphate For patients who have received insufficient bene-
salt should be used. Generally accepted guidelines fit from taping and padding therapy or are not con-
recommend that any given joint be injected a maxi- sidered candidates for surgery, shoe modifications
mum of once every 1 to 3 months and no more than may relieve symptoms.3-5, 30 An extra-depth shoe to
two to three times within a 12-month period.29 A accommodate the digital deformity combined with a
number of factors will influence the frequency of in- rocker-bottom sole with steel shank to decrease
jections; these include the type of steroid, the quanti- propulsive forces on the affected metatarsophalange-
ty of the steroid, and the clinical entity being treated. al joint may be effective. A viable alternative option
The authors strongly discourage the indiscriminate, for treating this condition is an extra-depth shoe with
repeated use of intra-articular steroids for predislo- a metatarsal bar added to the sole.
cation syndrome. In the authors’ opinion, the use of The authors’ typical regimen for the conservative
acetate steroids is rarely, if ever, indicated for the treatment of predislocation syndrome consists of a
treatment of predislocation syndrome. If intra-articu- combination of oral steroids, nonsteroidal anti-in-
lar steroids are employed, splinting of the toe to flammatory drugs, and taping or padding. In cases of
maintain position and prevent progressive disloca- significant symptomatology associated with moder-
tion is strongly recommended. ate to severe inflammation, oral steroids have been
In addition to pharmacologic therapy, offloading very effective. The authors typically use a tapering
the metatarsophalangeal joint and supporting the toe dose (from 60 to 10 mg in 10-mg increments) of pred-
in a proper position are essential to successful treat- nisone, administered in twenty-one 10-mg tablets;
ment. Taping of the digit into a slightly overcorrected this regimen is extremely effective clinically and
rectus position can prevent pain and progression of quite cost-effective. This is followed by a course of
the deformity. It is believed that this course of treat- nonsteroidal anti-inflammatory drug therapy in con-
ment stabilizes the digit enough to allow fibrosis of junction with splinting. Additionally, the authors
the periarticular structures to occur. However, there often employ physical therapy and aggressive manip-
are two main drawbacks to taping of the digit: 1) ulation to stretch out the dorsally contracted tissues
Taping may take several months before stability and and decrease inflammation.

192 April 2002 • Vol 92 • No 4 • Journal of the American Podiatric Medical Association
Crossover taping of the toe in proper position usu-
ally provides immediate relief of symptoms in the of-
fice (Fig. 10). The patient is permitted to be weight-
bearing and ambulate. If the patient is comfortable
with this taping, the diagnosis is confirmed. A meta-
tarsal sling pad fabricated from 1/8-inch adhesive
foam, moleskin, and tube gauze has been very effec-
tive for this condition and is worn for weeks to
months until the toe stabilizes or is corrected surgi-
cally (Fig. 11). Felt should not be used to fabricate
these pads because its bulkiness aggravates plantar
discomfort and usually makes it difficult for the pads
to fit into normal shoes. Foam does not seem to pose
this problem. Prefabricated pads are commercially
available for single-, double-, or triple-digit control. Figure 11. Metatarsal pad to maintain digital position
when metatarsophalangeal joint dislocation has not
Where the toe deviates in more than one plane, it can yet occurred is shown along with the materials used
be splinted to the adjacent toe with a multidigit pad to fabricate it. The materials are 1/8-inch foam, mole-
by securing the digital straps to one another. skin, and tube gauze. Prefabricated pads are readily
Varying degrees of success have been reported available.
with conservative treatment of lesser metatarsopha-
langeal joint instability. Mizel and Michelson4 treated
patients with a combination of a single intracapsular matory medications and extra-depth shoes with a
steroid injection and a laced shoe with a steel shank metatarsal support. Only one patient responded posi-
added to the outer sole. They reported a 70% success tively to this course of treatment; the remainder pro-
rate with this treatment regimen for a 75-month fol- gressed to surgical correction of the deformity. Cough-
low-up period. Trepman and Yeo30 also used a single lin1 treated a group of ten athletes with taping for a
intra-articular steriod injection and a rocker-sole period of 3 to 6 months combined with a reduction in
shoe modification to treat lesser metatarsophalange- activity level. Three of the ten patients responded
al joint synovitis. That study included 15 joints in 13 well to this treatment, and six of the remaining seven
patients with no known history of rheumatologic patients progressed to surgery.
conditions. At follow-up evaluation, an average of 18 One weakness of these studies is the lack of cor-
months later, nine joints (60%) were asymptomatic, relation of the stage of the deformity with the clinical
five joints (33%) were improved, and one joint (7%) outcome of conservative treatment. In the Coughlin
had progressed to surgery. Mann and Mizel9 treated a study, all of the patients already had a positive verti-
group of seven patients with nonsteroidal anti-inflam- cal stress test result, and eight of the ten patients had
either transverse or sagittal plane deviation at the
time of initial presentation. It would seem likely that
the number of patients requiring surgical correction
would be lower if the disorder was identified and
treated prior to obvious or frank subluxation. None-
theless, the literature bears out that conservative treat-
ment will only prevent progression of the deformity
and does not permit realignment of the digit once sig-
nificant malalignment or instability has occurred.
In the authors’ experience, significant alleviation
of symptoms can be quickly achieved with conserva-
tive treatment; however, the complete resolution of
all symptoms is less predictable. Digital splinting and
pharmacologic management are used for several
months in most cases. Early discontinuation of
splinting is likely to result in progressive subluxation
and dislocation of the metatarsophalangeal joint. As
Figure 10. Crossover tape used as a diagnostic tool this occurs, secondary contracture of the extensor
to evaluate and assess predislocation syndrome. digitorum longus tendon develops along with dorsal

Journal of the American Podiatric Medical Association • Vol 92 • No 4 • April 2002 193
contracture of the metatarsophalangeal joint cap- Numerous authors have advocated other soft-tis-
sule. Once flexibility and reducibility of the deformi- sue procedures to correct subluxation or dislocation
ty are lost, surgical intervention becomes the logical of the lesser metatarsophalangeal joint. Ruch31 has
next treatment. described two different procedures in which the ex-
tensor digitorum brevis tendon is transferred to re-
Surgical Management establish muscle-tendon balance at the lesser metatar-
sophalangeal joint. Girdlestone35 is often credited as
In light of the frequent failure of conservative treat- being one of the first to advocate reduction of the
ment in dealing with subluxation/dislocation of the dislocated metatarsophalangeal joint by splitting the
lesser metatarsophalangeal joint, numerous surgical long flexor tendon and transferring it to the dorsum
procedures have been advocated to correct the de- of the proximal phalanx. Later, Kuwada and Dock-
formity.2, 3, 9, 31-37 While there is great divergence in the ery38 popularized the transfer of the long flexor ten-
strategies employed to correct the deformity, the don to the dorsal aspect of the proximal phalanx
basic goal of all of the procedures is release of peri- through a drill hole in the neck of the proximal pha-
articular contractures and decompression of the lanx. A recent modification of that procedure places
metatarsophalangeal joint to reestablish alignment of the drill hole in the base of the proximal phalanx.39
the digit. Surgical treatment should focus on restora- The benefits of this modification include better re-
tion of plantar plate function and release of the dor- duction of the dislocation owing to a more proximal
sally contracted structures. The design and extent of pull of the flexor tendon and the preservation of the
surgery will vary with the severity of the deformity ability to perform either a proximal interphalangeal
and the amount of time that has elapsed since the ini- joint arthroplasty or arthrodesis. The main drawback
tial diagnosis. of all flexor-to-extensor transfer procedures is the
Several authors have realized the importance of decreased dorsal range of motion at the metatarso-
the plantar plate and collateral ligaments in stabiliz- phalangeal joint that occurs postoperatively. This,
ing the metatarsophalangeal joint and have developed however, may also be construed as the main benefit
procedures to correct the deformity at this level. of the procedure, as the goal of treatment is to pre-
Some surgeons have chosen to correct this deformity vent dorsal migration and dislocation of the toe.
with direct primary repair of the plantar plate. A Some authors have used a combination of soft-tis-
plantar incisional approach is employed with direct sue and osseous procedures to correct the deformity.
visualization of the plantar plate. The attenuated or Daly and Johnson40 advocated resection of the bases
ruptured site is identified, and direct primary surgical of the second and third proximal phalanges with
repair is performed with or without concomitant subtotal webbing. They believed that subtotal web-
arthroplasty or arthrodesis of the proximal interpha- bing promoted digital stability that was lost as a re-
langeal joint and release of the dorsal contracted soft sult of the resection of the bases of the proximal pha-
tissue. langes. Coughlin and Mann11 have recommended
Ford et al33 have also advocated primary repair of relocation of the metatarsophalangeal joint by partial
the plantar plate to correct this deformity. Using a metatarsal head resection and release of the con-
biomechanical testing apparatus, they showed that tracted periarticular structures in severe cases of dis-
primary plantar plate repair is as effective as a flexor- location of the metatarsophalangeal joint. Cracchiolo
to-extensor tendon transfer in stabilizing the metatar- et al41 reported on 28 cases in which silicone implant
sophalangeal joint. However, the study concluded that arthroplasty was used to correct second metatarso-
performing both procedures together was more effec- phalangeal joint dislocation. They reported good re-
tive than performing either procedure individually. sults in 88% of patients presenting with multiple fore-
Deland et al32 noted the importance of the collat- foot deformities. The authors cited transfer lesions
eral ligaments in stabilizing the metatarsophalangeal as the most frequent complication of the procedure.
joint and proposed grafting of a partial thickness of Other osseous procedures that have been used in-
the interosseous tendon to reinforce the weakened clude oblique osteotomies of the head and neck of
collateral ligament. Although Deland et al only re- the metatarsal, with or without fixation. In a compar-
ported on the procedure as performed in cadaveric ison of the results of the Weil and Helal osteotomies
specimens, they stated that the benefits of this proce- in correcting dislocation of the lesser metatarsopha-
dure would include a good range of motion postoper- langeal joint, Trnka et al36 concluded that the Weil os-
atively, compared with a flexor-to-extensor tendon teotomy was far superior in maintaining the metatarso-
transfer, and the ability to control the transverse phalangeal joint in a corrected position and resulted
plane alignment of the digits. in fewer malunions. They suggest that this is because

194 April 2002 • Vol 92 • No 4 • Journal of the American Podiatric Medical Association
the Weil procedure uses stable internal fixation while sophalangeal joint for 5 to 7 weeks, allowing for soft-
the Helal procedure allows the metatarsal head to tissue adaptation and stabilization of the metatarso-
“float” or “seek its own level” after osteotomy. Van- phalangeal joint.
deputte et al42 studied the use of the Weil osteotomy If instability of the metatarsophalangeal joint per-
for both intractable plantar keratosis and chronically sists following soft-tissue release, a flexor tendon
dislocated metatarsophalangeal joints. Of the 59 transfer employing the flexor digitorum longus (more
metatarsophalangeal joints involved in the study, 33 commonly) or the flexor digitorum brevis is strongly
were chronically dislocated. At follow-up at an aver- recommended (Fig. 14). Arthrodesis of the proximal
age of 30 months postoperatively, patients rated the interphalangeal joint alone, although helping to cre-
results as excellent or good in 32 of 37 feet (86%). ate a rigid beam effect of the toe, may not be suffi-
Two symptomatic transfer lesions occurred, and re- cient to prevent dorsal subluxation/dislocation of the
current dislocations occurred in 5 joints. Vandeputte phalangeal base.
et al concluded that the Weil osteotomy is simple and Several types of flexor digitorum longus tendon
efficient for the reduction of chronically dislocated transfers have been well described in the orthopedic
lesser metatarsophalangeal joints. and podiatric literature. Several modifications the
The current authors do not have experience with current authors employ are worthy of discussion. Re-
direct surgical repair of the plantar plate, but have in- gardless of the technique of actual transfer, the au-
stead chosen to approach this problem by means of thors routinely harvest the tendon at the level of the
various soft-tissue and osseous procedures. A dorsal proximal interphalangeal joint that is undergoing
incisional approach is employed. An arthrodesis of arthrodesis. The plantar tissue is incised at the base
the proximal interphalangeal joint is typically per- of the middle phalanx from the dorsal incision. A
formed. The dorsal soft-tissue release is done in a se- secondary plantar incision, although commonly
quential manner until complete release and reloca- used, is not necessary. The split flexor digitorum bre-
tion of the toe at the metatarsophalangeal joint are vis tendon is retracted medially and laterally, and the
achieved. An extensor hood release/recession is fol- flexor digitorum longus tendon identified. The ten-
lowed by a dorsal/medial/lateral metatarsophalangeal don is retracted into the wound, tagged, and tran-
joint capsule release. It is not uncommon for a signifi- sected at the level of the middle phalangeal base.
cant amount of synovitis to be present within the The tendon is transferred by means of one of sev-
joint. Debridement of this inflammatory tissue is per- eral techniques. The standard technique is to split the
formed as needed (Figs. 12 and 13). The toe is stabi- tendon and then transfer each of the two halves
lized with a Kirschner wire driven across the metatar- around the proximal phalanx and suture them to-
gether dorsally. Another technique is to suture the
end of the tendon distally, split it proximally, and
then simply pass it over the proximal phalangeal neck
area, creating a “check-rein” type of configuration.

Figure 12. Intraoperative appearance of the second


metatarsophalangeal joint following complete reloca-
tion of the digit. Note the extensive synovitis within the
joint capsule. In addition to soft-tissue release and syn-
ovectomy, an arthrodesis of the proximal interphalan- Figure 13. Clinical photograph 6 months following
geal joint was performed. surgery of the patient shown in Figures 2 and 12.

Journal of the American Podiatric Medical Association • Vol 92 • No 4 • April 2002 195
Figure 14. Traditional technique of flexor digitorum Figure 15. Modified technique of flexor digitorum
longus tendon transfer. Tendon has been harvested longus tendon transfer through a drill hole at the base
at the proximal interphalangeal joint level, split, and of the proximal phalanx. The tendon is sutured to the
brought around the diaphysis of the proximal phalanx. dorsal periosteum and extensor digitorum longus ten-
A simultaneous arthrodesis of the proximal interpha- don. Simultaneous fusion of the proximal interpha-
langeal joint will further facilitate plantar stabilization langeal joint can be performed. As shown here, the
of the digit via the flexor digitorum longus and flexor Kirschner wire is driven through the tendon and across
digitorum brevis. the metatarsophalangeal joint if desired.

The authors’ current recommendation is to pass at the metatarsophalangeal joint that is available for
the tendon from plantar to dorsal through a drill hole normal walking.
at the proximal metaphyseal–diaphyseal junction In more resistant and recalcitrant cases, an osteot-
and secure it to the dorsal periosteal tissues (Figs. 15 omy of the metatarsal for shortening or angular cor-
and 16). When the digit is stabilized with a Kirschner rection will be necessary. A variety of techniques can
wire, the wire is driven through the transferred flex- be performed depending on the need for, and extent
or digitorum longus tendon and across the lesser of, shortening versus angular correction of the meta-
metatarsophalangeal joint. It is important to impart a tarsal segment. A through-and-through oblique oste-
physiologic tension to the transferred tendon, as ex- otomy from dorsal distal to plantar proximal allows
cessive tightness can limit the amount of dorsiflexion for shortening as well as rotation if needed. Small

A B

Figure 16. Preoperative (A) and postoperative (B) clinical appearance of the patient in Figure 15 following correc-
tion of the hallux abducto valgus deformity as well as arthrodesis and flexor digitorum longus tendon transfer of
the second digit.

196 April 2002 • Vol 92 • No 4 • Journal of the American Podiatric Medical Association
cortical screws are used to fixate the osteotomy. A lesser-toe metatarsophalangeal joint subluxation/dis-
shortening Z-osteotomy can also be employed. If location. The treatment of predislocation syndrome
only angular change of the metatarsal is needed in is dependent on the stage of the deformity. In early
the transverse plane, a small wedge osteotomy at the predislocation syndrome, conservative treatment
metatarsal base can be very effective; a rotational os- with anti-inflammatory medication, shoe modifica-
teotomy can also be employed. The authors generally tion, padding, and taping may be successful in pre-
reserve metatarsal osteotomy for the more severe venting progression of the deformity. In long-stand-
and fixed deformity where obvious structural defor- ing deformity of the lesser metatarsophalangeal joint,
mity (abnormal length or position) is present (Figs. surgical intervention is warranted when painful fixed
17 and 18). subluxation or dislocation is present and conserva-
tive treatment modalities have failed or when pro-
Conclusion longed conservative care is not desirable.

Instability of the lesser metatarsophalangeal joint is Acknowledgment. The Podiatry Institute and At-
becoming well recognized and accepted in podiatric lanta Slide Art Productions (Tucker, Georgia) for their
and orthopedic surgery. It is often the sequela of a assistance in the preparation of the photographs;
chronic inflammation resulting from increased and Michael Dujela, DPM, for his assistance with the
abnormal weightbearing stresses about the lesser
preparation of the manuscript.
metatarsophalangeal joint. The end result of this de-
rangement is attenuation or rupture of the plantar
plate, capsule, or collateral ligaments with subse- References
quent subluxation or dislocation of the digit. 1. C OUGHLIN MJ: Second metatarsophalangeal joint insta-
This clinical entity has been underrecognized as a bility in the athlete. Foot Ankle 14: 309, 1993.
cause of lesser metatarsalgia. Although the second 2. C OUGHLIN MJ: Subluxation and dislocation of second
digit is most commonly and most profoundly affect- metatarsophalangeal joint. Orthop Clin North Am 20:
ed (ie, crossover second-toe deformity), the adjacent 535, 1989.
3. F ORTIN PT, M YERSON MS: Second metatarsophalangeal
third and fourth digits can also be affected. Addition-
joint instability. Foot Ankle 16: 306, 1995.
ally, one or more lesser digits can be simultaneously 4. MIZEL MS, MICHELSON JD: Nonsurgical treatment of mono-
involved. articular nontraumatic synovitis of the second metatar-
The clinical signs and symptoms present on exam- sophalangeal joint. Foot Ankle 18: 424, 1997.
ination can help to stage and chart the evolution of 5. T HOMPSON FM, H AMILTON WG: Problems of the second

A B

Figure 17. Preoperative (A) and postoperative (B) an-


teroposterior radiographs demonstrating a shortening
Z-osteotomy of the second metatarsal and oblique ro-
tational osteotomy of the third metatarsal in a patient
with an abnormal metatarsal parabola as a significant
contributing factor to the development of digital disloca-
tions. The lines represent the metatarsal break angle.

Journal of the American Podiatric Medical Association • Vol 92 • No 4 • April 2002 197
A B

Figure 18. Preoperative clinical appearance of the patient shown in Figure 17. Note the complete dislocation of the
toes with multiplanar deviation (A) and severe plantar plate inflammation (B). The patient had complete resolution of
all symptoms postoperatively.

metatarsophalangeal joint. Orthopedics 10: 83, 1987. 18. DELAND JT, SUNG I: The medial crossover toe: a cadav-
6. Y U GV, J UDGE MS: “Predislocation Syndrome of the eric dissection. Foot Ankle 21: 375, 2000.
Lesser Metatarsophalangeal Joint: A Distinct Clinical 19. M C G LAMRY ED: “Lesser Ray Deformities,” in Compre-
Entity,” in Reconstructive Surgery of the Foot and Leg: hensive Textbook of Foot Surgery, 2nd Ed, Vol 1, ed by
Update ’95, ed by CA Camasta, NS Vickers, SR Carter, ED McGlamry, AS Banks, MS Downey, p 321, Williams
p 109, The Podiatry Institute, Tucker, GA, 1995. & Wilkins, Baltimore, 1992.
7. B RANCH HE: Pathologic dislocation of the second toe. 20. S ARRAFIAN SK, T OPOUZIAN LK: Anatomy and physiology
J Bone Joint Surg Am 19: 978, 1937. of the extensor apparatus of the toes. J Bone Joint Surg
8. D U V RIES HL: Dislocation of the toe. JAMA 160: 728, Am 51: 669, 1969.
1956. 21. BOJSEN-MOLLER F: Anatomy of the forefoot, normal and
9. M ANN RA, M IZEL MS: Monoarticular synovitis of the pathologic. Clin Orthop 142: 10, 1979.
metatarsophalangeal joint: a new diagnosis? Foot Ankle 22. J ARRETT BA, M ANZI JA, G REEN DR: Interossei and lum-
6: 18, 1985. bricales muscles of the foot: an anatomical and func-
10. MILLER SJ: “Transverse Plane Metatarsophalangeal Joint tional study. JAPA 70: 1, 1980.
Deformity: Another Etiology and Solution,” in Recon- 23. HATCH DJ, BURNS MJ: An anomalous tendon associated
structive Surgery of the Foot and Leg: Update ’98, ed with crossover second toe deformity. JAPMA 84: 131,
by SJ Miller, KT Mahan, GV Yu, et al, p 124, The Podi- 1994.
atry Institute, Tucker, GA, 1998. 24. KARPMAN RR, MACCOLLUM MS: Arthrography of the meta-
11. C OUGHLIN MJ, M ANN RA: “Lesser Toe Deformities,” in tarsophalangeal joint. Foot Ankle 9: 125, 1988.
Surgery of the Foot and Ankle, 7th Ed, Vol 1, ed by RA 25. YAO L, DO HM, CRACCHIOLO A, ET AL: Plantar plate of the
Mann, MJ Coughlin, p 351, CV Mosby, St Louis, 1999. foot: findings on conventional arthrography and MR
12. F ISHCO WD: “The Sub-Two Syndrome,” in Reconstruc- imaging. AJR Am J Roentgenol 163: 641, 1994.
tive Surgery of the Foot and Leg: Update ’98, ed by SJ 26. Y AO L, C RACCHIOLO A, F ARAHANI K, ET AL : Magnetic res-
Miller, KT Mahan, GV Yu, et al, p 107, The Podiatry In- onance imaging of plantar plate rupture. Foot Ankle
stitute, Tucker, GA, 1998. 17: 33, 1996.
13. R OOT ML, O RIEN WP, W EED JH: Normal and Abnormal 27. R EIS ND, K ARKABI S, Z INMAN C: Metatarsophalangeal
Function of the Foot, p 218, Clinical Biomechanics joint dislocation after local steroid injection. J Bone
Corp, Los Angeles, 1977. Joint Surg Br 71: 864, 1989.
14. COOPER PS: “Disorders and Deformities of the Lesser 28. N OYES FR, N USSBAUM NS, D E L UCAS JL, ET AL : Biome-
Toes,” in Foot and Ankle Disorders, Vol 1, ed by MS chanical and ultrastructural changes in ligaments and
Myerson, p 308, WB Saunders, Philadelphia, 2000. tendons after corticosteroid injection. J Bone Joint
15. J OHNSTON RB, S MITH J, D ANIELS T: The plantar plate of Surg Am 57: 876, 1975.
the lesser toes: an anatomical study in human cadav- 29. G RAY RG, G OTTLIEB NL: Intra-articular steroids: an up-
ers. Foot Ankle 15: 276, 1994. dated assessment. Clin Orthop 177: 235, 1983.
16. D ELAND JT, L EE KT, S OBEL M, ET AL : Anatomy of the 30. TREPMAN E, YEO S: Non-operative treatment of metatar-
plantar plate and and its attachments in the lesser sophalangeal joint synovitis. Foot Ankle 16: 771, 1995.
metatarsophalangeal joint. Foot Ankle 16: 480, 1995. 31. RUCH JA: “Use of the EDB Tendon for Muscle-Tendon
17. B HATIA D, M YERSON MS, C URTIS MJ, ET AL : Anatomical Balance of the Lesser MPJ,” in Reconstructive Surgery
restraints to dislocation of the second metatarsopha- of the Foot and Leg: Update ’95, ed by CA Camasta, NS
langeal joint and assessment of repair technique. J Vickers, RS Carter, p 114, The Podiatry Institute, Tucker,
Bone Joint Surg Am 76: 1371, 1994. GA, 1995.

198 April 2002 • Vol 92 • No 4 • Journal of the American Podiatric Medical Association
32. DELAND JT, SOBEL M, ARNOCZKY SP, ET AL: Collateral lig- 38. KUWADA GT, DOCKERY GL: Modification of the flexor ten-
ament reconstruction of the unstable metatarsophalan- don transfer procedure for the correction of flexible
geal joint: an in vitro study. Foot Ankle 13: 391, 1992. hammertoes. J Foot Ankle Surg 19: 38, 1980.
33. F ORD LA, C OLLINS KB, C HRISTENSEN JC: Stabilization of 39. SCHUBERTH JM, JENSEN R: “Flexor Digitorum Longus
the subluxed second metatarsophalangeal joint: flexor Transfer for Second Metatarsophalangeal Joint Disloca-
tendon transfer versus primary repair of the plantar tion/Subluxation,” in Reconstructive Surgery of the Foot
plate. J Foot Ankle Surg 37: 217, 1998. and Leg: Update ’97, ed by NS Vickers, SJ Miller, KT
34. GAZDAG A, CRACCHIOLO A: Surgical treatment of patients Mahan, et al, p 11, The Podiatry Institute, Tucker, GA,
with painful instability of the second metatarsophalan- 1997.
geal joint. Foot Ankle 19: 137, 1998. 40. DALY PJ, JOHNSON KA: Treatment of painful subluxation
35. GIRDLESTONE GR: Physiotherapy for the hand and foot. at the second and third metatarsophalangeal joints by
J Chartered Soc Physiotherapy 32: 167, 1947. partial proximal phalanx excision and subtotal web-
36. TRNKA HJ, MUHLBAUER M, ZETTL R: Comparison of the bing. Clin Orthop 278: 164, 1992.
Weil and Helal osteotomies for the treatment of meta- 41. C RACCHIOLO A 3 RD , K ITAOKA HB, L EVENTEN EO: Silicone
tarsalgia secondary to dislocation of the lesser metatar- implant arthroplasty for second metatarsophalangeal
sophalangeal joints. Foot Ankle 20: 72, 1999. joint disorders with and without hallux abducto valgus
37. RUCH JA: “A Surgical Technique for the Repair of the Pre- deformities. Foot Ankle 9: 10, 1988.
dislocation Syndrome,” in Reconstructive Surgery of the 42. VANDEPUTTE G, DEREYMAEKER G, STEENWERCKX A, ET AL:
Foot and Leg: Update ’97, ed by NS Vickers, SJ Miller, The Weil osteotomy of the lesser metatarsals: a clinical
KT Mahan, et al, p 7, The Podiatry Institute, Tucker, GA, and pedobarographic follow-up study. Foot Ankle 21:
1997. 370, 2000.

Journal of the American Podiatric Medical Association • Vol 92 • No 4 • April 2002 199

Potrebbero piacerti anche